Medicine MCQs for Medical Professionals - 3E (2013) [PDF]

April 4, 2018 | Author: Rohit Rajeev | Category: Red Blood Cell, Anemia, Haematopoiesis, Bone Marrow, Shock (Circulatory)


Comments



Description

MedicineM Qs for Medical Professionals C R G d V ti e U n - 9 9 i r h ta Medicine M Qs for Medical Professionals C Third Edition R G d V ti e U n Dr. Ajay Mathur Professor of Medicine - SMS Medical College and Hospital Jaipur, Rajasthan, India 9 9 i r h ta The Health Sciences Publishers New Delhi | London | Philadelphia | Panama Jaypee Brothers Medical Publishers (P) Ltd Headquarters Jaypee Brothers Medical Publishers (P) Ltd 4838/24, Ansari Road, Daryaganj New Delhi 110 002, India Phone: +91-11-43574357 Fax: +91-11-43574314 Email: [email protected] Overseas Offices J.P. Medical Ltd Jaypee-Highlights Medical Publishers Inc Jaypee Medical Inc 83 Victoria Street, London City of Knowledge, Bld. 237, Clayton The Bourse SW1H 0HW (UK) Panama City, Panama 111 South Independence Mall East Phone: +44 20 3170 8910 Phone: +1 507-301-0496 Suite 835, Philadelphia, PA 19106, USA Fax: +44 (0)20 3008 6180 Fax: +1 507-301-0499 Phone: +1 267-519-9789 Email: [email protected] Email: [email protected] Email: [email protected] Jaypee Brothers Medical Publishers (P) Ltd Jaypee Brothers Medical Publishers (P) Ltd 17/1-B Babar Road, Block-B, Shaymali Bhotahity, Kathmandu, Nepal Mohammadpur, Dhaka-1207 Phone +977-9741283608 Bangladesh Email: [email protected] Mobile: +08801912003485 Email: [email protected] Website: www.jaypeebrothers.com Website: www.jaypeedigital.com © 2014, Jaypee Brothers Medical Publishers The views and opinions expressed in this book are solely those of the original contributor(s)/author(s) and do not necessarily represent those of editor(s) of the book. All rights reserved. No part of this publication may be reproduced, stored or transmitted in any form or by any means, electronic, mechanical, photocopying, recording or otherwise, without the prior permission in writing of the publishers. All brand names and product names used in this book are trade names, service marks, trademarks or registered trademarks of their respective owners. The publisher is not associated with any product or vendor mentioned in this book. Medical knowledge and practice change constantly. This book is designed to provide accurate, authoritative information about the subject matter in question. However, readers are advised to check the most current information available on procedures included and check information from the manufacturer of each product to be administered, to verify the recommended dose, formula, method and duration of administration, adverse effects and contraindications. It is the responsibility of the practitioner to take all appropriate safety precautions. Neither the publisher nor the author(s)/editor(s) assume any liability for any injury and/or damage to persons or property arising from or related to use of material in this book. This book is sold on the understanding that the publisher is not engaged in providing professional medical services. If such advice or services are required, the services of a competent medical professional should be sought. Every effort has been made where necessary to contact holders of copyright to obtain permission to reproduce copyright material. If any have been inadvertently overlooked, the publisher will be pleased to make the necessary arrangements at the first opportunity. Inquiries for bulk sales may be solicited at: [email protected] Medicine MCQ’s for Medical Professionals First Edition: 2007 Second Edition: 2010 Third Edition: 2013 Reprint: 2014 ISBN 978-93-5152-551-6 Printed at Foreword As a professional who has been practicing medicine for over four decades now, I appreciate the value this book brings to the table in times like today. As we move from a largely descriptive era to the bullet-point generation, this academic initiative appears even more relevant than its first two editions. Many refinements have been made in this book bearing in mind the reception it has received in the last few years. The book has been a reference point for many medical entrance examinations and has left an impact on medical G professionals who look for high quality of academic material. R Harrison’s Principles of Internal Medicine, published by The McGraw-Hill Companies, Inc. is an epic in the world of medical science. This book serves as a faithful companion to the epic by assisting the readers draw most out of it V in the service of mankind. d Knowledge is a more processed form of information. Prof. Ajay Mathur stays true to his pledge by presenting well-digested bytes of knowledge across different fields of medicine. He relies on good old word-of-mouth to make ti e this book a success rather than blitzkrieg marketing. I recommend that you make this a must-have without a shadow of doubt. U n Dr Ramesh Roop Rai Professor & Head, Gastroenterology (Retd.) - SMS Medical College & Hospital, Jaipur Past President, Indian Society of Gastroenterology (2008) 9 9 i r h ta Preface Medicine, in all its vastness, needs to be understood in a way that makes most sense to how it is applied today. Memorizing each word is elusive and therefore, testing knowledge of a discipline remains an evergreen challenge. It is a widely accepted fact that taking a quiz soon after studying helps one retain information and knowledge better. The brain works in mysterious ways but a sure way of holding onto what the mind has already digested is to put lessons to test. Multiple Choice Questions are a quick and effective way of remembering the gist of the matter. This is precisely the reason why most examinations today follow this format. This book is committed to hone your skills for retaining knowledge; it is only axiomatic that excellence will follow when you acquire knowledge properly. In its third edition, this book incorporates the recent advances in medicine as well as my personal insights on how to learn better. Based on earlier and the 18th edition of Harrison’s Principles of Internal Medicine, published by The McGraw-Hill Companies, Inc., this book also comprises relevant studies from the leading medical journals from the world over. This book caters to medical professionals at all levels. Not only can this be used by aspiring doctors to prepare for medical entrance examinations but by seasoned medical professionals to update knowledge long after it has been acquired. The book is sign-posted with resources and references should the reader require elaboration on any given topic. Over ten thousand questions and still counting; I take it upon myself to continually refine the content of the book and chronicle the advances of medical science. Dr Ajay Mathur Professor of Medicine SMS Medical College & Hospital Jaipur Contents 1. Hematology 1 2. Cardiology 103 3. Respiratory 239 G 4. Nephrology 319 R 5. Gastroenterology 389 V d 6. Endocrinology 523 ti e 7. Neurology 643 8. Miscellaneous 827 n Genetics, HIV/AIDS, Tuberculosis, Malaria, Dengue, Typhoid, Syphilis, Skin, Rheumatoid U arthritis, Leptospirosis, Rabies - 9 9 i r h ta 1 Hematology MCQ’s FOR MCQ’s FOR MEDICAL MEDICAL PROFESSIONALS PROFESSIONALS BY PROF. AJAY MATHUR Cardiology 1 7 What percentage of all circulating RBC’s is replaced daily ? Chapter 57. Anemia and Polycythemia Harrison’s 18th Ed. 448 A. 0.2 to 0.4 % 1 According to WHO, anemia is defined as as a hemoglobin B. 0.4 to 0.6 % level of ? Harrison’s 18th Ed. 449 C. 0.6 to 0.8 % A. < 14 g/dL in men & < 13 g/dL in women D. 0.8 to 1% B. < 13 g/dL in men & < 12 g/dL in women Since the average red cell lives for 100 - 120 days, normal red cell production results in the daily replacement of 0.8 - 1% of all circulating red cells in the body, C. < 12 g/dL in men & < 11 g/dL in women D. < 11 g/dL in men & < 10 g/dL in women 8 Term “erythron” best relates to ? Harrison’s 18th Ed. 448 The World Health Organization (WHO) defines anemia as a hemoglobin level < 13 g/dL in men and < 12 g/dL in women. A. Erythroid / megakaryocyte progenitor B. Red cell destruction 2 Hematopoietic stem cell produce which of the following ? Harrison’s 18th Ed. 448 C. Organ responsible for red cell production A. Red cells D. Red blood cell mass B. All classes of granulocytes The organ responsible for red cell production is called “erythron” which consists of a rapidly proliferating pool of marrow erythroid precursor cells & the mass of mature circulating RBCs. C. Cells of the immune system D. All of the above 9 Erythropoietin is a ? Harrison’s 18th Ed. 448 Stem cells are capable of producing red cells, all classes of granulocytes, monocytes, platelets, and the cells of the immune system. A. Peptide hormone B. Glycoprotein hormone 3 In the absence of erythropoietin (EPO), committed erythroid progenitor cells undergo ? C. Steroid hormone Harrison’s 18th Ed. 448 D. None of the above A. Stunted growth EPO is a glycoprotein hormone that acts by binding to specific receptors on surface of marrow B. Halting of growth erythroid precursors, inducing them to proliferate & mature when iron is adequately available. C. Programmed cell death (apoptosis) 10 Erythropoietin is produced and released by ? D. Change to other series of hematopoietic cell Harrison’s 18th Ed. 448 For RBC production, EPO is the regulatory hormone. It is required for maintenance of committed A. Glomerular capillaries erythroid progenitor cells which undergoes programmed cell death (apoptosis) if EPO is absent. B. Proximal tubular cells 4 How many mature red cells are produced from a pronormoblast ? C. Peritubular capillary lining cells of kidney Harrison’s 18th Ed. 448 D. All of the above A. 1 to 16 Physiologic regulator of RBC production, glycoprotein hormone EPO, is produced & released by B. 16 to 32 highly specialized epithelial-like peritubular capillary lining cells within kidney. C. 32 to 48 11 Erythropoietin is also produced by ? D. 48 to 64 Harrison’s 18th Ed. 448 Pronormoblast undergoes 4 - 5 cell divisions resulting in the production of 16 - 32 mature RBC’s. A. Pancreas B. Spleen 5 Regulation of EPO production is linked to ? Harrison’s 18th Ed. 448 C. Hepatocytes A. O2 D. Al of the above B. CO 2 A small amount of EPO is produced by hepatocytes. C. Red cell mass 12 Key to EPO gene regulation is ? D. Hemoglobin concentration Harrison’s 18th Ed. 448 The regulation of EPO production is linked to O 2 availability. A. (HIF)-1 6 Which of the following about mature red cell is false ? B. (EIF)-1 Harrison’s 18th Ed. 448 C. (RIF)-1 A. Diameter is 8 µm D. (GIF)-1 B. Anucleate Key to EPO gene regulation is hypoxia-inducible factor (HIF)-1. C. Discoid in shape 13 EPO production is increased in ? D. None of the above Harrison’s 18th Ed. 448 Mature RBC is 8 µ in diameter, anucleate, discoid in shape, and extremely pliable for it to A. Anemia negotiate microcirculation successfully. 2 MCQ’s FOR MEDICAL PROFESSIONALS BY PROF. AJAY MATHUR Hematology B. Hypoxemia D. Bennet effect C. Renal artery stenosis Bohr effect refers to enhanced O 2 delivery through changes in O 2 –hemoglobin dissociation curve mediated by a decreased pH or increased CO 2. D. All of the above The fundamental stimulus for EPO production is the availability of O 2 for tissue metabolic 20 Signs of vascular instability appear with acute blood loss of ? needs. Anemia, hypoxemia, or renal artery stenosis can raise EPO production. Harrison’s 18th Ed. 449 A. 5 to 10 % of total blood volume 14 Normal level of Plasma EPO is ? Harrison’s 18th Ed. 448 B. 10 to 15 % of total blood volume A. 10 to 25 U/L C. 15 to 20 % of total blood volume B. 25 to 50 U/L D. 20 to 25% of total blood volume C. 50 to 75 U/L Signs of vascular instability appear with acute losses of 10 - 15% of the total blood volume. D. 75 to 100 U/L 21 Hypovolemic shock results if volume of blood lost is ? G Normal EPO level in plasma is 10 - 25 U/L. Harrison’s 18th Ed. 449 15 Plasma EPO levels increase when hemoglobin falls below ? A. > 25 % R Harrison’s 18th Ed. 449 Figure 57-2 B. > 30 % A. 10 to 12 g/dL C. > 35 % V B. 8 to 10 g/dL D. > 40 % d C. 6 to 8 g/dL If the volume of blood lost is >40% (>2 L in average-sized adult), signs of hypovolemic shock D. 4 to 6 g/dL including confusion, dyspnea, diaphoresis, hypotension and tachycardia appear. ti e When the hemoglobin level falls to 12 g/dL, plasma EPO levels increase logarithmically. When 22 Intravascular hemolysis with release of free hemoglobin hemoglobin concentration falls below 10 - 12 g/dL, plasma EPO levels increase in proportion to the severity of anemia. may be associated with ? n Harrison’s 18th Ed. 449 16 In circulation, EPO has a half-clearance time of ? A. Acute back pain Harrison’s 18th Ed. 448 B. Acute pain in lower limbs U A. 1 to 3 hours C. Acute pain in upper limbs - B. 3 to 6 hours D. Acute headache C. 6 to 9 hours Intravascular hemolysis with release of free hemoglobin may be associated with acute back 9 D. 9 to 12 hours pain, free hemoglobin in the plasma and urine, and renal failure. In circulation, EPO has a half-clearance time of 6 - 9 hours. 9 23 Which of the following may be associated with autoimmune r hemolysis ? 17 The mean hematocrit value for adult males is ? i Harrison’s 18th Ed. 449 Harrison’s 18th Ed. 448 A. Chronic lymphocytic leukemia h A. 42 % B. Infection B. 45 % ta C. Rheumatoid arthritis C. 47 % D. All of the above D. 49 % Chronic inflammatory states (infection, rheumatoid arthritis, cancer) are associated with mild Mean hematocrit value for adult males is 47% (± SD 7) and for adult females is 42% (± 5). to moderate anemia, whereas lymphoproliferative disorders (chronic lymphocytic leukemia and certain other B cell neoplasms) may be associated with autoimmune hemolysis. 18 Anemia is most often recognized by ? Harrison’s 18th Ed. 449 24 If palmar creases are lighter in color than surrounding skin, A. Attendant symptoms hemoglobin level is usually ? Harrison’s 18th Ed. 449 B. Attendant signs A. < 10 g/dL C. Abnormal screening laboratory tests B. < 8 g/dL D. All of the above C. < 6 g/dL Anemia is most often recognized by (incidental) abnormal screening laboratory tests. D. < 4 g/dL 19 O 2–hemoglobin dissociation curve relates to which of the If the palmar creases are lighter in color than the surrounding skin with hyperextended hand, following ? the hemoglobin level is usually < 8 g/dL. Harrison’s 18th Ed. 449 25 Femtoliters is the unit of expression of which of the A. Claude effect following ? B. Bohr effect Harrison’s 18th Ed. 449 C. Charles effect A. Mean cell volume (MCV) 3 Hematology MCQ’s FOR MCQ’s FOR MEDICAL MEDICAL PROFESSIONALS PROFESSIONALS BY PROF. AJAY MATHUR Cardiology 3 B. Mean cell hemoglobin (MCH) 32 Reliable estimate of red cell production is provided by ? Harrison’s 18th Ed. 452 C. Mean concentration of Hb per volume of RBCs (MCHC) A. EPO levels D. None of the above B. Reticulocyte count Mean cell volume (MCV) - femtoliters, mean cell hemoglobin (MCH) - picograms per cell, and mean concentration of hemoglobin per volume of red cells (MCHC) - grams per liter. C. RDW D. Polychromasia 26 Which of the following reflects iron supply ? Harrison’s 18th Ed. 450 Reticulocyte count provides a reliable measure of red cell production. A. Serum iron 33 “Shift” cells refer to ? B. Total iron-binding capacity (TIBC) Harrison’s 18th Ed. 453 C. Serum ferritin A. Older RBC’s D. All of the above B. Prematurely released reticulocytes Serum iron, total iron-binding capacity (TIBC), and serum ferritin are measurements of iron supply. C. Normoblasts D. Any of the above 27 Which of the following formula estimates MCV ? Harrison’s 18th Ed. 450 Table 57–2 In anemia, polychromatophilic macrocytes in PBF represent prematurely released reticulocytes referred to as “shift” cells. A. (Hematocrit x 10) / (red cell count x 10 ) 6 B. (Hemoglobin x 10) / (red cell count x 106) 34 Erythroid cells take about how many days to mature ? Harrison’s 17th Ed. 359 Figure 58-13 C. (Hemoglobin x 10) / hematocrit A. 2.5 D. (Hematocrit x 10) / hemoglobin B. 3.5 28 Anisocytosis is related to which feature of RBC ? C. 4.5 Harrison’s 18th Ed. 450 D. 5.5 A. Size Erythroid cells take ~4.5 days to mature. At normal hematocrit levels, they are released to the B. Shape circulation with ~1 day left as reticulocytes. C. Number 35 Absolute reticulocyte count is calculated by ? D. Colour Harrison’s 18th Ed. 452 Table 57–4 Variations in red cell size is called anisocytosis. A. Reticulocyte count x (Hemoglobin ÷ expected hemoglobin) 29 Poikilocytosis is related to which feature of RBC ? Harrison’s 18th Ed. 450 B. Reticulocyte count x (Hemoglobin + Hematocrit) A. Size C. Reticulocyte count x (Hemoglobin x Hematocrit) B. Shape D. Reticulocyte count / (Hemoglobin + Hematocrit) C. Number 36 Which of the following about macrocytes is false ? D. Colour Harrison’s 18th Ed. 451 Figure 57-5 A. Red cells are larger than a small lymphocyte Variations in red cell shape is called poikilocytosis. Poikilocytosis suggests a defect in the maturation of red cell precursors in bone marrow or fragmentation of circulating red cells. B. Well hemoglobinized red cells 30 RDW correlates with ? C. Often oval shaped Harrison’s 18th Ed. 450 D. None of the above A. Anisocytosis Macrocytes are red blood cells that are larger than a small lymphocyte and are well hemoglobinized. Often macrocytes are oval shaped (macro-ovalocytes). B. Poikilocytosis C. Polychromasia 37 Howell-Jolly bodies best relate to which of the following ? D. All of the above Harrison’s 18th Ed. 451 Figure 57-6 A. Uremia Degree of anisocytosis correlates with increases in RDW or the range of cell sizes. B. Nuclear remnants 31 Upon staining with supravital dye, reticulocytes are identified C. Foreign bodies in the circulation by what colour of punctate spots ? Harrison’s 18th Ed. 450 D. All of the above A. Red Howell-Jolly bodies refer to tiny nuclear remnants that are not removed from red cells due to absence of a functional spleen (splenectomy) and and with maturation/dysplastic disorders (excess B. Blue production). They remain as small homogeneously staining blue inclusions on Wright stain. C. Green 38 Teardrop-shaped red cells best relates to ? D. Yellow Harrison’s 18th Ed. 451 Figure 57-7 Reticulocytes are larger red cells, grayish blue in color on Wright-Giemsa stain, that are recently A. Uremia released from bone marrow & their color represents residual amounts of ribosomal RNA. B. Myelofibrosis 4 MCQ’s FOR MEDICAL PROFESSIONALS BY PROF. AJAY MATHUR Hematology C. Thalassemia Acanthocytes are contracted dense cells with irregular membrane projections that vary in length and width. Acanthocytes are present in severe liver disease, in patients with D. Liver disease abetalipoproteinemia, and in rare patients with McLeod blood group. Teardrop-shaped red cells are seen in myelofibrosis and extramedullary hematopoiesis. 45 The normal serum iron range is ? 39 Target red cells best relates to ? Harrison’s 18th Ed. 453 Harrison’s 18th Ed. 451 Figure 57-8 A. 10 to 50 µg / dL A. Uremia B. 50 to 150 µg / dL B. Myelofibrosis C. 150 to 250 µg / dL C. Thalassemia D. 250 to 450 µg / dL D. Acute hemolysis Normal serum iron ranges from 50 - 150 µg/dL Target red cells have a bull’s-eye appearance & are seen in thalassemia and liver disease. 46 The normal serum TIBC range is ? 40 Target cells are seen in ? Harrison’s 18th Ed. 453 Harrison’s 18th Ed. 451 Figure e17-13 A. 100 to 150 µg / dL A. Liver disease B. 150 to 300 µg / dL B. Thalassemia C. 300 to 360 µg / dL C. Hemoglobin C disease D. 350 to 450 µg / dL D. All of the above Normal TIBC is 300 - 360 µg/dL. Target cells are recognized by the bull’s-eye appearance of the red cell. Small numbers of target cells are seen with liver disease & thalassemia. Larger numbers are typical of hemoglobin C disease. 47 Percent transferrin saturation is calculated by ? Harrison’s 18th Ed. 453 41 Thermal injury may produce which of the following ? A. (TIBC x 100) ÷ Serum iron Harrison’s 18th Ed. 451 Figure 57-9 B. (Serum iron + TIBC) ÷ 100 A. Target cells C. (Serum iron x 100) ÷ TIBC B. Burr cells D. (Serum iron ÷ TIBC) x 100 C. Howell-Jolly bodies Percent transferrin saturation is derived by dividing serum iron level (x 100) by TIBC. Normal D. Red cell fragmentation transferrin saturation ranges from 25 - 50%. Presence of foreign bodies in the circulation (mechanical heart valves), or thermal injury may cause red cell fragmentation. 48 Adult males have average serum ferritin levels of about ? Harrison’s 18th Ed. 453 42 Burr cells are also called ? A. 10 µg / L Harrison’s 18th Ed. 451 Figure 57-10 B. 50 µg / L A. Echinocytes C. 100 µg / L B. Acanthocytes D. 200 µg / L C. Elliptocytes Serum ferritin is used to evaluate total-body iron stores. Adult males have serum ferritin levels D. Spherocytes that average ~100 µg/L, corresponding to iron stores of ~1 gram. Serum ferritin level of 10 - 15 µg/L represents depletion of body iron stores. Burr cells are also called echinocytes. Echinocytes have small, uniform, and evenly spaced membrane projections. 49 Adult females have average serum ferritin levels of about ? 43 Echinocytes are found in patients with ? Harrison’s 18th Ed. 453 Harrison’s 18th Ed. 451 Figure e17-16 A. 10 µg / L A. Severe uremia B. 30 µg / L B. Glycolytic red cell enzyme defects C. 60 µg / L C. Microangiopathic hemolytic anemia D. 90 µg / L D. All of the above Adult females have serum ferritin levels averaging 30 µg/L, reflecting lower iron stores (300 mg). Echinocytes are found in patients with severe uremia, in glycolytic red cell enzyme defects, and in microangiopathic hemolytic anemia. 50 Ferritin is also an ? Harrison’s 18th Ed. 453 44 Acanthocytes are present in which of the following conditions ? A. Enzyme Harrison’s 18th Ed. 451 Figure e17-16 B. Cytokine A. Severe liver disease C. Chemokine B. Abetalipoproteinemia D. Acute-phase reactant C. Patients with McLeod blood group Ferritin is also an acute-phase reactant. D. All of the above 5 Hematology MCQ’s FOR MCQ’s FOR MEDICAL MEDICAL PROFESSIONALS PROFESSIONALS BY PROF. AJAY MATHUR Cardiology 5 51 Erythroblasts containing what are called sideroblasts ? B. Abnormalities in heme synthesis Harrison’s 18th Ed. 454 C. Abnormalities in globin synthesis A. Hemosiderin D. Any of the above B. Ferritin Cytoplasmic maturation defects result from ‘severe’ iron deficiency or abnormalities in globin or C. Residual RNA heme synthesis. D. All of the above 58 Increased red cell mass is present when hematocrit in men In bone marrow smears, developing erythroblasts with small ferritin granules are sideroblasts. is ? Harrison’s 18th Ed. 456 52 Hemolysis is most likely cause if reticulocyte production A. > 45 % index is more than ? Harrison’s 18th Ed. 454 B. > 50 % A. 2.5 C. > 55 % B. 3.5 D. > 60 % C. 4.5 PCV >60% in men & >55% in women are invariably associated with an increased red cell mass. D. 5.5 59 Gaisbock’s syndrome relates to ? Reticulocyte production index of >2.5 indicates that hemolysis is most likely. Harrison’s 18th Ed. 456 A. Spurious polycythemia 53 Which of the following anemias is most frequent ? Harrison’s 18th Ed. 455 B. High altitude polycythemia A. Hypoproliferative C. Ectopic EPO production B. Maturation Disorders D. Familial polycythemia C. Hemoglobinopathies Gaisbock’s syndrome refers to spurious polycythemia due to a decrease in plasma volume. D. Hemolytic 60 EPO-producing neoplasms include ? At least 75% of all cases of anemia are hypoproliferative in nature. Hemolytic disease is among Harrison’s 18th Ed. 457 the least common forms of anemia. A. Hepatoma 54 A hypoproliferative anemia can result from ? B. Uterine leiomyoma Harrison’s 18th Ed. 455 C. Cerebellar hemangiomas A. Marrow damage D. All of the above B. Iron deficiency EPO-producing neoplasms include hepatoma, uterine leiomyoma, renal cancer or cysts and C. Inadequate EPO stimulation cerebellar hemangiomas. D. All of the above Majority of hypoproliferative anemias are due to mild to moderate iron deficiency or inflammation. They result from marrow damage, iron deficiency or inadequate EPO stimulation. Chapter 58. Bleeding and Thrombosis 55 Which of the following is the iron regulatory hormone ? Harrison’s 18th Ed. 455 61 In the human hemostatic system, procoagulant forces include ? A. Erythropoietin Harrison’s 18th Ed. 457 B. Transferrin A. Platelet adhesion C. Hepcidin B. Platelet aggregation D. All of the above C. Fibrin clot formation Hepcidin is the iron regulatory hormone that is increased in inflammation. D. All of the above 56 Nuclear maturation defects of RBC’s result from all except ? In the human hemostatic system, procoagulant forces include platelet adhesion and aggregation and fibrin clot formation. Harrison’s 18th Ed. 455 A. Vitamin B12 deficiency 62 The major components of the hemostatic system are ? B. Folic acid deficiency Harrison’s 18th Ed. 457 C. Iron deficiency A. Platelets, monocytes and red cells D. Methotrexate therapy B. Plasma proteins C. Vessel wall Nuclear maturation defects result from vitamin B 12 or folic acid deficiency, drug damage (methotrexate or alkylating agents), myelodysplasia and alcohol. D. All of the above 57 Cytoplasmic maturation defects of RBC’s result from ? Major components of the hemostatic system platelets & other formed elements of blood (monocytes & RBCs), plasma proteins (coagulation & fibrinolytic factors & inhibitors) and Harrison’s 18th Ed. 455 vessel wall. A. Severe iron deficiency 6 MCQ’s FOR MEDICAL PROFESSIONALS BY PROF. AJAY MATHUR Hematology 63 Formed element of blood that is not a component of the 69 Surface of each platelet has how many GpIIb / IIIa binding hemostatic system is ? sites ? Harrison’s 18th Ed. 457 Harrison’s 18th Ed. 457 A. Neutrophil A. ~ 10,000 B. Monocyte B. ~ 25,000 C. Red cell C. ~ 50,000 D. Platelet D. ~ 100,000 Major components of hemostatic system are platelets, monocytes & red cells, plasma proteins Surface of each platelet has ~50,000 GpIIb/IIIa binding sites. (coagulation & fibrinolytic factors & inhibitors) and vessel wall itself. 70 Tissue factor (TF) is present on ? 64 Platelet adhesion is mediated primarily by ? Harrison’s 18th Ed. 458 Harrison’s 18th Ed. 457 A. Subendothelial cellular components of vessel wall A. von Willebrand factor (vWF) B. Circulating microparticles from monocytes B. Gp IIb/IIIa C. Circulating microparticles from platelets C. Gp Ia/IIa D. All of the above D. Fibronectin TF is expressed on surfaces of subendothelial cellular components of vessel wall (smooth-muscle cells Platelet adhesion is mediated primarily by von Willebrand factor (vWF). & fibroblasts). TF is present in circulating microparticles shed from monocytes & platelets. 65 Which of the following is false about von Willebrand factor (VWF) ? 71 Which of the following is a serine protease factor ? Harrison’s 18th Ed. 457 Harrison’s 18th Ed. 458 A. Large multimeric protein A. III B. Present in plasma B. VIIa C. Present in extracellular matrix of subendothelial vessel C. IX wall D. X D. None of the above TF binds serine protease factor VIIa and their complex (TF+VIIa) activates factor X to factor Xa. Factor IXa also activates factor X to factor Xa. von Willebrand factor (VWF) is a large multimeric protein present in both plasma & extracellular matrix of subendothelial vessel wall. It serves as the primary “molecular glue” to withstand high levels of shear stress that would detach platelets due to flow of blood. 72 Essential cofactor for conversion of prothrombin to thrombin is ? 66 Which of the following is released from activated platelets ? Harrison’s 18th Ed. 458 Harrison’s 18th Ed. 457 A. Va A. Epinephrine B. VIIa B. Thrombin C. IX C. Adenosine diphosphate D. X D. All of the above Factor Xa converts prothrombin to thrombin, the pivotal protease of the coagulation system. Platelet adhesion is followed by platelet activation and aggregation which is enhanced & The essential cofactor for this reaction is factor Va. Factor Va is produced by thrombin-induced amplified by humoral mediators in plasma (epinephrine, thrombin), mediators released from limited proteolysis of factor V. activated platelets (adenosine diphosphate, serotonin) & vessel wall extracellular matrix constituents that come in contact with adherent platelets (collagen, VWF). 73 Which of the following in endothelial cells has antithrombotic effect ? 67 Most abundant receptor on the platelet surface is ? Harrison’s 18th Ed. 458 Harrison’s 18th Ed. 457 A. Prostacyclin A. Gp Ia/IIa B. Nitric oxide B. Gp Ib/IX C. EctoADPase/CD39 C. Gp IIb/IIIa D. All of the above D. Gp VI Prostacyclin, nitric oxide & ectoADPase/CD39 produced from endothelial cells act to inhibit Platelet glycoprotein (Gp) IIb/IIIa (IIb3) complex is the most abundant receptor on platelet surface. platelet binding, secretion and aggregation. 68 Platelet surface receptor GpIIB / IIIa binds to ? 74 Anticoagulant factor produced by endothelial cells is ? Harrison’s 18th Ed. 457 Harrison’s 18th Ed. 458 A. Thrombospondin A. Heparan proteoglycans B. Fibrinogen B. Antithrombin C. Collagen C. Thrombomodulin D. All of the above D. All of the above Platelet activation converts the normally inactive GpIIb/IIIa receptor into an active receptor Endothelial cells produce anticoagulant factors like heparan proteoglycans, antithrombin, TF enabling its binding to fibrinogen and vWF. pathway inhibitor and thrombomodulin. 7 Hematology MCQ’s FOR MCQ’s FOR MEDICAL MEDICAL PROFESSIONALS PROFESSIONALS BY PROF. AJAY MATHUR Cardiology 7 75 Which of the following produced by endothelial cells acts 81 Which of the following relates to protein S ? as an anticoagulant ? Harrison’s 18th Ed. 459 Harrison’s 18th Ed. 458 A. Cofactor A. Plasminogen activator inhibitor B. Glycoprotein B. Thrombomodulin C. Vitamin K dependent posttranslational modification C. Prostacyclin D. All of the above D. Nitric oxide Glycoprotein Protein S is a cofactor that undergoes vitamin K dependent posttranslational modification. It accelerates the reaction of activated protein C with factors V and VIII. 76 Endothelial cells activate fibrinolytic mechanisms through the production of ? 82 Which of the following inhibits TF / FVIIa / FXa complex ? Harrison’s 18th Ed. 458-9 Harrison’s 18th Ed. 459 A. Tissue plasminogen activator 1 A. Activated protein C B. Urokinase B. Protein S C. Plasminogen activator inhibitor C. Tissue factor pathway inhibitor (TFPI) D. All of the above D. All of the above Endothelial cells activate fibrinolytic mechanisms through the production of tissue plasminogen Tissue factor pathway inhibitor (TFPI) is a plasma protease inhibitor that regulates the TF– activator 1, urokinase, plasminogen activator inhibitor & annexin-2. induced extrinsic pathway of coagulation. TFPI inhibits the TF/FVIIa/FXa complex. 77 Which of the following best relates to heparin ? 83 Tissue factor pathway inhibitor (TFPI) be released by ? Harrison’s 18th Ed. 459 Harrison’s 18th Ed. 459 A. Antithrombin III A. Heparin B. Protein C B. Streptokinase C. Protein S C. Urokinase D. All of the above D. All of the above Antithrombin III inhibits thrombin by forming inactivating complexes that increase several folds TFPI is bound to lipoprotein and can also be released by heparin from endothelial cells, where in the presence of heparin. it is bound to glycosoaminoglycans, and from platelets. 78 Which of the following becomes an anticoagulant when it is 84 Plasminogen activators (tPA & uPA) cleave which bond of activated by thrombin ? plasminogen to generate the active enzyme plasmin ? Harrison’s 18th Ed. 459 Harrison’s 18th Ed. 459 A. Antithrombin III A. Arg460 - Val461 B. Protein C B. Arg560 - Val561 C. Protein S C. Arg660 - Val661 D. Tissue factor pathway inhibitor (TFPI) D. Arg760 - Val761 Protein C is a plasma glycoprotein that becomes an anticoagulant when activated by thrombin. Plasminogen activators (tissue type plasminogen activator & urokinase type plasminogen activator cleave Arg560-Val561 bond of plasminogen to generate the active enzyme plasmin which is the major 79 Which of the following about thrombomodulin is false ? protease enzyme of the fibrinolytic system, acting to digest fibrin to fibrin degradation products. Harrison’s 18th Ed. 458-9 A. Transmembrane proteoglycan binding site for thrombin on 85 “Fibrin specific” activity of plasmin is due to its ? Harrison’s 18th Ed. 459 endothelial cell surface A. Arginine-binding sites B. Thrombin-induced activation of protein C occurs B. Lysine-binding sites physiologically on thrombomodulin C. Valine-binding sites C. Anticoagulant factor from endothelial cell D. Leucine-binding sites D. None of the above The lysine-binding sites of plasmin (and plasminogen) permit it to bind specifically to fibrin and Thrombomodulin expressed on the surface of endothelial cells binds thrombin at low therefore physiologic fibrinolysis is “fibrin specific”. concentrations and inhibits coagulation through activation of the protein C pathway, leading to enhanced catabolism of clotting factors Va and VIIIa, thereby combating thrombus formation. 86 Physiologic regulation of fibrinolysis is done by ? 80 Activated protein C acts as an anticoagulant by cleaving Harrison’s 18th Ed. 460 and inactivating activated factor ? A. Plasminogen activator inhibitors (PAI-1 & PAI-2) Harrison’s 18th Ed. 459 B. Thrombin-activatable fibrinolysis inhibitor (TAFI) A. II C.  2-antiplasmin B. V D. All of the above C. VI D. X 87 Which of the following antiplasmin inhibits plasmin ? Harrison’s 18th Ed. 460 Activated protein C acts as an anticoagulant by cleaving & inactivating activated factors V & VIII. A. 1 antiplasmin 8 MCQ’s FOR MEDICAL PROFESSIONALS BY PROF. AJAY MATHUR Hematology B. 2 antiplasmin C. Horse chestnut C. 3 antiplasmin D. All of the above D. 4 antiplasmin Coumarin containing herbs include Motherworth (Leonurus cardiaca), Chamomile (Matricaria recutita, Chamaemelum mobile), Horse chestnut (Aesculus hippocastanum) Red clover (Trifolium PAI1 is the primary inhibitor of tPA & uPA and 2 antiplasmin is the main inhibitor of plasmin in pratense) Fenugreek (Trigonella foenum-graecum). human plasma, inactivating any nonfibrin clot associated plasmin. 94 Bruising or mucosal bleeding may be the presenting 88 Epistaxis is the most common symptom in ? complaint in ? Harrison’s 18th Ed. 460 Harrison’s 18th Ed. 461 A. Hemophilia A A. Liver disease B. Ehlers-Danlos syndrome B. Severe renal impairment, C. Hereditary hemorrhagic telangiectasia C. Hypothyroidism D. Cushing’s syndrome D. All of the above Epistaxis is the most common symptom in hereditary hemorrhagic telangiectasia & in boys with VWD. Bruising or mucosal bleeding may be the presenting complaint in liver disease, severe renal impairment, hypothyroidism, paraproteinemias or amyloidosis & bone marrow failure. 89 Menorrhagia is a common symptom in women with ? Harrison’s 18th Ed. 460 95 All coagulation factors are synthesized in ? A. VWD Harrison’s 18th Ed. 461 B. Factor XI deficiency A. Liver C. Symptomatic carriers of hemophilia A B. Kidney D. All of the above C. Lungs Menorrhagia is a common symptom in women with underlying bleeding disorders. It is seen in D. None of the above majority of women with VWD & factor XI deficiency & in symptomatic carriers of hemophilia A. All coagulation factors are synthesized in liver & hepatic failure results in combined factor deficiencies. 90 Which of the following is called a “life-threatening site of bleeding” ? 96 Which of the following coagulation factors is dependent on Harrison’s 18th Ed. 461 vitamin K for posttranslational modification ? A. Bleeding into oropharynx Harrison’s 18th Ed. 461 B. Bleeding into central nervous system A. Factor II C. Bleeding into retroperitoneum B. Factor VII D. All of the above C. Factor IX Life-threatening sites of bleeding include bleeding into oropharynx, into central nervous system, D. All of the above and into retroperitoneum. 97 Which of the following proteins is dependent on vitamin K 91 Which of the following about clopidogrel is false ? for posttranslational modification ? Harrison’s 18th Ed. 461 Harrison’s 18th Ed. 461 A. Thienopyridine A. Protein C B. Inhibits ADP-mediated platelet aggregation B. Protein S C. Can precipitate or exacerbate bleeding symptoms C. Protein Z D. None of the above D. All of the above Thienopyridines (clopidogrel and prasugrel) inhibit ADP-mediated platelet aggregation and like Coagulation factors II, VII, IX, X and proteins C, S, and Z are dependent on vitamin K for NSAIDs can precipitate or exacerbate bleeding symptoms. posttranslational modification. 92 Herb with potential anti-platelet activity is ? 98 Normal blood platelet count is ? Harrison’s 18th Ed. 461 Table 58–2 Harrison’s 18th Ed. 461 A. Ginger A. 50,000 to 100,000/µL B. Turmeric B. 100,000 to 250,000/µL C. Garlic C. 150,000 to 450,000/µL D. All of the above D. 250,000 to 550,000/µL Herbs with potential anti-platelet activity include Ginkgo, Garlic, Bilberry, Ginger, Dong quai, Normal blood platelet count is 150,000 to 450,000/µL. Feverfew, Asian, Siberian & American ginseng, Turmeric, Meadowsweet and Willow. 99 Thrombocytopenia results from ? 93 Which of the following is a Coumarin containing herb ? Harrison’s 18th Ed. 461 Harrison’s 18th Ed. 461 Table 58–2 A. Decreased production of platelets A. Motherworth B. Increased destruction of platelets B. Chamomile C. Sequestration of platelets 9 Hematology MCQ’s FOR MCQ’s FOR MEDICAL MEDICAL PROFESSIONALS PROFESSIONALS BY PROF. AJAY MATHUR Cardiology 9 D. Any of the above D. (PT ratio) International Sensitivity Index (ISI) Thrombocytopenia results from decreased production, increased destruction, &/or sequestration. International normalized ratio (INR) is calculated by the formula: INR = (PT patient/PTnormal mean)ISI. 100 Bleeding rarely occurs in isolated thrombocytopenia at 106 Activated partial thromboplastin time (aPTT) assesses which counts ? of the following factors ? Harrison’s 18th Ed. 461 Harrison’s 18th Ed. 462 A. < 50000 / µL A. Factor VIII B. < 80000 / µL B. Factor IX C. < 100000 / µL C. Factor X D. < 150000 / µL D. All of the above Bleeding rarely occurs in isolated thrombocytopenia at counts < 50000 / µL. aPTT assesses the intrinsic & common coagulation pathways, factors XI, IX, VIII, X, V, II, fibrinogen, and also prekallikrein, high-molecular-weight kininogen & factor XII. 101 Most procedures can be performed in patients with a platelet 107 Structure of Fibrinogen is ? count of ? Harrison’s 18th Ed. 458 Figure 58-2 Harrison’s 18th Ed. 461 A. Uninodular A. < 50000 / µL B. Binodular B. < 80000 / µL C. Trinodular C. < 100000 / µL D. Quadrinodular D. < 150000 / µL Fibrinogen is a trinodular structure consisting of 2 D domains and 1 E domain. Most procedures can be performed in patients with a platelet count of 50,000/µL. For major surgery, a count of about 80,000/µL is likely to be sufficient. 108 Cross-linking of the D domains on adjacent fibrinogen 102 The major risk factor for arterial thrombosis is ? molecules is done by ? Harrison’s 18th Ed. 458 Figure 58-2 Harrison’s 18th Ed. 461 A. FX A. Atherosclerosis B. FXI B. Hyperhomocysteinemia C. FXII C. Dysfibrinogenemia D. FXIIIa D. Hormonal therapy Cross-linking of D domains on adjacent fibrinogen molecules is done by FXIIIa. The major risk factor for arterial thrombosis is atherosclerosis. Risk factors for venous thrombosis are immobility, surgery, underlying medical conditions, malignancy, hormonal therapy, obesity, and genetic predispositions. 109 Which of the following about D-Dimers is false ? Harrison’s 18th Ed. 458 103 Most coagulation assays are performed in plasma A. Product of complete lysis of fibrin anticoagulated with ? B. Released when plasmin acts on fibrin Harrison’s 18th Ed. 462 A. Ethylenediamine tetraacetic acid (EDTA) C. Relatively specific test of fibrin degradation B. Sodium citrate D. Relatively specific test of fibrinogen degradation C. Heparin D-Dimers are the product of complete lysis of fibrin, maintaining the cross-linked D domains. When plasmin acts on covalently cross-linked fibrin, D-dimers are released. D-dimers in plasma are as a D. Any of the above relatively specific test of fibrin rather than fibrinogen degradation. D-Dimer assays are a sensitive marker of blood clot formation and coagulation activation. Most coagulation assays are performed in sodium citrate anticoagulated plasma that is recalcified for the assay. 110 Normal level of D-Dimer in blood is ? Harrison’s 17th Ed. 1653 104 PT assesses the factors except ? Harrison’s 18th Ed. 462 A. < 500 pg/mL A. Factor I B. < 500 ng/mL B. Factor II C. < 500 µg/mL C. Factor V D. < 500 mg/mL D. Factor VIII Normal level of D-Dimer in blood is < 500 ng/mL. PT assesses factors I (fibrinogen), II (prothrombin), V, VII, and X. PT only measures one 111 Plasma level of D-Dimer is ? aspect of hemostasis affected by liver dysfunction. Harrison’s 17th Ed. Appendix Table 1 105 International normalized ratio (INR) is calculated by ? A. 0.22 - 0.74 mg/mL Harrison’s 18th Ed. 462 B. 0.22 - 0.74 µg/mL A. PT ratio x International Sensitivity Index (ISI) C. 0.22 - 0.74 ng/mL B. PT ratio ÷ International Sensitivity Index (ISI) D. 0.22 - 0.74 pg/mL C. PT ratio + International Sensitivity Index (ISI) Plasma level of D-Dimer is 0.22 - 0.74 µg/mL. 10 MCQ’s FOR MEDICAL PROFESSIONALS BY PROF. AJAY MATHUR Hematology 112 D-dimer levels increase in patients with ? 118 Platelet surface receptor GpVI / FcRIIa binds to ? Harrison’s 17th Ed. Chapter 256, 96, 292 Harrison’s 16th Ed. 338 A. Myocardial infarction A. vWF B. Pneumonia B. Fibrinogen C. II or III trimester of pregnancy C. Collagen D. All of the above D. All of the above D-dimer levels increase in myocardial infarction, pneumonia, intestinal ischemia, sepsis, cancer, GpVI/Fc  RIIa binds collagen. postoperative state, initial infusion of human or humanized antibodies (rituximab, gemtuzumab, trastuzumab), and second or third trimester of pregnancy. D-dimer elevation is not as predictive of DVT in cancer patients as it is in patients without cancer. 119 After leaving bone marrow, what proportion of platelets are sequestered in spleen ? 113 Platelet interaction with vascular collagen is stabilized by ? Harrison’s 16th Ed. 673 Harrison’s 16th Ed. 337 A. One-third A. Gp Ia/IIa B. One-half B. Gp VI C. Two-thirds C. von Willebrand factor (vWF) D. Three-fourths D. FcRg 120 The life span of platelets in circulation is about ? von Willebrand factor (vWF) is an adhesive glycoprotein that allows platelets to remain attached Harrison’s 16th Ed. 673 to the vessel wall despite the high shear forces generated within the vascular lumen by stabilizing interaction between platelets with collagen. A. 1 to 3 days B. 3 to 5 days 114 Platelet surface receptor GpIb / IX binds to ? Harrison’s 16th Ed. 338 C. 5 to 7 days A. vWF D. 7 to 10 days B. Fibrinogen After leaving bone marrow, ~one-third of platelets are sequestered in spleen, while the other two-thirds circulate for 7 to 10 days. C. Collagen D. All of the above 121 During menstrual cycle, platelet count rise at what time ? Harrison’s 16th Ed. 673 115 vWF forms a link between collagen fibrils & which of the A. Following ovulation following platelet receptor ? Harrison’s 16th Ed. 337, 338 Figure 53-2 B. At the onset of menses A. Gp Ia/IIa C. After completion of menstrual flow B. Gp Ib/IX D. Before ovulation C. Gp VI 122 During menstrual cycle, platelet count fall at what time ? D. All of the above Harrison’s 16th Ed. 673 GpIb/IX complex binds vWF. Adhesion of platelets with vessel wall is stabilized by von Willebrand A. Following ovulation factor, which forms a bridge between collagen fibrils in the vessel wall & receptors on platelet B. At the onset of menses glycoprotein Ib/IX. Similarly, platelet aggregation is mediated by fibrinogen, which links adjacent platelets via receptors on the platelet glycoprotein IIb/IIIa complex. C. After completion of menstrual flow D. Before ovulation 116 Formation of thromboxane A 2 (TXA 2) from arachidonic acid is mediated by enzyme ? Platelet count varies in menstrual cycle, rising following ovulation & falling at onset of menses. Harrison’s 16th Ed. 338 A. Phospholipase C 123 Platelet counts are decreased in which of the following deficiencies ? B. Phospholipase A2 Harrison’s 16th Ed. 673 C. Cyclooxygenase A. Severe Iron deficiency D. All of the above B. Folic acid deficiency Formation of TXA 2 from arachidonic acid is mediated by the enzyme cyclooxygenase. C. Vitamin B12 deficiency D. All of the above 117 Platelet surface receptor GpIa/IIa binds to ? Harrison’s 16th Ed. 338 Platelet count are decreased in severe iron, folic acid or vitamin B12 deficiency. A. vWF 124 Secondary or reactive thrombocytosis is due to which B. Fibrinogen property of platelets ? C. Collagen Harrison’s 16th Ed. 674 D. All of the above A. Hormonal GpIa/IIa binds collagen. B. Acute-phase reactant C. Enzymatic 11 Hematology MCQ’s FOR MCQ’s FOR MEDICAL MEDICAL PROFESSIONALS PROFESSIONALS BY PROF. AJAY MATHUR Cardiology 11 D. All of the above D. Anti-CD22 Platelets are acute-phase reactants. Secondary or reactive thrombocytosis refers to an increase Rituximab is an anti-CD20 monoclonal antibody. Efficacy is established in ITP. It eliminates in platelet counts in patients with systemic inflammation, tumors, bleeding & mild iron deficiency. normal B cells that produce antiplatelet antibody. 125 Which of the following cytokines interleukins stimulate platelet production in acute inflammation ? Chapter 59. Enlargement of Lymph Harrison’s 16th Ed. 674 A. IL-3 Nodes & Spleen B. IL-6 131 Generalized adenopathy means involvement of how many C. IL-11 noncontiguous lymph node areas ? D. All of the above Harrison’s 18th Ed. 466 Cytokines interleukin IL-3, IL-6 & IL-11 stimulate platelet production in acute inflammation. A. One or more B. Two or more 126 Mechanism of thrombocytopenia include ? Harrison’s 16th Ed. 674 C. Three or more A. Decreased bone marrow production D. Four or more B. Increased splenic sequestration Generalized adenopathy is defined as involvement of three or more noncontiguous lymph node areas. Localized or regional adenopathy implies involvement of a single anatomic area. C. Accelerated destruction D. All of the above 132 Which of the following is a cause of generalized lymphadenopathy ? Thrombocytopenia is caused by decreased bone marrow production or increased splenic Harrison’s 18th Ed. 466 sequestration or accelerated destruction of platelets. A. AIDS 127 TAR syndrome means ? B. Systemic lupus erythematosus (SLE) Harrison’s 16th Ed. 674 C. Mixed connective tissue disease A. Thrombocytopenia with absent reticulocytes D. All of the above B. Thrombocytopenia with absent renin Generalized lymphadenopathy is frequently associated with infectious mononucleosis (EBV) or C. Thrombocytopenia with absent radii cytomegalovirus (CMV), toxoplasmosis, AIDS, systemic lupus erythematosus (SLE), and mixed D. Thrombocytopenia with abnormal vessels connective tissue disease. Acute and chronic lymphocytic leukemias and malignant lymphomas also produce generalized adenopathy in adults. Congenital amegakaryocytic hypoplasia & thrombocytopenia with absent radii (TAR syndrome) produce a selective decrease in megakaryocyte production. 133 Occipital lymphadenopathy accompanies which of the following ? 128 Acute ITP is common in ? Harrison’s 18th Ed. 466 Harrison’s 16th Ed. 675 A. Scalp infection A. Children B. Ear infection B. Adults C. Conjunctival infections C. Elderly D. Tooth infection D. All of the above Occipital adenopathy often reflects infection of the scalp. Acute ITP is a severe thrombocytopenia following recovery from a viral exanthem or upper respiratory illness. It is common in children & is responsible 90% of pediatric cases of immunologic thrombocytopenia. >90% cases recover within 3 to 6 months. 134 Preauricular lymphadenopathy accompanies which of the following ? 129 Which of the following appear in the red cells of asplenic Harrison’s 18th Ed. 466 individuals ? A. Scalp infection Harrison’s 16th Ed. 675 B. Ear infection A. Macroovalocytes C. Conjunctival infections B. Basophilic stippling D. Tooth infection C. Howell-Jolly bodies Preauricular adenopathy accompanies conjunctival infections and cat-scratch disease. D. Target cells Howell-Jolly bodies are tiny nuclear remnants that are normally removed by spleen. They 135 Most frequent site of regional lymphadenopathy is ? appear in blood after splenectomy & with maturation / dysplastic disorders. Harrison’s 18th Ed. 466 A. Axilla 130 Rituximab is what type of a monoclonal antibody ? Harrison’s 16th Ed. 676 B. Neck A. Anti-CD19 C. Groin B. Anti-CD20 D. Abdominal C. Anti-CD21 Most frequent site of regional lymphadenopathy is neck. Tuberculosis A. 466 Harrison’s 17th Ed. a gastrointestinal primary. All of the above Romaña’s sign is a classic finding in acute Chagas’ disease. 145 In Winterbottom’s sign. usually secondary to an inflammatory process. 1304 A. Breast B. Stretching of capsule D. All of the above Virchow’s node is an enlarged left supraclavicular node infiltrated with metastatic cancer from Lymph nodes containing metastatic cancer are hard. Asymmetric C. Most frequent site of regional lymphadenopathy is neck Harrison’s 18th Ed. Waldenström’s macroglobulinemia A. a ratio of long to short axis of <2.25 cm2 B. Inguinal D. nonspecific reactive causes. 466 A. 147 Enlarged. sarcoidosis & toxoplasmosis are causes of supraclavicular adenopathy. rubbery. Virchow’s node is enlarged right supraclavicular node C. Sarcoidosis can cause supraclavicular adenopathy D. firm. Toxoplasmosis C. testis or ovarian cancers. On Ultrasonography of cervical nodes. Polycythemia vera B.25 cm 2 (1. Any of the above D. b. Nontender D.12 MCQ’s FOR MEDICAL PROFESSIONALS BY PROF. nontender & nonmovable.00 cm2 Lymphadenopathy of the posterior cervical triangle.75 cm2 D. Testis C.50 cm2 C. and nontender. 143 Which parameter is used in ultrasonography of cervical 137 Metastases to supraclavicular nodes occur from ? nodes for distinguishing benign from malignant nodes ? Harrison’s 18th Ed. Sonodensity Metastases to supraclavicular nodes occur from lung. 466 A. Lung A.5 x 1. Generalized Lymph node tenderness occurs when the capsule is stretched during its rapid enlargement. 466 A. < 0. 466 Harrison’s 18th Ed. 3154 Harrison’s 18th Ed. breast. mobile. Nontender Tangier disease is associated with cholesterol accumulation in reticuloendothelial system with Lymph nodes involved by lymphoma tend to be large. in T. grayish yellow or orange tonsils are pathognomonic 141 Which of the following is not a characteristic of enlarged of ? lymph nodes in lymphoma ? Harrison’s 18th Ed. Balantidiasis TB. Hard always abnormal B. 466 B. Inflammation of capsule B. AJAY MATHUR Hematology 136 Which of the following about lymphadenopathy is false ? 142 Which of the following is characteristic of enlarged lymph Harrison’s 18th Ed. 467 A. Toxoplasmosis C. Discrete B. Area B. Sarcoidosis B.0 is useful in distinguishing benign & malignant nodes in patients with head & neck cancer. Enlargement of supraclavicular and scalene nodes is A. Babesiosis D. . Submental C. Wolman disease C. Acute Chagas’ disease A. hepatosplenomegaly & enlarged. lymphadenopathy & hepatosplenomegaly may develop. Ratio of long to short axis (L / S ratio) D. Rupture of capsule C. Nonmovable C.5 cm) is the best size limit to differentiate malignant or granulomatous from other causes of lymphadenopathy. Rubbery D. Volume C. 138 Which of the following is a cause of supraclavicular adenopathy ? 144 Winterbottom’s sign is a classic finding of ? Harrison’s 18th Ed. grayish yellow or orange tonsils. gambiense trypanosomiasis. or Winterbottom’s sign. Posterior cervical triangle B. location of lymph node enlargement is ? 139 Benign lymphadenopathy is assumed when lymph node area is ? Harrison’s 17th Ed. Human African Trypanosomiasis D. is a classic finding Lymph nodes <1. < 0. Lymph node size of 2. discrete.0 cm 2 in area (1 x 1 cm or less) are almost always secondary to benign. 1301 140 Tenderness of lymph node is due to ? Harrison’s 18th Ed. Giardiasis B. 146 Romana’s sign is a feature of ? Harrison’s 17th Ed. 466 nodes in metastatic cancer ? A. < 1. Visceral leishmaniasis C. < 0. 1304 Harrison’s 18th Ed. Cutaneous leishmaniasis B. and consists of unilateral painless edema of palpebrae & periocular tissues when conjunctiva is the portal of entry. symmetric. Tangier disease D. All of the above D. Axilla A. RBCs traverse sinusoidal pores B. Percussion B. or Barkun methods are used for splenic ? hemoglobin . 469 A. Pulp cords are dead ends C. 978 Harrison’s 18th Ed. White pulp is lymphoid in nature & contains B cell follicles. damaged. Red pulp areas include pulp sinuses & pulp cords. Harrison’s 18th Ed. and T cell rich areas sheathing arterioles. Heinz bodies D. ballotment. < 450 grams In Nixon’s method. White pulp is lymphoid in nature A.Heinz bodies) are pinched off while passing through slits. > 4 cm A. 469 153 Weight of a normal spleen is ? Harrison’s 18th Ed. Percussion begins at lower level of pulmonary resonance in posterior axillary line & proceeds toward lower midanterior costal margin. 467 B. It is a serum sickness like syndrome with tissue mass. None of the above upper border of dullness above costal margin is ? Harrison’s 18th Ed. AJAY MATHUR Cardiology 13 148 Painful preauricular lymphadenopathy is a feature of ? 154 Which of the following is false about spleen ? Harrison’s 17th Ed. and syphilis. and is normally not palpable. diameter of 13 cm by USG or maximum length of 12 cm and/or width of 7 cm by radionuclide scan. 468 A. 149 Accessory spleens are seen in what percentage of persons ? Harrison’s 18th Ed. None of the above D. Spleen is in the portal circulation 159 In Nixon’s method. or old red cells 157 Which of the following is false in spleen examination ? cannot enter the sinuses. Auscultation neutrophils Percussion for splenic dullness is accomplished with either Nixon. red blood cells must traverse tiny openings in the sinusoidal lining. hepatosplenomegaly & high peripheral blood eosinophilia. Maximum cephalocaudad diameter is 13 cm by USG D. fever. When these hillocks fail to unify into a single Acute schistosomiasis is called Katayama fever. Dullness >8 cm in adult indicates splenic enlargement. > 8 cm B. Palpation A. sporotrichosis. None of the above D. Fever B. lies entirely within rib cage. 150 Which of the following about structure of spleen is false ? 156 Middleton maneuver is used for splenic ? Harrison’s 18th Ed. Normal spleen contains significant no. border of dullness is normally 6-8 cm above costal margin. splenic enlargement is indicated when D. a process called pitting. Upper Normal spleen weighs <250 grams. or denatured 158 Nixon. 15 % C. All of the above Red cell inclusion bodies like parasites. Bimanual palpation in right lateral decubitus position adds nothing to supine examination A. In order to regain access to the circulation. Castell. patient is in right decubitus. Stiff. 468 A. Tularemia B. accessory spleens may develop in ~20% of persons. Syphilis D. tuberculosis. Inspection 152 Which of the following is false about spleen ? Harrison’s 18th Ed. 5% A. 467 B. Cords are dead ends. > 16 cm D. marginal zone around the follicles. Normal spleen contains ~one-third of total body platelets C. 20 % D. of marginated D. C. Palpation C. < 150 grams B. 467 155 Katayama fever is characterized by all except ? Harrison’s 17th Ed. 468 Figure 59-1 Harrison’s 18th Ed. < 250 grams C. RBC parasites C. generalized lymphadenopathy. 13 Hematology MCQ’s FOR MCQ’s FOR MEDICAL MEDICAL PROFESSIONALS PROFESSIONALS BY PROF. Spleen decreases in size with age. nuclear residua (Howell-Jolly bodies. . 1333 A. Lies entirely within rib cage C. Inspection B. Hepatosplenomegaly Embryologic origin of spleen is in dorsal mesogastrium at about 5 weeks’ gestation as a series of hillocks that migrates to left upper quadrant. Lymphocytosis D. < 350 grams D. has a maximum cephalocaudad scratch disease. > 12 cm C. 468 A. 468 A. called central arteries. Howell-Jolly bodies percussion C. and palpation from above artery. Generalized lymphadenopathy C. Cat-scratch disease A. Decreases in size with age B. Auscultation Spleen comprises many units of red and white pulp centered around small branches of splenic Palpation of spleen can be done by bimanual palpation. Auscultation may reveal venous hum or friction rub 151 Which of the following is a ‘Red cell inclusion body’ ? Harrison’s 18th Ed. To return to circulation. 10 % B. Reproducibility among examiners is better for palpation than B. or Barkun technique. Harrison’s 18th Ed. a (Middleton maneuver). Percussion D. Tuberculosis C. Maximum width of 14 cm by radionuclide scan Painful preauricular lymphadenopathy is unique to tularemia and distinguishes it from cat. Castell. Basophilic stippling weighs (drained) more than 1000 grams.e. 2 weeks 166 ‘Abscopal effect’ refers to ? C. Peritoneal seeding of splenic fragments . Iatrogenic splenic rupture B. > 6 cm below left costal margin C. 161 Massive splenomegaly is defined as spleen extending ? 168 Which out of the following is a contraindication for Harrison’s 18th Ed. 471 D. Polycythemia vera Frequency of a serious infection following splenectomy is highest within first 3 years. First 3 years A. as in endometriosis. 2001:345. kala-azar. A. 471 165 Causes of Massive splenomegaly include ? A. Myelofibrosis with myeloid metaplasia D. Midaxillary line D. 682). portal hypertension. Presence of bone marrow failure D. Autoimmune hemolytic anemia 172 In elective splenectomy. Ultrafiltration of abnormal RBCs Pneumococcal vaccine (23-valent polysaccharide vaccine) should be administered to all patients B. 471 Harrison’s 18th Ed. Chronic myelogenous leukemia 170 Frequency of a serious infection following splenectomy is B. > 4 cm below left costal margin B. infiltrative diseases such as Gaucher’s disease and Niemann-Pick disease. No increased risk of viral infection A. B. patient is supine. C. presence of multiple collections of spleen tissue not connected to portal circulation. percussion in the lowest intercostal space 167 Term “splenosis” best relates to ? is done in ? Harrison’s 18th Ed. Increased susceptibility to babesiosis B. poikilocytosis. This ectopic spleen tissue may cause pain or gastrointestinal obstruction. > 750 gram A. pneumococcal vaccine should be D. 471 chronic malaria. All of the above Harrison’s 18th Ed. Ectopic spleen tissue D. > 8 cm below left costal margin D. Increased tendancy of enlarged splenic rupture D. First 6 months B. Regression of systemic disease after splenectomy 2 weeks before elective splenectomy. > 1000 gram B. Hairy cell leukemia Harrison’s 18th Ed. Hairy cell leukemia 162 Massive splenomegaly is defined as spleen that weighs ? The only contraindication to splenectomy is the presence of marrow failure. Ultrafiltration of abnormal RBCs B. D. 469 enlarged spleen is the only source of hematopoietic tissue.14 MCQ’s FOR MEDICAL PROFESSIONALS BY PROF. All of the above A. AJAY MATHUR Hematology 160 In Castell’s method. percussion is done in lowest intercostal space in anterior splenosis i. Heinz bodies Massive splenomegaly refers to spleen that extends >8 cm below left costal margin and/or C. > 2 cm below left costal margin A. peritoneal seeding of splenic fragments can lead to In Castell’s method. 4 weeks A. including A. First 5 years B. 469 A. All of the above administered how many weeks before surgery ? Causes of massive splenomegaly fall into four main categories: infectious diseases such as Harrison’s 18th Ed. 471 D. and hematologic diseases. in which the Harrison’s 18th Ed. A. Howell-Jolly bodies D. presence of Howell- Harrison’s 18th Ed. 471 Table 59-3 Jolly bodies (nuclear remnants). basophilic stippling. C. > 250 gram 169 Chronic manifestations of splenectomy include ? B. Regression of systemic disease after splenectomy A. Increased susceptibility to capsulated bacterial infections Harrison’s 18th Ed. All of the above 163 Causes of massive splenomegaly include ? Chronic manifestations of splenectomy include anisocytosis. axillary line. > 500 gram Harrison’s 18th Ed. 471 A. Iatrogenic splenic rupture C. 469 splenectomy ? Harrison’s 18th Ed. Anterior axillary line C. Posterior axillary line At times in patients with splenic rupture. Sarcoidosis D. 471 Table 59-3 B. and leishmaniasis. Heinz bodies (denatured hemoglobin). Midclavicular line B. Gaucher’s disease 171 Which of the following is false in postsplenectomy period ? D. 1 weeks myeloproliferative and lymphoproliferative disorders (N Engl J Med. 471 C. None of the above C. A dull percussion note on full inspiration suggests splenomegaly. First 1 year 164 Causes of massive splenomegaly include ? Harrison’s 18th Ed. Chronic lymphocytic leukemia C. 3 weeks Harrison’s 18th Ed. Thrombocytopenia C. 471 Table 59-3 C. Lymphomas highest within ? C. hormones required for growth of monocytes & neutrophils in bone marrow. 300 to 400 182 Which of the following azurophil granules has broad D. vitamin B 12 . Promyelocyte D. Acid hydrolases In a 80 kg person. B.3 x 10 11 neutrophils are produced by hematopoietic system per day. 472 176 Colony-stimulating factors are produced by ? A. and D. All of the above C. Vitamin B12 binding protein many days ? D. 472 C. ~ 3. 472 177 In a 80 kg person. 472 A. None of the above 181 Primary or azurophil granules contain ? Leukocytes are derived from a common stem cell in bone marrow. AJAY MATHUR Cardiology 15 B. D. 2 days Myelocyte synthesises specific or secondary granules which contains lactoferrin. 472 B. A. Myeloblast B.500 at any one time. Myeloperoxidase Harrison’s 18th Ed. Derived from common stem cell in bone marrow A. ~ 1. All of the above stimulating factors & interleukins Classic lysosomal granules called the primary or azurophil granules are found in promyelocyte. 400 to 500 antimicrobial activity against bacteria. 472 A. 472 Figure 60-2 B.3 x 1011 C. Harrison’s 18th Ed.3 x 10 11 A. myeloperoxidase. Tissue macrophages D. B lymphocyte A. All of the above B. ~ 2. Natural killer (NK) cell 179 Which of the following is the largest cell in size ? Harrison’s 18th Ed.3 x 1011 D. All of the above A. Myelocyte C. Human blood monocytes. 7 days & monocytes D. Myeloblast hematopoietic system per day ? B. ~1.3 x 1011 184 Secondary granules contain all except ? Harrison’s 18th Ed. Metamyelocyte 174 Which of the following statements about leukocytes is false ? 180 Primary or azurophil granules are present in ? Harrison’s 18th Ed. how many neutrophils are produced by A. ~ 0. Monocytes C. and basophils. cathepsin G. Myelocyte Leukocytes include neutrophils. fungi. fungi & certain Minimum number of stem cells necessary to support hematopoiesis is 400 . Monocyte B. while maturation phase from 173 Which of the following is a leukocyte ? metamyelocyte to mature neutrophil takes ~1 week. cationic proteins. Cathepsin G hematopoiesis is ? C. Leukocyte maturation in the Harrison’s 18th Ed. 100 to 200 Primary granules contain hydrolases. B. Promyelocyte Harrison’s 18th Ed. Cathepsin G A. 472 A. natural killer (NK) cells. Promyelocyte committed to leukocyte production C. Defensins C. 472 D. T and B lymphocytes. Their maturation in marrow is regulated by colony. Hydrolases 175 Minimum number of stem cells necessary to support B. D. Disorders of granulocytes C. C. Myeloperoxidase B. Myelocyte A. 472 D. eosinophils. D. All of the above certain enveloped viruses. 472 marrow is under the regulatory control of colony-stimulating factors (CSFs) & interleukins (ILs). Threefourths of nucleated cells of bone marrow are B. 10 days Proliferation phase through the metamyelocyte takes ~1 week. 472 Harrison’s 18th Ed. monocytes. elastase. Lactoferrin 178 Maturation from metamyelocyte to neutrophil takes how C. Histaminase Harrison’s 18th Ed. Hydrolases Harrison’s 18th Ed. tissue macrophages & stromal cells produce CSFs. Stromal cells Defensins in azurophil granules have a broad antimicrobial activity against bacteria. Myeloblast C. Three-fourths of nucleated cells of bone marrow are committed to production of leukocytes. 183 Specific or secondary granules are present in ? Harrison’s 18th Ed. 5 days Chapter 60. enveloped viruses ? Harrison’s 18th Ed. 200 to 300 bactericidal/permeability-increasing protein & defensins. 15 Hematology MCQ’s FOR MCQ’s FOR MEDICAL MEDICAL PROFESSIONALS PROFESSIONALS BY PROF. Phagocytosis pool is in the circulation ? Harrison’s 18th Ed. All of the above A. Harrison’s 18th Ed. 12 to 24 hours Harrison’s 18th Ed. C. & remainder in tissues. Repeated blood transfusion B. Freely flowing pool is ~half the neutrophils in basal state and is composed of those cells that are in the blood and not in contact with endothelium. n e u t r o p h i l s s p e n d h o w m u c h t i m e i n D. Majority of granulocytes are bilobed 193 Diapedesis involves which of the following ? Harrison’s 18th Ed. 2 to 3 hours B. 473 C. neutrophil 187 Which of the following is false about ‘Pelger-Huet anomaly’ ? adhesiveness increases and they “stick” to the endothelium through integrins. Excessive segmentation (>5 C. 475 Normally. A. Selectins D. ~90% of the neutrophil pool is in bone marrow. CCAAT/enhancer binding protein- B. 473 A. CCAAT/enhancer binding protein- 191 “Rolling” of the neutrophil along the endothelial surface is B. Opsonins warts. 475 D. C. Iron deficiency Harrison’s 18th Ed. Spleen Normally. margination occurs because the capillaries are about the same size as a mature neutrophil. A. Thalassemia A. Vascular endothelial growth factor (VEGF) D. Integrins are leukocyte glycoproteins that bind to specific endothelial receptors [intercellular adhesion Harrison’s 18th Ed. Diapedesis involves platelet/endothelial cell adhesion molecule (PECAM) 1 (CD31) expressed on emigrating leukocyte & endothelial cells. 474 C. Marginated leukocytes are those 185 Packaging of secondary granule contents during that are in close physical contact with endothelium. Migration through mucous membrane of GI tract B. 24 to 36 hours B. Circulating pool of neutrophils exists in two dynamic compartments: one freely flowing and one marginated. Intercellular adhesion molecules D. Secondary granules do not contain acid hydrolases. ~ 70 % Harrison’s 18th Ed. ~ 90 % circulation ? Harrison’s 18th Ed. Apoptosis 189 Under normal conditions.16 MCQ’s FOR MEDICAL PROFESSIONALS BY PROF. ~ 90 % A. Anaphylatoxins Pelger-Hüet anomaly is a benign inherited disorder. ~ 20 % B. ~2% In the healthy adult. 473 C. 473 Figure 60-5 molecules (ICAM) 1 & 2]. Nucleus has a spectacle-like. Folate or vitamin B12 deficiency 192 Neutrophils “stick” to the endothelium through ? B. CCAAT/enhancer binding protein- A. neutrophils spend a short time in circulation (half-life. membrane components of reduced nicotinamide-adenine dinucleotide phosphate (NADPH) oxidase. AJAY MATHUR Hematology binding protein. Lungs D. Benign disorder B. A. ~ 20 % C. and laminin. Kidneys . C. In response to chemotactic stimuli from injured tissues or bacterial products. hypogammaglobulinemia. None of the above B. what proportion of neutrophil Process of migration into tissues is called diapedesis and involves crawling of neutrophils pool is in the bone marrow ? between postcapillary endothelial cells that open junctions between adjacent cells to permit Harrison’s 18th Ed. B. Due to extensive capillary bed (~1000 myelopoiesis is controlled by ? capillaries per alveolus) in pulmonary circulation. Nucleus has spectacle-like configuration A. ~ 50 % 195 N o r m a l l y. 473 leukocyte passage. Liver D. 473 A. 2 . histaminase. Opsonins 186 Excessive segmentation of nucleus of neutrophils is a manifestation of ? Selectins are glycoproteins expressed on neutrophils & endothelial cells and cause a low- affinity interaction resulting in “rolling” of neutrophil along the endothelial surface. Integrins Nucleus of neutrophils normally contains up to four segments. Harrison’s 18th Ed. 473 D. ~ 50 % 194 In the healthy adult. most neutrophils leave the body by migration through the mucous membrane of the gastrointestinal tract. 6 . Selectins D. most neutrophils leave the body by ? C. Majority of granulocytes are bilobed (hyposegmented neutrophils). CCAAT/enhancer binding protein- a function of ? Harrison’s 18th Ed. what proportion of neutrophil C. PECAM 1 D. Intercellular adhesion molecules nuclear lobes) is seen in folate or vitamin B 12 deficiency & congenital neutropenia syndrome of D.7 hours). Prostaglandins E & I 188 Under normal conditions. 6 to 7 hours 190 Marginated leukocytes are maximum in ? C. infections & myelokathexis (WHIM). Integrins Packaging of secondary granule contents during myelopoiesis is controlled by CCAAT/enhancer binding protein- .3% in circulation (freely flowing & marginated). 472 A. or “pince-nez” configuration. CXC. Shwachman-Diamond syndrome. Shwachman-Diamond syndrome C. Congenital forms of neutropenia include Kostmann’s syndrome. 2000 cells/µL B. Hemodialysis lymphocytes. < 1000 cells / µL D. All of the above Harrison’s 18th Ed. PKD-1 gene C. All of the above Senescent neutrophils are cleared from the circulation by macrophages in the lung and spleen. All of the above D. Clozapine C. & T cells. and CXXXC. CXC Harrison’s 18th Ed. A. . Depressed production A. 476 A. CCC peripheral pooling (Transient neutropenia) ? D. Elastase neutropenia due to peripheral destruction ? C. < 200 cells / µL B. Kostmann’s syndrome B. Antithyroid drugs D. sulfonamides). some phenothiazines. tranquilizers 197 Characteristic green color to pus is due to ? (meprobamate). mercurial diuretics. A. Kidneys A. when absolute neutrophil count (ANC) falls below ? 206 Kostmann’s syndrome is due to mutations in ? Harrison’s 18th Ed. antipsychotics (clozapine). antithyroid drugs. Mercurial diuretics 198 Which of the following is not a group of chemokines ? D. Increased peripheral destruction B. WHIM syndrome and hereditary cyclic neutropenia. CC chemokines (MIP-1) attract B. C. Potassium voltage-gated channel gene KCNA1 B. cartilage-hair hypoplasia syndrome. CXXXC chemokine (fractalkine) attracts neutrophils. Phenylbutazone inflammatory process by inactivating chemoattractants and immobilizing phagocytic cells. C. WHIM syndrome D. inflammatory process is absent. < 100 cells / µL A. 4000 cells/µL D. Kostmann’s syndrome B. cyclophosphamide). All of the above Neutropenia can be due to depressed production. 475 Harrison’s 18th Ed. anticonvulsants (carbamazepine). AJAY MATHUR Cardiology 17 196 Senescent neutrophils are cleared from the circulation by 202 Which of the following drugs cause neutropenia due to macrophages in ? decreased production ? Harrison’s 18th Ed. Collagenase 203 Which of the following drugs act as haptens to cause B. 476 Harrison’s 18th Ed. GJB2 gene When absolute neutrophil count (band forms & mature neutrophils combined) is <200/µL. 476 Table 60–1 D. mutations in the Shwachman-Bodian-Diamond syndrome gene SBDS. C chemokine (lymphotactin) is T cell tropic. 476 Table 60–1 A. CC. 5- flucytosine). chlorambucil. & basophils. CXC cytokines (IL-8) attract neutrophils. Shwachman-Diamond syndrome C. WHIM syndrome D. phenothiazines. 476 Table 60–1 A. Harrison’s 18th Ed. Anti-apoptosis gene HAX-1 D. 475 Drugs that act as haptens to cause neutropenia due to peripheral destruction include aminopyrine. Excessive peripheral pooling C. 475 anti-inflammatory agents. GI tract C. 17 Hematology MCQ’s FOR MCQ’s FOR MEDICAL MEDICAL PROFESSIONALS PROFESSIONALS BY PROF. Lung B. eosinophils. C alpha methyldopa. All of the above Susceptibility to infectious diseases increases sharply when neutrophil counts fall <1000 cells/µL. certain diuretics. antibiotics (chloramphenicol. monocytes. Thymus D. B. Cardiopulmonary bypass D. All of the above A. monocytes. All of the above 199 Susceptibility to infectious diseases increases sharply when neutrophil counts fall below ? 205 Congenital forms of neutropenia include ? Harrison’s 18th Ed. Myeloperoxidase Harrison’s 18th Ed. penicillins. 200 Inflammatory process is absent. 476 Harrison’s 18th Ed. < 500 cells / µL C. antimetabolites (methotrexate. Alpha methyldopa Myeloperoxidase confers the characteristic green color to pus and may participate in turning off the B. Overwhelming bacterial infection (acute endotoxemia) Four major groups of chemokines are recognized based on cysteine structure near N terminus: C. increased peripheral destruction. CC 204 Which of the following is a cause of neutropenia due to C. the Kostmann’s syndrome is due to mutations in anti-apoptosis gene HAX-1. drugs cause neutropenia due to decreased production include alkylating agents (nitrogen mustard. phenylbutazone. 207 Which of the following is associated with pancreatic 201 Neutropenia can be due to ? insufficiency ? Harrison’s 18th Ed. busulfan. Carbamazepine B. 1000 cells/µL A. or excessive Shwachman-Diamond syndrome is associated with pancreatic insufficiency and is due to peripheral pooling. 476 A. 3000 cells/µL C. 476 A. 476 Harrison’s 18th Ed. 6-mercaptopurine. Increased susceptibility to infection C. while large granular Patients with LAD1 have mutations in CD18. Defects in lysosomal transport protein LYST D. encoded by the gene CHS1 at 1q42. Hepatomegaly A. granulocyte colony- stimulating factor (G-CSF) can cause neutrophilia. None of the above Harrison’s 18th Ed. All of the above D. Defective margination C. CD18 C. 477 Harrison’s 17th Ed. blue-staining nongranular areas found in periphery of cytoplasm of neutrophil in infections & toxic states. Cluster determinant C. A.50. Abnormal packaging & disbursement of granules LAD 1 & 2 are autosomal recessive traits leading to inability of neutrophils to exit circulation to sites of infection. Aggregates of rough endoplasmic reticulum 212 Which of the following drugs can cause neutrophilia ? D. 477 glycosylation IIc (CDGIIc)” ? Harrison’s 18th Ed. 476 mutations in ? A. leukocytosis & increased susceptibility to infection. Autosomal dominant traits syndrome (CHS) ? Harrison’s 18th Ed.000/µL is called a leukemoid reaction. Leukocytosis B. LAD 2 is also known as congenital disorder of glycosylation IIc (CDGIIc) due to mutation in a GDP-fucose transporter (SLC35C1). 477 granules). Increased marrow release B. Circulating neutrophils are mature 217 Which of the following is false about ‘Dohle body’ ? Harrison’s 18th Ed. lymphocytes (LGL) can attack marrow neutrophil precursors. Lithium C. 477 B. In leukemoid reaction. Dohle bodies C. Increased neutrophil production A. > 75000 to 100000 / µL D. Discrete. B. 210 Persistent neutrophilia with what level of raised cell counts 216 In neutrophils. splenomegaly & neutropenia form the triad of Felty’s syndrome. Cell determinant (LAD) 1 & 2 is false ? Harrison’s 18th Ed. Leukocyte adhesion deficiency 3 (LAD 3) D. 215 Which of the following is called “Congenital disorder of 209 Neutrophilia results from ? Harrison’s 18th Ed. All of the above 211 Which of the following is false about leukemoid reaction ? In severe acute bacterial infection toxic granulations (immature or abnormally staining azurophil Harrison’s 18th Ed. 350 D. CD62 Rheumatoid arthritis. They are aggregates of rough endoplasmic reticulum. 473 Figure Figure 60-3 C. > 30000 to 50000 / µL B. > 10000 to 30000 / µL A. B. Meprobamate Döhle bodies are discrete. Neutrophil cell count > 30000 to 50000 / µL B. 377 A. 477 Table 60–2 A. None of the above Neutrophilia is due to increased neutrophil production & marrow release or defective margination. Inability of neutrophils to exit circulation A.18 MCQ’s FOR MEDICAL PROFESSIONALS BY PROF. glucocorticoids. Leukocyte adhesion deficiency 2 (LAD 2) C. CD31 D. 477 219 Which of the following is false about Chédiak-Higashi A. blue-staining nongranular areas found in D. Neutropenia C. which of the following is seen in severe acute is called leukemoid reaction ? bacterial infections ? Harrison’s 18th Ed. Toxic granulations B. lithium. “CD” stands for ? Harrison’s 16th Ed. None of the above periphery of cytoplasm of neutrophils Persistent neutrophilia with cell counts of 30. > 50000 to 75000 / µL C. None of the above CHS is a systemic disease with autosomal recessive inheritance due to defects in the lysosomal transport protein LYST required for normal packaging and disbursement of granules. nonsteroidal anti-inflammatory agents. Autosomal recessive inheritance C. Clonal determinant Epinephrine. B. Leukocyte adhesion deficiency 1 (LAD 1) B. 479 A. CD34 D. Rheumatoid arthritis Harrison’s 18th Ed. . Neutropenia is due to antibodies produced by spleen that shorten neutrophil life span. Circulating neutrophils are clonally derived A. AJAY MATHUR Hematology 208 Felty’s syndrome include all except ? 214 Patients with leukocyte adhesion deficiency 1 (LAD 1) have Harrison’s 18th Ed. CD18 gene is located on distal chromosome 21q. 479 B. . Characterized by the presence of giant lysosomes within leukocytes. Phenothiazines 218 For lymphocytes. Phenylbutazone A. Capsule determinant 213 Which of the following about leukocyte adhesion deficiency D.000 . circulating neutrophils are usually mature & not clonally derived. Splenomegaly B. Döhle bodies (cytoplasmic inclusions which are fragments of ribosome-rich endopla- smic reticulum) & large neutrophil vacuoles (pinocytosed or internalized membrane) are seen. D. Found in infections and other toxic states C. Large neutrophil vacuoles D. It plays significant role in the pathogenesis of gram-negative shock. 6 to 7 hours A. All of the above C. promonocytes. TNF-alpha B. All of the above Harrison’s 18th Ed. A. Acute infections D. Subacute bacterial endocarditis 224 Monocytes have a half-life in the blood of ? D. malaria. Harrison’s 18th Ed. AJAY MATHUR Cardiology 19 220 Patients with Chédiak-Higashi syndrome (CHS) may have B. 19 Hematology MCQ’s FOR MCQ’s FOR MEDICAL MEDICAL PROFESSIONALS PROFESSIONALS BY PROF. 480 chronic idiopathic neutropenias & granulomatous diseases like sarcoidosis & regional enteritis. All of the above A. 481 C. 480 Gain-of-function mutations in TNF- receptor cause TNF- receptor-associated periodic syndrome A. IL-1 A. All of the above A. Activating lymphocytes & neutrophils Harrison’s 18th Ed. 480 A. SABE. Tuberculosis Mononuclear phagocyte system is composed of monoblasts. Vitamin A. and an increased frequency of infections. All of the above Harrison’s 18th Ed. Malaria C. T receptor associated periodic syndromes Harrison’s 18th Ed. 480 B. C. IL-12 221 Leukocytes from patients with chronic granulomatous D. 12 to 24 hours B. None of the above B. Tumour associated periodic syndromes D. Acute myeloid leukemia D. Neutrophils B. 479 TNF. partial oculocutaneous albinism.IL-1 is ? D. C. is a pyrogen that duplicates many actions of IL-1. Seizure Functions of IL-1 include initiating fever in hypothalamus. Stress 223 Mononuclear phagocyte system is composed of ? D. D. Macrophages A. C. Tremors 227  duplicates the function of which of the following ? TNF- Harrison’s 18th Ed. They differentiate from blood monocytes that arrive in tissues. stress. B. All of the above Harrison’s 18th Ed. Rocky Mountain spotted fever. C. Monocytes 232 “TRAPS” stands for ? Harrison’s 18th Ed. TNF- receptor associated periodic syndromes Macrophages are called “big eaters”. 481 B.24 hours. Blindness marrow and activating lymphocytes & neutrophils. All of the above 225 “Big eaters” is the term used for ? Monocytosis is associated with TB. A. 480 A. IL-8 can cure patients of CHS. Monocytes Harrison’s 18th Ed. Glucocorticoid therapy Leukocytes from patients with CGD have severely diminished hydrogen peroxide production. Immune thrombocytopenic purpura (ITP) 229 Monocytopenia occurs with ? Harrison’s 18th Ed. Promonocytes 230 Monocytosis is associated with ? C. Thyroid associated periodic syndromes 226 Function of macrophage secreted product . . leukemias. hairy A. IL-18 disease (CGD) have severely diminished production of ? Harrison’s 18th Ed. 1 to 3 hours 231 Monocytosis is associated with ? Harrison’s 18th Ed. C. monocytes and B. mobilizing leukocytes from bone B. acute myeloid leukemia & use of myelotoxic drugs. brucellosis. Hemolytic anemias Monocytes have a half-life in blood of 12 . hemolytic anemias. Juvenile rheumatoid arthritis A. aplastic anemia. Monoblasts cell leukemia. Hydrogen peroxide B. Patients may develop a severe disabling peripheral neuropathy in adulthood. Discoid lupus B. Hairy cell leukemia C. Nystagmus D. 480 D. 479 D. Nitric oxide 228 Monocytopenia occurs with ? Harrison’s 18th Ed. Initiating fever in hypothalamus (TRAPS) characterized by recurrent fever without infection (persistent stimulation of TNF- receptor). Visceral leishmaniasis (kala azar) D. Hematopoietic cell transplantation B. visceral leishmaniasis (kala azar). 24 to 36 hours C. 480 Monocytopenia occurs with acute infections. glucocorticoid use. Aplastic anemia 222 Frequency of which of the following is increased in CGD ? D. IL-1 C supplementation is useful in Chédiak-Higashi syndrome. 480 C. Mobilizing leukocytes from bone marrow which of the following ? C. myeloproliferative syndromes. 480 Patients with CHS have nystagmus. Brucellosis tissue macrophages. allergies (hay fever. Familial cold urticaria D. 481 host defense in invasive helminthic infections. tissue eosinophils can 245 Which of the following is the dominant eosinophil growth factor ? recirculate. pancreas. All of the above 243 Eosinophilia is associated with which of the following diseases ? TNF. eczema. Eczema Harrison’s 18th Ed. Muckle-Wells syndrome C. A. eosinophils are not important penicillins & cephalosporins). All of the above B. and sterile pyogenic arthritis (PAPA syndrome) 242 Eosinophilia is due to allergic reaction which of the following is caused by mutations in CD2BP1. > 400 C. familial cold urticaria. eosinophils are not important but plays a central role in Harrison’s 18th Ed. Harrison’s 18th Ed. Pemphigus B. 481 235 Mutations in CD2BP1 cause which of the following ? A. Cancer uterus A. 481 A. CML. Cancer pancreas 238 Which of the following is false about eosinophils ? B. > 200 Harrison’s 18th Ed. Cancer ovary Harrison’s 18th Ed. Shorter half life than neutrophils D. 240 Charcot-Leyden crystal protein in eosinophil cytoplasm is ? 234 Mutations in cold-induced autoinflammatory syndrome 1 Harrison’s 18th Ed. A. AJAY MATHUR Hematology 233 Familial Mediterranean fever due to mutations in PYRIN is 239 Which of the following is false about eosinophil granule ? due to abnormal regulation of ? Harrison’s 18th Ed. serum sickness. Eotaxin C. allergic angiitis D. Hyaluronidase Harrison’s 18th Ed.20 MCQ’s FOR MEDICAL PROFESSIONALS BY PROF. certolizumab. C. Phospholipase A. Neonatal-onset multisystem autoinflammatory disease C. Serum sickness 237 Specific chemokine expressed by eosinophils is ? B. 481 A. eosinophilic fasciitis. 481 C. Unlike neutrophils. stomach. uterus). During most infections. allergic vasculitis. and etanercept. mycosis fungoides. Central role in defense against invasive helminthic & PAN). and helminthic infections. antagonists are infliximab. 481 A. IL-18 Eosinophil granule contains arginine-rich protein in its crystalline core which has histaminase Diseases with abnormal IL-1 regulation leading to fever include familial Mediterranean fever activity. 481 Eosinophils express a specific chemokine EOTAXIN. > 500 D. and Muckle-Wells syndrome. Iodides A. 481 Harrison’s 18th Ed. Muckle-Wells syndrome Eosinophil cytoplasm contains Charcot-Leyden crystal protein which is a lysophospholipase. All of the above malignancies ? Harrison’s 18th Ed. Arginine-rich protein content A. 480 B. IL-1 B. Histaminase activity B. 481 (CIAS1) lead to ? A. Eosinotaxin D. Eosinophilotaxin 244 Eosinophilia is associated with which of the following D. Adalimumab C. asthma. malignancies (Hodgkin’s disease. 481 A. D. blood ? Harrison’s 18th Ed. drugs ? Harrison’s 18th Ed. The syndrome of pyoderma gangrenosum. Infliximab B. None of the above D. collagen vascular diseases (RA. PAPA syndrome B. eosinophil peroxidase that catalyzes oxidation by hydrogen peroxide. Lysophospholipase B. Contain eosinophil peroxidase C. Etanercept D. Eosinophils have a longer half life than neutrophils. Familial Mediterranean fever Eosinophilia is the presence of >500 eosinophils per µL of blood. IL-1 . & pemphigus). During most infections. IL-12 D. Nitrofurantoin C. due to mutations in PYRIN. acne. IL-8 C. All of the above Mutations in cold-induced autoinflammatory syndrome 1 (CIAS1) lead to neonatal-onset 241 Eosinophilia refers to how many eosinophils per microliter of multisystem autoinflammatory disease. > 300 B. Ca. All of the above C. sulfonamides. lung. Eosinophils can recirculate Eosinophilia occurs in allergic reaction to drugs (iodides. Familial cold urticaria D. adalimumab. Aspirin B. Immunoglobulin E C. aspirin. 480 A. infections ovary. All of the above D. Job’s syndrome & CGD. 481 236  antagonist ? Which of the following is a TNF- Harrison’s 18th Ed. nitrofurantoin. Hyperimmunoglobulin E . Eosinophil count > 1000/µL Chapter 103. 254 Most common anemia among hypoproliferative anemias is ? rized by typical facies with broad nose. Glucocorticoids C. None of the above Harrison’s 18th Ed. D. None of the above C.recurrent infection syndrome is also called Job’s syndrome. Endotoxin challenge test D. Also called Job’s syndrome D. IL-5 C. A. None of the above There is no known adverse effect of eosinopenia. Nitroblue tetrazolium (NBT) dye test Iron absorbed from the diet or released from stores circulates in the plasma bound to transferrin. 482 A. Apart from recombinant G-CSF. C. 482 C. Dihydrorhodamine (DHR) oxidation test D.60 minutes in neutropenia due to impaired production ? D. Obstructive lung disease marrow damage. Epinephrine challenge test is for marginated circulating pool of cells & endotoxin challenge test is for their marginating ability. 844 In vivo assessment of inflammation is done by Rebuck skin window test or skin blister assay.5 B. Primary teeth erupt normally but do not deciduate requiring extraction. Anemias from marrow damage A. osteoporosis & eczema.90 minutes. Normocytic RBC’s 247 Which of the following is an adverse effect of eosinopenia ? Harrison’s 18th Ed. 844 Assessment of bone marrow reserves of WBC’s is done by steroid challenge test. 482 C. C. the iron transport protein. 844 A. glucocorticoids. NBT & DHR tests are for detecting deficiencies of oxidative metabolism. lithium & immunosuppressive therapy are used to restore myelopoiesis in patients with neutropenia due to impaired production. which measures the ability of leukocytes & inflammatory mediators to accumulate locally in A.0 . . All of the above B. Anemia of acute and chronic inflammation E–recurrent infection (HIE) syndrome ? B. kyphoscoliosis.10 minutes skin. Patients develop recurrent sinopulmonary Harrison’s 18th Ed. All of the above 255 Which of the following is called iron transport protein ? Harrison’s 18th Ed. hypometabolic states (protein malnutrition & endocrine deficiencies). Pulmonary fibrosis Anemias associated with normocytic and normochromic red cells and an inappropriately low reticulocyte response (reticulocyte index < 2. Myopathy D. characte. All of the above 256 Turnover or half-clearance time of transferrin-bound iron is ? Harrison’s 18th Ed. acute and chronic inflammation. Kyphoscoliosis Hypoproliferative anemias include early iron deficiency. Anemias associated with cancer Harrison’s 18th Ed. Responds to glucocorticoids 252 Which of the following is false about hypoproliferative anemias ? D. 60 . Transferrin 250 In vivo assessment of inflammation is done by ? Harrison’s 18th Ed. IL-3 B. Caused by ingesting contaminants in L-tryptophan Hypoproliferative Anemias containing products C. AJAY MATHUR Cardiology 21 B. 253 Which of the following is a hypoproliferative anemia ? Harrison’s 18th Ed. Rebuck skin window test D. 30 .5) are hypoproliferative anemias. Ferritin B. 5 . Reticulocyte index < 2.2.2. 246 Which of the following is false about eosinophilia-myalgia syndrome ? Harrison’s 18th Ed. A. Anemias associated with renal disease 249 Which of the following is performed to assess bone marrow B. All of the above D. All of the above B. B. 844 248 Which of the following is false about hyperimmunoglobulin A. Androgens Turnover or half-clearance time of transferrin-bound iron is 60 .30 minutes 251 Which of the following drugs is useful to restore myelopoiesis C. Cardiac arrhythmias C. ‘Cold’ skin abscesses C. All these are characterized by an abnormal erythropoietin response to the anemia. 10 .0 . androgens. Lithium D. Anemias associated with chronic inflammation reserves ? C. 482 D. Steroid challenge test anemia associated with chronic inflammation is the most common of the above mentioned B. Normochromic RBC’s A. and anemias from D. Divalent metal transporter 1 (DMT1) A. 21 Hematology MCQ’s FOR MCQ’s FOR MEDICAL MEDICAL PROFESSIONALS PROFESSIONALS BY PROF. renal disease. All of the above IL-5 is the dominant eosinophil growth factor.90 minutes Harrison’s 18th Ed. Iron Deficiency and Other B. Anemia of hypometabolic states Harrison’s 18th Ed. 481 A. 844 & cutaneous infections much less inflamed than expected (“cold abscesses”). Epinephrine challenge test hypoproliferative anemias. 482 B. Anemias associated with hypometabolic states A. Hephaestin C. 5 to 6 mg 266 Iron transport across the enteric absorptive cell membrane During the last two trimesters of pregnancy. Duodenal cytochrome B (Dcytb) B. 2 mg B.8 . 0. 844 A. while females in childbearing years border membrane of duodenal enterocytes is done by ? need 1.4 % A.8 % C. 1 mg A. Duodenal hepcidin requirements increase to ? B. Clathrin-coated pits D. 2 to 3 mg D. Diferric transferrin C. excess iron binds to which of the following to plasma transferrin through ? to form ferritin ? Harrison’s 18th Ed. Duodenal cytochrome B (Dcytb) B. Proximal small intestine A. 845 A.6 % B. Thus. Harrison’s 18th Ed.0 % D.22 MCQ’s FOR MEDICAL PROFESSIONALS BY PROF. Stomach Harrison’s 18th Ed. 0. Duodenal cytochrome B (Dcytb) A. 845 C. average RBC life span is 120 days.a ferrireductase. excess iron binds to Apoferritin to form ferritin. Apotransferrin D. 845 transferrin binding at the basolateral surface of gut cell ? Harrison’s 18th Ed. Diferric transferrin has the highest affinity for transferrin receptors. Duodenal ferroportin Harrison’s 18th Ed. Divalent metal transporter 1 (DMT-1) D. Harrison’s 17th Ed. 845 Harrison’s 18th Ed. 845 A. Hephaestin B.4 mg/day. Divalent metal transporter 1 (DMT-1) Normally.6 mg/day. 845 260 Transferrin-receptor complex is internalized via ? A. Monoferric transferrin B. 844 Figure 103-1 B. Transferritin C.6 to 0. 264 Iron absorption takes place largely in ? Apotransferrin does not carry iron and has very little affinity for transferrin receptors. Apoferritin B. D.4 to 0. 845 258 Daily requirement of dietary iron in adult man is ? A. 4 to 5 mg reduction of ferric (Fe3+) to ferrous (Fe2+) iron by duodenal cytochrome B (Dcytb) . 3 to 4 mg Dietary inorganic iron traverses brush border membrane of duodenal enterocytes via DMT1 after C.2 to 0. 267 Iron in gut cell is transported through its basolateral surface 261 In erythroid cell. Divalent metal transporter 1 (DMT-1) D. Megalin-coated pits C. Ferroportin (FPN) C. 3163 Figure 357-1 259 During the last two trimesters of pregnancy.8 to 1. 845 A. an adult male needs ~1 mg of elemental iron daily. D. 1 mg C. 0. Ferroportin C. is accomplished by ? Harrison’s 18th Ed. AJAY MATHUR Hematology 257 Which of the following has the highest affinity for transferrin 263 Each milliliter of red cells contain how much elemental iron ? receptors ? Harrison’s 18th Ed. Etharin-coated pits B. DMT-1 is a general cation transporter and is also known as natural resistance Transferrin-receptor complex is internalized via clathrin-coated pits and transported to an acidic macrophage-associated protein type 2 (Nramp 2) or DCT-1.which oxidizes iron to ferric form for transferrin binding. Iron moves from enterocyte into circulation via a process requiring basolateral iron exporter ferroportin (FPN) & iron oxidase hephaestin (Heph) . Iron-transferrin complex in plasma interacts with specific transferrin receptors on marrow erythroid cell surface.1. Duodenal hephaestin A. through membrane-embedded iron exporter. Distal small intestine B. where iron is released at a low pH. All of the above Each milliliter of red cells contains 1 mg of elemental iron. Coferritin D. 2 mg D. daily iron A. daily iron requirements increase to 5 . endosome. Hephaestin B. 0. Endoferritin In gut cell. 845 Harrison’s 18th Ed. ferroportin (FPN). 0. Duodenal cytochrome B (Dcytb) Harrison’s 18th Ed. 4 mg 265 Reduction of ferric (Fe 3+ ) to ferrous (Fe 2+ ) iron at brush Normally. Azalin-coated pits Iron transport across enteric absorptive cell membrane is achieved by Divalent metal transporter 1 (DMT-1). Large intestine C. . 3 mg Iron absorption takes place largely in proximal small intestinethrough a carefully regulated process. 3 mg C. 4 mg D.0 % of red cells turn over each day. Ferroportin D. iron may be stored as ferritin or released at basolateral surface to plasma transferrin In erythroid cell. 262 What proportion of red cells turn over each day ? 268 Which of the following oxidizes iron to ferric form for Harrison’s 18th Ed. Duodenum D. Mutations in the genes encoding HFE. All of the above D. Cheilosis Usual signs of iron deficiency anemia are fatigue. Ferritin Harrison’s 18th Ed. Ferritin D. Erythropoietin B. Peripheral blood smear reveals the first appearance of microcytic cells and hypochromic reticulocytes in circulation. Bone marrow B. Pallor C. When transferrin saturation is 10 . iron deficiency in adult male means gastrointestinal blood loss until proved otherwise. 845 in circulation in which of the following stages ? A. TfR2 & Aa a rule. hemoglobin & hematocrit begin to fall. 849 A. Cheilosis Bone marrow iron stores are absent when the serum ferritin level is <15 µg/L. and hepcidin lead to decreased hepcidin release and increased iron absorption. This is a Hepcidin is a 25-amino-acid peptide made by the liver. D. 10 to 40 µg / dL A. Fatigue B. Inadequate iron in diet B. < 25 µg/L B. TfR2 (transferrin receptor 2) B. (fissures at corners of mouth) & koilonychia are signs of advanced tissue iron deficiency. Iron-deficient erythropoiesis D. pallor & reduced exercise capacity. Negative iron balance C. < 35 µg/L C. Hyperchromic reticulocytes in circulation C. 846 Harrison’s 18th Ed. 20 to 25 % A. 23 Hematology MCQ’s FOR MCQ’s FOR MEDICAL MEDICAL PROFESSIONALS PROFESSIONALS BY PROF. All of the above Hepcidin responds to changes in body iron requirements by signals mediated by HFE. 846 A. It is a crucial molecule in iron metabolism. Acute inflammation Hepcidin (principal iron regulatory hormone) is involved in regulation of iron uptake by enterocytes B. 845 C.15%. Blood malignancy 273 Hepcidin responds to signals mediated by ? 279 Iron deficiency in adult male usually means ? Harrison’s 17th Ed. HFE A. hemoglobin synthesis is impaired. 847 Harrison’s 18th Ed. Gastrointestinal blood loss release from macrophages & serves as a central regulator of body iron traffic. Inadequate iron absorption C. Iron-deficiency anemia 272 Hepcidin principally acts on which of the following ? D. Transferrin saturation between 15 to 20 % D. reflecting iron-deficiency anemia. Reduced exercise capacity D. Impaired hemoglobin synthesis A. Ceruloplasmin Hephaestin is similar to ceruloplasmin. Erythropoietin Once the transferrin saturation falls to 15 . 276 Which of the following about iron-deficient erythropoiesis is false ? 270 Hepcidin is derived from ? Harrison’s 18th Ed. 846 B. 15 to 20 % Harrison’s 18th Ed. All of the above Harrison’s 18th Ed. 25 to 30 % B. 271 Which of the following is called “iron regulatory hormone” ? 277 Microcytic RBC’s & hypochromic reticulocytes first appear Harrison’s 18th Ed. hemojuvelin. Liver C. TfR2. resulting in hemochromatosis. 280 Which of the following is a sign of advanced tissue iron 274 Bone marrow iron stores are absent when serum ferritin level is ? deficiency ? Harrison’s 18th Ed. 845 Hb synthesis is impaired in iron-deficient erythropoiesis and PBF shows microcytic RBCs and A. 846 A. the copper-carrying protein. Chronic inflammation & iron release by RE cells. Spleen When transferrin saturation is between 15 to 20%. C-reactive protein is a period of iron-deficient erythropoiesis. hemojuvelin (HJV). Divalent metal transporter 1 (DMT-1) A. AJAY MATHUR Cardiology 23 269 Hephaestin is similar to which of the following ? B. Ferroportin Harrison’s 18th Ed. 10 to 15 % B. D. Hemojuvelin (HJV) C. 846 Harrison’s 18th Ed. period of iron-deficient erythropoiesis. B. 846 A. This C. Hepcidin C. hemoglobin synthesis becomes impaired. linking body stores with intestinal iron absorption. 20 to 80 µg / dL . Hephaestin 278 Appearance of iron deficiency in an adult male means ? C. < 45 µg/L D. 846 D. 275 Hemoglobin synthesis is impaired when transferrin saturation 281 Normal range for serum iron is ? falls to ? Harrison’s 18th Ed. Gastrointestinal blood loss D. Hepcidin represses ferroportin at basolateral surface as well as iron C. Microcytic RBCs first appear in PBF B.20%. 3163 Figure 357-1 Harrison’s 18th Ed. Duodenal cytochrome B (Dcytb) hypochromic reticulocytes. < 15 µg/L A. Transferrin A. 45 to 70 % A. Cell membrane 9 Harrison’s 18th Ed. Defective erythroblasts d D. Iron accumulates in mitochondria D. < 30 µg/dL A. Endoplasmic reticulum i C. 847 A. Circulating iron bound to transferrin Normally. 288 Adult females have serum ferritin values averaging about ? Harrison’s 18th Ed. Normal value for adult males & females average ~100 and 30 µg/L respectively. 10 % C. C. AJAY MATHUR Hematology C. Most common causes of increased red cell protoporphyrin levels are absolute or relative iron deficiency and lead poisoning. Developing erythroblasts B. Defective myeloblasts 284 Which of the following is the formula for calculating transferrin saturation ? 291 Normal percentage of sideroblasts in bone marrow is ? ti e Harrison’s 18th Ed. 400 µg / L Normal value of red cell protoporphyrin is < 30 µg/dL. All of the above G B. 300 to 360 µg / dL 289 Sideroblasts have granules consisting of ? C. 10 % D. < 120 µg/dL D.40% of developing erythroblasts called C. - 292 In ‘ringed sideroblasts’. 847 287 Adult males have serum ferritin values averaging about ? Harrison’s 18th Ed. Circulating free iron D. 14 % 293 Sideroblastic anemia usually points to the diagnosis of ? h D. 847 295 Which of the following is false about transferrin receptor protein (TRP) ? A.360 µg/dL. Circulating free iron + iron bound to transferrin sideroblasts are visible with ferritin granules in their cytoplasm. 300 µg / L D. 10 µg / L Harrison’s 18th Ed. 847 R D. V Transferrin saturation is normally 25 . 40 % D. 847 D. All of the above B. 750 to 1050 µg / dL A. 12 to 18 % D.24 MCQ’s FOR MEDICAL PROFESSIONALS BY PROF. Ringed sideroblasts are seen in myelodysplastic syndromes due to mitochondrial dysfunction. 847 Harrison’s 18th Ed. 50 to 150 µg / dL C. 40 µg / L 282 Normal range for TIBC is ? Harrison’s 18th Ed. 847 C. 25 to 50 % Harrison’s 18th Ed. 15 to 28 % 290 Sideroblasts are ? C. Nucleus A. 180 to 250 µg / dL B. A. 450 to 700 µg / dL Harrison’s 18th Ed. (Serum iron ÷ 100) x TIBC C. Ferritin B.150 µg/dL. Serum iron level represents amount of circulating iron bound to transferrin. 30 µg / L D. (TIBC x 100) ÷ Serum iron n B. 847 B. 75 % Transferrin saturation is calculated by as serum iron x 100 ÷ TIBC. 847 . (Serum iron x 100) ÷ TIBC A. in a bone marrow smear stained for iron. 847 Serum ferritin level correlates with total body iron stores. the accumulation of iron is around ? 285 Iron deficiency states is present when transferrin saturation Harrison’s 18th Ed. Developing myeloblasts Normal range for serum iron is 50 . Pernicious anemia A. A. 294 Normal value of red cell protoporphyrin is ? Harrison’s 18th Ed. 847 A. 5% B. 100 µg / L B. 20 % Harrison’s 18th Ed. < 90 µg/dL C. 847 is below ? A.50%. (TIBC ÷ 100) x Serum iron U D. Aplastic anemia 286 Serum iron level represents the amount of ? B. 30 to 100 µg / dL B. 847 A. Glycogen Harrison’s 18th Ed. Transferrin 283 Normal Transferrin saturation is ? C. 200 µg / L C. Normal range for TIBC is 300 . Any of the above in a necklace fashion around the nucleus of erythroblast. 847 ta Iron-deficiency is associated with transferrin saturation levels < 20 %. < 60 µg/dL B. 20 . Myelodysplasia Harrison’s 18th Ed. 6% 9 C. 20 µg / L D. Mitochondria r B. None of the above D.14 days Harrison’s 18th Ed. Weight (kg) x 0. reticulocyte count begin to increase ? Harrison’s 18th Ed.3 x (15 . 21 . Hypochromic microcytic anemia A. D. 10 µg / dL B. increased red cell protoporphyrin. It along with serum ferritin. ~ 300 mg D. 304 Endogenous erythropoietin production is inadequate for 298 Elemental iron per day for iron replacement therapy is ? the degree of anemia in ? Harrison’s 18th Ed. 500 µg / dL C. Hypoproliferative marrow 301 In normal iron tolerance test. Normocytic and normochromic anemia A. With chronic inflammation. Hypoproliferative marrow D. 847 Harrison’s 18th Ed.patients Hb) + 500 or 1000 mg B.patients Hb in grams/ iron levels & transferrin saturation with small RDW index which is elevated in iron deficiency. Low serum iron D. dL) + 500 or 1000 mg (for stores). normal or increased serum Amount of parenteral iron needed is calculated by weight (kg) x 2. Increased red cell protoporphyrin Typically. Weight (kg) x 1. 100 µg / dL Serum levels of transferrin receptor protein (TRP) is 4 . Just before a meal A. Chronic inflammation B. wheezing.7 days after initiation of oral iron C. TRP is released by erythroid cells into circulation B. Normal TIBC D. Elevated red blood cell distribution width (RDW) index D.20%. the infusion of iron should be stopped immediately. 849 . 849 A. or other systemic Ferritin level is normal or increased and percent transferrin saturation & TIBC are typically below symptoms occur. 297 Which of the following is false about anemia of chronic 303 Infusion of iron must be stopped immediately if which of inflammation ? the following develops ? Harrison’s 18th Ed. 25 Hematology MCQ’s FOR MCQ’s FOR MEDICAL MEDICAL PROFESSIONALS PROFESSIONALS BY PROF. Chest pain B. iron preparations should be taken on empty stomach. 296 Which of the following is false about thalassemias ? 302 Amount of parenteral iron needed is calculated by ? Harrison’s 18th Ed. Normal or increased serum ferritin levels B. In normal iron tolerance test.21 days A. ~ 100 mg B. Normal or increased serum iron levels B. transferrin saturation of 15 . Weight (kg) x 3. increases by at least 100 µg/dL indicating patient’s ability to absorb iron adequately. 75 µg / dL D. 7 . Fall in blood pressure D.3 x (15 . Hypometabolism D. ~300 mg of elemental iron is given per day. a fall in blood pressure. Following meals C. Wheezing C. if chest pain. serum iron distinguishes between iron deficiency & anemia of chronic inflammation. renal disease. 849 A. ~ 200 mg C.3 x (15 . Weight (kg) x 2.patients Hb) + 500 or 1000 mg D. endogenous EPO production is inadequate for the degree of anemia observed. 847 Harrison’s 18th Ed. 2 hours after 2 iron tablets on an empty stomach. or hypometabolism. 849 A. 848 Harrison’s 18th Ed. oral iron preparations should be taken ? Harrison’s 18th Ed.9 µg/L. 299 Ideally. Early in the infusion of iron. Below normal serum transferrin saturation C. All of the above Since foods may inhibit oral iron absorption. 849 B. reticulocyte count begin to increase within 4 .3 x (15 . Serum ferritin level is the most distinguishing feature between true iron-deficiency anemia & serum iron increases by ? iron-deficient erythropoiesis associated with inflammation which increase threefold over basal levels in inflammation due to effects of inflammatory cytokines & hepcidin. 849 306 Which of the following is the most distinguishing feature between true iron-deficiency anemia & iron-deficient A. 4 . ~ 30 mg A. Normal or increased serum ferritin therapy and peak at 1½ weeks. 849 C.7 days erythropoiesis associated with inflammation ? B. Along with meals B. 14 . AJAY MATHUR Cardiology 25 A. 848 305 Which of the following is a feature of anemia of chronic disease ? A. Features of anemia of chronic disease are a low serum iron. TRP levels are elevated in absolute iron deficiency C. Any of the above Anemia of chronic inflammation with inadequate iron supply is normocytic and normochromic.patients Hb) + 500 or 1000 mg Characteristics of thalassemia include hypochromic microcytic anemia. normal. All of the above For iron replacement therapy. Normal or increased serum ferritin D. Serum levels of TRP reflect total erythroid marrow mass A. 2 hours after 2 iron tablets. Harrison’s 18th Ed.patients Hb) + 500 or 1000 mg C. hypoproliferative marrow.3 x (15 . 300 How many days after initiation of oral iron therapy. Normal or increased serum transferrin saturation C. Renal disease C. & normal or increased serum ferritin. Empty stomach Harrison’s 18th Ed. Increased red cell protoporphyrin C.28 days B. None of the above A. 852 r A. 1 D. None of the above - B. Triple oedema syndrome Harrison’s 18th Ed. Ballantyne syndrome D. 250 to 300 mg B.  2 2 Harrison’s 18th Ed. All of the above B.  and  are -like genes U N Engl J Med 2006. Aluminum toxicity and beta-like genes (on chromosome 11). 100 to 150 mg Harrison’s 18th Ed. Mirror syndrome 313 Structure of HbF is ? C. Aluminum toxicity and hyperparathyroidism can also compromise EPO response.5 g / dL D.354:2034-45 A. A. C. IX i B. 1. 9 D.75 g / dL C. X Chapter 104.  2 2 A. 852 D. 2 Hemoglobinopathies are disorders affecting the structure. has the structure ? D. /  ) & two  globin genes on short arm of chromosome 11 (one per C. All of the above   2 2 Ballantyne syndrome is also known as mirror syndrome and Triple oedema syndrome. All of the above 316 Which of the following is false ? Harrison’s 18th Ed. 10 mU per milliliter D.  2 2 Harrison’s 18th Ed. LCR controlling  globin gene is modulated by ATRX Harrison’s 18th Ed. -like globin genes are on chromosome 16 D. -like globin genes on chromosome 11 R 309 Inadequate erythropoietin response is due to ? C. or production of hemoglobin C. Function oxygen molecules ? Harrison’s 18th Ed. consisting of a protoporphyrin IX ring complexed with a single iron atom in ferrous state (Fe 2+). Hyperparathyroidism ti e D. 25 to 30 mg 315 Which of the following about human hemoglobins is false ? B. 852 Figure 104-1 C. 40 mU per milliliter 317 Heme consists of which of the following protoporphyrin ring ? 9 Normal blood level of erythropoietin is 20 mU per milliliter.25 g / dL   2 2 A typical unit of packed red cells increases the hemoglobin level by 1 g/dL. Normal individual has two  globin genes 310 Normal blood level of erythropoietin is ? C.  2 2 A. XI D. or  /  ). 851 D. 4 Harrison’s 18th Ed. HbA. 3 312 The major adult hemoglobin. Normal individual has four  globin genes n B. One molecule of hemoglobin can A. B. 20 mU per milliliter A normal individual has 4  globin genes on short arm of chromosome 16 (two genes per chromosome . 852 C.  2 2 308 A unit of packed RBCs contains how much iron ? C. 1 g / dL 314 Structure of HbA 2 is ? Harrison’s 18th Ed. 851 C. None of the above V A. Harrison’s 18th Ed. Production A. Disorders of Hemoglobin h C. None of the above B. 0. Locus control region (LCR) LCR controlling the alpha- globin gene cluster is modulated by ATRX.  2 2 319 Severe fetal hydrops is related to which of the following ? C. 852 A. None of the above B. . Structure 318 Every molecule of hemoglobin can transport how many B. 852 D. 150 to 250 mg G A. function.26 MCQ’s FOR MEDICAL PROFESSIONALS BY PROF. Iron depletion Gene clusters encoding human hemoglobins are alpha-like globin genes (on chromosome 16). 0. 30 mU per milliliter chromosome.  2 2 transport up to four oxygen molecules. Harrison’s 18th Ed. The  -like genes (  &  ) are nearby on chromosome 11. B. 851 D. AJAY MATHUR Hematology 307 A typical unit of packed RBC increases hemoglobin by ? B. 852 A fall in Hb during EPO therapy signifies infection or iron depletion. d B. 852 Each heme moiety can bind a single oxygen molecule. XII ta 311 Hemoglobinopathies are disorders that affect which of the following parameters of hemoglobin ? Each globin chain has a single heme moiety. Whereas thalassemias are caused by gene 326 Almost exclusive synthesis of adult hemoglobin (HbA) in mutations resulting in decreased production of  or  globin chains. 30 weeks Harrison’s 18th Ed. 852 A. About 6 weeks A. About 18 weeks C. 852 321 Modulator of O2 affinity of heme molecules is ? A. 852 is false ? A. 853 B. Stiff RBC membrane D. 852 A. the sickle cell disorders are hemoglobinopathies caused by mutations in the  globin gene resulting in the production of a foetus occurs at about ? structurally abnormal  globin chain. 323 In which week of gestation red cells first appear in foetus ? 329 Which of the following statements about hemoglobinopathies Harrison’s 18th Ed. Thalassemia syndromes A. Thalassemia children more susceptible to infection with nonlethal Plasmodium vivax D. All of the above B. 223 GluVa1 B.3-bisphosphoglycerate (2.3-BPG) C. pH D. Plasmodium falciparum 324 Which of the following is an embryonic hemoglobin ? D. About 4 weeks A. About 6 weeks B.the most common structural hemoglobinopathy). Hb Gower II A. 226 GluVa1 At 10 . which of the following hemoglobinopathies occur ? Harrison’s 18th Ed. 853 Figure 104-2 B. Tissue PO2 Nearly exclusive synthesis of adult hemoglobin (HbA) occurs at ~38 weeks. None of the above Harrison’s 18th Ed. mRNA. chain. pH D. leading to deficient globin chain biosynthesis. AJAY MATHUR Cardiology 27 320 Hemoglobin-oxygen dissociation curve is between percent A. 852 . 27 Hematology MCQ’s FOR MCQ’s FOR MEDICAL MEDICAL PROFESSIONALS PROFESSIONALS BY PROF. 34 weeks A. Hb Gower I and Hb Gower II. Tissue PCO2 327 When mutations alter the amino acid sequence of a globin Hemoglobin-oxygen dissociation curve is between percent saturation of Hb and tissue PO 2 . 854 A. CO 2 chain therby altering physiologic properties of the variant hemoglobins (sickle cell anemia . 852 Harrison’s 18th Ed. HbS is best illustrated as ? Harrison’s 18th Ed. 38 weeks B. Temperature Structural hemoglobinopathies occur when mutations alter the amino acid sequence of a globin D. About 10 weeks B. Neutral pH C. Acquired hemoglobinopathies D. None of the above 325 In which week of gestation does fetal hemoglobin become predominant ? 331 In sickle cell syndrome. Alveolar PO2 D. The sickle cell syndromes are caused by a mutation in the beta-globin gene that changes the sixth amino acid from glutamic acid to valine. 224 GluVa1 C. 854 C. 28 weeks saturation of Hb and ? B. fetal hemoglobin (HbF) becomes predominant. 225 GluVa1 D. High pH D. 853 Figure 104-2 C. Acquired hemoglobinopathies C. Common in malaria endemic areas C. About 8 weeks C. Hereditary persistence of fetal hemoglobin (HPFH) B. About 24 weeks D. Any of the above Thalassemia syndromes arise from mutations that impair production or translation of globin The Bohr effect is the ability of hemoglobin to deliver more oxygen to tissues at low pH. C. Premature RBC destruction Embryonic hemoglobins are Hb Portland. Hereditary persistence of fetal hemoglobin (HPFH) C. Low pH B. Thalassemia syndromes B. Hb Portland 330 Which of the following is not a characteristic of Sickle Cell Syndromes ? B. Blood pH is the most important modulator of O 2 affinity (Bohr effect). Structural hemoglobinopathies A. which of the following hemoglobinopathies occur ? oxygen to tissues at ? Harrison’s 18th Ed. 328 When mutations impair production or translation of globin 322 Bohr effect is the ability of hemoglobin to deliver more mRNA. Microvascular vasoocclusion D. 853 Harrison’s 18th Ed. Hb Gower I Harrison’s 18th Ed. Structural hemoglobinopathies Harrison’s 18th Ed. About 2 weeks Harrison’s 18th Ed.11 weeks. C. Harrison’s 18th Ed. 2. Thalassemia naturally protects against infection with Red cells first appear at about 6 weeks after conception. 3 D. how many of the four -globin loci are deleted ? 9 C. D. C. Elfin facies B. 3 h 336 Which of the following drugs may elevate HbF ? D. 2 i hydration. Its widespread use has been prevented because of concerns about acute toxicity & carcinogenesis. 4 ti e C. B. 2 C. 859 334 Bone pain in sickle cell crisis is due to ? A. 2854 A. how many of the four -globin loci are deleted ? The bone pain in sickle cell crisis is due to bone and bone marrow infarction. 1 U disease ? B. Hydroxyurea -thalassemia-1 trait (with two deleted loci). 857 C. 859 D. Anagrelide D. All of the above minor ? Harrison’s 18th Ed. -thalassemia-1 trait deoxyazacytidine (decitabine) can elevate HbF with more acceptable toxicity.30 mg/kg per day) increases fetal hemoglobin. 857 -thalassemia-1 trait resembles  -thalassemia minor. 4 Harrison’s 18th Ed. 4 Muddy appearance of freshly drawn blood is characteristic of Methemoglobinemia. 4 G Sickle cell dactylitis or hand-foot syndrome caused by painful infarcts of digits & dactylitis is seen 340 In  -thalassemia-1 trait. Chipmunk facies A. C. Low doses of 5. Bone & bone marrow infarction C. AJAY MATHUR Hematology 332 HbC is best illustrated as ? B. 859 D. Sickle cell disease C.28 MCQ’s FOR MEDICAL PROFESSIONALS BY PROF. 1 r Hydroxyurea (10 . vascular wall adherence. 224 GluLys In thalassemic children. Danazol 342 In HbH disease. Tietze Syndrome A. are deleted ? R Harrison’s 18th Ed. 1 V Harrison’s 18th Ed. Sickle cell anemia silent carrier state. how many of the four  -globin loci in sickle cell disease & sickle cell thalassemia below the age 4 or 5 years & not in adults. Hyperuricemia D. Osteoporosis 341 In hydrops fetalis with Hb Bart’s. HbH disease (with three loci deleted) and hydrops B. Thalassemia Harrison’s 18th Ed. 2 Harrison’s 18th Ed. Fracture B. how many of the four  -globin loci 333 Hand-foot syndrome is related to which of the following ? are deleted ? Harrison’s 18th Ed. characteristic “chipmunk” facies occurs due to maxillary marrow hyperplasia C. 5-azacytidine fetalis with Hb Bart’s (all four loci deleted). 3 D. IFN-alpha Harrison’s 18th Ed. 5-deoxyazacytidine (decitabine) 343 Which of the following -thalassemia resembles  -thalassemia D. and suppresses granulocyte & reticulocyte counts. 859 D. Syringomyelia D. B. 857 ta The four classic  thalassemias are -thalassemia-2 trait (one of the four -globin loci is deleted). 854 C. 225 GluLys & frontal bossing.  -thalassemia-2 trait is an asymptomatic. Leonine facies Harrison’s 18th Ed. n Harrison’s 18th Ed. A. 1 B. HbH disease which of the following ? D. Plethoric moon facies Hb Barts is designated as  4 . 4 B. 3 - A. Methemoglobinemia 344 Hb Barts is designated as ? C. 859 335 Which of the following is useful in the treatment of sickle cell A. Hypertrophic Osteoarthropathy B. Hydroxyurea 9 A. 226 GluLys 339 In  -thalassemia-2 trait. 859 A. A. -thalassemia-2 trait 337 Muddy appearance of freshly drawn blood is characteristic of C. Hydrops fetalis with Hb Bart’s Harrison’s 18th Ed. 4 Harrison’s 18th Ed. 4 A. has favourable effects on RBC B. 859 Antitumor drug 5-azacytidine was the first agent found to elevate HbF. B. 2 d B. 223 GluLys D. 854 Harrison’s 18th Ed. All of the above A. 4 338 Which of the following facies is typical of thalassemia ? C. A. MCH > 27 pg.353:1135-46 conditions ? A. Diabetes mellitus Harrison’s 17th Ed. Serum iron level 357 Which of the following findings rules out thalassemia and no B. 15 years Variant hemoglobins that may be co-inherited with -thalassemia are Hemoglobin S. Only 3 conditions cause microcytic erythrocytosis . and glucose-6-phosphate dehydrogenase (G6PD) deficiency closely resemble that of malaria D. Patient who inherits a HbE mutation from one parent and a -thalassemia mutation from another (HbE/-thalassemia) will clinically be similar to a patient with -thalassemia intermedia or major. Thalassemia trait C. Hemoglobin C B. 5 years C. 673 B. Chapter 103 B. Pulmonary fibrosis resemble that of malaria ? D. All of the above C. Cardiomyopathy hemoglobin C disease. All of the above structurally abnormal it is produced in reduced quantities. Iron deficiency D. Microfractures In  -thalassemia.353:1135-46 Target cells (area of central pallor with dense center or bull’s eye) is typical of thalassemia. normal Hb electrophoresis Normal or increased serum iron & ferritin levels and transferrin saturation are characteristic of the C. MCH > 27 pg. Beta thalassemia intermedia B. Levels increase when iron stores are elevated D. Polycythemia vera (PV) Hemoglobin E is referred to as a “thalassemic hemoglobinopathy” because in addition to being D. Levels are high in patients with thalassemia major Geographic distributions of sickle cell disease. thalassemia. All of the above B. Hypoxic erythrocytosis D. 10 years D. 631 intermedia & thalassemia major. Any of the above D. Hemoglobin S age suggests a coexisting thalassemia ? Harrison’s 17th Ed. Thalassemia A. A. All of the above C. 20 years Presence of a palpable spleen in sickle cell disease after age 5 years suggests a coexisting 353 Which of the following is referred to as a “thalassemic hemoglobinopathy like thalassemia. MCV > 90 fL. 352 Variant hemoglobins that may be co-inherited with  - thalassemia are ? 346 In sickle cell disease. MCH > 27 pg. 29 Hematology MCQ’s FOR MCQ’s FOR MEDICAL MEDICAL PROFESSIONALS PROFESSIONALS BY PROF. Osteomalacia C. All of the above C. Hemoglobin E A. Glucose-6-phosphate dehydrogenase (G6PD) deficiency B. normal Hb electrophoresis D. AJAY MATHUR Cardiology 29 345 Which of the following is not a feature of thalassemia ? 351 Cooley’s anemia refers to ? Harrison’s 17th Ed. but can be seen in iron deficiency. Hepcidin levels normally 350 Which of the following is normal or increased in thalassemia ? increase when iron stores are elevated. . Hemoglobin C B. 375 B. Chapter 203 A. RDW is normal in beta-thalassemia. Hypoparathyroidism B.353:1135-46 B. Diabetes insipidus C.thalassemia trait. Inhibits iron absorption in small bowel C. Sickle cell disease 356 Which of the following about Hepcidin is false ? N Engl J Med 2005. hypoxic erythrocytosis & PV. presence of palpable spleen after what A. MCV > 80 fL. ovalocytosis. Hemoglobin S Harrison’s 17th Ed. Excessive melanin skin pigmentation 349 Geographic distributions of which of the following closely C. Avascular necrosis is not a feature of thalassemia because there is no sickling of red cells leading to thrombosis and infarction. Hemoglobin E A. Larger numbers are typical of A. B. Osteopenia D. 2179 A. Hepcidin levels are inappropriately low in thalassemia Harrison’s 17th Ed. Hypogonadism A. cholestatic liver disease or as an artifact. Hemoglobin E and Hemoglobin C. the  globin chains are structurally normal but quantitatively reduced. Transferrin saturation additional thalassemia testing is required ? C. normal Hb electrophoresis thalassemias. Serum ferritin level A. All of the above 355 Features of thalassemia include all except ? N Engl J Med 2005. MCV > 100 fL. Hypopituitarism Harrison’s 17th Ed. D. Avascular necrosis B. Thalassemia C. 354 Features of thalassemia include all except ? 348 Target cells in PBF can be seen in which of the following N Engl J Med 2005. hemoglobinopathy” ? 347 Which of the following is a cause of microcytic erythrocytosis ? A. Beta thalassemia major C. Beta thalassemia minor A. Cholestatic liver disease D. Also called “Storage iron regulator” Hepcidin is a small peptide that inhibits iron absorption in small bowel. while it is elevated in hypoxic erythrocytosis & PV. MCH > 27 pg. Porphyria thalassemia testing. A. > 100 units Cardiothoracic ratio in fetal ultrasound is used to assess risk of hemoglobin Bart’s hydrops i fetalis (normal < 0. Vitamin C 359 Which of the following is not applicable for estimating iron B. Butyrates A. 860 A. Hemolytic anemia 358 Patients with  -thalassemia trait have an elevated HbA2 of ? C. CT of liver d A. Ventricular system of fetal brain r 9 B. Deferasirox Estimated 5-year survival rate following allogeneic bone marrow transplantation is 90% if done D. Cadmium D. B. . 861 Patients with  -thalassemia trait have an elevated HbA2 of > 3.5 % Harrison’s 18th Ed. AJAY MATHUR Hematology D. 90 % C. > 3. Cytarabine B. None of the above before they develop hepatomegaly or portal fibrosis and if given adequate iron chelation therapy. Deferoxamine is iron-chelating agent & requires parenteral administration. Hydroxyurea Harrison’s 18th Ed. C.300 mg 367 Which parameter in fetal ultrasound is used to assess risk U of hemoglobin Bart’s hydrops fetalis ? A unit (250-300 ml) of packed RBCs contains 250 . C. Deferoxamine C. > 1. B. 861 A. Harrison’s 18th Ed. Butyrates also stimulate HbF production. Precipitates of unpaired a & b globin chains A. D.300 mg of iron (1 mg/mL).5 %. 860 C. Thalassemia A. Precipitates of unpaired  globin chains 360 A unit (250-300 ml) of packed RBCs contains how much iron ? B. All forms of Cobalamin (vit. Vitamin E overload in thalassemia patients ? C. Any of the above B. Nuchal translucency packed RBCs ? C.30 MCQ’s FOR MEDICAL PROFESSIONALS BY PROF. normal Hb electrophoresis 364 Lucarelli classification is used to classify ? N Engl J Med 2005. Abdominal circumference 361 Patients develop hemosiderosis after how many units of B. Megaloblastic Anemias synthesis ? Harrison’s 18th Ed. Anemia B.5 % 365 Which of the following should be avoided in an iron excess state ? D. C. Deferiprone D.353:1135-46 D. 150 . Precipitates of unpaired a globin chains ti e Harrison’s 18th Ed. Labile plasma iron estimation N Engl J Med 2005.53).250 mg D.150 mg n Precipitates of unpaired  chains form single large inclusions known as Heinz bodies. 50 % A. 30 % 362 Which of the following is an not an oral iron-chelating agent ? N Engl J Med 2005. Cobalt Reestablishing high levels of HbF synthesis can ameliorate symptoms of thalassemia. Calcium Hydroxyurea & cytarabine promote high levels of HbF synthesis by stimulating proliferation of F cell progenitors. 861 369 Element found at the center of corrin ring in cobalamin is ? A. All of the above C. 70 % B. > 2. Folic acid G N Engl J Med 2005. 50 . > 50 units D.5 % Lucarelli classification assess risk factors that predict outcome & prognosis in thalassemia. 250 . 100 .100 mg D.353:1135-46 D.353:1135-46 The finding of a normal MCV (>= 80 fL) with normal MCH (>= 27 pg) and normal Hb electrophoresis or HPLC rules out most cases of thalassemia and requires no additional A.5 % D. Copper C. > 0. 363 Which of the following promotes high levels of HbF Chapter 105. Cardiothoracic ratio 9 Harrison’s 18th Ed. > 200 units 368 Estimated 5-year survival rate following allogeneic bone h D. B 12) have a cobalt atom at the center of corrin ring. Magnetic susceptometry (SQUID) 366 Heinz bodies is best related to ? V C. Plant flavonoids A.353:1135-46 B. Serum ferritin R Vitamin C should not be supplemented in iron excess states because it generates free radicals. - A. 862 B. > 500 units marrow transplantation is ? ta Patients who receive >100 units of packed RBCs usually develop hemosiderosis. MCV > 110 fL. Methylcobalamin is the Dietary cobalamin combines rapidly with a salivary glycoprotein that belongs to the family of form of cobalamin in human plasma & in cell cytoplasm. IF-cobalamin attaches to it and enters the ileal cell. Daily requirement for cobalamin is ~ 1 . Gastric parietal cells C. Cannot be synthesized in human body A. AJAY MATHUR Cardiology 31 370 Which form of cobalamin is present in human plasma & in 376 Which of the following is a family of cobalamin-binding cell cytoplasm ? proteins ? Harrison’s 18th Ed. normal physiologic active mechanism is through ileum mediated by gastric intrinsic factor (IF). Cardiomyocyte B. Ileal mucosa C. 2 to 3 years A. Islet of Langerhans D. 862 cobalamin-IF complex is ? A. and dairy products). Specific congenital enzyme deficiencies C. Cubulin D. It cannot be synthesized in human body & must be supplied in diet of animal products (meat. 373 Body stores of cobalamin can suffice for how many years after supplies are completely cut off ? 379 Name of the receptor that mediates intestinal absorption of Harrison’s 18th Ed. Dietary deficiency Harrison’s 18th Ed. 862 following enzyme ? A.. All of the above Cobalamin absorption can be passive through buccal. haptocorrin is digested by pancreatic trypsin to release cobalamin which is transferred 372 Which of the following about cobalamin is false ? to IF. fish. IF is produced in the acid-secreting gastric parietal cells located in oxyntic gland of fundus & body of stomach. Adocobalamin D. Pancreatic trypsin C. 862 B. Gastric enterochromaffin cells corrin ring. Buccal mucosa A. an endocytic receptor protein that directs sublocalization and endocytosis of cubulin with its ligand IF-cobalamin complex. Daily requirement is ~ 1 . 5 to 7 years C. duodenal & ileal mucosa. Deficiency of cobalamin is almost always due to malabsorption. Renal proximal tubular epithelium C. All of the above D. Serine . 862 378 IF is produced in ? A. Copper atom is situated within a corrin ring Harrison’s 18th Ed.glycine hydroxymethylase B. Methionine synthase A. 862 A. 374 Deficiency of cobalamin is almost always due to ? Harrison’s 18th Ed. Methylcobalamin B. Cobalamin A. 31 Hematology MCQ’s FOR MCQ’s FOR MEDICAL MEDICAL PROFESSIONALS PROFESSIONALS BY PROF. Cubilin is a specific receptor on microvillus membrane of enterocytes. Cobalin Body stores of 2 . Humulin D. Pancreatic colipase Methylcobalamin is the cofactor for methionine synthase. Hydroxocobalamin C. Malabsorption A. cobalamin-binding proteins known as haptocorrins (HCs). All of the above Cobalamin (vitamin B12) exists in a number of different chemical forms. . 371 Methylcobalamin is the cofactor for ? 377 In intestine. Glycophorin A Dietary intake of cobalamin is more than adequate for body’s requirements.3 µg C. All of the above C. In intestine.3 µg. Gastric endocrine cells Cobalamin is a complex organometallic compound in which a “cobalt” atom is situated within a D. There is a permanent liver reserve of 1 mg. Pancreatic amylase D. 862 B. Cubulin acts through amnionless (AMN). 862 Harrison’s 18th Ed. Enterocorrins C. Its secretion parallels that of hydrochloric acid. Amnionless (AMN) C. 862 A. More efficient Cubilin also is present in yolk sac and renal proximal tubular epithelium. 862 Harrison’s 18th Ed. except in complete vegetarians & their breast-fed infants. 862 B. 1 to 2 years Harrison’s 18th Ed. Duodenal mucosa B.4 years if supplies are completely cut off essentially due to enterohepatic cycle & size of liver stores. Asialo-GM1 D. Pancreatic lipase D. Glucocorrins B. Spirulin C.3 mg are sufficient for 3 . where IF is destroyed. 862 380 Endocytic receptor protein related to cubulin is ? A. 3 to 4 years B. Only dietary source is animal products B. Haptocorrins D. 862 B. haptocorrin is digested by which of the Harrison’s 18th Ed. Cystathionine synthase Harrison’s 18th Ed. Harrison’s 18th Ed. Alcohol abuse B. Leptin receptor D. 375 Normal active physiologic mechanism of cobalamin absorption occurs in ? 381 Cubilin also is present in ? Harrison’s 18th Ed. Gastric chief cells D. Milk B. . All dietary folates are 387 Which of the following is primary dietary source of folic converted to 5-methyl THF (5-MTHF) within small-intestinal mucosa before entering portal plasma. periods of enhanced metabolic demand such as pregnancy. 602 A. Vitamin B12 A. 389 Normally. hemodialysis. Transcobalamin (TC) I B. Ascorbic acid B. 200 µg B. N15-methyltetrahydrofolate D. 602 Harrison’s 18th Ed. Kidney Cobalamin in food is released and forms a stable complex with gastric R binder that is found in Total-body folate in the adult is ~10 mg. 863 A. 863 B. 5 to 7 months U TC I is derived from specific granules in neutrophils.4 months. All of the above D. 300 µg G C. but this may be increased several fold during On entering the duodenum. yeast. which it binds tightly. Eggs A. Plasma folate is primarily in the form of N 5 -methyltetrahydrofolate. 863 Harrison’s 16th Ed. it is ~ two-thirds saturated with Total-body folate in adult is ~10 mg. polyglutamates to mono. Daily adult requirement is ~100 µg. Folic acid Harrison’s 18th Ed. placenta. Methionine to homocysteine B. acid ? Harrison’s 18th Ed. Transcobalamin (TC) I D.32 MCQ’s FOR MEDICAL PROFESSIONALS BY PROF. Normally. Diglutamate D. malignancy. 863 ti e A. a 50-kDa glycoprotein which catalyzes the conversion hematopoiesis (chronic hemolytic anemias). Meat C. Gastric juice B. secretions like saliva. 863 B. Conjugases (  -glutamyl carboxypeptidases) in gut lumen convert ta cobalamin in plasma & gives up cobalamin to marrow. liver containing the largest store. gastric juice and bile. for 3 . and other tissues. minimum daily requirement of folic acid is about ? 383 Intrinsic factor (IF) catalyzes the conversion ? Harrison’s 18th Ed. N5-methyltetrahydrofolate C. N10-methyltetrahydrofolate D. 863 9 Harrison’s 18th Ed. infancy. Pyridoxine C. and nuts. Folate concentrations is D. TC I does not enhance cobalamin entry into tissues. 863 393 N5-methyltetrahydrofolate is a type of ? A. TC II carries intestinal absorption. chronic exfoliative skin disorders. 3 to 4 months D. increased which then binds to intrinsic factor (IF). which are readily absorbed in proximal jejunum. milk. Liver B. Transcobalamin (TC) II h Folates in food are largely conjugated to a chain of glutamic acid residues which impair its Two main cobalamin transport proteins in human plasma are TC I & TC II. Bone marrow C. if severe folate deficiency develops rapidly. Bile C. Gastric R binder 9 B. Serine to glycine D. 386 The common name for pteroylmonoglutamic acid is ? 392 All dietary folates are converted to which of the following Harrison’s 18th Ed.& diglutamates. N 5 -methyltetrahydrofolate is a monoglutamate. Polyglutamate highest in liver. 862 Harrison’s 18th Ed. Glycine to serine R Daily requirement is normally about 100 µg. Monoglutamate C. 390 Body stores of folate can suffice for how many months after d 384 Most circulating cobalamin is bound to ? supplies are completely cut off ? Harrison’s 18th Ed. 862 A. Saliva A. - 385 Which of the following is not related to “absorption” of 391 Site of absorption of Folic acid is ? cobalamin in humans ? Harrison’s 18th Ed. 2 to 3 months C. Triglutamate Fruits and vegetables are the primary dietary source of folic acid. Gastric R binder A. Intrinsic factor (IF) D. releasing the cobalamin. Terminal Ileum i C. 1 to 2 months B. AJAY MATHUR Hematology 382 Gastric R binder is found in which of the following secretions ? 388 Which of the following is the major body store of folic acid ? Harrison’s 16th Ed. 400 µg D. Proximal jejunum B. 100 µg A. Stomach A. Transcobalamin (TC) II n C. Homocysteine to methionine C. other greens. Spleen D. so stores are sufficient cobalamin. 863 before entering portal plasma? A. spinach. the cobalamin-R binder complex is digested. N20-methyltetrahydrofolate Folic acid is the common name for pteroylmonoglutamic acid. Colon D. Intrinsic factor (IF) r C. Fruits and vegetables Harrison’s 18th Ed. V of homocysteine to methionine. Methionine D. Pyrimidine synthesis B. 863 A. For purine synthesis. and the methylation of homocysteine synthesis (Methylation of deoxyuridine monophosphate (dUMP) to thymidine monophosphate to methionine requires both methylcobalamin and 5-MTHF. the precursor of dTTP because folate is needed as the coenzyme 5. All of the above Folate coenzymes are essential in Formate activation. Low cell folate C. 863 5. the B. 645 D. there is failure to convert deoxyuridine monophosphate B. For methionine synthesis. The sources of these 1-carbon moieties is usually C. Factor for methionine synthase & methylmalonyl coenzyme A (CoA) synthase 400 Which of the following about megaloblastic anemias is false ? C. Deoxythymidylate monophosphate (dTMP) synthesis 397 Active form of folic acid is ? C. and methionine. In these reactions the cofactor is released as THF. homocysteine to methionine and forminoglutamic acid to glutamic acid in histidine catabolism). Cell division becomes sluggish but Harrison’s 16th Ed. All of the above D.10 methylenetetrahydrofolate. not as THF.glycine interconversion C. pyrimethamine. with an increased ratio of RNA to DNA. B.10-methylene . Hydrogen atoms used for this reduction come from the cofactor. Hematopoietic precursors & GI epithelial cells affected formyl groups to various organic compounds. B. Methotrexate A. and trimethoprim inhibit DHF reductase that prevents formation of active THF coenzymes from DHF. B. Dihydrofolate THF acquires 1-carbon fragment from serine which is converted to glycine. there is failure C. deoxythymidylate monophosphate (dTMP). Megaloblastic erythroid progenitors are destroyed in marrow whose A. Adocobalamin B. Megaloblastic cells have increased DNA to RNA ratio serine which reacts with tetrahydrofolate to produce glycine and N 5. D. All of the above Harrison’s 18th Ed. DHF has to be re-reduced to THF. Polyglutamates to mono & diglutamates cellularity is increased but production of RBC is decreased (ineffective erythropoiesis). 404 Which of the following abnormalities of folate metabolism 399 Which of the following drugs inhibit DHF reductase ? occur in cobalamin deficiency ? Harrison’s 18th Ed. 864 Conjugases (  -glutamyl carboxypeptidases) in the gut lumen convert polyglutamates to mono. dUTP to dUMP A. a cobalamin-requiring reaction. Amino acid interconversion (serine .THF polyglutamate for conversion of dUMP to dTMP. Disaccharides to monosaccharides Harrison’s 18th Ed. Polysaccharides to mono and disaccharides to convert ? D. so megaloblastic cells tend to be large. Positive purine precursor aminoimidazole carboxamide ribonucleotide (AICAR) excretion . Pyrimidine Methylmalonyl CoA isomerization requires adocobalamin. Conversion of succinyl CoA to methylmalonyl CoA A. Trimethoprim C. 398 Folate coenzymes are essential in which of the following biochemical reactions ? 403 Methylmalonyl CoA isomerization requires which of the Harrison’s 18th Ed. dTMP to dUMP 396 Folate is essential for the de-novo synthesis of ? C. Ineffective erythropoiesis 395 Gamma-glutamyl carboxypeptidases in gut lumen convert Megaloblastic anemias are caused by impaired DNA synthesis in cells with rapid turnover like ? hematopoietic precursors & gastrointestinal epithelial cells.10-methylene-THF is oxidized to DHF (dihydrofolate). 863 Table 105–2 following ? Harrison’s 18th Ed. the 1-carbon fragment is reduced from formaldehyde to a methyl group during transfer reaction. 864 A. dUMP to dTMP and diglutamates. All of the above 402 Which of the following is a cobalamin-requiring reaction ? Harrison’s 16th Ed. dUMP to dUTP Harrison’s 16th Ed. which is released. (dTMP). A. Methionine synthesis Harrison’s 16th Ed. then transferred to homocysteine. During the D. To participate further in the 1-carbon transfer cycle. Purines synthesis biosynthesis of these compounds.glycine interconversion. Trihydrofolate 1-carbon fragment is first oxidized to the level of formic acid. which are readily absorbed in the proximal jejunum. Purine synthesis (formation of glycinamide ribonucleotide and formylation of aminoimidazole carboxamide ribonucleotide (AICAR). the 1-carbon fragment is first reduced to the C. All of the above A. High serum folate B. Conversion of methylmalonyl CoA to succinyl CoA Harrison’s 17th Ed. a reaction catalyzed by dihydrofolate reductase. Caused by impaired DNA synthesis Primary function of folate compounds is to transfer single carbon moieties such as methyl and B. Deoxythymidylate monophosphate (dTMP) (dUMP) to deoxythymidine monophosphate (dTMP). 5-MTHF D. 602 D. then transferred to substrate. Active form of folate is tetrahydrofolate (THF). B. Purines In deficiencies of either folate or cobalamin. Pentahydrofolate production of dTMP from dUMP. Methotrexate. Pyrimethamine B. Enzyme DHF reductase converts DHF A. Methylcobalamin C. 33 Hematology MCQ’s FOR MCQ’s FOR MEDICAL MEDICAL PROFESSIONALS PROFESSIONALS BY PROF. serving as an intermediate carrier of 1-carbon fragments used in the A. 864 Harrison’s 18th Ed. which can immediately participate in another 1-carbon transfer cycle. 602 Folate is essential for the de novo synthesis of purines. 601 cytoplasmic development progresses normally. Tetrahydrofolate level of a methyl group. To transfer single-carbon moieties to organic compounds to THF. AJAY MATHUR Cardiology 33 394 The prime function of folate compounds is ? D. Purine synthesis A. C. but as dihydrofolate (DHF). 602 D. Serine . Diglutamates to monoglutamates 401 In deficiencies of either folate or cobalamin. elevated homocysteine cystathionine synthase. Raised serum methylmalonic acid. 606 411 Deficiency of which of the following enzymes can cause A. Raised serum methylmalonic acid. the gastrointestinal tract. Autoantibodies to folate receptors cell folate and positive purine precursor aminoimidazole carboxamide ribonucleotide (AICAR) excretion. 18 % Harrison’s 18th Ed. 0. Mutations in serine . reduced activity of the enzyme 5.4 mg daily incidence of which of the following ? Harrison’s 18th Ed. 865 Prophylactic folic acid in pregnancy reduces the subsequent incidence of acute lymphoblastic leukemia (ALL) in childhood. and Harrison’s 18th Ed. A. 865. 865 0. reduced homocysteine A. low C. folic acid supplements must D. First 12 weeks of pregnancy A. 865. 865 respiratory. Normal serum methylmalonic acid. Blood To prevent neural tube defects. First 4 weeks of pregnancy 415 Clinical features of cobalamin deficiency involve which of C. Reduced activity of 5. Folate deficiency is due to excessive utilization because of B. Reduced serum methylmalonic acid.10-methylene-THF reductase (MTHFR) has anemia due to folate deficiency is true ? been identified as the maternal folate metabolic abnormality. 0. All of the above B. B. Harrison’s 16th Ed. It reduces the incidence of neural tube defects (NTDs) B. Cystathionine synthase A. 865 the nervous system. Methionine synthase D. Epithelial cell surfaces of the mouth D. 25 % A. Hodgekin’s lymphoma C. First 8 weeks of pregnancy the following ? Harrison’s 16th Ed. reduced homocysteine 412 Individuals with which of the following enzyme deficiency D. Hemangioma be started at ? Harrison’s 18th Ed. Conception B. elevated have an increased risk of vascular disease ? homocysteine Harrison’s 18th Ed. In women who A. 869 C. A. Epithelial cell surfaces of the small intestine 413 Meta-analysis has suggested that folic acid supplementation Most frequently affected tissue in cobalamin and folate deficiencies is the bone marrow reduces the risk of stroke by ? followed by the epithelial cell surfaces of the mouth. Raised serum methylmalonic acid. Bone marrow Children with deficiency of enzyme methionine synthase. Folic acid (400 µg daily. A. Nervous system 410 In NTD fetuses. MHTFR Harrison’s 18th Ed. C. 4% 408 What dose of folic acid provides protective effect against B. Homocystinuria is a rare metabolic defect in the conversion of homocysteine to cystathionine. All of the above B. 865 C. elevated homocysteine homocystinuria ? B.4 mg daily of folic acid provides protective effect against NTDs at conception. and female genital tracts. should be given as a supplement before and throughout pregnancy. Astrocytoma 409 To prevent neural tube defects. or A.10-methylene-THF reductase (MTHFR) 405 Which of the following statements about megaloblastic In NTD fetuses. reduced homocysteine Harrison’s 18th Ed.1 mg daily Meta-analysis has suggested that folic acid supplementation reduces the risk of stroke by 18%. encephalocele. 0. C. 871 D. D. AJAY MATHUR Hematology D. Reduced serum methylmalonic acid. meningomyelocele. folic acid supplements must be started at the time of conception and in the first 12 weeks of pregnancy. reduced homocysteine compensatory increased conversion of homocysteine to methionine.2 mg daily C. and spina bifida) in the fetus by 70%. MHTFR or cystathionine synthase C. Acute lymphoblastic leukemia (ALL) have had a previous fetus with a neural tube defect. All of the above metabolic abnormality has been identified ? The clinical features of cobalamin deficiency involve the blood. 5 mg daily is recommended when pregnancy is contemplated and throughout the subsequent pregnancy. Mutations in methionine synthase .34 MCQ’s FOR MEDICAL PROFESSIONALS BY PROF. Gastrointestinal tract (anencephaly.glycine hydroxymethylase Abnormalities of folate metabolism that occur in cobalamin deficiency include high serum folate. elevated homocysteine B. D. Methionine synthase 407 Most frequently affected tissues in cobalamin and folate deficiencies is ? B. MTHFR. Reduced serum methylmalonic acid. Peripheral nerves have an increased risk of vascular disease. 865 A. 606 Severe homocystinuria may be due to deficiency of methionine synthase. MTHFR C. which of the following maternal folate D. urinary. Normal serum methylmalonic acid. 603 D. All of the above Harrison’s 16th Ed. stomach. 8% Neural Tube Defects (NTDs) at conception ? C. and small intestine and the Harrison’s 18th Ed. 0. Cystathionine synthase 406 Which of the following statements about megaloblastic anemia due to cobalamin deficiency is true ? D. B.3 mg daily 414 Prophylactic folic acid in pregnancy reduces subsequent D. Sphincter disturbances C. with macroovalocytes. Anemia A. Raised serum lactate dehydrogenase A. & leukocyte and platelet count may also be decreased. Hemolysis C. 35 Hematology MCQ’s FOR MCQ’s FOR MEDICAL MEDICAL PROFESSIONALS PROFESSIONALS BY PROF. Imerslund-Grasbeck disease extremities (the earliest neurologic manifestations). All of the above Sites of involvement include peripheral nerves. liver disease. D. > 100 fL D. Nuclear chromatin is more dispersed & it condenses in a peculiar 421 Which of the following provides evidence of ineffective fenestrated pattern that is very characteristic of megaloblastic erythropoiesis. Aplastic anemia D. Nuclear-cytoplasmic asynchrony in RBC precursors D. TC II deficiency D. 605 B. 603 Harrison’s 16th Ed. weakness. RBC precursors are abnormally large & have nuclei that single cell with a nucleus of six lobes or more raises suspicion of a megaloblastic anemia. Numbness and paresthesia in extremities Harrison’s 16th Ed. AML C. Reticulocyte index is low. the spinal cord. Pernicious anemia C. Abnormal mitoses may be seen. Demyelination Harrison’s 16th Ed. All of the above D. Hemolysis B. fully 417 Which of the following pathological situations can be seen hemoglobinized erythrocytes typical of megaloblastic anemias. normoblasts in which a megaloblastic nucleus is associated with severely hypochromic cytoplasm. 605 B. and a raised serum lactate dehydrogenase provides evidence of ineffective erythropoiesis. 418 Involvement of which of the following structures is uncommon in cobalamin deficiency ? 424 Megaloblastoid morphologic picture of RBC series is seen in ? Harrison’s 16th Ed. appear much less mature than would be expected from the development of cytoplasm (nuclear- cytoplasmic asynchrony). Macrocytosis is less marked with concurrent iron deficiency or thalassemia. alcoholism. D. There may be sphincter disturbances. Orotic aciduria Signs and symptoms of cobalamin deficiency include numbness and paresthesia in the D. 605 A. where the posterior and lateral Myelodysplasia produces a distinct morphologic picture most apparent in orthochromatic columns undergo demyelination. AJAY MATHUR Cardiology 35 416 Hematologic manifestations of cobalamin deficiency are due to ? 422 What value of MCV is diagnostic of megaloblastic anemia ? Harrison’s 16th Ed. in cobalamin deficiency ? Harrison’s 16th Ed. Fenestrated nuclear chromatin in RBC precursors In B 12 deficiency megaloblastic anemia. oval. Graves’ disease D. Hereditary TC I deficiency C. A erythroid ratio & abundant stainable iron. followed by axonal degeneration & eventual neuronal death. 605 A. although very rarely Significant and marked macrocytosis (MCV > 100 fL) suggests presence of a megaloblastic anemia. 603 423 Causes of macrocytosis include all except ? A. 866 A. and the cerebrum itself. CML B. Myxedema C. due to thrombocytopenia. 605 A. Neuronal death B. > 110 fL The hematologic manifestations are almost entirely the result of anemia. and ataxia. > 80 fL B. Megakaryocytes are decreased & show abnormal morphology. Increased myeloid / erythroid ratio C. Thrombocytopenia C. Peripheral nerves A. 603 425 Megaloblastic anemia is seen in all except ? A. Cerebrum D. Hypercellularity B. Leucopenia B. Multiple myeloma D. Axonal degeneration A. > 90 fL C. Positive urine hemosiderin Harrison’s 18th Ed. All of the above . Liver disease Initial pathology is demyelination. This variant is called “megaloblastoid” meaning presence of both nuclear & cytoplasmic maturation 419 Earliest neurologic manifestation of cobalamin deficiency is ? defects. purpura may appear. reduced haptoglobins and positive urine hemosiderin. Motor weakness A. Harrison’s 18th Ed. 605 Harrison’s 16th Ed. Thyroiditis Raised urine urobilinogen. Spinal cord B. Granulocyte precursors are also affected appearing as giant bands and erythropoiesis ? metamyelocytes. All of the above C. Ataxia B. Myelodysplasia D. 867 C. Hyperthyroidism Macrocytosis occurs in hemolysis. which are large. particularly in severely anemic patients. Reduced haptoglobins 427 Incidence of pernicious anemia is increased in ? B. Megaloblastic anemia A. 603 Harrison’s 16th Ed. PBF shows marked anisocytosis & poikilocytosis. 426 Which of the following is not a finding in bone marrow 420 Neutrophils nucleus of >=6 lobes is suggestive of ? examination in folate or B12 deficiency megaloblastic anemia ? Harrison’s 16th Ed. bone marrow is hypercellular with decreased myeloid / Hypersegmented nuclei of neutrophils is a characteristic finding of megaloblastic anemia.” Harrison’s 16th Ed. Cerebellum C. All of the above B. “Megaloblastoid” does not mean “mildly megaloblastic. hypothyroidism & aplastic anemia. A. Pentagastrin-fast achlorhydria 436 Disease due to mutation in cubulin receptor leading to C. 1. 0. due to atrophy of gastric mucosa or autoimmune destruction of parietal cells. Hypergastrinemia & pentagastrin-fast achlorhydria occur secondary to gastric atrophy. achlorhydria and serum IF antibodies are present. 867 D. thyroiditis. B.36 MCQ’s FOR MEDICAL PROFESSIONALS BY PROF. myxedema. Individuals acquire the worm by eating raw or partly cooked fish. Unusually common in agammaglobulinemia Combined deficiencies of cobalamin & folic acid is seen in tropical sprue.K +-ATPase. Vitiligo 434 Which of the following about juvenile pernicious anemia is C. Patients have normal amounts of IF & gastric acid as are other tests of intestinal absorption. Intestinal stasis B. All of the above A. Gastric atrophy does not affect antrum of stomach In Juvenile PA. Gastric epithelium atypia common D. ~80% of patients have anti-IF antibody 435 Removal of what length of terminal ileum causes malabsorption of cobalamin ? D. Destruction of the worm eliminates the D. Average age of presentation is 60 A.D. 867 D.2 meter antibodies are detected in gastric juice in ~80%. D. 867 Removal of 1. 648 of IF. Diphyllobothrium latum. Hypoparathyroidism false ? Harrison’s 18th Ed. Gastric achlorhydria B. A. vitiligo & hypoparathyroidism. Relatives have increased incidence of disease selective defect in cobalamin absorption is ? Harrison’s 18th Ed. Parietal cell antibodies present A. Average age of presentation is 60 years complex. Regional enteritis Harrison’s 18th Ed. . latum causes megaloblastic anemia due Pernicious anemia is unusually common in patients with agammaglobulinemia. 867 Imerslund-Gräsbeck syndrome is a congenital disorder (autosomal recessive) of selective A. Ratio of incidence of PA in men and women among whites is 1:1.8 meter 430 Which of the following about pernicious anemia is false ? D. pepsin. and IF is severely reduced. Most characteristic finding in pernicious to ? anemia is gastric atrophy affecting the acid. Stickler syndrome PA is associated with hypogammaglobulinemia. Pregnancy A. Defective release of cobalamin from food 433 Which of the following about pernicious anemia is false ? B. Mutation occurs in cubulin receptor that mediates intestinal absorption of cobalamin-IF B. Regional enteritis produces cobalamin deficiency by malabsorption due to terminal ileum C. due to competition by the worm for cobalamin. Achlorhydria C. with premature graying or blue eyes. Competition for cobalamin A. but anti-IF antibody is usually absent.15% of random patient population. Helicobacter pylori does not cause parietal cell destruction in pernicious anemia. gastric atrophy. Cogan’s syndrome 431 Which of the following is false about pernicious anemia ? D. B. 868 sparing the antrum. A. Addison’s disease. Hypergastrinemia B. B. None of the above 438 Fish tapeworm . It is uncommon in patients with agammaglobulinemia A.5 meter cell antibody is found in 10 . Hypergastrinemia common Megaloblastic anemia or cobalamin neuropathy is seen in persons heavily infested by fish C. 868 D. None of the above problem. Pregnancy produces folate deficiency due to increased requirements. Addison’s disease B. C. Menkes’ syndrome Harrison’s 18th Ed. None of the above Harrison’s 18th Ed.and pepsin-secreting portion of the stomach while Harrison’s 18th Ed. 0. B. 90% of patients have antiparietal cell antibody C. 0. Gastric atrophy spares the antrum malfunctioning.2 meter Harrison’s 18th Ed. Gastric polyps common tapeworm. Imerslund-Gräsbeck Syndrome Serum gastrin level is raised. C. 867 C. Rare under the age of 30 years defect in cobalamin absorption accompanied by nonspecific proteinuria but renal functions are normal.6. and serum pepsinogen I levels are low. Antiparietal B. AJAY MATHUR Hematology 428 Incidence of pernicious anemia is increased in ? Abnormalities in gastric epithelium appear as cellular atypia that must be distinguished from Harrison’s 18th Ed. Serum IF antibodies present 429 Which of the following about pernicious anemia is false ? Harrison’s 18th Ed. Helicobacter pylori infection is infrequent produces cobalamin deficiency by malabsorption due to defective release of cobalamin from food. Gastric atrophy Incidence of pernicious anemia is increased in patients with other diseases of immunologic origin like Graves’ disease. Inadequate production of intrinsic factor (IF) Harrison’s 18th Ed. Incidence of gastric polyps & stomach cancer is increased. while IF A. In patients with gastric atrophy without pernicious anemia. 867 D. Gastric achlorhydria (disease of elderly) & is rare < 30 years of age. 868 ~90% patients with PA have antiparietal cell antibody directed against H +. Tropical sprue 432 Which of the following about pernicious anemia is false ? C.2 meters of terminal ileum causes malabsorption of cobalamin. 867 cytologic abnormalities of gastric malignancy. C. Men more affected than women D. Caused by the absence of IF 437 Combined deficiencies of cobalamin and folic acid is seen in which of the following conditions ? Pernicious anemia is the most common cause of cobalamin deficiency. It is caused by absence Harrison’s 17th Ed. and in persons of blood group A. antiparietal cell antibody is found in 50%. but parietal cell antibodies are usually absent. Gastric output of hydrochloric acid. Congenital spherocytosis Harrison’s 18th Ed. cholestyramine & triamterene. Chloroform anesthesia and wine do not contain enough of vitamin for daily requirement. Harrison’s 18th Ed. Malabsorption A. Distilled spirits are devoid of folic acid. Inadequate intake B. Harrison’s 18th Ed. particularly in sickle cell B. Pentamidine Nitrofurantoin. 32 weeks of age deficiency. 869 Infestation is common around the lakes of Scandinavia. B. and congenital spherocytosis. All of the above A. Malabsorption D. Values between 100 & 200 A. autoimmune hemolytic anemia. 5 to 100 ng/L The mechanism of folic acid deficiency in tropical sprue is malabsorption. South America 446 Alcohol causes folic acid deficiency by which of the following mechnism ? Individuals acquire fish tapeworm. 869 C. Scandinavia disease. It is given orally or A. 868 D. Autoimmune hemolytic anemia common in which of the following countries ? C. Folinic Acid (5-Formyl-THF) is a stable form of fully reduced folate. D. and tetracycline. B. pyridoxine in pharmacologic doses (~300 mg/day) can be tried. Zidovudine Harrison’s 18th Ed. Japan. A. 869 For the megaloblastic forms of sideroblastic anemia. Folinic acid B. barbiturates). primidone. Nitrous oxide anesthesia D. 160 to 1000 ng/L 445 Folate deficiency frequently occurs in which of the following ? D. Harrison’s 18th Ed. 449 In megaloblastic anemia due to folate antagonists. 24 weeks of age when used in conjunction with sulphamethoxazole in patients with preexisting folate or cobalamin D. Rest of the above B. 869 D. AJAY MATHUR Cardiology 37 439 Infestations by fish tapeworm “Diphyllobothrium latum” is B.10-methylene THF. 869 B. Pyrimethamine Harrison’s 18th Ed. All of the above reductase ? Malabsorption of folate occurs in patients receiving salazopyrine. All of the above D. 12 weeks of age Methotrexate has the most powerful action against the human enzyme. sulphasalazine. Increased requirements Harrison’s 18th Ed. The activity of pyrimethamine is intermediate. 870 Antifolate drugs include anticonvulsant drugs (phenytoin. Neonatal folate level falls rapidly to the lowest values at about 6 weeks of age. Sickle cell disease ng/L are regarded as borderline. Germany. All of the above A. Diphyllobothrium latum by eating raw or partly cooked fish. Tropical sprue A. Increased demand C. Hemodialysis Harrison’s 18th Ed. 870 D.1000 ng/L. Azathioprine B. Pregnancy 450 The normal range of cobalamin in serum is ? C. while beer C. B. which of the following is given ? 443 Folic acid deficiency is due to ? Harrison’s 18th Ed. Triamterene 448 Which of the following drugs is an inhibitor of dihydrofolate D. Ether anesthesia 447 Which of the following drugs is an inhibitor of dihydrofolate Nitrous oxide irreversibly oxidizes methylcobalamin to an inactive precursor which inactivates reductase ? methionine synthase. Trimethoprim 442 Neonatal folate level falls rapidly to the lowest values at ? C. North America & Russia. 869 The normal range of serum cobalamin in serum is 160 . Methotrexate A. whereas trimethoprim is most active against the bacterial enzyme and is only likely to cause megaloblastic anemia C. Impaired metabolism B. 870 441 Malabsorption of folate is seen with which of the following drugs ? A. which circumvents the 444 Conditions that increase demand of folic acid are all except ? block in folate metabolism by providing a form of folate that can be converted to 5. Cholestyramine D. Harrison’s 18th Ed. Salazopyrine C. C. Halothane anesthesia Alcohol interferes with folate metabolism. 871 Harrison’s 18th Ed. South East Asia Folate deficiency frequently occurs in chronic hemolytic anemia. Inadequate intake 440 Acute megaloblastic anemia can be seen after ? B. 37 Hematology MCQ’s FOR MCQ’s FOR MEDICAL MEDICAL PROFESSIONALS PROFESSIONALS BY PROF. Pyridoxine C. Folic acid A. Chronic hemolytic anemias parenterally to overcome the toxic effects of methotrexate or other DHF reductase inhibitors. All of the above C. A. 870. All of the above Megaloblastic anemia due to folate antagonists that inhibit dihydrofolate reductase can be counteracted by folinic acid [5-formyl tetrahydrofolate (THF)] in a dose of 100 to 200 mg/day. C. South Africa D. 100 to 200 ng/L conditions lead to increased folic acid requirements and cause megaloblastic anemia. 1000 to 2500 ng/L Harrison’s 18th Ed. 6 weeks of age Drugs that inhibit DHF reductase include methotrexate. . 869 A. pyrimethamine & trimethoprim. Elevated D. Once a month D. 60 to 80 ng / mL 460 Oral dose of folic acid in folate deficiency is ? D. Homocysteine levels are not used for diagnosis of cobalamin or folate deficiency. Serum folate level 463 Pathway deranged in cobalamin deficiency is ? B. 80 to 100 ng / mL Harrison’s 18th Ed. nonphysiologic fatty acids containing an odd number of carbon 457 Which is a better index of folate stores ? atoms are synthesized and incorporated into neuronal lipids. 12 to 25 µg / L D. in women taking HRT or oral contraceptives. 25 to 45 µg / L Oral dose of folic acid in folate deficiency is 5-15 mg daily for ~4 months because all folate- deficient RBC’s will be eliminated in this time. propionyl CoA. 5 .to 7-day intervals. RBC folate level Harrison’s 16th Ed. hemolytic anemias and gluten- D. 870 A. This biochemical abnormality may also contribute to the neurologic complications of cobalamin deficiency. Chronic dialysis Harrison’s 16th Ed. 2 to 15 µg / L C. Hemolytic anemias A. 605 B. Because of the poorer retention. serum methylmalonate (MMA) level is raised. 603 456 What value of folic acid is diagnostic of folate deficiency ? A. Once every 3 months cobalamin deficiency ? Harrison’s 18th Ed. WBC folate level Harrison’s 18th Ed. 871 455 In folate deficiency due to drugs inhibiting dihydrofolate reductase. Pyridoxine deficiency 458 For cobalamin maintenance therapy. Any of the above induced enteropathy that does not respond to gluten-free diet. 5 . hydroxocobalamin IM is given ? pyridoxine deficiency. Values <=4 ng/mL are considered diagnostic B. steroid & cyclosporine therapy.38 MCQ’s FOR MEDICAL PROFESSIONALS BY PROF. Once every 2 months In patients with cobalamin deficiency sufficient to cause anemia or neuropathy. Serum methylmalonate (MMA) A. <=2 ng/mL D. Chronic renal disease Normal serum folic acid ranges from 6 to 20 ng/mL. 462 Adenosylcobalamin is required for the conversion of ? Harrison’s 16th Ed. in men than in premenopausal women. Hyperthyroidism of folate deficiency. alcoholism. Once every 6 months 453 Normal serum folic acid levels are ? Harrison’s 18th Ed. 42 to 75 µg / L 461 Long-term folic acid therapy is required in which of the Normal range of serum folate is 2 to 15 µg/L. 870 A. Glycine B. Reduced Long-term folic acid therapy is required in chronic dialysis. 870 D. 1000 µg A. C. 871 A. Measurement of RBC folate level is not subject to short-term fluctuations in folate intake & is better than serum folate as an index of folate stores. tissue folate concentrations remain normal. Levels are higher in Harrison’s 18th Ed. Platelet folate level A. 5 . C.15 mg for ~ 8 months B. Succinyl CoA to Methylmalonyl CoA A. AJAY MATHUR Hematology 451 Serum homocysteine is raised in all except ? C. Alcoholism D. following ? Harrison’s 18th Ed. 871 C. Once every 2 months 452 Levels of which of the following is raised in patients with C. smoking. CKD. For maintenance therapy. Methylmalonyl CoA to succinyl CoA Harrison’s 16th Ed.15 mg for ~ 4 months Harrison’s 18th Ed. Propionyl CoA to Methylmalonyl CoA B. Once every 3 months D. 602 . Lack D. 870 D. D. Various drugs can inhibit dihydrofolate reductase thereby producing folate deficiency. monthly.15 mg for ~ 12 months C. All of the above C. As a consequence. Harrison’s 18th Ed. <=3 ng/mL Adenosylcobalamin is required for the conversion of methylmalonyl CoA to succinyl CoA. cyanocobalamin is given 1000 µg IM. tissue folate concentrations are ? A. Iron cyanocobalamin IM is given ? Harrison’s 18th Ed. 1000 µg of Serum homocysteine is raised in early cobalamin & folate deficiency.15 mg for ~ 1 month 454 Normal range of serum folate is ? B. Gluten-induced enteropathy B. Once every 6 months A. 1000 µg of B. <=4 ng/mL of this cofactor leads to large increases in the tissue levels of methylmalonyl CoA and its precursor. Once a month B. 871 serum than in plasma. hypothyroidism. 870 Replenishment of body stores of cobalamin is complete with six 1000-µg IM injections of hydroxocobalamin given at 3. However. <=1 ng/mL C. 30 to 40 ng / mL C. B. Methylmalonyl CoA to Propionyl CoA C. 5 . A. Calcium 459 For cobalamin maintenance therapy. 606 B. 6 to 20 ng / mL hydroxocobalamin IM once every 3 months is satisfactory. Normal C. Dihydrofolate to tetrahydrofolate B. Membrane with folic acid deficiency. weblike network on inner surface of RBC membrane.15 % D. what percentage of RBC precursors are destroyed B. < 300 pg / mL D. 602 470 Clinically significant deficiency of cobalamin is present A. 10 . N -methenyltetrahydrofolate to N -methylenetetrahydrofolate 5 5 in bone marrow ? Harrison’s 16th Ed. Lipid B. Main cytoskeletal protein is spectrin. other factors contributing to anemia (infection. Neurologic B. Cyanocobalamin is biologically active D. 605 Harrison’s 17th Ed.1 / band 4. Anemia Due to Acute Blood Loss 467 Which of the following manifestations occur with cobalamin deficiency but not with folic acid deficiency ? 472 The main cytoskeletal protein is ? Harrison’s 16th Ed. is physically linked to cytoskeleton by proteins (ankyrin & band 4. coexisting iron and/or folate B. Cobalamin is essential for the conversion of methyltetrahydrofolate to tetrahydrofolate. Protamine reduces folate levels in blood D. Most abundant of the membrane proteins Gastrointestinal. Carbohydrate C. Others D. Day 10 Harrison’s 16th Ed. Spectrin Patients with folic acid deficiency are more malnourished than those with cobalamin deficiency. Hemolytic Anemias and Enhanced intramedullary destruction of erythroblasts results in an increase in unconjugated bilirubin and lactic acid dehydrogenase (isoenzyme 1) in plasma. Defect in DNA synthesis and D. 39 Hematology MCQ’s FOR MCQ’s FOR MEDICAL MEDICAL PROFESSIONALS PROFESSIONALS BY PROF. actin with Bands 4.30 % Due to IF deficiency.2). 606 Reticulocytosis begins 4 to 5 days after intramuscular cyanocobalamin therapy is started & A. Megaloblastic anemias are characterized by ineffective erythropoiesis. Isoenzyme 3 D. Spectrin. < 400 pg / mL 465 Cobalamin deficiency without anemia is common in ? Values <200 pg/mL indicate clinically significant cobalamin deficiency. Day 3 B. . Folate is lost in dialysate B. All of the above RBC cell membrane contain ~52% protein. 606 C. as many as 90% of RBC precursors may be destroyed before they are released into the 464 Which of the following about cobalamin is false ? bloodstream. Band 4. < 200 pg / mL C. < 100 pg / mL metabolically active forms B. 40% lipid & 8% carbohydrate by weight.1 & 4. C. Its absence deranges folate metabolism. Cobalamin is an essential cofactor for methionine when its levels are ? synthase & methylmalonyl-CoA synthase Harrison’s 16th Ed. 655 A. AJAY MATHUR Cardiology 39 A. In a severely megaloblastic patient. Patients on chronic hemodialysis require folate supplementation to replace that lost in dialysate. Protein A. Neurologic abnormalities do not occur are glycophorins & band 3 (anion transporter). Harrison’s 16th Ed.60 % methionine. Day 5 466 Which isoenzyme of plasma lactic acid dehydrogenase is increased in enhanced intramedullary destruction of C.2 together form a fibrillar. or hypothyroidism) need to be looked. Hematologic C. Therapeutically available as cyanocobalamin C.1 C. 605 A. Methylcobalamin and adenosylcobalamin are A. 606 B. 25 . 468 Folic acid supplementation is required in patients on chronic hemodialysis because ? 473 Which is the largest component of RBC cell membrane ? Harrison’s 16th Ed. Methyltetrahydrofolate to tetrahydrofolate 469 Normally. Day 7 erythroblasts ? D. Adolescents at about ? C. Adults Harrison’s 16th Ed. or if it is less brisk than expected from the level of hematocrit. If reticulocytosis does not occur. RBC cell membrane (7 nm thick) is a lipid bilayer. 50 . 605 A. Infants 471 Reticulocytosis after intramuscular cyanocobalamin therapy for megaloblastic anemia due to cobalamin deficiency peaks B. 606 D. Isoenzyme 2 deficiency. All of the above D. 75 . hematologic manifestations are similar. Isoenzyme 1 peaks at ~ day 7. Heparin reduces folate levels in blood C. cobalamine is deficient thereby impairing conversion of homocysteine to C. Harrison’s 16th Ed. compared with 10 to 15% in normal individuals. Gastrointestinal A.80 % megaloblastic maturation pattern in patients who are deficient in cobalamin is due to this mechanism (Folate trap hypothesis). N5-methylenetetrahydrofolate to Dihydrofolate A. Band 3 B. Isoenzyme 4 Chapter 106. Glycophorin D. Elderly A. Glutathione 477 Mutation in which of the following gene is not a cause of B.aldolase. Glucose-6-phosphate dehydrogenase (G6PD) hereditary spherocytosis ? C. None of the above B. Increased ratio of RBC surface area to volume C. Increase in MCHC is a characteristic feature. Clostridial infections C. a major C. which maintains B. ATP to ADP 479 Which of the following is false about hereditary B. All of the above A three-enzyme sequence concerned with ATP production is membrane bound . ANK1 G6PD protects RBC’s proteins from oxidative damage by generating NADPH. Phosphoglycerate kinase C.40 MCQ’s FOR MEDICAL PROFESSIONALS BY PROF. Pyruvate Harrison’s 17th Ed. NADP to NADPH Harrison’s 17th Ed. and gallstones. Snake envenomations D. glyceraldehyde-3 phosphate dehydrogenase (G-3PD) & phosphoglycerate kinase. phosphatidylethanol. oxygen delivery to tissues is increased. Osmotic fragility test B. A. 656 condition in which high MCHC is seen. 654 D. Aldolase A. Red cell survival study A. SPTA1 high levels of reduced glutathione. All of the above C. 653 A. NADPH then provides the reducing power that converts oxidized glutathione (GSSG) to reduced glutathione (GSH). Glutathione protects RBC’s against oxidant stress. Thus. 608 A. None of the above 486 Which of the following protects the patient from malaria ? Main clinical findings of HS are jaundice. 609 A. ANK1 D. Erythropoietin (EPO) A. C. EPB42 which of the following pathways ? Mutations of SPTA1 gene account for ~65% of Hereditary elliptocytosis (HE). Harrison’s 17th Ed. Harrison’s 17th Ed. A. 655 A. Oral folic acid should be given constantly A. Sphingomyelin A modified version of osmotic fragility test is called the “pink test”. NADPH to NADP A. SLC4A1 effector of hemoglobin-oxygen dissociation curve. an enlarged spleen. Hexose monophosphate shunt Harrison’s 16th Ed. Harrison’s 17th Ed. HS is the only Harrison’s 17th Ed. they 481 Pink test is a modified version of which of the following ? convert fructose diphosphate to 3-phosphoglycerate with production of ATP. SLC4A1 484 Glucose-6-phosphate dehydrogenase (G6PD) is related to D. Mean corpuscular volume usually normal G6PD is a HMP shunt enzyme. High reticulocytosis 476 Which of the following gene mutation accounts for majority of autosomal dominant hereditary spherocytosis (HS) ? B. Lecithin & sphingomyelin can substitute each other. RBC survival after splenectomy is normal breakdown of oxidant compounds like H 2O 2. Reduced glutathione protects against oxidant injury by catalyzing C. Purine salvage pathway B. RBC absolute values 475 Phospholipid of RBC membrane is ? B. sphingomyelin & phosphatidylserine. Due to defect in ankyrin / protein 3 / spectrin / palladin G6PD reduces NADP to NADPH while oxidizing glucose-6-phosphate (G6P) to 6-phosphogluconate B. Pigmented gallstones are common (6PG). All of the above Most of phospholipid of RBC membrane is phosphatidylcholine (lecithin). ADP to ATP spherocytosis ? C. D. A. Phosphatidylethanolamine D. G6PD Deficiency . SPTA1 Metabolic block at the last step in glycolysis increase bisphosphoglycerate (or DPG). D. Splenomegaly is very common D. Mean corpuscular hemoglobin concentration increased 485 Which of the following reactions is releted to G6PD ? Anemia in HS is normocytic. Glyceraldehyde-3 phosphate dehydrogenase (G-3PD) B. Cirrhosis liver B. 653 Figure 101-1 of autosomal dominant hereditary spherocytosis (HS). 610 regarded as an obligatory therapeutic measure in HS. D. Schilling test C.2 producing ankyrin protein accounts for majority Harrison’s 17th Ed. 655 Table 101-3 C. D. 655 A. Splenectomy is Harrison’s 16th Ed. EPB41 483 Which of the following protects RBC’s against oxidant stress ? Mutations in ANK1 gene on chromosome 8p11. 482 Which of the following is false about pyruvate kinase deficiency ? amine. AJAY MATHUR Hematology 474 Enzyme required for the production & utilization of ATP in 480 Spherocytes are seen in ? RBC cell membrane is ? Harrison’s 16th Ed. Together. Embden-Meyerhof pathway 478 Which of the following is false about hereditary spherocytosis ? B. Oxygen delivery to tissues is increased Harrison’s 17th Ed. Phosphatidylcholine (lecithin) C. All of the above D. 655 Table 101-3 D. It is the most common enzymatic disorder of RBC’s in humans. Raised LDH Vitamins that G6PD deficient persons should avoid include vitamin C & K. A. Zinc D. splenomegaly. Hereditary ovalocytosis C. 610 poikilocytes with RBC’s having unevenly distributed hemoglobin are called hemighosts. Primaquine B. reticulocytosis. All of the above C. but older RBC’s are markedly deficient. hemoglobinemia. 611 497 Which of the following element is essential for the activity A. X chromosome 493 Which of the following is a peripheral blood finding of B. Hemoglobinemia B. 656 A. giving rise to “bite cells”. and G6PD Union worldwide. causes oxidation of sulfhydryl groups of globin chains leading to denaturation of hemoglobin & formation of precipitates (Heinz bodies) which damage RBC membrane to cause intravascular hemolysis. Mefloquine C. always in a male. Any of the above B. Mode of transmission of G6PD deficiency is X- linked recessive. Vitamin E C. unconjugated hyperbilirubinemia. Heinz bodies D.((10 . Immature . 657 B. 656 C. History of neonatal jaundice (NNJ) D. Mercury 492 Which population of RBC is rapidly destroyed during D. C. 658 B. Selenium hemolysis in G6PD deficient subjects ? Harrison’s 17th Ed. D. All of the above expressed in all erythrocytes of the affected male. Vitamin A A. Autosome A. Vitamin D B. spherocytes. Type B A. Dimer Most common normal “wild type” G6PD enzyme is designated as G6PD-B.type is present in ~10% of American blacks. some deficient. G6PD Canton in China. 490 Which of the following ‘antimalarials’ can cause hemolysis hemoglobinuria. Cobalt C. macrophages pluck out or bite Heinz 488 The normal G6PD is designated as ? bodies. In heterozygous female. poikilocytosis. AJAY MATHUR Cardiology 41 B. Chloroquine Harrison’s 17th Ed. Hemoglobin S B. Acute hemolytic anemia due to G6PD deficiency leads to anisocytosis. Bizarre Harrison’s 16th Ed. Exposure to oxidants chromosomes. 611 496 Which of the following is false about CNSHA ? A. low or absent plasma haptoglobin. G6PD Vianchan & G6PD Mahidol in Southeast Asia. 656 Infantile poikilocytosis is due to deficiency of glutathione peroxidase (GSHPx) due to nutritional deficiency of selenium which is an essential element for the activity of GSHPx. Raised plasma haptoglobin D. Trimer designated G6PD A. Viral & bacterial infections of glutathione peroxidase (GSHPx) ? Harrison’s 17th Ed. Chronic hemolysis Deficiency of G6PD protects against malaria due to Plasmodium falciparum. Hemoglobinuria C. Naphthalene A. Y chromosome hemolysis in G6PD deficiency ? C. G6PD D. 487 The G6PD gene is located on ? Exposure to oxidants induces hemolysis of older red cells but not of younger ones. A. Mature C. Patient is always a male C. in G6PD deficient subjects ? Harrison’s 16th Ed. 489 Which of the following ‘vitamins’ can cause hemolysis in G6PD deficient subjects ? 495 Which of the following is false about acute hemolytic anemia Harrison’s 16th Ed. Enzymatically active form of G6PD is either a dimer or a tetramer of a single protein subunit of 514 amino acids. raised LDH. in G6PD deficient subjects ? Harrison’s 16th Ed. Quinine A. Any of the above B. 611 due to G6PD deficiency ? Harrison’s 17th Ed. Defect is D.60% of activity) and G6PD Mediterranean (<10% of normal activity). with history of NNJ. None of the above Chronic nonspherocytic hemolytic anemia (CNSHA) is a severe clinical phenotype of G6PD 491 Which of the following can precipitate an episode of hemolysis deficiency. All of the above G6PD enzyme activity is normal in reticulocytes. A. gallstones. lead to clinically significant hemolysis. polychromasia. Monomer D. Membrane damage induces formation of spherocytes. Vitamin E is protective. 41 Hematology MCQ’s FOR MCQ’s FOR MEDICAL MEDICAL PROFESSIONALS PROFESSIONALS BY PROF. Type A (-) Harrison’s 17th Ed. Bite cells or blister cells C. Two variants. 657 A. Type A (+) 494 The enzymatically active form of G6PD is a ? B. It follows that males are more vulnerable to oxidant injury than females are. Vitamin K D. Harrison’s 17th Ed. Metabolic acidosis B. All of the above D. Older D. All of the above Mediterranean is found in Middle East. two populations of red cells. are present owing to random inactivation of the X Hemolysis in G6PD deficiency is both intravascular & extravascular. Heinz bodies decrease RBC deformability. others normal. Spherocytes Gene for G6PD is located on X chromosome. dapsone. Dapsone AIHA is usually classified on the basis of thermal amplitude of the autoantibody. whereas cold AIHA is caused by an IgM antibody that fixes complement B. Hemolytic Uremic Syndrome (HUS) test (IAT) ? Autoimmune Hemolytic Anemia (AIHA) is the most common form of acquired hemolytic anemia Harrison’s 17th Ed. 659 in areas where malaria is not endemic. Enzymopathies C.microangiopathic hemolytic anemia. Direct agglutination test diagnosis of chronic Coombs-negative HA ? C. ANy of the above Harrison’s 17th Ed. 656 D. cisplatin cause hemolysis by their oxidative potential. IgG antibodies cannot cause direct RBC agglutination. A. Disseminated intravascular coagulation (DIC) Harrison’s 17th Ed. Warm AIHA is caused by IgG antibody. 659 Enzymopathies (G-6-PD deficiency. 659 D. Paroxysmal Cold Hemoglobinuria (PCH) 502 Which of the following is false about direct antiglobulin test (DAT) ? D. Reagent RBC’s are incubated in with serum Harrison’s 16th Ed. B. Nonimmune [Coombs test (–)] hemolytic anemia Some may develop overt hemolysis. Direct antibody test A. developing several days after diarrhea. Anti-IgG antihuman globulin (AHG) reagent is used A. Erythrocytes are washed before the test where malaria is not endemic is ? Harrison’s 17th Ed. Paroxysmal Cold Hemoglobinuria (PCH) but if RBC’s are coated with IgG antibodies. through mimicry. Reflects in-vivo antibody sensitization of RBC’s differential diagnosis of any chronic Coombs-negative HA. C. Basophilic stippling is highly distinctive feature of P5N deficiency. C. 659 D. A. C. IgM antibody 499 Drug that can cause hemolysis by depletion of ATP is ? D. Ribavirin to the surface of the RBC. Direct antiglobulin test 507 Which of the following should be considered in differential B. Methyldopa. pyruvate kinase deficiency). A failure due to thrombosis of glomerular capillaries. Cisplatin 505 Conditions associated with hemolysis and a negative DAT D. nitrates. B. Reflects in-vitro antibody sensitization of RBC’s when which of the following disease was prevalent ? C. AB Harrison’s 17th Ed. HA caused by lead poisoning is also characterized by basophilic stippling. chlorates. Leucopenia 506 RBC’s of which blood group are used in indirect antiglobulin Shiga toxin producing Escherichia coli O157:H7 is an etiologic agent of HUS. Reflects in-vitro antibody sensitization of RBC’s 508 Most common form of acquired hemolytic anemia in areas C. Chapter 313 Table 313-1 Ribavirin causes hemolysis by depletion of ATP. B. B. Direct antigen test Harrison’s 17th Ed. DIC). Hyperbaric oxygen. AHG reagent will cause them to agglutinate. AJAY MATHUR Hematology 498 Basophilic stippling is a highly distinctive feature of ? D. thrombocytopenia & acute renal A. B. they bind to their target antigens on reagent RBC’s. Acute hemolytic transfusion reaction Harrison’s 17th Ed. Hemoglobinopathies as a hapten & induces antibody production leading to hemolysis. Liver disease. Systemic lupus erythematosus (SLE) C. Used to detect IgG antibodies in serum 509 Paroxysmal cold hemoglobinuria (PCH) was more frequent B. B. Erythrocyte membranopathies (spherocytosis). All of the above 504 Cold Autoimmune Hemolytic Anemia (AIHA) is caused by ? P5N is a key enzyme in catabolism of nucleotides arising from degradation of nucleic acids that A. D. Microangiopathic hemolytic anemia Hemoglobinopathies (sickle cell disease. B. Penicillin acts A. pyruvate kinase deficiency) should be considered in the A. Erythrocyte trauma (mechanical heart valves). Thrombotic thrombocytopenic purpura (TTP) 500 Which of the following about HUS due to Shiga toxin producing Escherichia coli O157:H7 is false ? D. Pyrimidine 5'-Nucleotidase (P5N) Deficiency D. B 501 Direct Coombs test is also called ? C. Anti-IgG antihuman globulin reagent is not used Harrison’s 17th Ed.42 MCQ’s FOR MEDICAL PROFESSIONALS BY PROF. Methyldopa result include all except ? Harrison’s 17th Ed. thalassemia). After an incubation period. Autoimmune Hemolytic Anemia (AIHA) 503 Which of the following is false about indirect antiglobulin D. If Ab’s are present. 659 A. 614 . IgA antibody takes place in final stages of RBC maturation. HUS is a nonimmune [Coombs test (–)] hemolytic anemia with classical triad . Drug-induced hemolytic anemias Direct Coombs test is also called Direct antiglobulin test (DAT). Reagent RBC’s are incubated with serum that potentially contains antibodies. more common in test (IAT) ? children. Hypersplenism. B. Anti-IgG AHG reagent is then added and will cause IgG-coated RBC’s to agglutinate. C. Autoimmune Hemolytic Anemia (AIHA) sensitization). 658 Indirect antiglobulin test (IAT) is used to detect the presence of IgG antibodies in serum (in-vitro A. Thrombocytopenia Infectious diseases (Clostridium difficile infection). 659 Antierythrocyte membrane autoantibodies in SLE have positive direct Coombs’ test (60% prevalence). Familial Hemolytic Uremic Syndrome (HUS) RBC’s are washed to remove unbound antibodies. RBC’s are washed to remove unbound Ab’s & anti-IgG AHG reagent is then added. IgG antibody C. produces of an antibody (Rhesus antibody anti-e) against RBC antigen causing hemolysis. Erythrocyte enzymopathies (G-6-PD deficiency. O A. Conditions associated with hemolysis and a negative DAT result are Microangiopathic hemolytic anemias (TTP. methylene blue. Leucocytosis can cause in HUS. Paroxysmal Nocturnal Hemoglobinuria (PNH) DAT reflects in-vivo antibody sensitization of RBC’s. C. CD39 early block in the synthesis of glycosylphosphatidyl-inositol (GPI) anchors. CD59+. which tether many proteins to the cell surface. B. Intracorpuscular defect acquired at stem cell level B. PIG-A mutations cause an B. CD29 the X-linked gene called PIG-A (phosphatidylinositol glycan class A). CD55+ D. Analysis of GPI-linked proteins (CD59. All of the above proteins. the blood cells in PNH have a partial deficiency C. Antibody is produced by expanded clone of B lymphocytes and may show as a spike in plasma protein electrophoresis A. Sucrose lysis test C. venous thrombosis & bone marrow failure. Arterial thrombosis D. 661 Harrison’s 16th Ed. Terminal pathway is common to these three pathways 515 Which of the following is false about Paroxysmal Nocturnal leads to membrane attack complex that lyses cells. Inhibiting C3 convertases B. Harrison’s 17th Ed. 43 Hematology MCQ’s FOR MCQ’s FOR MEDICAL MEDICAL PROFESSIONALS PROFESSIONALS BY PROF. CD55 514 Which of the following is the action of CD59 ? is called decay-accelerating factor (DAF). and in the presence of complement when temperature becomes 37°C. CD55– is diagnostic of PNH. 616. and CD59. Donath-Landsteiner antibody Harrison’s 17th Ed. 661 RBC’s strongly at lower temperatures (cold exposure). Granulocytopenia & thrombocytopenia 511 Cold Agglutinin Disease (CAD) is related to which of the following ? D. mannose- binding lectin and alternative pathways. CD55 (decay-accelerating factor). B. Binds C3b A. Mannose-binding lectin pathway D. Protein calorie malnutrition D. When A. Tertiary syphilis B. 660 . Acidified serum lysis test (Ham’s test) B. BRCA1 membrane protein ? D. intravascular hemolysis & hemoglobinuria occurs. CD55– B. PIG-A 512 Which of the following is the most important protective RBC C. 661 519 Pathway of activation of the complement system is called ? A. DAF) D. CD59 blocks D. CD55– A. B. CD49 (type II) or a complete deficiency (type III) of GPI-linked proteins. AJAY MATHUR Cardiology 43 A. 619 A. Familial cold autoinflammatory syndrome (FCAS) thrombosis affects hepatic veins. 518 Which of the following tests is useful in diagnosing 513 Which of the following is diagnostic of PNH ? Paroxysmal Nocturnal Hemoglobinuria ? Harrison’s 17th Ed. Alternative pathway Erythrocytes defend themselves from membrane attack complexes with two membrane-bound D. and the continuous sloughing of these cells into the urine culminates in iron deficiency. which inhibits the insertion of C9 into the membrane.344:1058 B. The hemoglobin-drenched B. Systemic Mastocytosis 517 In Paroxysmal Nocturnal Hemoglobinuria. name of the defective gene is ? CAD is a form of chronic AIHA & affects elderly. CFTR Harrison’s 17th Ed. lysis of RBC’s.the classical. Hemolysis due to activation of complement RBC’s only at a low temperature (~4°C). RBC’s have an increased susceptibility to complement (C) due to the deficiency on their surface of CD59 & CD55 proteins that normally protect RBC’s from activated C. CD59+. Ki-67 monoclonal antibody 516 Which of the following is false about Paroxysmal Nocturnal Hemoglobinuria (PNH) ? D. Hemoglobin-drenched renal tubules lock iron of Hb into hemosiderin and continuous A. Classical pathway C. Gonorrhoea C. 660 Antibody in PCH is Donath-Landsteiner antibody which has anti-P specificity and binds to A. All of the above C. when Harrison’s 17th Ed. Anti-Smith antibody C. 660 In PNH. Urine is black when patient awakens. May present as Budd-Chiari syndrome Harrison’s 17th Ed. Hemosiderinuria is usually absent C. Anti-Jo-1 antibody sloughing of these cells into urine culminates in iron deficiency. All of the above Discrete population of cells that is CD59–. CD59 (membrane inhibitor of reactive lysis) is a protective RBC membrane protein. CD59 the formation of terminal complement complex (membrane-attack complex) on the cell surface. 660 PNH results from clonal expansion of hematopoietic stem cells that have somatic mutations in A. Thomas Hale Ham (1938) developed Ham’s test at Thorndike Laboratory of Boston City Hospital and reported that RBC’s in PNH are highly susceptible to lysis in an acidic environment. Consequently. CD59–. CD55+ C. Hemolysis often occurs during sleep. HNF4 resembling monoclonal gammopathy and may be related to Waldenström macroglobulinemia (WM). Hemoglobinuria (PNH) ? Harrison’s 17th Ed.complement-dependent acquired chronic intravascular hemolytic 510 Antibody in Paroxysmal cold hemoglobinuria (PCH) is ? anemia. not at all at 37°C. There are three pathways of activation of the complement system . CD59–. 660 decline in blood pH triggers activation of complement components. Budd-Chiari syndrome occurs. Cold urticaria renal tubules lock iron of hemoglobin into hemosiderin. Autoantibody (IgM with anti-I specificity) reacts with Harrison’s 17th Ed. Tuberculosis A. Inhibits insertion of C9 into cell membrane N Engl J Med 2001. Waldenström macroglobulinemia (WM) D. Hemolytic anemia C. Bone marrow failure PNH has three attributes . thereby preventing erythrocyte lysis and in vitro platelet activation. Intravascular hemolysis is the consequence of absence of GPI-linked complement regulatory protein CD59. which binds C3b. All of the above B. Low or absent is significant hemolysis B. 4 Harrison’s 16th Ed. N Engl J Med 2004. 527 Urine is positive with benzidine reaction in ? Harrison’s 16th Ed. None of the above 3 activation pathways of complement converge to generate C3 convertase that cleaves C3.0 g/L A. Severe hepatocellular disease Harrison’s 16th Ed. the level of unconjugated bilirubin never exceeds 4 to 5 mg/dL unless liver function is impaired. 49 ºC 529 Drug that cause immunohemolytic anemia is ? RBC membrane is unstable at temperatures > 49°C due to denaturation of cytoskeletal protein Harrison’s 16th Ed. malaria & babesiosis directly parasitize RBC & cause severe hemolysis. C. C1 B. Methylene blue A. Eculizumab is a recombinant humanized monoclonal antibody that binds specifically to terminal complement protein C5. is elevated by accelerated RBC destruction.44 MCQ’s FOR MEDICAL PROFESSIONALS BY PROF. C. Hematuria B. HELLP syndrome 525 Which of the following is false about ‘Haptoglobin’ ? C. Harrison’s 16th Ed. Crystal violet B. A. 38 ºC D. 2 Bartonellosis. thereby preventing release of 528 Which of the following stains is used to detect hemosiderin inflammatory mediator C5a & formation of the cytolyticpore C5b–C9. 615 LDH. C5 mononuclear phagocyte system D. Quinidine A. Serum concentration is ~1. -methyldopa 523 In hemolysis. 3 531 Spur cell anemia is due to ? D. 615 Haptoglobin binds specifically to the globin in hemoglobin. 4 to 5 mg / dL 530 Which of the following can directly parasitize RBC’s & cause severe hemolysis ? In patients with hemolysis. Decreased in hepatocellular disease 532 Surface membrane of a spur cell (acanthocytes) contain excess of ? D. 608 microorganisms & clearance of immune complexes.350:553 A. 608 C. 46 ºC a significant amount of circulating free hemoglobin has been filtered by the kidneys. Hemolytic-uremic syndrome A. Phospholipid . All of the above B. 615 A. Hemoglobin-haptoglobin complex is cleared rapidly by C. 1 to 2 mg / dL D. C4 C. AJAY MATHUR Hematology 520 Pathways of activation of the complement system converge 526 Which of the following is false about ‘Haptoglobin’ ? at the point of cleavage of ? Harrison’s 16th Ed. Serum AST (SGOT) may A. Malaria by accelerated RBC destruction ? Harrison’s 16th Ed. C3 C. All of the above D. particularly LDH-2. A. penicillin & quinidine. 608 521 Eculizumab binds specifically to ? Harrison’s 17th Ed. C4 D. 612 Table 93-7 spectrin. Hemoglobinuria A. C5 Urine is positive with benzidine reaction in hemoglobinuria. level of unconjugated bilirubin never exceeds ? B. Binds to heme in hemoglobin bizarre-shaped RBC occuring in severe hepatocellular disease (Laennec’s cirrhosis). Increased in inflammatory states Harrison’s 16th Ed. Penicillin Harrison’s 16th Ed. 42 ºC Prussian blue is used to detect presence of hemosiderin in urine. 2 to 3 mg / dL Drugs causing warm antibody immunohemolytic anemia are -methyldopa.  globulin Spur cells are irregularly spiculated acanthocytes. 1 D. 615 C. inhibiting its cleavage into C5a and C5b. Bartonellosis 524 Which fraction of lactate dehydrogenase (LDH) is elevated B. 608 D. C3 B. hematuria and myoglobinuria. Prussian blue 522 RBC membrane is unstable at temperatures above ? B. D. 608 N Engl J Med 2001.344:1058 A. C. Babesiosis A. B. Clostridium welchii infection be elevated but ALT (SGPT) is not. A. 608 C. C6 D. Spur cell anemia is a hemolytic anemia with B. Blockade of complement in urinary sediment ? cascade at C5 preserves early components of complement that are essential for opsonization of Harrison’s 16th Ed. 3 to 4 mg / dL D. Giemsa’s stain Harrison’s 16th Ed. 661. Positive result indicates that C. Myoglobinuria C. regularly spaced. All of the above Harrison’s 16th Ed. 889 MDS & leukemia. All of the above C. Hepatitis 535 Most cases of aplastic anemia are ? B. Myelodysplasia (MDS) Posthepatitis aplastic anemia is typically seronegative (non-A. Leukemia 544 Aplastic anemia is related to ? D. Teens D. C A. A. All of the above B. Uremia Harrison’s 17th Ed. Infectious mononucleosis Harrison’s 18th Ed. non-B. Idiopathic D. Eosinophilic fasciitis C. A. Harrison’s 18th Ed. C. All of the above Harrison’s 18th Ed. chloramphenicol. Due to drug exposure Parvovirus B19 is the cause of transient aplastic crisis in hemolytic anemias. MDS D. Congenital dyserythropoietic anemia 541 Cause of transient aplastic crisis in hemolytic anemias is ? Hypoproliferative anemia is a prominent feature of bone marrow failure states like aplastic Harrison’s 18th Ed. 887 C. B Harrison’s 18th Ed. non-C. The age distribution is biphasic. 615 Figure 93-8 539 Which of the following has consistent association with aplastic A. Chapter 107. are late effects of radiation. Aplastic anemia C. Glycoprotein A. 45 Hematology MCQ’s FOR MCQ’s FOR MEDICAL MEDICAL PROFESSIONALS PROFESSIONALS BY PROF. hydantoins. Schnitzler’s syndrome A. HELLP syndrome B. men & women are affected equally. Severe hepatocellular disease anemia ? B. 887 B. This decreases membrane fluidity & cell deformability forbiding their passage B. 663 Table 102-3 C. A 534 Which of the following is not a hypoproliferative anemia ? B. Pure red cell aplasia (PRCA) due to as yet undiscovered infectious agent. 887 C. In aplastic anemia. D. myelodysplasia (MDS). 889 anemia. In such RBC’s numerous. carbamazapine. Systemic lupus erythematosus (SLE) B. non-G) & probably C. All of the above Benzene. Children C. Fanconi’s anemia . Aplastic anemia D. Transfusion-associated graft-versus-host disease (GVHD) Harrison’s 18th Ed. small C. Echinocytes are a frequent artifact in PBF. C. Phenylbutazone Burr cells or echinocytes are seen in uremia. age affected is ? A. 663 Table 102-3 D. Parvovirus B19 A. 888 B. Cholesterol 538 Which of the following has consistent association with aplastic C. with the 543 Aplastic anemia is strongly associated with which of the following ? major peak in the teens and twenties and a second rise in older adults. None of the above B. Young adults Aplastic anemia is strongly associated with eosinophilic fasciitis which is characterized by D. Gold D. Myelodysplasia. pure red cell aplasia (PRCA) and myelophthisis. Chloramphenicol through filtering system of spleen. Aplastic Anemia. Teens & twenties and older adults painful induration of subcutaneous tissues. AJAY MATHUR Cardiology 45 B. Harrison’s 18th Ed. D. Due to immune diseases D. 889 A. Due to inherited disorders 542 Aplastic anemia is strongly associated with which of the following collagen vascular syndrome ? Most cases of aplastic anemia are idiopathic. Cimetidine spiny projections are seen. gold. Triglyceride anemia ? Harrison’s 17th Ed. phenylbutazone show most consistent association with AA. cimetidine. Carbamazapine 533 ‘Burr cells’ are found in ? D. DIC A. Wegener’s granulomatosis B. and Related Bone 540 Which of the following hepatitis virus infections most often precede posthepatitis marrow failure ? Marrow Failure Syndromes Harrison’s 18th Ed. but its total phospholipid content is normal. but probably not aplastic anemia. quinacrine. Eosinophilic fasciitis A. Benzene Surface membrane of a spur cell contains 50 to 70% excess cholesterol. Polyarteritis nodosa Harrison’s 18th Ed. 889 536 In aplastic anemia. 889 537 Which of the following is not a late effect of irradiation ? A. None of the above 549 Which cell in the bone marrow is helpful in differentiating aplastic anaemia from hypoplastic myelodysplastic syndromes ? Aplastic anemia is a disease of young. Malabsorption 554 Which of the following is false ? Harrison’s 18th Ed. aplastic anemia develops during childhood. are greatly diminished. TERC No immature myeloid forms and abnormal platelets. Normal lymphocyte number genes ? D.46 MCQ’s FOR MEDICAL PROFESSIONALS BY PROF. BM biopsy shows mainly fat under the microscope. Absolute reticulocyte count < 60. Lymphadenopathy Fanconi’s anemia is an autosomal recessive disorder. Weight loss 545 Which of the following about Fanconi’s anemia is false ? D. the autosomal dominant type is due to mutation in TERC. Mucosal leucoplakia Harrison’s 18th Ed. A. C. Risk of aplastic anemia B. Nail dystrophy 552 Which of the following is not a feature of blood smear in aplastic anemia ? D. Eosinophilia A. In all the single lineage failure syndromes. nail dystrophy and mucous membrane leukoplasia. CD38 positive stem cells Harrison’s 18th Ed. progression to compound heterozygous mutations in SBDS. Jaundice Harrison’s 18th Ed. C. with hematopoietic cells occupying <25% of marrow space. Reticulocytes are absent or few & lymphocyte numbers may be normal or reduced. A.000 / µL Bone marrow failure results from severe damage to hematopoietic cell compartment. which encodes the catalytic reverse transcriptase. B. Autosomal dominant disorder B. and TERT. All of the above A. B. In AA. 893 C. Megakaryocytes greatly reduced / absent D. café au lait spots 551 Which of the following is unusual in aplastic anemia ? C. Platelet count < 20. which encodes an RNA template. Bleeding Fanconi’s anemia presents as progressive pancytopenia. PBF shows large erythrocytes & reduced platelets & granulocytes. 889 Infection on presentation. 891 Dyskeratosis congenita is an inherited bone-marrow-failure syndrome in childhood & presents A. Systemic complaints and weight loss should point to other etiologies of pancytopenia. 891 D. Increased risk of malignancy A. C. pancytopenia or leukemia is unusual D. progression Harrison’s 18th Ed. Bone marrow from PNH patients show evidence of defective hematopoiesis and may later develop frank marrow aplasia & B. Lancet 2005. a marker of early hematopoietic cells. Absolute neutrophil count < 500 / µL D. 889 In AA. In Dyskeratosis congenita. Abnormal skin pigmentation anemia. Aplastic anemia is a disease of the young D. 890 D. lymphadenopathy & splenomegaly are atypical of aplastic A. 889. CD34 positive stem cells 555 Severe aplastic anemia is defined by ? B. and bone marrow failure due to C. TERT 553 Which of the following is not a feature of bone marrow D. Few or absent reticulocytes 547 Dyskeratosis is due to mutations in which of the following C. CD42 positive stem cells A. All of the above Harrison’s 18th Ed. malabsorption. Pancreatic insufficiency C. Any of the above cytology in aplastic anemia ? Harrison’s 18th Ed.365:1647–56 to pancytopenia or leukemia is unusual. Infection pancytopenia.000 / µL cells bearing CD34 antigen. Reduced platelets and granulocytes Harrison’s 18th Ed. Agranulocytosis is more frequent in elderly & women Schwachman-Diamond syndrome is an autosomal recessive disorder characterised by pancreatic exocrine dysfunction. telomerase. B. CD46 positive stem cells B. a component of the shelterin. DKC1 increased. AJAY MATHUR Hematology B. In single lineage failure syndromes. Type A is due to mutation in FANCA Harrison’s 18th Ed. Splenomegaly 546 Dyskeratosis congenita is characterized by all except ? D. also occur in dyskeratosis. Decreased mean corpuscular volume (MCV) with the triad of reticular hyperpigmentation. 889 In AA. Dilute aspirate (dyskerin) gene. MCV is commonly A. C. Hematopoietic cells occupying < 50 % of marrow Mutations in TNF2. Dyskeratosis congenita in aplastic anemia ? Harrison’s 18th Ed. bone marrow is readily aspirated but dilute on smear. proteins that bind the telomere DNA. Most common early symptom is bleeding. . B. committed & primitive progenitor cells are virtually absent. Pancreatic insufficiency B. 891. Paroxysmal nocturnal hemoglobinuria 550 Which of the following is the most common early symptom C. Short stature. C. Mild megaloblastic erythropoiesis 548 Shwachman-Diamond syndrome features include all except ? Harrison’s 18th Ed. metaphyseal dysostosis. Infection on presentation B. 891 C. 889 Aplastic anemia can have abrupt or insidious onset. 891 Dyskeratosis congenita is due to mutations in genes of the telomere repair complex that maintain telomere length in replicating cells: The X-linked variety is due to mutations in DKC1 A. A. Immunological deficiencies are common anemia should be between ? Harrison’s 18th Ed.000/µL & corrected reticulocyte count <1% (or absolute reticulocyte count <60. Bone Marrow Transplantation A. Zinc A. 559 With standard regimen of ATG + cyclosporine for AA. Glucocorticoids treatment include all except ? C. Hepatotoxicity cells by stem cell transplant. A. Horse ATG is given at 40 mg/kg per day for 4 days. 2 mg/kg per day Severe AA is defined by the presence of two of three parameters : absolute neutrophil count B. or suppression of immune system. 40 mg/kg per day for 1 day C. Nephrotoxicity Treatment options in severe acquired aplastic anemia are replacement of the absent hematopoietic B. 25 % D. 892 C. 10 % A. 20 C. All of the above A. 10 B. 2 months of treatment B. hypertension. 892 A. Azithromycin Harrison’s 17th Ed. C. 3 months of treatment C. 150 and 200 ng / mL D. 892 564 Dose of Horse ATG in treatment of aplastic anemia is ? A. MDS develops in 15 % of patients. Hematopoietic growth factors (HGFs) 563 Most important side effects of chronic cyclosporine B. AJAY MATHUR Cardiology 47 D. 561 In adults. 450 and 600 ng / mL 557 Which of the following have no value in treating severe D. iron chelators (deferoxamine & deferasirox) are given at around the fiftieth transfusion to avoid secondary hemochromatosis. MDS develops in 566 In chronic anemia. seizures & opportunistic infections.200 ng/mL. 892 Harrison’s 18th Ed. 40 mg/kg per day for 3 days Addition of cyclosporine to ALG (antilymphocyte globulin) or ATG (antithymocyte globulin) D. Very severe disease is defined as absolute neutrophil count <200/µL. Total reverse isolation With standard regimen of ATG + cyclosporine for AA. iron chelators are given after how many what percentage of patients ? blood transfusions ? Harrison’s 18th Ed. Vitamin E C. 891 Trough cyclosporine blood levels in treatment of AA should be between 150 . 893 A. 8 mg/kg per day <500/µL. 250 and 400 ng / mL C. 4 months of treatment D. increase response rates in aplastic anemia ? Harrison’s 18th Ed. 893 Harrison’s 17th Ed. 20 % C. Complete recovery can occur with effective immunosuppressive therapy B.000/µL). 1 month of treatment A. CD34+ cells greatly reduced 562 Trough blood levels of cyclosporine in treatment of aplastic B. Glucocorticoids are of no value. D. 892 Harrison’s 18th Ed. Prompt institution of parenteral. Hypertension adjuncts to immunosuppression are not clear. 650 and 800 ng / mL acquired aplastic anemia ? Harrison’s 18th Ed. Hematopoietic growth factors (HGFs) are not recommended as initial therapy for severe aplastic anemia. 40 mg/kg per day for 2 days D. Nonabsorbed antibiotics for gut decontamination D. Normal lymphocyte count A. initial oral dose of cyclosporine in treatment of 567 PRCA is characterized by ? aplastic anemia is ? Harrison’s 18th Ed. 892 D. 20 mg/kg per day 556 Which of the following about aplastic anemia is false ? Cyclosporine is administered orally at an initial dose of 12 mg/kg per day in adults Lancet 2005. broad-spectrum antibiotics B. platelet count <20. improvement in granulocyte number is generally apparent 565 The single best method of preventing the spread of infection within ? while treating patients of aplastic anemia is ? Harrison’s 18th Ed. 560 In aplastic anemia treatment responders. 667 B. 12 mg/kg per day D. and even their roles as C. 667 . the single best method of preventing the spread of infection. neglected practice. Hand washing C. In chronic anemia. 40 D. improvement in granulocyte number is Hand washing. Antithymocyte globulin (ATG) Harrison’s 18th Ed. Cyclosporine B. 15 % B. 40 mg/kg per day for 4 days increases response rates in AA. but remains a generally apparent within 2 months of treatment. 47 Hematology MCQ’s FOR MCQ’s FOR MEDICAL MEDICAL PROFESSIONALS PROFESSIONALS BY PROF.365:1647–56 A. Seizures 558 Which of the following is used along with ALG or ATG to Most important side effects of chronic cyclosporine treatment are nephrotoxicity. 50 In aplastic anemia treatment responders. 70 years) and. PRCA is characterized by anemia. Intravenous immunoglobulin therapy C. Ringed sideroblast B. 893 A. B. None of the above 573 B19 parvovirus tropism for human erythroid progenitor cells Idiopathic MDS is a disease of the elderly (mean age at onset . Absent or rare erythroid precursor cells in bone marrow D. and daclizumab (antibody to IL-2 receptor). refractory anemia with excess blasts in transformation (RAEB-t) Harrison’s 18th Ed. Hereditary pure red cell aplasia C. S D. Type 1 (RAEB-1) Harrison’s 18th Ed. Refractory anemia (RA) PRCA may be associated with thymoma. Clonal hematopoietic stem cell disorder Bone marrow in PRCA due to chronic parvovirus infection shows red cell aplasia & giant C. B. which is the cytopathic sign of B19 parvovirus infection. Diamond-Blackfan anemia D. Refractory anemia with excess blasts. RNA processing gene called RPS19. 893 Table 107–4 576 Which of the following is a myelodysplastic syndrome ? A. All of the above C. 894 Harrison’s 18th Ed. Chloramphenicol A. D. five entities of MDS are 571 PRCA may be associated with which of the following ? refractory anemia (RA). Fetal red blood cell aplasia B. Phenytoin Harrison’s 18th Ed. 894 Table 107–5 D. P C. Glucocorticoids B. Kostmann’s Syndrome C. All of the above B19 parvo viral tropism for human erythroid progenitor cells is due to its use of erythrocyte P antigen as a cellular receptor for entry. Ineffective blood cell production C.4 gram/kg daily for 5 days). All of the above A. Reticulocytopenia C. Acquired pure red cell aplasia D. Majority of patients with idiopathic PRCA respond responds to glucocorticoid treatment. L . Giant pronormoblast A. Cyclosporine D. 893 Table 107–4 A.is rare in children. Harrison’s 18th Ed. 893 and chronic myelomonocytic leukemia (CMML). Subcutaneous administration of erythropoietin estimated proportion of patients with MDS ? Harrison’s 18th Ed. Anemia B. Daclizumab Almost all patients with PRCA due to persistent B19 parvovirus infection respond to IV Ig Diamond-Blackfan anemia or congenital PRCA is diagnosed at birth or in early childhood & often therapy (0. Giant megakayroblast Harrison’s 18th Ed. 894 C. 894 Harrison’s 18th Ed. Harrison’s 18th Ed. Fanconi’s anemia A. Idiopathic MDS is a disease of elderly D. Chronic lymphocytic leukemia 577 Which of the following is the most frequent classe in WHO C. Thymoma (2002) classified Myelodysplastic Syndromes/Neoplasms differently. 574 PRCA patients with persistent B19 parvovirus infection 568 Congenital pure red cell aplasia is known as ? respond best to ? Harrison’s 18th Ed. World Health Organization (WHO) classification A. 893 A. Refractory anemia with ringed sideroblasts (RARS) D. procainamide. refractory anemia with ringed sideroblasts (RARS). azathioprine. Apoptosis of marrow cells is increased in MDS. Dysmorphic cellular bone marrow B. Idiopathic Myelodysplasias (MDS) are characterized by cytopenias. Refractory anemia with ring sideroblasts (RARS) C. chloramphenicol. reticulocytopenia & absent or rare erythroid precursor cells in bone marrow. Refractory anemia (RA) C. 894 B.48 MCQ’s FOR MEDICAL PROFESSIONALS BY PROF. M B. It is a clonal hematopoietic stem cell disorder leading to impaired cell proliferation and differentiation. Cytopenias A. According to French-American-British Cooperative Group (1983). Refractory anemia with excess blasts (RAEB) Drug that may cause PRCA include phenytoin. D. cyclophosphamide. is due to its use of which erythrocyte antigen ? Therapy-related MDS is not age-related. refractory anemia with excess blasts (RAEB). chronic lymphocytic leukemia. AJAY MATHUR Hematology A. cyclosporine. dysmorphic cellular bone marrow and 570 Drug that may cause PRCA is ? by ineffective blood cell production. hypogammaglobulinemic and subcutaneous administration of erythropoietin. All of the above isoniazid and erythropoietin. 893 578 Which of the following is false about myelodysplastic A. Uninuclear megakaryocyte syndrome ? B. large granular lymphocytosis. Isoniazid B. A minority of patients have etiologic mutations in a ribosomal favorably to immunosuppression with glucocorticoids. azathioprine. Apoptosis of marrow cells increased pronormoblasts. Shwachman-Diamond syndrome B. ATG. Refractory cytopenias with multilineage dysplasia (RCMD) 572 Pathognomonic cell in bone marrow of PRCA patients with B19 parvovirus infection is ? D. 569 Congenital pure red cell aplasia (Diamond-Blackfan syndrome) 575 Which of the following is a feature of myelodysplasias (MDS) ? is which variety of PRCA ? Harrison’s 18th Ed. All of the above D. Nitrosourea Harrison’s 18th Ed. solid tumors. Ringed sideroblasts Harrison’s 17th Ed. forming a ring. Circulating myeloblasts C. 5% B. congenital osteopetrosis. Iron-encrusted mitochondria surround the nucleus of erythroid cell. occur with Harrison’s 18th Ed. small ferritin granules are normally seen under oil immersion in 20 . increase in myeloblasts and reduced number of megakaryocytes C. A. Normal or hypercellularity Secondary MDS occurs as a late toxicity of cancer treatment. 896 MDS. Pancytopenia is seen despite very large numbers of circulating hematopoietic progenitor cells. D. ATG and anti- CD52 monoclonal antibody Campath is effective in younger MDS patients who bear the Harrison’s 17th Ed. DNA topoisomerase inhibitors myelodysplasia. Ineffective erythropoiesis cells are called sideroblasts. Cowden disease with disorganized nuclei. liver. Pernicious anemia erythropoiesis. 361 D. latent period is least following which of Acquired defects in heme synthesis is seen in myelodysplasia. A.40% of developing erythroblasts. Myeloid metaplasia D. or inflammatory bowel disease. Busulfan 586 Bone marrow in myelodysplasia may show all except ? C. Lenalidomide C. Nucleus 590 Which of the following is false about myelophthisis ? Harrison’s 18th Ed. 897 C. Azacytidine A. Hypogranulation in granulocytic precursors 581 Sideroblasts have granules consisting of ? B. Leukoerythroblastic blood smear Storage iron is in the form of ferritin or hemosiderin. 896 D.e. Sweet’s syndrome or hypercellular with ringed sideroblasts in erythroid lineage. Such C. Mitochondria A. In bone marrow smears. Defective erythroblasts D. 897 582 Sideroblasts are ? Harrison’s 17th Ed. With busulfan. 10 % 589 Which of the following is false about myelophthisis ? Harrison’s 18th Ed. 360 histocompatability antigen HLA-DR15. Aplastic anemia Pathophysiology of myelophthisis has three distinct features: proliferation of fibroblasts in B. Cell membrane B. 360 C. and lymph nodes) and ineffective C. hematologic malignancies. Decrease in myeloblasts A. extension of hematopoiesis into long bones and into extramedullary sites (spleen. the accumulation of iron is around ? Myelophthisis or secondary myelofibrosis is a reactive phenomenon. Fibrosis of BM occurs as a response to invading tumor cells. These ringed sideroblasts on marrow iron stain. Proportion of marrow blasts A. D. radiation therapy or treatment with radiomimetic drugs. 361 disease. Pancytopenia 585 Sideroblastic anemia usually points to the diagnosis of ? C. 897 C. Amifostine B. D. All of the above . Decitabine (15 mg/m 2 IV infusion. hypogranulation & hyposegmentation in granulocytic precursors. Increased circulating hematopoietic progenitor cells Harrison’s 17th Ed. prognosis strongly correlates with ? Unusual skin lesions. including Sweet’s syndrome (febrile neutrophilic dermatosis). Gaucher Harrison’s 17th Ed. myeloid metaplasia i. B. Ferritin D. Endoplasmic reticulum B. sarcoidosis. 49 Hematology MCQ’s FOR MCQ’s FOR MEDICAL MEDICAL PROFESSIONALS PROFESSIONALS BY PROF.7 years and with DNA topoisomerase inhibitors it is 2 years. A. Gardner syndrome No single characteristic feature of marrow morphology distinguishes MDS. TDS for three days). fungi. G-CSF + Erythropoietin have a role in MDS. BM is usually normal B. Secondary myelofibrosis D. None of the above A. Amifostine (blocks apoptosis). Procarbazine A. D. Glycogen usually correlate with marrow blast numbers. Mitochondrial iron loading the following cancer treatments ? occurs as iron taken up by mitochondria of developing erythroid cell is not incorporated into Harrison’s 18th Ed. Developing myeloblasts C. All of the above 588 Which of the following drugs has a role in treatment of MDS ? Harrison’s 18th Ed. Circulating myeloblasts B. 895 heme. C. None of the above 584 In ‘ringed sideroblasts’. 896 D. the latent period is 5 . Ringed sideroblasts procarbazine. or B. Hypogranulation & hyposegmentation in granulocytic 580 Which of the following skin conditions is related to MDS ? precursors Harrison’s 18th Ed. Lenalidomide (10 mg orally daily for 3 583 Normal percentage of sideroblasts in bone marrow is ? months). Myelodysplasia marrow space (myelofibrosis). Torre syndrome 587 In MDS. 360 A. AJAY MATHUR Cardiology 49 579 In secondary MDS. Defective myeloblasts Only stem cell transplantation offers cure in MDS. infections (Mycobacterium. in sideroblastic anemia almost always reflecting A. 40 % A. All of the above D. Azacitidine (75 mg/m 2 daily s/c for 7 days). nitrosourea. HIV). 75 % B. Developing erythroblasts B. Transferrin Prognosis strongly correlates with the proportion of marrow blasts. Tyrosine kinase 2 (TYK2) C. All of the above B. Chronic eosinophilic leukemia (CEL) 600 JAK3 associates with which of the following cytokine D. growth hormone. propensity to thrombosis & hemorrhage.368:161-70 B. or  c. F716V C. interleukin-6. interleukins 1 and 8. STAT5A. and tyrosine kinase 2 (TYK2). essential thrombocytosis (ET). None of the above 594 Which of the following conditions can transform into each other ? Cytokine (interferon. Transformation to acute leukemia is uncommon. JAK1 D. Clonal disorder D. 592 Which of the following is a Philadelphia (Ph) negative Tumor necrosis factor. Interleukin-2R chain Harrison’s 18th Ed. All of the above There are four JAKs: JAK1. Idiopathic myelofibrosis B. JAK3. erythropoietin. Thrombopoietin Myeloproliferative disorders share many characteristics like marrow hypercellularity. Marrow hypercellularity as their essential mode of signaling ? N Engl J Med 2013. transforming growth factor  and macrophage colony- myeloproliferative disorder ? stimulating factor do not use the JAK-STAT pathway as their essential mode of signaling. None of the above JAK3 result in severe combined immunodeficiency (SCID). growth hormone.50 MCQ’s FOR MEDICAL PROFESSIONALS BY PROF. JAK1 & JAK2 have roles that range from host defense and hematopoiesis to growth and neural development. CNL and CEL cannot transform into each other. But all three have a high rate of transformation into acute leukemia. Gain-of- function JAK2 mutations underlie polycythemia vera (PV).355:2452-66 A. STAT4. This process allows the selective binding of members of the STAT family: STAT1.368:161-70 Harrison’s 18th Ed. WBC & platelets Activated JAKs phosphorylate (P) and activate STATs and other pathways. V716F Other Myeloproliferative Diseases Mutation in tyrosine kinase JAK2 .368:161-70 C. C..replaces valine with phenylalanine (V617F) causes its constitutive activation (erythropoietin-independent) is central in the pathogenesis of PV. 898. Thrombosis & hemorrhage C. Polycythemia vera 598 Which of the following is a member of Janus kinases ( JAKs) ? N Engl J Med 2013.355:2452-66 597 Which out of the following do not use the JAK-STAT pathway A. Chronic neutrophilic leukemia (CNL) gene expression. the C. Interleukin-3R chain A. Essential thrombocytosis (ET) A. Interleukin-1R chain 595 Which of the following about polycythemia vera is false ? B. STAT3. Polycythemia vera A. N Engl J Med 2006. C. JAK2 The three main Ph-negative myeloproliferative disorders are polycythemia vera. Primary myelofibrosis (PMF) of transcription (STAT) is false ? N Engl J Med 2013. Absence of recognizable physiologic stimulus common interleukin-2R  chain. D. JAK2. Growth hormone Erythropoietin. Loss-of-function mutations of the genes encoding c and D. and a risk of leukemic transformation in the long term. DNA-binding proteins D. All of the above C. STAT5B. Some JAKs are associated with many different cytokine receptors. These DNA-binding proteins become tyrosine-phosphorylated. myeloproliferative disorders (MPDs) ? N Engl J Med 2006. A. Essential thrombocythemia A. and STAT6. V617F Chapter 108. thrombopoietin. All of the above B. Essential thrombocythemia B. 898 C. Chronic myelogenous leukemia (CML) STAT2. A. translocate to the nucleus. and regulate B. Erythropoietin D. 898 A. Interleukins 1 C. Increased phenotypically normal RBC. Interleukin-4R chain B. None of the above receptors ? N Engl J Med 2013. All JAKs selectively 593 Which of the following conditions can transform into each associate with cytoplasmic domains of various cytokine receptors. Regulate gene expression D. 898 receptors. interferon-. which allows them to dimerize. AJAY MATHUR Hematology C. prolactin. and leptin use JAK-STAT pathway. D. Harrison’s 18th Ed.368:161-70 CML. Mutation of TYK2 also results in immunodeficiency. Primary myelofibrosis . but JAK3 associates with only one subunit. Translocate to the nucleus PV. interleukin-7) binding activates JAKs. Risk of leukemic transformation in long term A. and interferon- . PMF and ET are capable of transforming into each other. which in turn phosphorylate cytokine Harrison’s 18th Ed. interleukin- 2. 596 Which of the following illustrates the mutation in JAK2 in 601 JAK2 mutations are associated with which of the following ? the pathogenesis of PV ? N Engl J Med 2013. N Engl J Med 2013.368:161-70 A. JAK3 & TYK2 are primarily other ? important for immune responses. Polycythemia vera (PV) 599 Which of the following about signal transducer and activator B. Polycythemia Vera and D. F617V B. essential thrombocythemia and idiopathic myelofibrosis.368:161-70 B. and 591 Which of the following is a characteristic feature of myelofibrosis (MF). baricitinib. All of the above D. Mutations in STAT3 also cause large granular lymphocytic (LGL) leukemia. Erythroid progenitor cells are resistant to apoptosis A. and their increase in Bcl-X L expression. granulocyte colony. hyperviscosity leading to venous or arterial thrombosis. Intracellular signaling by receptors for erythropoietin C. 899 susceptibility to mycobacteria and viruses. Chronic mucocutaneous candidiasis formation. hypoxic JAK3 Inhibitors include Tofacitinib. 899 D. Bartter’s syndrome are associated with rheumatoid arthritis. high Hb. Aquagenic pruritus following ? Splenomegaly. Intracellular signaling by receptors for thrombopoietin D. and hypersensitivity of PV erythroid progenitor cells to erythropoietin and other hematopoietic growth factors. AZD1480. essential thrombocythemia. 612 Which of the following is false about polycythemia vera ? 606 Which of the following is false about Janus kinase 2 (JAK2) ? Harrison’s 18th Ed. Only three situations cause microcytic erythrocytosis:  -thalassemia trait. Tofacitinib C. AZD1480. Susceptibility to mycobacteria Constitutive activation of JAK2 can explain the erythropoietin-independent erythroid colony B. An elevated stimulating factor (G-CSF) and granulocyte–macrophage colony-stimulating factor (GMCSF). JAK2 Inhibitors include Ruxolitinib. 898 JAK2 mutations are associated with myeloproliferative neoplasms. 899 A. Abundant bone marrow iron JAK2 is a nonreceptor “cytoplasmic” tyrosine kinase and is critical for instigating intracellular signaling by receptors for erythropoietin. High hematocrit 603 Polymorphisms of STAT3 are associated with which of the D. Intracellular signaling by receptors for GM-CSF polycythemia vera as the cause for erythrocytosis D. all of which are characteristic in PV. Short arm of chromosome 10 C. 1 to 11 mU / mL B. Geisbock’s syndrome 604 Which of the following is a JAK inhibitor ? N Engl J Med 2013. essential thrombocytosis 608 Which of the following typically distinguishes polycythemia (ET). systemic lupus erythematosus. uric acid stones are features of PV. All of the above 607 Which of the following is false in the pathogenesis of PV ? Harrison’s 18th Ed. and other autoimmune diseases and polymorphisms of STAT6 are associated with allergic disease. With the A. .368:161-70 systolic hypertension. 9p. Ankylosing spondylitis exception of aquagenic pruritus. tofacitinib. GLPG0634. pacritinib. Loss-of-function mutations A. ASP015K. C. Nuclear tyrosine kinase B. Short arm of chromosome 9 A. tofacitinib.368:161-70 610 Which of the following causes microcytic erythrocytosis ? Harrison’s 18th Ed. erythromelalgia. N Engl J Med 2013. AJAY MATHUR Cardiology 51 D. and PV. Rheumatoid arthritis 609 Spurious erythrocytosis is also termed as ? C. D. 21 to 54 mU / mL D. high PCV. Allergic disease A. Budd-Chiari syndrome Polymorphisms of STAT3 are associated with ankylosing spondylitis. Baricitinib A. 605 JAK2 gene is located on the ? 611 Normal range for plasma erythropoietin is ? Harrison’s 18th Ed. 898 Harrison’s 16th Ed. Hypersensitivity of PV erythroid progenitor cells to EPO 602 Loss-of-function mutations of STAT3 result in ? C. clonal cancers arising from hematopoietic progenitor cells. no symptoms distinguish PV from other causes of erythrocytosis. B. 4 to 26 mU / mL C. Elevated plasma erythropoietin level excludes C. Autosomal dominant loss-of-function mutations of STAT1 result vera from other causes of erythrocytosis ? in susceptibility to mycobacteria only. Massive splenomegaly of STAT3 result in the hyper-IgE syndrome. A.368:161-70 D. Erythropoietin-independent erythroid colony formation and primary myelofibrosis. -thalassemia trait B. due to mitotic recombination is the most common cytogenetic abnormality in PV. ASP015K. C. and myelofibrosis (MF). 34 to 90 mU / mL JAK2 gene is located on the short arm of chromosome 9. Systemic lupus erythematosus Harrison’s 18th Ed. None of the above A. Autonomous clonal form of erythrocytosis B. Ruxolitinib B. All of the above Gain-of-function JAK2 mutations underlie polycythemia vera (PV). B. C. lestaurtinib. baricitinib. Gray-platelet syndrome D. Autosomal recessive mutations of STAT1 cause Harrison’s 18th Ed. 903 N Engl J Med 2006. All of the above JAK1 Inhibitors include Ruxolitinib.355:2452-66 A. Autosomal dominant gain-of function mutations of STAT1 cause chronic mucocutaneous candidiasis. VX-509. Polycythemia vera (PV) D. Loss of heterozygosity on chromosome Normal range for plasma erythropoietin is 4 to 26 mU/mL. thrombopoietin. Hyper-IgE syndrome or Job’s syndrome their rapid terminal differentiation. which include polycythemia vera. erythropoietin level excludes PV as the cause of erythrocytosis. erythrocytosis. Hypoxic erythrocytosis C. polymorphisms of STAT4 B. Constitutive STAT3 and STAT5 activation is associated with many B. their resistance to apoptosis in vitro in the absence of erythropoietin. and aneurysms. Resistance of PV erythroid progenitor cells to apoptosis N Engl J Med 2013. Mutations of STAT5B result in a syndrome characterized by dwarfism and autoimmunity. interleukin-3. Absent marrow iron in the presence of marrow hypercellularity is a feature of PV. 627 A. gout. Long arm of chromosome 10 D. Long arm of chromosome 9 B. 51 Hematology MCQ’s FOR MCQ’s FOR MEDICAL MEDICAL PROFESSIONALS PROFESSIONALS BY PROF. High hemoglobin cancers. Fibroblasts in PMF are polyclonal and not part of the neoplastic clone. All of the above 615 Drug ‘Anagrelide’ is a ? The presence of teardrop-shaped red cells. generalized pruritus not responding to antihistamines can be treated with hydroxyurea. Salicylate isomer Harrison’s 18th Ed. intracranial hypertension. & psoralens with ultraviolet light in the A range (PUVA). 902 C. PUVA therapy A. Marrow fibrosis Harrison’s 18th Ed. 901 B. Mpl in addition to interleukin 3 (IL-3) and stem cell factor. Granulocyte-macrophage colony-stimulating factor D. to ? Harrison’s 18th Ed. while osteosclerosis is due to overproduction of osteoprotegerin. Myelodysplasia receptor Mpl occur in about 5%. 902 C. Cytotoxic agent B. 901 C. D. antinuclear antibodies. Extramedullary hematopoiesis A. Thrombocytosis B. Positive Coombs’ test hydroxyurea because it lacks marrow toxicity. Hydroxyurea occurrence of extramedullary hematopoiesis ? B. All of the above C. 50 % A. C. Female predominance Chronic PMF is a clonal disorder of a multipotent hematopoietic progenitor cell of unknown C. CML D. an D. 900 of the following ? A. extramedullary hematopoiesis. Overproduction of tissue inhibitors of metalloproteinases A. 900 620 Presence of which of the following establishes the A. or myelodysplasia can masquerade as essential thrombocytosis. Splenomegaly B. Harrison’s 18th Ed. Interferon (IFN)- Harrison’s 18th Ed. 902. Polycythemia vera JAK2 V617F is present in approximately 50% of PMF patients and mutations in the thrombopoietin C. Pulmonary hypertension C. Rheumatoid factor Anagrelide. 903 B. 903 C. 75 % B. All of the above osteoclast inhibitor. Leucocytosis A. Harrison’s 18th Ed. None of the above 617 JAK2 V617F is present in what percentage of PMF patients ? Very high platelet counts are associated primarily with hemorrhage due to acquired von Willebrand disease. Erythrocytosis Harrison’s 18th Ed. 901 A. polycythemia vera. and promyelocytes establishes the presence of extramedullary hematopoiesis. can reduce the platelet count and is preferable to C. Phosphodiesterase inhibitor 621 In PMF. 901 623 Disorders that can masquerade as essential thrombocytosis A. Anagrelide is protective against venous thrombosis. rheumatoid factor. or a positive Coombs’ test. rapid splenic enlargement & splenic infarctions. myelocytes. nucleated red cells. Anticoagulant A. Clonal disorder of unknown etiology D. All of the above C. 10 % include ? B. 900 A. D. skin nodules. a phosphodiesterase inhibitor. pericardial tamponade. pulmonary hypertension. & hyperuricemia & secondary gout. Erythropoietin C. Harrison’s 18th Ed. Overproduction of transforming growth factor 624 ‘Mpl’ is the receptor for ? Harrison’s 18th Ed. Megakaryocytopoiesis and platelet production depend upon thrombopoietin and its receptor. spinal cord compression. All of the above Thrombosis due to erythrocytosis is the most significant complication of PV. Spinal cord compression D. 25 % Harrison’s 18th Ed. D. which of the following is seen ? B.52 MCQ’s FOR MEDICAL PROFESSIONALS BY PROF. Nucleated red cells In PV. Overproduction of osteoprotegerin B. 900 A. Jaundice 622 Which of the following is false about essential thrombocytosis ? B. Thrombopoietin Fibrosis in PMF is due to overproduction of transforming growth factor and tissue inhibitors of metalloproteinases. and splenomegaly. Antinuclear antibodies D. intestinal 614 Intractable generalized pruritus in polycythemia vera responds or ureteral obstruction. Patients have hemorrhagic & thrombotic tendencies etiology characterized by marrow fibrosis. Exuberant extramedullary hematopoiesis can cause ascites. Teardrop-shaped red cells D. AJAY MATHUR Hematology 613 Thrombosis in polycythemia vera is due to ? 619 Exuberant extramedullary hematopoiesis can cause which Harrison’s 18th Ed. D. Marrow angiogenesis is due to increased production of vascular endothelial growth factor. Harrison’s 18th Ed. Intracranial hypertension D. Myelocytes and promyelocytes interferon-. All of the above 616 Chronic primary myelofibrosis is characterised by all except ? An intriguing feature of PMF is the occurrence of autoimmune abnormalities such as immune complexes. All of the above 618 Fibrosis and osteosclerosis in PMF is due to ? CML. All of the above B. Their subsequent development is also enhanced by the chemokine stromal cell-derived factor 1 (SDF-1). . 10 % B.000 people per year A. 15 % C. D. and an inverse correlation exists between the platelet count and plasma thrombopoietic activity. bone marrow.000 people per year B. Chloroquine Harrison’s 18th Ed. which acute myeloid leukemia is most common ? A. IFN- Anticancer drugs are the leading cause of therapy-associated AML occuring 4-6 years after exposure. 904 A.5 per 100. Pancreas C. and median age at diagnosis is 67 years. 53 Hematology MCQ’s FOR MCQ’s FOR MEDICAL MEDICAL PROFESSIONALS PROFESSIONALS BY PROF.7 years 627 Gene for thrombopoietin is located on which chromosome ? C. 3. b-mpl 631 Disease associated with an increased incidence of AML is ? C. 10 % Chapter 109. A reduction in platelet count increases the level of free TPO & thereby stimulates megakaryocyte & platelet production. 905 cutoff for a diagnosis of AML is ? Harrison’s 18th Ed. All of the above 626 Thrombopoietin is produced in ? Harrison’s 18th Ed. -aminocaproic acid 634 In WHO classification. 905 Genes for thrombopoietin and its receptor Mpl are located on chromosomes 3 and 1 respectively. It is 20% in WHO classification and 30% in FAB. A. Difference between WHO and FAB systems for a diagnosis of AML is the blast cutoff. Liver runt-related transcription factor 1 (RUNX1). 3 D. A. 905 D. and other tissues by neoplastic cells of the hematopoietic system. Bone marrow B. Chloramphenicol B. Ataxia telangiectasia.000 people per year Harrison’s 18th Ed.25 years B. AML is ? Harrison’s 18th Ed.Minimally differentiated leukemia C. Ataxia telangiectasia D. 905 A. 1. higher in men A. 1 D.15 years Harrison’s 18th Ed. TPO (c-mpl ligand) is secreted B. 4 633 Use of which of the following drugs may evolve into AML ? Harrison’s 18th Ed. All of the above B. the blast Harrison’s 18th Ed. 15 % Myeloid Leukemia C. All of the above D. chloroquine & methoxypsoralen can result in C. 903 Incidence of acute myeloid leukemia (AML) is 3. germ-line mutations of CCAAT/enhancer-binding protein (CEBPA).5 per 100.000 people per year Harrison’s 17th Ed. d-mpl A. Down syndrome Megakaryocyte receptor of TPO is a protooncogene c-mpl.7 years after exposure. 2. Salicylates D. Therapeutic radiation alone adds little risk of AML.Myeloblastic leukemia without maturation . c-mpl Harrison’s 18th Ed. Phenylbutazone 628 Bleeding associated with thrombocytosis responds to ? C. 30 % Myeloid leukemias are characterized by infiltration of the blood. Other tissues C. Anagrelide bone marrow failure that may evolve into AML. myeloproliferative syndromes. Acute and Chronic B. AML incidence increases with age. a-mpl than in women. Chloramphenicol. neoplastic cells of hematopoietic system infiltrate which of the following ? 635 In French-American-British (FAB) classification. M1. 2 High-dose radiation increase the risk of myeloid leukemias that peak 5 . C.5 per 100.5 per 100. 20 % D.3 years B. 630 Incidence of acute myeloid leukemia (AML) is ? Harrison’s 18th Ed. thrombopoietin is produced in both the liver and the kidneys. Thymus years after radiation exposure ? Harrison’s 18th Ed. B. Fanconi anemia continuously at a low level and binds tightly to circulating platelets. and tumor protein p53 (TP53) are associated with an increased incidence of AML. Lungs 632 Increase in the risk of myeloid leukemias peak how many D. AJAY MATHUR Cardiology 53 625 Megakaryocyte receptor of thrombopoietin (TPO) is ? D. M0 . 905 Like erythropoietin. A. 678 Table 104-1 B.000 people per year. C. Fanconi anemia. 4. 5 . 905 A. 20 % D. 2 . Blood A. the blast cutoff for a diagnosis of Bleeding associated with thrombocytosis usually responds to  -aminocaproic acid. B. Bloom syndrome.5 per 100. 20 . 903 Down syndrome. phenylbutazone. 10 . Kostmann syndrome. 30 % 629 In myeloid leukemias. 903 A. 905 636 Out of the following. 4%). They C. Juxtaposes PML with RAR 644 In determining complete remission (CR) in AML. 910 B. Absent Auer rods AML associated with younger age are t(8. t(15.17).9). Chronic myelomonocytic leukemia 637 Out of the following. and Auer rods should be absent.21) & t(l5. 910 D.17) Platelet counts <100. Bone marrow cellularity >20% with trilineage maturation C. All of the above common ? Auer rods are a hallmark of acute myeloid leukemia but occasionally are seen in myelodysplastic Harrison’s 17th Ed.17) Rasburicase.000/µL. Patients with complex karyotype.20%). Megakaryoblastic leukemia (M7 . 50 % 647 Which of the following chromosome findings in AML have a C. Myeloblastic leukemia without maturation (M1 .20%). Encodes a chimeric protein the following is not included ? Harrison’s 18th Ed. Contrast nephropathy D. M3 . which of A. Associated with disseminated intravascular coagulation A. 20 % B. catalyzes enzymatic oxidation of uric acid cytogenetic rearrangement ? into a soluble metabolite. B. M6 . Patients who achieve CR after one induction cycle have longer CR durations than those requiring multiple cycles.17). All of the above In determining CR. Erythroleukemia C. 905 A. Hemoglobin concentration have a very good prognosis.Hypergranular promyelocytic leukemia A. & 20% have counts >100.000/µL. Uric acid nephropathy (DiGuglielmo’s disease) (M6 . Neutrophil count D.17)(q22. A. Circulating blasts In determining CR. Older age the following is included ? Harrison’s 18th Ed. lower extremity weakness common C. blood neutrophil count must be >=1000/µL & platelet count >=100. 10 % A. AJAY MATHUR Hematology C. Hypergranular promyelocytic leukemia (M3 . 25 % B. Secondary AML after cytotoxic agents D. &/or thrombocytopenia for >3 months) preceding diagnosis of 641 Whar proportion of AML patients will have platelet count of AML is associated with lower CR rate & shorter survival time.Megakaryoblastic leukemia B. bone marrow cellularity should be >20% with trilineage maturation with <5% blasts. Rasburicase is contraindicated in G6PD deficiency.5%). Acute myeloid leukemia Minimally differentiated leukemia (M0 . 30 % C. 908 & pulmonary leukostasis contribute to poor outcome with initial therapy. which of B. D.000/µL) at presentation C.1%). Hyperleukocytosis (>100. It is used in uric acid nephropathy. Myelodysplastic syndromes Myeloblastic leukemia with maturation (M2 . 25 & 40% of patients have counts <5000/µL. M2 . 75 % very good prognosis ? Harrison’s 18th Ed.10%). Monocytic leukemia (M5 . Advancing age B. Associated with myeloid sarcomas A.21) circulating blasts should be absent. Back pain. D.000 per µL ? 646 In AML.000/µL & 639 Which of the following is false about AML with t(8.54 MCQ’s FOR MEDICAL PROFESSIONALS BY PROF. Diabetic nephropathy 638 Which of the following is false about AML with t(15. <5% blasts in bone marrow D. M4 .21) . which of the following predict poor outcome with Harrison’s 18th Ed. cytogenetic rearrangement ? Harrison’s 17th Ed. early CNS bleeding Harrison’s 18th Ed.Myelomonocytic leukemia 643 Rasburicase is used in ? B. & tumor lysis syndrome. Hyperleukocytosis (>100. Platelet count AML FAB M3 is called acute promyelocytic leukemia (APL) based on presence of either t(15. Harrison’s 18th Ed. Secondary AML due to cytotoxic less than 100.000/µL).000/µL are found at diagnosis in 75% of patients. 910 A.000 per µL ? agents is difficult to treat successfully. M5 . 678 Table 104-1 syndromes (MDSs) or chronic myelomonocytic leukemia. All of the above Median presenting leukocyte count is about 15. a recombinant uric-oxidase enzyme. Prolonged symptomatic interval with cytopenias (anemia. B. allantoin. 678 645 In determining complete remission (CR) in AML. A. which acute myeloid leukemia is most D. 910 C. <5% have no detectable leukemic cells in the blood. Granulocytic sarcoma or chloroma common B. leukopenia.10%). Salt-wasting nephropathy Myelomonocytic leukemia (M4 . Advancing age is associated with a poorer prognosis because AML in older patients differs biologically (multidrug resistance 1 (MDR1) efflux pump). 640 Whar proportion of AML patients will have leukocyte count of more than 100. Have a very poor prognosis B.30%). t(8. Extramedullary leukemia should not be present.q12) cytogenetic rearrangement or PML/RAR  product of translocation. DIC is associated with t(15.Monocytic leukemia Harrison’s 18th Ed. 40 % D. with older age are del(5q) and del(7q). 100 % A. D. 909 C. C.Myeloblastic leukemia with maturation 642 Auer rods are seen in ? D.000/µL.Erythroleukemia (DiGuglielmo’s disease) A. t(6. and about 25% have counts <25. M7 . inv(3) or 7 have a very poor prognosis. Hemoglobin level is not considered in determining CR. 908 initial therapy ? Harrison’s 18th Ed. 651 Toxicity with high-dose cytarabine includes ? Harrison’s 18th Ed. Dexrazoxane ABL is named after abelson murine leukemia virus gene. and significant and occasionally irreversible cerebellar toxicity. Iron C. attain CR should proceed to allogeneic stem cell transplant ? Harrison’s 18th Ed. G2-phase Harrison’s 18th Ed. Addition of etoposide may improve the CR duration.21) and A.000 people per year intracellularly to an active triphosphate form (1. Dexrazoxane is an antidote to doxorubicin-induced extravasation. Myelosuppression A. when it starts to ? B. Chromosomes 9 and 22 D. 911 656 In CML. Big cluster region Toxicity with high-dose cytarabine includes myelosuppression. All of the above B. Tretinoin cell cycle ? Harrison’s 18th Ed. there occurs a reciprocal translocation between ? Harrison’s 18th Ed. Incidence is higher in men than in women interferes with DNA synthesis. protects heart against anthracycline toxicity by preventing Fe-dependent free. Asparginase 648 Antimetabolite Cytarabine interferes with which phase of D. It’s an intracellular Fe chelator. Abelson murine leukemia virus A. Anderson murine leukemia virus B. Chromosomes 7 and 20 C. C. 914 A. 910 A. -D-arabinofuranylcytosine-triphosphate) that B.5 per 100. 3 C. Incidence decreased slightly between 1973 and 1991 649 Which of the following drugs are of use in AML ? D. Chromosomes 5 and 18 B. 914 Harrison’s 18th Ed. 914 A. Pulmonary toxicity B. Thiamine D. Idarubicin 655 Incidence of CML increases slowly with age until mid forties. AML patients who fail to Incidence of CML increases slowly with age until mid forties. Remains the same . Incidence is 1.9). 659 In CML.9) 653 Which of the following drugs is used in the treatment of D. Breakpoint cluster region C. Bridgepoint cluster region B. t(6. Head-to-tail . pulmonary toxicity. 914 Cytarabine is a cell cycle S-phase–specific antimetabolite that becomes phosphorylated A. C. 4 D. 912 AML patients with t(15. Rise rapidly Most commonly used CR induction regimens in AML (other than acute promyelocytic leukemia C. All of the above D. None of the above Harrison’s 18th Ed. 911 657 “BCR” stands for ? Harrison’s 18th Ed. 914 C. then rises rapidly. Irreversible cerebellar toxicity C. Daunorubicin A. those with no cytogenetic abnormality have a moderately favorable outcome (~40% cured).17) have a very good prognosis (~85% cured). Gemcitabine inv(16) a good prognosis (~55% cured). Harrison’s 18th Ed. Atkin murine leukemia virus C. Ascorbic acid B. BCR stands for breakpoint cluster region. 839 A. M-phase Chronic Myeloid Leukemia (CML) B. AJAY MATHUR Cardiology 55 C. Any of the above 650 After how many induction courses. G1-phase 654 Which of the following is false about CML ? D. Bilateral cluster region D. or -7 B. In CML. AML patients with a complex karyotype. inv(3). 910 A. Fall rapidly D. 1 A. Cytarabine Harrison’s 18th Ed. D. Etoposide have a very poor prognosis. those with t(8. S-phase C. 55 Hematology MCQ’s FOR MCQ’s FOR MEDICAL MEDICAL PROFESSIONALS PROFESSIONALS BY PROF. 2 B. Ashley murine leukemia virus D. t(6. inv(3) acute promyelocytic leukemia (APL) ? Harrison’s 18th Ed. 914 A. nature of fusion of BCR gene ABL gene is ? radical generation. clonal expansion of hematopoietic stem cell with reciprocal translocation between chromosomes 9 & 22 occurs. Chromosomes 11 and 22 Patients who fail to attain CR after two induction courses should immediately proceed to an allogeneic hematopoietic stem cell transplant (HSCT) if an appropriate donor exists. 652 Which of the following may protect heart against anthracycline 658 “ABL” is named after ? toxicity ? Harrison’s 18th Ed.APL) is a combination chemotherapy with cytarabine and Daunorubicin or Idarubicin. or blood or marrow blasts between 10 and 20%. Hepatomegaly B. disease acceleration is defined by ? in normal tissues. Chromosome 11 D. A. Chromosome 12 D. 915 (Philadelphia) chromosome and ABL-BCR on chromosome 9q+. Antiapoptotic BCR-ABL is exclusively cytoplasmic. D. 914 C. but when Breakpoint cluster region (BCR) gene is located on chromosome 22. Lymphadenopathy C. Reciprocal translocation between chromosomes 9 & 22 660 Untreated CML. or platelet count <100. A. Chromosome 9 B. Chromosome 11 D. D. cytogenetic clonal evolution. > 10 % B.349:1451-64 Harrison’s 18th Ed. Chromosome 12 Minimal to moderate splenomegaly is the most common physical finding Abelson (ABL) proto-oncogene normally located on chromosome 9. Increasing anemia 664 ABL-BCR fusion protein is found in ? B.22). ABL proapoptotic protein is found in both nucleus & cytoplasm. None of the above The classic BCR-ABL gene of CML results from the fusion of parts of two normal genes .000/µL.346:684 A.346:684 A. there is no evidence to suggest a viral etiology. presence of which of the following points towards poor prognosis ? 662 Breakpoint cluster region (BCR) gene is present on ? Harrison’s 18th Ed. Lack of sleep C. Both genes are ubiquitously expressed 670 In CML. or platelet count <100. Chromosome 22 C. Bony tenderness D. 4 years Untreated CML degenerates from a chronic phase to an accelerated phase & on to blast crisis 667 Which of the following is the most common physical finding in a median time of 4 years. 915 661 ABL proto-oncogene is present on ? N Engl J Med 2002.ABL gene on chromosome 9 & BCR gene on chromosome 22. translocation results in head-to-tail fusion of BCR gene on chromosome 22q11 with Harrison’s 18th Ed. Chromosomal instability is a rule A. Blood or marrow blasts between 10 and 20%. 2 years In CML. Chromosome 9q+ A. Platelet counts are always elevated at diagnosis B. Cytoplasm D. Splenomegaly A. Side-to-side Cigarette smoking accelerated the progression of CML to blast crisis. trisomy 8. 915 A. indicate a poor prognosis. Cell membrane Disease acceleration is defined by the development of increasing degrees of anemia unaccounted D. Any of the above 666 Which of the following about CML is false ? In CML. Hepatomegaly B. Nucleus marrow basophils >=20 %. Serum vitamin B12 & B12–binding proteins are raised D. 3 years malignant clone resulting in the acquisition of an additional t(9. Bony tenderness D. 914 ABL gene located on chromosome 9q34.000/µL B. Cigarette smoking C. 668 In CML.56 MCQ’s FOR MEDICAL PROFESSIONALS BY PROF. > 15 % C. blood or marrow basophils >=20%. Tail-to-head D. present.(TP53 loss) is a fundamental feature of CML. 671 In CML. Chromosome 22 C. 663 ABL-BCR fusion gene is present on ? 669 Which of the following is false about CML ? N Engl J Med 2003.349:1451-64 C. in CML ? Harrison’s 18th Ed. Harrison’s 18th Ed. Splenomegaly A. Alcohol ingestion B. or 17p. >5% A. blast crisis is defined as blood or marrow blasts more than ? 665 Progression of CML to blast crisis is accelerated by ? Harrison’s 18th Ed. There is direct evidence of a viral etiology B. Nuclear membrane for by bleeding or therapy. 914 A. Chromosome 22 B. Chromosome 12 Lymphadenopathy & myeloid sarcomas are unusual except late in the course of CML. goes into blast crisis in a median time of ? B. 1 year D. blood or A. Chromosomal instability of the C. Exercise . Lymphadenopathy C. Two novel fusion genes are found in CML . Bcr/Abl fusion protein is p210BCR/ABL Harrison’s 18th Ed. Chromosome 9 B. Cytogenetic clonal evolution N Engl J Med 2003. 915 Harrison’s 18th Ed.BCR-ABL on derivative 22q. Chromosome 11 C. All of the above C. AJAY MATHUR Hematology B. 915 N Engl J Med 2002. Leukocyte alkaline phosphatase is low C. hemoglobin & platelet counts and disappearance of splenomegaly.346:683 678 Sokal index staging system in CML includes all except ? A. A. complete hematologic remission means ? D. complete cytogenetic remission means no bone marrow metaphases with t(9.22) B. WBC <10. t(12. T cells D.346:684 D. Harrison’s 18th Ed. 80 tyrosine kinases (TK) In CML tt. 36 . Lengthened chromosome 9 (9q+) 680 CML is characterized by ? N Engl J Med 2002. C. Cytogenetic clonal evolution C. > 20 % B. Disappearance of splenomegaly A.000/µL. 915 B.22) 676 The human genome contains about ? N Engl J Med 2005. All of the above 682 In CML treatment. C. B. platelet count. Marrow hypercellularity in approximately ? C. as well as in B and T cells in some patients. 57 Hematology MCQ’s FOR MCQ’s FOR MEDICAL MEDICAL PROFESSIONALS PROFESSIONALS BY PROF. 75 percent CML is due to mutation in a pluripotent stem cell and is characterized by progressive B. Platelet count N Engl J Med 2002. C. normal blood morphology. No bone marrow metaphases with t(9.22)(q34.17) A. Percentage of circulating blasts 672 Cytogenetic hallmark of CML is ? Sokal index (based on chemotherapy-treated patients) includes percentage of circulating blasts. Hydroxyurea B. 85 percent granulocytosis. 915 D. Splenomegaly N Engl J Med 2002. Hemoglobin 684 Function of protein kinase enzymes is to transfer ? D.q11. t(11. with blood or marrow blasts 20%.22) B. 1 .22) 679 Hasford staging system in CML does not include which of the following ? C. Percentage of circulating blasts C. Normal blood morphology. 70 tyrosine kinases (TK) D. IFN- A. Phosphate & adenosine from ATP to specific amino acids .346:683 C.35% bone marrow metaphases with t(9. but is absent in all other cells. Progressive granulocytosis 674 In classic CML patients. 100 tyrosine kinases (TK) Harrison’s 18th Ed.346:683 Philadelphia chromosome refers to a shortened chromosome 22 (22q-). C. Age 673 Philadelphia chromosome can be addressed as ? C. hemoglobin & platelet counts 675 Ph chromosome is present in ? N Engl J Med 2002. Sex neutrophils may appear (Pelger-Huet anomaly). spleen size. 916 Table 109–5 Diagnostic hallmark is Ph chromosome.85% bone marrow metaphases with t(9. marrow hypercellularity and splenomegaly. age & percentage of eosinophils & basophils as prognostic indicators. All dividing cells of hematopoietic lineage D. All of the above C. Age Blast crisis is defined as acute leukemia.14) Harrison’s 18th Ed. which is present in all dividing cells of hematopoietic lineage. Hyposegmented C. 915 spleen size. 915 B.353:172-87 C. Philadelphia chromosome is present B. complete cytogenetic remission means ? Harrison’s 18th Ed. 90 tyrosine kinases (TK) 683 Which of the following drugs is effective in treatment of CML ? D. AJAY MATHUR Cardiology 57 D. Shortened chromosome 9 (9q-) circulating blasts. Spleen size D. t(9. age and cytogenetic clonal evolution as the most important prognostic indicators. platelet count. Imatinib mesylate Harrison’s 18th Ed. Phosphate from ATP to specific amino acids Harrison’s 18th Ed. 100 percent Harrison’s 18th Ed.22). WBC <10. complete hematologic remission. All of the above A. Shortened chromosome 22 (22q-) Hasford system (based on interferon (IFN)   – treated patients) for CML includes % of B. Spleen size Harrison’s 18th Ed. A.000 / µL B. t(10. D. A.. < 10% bone marrow metaphases with t(9. A. Cytogenetic clonal evolution Cytogenetic hallmark of CML is t(9. 95 percent 681 In CML treatment.2). 915 D. 916 Table 109–5 Ph chromosome is present in ~95 % of patients with classic CML. All of the above B.22) A.22) B. 916 677 Sokal index staging system in CML includes all except ? A. Lengthened chromosome 22 (22q+) D. Percentage of circulating blasts A. Adenosine from ATP to specific amino acids A. B cells In CML treatment. 916 N Engl J Med 2002. and c-kit (receptor for stem-cell factor). 688 Imatinib mesylate is a specific inhibitor of which of the following TKs ? 695 Which of the following is the mechanism of resistance to N Engl J Med 2002.346:686 A. the protein serine-threonine kinases and the is false ? protein tyrosine kinases. A+B Treatment of CML with Imatinib mesylate is currently recommended for life.346:686 B. Mast-cell tumors A. 916 B. Imatinib C. BCR-ABL C. 455 B. 916 A. 3 years N Engl J Med 2002.346:686 imatinib ? A. nilotinib. All of the above C. A. 918 A. Harrison’s 18th Ed. Gastrointestinal stromal tumors Harrison’s 18th Ed. Chronic myelomonocytic leukemia (CMML) cytogenetic remission ? D. Gene amplification C.346:683 C. 5 years A. Hydroxyurea B. 916 amino acids on substrate proteins. Tyrosine kinase receptors like epidermal growth factor receptor. < 600 mg / day D. All of the above Harrison’s 16th Ed.threonine kinases D. mutations at the kinase site. All of the above <300 mg/day of Imatinib mesylate is ineffective & may lead to development of resistance. Imatinib is administered orally 686 Which of the following are protein kinases ? N Engl J Med 2002. Chronic-phase CML C. Most common hematologic side effect is B. All of the above Imatinib inhibits PDGF receptor. all ABL tyrosine kinases. Protein tyrosine kinases C. None of the above 692 Treatment of CML with Imatinib mesylate is currently recommended for ? Protein kinases are enzymes that transfer phosphate from adenosine triphosphate to specific Harrison’s 18th Ed. Harrison’s 16th Ed. Nilotinib D. D. < 300 mg / day A. Life long B. Mutations at the kinase site D. All of the above Harrison’s 18th Ed. Blast-crisis phase CML Cytogenetic remissions with hydroxyurea are uncommon. Epidermal growth factor receptor D. CML resistant to E255K/V & Y253F/H while dasatinib is also resistant to X299L & F317L. Protein serine . D. Epidermal growth factor receptor FLT1 C.58 MCQ’s FOR MEDICAL PROFESSIONALS BY PROF. 916 B. All of the above 694 What dose of Imatinib mesylate is ineffective and may lead to development of resistance ? 687 The tyrosine kinase inhibited by imatinib is ? Harrison’s 18th Ed. Imatinib mesylate 691 Imatinib mesylate is most effective in ? Harrison’s 16th Ed. 455 696 CML with the T315I mutation is resistant to ? A. enhanced expression of multidrug exporter proteins and alternative signaling pathways functionally 689 The proto-oncogene c-kit is expressed in ? compensating for the imatinib-sensitive mechanisms. Gastrointestinal stromal tumors (GIST) 697 Therapy with which of the following in CML does not induce C. Epidermal growth factor receptor FLT3 D. All of the above . < 500 mg / day C. Dose is 400 mg per day A. ABL-related gene product (ARG) Harrison’s 18th Ed. PDGF receptor (PDGFR) B. 455 CML with the T315I mutation is resistant to imatinib. < 400 mg / day B. None of the above D. c-kit B. Dasatinib 690 Imatinib has effective activity in cases of ? D. Enhanced expression of multidrug exporter proteins D. Neuroblastoma B. Four mechanisms of resistance to imatinib are gene amplification. All of the above C. FLT1 & FLT3 are unaffected. c-KIT A. 1 year 685 Which of the following are subfamilies of protein kinases ? B. Protein kinase C myelosuppression C. B. ABL. All of the above C. None of the above 693 Which of the following statements about Imatinib mesylate Protein kinases are composed of two subfamilies. AJAY MATHUR Hematology D. Accelerated phase CML D. and dasatinib. Nilotinib is also A. IFN- A. Pel who both published papers in 1887 D. hypercalcemia.60 µm in diameter and display a large rim of cytoplasm and A. Second peak in late life lymph nodes on alcohol ingestion. 1951 and prominent nucleoli giving an “owl’s eyes” appearance. nephrotic syndrome.16 µm C. immune hemolytic anemia. B and T cell lymphomas 50% patients will have mediastinal adenopathy at diagnosis. thrombocytopenia.10 µm The concept of staging Hodgkin’s lymphoma was made at the Ann Arbor Conference in 1971. Imatinib mesylate etiology of Hodgkin’s disease B. a large cell with a bilobed nucleus A. C. AJAY MATHUR Cardiology 59 698 Therapy with which of the following in CML does not induce B.K. most patients have nodular sclerosing Hodgkin’s disease. 12 . carcinomas.20 µm 701 Which of the following statements about Hodgkin’s Disease is false ? D. and pain in C. 1832. Mixed-cellularity Hodgkin’s disease or lymphocyte-depletion Hodgkin’s disease are seen more frequently in patients infected by HIV. Typical bimodal age distribution Hodgkin’s disease can present with unusual manifestations that are severe & unexplained B. melanomas.35 years of age itching. “Ghost” appearance in which year ? D. 5 . All of the above in the same journal. A. All of the above A. 59 Hematology MCQ’s FOR MCQ’s FOR MEDICAL MEDICAL PROFESSIONALS PROFESSIONALS BY PROF.363:653-62 C.35 years A. Nodular sclerosing Hodgkin’s disease N Engl J Med 2010. In 1898 and 1902. erythema nodosum. Paraneoplastic cerebellar degeneration 708 Which of the following is false about classic Hodgkin’s disease ? C. In 1944. Jackson and Parker proposed A. 8 . B. with a peak at 10 . None of the above 703 Which of the following about Pel and Ebstein is false ? Classic Hodgkin’s disease has a typical bimodal age distribution.55:162–176 D. Both published papers in in the same journal in 1887 rapid hematologic responses ? about fever in Hodgkin’s disease N Engl J Med 2002. Pel of age and a second peak in late life. D. None of the above A. Pain in lymph nodes on alcohol ingestion J Clin Pathol 2002. ichthyosiform atrophy. supraclavicular area. following ? Malignancies of Lymphoid Cells Harrison’s 18th Ed. “Owl’s eyes” appearance the first comprehensive classification of the tumour. Thomas Sir Thomas Hodgkin presented seven autopsy cases in his famous paper “On some morbid 705 Nucleus in Reed-Sternberg cell is also described as ? appearances of the absorbent glands and spleen” to the Royal Medical and Chirurgical Society Harrison’s 18th Ed. 934 Figure 110-11 of London on January 10 and 24. Long-term dispute persisted between Pel & Ebstein on the A. nontender. 1961 706 What is the diameter of Reed-Sternberg (RS) cells ? C. John In the US. All of the above 702 Unusual manifestations of Hodgkin’s disease include ? RS cells are not exclusive to HL because similar elements can be seen in reactive lesions Harrison’s 18th Ed. Most patients present with palpable lymphadenopathy C. Carl Sternberg and Dorothy Reed independently described the typical “diagnostic” cells.346:687 C. A long-term dispute persisted between Pel and Ebstein on the etiology of the condition. though Pel published first by several months. 934 Reed-Sternberg (RS) measure 20 . Infectious mononucleosis Hodgkin’s disease does not appear to be increasing in frequency.K. Melanomas D. “Moon” appearance N Engl J Med 2010. B. “Spectacle” appearance 700 Ann Arbor staging system for Hodgkin’s Disease was made C. Subdiaphragmatic presentation of Hodgkin’s disease is unusual and more common in older males. Lymphocyte-depleted Hodgkin’s disease B. B. Subdiaphragmatic presentation is unusual 707 Reed-Sternberg (RS) cells are seen in ? J Clin Pathol 2002. 704 Most patients of Hodgkin’s disease have which of the Chapter 110. 934 (infectious mononucleosis).55:162–176 D. 20 . 934 A.60 µm Harrison’s 18th Ed. paraneoplastic cerebellar degeneration. IFN- D. Their names were Wilhelm Ebstein and P. Hydroxyurea Pel-Ebstein fever is named after Wilhelm Ebstein & P. and axilla). and sarcomas. D. Disease not increasing in frequency at least two nuclei with acidophilic or amphophilic nucleoli. 1981 A. Clarke and lymphocyte-depleted Hodgkin’s disease are rare. B. Lymphocyte-predominant C.55:162–176 D. B & T cell lymphomas. B. in neck. Erythema nodosum B. Micheal D. Most patients present with palpable lymphadenopathy (contiguous. Mixed-cellularity Hodgkin’s disease 699 What was the first name of Hodgkin ? B. covering more than 50% of the nuclear area. .363:653-62 A Reed-Sternberg cell is present near the center of the field. 1971 J Clin Pathol 2002. Lymphocyte-predominant Hodgkin’s disease A. None of the above C. One peak at 10 . A. MYC (8q24) A. None of the above Plasma cell disorders are monoclonal neoplasms. Idiotypes 714 Which of the following secondary translocations are D. E) & two light chain isotypes (kappa & lambda).germinal center B cell (mol wt ~ 25. A After Otto Kahler. Kahler’s disease response to the tumor. Their clinical manifestations are due to expansion of neoplastic cells. Waldenström’s macroglobulinemia. to symptomatic myeloma.364:1046-60 A.germinal center B cell the antigen-binding portion of the molecule are called ? D. develop from common progenitors in B lymphocyte lineage. vincristine. A. and vincristine). All of the above D. Kayle’s disease immunoglobulin molecule ? D. CD15 is a valuable marker for RS cells and is multiple myeloma ? detected in ~80% of patients with classical Hodgkin’s disease. Post .000). N A. 936 D.364:1046-60 Antibody molecules are composed of two heavy chains (mol wt ~ 50. etoposide. C. and prednisone). D. M Harrison’s 18th Ed. 1 heavy chain and 2 light chains 713 Multiple myeloma originates from ? D. lymphokines). AJAY MATHUR Hematology 709 Hodgkin’s disease is related to ? N Engl J Med 2011. prednisone. All of the above Secondary translocations involving MYC (8q24). Multiple myeloma There are 5 heavy chain isotypes (M. deletion in p18. Any of the above N Engl J Med 2011. Monoclonal antibody Ki-1 has reactivity restricted to RS cells. finally. Pre . B. Plasma Cell Disorders D. 2 heavy chains and 2 light chains Plasma cell disorders also called monoclonal gammopathies or paraproteinemias or plasma cell dyscrasias or dysproteinemias comprise of Multiple myeloma. Expression of the CD30 molecule by RS cells is seen in 715 Which of the following genetic abnormalities are found in more than 98% of classical Hodgkin’s disease. Hodgkin’s disease is a tumor characterized by rare neoplastic cells of germinal centre B cell origin in a tumor mass with largely polyclonal inflammatory infiltrate. 934 C. Polyclonal inflammatory infiltrate C. Post . D 712 Which of the following is not a plasma cell disorder ? C. 936 vinblastine. All of the above 716 Which of the following about Plasma Cell Disorders is false ? Chemotherapy regimens used in Hodgkin’s disease include ABVD (doxorubicin. Waldenstrom’s macroglobulinemia C. BEACOPP progression and drug resistance. B. Harrison’s 18th Ed.germinal center T cell Harrison’s 18th Ed. D. and BEACOPP (bleomycin. Isotypes that are derived from post-germinal-center B cells. Strohl’s disease 717 Which of the following is not a heavy chain isotype of C. Primary amyloidosis 718 Antibody molecule is composed of ? D. RS cells usually lack CD45 and epithelial membrane antigen (EMA) expression.364:1046-60 Harrison’s 18th Ed. 934 A. or loss of expression or mutation in TP53 are found only in multiple myeloma and play a key role in determining tumor C. MYC dysregulation Hodgkin’s disease ? Harrison’s 18th Ed.germinal center T cell 719 Antigenic determinants formed by the unique structure of C. B. Neoplastic cells of B cell origin B. MYC dysregulation. 936 A. G. B. All of the above B. Allotypes are distinct . Monoclonal neoplasms procarbazine. Manifest clinically by expansion of neoplastic cells Chapter 111. MOPP (mechlorethamine. Loss of expression or mutation in TP53 A. 2 heavy chains and 1 light chain Primary amyloidosis & Heavy chain diseases. doxorubicin. 936 Myeloma arises from an asymptomatic premalignant proliferation of monoclonal plasma cells A. Mutations of RAS or FGFR3 710 Which of the following chemotherapy regimens is used in B. MOPP Mutations of RAS or FGFR3. cyclophosphamide. Light chain diseases Harrison’s 18th Ed. to 711 Multiple myeloma is also known as ? secretion of cell products (immunoglobulin molecules or subunits. Allotypes and. and IRF4 (6p25) are common in multiple myeloma but rare in MGUS. MAFB (20q12). B. Monoclonal antibody Ki-1 reactivity to RS cells D. B. Progenitors in B lymphocyte lineage C. and dacarbazine).60 MCQ’s FOR MEDICAL PROFESSIONALS BY PROF. None of the above common in multiple myeloma but rare in MGUS ? Isotypes are those antigenic determinants that distinguish among the main classes of antibodies of a given species and are the same in all normal individuals of that species. Myeloma evolves mostly from a monoclonal gammopathy of undetermined clinical significance (MGUS) that progresses to smoldering myeloma B. N Engl J Med 2011. MAFB (20q12) B. C. Pre . bleomycin. ABVD D.000) and two light chains A. and to host’s A. 936 A. Migasha disease Harrison’s 18th Ed. procarbazine. IRF4 (6p25) C. 936 C. B. 936 C. sarcoidosis. lymphomas of B or T cell origin. Gamma region A. < 30 mg D. C. Gaucher disease. 10 % A. All of the above 729 In electrophoretic analysis. 20 % B. Idiotypes A. M component may be detected in ? the amino acid sequences of otherwise similar immunoglobulins. Myeloma B. Miscellaneous B. . they are detected by antibodies made in the same species. Monoclonal A. Beta region Harrison’s 18th Ed. usually in the globulin region. 936 A. breast cancer. Disulfide bond A. cirrhosis. These differences are determined Harrison’s 18th Ed. Sarcoidosis D. Plasma cell disorders are formed by the unique structure of the antigen-binding portion of the molecule. Phosphate bond Harrison’s 18th Ed. Delta region B. Chronic myeloid leukemia proteins ? D. Colon cancer D. 5% 727 In electrophoretic analysis. parasitic diseases. 0. leukemia. A. and pyoderma gangrenosum. and colon cancer. 61 Hematology MCQ’s FOR MCQ’s FOR MEDICAL MEDICAL PROFESSIONALS PROFESSIONALS BY PROF. M component may be detected in ? Harrison’s 18th Ed. Myasthenia gravis B. All of the above 721 Light & heavy chains in antibody molecule are linked by ? Harrison’s 18th Ed. myasthenia gravis. 0. 937 Figure 111-1 C. 936 “church spire” sharp peak. This corresponds to ~10 9 cells producing the antibody. M component may be detected in ? A. Alpha region 730 In serum electrophoresis. Sulphar bond B. < 20 mg C. cold agglutinin disease. Rheumatoid arthritis A.3 g / dL 732 Bence Jones protein in urine of myeloma patients represents ? B. Antibody fragment 725 In serum electrophoresis. 936 A. Altered antibody M stands for monoclonal. M components may be detected in chronic lymphocytic <10 mg of free light chains secreted by plasma cells is excreted by kidney per day. A. lichen myxedematosus. All of the above A. Any of the above what concentration to be detectable ? M component may be an intact antibody molecule of any heavy chain subclass. None of the above 724 “M” in M component stands for ? In monoclonal gammopathies. Normal individuals D. 2 % B. 0. 0. by definition. quantity of light chains excreted daily by kidney is ? Harrison’s 18th Ed. Altered antibody immunoelectrophoresis. 936 B.000 and 970. 936 by allelic genes. 936 altered antibody or fragment. 936 Harrison’s 18th Ed. Lichen myxedematosus C. Any of the above C. Gaucher disease The light and heavy chains are linked by disulfide bonds. They usually constitute about 20 per cent of all the plasma proteins. Chronic lymphocytic leukemia 720 Immunoglobulins constitute what percentage of plasma C. Idiotypes are unique to the molecules produced by a given clone of antibody-producing cells. < 10 mg B. chronic myeloid leukemia. Harrison’s 18th Ed. C. 936 728 In electrophoretic analysis. Breast cancer Immunoglobulins (Ig) have molecular weights between 160. Heavy chains Antibody must be present at a concentration of at least 0.000. the predominance of a product of a single cell produces a Harrison’s 18th Ed. Light chains D. Plasma cell disorders D. church spire sharp peak is seen in ? B. immunoglobulins form a broad peak in ? rheumatoid arthritis. and necrobiotic xanthogranuloma.4 g / dL Harrison’s 18th Ed. 723 In serum electrophoresis.6 g / dL B. AJAY MATHUR Cardiology 61 determinants that reflect regular small differences between individuals of the same species in 726 In electrophoretic analysis.5 g / dL A.5 gram/dL to be detectable by C. < 40 mg In addition to the plasma cell disorders. 936 C. the antibody must be present at D. Polyclonal immunoglobulinopathy Immunoglobulins move heterogeneously in electric field & form a broad peak in gamma region. Cirrhosis C. D. Intact antibody molecule of any heavy chain subclass D. or it may be an Harrison’s 18th Ed. Monoclonal gammopathy D. M component is a tumor marker. M component may be detected in ? 722 Normally. C. Monocomponent 731 M component is composed of ? Harrison’s 18th Ed. All of the above D. Spleen A. Harrison’s 18th Ed. Serum alkaline phosphatase is usually normal associated with a poor prognosis.14). and International Staging System stage I. and protein kinase C signaling cascades. destroy bone & suppression of osteoblasts that form new bone. Protein kinase C signaling cascades Lymphocytes and plasma cells are produced mainly in lymphogenous tissues like lymph D. which often is worse at night. It does not occur in children. Due to bony destruction by osteoclasts The most common translocations in multiple myeloma are t(11. 937 A. t(13. 738 In Stage III of International Staging System for Multiple Myeloma. AJAY MATHUR Hematology D. 5. 4 A.5 mg / liter In ~20% of myelomas. 937. Densely eosinophilic cytoplasm cells that they have been called B-cell stimulating factors or B-cell growth factors. N Engl J Med 2009. and unlike the pain of metastatic carcinoma.q32). is 70 years.14)(p16. 735 Which of the following interleukins are called B-cell stimulating factors or B-cell growth factors ? 741 Which of the following is not a feature of myeloma cells in A. None of the above serum  2-microglobulin or lactate dehydrogenase. None of the above The frequency of myelomas of a particular heavy chain class is roughly proportional to the Myeloma is uncommon under age 40. and therefore IgG myelomas are more common than IgA and IgD myelomas.14).  2-microglobulin levels is more than ? 744 Osteoclast activating factors (OAF) from myeloma cells are Harrison’s 18th Ed. Males are more commonly affected than females. such as t(4. a densely basophilic cytoplasm. IL-1 . normal levels of Bone lesions of myeloma are caused by proliferation of tumor cells. Ras/Raf/mitogen-activated protein kinase C. and International Staging System stage III. activation of osteoclasts that serum  2-microglobulin or lactate dehydrogenase. thymus. Rare under the age of 40 years B.5 mg / liter urine as Bence Jones proteins. Any of the above B. 937. t(12. Perinuclear clear zone containing Golgi apparatus 736 Which of the following interleukins may play a role in driving Myeloma cells in bone marrow are round or oval cells in shape.14). N Engl J Med 2011. Standard-risk disease is defined by the presence of hyperdiploidy or t(11. 6 B.14) 743 Which of the following about bony lesions in myeloma is false ? Harrison’s 18th Ed. Persistent localized pain in a A. 5. Located to back and ribs C. Thymus B. and a perinuclear clear zone myeloma cell proliferation ? (hof) containing Golgi apparatus. 2. respectively. spleen. B. None of the above glands. IgD myeloma C. increases in incidence with age. and chromosome 1 abnormalities. PI3-K/Akt D. and migration in multiple myeloma are mediated via Ras/Raf/mitogen- activated protein kinase. affecting nearly 70% of patients. 938. drug resistance. with eccentric nucleus having coarsely clumped chromatin. t(10. or deletion 17p13.14) patient with myeloma usually signifies a pathologic fracture. 1 742 Which of the following about bone pain in myeloma is false ? B. Worse at night 737 Most common translocation in multiple myeloma is ? Bone pain is the most common symptom in myeloma. 4. high levels of D. 941 Table 111–3. 3. t(11. Serum alkaline phosphatase is normal even with extensive bone involvement because of the absence of osteoblastic activity. Precipitated by movement Interleukin (IL)-6 may play a role in driving myeloma cell proliferation.14) C.364:1046-60 usually involves the back and ribs. Pain Harrison’s 18th Ed. 734 Plasma cells are produced in ? 740 Drug resistance in multiple myeloma is mediated by ? A. D. PI3-K/Akt. Affects ~ 70% of patients D. and Peyer’s patches. 5 Harrison’s 18th Ed. More common in males than females D. Round or oval shape D. Specific translocations in the immunoglobulin heavy chain region that are detected on fluorescence B. High risk disease and poor prognosis are defined by the presence of one of the following in each category: hypodiploidy. only light chains are produced and in most cases are secreted in the C. Eccentric nucleus with coarsely clumped chromatin Interleukins 4.5 mg / liter 733 Which is the most common form of myeloma ? 739 Which of the following about myeloma is false ? Harrison’s 18th Ed. t(4. deletion 17p13. Growth. 937 Figure 111-2 C. 6 A. tonsils.364:1046-60 mediated by ? Harrison’s 18th Ed.360:2645-54 A. Median age at diagnosis is 70 years C. 938 C. Lymph glands Harrison’s 18th Ed.364:1046-60 A. are C. N Engl J Med 2011. All of the above C. IgG myeloma A. Median age at diagnosis serum concentration. 938 B. D. 4 bone marrow ? B. 938 D. D.5 mg / liter A.62 MCQ’s FOR MEDICAL PROFESSIONALS BY PROF. 937 Harrison’s 18th Ed. None of the above D. N Engl J Med 2011. the pain of myeloma is precipitated by movement. 2 Harrison’s 18th Ed.q32) and t(4. IgA myeloma B. All of the above B. and 6 participate in the B-cell response and have such potent effects on B C. Suppression of osteoblastic new bone formation in situ hybridization (FISH).14) A.14)(q13. most common infection is ? C. Pseudomonas aeruginosa C. 938 745 Dickhoff-1 (DKK-1) produced by myeloma cells causes ? A. 1 to 2 . normochromic 748 In myeloma. and sternum. Klebsiella pneumoniae D. Protein in urine is nearly all light chains Harrison’s 18th Ed. In myeloma. 938 B. localized bone lesions may be palpated Harrison’s 18th Ed. 752 Which of the following regarding renal failure in myeloma patients is false ? 746 In multiple myeloma. Bone lesions are lytic in nature and are rarely associated with osteoblastic new bone formation due to their suppression by dickhoff-1 (DKK-1) produced by myeloma cells. Adult Fanconi syndrome A. None of the above C. The collapse of vertebrae may lead to spinal cord compression. Sternum D. & TNF. Osteoclastic suppression In MM. with loss of glucose & amino acids. along with defects in the ability of kidney to acidify & concentrate urine. All of the above D.8 D. 63 Hematology MCQ’s FOR MCQ’s FOR MEDICAL MEDICAL PROFESSIONALS PROFESSIONALS BY PROF. clavicles. AJAY MATHUR Cardiology 63 B. Thrombocytopenia very rare B. Abnormalities in complement functions D. Cholecystitis D. Hypercalcemia C. Clavicles D. Nephronophthisis B. lymphotoxin. Osteoclastic activation C. None of the above C. Granulocytopenia very rare A. 938 D. Pyelonephritis 754 Which of the following is false about anemia in myeloma patients ? In myeloma. Proteinuria is not accompanied by hypertension A.4 B. Granulocytopenia and thrombocytopenia are very rare. 938 A. the most frequent pathogen in lungs is ? Harrison’s 18th Ed. Legionella pneumophila the tumor. OAF Hypercalcemia is the most common cause of renal failure. most common infections are pneumonias and pyelonephritis. receptor activator of NF. 939 A. 755 Normal relative serum viscosity is ? Harrison’s 18th Ed. function of osteoblasts and osteoclasts. 939 Localized bone lesions in multiple myeloma may cause mass lesions that may be palpated on the skull. 1. A. Medullary cystic kidney disease Harrison’s 18th Ed. Excretion of light chains is almost activity is mediated by IL-1. Increased B. Otitis in retention of chloride. All of the above Normal relative serum viscosity is 1. Multiple myeloma lesion represents a purely osteolytic lesion with little or no osteoblastic activity. Normal Harrison’s 18th Ed. 751 The earliest manifestation of tubular damage in MM is ? Harrison’s 18th Ed. most frequent pathogens are Streptococcus pneumoniae. Harrison’s 18th Ed. All of the above 753 Anion gap in patients with myeloma is ? Harrison’s 18th Ed. Haemophilus influenzae Normocytic & normochromic anemia occurs in 80% of myeloma patients. Diffuse hypogammaglobulinemia B. 938 B. Osteomyelitis Patients with myeloma have a decreased anion gap because M component is cationic. The inhibition of the Wnt pathway suppresses osteoblasts.8 (i. Bone lesions are caused by an imbalance in the from the release of intracellular lysosomal enzymes. always present that causes tubular damage either directly from light chain toxic effects or indirectly macrophage inhibitory factor (MIP)-1. earliest manifestation of tubular damage is adult Fanconi syndrome (a type 2 proximal D. 1. Low granulocyte lysozyme content C. 938. Susceptible to acute renal failure if dehydrated B. All of the above C. whereas the amplification of the RANK pathway and the action of macrophage inflammatory protein 1  (MIP1 ) activate osteoclasts. 750 Renal failure in myeloma patients is mainly due to ? 756 Symptoms of hyperviscosity occur at a level of ? Harrison’s 18th Ed. Receptor activator of NF-B (RANK) ligand B. Staphylococcus aureus.2 A. 1. Recurrent Infections A. Decreased 747 In myeloma.. VEGF. It is due to replacement of normal marrow by expanding tumor cells. to inhibition of hematopoiesis by factors made by D. Figure 111-4 B. 938 Harrison’s 18th Ed. Infiltration of kidney by myeloma cells Osteoclasts are activated by osteoclast activating factors (OAF) from myeloma cells.e. Skull C. 939 749 Immune deficiency in myeloma patients is due to ? Harrison’s 18th Ed. 1. Osteoblastic activation renal tubular acidosis). Osteoblastic suppression D. Any of the above A. Hyperuricemia D.6 C. resulting B.B (RANK) ligand. and Klebsiella pneumoniae in the lungs and Escherichia coli in urinary tract. Tumor necrosis factor (TNF) C. C. 938 on ? A. Normocytic. serum is normally almost twice as viscous as water). 939 A. and to reduced production of erythropoietin by kidney. 940 B. it is IgG3 subclass that has the highest tendency to form both concentration. CD134+ ti e D. With long-term follow-up. abnormal kappa/lambda free light chain ratio. CD138+ D. Spleen C. Visual disturbances A. 940 ta Harrison’s 18th Ed. and a serum 758 In myeloma. The classic triad of myeloma is marrow plasmacytosis (>10%). IgG3 A. Serum M protein > 1. Serum and/or urine M component G Hyperviscosity may lead to headache.5 g/dL D.5 g/dL) are associated with higher incidence of progression of MGUS to myeloma.64 MCQ’s FOR MEDICAL PROFESSIONALS BY PROF. CD142+ Most commonly. R hyperviscosity ? Harrison’s 18th Ed. 5 g/dL for IgG3. 940 B. All myeloma is preceded by MGUS A. 940 . 4 to 5 B. leading to hyperviscosity and cold Serum alkaline phosphatase is usually normal even with extensive bone involvement because agglutination at lower serum concentrations. IgG A. Serum alkaline phosphatase is normal B. tumor expansion is dominantly in bone & bone marrow and rarely causes enlargement of spleen. and serum M protein > 15 g/L other isotypes. None of the above Among IgG myelomas. All of the above D. All of the above C. IgA B. IgA Harrison’s 18th Ed. visual disturbances. Thalidomide & Bortezomib therapy C. lytic bone lesions. lymph nodes or gut-associated lymphatic tissue. Non-IgG subtype B. 940 763 The classic triad of myeloma consists of all except ? A. Lymph nodes D. In myeloma. CD130+ d C. & 7 g/dL for IgA. IgG3 chains D. which of the following paraproteins leads to and/or urine M component. ~ 1 % per year B. a level usually reached at paraprotein D. IgD D. 757 Hyperviscosity may lead to which of the following ? Harrison’s 18th Ed. 3 to 4 A. IgM Harrison’s 18th Ed. 940 A. and IgA paraproteins of the M component favour development of hyperviscosity syndromes. Abnormal kappa/lambda free light chain ratio C. AJAY MATHUR Hematology B. 940. 759 Hyperviscosity is most common with which of the following paraproteins ? 765 What percentage of patients with MGUS go on to develop U Harrison’s 18th Ed. 940 Harrison’s 18th Ed. 943 B. 5 to 6 C. IgM C. Multiple myeloma A. IgG1 B. IgG4 D. ~ 2 % per year 9 C.4% i of patients with IgA and IgG M components.and temperature-dependent aggregates. IgM. of the absence of osteoblastic activity. Bone marrow plasma cells are CD138+ and monoclonal. IgG2 C. 941 myeloma ? - A. Diagnosis of IgM myeloma is reserved for n patients with lytic bone lesions & predominant infiltration with CD138+ plasma cells in bone marrow. Retinopathy B. 762 Myeloma causes enlargement of which of the following ? 768 Extramedullary plasmacytoma usually involves ? Harrison’s 18th Ed. 760 Hyperviscosity is most common with which of the following h 766 Which of the following statements about myeloma is false ? IgG subclass paraproteins ? Harrison’s 18th Ed. Sensory neuropathy is also a side effect of thalidomide & bortezomib therapy. 940 764 Bone marrow plasma cells are ? V A. Bence Jones proteins in urine D. fatigue. ~ 4 % per year r About half of patients with IgM paraproteins develop hyperviscosity compared with only 2 . ~1% per year of patients with MGUS go on to develop myeloma. Dipsticks for detecting proteinuria not reliable for light C. Headache Harrison’s 18th Ed. (1. 761 Sensory neuropathy occurs in which of the following ? 767 Which of the following factor is associated with higher Harrison’s 18th Ed. 940 incidence of progression of MGUS to myeloma ? Harrison’s 18th Ed.6. None of the above concentrations of ~4 g/dL for IgM. Monoclonal gammopathy of undetermined significance B. ~ 3 % per year 9 D. All of the above C. Marrow plasmacytosis (>10%) C. Gut-associated lymphatic tissue Symptoms of hyperviscosity occur at a level of 5 . 941 A. IgG3. All of the above Neuropathy associated with monoclonal gammopathy of undetermined significance (MGUS) and myeloma is more frequently sensory than motor neuropathy and is associated with IgM more than Non-IgG subtype. Lytic bone lesions D. and retinopathy. no myeloma-related organ or tissue impairment (no bone lesions) A. Lactate dehydrogenase A. AJAY MATHUR Cardiology 65 A. Serum levels of IL-6 & Soluble IL-6 receptors 772 In MGUS. 0. Hemoglobin Harrison’s 18th Ed. Vincristine B. 941 < 10%. % plasma cells in marrow C.86 to 2. C. 777 In myeloma. C-terminal cross-linked telopeptide of collagen I. M component B. > 10 % B. serum levels of soluble IL-6 receptor. which of the following is the single most powerful predictor of survival and can substitute for staging ? 771 All of the following is true for monoclonal gammopathies of Harrison’s 18th Ed.95 780 Which of the following drugs is useful in treatment of myeloma ? C. 941 D. >5% D. 65 Hematology MCQ’s FOR MCQ’s FOR MEDICAL MEDICAL PROFESSIONALS PROFESSIONALS BY PROF. 941 uncertain significance (MGUS) except ? A. High levels of lactate dehydrogenase A. C-terminal cross-linked telopeptide of collagen I only light chains in serum and urine. Serum calcium & degree of skeletal involvement A. Lung-associated lymphatic tissue B. < 3 g/dL of M components D.12 to 0. Serum Beta 2-microglobulin B.26 to 1.65 staging. Doxorubicin 774 For diagnosis of myeloma. IgG 779 In myeloma. B. which of the following is the single most powerful B. 0. 940 C. Urinary Bence Jones protein D. Serum  2 -microglobulin and albumin levels are the basis for the three-stage International Staging System (ISS). Serum calcium performance status. No bone lesions 778 In myeloma. All of the above . and IgD in 1%. M component & degree of skeletal involvement. Serum creatinine (TGF). Harrison’s 18th Ed. Hepatocyte growth factor A. calcium. 940 B. IgA in 25%. 0. % plasma cells in marrow. IgM predictor of survival and can substitute for staging ? C. Any of the above 775 For diagnosis of myeloma. circulating plasma cells. All of the above C. no evidence of other B cell proliferative disorders. Serum Beta 2-microglobulin Harrison’s 18th Ed. TGF-beta 773  ) ratio is ?  / The normal free light-chain ( D. > 20 % D. 940 769 Which of the following is a Myeloma staging system ? A. C-reactive protein. Beta-2 microglobulin is a surrogate marker for the overall body tumor burden. marrow plasmacytosis must be ? Harrison’s 18th Ed. transforming growth factor D. Ann Arbor staging system C. bone marrow clonal plasma cells Harrison’s 18th Ed. C. 941 C. > 15 % C. Syndecan-1 N Engl J Med 2006.65 Harrison’s 18th Ed.35 A. Biclonal gammopathies A. which of the following is the single most powerful predictor of survival and can substitute for staging ? Diagnostic Criteria for MGUS are M protein in serum < 3 g/dL. and syndecan-1. Serum albumin D. Submucosal lymphoid tissue of nasopharynx or sinuses D. IgA Harrison’s 18th Ed. which of the following are Extramedullary plasmacytomas usually involve the submucosal lymphoid tissue of the included ? nasopharynx or paranasal sinuses without marrow plasmacytosis. Hemoglobin Besides the above three. Elevated levels of IL-6 770 What other parameter besides serum  2-microglobulin is B.06 to 0. hepatocyte growth factor. 941 A. < 10% bone marrow plasma cells C. other factors may influence prognosis in myeloma patients like : number of cytogenetic abnormalities. High labeling index used in International Staging System (ISS) for myeloma ? Harrison’s 18th Ed. 1.355:2765-70 D. 941 D. 20% of patients have B.355:2765-70 Serum  2-microglobulin is the single most powerful predictor of survival and can substitute for A. Melphalan A. C-reactive protein N Engl J Med 2006. Binet staging system D. Harrison’s 18th Ed. Durie-Salmon staging system a patient of myeloma ? Durie-Salmon staging system is based on Hb. Serum Beta 2-microglobulin Serum M component is IgG in 53% of patients. RAI staging system 776 Which of the following is associated with poor prognosis in D. 940 B. Thymidine kinase B. Gut-associated lymphatic tissue C. All of the above B. which is the most common immunoglobulin affected ? C. Multiple myeloma D. 784 Hematologic diseases causing massive splenomegaly and anemia and without hyperviscosity include all except ? anemia with absent hyperviscosity include ? 9 N Engl J Med 2001. Tumor cell .345:682 A. Tumor cell N Engl J Med 2001. Amyloidosis (associated with plasma-cell dyscrasias) D. doxorubicin G anemia and hyperviscosity include all except ? B. POEMS syndrome In myeloma treatment. Heavy-chain diseases C. Waldenström’s macroglobulinemia A.345:682 B. Leishmaniasis B. 941 789 Hematologic diseases causeing massive splenomegaly. 941 anemia and without hyperviscosity include all except ? A.345:682 786 Hematologic diseases causing massive splenomegaly and A. Polycythemia vera D. Polycythemia vera R C. C. All of the above C.345:682 A.345:682 B. Heavy-chain diseases C. Lenalidomide N Engl J Med 2001. 941 D.bone marrow interaction A. Gaucher disease anemia with hyperviscosity include all except ? N Engl J Med 2001. Chronic lymphocytic leukemia or its prolymphocytic variant D. lenalidomide or thalidomide leads to high response rates without compromising collection of stem cells for transplantation. Bortezomib A. Multiple myeloma V C. Toxoplasmosis D.345:682 A.345:682 B. Leishmaniasis C. All of the above D. tumor cell .bone marrow interaction & bone marrow milieu. Waldenström’s macroglobulinemia C. Waldenström’s macroglobulinemia drugs for treatment of myeloma ? Harrison’s 18th Ed. Niemann-Pick disease B. alkylating agents like melphalan should be avoided as they damage stem cells. High-dose pulsed glucocorticoids anemia and hyperviscosity include all except ? N Engl J Med 2001. Hodgkin’s disease r B. ti e Harrison’s 18th Ed. A. Heavy-chain diseases C. Hodgkin’s disease B. POEMS syndrome splenomegaly” ? Harrison’s 16th Ed. D. dexamethasone with bortezomib. Bone marrow milieu B. Polycythemia vera In myeloma patients who are transplant candidates. POEMS syndrome 782 Dexamethasone is recommended with which of the following D.345:682 N Engl J Med 2001. Chronic myeloid leukemia 9 B.66 MCQ’s FOR MEDICAL PROFESSIONALS BY PROF. Gaucher’s disease N Engl J Med 2001. Agnogenic myeloid metaplasia (with myelofibrosis) i C. Heavy-chain diseases 794 Which of the following diseases can cause “massive D. Chronic lymphocytic leukemia Lenalidomide & bortezomib target tumor cell. B. All of the above B. All of the above . Agnogenic myeloid metaplasia (with myelofibrosis) C. Hodgkin’s disease d 783 Lenalidomide and/or bortezomib target which of the following ? 790 Hematologic diseases causeing massive splenomegaly. Polycythemia vera C. Melphalan B. POEMS syndrome U D. Vincristine. leading to decreased ability to collect stem cells for autologous transplant. Hairy cell leukemia A. Bortezomib D.345:682 D. Amyloidosis (associated with plasma-cell dyscrasias) h D. Multiple myeloma A. AJAY MATHUR Hematology 781 Which of the following drug must not be used in myeloma B. - 791 Hematologic diseases causeing massive splenomegaly.345:682 anemia with absent hyperviscosity include ? A. Chronic myeloid leukemia patients who are transplant candidates ? C. Thalidomide 788 Hematologic diseases causeing massive splenomegaly. Chronic malaria anemia with hyperviscosity include all except ? N Engl J Med 2001. Polycythemia vera Harrison’s 18th Ed. Chronic myeloid leukemia n C. All of the above 792 Causes of massive splenomegaly include all except ? ta 785 Hematologic diseases causing massive splenomegaly and N Engl J Med 2001. 347 787 Hematologic diseases causing massive splenomegaly and A. Kala-azar A. All of the above 793 Diseases that can cause “massive splenomegaly” are ? N Engl J Med 2001. 797 Waldenström’s Macroglobulinemia originates from ? Harrison’s 18th Ed. Hyperviscosity syndrome common C. and half of these patients are positive for anti-MAG antibody. Subarachnoid hemorrhage 796 Waldenström’s Macroglobulinemia resembles which of the 802 Which of the following drugs is useful in Waldenström’s following diseases ? macroglobulinemia ? Harrison’s 18th Ed. Myelin oligodendrocyte glycoprotein (MOG) hyperprolactinemia. Fludarabine (25 mg/m 2 per day for 5 days every 4 weeks) or cladribine (0. Ataxia D. All of the above D. WM does not cause bone lesions or hypercalcemia. 943 (MAG) protein. 943 798 Clinical course of Waldenström’s macroglobulinemia is similar to ? A. Polyneuropathy C. D. Post . Multiple sclerosis B. Endocrinopathy Waldenström’s Macroglobulinemia originates from a post–germinal center B cell that has D. Amenorrhea for which protein ? B. progressive sensorimotor). Chronic lymphocytic leukemia A. WM patients develop peripheral neuropathy. Impairment of vision is a major symptom Harrison’s 18th Ed. Gynecomastia Harrison’s 18th Ed. None of the above D.345:682 A. Multiple myeloma undergone somatic mutations & antigenic selection in lymphoid follicle and has characteristics of an IgM-bearing memory B cell. Does not cause hypercalcemia 807 Levels of which of the following is low in POEMS syndrome ? D. WM and IgM myeloma follow a similar clinical course. Harrison’s 18th Ed. 943 A. gynecomastia. IgG myeloma D. AJAY MATHUR Cardiology 67 795 Which of the following statements about Waldenström’s 801 Neurologic abnormalities that may be associated with macroglobulinemia is false ? Waldenström’s macroglobulinemia include all except ? Harrison’s 18th Ed. Hepatosplenomegaly & lymphadenopathy common B. multiple myeloma. 943 B. IgM myeloma organomegaly. endocrinopathy. but 804 Which of the following is not a feature of POEMS syndrome ? therapeutic options are different. Polyneuropathy Harrison’s 18th Ed. Post . Digital clubbing Harrison’s 18th Ed. Cladribine C. Harrison’s 18th Ed. bendamustine and lenalidomide have improved patient outcome.1 mg/kg per day for myeloma. 67 Hematology MCQ’s FOR MCQ’s FOR MEDICAL MEDICAL PROFESSIONALS PROFESSIONALS BY PROF. C. and lymphocytic lymphoma. bortezomib. Renal disease is not common A. A. Type 2 diabetes mellitus. Skin changes C. 942 N Engl J Med 2001. Orthostatic hypotension D. but 805 Endocrine manifestations seen in POEMS syndrome include ? therapeutic options are different. Neuropathy may precede appearance of the neoplasm. Peripheral neuropathy B. Waldenström’s macroglobulinemia and IgM myeloma follow a similar clinical course. Pre . Hypertrichosis macroglobulinemia is false ? C. hypertrichosis. Organomegaly A. 7 days every 4 weeks) are highly effective single agents. TGF- . C. Hyperpigmentation 800 Which of the following statements about Waldenström’s B.germinal center T cell A. Fludarabine B. Myeloma B. All of the above Waldenström’s Macroglobulinemia resembles related diseases like chronic lymphocytic leukemia. 943 D. 942 Harrison’s 18th Ed. All of the above B. Lymphocytic lymphoma C. Pre . Type 2 diabetes mellitus A. IgA myeloma C.germinal center T cell C. Rituximab (anti-CD20) D. D. Renal disease is uncommon Skin changes in POEMS syndrome are hyperpigmentation. IL-1 and 80% excrete kappa isotype light chain. 943 B. and skin changes (POEMS).germinal center B cell B. 942 803 Which of the following is not a feature of POEMS syndrome ? A. Lymphoplasmacytoid malignancy secreting IgM A. skin thickening & digital B. Lytic bone lesions are frequent clubbing. All of the above A. Encephalopathy C. 943 Unlike myeloma. 943 799 IgM in Waldenström’s Macroglobulinemia have specificity A.germinal center B cell Harrison’s 18th Ed. 943 C. All of the above 806 Skin changes seen in POEMS syndrome include ? IgM in Waldenström’s Macroglobulinemia have specificity for Myelin-associated glycoprotein Harrison’s 18th Ed. hypothyroidism and adrenal insufficiency. B. IgD myeloma The features of POEMS syndrome are polyneuropathy (severe. Myelin basic protein (MBP) D. Rituximab (anti-CD20). Myelin-associated glycoprotein (MAG) Endocrine manifestations of POEMS syndrome include amenorrhea. 345:682 - A. Mu heavy chain disease 817 Schnitzler’s syndrome includes all except ? D. Tangier disease A. These antibodies are called alloantibodies. Lymphadenopathy n A. which is characterized by urticarial skin lesions that are often nonpruritic as well as Most distinctive symptom of Gamma Heavy Chain Disease (Franklin’s Disease) is palatal by recurrent fever. TTP/HUS D. Polycythemia vera 808 Plasmapheresis is not beneficial in which of the following ? C. 943 D. A. C. Gaucher disease D. Serum IgM M component >1000 mg/dL i B. IL-6. Erythematous. Alpha heavy chain disease C. Gamma heavy chain disease D.345:682 A. Alloantibodies 813 Sandhoff disease is nearly identical to which of the following diseases ? B. Alpha heavy chain disease D. 951 .345:682 ti e A. 2453 C.68 MCQ’s FOR MEDICAL PROFESSIONALS BY PROF. 2423 In POEMS syndrome. Multiple Myeloma hepatosplenomegaly & enlarged. POEMS syndrome Tangier disease is associated with cholesterol accumulation in reticuloendothelial system with B. None of the above N Engl J Med 2001. Oligoantibodies B. Wolman disease Harrison’s 18th Ed. 812 Which of the following diseases is related to Mediterranean lymphoma ? Chapter 113. but 819 Antibodies resulting from “natural” exposure are ? hepatosplenomegaly & bony dysplasias are also present. Mu heavy chain disease can cause nodular and macular lesions similar to those seen in leukemia and lymphoma. Gamma heavy chain disease Schnitzler’s syndrome is the association of an IgM monoclonal protein with erythematous. and lymphadenopathy in conjunction with a value for the serum edema due to involvement of nodes in Waldeyer’s ring. Erythematous urticarial skin lesions r A. and levels of the inhibitory cytokine TGF. Waldenström’s Macroglobulinemia 815 Which of the following statements about Schnitzler’s syndrome is false ? Plasmapheresis does not appear to be of benefit in POEMS syndrome N Engl J Med 2001. Autoantibodies A. are lower than expected. None of the above d D. B. Bone pain 810 Seligmann’s disease is also called ? B. h urticarial skin lesions. Dermis is infiltrated by malignant lymphocytic and B. 943 C. Heteroantibodies Harrison’s 17th Ed. Mu heavy chain disease antigens of another individual are called ? D. B. bone pain. 951 A. None of the above Harrison’s 18th Ed. an IgM D. VEGF 814 Enlarged. Fabry disease Autosomal recessive Sandhoff disease is phenotypically similar to Tay-Sachs disease. TNF pathognomonic of ? Harrison’s 17th Ed. VEGF. 944 C. Gamma heavy chain disease plasmacytic cells V C. grayish yellow or orange tonsils are D. urticarial skin lesions R A. Urticarial skin lesions Harrison’s 18th Ed. None of the above monoclonal protein may be deposited in the skin and cause pruritic papules. 943 B.e. Transfusion Biology and Harrison’s 18th Ed. Neimann-Pick disease alloimmunization i. Mu heavy chain disease D. It is the most common of the heavy chain diseases. high circulating levels of the proinflammatory cytokines IL-1. D. Alopecia U C. It may produce respiratory compromise. Waldenström’s macroglobulinemia and TNF is documented. Tay-Sachs disease RBC’s and other cellular blood elements & plasma proteins are antigenic and can result in C. Gamma heavy chain disease 818 Production of antibodies directed against blood group C. Harrison’s 18th Ed. Schnitzler’s ta syndrome. production of antibodies directed against blood group antigens of another individual. Alpha heavy chain disease B. Associated with IgM monoclonal protein G 809 Franklin’s disease is also called ? Harrison’s 18th Ed. grayish yellow or orange tonsils. IgM monoclonal protein Alpha heavy chain disease is also called Seligmann’s disease. Macrocephaly & hyperacusis are present. Alpha heavy chain disease B. Infiltration of the dermis by malignant lymphocytic and plasmacytic cells C. None of the above 816 Clinical features of Schnitzler’s syndrome all except ? Gamma heavy chain disease is also called Franklin’s disease. IgM M component that is usually less than 1000 mg/dL. Infiltration of dermis by malignant lymphocytic & 9 plasmacytic cells 811 Palatal edema is the distinctive symptom of ? 9 Harrison’s 18th Ed. N Engl J Med 2001. AJAY MATHUR Hematology C. 944 Therapy A. 951 D. Naturoagglutinins A. None of the above formation of A or B blood groups ? Harrison’s 18th Ed. the essential substance is ? B. They are found on cellular A. 951 820 Antibodies that result from allogeneic exposure like A. 951 C. Proteins C. None of the above B. None of the above A. Anti-B Harrison’s 18th Ed. IgA 827 Genes that determine the A and B phenotypes are expressed in ? Harrison’s 18th Ed. Neither C. Homoagglutinins Harrison’s 18th Ed. C. 951 H substance is the immediate precursor on which the A and B antigens are added. Anti-B Harrison’s 18th Ed. Mendelian recessive manner C. Carbohydrates The genes that determine the A and B phenotypes are found on chromosome 9p and are expressed in a Mendelian codominant manner. None of the above A. Anti-A 824 Addition of N-acetylgalactosamine to H substance creates ? B. B. IgG D. IgM Addition of N-acetylgalactosamine creates A antigen. IgE A. Individuals who lack “A” and “B” transferases are phenotypically type “O” B. 8p A. 951 D. Anti-A 825 Addition of galactose to H substance creates ? B. C. IgG antibodies can cross placenta resulting in hemolytic disease of the newborn. C. 9p B. Lipids 828 Which of the following “transferases” contributes to the D. B. None of the above B. Fucose D. 951 H substance is formed by the addition of fucose to the glycolipid or glycoprotein backbone. 951 C. IgG D. chromosome ? Harrison’s 18th Ed. Fructose D. B. AB agglutinins B. P substance while those who inherit both transferases are type “AB”. A. 951 C. and are secreted into plasma and body fluids as glycoproteins. 951 A or B antigens are carbohydrates attached to a precursor backbone. while addition of galactose produces B antigen. Mannose 830 Isoagglutinin found in type AB individuals is ? Harrison’s 18th Ed. 69 Hematology MCQ’s FOR MCQ’s FOR MEDICAL MEDICAL PROFESSIONALS PROFESSIONALS BY PROF. 951 D. or hydrops fetalis. Anti-A + Anti-B A. A antigen D. Histone acetyl transferase (HAT) Harrison’s 18th Ed. Glycosyl transferases A. . AJAY MATHUR Cardiology 69 A. H substance Glycosyl transferases are gene products which confer enzymatic capability of attaching specific antigenic carbohydrate. Anti-A + Anti-B A. K substance 829 Naturally occurring anti-A and anti-B antibodies are termed ? Harrison’s 18th Ed. 6p transfusion or pregnancy are usually ? B. Type O individuals produce both anti-A & anti-B. IgM C. None of the above B. 951 C. 951 D. N-acetyl transferase (NAT) membrane either as glycosphingolipids or glycoproteins. Glutathione-S-transferase 822 Immediate precursor upon which A & B antigen are added is ? C. while type B individuals make anti-A isoagglutinins. Mendelian dominant manner Blood transfusion or pregnancy (allogeneic exposure) result in IgG antibody formation. Mendelian codominant manner 821 A and B antigens are ? Harrison’s 18th Ed. Galactose Naturally occurring anti-A & anti-B antibodies are termed isoagglutinins. B antigen Type A individuals produce anti-B. A antigen D. B antigen 831 Isoagglutinin found in type O individuals is ? C. A. Rh antigen isoagglutinin is found in type AB individuals. Rh antigen Harrison’s 18th Ed. Z substance D. Isoagglutinins 823 In the formation of H substance. 7p Harrison’s 18th Ed. IgE 826 Genes that determine A & B phenotypes are found on Antibodies directed against RBC antigens resulting from “natural” exposure are of IgM isotype. IgA D. Duffy RBC blood group systems are Rh (D. 951 C. 844 Which of the following gene is present on the X Kidd (Jk a /Jk b). C. Hemolysis D. which of the following as a haplotype. Usually IgM type 833 Nonsecretors of A & B antigens by cells in the circulation B. i. P antigen B. Lewis antigen - Harrison’s 18th Ed. 1 In PCH. C. Nonsecretors are susceptible to infections (C. 951 A. 952 A. H substance D. 952 835 Which of the following is Rh antigen ? A. E 9 Duffy antigens serve as receptors for Plasmodium vivax. MNSsU system 834 Gene determining Rh phenotypes is found on chromosome ? D. About 15 % B. Duffy antigen C. 951 B. I/i and MNSsU. i The three Rh genes. N. Lewis (Le a . Lewis system V A & B antigens are secreted by cells in the circulation. Malignancy Lewis antigens cannot cross placenta. 952 837 Which of the following is not a RBC blood group system ? A. Cause incompatibility during pretransfusion screening A. 951 B antigens and are therefore compatible only with other hh donors. parvovirus B19 ? Harrison’s 18th Ed. Chromosome 12 Harrison’s 18th Ed. Soluble blood group antigens block this binding. Lewis antigen Harrison’s 18th Ed. Erythrocyte-binding protein 195 (EBA-195) D. None of the above G B. 3 841 Which of the following antigen is the cellular receptor for Plasmodium vivax ? D. 951 C. Erythrocyte-binding protein 165 (EBA-165) A. Rh . About 30 % 843 Which of the following antigen is the cellular receptor of ~15% of individuals lack D alloantigen. B. C C. h microbial ligand interacts with host receptor Glycophorin A ? Harrison’s 18th Ed. All of the above Harrison’s 18th Ed. Kidd antigen A. meningitidis. D. H. A. 2 n C. Le b ). 952 838 The Lewis gene is located on ? Harrison’s 18th Ed. Erythrocyte-binding protein 185 (EBA-185) C. Lewis C. 1014 Table 120–1 836 What percentage of of people lack the D antigen ? ta Harrison’s 18th Ed. About 25 % D. Duffy (Fy a/Fy b ). influenzae) as they may bind to polysaccharides on cells. AJAY MATHUR Hematology 832 Individuals with Bombay phenotype produce antibodies to ? B. 4 U Harrison’s 18th Ed. Chromosome 10 B. E/e. and C/c. P antigen 9 B. Erythrocyte-binding protein 175 (EBA-175) B. >70% of persons in malaria-endemic r D. cDE or Cde. albicans. D. Duffy antigen C. All of the above 839 Which of the following statements about antibodies to Lewis antigens is false ? Individuals with Bombay phenotype produce antibodies to H substance as well as to both A & Harrison’s 18th Ed. Smith D. 951 A. S. AB A. A antigens Lewis gene product is a fucosyl transferase and maps to chromosome 19. autoantibody with biphasic properties (Donath-Landsteiner antibodies) to P is produced that binds to RBCs in cold & fixes complement upon warming. B antigens D. B. Kell (K/k). 952 Table 113–1 B. Kidd antigen A. Chromosome 15 A. Infections 840 Paroxysmal Cold Hemoglobinuria (PCH) is related to ? R D. C/c. 951 A. pneumoniae. Chromosome 19 B. About 20 % C. D D. Kell The P antigen is the cellular receptor of parvovirus B19. chromosome ? Harrison’s 18th Ed. Can cross placenta are susceptible to ? Harrison’s 18th Ed.70 MCQ’s FOR MEDICAL PROFESSIONALS BY PROF..e. P system d C. I system ti e Harrison’s 18th Ed. All of the above areas lack these antigens. E/e). are arranged in tandem on chromosome 1 and inherited 842 In Plasmodium falciparum infection. 953 A. investigated by ? Harrison’s 18th Ed. Hyperpheresis technology FFP contains stable coagulation factors and plasma proteins like fibrinogen. Fibrinogen 849 Which of the following is not a processed component of B. whole blood is first separated into PRBCs & platelet-rich plasma (PRP). Lewis D. 25. then separated D. C. 952 D. 952 C. Kidd Refractoriness to rise in post transfusion platelet counts is investigated by detecting anti-HLA C. A. A. Anti-HLA antibodies in recipient’s serum group antigens ? B. which of the following RBC blood B. Kell 850 Single-donor apheresis platelets (SDAP) is equivalent to D. 50. D. Kell antibodies in the recipient’s serum. Platelets Cryoprecipitate is a source of fibrinogen. 5. Duffy 853 Fresh-Frozen Plasma (FFP) contains all except ? McLeod syndrome is due to absence of Kell precursor protein that leads to acanthocytosis. albumin. M Single-donor apheresis platelets (SDAP) contain platelets equivalent to ~6 units of random B. A. Heat precipitate 855 Each unit of cryoprecipitate contains how much of factor VIII ? D. Factor VIII whole blood ? C. N donor (RD) platelets. 10. Apheresis technology 854 Cryoprecipitate is a source of ? Harrison’s 18th Ed. shortened RBC survival & progressive form of muscular dystrophy that includes cardiac defects. ~ 800 units by centrifugation. 4 persons express ? C. U without fever or infections is ? Anti-U antibodies are rare but problematic. and proteins C and S. 952 D. 953 all persons express U. Packed RBC (PRBC) B. . 2 845 Virtually every donor is incompatible because nearly all B. 953 847 McLeod syndrome is related to which of the following blood A. B. Anti-I autoantibodies in recipient’s serum B. Each unit of cryoprecipitate contains ~80 units of factor VIII. Lewis Threshold for prophylactic platelet transfusion in patients without fever or infections is 10000/ B. ~ 20 units various anticoagulants. Cryoprecipitate Harrison’s 18th Ed. 71 Hematology MCQ’s FOR MCQ’s FOR MEDICAL MEDICAL PROFESSIONALS PROFESSIONALS BY PROF. 952 Kell precursor protein is controlled by a gene on X. and fresh-frozen plasma (FFP) or cryoprecipitate.000 / µL Harrison’s 18th Ed. ~ 80 units platelets.000 / µL group system is most immunogenic ? C.000/µL for those with fever or infections & 50000/µL for those undergoing invasive procedures. 5. All of the above A. factor VIII & von Willebrand factor (vWF). antithrombin. MNSsU µL. Anti-Jk antibodies in recipient’s serum Harrison’s 18th Ed. 953 Collection of multiple units of platelets from a single donor is done by Apheresis technology. Lewis how many units of random donor (RD) platelets ? Harrison’s 18th Ed. Antithrombin A. Fibrinogen 848 Collection of multiple units of platelets from a single donor B. 952 C.000 / µL A. Most donated blood is processed into following components . Kell D. ~ 200 units centrifuged at high speed to yield one unit of random donor (RD) platelets & one unit of FFP. By slow centrifugation. AJAY MATHUR Cardiology 71 C. C. Proteins C and S B.PRBCs. 8 A. S 851 Threshold for prophylactic platelet transfusion In patients D. C. 954 Blood products intended for transfusion are routinely collected as whole blood (450 mL) in A. Prothrombin is called ? Harrison’s 18th Ed. Anti-U antibodies in recipient’s serum A. Neopheresis technology D. 6 Harrison’s 18th Ed. Duffy 852 Refractoriness to rise in post-transfusion platelet counts is The immunogenicity of Kell is third behind the ABO and Rh systems. C. Thrombopheresis technology D. von Willebrand factor (vWF) Harrison’s 18th Ed. Harrison’s 18th Ed.000 / µL 846 After ABO and Rh systems. Platelet-rich plasma is then C. Virtually every donor is incompatible because nearly Harrison’s 18th Ed. Cryoprecipitate is produced by thawing FFP to precipitate the plasma proteins. 952 D. Low-titer anti-HLA antibodies in donor plasma reactions associated with plasma transfusion ? C. A. It is highly resistant to treatment B. HPA-1a on the platelet glycoprotein IIIa receptor A. AJAY MATHUR Hematology 856 Infectious agents rarely associated with transfusion include ? C. Leucocytes 865 Cause of TRALI usually is ? Harrison’s 18th Ed. Duffy A. 955 Laboratory evaluation for posttransfusion hemolysis includes the measurement of serum d haptoglobin.4 weeks posttransfusion. All of the above 863 Transfusion-associated GVHD (TA-GVHD) is characterized by ? V Harrison’s 18th Ed. Alloantibodies directed against other B. Haptoglobin Platelet-specific antibodies are found in recipient’s serum. Antibodies directed against donor leukocyte & HLA Harrison’s 18th Ed. IgE deficiency mediators that increase capillary permeability. hepatitis A virus. R Plasmapheresis can be used to remove the antibodies.250 mg of iron.10 days after platelet transfusion. 954 Predominantly in women. RBC D. IgA deficiency TRALI usually results from the transfusion of donor plasma that contains high-titer anti-HLA B. 955 D. I Harrison’s 18th Ed. Rh Each unit of RBCs contains 200 . Allergic Reactions with immunosuppressive therapies like glucocorticoids. C. cyclosporine. Indirect bilirubin D.72 MCQ’s FOR MEDICAL PROFESSIONALS BY PROF. 956 transfusion and should therefore receive only IgA-deficient plasma and washed cellular blood components. Treatment with IVIg neutralizes the effector antibodies. Kell . Anaphylactic Reaction appear at 8 . G A. Trypanosoma cruzi (Chagas disease). Antibodies react to both donor B. Anaplasma phagocytophilum (human granulocytic ehrlichiosis). Babesia microti (babesiosis). Hypothermia 857 Laboratory evaluation for posttransfusion hemolysis C. 6 hours 859 Urticarial reactions are related to which of the following B. D. Kell. West Nile virus I and i are not allelic pairs but are carbohydrate antigens. 100 to 125 mg of iron B. B. 125 to 150 mg of iron 861 Alloantibodies directed against which of the following RBC C. 9 A. A. and most frequently recognized antigen is HPA-1a found on the platelet glycoprotein IIIa receptor. 24 hours i Harrison’s 18th Ed. Borrelia burgdorferi (Lyme disease). High-titer anti-HLA antibodies in donor plasma Harrison’s 18th Ed. Harrison’s 18th Ed. antithymocyte globulin & ablative therapy followed by allogeneic bone marrow transplantation. Plasma proteins D. IgM deficiency 866 Iron content in each unit of RBCs transfusion is ? Patients who are IgA-deficient are at risk for anaphylactic reactions associated with plasma Harrison’s 18th Ed. C. Platelets TRALI. occurs within 6 hours after blood transfusion & is characterized by dyspnea & hypoxia due to noncardiogenic pulmonary edema. N Engl J Med 1999. 150 to 200 mg of iron antigens does result in hemolysis ? Harrison’s 18th Ed. Febrile Nonhemolytic Transfusion Reaction (FNHTR) n TA-GVHD is characterized by marrow aplasia & pancytopenia. D. Marrow aplasia & pancytopenia B. IgG deficiency antibodies that bind recipient leukocytes which aggregate in pulmonary vasculature & release C. Graft-versus-host disease (GVHD) 864 Transfusion-related acute lung injury (TRALI) occurs within - Most frequent reaction associated with transfusion of cellular blood components is a febrile how many hours after blood transfusion ? nonhemolytic transfusion reaction (FNHTR). A. 200 to 250 mg of iron A. hence the delay. All of the above Harrison’s 18th Ed. lactate dehydrogenase (LDH). and human herpesvirus-8. Lactate dehydrogenase (LDH) & recipient platelets. and indirect bilirubin levels. Thrombocytopenia includes ? D. 955 parvovirus B-19. thrombocytopenia may occur 7 . an acute respiratory distress syndrome. Treponema pallidum 862 Which of the following can occur after blood / blood product D. All of the above transfusion ? Infectious agents rarely associated with transfusion include West Nile virus. 48 hours h B. High-titer anti-HLA antibodies in recepient plasma 860 Patients with which of the following are at risk for anaphylactic B. B. ta C. Graft-versus-host disease (GVHD) Treponema pallidum.340:442 antigens & cytokines released from cells within stored blood components mediate these reactions. All of the above A. 954 Table 113–3 D. Resistance to immunosuppressive therapies ti e 858 Most frequent reaction associated with transfusion of cellular blood components is ? C. and Duffy may result in hemolysis.10 days and death occurs at 3 . Hepatitis A virus RBC antigens like Rh. 955. A. Clinical manifestations appear at 8 .10 days Harrison’s 18th Ed. 955 Urticarial reactions are related to plasma proteins found in transfused components. 955 D. 12 hours 9 components of transfused blood ? r C. 952 Table 113–1 D. Clinical manifestations U C. 955 A. Other D. C. Babesia microti (babesiosis). Harrison’s 18th Ed. 956 D. N Engl J Med 1999. Beta blockers 874 Perfluorocarbons are best known as ? Harrison’s 16th Ed. A. C. Chromaffin cells of the fetal adrenal medulla 870 Which of the following carries the least risk of transmission D. Dengue. Glycoprotein 41 878 What are Pappenheimer bodies ? D. Pseudomonas. 10 873 Parasitic diseases that can be transmitted by blood transfusion B. Nuclear fragments acid amplification testing (NAT). HIV-2 C. 956 Chagas disease. All of the above Harrison’s 18th Ed. 956 B. Nuclear fragments 871 To detect HIV-1 infection.340:442 879 What is basophilic stippling or punctate basophilia ? A. All of the above C. All of the above infectious agents rarely associated with transfusion include . A. Acinetobacter. and Escherichia species have all been Harrison’s 18th Ed. AJAY MATHUR Cardiology 73 867 Symptoms and signs of iron overload are common after how Organism most commonly causing bacterial contamination of RBCs is Yersinia enterocolitica. Yersinia enterocolitica B. Transfusion-related acute lung injury (TRALI) A. Trypanosoma cruzi (Chagas 877 What are Howell–Jolly bodies ? disease). 100 A. Klebsiella pneumoniae A. parvovirus B-19.353:498-507 B. Infectious complications N Engl J Med 2005. variant Creutzfeldt-Jakob disease. hepatic. Calcium channel blockers A.Hepatitis A virus.353:2042-55 C. 73 Hematology MCQ’s FOR MCQ’s FOR MEDICAL MEDICAL PROFESSIONALS PROFESSIONALS BY PROF. HIV-1 B. Harrison’s 18th Ed. Staphylococcus epidermidis C. and HIV RNA using Nucleic A. Fat substitutes D. Borrelia burgdorferi (Lyme disease). 956 876 Specialized tissues that sense local oxygen tension include ? A. All of the above N Engl J Med 2005. are presently in various stages of clinical trials. 50 Harrison’s 18th Ed. Fetoplacental arteries D. Transfusion-associated GVHD 875 Specialized tissues that sense local oxygen tension include ? B. All of the above . and Treponema pallidum . Hemosiderin-containing granules 872 Bacteria most commonly implicated in contamination of red C. many units of RBCs transfusion ? Yersinia. Ductus arteriosus No cases of HIV-2 infection have been reported in the United States since 1992. and yellow fever vaccine virus can also be transmitted by blood transfusion.353:498-507 A. All of the above 868 Transient hypotension during blood transfusion is more common in those taking ? Parasitic diseases that can be transmitted by blood transfusion include malaria. Hepatitis C A. Hemosiderin-containing granules D. B. donor blood is tested for ? Harrison’s 18th Ed. Hepatitis B N Engl J Med 2005. p24 antigen C. Altered ribosomes B. 956 A. Risk of HIV-1 infection per transfusion episode is 1 in 2 million. Glycoprotein 120 D. N Engl J Med 2005. All of the above B. Anaplasma phagocytophilum.353:498-507 Donated blood is tested for antibodies to HIV-1. All of the above through blood transfusion ? Harrison’s 18th Ed. babesiosis. Altered ribosomes cells during blood transfusion is ? D. cardiac) are common after 100 units of RBC transfusion (total-body iron load of 20 grams). Babesiosis Symptoms and signs of iron overload (endocrine. Angiotensin-converting enzyme (ACE) inhibitors B. Serratia. 20 include ? C. such as perfluorocarbons and aggregated hemoglobin solution. Hemosiderin-containing granules A. Malaria B. Glomus cells of the carotid body D. All of the above B. Neuroepithelial bodies in the lungs C. Oxygen-carrying blood substitutes D.353:2042-55 B. chikungunya virus. Staphylococcus aureus N Engl J Med 2005. 956. HIV-1 p24 antigen. Smooth-muscle cells of the resistance pulmonary arteries C. Blood products contain bradykinin that is normally degraded by ACE. Iron chelators 869 Nucleic acid amplification testing (NAT) is used to prevent which of the following complications of transfusion ? Oxygen-carrying blood substitutes. 667 C. Protein substitutes Transient hypotension is noted in transfused patients taking ACE inhibitors due to increased bradykinin levels. Chagas’ disease D. 956 implicated in infections related to PRBC transfusion. Altered ribosomes D. human herpesvirus-8 and hepatitis G virus. Nuclear fragments C. EDTA 882 Synthesis of thrombopoietin is increased specifically by ? B. 50000 .40. 15000 . Harrison’s 18th Ed. Kidney D. 965 C. AJAY MATHUR Hematology 880 Microspherocytes can be found in ? 886 Which of the following statements is false ? N Engl J Med 2005. TPO binds to its receptor on platelets A. Three . Nitric oxide is a platelet inhibitor B. Burns C. Two .18 days Drugs are the most common cause of thrombocytopenia. 7 . circulation. TPO. Liver Pseudothrombocytopenia is an in vitro artifact resulting by blood collection. Synthesis is increased with inflammation and specifically by interleukin 6. Approximately one-third of the platelets reside in the spleen 885 Which of the following statements is false ? 891 Which of the following statements about petechiae is false ? Harrison’s 18th Ed. a reduction in platelet and megakaryocyte mass increases the level of Excessive bruising is seen in disorders of both platelet number and function. 965 C.74 MCQ’s FOR MEDICAL PROFESSIONALS BY PROF. Cold weather A. Endothelium-derived vasoconstrictor endothelin is a platelet activator. Chapter 115. .third C. All of the above D.000 / µL B. Inherited disorders A.10. 889 Which of the following is the most common cause of thrombocytopenia ? 883 Average life span of platelets in circulation is ? Harrison’s 18th Ed. Platelets adhere to exposed intimal surface through VWF D. Therefore. 11 .third A. which then stimulates platelet production. 965 A. 890 What is the platelet count required to maintain vascular 884 What proportion of platelets reside in the spleen ? integrity in the microcirculation ? Harrison’s 18th Ed. Microangiopathic hemolytic anemia B. 1 . All of the above B. nonblanching TPO binds to its receptor on platelets and megakaryocytes and is thus removed from the hemorrhages and are usually a sign of a decreased platelet number and not platelet dysfunction. Half B. None of the above Endothelium-derived vasodilator nitric oxide is a platelet inhibitor. Pseudothrombocytopenia D.353:498-507 Harrison’s 18th Ed.000 / µL D. Drugs C.000 / µL C.10 days D. None of the above D.thrombopoietin B. 25000 . Pinpoint D.20. Average life span of platelets in circulation is 7-10 days. Prostaglandin D2 C. TPO binds to its receptor on megakaryocytes B. Heparin A.10. Nonblanching hemorrhages B. Fasting B. 966 Harrison’s 18th Ed. One . Exercise 881 Major regulator of platelet production . 965 A. C.000 is required to maintain vascular integrity in microcirculation. 965 A.000 / µL A platelet count of ~5000 . Sodium citrate Harrison’s 18th Ed. Disorders of Platelets and Vessel Wall 887 Pseudothrombocytopenia is due to ? Harrison’s 18th Ed. Spherocytic hemolytic anemia A. Sign of a platelet dysfunction Petechiae first appear in areas of increased venous pressure and are pinpoint. 5000 . 965 A. 966 Harrison’s 18th Ed. Adipose tissue collected with which of the following anticoagulants ? Thrombopoietin (TPO) is a hormone synthesized in liver that regulates platelet production.fourth D. 965 A.7 days C. Endothelin is a platelet activator C. Platelets were identified in 1865 and their function was elucidated by Giulio Bizzozero in 1882. Interleukin 6 D. TNF alpha Pseudothrombocytopenia is an in vitro artifact resulting from platelet agglutination via antibodies when the calcium content is decreased by blood collection in ethylenediamine tetraacetic D.3 days B. Muscle 888 Pseudothrombocytopenia is most likely when blood is D.100. Myelodysplasia B. 966 Harrison’s 18th Ed. Interferon beta (EDTA) anticoagulated blood tubes. 3 . 965 A. Platelets are anucleate C. Laboratory artifact (TPO) is synthesized in ? Harrison’s 18th Ed. First appear in areas of increased venous pressure C. 894 Which out of the following drugs is most capable of 900 Which of the following statements about HIT is false ? inducing drug-dependent antibodies ? Harrison’s 18th Ed. . All of the above A. Lepirudin Thrombocytopenia (HIT) ? C. Heparin C. Viral infection A. 967 A. 10 times Wet purpura. More common in medical than in surgical patients B. Lepirudin. 967 A. 966 with the use of UFH ? A. HIT is not usually severe Early recognition.7 days C. and usually resolves in 7-10 days after drug withdrawal. AJAY MATHUR Cardiology 75 892 In wet purpura. Any of the above C. 967 C.21 days Immune-mediated thrombocytopenia (ITP2) in children usually follows a viral infection and almost always resolves spontaneously. 14 .5 days D. 967. Oral mucosa B. 5 times D. and trimethoprim-sulphamethoxazole. 75 Hematology MCQ’s FOR MCQ’s FOR MEDICAL MEDICAL PROFESSIONALS PROFESSIONALS BY PROF. Danaparoid Harrison’s 18th Ed. They are more common with quinine. None of the above 903 Secondary immune thrombocytopenic purpura is associated with which of the following diseases ? 897 In Heparin-Induced Thrombocytopenia (HIT). HIT is more frequent in females than in males C. 995 Harrison’s 18th Ed. Scalp C. 995 895 Drug induced thrombocytopenia usually resolves how many A. Sole Harrison’s 18th Ed.3 days D. 967. A. Quinine HIT is more common in surgical patients than in medical patients Classic drug-dependent antibodies are antibodies that react with specific platelet surface antigens and result in thrombocytopenia only when the drug is present. Platelet factor 4 (PF4) B. 3 . Palm A. Systemic lupus erythematosis (SLE) A. 968 formation is against which of the following ? Harrison’s 18th Ed. 968 length 21 days).21 days of Heparin induced thrombocytopenia and thrombosis (HITT) ? Drug induced thrombocytopenia typically occurs after a period of initial exposure (median Harrison’s 18th Ed. Drug ingestion B.3 days C. 901 Which of the following about HIT is false ? Harrison’s 18th Ed. Most specific diagnostic test is serotonin release assay days after drug withdrawal ? B. None of the above B. 967 many days ? A. Most patients develop HIT after exposure to heparin for 5 – 14 days. 1 . Amiadarone B. rifampicin. Platelet transfusions are not advised A. Digoxin D. HIT markedly increases risk of thrombosis effective in HITT. HIV B. 967 B. or upon reexposure. quinidine. Platelet factor 4 (PF4) and heparin complex D. 7 . Argatroban 896 Which of the following is false about Heparin-Induced B. Hepatitis C C. blood blisters form on ? 898 As compared to LMWH. 967 B. Danaparoid. HIT is how many times more common Harrison’s 18th Ed. 3 . blood blisters that form on the oral mucosa denote an increased risk of life- threatening hemorrhage in the thrombocytopenic patient. HIT is not associated with bleeding key in treatment of HIT. Endothelial cells Immune / idiopathic thrombocytopenic purpura ITP is termed secondary if it is associated with an underlying disorder like systemic lupus erythematosis (SLE). Occurs earlier if heparin given within past 3 months A.10 days 902 Which of the following drugs is effective in the treatment D. HIT is about 10 times more common with the use of unfractionated heparin (UFH) than after exposure to low-molecular-weight heparin (LMWH). D.14 days D. Venous thrombosis more common than arterial thrombosis Harrison’s 18th Ed. Bacterial infection Harrison’s 18th Ed. 5 . All of the above D. Argatroban. Bivalirudin & Fondaparinux are C. antibody Harrison’s 18th Ed. Platelet count rarely fall to <20000/µL D. 967 D. Thrice D. 14 . Cephalosporin C. 1 . Twice C. 893 Immune-mediated thrombocytopenia (ITP2) in children usually follows which of the following ? 899 Most patients develop HIT after exposure to heparin for how Harrison’s 18th Ed. prompt discontinuation of heparin and use of alternative anticoagulants is B. HIV and hepatitis C HIT results from antibody (IgG isotpe) formation against heparin-PF4 complex & to platelet Fc receptors. Neurologic changes are common in B. Wiskott-Aldrich C. D. 969 B. Despite thrombocytopenia. severe bleeds are uncommon. 4 gram / kg described by ? In ITP. Bernard Soulier syndrome D. 969 A. Balint 908 Which of the following drugs is useful in refractory ITP ? Harrison’s 18th Ed. and fever. Microangiopathic hemolytic anemia B. 969 B. and Fechtner syndromes have thrombocytopenia Harrison’s 18th Ed.000 / µL A. laboratory evidence of hemolysis & microvascular thrombosis.one administered subcutaneously (romiplostim) Harrison’s 18th Ed. Wiskott-Aldrich syndrome D. Rituximab C. 969 and another orally (eltrombopag). 969 A. 75. C. Bovine plasma B. Epstein’s syndrome in PBF. It includes thrombotic thrombocytopenic purpura (TTP) & hemolytic uremic syndrome (HUS). Autosomal recessive inherited thrombocytopenic disorders include congenital amegakaryocytic 905 Thrombocytopenia related mortality increases with platelet thrombocytopenia. Schistocytes are typically seen C. renal failure.000/µL appear not to have increased mortality related to the D.76 MCQ’s FOR MEDICAL PROFESSIONALS BY PROF. Verner-Morrison B. PT A. Bernard Soulier syndrome thrombocytopenia. Two thrombopoietin receptor agonists . thrombocytopenia with absent radii and Bernard Soulier syndrome. A. Romiplostim D. 30. Human plasma thrombocytopenic microangiopathies ? Harrison’s 18th Ed. D. Renal failure 909 Large platelets is a feature of which of the following ? C. Hepatic failure Harrison’s 18th Ed. Thrombocytopenia C. Autoimmune hemolytic anemia with ITP Harrison’s 18th Ed. Thrombocytopenia with absent radii D. Laboratory evidence of hemolysis D. D. Pig plasma C. neurologic findings.000 / µL Harrison’s 18th Ed. Congenital amegakaryocytic thrombocytopenia C. Horse plasma A. total dose of IVIgG is ? Thrombotic thrombocytopenic microangiopathies are characterized by thrombocytopenia. All of the above Autoimmune hemolytic anemia with ITP is termed as Evans’s syndrome. 968 include ? A. Eltrombopag TTP was initially described by Dr Eli Moschcowitz at the Mount Sinai Hospital in New York City in 1924. 969 microangiopathic hemolytic anemia. Abnormal PT and aPTT All IVIgG immunoglobulin preparations are derived from human plasma. 969 B. counts less than ? Harrison’s 18th Ed. Thrombotic thrombocytopenic purpura . 968 911 Which of the following is inherited as X-linked pattern in inherited thrombocytopenia ? A. May-Hegglin anomaly D. SLE with ITP A. renal failure in HUS & neurologic or renal changes are absent in HELLP. 906 All IVIgG immunoglobulin preparations are derived from ? Harrison’s 18th Ed. total dose of IVIgG is 2 gram / kg. have shown response in many patients with refractory ITP. HIV with ITP B.000 / µL C. X-linked inherited thrombocytopenic disorders include Wiskott-Aldrich syndrome. Harrison’s 18th Ed. 1 gram / kg & aPTT are characteristically normal in TTP or HUS and is used to differentiate TTP from DIC. 3 gram / kg 913 Thrombotic thrombocytopenic purpura (TTP) was first D. thrombocytopenia. 2 gram / kg TTP. All of the above Rituximab. AJAY MATHUR Hematology 904 Evans’s syndrome refers to ? 910 Autosomal recessive inherited thrombocytopenic disorders Harrison’s 18th Ed. an anti-CD20 (B cell) antibody. 969 B. Multiple transfusions B. Harrison’s 18th Ed. has shown efficacy in the treatment of refractory 914 Which of the following is not a feature of TTP ? ITP. All of the above 915 Upshaw-Schulman syndrome is related to ? May-Hegglin anomaly. and Sebastian. Epstein’s. Microvascular thrombosis 907 In ITP. Hepatitis C with ITP C. 50.000 / µL B. Sebastian syndrome TTP is characterized by a pentad of findings that include microangiopathic hemolytic anemia. A. Neurologic dysfunction B. 969 with large platelets & are inherited as autosomal dominant & are associated with mutations in nonmuscle myosin heavy chain MYH9 gene. May-Hegglin anomaly C. Thrombotic thrombocytopenic purpura (TTP) is also called Moschcowitz disease. 100. Eli Moschcowitz A. 969 912 Which of the following is false regarding thrombotic A. Fechtner syndrome Patients with platelet counts >30. Plasma exchange B. Rituximab Harrison’s 17th Ed. 679 D. Near normal fibrinogen level . 77 Hematology MCQ’s FOR MCQ’s FOR MEDICAL MEDICAL PROFESSIONALS PROFESSIONALS BY PROF. 723 917 Idiopathic TTP is more common in ? A. C. Harrison’s 16th Ed. HIV infection & in pregnant women. D. Major constituents of Weibel-Palade bodies are von toxicity (cyclosporine. Inherited TTP D. 722 A. Coagulation studies are essentially normal in TTP Harrison’s 17th Ed. 679 D. 969 B. ADAMTS 11 D. HIV infection D. Severe Coombs-positive hemolytic anemia ADAMTS 13 is a member of ADAMTS (a disintegrin with thrombospondin type 1 motifs) zinc B. Near normal FDP level D. All of the above Idiopathic TTP is more common in women. Arterioles 926 Which of the following is false about Weibel-Palade bodies B. quinine). Severe deficiency of ADAMTS causes TTP. hyaline thrombi without inflammatory changes in C. increased indirect bilirubin. 679 D. It is cleaved by ADAMTS13. Splenectomy C. D. It is due to inherited deficiency of or antibodies to the plasma metalloprotease ADAMTS13. Medication-related TTP Findings that support the diagnosis of TTP include increased LDH. Anemia 927 TTP is due to deficiency in the activity of ? B. Ticlopidine D. 969 C. Weibel-Palade bodies are secretory organelles used for 920 Severity of TTP is estimated by the degree of ? post-synthesis storage in endothelial cells. Women C. ADAMTS 14 A. Idiopathic TTP C. George Emil Palade won the Nobel Prize in Physiology or vessel wall may be found in ? Medicine in 1974 Harrison’s 16th Ed. Venules A. 723 916 Upshaw-Schulman syndrome is best related to ? A. TTP in pregnant women decreased haptoglobin. TTP is more common in HIV patients and pregnant women in the causation of TTP ? B. 923 Mainstay of treatment of idiopathic TTP is ? Harrison’s 17th Ed. All of the above coagulation hemostasis and inflammation Presence of hyaline thrombi in arterioles. Hereditary form of TTP is called Upshaw-Schulman syndrome. Near normal prothrombin time metalloproteinase family that cleaves vWF complexes & prevents vWF-platelet interaction. Capillaries (WPBs) ? C. None of the above A. Weibel-Palade bodies play a dual role in blood D. increased reticulocyte count. 5-10% of all TTP cases are due to Upshaw-Schulman syndrome. None of the above C. 924 Which of the following is false about TTP ? Harrison’s 18th Ed. Increased indirect bilirubin A. Are organelles in the vascular endothelium Medication-related TTP may be secondary to antibody formation (ticlopidine) or direct endothelial B. All of the above C. Uremia D. 969 918 Which of the following antiplatelet agent has been implicated A. Persistence of ultra-large vWF molecules produces pathogenic platelet adhesion & aggregation. capillaries & venules without any inflammatory B. ADAMTS 13 Harrison’s 16th Ed. All of the above A. 922 Which of the following supports the diagnosis of TTP ? Harrison’s 17th Ed. CHF Presence of severe Coombs-negative hemolytic anemia with schistocytes or fragmented RBC’s in PBF. AJAY MATHUR Cardiology 77 C. with thrombocytopenia & minimal activation of coagulation system confirm diagnosis of TTP. Pregnant women Plasma exchange remains the mainstay of treatment of ITP idiopathic TTP. All of the above B. Severe bleeds are usually absent B. Willebrand factor (vWF) and P-selectin 919 In TTP. Weibel-Palade bodies are the main source of vWF changes in the vessel wall is diagnostic of TTP. Increased lactate dehydrogenase Harrison’s 18th Ed. Upshaw Schulman syndrome is due to a hereditary deficiency of a metalloprotease that cleaves vWF which is normally secreted as ultra-large multimers. Carvadilol 925 Which of the following is false about Weibel-Palade bodies ? D. 722 B. negative direct antiglobulin test. Thrombocytopenia Harrison’s 18th Ed. C. mitomycin C. None of the above A. Serum LDH level A. IV Ig A. tacrolimus. Decreased haptoglobin B. ADAMTS 12 921 Which of the following is not a feature of TTP ? C. Aspirin C. schistocytes in PBF and polychromasia. Plasma exchange treatment is not appropriate for hemolytic-uremic syndrome. 935 Hemolytic Uremic Syndrome is a disease of ? Harrison’s 18th Ed. Seizures and fluctuating focal deficits B. Platelet concentrates 939 Bleeding symptoms or prevention of bleeding in patients B. cyclophosphamide. Glanzmann’s thrombasthenia C. Edward Glanzmann . ADAMTS-12 B. 932 Which of the following laboratory finding does not support cancer. Plasma exchange in inherited disorders of platelet dysfunction include ? C. 970 B. D. All of the above D. Increased reticulocyte count C. Deficiency of ADAMTS-13 is a finding for TTP D. vomiting and weakness A. ADAMTS-13 molecule depends upon zinc and calcium HUS is a syndrome characterized by acute renal failure. ADAMTS-13 C. infection) or underlying myeloproliferative process (essential thrombocythemia or PV). and thrombocytopenia. reactive thrombocytosis (inflammation. PBF shows schistocytes and polychromasia. Use of platelet concentrates and B. Increased lactate dehydrogenase platelet GpIIbIIIa receptor ? Harrison’s 18th Ed. None of the above Various studies support the clinical diagnosis of TTP when ADAMTS-13. ADAMTS-13 prevents inappropriate microvascular platelet B. 970 929 Which of the following is false about ADAMTS-13 ? A. Thrombocytopenia without leukopenia C. Heyde’s syndrome test. ADAMTS-11 A. vincristine. Solvent/detergent-treated plasma (SD-FFP) A. Thrombocytopenia B. Vincristine D. 970 938 In which of the following disorders. Microangiopathic hemolytic anemia A. Infancy and early childhood A. ADAMTS refers to “A Disintegrin-like And Metalloprotease with ThromboSpondin type 1 motif” C. Deficiency of ADAMTS-13 results in TTP. The relationship of reduced ADAMTS13 to the pathogenesis of TTP is known as the Furlan-Tsai hypothesis. increased reticulocyte count with negative direct antiglobulin D.78 MCQ’s FOR MEDICAL PROFESSIONALS BY PROF. Escherichia coli ions for activity O157:H7 is the most frequent etiologic serotype. Adolescence aggregation C. All of the above Plasma metalloprotease ADAMTS-13 (vWF-cleaving protease) is responsible for proteolytic cleavage of large hemostatically hyperactive vWf multimers that are synthesized and secreted Immunomodulatory therapies reported to be successful in refractory or relapsing TTP include by endothelial cells in plasma into smaller less adhesive multimers. Autosomal recessive inherited qualitative disorders of platelet function include Glanzmann’s thrombasthenia (absence of the platelet GpIIbIIIa receptor) and Bernard Soulier syndrome (absence 933 Which of the following is not used in the treatment of TTP ? of the platelet GpIb-IX-V receptor). Desmopressin (DDAVP) Desmopressin. Increased direct bilirubin A. Adults B. nausea. activity is less than 5% of the activity in normal human plasma. 970 A. there is absence of the A. Hermansky-Pudlak syndrome Laboratory finding that support diagnosis of TTP include increased LDH. increased “indirect bilirubin”. inappropriate microvascular platelet aggregation. ADAMTS-10 Harrison’s 18th Ed. Chronic renal failure 930 Which of the following is false about ADAMTS-13 ? B. Frequently preceded by an episode of diarrhea C. D. In most cases is preceded by an episode of diarrhea. decreased haptoglobin. Cyclophosphamide D. Elderly C. Calcium Thrombocytosis is almost always due to iron deficiency. and splenectomy. Prestorage leukodepleted platelets transfusion Plasma exchange remains the mainstay of treatment of TTP. Vitamin B12 B. ADAMTS-13 prevents rituximab. Fresh-frozen plasma (FFP) Harrison’s 18th Ed. Human ADAMTS-13 gene is on chromosome 9q34 D. or DDAVP in the treatment of TTP is contraindicated as it leads to extensive C. All of the above . Folic acid C. Iron D. Abdominal pain. D. None of the above 931 Findings in TTP include ? 937 Thrombocytosis is due to deficiency of ? Harrison’s 18th Ed. EACA or tranexamic acid platelet aggregation in the CNS. Decreased haptoglobin B. the vWf-cleaving 936 Features of Hemolytic Uremic Syndrome include all except ? protease. Bernard Soulier syndrome D. microangiopathic hemolytic anemia. 969 refractory or relapsing TTP include ? A. 970 N Engl J Med 2006. diagnosis of TTP ? Harrison’s 18th Ed. AJAY MATHUR Hematology 928 Which of the following is related to TTP ? 934 Immunomodulatory therapies reported to be successful in Harrison’s 18th Ed. Harrison’s 18th Ed.354:1927-35 A.Swiss physician (1918) A. 970 B. Rituximab C. vWf multimers is the physiologic substrate for ADAMTS-13 HUS is seen predominantly in children. 971 D. 676 Harrison’s 16th Ed. Type I C. Functional platelet disorders of granule release are inherited (Hermansky-Pudlak syndrome. drug induced In 1925. 676 B. French B. Glanzmann’s thrombasthenia D. 971 C. Dr. 676 A. Hermansky-Pudlak syndrome D. Glanzmann’s thrombasthenia A. thrombin or epinephrine. dysproteinemias. 950 Which of the following is not a functional platelet disorder 944 Normal plasma vWF level is ? of granule release ? Harrison’s 16th Ed. Chediak-Higashi syndrome C. Type IIA B. Bernard-Soulier syndrome B. Type IIA D. Spanish D. von Willebrand’s disease (vWD) D. Glanzmann’s thrombasthenia Harrison’s 18th Ed. 940 Acquired platelet dysfunction can occur in ? Harrison’s 18th Ed. Harrison’s 16th Ed. 971 947 Which of the following vWD type is most severe ? A. 676 943 Nationality of Dr. NSAIDs). Thrombin C. acquired vWD). Type IIB platelets. Endothelial cells B. Antifibrinolytic therapy (EACA or tranexamic acid) is used alone or in conjunction with DDAVP or platelet therapy. Type III Type I vWF disease is most common. Type I A. 676 A. myeloproliferative disorders. High dose penicillins A. D. 676 952 vWF precursor subunit is synthesized in ? Harrison’s 16th Ed. Bernard Soulier syndrome Functional platelet disorders of adhesion are either inherited (Bernard-Soulier syndrome. Conjugated estrogens Harrison’s 16th Ed. Desmopressin (DDAVP) increases plasma vWF & FVIII levels. D. Danish Functional platelet disorders of aggregation are either inherited (Glanzmann’s thrombasthenia. Hemophilia 949 Which of the following is a functional platelet disorder of von Willebrand Disease (vWD) is the most common inherited bleeding disorder with prevalence aggregation ? of ~1%. Erik von Willebrand of Finland observed a disorder that was due to a disturbed . respond to ? Harrison’s 18th Ed. All of the above A. Type I A. All of the above Except type III vWF disease. 79 Hematology MCQ’s FOR MCQ’s FOR MEDICAL MEDICAL PROFESSIONALS PROFESSIONALS BY PROF. von Willebrand Disease Willebrand’s disease). isolated dense (  ) granule deficiency. Epinephrine D. Agonists of platelet aggregation are adenosine diphosphate (ADP). 4 mg/L A. afibrinogenemia) or acquired (fibrin degradation product inhibition. With cardiopulmonary bypass B. Bernard-Soulier syndrome A. Patients are usually the offspring of two parents (usually asymptomatic) with mild type I disease.ticlopidine. von Willebrand’s disease (vWD) 942 Most common inherited bleeding disorder is ? B. Hermansky-Pudlak syndrome B. 10 mg/L D. 676 D. ADP B. Erik von Willebrand was ? A. Type III 941 Acquired platelet dysfunction due to uremia. 8 mg/L C. while hemophilia mainly affects males. 946 Which of the following vWD type is most common ? Harrison’s 16th Ed. 676 A. all forms are inherited as autosomal dominant traits. All of the above C. Uremia B. Chediak-Higashi syndrome C. A. vWD affects both males & females. risk of alloimmunization can be limited by using prestorage leukodepleted C. drug induced . AJAY MATHUR Cardiology 79 In platelet transfusion. There is mild to moderate decrease in plasma vWF. Chediak-Higashi syndrome. 676 951 Which of the following is a platelet aggregation agonist ? Harrison’s 16th Ed. Gp IIb/IIIa inhibitors) function of thrombocytes and a general lesion of capillary walls and called it “pseudohemophilia”. Gray-platelet syndrome The normal plasma vWF level is 10 mg/L. Type IIB D. 971 Type III vWF disease is a very severe form of vWD with no detectable vWF antigen or activity. dominant trait? Harrison’s 16th Ed. Dialysis B. Type IIB C. 6 mg/L B. Gray-platelet syndrome or 945 Which of the following vWD is not inherited as autosomal acquired (cardiopulmonary bypass. von C. Finnish C. Type IIA . Uremia Harrison’s 16th Ed. DDAVP 948 Which of the following is not a functional platelet disorder of adhesion ? C. Type III D. or acquired (Uremia.aspirin. 724 954 Which of the following vWD is due to a defect in factor VIII A. vWF D. which of the following is “adhesive 956 vWD may be acquired in which of the following situations ? glycoprotein” ? Harrison’s 16th Ed. serotonin) & vessel wall extracellular matrix constituents in contact with adherent platelets (collagen. which of the following is a feature Lab. Monoclonal gammopathies of undetermined significance A. Hermansky-Pudlak syndrome kb of genomic DNA on chromosome 12. Glanzmann’s thrombasthenia B. a parallel apart from aortic stenosis ? reduction in vWF biologic activity by ristocetin cofactor assay and reduced factor VIII activity. platelets have deficiency or dysfunction of the Gp Ib/IX complex ? A. Raynaud’s phenomenon binding site of vWF ? B. (absence of platelet GpIb-IX-V receptor). All of the above Inherited platelet function qualitative disorders include autosomal recessive disorders like Glanzmann’s thrombasthenia (absence of platelet GpIIbIIIa receptor) & Bernard Soulier syndrome A single large vWF precursor subunit is synthesized in endothelial cells & megakaryocytes. Patients with type I vWD are the best candidates for desmopressin therapy. diagnostic findings in vWD are prolonged BT. 962 In platelet granules. reduction in plasma vWF level. 971 958 In which of the following diseases. Type II Heyde’s syndrome refers to aortic stenosis with GI bleeding due to angiodysplasia of GI tract. 10 957 In which of the following diseases. All of the above B. 363 955 Patients of which type of vWD are the best candidates for A. where it is cleaved & assembled into disulfide-linked multimers in plasma. All of the above D. Type IIB by humoral mediators (epinephrine. Type IIn D. Waldenstrom’s macroglobulinemia B. Bernard-Soulier syndrome A. 959 Which of the following is not an autosomal recessive trait ? Harrison’s 17th Ed. Type 1 Harrison’s 17th Ed. Adenosine diphosphate (ADP) desmopressin therapy ? B. Thrombospondin B. 678 Harrison’s 17th Ed. 11 Harrison’s 17th Ed. D. Bernard-Soulier syndrome D. Gastrointestinal bleeding Harrison’s 16th Ed. Vascular subendothelium D. Type I D. This process is promoted C. N Engl J Med 2004. Thrombin Harrison’s 16th Ed. 677 C.351:683-94 A. Clubbing A. 12 A. Both present with bleeding symptoms in childhood. C.80 MCQ’s FOR MEDICAL PROFESSIONALS BY PROF. 724 A. Chediak-Higashi syndrome 964 Most common type of vWD is ? Harrison’s 18th Ed. 723 953 Which of the following is false in diagnostic pattern of vWD ? Harrison’s 16th Ed. mediators from activated platelets (ADP. thrombin). Bernard-Soulier syndrome C. D. Type IIA Platelet activation & then aggregation occurs after platelet adhesion. platelets. Harrison’s 17th Ed. All of the above Acquired vWD is mostly seen in patients with underlying lymphoproliferative disorders like 963 von Willebrand factor gene is located on which chromosome ? MGUS. Type 2A A. Type 2B B. Platelets C. platelets are deficient or defective in Gp IIb/IIIa complex ? B. Reduced factor VIII activity 960 In Heyde’s syndrome. AJAY MATHUR Hematology B. Glanzmann’s thrombasthenia . Chediak-Higashi syndrome D. 13 B. Glanzmann’s thrombasthenia von Willebrand factor is a large multimeric glycoprotein that is encoded by a gene spanning 178 C. 723 B. von Willebrand’s disease Type II C. von Willebrand’s disease Type III D. Type I D. Prolonged clotting time B. Reduced vWF biologic activity D. Pes cavus B. 723 C. Reduced vWF concentration C. Type III 961 Which of the following platelet aggregation agonists require fibrinogen for binding ? Type IIn vWD is due to a defect in factor VIII binding site of vWF. Epinephrine A. multiple myeloma & Waldenstrom’s macroglobulinemia. 676 A. & subendothelium. Harrison’s 17th Ed. Fibronectin C. Type III vWF). Multiple myeloma C. Hermansky-Pudlak syndrome C. 677 C. 2 and 3. Decreased plasma vWF concentration Patients with “O” blood group have vWF protein levels about one-half those of patients with AB Laboratory findings in vWD are variable. 972 A. 972 B. FVIII has a very short half-life and level is markedly decreased. Type 2 has four subtypes. Monoclonal gammopathies of undetermined significance C. CT. 965 Which of the following vWD is inherited as autosomal recessive trait ? 971 Which of the following best relates to Heyde’s syndrome ? Harrison’s 18th Ed. making it susceptible to serum proteases with loss of large multimer forms. Type 2B N Engl J Med 2004. reduction in its vWF biologic activity (ristocetin cofactor assay) and reduced FVIII activity. Most common diagnostic pattern is prolonged BT. Increased APTT D. Prolonged BT B. Type 2N C. 972 B.351:683-94 C. 967 Which of the following is called “severe vWD” ? Harrison’s 18th Ed. Shear stress on blood recessive trait in homozygous or compound heterozygous persons. passing through stenotic aortic valve produces change in vWF. Acquired vWD is seen most commonly in MGUS. Type 2B In type 2N vWD. Type 3 C. 972 Harrison’s 18th Ed. Uterine relaxant 969 Which type of vWD is termed “autosomal hemophilia” ? D. Harrison’s 18th Ed. Type 3 968 Which type of vWD reflects mutations in vWF that preclude Platelet counts are within the normal range in vWD except in type 2B vWD where mild thrombocytopenia may be seen and may be misdiagnosed as autoimmune thrombocytopenia. leading to acquired type 2 vWD. Multiple myeloma . The most D. Bleeding time is normal in hemophilia A. A. Type 2A Harrison’s 18th Ed.patients with virtually no vWF antigen and FVIII levels C. B B. decreased plasma vWF concentration. Type 2B A. Cirrhosis of liver B. Type 1 D. blood type. Type 2M A. Type 2B <10%. Type 2A D. Type 2A B. Ectopic spleen Heyde’s syndrome refers to aortic stenosis with gastrointestinal bleeding due to presence of Type 3 vWD. normal platelet count. vWF: Ag is normal ? in inherited haemophilia A. 972 970 Acquired vWD is seen most commonly in ? A. 966 vWF protein levels are lowest in which of the following blood groups ? 972 Which of the following is false in von Willebrand Disease ? Harrison’s 18th Ed. O D. 972 C. Pancreatic neoplasis D. Analgesic D. D. Type 2N B. Type 3 D. Type 2N vWD is due to abnormal FVIII binding sites. Type 2M B. AJAY MATHUR Cardiology 81 D. Type 1 A. Type 3 autosomal hemophilia. Normal platelet count C. 972 975 In which of the following vWD. Type 1 C. is transmitted as an autosomal angiodysplasia of gastrointestinal tract in patients with aortic stenosis. vWF: Ag is normal ? A. which accounts for 1 . This is termed D.351:683-94 A. Type 2A Type 3 vWD refers to severe vWD . 81 Hematology MCQ’s FOR MCQ’s FOR MEDICAL MEDICAL PROFESSIONALS PROFESSIONALS BY PROF. 972 types except type 2B vWD where a brisk platelet aggregation is seen even at low concentrations. Type 2N A. A A. It is distinguished from haemophilia A by the finding of normal vWF:Ag levels and limitation of the disorder to males 976 In which of the following vWD. PT. C. Type 2M Harrison’s 18th Ed. 972 N Engl J Med 2004. Digestive enzyme Harrison’s 18th Ed. Type 3 B. Type 1 types of vWD ? B. Waldenstrom’s macroglobulinemia vWD is classified into three major types 1. Type 2B B. Type 2B A. Type 2A 974 What is the nature of “Ristocetin” ? B. AB C. Angiodysplasia of gastrointestinal tract C. binding of FVIII ? RIPA (Ristocetin induced platelet aggregation) shows a low to absent response in all vWD Harrison’s 18th Ed. increased APTT. Type 2B C. Type 2A Harrison’s 18th Ed. Antibiotic Type 2N vWD reflects mutations in vWF that preclude binding of FVIII. None of the above common type of vWD is type 1 disease (80%). 972 973 Platelet count are modestly reduced in which of the following A.5 % percent of cases. Cogan’s syndrome & Kawasaki disease. Neck and upper limbs C. Type 2B Harrison’s 18th Ed. Small vessel vasculitis C. usually decreased in 2B and 2N and nearly to completely absent in type 3 vWD. or desmopressin is not indicated in which type of von Willebrand disease ? D. Type 2A 985 Henoch-Schönlein purpura lesions are most commonly C. Henoch-Schönlein purpura. DDAVP is a synthetic hormone given IV injection or by nasal spray. 3. D. 972.351:683-94 Henoch-Schönlein purpura A. Churg-Strauss syndrome. It has no activity on type 1 receptors. Glaucoma B. B. All of the above Henoch-Schönlein purpura is a small-vessel vasculitis. 30 µg / kg body weight IV infusion 984 Henoch-Schönlein purpura is also referred to as ? Harrison’s 18th Ed. Purple purpura 979 DDAVP. Chest and abdomen 980 DDAVP. 2797 C. Buttocks and lower extremities Type 1 vWD is best treated with DDAVP (desmopressin). Harrison’s 18th Ed. A. 2797 B. Type 2B distributed over ? Harrison’s 18th Ed. Anaphylactoid purpura Desmopressin is given in a dose of 0. Factor VIII–von Willebrand factor concentrates Normal vWF:Ag levels do not exclude vWD. Giant cell arteritis. Recombinant factor VIII type 2A and 2M. which results in release of vWF and FVIII from endothelial stores.0 µg / kg body weight IV infusion C. B. Platelet concentrates 977 Desmopressin acts through which of the following vasopressin receptors ? N Engl J Med 2004. Type 3 A. Emergent purpura which type of von Willebrand disease ? D. C. D.3 µg / kg body weight IV infusion Takayasu’s arteritis. B. Alopecia C. AJAY MATHUR Hematology D. palpable purpura lesions are most commonly distributed over A. 972 A. Medium vessel vasculitis D. Upper respiratory infection 981 Antifibrinolytic amino acids like Aminocaproic acid or D. Essential mixed cryoglobulinemia. Behçet’s syndrome. or desmopressin is most effective for treatment of C. 982 In type 3 vWD. Type 3 A. Type 3 B. Isolated vasculitis of CNS. normal in C. Microscopic polyangiitis. Type 1 buttocks and lower extremities. 972 In Henoch-Schönlein purpura. Henoch-Schönlein purpura D. 300 µg / kg body weight IV infusion A. Mostly in age between 4 . Conjunctivitis thrombocytopenia.82 MCQ’s FOR MEDICAL PROFESSIONALS BY PROF. Wegener’s granulomatosis D. 2797 D. vWF levels are just below normal in type 1. Type 2 983 Which of the following is a Primary Vasculitis Syndrome ? Harrison’s 18th Ed. 2797 A. B. All of the above 978 Dose of Desmopressin is ? Harrison’s 18th Ed. 2797 D. N Engl J Med 2004. 2797 D. 972 A. Type 1 B. which of the following is the treatment of first choice in patients with alloantibodies ? 988 Which of the following is false about Henoch-Schönlein purpura ? N Engl J Med 2004.3 µg / kg body weight IV infusioninfusion for 30 minutes. Type 2A 986 Henoch-Schönlein purpura is often preceded by ? C. Trauma Desmopressin or DDAVPis not recommended in type 2B where it is known to aggravate B. Type 1 Henoch-Schönlein purpura is also referred to as anaphylactoid purpura.351:683-94 Harrison’s 18th Ed. Face Harrison’s 18th Ed. Type 2M A. Exercise tranexamic acid used in treatment of vWD can cause which Henoch-Schönlein purpura is often preceded by an upper respiratory infection (streptococcal of the following complications ? pharyngitis) or is triggered by drug or food allergies.351:683-94 Primary vasculitis syndromes include Wegener’s granulomatosis.7 years . Large vessel vasculitis Antifibrinolytic amino acids are contraindicated in patients with upper UTI or gross hematuria because clots that do not lyse may cause ureteral obstruction. C. Sympathetic purpura Harrison’s 18th Ed. Polyarteritis nodosa. Ureteral obstruction A. Cholelithiasis 987 Henoch-Schönlein purpura is a ? Harrison’s 18th Ed. Type 4 B. Desmopressin A. Churg-Strauss syndrome Desmopressin is a synthetic derivative of antidiuretic hormone that acts through type 2 vasopressin receptors. 0. B. Idiopathic cutaneous vasculitis. with RBC casts occurs in the majority. Peak incidence in spring C. Rufus Stuart HSP is a subtype of acute LCV.Stephen Christmas C. immune deposits may be found D. AJAY MATHUR Cardiology 83 B. Usually resolves spontaneously without therapy B. mesangial tissues. Necrotizing vasculitis C. FVI A.Stephen Christmas D. An American group identified the same factor in Mr. Colicky abdominal pain .Stephen Christmas A. 992 Which of the following statements about renal involvement in Henoch-Schönlein purpura is false ? 998 Coagulation-related protein prekallikrein is also called ? Harrison’s 18th Ed. IgG A. Rarely. Systemic vasculitis D. Male-to-female ratio is 1.5:1. Progressive glomerulonephritis rare C. He died due to transfusion- related AIDS. 993 Which of the following statements about renal involvement in Henoch-Schönlein purpura is false ? 999 Factor IX or Christmas Factor is named after ? Harrison’s 18th Ed. 2797 997 Factor X or Stuart-Prower factor is named after ? A. Rufus Stuart D. IgG D. 2797 B. 2797 B. Audrey Prower of London.Ms. which of the following is the most B. Patients . Male-to-female ratio is 1. IgM 996 In Henoch-Schönlein purpura. Progressive glomerulonephritis frequent Fletcher Factor and Fitzgerald Factor are coagulation-related proteins prekallikrein and high- molecular-weight kininogen respectively. Gastrointestinal involvement capillaries.Ms. 2797 A. Bowel intussusception D. and small arterioles. 2797 A. D. IgA levels are elevated in ~half of patients. C3 B. 1000 An isolated abnormal prothrombin time (PT) suggests deficiency of ? 994 Which of the following statements about gastrointestinal Harrison’s 18th Ed. Rufus Stuart B. MacFarlane Factor C. the antibody class most often seen in the immune complexes is ? A.7 years of age. 973 involvement in Henoch-Schönlein purpura is false ? A. IgA C. Rufus Stuart (1957). Coagulation Disorders 991 Henoch-Schönlein purpura is best related to ? Harrison’s 18th Ed.Stephen Christmas B. who had life- plus intravascular thrombosis. Mild glomerulonephritis. a progressive glomerulonephritis will develop. Livedoid vasculitis B. All coagulation tests are normal. Palpable purpura HSP patients have an acute inflammatory reaction with IgA and complement components in B. Red blood cell casts D. None of the above Stephen Christmas (1947-1993) of Canada sufferd from hemophilia B.Ms. None of the above D. Renal involvement occurs in 10 . Audrey Prower & Mr. Proteinuria B. Peak incidence in spring has been noted. Small arterioles Harrison’s 18th Ed. FV Harrison’s 18th Ed. 2797 biopsies of HSP patients. Furie Factor D. Scientists . Livedoid vasculitis represents a combination of a vasculopathy Factor X. Doctors . Patient . Renal failure is the most common cause of death D. Rufus Stuart C. Chapter 116. microscopic hematuria.Ms.50% of HSP patients. Microscopic hematuria C. 2797 D. Leukocytoclastic vasculitis (LCV) A. All of the above A. Politician . palpable purpura is seen in virtually all patients. A.5:1 B. Audrey Prower & Mr. it resolves spontaneously without therapy. or Stuart-Prower factor was identified in Ms. Blood and mucus per rectum C. Glomerulonephritis and localized hemorrhage. long bleeding tendency. Arthritis In pediatric HSP. Doctor . IgE in ? IgA is the antibody class most often seen in immune complexes and demonstrated in renal Harrison’s 18th Ed. None of the above Most patients of HSP are between 4 . Fitzgerald Factor B. Politicians . Fletcher Factor A. Audrey Prower & Mr. 83 Hematology MCQ’s FOR MCQ’s FOR MEDICAL MEDICAL PROFESSIONALS PROFESSIONALS BY PROF. Peripheral glomerular capillary wall common presentation in HSP ? C. leading to increased vascular permeability C. Mesangial and paramesangial areas 990 In pediatric patients. mesangial immune deposits consists of ? Harrison’s 18th Ed. 2797 989 In Henoch-Schönlein purpura. 995 In Henoch-Schönlein purpura. All of the above C. Audrey Prower & Mr. proteinuria. IgA Harrison’s 18th Ed. Scientist . FX deficiency D. fibrinogen.361:1887-94 A. FVIII deficiency C. Deficiency of factor VII C. Inhibitor of factor VII. fibrinogen. FII N Engl J Med 2009. fibrinogen. All of the above . IX. FII deficiency A. A. Disseminated intravascular coagulation D. or factor X. FII or fibrinogen abnormalities. IX. 973 factor V. Hemorrhagic diathesis of liver disease 1010 World Hemophilia Day is observed on ? B. All of the above C. FV deficiency B.00000 males worldwide B.344:1773 B. or XI. April 17 C. Inhibitor of prothrombin. Prussian. D. Liver disease and Warfarin therapy could lead to a prolonged PT and normal aPTT. 973 prolonged PT ? N Engl J Med 2009. June 17 Acquired deficiencies of plasma coagulation factors are more frequent than congenital disorders. May 17 D. Inversion of intron 25 DNA sequence C. Inversion of intron 23 DNA sequence N Engl J Med 2009. AJAY MATHUR Hematology C. factor V. Vitamin K deficiency. 973 carrier of hemophilia ? A. in all ethnic groups.361:1887-94 D. FVIII 1001 Prolonged activated partial thromboplastin time indicates ? 1007 Which of the following could lead to a prolonged aPTT and Harrison’s 18th Ed. a clinically normal carrier of hemophilia. Unfractionated heparin Hemophilia is an X-linked recessive hemorrhagic disease due to mutations in the F8 gene (hemophilia A or classic hemophilia) or F9 gene (hemophilia B). Harrison’s 18th Ed. Von Willebrand’s disease D. 974 1006 Which of the following could lead to a prolonged PT and A. Unfractionated heparin and Direct thrombin inhibitors cause a prolonged aPTT and normal PT. Patients with hemophilia have normal bleeding times & platelet counts. Leopold.000000 males worldwide C. PT N Engl J Med 2001. A prolonged activated B. Qualitative platelet disorders partial thromboplastin time (aPTT) indicates most commonly hemophilia or FXI deficiency. aPTT A. or XI C. FX 1008 Which of the following is a disorder of primary hemostasis ? C. disseminated D. Inhibitor of factor VIII. Thrombin time (TT) B. Supratherapeutic doses of heparin or warfarin. All of the above C. Most common disorders include hemorrhagic diathesis of liver disease. Liver disease. The C. transmitted Harrison’s 18th Ed. FV B. Disseminated intravascular coagulation and Argatroban could lead to a prolonged aPTT and prolonged PT A. von Willebrand’s disease prolongation of both PT & aPTT suggests deficiency of FV.000 males worldwide normal PT ? N Engl J Med 2009. 1 in 10. Vitamin K deficiency D. FX. Queen Harper 1004 Which of the following is an example of acquired Queen Victoria. factor V or X B. Supratherapeutic doses of heparin or warfarin C. Von Willebrand’s disease. who had deficiencies of plasma coagulation ? hemophilia and two daughters. 973 the disease to the Russian. Blood coagulation is hampered by the deficiency of more than one clotting factor. 1 in 10. or factor X. 1 in 100.361:1887-94 B. who were carriers and who. Queen Elizabeth I D.000 males worldwide A. All of the above males worldwide. IX. had one son. D. 1 in 10. Alice and Beatrice. Thrombocytopenia An isolated abnormal prothrombin time (PT) suggests FVII deficiency. FVII Deficiency of factor VII. July 17 intravascular coagulation (DIC). 1012 Most common hemophilia A F8 mutations results from ? Harrison’s 18th Ed. or XI.84 MCQ’s FOR MEDICAL PROFESSIONALS BY PROF. Deficiency of factor VIII. and vitamin K deficiency. All of the above A. Deficiency of prothrombin. The disease affects 1 in 10. Inversion of intron 24 DNA sequence B. Vitamin K deficiency B. Warfarin therapy 40 to 50 percent of the mutations causing severe hemophilia A are due to an inversion of DNA sequences in intron 22 that disruptes factor VIII gene. 1011 What is the prevalence of hemophilia ? Harrison’s 18th Ed. All of the above 1003 Which of the following is a test for blood coagulation ? 1009 Which of the following royal persons was a clinically normal Harrison’s 18th Ed. Deficiency of factor VIII. Queen Marie C. D. Queen Victoria D. Disseminated intravascular coagulation (DIC) A. All of the above 1002 Prolongation of both PT & aPTT suggests deficiency of ? Deficiency of prothrombin.000 D. Inversion of intron 22 DNA sequence normal aPTT ? B. in turn. and Spanish royal families. 974 1005 Which of the following could lead to a prolonged aPTT and A.361:1887-94 A. PT mixed with normal plasma 1016 One unit of factor VIII is defined as the amount of factor VIII present in ? B. Patients with type I inhibitor should not receive factor VIII adequate hemostasis is ? Harrison’s 18th Ed. 4 to 8 hours D. 682 Family history of the disease is absent in ~30% of cases and arise from a spontaneous mutation. 100 mL normal plasma is abnormally prolonged. 1 to 4 hours C. < 3% drugs such as e-aminocaproic acid (EACA) or tranexamic acid enhance local hemostasis. Harrison’s 18th Ed. 85 Hematology MCQ’s FOR MCQ’s FOR MEDICAL MEDICAL PROFESSIONALS PROFESSIONALS BY PROF. < 4% 1022 Laboratory test required to confirm presence of an inhibitor When residual activity of FVIII or FIX is < 1%. D. 5% D. who carry a single mutated gene. in patients with mild to moderate hemophilia A but not in in severe hemophilia A as there B. A. In inhibitor patients. ~30 % D. Harrison’s 17th Ed. ~10 % B. Symptomatic patients have factor VIII levels of < 5 % C. A. 976 A. 680 Harrison’s 18th Ed. 20 A. alloantibodies to FVIII is ? 5%. Severe iron-deficiency anemia is uncommon 1014 The minimal level of most clotting factors needed for B. FVIII half-life of 8 to 12 hours. 40 C. Desmopressin (DDAVP) B. hemophilia is classified as severe. Athens assay C. as inhibitor neutralizes FVIII clotting activity of normal plasma. Between 1 . . 1000 mL normal plasma 1023 Alloantibody inhibitor formation to FVIII occurs after how One unit is by definition the amount of FVIII (100 ng/mL) or FIX (5 g/mL) in 1 mL of normal plasma. 10 mL normal plasma Laboratory test for confirming presence of an inhibitor is aPTT mixed with normal plasma. 12 to 16 hours a prevalence of 5-10%. All of the above A. 1024 Which of the following assay is used to define the specificity of the inhibitor and its titer ? 1018 Which of the following is false about antihemophilic factor Harrison’s 18th Ed. 976 Those with residual FVIII or FIX activity >25% of normal. Bethesda assay A. a 1:1 mix with normal plasma corrects aPTT. Prenatal diagnosis from chorionic villus biopsy or amniocentesis possible A. None of the above D. but not FIX. aPTT mixed with normal plasma A. 30 B. 1 mL normal plasma D. therefore requires twice a day administration to maintain therapeutic levels. < 2% are no stores to release. aPTT mixed with patients plasma B. ~40 % 1020 Which of the following is false about hemophilia A ? Hemophilia affects males. Oral antifibrinolytic C. Minnesota assay Harrison’s 17th Ed. 976 1017 Factor VIII has a half-life of ? A. Gene for factor VIII is on the X chromosome B. hemophilia is discovered only by bleeding after major trauma or during presurgery laboratory tests. ~20 % C. One Bethesda unit (BU) is D. < 1% DDAVP is a synthetic vasopressin analogue that causes a transient rise in FVIII and vWF. are generally asymptomatic. Circulates complexed to vWF protein Bethesda assay defines the specificity of the inhibitor and its titer. 8 to 12 hours Formation of alloantibodies to FVIII or FIX is a major complication of hemophilia treatment with D. Repeated dosing of DDAVP results in tachyphylaxis.  aminocaproic acid (EACA) 1015 Clinically. D. 974 D. 974 C. 10 % C. Between 6 . it is moderate. hemophilia is classified as severe when residual C. None of the above B. Half-life of FIX is about 24 hours. One unit of FVIII per kilogram of body weight increases the plasma FVIII level by 2%. 974 A. many cumulative days of exposure ? Harrison’s 18th Ed. 25 % bleeding in hemophilia patients ? Harrison’s 18th Ed. Regulates activation of factor X A. Tranexamic acid activity of FVIII is ? Harrison’s 18th Ed. 976 (AHF) ? A. Inhibitors appear early in life & after 10 cumulative days of exposure. None of the above the amount of antibody that neutralizes 50% of FVIII or FIX present in normal plasma after 2 hours of incubation at 37°C. 975 C.30%. In hemophilia. 10 Harrison’s 18th Ed. Glasgow assay B. 680 B. Synthesized in liver D. Multiple-chain protein C. 15 % 1021 Which of the following medicines is useful in managing D. PT mixed with patients plasma Harrison’s 18th Ed. aPTT on a 1:1 mix C.. AJAY MATHUR Cardiology 85 1013 Family history of hemophilia is absent in what percentage 1019 Which of the following is false about antihemophilic factor (AHF) ? of cases of hemophilia ? Harrison’s 17th Ed. 975 B. women. it is mild. Factor IIa levels of tissue factor. and elevated levels of fibrin degradation products (FDPs). All of the above D. Anti-CD20 monoclonal antibody (rituximab) D. Elevated levels of FDP Harrison’s 18th Ed. Hepatitis E virus A. FIX. Factor VII B. Christmas disease 1027 Which of the following is the major cause of morbidity and C. B. Synthesized in liver Findings in DIC include the prolongation of PT and/or aPTT. Hydroxide B. C. Together these abnormalities contribute to B. AJAY MATHUR Hematology 1025 Control of bleeding episodes in high-responder inhibitor 1031 Vitamin K deficiency is due to ? patients can be achieved by ? Harrison’s 16th Ed. Immunosuppression B.000/mm 3 . intestinal malabsorption. 976 Purpura fulminans is a severe form of DIC resulting from thrombosis of extensive areas of skin following viral or bacterial infection. Prolongation of PT and/or aPTT coagulation that activates FIX to FIXa. B. Uncontrolled generation of thrombin Hepatitis C virus (HCV) infection is the major cause of morbidity and the second leading cause B. FX (prothrombin complex concentrates). 979 Factor XI is a zymogen of an active serine protease (FXIa) in the intrinsic pathway of blood A. or the Following absorption. C. FVII. Inadequate dietary intake A. a cofactor in enzymatic carboxylation of glutamic acid residues on prothrombin complex proteins. . Hepatitis B virus 1034 Which of the following is the mechanism of DIC ? C. Immune tolerance induction (ITI) C. but not fibrinogen degradation products. metastatic carcinoma. None of the above 1033 Purpura fulminans is related best to ? Harrison’s 18th Ed. Abnormal fibrinolysis 1028 Factor IX is activated to IXa by ? D. Requires vitamin K for biologic activity presence of schistocytes (fragmented RBCs) in PBF. Addition of anti-CD20 monoclonal antibody (rituximab) A. It is more common in those with inherited A. All of the above Three major causes of vitamin K deficiency are inadequate dietary intake. None of the above 1036 Chronic DIC can occur in ? 1030 In liver. D-dimer test is more C. Sulphide C. Platelet counts <100. Loss of storage in liver C. Recombinant activated Factor VII (FVIIa) D. 977 DIC is the result of uncontrolled generation of thrombin by exposure of blood to pathologic A. Simultaneous suppression of physiologic anticoagulant mechanisms and abnormal fibrinolysis further accelerate the process. Factor II Harrison’s 18th Ed. Factor IX gene is on X chromosome specific for detection of fibrin. Control of bleeding is achieved by using concentrates enriched for prothrombin. Presence of schistocytes (fragmented red cells) in PBF A. Prothrombin complex concentrates (PCCs) B. Factor Va systemic fibrin deposition in small and mid-sized vessels. Factor X D. All of the above Harrison’s 18th Ed. Most effective strategy is immune tolerance induction (ITI). 979 Harrison’s 18th Ed. 976 A. Hepatitis A virus or acquired hypercoagulability due to deficiencies of components of protein C pathway.86 MCQ’s FOR MEDICAL PROFESSIONALS BY PROF. Suppression of physiologic anticoagulant mechanisms of death in hemophilia patients exposed to older clotting factor concentrates. 978 A. The most sensitive test for DIC is the FDP level. B. compensated DIC can occur in giant hemangioma. Giant hemangioma A. D. Vitamin K deficiency exposed to older clotting factor concentrates ? Harrison’s 18th Ed. High-responder inhibitor patients are those with initial inhibitor titer of >10 BU & do not respond to and loss of storage sites due to hepatocellular disease. FVIII or FIX concentrates. Metastatic carcinoma B. B. more in children. recombinant activated Factor VII. Intestinal malabsorption B. Activated prothrombin complex concentrates (aPCCs) C. 683 Harrison’s 18th Ed. Epoxide Low-grade. Vitamin K is converted into its active form called ? Harrison’s 18th Ed. All of the above D. 1032 Which of the following clotting factors has the shortest half- life ? 1026 Which of the following is not effective for eradication of the Harrison’s 18th Ed. Factor IX C.000/mm3 1029 Which of the following is false about Factor IX ? C. 977 D. immunosuppression is not effective. Butyrate D. platelet counts <100. Hemophilia as a coadjuvant to ITI is beneficial. 979 For eradication of the inhibitory antibody. Factor Xa Harrison’s 18th Ed. Dead fetus syndrome C. Factor XIa 1035 The most sensitive test for DIC is ? D. Disseminated intravascular coagulation (DIC) second leading cause of death in hemophilia patients D. 980 inhibitory antibody in hemophiliacs ? A. 979 D. vitamin K is converted to an active epoxide in liver microsomes & serves as dead fetus syndrome. Hepatitis C virus Harrison’s 18th Ed. 976 A. Congenital homocystinuria syndrome patients have 1041 Which of the following predict more bleeding in DIC ? Marfanoid habitus Harrison’s 16th Ed. schistocytes or fragmented RBC’s. 980 A. Procoagulant consumption phase D. All of the above D. Normal levels of plasma FDP or D-dimers D. which further aggravates the disease. Protein S C. Schistocytes C. None of the above B. Protein C B. Prolonged PT 1039 Laboratory findings in DIC include ? B. Schistocytes D. Platelet concentrates A. 980 A. Positive D dimer immunoassay 1047 Which of the following is false about ‘Hyperhomocysteinemia’ ? D. Purpura fulminans 1038 Which of the following is the first phase in DIC ? B. PT. Reduced fibrinogen level C. Normal platelet counts is a common finding. Lack nucleus C. elevated fibrin degradation products thromboembolism (FDP) and positive D dimer immunoassay. Antithrombin III A. plasma levels of FDP or D-dimers are elevated. Disc-shaped B. D. prolonged PT. Harrison’s 18th Ed. None of the above Coagulation inhibitors include protein C. Thrombotic phase C. Reduced fibrinogen level Arterial and Venous Thrombosis Low fibrinogen levels in DIC predict more bleeding. Prolonged thrombin time 1046 Bleeding in liver disease is best managed with ? D. Cryoprecipitate C. Thrombospondin . 684 Table 102-2 Harrison’s 18th Ed. During removal of a dead fetus Harrison’s 16th Ed. AJAY MATHUR Cardiology 87 1037 Findings evident in chronic DIC are all except ? C. Vitronectin A. Prolonged PTT Harrison’s 16th Ed. reduced fibrinogen level. Average lifespan of 7 to 10 days D. All of the above C. 1042 Inhibitor of coagulation is ? 1048 Which of the following about platelets is false ? Harrison’s 16th Ed. and in patients with low-grade DIC due to ? fibrinogen values are within normal range or high. 1049 Which of the following subendothelial components trigger platelet reactivity ? 1043 Which of the following has no role in the management of Harrison’s 18th Ed. Predisposes to the risk of venous & arterial Laboratory features of DIC include thrombocytopenia. Normal aPTT limited efficacy afforded by replacement of single factors and high risk of products containing traces of activated blood proteases (PCCs). Secondary fibrinolysis phase Heparin is indicated for the treatment of purpura fulminans. Normal platelet count Clotting factor concentrates are not recommended for control of bleeding in DIC because of the B. 983 hemorrhagic symptoms of DIC ? Harrison’s 18th Ed. Fresh-frozen plasma 1040 Laboratory findings in DIC include ? B. FFP B. Harrison’s 18th Ed. None of the above The early thrombotic phase of DIC is followed by a phase of procoagulant consumption and 1045 Laboratory findings in patients with liver disease include all except ? secondary fibrinolysis. PTT & thrombin time. Fibronectin B. Anticoagulation with heparin C. during the surgical resection of giant hemangiomas. All of the above Harrison’s 17th Ed. Prothrombin complex concentrates Harrison’s 16th Ed. 684 C. protein S and antithrombin III. Clotting factor concentrates Harrison’s 18th Ed. 979 D. All of the above Harrison’s 18th Ed. aPTT. D. C. 980 A. Low fibrinogen level C. Prolonged thrombin time Chapter 117. 684 A. 685 A. Normal fibrinogen 1044 Low doses of continuous heparin infusion may be effective In chronic compensated DIC. Mild thrombocytopenia B. and during removal of a dead fetus. Thrombocytopenia C. 684 A. Elevated fibrin degradation products (FDP) D. Thrombocytopenia D. Fibrinogen concentrates B. 87 Hematology MCQ’s FOR MCQ’s FOR MEDICAL MEDICAL PROFESSIONALS PROFESSIONALS BY PROF. During surgical resection of giant hemangiomas B. 980 A. 683 A. B. 983 A. Vitamin B12 deficiency produces high homocysteine levels A. Endothelial receptor 1059 Aspirin can produce antithrombotic effect at a dose of ? Harrison’s 16th Ed. At high doses (~1000 mg/day). B. Antiplatelet. 687 1052 P2X1. 984 B. P2Y1. aspirin also inhibits COX-2. 687 D. Thromboxane A2 can be produced in monocytes and C. 20 mg/day Activated platelets adhere to circulating leukocytes. and integrin 2 1. AJAY MATHUR Hematology D. and Fibrinolytic Drugs Harrison’s 18th Ed. All of the above Exposed subendothelial components that trigger platelet reactivity favouring adhesion include Circulating platelets are maintained in an inactive state by nitric oxide (NO) and prostacyclin collagen. P2Y 1 . Integrin  2 1 C. They include the ADP receptors. Thienopyridine derivatives. Receptors found on platelets regulate their functions. Activating platelets B. Endothelial cells also express CD39 on their surface. Chemokine receptors ment while drugs that inhibit thrombin & fibrin generation play a primary role in venous disease.88 MCQ’s FOR MEDICAL PROFESSIONALS BY PROF. GP IV A. GPIb-IX-V B. prostaglandin receptors. 983 Thrombosis Research 2007. All of the above adhesion. None of the above The platelet GPIb-IX-V complex adhesive receptor is central both to platelet adhesion and to the initiation of platelet activation. All of the above D. ADP receptors C. That inhibit thrombin generation A. ADP receptors are classified 1058 At what dose. 1050 Which of the following expressed on platelet surface regulate collagen-induced platelet adhesion ? 1055 Thromboxane A2 (TxA2) plays a role in ? Harrison’s 18th Ed. GP VI B. GPIIb-IX-VII B. Nitric oxide (NO) D. 989 induced platelet aggregation. and chemokine receptors. 30 mg/day (PSGL-1). Promoting atherogenesis by inducing proliferation of D. and P2Y 12. Protease-activated receptor-1 (PAR-1) A. vitronectin and thrombospondin. That inhibit platelet activation & aggregation Harrison’s 18th Ed. 985 D. GPIb-IX-V macrophages D. von Willebrand factor. 1060 The major thrombin receptor on human platelets is ? Anticoagulant. clopidogrel & prasugrel. GPIb-IX-V complex binds to the exposed von Willebrand 1057 Which of the following drugs is used in arterial disease factor. C. In mature human platelets. That inhibit fibrin generation B. ~ 750 mg / day ? Harrison’s 18th Ed. Integrin  2 1 Harrison’s 18th Ed. Platelet surface receptor Aspirin produces its antithrombotic effect by irreversibly acetylating and inhibiting platelet cyclooxygenase COX-1. 988 C. ~ 325 mg / day B. Glycoprotein (GP) IIb/IIIa 1054 Circulating platelets are maintained in an inactive state by ? B. Lipid receptors Drugs that inhibit platelet activation & aggregation play a primary role in arterial disease manage- D. Activation of both P2Y 12 & P2Y 1 receptors is essential for ADP. management ? Harrison’s 16th Ed. Prostacyclin Protease-activated receptor-1 (PAR-1) is the major thrombin receptor on human platelets. Harrison’s 18th Ed. Leukocyte receptor C. 50 mg/day Chapter 118. GPVI. are clinically utilized inhibitors of ADP-induced platelet aggregation. All of the above vascular smooth muscle cells Certain proteins are expressed on the platelet surface that regulate collagen-induced platelet D. This association leads to increased expression of CD11b/CD18 (Mac-1) on leukocytes. a membrane-associated ecto-adenosine diphosphatase (ADPase) that degrades ADP released from activated platelets. All of the above C.120: 337–346 A. causing platelets to adhere. CD39 . 367: 606–17 initiation of platelet activation ? A. lipid receptors. aspirin also inhibits COX-2 ? as P2X 1 . These include glycoprotein (GP) IV. 1056 Which of the following statements is false ? 1051 Which of these is central to platelet adhesion and to the Lancet 2006. ~ 500 mg / day 1053 P-selectin glycoprotein ligand 1 (PSGL-1) best relates with C. A. Constricting blood vessels C. 983 COX A. Platelets bind via P-selectin (CD62P) expressed on the surface of activated platelets to the leukocyte receptor. released by endothelial cells lining the blood vessels. C. fibronectin. All of the above A. GPIIb-IX-V D. Prostaglandin receptors D. COX-2 are absent B. 989 Figure 118-3 A. NSAIDs (ibuprofen/diclofenac) are reversible inhibitors of Harrison’s 18th Ed. 40 mg/day D. GPIb-IX-VII C. vWF–bound GPIb-IX-V transforms the GPIIb/IIIa receptor from an inactive low-affinity state to an active high-affinity receptor for fibrinogen. and P2Y 12 are which variety of receptors ? A. P-selectin glycoprotein ligand 1 B. ~ 1000 mg / day A. SR 28334 Thienopyridines (Ticlopidine. 991 C.100 mg B. 8. clopidogrel.75 mg A. Aspirin B. and stroke by 8. P2X 21 C.5 % receptor ? B. Gp Ib D. . All of the above On platelets. Glycoprotein IIb-IIIa When compared with aspirin in patients with recent ischemic stroke. Ticlopidine 1072 Which of the following is a GPIIb/IIIA receptor antagonist ? B. 989 Figure 118-3 A. or stroke. Clopidogrel B. Glycoprotein Ia-IIa is an active receptor for collagen. 13. 25 % C. P2Y21 D. SR 26334 C. All of the above Daily aspirin doses of 75 . Glycoprotein Ib-V-IX C. MI. in the risk of cardiovascular death. 5% A. or stroke by ? Harrison’s 18th Ed. a key ADP receptor on the platelet surface. Ticlopidine and clopidogrel C. CYP2C20*2 1065 Compared with aspirin. Dipyridamole Harrison’s 18th Ed. or stroke by ? of platelet activation ? Harrison’s 18th Ed. or peripheral arterial disease. recommended daily dose of aspirin is ? Harrison’s 18th Ed. Aspirin B. 989 Harrison’s 18th Ed. Clopidogrel was compared to aspirin in CAPRIE trial for effect on ischemic events in patients with recent stroke or MI and in those with symptomatic peripheral arterial disease. 1062 For most indications. CYP2C18*2 C. SR 25334 B.7%. Cyclooxygenase (COX) B. clopidogrel reduced the risk of cardiovascular death. aspirin led to a 25% reduction in risk of CV death. MI. 1064 Which of the following is a prodrug ? 1070 Subjects with which of the following allele exhibit decreased Harrison’s 18th Ed. AJAY MATHUR Cardiology 89 1061 Compared with placebo. A. MI. 991 A. thrombocytopenia.2 % B. 688 A. 688 A. 75 . clopidogrel produces a reduction Subjects with loss-of-function CYP2C19*2 allele exhibit decreased responsiveness to clopidogrel. 2. Tirofiban A. 989 Figure 118-3 Harrison’s 16th Ed. 989 1068 Which drug was studied in the ‘CAPRIE trial’ ? Harrison’s 16th Ed. Tirofiban and thrombotic thrombocytopenic purpura usually occur within the first few months of starting treatment. All of the above Parenteral GPIIb/IIIa receptor antagonists are abciximab. 35 % D.355:1531 C. 40 . 100 . Neutropenia. Clopidogrel inhibits ADP-induced platelet aggregation. 15 % B. Its main metabolite is SR 26334.300 mg D. Prasugrel C. Clopidogrel rarely precipitates TTP. and prasugrel) irreversibly block P2Y 12.100 mg are recommended for most indications. CYP2C17*2 B. Glycoprotein Ia-IIa D. P2X 12 A. MI. CYP2C19*2 D. 990 1071 Which of the following is a platelet membrane glycoprotein A.7 % The Lancet 2000. 1063 Ticlopidine.Glycoprotein Ib-V-IX is an 1066 Which of the following drugs is known to produce thrombotic active receptor for insoluble von Willebrand factor while the most abundant surface protein thrombocytopenic purpura (TTP) ? Glycoprotein IIb-IIIa requires conformational change during platelet activation to express receptor Harrison’s 18th Ed.150 mg C. Abciximab D. Expression of functionally active GpIIb/IIIa on platelet surfaces is the final common pathway of platelet activation regardless of initial stimulus.5 % A. 150 . 18. Gp IIb/IIIa Compared with placebo. D. Adenosine diphosphate (ADP) receptors C. D. eptifibatide. Clopidogrel D. SR 27334 D. C. clopidogrel. P2Y12 B. Ticlopidine A. 990 function. and prasugrel irreversibly block which of the following ? 1069 The main metabolite of Clopidogrel is ? Harrison’s 18th Ed. mainly for fibrinogen. and tirofiban. Cangrelor and ticagrelor are reversible inhibitors of P2Y 12. All of the above D. Eptifibatide Side effects of ticlopidine are gastrointestinal and hematologic. aspirin produces a reduction in 1067 Which of the following represents the final common pathway the risk of cardiovascular death. 89 Hematology MCQ’s FOR MCQ’s FOR MEDICAL MEDICAL PROFESSIONALS PROFESSIONALS BY PROF. 990 responsiveness to clopidogrel ? Harrison’s 18th Ed. MI. It is a member of serine protease inhibitor All of the GPIIb/IIIa antagonists are given as an IV bolus followed by an infusion. About 80. Heparin is a sulfated polysaccharide and is isolated from mammalian tissues rich in mast cells. Glycoprotein Ib-V-IX D. Porcine bone marrow Abciximab. Eptifibatide C. Bovine liver C. 992 A. Antithrombin III is mandatory for its action. Decasaccharide Abciximab is a Fab fragment of a humanized murine monoclonal antibody that binds to the activated GPIIb/IIIa receptor with high affinity and blocks the binding of adhesive molecules Heparin acts as an anticoagulant exclusively by activating antithrombin III. 992 A. Leucopenia B. WBC Parenteral anticoagulants include heparin. Tirofiban C. Porcine liver D. thrombin inhibitor. 1081 Commercial unfractionated heparin (UFH) is obtained from 1075 Deficiency of which of the following results in Glanzmann’s which of the following sources ? thrombasthenia ? Harrison’s 16th Ed. Dabigatran etexilate C. Bovine bone marrow D. GPIIb/IIIa is inactive on resting platelets D. Bovine lung A. 688 The Lancet 2000.355:1531 A. Glycine protease inhibitor C. it binds to von Willebrand factor. Per platelet. and rivaroxaban is an oral Factor Xa inhibitor. 992 Harrison’s 18th Ed.000 per platelet C.all target the GPIIb/IIIa receptor. Fondaparinux D. the number of GP Ia-IIa receptors is 900–2300 molecules.355:1533 A. 991 A. (serpin) superfamily. Tirofiban D. All of the above D. fibronectin. Serine protease inhibitor D. Most commercial heparin is derived from porcine intestinal mucosa and is a polymer of alternating D-glucuronic acid and N-acetyl-d-glucosamine residues. Heparin Harrison’s 18th Ed. low-molecular-weight heparin (LMWH). and fondaparinux. Cystine protease inhibitor B. Hexasaccharide C. Porcine lung B. RBC C. Glycoprotein IIb-IIIa B.90 MCQ’s FOR MEDICAL PROFESSIONALS BY PROF. 991. All of the above Antithrombin is a plasma cofactor for heparin. All of the above D. AJAY MATHUR Hematology 1073 Which of the following is false about GPIIb/IIIa adhesion 1079 Most serious complication of GPIIb/IIIa antagonist therapy is ? receptors ? Harrison’s 18th Ed. GPIIb/IIIa binds which of the following ? A. 992 Harrison’s 18th Ed. 1082 Commercial unfractionated heparin (UFH) is obtained from 1076 Which of the following target the GPIIb / IIIa receptor ? which of the following sources ? Harrison’s 18th Ed. Tirofiban C. Thrombin B. GP Ib-V-IX receptors is 25000 molecules. 992 1074 Once activated. GP IIb-IIIa receptors is 80000 molecules. 1077 Which of the following is a Fab fragment of a humanized murine monoclonal antibody ? 1083 Which unique sequence of UFH molecule binds to antithrombin ? Harrison’s 18th Ed. Presence of like fibrinogen. vitronectin and and thrombospondin. 992 A. Heptasaccharide D. Patients with Glanzmann’s thrombasthenia have undetectable platelet glycoprotein IIb-IIIa. 1080 Which of the following can be administered orally ? Harrison’s 18th Ed. Ornithine protease inhibitor A. Abciximab B. Fibrinogen D. 991 Harrison’s 18th Ed. eptifibatide and tirofiban . Glycoprotein Ia-IIa C. GPIIb/IIIa binds fibrinogen. Abciximab A. Eptifibatide B. Bovine intestinal mucosa B. Thrombocytopenia C. . None of the above Immune-mediated thrombocytopenia is the most serious complication of GPIIb/IIIa therapy. 992 Harrison’s 18th Ed. Also. Pentasaccharide B. Eptifibatide B. Heparin binds to Antithrombin III via a unique pentasaccharide sequence that is found on one-third of the chains of commercial heparin. Low-molecular-weight heparin (LMWH) B. Anemia A. 1078 Which of the following GPIIb/IIIa antagonists are given as 1084 Antithrombin is a member of ? an IV bolus followed by an infusion ? Harrison’s 18th Ed. Found on surface of platelets & megakaryocytes B. All of the above D. The Lancet 2000. All of the above Commercial UFH is obtained from bovine lung or porcine intestinal mucosa. Dabigatran etexilate is an oral Once activated. Only available oral anticoagulant is warfarin. Abciximab A. Porcine intestinal mucosa C. 2500 . 20. Exosite 1 Harrison’s 18th Ed. Factor II D. RBC’s B. after IV bolus of 70 units/ LMWH have a mean molecular weight of 4500 . 993 1089 What is the mean molecular weight of LMWH ? A.5000. At least 19 saccharide units. Factor Xa–dependent D. At least 17 saccharide units. heparin induces a conformational change in reactive center loop Plasma half-life of heparin ranges from 30 . 4500 . Clearance is mainly renal A. Anti-factor Xa levels is used to monitor heparin therapy C. 993 B. kg. D. 60 minutes B. antithrombin. acute-phase reactants.353:1028-40. Platelet factor 4 (PF4) D.000 fibrin. heparin binds to ? Harrison’s 18th Ed. 1086 UFH-antithrombin complex inactivates which of the following ? 1092 Plasma half-life of heparin given as IV bolus (100 U/kg) is ? Harrison’s 18th Ed. 15. of antithrombin that makes it readily accessible to its target proteases. 5000 . This conformational change respectively. Exosite 2 serves as the heparin-binding domain.30 units/kg per hour A. Derived from endothelium C. 180 minutes D.60 minutes with bolus IV doses of 25 and 100 U/kg. Formation of this ternary complex brings the enzyme in close apposition to the inhibitor.50 units/kg per hour B. Endothelium B. At least 18 saccharide units. 240 minutes Once bound to antithrombin III. 30.6000 Clearance of heparin is mainly extrarenal. 993 B. Exosite 1 acts as a dock for substrates such as C. Heparin must be given parenterally Harrison’s 18th Ed.5000 D. In the circulation.000 Thrombin-inhibiting drugs can block the action of thrombin by binding to three domains: the active site or catalytic site and two exosites. 12 .000 D. At least 16 saccharide units. None of the above 15 units/kg per hour. molecular weight 5400 D. 992 Harrison’s 18th Ed.15 units/kg per hour Harrison’s 18th Ed. Lung D. heparin of what nature simultaneously binds to antithrombin and thrombin ? B. platelet factor 4 (PF4). which internalize and depolymerize the long heparin chains and secrete shorter chains back into the circulation. Factor X B. 25. 993 A. C. 994 1090 Which of the following statements about tissue factor pathway inhibitor (TFPI) ? A. heparin serves as a template that binds antithrombin & thrombin simultaneously. Macrophages C. heparin binds to the endothelium. Platelets A. Intestinal mucosa C. 993 A. Harrison’s 18th Ed. molecular weight 5400 D.000 1095 Which of the following statements about heparin is false ? Heparin has a mean molecular weight of 15. Harrison’s 18th Ed. Liver B. Platelet factor 4 can neutralize anticoagulant activity of heparin B. Exosite 2 A. 50 .65 units/kg per hour C. 25 . 30 . To catalyze thrombin 1093 Which of the following internalize & depolymerize long heparin inhibition. Endothelium 1087 For thrombin inhibition. heparin is infused at rate of 12– D. Factor IX A. 993 C. All of the above B. .000. after IV bolus of 70 units/kg. Active site or catalytic site 1088 What is the mean molecular weight of heparin ? B. D. All of the above Antithrombin is synthesized in liver and acts as a suicide substrate for its target enzymes. heparin chain of at least 18 saccharide units (molecular weight 5400) simultaneously binds to antithrombin and thrombin. A. chains & secrete shorter chains back into the circulation ? thereby promoting the formation of a stable covalent thrombin-antithrombin complex.7500 1096 For therapeutic anticoagulation. enhances the rate at which antithrombin inhibits factor Xa by at least two orders of magnitude but has little effect on the rate of thrombin inhibition by antithrombin. Factor Xa C.4000 C. macrophages. 6000 . AJAY MATHUR Cardiology 91 1085 Antithrombin is synthesized in ? 1091 In the circulation. Macrophages Harrison’s 18th Ed. 993 A. 997 For thrombin inhibition. molecular weight 5400 Heparin binds to macrophages. molecular weight 5400 1094 Heparin-binding domain in thrombin is ? N Engl J Med 2005. Harrison’s 18th Ed. Contribute to antithrombotic activity of heparin For therapeutic anticoagulation. 120 minutes C. Kidney A. 992 Harrison’s 18th Ed. 91 Hematology MCQ’s FOR MCQ’s FOR MEDICAL MEDICAL PROFESSIONALS PROFESSIONALS BY PROF. 993 C. heparin is infused at rate of ? Harrison’s 18th Ed. 100 B. 1 mg of protamine sulfate neutralizes 100 units of heparin. Apparent heparin resistance is a result of raised factor VIII levels. All of the above The most common side effect of heparin is bleeding. Calcium B.92 MCQ’s FOR MEDICAL PROFESSIONALS BY PROF. 675 A. Heparin–PF-4 antibody complexes A. Vascular endothelial cells In Type I HIT. White blood cells D. Directly agglutinating platelets binding to ? Harrison’s 16th Ed. Directly agglutinating platelets B. Heparin-induced thrombosis A. IM C. Other complications include thrombocytopenia. Potassium D. Idiopathic 1102 “True” heparin resistance results from nonspecific heparin B. IV D. Heparin C. S/C B. 200 C. Citrate “True” heparin resistance results from nonspecific heparin binding to WBC. Horse serum C. Factor VII B. Factor II A. Warfarin-induced thrombosis B. Osteoporosis B. and the resultant protamine-heparin complexes are then cleared. Factor VIII C. 675 heparin ? Harrison’s 18th Ed. Factor X D. Salmon sperm D. Leucopenia D. 994 Harrison’s 16th Ed. 1106 “White clot syndrome” is also called ? 1100 1 mg of protamine sulfate neutralizes how many units of Harrison’s 16th Ed. 1104 Which of the following is a type of immunologic 1098 Which of the following is not a side effect of heparin ? thrombocytopenia ? Harrison’s 18th Ed. binds heparin with high affinity. Increased number of bone marrow megakaryocytes C. None of the above D. 1105 Which of the following is false about immunologic thrombocytopenia ? Harrison’s 16th Ed. Acute-phase proteins heparin and platelet-derived heparin-neutralizing protein. 994 A. Platelets coated with immune complexes A. Thrombocytopenia A. Idiopathic A. Horse urine B. D. and elevated levels of transaminases. AJAY MATHUR Hematology 1097 One USP unit of heparin is defined as the concentration of 1103 “Apparent” heparin resistance is a result of elevation of heparin that prevents 1 mL of citrated sheep plasma from which factor ? clotting for 1 hour after addition of ? Harrison’s 16th Ed. 994 A. osteoporosis. All of the above 1109 Characteristic of Platelet factor 4 is ? Harrison’s 16th Ed. a mixture of basic polypeptides isolated from salmon sperm. Idiopathic thrombocytopenic purpura (ITP) D. Anaphylactoid reactions can occur. 674 1099 Protamine sulfate is isolated from ? Harrison’s 18th Ed. Elevated levels of transaminases C. platelet factor 4. Viral or bacterial infections B. citrated sheep plasma from clotting for 1 hour after addition of calcium. All of the above D. 994 A. drugs (heparin) and idiopathic thrombocytopenic purpura (ITP). Offending antigen is a complex formed between C. Bovine urine Most common causes of immunologic thrombocytopenia are viral or bacterial infections. 400 Heparin-induced thrombosis is also called “white clot syndrome”. thrombocytopenia is produced by directly agglutinating platelets. Any of the above 1108 Mechanism of Heparin induced thrombocytopenia type II is ? Protamine sulfate is given as a slow IV infusion. Usually no splenomegaly B. an immune reaction causes thrombocytopenia. 675 A. Thiazide-induced thrombosis D. 674 A. Sodium C. Harrison’s 16th Ed. 1107 Mechanism of Heparin induced thrombocytopenia type I is ? Harrison’s 16th Ed. 675 1101 Mode of administration of protamine sulfate is ? Harrison’s 18th Ed. Such patients usually do not have Protamine sulfate. Heparin–PF-4 antibody complexes C. 688 C. Platelet-derived heparin-aggravating protein . Phenytoin-induced thrombosis C. vascular endothelial One USP unit of heparin is defined as the concentration of heparin that prevents 1 mL of cells & acute-phase proteins. In type II HIT. 994 A. All of the above B. Protamine splenomegaly & show increased number of bone marrow megakaryocytes. 300 D. 688 Harrison’s 18th Ed. 1113 The most specific diagnostic test for diagnosis of HIT is ? 1119 Coagulation monitoring with LMWH is done by ? Harrison’s 18th Ed. ~5 saccharide units D. 996 bivalirudin). 996 B.1.1. 996 A.4:1. Danazol B. Platelet-derived heparin-neutralizing protein 1115 LMWH have a mean molecular mass of ? Harrison’s 18th Ed.5 . PT B. 5 to 14 days C. It has little effect on thrombin inhibition. 0. LMWH can be administered IV if a rapid anticoagulant response is needed. LMWH cannot be administered IV B. Platelet-derived heparin-activating protein A. Anti-factor Xa levels D. Resistance to LMWH is rare HIT occurs 5 to 14 days after initiation of heparin therapy. 996 A. Harrison’s 18th Ed.PF- 4 antibody complexes bind the platelet Fc receptor to induce platelet activation & secretion. Severe of Type II HIT is because heparin . Sulfonamides A. LMWH is cleared by kidneys D. argatroban. D. 995 Harrison’s 18th Ed. 6 hour HIT is an antibody-mediated process triggered by antibodies directed against neoantigens on C. C. Thiazide diuretics C. But. 12 hour more common in surgical patients than in medical patients and. the most specific diagnostic preparations have little effect on aPTT. 1 to 3 days A. 12 to 25 days D. 996 C.5 . 995 C. 0.5 . aPTT C. 675 1116 LMWHs have an anti-factor Xa:antithrombin activity ratio of ? A. ELISA for antibodies against heparin-PF4 complexes A. 93 Hematology MCQ’s FOR MCQ’s FOR MEDICAL MEDICAL PROFESSIONALS PROFESSIONALS BY PROF.2. 2. 995 B. ~17 saccharide units Platelet factor 4 is a heparin-neutralizing protein. 2:1 thrombocytopenia (HIT) ? LMWH accelerates factor Xa inhibition by antithrombin. These antibodies are of the IgG isotype and bind simultaneously to heparin-PF4 complex and to platelet Fc receptors. 3.2 units/mL A. 3-4 hours later.2 units/mL. occurs more frequently in females than in males.3. 1112 HIT occurs how many days after initiation of heparin therapy ? 1118 Which of the following statements about LMWH is false ? Harrison’s 18th Ed. like many autoimmune disorders. or factor Xa inhibitors (fondaparinux). Usually given SC. LMWH has a plasma half-life of ~4 hours and is cleared almost exclusively by kidneys. 996 A.0. More frequent in males than in females B. Alternative anticoagulant to prevent prophylactic doses is ? or treat thrombosis is started like parenteral direct thrombin inhibitors (lepirudin. A. heparin addition induces platelet activation and serotonin release. 2 to 5 days B. LMWH is prepared from unfractionated heparin C. This test is performed by quantifying serotonin release when washed platelets loaded with labeled serotonin are exposed to patient serum in the absence or presence of varying concentrations of heparin. Platelet activation assays B. Serotonin release assay C.4. None of the above D. ~45 saccharide units 1110 Which of the following drugs does not cause thrombocytopenia ? LMWH have a mean molecular weight of 5000 (~ 17 saccharide units) Harrison’s 16th Ed. 994-995 LMWH have anti-factor Xa to anti-factor IIa ratios ranging from 2:1 .2 : 1 C. 1. Fondaparinux Therapeutic anti-factor Xa levels with LMWH range from 0. coagulation monitoring is done by anti-factor Xa levels because most LMWH heparin-PF4 complexes or with platelet activation assays. If the patient serum contains the HIT 1120 Range of therapeutic anti-factor Xa levels with LMWH is ? antibody.3 units/mL . 1:1 1111 Which of the following best relates to Heparin-induced D. 0. Unfractionated Heparin Harrison’s 18th Ed. Antibodies of the IgM isotype 1117 Plasma half-life of LMWH is ? B. 0. AJAY MATHUR Cardiology 93 B. All of the above 1121 Peak anti-factor Xa levels required when LMWH is given in Heparin must be stopped in suspected or documented HIT. Antibodies against neoantigens on PF4 Harrison’s 18th Ed.2 units/mL B.5 . More common in medical than surgical patients A.2 units/mL 1114 Which of the following drugs is useful in HIT ? Harrison’s 18th Ed. Platelet-derived heparin-destroying protein B. 995 Harrison’s 18th Ed. 8 hour PF4 that are exposed when heparin binds to this protein. Lepirudin C. 4 hour D. A. HIT is D. ~30 saccharide units D.1 . Harrison’s 18th Ed. or Harrison’s 18th Ed. test is the serotonin release assay. Any of the above The diagnosis of HIT is established using enzyme-linked assays to detect antibodies against In LMWH therapy.2 units/mL C.5 : 1 D. Bivalirudin D.5 . Protamine sulfate completely reverses anti-factor IIa N Engl J Med 2005. Argatroban D. Lepirudin D. Pregnancy D.dependent has longest duration of action ? coagulation protein ? Harrison’s 16th Ed. Fondaparinux A. 1127 Which of the following drugs used for thromboprophylaxis 1133 Which of the following is a vitamin K . Renal insufficiency B. DTIs can bind to and inhibit the activity of soluble thrombin with continuous IV UFH than with SC LMWH and also thrombin bound to fibrin D. 2728 A. 1128 Which of the following is a natural anticoagulant ? 1122 Indications for LMWH monitoring include ? N Engl J Med 2005. obesity. 997 1132 Warfarin was initially developed as a ? A. Fondaparinux binds only to antithrombin and is D. 689 Harrison’s 18th Ed. protein C & protein S & antithrombin. Obesity C. Rodenticide Fondaparinux does not cause HIT because it does not bind to PF4.0. Indications for LMWH monitoring include renal insufficiency. Consequently. fondaparinux catalyzes factor Xa inhibition by antithrombin and does not enhance the rate of thrombin inhibition. Patients at high risk for bleeding are more safely treated B.2 . Also. 996 A. Sedative D. Argatroban D. Factor X . Warfarin was initially developed as a rodenticide.120 units/kg. peak anti-factor Xa levels of 0. 50 . Major complication of LMWH is bleeding 1130 Which of the following statements about Direct Thrombin Inhibitors is false ? B.80 units/kg A. Activity of DTIs is independent of antithrombin C. 997. anti-factor Xa levels must be measured because most LMWH preparations have little effect on the aPTT. at a dose of 100 . If monitoring is necessary in patients receiving LMWH. 0. Plasma half-life of Fondaparinux is 17 hours A. 998 A. All of the above D.50 units/kg Harrison’s 18th Ed. Protein C and S B. All of the above Coagulation cascade is regulated by natural anticoagulants like tissue factor pathway inhibitor. Ximelagatran In unstable angina. Argatroban too short to bridge thrombin to antithrombin. This facilitates once- weekly dosing for primary and/or secondary prevention of thromboembolic events.8 . Diuretic C. hence can be used for the treatment of HIT. When LMWH is given in prophylaxis.2 . DTIs also have an antiplatelet effect 1125 Fondaparinux has a molecular weight of ? D. Heparin C. None of the above C. Rest of the above DTI’s are cleared by kidney. Harrison’s 18th Ed. 997 B. dose of LMWH is ? 1129 Which of the following “Direct Thrombin Inhibitors” is Harrison’s 18th Ed. infants or children.5 units/mL C. Ximelagatran C. LMWH is given SC. 100 . twice-daily. 996 A. injury. 1123 In unstable angina.5 units/mL are desirable.353:1028-40 B.94 MCQ’s FOR MEDICAL PROFESSIONALS BY PROF. 997 1131 aPTT is used to monitor the anticoagulant effect of all except ? A.100 units/kg B. Recombinant Hirudins Argatroban is predominantly cleared by hepatic metabolism and requires dose adjustments in 1124 Which of the following statements about LMWH is false ? patients with hepatic dysfunction. AJAY MATHUR Hematology B. 80 .353:1028-40 activity of LMWH A. Factor VII B.8 units/mL D. None of the above Harrison’s 18th Ed.353:1028-40 Harrison’s 18th Ed. 996 cleared by liver ? A. Bioavailability of SC Fondaparinux is 100% Harrison’s 18th Ed. 0. 1728 Harrison’s 18th Ed. 30 . Antibiotic D. 0. there is no cross- reactivity of fondaparinux with HIT antibodies. None of the above C. Fondaparinux does not cause HIT B. Tissue factor pathway inhibitor A.1. D.0. pregnancy (III trimester). Melagatran C.0 units/mL Idraparinux is a long-acting pentasaccharide with a half-life of 130 hours. Low-Molecular-Weight Heparin Fondaparinux has a molecular weight of 1728. 998 B. They restrict formation of hemostatic plug to the site of mechanical heart valves. 3728 B.0.120 units/kg C. 4728 C. N Engl J Med 2005.5 . 1126 Which of the following about Fondaparinux is false ? Harrison’s 18th Ed. Antithrombin C. Idraparinux B. > 4.3 (VKOR) thereby interfering with the synthesis of the vitamin K dependent clotting proteins (factor II.0.5 is recommended. 998 B. > 3.1 Warfarin is a water-soluble vitamin K antagonist. 999 Harrison’s 18th Ed.0 . Glutamic acid In atrial fibrillation. recommended. prothrombin complex concentrates can be used.5 A. Fresh-frozen plasma A. more active being the S isomer.. < 1. Harrison’s 18th Ed.0 D. Factor X D. Warfarin does not pass into the breast milk C. Any of the above C. 95 Hematology MCQ’s FOR MCQ’s FOR MEDICAL MEDICAL PROFESSIONALS PROFESSIONALS BY PROF. None of the above D. < 1. IX. < 2.7 C. 1145 Warfarin-induced skin necrosis occurs in which week of 1139 INR in patients with prosthetic mechanical heart valves should therapy ? be ? Harrison’s 18th Ed.5 to 4.0 prothrombin (factor II) and factors VII. 1134 Warfarin acts by inhibiting which of the following ? Harrison’s 18th Ed. the risk of cardioembolic stroke increases at what level of INR ? A.0 Harrison’s 18th Ed. VII. Vitamin K carboxylase B. LMWH or fondaparinux. M and N C. Warfarin is a racemic mixture of R and S isomers. Synthesis of vitamin K dependent anticoagulant proteins (proteins C & S) is also reduced by vitamin K antagonists. and X.5 Warfarin interferes with the synthesis of the vitamin K–dependent clotting proteins like D. > 3. and X).9 D. warfarin is administered in doses that produce a target INR of 2. All of the vitamin K–dependent clotting factors possess glutamic acid residues at their N termini.0 B. First week A.5. Tryptophan to <1. ~ 60 hours infusion till INR is in normal range.3. Vitamin K epoxide reductase (VKOR) Harrison’s 18th Ed.42 hours. For life- threatening bleeds.5 C. Vitamin K hydrolase C. Isoleucine D. 1. ~ 12 hours C. a target INR of 2. 998 B. Warfarin crosses the placenta B. A. For most indications. 1000 Harrison’s 18th Ed. C and D B. 998 A. Factor XI 1144 Patients on warfarin with serious bleeding are treated with ? Antithrombotic effect of warfarin depends on reduction in functional levels of factor X & prothrombin. R and S D. A minimum 5 day course of parenteral anticoagulation is CYP2C9 mediates oxidative metabolism of S isomer. 3. X and Y Initial treatment with warfarin is supported by concomitant administration of either heparin. Any of the above D.3. Prothrombin B. ~ 24 hours D. Fondaparinux C. 1137 Antithrombotic effect of warfarin depends on reduction in 1143 Which of the following statements about warfarin is false ? the functional level of ? Harrison’s 18th Ed.5 . 1000 Harrison’s 18th Ed.5 to 2. 1000 1138 The half-life of warfarin in plasma is ? A. < 2. Racemic warfarin has a plasma half-life of 36 . Warfarin is usually started at a dose of 5-10 mg. 998 A. 2. Treatment with vitamin K should be supplemented with fresh-frozen plasma (FFP) as a source of the vitamin K dependent clotting proteins. 999 which amino acid residues at their N termini ? A. 999 B. AJAY MATHUR Cardiology 95 C. is also reduced by vitamin K antagonists.7 and an increase in bleeding with INR values >4. LMWH B. Factor IX C.0 to 3. All of the above C. It inhibits vitamin K epoxide reductase D. IX. Prothrombin complex concentrates B. Synthesis of the vitamin K–dependent anticoagulant proteins. Heparin A. > 4. IV Vitamin K infusion Harrison’s 18th Ed. Leucine C. there occurs an increased risk of cardioembolic stroke when the INR falls D. Factor VIII B. 999 A. the risk of bleeding increases at what level of INR ? 1135 All of the vitamin K dependent clotting factors possess Harrison’s 18th Ed. 999. ~ 36 hours Patients on warfarin with serious bleeding should be given 10 mg of vitamin K by slow IV D.0 . proteins C and S. >97% of circulating warfarin is bound to albumin so only a small fraction of unbound warfarin is biologically active.5 to 3. 1142 Initial treatment with warfarin is supported by concomitant 1136 Warfarin is a racemic mixture of which of the following isomers ? administration of which of the following ? Harrison’s 18th Ed. 2. 998 1140 In atrial fibrillation. In liver. 1141 In atrial fibrillation. For patients with mechanical heart valves. Vitamin K reductase A. Activase Warfarin embryopathy consists of nasal hypoplasia and stippled epiphyses.96 MCQ’s FOR MEDICAL PROFESSIONALS BY PROF. 1151 Warfarin embryopathy is due to effect of warfarin on which bone matrix protein ? 1157 Which of the following is false about ‘Streptokinase’ ? Harrison’s 18th Ed. Atherosclerotic vascular disease A. First A. Rivaroxaban and apixaban are oral factor Xa inhibitors. acylated plasminogen streptokinase activator complex (anistreplase). 1002 Harrison’s 16th Ed. Any of the above C. Urokinase Alteplase. following can be given during pregnancy ? Harrison’s 18th Ed. First Harrison’s 18th Ed. or symptoms of renal infarction. Osteophysin Skin necrosis due to warfarin therapy is seen 2 . All of the above D. D. Any of the above D. Risk of embryopathy is highest if warfarin is given in the first trimester of pregnancy. Antithrombin C.5 days after initiation of therapy. Protein C or S B. abdominal pain. D. it has no plasminogen activator (PA) activity C. Anistreplase B. risk of warfarin embryopathy is greatest 1153 Recombinant derivative of recombinant tissue-type in which trimester of pregnancy ? plasminogen activator (rt-PA) is ? Harrison’s 18th Ed. recombinant tissue-type plasminogen activator (rt-PA). Dabigatran etexilate B. All of the above “Purple-toe syndrome” presents with atheroembolic symptoms like ischemic (purple) toes. tenecteplase and reteplase are fibrin-specific plasminogen activators while 1149 For prevention or treatment of thrombosis. Approved fibrinolytic agents are streptokinase. Obtained from cultures of b-hemolytic streptococci B. gangrene. also known as alteplase or activase & two recombinant derivatives of rt-PA. Diabetes mellitus C. Dabigatran etexilate is an oral thrombin inhibitor. urokinase. This complex induces a reveals cholesterol emboli in the purple-toe syndrome. 1001 A. Not fibrin-selective D. which of the streptokinase. 690 Harrison’s 18th Ed. Alteplase C. None of the above 1150 “Purple-toe syndrome” is seen in patients receiving warfarin with ? 1156 Streptokinase binds to which of the following ? Harrison’s 16th Ed. Osteotensin D. 690 A. Fourth week C. LMWH B. By itself. Instead. Hypertension B. Plasminogen C. Apixaban D. 1000 Harrison’s 18th Ed. Osteocalcin C. Third B. Skin biopsy streptokinase forms a 1:1 stoichiometric complex with plasminogen. Tissue factor pathway inhibitor A. Alteplase D. 690 . Plasmin B. conformational change in plasminogen that exposes its active site which then converts additional plasminogen molecules to plasmin to induce a systemic lytic state. 1001 A. Streptokinase C. Porphyria D. Anistreplase CNS abnormalities can occur with exposure to coumarins at any time during pregnancy. Any of the above D. Osteogenin Osteocalcin is a vitamin K–dependent bone matrix protein. Heparin A. anistreplase and urokinase are nonspecific plasminogen activators. livedo reticularis. 1001 B. 1146 Warfarin-induced skin necrosis is seen in patients with congenital or acquired deficiencies of ? 1152 Which of the following is an oral anticoagulant ? Harrison’s 18th Ed. Third C. Second A. 1000 activators ? A. Rivaroxaban C. Tenecteplase D. Fondaparinux C. AJAY MATHUR Hematology B. Fibrin D. Second B. Second week A. All of the above Warfarin-induced skin necrosis is seen in patients with congenital or acquired deficiencies of protein C or protein S. 1000 1155 Which of the following is false about ‘Streptokinase’ ? Harrison’s 16th Ed. tenecteplase & reteplase. Streptokinase is not an enzyme and does not directly convert plasminogen to plasmin. 1147 In pregnant women. 1000 Harrison’s 18th Ed. 1148 CNS abnormalities can occur with exposure to coumarins during which trimester of pregnancy ? 1154 Which of the following is a fibrin-specific plasminogen Harrison’s 18th Ed. Third week B. 1000 A. Alfimeprase C. C. B.000. 690 molecular weight of 34. Deep vein thrombosis in pregnant women The half-life of t-PA is ~5 minutes. Horse’s urine alfimeprase is inhibited by  2-macroglobulin. Cultured fetal kidney cells snake) that degrades fibrin & fibrinogen in a plasmin-independent fashion. 365: 1163–74 C. Alfimeprase must be delivered via a catheter directly into the thrombus. Deep vein thrombosis of arms C. D. ~ 15 minutes A. Streptokinase C. ~ 10 minutes Lancet 2005. Urokinase 1160 Half-life of tissue-type plasminogen activator (t-PA) is ? D. AJAY MATHUR Cardiology 97 A. ~ 15 minutes Harrison’s 16th Ed. 690 C. Older age B. Urokinase converts plasminogen to plasmin directly by cleaving the Arg560-Val561 bond. Streptokinase Harrison’s 18th Ed. Hypertension B. Recurrent ipsilateral deep vein . B. A. In circulation. ~ 20 minutes B. ~ 5 minutes D. 1453 D. Higher body weight A. Acts only when complexed with plasminogen 1161 Which of the following is a direct fibrinolytic agent ? Harrison’s 18th Ed. Streptokinase has no affinity for fibrin A. ~ 5 minutes 1164 Paget-Schroetter syndrome relates to ? B. Tissue plasminogen activator Harrison’s 16th Ed. Antigenic and anaphylactogenic B. Staphylokinase D. 1002 Alfimeprase is a metalloproteinase enzyme (isolated from venom of southern copperhead A. Cow’s urine D. TNK-rt-PA 1158 Urokinase is derived from ? D. 690 A. Female sex 1159 The half-life of Urokinase is ? Harrison’s 16th Ed. Acute deep vein thrombosis in cancer patients D. ~ 10 minutes 1163 Which of the following is a “bolus fibrinolytic” agent ? C. Tenecteplase (TNK) Half-life of Urokinase is ~20 minutes. All of the above 1162 Risk of intracranial hemorrhage with use of thrombolytic agents is increased in all except ? Urokinase is a two-chain serine protease derived from cultured fetal kidney cells with a Harrison’s 16th Ed. ~ 20 minutes A. 97 Hematology MCQ’s FOR MCQ’s FOR MEDICAL MEDICAL PROFESSIONALS PROFESSIONALS BY PROF. 1003 B. None of the above C. Notes : . ........... A 30 ........... B 159 ... D 188 ........ C 269 .......... D 3 ........................................................ D 51 .......... D 106 ....... D 266 .. D 280 ............ C 227 ....... D 199 ........ C 67 ......Answer MCQ’s FOR MCQ’s FOR MEDICAL MEDICAL PROFESSIONALS PROFESSIONALS BY PROF....... D 187 ...................... A 102 . B 58 ........... C 34 ........ A 235 .............. D 158 ..... D 200 ...... C 96 ........ A 238 .......................... C 232 ...... A 84 .................. A 5 ..... D 272 . B 281 ................ B 173 ... B 53 ............ D 258 . A 14 .......... D 103 ........................................ A 284 ...................... B 81 ............. D 122 .. A 43 ...... C 260 .......... D 12 .. D 224 ............................... B 68 . D 214 ........ D 186 ............................................ A 287 ......................................... D 155 ......... A 45 .............. B 208 ............................. B 229 .................................. D 4 ...................... D 237 ..................... D 225 ...... D 161 ..................... B 120 .................................. C 52 ......... A 286 .. C 28 . B 194 .......................... B 202 ............... B 11 ............ C 47 ...... D 9 ....... B 241 ....... C 105 ............................ C 204 ........................ B 82 ......... C 65 .. D 41 ...... D 15 .... C 23 ... A 54 ....................... A 40 ............. B 80 .. D 285 ................................ D 174 ... B 50 ... D 293 ................................. D 29 ............ B 33 ................. D 216 .... B 267 . A 6 .. B 184 ............ B 42 .................... B 221 ........... B 261 . D 141 ............................. D 220 ....................... C 185 .......... A 101 .... D 27 ................ D 85 ..... D 175 ................................................. D 206 ................ B 26 ......................... A 91 ... C 255 .......... B 78 .............. C 39 .. D 279 ....... D 107 .......................... A 282 .................................. B 171 ... D 169 .. A 108 ... A 248 ........... A 239 ... A 64 .... B 13 ................ B 257 ............................. A 242 ............... D 142 .................................................... D 252 ........................ C 16 ............... D 128 .. A 240 .......................... D 125 ............................................................ C 115 ..... C 178 .................................. D 251 ..... B 86 .................................................................................. B 228 ......... D 201 ...... C 116 ....... A 144 . D 271 ....................................... D 290 ............... C 294 ..... C 165 ............... A 166 . D 198 ......... D 21 .. D 213 . B 256 ............ D 212 ................... D 275 .... A 151 ..... C 32 ................. D 93 ......... B 152 ........... A 168 .... C 83 ................... B 69 ......................... A 79 ................ D 7 .. B 134 ...... C 273 ........................................ B 10 ... C 189 ....... A 76 ......................... D 2 ...................... C 274 ................................... C 44 ................................ D 222 ............. D 111 .... A ........... A 170 ... D 25 .. C 156 ............................. A 17 ..................................................... A 270 .... D 70 . D 181 ... B 94 ....... D 36 .......... B 73 ...................... D 203 ............ D 18 ................................................ D 146 ..... D 291 ................................... D 288 .................. B 205 ...... C 59 ..... C 60 ........................ A 262 ........................................ C 289 ................... C 35 ............ B 87 .............. C 118 ......... B 133 ............ D 253 .............. D 90 ....... A 195 ...................... B 100 ................. D 292 ......... AJAY MATHUR 99 ANSWERS HEMATOLOGY 1 ..... C 217 .. A 177 .................................... A 126 ................ D 250 ........ A 276 .......................................................................................................................... D 22 ............................ B 190 ................................. D 223 .... D 268 ..................................... A 46 .... B 245 .................... B 48 .... C 167 ................... D 8 . C 131 .................. A 121 ... D 135 . A 112 ... A 72 .................... C 57 ................. B 160 ...... D 114 .... D 56 .................... D 211 .... B 127 ............. C 180 ....... B 259 ........................ A 74 .... D 140 .. C 246 ... D 109 .................. C 137 .. A 243 .................................... C 97 .................. B 179 ...... D 277 ...... B 264 ............................. D 148 ....... D 234 ...... D 145 ......................... C 176 ................. B 98 .......... B 49 ............ A 132 ...................... A 63 .......................................... B 249 ........... B 38 .. B 164 ........ D 254 . D 265 ..................... D 231 . D 55 .......... B 31 ............. A 71 ...... A 77 ........ D 191 .......... C 193 ........... A 233 ............... D 236 ........................... C 147 ........... B 226 .................................................................. C 66 ............ C 219 .... D 119 ..... D 138 ............................................................ C 247 .... D 62 ................... D 110 ..... C 192 ............. D 283 .. C 207 ..... D 123 .............. B 129 .................. B 209 ......... D 143 ..... A 182 ..... D 75 ................................................. A 163 .... C 88 ........ D 130 ...................... D 196 .................................................... A 263 ................... D 99 ...... D 139 ............ C 162 ....... B 197 .. D 157 ...................................................... B 124 ............. D 37 ............................................... B 215 ............... D 153 ......... D 210 ... A 19 ..................... A 61 ...... C 183 . B 117 ....... D 89 . D 92 ...... D 230 .......................... A 149 ....... D 218 ....... D 154 ............. A 113 ... C 104 .. D 172 .......... B 20 ................ B 24 ...................... B 244 . B 136 ..... C 95 ................. A 150 ..... D 278 ....................... .................................... A 525 ........ B 475 .... D 299 ............ A 438 ................................ C 413 . D 320 ......... A 338 ... D 401 ...... D 396 .. B 359 ....... C 425 ....................................... B 336 ................ B 420 ......................... D 354 ............................. A 306 ............ D 368 ............................................... B 370 ................... B 340 ...... C 405 ....... C 356 ......... D ......... C 404 ..... B 565 ..................... C 317 .. B 486 ...................... C 391 .. B 321 ....... D 399 ................. B 556 ... D 482 ........... B 582 .. B 313 .. C 534 .................... A 489 ... D 384 ..................................... C 372 ........................................ D 588 ....... B 374 ................... A 406 .. D 379 ..... D 360 .... C 363 ...... A 512 .......... B 430 ....... D 471 . C 424 ....... C 304 ...... D 497 ................ A 458 ....... A 348 ..................... A 504 ............................................. A 435 ................. D 513 ............................. D 483 ........... D 357 .... D 477 .............. D 585 ... C 400 ....................... D 397 .. A 450 ...... C 329 ... C 449 ................ D 460 ....... A 515 . A 388 . C 547 ................................ A 508 ............ D 398 ...... B 402 .......................................... D 365 ........... C 422 ............................... D 464 ...................................... C 451 ... C 355 ............. D 523 ....... D 334 ............................... A 426 ............................... A 335 ............... AJAY MATHUR Answer ANSWERS HEMATOLOGY 295 ..................................... B 330 ......................... D 403 . C 393 ......... A 414 .... A 429 ............. D 472 .... D 453 .. A 577 ..... A 491 .......................... A 518 ....................... C 544 ........................ A 517 ................................... A 584 ........ A 511 ..................... D 344 .. D 353 ................. D 528 ... B 485 . D 490 ..... D 555 ............... D 457 ........... C 487 ........... B 456 ................. D 559 .................................... B 574 ............................. B 432 ......... C 434 ...... D 350 .................................................... D 524 .... C 428 ..... C 507 ............................................. B 315 .... A 503 .......................................... A 366 ............. C 297 ...... A 468 ........... A 536 .............. D 481 ......... D 492 ......... A 474 ..... C 553 ................ B 377 ........ C 418 ......... D 540 .......... D 570 ......... C 416 ...... D 390 .......................... D 358 ........ D 578 ................. D 572 ..... B 509 ....... A 394 . D 307 . D 549 ..................... A 421 ........... D 493 . D 311 ... D 546 ............................................................................. A 463 .... D 461 ..... A 465 . B 389 ......... C 560 ........ D 408 .................. A 586 ............................... C 328 ....... A 337 . B 325 .......... A 532 .. D 448 ..................... A 502 ................. D 505 ..... D 545 ......................... A 452 .......................... D 571 .............. B 548 ......... A 361 ...... B 309 ............... D 529 ................ A 469 . A 437 . B 533 .. B 316 ... C 387 ........... C 467 ......... D 381 . C 462 .. C 343 .................... D 323 ........................... D 375 .. D 327 ......... C 473 .......................... A 415 ..... B 543 . C 557 .............................. D 576 ......................................... D 331 ............................................ A 455 .... D 466 ........ C 530 ............ B 500 ...........100 MCQ’s FOR MEDICAL PROFESSIONALS BY PROF.. A 563 .............. A 318 ... C 501 ........... B 362 ........... B 440 .................................................. D 470 .................. D 561 ... D 417 ................................................. D 583 ...... D 333 ...................... D 412 .... D 332 .. C 539 ... A 443 ........ B 488 .......... D 300 .. D 447 ....... C 382 .......................................... D 324 ............................................... B 383 ...... D 479 ........ D 575 .......... D 459 .................................... A 301 . B 526 ......... D 326 ..... B 558 ........................ D 579 ...................................... A 436 .......... D 431 ............. D 495 ................... D 346 .... D 510 ................................................ D 364 ............. C 537 .................................... A 522 ............................. A 349 .. B 423 ..... A 562 ....... B 551 ... B 506 . D 494 ............. D 514 ........ A 305 ................ B 410 . D 345 ............... D 385 ........ D 342 .................... A 444 ........ D 454 .... B 369 .......... D 538 ...... B 496 .............. D 347 ......................... D 535 ...... A 308 ........................ D 303 ........... B 419 .... A 411 ............... D 484 ..... A 376 ....... A 427 ............................................... D 587 . D 378 . D 373 ................ C 351 .................. C 580 .............................. D 521 ......................... D 568 ............. C 499 ....................... A 371 ............. A 527 ................. B 498 ........... A 441 . B 519 ........ C 439 ...... D 552 .... B 554 ....... A 392 .............. A 550 ...................... A 442 .............. D 296 .. B 341 .......... B 581 ............................... D 352 ... D 367 ..... D 310 ................... D 409 ........ B 573 ......... B 302 ....................... D 298 .... C 520 ......................................... B 319 ....................... D 380 ............................ C 446 ............. C 314 ..... D 531 ........ B 566 ................... B 386 .............. B 433 ................... C 564 ....... A 569 .................... B 516 .............................. A 395 . D 407 .. D 567 ......... C 480 ..... C 339 .... D 478 ............. D 542 ................. C 541 ........... D 445 ................. B 312 .......................... D 322 ......................................... D 476 . D 787 ....................... A 871 ................................ C 726 .... B 713 ................ D 761 .. C 705 ....................... A 873 .......... D 757 ... D 684 ......... B 753 ............................. D 854 ..... C 666 ............................ B 711 ....................... D 791 ... A 662 .... D 737 ........... D 703 ................. AJAY MATHUR 101 ANSWERS HEMATOLOGY 589 ........ D 810 .................... C 746 ............... D 843 .. A 710 ....... D 750 ................................... D 764 ...... A 696 .......................... D 755 ......... B 689 ... D 776 ....Answer MCQ’s FOR MCQ’s FOR MEDICAL MEDICAL PROFESSIONALS PROFESSIONALS BY PROF. D 756 ...... C 727 . A 827 ......... A 767 .............. D 769 ........ D 687 ............................. A 665 ........ A 754 .............................. D 661 ...................................... D 680 ......................... C 635 ............................... B 625 .................. C 779 ............. D 863 ....... B 714 ................. D 851 ...... B 786 ........ D 806 ..... D 766 .................. A 762 ......... A 688 ..................................................... C 768 ....... D 631 .............. C 725 ..................... A 830 .. D 792 ......................... B 858 .. A 771 . C 820 ......... D 650 .............. D 638 ..... A 655 ........................ C 673 .......... D 801 ............. A 840 ............ D 855 . D 691 ....... D 719 ...... D 833 . B 646 ................... D 623 ............................ D 656 ............................... B 593 .. D 702 .. C 723 ....................... A 620 ............................................. D 648 ............... D 784 ... A 821 ..... B 789 ............. B 706 .... D 709 .. D 829 .... C 839 ................................. A 867 ........ A 716 ......... D 825 .................. D 732 ............ A 819 ................................................................... C 734 .. A 624 .............. D 742 ........ B 862 ........................ D 748 ... C 717 ..................... D 780 ............ A 722 ......... A 708 ..... B 848 ............ C 773 ........................... C 674 ............. B 634 .. B 797 ................................ C 692 . D 735 ... D 803 ........ A 826 ........ D 613 ........... D 745 .. A 743 . C 602 .... D 752 ........... A 664 ..... D 724 ...... B 660 ...... D 875 .. D 847 .... B 699 ... D 641 ................. C 603 ........................................ D 841 ..................... A 630 .... C 617 ........................... D 672 ...... A 736 ......... D 629 .............. C 772 ... D 632 ............................ A 878 ... A 712 ... D 838 .............................................. D 834 ............................. A 781 ....... D 643 ........... A 633 . C 774 .... D 610 ............ D 818 ..... D 715 .......................... D 831 .................. B 596 ............. D 659 .............. D 845 ............................... B 812 ..................... D 667 . A 861 ............................... A 686 ....................... D 795 .......................................... C 690 .................................................. B 609 ............................ C 685 . C 599 ...................................... B 649 .......... A 590 ................... A 592 .. D 700 .................. A 627 .............. A 675 ...... D 785 ............ A 654 ................ B 856 ... D 751 .. C 880 ............ D 626 ............. D 881 . A 869 ............ C 813 .............................. D 739 ......................................... B 770 ...................... C 790 ................. D 639 ............ B 637 ........................................................... D 647 ..... D 640 ............................. A 846 .............................. A 652 ... D 701 . D 807 . A 614 . B 860 ........ C 835 ............ D 601 ....... D 597 ... B 837 ....................... D 668 ................. B 823 ... D 729 ......................................... D 657 ................ D 718 ......... D 695 .... B 747 ............................... D 671 ................................... D 744 ... C 853 .......... C 817 ............ A 859 ... A 733 ....... A 728 ...................................... D 775 ..... D 811 ....... D 865 ..................................... C 621 ......... D 877 .................... C 651 ................................ D 816 ................... D 642 ................................ D 805 .................................. C 882 .................. D 866 ..... B 874 ... B 852 ............ D 612 ............................. D 694 .... B 836 ............ B 598 ............................................................ D 622 ................ D 793 .... B 595 ............................................................... D 804 ................................ A 707 ................... A 870 .................. D 758 .......... B 802 ................ A 782 ................................. D 653 ........ D 798 ......... B 868 ............................................ C 645 ...... D 832 ........... D 842 ...... B 849 ...................... B 611 ...... D 777 ... D 844 . C 876 ............ A 608 ...... C 850 .. D 636 ................................. D 808 ........ A 857 . C 749 ............. C 605 ..... A 760 . D 879 ....................... A 721 .. A 828 ........ D 765 ........ D 763 ......... D 669 ....................... D 824 . A 616 ............. D 864 .. C 606 ... D 815 ............... A 872 ... C 679 ......... D 644 ................... D 594 ........... D 730 .............................. D 741 ... B 697 ............................ A 731 ......................... D 788 ....... D 658 .. C 693 ........ B 704 ... D 738 .... D 720 ..... D 670 .................... C 822 ................................ B 800 ............... B 681 .......... C 628 ................................. B 607 .. D 778 .... D 682 ...... B 794 .. C 676 .. C 683 ... D 619 .................. C 600 .......................... D 698 ...... D 591 .. B 799 ............. C 615 ............. D 759 ... D 678 ... D 663 ........... C 809 ............. A 740 .......... D 618 . D 677 .... D 796 .... A ...................... D 604 ................. A 814 ... D 783 ... ... A 1028 ...... A 890 ............. A 896 .. D 1079 ...................................... D 1151 ............................ D 905 ....... D 1099 . D 884 ........................ C 1152 .... D 1011 ....... D 1043 .. D 1044 ................ C 1031 ....... B 926 ............. B 964 ....................... A 1002 .. D 1125 ........ D 1078 ........... A 1126 .............................. C 938 ..... D 1068 ....... A 919 ....................................... A 930 ........... D 927 ............................... A 1106 ....... C 941 ........................... A 903 ........................... D 1041 ............... A 1007 ........102 MCQ’s FOR MEDICAL PROFESSIONALS BY PROF..... C 1058 ..... C 946 ... D 978 ....................... A 1038 .. A 954 ....... C 1086 ........ A 907 ................. D 921 .... C 1029 ...... A 1145 . C 1109 ..... A 953 .... C 1136 ................................. D 1093 ........................ D 1014 .......................................... C 1008 ................................................. D 929 .... D 979 ... D 889 .. C 1082 ... C 1066 ......... B 1006 ....... D 1107 ......... C 1117 .. A 1040 ....... A 1033 ........ A 1013 . C 999 ............. D 1000 .. D 1127 .......... C 909 .............. D 1105 ............................ C 961 .... D 900 ................................. D 911 .... A 923 . D 1116 . D 993 ............................................................................ A 1039 ..... B 1118 .......... D 1054 ....... B 962 ..... D 908 .............. A 1132 .... D 886 ............... D 973 ... A 1095 .......... D 952 ................................. D 1147 . A 1059 ........ D 897 ........................ D 1154 .... A 1019 .................... C 887 . A 1146 .. C 924 . D 989 .................................. D 898 ....... C 991 ........... B 942 ............. B 1073 .................... A 922 ........... B 1160 ........................... A 1001 ... B 1023 ......................................... D 1113 ........... D 1037 ................ C 986 ........ D 943 ......... D 990 ........................ C 1035 ............... D 1018 ......... D 968 ............................. D 977 ..... A 937 .... D 1122 ......... C 1159 ......... A 1064 ... D 974 ........ D 1016 ................................ B 967 ............... D 1139 ... A 1149 ...... D 969 .......... D 1010 ....... D 1096 ........ B 1026 ......................... A 906 .. A 1048 .......... A 955 ................................ A 995 .... D 980 ....................... A 998 ..... D 1163 ....... A 925 ........ D 1088 .... A 984 ....... B 1158 ...... D 951 ............ A 1047 ........ C 976 ................ C 1022 .. D 994 .......... D 983 ... C 1162 ........ B 981 .... B .......................... D 1120 ............... D 947 ...... D 1056 ................. A 1004 .......... D 959 . A 1060 ............................... C 1071 .................... C 944 ... C 893 ............. B 1142 ... A 1065 ..................... C 1076 ..... C 932 ....... D 1069 ......... B 1144 ......... D 931 .. C 894 .......... B 1111 .......... A 1087 ... C 892 .. B 936 . B 910 ......... D 928 ....... D 958 ...... D 915 ..... C 1150 ..... B 902 ................... A 1072 ...... D 1017 .. AJAY MATHUR Answer ANSWERS HEMATOLOGY 883 .. D 1015 .... B 1156 ................... D 934 .............................. D 1005 ....... D 1097 .. D 1045 .................... D 1098 .. C 1049 ........ D 1089 .......... D 1057 .............. C 918 ............................................................... A 917 .................. C 891 ............ A 985 .... B 1024 ........ D 988 ...... B 1009 .................. B 885 ............. D 1104 ............ D 957 ....................... B 960 ................. B 949 ....... C 895 ........ C 1003 ................................ C 1021 ............ A 1101 ................... D 1080 ..... D 1055 .................................. C 1129 .............. C 1050 ..................... D 996 ................................................................... B 997 ......................................... D 966 .................. C 948 ........................ D 899 .......... A 1075 ......... A 1115 ............. D 1091 .. D 888 ............. B 912 ... A 1141 .......... B 1108 ...... D 975 ......... D 1133 ............................ A 1025 ............................................ D 1153 ... C 963 .... D 1083 ......................... D 1090 ...... D 1052 . D 1070 .. B 1138 ............................................ A 970 ...................................................................... C 1143 ...... B 920 . C 1123 . A 1067 ................................................................................ D 935 ................ D 1074 ........................ D 1063 .................... B 1102 ......... D 1094 ...... B 1030 .... B 1131 . B 1114 ... A 939 .......... D 1140 ...................................................................... C 1012 ...... A 1155 ... D 971 . C 1161 ................. B 1020 .... A 1130 ... A 933 .. B 1134 .. B 965 ...................... C 1135 ............... C 1148 ...... D 1121 ........... C 1077 ....... B 1112 ........ C 1062 ....... A 914 ................................ D 945 ............. D 1051 ................. B 1081 ............ C 1084 ............ A 987 ........... A 1036 ........ C 1110 .............. D 950 .................... A 1027 .. A 982 ................ C 1157 ....... C 1128 .................... A 901 ......... C 1119 .. A 1137 ...... B 1164 ............... B 956 . D 1053 ..................................... D 940 .......... A 1061 .. C 1092 ...... D 1085 . A 1124 .............................................. C 972 ... A 1034 ........... B 992 ....... D 1042 .... A 916 .. A 1046 ........ D 913 .... D 1103 ........... D 904 . C 1100 ............ D 1032 ... Asthma & COPD C. It points to the diagnosis of CHF. Emphysema C. spontaneous pneumothorax. Afferent fibers in the phrenic nerves B. Asthma A. 202 paroxysmal dyspnea of left ventricular failure ? A. Increased respiratory activity in brainstem 2 Sudden and unexpected dyspneic episodes at rest can be B. 201 C. Sitting C. 203 Harrison’s 16th Ed. chemoreceptors in brain.e. 201 A. Increased respiratory activity in brainstem B. anxiety. Lateral Severe kyphoscoliosis alters ventilation to produce chronic cor pulmonale & respiratory failure. 103 Cardiology MCQ’s FOR MCQ’s FOR MEDICAL MEDICAL PROFESSIONALS PROFESSIONALS BY PROF. Pleural effusion 13 Which of the following is called “cardiac asthma” ? Trepopnea is dyspnea that occurs only in a lateral decubitus position. 201 A. Dyspnea occuring in supine posture is termed orthopnea. Nocturnal episodes B. Sudden & deep breath rather than difficulty in exhaling. Paroxysmal nocturnal dyspnea (PND) . Bronchi A. 202 4 Orthopnea is seen in ? Harrison’s 16th Ed. This stimuli may come from intrathoracic receptors via vagal nerves. Sudden and unexpected C. which of the following is the predominant sensory experience ? 3 Which of the following is most characteristic of severe Harrison’s 16th Ed. most often in patients Harrison’s 16th Ed. afferent C. 201 A. Anxiety carotid bodies. Chemoreceptors in aortic and carotid bodies D. Pulmonary emboli D. asthma. Chemoreceptors in aortic and carotid bodies A. Sense of air hunger arises from increased respiratory activity within brainstem & sensation of chest tightness results from Laboured breathing is not synonymous with dyspnea. Ankylosing spondylitis C. Abnormal A. discomfort is expected to be transient. Inability to take in a sufficiently deep breath Harrison’s 16th Ed. Harrison’s 16th Ed. All of the above Despite the fact that severe limitation of expiratory flow & hyperinflation of lung are characteristic of chronic bronchitis. 202 A. Congestive heart failure B. All of the above D. Sudden & unexpected dyspnea at rest occur with pulmonary emboli. Platypnea is dyspnea that occurs only in upright position. Rheumatoid arthritis D. though aware of breathing. it is invariably accompanied by obstruction of airways. Orthopnea is characteristic of congestive heart failure. one is unaware of the act of breathing. A. All of the above D. or anxiety. Orthopnea D. hypercapnea secondary to breath holding. Chronic bronchitis B. COPD. abnormally uncomfortable. Stimulation of vagal-irritant receptors associated with all except ? Harrison’s 16th Ed. 201 A. 12 J (juxtacapillary) receptors are found in ? 6 Trepopnea most often occurs in patients with ? Harrison’s 16th Ed. Bilateral diaphragmatic paralysis D. Awareness C. Terminal brochiole B. sensory experience is often that of an inability to take in a sufficiently Nocturnal episodes of dyspnea are a typical feature of left ventricular failure. Severe kyphoscoliosis A. 203 with heart disease due to positional alterations in ventilation-perfusion relationships. 201 Harrison’s 16th Ed. cortical centers or afferent fibers in phrenic nerves. Uncomfortable B. 10 Obstruction of airways is an invariable finding in ? Harrison’s 16th Ed. All of the above D. Heart disease D. All of the above Emphysema is a parenchymal disease. Metabolic acidemia somatic nerves from respiratory muscles & chest wall. Stimulation of vagal-irritant receptors C. Spontaneous pneumothorax Dyspnea is characterized by an excessive or abnormal activation of respiratory centers in brainstem. Any of the above D. Upright B. at rest. Difficulty in exhaling B. AJAY MATHUR Cardiology 103 1 Which of the following features of breathing define dyspnea ? 7 The sense of air hunger arises from ? Harrison’s 16th Ed. unexpected dyspneic episodes at rest is more typical of pulmonary embolization. spontaneous pneumothorax. Alveolar interstitial space C. Supine D. 11 Chronic cor pulmonale & respiratory failure is more common in which of the following diseases ? 5 Platypnea is dyspnea that occurs in which position ? Harrison’s 16th Ed. Afferent fibers in the phrenic nerves Normally. Dyspnea is defined with prefixes before 8 The sensation of chest tightness probably results from ? awareness of breathing i. Pectus excavatum B. 202 A. or bilateral diaphragmatic paralysis. 203 Harrison’s 16th Ed. Difficulty in inhaling and exhaling C. COPD C. aortic & D. 9 In chronic bronchitis. With exercise. Asthma B. is rarely accompanied by dyspnea. Hyperventilation with metabolic acidemia stimulation of vagal-irritant receptors. 203 D. B. Endogenous vasoactive substances 24 Overdoses of which of the following heroin preparations is D. Platypnea edema” ? Harrison’s 16th Ed. C. During night. All of the above persons of which age ? G Chronic bronchitis causes nocturnal dyspnea due to bronchial mucus hypersecretion leading to Harrison’s 16th Ed. Central nervous system disorders B. total blood volume is increased due to fluid mobilization from edematous areas leading to pulmonary congestion. Asthma 20 High-altitude pulmonary edema (HAPE) is more common in D. Repolarization 9 C. Peripheral nervous system disorders 18 Pulmonary edema due to “imbalance of Starling forces” includes all of the following except ? C. All of the above A. administration of oxygen and/or return to B. Shock due to hemorrhagic pancreatitis U 16 In neurocirculatory asthenia. Left atrial dilatation Exposure to high altitude in association with severe physical exertion causes pulmonary edema in healthy unacclimatized persons. 30 to 60 years of left ventricular failure ? V Harrison’s 16th Ed. Chronic bronchitis called overperfusion pulmonary edema. Infants B. Orthopnea 19 Which of the following is termed “overperfusion pulmonary C. Steroid ta suggest which cause of dyspnea ? C. with recumbency. Cardiac lower altitudes reduces the incidence of high-altitude pulmonary edema (HAPE). AJAY MATHUR Cardiology B. Increased pulmonary capillary pressure Neurogenic pulmonary edema has been described in patients with CNS disorders and without B. B. Trepopnea A. Left ventricular hypertrophy ti e C. 205 Frequent sighing respirations & irregular breathing point to a psychogenic origin of dyspnea. ECG changes are most often seen during repolarization. diffuse pulmonary infections. 205 n Left atrial &/or left ventricular dilatation. Shock due to sepsis may be low at rest or may decline during exercise in patients with severe lung disease. 203 Toxic insult to lungs. Increased pulmonary venous pressure secondary to LVF C. Shock following cardiopulmonary bypass - changes are most often seen during ? D. B. 203 D. A. 2 agonist salmeterol h 17 Frequent sighing respirations & irregular breathing pattern B. Decreased plasma oncotic pressure apparent preexisting left ventricular dysfunction. 204 D. Diseases of chest wall or respiratory muscles Harrison’s 16th Ed. Chromolyn A. Heart failure Increased pulmonary capillary pressure secondary to increased pulmonary arterial pressure is B. Ipratropium bromide Harrison’s 16th Ed.104 MCQ’s FOR MEDICAL PROFESSIONALS BY PROF. particularly due to A. aspiration & shock. reduced LV ejection fraction & disorders of LV wall motion are clues to a left ventricular cardiac etiology. Reduced left ventricular ejection fraction 21 Which of the following diffuse pulmonary edema does not have D. 204 D. B. Morphine interstitial pressure. 205 Imbalance of Starling forces leading to pulmonary edema can be produced by increased pulmonary capillary pressure. < 25 years R 15 Echocardiographically. Central + peripheral nervous system disorders Harrison’s 16th Ed. Dextropropoxyphene . decreased plasma oncotic pressure or by increased negativity of A. > 75 years d A. Reduced right ventricular ejection fraction a hemodynamic origin ? Harrison’s 16th Ed. LVH. Any of the above incidence of high-altitude pulmonary edema (HAPE) ? i Harrison’s 16th Ed. A. C. Pulmonary 23 Neurogenic pulmonary edema has been described in ? D. Right ventricular ejection fraction A. Depolarization + Repolarization 22 Prophylactic inhalation of which of the following reduces the r D. Increased negativity of interstitial pressure associated with pulmonary edema ? Harrison’s 16th Ed. Any of the above A. Circadian variations increase bronchial sensitivity between 2 AM & 4 AM in asthma patients leading to episodes of nocturnal dyspnea. Increased pulmonary capillary pressure secondary to 14 “Nocturnal dyspnea” is a feature of which of the following ? increased pulmonary arterial pressure Harrison’s 16th Ed. 205 airway obstruction. are associated with 9 diffuse pulmonary edema that clearly does not have a hemodynamic origin. B. 203 D. 205 In neurocirculatory asthenia. the electrocardiographic C. kinins) alter alveolar- capillary membrane permeability (acute respiratory distress syndrome). Methadone C. A. Endogenous vasoactive substances (histamine. C. hemorrhagic pancreatitis & following cardiopulmonary bypass. which of the following is not a feature C. Psychogenic Prophylactic inhalation of  2 agonist salmeterol. All of the above Harrison’s 16th Ed. It is relieved by cough & expectoration. It is common in persons under the age of 25 years. Depolarization sepsis. Increased pulmonary venous pressure without LVF PND is also called cardiac asthma. 32 When hypoxia occurs consequent to respiratory failure.000 ft). Cardiac arrhythmia Large negative intrapleural pressures during acute severe asthma may cause interstitial edema. . Respiratory failure Rapid evacuation of a large pneumothorax causes increased negativity of interstitial pressure. death usually results from ? C. Any of the above Pasteur’s effect refers to switch from aerobic to anaerobic metabolism. None of the above Most common cause of respiratory hypoxia is ventilation-perfusion mismatch. Left C. 287 Parenteral & oral overdoses of legitimate preparations of morphine. Phosphoglycerate kinase A. 287 A. Glucose transporters Glut-1 and Glut-2 C. Partial seizure B. Lymphangitic carcinomatosis Harrison’s 18th Ed. 287 A. 287 A.channels D. 34 Cyanosis occurs upon ascent to an altitude of ? Harrison’s 18th Ed. TGA & Eisenmenger’s syndrome) resembles In pulmonary vascular smooth-muscle cells. All of the above D. 287 Harrison’s 18th Ed. 287 28 During hypoxia systemic arterioles dilate by opening of ? A. All of the above are about 85 & 50 mmHg. Cl. K+ channels C. 287 D. and by growth factors like vascular endothelial growth factor (VEGF) & erythropoietin (EPO). 3000 meters A. ClATP channels in vascular smooth-muscle cells Cyanosis is manifest in an ascent to 4000 m (13. Ventilation-perfusion mismatch C. 4000 meters B. All of the above A. 35 In which of the following conditions. B. 205 D. Abnormal hemoglobin derivative B. FIO 2 & alveolar PO 2 D. Flow rate in vessels D. 105 Cardiology MCQ’s FOR MCQ’s FOR MEDICAL MEDICAL PROFESSIONALS PROFESSIONALS BY PROF. Right B. acute hypoxia resembles acute alcoholism (impaired judgment. A. with greater quantities of O 2 released at any level of tissue PO 2 . All of the above D. Phosphofructokinase B. All of the above Hypoxia due to congenital cardiac malformations (TOF. 287 B. Autonomic failure Hypoxia and Cyanosis In severe hypoxia. PaCO2 rises & Hb-O 2 dissociation curve is displaced to right. inhibition of K + channels causes causing contraction. Hypoventilation B. 287 Harrison’s 18th Ed. 26 Pasteur’s effect relates to ? hemoglobin-oxygen dissociation curve is displaced to ? Harrison’s 18th Ed. Eisenmenger’s syndrome C. In hypoxia systemic arterioles dilate by opening of K ATP channels in vascular smooth-muscle cells. AJAY MATHUR Cardiology 105 D. Intrapulmonary right-to-left shunting D. Transposition of great arteries (TGA) B. 27 Which of the following gene is upregulated in adaptation to hypoxia ? 33 Most common cause of respiratory hypoxia is ? Harrison’s 18th Ed. Migraine A. All of the above 30 Acute hypoxia causes a clinical picture resembling ? Harrison’s 18th Ed. Acute severe asthma 31 In severe hypoxia. KATP channels in vascular smooth-muscle cells D. B. pulmonary vascular smooth-muscle cells to normal with inspiration of 100% O2 ? contract due to inhibition of ? Harrison’s 18th Ed. motor incoordination). When hypoxia occurs due to respiratory failure. PaO2 cannot be restored 29 During hypoxia. intrapulmonary right-to-left shunting & PaO2 cannot be restored to normal with 100% O2. respectively & SaO 2 is ~75% leaving more reduced Hb in arterial blood. Center D. Acute alcoholism Harrison’s 16th Ed. centers of brainstem are affected & death results from respiratory failure. PaO 2 declines. 287 Harrison’s 18th Ed. methadone and dextropropoxyphene can produce pulmonary edema. Rapid evacuation of a large pneumothorax Clinically. Lymphatic blockade due to lymphangitic carcinomatosis may lead to interstitial edema. 5000 meters C. Seizure D. Peripheral neuropathy 25 Which of the following leads to the development of interstitial edema ? C. Adaptations to hypoxia are mediated by upregulation of genes encoding glycolytic enzymes like phosphoglycerate kinase & phosphofructokinase & glucose transporters Glut-1 & Glut-2. Pulmonary arteriovenous fistulae C. Tetralogy of Fallot (TOF) A. NaATP channels in vascular smooth-muscle cells C. Switch from aerobic to anaerobic metabolism A. At this height. 2000 meters Harrison’s 18th Ed. C. Na+ channels B. venous blood tends to 43 Which of the following is suspected when blood remains brown have a high O2 ? after mixing in test tube & exposed to air ? Harrison’s 18th Ed. Normal A. As D. Cyanosis is also observed when nonfunctional hemoglobin (methemoglobin or sulfhemoglobin) is present in blood. 288 45 In peripheral cyanosis of extremities. . Marked polycythemia B. 46 Clubbing without cyanosis is frequent in ? ta Harrison’s 18th Ed. 287 ti e C. 37 In which of the following hypoxia’s. of reduced hemoglobin exceeds ? - Harrison’s 18th Ed. the PaO2 is ? 42 Differential cyanosis is a feature of ? Harrison’s 18th Ed. Pulmonic stenosis U Elevated pulmonary vascular resistance that produces cyanosis in the presence of intra. Methemoglobin R D. Circulatory hypoxia B. 3 gram / dL A. Infective endocarditis Harrison’s 18th Ed. the arterial blood is ? A.106 MCQ’s FOR MEDICAL PROFESSIONALS BY PROF. Cyanide poisoning Harrison’s 18th Ed. Normally saturated with oxygen 9 C. Carboxyhemoglobin (COHb) C. Patients with marked polycythemia become cyanotic at higher levels of SaO 2 than patients with normal hematocrit values. 288 d A. 230 A. cyanosis occurs in the lower but not in the upper extremities. absolute quantity In patent ductus arteriosus. 287 Harrison’s 17th Ed. Interstitial fluid volume D. Edema is defined as a clinically apparent increase in interstitial fluid volume. Any of the above D. Spectroscopy confirms the diagnosis. All of the above Most common congenital cardiac lesion with cyanosis in the adult is tetralogy of Fallot. As a result. Tetralogy of Fallot A. Tetralogy of Fallot B. AJAY MATHUR Cardiology 36 In anemic hypoxia. inflammatory bowel disease & in jackhammer operators. Sulfhemoglobin Clubbing without cyanosis is frequent in infective endocarditis. Cyanosis D. 288 Harrison’s 16th Ed. Patent ductus arteriosus B. Atrial septal defect In anemic hypoxia. Sulfhemoglobin V Diagnosis of methemoglobinemia is suspected if blood remains brown after mixing in a test 38 Example of “Histotoxic hypoxia” is ? tube & exposure to air. Intracellular fluid volume B. PaO 2 is normal but due to reduction of Hb concentration. D. 288 A. This condition is called histotoxic hypoxia. Increased C. absolute quantity of reduced Hb is still small and patients may not become cyanotic even with marked arterial desaturation. results. Over saturated with oxygen r D. Intracardiac communication Cyanide causes cellular hypoxia because tissues are unable to utilize O 2. Ventricular septal defect D. Ventricular septal defect C. that is. Raynaud’s phenomenon A. 289 9 B. Cyanide poisoning C. Any of the above h concentration of total Hb is markedly reduced in severe anemia. differential cyanosis of O 2 transported per unit volume of blood is diminished. pulmonary hypertension and right-to-left shunt. Methemoglobin D. Under saturated with oxygen i It is the absolute rather than relative quantity of reduced Hb that produces cyanosis. Jackhammer operators C. Excercise induced A. Atrial septal defect D. All of the above D. Edema 41 Most common congenital cardiac lesion associated with cyanosis in adult is ? Harrison’s 16th Ed. Inflammatory bowel disease B. Marked polycythemia G B. High altitude hypoxia B. 290 40 Cyanosis can be observed in all except ? A. 211 47 Edema is defined as a clinically apparent increase in ? Harrison’s 18th Ed. venous blood tends to have a high O 2 tension. Severe exercise 44 Which of the following is false in Eisenmenger syndrome ? B. 4 gram / dL B. Plasma volume C. Decreased B. Elevated pulmonary vascular resistance n C. 211 A. Carboxyhemoglobin (COHb) C. Patent ductus arteriosus C. 2 gram / dL Harrison’s 18th Ed. Harrison’s 18th Ed. 5 gram / dL C. Carbon monoxide intoxication D. 290 A.& 39 Cyanosis is apparent when the mean capillary concentration extracardiac communications without pulmonic stenosis is termed Eisenmenger syndrome. Cofactor C. Atrial myocytes D. AJAY MATHUR Cardiology 107 48 Which of the following is referred to as “tissue tension” ? 54 Renal effects of Angiotensin II are mediated by activation of Harrison’s 17th Ed. Aldosterone antagonism B. Acute. Renal efferent arteriolar constriction C. 291 59 Atrial natriuretic peptide (ANP) is stored in secretory granules A. Enzyme within ? Harrison’s 18th Ed. Renal afferent arteriolar constriction B. Compensated heart failure Diminished renal blood flow resulting in diminished stretch of juxtaglomerular cells lowers sodium chloride load reaching distal renal tubules signals juxtaglomerular cells to secrete renin. Reduced hepatic catabolism C. 291 Harrison’s 18th Ed. Kidney B. Renal glomerular capillary constriction D. 292 B. Increased secretion B. 53 Renin is which of the following kinds ? Harrison’s 18th Ed. All of the above D. 58 Mineralocorticoid escape phenomenon is best explained by ? 52 Angiotensinogen is synthesized by ? Harrison’s 18th Ed. All of the above C. Juxtaglomerular cells of kidney B. These renal effects are mediated by activation of Angiotensin II type 1 receptors. 232 A. stable. It is due to an increase in GFR (pressure natriuresis). Deficit in effective arterial blood volume A. Hydrostatic pressure within the vascular system Harrison’s 17th Ed. 291 constriction of which of the following ? A. 291 Harrison’s 17th Ed. aldosterone secretion is elevated due to ? 50 Conditions that reduce effective arterial blood volume cause Harrison’s 18th Ed. venous end of capillary & by way of lymphatics. Colloid oncotic pressure within the vascular system A. Pulmonary veins Renin is an enzyme with a molecular weight of ~40. Lung Administration of potent mineralocorticoids (deoxycorticosterone acetate or fludrocortisone) Angiotensinogen. Chronic heart failure C. 57 Activation of Renin-Angiotensin-Aldosterone (RAA) system is 51 Which of the following stimulates renin release ? most striking in which of the following ? Harrison’s 18th Ed. prolonged biologic half-life & reduced hepatic catabolism due to reduced hepatic blood flow. Liver C. a phenomenon known as mineralocorticoid escape wherein edema does not develop. Type 1 C. Stable heart failure D. severe heart failure & Activation of  -adrenergic receptors in juxtaglomerular cells by sympathetic nervous system & is less intense in patients with chronic. Pressure natriuresis C. 49 Movement of water & diffusible solutes from vascular space into the interstitial space occurs at ? 55 Aldosterone is produced by ? Harrison’s 18th Ed. Activation of RAA system is seen conspicuously in early phase of acute. Arteriolar end of capillaries A. renal efferent arteriolar constriction. D. 232 which type of Angiotensin II receptors ? A. despite continued exposure decapeptide angiotensin I. Hydrostatic pressure within the interstitial fluid B. 232 A. Macula densa cells of kidney C. 291 A. levels of aldosterone are raised due to increased secretion. severe heart failure B. Sinoatrial node C. Type 3 Plasma & interstitial fluid are two components of extracellular fluid regulate by Starling forces. circulating catecholamines stimulates renin release. Low sodium chloride load in distal renal tubules B. All of the above D. 290 A. 232 B. Zona glomerulosa of adrenal cortex D. nephrotic syndrome & cirrhosis reduce effective arterial blood volume and cause cardiac output.000 secreted by juxtaglomerular cells. an  2 globulin. Pancreas D. Angiotensin II produces renal vasoconstriction & salt and water retention. secondary to reduction in Heart failure. Renin converts angiotensinogen to a leads to salt & water retention. Fluid is returned from interstitial space into vascular system at the Angiotensin II. Zona reticularis of adrenal cortex Movement of water & diffusible solutes from vascular space into interstitial space occurs at the Aldosterone is produced by zona glomerulosa of adrenal cortex & its release is stimulated by arteriolar end of capillaries. Diminished stretch of the juxtaglomerular cells A. All of the above In heart failure. Venous end of capillaries B. which is broken down to an octapeptide angiotensin II. Hormone B. to steroid. is synthesized by liver. Type 2 D. 292 Harrison’s 18th Ed. 56 In heart failure. Type 4 Hydrostatic pressure within interstitial fluid is referred to as the tissue tension which promotes the movement of fluid into the vascular compartment. Blocking of epithelial sodium channels D. All of the above D. . Circulating catecholamines C. Prolongation of biologic half-life Harrison’s 17th Ed. This accumulation is self-limiting. Lymphatics C. 107 Cardiology MCQ’s FOR MCQ’s FOR MEDICAL MEDICAL PROFESSIONALS PROFESSIONALS BY PROF. Pro-hormone A. compensated heart failure. A. frequently accompanied by abdominal distention orthostatic retention of sodium & water after upright posture. Advanced heart failure D. Interleukin 2 A. Cerebellum A. Austin Flint n Harrison’s 18th Ed. Calcium emboli . Harrison’s 18th Ed. Calcium channel antagonists D. D. Clonidine the Cardiovascular System ti e D. D. A. None of the above 71 Which of the following retinal emboli is most common ? Idiopathic cyclic edema occurs almost exclusively in women. Trichinosis is a cause of FUO. Occurs in women with DM for >10 years B. All of the above B. 292 proliferative glomerulonephritis & acute pericarditis. Edema unrelated to menstrual cycle Early diabetic microaneurysms are found just temporal to fovea. All of the above B. Constrictive pericarditis BNP is stored in cardiac ventricular myocardium & is released when ventricular diastolic pressure rises. Its actions are similar to ANP. Janeway lesions C. 294 C. JVP is normal. Nikolsky’s sign B. 234 D. Hydralazine Chapter 227. Minoxidil V B. C. 293 Table 36-1 B. Subungual hemorrhages 9 Harrison’s 18th Ed. Platelet emboli B. Its actions are excretion of sodium & water by increasing GFR. AVP & sympathetic stimulation causing arteriolar & venous dilatation.108 MCQ’s FOR MEDICAL PROFESSIONALS BY PROF. Anabolic steroids ta Infective endocarditis may present as petechiae. In hepatic cirrhosis. Osler’s nodes and Janeway lesions. Yellow exudates 65 Which of the following is false about idiopathic edema ? Harrison’s 17th Ed. In the exfoliative stage. Circulating levels of ANP & BNP are elevated in CHF. Richard Cabot - C. 293 Table 36-1 A. Harrison’s 18th Ed. Trichinosis Polypeptide ANP is secreted by atrial myocytes secondary to atrial distention and/or sodium B. All of the above 60 Brain natriuretic peptide (BNP) is present in ? Apart from hypoproteinemia. Atenolol 62 Antihypertensive agents associated with edema formation 68 Stethoscope was first introduced by ? include all except ? A. Growth hormone Harrison’s 17th Ed. Allergic reactions load. causes of facial edema include trichinosis (Trichinella spiralis). constrictive pericarditis include all except ? R or tricuspid stenosis but is normal in cirrhosis. D. Myxedema actions of Angiotensin II. 293 Table 36-1 lateral pressure (Nikolsky’s sign). Cardiac ventricular myocardium B. Estrogens / Progestins Staphylococcal scalded-skin syndrome is seen in children and in immunocompromised adults. Generalized erythema is often evident during the prodrome of fever and malaise. the skin can be induced to form bullae with light Harrison’s 18th Ed. Glucocorticoids i C. Cerebral cortex 67 Venous pressure in upper extremities is elevated in all except ? Harrison’s 17th Ed. 293 Table 36-1 r B. ACE Inhibitors Stethoscope was first introduced by René Laennec in 1819. Tricuspid stenosis G D. along horizontal raphe & cotton- wool infarcts are found circularly around disc. René Laennec U B. Trichinosis can also lead to membranous glomerulopathy. Mineralocorticoids h D. 1382 all except ? A. Hollenhorst plaques 66 Causes of facial edema include ? Harrison’s 18th Ed. Nonproliferative retinopathy is found in almost all C. allergic reactions & myxedema. 1382 episodes of edema (unrelated to menstrual cycle). is characterized by periodic Harrison’s 17th Ed. Cyclosporine 70 Which of the following is false about diabetic retinopathy ? B. AJAY MATHUR Cardiology D. Alpha Adrenergic antagonists 69 Which of the following is not related to infective endocarditis ? 9 63 Steroid hormones associated with edema formation include Harrison’s 17th Ed. Osler’s nodes A. Nonproliferative retinopathy found in almost all individuals A. D. Cirrhosis liver 61 Antihypertensive agents associated with edema formation Venous pressure in upper extremities is elevated in advanced heart failure. Methyldopa C. A. 1382 C. profound tenderness 64 Which of the following is associated with edema formation ? of the skin is distinctive. Cotton-wool infarcts circularly around the disc C. Barlow A. 235 C. inhibiting sodium reabsorption in PCT & inhibiting release of renin & aldosterone. All of the above A. It also antagonizes vasoconstrictor C. Microaneurysms just temporal to fovea D. Occurs after upright posture individuals who have had DM for >20 years. Physical Examination of d C. Increased peripheral vascular resistance Harrison’s 17th Ed. fever. A. Calf pressure & increased peripheral vascular resistance. 1383 D. A. Cirrhosis liver 79 Pulsus parvus is found in which of the following conditions ? Harrison’s 17th Ed. First systolic peak in HOCM is called “tidal wave” A. thigh. C. Assessing chest expansion 72 When ascites is out of proportion to peripheral edema. peripheral AV fistula. Tidal wave is a smaller and slowly rising positive pulse wave produced by continued ventricular ejection and by reflected waves from the periphery.5 C. C. or foot. < 0. < 0. rest pain. Mitral stenosis buttock. anemia. ABI <0. Aortic valve stenosis 74 What resting ankle-brachial index is consistent with critical C. Decrease in peripheral vascular resistance 75 Which of the following statements is false ? Harrison’s 17th Ed. anacrotic shoulder is present on ascending limb. 1383 D. Uric acid emboli 78 “Trisection” is a method for ? Harrison’s 17th Ed. one each in systole & diastole A. B. Tricuspid regurgitation Trisection is a method for assessing the sharpness of upstroke. All of the above A. narrow pulse B. AJAY MATHUR Cardiology 109 D. Characteristic of hypertrophic cardiomyopathy (HOCM) 76 Pulse more evident in a peripheral artery is ? Harrison’s 17th Ed. calf. . 1383 Constrictive pericarditis is a possibility when ascites is out of proportion to peripheral edema. Pulsus bigeminus Bisferiens pulse has two systolic peaks is characteristic of AR (with or without stenosis) & HOCM. Peripherally. bounding or hyperkinetic pulse occurs in complete heart block. Assessing postural fall of blood pressure Harrison’s 17th Ed. anacrotic shoulder is less apparent & incisura is replaced by dicrotic notch. one each in systole & diastole. which B. Usually denotes a very low stroke volume B. 1383 D. C. Assessing pulse. Found in dilated cardiomyopathy D. Peripherally. All of the above A. Incisura coincides with the aortic valve closure regurgitation. Anacrotic shoulder is present on ascending limb Large. Buttock Pulsus parvus is seen in conditions with diminished left ventricular stroke volume.7 81 Bounding or hyperkinetic pulse is associated with ? Harrison’s 17th Ed. producing the “first systolic B. Found in restrictive cardiomyopathy C. and tissue loss. Descending A. In HOCM. 109 Cardiology MCQ’s FOR MCQ’s FOR MEDICAL MEDICAL PROFESSIONALS PROFESSIONALS BY PROF. 1383 A. Assessing character of arterial pulse of the following should be suspected ? C. < 0. B. Dicrotic pulse Dicrotic pulse has 2 palpable waves.3 Pulsus tardus is seen in aortic valve stenosis (delayed systolic peak due to obstruction to left ventricular ejection). Bisferiens pulse (in AR) & pulsus alternans are more evident in peripheral arteries. B. 1383 In normal central aortic pulse wave. rest pain & tissue loss ? D. 1383 Atherosclerosis of peripheral arteries of lower extremities produces intermittent claudication of A. exercise. Mitral regurgitation Harrison’s 17th Ed.9 is considered abnormal. Dicrotic pulse 83 Which of the following about dicrotic pulse is false ? Harrison’s 17th Ed. 1383 A. PDA. < 0. B. Aortic regurgitation ischemia. Has two systolic peaks limb has a sharp downward deflection called incisura coinciding with aortic valve closure. C. Pulsus bigeminus B. Constrictive pericarditis D. systolic peak and diastolic B. Increased left ventricular stroke volume Resting ABI < 0. Narrow pulse pressure intermittent claudication of ? C. Budd Chiari Syndrome slope of the arterial pulse. Characteristic of aortic regurgitation (AR) C. mitral regurgitation or ventricular septal defect & aortic B. None of the above Harrison’s 17th Ed. Any of the above Harrison’s 17th Ed. Thigh 80 Pulsus tardus is found in ? D.9 A. 1383 Platelet emboli in retinal artery is most common. 1382 D. BP and respiration A. Pulsus paradoxus D. Wide pulse pressure C. Pulsus alternans C. 1383 77 Pulse more evident in a peripheral artery is ? Harrison’s 17th Ed. 1382 D. Has 2 palpable waves. Pulsus paradoxus D. It denotes a very low stroke volume as in dilated cardiomyopathy. pulse wave upstroke rises rapidly and forcefully. anxiety.3 is consistent with critical ischemia. Diminished left ventricular stroke volume 73 Atherosclerosis of the peripheral arteries may produce B. Pulsus bisferiens peak called percussion wave”. incisura is replaced by dicrotic notch 82 Which of the following about bisferiens pulse is false ? D. Left atrium n C. rhythm is regular. D. v A. 30º. 1384 Harrison’s 17th Ed. edema involves right leg prior to left B. y . > 30 mm Hg with inspiration h D. 1384 inspiration. Indicates severe impairment of LV function B. Regular alteration of pressure pulse amplitude Harrison’s 17th Ed. It may occur during or following paroxysmal tachycardia or for several beats & estimation of CVP. a 9 Harrison’s 17th Ed. y B. All of the above increasing volume of blood in RA during ventricular systole. Right atrium A. Right ventricle B. Peripheral atherosclerosis is a risk factor for IHD inspiration ? Harrison’s 17th Ed. None of the above C. “y” descent (negative wave) is due to opening of tricuspid valve and rapid inflow of blood into RV. c Harrison’s 17th Ed. c and v positive waves Pulsus paradoxus is found in patients with pericardial tamponade. Edema is a late sign of heart failure A. c D. x r B. < 30º D. “x” descent (negative wave) is due to atrial relaxation and to downward displacement of tricuspid valve during ventricular systole. In PSVT D. a G B. Loud S3 is Right internal jugular vein is best for examination of neck veins for inspection of their waveform frequently heard. 1384 is inclined by ? A. “c” wave C. Normally. 1384 ti e Harrison’s 17th Ed. airway obstruction. All of the above U JVP reflects phasic pressure changes in right atrium and consists of a. “v” wave is a positive late systolic due to D. Aortic coarctation “a” wave is a positive presystolic dominant wave in JVP due to right atrial contraction. it involves the right leg prior to the left. 85 Pulsus bigeminus is found in ? 91 Which of the following waves in JVP is negative ? Harrison’s 17th Ed. May occur during or following paroxysmal tachycardia C. AJAY MATHUR Cardiology 84 Which of the following about pulsus alternans is false ? 90 Pulsation of the internal jugular vein is greatest when the trunk Harrison’s 17th Ed. > 20 mm Hg with inspiration i D. radial & femoral arterial pulses are virtually coincident except in aortic coarctation. It is due to alternating LV contractile force indicating severe impairment of LV function. but caused by premature ventricular contraction that follows each regular beat. c R D. 1305 95 Which of the following wave is dominant wave in JVP during A. Edema is a late sign of heart failure. 1384 B. 1384 B. 1384 A. 1491 B. 89 Which of the following statements is false ? Harrison’s 16th Ed.110 MCQ’s FOR MEDICAL PROFESSIONALS BY PROF. > 40 mm Hg with inspiration 94 Which of the following wave is produced at the time of right ventricular isovolumetric systole ? ta Pulsus paradoxus is defined as a decrease in systolic arterial pressure of >10 mm Hg with Harrison’s 17th Ed. x Peripheral atherosclerosis is an important risk factor for coincident ischemic heart disease. Airway obstruction C. a 88 Radial & femoral arterial pulses are not coincident in ? B. Aortic dissection D. Superior vena cava obstruction D. < 15º C. - 93 The wave produced due to atrial relaxation is ? 87 Pulsus paradoxus is defined as a decrease in systolic arterial Harrison’s 17th Ed. > 10 mm Hg with inspiration C. 1384 A. Pericardial tamponade B. A. < 60º In pulsus alternans. or and x and y negative waves. v V Regular alteration of pressure pulse amplitude occur in pulsus bigeminus. < 45º D. 1384 C. Left ventricle D. 1384 pressure of ? A. y C. y C. a C. 92 JVP waveforms reflect pressure changes in which of the d following cardiac chambers ? 86 Pulsus paradoxus is found in ? Harrison’s 17th Ed. HOCM is a positive wave due to bulging of tricuspid valve into RA during RV isovolumetric systole & by impact of carotid artery adjacent to jugular vein. Premature atrial contraction A. In ventricular tachycardia C. superior vena cava obstruction. v 9 A. Premature ventricular contraction B. Pulsation of internal jugular vein is greatest when trunk is inclined by < following a premature beat in patients without heart disease. but regular alteration of pressure pulse amplitude. In CHF. Irregular rhythm A. Caused by severe right-sided heart failure A venous pulse characterized by a sharp y descent. AV dissociation with VT 102 Center of right atrium lies ~5 cm. 111 Cardiology MCQ’s FOR MCQ’s FOR MEDICAL MEDICAL PROFESSIONALS PROFESSIONALS BY PROF. Harrison’s 17th Ed. 1306 A. Any of the above except ? Right internal jugular is the best vein for accurate estimation of CVP. Supine Large “a” waves occur during arrhythmias when RA contracts with tricuspid valve closed. Pulmonic stenosis B. When TR becomes B. Rapid ascent to baseline A. deep y descent in early diastole occurs with severe TR. Pulmonary hypertension A. which of the following is a typical feature of tricuspid right-sided heart failure. 1384 A. “a” wave is absent in atrial fibrillation due to ineffective contraction of atria. and a rapid ascent to the C. Frequent in left ventricular infarction a high venous pressure. Most common cause is right-sided heart failure 100 In JVP. combination of a prominent v wave & obliteration of x descent results in a single large positive systolic wave. A. B. AJAY MATHUR Cardiology 111 96 “Cannon” “a” waves may occur regularly in which of the D. Atrial fibrillation B. constrictive pericarditis and right ventricular infarction. Frequent in constrictive pericarditis baseline is seen in patients with constrictive pericarditis or with severe right-sided heart failure and D. Sharp “y” descent 106 Which of the following about Kussmaul’s sign is false ? B. 1384 angle is expressed in centimeters of blood. 1384 D. During junctional rhythm seen in Tricuspid regurgitation. (1. which of the following is seen in constrictive pericarditis ? Positive test is best defined as increase in JVP during 10 seconds of firm midabdominal compression followed by a rapid drop in pressure of 4 cm blood on release of the compression.36 cmH 2O = 1. stenosis ? Harrison’s 17th Ed. All of the above B. Prominent “v” wave & obliteration of “x” descent is A. In normal persons. All of the above A. Early systolic B. C. A. Tricuspid stenosis 103 CVP is normally expressed as ? Harrison’s 17th Ed. Rapid drop of 4 cm on release of compression D. C. A. Harrison’s 17th Ed. Firm pressure over midabdomen for >30 seconds Tricuspid regurgitation (TR) causes the v wave to be more prominent. 1384 below sternal angle “regardless of body position”. does not alter JVP significantly severe. deep “y” descent in early diastole Harrison’s 17th Ed. A rapid. Prominent “v” wave A. Centimeter of Mercury Large “a” waves indicate that RA is contracting against increased resistance as in TS. 1384 C. Deep “y” trough following conditions ? A slow “y” descent in JVP suggests an obstruction to right ventricular filling as occurs with Harrison’s 17th Ed. Elevated right atrial systolic pressure Increased delay occurs between “a” wave and carotid arterial pulse or “c” wave in patients with first-degree atrioventricular block. All of the above A. pulmonary hypertension or PS.0 mmHg). Millimeter of Mercury D. Deep “y” trough is seen in constrictive pericarditis. Kussmaul’s sign refers to an increase in CVP during inspiration. Elevated right ventricular systolic pressure 99 Which of the following is seen in tricuspid regurgitation ? Most common cause of a high jugular venous pressure is elevated right ventricular diastolic Harrison’s 17th Ed. C. Prominent “v” wave 105 Which of the following statements about abdominojugular test is false ? C. Millimeter of blood D. 45º recline 97 Large “a” waves may occur is all of the following conditions D. below sternal angle in which C. 1384 pressure. 1384 107 The normal left ventricular apex impulse is ? Harrison’s 16th Ed. 1384 C. “Cannon” “a” waves occur regularly during junctional rhythm and occur irregularly during AV B. Slow “y” descent B. Mid systolic C. 1384 D. 1384 tricuspid stenosis or right atrial myxoma. Elevated right ventricular diastolic pressure D. 2º heart block 104 Most common cause of a high jugular venous pressure is ? Harrison’s 17th Ed. Centimeter of blood 98 Cause of increased delay between “a” & “c” wave in JVP is ? Vertical distance between top of the oscillating jugular venous column and the level of sternal Harrison’s 17th Ed. Increase in CVP during inspiration D. Obliteration of “x” descent . Deep “y” trough Harrison’s 17th Ed. a deep y trough. Elevated right atrial diastolic pressure D. 1º heart block B. Center of RA lies ~5 cm. It’s causes include severe 101 In JVP. 3º heart block A. C. Complete heart block position of the patient ? Harrison’s 17th Ed. Rapid. 1384 C. All of the above B. 30º recline dissociation with VT or CHB. Obliteration of “x” descent B. dilation of the ascending aorta. This occurs in Pulsation of the right sternoclavicular joint may indicate a right-sided aortic arch or aneurysmal tachycardia. In MR. Position of mitral leaflets at onset of ventricular systole ta C. Reversed splitting of S1. This pulsation occurs distinctly D. 1306 A. Left midprecordium Harrison’s 16th Ed.112 MCQ’s FOR MEDICAL PROFESSIONALS BY PROF. while thrill of pulmonic stenosis radiates more to left side of the neck. All of the above D. Harrison’s 17th Ed. V or VI ICS A. standing increases systolic murmur of HOCM. Associated with heart murmurs Best felt with fingertips. High-frequency vibrations C. Tachycardia A.5 cm Harrison’s 17th Ed. Resistance to right ventricular filling B. Bisferiens pulse C. Pulmonary artery pulsation C. The farther apart they are at this time. expanding left atrium. Sustained handgrip exercise. III or IV ICS 114 Which of the following is not true about ‘Thrills’ ? B. MS and short PR interval. Frequent in patients with severe mitral regurgitation h Harrison’s 17th Ed. Any of the above D. 1385 A. 108 Normal location of left ventricular apex impulse is ? squatting diminishes & Valsalva maneuver. Left atrial myxoma Abnormal precordial pulsations occur during systole in patients with left ventricular dyssynergy C. IV or V ICS Harrison’s 17th Ed. thrill of AS crosses palm toward right side of neck. 1307 U C. Structural disease of mitral valve “later” than the LV apical impulse. Due to anterior displacement of RV by expanding LA C. 119 Hangout time is related to ? 113 Which of the following is false about hypertrophic Harrison’s 17th Ed. 1385 B. Short PR interval C. 1385 cardiomyopathy ? Harrison’s 16th Ed. most commonly in left midprecordium one or two interspaces above and/or 1 to 2 cm 9 medial to left ventricular apex. Tricuspid regurgitation Loudness of S1 depends upon the position of mitral leaflets at the beginning of ventricular systole. it is felt in III & IV ICS near left sternal border. 1306 117 Intensity of the first heart sound is influenced by all except ? A. Better felt with the palm of hand Thrills are palpable “low-frequency” vibrations associated with heart murmurs. n A. Left bundle branch block due to IHD. Resistance to pulmonary vascular bed A. Synchronous with ‘v’ wave in left atrial pressure curve A.5 cm. thrill is 109 Normal diameter of left ventricular apex impulse is about ? felt at the cardiac apex. 118 S1 is louder in which of the following conditions ? Harrison’s 17th Ed. A. louder will be the sound of S1. Palpable D.0 cm A. Mitral stenosis B. Any of the above B. Sustained handgrip exercise diminishes systolic murmur D. Apex 116 Reversed splitting of S1 may be present in patients with ? B. Aneurysmal dilation of ascending aorta B. i Harrison’s 16th Ed. Late systolic C. 5 to 10 milliseconds B. 1385 V D. 1385 C. Resistance to left ventricular outflow . D. 10 to 30 milliseconds ti e 110 Abnormal systolic precordial pulsations in patients with left D. Left lower sternal border B. two high-pitched components of S1 are split by 10 . Tricuspid stenosis D. 1385 112 Pulsation of right sternoclavicular joint may indicate ? Harrison’s 16th Ed.0 cm 115 Normal splitting of two components of S1 is by ? C. Left ventricular end diastolic pressure Typical left parasternal lift is seen in severe mitral regurgitation. It is due to anterior displacement of right ventricle by an enlarged. left atrial myxoma r false ? and left bundle branch block.30 milliseconds. D.5 cm R B. is synchronous with the v wave in left atrial pressure curve. C. None of the above Bisferiens pulse has 2 systolic peaks & is characteristic of HOCM. 1385 of S1 is mitral valve closure and the second is due to tricuspid valve closure. Rate of rise of left ventricular pressure pulse D. while in VSD. where 111 Which of the following statements about left parasternal lift is the mitral sound follows tricuspid sound. All of the above 9 Widening of S1 is due to complete right bundle branch block. AJAY MATHUR Cardiology C. normal left ventricular apex impulse is located at or medial to left midclavicular line in 4th or 5th intercostal space with a tapping character & early systolic timing. Double systolic apical impulse B. 1306 A. 3. is heard in severe mitral stenosis. 2. 25 to 40 milliseconds ventricular dyssynergy are most commonly felt in ? Normally. Base of the heart A. First component Harrison’s 17th Ed. 8 to 15 milliseconds d Early systolic outward thrust of left ventricular apex impulse is localized & the usual diameter is less than 2. When palm G Harrison’s 16th Ed. Severe mitral stenosis - D. 4. Occurs simultaneously with LV apical impulse B. 1306 of hand is placed over precordium. 3. 12 seconds A2-OS interval is inversely related to height of mean LA pressure & ranges from 0. AJAY MATHUR Cardiology 113 D. 1386 B. 0. 0. Resistance to left ventricular filling D. 0. hypertension except in patients with ? Harrison’s 17th Ed. MS P2 is coincident with incisura of PA pressure curve. 0. Pulmonic area A. They are best heard along the lower left sternal border & at left ventricular apex. It is 124 Reversed (paradoxic) splitting of S2 is seen in ? generally heard best at the lower left sternal border and radiates well to the base of the heart. 0. Tetralogy of Fallot D. As the severity of pulmonary hypertension Pulmonary ejection sound is loudest at the upper left sternal border and unlike most other right- increases. Nonejection or midsystolic clicks are due to prolapse of mitral valve leaflets. 1385 B. increased to decreased. Opening snap B.0. May occur at any time in systole best at the pulmonic area. 1385 Harrison’s 17th Ed. 126 Which of the following right-sided acoustical event is heard 120 In pulmonary hypertension. Harrison’s 16th Ed. VSD A. Tricuspid area C. 1385 A.04 . which is separated from RV pressure tracing P2 is normally softer than A2 in 2nd left intercostal space. Constrictive pericarditis B. 1385 A. in presence of semilunar valve stenosis & with dilation of aorta or pulmonary artery.10 seconds In reversed or paradoxical splitting of S2. Splitting is maximal in expiration and decreases during inspiration. Heard in mitral valve prolapse C. Mitral regurgitation 129 What is the character of opening snap (OS) ? Harrison’s 17th Ed.06 seconds C. Normal B. VSD B. Early systole B. Apex Harrison’s 17th Ed. closely following S1. 122 Abnormal splitting of S2 is best heard in which area ? 128 Which of the following statements about “nonejection clicks” Harrison’s 17th Ed. P2 louder than A2 suggests pulmonary seconds. Any of the above Due to volume and duration of right ventricular ejection remaining the same during inspiration Ejection sound is a sharp. Any of the above D. Any of the above Opening snap (OS) is a high-pitched. is heard better during expiration. Pulmonary ejection sound C. VSD Harrison’s 17th Ed. 1308 A. high-pitched event in early systole. Late systole D. They occur and expiration. D.08 to 0. PDA C.12 125 On cardiac auscultation. Events that delay closure of aortic valves paradoxical S2 splitting. except in patients with atrial septal defect. Left bundle branch block 130 The normal A2-OS interval is ? Harrison’s 17th Ed.04 to 0.08 seconds C. MR B. 1307 A. there is little inspiratory exaggeration of the splitting of S2 in ASD. All of the above C. P2 that is greater than A2 in this area by an interval called hangout time. Accentuated C. Aortic 1 area B. Absolute value of this interval reflects the resistance to suggests pulmonary hypertension. occurs in ? multiple & may occur at any time in systole but usually later than systolic ejection sound. pulmonary vascular bed.04 to 0. May be single or multiple Splitting of S2 that persists with expiration when the patient is in the upright position is heard D. sided acoustical events. 0. Low-pitched C. Harrison’s 17th Ed. ASD A. Diminished A. ventricular septal defect and constrictive pericarditis. the splitting of S2 may vary from normal.12 seconds . 1385 Harrison’s 16th Ed. S4 In pulmonary hypertension. Severe aortic outflow obstruction B. High-pitched D.04 to 0. ASD A. P2 component of S2 is loud. splitting of S2 may be ? better during expiration ? Harrison’s 17th Ed. D. 1386 B. 121 Fixed splitting of second heart sound diagnostic of ? 127 The timing of ejection sounds is ? Harrison’s 17th Ed. Right ventricular ectopic beat A. They may be single or 123 Early aortic valve closure (A2).08 seconds D. S3 D. 1386 B. 1385 is false ? A. 113 Cardiology MCQ’s FOR MCQ’s FOR MEDICAL MEDICAL PROFESSIONALS PROFESSIONALS BY PROF. P2 precedes A2. Heard best in pulmonary area D. Variable-pitched Aortic valve closes early in mitral regurgitation. The S2 splitting persists during expiration. 1385 131 What duration after A2 is the third heart sound (S3) heard ? A. All of the above C.04 to 0. early diastolic sound.04 to 0. Mid systole C. usually due to mitral stenosis. D. Fourth heart sound (S4) A. S3 soften & may disappear. Harrison’s 17th Ed. S1 D. murmur of HCM becomes louder h Harrison’s 16th Ed. 139 Murmur of HOCM becomes louder with ? S3 usually indicates impairment of ventricular function. I 132 S3 is normally heard in ? B. such as a reduction in preload or afterload (Valsalva. Murmur is produced by LV outflow obstruction & MR B. Benign grade 2 murmur 137 Loudness of cardiac murmurs is graded from ? B. A. HCM murmur becomes louder except ? C.16 seconds after A2 at the termination Harrison’s 17th Ed.14 . HOCM. but those of HOCM & MVP becomes louder. S2 - In Valsalva maneuver. Hypertrophic cardiomyopathy 143 Which of the following about Still’s murmur ? D. Opening snap (OS) 142 Which of the following about HOCM is false ? C. Maneuvers that increase preload (squatting. Pregnancy Loudness of cardiac murmurs is graded from I to VI. Passive leg raising A. 1386 subclavian & carotid arteries. 0. d With squatting & passive leg raising. 0 to V D. Squatting R Harrison’s 16th Ed. II Harrison’s 17th Ed. At lower left sternal border A. Exercise B. most murmurs decrease in intensity. None of the above ta B. 1386 G A. most cardiac murmurs diminish. S3 becomes louder. All of the above S3 is frequent in normal children & in patients with high cardiac output. C. vibratory mid-systolic murmur at the lower left sternal border in normal children and adolescents. IV B. Passive leg raising A. With standing. 1386 C. ti e D. murmur of HCM decreases ir 135 All of the following are heard best at the left ventricular apex B. Murmur increases with reduction in preload / afterload D. grade IV murmur is accompanied by a thrill while grade VI murmur is audible with the stethoscope removed from contact with the chest. Standing 133 Which of the following may radiate to subclavian and carotid arteries ? B. 1 to V . 1308 C. C. Third heart sound (S3) Harrison’s 17th Ed. 1 to VI D. Squatting Harrison’s 17th Ed. Grade I murmur is heard only with special C. 1386 D. OS is heard best at the lower left sternal border and radiates to the base of the heart. In elderly B. most murmurs become louder. Anemia effort.12 seconds after A2) & higher-pitched than normal occurs in patients Harrison’s 17th Ed. AJAY MATHUR Cardiology C. 1386 9 with constrictive pericarditis and is called pericardial knock. S2 With standing. With exercise. S4 9 141 Which of the following about murmur of HCM is false ? S3 that is earlier (0. IHD and acute MR. C. AS. 1386 S 4 is frequent in systemic hypertension. A. B. 1308 A.16 seconds 138 If thrill is palpable.14 to 0. 1386 U C. Aortic ejection sound D. Murmur of HOCM decreases with exercise. Aortic stenosis vasopressors) or that reduce contractility (beta blockers) decrease intensity of murmur. A. 1386 Murmur of HOCM increase in intensity with maneuvers that result in increasing degrees of outflow tract obstruction. Systemic hypertension vasodilators) or to an augmentation of contractility (inotropic stimulation). D. III A. All of the above Harrison’s 17th Ed. OS 140 Murmur of HOCM becomes louder with ? Both left-sided S3 & S4 sounds increase with isometric exercise and both may radiate to the Harrison’s 17th Ed. standing. and S4 are heard best at the left ventricular apex.10 . 1386 of rapid filling. S1 D. passive leg raising. 0. 0 to VI Still’s murmur refers to a benign grade 2.0. Murmur intensity does not exceed grade 3 Aortic ejection sound. S3. A. In normal children D. Murmur decreases with augmentation of contractility 136 S4 is frequently present in ? Harrison’s 17th Ed. volume administration) or afterload (squatting.14 seconds D. 0. Systolic murmur of HOCM & MVP C. In individuals >40 years.0. but those of HOCM & MVP C. cardiac murmur is of which grade ? S3 is a low-pitched sound produced in ventricle. A. 1386 C. Valsalva maneuver n 134 Pericardial knock is a variant of ? B. All of the above V B. Vibratory mid-systolic murmur Harrison’s 17th Ed.12 to 0. With Valsalva maneuver.114 MCQ’s FOR MEDICAL PROFESSIONALS BY PROF. Graham Steell D. 1386 C. Maladie de Roger describes patients Scottish cardiologist Graham Steell was an avid horseman and iconoclast. Henri-Louis Roger B. Left lower sternal border C. Graham Steell murmur D. None of the above C. restrictive VSDs (maladie de Roger) produces a very loud murmur due to significant & sustained systolic pressure gradient between LV & RV.363:22 B. Anxiety increases. Left lower sternal border D. 1386. Aortic regurgitation 156 Acute.363:22 A. Although the murmur is audible over the entire base of the heart. George Frederic Still C. Begin with S1 and end after S2 N Engl J Med 2010. at the left interscapular region. or cranial to the scapular spine. Left interscapular region eponymous rheumatic disorders: a juvenile febrile arthritis and a more typhoidal illness in adults. Graham Steell B. Systolic ejection murmur with a musical quality A. decrescendo diastolic murmur running off of an accentuated second sound that mimics the murmur of aortic insufficiency. A. anxiety. Gibson’s continuous murmur may be audible from the back. Chronic MR A. with asymptomatic ventricular septal defects. Apex English physician George Frederic Still. ASD 150 Which of the following is best heard at left upper sternal border ? B. Endocarditis plateau in configuration ? Key . 1386 similar to the sound of a saw cutting through wood. Apex 155 Which of the following is true for holosystolic murmurs ? Harrison’s 17th Ed. George Gibson Small.363:22 147 Which of the following is false about Gibson’s murmur ? A. is called father of British pediatrics ? N Engl J Med 2010. Small.363:22 C. The murmur grows louder as the child ages and arterial dilation B. 152 Who. All of the above N Engl J Med 2010. Fever continuous. Harrison’s 17th Ed. Patients with symptomatic ventricular septal defects. Begin before S1 and end before S2 Graham Steell murmur is a soft. Left upper sternal border murmur with a musical quality. It increases in intensity with fever. and the area of maximal intensity may migrate farther left. it is heard at the left lower sternal border and apex. the father of British pediatrics. Gibson’s murmur is best heard at the left upper sternal border. which cause cyanosis and progressive pulmonary hypertension. N Engl J Med 2010. MVP N Engl J Med 2010. beginning after the first heart sound and extends through the second heart sound. All of the above 145 “Maladie de Roger” relates to ? Harrison’s 17th Ed. Grows louder as the child ages 154 Still’s murmur increases in intensity with ? D. Aortic stenosis B. restrictive VSDs D. have Eisenmenger’s syndrome. A.Hodgkin murmur is a diastolic murmur of aortic regurgitation. Gibson’s murmur Holosystolic murmur of chronic MR is high pitched & plateau in configuration because of wide C.Hodgkin murmur is best related to ? D. Left upper sternal border C. It may diminish during diastole. Still’s murmur is a benign. Papillary muscle rupture in acute myocardial infarction . D. George Gibson C. PDA B. TR. Its cause is unknown. blowing. Begin after S1 and end before S2 murmur is best heard in a localized area at the left upper sternal border. C. 1386 D. Left interscapular region A. Congenital AS C. Roger’s murmur difference between LV & LA pressure throughout systole. It has a raspy quality. medium-to-long systolic ejection A. Begin with S1 and end with S2 149 Key . Gibson murmur is A. B.363:22 Most often seen in children. AJAY MATHUR Cardiology 115 144 Which of the following produces a cardiac murmur that is D. Begins after the first heart sound D.363:22 George Gibson’s murmur is caused by a persistent patent ductus arteriosus. 153 Which of the following is false about Still’s murmur ? N Engl J Med 2010. Heard at the left lower sternal border and apex B. George Frederic Still A. both called Still’s disease. VSD B. from the following. 115 Cardiology MCQ’s FOR MCQ’s FOR MEDICAL MEDICAL PROFESSIONALS PROFESSIONALS BY PROF. The Graham Steell B.363:22 B. from the following.363:22 151 Who. ASD A. Exercise 148 Graham Steell murmur is best heard at ? D. Most often seen in children N Engl J Med 2010. Pericarditis A. or exercise.363:22 Holosystolic murmurs (MR. Henri-Louis Roger N Engl J Med 2010. C.363:22 146 Gibson’s murmur is best heard at ? A. None of the above N Engl J Med 2010. Caused by a persistent patent ductus arteriosus C. VSD) at the area of maximal intensity begin with S1 & end after S2. is best known for his D. was from Scotland ? N Engl J Med 2010. severe MR occurs in ? Harrison’s 17th Ed. severe MR from papillary muscle rupture accompanies B. with radiation into neck. C. small muscular VSD. Mitral valvulitis 159 Most benign. C. Left parasternal Harrison’s 17th Ed. Infective endocarditis D. AJAY MATHUR Cardiology B. Apical S4 169 Frequent cause of late systolic murmurs is ? D. occur across aortic or pulmonic outflow tracts. rupture of chordae tendineae in myxomatous mitral valve disease (MVP). Paradoxical splitting of S2 Late systolic murmurs high-pitched apical murmurs that start after S1 & do not mask S1 or S2. Late systole B. 1386 D. Holosystolic 165 Midsystolic murmur occuring in mitral regurgitation suggests ? Harrison’s 17th Ed. C. severe MR occurs include papillary muscle rupture complicating with disproportionate radiation into right carotid artery ? acute MI. Lateral MI Harrison’s 17th Ed. Early systolic murmurs D. A. Midsystolic originates in LV cavity & is maximal at lower left sternal edge & apex. Supravalvular aortic stenosis 158 Majority of heart murmurs are ? D. 1386 . apical S 4 and a late-peaking systolic murmur. 1387 Majority of heart murmurs are midsystolic and soft (grades I to II/VI). TR with pulmonary hypertension A. Valvular aortic stenosis lateral MI and occurs 2 . Murmur starts shortly after S1 and ends before the closure of aortic or pulmonic leaflets. Pulmonary outflow tract Midsystolic murmur can occur in mitral regurgitation resulting from papillary muscle dysfunction. Early systole Harrison’s 17th Ed. functional murmurs are ? A. C. Base 162 Auscultatory findings of severe AS include all except ? C. Small muscular VSD Harrison’s 17th Ed. Early systolic D. Any of the above A. paradoxical splitting of S 2. 1386 B. Early systolic AS. Late systolic murmurs pulmonary hypertension. D. Late systolic D. 1386 endocarditis & blunt chest wall trauma. functional murmurs originate from ? B. Mitral valve prolapse B. 1386 D. Midsystolic D. severe MR from papillary muscle rupture usually accompanies an inferior. Papillary muscle dysfunction Harrison’s 17th Ed. 1386 160 Most benign. Tricuspid valve 166 Early systolic murmurs end in ? Harrison’s 17th Ed. A. Apex B. or B. 1386 C. Midsystole A. posterior. Any of the above 168 Late systolic murmurs are best heard at ? Midsystolic murmurs are also called systolic ejection murmurs. Acute mitral regurgitation B.7 days after presentation. functional murmurs are midsystolic and originate from the pulmonary outflow tract. 1386 in shape. Presystolic C. Late systolic 167 Causes of early systolic murmurs include all except ? Harrison’s 17th Ed. In supravalvular A. Soft or absent A2 B. Examples include large VSD with C. murmur radiates disproportionately into right carotid artery. midsystolic murmur B. They are crescendo-decrescendo Harrison’s 17th Ed. Posterior MI 164 Which of the following lesions produce midsystolic murmur with little radiation into carotid arteries ? C. infective Harrison’s 17th Ed. Midsystolic murmurs Early systolic murmurs begin with S1 and end in midsystole. Aortic outflow tract D. C. In HOCM. with relatively little radiation to carotids. Hypertrophic cardiomyopathy 157 Acute. TR without pulmonary hypertension and acute MR. Supravalvular aortic stenosis Harrison’s 17th Ed. murmur is maximal in 2nd right ICS. Large VSD with pulmonary hypertension 161 Systolic ejection murmur is the name given to ? B. A. Rupture of chordae tendineae in MVP Auscultatory findings of severe AS include a soft or absent A 2. 1386 In valvular AS. A. Xiphoid A. Any of the above C. 1387 C. All of the above A. Inferior MI B. All of the above 163 Which of the following lesions produce midsystolic murmur Clinical settings in which acute. AV ring dilatation A. All of the above Harrison’s 17th Ed. Early-peaking systolic murmur Harrison’s 17th Ed. 1386 Most benign. Hypertrophic cardiomyopathy Acute.116 MCQ’s FOR MEDICAL PROFESSIONALS BY PROF. 1386 D. Valvular aortic stenosis which MI ? C. Mitral valve D. 117 Cardiology MCQ’s FOR MCQ’s FOR MEDICAL MEDICAL PROFESSIONALS PROFESSIONALS BY PROF. 1310 B. Carey Harrison’s 16th Ed. Dock’s murmur is a continuous diastolic murmur. 10 % A.363:22 A. low pitched murmur. 1310. 178 Middiastolic murmurs may be generated across mitral valve by all except ? 172 By Doppler studies. PDA or VSD. Ventricular septal defect D. Decrescendo D. and across the tricuspid valve in TR or ASD due to rapid flow across AV valve. which of the following is more reliable as an sits leaning forward & holds breath in full expiration. Best heard over left midsternal border 2D Doppler echocardiography is indicated in patients with loud systolic murmurs (grades >=III/VI). 1310 171 Which of the following murmurs is low-pitched ? Harrison’s 17th Ed. Mitral regurgitation C. Mitral stenosis D. Papillary muscle dysfunction D. A. Atrial septal defect C. Middiastolic murmur of mitral stenosis is low-pitched. 1310 detected in what percentage of normal individuals ? A. 10 % B. presystolic accentuation. the duration of murmur is more reliable than its intensity as an index of the severity of valve obstruction. AV ring dilatation Aortic regurgitation is encountered much less frequently in normal persons. trivial tricuspid regurgitation (TR) can be detected in what percentage of normal individuals ? 179 Which of the following statements about Carey-Coombs Harrison’s 16th Ed. but may follow an S3 gallop. Middiastolic murmur C. 45 % C. 4 cm left of the sternum in the third intercostal space. tricuspid regurgitation in ~ 70% and pulmonic regurgitation in ~ 88%. It is likened to that of the Harrison’s 16th Ed. Pulmonic regurgitation due to pulmonary hypertension In mitral stenosis. Character of murmur B. 88 % A. It may be in a sharply localized area. Tricuspid regurgitation . Stenosis of the left anterior descending artery Trivial mitral regurgitation can be detected by 2D echocardiography & Doppler studies in ~ 45% of normal individuals. decrescendo. Holosystolic aortic regurgitation ? Harrison’s 16th Ed. especially those that are holosystolic or late systolic & in those with diastolic or continuous murmurs. detectable only when the patient is sitting upright. All of the above Late systolic murmurs are frequently caused by papillary muscle dysfunction due to myocardial 176 2D Echocardiography is always recommended in an infarction or ischemia. 173 By Doppler studies. 1309 A. D. asymptomatic person with which of the following murmur ? Harrison’s 16th Ed. Pulmonic regurgitation C. trivial pulmonic regurgitation (PR) can be Carey-Coombs murmur is a soft. Mitral regurgitation arteries. Patent ductus arteriosus B. Duration of murmur C. and its incidence increases with advancing age. 1303 170 Which of the following is false about early diastolic murmur of A. 65 % N Engl J Med 2010. MR in MVPS produces late systolic murmurs following midsystolic clicks. trivial mitral regurgitation (MR) can be Harrison’s 16th Ed. with a presystolic peak. index of severity of valve obstruction ? Harrison’s 16th Ed. Mitral regurgitation Harrison’s 16th Ed. None of the above 174 By Doppler studies. Diastolic A. D. best heard over left midsternal border when patient 177 In mitral stenosis. 1309 murmur is false ? Harrison’s 16th Ed. Holosystolic murmur of MR is high pitched. heard best at the apex. N Engl J Med 2010. It detected in what percentage of normal individuals ? is heard in patients with acute rheumatic fever due to inflammation of mitral valve cusps. Heard in patients with acute rheumatic fever D. a pattern consistent with blood flow through the coronary A. All of the above C. 70 % D. B. middiastolic. Pericarditis D. or a loud first sound. Loudness of S1 A. Intensity of murmur D. Aortic regurgitation B. 25 % C. Continuous B. AJAY MATHUR Cardiology 117 A. which of the following is least detected in normal individuals ? Dock’s murmur is due to stenosis of left anterior descending artery. 1309 Coombs murmur does not have an opening snap. Endocarditis 175 By Doppler studies. 75 % Middiastolic murmurs are generated across mitral valve in MR. Soft B.363:22 D. 10 % B. Aortic regurgitation B. Low pitched C. 1309 bruits heard over stenosed renal and hepatic arteries. C. Rupture of sinus of Valsalva B. 1386 A. 25 % B. 22 % 180 Dock’s murmur is best related to ? C. Early diastolic murmurs of AR and PR due to pulmonary hypertension are high pitched. Mitral valvulitis C. 45 % D. Best heard with patient sitting & leaning forward Murmur of AR is high-pitched. VSD C. 191 Intensity of murmurs is described using which grading scale ? 185 Transient external compression of both arms to levels above A. IV/VI – loud with palpable thrill. Right ventricular sounds eliminate the diastolic portion of murmur It is important to remember to auscultate areas such as left axilla and back (MR). Ebstein’s anomaly (due to D. Heard best at left 2nd intercostal space in expiration D. The intensity of murmurs is described using the Levine grading scale. pitch. 1388 A. Mitral regurgitation A. Represents augmented venous flow through engorged breasts B. Becomes audible during the late pregnancy C. Low frequency sounds (S3. 183 All are true about cervical venous hum except ? Harrison’s 16th Ed. Lennaec peak SBP augments murmurs of ? B. MR) are best heard with the diaphragm. MS rumble Continuous murmur in the “back” may be present in coarctation of the aorta. All of the above Cervical venous hum is a continuous murmur audible over medial aspect of right supraclavicular fossa with patient upright. S4. S3 B. Result from rubbing of pericardium against pleura Crescendo presystolic murmur follows atrial contraction that fills the ventricules in sinus rhythm in AV valve stenosis. Ebstein’s anomaly A. Ventricular arrhythmias B. Leukemia C. 1311 189 A patient should be examined in the left lateral decubitus position to bring out findings at the apex of ? A. S4 C. It is louder during diastole and is abolished by digital compression of Patient should be examined in the left lateral decubitus position to bring out findings at the apex ipsilateral internal jugular vein. . systolic pressure augments murmurs of MR. subxiphoid D. whereas high frequency sounds (clicks. II/VI – soft but easily detected. and top of the head (if AR is suspected). MR D. V/VI – heard with edge of stethoscope head. OS and MS. All of the above C. MS rumble) are best heard with the bell applied lightly. a systolic pleuropericardial friction rub (Means-Lerman scratch) may be heard at left 2nd intercostal space during expiration due to hyperdynamic cardiac motion. OS C. Heard in second to sixth anterior intercostal spaces (RV sounds). Firm pressure with diaphragm of stethoscope can D. “Mammary souffle” is an innocent murmur heard over breasts during late pregnancy and in early postpartum period. AR Murmurs should be described by noting intensity. Aortic regurgitation B. 230 B. They also appear in leukemia & diabetes. Audible in healthy children and young adults A. Aortopulmonary septal defect B. 1307 C. AJAY MATHUR Cardiology 181 In sinus rhythm. MS D. III/VI – prominent. S3 B. Roth’s spots are white centered retinal hemorrhages pathognomonic for subacute bacterial endocarditis. Levine A. Diabetes mellitus Splitting of S1 may be normal or may be associated with RBBB. Abolished by compression over internal jugular vein C. Coarctation of aorta C. presystolic murmur is most typical of ? 187 Which of the following statements about ‘Means-Lerman Harrison’s 16th Ed. quality. supraclavicular (venous hums). It is most characteristic of tricuspid stenosis with sinus rhythm. Systolic scratchy sound C. Audible in right supraclavicular fossa B. SABE D. Mammary souffle A. 192 Splitting of S1 may be due to ? 186 Roth’s spots are a feature of ? A. VSD & AR. Opening snap D. Theodore Harrison’s 16th Ed. All of the above abnormality of the tricuspid valve). shape. All of the above special effort. OS. Aortic stenosis C. Mitral regurgitation A. or ventricular arrhythmias. Proximal coronary artery stenosis D. palpable thrill and VI/VI – heard with Transient external compression of both arms by bilateral cuff inflation to 20 mm Hg over peak stethoscope head removed from chest. It may be systolic or continuous. of S3. Tricuspid stenosis B. Systolic louder than diastolic component D. In hyperthyroid patients. RBBB Harrison’s 18th Ed.118 MCQ’s FOR MEDICAL PROFESSIONALS BY PROF. Aortic regurgitation Harrison’s 17th Ed. duration. and timing. 184 All of the following are true about mammary souffle except ? 190 Top of the head is auscultated for ? Harrison’s 17th Ed. McCuine B. Venous hum C. I/VI – heard only with D. 1500 B. Found in hypothyroidism D. 1311 stetoscope ? A. 182 All of the following cause continuous murmur over precordium except ? 188 Which of the following is best heard with the diaphragm of a Harrison’s 16th Ed. palpable thrill. 1310 scratch’ is false ? A. Phase 1 A. Hypercalcemia C. T wave & U or hypercalcemia. ST-T-U complex tend to increase QRS duration. Amiodarone A. T wave D. End of ST segment & beginning of T wave C. PSVT C. 1831 A. AV node and His-bundle to the onset of QRS. Cardiomyopathy B. PSVT Chapter 228. Phase 1 C. 1831 Harrison’s 16th Ed. onset of QRS is called ? Harrison’s 18th Ed. Differences in electrical potential between electrode & D. 1831 B. hypocalcemia) increase QT interval. Rapid upstroke (phase 0) of the action potential corresponds B. Wandering pacemaker 193 ECG leads displays ? 199 Atrial repolarization wave may become apparent in ? Harrison’s 18th Ed. Four phases D. AV node. 1831 194 Electric currents that spread through the heart can be produced A. Acute rheumatic fever electrodes B. Two phases B. 203 The action potential of the His-Purkinje system and ventricular 197 J point is the junction between ? myocardium has ? Harrison’s 18th Ed. 1333 A. Three phases C. Plateau (phase 2) corresponds to the isoelectric ST segment. maximum negative charge 200 Rapid upstroke of the action potential that corresponds to the ECG leads display instantaneous differences in potential between electrodes. His-bundle and fascicles D. but it may become apparent D. 1831 Depolarization of the heart is the initiating event for cardiac contraction. Action potential of the His-Purkinje system has five phases. Instantaneous differences in electrical potential between A. and heart muscle. End of T wave & beginning of U wave D. ST segment C. wave) represents ventricular repolarization. 1831 Harrison’s 18th Ed. 1333 A. Shortening of ventricular repolarization (phase 2) by digitalis administration QRS complex represents ventricular depolarization& ST-T-U complex (ST segment. Atrioventricular (AV) node The QRS-T waveforms of the surface ECG correspond with the different phases of simultaneously obtained ventricular action potentials. Phase 3 A. Phase 2 B. Phase 2 195 AV junction is constituted by ? Harrison’s 18th Ed. 1831 B. Heart muscle 201 Plateau of the action potential that corresponds to isoelectric D. Cardiac pacemaker cells C. 1831 Harrison’s 16th Ed. and active repolarization (phase 3) corresponds to the inscription of the T wave. Harrison’s 18th Ed. U wave Factors that decrease slope of phase 0 by impairing influx of Na + (hyperkalemia & flecainide) D. 1831 D. AV node. End of QRS complex & beginning of ST segment B. 198 Atrial repolarization wave may become apparent in ? 204 Resting membrane potential in ventricular myocardium is ? Harrison’s 18th Ed. Specialized conduction tissue D. Acute pericarditis A. The electric currents that spread through the heart are produced by cardiac pacemaker cells. Hyperkalemia Harrison’s 18th Ed. Phase 0 tissue. specialized conduction A. Digitalis B. End of P wave & beginning of QRS complex A. Phase 3 C. B. abbreviates ST segment. Phase 2 . C. The ECG records only the depolarization (stimulation) and repolarization (recovery) potentials generated by the atrial and ventricular myocardium. Atrial infarction maximum positive charge Atrial repolarization is usually too low in amplitude to be detected. Phase 1 B. Phase 0 by ? Harrison’s 18th Ed. Five phases J point is the junction between the end of QRS complex and the beginning of ST segment. All of the above ST segment is called ? Harrison’s 18th Ed. Cardiomyopathy B. 1831 196 Ventricular repolarization is represented by ? A. 119 Cardiology MCQ’s FOR MCQ’s FOR MEDICAL MEDICAL PROFESSIONALS PROFESSIONALS BY PROF. Differences in electrical potential between electrode and neutral C. AJAY MATHUR Cardiology 119 C. Electrocardiography D. fascicles & Purkinje fibres 202 Which of the following prolong phase 2 of the cardiac muscle action potential and increase the QT interval ? AV nodal region & His-bundle area together constitute AV junction. Conditions that prolong phase 2 (amiodarone. Differences in electrical potential between electrode & in acute pericarditis or atrial infarction. His-bundle. of depolarization spreads toward the positive pole of that lead. V5 = V ICS anterior axillary line. Tripolar leads D. Harrison’s 18th Ed. AJAY MATHUR Cardiology C. AV node C. V3 = midway between V2 and V4. It is the property of a cardiac cell that causes it to depolarize Harrison’s 18th Ed. Specialized fibers of His-Purkinje system D. 1389 A. Left arm-right arm voltages B. Unipolar leads B. Diagonal plane D. A. Diagonal plane D. Left leg-right arm voltages C. and aVF are called ? Harrison’s 17th Ed. Six extremity leads are 3 bipolar leads (I. All of the above ECG leads are configured so that a positive (upright) deflection is recorded in a lead if a wave PR interval measures the time (normally 120 . Some specialized atrial fibers IV and V intercostal spaces. 1832 215 ECG leads aVR. D. V6 = V ICS midaxillary line. V2 = IV ICS. Negative wave will be recorded B. 1312 C. Phase 4 211 Number of intercostal spaces used for placing unipolar chest Rapid depolarizing current (phase 0) is due to influx of Na + into myocardial cells followed by influx of Ca++. Unipolar leads B. Atrial musculature conduction B. and III are called ? Harrison’s 17th Ed. Lead III A. 80 mV Each lead axis is designated by their angular position relative to the positive pole of lead I (0°). 1832 A. Positive wave will be recorded A. 0° corresponds to which lead ? some specialized atrial fibers. Bipolar leads C. 4 C. C. 2 A. Vertical plane B. which of the following will happen ? 207 PR interval is related to ? Harrison’s 18th Ed. 1389 A. Vertical plane B. Multipolar leads 209 ECG chest leads record potentials transmitted onto ? Harrison’s 18th Ed. If the mean orientation of depolarization vector is at right angles to a given lead axis. aVL. 1334 B.200 ms) between atrial & ventricular depolarization. Posterior axillary line C. 208 Extremity ECG leads record potentials transmitted onto ? Harrison’s 18th Ed. Anterior axillary line A. 100 mV at right angles to a given lead axis. 1832 214 ECG leads I. 3 B. AV junction area D. II. Scapular line D. 210 ECG Lead V6 is placed at ? Harrison’s 18th Ed. Resting membrane potential is phase 4. Lead I 206 Polarized resting myocardial cells carry an electric charge on B. Lead II their surface of about ? Harrison’s 16th Ed. aVL& aVF) leads. Figure 228-4 spontaneously during phase 4 of the action potential leading to the generation of an impulse. wave spreads toward the negative pole. Sinus node C. a biphasic (equally positive & negative) deflection is recorded. Repolarization phases of action potential (phases 1 to 3) are due to outward flux leads in ECG are ? of K +. Frontal plane A. and a negative deflection if the which includes the physiologic delay imposed by stimulation of cells in the AV junction area. 1832 Harrison’s 18th Ed. Frontal plane A. Biphasic wave will be recorded C. 1 205 Automaticity is normally observed in ? Harrison’s 16th Ed. 90 mV 213 In ECG. Any of the above B. Phase 3 just to the left of sternum. Left leg-right leg voltages Positions of six unipolar chest leads are : V1 = IV ICS just to the right of sternum. 70 mV D. 1832 216 Bipolar lead I measures the difference in potential between ? Harrison’s 17th Ed. All of the above 212 In the hexaxial diagram representing frontal plane (limb or Automaticity is normally observed in sinus node.120 MCQ’s FOR MEDICAL PROFESSIONALS BY PROF. Multipolar leads Six extremity leads record potentials transmitted onto the frontal plane & the six chest leads record potentials transmitted onto the horizontal plane. specialized fibers of His-Purkinje system & extremity) leads. if the mean orientation of the depolarization vector is D. II& III) and 3 unipolar leads (aVR. 1389 A. Midaxillary line B. Each bipolar lead measures the difference in potential between electrodes at two extremities. 1832 A. Horizontal plane C. D. Horizontal plane C. Bipolar leads C. Flat wave will be recorded D. 1832. . Left leg-left arm voltages D. V4 = V ICS midclavicular line. Tripolar leads D. Upward and toward right Harrison’s 18th Ed. Left leg-right arm voltages C. The right leg electrode functions as a ground. 1834 Harrison’s 17th Ed. Added D. B. All of the above . + 100º A. 1833. the QRS axis ranges from –30º to +100º. normal P wave will be negative in aVR. 1389 C. Its first phase (vector 1) is depolarization B. Left leg-right leg voltages 225 Above what degree of QRS axis is called right axis deviation ? Each bipolar lead measures the difference in potential between electrodes at two extremities: Harrison’s 18th Ed. Rightward and posteriorly Harrison’s 17th Ed. – 20º B. dextrocardia. Left leg-right leg voltages 224 Above what degree of QRS axis is left axis deviation defined ? 218 Bipolar lead III measures the difference in potential between ? Harrison’s 18th Ed. Left pneumothorax Opposite will occur if an ectopic pacemaker is in the lower part of either atrium or in the AV C. and aVF = left leg. Leftward and anteriorly D. All of the above D. Procainamide A. QRS axis more negative than – 30º is C. 1389 A. All of the above C. Due to reversal of left and right arm electrodes 221 The normal atrial depolarization vector is oriented ? Harrison’s 18th Ed. Infarction of lateral wall of left ventricle B. 1833. Apparent 220 Electrical augmentation of unipolar potentials in unipolar 226 Left axis deviation occurs in ? Harrison’s 18th Ed. 121 Cardiology MCQ’s FOR MCQ’s FOR MEDICAL MEDICAL PROFESSIONALS PROFESSIONALS BY PROF. Disopyramide B. Rightward and anteriorly Harrison’s 18th Ed. Left arm-right arm voltages B. aVL = left arm. Downward and toward right 228 Right axis deviation (RAD) occurs in ? D. 1389 D. 1834 223 QRS vector 2 points towards ? A. A. Left leg-left arm voltages D. Left leg-left arm voltages to the more massive left ventricle. Since this vector points toward the negative pole of lead aVR. As a normal variant A. 50 % C. + 110º B. Augmented Normally. 75 % D. 1834 A. + 80º 219 In unipolar leads. 1834 lead I = left arm–right arm voltages. the lowercase “a” stands for ? B. Figure 228-6 reversal of left & right arm electrodes. indicating A. Right ventricular overload A. Leftward and anteriorly C. Second phase (vector 2) results from the simultaneous depolarization of right & left ventricles that points leftward & posteriorly due C. Left posterior hemiblock junction region. left pneumothorax or left posterior A. Upward and toward left D. infarction of lateral wall of left ventricle. while an axis more positive than + 100º is referred to as right axis deviation. Thus. and lead III = left leg–left arm. as a spurious finding due to Harrison’s 18th Ed. + 90º Harrison’s 17th Ed. Inferior myocardial infarction C. Quinidine Harrison’s 18th Ed. 1834 leads is by ? Harrison’s 17th Ed. D. Leftward and posteriorly fascicular block. Figure 228-6 B. or inferior myocardial infarction. Dextrocardia spread of depolarization from sinus node to right & then left atrial myocardium. – 30º C. Left arm-right arm voltages QRS complex represents depolarization of ventricles. left anterior fascicular hemiblock. AJAY MATHUR Cardiology 121 217 Bipolar lead II measures the difference in potential between ? C. aVR = right arm. D. 100 % Left axis deviation may occur as a normal variant but is more commonly associated with left ventricular hypertrophy. lead II = left leg–right arm. Leftward and posteriorly C. 1389 A. Left ventricular hypertrophy B. 1834 Normal atrial depolarization vector (P wave) is oriented downward & toward left. All of the above 222 QRS vector 1 points towards ? RAD may occur as a normal variant (in children & young adults). Rightward and anteriorly A. Arithmatic referred to as left axis deviation. Unipolar leads measure the voltage at one locus relative to an electrode (called central terminal or indifferent electrode) that has approximately 227 Right axis deviation occurs in ? zero potential. Harrison’s 18th Ed. Downward and toward left C. – 40º D. or in right ventricular overload (acute or chronic). Left leg-right arm voltages of interventricular septum from left to right & anteriorly. – 10º A. B. The lowercase “a” indicates that these unipolar potentials are electrically augmented by 50%. 25 % B. D. Rightward and posteriorly 229 An abnormal increase in U-wave amplitude is due to which of the following drugs ? D. 1832 B. . owing to hyperaeration of the lungs. Inversion of U wave in precordial leads is abnormal & may be a subtle sign of ischemia.5 mm 238 In ECG. 1834 D. Torsades de pointes (TDP) B. Sensitivity of conventional voltage criteria for LVH is decreased in obese persons & in smokers. Acute cor pulmonale due to pulmonary embolism Abnormal increase in U-wave amplitude is mostly due to hypokalemia or drugs like dofetilide. 1834 D. 1835 RVH due to a pressure load (PS. with or without increased precordial voltage. RaVF>=20 mm or R1 + SIII>=25 mm C. amiodarone. quinidine. ECG evidence for LVH is a major noninvasive marker of increased risk B. Sinus A. R1+S III >= 25 mm. B. PSVT Harrison’s 18th Ed. This is due to downward displacement of diaphragm and A. with right axis deviation. 1835 Ventricular “strain” pattern is due to repolarization abnormalities in hypertrophied muscle. >=2. >=1. None of the above Harrison’s 18th Ed. Q wave in lead III. R >= S wave in lead V1 RaVL >= 11 to 13 mm. Left ventricular hypertrophy 234 Which of the following is not a common feature in right C. Hyperaeration of lungs Very prominent U waves are a marker of increased susceptibility to TDP type of ventricular tachycardia. A. Atrial flutter following electrolyte disturbance ? D. Right axis deviation C. Sensitivity of conventional voltage criteria for LVH is decreased in thin persons 235 In acute cor pulmonale due to pulmonary embolism. which of the following is the most common arrhythmia ? D. 1834 D. PHT) is characterized by tall R wave in lead V1 (R > S wave). None of the above A. 1835 D.5 mm Harrison’s 18th Ed. 1834 Poor R-wave progression is noted in chronic cor pulmonale due to obstructive lung disease despite right ventricular hypertrophy.e. Any of the above A. B. 1834 S1Q3T3 pattern i. Hypokalemia 236 S1Q3T3 pattern is seen in ? Harrison’s 18th Ed. B.0 mm indicates ? D. Low-voltage complexes are commonly present. procainamide. including sudden cardiac death. T-wave inversion in (R) to midprecordial leads hypertrophy (LVH) is false ? Harrison’s 18th Ed. Hyperkalemia C. Biventricular hypertrophy hypertrophied muscle ? Harrison’s 18th Ed. Acute dissection of aorta to which of the following ? Harrison’s 18th Ed. >=1. A. Hyponatremia tachycardia is the most common arrhythmia. Atrial fibrillation or flutter may occur. Right ventricular hypertrophy B. Right ventricular hypertrophy with right ventricular volume overload (ostium secundum ASD) is commonly associated with incomplete or complete B. ECG evidence of LVH is a marker of sudden cardiac death Harrison’s 18th Ed. B. RaVL>=11 to 13 mm. 1834 Presence of left atrial abnormality increases the likelihood of underlying LVH. Repolarization & depolarization indicates ? D. A. T-wave inversion in lead III. 1834 Acute cor pulmonale due to pulmonary embolism may be associated with a normal ECG. Left atrial abnormality increases likelihood of underlying LVH also often present (ventricular strain pattern). Myocardial ischemia 232 P wave amplitude of how many millimeter suggests right atrial D.122 MCQ’s FOR MEDICAL PROFESSIONALS BY PROF. B. Incomplete or complete right bundle branch block pattern 240 Which of the following statements about left ventricular D. All of the above A.0 mm heart.5 mm). indicates left ventricular hypertrophy. Right bundle branch block C. disopyramide. 1835 Right atrial overload (acute or chronic) may lead to an increase in P wave amplitude (>=2. B. >=2. Atrial fibrillation of cardiovascular morbidity & mortality. is seen in acute cor pulmonale due to pulmonary embolism. RaVF >= 20 mm. Sinus tachycardia to incomplete or complete LBBB. LVH often progresses A. Severe mitral stenosis 231 Very prominent U waves are a marker of increased susceptibility D. AJAY MATHUR Cardiology 230 An abnormal increase in U-wave amplitude is due which of the C. C. None of the above A. ST depression. Repolarization 239 In ECG. Ventricular flutter to obstructive lung disease is due to ? Harrison’s 18th Ed. ST depression and T-wave inversion in right to midprecordial leads are A. Downward displacement of diaphragm and heart C. All of the above overload ? Harrison’s 18th Ed. AIVR 237 Poor R-wave progression seen in chronic cor pulmonale due C. Left ventricular hypertrophy 233 Ventricular “strain” pattern is due to which abnormality in C. Depolarization SV 1 + (RV 5 or RV 6) >= 35 mm or (RV5 or RV 6) >= 25 mm indicate LVH by voltage criteria. prominence of S wave in lead I. Biventricular hypertrophy ventricular hypertrophy due to pressure load ? Harrison’s 18th Ed. Right ventricular hypertrophy B. LVH often progresses to incomplete or complete LBBB right bundle branch block pattern with a rightward QRS axis. SV1 + (RV5 or RV6)>=35 mm or (RV5 or RV6)>=25 mm C. sotalol. Posteriorly and to left Intraventricular conduction delays can be caused by toxic factors that slow ventricular conduction In bundle branch blocks. AJAY MATHUR Cardiology 123 241 Which of the following about bundle branches is true ? LBBB is a marker of increased risk of cardiovascular morbidity & mortality (coronary heart Harrison’s 16th Ed. 1835 A. Cause a voltage gradient between normal & ischemic zones C. the major QRS vector is oriented ? Harrison’s 18th Ed. the terminal QRS vector is oriented ? Harrison’s 18th Ed. >= 120 ms in duration D. Anteriorly and to right B. Left & right bundle branches are broad sheets of fibers A. 1835 Harrison’s 18th Ed. >= 100 ms in duration B. Figure 228-12 Harrison’s 18th Ed. In RBBB. Anteriorly and to left C. Bundle branch block may be rate-related D. Relatively short PR interval B. Left is narrow & cable-like. Posteriorly and to right D. the QRS interval is between 100 and 120 ms. 1835 A. LBBB is more common myocardial cells than RBBB B. depolarization is delayed. Major QRS vector is toA form of ventricular preexcitation via a bypass tract. Stenosis of right coronary artery D. All of the above . 1835 A. Leads to flow of currents of injury D. AV Dissociation With complete bundle branch blocks the QRS interval is >=120 ms in duration. Posteriorly and to left Prolongation of QRS duration does not necessarily indicate a conduction delay but may be due LBBB alters both early & later phases of ventricular depolarization. aortic valve disease. Cardiomyopathy D. Anteriorly and to left C. RBBB is seen more commonly than LBBB. Bifascicular disease B. Normal early left to right pattern of septal activation is disrupted is diagnosed by a triad consisting of a wide QRS complex associated with a relatively short PR so septal depolarization proceeds from right to left. 1835 C. Stenosis of posterior descending coronary artery B. 1333 disease. Bundle branch blocks may be chronic or intermittent C. LBBB pattern is seen in electronic right ventricular pacing Severe. 249 Intraventricular conduction delays can be caused by ? 243 In RBBB. QRS vector is oriented in the direction of myocardial region where (hyperkalemia. Slurring of initial part of QRS (delta wave) C. Severe. D. In subjects without structural heart disease. Stenosis of left circumfles coronary artery C. right is a broad sheet of fibers 247 Examples of bifascicular block include ? B. Trifascicular disease C. predominantly negative interval and slurring of the initial part of the QRS (delta wave). Stenosis of left anterior descending coronary artery A. hypertensive heart disease. Wide QRS complex A. Left & right bundle branches are narrow & cable-like B. Ischemic heart disease B. tricyclic antidepressants. Complete heart block D. RBBB with LAHB Bundle of His gives rise to a broad sheet of fibers that course over the left side of interventricular C. the terminal QRS vector is oriented anteriorly and to the right (rSR´ in V1 & qRS in V6). preceded by a sharp spike. Complete LBBB septum to form left bundle branch & a narrow cable-like structure on the right side that forms the right bundle branch. acute ischemia lengthens duration of action potential B. LBBB has wide. 251 Which of the following about myocardial ischemia is false ? 245 Which of the following about bundle branch blocks is false ? Harrison’s 18th Ed. is seen in cases of electronic right ventricular pacing because of the relative delay in left ventricular activation. 1836 Harrison’s 18th Ed. Phenothiazines C. and cardiomyopathy). 1836 Harrison’s 18th Ed. phenothiazines). Wolff-Parkinson-White (WPW) syndrome directed to left & posteriorly. 1835 A. 123 Cardiology MCQ’s FOR MCQ’s FOR MEDICAL MEDICAL PROFESSIONALS PROFESSIONALS BY PROF. 250 Which of the following is diagnostic of Wolff-Parkinson-White syndrome ? 244 In LBBB. With incomplete Alternation of RBBB & LBBB is a sign of trifascicular disease. that of LBBB. Complex time-dependent effects on electrical properties of A. >= 90 ms in duration A. with the latter effect being due to (QS) complexes in lead V1 & entirely positive (R) complexes in lead V6. A pattern identical to aberrant activation of ventricular myocardium. All of the above D. Without structural heart disease. class 1 antiarrhythmic agents. Left is a broad sheet of fibers & right is narrow & cable-like Harrison’s 18th Ed. All of the above 242 With complete bundle branch blocks the QRS interval is ? 248 Alternation of right & left bundle branch block is a sign of ? Harrison’s 18th Ed. >= 110 ms in duration C. All of the above D. acute ischemia lowers the resting membrane potential & shortens duration of action potential. A. Anteriorly and to right B. Posteriorly and to right D. blocks. Severe aortic valve disease C. 1836. Tricyclic antidepressants B. RBBB and LPHB D. 252 Wellens T waves are usually associated with ? 246 Left bundle branch block is often a marker of ? Harrison’s 18th Ed. 1835 Harrison’s 18th Ed. 1835 A. Hyperkalemia A. B. Hypocalcemia D. Hyperkalemia particularly subarachnoid hemorrhage and are called as “CVA T-wave” pattern. Narrowing & peaking (tenting) of T waves Harrison’s 18th Ed. Left bundle branch block D. 1838. Hypothermia phenothiazines. Tricyclic antidepressant overdose electronic ventricular pacemaker patterns & WPW preexcitation. 1838 A. Brugada syndrome Systemic hypothermia also prolongs repolarization with a distinctive convex elevation of the J D. 1838 a high-grade stenosis of the left anterior descending coronary artery. Left ventricular volume overload (MR or AR) C. All of the above sinusoidal type of mechanism (“sine-wave” pattern) followed by asystole. This pattern referred to as Wellens T waves is usually associated with Harrison’s 18th Ed. WPW preexcitation 261 QRS & QT prolongation along with sinus tachycardia can be D. 1838. found in ? Harrison’s 18th Ed. Cerebrovascular injury B. 1838 or “hump”at the J point referred to as Osborn wave. PR-segment deviations D. Inferior MI A. Acute pericarditis C. D. All of the above B.124 MCQ’s FOR MEDICAL PROFESSIONALS BY PROF. Lateral MI Harrison’s 18th Ed. sotalol. 265 Abbreviation of ST segment and shortening of QT interval is hyperkalemia. 1837 Harrison’s 18th Ed. normal variants. Diminution in P-wave amplitude 253 Atrial infarction may be associated with ? B. Anterior MI by ? B. Heat stroke Drugs that prolong QT are quinidine. Depolarization Harrison’s 18th Ed. All of the above D. Table 228-1 D. Changes in P-wave morphology ECG changes in hyperkalemia usually begin with narrowing and peaking (tenting) of the T C. Myocarditis D. or atrial arrhythmias. Quinidine excess 255 Osborn wave in ECG is found in ? C. B. 1838 A. 1395 Table 221-2 254 Diagnostic changes of acute or evolving ischemia are masked A. Repolarization A. A. B. AV conduction disturbances B. Figure 228-16 Diagnostic changes of acute or evolving ischemia can be masked by presence of LBBB. 1838 257 ST-segment elevations simulating ischemia may occur with ? A. Hypercalcemia Marked QT prolongation. C. Marked QT prolongation potential characteristics. 1838. 258 Tall. Hyperkalemia A. Wide T-wave inversions A. Marked PR prolongation 256 ST-segment elevations simulating ischemia may occur with ? C. Widening of QRS interval A. diminution in P-wave amplitude. Further elevation of extracellular K + leads to AV conduction disturbances. Deep T-wave inversions Harrison’s 18th Ed. usually with a distinctive convex elevation Harrison’s 18th Ed. Depolarization + repolarization B. tricyclic antidepressants. Hypocalcemia . B. Table 228-1 D. Hyperkalemia A. All of the above seen in ? Harrison’s 18th Ed. Severe hyperkalemia causes a slow D. with deep & wide T-wave inversions occur with intracranial bleeds. 1839. Atrial infarction may be associated with PR-segment deviations due to an atrial current of injury. positive T waves may be seen in hyperacute ischemic changes. All of the above Tall. procainamide. Pneumothorax 262 “CVA T-wave” pattern consists of all except ? Systemic hypothermia also prolongs repolarization. Table 228-1 B. disopyramide. and widening of the QRS interval. Subarachnoid hemorrhage Harrison’s 18th Ed. amiodarone. Hypokalemia C. 1837 C. positive T waves are seen in ? 264 Which of the following prolongs the QT interval ? Harrison’s 18th Ed. Cerebrovascular accident tachycardia occur in tricyclic antidepressant overdose. 260 ECG changes in hypertrophic cardiomyopathy may simulate ? Harrison’s 17th Ed. Hypothermia 263 Systemic hypothermia prolongs ? D. QRS & QT prolongation along with sinus C. 1837 C. All of the above Harrison’s 18th Ed. Electronic ventricular pacemaker patterns C. It is due to altered ventricular action A. changes in P-wave morphology. None of the above C. All of the above point called Osborn wave. Hypothermia B. Atrial arrhythmias waves. ibutilide. cerebrovascular injury & left ventricular volume overload due to mitral or aortic regurgitation. A. AJAY MATHUR Cardiology Severe anterior wall ischemia (with or without infarction) may cause prominent T-wave inversions 259 The first ECG change in hyperkalemia is ? in the precordial leads. SLE of T-wave inversions in leads V 1 . 1321 D. Myocarditis turbulent blood flow ? C. 1838 B. 1396 ST–T-wave complex (digitalis effect). which colour represents D. Green A.V3 267 Q waves in ECG can be seen in all of the following except ? B. Myocardial ischemia 276 In color flow Doppler imaging of heart. hyperventilation. Long QT syndrome 274 In color flow Doppler imaging of heart. total electrical alternans (P-QRS-T) with sinus B. Digitalis D. Hyperkalemia 272 Chronic renal failure (CRF) is suspected if ECG shows ? Harrison’s 18th Ed. Long QT Harrison’s 18th Ed. 1839 D. Yellow Harrison’s 18th Ed. Mechanical instability of heart A. A. Blue Harrison’s 18th Ed. Disopyramide CRF can have the following ECG features as a triad .V4 Harrison’s 17th Ed. transient nonspecific repolarization changes occur following all except ? A. Parallel to the jet B. Hyperventilation 275 In color flow Doppler imaging of heart. D. often with a characteristic “scooping” of the Harrison’s 17th Ed. which colour represents D. T-wave inversions in leads V1 . Meals C. Left pneumothorax Imaging: Echocardiography. 1839 determine mean gradient across a stenotic cardiac valve ? Harrison’s 16th Ed. Peaked T waves 266 Which of the following shortens the QT interval ? B. Sarcoidosis D. Hyponatremia D. or exercise in healthy individuals. D. Ibutilide due to ST segment lengthening (hypocalcemia) & LVH (systemic hypertension). Red A. while hypercalcemia shortens it. Electrical instability of heart B. 1395 Table 221-2 C.V3. 1321 meal or with postural (orthostatic) change. Red tachycardia is a relatively specific sign of ? C. Postural (orthostatic) change D. Serious pulmonary disease C. long QT C. 1839 Hypocalcemia typically prolongs the QT interval (ST portion). C. Procainamide 273 “Persistent juvenile T-wave pattern” consists of ? Digitalis glycosides also shorten the QT interval. T-wave inversions in leads V1 . Blue 270 In ECG. Left ventricular hypertrophy (LVH) A. Chagas’ disease 268 Q waves in ECG can be seen in all of the following except ? Chapter 229. AJAY MATHUR Cardiology 125 B. which colour represents B. 1395 Table 221-2 A. Yellow tamponade due to periodic swinging motion of heart in effusion at a frequency exactly half of heart rate. Tangential to the jet D. Green C. Blue Total electrical alternans (P-QRS-T) with sinus tachycardia (beat-to-beat alternation in one or B. and MRI/CT Imaging C. A. All of the above .peaked T waves (hyperkalemia). 1838 C.V5 A. T-wave inversions in leads V1 . Hypercalcemia Repolarization (ST-T or U wave) alternans is a sign of electrical instability & may precede ventricular tachyarrhythmias. Nuclear B. Sleep blood flow away from the transducer ? Transient nonspecific nonspecific ST–T-wave repolarization changes may occur following a Harrison’s 16th Ed. Pneumothorax A. All of the above B. 125 Cardiology MCQ’s FOR MCQ’s FOR MEDICAL MEDICAL PROFESSIONALS PROFESSIONALS BY PROF. Yellow B. Noninvasive Cardiac Harrison’s 17th Ed. Pericardial effusion Harrison’s 16th Ed. Dextrocardia Cardiology. Green 271 Repolarization (ST-T or U wave) alternans is a sign of ? 277 How should a continuous-wave Doppler beam be placed to Harrison’s 18th Ed. Red more components of ECG) is a relatively specific sign of pericardial effusion with cardiac C. Scleroderma “Persistent juvenile T-wave pattern” a normal variant in healthy young adult woman consisting C. 1322 A. T-wave inversions in leads V1 . A.V6 B. Wolff-Parkinson-White syndrome blood flow towards the transducer ? Harrison’s 16th Ed. Serious hepatic disease D. Perpendicular to the jet C. 1321 269 In ECG. D. 1 in 5. 100 .10 mm Hg 283 Which of the following is recommended in those undergoing C. 60 . Right ventricular C.100 mm Hg Risk of death from elective cardiac catheterization is ~1 in 10.000 (0. 4 . 1 in 10. 1 .105 mm Hg D. 1406. Oral aspirin (325 mg/day) is given to patients undergoing diagnostic B. Left atrial B. 1322 Chronically anticoagulated patients with warfarin should discontinue it at least 48 hours prior to A. C. D. 70 .15 mm Hg B.20 mm Hg C. 130 .5 mm Hg . Antero-posterior projection D. 1405 A.3 mm Hg B. Adequate prehydration B.5 mm Hg agent.or bradyarrhythmias. indicators of myocardial C. 3 . 50 . 1405 D. 100 . 1 in 10. transient tachy. 2 . Warfarin 290 Normal value of “v” wave left atrial pressure is ? Harrison’s 17th Ed. 1 . All of the above 289 Normal value of “a” wave left atrial pressure is ? Harrison’s 17th Ed.15 mm Hg diagnostic catheterization for suspected coronary disease ? D. Table 223-1 B. AJAY MATHUR Cardiology 278 During a stress echocardiogram. Right anterior oblique projection C. 100 . left ventriculography is following pressures ? Harrison’s 17th Ed.or bradyarrhythmias from elective cardiac catheterization is about ? 287 Normal value of left ventriclar peak systolic / end-diastolic Harrison’s 17th Ed. Increase in end-systolic volume prophylactic antibiotics are not necessary. 1405 A.90 mmHg 280 Risk of death from elective cardiac catheterization is about ? Harrison’s 17th Ed. 1 in 100. 1406.000 B. Diagnostic Cardiac pressure is ? Harrison’s 17th Ed. 1406.000 D. preprocedure N-acetylcysteine. Table 223-1 Catheterization and Coronary A. 90 .000 C.126 MCQ’s FOR MEDICAL PROFESSIONALS BY PROF.25 mm Hg D.90 mmHg A.110 / 60 . 1406. Left anterior oblique projection B. 1 in 1000 286 Normal value of mean systemic arterial pressure is ? B. 1405 pressure is ? A. 288 Normal value of mean left atrial or pulmonary capillary wedge pressure is ? 282 Transient deterioration in renal function in patients Harrison’s 17th Ed. or bruising or bleeding at the catheter insertion site. 80 . B.90 mmHg Angiography B. All of the above 284 Pulmonary wedge pressure approximates which of the 279 To assess mitral and aortic valves. Table 223-1 C.5 mm Hg C. Table 223-1 A. or use of an isoosmolar contrast A. Use of isoosmolar contrast agent D. Table 223-1 Transient deterioration in renal function in patients undergoing cardiac catheterization is reduced by adequate prehydration.160 / 60 . Oral aspirin ischemia include ? D.000 Harrison’s 17th Ed. Postero-anterior projection Pulmonary wedge pressure approximates left atrial pressure. C.90 mm Hg D. All of the above Harrison’s 16th Ed. 6 . 1331 A.140 / 1 . Preprocedure N-acetylcysteine C.000 B. As cardiac catheterization is a sterile procedure.20 mm Hg Harrison’s 17th Ed.140 / 2 . 2 .140 / 0 . since aspirin pretreatment is desired if a coronary intervention is to be performed. Left ventricular D. 4 . transient tachy.000 A. 285 Normal value of systemic arterial peak systolic / end-diastolic Chapter 230. Right atrial A. New regional wall motion abnormality procedure (INR <2).12 mm Hg Elective cardiac catheterization carry a ~1 in 1000 risk of stroke or MI. 100 .140 / 3 . 1406.90 mmHg C.10 mm Hg A. 1 in 5.8 mm Hg D. 8 . 140 .160 / 60 . 1406 usually performed in which projection ? Harrison’s 16th Ed. 1406. 1 in 1000 Harrison’s 17th Ed. MI. Table 223-1 undergoing cardiac catheterization is reduced by ? Harrison’s 17th Ed.110 mm Hg 281 Risk of stroke.01%). 1 in 100. 100 . Prophylactic antibiotics A. Decline in ejection fraction catheterization for suspected coronary disease.140 / 60 . 2 . 1406.600 [(dyn•s)/cm5] C. 15 .1600 [(dyn•s)/cm5] D. Table 223-1 D. 127 Cardiology MCQ’s FOR MCQ’s FOR MEDICAL MEDICAL PROFESSIONALS PROFESSIONALS BY PROF. Table 223-1 B. 110 . Height A.250 [(dyn•s)/cm5] pressure is ? Harrison’s 17th Ed.10 mm Hg by ? C.2 [(L/min)/m2] D.600 [(dyn•s)/cm5] A. 25 .15 mm Hg the following formula ? D.8 / 4 . 2 . Body surface area B. 80 (MAP .22 mm Hg B.12 mmHg 299 Normal value of cardiac index is ? B.50 (mL/L) D.20 / 4 . Systemic vascular resistance Harrison’s 17th Ed. 4 .20 mm Hg A.150 [(L/min)/m2] 293 Normal value of right ventriclar peak systolic / end-diastolic pressure is ? D. 2 . 2 . 1 .8 mmHg A. 1406 A. 1 . 1406.50 [(dyn•s)/cm5] D. 1406. 100 . Mean aortic pressure .12 mmHg Harrison’s 17th Ed. 15 . 1406.8 mm Hg B. 10 .RA) / SBF C. 1406.5 mm Hg 303 For calculating cardiac index (CI). 3 .40 / 4 .6 . 5 .2 [(L/min)/m2] 292 Normal value of mean pulmonary artery pressure is ? Harrison’s 17th Ed. 50 (MAP .6 .6 . Weight B.20 mm Hg B.6 . 20 . 1406 B. Body mass index 296 Normal value of “v” wave right atrial pressure is ? Harrison’s 17th Ed. 1406 D. 20 . 1400 . 150 . 10 .40 / 2 .15 mm Hg Harrison’s 17th Ed.8 mmHg B.4.RA) / SBF Harrison’s 17th Ed. 5 . 3 . Table 223-1 301 Normal value of arteriovenous oxygen difference is ? A. 1000 . 1406.15 mm Hg A.4. 2 .10 mm Hg 298 Normal value of pulmonary vascular resistance is ? Harrison’s 17th Ed.4 mm Hg Harrison’s 17th Ed. 4 . Arteriovenous oxygen difference 295 Normal value of “a” wave right atrial pressure is ? D.20 mm Hg Harrison’s 17th Ed. Table 223-1 C.8 mmHg Harrison’s 17th Ed. Oxygen consumption index C.2 [(L/min)/m2] A. Cardiac output D. 70 (MAP . 2. AJAY MATHUR Cardiology 127 B.30 (mL/L) C.50 / 2 . 11 . 80 .12 mmHg B. 1406. 10 . 4 . Table 223-1 C. Table 223-1 C. 50 .2000 [(dyn•s)/cm5] 305 Which of the following variables is not included in calculating D. 3.20 / 2 .2600 [(dyn•s)/cm ] 5 systemic vascular resistance (SVR) ? Harrison’s 17th Ed.RA) / SBF 297 Normal value of systemic vascular resistance is ? B.6 mm Hg A. 1406. 0. 50 . 7 . 1406.5 mm Hg D. cardiac output (Q) is divided B. Table 223-1 C. 30 .4.4. 300 . Table 223-1 D. 1406.30 / 4 .110 (mL/L) Harrison’s 17th Ed.100 [(L/min)/m2] C.12 mmHg A. 5 . A.10 mm Hg C. 2 .RA) / SBF B. 700 .10 mm Hg 304 Systemic vascular resistance (SVR) is calculated by which of C.12 mm Hg 300 Normal value of oxygen consumption index is ? B. 1406. 20 . 1. 3 . 9 . Table 223-1 Cardiac output is estimated by Fick principle or by thermodilution method using thermistor on catheter.30 / 2 . 60 (MAP . 50 . Table 223-1 302 Fick principle is used to measure ? A. 1406.2 [(L/min)/m2] C.80 (mL/L) 294 Normal value of mean right atrial pressure is ? D.250 [(L/min)/m2] Harrison’s 17th Ed.18 mm Hg A.15 mm Hg Harrison’s 17th Ed. 10 .8 mmHg C.130 [(dyn•s)/cm5] 291 Normal value of pulmonary artery peak systolic / end-diastolic C.50 [(L/min)/m2] D. Table 223-1 A. 2 . Table 223-1 A. during diastole. Einthoven Harrison’s 17th Ed. Left atrium B. Body surface ECG is the timed sum of cellular action Harrison’s 16th Ed. Oxygen consumption index A. Systemic blood flow C. normal value for left ventricular end-systolic volume A. resulting in the P wave of the ECG. left ventricular pressure should be equal to that in ? Chapter 231. 1331 potentials in atria & ventricles A.1600 ms . 52 ± 15 mL/m2 SA node is composed of a cluster of small fusiform cells located in sulcus terminalis on epicardial surface of heart at right atrial . 62 ± 15 mL/m2 D. 20 ± 8 mL/m2 Electrical impulse from SA node is transmitted slowly through nodal tissue to atria. Right ventricle 314 Electrocardiogram (ECG) was developed by ? C. QT interval is duration of activation & recovery of ventricles 312 Assessment of hypertrophic cardiomyopathy during left Repolarization occurs first on the epicardial surface. Antero-posterior C. Systemic blood flow 313 Assessment of ventricular septal defect during left 306 Gorlin formula is used to calculate which of the following ? ventriculography done in which projection ? Harrison’s 16th Ed. White A. Right ventricle C. Pulmonary artery A. Postero-anterior D. AJAY MATHUR Cardiology B. 1406 A. 20 . 1860 310 In adults. Both atria + Atrioventricular node (AVN) A. Antero-posterior C. 1406 Electrophysiology A. Right anterior oblique B. 72 ± 15 mL/m2 316 PR interval of ECG represents time needed for activation of ? Harrison’s 18th Ed. Antero-posterior D. Pulmonary artery A. 1860 D. Principles of Harrison’s 17th Ed. Valve orifice area D. B.superior vena caval junction where they envelop SA nodal artery. then proceeds to the endocardium. Aorta Harrison’s 18th Ed. left atrial (pulmonary capillary wedge) pressure should be equal to that in ? C. and rapidly C. D.40 ms B. during systole. Aorta 315 Sinoatrial (SA) node is located at the junction of ? Harrison’s 18th Ed. Hewlet B. Atrioventricular node (AVN) Harrison’s 16th Ed. 80 . Repolarization occurs first on endocardium then proceeds B. Postero-anterior 307 In a normal heart. Postero-anterior D. 1331 C. 10 ± 8 mL/m2 D. Left atrial mean pressure D. Table 223-1 D. 1331 318 Duration of action potential in cardiac myocytes is ? Harrison’s 18th Ed. Both atria is ? B. Right atrial pressure C. 1406 D. Right atrium and superior vena cava 309 In adults. Right atrium and coronary sinus A. ventriculography done in which projection ? Harrison’s 16th Ed. Right atrium and left atrium Harrison’s 17th Ed. 1860 ventriculography done in which projection ? A. 42 ± 15 mL/m2 B. Right atrium and inferior vena cava is ? C. C. normal value for left ventricular end-diastolic volume B. 1331 Harrison’s 17th Ed. 1860 D. Left anterior oblique A. Right anterior oblique B. Cardiac output B. Left anterior oblique to epicardium C. Judkin 308 In a normal heart.128 MCQ’s FOR MEDICAL PROFESSIONALS BY PROF. 1406. Activation of atria & AV node is PR interval D. 40 ± 8 mL/m 2 317 Which of the following statements is false ? 311 Assessment of mitral and aortic valves during left Harrison’s 18th Ed. 30 ± 8 mL/m2 on to atrioventricular node (AVN). Left anterior oblique C. 1860 A. Both atria + AVN + His Purkinje system B. Left ventricle ECG was developed by Einthoven at the turn of the twentieth century. Right anterior oblique B. Pumps. spontaneous diastolic depolarization) is characteristic of sinoatrial (SA) & atrioventricular (AV) nodes. 1860 A.400 ms) compared to neurons or skeletal muscle cells (1 . Phases 0 and 1 of the action potential 321 Which of the following gene is responsible for ventricular B.800 ms Inactivation of the calcium current with persistent activation of potassium currents (predominantly I Kr and I Ks) causes phase 3 repolarization. phase 1 is early repolarization. 1862 Harrison’s 18th Ed. 1861 Figure 231-1 D. IKs 329 Cellular feature common to induction of DADs is the presence of ? 323 Which of the following determines the resting membrane Harrison’s 18th Ed. IK1 Afterdepolarizations are spontaneous depolarizations due to membrane voltage oscillations in B. Phases 1 and 2 of the action potential repolarization ? C. Phase 2 Property of automaticity (pacemaking i. 2 B. 1861 pores (ion channels). interrupting repolarization. Early afterdepolarizations (EAD) occur before the end of the action potential C. Ito cardiac myocytes. His-Purkinje system 320 Diastole corresponds to which phase of action potentials ? Harrison’s 18th Ed. CACNA1C A. Ion channels B. 1 A. 200 . All of the above Sodium-calcium exchanger transports three Na + for one Ca 2+. A. A. Figure 231-3 repolarization. All of the above B. Phase 2 of action potential 322 Which of the following is principal current during phase 4 ? C.e. Coronary sinus A. Myocardial ischemia C. Potassium current (I K1 ) is the principal current during phase 4 and determines the resting membrane potential of the myocyte. 3 C. 1861 Figure 231-1 B. 1860 potential ? Harrison’s 18th Ed. Delayed afterdepolarizations (DAD) occur during phase 4 of the action potential after completion of repolarization. 1861 Figure 231-1 D. 1862 potential of the myocyte ? A. Persistent activation of IKr C. transporters C. IKr The cellular feature common to induction of DADs is the presence of an increased Ca 2+ load in D. and phase 4 is diastole. transporters).5 ms). Phase 3 veins. Persistent activation of IKs D.400 ms D. Phase 3 of action potential Harrison’s 18th Ed. Phase 1 of action potential D. Catecholamines B. 1863. IK1 C. phase 2 is plateau. Increased Mg 2+ load in the cytosol C. phase 3 is late Harrison’s 18th Ed. SCN5A 328 Delayed afterdepolarizations occur after completion of ? Harrison’s 18th Ed. Phases 3 and 4 of the action potential A. Figure 231-3 C. D. actively transport ions against their electrochemical gradient (pumps. Increased Ca 2+ load in the cytosol Harrison’s 18th Ed. Phase 4 of action potential A. Digitalis glycoside toxicity A. Exchangers D. The action potential of cardiac myocyte results from the activity of multiple distinctive time- and voltage-dependent ionic currents. KCNJ2 B. coronary sinus & pulmonary C. Ito D. 330 Which of the following enhance Ca2+ loading sufficiently to produce DADs ? 324 Which of the following causes phase 3 repolarization ? Harrison’s 18th Ed. IKr (phases 2 and 3). IKs the cytosol and sarcoplasmic reticulum. Increased K+ load in the cytosol B. Phase 4 327 Early afterdepolarizations occur during ? Phase 0 is rapid upstroke. Inactivation of the calcium current B. The currents in turn are carried by transmembrane 326 Automaticity is a property of ? proteins that passively conduct ions down their electrochemical gradients through selective Harrison’s 18th Ed. AJAY MATHUR Cardiology 129 C. All of the above . 1861 Figure 231-1 C. Increased Na+ load in the cytosol A. D. 4 D. All of the above D. Phase 1 D. Phases 2 and 3 of the action potential Harrison’s 18th Ed. 500 . 1861 Figure 231-1 A. Sinoatrial (SA) node B. His-Purkinje system. SLC8A1 B. 129 Cardiology MCQ’s FOR MCQ’s FOR MEDICAL MEDICAL PROFESSIONALS PROFESSIONALS BY PROF. 1863. or electrogenically exchange ionic species (exchangers). Cardiac myocytes have a characteristically long action potential (200 . 325 Sodium-calcium exchanger transports how many Na+ for one 319 Which of the following participate in the cardiac myocyte action Ca2+ ? Harrison’s 18th Ed. Long QT syndrome D. All of the above B. All of the above 335 Which of the following methods is related to autonomic HUT is relatively contraindicated in the presence of severe CAD with proximal coronary stenoses. in hypertrophic cardiomyopathy. C. All of the above B. 1865 C. Hypertrophic cardiomyopathy Harrison’s 18th Ed. Harrison’s 18th Ed. and obstruction to left nervous system influence on the heart ? ventricular outflow (aortic stenosis). 334 Which of the following increases mortality rates in patients after myocardial infarction ? 340 Head-up tilt (HUT) testing is relatively contraindicated in ? Harrison’s 18th Ed. Transposition of the great arteries 331 Which of the following about connexins is false ? Harrison’s 18th Ed. HUT is also a useful tool in the D. and in infiltrative diseases of depolarizing current like sarcoidosis. Interference with calcium conductance with ? Harrison’s 18th Ed. Decrease in heart rate variability (HRV) C. D. chronic fatigue syndrome. 1864 A. Delayed repolarization by K+ current inhibition or activation Ventricular tachyarrhythmias occur more frequently in patients with ventricular systolic dysfunction and chamber dilation. Connexin 31 Exaggerated activation of a central reflex in response to Head-up tilt (HUT) testing produces a C. Mutations in connexin 26 causes progressive hearing loss Supraventricular arrhythmias may be associated with Ebstein’s anomaly. They pair across adjacent cells. 1865 cell communication. 1864 B. Late potentials 341 Which of the following relates best with Vaughan-Williams class I class of antiarrhythmic drugs ? B. 1864 Harrison’s 18th Ed. 1864 342 Which of the following relates best with Vaughan-Williams class A. Interference with calcium conductance . and repeated falls of unknown etiology in the elderly. Connexin 26 D. severe mitral stenosis. Sarcoidosis A. Local anesthetic effect due to blockade of Na + current D. Ventricular systolic dysfunction II class of antiarrhythmic drugs ? B. Interference of catecholamine action at -adrenergic receptor C. Late reduction in heart rate A. Arrhythmogenic right ventricular dysplasia HUT is used most often in patients with recurrent syncope. All of the above C. catecholamines. then a drop in blood pressure followed by a reduction in heart rate characteristic of neurally mediated hypotension. and ischemia all can enhance Ca 2+ loading 337 Supraventricular arrhythmias may be associated with ? sufficiently to produce DADs. A decrease in HRV is associated with increased sympathetic nervous system tone and increased mortality rates in patients after myocardial infarction. None of the above 338 Which of the following is a response to Head-up tilt (HUT) ? Connexins are gap junction membrane-spanning structural proteins that are important in cell-to. severe cerebrovascular disease. Aortic stenosis B. Ebstein’s anomaly A. Brugada syndrome diagnosis of and therapy for recurrent idiopathic vertigo. Recurrent idiopathic vertigo Harrison’s 18th Ed. 1864 B. Connexin 43 Chronically ischemic myocardium exhibits a downregulation of the gap junction channel protein 339 Head-up tilt (HUT) is a useful tool in the diagnosis of and (connexin 43) that carries intercellular ionic current. recurrent transient ischemic attacks. Gap junction membrane-spanning protein C. Ventricular septal defects (VSD) C. Severe cerebrovascular disease D. 1865 C. Harrison’s 18th Ed. D. QT interval variability (QTV) A. 1865 333 Epsilon waves in ECG are a feature of ? A. Local anesthetic effect due to blockade of Na + current D. therapy for ? Harrison’s 18th Ed. Delayed repolarization by K+ current inhibition or activation of depolarizing current 336 Ventricular tachyarrhythmias occur more frequently in patients D. D. Harrison’s 18th Ed. T wave alternans (TWA) at low heart rates B. Late potentials A. Drop in blood pressure Harrison’s 18th Ed. Interference of catecholamine action at -adrenergic Heart rate variability (HRV) & QT interval variability (QTV) provide noninvasive methods to assess receptor autonomic nervous system influence on heart. T wave alternans (TWA) at low heart rates Harrison’s 18th Ed. Initial increase in heart rate 332 Chronically ischemic myocardium exhibits a downregulation of ? B. 1865 A. Chronic fatigue syndrome A. A. All of the above B. Recurrent transient ischemic attacks B. AJAY MATHUR Cardiology Digitalis glycoside toxicity. 1864 A. All of the above D.130 MCQ’s FOR MEDICAL PROFESSIONALS BY PROF. Wolff-Parkinson-White (WPW) syndrome C. Severe mitral stenosis C. Ostium primum atrial septal defect (ASD) B. Connexin 32 stereotypic response of an initial increase in heart rate. 1864 C. CMT1X is caused by mutations in connexin 32 gene D. Fewer myofibrils 347 Ventricular tachycardia is defined as ? B. Interference of catecholamine action at -adrenergic receptor C. no intercalated C. Zipes A. None of the above 355 Membrane potential of SA nodal cells is ? Harrison’s 18th Ed. Rest angina within 48 hours 353 Which of the following is not a feature of SA nodal cells ? Harrison’s 18th Ed. Significantly decreased Harrison’s 18th Ed. B. >= 3 consecutive VPCs at a rate > 200 per minute disks. Local anesthetic effect due to blockade of Na + current C. Hille C. 1865 B. 1867 repolarization due to inhibition of K+ current or activation of depolarizing current and class IV interfere with calcium conductance. Most rapid phase 4 depolarization 345 First catheter ablation using a DC energy source was performed in the early 1980s by ? C. and phase 4 diastolic depolarization. That does not produce retrograde concealed conduction B.60% and left circumflex artery in 40 . D. Well developed sarcoplasmic reticulum A. 1867 D. Delayed repolarization by K+ current inhibition or activation of depolarizing current 350 In acute myocardial infarction. Josephson 352 Nodal cell action potentials exhibit which of the following ? Harrison’s 18th Ed. class 351 Which of the following is false about nodal cells ? II interfere with the action of catecholamines at -adrenergic receptor. Posterior descending artery stops the oncoming sinus impulse SA nodal artery arises from RCA in 55 . Action potential mediated by Na+ rather than Ca++ A. 1437 Nodal cell action potentials exhibit more depolarized resting membrane potentials. Remined unchanged B. Left circumflex artery B. 1865 C. Interference with calcium conductance Class I exert their antiarrhythmic action by local anesthetic effect by blockade of Na + current. 131 Cardiology MCQ’s FOR MCQ’s FOR MEDICAL MEDICAL PROFESSIONALS PROFESSIONALS BY PROF. Local anesthetic effect due to blockade of Na + current D. poorly developed sarcoplasmic reticulum. class III cause delay of Harrison’s 18th Ed. 1868 349 What was the impact of antiarrhythmic agents on mortality in A. Significantly increased A. Less negative resting membrane potential B. 1343 A. No T-tubules B. All patients IV class of antiarrhythmic drugs ? B. That produces retrograde concealed conduction and D. Slower phase 0 upstrokes and colleagues. Slower phase 4 diastolic depolarization 346 Contraindications to exercise stress testing include all except ? D. >= 3 consecutive VPCs at a rate > 100 per minute D. A. Scheinman In nodal cells. >= 3 consecutive VPCs at a rate > 150 per minute SA nodal cells have fewer distinct myofibrils than surrounding atrial myocardium. 1344 344 Which of the following relates best with Vaughan-Williams class A. 1344 III class of antiarrhythmic drugs ? A. None of the above C. action potential mediated by Ca ++ rather than Na + current. Young patients with complicated infarctions Harrison’s 18th Ed. prophylactic antiarrhythmic therapy is recommend in ? D. Patients with hypertension B. SA nodal artery arises from ? Harrison’s 18th Ed. Active infective endocarditis A. >= 3 consecutive VPCs at a rate > 250 per minute 354 In majority. That produces an inverted P wave by retrograde conduction D. Interference of catecholamine action at -adrenergic receptor D. Delayed repolarization by K+ current inhibition or activation of depolarizing current Chapter 232. and no T-tubules.45% of persons. C. The Bradyarrhythmias D. B. 1343 C. Acute pericarditis C. –40 to –60 mV . All of the above Harrison’s 16th Ed. 1867 D. 1867 348 Interpolated VPC is defined as a VPC ? Harrison’s 16th Ed. Interference with calcium conductance Harrison’s 16th Ed. No intercalated disks Harrison’s 16th Ed. slower A. Severe aortic stenosis phase 0 upstrokes. Slow action potential upstrokes (phase 0) Harrison’s 18th Ed. 1865 D. C. AJAY MATHUR Cardiology 131 343 Which of the following relates best with Vaughan-Williams class Harrison’s 16th Ed. More depolarized resting membrane potentials First catheter ablation using a DC energy source was performed in the early 1980s by Scheinman B. Patients with reinfarction A. Right coronary artery A. –20 to –40 mV the Cardiac Arrhythmia Suppression Trial (CAST) ? B. Left anterior descending artery or influence the oncoming sinus impulse C. Failure of conduction of sinus impulses at AV node D. 1869 C.132 MCQ’s FOR MEDICAL PROFESSIONALS BY PROF. C. tachycardia (in SSS). Botulism surrounding atrial tissue is called ? Harrison’s 18th Ed. B. Advanced liver disease C. Alkalosis B. None of the above 358 Sinus bradycardia is associated with all except ? Harrison’s 16th Ed. D. None of the above Harrison’s 16th Ed. sinus arrest. B. Increased intracranial pressure 363 Intermittent absence of P waves on ECG is due to ? D. sinus pauses. 1868 D. Typhoid fever 366 Lack of atrial activity or the presence of an ectopic subsidiary B. 368 In sinus arrest and in sinus exit block. Sinus arrhythmia D. 1869 A. slow phase 0 upstroke. –60 to –80 mV 362 Electrocardiographic manifestations of SA node dysfunction D. Third-degree sinoatrial exit block 359 Sinus bradycardia is associated with all except ? D. Tachycardia Harrison’s 18th Ed. tachycardia (in SSS). Second-degree sinoatrial exit block 360 Sinus arrest means ? C. Second-degree SA block Increased intracranial pressure producing SA nodal dysfunction is called Cushing’s response. 1418 A. Sinus arrest node disease ? Harrison’s 18th Ed. Sick sinus syndrome A. Sinus arrest C. Trypnosomiasis A. 1869 C. Acute hypertension atrial pacemaker is called ? Harrison’s 18th Ed. Third-degree sinoatrial exit block D. 1868 D. Dermopathy Harrison’s 18th Ed. Complete or third-degree SA block results in no P waves on the ECG. and chronotropic incompetence. In type II second-degree SA block. without P waves visible on ECG. Second-degree sinoatrial exit block Neuromuscular diseases like Kearns-Sayre syndrome (ophthalmoplegia. Second-degree SA block appears on the ECG as an intermittent absence of P C. Block of conduction of sinus impulses beyond AV node 361 Sinus exit block means ? 367 Sick sinus syndrome refers to a combination of all except ? Harrison’s 16th Ed. Hypercapnia A. All of the above A. sinus exit block. Failure of sinus impulse formation Type I second-degree SA block results from progressive prolongation of SA node conduction B. AJAY MATHUR Cardiology C. Hypothyroidism surrounding atrial tissue is called ? Harrison’s 18th Ed. 1335 Harrison’s 17th Ed. Sinus bradycardia B. Block of conduction of sinus impulses to atrial tissue with intermittent failure of the impulses originating in the sinus node to conduct to the surrounding atrial tissue. causing a pause heart rate in response to exercise or other stress appropriately). Kearns-Sayre syndrome B. and relatively rapid phase 4 diastolic depolarization compared to the action potentials recorded in cardiac muscle cells. 1335 D. Visual blurring Electrocardiographic manifestations of SA node dysfunction include sinus bradycardia. None of the above A. Second-degree sinoatrial exit block C. First-degree sinoatrial exit block D. 1869 B. pigmentary degeneration of retina & cardiomyopathy) & myotonic dystrophy produce conducting system & SA node disease. Lambert-Eaton syndrome A. First-degree sinoatrial exit block D. sinus pauses. Sinus pauses potential of –40 to –60 mV. First-degree sinoatrial exit block D. Block of conduction of sinus impulses at AV node waves. Intermittent failure of conduction from SA node produces sinus exit block. Sinoatrial block atrial tissue C. Accelerated hypertension C. 1335 A. sinus arrest. Sinus exit block 357 Which of the following neuromuscular disease produce SA C. Failure of conduction of sinus impulses beyond AV node ECG manifestations of SA node dysfunction include sinus bradycardia. Myasthenia gravis 364 A prolongation of conduction time from SA node to B. sinus exit block. atrial asystole is of what duration ? . there is no change in SA node conduction before the pause. B. Failure of conduction of sinus impulses to surrounding B. 1869 The action potentials of SA nodal cells are characterized by a relatively depolarized membrane A. Sinus exit block 356 Cushing’s response is best related to ? C. –80 to –100 mV include ? Harrison’s 18th Ed. 1335 365 The intermittent failure of conduction of sinus impulses to the A. and chronotropic incompetence (inability to increase Sinus pauses & sinus arrest result from failure of SA node to discharge. Third-degree sinoatrial exit block Harrison’s 16th Ed. Failure of sinus impulse formation A. Brucellosis B. Failure to increase the heart rate with exercise is called chronotropic incompetence. 1869 A. Theophylline increases heart rate. Atrial fibrillation pacing of right atrium near SA node (normal <1500 mseconds or. node function ? Harrison’s 18th Ed. In Mobitz type I SA nodal exit block. 55 % C. 133 Cardiology MCQ’s FOR MCQ’s FOR MEDICAL MEDICAL PROFESSIONALS PROFESSIONALS BY PROF.37 x age) beats per minute C. Exercise testing is useful in discriminating chronotropic incompetence from resting bradycardia. 1336 anterior to coronary sinus A. 117. 1870 371 Chronotropic incompetence is failure to reach what percentage A. SA node dysfunction may become 370 Which of the following is false about Mobitz type I SA nodal manifest in the presence of which of the following ? Harrison’s 16th Ed.53 x age) beats per minute B. Base of the interventricular septum just above tricuspid D. > 3 seconds 374 Which of the following is a sensitive and specific indicator of Sinus pauses of up to 3 seconds are common in the awake athlete.(0. SA node rate is constant with progressive delay in exit from the node and activation of atria. amiodarone may unmask evidence of sinus node dysfunction in susceptible individuals. Atrial tachycardia D.(0. A. Combination of an abnormal SNRT.(0.(0. Harrison’s 18th Ed.1 . Class I Antiarrhythmics of predicted maximal heart rate at peak exercise? Harrison’s 18th Ed.5 seconds disease. Amiodarone B. <550 mseconds). D. AJAY MATHUR Cardiology 133 Harrison’s 17th Ed. Digitalis C. corrected for sinus cycle length.2 – (0. & pauses of this duration intrinsic SA node disease ? or longer may be observed in asymptomatic elderly subjects.53 x age) beats per minute C. atrial flutter.04 mg/kg atropine is 117. 1870. 119.53 x age) beats per minute A. Automaticity always A. 116.57 x age) beats per minute D. 1870 than two standard deviations below that of an age-matched control population. Low intrinsic heart rate (IHR) A. Base of interatrial septum just below tricuspid annulus and Harrison’s 16th Ed. 375 In susceptible individual. 118. All of the above D. Sinoatrial conduction time (SACT) Harrison’s 18th Ed. Regularly irregular heart rhythm B. and antiarrhythmic drugs with class I and III action may promote SA node exit block. Digitalis shortens D. 65 % D. > 1 seconds Determining the intrinsic heart rate (IHR) may distinguish SA node dysfunction from slow heart B. > 2.2 – (0. Any of the above intrinsic sinus cycle length and a noncompensatory pause after a premature atrial stimulus (normal <125 mseconds). which of the following A. None of the above . 75 % Beta blockers and calcium channel blockers increase SNRT in patients with SA node dysfunction. 1869 B. tachyarrhythmias. None of the above Cardiac glycosides. Isoproterenol or atropine administered IV may increase sinus rate acutely. Cardiac glycosides A.47 x age) beats per minute posterior to coronary sinus C.2 – (0. 118. Calcium channel blockers C. Normal IHR after administration of 0.2 – (0.53 x age) beats per minute D. 118.1 . Sinus node recovery time (SNRT) is the most common tachycardia ? B. 118.53 x age) in beats/minute.2 – (0. Base of interatrial septum just above tricuspid annulus and 372 Normal values of intrinsic heart rate are calculated by the anterior to coronary sinus formula ? B. Base of interatrial septum just above tricuspid annulus and B. Alternatively defined as a failure to reach 85% of predicted maximal heart rate at peak exercise or failure to 377 The AV node lies at the ? achieve a heart rate >100 beats/minute with exercise or a maximal heart rate with exercise less Harrison’s 18th Ed. No automaticity C. 1418 D. This produces subtly decreasing P-P intervals before the pause. and a low IHR is a Tachycardia-bradycardia syndrome manifests as alternating sinus bradycardia & atrial sensitive and specific indicator of intrinsic SA node disease. Class III Antiarrhythmics A. Although atrial tachycardia. Figure 232-4 A. calcium channel blockers. > 1. there is grouped beating producing a regularly irregular heart rhythm. 1870 369 In Tachycardia-bradycardia syndrome. All of the above B. Decreasing P-P intervals before the pause C. Atrial flutter Sinus node recovery time (SNRT) is defined as the longest pause after cessation of overdrive C. 1869 C. Sinoatrial conduction time (SACT) is defined as one-half the difference between D. and atrial fibrillation may be observed.2 mg/kg propranolol & 0. 85 % SNRT in SA node dysfunction. the latter is the most common tachycardia. and the 376 Which of the following pharmacologic agents may improve SA pause is less than twice the cycle length of the last sinus interval. beta-blockers.5 seconds rates resulting from high vagal tone. an abnormal SACT. 118. Low IHR is indicative of SA C. Pause < twice the cycle length of last sinus interval D. 1342 Harrison’s 18th Ed.1 . 1335 exit block ? Harrison’s 18th Ed.1 . Calcium channel blockers B. Automaticity sometimes B.67 x age) beats per minute annulus and anterior to coronary sinus 373 Normal values of intrinsic heart rate are calculated by the 378 Normal AV node in vivo possesses ? formula ? Harrison’s 16th Ed. Sarcoidosis 381 Which of the following about His bundle & bundle branches is B. and hemochromatosis may produce AV conduction block D. sarcoidosis. and phase 4 diastolic depolarization. 1871 A. .134 MCQ’s FOR MEDICAL PROFESSIONALS BY PROF. SLE D. KCNQ1 A. Septal mitral valve annulus D. high-input resistance and relative insensitivity to external [K+]. Tuberculosis A. 1871 C. None of the above 386 Conditions that can produce AV conduction block include ? 380 Boundaries of triangle of Koch include all except ? Harrison’s 18th Ed. 1871. INa B. I Ca-L. amyloidosis C. SCN5A C. 389 AV conduction block has been associated with which of the following ? 383 AV nodal cells lack which of the following ? Harrison’s 18th Ed. Chagas disease A. . 1871. Kearns-Sayre syndrome A. Carotid sinus hypersensitivity.70 mV D. Chagas’ disease. Coronary sinus ostium B. A. 1871. Located at posterior-inferior right atrium C. Carotid sinus hypersensitivity 391 Congenital AV block in a structurally normal heart is seen in B. 1871 membrane potential of about ? Harrison’s 18th Ed.80 mV Accelerated forms of progressive familial heart block have been identified in families with mutations in cardiac sodium channel gene (SCN5A). . Table 232–2 Harrison’s 18th Ed. Systemic lupus erythematosus false ? C. MCTD. 1870 A. 1871 D. Cough children born to mothers with ? Harrison’s 18th Ed. I Ca-L is responsible for phase 0. Hemochromatosis 385 Conditions that can produce AV conduction block include ? Congenital AV block in a structurally normal heart is seen in children born to mothers with SLE. AV node is a subendocardial B.50 mV B. AJAY MATHUR Cardiology 379 Which of the following statements is false ? Harrison’s 18th Ed. Diphtheria C. All of the above D. Autoimmune and infiltrative diseases like SLE. Toxoplasmosis B. 1870 A. vasovagal syncope. Tendon of Todaro 387 Conditions that can produce AV conduction block include ? Triangle of Koch is bordered by coronary sinus ostium posteriorly. Ventricular septal defect AV conduction block ? Harrison’s 18th Ed. A. None of the above 388 Mutation in which of the following gene causes accelerated 382 Myocytes that constitute the compact node have a resting forms of progressive familial heart block ? Harrison’s 18th Ed. Table 232–2 annulus anteriorly & tendon of Todaro superiorly. Have most rapid conduction in heart Infectious diseases that lead to conducting system disturbances include Lyme disease. Ostium primum atrial septal defect 384 Which of the following is associated with SA node slowing and C. Lyme disease B. and cough and micturition syncope may C. Sarcoidosis Heightened vagal tone during sleep or in well-conditioned individuals can lead to all grades of AV block. Transposition of the great arteries B. scleroderma. All of the above B. Facioscapulohumeral muscular dystrophy C. Phase 4 depolarization reflects Harrison’s 18th Ed. Action potentials have low amplitudes.60 mV C. RA. Insulated from ventricular myocardium (primary & secondary). . and syphilis. If 390 Congenital AV block may be seen in which of the following ? AV nodal cells lack I K1 and I Na. KCNH2 (HERG) B. Rheumatoid arthritis Harrison’s 18th Ed. 1871 the composite activity of the depolarizing currents I f. Table 232–2 Harrison’s 18th Ed. Table 232–2 Harrison’s 18th Ed. . Located at apex of triangle of Koch D. All of the above A. slow upstrokes of phase 0 (<10 V/s). Myocytes that constitute the compact node are depolarized with a resting membrane potential of about –60 mV. Micturition syncope A. Rheumatoid arthritis be associated with SA node slowing and AV conduction block. ICa-L C. and INCX and the repolarizing currents I Kr and IKAC h. Syphilis C. I Ca-T. All of the above A. All of the above D. Minimally influenced by autonomic tone B. 1871 A. 1870-1 D. All of the above D. ANK2 D. Septal tricuspid valve annulus C. 1871. septal tricuspid valve Harrison’s 18th Ed. ICa-T D. D. Myotonic dystrophy B. Karel Frederik Wenckebach C. Russian German C. Atrial activation Harrison’s 18th Ed. His-Purkinje activation B. Pause that follows is fully compensatory I° AV block (PR interval > 200 ms) is a slowing of conduction through AV “junction”. Due to Digitalis. Seen in normal individuals with heightened vagal tone B. A. Leads to complete heart block C. Anglo Indian degree AV block is false ? Harrison’s 16th Ed.21 second B. less commonly. > 0. & a pause that is < 2 times the immediately preceding RR interval. AV block transiently develops in 10 . difference between the longest & shortest PR intervals exceeds D. Distal AV node degree AV block is false ? D. AV block transiently develops in what percentage 399 When some atrial impulses fail to conduct to ventricles. Usually the difference between the longest & shortest PR narrow QRS suggests delay in the AV node proper or. British American D. Robert Silverman In Mobitz type 1 II° AV block.10 % A. Jack Upshaw 100 mseconds. 393 PR interval is determined by ? Harrison’s 17th Ed. Progressive PR prolongation prior to block of an atrial impulse D.24 second D. second degree AV block is false ? A. and Ca ++ channel blockers A. Jack Upshaw degree AV block is false ? Harrison’s 16th Ed. Even when it does.45 % D. slow. 20 . Progression to complete heart block is uncommon.22 second C. except in acute A. Mid AV node 401 Which of the following statements about Mobitz type I second C. 1872 lengthening PR interval. It is almost always localized to AV node & associated with a normal QRS duration. > 0. B. 5 . Proximal AV node preceding the pause. 1420 400 Which of the following statements about Mobitz type I. or His-Purkinje system. delay within AV A. PR interval of first conducted impulse is shorter than last delay is typically in AV node but may be in atria. 1337 Woldemar Mobitz (1889 . 1872 B. lower escape pacemaker . All of the above C. Also called AV Wenckebach block C. AV node bundle of His. Sick sinus syndrome In acute MI. shortening of RR interval. When Mobitz type II block occurs with a normal QRS C.23 second duration. Block is almost always localized to AV node A. AJAY MATHUR Cardiology 135 392 In acute MI. C. Karel Frederik Wenckebach inferior wall myocardial infarction. First-degree heart block B.1951) was a Russian-German physician. 1872 A. Third-degree heart block D. 1337 preceding change in PR intervals A. Spanish 402 Which of the following statements about Mobitz type I second D.25 % B. ECG complex after the pause exhibits a shorter PR interval that immediately A. Robert Silverman 403 Which of the following statements about Mobitz type II second D. 1337 I° AV block with wide QRS is suggestive of delay in the distal conduction system. In II° AV block there is an intermittent failure of electrical impulse conduction from atrium to ventricle. 1337 398 In presence of a normal duration QRS complex.35 % C. John Hay rarely requiring aggressive therapy. > 0. It is seen most often as a transient abnormality with inferior wall infarction or with drug intoxication (digitalis. The site of D. 135 Cardiology MCQ’s FOR MCQ’s FOR MEDICAL MEDICAL PROFESSIONALS PROFESSIONALS BY PROF. whereas a A. Amiodarone therapy is a frequent cause C. 30 . intervals exceeds 100 mseconds 395 Nationality of Woldemar Mobitz was ? B. High incidence of progression to complete heart block with an unstable. John Hay D. beta and calcium channel blockers) or in normal individuals with 397 Mobitz Type II is also named after ? heightened vagal tone. Second-degree heart block C. 1872 Harrison’s 18th Ed. conducted atrial impulse prior to blocked P wave 394 I° AV block with wide QRS suggests delay in ? In Mobitz type 1 II° AV block. Due to disease of His-Purkinje system and associated with prolonged QRS duration B. AV nodal activation A. periodic failure of conduction occurs characterized by a progressively Harrison’s 18th Ed. Usually associated with a normal QRS duration B. Conduction fails suddenly and unexpectedly without a node is the cause of prolonged PR interval if it is ? Harrison’s 16th Ed. the of patients ? type of AV block is ? Harrison’s 18th Ed. > 0.25 % of patients. in the bundle of His. -blockers. an intra-AV node block should be expected D. Most often occurs transiently with inferior MI 396 Mobitz Type I is also named after ? B. this heart block is well tolerated because the escape pacemaker usually arises in the proximal His bundle & provides a stable rhythm & B. 10 . Distal conduction system Harrison’s 16th Ed. D. The finding of a His bundle electrogram after every atrial electrogram indicates that block is occurring in the 413 Which of the following improve conduction through AV node & distal conduction system.136 MCQ’s FOR MEDICAL PROFESSIONALS BY PROF. P waves following QRS complex valve annulus provides information about conduction at all levels of AV conduction axis. QRS duration is prolonged Harrison’s 18th Ed. Typically occurs in distal or infra-His conduction system 411 Which of the following about Lev’s disease is false ? D. impair infranodal conduction ? Harrison’s 18th Ed. Calcification & sclerosis of fibrous cardiac skeleton changes in the preceding PR or RR intervals. 1337. QRS duration is prolonged changes in the preceding PR intervals Complete failure of conduction from atrium to ventricle is called complete or third-degree AV block. It implies significant conduction system disease 415 AH interval in the His bundle recording represents an indirect and is an indication for permanent pacing. Occur in primary or secondary sclerodegenerative or AV block and like CHB implies advanced AV conduction system disease. It is more likely to proceed to higher grades of AV block than is type I second-degree AV block. 1872 D. 1419 Table 225-2 Type 2 II° AV block is characterized by intermittent failure of conduction of the P wave without A. Sinus node activation time . 1337. 1872 D. calcific disorders of fibrous skeleton of heart C. Pre third degree AV block degree AV block is false ? C. Produces AV block block when AV block is ? 412 Which of the following about Lenegre’s disease is false ? Harrison’s 18th Ed. Cause of isolated chronic heart block in adults When AV block is 2:1. 1872 B. the electrode often associated with intraventricular conduction delays (bundle branch block). Exercise B. It typically occurs in distal or infra-His conduction B. High-grade AV block C. Across the pulmonary valve Harrison’s 18th Ed. No involvement of myocardium C. it may be difficult to distinguish type I from type II block. Associated with intraventricular conduction delays D. Involves central fibrous body & summit of ventricular septum 406 It may be difficult to distinguish between type I from type II D. C. Block is usually in AV node 410 Which of the following about Third-degree AV block is false ? Harrison’s 18th Ed. Harrison’s 17th Ed. May proceed to higher grades of AV block Atropine. is often associated with intraventricular conduction delays (bundle branch block). Intermittent failure of conduction of the P wave without D. 1872 B. C. In superior vena cava C. 1873 408 “Paroxysmal AV block” best relates to ? A. No atrial impulse propagates to ventricles 405 Which of the following statements about Mobitz type II second B. None of the above Type II second-degree AV block typically occurs in distal or infra-His conduction system and is 414 To obtain a recording from the bundle of His. None of the above Harrison’s 16th Ed. Block is usually in His-Purkinje system A. 1419 Table 225-2 A. AV nodal conduction time degree is referred to as ? B. while a narrow QRS rhythm implies block in the AV node or changes in the preceding RR intervals proximal His and an escape rhythm originating in the AV junction. It is most often distal to the AV node. Occur in anteroseptal myocardial infarction AV block that is intermediate between second degree & third degree is referred to as high-grade B. A wide QRS escape rhythm implies block in the B. In AV nodal block. and is more likely to proceed to higher grades of AV block. Primary sclerodegenerative disease in conducting system B. 1872 A. 2:1 A. referred to as paroxysmal AV block. 1872 A. Congenital complete AV block is localized to AV node degree AV block is false ? C. 4:1 C. In coronary sinus B. Frequently involves pulmonary & tricuspid valves system. 3:1 B. 5:1 D. Post second degree AV block Harrison’s 18th Ed. catheter is positioned ? Harrison’s 18th Ed. Involves fibrous skeleton of heart D. Atropine Harrison’s 18th Ed. In His bundle block. P waves buried in QRS complex Recording of His bundle electrogram by a catheter positioned at superior margin of tricuspid D. Atrial conduction time Harrison’s 18th Ed. Paroxysmal AV block A. Isoproterenol A. AJAY MATHUR Cardiology 404 Which of the following statements about Mobitz type II second B. All of the above C. 1872 Harrison’s 16th Ed. 1873 409 AV block that is intermediate between second degree & third A. Occurs in distal or infra-His conduction system C. Harrison’s 17th Ed. Second-degree AV block (particularly type II) may be associated with a series of nonconducted P waves. 1873 407 Which of the following about Type II second-degree AV block is false ? A. Series of nonconducted P waves D. Self correcting AV block C. Intermittent failure of conduction of the P wave without distal His or bundle branches. isoproterenol & exercise improve conduction through AV node & impair infranodal conduction. method of assessing ? Harrison’s 18th Ed. Across the tricuspid valve A. Endocardium to epicardium 423 Intrinsic discharge rate is highest of which of the following potential cardiac pacemakers ? 417 Normal HV interval in the His bundle recording is ? Harrison’s 16th Ed. Sinus node A. or the bradycardia-tachycardia syndrome do not require electrophysiologic tests for diagnosis. 35 to 55 ms D. 1336 B. It is an index of intraatrial conduction time & should be 50 ms. 419 Normal AH interval in the His bundle recording is ? Harrison’s 17th Ed. 1421 425 Escape pacemaker following AV nodal block is usually in ? A. 10 to 50 ms Harrison’s 16th Ed. 1336 C. D. 1334 426 Which of the following statements about escape pacemaker in A. Ventricular myocardium 420 Anisotropic conduction means impulse propagation is more rapid ? Escape pacemaker following AV nodal block is usually in the His bundle. Bundle branches D. Specialized fibers of His-Purkinje system B. nonpropagated response. 424 Which of the following conditions do not require electro- 418 Normal PA interval in the His bundle recording is ? physiologic tests for diagnosis in ‘symptomatic’ patients with Harrison’s 17th Ed. Harrison’s 16th Ed. Has a rate of 25 to 45 beats per minute D. Wide QRS complex of prolonged duration Effective refractory period is that part of the action potential during which a stimulus can evoke C. nonpropagated response Harrison’s 16th Ed. Purkinje fibres D. Bradycardia-tachycardia syndrome PA interval is the time from the earliest onset of P wave on surface ECG to onset of atrial deflection D. 40 ms A. 125 to 180 ms B. 15 to 35 ms C. 1334 427 Which of the following statements about escape pacemaker in A. Across the tricuspid valve time through ? Harrison’s 18th Ed. 1873 C. C. Symptomatic patients with ECG documentation of asystole. Is unstable 421 “Effective” refractory period is defined as that portion of action Escape pacemaker in His bundle is stable with discharge rate of 40 to 60 beats/minute. Parallel to fiber orientation than transverse to it His bundle is false ? B. Sinoatrial block or arrest D. 1335 Harrison’s 18th Ed. Harrison’s 16th Ed. AV node D. In the superior vena cava B. AJAY MATHUR Cardiology 137 D. 50 ms B. 10 to 20 ms B. Weaker stimulus can evoke a response B. Is the escape pacemaker following AV nodal block property is termed anisotropic conduction. sinoatrial block or arrest. In a particular fiber length B. Has a rate of 40 to 60 beats per minute D. 30 ms Harrison’s 16th Ed. All of the above on His bundle catheter. 1334 electrogram (AH interval) represents conduction through the AV node and is normally <130 ms. 137 Cardiology MCQ’s FOR MCQ’s FOR MEDICAL MEDICAL PROFESSIONALS PROFESSIONALS BY PROF. potential during which ? associated with a QRS complex of normal duration. Is unstable . 1336 B. None of the above 422 To obtain a recording of left atrial activity. 180 to 250 ms C. Stimulus can evoke a local. His bundle C. 1873 A. Is the escape pacemaker following AV nodal block only a local. represents the conduction time through the His-Purkinje system and is normally 55 ms. 1336 C. Transverse to fiber orientation than parallel to it Harrison’s 16th Ed. Stronger stimulus is required to evoke a response A. In a particular fiber type A. No stimulus can evoke another response distal His-Purkinje system is false ? B. Asystole C. In the coronary sinus A. Is associated with a QRS complex of normal duration Impulse propagation is more rapid parallel to fiber orientation than transverse to it. His-Purkinje system D. 60 to 125 ms A. Across the pulmonary valve 416 HV interval in the His bundle recording represents conduction B. His bundle Left atrial activity is recorded directly via a catheter placed across a patent foramen ovale or indirectly using a catheter inserted into the coronary sinus. 60 ms C. the electrode catheter is positioned ? Time from the most rapid deflection of the atrial electrogram in the His bundle recording to the His Harrison’s 16th Ed. A. This C. 60 to 75 ms SA node is normally the dominant cardiac pacemaker because its intrinsic discharge rate is the The time from the His electrogram to the earliest onset of the QRS on the surface ECG (HV interval) highest of all potential cardiac pacemakers. D. Some specialized atrial fibers C. 1421 ECG documentation of ? A. None of the above D. dual. Programmability and rate modulation D. 3 Pacemaker syndrome is a constellation of signs and symptoms associated with any mode of pacing that does not maintain or restore AV synchrony. dual. atrium. > 110 beats/minute D. none. the second letter indicates the chamber(s) in which sensing occurs (O. A. single). 1875 A. Documented symptomatic bradycardia 430 The first letter in the pacing code indicates ? B. 1875 B. The response to a sensed electric signal 437 Tachyarrhythmias typically refer to ? D. Failing battery of pacemaker D. Chamber in which electrical activity is sensed C. 1878 A. B.138 MCQ’s FOR MEDICAL PROFESSIONALS BY PROF. Abnormal automaticity is responsible for APCs & VPCs A. Who do not maintain AV synchrony D. All of the above 428 Which of the following is false ? Rotation of the pacemaker pulse generator in its subcutaneous pocket. None of the above B. D. > 120 beats/minute In pacing code . none. S. Pacing does not provoke automatic rhythms B. the third letter indicates the response to a sensed event (O. or symptomatic chronotropic incompetence. 1875 Harrison’s 18th Ed. Interference by external stimuli on pacemaker . Pacing provoke tachycardias due to triggered activity 434 Twiddler’s syndrome best relates to ? Harrison’s 18th Ed. A. 4 C. D. ventricle. D. D. All of the above Harrison’s 18th Ed. Chamber(s) paced Chapter 233. Who have chronic illnesses (HTN. Sinus node dysfunction associated long-term drug therapy B. triggered. Definition of tachycardia is rhythm that produces a ventricular rate >100 beats/minute. S. The response to a sensed electric signal C. pace + shock). Chamber in which electrical activity is sensed for which there is no alternative C. 1875 A. the 439 Which of the following statements is false ? fourth letter refers to the programmability or rate response (R. the first letter indicates the chamber(s) that is paced (O. 1879 refers to the existence of antitachycardia functions if present (O. All of the above 433 The fourth letter in the Pacing Code indicates ? 438 In tachycardia. P. antitachycardia pacing. sinus node dysfunction associated long-term drug therapy for which there is no alternative. 1875 Table 232-3 Pacemaker modes & function are named using a five-letter code. often referred to as “twiddler’s syndrome” producing dislodgment with failure to sense or pace the heart. Who do not have adequate cardiac output 429 How many letter codes are used for describing pacemaker C. manifest wide QRS complexes with prolonged duration and are unstable. atrium. None of the above B. the ventricular rate should be ? Harrison’s 18th Ed. and the fifth Harrison’s 18th Ed. either intentionally or Harrison’s 17th Ed. > 100 beats/minute C. The response to a sensed electric signal D. Programmability and rate modulation D. > 90 beats/minute B. S. I. 1878 432 The third letter in the Pacing Code indicates ? A. inhibition. Isolated premature complexes (depolarizations) Harrison’s 18th Ed. Syncope of unexplained origin with major abnormalities of Harrison’s 18th Ed. The Tachyarrhythmias B. inhibition + triggered). Chamber in which electrical activity is sensed B. Chamber in which electrical activity is sensed C. Sustained forms of tachycardia originating from myocardial C. Chamber(s) paced myocardial foci or reentrant circuits B. 1875 C. shock. A. Mobitz type 1 second-degree AV block is intranodal 435 Pacemaker syndrome is associated in those ? Harrison’s 18th Ed. Symptomatic chronotropic incompetence D. 6 Harrison’s 18th Ed. Chamber(s) paced C. 431 The second letter in the Pacing Code indicates ? Harrison’s 18th Ed. Programmability and rate modulation Harrison’s 18th Ed. AJAY MATHUR Cardiology Escape rhythms arising in the distal His-Purkinje system have lower intrinsic rates (25 to 45 C. The response to a sensed electric signal foci or reentrant circuits D. V. T. none. A. Chamber(s) paced A. V. 1422 inadvertently. rate responsive). DM) modes & function ? D. 1875 C. ventricle. First-degree AV block is intranodal B. none. single). D. 5 436 Class I indication for pacing in SA node dysfunction include all except ? D. Nonsustained forms of tachycardia originating from A. Mobitz type 2 second-degree block is infranodal A. 1875 SA node dysfunction A. A. Programmability and rate modulation Class I indications for pacing in SA node dysfunction include documented symptomatic bradycardia. Rotation of pacemaker pulse generator in its pocket beats/minute). Myocardial cells do not possess pacemaker activity D. Junctional tachyarrhythmias B. 1880 D. 1342 A. P wave of APC differs from sinus P wave morphology D. Left atrium morphology ? C. generalized genetically determined myocardial ion channel abnormalities (long QT syndrome) or A. 1342 leading to reentry is exaggerated by ? A. Asymptomatic Harrison’s 18th Ed. Triggered activity is related to cellular afterdepolarizations that occur at the end of the action potential. Reentry 446 Which of the following is a genetically determined ion channel B. Altered sodium transport B. Intracellular potassium deficiency D. It is the vertical crest on interior to ? wall of right atrium that separates nontrabeculated posterior right atrium from rest of the Harrison’s 18th Ed. Harrison’s 18th Ed. Reexcitation of the initially blocked pathway Afterdepolarizations are attributable to an increase in intracellular calcium accumulation. tobacco or adrenergic stimulants precipitate APCs C. Altered potassium transport C. and fascicular tachyarrhythmias. Right atrial appendage . Catecholaminergic polymorphic VT phase 3 (early afterdepolarizations). None of the above A. 139 Cardiology MCQ’s FOR MCQ’s FOR MEDICAL MEDICAL PROFESSIONALS PROFESSIONALS BY PROF. Can be found in over 90 % of normal adults B. Frequently increases with age & structural heart disease 445 Crista terminalis is located in ? C. None of the above D. 1880 Inhomogeneities can be exaggerated by the presence of extra pathways (WPW syndrome). 1879 sustained paroxysmal tachyarrhythmia is false ? Harrison’s 16th Ed. May originate from any location in either atrium C. Intracellular calcium accumulation A. Extra pathways abnormality ? C. 1879 D. Left ventricle A. 448 Which of the following statements about mechanism of 442 Early afterdepolarizations is responsible for ? tachyarrhythmia is false ? Harrison’s 18th Ed. during phase 4 (late afterdepolarizations). Triggered activity is caused by early afterdepolarizations only caused by digoxin toxicity and for catecholamine-sensitive VT originating in the outflow tract. All of the above C. Fascicular tachyarrhythmias C. 1879 Harrison’s 16th Ed. All of the above 451 Which of the following about APCs is false ? Reentry is due to inhomogeneities in myocardial conduction and/or recovery properties. Reentrant arrhythmias can be reproducibly initiated and B. 1879 trabeculated right atrium. B. LQTS Abnormal impulse formation is due to the development of triggered activity. Alcohol. AJAY MATHUR Cardiology 139 440 Abnormal impulse formation due to triggered activity is related Crista terminalis is a natural anatomic barrier of conduction. 449 Which of the following statements about atrial premature 443 Late afterdepolarizations in digoxin toxicity may lead to ? complexes (APCs) is false ? Harrison’s 18th Ed. junctional. during C. 1879 D. Conduct to ventricles when they occur late in cardiac cycle Late afterdepolarizations caused by digoxin toxicity may lead to atrial. Myocardial ion channel abnormalities Harrison’s 18th Ed. Myocardial fibrosis D. Unidirectional block in one pathway C. Atrial tachyarrhythmias A. Presence of extra pathways B. Sum of pre-and postextrasystolic PP intervals is less than Harrison’s 18th Ed. Cellular afterdepolarizations A. Brugada syndrome B. or after the action potential. Tachycardia caused by automaticity cannot be started or Early afterdepolarizations may be responsible for VPCs that trigger torsades des pointes stopped by pacing (TDP). 1879 the sum of two sinus PP intervals A. 1879 Harrison’s 16th Ed. D. Myocardial ion channel abnormalities C. Right atrium 452 APCs from which of the following may mimic the sinus P wave B. 1342 A. 1342 A. Late afterdepolarizations are responsible for atrial. All of the above 441 Afterdepolarizations are due to an increase in ? 447 Which of the following statements about mechanism of Harrison’s 18th Ed. A. 450 Which of the following statements about atrial premature complexes (APCs) is false ? 444 Inhomogeneities in myocardial conduction and/or recovery Harrison’s 16th Ed. All of the above D. Electrophysiologic inhomogeneity B. Fast conduction over an alternative pathway D. Right ventricle Harrison’s 18th Ed. Most common arrhythmia by the interruption of normal myocardial patterns of activation due to the development of fibrosis. Catecholamine-sensitive VT A. junctional & fascicular tachyarrhythmias D. QRS complex following most APCs is normal B. Torsades des pointes B. Atrial tachyarrhythmias caused by digoxin toxicity terminated by premature complexes and rapid stimulation C. Originate from AV node & His bundle region C. when the left atrial diameter exceeds ? AF is the most common sustained arrhythmia. Heart rate increases out of proportion to stress/exercise B. Right atrium P wave contour differs from that seen during sinus rhythm. Acute alcoholic intoxication B. Digitalis Sinus tachycardia. Atrial scarring A. Complexes originate from AV node & His bundle region & may produce retrograde atrial activation with P wave distorting the initial or terminal portions of QRS complex producing pseudo Q or S waves in leads II. Electrical cardioversion C.140 MCQ’s FOR MEDICAL PROFESSIONALS BY PROF. 1346 C. Maintaining hydration. The resulting sum of the pre.& post-APC RR is < 2 sinus PP intervals B. APCs from right atrial appendage. Superior aspect of crista terminalis A. rate varies between 120 & 160 beats/minute. Atrialized musculature that enters pulmonary veins B. Tachycardia . 1880 A. rapid & irregular atrial activation with irregular ventricular response. Accessory atrio-ventricular tracts C. which of the following is the treatment of choice ? Harrison’s 16th Ed. 454 Which of the following about junctional premature complexes 460 Which of the following relates best to AF initiation & maintenance ? Harrison’s 18th Ed. 1345 457 Which of the following is the most common sustained arrhythmia ? A. Carotid sinus pressure produces slowing with sudden Harrison’s 16th Ed. JPCs are extremely uncommon. 1881 is false ? Harrison’s 18th Ed. Present when heart rate exceeds 100 beats/min C. III. Heart rate increases spontaneously A. 1881 463 Which of the following is a precipitating factor for acute AF ? Harrison’s 16th Ed. 250 and 280 bpm APCs characteristically reset the sinus node. 461 ‘Lone AF’ often represents the tachycardia phase of ? Harrison’s 16th Ed. 1881 B. salt loading. 150 and 180 bpm C. if a patient’s clinical status is severely compromised. C. Sum of pre. 1345 A. Atrial Fibrillation termination. 120 and 160 bpm B. Class IC antiarrhythmic agents may eliminate APCs C.  adrenergic blocker Harrison’s 18th Ed. Depending on conduction properties of AV junction. 464 In AF. the ventricular rate is usually ? Harrison’s 18th Ed. Class IC antiarrhythmic agents may eliminate the AF produces disorganized. and aVF. None of the above D. Digitalis B. 1344 A. Acute alcoholic intoxication is false ? Harrison’s 18th Ed. Produce retrograde atrial activation D. 1345 return to previous rate upon cessation A. 1345 455 Which of the following is not true for physiologic sinus A. . None of the above 465 It is difficult to convert AF to sinus rhythm and/or maintain it. Anxiolytics are the treatment of choice D. Atrial flutter C. Extremely uncommon B. Pulmonary emboli B. Superior vena cava (SVC) 458 Paroxysmal AF is triggered by automatic foci located in ? Harrison’s 16th Ed.bradycardia syndrome Harrison’s 16th Ed. Left atrium D. Pulmonary veins node may mimic sinus P wave morphology. heart rate increases either spontaneously or out of proportion to the degree of physiologic stress/exercise. Has a gradual onset and offset 462 ‘Tachycardia-induced cardiomyopathy’ is cause by ? D. 1881 A. 200 and 260 bpm D. and beta blockers minimizes symptoms. Part of postviral dysautonomia C. All of the above D. D. All of the above 453 Which of the following about APCs is false ? 459 In atrial fibrillation. All of the above In inappropriate sinus tachycardia. AV nodal reentrant tachycardia (AVNRT) D.12 months (postviral dysautonomia). Thyrotoxicosis 456 Which of the following about inappropriate sinus tachycardia D. 1880 Harrison’s 18th Ed.and post-APC RR interval is less than two sinus PP intervals. Calcium channel antagonist A. carotid sinus pressure produces modest & transient slowing but no abrupt B. Thyrotoxicosis tachycardia ? B. Atrial fibrillation D. despite therapy. CHF C. None of the above Drivers responsible for initiation and maintenance of AF originate from atrialized musculature that enters the pulmonary veins. superior vena cava (SVC) and superior aspect of the crista terminalis in the region of sinus C. Pericarditis D. Heart rate rarely exceeds 200 beats/min D. APCs but should be avoided if structural heart disease is present. It may occur after a viral illness and resolve spontaneously over 3 . All of the above C. AJAY MATHUR Cardiology B. APCs characteristically reset the sinus node A. All of the above B. Amiodarone A. At least 200 W D.200 J D. Conversion rates using a 200-J biphasic shock delivered synchronously with the precluding the ability of the atria to fibrillate. > 6 hours B.5 cm Organized atrial flutter activity can frequently be terminated with low-energy external cardioversion D. 4 weeks direct current (DC) cardioversion. 1 week D. Macroreentrant arrhythmias involving atrial myocardium are A. pretreatment with which of the following drugs facilitates cardioversion ? 468 In AF. 3. At least 300 W D.8 on at least two separate occasions prior to attempts at cardioversion. R wave synchronous DC cardioversion using 100 . 1881 B. 2 weeks D. At least 40 J C. QRS complex typically are >90%. patients without evidence of structural heart disease ? Harrison’s 18th Ed.40 J B. conversion to sinus rhythm by cardioversion should be Harrison’s 18th Ed. 1356 B. 100 . Dofetilide B.100 J A. Ibutilide Harrison’s 18th Ed. 2 weeks In AF patients without evidence of structural heart disease. is begun if duration of AF is >12 hours & risk factors for stroke with AF are present. Risks include pulmonary vein stenosis. 1885 delivery of ? A. 50 . Flecainide Harrison’s 18th Ed. Digoxin may add to the rate-controlling benefit of other agents but is uncommonly used as a stand-alone agent. Amiodarone B. atrioesophageal fistula. verapamil or diltiazem. 1885 Harrison’s 18th Ed. Calcium antagonist B. 141 Cardiology MCQ’s FOR MCQ’s FOR MEDICAL MEDICAL PROFESSIONALS PROFESSIONALS BY PROF. The Cox-Maze procedure Direct current transthoracic cardioversion during short-acting anesthesia is a reliable way to is designed to interrupt all macroreentrant circuits that might potentially develop in the atria. At least 100 J D. 1882 469 In AF. AJAY MATHUR Cardiology 141 Harrison’s 16th Ed. At least 200 J Most ablation strategies isolate atrial muscle sleeves entering the pulmonary veins which are a D.5 cm C. thereby terminate AF. 5.40 J called AFL . 1881 B.5 cm of 50 . especially in acute AF.150 J B. PSVT B. 20 . 3 weeks IV ibutilide is more effective and may be used in selected patients to facilitate termination with D.5 cm D. All of the above C. 1886 A. Digitalis C. 150 . At least 100 W C. 200 . 1881 473 What energy should be used for initial attempts at terminating VF ? A.300 J IV heparin tt. 2. 1345 B. 4. VT A. Beta blocker A. AF Harrison’s 18th Ed. anticoagulation must be A. if the duration of AF is ? 472 Electrical cardioversion for atrioventricular nodal reentrant Harrison’s 18th Ed. > 3 hours A. 477 Which of the following statements is false about macroreentrant 471 Electrical cardioversion for atrial flutter is accomplished arrhythmias ? through delivery of ? Harrison’s 18th Ed. 50 . use of flecainide or propafenone C. For patients who do not warrant early cardioversion of AF. 466 IV heparin treatment is begun. D.100 J C. Sotalol maintained for ? B. > 9 hours C. 1882 done after proper anticoagulation for ? A. 100 . At least 400 W Ventricular rate control for acute AF is best established with beta blockers and/or calcium channel blocking agents. 1888 A. 3 weeks (class IC) is well tolerated and does not have significant proarrhythmia risk. 20 . At least 400 J source of triggers responsible for initiation of AF. 474 In AF. 1881 C.100 J. Adenosine Harrison’s 16th Ed. 467 Ventricular rate control for acute AF is best established with ? Harrison’s 18th Ed. 1881 tachycardia (AVNRT) is accomplished through delivery of ? Harrison’s 18th Ed.300 J C. > 12 hours D. 4 weeks 476 The Cox-MAZE procedure is a surgical approach to cure ? 470 Electrical cardioversion for AF is accomplished through Harrison’s 18th Ed. following cardioversion.200 J can terminate atrioventricular nodal reentrant tachycardia (AVNRT). systemic embolic events & perforation/tamponade. 1 week C. anticoagulation should be maintained 475 Which drug is preferred in AF to maintain sinus rhythm in for at least 3 weeks with the INR confirmed to be >1. Digoxin C. Beta blockers A. Left atrium C. They may respond to adenosine. 150 and 250 bpm Harrison’s 18th Ed. Absence of any intervening sinus rhythm right AFL with superiorly directed activation of interatrial septum produces saw-toothed appearance of P waves in ECG leads II. None of the above predominantly positive P waves in leads II. Aortic valve annulus B. 482 Which of the following drugs is particularly effective for 488 Which of the following is false about automatic atrial conversion of atrial flutter to sinus rhythm ? tachycardias ? Harrison’s 18th Ed. All of the above response at 2:1 or 130 . amiodarone or ibultilide is appropriate. Defined as >= 3 consecutive P waves of different Harrison’s 18th Ed. Irregular ventricular rate C. Response to adenosine C. P wave initiating tachycardia has different morphology than P wave during sustained AT . 481 In atrial flutter. All of the above D. Multifocal atrial tachycardia D. A.150 beats/minute. Calcium antagonist C. the atrial rate is ? Harrison’s 18th Ed. verapamil). 1886 A. Response to verapamil Classic or typical right AFL has an atrial rate of 260 . 1886 Harrison’s 18th Ed. Provoked by isoproterenol infusion C. 1886 B. and/or Automatic ATs start with a “warm-up” period over the first 3 . AFL is poorly tolerated than AF D. Initiate with spontaneous premature beats B. rate control with calcium antagonists (diltiazem. 479 Typical AFL circuit rotates around ? 485 Which of the following is the signature tachycardia of Harrison’s 18th Ed. 483 Which of the following drugs is useful in preventing 489 Which of the following is false about focal reentrant atrial recurrences of atrial flutter ? tachycardias ? Harrison’s 16th Ed. 1886 C.300 beats/minute with a ventricular D. Pulmonary valve annulus C. Counterclockwise C. III. flutter waves of atrial flutter are most prominent in ? Harrison’s 18th Ed. In patients with high anesthestic risk. Amiodarone C.10 complexes and slow in rate digoxin is difficult. Macroreentrant ATs represent continuous atrial activation and isoelectric baseline between P waves is frequently absent. Nonfocal source of an atrial arrhythmia C. remaining waves. None of the above In AFL. 1886 A. Antiarrhythmic drug therapy may enhance be provoked by isoproterenol infusion. 300 and 400 bpm B. III and aVF. and aVF. Mitral valve annulus Harrison’s 18th Ed. Atrial fibrillation D. Clockwise rotation of right atrial circuit produces D. 1885 487 Focal ATs can be distinguished by ? A. Quinidine A. Both atria and AV node There is a high incidence of AF in patients with MAT. 1347 Harrison’s 18th Ed. beta blockers. First P wave of AT has same morphology as remaining waves D. Beta blocker B. Sotalol B. 1350 478 Atrial flutter is a form of atrial reentry localized to the ? A.142 MCQ’s FOR MEDICAL PROFESSIONALS BY PROF. 1885 significant pulmonary disease ? A. AV Nodal reentrant tachycardia 480 In ECG. High incidence of VF D. None of the above 484 Which of the following statements about multifocal atrial Macroreentrant nonfocal arrhythmias involving the atrial myocardium are referred to collectively tachycardia (MAT) is false ? as AFL. Harrison’s 16th Ed. Focal ATs can be distinguished by observations made at AT initiation & in response to adenosine. Initiation of automatic ATs frequently can procainamide. Tricuspid valve annulus A. 400 and 550 bpm C. First P wave of tachycardia has same morphology as efficacy of direct current cardioversion & maintenance of sinus rhythm after cardioversion. Isoelectric baseline on ECG Typical AFL circuit rotates in right atrium around tricuspid valve annulus. Common following theophylline administration B. Signature tachycardia of significant pulmonary disease D. Digitalis D. 1886 B. Lateral leads B. Posterior leads A. Ibutilide A. pharmacologic cardioversion with before termination. Right atrium B. AJAY MATHUR Cardiology B. 260 and 300 bpm A. Response to amiodarone D. Both atria D. Anterior leads Harrison’s 18th Ed. 1885 Multifocal AT (MAT) is the signature tachycardia of patients with significant pulmonary disease. Inferior leads 486 Which of the following about multifocal atrial tachycardia is false ? B. Atrial flutter C. 1885 morphologies at rates >100 beats/minute A. Respond to adenosine B. high-frequency form of electrical energy 493 Which of the following symptoms is uncommon in B.354:1039-51 D. 150 cases per 100. SVT is usually associated with structural heart disease AV nodal reentrant tachycardia is the most common paroxysmal regular SVT.354:1039-51 D. low-frequency form of electrical energy supraventricular tachycardia ? C. 10 per 1000 N Engl J Med 2006. Palpitations Catheter ablation.000 persons per year B. 2. All of the above A. B. Coronary venous thrombosis 491 Incidence of supraventricular tachycardia (SVT) is about ? N Engl J Med 2006. Jugular veins D.354:1039-51 D. C. All of the above A. AJAY MATHUR Cardiology 143 C. B. is very B. Intravenous Procainamide reentrant ATs will produce AV block but typically do not slow and/or terminate. Myocardial perforation 492 Prevalence of supraventricular tachycardia is about ? C. High-voltage.354:1039-51 A. 70 cases per 100. 1888 B. Do not slow and/or terminate with adenosine D. Syncope tachycardia is false ? Harrison’s 18th Ed. Most have a reentry mechanism B. A. All of the above D. 100 cases per 100.354:1039-51 P wave initiating tachycardia has a different morphology than P wave during sustained AT. Limbus of fossa ovalis N Engl J Med 2006.000 persons per year 499 Danger of radiofrequency catheter ablation of accessory B. 1887 D. 35 cases per 100.25 per 1000 B. directed at elimination or modification of slow pathway conduction. Valvular damage N Engl J Med 2006. It is more common in women than in men (>2:1). Low-voltage. valve annuli or limbus of the fossa ovalis. ~ 60 % are due to AV nodal reentry circuit D. Typically manifests in II to IV decades of life C. adenosine is contraindicated in patients with ? and tends to occur in the absence of structural heart disease. 143 Cardiology MCQ’s FOR MCQ’s FOR MEDICAL MEDICAL PROFESSIONALS PROFESSIONALS BY PROF. All of the above A. Nail beds N Engl J Med 2006. 5 per 1000 500 Characteristic of radiofrequency current used for catheter ablation of accessory pathways in SVT is ? C.354:1039-51 D. Intravenous verapamil A. None of the above 497 Which of the following drugs are used in treatment of SVT ? Focal reentrant AT initiate with programmed atrial stimulation or spontaneous premature beats. Damage to an artery. In patients with severe obstructive lung disease A. Abdominal pulsations 496 Which of the following drugs are used in treatment of SVT ? D. Atrioventricular block that requires pacemaker therapy Anatomic ridges of heart are crista terminalis. Pulmonary embolism D. Intravenous beta-blocker B. Intravenous Propafenone Harrison’s 18th Ed. Dyspnea effective in permanently eliminating AVNRT.354:1039-51 C. N Engl J Med 2006. Low-voltage. high-frequency form of electrical energy N Engl J Med 2006. Typically manifests in second to fourth decades of life 495 In SVT. arteriovenous fistula C. N Engl J Med 2006. Valve annuli pathway in SVT is ? C. High-voltage. In heart-transplant recipients Harrison’s 18th Ed. Most common paroxysmal regular SVT A. it is usually well tolerated. P wave in AT is characteristically distinct from the sinus P-wave morphology. Intravenous Ibutilide 490 Which of the following is an anatomic ridge in heart ? C. Adenosine Harrison’s 18th Ed. More common in men than in women B. All of the above A.354:1039-51 502 “Frog sign” is best related with ? A.354:1039-51 D.000 persons per year A. Occur in the absence of structural heart disease D. 1888 B. Crista terminalis 498 Danger of radiofrequency catheter ablation of accessory B. In patients with tachycardia with a wide QRS complex B. low-frequency form of electrical energy A.354:1039-51 503 “Frog sign” is best related with ? A. Arterial pulse C. All of the above C. Atrioventricular nodal reentrant tachycardia . Atrial fibrillation C. With adenosine. Light-headedness 501 Which of the following about atrioventricular nodal reentrant D. bleeding. 1888 494 Which of the following statements about SVT is false ? N Engl J Med 2006.000 persons per year pathway in SVT is ? N Engl J Med 2006. 20 per 1000 A. Classified according to location of reentry circuit C. conduction is through both pathways. During sinus B. D.367:1438-48 Sinus tachycardia is gradual in onset and recession. Most common cause is AV nodal reentrant tachycardia atrioventricular node and the atrial tissue. B. whereas pathway A. the second most common pathologic supraventricular tachycardia. A. atrial flutter (AFL). During sinus rhythm. Rate In atrioventricular nodal reentrant tachycardia. Flutter waves are usually obscured by T waves. 5% from other supraventricular tachycardias ? B. neck pulsations are felt due to simultaneous C. Orthodromic AVRT is more common than antidromic AVRT. B. and a wide irregular QRS complex (atrial fibrillation). C. Atrial flutter tachycardia ? Harrison’s 18th Ed. whereas atrioventricular reciprocating tachycardia occurs more frequently in pediatric population.367:1438-48 D. Repetitive activation up slow and down fast pathway B. Regularity D. Tricuspid valve conduction down the atrioventricular node and retrograde conduction through the bypass tract). Aortic valve through the atrioventricular node). results from a of less than ? reentrant circuit around the tricuspid valve in the right atrium. conduction down the bypass tract and retrograde conduction C. The heart rate is regular and classically does not exceed 220 beats per minute minus the patient’s age. atrioventricular nodal reentrant tachycardia (AVNRT). Repetitive activation down fast pathway A. Sudden in onset and recession (AVRT). Irregular supraventricular tachycardias include atrial fibrillation (AF). None of the above N Engl J Med 2012. P waves A. Rate < 220 bpm minus the patient’s age D. atrioventricular reciprocating tachycardia A. and a “frog sign” is identified on physical examination during arrhythmia. Mitral valve Three arrhythmias are seen with bypass tracts: a narrow regular QRS complex (orthodromic. Regular patient with variable AV block. B. Bundle of His 506 Which of the following statemtents about SVT is false ? D.2 In sinus tachycardia. C. Ventricular tachycardia A. Atrioventricular reciprocating tachycardia (AVRT) D.367:1438-48 509 Which of the following parameter helps distinguish atrial flutter A. Atrioventricular nodal reentrant tachycardia lower in AV node region conducts more slowly and has a shorter refractory period. Orthodromic AV reentry precede the QRS complex. Atrial tachycardia (AT) D. a heart rate of 150 beats per minute is highly suggestive of this tachyarrhythmia. APCs that initiate the SVT has a normal PR interval B. More commonly observed in women 512 Regular supraventricular tachycardias include all except ? C. Wide irregular QRS complex (atrial fibrillation) N Engl J Med 2012. a wide regular QRS complex (antidromic. Ventricular tissue reentrant tachycardia. Presents as regular narrow QRS complex tachycardia @ N Engl J Med 2012. Pulmonary valve 514 Heart rate regularity is defined as beat-to-beat timing variation Atrial flutter. Atrioventricular reciprocating tachycardia rhythm.367:1438-48 120 to 250 bpm A. A.144 MCQ’s FOR MEDICAL PROFESSIONALS BY PROF.367:1438-48 D. conduction is through both pathways population ? D. Repetitive activation down slow pathway N Engl J Med 2012. 10 % N Engl J Med 2012. Atrial tissue Repetitive activation down the slow and up the fast pathway results in typical AV nodal B.367:1438-48 . Atrioventricular nodal reentrant tachycardia (AVNRT) APC initiating AVNRT is characteristically followed by a long PR interval consistent with conduction via the slow pathway. AFL when it occurs in a B. and atrial tachycardia (AT). All of the above A. All of the above B.367:1438-48 Fast pathway is in superior part of AV node and has a longer refractory period. Multifocal atrial tachycardia (MAT) 507 Which of the following about sinus tachycardia is false ? N Engl J Med 2012. making the surface ECG tracing for this tachycardia difficult to distinguish from that of other supraventricular tachycardias. Blood pressure atrial & ventricular contraction. 1888 Atrioventricular nodal reentrant tachycardia is most common in persons >20 years of age. AJAY MATHUR Cardiology C. C. Slow pathway lower in the AV node region 510 Which of the following occurs more frequently in the pediatric C. C. Antidromic AV reentry 508 Atrial flutter results from a reentrant circuit around ? C. multifocal atrial tachycardia (MAT) & sinus rhythm with multiple atrial premature beats. 1888 beats per minute. N Engl J Med 2012. 1888 Atrioventricular nodal reentrant tachycardia is caused by a reentrant loop that involves the A. P waves precede the QRS complex 513 Which of the following arrhythmias are seen with bypass tracts ? N Engl J Med 2012. Fascicles Harrison’s 18th Ed. B. D.367:1438-48 Regular supraventricular tachycardias include sinus tachycardia. Repetitive activation down slow and up fast pathway 511 Atrioventricular nodal reentrant tachycardia is caused by a reentrant loop that involves atrioventricular node and ? C. Fast pathway has a longer refractory period However. Jugular venous pulsations Atrial flutter is an organized regular rhythm that is characterized by an atrial rate of 280 to 300 504 Which of the following about AVNRT is false ? beats per minute and with 2:1 conduction in AV node results in a ventricular rate of 140 to 150 Harrison’s 18th Ed. Atrial tachycardia 505 Which of the following occurs in a typical AV nodal reentrant D. All of the above 517 Vagomimetic maneuvers include ? AVNRT prevention may be achieved with drugs that slow conduction in the antegrade slow N Engl J Med 2012. Ventricular tachycardia J can terminate atrioventricular nodal reentrant tachycardia. A. Torsades de pointes slow pathway in atrioventricular nodal reentrant tachycardia ? D. Beta blockers reciprocating tachycardias as well as up to 80% of atrial tachycardias. Beta blockers bundle-branch block). Flecainide N Engl J Med 2012. All of the above N Engl J Med 2012. and a ventricular paced rhythm from a pacemaker. C. C.367:1438-48 In patients who have a history of exercise-precipitated AVNRT. use of beta blockers frequently A. atrial tachycardia. A. 1888 A. P waves follow the QRS complex in atrioventricular nodal reentrant tachycardia and atrioventricular reciprocating B. and calcium channel blockers. Atrioventricular nodal reentrant tachycardias Harrison’s 16th Ed. Calcium channel blockers 519 Adenosine should not be given in ? D. such as digitalis. 1888 P waves immediately precede the QRS complex in sinus tachycardia. Beta blockers of the face in ice water They increase vagal tone and block the atrioventricular node. B. . Heart failure Regularity is defined as variation of less than 10% in beat-to-beat timing. May cause cannon ‘a’ waves 520 Arrhythmias that cause which of the following require urgent The site of origin of AV junctional complexes is in the bundle of His. Associated with digitalis intoxication but it should be given only when these tachycardias are regular.367:1438-48 pathway. Carotid sinus massage 523 Which of the following drugs is useful in preventing exercise- B. ventricular A.200 J) N Engl J Med 2012. Can conduct both antegradely and retrogradely tachycardias may be rendered unstable after the administration of adenosine. Hypotension B. AJAY MATHUR Cardiology 145 C. Intravenous calcium channel therapy 516 Which of the following is a wide-complex tachycardia ? D. 145 Cardiology MCQ’s FOR MCQ’s FOR MEDICAL MEDICAL PROFESSIONALS PROFESSIONALS BY PROF. D. All of the above Harrison’s 18th Ed. terminates nearly all atrioventricular nodal reentrant tachycardias and atrioventricular B. B. bearing down. R-wave synchronous DC cardioversion using 100–200 A. Digitalis Vagal maneuvers include Valsalva maneuver. Digitalis fibrillation. Valsalva maneuver precipitated atrioventricular nodal reentrant tachycardia ? C. Intravenous beta blockade tachycardia. 15 % N Engl J Med 2012. a very short-acting endogenous nucleotide that blocks atrioventricular nodal conduction. B. All of the above A.367:1438-48 D. carotid sinus massage. Digitalis Adenosine. but most regular tachycardias actually vary by less than 5%. C.367:1438-48 If hemodynamic compromise is present. Ventricular fibrillation 522 Which of the following drugs slow conduction in the antegrade C. Atrioventricular reciprocating tachycardias A. which of the following C. Atrioventricular nodal reentrant tachycardia is preferred to terminate atrioventricular nodal reentrant tachycardia ? D. 20 % A. Immersion of face in cold water Harrison’s 18th Ed. Calcium channel blockers ventricle from bypass tract. 1342 D. Multifocal atrial tachycardia 521 If hemodynamic compromise is present. B.367:1438-48 Arrhythmias causing hemodynamic instability (hypotension. a bypass tract (Wolff–Parkinson–White syndrome). since the normal AV node electrical cardioversion ? in vivo possesses no automaticity. with depolarization of C. Irregular wide-complex tachycardias 525 Which of the following about AV junctional complexes is false? C. Atrial tachycardia B. R-wave synchronous DC cardioversion (100 . and multiple atrial premature contractions. All of the above 515 P waves immediately precede the QRS complex in all except ? N Engl J Med 2012. All of the above Harrison’s 18th Ed. Calcium channel blockers 518 Adenosine terminates which of the following ? D. 1888 Wide-complex tachycardias are caused by ventricular arrhythmia (ventricular tachycardia. Atrioventricular reciprocating tachycardias 524 Which of the following drugs slow conduction in the antegrade C. Site of origin is AV node Adenosine is also useful in the differential diagnosis and treatment of wide-complex tachycardias. heart failure.367:1438-48 In patients who do not respond to drug therapy directed at antegrade slow pathway. Multiple atrial premature contractions Harrison’s 18th Ed. D. and immersion B. C. Regular wide-complex tachycardias with class IA or IC agents directed at altering conduction of the fast pathway may be considered. beta blockers. or coronary ischemia) require urgent electrical cardioversion. A. since irregular wide-complex C. Intravenous adenosine multifocal atrial tachycardia. 1888 D. treatment A. Atrioventricular nodal reentrant tachycardias eliminates symptoms. Coronary ischemia D. Atrial tachycardias slow pathway in atrioventricular nodal reentrant tachycardia ? D. and torsades de pointes or polymorphic ventricular tachycardia) or supraventricular tachycardias with aberrant conduction resulting from disease in His–Purkinje system (left or right B. Left ventricular hypertrophy A. Slurred QRS upstroke (delta wave) angle of jaw for up to 10 seconds C. PR depression 531 Which of the following about atriofascicular accessory D. produces the ECG pattern described by Wolff. Patient should be supine with neck extended D. All of the above A. Focal atrial tachycardias C. Short PR interval C. None of the above C. There may be no delta wave in the ECG of concealed APs is ? C. The classic accessory pathway is the AV digoxin toxicity ? bypass tract or bundle of Kent in WPW that directly connects atrial and ventricular myocardium. APs connect the atria with the ventricles 535 Most common arrhythmia associated with WPW syndrome B. James fibers 527 Junctional tachycardia due to abnormal automaticity can be B. Have decremental antegrade conduction B. Acute pericarditis D. B. B. Bundle of Kent treated pharmacologically with ? Harrison’s 18th Ed. C. Beta blockers Preexcitation through an AV bypass tract. Prolonged QRS interval of > 0. 1889 A. Marfan’s syndrome 532 Which of the following best relates to normal PR interval with a delta wave in ECG ? 538 The WPW ECG may mimic the following ? Harrison’s 18th Ed. Atrioventricular nodal reentrant tachycardia 530 The most common accessory pathway (AP) connects which of C. Conduct more slowly C. Atrioventricular reciprocating tachycardia B. Shortened PR interval of < 0. connect atrium to distal or compact AV node. 1889 D. Right atrium to left ventricle A. Brechenmacher fibers .349:1787 D. Bundle of Kent C. Performed by applying firm pressure just underneath the B. Decremental conduction means increase in conduction time of the impulse propagating through the AV node as the cycle length shortens Junctional tachycardia due to abnormal automaticity can be treated with beta blockers. Mahaim fibers connect the AV node to the fascicles. Presence of delta wave The most common AP connects the left atrium to the left ventricle. All of the above D. Calcium channel blockers atrial & ventricular myocardium.349:1787 B. Prolonged QRS complex D. Not performed in patients with carotid arterial bruits N Engl J Med 2003. 1888 C. classic electrocardiographic triad is a short PR interval.12 seconds right free wall.146 MCQ’s FOR MEDICAL PROFESSIONALS BY PROF. Mahaim fibers D. B. 1888 bypassing the AV node/His-Purkinje system. 1889 D. Right atrium to right ventricle C. James fibers or atrionodal tracts. AV nodal reentrant tachycardia A. Massage one carotid bulb at a time A. AV conduction is nondecremental and more rapid through the accessory pathway than through the AV node. Multifocal atrial tachycardia A. followed by posterior septal.12 seconds pathways is false ? Harrison’s 18th Ed. Bundle branch block C. Right ventricular hypertrophy B. A. 528 Which of the following is false for carotid sinus massage ? 534 Classic electrocardiographic triad in WPW syndrome includes Harrison’s 16th Ed. Digitalis D. 1344 all except ? A. and anterior septal APs. Left atrium to right ventricle 536 The following are ECG features of WPW syndrome except ? D. Parkinson. James fibers B. (heart rate increased). ST elevation myocardial infarction B. Atrial tachycardia the following ? Harrison’s 18th Ed. 1889 537 Which of the following are known conditions associated with A. Mahaim fibers A. Brechenmacher fibers B. the bundle of Kent. Harrison’s 18th Ed. a slurred QRS pathways (APs) is correct ? upstroke (delta wave). All of the above difference that is increased at fast heart rates. Bundle of Kent in WPW directly connects C. Originate from the right atrium WPW syndrome ? A. Left atrium to left ventricle Most common arrhythmia associated with WPW syndrome is atrioventricular reciprocating tachycardia. Multifocal atrial tachycardia His bundle. bypassing the AV node/His-Purkinje system. a D. AV junctional tachycardias 533 Wolff-Parkinson-White syndrome is best related to ? Harrison’s 18th Ed. Mahaim fibers typically manifest a normal PR interval with a delta wave. Brechenmacher fibers (atrio-Hisian tracts) connect the atrium to A. Ebstein anomaly D. Prolonged QT interval 529 Which of the following statements regarding accessory In WPW syndrome. AJAY MATHUR Cardiology 526 Which of the following arrhythmia may be a manifestation of There are several types of accessory pathways. and White in 1930. More than 50% of APs are located at the left free wall N Engl J Med 2003. and a prolonged QRS complex. Prolonged QT interval A VPC has a wide QRS complex that is typically >140 ms in duration. bizarre QRS C. Sustained AIVR is seen in acute MI & postoperatively. May occur in up to 80 % of patients with previous MI B. Atrial flutter and AF are common C. 1351 trigeminy. 1890 A. Antiarrhythmic agents can produce lethal arrhythmias Harrison’s 16th Ed. Quinidine or flecainide slow conduction and increase refractoriness primarily of the AV node 548 Which of the following about AIVR is false ? Harrison’s 18th Ed. in which two sinus beats are followed by a VPC. 547 Accelerated idioventricular rhythm (AIVR) has overlap features White syndrome ? with ? Harrison’s 16th Ed. VPC produces slow ventricular activation and a wide QRS complex that is typically >140 ms in duration. Short PR interval (< 0. They result in a fully compensatory pause postoperative cardiac surgery. ventricular parasystole C. VSD VPC typically does not conduct to the atrium. 1350 A. D. > 10 seconds A. Atrial-like muscle 545 Which of the following statements about VPCs is false ? B. 147 Cardiology MCQ’s FOR MCQ’s FOR MEDICAL MEDICAL PROFESSIONALS PROFESSIONALS BY PROF. Ventricular-like muscle Harrison’s 18th Ed. 1891 543 Which of the following statements about VPCs is false ? Harrison’s 18th Ed. 541 Which of the following is not a feature of Wolff-Parkinson-White 546 QRS duration of a ventricular premature complex (VPC) is ? (WPW) syndrome ? Harrison’s 18th Ed. TOF retrograde to atrium to reset the sinus node. Two successive VPCs are termed pairs or couplets. Wide QRS complex D. acute myocarditis. > 120 ms B. Polymorphic VT B. Heart rate ranges from 60 to 120 beats/min A. 549 Sustained ventricular tachycardia is defined as VT that persists for ? 544 Which of the following statements about VPCs is false ? Harrison’s 18th Ed. Two successive VPCs are termed pairs or couplets Hemodynamically unstable VT that requires termination before 30 seconds or VT that is terminated by therapy from an implantable defibrillator is also typically classified as sustained. During PSVT in WPW. in which every sinus beat is followed by a VPC. > 30 seconds B. When they arise in specialized conduction system (fascicles). 1890 C. D. None of the above unusually rapid and may cause VF By definition there is an overlap between AIVR and “slow” VT. > 130 ms C. > 15 seconds VPCs has a common denominator. VPCs are associated with a “fully compensatory pause” i. VPC that does not produce retrograde concealed conduction and fails to influence oncoming sinus impulse 540 AV bypass tracts are associated with which of the following is termed interpolated VPC congenital abnormalities ? C. Three or more consecutive VPCs are termed VT when the rate is >100 beats/min. or Harrison’s 16th Ed. Occasionally VPC can occur early enough & conduct C. Transient AIVR is frequently seen in acute myocardial infarction.12 s). > 110 ms A. AIVR refers to a ventricular rhythm that is characterized by three or more complexes at a rate they may be < 0. ASD D. Pause that results will be less than compensatory. impulse is conducted antegradely over A. 542 Which of the following is not true regarding Wolff-Parkinson. cocaine intoxication.) B. 1350 A. 1350 Harrison’s 18th Ed. duration between last QRS before PVC and next QRS complex is equal to twice the sinus rate. >= 3 consecutive VPCs are termed ventricular tachycardia when the rate exceeds 160 beats/min 550 Which of the following is not true for sustained VT ? VPCs may occur in patterns of bigeminy. AJAY MATHUR Cardiology 147 539 AV bypass tract in ventricular preexcitation are composed of ? and are thus referred to as multiformed. D. Occurs in acute MI B. Ventricular impulses are never conducted retrogradely to atrium D. Treatment consists of atropine and atrial pacing. 1891 Harrison’s 18th Ed. 1891 A. Harrison’s 16th Ed.12 s in duration >40 and <120 beats/minute due to abnormal automaticity. 1350 A. 1890 Harrison’s 16th Ed. Monomorphic VT normal AV system and retrogradely through bypass tract B. Treatment consists of  blockers C. VPCs may have different morphologies . Usually transient complexes not preceded by P waves D. digoxin intoxication & D. Slow VT C. C.12 sec. On ECG show wide (usually > 0. > 140 ms D. Trigeminy means two sinus beats followed by a VPC A time duration of 30 seconds is frequently used to distinguish sustained from nonsustained VT. Specialized conduction tissue A. Ventricular responses during atrial flutter or fibrillation is D. > 20 seconds is said to be present D. 1890 A. Slurred upstroke of QRS complex (delta wave) C.e. When fixed coupling is not present and interval between B. Ebstein’s anomaly VPCs that fail to influence the oncoming sinus impulse are termed interpolated VPCs. All of the above B. Prophylactic antiarrhythmic therapy is recommended only for young patients with complicated MI B. 1894 556 Characteristics ECG findings that suggest VT are all except ? Harrison’s 18th Ed. 1351 A. 1893 Harrison’s 16th Ed. 1894 Harrison’s 18th Ed. acute IV administration of lidocaine. 1891 A. IV lidocaine A. 100 to 150 beats/minute C. 1892. bizarre QRS complex that does not mimic the C. Monomorphic VT B. 1893 551 The heart rate in ventricular flutter usually is ? A. IV amiodarone C. and slurring of initial portion of QRS. Ventricular fibrillation D. AV dissociation B. Intravenous nitroglycerine C. 1891 B. Not associated with SCD B. AJAY MATHUR Cardiology A. ventricular flutter. VT flow C. slurring of initial portion of QRS. Torsades de pointes In patients with recurrent monomorphic VT. Bidirectional tachycardia D. Concordance of QRS pattern in all precordial leads C. Ventricular flutter appears as a sine wave on the ECG and has a rate of >250 beats/minute. Atrial flutter B. procainamide. 1892 A. Benign VT plane axis. Polymorphic ventricular arrhythmias B. Almost always symptomatic D. Inferior QRS frontal plane axis B. VT storm Harrison’s 18th Ed. Ventricular flutter C. 1894 B. 1893 the following arrhythmia ? Harrison’s 18th Ed. IV procainamide B. Polymorphic ventricular arrhythmias. 150 to 300 beats/minute D. All of the above ~80% of outflow tract VTs originate in RV & ~20% in LV outflow tract regions. Cycle length oscillations are common C. Any of the above D. VT paroxysm A. Mostly associated with marked hemodynamic compromise VT on ECG is suggested by presence of QRS duration >140 ms in the absence of drug therapy. Acute ischemia is responsible for most recurrent episodes of sustained uniform VT 557 Repeated VT episodes requiring defibrillation is called ? Harrison’s 18th Ed. superior frontal D. III & aVF A. Polymorphic VT C. 450 to 600 beats/minute ICD shock therapy are referred to as VT storm. Ventricular flutter C. More commonly in men . Superior and rightward QRS frontal plane axis B. Intravenous calcium channel blocker B.148 MCQ’s FOR MEDICAL PROFESSIONALS BY PROF. Idiopathic VT D. VT in the absence of structural heart disease is called idiopathic VT. Intravenous digoxin D. D. Table 233–6 560 VT in the absence of structural heart disease is called ? A. Casual VT Ventricular tachycardia on ECG shows AV dissociation. and large S wave in V 6. QRS complex > 0. Intravenous beta blocker A. Torsades de pointes D. or amiodarone can prevent recurrences. Produces monophasic R waves in leads II. 559 Which of the following can prevent recurrences in patients with recurrent monomorphic VT ? 553 VT that shows an alternation in QRS axis is called ? Harrison’s 18th Ed. Bizarre QRS complex mimiking LBBB or RBBB QRS C. 554 Characteristics ECG findings that suggest VT are all except ? Harrison’s 18th Ed. 555 Which of the following always produce hemodynamic collapse 561 Which of the following about idiopathic outflow tract VT is false ? if allowed to continue ? Harrison’s 18th Ed. 558 Which of the following should be considered for polymorphic 552 It is not possible to assign a specific morphology to which of VT storm ? Harrison’s 18th Ed. and VF always produce hemodynamic collapse if allowed to continue. Almost always leads to development of myocardial ischemia characteristic QRS pattern associated with LBBB or RBBB. wide QRS >200 ms. 1891 A. Adenosine & beta blockers terminate them D. WPW syndrome with AF Intravenous beta blockade therapy should be considered for polymorphic VT storm. Slurring of the initial portion of QRS D. A rapid rate with sine wave oscillations of ventricular flutter make it impossible to assign it a specific morphology and in some cases to distinguish it from rapid VT. VT battle B. 562 Which of the following about idiopathic outflow tract VT is false ? Harrison’s 18th Ed. AV dissociation A. Figure 233-10. Typically occur as nonsustained bursts of VT B. Vagal maneuvers terminate them C.20 seconds Harrison’s 18th Ed. 300 to 450 beats/minute Repeated VT episodes requiring external cardioversion/defibrillation or repeated appropriate D. Functional VT C. superior & rightward QRS frontal plane axis. ~80% of outflow tract VTs originate in LV A. PRKAG2 mutation D. bundle branch reentrant VT presents as an incomplete left bundle block. Good short-term prognosis cardiomyopathy associated with which mutation ? B. Tricuspid valvular region Characteristically. AJAY MATHUR Cardiology 149 Acute medical therapy for idiopathic outflow tract VT is rarely required because the VT is B. Torsades de pointes 567 In ECG. CRYAB 563 Which of the following can terminate the idiopathic outflow WPW syndrome is seen in patients with hypertrophic cardiomyopathy associated with PRKAG2 tract ventricular tachycardia ? mutations. WPW syndrome with AF A. 1354 568 WPW syndrome is observed in patients with hypertrophic A. Ventricular tachycardia A. Chagas disease C. exudation & retinal C. Emery-Dreyfuss muscular dystrophy cardiomyopathy can be mapped to ? Harrison’s 18th Ed. Table 233-7 C. IV amiodarone A. Incomplete left bundle block IHD is mostly responsible for VF which also occur with antiarrhythmic drugs. 1895 C. 1894 A. Valsalva maneuver or adenosine D. Verapamil is associated with hemodynamic collapse when administered to patients with structural heart disease and VT. Good long-term prognosis Harrison’s 18th Ed. All of the above administration. Complete left bundle block 574 Patients who have primary VF within first 48 hours of onset of acute infarction have ? In sinus rhythm. 149 Cardiology MCQ’s FOR MCQ’s FOR MEDICAL MEDICAL PROFESSIONALS PROFESSIONALS BY PROF. TDP and WPW syndrome who develop AF with an extremely rapid ventricular response. Mitral valvular region D.or polymorphic. Facioscapulohumeral muscular dystrophy 566 Most uniform sustained VT associated with LV dilated B. Harrison’s 18th Ed. Sarcoidosis D. IV administration of verapamil and/or adenosine is not recommended as a diagnostic test. They are more common in women. Limb-girdle muscular dystrophy A. Harrison’s 18th Ed. Amyloidosis Most VTs do not respond to carotid sinus massage. 1895 C. IV lidocaine 569 Infiltrative / Inflammatory & neuromuscular disorders B. Atrial fibrillation C. Myotonic muscular dystrophy Intravenous beta blockers frequently terminate idiopathic outflow tract ventricular tachycardia. S135L mutation hemodynamically tolerated and is typically nonsustained. A. 1896. Myopathic Harrison’s 16th Ed. All of the above B. C. ACTN2 D. IV beta blockers B. Intravenous beta blockers frequently terminate the tachycardia. All of the above ventricular arrhythmia risk include all except ? Idiopathic LV septal VT responds uniquely to IV verapamil. Idiopathic LV septal VT C. Fabry disease D. C. 1895 570 Infiltrative / Inflammatory & neuromuscular disorders associated with increased ventricular arrhythmia risk include ? A. Table 233-7 B. Duchenne’s muscular dystrophy B. All of the above 573 Ventricular fibrillation can occur due to ? VT associated with LV dilated cardiomyopathy may be mono. 1895 C. Pulmonary valvular region detachment) also occurs. Becker’s muscular dystrophy Harrison’s 18th Ed. Harrison’s 16th Ed. Antiarrhythmic drugs B. Accessory pathway mediated tachycardias B. None of the above . Table 233-7 verapamil ? A. Kearn-Sayre syndrome 564 Verapamil is least effective / contraindicated in which of the D. Friedreich’s ataxia B. 1896. Bundle Branch Reentrant VT 572 Neuromuscular disorders associated with increased D. 1354 process has a predilection for fibrosis around mitral & aortic valvular regions. Idiopathic LV septal VT Harrison’s 18th Ed. VT associated with LV dilated cardiomyopathy D. 1896. Complete right bundle block D. 1896. Incomplete right bundle block D. IV procainamide associated with increased ventricular arrhythmia risk include ? Harrison’s 18th Ed. D. Table 233-7 A. 1895 B. All of the above C. All of the above following ? Harrison’s 18th Ed. Poor long-term prognosis A. 571 Infiltrative / Inflammatory & neuromuscular disorders associated with increased ventricular arrhythmia risk include ? 565 Which of the following is unique in its suppression with Harrison’s 18th Ed. bundle branch reentrant VT presents as ? Harrison’s 18th Ed. Labile hypertension & nerve deafness is common. Coats’ disease (telangiectasia. Most uniform sustained VT are mapped to these regions. FSH muscular dystrophy does not involve other organ systems. Hemochromatosis A. C. The 577 In arrhythmogenic RV cardiomyopathy/dysplasia (ARVCM/D). B. Long QT syndrome (LQTS) A. AJAY MATHUR Cardiology Most patients who have primary VF within the first 48 hours of the onset of acute myocardial B. 1897 Schwartz scoring system for LQTS includes ECG features. Jervell and Lange-Nielsen first described a family with long QT syndrome (LQTS). Arrhythmogenic RV dysplasia 585 Schwartz scoring system for diagnosis of LQTS includes all B. QT x Square root of RR 576 Epsilon wave is characteristic ECG finding of ? QT interval is from the onset of QRS complex to the end of the T wave. Parapsoriasis A. Lack of sleep Harrison’s 18th Ed. QT / Square root of RR ARVCM/D due to a genetically determined dysplastic process or after a suspected viral myocarditis is also associated with VT/VF. four of whom were deaf with frequent fainting attacks precipitated by acute emotional arousal and exercise. Genetic information D. Family history 580 Which of the following about digoxin toxicity is false ? Harrison’s 18th Ed. None of the above 575 Which of the following is related to arrhythmogenic RV The signature VT associated with digoxin toxicity is bidirectional VT due to triggered activity cardiomyopathy/dysplasia (ARVCM/D) ? associated with calcium overload resulting from inhibition of Na + /K + ATPase by digoxin. personal & family history. 5. C. family consisted of unrelated parents and their six children. QT x Square root of PR cardiomyopathy is also seen. reported separate families with QT prolongation in one parent and several children. Sporadic A. echocardiography shows RV enlargement with RV wall motion abnormalities and RV apical aneurysm formation. In 1963 and 1964.150 MCQ’s FOR MEDICAL PROFESSIONALS BY PROF.358:169-76 MRI in arrhythmogenic RV cardiomyopathy/dysplasia (ARVCM/D) shows fatty replacement of A. C. 1896 D. B. Levine and Woodworth C. Sporadic nonfamilial/nondysplastic form of RV C. Fat sudden death (autosomal dominant mode of inheritance). RV enlargement polymorphic ventricular tachycardia called torsades de pointes. Myocardial cell calcium overload . In 1957. Genetically determined QRS axis. Woolly hair B. Arrhythmogenic RV cardiomyopathy / dysplasia B. Mucopolysaccharide 583 Syncope in long-QT syndrome is attributed to ? D. Electrolyte disturbances C. Bidirectional VT from left anterior & posterior fascicles infarction have a good long-term prognosis. RV wall motion abnormalities C. Personal history D. QT / Square root of PR D. Bazett’s formula (QT Harrison’s 18th Ed. B. and 9 years. and associated wall motion abnormalities. with a very low rate of recurrence or sudden cardiac death. Brugada Syndrome 582 Long QT syndrome (LQTS) was first described by ? C. 1896 Bidirectional VT originates from left anterior & posterior fascicles creating a relatively narrow QRS right bundle branch configuration with beat-to-beat alternating right & left frontal plane A. 1896 Syncope in patients with the long-QT syndrome is generally attributed to the form of A. B. 1896 interval is adjusted for heart rate) for the calculation of QTc is QT interval (in seconds) divided by the square root of the RR interval (in seconds). Physical stress C. but does not take into account genetic information. After viral myocarditis 581 Bazett’s formula for the calculation of QTc is ? C. Jervell and Lange-Nielsen Epsilon wave is the terminal notching of QRS complex & is separated from the QRS complex. Palmar-plantar keratosis except ? N Engl J Med 2008. Digoxin toxicity B. thinning of the RV free wall with increased fibrosis. Any of the above B. Romano Harrison’s 18th Ed.358:169-76 D. Narrow QRS right bundle branch configuration D. Death is usually due to ventricular fibrillation. Diving and swimming 579 Which of the following about Naxos disease is false ? Harrison’s 18th Ed. Emotional stress In patients with ARVCM/D. Three of the four deaf ventricles have an excess of ? children died suddenly while playing at the ages of 4. ECG features Naxos disease consists of arrhythmogenic RV dysplasia with palmar-plantar keratosis & woolly hair with a high risk of SCD in adolescents & young adults. C. All of the above A. 1896 and Ward respectively. A. Emotional or physical stress the ventricle. It is seen in ARVCM/D. Glycogen LQTS families. Romano and Ward D. RV apical aneurysm formation 584 LQT1-specific trigger is ? N Engl J Med 2008. all of whom possessed normal hearing but experienced recurrent syncope and A. In 1979 Moss and Schwartz established the prospective International LQTS Registry for enrollment and follow-up of proband-identified B. D. All of the above A. A.358:169-76 C. Epsilon waves are due to marked delay in ventricular activation in RV D. Harrison’s 18th Ed. Moss and Schwartz free wall near the base of tricuspid & pulmonic valves which undergo extensive fibrosis. Torsades de pointes 578 Echocardiographic finding in arrhythmogenic RV cardiomyopathy/dysplasia (ARVCM/D) is ? D. All of the above N Engl J Med 2008. None of the above Besides LQTS. The risk of 595 Which of the following do not cause QT prolongation ? cardiac events is higher in males before puberty and higher in females during adulthood. Holter monitoring 593 Which of the following is false about LQT3 ? Harrison’s 18th Ed. D. Holter monitoring and electrophysiological testing show no abnormalities in LQTS. Due to a mutation in cardiac sodium channel gene on Harrison’s 18th Ed. Disopyramide. Due to a mutation in a potassium-channel gene on C. while opposite was true in LQT3. Harrison’s 17th Ed. Marked lengthening of the QT interval to > 500 ms is clearly associated with a greater arrhythmia risk in patients with the LQTS. T wave is broad Class IB antiarrhythmic agents (lidocaine) do not cause QT prolongation. 1898. All of the above B. None of the above Dofetilide. QTc duration Harrison’s 18th Ed. All of the above D. Exercise is not restricted in LQT3 Physical examination. 1898 598 In short QT syndrome. 1884. 1898 chromosome 3 A. Echocardiography B. Respond to beta blocker therapy B. < 320 ms B. QT interval is ? Harrison’s 18th Ed. 589 Which of the following is false about LQT1 ? Harrison’s 18th Ed. LQT3 is caused by a mutation in sodium channel gene on chromosome 3 (SCN5A) D. Most common genotypic abnormality in LQTS C. Beta blockers are not recommended D. Male patients have worst prognosis in LQT3 The most powerful predictor of risk is the QTc duration. Lidocaine B. Ibutilide Harrison’s 18th Ed. < 340 ms C. notched and bifid B. None of the above A. QT interval fails to shorten with exercise D. 1898 C. 1898 QT syndrome ? A. 592 Which of the following is false about LQT3 ? 599 Mutations in which of the following gene is the cause of short Harrison’s 18th Ed. 1441 594 Which of the following is false about LQT3 ? A. Due to mutation in potassium-channel gene (KCNQ1) on C. None of the above 597 Which of the following is a repolarization abnormality ? 590 Which of the following is false about LQT2 ? Harrison’s 18th Ed. Arrhythmogenic right ventricular cardiomyopathy B. Quinidine.358:169-76 D. & short QT syndrome. 1898 A. Risk of syncope & sudden death increased in postpartum period A. HERG B. All of the above D. arrhythmogenic right ventricular cardiomyopathy. Most events in LQT3 patients occur during sleep C. Exercise is the most common trigger for arrhythmias A. Table 233-5 588 Which of the following is false about LQT1 ? A. Respond to beta blocker therapy B. KvLQT1 . Harrison’s 18th Ed. Brugada syndrome A. Sotalol prolong QT interval prolong QT and may precipitate torsades des pointes. 591 Which of the following is false about LQT2 ? Harrison’s 18th Ed. Hypokalemia chromosome 11 D. Emotional stress /startle triggers arrhythmias C. Disopyramide C. and no sex bias occurs in LQT1. 151 Cardiology MCQ’s FOR MCQ’s FOR MEDICAL MEDICAL PROFESSIONALS PROFESSIONALS BY PROF. T wave is low amplitude. History of syncope B. Syncope and sudden death are most frequent in childhood and adolescence. QT shortening occurs with mexiletine D. Hypocalcemia B. None of the above A QT interval < 320 ms is required to establish the diagnosis of Short QT syndrome. 1884 A. Electrophysiological testing A. 1898 B. Sleep or auditory stimulation triggers arrhythmias D. Gender C. C. Late-onset peaked biphasic/asymmetric T waves B. Hypothyroidism C. Procainamide. < 360 ms chromosome 7 (KCNH2 or HERG) D. Sotalol A. None of the above 587 Most powerful predictor of risk in LQTS is ? N Engl J Med 2008. 1898 B. 1898 596 Which of the following prolongs QT interval ? Harrison’s 18th Ed. AJAY MATHUR Cardiology 151 586 Which of the following is not useful in diagnosis of LQTS ? C. None of the above females fared worse than their males. MRI.358:169-76. Family history A. In LQT2. Short QT syndrome C. echocardiography. other repolarization abnormalities are Brugada syndrome (opposite of LQT3). Ibutilide. Prognosis for LQT3 is the poorest of all LQTs N Engl J Med 2008. 1898 A. D. < 380 ms D. Pulmonary embolism C. ST segment usually upsloping C. 606 Which of the following is the cause of ST-segment elevation ? N Engl J Med 2003. Hyperkalemia D. with ST segment usually downsloping. KvLQT1 & KCNJ2 genes cause of Short QT syndrome. Short-QT syndrome A. Prominent U waves 607 ST-segment elevation in normal healthy young men (male In Short QT syndrome. Ajmaline B.353:2492-501 605 Which of the following is false about Brugada syndrome ? A. a shorter-than-expected QT interval was noted in patients with chronic fatigue A. Transient point.349:2128-35 A.349:2128-35 603 ECG manifestations of Brugada syndrome are provoked with ? Harrison’s 18th Ed. reciprocal ST- segment depression in aVR. hyperkalemia. androgen use. D. Low-amplitude or absent P waves B. 1898 D. N Engl J Med 2003. tented T waves A. Patients with the N Engl J Med 2003.349:2128-35 600 Which of the following is a feature of Short QT syndrome ? Harrison’s 18th Ed. Any of the above Procainamide & flecainide exacerbate Brugada syndrome. with notching at J B. tented T waves. acidosis. It is syndrome of RBBB & nonischemic ST-segment elevations. when limb leads are C. Procainamide D. digitalis use.349:2128-35 syndrome are predisposed to both AF and VF. B. T waves tends to be tall and peaked especially in leads V2 to V4 and pattern) is seen in ? may be interpreted as R waves by an implantable cardioverter-defibrillator. V3 B. V4 A. All of the above In acute pericarditis. V3 Harrison’s 18th Ed. AJAY MATHUR Cardiology C. V2 A. Tall and peaked T waves D. Hypercalcemia C. 1898 A. Long-QT syndrome Harrison’s 18th Ed. Short PR interval B. Procainamide & flecainide are beneficial D. Acute pericarditis A. All of the above D. Manifest ST-segment elevation due to early repolarization is most marked in V4. not in aVL.152 MCQ’s FOR MEDICAL PROFESSIONALS BY PROF. Flat P waves C. V2 602 In Brugada syndrome. Left bundle branch block pattern C. Terminal T-wave inversion in leads V1 – V3 In Hyperkalemia. flecainide. All of the above C. peaked. N Engl J Med 2003.349:2128-35 Interestingly. V1 syndrome. peaked. not in aVL and PR-segment depression. V4 C. D. All of the above D. Most common in young Asian male C. Reciprocal ST-segment depression in aVR. 1898 A. ECG shows diffuse ST-segment elevation (seldom >5 mm). Affected region is RV outflow tract epicardium D. Any of the above 609 Which of the following is an ECG feature of acute pericarditis ? N Engl J Med 2003. reciprocal ST depression in aVR. tall. ST segment elevation in V1 . ST elevation leads V1 – V3 D. Mutations in the HERG. The major clinical features of autosomal dominant Brugada syndrome include manifest. Quinidine is helpful. A. low-amplitude or absent P waves. Hyperkalemia B. increased vagal tone. All of the above is most marked in V2. right bundle-branch block (rSR’) and downsloping ST-segment elevation in lead V1 is 611 Which of the following “Channelopathies” can cause Ventricular typical of Brugada syndrome.V3 may be ? C. Brugada syndrome B. V1 601 Which of the following can shorten QT interval ? B. upright T waves. or concealed ST segment elevation in V 1 to V 3 that can typically be provoked with sodium channel-blocking drugs ajmaline. Conditions that can shorten QT interval include hypercalcemia (with an accompanying prolonged 608 ST-segment elevation in early repolarization is seen in ? PR interval and a wide QRS complex). after ventricular fibrillation (due to increased intracellular calcium). transient. not in aVL B. Diffuse ST-segment elevation A. Due to mutation in cardiac sodium channel SCN 5A In ECG. Concealed involved. Widened QRS 604 Which of the following is false about Brugada syndrome ? Harrison’s 18th Ed. Fibrillation–Induced Cardiac Arrest ? N Engl J Med 2005. 1898 B. 1898 B. tall. ECG shows widened QRS. Patients do not benefit from beta blocker therapy . cocaine and procainamide and a risk of polymorphic 610 Which of the following is not an ECG feature of hyperkalemia ? ventricular arrhythmias. Tall. PR-segment depression C. Acidosis Concave ST-segment elevation of 1 .3 mm seen in ~90 % of healthy young men (male pattern) D. Flecainide C. KCNJ2 D. Hypertrophic cardiomyopathy C. The arrhythmias are precipitated by exercise & emotional D. Ibutilide B. Recurrent VF B. B. Sepsis) and Neurologic (Seizure. which controls intracytoplasmic Harrison’s 16th Ed. Accumulation of multiple classes in Vaughan-Williams classification ? intracellular calcium potentiates delayed afterdepolarizations and triggered activity. Phenytoin C. All of the above 618 Which of the following drug does not fit in Vaughan-Williams Noncardiac causes of Ventricular Fibrillation Induced Cardiac Arrest are Respiratory (Bronchospasm. Bronchospasm B. 623 Which of the following ‘Genetically determined arrhythmia 616 Which of the following is useful for patients with congenital syndromes’ have autosomal dominant inheritance ? prolonged QT interval syndrome ? Harrison’s 16th Ed. PKA dissociates calstabin from RyR2. 153 Cardiology MCQ’s FOR MCQ’s FOR MEDICAL MEDICAL PROFESSIONALS PROFESSIONALS BY PROF. Cardiac cellular excitatory current Two ECG markers of abnormalities in repolarization are QT-interval dispersion & T-wave alternans. Excessive plasma catecholamine levels B. Automaticity T-wave alternans is defined as alternating T-wave amplitude from beat to beat on special ECG recording techniques. Regulatory protein. nonsustained polymorphic VT. Primary pulmonary hypertension. 1355 poisoning. 1355 [Ca2+] and leads to local changes in intracellular [Ca 2+] called calcium sparks. B2-Bradykinin receptor B. Adenosine 613 Which of the following is an ECG marker of abnormality in C. Antiarrhythmic drugs B. Treatment with beta blockers & ICD implantation is recommended. All of the above stress. D. Cardiac cellular excitatory current B. Amiodarone and cardiac sympathetic neuronal release of norepinephrine cause hyperphosphorylation of PKA. Nonsustained polymorphic VT A. Cerebrovascular accident: intracranial hemorrhage.345:1476 D. Action potential duration They identify patients who are at risk for sudden death from arrhythmia. 1355 Harrison’s 18th Ed. 1355 B. A. Ryanodine receptor C. C. Propafenone 615 Patients of catecholaminergic polymorphic VT may present Amiodarone is consistent with multiple classes. 1899 622 Which of the following is included in ‘Genetically determined A. LQTS A. and also triggers ventricular arrhythmias. 1355 N Engl J Med 2005. All of the above 614 Catecholaminergic polymorphic VT is best related to ? 620 Class I and IV antiarrhythmics exert their action through ? Harrison’s 16th Ed. or ischemic stroke). QT-interval dispersion 619 Class III antiarrhythmics exert their action through ? C. Brugada syndrome B. Bretylium repolarization ? N Engl J Med 2001. D.353:2492-501 A. A. Seizure D. All of the above D. 1353 A. or recurrent VF. Vasopressor drugs D. QT-interval dispersion refers to difference between maximal & minimal QT intervals from various leads on standard ECG. which creates a “leak” in calcium from sarcoplasmic reticulum (SR). 1899 A. classification of antiarrhythmic drugs ? Metabolic or toxic (Electrolyte disturbances. Action potential duration C. All of the above In catecholaminergic polymorphic VT there occurs a mutation of myocardial ryanodine release 621 Which of the following drug exhibits properties consistent with channel. LQTS C. Quinidine calstabin 2 inhibit RyR2 and thereby release of Ca 2+ from SR. All of the above . Pulmonary embolism). 1355 Harrison’s 16th Ed. 1355 D. Hypertrophic cardiomyopathy Patients of catecholaminergic polymorphic VT can manifest bidirectional VT. with ? Harrison’s 18th Ed. leading to heart failure. AJAY MATHUR Cardiology 153 612 Noncardiac causes of Ventricular Fibrillation Induced Cardiac 617 Vaughan-Williams classification is used to classify ? Arrest include ? Harrison’s 16th Ed. All of the above A. Automaticity D. Sleep apnea. Vasodilator drugs C.ryanodine receptor (RyR2). Cervicothoracic sympathectomy D. Ca 2+ is released from SR through a Ca2+ release channel . Mexiletine contraction. Aspiration. PR-interval dispersion Adenosine does not fit into VW classification. Cardiotonic drugs A. Environmental Harrison’s 16th Ed. enhancing Ca 2+ release and thereby myocardial contractility. Ang II type I receptor A. Propafenone A. J-point dispersion Harrison’s 16th Ed. Sleep apnea C. Brugada syndrome D. The latter depletes SR Ca 2+ stores and thereby impairs cardiac C. Medications or drug ingestion. B. All of the above D. leading to calstabin 2–depleted RyR2. Bidirectional VT arrhythmia syndrome’ ? Harrison’s 16th Ed.  adrenergic blocking agents B. Any of the above C. 50 per minute Harrison’s 17th Ed. Rapid defibrillation C. Pulmonary infiltrates N Engl J Med 2001. Any of the above erythematosus like syndrome’ ? In cardiopulmonary resuscitation. 10 : 1 630 Which of the following antiarrhythmic drugs can cause taste disturbance ? B. 2 to 3 cm B. 10 : 2 Harrison’s 17th Ed. Amiodarone The sequence of rapid access. QT prolongation & hypokalemia or hypomagnesemia and at the time of conversion from atrial fibrillation to sinus rhythm. Bronchospasm C. Mexiletine B. Upper half of the sternum C. the ratio should be five compressions to one ventilation. 1430 Table 226-4 the chest on the lower half of the sternum. Procainamide 634 In cardiopulmonary resuscitation in adults.344:1311 B. Mickey Isenberg C. 1430 Table 226-4 D.154 MCQ’s FOR MEDICAL PROFESSIONALS BY PROF. chest compression should Harrison’s 17th Ed. the rate of chest C. 15 : 2 B. Lower half of the sternum 627 Which of the following antiarrhythmic drugs can cause ‘Lupus D. Mexiletine A. 1356 Table 214-8 defibrillator ? A. Rapid access A. Claude Beck is one of the inventors of cardiac defibrillation. Disopyramide When cardiopulmonary resuscitation is performed by two persons. After endotracheal intubation. 625 Which of the following antiarrhythmic drugs is least likely to 632 The first intervention in the “chain of survival” is ? produce bradycardia ? N Engl J Med 2001. the depth of chest compression should be 4 to 5 cm in adults. Lidocaine N Engl J Med 2001. 5 to 6 cm A. 15 : 1 A. Mexiletine N Engl J Med 2001. Procainamide D. 1430 Table 226-4 be administered in the center of the chest on the ? A. Bardy GH D.344:1311 Harrison’s 17th Ed. Rapid advanced care D. Rapid cardiopulmonary resuscitation B. Claude Beck D. A.344:1311 D. Lidocaine compression should be approximately ? D.344:1311 A. 3 to 4 cm 628 Which of the following antiarrhythmic drugs can cause seizures ? C. 1430 Table 226-5 A. rapid defibrillation. Sotalol the ventilatory breath. Photosensitivity B. 75 per minute A. Propafenone to breaths remains 15:2 with pause during ventilations. chest compression should be administered in the center of Harrison’s 17th Ed. . Disopyramide compression should be ? C. Disopyramide 635 In cardiopulmonary resuscitation in adults. the ratio of chest compressions C. Hypokalemia B. 100 per minute B. rapid cardiopulmonary resuscitation.344:1311 A. Procainamide D.344:1311 B. and rapid advanced care is termed the chain of survival. Terry Mengert TDP occurs most often with slow heart rates. Flecainide C. Middle of the sternum D. 4 to 5 cm Harrison’s 17th Ed. Procainamide In CPR in adults. 1430 Table 226-4 C. Slow heart rate N Engl J Med 2001. 30 per minute 629 Which of the following antiarrhythmic drugs can cause congestive heart failure ? B. was one of the inventors of cardiac Harrison’s 16th Ed. Mexiletine N Engl J Med 2001. Propafenone D. and there should be no pause in chest compressions for D. Disopyramide 636 In cardiopulmonary resuscitation performed by two persons. All of the above C. B. Hepatitis A. QT prolongation A. 626 Which of the following is not an adverse effect of amiodarone ? 633 In cardiopulmonary resuscitation. 1430 Table 226-4 C. AJAY MATHUR Cardiology 624 Torsades de pointes occurs most often in the setting of ? 631 Who. from the following. Lidocaine the ratio of compressions to breaths is ? D. C. the depth of chest B. 24 cm 645 In heart failure. Impaired renal function C. All three shocks should be given in quick succession. diastolic failure. carotid sinus and aortic arch. renin-angiotensin-aldosterone system and cytokine system. 200 J protein. Increased vascular stiffness B. 1903 Harrison’s 18th Ed. Fourth ICS. Right parasternally at the level of fourth rib C. 1356 C. 20 % C. 12 cm Conditions that lead to a high cardiac output like arteriovenous fistula and anemia seldom lead to HF in a normal heart. Fifth ICS. A. Vinculin D. Aortic arch C. Desmin B.50%) i. midclavicular line C. 1902 Harrison’s 16th Ed. Atrial and brain natriuretic peptides (ANP & BNP) C. a compensatory mechanisms are activated that 640 One paddle during external cardioversion is placed at ? includes adrenergic nervous system. 1356 A. Fourth ICS. Nitric oxide (NO) D. Hypertension B. 1901 A. 648 Which of the following mechanism may be responsible in the development of HF with preserved EF ? 642 Which of the following is a cadiac cytoskeletal protein ? Harrison’s 18th Ed. initial shock with a monophasic wave. Fifth ICS. 1903 Harrison’s 18th Ed. 641 What percentage of patients who develop Heart failure (HF) 647 Baroreceptors are located in which of the following locations ? Harrison’s 18th Ed. Laminin is a nuclear membrane B. prostaglandins (PGE2 & PGI2) and nitric oxide (NO). Myocyte hypertrophy A. Myosin form defibrillator should be ? C. 155 Cardiology MCQ’s FOR MCQ’s FOR MEDICAL MEDICAL PROFESSIONALS PROFESSIONALS BY PROF. All of the above D. 50 % High-pressure baroceptors are located in left ventricle. Epidemiologic studies show that approximately one-half of patients who develop HF have a normal or preserved EF (EF 40 . Arteriovenous fistula B. After this initial decline in pumping capacity. Adrenergic nervous system A. Altered contractile properties of myocyte . 1901 A. followed by a third shock of 360 J if still necessary. 1902 be 200 J. Left ventricle A. Laminin A. Infiltrative disorders 638 Paddle diameter for external cardioversion is ? Harrison’s 16th Ed. left anterior axillary line Heart failure begins after an index event produces an initial decline in the heart’s pumping capacity. cardiac myosin & vinculin. If the arrhythmia persists. the initial shock should Harrison’s 18th Ed. left anterior axillary line D.e. 100 J Cytoskeletal proteins include desmin. C. C. 18 cm D. which of the following compensatory mechanisms is activated after an index event ? 639 One paddle during external cardioversion is placed at ? Harrison’s 18th Ed. Prostaglandins (PGE2 & PGI2) D. Right parasternally at the level of first rib 646 Which of the following is a vasodilatory molecule ? Harrison’s 18th Ed. Chagas’ disease A. 300 J D. Vinculin N Engl J Med 2001. All of the above D. 1901 A. 10 cm D. B. Right parasternally at the level of second rib A. Myosin C. there is activation of a family of compensatory vasodilatory molecules like atrial & brain and Cor Pulmonale natriuretic peptides (ANP & BNP). 1902 B. Figure 234-2 have normal or preserved EF ? Harrison’s 18th Ed. All of the above 649 Which of the following contribute to LV remodeling ? 643 Which of the following is not a cadiac cytoskeletal protein ? Harrison’s 18th Ed. 360 J 644 Which of the following condition does not lead to HF in a normal heart ? When a monophasic wave-form defibrillator is used for ventricular fibrillation. Carotid sinus B.344:1311 D. Renin-angiotensin-aldosterone system B. 1903. Heart Failure In HF. Harrison’s 16th Ed. Diastolic dysfunction A. AJAY MATHUR Cardiology 155 637 In ventricular fibrillation. 1356 A. Cytokine system C. 35 % D. midclavicular line B. 10 % B. that oppose excessive peripheral vascular vasoconstriction. Desmin B. All of the above Chapter 234. All of the above D. a second shock of 200 to 300 J should be given. Right parasternally at the level of third rib B. 1904 D. Aortic regurgitation B. All of the above 656 Orthopnea is caused due to redistribution of fluid from ? 650 Which of the following act as a biologic stimuli in LV remodeling ? Harrison’s 18th Ed.3 hours after the patient retires. CD2-associated protein (CD2AP) D. Juxtacapillary K receptors C. By 2-D echocardiogram/Doppler. 1904 Harrison’s 18th Ed. Circulating neurohormones C. interstitial the rapid. resulting in decreased calcium uptake into SR. 1906 A. 1904 A. 1904 B. LV volume D.156 MCQ’s FOR MEDICAL PROFESSIONALS BY PROF. None of the above Most important mechanism of dyspnea in HF is pulmonary congestion with accumulation of interstitial or intra-alveolar fluid. results from redistribution of fluid from Biologic stimuli leading to LV remodeling include mechanical stretch of myocyte. Scrotum A. SERCA2A B. which activates juxtacapillary J receptors. 1906 B. Majority of patients with chronic HF do not have evidence of pulmonary hypertension.4 hours C. N-terminal pro-BNP D. 15 . edema. together . 654 Dyspnea in HF may become less frequent with the onset of ? 660 Which of the following is useful in HF patients with a preserved Harrison’s 18th Ed. apoptosis. inflammatory cytokines (tumor necrosis factor). leading to calcium leakage from SR. B-type natriuretic peptide (BNP) C. Circulating inflammatory cytokines D. All of the above A. and/or pulmonary edema due to increased capacity of the lymphatics to remove interstitial and/or pulmonary fluid. and hyperphosphorylation of ryanodine receptor. Juxtacapillary M receptors D. 3 . D. 1904 B. AJAY MATHUR Cardiology C. Harrison’s 18th Ed. peptides and growth factors (endothelin) and reactive oxygen species (superoxide). Pulmonary circulation B. with a neurohormones (norepinephrine. 1904 A. resultant increase in pulmonary capillary pressure. Mechanical stretch of myocyte B. edema may be found in ? 652 Left ventricular remodeling refers to changes in which of the Harrison’s 18th Ed. Early satiety of extracellular matrix (organized structural collagen around myocytes being replaced by interstitial collagen matrix that is unable to provide structural support to myocytes). Tricuspid regurgitation A. Juxtacapillary J receptors pulmonary edema B. usually 1 . Interscapular region Harrison’s 18th Ed. volume. All of the above Orthopnea (dyspnea occurring in recumbent position). Cerebral circulation D. LV mass C. LV remodeling is a result of myocyte hypertrophy. shallow breathing characteristic of cardiac dyspnea. circulating splanchnic circulation and lower extremities into central circulation during recumbency. and autophagic cell death. 1905 following ? A. Nape of the neck B. beta- adrenergic desensitization. Magnetic resonance imaging (MRI) is the gold standard C.30 minutes A. LV shape In bedridden HF patients. and shape and the composition 659 Which of the following statements is false ? of heart that occur after cardiac injury and/or abnormal hemodynamic loading conditions. Nocturia Adaptive changes within the myocardium is collectively referred to as LV remodeling. Juxtacapillary L receptors for assessing LV mass & volumes D. 658 In bedridden HF patients. 5 . Left atrial dilation by 2D Echo Dyspnea in HF may become less frequent with the onset of right ventricular (RV) failure and D. Pulmonary regurgitation C. Mitral regurgitation Harrison’s 18th Ed. Progressive loss of myocytes 655 Which of the following is a symptom of HF ? Harrison’s 18th Ed. paroxysmal nocturnal dyspnea (PND) occurs how many 651 Which of the following is best related to a failing cardiac hours after the patient retires ? myocyte ? Harrison’s 18th Ed. presence of left atrial dilation and LV hypertrophy. and/or A. All of the above Left ventricular remodeling refers to changes in LV mass.3 hours C. Giant cell line-derived neutrophilic factor (GDNF) PND refers to acute episodes of severe shortness of breath and coughing that awaken the Changes that regulate excitation-contraction include decreased function of sarcoplasmic reticulum patient from sleep. 1 . All of the above C. which in turn stimulate Majority of patients with chronic HF do not have evidence of pulmonary hypertension. angiotensin II). 657 In HF.6 hours D. altered contractile properties of myocyte. B. NCK-adaptor protein B. A normal ECG excludes LV systolic dysfunction 653 Dyspnea in HF is best related to ? Harrison’s 18th Ed. All of the above tricuspid regurgitation. interstitial edema. 1904 EF ? A. abnormal myocardial energetics and metabolism and reorganization C. 1904 Harrison’s 18th Ed. Ca 2+ adenosine triphosphatase (SERCA2A). D. edema may be found in sacral area (presacral edema) and scrotum. Nocturnal cough progressive loss of myocytes through necrosis. Coronary circulation A. Splanchnic circulation C. Upward titration at 2-week intervals A. Warfarin 663 Loop diuretics increase fractional excretion of sodium by ? D. Kinin potentiation D. 1911 B. Maximum dose as reported effective in clinical trials B.50 % C. pulmonary fibrosis and hepatitis. 157 Cardiology MCQ’s FOR MCQ’s FOR MEDICAL MEDICAL PROFESSIONALS PROFESSIONALS BY PROF. C.25 %. All of the above increase it by 5 . 1907 671 Which of the following is a precipitating cause of acute A. 1911 D. 667 Which of the following statements about beta blockers in HF is B-type natriuretic peptide (BNP) and N-terminal pro-BNP are elevated in HF patients with a preserved EF.elevated LV filling pressures with decreased perfusion and Profile L - B. 1911 B. Stimulating guanylyl cyclase perfusion. Hepatitis Loop diuretics increase the fractional excretion of sodium by 20 . None of the above C.normal LV filling pressure with normal perfusion. Renal impairment D. AJAY MATHUR Cardiology 157 with abnormalities of LV diastolic filling is useful for the assessment of HF with a preserved EF. Any of the above A. All of the above Harrison’s 18th Ed. arrhythmias. B. A. B. 1906 B. Eplerenone A. Adverse effects of amiodarone include hyperthyroidism. Occult myocardial ischemia 665 Which of the following can attenuate the effects of ACE D. 1908 dietary indiscretion. A. and environmental and/or emotional stress. Interference with adrenergic nervous system normal or low LV filling pressure with decreased tissue perfusion. false ? Harrison’s 18th Ed. Nitroglycerin . 20 . 1910 Natriuretic peptide levels increase with age & renal impairment. Hyperthyroidism C. although to a lesser degree. Profile A Fluid retention can attenuate the effects of ACE inhibitors. Profile B includes most patients with acute pulmonary edema. All of the above inhibitors ? Precipitating causes of of acute decompensated HF (ADHF) include infection. C-reactive protein B.20 % A. Should be initiated in low doses Harrison’s 18th Ed. Spironolactone decompensated HF (ADHF) ? Harrison’s 18th Ed. All of the above A. Phenytoin Levels of troponin T and I. Harrison’s 18th Ed.elevated LV filling pressure with normal A.10 %. Profile A .5 mg/dL B. Hypothyroidism D. are more elevated in women. All of the above 668 Aldosterone antagonists are not recommended when ? Harrison’s 18th Ed. TNF receptors. 664 Which of the following diuretics lead to hyperkalemia ? Harrison’s 18th Ed. Triamterene B.25 % B.15%) and angioedema (1%). TNF receptors 669 Amiodarone increases the level of which of the following ? C. Uric acid Harrison’s 18th Ed. 1907 670 Which of the following is an adverse effect of amiodarone ? A. 1912 D. Pulmonary embolism D. 1909 661 Natriuretic peptide levels are more in ? A. Profile C 666 Monproductive cough as a side effect of ACE inhibitors is related to ? D. C-reactive protein. Physical exercise 672 Most patients with acute pulmonary edema belong to which of C. Profile B C. Infection C. All of the above C. 5 . Thiazide diuretics D. occult myocardial ischemia/ A. All of the above 673 Which of the following drugs exerts dilating effect on arterial resistance and venous capacitance vessels ? The side effects of ACE inhibitors related to kinin potentiation include a nonproductive cough Harrison’s 18th Ed. 10 . All of the above A. infective endocarditis. Aged C. Profile C . pulmonary embolism. Serum creatinine is > 2. and can be elevated in right HF from any cause. It is preferable to optimize the dose of diuretic before starting the ACE inhibitor. 1912 (10 . Creatinine clearance is < 30 mL/minute 662 Levels of which of the following may be elevated in HF ? C.10 % Harrison’s 18th Ed. Digoxin HF and provide important prognostic information. 25 . Profile B . Anxiety infarction. Serum potassium is > 5 mmol/L Harrison’s 18th Ed. may be elevated in B. C. and uric acid. 1909 Patients with acute HF present with one of four basic hemodynamic profiles. Profile L Harrison’s 18th Ed. Fluid retention the following profiles ? Harrison’s 18th Ed. 1906 D. hypothyroidism. Women D. Nitroprusside Harrison’s 18th Ed. and ? Harrison’s 18th Ed. 2218. Any of the above 675 Nesiritide is best related to ? Harrison’s 18th Ed. 2218. Cardiac output / Body mass index C.RAP) ÷ CO] x 80 Harrison’s 18th Ed. Left heart failure D. with little effect on  1 receptors. A. Cardiac output / Body mass index 677 Which of the following is a phosphodiesterase III inhibitor ? Harrison’s 18th Ed.PCWP) ÷ CO] + 80 D. and nesiritide exert dilating effects on arterial resistance and venous capacitance vessels. Milrinone . Heart rate (HR) Harrison’s 18th Ed. Positive-pressure (mechanical) ventilation D. Tolvaptan D. which is an endogenous 682 The blood pressure (BP) is the product of cardiac output (CO) peptide secreted primarily from LV in response to an increase in wall stress. Nesiritide A. Cardiac output / Peripheral vascular resistance B. All of the above 683 Cardiac index (CI) is calculated by formula ? Harrison’s 18th Ed. Dopaminergic receptors A. Dopamine C Stroke volume B. [(MAP x RAP) ÷ CO] x 80 Dopamine is an endogenous catecholamine that stimulates 1 and  1 receptors and dopaminergic receptors (DA 1 and DA 2) in the heart and circulation. A. Cardiac output / Peripheral vascular resistance A.  1 receptors 685 Systemic vascular resistance (SVR) is calculated by the B. 2218. 1914 C. > 250 µg / minute for over 24 hours A. C.RAP) x CO] x 80 679 Which of the following is administered orally ? D. Tolvaptan Harrison’s 18th Ed. Cardiac output / Heart rate Milrinone is a phosphodiesterase III inhibitor that leads to increased cyclic AMP by inhibiting its breakdown. Pulmonary vascular disease Cyanide toxicity (gastrointestinal & central nervous system) in nitroprusside therapy occurs at doses > 250 µg / minute for over 48 hours. It exerts its effects by stimulating  1 and  2 receptors. Peripheral vascular resistance (PVR) A. C. 1914 C. > 150 µg / minute for over 24 hours B. Cardiac output / Body surface area B. Dobutamine 684 Stroke volume (SV) is calculated by the formula ? C. [(MAP + RAP) ÷ CO] x 80 D. AJAY MATHUR Cardiology B.158 MCQ’s FOR MEDICAL PROFESSIONALS BY PROF. 1913 C. Milrinone A. 1913 Decompensation of chronic cor pulmonale can be aggravated by events that induce pulmonary vasoconstriction and RV afterload. Nesiritide formula ? B. [(PAPm .  1 receptors formula ? Harrison’s 18th Ed. > 150 µg / minute for over 48 hours 681 Most common cause of right-heart failure is ? Harrison’s 18th Ed. Obesity hypoventilation syndrome (OHS) D. Table 270–2 D. All of the above B. 1913 B. Table 270–2 C. 2218. Table 270–2 Dobutamine is an inotropic agent for the treatment of acute HF. Atrial-type natriuretic peptide B. 1913 686 Pulmonary vascular resistance (PVR) is calculated by the A. Table 270–2 C. Dopamine D. Dobutamine D. None of the above C. Pulmonary parenchymal disease D. nitroglycerin. 1913 D. 2218. B. Tolvaptan A. 1913 Most common cause of right-heart failure is not pulmonary parenchymal or vascular disease but left heart failure. All of the above 674 Cyanide toxicity in nitroprusside therapy occurs at doses ? Harrison’s 18th Ed. Cardiac output / Body surface area 678 Dopamine stimulates which of the following ? Harrison’s 18th Ed. All of the above B. Acute pulmonary emboli By stimulating guanylyl cyclase within smooth-muscle cells. Brain-type natriuretic peptide C. Nitroprusside 680 Decompensation of chronic cor pulmonale is aggravated by ? Harrison’s 18th Ed. Phosphodiesterase III inhibitor Myocardial function D. A. Table 270–2 676 Which of the following is an inotropic agent ? A. [(MAP . nitroprusside. [(MAP . Endogenous catecholamine Nesiritide is a recombinant form of brain-type natriuretic peptide. > 250 µg / minute for over 48 hours B. C. Cardiac output / Heart rate A. G Bardy C. Bronchi B. Non selective  blocker C. 1d B. [(PAPm . Nisoldipine B. Metoprolol following tissues except ? D. Mibefradil 687 In hypovolemic shock. All of the above C. Systemic Vascular Resistance D. Propranolol 700 Amlodipine binds which subunit of voltage gated L-type B. Propranolol D. Diltiazem C.355:608 Harrison’s 16th Ed. which of the following is decreased ? Harrison’s 18th Ed. Vasculature D. 2219. Nadolol A. Atenolol calcium channels ? N Engl J Med 2006.PCWP) ÷ CO] . Sotalol B. Cardiac myocytes 694 Which of the following is true for Carvedilol ? B. All of the above D. 1a D. M Packer B. Adipose tissue C. K-type C. Amlodipine Harrison’s 18th Ed.  mediated vasodilator C. Cardiac Output 696 Which of the following is a dihydropyridine class of calcium- C. Heart 699 Calcium-channel antagonists block which voltage gated 691 The credit of developing  blockers goes to ? calcium channel ? A.80 C.PCWP) ÷ CO] x 80 D. Nifedipine A. Diltiazem D.355:608 A. Venous O2 Saturation D. Nicardipine A. Table 270–4 C. None of the above 698 Which of the following is the phenylalkaylamine class of calcium-channel blocker ?  3 receptors are found predominantly in ? N Engl J Med 2006. 2219. Lionel H Opie N Engl J Med 2006. Nifedipine A. Systemic Vascular Resistance D. Nifedipine B. L-type D. Anti-oxidant D. Isradipine Harrison’s 18th Ed. M-type D. Verapamil C.  1c A.355:608 C. [(PAPm . 2219.355:608 A. Nimodipine B. Amlodipine 688 In cardiogenic shock. AJAY MATHUR Cardiology 159 B. CVP and PCWP C. Pancreatic beta cells B. Systemic Vascular Resistance channel blocker ? D. which of the following is increased ? A. 159 Cardiology MCQ’s FOR MCQ’s FOR MEDICAL MEDICAL PROFESSIONALS PROFESSIONALS BY PROF. James Black A. 1b 693 Which of the following  blockers has low hepatic clearance ? C. Venous O2 Saturation N Engl J Med 2006. Table 270–4 B. 423 A.355:608 B. CVP and PCWP C.PCWP) ÷ CO] ÷ 80 695 Which of the following is a T-type calcium channel blocker ? [(PAP m – PCWP)/CO] x 80 A. which of the following is increased ? B. Atenolol N Engl J Med 2006. Vascular smooth muscle A. Carvedilol 701 L-type calcium channels are found abundantly in all of the C. CVP and PCWP D. [(PAPm . Amlodipine A.355:608 689 In neurogenic shock. Brain . Venous O2 Saturation 697 Which of the following is the benzothiazepine class of calcium- D. Table 270–4 B. Nimodipine B. All of the above channel blocker ? N Engl J Med 2006. N-type 692 Which of the following is a lipid soluble  blocker ? A. 2 µm A. Arterial pressure D. G C. Ventricular radius D. Ratio of stroke volume to end-systolic volume Harrison’s 18th Ed. Ratio of stroke volume to end-diastolic volume A.5 µm D. Troponin I 713 Cardiac function variable that is influenced by ventricular C. Myosin C. Ventricular volumes A. Cathotropic D. 60 ± 20 mL/m2 Harrison’s 18th Ed. Ratio of end-systolic volume to stroke volume B. All of the above Harrison’s 16th Ed. Cardiac output A. the force of ventricular contraction is B. All of the above Harrison’s 18th Ed. Cardiac output contraction is approximately ? B. H D. Atrioventricular dissociation a function of ? C. All of the above A. All of the above . Wall thickness D. Mid-systolic length of cardiac muscle B. 2000 times B. 1906 to ? A. End-diastolic length of cardiac muscle B. Ca 2+ becomes attached Harrison’s 18th Ed. Mid-diastolic length of cardiac muscle A. 10000 times D.2 µm 714 Atrial contribution to ventricular filling is reduced by ? C. Troponin T loading conditions is ? D. Dromotropic A. Aortic flow velocity B.7 µm Harrison’s 16th Ed. 1362 705 Acceleration of cardiac relaxation occurs by phosphorylation A.160 MCQ’s FOR MEDICAL PROFESSIONALS BY PROF. E A. 80 ± 20 mL/m2 B. Ejection fraction Harrison’s 18th Ed. End-systolic length of cardiac muscle 715 Afterload is determined by ? Harrison’s 18th Ed. Lusitropic C. LV volume 708 During exercise. 3. 1805 C. 1806 A. 1362 706 The sarcomere length associated with the most forceful A. 1803 A. LV end-diastolic pressure 716 Which of the following parameter is included in Laplace’s law ? Harrison’s 18th Ed. Thickness of LV wall relatively little change ? D.355:608 A. Intracavitary ventricular pressure C. 1805 B. 1363 D. 1. 2. Titin B. 20000 times 710 Which protein series mediates the activation of adenylate 703 Stretching capability of which myofibrillar protein contributes cyclase by -adrenergic agonists ? to the elasticity of heart ? Harrison’s 16th Ed. 2. Troponin I D. 1805 D. All of the above 712 Normal value of ventricular end-diastolic volume is ? Harrison’s 16th Ed. Ionotropic B. 50 ± 20 mL/m2 by cyclic AMP of which protein ? B. Ratio of end-diastolic volume to stroke volume C. Troponin C C. AJAY MATHUR Cardiology 702 The extracellular concentration of calcium is how many times 709 Drugs that enhance relaxation of myocardium are called ? more than the intracellular concentration ? Harrison’s 16th Ed. 1362 Harrison’s 18th Ed. Atrial fibrillation 707 In Starling’s law of heart. which of the following parameters show C. 70 ± 20 mL/m2 A. 1803 B. 1805 C. Actin D. All of the above 711 Ejection fraction is the ? 704 During activation of the myocyte. All of the above B. Troponin T D. F B. Troponin C D. 5000 times C. 1803 C. Rate of ventricular pressure development C. Prolongation or abbreviation of P-R interval Harrison’s 18th Ed. 1362 N Engl J Med 2006. Piretanide C. 1805 C.345:588 D. Vasopressin B. Omapatrilat C. 600 to 800 pg per milliliter B.345:588 D. Endothelin A. Nesiritide vasopressin concentrations in the range of ? N Engl J Med 2001. Cardiac ventricle 720 Diastolic heart failure is differentiated from systolic heart B. Relative wall thickness 728 Brain natriuretic peptide (BNP) is secreted by cardiac ventricles C. Maladaptive changes in extracellular matrix 729 What level of BNP indicates that heart failure is likely ? N Engl J Med 2005. 1371 D. N-terminal BNP B. Pre pro-BNP 719 Which of these loop diuretics is associated with deafness ? B. Nesiritide 724 In vasodilatory shock. Atrial natriuretic peptide B. 428 C. > 2000 pg per milliliter A.351:1097-105 D. Maladaptive changes in surviving myocytes C. Ejection fraction B. > 500 pg per milliliter 722 Plasma concentrations of all of the following substances are B. Pro-BNP Harrison’s 15th Ed. Activation of ATP-sensitive potassium channels Harrison’s 18th Ed. Activation of calcium-regulated potassium channels A. AJAY MATHUR Cardiology 161 717 Laplace’s law indicates ? B. Torsemide A. > 1000 pg per milliliter markedly increased in septic shock except ? C. 161 Cardiology MCQ’s FOR MCQ’s FOR MEDICAL MEDICAL PROFESSIONALS PROFESSIONALS BY PROF. Omapatrilat 723 Vasoconstrictor effects of vasopressin occur at plasma C. All of the above A.345:588 D. 200 to 400 pg per milliliter A. Cerebellum N Engl J Med 2004. Torsemide A. resistance is observed to all of the following except ? D. Metolazone D. Adenosine D. Decreased intracardiac pressures Lancet 2005. 1907 C. All of the above A. Calcitonin gene-related peptide 730 Which of the following is a brain natriuretic peptide (BNP) ? Harrison’s 18th Ed. Fenoldopam A. 1913 C.353:2788-96 D. > 1500 pg per milliliter N Engl J Med 2001. Adenosine A. . All of the above B. Angiotensin II Harrison’s 18th Ed. 365: 1877-89 C. 1 to 7 pg per milliliter 731 Which of the following is a dopamine receptor stimulator ? B.353:2788-96 D. Bradycardia B. 10 to 200 pg per milliliter Harrison’s 16th Ed. Cerebral cortex failure by ? C. All of the above A. Norepinephrine agent. End diastolic volume in response to ? N Engl J Med 2005.345:588 Fenoldopam is a dopamine a-1 specific agonist approved as a parenteral antihypertensive A. Length of myocardial fiber D. Furosemide D. Bumetanide 726 Brain natriuretic peptide (BNP) is formed form ? Harrison’s 16th Ed. N-terminal-pro-BNP A. Fenoldopam N Engl J Med 2001. All of the above A. Tension of myocardial fiber C. Wall stretch 721 Ventricular “remodelling” is a result of ? B. Bumetanide C. Loss of myocytes D.353:2788-96 D. Ethacrynic acid 727 Brain natriuretic peptide (BNP) is secreted predominantly by ? N Engl J Med 2005. Vasopressin B. 1478 C. Increased nitric oxide synthesis & depletion of vasopressin 718 Which of the following is not a loop diuretic ? B. Hypotension A. Adenosine D. Diameter of myocardial fiber 725 Mechanism of vasodilatory shock include ? N Engl J Med 2001. left arch 4 as the aortic arch and right subclavian C. 10 percent A. Umbilical vein A. Congenital valvular AS. 40 percent D. . 1921 A. Congenital valvular AS C. Complete transposition of great vessels 733 Following are the radiographic features of cardiogenic ASD occurs more frequently in females. AJAY MATHUR Cardiology 732 Pulmonary edema may not be visible on chest X-ray until the 738 Which of the following congenital heart disease is more amount of lung water increases by ? common in females ? N Engl J Med 2005. along right sternal border Sinus venosus receives the umbilical. D. Gastrointestinal system B. Sinus venosus ASD C. Ostium secundum ASD The most common birth defects are cardiovascular in origin. Absent peribronchial cuffing B. Greater than normal heart size pulmonary venous connection ? B. 1920 in which of the following ? A. D. ~4% 742 Which out of the following ASD is most common ? CHD complicates ~1% of all live births in general population. ~1% B. 4 A. 1921 with CHD. 1920 D. Ostium secundum ASD Harrison’s 18th Ed. All of the above C. 3rd intercostal space. Septal lines (Kerley’s B lines) A. Sinus venosus ASD B. Common cardinal vein B. Ostium primum ASD arteries ? C. 1921 C. Sinus venosus ASD 736 Which of the following arches develop as the internal carotid B. Ostium primum ASD Truncus arteriosus & aortic sac initially develop six paired symmetric arches. Development of arch 3 results in internal carotid arteries. 20 percent B. Ostium secundum ASD D. Coarctation of aorta and Complete transposition of great vessels is more frequent in males. 3 D. Patent foramen ovale A. 30 percent C. 3rd intercostal space. Patent foramen ovale D.162 MCQ’s FOR MEDICAL PROFESSIONALS BY PROF. ASD B. 734 The most common birth defects have origin in ? 740 ASD occurs in the basal portion of the interventricular septum Harrison’s 18th Ed. 4th intercostal space. Congenital Heart Disease ASD in sinus venosus type is high up in the atrial septum near the entry of superior vena cava in the Adult into right atrium. but occurs in 4% of offspring of women Harrison’s 18th Ed. 1922 A. along left sternal border B. and part of arch 6 as the patent ductus arteriosus. ~2% C. Patent foramen ovale 737 Sinus venosus receives which of the following veins ? Harrison’s 18th Ed. 1921 B. pulmonary oedema except ? N Engl J Med 2005. 4th intercostal space. 1921 C. ~3% D. 1921 live births ? Harrison’s 18th Ed. vitelline. Patent foramen ovale Chapter 236. Increased width of vascular pedicle Harrison’s 18th Ed.353:2788-96 739 Which of the following ASD type is associated with anomalous A. Ostium secundum ASD C. 1920 744 Mid-diastolic rumbling murmur in ASD is loudest at ? Harrison’s 18th Ed. Cardiovascular system Harrison’s 18th Ed. Patent foramen ovale 735 Congenital heart disease complicates what percentage of all 741 Which of the following ASD is more common in Down syndrome ? Harrison’s 18th Ed. along right sternal border D. Urogenital system C. Ostium secundum ASD artery. A. Ostium primum ASD B. Central nervous system A.353:2788-96 Harrison’s 18th Ed. Sinus venosus ASD A. 2 743 Which out of the following ASD involves the fossa ovalis ? Harrison’s 18th Ed. It is associated frequently with anomalous pulmonary venous connection from the right lung to the superior vena cava or right atrium. 1 B. Coarctation of aorta D. and common cardinal veins. Vitelline vein C. Ostium primum ASD C. along left sternal border D. Ostium primum ASD D. 1920 A. Sinus venosus ASD D. Clockwise rotation of sagittal plane QRS loop correction of shunt. Pulmonary arterial dilatation D. the RV conduction defect is accompanied by left superior axis deviation types of compensated ASD ? and counterclockwise rotation of the frontal plane QRS loop. Tricuspid stenosis 753 In Eisenmenger’s syndrome. or echocontrast. and A. D. In ASD. 1922 755 In patients with Eisenmenger syndrome. ventricular. ASD may be visualized directly by two-dimensional imaging. Mid-diastolic murmur at fourth intercostal space and along left sternal border indicates increased flow across tricuspid valve. Harrison’s 18th Ed. Chest pain D. i. Counterclockwise rotation of frontal plane QRS loop 756 Pulmonary vascular disease does not progress after operative C. second heart sound is widely split and is relatively fixed in relation to respiration. Counterclockwise rotation of sagittal plane QRS loop than systemic vascular resistance by ? . pulmonary-to-systemic flow ratios 2:1. Ostium secundum ASD cases rarely die before 5th decade D. 163 Cardiology MCQ’s FOR MCQ’s FOR MEDICAL MEDICAL PROFESSIONALS PROFESSIONALS BY PROF.e. clubbing. second heart sound is widely split and is relatively (paradoxical) ventricular septal motion in the presence of a significant right heart volume overload. 1922 In patients with Eisenmenger syndrome. is true ? Harrison’s 18th Ed. Flow is bidirectional or predominantly right-to-left because of high resistance and obstructive pulmonary hypertension.. syncope. Risk of infective endocarditis is quite low unless ASD is complicated by valvular regurgitation or has recently Harrison’s 18th Ed. A. Mitral stenosis B. 1922 C. and hemoptysis. Right-axis deviation leads to ? B. Sinus venosus ASD cases rarely die before 5th decade C. 1922 A. Patients with findings may be confused with ? sinus venosus or ostium secundum ASDs rarely die before the fifth decade. Apical holosystolic murmur A. Midsystolic pulmonary ejection murmur B. ECG shows right-axis deviation and an rSr’ pattern in right precordial C. which of the following is true ? D. chest pain. All of the above indicates associated mitral or tricuspid regurgitation or a ventricular septal defect (VSD). Middiastolic rumbling murmur A. Any of the above 748 Which of the following statements about ostium secundum type Eisenmenger’s syndrome refers to a large communication between systemic and pulmonary of ASD is false ? circulations at the aortopulmonary. Left superior axis deviation Harrison’s 18th Ed. Mid-diastolic left parasternal murmur Harrison’s 18th Ed. Pulmonary stenosis Harrison’s 18th Ed. which of the following In patients with Eisenmenger syndrome. Syncope In ostium secundum ASD. 1922 B. Clubbing C. Midsystolic pulmonary outflow murmur 751 Which of them is a finding in echocardiogram of a patient of ASD ? B. Abnormal (paradoxical) ventricular septal motion murmur. RV and RA dilatation Increased flow across the pulmonic valve is responsible for a midsystolic pulmonary outflow C. Left-axis deviation B. 1922 A. 1923 D. echocardiography reveals pulmonary arterial and RV and RA dilatation with abnormal 746 In ASD. fixed in relation to ? Harrison’s 18th Ed. right-to-left shunt leads to cyanosis. Heart rate B. In ostium primum ASD. physical findings may be confused with mitral B. or atrial levels. Atrial level stenosis and the mitral “opening snap. D. Cyanosis D. 1922 been repaired with a patch or device. 1923 C. All of the above B. D. if pulmonary vascular resistance is less D. a large communication between systemic and pulmonary circulation exists at ? C. None of the above Closure by an appropriate method should be advised in uncomplicated secundum ASD with 747 In adults with an ASD and atrial fibrillation. Risk of infective endocarditis in ASD is low In ASD. 1923 C. right-to-left shunt A. the physical significant left-to-right shunting. All of the above Harrison’s 18th Ed.” respectively. Erythrocytosis 750 In ECG of cases of ostium primum ASD. color-flow imaging. Respiration Harrison’s 18th Ed. Hemoptysis leads representing enlargement of the RV outflow tract. S2 widely split and relatively fixed Harrison’s 18th Ed. Aortopulmonary level In adults with an ASD and atrial fibrillation. Left inferior axis deviation A. B. Posture A. All of the above C. 1922 752 Which of the following about atrial septal defect is false ? A. All of the above 749 In ECG of cases of ostium primum ASD. symptoms in adult life consist of ? B. Clockwise rotation of frontal plane QRS loop erythrocytosis. ECG shows left axis deviation 754 In Eisenmenger syndrome. symptoms in adult life consist of exertional dyspnea. apical holosystolic murmur D. Harrison’s 18th Ed. AJAY MATHUR Cardiology 163 745 Which of the following cardiac murmurs can be heard in various In ostium primum ASD. Ventricular level stenosis with pulmonary hypertension because the tricuspid diastolic flow murmur and widely split second heart sound may be mistakenly thought to represent the diastolic murmur of mitral C. Mitral regurgitation A. 1923 C. This is called differential cyanosis. 1924 syndrome ? Harrison’s 18th Ed. 1924 systemic value. Congenital Aortic Stenosis C. Congenital aneurysm of aortic sinus of Valsalva with fistula A. Right atrium B. when the noncoronary cusp is ruptured. 759 Differential cyanosis is best related to ? Harrison’s 18th Ed. Cardiac arrhythmia C. Rupture of aneurysm of an aortic sinus of Valsalva 757 Which of the following is a consequence of chronic hypoxemia B. usually coronary sinus. C. severe pulmonary vascular disease results in reversal of flow through ductus therby unoxygenated blood is shunted to descending aorta leading to cyanosis and clubbing of the D. Right atrium A. Right ventricle C. All of the above 764 In coronary arteriovenous fistula. 1924 B. or right ventricle. Two-third between right coronary cusp and the right ventricle. Tetralogy of Fallot A. 1924 A. AJAY MATHUR Cardiology Harrison’s 18th Ed. Ends at aorta just distal to left subclavian artery C. Left atrium A. But if the shunt is large. Best heard at lower or midsternal border In PDA. A. Subaortic Stenosis D. right atrium. All of the above C. a continuous murmur accentuated in diastole. Any of the above D. Infective endocarditis B. Continuous murmur best heard at upper left sternal edge D. Right ventricle . is not usually compromised. and anomalous origin of the left coronary artery (LCA) from the pulmonary trunk. Loud and superficial C. Secondary erythrocytosis B. All of the above an aortic sinus of Valsalva with fistula. ASD arteriovenous fistula ? Harrison’s 18th Ed. 1924 767 In which of the following condition. When the noncoronary cusp is involved. None of the above Coronary arteriovenous fistula consists of a communication between a coronary artery and another cardiac chamber. Coronary arteriovenous fistula A. 762 In aneurysm of an aortic sinus of Valsalva. 1925 B. Left ventricle B.164 MCQ’s FOR MEDICAL PROFESSIONALS BY PROF. the fistula drains into the right atrium. there may be a coronary “steal” syndrome with myocardial ischemia and possible angina or ventricular arrhythmias. Originates from bifurcation of pulmonary artery B.right-heart shunt ? Harrison’s 18th Ed. 1924 A. Supravalvular Aortic Stenosis The three most common causes of aortic root–to-right-heart shunts are congenital aneurysm of D. 1924 Harrison’s 18th Ed. Coronary sinus A. 1923 D. Patent ductus arteriosus B. coronary arteries are subjected to elevated systolic pressures from left ventricle ? A. VSD A. 1924 765 Which of the following is true for the murmur due to coronary A. Coronary arteries are subjected to elevated systolic pressures from the left ventricle in supravalvular aortic stenosis. Three-fourth The degree to which pulmonary vascular resistance is elevated before operation is a critical 763 Cause of continuous murmur that accentuates in diastole is ? factor determining prognosis. Tetralogy of Fallot C. but not fingers. One-half Rupture of aneurysm of an aortic sinus of Valsalva leads to aorticocardiac fistula mostly C. the fistula drains into ? 768 Which of the following is not a feature of Williams-Beuren Harrison’s 18th Ed. Continuous D. 1925 A. One-third D. Anomalous origin of LCA from pulmonary trunk B. Coronary arteriovenous fistula B. progression of pulmonary vascular disease after operation is unusual. If the pulmonary vascular resistance is one-third or less of the Harrison’s 18th Ed. Congenital aneurysm of aortic sinus of Valsalva with fistula Harrison’s 18th Ed. 1924 Harrison’s 18th Ed. there occurs communication between a coronary artery and ? 758 Which of the following about Ductus Arteriosus is false ? Harrison’s 18th Ed. shunt is usually of small magnitude and myocardial blood flow Leading causes of death in adults with patent ductus are cardiac failure & infective endocarditis. Anomalous origin of left coronary artery from pulmonary trunk C. Hyperviscosity D. coronary arteriovenous fistula. “Elfin” facies B. Abnormal hemostasis Abrupt rupture causes chest pain. 761 Which of the following is aortic root . VSD in cyanotic CHD ? C. and volume overload of the heart. All of the above toes. All of the above D. 766 Coronary “steal” syndrome best relates to ? 760 Leading cause of death in adults with PDA is ? Harrison’s 18th Ed. Brain abscess D. Patent ductus arteriosus Harrison’s 18th Ed. bounding pulses.to . D. Restrictive lung disease In coronary arteriovenous fistula. B. notching of the third to ninth ribs is an important radiographic sign. and the aortic arch and knob may be on the right side. 7% B. enlarged and pulsatile collateral In tetralogy of Fallot. Enlarged heart In coarctation of the aorta. 3rd to 7th ribs C. Rarely. 3rd to 12th ribs Harrison’s 18th Ed. Circle of Willis aneurysms A. Turner syndrome 776 Which of the following may also be present in TOF ? B. 770 Which of the following is more common in coarctation of the aorta ? aortic override of the VSD. aortic dissection and rupture. cheerful demeanor. 1926 D. 1925 A. the chief hazards of proximal aortic severe hypertension include with congenital heart disease ? cerebral aneurysms and hemorrhage. Mental retardation with retained language skills 774 Which of the following is the hazard of coarctation of aorta ? Harrison’s 18th Ed. Coarctation of the aorta occurs in 7% of patients with congenital heart disease and is more common in males than females. typically comprising of “elfin” facies. there is unilateral absence of a pulmonary Harrison’s 18th Ed. 5% A. Intercostal spaces B. “3” sign relates to which of the following ? D. Supravalvular aortic stenosis A. boot-shaped heart (coeur en sabot) with a A. Pulmonary valve stenosis D. Tetralogy of Fallot A. 1925 artery (usually left). radiographic notching of which of D. Concavity in the region of pulmonary conus Harrison’s 18th Ed. enlarged and pulsatile collateral vessels may be palpated in the B. 1925 Reoperation in adults is most commonly for severe pulmonary regurgitation. 1925 In Tetralogy of Fallot. Pulmonary vascular markings are typically diminished. 3rd to 5th ribs arrhythmias require treatment. obstruction to RV outflow. Supravalvular Aortic Stenosis C. premature coronary arteriosclerosis. in the axillae. Unilateral absence of a pulmonary artery Coarctation of the aorta is frequent in patients with gonadal dysgenesis (Turner syndrome) and C. C. 1926 C. AJAY MATHUR Cardiology 165 B. 165 Cardiology MCQ’s FOR MCQ’s FOR MEDICAL MEDICAL PROFESSIONALS PROFESSIONALS BY PROF. Right-sided aortic arch & descending thoracic aorta is associated with bicuspid aortic valve (75%) and Circle of Willis aneurysms. D. Endocarditis risk eliminated after surgical repair the following ribs is seen ? Most adults with tetralogy of Fallot have had some form of previous surgical intervention. Harrison’s 18th Ed. Boot-shaped heart (coeur en sabot) intercostal spaces anteriorly. A right-sided aortic arch and descending thoracic aorta occur in 25%. and RV hypertrophy due to the RV’s response to aortic pressure via Harrison’s 18th Ed. Ventricular and atrial arrhythmias common with pre. and transient hypercalcemia. chest x-ray shows a normal-sized. 1925 C. Aortic override of the VSD Narrowing or constriction of the lumen of the aorta may occur anywhere along its length but is most common distal to the origin of the left subclavian artery near the insertion of the ligamentum C. Oligemic lung fields 772 In Chest x-ray. Ventricular & atrial A. All of the above 769 Coarctation of the aorta occurs in what percentage of patients In coarctation of the aorta. 1926 C. low nasal bridge. and LV failure. Aortic root has a medial tissue defect & it is commonly enlarged & associated with aortic regurgitation. Indentation of the aorta at the site of coarctation B. The severity of RV outflow obstruction determines the clinical presentation. Transient hyperkalemia B. severity of hypoplasia of the RV outflow tract varies from mild to vessels may be palpated in ? complete (pulmonary atresia). Axillae 777 Which of the following about Chest x-ray findings in Tetralogy of Fallot is false ? C. Interscapular area Harrison’s 18th Ed. 1927 D. B. Coarctation of the aorta 778 Which of the following about treatment in TOF is false ? Harrison’s 18th Ed. VSD . Aortic dissection and rupture Supravalvular aortic stenosis is associated cardiac defect in Williams-Beuren syndrome. Cerebral aneurysms and hemorrhage D. 1927 In coarctation of the aorta. mental retardation with retained C. All of the above 771 In coarctation of the aorta. ASD This is due to inferior rib erosion by dilated collateral vessels. Premature coronary arteriosclerosis language skills and love of music. severity of which of the following B.and poststenotic dilatation (“3” sign) along left paramediastinal shadow are pathognomonic. 1925 the large VSD. 3rd to 9th ribs 779 Which is the most common communication in complete transposition of the great arteries ? D. D. A. 2% 775 In Tetralogy of Fallot. Size of VSD D. All of the above A. 3% determines the clinical presentation ? Harrison’s 18th Ed. RV outflow obstruction arteriosum. D. C. or posteriorly in the interscapular area. RV hypertrophy The four components of the tetralogy of Fallot are malaligned VSD. Reoperation in adults is mostly for severe PR Chest x-ray in Coarctation of the aorta shows a dilated left subclavian artery high on the left mediastinal border and a dilated ascending aorta. Harrison’s 18th Ed. A. A. Endocarditis remains a risk despite surgical repair. Bicuspid aortic valve Harrison’s 18th Ed. B. All of the above B. Aortic regurgitation common 773 In coarctation of the aorta. Pulmonary valve stenosis and supravalvular and peripheral pulmonary arterial obstruction may coexist. Patent ductus arteriosus prominent right ventricle and a concavity in the region of pulmonary conus. 1927 B. Shprintzen PDA ? Harrison’s 16th Ed. Alcohol D. Cutis laxa B. Catch-22 (DiGeorge) D. Thalidomide A. Noonan ASD ? C. Pseudoxanthoma elasticum A. MR D. due to anomalous attachment of tricuspid leaflets. 1382 785 Heritable syndromes with associated Tetralogy of Fallot A. Holt-Oram D. Holt-Oram syndrome A. AJAY MATHUR Cardiology C. Marfan’s stenosis is ? B. and about one-third have an associated VSD. All of the above Harrison’s 16th Ed. Pseudoxanthoma elasticum A. 1383 C. Maroteaux-Lamy B. Shprintzen (velocardiofacial) Harrison’s 16th Ed. PDA C. Ellis-van Creveld A. 1382 D. Noonan 792 Which of the following connective tissue disorders is associated with peripheral coronary arterial disease ? D. TS 788 Chromosomal abnormalities with associated VSD include ? D. Trisomy 13 (D) C. Dilantin 782 Heritable syndromes with associated ASD include all except ? Harrison’s 16th Ed. Cri du chat ventricle. All of the above B. 1382 C. 1382 780 Most common finding in Ebstein’s Anomaly is ? A. Aortic or mitral stenosis (moderate) Harrison’s 16th Ed. TAR (Thrombocytopenia-absent radius) syndrome 783 Heritable syndromes with associated VSD include all except ? Harrison’s 16th Ed. Osteogenesis imperfecta Harrison’s 16th Ed. Multiple lentigines (LEOPARD) syndrome 793 Which of the following connective tissue disorders is associated with peripheral pulmonic stenosis ? D. Ellis-van Creveld syndrome B. Crouzon syndrome 784 Heritable syndromes with associated PDA include all except ? B. Lithium C. All of the above D. Holt-Oram Harrison’s 16th Ed. congenital cardiac malformations that are “well” tolerated (NYHA class I) include all except ? D. Williams D. Crouzon Most patients of complete transposition of the great arteries have an interatrial communication. Kartagener D. Scheie A. 1382 C. TAR (thrombocytopenia-absent radius) C. Conradi-Hunermann syndrome A. Pulmonary stenosis (mild-moderate) B. Trisomy 21 (Down syndrome) 781 During pregnancy. 1382 C. Morquio Harrison’s 18th Ed. TAR (thrombocytopenia-absent radius) 790 Which of the following heritable syndromes is associated with B. tissue of which is dysplastic. 1382 786 Heritable syndrome with associated supravalvular aortic A. Conradi-Hunermann A. Osteogenesis imperfecta Harrison’s 16th Ed. 1383 789 Ebstein’s anomaly occurs with which of the following A. 787 Heritable syndromes with associated aortic regurgitation include ? Harrison’s 16th Ed. Laurence-Moon-Biedl-Bardet . Crouzon Harrison’s 16th Ed. 1382 Ebstein’s Anomaly is characterized by a downward displacement of tricuspid valve into right A. Incontinentia pigmenti C. B. This results in tricuspid regurgitation. Aortic or mitral regurgitation (mild-moderate) teratogenic drugs ? B. All of the above C. 1382 C. Pulmonary or tricuspid regurgitation A. Apert 791 Which of the following heritable syndromes is associated with C. Ehlers-Danlos syndrome B. 1382 D. Incontinentia pigmenti Harrison’s 16th Ed. TR Harrison’s 16th Ed. Marfan’s include all except ? B. 1382 D. Conradi-Hunermann D. two-thirds have a patent ductus arteriosus.166 MCQ’s FOR MEDICAL PROFESSIONALS BY PROF. MS C. All of the above B. 2 cm2 800 What proportion of all patients with rheumatic heart disease & When the mitral valve opening is reduced to < 1 cm 2 it is referred to as “severe” MS. VSD 804 Mitral stenosis is called “moderate”. 1387 A. Circumference 797 Which of the following heritable syndromes is associated with C.1. Catch-22 syndrome valve orifice is 4-6 cm2 ? Harrison’s 18th Ed. 3 cm2 Harrison’s 16th Ed. 1382 Harrison’s 18th Ed.5 cm2 .5 cm2 Rheumatic fever is the leading cause of mitral stenosis (MS). 1382 In normal adults. C. Mitral commissures 796 Which of the following heritable syndromes is associated with Thrombus formation & arterial embolization may arise from calcific valve itself. left atrial myxoma. 1929 Mitral stenosis is called “moderate”. and infective endocarditis with large vegetations. thrombi arise more with coarctation of aorta ? frequently from ? Harrison’s 16th Ed. Coarctation of aorta Mitral valve orifice area of < ~2 cm2 is considered “significant obstruction” because blood can flow from LA to LV only if propelled by an abnormally elevated left atrioventricular pressure gradient. A. 1382 A. 1929 C. 1929 D. which of the following parameter of the mitral B. 1382 B.0 cm2 Harrison’s 18th Ed. Turner syndrome A. 1. 4 cm2 B. when the mitral valve orifice is 1. Trisomy 18 C. rheumatoid C.0 cm2 . 40 % C. < 0.5 cm2 arthritis. Tetralogy of Fallot? particularly from within the left atrial appendage. Multiple sites of narrowing of peripheral pulmonary arteries D. 10 % Harrison’s 16th Ed. Down syndrome D. < 1 cm2 annular calcification with extension onto the leaflets. cor triatriatum.0 cm 2 . 2. Left atrial appendage D. Chordae tendineae C. Shprintzen syndrome A. Systemic lupus erythematosus B. systemic lupus erythematosus. the area of the mitral valve orifice is 4-6 cm 2 .2. Wegeners granulomatosis is reduced to ? Harrison’s 18th Ed. Cor triatriatum A. a history of rheumatic fever have pure or predominant MS ? . Turner syndrome C. Other less common etiologies of obstruction to left atrial outflow include congenital mitral valve stenosis. Cri du chat syndrome D. Rheumatoid arthritis 805 Mitral stenosis is called “severe”. 2 cm2 embryopathy ? C.5 cm2 B.5 cm2 atrial outflow ? D. Cri du chat syndrome history of rheumatic fever. 795 Which of the following chromosomal disorders is associated 801 In patients of MS with atrial fibrillation (AF). D. Tricuspid atresia D.0 cm2 799 Which of the following is not a cause of obstruction to left C. < 1. Catch-22 syndrome 803 Significant obstruction of mitral valve orifice is considered C. 2. 1929 A. 1929 D. Trisomy 8 Pure or predominant MS occurs in ~40% of all patients with rheumatic heart disease and a D.2. Down syndrome B. when the mitral valve opening C. 167 Cardiology MCQ’s FOR MCQ’s FOR MEDICAL MEDICAL PROFESSIONALS PROFESSIONALS BY PROF. A. Turner syndrome B.3. but in patients with atrial fibrillation (AF).5 cm 2 . 1 cm2 798 Which congenital malformation is typical of rubella B.5 cm2 . 1929 Chapter 237. when the mitral valve orifice is ? Harrison’s 18th Ed. 1929 with Tetralogy of Fallot ? A. All of the above B. < 0. Area supravalvular aortic stenosis ? D.1. Turner syndrome 802 In normal adults.0 cm2 . Diameter D. Radius Harrison’s 16th Ed. thrombi arise more frequently from the dilated left atrium (LA). Shprintzen syndrome when the orifice is less than approximately ? Harrison’s 18th Ed. Mitral valve leaflets B. Williams syndrome A. Harrison’s 16th Ed. mitral B. 1. Valvular Heart Disease A. AJAY MATHUR Cardiology 167 794 Which of the following chromosomal disorders is associated Harrison’s 18th Ed. 20 % A. 30 % B. Transvalvular pressure gradient Hemoptysis results from rupture of pulmonary-bronchial venous connections secondary to pulmonary venous hypertension in patients who have elevated LA pressures without markedly B. 60 mm Hg A. as well as right-sided heart failure. A. ~ 15 mm Hg 813 Which of the following about mitral stenosis is false ? Harrison’s 18th Ed. Transvalvular pressure gradient A. RV enlargement Systemic embolization. Total lung capacity The clinical and hemodynamic features of MS are influenced importantly by the level of the PAP. 1929 C. B. B. ~ 25 mm Hg development of symptoms due to MS is ~ 2 decades In severe MS. Pulmonary arteriolar constriction C. particularly in enlarged atrial appendages of patients with MS. Bronchial arterioles 808 Hemodynamic variable of importance in Mitral Stenosis is ? Harrison’s 18th Ed. 1929 D. B. Pulmonary arterial pressure (PAP) B. Pulmonary arterioles C. All of the above 810 “Second stenosis” refers to which of the following ? In addition to interstitial edema in walls of small pulmonary vessels and organic obliterative Harrison’s 18th Ed. and oxygen uptake per unit of ventilation are reduced. 1930 C. Latent period between initial attack of rheumatic carditis & D. Cardiac output 816 Which of the following PFT variable is reduced in MS ? B. None of the above Harrison’s 18th Ed. B. 40 mm Hg Harrison’s 18th Ed. systemic embolization occurs more frequently in ? Harrison’s 18th Ed. Passive backward transmission of elevated LA pressure occurs commonly in MS. Two-third Harrison’s 18th Ed. Heart rate 815 In MS. All of the above Harrison’s 18th Ed. 1930 D. Vital capacity D.05 . All of the above Harrison’s 18th Ed. Hemoptysis is rarely fatal artery systolic pressure (PASP) is more than ? D. Pulmonary infections importantly by ? D. A2 . Recurrent pulmonary emboli 809 Clinical and hemodynamic features of MS are influenced C. Pulmonary-bronchial venous connections which depends on the cardiac output and heart rate. . Organic obliterative changes in pulmonary vascular bed A. Three-fourth A. 1929 D. secondary tricuspid regurgitation (TR) and pulmonic regurgitation (PR). Cardiac output elevated pulmonary vascular resistances and is rarely fatal. 80 mm Hg B. ~ 10 mm Hg B. Pulmonary-bronchial venous connections A. ~ 20 mm Hg A.OS time interval varies inversely with the severity of the MS. total lung capacity. OS follows A2 by 0. occurs more frequently in patients with AF. S1 is accentuated and delayed Severe pulmonary hypertension results in RV enlargement. Figure 237-1 A. hemoptysis results from ? D. 1930 Hemodynamic severity of MS depends on transvalvular pressure gradient and the flow rate A. incidence of which is 10–20%. maximal breathing capacity. Maximal breathing capacity D. In older patients pulmonary vessels and organic obliterative changes in pulmonary vascular bed. hemoptysis results from rupture of ? B. 1929 A. in older patients (>65 years). 1390 D. C. LA pressure required to maintain a normal B. One-third cardiac output is ? C. AJAY MATHUR Cardiology 806 In severe MS. fibrous thickening of walls of alveoli & pulmonary capillaries A. 1929 changes in pulmonary vascular bed. All of the above A. pulmonary arteriolar constriction (“second stenosis”).12 seconds C. Interstitial edema in walls of small pulmonary vessels 817 In MS. C.OS interval varies directly with severity of MS 812 What proportion of all patients with mitral stenosis are female ? Harrison’s 16th Ed. Secondary tricuspid regurgitation (TR) C. LA pressure of ~25 mmHg is required to maintain a normal cardiac output. All of the above Harrison’s 18th Ed.168 MCQ’s FOR MEDICAL PROFESSIONALS BY PROF. 1929 Thrombi form in left atria. None of the above A.0. In those with a reduced CO 811 Severe pulmonary hypertension results in ? D. Pulmonic regurgitation (PR) 818 Which of the following is false in auscultation in MS ? Harrison’s 18th Ed. and in those with a reduced CO. B. C. Patients with AF Pulmonary hypertension results from passive backward transmission of elevated LA pressure. 1932. 20 mm Hg 814 In MS. 1930 D. 1930 C. Infective endocarditis is rare in isolated MS 807 Pulmonary hypertension is considered severe if the pulmonary C. One-fourth A 2 . Alveolar capillaries D. interstitial edema in walls of small B. Vital capacity. Heart rate Harrison’s 18th Ed. Left atrial myxoma D. A. 1931 Harrison’s 18th Ed. produced by functional TR may be audible along the left sternal border. results from dilation of pulmonary valve ring due to severe pulmonary hypertension. lung fields resulting from distention of interlobular septae and lymphatics with edema when resting mean LA pressure > ~20 mmHg. Kerley B lines on X-Ray chest are apparent when the 829 Auscultatory findings change markedly with body position in resting mean LA pressure exceeds approximately ? which of the following ? Harrison’s 18th Ed. All of the above A. Apical D. 1931 A. M-mode Echocardiography A. 1931 A. Linear opacities on CxR chest A. Mitral Stenosis B. Atrial septal defect Kerley B lines are fine. Represent Kerley’s B lines en face failure. particularly in right pleural cavity. 830 Levels of which of the following is elevated in left atrial myxoma ? 824 Which of the following about Kerley’s A lines is false ? Harrison’s 18th Ed. 1931 A. rumbling. None of the above B. Soft. diastolic. 169 Cardiology MCQ’s FOR MCQ’s FOR MEDICAL MEDICAL PROFESSIONALS PROFESSIONALS BY PROF. ECG. Caused by distention of anastomotic channels between Harrison’s 18th Ed. grade I or II/VI systolic murmurs are Apical mid-diastolic murmur associated with severe AR (Austin Flint murmur) is differentiated commonly heard at the apex or along the left sternal border in patients with pure MS and do not from MS because it is not intensified in presystole & becomes softer with administration of necessarily signify the presence of MR. All of the above Atrial septal defect may be mistaken for MS as in both conditions there is clinical. 10 mmHg A. Not intensified in presystole hypertension. Tricuspid Stenosis C. 822 Which of the following echocardiographic technique is most 828 Which of the following features differentiates atrial septal defect useful in evaluating a patient of MS ? from MS ? Harrison’s 18th Ed. D. All of the above 825 Which of the following about Kerley’s B lines is false ? Duration of low-pitched. diastolic murmur correlates with the severity of the stenosis in patients with preserved CO. 1931 peripheral & central lymphatics A. With severe pulmonary C. None of the above sternal border ? Kerley’s C lines are reticular opacities at the lung base. Situated perpendicularly to pleural surface at lung base 820 In patients with MS with RV failure. 20 mmHg C. All of the above D. Mid-diastolic RV tap along the left sternal border signifies an enlarged RV. 823 In MS. diastolic. High-pitched. grade I or II/VI systolic murmurs A. and chest x-ray evidence of RV enlargement and accentuation of pulmonary vascularity. louder during inspiration. They represent edema of the interlobular septa. a high-pitched. Suggest cardiogenic pulmonary edema 821 In MS. dense. Short horizontal lines on CxR chest B. Interleukin 2 (IL-2) . 1931 Harrison’s 18th Ed. Absence of LA Transthoracic echocardiography (TTE) with color flow and spectral Doppler imaging provides enlargement and of Kerley B lines and the demonstration of fixed splitting of S2 with a grade 2 or 3 critical information in evaluation of a patient of MS. B. pleural effusion is on which C. Loudness of opening snap D. a pansystolic murmur. Absence of LA enlargement B. horizontal lines most prominent in the lower and mid- The auscultatory findings in left atrial myxoma change markedly with body position. 25 mmHg D. amyl nitrite. None of the above A. 2D Echocardiography with Colour Doppler studies B. Duration of mid-diastolic murmur Kerley’s A lines are linear opacities extending from periphery to hila caused by distention of C. Extend from periphery to hila stenosis ? C. All of the above B. Transthoracic echocardiography (TTE) with color flow C. C. which of the following findings can be found along left D. Bilateral 826 Which of the following about Kerley’s C lines is false ? D. Fixed splitting of S2 and spectral Doppler imaging D. None of the above decrescendo blowing murmur along the left sternal border. 15 mmHg B. Absence of Kerley B lines C. Right Kerley’s B lines are short horizontal lines situated perpendicularly to pleural surface at lung base. RV tap 827 Which of the following is false about Austin Flint murmur ? B. A. 1931 D. opaque. Soft. 1931 C. Graham Steell murmur of PR. may occur in patients with MS and RV B. Represent edema of interlobular septa side ? Harrison’s 18th Ed. Left C. Duration of presystolic accentuation anastomotic channels between peripheral & central lymphatics. decrescendo blowing murmur Harrison’s 18th Ed. AJAY MATHUR Cardiology 169 819 Which of the following correlates with the severity of mitral B. Reticular opacities at lung base Pleural effusion. D. mid-systolic murmur at the mid to upper left sternal border all favor atrial septal defect over MS. 1931 suggesting cardiogenic pulmonary edema. representing Kerley’s B lines en face Harrison’s 18th Ed. All of the above B. Table 237-3 A. Blunt chest wall trauma isolated MS whose effective orifice is less than ? Harrison’s 18th Ed. AJAY MATHUR Cardiology B. 2. 2 week B. mitral valve prolapse (MVP). 1 week A. Aortic valve repair B. with MS who have atrial fibrillation ? Harrison’s 18th Ed. At least 6 months C. At least 6 months B. At least 1 year C. All of the above 841 Acute MR can occur in which of the following ? Conversion to sinus rhythm is rarely successful or sustained in patients with severe MS. PMBV is the preferred strategy and is performed with TEE and no or minimal x-ray exposure. following blunt chest wall trauma. congenital valve defects. LA is enlarged D. MRI . 1932 A. Hypertrophic obstructive cardiomyopathy (HOCM) C. percutaneous mitral balloon valvuloplasty (PMBV) is performed with ? D. Chordae tendineae 834 In patients with MS and AF. 1931 C. 1932 D. Harrison’s 18th Ed. Mitral valve prolapse (MVP) 836 In pregnant patient with MS. Interleukin 6 (IL-6) D. 837 Successful mitral valvotomy is defined as a reduction in the 831 Anticoagulants are administered for how long to MS patients mean mitral valve gradient by ? who have suffered systemic and/or pulmonary embolization ? Harrison’s 18th Ed. B. At least 1 year D. Table 237-3 Harrison’s 18th Ed. Dilated cardiomyopathy Harrison’s 18th Ed. extensive mitral A. conversion to sinus rhythm is rarely sustained when ? B. 1934 D. elevated levels of serum IgG and interleukin 6 (IL-6) are found. Mitral valve repair C.5 cm2/m2 body surface area A. 1934 A. Aortic valve replacement with coronary artery bypass B.170 MCQ’s FOR MEDICAL PROFESSIONALS BY PROF. 1932 Harrison’s 16th Ed. Subjacent myocardium A. chordae tendineae. Mitral valve replacement D. 1392 A. 1932 838 Operative mortality is maximum in which of the following ? Harrison’s 18th Ed. Any of the above D. Life long D. Papillary muscles Harrison’s 18th Ed. Severe MS papillary muscles. C.0 cm 2/m 2 body surface area or <1. At least 3 months B. A. Acute myocardial infarction with papillary muscle rupture 835 Mitral valvotomy is indicated in the symptomatic patient with C. hypertrophic obstructive cardiomyopathy (HOCM). 20 % B. 1. 1933. 1932 Chronic MR can result from rheumatic disease. Pulmonic valve replacement Cardioversion should be undertaken after the patient has had at least 3 consecutive weeks of 840 Which of the following is a component of mitral valve apparatus ? anticoagulant treatment to a therapeutic INR. Interleukin 8 (IL-8) In the pregnant patient with MS not responding to intensive medical treatment. D.5 cm 2 in normal-sized adults. 3 week C. In LA myxoma. B. Infective endocarditis B. and dilated cardiomyopathy. annulus. Interleukin 4 (IL-4) C. 50 % D. AF has been present for more than 1 year Five functional components of the mitral valve apparatus are leaflets. TEE C. Dysfunction in any of these may cause MR.5 cm2/m2 body surface area C. CT annular calcification. 30 % C. Harrison’s 18th Ed. Tricuspid valve surgery D.0 cm2/m2 body surface area Acute MR can occur in infective endocarditis. 4 week D. 10 % A. B. Life long Successful mitral valvotomy is defined by a 50% reduction in the mean mitral valve gradient 832 Anticoagulants should be administered for how long to patients and a doubling of the mitral valve area. 1. Ankylosing spondylitis Mitral valvotomy is indicated in symptomatic patients with isolated MS whose effective orifice is < ~1. and subjacent myocardium. papillary muscle rupture (post Acute MI). 2. All of the above A. 1934 particularly those in whom the LA is especially enlarged or in whom AF has been present for more than 1 year. 1933. At least 3 months A. cardioversion should be undertaken after patient has had anticoagulant treatment for ? 839 Operative mortality is maximum in which of the following ? Harrison’s 18th Ed.0 cm2/m2 body surface area 842 Chronic MR can result from all except ? Harrison’s 18th Ed. Mitral valve replacement with coronary artery bypass C. and rupture of chordae tendineae in MVP & IE. Aortic valve replacement 833 In MS with AF of relatively recent origin. Murmur resolves with treatment of anemia. 171 Cardiology MCQ’s FOR MCQ’s FOR MEDICAL MEDICAL PROFESSIONALS PROFESSIONALS BY PROF. Harrison’s 17th Ed.40 cm 2. Regurgitant volume >=60 mL/beat C.0 cm 849 Systolic murmur of MR. it was finally accepted & published in 1968 by British Heart Journal. soft.” After D. Acute pulmonary edema is 852 Barlow’s syndrome refers to ? common. 25 % 845 Which of the following defines severe MR ? Harrison’s 18th Ed. Short. v wave in the LA pressure pulse D.40 cm2 Rheumatic heart disease is the cause of chronic MR in only about one-third of cases and occurs more frequently in males. Midsystole C. D. rumbling. Posterior mitral leaflet intensified by isometric exercise (handgrip) but is reduced during the strain phase of the C. United Kingdom A. South Africa 848 Which of the following is not ordinarily heard with isolated MR ? Harrison’s 18th Ed. Both mitral leaflets dimension reaches 6. Effective regurgitant orifice area >=0. transmitted to base of heart indicates B.3 cm 2 . Frequency of pulmonary edema Cabot . and billowing mitral leaflet syndrome. Canada C. Richard Cabot was an American physician and Frank Locke was his colleague.0. In this situation. floppy-valve syndrome. 1935 B. Systolic murmur of chronic MR not due to MVP is B. S4 South African physician John Barlow first submitted his work on mitral-valve prolapse to the journal Circulation. pulling it away from the mitral orifice & thereby aggravating valvular dysfunction. In chronic severe MR. dilated cardiomyopathy when left ventricular end-diastolic chordae tendineae and valve leaflets. Pericarditis A.17 seconds after A 2 is caused by sudden tensing of papillary muscles. All of the above N Engl J Med 2010.0 cm A. enlargement of LA places tension on ? in patients with ruptured chordae tendineae or primary involvement of the posterior mitral leaflet Harrison’s 18th Ed. 50 % B. Atrial myxoma 847 In mitral regurgitation due to papillary muscle dysfunction. 5. 33 % A. 1934 B. Anterior mitral leaflet chordae tendineae. D. A. regurgitant fraction 851 Which of the following best relates to Cabot . D. 1935 D. HOCM A. LA Harrison’s 18th Ed. resulting in wide but physiologic splitting of S 2. or buried in the holosystolic murmur. systolic murmur may have a cooing or “sea gull” quality. AJAY MATHUR Cardiology 171 843 MR occurs universally in patients with nonischemic forms of aortic valve may close prematurely. Hyperthyroidism 846 Acute severe MR & chronic severe MR are differentiated by ? B. Low-pitched S 3 occurring 0. B. & strikes LA wall adjacent to aortic root. Holosystolic murmur C.Locke murmur is a diastolic murmur heard best at the left sternal border. S3 D. Configuration of systolic murmur similar to aortic insufficiency but does not have a decrescendo. but the manuscript was refused for its “overstated conclusion. 66 % C. 1934 with prolapse or flail. regurgitant jet is eccentric.363:22 A. S 4 is heard in acute severe MR in sinus rhythm. the systolic murmur commences in ? C. Murmur is classically holosystolic in timing and plateau in configuration. Chordae tendineae Valsalva maneuver.Locke murmur ? (RF) >=50%. 1937 v wave is relatively less prominent. Barlow’s syndrome. and effective regurgitant orifice area >=0. In chronic MR. Regurgitant fraction (RF) >=50% D. All of the above 850 Rheumatic heart disease is the cause of chronic MR in what Chronic severe MR is progressive since enlargement of LA places tension on posterior mitral percentage of cases ? leaflet. All of the above Severe nonischemic MR is defined by a regurgitant volume >=60 mL/beat. Murmur is early in timing and decrescendo in configuration. Posterior mitral leaflet Annular dilatation and ventricular remodeling contribute to MR that occurs universally among patients with nonischemic forms of dilated cardiomyopathy once the left ventricular end-diastolic C.0 cm Harrison’s 18th Ed. Presystolic murmur considerable abbreviation of the paper. All of the above In acute severe MR. 3. D. even in the dimension is ? absence of structural MS may follow. . A Harrison’s 18th Ed.12 . Anterior mitral leaflet B. In patients with ruptured A.363:22 associated with an effective regurgitant orifice area of >0. 1934 C. v wave in LA pressure pulse is prominent. Early systole MVP is also termed systolic click-murmur syndrome. In severe MR. A. directed anteriorly. Endocarditis B. 1469 A. systolic murmur is transmitted to base of heart. Anemia Harrison’s 18th Ed. while a flail leaflet may cause a murmur with a musical quality. 6. mid-diastolic murmur. 1934 presystolic murmur is not ordinarily heard with isolated MR. 1935 D. A. Severe ischemic MR is usually N Engl J Med 2010. Late systole 853 John Barlow belonged to which country ? D. S1 is generally absent. It sounds C. 4. Bicuspid aortic stenosis B. Mitral valve prolapse Harrison’s 18th Ed.0 cm. Mitral valve annulus 844 In severe chronic MR. Australia B.0 cm primary involvement of ? C. Late systolic murmur heard best at apex A. MVP causes ventricular arrhythmias due to excessive stress on papillary muscles which lead to dysfunction & ischemia of papillary muscles and subjacent ventricular myocardium. systolic click & murmur occur earlier with standing. C.365:507 A. 860 Which of the following valvular heart disease is more common 866 Which view in echocardiography is best to diagnose mitral in female ? valve prolapse ? Harrison’s 18th Ed. B. Adult patients with symptomatic valvular AS A. Adult patients with symptomatic valvular AS Harrison’s 18th Ed. 1937 C. During the strain phase of Valsalva maneuver B. 1937 Harrison’s 18th Ed. 1472 859 Which of the following valvular heart disease is more common A. 1937 D. Infective endocarditis in male ? Harrison’s 18th Ed. Systolic clicks may be multiple B. which increase LV volume. crescendo-decrescendo configuration (“whooping” or “honking”) and is heard best at the apex. Rheumatic MR C. 1937 MVP is a frequent finding in patients with heritable disorders of connective tissue. All of the above 863 Which of the following about MVP is false ? Harrison’s 18th Ed. Reduced production of type III collagen B. Osteogenesis imperfecta 862 In MVP. Sudden cardiac death A. High-pitched. Patients with pure or predominant valvular AR A.13 seconds Harrison’s 18th Ed. A.14 seconds A. Squatting A. Tricuspid Stenosis Harrison’s 18th Ed. Mitral stenosis (MS) D. 1937 C. Ventricular septal defect D. diminish click-murmur complex of MVP thus moving it away from S1. Patent ductus arteriosus 864 Systolic click & murmur of MVP occur earlier in all of the following except ? Mitral valve prolapse (MVP) is associated with 20% of patients with ostium secundum ASD. Parasternal short-axis view . Various cardiomyopathies D. 1937 B. 1937 MVP may occur as a sequel to acute rheumatic fever. 1937 B. MVP Sudden death is a rare complication & occurs in patients with severe MR & depressed LV systolic function. 0. Mitral annulus C. None of the above B. nonejection systolic click occurs how many seconds C. None of the above A. Standing Harrison’s 18th Ed. Harrison’s 18th Ed. 865 Most frequent serious sequelae of mitral valve prolapse is ? Harrison’s 17th Ed. late systolic crescendo-decrescendo 857 Mitral valve prolapse (MVP) is associated with which of the murmur following ? Harrison’s 18th Ed. Posterior leaflet is more affected than anterior Harrison’s 18th Ed. 1937. 1937 858 Ventricular arrhythmias in MVP result from stress placed on ? A. MVP is the commonest cause of isolated severe MR requiring surgical D. All of the above C. during strain phase of Valsalva maneuver. 0.14 seconds or more after S 1 and B. MVP B.12 seconds 856 Mitral valve prolapse (MVP) may occur as a sequel to ? C. AJAY MATHUR Cardiology 854 Which of the following statements about MVP is false ? C. Hemophilia Harrison’s 18th Ed. and in various cardiomyopathies. Severe mitral regurgitation B. None of the above C.172 MCQ’s FOR MEDICAL PROFESSIONALS BY PROF. elongated chordae tendineae or due to prolapsing mitral leaflet when it reaches its maximum excursion. Parasternal long-axis view B. Chordae tendineae In MVP. Ostium primum atrial septal defect D. Rheumatic MR A. the late systolic murmur is high-pitched. Conversely. Acute rheumatic fever In MVP. Down syndrome D. the mid. C. including A. Papillary muscles and with any intervention that decreases LV volume. 1937 D.or late (nonejection) systolic click occurs 0. 1937 D.11 seconds Marfan syndrome. Patients with pure or predominant valvular AR B. Lancet 2005. squatting & isometric exercises. Ischemic heart disease is due to sudden tensing of slack. AR with associated mitral valve disease treatment. osteogenesis imperfecta & Ehler-Danlos syndrome. in ischemic heart disease. Rheumatic MR 855 Mitral valve prolapse has association with heritable disorder like ? B. Posterior mitral leaflet D. Increased concentrations of acid mucopolysaccharide 861 Which of the following valvular heart disease is more common in female ? C. Ostium secundum atrial septal defect In MVP. 0. Turner syndrome after S1 ? D. 0. D. Bicuspid aortic valve (BAV) Lancet 2005. Chronic trileaflet aortic valve deterioration A. Myofibroblast Lancet 2005. A. valvular myofibroblasts differentiate phenotypically C. All of the above C. Prolapse > 2 mm beyond long-axis annular plane D. 873 In MVP.365:507 many features with ? A. 1937 B. Increased left-ventricular diameter Harrison’s 18th Ed. Atherosclerosis C. most cases have bicuspid aortic valve (BAV) disease. 173 Cardiology MCQ’s FOR MCQ’s FOR MEDICAL MEDICAL PROFESSIONALS PROFESSIONALS BY PROF. 30 % Harrison’s 18th Ed. Structurally abnormal chordae tendinae AS in adults is due to degenerative calcification of the aortic cusps and occurs most commonly on an existing disease like bicuspid aortic valve. Posterior leaflet prolapse B. Middle scallop prolapse In aortic valve deterioration & calcification. Thickening of the valve leaflets 80% of adult patients with symptomatic valvular AS are male. D. 80 % C. All of the above . AJAY MATHUR Cardiology 173 C. All of the above Echocardiographic definition of MVP is systolic displacement (in the parasternal long axis view) 875 Degenerative calcification in AS occurs most commonly over of mitral valve leaflets by at least 2 mm into LA superior to the plane of the mitral annulus. 1938 complications are ? Lancet 2005. All of the above D. Associated AR C. Moderate-severe mitral regurgitation C. All of the above A. All of the above AS are male ? Harrison’s 18th Ed. Out of these. and above which of the following ? Harrison’s 18th Ed.365:507 atherosclerosis. Macrophage B. Foam cell 871 Transthoracic echocardiography (TTE) does not exclude ? B. Medial scallop prolapse D.365:507 Harrison’s 18th Ed. cytokine release. Mitochondrial diseases D. Leaflet thickness B. All of the above rheumatic inflammation. inflammatory cell activation. Neoplasia 870 Which of the following echocardiographic parameters is useful to risk stratify patients with mitral valve prolapse ? The process of aortic valve deterioration & calcification shares many features with vascular Lancet 2005. Increased left ventricular dimensions C. 1938 868 Which of the following is true in mitral valve prolapse syndrome ? A.365:507 A. including endothelial dysfunction. 1938 B. 869 Non-prolapse related systolic clicks are documented in ? 876 Process of aortic valve deterioration and calcification shares Lancet 2005. Pericarditis B. All of the above C. chronic trileaflet deterioration. Structural alterations of collagen in all components of leaflet D. Bicuspid aortic stenosis Harrison’s 18th Ed. Previous rheumatic inflammation of aortic valve B. Atrial myxoma A. and upregulation of several signaling pathways. Lateral scallop prolapse into osteoblasts and actively produce bone matrix proteins that allow for the deposition of calcium hydroxyapatite crystals. Presence of thickened leaflets A. 1938 D. Mitral leaflet thickness greater than 5 mm D. 60 % B. Congenital Mitral stenosis D. Bicuspid aortic valve (BAV) C. Depressed left ventricular systolic function B. which of the following echocardiographic finding 879 Which of the following is the most common congenital heart predicts a greater risk of severe MR ? valve defect ? Lancet 2005. None of the above 878 A rheumatic etiology of aortic stenosis is favored by ? 872 Patients with mitral valve prolapse at increased risk of Harrison’s 18th Ed. Rheumatic involvement of mitral valve B. 50 % A. or previous D. lipid accumulation. Expansion of spongiosa layer by proteoglycans C. Four-chamber view 874 What percentage of adult patients with symptomatic valvular D. Redundancy 877 Which of the following is central in the pathogenesis of aortic valve deterioration & calcification ? C. D.365:507 B.365:507 C. 1937 867 “Classic” mitral valve prolapse is defined as ? A. Endothelial cell A. Prolapse of mitral valve C. History of rheumatic fever A. All of the above Rheumatic AS is almost always associated with involvement of the mitral valve and with AR. Apoptosis D. Congenital Pulmonic stenosis B. 1938 A. ~5% A. 1938 C. 1939 B. Epicardium C. Holt-Oram syndrome D. 1939 881 Bicuspid aortic valve disease may be associated with ? Harrison’s 18th Ed. compressive forces are increased.8 cm2/m2 body surface area Harrison’s 18th Ed. NOTCH3 C. 1939 A. Myocardium B. Exertional dyspnea B. AF in AS suggests the possibility of associated mitral valve disease. 1938 A. In suprasternal notch B. 1938 Harrison’s 18th Ed. 1938 Harrison’s 18th Ed. Capillary density is reduced relative to wall thickness. P = pressure. NOTCH1 A. elevated LV end-diastolic pressure reduces the coronary driving pressure. Systolic wall stress = Pressure x Radius / Wall thickness Harrison’s 18th Ed. 1939 Harrison’s 18th Ed. Along carotid arteries C. Angina pectoris C. ~ 20 % D..4:1 male to female predominance. ischemia of ? Harrison’s 18th Ed. 882 Prevalence of BAV disease among first-degree relatives of an affected individual is ? 888 Cardinal symptom in aortic stenosis is ? Harrison’s 18th Ed. Turner’s syndrome Hypertrophied LV in AS causes an increase in myocardial oxygen requirements. 1939 0. Autosomal recessive with incomplete penetrance D. 0. 1938 891 Which of the following is false about aortic stenosis murmur ? A. Inheritance pattern of bicuspid aortic valve disease is autosomal dominant with incomplete 887 In AS. Rough & rasping in character D. Systolic wall stress = Pressure + Radius / Wall thickness C. angina pectoris and syncope are the three cardinal symptoms in patients with pure or predominant AS. Autosomal recessive with complete penetrance A mean systolic pressure gradient >40 mmHg with a normal CO or an effective aortic orifice C. Autosomal dominant with complete penetrance of the normal orifice is considered to represent severe obstruction to LV outflow. ~ 15 % C. 1. Ellis-van Creveld syndrome B. Exertional dyspnea. Absence of systolic thrill C. r = radius.5–1. in suprasternal notch & along carotid arteries 885 Laplace relation is ? (more commonly left).2 cm2/m2 body surface area B. and the X-linked inheritance is suggested by prevalence of BAV disease in patients with Turner’s syndrome. 884 Which of the following occur commonly among patients with 890 In aortic stenosis. 1938 A. Ejection systolic Systolic stress developed by myocardium is predicted by Laplace relation (S = Pr/h. All of the above Mutation in NOTCH1 gene has been described in some families suffering from BAV disease.e. Atrial fibrilation B. 1939 A. Noonan syndrome C. High pitched C. severe obstruction to LV outflow is considered when effective aortic orifice area is ? A bicuspid aortic valve (BAV) is the most common congenital heart valve defect and occurs in Harrison’s 18th Ed.6 cm2/m2 body surface area 880 Inheritance pattern of bicuspid aortic valve disease is ? B.0 cm2 (or ~<0. Autosomal dominant with incomplete penetrance area < ~1. Syncope D. where S = D. hypertrophied LV compresses coronary arteries causing penetrance. All of the above D. 0. AJAY MATHUR Cardiology D. 883 Mutation in which of the following gene may be associated 889 Which of the following excludes the possibility of pure or with BAV disease ? predominant severe aortic stenosis ? Harrison’s 18th Ed.4% of the population with a 2 .174 MCQ’s FOR MEDICAL PROFESSIONALS BY PROF. All of the above D. A. .6 cm2/m 2 body surface area in a normal-sized adult) i. Harrison’s 18th Ed. Systolic wall stress = Wall thickness x Radius / Pressure B. Aortic dissection D. All of the above Prevalence of BAV disease among 1º relatives of an affected individual is ~10%. Subendocardium A. Congenital Tricuspid stenosis 886 In normal sized adults. NOTCH2 B. systolic thrill is generally present at ? BAV disease ? Harrison’s 18th Ed. 1. NOTCH4 D. < ~ one-third D. Subendocardium is especially vulnerable to ischemia by this mechanism.0 cm2/m2 body surface area A. Loudest in second right intercostal space systolic wall stress. Systolic wall stress = Pressure x Wall thickness / Radius A. Ascending aortic aneurysm C. ~ 10 % B. Aortic coarctation B. Base of heart A. All of the above Systolic thrill is present at the base of heart. Bisferiens pulse D. and h = wall thickness). 0 cm . <=3. Proximal ascending aorta not dilated following ? C. Absence of valvular calcification Harrison’s 18th Ed. 1941 The intensity of a cardiac murmur is graded according to the Levine scale. None of the above leads to early death ? Harrison’s 18th Ed. Hypertension A.1. increasing in 898 Which of the following Chest X-Ray findings indicate that intensity to reach a peak toward middle of ejection & ending just before aortic valve closure. Apex B. > 10 mm Harrison’s 18th Ed.0 cm2 . when the aortic valve area is ? A.0 cm2 to primary valve disease ? Harrison’s 18th Ed.0 cm2 . A. Aortitis 896 Aortic stenosis is called “severe”. > 15 mm A. Syphilis C. Causes of AR D. C. Angina pectoris almost all patients with severe obstruction & preserved CO.5 meters/second B. 1930. traumatic. Little overall cardiac enlargement 892 In Gallavardin effect.5 cm2 C. Bicuspid aortic valve A. Morrison scale After onset of symptoms in AS.5 cm2 C.2 cm 2 due to primary aortic root disease include aortic dissection. It valvular AS is not severe ? is characteristically low-pitched. <=5.5 meters/second C. None of the above A.5 to 2 years. bicuspid aortic valve. Marfan’s syndrome D. Congenital AR B. 1939 D. 2.5 meters/second D. when the aortic valve area is ? B.5 cm2 . 1940 B. Mogagni scale dyspnea .2 years and congestive heart failure . the murmur of AS is at least grade III/VI. Table 237-1 A. cystic medial degeneration (Marfan’s syndrome. 2. . endocarditis. Carotids In chest X-Ray.0 cm 2 . <=2. syphilis and ankylosing spondylitis. < 0.2. Aortic dissection Harrison’s 18th Ed. myxomatous (prolapse).0 cm2 Severe LV hypertrophy is present when a wall thickness is > 15 mm. Marfan’s syndrome C. Table 237-1 895 Aortic stenosis is called “moderate”. <=4. C.1. Rheumatic fever C.2. nonsyndromic familial aneurysm). 1.2. Marfan’s syndrome B. absence of valvular calcification in adults suggests that severe valvular AS is not present.5 cm 2 2 D. All of the above Aortic sclerosis is defined echocardiographically as focal thickening or calcification of the valve Coexistence of hemodynamically significant AS with AR occurs almost exclusively in patients cusps with a peak Doppler transaortic velocity of <=2.5 cm2 Causes of AR due to primary valve disease include congenital (bicuspid). 897 Aortic sclerosis is defined echocardiographically as focal thickening or calcification of valve cusps with a peak Doppler 903 Coexistence of hemodynamically significant AS with AR occurs transaortic velocity of ? in which of the following ? Harrison’s 18th Ed. 1. 2. Syncope C. Back 899 Following onset of symptoms in AS. Congestive heart failure 893 Intensity of a cardiac murmur is graded (1-6) according to ? D.5 cm2 . aortitis and hypertension. where it is loudest with musical quality (Gallavardin effect) and may be confused with systolic murmur of MR. 1930. 1943 A. 1.5 meters/second at a peak gradient <25 mmHg.0 cm2 .5 cm2 902 Which of the following is the cause of aortic regurgitation due D.1. < 0. It is transmitted along carotid arteries. 1940 Occasionally. < 1. which of the following D. Harrison’s 18th Ed. A. life expectancy in angina pectoris & syncope is 3 years. rheumatic fever.1.0 cm 2 D.2. murmur of AS radiates to which of the B. < 1.3.5 cm2 901 Which of the following is the cause of aortic regurgitation due D. Osler scale Harrison’s 18th Ed. when aortic valve area is ? Harrison’s 18th Ed. Levine scale 900 Severe LV hypertrophy is present when a wall thickness is ? D. > 5 mm 894 Aortic stenosis is called “mild”. In A. B.5 cm2 .3. 1940 C. with rheumatic or congenital AR. 175 Cardiology MCQ’s FOR MCQ’s FOR MEDICAL MEDICAL PROFESSIONALS PROFESSIONALS BY PROF. 1940 Harrison’s 18th Ed. 1. AJAY MATHUR Cardiology 175 Murmur of AS is ejection (mid) systolic murmur commencing shortly after S 1. murmur of AS may be transmitted downward to the apex. 1940 2nd right intercostal space. B. Absence of pulmonary congestion A.5 cm2 .5 cm 2 . when the mitral valve orifice is 1.0 cm2 to both primary valve disease & primary aortic root disease ? Mitral stenosis is called “moderate”. Ankylosing spondylitis B. C. > 20 mm B. 1940 B.0 cm2 D.5 meters/second A. 2. rough & rasping in character & loudest at the base of heart in Harrison’s 18th Ed. HOCM D. arteries is called ? Harrison’s 18th Ed. Reduction in LVEF B. 1944 908 A large fraction of coronary blood flow occurs during ? A. D. Quincke’s sign (pulsation Myocardial ischemia. Quincke’s pulse A. 1943 B. 1943 A. Ankylosing spondylitis C. Deterioration of LV function often precedes the development of symptoms. Aortic dissection B. 1943 Harrison’s 18th Ed. Elevation of LA pressure A. and elevated LV systolic tension. All of the above Other peripheral signs of AR include Corrigan’s pulse (rapid upstroke & collapse of carotid artery pulse). 911 Which of the following about AR is false ? Harrison’s 18th Ed. Lighthouse sign (blanching & flushing of forehead). heard over femoral artery when it is gradually compressed). Duroziez C. aortic dissection. Traube’s sign B. Systole C. Vomiting A. booming “pistol-shot” sound heard over femoral arteries is termed as Traube’s sign. hypertrophy. 1943 Harrison’s 18th Ed. paroxysmal nocturnal dyspnea. 1943 B. Corrigan’s pulse Harrison’s 18th Ed. Watson A. weighing >1000 grams. Ankylosing spondylitis D. 1943 A. All of the above D. Corrigan Harrison’s 18th Ed. AS B. Duroziez’s sign C. 1943 905 Which of the following condition predisposes to AR ? A. or trauma (Nonpenetrating Both syphilis & ankylosing spondylitis affect aortic valves. leading to aortic dilation. LV dilation C. Any of the above A large fraction of coronary blood flow occurs during diastole. They may also be associated with cardiac injury leading to rupture or avulsion of the aortic valve). MR In chronic AR. particularly of the subendocardium. Syphilis B. Diastole D. both LV preload & afterload increase. occurs in AR because of myocardial of capillary bed in nails). booming “pistol-shot” sound heard over femoral early sign of LV dysfunction is a reduction in the EF. Exertional dyspnea is the first symptom of diminished cardiac C. Elevation of LV end-diastolic pressure D. Landolfi’s sign (alternating constriction & dilatation of pupil). 1943 following valvular heart disease ? Harrison’s 18th Ed. 3/4th of patients with predominant valvular AR are men Harrison’s 18th Ed. Vertigo C. Dilated cardiomyopathy Exertional dyspnea is followed by orthopnea. 912 Which of the following is a symptom of chronic severe AR ? 906 At autopsy. Chronic AR C. Becker’s sign (pulsations . Elevation of PA wedge pressure C. Duroziez’s sign (a double sound and coronary blood flow may be compromised due to low arterial pressure. Nonpenetrating cardiac injury C. 1943 A. & severe aortic regurgitation. Elevated LV systolic tension D. Primary valvular AR with rheumatic mitral valve disease more common in women B. hearts of patients with chronic AR may be among the largest encountered sometimes may be accompanied by marked diaphoresis. and excessive diaphoresis. ankylosing spondylitis & VSD. An 914 In free AR. end-diastolic volume rises further and the forward stroke volume and EF decline. When adaptive measures fail. aneurysm formation. Gait disturbance B. AJAY MATHUR Cardiology 904 Which of the following is the cause of aortic regurgitation due 910 Which of the following is a cause of acute severe AR ? to both primary valve disease & primary aortic root disease ? Harrison’s 18th Ed. de Musset B. Infective endocarditis A. Chronic AS B. heard over femoral artery when it is compressed distally). AR C. Bicuspid aortic valve C. Marfan syndrome B. Ventricular septal defect reserve D. Traube’s sign (systolic & diastolic murmurs described as ‘pistol shots’ oxygen requirements are elevated by LV dilation. 915 Large-volume ‘collapsing’ water hammer peripheral pulse seen in AR is named after ? 909 Myocardial ischemia occurs in AR because of ? A. MS D. LV function deteriorates. Low arterial pressure Large-volume ‘collapsing’ water hammer peripheral pulse seen in AR is named after Watson. Diaphoresis D. All of the above Acute severe AR results from infective endocarditis. Systole + diastole In free AR.176 MCQ’s FOR MEDICAL PROFESSIONALS BY PROF. cellular infiltration & scarring of media of thoracic aorta. heart of which of the following lesions is heaviest ? Harrison’s 18th Ed. 907 Early sign of LV dysfunction in chronic AR is ? 913 Diaphoresis is a predominant symptom of which of the Harrison’s 18th Ed. None of the above Conditions that predispose to AR are Marfan syndrome. de Musset’s sign (head nodding in time with heart beat). All of the above D. D. 1943 A. Nocturnal angina that does not respond satisfactorily to sublingual nitroglycerin At autopsy. 1944 D. Sherman sign (dorsalis pedis pulse is quickly located & unexpectedly prominent in age >75 years). decrescendo diastolic murmur. Due to severe hypertension A. ST depression & T-wave inversion in I. 1944 926 In AR. QRS prolongation 916 The murmur of AR is typically heard best in ? Harrison’s 18th Ed. 1944 D. which of the following findings Surgery should be carried out in asymptomatic patients with severe AR and progressive LV usually signify a poor prognosis ? dysfunction defined by an LVEF <50%. Gerhardt’s sign (enlarged spleen). II left ICS parasternally B. Marfan’s syndrome D. 1944 A. Intravenous diuretics 919 Which of the following murmur is heard in chronic AR ? B. central jet width assessed by color flow Doppler imaging exceeds 65% of left B. Australia D. Soft. early diastolic murmur of AR B. III left ICS parasternally 923 On Echocardiogram. AJAY MATHUR Cardiology 177 of retinal vessels). Long-acting nifedipine aneurysm is ? N Engl J Med 2004. Harrison’s 18th Ed. 1944 >55 mL/m 2. Intraaortic balloon counterpulsation is contraindicated. Mid-systolic ejection murmur D. Increased systolic excursion of posterior LV wall Murmur of chronic AR is typically high-pitched. Mayen’s sign (diastolic drop of BP >15 A.a >20 mm Hg difference in popliteal & brachial systolic cuff pressures in chronic B. short. or an LV diastolic dimension >75 mm. It seldom causes thrill. All of the above diagnosis. Nondihydropyridine calcium channel blockers 921 Most common pathologic condition associated with aortic D. blowing. LVEF < 50% B. 177 Cardiology MCQ’s FOR MCQ’s FOR MEDICAL MEDICAL PROFESSIONALS PROFESSIONALS BY PROF. C. it suggests that AR is caused by aneurysmal dilatation of aortic root. Vasodilators (sodium nitroprusside) Harrison’s 18th Ed. operation should be carried out in ? Harrison’s 18th Ed. B. Accompanied by significant MS B. regurgitant volume is >=60 mL/beat. American With severe AR. severe AR D. Regurgitant fraction > 50% 918 Austin Flint (1812-86) was a physician of which country ? D. Rosenbach’s sign (pulsatile liver). which of the following can be found in a C. Decrescendo diastolic murmur C. 920 Which of the following is a feature of acute severe AR ? Harrison’s 18th Ed. All of the above C. Surgery is the treatment of choice and is usually necessary within 24 hours of D. an American physician (1812-1886) was a pioneer in the use of the stethoscope. heard best in 3rd intercostal space along left sternal border. and diastolic flow reversal in the proximal descending thoracic aorta is seen. All of the above 922 In ECG of patient with AR. 1946 A. which of the following drugs can delay the need for surgery ? A. Digitalis glycosides C. A. Acute.351:1539-46 927 In AR. C. Central jet width > 65 % of left ventricular outflow tract When murmur of AR is heard best along right sternal border. Atherosclerosis B. Regurgitant volume > 60 mL/beat C. United Kingdom ventricular outflow tract. Dilatation of aortic annulus 917 When murmur of AR is heard best along right sternal border. LV end-systolic volume > 55 mL/m 2 C. Müller’s sign (pulsations of uvula). Tuberculosis C. Diuretics D. Canada 925 Which of the following is contraindicated in the treatment of Austin Flint. Intraaortic balloon counterpulsation B. Beta blockers are also best avoided as they may reduce CO further. regurgitant fraction is >=50%. A. Accompanied by Infective endocarditis 924 Which of the following findings in color flow Doppler imaging C. High-frequency diastolic fluttering of anterior mitral leaflet C. aVL severe AR. it D. 1401 B. Mid-to-late diastolic murmur Patients with acute severe AR may respond to IV diuretics and vasodilators like sodium nitroprusside. ST depression & T-wave inversion in V 5 & V 6 D. His “A acute aortic regurgitation ? Treatise on the Principles and Practice of Medicine” (1866) was a leading textbook of medicine. 1944 Left axis deviation and/or QRS prolongation denote diffuse myocardial disease and signify a poor prognosis. 1944 A. IV left ICS parasternally patient of AR ? Harrison’s 18th Ed. All of the above A. Pulse pressure not wide A. . Lincoln sign (pulsatile popliteal artery). Surgery C. 1944 A rapid. No electrocardiographic abnormalities mm Hg with arm raised). high-frequency fluttering of the anterior mitral leaflet produced by the impact of the regurgitant jet is a characteristic finding in 2D echocardiogram of AR patient. All of the above suggests that AR is ? Harrison’s 18th Ed. Syphilis A. Due to aneurysmal dilatation of aortic root suggest severe AR ? Harrison’s 18th Ed. Hill’s sign . B. an LV end-systolic dimension >55 mm or end-systolic volume Harrison’s 18th Ed. Soft S1 Harrison’s 16th Ed. V left ICS parasternally A. > 25 mm Hg A. All of the above D. Absence of ECG evidence of RVH B. jugular veins may show ? 936 Functional tricuspid regurgitation (TR) may be observed in ? Harrison’s 18th Ed. Slow ‘y’ descent C. reduced during expiration and during the strain phase of Valsalva maneuver. Pulmonary A diastolic pressure gradient between the RA and RV defines TS. Nonrheumatic causes of TS are rare. Less evidence of pulmonary vascular congestion D. TR C. Rheumatic TS is (RVH) in a patient with right-sided heart failure. 929 Systemic venous congestion sets in when mean RA . radiation. tricuspid valve prolapse. Presystole C. 1948 A. 1 mm Hg A. 937 Severe pulmonary artery hypertension is defined when pulmonary artery systolic pressure is ? Harrison’s 18th Ed. 930 In Tricuspid stenosis (TS). Ischemic cardiomyopathy C. In combined MS & TS there is absence of ECG evidence of right ventricular hypertrophy It does not occur as an isolated lesion and is usually associated with MS. All of the above Murmur of TS is most prominent during presystole in patients with sinus rhythm. Pulmonic stenosis C. 1948 A. 1948 931 Which of the following statements is false ? Harrison’s 18th Ed. murmur of TS is most prominent during ? 939 Carcinoid syndrome may cause ? Harrison’s 18th Ed. Less conspicuous ‘v’ wave B. 3 mm Hg C. than in other positions. 1946 Harrison’s 18th Ed. TS is almost always accompanied by significant TR. TS is almost always accompanied by significant TR D. 1947 A. Idiopathic dilated cardiomyopathy D. > 35 mm Hg B. . None of the above Tricuspid stenosis (TS) is generally rheumatic in origin and more common in women than men. Development of MS precedes that of TS B. 1946-47 A. It is augmented Carcinoid syndrome may cause TR. 1948 A. 933 In sinus rhythm. superior vena cava without much enlargement of PA with less evidence of pulmonary vascular congestion. “v” waves are less conspicuous with a slow “y” descent. Aortic C. 1947 Harrison’s 18th Ed. More common in women than men B. A mean diastolic pressure gradient of 4 mmHg is usually sufficient to elevate the mean RA pressure to levels that result Mechanical valves in the tricuspid position are more prone to thromboembolic complications in systemic venous congestion (hepatomegaly. Mitral B. Presence of TS can mask hemodynamic & clinical features of MS. Inferior MI B. None of the above Severe pulmonary artery hypertension is defined when pulmonary artery systolic pressure is > 55 mm Hg. > 55 mm Hg D. 2 mm Hg B.RV diastolic pressure gradient is ? 935 Thromboembolic complications are most when mechanical Harrison’s 18th Ed. All of the above Infarction of RV papillary muscles. 1946 Harrison’s 18th Ed. 938 Carcinoid heart disease predominantly affects which of the 932 Murmur of which of the following valvular heart disease is following heart valves ? Harrison’s 18th Ed. > 45 mm Hg C. All of the above In TS with sinus rhythm. edema). Early diastole A. All of the above D. Does not occur as an isolated lesion C. Mitral A. pulmonic stenosis and/or regurgitation. and trauma may produce Murmur of TS is best heard along left lower sternal margin and over xiphoid process. TS B. infective endocarditis. during inspiration. Mid systole B. Tricuspid C. ascites. VSD D. Presence of TS can mask clinical features of MS C. carcinoid heart disease. 1947 A. Prominence of RA & SVC without enlarged PA on CxR C. 1946 valves are used for which valve replacement ? Harrison’s 18th Ed. Tricuspid regurgitation B. Aortic D. Tricuspid B. 4 mm Hg D. jugular veins are distended with giant “a” waves. Giant ‘a’ wave A. TR. endomyocardial fibrosis. AJAY MATHUR Cardiology 928 Which of the following about tricuspid stenosis is false ? 934 Which of the following is false about combined MS & TS ? Harrison’s 18th Ed. Pulmonic regurgitation D. Development of MS generally precedes that of TS. Generally congenital in origin A. 1948 heard best over the xiphoid process ? Harrison’s 18th Ed. Pulmonary D.178 MCQ’s FOR MEDICAL PROFESSIONALS BY PROF. Rheumatic TS commonly associated with TR D. Chest x-ray shows prominence of RA & commonly associated with some degree of TR. 948 Pulmonic regurgitation occurs universally among patients 942 Carvallo’s sign is related to which of the following ? who have undergone which of the following ? Harrison’s 18th Ed. which may be intensified during inspiration & reduced during Pulmonic regurgitation occurs universally among patients who have undergone childhood repair expiration or the strain of the Valsalva maneuver (Carvallo’s sign). Rapid y descent C. All of the above Aortic insufficiency can be caused by serotonin reuptake inhibitors. PS 951 Bioprosthetic valves are indicated in ? Harrison’s 18th Ed. Marfan’s syndrome A. 179 Cardiology MCQ’s FOR MCQ’s FOR MEDICAL MEDICAL PROFESSIONALS PROFESSIONALS BY PROF. Mustard operation B. 1948 A. Tricuspid C. Senning operation D. Constrictive pericarditis D. 1948 B. c-v wave with a rapid y descent. decrescendo. Pulmonary In severe TR. TS D. Tricuspid regurgitation C. Mitral regurgitation B. 1948 Harrison’s 18th Ed. All of the above On Doppler examination. D. diastolic B. 1948 A. No x descent A. SVC obstruction contraindicated. 1948 Harrison’s 18th Ed. All of the above Graham Steell murmur due to severe pulmonary hypertension is a high-pitched. AJAY MATHUR Cardiology 179 940 Which of the following is seen in jugular venous pulsations in 946 Which of the following heart valves is least affected by infective tricuspid regurgitation ? endocarditis ? Harrison’s 18th Ed. Harrison’s 18th Ed. for women who expect to become pregnant & for those who refuse to take anticoagulation or for whom anticoagulation is C. All of the above B. Aortic D. 1949 D. None of the above D. Carcinoid heart disease Harrison’s 18th Ed. TR C. Tricuspid regurgitation Bioprostheses are preferred for patients >65 years. severe TR is accompanied by hepatic vein systolic flow reversal. prominent c-v wave with rapid y descent 947 Which of the following is false about ‘Graham Steell murmur’ ? is typical of ? Harrison’s 18th Ed. Women who expect to become pregnant is a feature of ? Harrison’s 18th Ed. Tricuspid regurgitation C. All of the above A. 1948 C. Cisplatin D. systolic pulsations of liver. Pericarditis D. pleural effusions. Constrictive pericarditis D. and a positive 950 Which of the following can produce mixed valve lesions ? hepatojugular reflex. TR C. 1948 Harrison’s 18th Ed. TR. PS D. Repair of tetralogy of Fallot C. Prominent c-v wave B. edema. TS C. Penicillin B. All of the above B. Ross procedure A prominent RV pulsation along the left parasternal region and a blowing holosystolic murmur along the lower left sternal margin. 1948 C. specifically medications containing fenfluramine or dexfenfluramine isotopes. RA pressure pulse may exhibit no x descent during early systole but a prominent diastolic blowing murmur along the left sternal border. 941 In RA pressure pulse. Decrescendo. 1948 A. are characteristic findings of of tetralogy of Fallot with reconstruction of the RV outflow tract. Aortic regurgitation A. Elderly (> 65 years) 945 On Doppler examination. it is uncommonly a nidus for infective endocarditis. “hepatic vein systolic flow reversal” B. systemic venous congestion causes right-sided heart failure manifesting as marked hepatomegaly. In severe TR. 1948 944 Hepatic vein systolic flow reversal on Doppler A. RA pressure pulse shows no x descent during early systole but a prominent c-v Pulmonic valve is affected by rheumatic fever far less frequently than are the other valves. Mitral B. In TR. In whom anticoagulation is contraindicated A. . Fenfluramine A. Mediastinal radiation echocardiography is a feature of ? B. ascites. 949 Which of the following drug can cause aortic insufficiency ? 943 A positive hepatojugular reflex is seen in ? A. and wave with a rapid y descent. Ebstein’s anomaly B. Methysergide Harrison’s 18th Ed. High-pitched A. and dopamine agonists. All of the above D. Along left sternal border C. Mitochondrial genes Harrison’s 18th Ed. Restrictive cardiomyopathy C. tropomyosin.180 MCQ’s FOR MEDICAL PROFESSIONALS BY PROF. Sarcomeric proteins C. Desmin forms intermediate filaments that connect the nuclear and plasma membranes. All of the above D. coronary or pericardial abnormalities. 955 Defects in sarcomeric proteins of myosin. Desmin mutations impair the transmission 963 Mostly. Genes encoding sarcomeric proteins 957 Which of the following is a sarcomeric protein ? C. Desmosome complex Cardiomyopathies are a group of diseases that affect heart muscle itself & are not the result of hypertension or congenital or acquired valvular. 954 Most familial cardiomyopathies are inherited in which of the following pattern ? 960 Abnormal dystrophin can be acquired in infection by ? Harrison’s 18th Ed. Herpes simplex virus (HSV ) type 1 D. Desmosome complex D. All of the above Prominent atrial enlargement is a distinguishing feature of restrictive cardiomyopathy. 1481 Harrison’s 18th Ed. Myosin Myocarditis C. Congenital or acquired valvular abnormality A. Hypertension connecting the cell membrane to intracellular structures” ? Harrison’s 18th Ed. Tropomyosin D. 1951 A. 1956. 1 /4 A. Dilated cardiomyopathy A. 3 . X-linked inheritance C. and troponin are mostly associated with consequence of coronary artery disease. autosomal recessive and X-linked inheritance. Autosomal dominant A. Genes encoding nuclear envelope protein lamin A/C B. 1 /2 B. 1952 A. Left ventricular wall thickness C. Most of the identified genetic defects in the Z-disk and cytoskeleton are associated with dilated cardiomyopathy. actin. Sarcomeric proteins include actin. Arrhythmia D. myosin. Dilated cardiomyopathy B. 4 / D. familial forms of DCM are due to mutations in ? of force and signaling for both cardiac & skeletal muscle leading to a peripheral myopathy and Harrison’s 17th Ed. Dystrophin complex C. All of the above 952 Cardiomyopathy can be a result of which of the following ? Harrison’s 18th Ed. Dystrophin complex D. All of the above Up to one-third of cases of dilated cardiomyopathy may be familial. D. 1956. 1 /2 C. with the remainder the Defects in sarcomeric proteins of myosin. Hypertrophic cardiomyopathy D. with occasional Abnormal dystrophin can be acquired when Coxsackie virus cleaves dystrophin in viral myocarditis. Figure 238-1 A. 1 /4 B. Sarcomeric proteins B. 1952 A. which of the 959 Which of the following is related to “cell-cell connections and following has most relevance ? myocyte stability” ? Harrison’s 18th Ed. 1481 a dilated cardiomyopathy. and the intercalated disks between muscle cells. 4 / About one in three cases of CHF is due to dilated cardiomyopathy. Figure 238-1 D. 1956. 1 /3 C. 1953 cytoskeleton are associated with ? Harrison’s 18th Ed. 1951 Harrison’s 18th Ed. Epstein-Barr virus Most familial cardiomyopathies are inherited in an autosomal dominant pattern. Atrial size A. hypertrophic cardiomyopathy. A. 1951 Harrison’s 18th Ed. Any of the above D. 3 . AJAY MATHUR Cardiology A. All of the above 953 In the diagnosis of restrictive cardiomyopathy. Dystrophin complex stabilizes and connects the cell membrane to intracellular structures. Hypertrophic cardiomyopathy C. Desmosome complexes are associated with cell-cell connections and stability. Restrictive cardiomyopathy B. Cardiomyopathy and B. 956 Most of the identified genetic defects in the Z-disk and 962 What proportion of dilated cardiomyopathy are familial ? Harrison’s 18th Ed. Z- lines. and the associated regulatory proteins. None of the above . actin & troponin are 961 What proportion of congestive heart failure (CHF) is due to associated with ? dilated cardiomyopathy ? Harrison’s 17th Ed. Left ventricular diastolic dimension B. Coxsackie virus B. Varicella-zoster virus C. 1951 958 Which of the following is related to “myocyte stabilizing and A. Pericardial abnormalities B. None of the above C. Figure 238-1 B. Actin Chapter 238. Autosomal recessive B. 1 /3 D. cardiomyopathy. Intraventricular conduction defects sudden cardiac death (SCD). Most patients should be Harrison’s 16th Ed. C. delayed motor A. diuretics. 320 grams cardiomyopathy ? B. Figure 238-4 971 Which of the following drugs should be avoided in a patient of A. Obesity & sleep apnea increases the risk of developing heart failure. thyroid disease. the long arm of X Harrison’s 17th Ed. C. Fraser syndrome 968 In which of the following conditions. chronic fatigue. Atrial fibrillation B. It is found exclusively in males. Pregnancy compromised immune system. Syndrome consists of metabolism distortion. cocaine use. heavy chain alpha-tropomyosin & troponins T. It is due to mutations in BTHS gene. ACE inhibitors or angiotensin II receptor 966 Which of the following is a feature of cardiac remodeling ? blocker. stamina deficiency. Calcium channel blockers Upper limit of normal of the weight of human heart is 360 grams. Alcohol D. 1954 969 ECG of a DCM patient may show all of the following except ? A. Figure 238-2 970 Which of the following should be avoided in dilated A. It is also known as 3-Methylglutaconic aciduria type II and Cardiomyopathy-neutropenia is reversible ? syndrome. delayed growth. 1957. All of the above A. AJAY MATHUR Cardiology 181 Most commonly. Mutations in mitochondrial genes have also been reported in DCM. D. Harrison’s 18th Ed. Ventricular arrhythmias proteins like alpha-cardiac actin. Alcohol abuse skills. Enlarged. 1481 chromosome and is associated with cardiolipin molecules in the electron transport chain & mitochondrial membrane structure. Increased myocyte size Harrison’s 17th Ed. Infectious agents A. ECG in a case of DCM shows sinus tachycardia or atrial fibrillation.& alpha-myosin. and digitalis produces symptomatic improvement. All of the above produced by which of the following ? Harrison’s 17th Ed. A. familial forms of DCM are due to mutations in genes encoding sarcomeric B. Myotonic dystrophy enlargement and hypertrophy. Calcium channel blocker interstitial fibrosis and myocyte hypertrophy characterized by increased myocyte size and enlarged. irregular nuclei. 1408 treated with a beta adrenergic blocker. beta. Metabolic factors 973 Cardiomyopathy-neutropenia syndrome is also called ? C. or toxic agents. Thyroid disease D. Cardiac hypertrophy conditions ? Harrison’s 17th Ed. 1482 C. leading to progressive cardiac B. 380 grams B. and B. Friedreich’s ataxia 967 Myocardial damage leading to dilated cardiomyopathy can be D. Barth syndrome DCM is either familial or the end result of myocardial damage produced by known or unknown C. Viral myocarditis Harrison’s 17th Ed. and chronic uncontrolled tachycardia. calcium channel blockers & NSAIDs should be avoided in DCM. All of the above 965 Histopathological feature of a dilated cardiomyopathy specimen is ? Alcohol. irregular nuclei A. hypotonia. diffuse nonspecific ST-T-wave abnormalities & sometimes intraventricular 964 Upper limit of normal of the weight of human heart is ? conduction defects and low voltage. Duchenne’s progressive muscular dystrophy Left and/or right ventricular systolic pump function is impaired. and C. I. left atrial abnormality. Spironolactone Standard therapy of heart failure with salt restriction. Mutations in gene encoding nuclear envelope protein lamin A/C are C. Cardiac enlargement 972 Cardiac involvement is common in which of the following C. 181 Cardiology MCQ’s FOR MCQ’s FOR MEDICAL MEDICAL PROFESSIONALS PROFESSIONALS BY PROF. Pompe disease D. A. All of the above B. Sarcoidosis . 340 grams Harrison’s 17th Ed. 1482 D. Harrison’s 18th Ed. 1481 Cardiac involvement is common in Duchenne’s progressive muscular dystrophy. Spironolactone should be added for most patients with recent or current advanced heart failure. tafazzin (TAZ) located at Xq28. 1958. All of the above B. Impaired ventricular systolic pump function B. Myotonic dystrophy and Friedreich’s ataxia. Peter Barth (Netherlands). Toxic agents B. Interstitial fibrosis dilated cardiomyopathy ? B. 1482 C. a process called remodeling. Holt-Oram syndrome infectious. Beta adrenergic blocker Microscopically. 974 Ventricular tachyarrhythmias may be a feature of ? Harrison’s 18th Ed. C. a dilated cardiomyopathy specimen would show nonspecific changes of C. 1482 A. ventricular arrhythmias. dilated cardiomyopathy Barth syndrome (BTHS) is named after Dr. 360 grams A. High voltage responsible for DCM associated with atrioventricular (AV) conduction disorder that may cause D. metabolic. D. Digitalis D. All of the above Myocarditis Reversible form of dilated cardiomyopathy may be found with alcohol abuse. pregnancy. Nonsteroidal anti-inflammatory drugs D. Aortic stenosis Chagas disease caused by protozoan Trypanosoma cruzi is transmitted by the bite of insect vector D. In Lyme disease. Schistosoma haematobium B. Conduction system abnormalities B. giant cell myocarditis. Third Features typical of Chagas’ disease are conduction system abnormalities. Benznidazole C. 980 Multiple left ventricular aneurysm formation is a feature of 986 Giant cell myocarditis may occur in association with ? which of the following ? Harrison’s 18th Ed. Radiation C. 977 Chagas disease is caused by ? Harrison’s 18th Ed. conduction abnormalities are the most common manifestations of cardiac involvement. Blood transfusion Harrison’s 17th Ed. 1958 A. 1486 B. Right ventricular dysfunction Transmission of protozoan T. All of the above D. Influenza A. Thyroiditis B. Hypersensitivity to drugs B. Pericarditis C. and occasionally orally. HIV D. Mitral regurgitation D. Mitral stenosis C. Melarsoprol While almost every infectious agent is capable of producing myocarditis. aneurysm formation in left ventricular apex and mural thrombi that may embolize to pulmonary and systemic circulations. All of the above C. drug hypersensitivity. Conduction abnormalities Myocarditis results from infections. Cardiomegaly & severe CHF in diphtheritic myocarditis appear after the first week of illness. Thymoma A. 1957 984 In HIV myocarditis. Infectious process A. 1958 A. Myopericarditis B. Ventricular tachyarrhythmias A. Toxoplasma gondii Harrison’s 17th Ed. Atrial fibrillation Harrison’s 17th Ed. Atrial fibrillation and ventricular tachyarrhythmias also occur. Left ventricular dysfunction D. fibrosis & thinning of ventricular wall. Dermatobia hominis C. Mother to fetus B. Giant cell myocarditis C. Lyme carditis Ventricular tachyarrhythmias dominate the presentation in viral myocarditis. All of the above D. 1486 disease ? Harrison’s 18th Ed. Conduction abnormalities from mother to fetus. cruzi can also occur through blood transfusion.182 MCQ’s FOR MEDICAL PROFESSIONALS BY PROF. Suramin D. Chagas disease D. 1958 985 Cardiomegaly & severe CHF in diphtheritic myocarditis appear after how many weeks of illness ? A. First C. Myocarditis in patients with HIV . 1487 B. sarcoidosis and Cardiac involvement in Chagas heart disease is characterized by dilatation of cardiac chambers. especially coxsackievirus B. clinically significant acute myocarditis in USA is caused by viruses. 975 Which of the following viruses most often produces clinically significant acute myocarditis ? 981 Which of the following drug is used in the treatment of Chagas’ Harrison’s 17th Ed. In HIV patients. Harrison’s 18th Ed. the most common finding is left ventricular dysfunction that in some cases 979 Which of the following is a typical feature of Chagas’ disease appears to be due to infiltration of myocardium by virus. Organ donation A. Left ventricular dysfunction C. 1957 983 On auscultation in a case of myocarditis. Coxsackievirus B C. Diphtheritic myocarditis B. 1959 A. AJAY MATHUR Cardiology C. radiation. Fourth node and atrioventricular (AV) node dysfunction and right bundle branch block. which of the following murmur is frequently heard ? A. Right ventricular dysfunction D. 1486 Harrison’s 18th Ed. Aortic regurgitation reduvid bug. Antiparasitic therapy for Chagas’ disease include benznidazole and nifurtimox. 1487 B. 976 Myocarditis can result from ? 982 Cardiac abnormality most common in Lyme disease is ? Harrison’s 17th Ed. Pentamidine B. Pericardial rub may be heard in patients with associated pericarditis. Nonspecific ST-T abnormalities C. organ donation. 1961 Harrison’s 18th Ed. chemicals or physical agents. Auscultatory findings in a case of myocarditis include muffled S1 with S3 & a murmur of mitral 978 Protozoan Trypanosoma cruzi can be transmitted by ? regurgitation. particularly sinus D. the most common finding is ? A. Trypanosoma cruzi A. Second D. D. Coxsackievirus A B. in ECG ? Harrison’s 18th Ed. C. 1410 D. Second heart syndrome” ? Harrison’s 17th Ed. atrial fibrillation is the most common arrhythmia.40 grams / day 991 Risk factor for peripartum cardiomyopathy is ? D. All of the above B. Pernicious anemia A.  1 . Tricyclic antidepressants is ? B. Harrison’s 18th Ed. malnutrition. > 30 years 996 What quantity of alcohol consumption has been shown to be cardioprotective ? C. of African ancestry B. Atrial fibrillation Cardiac dilatation & CHF of unexplained cause may develop during last trimester of pregnancy or within 6 months after delivery.30 grams/day) appears to be cardioprotective. Multiparous B. 3 ounces of pure ethanol daily for 5 . All of the above 987 Myocarditis may result from hypersensitivity to which of the Polymorphisms of the genes encoding alcohol dehydrogenase and the angiotensin-converting following drugs ? enzyme increase the likelihood of alcoholic cardiomyopathy. C. Ventricular tachycardia 990 Likelihood of peripartum cardiomyopathy is more in ? Harrison’s 17th Ed. All of the above B. 1961 C. Antipsychotics A. First 995 Which of the following arrhythmias is most frequent in “Holiday B. D. Methyldopa A. Beta-adrenergic blockers 992 Polymorphisms of which of the following genes increases the D. thiazides. pernicious anemia.adrenergic receptor Associated conditions of giant cell myocarditis are thymomas. C. and preeclampsia or toxemia B. Tobacco Myocarditis may result from hypersensitivity to antibiotics.10 years Myocarditis may result from hypersensitivity to drugs like tricyclic antidepressants. 1482 In holiday heart syndrome. Beta-adrenergic blockers should be avoided. 1961 A. Antibiotics Harrison’s 17th Ed. Preeclampsia or toxemia of pregnancy cocaine-induced cardiotoxicity ? Harrison’s 16th Ed. 20 . AJAY MATHUR Cardiology 183 C. followed by atrial flutter and ventricular premature depolarizations. Atrial flutter D. Angiotensin-converting enzyme D. Ventricular premature depolarizations D. twin pregnancy.10 years 988 Myocarditis may result from hypersensitivity to which of the Alcohol consumption necessary to produce cardiomyopathy in an otherwise normal heart is following drugs ? about 4 ounces of pure ethanol daily for 5 .30 grams / day and >30 years of age. Calcium channel blockers of pregnancy. antibiotics C. D. C. 1961 994 “Holiday heart syndrome” is the term used for cardiotoxicity A. All of the above A. 10 . D. C. anticonvulsants. 30 . Nitrates Risk factors for peripartum cardiomyopathy are increased maternal age. and methyldopa. All of the above A. . A. 1482 D. All of the above B. 2 ounces of pure ethanol daily for 5 . Of African ancestry Harrison’s 17th Ed. Harrison’s 17th Ed. Benzodiazepines likelihood of alcoholic cardiomyopathy ? Nitrates. Cocaine indomethacin. 4 ounces of pure ethanol daily for 5 . Amphetamine 989 Peripartum cardiomyopathy usually develops in which Holiday heart syndrome is a state of alcoholic cardiotoxicity presenting as recurrent trimester of pregnancy ? supraventricular or ventricular tachyarrhythmias without overt heart failure due to alcoholic binge drinking. Thiazides produced by ? B. increased parity.  2 . use of tocolytic therapy for premature labor. 1961 Alcohol consumption in moderate quantity (20 .20 grams / day Patient who develops peripartum cardiomyopathy typically is multiparous.adrenergic receptor other autoimmune diseases. Use of tocolytic therapy for premature labor 997 Which of the following drugs should be avoided while treating C. 1486 993 Alcohol consumption necessary to produce cardiomyopathy A. Antibiotics Harrison’s 18th Ed. Alcohol D. calcium channel blockers (to reduce coronary spasm) & benzodiazepines are used to Harrison’s 18th Ed. 40 . 183 Cardiology MCQ’s FOR MCQ’s FOR MEDICAL MEDICAL PROFESSIONALS PROFESSIONALS BY PROF. 1 ounce of pure ethanol daily for 5 . 1961 treat cocaine-induced cardiotoxicities.50 grams / day Harrison’s 18th Ed.10 years & antipsychotics.10 years. A. 1482 C. Harrison’s 18th Ed. 1482 C.10 years D. All of the above B. thyroiditis. Third A. Increased parity B. Cirrhosis . 1962 Harrison’s 17th Ed. Cyclophosphamide A.184 MCQ’s FOR MEDICAL PROFESSIONALS BY PROF. Reduced ECG voltage A. & warm extremities. Cyclophosphamide with tachycardia. Ifosfamide “Wet beri-beri heart disease” is characterized by high cardiac output failure state & is associated C. Selenomethionine failure with “preserved” ejection fraction. wide pulse pressure. Response to thiamine in “wet beri-beri heart disease” is dramatic & must be accompanied by diuretics. S3. Copper A. Selenothiamine C. Trastuzumab (Herceptin) D. Ifosfamide B. Harrison’s 17th Ed. Diabetes C. 1488 A. Selenium B. Kashin-Beck Disease (Big Bone disease) and Myxedematous Endemic Cretinism. 1001 Which of the following drugs can cause cardiotoxicity with 1007 Which of the following diseases is associated with selenium chronic use ? deficiency ? Harrison’s 18th Ed. Hyperthyroidism J Am College of Nutr 2001. Heart failure D. 1488 A. Dexrazoxone Pathogenetic factors in peripartum cardiomyopathy include low socioeconomic status. 419 A. Erythropoietin D. All of the above Specific diseases associated with selenium deficiency are Keshan Disease. 1962 A. Emetine D. Pulmonary hypertension A. prolonged lactation. and some other alkylating agents can cause recurrent coronary spasm. 5000 µg/day D. Hypothyroidism A. All of the above D. 1962 selenium ? A. Cyclophosphamide 1005 “Wet beri-beri heart disease” is characterized by all except ? Iron chelator dexrazoxone is cardioprotective and reduces risk of doxorubicin (Adriamycin) cardiotoxicity. Chloroquine C. Theophyllin C. Hydroxychloroquine B. Selenoleucine Diabetes is a typical factor. Cold extremities Harrison’s 18th Ed. Diuretics 5-Fluorouracil. Arrhythmias C. which recurrent coronary spasm ? of the following drugs should be accompanied ? Harrison’s 18th Ed. 1002 Heart failure with “preserved” ejection fraction is best applied to ? 1008 Which of the following is best absorbed & utilized form of Harrison’s 18th Ed. Selenoarginine D. All of the above diffuse T-wave abnormalities & prolongation of QT interval. and female gender. 50 µg/day B. cisplatin. AJAY MATHUR Cardiology 998 Which of the following is a cardioprotective agent when A. Recommended Dietary Allowances (RDA) for selenium for adults is 50-200 µg/day. 1004 Deficiency of which of the following plays a pathogenetic role 1010 Clinical syndrome of Hemochromatosis includes ? in peripartum cardiomyopathy ? Harrison’s 18th Ed. high C. 500 µg/day C. Zinc Harrison’s 17th Ed. 1003 Which of the following conditions is associated with a high 1009 Recommended Dietary Allowances (RDA) for selenium for risk of death in patients with viral myocarditis ? adults is ? Harrison’s 16th Ed. Quinidine D. Myxedematous Endemic Cretinism C. All of the above D. along with hypertension. excessive dietary salt intake & selenium deficiency. 50000 µg/day Patients with viral myocarditis & pulmonary hypertension are at a particularly high risk of death. D. D. Keshan Disease A. Amiodarone parity. ECG shows reduced voltage. 1488 A. Prolongation of QT interval B. Recovery of cardiac function occurs with aggressive use of ACE inhibitors. advanced age. Cisplatin C. Selenomethionine is considered to be the best absorbed and utilized form of selenium. Obesity B. 1962 C. 1413 Harrison’s 16th Ed. 1963 Harrison’s 17th Ed. Pericardial friction rub B. Digoxin B. in heart D. Gold doxorubicin (Adriamycin) is given to a patient ? B.20:1-4 B. Third heart sound 999 Which of the following drug causes dilated cardiomyopathy ? B. 1000 Which of the following chemotherapeutic agents can cause 1006 While treating “wet beri-beri heart disease” with thiamine. 1483 C. Kashin-Beck Disease B. None of the above Also known as apical ballooning syndrome. A. Changes are reversible in 3 . It Harrison’s 18th Ed. 1963 in left ventricle distal to papillary muscles. 1963 communicating with ventricular cavity. Strabismus cardiomyopathy ? Harrison’s 18th Ed. A. 1963 1020 Which of the following is false about Tako-Tsubo (stress) A. Restanin RV dysplasia is a familial cardiomyopathy marked by progressive replacement of right ventricular B. Hypothyroidism A. Japanese fishing net 1014 Which of the following best relates to genetic defect in the “Tako-tsubo” in Japanese language refers to a Japanese pot for fishing for octopus. 1964 B. It is associated with multiple genetic variants in sarcomeric and other proteins such as tafazzin.7 days without consequences. Desmosomes B. All of the above The three cardinal clinical features of left ventricular noncompaction are ventricular arrhythmias. A. AJAY MATHUR Cardiology 185 B. desmosomal protein ? Harrison’s 18th Ed. Nuclear membrane B. 2 layered structure of endomyocardium leading to LV A. Ventricular arrhythmias are common & sudden death is a constant threat. D. C. Japanese baloon D. creating a “spongy” appearance of apex. Woolly hair B. ST elevations &/or T-wave inversions in chest leads B. Mucopolysaccharides A. Coronary angiography shows no obstruction in epicardial arteries. Normal coronary angiography Mitochondrial myopathies show the characteristic “ragged red fiber” appearance. and heart failure. Adipose tissue Harrison’s 18th Ed. 185 Cardiology MCQ’s FOR MCQ’s FOR MEDICAL MEDICAL PROFESSIONALS PROFESSIONALS BY PROF. 1963 A. Gait disturbance A. Mitochondria Q J Med 2003. Mitochondrial myopathies Harrison’s 18th Ed. C. embolic events. thromboembolism or ventricular arrhythmias due to arrest of normal embryogenesis. None of the above D. Diabetes C. Multiple deep trabeculations into myocardium D. Troponins are mildly elevated. 1964 D. Hemochromatosis 1021 Which of the following is false about Tako-Tsubo (Stress) B. structures that maintain normal contacts between cells C. 96:563–573 A. Ventricular arrhythmias Severe akinesia of distal portion of left ventricle with EF reduction occurs. None of the above 1016 Cardinal clinical feature of left ventricular noncompaction is ? Harrison’s 18th Ed. diabetes & hypogonadism. Hypogonadism D. Embolic events . 1963 ECG in Tako-Tsubo (stress) cardiomyopathy shows ST-segment elevations and/or deep T-wave inversions in chest leads. 1963 Harrison’s 18th Ed. Endoplasmic reticulum 1019 What does “tako tsubo” in Japanese language mean ? C. May be due to iron overload as in hemolytic anemia A. Gibazin 1013 Which of the following is mostly affected in Arrhythmogenic D. Japanese pot for fishing octopus leading to detachment of myocytes with consequent myocyte apoptosis and fibrofatty replacement. there is progressive replacement Left ventricular noncompaction (LVNC) is a congenital cardiomyopathy that may present at any of right ventricular wall with ? age with symptoms of CHF. Congenital cardiomyopathy B. Transferrin saturation > 60 % for men C. 1963 occurs mostly in women >50 years. Some patients with mitochondrial myopathy have characteristic drooping eyelids. Iron deposited in perinuclear area of cardiomyocytes B. C. B. leading in to a distinct syndrome of “woolly hair” and thickened palms and soles. Elastic tissue contractile dysfunction. Duchenne’s and Becker’s dystrophy Cardiomyopathy ? C. Occurs mostly in women >50 years leading to replacement of myocardium by deposits of fat. Japanese parachute ARVC/D is an autosomal dominant disorder caused by multiple mutations of several genes encoding proteins that constitute desmosomes. Also known as apical ballooning syndrome C. D. 1017 Which of the following is false about Left Ventricular 1011 Which of the following is false about Hemochromatosis ? Noncompaction ? Harrison’s 18th Ed. ECG & Echo CG changes revert in 3-7 days D. with persistence of multiple deep trabeculations into myocardium Harrison’s 18th Ed. Frufin Right Ventricular Cardiomyopathy/Dysplasia (ARVC/D) Diagnostic criteria of left ventricular noncompaction includes presence of multiple trabeculations Harrison’s 18th Ed. B. 1963 D. Heart failure C. Severe chest discomfort preceded by stressful emotional or D. Tako-Tsubo (stress) cardiomyopathy is characterized 1015 Drooping eyelids is a feature of ? by abrupt & severe chest discomfort preceded by a stressful emotional or physical event. All of the above D. It is diagnosed echocardiographically. Tafazzin wall with adipose tissue. Abnormal embryogenesis C. Collagen tissue 1018 Left ventricular noncompaction is best related to ? C. Cyanosis physical event Genetic defects in proteins of the desmosomal complex disrupt myocyte junctions & adhesions. This same protein also affects hair & skin. The clinical syndrome of Hemochromatosis includes cirrhosis. None of the above 1012 In right ventricular dysplasia. hypereosinophilic disease. They range from barely visible to several millimeters in D. Harrison’s 18th Ed. C. Calcitonin Fabry’s disease results from a deficiency of the lysosomal enzyme alpha-galactosidase A. Hallmark of restrictive cardiomyopathies is abnormal diastolic function. Autonomic neuropathy A. endomyocardial Harrison’s 18th Ed. Mucopolysaccharidoses 1033 Which of the following about angiokeratomas in Fabry’s D. IVC . & small-vessel disease of kidney. autonomic neuropathy. 3191 A. 1964 B. C. All of the above D. Alpha-galactosidase A B. corneal & lenticular opacities. Beta-galactosidase 1025 Familial amyloidosis results from an autosomal dominant C. A. 3191 D. 1029 2D echocardiogram that shows a thickened myocardial wall Infiltrative diseases leading to secondary RCM may show impairment of systolic function. Hemochromatosis Myocardial involvement with amyloid is a common cause of secondary RCM. Do not blanch with pressure 1027 Amyloid fibrils infiltrate the myocardium. following mediastinal irradiation. Asymmetric C. especially around ? Harrison’s 18th Ed. All of the above C. Conduction block Harrison’s 17th Ed. B. Amyloidosis B. Leg lymphedema without hypoproteinemia 1026 Amyloidosis is best related to which of the following ? D. and scleroderma. 1965 is abnormal ? A. Ventricular walls are excessively rigid & impede ventricular filling due to myocardial fibrosis. AJAY MATHUR Cardiology Restrictive cardiomyopathy Amyloid fibrils infiltrate myocardium. hypohidrosis. Systolic + diastolic dysfunction Typical clinical features are conduction block. 1965 1023 Which of the following is a feature of restrictive cardiomyopathy ? A. hypertrophy or infiltration. B. End-diastolic pressures are elevated in both ventricles D. 1965 B. Gelsolin 1032 Which of the following about Fabry’s disease is false ? C. and in neoplastic infiltration. dark red to blue-black B. Refractile brightness in septum on echocardiography B. glycogen deposition. and D. Both atria are enlarged C. Entry of SVC. 1485 B. Clinically. a carrier B. 1964 D. Renal involvement B. Hypereosinophilic syndrome disease is false ? Congo red stain can be used to highlight amyloid. and usually symmetric. flat or slightly raised. Congo red stain heart & brain. especially around conduction system & coronary vessels. 1966. dark red to blue-black. Endomyocardial fibrosis Harrison’s 18th Ed. 1965 fibrosis. 1964 A. although restriction 1031 Fabry’s disease results from a deficiency of ? is also seen in the transplanted heart. Amyloid  protein A. Preserved cardiac output 2D echocardiogram that shows a thickened myocardial wall with a distinctive “speckled” D. Transthyretin Harrison’s 18th Ed. 1028 Typical feature of amyloidosis is ? 1022 The hallmark of the late secondary restrictive cardiomyopathies Harrison’s 18th Ed. Any of the above occasionally thickened skin lesions. renal involvement. Scleroderma D. Diastolic dysfunction D. All of the above appearance is suggestive of amyloidosis. with a distinctive “speckled” appearance is suggestive of ? Harrison’s 18th Ed. Eosinophilic endomyocardial disease A. 1024 Which of the following is the more common cause of secondary 1030 Which of the following go in favour of amyloidosis ? restrictive cardiomyopathy ? Harrison’s 18th Ed. Sphingomyelinase mutation in ? Harrison’s 18th Ed. Autosomal dominant disorder Familial amyloidosis results from an autosomal dominant mutation in transthyretin. Glycosphingolipid female carriers. 1965 Harrison’s 18th Ed. in hemochromatosis. Disorder of glycosphingolipid metabolism protein for thyroxine and retinol. Figure 238-12 Fabry’s disease is an X-linked recessive disorder that may also cause clinical disease in A. Both atria are dilated C. Sarcoidosis C. Cardiomyocyte C. acroparesthesia. Systolic dysfunction C. Mostly between umbilicus & knees “bathing suit area” A. Primary cardiac amyloidosis B. Coronary vessels D. it manifests with angiokeratomas (telangiectatic skin lesions). All of the above C. Lymphatics Angiokeratomas are punctate. Punctate. sarcoidosis. Low voltage ECG A. Enzyme therapy useful Harrison’s 18th Ed. Neuraminidase A.186 MCQ’s FOR MEDICAL PROFESSIONALS BY PROF. they do not blanch with pressure. Phentermine 1034 Cardiac Danon’s Disease is due to mutations in ? Harrison’s 18th Ed. Outflow portion of right or left atria (or both) Echocardiographic studies have confirmed that by age 20 years. 1412 C. Harrison’s 17th Ed. 1968 or left ventricle (or both) & often involves atrioventricular valves. 187 Cardiology MCQ’s FOR MCQ’s FOR MEDICAL MEDICAL PROFESSIONALS PROFESSIONALS BY PROF. Chromosome 14 A. Elderly D. 10 years involve which of the following areas of heart ? C. Cardiac myosin light chain gene Harrison’s 17th Ed. Others involve alpha-myosin heavy chains.the “bathing suit area”. B. Deficiencies of LAMP2 and protein kinase. LAMP1 Carcinoid syndrome. Systolic dysfunction Harrison’s 18th Ed. most common mutations occur in which of the muscle. MVP 1039 Endocardial fibrosis results from which of the following ? B. and T. 20 years Harrison’s 17th Ed. Children and young adults D. Cardiac alpha-myosin heavy chain gene 1036 Differentiation of constrictive pericarditis from restrictive B. A. most common mutations of cardiac beta-myosin heavy chain gene is on chromosome 14. Middle aged In HCM. Chromosome 12 Harrison’s 16th Ed. 1486 C. D. 30 years A. 1486 D. Carcinoid syndrome between umbilicus & knees . 5 years 1038 Fibrous endocardial lesions in endomyocardial fibrosis B. cardiac troponins C. particularly ventricular preexcitation and following genes ? conduction defects distinguish them from patients with hypertrophic cardiomyopathy resulting Harrison’s 18th Ed. They are most dense A. B. Systolic + diastolic dysfunction B. I. Outflow portion of right or left ventricle (or both) 1044 Which of the following is the most common cause of SCD in young competitive athletes ? Endomyocardial fibrosis is characterized by fibrous endocardial lesions of inflow portion of right Harrison’s 18th Ed. 1484 A. None of the above C. most common mutations of cardiac beta-myosin heavy Apex impulse is easily palpable & mitral regurgitation is more common in RCM. C. Diastolic dysfunction A. 1967 cardiomyopathies with LVH shows ventricular preexcitation ? A. Their electrophysiologic abnormalities. Inflow portion of right or left atria (or both) B. 1966 D. 1968 from defects in sarcomere-protein genes. Fenfluramine C. Apex impulse B. Cardiac beta-myosin heavy chain gene cardiomyopathies is done by which of the following ? C. most common are mutations of the cardiac beta-myosin heavy chain gene. characterized by D. Any of the above 1043 At what age. C. Pericardial chain gene is on ? calcification on Chest x-ray/CT/MRI occurs commonly in constrictive pericarditis. adenosine monophosphate (AMP)-activated gamma 2 noncatalytic subunit (PRKAG2) result in the accumulation of glycogen in heart and skeletal 1041 In HCM. Fabry Disease C. Mitral regurgitation In HCM. A. All of the above 1042 In HCM. LAMP4 Hypertrophic cardiomyopathy (HCM) Cardiac Danon Disease is caused by mutations in an X-linked lysosome-associated membrane protein (LAMP2). Friedreich’s Ataxia D. ECG in Cardiac Danon Disease shows severe LV hypertrophy & ventricular preexcitation. LAMP3 D. use of anorexic agents like fenfluramine & phentermine result in endocardial fibrosis & stenosis and/or regurgitation of the tricuspid and/or pulmonary valve. producing valvular regurgitation. and T gene A. Glycogen Storage Cardiomyopathy increased stiffness of hypertrophied muscle. I. LAMP2 C. Chromosome 10 1037 Endomyocardial fibrosis most commonly affects which of the following age groups ? B. 1966 B. myosin light chains & titin. AJAY MATHUR Cardiology 187 diameter and have a tendency to increase in size and number with age. Angiokeratomas also occur in Fordyce scrotal angiokeratoma and several other very rare lysosomal storage diseases. ubiquitous pathophysiologic abnormality is ? 1035 ECG of which of the following inherited metabolic Harrison’s 18th Ed. cardiac myosin-binding C. This results in elevated diastolic filling pressures & is present despite of a hyperdynamic left ventricle. full genetic expression occurs in first-degree relatives of patients with familial HCM ? Endomyocardial fibrosis is a progressive disease of unknown cause that occurs mostly in Harrison’s 17th Ed. HCM . particularly Uganda & Nigeria. Cardiac troponins C. 1484 children & young adults residing in tropical & subtropical Africa. All of the above A. full genetic expression occurs in about half of the first-degree relatives of patients with familial HCM. Chromosome 16 B. Cardiac Danon Disease Ubiquitous pathophysiologic abnormality in HCM is diastolic dysfunction. A. actin. Long QT syndrome Harrison’s 17th Ed. 1040 In HCM. Pericardial calcification protein C. Inflow portion of right or left ventricle (or both) D. 1487 D. Small Familial clustering due to Mendelian transmission of single-gene defects may occur in B. Tachycardia Echocardiographic studies have confirmed that about one-third of the first-degree relatives of patients with familial HCM have evidence of disease. AJAY MATHUR Cardiology D. HCM echocardiogram. Symptoms are not closely related to the presence or severity of an outflow pressure gradient. Nitroglycerin In symptomatic HCM patients. 1968 C. Interventions that increase myocardial contractility like exercise. abnormal blood pressure response C. One-half Harrison’s 18th Ed. angina pectoris & fatigue. D. Pulmonary area 1045 Which of the following is the most common cause of sudden death among young athletes ? D. most common complaint is dyspnea. There is no correlation between the risk of sudden death and the severity of symptoms.1 times to exercise & a family history of sudden death. sustained Harrison’s 16th Ed. 1968 . the most common complaint is ? Harrison’s 18th Ed. Essential hypertension or coronary atherosclerosis are often polygenic disorders. Murmur is best heard at lower left A. 0. interventricular septum is how B. Syncope B. Any of the above 1049 Risk of sudden death in HCM correlates with all except ? Left ventricular cavity typically is small in HCM. 1. Passive leg raising 1047 In symptomatic HCM. 1970 reduced septal excursion. marked ventricular hypertrophy (ventricular septal thickness >30 mm). may all cause an increase in the gradient & the murmur. 1968 D. syncope (in children). C. 1050 Systolic murmur in obstructive HCM is best heard at ? Harrison’s 18th Ed. A1 area HCM is the most common cause of SCD in young competitive athletes. nitroglycerin. Conversely. sudden 1048 Mendelian transmission of single-gene defects may occur in ? standing. Passive leg raising pulmonary capillary pressures. MS HCM have evidence of the disease ? Harrison’s 16th Ed. Tetralogy of Fallot A. 0. genetic mutations associated with an increased risk. Dyspnea include all except ? B. Ventricular tachycardia on ambulatory monitoring 1055 In echocardiography of HCM. Sustained handgrip C. A. 1968 D. Normal hypertrophic cardiomyopathy. One-fourth 1052 Maneuvers that increase the murmur of obstructive HCM B. where it is often more holosystolic & blowing in quality due B. and those that reduce ventricular volume like Valsalva maneuver. elevation of arterial pressure by phenylephrine. 1968 Hypertrophic cardiomyopathy is the most common cause of sudden death among young athletes.8 times testing. due to diastolic ventricular dysfunction which impairs ventricular filling & leads to elevated LV diastolic. AR 1046 What proportion of first-degree relatives of patients with familial C.188 MCQ’s FOR MEDICAL PROFESSIONALS BY PROF. with septum 1. Anxiety Harrison’s 18th Ed. with vigorous posterior wall motion but Harrison’s 18th Ed. augmentation of venous return by passive leg raising & expansion of the blood A. Squatting D. Thyrotoxicosis 1051 Which of the following accompanies obstructive HCM ? D. Severity of symptoms Harrison’s 17th Ed. demonstrates LVH. A2 area C. Marfan syndrome & sudden death associated with a prolonged C. Mitral valve prolapse syndrome sternal border as well as at apex. Angina pectoris A. 1. since ejection is unimpeded early in systole. B. Hallmark of obstructive HCM is a systolic murmur which is typically harsh. Hypertrophic cardiomyopathy to mitral regurgitation that usually accompanies obstructive HCM. One-third include all except ? C. Large QT syndrome. amyl nitrite or tachycardia. ventricular tachycardia on ambulatory monitoring or at electrophysiologic B. A. Lower left sternal border N Engl J Med 2003. sympathomimetic amines & digitalis glycosides. Marked ventricular septal hypertrophy (>30 mm) many times thicker than high posterior left ventricular free C. squatting. 1302 handgrip. Other symptoms include syncope. All of the above A. MR A. Three-fourth A.3 times with outflow gradients. Long QT syndrome Harrison’s 18th Ed. 1411 D. Marfan syndrome 1054 Left ventricular cavity in HCM is typically ? C.349:1064-75. Sudden standing D. 1484 HCM patients at higher risk of sudden death are those with a history of resuscitation from A. Valsalva maneuver B. Abnormal blood pressure response to exercise wall ? D. B. Hypertrophic cardiomyopathy volume all increase ventricular volume & ameliorate the gradient & murmur. but there is an increased risk of death in patients D.5 times sudden cardiac death. C. diamond-shaped & begins after S1. AS B. left atrial & D. 1968 1053 Maneuvers that decrease the murmur of obstructive HCM A.3 or more times the thickness of high posterior left ventricular free wall. Palpitation Harrison’s 18th Ed. Pericardial Disease should be avoided. HCM with AR A. Digitalis Harrison’s 18th Ed. increase exercise tolerance & reduces severity of outflow tract pressure gradients. . Visceral pericardium is a serous membrane that is separated by a small quantity (15 to 50 mL) of fluid. from the fibrous Digitalis. None of the above N Engl M Med 2004. Digitalis B. AJAY MATHUR Cardiology 189 1056 Which of the following investigation is best for accurate C. All of the above B.350:1320-7 HCM is found in ~1 in 500 of the general population. neoplastic & uremic pericarditis. Myocardial perfusion defects on thallium 201 radionuclide scintigraphy frequent 1057 Complications of HCM include ? D. Disopyramide In partial left pericardial defects. Pulmonary vein D. Nondihydropyridine calcium blockers D. Left ventricle C. Systolic anterior motion of mitral valve is characteristic CMRI is superior to echocardiography in providing accurate measurements of regional hypertrophy and in identifying sites of regional fibrosis. 1971 N Engl M Med 2004. Digitalis C. 1411 A. CMRI A. 1484-5 C. HCM progressing to DCM with disappearance of preexisting avoided in cases of obstructive HCM ? outflow pressure gradient Harrison’s 16th Ed. particularly in patients with known parietal pericardium. Disopyramide measurements of regional hypertrophy and in identifying sites D. 1970 Propensity for sudden death in HCM is genetic. D. an ultrafiltrate of plasma. All of the above Harrison’s 18th Ed. Left atrium B. C. All of the above Verapamil & diltiazem reduces stiffness of ventricle. HCM not manifesting in offsprings infective endocarditis & end-stage heart failure. Nitrates D. A. diuretics. 1064 Pericardial cavity normally contains how much fluid ? 1059 Which of the following drugs should be avoided in cases of Harrison’s 18th Ed. 1411 D. Dynamic LV subaortic outflow tract A. 1969 C. 5 to 10 ml of fluid A. Verapamil Progression of HCM to left ventricular dilatation & dysfunction (DCM) with wall thinning & B. 1065 In partial left pericardial defects.350:1320-7 A. vasodilators are best avoided. Nifedipine Chapter 239. 10 to 20 ml of fluid B. B. Uremic pericarditis A. Diltiazem disappearance of a preexisting outflow pressure gradient is called burnt out HCM. Diuretics C. A. It occurs in C. Other complications include atrial fibrillation. Neoplastic pericarditis Harrison’s 18th Ed. All of the above Normal pericardium is a double-layered sac. HCM disappearing with advancing age 1058 Which of the following calcium channel blockers should be C. left ventricular outflow tract pressure gradients. postirradiation. C. Sudden death 1063 What is meant by “burnt out HCM” ? D. Postirradiation pericarditis 1061 Which of the following drug is protective against SCD in HCM ? B. LV Angiography B. A. reduces elevated diastolic pressures. Radionuclide scintigraphy with thallium 201 frequently B. Beta blockers block the effects of catecholamines that exacerbate outflow tract obstruction & to slow the heart rate so that diastolic filling is enhanced. Amiodarone D. 15 to 50 ml of fluid C. 1971 obstructive HCM ? Harrison’s 16th Ed. Found in ~1 in 500 of general population D.10% of patients & may be associated with nonresponsive CHF requiring cardiac transplantation. Beta-blockers B. 189 Cardiology MCQ’s FOR MCQ’s FOR MEDICAL MEDICAL PROFESSIONALS PROFESSIONALS BY PROF. 1484 Amiodarone is effective in reducing the frequency of supraventricular as well as life-threatening ventricular arrhythmias and anecdotal data suggest that it may reduce the risk of SCD. nitrates. which of the following may 1060 Which out of the following drugs should not be used in a case bulge through the defect ? of obstrutive HCM ? Harrison’s 18th Ed. 50 to 100 ml of fluid D. main pulmonary artery & left atrium may bulge through the defect. 1971 increases left ventricular outflow tract pressure gradients by increasing myocardial contractility. Disopyramide exerts negative 1066 Chest pain is often absent in which of the following ? inotropic effects that decreases the outflow gradient and thereby improve symptoms. Diltiazem of regional fibrosis ? Harrison’s 17th Ed. Echocardiography 1062 Which of the following is false about HCM ? Harrison’s 17th Ed. Infective endocarditis shows myocardial perfusion defects even in asymptomatic patients. Nifedipine 5 . ECG B. Beta-adrenergic blockers Chest pain is absent in slowly developing TB. Aorta A. Atrial fibrillation pressure gradient due to midsystolic proximity of anterior mitral valve leaflet against hypertrophied septum is typical of obstructive HCM. Bamberger-Pins-Ewart Sign or Pins’ Syndrome. Also known as at end-expiration with the patient upright & leaning forward throughout the respiratory cycle. this ratio is higher in acute pericarditis. ST segments return to normal and Harrison’s 18th Ed.retraction of thoracic wall. or left shoulder. idiopathic pericarditis & renal failure. 1971 N Engl J Med 2004. Ewart’s Sign B. 1077 In cardiac tamponade. 1971 A. Stage III D. Tear drop B. arms. N Engl J Med 2004.16 B. Auenbrugger’s sign .percussion dullness in fifth intercostal space on the right. an A. involving two or three standard limb leads and V 2 to V 6. T waves are usually tall and the ST/T ratio is <0. Exacerbated by inspiration B. Beck’s triad consists of all except ? segment depression with no significant changes in QRS. Pulsus paradoxus A. AJAY MATHUR Cardiology 1067 Which of the following statements about chest pain of acute 1073 The most reliable ECG distinguishing feature between acute pericarditis is false ? pericarditis and acute myocardial infarction is ? Harrison’s 18th Ed. 1971 C.351:2195-202 sudden collapse of the previously distended veins of neck at each diastole. 100% of patients indrawing of the intercostal spaces. 75% of patients 1075 Which of the following is the name given to the configuration C. In stage 2. Stage II Chest roentgenogram in pericardial effusion may show a “water bottle” configuration of the C. 1971 A. aVL an absent “y” descent C. as well as PR. Ratio of ST segment elevation to T-wave height in V6 of > 0. All of the above sounds. Right sternal border D. Water bottle Harrison’s 18th Ed. Hypotension 1072 In acute pericarditis. 1971 A.351:2195-202. All of the above Patch of dullness on auscultation beneath angle of left scapula due to compressive atelectasis of Pericardial friction rub is high-pitched. Soft or absent heart sounds Harrison’s 18th Ed. Pericardial pain is relieved by sitting up & leaning forward & is intensified by lying supine. Stage IV 3 commonest causes of tamponade are neoplastic disease. soft or absent heart D. aVR D. Broadbent’s sign . Harrison’s 18th Ed. Ebstein’s sign - 1069 “Triphasic” pericardial friction rub is heard in about ? obtuseness of the cardiohepatic angle on percussion. aVF Three main features of cardiac tamponade (Beck’s triad) are hypotension. 1971 A. ECG in stage 1 of acute pericarditis shows widespread elevation of ST segments. Neoplastic disease A. Stage I D. Friedreich’s sign . 1074 Patch of dullness on auscultation beneath angle of left scapula in pericardial effusion is called ? 1068 Pericardial friction rub is best heard at ? Harrison’s 18th Ed. Apex B. Pain of acute pericarditis is often severe. Synonym: Kreysig’s sign. Ratio of ST segment elevation to T-wave height in V6 of > 0. synchronous with cardiac systole.20 C. Weeks or months later. elicited sometimes only at the left lower sternal border left lung base by pericardial fluid is called Ewart’s Sign (William Ewart. and jugular venous distention with a prominent “x” descent but an absent “y” descent.190 MCQ’s FOR MEDICAL PROFESSIONALS BY PROF. Renal failure D. Stage III cardiac silhouette but may be normal. A. ECG in acute pericarditis evolves through 4 stages. Hour glass 1070 PR-segment depression is seen in which stage of acute pericarditis ? B. In stage 1. Stage IV 1076 Which out of the following is the less common cause of cardiac tamponade ? 1071 Widespread T-wave inversions are found in which stage of acute Harrison’s 18th Ed. aggravated by inspiration. Broadbent’s sign C. 25% of patients Harrison’s 18th Ed. UK). 1971 C. Dust bin A. Retrosternal and left precordial A. B. Pain may radiate to one or both trapezius muscle ridges In early repolarization.In adherent pericardium. D. there is widespread elevation of ST segments with upward concavity. involving two or three standard limb leads and V2 to V6. as well as depression of the PR segment below the TP segment cardiac tamponade ? reflecting atrial involvement without significant changes in QRS complexes. 1972 then T waves become inverted (stage 3). Idiopathic pericarditis C. synchronous with cardiac systole. Ratio of ST segment elevation to T-wave height in V6 of > 0. 1972 . 50% of patients of the cardiac silhouette in pericardial effusion ? D. Worse when the patient sits upright and leans forward and C. with reciprocal depressions only in aVR and sometimes V 1 . ECG returns to normal in stage 4. 1972 pericarditis ? Harrison’s 18th Ed. Harrison’s 18th Ed. retrosternal & left precordial.25. Rotch’s sign . Tuberculosis B. coughing & changes in body position. visible in left posterior axillary line is a sign of adherent pericardium. Jugular venous distention with a prominent “x” descent but B. Stage I B. Ratio of ST segment elevation to T-wave height in V6 of > 0. 1971 Harrison’s 18th Ed. referred to neck. Ebstein’s sign D.28 D. reciprocal depression is seen in ? B.epigastric prominence seen in marked pericardial effusion. Stage II C. with upward 1078 Presence of which of the following should raise suspicion of concavity. Left lower sternal border C.24 improves in supine position D. Heim-Kreysig sign . Auenbrugger’s sign A.In adherent pericardium. Often pain is pleuritic. Right ventricular myocardial infarction (RVMI) decline in systolic arterial pressure. a pericardial knock. Restrictive cardiomyopathy C. > 40 mm Hg with inspiration C. y descent. Restrictive cardiomyopathy Paradoxical pulse occurs not only in cardiac tamponade but also in approximately one-third of patients with constrictive pericarditis. 191 Cardiology MCQ’s FOR MCQ’s FOR MEDICAL MEDICAL PROFESSIONALS PROFESSIONALS BY PROF. 1087 Pulsus paradoxus is most common in ? tricuspid stenosis. Mitral valve 1084 Prominent “x” descent is rare in ? D. 1975. When jugular veins are distended and venous pressure fails to decline during inspiration is called Kussmaul’s sign. 1975. Pulmonary embolus D. Constrictive pericarditis A. Constrictive pericarditis A. Doppler ultrasound shows that tricuspid & pulmonic valve flow velocities A. In constrictive pericarditis. occurring at the cardiac apex 0. Cardiac tamponade C. > 10 mm Hg with inspiration Harrison’s 18th Ed. Aortic valve Harrison’s 18th Ed. Right ventricular myocardial infarction (RVMI) 1082 On echocardiography. AJAY MATHUR Cardiology 191 A. Cardiac tamponade after aortic valve closure. C. right & left atrial pressure pulses display an M-shaped contour. Restrictive cardiomyopathy B. electrical alternans may be present in ? D. RVMI Harrison’s 18th Ed. 1972 Harrison’s 18th Ed. > 20 mm Hg with inspiration A. Constrictive pericarditis cessation of ventricular filling. B. Harrison’s 18th Ed. following condition ? Harrison’s 18th Ed. Viral or unknown (idiopathic) In constrictive pericarditis. with prominent x and y descents. is often conspicuous in constrictive pericarditis. which is absent or diminished in cardiac tamponade. 1975. Electrical alternans of QRS or T waves B. y descent is interrupted by a rapid rise in atrial pressure during early diastole. 1973 B. RVMI N Engl J Med 2004. Cardiac tamponade Harrison’s 18th Ed. Electrical alternans of P wave A. 1972 1081 Right ventricular size is small in which of the following ? A. Restrictive cardiomyopathy 1090 Most common cause of acute pericarditis is ? D. Restrictive cardiomyopathy B. Pulmonic valve C. D. Constrictive pericarditis D. Acute and chronic obstructive airways disease C. and restrictive cardiomyopathy. Table 239-2 B. whereas pulmonic vein. RVMI pressure of ? Harrison’s 18th Ed. Table 239-2 1089 Doppler ultrasound in cardiac tamponade shows marked A.. Cardiac tamponade A. > 30 mm Hg with inspiration B.e. Restrictive cardiomyopathy 1079 In ECG. RVMI C. 1972 1080 Kussmaul’s sign is absent in which of the following ? A. Table 239-2 B. Table 239-2 B. It reflects rapid early filling of the ventricles. Table 239-2 In cardiac tamponade. right ventricular infarction. is B. Cardiac tamponade increase in flow velocities during inspiration across ? Harrison’s 18th Ed. All of the above C. All of the above C. 1975. right atrial collapse and right ventricular 1088 Pulsus paradoxus is observed in which of the following diastolic collapse (RVDC) is present in which of the following ? condition ? Harrison’s 18th Ed. It is absent in cardiac tamponade and is seen in chronic pericarditis. Hypovolemic shock B. It is due to an abrupt B. Constrictive pericarditis B. All of the above Pulsus paradoxus is not pathognomonic of pericardial disease as it may be observed in some 1083 Prominent “y” descent is usually present in which of the cases of hypovolemic shock. 1975. Tuberculosis . when C.351:2195-202 A.12 s A. mitral & aortic flow velocities diminish. Right ventricular myocardial infarction (RVMI) D. 1975. Table 239-2 A. Cardiac tamponade C. Constrictive pericarditis D. Pulmonic vein D. Table 239-2 An early third heart sound i. Trauma to the chest the most prominent deflection in constrictive pericarditis. ventricular pressure pulses in both ventricles exhibit characteristic “square root” signs during diastole. Neoplastic invasion of the pericardium ventricular filling is impeded by constricting pericardium. Restrictive cardiomyopathy The presence of cardiac tamponade consists of a greater than normal (10 mmHg) inspiratory D.09–0. Cardiac tamponade C. Constrictive pericarditis C. Reduction in amplitude of QRS complexes 1085 Pericardial knock is often present in ? Harrison’s 18th Ed. Restrictive cardiomyopathy 1086 Pulsus paradoxus is defined as a decrease in systolic arterial D. Cardiac tamponade increase markedly during inspiration. acute & chronic obstructive airways disease & pulmonary embolus. D. Constrictive pericarditis D. 1975. 1100 Drugs implicated in the causation of pericarditis include ? The y descent is interrupted by a rapid rise in atrial pressure during early diastole.351:2195-202 C.351:2195-202 D. Systemic arterial hypotension B. B. colchicine is used. Dantrolene D. All of the above In constrictive pericarditis. Minoxidil A. M .351:2195-202 D. Elevated cholesterol N Engl J Med 2004. Pericardial constriction N Engl J Med 2004.192 MCQ’s FOR MEDICAL PROFESSIONALS BY PROF. A. Cardiac tamponade D.shape B. C. Elevated jugular venous pressure Harrison’s 16th Ed. > 250 ml 1104 Interferon alpha is reported to be beneficial in pericarditis caused by ? C. A. Colchicine may prevent recurrences.351:2195-202 In chronic constrictive pericarditis. when Harrison’s 16th Ed. > 450 ml A. All of the above A. 1976 1099 Drugs implicated in the causation of pericarditis include ? A. 20 to 25 percent 1094 The possible sequelae of pericarditis include ? C. Recurrent pericarditis 1102 A large pericardial effusion is diagnosed if the echo-free space on 2D echocardiography is ? C. Phenytoin D. Pulsus paradoxus A. Adenovirus A. > 350 ml Harrison’s 16th Ed. > 10 mm 1095 The presence of which of the following is most sensitive B. AJAY MATHUR Cardiology 1091 Which of the following drugs is not indicated in treatment of B. Restrictive cardiomyopathy 1101 Plasma troponin concentrations are elevated in what percent of patients with pericarditis ? D. Ibuprofen B. RVMI N Engl J Med 2004. Isoniazid 1093 Broadbent’s sign is a feature of ? B. Parvovirus . 1977 C. Methysergide If treatment with aspirin is ineffective. Cytomegalovirus 1097 Drugs implicated in the causation of pericarditis include ? B. Colchicine 1098 Drugs implicated in the causation of pericarditis include ? N Engl J Med 2004.shape N Engl J Med 2004. > 30 mm N Engl J Med 2004. All of the above A. Exudate 1096 The appearance of cardiomegaly on chest radiography B. A . All of the above B. > 150 ml B. Procainamide D. the apical pulse is reduced and may retract in systole . Doxorubicin idiopathic acute pericarditis ? C.351:2195-202 B.shape C. All of the above A. 1492 D. 35 to 50 percent N Engl J Med 2004. Phenylbutazone C. right & left atrial pressure pulses show M-shaped contour. Isoniazid D. pericardial fluid has which of the following features ? C. Phenylbutazone B. > 40 mm A. 1492 D. The y descent. 50 to 60 percent A. Tachycardia 1103 In rheumatoid arthritis. with prominent x and y descents. reflects rapid early filling of ventricles (absent or diminished in cardiac tamponade) is the most prominent deflection in constrictive pericarditis. 1493 ventricular filling is impeded by the constricting pericardium. 10 to 15 percent Broadbent’s sign. Coxsackie B N Engl J Med 2004. Decreased concentrations of complement and glucose indicates a pericardial effusion of ? C.351:2195-202 D. Hydralazine Harrison’s 16th Ed.351:2195-202 C. 1424 D. Prednisone A. W . Cromolyn Harrison’s 18th Ed.351:2195-202 D. Cardiac tamponade B. right & left atrial pressure pulses D. V . Pergolide 1092 In constrictive pericarditis. > 20 mm indicator of cardiac tamponade ? C. Constrictive pericarditis C.shape A. All of the above display a contour of the shape of ? Harrison’s 18th Ed. 1979 D. Lipoma D. Lipoma Harrison’s 18th Ed. Rhabdomyomas are strongly associated with tuberous sclerosis. Carcinoma breast B. A. Atrial myxoma Relative incidence of cardiac metastases is high in malignant melanoma. Melanoma 1116 In cardiac metastases. Pericardium B. 1979 A. and/or breast). Carcinoma thyroid 1109 LAMB syndrome includes all except ? C. Blue nevi Harrison’s 18th Ed. A. AJAY MATHUR Cardiology 193 1105 Hyperimmune globulin is reported to be beneficial in D. adenovirus & parvovirus pericarditis. C. 1981 NAME syndrome consists of nevi. Spotty skin pigmentation Hemangiomas and mesotheliomas most often are intramyocardial in location. C. Cardiac metastases pericarditis caused by ? Papillary fibroelastomas are the most common tumors of the cardiac valves. and/or pituitary adenomas with gigantism or acromegaly). 1980 Myxomas are the most common type of primary cardiac tumor in all age groups. Rhabdomyoma 1107 Carney syndrome is characterized by all except ? C. which of the following is least involved ? D. Atrial myxoma Most common primary originating sites of cardiac metastases are carcinoma of breast & lung. Ephelides Harrison’s 18th Ed. and may cause B. Carcinoma testes B. with a female predilection. Adenovirus heart in adults ? Harrison’s 18th Ed. Hemangioma Harrison’s 18th Ed. B. atrial myxoma. 1980 A. most commonly A. testicular tumors. Lymphoma 1108 NAME syndrome includes all except ? C. A. Myxoma D. pericardium is most often involved. Leukemia B. skin. Myxomas atrioventricular (AV) conduction disturbances. C. Carcinoma pancreas Harrison’s 18th Ed. Myxoma in third to sixth decade. Myocytic hamartoma Hyperimmune globulin is beneficial in cytomegalovirus. Cataract 1114 Relative incidence of cardiac metastases is highest in which D. Endocardium Harrison’s 18th Ed. Tumors and Trauma of the heart in children ? Heart Harrison’s 18th Ed. Hemangioma A. All of the above B. Cytomegalovirus 1111 Which of the following is the most common primary tumor of B. followed by myocardial and rarely by involvement of C. 1981 nevi and endocrine overactivity (primary nodular adrenal cortical disease with or without Cushing’s syndrome. while interferon alpha has been reported to be so in coxsackie B pericarditis. Sarcoma endocardium or cardiac valves. C. Myxoma Rhabdomyomas and fibromas are the most common cardiac tumors in infants and children and B. . C. Pituitary adenomas of the following malignancies ? Carney complex comprises of myxomas (cardiac. 1492 A. Nevi D. Lipoma D. Hemangioma A. Lentigines D. atrial myxoma and blue nevi. Fibroma 1113 Which of the following heart tumour is intramyocardial in location ? D. 1981 LAMB syndrome consists of lentigines. Myocardium 1110 Most common tumor of the cardiac valves is ? C. 1979 D. Rhabdomyoma heart in adults ? C. Parvovirus A. 193 Cardiology MCQ’s FOR MCQ’s FOR MEDICAL MEDICAL PROFESSIONALS PROFESSIONALS BY PROF. Rhabdomyoma usually occur in ventricles. lentigines and/or pigmented Harrison’s 18th Ed. Harrison’s 16th Ed. myxoid neurofibroma and ephelides. 1980 A. None of the above B. Malignant melanoma Harrison’s 18th Ed. Papillary fibroelastoma Cardiac metastases occur via hematogenous or lymphangitic spread or by direct tumor invasion. Myxoedema 1115 Most common primary originating site of cardiac metastases is ? D. All of the above B. Myxoma 1106 Which of the following is the most common primary tumor of B. 1980 C. Lipoma Harrison’s 18th Ed. Inflammatory psuedotumor 1112 Which of the following is the most common primary tumor of Chapter 240. 1979 A. Prompt defibrillation saves life. 1983 D. Impact to chest wall overlying heart B. Kinking of coronary artery D. Right ventricular outflow tract 1124 Which of the following slows the egress of lipid-rich particles from the arterial wall intima ? 1119 What is meant by “commotio cordis” ? Harrison’s 18th Ed. and play a pathogenic role in atherogenesis. This links oxidative stress with impaired reverse cholesterol transport as a likely mechanism of the antiatherogenic action of HDL. 1121 Which of the following is the most common vascular 1126 Which of the following reactions modifies high-density deceleration injury ? lipoprotein (HDL) particles to become poor cholesterol Harrison’s 18th Ed. is the most common vascular deceleration injury with clinical presentation similar to that of aortic dissection. 1983 Harrison’s 18th Ed. Necrotic core 1118 Characteristic endocardial lesions of SLE (Libman & Sacks) B. 1424 D. and aldehydic breakdown products of fatty elevated cardiac enzymes without significant epicardial coronary artery disease. Carotid bifurcation A. All of the above Rupture of aorta. Carnitine biosynthesis D. Oxysterols Tako-Tsubo syndrome or apical ballooning syndrome is due to sudden emotional or physiologic D. 1127 Hypochlorous acid is produced in extracellular space of the Prevention.194 MCQ’s FOR MEDICAL PROFESSIONALS BY PROF. Hydroperoxides C. it presents with chest pain. 1983 A. oxysterols. 1981 C. All of the above D. Proximal portions of the renal arteries Harrison’s 18th Ed. Chapter 241. 1983 Cardiac MRI plays a central role in diagnostic evaluation of cardiac metastases & cardiac tumors. nonpenetrating injuries to chest that trigger ventricular fibrillation is referred to as matrix. favorable & complete & spontaneous resolution of ventricular dysfunction occurs within weeks. Rupture of the aorta D. Hypochlorous acid mediated chlorination C. Lysophospholipids D. Angiography atherosclerosis ? Harrison’s 18th Ed. usually just above aortic valve or at the site of ligamentum arteriosum. Prognosis is acids and phospholipids. “fatty streak” represents the initial lesion of atherosclerosis. Flavonoids B. lysophospholipids. 1982 acceptors ? Harrison’s 18th Ed. These B. CT C. 1983 Atherosclerosis A. Hyperdynamic function at ventricular base A. Myeloperoxidase . Monoamine oxidase following locations ? Harrison’s 18th Ed. Succinic acid dehydrogenase 1122 Stenoses due to atherosclerosis occurs focally in which of the C. Fatty streak are most often located at ? C. Left ventricular outflow tract D. Anterior ST-segment elevation B. All of the above trauma leading to dysfunction of mid-portion & apex of LV with hyperdynamic function at ventricular Oxidative modification of lipoproteins in the extracellular space of the intima give rise to base. Glutathione peroxidase B. Lipoproteins when associated with glycosaminoglycans of arterial extracellular matrix lead commotio cordis. AJAY MATHUR Cardiology 1117 Which of the following investigation is most useful in diagnostic A. 1120 Which of the following is false about Tako-Tsubo syndrome ? 1125 Oxidative modification of lipoproteins in the extracellular Harrison’s 18th Ed. Pyruvate -ketoglutarate B. 1983 B. anterior ST-segment elevation & mildly hydroperoxides. The Pathogenesis. Microthrombi rich in platelets A. A. Proximal left anterior descending coronary artery evaluation of cardiac metastases & cardiac tumors ? B. All of the above B. and Treatment of intima of arteries by ? Harrison’s 18th Ed. None of the above C. Microscopic breaches in endothelial integrity Harrison’s 16th Ed. Ventricular surface of mitral valve arise from focal increases in the content of lipoproteins within regions of the intima. Impact to chest wall overlying heart during the susceptible phase of to their slow from the intima. Fatty streak formation begins beneath a morphologically intact endothelium. These lipoprotein particles undergo further oxidative modifications repolarization just prior to peak of T wave is the cause. Atria In human atherosclerosis. 1981 A. C. Glycosaminoglycans A. Aneurysmal rupture of coronary artery Lipoproteins collect and stay in the arterial wall intima by binding to constituents of extracellular Blunt. Post-pericardiotomy syndrome A. Tako-Tsubo syndrome B. More common in women. Sudden emotional trauma Harrison’s 18th Ed. Torsion injury of heart C. High-density lipoprotein (HDL) particles modified by Hypochlorous acid (HOCl)-mediated chlorination function poorly as cholesterol acceptors. Vitamin K C. Echocardiography D. 1982 space of the intima give rise to ? A. Cardiac MRI 1123 Which of the following represents the initial lesion of D. Commotio cordis C. C. Nicotinic acid and remodeling. 1985 D. 40 years of age A. All of the above 1136 Which of the following is a Niemann-Pick C1-like 1 protein Atherosclerotic plaques often contain fibrin and hemosiderin. 1986 Member of phospholipase family. Thin fibrous caps proinflammatory lipids. All of the above 1135 Current ATP III guidelines recommend lipid screening in all adults above ? Exposure of endothelial cells to laminar shear stress increases the transcription of Krüppel-like Harrison’s 18th Ed. B. All of the above Harrison’s 18th Ed. 1983 direction leading to increase in diameter of the vessel. 10 years of age endothelial nitric oxide synthase. 20 years of age 1130 Atherosclerotic plaques contain which of the following ? C. 30 years of age Harrison’s 18th Ed. 1985 D. 1989 D. Retinaldehyde D. Osteopontin inhibiting an enterocyte cholesterol transporter denoted Niemann-Pick C1-like 1 protein (NPC1L1). and a high content of macrophages. Ubiquinone of atherosclerotic plaque exceeds ~40% of the area encompassed by the internal elastic lamina. Growing atheroma does not encroach on arterial lumen until the burden A. ~ 80 % proinflammatory lipids ? During initial phases of atheroma development. ~ 40 % within the plaque yields chlorinated species such as chlorotyrosyl moieties. LDL cholesterol. A. Cholesterol in bone also localize in atherosclerotic lesions. 1989 Proteins like osteocalcin. Weight loss 1133 Growing atheroma does not encroach on arterial lumen until D. B. ~ 20 % . 1988 plaques also accumulate calcium. Calcium D. relatively large lipid cores. Sildenafil A. Neovascularization 1138 Which of the following can raise HDL levels ? Harrison’s 18th Ed. LDL 1132 Which of the following contribute to atheromata lesion formation ? C. C. Osteocalcin Ezetimibe is a LDL-lowering medication that reduces cholesterol absorption from proximal small bowel by B. KLF2 augments the activity of A. and bone morphogenetic proteins that are usually found A. Hemosiderin include a fasting lipid profile (total cholesterol. atherosclerotic Harrison’s 18th Ed. As they advance. cell proliferation and cell death. as well as calcification and neovascularization. 1985 D. 195 Cardiology MCQ’s FOR MCQ’s FOR MEDICAL MEDICAL PROFESSIONALS PROFESSIONALS BY PROF. Physical activity During the evolution of atherosclerotic plaque. Nicotinic acid Harrison’s 18th Ed. Idebenone 1134 Which of the following is a characteristic feature of the culprit C. in an abluminal Harrison’s 18th Ed. 1984 A. B. All of the above A. C. ~ 60 % 1128 Which of the following leads to the formation of D. lipoprotein-associated phospholipase A2 (LpPL A 2) can generate A. a complex balance between entry and egress of lipoproteins and leukocytes. C. Thioredoxin Characteristic features of the culprit lesion that causes acute MI include thin fibrous caps. Fibrin Current ATP III guidelines recommend lipid screening in all adults >20 years of age and should B. All of the above Harrison’s 18th Ed. Ezetimibe atherosclerotic lesions ? C. Relatively large lipid cores C. triglycerides. Biotin having markers of inflammatory activation predominate and contain relatively few smooth muscle cells at sites of plaque rupture. Triglycerides Harrison’s 18th Ed. contribute to lesion formation. including lysophosphatidyl choline-bearing oxidized lipid moieties from oxidized phospholipids found in oxidized low-density lipoproteins (LDLs). 1987 factor 2 (KLF2) and reduces the expression of a thioredoxin-interacting protein (Txnip) that inhibits the activity of the endogenous antioxidant thioredoxin. particularly in combination with Harrison’s 18th Ed. osteopontin. VLDL A. Aminophylline 1131 Which of the following proteins found in bone also localize in B. Bone morphogenetic proteins 1137 Blood HDL levels vary inversely with those of ? D. AJAY MATHUR Cardiology 195 Local production of hypochlorous acid by myeloperoxidase associated with inflammatory cells B. C. A. This type of vascular remodeling is called compensatory enlargement. can robustly raise HDL. and HDL cholesterol) repeated every 5 years. Nicotinic acid. extracellular matrix production. B. and reduced Txnip levels boost the function of thioredoxin. B. High content of macrophages 1129 Which of the following is an endogenous antioxidant ? D. Macrophages & T lymphocytes B. All of the above Harrison’s 18th Ed. Extracellular matrix production Blood HDL levels vary inversely with those of triglycerides. All of the above the burden of atherosclerotic plaque exceeds ? Weight loss and physical activity can raise HDL. Calcification C. B. an indication that episodes of (NPC1L1) inhibitor ? intraplaque hemorrhage contribute to plaque complications. plaque usually grows outward. Lipoprotein-associated phospholipase A2 (LpPL A2) lesion that causes acute MI ? D. 1986 statins. 3146. Interstitial fluid C. Triglycerides D. 1151 Apo E is synthesized mainly in hepatocytes and also in ? Harrison’s 16th Ed. 3146. Apo CII C. Apo B48 B. Chylomicrons Harrison’s 18th Ed. All of the above D. Dietary cholesterol B. VLDLs. Density B. Apo B48 A. Phospholipids Triglycerides of chylomicrons are hydrolyzed by LPL & free fatty acids are released. 3147 1143 Which of the following is not a hydrophobic lipid ? A. low-density lipoproteins (LDLs). triglycerides. All of the above Five major classes of plasma lipoproteins are chylomicrons. 1147 Major apolipoprotein of VLDL. Fat-soluble vitamins D. 3145 B. Apo E C. Apo B100 A. ApoC-II. Fat-soluble vitamins C. Table 356-1 Harrison’s 18th Ed. Apo AI B. All of the above D. long-chain fatty acids. 1146 Which of the following is a Lipoprotein ? 1140 Lipoproteins transport hydrophobic lipids through which of Harrison’s 18th Ed. 3145 C. esterified & unesterified cholesterol & triglycerides. cholesterol & fat-soluble vitamins from liver to peripheral tissues and transport of cholesterol from peripheral tissues to liver. VLDL from liver is ? Harrison’s 18th Ed. Apo AI D. 3145 Harrison’s 18th Ed. All lipoprotein classes contain phospholipids. Diameter D. IDLs. unesterified cholesterol) and proteins. Triglycerides B. Cell membrane configuration Lipoproteins are complexes of lipids & proteins essential for transport of cholesterol. cholesterol & fat-soluble vitamins) density lipoproteins (HDLs). IDL & LDL. lipase (LPL) is ? Harrison’s 18th Ed. 3145. and LDL is ? 1141 Lipoproteins are essential in the absorption of ? Harrison’s 18th Ed. through body fluids (plasma. intermediate-density lipoproteins (IDLs). 3145 A.apoB-48 in C. D. D. Lipoproteins are classified by density and size. Lymph D.196 MCQ’s FOR MEDICAL PROFESSIONALS BY PROF. & LDLs . Apo E B. Long-chain fatty acids C. IDL. very low density lipoproteins (VLDLs). AJAY MATHUR Cardiology 1139 Lipoproteins are essential for the transport of ? 1145 Lipoproteins have been classified on the basis of their ? Harrison’s 18th Ed. Apo B100 following to and from liver to peripheral tissues ? B. All of the above 1148 Apolipoprotein that is essential for assembly & secretion of Lipoproteins are essential in the absorption of dietary cholesterol. which are inversely related. Triglycerides D. Cholesteryl esters Liver synthesizes apoB-100 and the intestine makes apoB-48. Apo B100 Harrison’s 18th Ed. 3145 A. Unesterified cholesterol D. 3145 1142 Lipoproteins are essential for the transport of which of the A. D. Volume C. Cholesterol ApoB is the major structural protein of chylomicrons. Plasma B. Triglycerides C. Very low density lipoproteins (VLDL) B. 370 . Table 356-1 the following ? A. Table 356-1 B. Cholesterol A. Apo B48 A. All of the above Lipoproteins contain a core of hydrophobic lipids (triglycerides & cholesteryl esters) surrounded 1150 Apolipoprotein that is essential activator of enzyme lipoprotein by hydrophilic lipids (phospholipids. Apo AI C. All of the above which is transferred to circulating chylomicrons from HDL. Low-density lipoproteins (LDL) C. Apo B48 Harrison’s 18th Ed. All of the above 1149 Apolipoprotein that is essential for assembly & secretion of Lipoproteins are essential for transport of triglycerides. acts as a cofactor for LPL. Apo E A. interstitial fluid & lymph) to & from tissues. 3146 1144 Major lipid of lipoproteins is ? A. Figure 356-1 A. Apo E B. Cholesterol C. and fat-soluble vitamins. chylomicrons is ? Harrison’s 18th Ed. Apo B100 Harrison’s 18th Ed. and high- Lipoproteins transport hydrophobic lipids (triglycerides. Fat-soluble vitamins chylomicron and apoB-100 in VLDL. and fat-soluble vitamins in the proximal small intestine. 3146. metformin possesses the best evidence base for A. 2287 C. HDL cardiovascular event reduction. Any of the above 1155 Which of the following enzymes are involved in lipoprotein Studies of human atherosclerosis suggests that the “fatty streak” represents the initial lesion metabolism ? of atherosclerosis. ApoA- 1154 Enzyme lecithin:cholesterol acyltransferase (LCAT) which II is the second most abundant HDL apolipoprotein & is on about two-thirds of all HDL particles. 3148 1158 The surface coat of the chylomicron is composed of ? Harrison’s 18th Ed. apo AIV D. apo AI. 130/80 mmHg B. Apo AI A. 3146. VLDL recommends a blood pressure goal of ? D. 120/70 mmHg 1157 Normolipidemic individuals dispose of most dietary fat in the bloodstream within ? B. Cholesteryl ester transfer protein (CETP) A. Chylomicrons Harrison’s 17th Ed. Sulfonyurea 1156 Cholesteryl ester transfer protein (CETP) circulates in plasma C. LDL 1163 In diabetic populations. Apo B100 C. Macrophages D. Obesity C. LDL B. 120/80 mmHg A. LDL C. Harrison’s 18th Ed. Apo B48 B. Bile acids B. apo B48. 3146 A. 2 hours of the last meal D. Glial cells D. 2288 A. All of the above B. American Diabetes Association C. esterifies free cholesterol in plasma is activated by ? Harrison’s 16th Ed. VLDL C. Lipoprotein lipase (LPL) 1162 Among oral hypoglycemic agents. Apolipoproteins . Fatty streak D. cholesteryl esters. phospholipids and cholesterol to form chylomicrons. B. 197 Cardiology MCQ’s FOR MCQ’s FOR MEDICAL MEDICAL PROFESSIONALS PROFESSIONALS BY PROF. 12 hours of the last meal 1164 Fredrickson classification is used to classify ? Harrison’s 18th Ed. HDL A. IDL ApoA-I. Fibrous plaque B. All of the above 1160 The crucial structural apoprotein for HDL is ? 1153 apo AI. IDL D. apo AII. 2289 Harrison’s 16th Ed. 3146 A. Phospholipid B. Apo E D. apo AI B. VLDL D. which of the following has the best evidence base for cardiovascular event reduction ? B. HDL B. C. 1507 C. 130/70 mmHg Harrison’s 16th Ed. Free cholesterol C. Apo AIV C. Apo AII B. Intimal thickening C. Hepatic triglyceride lipase (HTGL) Harrison’s 17th Ed. C. Acarbose Harrison’s 18th Ed. apo AII and apo AIV are found primarily on ? Harrison’s 18th Ed. None of the above D. Pioglitazone in association with ? D. Neurons Longer-chain fatty acids (>12 carbons) are incorporated into triglycerides & packaged with apoB-48. Apo AI A. 1507 A. 2288 1161 Which of the following represents the initial lesion of atherosclerosis ? A. 4 hours of the last meal Recently updated American Diabetes Association blood pressure goal is 130/80 mmHg in diabetic populations. apo AII C. 6 hours of the last meal D. 3146 Harrison’s 18th Ed. and apo AIV D. Table 356-2 A. AJAY MATHUR Cardiology 197 A. retinyl esters. Hyperlipoproteinemia A. which is synthesized in liver & intestine. All of the above 1159 In plasma. Metformin D. apo C proteins are transferred to chylomicron from which lipoprotein ? 1152 Which of the following Apo A is synthesized only in small intestine ? Harrison’s 16th Ed. is found on virtually all HDL particles. All of the above B. 3147 Among the oral hypoglycemic agents. Broad-beta disease B. & 23% of chylomicrons. All of the above . Elevations of VLDL B. GpI-IX D. 3149 D. 0. 337 D. Type IIa C. All of the above 1167 Type III hyperlipidemia is also known as ? Harrison’s 18th Ed. most likely cause is ? B. It is a spherical particle of 250 Å diameter Harrison’s 18th Ed. Human immunodeficiency virus protease inhibitors D. All of the above disease. Table 356-3 A. AJAY MATHUR Cardiology 1165 Which of the following hyperlipidemias is not characterized 1171 Conditions that cause hyperlipidemia are all except ? by elevated triglycerides ? Harrison’s 18th Ed. 337 D. triglycerides comprise ~86%. B. Type IIb D. Type I D. 3149 C. Hypothyroidism C. 2294 D. Tg is present in LDL & HDL in smaller quantities of <10%.198 MCQ’s FOR MEDICAL PROFESSIONALS BY PROF. 0. or broad-beta D. All of the above C. type IIa hyperlipidemia does not have Harrison’s 18th Ed. Serum cholesterol levels are normal. A. Type V hyperlipidemia A. GpIb located on circulating platelets When triglyceride levels >1000 mg/dL. Nephrotic syndrome 1166 Which of the following Type I hyperlipidemia does not increase B. GpIVb located on circulating platelets A. Consists of apo B-100 and apo (a) 1169 When triglyceride levels >1000 mg/dL. Dysbetalipoproteinemia Harrison’s 16th Ed. Elevations of triglycerides C. 2294 B. Type II hyperlipidemia D. respectively. Type IIb Harrison’s 16th Ed. Alcohol ingestion risk for developing coronary artery disease ? Harrison’s 18th Ed. A. Diabetes mellitus B.02586 C. Type I B. Pheochromocytoma D.05863 A. It closely resembles HDL A. Fibrinogen B. 3148. and triglyceride levels are almost Harrison’s 18th Ed. most likely cause is type V hyperlipidemia. VLDLs. Type III hyperlipidemia C. 3147 always <1000 mg/dL. Table 356-3 C. Table 356-3 A. Oral progesterone A. VLDL Harrison’s 16th Ed. IV & V are characterized by elevated triglycerides. GpIIb located on circulating platelets 1170 Triglyceride content is least in which of the following ? C. C. Beta-adrenergic antagonists C. remnant removal disease. Lp(a) has proatherosclerotic and prothrombotic actions B. C. Cyclosporine Type III is also known as dysbetalipoproteinemia. 55%. Thiazide diuretics D.08651 B. 3150 hypertriglyceridemia. Fibronectin By dry weight. IDL A. All of the above 1176 What is not true for Lipoprotein (a) ? Type IV is characterized by abnormal elevations of VLDL. 3148 1174 Drug that causes hyperlipidemia include ? A. 0. 3148. Oral estrogen B. 3148. Types I. Normal serum cholesterol D. Chylomicrons 1178 Which of the following are essential to increase the cell-to-cell B. GpIIIb located on circulating platelets Harrison’s 18th Ed. LDL contact and facilitate platelet aggregation ? C. Sertraline hydrochloride Type I is the only form of hypertriglyceridemia that does not confer an increased risk for developing coronary artery disease. Type III A. and IDLs. Type IV hyperlipidemia 1177 Surface-bound von Willebrand factor binds to ? Harrison’s 16th Ed.02861 D. Isotretinoin B. 1175 To convert the values for LDL cholesterol to millimoles per 1168 Type IV hyperlipidemia is characterized by ? liter. 3151 Harrison’s 18th Ed. Type III 1172 Conditions that cause hyperlipidemia are all except ? According to Fredrickson classification. IIb. Type IIa 1173 Drug that causes hyperlipidemia include ? C. Obesity A. Remnant removal disease A. 0. III. multiply by ? Harrison’s 18th Ed. 16 1191 Major features of metabolic syndrome include all except ? A. Protein C B. Warfarin Harrison’s 18th Ed. Hypertension more likely in men A. CYP 2C9 A. Phenyl B. CYP 2D6 C. Fasting triglycerides >150 mg/dL more likely in women antioxidant properties ? Lancet 2004. 1992 B. The Metabolic Syndrome D. hypertriglyceridemia. Low high-density lipoprotein (HDL) cholesterol 1185 Heparin cofactor is also called ? Harrison’s 16th Ed. CYP 2D6 C. CYP 2C9 B. 1183 The major hepatic isoenzyme involved in quinidine metabolism is ? 1190 Metabolic syndrome increases the risk of ? Harrison’s 16th Ed. Angiotensin II Harrison’s 16th Ed. 1992 D. Stroke D. Lung A. Thromboxane A2 1187 Angiotensin I is converted by angiotensin-converting enzyme to angiotensin II mainly in ? D. Syndrome X C. Quinidine A. Angiotensin III A. Hypertriglyceridemia D. AJAY MATHUR Cardiology 199 1179 Platelet aggregation and further activation is done by ? B. High low-density lipoprotein (LDL) cholesterol D. Adrenaline C. Mexiletine B. Angiotensinogen 1181 The major hepatic isoenzyme involved in nifedipine B. the following drugs ? Harrison’s 16th Ed. APLA A.364:985-996 D. CYP 2C19 C. All of the above 1184 CYP enzyme activity inducers lower plasma levels of which of Metabolic syndrome consists of a constellation of metabolic abnormalities that confer increased risk of cardiovascular disease (CVD) and diabetes mellitus (DM). CYP 3A 1182 Major hepatic isoenzyme involved in lidocaine metabolism is ? 1189 The metabolic syndrome is also called ? Harrison’s 18th Ed. low high-density lipoprotein (HDL) cholesterol. 1992 Harrison’s 16th Ed. Antithrombin III Harrison’s 18th Ed. Tyrosine . All of the above C. 676 C. CYP 2C19 B. Liver Harrison’s 16th Ed. Heart B. 1992 A. CYP 2C19 C. Diabetes mellitus C. Methyl A. CYP 2C9 D. Syndrome M B. CYP 3A D. CYP 2D6 Chapter 242. CYP 2C19 B. 340 Five major features of the metabolic syndrome include central obesity. Protein S 1192 Which of the following about metabolic syndrome is false ? C. Hypertension B. Increases in waist circumference predominate in women 1186 Which component of red wine has anti-inflammatory and C. Prevalence increases with age B. Kidney atorvastatin metabolism is ? B. Angiotensin I metabolism is ? C. 16 C. Syndrome Y D. Phenolic Harrison’s 16th Ed. All of the above Harrison’s 16th Ed. CYP 2C9 A. CYP 3A D. 16 Harrison’s 18th Ed. CYP 2D6 1188 Which of the following is a ‘heptapeptide’ ? Harrison’s 16th Ed. 16 A. ADP D. 199 Cardiology MCQ’s FOR MCQ’s FOR MEDICAL MEDICAL PROFESSIONALS PROFESSIONALS BY PROF. 2131 1180 The major hepatic isoenzyme involved in simvastatin / A. CYP 3A A. hyperglycemia. Syndrome Z The metabolic syndrome is also called syndrome X or insulin resistance syndrome. Central obesity C. and hypertension. 2130 D. 16 D. A. 1994. Figure 242-2 Adiponectin is an anti-inflammatory cytokine that enhances insulin sensitivity. ) . which of the following effect resistance ? of insulin is lost ? Harrison’s 18th Ed. it inhibits expression of gluconeogenic enzymes and rate of A. Resistin adipose tissue mass. Increased activity of sympathetic nervous system B. Figure 242-2 C. 1994 A. insulin is a vasodilator with secondary effects on sodium circulating fatty acids.200 MCQ’s FOR MEDICAL PROFESSIONALS BY PROF. Liver glucose production. Lentigo 1198 Which of the following is an anti-inflammatory and insulin- B. Postprandial hyperinsulinemia A. free fatty acids (FFAs) are released in abundance from an expanded B. Pancreas 1203 Which of the following is a physical finding in metabolic D. Sodium reabsorption in kidney C. Increased production of glucose 1202 Levels of which of the following is reduced in the metabolic B. hyperglycemia. 1994. All of the above pass through endothelial basement membrane & adhere to glycosaminoglycans. Adiponectin 1204 Which of the following is associated with metabolic syndrome ? D. None of the above D. 1994 1193 Lipodystrophic disorder associated with the metabolic A. it increases glucose transport and enhances fatty acid oxidation. increases in waist circumference predominate in women. All of the above A. Reduced A. Lung syndrome ? C-reactive protein (CRP) is produced in liver. Kidney metabolic syndrome. 1994. Vasodilatory D. Interleukin 6 (IL-6) D. ~ 130 mg/dL A. A. In insulin resistance. Small dense LDLs are toxic to endothelium and are able to D. Onset of insulin resistance is heralded by postprandial hyperinsulinemia. 1993 1201 Which of the following is a proinflammatory cytokine ? Harrison’s 18th Ed. resistin. HIV-related lipodystrophy in patients on HAART There is almost always a predominance of more atherogenic small dense LDLs when fasting serum triglyceride is ~180 mg/dL. 1995 Adiponectin is an anti-inflammatory cytokine produced exclusively by adipocytes. In muscle. Hyperglycemia C. FFAs result in an increased production of glucose and triglycerides and secretion of very low density lipoproteins (VLDLs). free fatty acids (FFAs) result in ? Proinflammatory cytokines include interleukin (IL)-1. Interleukin-18 1197 C-reactive protein (CRP) is produced in ? Harrison’s 18th Ed. C. 1995 A. Insulin stimulates lipoprotein lipase (LPL) in adipose tissue B. reabsorption in the kidney. Fasting hyperinsulinemia B. Resistin Lipoatrophy or acanthosis nigricans is a physical finding in metabolic syndrome. IL-6. Increased secretion of very low density lipoprotein (VLDL) Harrison’s 18th Ed. Increased production of triglycerides syndrome ? C. In liver. IL-18. All of the above An early major contributor to the development of insulin resistance is an overabundance of Under normal physiologic conditions. Lipolysis produces fatty acids C. AJAY MATHUR Cardiology In metabolic syndrome. Ephelide (freckle) B. 1995 A. Acanthosis nigricans sensitizing cytokine ? Harrison’s 18th Ed. C. C-reactive protein (CRP) D. when fasting serum triglyceride is ? Harrison’s 18th Ed. vasodilatory effect of insulin is lost but the followed by fasting hyperinsulinemia and. Harrison’s 18th Ed. 1993 Harrison’s 18th Ed. Café au lait macule A. C-reactive protein (CRP) Harrison’s 18th Ed. Berardinelli-Seip congenital lipodystrophy C. and C-reactive protein (CRP). whereas 1199 There is almost always a predominance of small dense LDLs fasting triglycerides >150 mg/dL and hypertension are more likely in men. Nonalcoholic steatohepatitis (NASH) production of this insulin-sensitizing cytokine adiponectin is associated with the metabolic syndrome. 1993 B. Resistin C. 1194 Which of the following is central to the development of insulin 1200 In the setting of insulin resistance. Adiponectin is reduced in the B. Insulin inhibits lipolysis in adipose tissue A. C-reactive protein (CRP) D. ~ 100 mg/dL syndrome is ? Harrison’s 18th Ed. Polycystic ovarian disease (PCOS) . IL-18 B. It is produced exclusively by adipocytes. C. Dunnigan familial partial lipodystrophy D. ~ 180 mg/dL C. due to activation of adenosine monophosphate (AMP) kinase. Figure 242-2  (TNF. 1195 Which of the statements is false ? Harrison’s 18th Ed. Adiponectin In metabolic syndrome. renal effect on sodium reabsorption is preserved. In liver. ultimately. 1995 D. tumor necrosis factor- Harrison’s 18th Ed. ~ 150 mg/dL B. Insulin induced increases the activity of sympathetic nervous system is preserved in insulin resistance. All of the above 1196 In the liver. Overabundance of circulating fatty acids D. B. All of the above Fibrates (gemfibrozil or fenofibrate) is the drug of choice to lower fasting triglycerides (35 . AJAY MATHUR Cardiology 201 C. 1998 Harrison’s 18th Ed. For each Harrison’s 18th Ed. nonalcoholic fatty liver disease (NAFLD) and/or nonalcoholic steatohepatitis (NASH). contractility. Asymmetric dimethylarginine (ADMA) D. 1996 D. Large epicardial arteries (R1) 1208 In metabolic syndrome treatment. cytochrome P450 system (including some statins) greatly increases the risk of myopathy. including leukocyte Chapter 243. 1434 fasting triglycerides are >200 mg/dL. ADMA acts as a competitive inhibitor of NO synthase. 3500 The major determinants of myocardial oxygen demand (MVO 2 ) are heart rate. In metabolic synfrome and diabetes. Ischemic Heart Disease adhesion. the major determinant of coronary resistance is found in R 2 and R 3. There is no effect on HDL cholesterol with ezetimibe or omega-3 fatty acids. other metabolic alterations specifically associated with metabolic of omega-3 fatty acids. Omega-3 fatty acids Prinzmetal’s angina . All of the above D. LDL cholesterol should be reduced to <100 mg/dL. 1996-97 C. asymmetric dimethylarginine. ~8% Blood flows through coronary arteries in a phasic fashion. Large epicardial coronary arteries are conductance vessels 1209 Drug that lowers triglycerides is ? B. Diastole B. trans-fats (as few as possible). NO has vasodilator activity. R1 is trivial. HMG-CoA reductase inhibitors B. Prediastole D. Fibrate Prinzmetal’s angina B. and obstructive sleep apnea (OSA). ~2% B. prothrombotic factors (fibrinogen. ADMA is metabolised by dimethylarginine dimethylaminohydrolase (DDAH) to L-citrulline and dimethylamine. Systole A. Nicotinic acid D. 1211 Which of the following is a determinant of myocardial oxygen demand (MVO2) ? 1206 How many kilocalories equal one pound of fat ? Harrison’s 18th Ed. Phenylbenzimidazole sulfonic acid 10%). there is only ~6% additional lowering of LDL cholesterol. 1995 C. Cholestyramine intramyocardial capillary vessels (R 3). and vascular smooth muscle cell proliferation. Arteriolar and intramyocardial capillary vessels (R 3) Harrison’s 18th Ed. They should not be administered when Harrison’s 16th Ed. Myocardial wall tension (stress) C. Ezetimibe About 75% of the total coronary resistance to flow occurs across three sets of arteries: (1) large epicardial arteries (Resistance 1 = R 1). In the absence of significant flow-limiting atherosclerotic obstructions. proinflammatory cytokines. Prearteriolar vessels (R2) the following can increase triglycerides ? C. Ezetimibe A. Nicotinic acid B. serum viscosity. D. white blood cell count. 1996 A. and cholesterol (<200 mg daily) should be applied aggressively. there is an additional 1212 Majority of blood flow through coronary arteries is during ? lowering of LDL cholesterol by ? Harrison’s 18th Ed. Also. If LDL cholesterol remains above goal. Concomitant administration with drugs metabolized by the 3A4 syndrome include increases in apoB and apoC-III. Dimethyl sulfoxide To increase HDL cholesterol. Fibrates Bile acid sequestrants cholestyramine & colestipol must be used with caution in metabolic 1214 Which of the following statements is false ? syndrome because they can increase triglycerides. D. CRP. Other drugs that lower triglycerides include statins. A. 2500 D. Homosalate Endothelial NOS converts amino acid L-arginine into L-citrulline and NO. Presystole C. Abnormal constriction of resistance vessels can cause C.50% reduction). (2) prearteriolar vessels (R2). fibrates. 1998 doubling of the statin dose. 1997 A. ~4% C. plasminogen activator inhibitor 1). and (3) arteriolar and C. uric acid. polycystic ovarian disease 1210 Which of the following has no effect on HDL cholesterol ? (PCOS). 500 B. Nicotinic acid is the only currently available drug with predictable HDL cholesterol-raising properties. Harrison’s 18th Ed. microalbuminuria. 1500 C. Myocardial contractility B. Obstructive sleep apnea (OSA) D. platelet aggregation. 1207 For each doubling of statin dose. All of the above D. and myocardial wall tension (stress). ~6% D. administration of which of B. R2 + R3 A. Fibrates 1205 Which of the following is an endogenous inhibitor of nitric oxide synthase (eNOS) ? B. and bile acid sequestrants have modest effects (5 - C. nicotinic acid. and high doses Apart from insulin resistance. statins. with majority occurring during diastole. Intramyocardial arterioles are resistance vessels Harrison’s 18th Ed. Asymmetric dimethylarginine (ADMA) is an early marker of atherosclerotic vascular disease. statins 1213 Major determinant of total coronary resistance is found in ? (HMG-CoA reductase inhibitors) may produce a 20–60% lowering of LDL cholesterol. NO inhibits key processes involved in vascular disease. homocysteine. Heart rate A. 201 Cardiology MCQ’s FOR MCQ’s FOR MEDICAL MEDICAL PROFESSIONALS PROFESSIONALS BY PROF. Abnormal constriction of conductance vessels can cause A. A. Diets restricted in saturated fats (<7% of calories). Bile acid sequestrants Harrison’s 18th Ed. myocardial ~3500 kcal = 1 lb of fat. 1996 A. 1998 Harrison’s 18th Ed. Abnormal constriction or failure of normal dilation of coronary resistance the form of inversion of T waves and. especially to the ulnar surfaces of forearm and hand. Any of the above 1220 In ECG. < 20 years B. Angina is usually crescendo-decrescendo in nature B. Lactate B. Epicardial ischemia C.30 years C. radiate to left shoulder and to both arms. in C. Fatty acids Harrison’s 18th Ed. when more severe. Transmural ischemia Harrison’s 18th Ed. substernal discomfort in angina pectoris Harrison’s 18th Ed. central. vessels causes ischemia termed as microvascular angina. Pain can radiate to both arms D. substernal discomfort indicative of angina pectoris is termed as Levine’s sign. > 25 minutes B. All of the above D. while ST-segment elevation is caused by more severe transmural ischemia. such whether it is permanent. Subendocardial ischemia 1226 Atypical presentations of angina pectoris are more frequent in ? B. Levine’s sign D. 1999 Harrison’s 18th Ed. Recumbent D. Landolfi’s sign C. Typically lasts for 10 to 15 minutes C. Jaw C. Angina is rarely localized below the umbilicus or above the mandible. Epicardial ischemia healthy persons they serve as conduits and are referred to as conductance vessels. 1999 A.40 years D. Amino acids A. with subsequent myocardial necrosis (>20 minutes). 1215 Which of the following sites has the most predilection for atherosclerotic plaques to develop ? 1221 Coronary atherosclerosis begins to develop at what age ? Harrison’s 18th Ed. Origin of epicardial arteries A. N Engl J Med 2005. 2000 A. teeth.202 MCQ’s FOR MEDICAL PROFESSIONALS BY PROF. and epigastrium. and can uptake of sodium by myocytes as well as an increase in cytosolic calcium. 1999 Harrison’s 18th Ed. Increase in cytosolic calcium C. 1217 During myocardial ischemia. 2000 A. D. 2000 B. Transmural ischemia B. All of the above resistance vessels.352:2524-33 . typically lasts 2 to 5 minutes. 1999 A. Transient ST-segment depression reflects patchy subendocardial ischemia. transient ST-segment depression reflects ? 1225 “Angina decubitus” refers to anginal pain occuring when the Harrison’s 18th Ed. Carvolo’s sign Squeezing. 1999 is termed ? A. impairment of cell membrane function leads to ? 1223 Which of the following statements about angina is false ? Harrison’s 18th Ed. Uptake of sodium by myocytes B. Any of the above D. 20 . a radiation pattern is more typical of pericarditis. 1216 Normal myocardium metabolizes which of the following ? 1222 Squeezing. 1219 In ECG. 1999 Harrison’s 18th Ed. interscapular region. 30 . > 20 minutes A. root of the neck. All of the above D. Branch points in epicardial arteries B. Leakage of potassium from myocytes A. Lhermitte’s sign Normal myocardium metabolizes fatty acids and glucose to carbon dioxide and water. Epigastrium D. > 35 minutes D. AJAY MATHUR Cardiology Although the large epicardial coronary arteries are capable of constriction and relaxation. Trapezius muscles Severity & duration of imbalance between myocardial oxygen supply & demand determine whether damage is reversible (<=20 minutes for total occlusion in the absence of collaterals) or Chest discomfort due to myocardial ischemia does not radiate to the trapezius muscles. Abnormal constriction of the conductance vessels can cause severe ischemia Ischemia also causes characteristic changes in ECG such as repolarization abnormalities in in Prinzmetal’s angina. Sitting with legs hanging C. Interscapular region B. transient ST-segment elevation reflects ? Episodes of angina that occur at night while the patient is recumbent is termed as angina decubitus. > 30 minutes C. 1999 patient is in which of the following positions ? Harrison’s 18th Ed. 2000. while the intramyocardial arterioles normally exhibit changes in tone and are therefore referred to as D. central. Subendocardial ischemia A. by displacement of ST segments. Terminal regions of epicardial arteries C. Rarely localized below umbilicus or above mandible Impaired myocyte cell membrane function due to ischemia leads to leakage of potassium and Angina is usually crescendo-decrescendo in nature. All of the above C. Squatting B. 1218 Minimum duration of total occlusion of epicardial vessel in absence of collaterals for development of myocardial necrosis 1224 Radiation of chest pain towards which of the following is more is ? typical of pericarditis ? Harrison’s 18th Ed. Harrison’s 18th Ed. age 20 and is widespread even among adults who were asymptomatic during life. > 40 years Atherosclerotic plaques have a predilection to develop at sites of increased turbulence in Postmortem studies have shown that coronary atherosclerosis often begins to develop prior to coronary flow like branch points in the epicardial arteries. It can also arise in or radiate to the back. jaw. 2000 A. Severe aortic stenosis stress testing except ? D. or the development of a ventricular tachyarrhythmia. Severe mitral stenosis Harrison’s 18th Ed. Older patients D. Diabetes mellitus D. Palpitation D. 2004 C. Hypertrophic cardiomyopathy 1235 Modified (heart rate limited) exercise stress tests can be earliest performed safely in patients of uncomplicated MI after how B. Chest discomfort 1236 Contraindications to exercise stress testing include all except ? B. Incidence of false-positive tests is significantly increased in patients with low probabilities of IHD like asymptomatic men <40 years or in premenopausal women with no risk factors for 1229 Disorders that may cause angina in absence of coronary premature atherosclerosis. unstable rhythm. Severe shortness of breath Harrison’s 18th Ed. severe aortic stenosis. Fatigue C. All of the above 1233 Overall sensitivity of exercise stress electrocardiography is ? Harrison’s 18th Ed. Pulmonary hypertension Harrison’s 18th Ed. 9 days 1230 All of the following are indications of discontinuing exercise D. severe shortness of breath. Failure of blood pressure to increase C. AJAY MATHUR Cardiology 203 A. ventricular hypertrophy. 2004 1238 Adverse prognostic signs in exercise stress testing include A. Women D. It is also increased in patients taking cardioactive drugs like digitalis & antiarrhythmic agents. Rest angina within 48 hours D. Right coronary artery B. ~ 85 % D. 2004 1227 Anginal “equivalents” are all except ? A. All of the above Anginal “equivalents” are symptoms of myocardial ischemia other than angina. 3 days Aortic stenosis. Systemic hypertension A. resting ST- atherosclerosis are all except ? segment & T-wave abnormalities. severe fatigue. Active infective endocarditis in systolic blood pressure >10 mmHg. limited to which coronary artery ? Harrison’s 18th Ed. uncontrolled heart failure. Patients with ventricular hypertrophy C. Faintness Overall sensitivity of exercise stress electrocardiography is only ~75%. Dyspnea B. Unstable rhythm 1231 All of the following are indications of discontinuing exercise C. Patients taking digitalis and antiarrhythmic agents B. ~ 55 % Harrison’s 18th Ed. dizziness. Acute myocarditis C. 2001 safely in patients as early as 6 days after uncomplicated MI. pulmonary hypertension and hypertrophic cardiomyopathy B. Development of ventricular tachyarrhythmia C. Development of angina . Patients with abnormal serum potassium levels D. Rise in systolic blood pressure > 40 mmHg B. severe pulmonary 1232 False-negative stress test is usual in obstructive disease hypertension and active infective endocarditis. A. or in those with intraventricular conduction disturbances. 2004 D. C. These include dyspnea. Dizziness A. a fall D. Diabetics Obstructive disease limited to circumflex coronary artery results in a false-negative stress test since lateral portion of heart which it supplies is not well represented on the surface 12-lead ECG. ST-segment depression > 0. Contraindications to exercise stress testing include rest angina within 48 hours. Development of supraventricular tachyarrhythmia B. Smoker B. 2000 A. or abnormal serum potassium levels. 2004 B. 12 days stress testing except ? Modified (heart rate limited rather than symptom-limited) exercise tests can be performed Harrison’s 18th Ed. 1228 Anginal “equivalents” are more common in ? 1234 Incidence of false-positive exercise stress electrocardiography Harrison’s 18th Ed. fatigue and faintness and are more common in the elderly and in diabetic patients.2 mV (2 mm). Left anterior descending coronary artery all except ? Harrison’s 18th Ed. acute myocarditis. ~ 75 % C. 2001 A. C. 2001 A. Hypertension C. Fall in systolic blood pressure > 10 mmHg A. 2004 A. Posterior descending artery B. Aortic regurgitation many days ? C. Harrison’s 18th Ed. 2004 B. 6 days may cause angina in the absence of coronary atherosclerosis. Uncontrolled heart failure D. ST-segment depression > 0. Hypertension Treadmill exercise stress test is discontinued upon evidence of chest discomfort. ~ 65 % B. aortic regurgitation.2 mV (2 mm) 1237 Contraindications to exercise stress testing include all except ? Harrison’s 18th Ed. 203 Cardiology MCQ’s FOR MCQ’s FOR MEDICAL MEDICAL PROFESSIONALS PROFESSIONALS BY PROF. Left circumflex coronary artery A. Women A. 2000 is significantly increased in ? Harrison’s 18th Ed. B. and/or ST-segment depression that persists for “>5 minutes” after the termination of exercise increases the specificity of the test and suggests severe IHD and a high risk of future adverse events. 6 hours 1243 Which of the following is associated with coronary artery D. A. Regional wall motion abnormalities C. and sunlight. Guanosine triphosphate (GTP) N Engl J Med 2005. patient should be kept free of C. Guanosine diphosphate (GDP) 1242 Glagov effect. Harrison’s 17th Ed. 4 hours C. Total triglyceride levels are an independent risk factor for CHD in women but C. organic nitrates release nitric oxide (NO) that binds to guanylyl cyclase in vascular smooth muscle cells. Paced ventricular rhythm 1245 Which of the following is an independent risk factor for CHD in women but not in men ? When resting ECG is abnormal (Wolff-Parkinson-White syndrome. Bacterial endocarditis Failure of BP to increase or an actual decrease with signs of ischemia during the exercise test Vasculitis of the coronary arteries (intimal proliferation and infiltration of the vessel wall with is an important adverse prognostic sign. Adenosine not in men. and severe atherosclerosis that does not encroach on the lumen may go undetected. coronary obstruction at different times 1248 To minimize nitrate tolerance. B. low high-density lipoprotein (HDL) B.it accelerates D. Platelet guanylyl cyclase diameter of lumen to be maintained When metabolized. Cyclic guanosine monophosphate A. It provides no information A. Wolff-Parkinson-White syndrome C. since it may reflect ischemia induced global left mononuclear cells) occurs in Kawasaki disease leading to beadlike aneurysms and thromboses. All of the above C. Development of angina and/or severe (>0. paced ventricular rhythm). Air 1241 Agatston score is used for ? Harrison’s 18th Ed. and lack of physical activity are important risk factors for CHD in both men and women. Hypertension B. As coronary plaque burden increases. or positive remodeling refers to ? B. Dipyridamole Elevated cholesterol levels. Obesity B. Kawasaki disease . 2005 D. A. AJAY MATHUR Cardiology C. and is associated with an earlier age of menopause. Elevated total triglyceride A. 29 exercise test in all of the following conditions except ? A. 2 hours about the arterial wall. All of the above Harrison’s 18th Ed. Dopamine cholesterol levels. 1244 Which of the following in not a risk factor for CHD in both men & women ? 1239 Stress myocardial perfusion imaging is more informative than Harrison’s 16th Ed. 29 depression. 2010 Coronary arteriography outlines the lumina of the coronary arteries. 2005 In order to minimize the effects of nitrate tolerance. 2010 area & density of calcification. Coronary plaque tends to produce different levels of which causes relaxation of vascular smooth muscle. Intravenous dipyridamole or adenosine can be used in place of exercise for noninvasive stress testing to create a coronary “steal” by temporarily increasing flow in nondiseased segments of the coronary vasculature at the expense of diseased segments. 2005 B. Right bundle branch block D. moisture. obesity. Coronary calcium quantification Nitroglycerin deteriorates with exposure to air. smoking. D. All of the above atherosclerosis. Smoking C. >1 mm of resting ST segment Harrison’s 16th Ed. Moisture A. Coronary plaque is unduly unstable the drug for a minimum of how many hours each day ? D. leading to an increase in cyclic guanosine monophosphate. Sunlight B. ventricular dysfunction. Smoking is an important risk factor for CHD in women. hypertension. 8 hours aneurysms ? Harrison’s 18th Ed. ST-segment depression persisting for > 3 minutes after C.2 mV) ST-segment depression myocardial ischemia and infarction. exerts direct negative effects on cardiac function. Elevated cholesterol levels Harrison’s 18th Ed. Antiphospholipid antibody syndrome D. minimum effective dose should be used and a minimum of 8 hours each day kept free of the drug. diabetes. Elevated total triglyceride A.204 MCQ’s FOR MEDICAL PROFESSIONALS BY PROF.352:2524-33 C. A. 2005 B. Left bundle branch block D. which allows D. atherosclerotic mass tends to stay external to lumen. Myocardial ischemia quantification D. Graded incremental infusion of 1246 Nitroglycerin deteriorates with exposure to ? dobutamine may be administered to increase MVO 2. Collateral blood flow quantification 1247 Increase in which of the following causes relaxation of vascular Coronary calcium detected by electron beam computed tomography (EBCT) & multidetector smooth muscle ? computed tomography (MDCT) is quantified using the Agatston score which is based on the Harrison’s 18th Ed. Syphilis termination of exercise D. left bundle branch block. Diabetes Harrison’s 18th Ed. at a low workload (before completion of stage II of Bruce protocol). information gained from an exercise test can be enhanced by stress myocardial perfusion imaging. Low HDL cholesterol 1240 Which of the following drugs is used for stress echocardiography ? C. 1522 A. Severe ST-segment depression at low workload B. Low HDL-cholesterol and diabetes are more important risk factors for CHD in women than in men. Nifedipine (Immediate release) A. 2016 C. Betaxolol of patients with acute myocardial infarction (MI) with ST-segment elevation on their presenting electrocardiogram (STEMI) and those with unstable angina (UA) and non-ST-segment elevation B. 5% D. 5% B. A. 15 % 1254 Which of the following calcium channel blocker may be associated with increased risk of mortality if administered C. dynamic obstruction [coronary spasm. Unstable angina (UA) 1250 Which of the following drug is useful in the management of D. Felodipine features of UA develops evidence of myocardial necrosis. as reflected in elevated cardiac biomarkers. 2015 A. 205 Cardiology MCQ’s FOR MCQ’s FOR MEDICAL MEDICAL PROFESSIONALS PROFESSIONALS BY PROF. 1526 ST-Segment Elevation Myocardial A. particularly in the absence of beta blockers. 40 % Harrison’s 18th Ed. 40 % . Nicardipine A. 2009. 2008. Table 243–6 A. Nisoldipine Harrison’s 18th Ed. ACS is composed A. progressive mechanical obstruction [rapidly advancing coronary atherosclerosis or restenosis following percutaneous coronary intervention (PCI)] & secondary duration of action ? UA related to increased myocardial oxygen demand and/or decreased supply (anemia). Occurs with a crescendo pattern 1251 Which of the following is not a dihydropyridine calcium channel blocker ? D. 15 % Nifedipine (Immediate release) may be associated with increased risk of mortality if administered C. Table 243–6 B. Nifedipine D. Diabetics with autonomic dysfunction 1255 Acute coronary syndrome (ACS) includes ? D. Amlodipine with a crescendo pattern. C. Patients with higher pain thresholds Infarction B. Diltiazem 1257 Which of the following is the most common pathophysiologic D. More Harrison’s 18th Ed. ST-segment elevation MI Asymptomatic persons with severe coronary atherosclerosis who exhibit ST-segment changes during activity include those who exhibit higher thresholds to electrically induced pain. Verapamil Pathophysiologic processes leading to UA are plaque rupture or erosion with superimposed nonocclusive thrombus (most common cause). short-acting dihydropyridines should be avoided because of the risk of precipitating infarction. as in 1253 Which of the following calcium channel blocker has the longest Prinzmetal’s variant angina). Amlodipine 1259 Among patients with UA/NSTEMI. 2015 Betaxolol is a selective beta 1 blocker. All of the above Harrison’s 18th Ed. Table 243–6 than one of these processes may be involved in many patients.those with stable angina secondary to chronic coronary artery disease and patients with acute coronary syndromes (ACS). and/or it occurs A. higher B. D. what percentage would have B. 30 % during acute myocardial infarction ? D. New onset angina pectoris C. Pentaerythritol tetranitrate 1256 Which of the following is true for unstable angina (UA) ? D. 2009. Diltiazem (Immediate release) Harrison’s 18th Ed. The diagnosis of NSTEMI is established if a patient with the clinical B. Amlodipine 1258 Among patients with UA/NSTEMI. Dynamic obstruction A. Pentaerythritol tetranitrate is used sublingually for its antianginal effect. Non-ST-segment elevation MI endorphin levels and diabetics with autonomic dysfunction. In general. 30 % during acute myocardial infarction. Progressive mechanical obstruction B. Unstable Angina and Non- Harrison’s 17th Ed. AJAY MATHUR Cardiology 205 1249 Asymptomatic persons with severe coronary atherosclerosis include ? Chapter 244. Table 243–4 Patients with IHD fall into two large groups . Table 243–6 UA is defined as angina pectoris or equivalent ischemic discomfort with at least one of three features: it occurs at rest usually lasting > 10 minutes. All of the above Harrison’s 18th Ed. 2009. Plaque rupture with superimposed nonocclusive thrombus channel blocker ? Harrison’s 18th Ed. Occurs at rest lasting > 10 minutes of late inward sodium current. 2016 D. 2015 1252 Which of the following is a nondihydropyridine calcium A. Increased myocardial oxygen demand and/or decreased supply C. All of the above angina pectoris ? Harrison’s 18th Ed. Nisoldipine UA/NSTEMI can be caused by a reduction in oxygen supply and/or by an increase in myocardial oxygen demand (tachycardia or severe anemia) superimposed on a coronary obstruction. 2009. it is severe and of new onset. Nicardipine C. D. Ranolazine MI (UA/NSTEMI). Patients with higher endorphin levels C. All of the above Harrison’s 18th Ed. C. Isradipine left main stenosis coronary artery disease ? C. Isradipine process in the development of UA ? Harrison’s 18th Ed. B. what percentage would have three-vessel coronary artery disease ? B. Verapamil (Immediate release) B. Ranolazine casts its antianginal effect through blockage A. C. 1261 Among patients with UA/NSTEMI. 1270 With prasugrel therapy. MI. 15 % C. 6 hours D. Hypertension D. 1265 Risk factor for CAD include ? 1271 Which of the following is a reversible ADP inhibitor ? Harrison’s 18th Ed. Prasugrel C. 1269 Prasugrel is contraindicated in patients with ? 1263 Cardiac biomarkers include ? Harrison’s 18th Ed. All of the above With prasugrel therapy. All of the above D. sometimes in epigastrium. and/or the left arm. 30 % D. brain natriuretic peptides & C-reactive protein. Patients with previous stroke or transient ischemic attack C. other risk factors are diabetes mellitus. C-reactive protein B. some of the latter have Prinzmetal’s Absolute contraindications to the use of nitrates are hypotension or the use of sildenafil variant angina. 2017 B. the underweight. 2019 A. what percentage would have no critical coronary artery stenosis ? 1267 Nitrates must not be administered if sildenafil (Viagra) has been Harrison’s 18th Ed. 40 % C. and 10% have no critical coronary stenosis. Dose is 60 mg load followed by 10 mg/day for up to 15 months. It reduces the risk of cardiovascular death. Ticagrelor D. 5% Harrison’s 18th Ed. Tuberculosis C. (Viagra) or other drugs in that class within the previous 24 hours. 1262 The clinical hallmark of UA/NSTEMI is chest pain that is ? 1268 Letter “C” in CURE trial stands for ? Harrison’s 18th Ed. It is contraindicated in There is a direct relationship between the degree of troponin elevation & mortality. 1528 Harrison’s 18th Ed. Cardiac Clinical hallmark of UA/NSTEMI is chest pain. the elderly. Elevated levels of creatinine A. or stroke by 16% compared with creatinine.361:941 Harrison’s 18th Ed. typically located in the substernal region or CURE is CURE trial stands for Clopidogrel in Unstable Angina to Prevent Recurrent Events. left ventricular dysfunction & elevated levels of P2Y12. Any of the above D. and higher level of platelet inhibition than Elevated levels of CK-MB & troponin distinguish patients with NSTEMI from those with UA. myocarditis or pulmonary embolism. All of the above Besides the seven independent risk factors in Thrombolysis in Myocardial Infarction (TIMI) Ticagrelor is a reversible and direct-acting oral antagonist of the adenosine diphosphate receptor Trials. ~5% have left main stenosis. 1445 A. Clopidogrel D. 5% B. Brain natriuretic peptides B. 2016 A. Coronary B. which of the following was prone to serious bleeding ? 1264 Minor troponin elevations can be caused by ? N Engl J Med 2009. Retrosternal B. Congestive heart failure B. AJAY MATHUR Cardiology 1260 Among patients with UA/NSTEMI. Plaque rupture C. 10 % A. C-reactive protein C. 3 hours C. clopidogrel in ACS patients without increasing the risk of total bleeding. 2016 used by the patient within the previous ? A. Carotid C. 30 % B. Prior stroke or transient ischemic attack B. . 30% have two-vessel disease. 15% D. 40 % C-reactive protein is a marker of vascular inflammation and B-type natriuretic peptide is a marker of increased myocardial wall tension. B-type natriuretic peptide C. Peripheral artery disease A. Vascular inflammation A. 40% have single- vessel disease. 12 hours Among patients with UA/NSTEMI studied at angiography. Other cardiac patients with prior stroke or transient ischemic attack. biomarkers C-reactive protein. Pulmonary embolism D. Increased myocardial wall tension B.206 MCQ’s FOR MEDICAL PROFESSIONALS BY PROF. 2017 A. 2016 Harrison’s 18th Ed. B-type natriuretic peptide & CD-40 ligand correlate independently with increased mortality & recurrent cardiac events in patients presenting with UA/NSTEMI. All of the above Thienopyridine drug prasugrel has a more rapid onset. Suprasternal C. Substernal A. CD-40 ligand D. three subgroups appeared to be particularly prone to serious bleeding - Minor troponin elevations are due to congestive heart failure. 24 hours have three-vessel coronary artery disease. clopidogrel. All of the above D. 2016 Harrison’s 18th Ed. that radiates to the neck. what percentage would have 1266 B-type natriuretic peptide is a marker of ? single-vessel coronary artery disease ? Harrison’s 17th Ed. Elderly A. Clopidogrel B. 2016 A. left shoulder. and patients with a previous stroke or transient ischemic attack. Myocarditis C. Underweight B. 2019 Harrison’s 16th Ed. All of the above D. 1273 Laboratory test used to measure the antiplatelet effects of aspirin is ? Chapter 245. Mortality is approximately fourfold higher in elderly patients (over age 75) compared with younger patients. Diffuse spasm of an intramyocardial coronary artery Described by Prinzmetal in 1959. epinephrine and serotonin promote platelet activation. ~ 10 % 1274 Prinzmetal’s variant angina is due to ? Harrison’s 18th Ed. Raynaud’s phenomenon (NSTEMI) ultimately develop Q-wave on ECG (QwMI) C. D. Of patients with ST-segment elevation. ST-Segment Elevation A. ~ 20 % A. 2019 plaque include all except ? A. Serotonin establish the diagnosis of Prinzmetal’s variant angina. ADP. Epinephrine Ergonovine. 2019 B. Heparin C. and it may occur at one or more sites in D. Rich lipid core and thin fibrous cap C. Bivalirudin D. leukotrienes. Right coronary artery Harrison’s 18th Ed. Harrison’s 18th Ed. D. Majority of patients without ST-segment elevation B. Nitrates and Ca++ channel blockers are main treatments Harrison’s 18th Ed. or serotonin. Poor lipid core and thin fibrous cap Focal spasm is most common in right coronary artery. 2019 . has also been found to be of value in some patients. 2022 B. Migraine B. Focal spasm of an intramyocardial coronary artery D. ~ 30 % B. Aspirin may increase the severity of ischemic episodes C. Skin bleeding time C. Acetylcholine A. 2022 B. Ca++ channel blockers given in maximally tolerated doses A. there are production and release of thromboxane A2 (a potent local vasoconstrictor). ~ 40 % C. and potential resistance to thrombolysis. ATP D. Optical platelet aggregation Myocardial Infarction B. Poor lipid core and thick fibrous cap one artery or in multiple arteries simultaneously. 2019 develop Q-wave on ECG (QwMI) A. 2021 A. Bivalirudin is a direct thrombin inhibitor. 2021. Harrison’s 18th Ed. Enoxaparin is a blocker. of ischemic episodes. All of the above C. AJAY MATHUR Cardiology 207 1272 Which of the following is a direct thrombin inhibitor ? A. 207 Cardiology MCQ’s FOR MCQ’s FOR MEDICAL MEDICAL PROFESSIONALS PROFESSIONALS BY PROF. Harrison’s 18th Ed. Hyperventilation B. the majority ultimately develop a Q-wave on the 1276 In Prinzmetal’s variant angina. The response to beta blockers is variable. It is used to D. Enoxaparin Calcium antagonists are extremely effective in preventing coronary artery spasm of variant angina & they should be prescribed in maximally tolerated doses. Collagen C. All of the above Harrison’s 18th Ed. After an initial platelet monolayer forms at the site of the ruptured plaque. Prinzmetal’s variant angina is due to focal spasm of an epicardial 1280 Which of the following statements about acute coronary coronary artery. a selective alpha adrenoreceptor Fondaparinux is a Factor Xa inhibitor. Prazosin is of no therapeutic value B. leading to severe myocardial ischemia. Left circumflex coronary artery B. Prazosin. angina ? further platelet activation. Majority of patients presenting with NSTEMI do not develop a Q-wave on the ECG. various agonists like collagen. Figure 245-1 1275 Prinzmetal’s variant angina may be associated with ? A. Ergonovine Harrison’s 18th Ed. It may be related to hypercontractility of syndromes is false ? vascular smooth muscle due to vasoconstrictor mitogens. acetylcholine & hyperventilation can provoke focal coronary stenosis. Raynaud’s phenomenon or aspirin-induced asthma. 2019 B. focal spasm is most common in ? ECG (QwMI). Aspirin-induced asthma C. After agonist stimulation 1278 Which of the following is false regarding Prinzmetal’s variant of platelets. Majority of patients with ST-segment elevation ultimately Harrison’s 18th Ed. Diffuse spasm of an epicardial coronary artery Early (30-day) mortality rate from AMI is ~30%. while aspirin may increase the severity low molecular weight heparin (LMWH). spasm in Prinzmetal’s variant angina ? 1282 Agonists that promote platelet activation at the site of ruptured Harrison’s 18th Ed. Fondaparinux Nitrates & calcium channel blockers are the main treatments for Prinzmetal’s variant angina. Mortality due to AMI is fourfold higher in elderly (>75 years) D. Rich lipid core and thick fibrous cap D. Focal spasm of an epicardial coronary artery C. coronary plaques prone to disruption are those with ? A. None of the above Prinzmetal’s variant angina is a manifestation of a vasospastic disorder and is associated with migraine. with more than half of these deaths occurring before the patient reaches hospital. Histologic studies indicate that the coronary plaques prone to disruption are those with a rich 1277 Which of the following can be used to provoke focal coronary lipid core and a thin fibrous cap. Posterior descending coronary artery C. Urinary 11-dehydrothromboxane B2 1279 Early (30-day) mortality rate from acute MI is ? D. 2019 1281 Histologically. All of the above as compared with younger patients D. Left anterior descending coronary artery A. 2022 subside with cessation of activity. Anterior infarction C. Adhesive protein in diabetes mellitus. Harrison’s 18th Ed. STEMI may present as sudden-onset B. Combination of substernal chest pain lasting >30 minutes & diaphoresis suggests STEMI. It is similar in character to discomfort of angina pectoris but is more severe. Collagen vascular disease common in ? Harrison’s 18th Ed. Does not radiate below umbilicus A. First few hours to 3 days Pain is the most common presenting complaint in STEMI. circadian variations have been reported such that clusters are seen in the morning within a few hours of awakening. Attention to time myocardial infarction ? Harrison’s 18th Ed. 3 to 10 days B. C. The weak link B. within a few hours after awakening pressure declines by how much from the preinfarction state ? Harrison’s 18th Ed. Late evening. Lateral infarction increases risk of developing STEMI ? Harrison’s 18th Ed. 10 to 21 days D. in contrast to angina pectoris. 2022 1290 Parasympathetic nervous system hyperactivity is more A. AJAY MATHUR Cardiology 1283 Which of the following best describes thromboxane A2 ? Pain of STEMI may radiate as high as occipital area but not below umbilicus.15 mm Hg Although STEMI may commence at any time of the day or night. 2022 A. Typically. Local vasoconstrictor breathlessness. Alcohol abuse C. Painless STEMIs is greater in diabetics D. following return from work B. Chain of survival A. First few hours D. and it increases with age. systolic B. 2023 Harrison’s 18th Ed. Inferior infarction D. 2022 C. Resistance to fibrinolysis 1289 Sympathetic nervous system hyperactivity is more common in ? D. 25% patients with anterior infarction have manifestations of sympathetic nervous system hyperactivity (tachycardia and/or hypertension). and intracardiac thrombi or masses that can produce coronary emboli.208 MCQ’s FOR MEDICAL PROFESSIONALS BY PROF. ~ 20 . lasts longer and does not C. epinephrine. ~ 10 . A. 7 to 15 days C.25 mm Hg 1287 Which of the following is false regarding ‘pain’ in STEMI ? Harrison’s 18th Ed. Unstable angina A. 2022 A. Decreased intensity of S1 D. few hours before awakening 1292 In most patients with transmural myocardial infarction. which is the least likely condition that D. In elderly. collagen vascular disease. ADP. May radiate to trapezius B. Radiation of discomfort to trapezius is not seen in patients with STEMI & distinguishes it from pericarditis. 2023 C. Pain does not Harrison’s 18th Ed. Harrison’s 18th Ed. ~ 15 . serotonin promote A. 2022 In most patients with transmural myocardial infarction. and up to one-half with inferior infarction show parasympathetic 1285 Which of the following best relates to management of STEMI ? hyperactivity (bradycardia and/or hypotension). C. systolic pressure declines by ~ 10 . pain involves central portion of chest and/ or epigastrium & on occasion it radiates to arms. Deep and visceral 1293 In temporal staging of myocardial infarction. Less common underlying medical D. healing stage is ? Harrison’s 18th Ed. Paradoxical splitting of S2 Management of STEMI aims at providing expeditious implementation of a reperfusion strategy through a highly integrated system termed as “chain of survival” which involves prehospital care C.20 mm Hg D. The linking theory 1291 Which of the following is not an auscultatory finding in B. Pain does not subside with cessation of activity C. cocaine abuse. May radiate as high as occipital area B. Pain is deep & visceral. 2022 B. Painless STEMIs is more A. 2022 C. First few hours to 7 days 1288 Which of the following is false regarding ‘pain’ in STEMI ? 1294 In temporal staging of myocardial infarction. Different in character to discomfort of angina pectoris A. D. Cocaine abuse B. Most common presenting complaint B. agonists like collagen. ~ 5 . Early systolic apical systolic murmur that extends to early hospital management. D.15 mmHg from the preinfarction state. 7 to 28 days . 2022 A. C. Posterior infarction Patients at increased risk of developing STEMI include those with multiple coronary risk factors and those with unstable angina or Prinzmetal’s variant angina. Morning. After meals A.10 mm Hg D. Early morning. acute stage is ? Harrison’s 18th Ed.a potent local vasoconstrictor is released B. 2022 At the site of disrupted plaque. First few hours to 5 days usually subside with cessation of activity. Anterior infarction platelet activation and from them thromboxane A 2 . which may progress to pulmonary edema. Pericardial friction rub 1286 STEMI tends to occur most commonly at ? A transient midsystolic or late systolic apical systolic murmur due to dysfunction of the mitral valve apparatus may be present in myocardial infarction. Lateral infarction conditions predisposing patients to STEMI include hypercoagulability. Inferior infarction and potential resistance to fibrinolysis develops. Posterior infarction 1284 Out of the following. Platelet aggregator Harrison’s 18th Ed. for the first 6 to 12 hours after infarction and then according to the situation. 2024 D. None of the above C. All of the above a higher peak value.2 1298 Cardiac biomarkers that are released into the interstitium are first cleared by ? B. stroke. Blood levels return to normal within 24 hours A. Are detected sooner Harrison’s 18th Ed. O2 is administered by nasal prongs or face mask at the rate of ? 1300 Which of the following is true for levels of creatine Harrison’s 18th Ed.4 L/minute CK rises within 4 to 8 hours and generally returns to normal by 48 to 72 hours. 1302 Potential sources of total CK elevation are all except ? Harrison’s 18th Ed. >=1. supplemental O 2 is of limited clinical benefit. D. Figure 245-2 C. It is rapidly excreted in urine therefore blood levels return to normal range within 24 hours of the onset of myocardial infarction. 4 . Transmurality of myocardial infarction C. >=2. surgery. None of the above 1303 Potential sources of total CK elevation are all except ? Harrison’s 16th Ed. High cardiac specificity phosphokinase (CK) in STEMI ? C. >=2. Rises within 10 to 30 minutes Myoglobin is one of the first serum cardiac markers that rises after STEMI but it lacks cardiac B. Returns to normal by 24 to 48 hours C. Returns to normal by 48 to 72 hours D. 6 . skeletal C. Prolonged immobilization A. 209 Cardiology MCQ’s FOR MCQ’s FOR MEDICAL MEDICAL PROFESSIONALS PROFESSIONALS BY PROF. 7 to 10 days muscle damage secondary to trauma. When hypoxemia is present. Capillaries skeletal muscle source for the CKMB elevation. 10 to 21 days 1304 What ratio (relative index) of CKMB mass:CK activity suggests Levels of cardiac-specific troponin T (cTnT) and cardiac-specific troponin I (cTnI) detected by myocardial infarction ? highly specific monoclonal antibodies may remain elevated for 7 to 10 days after STEMI. healing (7 to 28 days) and healed (>=29 days). Detected in serum C. Myopathies A. electrical cardioversion. 2025 phosphokinase (CK) in STEMI ? A. 2023 B. Hypothyroidism days after STEMI ? C. Polymyositis B. hypothyroidism. 1451 Serum cardiac biomarkers are proteins released from necrotic heart muscle after STEMI. 1 to 3 days Potential sources of total CK elevation are skeletal muscular diseases (muscular dystrophy. which of the 7 days). Rises within 4 to 8 hours 1306 In acute MI patients with hypoxemia. cardiac biomarkers are detected sooner. . Lymphatics 1305 Which of the following is false about myoglobin in STEMI ? Cardiac biomarkers that are released into the interstitium are first cleared by lymphatics Harrison’s 18th Ed. but decline more rapidly. A. C. Rise to a higher peak value A. First serum cardiac markers to rise after STEMI 1299 Which of the following is true for levels of creatine B. Returns to normal within 24 hours B. whose arterial O 2 saturation is normal. convulsions and prolonged immobilization. Thickness of the opposite myocardial wall When patients with STEMI undergo reperfusion.8 Harrison’s 18th Ed. A. Rises within 1 to 4 hours D. 2 to 5 days myopathies and polymyositis). Rises within 30 minutes to 1 hour specificity. 2023. AJAY MATHUR Cardiology 209 Myocardial infarction (MI) progresses through three temporal stages . >=1.4 L/minute Harrison’s 18th Ed. O 2 is given by nasal prongs or face mask @ 2 . 2023 D. 2024 B. Contemporary studies using MRI suggest that the development of a Q wave on the ECG is more dependent on the volume of infarcted myocardial tissue rather than transmurality of infarction. 2024 1296 Which of the following about cardiac biomarkers is false ? A.5 “suggests” myocardial rather than a C. Muscular dystrophy Harrison’s 18th Ed. 8 .10 L/minute D. Released from necrotic heart muscle D.2 A. 2024 A. D. rise to D.acute (first few hours to 1301 When patients with STEMI undergo reperfusion. Volume of infarcted myocardial tissue D. Cardiac biomarkers are proteins D. following is false about cardiac biomarkers ? Harrison’s 18th Ed. B. Hyperthyroidism C. Alcohol binge 1297 Levels of cTnI and cTnT may remain elevated for how many B. 2023 B. 2 . Rapidly excreted in urine Harrison’s 18th Ed. Returns to normal by 72 to 96 hours In STEMI patients. 2024 1295 Development of a Q wave on ECG is more dependent on ? A. Decline more rapidly B.5 B. Stroke Harrison’s 18th Ed.8 L/minute C. Harrison’s 18th Ed.6 L/minute A. Venules A ratio (relative index) of CKMB mass : CK activity >=2. 2024 followed subsequently by spillover into the venous system. Arterioles D. third of patients with STEMI may achieve spontaneous reperfusion of the infarct- C. Three hours 1314 In STEMI. D. IV Atropine C. fibrinolytic therapy should ideally be initiated within ? A. These side effects respond to atropine. Useful in the control of pain of STEMI center” if it does how many percuteneous coronary intervention B. three doses of sublingual nitroglycerin (0.4 mg) C. because D. the golden hour refers to ? D. 2027 D. >=75 absorption may be unpredictable by subcutaneous route.4 mg) rather than by subcutaneous administration of a larger quantity.half A. All of the above A. 2026. First 180 minutes A. 60 minutes promptly by ? C. Intravenous beta blockers reduce risks of reinfarction & ventricular fibrillation. the goal is to keep total ischemic time within ? Sublingual nitroglycerin can be given safely to most patients with STEMI. Glucocorticoids and nonsteroidal anti-inflammatory agents (other than aspirin). D.third Harrison’s 18th Ed. A. 90 minutes Harrison’s 18th Ed. within 24 hours ? Harrison’s 18th Ed. B. Harrison’s 18th Ed. None of the above A. C. 2026 1318 In STEMI. >=25 D. B. 30 minutes 1309 Idiosyncratic hypotensive reaction to nitrates is reversed B. 2025 C. >=36 1312 What proportion of patients with STEMI may achieve spontaneous reperfusion of infarct-related coronary artery A cardiological unit is considered as a “dedicated medical center” if it does >=36 percuteneous coronary intervention (PCI) cases per year. the goal is to keep total ischemic time within 120 minutes. >=5 D. 2027 C. Up to three doses of Harrison’s 18th Ed. consisting of sudden marked hypotension can be reversed A. IV Fluids Harrison’s 18th Ed. >=50 Morphine is routinely administered by repetitive (every 5 min) intravenous injection of small doses (2 . One . 2025 D. Glucocorticoids hours since it may potentiate the hypotensive effects of nitrates. First 15 minutes promptly by the prompt administration of intravenous atropine. One . B. First 60 minutes Harrison’s 18th Ed. Shoule be given IV rather than subcutaneously Harrison’s 18th Ed. One .fourth Harrison’s 18th Ed. B.4 mg should be administered at about 5 minute intervals. 2026 elevated jugular venous pressure. 120 minutes A. AJAY MATHUR Cardiology 1307 Therapy with nitrates in STEMI should be avoided in ? B. 2025 D. 2025 D. 2025 1317 A cardiological unit is considered as a “dedicated medical A. Figure 245-4 An idiosyncratic reaction to nitrates. >=25 Morphine has vagotonic effect & may cause bradycardia or advanced degrees of heart block particularly in patients with posteroinferior infarction. None of the above B. 2027 . >=15 Intravenous beta blockers are useful in the control of pain of STEMI by diminishing myocardial O2 demand & ischemia.210 MCQ’s FOR MEDICAL PROFESSIONALS BY PROF. and their use may result in a larger infarct scar. May cause advanced heart block particularly in patients he has does how many percuteneous coronary intervention with anterior wall infarction (PCI) cases per year ? C. thereby potentially reducing flow to ischemic myocardium. Short-acting dihydropyridines calcium antagonists should be administered at an interval of ? Harrison’s 18th Ed. >=15 B. Nonsteroidal anti-inflammatory agents 1308 In STEMI. These also increase coronary vascular C.third B. Low systolic arterial pressure (< 90 mm Hg) D. clear lungs & hypotension). A cardiologist is considered as an “experienced operator” if he does >=75 percuteneous coronary intervention (PCI) cases per year. Reduce the risks of ventricular fibrillation A. Morphine has a vagotonic effect 1316 A cardiologist is considered as an “experienced operator” if B. First 30 minutes 1310 Which of the following about morphine in acute MI is false ? C. 2026. 30 minutes should be avoided in STEMI as they can impair infarct healing & increase the risk of myocardial rupture. Two . Figure 245-4 0. When there is clinical suspicion of RV infarction Up to one . All of the above 1313 Which of the following drugs should be avoided in patients Therapy with nitrates is avoided in MI patients who have low systolic arterial pressure (<90 with STEMI ? mmHg) or in whom there is clinical suspicion of RV infarction (inferior infarction on ECG. 1311 Which of the following about IV -blockers in acute MI is false ? Harrison’s 18th Ed. 5 minutes Short-acting dihydropyridine calcium antagonists may be associated with an increased mortality risk in STEMI. IV Norepinephrine 1315 In STEMI. Reduce the risks of reinfarction (PCI) cases per year ? Harrison’s 18th Ed. Who have taken ‘sildenafil’ within preceding 24 hours related coronary artery within 24 hours. Nitrates should not be administered to patients who have taken phosphodiesterase-5 inhibitor sildenafil within the preceding 24 A. C. IV Calcium gluconate In STEMI. One hour resistance. C. 211 Cardiology MCQ’s FOR MCQ’s FOR MEDICAL MEDICAL PROFESSIONALS PROFESSIONALS BY PROF.complete occlusion of infarct-related IIb/IIIa inhibitor with a reduced dose of a fibrinolytic agent. Percutaneous coronary intervention (PCI) Qualitative scale to angiographically assess flow in the culprit coronary artery is the thrombolysis Combination reperfusion regimens for coronary reperfusion combine an intravenous glycoprotein in myocardial infarction (TIMI) grading system. streptokinase. grade 1 . Reteplase (rPA) B. Streptokinase (STK) D. Intravenous glycoprotein IIb/IIIa inhibitor D. 2027 D. 30 minutes of presentation determining the rate of entry and exit of contrast dye from microvasculature in myocardial infarct zone (TIMI myocardial perfusion grade). Tissue plasminogen activator (tPA) TNK is given as a “single” weight-based intravenous bolus of 0. Tenecteplase (TNK) C. 1327 Which of the following fibrinolytic agent is administered in a double-bolus regimen ? 1322 Angiographically flow in the culprit coronary artery is assessed Harrison’s 18th Ed. Tissue plasminogen activator (tPA) TIMI grade 3 indicates full perfusion of the infarct vessel with normal flow which is the goal of B. Reteplase (rPA) B. 60 minutes of presentation D. Tissue plasminogen activator (tPA) A. 10 % lyses fibrinthrombi. TIMI 4 A. TIMI frame count B. Streptokinase (STK) D.5 million units When assessed angiographically. Grade 0 . Tissue plasminogen activator (tPA) 1326 Which of the following is given as a “single” weight-based intravenous bolus ? 1321 Which of the following is a ‘bolus fibrinolytic’ agent ? Harrison’s 18th Ed. Tenecteplase (TNK) when administered within the first hour of the onset of symptoms. 2027 Harrison’s 18th Ed. C. Tenecteplase (TNK) 1325 In STEMI. Tenecteplase (TNK) & reteplase (rPA) are bolus fibrinolytics since their administration does not require a prolonged intravenous infusion. Clopidogrel C. Benefit is maintained for at least 10 years. Tenecteplase (TNK) C. Streptokinase reperfusion therapy. TIMI 1 B. flow in the culprit coronary artery is described by a simple (MU) intravenously over 1 hour.perfusion of entire infarct vessel into distal bed. All of the above D. Urokinase D. TIMI flow grade A. 50 % Harrison’s 18th Ed. fibrinolytic therapy can reduce the relative risk of in- D. 10-MU bolus given over 2 . 75 % A. 2027 A. 2027 A. but with flow that is delayed compared with that of a normal artery and grade 3 . A. which subsequently A. fibrinolytic therapy should ideally be initiated within 30 minutes of presentation (door- Harrison’s 18th Ed. 10 minutes of presentation of efficacy of fibrinolysis include counting the number of frames on cine film required for dye to flow from the origin of the IRA to a landmark in distal vascular bed (TIMI frame count) and B. 2027 to-needle time 30 minutes) with a principal goal to promptly restore full coronary arterial patency (TIMI grade 3). 2027 Tissue plasminogen activator (tPA). rPA is administered in a double-bolus regimen consisting of a qualitative scale called the thrombolysis in myocardial infarction (TIMI) grading system. 2027 A. All of the above hospital death by up to ? Harrison’s 18th Ed.some penetration of contrast material beyond the point of obstruction but without perfusion of distal coronary bed. TIMI 3 Harrison’s 18th Ed. TIMI 2 1319 Which of the following fibrinolytic agent acts by promoting conversion of plasminogen to plasmin ? C.3 minutes. CAPRIE tPA is given as a 15 mg bolus followed by 50 mg intravenously over the first 30 minutes. 1323 Method of angiographic assessment of the efficacy of 1328 Combination reperfusion regimens involve giving a fibrinolysis is ? fibrinolytic agent with which of the following ? Harrison’s 18th Ed. Additional methods of angiographic assessment except ? . Streptokinase B. B. tenecteplase (TNK). Streptokinase In STEMI. CURE D. Urokinase D. ISIS C.53 mg/kg over 10 seconds. TIMI myocardial perfusion grade C. Streptokinase is administered as 1. Reteplase (rPA) C. C. artery (IRA). AJAY MATHUR Cardiology 211 A. grade 2 . 90 minutes of presentation 1324 Which TIMI flow grade indicates full perfusion of infarct vessel with normal flow following reperfusion therapy ? In STEMI. followed by 35 mg over the next 60 minutes. fibrinolytic therapy can reduce the relative risk of in-hospital death by up to 50% B. Aspirin B. TIMI B. 2027 by which of the following grading system ? Harrison’s 18th Ed. 2027 D.full 1329 Clear contraindications to the use of fibrinolytic agents are all perfusion of the infarct vessel with normal flow. followed by a second 10-MU bolus 30 minutes later. and reteplase (rPA) are fibrinolytic agents that act by promoting conversion of plasminogen to plasmin. 25 % 1320 Which of the following is a ‘bolus fibrinolytic’ agent ? C. 2027 Harrison’s 18th Ed. Tissue plasminogen activator (tPA) A. A. Atropine C. AJAY MATHUR Cardiology Harrison’s 18th Ed. 21 % . patients should be encouraged. diet rich in which of the following is recommended ? B.0. By day 3 after MI. 48 hours Harrison’s 18th Ed. 5 days to 2 years Harrison’s 18th Ed. To maintain patency of infarct-related artery (IRA) C. a hemorrhagic ophthalmic condition (hemorrhagic diabetic retinopathy).212 MCQ’s FOR MEDICAL PROFESSIONALS BY PROF. Active internal bleeding 1335 Patients with confirmed STEMI and low risk may be safely transferred out of the coronary care unit within ? Clear contraindications to the use of fibrinolytic agents include a history of cerebrovascular Harrison’s 18th Ed. CHF. D. H 2 blockers & narcotics can produce delirium. Magnesium Because of the risk of an allergic reaction. 1.5 . D. 12 hours ulcer disease. Potassium D. particularly in the elderly. occurs in ~0. 12 hours 1330 In STEMI. a recent (<2 weeks) invasive or surgical B. All of the above 1333 Minor degree of hypotension occurs in what percentage of Atropine. 2028 reduction in mortality of how much in patients with MI receiving antiplatelet agents ? A. To reduce likelihood of deep venous thrombosis Minor hypotension occurs in 4 . the most serious complication.10 % agents is ? Harrison’s 18th Ed. foods that are high in potassium.1. D. 10 .5 . 3% B. 0.40 % C. Pregnancy C. Hemorrhagic diabetic retinopathy 1336 Patients with STEMI should be kept at bed rest for the first ? Harrison’s 18th Ed.5 . active peptic C. Cerebrovascular hemorrhage at any time Hemorrhagic stroke.1 . In the absence of complications.9 % A.30 % B. 2028 D. known bleeding diathesis. which of the following is a relative contraindication C. percentage of patients who receive it ? Harrison’s 18th Ed. but low in sodium are recommended. under supervision. 2028 should increase their ambulation progressively to a goal of 185 meters (600 ft) three times a day.0. goal of treatment with antiplatelet and anticoagulant A. 2028 1339 In STEMI. Suspicion of aortic dissection D. pregnancy. which require clinical consideration of risk:benefit ratio include current use of anticoagulants (INR >=2).4 % A. 6 hours aortic dissection.5 % B. Narcotics Allergic reactions to streptokinase occur in ~2% of patients who receive it. 2028 A. All of the above 1334 Hemorrhagic stroke occurs in about what percentage of 1340 Data from Antiplatelet Trialists’ Collaboration shows a relative patients who receive streptokinase ? Harrison’s 18th Ed. 2029 B. streptokinase should not given. marked hypotension occurs rarely. suspicion of A. patients who receive streptokinase ? Harrison’s 18th Ed. 24 hours to fibrinolytic therapy ? D. Fiber agent had been received within the preceding 5 days to 2 years.9% of patients receiving B. 24 hours 1331 For what period. 12 % C. All of the above 1332 Allergic reactions to streptokinase occur in about what In STEMI. patients should not receive streptokinase if that C. 2029 C. To reduce likelihood of mural thrombus formation D. B.10 % of patients given STK. 4 .0. hypotension or cardiac arrhythmias) may be safely transferred out of CCU within 24 hours. 16 . 2% A. H2 blockers D. 1 to 2 years A.2. Nonhemorrhagic stroke at any time fibrinolytic agents. C.5 % Harrison’s 18th Ed. to resume an upright posture by dangling their has received it in past ? feet over the side of the bed & sitting in a chair within the first 24 hours. particularly in the elderly ? Harrison’s 18th Ed. and active internal bleeding (excluding menses). Never in life 1337 In STEMI. 2028 1338 Which of the following drugs used in coronary care unit can A. 2029 B. 0. 3 to 5 years B. Rate of intracranial hemorrhage with tPA or rPA is slightly higher than with STK. D. 3 hours Relative contraindications to fibrinolytic therapy. and fiber. 2029 D. marked hypertension (>180/110 mmHg) at any time during acute presentation. magnesium.9 . B. a nonhemorrhagic stroke or other cerebrovascular event within the past year. 4% C. 30 . 1% produce delirium. 6 hours procedure or prolonged (>10 minute) cardiopulmonary resuscitation. patients Harrison’s 18th Ed. 0. All of the above A. 2029 hemorrhage at any time. Current use of anticoagulants (INR >=2) Patients with confirmed STEMI & low risk (no prior MI & no persistent chest discomfort. 2029 B. and a history of severe hypertension that is currently adequately controlled.16 % A. if the patient Patients with STEMI should be at bed rest for the first 12 hours. 2 times the control value. Lateral A. 4000 U A. 2030 C. S3 gallop. pulmonary edema and class IV . 1456 D. 20 % B. In-hospital mortality rate is >50%. About 30 % to UFH for anticoagulation ? C. history of embolism.48 hours after the onset of myocardial infarction ? Harrison’s 18th Ed. 2500 U Harrison’s 18th Ed. All of the above Harrison’s 16th Ed. or signs of right heart failure. Angiotensin-converting enzyme (ACE) inhibitors D. 1613 Advantages of low-molecular-weight heparin (LMWH) preparations include high bioavailability A. 5% time during maintenance therapy should be ? Harrison’s 18th Ed. severe LV dysfunction. 1000 U / hour 1348 In STEMI. 5000 U B. AJAY MATHUR Cardiology 213 C. 2031 thromboplastin time during maintenance therapy should be 1. B. activated partial thromboplastin B. 34 % C. 7500 U C. All of the above first 24 . with the use of UFH. Generally associated with mortality rate of >50% Harrison’s 18th Ed. what proportion of patients present with cardiogenic C. About 20 % 1344 In STEMI. peripheral vasoconstriction. 1500 U / hour shock on admission ? D. Killip class IV 1342 In STEMI. 2000 U / hour Harrison’s 18th Ed. 1457 .5 times the control value D.5 . 500 U / hour <90 mmHg. mental confusion. Sustained systolic arterial pressure of <60 mmHg permitting administration subcutaneously. About 35 % Harrison’s 18th Ed. Killip class II D.2% in control patients to 10. 1% 1343 In STEMI. C. class II . B. Infarction of >=40% of left ventricle results in cardiogenic shock. Killip class III D. 1341 In STEMI. S 3 gallop. C. 27 % B. which of the following can be used as an alternative B.3 times the control value D. About 40 % A. Unfractionated heparin (UFH) mortality rate. 3 . 1. Intravenous nitroglycerin Harrison’s 16th Ed. 2031 A. the maximum recommended dose of UFH given as an initial bolus is ? 1347 Patients with rales at lung bases. Associated with occlusion of proximal RCA Patients with anterior myocardial infarction. peripheral cyanosis. B.shock with systolic pressure A.severe heart failure. A.rales at lung bases. heart failure.2 L/(min/m2) C. the recommended dose of UFH is an initial bolus of 60 U/kg (maximum 4000 U) cardiogenic shock ? followed by an initial infusion of 12 U/kg per hour (maximum 1000 U/hour). ST-segment elevation of >1 mm in lead V4R risk of systemic or pulmonary thromboembolism. class III . 2030 D.2 (L/min)/m 2 ] & sustained systolic arterial hypotension (<90 mmHg). and oliguria. Posterior 1351 What is true for right ventricular myocardial infarction ? Harrison’s 16th Ed. Class I . and greater antiXa:IIa activity. Upright T wave in lead V4R during initial hours 1346 Which of the following medication is least useful during the D. or signs Harrison’s 18th Ed.2 times the control value Only 10% of patients with STEMI present with cardiogenic shock on admission. Fondaparinux Hemodynamic evidence of abnormal LV function appears when contraction is seriously impaired B. Activated partial Harrison’s 18th Ed. from 14.4% in patients receiving antiplatelet agents. 2030 congestion. Benefits of routine use of intravenous nitroglycerin are less in contemporary era where beta- adrenoceptor blockers & ACE inhibitors are routinely prescribed for patients with STEMI. the maximum recommended dose of UFH given after the initial bolus is ? Killip classification divides patients into four groups. 2030 1352 Ventricular tachycardia or fibrillation refractory to electroshock is more responsive after patient is treated with ? A.no signs of pulmonary or venous Harrison’s 18th Ed.5 times the control value 1349 Infarction of what percentage of left ventricle results in In STEMI. 2D echocardiographic evidence of mural thrombus or atrial fibrillation are at increased B. Inferior C. 90% develop it during hospitalization. Elevated pulmonary capillary wedge pressure (>18 mmHg) 1345 Which of the undermentioned myocardial infarction has an increased risk of systemic or pulmonary thromboembolism ? D. Bivalirudin in 20 to 25% of the left ventricle. Cardiac index < 2. tachypnea.3.5 . 2031 B. 213 Cardiology MCQ’s FOR MCQ’s FOR MEDICAL MEDICAL PROFESSIONALS PROFESSIONALS BY PROF. 2030 of right heart failure belong to which Killip class ? A. reliable anticoagulation without monitoring. 2. 10 % A.5 . C. Beta blockers Data from Antiplatelet Trialists’ Collaboration patients with MI showed a relative reduction of 27% in D. Killip class I C. 2030 Cardiogenic shock (CS) is characterized by systemic hypoperfusion due to severe depression A. Anterior of cardiac index [<2. tachypnea. despite an elevated filling pressure (PCWP > 18 mmHg). 1. Enoxaparin 1350 Which of the following is not true for cardiogenic shock ? D. V2. 1542 N Engl J Med 2003. steerable guidewire put into coronary artery lumen has a diameter of ? A. It is even higher in C. Christian Bernard 1353 Implantable cardioverter/defibrillator (ICD) is recommended In 1977.348:933-40 B. 1544 B. AJAY MATHUR Cardiology A. D. Aggressive Treatment Panel A. Adult Treatment Panel 1363 Proximal occlusion of the left anterior descending artery is indicated by ? B. Greater ST-segment elevation in lead II & III than in lead 1355 Usual duration of hospitalization for an uncomplicated STEMI aVF & ST-segment depression of >1 mm in leads I & aVL is about ? Harrison’s 18th Ed. < 1 mm insertion of an implantable cardioverter/defibrillator (ICD) if the LVEF is <30–40% with NYHA class II-III or if LVEF is <30–35% with NYHA class I functional status. and V3 D. Anterior wall of left ventricle Usual duration of hospitalization for an uncomplicated STEMI is about 5 days. < 0. More frequent in those who have successful fibrinolysis ST-segment depression of >1 mm in leads I & aVL C. ST elevation in V1.4 mm. ST elevation in V1. Atrial infarction 1356 Intravenous antiplatelet drugs include ? 1362 Most sensitive electrocardiographic sign of right ventricular Harrison’s 16th Ed. Repeat fibrinolysis is an alternative B. steerable guidewire put into coronary artery lumen has a diameter of < 0. Abciximab N Engl J Med 2003. 1360 What suggests involvement of right coronary artery rather than 1354 Which of the following is false about recurrent post STEMI angina ? left circumflex artery as the culprit vessel in inferior MI ? Harrison’s 17th Ed.214 MCQ’s FOR MEDICAL PROFESSIONALS BY PROF.348:933-40 C. Patients with preserved left During PCI. Anterior wall MI. Greater ST-segment elevation in lead aVF than in lead II or patients who undergo successful fibrinolysis. Andreas Gruntzig introduced percutaneous transluminal coronary angioplasty (PTCA). 10 days C. bretylium (5-mg/kg bolus) or amiodarone (75 to 150-mg bolus). Figure 245-6 1359 During percutaneous coronary intervention (PCI). All of the above B. Develops in ~25% of patients hospitalized for STEMI A. Eptifibatide A. LVEF is <30-40% and NYHA class II Harrison’s 17th Ed. LVEF is <30-35% and NYHA class I B. Eptifibatide (synthetic cyclic heptapeptide C. ST-segment elevation of >1 mm in V 4R with upright T wave with a KGD sequence) and Tirofiban (synthetic peptidomimetic based on the RGD sequence). 2033. ST-segment elevation of >1 mm in V 4R with inverted T wave during first 12 hours Three potent parenteral GpIIb/IIIa inhibitors are Abciximab (chimeric monoclonal Fab fragment of human & murine protein that binds to GpIIb/IIIa). 7 days B. 5 days A. 2034 1361 In inferior MI.4 mm C. Repeat administration of a fibrinolytic agent is an III & ST-segment depression of >1 mm in leads I & aVL alternative to early mechanical revascularization.348:933-40 B. LVEF is <30-40% and NYHA class III A. 1544 C. after first 12 hours 1357 ATP in “National Cholesterol Education Project ATP III” stands D. < 0. ST-segment depression in leads V1 and V2 suggest A.348:933-40 A. Amiodarone A. 688 infarction is ? A.6 mm D. ST-segment elevation of >1 mm in V 4R with upright T wave C. Posterior wall of left ventricle D. Lateral wall of left ventricle C.8 mm Patients with depressed left ventricular function at least 40 days post-STEMI are referred for D.000 solution via the intracardiac route). Micheal DeBakey after patient is treated with epinephrine (1 mg intravenously or 10 mL of a 1:10. < 0. ST-segment elevation of >1 mm in V 4R with inverted T for ? wave after first 12 hours Harrison’s 16th Ed. ventricular function (LVEF >40%) do not receive an ICD regardless of NYHA functional class. Angina Treatment Panel B. Greater ST-segment elevation in lead II than in lead III & B. Tirofiban during first 12 hours D. . 3 days concomitant infarction of ? N Engl J Med 2003. and V3 & aVL Adult Treatment Panel III (ATP III). None of the above ST-segment depression of >1 mm in leads I & aVL Recurrent angina develops in ~25% of patients hospitalized for STEMI. Advanced Treatment Panel N Engl J Med 2003. D. Greater ST-segment elevation in lead III than in lead II & D. 1430 A. D. Anterior wall MI. Andreas Gruntzig D. for which of the following ? Harrison’s 18th Ed. Denton Cooley Ventricular tachycardia or fibrillation that is refractory to electroshock may be more responsive C. All of the above C. V2. Bretylium angioplasty” (PTCA) was first introduced by ? Harrison’s 17th Ed. All of the above B. Epinephrine 1358 The technique “Percuteneous transluminal coronary B. ~ 65 years artery in 25 % D. after which it tends to decrease ? artery in 50 % Harrison’s 18th Ed. None of the above Hypertension prevalence is related to dietary NaCl intake. Anterior wall MI.8 times D. Supplied by RCA in 60 % of people and by the left circumflex artery in 40 % C. ~ 75 years D. Supplied by RCA in 75 % of people and by left circumflex C. >=160/100 mm Hg B. ST-segment resolution during first 90 minutes Harrison’s 18th Ed. Supplied by RCA in 80 % of people and by left circumflex of >=140 mm Hg and a diastolic pressure below ? artery in 20 % N Engl J Med 2006. Supplied by RCA in 60 % of people and by left circumflex A. T-wave inversion in < 4 hours after MI 1374 High BP before age 55 occurs how many times more frequently among persons with a positive family history of hypertension ? B. Supplied by septal branch of left circumflex artery Probability that a middle-aged / elderly individual will develop hypertension in lifetime is 90%. AJAY MATHUR Cardiology 215 C. III and aVF 1369 According to JNC 7. Supplied by RCA in 90 % of people and by left circumflex artery in 10 % Among adults. D. ~ 55 years C. Supplied by atrioventricular nodal branch of left circumflex artery + septal perforators of LAD D. Supplied by atrioventricular nodal branch of LAD A.8 times D.348:933-40 develop hypertension in his or her lifetime is ? Harrison’s 18th Ed. Supplied by atrioventricular nodal branch of RCA + septal B. Supplied by septal perforators of RCA B. V3 & aVL with ST depression of >1 mm in lead aVF Chapter 247. All of the above B. 2042 B. stage 2 hypertension is ? 1364 Which of the following statements about blood supply of sinus N Engl J Med 2006. 2042 C. 1366 Which of the following statements about blood supply of 1372 The probability that a middle-aged or elderly individual will bundle of His is true ? N Engl J Med 2003. after which it tends to decrease. 1. Supplied by septal perforators of left circumflex artery C. 140-159/90-99 A.348:933-40 A. Supplied by RCA in 40 % of people and by left circumflex D. V2.355:385-92 node is true ? A. 2042 A. 180/110 mm Hg artery in 60 % 1370 Isolated systolic hypertension is defined as a systolic pressure C. & age-related increase of BP is 1368 Which of the following is an electrocardiographic predictor of augmented by high NaCl intake. Low dietary intakes of calcium & potassium contribute to risk myocardial reperfusion following thrombolysis ? of hypertension. Augmented by high NaCl intake B. Accelerated idioventricular rhythm A. 100 mm Hg N Engl J Med 2003. Augmented by low dietary intakes of potassium D. 10 % B. 60 % C.8 times C. Augmented by low dietary intakes of calcium C. 1367 Which of the following statements about blood supply of right 1373 Which of the following statements about age-related increase bundle branch is true ? of blood pressure is false ? N Engl J Med 2003. Supplied by septal perforators of LAD A. N Engl J Med 2003. V3 & aVL with ST Hypertensive Vascular Disease depression of >1 mm in lead II. 90 % D. 30 % perforators of LAD C. 3. diastolic blood pressure increases progressively with age until ~55 years.348:933-40 1371 Among adults. V2. 90 mm Hg atrioventricular node is true ? D. 215 Cardiology MCQ’s FOR MCQ’s FOR MEDICAL MEDICAL PROFESSIONALS PROFESSIONALS BY PROF. diastolic blood pressure increases A.355:385-92 D.348:933-40 B. Anterior wall MI. ST elevation in V1. 70 mm Hg decending artery in 40 % B. 2042 A. ST elevation in V1. None of the above D. 120/80 mm Hg N Engl J Med 2003. 4. 2. 80 mm Hg 1365 Which of the following statements about blood supply of C.8 times .348:933-40 Harrison’s 18th Ed. Supplied by RCA in 60 % of people and by left anterior A. Supplied by RCA in 50 % of people and by left circumflex progressively until what age. ~ 45 years artery in 40 % B. 2042 1381 When activated by catecholamines. Salt-wasting disorders are associated with low blood pressure C.  2-adrenergic receptors 1377 Which of the following statements about genesis of n In kidney. AJAY MATHUR Cardiology High blood pressure before age 55 occurs 3. 2043 C. 1 9 levels D. 2 r D. 2 Alpha adducin gene encodes a cytoskeletal protein important for renal tubular sodium absorption. Nonchloride salts of sodium have little or no effect on blood A. 2043 C. 1378 The autonomic nervous system maintains cardiovascular h homeostasis via which of the following signals ? 1384 Which of the following is false about Renin ? Harrison’s 18th Ed.8 times more frequently among persons with a C. 2 9 C. 1385 Which of the following is not a primary stimuli for renin secretion ? 1379 Which of the following is an endogenous catecholamine ? Harrison’s 18th Ed. R Harrison’s 18th Ed. None of the above The autonomic nervous system maintains cardiovascular homeostasis via pressure. Norepinephrine of Henle B. gene ? Harrison’s 18th Ed. They D. and chemoreceptor signals. It is thought to be associated with increased renal tubular absorption of sodium. D. aldosterone C. Decreased pressure or stretch within renal afferent arteriole The three endogenous catecholamines are norepinephrine. Vascular volume is a primary determinant of arterial pressure relaxes vascular smooth muscle leading to vasodilation ? over the long term Harrison’s 18th Ed. and dopamine. hypertension is false ? Harrison’s 18th Ed. NAT2 gene D. 2043 1383 Activation of which of the following receptors by epinephrine U A. Plasma contains 2 . activation of which of the following increases renal tubular reabsorption of sodium ? V B. Dopamine Henle D. Receptor Tyrosine Kinase renin-secreting cells via beta 1 adrenoreceptors.216 MCQ’s FOR MEDICAL PROFESSIONALS BY PROF. All of the above A. 2043 - B. 2043 A. and sympathetic nervous system stimulation of A. Serine Kinase 1375 Which of the following is considered as a hypertension-related The activities of adrenergic receptors are mediated by guanosine nucleotide-binding regulatory proteins (G proteins) & by intracellular concentrations of downstream second messengers. epinephrine. Sympathetic nervous system stimulation of renin-secreting play important roles in tonic and phasic cardiovascular regulation. Gene encoding  2 adrenoreceptor A.  1-adrenergic receptors synthase. volume. 1 of this gene may be associated with hypertension and salt sensitivity of blood pressure. None of the above i Activation of  2 receptors by epinephrine relaxes vascular smooth muscle & results in vasodilation. 2042 A. B. inhibiting B. activation of 1-adrenergic receptors increases renal tubular reabsorption of sodium. 1 B. Epinephrine B. Gene encoding AT1 receptor 1382 In the kidney. 2 G 1376 Which of the following genes is possibly related to When activated by catecholamines. Alpha-adducin gene receptors act as negative feedback controllers. inhibiting hypertension ? further norepinephrine release. D. cells via beta1 adrenoreceptors 1380 The activities of the adrenergic receptors are mediated by ? Three primary stimuli for renin secretion are decreased NaCl transport in thick ascending limb of loop of Henle (macula densa mechanism). 2-adrenergic receptors ti e Genes possibly related to hypertension include genes encoding the AT 1 receptor. and the  2 adrenoreceptor. Synthesized in renal afferent renal arteriole C. Volume signals B. and variants C. Guanosine nucleotide-binding regulatory proteins positive family history of hypertension. Decreased NaCl transport in thick ascending limb of loop A. 2044 Harrison’s 18th Ed. 2043 Harrison’s 18th Ed. MDR1 gene further norepinephrine release ? Harrison’s 18th Ed. decreased pressure or stretch within renal Harrison’s 18th Ed. 1-adrenergic receptors d D. D. 2043 afferent arteriole (baroreceptor mechanism). All of the above C. which of the following A. Increased NaCl transport in thick ascending limb of loop of C. Cytokine Receptor–Linked Kinase . 2044 ta A. Gene encoding aldosterone synthase Harrison’s 18th Ed. All of the above D. 2 receptors act as negative feedback controllers.5 times more prorenin than renin D. Pressure signals A. 1 pressure B. Renin secretion is inhibited by increased NaCl transport in thick ascending limb of loop of Henle. All of the above B. Prorenin is an enzymatically inactive precursor B. Chemoreceptor signals C. and angiotensin II are also synthesized locally in brain. Angiotensin I is an active octapeptide C. Sodium excretion following ? C. All of the above 1391 Renin-producing carcinomas may be found in ? Harrison’s 18th Ed. colon and adrenals. I. heart. located primarily in pulmonary circulation. vascular inflammation Renin-producing carcinomas have been described in lung. Pancreas mineralocorticoid receptor. Angiotensin II D. 2044 A. All of the above D. 2044 Harrison’s 18th Ed. pancreas. renal carcinomas & Wilms’ tumors. angiotensin like tonin. and sweat glands. Salivary glands Angiotensin II type 2 (AT 2 ) receptor is widely distributed in kidneys and has the opposite functional effects of AT 1 receptor. Stimulates secretion of aldosterone C. chymase. arteries. 1393 Angiotensin II in tissues may be formed by the enzymatic activity of ? 1387 Which of the following statements is false ? Harrison’s 18th Ed. and remodeling are the effects of which of the following ? Harrison’s 18th Ed. Brain B. 2044 Aldosterone is a potent mineralocorticoid that increases sodium reabsorption by amiloride- sensitive epithelial sodium channels (ENaC) on the apical surface of the principal cells of the A. Vasodilation 1395 Mineralocorticoid receptors are expressed in which of the B. spleen. adrenal glands. Aldosterone synthesis is also dependent on potassium and acute elevations of adrenocorticotropic hormone (ACTH). AJAY MATHUR Cardiology 217 1386 Which of the following directly inhibits renin secretion ? Harrison’s 18th Ed. Lung renal cortical collecting duct. Spleen C. Angiotensin II directly inhibits renin secretion due to angiotensin II type 1 receptors on juxtaglomerular cells. Cathepsins C. Aldosterone D. ACE-kininase II is located in pulmonary circulation D. 217 Cardiology MCQ’s FOR MCQ’s FOR MEDICAL MEDICAL PROFESSIONALS PROFESSIONALS BY PROF. All of the above A. All of the above 1390 Which of the following is a renin-secreting tumor ? Mineralocorticoid receptors also are expressed in the colon. liver. nephrosclerosis. Harrison’s 18th Ed. 2045 A. Cortisone D. Angiotensin II C. Choriocarcinoma A. D. Inhibition of cell growth & matrix formation Harrison’s 18th Ed. Cortisol also binds to mineralocorticoid receptors but is a less potent mineralocorticoid than aldosterone because cortisol is converted to cortisone by the B. Cortisol Renin-secreting tumors include benign hemangiopericytomas of the juxtaglomerular apparatus. leukocytes. Sweat glands inhibition of cell growth and matrix formation. 2044 A. Liver enzyme 11 b-hydroxysteroid dehydrogenase type 2. ovaries. Renin secretion increases in response to pharmacologic blockade of either ACE or angiotensin II receptors. Cortisone has no affinity for the C. Potassium B. All of the above Angiotensinogen. testes. angiotensin II. uterus. 2044 A. 2045 Harrison’s 18th Ed. renin. Cortisol . Harrison’s 18th Ed. and skin. ACE-kininase II is a converting enzyme. Angiotensin I B. Adrenocorticotropic hormone (ACTH) C. sodium excretion and C. Colon B. AT 2 receptor induces vasodilation. and cathepsins. Acts on angiotensin II type 1 receptors on cell membranes A. C. Benign hemangiopericytoma receptor ? Harrison’s 18th Ed. It also inactivates vasodilator bradykinin. Aldosterone synthesized locally in ? B. 2045 D. and angiotensin II are also A. pituitary. 2044 A. Osteoblastoma B. Chymase B. Angiotensin I is an inactive decapeptide B. Prorenin D. renin. kidneys. Germinoma 1396 Which of the following has no affinity for the mineralocorticoid B. All of the above 1389 Which of the following is a functional effect of angiotensin II Angiotensin II is the primary trophic factor regulating synthesis & secretion of aldosterone by type 2 receptor (AT2) ? zona glomerulosa of adrenal cortex. adipocytes. converts angiotensin I to active octapeptide. D. Tonin A. All of the above 1397 Myocardial fibrosis. Potent pressor substance B. 1394 Which of the following is the primary trophic factor regulating synthesis & secretion of aldosterone by zona glomerulosa of 1388 Which of the following statements is false about angiotensin II ? adrenal cortex ? Harrison’s 18th Ed. 2045 1392 Angiotensinogen. salivary glands. Uterus A. 2045 C. ACE-kininase II activates bradykinin Angiotensin II in tissues may be formed by the enzymatic activity of renin or by other proteases In circulation. active renin cleaves angiotensinogen to form an inactive decapeptide. aorta. None of the above C. All of the above D. 2045 Harrison’s 18th Ed. Dopamine A. cardiovascular disease risk increases by ? Harrison’s 18th Ed. Norepinephrine blood pressures by ? Harrison’s 18th Ed.5 times B. 2046 hypertension. leading to myocardial fibrosis. which of the following is the least powerful predictor of cardiovascular disease ? V Harrison’s 18th Ed. hypertension-related vascular lesions in the kidney primarily affect preglomerular arterioles. 2045 Harrison’s 18th Ed. Na+ . low-dose spironolactone reduces the risk of progressive heart failure and sudden death from cardiac 1404 For every 20-mm Hg increase in SBP & 10-mm Hg increase in causes by ? DBP. More severe in glomerular than in interstitial diseases B. Harrison’s 17th Ed. Atrial natriuretic factor C.218 MCQ’s FOR MEDICAL PROFESSIONALS BY PROF. Myocardial infarction and stroke are more frequent in the early morning hours. low-dose spironolactone reduces the risk of progressive heart failure and sudden death increase in diastolic pressure. D. Fibrinoid necrosis of glomerular capillaries D. 5 . 40 % D. Na+ . Evenings 9 D. All of the above C. Epinephrine 1407 Generally. U 1400 Which of the following is increased in hypertension ? 1406 Blood pressure tends to be higher during which of the Harrison’s 18th Ed. Systolic blood pressure B. 20 % B. Pulse pressure D. nephrosclerosis & vascular inflammation and remodeling.20 % Adrenergic neurons synthesize norepinephrine & dopamine. 30 . 2047 C. Following meals C.30 % Epinephrine is synthesized in adrenal medulla & released into circulation upon adrenal stimulation. kidney & blood vessels. All of the above D. All of the above r D. All of the above heart.40 % 1402 Atherosclerotic. 2047 A. 2 times C. Low-renin patients have volume-dependent hypertension C. Preglomerular arterioles A. Postglomerular arterioles C.dependent HCO 3– . Diastolic blood pressure ti e C. 2046 1408 Which of the following about hypertension is false ? Harrison’s 18th Ed. Renal lesion associated with malignant hypertension consists of fibrinoid necrosis of the afferent arterioles. 1398 In CHF. 2. D. 20 . hypertension-related vascular lesions in the Nighttime blood pressures are generally 10 . kidney primarily affect ? Harrison’s 18th Ed. as a consequence of oxidative stress. 1550 A. Fibrinoid necrosis of afferent arterioles . from cardiac causes by 30% R 1399 Intracellular pH (pHi) is regulated by ? 1405 Among older individuals. Cation-independent HCO 3 . soon after waking. 1. 2045 following times ? - Harrison’s 18th Ed.95% of hypertensive patients have “essential” hypertension 1403 Renal lesion associated with malignant hypertension is ? Individuals with white coat hypertension are at increased risk for developing sustained Harrison’s 18th Ed. All of the above hypertension Atherosclerotic. All of the above B.Cl– exchange C. 2047 A. 10 . When neuron is stimulated.H+ exchanger 9 B. Activity of the Na + . Glomerular capillaries B.H + exchange d A. night time blood pressures are lower than day time ta B. Cortisone B. systolic blood pressure & pulse pressure are more powerful predictors HCO 3–-Cl – exchange and cation-independent HCO 3– Cl – exchange. A. Na +-dependent Among older individuals. 10 % A. None of the above n Intracellular pH (pH i ) of vascular smooth muscle cells determine vascular tone and vascular growth. White coat hypertension does not develop into sustained D. Sympathetic outflow A. these neurotransmitters are released into synaptic cleft.20% lower than daytime blood pressures. of cardiovascular disease than diastolic blood pressure. AJAY MATHUR Cardiology C. Fibrinoid necrosis of efferent arterioles Aldosterone and/or mineralocorticoid receptor activation induces structural & functional alterations in D. 2045 A.10 % D. 2047 A. than at other released into circulation upon adrenal stimulation ? h times of day. Ion transport mechanisms that regulate pH i include Na +-H + exchange. which are stored in vesicles within neuron. ~80 .Cl – exchange B.5 times D. These effects are amplified by a high salt intake. C. resulting in ischemic changes in the glomeruli and postglomerular structures. Night i 1401 Which of the following is synthesized in adrenal medulla and Blood pressure tends to be higher in the early morning hours. 3 times G Cardiovascular disease risk doubles for every 20 mm Hg increase in systolic and 10 mm Hg In CHF. 30 % C. 2047 Harrison’s 18th Ed. Early morning hours soon after waking B. 2049 C. Pheochromocytoma D. Intimal fibroplasia D. hypertension may be treated with low-dose glucocorticoids. Mid Aldosterone antagonists. 1416 Plasma aldosterone is suppressed by all except ? Harrison’s 18th Ed. Glucocorticoid-remediable hyperaldosteronism 1412 In renal artery obstruction. Consequently. < 16 ng/dL B. angiotensin receptor antagonists. Proximal D. these individuals do not mature sexually. > 90 % of the lumen pseudohermaphroditism and females with primary amenorrhea and absent secondary sexual characteristics. perimedial fibroplasia. functionally significant lesions occlude ? B. Hypertension and hypokalemia are consequences of increased synthesis of mineralocorticoids proximal to the enzymatic block. < 48 ng/dL D. Contrast arteriography 1418 In which of the following conditions. Aldosterone biosynthesis more responsive to ACTH B. Any of the above Lesions of fibromuscular dysplasia are frequently bilateral and in contrast to atherosclerotic 1417 Which of the following is false about aldosterone-producing renovascular disease tend to affect more distal portions of renal artery. IV infusion of isotonic saline. Sodium retention Harrison’s 18th Ed. hypertensive headache occurs in morning & is localized to occipital region. Glucocorticoid-induced hypertension C. 2053 Harrison’s 18th Ed. Captopril A. Temporal region C. hypokalemic hypertension may be a consequence of secondary aldosteronism. 2049 B. or captopril suppress aldosterone.and glucocorticoid-induced hypertensive disorders. hence. medial hyperplasia & intimal fibroplasia. Morning C. Males may present with D. None of the above C. Perimedial fibroplasia B. increased. Occipital region A. < 32 ng/dL C. All of the above Characteristically. 2049 1419 “Hypertensive headache” occurs during ? A. 219 Cardiology MCQ’s FOR MCQ’s FOR MEDICAL MEDICAL PROFESSIONALS PROFESSIONALS BY PROF. 2050 1410 Lesions of fibromuscular dysplasia mostly affect which portion A. Afternoon D. Because cortisol-induced negative feedback on pituitary ACTH production is Functionally significant renal artery lesions generally occlude more than 70% of the lumen of diminished. 1413 Which of the following is not a feature of primary aldosteronism ? Harrison’s 18th Ed. ACTH-stimulated adrenal steroid synthesis proximal to the enzymatic block is the affected renal artery. Primary aldosteronism B. oral NaCl load. synthesis of sex hormones & cortisol is decreased. High PRA C. 17-hydroxylase deficiency A. DTPA scan C. > 70 % of the lumen With 17  -hydroxylase deficiency. Table 247–4 A. > 30 % of the lumen D. Gadolinium-contrast magnetic resonance angiography D. 11-hydroxylase deficiency Harrison’s 18th Ed. 2050 1411 Which of the following serves as “gold standard” for evaluation and identification of renal artery lesions ? A. Oral NaCl load of renal artery ? B. and 1421 Width of the blood pressure bladder cuff should equal at least pheochromocytoma. Distal renin. adrenal adenoma ? Harrison’s 18th Ed. Tumor is almost always unilateral Harrison’s 18th Ed. 2049 C. Night system and leads to sodium retention. Medial fibroplasia is the most common variant (66%). Harrison’s 18th Ed. Medial fibroplasia A. males may present with D. Fludrocortisone Harrison’s 18th Ed. < 72 ng/dL Histologic variants of fibromuscular dysplasia include medial fibroplasia. 2051. Table 2 A. in diameter A. Medial hyperplasia C. 2049 A. particularly desoxycorticosterone. and low PRA. Evening Excess aldosterone production in primary aldosteronism is independent of the renin-angiotensin D. Increased steroid production and. Any of the above D. fludrocortisone. Besides primary aldosteronism. Appendix: Laboratory Values of Clinical Importance. D. AJAY MATHUR Cardiology 219 1409 Which is the most common histologic variant of fibromuscular 1415 Plasma aldosterone (adult) level in supine position with patient dysplasia ? on normal sodium diet normally is ? Harrison’s 18th Ed. Hypokalemia B. Hypertension A. > 50 % of the lumen C. Aldosterone antagonists may increase aldosterone. other mineralocorticoid. Spironolactone B. hypertension.. 2049 Harrison’s 18th Ed. 2053 B. what percentage of the arm circumference ? . Measures < 3 cm. 11-hydroxysteroid dehydrogenase deficiency B. hypokalemia. Doppler ultrasound of the renal arteries pseudohermaphroditism and hypertension ? Contrast arteriography remains the “gold standard” for evaluation and identification of renal artery lesions. 1420 “Hypertensive headache” is localized to ? 1414 Hypokalemic hypertension is seen in ? Harrison’s 18th Ed. and ACE inhibitors may increase C. Frontal region B. 2056 B. 2056 D. Thiazides inhibit the Na +/Cl– pump in the distal convoluted tubule and hence increase sodium excretion. 20 % Harrison’s 18th Ed. Calcium antagonists i D. Guanfacine D. All of the above the length of the cuff bladder should encircle at least 80% of the arm circumference. ~ 14 grams Harrison’s 18th Ed. Death and survival in hypertension B. Potassium B. Eplerenone is an aldosterone antagonist. Decrease production of angiotensin II C. 40 % A. Harrison’s 18th Ed. 20 % D. Autonomic neuropathy Harrison’s 18th Ed. 2056 A. Increase bradykinin levels D. 2053 B. Table 247–7 ACEIs and ARBs improve insulin action and ameliorate the adverse effects of diuretics on glucose metabolism. Lisinopril 1426 Supplementation of which of the following may be associated C. Captopril with reduced stroke mortality ? D. Reduce sympathetic nervous system activity G Width of the blood pressure bladder cuff should equal at least 40% of the arm circumference. Valsartan DASH (Dietary Approaches to Stop Hypertension) trial. AT2 receptors Attenuated nighttime blood pressure “dip” is associated with increased cardiovascular disease C. C. ACEIs and ARBs D. Eplerenone B. B. 2053 following mechanism ? A. AT4 receptors & in African Americans. 2054. 40 % channels in the distal nephron. 80 % C. 1422 Length of blood pressure bladder cuff bladder should encircle at least what percentage of the arm circumference ? 1428 Angiotensin-converting enzyme inhibitors act by which of the Harrison’s 18th Ed. act by inhibiting epithelial sodium D. 1553 1429 Angiotensin receptor blockers (ARB) provide selective A. AJAY MATHUR Cardiology Harrison’s 18th Ed. Ramipril Harrison’s 18th Ed. 2054 A. ~ 10 grams diuretics on glucose metabolism ? B. Alpha antagonists 1425 DASH trial stands for ? h Harrison’s 18th Ed. R ACEIs decrease the production of angiotensin II. Doxazosin Potassium supplementation may be associated with reduced stroke mortality. ta A. Thiazides 1433 Which of the following about spironolactone is false ? Harrison’s 18th Ed. increase bradykinin levels. A.220 MCQ’s FOR MEDICAL PROFESSIONALS BY PROF. autonomic neuropathy D. Duration and severity of hypertension A. D. African Americans populations D. ~ 24 grams 9 B. Beta blockers r One standard drink contains ~14 grams of ethanol. 60 % B. nonpeptide competitive inhibitors of the enzymatic activity of renin. 2056 C. Diastolic and systolic hypertension 1431 Which of the following reduces the risk of developing diabetes C. 1423 Blunting of the day-night blood pressure pattern occurs in ? V Harrison’s 17th Ed. C. All of the above B. Aliskiren 1427 Which of the following act by inhibiting epithelial sodium D. 2054 Valsartan (an ARB) reduces the risk of developing diabetes in high-risk hypertensive patients. and the effect of angiotensin II on unblocked U 1424 One standard drink contains how many grams of ethanol ? AT 2 receptors may augment their hypotensive effect. Alcohol B. 2056 . Sleep apnea blockade of ? d B. AT1 receptors ti e C. and reduce sympathetic nervous system activity. Loop diuretics inhibit the Na +-K+-2Cl – cotransporter in the thick ascending limb of the loop of Henle. Sodium A. Furosemide A. 2054 - 1430 Which of the following diminishes the adverse effects of A. Amiloride C. Calcium 1432 Which of the following is a direct renin inhibitor ? Harrison’s 18th Ed. 2056 9 C. 10 % C. Harrison’s 18th Ed. AT3 receptors n risk. amiloride and triamterene. Dietary approaches to stop hypertension in high-risk hypertensive patients Harrison’s 18th Ed. ~ 18 grams A. Blunting of day-night blood pressure pattern occurs in sleep apnea. ARBs provide selective blockade of AT 1 receptors. Terazosin channels in the distal nephron ? Aliskiren is an oral. 30 % Potassium-sparing diuretics. side effects may include gynecomastia. 2056 A. Diuretics Phenoxybenzamine is a nonselective alpha antagonist. Hydralazine C. Edema is progesterone and androgen receptors. 2057 A. 1437 Selective alpha antagonists include all except ? 1443 Patients with high-renin hypertension may be more responsive Harrison’s 18th Ed. Aliskiren C. It is particularly effective agent in patients with low-renin essential hypertension. drugs. Binds to progesterone D. Antioxidant B. Beta blockers D. 2056 Harrison’s 18th Ed. Benzothiazepines B. 2056 to which of the following ? Harrison’s 18th Ed. Terazosin B. 2056 B. Candesartan D. All of the above D. which of the following ACE inhibitors was studied ? B.channel blockade B. N . Nitric-oxide enhancer C. 1434 Spironolactone is effective in patients with ? 1440 Edema with dihydropyridine calcium channel blockers is due Harrison’s 18th Ed. All of the above C. Net salt retention C. Beta blockers C. Lisinopril . isradipine. None of the above and nisoldipine). 2056 Harrison’s 18th Ed. Eplerenone D. Carvedilol B. Prazosin B. 1441 Which of the following is true about Hydralazine ? 1435 Which of the following is a selective aldosterone antagonist ? Harrison’s 18th Ed. 2056 to ? A. impotence. Patients with high-renin hypertension may be more responsive to ACE inhibitors & angiotensin receptor blockers. nicardipine. All of the above with a thiazide diuretic.phenylalkylamines (verapamil). Low-renin essential hypertension Harrison’s 18th Ed. Methyldopa D. Nonselective aldosterone antagonist C. Doxazosin A. which reduces Younger patients may be more responsive to beta blockers & ACE inhibitors. Net water retention D. Binds to androgen receptors Calcium antagonists are of three classes . Dihydropyridines C. 2056 Harrison’s 18th Ed. Calcium antagonists D. Harrison’s 18th Ed. 1442 Which of the following drug is used in patients with renal 1436 Which of the following blocks both  receptors and peripheral insufficiency who are refractory to all other drugs ?  -adrenergic receptors ? Harrison’s 18th Ed. age 50 may be more responsive to diuretics & calcium antagonists. Direct vasodilator A. Eplerenone C. ACE inhibitors C. 1444 Patients with low-renin hypertension are more responsive to 1438 Calcium antagonists reduce vascular resistance through ? which of the following ? Harrison’s 18th Ed. Increase in transcapillary pressure gradients Spironolactone is a nonselective aldosterone antagonist that may be used alone or in combination D. whereas patients over intracellular calcium and blunts vasoconstriction. K . Phenylalkylamines 1445 In HOPE (Heart Outcomes Prevention Evaluation) trial. Phenylpyridines The American Journal of Medicine 2005. Minoxidil Hydralazine is a potent direct vasodilator that has antioxidant and nitric-oxide enhancing actions. Primary aldosteronism B. Because spironolactone binds to Edema with dihydropyridine use are related to their potencies as arteriolar dilators. AJAY MATHUR Cardiology 221 A. not to net salt and water retention. 221 Cardiology MCQ’s FOR MCQ’s FOR MEDICAL MEDICAL PROFESSIONALS PROFESSIONALS BY PROF. 695-705 A. Direct vasodilators Calcium antagonists reduce vascular resistance through L-channel blockade. Minoxidil A. Phenoxybenzamine C. Calcium antagonists D. Minoxidil Minoxidil is used most frequently in patients with renal insufficiency who are refractory to all other Carvedilol and labetalol block both  receptors and peripheral  -adrenergic receptors. benzothiazepines (diltiazem). 2057 A.channel blockade A. M . and due to an increase in transcapillary pressure gradients.channel blockade D. menstrual abnormalities. resistant hypertension. whereas 1439 Which of the following is not a class of calcium antagonists ? patients with low-renin hypertension are more responsive to diuretics and calcium antagonists. ACE inhibitors B. and 1. Eplerenone is a selective aldosterone antagonist. Resistant hypertension A. Aliskiren B.118.4-dihydropyridines (amlodipine. nifedipine D. L .channel blockade C. and primary aldosteronism. 2056 A. Aliskiren B. 2056 A. felodipine. 2058 B. Stroke D. exudates. Absolute level of blood pressure B. the maneuver is called ? ir Harrison’s 18th Ed. 25 % Osler maneuver refers to the radial pulse that remains palpable despite occlusion of the C. Roger maneuver reduce mean arterial blood pressure by no more than ? h B. 2058 B. Captopril Harrison’s 18th Ed. All of the above C. AJAY MATHUR Cardiology B. 2058 B. and encephalopathy. Higher office and High home blood pressures C. G 1447 Which of the following drug has renal protective effects ? Harrison’s 18th Ed. Losartan A. Osler maneuver Harrison’s 18th Ed. Calcium channel blockers B. Microangiopathic hemolytic anemia - D. 2058 Harrison’s 18th Ed. Underlying hypertension A. which of the following ACE hypertension ? inhibitors was studied ? Harrison’s 18th Ed. 1449 If the radial pulse remains palpable despite occlusion of the hemorrhages. 2059 A. brachial artery by the cuff. Enalapril D. 2057 1453 Which of the following occurs pathologically in malignant hypertension ? A.118. 1457 Which of the following drug is contraindicated in pregnancy ? Harrison’s 18th Ed. 2058. and papilledema). >140/90 mmHg B. >150/90 mmHg C. 695-705 A. and fibrin deposition in arteriolar walls. Happit maneuver D. >130/90 mmHg A. malignant hypertension is associated with diffuse necrotizing vasculitis. Weber maneuver B. ARBs B. arteriolar 1448 Better stroke protection is provided by ? thrombi. Aliskiren R Harrison’s 18th Ed. in malignant hypertension. Arteriolar thrombi D. Aliskiren B. Beta blockers U A. Ramipril C. All of the above d C. Adrenergic crisis Resistant hypertension refers to patients with BP persistently >140/90 mmHg despite taking three or more antihypertensive agents. 2058 The American Journal of Medicine 2005. High office and lower home blood pressures D. ACE inhibitors Harrison’s 18th Ed. 2055. Lower office and low home blood pressures D.110 mmHg. deteriorating renal function with proteinuria. renal protection with aliskiren is comparable to that with ACEIs & D. Lisinopril B. in reasonable combination and at full doses ? 1456 Esmolol is the preferred parenteral drug for ? Harrison’s 18th Ed. including a diuretic. Diffuse necrotizing vasculitis V C. 2058 1455 In malignant hypertension. Preeclampsia/eclampsia of pregnancy C. 60 % 1450 Resistant hypertension refers to BP persistently above what In malignant hypertension. 10 % ta C. Lower office and High home blood pressures C. the initial goal of therapy is to A. All of the above channel blockers provide more stroke protection than either ACE inhibitors or beta blockers. Enalapril D.222 MCQ’s FOR MEDICAL PROFESSIONALS BY PROF. 2057 n 1454 Malignant hypertension is associated with ? A. Harrison’s 18th Ed. Ramipril C. the initial goal of therapy is to reduce mean arterial blood pressure by no more than 25% within minutes to 2 hours or to a blood pressure in the range of 160/100 . Pathologically. including a diuretic. Aortic dissection B. Perindopril In malignant hypertension. Progressive retinopathy C. ACE Inhibitors A. Encephalopathy ACE inhibitors provide better coronary protection than calcium channel blockers. 9 Clinically. Table 247–8 1451 “Pseudoresistance” refers to ? A. Perindopril “Pseudoresistance” refers to high office blood pressures & lower home blood pressures. whereas calcium D. Rate of rise of blood pressure C. Table 247–9 A. 9 microangiopathic hemolytic anemia. there occurs progressive retinopathy (arteriolar spasm. Fibrin deposition in arteriolar walls In hypertension & diabetes. D. 40 % brachial artery by the cuff. Response to treatment D. 1446 In ALLHAT (Antihypertensive and Lipid-Lowering Treatment 1452 Which of the following is most important in malignant to Prevent Heart Attack Trial). in reasonable combination & at full doses. All of the above . >160/90 mmHg D. level despite taking 3 or more antihypertensives. absolute level of blood pressure is not as important as its rate of rise. All of the above ti e ARBs. 338:1131 1464 High sodium intake is defined by ? A. Factor V Leiden mutation C. Calcium channel blocker C. Chloroquine Harrison’s 16th Ed. All of the above A. Table 247–9 D. Obesity N Engl J Med 2006.355:385-92 D. Urinary sodium excretion of >50 mmol per day C. Labetalol A. Labetalol 1467 Which of the following suggest aldosteronism as cause of C. Appetite stimulants N Engl J Med 1998. AJAY MATHUR Cardiology 223 1458 Which of the following antihypertensive agents is given as “IV B. Recent increase in severity of hypertension Harrison’s 18th Ed. Table 247–10 D. Nulliparity D. 2058. Thyroid hormone A. Beta-adrenergic–receptor antagonists A. Antiphospholipid antibody syndrome 1469 To prevent hypertensive crisis. Labetalol N Engl J Med 2006. 2067 1463 Medications or substances that can raise blood pressure or A. Angiotensinogen gene T235 D.338:1131 1465 Frequency of salt sensitivity is increased among ? N Engl J Med 2006. which of the following drugs would be safest ? D. 32 C. Abnormal ratio of aldosterone to renin 1460 Risk factors for preeclampsia include all except ? B. Catecholamines N Engl J Med 2006. Black or obese bolus” in hypertensive emergencies ? C.355:385-92 1461 Risk factors for preeclampsia include all except ? A. > 60 years of age B. All of the above C. 32 B. Smoking avoid ? N Engl J Med 2005. Elevated plasma metanephrines B. Excessive alcohol use C. Phentolamine renovascular disease ? N Engl J Med 2006. Hypokalemia preeclampsia / eclampsia of pregnancy is ? C. Abdominal bruit 1459 Preferred parenteral drug for hypertensive emergency during B. Dopamine antagonize effects of antihypertensive drugs include all except ? B. Prolonged renal failure in neonates A. High sodium intake 1471 In hypertension during pregnancy. Prazosin B. 223 Cardiology MCQ’s FOR MCQ’s FOR MEDICAL MEDICAL PROFESSIONALS PROFESSIONALS BY PROF. None of the above effects can happen ? N Engl J Med 1998. Abnormal response to sodium loading Harrison’s 16th Ed.355:385-92 B. Urinary sodium excretion of >150 mmol per day 1472 If ACE inhibitors are given during pregnancy. what teratogenic D. Adrenal adenoma on CT or MRI A. Decreased skull ossification . Nonsteroidal antiinflammatory drugs D. Anabolic steroids D.355:385-92 A. Nicardipine secondary hypertension ? N Engl J Med 2006. Norepinephrine >80 µg/24 hours Harrison’s 16th Ed. Tryptophan diet A. All of the above C.355:385-92 C.355:385-92 D. Hydralazine B. Esmolol 1466 Which of the following suggest a diagnosis of atherosclerotic C. Ginseng C. Urinary sodium excretion of >100 mmol per day D. Hydralazine B. 2058. Erythropoietin 1470 Which of the following is amino acid derived hormone ? D. Phenylalanine diet N Engl J Med 2006. All of the above A. Renal impairment Harrison’s 18th Ed. Diabetes mellitus C. patients taking MAOIs must D.355:385-92 B. Tyramine diet B. All of the above B. All of the above B.353:1819-34 1462 Medications or substances that can raise blood pressure or antagonize effects of antihypertensive drugs include all except ? A. Anemia 1468 Which of the following suggest pheochromocytoma as cause of secondary hypertension ? D. VMA >5 mg/24 hours A. All of the above A. Alcohol C. and an internal elastic lamina. Warfarin 1482 Kommerell’s diverticulum is an anatomic remnant of ? B. None of the above ta C. aortic diameter at the origin is approximately ? degenerative aortic aneurysms ? Harrison’s 18th Ed. William Osler D. Endothelium A. intimal & medial layers are disrupted & dilatation is lined by adventitia only.351:2310-7 B. 1485 Which of the following factors is not associated with 1479 In adults. Renal tubular dysgenesis In adults. 2060 A. 2. Severe nasal hypoplasia Harrison’s 18th Ed. Adventitia B. Tunica media C. Chapter 248.14:107-114 Harrison’s 18th Ed.5 cm in descending portion in thorax. Endothelium R C. Calpains A. 35 following structures of aorta ? G A. Internal elastic lamina C. Ductus arteriosus U 1476 Which of the following is a vasoconstrictor ? Lancet 2005:365:417-430 D. Endothelin 1483 In a pseudoaneurysm. Hypercholesterolemia . Angiotensin II disrupted ? 9 D. Subendothelial connective tissue B. Adventitia V D. 2061 A. subendothelial connective tissue. All of the above d 1475 Which of the following anticoagulants does not cross placenta ? Harrison’s 16th Ed.224 MCQ’s FOR MEDICAL PROFESSIONALS BY PROF. Anomalies of teeth and bone C. All of the above 1473 If Warfarin is given during pregnancy. Bilirubin D. Chiari type I malformation D. 2060 r A. Alexender Fleming 1484 A fusiform aneurysm affects what proportion of the 1478 Which of the following is called “Age pigment” ? circumference of a segment of the vessel ? Heart Lung and Circulation 2005. Lipofuscin C. and 1. B. D. Intima i 1477 Hypertensive retinopathy was first described by ? N Engl J Med 2004. 1474 Mothers who took warfarin during 1st trimester could have all of the following congenital malformations in infants except ? 1481 Vasa vasorum and nervi vascularis are located in which of the Harrison’s 16th Ed. Anisindione Harrison’s 18th Ed. 2060 B. Diseases of the Aorta resulting in an outpouching of the vessel wall. 2060 Harrison’s 18th Ed. AJAY MATHUR Cardiology C. 680 Vasa vasorum & nervi vascularis are located in adventitia composed of connective tissue. Right aortic arch C. 75 % D. 3 cm B. Left atrial appendage - A. 2060 can happen ? N Engl J Med 1998. Neonatal meconium ileus B. which of the following layers is not 9 C. 100 % A fusiform aneurysm affects the entire circumference of a segment of the vessel. 2 cm A. 5 cm D. 2060 C. 4 cm C. A true aneurysm involves all three layers of the vessel wall. what teratogenic effect 1480 Aortic wall intima is composed of all except ? Harrison’s 18th Ed. ti e A. Media h A.8 to 2 cm in abdomen. Stippled epiphyseal calcifications A. All of the above Harrison’s 18th Ed. Smooth muscle cells D. Heparin A. None of the above B. Left umblical artery n D. Agenesis of the corpus callosum B. Keith Wegener In pseudoaneurysm. Aging B. Cigarette smoking D. D. Dandy–Walker syndrome Aortic wall thin intima is composed of endothelium. 50 % C. 25 % B.338:1131 A. A saccular aneurysm involves only a portion of circumference. Ebstein’s anomaly D. Melanin B. resulting in a diffusely dilated artery. Norepinephrine Kommerell’s diverticulum is an anatomic remnant of a right aortic arch. aortic diameter at the origin is ~3 cm. Marcus Gunn C. TNXB C. COL5A1 B. Cystic medial necrosis is particularly prevalent in patients with Marfan syndrome. relapsing D. D. Tuberculosis D. Hypertension C. cigarette smoking. 1493 Syphilitic aneurysms are mostly located in ? Harrison’s 18th Ed. Cystic medial necrosis C. Behcet’s disease C. Ehlers-Danlos syndrome type IV D. 2061 B. Loeys-Dietz syndrome Spondyloarthropathies like ankylosing spondylitis. PLOD1 Atherosclerosis is the most frequent cause of aneurysms of aortic arch & descending thoracic Loeys-Dietz syndrome is due to mutations in genes that encode TGF- 1 (TGFBR1) & 2 (TGFBR2). Atherosclerosis D. 2062 B. Marfan syndrome C. congenital bicuspid aortic valves. AJAY MATHUR Cardiology 225 Factors associated with degenerative aortic aneurysms include aging. dilatation and development of fusiform aneurysm. Cleft palate B. Intima B. 2061 A. Direct extension of tubercular infection D. 1491 Loeys-Dietz syndrome (LDS) is characterized by all except ? 1497 Most common pathological condition for distal abdominal aortic aneurysm below renal arteries is ? A. Ascending aorta Harrison’s 18th Ed. Hypertension Tuberculous aneurysms typically affect thoracic aorta. Ascending aorta Harrison’s 18th Ed. Hypertension C. Abdominal aorta B. 2061 1488 Cystic medial necrosis is prevalent in patients with ? A. Harrison’s 18th Ed. and a family history of aortic aneurysms. 2061 B. Cystic medial necrosis Approximately 90% of syphilitic aneurysms are located in ascending aorta or aortic arch. Atherosclerosis B. and 1495 All of the following conditions can lead to dilatation of the familial thoracic aortic aneurysm syndromes. Marfan syndrome C. Tuberculosis Syphilis is a relatively uncommon cause of aortic aneurysm. cleft palate & hypertelorism. Ehlers-Danlos syndrome type IV. Cystic medial necrosis characteristically affects the proximal aorta. 2061 1489 Mutations of the gene that encodes fibrillin-1 are present in patients with ? A. Osteogenesis Imperfecta (OI) polychondritis and Reiter’s syndrome are associated with dilatation of the ascending aorta while Behcet’s disease and Cogan’s syndrome cause thoracic and abdominal aortic aneurysms. hypercholesterolemia. Rheumatoid arthritis A. 2061 Harrison’s 18th Ed. Thoracic aorta A. Ankylosing spondylitis Harrison’s 18th Ed. Ehlers-Danlos type I syndrome A. psoriatic arthritis. Aortic aneurysms Harrison’s 18th Ed. Hypertelorism C. pathologic condition associated with degenerative aortic aneurysms is atherosclerosis. 2061 1487 Most common pathological condition for ascending aortic aneurysm is ? A. Adventitia D. 1492 Mutations of type III procollagen results in ? 1486 Cystic medial necrosis affects which component of the aortic wall ? Harrison’s 18th Ed. TGF- receptors 1 (TGFBR1) D. 1496 Most common pathological condition associated with 1490 Loeys-Dietz syndrome is caused by mutations in the genes aneurysms of aortic arch & descending thoracic aorta is ? that encode ? Harrison’s 18th Ed. hypertension. male sex. Most common LDS is related to Marfan Syndrome. All of the above Mutations of type III procollagen have been implicated in Ehlers-Danlos type IV syndrome. 225 Cardiology MCQ’s FOR MCQ’s FOR MEDICAL MEDICAL PROFESSIONALS PROFESSIONALS BY PROF. Loeys-Dietz syndrome. Ehlers-Danlos type II syndrome B. Hypertension A. 2061 B. ascending aorta except ? Harrison’s 18th Ed. rheumatoid arthritis. 2061 A. 2061 Harrison’s 18th Ed. Loeys-Dietz syndrome (LDS) is characterized by aortic aneurysms. All of the above C. Reiter’s syndrome B. Mutations of the gene that encodes fibrillin-1 are present in patients with Marfan syndrome. Cystic medial necrosis . Media C. All of the above C. Abdominal aorta B. Ehlers-Danlos type III syndrome C. Cystic medial necrosis is a histopathologic term used to describe degeneration of collagen & elastic fibers in tunica media of aorta. Hearing loss A. Thoracic aorta A. aorta. Ehlers-Danlos syndrome type IV D. leading to circumferential 1494 Tuberculous aneurysms are mostly located in ? weakness. Ehlers-Danlos type IV syndrome D. Atherosclerosis D. All of the above from hilar lymph nodes or contiguous abscesses or from bacterial seeding resulting granulomatous destruction of medial layer is the pathogenetic mechanism. parenteral beta-adrenergic blockers (IV propranolol. Femoral artery . Hydralazine substantially beyond ? D. All of the above Harrison’s 18th Ed. Left lateral wall of ascending aorta Harrison’s 18th Ed. < 0. Fibromuscular dysplasia C. Peak incidence is in sixth & seventh decades extremity. Raynaud’s phenomenon. 2063 1506 In patients with peripheral arterial disease. 2064 most of these aneurysms are below the level of the renal arteries. Isolated use of direct vasodilators like diazoxide and hydralazine is contraindicated C. metoprolol. Sodium nitroprusside B. A. Giant cell (temporal) arteritis following period of pregnancy ? “String of beads” appearance on angiography is characteristic of fibromuscular dysplasia caused by Harrison’s 18th Ed. Claudication of affected extremity D. systolic blood pressure in leg is decreased. < 0. It often occurs along the right lateral wall of the ascending aorta where the hydraulic shear stress is high. <1 ligamentum arteriosum. Renal artery stenosis B. 2063 Clinical features of thromboangiitis obliterans include a triad of claudication of the affected A. Third trimester of pregnancy 1508 Clinical features of thromboangiitis obliterans include ? Harrison’s 18th Ed. AJAY MATHUR Cardiology D. > 5 cm infusion is used to lower SBP to <120 mmHg. Harrison’s 18th Ed. Internal carotid artery lesions C. Right lateral wall of thoracic descending aorta A. and Harrison’s 18th Ed. less-involved portions of the arterial wall. diltiazem or enalaprilat may be used parenterally.8 intima. below the renal arteries. Verapamil. 2069 C. > 6 cm because these agents can increase hydraulic shear and may propagate dissection. below the renal arteries ? Harrison’s 17th Ed. Men are more affected than women 1509 In lower extremities. Second trimester of pregnancy B. Raynaud’s phenomenon Incidence of aortic dissection is increased in normal women during 3rd trimester of pregnancy.6 Aortic dissection is caused by a circumferential or. > 7 cm Risk of rupture increases substantially for ascending aortic aneurysms >6 cm and descending thoracic aneurysms >7 cm. In hemodynamically significant arterial stenoses. emboli lodge most frequently in ? C. Migratory superficial vein thrombophlebitis 1503 Which of the following about aortic dissection is false ? D. 33 % 1505 Which of the following drugs should be avoided while C. risk of rupture increases C. 2069 D. Chapter 249. Sodium nitroprusside 1499 For descending thoracic aneurysms. Another common site is the descending thoracic aorta just below the D. Aortic dissection A. Intrapartum period A. 2063 thickened fibromuscular ridges contiguous with thin. transverse tear of the C. Thromboangiitis obliterans D. Acute aortic regurgitation is common Harrison’s 18th Ed. the ankle:brachial A. Peripheral vascular disease 1507 “String of beads” appearance on angiography is characteristic of ? Harrison’s 18th Ed. D. B. Takayasu’s arteritis 1502 Incidence of aortic dissection is increased during which of the D. C. 1563 C. 2063 A.0 cm are affected by atherosclerosis. Beta adrenergic blockers 1498 What percentage of atherosclerotic aneurysms are located in B. 2064 D. Diazoxide A.226 MCQ’s FOR MEDICAL PROFESSIONALS BY PROF. None of the above A. 1482 A. 50 % managing hypertension in acute aortic dissection ? Harrison’s 18th Ed. Verapamil Harrison’s 16th Ed. Right lateral wall of ascending aorta index (ABI) is ? B. Ratio of ankle and brachial artery pressures (ankle:brachial index or ABI) is >=1. < 0. B. > 4 cm For acute dissection. 75 % A. Tuberculosis 1504 Which of the following drugs should be avoided while managing hypertension in acute aortic dissection ? At least 90% of all abdominal aortic aneurysms >4. labetalol or esmolol) should be administered to achieve a heart rate of ~60 beats/min. Post partum period B. Vascular Diseases of the Extremities 1500 Aortic dissection commonly occurs at ? Harrison’s 18th Ed. less frequently.0 in normal individuals and 1501 Stanford & DeBakey classifications are used to classify ? <1. 2069 B.4 D. and migratory superficial vein thrombophlebitis.0 in patients with peripheral arterial disease. Sodium nitroprusside distal abdominal aorta. Diltiazem B. a ratio of <0. A. Right lateral wall of thoracic descending aorta B. 25 % D.5 is consistent with severe ischemia. 2067 C. Labetalol 75 % of atherosclerotic aneurysms are located in distal abdominal aorta. weakness. 2071 B. A. Cisplatin 1514 On cold exposure. Radial. Women are affected about five times more often than men. In addition to rubor D. 50% with Raynaud’s phenomenon have Raynaud’s disease Harrison’s 18th Ed. Scleroderma Raynaud’s phenomenon is characterized by episodic digital ischemia. Dermatomyositis or polymyositis . B. vinblastine and cisplatin. Frostbite B. Methysergide Harrison’s 18th Ed. Vinblastine called Nicoladoni-Branham sign. None of the above A. All of the above Raynaud’s phenomenon ? Harrison’s 18th Ed. Hyperabduction maneuver causing loss of pulses in arm A. This is B. followed by iliac artery. and pedal pulses are normal. Thoracic Outlet Compression Syndrome C. C. Table 249–1 D. Toes are involved more frequently than fingers D. Fingers and toes perspire excessively between attacks C. patients experience throbbing pain C. Compression of large arteriovenous fistula causing reflex D. manifested clinically by sequential development of digital blanching or pallor. Cyanosis aorta. Harrison’s 18th Ed. beta- adrenergic receptor blockers. The fingers and toes may be cool D. Hyperabduction maneuver C. Arteriovenous Fistula and warmth. All of the above between attacks and may perspire excessively. Age of presentation is between 20 and 40 years C. and popliteal and tibioperoneal arteries. This “reactive hyperemia” imparts a bright red color to the digits. Systemic lupus erythematosus following cold exposure & subsequent rewarming. Fingers are involved more frequently than the toes. Iliac artery 1515 In Raynaud’s phenomenon. 2071 Drugs that can cause Raynaud’s phenomenon include ergot derivatives. Atheroembolism the digital vasospasm resolves. Rubor 1510 “Blue toe” syndrome is best related to ? D. Radial. Venous compression causes thrombosis of subclavian & axillary veins. Costoclavicular maneuver Raynaud’s phenomenon occurs frequently in patients who also have migraine headaches or variant angina. Bleomycin Compression of a large arteriovenous fistula may cause reflex slowing of the heart rate. None of the above Harrison’s 18th Ed. Tenderness in supraclavicular fossa C. Patients with arterial compression experience claudication. bleomycin. 1517 Which of the following about Raynaud’s phenomenon is false ischemic tissue loss & gangrene. Popliteal artery during which phase ? Harrison’s 18th Ed. 20 and 40 years. C. Women are affected 5 times more than men A. in which affected arm is abducted by 90° & shoulder is externally rotated. and blood flow into the dilated arterioles and capillaries increases dramatically. Atheroembolism D. Pallor B. ulnar and pedal pulses are abnormal B. During ischemic phase. -adrenergic receptor antagonists A. C. and the age of presentation is usually between Thoracic outlet compression syndrome may be divided into arterial. 1516 Which of the following about Raynaud’s disease is false ? Harrison’s 18th Ed. cyanosis. 2072. 227 Cardiology MCQ’s FOR MCQ’s FOR MEDICAL MEDICAL PROFESSIONALS PROFESSIONALS BY PROF. Frostbite Blanching or pallor represents the ischemic phase of Raynaud’s phenomenon and results from vasospasm of digital arteries. 2072 D. often associated with effort & is referred to as Paget-Schroetter syndrome. Tibioperoneal arteries A. C. Arteriovenous Fistula Over 50% of patients with Raynaud’s phenomenon have Raynaud’s disease. 2070-71 A. 2072 1513 Nicoladoni-Branham sign refers to ? A. 2071 1511 Paget-Schroetter syndrome is best related to ? A. Raynaud’s phenomenon results from deoxygenated blood that is present in these vessels. Occurs frequently in patients who also have variant angina Harrison’s 18th Ed. ulnar. Costoclavicular phenomenon ? maneuver (posterior rotation of shoulders) & hyperabduction maneuver (raising arm to 180°). painful sensation during the hyperemic phase. Rubor phenomenon ? Harrison’s 18th Ed. Scalene maneuver D. Maneuvers used for diagnosis of thoracic outlet compression syndrome include abduction & external rotation test. -adrenergic receptor antagonists B. Pallor In lower extremities. 2070 A. 1518 Which of the following drugs does not cause Raynaud’s Scalene maneuver (extension of neck & rotation of head to side of symptoms). 2071 D. emboli lodge most frequently in femoral artery. venous & neurogenic forms. which of the following is the first to occur in D. A sensation of cold or numbness or paresthesia of the digits often accompanies the phases of pallor and cyanosis. Raynaud’s phenomenon. & paresthesias. & rubor of fingers or toes B. Occurs frequently in patients who also have migraine thoracic outlet compression syndrome ? B. Atheroembolism causes cyanotic discoloration & impending necrosis of the toes termed as “blue toe syndrome”. Cyanosis 1520 Which of the following diseases can present as Raynaud’s C. AJAY MATHUR Cardiology 227 B. patients often experience a throbbing. 2070 B. methysergide. None of the above slowing of heart rate 1519 Which of the following drugs cause Raynaud’s phenomenon ? C. ? Harrison’s 18th Ed. 2071 1512 Which of the following maneuvers is useful in diagnosis of A. Patients with neurogenic thoracic outlet compression may develop shoulder & arm pain. capillaries & venules dilate and cyanosis B. Hyperabduction maneuver causing subclavian bruits Harrison’s 18th Ed. With rewarming. Prazosin D. there is arterial vasoconstriction & secondary dilation of capillaries & venules with resulting persistent cyanosis of hands & feet. All of the above C. use of vibrating hand tools. rheumatoid arthritis. 2073 A. pulmonary hypertension. Methyldopa 1528 Which of the following best relates to Pernio (Chilblains) ? D. All of the above C. Lung cancer environment C. sized arteries by a thrombus or embolus. anticardiolipin antibodies. Erythromelalgia A. Acrocyanosis Harrison’s 18th Ed. Brain tumour D. May be secondary to myeloproliferative disorders 1524 Secondary acrocyanosis is associated with ? B. primary hyperviscosity. connective tissue diseases. and pergolide. Electric shock injury to the hands or frostbite may lead to the later development of 1527 The other name of ‘Pernio’ is ? Raynaud’s phenomenon. and they may blister and ulcerate. cold agglutinins. such as calcium channel D. Frostbite C. Anticardiolipin antibodies Raynaud’s phenomenon occurs in patients with systemic sclerosis (scleroderma). and is associated are relieved by exposing the affected area to cool air or water or by elevation. These are associated with pruritus and a burning sensation. 2072 B. isradipine. Persistent cyanosis of the hands C. The symptoms atheroembolism. cryoglobulinemia & Sneddon’s syndrome (ischemic stroke & livedo reticularis). Secondary livedo reticularis can occur with atheroembolism. Myxoedema dermopathy B. Pulses Harrison’s 18th Ed. bromocriptine. Patients complain of burning in the extremities that is precipitated Secondary acrocyanosis may result from hypoxemia. felodipine & amlodipine) are useful. Thromboangiitis obliterans A. All of the above lupus erythematosus (SLE). Atheroembolism A. A. Secondary livedo reticularis Venulitis occurring in thromboangiitis obliterans. 2073 A. with anorexia nervosa and orthostatic tachycardia syndrome. 1491 A. atherosclerosis of extremities. 2073 1521 Which of the following drug is useful in Raynaud’s phenomenon ? A. Behçet’s syndrome B. Postsynaptic alpha 1-adrenergic antagonist prazosin. Feet involved more frequently than hands Harrison’s 18th Ed. None of the above 1530 Which of the following is false about erythromelalgia ? In acrocyanosis. Chilblains B. Phenoxybenzamine Pernio (Chilblains) is a vasculitic disorder associated with exposure to cold. Sympatholytic agents methyldopa. May occur as an adverse effect of bromocriptine A. AJAY MATHUR Cardiology D. Harrison’s 18th Ed. Anorexia nervosa blockers. Normal pulses D. thoracic outlet compression syndrome. Infective hepatitis Secondary erythromelalgia occurs with myeloproliferative disorders such as polycythemia vera and essential thrombocytosis. 2072 A. and males are affected more frequently than females. doxazosin & terazosin may be 1529 Which of the following is false about erythromelalgia ? effective. Vasculitic disorder B. blood dyscrasias. All of the above D. All of the above Harrison’s 18th Ed. D. Raised erythematous D. All of the above lesions develop on lower part of legs & feet in cold weather. Trophic skin changes and ulcerations do not occur Erythromelalgia is characterized by burning pain and erythema of the extremities. Dihydropyridine calcium channel antagonists (nifedipine. pianists and keyboard operators. Burning sensation is precipitated by exposure to cold B. 2073 1522 Which of the following drug is useful in Raynaud’s phenomenon ? A. Atheroembolism Harrison’s 18th Ed. 1531 Venulitis is a feature of ? 1525 “Atrophie blanche en plaque” is the term given to ? Harrison’s 18th Ed. Deoxygenated hemoglobin in stagnant veins giving B. 2072 C. Pain. Behçet’s syndrome and homocystinuria may Primary livedo reticularis with ulceration is called atrophie blanche en plaque. Less-common causes include drugs. 2073 are normal & blanching does not occur. Harrison’s 18th Ed. May occur as an adverse effect of nifedipine Harrison’s 18th Ed. ulcers & gangrene do not occur. Characterized by burning pain and erythema of extremities 1523 Which of the following is false about acrocyanosis ? B. Nifedipine C. thromboangiitis obliterans. Amlodipine C. Terazosin D. guanethidine & phenoxybenzamine may be useful. 1526 Secondary livedo reticularis can occur with ? 1532 ‘Phlegmasia cerulea dolens’ refers to ? Harrison’s 18th Ed. 2072 B. cause venous thrombosis. The feet are involved more frequently than hands. dermatomyositis or polymyositis. antiphospholipid antibodies. or cryoglobulins. 2073 Harrison’s 18th Ed. Males affected more frequently than females B. Mechanical trauma A. by exposure to a warm environment and aggravated by a dependent position. SLE cyanotic hue to limb .228 MCQ’s FOR MEDICAL PROFESSIONALS BY PROF. SLE. such as SLE and paraneoplastic syndromes. Homocystinuria C. systemic D. Digital sympathectomy is helpful in those who are unresponsive to medical therapy. neuropathies. acute occlusion of large and medium. Thyroid acropachy C. 2073 Harrison’s 16th Ed. Primary livedo reticularis with ulceration D. None of the above C. connective tissue diseases. Exertional dyspnea B. 1 year During cardiac catheterization. AJAY MATHUR Cardiology 229 B. 2075 D. 1491 Harrison’s 18th Ed. A. Increased angiopoietin expression in smooth-muscle cells A. Klinefelter syndrome A. vascular lumen. 1577 disease) may be inherited in an autosomal dominant manner with variable penetrance. Mid respiration 1536 Lymphedema tarda usually begins after the age of ? Harrison’s 18th Ed. Intestinal lymphangiectasia syndrome C. mutations in C. 2076 B. pressures must be recorded only at ? D. There is a loss of apoptosis of smooth-muscle cells allowing their proliferation. Bacterial lymphangitis 1543 Pathobiologic process that result in pulmonary arterial hypertension is ? Familial forms of congenital lymphedema (Milroy’s disease) and lymphedema praecox (Meige’s Harrison’s 17th Ed. Filariasis In PAH. angina pectoris (RV ischemia). Noonan syndrome B. increased serotonin uptake in smooth-muscle cells. near syncope & peripheral 1534 Primary lymphedema may be associated with ? edema. physical examination shows increased jugular venous pressure. B. 2077 Primary lymphedema may be associated with Turner’s syndrome. and a palpable RV impulse. Any of the above A. Klinefelter’s syndrome. and lymphangiomyomatosis. 2075 1540 Which of the following is a feature of pulmonary hypertension ? A. Pulmonary Hypertension C. Pallor in markedly edematous DVT affected legs Most common symptom attributable to pulmonary hypertension is exertional dyspnea. Turner syndrome Harrison’s 18th Ed. Lymphedema tarda D. 2076 A. pressures must be recorded only at end expiration. All of the above Harrison’s 18th Ed. Reduced carotid pulse D. Pain on antigravity movement of DVT affected limb C. All of the above A. All of the above C. . DVT affected veins that are likely to embolize D. right-sided S4. 2075 D. lymphedema praecox has its onset at the A. Radiation therapy for breast carcinoma the bone morphogenetic protein-2 receptor. End expiration Noonan’s syndrome. Harrison’s 18th Ed. Other symptoms are fatigue. Lymphedema praecox which is a fall in mean pulmonary artery pressure (MPAP) >=10 mmHg & a final MPAP <40 mmHg. Increased serotonin uptake in smooth-muscle cells ? Harrison’s 18th Ed. Deoxygenated hemoglobin in stagnant veins giving A. All of the above Harrison’s 18th Ed. IV adenosine & IV epoprostenol reduce pulmonary artery pressure acutely B. increased angiopoietin expression in smooth-muscle cells & excessive thrombin deposition related D. B. Fatigue cyanotic hue to limb B. Most patients have an increased pulmonic component S2. Increased jugular venous pressure C. Syncope C. A. Inhaled nitric oxide time of puberty and lymphedema tarda usually begins after age 35. Lymphangiomyomatosis 1541 During cardiac catheterization. and emergence of apoptosis-resistant endothelial cells which can obliterate the Most common cause of secondary lymphedema worldwide is filariasis. Intravenous adenosine 1537 Milroy’s disease is best described as ? C. Yellow nail syndrome pulse. End Inspiration C. 2077 Congenital lymphedema appears shortly after birth. Pain on antigravity movement of DVT affected limb Chapter 250. 2075 C. a reduced carotid A. Palpable RV impulse 1535 Primary lymphedema may be associated with ? D. Chronic venous insufficiency 1539 Which of the following is the most common symptom 1533 ‘Phlegmasia alba dolens’ refers to ? attributable to pulmonary hypertension ? Harrison’s 16th Ed. intestinal lymphangiectasia syndrome. abnormalities in molecular pathways regulating the pulmonary vascular endothelial and smooth-muscle cells include inhibition of voltage-regulated potassium channel. Congenital lymphedema Inhaled nitric oxide. Peripheral edema D. Intravenous epoprostenol Harrison’s 18th Ed. Pregnancy to a procoagulant state. 25 years acutely with little effect on systemic vascular bed ? D. Bacterial lymphangitis D. and tricuspid regurgitation. 229 Cardiology MCQ’s FOR MCQ’s FOR MEDICAL MEDICAL PROFESSIONALS PROFESSIONALS BY PROF. All of the above B. 14 years 1542 Which of the following reduces pulmonary artery pressure C. B. Inhibition of voltage-regulated potassium channel 1538 Most common cause of secondary lymphedema worldwide is B. B. 35 years Harrison’s 18th Ed. syncope. 2075 In PAH. C. DVT affected veins that are likely to embolize D. yellow nail syndrome. Portal Hypertension of the following in pulmonary vascular smooth muscle ? Harrison’s 18th Ed.4. Esophageal involvement. Sclerodactyly. 1546 Which of the following is an ‘Endothelin Receptor Antagonist’ used in the treatment of idiopathic pulmonary arterial 1550 CREST syndrome includes all except ? hypertension ? Harrison’s 18th Ed. Calcinosis A. All of the above B. Bosentan D. AJAY MATHUR Cardiology 1544 Mutation involving the gene that code for which of the following C.5'-guanosine monophosphate (cyclic GMP) in pulmonary vascular smooth muscle which induces relaxation A. B. Microfibril-associated glycoproteins (MAGPs) PAH ? Harrison’s 18th Ed. Obstructive sleep apnea (OSA) A. Cyclic GMP C. 2080 Harrison’s 18th Ed. Calcium channel blockers A. Dermatomyositis arterial pressure in response to short-acting vasodilators at the time of cardiac catheterization should be treated initially with calcium channel blockers. Sclerodactyly C. Raynaud’s phenomenon B. All of the above Harrison’s 18th Ed. dermatomyositis. 2080 C. Iloprost account for most cases of familial IPAH. Endothelin receptor antagonists bosentan and ambrisentan are approved for treatment of PAH. Tenascin X of smooth muscle. Thrombosis D. Type II bone morphogenetic protein receptor (BMPR II) C. vardenafil. Raynaud’s phenomenon. Sjögren’s syndrome. A. 2078 A. Inositol 1. Sildenafil is a 1551 Which of the following can produce pulmonary hypertension phosphodiesterase-5 inhibitor. Nitric oxide increases the production of cyclic GMP.5-trisphosphate (IP3) (IPAH) ? Harrison’s 18th Ed. 2079 B. Treprostinil C. ? Harrison’s 18th Ed. Rheumatoid arthritis patients with PAH. Connective tissue growth factor (CTGF) 1548 Which of the following prostacyclins is used for treatment of D. Treprostinil includes ? D. PAH is commonest in ? B. B. Diuretic therapy relieves peripheral edema & reduces RV volume overload in presence of tricuspid regurgitation. Inhibitors of PDE-5 such as sildenafil. and Telangiectasia. It is common with CREST syndrome & scleroderma. It is less frequent in SLE. Sickle cell disease B. 2080 1547 Phosphodiesterase-5 is responsible for the hydrolysis of which A. Epoprostenol 1545 Conventional therapy for pulmonary arterial hypertension C. Cyclic AMP D. Patients who have substantial reductions in pulmonary D.230 MCQ’s FOR MEDICAL PROFESSIONALS BY PROF. and tadalafil reduce the breakdown of cyclic GMP. Sildenafil CREST syndrome includes Calcinosis. Diuretics Harrison’s 18th Ed. Scleroderma Anticoagulant therapy is advocated for all patients with PAH as warfarin increases survival of C. Epoprostenol B. Bosentan is contraindicated in patients who are on cyclosporine or glyburide concurrently. polymyositis & RA. 2077 Phosphodiesterase-5 (PDE-5) is responsible for the hydrolysis of cyclic 3'. Liver functions must be monitored monthly throughout the duration of use. Systemic lupus erythematosus D. a member of transforming growth factor (TGF)  superfamily. Endothelin-1 is related to familial Idiopathic pulmonary arterial hypertension D. Treprostinil is an analogue of epoprostenol. All collagen vascular diseases may be associated with PAH. 2079 A. Anticoagulation 1549 Of the following collagen vascular diseases. All of the above . 2079 Heterozygous germ-line mutations involving the gene that code the type II bone morphogenetic protein receptor (BMPR II). .. B 31 ..... A 65 ....... B 124 ......................... A 130 ...... B 198 .. C 80 .. A 87 ......... B 83 ........................................ D 163 .......................... B 143 .......... D 175 ........... D 153 ............ B 92 .............................. A 131 ........ D 99 ......... B 33 .. D 28 ............... B 168 .......... D 32 ..... A 120 ........... C 72 ............. C 167 .. D 57 .................................................. B 49 ..... A 55 .............. A 82 ................. C 102 . D 9 ... D 149 . C 144 ...................................... D 114 .................. A 138 ... B 62 ... A 74 .......................................... D 112 ... B 141 ...... D 178 ... C 150 . D 135 ... D 188 ......................... C 35 ....... A 56 .......................... D 19 .. B 75 .............................. A 140 .......... D 106 . A 125 ... C 52 .................... C 105 ....... A 42 ........................ D 186 ... A 38 .... C 63 .... D 115 ........... D 98 .......... A 78 ..................................... D 108 . D 156 ...................... D 171 ......... B 85 . D 190 ............ A 197 ........ A 40 ...................... D 15 ................... C 8 .... D 34 ............................ A 50 .......... AJAY MATHUR 231 ANSWERS CARDIOLOGY 1 ..... D 122 ............................ D 79 ...... A 93 ...... C 23 ...................... D 136 ............................................ A 44 ........................... C 14 ................................................... C 116 ............. D 169 ............... D 100 . A 6 ................... B 117 ........................... B 192 ........ A 107 ................. A ............ D 146 ................................... D 183 ................. D 139 .. C 24 ................. A 154 .... B 162 ............................ D 21 ... C 126 .................. A 69 ................ C 17 ..... B 53 ............................................... B 161 ............. D 30 ... C 51 .. D 84 .... A 129 ....... D 25 ............................... D 22 ........ D 170 .. A 179 ................................ D 12 ........................................ D 119 ........................................ A 160 ...... C 88 ...... B 71 . A 46 .......... B 111 ...... D 68 .................................................. C 7 ................................................ C 39 ................. D 60 ................. B 41 ......... D 157 ....... A 173 .. B 159 .............................. D 54 ......................................... D 164 .............. D 97 .... D 47 ..... B 158 .................... B 134 ................ A 172 .. C 193 ... B 182 .......... D 43 ......................... B 127 ... C 11 ... C 166 . C 101 ............................................... D 174 ........................................... B 2 ... B 155 ............... C 45 ................................ D 89 ..................... D 94 ........ D 180 ............ C 95 ......... B 147 ................... B 96 ............. B 123 .............................................. C 76 .... A 145 .. D 133 .... B 61 .. D 77 .......... C 70 .. A 104 ......................................... D 58 ........ B 118 ...... B 5 .......................... D 187 .. B 109 ...Answer MCQ’s FOR MCQ’s FOR MEDICAL MEDICAL PROFESSIONALS PROFESSIONALS BY PROF............................................................................ D 37 ....... C 196 ... B 191 . A 181 ................ D 176 ........................................ A 29 .... A 59 .. C 148 ............................. D 151 ...................... C 184 .................... D 165 ........ A 128 .. C 13 ....... D 48 .................... D 132 ....... B 16 ..... C 3 .. A 142 ...... A 20 .......... A 152 ............... C 91 ....................................... C 18 .......... A 36 ..................................... D 195 ....................................... B 194 ............................................. B 121 ......................... A 90 .............. C 177 ..................................... A 86 ... C 137 .. D 10 . B 73 ...................................... C 27 ........... D 189 ............................................... B 110 ............................ A 64 .............. C 67 ............. B 185 ..... B 26 . D 103 ............... B 66 ..... C 81 .. A 4 ............ B 113 ... ........................................................ B 274 ...................... B 328 ......... A 294 ......... A 248 . A 249 ..... B 308 .................... C 325 ................................................. B 330 ... A 344 ..................................... D 372 ........................... B 379 .. D 352 .. B 288 ...... B 218 ............ C 349 ........................ D 297 ...... A 276 ..... C 347 ........................... D 363 ..... AJAY MATHUR Answer ANSWERS CARDIOLOGY 199 .......... C 324 ......... D 367 .. D 223 ..... C 316 ..... C 234 .......... A 354 . D 237 ...................................... B 345 . A 278 ..... B 271 .......... B 266 ....... D 390 ...... D 259 .. C 366 . B 388 ..... B 243 ......... A 374 ...... D 286 .. B 250 ........ A 348 .......................................... D 283 .......................................................................................... A 307 ........ A 213 ............................ C 323 ..... D 385 . D 280 ...... D 230 . D 255 ............... B 209 ... C 322 .. B 317 ............ D 275 ........... A 268 ....................... A 231 ..................... D 305 . A 362 ...................... B 272 ....... C 376 ............................ D 225 ........................... C 303 ....... D 211 .... B 326 ........... A 264 ............ C 361 .... D 315 .. C 386 ................... A 219 ........ B 252 .. D 238 .... C 300 .. A 321 .................... D 260 .............................. A 299 .............. C 257 .................. B 346 . D 357 .............. B 201 ... A 314 ...... C 332 ........ D 222 ............ D 279 . B 270 ...................... C 267 .................... A 356 ...... D 339 .................... D 262 .... C 327 .................. A 306 .......................................................... C 351 ............................................. B 375 ....... B 284 .................... A 310 ... A 378 .......... A 285 ................................... D 371 ..... C 214 .................. A 355 ........ C 273 ... C 224 ................................................... C 216 .............. C 242 .... B 304 ........ D 359 ........... D 232 ............................. B 334 .... C 313 ... B 281 .................................... B 329 ........................... A 241 . B 312 ............................................ C 202 ................ C 333 .......... A 396 ...................... C 205 .......... A 336 ......... D 263 ............................................. D 337 ................. C 391 .............. B 292 ........ A 380 ........ B 335 . A 341 .............. D 365 .. D 369 ............................. D 221 .......................... A 227 ........... D 384 ..... C 358 ............ D 373 ........232 MCQ’s FOR MEDICAL PROFESSIONALS BY PROF............ A 387 ......................... A 381 ........ D 364 ... C 246 ......... D 215 ...... C 338 .................................................... A 254 .................. D 290 ................... B 247 ................................................................................ B 331 .. C 301 .... A 245 ...... B 383 .... D 277 ......... D 265 ........................ D 282 . D 377 ..... B 343 ............. A ...... C 360 ..................... D 392 ... D 206 ..................... C 368 ... D 293 ... D 207 .. B 370 ....... D 204 ..... D 395 ...... D 320 .. A 256 ..... C 208 .......... A 200 ........... B 350 ...... D 229 ................ B 296 ................... D 217 .............. D 261 .. D 226 ........ A 269 .......................... C 340 ............ B 295 .......... D 311 ... D 236 ................................... D 210 . B 298 .... C 251 ......................................... B 382 ......................................... B 253 ...... B 394 ....................... B 235 ................. B 244 . D 212 ..... D 353 ..... A 233 ....................................... D 203 ............... D 287 .......... D 302 ................... D 342 ............... C 239 ........................... C 240 .............. B 318 ........ A 393 .................. D 220 ................ D 309 ........ C 258 . C 389 .. B 228 ............. C 319 ...................................... D 289 ... D 291 ... ........... D 510 ... B 512 ................................... B 543 ............ A 568 ............. D 477 .... B 490 .......... D 470 ...... A 492 .......... B 494 .. C 433 ..................... B 410 ........... D 517 ...... A 540 . D 526 ........ C 505 ......... D 483 ... D 466 . D ......... C 475 ............................................ B 556 .. B 506 ........................ C 467 ........... C 465 .... C 419 .. A 563 .......... A 439 ... D 429 . B 500 ......................... A 590 ................... A 478 .............. D 434 .................... D 551 ............................................................................ D 399 ................... D 519 ............ D 476 ............. A 533 ............... D 422 ................................... A 572 ...... C 559 .............. C 415 ........................ D 537 ............. C 418 .... D 435 . D 488 .... C 436 ... C 462 .................................... A 413 ... B 444 ...... A 412 .......................... B 493 .................................. D 539 ................................. C 503 ...................... C 586 ... A 544 ..Answer MCQ’s FOR MCQ’s FOR MEDICAL MEDICAL PROFESSIONALS PROFESSIONALS BY PROF............... A 463 .... D 428 ........................ A 442 ... D 438 ............................. B 464 ......... D 579 .......... B 469 ............. D 406 ................................ D 489 .. D 407 ......................... B 417 ................ B 585 ............... D 485 ............................ D 592 ... C 561 .. B 534 .................... D 426 ................. D 550 ...... D 481 .................................... D 431 .............. D 554 ........ D 479 ...... B 454 ........................... D 528 ...... D 588 .................. D 516 ... B 432 .. A 401 .... D 562 .......... D 582 ............... D 583 .......... D 484 .... A 474 ............................. C 580 ........ C 450 .. B 547 .............................................. D 575 ............. B 584 ............. C 587 .. D 555 .. D 424 ...... D 573 .......... D 459 ............. D 496 .. D 398 ............ B 447 ......................... B 542 ....................... A 509 ........................... D 571 . D 577 .................................... A 558 ..................... C 455 ............ B 576 ......................... C 443 ............................ D 553 ......... D 420 .... B 589 ................ B 593 ...... D 427 ............ D 532 ........................ B 552 .................................. B 430 ............................................ A 520 .............. D 440 .................................................... C 437 ...................... C 445 ..... B 514 ............................................. D 502 ......... C 480 ........ A 423 ............................ A 421 .............. D 567 ........ D 531 ............ AJAY MATHUR 233 ANSWERS CARDIOLOGY 397 .... B 452 .. D 461 .................................. C 460 .... A 408 .. D 487 ......... D 499 ............. C 404 .......... C 498 ...... D 409 ... D 549 ......... B 400 ............................. D 425 ....... D 472 ........................ D 473 ......... D 557 ................. D 545 .. A 453 . D 497 . A 578 ......... D 522 ...... D 581 ...... C 501 .......... A 524 .. D 564 ...... B 536 ............. B 566 ......... A 511 ................................... D 446 ................... C 491 ........... B 441 .... D 574 .. B 504 ...... C 565 ......... B 570 .... A 448 ........ A 507 .... B 403 .... C 548 ........ A 456 ..... C 518 .. D 416 ....................... D 486 ... A 402 ......... C 569 ............... B 495 .............. D 411 ................................................ D 529 ....... A 482 .... C 451 .............................. D 405 ...... B 538 .......... A 525 ................ B 560 ..... A 594 ..... D 527 ............................................ A 468 ........................................... D 457 ................. A 591 ...... D 521 .... D 414 ....................................................................................................... B 508 ...... C 449 ........ B 541 ................. A 515 ......... D 523 ............ D 546 .. B 535 . A 513 .... A 458 ........... B 471 .. D 530 ...... ...... D 635 ................................ D 704 .... D 605 ............ D 634 .................................... C 664 ...................... D 748 .......... A 772 ................ B 671 ..... D 786 .. D 753 ......................... C 633 ...... B 640 . D 693 .. D 713 ................. D 632 ................ D 655 . C ........ C 743 ....................... A 725 .... D 710 ....... D 683 .............................. D 614 ... B 670 .......... D 682 ........ D 642 .......... D 790 ............... B 673 ......... A 728 . D 726 ......................... C 699 .............. A 761 .............................. D 636 ................................ D 656 .................. B 783 .... A 613 ......... A 760 ............................................... D 701 .................. B 724 ............................................ A 599 .. D 727 ............................. B 739 ................. A 686 ................................. C 775 .. A 660 ... A 767 ............. A 779 ................ D 649 ... A 740 ............ B 773 ...... D 616 ................................... D 604 .. B 652 ........ B 738 . D 681 .. D 623 ...... B 690 ................. D 778 .... D 788 ................... D 657 . D 755 . D 791 .................. A 658 ................. D 764 ..... C 628 ... D 668 ............... C 611 ................................. D 612 . D 676 ........... D 627 ......... C 721 ... A 781 ..... C 622 ....................... C 777 .......................... C 735 ................. C 667 ... C 733 ............ B 709 .... C 689 . D 697 ........................ A 736 .............. B 646 ....................................... B 617 ....... C 769 ... B 651 .............. B 691 .......... D 694 ....... D 685 ................................. A 737 .... C 763 ..................... D 645 ........................................................................................................ A 751 ..... C 608 .................................................. D 626 .......... D 711 ....... A 771 .................. B 705 ........ C 745 .................... D 700 ................... D 718 ........ C 742 ...... D 597 ................... D 644 ..... B 774 ..... D 746 ................. B 618 ...... C 712 ... D 707 ........................... A 615 .. D 703 ............ D 730 ... D 674 . C 719 .......................................... B 756 ............ C 620 ...................... B 708 ................. A 716 ................. D 630 .. A 789 ................................. D 722 .. B 654 ... D 610 ............. D 770 ........... D 600 ..... A 780 ................ D 639 ........... B 687 ............ D 747 ..... D 692 ............................... D 648 .......... D 754 ........................... D 624 ... D 706 ....... A 596 ...................... C 776 ...................... D 757 ......... C 792 ......................... B 602 ......... B 782 .................... D 638 ................... D 601 ...................... D 629 ........................... D 785 ....................... B 650 .. D 734 ........... D 679 .. D 749 .................................................. A 653 ....... C 609 ................................... C 752 ... C 720 .................. D 784 .. D 643 ...... D 695 ................ B 732 ........ A 759 ............. C 765 ............ AJAY MATHUR Answer ANSWERS CARDIOLOGY 595 ...................... C 661 ................................ C 696 ..... D 678 .......................... A 717 . D 641 ...... D 758 ................ A 665 ..... C 698 .. D 677 ......... B 715 .. B 723 ....... D 787 ........ D 675 .................................. C 731 ...............234 MCQ’s FOR MEDICAL PROFESSIONALS BY PROF.. A 741 ......... D 669 .. B 684 .... A 637 .... D 702 . B 750 .. C 621 ..... D 606 ......... D 607 ....................................... A 762 ...... A 598 .......... D 766 .................................. D 688 ..... D 659 .... C 603 .... B 662 ............... C 714 ..................................................................... D 672 .............. D 680 ............ C 666 ........ D 729 .......... B 744 ..... B 625 .. C 663 ...................................................... A 768 ......... A 619 .. C 647 ..... A 631 .... ..................... C 988 ................... A 796 .......................................................................................... C 807 ... C 975 ........ D 901 .................................. B 820 .............................. D 879 .................... D 922 .......................... C 904 ................. D 959 ...... C 815 ................................... D 874 ..... C 862 ...... D 844 .... D 804 ......... B 877 . B 836 .......... A 828 .................... B 941 .... A 925 ......... B 799 .................. D 937 ..................... A 821 .... D 806 ....................... D 967 ................................ B 890 . C 834 .......... D 957 ..... B 981 ..... A 908 ......................... C 856 .............................. B 817 .... D 810 .. D 818 ............... B 910 ...................... D 983 .. D 859 .......... A 909 ....... D 803 ................... D 900 ...... A 919 . C 872 ...... B 971 ........ D 945 .......................... D 823 ........... D 867 .......... D 917 .... B 882 ............... D 969 .......... D 985 ... D 801 ......... B 893 .... B 794 ....................................... D 960 . C 863 ......... D 911 ................................................... A 989 ...................... C 797 .............. D 923 ..................................... B 813 .................. B 940 ........ D 990 .. A 965 ......... D 889 ............... D 873 .. D 964 .... A 986 .... D 881 ...................... C 932 ................................................. C 885 ........... D 972 ....................... B 912 ............ B 935 ..... D 865 .. C 913 .. A 899 .... D 870 .. D 903 .. D 980 ..... A 924 ............................. D 829 ..... D 887 ......................... D ............. D 906 . C 811 ............... D 848 ...................................... D 979 ..... B 948 .. D 973 ........... D 891 ........... C 798 . B 982 .... C 800 ............................................................. B 838 .... B 954 .. D 984 ..... D 833 ........................................... B 929 ............ D 896 . C 933 .... D 876 .................................... C 851 .... D 858 .............................. B 827 .............. A 928 ... C 816 .......... D 962 ... B 946 ............ A 930 ..... B 978 ..... D 839 .......................Answer MCQ’s FOR MCQ’s FOR MEDICAL MEDICAL PROFESSIONALS PROFESSIONALS BY PROF............................. D 808 . D 822 .................. D 952 ............ D 942 . A 875 ..................... C 845 ......... C 795 ...... D 956 .. D 938 . C 825 ...................... C 866 .......... C 958 . B 871 ................ B 974 .................. A 831 ................... D 846 ...... B 894 .... D 895 ....... C 966 .......... D 842 ............................... C 835 ............... C 898 ......................... D 850 ........... B 915 ................ C 931 .. D 902 ....................... D 824 ........ D 944 ........ D 921 ................... C 855 ........ C 805 .. B 888 .... C 947 ..... A 916 .......... B 852 .. C 840 ............... D 963 .................. C 926 ........... B 809 .......... AJAY MATHUR 235 ANSWERS CARDIOLOGY 793 ..... B 819 .............. B 927 .............................. B 883 . A 951 .. B 843 ............... B 968 ......... D 905 ..................... D 802 .. D 857 ................... A 886 . D 914 ............................. A 868 . A 920 ....... D 943 .................. B 880 ........... D 878 .......................... B 949 ............. D 861 ............. D 907 ................... A 892 ...................... B 884 ............ C 950 ................... D 849 .................. D 826 ........... A 961 ........ D 953 ......... D 812 ........... B 939 ......... C 832 ...................................................................... C 864 ..................... A 936 ............... D 955 . D 847 ... B 897 .. A 853 ...................... C 934 ........ B 977 ....................... D 814 ......................... D 841 .......................................... D 830 ... A 837 ............ D 970 ......... D 854 ............................................. D 860 ....................... C 869 ............................ A 918 .......... B 976 ................. A 987 ........ ............... D 1179 .... B 1116 ....... A 1119 ........... A 1070 ............. B 1125 .................. D 1044 ............... A 1000 ......... C 1142 ................ A 1152 ... A 1064 ..................... D 1014 . D 1066 ................ D 1004 ...... C 1055 ............ C 997 .. C 1140 ............................. D 1063 ....... A 1027 ... D 1035 ........... D 1034 . B 1006 ....... D 1005 ............ C 1068 ................ C 1076 ............. D 1085 ...................................................... D 1165 ....... D 1040 .... C 1021 .... C 1075 . C 1104 .... D 993 . D 1039 ............. D 1127 .... D 1056 .......................... A 1042 ... D 1111 .. A 998 .................... B 1129 ......... D 1037 ......................... A 1157 ... C 1033 ....... B 1067 ................... B 1059 ...................... B 1145 ......... B 1181 ....... D 1184 ..................... D 1136 .... C 1106 ...... D 1081 .............. B 1169 ................................ A 1065 ....... C 1045 ............... C 1147 ................. D 1053 ..... C 1011 ...................... B 1168 .......... D 1049 .. D 1024 ..... B 1079 ... A 1013 ............................ C 1048 .............. D 1083 ..... D 1173 ........ D 1009 .. D 1117 ......... D 1188 ........... D 1090 ........... C 1149 .... C 1110 ..... D 1163 ........... D 1082 ............................... A 995 ........ D 1172 .................. C 1144 . B 1176 ............................ D 1088 ...... B 1107 .. D 1131 .......... B 1159 .. C 1047 ............... C 1102 ........ D 1026 . C 1161 .............. B 1166 ..... B 1098 .......... B 1093 . B 1175 ............ D 1054 ................ B 1143 .. A 1124 .................................. D 1017 ..................... A 1148 ........ A 1139 ..................... D 1050 ..... D 1061 .. C 1185 . D 1072 ..................................................................... B 1074 ......... D 1046 .. D 1020 . D 1146 . B 1031 ......... A 1103 .. A 1114 ........................................ B 1137 ............. A 1120 ........................... C 1134 .. C 1086 ... D 1171 .. B 1151 ...................... D 1158 ...... A 1023 .... D 1183 ........ A 1178 ................................ B 1126 ...... A 992 ...... A 996 ............ B 1029 .............................. C 1097 .... A 1036 ..... C 1008 ............ C 1150 ... B 1135 ................................. B 1028 .... C 1030 .......... A 1002 .............. A 1122 ............................ A 1113 ..... D 1069 ..... D 1095 .... D 1032 . D 1174 ................ D 1043 ......... B 1156 .............................. C 1062 ............................ B 1073 .......... D 1160 ...... D 1003 ... B 1060 ........... B 1001 ... A 1115 ....... D 1132 .... C 1071 ................. D 1041 ................................ D 1177 ... A 1087 .............................. C 1100 ..... A 1112 ...... B 1038 .......... D 1084 ... C 1012 .............. B 1101 .. D 1155 ........ D 1128 .......... D 1010 ..... D 999 ................ C 1052 ........ B 1022 .236 MCQ’s FOR MEDICAL PROFESSIONALS BY PROF. B 1118 .... D 1007 . C 1096 ........ D 1138 ................ C 1077 ................ D 1099 ......... A 1162 .................... D ............................................................................. A 1025 ...................... B 1051 ................. A 1186 ..................... A 1123 ......... B 1109 ................ D 1153 ........... D 994 ........ C 1170 ............ B 1108 ................... A 1182 .... D 1121 .. A 1057 ................... D 1018 ...... D 1167 ................ A 1080 ........................................... D 1164 ........................ C 1141 .. B 1078 .... C 1091 ............. D 1016 .... D 1154 .... A 1187 ............ A 1094 ................. A 1105 ...................................... AJAY MATHUR Answer ANSWERS CARDIOLOGY 991 . C 1058 .................... C 1089 .............. D 1092 ............. D 1015 .............. D 1133 ........ D 1130 ............. D 1019 ....... A 1180 ............... ...................................... C 1224 .... D 1306 ........ D 1288 ........... B 1367 ......................... D 1356 .... D 1363 ....... B 1376 ........ D 1201 ........... B 1202 ........ D 1337 ....... D 1208 ............... B 1197 ............ B 1364 ..................... D 1297 ......... B 1256 .................................... C 1205 ................. D 1317 ........ B 1321 .......... AJAY MATHUR 237 ANSWERS CARDIOLOGY 1189 .. A 1211 ........................... D 1346 .................. D 1347 .. B 1273 .. B 1253 ................. D 1285 ... C 1304 .. A 1203 .. D 1372 ....... B 1217 ... D 1247 ........... A 1251 ....... B 1360 ........... B 1343 ............... A 1339 ................................ D 1244 ............ A 1200 .... A 1254 ...... B 1362 ......... B 1248 ... A 1383 ... D 1283 ...................... D 1349 . D 1352 ....... B 1222 ................... D 1350 .......... C 1280 .................. B 1207 ....... D 1214 ................................. D 1385 .. A 1307 .. D 1269 .... C 1216 ......... A 1290 ...... D 1303 ... A 1378 ............... C 1348 . C 1206 ................................ C 1318 .. D 1263 ....... D 1296 .... C 1324 . D 1298 ............... B 1265 ........ A 1195 ....... B 1328 ... D 1371 ................... D 1331 ............................. A 1355 .............................................. A 1309 ........... D 1344 . D 1204 ... B 1294 . C 1320 ..... D 1215 ............................. A 1295 ........ D 1239 ..... D 1335 .................................. D 1281 ........... C 1369 ........................ B 1338 ........... B 1342 ............. C 1381 ......... D 1382 . C 1270 ............. D 1334 ... D 1278 ............... B 1305 ...... A 1249 .. D 1228 ......... D 1192 ..... D 1212 .............. C 1301 ........................ C 1260 ............. B ..................... B 1223 .................. D 1259 .... D 1242 .. B 1292 . B 1353 ... A 1198 ..... C 1361 . C 1258 .................................. D 1190 .... A 1379 ........... B 1365 ............... D 1308 ............................... D 1384 ........ D 1267 ..................... A 1218 ....................... A 1194 .................................... D 1300 ..... B 1221 ...... C 1199 ..................................... B 1374 ..... B 1323 ................. C 1241 ..... D 1370 ......... B 1236 ...... B 1351 .....Answer MCQ’s FOR MCQ’s FOR MEDICAL MEDICAL PROFESSIONALS PROFESSIONALS BY PROF...... C 1233 ............. D 1234 . A 1252 ..... B 1315 ............................. D 1213 .............................. C 1368 .............. D 1262 ....... D 1220 ................ C 1359 .......... D 1250 .......... D 1227 ........... C 1210 .............. D 1326 ........ D 1226 ..... B 1268 ................... D 1237 .......... D 1293 ....... A 1341 ...... A 1366 ........... C 1240 ....................... C 1243 ................... B 1299 ................... D 1340 ..................... B 1325 . A 1230 ........... C 1358 ................. C 1264 ......... B 1277 ............... D 1386 ............ D 1261 . D 1329 ................. A 1336 ................................. D 1282 ..... C 1266 .......... A 1276 ... B 1319 ........... D 1209 . B 1196 ............ D 1310 ........................... A 1238 .. D 1219 ................................... B 1354 ............ D 1229 ............... C 1357 ...... D 1257 ......... D 1232 ....... B 1345 ............ D 1327 ..... B 1191 ............ C 1330 ............. C 1274 ................................... B 1313 ................ B 1255 ..... B 1245 ......... C 1284 ............................. C 1311 .................. A 1291 ............ D 1246 ... B 1316 ... B 1302 .............. C 1373 ...... D 1289 ................ C 1287 .................................. A 1193 ................. A 1322 ...... C 1312 ..... B 1380 ... C 1225 ..................................... B 1375 ... A 1235 . C 1231 ...... C 1333 ..... D 1332 ............... A 1275 ................ D 1271 .. D 1377 .......... C 1272 ....... D 1279 ......... A 1314 ............... A 1286 .. .......................... B 1438 ............. D 1422 ............. D 1449 ..... A 1462 ................. A 1510 ..................... A 1452 .. D 1460 ...................... D 1424 ......... D 1541 ............. B 1551 ............... D 1454 ...................... C 1497 ........................ D 1539 ......... C 1538 ..... C 1451 ..... C 1481 .............. D 1388 ..... C 1523 ..................................................... D 1411 . B 1524 . D 1420 ............................... A 1416 ........ D 1392 .............................. D 1513 .................. C 1415 ............ A 1489 .......................238 MCQ’s FOR MEDICAL PROFESSIONALS BY PROF.......................... D 1531 ...... D 1505 ..... A 1476 ......... A 1410 ..... D 1517 .. C 1427 .................... A 1479 ........................ B 1535 .................... D 1536 .... D 1506 . C 1520 ............ C 1502 ................ B 1519 ................. C 1396 ..... B 1413 ......................... D 1494 ......... C 1444 ........ A 1522 . A 1453 ...... D 1490 ......................... C 1443 ................... D 1393 ... B 1529 ...................... C 1508 ................................ B 1482 ................. C 1441 ............. C 1543 .......... D 1409 .............. D 1461 ...... A 1469 ...... D ...................... B 1408 ........ D 1466 ....... D 1404 ............. B 1530 .................. B 1527 .... C 1514 ......... A 1435 .... D 1428 .. D 1496 ..... C 1458 ............................................................................. A 1463 .............. B 1512 ....... C 1516 . D 1432 ...... B 1546 ............... D 1389 ....................... B 1407 ............ D 1398 ............... B 1534 ..................... D 1509 ..... A 1430 ... A 1533 .. D 1450 .... D 1492 ...... B 1440 ....... D 1532 ... A 1406 ......................... B 1478 ............. D 1480 . C 1473 .... B 1423 ... D 1500 ....... D 1447 ... A 1542 .............. D 1545 .................... D 1499 ....... C 1431 ......... B 1437 ............ A 1464 . D 1400 ................................ D 1417 ..................... B 1472 ....... C 1445 .... D 1405 ...... B 1518 ..... D 1455 ........... B 1418 ............. D 1501 ....... A 1439 ......................... B 1515 ................ D 1395 ... D 1456 ... D 1525 .................. B 1484 . C 1465 ................... C 1429 . A 1547 ........ B 1544 ........ D 1495 ............. B 1483 ... D 1390 .... D 1412 ......... C 1485 .................................. AJAY MATHUR Answer ANSWERS CARDIOLOGY 1387 .. D 1399 ......................... D 1467 . A 1434 .............. D 1521 . D 1503 ... A 1436 . D 1487 ................... C 1397 ............ A 1448 ...................... B 1468 .... D 1391 ... C 1491 ..... D 1402 . D 1425 .. D 1426 .... B 1457 .. A 1401 .......... A 1459 . A 1504 .......... D 1474 .............................. D 1403 ...................... D 1507 ................ D 1419 .. A 1442 .... C 1477 .. D 1433 ..... B 1488 ......... B 1471 . A 1526 .... C 1511 ............................... D 1446 ............. C 1550 ................................... D 1486 .. D 1549 . C 1537 . C 1493 ..................... D 1414 ......................................... A 1475 ...... D 1421 .................... C 1548 .. D 1498 ...................... D 1470 .... A 1540 ............................... C 1394 ...... D 1528 . 1496 C. 2084 D. 2 weeks airway obstruction. Bronchiectasis . All of the above A. Unilateral phrenic nerve paralysis Harrison’s 18th Ed. Stridor occurs primarily during inspiration A. Pulmonary venoocclusive disease A. Pneumothorax 12 Which of the following about stridor is false ? D. Bronchitis and Tracheitis is included A. 1496 Obstructive lung diseases include disorders of the airways such as asthma. Asthma COPD. 2085 Acute shortness of breath is usually associated with laryngeal edema. Table 251–1 D. All of the above Harrison’s 18th Ed. A. 2085. C. Pulmonary vascular diseases D. Obstructive lung diseases are most common. 8 weeks Harrison’s 18th Ed. Parietal pleura 1 Which of the following is the most common of diseases of the B. 1496 C. neoplasia. None of the above C. bronchospasm. Chest pain from respiratory disorders results B. Bronchial airways A. Bronchiectasis desquamative interstitial pneumonitis (DIP). restrictive disorders and abnormalities of pulmonary vasculature. AJAY MATHUR Cardiology 239 7 Chest pain caused by diseases of respiratory system usually Chapter 251. Rhonchi is the sound created when there is free liquid in A. 2084 D. B. bronchiectasis. Heard over the neck as chronic cough ? D. Restrictive disorders from either diseases of parietal pleura (pneumothorax) or pulmonary vascular diseases (pulmonary hypertension). Pulmonary embolism D. Crackles are created when alveoli & small airways open & D. focal inspiratory wheeze heard over the neck. Bronchiolitis D. Crackles are typically inspiratory sounds 4 Which of the following is a disorder of pulmonary vasculature ? C. spontaneous pneumothorax. Unilateral parenchymal or pleural disease 3 Which of the following is a type of restrictive lung disease ? C. Pulmonary Langerhans cell histiocytosis Harrison’s 18th Ed. Pulmonary parenchyma Harrison’s 18th Ed. B. Neuromuscular disease A. B. myocardial A. Harrison’s 17th Ed. Pulmonary hypertension Harrison’s 16th Ed. Myocardial infarction the airway lumen B. 2084 C. It is a manifestation of upper A. Focal inspiratory wheeze 6 Cough that persists for more than how many weeks is defined C. pulmonary embolism. Abnormalities of chest wall and pleura Harrison’s 16th Ed. 2085 Stridor or a low-pitched. B. Wheezes are more prominent during expiration Harrison’s 18th Ed. pulmonary Langerhans cell histiocytosis & pulmonary hemorrhage with Goodpasture’s syndrome are associated with smoking. All of the above 9 Asymmetric expansion of the chest is due to ? Harrison’s 16th Ed. A. 239 Respiratory MCQ’s FOR MCQ’s FOR MEDICAL MEDICAL PROFESSIONALS PROFESSIONALS BY PROF. Parenchymal lung diseases 10 Which of the following about lung auscultation is false ? B. Obstructive lung diseases Lung parenchyma is not innervated with pain fibers. Manifestation of upper airway obstruction Harrison’s 18th Ed. 2084 D. High-pitched infarction. Visceral pleura respiratory system ? C. All of the above close with respiration 5 Acute shortness of breath is usually associated with ? B. Spontaneous pneumothorax B. 2084. or pneumothorax. Wheeze & rhonchi have same meaning D. Neoplastic diseases of lung 8 Which of the following is associated with smoking ? Harrison’s 18th Ed. 2085 A. 2085 Cough that persists for more than eight weeks is defined as chronic cough. Respiratory bronchiolitis . respiratory bronchiolitis–interstitial lung disease. 1583 Diseases of respiratory system are obstructive lung diseases.interstitial lung disease 2 Which of the following is included in the category of obstructive lung diseases ? C. Endobronchial obstruction of a large airway under the infectious pathology. 6 weeks of ? D. chronic obstructive pulmonary disease (COPD). C. All of the above A. 4 weeks 13 Crackles sounding like Velcro being ripped apart is a feature C. Approach to the Patient originates from involvement of ? with Disease of the Respiratory System Harrison’s 18th Ed. Pulmonary embolism 11 Which of the following about lung auscultation is false ? B. Wheezes reflect oscillation of airway walls with airflow limitation Harrison’s 18th Ed. and bronchiolitis. All of the above B. Diseases that result in fibrosis of the interstitium (IPF) result in crackles sounding like Velcro being ripped apart. Ptosis D. Primary or familial hypertrophic osteoarthropathy 17 Schamroth’s sign is seen in ? B. Lung cancer primary and secondary forms of HOA. 2856 Harrison’s 18th Ed. 19 Which of the following is a feature of Hypertrophic osteoarthropathy (HOA) ? 24 Cutis verticis gyrata is a feature of ? Harrison’s 18th Ed. All of the above D. and with interstitial fibrosis. Arthropathy of acromegaly A. Synovial effusions C. RBC A. periostitis. Tactile fremitus D. Wegener’s granulomatosis A. Interscapular region D. Crohn’s disease B. 2857 A. Pulmonary edema HOA is characterized by clubbing of digits and. Stimulation Harrison’s 16th Ed. Egophony is auscultation of sound “AH” instead of “EEE” when a patient arthralgias. Periosteal new bone formation B. 2857 A. Axillary region C. clubbing. All of the above D. Egophony helps to distinguish between crackles associated with alveolar fluid and those associated hyperhydrosis. Hemophilic arthropathy Hutchison’s Hunter C. Deep nasolabial folds develop. The skin becomes investigation for causes of clubbing. or rales are a sign of alveolar disease. and unusual skin features. in more advanced stages. and the forehead may become furrowed. Clubbing B. Egophony Primary or idiopathic hypertrophic osteoarthropathy is an autosomal dominant disorder characterized by periosteal new bone formation of distal extremities with irregular periosteal surface. Empyema Harrison’s 18th Ed. and its presence should alert clinician to initiate an Primary HOA is characterized by clubbing. Primary HOA C. Arthropathy of hemochromatosis B. Bases of lung Recent studies have suggested a role for platelets in the development of HOA. get fragmented in their passage through Chest hyperresonance. Air entry B. 2085 A. 20 Which of the following about primary hypertrophic 14 Which of the following is useful in distinguishing crackles osteoarthropathy is false ? Harrison’s 18th Ed. Percussion note C. IPF D. None of the above 23 Which of the following is not a feature of primary hypertrophic 18 Clubbing of the digits is not a sign of ? osteoarthropathy (HOA) ? Harrison’s 18th Ed. and there may be excessive sweating of the hands and feet. 21 Role of which of the following explains acquired HOA ? 15 Chest hyperresonance in pneumothorax is best appreciated Harrison’s 18th Ed. Even cyanosis is a late finding in cor pulmonale & is secondary thickened and coarse. 2086 Harrison’s 18th Ed. Anhidrosis Clubbing of the digits is not a sign of COPD. Smooth & undulating periosteal surface C. In secondary hypertrophic osteopathy due to pulmonary disorders. Periosteal new bone formation B. 1496 of fibroblasts by PDGF & transforming growth factor results in cell growth and collagen synthesis. Platynychia Primary or familial HOA is called pachydermoperiostitis or Touraine-Solente-Golé syndrome. normal lung. Hyperhydrosis D. Pleuritis C. Scleroderma B. WBC C. Platelet-endothelial activation in distal portion of extremities results in release of platelet- 16 All of the following can cause clubbing except ? derived growth factor (PDGF) leading to proliferation of connective tissue & periosteum. 2857 due to alveolar fluid and those due to interstitial fibrosis ? Harrison’s 18th Ed. Diabetes mellitus . 2857 D. lung and will be present in pneumonia but not in IPF. Crackles. these large platelet particles bypass lung and reach distal extremities. Clubbing begins insidiously at puberty. indicating endothelial activation or damage. Autosomal dominant disorder A. Bacterial endocarditis C.” This change in note is due to abnormal sound transmission through consolidated new bone formation occurs with smooth and undulating periosteal surface. present in venous circulation. this autosomal dominant trait with variable expression and is nine times more common in boys than in girls. AJAY MATHUR Respiratory B. 2857 at ? Harrison’s 18th Ed. Periostitis C. Clubbing of digits A. indicates air in pleural space (pneumothorax). particularly at apex. Interstitial lung disease D. 2086 A. The skin is often greasy. thickening of skin of face & forehead. ILD 22 Touraine-Solente-Golé syndrome refers to ? C. to a low cardiac output with systemic vasoconstriction & ventilation-perfusion mismatches in lung. Apex B. by periosteal new bone formation and synovial effusions. Megakaryocytes & large platelet particles. Koilonychia D.240 MCQ’s FOR MEDICAL PROFESSIONALS BY PROF. Elevated plasma levels of von Willebrand factor antigen have been found in patients with both A. Patients may have heavy-appearing eyelids and ptosis. exuberant periosteal phonates “EEE. COPD A. D. B. Platelets B. where they interact with endothelial cells. Usually C. In patients with cyanotic congenital heart disease and in right-to-left shunts. progressive enlargement of hands & feet. Hypertrophic gastropathy 31 Out of the following malignancies. Periostitis of hand bones Harrison’s 18th Ed. Ankles Clubbing of a single digit may follow trauma. Aneurysms of major extremity arteries C. All of the above Clubbing of the toes but not fingers is seen in an infected abdominal aortic aneurysm & PDA. Budd Chiari syndrome C. 2858 Table 336-3 C. Bone marrow failure A. 2857 A. drumstick appearance. Anemia Harrison’s 18th Ed. Arteriovenous fistulas of brachial vessels Patients with clubbing may experience a burning sensation in their fingertips. 2858 Pressure applied over the distal forearms and legs may be quite painful. Harrison’s 18th Ed. excessive sweating and oiliness of the skin and thickening of the facial skin are uncommon in secondary HOA. VSD A. Cystic fibrosis Harrison’s 18th Ed. The pain can be quite incapacitating and is aggravated by dependency and relieved by elevation of the 35 Clubbing of a single digit may be found in all except ? affected limbs. Oiliness of the skin In primary HOA. knees & ankles. With advanced B. Sarcoidosis A. Wrists Harrison’s 18th Ed. 34 Clubbing of toes but not fingers is found in ? 28 Which of the following is present in clubbing ? Harrison’s 18th Ed. Drumstick appearance C. with infected arterial grafts & with arteriovenous fistulas of brachial vessels. Hip 36 Which of the following is not a feature of thyroid acropachy ? D. Synovial hypertrophy Harrison’s 18th Ed. and the distal interphalangeal joint can be hyperextended. and cranial suture defects. Harrison’s 18th Ed. Hypothyroidism is observed. in tophaceous gout and sarcoidosis. wrists & elbows A. Periostitis of feet bones A. 2858 Table 336-3 Harrison’s 18th Ed. Clubbing A. Budd Chiari syndrome An objective measurement of finger clubbing can be made by determining the diameter at the base of the nail and at the distal interphalangeal joint of all 10 digits. Tetralogy of Fallot of the individual digit ratios is >10. gynecomastia. 2858 With clubbing. A. and the skin slightly erythematous. Coarctation of aorta C. clubbing of digits appears in virtually all patients with CF. Syphilis B. Marked periungual erythema is usually present. The overlying soft tissue may be swollen. bone marrow failure. Tetralogy of Fallot D. Clubbing 30 Which of the following is uncommon in secondary HOA ? C. Thickening of facial skin D. When clubbing is advanced. Patent ductus arteriosus B. Periosteal involvement in the distal extremities may produce a burning or deep-seated aching pain. pain & symmetric arthritis-like changes in shoulders. C. female escutcheon. Excessive sweating . Pleural tumors Associated abnormalities in primary HOA include hypertrophic gastropathy. Cyanosis 33 Clubbing of toes but not fingers is found in ? B. All of the above C. Trauma 29 In a patient with clubbing. Infected arterial grafts D. Thymoma wrists. AJAY MATHUR Cardiology 241 D. Hyperextended distal interphalangeal joint D. 2857 D. 241 Respiratory MCQ’s FOR MCQ’s FOR MEDICAL MEDICAL PROFESSIONALS PROFESSIONALS BY PROF. knees. C. Burning Harrison’s 18th Ed. 2857 D. the finger may have a Differential cyanosis refers to cyanotic and clubbed toes but not the fingers. B. Carcinoma of esophagus D. Knees lung disease. 2857 32 Unilateral clubbing may be found in which of the following ? A. Jaundice B. Periungual erythema B. All of the above 27 Fishman’s classification is used for ? Unilateral clubbing is seen in association with aneurysms of major extremity arteries. 2857 C. 2858 Table 336-3 B. Female escutcheon B. ankles. 2857 A. Harrison’s 18th Ed. All of the above 25 Associated abnormalities observed in primary HOA include all except ? Unlike primary HOA. skin over scalp becomes very thick & corrugated (cutis verticis gyrata). Lung metastases infrequently 26 The sensation experienced by patient in clubbed fingers is ? cause HOA. Itching A. Clubbing is present when the sum D. Tingling B. D. Infected abdominal aortic aneurysm D. Noninflammatory effusions occur in D. HOA is most common in bronchogenic carcinoma and pleural tumors. 2857 A. Tophaceous gout following joints except ? C. HOA is most common in ? B. Lung metastases C. pain may be present in all of the B. Harrison’s 18th Ed. Synovial hypertrophy is not found. ~ 20 % D. 47 Total lung capacity (TLC) is a sum of ? Harrison’s 17th Ed. Respiratory Function 44 The volume of gas that is exhaled from lungs in going from TLC to RV is ? 38 What is the approximate lung surface area ? Harrison’s 17th Ed. 9 Liters A. All of the above Average expiratory flow rate during the middle 50% of VC [forced expiratory flow (FEF) between 25 Respiratory system includes lungs. Expiratory reserve volume (ERV) A. 1586 Harrison’s 18th Ed. 50 m2 B. AJAY MATHUR Respiratory Hyperthyroidism (Graves’ disease) associated with clubbing & periostitis of bones of hands & feet D.V T . Tidal volume (VT) D. VC + FRC A. It is strongly associated with thyroid dermopathy. how many capillaries are present Harrison’s 17th Ed. 400 mL D. 1586 Harrison’s 18th Ed. 1586 Figure 246-1 41 What portion of the fresh air inspired with each breath does A. VC + ERV not reach alveoli but remains in conducting airways of lung ? Harrison’s 17th Ed. Lungs B. 5 Liters 49 Vital capacity (VC) is a sum of ? B. D. 60 m2 C. VC + FRC A. extensive capillary bed with ~1000 capillaries per alveolus exists. ~ 10000 Spirographic tracings consist of 4 lung capacities (TLC. 46 Total lung capacity (TLC) is a sum of ? 40 In pulmonary circulation. VC. Maximal early expiratory flow rate A. Functional residual capacity (FRC) B. IC . VC + RV Harrison’s 16th Ed. FRC + V T A. 45 FEF25–75% is also called ? 39 The respiratory system includes ? Harrison’s 17th Ed. IC + ERV + RV Harrison’s 17th Ed. Neuromuscular system C. 500 mL Chapter 252. 1589 B. Inflammatory bowel disease A. 80 m2 Volume of gas exhaled from lungs in going from TLC to RV is vital capacity.242 MCQ’s FOR MEDICAL PROFESSIONALS BY PROF. 1586 Figure 246-1 ~30% of inspired fresh air of each breath does not reach alveoli but remains in the conducting A. 2087 A. ~ 1000 D. 300 mL D. B. 37 Clubbing without cyanosis is frequent in ? Harrison’s 18th Ed. VC + ERV airways of lung (anatomic dead space). ERV & VT). Total ventilation each minute is ~7 liters (2 liters/minute of dead space ventilation and 5 liters/minute of alveolar ventilation). 1586 Figure 246-1 per alveolus ? A. VC + ERV B. All of the above C. VC + IC C. FRC + IC C. Infective endocarditis Harrison’s 17th Ed. 70 m2 D. IC & FRC) & 3 lung volumes (RV. ~ 10 % C. muscles) & pulmonary circulation. Disturbances of Normal individual at rest inspires ~12 to 16 times per minute. chest wall (diaphragm & intercostal & 75% of VC. Vital capacity (VC) C. Maximal pan expiratory flow rate D. each breath has a tidal volume of ~500 mL. 7 Liters Harrison’s 17th Ed. Maximal mid expiratory flow rate B. Chest wall (diaphragm & intercostal muscles) D. ~ 40 % Harrison’s 17th Ed. or FEF25–75% is also called maximal midexpiratory flow rate (MMFR)]. neuromuscular system. ~ 100 C. 1589 B. ~ 30 % 48 Total lung capacity (TLC) is a sum of ? D. 11 Liters is called thyroid acropachy. ~ 100000 In pulmonary circulation. Maximal late expiratory flow rate C. 1589 D. VC + FRC 42 Total ventilation each minute is approximately ? C. FRC + Tidal volume (VT) B. Alveolar surface area is ~70 m2 and volume of thoracic cavity is 7 liters. 200 mL C. 1586 Figure 246-1 C. 290 43 Each breath has a tidal volume of approximately ? A. 350 B. 2087 A. Jackhammer operators B. Tidal volume (VT) 58 Helium dilution & body plethysmography are used for B. ERV 61 Maximal inspiratory pressure (MIP) and maximal expiratory C. IC pressure (MEP) are useful in assessing ? Harrison’s 17th Ed. 0. Strength of respiratory muscles 55 Which of the following respiratory functions cannot be B. 1587 D. and inspiratory capacity (IC) are measured by having the Harrison’s 17th Ed. 1587 D. A. 1588 patient breathe into and out of a spirometer. 0. 0. RV B. 1587 Helium dilution and body plethysmography are used to measure RV. A. Midpoint of inspiration 54 Which of the following respiratory functions cannot be C. Lung recoil measured by a spirometer ? C.80 measured by a spirometer ? D. IC A. None of the above Harrison’s 17th Ed. Expiratory reserve volume (ERV) Harrison’s 16th Ed.75 53 Which of the following respiratory functions cannot be C. ERV and the maximal expiratory pressure (MEP). All of the above A. FRC. A. AJAY MATHUR Cardiology 243 B. 1587 D. Expiratory reserve volume (ERV) Harrison’s 17th Ed. inspiratory flow is most rapid at midpoint of inspiration. All of the above B. Pleural pressure . ERV C. IC Harrison’s 17th Ed. Terminal point of inspiration measured by a spirometer ? D. Endinspiratory . Functional residual capacity (FRC) A. 1500 C.endinspiratory pleural pressure 57 Helium dilution & body plethysmography are used for estimating ? D. 1587 A. ERV Harrison’s 17th Ed. ERV Harrison’s 17th Ed. IC + ERV estimating ? Harrison’s 17th Ed. Residual Volume (RV) B.endexpiratory pleural pressure Harrison’s 17th Ed. 1587 C. expiratory reserve volume (ERV). A. Functional Residual Capacity (FRC) A. 1587 During maximal inspiration from RV to TLC. Beginning of inspiration D. FRC & TLC. Increased FEV1/VC B. 1587 D. Inspiratory capacity (IC) A. VC 50 Transpulmonary pressure (PTP) refers to ? B. C. RV. 243 Respiratory MCQ’s FOR MCQ’s FOR MEDICAL MEDICAL PROFESSIONALS PROFESSIONALS BY PROF.00 Harrison’s 17th Ed. IC A.25 to 0. VC FRC is defined as the volume of gas in the lungs at the end of a normal exhalation. TLC A. IC 62 Abnormality noted on routine testing of pulmonary function D. Endexpiratory . B. FRC by a spirometer ? Harrison’s 17th Ed. Residual volume (RV) estimating ? C. ERV 51 Volume of gas in the lungs at the end of a normal exhalation is denoted by ? C. VC B. VC Strength of respiratory muscles is determined by estimating the maximal inspiratory pressure (MIP) B. Depression in FEF25–75% . IC 52 Which of the following respiratory functions can be measured D. 1587 Transpulmonary pressure or PTP is defined as alveolar pressure minus pleural pressure. IC + V T 56 Helium dilution & body plethysmography are used for C. 0.50 D. Vital Capacity (VC) 59 Normal value for the ratio FEV1 / FVC is ? B. and TLC cannot be measured by a spirometer because they include the volume of gas present in the lungs even after a maximal expiration. 1586 C. inspiratory flow is most rapid at ? B. 1587 D.75 to 0.50 to 0.pleural pressure D.80 to 1. Airway obstruction Harrison’s 17th Ed.ERV A. IC . VC 60 During maximal inspiration from RV to TLC. Alveolar pressure .alveolar pressure C. Total Lung Capacity (TLC) in early obstructive disease is ? VC. VC B. 10 mmHg RBC’s through pulmonary capillary bed. 1 /2 66 Normal mean pulmonary artery pressure is ? C.34 mL of O2 B. pulmonary arterial pressure (PAP) is C. 80 mmHg lowest at ? Harrison’s 17th Ed. All of the above D. A. Right middle lobe carrying ? Harrison’s 17th Ed. Coved early portion of flow-volume curve C. 5 mmHg Normally. Only abnormalities noted may be depression In an upright person. 1588 D. 1588 C. Increased VC C. None of the above D.150 dyn  s/cm5. PVR = 80(PAP – PCW)/CO A. CO is obtained by thermodilution method. are higher at lung apices than at the lung bases. Equilibration is complete within one-third of transit time of B. 1590 Normal mean pulmonary artery pressure is 15 mmHg. 1588 D. B. Harrison’s 17th Ed. Apex B. 15 mmHg 73 Hemoglobin is almost fully (~90%) saturated at a PO2 of ? D. Right middle lobe D. Lung base When fully saturated. All of the above In pulmonary parenchymal restrictive disease. Expiratory flow rates are preserved. 70 mmHg 67 In an upright person. PAP & PCW are measured directly ith flow-directed Swan-Ganz catheter in pulmonary artery. 1588 A. 2. Ratio RV / TLC increased B. B. VC & RV are decreased. FEV1/VC may be normal. 0. C. 250 to 500 dyn s/cm 5 B. Apical segment of lower lobe With early obstructive disease. Apical segment of lower lobe In obstructive disease. 1588 Harrison’s 17th Ed. O2 & CO2 diffusion equilibration is complete in what D. TLC is normal or increased. pulmonary perfusion is least at ? Harrison’s 17th Ed. All of the above time of RBC’s transit through pulmonary capillary bed ? With extraparenchymal pulmonary disease due to neuromuscular disease (respiratory muscle Harrison’s 17th Ed. each gram of Hb is capable of carrying 1. 63 Which of the following is true in obstructive lung disease ? 69 In upright position. perfusion is least at apex of lung & greatest at base. D. Decreased TLC A.34 mL of O2 68 In an upright person. 50 to 150 dyn s/cm 5 causing pulmonary extraparenchymal restrictive lung disease ? Harrison’s 17th Ed. 90 mmHg A.34 mL of O2 A. RV is not significantly affected & MIP is decreased.34 mL O2 . 1588 64 Which of the following is decreased in pulmonary parenchymal A. TLC D. 3. Apex of lung D. pulmonary arterial pressure (PAP) is B. TLC.34 mL of O2 highest at ? Harrison’s 17th Ed. Lung base 74 When fully saturated. Apical segment of lower lobe A. VC Pulmonary vascular resistance (PVR) = 80[Pulmonary arterial pressure (PAP) . 150 to 250 dyn s/cm 5 A. O2 & CO2 diffusion is rapid. FEF rates are 71 Normal value for pulmonary vascular resistance is ? preserved. AJAY MATHUR Respiratory C. 25 to 50 dyn s/cm 5 65 Which of the following is decreased in neuromuscular weakness B. Ventilation-perfusion ratios in FEF25–75% & coved terminal portion of forced expiratory flow-volume curve. PVR = 90(PAP – PCW)/CO B. 1588 C. VC Normal value for pulmonary vascular resistance is ~50 . 60 mmHg mean aortic pressure. VC is frequently decreased. TLC is decreased. Throughout transit time A. 1. Lung base C. residual volume is elevated and RV / TLC is 70 Pulmonary vascular resistance (PVR) is calculated by ? increased.pulmonary capillary C. PVR = 60(PAP – PCW)/CO restrictive lung disease ? B. as compared to ~95 mmHg for the normal A. 1590 D. TLC D.244 MCQ’s FOR MEDICAL PROFESSIONALS BY PROF. Apex of lung Hemoglobin is almost fully (~90%) saturated at a P O2 of 60 mmHg. 3 /4 Harrison’s 17th Ed. PVR = 70(PAP – PCW)/CO Harrison’s 17th Ed. 1 /3 B. Right middle lobe D. Harrison’s 17th Ed. C. each gram of hemoglobin is capable of C. RV wedge pressure (PCW)] / cardiac output (CO). 20 mmHg Harrison’s 17th Ed. 1588 C. 1589 weakness). MIP 72 Normally. 1588 A. 85 Disease associated with lowered DLCO is ? Harrison’s 17th Ed.3% of carbon monoxide is Amount of O2 transported dissolved in plasma is ~0. height 175 cm.75 % D. Goodpasture’s syndrome 80 Specific relationship between PaO2 & SaO2 depends on ? B. Methemoglobin B. Hb in RBCs in alveolar lumen binds CO leading to increased DLCO. 40 mmHg A. All of the above C.5 % C. ~ 3. 1590 B. Erythrocyte concentration of 2. Shunting D. Arterial P O2 of 60 mmHg corresponds to SaO2 of 90%. < 45 mmHg D. In healthy person breathing room air. Specific relationship between Pa O2 & SaO2 depends on temperature. Amount of O2 transported bound to hemoglobin is ~20 mL per deciliter of blood. 245 Respiratory MCQ’s FOR MCQ’s FOR MEDICAL MEDICAL PROFESSIONALS PROFESSIONALS BY PROF. Oxyhemoglobin Harrison’s 17th Ed. 60 mmHg B. Abnormalities in diffusion A.0 Litres . Appendix 14 81 CO-oximeter can distinguish which of the following ? A. 4. Decrease in PaO2 D.3 mL per deciliter of blood. 78 In a healthy young person breathing room air. 2 C. 1592 Harrison’s 17th Ed. 80 mmHg C. 0. Interstitial lung disease Harrison’s 17th Ed. ~ 0. All of the above C.3-diphosphoglycerate Goodpasture’s syndrome. inhaled in a single breath & held for ~10 seconds. 1592 79 Pulse oximeter calculates O 2 saturation by measuring absorption of how many wavelengths of light ? A. 1590 Harrison’s 17th Ed.3 mL O2 per deciliter of blood C. 82 What concentration of carbon monoxide is inhaled to assess 76 Amount of O2 transported dissolved in plasma is ? DLCO ? Harrison’s 17th Ed. 0. Elevation in PaO2 In mixed venous blood. D. Harrison’s 17th Ed. AJAY MATHUR Cardiology 245 75 Amount of O2 transported bound to hemoglobin is ? A. 1590 C. the PO2 is normally ~40 mmHg Hallmark of hypoventilation as a cause of hypoxemia is an elevation in PaCO2. ~ 10 mL per deciliter of blood C.3 % B. 77 In mixed venous blood.3 mL O2 per deciliter of blood D. Hypoventilation C. Temperature D. ~ 30 mL per deciliter of blood CO-oximeter uses at least four wavelengths of light and is capable of distinguishing oxyhemoglobin. the PO2 is normally about ? 83 The hallmark of hypoventilation as a cause of hypoxemia is ? Harrison’s 17th Ed. Emphysema A. 0. pH Diffusing capacity may be elevated in congestive heart failure and in alveolar hemorrhage as in C. weight 75 kg. 0. 1590 B. 1590 Figure 246-5 A. All of the above B. 100 mmHg D. ~ 2. < 60 mmHg Abnormalities in diffusion are rarely responsible for hypoxemia. PAO2 – PaO2 is normally <15 mmHg for subjects <=30 years and increases by ~3 mmHg per decade after age 30. the PAO2 . Elevation in PaCO2 C. Percentage of Hb saturated with oxygen . 3.3 mL O2 per deciliter of blood A. carboxyhemoglobin & methemoglobin. Emphysema A.SaO2 is displayed. Interstitial lung disease Harrison’s 17th Ed.3 mL O2 per deciliter of blood B. Decrease in inspired PO2 B. D.2 Litres Harrison’s 17th Ed.PaO2 is 84 Which of the following is rarely responsible for hypoxemia ? normally ? Harrison’s 17th Ed. Decrease in PaCO2 B. 5 D. what is the expected value of FVC ? Harrison’s 16th Ed. All of the above 87 In a normal man aged 40 years. ~ 20 mL per deciliter of blood D. emphysema & pulmonary vascular disease. 1590 Harrison’s 17th Ed. 1592 A. Pulmonary vascular disease B. 1591 A. Carboxyhemoglobin A. D. ~ 40 mL per deciliter of blood deoxygenated hemoglobin.1 % To assess the diffusing capacity of lung for carbon monoxide (DL CO). 8 Lowered DLCO is found in interstitial lung disease. pH & RBC concentration of 2. 1590 B. 1590 A. 0. 12 86 Diffusing capacity (DLCO) may be elevated in ? Pulse oximeter calculates O2 saturation by measuring absorption of TWO wavelengths of light by oxygenated & nonoxygenated Hb.3-DPG. ~ 1. < 15 mmHg B. < 30 mmHg C. Cyanosis C. the FIO2 is about ? Harrison’s 16th Ed. 76 % B. 140 mmHg what is the expected value of FEV1/FVC ratio ? Harrison’s 16th Ed. moist palms. reversal of flow through ductus results in shunting .6 Litres C. rather than the relative. 4. after rewarming. 86 % C. Carbon monoxide poisoning Harrison’s 17th Ed. Rubor or erythema D. height 175 cm. Normal pulses 92 In which condition. 138 mmHg 90 For cyanosis to occur. Wheal-and-flare formation 93 Which of the following site is best for detecting central cyanosis ? C. cyanosis does not occur ? In acrocyanosis . 4 gm % B. intensified by cold exposure. 66 % A. Ear lobule Harrison’s 17th Ed. 210 A.0 Litres A.8 Litres B. 5 gm % C. Blanching or pallor B. Nails superficial blisters. Trophic skin changes & ulcerations do not occur D. 1572 A.246 MCQ’s FOR MEDICAL PROFESSIONALS BY PROF. which of the following occurs ? C.8 Litres 95 At a height of 8000 feet. 100 mmHg D. 6000 meters 88 In a normal man aged 40 years. quantity of reduced hemoglobin that is Raynaud’s phenomenon is characterized by episodic digital ischemia & a triphasic color response important in producing cyanosis. 5. Cyanosis in toes but not fingers 94 Cyanosis usually becomes manifest at an altitude of ? C. 85 mmHg C. Normothermic C. edema. no trophic skin changes & ulcerations. Hyperthermic B. Cyanosis & erythema B. 80 mmHg C. 91 In peripheral cyanosis. B. Oedema Cyanosis becomes apparent when the concentration of reduced hemoglobin in capillary blood exceeds 4 gm/dL. weight 75 kg. Periheral circulatory failure pulses. Cyanosis in fingers but not toes the detection of cyanosis.2 Litres C. C. 3. 2183 98 Which of the following is false about acrocyanosis ? Harrison’s 17th Ed. 2.000 ft). manifested clinically by sequential development of digital blanching (vasoconstriction). 6 gm % D. It is the absolute. Cyanosis intensified by cold C. 1000 meters In PDA with severe pulmonary vascular disease. Edema & superficial blisters Harrison’s 17th Ed. hemoglobin concentration should be more than ? 97 Which of the following comes first in clinical manifestation of Harrison’s 17th Ed. 1573 D. 3 gm % A. 4000 meters D. All of the above A. 210 B. AJAY MATHUR Respiratory C. 96 % D. Appendix 14 96 At a height of 16000 feet. B. normal A. height 175 cm. 62 mmHg B. Any of the above D. weight 75 kg. 230 D. 1461 Examination of mucous membranes in oral cavity & conjunctivae rather than skin is more helpful in A. 230 Raynaud’s phenomenon ? Harrison’s 17th Ed. cyanosis (ischemia) & rubor (reperfusion) of fingers or toes following cold exposure & subsequent rewarming. Skin In frostbite. Cyanotic congenital heart disease 99 In frostbite. 1573 A. 115 mmHg D. 5. 120 mmHg 89 In a normal man aged 40 years. Oral mucous membranes 100 Differential cyanosis in patent ductus arteriosus means ? D. the extremities are ? Harrison’s 17th Ed. 230 D. what is the expected value of FEV1 ? Cyanosis usually becomes manifest at an altitude of 4000 meters (13. Harrison’s 16th Ed. wheal-and-flare formation.8 Litres B. 3. 211 resulting persistent peripheral cyanosis of the hands & feet. after rewarming there is cyanosis & erythema. All of the above A. 2000 meters D. None of the above A. D. Appendix 14 A. the FIO2 is about ? Harrison’s 16th Ed. and B. Hypothermic A. Episodic peripheral cyanosis of the hands B. there is arterial vasoconstriction & secondary dilation of capillaries & venules with Harrison’s 16th Ed. Cyanosis in lips & cheek but not fingers & toes Harrison’s 17th Ed. A. pulmonary parenchyma B. 247 Respiratory MCQ’s FOR MCQ’s FOR MEDICAL MEDICAL PROFESSIONALS PROFESSIONALS BY PROF. Intercostal muscles D. Hypoglycemia B. AJAY MATHUR Cardiology 247 of unoxygenated blood to descending aorta. Charcot’s aneurysm A. bronchiectasis. B. None of the above 106 Which of the following statements about stridor is false ? Harrison’s 16th Ed. A. Occurs in a patient with retropharyngeal growth A. lung abscess. Vincent’s angina Harrison’s 17th Ed. 213 A. Vincent’s angina arteriovenous malformations). Superior vena cava syndrome C. Affects III. Laryngitis D. Ludwig’s angina A. IV and V costochondral joints. All of the above Harrison’s 17th Ed. 212 D. 1010 characterized by painful. Beyer’s aneurysm B. Pneumothorax In pulmonary tuberculosis. coughing. All of the above A. Acute epiglottitis 101 Which of the following is false about hemoptysis ? D. Costochondral articulations C. inflamed gingiva with ulcerations of interdental papillae that bleed easily. idiopathic pulmonary hemosiderosis). Massive hemoptysis is > 600 mL/day Harrison’s 17th Ed. Pneumothorax D. bronchogenic carcinoma or bronchial carcinoid tumors). Pontomesencephalic lesions 112 Which of the following is false about costochondritis ? C. Chapter 340 C. Ludwig’s angina 103 A dilated vessel in a pulmonary tuberculosis cavity is called ? Vincent’s angina (acute necrotizing ulcerative gingivitis or trench mouth) is a form of gingivitis Harrison’s 17th Ed. Airways Ludwig’s angina is a rapidly progressive cellulitis of sublingual & submandibular spaces following B. C. B. Rasmussen’s aneurysm 110 Harrison’s groove is characteristic of ? B. None of the above A. Lemierre’s disease (pneumonia. All of the above Harrison’s 17th Ed. 1496 Costochondritis is pain of costochondral articulations without swelling. Aspergillosis. Occurs in an infant with croup Harrison’s 16th Ed. Rickets cavity (Rasmussen’s aneurysm). C. Bouchart’s aneurysm Harrison’s 15th Ed. Harrison’s 17th Ed. D. Pulmonary parenchyma infected or recently extracted tooth (lower II & III molars). tuberculosis. Pleural effusion 104 Kussmaul’s breathing is seen in ? Harrison’s 17th Ed. 1583 D. Pigeon-breast Harrison’s 17th Ed. Occurs primarily during inspiration B. pulmonary C. IV and V costochondral joints D. Alkaline pH 108 “Hot potato” voice is a feature of ? C. B. Goodpasture’s syndrome. Patients speak in a “hot potato” voice. Herpes zoster Harrison’s 17th Ed. Oropharyngeal candidiasis (thrush) Massive hemoptysis is defined as >600 mL of blood produced in 24 hours. Observed mostly in women over age 40 years Rapid. 1718 111 Tietze’s syndrome refers to painful swelling of ? Harrison’s 17th Ed. Lemierre’s disease 102 Hemoptysis can originate from disease of ? C. 2183 A. Croup syndrome differential cyanosis. Toes. deep (Kussmaul) breathing usually implies metabolic acidosis but may also occur with pontomesencephalic lesions. deep inspirations or twisting motions of chest. but not fingers become cyanotic and clubbed . A. or pulmonary vasculature (pulmonary thromboembolic disease. 212 Hemoptysis can originate from disease of airways (acute or chronic bronchitis. Pulmonary vasculature 109 “Trench mouth” is related to ? D. observed mostly in women over age 40 years and tends to affect the III. Rachitic rosaries B. 1735 Neck radiographs in acute epiglottitis (supraglottitis) typically reveal an enlarged edematous epiglottis A. Bacterial meningitis B. A. All of the above 105 Kussmaul breathing may also occur with ? Tietze syndrome is manifested by painful swelling of one or more costochondral articulations. Bright red colour referred to as the thumbprint sign. D. Ribs B. Pain of costochondral articulations without swelling B. 227. Kyphoscoliosis 107 Radiographic “thumbprint sign” is seen in ? C. Oropharyngeal candidiasis (thrush) cystic fibrosis. aggravated by sneezing. 1718 usually II or III costochondral joint presenting as anterior chest pain radiating to arms or shoulders. Barrel chest . Represents flow through a narrowed lower airway 113 Pectus carinatum is also called ? C. C. Emphysema D. massive hemoptysis may occur from rupture of a dilated vessel in a C. 2183 D. 1569 D. Diabetic ketoacidosis A. Also called Funnel chest all except ? N Engl J Med 2005. Smooth-muscle cells of resistance pulmonary arteries B. C. Pneumomediastinum D. Pulmonary function tests are nearly normal A. Chromaffin cells of fetal adrenal medulla are uncommon & pulmonary function tests are nearly normal. Kyphoscoliosis D. 1210 N Engl J Med 2005.353:2042-55 B. Vomiting C. Tonically held tongue 120 All of the following can cause diffuse nodular opacities in chest C. Intestines Pleurodynia presents with acute onset of fever & spasms of pleuritic chest or upper abdominal pain. AJAY MATHUR Respiratory Pectus carinatum is also called Pigeon-breast. Pneumoconiosis sternum protruding anteriorly. Pain is usually left submammary C. Angina inversa D. which results in a depressed panel in the anterior chest. Lower sternum commonly involved B. None of the above C. lower portion of sternum is displaced posteriorly and the anterior ribs are markedly bowed. Respiratory symptoms D. Metastatic neoplasm . Herpes zoster infection of thoracic nerve roots C. Hodgkin’s disease D.248 MCQ’s FOR MEDICAL PROFESSIONALS BY PROF. B. pain occurs after excercise B. It is the reverse of pectus excavatum with the B. Confusion A. Kidneys D. Xiphoidalgia 124 Vasoconstrictors produced by endothelium are all except ? 117 Which of the following is false about Da costa’s syndrome ? N Engl J Med 2005. Bilateral Asterixis Harrison’s 17th Ed. 123 Vasodilators produced by endothelium are all except ? N Engl J Med 2005. Endothelin B. Voluntary limb motion X-Ray except ? D. Pulmonary vascular endothelium B. Neuroepithelial bodies in lungs D. Chronic anxiety A. All of the above Harrison’s 16th Ed. Most cases are due to coxsackievirus B and occur during epidemics. Ductus arteriosus Sulfur granules are characteristic of actinomycosis.353:2042-55 116 Mondor’s disease refers to ? A. Hepatic vein C. Fetoplacental arteries D. 122 Specialized tissues that sense the local oxygen tension include 115 ‘Sulphar granule pus’ is seen in infection with ? all except ? Harrison’s 16th Ed.353:2042-55 A. Also called epidemic pleurodynia A. Diaphragmatic paralysis C. 937 N Engl J Med 2005. Tension pneumothorax 127 Asterixis can be found in ? Harrison’s 16th Ed. Both of the above C. Subcutaneous emphysema in suprasternal notch and Hamman’s sign (crunching or clicking noise synchronous with heartbeat A. Pain is like that of pleurisy B. 1624 In pneumomediastinum. Sarcoidosis 114 Which of the following is false about “Pectus excavatum” ? Harrison’s 16th Ed. Systemic veins In Pectus excavatum.353:2042-55 A. Any tonically held posture & best heard in left lateral decubitus position) are characteristic features. 1569 121 Specialized tissues that sense the local oxygen tension include A. Nitric oxide A. Prostacyclin B. Mycoplasma B. 1661 B. Tuberculosis A. None of the above 118 Which of the following is false about Bornholm disease ? 125 Angiotensin-converting enzyme (ACE) is present in ? Harrison’s 17th Ed. Thromboxane A2 C. Eosinophilic granuloma D. 1497 A. Liver C. 1624 119 Hamman’s sign is characteristic of ? A. Both of the above D. Seizure B. It is associated with congenital ASD or VSD & severe prolonged childhood asthma. Actinomycosis C. Subcuteneous anterior thoracic phlebitis B. None of the above D. there is gas in the interstices of mediastinum. 126 Most specific signs of a metabolic encephalopathy is ? Harrison’s 16th Ed.353:2042-55 A. None of the above D. Due to Coxsackie virus C. None of the above D. Glomus cells of carotid body C. Often. GPRA Severity of asthma does not vary significantly within a given patient. Most patients with asthma in affluent countries are atopic B. T lymphocytes A. Goblet cell metaplasia C. Thickening of basement membrane D. characteristic histologic finding in airways is ? 133 Which of the following is called ‘house-dust mite’ ? Harrison’s 17th Ed. Onset of asthma in adulthood rarely become permanently A. 1597 Harrison’s 18th Ed. 139 In asthma. 1508 A. 2102 subepithelial collagen deposition is a feature of ? A. 2104 B. Betula verrucosa C. Large airway 137 Which of the following is strongly linked with asthma in genetic association studies ? B. DPP-10 D. Dermatophagoides pteronyssinus B. but in adulthood the sex ratio is equalized. characteristic finding is thickening of basement membrane due to subepithelial types III Dermatophagoides pteronyssinus is called house dust mite. Novel genes associated with asthma include ADAM-33. ORMDL3 131 The term “atopy” in Greek means ? C. Terminal bronchiole A.Asthma D. Harrison’s 16th Ed. and activation of mucosal mast cells. Commonly have nasal polyps 129 Which of the following about asthma is false ? Patients with nonatopic or intrinsic asthma usually have more severe. Phleum pratense D. Normal parenchymal attachments A. Sex ratio in adults is equal A. IgA antibody Harrison’s 18th Ed. 249 Respiratory MCQ’s FOR MCQ’s FOR MEDICAL MEDICAL PROFESSIONALS PROFESSIONALS BY PROF. ~ 80 % Autopsy results from patients of established asthma show airway mucosa infiltrated with activated D. Respiratory bronchiole B. C. DPP-10 and GPRA. AJAY MATHUR Cardiology 249 C. IgG antibody 254 . All of the above C. A. D. later onset and persistent Harrison’s 18th Ed. Atopic asthma . ORMDL3 is a member of a gene family that encodes transmembrane proteins anchored in the C. 134 Atopy is due to the genetically determined production of ? 140 Thickening of basement membrane in airway mucosa due to Harrison’s 18th Ed. male:female ratio is 2:1. B. 2102 asthma. Mischief 138 Airway mucosa in asthma is infiltrated with ? 132 Allergic rhinitis is found in what percentage of asthmatic Harrison’s 18th Ed. ~ 40 % C. 10 . Inflammatory disease of airways C. 2102 135 Which of the following statements about nonatopic or intrinsic A. All of the above 130 Asthma is a disease of ? Asthma is polygenic. Severity of asthma varies significantly within a patient 136 Novel gene associated with asthma is ? Harrison’s 18th Ed. Allergic rhinitis is found in over 80% of asthmatic patients. Normal serum IgE levels In childhood asthma. 2104 patients ? A. Explosive endoplasmic reticulum and asthma patients have elevated expression levels of this gene. Activated eosinophils Harrison’s 18th Ed. Medium airway Harrison’s 18th Ed. IL23R A.344:30 D. ADAM-33 asymptomatic B. TNFAIP3 D. 2478 C. None of the above B. 2102 B. Ambrosia artemisiifolia In asthma.12% adults and 15% children affected by asthma asthma is false ? B. Degree of inflammation is poorly related to disease severity. Peak age of presentation is 3 years Harrison’s 18th Ed. ~ 60 % D. Negative skin test to common inhalant allergens D. ~ 100 % eosinophils and T lymphocytes. 2103 C. PTPN2 N Engl J Med 2001. Thickened airway smooth muscle B. Activated mucosal mast cells B. Inert Asthma is caused by a combination of poorly understood genetic & environmental factors. IgE antibody A. 2102 A. 2103 C. D. IgM antibody Atopy is due to the genetically determined production of specific IgE antibody 128 Which of the following statements about asthma is false ? Harrison’s 18th Ed. Have mild asthma D. and V collagen deposition. Out of place Orosomucoid like 3 (ORMDL3) has been strongly linked with asthma in genetic association studies. LTD 4 Activated mast cells are found at the airway surface and also in the airway smooth-muscle layer in C. Patients with eosinophilic bronchitis A. Induction of Haptines 144 In asthmatic patients. Aspirin-sensitive asthma responds to usual therapy with ICS. Harrison’s 18th Ed. and pediatric asthma. Goblet cell dysplasia D. All of the above D. and the  and  chains C. occupational. IgE gets attached to human mast cells and ? Harrison’s 16th Ed. CD 8+ Harrison’s 18th Ed. Prostaglandin D2 .357:1841-54 Harrison’s 18th Ed. All of the above A. Asthma is triggered by COX inhibitors. All of the above B.250 MCQ’s FOR MEDICAL PROFESSIONALS BY PROF. prepares D. a process termed sensitization. 1947 141 Which of the following is a histopathologic feature of a small airway in fatal asthma ? A. PGI2 A. 2104 A. characteristically provokes rhinorrhea. Harrison’s 16th Ed. All of the above A. Normal subjects Harrison’s 16th Ed. D. Cysteinyl-leukotrienes C. Monocytes B. 147 In “sensitization”. nonatopic. urticaria. and wheezing. which together cross the B. even in small doses. Although antileukotrienes should be effective in these patients. All of the above B. lumen 148 Which chain of FcRI is solely responsible for IgE binding ? occlusion with a mucous plug. All of the above Mast cells release bronchoconstrictor mediators like histamine. Thromboxane B. 1508 and systemic mastocytosis. mast cells are localized to airway smooth muscle layer. Eosinophils A. Structural inflammatory cell A. 1947 vasodilation and increased numbers of blood vessels (angiogenesis). All nonselective COX inhibitors should be avoided. B. thickened airway wall due to subepithelial collagen deposition. All of the above 146 Mast cells release which of the following bronchoconstrictor 152 Cysteinyl leukotrienes were earlier known as ? mediators ? N Engl J Med 2007. LTE 4 asthma patients. 2104 Figure 254-1 B. 1947 D. LTC 4 D. AJAY MATHUR Respiratory B. Pathology of asthma is remarkably uniform in different types of asthma. 1598 C. CD34+ D. Goblet cell aplasia C. Urticaria plasma membrane seven times. Asthma patients 151 Which of the following is a cysteinyl leukotriene ? B. Occasionally. Lymphocytes C. Production of T cell derived cytokines A. increase in basement membrane thickness and airway smooth muscle. Harrison’s 16th Ed. Any of the above these cells for subsequent antigen-specific activation. Parenchyma There is a genetic predisposition to increased production of cysteinyl-leukotrienes with functional polymorphism of cys-leukotriene C synthase. Fibroblasts Aspirin-sensitive asthma is a well defined subtype of asthma that is usually preceded by perennial C. 145 Activated mast cells are found at the airway surface in ? they are no more effective than in allergic asthma. aspirin-sensitive. Smooth muscle rhinitis and nasal polyps in nonatopic patients with a late onset of the disease. Slow-reacting substance of anaphylaxis B. Leukotrienes 143 Human mast cells originate from ? B. Involvement of airways may A. and neurotrophins. one . Histamine B. anaphylaxis. A. Aspirin-sensitive asthma D. CD 4+ 150 Mechanism by which aspirin produces bronchospasm is ? C. PGI1 Harrison’s 16th Ed. Basophils Harrison’s 18th Ed. conjunctival irritation. goblet cell metaplasia. chemokines.  chain 142 Mast cell is the key effector cell of the biologic response in ? C. Anaphylaxis provide signal transduction. Nonatopic asthma C. facial flushing. 1947 C. but selective COX2 inhibitors are safe to use when an anti-inflammatory analgesic is needed. including atopic. Goblet cell metaplasia The fixation of IgE to human mast cells and basophils. Epithelial cells D. aspirin desensitization is necessary. 2104 but this should only be undertaken in specialized centers. Chronic overexcretion of cysteinyl leukotrienes Harrison’s 17th Ed. D. but not in normal subjects or patients with eosinophilic bronchitis. even in their absence. and two disulfide-linked  chains.  chain Harrison’s 16th Ed. growth factors. 1514 C. prostaglandin D 2. Histopathologic feature in a small airway in fatal asthma include shed or friable epithelium. 2115 D. D. but is persistent In asthmatic patients.  chain be patchy. All of the above 149 Bronchial asthma is associated with increased levels of ? Mast cell is the key effector cell of the biologic response in allergic rhinitis. Allergic rhinitis FcRI is composed of one . A. mast cells are localized to airway ? B. Aspirin. and cysteinyl- leukotrienes besides several cytokines. The  chain is responsible for IgE binding. Monocytes C. 2105 D.augmenting protein (FLAP) C. All of the above Synthesis of leukotrienes from substrate arachidonic acid is initiated by 5-lipoxygenase enhanced by 5-lipoxygenase . whereas CCL17 (TARC) and CCL22 (MDC) from epithelial cells attract TH2 cells via CCR4.344:30 Blocking antibodies to IL-5 causes a profound & prolonged reduction in circulating & sputum A.activating protein (FLAP).associated protein (FLAP) B. T H1 Fc portion of circulating IgE binds to high-affinity receptors (FcRI) present on the surfaces of mast B. Chemokines are involved in attracting inflammatory cells from the bronchial circulation into the airways. All of the above Cysteinyl leukotrienes C4. Interleukin-13 N Engl J Med 2001. Initiate inflammatory response C.assisted protein (FLAP) D.WBCs & “trienes” . N Engl J Med 2007. and lipoxins. Lipoxins Harrison’s 18th Ed. IL-4 B. Basophils B. 5-hydroxyeicosatetraenoic acid 161 Blocking antibodies to which of the following causes a B. Eosinophils Harrison’s 18th Ed. 2105 A.activating protein (FLAP) A. They also release anti-inflammatory mediators (IL-10). and migrate to local lymph nodes to present allergenic peptides to uncommitted T-lymphocytes for programming to allergen-specific T cells. 2106 D. IL-12 B. 2105 Harrison’s 18th Ed. Dendritic cells take up allergens. Three sulphar molecules D. 154 Synthesis of leukotrienes from arachidonic acid is initiated by 5-lipoxygenase in concert with ? 160 In asthmatic patients. All of the above A. Specialized eosinophil-like cells B. T H2 cells and basophils. but is not associated with reduced AHR or asthma symptoms. All of the above A. B. Exacerbation molecule D. 2105 A. Three metabolic pathways C. 2106 Harrison’s 18th Ed. Interleukin-4 5-oxo-eicosatetraenoic acid. D. 153 “Trienes” in word leukotriene stand for ? N Engl J Med 2007. 251 Respiratory MCQ’s FOR MCQ’s FOR MEDICAL MEDICAL PROFESSIONALS PROFESSIONALS BY PROF. Crystal structure molecule C. T H3 D. which are the major antigen-presenting cells. Thymic stromal lymphopoietin (TSLP) D.357:1841-54 A. Basophils 162 Which of the following cells predominate in normal airways ? C. Interleukin-2 Products of the 5-lipoxygenase pathway besides leukotrienes are 5-hydroxyeicosatetraenoic acid. AJAY MATHUR Cardiology 251 C. Dendritic cells are specialized macrophage-like cells in airway epithelium. 5-oxo-eicosatetraenoic acid profound and prolonged reduction in circulating and sputum eosinophils ? C. Specialized macrophage-like cells D. Activated by allergens B. which of the following instructs dendritic N Engl J Med 2007.3 conjugated double bonds) is a family of products of 5- lipoxygenase pathway of arachidonic acid metabolism. IL-5 A. D4 & E4 were previously termed “slow-reacting substance of anaphylaxis”. T H4 157 Macrophages are derived from ? 163 Which of the following cells predominate in asthmatics ? Harrison’s 18th Ed. 2106 A. C.357:1841-54 159 Which of the following about macrophages is false ? Harrison’s 18th Ed. Eotaxin (CCL11) is selectively attractant to eosinophils via CCR3 and C.357:1841-54 cells to release chemokines that attract TH2 cells into airways ? Harrison’s 18th Ed. B. process them to peptides. Capacity to form leukotrienes from Macrophages are activated by allergens via low affinity IgE receptors (FcRII). 5-lipoxygenase . 2105 A. None of the above Leukotrienes (“leuko” . inflammatory response via release of certain cytokines. 5-lipoxygenase . Interleukin-5 156  is intracellular in which of the following ? FcRI- D. Release anti-inflammatory mediators D. Eosinophils A. 5-lipoxygenase . Three cysteinyl molecules B. Cytokine thymic stromal lymphopoietin (TSLP) released from epithelial cells in asthmatic 155 Products of 5-lipoxygenase pathway besides leukotrienes are ? patients instructs dendritic cells to release chemokines that attract TH2 cells into airways. Macrophages initiate arachidonate is confined to leukocytes. Any of the above D. T H2 C. 164 Which of the following leads to eosinophilic inflammation in asthmatics ? 158 Dendritic cells are ? Harrison’s 18th Ed. Tumor necrosis factor (TNF-) C. Mast cells eosinophils. T H3 is expressed by epithelial cells of asthmatics. T H1 B. T H4 Macrophages are derived from blood monocytes. 5-lipoxygenase . Specialized basophil-like cells . Three conjugated double bonds A. 2108 A. D. Interleukin 8 165 Differentiation of Th1 cells depends on ? N Engl J Med 2002. Mucus Hypersecretion A. Interleukin 3 The naïve immune system and immune system of asthmatics express TH2 phenotype. 169 Which of the following cytokines may be deficient in asthma ? 175 Which of the following favours Th2 phenotype ? Harrison’s 18th Ed.346:857 D. Interleukin-12 172 Evidence for increased oxidative stress in asthma is provided B. Interferon- D. c-maf C. 9-isoprostane A. Pulmotaxin D. Interleukin 4 eosinophilic inflammation and. N Engl J Med 2001. 174 Level of NO in expired air in asthma is related to ? Harrison’s 18th Ed. C. Interleukin-4 from Th2 cells inhibits Th1 cells D. All of the above Cytokines IL-10 & IL-12 are anti-inflammatory and may be deficient in asthma. Eotaxin A. is attractant to eosinophils via CCR3. Its level NO D. particles that are released can trigger severe asthma exacerbations (thunderstorm asthma). expressed by epithelial cells of asthmatics. are associated with B. . Interferon- exhaled breath condensates ? Harrison’s 18th Ed. All of the above A. are associated with increased IgE formation. Eosinophilic inflammation B. Proinflammatory transcription factors like nuclear factor kB & activator protein 1 (AP-1) are activated in asthmatic airways leading to expression of multiple inflammatory genes. pollen grains are disrupted & CCR4. IL-4 A. whereas proinflammatory cytokines tumor necrosis factor alpha and IL-1beta amplify the inflammatory response. Interleukin-13 Its level in expired air of asthma patients is higher than normal due to eosinophilic inflammation.252 MCQ’s FOR MEDICAL PROFESSIONALS BY PROF.346:857 B. Interleukin-2 C. IL-5 B. Interleukin-5 Nitric oxide (NO) is produced by airway epithelial cells & macrophages in the airway by NO synthases. 2106 Harrison’s 17th Ed. None of the above in expired air in asthmatics is higher than normal and is related to eosinophilic inflammation. Airway Hyperresponsiveness (AHR) C. Cockroaches Chemokine Eotaxin (CCL11). 8-isoprostane N Engl J Med 2002. 1600 A. C. Bronchotaxin C. in asthmatics ? Harrison’s 18th Ed. Pollen grains D. Interleukin 5 D. Fibroblasts A. TH2 cytokines IL-4. A. All of the above A. B. Transcription factor GATA-3 B. whereas CCL17 (TARC) and CCL22 (MDC) from epithelial cells attract TH2 cells via Pollens cause allergic rhinitis rather than asthma. 2106. 2106 176 Thunderstorm asthma is due to ? Harrison’s 18th Ed. Specific transcription 170 Which of the following is selectively attractant to eosinophils factor GATA-3 regulates expression of TH2 cytokines in T cells. TH2 cells. 2106 167 Which of the following is not produced by Th2 cells ? A. Chemotaxin B. Interleukin-4 by increased concentrations of which of the following in C. 2106 168 Which of the following in bronchial asthma is false ? A. AJAY MATHUR Respiratory C. IL-13 171 Stimulating factor for eosinophils is ? Harrison’s 16th Ed. 6-isoprostane IL-10 & IL-12 are anti-inflammatory and may be deficient in asthma. Bronchoconstriction N Engl J Med 2002. 7-isoprostane 166 Differentiation of Th2 cells depends on ? C. Smooth muscle B.346:857 Interleukin 5 stimulates the release of eosinophils into the circulation & extends their survival. through the release of IL-5. through the release of IL-4 and IL-13. Interferon- from Th1 cells inhibits Th2 cells C.344:30 B. 1509 D. Interleukin-12 Evidence for increased oxidative stress in asthma is provided by increased concentrations of 8- B. IL-13 D. Interleukin-4 D. Domestic pets C. 2106 D. All of the above 173 In airways. Epithelial cells Harrison’s 18th Ed. Bronchial lymphocytes in asthma lack T-bet transcription factor Nitric oxide (NO) is produced particularly by airway epithelial cells and macrophages. IL-5 & IL-13 mediate allergic inflammation. Prostaglandin E2 D. In thunderstorms. whereas in normal airways TH1 cells predominate. Fungal spores B. All of the above C. Interleukin-4 isoprostane (product of oxidized arachidonic acid) in exhaled breath condensates and increased ethane (product of lipid peroxidation) in the expired air of asthmatic patients. IL-12 C. Nitric Oxide (NO) is produced mainly by ? Harrison’s 18th Ed. Adrenergic reflex (EIA) ? C. tuberculosis and produce the characteristic cough. humid conditions. None of the above 187 The characteristic symptom of asthma is ? Harrison’s 18th Ed. and sulfur dioxide (via a cholinergic reflex). All of the above Prodromal symptoms that may precede an asthma attack are itching under the chin. 183 Patients with aspirin-induced asthma may benefit from ? 189 Which of the following symptoms is considered ‘sine qua non’ Harrison’s 18th Ed. B. None of the above 188 Prodromal symptom that may precede an asthma attack is ? 182 Which of the following may also be a trigger of asthma ? Harrison’s 18th Ed. dyspnea & coughing. 2108 D. 2109 181 Which of the following is false about exercise induced asthma (EIA) ? A. 2109 Harrison’s 18th Ed. Wheezing Harrison’s 18th Ed. Itching under the axilla A. B. AJAY MATHUR Cardiology 253 177 Which of the following upper respiratory tract virus is the most C. Does not increase airway reactivity Most of the triggers for asthma symptoms act indirectly. Itching behind the ears D. All of the above Harrison’s 18th Ed. Type I interferons develops within 1 hour of drug administration. Rhinovirus B. Cysteinyl leukotrienes 185 Which of the following is supposedly protective for the occurrence of childhood-onset asthma ? D. hyperventilation. Pepper free diet A. Best prevented by inhaled glucocorticoids The characteristic symptoms of asthma are wheezing. Itching under the chin C. D. Kinins C. pylori with childhood-onset asthma. and atopic disorders. aspirin-induced asthma develops C. H. Respiratory syncytial virus 184 After what length of time. Sulphar free diet Harrison’s 18th Ed. Mast cell activation 180 Which of the following is false about exercise induced asthma B. including aspirin. respiratory syncytial virus. B. 2108 A. Salt free diet Harrison’s 16th Ed. All of the above B. 2108 for asthma ? A. Begins after exercise has ended C. Within 12 hours 178 Cough associated with angiotensin-converting enzyme inhibitors best relates to ? D. Urticaria/angioedema may be delayed up to 24 hours. C. 1511 B. Dyspnea A. Chlamydia trachomatis 179 Severity of exercise induced asthma depends on ? C. Cholinergic reflex Harrison’s 18th Ed. cause immediate. discomfort Laughter may also be a trigger of asthma. B. pylori Harrison’s 18th Ed. Coronavirus after drug administration ? Harrison’s 18th Ed. 2108 Most NSAIDs. Salicylate free diet common trigger of acute severe asthma exacerbation ? D. Coughing B. Allergens. Humidity of inspired air D. Whether H. allergy-like symptoms in susceptible individuals. exercise. 436 D. dry climates than in hot. No long-term sequelae D. Attacks follow exertion bronchoconstrictors are histamine & methacholine. Within 6 hours C. 2108 Patients with aspirin-induced asthma may benefit from a salicylate-free diet. Within 24 hour Harrison’s 18th Ed. between scapulae or inexplicable fear (impending doom). which are bronchoconstrictors A. All of the above C. 2108 B. Anger B. All of the above 186 Sulfur dioxide triggers asthma symptoms by ? Harrison’s 18th Ed. Level of ventilation achieved Recent studies have shown an inverse association of cagA+ H. Direct C. Orgasm C. and coronavirus are the most common triggers of acute severe asthma exacerbations. A. fog (via mast cell activation). Recovers spontaneously within ~30 minutes D. Temperature of environment hay fever. Within 1 hour Rhinovirus. pylori status is merely a marker or is causally associated with protection against these diseases remains to be determined. Dyspnea . 1263 Angiotensin-converting enzyme inhibitors inhibit breakdown of kinins. Laughter D. All of the above A. 2108 A. irritant dusts. 2109 EIA is worse in cold. M. Itching over epigastrium B. 253 Respiratory MCQ’s FOR MCQ’s FOR MEDICAL MEDICAL PROFESSIONALS PROFESSIONALS BY PROF. The rhinosinusitis-asthma syndrome generally A. All of the above A. A. D. side effects of chlorpropamide. Zollinger-Ellison syndrome In acutely ill asthmatic patients. 1509 191 Bronchial reactivity reaches its peak around ? A. 2793 Most conspicuous feature of lungs of asthma patients at necropsy is their gross overdistention. reactions to alcohol or glutamate. 100 % of normal history of asthma. Tubercular cavity B. characterized by a family A. All of the above A. All of the above D. 3062 D. Airway smooth muscle B. Gross overdistention of lungs Churg-Strauss syndrome (allergic angiitis & granulomatosis). All of the above Cough due to asthma in the absence of wheezing. residual volume is about ? D. Cogan’s Syndrome Airflow limitation is confirmed by reduced FEV 1. All of the above B. in a case of acute asthma. feature not found is ? C. chronic myeloid leukemia with increased histamine release. C. Nocturnal awakening with dyspnea and/or wheeze channel blockers. A. Atopic dermatitis (AD) failure to collapse when pleural cavities are opened and numerous gelatinous plugs of exudate in most of the bronchial branches down to the terminal bronchioles. B. Hypoxia . All of the above asthma ? Harrison’s 16th Ed. 300 % of normal Harrison’s 18th Ed. Wheezing A. and PEF. wheezing or asthma-like symptoms (8 - 18%) and pellagra-like skin lesions (2 .25%) are also seen. 6 AM C. 12 Noon D.” Children particularly may present with a predominant 199 Airflow limitation is represented by ? nonproductive cough or cough-variant asthma that ties the onset of cough to typical triggers for Harrison’s 18th Ed. and chest tightness is referred to as “cough-variant asthma. Cough-variant asthma typically responds well to inhaled glucocorticoids and intermittent use of inhaled beta-agonist A. All of the above Harrison’s 16th Ed. calcium B. 283 Harrison’s 16th Ed. B. A. 192 Cough-variant asthma is seen in ? 198 Curschman’s spirals in sputum is seen in ? Harrison’s 18th Ed. Elderly D. Reduced FEV1 / FVC ratio 193 Asthma is a feature of which of the following ? C. All of the above A. Polyarteritis Nodosa 200 At necropsy. Constriction feeling in chest 197 Creola bodies are made up of ? D. FEV1/FVC ratio. B. Seborrheic Dermatitis 201 In acutely ill asthmatic patient. diarrhea & cardiac manifestations. and nicotinic acid. and vasculitis of multiple organ systems. Churg-Strauss syndrome Harrison’s 16th Ed. Perspiration Harrison’s 16th Ed. 3064 C. Menopause of asthma ? Harrison’s 16th Ed. Adults C. 1511 Atopic dermatitis (AD) is the cutaneous expression of the atopic state. Verner-Morrison syndrome C. Cough Flushing occurs in systemic mastocytosis. allergic rhinitis. PEF Harrison’s 18th Ed. peripheral B.254 MCQ’s FOR MEDICAL PROFESSIONALS BY PROF. 1511 A. 2109 asthma and resolution of cough upon withdrawal from exposure to them. AJAY MATHUR Respiratory B. menopause. 400 % of normal A. Adolescents B. 1511 D. 12 Midnight Airway epithelium which is sloughed into bronchial lumen appears in the form of Creola bodies. B. B. extravascular granuloma formation. Bronchiectasis D. or eczema. 1511 D. Cough 196 Flushing is a feature of which of the following ? Harrison’s 18th Ed. Thick and stringy mucus C. 2793 D. while functional residual capacity doubles. Reduced FEV1 bronchodilators. Children A. Chronic myeloid leukemia 190 Which of the following is most common & characteristic feature C. Henoch-Schönlein Purpura A. C. None of these conditions cause increased urinary 5-HIAA. Allergic Contact Dermatitis (ACD) C. is characterized by asthma. Bronchial Asthma C. residual volume approaches 400% of normal. shortness of breath. Carcinoid syndrome 202 Which of the following is found during acute exacerbation of D. 2109. Systemic mastocytosis D. Destructive emphysema Harrison’s 18th Ed. Gelatinous plugs of exudate in terminal bronchioles 194 Asthma is related to which of the following ? D. 3 AM B. Failure of lungs to collapse when pleural cavities are opened & tissue eosinophilia. Airway epithelium A. 250 % of normal 195 Asthma is a feature of which of the following ? C. 1511 Apart from flushing. Adenylyl cyclase minutes after inhaled short-acting beta2-agonist of ? C. 2110 Reversibility of asthma means a >12% or 200 mL increase in FEV1 15 minutes after inhaled short- acting beta2-agonist or. < 15 % D. B. Respiratory rate 212 Which of the following is not a bronchodilator drug ? B. All of the above Harrison’s 18th Ed. B. C. SABAs are useful in preventing EIA calculation of provocative concentration that reduces FEV1 by 20% (PC20). 203 Ominous finding during acute exacerbation of asthma is ? Harrison’s 16th Ed. > 12 % or 200 mL 214 Actions of Beta2-agonists include all except ? Harrison’s 18th Ed. High pitched wheezing A. Paradoxical pulse C. > 12 % or 50 mL Beta 2-agonists activate beta 2-adrenergic receptors which are coupled through a stimulatory G B. which relaxes smooth muscle cells. 1511 210 Which of the following is used in the prevention of EIA ? A.to 4-week trial of oral glucocorticoids (prednisolone 30-40 mg daily). beta 2-agonist have no effects on inflammatory cells in airways and B. Visibly active accessory muscles of respiration B. including Churg-Strauss syndrome and polyarteritis Harrison’s 16th Ed. Inhibition of sensory nerve activation A. Cyanosis B. 2110 C. > 20% increase in FEV1 after 4 puffs of  agonists A. Polyarteritis nodosa C. Respiratory alkalosis C. All of the above 205 All of the following have relation with severity of asthma except ? Eosinophilic pneumonias and systemic vasculitis. 2109 D. 2110 D. A. All of the above C. LABAs should not be given without ICSs 209 Aim of asthma therapy is to reduce peak expiratory flow (PEF) C. Beta2-adrenergic agonists D. All of the above obstruction in asthma ? 211 Which of the following may be associated with wheezing ? Harrison’s 16th Ed. Nitric Oxide (NO) there is no reduction in AHR. Methacholine 215 Which of the following about beta2-agonists is false ? D. <5% . AJAY MATHUR Cardiology 255 B. Inhibition of mast cell mediator release by ? C. G protein 207 Reversibility of asthma is defined as an increase in FEV1 15 B. Hypocapnia B. 255 Respiratory MCQ’s FOR MCQ’s FOR MEDICAL MEDICAL PROFESSIONALS PROFESSIONALS BY PROF. All of the above D. Inhaled Corticosteroids (ICSs) D. Cyclic adenosine monophosphate (AMP) Harrison’s 18th Ed. 2109 A. < 20 % Aim of asthma therapy is to reduce PEF circadian variation to < 20 %. Eosinophilic pneumonias B. A. may be associated with wheezing. > 15% increase in FEV1 after 4 puffs of  agonists 213 Which of the following is related to beta2-agonists ? C. Churg-Strauss syndrome D. None of the above Harrison’s 18th Ed. Pulsus paradoxus Harrison’s 18th Ed. All of the above D. increased airway responsiveness (AHR) is measured B. < 10 % C. resulting in increased intracellular cyclic adenosine monophosphate (AMP). anticholinergics & theophylline. Short-acting Beta2-agonists (SABAs) C. Normal arterial carbon dioxide tension Harrison’s 17th Ed. Carbon dioxide In contrast to corticosteroids. Mast cell tolerance prevented by concomitant ICSs circadian variation to ? D. by a 2. Table 254-2 A. Reduction in airway hyperresponsiveness (AHR) 208 In asthma. 1603 B. >= 20% increase in FEV1 after 2 puffs of  agonists Harrison’s 18th Ed. Anticholinergics 206 Reversibility of asthma is defined as ? C. 1512 D. Metabolic acidosis A. 1511 Harrison’s 18th Ed. Use of accessory muscles of respiration B. Inhaled Corticosteroids (ICSs) Harrison’s 16th Ed. > 12 % or 100 mL protein to adenylyl cyclase. >= 15% increase in FEV1 after 2 puffs of  agonists Three classes of bronchodilators are Beta 2-adrenergic agonists. 2110. 2110 Increased airway responsiveness (AHR) is measured by methacholine or histamine challenge with A. Theophylline A. 1515 nodosa. C. All of the above A. > 12 % or 150 mL D. Reduction in plasma exudation Harrison’s 18th Ed. Cromolyn sodium 204 Which of the following indicate severe and prolonged bronchial D. 2109 D. Inability to speak A. cimetidine. 1513 ICSs are the first-line therapy for persistent asthma. D. Proximal myopathy A. Mast cells like erythromycin. 20 mg/L Harrison’s 18th Ed. which is a critical mechanism for switching off activated inflammatory genes. All of the above A. Muscarinic receptor antagonis asthma ? Harrison’s 18th Ed. 20 . allopurinol. Cromolyn sodium Nausea. Ciprofloxacin Proximal myopathy is a major problem with intramuscular triamcinolone acetonide therapy. Inhaled Corticosteroids (ICSs) D. AJAY MATHUR Respiratory 216 Theophylline activates which of the following ? B. Zafirlukast 228 In asthma. 1513 D. All-trans-retinoic acid (ATRA) A. If symptoms are not controled. 2111 D. 2111 of symptoms through relaxation of airway smooth muscle. 2112 Theophylline is metabolized by CYP450 in liver. Inhibition of transcription factor activator protein (AP)-1 C. 2111 C. Macrophages Harrison’s 16th Ed. Neutrophils 222 All of the following drugs increase theophylline level except ? C. All of the above Harrison’s 18th Ed. Long-acting beta 2-agonists (LABAs) D. 2112 D. Erythromycin B.40 µg/mL intramuscular triamcinolone acetonide ? C.15 µg/mL 227 Which of the following is a major problem in therapy with B. 20 . Basophils A. Recruitment of histone deacetylase-2 (HDAC2) to 219 Theophylline side effects are rare at plasma concentrations inflammatory gene complex below ? Harrison’s 18th Ed.15 µg/mL Harrison’s 18th Ed. Erythromycin Harrison’s 18th Ed. 2111 C. Inhibition of transcription factor NF-B Theophylline side effects like cardiac arrhythmias & epileptic seizures occur due to adenosine A1 receptor antagonism. vomiting & headaches produced by theophylline are D. Allopurinol Theophylline activates key nuclear enzyme histone deacetylase-2. Rifampin Cysteinyl-leukotrienes are produced predominantly by mast cells in asthma.40 µg/mL A. 30 mg/L A. Cataracts Harrison’s 18th Ed. C. 2111. B. 5 . Its plasma concentrations may be elevated by drugs A. Theophylline 220 Therapeutic plasma concentration of theophylline is ? Harrison’s 16th Ed. 10 mg/L 226 First-line therapy for patients with persistent asthma is ? B. 2111 D. vomiting & headaches produced by theophylline are due to phosphodiesterase inhibition. ciprofloxacin. Cimetidine B. C. 2111 C.related receptor tyrosine kinase D. Osteoporosis B. All of the above A. Muscarinic receptor antagonis A. Growth factor . Histone deacetylase-2 C. Phosphodiesterase inhibition 225 Which of the following is an action of corticosteroids in C. B. zafirlukast. Bronchodilators provide rapid relief Harrison’s 18th Ed. 1513 D. Antileukotrienes 218 Cardiac arrhythmias produced by theophylline is due to ? Drugs for asthma are divided into bronchodilators & controllers. 2111 224 Which of the following does not belong to the controller therapy A. 5 . DNA methyltransferase 223 All of the following drugs increase theophylline level except ? Harrison’s 16th Ed. Beta2-adrenergic agonists D.256 MCQ’s FOR MEDICAL PROFESSIONALS BY PROF. B. Anticholinergics Theophylline side effects are rare at plasma concentrations < 10 mg/L. LABA is added. Inhaled corticosteroids (ICSs) C. Adenosine receptor antagonism of asthma ? B. Depression 221 Plasma concentrations of theophylline may be elevated by ? C. Propranolol C. Phenytoin due to ? Harrison’s 18th Ed. cysteinyl-leukotrienes are produced mainly by ? D. Phosphodiesterase inhibition Harrison’s 18th Ed. Cephalosporins 217 Nausea. Ciprofloxacin A. All of the above B. A. Adenosine A1 receptor antagonism B. 40 mg/L B. while controllers inhibit inflammatory process. A. Table 254-4 D. . zileuton. 257 Respiratory MCQ’s FOR MCQ’s FOR MEDICAL MEDICAL PROFESSIONALS PROFESSIONALS BY PROF. AJAY MATHUR Cardiology 257 229 Antileukotrienes (montelukast & zafirlukast) block which of D. > 700 IU/L the following ? Omalizumab cannot be given if the total IgE is > 700 IU/L, which effectively excludes highly atopic Harrison’s 18th Ed. 2112 patients. Recommended dose is 0.016 mg /per kg body weight per international unit of IgE every A. cys-LT 1-receptors four weeks, administered subcutaneously at either two-week or four-week intervals. Dose is based on estimated amount of drug that is required to reduce circulating free IgE levels to < 10 IU/mL. B. cys-LT 2-receptors C. cys-LT 3-receptors 235 Speleotherapy is best related to ? D. All of the above A. Mountain B. Lagoon Cysteinyl-leukotrienes are potent bronchoconstrictors, cause microvascular leakage, and increase eosinophilic inflammation through activation of cys-LT 1-receptors. Antileukotrienes (montelukast & C. Cave zafirlukast) block cys-LT1-receptors. D. Iceberg 230 Which of the following medicine is used as steroid-sparing Speleotherapy comes from a Greek word meaning cave. Speleotherapy was first formulated by therapy ? Polish physician F. Bochkowsky who noticed that salt miners had a very low incident of respiratory Harrison’s 18th Ed. 2112 diseases. A. Methotrexate 236 How frequent use of a reliever medication indicates the need B. Cyclosporin A for regular controller therapy ? C. IV gamma globulin Harrison’s 18th Ed. 2113 D. All of the above A. > once a week B. > three times a week Immunomodulators have been used to reduce requirement for OCS in patients with severe asthma. Methotrexate, cyclosporin A, azathioprine, gold, and IV gamma globulin are used as steroid-sparing C. > five times a week therapies, but none of these treatments has any long-term benefit and each is associated with a relatively high risk of side effects. D. > seven times a week Use of a reliever medication > three times a week indicates need for regular controller therapy. 231 Anti IgE monoclonal antibody used in bronchial asthma is ? N Engl J Med 2006;354:2689-95 237 The mainstay of treatment in acute severe asthma is ? A. Mepolizumab Harrison’s 18th Ed. 2113 B. Omalizumab A. High dose short-acting inhaled beta2-agonists C. Keliximab B. High dose inhaled corticosteroids (ICSs) D. Altrakincept C. Intravenous beta2-agonists Humanized anti-IgE monoclonal antibody omalizumab blocks IgE. Approved for persistent allergic D. Intravenous corticosteroids asthma not controlled by inhaled glucocorticoid therapy. The mainstay of treatment is high doses of short-acting inhaled Beta2-agonists. 232 Omalizumab is given in bronchial asthma by which route ? N Engl J Med 2006;354:2689-95 238 Treatment of choice in severely ill asthma patients with impending respiratory failure is ? A. Subcutaneous Harrison’s 18th Ed. 2113 B. Intravenous A. High dose short-acting inhaled beta2-agonists C. Intramuscular B. High dose inhaled corticosteroids (ICSs) D. Aerosol C. Intravenous beta2-agonists Omalizumab is usually given as a subcutaneous injection every 2 - 4 weeks. D. Intravenous corticosteroids 233 Which of the following about Omalizumab is false ? In severely ill patients with impending respiratory failure, intravenous Beta2-agonists may be given. N Engl J Med 2006;354:2689-95 239 Refractory asthma is defined as difficult to control asthma A. Recombinant humanized IgG1 monoclonal anti-IgE antibody despite ? B. Not anaphylactogenic Harrison’s 18th Ed. 2113 C. Total serum IgE levels will increase during treatment A. Maximal inhaled therapy D. None of the above B. Maximal inhaled + oral therapy Omalizumab is a recombinant humanized IgG1 monoclonal anti-IgE antibody that binds to IgE C. Maximal inhaled + oral + IV therapy molecule at the same epitope on Fc that binds to FcRI. It is not anaphylactogenic since it cannot interact with IgE that is already bound to cell surfaces and cannot induce degranulation of mast cells D. All of the above or basophils. Omalizumab binds to circulating IgE forming small, biologically inert IgE - anti-IgE complexes without activating complement cascade. Approximately 5% of asthmatics are difficult to control despite maximal inhaled therapy. 234 Omalizumab cannot be given if the total IgE is ? 240 The most common reason for poor control of asthma is ? BMJ 2009;338:b494 Harrison’s 18th Ed. 2114 A. > 100 IU/L A. Continued exposure to allergens B. > 300 IU/L B. Use of cyclooxygenase (COX) inhibitors C. > 500 IU/L C. Noncompliance with medication 258 MCQ’s FOR MEDICAL PROFESSIONALS BY PROF. AJAY MATHUR Respiratory D. Gastroesophageal reflux B. Sodium salicylate Most common reason for poor control of asthma is noncompliance with medication (ICS). C. Naproxen D. Propoxyphene 241 Premenstrual worsening of asthma is treated by ? Harrison’s 18th Ed. 2114 Aspirin-sensitive asthma responds to usual therapy with an ICS. A. Corticosteroids 247 Drugs for asthma that are safe and without teratogenic potential B. Progesterone include all except ? C. Oestrogen Harrison’s 18th Ed. 2115 D. Anticholinergics A. Short-acting beta2-agonists B. Long-acting beta2-agonists Severe premenstrual worsening of asthma, unresponsive to corticosteroids requires treatment with progesterone or gonadotropin-releasing factors. C. ICSs D. Theophylline 242 Corticosteroid resistant asthma is defined as failure to respond to what dose & duration of oral prednisolone ? Drugs for asthma that are safe and without teratogenic potential include SABAs, ICSs & theophylline. Harrison’s 18th Ed. 2114 There is less safety information about LABAs, antileukotrienes & anti-IgE. A. 40 mg OD for 2 weeks 248 Early onset bronchial asthma is associated with all except ? B. 60 mg OD for 2 weeks Harrison’s 16th Ed. 1508 C. 80 mg OD for 2 weeks A. Rhinitis D. 100 mg OD for 2 weeks B. Family history of urticaria Corticosteroid resistant asthma is defined as failure to respond to 40 mg OD for 2 weeks of oral C. Personal history of eczema prednisolone. D. Normal levels of IgE in serum 243 Mechanism of corticosteroid resistant asthma may be ? 249 Heightened airway responsiveness in bronchial asthma can Harrison’s 18th Ed. 2114 be demonstrated by the use of ? A. Excess of transcription factor AP-1 Harrison’s 16th Ed. 1512 B. Increase in glucocorticoid receptor (GR)- A. Histamine C. Reduction in histone deacetylase activity B. Methacholine D. All of the above C. Isocapnic hyperventilation of cold air Mechanism of corticosteroid resistant asthma may be excess of transcription factor AP-1, increase D. All of the above in glucocorticoid receptor-, abnormal pattern of histone acetylation in response to corticosteroids, defect in IL-10 production and reduction in histone deacetylase activity. 250 When sleep disruption is a prominent side effect of asthma treatment, which drug is preferred ? 244 Treatment of choice in type 2 brittle asthma is ? Harrison’s 18th Ed. 218 Harrison’s 18th Ed. 2114 A. Theophylline sustained release preparations A. Oral corticosteroids B. Inhaled beclomethasone B. Continuous infusion of beta2-agonists C. Inhaled Adrenergic agonists C. Continuous infusion of corticosteroids D. Oral glucocorticoids D. Subcutaneous epinephrine Prominent daily variation in airway resistance in asthmatics results in marked increases in asthmatic Type I brittle asthma - chaotic variations in lung function despite appropriate therapy. Some show a symptoms at night, especially during sleep. Treatment with theophylline, adrenergic agonists or persistent pattern of variability and may require oral corticosteroids or, at times, continuous infusion glucocorticoids can independently disrupt sleep. When sleep disruption is a side effect of asthma of Beta 2-agonists. Type II brittle asthma - normal or near-normal lung function but precipitous, treatment, inhaled glucocorticoids (beclomethasone) that do not disrupt sleep may be useful. unpredictable falls in lung function that may result in death, do not respond well to corticosteroids and inhaled bronchodilators, most effective therapy is subcutaneous epinephrine. 251 Most successful means of bronchial asthma management is ? Harrison’s 16th Ed. 1512 245 Aspirin sensitive asthma is associated with ? Harrison’s 18th Ed. 2115 A. Glucocorticoids A. Extrinsic asthma B. Glucocorticoids +  2 agonists B. Urticaria C. Desensitization / Immunotherapy C. Nasal polyp D. Elimination of provocative agent D. Obesity 252 Bronchial asthma occuring in women at specific predictable Aspirin-sensitive asthma is a well-defined subtype of late onset asthma that is preceded by time during menstrual cycle is called ? perennial rhinitis & nasal polyps in nonatopic patients. A. Functional asthma 246 In aspirin sensitive asthma, which of the following drugs is B. Periodic asthma not well tolerated ? C. Mittelschmerz asthma Harrison’s 16th Ed. 1510 D. Catamenial asthma A. Acetaminophen 259 Respiratory MCQ’s FOR MCQ’s FOR MEDICAL MEDICAL PROFESSIONALS PROFESSIONALS BY PROF. AJAY MATHUR Cardiology 259 253 All are “quick relief medications” for bronchial asthma C. Fenoterol except ? D. Albuterol Harrison’s 16th Ed. 1512 A.  adrenergic agonists 260 Which of the following is a saligenin ? Harrison’s 16th Ed. 1512 B. Methylxanthines A. Isoetharine C. Anticholinergics B. Terbutaline D. Glucocorticoids C. Fenoterol “Quick relief medications” for bronchial asthma are  -adrenergic agonists, methylxanthines and anticholinergics. D. Albuterol 254 All are “long-term control medications” for bronchial asthma 261 Which of the following catecholamines can be given by routes except ? other than inhalational or parenteral ? Harrison’s 16th Ed. 1512 Harrison’s 16th Ed. 1512 A. Glucocorticoids A. Epinephrine B. All  adrenergic agonists B. Isoproterenol C. Mast cell stabilizing agents C. Isoetharine D. Leukotriene modifiers D. None of the above “Long-term control medications” for bronchial asthma are glucocorticoids, long-acting  2-agonists, 262 Major side effect of 2 adrenergic agonists is ? combined medications, mast cell stabilizing agents, leukotriene modifiers and methylxanthines. Harrison’s 16th Ed. 1512 255 Which of the following is a bronchodilator ? A. Tachycardia Harrison’s 17th Ed. 1602 B. Hyperglycemia A.  2-adrenergic agonists C. Tremors B. Anticholinergics D. Hypertension C. Theophylline 263 In patients of bronchial asthma with coexistent heart disease, D. All of the above which of the following drugs is preferred ? Three classes of bronchodilator are 2-adrenergic agonists, anticholinergics & theophylline. Harrison’s 16th Ed. 1513 A. Anticholinergic drugs 256 Non-bronchodilator effect of beta-2 agonists is ? Harrison’s 17th Ed. 1602 B. Methylxanthines A. Inhibition of mast cell mediator release C.  adrenergic stimulants B. Reduction in plasma exudation D. None of the above C. Inhibition of sensory nerve activation 264 Which of the following is a nonstandard bronchodilator ? D. All of the above Harrison’s 16th Ed. 1514 Non-bronchodilator effects of beta-2 agonists are inhibition of mast cell mediator release, reduction A. Magnesium sulfate in plasma exudation & inhibition of sensory nerve activation. B. Methotrexate 257 Which of the following about asthma treatment is false ? C. Gold salts Harrison’s 17th Ed. 1602 D. Colchicine A. Increased use of SABAs means asthma is not controlled 265 Which of the traditional medicine was used in the treatment of B. LABAs do not control underlying inflammation asthma ? C. LABAs should not be given without ICS therapy Harrison’s 18th Ed. Chapter e2 D. None of the above A. Ginkgo biloba B. Ma huang 258 Adrenergic stimulants include ? Harrison’s 16th Ed. 1512 C. Hypericum perforatum A. Catecholamines D. All of the above B. Resorcinols Ephedra sinica, or ma huang, a product used in traditional Chinese medicine was used for short-term treatment of asthma and bronchial congestion. C. Saligenins D. All of the above 266 Which of the following promotes histamine release from tissue mast cells & may worsen bronchospasm in patients with 259 Which of the following is a resorsinol ? asthma ? Harrison’s 16th Ed. 1512 Harrison’s 18th Ed. 2211 A. Isoetharine A. Ketamine B. Terbutaline B. Morphine 260 MCQ’s FOR MEDICAL PROFESSIONALS BY PROF. AJAY MATHUR Respiratory C. Fentanyl 274 Phosphodiesterase inhibitors used in bronchial asthma are D. Propofol all except ? A. Ariflo Morphine can promote histamine release from tissue mast cells and may worsen bronchospasm in patients with asthma. B. Roflumilast C. Urodilatin 267 The most potent anti-inflammatory agent available for bronchial asthma is ? D. Cilostazol Harrison’s 16th Ed. 1513 275 Monoclonal antibody to IL-5 is ? A. Glucocorticoids A. Mepolizumab B. Nedocromil B. Omalizumab C. Ketotifen C. Keliximab D. Zafirlucast D. Altrakincept 268 Oral and intravenous administration produces the same effect 276 Anti CD 4 monoclonal antibody used in bronchial asthma is ? in case of ? BMJ 2009;338:b494 Harrison’s 16th Ed. 1513 A. Mepolizumab A. Methylprednisolone B. Omalizumab B. Prednisolone C. Gomilumab C. Prednisone D. Altrakincept D. Hydrocortisone Gomilumab is a humanised monoclonal antibody against tumour necrosis factor. 269 The 5-lipoxygenase inhibitor used in asthma is ? Harrison’s 18th Ed. 44 277 Immunomodulators used in bronchial asthma are ? A. Bupropion A. Mycophenolate mofetil B. Zolpidem B. Leflunomide C. Zileuton C. Brequinar sodium D. Zafirlucast D. All of the above 270 “Balance oxygen” used for treatment of severe airway 278 Which oral  agonist is administered once daily in bronchial obstruction is ? asthma ? Harrison’s 16th Ed. 1515 A. Albuterol A. 70 - 80 % Hydrogen B. Bambuterol B. 70 - 80 % Nitrogen C. Formoterol C. 70 - 80 % Oxygen D. Salmeterol D. 70 - 80 % Helium 279 Pneumococcal polysaccharide (PPSV) vaccination is indicated Treatment with 70 to 80% helium (balance oxygen) may be beneficial in severe airway obstruction. for patients with ? Harrison’s 18th Ed. 1034 271 Which is a “Soft steroid” used in bronchial asthma ? A. Asthma A. Budesonide B. Nephrotic syndrome B. Flunisolide C. Cirrhosis C. Ciclesonide D. All of the above D. Dexamethasone Pneumococcal polysaccharide (PPSV) vaccination is indicated for patients with chronic lung disease 272 Potassium channel opener used in bronchial asthma is ? (including asthma), chronic cardiovascular diseases, diabetes mellitus, chronic liver diseases; cirrhosis, chronic alcoholism, sickle cell disease, splenectomy, chronic renal failure, nephrotic A. Levcromakalim syndrome) and cochlear implants and cerebrospinal fluid leaks. B. Nicorandil C. Minoxidil 255 - Hypersensitivity Pneumonitis & D. All of the above Pulmonary Infiltrates with Eosinophilia 273 Atrial Natriuretic Peptide (ANP) analogue used in bronchial asthma is ? 280 Hypersensitivity pneumonitis (HP) is also called ? A. Urodilatin Harrison’s 18th Ed. 2116 B. Nesiritide A. Extrinsic allergic alveolitis C. Ariflo B. Intrinsic allergic alveolitis D. Roflumilast C. Immunologic alveolitis 261 Respiratory MCQ’s FOR MCQ’s FOR MEDICAL MEDICAL PROFESSIONALS PROFESSIONALS BY PROF. AJAY MATHUR Cardiology 261 D. Decaying alveolitis D. Apical sparing Hypersensitivity pneumonitis (HP) is also called extrinsic allergic alveolitis. Eosinophilia is not a CxR abnormalities rarely seen in HP are pleural effusion or thickening and hilar or mediastinal LNpathy. feature of HP. 287 Which of the following can be found in BAL of HP patients ? 281 Hypersensitivity pneumonitis (HP) is also called ? Harrison’s 18th Ed. 2118 Harrison’s 18th Ed. 2116 A. Lymphocytic alveolitis A. Gardener’s lung B. Alveolar neutrophilia B. Mountain lung C. Bronchoalveolar mastocytosis C. Farmer’s lung D. All of the above D. Pollen lung BAL in HP patients shows marked lymphocytic alveolitis, alveolar neutrophilia in acute cases and Term “Farmer’s lung” is used for HP due to inhalation of antigens present in moldy hay, such as bronchoalveolar mastocytosis. thermophilic actinomyces, Micropolyspora faeni, and Aspergillus species. 288 Which of the following is false in lung biopsy of HP patients ? 282 Hot tub lung is best related to ? Harrison’s 18th Ed. 2118 Harrison’s 18th Ed. 2116 A. Mononuclear bronchiolitis A. Molds in air conditioners or humidifiers B. Interstitial infiltrates of lymphocytes & plasma cells B. Isocyanates C. Necrotizing granulomas C. Mycobacterium avium complex D. Thickened interstitium D. Cigarette smoking Granulomas in HP are nonnecrotizing. Hot tub lung refers to a hypersensitivity reaction to Mycobacterium avium complex, which is present in hot tubs or whirlpools and is differentiated from actual infection. 289 The most important tool in diagnosing HP is ? Harrison’s 18th Ed. 2119 283 The clinical presentation of HP can be ? Harrison’s 18th Ed. 2116 A. Pulmonary function tests A. Acute B. Radiographic tests B. Subacute C. BAL C. Chronic D. High index of suspicion D. Any of the above Most important tool in diagnosing HP is high index of suspicion. The clinical presentation of HP can be acute, subacute, or chronic. 290 Which of the following about Organic dust toxic syndrome is false ? 284 In acute HP, symptoms occur within how many hours after Harrison’s 18th Ed. 2119 exposure to antigen ? Harrison’s 18th Ed. 2116 A. More common than HP A. 6 to 8 hours B. Follows heavy exposure to organic dusts B. 12 to 24 hours C. Transient fever & muscle aches C. 24 to 48 hours D. None of the above D. 48 to 72 hours 291 Which of the following about Organic dust toxic syndrome is In acute HP, symptoms (cough, fever, chills, malaise, dyspnea) occur 6 to 8 hours after exposure false ? to antigen and usually clear within a few days if there is no further exposure to antigen. Harrison’s 18th Ed. 2119 A. Serum precipitins are absent 285 Which of the following is false about acute hypersensitivity pneumonitis (HP) ? B. Chest x-ray is usually normal Harrison’s 18th Ed. 2118 C. Self-limited disorder without long-term sequelae A. Neutrophilia D. None of the above B. Eosinophilia Organic dust toxic syndrome (ODTS) is more common than HP. It follows heavy exposure to organic C. Lymphopenia dusts. Transient fever & muscle aches, with or without dyspnea & cough occur. Serum precipitins are absent, CxR is normal. ODTS is a self-limited disorder without significant long-term sequelae. D. Presence of rheumatoid factor 292 Tropical eosinophilia is usually caused by ? Following acute exposure to antigen, neutrophilia & lymphopenia are present. Eosinophilia is not a feature. Raised ESR, C-reactive protein, rheumatoid factor, and serum immunoglobulins also occur. Harrison’s 18th Ed. 2120 A. Wuchereria bancrofti 286 Abnormality rarely seen in chest X-Ray of HP patient is ? B. Ascaris Harrison’s 18th Ed. 2118 C. Ancyclostoma A. Diffuse reticulonodular infiltrate D. Strongyloides stercoralis B. Honeycombing Tropical eosinophilia is caused by filarial infection (Wuchereria bancrofti or W. malayi). It is treated C. Pleural effusion successfully with diethylcarbamazine. Eosinophilic pneumonias also occur with Ascaris, Ancyclostoma sp., Toxocara sp. & Strongyloides stercoralis. 262 MCQ’s FOR MEDICAL PROFESSIONALS BY PROF. AJAY MATHUR Respiratory 293 Drug-induced eosinophilic pneumonias can be caused by ? C. Leukotriene-modifying agents Harrison’s 18th Ed. 2120 D. Anticholinergics A. Nitrofurantoin B. Sulfonamides 300 Most common route of entry of aspergillus spores is ? N Engl J Med 2009;360:1870-84 C. Penicillin A. Inhalation D. All of the above B. Ingestion 294 Drug-induced eosinophilic pneumonias can be caused by ? C. Urinary tract Harrison’s 18th Ed. 2120 D. Skin A. Thiazides Invasive aspergillosis principally involves sinopulmonary tract. Inhalation is the most common route B. Tricyclic antidepressants of entry of aspergillus spores. C. Isoniazid 301 Invasive aspergillosis of lung tissue is found in ? D. All of the above Harrison’s 16th Ed. 1188 Drug-induced eosinophilic pneumonias can be caused by nitrofurantoin, sulfonamides, penicillin, A. Granulocyte count in peripheral blood of <500/uL chlorpropamide, thiazides, tricyclic antidepressants, hydralazine, gold salts, isoniazid, indomethacin. B. Treatment with supraphysiologic doses of glucocorticoids 295 BAL fluid of normal nonsmoker individual, percentage of C. History of treatment with cytotoxic drugs eosinophils is ? D. All of the above Harrison’s 18th Ed. 2120 A. <2% 302 Aspergillus infection in a neutropenic patient is characterized by ? Harrison’s 16th Ed. 1188 B. <4% A. Hyphal invasion of blood vessels C. <6% B. Thrombosis, necrosis D. <8% C. Hemorrhagic infarction BAL fluid of normal nonsmoker individual, percentage of eosinophils is less than 2 %. D. All of the above 296 In diagnosis of hypereosinophilic syndrome, eosinophil count should be over ? 303 Term allergic bronchopulmonary aspergillosis (ABPA) denotes Harrison’s 18th Ed. 2120 the condition of patients with preexisting asthma who have ? Harrison’s 17th Ed. 1610 Table 249-3 A. 1000 / µL of peripheral blood for 3 months or longer A. Eosinophilia B. 1500 / µL of peripheral blood for 3 months or longer B. IgE antibody to Aspergillus C. 1000 / µL of peripheral blood for 6 months or longer C. Fleeting pulmonary infiltrates from bronchial plugging D. 1500 / µL of peripheral blood for 6 months or longer D. All of the above Hypereosinophilic syndrome is characterized by presence of >1500 eosinophils per microliter of peripheral blood for 6 months or longer. Main diagnostic criteria of Allergic Bronchopulmonary Aspergillosis (ABPA) include bronchial asthma, pulmonary infiltrates, peripheral eosinophilia (>1000/µL), immediate wheal-and-flare response to A fumigatus, serum precipitins to A. fumigatus, elevated serum IgE and central bronchiectasis. 297 In hypereosinophilic syndrome, the cardiac abnormality is ? Harrison’s 18th Ed. 2120 304 Endobronchial saprophytic pulmonary aspergillosis presents A. Tricuspid valve abnormalities in a patient with prior ? B. Endomyocardial fibrosis Harrison’s 16th Ed. 1188 C. Restrictive, biventricular cardiomyopathy A. Pulmonary tuberculosis D. All of the above B. Sarcoidosis C. Histoplasmosis 298 Besides lung, which other organ is involved in allergic angiitis D. All of the above and granulomatosis of Churg and Strauss ? Harrison’s 16th Ed. 1520 305 In allergic bronchopulmonary aspergillosis (ABPA), fleeting A. Skin eosinophilic infiltrates involve which lobes of lung ? B. Kidney Harrison’s 17th Ed. 1606 C. Nervous system A. Upper lobes D. All of the above B. Middle lobes C. Lower lobes 299 In asthma, use of which of the following drugs is associated D. Any of the above with Churg-Strauss syndrome ? Harrison’s 16th Ed. 1520 In ABPA, fleeting eosinophilic infiltrates typically involve upper lobes of lungs. A. Beta agonists 306 The earliest CT finding in invasive aspergillosis is ? B. Chromolyns Harrison’s 16th Ed. 1188 A. Pulmonary nodules 263 Respiratory MCQ’s FOR MCQ’s FOR MEDICAL MEDICAL PROFESSIONALS PROFESSIONALS BY PROF. AJAY MATHUR Cardiology 263 B. Pulmonary infiltrate B. Liposomal amphotericin B C. Pleural effusion C. Itraconazole D. Pneumothorax D. All of the above 307 Halo sign & crescent sign are radiological findings in ? 313 Drugs for treatment of aspergillosis include ? N Engl J Med 2009;360:1870-84, Harrison’s 16th Ed. 1189 Harrison’s 16th Ed. 1189 A. Invasive aspergillosis of the lung A. Intravenous voriconazole B. Bronchiectasis B. Intravenous caspofungin C. Bronchogenic carcinoma C. Itraconazole D. Atelectasis D. All of the above Earliest radiologic sign of invasive aspergillosis is a nodule. “Halo sign” is defined as a macronodule surrounded by a perimeter of ground-glass opacity corresponding to alveolar hemorrhage. 314 Which of the following is not effective in treatment of aspergillosis ? Harrison’s 17th Ed. 1243 308 Detection of galactomannan antigen in serum suggests the diagnosis of ? A. Fluconazole Harrison’s 16th Ed. 1189 B. Voriconazole A. Invasive aspergillosis of the lung C. Caspofungin B. Bronchiectasis D. Itraconazole C. Bronchogenic carcinoma Fluconazole is not effective for the treatment of aspergillosis D. Atelectasis Antigen-based diagnosis relies on serum detection of either galactomannan or beta-D-glucan, two constituents of fungal-cell walls. Chapter 256. Occupational and 309 Which of the following can give false positive results for Environmental Lung Disease galactomannan antigen ? N Engl J Med 2009;360:1870-84 315 “Fugitive dust” includes ? A. Concomitant use of piperacillin–tazobactam antibiotic Harrison’s 16th Ed. 1521 B. Concomitant use of beta-lactam antibiotic A. Pollens C. Gluconate containing IV fluids B. Windblown dust D. All of the above C. Dust from mechanical industrial processes False positive results for galactomannan antigen are due to concomitant use of piperacillin– D. All of the above tazobactam, other beta-lactam antibiotics, and gluconate containing IV fluids. 316 Particles below what size can be carried to lower airways ? 310 Drug of choice as primary therapy for invasive aspergillosis Harrison’s 18th Ed. 2122 is ? A. 10 µm N Engl J Med 2009;360:1870-84 B. 7.5 µm A. Posaconazole C. 5 µm B. Itraconazole D. 2.5 µm C. Voriconazole Respirable particles that can deposit & be carried to lower airways, is of the size <2.5 µm (fine-mode D. Amphotericin B fraction). Drug of choice as primary therapy for invasive aspergillosis is Voriconazole. 6 mg/kg IV BD for 2 doses, then 4 mg/kg BD as initial therapy. Oral therapy for adults - 200 mg BD or 4 mg/kg BD. 317 Coarse-mode fraction contains particles of what size ? Harrison’s 18th Ed. 2122 311 Which of the following is an “Echinocandin” ? A. ~ 2.5 - 10 µm N Engl J Med 2009;360:1870-84 B. ~ 10 - 19.9 µm A. Caspofungin C. ~ 20 - 20.9 µm B. Micafungin D. ~ 30 - 30.9 µm C. Anidulafungin Particles of silica, aluminum and iron are of the size of ~2.5 - 10 µm and represent coarse-mode D. All of the above fraction. They mostly deposit in tracheobronchial tree. Fungal beta-glucans are ubiquitous cell-wall constituents of fungi and trigger inflammatory responses in macrophages through their exposure on surface of germinating aspergillus conidia. Dectin-1 and 318 Asbestos includes ? TLR2 recognize distinct beta-glucan motifs and stimulate proinflammatory cytokine production. Harrison’s 18th Ed. 2122 Echinocandins are antifungal agents acting on the cell wall by inhibiting beta-glucan synthesis. A. Chrysolite 312 Drugs for treatment of aspergillosis include ? B. Amosite Harrison’s 16th Ed. 1189 C. Crocidolite A. IV amphotericin B 264 MCQ’s FOR MEDICAL PROFESSIONALS BY PROF. AJAY MATHUR Respiratory D. All of the above D. Seronegative Rheumatoid arthritis + bronchial asthma Asbestos is a generic term for mineral silicates like chrysolite, amosite, anthophyllite & crocidolite. Caplan’s syndrome occurs in patients with silicosis includes seropositive rheumatoid arthritis with characteristic pneumoconiotic nodules leading to progressive massive fibrosis (PMF). 319 Major health effect from exposure to asbestos is ? Harrison’s 18th Ed. 2123 325 Histopathological lesion of which of the following environment- associated disease is similar to sarcoidosis ? A. Pleural fibrosis Harrison’s 18th Ed. 2125 B. Diffuse interstitial pulmonary fibrosis A. Asbestosis C. Cancers of respiratory tract, pleura & peritoneum B. Berylliosis D. All of the above C. Coal worker’s pneumoconiosis (CWP) Major health effects from asbestos exposure are pleural & diffuse interstitial pulmonary fibrosis & D. Silicosis cancers of respiratory tract, pleura and rarely peritoneum. Beryllium is commonly associated with a chronic granulomatous inflammatory disease that is 320 The clinical & pathological features of acute silicosis are similar to sarcoidosis. Hilar adenopathy is less common though. similar to those of ? Harrison’s 18th Ed. 2124 326 Which of the following confers increased risk of chronic beryllium disease (CBD) ? A. ABPA Harrison’s 18th Ed. 2125 B. Asthma A. Glu69 polymorphism of HLA-DP chain C. Acute pneumonitis B. Glu79 polymorphism of HLA-DP chain D. Pulmonary alveolar proteinosis C. Glu89 polymorphism of HLA-DP chain Clinical & pathological features of acute silicosis are similar to those of pulmonary alveolar proteinosis. D. Glu99 polymorphism of HLA-DP chain 321 “Crazy paving” HRCT pattern is characteristic of ? Glu69 polymorphism of the HLA-DP chain confers increased risk of CBD. Harrison’s 18th Ed. 2124 Figure 256-2 327 “Monday chest tightness” is seen in ? A. Asbestosis Harrison’s 18th Ed. 2126 B. Berylliosis A. Bagassosis C. Silicosis B. Byssinosis D. Coal worker’s pneumoconiosis (CWP) C. Berylliosis Silicosis chest radiograph shows profuse miliary infiltration or consolidation and “crazy paving” is a D. Asbestosis characteristic HRCT pattern (diffuse ground-glass densities with thickened intralobular & interlobular septa, producing polygonal shapes). Cotton dust exposure (Byssinosis) is characterized clinically by chest tightness toward the end of first day of workweek (“Monday chest tightness”). 322 “Eggshell” pattern of hilar node calcification is seen in ? Harrison’s 18th Ed. 2124 328 “Monday chest tightness” responds best to ? A. Asbestosis Harrison’s 17th Ed. 1616 B. Berylliosis A. Bronchodilators C. Silicosis B. Antihistamines D. Coal worker’s pneumoconiosis (CWP) C. Oxygen therapy D. All of the above Calcification of hilar nodes produces a characteristic “eggshell” pattern in simple silicosis. 329 Thermophilic actinomycetes can cause all except ? 323 Potential clinical complication of silicosis is ? Harrison’s 18th Ed. 2126 Harrison’s 18th Ed. 2125 A. Bagassosis A. Mycobacterium tuberculosis infection B. Farmer’s lung B. Rheumatoid arthritis C. Mushroom worker’s lung C. Scleroderma D. Tobacco worker’s lung D. All of the above Silica is cytotoxic to alveolar macrophages. Lung infections (Mycobacterium tuberculosis, atypical 330 Which of the following is not a feature of acute farmer’s mycobacteria & fungi) are therefore common. Autoimmune connective tissue disorders like rheumatoid lung ? arthritis or scleroderma also occur. Harrison’s 18th Ed. 2126 324 Caplan’s syndrome is ? A. Onset 4 - 8 h after exposure Harrison’s 18th Ed. 2125 B. Fever with chills A. Seronegative Rheumatoid arthritis + Neutropenia C. Cough and dyspnea B. Seropositive Rheumatoid arthritis + splenomegaly D. Wheezing C. Seropositive Rheumatoid arthritis + progressive massive Acute farmer’s lung presents 4 - 8 hours after exposure with fever, chills, malaise, cough & dyspnea fibrosis (PMF) without wheezing. 265 Respiratory MCQ’s FOR MCQ’s FOR MEDICAL MEDICAL PROFESSIONALS PROFESSIONALS BY PROF. AJAY MATHUR Cardiology 265 331 Which of the following agent has been implicated in “Tight- B. Hydrogen sulphide building syndrome” ? C. Ozone Harrison’s 18th Ed. 2128 D. Ammonia A. Dust mites B. Perfumes C. Latex particles Chapter 257. Pneumonia D. None of the above Nonspecific responses occur in “tight-building syndrome” and no particular agent has been implicated 339 Pneumonia is an infection of ? in its causation. Harrison’s 18th Ed. 2130 A. Pulmonary parenchyma 332 Which of the following is false about “nuisance dusts” ? Harrison’s 16th Ed. 1524 B. Terminal bronchiole A. Do not affect architecture of terminal bronchioles/acini C. Respiratory bronchiole B. Clinical effects are reversible D. All of the above C. Pulmonary function tests are usually normal Pneumonia is an infection of the pulmonary parenchyma. D. None of the above 340 HCAP stands for ? 333 Siderosis is due to ? Harrison’s 18th Ed. 2130 Harrison’s 16th Ed. 1524 A. Hospital-community acquired pneumonia A. Iron & iron oxides B. Hospital-community associated pneumonia B. Tin oxide C. Health care - associated pneumonia C. Barium sulfate D. Health care - acquired pneumonia D. Antimony salts Pneumonia is categorized as either community-acquired pneumonia (CAP) or health care - associated pneumonia (HCAP). HCAP includes hospital-acquired pneumonia (HAP) & ventilator-associated 334 Baritosis is due to ? pneumonia (VAP). Harrison’s 16th Ed. 1524 A. Iron & iron oxides 341 Which of the following is the most common route for bacterial pneumonia ? B. Tin oxide Harrison’s 18th Ed. 2130 C. Barium sulfate A. Aerosolization D. Antimony salts B. Contiguous extension 335 Stannosis is due to ? C. Microaspiration of oropharyngeal secretions Harrison’s 16th Ed. 1524 D. Hematogenous spread A. Iron & iron oxides Most common mechanism by which microorganisms gain access to lower respiratory tract is by B. Tin oxide aspiration from the oropharynx. C. Barium sulfate 342 Which of the following is instrumental in clearing and killing D. Antimony salts pathogens at the the alveolar level ? Harrison’s 18th Ed. 2130 336 Hypereosinophilic syndrome is characterised by TEC ? A. Resident alveolar macrophages Harrison’s 16th Ed. 1520 B. Surfactant protein A A. > 250 / µL C. Surfactant protein D B. > 500 / µL D. All of the above C. > 1000 / µL Resident alveolar macrophages are assisted by local proteins (surfactant proteins A and D) in D. > 1500 / µL clearing and killing pathogens at the the alveolar level. 337 For diagnosis of hypereosinophilic syndrome, the duration of 343 Which of the following triggers the clinical syndrome of eosinophilia must be ? pneumonia ? Harrison’s 16th Ed. 1520 Harrison’s 18th Ed. 2130 A. > 2 weeks A. Host inflammatory response B. > 2 months B. Proliferation of microorganisms C. > 4 months C. Alveolar capillary leak D. > 6 months D. All of the above 338 All can cause chronic bronchitis except ? Alveolar macrophages initiate the inflammatory response. The host inflammatory response, rather Harrison’s 18th Ed. 2127 Table 256-2 than the proliferation of microorganisms, triggers the clinical syndrome of pneumonia. A. Phosgene 266 MCQ’s FOR MEDICAL PROFESSIONALS BY PROF. AJAY MATHUR Respiratory 344 Release of which of the following results in fever ? Due to microaspiration mechanism, bronchopneumonia pattern is most common in nosocomial Harrison’s 18th Ed. 2130 pneumonias. Lobar pattern is more common in bacterial CAP. A. Interleukin (IL) 1 350 Which of the following is the most common cause of CAP in B. Interleukin (IL) 8 OPD, Non-ICU and ICU ? C. Granulocyte colony-stimulating factor Harrison’s 18th Ed. 2131 Table 257-2 D. All of the above A. Streptococcus pneumoniae Release of inflammatory mediators interleukin (IL) 1 & tumor necrosis factor (TNF) results in fever. B. Haemophilus influenzae Chemokines IL-8 & granulocyte colony-stimulating factor stimulate release of neutrophils & their attraction to lung. C. Mycoplasma pneumoniae D. H. influenzae 345 Macrophage is the dominant cell type in alveolar space in which Streptococcus pneumoniae is most common of CAP in OPD, Non-ICU and ICU. of the following pathological phase of pneumonia ? Harrison’s 18th Ed. 2131 351 Which of the following complicates influenza infection ? A. Edema Harrison’s 18th Ed. 2131 B. Red hepatization A. Streptococcus pneumoniae C. Gray hepatization B. Haemophilus influenzae D. Resolution C. Staphylococcus aureus In the resolution phase, macrophage is the dominant cell type in alveolar space. D. H. influenzae S. aureus pneumonia is well known to complicate influenza infection. 346 Neutrophil is the dominant cell type in alveolar space in which of the following pathological phase of pneumonia ? 352 Which of the following is not an independent risk factor for Harrison’s 18th Ed. 2131 community-acquired pneumonia ? A. Edema Harrison’s 18th Ed. 2132 B. Red hepatization A. Alcoholism C. Gray hepatization B. Asthma D. Resolution C. Immunosuppression In the third phase i.e. gray hepatization, neutrophil is the predominant cell, fibrin deposition is D. Age > 60 years abundant, and bacteria have disappeared. Risk factors for CAP include alcoholism, asthma, immunosuppression, institutionalization, and an age of >=70 years. 347 Successful containment of infection & improvement in gas exchange occurs in which of the following pathological phase 353 Which of the following is the strongest independent predictor of pneumonia ? of invasive pneumococcal disease in immunocompetent young Harrison’s 18th Ed. 2131 adults ? A. Edema Harrison’s 17th Ed. 1621 Harrison’s 16th Ed. 1530 B. Red hepatization A. Alcoholism C. Gray hepatization B. Cigarette smoking D. Resolution C. Chronic obstructive pulmonary disease Phase of gray hepatization corresponds with successful containment of the infection and improvement D. Male gender in gas exchange. Risk factors for pneumococcal pneumonia include dementia, seizure disorders, heart failure, cerebrovascular disease, alcoholism, tobacco smoking, chronic obstructive pulmonary disease, 348 Four pathological phases of pneumonia are best described and HIV infection. Cigarette smoking is the strongest independent predictor of invasive pneumococcal for ? disease among immunocompetent young adults. Harrison’s 18th Ed. 2131 A. Pneumococcal pneumonia 354 A recent hotel stay or ship cruise predisposes a person to pneumonia due to ? B. Viral pneumonia Harrison’s 18th Ed. 2132 C. Pneumocystis pneumonia A. Legionella D. All of the above B. Enterobacteriaceae Four pathological phases of pneumonia are described best for lobar pneumococcal pneumonia and C. CA-MRSA may not apply to pneumonias of all etiologies, especially viral or Pneumocystis pneumonia. D. P. aeruginosa 349 Bronchopneumonia pattern is most common in ? Risk factors for Legionella infection include diabetes, hematologic malignancy, cancer, renal disease, Harrison’s 18th Ed. 2131 HIV, smoking, male, and a recent hotel stay or ship cruise. A. Bacterial CAP 355 Pneumatoceles on chest radiography suggest infection with ? B. Nosocomial pneumonia Harrison’s 18th Ed. 2132 C. Viral pneumonia A. Enterobacteriaceae D. Pneumocystis pneumonia B. P. aeruginosa 267 Respiratory MCQ’s FOR MCQ’s FOR MEDICAL MEDICAL PROFESSIONALS PROFESSIONALS BY PROF. AJAY MATHUR Cardiology 267 C. M. tuberculosis 362 Number of variables included in the CURB-65 criteria is ? Harrison’s 18th Ed. 2133 D. S. aureus A. 3 In CAP, pneumatoceles suggest infection with S. aureus. B. 4 356 To be adequate for culture, sputum sample should have ? C. 5 Harrison’s 18th Ed. 2133 D. 6 A. >25 PMN & <10 squamous cells per low power field CURB-65 criteria include five variables - confusion (C), urea >7 mmol/L (U), respiratory rate 30/ B. >25 PMN and <20 squamous cells per low power field minute (R), blood pressure, systolic 90 mmHg or diastolic 60 mmHg (B) and age 65 years (65). C. >50 PMN and <10 squamous cells per low power field 363 Patients should be admitted to the hospital when the CURB-65 D. >50 PMN and <20 squamous cells per low power field score is more than ? To be adequate for culture, a sputum sample must have >25 neutrophils (PMN) and <10 squamous Harrison’s 18th Ed. 2133 epithelial cells per low-power field. A. 0 357 Urinary antigen detection by ELISA is done for which of the B. 1 following lung infections ? C. 2 Harrison’s 18th Ed. 2133 D. 3 A. Legionella pneumophila serogroup 1 With a CURB-65 score of 2, 30-day mortality rate is 9.2% & patients should be admitted in hospital. B. C. burnetii C. Histoplasma capsulatum 364 Methicillin resistance in S. aureus is determined by ? Harrison’s 18th Ed. 2134 D. Neisseria meningitidis A. ermB gene 358 Urinary antigen detection by ELISA is done for which of the B. mecA gene following lung infections ? Harrison’s 18th Ed. 2133 C. gyrA gene A. S. pneumoniae D. parC gene B. Mycobacterium tuberculosis Methicillin resistance in S. aureus is determined by the mecA gene, which encodes for resistance to all  -lactam drugs. C. Histoplasma capsulatum D. Neisseria meningitidis 365 CA-MRSA has which type of SCCmec element ? Harrison’s 18th Ed. 2134 Pneumococcal & Legionella pneumophila serogroup 1 antigens can be detected in urine. A. Type I 359 In pneumonia, following etiologic agents can be diagnosed B. Type II serologically except ? C. Type III Harrison’s 16th Ed. 1533 D. Type IV A. Mycoplasma pneumoniae Out of the five staphylococcal chromosomal cassette mec (SCCmec) types described, typical B. Chlamydia pneumoniae hospital-acquired strains have type II or III, whereas CA-MRSA has a type IV SCCmec element. C. Chlamydia psittaci D. Histoplasma capsulatum 366 Which of the following is a superantigen in CA-MRSA strains ? Harrison’s 18th Ed. 2134 360 In pneumonia, following etiologic agents can be diagnosed A. Enterotoxins B serologically except ? B. Enterotoxins C Harrison’s 16th Ed. 1533 C. Panton-Valentine leukocidin A. Legionella spp. D. All of the above B. Coxiella burnetii CA-MRSA strains also carry genes for superantigens, such as enterotoxins B and C and Panton- C. Adenovirus Valentine leukocidin that can create cytolytic pores in polymorphonuclear neutrophils, monocytes, D. Coccidioides immitis and macrophages. 361 Number of variables included in the Pneumonia Severity Index 367 Which of the following bacteria is typically resistant to (PSI) is ? cephalosporins ? Harrison’s 18th Ed. 2133 Harrison’s 18th Ed. 2134 A. 8 A. Escherichia coli B. 15 B. Enterobacter spp. C. 20 C. CA-MRSA D. 26 D. All of the above To determine the PSI, points are given for 20 variables. Enterobacter spp. are typically resistant to cephalosporins. Drugs of choice for these are usually fluoroquinolones or carbapenems. 268 MCQ’s FOR MEDICAL PROFESSIONALS BY PROF. AJAY MATHUR Respiratory 368 If CA-MRSA pneumonia infection is suspected, which of the A significant pleural effusion in pneumonia should be tapped if fluid has a pH of <7, glucose <2.2 following should be added to initial empirical regimen ? mmol/L and LDH >1000 U/L or if bacteria are seen or cultured. Harrison’s 18th Ed. 2135 A. Tobramycin 374 Lung abscess may occur in association with ? Harrison’s 18th Ed. 2136 B. Meropenem A. Aspiration C. Linezolid B. CA-MRSA infection D. Piperacillin/tazobactam C. P. aeruginosa infection If CA-MRSA infection is suspected, either linezolid or vancomycin should be added to the initial empirical regimen. D. All of the above Lung abscess may occur in association with aspiration or with infection caused by a single CAP 369 In empirical treatment of community-acquired pneumonia, pathogen, such CA-MRSA, P. aeruginosa, or S. pneumoniae. which of the following has significantly higher mortality rate ? Harrison’s 16th Ed. 1534 375 In treated pneumonia, a follow-up radiograph is done after how A. Macrolide + second-generation cephalosporin many weeks ? Harrison’s 18th Ed. 2136 B. Second- or third-generation cephalosporin alone A. One week C. Fluoroquinolone B. 3 weeks D. Aminoglycoside + any other antibiotic C. 4 - 6 weeks Combination of an aminoglycoside & any other antibiotic has a significantly higher mortality rate. D. 8 - 12 weeks 370 Therapy of longer duration is recommended for all of the following Chest radiographic abnormalities are slowest to resolve & may require 4 - 12 weeks to clear. A pathogens causing community-acquired pneumonia except ? follow-up radiograph can be done ~4 - 6 weeks later. Harrison’s 16th Ed. 1535 Table 239-9 376 Clinical Pulmonary Infection Score (CPIS) was developed for A. Legionella spp. the diagnosis of ? B. P. aeruginosa Harrison’s 18th Ed. 2139 C. S. pneumoniae A. CAP D. Enterobacteriaceae B. HAP Pneumonia due to Legionella spp., P. aeruginosa, or Enterobacteriaceae usually requires therapy of C. HCAP longer duration (often up to 21 days). D. VAP 371 Which of the following is intrinsically resistant to many of the Clinical Pulmonary Infection Score (CPIS) was developed for the diagnosis of VAP. empirical antibiotic regimens ? Harrison’s 18th Ed. 2139 377 Which of the following is intrinsically resistant to many of the A. Acinetobacter empirical antibiotic regimens ? Harrison’s 18th Ed. 2139 B. Stenotrophomonas maltophilia A. Acinetobacter spp. C. Burkholderia cepacia B. Stenotrophomonas maltophilia D. All of the above C. Burkholderia cepacia Acinetobacter, Stenotrophomonas maltophilia, and Burkholderia cepacia are intrinsically resistant to many of the empirical antibiotic regimens D. All of the above Acinetobacter spp., Stenotrophomonas maltophilia and Burkholderia cepacia are intrinsically resistant 372 Drotrecogin alfa (activated) should be considered for CAP to many of the empirical antibiotic regimens employed. patients if ? Harrison’s 18th Ed. 2136 378 The most sensitive component of the CPIS is improvement in ? A. Persistent septic shock Harrison’s 18th Ed. 2140 B. APACHE II scores of >=25 A. Oxygenation C. Infection by S. pneumoniae B. Body temperature D. All of the above C. Leucocytosis D. Tracheal aspirate culture Immunomodulatory therapy with drotrecogin alfa (activated) should be considered for CAP patients with persistent septic shock & APACHE II scores of >=25, particularly if infection is by S. pneumoniae. The most sensitive component of the CPIS is improvement in oxygenation. 373 In community-acquired pneumonia, complicating pleural 379 In VAP, clinical improvement, if it occurs, is usually evident effusion should be drained for all the following reasons except ? within how many hours of initiation of antimicrobial treatment ? Harrison’s 18th Ed. 2136 Harrison’s 18th Ed. 2140 A. If the fluid has a pH of <7 A. 6 - 12 hours B. If the fluid has a glucose level of < 2.2 mmol/L B. 12 - 24 hours C. If the fluid has a LDH content of > 500 units C. 24 - 48 hours D. If the fluid has is positive on Gram’s staining or culture 269 Respiratory MCQ’s FOR MCQ’s FOR MEDICAL MEDICAL PROFESSIONALS PROFESSIONALS BY PROF. AJAY MATHUR Cardiology 269 D. 48 - 72 hours B. Nocardia In VAP, clinical improvement, if it occurs, is usually evident within 48–72 h of the initiation of C. Legionella spp. antimicrobial treatment. D. All of the above 380 Pneumonia caused by which pathogen signifies that patients Mycobacterium, Nocardia & Legionella spp. are resistant to the microbicidal activity of phagocytes. immune system is so compromised that death is almost inevitable ? 386 Virulence factor ‘pneumolysin’ interacts with any cell whose Harrison’s 18th Ed. 2140 membrane contains ? Harrison’s 16th Ed. 1529 A. P. aeruginosa B. S. maltophilia A. Peptides C. Acinetobacter spp. B. Cholesterol D. C. Triglyceride D. Amino acids Pneumonia caused by S. maltophilia is a marker for a patient whose immune system is so compromised that death is almost inevitable. All pneumococci produce pneumolysin that interacts with any cell whose membrane contains cholesterol. Pneumococci also produce neuraminidase, hyaluronidase, and IgA1 protease. 381 Which of the following are active in opsonization of bacteria in lower respiratory tract ? 387 In patients with community-acquired pneumonia, which of the Harrison’s 16th Ed. 1529 following is an independent risk factor for a fatal outcome ? Harrison’s 16th Ed. 1529 A. Surfactant B. IgG A. Lymphotoxin- AA genotype C. Fibronectin B. TNF- 308:LT- +250 GC haplotype D. All of the above C. TNF- 238 GA genotype D. DRB1*1501/DQB1*0602 haplotype Surfactant is bactericidal to certain pathogens & along with IgG & fibronectin can opsonize bacteria. 388 In patients with community-acquired pneumonia, which of the 382 Which of the following statements about alveolar macrophages following is a risk factor for septic shock ? is false ? Harrison’s 16th Ed. 1529 Harrison’s 16th Ed. 1529 A. Lymphotoxin- +250 AA genotype A. Density is one per alveolus B. TNF- 308:LT- +250 GC haplotype B. Play a role in both innate and acquired immunity C. TNF- 238 GA genotype C. Have a short life span D. DRB1*1501/DQB1*0602 haplotype D. Have ability to phagocytose multiple times Alveolar macrophages are present at a density of one per alveolus. They play a role in innate & 389 In patients with community-acquired pneumonia, which of the acquired immunity and have a long life span (20 to 80 days) and can phagocytose multiple times. following is protective against septic shock ? Harrison’s 16th Ed. 1529 383 Which of the following does not cause pneumonia through A. Lymphotoxin- AA genotype aerosolization route ? Harrison’s 16th Ed. 1529 B. TNF- 308:LT-a +250 GC haplotype A. Mycobacterium tuberculosis C. TNF- 238 GA genotype B. Histoplasma capsulatum D. DRB1*1501/DQB1*0602 haplotype C. Escherichia coli In patients with CAP, the TNF- 238 GA genotype is an independent risk factor for a fatal outcome, lymphotoxin- (LT-) +250 AA genotype is a risk factor for septic shock, and the TNF- 308:LT- D. Legionella +250 GC haplotype is protective against septic shock. Aerosolization is the route by which M. tuberculosis, Coccidioides immitis, Blastomyces dermatitidis, Histoplasma capsulatum, Legionella spp., Coxiella burnetii, influenza viruses A and B reach the 390 In the clinical entity designated ALPS, ‘S’ stands for ? lungs. Escherichia coli commonly originates from urinary tract infections. Harrison’s 16th Ed. 1530 A. Smoking 384 Which of the following produces ciliostatic factor ? Harrison’s 16th Ed. 1529 B. Sepsis A. Mycoplasma pneumoniae C. Sedantry B. Chlamydia pneumoniae D. Supine C. Influenza virus Clinical entity designated ALPS (alcoholism, leukopenia, pneumococcal sepsis) is associated with a mortality rate of 80%. D. Neisseria meningitidis Chlamydia pneumoniae produces a ciliostatic factor to counteract host defenses.. 391 ‘PORT risk score’ is used to classify ? Harrison’s 16th Ed. 1532 385 Which of the following are resistant to microbicidal activity of A. Community-acquired pneumonia phagocytes ? B. Bronchiectasis Harrison’s 16th Ed. 1529 C. Interstitial lung disease A. Mycobacterium 270 MCQ’s FOR MEDICAL PROFESSIONALS BY PROF. AJAY MATHUR Respiratory D. Pulmonary thromboembolism Certain microorganisms, if isolated from sputum, should always be considered pathogens. These include M. tuberculosis, Legionella spp., B. dermatitidis, H. capsulatum, and C. immitis. PORT stands for Pneumonia Patient Outcomes Research Team. PORT risk class stratifies mortality rate in Community-Acquired Pneumonia. 398 Aerobic pathogen most often implicated in causation of 392 The single most useful clinical sign of severity of pneumonia aspiration-associated lung abscesses is ? in a person without underlying lung disease is ? Harrison’s 16th Ed. 1537 Harrison’s 16th Ed. 1532 A. Streptococcus milleri A. Respiratory rate >30/minute B. Bacteroides fragilis B. Pulse rate > 100/minute C. Prevotella oralis C. Hypothermia D. Fusobacterium nucleatum D. Postural fall of SBP > 10 mm Hg Most aspiration-associated lung abscesses are due to a combination of aerobic and anaerobic bacteria, with an average of six or seven bacterial species identified in an individual case. The single most useful clinical sign of the severity of pneumonia is a respiratory rate of >30/minute Streptococcus milleri is one of the principal aerobic pathogens. in a person without underlying lung disease. 399 Gastric acid aspiration leading to aspiration pneumonitis 393 Which of the following bacterial pneumonia has the highest requires following characteristics of gastric aspirate ? mortality rate ? Harrison’s 16th Ed. 1538 Harrison’s 16th Ed. 1532 A. pH < 2.5 & gastric aspirate volume of > 0.3 mL/kg A. P. aeruginosa B. pH < 5 & gastric aspirate volume of > 0.3 mL/kg B. Klebsiella spp. C. pH < 2.5 & gastric aspirate volume of > 0.6 mL/kg C. E. coli D. pH < 5 & gastric aspirate volume of > 0.6 mL/kg D. S. aureus Gastric aspirate pH of <2.5 & volume >0.3 mL/kg (20 to 25 mL in adults) is required for the Mortality rate is highest (>50%) for pneumonia due to P. aeruginosa, followed by Klebsiella spp., E. development of aspiration pneumonitis. coli, S. aureus, and Acinetobacter spp. (30 to 35%). 400 Which of the following is a respiratory fluoroquinolone ? 394 Which capsular serotype pneumococcus is associated with a Harrison’s 18th Ed. 2135 Table 257-4 higher mortality rate ? Harrison’s 16th Ed. 1532 A. Moxifloxacin A. 1 B. Gemifloxacin B. 2 C. Levofloxacin C. 3 D. All of the above D. 4 Respiratory fluoroquinolones include moxifloxacin, gemifloxacin and levofloxacin. Capsular serotype 3 pneumococcus is associated with a much higher mortality rate than serotype 1. 401 Hospital-acquired pneumonia (HAP) is defined as pneumonia occurring how many hours after hospital admission ? 395 In a case of pneumonia, pneumatoceles in chest radiograph Harrison’s 16th Ed. 1538 suggests infection with ? Harrison’s 16th Ed. 1532 A. At least 12 hours A. Mycobacterium tuberculosis B. At least 24 hours B. S. aureus C. At least 36 hours C. Escherichia coli D. At least 48 hours D. Legionella HAP is defined as pneumonia occurring at least 48 hours after hospital admission and not incubating at the time of admission. 396 Microorganisms isolated from sputum that should always be considered pathogens include ? 402 Cefoxitin, Cefotetan, and Cefmetazole belong to which Harrison’s 16th Ed. 1533 generation of cephalosporins ? Harrison’s 16th Ed. 1538 A. Mycobacterium tuberculosis A. First B. Legionella spp. B. Second C. Blastomyces dermatitidis C. Third D. All of the above D. None of the above 397 Microorganisms isolated from sputum that should always be Cephamycins (cefoxitin, cefotetan, & cefmetazole) are structurally different from true cephalosporins considered pathogens include ? & display enhanced stability in the presence of ESBLs. Some consider cephamycins to be subset of Harrison’s 16th Ed. 1533 second-generation cephalosporins. A. Blastomyces dermatitidis 403 ESBL-producing isolates should be considered resistant to ? B. Histoplasma capsulatum Harrison’s 17th Ed. 938 C. Coccidioides immitis A. All penicillins D. All of the above B. Cephalosporins 271 Respiratory MCQ’s FOR MCQ’s FOR MEDICAL MEDICAL PROFESSIONALS PROFESSIONALS BY PROF. AJAY MATHUR Cardiology 271 C. Aztreonam A. RTH Laënnec D. All of the above B. Lynne Reid ESBL-producing isolates should be considered resistant to all penicillins, cephalosporins & aztreonam. C. Jean Athanase Sicard D. PJ Cole 404 Which of the following organisms have been designated as “core pathogens” in the American Thoracic Society’s Different patterns of bronchiectasis were described by Reid in 1950. guidelines for treatment of nosocomial pneumonia ? Harrison’s 16th Ed. 1538 410 Which of the following is false about bronchiectasis ? Harrison’s 18th Ed. 2142 A. S. pneumoniae A. Abnormal & permanent dilatation of bronchi B. H. influenzae B. Affects older individuals C. S. aureus C. Affects women more than men D. All of the above D. None of the above 405 According to American Thoracic Society (ATS) guidelines for treatment of nosocomial pneumonia “core pathogens” include 411 Which of the following forms of bronchiectasis is due to an all except ? underlying systemic or infectious disease process ? Harrison’s 16th Ed. 1539 Harrison’s 18th Ed. 2142 A. Staphylococcus aureus A. Cylindrical or tubular bronchiectasis B. Pneumococcus B. Varicose bronchiectasis C. Pseudomonas aeruginosa C. Focal bronchiectasis D. E. coli D. Diffuse “Core pathogens” in American Thoracic Society’s guidelines for the treatment of nosocomial pneumonia Diffuse bronchiectasis is characterized by widespread bronchiectatic changes throughout the lung are S. pneumoniae, H. influenzae, S. aureus, and enteric gram-negative bacilli (E. coli, Klebsiella and often arises from an underlying systemic or infectious disease process. spp., Proteus spp., and Serratia marcescens). 412 Bronchiectasis more pronounced in upper lung fields is most 406 British Thoracic Society (BTS) criteria for severe community common in ? acquired pneumonia includes all parameters except ? Harrison’s 18th Ed. 2142 Harrison’s 16th Ed. 1532 A. Cystic fibrosis (CF) A. Pulse rate B. Scleroderma B. Blood pressure C. Idiopathic pulmonary fibrosis C. Respiratory rate D. Due to Mycobacterium avium-intracellulare complex (MAC) D. Blood urea level More pronounced bronchoectatic involvement of the upper lung fields is most common in cystic British Thoracic Society Rule for definition of severe community-acquired pneumonia includes fibrosis (CF), in postradiation fibrosis, corresponding to the lung region radiated. confusion, urea, respiratory rate and blood pressure. 413 Bronchiectasis more pronounced in lower lung fields is most 407 In a case of pneumonia, urine examination can help in the common in ? etiological diagnosis in ? Harrison’s 18th Ed. 2142 Harrison’s 16th Ed. 1533 A. Cystic fibrosis (CF) A. Mycoplasma B. Scleroderma B. Chlamydia C. Idiopathic pulmonary fibrosis C. Coxiella D. Due to Mycobacterium avium-intracellulare complex (MAC) D. L. pneumophila serogroup 1 414 Bronchiectasis more pronounced in lower lung fields is most 408 In a case of pneumonia, urine examination can help in the common in ? etiological diagnosis in ? Harrison’s 18th Ed. 2142 Harrison’s 16th Ed. 1533 A. Scleroderma A. Mycoplasma B. Idiopathic pulmonary fibrosis B. Chlamydia C. Hypogammaglobulinemia C. Coxiella D. All of the above D. S. pneumoniae Bronchiectasis with predominant involvement of the lower lung fields is due to chronic recurrent aspiration (due to esophageal motility disorders like scleroderma), end-stage fibrotic lung disease (traction bronchiectasis from idiopathic pulmonary fibrosis), or recurrent immunodeficiency-associated Chapter 258. Bronchiectasis and Lung infections (hypogammaglobulinemia). Abscess 415 Bronchiectasis more pronounced in mid lung fields is most common in ? Harrison’s 18th Ed. 2142 409 Patterns of bronchiectasis were described by ? Harrison’s 16th Ed. 1541 A. Cystic fibrosis (CF) 272 MCQ’s FOR MEDICAL PROFESSIONALS BY PROF. AJAY MATHUR Respiratory B. Scleroderma C. Sjogren’s syndrome C. Idiopathic pulmonary fibrosis D. All of the above D. Due to Mycobacterium avium-intracellulare complex (MAC) Bronchiectasis occurs rarely in ulcerative colitis, rheumatoid arthritis & Sjogren’s syndrome. Bronchiectasis due to noninfectious causes may occur in immune-mediated reactions that damage Bronchiectasis due to infection by nontuberculous mycobacteria (NTM), mostly Mycobacterium the bronchial wall (Sjögren’s syndrome & rheumatoid arthritis). avium-intracellulare complex (MAC) preferentially affects midlung fields. Congenital causes of bronchiectasis with predominant midlung field involvement include the dyskinetic/immotile cilia syndrome. 422 Which of the following is false about bronchiectasis ? Harrison’s 17th Ed. 1629 416 Bronchiectasis of relatively proximal airways suggests ? A. Parenchyma supplied by affected airways is abnormal Harrison’s 18th Ed. 2142 B. Vascularity of bronchial wall is increased A. Infection with M. avium complex C. Broncho-pulmonary arterial anastomoses B. ABPA D. None of the above C. Tuberculosis D. Cystic fibrosis 423 In which pattern of bronchiectasis, bronchi end in blind sacs without recognizable distal bronchial structures ? Bronchiectasis accompanying ABPA involves proximal airways & is associated with mucoid impaction. Harrison’s 17th Ed. 1629 An immune-mediated reaction to Aspergillus damages the bronchial wall. A. Cylindrical or tubular 417 Which of the following is a congenital cause of B. Varicose bronchiectasis ? Harrison’s 18th Ed. 2142 C. Saccular A. McCune-Albright syndrome D. All of the above B. Laurence-Moon-Biedl syndrome In saccular (cystic) bronchiectasis, bronchi end in blind sacs without recognizable bronchial structures distal to the sacs. C. Kallmann syndrome D. Mounier-Kuhn syndrome 424 Which of the following is the most severe form of bronchiectasis ? 418 Which of the following is a congenital cause of N Engl J Med 2002;346:1383 bronchiectasis ? A. Cylindrical or tubular Harrison’s 18th Ed. 2142 B. Varicose bronchiectasis A. Pallister-Hall syndrome C. Saccular or cystic B. Prader-Willi syndrome D. None of the above C. Laron syndrome D. Williams-Campbell syndrome 425 Traction bronchiectasis as seen in HRCT is a feature of ? N Engl J Med 2002;346:1383 Congenital causes of central airway predominant bronchiectasis resulting from cartilage deficiency include tracheobronchomegaly (Mounier-Kuhn syndrome) and Williams-Campbell syndrome. A. Asthma B. Chronic bronchitis 419 In bronchiectasis, which of the following components of the wall of airways is destroyed ? C. Pulmonary fibrosis Harrison’s 18th Ed. 2143 D. Allergic bronchopulmonary aspergillosis A. Cartilage 426 Which of the following is the least common cause of B. Muscle bronchiectasis ? C. Elastic tissue Harrison’s 17th Ed. 1629 D. All of the above A. Adenovirus Normal structural components of larger-airway wall - cartilage, smooth muscle & elastic tissue B. Staphylococcus aureus are destroyed and may be replaced by fibrous tissue due to significant small-airway wall C. Tuberculosis inflammation. D. Mycoplasma 420 Which of the following is an action of  1 antitrypsin ? Adenovirus, influenza virus, Staph. aureus, Klebsiella, anaerobes, Bordetella pertussis, M. tuberculosis Harrison’s 18th Ed. 2143 are important causes of bronchiectasis. A. Antiprotease 427 Which of the following is not a feature of Kartagener’s syndrome B. Neutralizes damaging effects of neutrophil elastase ? C. Enhances bacterial killing Harrison’s 17th Ed. 1629 D. All of the above A. Situs inversus B. Colonic diverticula 421 Bronchiectasis may occur with which of the following ? Harrison’s 18th Ed. 2143, Harrison’s 16th Ed. 1542 C. Bronchiectasis A. Ulcerative colitis D. Sinusitis B. Rheumatoid arthritis Kartagener’s syndrome includes situs inversus, bronchiectasis and sinusitis. 273 Respiratory MCQ’s FOR MCQ’s FOR MEDICAL MEDICAL PROFESSIONALS PROFESSIONALS BY PROF. AJAY MATHUR Cardiology 273 428 Which of the following is related to bronchiectasis ? D. Any of the above Harrison’s 18th Ed. 2143 Bronchiectasis with upper lobe involvement may be suggestive of either tuberculosis or ABPA. A. Tram tracks B. Signet-ring sign 434 Multiple small pulmonary abscesses in contiguous areas of C. “Tree-in-bud” pattern lung is called ? Harrison’s 18th Ed. 2144 D. All of the above A. Lung chancre In chest radiographs, presence of “tram tracks” indicate dilated airways and is consistent with bronchiectasis. Chest CT findings include airway dilation that is detected as parallel “tram tracks” or B. Lung carbuncle as “signet-ring sign” i.e. a cross-sectional area of the airway with a diameter at least 1.5 times that C. Lung gangrene of the adjacent vessel, lack of bronchial tapering (including the presence of tubular structures within 1 cm from the pleural surface), bronchial wall thickening in dilated airways, inspissated secretions D. Lung pemphigus (e.g., the “tree-in-bud” pattern), or cysts emanating from the bronchial wall (especially pronounced in cystic bronchiectasis. Lung abscess refers to microbial infection of lung resulting in necrosis of pulmonary parenchyma. Necrotizing pneumonia or lung gangrene refers to multiple small pulmonary abscesses in contiguous areas of the lung, usually resulting from a more virulent infection. 429 “Tram tracks” and “ring shadows” are chest radiological signs seen in ? 435 Most common cause of lung abscess is ? Harrison’s 17th Ed. 1630 Harrison’s 17th Ed. 1631 A. Bronchopneumonia A. Aspiration B. Bronchiectasis B. Periodontal disease C. Bronchogenic carcinoma C. Alcoholism D. Aspergillosis D. Immunoglobulin deficiency Dilated airways with thickened walls due to peribronchial inflammation are often crowded together in parallel. When seen longitudinally, they appear as “tram tracks”, when seen in cross-section, they Aspiration is the most common cause of lung abscess. produce “ring shadows”. 436 Most common causative organisms for lung abscess is ? 430 Nodular bronchiectasis is suggestive of ? Harrison’s 18th Ed. 2145 Harrison’s 16th Ed. 1542 A. Anaerobic bacteria A. Infection with Mycobacterium avium complex B. S. aureus B. ABPA C. Klebsiella pneumoniae C. Cystic fibrosis D. M. tuberculosis D. P. aeruginosa infection Anaerobic bacteria are the most common causative organisms for lung abscess. Presence of multiple small pulmonary nodules (nodular bronchiectasis) suggests infection with M. avium complex. 437 Of the following, which is the most common pulmonary site of lung abscess ? 431 FEV1 declines by how much per year in healthy individuals ? Harrison’s 17th Ed. 1631 Harrison’s 18th Ed. 2144 A. Anterior segments of upper lobes A. 10 - 20 mL B. Posterior segments of lower lobes B. 20 - 30 mL C. Right middle lobe C. 30 - 40 mL D. None of the above D. 40 - 50 mL Dependent areas of lung, particularly upper lobes & posterior segments of lower lobes are the most Forced expiratory volume in 1 second (FEV1) declines by 20 - 30 mL per year in healthy subjects. common pulmonary site of lung abscess. 432 Which of the following is not a feature of ‘Yellow nail syndrome’ 438 Acute lung abscess in a healthy young patient, along with ? influenza, is likely to be due to ? Harrison’s 17th Ed. 1629 Harrison’s 18th Ed. 2145, Table 258-2 A. Lymphedema A. Klebsiella pneumoniae B. Pleural effusion B. Staphylococcus aureus C. Mediastinal lymphadenopathy C. M. tuberculosis D. Yellow discoloration of nails D. All of the above Yellow nail syndrome is due to hypoplastic lymphatics. The triad of lymphedema, pleural effusion Acute lung abscess in a healthy young patient, especially in conjunction with influenza, is likely to and yellow discoloration of nails is accompanied by bronchiectasis in ~40% of patients. be due to Staphylococcus aureus. Other bacteria include S. milleri, K. pneumoniae, Group A Streptococcus, Gemella, Legionella, and Actinomyces spp. Parasites include Entamoeba histolytica, Paragonimus westermani, Strongyloides stercoralis. 433 Which lobe of lung is involved in “Dry” bronchiectasis ? Harrison’s 17th Ed. 1630 439 Which of the following agent is found almost exclusively in A. Upper lobe patients with defective cell-mediated immunity ? B. Middle lobe Harrison’s 18th Ed. 2145, Table 258-2 C. Lower lobe A. Nocardia asteroides 274 MCQ’s FOR MEDICAL PROFESSIONALS BY PROF. AJAY MATHUR Respiratory B. Burkholderia pseudomallei C. Metronidazole C. Paragonimus westermani D. TMP-SMZ D. S. milleri Penicillin “was” the mainstay of empiric antibiotic therapy for lung abscess. Due to frequent  - lactamase producing organisms, clindamycin is now standard therapy. Agents that are found almost exclusively in patients with defective cell-mediated immunity are Nocardia asteroides and Rhodococcus equi. Other agents include M. tuberculosis, Legionella spp., P. aeruginosa, Enterobacteriaceae (Klebsiella pneumoniae), Aspergillus spp., Cryptococcus spp. 446 Which of the following should not be used for pulmonary infections ? 440 Lemierre’s disease best relates to ? Harrison’s 18th Ed. 2146 Harrison’s 18th Ed. 2145, Table 258-2 A. Linezolid A. Strongyloides stercoralis B. Daptomycin B. Blastomyces C. Aerosolized colistin C. Fusobacterium necrophorum D. Aminoglycosides D. Gemella Daptomycin should not be used for pulmonary infections. 441 Which of the following clinical features of nonspecific lung abscess is uncommon ? Harrison’s 18th Ed. 2145 Chapter 259. Cystic Fibrosis A. Fatigue B. Cough 447 The median survival for patients with CF is about ? Harrison’s 18th Ed. 2147 C. Sputum production A. 27 years D. Fever with chills B. 37 years Chills are uncommon. C. 47 years 442 Foul odor of sputum is due to the organisms’ production of ? D. 57 years Harrison’s 18th Ed. 2145 Due to improvements in therapy, the median survival is >37.4 years for patients with CF. A. Short-chain fatty acids B. Medium-chain fatty acids 448 CF is characterized by ? Harrison’s 18th Ed. 2147 C. Long-chain fatty acids A. Bronchiolectasis D. All of the above B. Exocrine pancreatic insufficiency Foul odor of sputum is due to the organisms’ production of short-chain fatty acids, such as butyric or succinic acid. C. Intestinal dysfunction D. All of the above 443 Which of the following is uncommon in lung abscess ? Harrison’s 18th Ed. 2145 CF is characterized by chronic bacterial infection of airways that leads to bronchiectasis and bronchiolectasis, exocrine pancreatic insufficiency and intestinal dysfunction, abnormal sweat A. Fever with chills gland function, and urogenital dysfunction. B. Lymphadenopathy 449 Cystic fibrosis (CF) is characterized by all except ? C. Anaerobic bacteria in expectorated sputum cultures Harrison’s 18th Ed. 2147 D. All of the above A. Polygenic disorder Lymphadenopathy is not associated with bacterial lung abscess and suggests an alternative B. Autosomal recessive diagnosis. Anaerobic bacteria, the most common causes of primary lung abscess, are not detected in expectorated sputum cultures. C. Bronchiolectasis D. Exocrine pancreatic insufficiency 444 Which of the following in lung abscess indicates anaerobic infection ? Cystic fibrosis (CF) is a monogenic autosomal recessive disorder. Harrison’s 18th Ed. 2145 A. Putrid breath 450 CFTR stands for ? Harrison’s 18th Ed. 2147 B. Sputum A. CF transmembrane resistance regulator C. Empyema fluid B. CF transmembrane conductance regulator D. All of the above C. CF transport regulator In lung abscess, putrid breath, sputum, or empyema fluid indicates anaerobic infection. D. CF transcription regulator 445 Which of the following drugs is now standard therapy for lung CFTR stands for CF transmembrane conductance regulator. abscess ? Harrison’s 18th Ed. 2146 451 CFTR gene is located on chromosome ? Harrison’s 18th Ed. 2147 A. Penicillin A. 4 B. Clindamycin B. 7 275 Respiratory MCQ’s FOR MCQ’s FOR MEDICAL MEDICAL PROFESSIONALS PROFESSIONALS BY PROF. AJAY MATHUR Cardiology 275 C. 12 B. Nuclear membrane D. 13 C. Cytoplasm CF is due to mutations in the gene located on the long arm of chromosome 7 that encodes CF D. Rough endoplasmic reticulum transmembrane conductance regulator (CFTR) protein. Fully processed CFTR is found in the plasma membrane in normal epithelia. 452 Which class of CF is least severe ? Harrison’s 18th Ed. 2147 458 Cellular function ascribed to CFTR is ? N Engl J Med 2005;352:1992-2001 A. I A. Conducts chloride across cell membrane B. II B. Down-regulates transepithelial sodium transport C. III C. Regulates Ca++-activated chloride & K+ channels D. IV D. All of the above Mutations in CFTR gene fall into five major classes. Classes I - III CFTR gene mutations are “severe” with pancreatic insufficiency and high sweat NaCl values. Class IV mutations can be “mild” CFTR conducts chloride across the cell membrane & is regulated by protein kinase A (PKA) in a associated with pancreatic sufficiency & intermediate/normal sweat NaCl values. cAMP-dependent fashion. Other cellular functions of CFTR are down-regulation of transepithelial sodium transport, regulation of calcium-activated chloride & potassium channels. 453 Which of the following is abnormal in Class IV CFTR gene mutation ? 459 Diagnostic biophysical hallmark of CF airway epithelia is ? Harrison’s 18th Ed. 2148 Harrison’s 18th Ed. 2147, Figure 259-1 A. Defective protein synthesis A. Lowered transepithelial electric potential difference B. Defective processing B. Raised transepithelial electric potential difference C. Defective regulation C. Plasma membrane fenestration D. Defective conduction D. Plasma membrane rigidity Diagnostic biophysical hallmark of CF airway epithelia is raised transepithelial electric potential Class I - Defective protein synthesis, Class II - Defective processing, Class III - Defective difference which reflects both the rate of active ion transport and epithelial resistance to ion flow. regulation, Class IV - Defective conduction, Class V - Reduced functioning CFTR protein. 454 The most common mutation in CF is termed as ? 460 Cystic fibrosis (CF) is characterized by all except ? Harrison’s 18th Ed. 2148 Harrison’s 17th Ed. 1632, N Engl J Med 2005;352:1992-2001 A.  F508 A. Increased airway Na+ absorption B. N1303K B. Increased airway Cl- secretion C. G85E C. Reduced salt & water content of mucus D. G91R D. Decreased volume of periciliary liquid Capacity to initiate cyclic AMP mediated Cl– secretion is diminished in CF airway epithelia due to Most common mutation in CF is termed F508. CFTR with F508 mutation lacks a phenylalanine absence/dysfunction of CFTR Cl– channel. Accelerated Na + absorption in CF reflects absence of (F) residue at position 508. Other clinically important mutations - N1303K, G85E, and G91R lead to CFTR inhibitory effects on Na+ channels. misfolded CFTR protein that is prematurely degraded. 455 CF patients homozygous for  F508 have normal sweat 461 In a normal cell, CFTR is synthesized in ? Harrison’s 16th Ed. 1544 Figure 241-1 electrolytes in the presence of which second mutation ? N Engl J Med 1997;336:487 A. Golgi apparatus A. R553Q B. Rough endoplasmic reticulum B. R554Q C. Mitochondria C. R555Q D. Nucleus D. R556Q CFTR is synthesized in rough endoplasmic reticulum (RER),glycosylated in Golgi apparatus & functions as a Cl- channel & regulator of other ion channels when located in plasma membrane. Patients homozygous for  F508 mutation have normal sweat electrolyte concentrations if a second mutation - R553Q is also present. 462 Which of the following is best related to airway surfaces in CF ? Harrison’s 18th Ed. 2148 456 The CFTR protein contains how many amino acids ? Harrison’s 18th Ed. 2147 A. Edematous A. 1280 B. Atrophic B. 1380 C. Hypertrophic C. 1480 D. Dehydrated D. 1580 Due to faulty regulation of Na+ absorption and inability to secrete Cl– via CFTR reduce the volume of liquid on airway surfaces and appear “dehydrated”. CFTR protein is a single polypeptide chain with 1480 amino acids. It functions as a cyclic AMP - regulated Cl– channel and as a regulator of other ion channels. 463 Which of the following is mostly infected in CF ? Harrison’s 18th Ed. 2148 457 Fully processed form of CFTR is found in ? Harrison’s 18th Ed. 2147 A. Mucus layer A. Plasma membrane B. Epithelia 276 MCQ’s FOR MEDICAL PROFESSIONALS BY PROF. AJAY MATHUR Respiratory C. Airway wall D. Any of the above D. All of the above In CF, sweat emerging on the skin surface contains a high level of salt. Infection in CF airways involves mucus layer rather than epithelial or airway wall. 470 Elevated electrolyte levels in sweat can be due to ? N Engl J Med 1997;336:487 464 Which of the following is false about mucus layer in CF airways ? A. Cystic fibrosis Harrison’s 18th Ed. 2148 B. Fucosidosis A. Mucus stasis C. Glycogen storage disease type 1 B. Mucus hypoxemia D. All of the above C. Mucus dehydration 471 Elevated electrolyte levels in sweat can be due to ? D. All of the above N Engl J Med 1997;336:487 O 2 tension is very low in CF mucus (mucus hypoxemia). Mucus stasis is due to dehydration of A. Mucopolysaccharidosis mucus and periciliary liquid layers. It produces adhesion of mucus to airway surface causing failure to clear mucus from airways by ciliary and cough-dependent mechanisms. B. Hypothyroidism C. Vasopressin-resistant diabetes insipidus 465 CF airways are predisposed to chronic infection by ? Harrison’s 18th Ed. 2148 D. All of the above A. Staphylococcus aureus 472 Elevated electrolyte levels in sweat can be due to ? B. Pseudomonas aeruginosa N Engl J Med 1997;336:487 C. Strict anaerobes A. Adrenal insufficiency D. All of the above B. Familial cholestasis C. Familial hypoparathyroidism 466 CF patients fail to secrete which of the following in pancreatic duct ? D. All of the above Harrison’s 18th Ed. 2148 473 Recovery of which bacteria in bronchoalveolar lavage fluid A. Na + supports the diagnosis of CF ? B. HCO 3– N Engl J Med 1997;336:487 C. Water A. Klebsiella pneumoniae D. All of the above B. Proteus mirabilis In CF, failure to secrete Na , HCO 3 and water leads to retention of enzymes in pancreas & + – C. Escherichia coli destruction of virtually all pancreatic tissue. D. Pseudomonas aeruginosa 467 Cholelithiasis occurs with increased frequency in ? Recovery of Pseudomonas aeruginosa in BAL fluid supports the diagnosis of CF. Harrison’s 17th Ed. 1633 A. Cystic fibrosis (CF) 474 In lower respiratory tract, the first symptom of CF is ? Harrison’s 18th Ed. 2149 B. Crohn’s disease (CD) A. Cough C. Wilson Disease B. Dyspnoea D. All of the above C. Pain chest In addition to the above three illnesses, cholelithiasis occurs with increased frequency in polyglandular autoimmune syndrome Type I. D. Hemoptysis In the lower respiratory tract, the first symptom of CF is cough. 468 Which of the following can occur in CF ? Harrison’s 18th Ed. 2149 475 Which of the following is often the first organism recovered A. Thickened biliary secretions from lung secretions in newly diagnosed CF patients ? Harrison’s 18th Ed. 2149 B. Focal biliary cirrhosis A. P. aeruginosa C. Bile-duct proliferation B. Haemophilus influenzae D. All of the above C. Aspergillus fumigatus In CF, defective hepatic ductal salt (Cl–) & water secretion causes thickened biliary secretions, focal biliary cirrhosis, and bile-duct proliferation in ~ 25 - 30% of patients. Chronic cholecystitis and D. Klebsiella cholelithiasis also occur. Haemophilus influenzae and S. aureus are often the first organisms recovered from lung secretions in newly diagnosed CF patients. 469 In CF, sweat emerging on the skin surface contains ? N Engl J Med 2005;352:1992-2001 476 Infection with which of the following is rare in CF ? A. High level of salt Harrison’s 18th Ed. 2149 B. Low level of salt A. Haemophilus influenzae C. Normal level of salt B. Mycobacterium tuberculosis 277 Respiratory MCQ’s FOR MCQ’s FOR MEDICAL MEDICAL PROFESSIONALS PROFESSIONALS BY PROF. AJAY MATHUR Cardiology 277 C. P. aeruginosa D. Malabsorption D. Aspergillus fumigatus DIOS can be confused with appendicitis, whose frequency is not increased in CF. Gastrointestinal malignancy is increased in incidence in CF. CF patients have an increased incidence of Mycobacterium tuberculosis is rare in patients with CF. osteoarthropathy, renal stones, and osteoporosis, particularly following transplant. 477 First lung-function abnormalities seen in CF children is ? 483 Which of the following can also cause pulmonary disease and Harrison’s 18th Ed. 2149 azoospermia ? A. Decreased forced vital capacity (FVC) N Engl J Med 1997;336:487 B. Decreased FEV1 A. Chédiak-Higashi syndrome C. Decreased FEV25-50 B. Young’s syndrome D. Increased ratios of RV / TLC C. Felty’s syndrome First lung-function abnormality seen in CF children is increased ratios of residual volume to total D. Job’s Syndrome lung capacity reflecting small-airways disease. Young’s syndrome can cause pulmonary disease and azoospermia. > 95% of male patients with CF are azoospermic, reflecting obliteration of the vas deferens due to defective liquid secretion. 478 Earliest chest x-ray change in CF lungs is that of ? Harrison’s 18th Ed. 2149 484 All of the following are useful tests for CF except ? A. Hyperinflation Harrison’s 18th Ed. 2149 B. Bronchial cuffing A. Sweat chloride concentration C. Bronchiectasis B. Semen Analysis D. Pleural effusion C. Serum lipids Earliest chest x-ray change in CF lungs is hyperinflation, reflecting small-airways obstruction. D. Nasal Potential-Difference Measurements A combination of clinical criteria & abnormal CFTR function ( sweat tests, nasal PD measurements, 479 Which part of the lung displays earliest & most severe changes and CFTR mutation analysis) are required for diagnosis of CF. in CF ? Harrison’s 18th Ed. 2149 485 What value of sweat Cl– concentration is typical in CF adults ? Harrison’s 18th Ed. 2149 A. Right upper lobe A. > 40 meq/L B. Left upper lobe B. > 50 meq/L C. Right lower lobe C. > 60 meq/L D. Left lower lobe D. > 70 meq/L Right upper lobe displays the earliest and most severe changes in CxR in CF. Typically in adults a sweat Cl– concentration of >70 meq/L is pathognomonic for CF. 480 Which of the following can occur in CF ? Harrison’s 18th Ed. 2149 486 Normal mean nasal potential-difference is ? N Engl J Med 1997;336:487 A. Pneumothorax A. – 4.7 mV B. Cor pulmonale B. – 24.7 mV C. Clubbing of digits C. – 44.7 mV D. All of the above D. – 64.7 mV Pneumothorax occurs in >10% of CF patients. Hemoptysis is common. Clubbing of digits, respiratory failure and cor pulmonale point to advanced CF. Normal mean nasal potential-difference is – 24.7 mV. In CF, nasal transepithelial PD is raised 481 Distal intestinal obstruction syndrome (DIOS) consists of all 487 In CF, information of nasal potential-difference measurements except ? can be augmented by the use of ? Harrison’s 18th Ed. 2149 N Engl J Med 1997;336:487 A. Left lower quadrant pain A. Amiloride B. Loss of appetite B. Chloride-free solution C. Emesis C. Isoproterenol D. Palpable mass D. All of the above In children & young adults, meconium ileus equivalent or distal intestinal obstruction syndrome (DIOS) Nasal potential-difference measurements (including responses to amiloride, chloride-free solution, occurs consisting of right lower quadrant pain, loss of appetite, emesis & often a palpable mass. and isoproterenol) may demonstrate abnormal CFTR function more reliably than the sweat test. 482 Which of the following does not occur with increased frequency 488 What is the strength of hypertonic saline that is recommended in CF ? for inhalation in CF ? Harrison’s 18th Ed. 2149 Harrison’s 18th Ed. 2150 A. Diabetes mellitus A. 2% B. Appendicitis B. 3% C. Cholelithiasis C. 5% 278 MCQ’s FOR MEDICAL PROFESSIONALS BY PROF. AJAY MATHUR Respiratory D. 7% 494 Which allele is associated with markedly reduced  1 antitrypsin levels ? Inhaled hypertonic saline (7%) has demonstrated efficacy in CF patients by restoring mucus Harrison’s 18th Ed. 2152 clearance and pulmonary function. A. S 489 In CF patients who have undergone lung transplantation, death B. M principally results from ? Harrison’s 18th Ed. 2150 C. Z A. Massive hemoptysis D. Null B. Secondary infection Z allele is associated with markedly reduced 1AT levels. Null alleles lead to absence of any 1AT. Individuals with two Z alleles or one Z and one null allele are referred to as PiZ, which is the most C. Obliterative bronchiolitis common form of severe 1AT deficiency. D. HIV complications   1 antitrypsin is mainly produced in ? Only effective treatment for respiratory failure in CF is lung transplantation. 2-year survival for lung Lancet 2005;365:2225-36 transplantation exceeds 60%. Transplant-patient deaths result principally from obliterative bronchiolitis. A. Lung B. Liver Chapter 260. Chronic Obstructive C. Kidney Pulmonary Disease D. Spleen 1-antitrypsin deficiency is a genetic disorder, clinically characterised by liver disease & early-onset emphysema. 1 AT is mainly produced in liver. Its main function is to protect lung against proteolytic 490 COPD includes ? damage from neutrophil elastase. Harrison’s 18th Ed. 2151 A. Emphysema   1 antitrypsin is the prototypic member of which superfamily B. Chronic bronchitis of proteins ? Lancet 2005;365:2225-36 C. Small airways disease A. Glutamine protease inhibitor D. All of the above B. Methionine protease inhibitor COPD includes emphysema, chronic bronchitis and small airways disease all characterized by airflow limitation that is not fully reversible. C. Threonine protease inhibitor D. Serine protease inhibitor 491 Asthma, chronic bronchitis and emphysema are variations of the same basic disease is hypothesised by ? 497 Mutation that causes severe  1-antitrypsin deficiency arises Harrison’s 18th Ed. 2152 in which gene ? Harrison’s 18th Ed. 2152, Lancet 2005;365:2225-36 A. American hypothesis B. British hypothesis A. SERPINA 1 B. SERPINA 2 C. Canadian hypothesis C. SERPINA 3 D. Dutch hypothesis D. SERPINA 4 Dutch hypothesis suggests that asthma, chronic bronchitis & emphysema are variations of the same basic disease which is modulated by environmental & genetic factors to produce these Most frequent mutation that causes severe 1-AT deficiency arises in SERPINA1 gene & gives rise pathologically distinct entities. British hypothesis states that asthma & COPD are fundamentally to Z allele. This mutation reduces concentrations in serum of  1 AT by retaining polymerised different diseases. molecules within hepatocytes. 492 Obstructive lung disease is diagnosed if ? 498 Risk for emphysema increases when level of serum 1-AT is ? Harrison’s 16th Ed. 1551 Lancet 2005;365:2225-36 A. FEV1 / FVC < 0.7 A. < 11 µmol/L B. FEV1 / FVC > 0·7 B. < 21 µmol/L C. FVC / FEV 1 < 0·7 C. < 31 µmol/L D. FVC / FEV 1 > 0·7 D. < 41 µmol/L COPD is characterized by an FEV1 value that is <80 % of the predicted normal value and an FEV1/ An amount of 1-AT below the serum protective threshold of 11 µmol/L increases risk for emphysema. FVC ratio of <0.70. 499 SERPINA1 gene was formerly known as ? 493 Which allele is associated with normal 1 antitrypsin levels ? Harrison’s 18th Ed. 2152, Lancet 2005;365:2225-36 Harrison’s 18th Ed. 2152 A. PR A. S B. PI B. M C. PX C. Z D. PZ D. Null SERPINA1 gene was formerly known as protease inhibitor (PI). Protease inhibitor (PI or SERPINA1) locus encodes 1AT. M allele is associated with normal 1AT levels. S allele is associated with slightly reduced 1AT levels. 279 Respiratory MCQ’s FOR MCQ’s FOR MEDICAL MEDICAL PROFESSIONALS PROFESSIONALS BY PROF. AJAY MATHUR Cardiology 279   1 antitrypsin deficiency is inherited as ? B. Rocket immunoelectrophoresis Lancet 2005;365:2225-36 C. Radial immunodiffusion A. Autosomal co-dominant D. All of the above B. Autosomal recessive Testing for 1-AT deficiency in serum is done by nephelometry, rocket immunoelectrophoresis or C. X linked radial immunodiffusion. Clinical laboratory test used most frequently to screen for 1AT deficiency is measurement of the immunologic level of 1AT in serum D. None of the above 1-antitrypsin deficiency is inherited as an autosomal co-dominant disorder, for which more than   1 antitrypsin plays a major role in inactivating ? 100 alleles have been identified. Lancet 2005;365:2225-36 A. Macrophage elastase 501 SERPINA1 gene is located on ? Lancet 2005;365:2225-36 B. Neutrophil elastase A. Long arm of chromosome 12 C. Lymphocyte elastase B. Long arm of chromosome 13 D. Eosinophil elastase C. Long arm of chromosome 14 1 AT is the prototypic member of serine protease inhibitor (serpin) superfamily of proteins, which have a major role in inactivating neutrophil elastase to maintain protease-antiprotease balance. D. Long arm of chromosome 15 SERPINA1 gene is located on long arm of chromosome 14. It spans 12·2 kb & is organised into four   1 antitrypsin deficiency, conformational instability of coding (2, 3, 4, 5) & three non-coding (1a, 1b, 1c) exons. which structure of serpins leads to mutations and polymerisation ? 502 Normal  1-antitrypsin concentrations in serum is ? Lancet 2005;365:2225-36 Lancet 2005;365:2225-36 A. -sheet A. 5 - 13 mol/L B. -sheet B. 13 - 20 mol/L C. -sheet C. 20 - 53 mol/L D. -sheet D. 65 - 90 mol/L Conformational instability of the  -sheet structure of the serpins underlies their susceptibility to 503 Normal  1-antitrypsin concentrations in serum is ? mutations and polymerisation. Lancet 2005;365:2225-36 Table 2   1 antitrypsin deficiency can predispose to which of the A. 15 - 50 mg/dL following liver disorders ? B. 50 - 150 mg/dL Lancet 2005;365:2225-36 C. 150 - 350 mg/dL A. Chronic hepatitis D. 350 - 550 mg/dL B. Cirrhosis Normal alleles are typified by 1-antitrypsin concentrations in serum of 20 - 53 µmol/L. Typically, C. Hepatoma deficient alleles lead to 1 AT levels <20 µmol/L. D. All of the above 504 Dysfunctional variants leading to abnormal function of  1   1 antitrypsin deficiency can predispose to which of the antitrypsin can cause ? following skin disorders ? Lancet 2005;365:2225-36 Lancet 2005;365:2225-36 A. Emphysema A. Pemphigus B. Cirrhosis liver B. Panniculitis C. Bleeding diathesis C. Psoriasis D. All of the above D. All of the above 505 Dysfunctional variants leading to abnormal function of  1  1-AT deficiency can predispose to lung disease (emphysema, bronchiectasis), liver disorders antitrypsin can cause ? (chronic hepatitis, cirrhosis, hepatoma), skin disease (panniculitis), and vasculitis (anticytoplasmic antibody-positive vasculitis - Wegener’s granulomatosis). Lancet 2005;365:2225-36 A. c-ANCA-positive vasculitis 511 Emphysema associated with  1 antitrypsin deficiency, B. Cirrhosis liver distinctive features include ? Lancet 2005;365:2225-36 C. Bleeding diathesis A. Early onset (fourth and fifth decade) D. All of the above B. Panacinar pathology Dysfunctional variants lead to abnormal function of 1 antitrypsin with reduced binding to neutrophil elastase (as in the F variant) or, as with Pittsburgh, structural abnormality that causes the protein C. Disproportionate involvement of lung bases to serve as a thrombin inhibitor rather than as an antielastolytic protein, causing a bleeding diathesis. D. All of the above 506 Testing for  1 antitrypsin concentration in serum is done by ? Emphysema associated with 1-AT deficiency is early onset (IV & V decades), panacinar pathology, Harrison’s 18th Ed. 2152, Lancet 2005;365:2225-36 & disproportionate emphysematous involvement of lung bases (compared with more apical distribution seen in 1 AT replete COPD). A. Nephelometry 280 MCQ’s FOR MEDICAL PROFESSIONALS BY PROF. AJAY MATHUR Respiratory 512 In 1 antitrypsin deficiency, retained 1 antitrypsin aggregates A. Chronic hypoxemia (Pa O2 < 75 mmHg) are found in which orgenelle of hepatocytes as inclusions ? B. Chronic hypoxemia (Pa O2 < 65 mmHg) Lancet 2005;365:2225-36 C. Chronic hypoxemia (Pa O2 < 55 mmHg) A. Endoplasmic reticulum D. Chronic hypoxemia (Pa O2 < 45 mmHg) B. Mitochondria PHT severe enough to cause cor pulmonale & RVF due to COPD occurs only in those who have C. Golgi apparatus marked decreases in FEV1 (<25% of predicted) together with chronic hypoxemia (PaO2 <55 mmHg). D. All of the above 519 Which of the following accounts most for the reduction in PaO2 513 Which amino acid is essential in the reactive center of  1- that occurs in COPD ? antitrypsin to inhibits neutrophil elastase ? Harrison’s 18th Ed. 2154 N Engl J Med 2002;346:45 A. Bronchospasm A. Serine B. Shunting B. Methionine C. Ventilation / perfusion mismatching C. Threonine D. Bronchial tree plugging D. Lysine Ventilation/perfusion mismatching accounts for all of the reduction in Pa O2 that occurs in COPD. Inhibition is initiated by docking of protease with serpin. Alpha1-antitrypsin with a methionine at its Shunting is minimal. reactive center inhibits neutrophil elastase. 520 Which of the following is responsible for physiologic 514 Which of the following increase the risk of COPD in PiZ subjects ? alterations that occur with COPD ? Harrison’s 18th Ed. 2152 Harrison’s 18th Ed. 2154 A. Asthma A. Large airways B. Male gender B. Medium airways C. Cigarette smoking C. Small airways and alveolar space D. All of the above D. All of the above 515 Which of the following is the most typical finding in COPD ? Changes in large airways cause cough and sputum, while changes in small airways and alveoli are responsible for physiologic alterations. Harrison’s 18th Ed. 2153 A. Increase in the residual volume 521 Which of the following is responsible for increased resistance B. Persistent reduction in forced expiratory flow rates in most individuals with COPD ? Harrison’s 18th Ed. 2154 C. Increase in residual volume/total lung capacity ratio A. Large airways D. Nonuniform distribution of ventilation B. Medium airways Persistent reduction in forced expiratory flow rates is the most typical finding in COPD. Increases in residual volume & RV/TLC ratio, nonuniform distribution of ventilation & ventilation-perfusion C. Small airways mismatching also occur. D. All of the above 516 In COPD, at rest, PaO2 usually remains near normal until the Major site of increased resistance in COPD is in airways <=2 mm diameter. FEV1 is decreased to ? Harrison’s 18th Ed. 2154 522 Cells secreting surfactant are called ? A. ~ 25 % of predicted Harrison’s 18th Ed. 2154 B. ~ 33 % of predicted A. Kulchitsky cell C. ~ 50 % of predicted B. Clara cell D. ~ 75 % of predicted C. Langerhans cell D. C cell PaO2 usually remains near normal until the FEV1 is decreased to ~50% of predicted at rest. In COPD, characteristic cellular changes include goblet cell metaplasia, with these mucus-secreting 517 In COPD, PaCO2 usually remains near normal until the FEV1 is cells replacing surfactant-secreting Clara cells. Reduced surfactant may increase surface tension at the air-tissue interface, predisposing to airway narrowing or collapse. decreased to ? Harrison’s 18th Ed. 2154 523 Gas exchanging airspaces include ? A. ~ 25 % of predicted Harrison’s 18th Ed. 2154 B. ~ 33 % of predicted A. Respiratory bronchiole C. ~ 50 % of predicted B. Alveolar duct D. ~ 75 % of predicted C. Alveoli In COPD, elevation of PaCO2 is not expected until FEV1 is <25% of predicted. D. All of the above Gas-exchanging airspaces are respiratory bronchioles, alveolar ducts and alveoli. Emphysema is 518 Pulmonary hypertension leading to cor pulmonale & CHF due characterized by destruction of all these three. to COPD occurs in those with FEV1 (<25% of predicted) and ? Harrison’s 18th Ed. 2154 281 Respiratory MCQ’s FOR MCQ’s FOR MEDICAL MEDICAL PROFESSIONALS PROFESSIONALS BY PROF. AJAY MATHUR Cardiology 281 524 Which of the following is false in smokers ? 530 In COPD, Hoover’s sign refers to ? Harrison’s 18th Ed. 2154 Harrison’s 18th Ed. 2156 A. In BAL fluid, macrophages are > 95% of total cell count A. Use of accessory muscles B. Neutrophils account for 1 to 2 % of cells B. Pursed-lip breathing C. CD8+ T lymphocytes are increased in alveolar space C. Diffuse loss of subcutaneous adipose tissue D. None of the above D. Paradoxical inward movement of rib cage with inspiration In smokers’ BAL fluid, macrophages comprise >95% of the total cell count, and neutrophils, nearly Hoover’s sign refers to paradoxical inward movement of rib cage with inspiration. absent in nonsmokers’ lavage, account for 1–2% of the cells. CD8+ lymphocytes are also increased in alveolar space of smokers. 531 Which of the following is not a sign of COPD ? Harrison’s 18th Ed. 2156 525 Which of the following is false ? Harrison’s 18th Ed. 2154 A. Expiratory wheezing A. Panacinar emphysema is seen in  AT deficiency B. Bitemporal wasting B. Centriacinar emphysema is seen in cigarette smokers C. Clubbing of the digits C. Panacinar emphysema has predilection for upper lobes D. Right heart failure D. Centriacinar emphysema is most prominent in the upper Clubbing of the digits is not a sign of COPD, and its presence should alert the clinician to rule out lung cancer or else. lobes and superior segments of lower lobes Panacinar emphysema is usually observed in patients with alpha1AT deficiency, and has a predilection 532 Which of the following is the basis for Gold criteria for COPD for the lower lobes. severity ? Harrison’s 18th Ed. 2156 526 Principal feature of advancing COPD is ? Harrison’s 18th Ed. 2156 A. Airflow obstruction A. Worsening dyspnea on exertion B. Exercise performance B. Cough C. Arterial blood gases and oximetry C. Sputum production D. Body mass index D. All of the above The hallmark and important prognostic factor of COPD is airflow obstruction and is the basis for the Global Initiative for Lung Disease (GOLD) classification for COPD severity. Three most common symptoms in COPD are cough, sputum production & exertional dyspnea. As COPD advances, the principal feature is worsening dyspnea on exertion. 533 Which of the following interventions does not influence the natural history of patients with COPD ? 527 Sitting in “tripod” position, facilitates action of which of the Harrison’s 18th Ed. 2157 following muscles ? Harrison’s 18th Ed. 2156 A. Antibiotics A. Sternocleidomastoid muscle B. Smoking cessation B. Scalene muscle C. Oxygen therapy in chronically hypoxemic patients C. Intercostal muscles D. Lung volume reduction surgery D. All of the above Only three interventions i.e.smoking cessation, oxygen therapy in chronically hypoxemic patients, and lung volume reduction surgery in selected patients with emphysema influence the natural Use of accessory muscles of respiration is done by sitting in characteristic “tripod” position that history of patients with COPD. facilitates the actions of sternocleidomastoid, scalene, and intercostal muscles. 534 Cigarette smoking increases risk for which of the following ? 528 Which of the following is a poor prognostic factor in COPD ? Lancet 2004;364:791-802 Harrison’s 18th Ed. 2156 A. Atherosclerotic vascular disease A. Diffuse loss of subcutaneous adipose tissue B. Osteoporosis B. Significant weight loss C. Peptic ulcer disease C. Bitemporal wasting D. All of the above D. All of the above Cigarette smoking increases atherosclerotic vascular disease risk, osteoporosis & peptic ulcer Independent poor prognostic factors in advanced COPD are systemic wasting, significant weight disease. loss, bitemporal wasting & diffuse loss of subcutaneous adipose tissue. 535 The other action of ‘Bupropion’ used in nicotine replacement 529 Systemic wasting in advanced COPD is related to ? therapy is ? Harrison’s 18th Ed. 2156 Lancet 2004;364:791-802 A. Neutrophil elastase A. Antihypertensive B. Matrix metalloproteinase-12 (MMP-12) B. Antidepressant C. TNF- C. Anticholinergic D. Transforming growth factor  (TGF-) D. Analgesic Systemic wasting in advanced COPD is related to both inadequate oral intake and elevated levels Antidepressant bupropion is shown to be effective in smoking cessation. of inflammatory cytokines (TNF- ). 282 MCQ’s FOR MEDICAL PROFESSIONALS BY PROF. AJAY MATHUR Respiratory 536 Bupropion use is contraindicated in patients of ? 542 Besides bronchodilator action, theophylline also has which Lancet 2004;364:791-802 of the following therapeutic effects ? A. Diabetes mellitus Lancet 2004;364:791-802 B. Hypertension A. Anti-inflammatory effects C. Seizure B. Inotropoic & diuretic effects D. Hypothryoidism C. Augmenting skeletal muscle strength D. All of the above Bupropion is contraindicated in those with increased risk of seizures. Besides the bronchodilator effects of theophylline in COPD, it has anti-inflammatory, inotropoic and 537 Which of the following drugs have been found useful in diuretic effects and has effects in augmenting skeletal muscle strength. smoking cessation ? Harrison’s 18th Ed. 738, Lancet 2004;364:791-802 543 Only therapy demonstrated to decrease mortality in COPD patients is ? A. Clonidine Harrison’s 18th Ed. 2158 B. Nortriptyline A. Supplemental oxygen C. Varenicline B. Beta agonists D. All of the above C. Theophylline Clonidine & tricyclic antidepressant nortriptyline have been found useful in smoking cessation as D. Glucocorticoids second-line treatments. Varenicline is an 4 2 nicotinic acetylcholine receptor partial agonist. Supplemental O2 is the only pharmacologic therapy demonstrated to decrease mortality in COPD. 538 Adverse effect of Varenicline is ? Harrison’s 18th Ed. 738 544 Which of the following is an action of N-acetyl cysteine ? A. Dysphagia Harrison’s 18th Ed. 2158 B. Suicidal ideation A. Antioxidant C. Hemoptysis B. Anti-inflammtory D. Seizure C. Bronchodilator D. Mast cell stabilizer Therapy with an antidepressant bupropion or nicotine replacement therapy with varenicline are approved by USFDA as first-line treatments for nicotine dependence. Both drugs increase N-acetyl cysteine is used in COPD patients for its mucolytic and antioxidant properties. suicidal ideation and must be used with caution. Varenicline is more efficacious than bupropion or placebo. 545 Eligibility for  1 antitrypsin augmentation therapy requires a 539 Nicotine replacement therapy can be carried out by all routes serum  1 antitrypsin level of ? except ? Harrison’s 18th Ed. 2158 Harrison’s 18th Ed. 2157 A. < 6  A. Per rectal B. < 8  B. Transdermal patches C < 10  C. Inhaler D. < 11  D. Nasal spray Eligibility for 1 antitrypsin augmentation therapy requires a serum 1 antitrypsin level < 11  (~ 50 mg/ dL) PiZ & null-null individuals and not in those with normal pulmonary function and a normal chest CT scan. Nicotine replacement therapy is available as gum, transdermal patches, inhaler & nasal spray. 546 Which of the following vaccines is recommended before 540 Bronchodilator effect of anticholinergics is due to blocking administering  1 antitrypsin augmentation therapy ? the action of acetylcholine on ? Harrison’s 18th Ed. 2158 Lancet 2004;364:791-802 A. Influenza vaccine A. M1 muscarinic receptors B. Polyvalent pneumococcal vaccine B. M2 muscarinic receptors C. Hepatitis B vaccination C. M3 muscarinic receptors D. All of the above D. M4 muscarinic receptors Despite sterilization procedures for blood-derived products and the absence of reported cases of Anticholinergics are effective bronchodilators because they block the action of acetylcholine on M3 viral infection from 1 antitrypsin augmentation therapy, some physicians recommend hepatitis B muscarinic receptors that induce contraction on airway smooth muscle. vaccination prior to starting augmentation therapy. 541 Which of the following is a long-acting 2 adrenergic receptor 547 LVRS is used in the management of ? agonist ? Harrison’s 18th Ed. 2158 Lancet 2004;364:791-802 A. Bronchial asthma A. Formoterol fumarate B. ILD B. Albuterol sulfate C. Chronic bronchitis C. Tiotropium bromide D. Emphysema D. None of the above Lung volume reduction surgery (LVRS) reduce volume of lung in emphysema. 283 Respiratory MCQ’s FOR MCQ’s FOR MEDICAL MEDICAL PROFESSIONALS PROFESSIONALS BY PROF. AJAY MATHUR Cardiology 283 548 Lung volume reduction surgery (LVRS) is not recommended if 554 Which of the following is true as “Lung regeneration agent” ? pulmonary artery systolic pressure is ? Lancet 2004;364:985-996 Harrison’s 18th Ed. 2158 A. Retinoic acid (all-trans) A. > 15 mmHg B. Retinoic acid receptor  agonists B. > 25 mmHg C. Stem cells C. > 35 mmHg D. All of the above D. > 45 mmHg Lung regeneration agents include Retinoic acid (all-trans), Retinoic acid receptor  agonists & Stem cells. LVRS is not recommended if patients of emphysema have significant pleural disease, a pulmonary artery systolic pressure >45 mmHg, extreme deconditioning, congestive heart failure, or other 555 Which of the following is used in the “Augmentation therapy” severe comorbid conditions. of COPD ? Lancet 2005;365:2225-36 549 Patients with which of the following are not candidates for LVRS ? Harrison’s 18th Ed. 2158 A. Smoking cessation A. FEV1 < 20% of predicted B. Vaccines B. Diffusely distributed emphysema on CT scan C. Human plasma 1 antitrypsin C. DLCO < 20% of predicted D. Supplemental oxygen D. All of the above In COPD, specific treatment of  1-AT deficiency consists of infusion of purified pooled human plasma 1 antitrypsin - known as augmentation therapy. 550 Patients with which of the following are most likely to benefit from LVRS ? 556 Pooled human plasma is the source of which of the following Harrison’s 18th Ed. 2158 preparations of purified  1 antitrypsin ? Lancet 2005;365:2225-36 A. Upper lobe predominant emphysema A. Prolastin B. Middle lobe predominant emphysema B. Aralast C. Lower lobe predominant emphysema C. Zemaira D. Any of the above D. All of the above Patients with upper lobe predominant emphysema and a low postrehabilitation exercise capacity are most likely to benefit from LVRS. 557 Which season is appropriate for giving influenza vaccination yearly in patients of COPD ? 551 Bacteria frequently implicated in COPD exacerbations include Lancet 2004;364:791-802 all except ? Harrison’s 18th Ed. 2159 A. Autumn A. Streptococcus pneumoniae B. Winter B. Haemophilus influenzae C. Spring C. Staphylococcus aureus D. Summer D. Moraxella catarrhalis Influenza vaccination reduces serious illness & death in COPD patients by 50%. Vaccination should be given once in autumn. Bacteria frequently implicated in COPD exacerbations include S. pneumoniae, H. influenzae, Moraxella catarrhalis, M. pneumoniae or Chlamydia pneumoniae. 558 Etiological factor in acute exacerbations of COPD includes ? Lancet 2004;364:883-895 552 Which of the following is not a parameter in ‘BODE score’ used as a predictor for COPD mortality ? A. Viral infections Lancet 2004;364:985-996 B. Bacterial infections A. Breathing rate C. Particulate air pollutants B. Airflow obstruction D. All of the above C. Dyspnoea Main aetiological factors in acute exacerbations of COPD are viral & bacterial infections & air pollutants. D. Exercise capacity 559 Risk factors predicting increased hospital admission for acute 553 Which of the following drugs is not included in the “not exacerbations in COPD include all except ? recommended or contraindicated” list in GOLD guidelines ? Lancet 2004;364:883-895 Lancet 2004; 364: 791–802 Table 2 A. Low body-mass index A. Mucolytics B. High arterial carbon dioxide tension (PaCO2) B. Narcotics C. Degree of hypoxaemia C. Antitussives D. High mean pulmonary arterial pressure D. Vasodilators 560 Risk factors predicting increased hospital admission for acute Use of mucolytic, antioxidant, immunoregulators, respiratory stimulants is not recommended in pharmaco- logical treatments of COPD. Antitussives & vasodilators are contraindicated. Narcotics can relieve exacerbations in COPD include all except ? dyspnoea (potentially dangerous due to ventilatory suppression) & is used to manage symptoms in Lancet 2004;364:883-895 terminal situations. A. Gas-exchange impairment 284 MCQ’s FOR MEDICAL PROFESSIONALS BY PROF. AJAY MATHUR Respiratory B. Severity of airflow obstruction 566 Granulomas in granulomatous lung disease consists of ? Harrison’s 18th Ed. 2160 C. High arterial carbon dioxide tension (PaCO 2) A. T lymphocytes D. High mean pulmonary arterial pressure B. Macrophages Neither the severity of airflow obstruction nor the degree of hypoxaemia are predictive of hospital admission in COPD C. Epithelioid cells D. All of the above 561 Which of the following is an effective respiratory stimulant ? Lancet 2004;364:883-895 Granulomas in granulomatous lung disease consists of T lymphocytes, macrophages, and epithelioid cells organized into discrete structures in the lung parenchyma. Granulomatous lesions can progress A. Doxapram to fibrosis. Sarcoidosis and hypersensitivity pneumonitis are prime examples. B. Cilomilast 567 The initial insult leading to inflammation and fibrosis in ILD is C. Gefitinib to ? D. Batimastat Harrison’s 18th Ed. 2160 Doxapram is an effective respiratory stimulant and can provide only minor short-term improvement A. Vascular endothelium in blood gas tensions in COPD. B. Epithelial surface C. Perivascular tissues Chapter 261. Interstitial Lung Diseases D. Lymphatic tissues The initial insult is an injury to the epithelial surface that causes inflammation in the air spaces and alveolar walls. Inflammation spreads to adjacent portions of interstitium and vasculature and 562 ILDs involves which of the following in lung parenchyma ? eventually causes interstitial fibrosis. Harrison’s 18th Ed. 2160 A. Capillary endothelium 568 Myofibroblasts produce which of the following ? Harrison’s 18th Ed. 2163, Figure 261-2 B. Perivascular tissues A. Angiotensinogen C. Lymphatic tissues B. Gelatinases D. All of the above C. Collagens ILDs involve parenchyma of lung - alveoli, alveolar epithelium, capillary endothelium, & spaces between these structures, as well as perivascular & lymphatic tissues. D. All of the above 563 Lung response is alveolitis, interstitial inflammation and 569 Patients with idiopathic pulmonary fibrosis typically have all fibrosis in all of the following ILDs except ? the following features except ? Harrison’s 18th Ed. 2161, Table 261-1 N Engl J Med. 2001; 345:517 A. Hypersensitivity pneumonitis A. Exertional dyspnea B. Goodpasture’s syndrome B. Productive cough C. Idiopathic pulmonary hemosiderosis C. Fine bibasilar inspiratory crackles D. Asbestosis D. An abnormal chest radiograph on presentation ILDs are groups in two based on the major underlying histopathology i.e. they are either associated 570 Which of the following initial features is most frequent in ILD with predominant inflammation and fibrosis or they have predominantly granulomatous reaction in interstitial or vascular areas. Hypersensitivity pneumonitis belongs to the later group. patients ? Harrison’s 17th Ed. 1643 564 Etiology of which of the following interstitial lung diseases is known ? A. Wheezing Harrison’s 18th Ed. 2160 B. Chest pain A. Sarcoidosis C. Progressive exertional dyspnea B. Idiopathic pulmonary fibrosis (IPF) D. Hemoptysis C. Pulmonary fibrosis associated with CTDs Patients with ILDs seek medical attention mainly due to progressive exertional dyspnea or a D. None of the above persistent, nonproductive cough. Hemoptysis, wheezing, and chest pain may be present. Sarcoidosis, IPF and pulmonary fibrosis associated with connective tissue diseases (CTD) are the most common ILDs of unknown etiology. Among the ILDs of known cause, the largest group 571 ILDs with symptoms and signs form a chronic presentation includes occupational & environmental exposures. include all except ? Harrison’s 18th Ed. 2161 565 Which of the following statements about ILDs is false ? Harrison’s 18th Ed. 2160 A. Sarcoidosis A. Non-malignant disorders B. Pulmonary Langerhans cell histiocytosis (PLCH) B. Not caused by identified infectious agents C. Churg-Strauss syndrome C. May have granulomatous or inflammatory/fibrosis pattern D. IPF D. None of the above ILDs with s/s as a chronic presentation (months to years) include IPF, sarcoidosis, pulmonary Langerhans cell histiocytosis (PLCH), pneumoconioses, and CTDs. ILDs are nonmalignant disorders not caused by identified infectious agents. Two major histopathologic patterns are granulomatous or inflammation & fibrosis. 285 Respiratory MCQ’s FOR MCQ’s FOR MEDICAL MEDICAL PROFESSIONALS PROFESSIONALS BY PROF. AJAY MATHUR Cardiology 285 572 ILDs with acute presentation include all except ? D. Goodpasture’s syndrome Harrison’s 18th Ed. 2161 Patients with PLCH, desquamative interstitial pneumonia (DIP), Goodpasture’s syndrome, respiratory A. Sarcoidosis bronchiolitis, and pulmonary alveolar proteinosis are almost always current or former smokers. Two- thirds to 75% of patients with IPF have a history of smoking. B. Eosinophilic pneumonia C. Hypersensitivity pneumonitis 578 Which of the following is usually not a clinical feature of D. Churg-Strauss syndrome sarcoidosis ? Harrison’s 18th Ed. 2162 Acute presentation (days to weeks) occurs with allergy (drugs, fungi, helminths), acute interstitial pneumonia (AIP), eosinophilic pneumonia, & hypersensitivity pneumonitis. Episodic presentations A. Dyspnea include eosinophilic pneumonia, hypersensitivity pneumonitis, COP, vasculitides, pulmonary hemorrhage, and Churg-Strauss syndrome. B. Wheezing C. Hemoptysis 573 ILDs that present at age < 50 years include all except ? D. Substernal chest pain Harrison’s 18th Ed. 2161 Dyspnea is a common and prominent complaint in patients with ILD, especially sarcoidosis. A. IPF Wheezing and clinically significant chest pain, though uncommon in ILD, is noted in patients with B. Lymphangioleiomyomatosis (LAM) sarcoidosis. Frank hemoptysis rarely a presenting manifestations of sarcoidosis. C. PLCH 579 Differential diagnosis of sudden worsening of dyspnea with D. Sarcoidosis acute chest pain due to spontaneous pneumothorax in a case of ILD includes ? Most patients with sarcoidosis, ILD associated with CTD, lymphangio-leiomyomatosis (LAM), PLCH, and inherited forms of ILD (familial IPF, Gaucher’s disease, Hermansky-Pudlak syndrome) Harrison’s 18th Ed. 2162 present between the ages of 20 and 40 years. Most patients with IPF are older than 50 years. A. PLCH 574 Interstitial lung disease in which of these conditions is more B. Tuberous sclerosis common in men ? C. LAM Harrison’s 18th Ed. 2161 D. All of the above A. Lymphangioleiomyomatosis (LAM) Sudden worsening of dyspnea, especially if associated with acute chest pain, may indicate a B. Tuberous sclerosis spontaneous pneumothorax, which occurs in PLCH, tuberous sclerosis, LAM & neurofibromatosis. C. Hermansky-Pudlak syndrome D. Rheumatoid arthritis 580 Frank hemoptysis in ILD suggests the possibility which of the following ILDs? LAM & pulmonary involvement in tuberous sclerosis occur exclusively in premenopausal women. ILD in Hermansky-Pudlak syndrome & in CTDs is more common in women. ILD in rheumatoid Harrison’s 18th Ed. 2162 arthritis (RA) is more common in men. A. Diffuse alveolar hemorrhage (DAH) syndromes 575 Familial lung fibrosis is associated with mutation in ? B. Lymphangioleiomyomatosis (LAM) Harrison’s 18th Ed. 2161 C. Tuberous sclerosis A. Surfactant protein C gene D. Any of the above B. Surfactant protein A2 gene Frank hemoptysis and blood-streaked sputum are rarely presenting manifestations of ILD but can C. ATP-binding cassette transporter A3 gene be seen in the diffuse alveolar hemorrhage (DAH) syndromes, LAM, tuberous sclerosis, and the granulomatous vasculitides. D. All of the above 581 Which of the following is a nonspecific finding common to ILDs ? Familial lung fibrosis is associated with mutations in surfactant protein C gene, surfactant protein A2 gene and ATP-binding cassette transporter A3 gene. Harrison’s 18th Ed. 2163 A. Raised LDH 576 Autosomal recessive pattern of inheritance occurs in all B. Antinuclear antibodies except ? Harrison’s 18th Ed. 2161 C. Anti-immunoglobulin antibodies (rheumatoid factors) A. Niemann-Pick disease D. Circulating immune complexes B. Gaucher’s disease Antinuclear antibodies, anti-immunoglobulin antibodies (rheumatoid factors), and circulating immune complexes are identified in some patients, even in the absence of a defined CTD. A raised LDH is C. Neurofibromatosis a nonspecific finding common to ILDs. D. Hermansky-Pudlak syndrome 582 Which of the following laboratory abnormality is common in Family associations (with an autosomal dominant pattern) occur in tuberous sclerosis and sarcoidosis ? neurofibromatosis. An autosomal recessive pattern of inheritance occurs in Niemann-Pick disease, Gaucher’s disease, and the Hermansky-Pudlak syndrome. Harrison’s 18th Ed. 2163 A. Elevated serum angiotensin-converting enzyme level 577 Patients with all of the following are almost always current or B. Presence of serum precipitins former smokers except ? Harrison’s 18th Ed. 2162 C. Antineutrophil cytoplasmic antibodies A. Pulmonary Langerhans cell histiocytosis (PLCH) D. Anti-basement membrane antibodies B. Lymphangioleiomyomatosis (LAM) Elevation of serum ACE level is common in sarcoidosis. C. Desquamative interstitial pneumonia (DIP) 286 MCQ’s FOR MEDICAL PROFESSIONALS BY PROF. AJAY MATHUR Respiratory 583 In which of the following ILDs, nodular opacities in upper lung C. Reduction in residual volume zones is frequent ? D. All of the above Harrison’s 18th Ed. 2163 Most forms of ILD produce a restrictive defect with reduced total lung capacity (TLC), functional A. Sarcoidosis residual capacity, and residual volume. FEV1 and FVC are reduced, but these changes are related B. PLCH to the decreased TLC. FEV1/FVC ratio is usually normal or increased. Lung volumes decrease as lung stiffness worsens with disease progression. C. Chronic hypersensitivity pneumonitis D. All of the above 589 Which of the following about decreased DlCO in most ILDs is false ? ILDs that show nodular opacities with a predilection for the upper lung zones include sarcoidosis, Harrison’s 18th Ed. 2164 PLCH, chronic hypersensitivity pneumonitis, silicosis, berylliosis, RA (necrobiotic nodular form), and ankylosing spondylitis. A. Common nonspecific finding in most ILDs B. Due to mismatching of ventilation & perfusion (V./Q.) 584 Which of the following chest x-ray findings is indicative of a poor prognosis in a case of ILD ? C. Does not correlate with disease stage Harrison’s 18th Ed. 2163 D. None of the above A. Bibasilar reticular pattern 590 Which of the following is rare in arterial blood gas analysis in B. Honeycombing ILDs ? C. Nodular pattern of alveolar filling Harrison’s 18th Ed. 2164 D. Mixed pattern of alveolar filling A. Normal resting arterial blood gas Most common chest radiograph abnormality in ILD is a bibasilar reticular pattern. Radiographic B. Hypoxemia finding of honeycombing suggests small cystic spaces & progressive fibrosis which carry a poor prognosis. C. Carbon dioxide (CO2) retention D. Respiratory alkalosis 585 Pathologic changes that characterize idiopathic pulmonary Carbon dioxide (CO2) retention is rare and is a manifestation of end-stage ILD. fibrosis are all except ? N Engl J Med. 2001; 345:517 591 Which of the following is a bronchoalveolar lavage (BAL) A. Predilection for peripheral subpleural parenchyma finding in sarcoidosis ? B. Fibrotic zones with associated honeycombing alternate Harrison’s 18th Ed. 2165, Table 261-3 with areas of relatively unaffected lung tissue A. Eosinophils > 25% C. Fibrotic areas vary in age and activity B. Hemosiderin-laden macrophages D. “Fibroblast foci” occur at sites of chronic lung injury C. CD4 : CD8 ratio > 3.5 “Fibroblast foci” occur at sites of acute lung injury. D. Atypical hyperplastic type II pneumocytes 586 HRCT in idiopathic pulmonary fibrosis shows all except ? 592 Decreased CD4:CD8 ratio in bronchoalveolar lavage (BAL) is a Harrison’s 18th Ed. 2164, Figure 261-3 feature of ? Harrison’s 18th Ed. 2165, Table 261-3 A. Bibasal peripheral lower lobe reticular opacities A. Organizing pneumonia B. Peripheral “honeycombing” B. Eosinophilic lung disease C. Traction bronchiectasis C. Hypersensitivity pneumonitis D. Pleural thickening D. Pulmonary Langerhans cell histiocytosis HRCT lung scans in IPF show patchy, predominantly basilar, subpleural reticular opacities, with traction bronchiectasis and honeycombing. 593 Which of the following is false about idiopathic pulmonary fibrosis ? 587 HRCT finding that goes against the diagnosis of idiopathic Harrison’s 18th Ed. 2165 pulmonary fibrosis is ? Harrison’s 18th Ed. 2165 A. Most common form of idiopathic interstitial pneumonia A. Extensive ground-glass abnormality B. Has a distinctly poor response to therapy B. Nodular opacities with upper or mid-zone predominance C. Has a bad prognosis C. Prominent hilar or mediastinal lymphadenopathy D. None of the above D. All of the above 594 Usual interstitial pneumonia (UIP), nonspecific interstitial HRCT findings that suggest an alternative diagnosis to IPF include extensive ground-glass pneumonia, organizing pneumonia etc are categories of ILD abnormality, nodular opacities, upper or mid-zone predominance & prominent hilar or mediastinal based on ? lymphadenopathy. Harrison’s 18th Ed. 2160 588 Pulmonary-function test in patients with idiopathic pulmonary A. Clinical presentation fibrosis will show ? B. Radiological findings Harrison’s 18th Ed. 2164 C. Histopathologic patterns A. Reduction in total lung capacity D. Response to treatment B. Reduction in functional residual capacity 287 Respiratory MCQ’s FOR MCQ’s FOR MEDICAL MEDICAL PROFESSIONALS PROFESSIONALS BY PROF. AJAY MATHUR Cardiology 287 Important histopathologic patterns found in ILDs include usual interstitial pneumonia (UIP), nonspecific D. Pleural calcification interstitial pneumonia, respiratory bronchiolitis/desquamative interstitial pneumonia, organizing pneumonia, diffuse alveolar damage (acute or organizing), and lymphocytic interstitial pneumonia. HRCT of NSIP shows bilateral, subpleural ground-glass opacities. Honeycombing is unusual. Key histopathologic features are uniformity of interstitial involvement across biopsy section. 595 Histologic hallmark of Usual interstitial pneumonia (UIP) is ? 601 Hamman Rich Syndrome is the name given to ? Harrison’s 18th Ed. 2165 Harrison’s 18th Ed. 2166 A. Alternating areas of normal lung & interstitial inflammation A. Acute interstitial pneumonia (AIP) B. Foci of proliferating fibroblasts B. Hypersensitivity pneumonitis C. Dense collagen fibrosis C. Desquamative interstitial pneumonia (DIP) D. All of the above D. Respiratory bronchiolitis The histologic hallmark and chief diagnostic criterion of UIP is a heterogeneous appearance at low magnification with alternating areas of normal lung, interstitial inflammation, foci of proliferating Hamman Rich Syndrome refers to Acute interstitial pneumonia (AIP). fibroblasts, dense collagen fibrosis, and honeycomb changes affecting peripheral, subpleural parenchyma most severely. Lesion is idiopathic and not associated with another condition. 602 Which of the following is a fulminant form of ILD ? Harrison’s 18th Ed. 2166 596 Treatment options in Usual interstitial pneumonia (UIP) patient A. Acute interstitial pneumonia (AIP) include all except ? Harrison’s 16th Ed. 1557 B. Hypersensitivity pneumonitis A. Azathioprine C. Desquamative interstitial pneumonia (DIP) B. Interferon alpha D. Respiratory bronchiolitis C. Colchicine AIP is a fulminant form of lung injury characterized histologically by diffuse alveolar damage on lung biopsy. Onset of drug-Induced ILD may be abrupt and fulminant, or insidious. D. Pirfenidone 603 AIP is similar in presentation to ? 597 In the treatment of Usual interstitial pneumonia (UIP), which of Harrison’s 18th Ed. 2166 the following is an antifibrotic agents ? Harrison’s 16th Ed. 1557 A. Spontaneous pneumothorax A. Colchicine B. Acute respiratory distress syndrome (ARDS) B. Pirfenidone C. Bronchial asthma C. Interferon gamma-1b D. CHF D. All of the above AIP is similar in presentation to acute respiratory distress syndrome (ARDS). Treatment options for Usual interstitial pneumonia (UIP) include glucocorticoids, cytotoxic agents like azathioprine, cyclophosphamide & antifibrotic agents like colchicine, pirfenidone, or interferon -1b. 604 Which of the following is known as idiopathic BOOP ? Harrison’s 18th Ed. 2166 598 Which of the following drug is effective in acute exacerbations A. Cryptogenic Organizing Pneumonia of IPF ? B. Desquamative interstitial pneumonia Harrison’s 18th Ed. 2166 C. Respiratory bronchiolitis A. Glucocorticoids D. Lymphocytic interstitial pneumonia B. Azathioprine Cryptogenic Organizing Pneumonia (COP) is also known as idiopathic BOOP (bronchiolitis obliterans C. Interferon gamma-1b with organizing pneumonia). D. None of the above 605 Which of the following ILD is associated with cigarette smoking ? No therapy has been found to be effective in the management of acute exacerbations of IPF. Harrison’s 18th Ed. 2167 599 Which of the following about Idiopathic Nonspecific Interstitial A. Desquamative Interstitial Pneumonia (DIP) Pneumonia (NSIP) is false ? B. Respiratory Bronchiolitis - Associated ILD (RB-ILD) Harrison’s 18th Ed. 2166 C. Pulmonary Langerhans Cell Histiocytosis (PLCH) A. Good prognosis D. All of the above B. Presents at a younger age C. Occurs in women who have never smoked 606 Which of the following is false about Desquamative interstitial pneumonia (DIP) ? D. None of the above Harrison’s 18th Ed. 2167 Idiopathic NSIP is a subacute restrictive process with presentation similar to IPF, most common in A. Found exclusively in cigarette smokers women of younger age who have never smoked. B. Macrophages in intraalveolar spaces 600 Laboratory feature of NSIP is ? C. Minimal interstitial fibrosis Harrison’s 18th Ed. 2166 D. Has a worse prognosis than IPF A. Honeycombing DIP occurs exclusively in cigarette smokers. Histologic hallmark is extensive accumulation of B. Uniformity of interstitial involvement macrophages in intraalveolar spaces with minimal interstitial fibrosis. DIP has a better prognosis than IPF. C. Traction bronchiectasis 288 MCQ’s FOR MEDICAL PROFESSIONALS BY PROF. AJAY MATHUR Respiratory 607 Which of the following about Pulmonary Langerhans Cell 613 Which of the following about pulmonary alveolar proteinosis Histiocytosis (PLCH) is false ? (PAP) is false ? Harrison’s 18th Ed. 2167 Harrison’s 18th Ed. 2168 A. Smoking-related A. Impaired ability to process surfactant B. HRCT shows nodules & thin-walled cysts B. Accumulation of PAS positive lipoproteinaceous material in distal air spaces C. Increased DLCO C. Extensive lung inflammation D. Persistent / progressive disease D. Preserved lung architecture Most frequent pulmonary function abnormality is a markedly reduced DL CO along with restrictive disease pattern, airflow limitation & diminished exercise capacity. There is a defect in macrophage function with impaired ability to process surfactant. Amorphous, PAS-positive lipoproteinaceous material accumulates in distal air spaces. There is little or no lung inflammation, and the underlying lung architecture is preserved. 608 Radiographic features of PLCH include all except ? Harrison’s 18th Ed. 2167 614 In pulmonary alveolar proteinosis (PAP), the neutralizing IgG A. Reticular or nodular opacities antibody is against which of the following ? B. Bizarre-shaped, thin-walled upper zone cysts Harrison’s 18th Ed. 2168 C. Honeycombing A. Surfactant D. Sparing of the costophrenic angles B. GM-CSF CxR of PLCH include a ill-defined or stellate nodules, reticular or nodular opacities, bizarre-shaped C. PAS positive lipoproteinaceous material upper zone thin-walled cysts & sparing of costophrenic angles. Honeycombing is a feature of IPF. D. All of the above 609 Most common form of pulmonary involvement in connective PAP is an autoimmune disease with neutralizing antibody of IgG isotype against granulocyte- macrophage colony-stimulating factor (GM-CSF). Neutralization of GM-CSF bioactivity by the tissue disorders is ? antibody causes dysfunction of alveolar macrophages, which results in reduced surfactant clearance. Harrison’s 18th Ed. 2167 Elevated serum anti-GM-CSF titer is highly sensitive & specific for diagnosis of acquired PAP. A. Respiratory bronchiolitis 615 Which of the following is the most common class of Pulmonary B. Desquamative interstitial pneumonia Alveolar Proteinosis (PAP) ? C. Cryptogenic organizing pneumonia Harrison’s 18th Ed. 2168 D. Nonspecific interstitial pneumonia A. Acquired Commonest form of pulmonary involvement in connective tissue disorders is nonspecific interstitial B. Congenital pneumonia. C. Secondary 610 Rheumatoid pneumoconiosis is also called ? D. None of the above Harrison’s 18th Ed. 2167 There are three distinct classes of PAP. Acquired (>90%), congenital, and secondary. A. Hermansky-Pudlak syndrome B. Churg-Strauss syndrome 616 Secondary pulmonary alveolar proteinosis (PAP) is caused by which of the following malignancies ? C. Caplan’s syndrome Harrison’s 18th Ed. 2168 D. Goodpasture’s syndrome A. Lung Caplan’s syndrome is the term given to rheumatoid pneumoconiosis. B. Pleural C. Hematopoietic 611 Most common pulmonary manifestation in Systemic Lupus Erythematosus (SLE) is ? D. Hepatic Harrison’s 18th Ed. 2167 617 Secondary PAP is caused by ? A. Atelectasis Harrison’s 18th Ed. 2168 B. Pleuritis A. Lysinuric protein intolerance C. Pulmonary vascular disease B. Acute silicosis D. Infectious pneumonia C. Hematopoietic malignancies Pleuritis with or without effusion is the most common pulmonary manifestation in SLE. D. All of the above Secondary PAP is caused by lysinuric protein intolerance, acute silicosis, immuno-deficiency 612 ILD occurs more commonly in the subgroup of polymyositis & disorders, hematopoietic malignancies and hematopoietic disorders. dermatomyositis patients having which antibody ? Harrison’s 18th Ed. 2168 618 Congenital PAP is caused by mutation in ? A. Anti-Smith antibody Harrison’s 18th Ed. 2168 B. Anti-Ro antibody A. SP-A gene C. Anti-Jo-1 antibody B. SP-B gene D. Anti-La antibody C. SP-C gene 80% of patients of PM/DM with anti-Jo-1 antibodies (directed to histidyl tRNA synthetase) have D. SP-D gene interstitial lung disease. Congenital PAP is transmitted in autosomal recessive manner & is caused by mutation in SP-B gene. 289 Respiratory MCQ’s FOR MCQ’s FOR MEDICAL MEDICAL PROFESSIONALS PROFESSIONALS BY PROF. AJAY MATHUR Cardiology 289 619 In PAP, which of the following is frequent ? D. All of the above Harrison’s 18th Ed. 2168 Oophorectomy, progesterone, sirolimus, tamoxifen & luteinizing hormone-releasing hormone analogues A. Polycythemia have been used in pulmonary LAM. Lung transplantation offers the only hope for cure. B. Hypergammaglobulinemia 625 Which of the following is not a type of diffuse interstitial C. Increased LDH disease ? D. All of the above Harrison’s 16th Ed. 1497 Apart from elevated serum levels of lung surfactant proteins A & D, polycythemia, A. Pneumoconiosis hypergammaglobulinemia and increased LDH levels are frequent in PAP. B. Hypersensitivity pneumonitis 620 Radiographically, ‘bat-wing’ distribution of lung opacities is C. Eosinophilic granuloma suggestive of which of the following ? D. Acute respiratory distress syndrome Harrison’s 18th Ed. 2168 A. Churg-Strauss syndrome 626 Which of the following is not a type of diffuse alveolar disease ? Harrison’s 16th Ed. 1497 B. Pulmonary lymphangioleiomyomatosis (LAM) A. Cardiogenic pulmonary edema C. Pulmonary alveolar proteinosis (PAP) B. Acute respiratory distress syndrome D. Goodpasture’s syndrome C. Sarcoidosis Radiographically, bilateral symmetric alveolar opacities located centrally in mid and lower lung zones result in a ‘bat-wing’ distribution in PAP. D. Eosinophilic granuloma 621 Pulmonary lymphangioleiomyomatosis (LAM) is seen in ? 627 Injury to which of the following leads to hemoptysis in Diffuse Harrison’s 18th Ed. 2168 Alveolar Hemorrhage ? Harrison’s 18th Ed. 2168 A. Female child A. Arterioles B. Premenopausal women B. Venules C. Postmenopausal women C. Capillaries D. Any of the above D. Any of the above Pulmonary LAM afflicts premenopausal women and should be suspected in young women with emphysema, recurrent pneumothorax, or chylous pleural effusion. The disease accelerates during Syndromes of ILD with diffuse alveolar hemorrhage (DAH) may arise due to injury to arterioles, pregnancy and abates after oopherectomy. venules, and the alveolar septal capillaries. Bleeding into alveolar spaces results in hemoptysis secondary to disruption of alveolar-capillary basement membrane. 622 In Pulmonary lymphangioleiomyomatosis (LAM), there occurs proliferation of which of the following ? 628 Which of the following is increased in Diffuse Alveolar Harrison’s 18th Ed. 2168 Hemorrhage (DAH) ? Harrison’s 18th Ed. 2168 - 69 A. Type 1 pneumocytes A. PaO 2 B. Type 2 pneumocytes B. PAO2 C. Pulmonary lymphatic vessels C. Dl CO D. Atypical pulmonary interstitial smooth muscle D. PaCO 2 Pathologically, pulmonary LAM is characterized by the proliferation of atypical pulmonary interstitial smooth muscle and cyst formation. In DAH, DlCO may be increased due to increased hemoglobin within alveoli compartment. 623 Atypical smooth-muscle cells that proliferate in pulmonary 629 Immune complexes are absent in examination of lung or renal lymphangioleiomyomatosis (LAM) react with which of the tissue by immunofluorescent techniques in ? following monoclonal antibody ? Harrison’s 18th Ed. 2169 Harrison’s 18th Ed. 2168 A. Granulomatosis with polyangiitis (Wegener’s) A. HMB42 B. Microscopic polyangiitis pauci-immune B. HMB43 glomerulonephritis C. HMB44 C. Isolated pulmonary capillaritis D. HMB45 D. All of the above Immature-appearing smooth-muscle cells react with monoclonal antibody HMB45, which recognizes Immune complexes are absent (pauci-immune) in examination of lung or renal tissue by gp100 originally found in human melanoma cells. immunofluorescent techniques in granulomatosis with polyangiitis (Wegener’s), microscopic polyangiitis pauci-immune glomerulonephritis & isolated pulmonary capillaritis. 624 Which of the following modalities of treatment is useful in Pulmonary lymphangioleiomyomatosis (LAM) ? 630 Linear deposition of immune complexes is seen in examination Harrison’s 18th Ed. 2168 of lung or renal tissue by immunofluorescent techniques in ? Harrison’s 18th Ed. 2169 A. Oophorectomy A. SLE B. Progesterone B. Goodpasture’s syndrome C. Tamoxifen C. Henoch-Schönlein purpura 290 MCQ’s FOR MEDICAL PROFESSIONALS BY PROF. AJAY MATHUR Respiratory D. Wegener’s granulomatosis C. Vascular malformation Linear deposition of immune complexes is seen in examination of lung or renal tissue by D. Sarcoidosis immunofluorescent techniques in Goodpasture’s syndrome. Granular pattern is found in CTDs, particularly SLE. Granular deposition of IgA-containing immune complexes is seen in Henoch- Causes of solitary circumscribed density.nodule on CxR are primary or metastatic neoplasm, Schönlein purpura. localized infection, Wegener’s granulomatosis, Rheumatoid nodule, Vascular malformation and Bronchogenic cyst. Sarcoidosis leads to diffuse interstitial disease. 631 Inherited diseases that produce interstitial lung disease include ? 637 Which of the following is not a cause of diffuse nodular Harrison’s 18th Ed. 2169 pulmonary disease ? Harrison’s 17th Ed. 1585 Table 245-2 A. Tuberous sclerosis A. Metastatic neoplasm B. Neurofibromatosis B. Sarcoidosis C. Niemann-Pick disease C. Pneumoconiosis D. All of the above D. Eosinophilic granuloma 632 Inherited diseases that produce interstitial lung disease Causes of diffuse nodular disease on CxR include metastatic neoplasm, hematogenous spread of include ? infection, pneumoconiosis and eosinophilic granuloma. Harrison’s 18th Ed. 2169 A. Gaucher’s disease 638 Pulmonary hemorrhage syndromes include ? Harrison’s 17th Ed. 1643 Table 255-1 B. Hermansky-Pudlak syndrome A. Goodpasture’s syndrome C. Niemann-Pick disease B. Idiopathic pulmonary hemosiderosis D. All of the above C. Isolated pulmonary capillaritis Inherited disorders associated with ILD include phakomatoses, tuberous sclerosis, neurofibromatosis, Niemann-Pick disease, Gaucher’s disease and Hermansky-Pudlak syndrome. D. All of the above 633 Which of the following is false about bronchocentric 639 Gastrointestinal / liver diseases that can produce interstitial granulomatosis (BG) ? lung disease include ? Harrison’s 17th Ed. 1643 Table 255-1 Harrison’s 18th Ed. 2169 A. Descriptive clinical term A. Crohn’s disease / ulcerative colitis B. Hypersensitivity reaction to Aspergillus B. Primary biliary cirrhosis C. Peripheral blood eosinophilia C. Chronic active hepatitis D. CxR may show irregularly shaped mass lesions D. All of the above Rather than a specific clinical entity, BG is a descriptive histologic term. Chapter 329. Sarcoidosis 634 Bronchocentric granulomatosis (BG) is associated with which of the following pulmonary condition ? Harrison’s 18th Ed. 2169 640 Besnier-Boeck-Schauman disease refers to ? A. Asthma Boeck C. Multiple benign sarcoid of the skin. J Cutan Genitourin Dis 1899;17:543-50. B. Bacterial pneumonia A. Hydatid disease of lung C. Pleural effusion B. Syphilis of lung D. Primary spontaneous pneumothorax C. Sarcoidosis BG is a pathologic nonspecific response to airway injuries. BG can be caused by a hypersensitivity D. Miliary tuberculosis reaction to Aspergillus in patients with asthma. 641 Condition known to cause granulomas is ? 635 Treatment of choice in bronchocentric granulomatosis is ? Harrison’s 18th Ed. 2805 Harrison’s 18th Ed. 2169 A. Fungal infections A. Glucocorticoids B. Malignancy B. Chemotherapy C. Beryllium C. Surgery D. All of the above D. Radiation Condition known to cause granulomas are sarcoidosis, mycobacterial and fungal infections, malignancy and environmental agents such as beryllium. Glucocorticoids are treatment of choice in bronchocentric granulomatosis (BG). 642 Which of the following is false about sarcoidosis ? 636 Which of the following is not a cause of solitary circumscribed Harrison’s 18th Ed. 2805 pulmonary nodule ? Harrison’s 17th Ed. 1585 Table 245-2 A. Inflammatory disease A. Wegener’s granulomatosis B. Noncaseating granulomas B. Bronchogenic cyst C. Multisystem involvement Central nervous system D. heart. DRB1*1101 Lupus pernio is the term for chronic sarcoidosis-related indurated. Erythema nodosum B. 2136 A. Langerhans cell histiocytosis. 2806 A. Diseases that affect the upper lobe are sarcoidosis. fingers & knees. Sarcoidosis rarely involve breast. All of the above is also common. Hyper. bone. DQB1*0201 Sarcoidosis is seen worldwide. blue-purple lesions HLA-DRB1*1101 is associated with an increased risk for developing sarcoidosis. presence of involvement in two or more organs is required. GM-CSF . Lupus pernio A. silicosis. hilar adenopathy and uveitis. 646 Which of the following organs is least affected in sarcoidosis ? 652 The classic cutaneous lesion in sarcoidosis is ? Harrison’s 18th Ed. testes. DRB1*1501 D. Involvement of skin. 2810 Harrison’s 18th Ed.and hypopigmentation C. AJAY MATHUR Cardiology 291 D. Arthritis B. Bilateral hilar adenopathy D. Elbows D. Bacterial Löfgren’s syndrome refers to an acute presentation of sarcoidosis consisting consists of erythema components (MDP & IT27 antigen). eye. Eye B. Lymph node C. and subcutaneous nodules. hyper- are at greatest risk. 647 Which of the following HLA haplotype is associated with an 653 The location of lesions in “lupus pernio” is on ? increased risk for developing sarcoidosis ? N Engl J Med 2007. Propionibacter acnes B. and central nervous system Classic cutaneous lesions in sarcoidosis include erythema nodosum. 644 Highest prevalence of sarcoidosis is reported in ? 650 Which of the following HLA haplotype is highly associated Harrison’s 18th Ed. and hypopigmentation. HLA-DQB1*0201 is highly associated with Löfgren’s syndrome. Heart A. with highest prevalence reported in the Nordic population. demonstrated in sarcoid granulomas. Keloid formation D. 645 Organ most frequently affected in sarcoidosis is ? Harrison’s 18th Ed. eye. Eotaxin For a specific diagnosis. ears. cheeks. and subcutaneous nodules. DRB1*0402 B. 2805 Harrison’s 18th Ed. Pseudomonads D. Heel C. 643 Which of the following has been implicated in the possible etiology of sarcoidosis ? 649 Löfgren’s syndrome is characterized by ? Harrison’s 18th Ed. 291 Respiratory MCQ’s FOR MCQ’s FOR MEDICAL MEDICAL PROFESSIONALS PROFESSIONALS BY PROF. DQB1*0302 (Type 1 DM). Ovary C. liver & lymph nodes D. All of the above Propionibacterium acnes is so far the only bacterium to be found in sarcoid lymph nodes. violaceous. Hispanic population B. Liver B. Burkholderia pseudomallei A. lips. Capnocytophaga canimorsus C. Hypersensitivity pneumonitis C. Lymphoma cutis B. keloid formation. Cheek and nose B. ovary. DRB1*0402 B. HLA-DRB1*03 was found in two- thirds of Scandinavian patients with Löfgren’s syndrome. DRB1*0402 on nose. 2806 with Löfgren’s syndrome? Harrison’s 17th Ed. but lung. hypersensitivity pneumonitis. Lung Harrison’s 18th Ed. Red Indian tribes D. 2808 A. Nape of the neck A. Erythema nodosum C. acnes is indigenous to skin of healthy humans. D. DQB1*0302 C. 2810 651 Disease that affect the upper lobe is ? A. tuberculosis & Pneumocystis pneumonia. liver. 2806 A. Involvement in >= 3 organs for a specific diagnosis C. 309 Harrison’s 18th Ed. Aborigines in Australia C. P. ~20% of patients with sarcoidosis develop a chronic persistent form of the disease and is associated with the secretion of high levels of IL-8. 648 Persistent chronic form of sarcoidosis is associated with the 654 Violaceous papules and plaques are seen in ? secretion of high levels of ? Harrison’s 16th Ed. or stomach. strongly suggest the causative nodosum. MCP-1 Sarcoidosis is an inflammatory multisystem disease characterized by noncaseating granulomas.357:2153-65 Harrison’s 18th Ed. maculopapular lesions. DRB1*1501 (Multiple sclerosis). IL-8 B. agent of sarcoidosis to be derived from some bacteria. Langerhans cell histiocytosis Organ most frequently affected in sarcoidosis is lung. Silicosis D. Nordic population A. DRB1*1501 D. 2807 B. lumpy. 2806 A. Skin A. All of the above Any organ can be threatened by sarcoidosis. It can be disfiguring by eroding into underlying cartilage & (Pemphigus vulgaris). DQB1*0302 C. 357:2153-65 B. Elevation of transaminase levels A. Thrombocytopenia C. D. myelopathy. MIP-1. Eye cranial-nerve palsies. Complete hypopituitarism & infertility are rare. Lymph nodes in sarcoidosis are firm. Parotid enlargement D. 658 Which out of the following is the most common neurologic the nodes do not ulcerate. bears some similarities to 659 Which cranial nerve is most commonly involved in sarcoidosis ? childhood sarcoidosis. basilar meningitis. 2809 and eye involvement but the lungs are not involved. Children with Blau’s syndrome present with granulomatous arthritis and skin Harrison’s 18th Ed. seizures & cognitive Violaceous papules and plaques are seen in lupus pernio. D. AJAY MATHUR Respiratory C. Ulcerate Mechanism of hypercalcemia and/or hypercalciuria in sarcoidosis is Increased production of 1. Elevation of alkaline phosphatase level Harrison’s 18th Ed. VII A. Unlike tuberculosis. III 665 In Sarcoidosis. Skin D. lymphoma cutis & cutaneous lupus. All of the above C. CD4+ T cells Seventh cranial nerve involvement with unilateral facial paralysis is most common in sarcoidosis. ataxia. weakness & seizures. and GM-CSF. Macrophage inflammatory protein 1 (MIP-1) 660 Which of the following can be a feature of neurosarcoidosis ? D. and -18. secrete interleukin-12. painless.25-dihydroxyvitamin D by 663 Which of the following is not a feature of lymphadenopathy in sarcoidosis ? granuloma Harrison’s 16th Ed. Seizures C. 2809 662 Which of the following is not a feature of Heerfordt- A. Fever C. VI N Engl J Med 2007. D. Ataxia A. Infertility consistent with an obstructive pattern. Increased production of 1. Posterior uveitis Most common hematologic problem in sarcoidosis is lymphopenia. 655 Most common abnormality of liver function in sarcoidosis is ? 661 Endocrine abnormality most commonly seen in sarcoidosis Harrison’s 18th Ed.357:2153-65 D. 2808 is ? A. are D.25.292 MCQ’s FOR MEDICAL PROFESSIONALS BY PROF. Harrison’s 18th Ed. Joints C. anterior sarcoidosis is ? uveitis & facial nerve palsy. Hypothalamic-pituitary axis is most commonly involved in sarcoidosis & presents as diabetes insipidus. presentation in sarcoidosis ? N Engl J Med 2007. 2019 B. C. V initiating the formation and maintenance of granulomas ? C. Lymphopenia A. Leukopenia B. Lungs Most common neurological presentations in sarcoidosis. 2809 A. Increased ACE levels B. also called Blau’s syndrome. All of the above Areas of nervous system affected in neurosarcoidosis include cranial nerve involvement. headache. Addison’s syndrome has been described. cognitive dysfunction. Optic neuritis is also a cranial nerve manifestation of sarcoidosis. 656 Most common hematologic problem in sarcoidosis is ? Harrison’s 18th Ed. CD4+ T cells release IL-2 & interferon-alpha. 2020 B. GM-CSF Harrison’s 18th Ed. C. Myelopathy the formation & maintenance of granulomas. Anterior hypothalamic disease . which of the following is instrumental in B. 2809 B. Painless D. Cognitive dysfunction B. Tumor necrosis factor alpha B. Familial juvenile systemic granulomatosis. 2809 Antigen-presenting cells (APC). Addison’s syndrome The most common abnormality of liver function is an elevation of the alkaline phosphatase level.357:2153-65 664 Which of the following is not involved in Familial juvenile systemic granulomatosis or Blau’s syndrome ? A. A. anterior hypothalamic disease (diabetes insipidus). Diabetes insipidus C. Facial nerve palsy 657 Mechanism of hypercalcemia and/or hypercalciuria in Heerfordt-Waldenstrom syndrome describes individuals with fever. -15. Anemia Waldenstrom syndrome ? Harrison’s 16th Ed. in addition to producing high levels of tumor necrosis factor alpha. Nonadherent dihydroxyvitamin D by granuloma. A. Elevation of bilirubin level B. listed in decreasing order of frequency. Suppressed parathyroid hormone (PTH) level A. Cutaneous lupus D. All of the above D. Complete hypopituitarism D. The Kveim–Siltzbach skin test is negative. nonadherent with a rubbery texture. Cranial-nerve palsies N Engl J Med 2007. monocyte chemotactic protein 1 (MCP-1). None of the above C. parotid enlargement. changes. Basilar meningitis A cardinal feature of sarcoidosis is the presence of CD4+ T cells that interact with APCs to initiate B. Rubbery texture C. stage 1 is bilateral hilar lymphadenopathy without infiltration. Hyperglobulinemia infiltration of heart muscle by granulomas. respiratory infection & 669 Which of the following is typically unilateral in sarcoidosis ? failure. 2809 diseases. Anterior hypothalamic disease sarcoidosis is ? N Engl J Med 2007. Sarcoidal granulomas may persist. paraplegia. renal failure. Parotid enlargement is a classic feature of sarcoidosis & bilateral involvement C. stage 2 is Harrison’s 18th Ed. C. Large numbers of activated TH1 cells in affected organs D. Pulmonary or skin involvement the diagnosis of sarcoidosis over other ILDs ? D. Myelopathy Sarcoidal granulomas are organized. Paraplegia Fibrosis and the resulting obliteration of the pulmonary vessels is the most common mechanism for C. Meningeal enhancement indicate disease chronicity or correlate with changes in pulmonary function.357:2153-65 D. often with involvement of the conducting system. These stages do not A. Stage 3 sarcoidosis refers to which of B. 2810 C. D. B. Granulomatous infiltration of pulmonary arterioles Harrison’s 18th Ed. epithelioid cells. None of the above 672 The most common location for sarcoid granulomas and scars T H1 lymphocytes accumulate at sites of disease and proliferate at an exaggerated rate that is in the heart is ? maintained by spontaneous release of interleukin (IL) 2. Inter ventricular septum 667 Which of the following is an infrequent constituent of sarcoidal C. A. Basilar meningitis 668 The most common mechanism for pulmonary hypertension in D. Multiple enhancing white matter abnormalities in MRI suggest multiple sclerosis. Fibrosis with obliteration of pulmonary vessels 674 Which of the following is a complication of sarcoidosis ? B. Presence of meningeal enhancement or hypothalamic B. bronchiectasis. Inter atrial septum blood of affected individuals. Infiltration alone is the rule.5 cm D. T cells in nonaffected organs are quiescent Cardiac disease in sarcoidosis presents as CHF. B. Bilateral hilar lymphadenopathy without infiltration D. 293 Respiratory MCQ’s FOR MCQ’s FOR MEDICAL MEDICAL PROFESSIONALS PROFESSIONALS BY PROF. Renal failure pulmonary hypertension in sarcoidosis. hilar retraction. Macrophage A. In the involved organs. bronchiectasis and diaphragmatic tenting. Numbers of TH1 cells in blood are normal or slightly reduced. Epithelioid cells The most common location for granulomas and scars is the left ventricular free wall.0 cm involvement and evidence of extraneurologic disease such as pulmonary or skin involvement suggests neurosarcoidosis. AJAY MATHUR Cardiology 293 666 Which of the following about active sarcoidosis is false ? B. B. structured masses composed of macrophages & their derivatives. hilar retraction. Parotid enlargement the following ? N Engl J Med 2007. 2020 A.357:2153-65 C. Erythema nodosum B.357:2153-65 A link between fatigue & small peripheral nerve fiber disease is noted in sarcoidosis. bullae. followed by the intraventricular septum. A. fatigue is linked to which of the following ? Harrison’s 18th Ed. All of the above Harrison’s 18th Ed. T cell growth factor by activated TH1 cells N Engl J Med 2007.0 cm 671 Which of the following occurs due to cardiac involvement in sarcoidosis ? Adenopathy >2 cm in CT short axis favours diagnosis of sarcoidosis over other interstitial lung Harrison’s 18th Ed. resolve. bullae. All of the above Sarcoidosis complications include blindness. Hilar lymphadenopathy is almost always bilateral. T cells C. Bilateral hilar lymphadenopathy with infiltration Pleural involvement in sarcoidosis almost always is unilateral pleural effusion. 2810 A. D. Small peripheral nerve fiber disease C. Hilar lymphadenopathy 675 By chest radiographs. CxR wise. > 1. the ratio of CD4+ to CD8+ T cells may be high (10:1) compared to ratio of 2:1 found in normal tissues or in A. Any of the above B.357:2153-65 locally. Blindness D. stage 3 is infiltration alone and stage 4 is fibrotic bands. cardiac arrhythmias or conduction blocks due to C. or lead to fibrosis. giant cells & T cells. Pleural effusion A. 2019 C. > 0. > 1. Hyperviscosity A. Harrison’s 16th Ed. Hypothalamic involvement 676 What measure of lymphadenopathy in CT short axis supports C. > 2. Cardiac arrhythmias Harrison’s 16th Ed. Left ventricular free wall granuloma ? N Engl J Med 2007. Heart blocks A. B cells 673 In sarcoidosis. and diaphragmatic tenting 670 Which of the following observation helps to differentiate neurosarcoidosis from multiple sclerosis ? In sarcoidosis. 2809 bilateral hilar lymphadenopathy with infiltration. D. although granulomatous infiltration of the pulmonary arterioles can cause pulmonary hypertension in the absence of pulmonary fibrosis. Congestive heart failure (CHF) . B.5 cm Optic neuritis occurs in both neurosarcoidosis & multiple sclerosis. Right ventricular free wall A. Active sarcoidosis is also characterized by hyperglobulinemia. Erythema nodosum comprises of bilateral. Cardiac dysfunction B. Fibrotic bands. 2810 D. tender red nodules on anterior surface of legs. bronchoalveolar lavage typically shows an C. Neutrophils 680 Lower than normal levels of ACE is seen in ? D.5 679 Significant elevation of ACE levels is seen in ? D. CD4+T lymphocytes Mild elevation of ACE is seen in diabetes. A.5 is strongly supportive of sarcoidosis. C. Liver 681 Biopsy from which of the following can be obtained for the B. Leprosy Remaining cells are mostly alveolar macrophages. 2810-11 C. 2811 BAL typically shows an increased proportion of activated TH1 subset of CD4+T lymphocytes. Eye & sarcoidosis ? adnexa are involved in 25 to 80% of patients with sarcoidosis. skin lesion. Muscle D.294 MCQ’s FOR MEDICAL PROFESSIONALS BY PROF. A. Two-third is strongly supportive of sarcoidosis ? Harrison’s 18th Ed. Eosinophils are rare. lymphocytes represent <20% of the cell B. Any of the above 687 In Kveim-Siltzbach test. CD4 / CD8 ratio more than which of the following C. Gaucher’s disease 685 In sarcoidosis. Calcium metabolism is disturbed . 24-hour urinary excretion of calcium should be measured in all patients with sarcoidosis. Miliary tuberculosis population. B. Elevations of >50% of upper limit of normal are seen in sarcoidosis. AJAY MATHUR Respiratory 677 Which of the following is least helpful in defining active because sarcoidal macrophages possess 25-hydroxyvitamin D–1alpha-hydroxylase. necessitating routine slit-lamp and Harrison’s 18th Ed. leprosy. 2810 funduscopic examination. 2811 A. Eosinophils Harrison’s 18th Ed. Sarcoidal granulomas produce angiotensin converting enzyme. 2811 D. 2 weeks N Engl J Med 2007. 2810 Positive gallium scan can support the diagnosis of sarcoidosis if increased activity is noted in parotids & lacrimal glands (Panda sign) or in right paratracheal & left hilar area (lambda sign). C. Gallium-67 lung scan with sarcoidosis. 2811 D. papule that develops at the site of injection is biopsied how many weeks later ? Biopsy from lung. Alveolar macrophages such as miliary tuberculosis. hyperthyroidism. Three-fourth A. Extrathoracic lymph node In Kveim-Siltzbach skin test. Lymphoma sarcoidosis extract is from ? ACE level in lymphoma and patients on ACE inhibitor is lower than normal. the intradermal injection of a heat-treated suspension of a sarcoidosis extract is from spleen of a known sarcoidosis patient. CT B.357:2153-65 C. All of the above A. A. Gaucher’s disease. extrathoracic lymph node. ACE level is not elevated in malignancy. 1 week 682 Which of the following tests is least relevant in sarcoidosis ? B.357:2153-65 A.5 Elevated levels of ACE are reported in 60% of patients with acute disease and only 20% of patients B. Hyperthyroidism increased proportion of which of the following ? Harrison’s 16th Ed. Harrison’s 18th Ed. source of intradermal injection D. > 1. 24-hour urinary excretion of calcium D. Lymph node B. > 0. Anterior uveitis is more common than posterior uveitis. 2022 D. Hyperthyroidism 686 In Kveim-Siltzbach skin test. & ACE levels are elevated in 60% of patients A. One-half 684 In BAL fluid. Gallium scan 678 Serum levels of ACE are elevated in approximately what D. hypercalciuria & renal calculi occur in sarcoidosis. The papule that develops at the site of D. In significant fibrosis. Lung diagnosis of sarcoidosis ? Harrison’s 18th Ed. Computed tomography Harrison’s 18th Ed. which converts 25-hydroxyvitamin D to more active vitamin D metabolite. 3 weeks A. BAL chest is rarely helpful for either diagnosis or prognosis but can identify early fibrosis. > 2. Serum angiotensin-converting enzyme injection is biopsied 4 weeks later. Hypercalcemia. PET percentage of patients with acute sarcoidosis ? Harrison’s 18th Ed. 1. 24-hour proteinuria C.5 Harrison’s 18th Ed. It is performed by injecting homogenate of human sarcoid tissue extract intradermally. One-third B. CxR Gallium-67 lung scans. Leprosy B. 2810 In BAL fluid. and funduscopic examinations Kveim-Siltzbach test has been used for the diagnosis of sarcoidosis. tonometric. and disseminated granulomatous infections B. endomyocardium or muscle can be obtained for the diagnosis of sarcoidosis. N Engl J Med 2007. Liver D. In normal individuals.5 with chronic sarcoidosis. 4 weeks B. Slit-lamp. Serum level of ACE A. Bronchoalveolar lavage findings 683 Panda sign and lambda sign relate to ? C. C.25 dihydroxyvitamin D. C. BAL & serum ACE level help in defining sarcoidosis disease activity. CD4/CD8 ratio of > 3. liver. C. > 3. a few neutrophils are also found. Spleen A. tacrolimus. Cyclophosphamide of PE is only 8%. decision making on therapy depends on the extent and activity of the inflammatory process in lung. 10 % C. B.357:2153-65 High-dose bolus intravenous glucocorticoids in sarcoidosis are not as effective as oral therapy.5 points B. First 12 . COPD B. 699 Frequency of occurrence of DVT is how many times more than PE ? Other drugs in refractory cases include indomethacin. brucellosis. followed by a slow taper over 2 to 3 months. levamisole. and bronchogenic carcinoma.5 years C. chloroquine. Deep Venous Thrombosis D. thalidomide. AJAY MATHUR Cardiology 295 688 Chronic sarcoidosis is likely if the acute form does not resolve within ? 694 Sarcoidosis clears spontaneously in about what percentage Harrison’s 18th Ed. 1561 Table 244-1 693 Usually. which of C. 2023 B. Methotrexate and Pulmonary Thromboembolism Hydroxychloroquine is used for hypercalcemia. A.5 points Harrison’s 18th Ed. the likelihood D. 75 % Chronic sarcoidosis is likely if acute form does not resolve within first 2 .5 months Harrison’s 16th Ed. First 6 . Intravenous 691 For the treatment of hypercalcemia in sarcoidosis. A. Lupus pernio 695 Prednisone therapy in sarcoidosis is usually slowly tapered B. 12. tuberculosis. First 2 . 1 to 2 months D. for 4 to 6 weeks. 2811 A. Inhaled N Engl J Med 2007. 15. Hydroxychloroquine D. infliximab. Bone cysts over how many months ? Harrison’s 16th Ed. cardiac or neurologic disease & presence of renal calculi due to hypercalciuria. Harrison’s 18th Ed. Cardiac sarcoidosis A.5 years. oxyphenbutazone. 2812 B. Cutaneous sarcoidosis which of the following routes of administration ? Harrison’s 16th Ed. 1651 allopurinol. B. 3 to 4 months D. Hydroxychloroquine C. azothioprine. First 2 . Pulmonary sarcoidosis B. pentoxifylline. skin disease & neurologic involvement. ILD A.5 points C. 1 mg/kg. 2023 A. 25 % D. Oral Dosage of glucocorticoids is higher for neurosarcoidosis & lower for cutaneous disease. 295 Respiratory MCQ’s FOR MCQ’s FOR MEDICAL MEDICAL PROFESSIONALS PROFESSIONALS BY PROF. 2023 C. p-aminobenzoate. Sarcoidosis B. heart. eye. Intramuscular the following drugs is effective ? D. etanercept. Neurosarcoidosis 696 Glucocorticoids in sarcoidosis are most effective through C. 4 to 6 months 690 Dosage of glucocorticoids is higher for ? Harrison’s 18th Ed. Azothioprine The Wells Scoring System has a maximum of 12. Sarcoidosis clears spontaneously in ~50% of patients. Brucellosis D. bone cysts. If the score is <= 4 points. coccidioidomycosis. lymphoma. Bilateral hilar adenopathy is seen in sarcoidosis. and CNS. after glucocorticoids. and cyclophosphamide. 2811 of patients ? A. As permanent organ derangements often do 689 Risk factor at presentation for chronic sarcoidosis is ? not improve with glucocorticoid treatment. which of the following is the most preferred drug in the treatment of sarcoidosis ? A. Pulmonary embolism C.12 months A. Harrison’s 17th Ed. 2812 Usual therapy for sarcoidosis is prednisone. Methotrexate C. C. 2 to 3 months Chronic sarcoidosis can be identified by risk factors at presentation like fibrosis on CxR. hydroxychloroquine. 50 % D.5 points A. Lymphoma C. Cystic fibrosis D. Azathioprine There is no evidence that inhaled glucocorticoids are efficacious. 8. Therapy of choice for sarcoidosis is glucocorticoids. 10. All of the above Wells Diagnostic Scoring System is a semi-quantitative clinical scoring system for suspected pulmonary embolism (PE). 697 Wells diagnostic scoring system is used for ? Harrison’s 16th Ed. A. 1561 692 Bilateral hilar adenopathy is seen in ? Harrison’s 16th Ed. lupus pernio. Renal calculi due to hypercalciuria A. All of the above B. Azathioprine is recommended in sarcoidosis with intracerebral involvement. Thalidomide & Methotrexate are recommended in Lupus pernio. Methotrexate is the second-line medication. Thalidomide Chapter 262. 2023 D.18 months B. 698 Wells Scoring System has a maximum of ? Harrison’s 16th Ed.5 points. 2 . Hemoptysis Autosomal dominant genetic mutations that contribute to the likelihood of VTE are factor V Leiden and the prothrombin gene mutations. 704 In a case of pulmonary embolism. DVT occurs about 3 times more often than PE. 2171 A. Impaired gas exchange B. Second or third trimester of pregnancy D. increased airway resistance & decreased pulmonary compliance. Dyspnea is the most frequent symptom of PE. surgery. Pleuritic pain. Systemic arterial hypertension D. All of the above D. 2170 A. Isolated calf vein thrombi C. alveolar hyperventilation. ~25% pregnancy with DVT carry factor V Leiden allele D. AJAY MATHUR Respiratory B. 2172 Harrison’s 18th Ed. postoperative exchange. 2172 A. 706 Eponym “The Great Masquerader” is used for ? 700 Which of the following is false about postphlebitic syndrome ? Harrison’s 18th Ed. T wave inversion in V1-4 A. without PE. trauma. Cough B. Factor V Leiden mutation A. hypotension or cyanosis indicates a massive PE. All of the above Effects of pulmonary embolism are : Increased pulmonary vascular resistance. syncope. pregnancy. and tachypnea is its most frequent sign. It is also known as postthrombotic syndrome or chronic venous insufficiency. Methylenetetrahydrofolate reductase C677T mutation C. 2172 702 Which of the following about pregnancy & DVT is false ? A. Increased airway resistance D.296 MCQ’s FOR MEDICAL PROFESSIONALS BY PROF. Alveolar hypoventilation C. Severe thigh swelling Harrison’s 18th Ed. Pleuritic pain C. Caused by permanent damage to venous valves of leg C. Pleuritic pain B. New onset atrial fibrillation B. Harrison’s 18th Ed. state. Dyspnea D. Dyspnea A. oral contraceptives. Any of the above D. Increased pulmonary vascular resistance A. Pelvic vein thrombosis B. obesity. Proximal leg deep venous thrombosis . cancer. Myocardial infarction B. 705 Which of the following is the most common source of 711 ECG change in pulmonary embolism include ? paradoxical embolism ? Harrison’s 18th Ed. It is a late adverse effect of DVT. postmenopausal hormone replacement. cough or hemoptysis suggests a small embolism located distally near the pleura. Isolated calf vein thrombi pose a lower risk of Venous thromboembolism (VTE) includes deep venous thrombosis (DVT) & pulmonary embolism PE but they are the most common source of paradoxical embolism. 2170 A. DVT occurs in ~1 in 2000 pregnancies B. All of the above edematous. Antiphospholipid antibody syndrome C. 2172 A. DVT is unlikely. Diffusely edematous leg B. systemic arterial hypertension & COPD. Severe thigh swelling & marked tenderness in inguinal area & common femoral vein occurs in massive DVT. If leg is diffusely D. Sudden. Skin ulceration on medial malleolus of leg D. cancer. and second or third trimester of pregnancy. cigarette smoking. 707 Which of the following is likely in massive DVT ? Harrison’s 18th Ed. severe calf discomfort suggests a ruptured Baker’s cyst. is postphlebitic syndrome. impaired gas Levels of D-dimer increase in myocardial infarction. 3 C. Venous thromboembolism (VTE) C. No effective medical therapy Apart from PE. Any of the above C. (PE). 5 50% of patients with pelvic vein thrombosis or proximal leg deep venous thrombosis (DVT) have asymptomatic pulmonary thromboembolism (PTE). severe calf discomfort likelihood of VTE ? B. Chronic obstructive pulmonary disease Sudden. Acquired predispositions to VTE include long-haul air travel. Medical 708 Which of the following suggests a small pulmonary embolism ? conditions include APLA syndrome. Hypotension D. Dyspnea. Sepsis C. Early adverse effect of DVT B. pneumonia. 2171 Harrison’s 18th Ed. which occurs in more than half of DVT cases. 2171 701 Which of the following medical conditions contribute to the A. All of the above D. None of the above 709 Which of the following suggests a massive pulmonary 703 Genetic mutations associated with DVT in pregnancy include ? embolism ? Harrison’s 18th Ed. term masquerader has been used for acute appendicitis & pheochromocytoma. which of the following is false ? 710 Levels of D-dimer increase in ? Harrison’s 18th Ed. Pulmonary embolism (PE) B. Major adverse outcome of DVT alone. Prothrombin G20210A mutation B. sepsis. 2171 Harrison’s 18th Ed. Syncope C. Deep venous thrombosis (DVT) A. DVT more common in left than right leg C. rather acute exacerbation of venous insufficiency due to postphlebitic syndrome is likely. 4 D. 2173 A. Normal venous ultrasound does not exclude PE C. 2173 A. AJAY MATHUR Cardiology 297 C. 2174 B. Although. S1Q3T3 pattern C. Peripheral wedged-shaped density above diaphragm C. a Q wave in lead III & an inverted T wave in lead III. New-onset atrial fibrillation Echocardiography is not a reliable diagnostic imaging tool for acute PE because most patients with PE have normal echocardiograms. Focal oligemia 721 Which of the following is false ? Harrison’s 18th Ed. Peripheral wedged-shaped density above diaphragm C. invasive catheter-based diagnostic testing is reserved A. Hampton’s hump right ventricular apical motion and is specific for PE. Four or more A. All of the above A. B. Enlarged right descending pulmonary artery 722 Definitive diagnostic test for pulmonary embolism is ? Harrison’s 16th Ed. Chest x-ray B. Normal or nearly normal chest x-ray often occurs in PE D. A. C. None of the above Harrison’s 18th Ed. 2173 D. Enlarged left descending pulmonary artery B. or an enlarged right descending pulmonary artery (Palla’s sign). Acute myocardial infarction Classic ECG abnormalities in PE include sinus tachycardia. new-onset atrial fibrillation or flutter & an S wave in lead I. Enlarged left descending pulmonary artery A. Multidetector-row spiral CT scanners can image small peripheral emboli. Echocardiography is not reliable diagnostic tool for acute PE B. Peripheral wedged-shaped density above diaphragm C. Infective endocarditis A. B. 2172 D. Focal oligemia PE is unlikely in normal/nearly normal lung scan. Aortic dissection A. S1Q3T3 pattern 718 Echocardiography is the least useful diagnostic tool for ? Harrison’s 17th Ed. Chest CT C. 1654 D. McConnell’s sign is specific for which of D. One or more for patients with technically unsatisfactory chest CTs and those in whom an interventional procedure such as catheter-directed thrombolysis or embolectomy is planned. B. T-wave inversion in leads V 1 to V 4 the following ? Harrison’s 18th Ed. 297 Respiratory MCQ’s FOR MCQ’s FOR MEDICAL MEDICAL PROFESSIONALS PROFESSIONALS BY PROF. CT Chest 717 Pulmonary perfusion scan that has a high probability for PE D. Focal oligemia B. 2173 A. HOCM 713 Which of the following is a feature of pulmonary embolism on B. Lung scanning is the second-line diagnostic test for PE 716 Hampton’s hump refers to ? D. MR Angiography (Hampton’s hump). 2173 714 Palla’s sign refers to ? Harrison’s 18th Ed. Enlarged right descending pulmonary artery D. Pulmonary embolism C. Enlarged right descending pulmonary artery A. Pulmonary embolism chest X-Ray ? C. T-wave inversion in leads V1 to V4 is the most frequent change (right ventricular strain). 2174 D. Three or more 723 Primary therapy of PE is ? Harrison’s 18th Ed. All of the above diagnosis of PE ? Harrison’s 18th Ed. Acute rheumatic fever Harrison’s 18th Ed. Other abnormalities include focal oligemia (Westermark’s sign). a normal or near-normal chest x-ray suggests PE. Sixth-order branches can be visualized with resolution superior to that of conventional invasive contrast pulmonary 715 Westermark’s sign refers to ? angiography. Anticoagulation with warfarin C. 2173 pulmonary angiography as a diagnostic test. QRS axis is >90°. chest CT with contrast has replaced invasive Harrison’s 18th Ed. Harrison’s 18th Ed. 2174 In PE. QRS axis greater than 90º 719 In echocadiography. Thrombolysis . Two or more C. C. V/Q scan In a dyspneic patient. Echocardiography D. Pericardial tamponade 712 Most frequent ECG finding in pulmonary embolism is ? Harrison’s 18th Ed. Westermark’s sign Echocardiographic McConnell’s sign refers to right ventricular free wall hypokinesis with normal B. Enlarged left descending pulmonary artery Computed tomography of chest with intravenous contrast is the principal imaging test for diagnosis of PE. Anticoagulation with heparin A high probability pulmonary perfusion scan for PE is defined as having two or more segmental perfusion defects in the presence of normal ventilation scan. B. Selective pulmonary angiography should have how many segmental perfusion defects in the Selective pulmonary angiography demonstrating intraluminal filling defect in more than one projection presence of normal ventilation scan ? is the most specific examination available for establishing the definitive diagnosis of PE and can detect emboli as small as 1 to 2 mm. a peripheral wedged-shaped density above the diaphragm B. Palla’s sign 720 Which of the following is the principal imaging test for the D. Lung Scanning A. 1563 D. 2176 B. 7. “bridging” with a in PE is ? parenteral anticoagulant is not required. All of the above Harrison’s 18th Ed. Harrison’s 18th Ed. Anticoagulation with heparin and warfarin or placement of an inferior vena caval filter constitutes A. 10 mg. 2174 Direct thrombin inhibitors are argatroban. Warfarin D. B. D. or elevation of the troponin level due to A. immediately effective anticoagulation is initiated with a parenteral drug . 2176 B. LMWH (Enoxaparin. Increased levels of D-dimer A. C. 2 to 3 days VII. or fondaparinux. 2175 D. S1Q3T3 pattern in ECG 731 Warfarin acts by preventing carboxylation activation of which C. or bivalirudin and should be used in patients with A. 2175 A. which of the following is the most widely used approach D.5 mg of Fondaparinux is ised to prevent VTE. which of the following identify high. Vitamin K D. Argatroban secondary prevention of recurrent PE. 80 U / kg A. 120 U / kg C. Right ventricular dysfunction 730 What is the dose of Fondaparinux in patients weighing between C. 2175 Primary therapy consists of clot dissolution with thrombolysis or removal of PE by embolectomy. C. hour. B.3 times the upper limit of laboratory normal. sex. Dabigatran is a direct thrombin inhibitor. Recombinant factor VIIa UFH is dosed to achieve a target activated partial thromboplastin time (aPTT) that is 2 . II In PE. 2174 Patients weighing <50 kg receive 5 mg. the parenteral and oral B. 2175 Hemodynamic instability. C. Concomitant drugs 727 In massive pulmonary embolism.UFH. Bivalirudin risk patients ? D.5 mg right ventricular microinfarction identify high-risk PE. 5 to 6 days A. Drug-drug and drug-food interactions Warfarin requires 5-7 days to achieve a therapeutic effect. followed by an initial infusion rate of 18/kg per There is no specific antidote for bleeding from fondaparinux or direct thrombin inhibitors. 100 U / kg B. and X. A. 5 mg on chest CT. 2175 733 “Bridging” with a parenteral anticoagulant is required when A. Elevation of troponin level B. a typical initial intravenous bolus is 80 U/kg. 2. 60 U / kg 734 Antidote for bleeding from fondaparinux is ? Harrison’s 18th Ed. right ventricular dysfunction. B. VII 726 Heparin should be overlapped with oral anticoagulation for at C. 2175 D. weight agents are overlapped. and dabigatran is a direct thrombin inhibitor. During that period. Harrison’s 18th Ed. All of the above anticoagulation is initiated with ? Harrison’s 18th Ed. X least how many days ? D. IX. Low molecular weight heparin (LMWH) A.100 kg ? D. 2175 Warfarin is a vitamin K antagonist that prevents carboxylation activation of coagulation factors II. Dabigatran C. Age. 50-100 kg patients receive 7. Rivaroxaban In massive PE. Hemodynamic instability proven or suspected heparin-induced thrombocytopenia. B.5 mg 725 In PE. All of the above Rivaroxaban is a factor Xa inhibitor. Protamine sulfate C. . A. Detection of RV hypokinesis by echocardiography coagulation factor ? Harrison’s 18th Ed. Lepirudin 724 In pulmonary embolism. 2. to risk stratification ? Fondaparinux is an anti-Xa pentasaccharide. 4 to 5 days Harrison’s 18th Ed. immediately effective D. Fondaparinux B. None of the above UFH. All of the above Harrison’s 18th Ed. All of the above Harrison’s 18th Ed. AJAY MATHUR Respiratory D.298 MCQ’s FOR MEDICAL PROFESSIONALS BY PROF. Elevation of troponin level 50 . It is administered by once-daily subcutaneous injection. Unfractionated heparin (UFH) which of the following is used ? Harrison’s 18th Ed. For D. Placement of inferior vena caval filter 729 Which of the following is a direct thrombin inhibitor ? Harrison’s 18th Ed. Because of these 728 A typical initial intravenous bolus of unfractionated heparin drugs’ rapid onset of action & relatively short half-life compared with warfarin. 3 to 4 days 732 Which of the following affect warfarin metabolism ? C. lepirudin.5 mg. Any of the above C. This is usually equivalent to an aPTT of 60 .80 seconds. detection of RV hypokinesis on echocardiography is most widely used for risk stratification. RV enlargement on chest CT indicates a 5x increased likelihood of death within next 30 days compared with PE patients with normal RV size B. and patients weighing >100 kg receive 10 mg. Tinzaparin). 2176 D. 3 days 739 Duration of anticoagulation for PE following surgery or trauma is ? C. Osteoporosis A. B. 2176 Preferred fibrinolytic regimen is 100 mg of tPA administered as a continuous peripheral intravenous A. B. B. Plasma D-dimer ELISA assay has high (-) predictive value Duration of anticoagulation for PE with moderate or high levels of anticardiolipin antibodies is indefinite. Twelfth to sixteen weeks of gestation 744 The preferred fibrinolytic regimen in PE is ? Harrison’s 18th Ed. 3 . Indefinite Harrison’s 18th Ed. Urokinase 738 Warfarin can be administered safely during ? D. Patients with idiopathic VTE B. Twenty to twenty four weeks of gestation A. 6 . 14 days A. Breast feeding after the PE has occurred ? D. 1562 D. Skin necrosis 743 Successful fibrinolytic therapy in PE leads to ? C. B. Vasopressin 736 Most common nonbleeding side effect of warfarin is ? Harrison’s 17th Ed. 7 days Harrison’s 18th Ed. 3 .18 months 747 All are true for pulmonary embolism except ? Harrison’s 16th Ed. All of the above D. Alteplase Harrison’s 18th Ed. 2176 A. Sixth to twelfth weeks of gestation C. 2176 D. B. protamine sulfate can be Harrison’s 18th Ed.6 months. Vascular compression stockings B. Recombinant tissue plasminogen activator (tPA) Warfarin embryopathy is most common with exposure during 6th to 12th weeks of gestation.18 months 746 Which of the following prevents postphlebitic syndrome ? D.40 mmHg vascular compression stockings.12 months C. fresh-frozen plasma can be used. For minor bleeding or to manage an excessively high INR in the absence of bleeding. Streptokinase In the second and third trimesters. Prevention of recurrent PE 737 Warfarin embryopathy occurs with warfarin exposure during ? Harrison’s 18th Ed. 2176 administered. All of the above A. Lowers rate of death C. Vitamin K D.6 months PE patients respond to fibrinolysis for up to 14 days after the PE has occurred. With non-life threatening bleeding. Patients with high levels of anticardiolipin antibodies A. 2176 D. 12 . Fresh-frozen plasma B. A. AJAY MATHUR Cardiology 299 735 Catastrophic bleeding associated with warfarin administration 741 Indefinite-duration anticoagulation is recommended for ? is best treated by ? Harrison’s 18th Ed.12 months C. Recombinant human coagulation A. 2176 D. B. All of the above 742 First-line inotropic agent for treatment of PE-related shock is ? For life-threatening or intracranial hemorrhage due to heparin or LMWH. Warfarin of anticardiolipin antibodies is ? Harrison’s 18th Ed. even if initial VTE was provoked by trauma or surgery. 299 Respiratory MCQ’s FOR MCQ’s FOR MEDICAL MEDICAL PROFESSIONALS PROFESSIONALS BY PROF. B. Warfarin-induced skin necrosis is rare. Clopidogrel A. Norepinephrine A. A. 2177 Duration of anticoagulation for PE following surgery or trauma is 3 . warfarin may cause fetal optic atrophy and mental retardation. 12 . Major bleeding from warfarin is best managed with prothrombin complex concentrate. Recombinant factor VIIa therapy (rFVIIa) C. Indefinite A. 1656 D. Aspirin 740 Duration of anticoagulation for PE with moderate or high levels C. B. Warfarin therapy is contraindicated in first trimester due to its association with fetal chondrodysplasia punctata. 6 . B. Second to sixth weeks of gestation Successful fibrinolytic therapy in PE rapidly reverses right heart failure and leads to a lower rate of death and recurrent PE. Postpartum period 745 PE patients respond to fibrinolysis for up to how many days C. C. All of the above Harrison’s 18th Ed. 2176 Warfarin is safe during second trimester. Dopamine / dobutamine factor VIIa (rFVIIa) is used to manage catastrophic bleeding from warfarin.6 months Only therapy to prevent postphlebitic syndrome is daily use of below-knee 30 . ABG lacks diagnostic utility . postpartum and breast feeding period. 1 day B. Alopecia Dopamine and dobutamine are first-line inotropic agents for treatment of PE-related shock. Rapidly reversal of right heart failure Most common nonbleeding side effect of warfarin is alopecia. Seizure Harrison’s 18th Ed. 2176 D. oral vitamin K may be administered. Phenylephrine C. Patients with cancer and VTE C. 2176 Harrison’s 18th Ed. Second trimester of pregnancy infusion over 2 hours. fluid is removed via lymphatics situated in D. Bacterial pneumonia effusions). Garland triangle (small area of resonance next to spine in patients with large unilateral pleural B. fluid enters pleural space from capillaries in 758 Parapneumonic effusions are associated with ? parietal pleura Harrison’s 18th Ed. 5 to 10 mL 754 Which of the following is true for transudative pleural B. A. 2178 A. 2178 C. 500 . exudative pleural effusions meet at least one. by physical examination ? Cleveland Clinic Journal Of Medicine 2008.500 mL A. 2178 and Mediastinum A. Pleural fluid protein / serum protein >0. Plasma D-dimer ELISA has a high negative predictive value (99. None of the above Cleveland Clinic Journal Of Medicine 2008. 50 to 100 mL A. Pleural fluid protein/serum protein >0. 25 to 50 mL effusions ? Harrison’s 18th Ed. D-dimer assay D.75:303 757 Which of the following is false for ‘Hepatic Hydrothorax’ ? A. Normally. 2179 D. Disorders of the Pleura Harrison’s 18th Ed. Westermark’s sign in chest X-Ray is focal oligemia B. than is normally formed in pleural spaces Arterial blood gases lack diagnostic utility for PE. 2179 A. pleural space contains how much of pleural C. 500 mL holes in diaphragm into pleural space At least 500 mL of fluid must be present to be detected an effusion by physical examination. 350 mL B. Pleural fluid sodium D. Garland triangle 755 Most common cause of pleural effusion is ? Harrison’s 18th Ed. Lymphatics have a capacity to absorb 20 times more fluid than is normally formed.1 g/dL.6 In healthy adults. Effusion is usually left-sided D.4%. C. C.5. Pleural fluid LDH / serum LDH >0. > 1. Pleural fluid N-terminal pro-brain natriuretic peptide with significant asymmetry of chest expansion. Left ventricular failure angle triangle of dullness found over posterior region of chest opposite a large pleural effusion).6%) and can be used to help exclude PE. whereas A.500 mL 756 Which of the following test is virtually diagnostic of pleural B. pleural fluid LDH/ serum LDH >0. Still.1000 mL Harrison’s 18th Ed. pleural space contains ~ 5 to 10 mL of pleural fluid (0. Bacterial pneumonia .75:303 A. pleural effusion is transudative. Palla’s sign is enlarged (L) descending pulmonary artery parietal pleura Quantitative ELISA plasma D-dimer level is elevated (>500 ng/mL) in more than 90% of patients with C. Viral infection of pleural effusion is ? Most common cause of pleural effusion is LVF. Isolated right-sided pleural effusions are more Cleveland Clinic Journal Of Medicine 2008. Grocco triangle (right. 750 .Out of the following. AJAY MATHUR Respiratory C. Lymphatics have the capacity to absorb 2 times more fluid is not specific and levels increase in patients with MI. It has a sensitivity of 96. Fluid can also enter pleural space from peritoneal cavity PE. Pleural fluid protein / serum protein >0. 100 to 150 mL B. if B. Pleural fluid LDH Physical findings are normal if <300 mL of fluid is present. 100 mL Harrison’s 18th Ed. Pleural fluid LDH / serum LDH >0. C.75:303 common than left-sided effusions in heart failure.300 MCQ’s FOR MEDICAL PROFESSIONALS BY PROF. sepsis etc. 300 . All of the above Cleveland Clinic Journal Of Medicine 2008. 150 mL A. 2179 B. Calots triangle clinically transudative effusion is strongly suspected. Pleural fluid LDH > two-thirds normal upper limit for serum 749 Which of the following is not related to pleural effusion ? D. Skodaic resonance transudative pleural effusions meet none. difference between albumin levels in serum and pleural fluid is estimated. contralateral tracheal or mediastinal shift and bulging of intercostal spaces. Pleural fluid LDH > two-thirds normal upper limit for serum fluid ? D.5 B.1 mg/kg body weight). 753 Which of the following is true for exudative pleural effusions ? Chapter 263. Grocco triangle D. Due to direct movement of peritoneal fluid through small D. C. Large effusions (>1500 mL) is associated B.6 748 In healthy adults. 2178 Skodaic resonance (area of hyperresonance above a pleural effusion). Occur in ~5% of patients with cirrhosis & ascites C. Harrison’s 18th Ed. Succussion splash (splashing sound produced by violently shaking patients with hydropneumothorax). If this difference is >3.75:303 Light’s criteria .5 D. Cirrhosis liver 750 Contralateral tracheal or mediastinal shift occurs when the size D.750 mL effusion secondary to congestive heart failure ? C.6 and pleural fluid LDH more than two-thirds normal upper limit for serum. Transjugular intrahepatic portal systemic shunt helpful 752 Which of the following statements is false ? Hepatic Hydrothorax is usually “right-sided”. Pleural fluid osmolality 751 What quantity of pleural fluid must be present to be detected Pleural fluid NT-proBNP >1500 pg/mL is virtually diagnostic of an effusion secondary to CHF. A. C. reflecting plasmin’s breakdown of fibrin and indicating endogenous thrombolysis. Normally. AIDS lung carcinoma. Viral infection C. 763 Which out of the following is the commonest cause of 769 The diagnosis most commonly overlooked in differential malignant pleural effusion ? diagnosis of an undiagnosed pleural effusion is ? Harrison’s 18th Ed. Pulmonary embolism B. Bronchiectasis A. Hepatoma B. 2179 false ? A. and a shrunken hemithorax.20. 764 The only symptom that can be attributed to the malignant pleural effusion itself is ? . (+) Gram stain or thickening. brisk leukocytosis. Left ventricular failure C. 2179 A. > 5 mm Harrison’s 18th Ed. 2179 Harrison’s 18th Ed. generalized pleural loculated pleural fluid. None of the above mild anemia. Thyroid D. Opiates B. gross pus in pleural space 768 Shortness of breath in mesothelioma should be treated with ? 762 Which out of the following is the commonest cause of Harrison’s 18th Ed. Most malignancies associated with PE are not curable Patients with “anaerobic infections” present with a subacute illness. 2179 B. Hemoptysis Only symptom that can be attributed to pleural effusion itself is dyspnea. 2179 A. > 15 mm B. Indicates disseminated disease D. culture. Lymphoma Shortness of breath in mesothelioma is treated with oxygen and/or opiates. Cough Parapneumonic effusions are associated with bacterial pneumonia. Pleural fluid glucose <60 mg/dL C. breast carcinoma & lymphoma are the cause of ~75% of all malignant pleural effusions. Hepatoma A. Ca. pleural fluid pH < 7. weight loss. > 20 mm C. Shrunken hemithorax D. weight loss. Mesotheliomas C. pleural fluid glucose <60 mg/dL. 761 Factors indicating the likely need for a procedure more invasive than a thoracentesis are all except ? 767 Chest radiograph of patients with mesothelioma reveals ? Harrison’s 18th Ed. Dyspnea D. or bronchiectasis. Bacterial infection Harrison’s 18th Ed. AJAY MATHUR Cardiology 301 B. a therapeutic thoracentesis should be performed. Lung abscess Harrison’s 18th Ed. Malignancy D. Glucocorticoids C. 2179 A. lung abscess. Generalized pleural thickening C. > 10 mm A. Benzene D. Silica C. Diagnosis most commonly overlooked in differential diagnosis of an undiagnosed pleural effusion is pulmonary embolism. separates lung from chest wall by ? Harrison’s 18th Ed. Bronchodilators Harrison’s 18th Ed. Ca. D. 301 Respiratory MCQ’s FOR MCQ’s FOR MEDICAL MEDICAL PROFESSIONALS PROFESSIONALS BY PROF. Dyspnea is frequently out 759 Which of the following is the likely pathogen in pneumonia of proportion to the size of pleural effusion. Carbon If pleural free fluid separates lung from the chest wall by >10 mm. ACE inhibitors D. Grossly purulent effusion is termed Empyema. 2179 766 Most malignant mesotheliomas are related to exposure to ? A. presenting with a subacute illness. 2179 Harrison’s 18th Ed. & predisposition for aspiration. Anaerobic infection A. Malignant mesotheliomas are primary tumors that arise from the mesothelial cells that line the pleural cavities. Patients with malignant pleural effusion are treated symptomatically as presence of effusion indicates disseminated disease & most malignancies associated with pleural effusion are not curable with 760 Therapeutic thoracentesis is indicated if pleural free fluid chemotherapy. Breast D. Pleural effusion B. D. Leukemia B. Loculated pleural fluid A. Pain C. brisk leukocytosis and possibility of aspiration ? 765 Which of the following about malignant pleural effusion is Harrison’s 18th Ed.40 B. To be treated symptomatically C. Most are related to asbestos exposure. Pancreas C. 2179 B. All of the above Factors indicating the likely need for a procedure more invasive than a thoracentesis include Chest radiograph in patients with mesothelioma reveals a pleural effusion. 2179 malignant pleural effusion ? A. 2179 C. Pleural fluid pH < 7. Positive Gram stain or culture of pleural fluid D. Fungal infection B. All of the above B. Asbestos D. Ca. Hematogenous spread of tubercular bacilli 778 The treatment of choice for most chylothoraces is ? B. HDL < 40 mg/dL If pleural effusion secondary to PE increases in size after anticoagulation. Contiguous spread of tubercular bacilli Harrison’s 18th Ed. Ligation of thoracic duct Treatment of choice for most chylothoraces is insertion of a chest tube plus administration of 773 Which of the following is false about tuberculous pleural octreotide. Dyspnea 776 The most common cause of chylothorax is ? B. Recommended treatment of pleural & pulmonary C. Mesothelioma C. 2180 B. Thoracotomy C. The most common cause of chylothorax is trauma. Total cholesterol > 300 mg/dL 772 Tuberculous pleural effusions are due to ? In chylothorax. 1523 A. Pleural effusions are uncommon B. Pleural infection A. the Chylothorax occurs when thoracic duct is disrupted and chyle accumulates in pleural space. interferon  is > 140 pg/mL. All of the above B. and primary effusion lymphoma. TB markers in pleural fluid like ADA is > 40 IU/L. Developmental anomaly D. > 50 % tuberculosis is not identical D. Necrotizing infection 771 In pleural effusion secondary to pulmonary embolization. Tuberculosis D. cryptococcosis. patient has a hemothorax. LDL > 120 mg/dL D. Malignancy D. Trauma A. Cough A. Metastasis to pleura Most hemothoraces are due to trauma. recommended treatment of pleural and pulmonary tuberculosis is “identical”. 2179 777 Which of the following is characteristic of pleural fluid A. Trauma Dyspnea is the most common symptom in with pleural effusion due to PE. 1659 A. 2180 Harrison’s 17th Ed. Pleural effusions Harrison’s 18th Ed. All of the above C.80 C. a pleuroperitoneal shunt is placed unless patient has chylous ascites. Recurrent emboli biochemical analysis in a case of chylothorax ? B. Triglyceride > 110 mg/dL C. if pleural effusion increases in size after anticoagulation. D. Tube thoracostomy with chest tube drainage space B. Contains predominantly small lymphocytes how much of the peripheral blood hematocrit ? Harrison’s 18th Ed. Pleurodesis B. Tuberculosis B. A. Most common cause is Kaposi’s sarcoma C. D. Tumor C. > 25 % C. Tube thoracostomy with chest tube drainage A. The If hematocrit of bloody pleural fluid is >50% that of the peripheral blood. 2179 779 Hemothorax is diagnosed when hematocrit of pleural fluid is A. Hemothorax Harrison’s 18th Ed. 780 Most hemothoraces are the result of ? 774 Pleural effusion that produces no shift of mediastinal Harrison’s 18th Ed.302 MCQ’s FOR MEDICAL PROFESSIONALS BY PROF. If these fail. thoracentesis reveals milky fluid and biochemical analysis reveals a triglyceride level of > 110 mg/dL. or positive PCR for tuberculous DNA. AJAY MATHUR Respiratory 770 Which is the most common symptom in cases of pleural effusion Pleural effusions are uncommon in AIDS patients. C. Interferon  level >140 pg/mL B. Harrison’s 18th Ed. possibility is ? Harrison’s 18th Ed. secondary to pulmonary embolization ? Other common causes are TB. probability is of recurrent emboli or a hemothorax or a pleural infection. Other causes include rupture of a blood vessel or tumor. Pleurodesis D. carinii infection D. Blood transfusion D. Rupture of a blood vessel B. Hemoptysis B. > 75 % Tuberculous pleural fluid is an exudate with predominantly small lymphocytes. Pulmonary adenocarcinoma D. effusion ? Harrison’s 18th Ed. 775 Which of the following is false about AIDS & pleural effusions 781 The treatment of choice for most hemothorax is ? ? Harrison’s 18th Ed. 2180 C. 2180 C. None of the above . 2180 structures to opposite side is suggestive of ? Harrison’s 16th Ed. 2179 . Adenosine deaminase level > 40 IU/L A. The most common cause is Kaposi’s sarcoma. Pain Harrison’s 18th Ed. Implantation of a pleuroperitoneal shunt Tuberculous pleural effusions are due to a hypersensitivity reaction to tuberculous protein in pleural space. > 33 % D. Pleural effusions very uncommon with P. Hypersensitivity reaction to tuberculous protein in pleural A. 2179 A. 2179 are very uncommon with Pneumocystis carinii infection. Secondary spontaneous pneumothorax occurs in presence of underlying lung disease D. All of the above C. Approximately one-half of patients with an initial primary spontaneous pneumothorax will have a recurrence. Recurrent pneumothorax A. Table 263-1 trauma to thorax A. Left-sided A. Bromocriptine. 785 Pleural effusions that occur following coronary artery bypass surgery within the first weeks are which sided ? 791 Which of the following is false about primary spontaneous Harrison’s 18th Ed. Nitrofurantoin C. 2181 C. 303 Respiratory MCQ’s FOR MCQ’s FOR MEDICAL MEDICAL PROFESSIONALS PROFESSIONALS BY PROF.81 pneumothorax ? A. 2180. which allows continuous 788 In “Pulmonary Lymphangiomyomatosis (LAM)”. endoscopic variceal C. Stapling of blebs Medical manipulations that induce pleural effusions include abdominal surgery. thoracotomy is considered. liver or lung transplantation. During mechanical ventilation or resuscitative efforts D. 50 % of patients with an initial primary spontaneous Pleural effusions commonly occur within weeks following CABG surgery are left-sided & bloody. Right-sided Harrison’s 18th Ed. Tamoxifen B. Bilateral B. Cephalosporins B. Monocyte D. All of the above lines. 2180 . 2181 B. AJAY MATHUR Cardiology 303 Most patients with hemothorax are treated with tube thoracostomy. All of the above B. Methysergide Harrison’s 18th Ed. Asthma 793 Secondary spontaneous pneumothoraces are most frequently B. pressure in pleural space is positive throughout the respiratory cycle. Dantrolene D. Chylous pleural effusion B. Proliferation of surfactant producing alveolar cells B. Harrison’s 16th Ed. Dantrolene. Oophorectomy Harrison’s 18th Ed. Neutrophil C. All of the above C. Intra-vascular insertion of central lines A. Occur almost exclusively in smokers D. Methysergide. 1559 783 Drug-induced pleural disease can be due to all except ? A. Bromocriptine absence of underlying lung disease C. pressure in pleural space is Drug-induced Exudative pleural disease can be due to Nitrofurantoin. Lymphocyte D. with pneumothorax will have a recurrence large numbers of eosinophils & respond to therapeutic thoracenteses. They occur almost exclusively in smokers. Radiation therapy Harrison’s 18th Ed. Emphysema found in ? Harrison’s 18th Ed. 1559 A. If pleural hemorrhage exceeds 200 mL/hour. If the 787 All of the following are features of “Pulmonary lung does not expand or if patient has recurrent pneumothorax. pathological lesion is ? Harrison’s 16th Ed. Usually due to rupture of apical pleural blebs C. Table 263-1 B. Simple aspiration D. Harrison’s 16th Ed. Proliferation of atypical pulmonary interstitial smooth muscle A. radiation therapy. Proliferation of lung lymphatic vessels Harrison’s 18th Ed. Primary spontaneous pneumothoraces are due to rupture of apical pleural blebs that lie within or 786 Which of the following medical manipulations can cause pleural immediately under the visceral pleura. Amiodarone D. None of the above few weeks are left-sided & clear yellow with more of small lymphocytes & tend to recur. Endoscopic variceal sclerotherapy pneumothorax surely prevents recurrences ? B. 2180. Pleural abrasion sclerotherapy. Procarbazine. effusion ? Harrison’s 18th Ed. Primary spontaneous pneumothorax occurs in the B. Eosinophil 789 Which of the following treatment modalities have a beneficial role in “Pulmonary Lymphangiomyomatosis (LAM)” ? Several drugs can cause pleural effusion. Progesterone A. thoracoscopy with stapling of blebs and pleural abrasion is indicated. 2180. Spontaneous pneumothorax occurs without antecedent Harrison’s 18th Ed. Procarbazine C. Asthma . Chloroquine 790 Which of the following about pneumothorax is false ? D. characteristic quantification of bleeding. Table 263-1 792 Which of the following treatments for primary spontaneous A. Any of the above C. or intravascular insertion of central D. 2181 B. Initial recommended treatment for primary spontaneous pneumothorax is simple aspiration. Associated fluid is usually eosinophilic. 2181 C. positive at the end of inspiration In tension pneumothorax. Thoracoscopy or thoracotomy with pleural abrasion is ~100% Lymphangiomyomatosis (LAM)” except ? successful in preventing recurrences. 2181 784 Drug-induced pleural disease can be due to all except ? A. Effusions occurring after first D. In tension pneumothorax. 1559 782 Which WBC predominates in drug-induced pleural effusion ? A. Amiodarone. 2181 Harrison’s 18th Ed. Chronic mediastinitis D. 2181 A. COPD D. thoracentesis and insertion of central intravenous catheters. Simple aspiration contains descending thoracic aorta. Posterior be treated in the first instance with ? Harrison’s 18th Ed. A large-bore needle is inserted into pleural space through D. 5 Physical examination in pneumomediastinum reveals subcutaneous emphysema in suprasternal Mediastinum is the region between pleural sacs. and pleuropericardial and bronchogenic cysts. Posterior C. All of the above D. C. Middle A. 2 B. Supine B. 2181 803 Hamman ’s sign is characteristic of ? A. a large-bore needle is meningomyeloceles. 2181 Most secondary spontaneous pneumothoraces are due to COPD. Middle 794 Patients with secondary spontaneous pneumothorax should C. Upright A. Prone D. Nearly all patients with secondary pneumothorax should be treated with tube thoracostomy. teratomatous Tension Pneumothorax usually occurs during mechanical ventilation or resuscitative efforts. None of the above C. mediastinal air is absorbed faster if Harrison’s 18th Ed. Third anterior intercostal space A. Insertion of central intravenous catheters C. which is a crunching or clicking noise synchronous with heartbeat & best heard in left lateral decubitus position. High concentrations of oxygen C. Needle is left in place until a thoracostomy tube can be inserted. ascending & transverse arches of aorta. 2182 B. It D. 2181 D. notch & Hamman’s sign. In the posterior mediastinum. Fifth anterior intercostal space C. meningoceles. Pneumomediastinum C. main bronchi & their contiguous lymph nodes & pulmonary arteries & veins. esophagus. Lung malignancy The most common lesions in the anterior mediastinum are thymomas. Lung malignancy compartment of mediastinum ? Harrison’s 18th Ed. Posterior mediastinum is bounded by pericardium & trachea anteriorly & vertebral column posteriorly. Middle C. 2181 A. Posterior B. COPD 800 Azygos and hemiazygos veins are present in which D. Tube thoracostomy Anterior mediastinum extends from the sternum anteriorly to pericardium & brachiocephalic vessels posteriorly. Bleb resection & veins. 2182 B. neoplasms. and esophageal diverticula are commonly found. Asthma C. Any of the above Leading causes of iatrogenic pneumothorax are transthoracic needle aspiration. brachiocephalic arteries & veins. separated in 3 compartments. phrenic nerves. 797 In treating ‘Tension pneumothorax’. Second anterior intercostal space Harrison’s 18th Ed. 802 Lymph node enlargement from metastases are most common lesion in which mediastinal compartment ? 796 Tension pneumothorax is most frequently found in ? Harrison’s 18th Ed. lymph node enlargement from metastases or granulomatous disease. lymphomas. 3 C. The most common masses in the middle mediastinum are vascular masses. Thoracentesis B. Left lateral C. 4 D. 2181 compartment ? Harrison’s 18th Ed. IV Hypertonic saline . Anterior A. 2181 A. Traumatic Pneumothorax 2nd anterior intercostal space. 804 Hamman’s sign is best elicited in which position ? 798 Mediastinum is separated into how many compartments ? Harrison’s 18th Ed. Anterior B. AJAY MATHUR Respiratory C. CO2 inhalation D. 2181 the patient is given ? A. During mechanical ventilation/resuscitative efforts B. Pleural abrasion trachea. Posterior D. inserted into pleural space through ? Harrison’s 18th Ed. Transthoracic needle aspiration A. It contains thymus gland. Anterior reported with virtually every lung disease. anterior mediastinal lymph nodes & internal mammary arteries B. 2182 Harrison’s 18th Ed. azygos & hemiazygos veins & posterior group of mediastinal lymph nodes. B. 799 Thoracic duct is present in which compartment of mediastinum ? 805 In pneumomediastinum.304 MCQ’s FOR MEDICAL PROFESSIONALS BY PROF. Anterior Harrison’s 18th Ed. gastroenteric cysts. thoracic duct. Any of the above D. neurogenic tumors. but pneumothoraces have been A. None of the above A. venae cavae. Middle mediastinum lies between anterior & posterior mediastina & contains heart. 795 Which of the following is a cause of iatrogenic pneumothorax ? 801 Lymphomas are most common lesion in which mediastinal Harrison’s 18th Ed. Fourth anterior intercostal space B. Tension Pneumothorax Tension pneumothorax is a medical emergency. Middle B. and thyroid masses. All of the above B. Carbon dioxide production Harrison’s 16th Ed. A. which results in a depressed panel in anterior A. hypoxic vasoconstriction. Toxins Chapter 264. AJAY MATHUR Cardiology 305 D. Anterior horn disease 813 Which of the following pectus excavatum is false ? Harrison’s 16th Ed. 1569 C. Carbon dioxide excretion kyphoscoliosis ? B. Goat A. it is the reverse of pectus excavatum Most patients with severe diaphragmatic weakness present with hypercapnic respiratory failure. Muscular dystrophy A. B. B. atelectasis. 1569 angle of curvature is <60°. 1569 D. 38 & 44 mmHg D. 1569 A. and muscular dystrophy. the paralyzed diaphragm will move paradoxically upward 816 In the equation PaCO2 = (k)(V. D. It describes change in pronounced sound of B. Temper B. Unilateral paralysis of diaphragm B. Nerve invasion from malignancy D. V. 1569 Pectus excavatum (Funnel chest) is a congenital condition.75:303 ventilation-perfusion imbalances that result in chronic alveolar hypoventilation. > 90º 807 Which of the following is the cause of bilateral diaphragmatic paralysis ? The severity of the cardiopulmonary disease correlates roughly with the degree of scoliosis. Disorders of Ventilation The most common cause of unilateral paralysis of the diaphragm is nerve invasion from malignancy. > 70º E to A. usually a bronchogenic carcinoma. Nasal polyp D. Respiratory symptoms are uncommon Most common causes of bilateral diaphragmatic paralysis include high spinal cord injury. Harrison’s 18th Ed. Cat Harrison’s 16th Ed. marked ventilatory abnormalities develop commonly ? C. while if it is >90°. Silence 812 In kyphoscoliosis. All of the above A. 35 & 41 mmHg A. High spinal cord injury C. Associated with congenital atrial or ventricular septal defects frequently complicated by cor pulmonale and right ventricular failure. A. If the Harrison’s 16th Ed. at what angle of curvature. Respiratory symptoms are uncommon. All of the above 806 Word egophony comes from Greek “ego” which means ? The major pathophysiologic effects of severe kyphoscoliosis are restrictive lung disease and Cleveland Clinic Journal Of Medicine 2008. The lower portion of sternum is displaced posteriorly and anterior ribs are markedly bowed. Structurally. If the patient does not have malignancy. Diagnosis of unilateral paralysis of diaphragm is suggested by finding an elevated hemidiaphragm on the chest roentgenogram. Cor pulmonale concentrations of oxygen. Pneumonia Harrison’s 16th Ed. marked ventilatory abnormalities develop commonly. Hypercapnic respiratory failure chest. Thoracic trauma sternum protruding anteriorly. Confirmation is best established with the “sniff test”. C. 2182 Harrison’s 16th Ed. OSA In health the arterial level of carbon dioxide (PaCO2) is maintained between 37 and 43 mmHg at sea level. CO2 represents ? due to the negative intrathoracic pressure. 2182 811 Which of the following is a pathophysiologic effect of severe A. Restrictive lung disease . arterial level of carbon dioxide (PaCO2) is maintained between ? 810 “Sniff test” is used for the confirmation of ? Harrison’s 18th Ed. This deformity is associated with congenital atrial or ventricular septal defects and severe prolonged childhood asthma. When a patient is observed with fluoroscopy while sniffing. 36 & 42 mmHg B. Mechanism in massive pleural effusions is upward displacement & compression or consolidation of lung at the top of effusion. 1569 C. Pulmonary function tests are nearly normal D. Carbon dioxide excess A. multiple sclerosis. A. and eventually pulmonary arterial hypertension and cor pulmonale. High doses of IV glucocorticoids B. > 60º Word egophony comes from Greek “ego” meaning goat. > 80º D. None of the above Harrison’s 16th Ed. ventilatory impairment is rare. and pulmonary function tests are nearly normal. thoracic trauma (including cardiac surgery). 1569 D. then usually no cause for the paralysis is found. the mediastinal air will be absorbed faster if the patient inspires high C. 305 Respiratory MCQ’s FOR MCQ’s FOR MEDICAL MEDICAL PROFESSIONALS PROFESSIONALS BY PROF. None of the above 808 Most patients with severe diaphragmatic weakness may present with ? Harrison’s 16th Ed.CO2)/V. 37 & 43 mmHg C. Bilateral diaphragmatic paralysis C. Ventilation-perfusion imbalance In pneumomediastinum. anterior horn disease. 1569 Pectus carinatum (Pigeon breast) This condition is the reverse of pectus excavatum with the A. Congenital condition D. C. B. C. and pneumonia. Multiple sclerosis B. Severe prolonged childhood asthma 809 Most common cause of unilateral paralysis of diaphragm is ? D. 815 In health and at sea level. Atelectasis 814 Which of the following about pectus carinatum is false ? C. 84 D. 2182 Harrison’s 18th Ed. A can be calculated as minute ventilation x(1-Vd/Vt) where Vd/Vt represents dead space fraction. B. Vagus nerve A. 2182 Harrison’s 18th Ed. An arterial blood gas demonstrating elevated Pa CO2 with a normal pH confirms chronic alveolar A. All of the above Harrison’s 18th Ed. Pre-Bötzinger complex D. pH hypoventilation ? D. 2182 Elevated plasma bicarbonate in the absence of volume depletion is suggestive of hypoventilation. Pa CO2 . Pa O2 hypoventilation. which is particularly important for the generation of active expiration. Pleura B. STOP-Bang questionnaire D. Normal pH 820 DRG acts as the initial integration site for which of the D. STOP-Bang survey is used in preoperative clinics to identify patients at risk of having OSA. k is a constant and V. All of the above receptors & juxtapulmonary-capillary receptors in lung parenchyma & chest wall to DRG ? 827 Which out of the following measures daytime sleepiness ? Harrison’s 18th Ed. 823 Which of the following is responsible for the generation of 817 V. During normal sleep there is an attenuated Harrison’s 18th Ed. Epworth Sleepiness Scale (ESS) C. particularly important area within the VRC is the so-called pre-Bötzinger complex. DRG respiratory muscle involvement in diseases with progressive muscle weakness ? B. 2182 Harrison’s 18th Ed. Pa CO2 826 Which of the following is used to identify patients of alveolar C. and lesioning of the pre- 819 Dorsal respiratory group (DRG) & ventral respiratory column Bötzinger complex leads to the complete cessation of breathing. Parafacial respiratory group (pFRG) B. DRG B. Phrenic nerve B. Berlin Questionnaire pressure of arterial oxygen (Pa O2 ). 824 Which of the following is responsible for the generation of expiratory activity ? 818 Disturbances of PaCO2 reflect which of the following ? Harrison’s 18th Ed. Parafacial respiratory group (pFRG) C. 822 Pre-Bötzinger complex is a part of ? Harrison’s 18th Ed. Medulla oblongata A. VRC . Berlin Questionnaire B. 2182 A. AJAY MATHUR Respiratory D. and blood pressure from carotid and aortic chemoreceptors and baroreceptors to the central nervous system (CNS).306 MCQ’s FOR MEDICAL PROFESSIONALS BY PROF. Bronchial airways hypoventilation ? Harrison’s 18th Ed. Pre-Bötzinger complex C. All of the above Vagus nerve relays information from stretch receptors and juxtapulmonary-capillary receptors in the Berlin Questionnaire has been validated in a primary care setting and identifies patients likely to lung parenchyma and chest wall to the DRG. All of the above ventilation. ESS measures daytime sleepiness. 2182 828 Which of the following parameter in PFT is used to monitor for A. Lung parenchyma 825 Which of the following confirms chronic alveolar C. Minute ventilation C. minute ventilation. All disturbances of PaCO2 must respiratory group (pFRG). STOP-Bang questionnaire 821 Which of the following nerves relay information from stretch D. A. Parafacial respiratory group (pFRG) V. Minute ventilation x (1-Vd/Vt) A. 2182 response to hypercapnia and hypoxemia resulting in mild nocturnal hypoventilation that corrects upon awakening. DRG A. Minute ventilation + (1-Vd/Vt) C. A is calculated by which of the following formula ? inspiratory activity ? Harrison’s 18th Ed. Epworth Sleepiness Scale (ESS) C. Elevated plasma bicarbonate The spontaneous cycle of inspiration & expiration is automatically generated in medulla through dorsal respiratory group (DRG) and ventral respiratory column (VRC). Altered CO2 production B. A is fresh gas alveolar D. All of the above The respiratory rhythm is generated within the VRC. 2184 DRG acts as the initial integration site for afferent nerves bringing in information about partial A. Dead space fraction D. B. All of the above D. 2184 A. B. CO2 represents the carbon dioxide production. 2182 A. All of the above V. Carbon dioxide retention C. 2183 . All of the above D. Elevated PaCO2 C. Minute ventilation / (1-Vd/Vt) B. This area is responsible for the generation of various forms of inspiratory activity. One reflect altered CO2 production. Minute ventilation . Intercostal nerves C. have OSA. as well as the more rostrally located parafacial All disorders of ventilation result in abnormal measurements of PaCO2. All of the above following ? Harrison’s 18th Ed.(1-Vd/Vt) D. The neural output of these medullary respiratory networks can be voluntarily suppressed or augmented by input from higher (VRC) are located in ? brain centers and the autonomic nervous system. pH. or dead space fraction. abolished have been implicated in the pathogenesis of congenital central hypoventilation syndrome. PaO2 < 80 mmHg 836 How many obstructive events & arousals per hour of sleep D. 2185 D. At least 9 second Harrison’s 18th Ed. Five obstructed apneas per hour of sleep A. Measurement of maximum inspiratory and expiratory pressures or forced vital capacity (FVC) can be used to monitor for respiratory muscle involvement in diseases with A. Body mass index (BMI) >=30 kg/m2 Hypopneas are 10 second events where there is continued breathing but the ventilation is reduced B. a transcription factor with a role in neuronal development. 20 % B. 2184 D. PHOX4b B. nasal continuous positive airway pressure (CPAP) is the treatment of choice. Chronic daytime alveolar hypoventilation define severe OSA ? Harrison’s 18th Ed. Forced vital capacity (FVC) five obstructed breathing events (apnea or hypopnea) per hour of sleep. ventilation is reduced from the previous baseline during sleep by at least ? D. Laryngopharynx Harrison’s 18th Ed. Unexplained excessive daytime sleepiness Harrison’s 16th Ed. Maximum inspiratory pressure D. D. airflow ceases despite respiratory drive D. > 30 A. 1573 called Ondine’s curse or congenital central hypoventilation syndrome (CCHS). In CSA. neural drive to respiratory muscles is transiently Abnormalities in the gene encoding PHOX2b. 2184 C. 2184 D. > 20 Harrison’s 18th Ed. All of the above A. 833 Obstructive sleep apnea/hypopnea syndrome (OSAHS) relates to ? 839 The definitive event in obstructive sleep apnea is occlusion of Harrison’s 18th Ed. 2185 D. sleep-disordered breathing and chronic daytime alveolar hypoventilation. In CSA. 2186 hypercapnia develops. Oropharynx 832 Which of the following gene has been implicated in the B. PHOX3b A. Five obstructed hypopneas per hour of sleep A. C. 50 % A. B. Nasopharynx B. and PaO2 < 70 mmHg in the absence of other known causes of hypercapnia. > 10 831 Ondine’s curse is best related to ? C. Central hypoventilation syndrome (CCHS) 837 Sleep apnea is defined as an intermittent cessation of airflow D. Maximum expiratory pressure OSAHS may be defined as coexistence of unexplained excessive daytime sleepiness with at least C. Acetazolamide Apneas are defined in adults as breathing pauses lasting 10 second. None of the above Chapter 265. neural drive to all respiratory muscles is transiently abolished. A. All of the above B. At least 7 second 829 Which of the following stimulates respiration ? C. C. 2186 upper airway usually at the level of ? A. Obstructive sleep apnea syndrome sleep). PHOX2b 838 Which of the following statements is false ? Harrison’s 16th Ed. At least 6 second progressive muscle weakness. 2186 Pharmacologic agents that stimulate respiration are medroxyprogesterone and acetazolamide. In OSA airflow ceases despite continuing respiratory drive because of occlusion of oropharyngeal airway. Oropharynx . Obesity Hypoventilation Syndrome at the level of ? Central Hypoventilation Syndrome can present later in life or in the neonatal period where it is often Harrison’s 16th Ed. 1573 C. At least 10 second A. Nasopharynx pathogenesis of congenital central hypoventilation syndrome ? C. AJAY MATHUR Cardiology 307 Harrison’s 18th Ed. Hyperventilation In severe OSA (significant daytime sleepiness or >30 obstructive events and arousals per hour of B. 2184 Diagnosis of obesity hypoventilation syndrome (OHS) requires body mass index (BMI) >=30 kg/m2. All of the above 834 Apneas of what duration are considered significant ? Respiratory muscle weakness has to be profound before lung volumes are compromised and Harrison’s 18th Ed. A. Norepinephrine Harrison’s 18th Ed. Sleep apneas can be central or obstructive D. defined as Pa CO2 >=45 A. PHOX1b B. 307 Respiratory MCQ’s FOR MCQ’s FOR MEDICAL MEDICAL PROFESSIONALS PROFESSIONALS BY PROF. PaCO2 >=45 mmHg by at least 50% from the previous baseline during sleep. B. Sleep Apnea Sleep apneas can be central or obstructive. C. Testosterone 835 In hypopnea. 30 % 830 Which of the following is false about diagnosis of Obesity Hypoventilation Syndrome ? C. B. 40 % Harrison’s 18th Ed. >5 mmHg. 1573 B. In OSA. Oestrogen C. OSA manifestations are cognitive disturbances C. 1574 Harrison’s 16th Ed. myotonic dystrophy. Theophylline Factors predisposing to OSAHS include obesity. 852 Which out of the following drug is useful in OSA ? 845 Risk factors for sleep apnea include all except ? Harrison’s 16th Ed. A. Increased neck circumference C. Male gender A. Laryngopharynx 846 Risk factors for sleep apnea include all except ? N Engl J Med 2002. Mental retardation 840 Main factor leading to collapse of upper airway in OSA is ? Harrison’s 16th Ed.347:498 D. Majority of snoring individuals do not have OSA disorder A. Shortening of mandible B. 1573 D. Retrognathia D. Narcolepsy C. Motor vehicle accidents in Harrison’s 16th Ed. retrognathia & macroglossia aggravate OSA. C. Lambert-Eaton myasthenic syndrome A. male gender. 1574 A. All of the above B. 1573 OSA are 2 to 7 times more as compared with normal. 2187 C. 848 Which of the following statements is false ? 841 Which of the following predispose to the development of OSA ? Harrison’s 16th Ed. All of the above A. hypothyroidism. B. 2186 electroencephalogram.308 MCQ’s FOR MEDICAL PROFESSIONALS BY PROF. middle age (40-65 years). Obesity Harrison’s 18th Ed. Submental electromyogram D. Obesity reduces size of upper airways by increasing fat deposition in soft tissues of pharynx D. Hall Structural compromise due to adenotonsillar hypertrophy.347:498 A. All of the above muscles to maintain airway stability. Snoring is not associated with long-term health risks B. A. 1575 C. Protriptyline D. AJAY MATHUR Respiratory C. Codeine D. . electrooculogram. Spasm of airway adductor muscles A. Domperidone & smoking. Ehlers Danlos syndrome B. Epworth Alcohol depresses activity of upper airway muscles & arousal response that terminates each apnea. Obesity B. All of the above B. Hypothyroidism In obstructive sleep apnea (OSA). and submental electromyogram. Theophylline C. All of the above accidents as normal individuals 842 Which of the following predispose to the development of OSA ? Most pervasive manifestation of OSA is excessive daytime sleepiness. Elderly D. Acetazolamide A. Adenotonsillar hypertrophy C. All of the above 844 Factors that predispose to OSAHS include all except ? Definitive investigation for suspected OSA is polysomnography. Smoking 851 Which out of the following drug is useful in OSA ? B. Electrooculogram C. Alcohol A. Fluoxetine C. Electroencephalogram B. Craniofacial abnormalities Medications are generally ineffective in management of OSA. Critical sub-atmospheric pressure in inspiration 847 Which of the following can cause day time sleepiness ? B.347:498 C. Ipratropium bromide B. Patients with OSA are equally prone to motor vehicle D. Ehlers Danlos syndrome D. shortening of mandible &/or maxilla. Dereck D. acromegaly. Short neck D. Circadian-rhythm abnormality D. It includes recording of Harrison’s 18th Ed. airflow ceases despite continuing respiratory drive because of occlusion of oropharyngeal airway. 1575 N Engl J Med 2002. 843 Factors that predispose to OSAHS include all except ? 850 Electrographic variable studied in polysomnography for OSA is ? Harrison’s 18th Ed. Myotonic dystrophy Harrison’s 16th Ed. 2186 Harrison’s 16th Ed. Critical supra-atmospheric pressure in inspiration N Engl J Med 2002. Obesity A. except in patients with predominantly rapid eye movement sleep related events in whom protriptyline or fluoxetine may be beneficial. Shortening of maxilla C. Periodic limb movement disorder The immediate factor leading to collapse of upper airway in OSA is generation of a critical subatmospheric pressure during inspiration that exceeds the ability of airway dilator and abductor D. Epworth Sleepiness Score helps to detect troublesome sleepiness. Alcohol B. 1573 A. Macroglossia 849 Sleepiness score is named after ? B. Garcia or by compressing the pharynx by superficial fat masses in neck. Acromegaly C. Hypoperfusion of respiratory muscles B. Pulmonary vasculature 856 About 50% mortality is expected when APACHE II score in nonoperative cases is ? 863 Rapid-shallow-breathing index (RSBI) is calculated by ? Harrison’s 18th Ed. 2198 B. Figure 267-1 Harrison’s 16th Ed. Respiratory bronchiole PCO2 level of wakefulness is lower than that required for rhythm generation in sleep. Hypercarbic disorders ? Harrison’s 16th Ed. Any of the above B. APACHE III system A.1 test is used to test which component of respiratory system ? Harrison’s 16th Ed. Alveolar duct D. Lung atelectasis C. Systemic vascular resistance (SVR) 865 Type II respiratory failure best relates to which of the Mean arterial pressure (MAP) is the product of cardiac output & systemic vascular resistance (SVR). Hypoperfusion of respiratory muscles C. 1588 D. Resection of Uvula & Posterior palate C. 1588 D. Reorientation of tonsillar pillars 859 Respiratory failure can be ? Harrison’s 16th Ed. Anaphylaxis .34 D. Heart rate B. Alveolar hypoventilation A. thyrotoxicosis. Approach to the Patient with respiratory system ? Critical Illness Harrison’s 16th Ed. Pulmonary vasculature A. All of the above A. Airways D. Alveoli 861 CO 2 challenge test is used to test which component of 267.19 A. following ? Harrison’s 18th Ed. Resection of tonsils failure. 1575 C. 1588 A. 2200 858 Most common cause of high cardiac output hypotension is ? A.24 B. Muscles of breathing APACHE II scoring system is the most commonly used severity-of-illness (SOI) scoring system in North America. anaphylaxis. 20 . PO2 level 860 Alveolar unit consists of all except ? C. severe pancreatitis. B. Other causes include liver A. Respiratory rate / FVC B. Combined A.347:498 The most common cause of high cardiac output hypotension is sepsis. Pulse pressure D. Respiratory rate / minute volume 857 Mean arterial pressure (MAP) is the product of cardiac output 864 Type I respiratory failure best relates to which of the following ? and ? Harrison’s 18th Ed. Respiratory rate / IC C. 2196 A. Peripheral arteriovenous shunts N Engl J Med 2002. Lung atelectasis D. APACHE II system 862 P0. 1588 A. Nervous system 855 Which of the following is the most commonly used severity-of. and peripheral arteriovenous shunts. APACHE I system B. Airways Harrison’s 18th Ed. Nervous system D. APACHE IV system B. Alveolar hypoventilation Harrison’s 18th Ed. Alveolar flooding A. C. AJAY MATHUR Cardiology 309 853 Uvulopalatopharyngoplasty in OSA involves ? D. All of the above A. Respiratory rate / tidal volume D. Alveolar flooding B. Muscles of breathing illness (SOI) scoring system in North America ? C. 15 . 30 . 309 Respiratory MCQ’s FOR MCQ’s FOR MEDICAL MEDICAL PROFESSIONALS PROFESSIONALS BY PROF. PCO2 level D. Thyrotoxicosis D. pH level Harrison’s 16th Ed. Common to all B. Sepsis C. 2196 D.29 C. Stroke volume C. C. B. burns and other trauma that elicit the systemic inflammatory response syndrome (SIRS). 2198. 2199 Harrison’s 18th Ed. Terminal bronchiole CSA disorders is a PCO2 level during sleep that falls transiently below the critical PCO2 required for respiratory rhythm generation. 1588 C. Hypoxemic 854 Which of the following is central in the pathogenesis of CSA B. 25 . Type I C. Decreased PaO 2. All of the above C. Shock Harrison’s 16th Ed. Respiratory rate > 35/minute for >5 minutes . increased PaCO2 C. Type III following ? D. decreased PaCO2 C. Type III following ? D. decreased PaCO 2 878 The ratio of partial pressure arterial oxygen and fraction of B. Decreased PaO 2. 1611 A. Hypoperfusion of respiratory muscles A. 1611 A. Lung atelectasis B. Type IV Harrison’s 18th Ed. Type II 867 Type IV respiratory failure best relates to which of the C. is a comparison between the oxygen level in the blood and the oxygen concentration B. 1611 A. Decreased PaCO2 A. Abraham index 871 Type II respiratory failure is seen in ? D. Alveolar hypoventilation 874 In multiple blood transfusions. which type of respiratory failure occurs ? Harrison’s 18th Ed. Increased PaO2. AJAY MATHUR Respiratory 866 Type III respiratory failure best relates to which of the C. Decreased PaO2. ARDS 879 Intubated patients having which of the following should D. increased PaCO 2 A. 2200 A. Type II D. Decreased PaO2 876 Perioperative respiratory failure is also called ? Harrison’s 18th Ed. Decreased PaO 2. increased PaCO 2 inspired oxygen is called ? A. Type I C. Carrico index C. Type I Harrison’s 18th Ed.120 minutes : stable oxygenation (PaO2/FIO2 >200 & PEEP <=5 cmH2O). which type of respiratory failure occurs ? A. ARDS C. Type II D. called P/F ratio or Carrico index. Decreased PaO 2. Normal PaO 2. Type I D. 873 In Guillain-Barre syndrome. Hypoperfusion of respiratory muscles B. cough & airway reflexes intact. CRF that is breathed. 2200 A.310 MCQ’s FOR MEDICAL PROFESSIONALS BY PROF. Pulmonary fibrosing alveolitis undergo spontaneous breathing trial ? Harrison’s 18th Ed. Normal or decreased PaCO2 877 Respiratory failure type IV is seen in ? Harrison’s 18th Ed. 2200 B. Alveolar flooding A. PEEP <= 5 cmH 2O A. Alveolar flooding occurs ? Harrison’s 18th Ed. 2200 B. there is ? D. Type IV Harrison’s 16th Ed. decreased PaCO 2 B. Type IV Harrison’s 18th Ed. Pronovost index Harrison’s 18th Ed. Type IV Harrison’s 16th Ed. which type of respiratory failure occurs ? 880 Spontaneous breathing trial is declared a failure and must be Harrison’s 18th Ed. Normal A-a gradient C. normal PaCO 2 B. COPD D. and no vasopressor agents or sedatives. 2200 872 In hypothyroidism. 2201 B. Girard index D. PaO2/FiO2 ratio <= 200 mmHg gets diagnosis of ARDS. PaO2 / FIO2 > 200 Harrison’s 18th Ed. Normal PaCO2 B. COPD with corpulmonale The ratio of partial pressure arterial oxygen and fraction of inspired oxygen. 2200 A. C. Type II D. Type II A. Type III 869 Type I respiratory failure consists of ? D. Type IV Intubated patients having the following should undergo spontaneous breathing trial for 30 . Type III 868 All are true about Type I respiratory failure except ? D. which type of respiratory failure B. Alveolar hypoventilation 875 In ARDS. Lung atelectasis C. Type III D. 2199 C. 2200 B. 2200 stopped if ? A. CRF 870 In type II respiratory failure. Type I C. Increased PaO2. Cough & airway reflexes intact B. 2199 B. 2201 A. evaluation of respiratory gas exchange is done by arterial blood-gas C. heart rate >140/min or a 20% increase or C. < 135 B. 2205 C.76 mL O 2 per dL blood C. 8% D. 15 % Use of neuromuscular blocking agents to facilitate mechanical ventilation may result in prolonged C. D. 2202 B. at wavelengths of 660 nm. 2201 A. 10 mL per cmH2O D. whereas the reverse is true in the infrared spectrum (940 nm). PaO2 / FiO2 <=200 mm Hg D. 33 % 883 Multiorgan system failure is defined by the simultaneous The normal tissue extraction ratio for O2 is Cao2 – Co2/Cao2 ([21. presence of physiologic dysfunction and/or failure of ? Harrison’s 18th Ed. 150 mL per cmH2O End-inspiratory pause (plateau pressure) is a static measurement. Pulmonary capillary wedge pressure <= 18 mm Hg Harrison’s 18th Ed. 2205 Syndrome of multiorgan system failure is defined by the simultaneous presence of physiologic dysfunction and/or failure of two or more organs. Organ failure must persist beyond 24 hours. 860 nm A.16–15. Acute onset 884 The “gold standard” for monitoring in the ICU is ? C. 2202 C. SIRS A. 2201 D. Circulatory Status A. patient can be extubated. 25 % weakness . Oxygen delivery (Q o2) is a function of cardiac output and content of O 2 in arterial blood (Ca o2). Respiratory system mechanics B. AJAY MATHUR Cardiology 311 B. Acute onset analysis which is the “gold standard”. Pulmonary capillary wedge pressure <= 18 mm Hg 885 Oxyhemoglobin reflects light more effectively than PaO2 / FiO2 <=300 mm Hg is typical of Acute lung injury (ALI). Harrison’s 18th Ed.76 mL O 2 per dL blood If. Arterial blood-gas analysis Harrison’s 18th Ed. 2205 C. 720 nm to ARDS ? Harrison’s 18th Ed. A pulse C. Pneumothorax Harrison’s 18th Ed. content of O2 in mixed venous blood (Co2) is 15. Four or more organs D. Pulse oximetry 890 Diagnostic criteria for ARDS include all except ? B. the ratio of the respiratory rate and tidal volume in D. Acute onset and PCWP <= 18 mm Hg deoxyhemoglobin at wavelengths of ? being the same as in ARDS. not airway resistance. O 2 saturation <90%. Three or more organs Distress Syndrome C. oxyhemoglobin reflects light more effectively than does deoxyhemoglobin. Normally. 50 mL per cmH2O Spontaneous breathing trial is declared a failure and stopped if any of the following occur. 2201 A. and the relative intensity of light D. Respiratory rate >35/minute for >5 minutes. . < 165 C. 882 Postparalytic syndrome refers to ? Normally. PaO2 / FiO2 <=200 mm Hg is a common basis for multiorgan system failure.76]/21. since the mixed venous blood is 75% saturated. at the end of the spontaneous breathing trial. HR>140/min or 20% increase or decrease from baseline A. systolic blood pressure <90 mmHg or >180 mmHg and increased anxiety or diaphoresis. respiratory system compliance is ~100 mL per respiratory rate & tidal volume in liters (f/VT) is ? cmH 2 O. 14.76 mL O 2 per dL blood liters (f/VT) is <105. Pneumonia B. 13. Neuropathy 888 Normal tissue extraction ratio for O2 is ? B. Acute Respiratory B. Five or more organs 889 Diagnostic criteria for ARDS is ? Harrison’s 18th Ed. D. Two or more organs Chapter 268. Burns oximeter passes both wavelengths of light through a finger. 100 mL per cmH2O decrease from baseline. Near drowning transmission at these two wavelengths is analysed to derive relative percentage of oxyhemoglobin. O2 saturation < 90% 886 Normally. affected only by respiratory 881 Successful spontaneous breathing trial means that the ratio of system compliance.a myopathy known as the postparalytic syndrome. 12. 15.76 mL O2 per dL blood. PaO2 / FiO2 <=300 mm Hg For monitoring in the ICU. 660 nm 891 Which of the following is the most common situation leading B. 2201 887 Normally. Harrison’s 18th Ed. < 55 Harrison’s 18th Ed. 311 Respiratory MCQ’s FOR MCQ’s FOR MEDICAL MEDICAL PROFESSIONALS PROFESSIONALS BY PROF. Multiple bone fractures D. All of the above A. 940 nm B. < 105 A. Head trauma In pulse oximetry. respiratory system compliance is about ? Harrison’s 18th Ed. Harrison’s 18th Ed. 2202 C.76 mL O 2 per dL blood D. D. Any of the above B.16) or 25 %. the content of O2 in mixed venous blood (Co2) is ? A. Myopathy A. B. Injury to alveolar capillary endothelial cells A. 893 Phases of ARDS are all except ? 899 Chest radiograph in exudative phase of ARDS shows alveolar Harrison’s 18th Ed. Chest x-ray in ARDS rarely shows cardiomegaly A. Neutrophil-rich leukocytic infiltration B. 0 to 81 C. 2205 Figure 268-1 Harrison’s 18th Ed. 894 Which of the following is a lipid mediator ? Harrison’s 18th Ed. 897 Which of the following is false about early ARDS ? Harrison’s 18th Ed. Two-thirds of lung fields D. One half of lung fields D. Diffuse pneumonia D. 2205 Harrison’s 18th Ed. Exudative phase encompasses first 7 days of illness after exposure to a precipitating ARDS risk factor. pulmonary dead space is prominent. Eosinophil predominant . 0 to 71 B. radiation pneumonitis and neurogenic pulmonary edema. Cardiomegaly C. pleural effusions or pulmonary vascular redistribution. 2207 B.e. Exudative Harrison’s 18th Ed. First 1 day B. Pleural effusions D. Less frequent diagnoses include acute interstitial lung diseases. near diminished aeration & atelectasis. pulmonary contusion. 2206 903 Histologically. in proliferative phase of ARDS. most common disorders are cardiogenic pulmonary edema. Age. Proliferative B. Day 7 to 21 Proliferative phase of ARDS usually lasts from day 7 to day 21. Fibrotic C. CxR in exudative phase of ARDS reveals alveolar & interstitial opacities involving at least three- quarters of the lung fields. Leukotriene B4 C. Pulmonary vascular redistribution D. First 5 days D. toxic inhalation. each having its own characteristic clinico-pathologic features is marked by three phases i.5-fold increase in risk of ARDS. 895 Which of the following is not a feature of exudative phase of ARDS ? 901 In D/D of ARDS. 2207 A. and alveolar hemorrhage. alveolar edema involves “dependent” portions of lung. Prominent interstitial inflammation C. whereas head trauma. AJAY MATHUR Respiratory Among patients with trauma. CxR rarely shows cardiomegaly. First 3 days C. 2205 & interstitial opacities involving ? A. and tumor necrosis factor alpha) and lipid mediators (leukotriene B4) are present in the lung in early exudative phase of ARDS.348:683-93 Harrison’s 17th Ed. Tumor necrosis factor  B. Proliferation of type II pneumocytes C. Lymphocyte predominant C. Three-quarters of lung fields Natural history of ARDS. hypercapnia secondary to an increase in drowning. lungs with subsequent formation of hyaline membranes from diffuse alveolar damage. Injury to type I pneumocytes B. and burns are rare causes. Interleukin 8 A. Those with a score >20 have an incidence of ARDS that is 3 fold greater than those with APACHE II scores <=9. Chest x-ray in exudative phase of ARDS rarely shows cardiomegaly. and Chronic Health Evaluation (APACHE II) can range from ? 898 After exposure to a precipitating ARDS risk factor. 892 Scores for Acute Physiology. which one of the following is least likely ? Harrison’s 18th Ed. pulmonary A. multiple bone fractures. diffuse pneumonia. 0 to 91 D. Hypersensitivity pneumonitis C. 2205 900 Chest x-ray in exudative phase of ARDS rarely shows ? A. Day 7 to 14 C. proliferative. 2207 B. All of the above Significant concentrations of cytokines (interleukin 1. One-third of lung fields C. None of the above B. Cardiogenic pulmonary edema B. With severe hypoxemia. leading to flail chest are the most frequently reported surgical conditions in ARDS. Hypercapnia is prominent in early ARDS Harrison’s 18th Ed. 0 to 61 A. 2207 A. Interleukin 1 Harrison’s 18th Ed. Alveolar edema A. 2207 C. and fibrotic. Serofibrinous A. and chest wall trauma/ In exudative phase of ARDS. Alveolar hemorrhage Exudative phase is notable for early alveolar edema and neutrophil-rich leukocytic infiltration of the In D/D of ARDS. exudative. interleukin 8. 896 Which of the following is not a feature of exudative phase of ARDS ? 902 Proliferative phase of ARDS usually lasts from ? Harrison’s 18th Ed.312 MCQ’s FOR MEDICAL PROFESSIONALS BY PROF. exudative N Engl J Med 2003. 2206 A. Alveolar edema involves dependent portions of lung infiltrate is ? B. Day 7 to 18 D. hypersensitivity pneumonitis. Day 7 to 10 B. Formation of hyaline membrane whorls D. Neutrophil predominant D. First 7 days Trauma patients with APACHE II score >=16 have a 2. 1680 phase encompasses how many days ? Harrison’s 18th Ed. Vascular obliteration by microthrombi & fibrocellular proliferation D. pleural effusions or pulmonary vascular redistribution. Fibrotic phase B. ARDS B. Proliferation of type II pneumocytes A. Markedly increased C. B. Increased hydrostatic pressure B. 2207 912 Mechanical ventilation strategies used for ARDS are ? A. 2207 A. Cuboidal type II cells make up 10 % of alveolar surface Harrison’s 16th Ed. what measure of tidal volume is preferred ? 916 All are true about ARDS except ? Harrison’s 18th Ed. D. C.342:1335 C. 12 mL/kg ideal body weight 904 Which of the following represents a reparative process in Significantly lower mortality was observed in large-scale. which of the following 913 Cause of pulmonary oedema in ARDS is ? occurs ? Harrison’s 16th Ed. 12 .18 mm Hg 905 Presence of procollagen III in alveolar space is associated with increased risk of death in patients of ? In ARDS. Sepsis 908 Which of the following is false about normal alveolar epithelium ? N Engl J Med 2002. 2207 911 Inverse ratio ventilation refers to ? A. in fibrotic phase of ARDS. High-frequency ventilation (HFV) D. Normal D. organization C. Basophil predominant B. inspiratory time is lengthened so that it is longer than 906 Increased risk of pneumothorax is present in which phase of expiratory time. Bronchial asthma A. Organization of alveolar exudates 910 In ARDS. 1592 Harrison’s 18th Ed. Proliferative phase A. ARDS ? Harrison’s 18th Ed. A. 10 mL/kg ideal body weight of alveolar exudates & a shift from a neutrophil to a lymphocyte-predominant pulmonary infiltrate. Multiple blood transfusions A. first signs of resolution are evident with initiation of lung repair. Harrison’s 18th Ed. Extracorporeal membrane oxygenation (ECMO) Increased risk of pneumothorax is present in fibrotic phase of ARDS. With diminished time to exhale. which of the following is a theoretical “optimal PEEP” B. Widespread atelectasis . AJAY MATHUR Cardiology 313 D. Pneumoconiosis Harrison’s 18th Ed. Lymphocyte-predominant pulmonary infiltrate for alveolar recruitment ? Harrison’s 18th Ed. leading to emphysema- like changes with large bullae. 1592 A. Harrison’s 18th Ed. 2208 B. None of the above D. 1592 alveolar duct & interstitial fibrosis. Partial liquid ventilation (PLV) C. 2208 B. 1592 area and are more resistant to injury A. 313 Respiratory MCQ’s FOR MCQ’s FOR MEDICAL MEDICAL PROFESSIONALS PROFESSIONALS BY PROF. All of the above 914 All are predisposing factors of ARDS except ? Histologically. 2207 A. 5 . Extensive interstitial fibrosis C. All of the above C. Type II cell functions include surfactant production. randomized controlled trials in the low tidal proliferative phase of ARDS ? volume patients (6 mL/kg predicted body weight) as compared to conventional tidal volume patients (12 mL/kg). D.5 mm Hg D. Status asthmaticus are easily injured 915 Usually. 2208 C. dynamic hyperinflation leads to increased end- expiratory pressure. there is a proliferation of type II pneumocytes along alveolar basement membranes. In inverse ratio ventilation technique. Increased vascular permeability A. Ventilating at extremely high respiratory rates a protracted clinical course & increased mortality from ARDS. Lymphatic obstruction C. alveolar edema & inflammatory exudates of earlier phases convert to extensive Harrison’s 16th Ed. 12 . Decreased 909 In ARDS. 2207 Harrison’s 16th Ed.15 mm Hg D. Exudative phase Harrison’s 18th Ed. similar to ventilator-prescribed PEEP. 0 . Moderately increased and proliferation & differentiation to type I cells after injury C. All of the above C. 6 mL/kg ideal body weight A. ion transport.12 mm Hg As part of the reparative process. 8 mL/kg ideal body weight In proliferative phase of ARDS. Flat type I cells make up 90 % of alveolar surface area and D. Expiratory time is longer than inspiratory time Presence of alveolar type III procollagen peptide (marker of pulmonary fibrosis). Prone-position ventilation C. All of the above 907 Histologically. Fat embolism B. pulmonary capillary wedge pressure in ARDS is ? B. Inspiratory time is longer than expiratory time D. Acinar architecture is markedly disrupted. is associated with D. Physiological D.15 mmHg is a theoretical “optimal PEEP” for alveolar recruitment. All of the above B. Extensive alveolar duct B. 15 . Large bullae D. Pulmonary Langerhans Cell Histiocytosis 918 Secondary factors that increases the risk for ARDS include ? C. phosgene. Cryptogenic Organizing Pneumonia N Engl J Med 2000 .342 :1335 D. Low serum pH C. aldehydes. 920 Pneumonia caused by which of the following may be clinically a sedative-hypnotic. Chronic lung disease B. 2227 A. Pneumocystis D. Normal PCWP Harrison’s 18th Ed. metal oxides. an opioid. 1592 921 Which of the following is similar in presentation to acute A. Chronic alcohol abuse A.314 MCQ’s FOR MEDICAL PROFESSIONALS BY PROF. Neisseria meningitidis C. Haemophilus influenzae . cyanide. Chronic lung disease AIP is a fulminant form of lung injury characterized histologically by diffuse alveolar damage. Hypoxemia A. mercury. AJAY MATHUR Respiratory B. High serum pH D. or salicylate. All of the above ARDS can be caused by poisoning with carbon monoxide. indistinguishable from ARDS ? ammonia. Harrison’s 16th Ed. hydrogen sulfide. Escherichia coli 917 All are true about ARDS except ? Pneumonia caused by viruses or by Pneumocystis may be clinically indistinguishable from ARDS. Chronic alcohol abuse Acute Interstitial Pneumonia (Hamman-Rich Syndrome) is similar in presentation to acute respiratory distress syndrome (ARDS). or shock. fumes. paraquat. Ground glass mottling in X-Ray chest C. All of the above 922 Adult respiratory distress syndrome (ARDS) can be caused by 919 Secondary factors that increases risk for ARDS include except ? which of the following ? Harrison’s 18th Ed. Harrison’s 18th Ed. Cyanosis D. chlorine. Increased surfactant B. Desquamative Interstitial Pneumonia A. or vapors (acids and alkali. polymers). phencyclidine. B. D.342:1335 A. Low protein pulmonary oedema B. AIP probably corresponds to the subset of cases of idiopathic ARDS. or by prolonged anoxia. Opioid C. Most C. by inhalation of irritant gases. Low serum pH patients are >40 years of age. Prolonged anoxia D. hyperthermia. 2166 C. Carbon monoxide B. isocyanates. Pulmonary hypertension respiratory distress syndrome (ARDS) ? B. Acute Interstitial Pneumonia D. Chapter e50 N Engl J Med 2000. ............... B 173 .............. A 72 ...................... B 168 ....... C 127 ........ C 154 .................. C 18 ....... A 155 ... D 220 ... C 197 ... B 113 ............ D 27 .................. A 118 ... D 194 .............................. C 162 ........................ C 105 ....................... D 47 ........................................... C 221 ....... D 50 .. B 73 . A 187 ............. A 136 .......................... A 112 ... B 114 ...... A 142 ...... A 57 ..... D 94 ...................... D 147 . A 106 .... C 32 ........ C 98 ............. A 3 .......... D 86 . D 153 ........................................... A 61 ...... D 175 ...... C 146 ....... C 59 .................. D 179 ...................................... A 164 .................................... A 125 .............. C 169 ..................... A 231 ... D 151 ........................ D 93 .. B 63 ... C 210 .................... D 16 .... A 34 .......... B 144 ..................... B 128 ............. D 5 .... D 120 ................... A 201 ................ A 38 ....... D 226 .............................. A 100 .. A 40 ............... D 166 .............. C 225 ... D 41 ............ D 138 ................. D 190 ...... B 150 ....................... A 165 ................................... A 12 ...... A 188 ............................ C 77 ... D 21 .... A 223 ...................... A 193 .. C 149 ..................................... A 35 .. C 29 .......... B 158 ............... B 189 .... B 99 ............................................. C 133 ........... D 177 .......................... C 56 .... B 135 . D ................................................................. B 22 ................................................ D 39 ........ A 116 ..... C 152 ......................... B 4 ........................ B 79 .................................. B 176 ....................... A 46 ..... A 184 ............................................ C 148 ................... D 185 ........ A 26 .................... A 143 ................................................................. D 219 .................................. D 58 ................................ D 198 ........... A 206 .. A 25 . A 204 ........... B 64 ................. B 202 .................... A 85 ....................... A 30 . C 88 ....... A 49 ......................... C 131 ....... C 208 ........ C 171 ........... A 212 .. C 200 ......................... C 163 ... D 15 ........ D 156 ................... D 82 ... D 36 ........... D 218 ........ C 222 .............................. D 19 ........... D 17 ................................ C 228 .......................... B 107 ... D 214 .......................................... D 78 ........ D 76 .......................... D 7 ... B 145 ............ A 192 ............... A 203 .............. A 51 ............ D 71 ............ D 9 ................... C 199 ............. D 53 .......... C 234 ......................................................... D 103 ......... C 87 ........ B 229 ............ C 74 ......... D 181 ........... B 205 . B 81 .............................. B 84 ... C 196 ................................................ AJAY MATHUR 315 ANSWERS RESPIRATORY 1 ...................... A 24 ... D 167 ..... D 159 ........ D 8 .. D 90 ..................... D 31 ....... D 97 .......................................................................... A 121 ...................... D 48 ........... D 62 .............. A 28 ..... D 216 .......................... D 95 ........................ B 139 ....... A 108 .. D 141 .......................................................... D 140 ........ A 117 ... C 213 .. C 183 ................................... D 124 ... D 33 .... A 75 ... B 102 ... C 232 .... A 11 .Answer MCQ’s FOR MEDICAL PROFESSIONALS BY PROF......................... B 37 ............ D 2 ...... B 161 ..... C 174 ... D 45 ...... C 215 .... A 209 ................................. C 68 ... C 70 ............ D 6 ...... B 217 .......... B 101 ..... D 91 .. D 178 ....................... B 129 ............... A 137 ........... D 42 ... C 126 ........................................ D 55 ................................ A 54 ..... A 123 .. A 186 ......... D 157 ................................... D 69 ................. C 83 ............. B 191 ........... B 60 .... C 109 .................................... D 92 ............... D 104 ....................... C 52 ............. A 89 ................................... C 160 .. D 44 ...................... C 195 ......... A 111 ............ A 233 .. B 122 ............................................................. C 227 ............ A 182 ..... A 230 ................................... A 20 ........ C 224 .... A 172 ... D 119 ......... C 134 ....... B 110 .............................. D 67 ..... B 23 .. A 130 ........... C 66 ........................ C 132 ............ C 211 ......... C 14 ...... D 10 ............ A 115 ... D 96 ......... D 207 ..... D 13 ............ B 170 ...................... C 80 .......... A 65 ............ C 180 ........... D 43 ...... .......... D 391 ....... B 392 .......... D 309 ............. D 405 ........................ B 265 ....................... B 253 ....... D 352 ................................. D 330 ................................. D 287 ...... D 332 ....................................... D 326 . C 412 ..... D 288 .......... D 294 ................. A 437 ............ B 425 . A 255 ..... D 272 ....... B 388 ............... D 322 .......... B 364 .................... C 464 .......... D 394 . D 371 .................. C 380 ........................................... D 370 ....................................... D 408 .. C 463 .................................................................................... D 424 ....... B 304 ............... D 337 . C 277 . B 448 .............. B 243 ......................... D 295 .... D 398 .......................... C 246 ........... B 419 . D 270 ......... A 261 .. D 271 ... B 428 .................... A 431 ........................ B 406 ...... D 441 ......... D 343 ... B 374 . D ................ D 355 ........................ A 263 .... B 305 ............ C 301 ... B 252 ................................. B 434 ..................................................................... D 456 ................ D 399 ........................................ C 361 ......... D 333 ..... C 274 ............ B 423 ..... C 385 ........... B 403 ............................... A 427 .. D 283 ..... B 429 ............... B 266 ........................................ B 468 ............. D 336 ..................................................... D 458 .......... D 461 . D 268 .. C 379 ... A 356 ..... D 446 ........... A 312 .......... D 290 ... D 354 ....... A 278 ..... C 386 ..... B 257 .. D 384 ........... B 275 ..... A 334 ............... B 317 ............ A 236 .. A 276 ... D 442 ... C 409 . B 289 . B 256 .. A 372 .. C 363 ... C 422 ................. D 454 ...................... A 314 ... D 296 ...................................... A 387 ...................... C 414 ...... C 421 .................. C 238 ..................... D 439 ........................... A 353 .... B 367 .......... D 341 ... D 435 .................. A 306 . C 407 ...... D 368 .......... A 378 ........ C 279 .................... A 383 ..................... A 389 ................ C 262 .... B 280 .............. A 430 ........................................... D 316 .................................... C 298 ........ C 307 .......................... D 359 ......... D 292 ................................ D 376 ... C 325 ... D 358 . B 251 ................................... A 438 ... C 349 ........................ D 254 ...... D 249 . D 348 ........... D 447 ..................... D 357 . D 375 ................ A 303 ............................ D 264 ................ B 324 ...................... A 339 .. D 340 ...................... B 466 ....... D 335 ....... A 302 ............................... C 400 ........ A 345 . D 404 ............................................ D 453 .............................................................. C 313 ............... A 248 ........................ A 432 ............. D 416 . B 455 ..................... A 393 ..... A 323 ...... A 395 ........ C 338 ................... A 242 ....... D 362 ................. D 443 .............. A 260 .......... D 450 .. A 445 ........ A 397 ............... A 319 ... D 350 .... C 284 . A 239 .......................................... D 300 ....... D 457 .... D 420 ............. D 318 .................. A 417 ............................................................................................... D 460 ....................... D 291 . D 247 .................... A 346 ..................... C 373 ..... A 451 .... D 282 ................. C 444 ............316 MCQ’s FOR MEDICAL PROFESSIONALS BY PROF.......... C 377 ............. C 308 ... D 328 . A 382 ............... D 440 ....................... D 415 ...... A 311 ...... D 467 ... C 240 ................ D 411 . D 465 ... C 462 ............................................................ A 344 .................... D 449 .......................... C 390 ........ D 459 ... B 267 . C 315 ................................ D 259 ........ A 245 ................. C 401 .. C 327 .. C 320 ..... A 281 ....... C 426 ...... D 250 .................................... AJAY MATHUR Answer ANSWERS RESPIRATORY 235 ..... A 365 . D 258 ........ C 360 ................. D 369 . D 436 .... B 286 ............. B 366 . D 273 ... D 244 ..... C 285 ....................... D 297 ........ A 396 ................. C 402 ..... D 342 ...................... D 299 ................................................................. A 347 ................. C 433 ................................... C 321 ..... C 310 ... B 452 .... A 331 . D 410 .. D 329 ........... D 351 . B 293 ................ A 241 ............ B 237 . D 381 ................................ D 418 .... D 413 ...... A 269 ....................................................... ... C 633 ...................................................... D 477 ................. D 688 .... B 623 ..................................... A 600 ....... C 540 ..... D 563 .. D 519 ............................................ C 475 ........... D 689 ...... B 669 .. A 582 ......... A 589 ......... C 615 ............. B 479 ................................ A 518 ...................... D 524 .................................................................... D 529 .. D 670 ........ A 499 ................ A 694 ....... A 491 . C 701 . A 683 ...................... B 547 ....... B 546 .... C 483 ..................... D 678 ........................ D 656 .. D 693 ........ D 588 ................................ D 628 ....... D 581 .... B 574 .......... D 662 .......... C 649 ......... C 646 . C 568 ........... D 644 . D 486 ........ B 532 ............. D 543 ................................ B 695 .......... A 641 .. A 697 . D 569 .............. B 577 .... C 533 ....... D 490 ................................ D 530 .......... C 557 ........................ B 657 ................. D 637 ................ D 493 ... C 617 ......... C 500 .... D 550 .................................................. D 627 .................. C 579 ............. A 513 ................... D 498 ......... D 681 ....................... C 470 .... C 685 ..... D 516 .................Answer MCQ’s FOR MEDICAL PROFESSIONALS BY PROF.. C 481 ..................... A 506 ....... A 591 ... D 658 .. B 488 .. A 572 ....... D 619 ...... B 692 ................. D 509 ................ B 505 ............................. D 677 ... B 599 .... C 542 .. B 496 ................ B 676 .................................. D 671 ..... A 631 ....................................... D 510 ............................... C 564 ..... D 527 ...... C 590 .. B 660 ........... B 606 ........................... A 696 .. D 664 . B 653 ........................ C 702 ......... D 487 .......................... C 492 ........................... A 611 .... B 682 . D 665 .......... D 626 ....... C 559 ...... B 642 .... C 560 ................. D 544 . A 643 .......... B 471 ..... B 497 . C 555 ... C 659 ........ D 512 ........................ C 575 ..................... D 562 ................. B 521 ...... AJAY MATHUR 317 ANSWERS RESPIRATORY 469 .................................. B 592 . A 602 ......... B 561 ............................................ B 608 .......... C 473 ............................... D 511 . B 650 ..... C 655 .................... D 666 ....................................... D 604 ................... C 651 .... A 654 ....... A 578 ......... A 612 ..................... D 603 .... B 525 ................................. A 672 ...... D 636 ............ D 584 .. C 610 ................................................................. A 580 ................... D 566 ......... C 598 ................ A 502 ........... D 553 ................. C 541 ........ D 648 ..................... D 680 . D 601 ........ D 595 ............... A 517 ................. C 523 ..................................................... A 620 ................. C 484 ................... D 573 .............. C 629 ...... D 638 ... A 684 .. A 556 .................................. D 485 ..... B 501 . C 558 ..... B 552 ...... A 640 ....... A 583 ............ D 593 ................ A 474 ... C 503 ..... D 597 ... C 667 ................................................... D 622 .................... A 674 ...................... C 609 .. D 614 ................................ D 472 .... D ....... D 476 ........ A 621 ...................................... C 504 ..................... D 675 .................. D 494 .. D 535 .. D 661 ............ D 480 ............... D 482 . D 586 .. B 549 .. D 607 .......................... C 647 ... D 495 . B 534 .............. C 630 ...................... C 570 ............ D 624 ...................... B 639 ............... C 528 ................. D 634 ..................................................................... D 699 . D 686 ..... B 522 .. C 652 ........................ A 700 .... A 520 ................... D 625 ............. D 613 . A 565 .. B 554 ..................... D 618 .................. A 539 ..... A 537 ....... D 526 ...................... D 698 ........................................ D 567 ......... D 545 ............... A 571 ....... B 538 ... D 548 ... D 605 ................. D 645 .................... A 536 ............... D 587 ........................................................... C 679 .. A 551 ..... A 531 .................... C 478 ....................... C 594 ..... D 691 ............... D 507 ..... A 673 .............. D 690 ...... B 635 ................. C 585 ........................................ A 508 ....... A 596 ...................................... D 632 ........ D 576 ................... A 687 ... B 515 .......... D 668 ......... C 489 ................. B 616 .................. B 514 ................... D 663 ......... .............. B 757 .......................... D 724 ..................................... D 732 ................................. C 811 .................. D 723 ... B 824 ............ D 858 ....... D 720 ....... D 887 ....................... D 825 . D 763 ........................ B 827 ....... AJAY MATHUR Answer ANSWERS RESPIRATORY 703 ...... D 777 ........ D 886 ............ B 788 ........................ D 784 .............. D 846 ..... A 844 .. C 796 ............... A 836 ............................. D 771 ......... A 754 .................... B 837 ............................................................ D 797 ..... A 908 ................ C 743 .................................. D 919 ............ C 721 .......................................... B 758 ........ D 916 ........ B 911 ...................... B 772 ............................. A 917 .. D 918 . B 894 .................... B 816 ........ A 876 ................. B 776 ........ C 712 ........ B 878 ......................... D 770 ..... C 854 ........... D 864 .......... A 840 .... A 808 ................................................ C 906 .. D 907 ...... B 774 ...................... D 742 ........................... A 715 ............................. A 778 .................... A 849 . D 828 ...... A 914 .......... D 801 ............ A 794 .......................... C 764 ........ C 831 ........ D 880 ......... B 903 .......................... C 726 ................................ D 889 ....... D 706 ... D 830 ........................................ D 881 ................................ D 790 ........... A 730 .......318 MCQ’s FOR MEDICAL PROFESSIONALS BY PROF......... D 760 .......... C 843 ..... A 736 ......................... D 780 .............. D 708 ..... C 744 . B 799 ......... B 737 ..... C 762 ... C 740 .... A 820 ............................ B 745 ...... B 909 ......................... B 888 .. D 782 .. C 711 ............. B 815 ....... C 769 ........... B 813 .................. A 823 ...... B 855 ... D 749 ............................................................ D 725 .. B 755 .................. B 890 ......... B 707 ... B 863 ............... B 841 ............ A 826 ................... B 800 .. D 704 ..... B 722 .................. D 875 .... D 750 ................ D 847 . D 806 ....................................... C 759 ... D 734 ....... A 833 ............. D ................................. D 821 ...... A 904 ..... D 853 ............. A 922 ................. D 779 .... C 731 .................. A 719 .... B 920 ................. A 913 ........................................ D 896 ........ D 718 ......... C 869 .......... C 710 .............. C 895 ........................ D 874 ........ B 735 .... D 898 ...... D 873 ............. B 912 .............................. A 897 ............................................................... D 859 . A 868 ........ C 885 ........ A 879 ............ D 793 ........... D 714 ................................. D 857 ........................... A 781 .......... C 838 ................. D 834 ................................ C 766 .. A 842 ... D 705 ...... D 739 ......................................... D 829 .................. C 807 ..... D 773 ...... C 818 ...... A 727 ..... D 752 .. D 877 ... B 717 ........... B 798 . D 804 ..... B 747 . A 787 ............. D 872 ................ A 819 ....... D 728 .......................... D 814 ...... D 867 ... D 850 . D 791 .... B 746 .......... D 792 ............. B 709 .............. D 882 ...... A 856 ................... A 817 ................................ D 789 ... D 865 ...... D 751 ............................... D 812 .... A 901 ... C 822 ......................... D 765 ..... A 783 .............. A 835 ............ A 809 .. D 786 .... D 713 ...... D 852 ..... C 870 ...................... D 905 ........................................ C 884 ....................................................... C 753 ...... B 871 ...................... C 883 .... D 768 ......... A 910 ............................. C 915 ............ D 802 .... B 805 ....... D 892 ... C 848 ...................... D 761 . C 716 ............. C 862 ...... C 733 ..................... D 845 .................. A 900 .... A 785 .. B 795 .................. C 832 ..................... D 810 ............................................ C 748 .. D 891 ................ A 860 ............................. A 803 ... C 741 ........................... B 866 ..... B 756 .. B 729 ........ B 738 ..................... C 839 ...................................... A 893 .............. B 767 ........................... B 899 ...... C 921 .... A 775 ... D 851 ... B 861 ..... C 902 ................ 250 to 400 cm 3 Normal combined kidney volume is about 250 to 400 cm3 8 Which of the following is not used to estimate GFR ? Harrison’s 18th Ed 334 2 Which of the following organs has the maximum blood flow A.15:1 in oliguric acute renal failure. B. < 15 mL / minute BUN/PCr ratio in prerenal azotemia is > 20 : 1. 319 Nephrology MCQ’s FOR MCQ’s FOR MEDICAL MEDICAL PROFESSIONALS PROFESSIONALS BY PROF. It provides a more sensitive indicator of GFR than the plasma creatinine B. AJAY MATHUR Cardiology 319 7 For calculating GFR by Cockcroft-Gault formula. >5:1 4 In general. mL / kg / minute 10 The BUN / PCr ratio in prerenal azotemia is ? Harrison’s 18th Ed 337 Table 44-2 Creatinine clearance is calculated for a defined time period (24 hours) & is expressed in mL/minute. C. 150 to 200 cm3 D. 100 mm Hg A. Structural glomerular injury concentration. It is 10 . 125 I-iothalamate C. 0. Azotemia and Urinary multiplied by what factor for women ? Abnormalities Harrison’s 18th Ed 334 A. mL / day D. Ammonium excretion of sodium (FENa) ? D.55 1 Normal combined kidney volume is about ? B. UNa x P cr x 100 + (PNa x Ucr) C. Brain C.75 B. 0. Production not affected by diet or nutritional status 13 Evidence of which of the following is present in ATN ? D. < 90 mL / minute Harrison’s 18th Ed 337 Table 44-2 A. Creatinine 12 Which of the following formula is correct to calculate fractional C. mL / minute D. the value is Chapter 44. patients do not develop symptomatic uremia until renal insufficiency is usually quite severe (GFR < 15 mL/minute). Kidney pressure falls below ? Harrison’s 18th Ed 337 Renal blood flow drains ~20% of cardiac output or 1000 mL/minute. Structural collecting duct injury relatively constant rate from all nucleated cells.85 C. < 45 mL / minute C. Urine osmolality >500 mosmol/L H2O 5 Cystatin C estimation is a renal function equivalent of ? C. mL / hour Once the mean arterial pressure falls below 80 mmHg. 200 to 300 cm 3 For calculating GFR by Cockcroft-Gault formula. 100 to 150 cm3 C. UNa + Ucr x 100 ÷ (PNa + Pcr) 6 Which of the following is false about Cystatin C ? Harrison’s 18th Ed 335 B. B. 90 mm Hg Harrison’s 18th Ed 334 C. there is a steep decline in GFR. 0. produced at a A. MDRD (modification of diet in renal disease) Harrison’s 17th Ed 1742 A. Produced from all nucleated cells D. UNa x Pcr x 100 ÷ (PNa x Ucr) A. Cystatin C production is not affected by diet or nutritional status.85 for women a lower fraction of the body weight is composed of muscle. Heart D. C. < 60 mL / minute 11 Which of the following is false in prerenal azotemia ? D. B. Inulin Harrison’s 18th Ed 337 Table 44-2 A. D. the value is multiplied by 0. None of the above Harrison’s 18th Ed 337 Cystatin C is a member of cystatin superfamily of cysteine protease inhibitors.354:2473-83 D. 80 mm Hg 3 Value of creatinine clearance is expressed in ? B.(PNa x Ucr) B. patients do not develop symptomatic uremia until B. Structural tubule injury . Member of cysteine protease inhibitors C. Fractional excretion of sodium (FENa) >2% N Engl J Med 2006. > 10 : 1 GFR is ? C. 0. Urine sodium (UNa) <20 meq/L In general. Urea In prerenal azotemia. A.65 A. > 15 : 1 Harrison’s 18th Ed 334 D. A. > 20 : 1 A. 110 mm Hg B. Fractional excretion of sodium (FE Na) is <1%. Insulin clearance per gram ? B. UNa x P cr x 100 . Urine/plasma creatinine (UCr/PCr) >40 A. Liver 9 There occurs a steep decline in GFR when mean arterial D. Cockcroft-Gault formula B. C. < 30 kDa Harrison’s 18th Ed 337 C. < 150 mg / day Harrison’s 18th Ed 337 B. < 500 mL Harrison’s 18th Ed 338 Oliguria refers to a 24-hour urine output of < 400 mL. C. Nonanuria 26 Size of pores of normal glomerular endothelial cell is ? C. Urokinase A. IgA C. 5% D. < 200 mL D. Tamm-Horsfall protein B. A. < 40 kDa A. < 20 kDa 15 Stigmata of atheroemboli is ? B. B. Harrison’s 18th Ed 338 16 Which of the following statements is false ? A. distal peripheral infarcts. < 300 mg / day C. < 200 mg / day A.320 MCQ’s FOR MEDICAL PROFESSIONALS BY PROF. False anuria B. Urokinase 19 Urine output of > 400 mL/day in acute or chronic azotemia is called ? D. 15 % D. None of the above Harrison’s 18th Ed 338 Eosinophils in urine suggest allergic interstitial nephritis or atheroembolic renal disease. apoproteins. < 50 mL A. Nonoliguria Harrison’s 18th Ed 338 D. False azotemia A.0. enzymes. 10 % Dipstick measurement detects albumin and gives false-positive results when urine pH > 7. False-positive when urine is very concentrated output ? Harrison’s 18th Ed 338 C. All of the above B. A. < 100 mg / day 17 Oliguria refers to a 24-hour urine output of ? Harrison’s 18th Ed 338 C. < 200 mg / day A. ~100 nm C. < 400 mL 24 Which of the following is secreted by renal tubules ? D. AJAY MATHUR Nephrology D. Beta 2-microglobulin Harrison’s 18th Ed 338 Tamm-Horsfall. Livedo reticularis D. False-positive when urine is contaminated with blood A. Tamm-Horsfall protein Anuria is the complete absence of urine formation (< 100 mL). eosinophilia. Eosinophilia D. < 30 mg / day B. < 50 kDa B. C. A. < 100 mL Harrison’s 18th Ed 338 D. B. or 1000 mL/minute. A. A. < 300 mg / day B. ~150 nm . Small amounts of beta 2- microglobulin. IgA. and urokinase in urine is secreted by the tubules. urine is very concentrated or contaminated with blood. IgA 18 Anuria refers to a 24-hour urine output of ? Harrison’s 18th Ed 338 C. < 300 mL Normal individuals excrete <150 mg/day of total protein and <30 mg/day of albumin. Renal artery thrombosis leads to mild proteinuria & hematuria C. Renal vein thrombosis leads to heavy proteinuria & hematuria D. < 300 mL D. < 200 mL 25 Which of the following is not secreted by renal tubules ? C. ~50 nm Nonoliguria refers to urine output >400 mL/day in patients with acute or chronic azotemia. and peptide hormones are filtered. False-positive when urine pH is > 7. All of the above 20 Which of the following is false about dipstick measurement of proteinuria ? Prolonged renal hypoperfusion can lead to acute tubular necrosis (ATN) and evidence of structural Harrison’s 18th Ed 338 tubule injury is present in ATN. < 250 mg / day B.0 14 Renal blood flow constitutes what percentage of cardiac B. 25 % 21 Proteins of what size are freely filtered by the kidneys ? Harrison’s 18th Ed 338 Renal blood flow is approximately 25% of cardiac output. Eosinophils in urine suggest atheroembolic renal disease 23 Normal individuals excrete what amount of albumin in urine ? D. None of the above B. All of the above 22 Normal individuals excrete what amount of total protein in urine ? Stigmata of atheroemboli are livedo reticularis. Distal peripheral infarcts Smaller proteins (<20 kDa) are freely filtered but are readily reabsorbed by the proximal tubule. 33 Hypercoagulable state in nephrotic syndrome is due to urinary 27 Disease that allows “selective” loss of albumin is ? loss of ? Harrison’s 18th Ed 339 Harrison’s 18th Ed 338 A.15 D. AJAY MATHUR Cardiology 321 D. Tubule obstruction (cast nephropathy) C. 321 Nephrology MCQ’s FOR MCQ’s FOR MEDICAL MEDICAL PROFESSIONALS PROFESSIONALS BY PROF. D. hereditary nephritis. Cholecalciferol-binding protein D. all of which promote excessive renal salt & water 38 Isolated glomerular hematuria can occur in ? reabsorption. transferrin. Gross hematuria D. Hypertension D. 8 . 12 . All of the above Hematuria is defined as 2 to 5 RBCs per high-power field (HPF) & can be detected by dipstick. metal- binding proteins. 30 Mechanism of renal failure due to plasma cell dyscrasias is ? Harrison’s 18th Ed 339 36 Persistent or significant hematuria means ? Harrison’s 18th Ed 339 A. Hypertension. 5-8 B. Diabetes & FSGS lead to “nonselective” proteins S & C and fibrinogen. Membranoproliferative glomerulonephritis (MPGN) D. > 3 RBCs/HPF on three urinalyses B. ~ 4 million RBCs per day C. fibrinogen and IgG are lost. All of the above C. ~ 5 million RBCs per day D. Focal segmental glomerulosclerosis (FSGS) B. Renin-angiotensin system A. or a single urinalysis with >100 RBCs. IgA nephropathy Harrison’s 17th Ed 272 B. Urogenital neoplasms B. Mechanisms include activation of renin-angiotensin system. Persistent or significant hematuria means >3 RBCs/HPF on three urinalyses. and thin basement membrane disease. 2-5 A. All of the above . Harrison’s 18th Ed 339 32 In nephrotic syndrome. ~ 2 million RBCs per day A. Focal segmental glomerulosclerosis (FSGS) B. which of the following protein is lost ? A. tubule catabolism of filtered albumin. Fibrinogen D. 28 Urinary protein excretion of >3. antithrombin III. 31 Mechanisms of edema formation in hypoalbuminemia is the activation of ? 37 Isolated hematuria may be a feature of all except ? Harrison’s 18th Ed 339 Harrison’s 18th Ed 339 A. Minimal change disease D. Transferrin Common etiologies of isolated glomerular hematuria are IgA nephropathy. Diabetes B. thyroxine-binding globulin. Hereditary nephritis A. Hypercalciuria C. Antidiuretic hormone B. Proteins S & C C. Sympathetic nervous system C.12 C. Membranous glomerulopathy C. Hyperoxaluria D. & sympathetic nervous system. “Selective” loss of albumin is seen in minimal change disease. cholecalciferol-binding protein. Thyroxine-binding globulin C. All of the above Renal failure from plasma cell dyscrasias (multiple myeloma) occurs due to tubule obstruction (cast nephropathy) and light chain deposition. All of the above Hypercoagulable state in severe nephrotic syndrome is due to urinary losses of antithrombin III. or gross hematuria. Disruption of basement membrane & slit diaphragms B. Single urinalysis with >100 RBCs C. Diabetes A. Minimal change disease C. CRF cause increased tubular protein loss. Normal glomerular endothelial cell forms a barrier composed of pores of ~100 nm that hold back blood cells but allow passage of most proteins.5 grams/day can occur without 35 Hematuria is defined as how many RBCs per high-power field ? the features of nephrotic syndrome in ? Harrison’s 17th Ed 272 Harrison’s 18th Ed 338 A. Hyperuricosuria Urogenital neoplasms present as isolated painless hematuria with nondysmorphic RBCs. All of the above Normal red blood cell excretion is up to 2 million RBCs per day. Hypercalciuria Hypoalbuminemia in nephrotic syndrome is due to excessive urinary losses & increased proximal & hyperuricosuria are risk factors for unexplained isolated hematuria in children & adults. loss of all plasma proteins. 29 Urinary protein excretion of >3. Antithrombin III B. Thin basement membrane disease B. Edema forms from renal sodium retention & reduced plasma oncotic pressure. proteins S & C. Fusion of glomerular epithelial cell foot processes A. All of the above D. Amyloidosis C. antidiuretic hormone.5 grams/day can occur without 34 Normal red blood cell excretion in urine is ? Harrison’s 17th Ed 272 the features of nephrotic syndrome in ? Harrison’s 18th Ed 338 A. ~200 nm In nephrotic syndrome. All of the above D. ~ 3 million RBCs per day B. > 5 L/day A. Broad cast Harrison’s 18th Ed 340 Urinary sediment of ATN is filled with cellular debris & dark (muddy brown) granular casts.600 mosmol / day diagnostic of glomerulonephritis. Bartter’s syndrome. ~ 70 % . Cystic renal diseases B. All of the above Harrison’s 18th Ed 340 Hematuria with dysmorphic RBCs. ~ 60 % B. Normal individuals excrete < 150 mg/day of total protein C. and protein excretion >500 mg/day is virtually A. C. 600 . or a resolving ATN are salt-wasting disorders. Membranoproliferative glomerulonephritis (MPGN) D. Cellular debris In polyuria (>3 L/day) with dilute urine (<250 mosmol/L). degenerated cellular casts called waxy casts can be seen in the urine. Degenerated cellular casts 49 Which of the following is a “salt-wasting disorder” ? D.322 MCQ’s FOR MEDICAL PROFESSIONALS BY PROF. All of the above B.800 mosmol of solutes per day. B. Normal individuals excrete ~ 30 mg/day of albumin D. Resolving ATN A.1400 mosmol / day Harrison’s 17th Ed 273 D. Urea A. Protein excretion >500 mg/day 46 An average person excretes how much of solutes per day ? D. Polyuria is defined as urine output of > 3 L/day 51 Water comprises what percentage of body weight in men ? Oliguria refers to a 24-hour urine output of < 500 mL. the cause ca be polydipsia. Glucose B. All of the above B. 50 Water comprises what percentage of body weight in women ? 44 Which of the following statements is false ? Harrison’s 18th Ed 341 Harrison’s 16th Ed. ~ 60 % A. ~ 70 % C. B. Oval fat bodies Poorly reabsorbed solutes are glucose. Transplant rejection cause is ? D. > 2 L/day C. > 4 L/day Harrison’s 18th Ed 339 D. C. Oliguria refers to a 24-h urine output of < 400 mL B.800 mosmol / day 40 In urine. SLE A. RBC casts C. Nephrogenic diabetes insipidus Harrison’s 18th Ed 337 D. WBC’s &/or WBC casts are seen in ? C. A. B. or nephrogenic diabetes insipidus. Lipid bodies D. Harrison’s 18th Ed 341 45 Polyuria is defined as a 24-hour urine output of ? A. the C. mannitol or urea. Any of the above A. ~ 50 % A. Mannitol 42 Waxy casts refer to ? Harrison’s 18th Ed 339 C. Hematuria with dysmorphic RBCs Polyuria is defined as a 24-hour urine output of >3 L/day. 1400 . Pyelonephritis D. 200 . primarily as urea & electrolytes. RBC cast insipidus. All of the above Composition of body fluids Broad casts arise in dilated tubules of enlarged nephrons that have undergone compensatory hypertrophy in response to reduced renal mass as in chronic renal failure. ~ 80 % D. RBC casts. Waxy cast 48 Which of the following is a poorly reabsorbed solute ? D. A. Central diabetes insipidus 41 Urinary sediment of ATN has which of the following casts ? C. Chronic renal failure Cystic renal diseases.1800 mosmol / day A. > 3 L/day glomerulonephritis ? C. Polydipsia and transplant rejection. 250 A. central diabetes B. All of the above Harrison’s 18th Ed 340 WBC’s &/or WBC casts in urine are seen in tubulointerstitial diseases like interstitial nephritis. 800 . C. Systemic lupus erythematosus 47 In polyuria (>3 L/day) with dilute urine (<250 mosmol/L). Bartter’s syndrome 43 Broad casts are diagnostic of ? Harrison’s 18th Ed 339 C. AJAY MATHUR Nephrology 39 Which of the following urinary feature is diagnostic of B. Interstitial nephritis Average person excretes 600 . Any of the above Harrison’s 18th Ed 340 In chronic renal diseases. B. ~ 50 % Harrison’s 18th Ed 340 B. glutamate. 57 Which of the following is a reflection of ECF volume ? 64 Average osmotic threshold for thirst is approximately ? Harrison’s 17th Ed 275 Harrison’s 18th Ed 342 A. All of the above fluid (ECF) ? Solutes like urea do not contribute to water shift across cell membranes and are known as Harrison’s 18th Ed 341 ineffective osmoles. Total body Na + A. 15 . ECF osmolality < ICF osmolality C. Harrison’s 18th Ed 341 59 Which of the following is known as “ineffective osmole” ? A.55 % A. 75 . D.. Decrease in blood pressure C. 58 A change in ICF osmolality is due to a change in ? 65 Arginine vasopressin is synthesized in ? Harrison’s 18th Ed 341 Harrison’s 17th Ed 275 A. located in the anterolateral hypothalamus. 62 Thirst is mediated by ? 55 Which of the following is correct ? Harrison’s 18th Ed 341 Harrison’s 18th Ed 341 A. 55 . 293 mosmol/kg D.45 % Harrison’s 18th Ed 342 C. myo-inositol & taurine. 500 mosmol/kg D. Hypothalamus . Total body Cl . 1:2 A. Betaine B. C. A. 15 . 55 . 291 mosmol/kg C. Decrease in ECF volume B. ICF Na + content Water comprises ~50% of body weight in women and ~60% in men.35 % Harrison’s 18th Ed 341 B. ~ 80 % B. C. also called osmolytes are creatine. ECF osmolality > ICF osmolality D. 150 mosmol/kg D. ECF osmolality = ICF osmolality B. All of the above D. 3:1 D. 1000 mosmol/kg D. All of the above Osmoreceptors. are stimulated by a rise in tonicity. 100 mosmol/kg C. 200 mosmol/kg Ratio of intravascular (plasma water) & extravascular (interstitial) ECF water is 1 : 3. C. 1:3 C. 50 mosmol/kg B. or a decrease in ECF volume or blood pressure. Posterior pituitary B. 295 mosmol/kg Average osmotic threshold for thirst is ~ 295 mosmol/kg & varies among individuals. ICF water content A. 25 . Creatine C. All of the above D. Total body Na+ content reflects ECF volume as it is restricted to extracellular compartment. 289 mosmol/kg B. 45 . Ineffective osmoles like urea & glucose do not play a role in stimulating thirst. Any of the above Primary stimulus for water ingestion is thirst mediated either by an increase in effective osmolality Osmotic equilibrium refers to ECF osmolality = ICF osmolality.content 52 What percentage of total body water is found as intracellular fluid (ICF) ? A change in ICF osmolality is usually due to a change in ICF water content.55 % A.95 % C. betaine. AJAY MATHUR Cardiology 323 D. 800 mosmol/kg 55-75% of body water is present as intracellular fluid (ICF) & 25-45% as extracellular fluid (ECF). 35 . 323 Nephrology MCQ’s FOR MCQ’s FOR MEDICAL MEDICAL PROFESSIONALS PROFESSIONALS BY PROF. 2:1 B. Increase in effective osmolality A.25 % 60 Maximal urine osmolality can be ? B. Total body HCO3.75 % B. Anterolateral hypothalamus C. 1200 mosmol/kg 54 Ratio of water in intravascular and extravascular spaces is ? Harrison’s 18th Ed 341 61 Minimum urine osmolality in humans can be ? Harrison’s 18th Ed 342 A. Organic solutes. 56 Which of the following is an ‘osmolyte’ ? 63 ‘Osmoreceptors’ are located in ? Harrison’s 18th Ed 347 Harrison’s 17th Ed 275 A.75 % B. Posteromedial hypothalamus D. ICF Cl. Inositol C. Urea 53 What percentage of total body water is found as extracellular D. Total body K+ B. Phospholipids D. Myo-inositol A. ICF K+ content D. Effective arterial volume D.500 mL / day Harrison’s 17th Ed 275 Fecal fluid loss is 100 . Thick ascending limb of the loop of Henle (TALH) AVP binds to V2 receptors on basolateral membrane of principal cells in the collecting duct activating the water channels which are encoded by aquaporin-2 gene.200 mL/day. V2 receptor sensitive Na+ . ~ 7 liters A.200 mL / day nausea. 100 . 5% occurs in DCT mediated by thiazide-sensitive Na+-Cl– A. 10 to 20 mmol/L B. . Aquaporin-3 gene Harrison’s 18th Ed 342 D.2Cl- B. hypoglycemia. C. All of the above A. 260 to 270 mosmol/kg co-transporter. U2 receptor 74 Reabsorption of filtered sodium mediated by ‘thiazide - B. 2 by ingestion & 7 liters by secretion. sodium concentration of sweat is ? C. Proximal convoluted tubule B. Anterior pituitary gland C. Hypothalamus Tubule Na reabsorption. GFR D. Tubule water reabsorption Harrison’s 18th Ed 341 D. 270 to 280 mosmol/kg C. All of the above B. 25-30% occurs in TALH via apical Na+-K+-2Cl–co-transporter. All of the above Harrison’s 18th Ed 343 Nonosmotic factors that regulate AVP secretion include effective circulating (arterial) volume. X2 receptor A. 200 . W2 receptor Harrison’s 18th Ed 342 D. stress. Posterior pituitary gland A. ~ 5 liters 70 Nonosmotic factors that regulate AVP secretion include ? Harrison’s 17th Ed 275 C.Cl. All of the above Harrison’s 18th Ed 342 A. Cortical & medullary collecting ducts A. Distal convoluted tubule 67 Receptor to which AVP binds in renal collecting ducts is ? Harrison’s 18th Ed 341 D. Posterior pituitary gland 73 Majority of filtered sodium is reabsorbed in ? D. pregnancy & numerous drugs. pain.324 MCQ’s FOR MEDICAL PROFESSIONALS BY PROF. Aquaporin-1 gene 75 Reabsorption of filtered sodium mediated by ‘Na+ . Cortical and medullary collecting ducts A. Distal convoluted tubule encoded by ? Harrison’s 18th Ed 341 D. Aquaporin-4 gene A. ~ 250 mmol Harrison’s 17th Ed 276 D. A. Proximal convoluted tubule B. C. + B. Distal convoluted tubule 69 Osmotic threshold for AVP release is ? D. Thick ascending limb of loop of Henle (TALH) 68 Water channels in collecting ducts activated by AVP are C. Aquaporin-2 gene cotransporter’ occurs in ? C. Hypoglycemia 77 Fecal fluid loss is ? D. Cortical and medullary collecting ducts Harrison’s 18th Ed 341 Almost two-thirds of filtered Na+ is reabsorbed in the proximal convoluted tubule. ~ 150 mmol 78 Normally.cotransporter’ occurs in ? C. ~ 500 mmol A. Thick ascending limb of loop of Henle C.K+ .2 % range. A. 400 . Proximal convoluted tubule Arginine vasopressin (AVP) or antidiuretic hormone (ADH) is a polypeptide synthesized in supraoptic & paraventricular nuclei of hypothalamus & secreted by posterior pituitary gland. AJAY MATHUR Nephrology B. and not GFR. 300 . 280 to 290 mosmol/kg 76 What quantity of fluid enters gastrointestinal tract daily as secretions ? D. Na+ reabsorption also occurs in cortical and medullary collecting ducts. Tubule Na + reabsorption 66 Arginine vasopressin (AVP) is secreted by ? C. Pregnancy ~ 9 liters of fluid enters GI tract daily.300 mL / day 71 Individuals eating a typical western diet consume what amount C. ~ 9 liters B. A. is the major regulatory mechanism controlling Na+ excretion. 20 to 50 mmol/L Individuals eating a typical western diet consume ~150 mmol of NaCl daily. B. 290 to 300 mosmol/kg Harrison’s 18th Ed 343 Osmotic threshold for AVP release is 280-290 mosmol/kg. B.400 mL / day of NaCl daily ? D. It maintains plasma osmolality within a 1 . Anterior pituitary gland 72 Major regulatory mechanism controlling sodium excretion is ? Harrison’s 18th Ed 342 C. ~ 3 liters B. ~ 50 mmol B. B. Pheochromocytoma 81 An increased BUN:creatinine ratio may be due to ? Adrenal insufficiency & hypothyroidism may cause hyponatremia. seizures. 20 liter / day Normally. Beer potomania C. ~ 450 mosmol/day B. GI bleeding 87 Renal excretory capacity is ? Harrison’s 17th Ed 277 C. the BUN:creatinine ratio is about ? Syndrome of inappropriate antidiuretic hormone secretion (SIADH) is the most common cause of normovolemic or euvolemic hyponatremia. [Na+] x [K+] C. Aldosterone antagonists D. Prerenal azotemia B. 1. 20:1 B. 18 liter / day D.4 mmol/L for every 100 mg/dL rise in plasma glucose concentration. [Na ] x [K ] ÷ [Cl ] + + . Plasma Na falls by 1.8 mmol / L Osmotic diuresis refers to a solute excretion rate of > ~750 mosmol/day. 79 Enhanced reabsorption of Na+ by collecting duct occurs due to ? Harrison’s 18th Ed 342 85 ‘Syndrome of inappropriate antidiuretic hormone secretion’ A. 10:1 Harrison’s 18th Ed 345 C. + 89 Phenomenon of ‘hyponatremia’ in beer drinkers is called ? Harrison’s 18th Ed 346 83 Urine tonicity is calculated by ? Harrison’s 17th Ed 277 A. 50 to 100 mmol/L C. Any of the above 80 Normally. C. 84 ‘Diuretic-induced hyponatremia’ is almost always due to ? 90 Stupor. increased urea production as in hyperalimentation (high-protein).6 mmol / L D. Suppressed atrial natriuretic peptide secretion A. Harrison’s 17th Ed 276 A. glucocorticoid therapy. + + potomania. 120 to 150 mmol/L D. 325 Nephrology MCQ’s FOR MCQ’s FOR MEDICAL MEDICAL PROFESSIONALS PROFESSIONALS BY PROF.2 mmol / L B. Normovolemic hyponatremia D. the loss of water exceeds that of Na+. Cushing’s disease D. C. Increased aldosterone (SIADH) causes ? B. AJAY MATHUR Cardiology 325 C. 5:1 86 Which of the following conditions can cause hyponatremia ? B. Beer flush B. All of the above B. 1. which may exceed renal excretory capacity and result in hyponatremia. Hypothyroidism decreases cardiac Harrison’s 18th Ed 344 output and GFR and thus increased AVP secretion leading to hyponatremia. D. ~ 250 mosmol/day A. Loop diuretics decrease tonicity of medullary interstitium & impair maximal urinary concentrating capacity which limits ability of AVP to promote water retention. Hypothyroidism D. increased C.4 mmol / L C. depletion & AVP-mediated water retention. 15:1 A. [Na ] + [K ] . Beer psychosis D. All of the above B. 15 liter / day Increased BUN (relative to creatinine) may be due to prerenal azotemia. A. D. the BUN:creatinine ratio is about 10:1. ~ 750 mosmol/day D. 1. Increased AVP secretion Harrison’s 18th Ed 345 C.[Cl ] + + . Loop diuretics A. and gastrointestinal bleeding. Hypervolemic hyponatremia AVP secretion & suppressed atrial natriuretic peptide secretion. Hypovolemic hyponatremia Enhanced reabsorption of Na+ by collecting duct is in response to increased aldosterone. and coma occur when plasma sodium Harrison’s 18th Ed 344 concentration falls acutely below ? A. 123 mmol/L . 1. [Na ] + [K ] + + Beer drinkers have poor dietary intake of protein & electrolytes & consume large volumes of beer. Beer syndrome A. Glucocorticoid therapy A. 12 liter / day D. Hyponatremia 88 ‘Osmotic diuresis’ is defined as a solute excretion rate of more than ? 82 Plasma sodium concentration falls by how much for every 100 Harrison’s 18th Ed 347 mg/dL rise in plasma glucose concentration ? Harrison’s 17th Ed 277 A. Hyperthyroidism Normally. renal excretory capacity is large of 12 liters per day. ~ 550 mosmol/day C. B. All of the above Na concentration of sweat is 20-50 mmol/L and decreases with profuse sweating due to the action + Diuretic-induced hyponatremia is almost always due to thiazide diuretics which lead to Na + & K + of aldosterone. Since sweat is hypotonic. This is referred to as beer Urine tonicity is calculated as the sum of concentrations of Na and K . Thiazide diuretics Harrison’s 17th Ed 277 B. Naptans U A. adrenal & thyroid function & usually have normal K+ & acid-base balance.371:1624-32 A. Three arginine-vasopressin-receptor subtypes A.326 MCQ’s FOR MEDICAL PROFESSIONALS BY PROF. treatment with diuretics & Harrison’s 18th Ed 348 SIADH. Hypoosmotic hyponatremia V1a. C. Patients are typically normovolemic & have normal Na + balance. All of the above A. . None of the above 9 99 V(1A) receptors located in ? SIADH is characterized by hypoosmotic hyponatremia in the setting of an inappropriately concentrated Lancet 2008. Collecting tubules Lancet 2008. 6% to 8% B. SIADH 91 The finding of urine sodium concentration >20 mmol/L in D. glycerophosphoryl- choline.371:1624-32 D. hypovolemic patients C. Normal potassium and acid-base balance - Vaptans are orally & intravenously active non-peptide vasopressin receptor antagonists. cirrhosis liver. They tend to be mildly volume-expanded secondary to water retention & have a urine Na+ A. Vascular smooth muscle cells & myocardium 95 Physiologic functions of AVP include ? B. hypoaldosteronism or vomiting. All of the above renal-collecting-duct system & mediate free water absorption. 2007. V1b. Hypervolemia ti e D. By definition. Collecting tubules in SIADH is due to uricosuric state induced by volume expansion. Salt-wasting nephropathy B. whereas V2 receptors are linked to adenylate cyclase signaling pathway.371:1624-32 A. mediated primarily by V(1A) and V(2) receptors. they have i normal renal. Normal sodium balance 98 Non-peptide vasopressin receptor antagonists are called ? Harrison’s 18th Ed 349 Patients with SIADH are usually euvolemic. D. Cardiovascular & renal effects of AVP are Physiologic functions of AVP or antidiuretic hormone include contraction of vascular smooth muscle. V2 R 92 Which of the following statements about SIADH is false ? D. 122 mmol/L 96 Inappropriately elevated levels of AVP are found in ? Drugs.371:1624-32 B. glutamine. A. V(2) receptors located in principal cells of D. Congestive heart failure D. AJAY MATHUR Nephrology B. C. V1b diuretic therapy. betaine.371:1624-32 9 urine (urine osmolality >100 mosmol/kg). D. All of the above hypovolemic hyponatremia implies ? Inappropriately elevated levels of AVP occur in CHF. Captans n 93 Which of the following statements about SIADH is false ? B. Collecting tubules D. All of the above ta 94 What percentage of increase in brain volume can be fatal ? A. Vaptans Harrison’s 18th Ed 347 C. Hypoaldosteronism Lancet 2008. regulation of corticotropin release and renal antidiuresis. with intracellular calcium acting as the B. 121 mmol/L A. and V2 belong to the large rhodopsin-like G-protein-coupled receptor family. 4% to 6% A. water diffuses into brain by osmosis causing dilution of extracellular fluid in brain & swelling of brain cells. Vascular smooth muscle cells & myocardium C. Normal renal. Cirrhosis of liver Stupor. All of the above Harrison’s 18th Ed 347 V Arginine vasopressin (AVP) is a neuropeptide hormone.67(6):847-58 C. creatine & taurine to restore osmolality of cerebral ECF & limiting swelling of brain cells. In contrast. An increase in brain volume of 8% to 10% can be fatal. AVP receptor C. Saptans B. Anterior pituitary h tend to be hyperuricemic secondary to increased proximal urate reabsorption. Regulation of corticotropin release Activation of V(1A) receptors located in vascular smooth muscle cells & myocardium results in vasoconstriction & increased afterload & hypertrophy. Anterior pituitary B. C. V1A & V1B receptors are linked to phosphoinositol signaling pathway. V1a G Urine Na concentration >20 mmol/L in hypovolemic hyponatremia implies a salt-wasting nephropathy. and thyroid function D. adrenal. All of the above osmoregulatory (adaptive) mechanism brain cells pump out myoinositol. V(1B) receptors are located in anterior pituitary & mediate adrenocorticotropin hormone release. Stimulation of liver glycogenolysis D. All of the above C. stimulation of liver glycogenolysis. 8% to 10% C. By cerebral D. Diuretic therapy 97 Which of the following is a arginine-vasopressin-receptor subtype ? C. Contraction of vascular smooth muscle C. seizures & coma occur when plasma Na+ concentration falls acutely below 120 mmol/L. Anterior pituitary If plasma is dilute. Acute hyponatremia may cause cerebral edema due to underlying osmotic interactions 101 V(1B) receptors located in ? between brain cells and the extracellular fluid compartment. A. + B. an effect termed aquaresis. Vascular smooth muscle cells & myocardium r excretion rate equal to intake (urine Na+ concentration usually >40 mmol/L). Lancet 2008. Hypouricemia B. Urine osmolality >100 mosmol/kg d second messenger. Hypouricemia antagonists induces free water diuresis without natriuresis or kaliuresis. with intracellular cyclic adenosine monophosphate (cAMP) acting as second messenger. 120 mmol/L B. 2% to 4% 100 V(2) receptors located in ? Lancet 2008. 371:1624-32 D. Conivaptan by multiplying the deficit in plasma Na+ concentration by total body water. Mozavaptan B. 77 mEq/L 107 Vaptans are metabolised by which of the following hepatic B. CYP3A4 Harrison’s 18th Ed 349 D. Euvolaemic hyponatraemia Lancet 2008. Iso-osmolar & isotonic hyponatremia These are effective in the treatment of euvolaemic & hypervolaemic hyponatraemia. N Engl J Med 2000. High A. Increase in solid phase of plasma Lancet 2008. C. Faeces B.371:1624-32 C. Tolvaptan D. Lixivaptan is a Benzodiazepine derivative. 256 mEq/L Lancet 2008. whereas the other three agents are for Harrison’s 17th Ed 278 oral administration. or high blood tonicity. Weight 106 Which of the following is a Benzodiazepine derivative ? B. Tolvaptan hypertriglyceridemia or paraproteinemia increases substantially the solid phase of plasma. 154 mEq/L cytochrome enzymes ? C. 513 mEq/L A. Tolvaptan Quantity of Na required to increase plasma Na+ concentration by a given amount can be estimated + C. Conivaptan D.342:1581 dysmenorrhoea. Urine output in liters B.371:1624-32 Lancet 2008.9% (normal. Harrison’s 18th Ed 347 V2-receptor antagonists .mozavaptan. Lixivaptan 110 Blood tonicity associated with hyponatremia is ? Relcovaptan is a selective V1a-receptor antagonist used in the treatment of Raynaud's disease. D. induce a highly hypotonic diuresis without substantially affecting excretion of electrolytes (by contrast with effects of diuretics).342:1581. and tocolysis. isotonic) saline is ? Tolvaptan.371:1624-32 C. Tolvaptan C. normal. D. A. since it is an aquaretic. B. All of the above D. plasma Na+ concentration during the first 24 hours should not be more than ? C.371:1624-32 D. A. Height Lancet 2008.371:1624-32 103 Which of the following is a selective V(1A) receptor antagonist A. CYP2C9 114 In hyponatremia. following conditions ? Lancet 2008. Tolvaptan Conivaptan & other V2-receptor antagonists are contraindicated in patients with hypovolaemic C. A. All of the above 111 Which of the following about pseudohyponatremia is false ? N Engl J Med 2000. Mozavaptan D. Relcovaptan hyponatraemia. Hypervolaemic hyponatraemia A. lixivaptan. Lixivaptan 109 V2-receptor antagonists are contraindicated in which of the Conivaptan is a combined V(1A) / V(2)-receptor antagonist (non-selective) that induces diuresis as well as haemodynamic improvement. CYP2C19 A. Lungs C. Normal Lancet 2008. 6 mmol/L B. satavaptan & tolvaptan. AJAY MATHUR Cardiology 327 102 Which of the following is a combined V(1A) / V(2)-receptor 108 Vaptans are excreted through ? antagonist ? Lancet 2008.371:1624-32 C. Severe hypertriglyceridemia or paraproteinemia 105 Which of the following is administered intravenously ? C. Mozavaptan & Conivaptan are benzazepine derivative. Mozavaptan D. Urine A. Conivaptan 112 Sodium required to correct hyponatremia can be estimated by D. Mozavaptan D. 8 mmol/L . Total body water A. Lixivaptan 113 Quantity of sodium in 0. 327 Nephrology MCQ’s FOR MCQ’s FOR MEDICAL MEDICAL PROFESSIONALS PROFESSIONALS BY PROF. CYP2D6 B. A. Satavaptan Hyponatremia can be associated with low. None of the above A.371:1624-32 A. C. Hypovolaemic hyponatraemia ? B. None of the above B. Any of the above B. Satavaptan is N-arylsulfonyl Harrison’s 18th Ed 344 oxindole derivative. Lixivaptan multiplying the deficit in plasma sodium concentration by ? Conivaptan is available for intravenous (IV) administration. Low 104 Which of the following is a V2-receptor antagonist ? B. Mozavaptan Pseudohyponatremia is a spurious form of iso-osmolar & isotonic hyponatremia when severe B. C. Results in ICF volume contraction C. Cholera D. Prior cerebral anoxic injury A. urine osmolality should increase by how much in CDI ? Hypernatremia is defined as a plasma Na + concentration >145 mmol/L. hypercalcemia & hyperkalemia cause sporadic NDI r D. All of the above due to mutation in autosomal aquaporin-2 gene 9 Besides. 135 mmol/L D. Hypernatremia 125 ‘Solute excretion rate’ is ? Harrison’s 17th Ed 279 119 Hypernatremia is defined as a plasma sodium concentration A. prepubescent children. Harrison’s 17th Ed 280 . n 117 Risk factors for ODS include ? 123 Which of the following about nephrogenic diabetes insipidus is false ? U Harrison’s 18th Ed 347 A. Congenital Nephrogenic diabetes insipidus (NDI) is X-linked recessive trait due to mutations in V2 receptor gene C. A. D. None of the above A. 420 mmol 121 Which of the following is not a secretory diarrhea ? B. Disruption of BBB results in an influx of activated complement into brain which is toxic to C. All of the above pseudobulbar palsy & spastic quadriparesis. hypercalcemia and hypokalemia can cause sporadic NDI. 10 mmol/L 120 Which of the following about hypernatremia is false ? Harrison’s 17th Ed 279 D. All of the above D. weakness. Cerebral dehydration 124 Major symptoms of hypernatremia are ? C. 150 mmol/L 126 After administering ‘desmopressin’. Urine volume x Urine sodium Harrison’s 18th Ed 349 C. hypokalemia. C. A. Carcinoid To raise plasma Na+ concentration from 110 to 120 mmol/L in a 70-kg man requires 420 mmol [(120 – C. Congenital Nephrogenic diabetes insipidus (NDI) may be 9 D. 140 mmol/L The product of urine volume & osmolality is called solute excretion rate. VIPoma 116 Osmotic demyelination syndrome (ODS) includes ? R Harrison’s 18th Ed 347 122 Which of the following about diabetes insipidus is false ? Harrison’s 18th Ed 350 A. focal neurologic deficits & coma or seizures. B. rapid or overcorrection of hyponatremia. 145 mmol/L D. to raise plasma Na+ concentration from 110 to 120 mmol/L. Lactulose induced G 110)x70x0. may develop NDI due i 118 Which of the following is related to central pontine myelinolysis ? to excessive vasopressinase from placenta h Harrison’s 17th Ed 278 Lithium. Familial CDI is due to mutation in propressophysin gene If hyponatremia is corrected too rapidly. in II or III trimester. The classic presentation is D. Total body water is 50 & 60% of lean body weight in women or men respectively. plasma Na+ concentration during the first 24 hours should not be more than 12 mmol/L and less than 6 mmol/L/day thereafter. 480 mmol B. Dysphagia C. 440 mmol Harrison’s 17th Ed 279 C. NDI results from renal resistance to actions of AVP d C. Familial CDI is inherited as autosomal recessive ti e D. propressophysin (AVP precursor) gene. Cerebral atrophy Harrison’s 18th Ed 350 D. State of iso-osmolality In hyponatremia. Cerebral oedema ta B. risk factors for ODS include prior cerebral anoxic injury. dysarthria and dysphagia (central pontine myelinolysis). and malnutrition. Respiratory oligodendrocytes that produce and maintain myelin in CNS. 460 mmol A. Stimulates thirst 115 In a 70 kg man. Flaccid paralysis V A. Pregnant women. Hypokalemia B.6] of Na+. Rapid or overcorrection of hyponatremia Harrison’s 18th Ed 350 - B. Urine volume x Urine potassium A. 12 mmol/L A. Excretion of minimum volume of maximally concentrated urine Harrison’s 18th Ed 349 Hypernatremia is a state of hyperosmolality. CDI is characterized by impaired AVP secretion B. neuromuscular irritability. Dysarthria B. Cardiac B. Clinical symptoms of central pontine myelinolysis do not occur until a few days after rapid Na + correction. Major symptoms of hypernatremia are neurologic like altered mental status. Lithium. Urine volume x Osmolality more than ? B. menstruating women. Urine volume x Urine bicarbonate B. Patients Familial form of CDI is inherited as autosomal dominant and is attributed to mutations in with chronic hyponatremia are most susceptible to development of ODS. AJAY MATHUR Nephrology C. Neurologic Acute brain dehydration produced by rapid correction disrupts the tight junctions of blood-brain barrier.328 MCQ’s FOR MEDICAL PROFESSIONALS BY PROF. what is the amount of Na+ required ? D. osmotic demyelination syndrome (ODS) develops which is characterized by flaccid paralysis. High renin A. Proximal convoluted tubule D. and the passive outward diffusion of K+ is quantitatively the most important factor that generates the A. ratio of ICF to ECF K+ concentration is 38:1 due to resting membrane potential and is crucial for normal neuromuscular function. A. B. Clofibrate 135 Potassium delivery to distal nephron approximates ? C. ratio of ICF to ECF K+ concentration is ? Harrison’s 17th Ed 280 B.72 hours. Carbamazepine Harrison’s 17th Ed 280 D. Phenytoin A. AJAY MATHUR Cardiology 329 A.120 mmol/day C. Chlorpropamide potassium balance occurs in ? Harrison’s 17th Ed 280 C. urine osmolality should D. At least 50 % C.40 mmol/day In hypernatremia due to water loss. Slowly over at least 48 to 72 hours A. 2:3 careful water restriction. All of the above D.5. 50 mmol/L B. or approximately 1 mmol/kg per day. 28:1 C. + Harrison’s 18th Ed 350 A. B. L.550 mmol/day Harrison’s 18th Ed 346 Table 45-1 K+ intake in an average western diet is 40 .5 . Chlorpropamide of which is absorbed by the gastrointestinal tract.4500 mmol) is in ECF. B. Carbamazepine B. Clofibrate A. D. Daily requirement have to take lithium ? K delivery to the distal nephron [DCT + cortical collecting duct (CCD)] approximates dietary intake. 90% A. At least 25 % A. Dietary deficiency 130 Which of the following drugs is useful in NDI patients who D. Dietary excess Drugs that either stimulate AVP secretion or enhance its action on kidney are chlorpropamide. At least 10% B. At least 35% Harrison’s 17th Ed 280 D. High angiotensin II B. B. 38:1 D. .0 mmol/L. Over 12 to 24 hours 134 K+ intake in an average western diet is ? C. Nephrotoxicity of lithium requires it to be C. water deficit should be D.120 mmol/day. 1:2 CDI & NDI can be distinguished by administering AVP analogue desmopressin (10 µg intranasally) after B. Rapidly resting membrane potential. Loop of Henle Amiloride is useful NDI patients who have to take lithium. At least 10 % 132 Normally. Hyperkalemia C. 48:1 127 After administering ‘desmopressin’. 10 . water deficit should be corrected slowly over 48 . carbamazepine and NSAIDs. 150 mmol/L D. 200 mmol/L Aldosterone is secreted by the zona glomerulosa cells of the adrenal cortex in response to high renin Normal plasma K+ concentration is 3. Distal nephron taken up into collecting duct cells via amiloride-sensitive Na + channel. 350 . <2% of total body K+ content (2500 . 3:4 128 In hypernatremia due to water loss. 329 Nephrology MCQ’s FOR MCQ’s FOR MEDICAL MEDICAL PROFESSIONALS PROFESSIONALS BY PROF.350 mmol/day 129 Drugs that either stimulate AVP secretion or enhance its action on the kidney includes all except ? D. At least 35 % B. Hypokalemia 137 Aldosterone is secreted by zona glomerulosa cells of adrenal cortex in response to ? 131 Normal plasma potassium concentration inside cells is about ? Harrison’s 17th Ed 280 Harrison’s 17th Ed 280 A. K+-ATPase pump actively transports K+ in and Na+ out of the cell in a 2:3 ratio. 100 mmol/L C. All of the above Virtually all regulation of renal K+ excretion & total body K+ balance occurs in distal nephron. Over 24 to 36 hours Harrison’s 17th Ed 280 D. 120 . whereas that inside cells is about 150 mmol/ and angiotensin II or hyperkalemia. Amiloride 136 All regulation of renal potassium excretion & total body B. 4:5 corrected in which of the following ways ? Harrison’s 18th Ed 350 The basolateral Na+. C. 40 . Urine osmolality should increase by at least 50% in CDI & will not change in NDI. None of the above A. 58:1 increase by how much in NDI ? Harrison’s 17th Ed 280 Normally. Dietary intake clofibrate. At least 25% 133 The basolateral Na-K ATPase pump actively transports potassium in & sodium out of the cell in a ratio of ? C. C. B. Uncontrolled hyperglycemia 148 ‘Syndrome of apparent mineralocorticoid excess’ is due to r deficiency of ? i D.8 mmol/L A. Ingestion of excess proteins aldosterone secretion by ? C. Excessive sweating C. Adrenal adenoma B.10 mmol/L and is a less likely cause of hypokalemia. All of the above Harrison’s 17th Ed 281 Renin-secreting tumors of juxtaglomerular apparatus. 11-HSDH ta A. A+B 139 Which of the following is a cause of decreased potassium D. Ingestion of excess fats Harrison’s 17th Ed 281 R D. B. < 3. and 9 A.6 mmol/L. both promote kaliuresis. Treatment of DKA with insulin causes hypokalemia due movement of K + inside the cells with glucose like anabolic states. Loss of gastric secretions C. Hyperaldosteronism D. Volume depletion Hypokalemia is defined as a plasma K+ concentration <3. 12-HSDH C. 11 -HSDH D. All of the above deficiency or suppression allows cortisol to bind to the aldosterone receptor & leads to syndrome of apparent mineralocorticoid excess. Treatment of DKA with insulin 9 C. None of the above Harrison’s 18th Ed 351 K concentration of gastric fluid is only 5 . All of the above 150 Which of the following is false about ‘Liddle’s syndrome’ ? 144 Which of the following is the least likely cause of hypokalemia ? Harrison’s 17th Ed 281 Harrison’s 17th Ed 281 A. C. All of the above Harrison’s 18th Ed 352. D. Autosomal dominant A. 11-HSDH Harrison’s 17th Ed 281 B. A. All of the above Harrison’s 17th Ed 281 h 142 Which of the following is a cause of hypokalemia ? A. Uncontrolled hyperglycemia causes K + depletion due to osmotic 149 -HSDH’ include ? Conditions that inhibit the activity of ‘11 diuresis. Metabolic acidosis (Conn’s syndrome) or carcinoma or to adrenocortical hyperplasia. Laxative abuse of ? D. + A. All of the above C.5 mmol/L B. Adrenal carcinoma V Ingestion of clay (geophagia) is a cause of decreased K+ intake as it binds dietary K+ & iron. renal cell carcinoma. Massive transfusion with thawed washed RBCs Cortisol is converted to cortisone by 11 -hydroxysteroid dehydrogenase (11  -HSDH). Metabolic alkalosis Wilms’ tumor produce renin. AJAY MATHUR Nephrology 138 Hypokalemia is defined as a plasma potassium concentration C. Respiratory alkalosis A. ovarian carcinoma. Wilms’ tumor - 141 Which of the following is a cause of hypokalemia ? D. Renal cell carcinoma U Metabolic alkalosis is associated with hypokalemia due to K+ redistribution as well as excessive renal K+ loss. B. Hypokalemic metabolic alkalosis . < 3. < 3. B. Respiratory acidosis 147 Tumor that produce renin include ? n Harrison’s 17th Ed 281 D.7 mmol/L Harrison’s 17th Ed 281 D. Laxative abuse Glycyrrhetinic acid (in licorice). nor B intake ? Harrison’s 17th Ed 281 Loss of gastric contents results in volume depletion & metabolic alkalosis. Table 45-4 ti e Primary hyperaldosteronism is due to dysregulated aldosterone secretion by an adrenal adenoma A. Patients of neutropenia treated with GM-CSF D. Metabolic alkalosis C. Patients of pernicious anemia treated with vitamin B12 C. Metabolic alkalosis C. Glycyrrhetinic acid in licorice Harrison’s 17th Ed 281 B. 12-HSDH B. Neither A.6 mmol/L 145 Hypokalemia due to loss of gastric contents is due to ? C. Excessive sweating B. Adrenocortical hyperplasia d 140 Hypokalemia is associated frequently with ? D.330 MCQ’s FOR MEDICAL PROFESSIONALS BY PROF. Chewing tobacco A. Carbenoxolone B. Ingestion of clay (geophagia) A. Harrison’s 17th Ed 281 143 Which of the following is a cause of hypokalemia ? A. Ingestion of excess carbohydrates 146 Primary hyperaldosteronism is due to dysregulated G B. Hypertension B. chewing tobacco & carbenoxolone inhibit activity of 11 -HSDH. Ovarian carcinoma C. < 3. D. Diuretic use A. 1 Harrison’s 17th Ed 283 B. Hypocalcemia A. Potassium bicarbonate D. 50 to 100 mmol C. AJAY MATHUR Cardiology 331 D. It is generally safer to correct hypokalemia via the oral route. Prominent U wave Harrison’s 17th Ed 283 B. D. Any of the above B. 200 to 400 mmol 152 Which of the following laboratory abnormalities is seen only A decrement of 1 mmol/L in plasma K+ concentration reflects total body K+ deficit of 200 . Harrison’s 17th Ed 282 A. 155 Hypokalemia with minimal renal potassium excretion suggests Potassium bicarbonate and citrate (metabolized to HCO3–) tend to alkalinize and is more appropriate for hypokalemia associated with chronic diarrhea or RTA. metabolic alkalosis. Early changes include flattening or inversion of T wave. Radiographic evidence of renal osteodystrophy C. TTKG is the ratio of K+ concentration in lumen of CCD ([K+]CCD) to that in peritubular capillaries or plasma ([K+]P). Potassium chloride is preferred for more rapid correction of hypokalemia and metabolic alkalosis. 100 to 200 mmol Bartter’s syndrome is characterized by hypokalemia. hyperreninemic hyperaldosteronism secondary to ECF volume contraction & juxtaglomerular apparatus hyperplasia. 80 mmol/hour 156 Hypokalemia with Transtubular K + concentration gradient Rate of K + infusion should not exceed 20 mmol/hour unless paralysis or malignant ventricular arrhythmias are present. 20 mmol/L . Delayed ventricular repolarization 159 It is generally safer to correct hypokalemia via which route ? Harrison’s 17th Ed 283 B. 100 mmol C. prominent D. 60 mmol/hour via the skin or gastrointestinal tract or that there is a remote history of vomiting or diuretic use. Skin not exceed ? B. Inversion of T wave A. 247 158 Plasma potassium levels of < 3 mmol/L require how much potassium to correct the deficit ? A. Hyperphosphatemia B. 331 Nephrology MCQ’s FOR MCQ’s FOR MEDICAL MEDICAL PROFESSIONALS PROFESSIONALS BY PROF. Hyperreninemic hyperaldosteronism A. and prolonged QU interval. 154 Which of the following ECG changes denote severe K+ depletion ? Harrison’s 17th Ed 282 160 Which is the preparation of choice for correction of hypokalemia with metabolic alkalosis? A.0 mmol/L require >600 mmol of K + to correct the deficit. that potassium loss is through ? Harrison’s 17th Ed 282 161 Rate of IV infusion of potassium in severe hypokalemia should A. Peripheral vein D. 3 Liddle’s syndrome is an autosomal dominant disease characterized by hypertension. Excess renin and aldosterone secretion C. Per rectum U wave. Potassium chloride C. Prolonged PR interval B. Gastrointestinal tract Harrison’s 17th Ed 283 C. 400 mmol 153 Which of the following is false regarding ECG changes of D. Anemia Harrison’s 17th Ed 283 B. ST-segment depression. 200 mmol D. Do not correlate well with plasma K levels + A. 600 mmol hypokalemia ? Patients with plasma levels <3. 151 Bartter’s syndrome is characterized by ? Harrison’s 17th Ed 282 157 A decrement of 1 mmol/L in plasma potassium concentration A. hypokalemic D. 2 A. Hypokalemia may represent a total body potassium deficit of ? B. None of the above QRS complex and ventricular arrhythmias. 40 mmol/hour Hypokalemia with minimal renal K+ excretion (<15 mmol/day of K+ in urine) suggests that K+ was lost C.400 mmol. Potassium citrate Severe K+ depletion may result in a prolonged PR interval. 20 to 40 mmol D. All of the above B. Prominent U wave B. 4 metabolic alkalosis. maximum concentration of Harrison’s 17th Ed 282 administered K+ should be no more than ? A. Shortened QU interval C. decreased voltage and widening of the D. Metabolic alkalosis Harrison’s 17th Ed 283 C. renal K+ wasting & suppressed renin & aldosterone secretion. 20 mmol/hour D. in chronic renal failure ? Harrison’s 16th Ed. (TTKG) greater than how much suggests renal K+ loss due to increased distal K+ secretion ? 162 Through a peripheral vein. Hypokalemia with a TTKG >4 suggests renal K+ loss due to increased distal K+ secretion. Central vein ECG changes of hypokalemia are due to delayed ventricular repolarization and do not correlate well with the plasma K+ concentration. Prolonged QU interval C. Oral C. Intravascular hemolysis B.0 mmol/L Harrison’s 17th Ed 283 C. Normal saline B. 40 mmol/L 168 Hyporeninemic hypoaldosteronism is seen in ? Harrison’s 17th Ed 283 C. All of the above 165 Pseudohyperkalemia can result from ? - Harrison’s 17th Ed 283 Heparin (UFH & LMWH) inhibits production of aldosterone by zona glomerulosa & can lead to severe hyperkalemia in those with renal disease. Prolonged tourniquet use. Those receiving ACE inhibitors U Hyperkalemia is defined as a plasma K+ concentration >5. on K+-sparing diuretics. Renal sodium wasting concentration is normal. Spironolactone is a competitive mineralocorticoid antagonist. Metabolic alkalosis Pseudohyperkalemia refers to an artificially elevated “plasma” K+ concentration due to K+ movement out of cells on venipuncture in asymptomatic patient with no obvious underlying cause. All of the above i B. Spironolactone C.5 mmol/L A. hemolysis & marked leukocytosis or thrombocytosis contribute to its occurrence. All of the above 163 Ideally. Pseudohypoaldosteronism is characterized by hyperkalemia. decreased effective circulating Harrison’s 17th Ed 283 arterial volume. Diabetes mellitus G Ideally. Renal sodium wasting h Harrison’s 17th Ed 283 D. None of the above C. Amiloride and triamterene block the Genetic defect is due to mutation in gene for skeletal muscle Na + channel. Ringer solutions hyperkalemia include all except ? Harrison’s 17th Ed 283 D. High renin & aldosterone levels r D. Concurrent use of NSAIDs Hyperkalemia V ACE inhibitors block conversion of angiotensin I to II. DM. and rhabdomyolysis all D. Episodic weakness / paralysis precipitated by exercise A. precipitated by stimuli that normally lead to mild hyperkalemia (exercise).0 mmol/L B. > 6. A. > 5. Those receiving potasium sparing diuretics C.0 mmol/L D. Hypertension ta A. Amiloride D. Intravascular hemolysis. Angiotensin receptor antagonists directly inhibit actions of angiotensin II on AT1 angiotensin II receptors and result in impaired aldosterone release. apical Na+ channel of principal cell of kidney. A. Serum K+ C. C. Any of the above A. and end-organ resistance to aldosterone. hypotension. Decreased effective circulating arterial volume C. Hypotension lead to K+ release from cells as a result of tissue breakdown. Triamterene Hyperkalemic periodic paralysis is an autosomal dominant disorder characterized by episodic D. Mild renal insufficiency D. 167 Which of the following regarding hyperkalemic periodic paralysis is false ? 173 Which of the following is a competitive mineralocorticoid Harrison’s 17th Ed 283 antagonist ? A. > 5. All of the above B. Due to mutation in gene for skeletal muscle Na+ channel B. > 4. 80 mmol/L B. renal insufficiency. B.332 MCQ’s FOR MEDICAL PROFESSIONALS BY PROF. All of the above weakness or paralysis. . End-organ resistance to aldosterone 166 Pseudohyperkalemia can result from ? C. 60 mmol/L A. bilateral renal artery stenosis. Hemolysis 171 Pseudohypoaldosteronism is characterized by all except ? Harrison’s 17th Ed 283 9 C. Tumor lysis syndrome 172 Pseudohypoaldosteronism is characterized by all except ? Harrison’s 17th Ed 283 C. Rhabdomyolysis A. AJAY MATHUR Nephrology B. Hyperkalemia D. ti e A. high renin and aldosterone levels. Patients at increased risk of ACE inhibitor or angiotensin receptor antagonist–induced d 164 Hyperkalemia is defined as a plasma K+ concentration of ? hyperkalemia include those with diabetes mellitus. Prolonged use of tourniquet 9 B. Autosomal dominant disorder Harrison’s 17th Ed 283 B. Chronic tubulointerstitial disease D. renal Na+ wasting. KCl is mixed in normal saline as dextrose solutions may initially exacerbate hypokalemia due to insulin-mediated movement of K + into cells. Dextrose solutions 169 Patients at increased risk of ACE inhibitor induced C. Diabetic nephropathy The maximum concentration of administered K+ should be no more than 40 mmol/L via a peripheral vein or 60 mmol/L via a central vein. Diabetes mellitus n D. tumor lysis syndrome.5 mmol/L 170 Heparin can lead to severe hyperkalemia in patients with ? B. metabolic acidosis. ACE inhibitors or NSAIDs. Marked leukocytosis A. Unilateral renal artery stenosis R D. diabetic nephropathy or chronic tubulointerstitial disease. for infusion KCl should be mixed in ? Harrison’s 17th Ed 283 Hyporeninemic hypoaldosteronism is seen in mild renal insufficiency. or concurrent use of K+-sparing diuretics or NSAIDs. plasma K+ & ECG abnormalities.insulin therapy B. Potentially fatal hyperkalemia rarely occurs unless plasma K+ concentration exceeds 7. QRS + T B. Usual dose is 25-50 grams in 100 mL of 20% sorbitol orally to prevent constipation. Half to 1 minute B. Symptoms B. 182 In severe hyperkalemia. Normal GFR C. High renin D. normal GFR. Usual dose is 25-50 gram orally C. 7. 1 to 2 minutes C. Lupus nephritis B. It is refractory to kaliuretic effect of exogenous infused over 2-3 minutes.5 mmol/L C. 2 to 3 minutes D. Dose can be repeated if no change in mineralocorticoids. 177 The earliest ECG change in hyperkalemia is ? 183 Which of the following is used for lowering potassium levels Harrison’s 17th Ed 284 in severe hyperkalemia ? A. At least 200 mmol D. Usual dose is 10 mL of a 10% solution with suppressed renin & aldosterone levels. 5. Its effect is short-lived (30-60 min). All of the above D. Alkali therapy with IV NaHCO3 C. Insulin-glucose infusion C. metabolic acidosis. All of the above 178 Sine wave pattern seen in severe hyperkalemia is due to merging of ? 184 Which of the following reduce plasma K+ levels by shifting K+ Harrison’s 17th Ed 284 into cells ? Harrison’s 17th Ed 284 A. Sickle cell disease C.  2 adrenergic agonists D. in the gastrointestinal tract. AJAY MATHUR Cardiology 333 174 Which of the following blocks the apical sodium channel of 180 The appropriate renal response to hyperkalemia is to excrete principal cell ? how much of K+ daily ? Harrison’s 17th Ed 283 Harrison’s 17th Ed 284 A. lupus nephritis. 2-adrenergic agonists promote cellular uptake of K+. All of the above The earliest ECG changes in hyperkalemia include increased T-wave amplitude. Prolonged PR interval A. 175 Nephropathy associated with impaired potassium excretion 181 Potentially fatal hyperkalemia occurs when plasma potassium include ? concentration exceeds ? Harrison’s 17th Ed 283 Harrison’s 17th Ed 284 A. P + QRS + T C. Pentamidine C. Plasma K+ concentration C. P + QRS A. AV conduction delay and loss of P waves. Alkali therapy with IV NaHCO3 shifts K+ into cells. 333 Nephrology MCQ’s FOR MCQ’s FOR MEDICAL MEDICAL PROFESSIONALS PROFESSIONALS BY PROF. At least 50 mmol B. Each gram binds 1 mmol of K+ and releases 2–3 mmol of Na+. At least 300 mmol Trimethoprim & pentamidine impair K+ secretion by blocking distal nephron Na+ reabsorption. The terminal event is ventricular fibrillation or asystole. Anion-exchange resin A. sickle cell disease and diabetic nephropathy. Drug-induced interstitial nephritis A. Hyperkalemia A. Amiloride A. 8. 10 mL of 10% calcium gluconate 176 Gordon’s syndrome includes all except ? should be infused over ? Harrison’s 17th Ed 284 Harrison’s 17th Ed 284 A. volume-expansion IV Calcium gluconate decreases membrane excitability. ECG abnormalities D. All of the above D. Prolonged QRS duration B. 5 to 10 minutes Gordon’s syndrome is characterized by hyperkalemia.5 mmol/L. Any of the above D.5 mmol/L Nephropathies associated with impaired K + excretion include drug-induced interstitial nephritis. It can also be administered as a retention enema. Beta2-adrenergic agonists D. All of the above Sodium polystyrene sulfonate is a cation-exchange resin that promotes the exchange of Na+ for K+ Severity of hyperkalemia is determined by symptoms. Trimethoprim B. Alkali therapy with IV NaHCO3 D. ECG is seen after 5-10 minutes. At least 100 mmol C. 185 Which of the following is false regarding sodium polystyrene sulfonate ? 179 Severity of hyperkalemia is determined by ? Harrison’s 17th Ed 284 Harrison’s 17th Ed 284 A. Metabolic acidosis B. Glucose . More severe degrees result in prolonged PR interval and QRS duration. Progressive widening of QRS complex and merging with T wave produces a sine wave pattern. Can be administered as retention enema D.5 mmol/L D. Increased T-wave amplitude Harrison’s 17th Ed 284 B. . 6. The appropriate renal response to hyperkalemia is to excrete at least 200 mmol of K+ daily.5 mmol/L B. One gram binds 1 mmol of K+ & releases 2-3 mmol of Na+ B. Loss of P waves C. All of the above D. Hypercalcemia of malignancy D.9. causing hypercalcemia as a result of the combination of enhanced osteoclastic bone resorption and enhanced intestinal absorption of calcium. 6-7 B. Insulin-glucose infusion B. 1.25-dihydroxyvitamin D (1. resulting in familial hypocalciuric hypercalcemia (FHH). AJAY MATHUR Nephrology 186 The most rapid & effective way of lowering plasma potassium 192 Which of the following is the action of PTH ? concentration is ? Harrison’s 17th Ed 285. Serum K or carcinoma. . Hypokalemia not resulting in a TTKG<2 suggests renal loss Harrison’s 17th Ed 285 as cause A. Figure 47-1 A. Sarcoidosis Hypercalcemia in sarcoidosis or lymphomas is caused by enhanced conversion of 25(OH)D to potent 1. Sarcoidosis h Many solid tumors produce PTH-related peptide (PTHrP) which binds PTH receptor & mimicks effects 190 Which of the following statements is false ? of PTH on bone & kidney like PTHrP-mediated hypercalcemia of malignancy & suppression of PTH.352:373-9 Harrison’s 17th Ed 285. Familial hypocalciuric hypercalcemia (FHH) ir TTKG in a normal person on a normal diet is 8 . Figure 47-1 Harrison’s 17th Ed 284 A. Peritoneal dialysis D. All of the above Some lymphomas secrete the active form of vitamin D. Breast cancer A. V To calculate following parameters are required : Serum Osmolality (mOsm/kg). Milk-alkali syndrome 9 D. Serum Osmolality C.25(OH)2D that enhances intestinal calcium absorption resulting in hypercalcemia & suppressed PTH. pH Excess PTH production not appropriately suppressed by increased serum calcium concentrations R occurs in primary neoplastic disorders of parathyroid glands like parathyroid adenomas. Squamous-cell cancer B. Sarcoidosis G B.25(OH)2D leading to B. Hemodialysis 193 Excess PTH production not appropriately suppressed by The most rapid and effective way of lowering the plasma K+ concentration is hemodialysis. Stimulates renal 1. Hypercalcemia of malignancy C. 194 Which of the following is related to familial hypocalciuric hypercalcemia (FHH) ? d 188 Formula for calculating Transtubular Potassium Gradient Harrison’s 17th Ed 285 (TTKG) is ? ti e A. Serum K (mEq/L) and Urine K (mEq/L). Urine Osmolality D. (POsm x U Potassium) / (P Potassium x U Osm) D. Increased tubular reabsorption of calcium by kidney A. (PPotassium x U Potassium) / (P Osm x U Osm) B. Parathyroid adenoma A. D.25(OH) 2D). (POsm x P Potassium) / (UPotassium x U Osm) C. Hyperthyroidism C. Peritoneal increased serum calcium concentrations occurs in ? dialysis is only 15-20% as effective as hemodialysis. Calcium gluconate infusion C. Ovarian cancer C.25(OH)2D B. 4-6 A. ta A.25(OH)2D production D. Lymphoma D. Exogenous calcium overload D. 2 . (PPotassium x U Osm) / (P Osm x U Potassium) Inappropriate PTH secretion for existing level of serum calcium occurs with heterozygous inactivating U calcium sensor receptor (CaSR) mutations. Urine Osmolality (mOsm/kg). Milk-alkali syndrome renal tubular acidosis C. PTH-related peptide (PTHrP) A.25(OH)2 D occurs in which of the following ? 191 Decrease in serum calcium leads to ? N Engl J Med 2005. D. hyperplasia. glands and kidneys. Increase in parathyroid hormone (PTH) B. which impair extracellular calcium sensing by parathyroid Correct formula for TTKG is (POsm x UPotassium) / (PPotassium x UOsm). - 189 TTKG in a normal person on a normal diet is ? 195 Which of the following is related to PTH-related peptide (PTHrP) ? A. 8-9 C. 1. Resorption of calcium from bone B. Hypercalcemia of malignancy TTKG ? B. Resorption of calcium from bone C. Hypokalemia should result in a TTKG < 2 196 Enhanced conversion of 25(OH)D to 1.4 Harrison’s 17th Ed 285 9 B. Hyperkalemia should result in a TTKG>10 hypercalcemia is related to ? C. Harrison’s 17th Ed 285 187 Which of the following parameter is not required in calculating A. Familial hypocalciuric hypercalcemia (FHH) Hyperkalemia not resulting in a TTKG>10 suggests type IV renal tubular acidosis D. It is valid only when Uosm >300 & UNa >25. Hyperkalemia not resulting in a TTKG>10 suggests type I B. Hypercalcemia 197 Hypercalcemia due to excess secretion of 1. Stimulates renal 1.25(OH)2D production C. Calcium sensor receptor (CaSR) mutations n C.334 MCQ’s FOR MEDICAL PROFESSIONALS BY PROF. calcitriol. and enhanced intestinal absorption of calcium (in 1.5 mg/dL D. Vitamin D C. All of the above D. 0. 14 mg/dL Severe acute hypercalcemia (>12–13 mg/dL) may result in lethargy. Elevated serum level of nonionized calcium C. 12 mg/dL C. Humoral hypercalcemia of malignancy B. All of the above D.11.0 mg/dL D.352:373-9 A. coma or pancreatitis. enhanced renal tubular reabsorption of calcium (in HHM & ectopic 210 What percentage of serum total calcium is ionized ? hyperparathyroidism).9.10.11. Enhanced renal tubular reabsorption of calcium B. Bradycardia A.5 mg/dL dL and the calcium . Nephrolithiasis C. ST segment True hypercalcemia is due to enhanced osteoclastic bone resorption (in local osteolytic hypercalcemia.352:373-9 A. Heart rate B.5 to 3.0 mg/ D. AV block& short QT interval. 335 Nephrology MCQ’s FOR MCQ’s FOR MEDICAL MEDICAL PROFESSIONALS PROFESSIONALS BY PROF. 2. Short QT interval C. humoral hypercalcemia of malignancy (HHM). AJAY MATHUR Cardiology 335 198 Type of hypercalcemia associated with cancer is ? 204 Medication that may independently lead to hypercalcemia is ? N Engl J Med 2005. 11 mg/dL B. serum phosphorus level should be kept in the range of 2. stupor. AV block B. 10 % .352:373-9 200 Serum levels of calcium in mild hypercalcemia are upto ? A.phosphorus product below 40. 2. Local osteolytic hypercalcemia A. vitamin D & thiazides. 5 Harrison’s 17th Ed 285 B. & ectopic hyperparathyroidism).352:373-9 Harrison’s 17th Ed 286 A. 207 Which of the following occurs in acute severe hypercalcemia 201 Serum levels of calcium in severe hypercalcemia is more than (>12-13 mg/dL) ? Harrison’s 17th Ed 286 ? N Engl J Med 2005.0 mg/dL A. serum phosphorus level should 199 Hypercalcemia with suppressed PTH secretion in be kept in the range of ? hyperthyroidism is due to ? N Engl J Med 2005. calcium . All of the above D.0 to 2. Enhanced osteoclastic bone resorption A.352:373-9 Harrison’s 17th Ed 285 A.phosphorus product to hypercalcemia with suppressed PTH secretion. should be kept ideally below ? N Engl J Med 2005. Serum levels of calcium in severe hypercalcemia is > 14. 7 .5 mg/dL While treating hypercalcemia. Pancreatitis A. QT interval D. Increased parathyroid cell mass Hyperthyroidism or osteolytic metastases directly increase calcium mobilization from bone leading 206 While treating hypercalcemia. 13 mg/dL D.25(OH)2D-secreting lymphomas C.8. 208 ECG changes in hypercalcemia include all except ? Harrison’s 17th Ed 286 202 True hypercalcemia refers to ? N Engl J Med 2005. Enhanced intestinal absorption of calcium C. 10 A. 9 . Changes in serum calcium can be monitored by following the QT interval. A. Serum levels of calcium in mild hypercalcemia is upto 11 .352:373-9 N Engl J Med 2005. Enhanced intestinal calcium absorption B. 1.5 to 3. 1. Peptic ulcer disease B.5 mg/dL C. Prolonged PR interval D. 209 Changes in serum calcium is preferrably monitored by which 203 Basic mechanism of true hypercalcemia is ? of the following in ECG ? N Engl J Med 2005.25(OH) 2 D secreting lymphomas. Elevated serum level of total calcium B.0 mg/dL. Lithium B.0 to 2.5 to 1. Elevated serum level of ionized calcium is true hypercalcemia.0 mg/dL B. Thiazides D. All of the above In addition to the above three. QRS amplitude C. 1. 11 . ideally in the range of 30. ectopic hyperparathyroidism due to ectopic secretion of authentic Medications that independently lead to hypercalcemia are lithium. 20 B. Increased calcium mobilization from bone C. 8 .5 mg/dL C. Elevated serum level of ionized calcium D.25(OH) 2D-secreting Harrison’s 17th Ed 286 lymphomas & ectopic hyperparathyroidism). Any of the above ECG changes in hypercalcemia include bradycardia. 205 While treating hypercalcemia.5 mg/dL and is usually asymptomatic. 30 C. PTHrP-mediated hypercalcemia D.352:373-9 A. PTH is a rare cause of hypercalcemia. 1 mg/dL A.500 ml/ hour. AJAY MATHUR Nephrology B.25(OH)2D-mediated hypercalcemia. < 0. plasma PTH level varies inversely with plasma calcium level.352:373-9 222 Dose of Zoledronic acid is ? Harrison’s 17th Ed 286 A. 0. Malignancy D. Primary hyperparathyroidism C. Impaired renal function C. and conversely for elevations in serum albumin.01 is suggestive of FHH. diagnosis is almost always C. Loop diuretics should not be administered until after n hypercalcemia due to an underlying malignancy. 0. 213 Increases in PTH are often accompanied by ? U Harrison’s 17th Ed 286 219 Drug that inhibits bone resorption is ? Harrison’s 17th Ed 286 A. Sarcoidosis primary hyperparathyroidism. Mithramycin D.352:373-9 C. All of the above D. Saline itself is calciuretic. Normal serum calcium is 8. All of the above B. They work by blocking osteoclastic bone 214 Lab.10. 2 mg IV infusion over 30-minutes B. metabolic alkalosis & impaired renal function are classic laboratory findings in G Serum albumin levels are determined to obtain “correct serum calcium”. Ectopic PTH secretion is extremely rare . h Harrison’s 17th Ed 286 A.6 mg/dL D. Ibandronate and Clodronate are the other two besides the above three. Pamidronate C. All of the above 216 Which of the following statements is false ? Harrison’s 17th Ed 286. Ectopic secretion of authentic PTH is a rare cause of hypercalcemia. Zoledronic acid d B. Hyperphosphatemia B. 50 % D. Zoledronic acid B. Harrison’s 17th Ed 286 A.25(OH)2D production. Low serum phosphorus Harrison’s 17th Ed 286 C. Medication use B.9 . glucocorticoids are the preferred therapy. Elevated serum calcium 220 Glucocorticoids are the preferred therapy in hypercalcemia ta due to ? B. Etidronate D. r Intravenous bisphosphonates are the best studied. depending on patient’s cardiovascular status & renal function) as hypercalcemia invariably Primary hyperparathyroidism is the most common cause of chronic hypercalcemia. Increased PTH level A. Familial hypocalciuric hypercalcemia (FHH) 215 In familial hypocalciuric hypercalcemia (FHH). In patients with 1. Calcitonin D. followed by leads to dehydration. Hypercalcemia A. Hypercalcemia invariably leads to dehydration C.8 mg/dL Hypercalcemia.336 MCQ’s FOR MEDICAL PROFESSIONALS BY PROF. Hypokalemia - A. 80 % Normally. 0. 0. findings in primary hyperparathyroidism include ? resorption. Hypernatremia D. 0. to total calcium for every decrement in serum albumin of 1. Severe hyperparathyroidism If PTH level is increased with elevated serum calcium & low phosphorus. < 0. < 0. D.2 B. N Engl J Med 2005.0 g/dL below the reference value of 4. Underlying malignancy D.0 g/dL. to date. what quantity 217 Classic laboratory finding in milk alkali syndrome is ? should be added to serum total calcium ? N Engl J Med 2008.8 mg/dL should be addd patients with milk alkali syndrome in those with excess oral intake of calcium and milk. 211 With a decrease in serum albumin of 1.1 g/dL for albumin. Metabolic alkalosis B. D. Calcitonin Calcium/creatinine clearance ratio (urine calcium/serum calcium divided by urine creatinine/serum C. as calcium / creatinine clearance ratio is ? they decrease 1. having been well documented in only eight patients ~50% of total calcium is ionized & rest is bound principally to albumin. Thiazide diuretics should not be used as they stimulate renal calcium reabsorption. safest and most effective agents for use in i patients with hypercalcemia associated with cancer. < 0. Gallium nitrate creatinine) of <0.01 221 Pharmacologic therapy for hypercalcemia associated with cancer include ? B. Glucocorticoids ti e C. All of the above 9 Increases in PTH are often accompanied by hypophosphatemia.2 mg/dL B. Sarcoidosis Initial therapy of significant hypercalcemia begins with volume expansion with saline (200 . full hydration has been achieved. Hypophosphatemia 9 C. 25 % C. None of the above D.1 A. Volume expansion with IV saline V Harrison’s 17th Ed 286 A.4 mg/dL C. 218 Initial therapy of significant hypercalcemia begins with ? R Harrison’s 17th Ed 286 212 Chronic hypercalcemia is most commonly caused by ? A.358:1952-6 Harrison’s 17th Ed 286 A.05 N Engl J Med 2005. Harrison’s 17th Ed 286 A. All of the above Control of arterial CO 2 tension (Pa CO2) by central nervous and respiratory systems & control of 226 Chvostek’s sign is present in what percentage of normal plasma bicarbonate by kidneys stabilize arterial pH by excretion or retention of acid or alkali individuals ? maintains systemic arterial pH between 7. ~ 10 % C.0301)) C. Harrison’s 17th Ed 287 A. Acidosis & Alkalosis 225 Degree of hypocalcemia is most in which of these disorders ? Harrison’s 17th Ed 286 231 Systemic arterial pH is maintained between 7.25(OH)2D levels 224 Main defense against hypocalcemia is ? Harrison’s 17th Ed 286 C.e. D. Central nervous system C. Hypoparathyroidism Harrison’s 17th Ed 287 B. 4 mg IV infusion over 30-minutes B. Serum calcium levels A. Serum 25-hydroxyvitamin D levels B. All of the above B. Vitamin D resistance C./ (PaCO2 x 0. Neural respiratory control A. Hydroxychloroquine A. and 223 Drugs that decreases 1.35 & 7. ~6% A.1 + log (HCO3. 20 mm Hg Henderson-Hasselbalch equation includes metabolic & respiratory components that regulate systemic pH. carpopedal spasm. Seizures B. Vitamin D Nutritional vitamin D deficiency is best assessed by obtaining serum 25-hydroxyvitamin D levels. PTH 1.1 + log (HCO3. Compensatory response to alteration in plasma [HCO 3-] . PaCO 2 C. Impaired 1. and hydroxychloroquine.0301)) B. 337 Nephrology MCQ’s FOR MCQ’s FOR MEDICAL MEDICAL PROFESSIONALS PROFESSIONALS BY PROF. PTH-related peptide (PTHrP) PTH is the main defense against hypocalcemia./ (PaCO2 x 0.25(OH)2D levels are informative. 15 mm Hg D. 6 mg IV infusion over 30-minutes C. Serum 1. 6. Trousseau’s sign refers to carpal spasm in hypocalcemia induced by inflation of a BP cuff to 20 mm Hg above patient’s SBP for 3 minutes.45 by ? A. D. 5 mm Hg B./ (PaCO2 x 0. Vitamin D deficiency A. ~4% 232 Henderson-Hasselbalch equation is used to determine ? Harrison’s 17th Ed 287 B. ~8% B./ (PaCO2 x 0. twitching of circumoral muscles in response to tapping of facial nerve just anterior to the ear is present in ~10% of normal individuals. laryngospasm. Short QT interval Severe hypocalcemia can induce seizures. Systemic pH Chvostek’s sign i.35 & 7.0301)) A. may decrease 1.25(OH)2D production and are used occasionally.1 + log (HCO3. 8 mg IV infusion over 30-minutes D.25(OH)2D production is ? prolongation of the QT interval. D. Calcium D.1 + log (HCO3. 10 mm Hg C. Anion gap 227 For eliciting Trousseau’s sign. Ketoconazole 229 Nutritional vitamin D deficiency is best assessed by ? B. Serum PTH levels B.45. serum C. Respiratory system D. which reflect vitamin D stores. chloroquine. Laryngospasm D. In renal insufficiency or suspected vitamin D resistance. Serum 25-hydroxyvitamin D levels D. bronchospasm.25(OH)2D levels C. Serum calcium levels Other drugs. Chloroquine Harrison’s 17th Ed 287 C. Serum 1. Serum PTH levels 230 Vitamin D resistance is best assessed by ? Hypocalcemia Harrison’s 17th Ed 287 A. PaO 2 D. D. Kidney Degree of hypocalcemia is most in hypoparathyroidism. such as ketoconazole. AJAY MATHUR Cardiology 337 B.0301)) D. 5. Bronchospasm C. 234 Increases or decreases in PaCO2 represent derangements of ? Harrison’s 17th Ed 288 228 Which of the following is false about severe hypocalcemia ? Harrison’s 17th Ed 287 A. BP cuff is inflated how much 233 Henderson-Hasselbalch equation is ? Harrison’s 17th Ed 287 above patient’s systolic blood pressure ? Harrison’s 17th Ed 287 A.25(OH)2D production B. 4. 7. All of the above Hasselbalch equation. PaCO 2 regulated primarily by neural respiratory factors Usual steady-state PaCO2 is maintained at 40 mmHg. 40 . Thus.338 MCQ’s FOR MEDICAL PROFESSIONALS BY PROF.5 x [HCO3 -]) + 8 D. AJAY MATHUR Nephrology C. Hypercapnia is the result of hypoventilation C. NH4+ production & excretion are impaired in chronic renal failure. Due to independently coexisting disorders B.100 mmol B.ions per day ? 245 In simple metabolic acidosis. D. None of the above nephron ? Harrison’s 17th Ed 288 235 Which of the following statements is false ? A.120 mmol C. NH 4+ Harrison’s 16th Ed. About 3000 mmol C. 100 . + D. None of the above B. About 3000 mmol A. Distal convoluted tubule D. Hyperkalemia D. About 4000 mmol 246 Which of the following statements about mixed acid-base D.25 mm Hg for each mmol/L decrease in [HCO 3–]. 2. PaCO 2 = (1. B. C.5 x [HCO3–]) + 8 ± 2. the Henderson-Hasselbalch equation ? Harrison’s 17th Ed 288 241 NH4+ production & excretion are impaired in ? A. Formation of titratable acid B PaCO 2 = (1. pH Harrison’s 17th Ed 288 B. 263 B. Usual steady-state PaCO 2 is 60 mm Hg C. Pa CO2 A. About 2000 mmol Harrison’s 17th Ed 288 B. About 5000 mmol disorders is false ? Harrison’s 17th Ed 288 Kidney filters ~4000 mmol of HCO3– per day and to reabsorb this filtered load of HCO3–.90% of HCO3– is reabsorbed in the proximal convoluted tubule of kidney. pH and Pa CO2 are measured and [HCO 3–] is calculated from Henderson- D. A. A+B 242 Normally.4 x [HCO3 -]) + 8 C. Primary alteration of PaCO2 evokes renal adaptation D. [HCO3–] B.75 mmHg D.00 mmHg 239 Kidneys excrete how much H+ ions per day ? Harrison’s 17th Ed 288 Degree of respiratory compensation in simple metabolic acidosis is predicted by Pa CO2 = (1. which of the following is calculated from 80 . CO2 overexcretion causes hypocapnia Distal nephron reabsorbs remainder of filtered HCO3– and secretes 40 . respectively).60 mmol/day of H+.ions ? Harrison’s 17th Ed 288 C. hyperkalemia & renal tubular acidosis.25 mmHg C. All of the above D. Relationship is not as compensatory responses 240 Which part of nephron reabsorbs most of HCO3. None of the above 237 Kidneys regulate plasma [HCO3-] by ? 244 Which of the following formula is correct ? Harrison’s 17th Ed 288 Harrison’s 17th Ed 288 A.80 mmol A. Proximal convoluted tubule D. Reabsorption of filtered HCO3. 1.60 mmol Harrison’s 17th Ed 288 B. About 5000 mmol C. About 2000 mmol Values for PaCO2 <24 or >28 mmHg define a mixed disturbance (metabolic acidosis and respiratory alkalosis or metabolic alkalosis and respiratory acidosis. Pa CO2 decreases by 1. PaCO 2 = (1. About 4000 mmol B. A. CO2 underexcretion produces hypercapnia D. Titratable acid A. Loop of Henle Mixed acid-base disorders are due to independently coexisting disorders and not merely as compensatory responses. 243 Secreted H+ is represented in the urine as ? Harrison’s 17th Ed 288 236 Which of the following statements is false ? A. what quantity of H + is secreted daily by distal D. . PaCO 2 changes result from alteration in plasma [HCO3 -] Secreted H is represented in urine as titratable acid and NH4+. 80 . 1. A. Excretion of NH4+ in urine C. 1. Titratable acid & NH 4+ B. 60 . All of the above C.6 x [HCO3 -]) + 8 238 Kidneys excrete how much HCO3.3 x [HCO3 -]) + 8 B. Chronic renal failure C. Collecting ducts 247 In clinical laboratory. All of the above C. Can lead to dangerous extremes of pH A. PaCO 2 = (1. They can lead to dangerous extremes of pH. PaCO2 would decrease by how Harrison’s 17th Ed 288 much for each mmol/L decrease in [HCO3-] ? A.50 mmHg D. renal tubules must secrete 4000 mmol of hydrogen ions. Renal tubular acidosis In clinical laboratory. Acidic Harrison’s 17th Ed 290 C.(Cl . Unmeasured Clinical metabolic acidosis is of two types . Peripheral arterial vasodilation C. AG = Na+ + (Cl. All of the above AG represents unmeasured anions in plasma (normally 10 to 12 mmol/L) and is calculated as AG = Fall in blood pH is accompanied by increase in tidal volume (Kussmaul respiration). Citrate C. All of the above A. Alkaline A. Patient’s AG ÷ 10 D. Hyperchloremic acidosis B. 339 Nephrology MCQ’s FOR MCQ’s FOR MEDICAL MEDICAL PROFESSIONALS PROFESSIONALS BY PROF. 3. Increase in unmeasured anions B. AG = Na+ + (Cl. Patient’s AG – 10 C. Chloride Metabolic acidosis occurs due to an increase in endogenous acid production (lactate & ketoacids). cations include calcium. and organic anions. - D. A. Sulphate Harrison’s 17th Ed 290 260 In a pure AG severe metabolic acidosis. Central venoconstriction D. 256 ) in AG ( Increment ( AG) is estimated by ? 250 Increase in AG is most often due to ? Harrison’s 17th Ed 290 Harrison’s 17th Ed 289 A. All of the above B. - B. sulfate.5 Harrison’s 17th Ed 290. AJAY MATHUR Cardiology 339 248 Anionic gap (AG) is calculated by ? 254 Fall in blood pH is accompanied by ? Harrison’s 17th Ed 289 Harrison’s 17th Ed 290 A. 292 D. 249 Anion gap represents “unmeasured” anions like ? 255 Normal-AG metabolic acidosis is also called ? Harrison’s 17th Ed 289 Harrison’s 17th Ed 290 A. Hypokalemic acidosis Unmeasured anions include anionic proteins. magnesium & potassium. phosphate. Phosphate B.5 g/ dL) decreases the anion gap by ? A.0 meq/L 258 Shohl’s solution is ? A fall in serum albumin by 1 g/dL from normal value (4. -hydroxybutyrate Harrison’s 17th Ed 289 B. and nervous systems. Increase in ventilation B. 15 meq/L Metabolic acidosis has profound effects on the respiratory. Decrease in measured cations Potential [HCO3 ] can be estimated from the increment () in the AG (AG = patient’s AG – 10). Hypochloremic acidosis C. goal is to increase the A. 257 Which of the following acid anion in plasma is metabolizable ? Harrison’s 17th Ed 290 251 A fall in serum albumin by 1 g/dL from the normal value (4.5 g/dL) decreases anion gap by 2. Hyperkalemic acidosis D. 292 meq/L. Acetoacetate A. 2. 1. Increase in endogenous acid production D. Mg++. C. Bicarbonate 253 Metabolic acidosis has profound effect on which of the following systems ? D. All of the above C. peripheral Na+ – (Cl– + HCO3–). AG = Na . Loss of bicarbonate C. Anionic proteins (Albumin) A. loss of bicarbonate (diarrhea). Increase in measured anions C.(Cl + HCO3 ) + . Neutral 252 Metabolic acidosis can occur because of ? B. or accumulation of endogenous acids (renal failure).5 meq/L Metabolizable acid anion in plasma are  -hydroxybutyrate. Patient’s AG x 10 A. acetoacetate & lactate.high-AG and normal-AG (hyperchloremic acidosis). – Increase in AG is most often due to increase in unmeasured anions and less commonly due to decrease in unmeasured cations (Ca++.0 meq/L D. D. Decrease in unmeasured cations D. 10 meq/L D. All of the above D. 2.HCO3 ) + .. AG = Na . B.5 meq/L C. Nervous A. Any of the above B.+ HCO3-) C. Lactate B. K+). Patient’s AG + 10 B. Accumulation of endogenous acids 259 Sodium salt in Shohl’s solution is ? Harrison’s 17th Ed 290. arterial vasodilation & central venoconstriction.HCO3-) A. Sulfate C. Respiratory [HCO3–] to ? B. Cardiac Harrison’s 17th Ed 290 C. cardiac. 20 meq/L D. 25 meq/L . 25 C. 7. NRTI drugs. Ingested toxins & their metabolites (carbon monoxide. Nitroprusside ketone reaction does not detect - Normal serum lactate level is 0. 3. starvation and D. 7. vomiting. Ethanol B. pH < 7. 7. Shock 273 Which of the following is exacerbated by glucose infusion for 267 Type B lactic acidosis is due to all except ? treatment of AKA ? Harrison’s 17th Ed 290 Harrison’s 17th Ed 291 A. cyanide) or to aerobic disorders (type B) . goal is to increase the 268 Which of the following can cause Type B lactic acidosis ? pH to ? Harrison’s 17th Ed 290 Harrison’s 17th Ed 290 A. AZT analogues acidosis ? Harrison’s 17th Ed 290 C. ethanol. Circulatory failure C. D. All of the above D. seizures. 263 Which of the following ‘toxins’ can cause high-anion-gap 270 The nitroprusside ketone reaction can detect ? metabolic acidosis ? Harrison’s 17th Ed 290 Harrison’s 17th Ed 290 A. . renal or hepatic failure.5 . If severe. Renal failure A.5-1. Glucagon levels are increased 265 Normal serum lactate level is ? A. Ethylene glycol B. Hypophosphatemia B. Severe anemia Chronic alcoholics can develop ketoacidosis when alcohol consumption is abruptly curtailed and B.malignancies. All of the above Nitroprusside ketone reaction can detect acetoacetic acid but not Beta-hydroxybutyrate. Carbon monoxide poisoning D. vomiting. may induce rhabdomyolysis. so the degree of ketosis and ketonuria can be underestimated.1. Diabetic ketoacidosis A.7 mmol/L D. Diabetes mellitus hormone levels are increased. Hypomagnesemia D.15 and not Harrison’s 17th Ed 290 to increase these values to normal. 1.e. ketoacidosis D. Typically. glucagon and growth C. Beta-hydroxybutyrate B. Starvation ketoacidosis B. Biguanides A. A+B C. propylene glycol. severe anemia.4. AKA is usually associated with binge drinking. All of the above B. Methanol C. cardiac failure). isoniazid.45 269 Which of the following can cause Type B lactic acidosis ? In severe acidosis i. Renal failure Increase in plasma L-lactate may be secondary to poor tissue perfusion (type A) .7 mmol/L Harrison’s 17th Ed 290 D. Salicylates D. Triglyceride levels are increased A. All of the above methanol. Acetoacetic acid A. 7. Methanol C. and fructose). hydroxybutyrate 266 Type A lactic acidosis is due to all except ? C.5 . abdominal pain. DM.3 mmol/L 272 Which of the following about alcoholic ketoacidosis is false ? C.7 . infections. and triglyceride. Associated with binge drinking. Growth hormone levels are decreased B. and inhibitors C. 264 Which of the following ‘ketoacidosis’ producing states can cause high-anion-gap metabolic acidosis ? 271 Which of the following about alcoholic ketoacidosis is false ? Harrison’s 17th Ed 290 Harrison’s 17th Ed 290 A. D. AJAY MATHUR Nephrology 261 In a pure AG severe metabolic acidosis.7 mmol/L. Hepatic failure B. A. All of the above Hypophosphatemia may be exacerbated by glucose infusion. goal is to increase [HCO3–] to 10 meq/L and pH to 7. Occurs in chronic alcoholics when alcohol consumption B. Fructose A. thiamine deficiency.340 MCQ’s FOR MEDICAL PROFESSIONALS BY PROF. All of the above volume depletion C. Acidosis is because of elevated -hydroxybutyrate B. Hypokalemia C.2. 2.5 .7 mmol/L A.3. Mitochondrial enzyme defects nutrition is poor. mitochondrial enzyme defects. Lactic acidosis.circulatory insufficiency (shock. Alcoholic ketoacidosis is abruptly curtailed C. 0.15 B. None of the above D. cortisol. Isoniazid 262 Which of the following can cause high anion-gap metabolic B.20. insulin levels are low. or drugs/toxins (biguanides. starvation and volume depletion. Insulin levels are high Harrison’s 17th Ed 290 D.35 D. glycolic acid. B. AJAY MATHUR Cardiology 341 274 Plasma osmolality (Posm) is calculated by ? C. Urea A. [Na+ + K+]u – [Cl–]u. Any of the above is generated by ? Harrison’s 17th Ed 291 286 Urine anion gap (UAG) is calculated by the formula ? A. osmotic pressure of blood D. Glucose B. Metabolites include formaldehyde and formic acid D. Sodium Harrison’s 17th Ed 292 B. Reduced rate of NH4+ production and excretion Ethylene glycol. 341 Nephrology MCQ’s FOR MCQ’s FOR MEDICAL MEDICAL PROFESSIONALS PROFESSIONALS BY PROF. C. Lactic acid renal mass. Isopropyl alcohol Harrison’s 17th Ed 291 D. suggesting a renal cause of acidosis. When urine anion gap is positive. [HCO3-] levels rarely < 15 mmol/L methanol cause a high-AG acidosis. B. Ethylene glycol In Isopropyl alcohol ingestion. Zero urine anion gap 279 Under most physiologic conditions. and other organic acids. It is used to treat Alcohol- Harrison’s 17th Ed 291 induced acidosis in a IV loading dose of 7 mg/kg. C. Its metabolites are oxalic acid. [Na+ + K+]u x [Cl-]u D. or Posm = 2Na+ + Glucose (mg/dL)/18 + BUN (mg/dL)/2. Ethylene glycol ingestion Unit of osmolality is mOsm/kg/H 2O. 2Na+ + Glucose (mmol/L) + (3 x Albumin) 282 Which of the following about methanol is false ? Harrison’s 17th Ed 291 C. A. All of the above C. Anion gap (AG) rarely > 20 mmol/L 278 Which of the following is a metabolite of ethylene glycol ? D. Ethylene glycol and B. B. Posm = 2Na + Glucose (mmol/L) + (3 x Albumin) + A. All of the above A. Alcohol dehydrogenase inhibitor Plasma osmolality (P osm ) = 2Na + + Glucose (mmol/L) + BUN (mmol/L). C. mOsm/kg/H 2O cause optic nerve & CNS damage. Ketosis A. Isopropyl alcohol ingestion C.[Cl-]u D. C. Posm = 2Na+ + Glucose (mmol/L) + BUN (mmol/L) 281 ‘Fomepizole’ is a ? Harrison’s 17th Ed 291 C. heart. [Na+ + K+]u + [Cl-]u 280 The chemical name of ‘Fomepizole’ is ? Urinary NH4+ levels are estimated by urine anion gap (UAG) i. urine ammonium level is low. Aldehyde dehydrogenase inhibitor D. 4-methylpyrazole Harrison’s 17th Ed 291 D. Negative urine anion gap (UAG) Ethylene glycol (used in antifreeze) causes metabolic acidosis and damage to CNS. acidosis is not present because acetone is rapidly excreted. None of the above Harrison’s 17th Ed 291 Uremic acidosis is characterized by reduced rate of NH4+ production & excretion due to decreased A. B. [HCO3–] rarely falls to <15 mmol/L and AG is rarely >20 mmol/L. Posm = 2Na+ + Glucose (mmol/L) + Urea (mmol/L) B. 2Na+ + 3Glucose (mmol/L) A. Intoxication cause severe optic nerve damage 276 Unit of osmolality is ? Methanol or wood alcohol causes metabolic acidosis. None of the above 275 Osmolar gap is calculated by ? Fomepizole or 4-methylpyrazole is an alcohol dehydrogenase inhibitor. Methanol 284 Which of the following about uremic acidosis is false ? C. methanol and isopropyl alcohol cause an elevated osmolal gap. the UAG is negative which indicates that urine ammonium level is appropriately increased. Calculated osmolality . Also called ‘wood alcohol’ D. [Na+ + K+]u / [Cl-]u C. mOsm/kg/Blood A.measured osmolality B. Methanol ingestion 277 Which of the following can cause an elevated osmolal gap ? Harrison’s 17th Ed 291 D. Hydrochloric acid A. Tricarboxylic acid 285 ‘Extrarenal cause’ of acidosis is suggested by ? C.8. Oxalic acid Harrison’s 17th Ed 292 D.e. mOsm/kg/Plasma 283 Acidosis is not present in which of the following ? C. mOsm/kg/Serum Harrison’s 17th Ed 291 D. 2-methylpyrazole suggesting an extrarenal cause of the acidosis. [Na+ + K+]u . Ketone dehydrogenase inhibitor D. When [Cl–]u > Harrison’s 17th Ed 291 [Na+ + K+]. B. 2Na+ + Glucose (mmol/L) + Urea (mmol/L) B. Has high molecular weight Difference between calculated osmolality and measured osmolality is called osmolar gap and is proportional to concentration of unmeasured solute. Its metabolites formaldehyde & formic acid A. Positive urine anion gap and kidneys. 3-methylpyrazole . lungs B. 5-methylpyrazole A. Posm = 2Na + + 3Glucose (mmol/L) B. It has a low molecular weight (32 Da). A. suggest ? ethylene glycol toxicity. Renal insufficiency If urine is acidic and has low concentrations of Na+. in older adults with 298 Which of the following is false about Liddle’s syndrome ? diabetes mellitus or tubulointerstitial disease and renal insufficiency. Prior diuretic ingestion B. and Cl–. diabetic & alcoholic ketoacidosis. Harrison’s 17th Ed 293 A. Hypertension A. magnesium deficiency. K+. low [Cl]u C. increased serum [HCO 3–] & increased PaCO2 due to compensatory alveolar hypoventilation. 4 mm Hg A. Diuretic ingestion A. or diuretic ingestion. All of the above Harrison’s 17th Ed 293 289 The typical findings in classic distal RTA (type 1 RTA) include A. Alkaline urine. Useful to memorize causes of anion gap metabolic acidemias. nonedematous patient can be due to Bartter’s C. Low urinary NH4+ excretion C. Urine pH > 5. paraldehyde. hyperchloremic acidosis.5). elevated [Na]u and [K]u. Harrison’s 17th Ed 293 290 Mnemonic “MUDPILES” is to remember cause of ? A. D. 2 mm Hg Harrison’s 17th Ed 292 B. A. Increase in serum [HCO3 ] . low vomiting ? urinary NH4+ excretion (positive UAG. Harrison’s 17th Ed 294 A. and high urine pH (pH > 5. low [Cl]u B. elevated [Na]u and [K]u. All of the above B. Hypercalcemia 293 The PaCO2 increases how much for each 10 mmol/L increase in [HCO3-] above normal ? B. 297 Acid urine with low urinary Na+. Posthypercapnic state A. Hyperkalemia D. Letters in MUDPILES represent methanol. All of the above B. D. All of the above C. Hyperkalemia . Glycosuria C.342 MCQ’s FOR MEDICAL PROFESSIONALS BY PROF. Bartter’s syndrome all except ? B. Increase in PaCO2 levels. vomiting. All of the above 294 Hypokalemia and alkalosis in a normotensive. and salicylates. 6 mm Hg B. the possibilities are prior vomiting. Exogenous alkali A. low [Na] u and [K] u. Low urinary NH4+ excretion or Gitelman’s syndrome. low [Na]u and [K]u. 267 C. isoniazid (INH). nonedematous patient suggests ? Fanconi syndrome refers to glycosuria. All of the above Hyporeninemic hypoaldosteronism causes hyperchloremic metabolic acidosis. low urine [NH4+]). AJAY MATHUR Nephrology 287 Fanconi syndrome is characterized by ? A. Diabetes mellitus C. Metabolic alkalosis manifests as elevated arterial pH. with elevated [Na+] & [K+] but low [Cl–]. Phosphaturia D. diagnosis is either vomiting or alkali ingestion. nonedematous patient suggests ? D. Alkaline urine. lactic acidemia. Generalized aminoaciduria D. Harrison’s 17th Ed 294 A.5 295 Hypokalemia and alkalosis in a normotensive. Acidic urine. generalized aminoaciduria. and Cl. uremia. Hyperchloremia Liddle’s syndrome results from increased activity of collecting duct Na+ channel (ENaC) manifests as hypertension due to volume expansion leading to hypokalemic alkalosis & normal aldosterone D. Increased activity of collecting duct sodium channel 292 Metabolic alkalosis is manifested by all except ? Harrison’s 17th Ed 292 B. Respiratory acidosis D. low [Cl] u D. Prior vomiting 291 Hyporeninemic hypoaldosteronism can be seen in ? Harrison’s 17th Ed 292 B. Gitelman’s syndrome Harrison’s 17th Ed 292 C. Paraldehyde can be replaced with propylene glycol. or prior diuretic ingestion. Metabolic alkalosis C. Elevated aldosterone levels C. Hypokalemia D. It is often accompanied by hypochloremia & 299 Presentation of metabolic alkalosis is similar to ? hypokalemia. Acidic urine. Elevated arterial pH C. High urine pH 296 Which of the following is true for urine in patients who are Typical findings of classic distal RTA (type 1 RTA) include hypokalemia. exogenous alkali. D. K+. Hypokalemic alkalosis B. Metabolic acidosis B. 8 mm Hg C. low [Cl]u A. and phosphaturia due to Harrison’s 17th Ed 293 generalized proximal tubular dysfunction (type 2 RTA). posthypercapnic state. Vomiting Harrison’s 17th Ed 292 C. Magnesium deficiency 288 Typical findings in classic distal RTA (type 1 RTA) include ? B. Respiratory alkalosis If urine is alkaline. Hyperchloremic acidosis Combination of hypokalemia & alkalosis in a normotensive. Tubulointerstitial disease D. Hypocalcemia Harrison’s 16th Ed. papilledema is due to ? A.5 mmol/L drop in [HCO3– C. [HCO3 ] levels increase by how . 5 mmol/L D. Vasoconstriction Changes in central and peripheral nervous system function with metabolic alkalosis are similar to B. D. In chronic respiratory acidosis (>24 hours). 5 mmol/L D. alkalosis ? Harrison’s 17th Ed 295 306 In respiratory acidosis.003 rise in pH. Gitelman syndrome In chronic respiratory alkalosis a 1 mm Hg fall in PaCO2 causes a 0. 3 mmol/L D. K+ . Liddle’s syndrome Harrison’s 17th Ed 295 D. 12 mmol/L above ? Harrison’s 17th Ed 294 B. 38 mmol/L D. by definition. C. 1 mmol/L B. Na + Harrison’s 17th Ed 294 B. Conn’s Syndrome Diagnosis of respiratory acidosis requires. 0. 4 mmol/L 310 [HCO3–] falls by how much for each 10-mmHg decrease in PaCO2 304 In chronic respiratory acidosis. 2 mmol/L C. 18 mmol/L C. serum HCO3– usually does not increase above 38 mmol/L. 2 mmol/L C. in chronic hypocapnia ? much for every 10 mm Hg increase in PaCO2 ? Harrison’s 17th Ed 295 Harrison’s 17th Ed 294 A. 0. AJAY MATHUR Cardiology 343 D.50:122-162 D. 28 mmol/L C. In acute respiratory acidosis. Pa O2 301 High urine chloride metabolic alkalemia with hypertension includes all except ? C. 311 In pure respiratory alkalosis.4 mmol/L A. Bartter’s syndrome ] and a 0. Gitleman syndrome 307 Diagnosis of respiratory acidosis requires measurement of ? C. Liddle’s syndrome 309 [HCO3–] falls by how much for each 10-mmHg decrease in PaCO2 303 In acute respiratory acidosis. All of the above A. Pa CO2 B. 58 mmol/L 312 Which of the following shift inside the cell in acute respiratory In respiratory acidosis. D. the [HCO3–] increases by 4 mmol/L for every 10-mmHg increase in PaCO2. [HCO3 ] levels increase by how . 4 mmol/L C. 4 mmol/L C. serum HCO3– usually does not increase A. 4 mmol/L In acute hypocapnia. Vasodilatation those of hypocalcemia. B. Excess bicarbonate administration D. 16 mmol/L B. there is an increase of 1 mmol/L of HCO 3– for every 10-mm Hg increase in PaCO2. 2 mmol/L A. 14 mmol/L A. 48 mmol/L It is unusual to see a plasma HCO3– < 12 mmol/L due to pure respiratory alkalosis.4 to 0. 343 Nephrology MCQ’s FOR MCQ’s FOR MEDICAL MEDICAL PROFESSIONALS PROFESSIONALS BY PROF. plasma HCO3– is usually more than ? Harrison’s 17th Ed 295 305 In respiratory acidosis. measurement of PaCO2 & arterial pH.50:122-162 C. 3 mmol/L B. 2 mmol/L A.3 to 0. [HCO3–] Disease-a-Month 2004. Liddle’s syndrome Harrison’s 17th Ed 295 302 High urine chloride metabolic alkalemia without hypertension A. 0. All of the above A. [HCO3–] falls by 2 mmol/L and 4 mmol/L for each 10-mmHg decrease in PaCO2 in acute and chronic hypocapnia respectively. Bartter’s syndrome 308 In chronic respiratory alkalosis a 1-mm Hg fall in PaCO2 causes a drop in [HCO3–] of ? D.2 mmol/L includes all except ? B. All of the above A. in acute hypocapnia ? much for every 10 mm Hg increase in PaCO2 ? Harrison’s 17th Ed 295 Harrison’s 17th Ed 294 A. papilledema is due to vasoconstriction secondary to loss of the vasodilator A. 1 mmol/L B. 3 mmol/L D. 0. Hyperviscosity 300 High urine chloride metabolic alkalemia includes ? D.4 to 0. Bartter syndrome effects of CO 2.50:122-162 In respiratory acidosis. B. Renal failure Disease-a-Month 2004.3 mmol/L Disease-a-Month 2004.2 to 0. Cushing disease C. 3 mmol/L B.1 to 0.5 mmol/L B. 30 meq/day C.260 meq/day A.6 . 0. 30 C. or hypotension develops. 50 .1. Table 7 Respiratory alkalosis is often an early finding in gram-negative septicemia. Table 7 B. 50 B. the plasma potassium rises by ? Harrison’s 16th Ed.3000 mg/day 140 mEq/L. and HCO3 is 20 mEq/L ? Harrison’s 16th Ed.25 meq/day Harrison’s 17th Ed 295 B. Acute respiratory acidosis D.10. 0. 30 A. 60 . 200 . PO 4– B.3. 10 Harrison’s 17th Ed. number of different cell types found is about ? D. Acute respiratory alkalosis 322 Normal range of urine uric acid is ? C. 264 Chapter 277. 100 . 800 . 25 B.000 . Cellular and Molecular A. Fever C. 1800 . 2. 250 . 25 . 1. Table 7 Harrison’s 18th Ed 2280 A. Respiratory alkalosis D. Table 7 D. 2.800 mg/day C.344 MCQ’s FOR MEDICAL PROFESSIONALS BY PROF.300 meq/day C. 320 Normal range of urine potassium is ? 313 Which of the following is an early finding in gram-negative Harrison’s 17th Ed. Inositol Harrison’s 16th Ed. Urea A.0 . 150 . 300 .87 g/day 316 Which of the following is an ineffective osmole ? Harrison’s 16th Ed.6 g/day Acute respiratory alkalosis causes intracellular shifts of Na . A. 1. 100 . 10 . 100. 10 .0 g/day hypokalemia is usually minor. AJAY MATHUR Nephrology C. All of the above C. Cl.1500 mosmol/kg 317 For each decrease in blood pH of 0.6 mmol/L C. hypoxemia.250 mg/day Chronic respiratory alkalosis is the most common acid-base disturbance in critically ill patients. 50 . Glutamine C.Hypocapnia-induced + + – D.100 meq/day A.900 mosmol/kg D.57 g/day D. 80 C. 15 Harrison’s 17th Ed.17 g/day C.37 g/day D.8 mmol/L Harrison’s 18th Ed 2280 318 Normal range of urine ammonia is ? A. 17 . Chronic respiratory acidosis Harrison’s 17th Ed. 100 .150 meq/day 326 How many glomeruli does each kidney contain in a normal- 319 Normal range of urine creatinine is ? birth-weight adult ? Harrison’s 17th Ed.is 100 mEq/L. 30 .1800 mg/day 315 What is the anion gap in a patient if albumin is 2 gm/dL. 5 . 0. Na+ is D.2 g/day D. before fever.200 meq/day B.50 meq/day D. K . 0. 20 A. 37 . 57 . Hypoxemia D.300 mosmol/kg D. 252 324 Normal range of urine osmolality is ? A.6 g/day A.2.2. Table 7 septicemia ? A. D. and PO 4 . 265 323 Normal range of urine urea nitrogen is ? A. B.2 .150 mosmol/kg C.60 meq/day B. Betaine B.100 meq/day 314 Most common acid-base disturbance in critically ill patients is ? Harrison’s 17th Ed 295 C. Chronic respiratory alkalosis A. Table A-5 B. 900 .100 meq/day Kidney is one of the highly differentiated organs in body containing nearly 30 different cell types. Table 7 B. 6 . 50 . Hypotension 321 Normal range of urine sodium is ? Harrison’s 17th Ed. 260 .6 .460 meq/day B.5 mmol/L Biology of the Kidney B.7 mmol/L 325 In kidneys. Harrison’s 18th Ed 2281 A. Angiotensin II space ? B. Autonomous vasoreactive reflex in afferent arteriole C. 400. Nitric oxide B. Renal interstitium B. AJAY MATHUR Cardiology 345 B. ~ 80 % 338 Juxtaglomerular apparatus is located in which of the ~20% of the renal plasma flow is filtered into Bowman’s space.000 A. Tubuloglomerular feedback 329 Which of the following about kidney structure is false ? Harrison’s 18th Ed 2281 C. ~ 60 % Angiotensin II & reactive oxygen species enhance. Cortical nephrons have short loops of Henle D. All of the above nephrons create a hyperosmolar gradient that allows for production of concentrated urine. None of the above A. 1 C. Distal tubule B. Myogenic reflex in afferent arteriole B. and angiotensin II mediated vasoconstriction of 328 Which of the following about kidney structure is false ? the efferent arteriole. 4 Henle called macula densa that act as sensors of solute concentration & flow of tubular fluid. the driving force for 327 Contractile actin-myosin fibers are found in which of the filtration falls to zero before reaching the efferent arteriole. while nitric oxide blunts tubuloglomerular feedback. 3 Tubuloglomerular feedback is mediated by specialized cells in thick ascending limb of the loop of D. Thick ascending limb of the loop of Henle A. Tubuloglomerular feedback D. Adenosine C. 900. Angiotensin II mediated vasoconstriction of efferent arteriole D. Angiotensin II mediated vasoconstriction of efferent arteriole A. following tissues of kidney ? Harrison’s 18th Ed 2280 333 Autoregulation of glomerular filtration is the result of ? A. Zero C. Juxtaglomerular apparatus Majority of nephrons are cortical. 337 Which of the following blunts tubuloglomerular feedback ? Harrison’s 18th Ed 2282 331 What percentage of renal plasma flow is filtered into Bowman’s A. Juxtamedullary C. None of the above B. Cortical nephrons perform most of glomerular filtration Harrison’s 18th Ed 2281 D.000 C. ~ 20 % C. Cortical nephron has two capillary beds arranged in series separated by efferent arteriole that regulates hydrostatic pressure in both capillary beds. All of the above C. Peritubular capillaries surrounding cortical nephrons are 335 Tubuloglomerular feedback is mediated by ? shared among adjacent nephrons Harrison’s 18th Ed 2281 D. Reactive oxygen species Harrison’s 18th Ed 2281 A. tubuloglomerular feedback. Juxtamedullary D. None of the above A. Majority of nephrons are cortical 334 Which of the following is the first line of defense against fluctuations in renal blood flow in autoregulation of glomerular B. 336 Macula densa is located in which of the following ? 330 Cortical nephron has how many capillary beds arranged in Harrison’s 18th Ed 2281 series ? A. 1200.000 B. Autoregulation of glomerular filtration is the result of an autonomous vasoreactive (myogenic) reflex in the afferent arteriole. Cortical nephrons perform most of the glomerular filtration & their afferent arterioles are larger than their respective efferent arterioles. ~ 45 % D. with glomeruli located in mid-to-outer cortex. Majority of glomeruli located in mid-to-outer cortex filtration ? C. Granular cells in the wall of afferent arteriole nephrons use separate capillaries called vasa recta. Cortical nephrons B.000 glomeruli in each kidney in normal-birth-weight adults. D. C. 2 D. Peritubular capillaries surrounding cortical nephrons are shared among adjacent nephrons. Collecting duct C. Proximal tubule Harrison’s 18th Ed 2281 B. whereas juxtamedullary nephrons have long loops of Henle. Mesangial cells contain contractile actin-myosin fibers to maintain structural integrity. 30 As the oncotic pressure rises along the length of the glomerular capillary. Juxtamedullary nephrons have long loops of Henle Myogenic reflex is a first line of defense against fluctuations in renal blood flow. 20 There are ~900. Glomeruli Harrison’s 18th Ed 2281 B. Macula densa have short loops of Henle. D. Mesangial cells A. All of the above Glomerular capillaries are in the midst of a mesangial matrix and together form the Bowman’s capsule. Afferent arteriole Harrison’s 18th Ed 2281 . following ? Harrison’s 18th Ed 2282 332 Driving force for glomerular filtration at efferent arteriole is ? A. 10 D. 345 Nephrology MCQ’s FOR MCQ’s FOR MEDICAL MEDICAL PROFESSIONALS PROFESSIONALS BY PROF. Active transport Harrison’s 18th Ed 2282 B. Ascending thick limb Ion-translocating ATPases that mediate active transport (“pumps”) are the ubiquitous Na +/K +- Loop of Henle consists of 3 major segments . antiporters or exchangers are related to ? Renin is a proteolytic enzyme that catalyzes conversion of angiotensinogen to angiotensin I. Bicarbonate exits the cell through a basolateral Na+/HCO3– co-transporter. Salicylates D. Angiotensin I to angiotensin II D. K+ B. All of the above either in the same direction (symporters or co-transporters) or in opposite directions (antiporters or exchangers) across the cell membrane. cephalosporins. Juxtaglomerular apparatus is located within the wall of afferent arteriole near macula densa and its granular cells release renin. Facilitated diffusion B. Na+/K+-ATPase pump is on basolateral membranes C. transporters (facilitated diffusion). Proximal tubule contains leaky epithelia. Angiotensin I is converted to angiotensin II by angiotensin-converting enzyme (ACE). Distal convoluted tubule B. Harrison’s 18th Ed 2283 340 Angiotensin-converting enzyme catalyzes conversion of ? A. Descending thin limb B. Collecting duct C. Carbonic acid diffuses into the cell Carbonic acid is metabolized by brush border carbonic anhydrase to water and carbon dioxide. Transporters B. AJAY MATHUR Nephrology B. whereas distal nephron segments. Distal convoluted tubule . Channels A. Filtered bicarbonate is converted to carbonic acid in lumen D. All of the above D. and Cl–. Proximal convoluted tubule A. All of the above D. All of the above D. Angiotensinogen to angiotensin I C. channels (passive transport). Angiotensin I to angiotensin II D. Cephalosporins D. Active transport Harrison’s 18th Ed 2282 B. and Ca2+-ATPases. Penicillins C. 345 Carriers or uniporters are related to ? Harrison’s 18th Ed 2283 339 Renin catalyzes the conversion of ? A. Aquaporin-1 present on apical & basolateral membranes B. K+. and salicylates are not filtered at the glomerulus and are secreted by active transport). Na+. proximal tubule by specific transporters. Distal tubule D. Proximal convoluted tubule A. Many transporters operate by translocating two or more ions/solutes D. Secondary active transport C. 341 Leaky epithelia is present in ? 347 Which of the following statements is false ? Harrison’s 18th Ed 2287 Harrison’s 18th Ed 2282 A. All of the above Movement of solutes & water across cell membranes is made possible by pumps (active transport). Angiotensin II to angiotensin III Facilitated diffusion is a specialized type of passive transport mediated by simple transporters called carriers or uniporters. Co-transporters C. Cl – C. Secondary active transport C.descending & ascending thin limb & ascending thick ATPase. where by cytoplasmic carbonic anhydrase reforms tubule & collecting duct contain tight epithelia with little water permeability.346 MCQ’s FOR MEDICAL PROFESSIONALS BY PROF. Ca 2+ -ATPase C. Passive transport A. Na + C. Collecting duct In kidney. Na+/K+-ATPase A. H+-ATPase B. All of the above 346 Symporters or co-transporters. 344 Which of the following solute moves by simple diffusion or 350 Site of action of loop diuretics is ? passive transport ? Harrison’s 18th Ed 2285 Harrison’s 18th Ed 2283 A. such as distal convoluted Dissolved carbon dioxide diffuses into the cell. 343 Ion-translocating ATPases include ? 349 Which of the following is not a major segment of loop of Henle ? Harrison’s 18th Ed 2283 Harrison’s 18th Ed 2285 A. and co-transporters (secondary Penicillins. limb. Loop of Henle B. Angiotensin II to angiotensin III D. 342 Facilitated diffusion occurs through which of the following ? Harrison’s 18th Ed 2283 348 Which of the following drugs is not filtered at the glomerulus ? A. the H+-ATPases. Pumps Harrison’s 18th Ed 2285 B. Passive transport A. Facilitated diffusion B. Efferent arteriole C. carbonic acid which dissociates into H+ and bicarbonate anions. movement by simple diffusion or passive transport occurs of water. Angiotensinogen to angiotensin I C. Ascending thin limb D. Descending thick limb C. All of the above Descending thin limb of loop of Henle is highly water-permeable due to dense expression of aquaporin-1 water channels.a salt-wasting disorder associated with hypokalemic alkalosis and hypocalciuria. Gitelman’s syndrome 360 Loss-of-function mutations of SLC12A3 encoding the apical B. TRPM6 and TRPM7 353 Tubular concentration of K+ is ? A molecular complex of TRPM6 and TRPM7 proteins is critical for Mg 2+ reabsorption in the thick Harrison’s 18th Ed 2285 ascending limb of Henle. Ascending thick limb B. ~ 4 meq/L Harrison’s 18th Ed 2285 C. Water permeability is negligible in the ascending limb. AJAY MATHUR Cardiology 347 D.e. Na+/K+-ATPase B. about 4 meq/L. Potassium recycling contributes to a positive electrostatic charge in lumen relative to the interstitium. Collecting duct D. Mutations in CLDN16 encoding paracellin-1. Bartter’s syndrome Bartter’s syndrome is an inherited disorder of thick ascending limb. All of the above Site of action of loop diuretics is loop of Henle. Bartter’s syndrome Loss-of-function mutations in genes encoding components of Na+/K+/2Cl– co-transporter (NKCC2). Liddle’s syndrome B. Mg2+ reabsorption collecting duct is false ? B. Liddle’s syndrome 355 Loss-of-function mutations in which of the following can cause Bartter’s syndrome ? Loss-of-function mutations of SLC12A3 encoding the apical Na+/Cl– co-transporter cause Gitelman’s syndrome . Apical Ca++-selective channels (TRPV5) C. WNK-1 & WNK-4 B. It leads to a salt-wasting renal disease with hypokalemia & metabolic alkalosis. KCNJ1 cause ? Harrison’s 18th Ed 2285 C. CLCNKB A. Hypertonic medullary interstitium Tubular concentration of K is similar to plasma i. D. Gitelman’s syndrome D. D. Ca2+ reabsorption Harrison’s 18th Ed 2286 C. Gordon’s syndrome A. TRPM3 and TRPM4 D. 351 Water-permeability is highest in ? 357 Which of the following is a transmembrane protein located Harrison’s 18th Ed 2285 within the tight junction complex ? A. Bartter’s syndrome Na+/Cl– co-transporter cause ? Harrison’s 18th Ed 2285 C. NKCC2 361 Mutations in genes encoding WNK kinases. Paracellin-1 D. Na+/Cl– co-transporter critical for Mg2+ reabsorption in thick ascending limb of Henle ? Harrison’s 18th Ed 2286 B. Isotonic medullary interstitium D. Any of the above B. Main Na + reabsorbing cells . Potassium recycling A. Hypotonic medullary interstitium D. 352 Which of the following is the primary target for loop diuretics ? Harrison’s 18th Ed 2285 358 A molecular complex of which of the following proteins is A. Gordon’s syndrome D. TRPM5 and TRPM6 The Na+/K+/2Cl– co-transporter is the primary target for loop diuretics. Na+/K+/2Cl– co-transporter A. Harrison’s 18th Ed 2285 A. a transmembrane protein located within the tight junction complex. Hensin C. + C. Harrison’s 18th Ed 2285 A. leads to familial hypomagnesemia with hypercalcuria and nephrocalcinosis. Descending thin limb Harrison’s 18th Ed 2285 B. C. ~ 6 meq/L A. A. apical K + channel (KCNJ1) or basolateral Cl – channel (CLCNKB. TRPM1 and TRPM2 C. BSND) can cause Bartter’s C. WNK-1 and WNK-4 cause pseudohypoaldosteronism interstitium is caused by ? type II or Gordon’s syndrome characterized by familial hypertension with hyperkalemia. Liddle’s syndrome 356 Positive electrostatic charge in the lumen relative to Mutations in genes encoding WNK kinases. Proximal convoluted tubule C. Gordon’s syndrome syndrome. ~ 8 meq/L B. Any of the above 354 Which of the following is an inherited disorder of the thick ascending limb ? Loop of Henle and and vasa recta establishes a hypertonic medullary interstitium by phenomenon called countercurrent multiplication. ~ 2 meq/L 359 “Countercurrent multiplication” leads to ? B. Ascending thin limb A. Harrison’s 18th Ed 2285 362 Which of the following about principal cells of cortical A. 347 Nephrology MCQ’s FOR MCQ’s FOR MEDICAL MEDICAL PROFESSIONALS PROFESSIONALS BY PROF. Gitelman’s syndrome D. Nedd4-2 D. D. Secreted by renal tubular epithelia B. 371 Urodilatin resembles which of the following ? 365 Activating mutations occur in which of the following channels Harrison’s 18th Ed 2286 in Liddle’s syndrome ? A. Extracellular protein increases activity of apical membrane Na+ channel. and spironolactone. 368 Which of the following water channels “aquaporin” is not 374 Aldosterone mediates its effects by which of the following present in inner medullary collecting duct cells ? genes ? Harrison’s 18th Ed 2286 Harrison’s 18th Ed 2288 A. 3. Aquaporin 2 A. In + – Aldosterone binds to cytoplasmic mineralocorticoid receptors in principal cells of collecting duct & bicarbonate excess with alkalemia type B intercalated cells predominate. D. Increase activity of basolateral Na+/K +-ATPase D. Hypernatremia D. SGK1 . Cl - D. 373 Aldosterone leads to which of the following in principal cells 367 Extracellular protein hensin is best related to ? of the collecting duct ? Harrison’s 18th Ed 2286 Harrison’s 18th Ed 2288 A. Aquaporin-2 B. K+-sparing diuretics. Acid & bicarbonate secretion collecting duct cells D. Atrial natriuretic peptide Harrison’s 18th Ed 2286 B. 372 Which of the folowing TRPV channels is osmoreceptive ? Harrison’s 17th Ed 1747 366 All of the following are features of Liddle’s syndrome except ? A. Site of action of K -sparing diuretics + Inner medullary collecting duct cells have vasopressin-regulated water channels (aquaporin-2 on apical & aquaporin-3 & 4 on basolateral membrane). Na + C. and 4 are regulated by vasopressin in the collecting duct. Podocytes C. Acid & bicarbonate secretion Aquaporin 1 is active in all water-permeable segments of proximal & distal tubules. AJAY MATHUR Nephrology B. All of the above In acidemia. TRPV4+ C. Principal cells are the main Na+ reabsorbing cells and the site of action of aldosterone. Aquaporin-1 A. kidneys uses type A intercalated cells to secrete excess H & generate more HCO3 . Acid & bicarbonate reabsorption 2. Acid & bicarbonate reabsorption C. TRPV3+ B. while aquaporins D. Metabolic alkalosis TRPV4+ neuronal cells connected to supraoptic and paraventricular nuclei in hypothalamus are osmoreceptive. None of the above Type B intercalated cells mediate bicarbonate secretion and acid reabsorption. Principal cells A. CLCN5 C. Aquaporin 3 B. K+ D. hypertension. Aldosterone A. Acid secretion & bicarbonate reabsorption A. Increase activity of apical membrane Na+ channel B. Aquaporin-1 water channels are densely D. HCO3. None of the above expressed in descending thin limb of loop of Henle. TRPV2+ A. Bicarbonate secretion & acid reabsorption D. Bicarbonate secretion & acid reabsorption B. K+-ATPase. SLC2A2 B.348 MCQ’s FOR MEDICAL PROFESSIONALS BY PROF. They modulate release of vasopressin by posterior lobe of the pituitary gland. Interacts with apical receptors on inner medullary C. Increase activity of apical membrane K+ channel C. and metabolic alkalosis. Acid secretion & bicarbonate reabsorption C. Aquaporin 4 C. Site of action of aldosterone D. Aquaporin-4 C. Intercalated cells B. PGE2 C. 364 Which of the following is true for type B intercalated cells of 370 Which of the following is false about renal natriuretic peptide cortical collecting duct ? (urodilatin) ? Harrison’s 18th Ed 2286 Harrison’s 18th Ed 2286 A. In Liddle’s syndrome. Hypokalemia C. apical membrane K+ channel & basolateral Na+/ ‘hensin’ mediates this adaptation. Aquaporin 1 Harrison’s 18th Ed 2286 B. Mesangial cells D. Hypertension Vanilloid receptors Transient receptor potential (TRPV) channels respond to changes in tonicity. Attenuates net Na+ reabsorption Type A intercalated cells of cortical collecting duct mediate acid secretion and bicarbonate reabsorption. TRPV1+ Harrison’s 18th Ed 2286 B. 369 Which of the following aquaporin is not regulated by vasopressin in the collecting duct ? 363 Which of the following is true for type A intercalated cells of Harrison’s 18th Ed 2286 cortical collecting duct ? A. Aquaporin-3 C. activating mutations occur in epithelial Na + channel causing increase in Na + reclamation that produces hypokalemia. Renin B. Prerenal ARF of Medicine (1960). Persistent renal hypoperfusion leads to ischaemic ATN D.365:417-430 379 Delta metabolic acidosis occurs when GFR falls below ? A. most of the blood supply is directed to ? Lancet 2005. HG Rennke A. Nephrotic syndrome 378 Urine specific gravity of 1. Calcium 383 RIFLE system is used to classify ? B. CASR Chapter 279 & 280. Acute Kidney Injury Aldosterone mediates its effects in part by serum/glucocorticoid-induced kinase 1 (SGK1). D. 1752 An exposition of the Intact Nephron Hypothesis was published by Bricker et. 75 mL/minute 386 In kidneys. C.5 377 Bricker’s trade-off hypothesis best relates to ? mg/dL Harrison’s 18th Ed 2289 D.5 mg/ hemodynamics associated with renal ablation are accompanied by structural lesions and suggest that sustained single nephron hyperfiltration (SNGFR) may have maladaptive consequences by dL. A. Adaption of the Kidney to 380 “Azo” means ? Renal Injury Harrison’s 18th Ed 2294 A. 25 mL/minute B. Neal Bricker Word azotemia is derived from “azo” meaning nitrogen. Dr. 40 mL/minute C.010 equals how many mosmol/L of 384 Which of the following about prerenal azotaemia is false ? urine osmolality ? Lancet 2005. 349 Nephrology MCQ’s FOR MCQ’s FOR MEDICAL MEDICAL PROFESSIONALS PROFESSIONALS BY PROF. al. 60 mL/minute D. 350 D. phosphorus & PTH levels. Sloughing of tubular cells into the lumen B. Appropriate physiological response to renal hypoperfusion B. Loss of brush border in proximal tubular cells Harrison’s 18th Ed 2292 A. 375 385 Typical histological features of human acute tubular necrosis Urine specific gravity of 1.365:417-430 C. All of the above D.365:417-430 C. Acute renal failure D. Interstitial oedema C. None of the above A. organic acids accumulate producing a delta metabolic acidosis. Barry Brenner of nitrogenous and non-nitrogenous waste products Barry Brenner in his hyperfiltration hypothesis in 1982 demonstrated that alterations in glomerular B. in American Journal A. Waste B. All of the above B. A. Increase in serum creatinine of > 20% if the baseline is > 2. Herbert Lubowitz D. 300 B. Urea B. Bricker defined the "intact nephron hypothesis" that the number of functioning nephrons is reduced in chronic renal disease and that the remaining nephrons undergo adaptations B. All of the above Harrison’s 18th Ed 2289 Prerenal acute renal failure is also called azotemia. Renal cortex . Elevations in PTH that occur in CKD restore calcium & phosphorus levels but lead to hyperplasia of parathyroid gland & secondary hyperparathyroidism (trade-off).365:417-430 When GFR falls below 25 mL/minute. Intrinsic ARF that maintain renal homeostasis. if the baseline is < 2. Phosphorus Lancet 2005. Meyer 382 Which of the following about acute renal failure is false ? Lancet 2005. Timothy W. Chronic renal failure Bricker’s trade-off hypothesis in 1972 is based on the effects of uremia on the balance between C. Acute & sustained increase in serum creatinine of 0. Fred Weisser Harrison’s 17th Ed. Doris Rolf 381 Term azotemia is used mostly for ? D. & Chronic Kidney Disease Chapter 278. Postrenal ARF 376 Who proposed the “Hyperfiltration hypothesis” ? D.010 equals ~350 mosmol/L of urine osmolality. Nitrogen A. JL Olson B. Glomerulonephritis calcium. Certain drugs can provoke acute prerenal failure C. 325 C. C. C.5 mg/dL damaging remnant glomeruli. include ? Lancet 2005. Ammonia 375 Whose name is associated with intact nephron hypothesis ? Harrison’s 18th Ed 2289 C. PTH A. Abrupt & sustained decrease in renal function with retention D.365:417-430 Harrison’s 18th Ed 2291 A Integrity of renal tissue is disturbed A. AJAY MATHUR Cardiology 349 D. ACE inhibitors 391 Which of the following about urine findings in pre-renal ARF is false ? C. Fractional excretion of sodium (%) > 2 More severe or prolonged prerenal ARF (renal hypoperfusion) may lead to ischemic injury of kidneys leading to acute tubular necrosis (ATN).365:417-430 D. Urine output < 200 mL/day with ECF overload 392 Which of the following about urine findings in pre-renal ARF is false ? C. D. Adenosine A. Fractional excretion of urea (%) < 35 .365:417-430 D. Fractional excretion of Lithium (%) > 20 acute tubular necrosis are ? C. Medullary rays false ? Lancet 2005.365:417-430 D. Urine osmolality > 300 blood supply occurs in ? C. Specific gravity about 1. None of the above A.365:417-430 C. Fractional excretion of uric acid (%) < 7 Oliguria is defined as urine output of <400 mL/day contributing to extracellular fluid overload. Prerenal ARF 393 Which of the following about urine findings in renal ARF is false ? B. Leukotrienes C4 and D4 B. None of the above A. Urinary interleukin 18 389 Vasoconstrictors implicated in the reduced renal blood flow in B. N-acetyl--glucosaminidase & alanine aminopeptidase Lancet 2005. Fractional excretion of urea (%) > 35 A. Urine output < 300 mL/day with ECF overload Lancet 2005.365:417-430 D. Urine osmolality > 500 include ? Lancet 2005. Low brush border enzyme levels 400 Acute tubular necrosis (ATN) relates best with ? D.365:417-430 D. All of the above B.365:417-430 A. 1752 A. All of the above A.365:417-430 D. Angiotensin-II-receptor blockers Lancet 2005. Outer medulla 395 Which of the following about urine findings in renal ARF is C. Renal cortex B. Angiotensin II B. Outer medulla 394 Which of the following about urine findings in renal ARF is C. Intrinsic ARF Lancet 2005. Urine sodium > 20 mmol/kg 387 In established acute tubular necrosis. Urine osmolality > 300 D. Urinary interleukin 18 D. Fractional excretion of urea (%) > 35 B. selective reduction in B. Medullary rays false ? Lancet 2005. Postrenal ARF B. High brush border enzyme levels Lancet 2005. None of the above A. Urine output < 400 mL/day with ECF overload A. None of the above Harrison’s 17th Ed. Fractional excretion of Lithium (%) < 7 C.365:417-430 D.365:417-430 D. Specific gravity about 1.365:417-430 C. Fractional excretion of sodium (%) > 2 A.010 C. N-acetyl--glucosaminidase false ? Lancet 2005. Kidney injury molecule 1 390 Which of the following about urine findings in pre-renal ARF is C. All of the above A. Fractional excretion of sodium (%) < 1 Lancet 2005. All of the above C. B. Urine sodium < 10 mmol/L B. All of the above A. Fractional excretion of urea (%) < 35 B.350 MCQ’s FOR MEDICAL PROFESSIONALS BY PROF. AJAY MATHUR Nephrology B. Fractional excretion of uric acid (%) > 15 388 Vasoconstrictors implicated in reduced renal blood flow in B.365:417-430 D. Thromboxane A2 396 Biomarkers proposed for the early diagnosis of acute renal C. 2301 C. NSAIDs D. Alanine aminopeptidase A. Prostaglandin H2 failure include ? Lancet 2005. Urine osmolality > 500 399 Oliguria is defined as ? Harrison’s 18th Ed.020 398 Agents that impair autoregulation of renal blood flow B. Fractional excretion of sodium (%) < 1 A. Endothelin 1 397 Biomarkers specifically higher in ischaemic ATN is ? Lancet 2005. Urine output < 100 mL/day with ECF overload D. Intestinal form of alkaline phosphatase acute tubular necrosis are ? C. Type I HRS is the more aggressive form Harrison’s 17th Ed 1755 A. C. Tacrolimus A. Prerenal ARF D. All of the above ischemic ATN ? Harrison’s 17th Ed 1754 Hypovolemia triggers neurohormonal responses that include activation of the sympathetic nervous system and renin-angiotensin-aldosterone system. Preferential constriction of efferent arterioles D. Systemic vasodilatation diseases of renal microcirculation & glomeruli and diseases of larger renal vessels. Ischemic or nephrotoxic tubular injury Endogenous nephrotoxins include calcium. All of the above A. A. urate. systemic A. Initiation 404 Which of the following drugs cause impairment of renal B. Renin-angiotensin-aldosterone system ascending limb of the loop of Henle. C. Release of arginine vasopressin 409 Urine output is lowest in which of the following phases of D. All of the above A. C. Angiotensin II also induces preferential constriction of efferent arterioles. Radiocontrast agents B. Extension 403 Which of the following is an action of Angiotensin II ? Harrison’s 17th Ed. Oxalate B. Increased biosynthesis of Prostaglandin E2 D. 2294 D. Maintenance A. Kidneys are structurally normal D. C.2 weeks. Myeloma light chains Harrison’s 17th Ed 1754 D. hemoglobin. low cardiac output. Urate A. Postrenal ARF Harrison’s 17th Ed 1755 Hepatorenal syndrome (HRS) is a unique form of prerenal ARF. Cyclosporine 405 Hepatorenal syndrome (HRS) is a form of ? Harrison’s 17th Ed 1753 C. Low cardiac output Intrinsic causes of ARF can be ischemic or nephrotoxic tubular injury. Intratubular obstruction 412 Endogenous nephrotoxins include all except ? D. Calcium B. B. Kidneys fail due to intense renal vasoconstriction 413 Which of the following is an endogenous nephrotoxin ? C. and release of arginine vasopressin. S1 segment of proximal tubule vasodilatation or selective intrarenal vasoconstriction. C. Angiotensin-converting enzyme inhibitors Diuresis occurs in the recovery phase. Sympathetic nervous system Ischemic renal injury is most prominent in S3 segment of PCT and in medullary portion of the thick B. tubulointerstitial diseases. Cortical thick ascending limb of loop of Henle A. Selective intrarenal vasoconstriction 408 Ischemic renal injury is most prominent in ? D. None of the above B. Initiation B. 351 Nephrology MCQ’s FOR MCQ’s FOR MEDICAL MEDICAL PROFESSIONALS PROFESSIONALS BY PROF. Recovery B. All of the above Harrison’s 17th Ed 1754 Prerenal ARF can complicate any disease that induces hypovolemia. B. 1753 C. urine output is lowest due to reduced GFR and uremic complications may appear. A. Recovery B. Increased biosynthesis of prostacyclin During maintenance phase that lasts for 1 . Tubulointerstitial diseases . ACE inhibitors & angiotensin II receptor blockers. Harrison’s 17th Ed 1754 A. 1753 D. AJAY MATHUR Cardiology 351 401 Prerenal ARF can complicate which of the following ? C. All of the above B. Diseases of renal microcirculation & glomeruli Harrison’s 18th Ed. Angiotensin II receptor blockers 411 Which of the following cause kidney injury through intrarenal D. myoglobin. All of the above vasoconstriction ? Harrison’s 17th Ed 1755 Impairment of renal autoregulatory responses is caused by COX inhibitors (NSAID’s). B. Hemoglobin 407 Which of the following leads to intrinsic ARF ? C. oxalate & myeloma light chains. Intrinsic ARF Radiocontrast agents (cyclosporine & tacrolimus) cause kidney injury through intrarenal vasoconstriction. Myoglobin D. Magnesium 406 Which of the following is false about hepatorenal syndrome ? Harrison’s 17th Ed 1753 C. A. Maintenance A. S3 segment of proximal tubule Harrison’s 17th Ed. S2 segment of proximal tubule 402 Hypovolemia leads to activation of ? C. 1753 Table 273-1 C. All of the above 410 Diuretic phase occurs in which of the following phases of Angiotensin II increases biosynthesis of vasodilator prostaglandins (Pg E2 & prostacyclin) resulting in ischemic ATN ? afferent arteriolar vasodilation. Nonsteroidal anti-inflammatory drugs D. Extension autoregulatory responses ? Harrison’s 17th Ed. Patients with prerenal ARF typically have a C.365:417-430 A. All of the above Hyperkalemia. 1758 Harrison’s 17th Ed. Hyperkalemia A. 1756 Table 273-2 A. Renal artery thrombosis B. thrombosis. HUS / TTP. Pigmented “muddy brown” granular cast is characteristic of ATN due to ischemic or nephrotoxic etiology. hematuria & myoglobinuria. 1758 A. Any of the above Broad granular casts are characteristic of chronic kidney disease. [(urine sodium x plasma creatinine)÷ (plasma sodium x urine A. Ischemic or nephrotoxic ATN C. Ischemic or nephrotoxic ATN C. 420 Oxalate crystal is best related with ? Harrison’s 17th Ed.352 MCQ’s FOR MEDICAL PROFESSIONALS BY PROF. Renal artery thrombosis B. Thrombotic microangiopathy D.0% in prerenal ARF if patients are on diuretics or with preexisting CKD. Hemoglobinuria A.0%. 1758 Harrison’s 17th Ed. Round-shaped D. Renal artery thrombosis A. Glomerular injury A. Envelope-shaped Harrison’s 17th Ed. Acute tubulointerstitial nephritis B. Allergic interstitial nephritis C. Needle-shaped D. Postrenal ARF D. All of the above D. [(urine creatinine x plasma creatinine)÷ (plasma sodium x urine sodium)] x 100 D. Multiple myeloma C. 1756 Table 273-2 Lancet 2005. All of the above Tamm-Horsfall protein is secreted by epithelial cells of the loop of Henle. 1756 D. Atheroembolic disease . HUS / TTP A. multiple myeloma. salt-wasting syndromes or adrenal insufficiency. Needle-shaped A. severe anemia in the absence of hemorrhage suggests hemolysis. Elliptical-shaped of acute renal failure ? Harrison’s 17th Ed. AJAY MATHUR Nephrology 414 Increased serum LDH is found in which of the following causes D. or thrombotic microangiopathy. Loop of Henle FENa relates sodium clearance to creatinine clearance. HUS / TTP C. 415 Eosinophilia is found in which of the following causes of acute 422 Urine is strongly positive for heme by dipstick in ? renal failure ? Harrison’s 17th Ed. 1758 A. Round-shaped B. severe anemia in the absence of 419 Broad granular casts are characteristic of ? hemorrhage suggests ? Harrison’s 17th Ed. Hemolysis B. Hyperphosphatemia B. Prerenal ARF B. Ethylene glycol ingestion creatinine)] x 100 C. [(urine sodium x urine creatinine)÷ (plasma sodium x plasma 417 Tamm-Horsfall protein is secreted by epithelial cells of ? creatinine)] x 100 Harrison’s 17th Ed. 416 Hypocomplementemia is found in which of the following causes of acute renal failure ? 423 Fractional excretion of sodium (FENa) is calculated as ? Harrison’s 17th Ed. 425 In acute renal failure. All of the above D. FENa may be >1. Hypocalcemia C. hemolysis. 1758 A. Envelope-shaped C. In acute renal failure. Hemolysis B. systemic eosinophilia suggests ? A. hypocalcemia. Atheroembolic disease C. Distal convoluted tubule FENa of <1. 1756 Table 273-2 421 Hippurate crystal is best related with ? Harrison’s 17th Ed. Ethylene glycol toxicity causes oxalate (envelope-shaped) & hippurate (needle-shaped) crystals. [(urine sodium x plasma sodium)÷ (plasma creatinine x urine B. hyperuricemia and raised creatine kinase (MM isoenzyme) suggest a diagnosis of rhabdomyolysis. Proximal convoluted tubule creatinine)] x 100 B. 424 Presence of which of the following suggests a diagnosis of acute renal failure due to rhabdomyolysis ? 418 Pigmented “muddy brown” granular cast is characteristic of ? Harrison’s 17th Ed. Atheroembolic disease Urine is strongly positive for heme by dipstick in hemoglobinuria. acute urate nephropathy & tumor lysis syndrome. Hemolysis D. Chronic kidney disease D. Hematuria B. 1758 426 In acute renal failure. hyperphosphatemia. D. Multiple myeloma B. Elliptical-shaped Increased serum LDH is found in acute renal failure due to renal artery. 1758 Harrison’s 17th Ed. Myoglobinuria C. All of the above C. 1758 B. 1759 D. Glomeruler filtration rate (GFR) A. hypokalemia. Hemolysis 436 In acute renal failure. Hypocalcemia C. Urinary acidification mechanisms 434 Which of the following is an oral phosphate binder ? Harrison’s 17th Ed. 1761 D. Evidence of renal osteodystrophy . 1755 C. Urinary anion gap A. 1759 vasculature is characteristically normal. hypermagnesemia and metabolic acidosis. All of the above Vigorous diuresis during recovery phase of ARF may lead to intravascular volume depletion causing hypernatremia. B. for estimation of ? detachment of cells from basement membrane and occlusion of tubule lumens. All of the above A. 1755 A.365:417-430 D. Hyperkalemia C. Polyarteritis nodosa 433 Which of the following may prevent or attenuate ARF ? Harrison’s 17th Ed. B. and B. 1761 A. Low production of interleukin 10 430 Which of the following can occur during recovery phase of D. calcium acetate. AJAY MATHUR Cardiology 353 C. Fractional excretion of sodium D. Serum anion gap B. Metabolic acidosis A. Desmopressin ARF is complicated by hyponatremia. B. Estrogens C. Any of the above A. Absolute indications for dialysis are symptoms or signs of uremic syndrome. Multiorgan failure Hyperkalemia in acute renal failure may be severe in patients with rhabdomyolysis. Acidosis D. All of the above A. Anemia D. Aluminum hydroxide 428 Which of the following is not a feature of acute renal failure ? Harrison’s 17th Ed. Forced alkaline diuresis C. Divalent cation homeostasis C. potassium. 1759 D. 1759 437 Absolute indication for dialysis in ARF is ? Harrison’s 17th Ed. or acidosis. hyperphosphatemia. hyperkalemia. Hyperphosphatemia Hyperphosphatemia can be controlled by oral phosphate binders like calcium carbonate. Rasburicase. Sevalamer C. sevalamer & aluminum hydroxide. All of the above A. Tumor lysis syndrome Lancet 2005. B. Calcium carbonate ARF impairs renal excretion of sodium. C. Hypernatremia A. forced alkaline diuresis. High concentration of TNF- tumor lysis syndrome. hypocalcemia. estrogens or dialysis. Normal renal vasculature D. Patchy & focal necrosis of tubular epithelium B. Rhabdomyolysis Uremic bleeding may respond to desmopressin. mortality is high when there is ? C. hemolysis. All of the above D. Fractional excretion of sodium B. 431 Cockcroft . All of the above 432 Modification of Diet in Renal Disease (MDRD) equation is used Classic pathologic features of ischemic ATN are patchy & focal necrosis of the tubular epithelium. 1758 D. hypophosphatemia and hypocalcemia. Serum anion gap Harrison’s 17th Ed. 1760 D. hypomagnesemia. and water and perturbs divalent cation homeostasis and urinary acidification mechanisms. All of the above A. 1758 D. refractory hypervolemia. Allopurinol In acute renal failure. Glomeruler filtration rate (GFR) C. Urinary anion gap 439 Which of the following suggests chronic kidney disease ? B. Rasburicase 427 Acute renal failure impairs which of the following ? Harrison’s 17th Ed. B. B. Renal excretion of sodium. All of the above A. hyperkalemia. Dialysis 429 Hyperkalemia in acute renal failure is severe in ? Harrison’s 17th Ed. Hypokalemia B. All of the above ARF ? Harrison’s 17th Ed. potassium & water Allopurinol. N-acetylcysteine help in preventing development of ARF due to various causes. 353 Nephrology MCQ’s FOR MCQ’s FOR MEDICAL MEDICAL PROFESSIONALS PROFESSIONALS BY PROF. systemic eosinophilia suggests allergic interstitial nephritis.Gault equation is used for estimation of ? Harrison’s 17th Ed. atheroembolic disease or polyarteritis nodosa. 1759 438 Classic pathologic features of ischemic ATN include ? Harrison’s 17th Ed. Normal glomeruli C. Hypomagnesemia 435 Uremic bleeding may respond to administration of ? Harrison’s 17th Ed. Uremic syndrome B. Glomeruli & renal Harrison’s 17th Ed. A. Hypophosphatemia C. plasma levels of phosphate. HIV associated nephropathy D. despite unrestricted intake of these substances (curve C). 1639 451 Plasma concentration of which of the following remains normal A. 1755 A. All of the above D. Proximal convoluted tubule 446 The molecular weight of inulin is about ? B. Potassium D. Urate B. Sodium Lancet 2005. Loop of Henle Harrison’s 16th Ed. AJAY MATHUR Nephrology C. Their secretion contributes little to overall excretion. Calcium 445 Decreased GFR occurs due to ? Harrison’s 16th Ed. 444 Decreased GFR occurs due to ? Harrison’s 16th Ed. Haemodialysis hydrogen ions usually do not rise until the GFR falls to a small percentage of normal. Sodium C. B. Uremic pericarditis D. plasma concentrations remain normal throughout the course of CRF. Reduced Glomerular hydraulic pressure throughout the course of CRF ? B. 80 ml D. Potassium A. 5400 Findings that suggest chronic kidney disease include anemia. Diabetic nephropathy B. Reduced Glomerular Permeability C. Hydrogen B. 60 ml B. urate. All of the above B. All of the above D. All of the above D.354 MCQ’s FOR MEDICAL PROFESSIONALS BY PROF. Reduced Glomerular blood flow For sodium chloride (NaCl). 1639 C. The clinical course of CRF disease and HIV associated nephropathy. usually also approximates curve A. Small scarred kidneys C. Urate C. These depend largely on glomerular filtration for Kidney size may be normal or increased in diabetic nephropathy. Amyloidosis C. Intravenous calcium C. None of the above C. Diminished Filtration surface area 452 Site where AVP exerts its principal effect is ? Harrison’s 16th Ed 1641 D. Potassium & Hydrogen ions C. oral acetylcysteine with hydration significantly lowers the risk of ? A. Urea C. Distal convoluted tubule A. 4800 D. and potassium and D.365:417-430 B. Hydrogen B. All of the above Harrison’s 16th Ed 1640 443 Most rapid way to remove potassium in severe hyperkalemia is ? A. approximately the same concentration in Bowman’s space as in plasma. polycystic kidney urinary excretion. Infusion of glucose & insulin D. Hydrogen A. With progressive renal failure this pattern of response (curve B) reflects the participation of tubule transport mechanisms in the excretion of these substances. appearing at (radiologic or laboratory) and small scarred kidneys. Na–K exchange resin (sodium polystyrene sulfonate) In contrast to solutes of the curve A type. Uremic encephalopathy 450 Which of the following is a curve C-type solute ? D. evidence of renal osteodystrophy Molecular weight of inulin is ~5200. 5000 . 90 ml 449 Which of the following is not a curve B-type solute ? Harrison’s 16th Ed 1640 442 In a case of CRF. 70 ml C. Sodium Lancet 2005.73 m2 is defined as chronic kidney disease ? Harrison’s 16th. 1653 A. Cortical & papillary portions of collecting duct B. Urate B. Contrast nephropathy C. Elevated Bowman’s space hydraulic pressure Harrison’s 16th Ed 1640 C. Ed. Sodium A. 441 A persistent reduction in GFR to less than what level per minute 448 Which of the following are curve B-type solutes ? Harrison’s 16th Ed 1640 per 1. Potassium A. 440 Kidney size may be increased in which of the following chronic 447 Which of the following is curve A-type solute ? renal diseases ? Harrison’s 16th Ed 1640 Harrison’s 17th Ed. All of the above A. Phosphate (PO43-) A. It pass freely across glomerular filtration barrier. 1639 D. Rise in Plasma colloid osmotic pressure A. 5200 D.365:417-430 B. amyloidosis. All of the above Curve A-type solutes include urea and creatinine. Antacid overdose D. 1642 A. Glucose D. Two third D. Oncotic pressure exceeds hydraulic pressure C. About 4 million D. in glomerulus. Ethylene glycol ingestion In the earliest portion of PCT. All of the above . Hydrogen A. 300 to 350 nm 457 Site of action of loop diuretics is ? C. AJAY MATHUR Cardiology 355 453 What proportion of glomerular ultrafiltrate is reabsorbed in 459 The principle site of action of Parathyroid hormone (PTH) is ? the proximal tubules ? Harrison’s 16th Ed 1642 Harrison’s 16th Ed 1640 A. 200 to 250 nm At glomerulus. All of the above B. which of the following is true ? B. Hypoosmotic 454 In earliest portion of proximal tubule. Loop of Henle B. One third B. Medullary cystic disease D. Fat embolism 455 Early proximal convoluted tubule is the major site of 462 Each kidney contains how many glomeruli in the renal cortex ? reabsorption of ? N Engl J Med 2006. hydraulic pressure exceeds oncotic pressure favoring filtration. 400 to 450 nm Harrison’s 16th. which of the following is the principal ion that accompanies the reabsorption of C.354:1387-401 tubule by cotransport mechanisms. Hyperosmotic sodium ? D. Fenestrated endothelium 456 Normally. Isoosmotic B. Collecting duct D. Lactate B. Terminal duct ascending limb of Henle. All of the above B. and organic solutes like lactate are extensively reabsorbed in the proximal N Engl J Med 2006. None of the above 458 Normally. 1641 D. amino acids. Proximal convoluted tubule A. Distal convoluted tubule C. Ed. Proximal convoluted tubule 465 The terminal segment of the distal nephron is ? B. About 1 million A. A. Proximal convoluted tubule A. Polycystic disease C. 355 Nephrology MCQ’s FOR MCQ’s FOR MEDICAL MEDICAL PROFESSIONALS PROFESSIONALS BY PROF.354:1387-401 Harrison’s 16th Ed 1640 A. Ed. About 2 million B. Ed. Glomerular basement membrane Harrison’s 16th Ed 1641 C. A. This process occurs via a Na/H exchanger at the luminal brush border and is dependent on the activity of carbonic anhydrase. 500 to 550 nm A. None of the above Harrison’s 16th Ed 1640 A. Three fourth 460 Fluid that enters the distal convoluted tubule is always ? ~ Two-thirds of glomerular ultrafiltrate is reabsorbed isosmotically in proximal tubule with little Harrison’s 16th Ed 1641 change in osmolality or sodium concentration of unreabsorbed fraction. With water absorption. Chloride 461 Calcium oxalate monohydrate crystals in the urinary sediment are typical of ? B. Distal convoluted tubule C.354:1065-72 C. major quantity of phosphate is reabsorbed in ? 466 Which of the following is a “salt-wasting nephropathy” ? Harrison’s 16th Ed 1641 Harrison’s 16th. Chronic pyelonephritis B. Interdigitating podocyte foot processes A. About 3 million C. Any of the above A. Amino acids C. sodium along with chloride & bicarbonate are absorbed actively to keep ultrafiltrate iso-osmotic. Blunt abdominal trauma sodium. D. Hydraulic pressure exceeds oncotic pressure D.354:1387-401 D. and its mutations give rise to Bartter’s syndrome. Collecting duct D. Loop of Henle B. All of the above 463 Filtration barrier of capillary wall in glomerulus contains ? Glucose. B. Bicarbonate N Engl J Med 2006. D. 1642 C. Phosphate B. Papillary collecting duct Na:K:2Cl cotransporter is the site of action of the powerful loop diuretics in the medullary thick C. One half C. Thin ascending limb of Henle’s loop Harrison’s 16th. bicarbonate is the principal anion that accompanies reabsorption of C. Cortical collecting tubule D. Oncotic pressure equals hydraulic pressure 464 Glomerular basement membrane has a thickness of ? N Engl J Med 2006. Medullary thick ascending limb of Henle A. 4 C.157 D. 1-3 469 To convert the values for urea nitrogen to millimoles/liter. 1762 D. Nidogen B. 3 B.300 481 Normal annual mean decline in GFR with age is ? C.154 x (age) –0. Congenital bone disease Harrison’s 17th Ed. 1. 0. 6 468 Pierson’s syndrome is a form of ? N Engl J Med 2006.457 477 End-stage renal disease denotes which stage of CKD ? Harrison’s 17th Ed. D. 478 Modification of Diet in Renal Disease (MDRD) formula for multiply by ? estimating GFR (mL/min per 1.203 D.400 Harrison’s 17th Ed. D. 1. 3 C. 3-5 A. 1. 1762 470 To convert the values for creatinine to micromoles/liter. 1762 B. 0. 1500 & 3000 Da D. 0.73 m2) is ? A. (150-age x body weight in kg) / (72 x plasma creatinine) multiply by ? A.348:2543-56 by National Kidney Foundation ? A. 5 D.2229 A. 0. 78.1229 480 Middle molecules have a molecular mass between ? Harrison’s 17th Ed. 0.357 and typically corresponds to CKD stages 3 .250 A. 5000 & 8000 Da multiply by ? Compounds with a molecular mass between 500 & 1500 Da are middle molecules. 0. 4 D. (140-age x body weight in kg) / (72 x plasma creatinine) 473 To convert the values for phosphorus to millimoles per liter.4 B.3229 B.4 End-stage renal disease denotes stage 5 of CKD. A.5.150 (ml/minute) is ? Harrison’s 17th Ed.154 x (age) –0.56 x (PCr)–1. D. 0.4551 Harrison’s 17th Ed. 0. Congenital nephrotic syndrome B.86 x (PCr)–1. multiply by ? A. 2 A.356 MCQ’s FOR MEDICAL PROFESSIONALS BY PROF. 0. B.203 D. D. 0.500 A. 0.257 Term CRF applies to the process of continuing significant irreversible reduction in nephron number C. 0.73 m2 . AJAY MATHUR Nephrology 467 Main components of glomerular basement membrane are ? 475 Chronic kidney disease (CKD) is divided into how many stages N Engl J Med 2003.203 C. 68.7551 C. 2-4 multiply by ? C. 500 & 1500 Da C.154 x (age) –0.4 C. (120-age x body weight in kg) / (72 x plasma creatinine) C.5551 A. multiply by ? 479 Cockcroft-Gault equation for estimating creatinine clearance A.203 472 To convert the values for calcium to millimoles per liter. 0. Laminin A. 0. Congenital eye disease A.354:1387-401 CKD is divided into six stages by National Kidney Foundation [Kidney Dialysis Outcomes Quality Initiative (KDOQI)] according to the estimated GFR.6551 B.4 D. Congenital heart disease 476 Chronic renal failure typically corresponds to which of the following CKD stages ? C.154 x (age) –0. 471 To convert the values for glucose to millimoles per liter. 88.200 B. 98. 0. 0.450 C. 0. All of the above C. (130-age x body weight in kg) / (72 x plasma creatinine) D. B.4229 C. 1762 Table 274-2 B. 4-5 B. Type IV collagen Harrison’s 17th Ed. A. 1. 0. 0. ~1 mL/min per year per 1. 1762 Table 274-2 B. 3000 & 5000 Da 474 To convert the values for magnesium to millimoles per liter. 0. 5 D.350 B.66 x (PCr)–1.76 x (PCr)–1. 1763 A. ~1. 1654 Harrison’s 17th Ed. Hypertension B. Hepatitis A. 1762 Table 274-1 492 Plasma levels of which of the following hormones rise with A. Harrison’s 17th Ed. Benzoates Persistence in urine of >17 mg of albumin per gram of creatinine in adult males and 25 mg albumin per gram of creatinine in adult females signifies CKD. which is the most common cause of 493 Clinical abnormalities in uremia that develop only after chronic kidney disease ? initiation of dialysis therapy are all except ? Harrison’s 17th Ed.73 m2. All of the above . Prolactin GFR in CKD (mL/min per 1. persistence in urine of how much albumin A.73 m2 C. 1763 Table 274-3 Harrison’s 17th Ed. > 15 mg 490 Which of the following is a nitrogenous excretory product ? D. Renal excretory failure B. 80 mL/min per 1. Myoinositol Harrison’s 17th Ed. 1762 C. Indoles C. Muscular irritability D. Luteinizing hormone . D. 30 .5 mL/min per year per 1. B. Cystic kidney disease D. stage 2 = 60 . 60 . Dialysis disequilibrium syndrome C. Hypothermia Most frequent cause of CKD is diabetic nephropathy. > 23 mg 491 Which of the following is a nitrogenous excretory product ? Harrison’s 16th Ed. secondary to type 2 diabetes mellitus. 100 mL/min per 1. stage 3 = 30 2 C. Glomerulonephritis C. All of the above B. persistence in urine of how much albumin per B.Stage 0 = >90. Parathyroid hormone (PTH) D. > 13 mg Apart from the above ones. Peritonitis C.89 B.73 m2 Normal annual mean decline in GFR with age is ~1 mL/min per year per 1. loss of fluid & electrolyte homeostasis & hormone regulation and progressive systemic inflammation with vascular C.73 m2 B. 1762 C. All of the above D. Hypertensive nephropathy is a common cause of CKD in the elderly. 1763 Table 274-3 A. C. 1762 Harrison’s 17th Ed. End products of aliphatic amine metabolism B. Loss of hormone regulation D. Diabetes mellitus A.73 m2 & nutritional consequences. End products of aromatic amino acid metabolism C.73 m2 at the age of 70 years.73 m2 D. 70 mL/min per 1. Polyamines per gram of creatinine signifies chronic renal damage ? B. > 21 mg C. > 18 mg D. Phenols A. AJAY MATHUR Cardiology 357 B. 1654 C. > 17 mg Harrison’s 16th Ed. Urea 483 In adult males. reaching a mean 489 Uremic “Toxins” include ? value of 70 mL/min per 1.89.73 m2) in stage 5 of chronic kidney disease (CKD) ? D.73 m2 Uremic syndrome manifests as accumulation of toxins due to renal excretory failure. All of the above B. 357 Nephrology MCQ’s FOR MCQ’s FOR MEDICAL MEDICAL PROFESSIONALS PROFESSIONALS BY PROF. D. stage 4 = 15 .73 m ) . stage 1 = >=90.29 Harrison’s 16th Ed. Creatine gram of creatinine signifies chronic renal damage ? Harrison’s 17th Ed. Adynamic osteomalacia B. All of the above 486 Out of the following. ~2 mL/min per year per 1. By-products of protein & amino acid metabolism A. > 25 mg A. 1765 A. Urates and hippurates A.73 m2 488 Uremic “Toxins” include ? 482 Mean value of GFR at the age of 70 years is ? Harrison’s 16th Ed. Idiopathic ascites B. 90 mL/min per 1. 15 . Progressive systemic inflammation C. < 15 renal failure ? B. Hypotension & arrhythmias D.59 A. 1654 D. PTH is also a uremic toxin. 494 Clinical abnormalities in uremia that develop only after initiation of dialysis therapy are all except ? 487 Which of the following dysfunctions occur in uremic syndrome ? Harrison’s 17th Ed. All of the above Harrison’s 17th Ed. 1762 A. 1654 484 In adult females. Phenols 485 What is the GFR value (mL/min per 1.29 and stage 5 = <15.5 mL/min per year per 1. > 11 mg D.59. ~2. 1763 Table 274-3 B. Dialysis disequilibrium syndrome 499 Gastrointestinal disturbances in uremia that develop only after D. Amenorrhea C. Pallor with optimal dialysis & related therapy are all except ? Harrison’s 17th Ed. Idiopathic ascites A. Peritonitis B. Pericarditis D. 1763 Table 274-3 B. Uremic frost D. Pallor D. Carbohydrate intolerance Harrison’s 17th Ed. Leukopenia Harrison’s 17th Ed. Hypotension & arrhythmias improve with optimal dialysis & related therapy are all except ? 508 Dermatologic disturbances in uremia that usually improve with Harrison’s 17th Ed. 1763 Table 274-3 B. Carbohydrate intolerance Harrison’s 17th Ed. 1763 Table 274-3 B. Adynamic osteomalacia A. Anemia A. Secondary hyperparathyroidism B. Muscle cramps C. Myopathy initiation of dialysis therapy are all except ? Harrison’s 17th Ed. Anorexia. Myoclonus D. Increased Lp(a) level C. Vitamin D-deficient osteomalacia C. Vitamin D-deficient osteomalacia A. Muscle cramps B. Fatigue C. None of the above erythropoietin therapy are all except ? Harrison’s 17th Ed. Impaired mentation A. Infertility and sexual dysfunction D. All of the above C. 1763 Table 274-3 improve with optimal dialysis & related therapy are all except ? A. Sleep disorder with optimal dialysis & related therapy are all except ? Harrison’s 17th Ed. Bleeding diathesis B. Decreased high-density lipoprotein level D. Uremic fetor A. Fatigue therapy are all except ? Harrison’s 17th Ed. Muscular irritability D. Leukopenia C. Pruritus . 1763 Table 274-3 persist or even progress. 1763 Table 274-3 B. AJAY MATHUR Nephrology 495 Clinical abnormalities in uremia that develop only after 502 Endocrine-metabolic disturbances in uremia that tends to initiation of dialysis therapy are all except ? persist or even progress. Ecchymoses C. Asterixis after initiation of dialysis therapy are all except ? 507 Cardiovascular & pulmonary disturbances in uremia that usually Harrison’s 17th Ed. CHF / pulmonary edema C. Impaired mentation D. Lethargy B. 1763 Table 274-3 506 Neuromuscular disturbances in uremia that improve with dialysis and erythropoietin therapy are all except ? A. 1763 Table 274-3 505 Neuromuscular disturbances in uremia that develops only after initiation of dialysis therapy are all except ? A. 1763 Table 274-3 therapy are all except ? A. despite optimal dialysis & related A. 1763 Table 274-3 B. Idiopathic ascites C. Hypernatremia & hyponatremia C. Lethargy 500 Endocrine-metabolic disturbances in uremia that develop only D. 1763 Table 274-3 optimal dialysis & related therapy are all except ? A. Sleep disorders C. Hypocomplementemia A. nausea and vomiting Harrison’s 17th Ed.358 MCQ’s FOR MEDICAL PROFESSIONALS BY PROF. Coma 498 Gastrointestinal disturbances in uremia that usually improve D. Impaired growth & development D. Hypothermia B. Hepatitis Harrison’s 17th Ed. None of the above 496 Clinical abnormalities in uremia that improve with dialysis and erythropoietin therapy are all except ? 503 Endocrine-metabolic disturbances in uremia that tends to Harrison’s 17th Ed. Gastroenteritis B. 1763 Table 274-3 504 Neuromuscular disturbances in uremia that usually improve A. 1763 Table 274-3 B. Uremic fetor C. despite optimal dialysis & related Harrison’s 17th Ed. Uremic lung 501 Endocrine-metabolic disturbances in uremia that usually D. Asterixis C. Infertility & sexual dysfunction 497 Clinical abnormalities in uremia that improve with dialysis and D. at times with hemorrhagic elements giving it brown color. Diabetes mellitus 515 Which of the following is a consequence of a very low PTH B. low BP. vitamin K–dependent regeneration of matrix GLA protein which prevents vascular calcification. 1765 late-stage CKD patients ? Harrison’s 17th Ed. Warfarin B. hence the term brown tumor. In late-stage CKD. 514 Which of the following is a phosphate binder ? 520 Normocytic. the highest capacity of inhibiting soft tissue calcification. BP should be reduced to 125/75 mm Hg. Fracture In CKD. Diffuse monoclonal hyperplasia (“adenoma” or tertiary A. Harrison’s 17th Ed. Sevelamer C. Harrison’s 17th Ed. normochromic anemia is almost always present Harrison’s 17th Ed. Liver D. Transthyretin autonomous hyperparathyroidism) B. 1764 A. 140 / 90 mm Hg exposure in the form of calcium-containing phosphate binders or high-calcium dialysis solutions. Ectopic calcification D. obstructive uropathy & sickle cell nephropathy. C. Fibrinogen with CKD. Sickle cell nephropathy A. 359 Nephrology MCQ’s FOR MCQ’s FOR MEDICAL MEDICAL PROFESSIONALS PROFESSIONALS BY PROF. Warfarin therapy decreases Side effects of minoxidil include hypertrichosis and pericardial effusion. Stage 1 B. Albumin Owing to increased PTH production from parathyroid cells. Prazosin C. 1765 518 In CKD patients with diabetes or proteinuria > 1 gram/day. Any of the above C. Obstructive uropathy level ? Harrison’s 17th Ed. Nodular growth (monoclonal) within diffuse hyperplasia Harrison’s 17th Ed. 511 Brown tumor relates best with ? 517 Which of the following is associated with poor prognosis in Harrison’s 17th Ed. Calcium carbonate B. All of the above hypoaldosteronism ? Phosphate binders are calcium acetate. AJAY MATHUR Cardiology 359 509 Which of the following leads to hyporeninemic D. In CKD patients with diabetes or proteinuria > 1 gram/day. Reduced body mass index C. potassium levels may rise out of proportion to decline in GFR in conditions that lead to hyporeninemic hypoaldosteronism like diabetes. blood A. 512 Adynamic bone disease can result from ? Harrison’s 17th Ed. Aspirin Harrison’s 17th Ed. 135 / 85 mm Hg Adynamic bone disease is a state of reduced bone volume & mineralization. 1767 A. It has hyperparathyroidism). All of the above C. reduced BMI & hypolipidemia indicate the presence of a malnutrition- inflammation state. target PTH level should be between 150 and 300 pg/mL. All of the above C. Growth patterns may be diffuse hyperplasia (polyclonal). Diffuse hyperplasia (polyclonal) 516 Which of the following is not a negative acute-phase reactant ? B. nodular growth (monoclonal) within diffuse hyperplasia or diffuse monoclonal hyperplasia (“adenoma” or tertiary autonomous Fetuin (AHSG). Fetuin (AHSG) D. 1765 519 Pericardial effusion is a side effect of ? A. It results from excessive PTH suppression from the use of vitamin D preparations or from excessive calcium D. Calcium acetate A. Use of vitamin D preparations pressure should be reduced to ? Harrison’s 17th Ed. its cell mass increases progressively D. 1767 B. 513 Calciphylaxis is best related to which drug ? Harrison’s 17th Ed. High-calcium dialysis solutions B. All of the above B. 110 / 70 mm Hg C. Renoprotective effect of antihypertensive medications is gauged by consequent reduction of proteinuria. Subcuteneous tissue A. 125 / 75 mm Hg D. with poor prognosis. 1766 C. Excessive calcium-containing phosphate binders A. Brain C. Very low PTH levels cause adynamic bone disease. Hypolipidemia D. 1767 B. 1766 C. a glycoprotein is synthesized by hepatocytes & is present in the circulation. Eplerenone Calciphylaxis (livedo reticularis & ischemic necrosis) is seen almost exclusively in advanced CKD D. Labetalol due to vascular occlusion secondary to extensive vascular calcification. Stage 3 . Bone B. Minoxidil D. 1767 A. calcium carbonate and Sevelamer. fracture and ectopic calcification. All of the above 510 Which of the following growth patterns is seen when cell mass of parathyroid glands increases with CKD ? In CKD. Statins A. Low blood pressure B. 1766 by which stage of CKD ? Harrison’s 17th Ed. All of the above Bone histology in hyperparathyroidism may show bone cysts. 1764 A. Adynamic bone disease D. Stage 2 C. Stage 4 Harrison’s 17th Ed. 1662 A. Erythropoietin (EPO) therapy dermopathy or nephrogenic systemic fibrosis (NSF) between 5 & 75 days following exposure. 1768 D. 1767 C. B. 1768 C. A. nausea. All of the above B. anorexia. Unresponsive hyperkalemia Uremic fetor (urine-like odor on breath) is due to breakdown of urea to ammonia in saliva. Linear scleroderma A. Saliva 532 Clear indications for initiation of renal replacement therapy include ? B. Progressive neuropathy due to uremia 524 Which of the following is true for peripheral neuropathy of C.5 cm) favors CKD.0 cm which stage of CKD ? C. malnutrition. Pericarditis B. Stage 1 Finding of bilaterally reduced kidney size (<8. < 9. Unresponsive volume overload D. Progressive metabolic acidosis D. AJAY MATHUR Nephrology D. Anorexia & nausea not relieved by protein restriction C. seen in CKD. Diffuse fasciitis with eosinophilia B. gadolinium may precipitate nephrogenic fibrosing C. 1771 Peripheral neuropathy usually becomes clinically evident after stage 4 of CKD. A. even on dialysis due to deposition of retained pigmented metabolites called urochromes. IV conjugated estrogens Exposure to magnetic resonance contrast agent. Stage 2 C. 1769 Normocytic. B. C. 1768 D. 1769 intractable muscle cramping. Metformin Harrison’s 17th Ed. Harrison’s 17th Ed. Desmopressin (DDAVP) D.5 cm Harrison’s 17th Ed. Anemia of chronic disease & legs similar to scleromyxedema. Iron deficiency Nephrogenic fibrosing dermopathy. blood transfusions & EPO. Lower extremities involved more than upper Harrison’s 17th Ed. 1769 522 Abnormal bleeding time and coagulopathy in renal failure may A. Stage 4 527 Nephrogenic fibrosing dermopathy is similar to ? Harrison’s 17th Ed. Lung Harrison’s 17th Ed. Bone marrow fibrosis 528 Which of the following can precipitate nephrogenic fibrosing Primary cause of anemia in patients with CKD is insufficient production of erythropoietin (EPO) by dermopathy ? the diseased kidneys. 1771 C. History of hiccoughing Patients become more pigmented in advanced CKD. D. 1771 C. 1768 D. normochromic anemia is almost always present by stage 4 of CKD. < 9. All of the above B. Renochromes Harrison’s 16th Ed. Gadolinium B. 1770 IV conjugated estrogens. is a progressive subcutaneous induration on arms C. Stage 3 530 Clear indications for initiation of renal replacement therapy include ? D. Harrison’s 17th Ed. Scleromyxedema Harrison’s 17th Ed. Urochromes complications are ? D. A. All of the above A. All of the above 526 Skin pigmentation in advanced CKD is due to ? Clear indications for initiation of renal replacement therapy in CKD include pericarditis. encephalopathy. Harrison’s 17th Ed.5 cm 523 Peripheral neuropathy of CKD becomes clinically evident after B. B. Aluminum A. Guttate morphea 521 Primary cause of anemia in patients with CKD is ? B. Distal parts of extremities involved more than proximal A. Insufficient erythropoietin (EPO) D. & refractory fluid & electrolyte A. Mucous membranes of mouth A. All of the above A. Chromogens abnormalities (hyperkalemia). All of the above 529 CKD is likely if bilateral kidney size is ? Abnormal BT & coagulopathy in renal failure may be reversed temporarily with DDAVP. Chromolytes 533 Clinical clues indicating the imminent development of uremic C. < 10 cm A. D. Vitamin D be reversed temporarily with ? B.360 MCQ’s FOR MEDICAL PROFESSIONALS BY PROF. Encephalopathy CKD ? Harrison’s 17th Ed. Muscle irritability D. Sensory nerves involved more than motor 531 Clear indications for initiation of renal replacement therapy include ? B. cryoprecipitate. Refractory fluid & electrolyte abnormalities 525 Uremic fetor is due to breakdown of urea to ammonia in ? Harrison’s 17th Ed. Intractable pruritus . < 8. 2244 A. 1319 C. NaHPO4 – B. 1.5 mg/dL A. Dialysis in the Treatment D.25(OH)2D B.9 mg/dL B. cellulosynthetic and synthetic. 2241 Harrison’s 18th Ed. Asterixis C. 1.7 to 1.4 mg/dL A. while that of creatinine is 113 Da. All of the above membrane surface.5 . Paracellin-1 C. All of the above of Renal Failure 535 Calcium reabsorption in cortical thick ascending limb of Henle’s loop (cTAL) requires which of the following ? 542 What is the size of urea molecule ? Harrison’s 16th Ed.0 m2 Harrison’s 16th Ed. 1319 D.5 mg/dL D. . Hypokalemia 534 Clinical clues indicating the imminent development of uremic B. 2245 A.4. Hypocalcemia Cellulosic membranes are bioincompatible because of the presence of free hydroxyl groups on D. 5. 2.5 . 3. All of the above A.0 m2 with hypomagnesemia ? D. Harrison’s 16th Ed.0 m2. 113 Da C. 2322 A. Hypercalcemia A.2 mg/dL 545 Surface area of modern dialysis membranes in adult patients B.3. 132 Da 537 In adults. 2. All of the above A. 2.6 mg/dL is ? Harrison’s 18th Ed.2.5 . 2241 Harrison’s 18th Ed. 50 Da B. 0. 100 Da B. H2PO4– A. More free sulphydral groups on membrane surface A.5 mg/dL B. All of the above 538 Normally. 1662 C.0 m2 539 Which of the following should be supplemented in patients C. PTH A.0 m2 D.7 to 2. substituted cellulose. Subarachnoid hemorrhage Bioincompatibility is defined as the ability of dialysis membrane to activate complement cascade. More free hydroxyl groups on membrane surface 540 Which of the following prolong QT interval ? B. Calbindin-D28k D.2. Muscle twitching & cramps D. 2322 A.2 to 1. the concentration of magnesium in serum is ? Categories of dialysis membranes are cellulose. 2322 C. 4. above what level of fasting serum phosphate Size of urea molecule is 60 Da. Non-compliance of treatment Chapter 281. 361 Nephrology MCQ’s FOR MCQ’s FOR MEDICAL MEDICAL PROFESSIONALS PROFESSIONALS BY PROF. C. AJAY MATHUR Cardiology 361 C. HPO4 2– C. All of the above D. Synthetic D. 2243 544 Which of the following is a category of dialysis membrane ? Harrison’s 18th Ed. phosphorus exists in which form ? 543 What is the size of creatinine molecule ? Harrison’s 16th Ed.1. Phosphate dialysis membrane ? Harrison’s 18th Ed. 1. Cellulosynthetic C.5 . 123 Da D. Quinidine D. Less free hydroxyl groups on membrane surface Harrison’s 16th Ed. concentration is called Hyperphosphatemia ? Harrison’s 16th Ed. 0. Hyperkalemia complications are ? Harrison’s 16th Ed. Less free sulphydral groups on membrane surface B. B. 80 Da 536 In cells & in ECF. Hypothermia B. 70 Da D. 2322 D. Calcium Surface area of modern dialysis membranes in adult patients is ~1.4 to 2. 60 Da C.5 mg/dL C.5 . Morning nausea & vomiting 541 “Sine-wave” pattern in ECG is seen in ? Harrison’s 16th Ed. 3. Potassium 546 Which of the following improve the ‘Biocompatibility’ of a C. 2322 A. Cellulose B. 1. 500 to 750 mL/min D. how many hours of machine ranges from ? dialysis is required each week ? Harrison’s 18th Ed. Dialysis access A. 0. AJAY MATHUR Nephrology 547 Which of the following is a form of synthetic membrane ? 553 In hemodialysis. 120 L B. 2324 A. Current target is a urea reduction ratio (fractional reduction in BUN per hemodialysis session) of >65 . All of the above D. its use as buffer in dialysate is hyperplasia at anastomosis between graft & recipient vein. Bicarbonate C. a common cause of hypotension. Formaldehyde A. particularly among diabetics. Infection A. 500 to 800 mL/min D.2 D. 45 % C. dialysate flows in an opposite counter-current Harrison’s 18th Ed. 35 % B. 2323 Harrison’s 18th Ed. Ventriculo-atrial shunt B.2. presence of which of the following is a common 552 Most common dialysis access-related complication is ? cause of hypotension ? Harrison’s 18th Ed. 2322 direction at the rate of ? A. 551 ‘Breschia . glutaraldehyde and bleach are used as reprocessing agents. Polysulfone Harrison’s 18th Ed. 2324 A. 1. patient is exposed to approximately 555 What level of KT/V per treatment. Polymethylmethacrylate A. access.800 mL/minute. 9 to 12 hours C. 2324 B. 2323 minimal standards for adequacy among ESRD patients ? Harrison’s 18th Ed. 554 What level of urea reduction ratio (URR) per treatment. 12 to 24 hours D. Anaphylactoid reactions In Brescia-Cimino fistula. 55 % D. 180 L C. Polyacrylonitrile B. 2324 Harrison’s 18th Ed. due to intimal Because of the vasodilatory & cardiodepressive effects of acetate. polymethylmethacrylate. Thrombosis B. 1. Acetate B. Hypotension C. Peripheral arterial bypass D. 24 to 36 hours Blood flow rate in extracorporeal circuit in hemodialysis machine ranges from 250 .362 MCQ’s FOR MEDICAL PROFESSIONALS BY PROF. Perforation D. 100 to 300 mL/min C. cephalic vein is anastomosed end-to-side to radial artery for dialysis Hypotension is the most common acute complication of hemodialysis. 250 L D. 300 to 500 mL/min D.70% 549 During each hemodialysis. Calcium D. peracetic acid–hydrogen peroxide. 2324 A. 2323 Harrison’s 18th Ed.0 C. 1. Peritono-caval shunt C. and polyacrylonitrile membranes. defines minimal standards for how much water as dialysate ? adequacy among ESRD patients ? Harrison’s 18th Ed. Glutaraldehyde C. usually divided into three equal sessions. 50 to 100 mL/min A.4 Patients are exposed to ~120 liters of water during each dialysis treatment. All of the above C. 100 to 200 mL/min B. Aluminium Most important complication of AV grafts is thrombosis of the graft & graft failure. 2324 A. 2323 Harrison’s 18th Ed. Embolism C. minute.8 B. Current target is body water–indexed clearance x time product (KT/V) above 1. 800 to 1000 mL/min Examples of synthetic membranes are polysulfone. These are more biocompatible because of the absence of free Dialysate flows in an opposite counter-current direction at 500 . . 250 to 500 mL/min C. 80 L A. 65 % Formaldehyde. hydroxyl groups. 3 to 9 hours B. 558 In dialysate. defines 548 Reprocessing agent used in a hemodialyzer include ? Harrison’s 18th Ed. between 9 and 12 h of dialysis are required each week. 2324 A. Anemia D.Cimino fistula’ is used for ? 557 Most common acute complication of hemodialysis is ? Harrison’s 18th Ed. Infection B. 550 Blood flow rate in the extracorporeal circuit in hemodialysis 556 For the majority of patients with ESRD.500 mL/ For the majority of patients with ESRD. Peracetic acid – hydrogen peroxide B. Drainage of spent dialysate is performed manually B. 2325 C. None of the above C. Nocturnal intermittent peritoneal dialysis (NIPD) B. 2325 564 In peritoneal dialysis. Better tolerated hemodynamically D. 100 to 250 mL of isotonic saline A. Continuous arteriovenous hemodiafiltration with dialysis B. catheter-associated nonperitonitis infections. All of the above D. Ultrafiltration modeling C. 1667 560 Hypotension during dialysis can be prevented by ? A. Reducing volume removal during dialysis C. Withholding of antihypertensive medications D. the last exchange remains in abdomen A. 2324 B. All of the above C.3 liters of a dextrose-containing solution is infused into peritoneal cavity dialysis ? & allowed for 2 . 2326 A. In CCPD. All of the above Harrison’s 18th Ed. All of the above . Continuous veno-venous hemodiafiltration with dialysis D. All of the above B.250 mL of isotonic saline or 10 mL of 23% saturated hypertonic saline. All of the above 568 Which of the following is false for CAPD ? Harrison’s 16th Ed. 10 to 12 L Harrison’s 18th Ed. Cooling of dialysate during dialysis treatment D. Continuous cyclic peritoneal dialysis (CCPD) A. 1667 561 Hypotension during dialysis can be prevented by ? A. for how many hours dextrose-containing A. 2324 B. 4 to 6 L Lactate is the preferred buffer in peritoneal dialysis solutions. 4 to 6 hours D. Antibiotics Harrison’s 18th Ed. 2 to 4 hours Additives to peritoneal dialysis solutions are heparin. Residual uremia A. the abdomen is left dry during the day B. exchanges are performed in automated fashion hemodialysis ? Harrison’s 18th Ed. All of the above B. Highly effective in removing fluid D. Peritonitis 565 Advantage of Continuous renal replacement therapy (CRRT) over intermittent hemodialysis in ARF is ? B. 1 to 3 L D. 1666 A. Continuous arteriovenous hemodiafiltration without dialysis C.5 . 1. Gradual correction of biochemical abnormalities Complications of PD are peritonitis. All of the above 569 Which of the following statements is false ? Harrison’s 16th Ed. 100 . 10 mL of 23 % saturated hypertonic saline B. antibiotics and insulin. 2325 C. Use of higher concentrations of sodium in dialysate D. 6 to 8 L 571 Additive to peritoneal dialysis solutions may be ? D. In NIPD. 1667 562 Which of the following may prevent muscle cramps during A.4 hours. Avoiding heavy meals during dialysis D. 2324 B. Careful evaluation of dry weight C. Acetate A. C. Salt-poor albumin C. Harrison’s 18th Ed. Dialysis solution remains in peritoneal cavity through night A. 567 Which of the following is a type of ‘Peritoneal dialysis’ ? Harrison’s 16th Ed. 2325 C. Dialysis solution is manually infused into peritoneal cavity Harrison’s 18th Ed. how much dextrose-containing solution is infused in the peritoneal cavity ? B. All of the above B. None of the above C. 363 Nephrology MCQ’s FOR MCQ’s FOR MEDICAL MEDICAL PROFESSIONALS PROFESSIONALS BY PROF. Lactate 563 In peritoneal dialysis. 6 to 8 hours 572 Which of the following is a complication of peritoneal In peritoneal dialysis. Insulin A. 2324 Harrison’s 16th Ed. Heparin solution is allowed to remain in the peritoneal cavity ? B. 566 Continuous renal replacement therapy (CRRT) techniques administration of which of the following is useful ? include ? Harrison’s 18th Ed. 2325 A. Continuous ambulatory peritoneal dialysis (CAPD) Harrison’s 18th Ed. Weight gain Harrison’s 18th Ed. Midodrine D. metabolic disturbances and residual uremia. Bicarbonate Harrison’s 18th Ed. AJAY MATHUR Cardiology 363 559 In the management of hypotension during dialysis. weight gain. All of the above Hypotension during dialysis is managed with discontinuing ultrafiltration. and administration of salt-poor albumin. In CCPD. C. 1 to 2 hours D. C. Quinine sulfate before treatment 570 Preferred buffer in peritoneal dialysis solution is ? D. capillaries. Lupus nephritis A. 2326 Harrison’s 18th Ed. 2326 Harrison’s 18th Ed. always accompanied by ? pore membranes have a radius of ? Harrison’s 18th Ed.364 MCQ’s FOR MEDICAL PROFESSIONALS BY PROF. Chapter 283. Glomerular basement membrane (GBM) C. Chamber infections Inflammation of the glomerular capillaries is called glomerulonephritis. 40 / mm3 B. Crucin B. and idiopathic membranous nephritis typically are associated with immune deposits along the GBM. 574 Most common culprit organism in peritonitis as a complication of peritoneal dialysis is ? 580 Congenital nephrotic syndrome occurs due to mutations in ? Harrison’s 18th Ed. 1. Optimin C. 584 Persistent glomerulonephritis that worsens renal function is 578 Pores in the glomerular basement membrane (GBM) and slit.2 million D. 2335 A. 1667 Harrison’s 18th Ed. 2 nm B. Gram-positive bacilli B. All of the above Chemokines attract neutrophils. 2334 A. None of the above D. All of the above D. and tubular atrophy. Channel infections D. 582 In glomerulonephritis.8 million D. 2326 A. Mesangium B. . Funnel infections B. cocci. lupus nephritis. About 1. 10 / mm 3 A. All of the above D.6 million C. 0. Bicarbonate in PD solution can precipitate calcium B. AJAY MATHUR Nephrology 573 What value of peritoneal fluid leukocyte count denotes 579 Which of the following is instrumental in reclaiming filtered peritonitis ? albumin along the proximal tubule ? Harrison’s 18th Ed. 8 nm Persistent glomerulonephritis that worsens renal function is always accompanied by interstitial Pores in the GBM and slit-pore membranes have a radius of 4 nm.8 million glomerular capillary tufts are found in the two human kidneys. PAX-8 D. Tubular atrophy C. 1. 581 Glomerulonephritis refers to inflammation of ? 575 Nonperitonitis catheter-associated infections are termed as ? Harrison’s 18th Ed. Glomerular Diseases 583 Which of the following is associated with immune deposits along the GBM ? 577 Number of glomerular capillary tufts in the two human kidneys Harrison’s 18th Ed. Idiopathic membranous nephritis B. Capillaries C. Gram-negative bacilli D. Megalin D. Reglin Peritonitis is defined by a raised peritoneal fluid leukocyte count (100/mm 3. Glomerular capillaries A. 4000 to 9000 mg/day of albumin is filtered and is reclaimed by megalin and cubilin receptors along the proximal tubule. renal fibrosis. 2335 is about ? A. PROP-1 B. 2335 A. Nonperitonitis catheter-associated infections are often termed tunnel infections. 80 / mm 3 C. cytokines & proteases damage which 576 Which of the following statements is false ? of the following ? Harrison’s 16th Ed. 2334 A. These react with antigens & epitopes producing more cytokines & proteases that damage mesangium. Bicarbonate in PD solution can caramelize glucose C. Glomerular arterioles B. All of the above C. 2. Tunnel infections C. Gram-negative cocci C. PIT-1 Most common culprit organisms in peritonitis as a complication of peritoneal dialysis are gram-positive Congenital nephrotic syndrome occurs due to mutations in NPHS1 (nephrin) and NPHS2 (podocin). 2335 Harrison’s 18th Ed. Poststreptococcal glomerulonephritis Harrison’s 18th Ed. Renal fibrosis B. and/or GBM. 50% are PMN). 2335 Harrison’s 18th Ed. 100 / mm3 D. Acetate in PD solution can accelerate peritoneal sclerosis A.4 million Poststreptococcal glomerulonephritis. NPHS1 C. Gram-positive cocci A. macrophages & T cells into glomerular tuft. 2334 B. Interstitial nephritis A. nephritis. GBM D. 6 nm D. 4 nm C. Subacute bacterial endocarditis glomerulonephritis. Sitting position A. Lupus nephritis . 2337 Table 283-1 C. 2335 A. Upright position glomerulonephritis (RPGN) ? B. Papillary necrosis C. All of the above A. < 300 mg/day 594 Asymptomatic hematuria is due to ? Harrison’s 16th Ed. or renal vascular injury. interstitial Harrison’s 18th Ed. 2339 Table 283-2 D. sleep apnea. Half-moon-shaped collection of cells in Bowman’s space D. functional or transient proteinuria occurs in normal population. Crescentic glomerulonephritis is also called rapidly 589 Sustained isolated proteinuria is found in ? progressive glomerulonephritis Harrison’s 18th Ed. 2339 Table 283-2 nephritis. and <1 gram/ day. < 150 mg/day D. mesangioproliferative B. C. Mesangioproliferative glomerulonephritis Proteinuria only seen with upright posture is called orthostatic proteinuria. 2337 Table 283-1 D.30 % B. Henoch-Schönlein purpura 587 Normal 24-hour urine protein is ? B. Poststreptococcal glomerulonephritis D. Recumbent position Harrison’s 18th Ed. Interstitial nephritis B. gross hematuria is present. Proteinuria during fever.50 % 595 Which of the following is false about a ‘crescent’ ? Harrison’s 18th Ed. cystic kidney diseases. 2339 Table 283-2 C. 2339 C. < 30 mg/day D. None of the above A. IgA nephropathy C. Tubulointerstitial nephritis B. C. renal stones. obesity. Poststreptococcal glomerulonephritis Sustained isolated proteinuria is found in diabetic nephropathy. Cystic kidney diseases D. Proteinuria in UTI C. papillary necrosis. AJAY MATHUR Cardiology 365 585 Renal failure in glomerulonephritis best correlates 591 Functional proteinuria refers to ? histologically with the appearance of ? Harrison’s 18th Ed. Henoch-Schönlein purpura Harrison’s 18th Ed. exercise. Proteinuria in females A. Focal segmental glomerulosclerosis A. 2339 Table 283-2 Harrison’s 18th Ed. Subacute bacterial endocarditis D. Thin basement membrane (TBM) disease in the urine ? C. Antiphospholipid syndrome Harrison’s 18th Ed. Fever. Mesangioproliferative glomerulonephritis B. IgA nephropathy (Berger’s disease) 588 Albumin comprises what percentage of total proteins excreted B. Papillary necrosis C. 30 . All of the above A. Nail-patella syndrome B. A. 365 Nephrology MCQ’s FOR MCQ’s FOR MEDICAL MEDICAL PROFESSIONALS PROFESSIONALS BY PROF. and FSGS. Membranoproliferative glomerulonephritis D. C.90 % B. All of the above Benign. 2337 597 Which of the following does not present as rapidly progressive A. 592 Which of the following can present with gross hematuria ? 586 Cause of microscopic hematuria is ? Harrison’s 18th Ed. A. 1690 Normal 24-hour urine protein is < 150 mg/day. 20 . < 100 mg/day In IgA nephropathy & sickle cell disease. 40 . Sickle cell disease A. 10 . Mesangioproliferative glomerulonephritis 596 Which of the following does not present as rapidly progressive glomerulonephritis (RPGN) ? C. nil lesion. Proteinuria in upright posture D. nonsustained. Wegener’s granulomatosis 590 Orthostatic proteinuria is evident only in ? D.70 % A. 2337 Harrison’s 18th Ed. 2337 A. Diabetic nephropathy B. infiltrating macrophages C.70 % of the total protein excreted in the urine. Cystic kidney disease D. nephritis. FSGS Harrison’s 18th Ed. Composed of proliferating parietal epithelial cells and Albumin represents 30 . Lateral position B. emotional stress & congestive heart failure cause Renal failure in glomerulonephritis best correlates histologically with the appearance of tubulointerstitial transient proteinuria. All of the above 593 Which of the following can present with gross hematuria ? Microscopic hematuria may appear with the onset of benign prostatic hypertrophy. Alport’s syndrome Harrison’s 18th Ed. 2337 D. emotional stress B. All of the above D. 2 .366 MCQ’s FOR MEDICAL PROFESSIONALS BY PROF. 1 A.3 weeks Harrison’s 18th Ed. 602 Which of the following stains is used to enhance basement lupus nephritis. Periodic acid-Schiff (PAS) A. Congo red for amyloid deposits. Hypertension Harrison’s 18th Ed. membrane structure in renal biopsy ? Harrison’s 18th Ed. else diffuse. All of the above irreversibility and progression to renal failure ? Harrison’s 18th Ed. D. < 60 % of clinical glomerulonephritis ? C. 2340 599 Wegener’s granulomatosis has which of the following patterns A. Pulmonary-Renal Syndrome D. ideally 20 glomeruli should be reviewed individually for discrete lesions. All of the above Circulating immune-complex formation and deposition within the glomerulus is seen with SABE. PAS for carbohydrate D. 610 Poststreptococcal glomerulonephritis due to impetigo develops how many weeks after the skin infection ? 604 ‘Focal glomeulonephritis’ is said to occur when lesion involves Harrison’s 18th Ed.3 weeks B. 1 . ideally how many glomeruli should be 609 Poststreptococcal glomerulonephritis develops how many reviewed individually for discrete lesions ? weeks after streptococcal pharyngitis ? Harrison’s 18th Ed. AJAY MATHUR Nephrology 598 Henoch-Schönlein purpura has which of the following patterns B.6 weeks D. 2340 600 Cryoglobulinemia has which of the following patterns of clinical glomerulonephritis ? A. Lobar glomerulonephritis Renal biopsy specimen are stained with H&E for cellularity & architecture. 10 C. 2 . All of the above 606 Acute nephritic syndromes classically present with ? Harrison’s 18th Ed. All of the above C. All of the above pyuria. red blood cell casts. lesion is considered focal. Glomerular Vascular Syndrome C. 1 . Hepatitis-C-related cryoglobulinaemia D. 2339 Table 283-2 B. Mesangiocapillary glomerulonephritis D. Glomerular Vascular Syndrome 605 In renal biopsy.8 weeks By light microscopy. Glomerular Vascular Syndrome Acute nephritic syndromes classically present with hypertension. which of the following is an ominous sign of D. Acute Nephritic Syndrome C. < 80 % A. Pyuria B. & hepatitis-C-related cryoglobulinaemia. 601 Churg-Strauss syndrome has which of the following patterns 607 Circulating immune-complex formation and deposition within of clinical glomerulonephritis ? the glomerulus is seen in ? Harrison’s 18th Ed. 603 By light microscopy. 2339 Harrison’s 18th Ed. B. 5 B. Lupus nephritis C. Glomerular Vascular Syndrome Interstitial fibrosis is an ominous sign of irreversibility & progression to renal failure. Hematoxylin and eosin Harrison’s 18th Ed. Masson’s trichrome C. Segmental necrotizing glomerulonephritis moieties in glomerular tuft & tubule membranes. 2339 Table 283-2 D. Pulmonary-Renal Syndrome D. Acute Nephritic Syndrome If <50% glomeruli are involved. 4 . Synechiae of clinical glomerulonephritis ? Harrison’s 18th Ed. Hematuria A. All of the above C. Interstitial fibrosis A. Pulmonary-Renal Syndrome B. Crescents B. 2339 B. Subacute bacterial endocarditis (SABE) B. 20 D. 2340 A. < 50 % . & Masson’s trichrome to identify collagen Poststreptococcal glomerulonephritis is prototypical for acute endocapillary proliferative deposition to assess degree of glomerulosclerosis & interstitial fibrosis. and mild to moderate proteinuria. hematuria. 2339 Table 283-2 Harrison’s 18th Ed. 3 . 2340 what proportion of glomeruli ? A. 2340-41 A.6 weeks A. Acute Nephritic Syndrome C. < 70 % Harrison’s 18th Ed. D. Pulmonary-Renal Syndrome C. Jones-methenamine silver to enhance basement membrane structure.6 weeks C. 2339 Table 283-2 B. glomerulonephritis. Acute Nephritic Syndrome A. Jones-methenamine silver B. 2339 608 Poststreptococcal glomerulonephritis is prototypical for ? A. 2340 B. Acute endocapillary proliferative glomerulonephritis C. 8 weeks A. Absent ANCA 613 Poststreptococcal glomerulonephritis develops how many D. Lung hemorrhage C. Normal levels of C4 syndrome ? Harrison’s 18th Ed. 4 weeks D.5. Alport’s Syndrome Harrison’s 18th Ed.4. Ernest 611 In the first week of poststreptococcal glomerulonephritis. D. 1680 619 Which of the following is true for ‘anti-GBM disease’ ? A. which of the following findings is false ? D.6(IV) C. 10 days Harrison’s 18th Ed.  Lupus nephritis is two + times more in female than among male population. 1 D. Lupus nephritis B. Eric Harrison’s 18th Ed. 20 days A. A. 2341 antigen’ ? A. Glomerulonephritis in SLE D.1. Goodpasture’s syndrome A. 367 Nephrology MCQ’s FOR MCQ’s FOR MEDICAL MEDICAL PROFESSIONALS PROFESSIONALS BY PROF. Ellis C.4. 3 D. Anti-GBM nephritis B.5(IV) B.3 weeks after streptococcal pharyngitis. Chinese young male B.6(IV). 3. None of the above A. 2351 B. Detectable circulating anti-GBM autoantibodies C. 1. Albert Poststreptococcal glomerulonephritis due to impetigo develops 2 . 30 days B.6 weeks after skin infection and 1 . 1680 Harrison’s 18th Ed.348:2543-56 D.13:177-97”.6 weeks 617 First name of Goodpasture was ? N Engl J Med 2003. 1681 A.  D.2(IV). AJAY MATHUR Cardiology 367 C. Australian aboriginals A. Decreased levels of C3 618 Which of the following is not a basement membrane C. 3 weeks C. Alport’s Syndrome 90% patients of poststreptococcal glomerulonephritis will have depressed CH50 & decreased levels of C3 with normal levels of C4 in first week of symptoms. Depressed total hemolytic complement (CH50) B. 2 weeks B. 2342 B. Nail-patella syndrome Harrison’s 16th Ed. Acute poststreptococcal glomerulonephritis B. 2 Lupus nephritis is most severe in African-American female adolescents as a complication of SLE.  .348:2543-56 Harrison’s 16th Ed.5.5(IV).2(IV) A.1. C. African-American female adolescents Goodpasture’s syndrome ? B. Autoimmune disease D. 5. All of the above 614 “Starry sky” appearance on immunofluorescence microscopy 621 Which of the following is a protomer of collagen IV ? of renal biopsy is seen in ? N Engl J Med 2003. 4 616 Which of the following glomerulonephritis is more common in female than in male population ? 623 Which chain of type IV collagen is called ‘Goodpasture Harrison’s 18th Ed. 2342 A. 2340 Ernest Goodpasture published his observation as “The pathology of pneumonia accompanying influenza. U S Naval Med Bull 1919. Churg-Strauss syndrome 612 Poststreptococcal glomerulonephritis develops how many days after pharyngitis ? D. 40 days C. Nail-patella syndrome C. 2350 D. Lupus nephritis C. Goodpasture’s syndrome C. White elderly Harrison’s 18th Ed. 2342 C. All of the above D. 4 . 3 C. 1 week A. B. All of the above weeks after skin infection ? 620 Which of the following is true for Goodpasture’s syndrome ? Harrison’s 16th Ed. Harrison’s 18th Ed. 3. 2340 622 Which chain of type IV collagen is the target antigen in A. B. and 5. 3 . Goodpasture’s syndrome The six chains of collagen IV form only three sets of triple helical molecules called protomers 615 Lupus nephritis is most severe in ? designated as 1. Normal serum complement levels D. 368 MCQ’s FOR MEDICAL PROFESSIONALS BY PROF.5(IV) C. 2351 Harrison’s 18th Ed. progressive renal failure basement membrane antibodies. Tongue B. SLE C. Harrison’s 18th Ed. Mutations in COL4A5 (X-linked). 2342 and cyclophosphamide therapy. COL4A3. Nose D. 627 Antibodies against the 1(IV) NC1 domain is seen in patients 633 Out of the following. 2342 A. All of the above IgA nephropathy (Berger disease) is a primary glomerular disease.70s C. Ocular lens A. 2351 A. Barratt disease A. Spleen C. 3(IV) NC1 A. Wegener’s granulomatosis . Hematuria. 1. None of the above In Alport’s syndrome. >50% crescents on renal biopsy C. Occurs in young men in late 20s B. Thymus D. A few such T cells escape deletion and can engage in the production of anti–glomerular Alport’s syndrome consists of hematuria. Recurrent episodes of macroscopic hematuria B.   3 (IV) NC1 peptides are produced by ? N Engl J Med 2003. Usually X linked 3. 3. All of the above 636 IgA nephropathy resembles which of the following diseases ? Patients of Goodpasture’s syndrome respond to plasmapheresis. IgA nephropathy B. there is a male preponderance. AJAY MATHUR Nephrology 624 Which of the following is known as “Goodpasture protomer” ? 630 Which of the following is false about Alport’s syndrome ? N Engl J Med 2003. Berger first described it in 1968. Oliguria B. Rapidly progressive/crescentic glomerulonephritis D. Occurs in men & women in 60 . 2342 Harrison’s 18th Ed.2(IV) D. Nails C. and sensorineural deafness. 2342 629 Patients of Goodpasture’s syndrome respond to which of the following treatments ? A. 5. 2342 Harrison’s 18th Ed. Serum creatinine >5. IgA nephropathy is the most common form of glomerulonephritis worldwide. 628 In Goodpasture’s syndrome.5. serum creatinine >5. proteinuria (<1 to 2 gram/day). D. Tomino disease C. or COL4A4 (both autosomal recessive) genes result in defective synthesis of collagen type IV with consequent dysfunction and inflammation of the glomerular basement membrane. 2351 A. Sensorineural deafness C. A. Jaundice Thymus expresses 3 (IV) NC1 peptides that can cause elimination of autoreactive CD4+ helper T cells. lenticonus of anterior lens capsule is termed as positive “oil droplet sign”. Female preponderance Harrison’s 18th Ed. Between episodes.348:2543-56. Glassock disease D. 626 Which of the following is false about Goodpasture’s 632 Positive “oil droplet sign” in Alport’s syndrome pertains to ? syndrome ? N Engl J Med 2003. 2342 A. which of the following indicate 634 IgA nephropathy is also called ? worse prognosis at presentation ? Harrison’s 18th Ed.5(IV) protomer is a basement-membrane molecule of family of type IV collagens in epithelial Chronic glomerulosclerosis is a feature of Alport’s syndrome. Plasmapheresis C.7 mg/dL D.4. Paraneoplastic syndrome B. prognosis at presentation is worse if there is oliguria.7 mg/dL or a need for dialysis. 635 Which of the following is false about IgA nephropathy ? Harrison’s 18th Ed. Ataxia B. 2342 B. tissues and is called the “Goodpasture protomer”.4. urinalysis is normal C. Peak incidence in II & III decades of life A. All of the above D. accompanied by oral prednisone Harrison’s 18th Ed. Inherited tubulointerstitial disease B. In Goodpasture’s syndrome.1. Due to mutation in COL4A5 gene C. Cyclophosphamide In IgA nephropathy. Oral prednisone D. advanced fibrosis or >50% crescents on renal biopsy. Membranous glomerulonephritis Antibodies against the 1(IV) NC1 domain is seen in patients with paraneoplastic syndrome. Hemoptysis is largely confined to smokers D. Bone marrow B. glomerulosclerosis & renal failure D. Alopecia D.6(IV) B.348:2543-56 631 Which of the following is related to Alport’s syndrome ? Harrison’s 18th Ed. Leukemia A. Berger disease B. Lymph node A. 2342 A.348:2543-56 Harrison’s 18th Ed. which is the most common form of with ? glomerulonephritis ? Harrison’s 18th Ed. Focal sclerosing glomerulonephritis C. cutaneous purpura. P. Bone marrow D. AJAY MATHUR Cardiology 369 B. Type I membranoproliferative glomerulonephritis D. Chronic liver disease A. Harrison’s 18th Ed. Glomerular capillaries B. idiopathic interstitial pneumonia. Wegener’s syndrome A. However. Persistent hepatitis C infections dermatitis herpetiformis. asthma. mycosis fungoides. 2343 648 Mesangioproliferative glomerulonephritis is seen in ? A. pauci-immune D. Crohn’s disease. 2344 C. Wegener’s disease Harrison’s 18th Ed. 2342 645 Mesangiocapillary glomerulonephritis or lobar A. All of the above associated with ? Harrison’s 18th Ed. Ankylosing spondylitis Membranoproliferative glomerulonephritis (MPGN) is also called mesangiocapillary glomerulonephritis or lobar glomerulonephritis. All of the above glomerulonephritis ? Type I MPGN is commonly associated with persistent hepatitis C infections. 1788 B. Crohn’s disease Harrison’s 18th Ed. IgA nephropathy is characterized by diffuse deposition of IgA in glomerular mesangium. All of the above D. falciparum malaria .. All of the above B. gastrointestinal adenocarcinoma. or neoplastic diseases. Henoch Schonlein purpura Presentation without renal involvement is termed limited Wegener’s granulomatosis. 2344 C. Membranous glomerulonephritis C. Harrison’s 18th Ed. Microscopic polyangiitis Harrison’s 18th Ed. allergic rhinitis. 638 In IgA nephropathy. 2344 Deposits of IgA in glomerular mesangium are found in chronic liver disease. Harrison’s 18th Ed. Churg-Strauss Syndrome is a small-vessel vasculitis associated with peripheral eosinophilia. 2342 D. also called dense deposit disease. Lymph node C. All of the above 643 In microscopic polyangiitis. B. Wegener’s granulomatosis 647 Which of the following is called “dense deposit disease” ? B. C. IgA nephropathy C. Pulmonary infiltrates C. Liver they rarely have significant lung disease or destructive sinusitis. Goodpasture’s syndrome D. A. Sjögren’s syndrome 646 Type I Membranoproliferative glomerulonephritis (MPGN) is D. D. Focal segmental glomerulosclerosis D. autoimmune diseases Harrison’s 18th Ed. All of the above B. ankylosing spondylitis. Type III Membranoproliferative glomerulonephritis more common in microscopic polyangiitis or Churg-Strauss syndrome. Hematuria Harrison’s 17th Ed. 2344 B. Renal involvement C. Isolated Wegener’s granulomatosis B. Spleen Patients of microscopic polyangiitis are similar to those with Wegener’s granulomatosis. 2342 from other small-vessel vasculitis ? Harrison’s 18th Ed. mononeuritis. Proteinuria A. 369 Nephrology MCQ’s FOR MCQ’s FOR MEDICAL MEDICAL PROFESSIONALS PROFESSIONALS BY PROF. deposition of IgA occurs in ? 644 Which of the following distinguishes Churg-Strauss Syndrome Harrison’s 18th Ed. Lupus inflammation or renal dysfunction. Glomerular mesangium A. hypergammaglobulinemia. Idiopathic interstitial pneumonia C. Limited Wegener’s granulomatosis C. leprosy. D. Leprosy glomerulonephritis is also called ? B. All of the above Morphologically. except that D. which of the following is Clinical and laboratory evidence suggests close similarities between Henoch-Schönlein purpura and uncommon ? IgA nephropathy. None of the above 642 Wegener’s granulomatosis presenting without renal Low serum C3 & dense thickening of GBM containing ribbons of dense deposits and C3 characterize involvement is termed ? Type II MPGN. Churg-Strauss syndrome A. 2344 637 Most IgA deposited in the kidney are derived from ? A. chronic obstructive bronchiectasis. Membranoproliferative glomerulonephritis 640 Deposits of IgA in glomerular mesangium are found in ? Harrison’s 18th Ed. Glomerular basement membrane B. A. 2344 A. Neoplastic diseases 641 Which of the following is an ANCA-positive. IgA deposition does not produce clinically significant glomerular B. Type II Membranoproliferative glomerulonephritis Anti-PR3 antibodies are more common in Wegener’s granulomatosis & anti-MPO antibodies are C. Significant lung disease or destructive sinusitis B. relapsing polychondritis. and Sjögren’s syndrome. Mesangioproliferative glomerulonephritis A. Glomerulonephritis D. Peripheral eosinophilia C. 2343 like lupus or cryoglobulinemia. and rheumatoid factor. elevated serum 639 Deposits of IgA in glomerular mesangium are found in ? IgE. Corticomedullary junction A.250 mg/mmol D. 1790 B. Am J Kidney Dis 1996. and Class II lupus nephritis. hepatitis B.350 mg/mmol indicates nephrotic range proteinuria. 50 . Acellular urinary sediment Membranous glomerulonephritis (MGN) is the most common cause of nephrotic syndrome in the elderly. proteinuria is selective and largely composed of albumin. Harrison’s 18th Ed. 2345 B. 2345 proteinuria ? A. Focal segmental glomerulosclerosis (FSGS) A. MCD patients with steroid resistance can develop FSGS on repeat biopsy. allergies.90% of nephrotic syndrome in childhood but only 10 . Minimal Change Disease (MCD) 652 Which of the following is false about minimal change disease ? B. Nil disease B. Mesangial interposition C. Mesangioproliferative glomerulonephritis D. Lipoid nephrosis C. schistosomiasis. 650 Minimal Change Disease (MCD) is associated with ? 656 What value of protein:creatinine ratio indicates nephrotic range Harrison’s 18th Ed. All of the above Pathologic changes of FSGS are most prominent in glomeruli located at corticomedullary junction. (FSGS) are most prominent in glomeruli located at ? Harrison’s 18th Ed. lung. Focal segmental glomerulosclerosis (FSGS) Harrison’s 18th Ed. Allergies A.75 % lupus. Hodgkin’s disease Harrison’s 17th Ed. Use of nonsteroidal anti-inflammatory agents B. resolving 655 Pathologic changes of focal segmental glomerulosclerosis postinfectious glomerulonephritis.150 mg/mmol C. Any of the above is the most common cause in black patients. Membranous glomerulonephritis (MGN) B. Commonest cause of nephrotic syndrome in adults D. Membranoproliferative glomerulonephritis (MPGN) D.350 mg/mmol MCD usually presents as a primary renal disease but can be associated with Hodgkin’s disease. It is rare in childhood. Mesangial proliferation B. > 200 . All of the above MCD on electron microscopy of renal biopsy consistently shows an effacement of the foot process 658 Most common cause of nephrotic syndrome in the elderly is ? supporting epithelial podocytes with weakening of slit-pore membranes.27:647-51 653 MCD patients with steroid resistance can develop ? A. Minimal change disease (MCD) A. All of the above Mesangioproliferative glomerulonephritis is seen in IgA nephropathy. D. Microscopic polyangiitis Membranous nephropathy is the most common cause of nephrotic syndrome in white. and early relapse predicts multiple subsequent relapses. Focal segmental glomerulosclerosis (FSGS) C. malaria. Minimal Change Disease (MCD) Harrison’s 18th Ed. All of the above C. 2345 Harrison’s 18th Ed. AJAY MATHUR Nephrology C. Mesangioproliferative glomerulonephritis C. Solid tumors of breast. Outer cortex B. 2345 649 Minimal change disease (MCD) is also called ? Harrison’s 18th Ed. 70 . A protein:creatinine ratio value > 300 . is ? Korbet SM et al. or use of NSAIDagents. Known as nil lesion C. . Membranous glomerulopathy C. 30 . In MCD. P.40 % D. Focal and segmental glomerulosclerosis (FSGS) B. 10 . colon. relapses occur in what proportion of children after the Harrison’s 18th Ed.60 % MGN can be secondary to solid tumors of breast. Effacement of the foot process D. Hepatitis B A.300 mg/mmol D. while FSGS D. Malaria B. 2347 first remission ? A.20 % C. lung. 651 Minimal Change Disease (MCD) on electron microscopy of 657 Majority of children with nephrotic syndrome are due to ? renal biopsy consistently shows ? Harrison’s 18th Ed. 2345 B. > 100 . All of the above D. 660 Membranous glomerulonephritis (MGN) can be secondary to ? 654 In MCD. colon Harrison’s 18th Ed. All of the above C. 2345 A. Middle cortex C. & D. Foot process disease D.370 MCQ’s FOR MEDICAL PROFESSIONALS BY PROF. Selective proteinuria MCD causes 70 . > 250 . 2347 A. 2345 A.75% of children after the first remission. > 300 . D. Membranous glomerulonephritis (MGN) B. 2345 A.15% of nephrotic syndrome 659 Most common cause of nephrotic syndrome in black patients in adults. falciparum malaria. Mesangioproliferative glomeru2 €phritis C. Resolving postinfectious glomerulonephritis Relapses occur in 70 . Minimal change disease (MCD) A. 2348 B. Thickening of GBM D. 20 . Membranous glomerulopathy A. 669 In diabetes. 3 . highest incidences of renal vein thrombosis. 1792 following histopathological findings in diabetic kidney ? Harrison’s 18th Ed. 15 . Natural history of diabetic nephropathy C.5 gram/dL. morphologic changes appear in the kidney. thrombosis. 1792 years after the onset of diabetes ? A. 2348 A. it usually takes 10 . 2348 B. 10 . >90% of Type 1 diabetics with B. Right-sided varicocele B.4 years D. Sudden onset of flank or abdominal pain Harrison’s 18th Ed. D.300 mg/day is called microalbuminuria. Acute decline in GFR C. Natural history of diabetic nephropathy in patients with Types 1 and 2 diabetes is similar.9 years Within 1 . > 30 years B. 20 . PAS + nodules. 2348 A. From the stage of C. Presence of diabetic retinopathy 673 Congo red stains are positive ? Harrison’s 18th Ed. 7 .2. D. Harrison’s 18th Ed. Nodular glomerulosclerosis D. 30 . 1 .20 670 From the earliest stages of microalbuminuria. Diabetic nephropathy D. 5 .400 mg/day Thrombotic complications occur in nephrotic syndrome. > 1000 mg/day A. and deep vein thrombosis ? 667 In microalbuminuria. Onset of microalbuminuria D.30 years A.10 years after the onset of Types 1 or 2 diabetes. Expansion of mesangial matrix After proteinuria level of >500 mg/day. Thickening of GBM B. 2348 morphologic changes appear in kidneys ? A. Nodular glomerulosclerosis microalbuminuria. 2349 Renal biopsies from patients with Types 1 and 2 diabetes are largely indistinguishable. Expansion of mesangial matrix C. > 30 mg Harrison’s 18th Ed. > 500 mg C.15 D. All of the above 671 Renal vein thrombosis is common in patients with nephrotic 665 Kimmelstiel-Wilson lesion relates best with which of the syndrome due to ? Harrison’s 17th Ed.10 years diabetic kidneys ? B.200 mg/day C. 2348 664 Which of the following histopathological findings occur in A.2 years after the onset of clinical diabetes. MGN has the highest incidence of renal vein Albuminuria in the range of 30 . Harrison’s 18th Ed. > 500 mg/day Harrison’s 18th Ed. < 150 mg/day B. microalbuminuria appears how many Harrison’s 17th Ed.10 D.10 years after its onset. All of the above D. 5 . microalbuminuria appears 5 . 40 . Fibrillary-Immunotactoid Glomerulopathy . AA & AL amyloidosis appears 5 .300 mg/day D. 2348 C. Amyloidosis C. 668 Dipstick positive level of albuminuria is ? 662 How many years after the onset of clinical diabetes. Microalbuminuria A. renal function decline relentlessly. 2348 B. Membranoproliferative glomerulonephritis B.2 years C. 1684 666 Which of the following is different between Types 1 & 2 diabetes ? A.7 years Dipstick positive level of proteinuria is > 300 mg albuminuria. pulmonary embolism. pulmonary embolism & deep vein thrombosis. Focal segmental glomerulosclerosis (FSGS) B. the range of albuminuria is ? Harrison’s 18th Ed. Risk of venous thrombosis is higher when serum albumin is <2 . 5 . > 150 mg A. it takes how many years to reach end-stage renal disease (ESRD)? In Types 1 or 2 diabetes. Renal biopsy findings D. > 1500 mg/day B. include all except ? Harrison’s 16th Ed. 2347 Harrison’s 18th Ed. Membranous glomerulonephritis (MGN) C. Gross hematuria A. 371 Nephrology MCQ’s FOR MCQ’s FOR MEDICAL MEDICAL PROFESSIONALS PROFESSIONALS BY PROF. 10 .20 years to reach ESRD. > 300 mg B. renal functions unrelentingly decline after what level of proteinuria ? 663 In Types 1 or 2 diabetes.20 years Harrison’s 18th Ed. Hyaline arteriosclerosis 672 Clinical features that suggest acute renal vein thrombosis Kimmelstiel-Wilson nodules or nodular glomerulosclerosis are eosinophilic. > 2500 mg/day C. AJAY MATHUR Cardiology 371 661 Which of the following causes of nephrotic syndrome has the nephropathy have diabetic retinopathy while only 60% of Type 2 diabetics with nephropathy have diabetic retinopathy. 2-5 C. AJAY MATHUR Nephrology C. neutral glycosphingolipids accumulate in which orgenelle of glomerular cells ? 686 HIV patients with FSGS present with nephrotic-range Harrison’s 18th Ed. 2353 D. Membranous glomerulopathy B. corneal and lenticular opacities. heart. Over forehead A. Between umbilicus and knees 684 Term HIV-associated nephropathy (HIVAN) is used for ? Harrison’s 18th Ed.5 years after HIV infection. Edema D. 2349 C. Over nape of the neck D. 2353 C. All of the above A. 2351 Fabry disease is an X-linked inborn error of globotriaosylceramide metabolism that results from mutations in the  -galactosidase gene leading to excessive intracellular storage of A. Hypertension C. Angiokeratomas D. None of the above C. hypohidrosis. 3191 proteinuria & hypoalbuminemia but without ? A. angiokeratomas are most dense ? B. A. -Mannosidosis 683 HIV infection is most commonly associated with which of the C. All of the above B. Mitochondria Harrison’s 18th Ed. More common in blacks In Fabry’s disease. Acute diffuse proliferative glomerulonephritis Harrison’s 18th Ed. B. -Mannosidosis (HIVAN) ? Harrison’s 18th Ed. Abnormal gene is located on long arm of chromosome 9 Harrison’s 18th Ed. fibrillar collagen deposits 675 Clinical manifestations of Fabry’s disease include all except ? B. ~ 6 months C. Harrison’s 18th Ed. IgA nephropathy A.354:1387-401. ~ 1 year 676 Angiokeratomas is a clinical feature of ? D ~ 2. LMX1B is expressed in kidney primarily by podocytes A. All of the above . 2353 D. Corneal and lenticular opacities C. A. C. All of thr above syndrome’ is false ? N Engl J Med 2006. angiokeratomas are most dense between umbilicus and knees ”the bathing suit area” but may occur anywhere. Fabry’s disease 685 Which of the following is false for HIV-associated nephropathy B. Membranous glomerulopathy A. Over finger tips C.372 MCQ’s FOR MEDICAL PROFESSIONALS BY PROF. electron microscopy of renal biopsy shows enlarged glomerular visceral epithelial cells packed with vacuoles containing globotriaosylceramide in parallel arrays called zebra bodies. It an autosomal dominant disease 674 Which of the following best relates to Fabry’s Disease ? B. 3193 Table 361-1 HIV-associated nephropathy begins ~ 2. B. Ceramide syndrome’ is false ? N Engl J Med 2006. Aggresive focal segmental glomerulosclerosis In Fabry’s disease. Fucosidosis following glomerulopathies ? Harrison’s 18th Ed. Golgi apparatus A. Small-vessel disease of the kidney 682 After what duration does HIV-associated nephropathy begin ? Harrison’s 17th Ed. Aggresive focal segmental glomerulosclerosis 677 In Fabry’s disease. Globotriaosylceramide 681 Which of the following statements about ‘Nail–patella D. Nucleus C. Glucosylceramide knees. Fabry’s disease manifests as angiokeratomas (telangiectatic skin lesions). May be the first manifestation of HIV infection D. 2353 B. ~ 3 months Clinically. Fabry’s disease B. Glomerular basement membrane is thickened with splitting and globotriaosylceramide. 3191 C. Light chain deposition disease 680 Which of the following statements about ‘Nail–patella D. Caused by loss-of-function mutations in LMX1B Harrison’s 18th Ed. Multiple osseous abnormalities primarily affect elbows and A. and small-vessel disease of kidney. 1796 D.5 years Harrison’s 18th Ed. Slow clinical course 679 In Fabry’s disease. Hyperlipidemia D. Lysosome B. 3191 C. including mucosal surfaces. More frequent in intravenous drug abusers D. Macrocephaly A. Fucosidosis A. IgA nephropathy 678 “Zebra bodies” is a histopathological hallmark of ? Harrison’s 18th Ed. acroparesthesia.354:1387-401. 2351 AA and AL amyloid fibrils are detectable with Congo red. & brain. Galactosylceramide D. None of the above B. and nail dysplasia B. 2350 D. Harrison’s 18th Ed. Acute diffuse proliferative glomerulonephritis C. Nitric oxide C. 1674 695 Which of the following is least common among patients with A. 1678 D. Crescentic glomerulonephritis A. Diffuse proliferative glomerulonephritis Harrison’s 16th Ed. Visceral epithelial cells Harrison’s 16th Ed. AJAY MATHUR Cardiology 373 HIV patients with FSGS typically present with nephrotic-range proteinuria and hypoalbuminemia. Glomerular proteinuria results from leakage of plasma proteins 693 Intrarenal vasoconstrictors include all except ? through a perturbed glomerular filtration barrier Harrison’s 16th Ed. that are present in excess in D. Prostacyclin C. Membranous glomerulopathy 697 Which of the following is true for Henoch Schonlein purpura ? B. Focal proliferative glomerulonephritis A. All of the above B. All of the above A. or hyperlipidemia. Negative anti-GBM serology C. Idiopathic renal-limited crescentic glomerulonephritis 692 Intrarenal vasoconstrictors include all except ? B. All of the above B. 1684 D. 1678 B. None of the above Harrison’s 16th Ed. Platelet-activating factor A. Circulating ANCA present Harrison’s 16th Ed. Endothelial cells glomerulonephritis’ ? C. Minimal change disease (MCD) Harrison’s 16th Ed. Thromboxanes 700 Which of the following statements is false ? Harrison’s 16th Ed. Titers of ANCA 689 Nephrotic-range proteinuria can be found in all except ? Harrison’s 16th Ed. Endothelins of low-molecular-weight plasma proteins B. Proliferating parietal epithelial cells 699 Which of the following is a type of pauci-immune C. 1678 A. Pauci-immune glomerulonephritis 688 Which of the following is false about inflammatory glomerulopathy ? D. 1680 C. Serum C3 level D. 1674 D. RBC cast in urine A. 1674 696 Serologic markers that predict the immunofluorescence A. Fibrin glomerulonephritis ? Harrison’s 16th Ed. Pauci-immune glomerulonephritis 687 Which of the following is an inflammatory glomerulopathy ? D. Absent circulating anti-GBM autoantibodies B. Wegener’s granulomatosis A. Nephrotic-type microscopy findings in nephritic syndrome and RPGN include ? B. Proteinuria > 150 mg per 24 h is abnormal . All of the above A. Normal complement levels D. Thromboxanes immunoglobulin light chains. RBC in urine Harrison’s 16th Ed. 373 Nephrology MCQ’s FOR MCQ’s FOR MEDICAL MEDICAL PROFESSIONALS PROFESSIONALS BY PROF. edema. None of the above 694 Which of the following is most common in ‘acute nephritic syndrome’ ? 701 Which of the following statements is false ? Harrison’s 16th Ed. Tubular proteinuria results from failure of tubular reabsorption A. Platelet-activating factor circulation D. Overflow proteinuria results from filtration of proteins. Normal serum complement levels 691 ‘Crescents’ are composed of ? C. Focal and segmental glomerulosclerosis (FSGS) B. Titers of anti-GBM antibody C. Focal proliferative glomerulonephritis RPGN ? B. but B. Immune-complex glomerulonephritis A. Negative ANCA serology 690 Which of the following cells of kidney do not proliferate ? Harrison’s 16th Ed. None of the above Harrison’s 16th Ed. Anti-GBM disease without hypertension. Anti-GBM disease C. Leucocyte in urine B. usually C. Mesangial cells 698 Which of the following is true for ‘pauci-immune B. C. Infiltrating monocytes D. All of the above A. 1680 Harrison’s 16th Ed. 1680 D. Leukotrienes D. 1684 A. All of the above B. Microscopic polyangiitis nodosa (PAN) Harrison’s 16th Ed. All of the above A. 1678 C. Immune-complex glomerulonephritis D. 1680 C. 1683 D. 1680 C. 1691 D. 2 to 4 months 710 HCV-induced immune-complex disease is associated with which of the following ? C. Membranous glomerulopathy Disease and Other Inherited Tubular Disorders 706 Morphologic lesion in warfarin induced glomerular disease is ? Harrison’s 16th Ed. ARPKD is mainly a disease of adults 707 Morphologic lesion in thiazide induced glomerular disease is ? C. Rapidly progressive glomerulonephritis B. Membranous glomerulopathy A. 1691 D. They infrequently cause kidney failure Harrison’s 16th Ed. 1692 and light chains A. 1691 A. Rapidly progressive glomerulonephritis Autosomal dominant polycystic kidney disease (ADPKD) is seen predominantly in adults. None of the above B. Pauci-immune necrotizing glomerulonephritis Harrison’s 18th Ed. All of the above A. MPGN patients with proteinuria following gold therapy ? C. Membranous glomerulopathy A. Membranous glomerulopathy B. 6 to 8 months A. Rapidly progressive glomerulonephritis 713 Which of the following statements is false about polycystic kidney disease ? B. Cysts are distributed throughout cortex & medulla C. Pauci-immune necrotizing glomerulonephritis autosomal recessive polycystic kidney disease (ARPKD) is mainly a disease of childhood. Focal segmental glomerulosclerosis A. 1 to 2 months B. Rapidly progressive glomerulonephritis B. whereas B. Long duration (> 10 years) 705 Morphologic lesion in rifampin induced glomerular disease B. All of the above .3 A. MPGN C. Proliferative glomerulonephritis with vasculitis 714 Which of the following statements about ‘Autosomal dominant polycystic kidney disease (ADPKD)’ is false ? 708 Glomerular lesion associated with HBV infection include ? Harrison’s 17th Ed. They frequently causes kidney failure. 1694 D. Focal segmental glomerulosclerosis Harrison’s 18th Ed. C. IgA nephropathy D. Cryoglobulinemic proliferative glomerulonephritis 703 Renal biopsy typically reveals which of the following in B. Membranous glomerulopathy Harrison’s 16th Ed. Pauci-immune necrotizing glomerulonephritis C. ADPKD-1 gene is on chromosome 16p13. Destructive arthropathy Harrison’s 16th Ed. ADPKD-2 gene is on chromosome 4q21-23 B. Focal segmental glomerulosclerosis Harrison’s 16th Ed. Tubular proteinuria never exceeds 2 gram per day 709 Which out of the following is the most common glomerular C. 2355 C. AJAY MATHUR Nephrology B. MCD disease is ? C.374 MCQ’s FOR MEDICAL PROFESSIONALS BY PROF. IgA nephropathy Harrison’s 16th Ed. MPGN 702 Proteinuria develops after what duration of gold therapy ? C. ADPKD is seen predominantly in childhood D. 1693 D. Polycystic Kidney D. Sulfosalicylic acid precipitation test detects both albumin lesion associated with HBV infection ? Harrison’s 16th Ed. Pauci-immune necrotizing glomerulonephritis 711 Which of the following glomerulopathies is most commonly associated with Hodgkin’s lymphoma ? C. Focal segmental glomerulosclerosis D. Circulating rheumatoid factor is ? C. Essential mixed cryoglobulinemia A. 2345 D. Focal segmental glomerulosclerosis Chapter 284. Membranous glomerulopathy D. 1692 A. 4 to 6 months Harrison’s 16th Ed. 1691 D. All of the above A. MPGN Harrison’s 16th Ed. Erythropoietin production is low D. FSGS 704 Morphologic lesion in ciprofloxacin induced glomerular B. 1691 D. Membranous glomerulopathy B. Membranous glomerulopathy A. Rapidly progressive glomerulonephritis B. 1797 Harrison’s 16th Ed. All of the above A. 1691 D. Pauci-immune necrotizing glomerulonephritis 712 Amyloidosis is more frequent in patients of rheumatoid arthritis with ? C. Thick ascending limb of Henle’s loop In ADPKD. In early childhood Harrison’s 18th Ed. Standard diagnostic criteria is at least 3 to 5 cysts in each kidney B. PKD-1 encodes polycystin-1. Unilateral abdominal mass is common A. All of the above Harrison’s 18th Ed. Fibrocystin is called ? B. A. >=3 in one & 2 cysts in other kidney . In utero 724 Protein expressed by PKHD1 is termed ? B. arachnoid membranes & seminal vesicles in men. All of the above ESRD in childhood ? Cysts can also develop in liver. Inversin In ADPKD. B. Caveolin 3 A. 716 Which of the following is a PKD-1 encoded protein ? Harrison’s 18th Ed. <5% of total nephrons are involved. C. >=3 C. Nephrocystin D. inguinal hernias occur with a higher frequency than in the general population in ADPKD. Portal hypertension & esophageal varices are frequent 720 In ADPKD. Polycystin-3 718 What percentage of total nephrons are involved in ADPKD ? Harrison’s 18th Ed. 375 Nephrology MCQ’s FOR MCQ’s FOR MEDICAL MEDICAL PROFESSIONALS PROFESSIONALS BY PROF. Bartter’s syndrome B. cardiac valvular abnormalities (MVPS. Arachnoid membranes 727 Which of the following is the most common genetic cause of D. D. 2358 C. Cortical & medullary collecting ducts D. >=4 D. FMR-1 protein B. AJAY MATHUR Cardiology 375 715 Which of the following statements about ‘Autosomal dominant 722 Diagnostic criteria for ADPKD in > 60 year old include ? polycystic kidney disease (ADPKD)’ is false ? Harrison’s 18th Ed. All of the above A. Colonic diverticulae polycystic kidney disease (ARPKD)’ is false ? Harrison’s 18th Ed. whereas PKD-2 gene product is polycystin-2. <2% Harrison’s 18th Ed. 2359 A. Autosomal recessive polycystic kidney disease 721 Diagnostic criteria for ADPKD in young include ? Harrison’s 18th Ed. spleen. ARPKD-2 C. 2358 C. AR). hepatic cysts. cyst formation begins in utero. cysts can also develop in ? C. cyst formation begins ? gene. 2358 A. >=2 B. 2358 Harrison’s 18th Ed. Mitral valve prolapse found in 25% of patients D. ARPKD-1 D. Pancreas Enlarged kidneys are detected soon after birth as bilateral abdominal masses. Medullary sponge kidney (MSK) A. pancreas. 2359 D. >=2 in one & 1 cysts in other kidney C. C. Fibrocystin D. PKHD1 ADPKD results from mutations in either PKD-1 (85% of cases) or PKD-2 gene. <5% B. Distal tubules & collecting ducts are dilated Intracranial aneurysm. Cardiac valvular abnormalities 726 Which of the following statements about ‘Autosomal recessive C. >=1 A. >=2 in one & 2 cysts in other kidney Nephronophthisis (NPHP) is the most common genetic cause of ESRD in childhood & adolescence. Biliary & pancreatic duct epithelia 719 Extrarenal manifestation of ADPKD is ? D. Harrison’s 18th Ed. <3% A. Intracranial aneurysms present in 5 to 10 % of patients Diagnostic criteria for ADPKD require two or more cysts in one kidney and at least one cyst in contralateral kidney in young subjects. Liver D. 2355 Protein expressed by PKHD1 is termed fibrocystin (polyductin). 2358 B. D. 2355 A. In adulthood B. In adoloscence A. Polycystin-1 Harrison’s 18th Ed. but four or more in subjects older than 60 years. A. 2358 C. colonic diverticulae. thyroid C. Nephronophthisis (NPHP) C. 2356 Fibrocystin (polyductin) is found in cortical & medullary collecting ducts & thick ascending limb of Henle’s loop in kidney and in biliary & pancreatic duct epithelia. 2355 723 Autosomal recessive polycystic kidney disease (ARPKD) gene A. >=3 in one & 1 cysts in other kidney D. <1% 725 Fibrocystin is found in ? B. Death in neonates is due to pulmonary hypoplasia B. Cysts may be found in brain. C. Harrison’s 18th Ed. Intracranial aneurysm B. PKHD2 ARPKD gene on chromosome 6p21 is called PKHD1 (polycystic kidney & hepatic disease 1) 717 In ADPKD. All of the above D. Von Hippel-Lindau Disease . Bartter’s syndrome 739 Carpopedal spasm. 734 Protein affected in Bartter’s syndrome is ? best relates to ? Harrison’s 18th Ed. All of the above Bartter’s syndrome is the result of mutations in ion transport proteins in TAL. fever. Loss of cell-positive electrical transport potential Most patients with medullary cystic kidney disease 2 (MCKD2) have severe hyperuricemia and D. Renal cysts A. Colonic diverticulae D. 729 Presence of hyperuricemia and gout point towards the diagnosis of which of the following ? 735 Which of the following electrical situations occur in Bartter’s Harrison’s 18th Ed. 732 Which of the following is false for Bartter’s syndrome ? 738 Gitelman’s syndrome is distinguished from Bartter’s syndrome Harrison’s 18th Ed. vomiting and diarrhea from PGE2 overproduction. Inherited as autosomal recessive trait Harrison’s 18th Ed. Metabolic alkalosis D. AJAY MATHUR Nephrology 728 Senior-Loken syndrome. Liddle’s syndrome Hyperprostaglandin E syndrome is a severe form of Bartter’s syndrome. Elevated renin and aldosterone levels D. Type 2 B. 2361 B. Gitelman’s syndrome D. Neonates present with pronounced volume depletion. In Bartter’s syndrome. 2359 syndrome ? A. Apical loop-diuretic sensitive Na-K-Cl co-transporter B. 2360 736 Gitelman’s syndrome is due to mutations in ? Harrison’s 18th Ed. Thiazide-sensitive Na-Cl co-transporter (NCCT) D. D. 2359 A. type 2 is due to Senior-Loken syndrome. All of the above D. type 3 is due to basolateral chloride channel ClC-Kb and Bartter type 4 results from mutations in barttin. Hypokalemia B. Hypokalemia C. Apical potassium channel C. NKCC2 A. Type 3 C. 2360 B. Loss of cell-negative electrical transport potential precocious onset of gout. Barttin C. renal angiomyolipomas & renal cell carcinoma. All of the above C. Medullary cystic kidney disease 2 A. Renal cell carcinoma C. apical potassium channel ROMK. following forms ? Harrison’s 18th Ed. an essential subunit of ClC-Ka and ClC-Kb that enables transport of the chloride channels to the cell surface. Bartter’s syndrome B. Severe magnesium wasting occurs in Gitelman’s syndrome. Liddle’s syndrome B. hypercalciuria & mild 730 Renal tuberous sclerosis (TS) may occur in which of the hypomagnesemia. Bartter’s syndrome C. ROMK B. besides juvenile nephronophthisis. the syndrome is called loop-diuretic sensitive sodium-potassium-chloride co-transporter protein NKCC2. Elevated renin and aldosterone levels In Bartter’s syndrome. Gitelman’s syndrome is due to mutations in thiazide-sensitive Na-Cl co-transporter (NCCT) in DCT. Type 1 is due to apical When juvenile NPHP has retinitis pigmentosa as an extrarenal manifestation. Gitelman’s syndrome A. 2360 by ? A. Renal angiomyolipomas B. Hypercalciuria marked overproduction of renal prostaglandins 733 Hyperprostaglandin E syndrome is related to ? (PGE2) occurs in Bartter’s syndrome. Type 1 A. Basolateral chloride channel ClC-Kb Renal TS occurs as renal cysts. 2361 A. 2360 Harrison’s 18th Ed. Due to abnormality in renal tubule transport proteins A. Gitelman’s syndrome is distinguished from Bartter’s syndrome by presence of severe hypomagnesemia & hypocalciuria. 2361 A. Loss of lumen-negative electrical transport potential D.376 MCQ’s FOR MEDICAL PROFESSIONALS BY PROF. Metabolic alkalosis C. cramps & tetany are clinical features of ? Harrison’s 18th Ed. Prostaglandin E production is low B. Hypomagnesemia Bartter type 4 is due to mutations in barttin which is also expressed in inner ear. calcium & magnesium causes NaCl wasting. Deafness is invariably associated with Bartter type 4. Type 4 D. loss of lumen-positive electrical transport potential that normally drives paracellular reabsorption of sodium. Intracranial aneurysm B. 731 Deafness is invariably associated with which type of Bartter’s syndrome ? 737 Gitelman’s syndrome is distinguished from Bartter’s syndrome Harrison’s 18th Ed. Harrison’s 18th Ed. Metabolic abnormalities similar to diuretic abuse C. 2361 B. Hypocalciuria marked overproduction of renal prostaglandins (PGE2). 2360 by ? Harrison’s 18th Ed. Autosomal recessive polycystic kidney disease Harrison’s 18th Ed. Loss of lumen-positive electrical transport potential C. Nephronophthisis (NPHP) C. 2360 A. Retinitis pigmentosa C. RAS activation causes increased levels of cyclooxygenase 2 (COX-2) & D. failure to thrive. 377 Nephrology MCQ’s FOR MCQ’s FOR MEDICAL MEDICAL PROFESSIONALS PROFESSIONALS BY PROF. glycogen storage disease type I. Autosomal recessive familial hypomagnesemia with diabetes insipidus (NDI) ? hypercalciuria and nephrocalcinosis (FHHNC). 2363 hypocalcemia (HSH). Medullary cystic kidney disease (MCKD) (Proximal) RTA ? C. C. ADPKD 751 Which of the following inherited disorders produce Type 2 B. and autosomal dominant hypoparathyroidism can also produce such presentation. All of the above Harrison’s 18th Ed. 2 . All of the above B. Chronic active hepatitis 744 During antenatal period. All of the above A. 2363 B. A. tyrosinemia. Chloride-independent sodium reabsorption D. B. Medullary cystic kidney disease (MCKD) 750 Systemic disorder that cause type 1 RTA include ? C. 1699 D. C. 2364 D. Polyuria A. hereditary fructose intolerance. Sodium-bicarbonate co-transporter A. dehydration. High aldosterone levels D. Bartter’s syndrome Harrison’s 18th Ed. 2363 741 Which of the following is false for Liddle’s syndrome ? A. 2364 D. Gitelman’s syndrome A. Fabry’s disease 743 Polyuria is a prominent feature of all except ? Harrison’s 18th Ed. B. 2 Liddle’s syndrome ? Harrison’s 18th Ed. Lupus Harrison’s 18th Ed. combined with salt restriction. and Lowe’s syndrome. 3 B. Hyponatremia 740 Which of the following best relate to Liddle’s syndrome ? Harrison’s 18th Ed. Seizures & mental retardation B. A. Wilson’s disease A. AJAY MATHUR Cardiology 377 Gitelman’s syndrome is characterized by hypocalciuria. 2362 Table 284-3 Inherited disorders that produce Type 2 (Proximal) RTA are Wilson’s disease. Cystinosis syndrome. Wilson’s disease In antenatal period. Sjogren’s syndrome B. 4 C. Glycogen storage disease type I 745 ENaC best relates to ? D. 2363 C. 1 742 Which of the following drug is ineffective in treatment of B. autosomal dominant hypomagnesemia. cystinosis. All of the above 747 Which of the following renal tubular acidosis type is rare ? Harrison’s 18th Ed. provide effective therapy for hypertension and hypokalemia. Triamterene D. A. Na-K-2Cl co-transporter 752 Serum potassium is high in which of the following RTA ? Harrison’s 18th Ed. Overactive ENaC in cortical collecting duct Hereditary nephrogenic diabetes insipidus (NDI) presents in infancy with severe vasopressin- resistant polyuria. Spironolactone D. Features include early and severe hypertension. Hypokalemia D. Gitelman’s syndrome A. 2363 A. Ehler-Danlos syndrome B. High plasma levels of vasopressin B. C. Plasma aldosterone & renin levels are low. Bartter’s Syndrome Harrison’s 16th Ed. Amiloride-sensitive epithelial sodium channel galactosemia. severe hypomagnesemia resulting in 746 Which of the following is false about hereditary nephrogenic carpopedal spasm. fetal polyuria may cause maternal polyhydramnios & premature labor in Bartter’s B. 4 A. Hypertension C. 1 Harrison’s 18th Ed. and dilute urine despite the presence of hypernatremia. 2363 C. Harrison’s 18th Ed. cramps and tetany. Amiloride Inherited renal tubular acidosis presents as nonanion gap (hyperchloremic) metabolic acidosis from proximal tubular bicarbonate wasting or impaired distal net acid excretion. 2361 D. Thiazide-sensitive Na-Cl co-transporter C. All of the above B. Metabolic alkalosis 748 Serum anion gap is normal in which of the following renal tubular acidosis (RTA) type ? Liddle’s syndrome mimics a state of aldosterone excess. All of the above 749 Hereditary diseases that cause type 1 RTA include ? Harrison’s 16th Ed. Unregulated sodium reabsorption D. 1 ENaC is an amiloride-sensitive epithelial sodium channel. ADPKD D. 1699 Amiloride or triamterene block ENaC and. 2 A. autosomal recessive hypomagnesemia with secondary Harrison’s 18th Ed. 2361 C. fetal polyuria causes maternal polyhydramnios and premature labor in ? C. hypokalemia & metabolic alkalosis. 0 C. Cystine C.5 even after acid loading with oral 765 Acquired disorders associated with Fanconi syndrome include ? ammonium chloride or calcium chloride. short stature. Hartnup Disease B. D. Harrison’s 17th Ed. hypercalciuria. Tyrosinemia D. Rickets or osteomalacia In Hartnup disease.15 mmol/kg/day along with supplemental potassium & B. Dent’s disease D. 764 Inherited systemic disorders associated with Fanconi 758 In Type 1 (Distal) RTA. 1 . Consequent niacin deficiency leads to skin manifestations. nephrolithiasis. 0. Type 1 RTA may present as “marble-brain disease” 761 Renal glucosuria is due to mutations in which gene ? Harrison’s 18th Ed. Dent’s disease D. All of the above A. All of the above In Type 1 (Distal) RTA. Type 1 RTA is due to impaired H ion secretion or HCO3 + . None of the above A. 2364 A. Uric acid Type 4 RTA is acquired in association with moderate renal dysfunction & is characterized by hyperkalemia. tubule C. patients fail to acidify urine to pH ? syndrome include ? Harrison’s 18th Ed. 763 Cerebellar ataxia & pellagra-like skin lesions are a feature of ? 757 Which of the following is not a feature of Type 1 (Distal) RTA ? Harrison’s 18th Ed. CA2 B. 1805 Increased proximal citrate absorption leads to hypocitraturia and together with hypercalciuria predisposes to nephrocalcinosis & calcium phosphate stone formation. Mental retardation gene that encodes sodium-glucose co-transporter SGLT2 in the proximal renal tubule. D. Hypocalciuria D. 4 759 Which of the following stone is formed in Type 1 (Distal) RTA ? Harrison’s 18th Ed. Chronic untreated acidosis may lead to rickets or osteomalacia.5 B. patients fail to acidify urine to pH <5. Short stature Renal isolated glucosuria with normal blood glucose concentration is due to mutations in SLC5A2 C. NSAIDs B. Calcium oxalate 753 Drug-induced type 4 RTA is due to which of the following drugs ? D. SLC7A9 B. While. hypocitraturia. 2364 Harrison’s 18th Ed. nephrocalcinosis. Type 1 (Distal) RTA presents as hypokalemia. All of the above A. 2365 A. in Type 2 (proximal) RTA the dose is 5 . tryptophan is retained in intestinal lumen & converted to indole compounds that are toxic to CNS. 2364 C. 2364 D. nephrocalcinosis and nephrolithiasis are features of Dent’s disease. Hartnup Disease B. < 7. Osteopetrosis D.10 mmol/kg/day reabsorption in distal nephron In Type 1 (Distal) RTA. 2364 C.5 D. Hypokalemia B. ACE inhibitors 760 Bicarbonate replacement dose in Type 1 (Distal) RTA is ? C. SLC6A19 755 Marble-brain disease consists of ? B. AJAY MATHUR Nephrology C. Multiple myeloma . bicarbonate replacement dose is 1 .1 mmol/kg/day B.reabsorption in proximal vitamin D. All of the above 762 Low-molecular-weight proteinuria is a feature of ? 756 Marble-brain disease is due to mutations in ? Harrison’s 18th Ed. Type 2 RTA is due to impaired HCO3.3 mmol/kg/day in divided doses. 2364 D. Calcium phosphate Harrison’s 17th Ed. SLC3A1 C. SLC5A2 Harrison’s 18th Ed. Renal glucosuria A. < 6. B. and mental retardation with dRTA.378 MCQ’s FOR MEDICAL PROFESSIONALS BY PROF. Cystinuria C. CLCN5 Low-molecular-weight proteinuria. < 5. Cystinuria C. A. Hypocitraturia C. SLC3A1 A. It is due to mutations in CA2 (carbonic anhydrase II). hypercalciuria. 2365 Harrison’s 18th Ed.3 mmol/kg/day 754 Which of the following statements is false ? Harrison’s 18th Ed.5 mmol/kg/day A. 2365 Harrison’s 18th Ed. 2364 D. Renal glucosuria A. 2364 A.5 A.2 . < 7. Galactosemia C. Heparin Harrison’s 18th Ed. 1805 A. 5 . 3 . Marble-brain disease consists of osteopetrosis. SLC6A19 D. Wilson’s disease B. Gout hypertension. Fibromuscular dysplasias C. A. Medullary sponge kidney A. Inflammatory disorders A. Progressive renal failure insufficiency. Hypertension associated with renal insufficiency reabsorption of filtered urate leading to acute gouty arthritis called saturnine gout. and drug-resistant B. embolic. Fibromuscular diseases 776 Which of the following is termed as renal enzyme ? Harrison’s 17th Ed.361:1972-8 Chapter 286. Serum aspartate aminotransferase (AST) C. 1811 B. Lead nephropathy 772 What percentage of cases of hypertension are caused by renal artery stenosis (RAS) ? C. 1808 (BP>160/100 mm Hg) after 50 years of age. Primary hematologic disorders B. Usually male calcifications surrounding the central sinus complex in a “garland” pattern. Plasma Cell Dyscrasias Harrison’s 17th Ed. Computed tomographic angiography 770 Which of the following threaten blood supply of the kidney ? D. ~5% nephropathy & sickle cell disease. Tubulointerstitial A. A. Polycystic kidney disease B. Usually young (<40 yr) “Ring sign” (radiolucent sloughed papilla surrounded by radiodense contrast material in calyx) on pyelogram is pathognomonic of papillary necrosis due to analgesic nephropathy. Hypercalcemia include all except ? N Engl J Med 2009. and primary hematologic disorders. All of the above . 2375 Chapter 285. Atherosclerotic disease 766 Tubulointerstitial disease affecting predominantly medullary D. C. Onset of stage 2 hypertension >30 years of age D. ~7% 767 “Ring sign” on the pyelogram is pathognomonic of ? D. ~1% Tubulointerstitial diseases that predominantly affect medullary & papillary structures are analgesic B. A. fibromuscular diseases. Magnetic resonance angiography C. Good BP response to revascularization with chronic lead nephropathy ? Patients of RAS due to fibromuscular dysplasia are usually female (15 . 1807 Large vessel renal artery occlusive disease results most commonly from atherosclerotic disease and also from extrinsic compression of the vessel and fibromuscular dysplasias. inflammatory. All of the above A. HTN with renal A. Hyperuricemia A. Chemotherapeutic drugs D. CT reveals papillary B. C. Absence of family history of hypertension Patients with chronic lead nephropathy are characteristically hyperuricemic due to increased C. C. 2375 A. Papillary necrosis 773 Which of the following is not a characteristic of renal artery C. 1807 A. D. Dyslipidemia 775 Which of the following diagnostic imaging tests for renal- D.361:1972-8 C. C. AJAY MATHUR Cardiology 379 B. Hypokalemia 774 The clinical clues for the diagnosis of renal-artery stenosis B. ~ 10 % Harrison’s 17th Ed. Duplex ultrasonography Vascular Injury to the Kidney B. All of the above 771 Large-vessel renal artery occlusive disease result most commonly from ? Harrison’s 18th Ed. Hyponatremia B. Renal function worsens after ACE inhibitors 769 Which of the following is not a feature of lead intoxication ? Classic clinical clues that suggest the diagnosis of RAS are onset of stage 2 hypertension Harrison’s 17th Ed. Transitional cell carcinoma stenosis due to fibromuscular dysplasia ? N Engl J Med 2009. Extrinsic compression of the vessel Diseases of the Kidney B.50 yrs). 1811 D. Heavy metal toxicity Disorders that threaten blood supply of kidney include large vessel atherosclerosis. absence of family history of HTN. 379 Nephrology MCQ’s FOR MCQ’s FOR MEDICAL MEDICAL PROFESSIONALS PROFESSIONALS BY PROF. Middle or distal lesion location 768 Which of the following is a characteristic finding in patients D. None of the above & papillary structures is ? Harrison’s 17th Ed. Serum lactate dehydrogenase (LDH) D. Hypertension artery stenosis provide best image quality & anatomical information ? N Engl J Med 2009. 1807 ~5% of cases of hypertension are caused by renal artery stenosis (RAS).361:1972-8 D. HTN with repeated hospital admissions for heart failure. Analgesic nephropathy B. Digital-subtraction angiography Harrison’s 18th Ed. renal function worsens after administration of renin–angiotensin–aldosterone system blocker. Harrison’s 17th Ed. Uremia C. Cerebrovascular accident B. Efferent arterioles D. P-ANCA Harrison’s 17th Ed. 1814 atheroembolic renal disease include all except ? A. 777 Most common cause of cholesterol crystal embolization in atheroembolic renal disease is ? 783 Which of the following is false regarding “onion-skin lesion” ? Harrison’s 17th Ed. “Purple” toes D. AJAY MATHUR Nephrology C. Anticoagulation with heparin C. 1813 Harrison’s 17th Ed. CHF embolization in atheroembolic renal disease ? Harrison’s 17th Ed. Flea-bitten appearance of kidneys is due to hemorrhages in surface capillaries. Harrison’s 17th Ed. All of the above B. abruptio placentae & preeclampsia. Afferent arterioles D. None of the above Arteriographic procedures are the most common cause of cholesterol crystal embolization in 784 In the natural course of malignant hypertension. Nephrolithiasis walls & narrowing of vascular lumen due to deposition of homogeneous eosinophilic material (hyaline arteriolosclerosis) resulting in ischemic injury to glomeruli & tubules. Its natural course includes a death rate of 80 . All of the above C. Smooth-muscle antibody (SMA) A. and CNS involvement. Abruptio placentae pathology ‘Hyaline arteriolosclerosis’ is in ? Harrison’s 17th Ed. fluid retention. Thrombotic thrombocytopenic purpura (TTP) Renal enzymes elevated in renal infarction are aspartate aminotransferase (AST). Arteriography B. 1814 A. always due to ? Harrison’s 17th Ed. 1812 B. findings in cholesterol crystal embolization in Harrison’s 17th Ed. Livedo reticularis C. Cholesterol-lowering agents SRC is a rapid deterioration in renal function. Rapid deterioration in renal function Harrison’s 17th Ed. Septic abortion 781 In ‘Benign arteriolar nephrosclerosis’ the characteristic B. C. findings in cholesterol crystal embolization in atheroembolic renal disease include rising BUN 786 Scleroderma renal crisis (SRC) includes ? & creatinine. Hypocomplementemia Lab. microangiopathic hemolytic anemia. 1812 B. Seen in malignant arteriolar nephrosclerosis B. Anti-RNA POL3 C. All of the above 787 Acute bilateral cortical necrosis is associated with ? Withdrawal of anticoagulation may be beneficial. Malignant arteriolar nephrosclerosis . Serum alkaline phosphatase B. anemia & hypocomplementemia. 1815 A. death is almost atheroembolic renal disease. almost always due to uremia. Toe gangrene Malignant hypertension is a medical emergency. Microangiopathic hemolytic anemia A. 1812 A.90% within 1 year of onset. Rising blood urea nitrogen & creatinine C. Preeclampsia A. Malignant hypertension 780 Which of the following approaches in atheroembolic renal disease is not beneficial ? B. Calcium A. D. Anemia Anti-RNA POL3 antibodies are strongly associated with scleroderma renal disease D. elevated ESR. 1813 C.380 MCQ’s FOR MEDICAL PROFESSIONALS BY PROF. Harrison’s 17th Ed. proteinuria. All of the above B. Ischemic heart disease A. Steroid therapy oliguria. eosinophilia. Renal infarction Renal infarction secondary to cholesterol embolization is rare. C. 1812 C. Hyperplastic arteriolitis D. Capillaries Acute bilateral cortical necrosis is associated with septic abortions. usually accompanied by malignant hypertension. anti-Jo-1 antibody B. Anticoagulation D. 788 The element in ‘Struvite’ is ? 782 ‘Flea-bitten appearance’ of kidneys is seen in ? Harrison’s 17th Ed. 1813 A. eosinophiluria. 785 Which of the following antibodies are strongly associated with scleroderma renal disease ? 779 Lab. All of the above Characteristic pathology of arteriolar nephrosclerosis is in afferent arterioles that have thickened Chapter 287. Vascular surgery A. lactate D. Thrombolytic therapy D. Eosinopenia D. Involves interlobular arteries C. Hemolytic uremic syndrome (HUS) D. 1815 Harrison’s 17th Ed. leukocytosis. All of the above dehydrogenase (LDH). D. and alkaline phosphatase. 1813 778 Which of the following is rare in cholesterol crystal A. 1816 798 Urine may appear ‘milky’ due to which of the following ? A. Hypothyroidism Harrison’s 17th Ed. C. Cystine D. Any of the above The chemical formula of struvite is MgNH4PO4 Multiple papillary calcifications are called nephrocalcinosis. AJAY MATHUR Cardiology 381 B. 1820 793 Urine crystals that are rectangular prisms and resemble ‘coffin A. vitamin D intoxication. Uric acid C. In urine specimens obtained with indwelling catheter. 1816 Table 281-1 Infection due to bacteria that possess the enzyme urease can cause stones composed of struvite A. sarcoidosis. Uric acid 790 Causes of ‘Hypercalciuria’ include ? Harrison’s 17th Ed. Vitamin D intoxication B. Zinc D. and Prostatitis B. 1710 Chapter 288. Rapidly progressive bone disease 797 Radiopacity of Cystine stones is due to ? Harrison’s 16th Ed. cystine & struvite stones are radiopaque on standard x-rays. 1816 infection ? A. Uric acid C. a growth of >=105 organisms per milliliter from a properly collected midstream “clean-catch” 795 Nephrocalcinosis best relates to ? urine sample indicates infection. 1710 C. Cushing syndrome. Multiple cortical calcifications . Calcium oxalate crystals D. Alumina Pyelonephritis. Magnesium C. Cystine pyelonephritis. Struvite B. Uric acid crystals C. Occur mainly in men Harrison’s 16th Ed. Uric acid Acute urinary tract infections are either lower tract (urethritis & cystitis) or upper tract (acute C. Pyelonephritis Harrison’s 16th Ed. 1820 B. Occur in patients with chronic bladder catheterization A. 789 Causes of ‘Hypercalciuria’ include all except ? 796 Infection due to bacteria that possess enzyme urease can cause Harrison’s 17th Ed. A. Struvite D. 1710 C. Zinc content D. Calcium content Hyperthyroidism. Perinephric abscess B. 1816 of 102–104/mL indicates infection. Multiple papillary calcifications D. 381 Nephrology MCQ’s FOR MCQ’s FOR MEDICAL MEDICAL PROFESSIONALS PROFESSIONALS BY PROF. May grow to produce a “staghorn” appearance C. Urinary Tract Infections. rapidly progressive bone disease. > 104 Calcium. Due to UTI with urease-producing bacteria B. Magnesium content 791 Which of the following is false about ‘Struvite stones’ ? Harrison’s 17th Ed. Cystine A. malignant tumors. > 105 In UTI. Struvite upper urinary tract” ? Harrison’s 17th Ed. 792 Basic constituents of most kidney stones include all except ? Harrison’s 16th Ed. Cystitis lids’ are of ? B. 1817 B. Calcium D. Immobilization C. Cystine 799 Which of the following is not included in “Acute infections of D. All of the above B. Cystine crystals D. Uric acid stones are radiolucent. Sarcoidosis B. colony counts Harrison’s 17th Ed. prostatitis & intrarenal & perinephric abscesses) infection. Calcium phosphate crystals Struvite stones occur mainly in women. 1711 B. D. Cushing syndrome A. immobilization. > 102 C. Prostatitis A. Calcium C. > 103 D. 1816 Table 281-1 stones composed of ? A. Calcium Harrison’s 17th Ed. Struvite 800 How many colony counts per mL grown from a properly 794 Which of the following stone is radiolucent ? collected midstream “clean-catch” urine sample indicates Harrison’s 17th Ed. Aluminium B. and Paget’s disease cause hypercalciuria. Multiple medullary calcifications C. Sulfur content D. A. Paget’s disease A. D. Persistent intestinal infection C. 1822 D. cytokines & chemokines from bladder epithelial cells provide antibacterial properties to urine. Bacteriuria C. 1821 A. properties of urine’ ? urealyticum. 806 Predisposition to upper urinary tract infection during interstitial nephritis or polycystic disease. hemolysin and aerobactin and are resistant to the bactericidal action of human serum. K. Hematuria 804 Vaginal introitus & distal urethra are normally colonized by all D. A. AJAY MATHUR Nephrology 801 “Same-strain” recurrent infection evident within 2 weeks of 807 E. Production of aerobactin D. . prostatic secretions. C. All of the above Sterile pyuria may be seen in calculi. Insignificant bacteriuria B. and H serogroups. All of the above A. Decreased ureteral peristalsis A. and temporary incompetence of vesicoureteral valves. U. 1823 D. B. Hematuria D.382 MCQ’s FOR MEDICAL PROFESSIONALS BY PROF. trachomatis. Prostatic secretion 811 “Sterile pyuria” may be demonstrated in ? Harrison’s 17th Ed. uropathogenic E.coli Harrison’s 17th Ed. 1820 facilitates its infectivity ? A. urealyticum lactobacilli & staphylococcal species but not by enteric gram-negative bacilli that commonly cause C. H D. U. Unresolved renal or prostatic infection A. All of the above C. upper UTI is due to decreased ureteral tone. coli strains causing symptomatic UTIs in noncatheterized cessation of therapy is due to ? patients belong which of the following serogroups ? Harrison’s 16th Ed. Urgency A. Pyuria D. 1821 In the urine of acute pyelonephritis patients. coli strains that cause symptomatic UTIs in noncatheterized patients belong to a small unresolved renal or prostatic infection (relapse) or of persistent vaginal or intestinal colonization. All of the above D. coli strains produce cytotoxins. Temporary incompetence of vesicoureteral valves B. Frequency B. Harrison’s 17th Ed. Urgency Pyelonephritis’ ? Harrison’s 17th Ed. Interstitial nephritis High urea concentration & high osmolarity. 1821 812 Leukocyte esterase “dipstick” test is done for ? A. All of the above except ? Harrison’s 17th Ed. All of the above 805 Which of the following contributes to the ‘antibacterial Pyuria in the absence of bacteriuria (sterile pyuria) may indicate infection with C. High urine osmolarity B. Any of the above “Same-strain” recurrent infections appearing within 2 weeks of cessation of therapy can be due to Most E. O B. urgency and frequency unaccompanied by significant bacteriuria is termed acute urethral syndrome. Renal angle tenderness Symptoms of dysuria. 802 Symptom related to acute urethral syndrome is ? 808 Which of the following property of uropathogenic E. number of specific O. Production of hemolysin D. 1822 B. Mycobacterium tuberculosis or fungi. Fimbriae C. Persistent vaginal infection B. High urine urea concentration A. A. Vesicoureteral reflux D. decreased ureteral peristalsis. Leukocyte casts in urine C. All of the above 803 Which of the following is true for ‘acute urethral syndrome’ ? Harrison’s 17th Ed. trachomatis Vaginal introitus & distal urethra are normally colonized by diphtheroids. K C. Dysuria Harrison’s 17th Ed. Nephrocalcinosis C. D. Lactobacilli infection with ? C. leukocyte casts are pathognomonic. Dysuria 809 Which of the following is pathognomonic for ‘Acute C. 1822 A. Mycobacterium tuberculosis UTIs. streptococcal species. 1715 Harrison’s 17th Ed. All of the above C. B. All of the above C. 1820 Besides fimbriae. Staphylococcal species Harrison’s 17th Ed. 1822 B. Leukocyte esterase “dipstick” method is useful in identifying pyuria. vesicoureteral reflux. Escherichia coli A. 1822 B. nephrocalcinosis. Decreased ureteral tone Harrison’s 17th Ed. All of the above During pregnancy. anatomic abnormality. pregnancy is due to ? Harrison’s 17th Ed. D. Streptococcal species 810 Pyuria in the absence of bacteriuria (sterile pyuria) indicates B. A. 1825 Patients with diabetes. Sickle cell disease C. Diabetes mellitus leukocytosis. Amoxicillin susceptible to papillary necrosis. coli causes most cases. B. 1720 Emphysematous pyelonephritis almost always occur in diabetics. E. 383 Nephrology MCQ’s FOR MCQ’s FOR MEDICAL MEDICAL PROFESSIONALS PROFESSIONALS BY PROF. 1720 D. coli In pregnancy. C. Nitrofurantoin 815 Emphysematous pyelonephritis is most often due to ? C. E. Chronic alcoholism . chronic alcoholism. It is characte-rized by high fever. and vascular disease are peculiarly A. All of the above A. B. AJAY MATHUR Cardiology 383 813 Acute cystitis in pregnancy can be treated with ? D. acute cystitis can be managed with 7 days of treatment with amoxicillin. nitrofurantoin. Cephalosporin Harrison’s 16th Ed. Proteus or a cephalosporin. All of the above Harrison’s 17th Ed. Pseudomonas Harrison’s 16th Ed. B. Klebsiella 814 Patients of UTI susceptible to papillary necrosis include ? D. sickle cell disease. renal parenchymal necrosis and accumulation of fermentative gases in kidney & perinephric tissues. Notes : . Answer MCQ’s FOR MCQ’s FOR MEDICAL MEDICAL PROFESSIONALS PROFESSIONALS BY PROF. AJAY MATHUR 385 ANSWERS NEPHROLOGY 1 .................. D 36 ................ D 71 ................ B 106 .............. D 141 .............. D 176 .............. D 2 .................. D 37 ................ C 72 ................ B 107 .............. C 142 .............. D 177 .............. A 3 .................. A 38 ................ D 73 ................ A 108 .............. B 143 .............. D 178 .............. B 4 .................. A 39 ................ D 74 ................ C 109 .............. A 144 .............. B 179 .............. D 5 .................. B 40 ................ D 75 ................ B 110 .............. D 145 .............. C 180 .............. C 6 .................. D 41 ................ A 76 ................ C 111 .............. D 146 .............. D 181 .............. C 7 .................. D 42 ................ C 77 ................ A 112 .............. C 147 .............. D 182 .............. C 8 .................. A 43 ................ B 78 ................ B 113 .............. B 148 .............. B 183 .............. D 9 .................. A 44 ................ A 79 ................ D 114 .............. D 149 .............. D 184 .............. D 10 ................ D 45 ................ B 80 ................ B 115 .............. A 150 .............. D 185 .............. A 11 ................ C 46 ................ B 81 ................ D 116 .............. D 151 .............. D 186 .............. D 12 ................ B 47 ................ D 82 ................ B 117 .............. D 152 .............. D 187 .............. C 13 ................ C 48 ................ D 83 ................ D 118 .............. B 153 .............. D 188 .............. C 14 ................ D 49 ................ D 84 ................ A 119 .............. C 154 .............. C 189 .............. D 15 ................ D 50 ................ A 85 ................ A 120 .............. A 155 .............. D 190 .............. D 16 ................ D 51 ................ B 86 ................ A 121 .............. C 156 .............. D 191 .............. D 17 ................ C 52 ................ C 87 ................ A 122 .............. C 157 .............. D 192 .............. A 18 ................ C 53 ................ B 88 ................ D 123 .............. C 158 .............. D 193 .............. B 19 ................ C 54 ................ C 89 ................ C 124 .............. B 159 .............. A 194 .............. C 20 ................ D 55 ................ A 90 ................ D 125 .............. A 160 .............. A 195 .............. A 21 ................ A 56 ................ D 91 ................ D 126 .............. D 161 .............. A 196 .............. D 22 ................ A 57 ................ A 92 ................ C 127 .............. D 162 .............. B 197 .............. D 23 ................ A 58 ................ A 93 ................ D 128 .............. D 163 .............. A 198 .............. D 24 ................ D 59 ................ C 94 ................ D 129 .............. D 164 .............. B 199 .............. B 25 ................ D 60 ................ D 95 ................ D 130 .............. A 165 .............. D 200 .............. D 26 ................ B 61 ................ A 96 ................ D 131 .............. C 166 .............. D 201 .............. D 27 ................ C 62 ................ D 97 ................ D 132 .............. B 167 .............. D 202 .............. C 28 ................ D 63 ................ B 98 ................ B 133 .............. B 168 .............. D 203 .............. D 29 ................ D 64 ................ D 99 ................ A 134 .............. B 169 .............. C 204 .............. D 30 ................ C 65 ................ A 100 .............. B 135 .............. B 170 .............. D 205 .............. D 31 ................ D 66 ................ C 101 .............. C 136 .............. C 171 .............. D 206 .............. D 32 ................ D 67 ................ B 102 .............. C 137 .............. D 172 .............. B 207 .............. A 33 ................ D 68 ................ B 103 .............. C 138 .............. B 173 .............. A 208 .............. D 34 ................ A 69 ................ C 104 .............. D 139 .............. D 174 .............. D 209 .............. C 35 ................ A 70 ................ D 105 .............. C 140 .............. B 175 .............. D 210 .............. C 386 MCQ’s FOR MEDICAL PROFESSIONALS BY PROF. AJAY MATHUR Answer ANSWERS NEPHROLOGY 211 .............. D 246 .............. D 281 .............. B 316 .............. B 351 .............. A 386 .............. A 212 .............. B 247 .............. C 282 .............. B 317 .............. B 352 .............. B 387 .............. B 213 .............. C 248 .............. D 283 .............. B 318 .............. B 353 .............. B 388 .............. D 214 .............. D 249 .............. D 284 .............. D 319 .............. A 354 .............. B 389 .............. D 215 .............. A 250 .............. A 285 .............. A 320 .............. B 355 .............. D 390 .............. D 216 .............. D 251 .............. C 286 .............. C 321 .............. C 356 .............. C 391 .............. C 217 .............. D 252 .............. D 287 .............. D 322 .............. B 357 .............. B 392 .............. D 218 .............. A 253 .............. D 288 .............. D 323 .............. A 358 .............. D 393 .............. D 219 .............. D 254 .............. D 289 .............. A 324 .............. C 359 .............. B 394 .............. C 220 .............. C 255 .............. A 290 .............. A 325 .............. B 360 .............. A 395 .............. D 221 .............. D 256 .............. C 291 .............. D 326 .............. C 361 .............. C 396 .............. D 222 .............. B 257 .............. D 292 .............. C 327 .............. B 362 .............. D 397 .............. B 223 .............. D 258 .............. C 293 .............. C 328 .............. D 363 .............. A 398 .............. D 224 .............. C 259 .............. B 294 .............. D 329 .............. D 364 .............. B 399 .............. D 225 .............. A 260 .............. A 295 .............. D 330 .............. B 365 .............. A 400 .............. B 226 .............. D 261 .............. A 296 .............. D 331 .............. A 366 .............. B 401 .............. D 227 .............. D 262 .............. D 297 .............. D 332 .............. A 367 .............. B 402 .............. D 228 .............. D 263 .............. D 298 .............. D 333 .............. D 368 .............. A 403 .............. D 229 .............. A 264 .............. D 299 .............. B 334 .............. A 369 .............. A 404 .............. D 230 .............. B 265 .............. A 300 .............. D 335 .............. B 370 .............. D 405 .............. A 231 .............. D 266 .............. C 301 .............. C 336 .............. B 371 .............. A 406 .............. D 232 .............. C 267 .............. C 302 .............. D 337 .............. C 372 .............. D 407 .............. D 233 .............. C 268 .............. D 303 .............. A 338 .............. A 373 .............. D 408 .............. C 234 .............. C 269 .............. D 304 .............. D 339 .............. A 374 .............. C 409 .............. C 235 .............. A 270 .............. A 305 .............. B 340 .............. B 375 .............. B 410 .............. D 236 .............. D 271 .............. D 306 .............. A 341 .............. A 376 .............. D 411 .............. D 237 .............. D 272 .............. C 307 .............. A 342 .............. C 377 .............. D 412 .............. B 238 .............. C 273 .............. A 308 .............. D 343 .............. D 378 .............. C 413 .............. D 239 .............. C 274 .............. B 309 .............. A 344 .............. D 379 .............. A 414 .............. D 240 .............. A 275 .............. A 310 .............. C 345 .............. C 380 .............. C 415 .............. D 241 .............. D 276 .............. A 311 .............. A 346 .............. C 381 .............. A 416 .............. D 242 .............. A 277 .............. D 312 .............. D 347 .............. D 382 .............. D 417 .............. B 243 .............. C 278 .............. C 313 .............. C 348 .............. D 383 .............. A 418 .............. B 244 .............. C 279 .............. D 314 .............. D 349 .............. B 384 .............. A 419 .............. D 245 .............. A 280 .............. C 315 .............. C 350 .............. B 385 .............. D 420 .............. A Answer MCQ’s FOR MCQ’s FOR MEDICAL MEDICAL PROFESSIONALS PROFESSIONALS BY PROF. AJAY MATHUR 387 ANSWERS NEPHROLOGY 421 .............. B 456 .............. A 491 .............. D 526 .............. C 561 .............. D 596 .............. C 422 .............. D 457 .............. C 492 .............. D 527 .............. B 562 .............. D 597 .............. B 423 .............. D 458 .............. A 493 .............. D 528 .............. D 563 .............. A 598 .............. D 424 .............. D 459 .............. A 494 .............. D 529 .............. A 564 .............. B 599 .............. D 425 .............. D 460 .............. B 495 .............. D 530 .............. D 565 .............. D 600 .............. D 426 .............. D 461 .............. B 496 .............. D 531 .............. D 566 .............. D 601 .............. D 427 .............. D 462 .............. A 497 .............. D 532 .............. D 567 .............. D 602 .............. C 428 .............. C 463 .............. D 498 .............. D 533 .............. D 568 .............. D 603 .............. D 429 .............. D 464 .............. B 499 .............. D 534 .............. D 569 .............. D 604 .............. A 430 .............. D 465 .............. B 500 .............. B 535 .............. C 570 .............. A 605 .............. B 431 .............. C 466 .............. D 501 .............. D 536 .............. D 571 .............. D 606 .............. D 432 .............. C 467 .............. D 502 .............. D 537 .............. D 572 .............. D 607 .............. D 433 .............. D 468 .............. A 503 .............. D 538 .............. C 573 .............. D 608 .............. A 434 .............. D 469 .............. C 504 .............. D 539 .............. D 574 .............. A 609 .............. A 435 .............. D 470 .............. C 505 .............. B 540 .............. D 575 .............. B 610 .............. B 436 .............. D 471 .............. B 506 .............. D 541 .............. B 576 .............. D 611 .............. D 437 .............. D 472 .............. B 507 .............. D 542 .............. B 577 .............. C 612 .............. A 438 .............. D 473 .............. C 508 .............. D 543 .............. B 578 .............. B 613 .............. B 439 .............. D 474 .............. D 509 .............. D 544 .............. D 579 .............. C 614 .............. A 440 .............. D 475 .............. D 510 .............. D 545 .............. B 580 .............. B 615 .............. A 441 .............. A 476 .............. C 511 .............. B 546 .............. B 581 .............. A 616 .............. C 442 .............. A 477 .............. D 512 .............. D 547 .............. D 582 .............. D 617 .............. C 443 .............. D 478 .............. D 513 .............. C 548 .............. D 583 .............. D 618 .............. C 444 .............. D 479 .............. C 514 .............. D 549 .............. B 584 .............. D 619 .............. D 445 .............. D 480 .............. A 515 .............. D 550 .............. C 585 .............. A 620 .............. D 446 .............. C 481 .............. A 516 .............. D 551 .............. A 586 .............. D 621 .............. D 447 .............. C 482 .............. A 517 .............. D 552 .............. B 587 .............. C 622 .............. C 448 .............. D 483 .............. D 518 .............. B 553 .............. C 588 .............. C 623 .............. A 449 .............. A 484 .............. D 519 .............. B 554 .............. D 589 .............. D 624 .............. C 450 .............. A 485 .............. A 520 .............. D 555 .............. C 590 .............. A 625 .............. D 451 .............. B 486 .............. A 521 .............. A 556 .............. B 591 .............. B 626 .............. D 452 .............. D 487 .............. D 522 .............. D 557 .............. B 592 .............. C 627 .............. B 453 .............. C 488 .............. D 523 .............. D 558 .............. A 593 .............. C 628 .............. D 454 .............. B 489 .............. D 524 .............. D 559 .............. D 594 .............. D 629 .............. D 455 .............. D 490 .............. D 525 .............. A 560 .............. D 595 .............. D 630 .............. A 388 MCQ’s FOR MEDICAL PROFESSIONALS BY PROF. AJAY MATHUR Answer ANSWERS NEPHROLOGY 631 .............. B 666 .............. D 701 .............. D 736 .............. C 771 .............. C 806 .............. D 632 .............. A 667 .............. C 702 .............. C 737 .............. D 772 .............. B 807 .............. D 633 .............. A 668 .............. C 703 .............. D 738 .............. D 773 .............. B 808 .............. D 634 .............. B 669 .............. A 704 .............. B 739 .............. B 774 .............. A 809 .............. B 635 .............. A 670 .............. B 705 .............. A 740 .............. D 775 .............. D 810 .............. D 636 .............. C 671 .............. D 706 .............. A 741 .............. C 776 .............. D 811 .............. D 637 .............. B 672 .............. C 707 .............. D 742 .............. A 777 .............. B 812 .............. B 638 .............. B 673 .............. A 708 .............. D 743 .............. D 778 .............. D 813 .............. D 639 .............. D 674 .............. C 709 .............. A 744 .............. C 779 .............. B 814 .............. D 640 .............. D 675 .............. D 710 .............. D 745 .............. A 780 .............. A 815 .............. A 641 .............. D 676 .............. D 711 .............. B 746 .............. B 781 .............. A 642 .............. D 677 .............. C 712 .............. D 747 .............. C 782 .............. D 643 .............. C 678 .............. A 713 .............. D 748 .............. D 783 .............. D 644 .............. B 679 .............. C 714 .............. D 749 .............. D 784 .............. D 645 .............. C 680 .............. D 715 .............. B 750 .............. D 785 .............. D 646 .............. D 681 .............. D 716 .............. B 751 .............. D 786 .............. D 647 .............. B 682 .............. D 717 .............. A 752 .............. C 787 .............. D 648 .............. D 683 .............. A 718 .............. D 753 .............. D 788 .............. C 649 .............. D 684 .............. A 719 .............. D 754 .............. D 789 .............. A 650 .............. D 685 .............. D 720 .............. D 755 .............. D 790 .............. D 651 .............. C 686 .............. D 721 .............. A 756 .............. A 791 .............. A 652 .............. B 687 .............. D 722 .............. D 757 .............. C 792 .............. A 653 .............. A 688 .............. A 723 .............. C 758 .............. A 793 .............. D 654 .............. D 689 .............. C 724 .............. C 759 .............. D 794 .............. D 655 .............. A 690 .............. C 725 .............. D 760 .............. B 795 .............. C 656 .............. D 691 .............. D 726 .............. C 761 .............. B 796 .............. C 657 .............. A 692 .............. B 727 .............. D 762 .............. D 797 .............. A 658 .............. C 693 .............. B 728 .............. B 763 .............. B 798 .............. D 659 .............. B 694 .............. A 729 .............. B 764 .............. D 799 .............. A 660 .............. D 695 .............. B 730 .............. D 765 .............. D 800 .............. D 661 .............. C 696 .............. D 731 .............. D 766 .............. A 801 .............. D 662 .............. A 697 .............. D 732 .............. C 767 .............. B 802 .............. D 663 .............. B 698 .............. D 733 .............. A 768 .............. C 803 .............. D 664 .............. D 699 .............. D 734 .............. D 769 .............. C 804 .............. D 665 .............. C 700 .............. D 735 .............. A 770 .............. D 805 .............. D 389 Gastroenterology MCQ’s MCQ’s FOR FORPROFESSIONALS MEDICAL MEDICAL PROFESSIONALS BY PROF. AJAY MATHUR Cardiology 389 D. Misdirection of food 292 - Diseases of the Esophagus Inhibition that precedes the peristaltic contraction is called deglutitive inhibition. 1 Dysphagia is defined as a sensation of “sticking” or 7 Peristalsis that begins at the point of oesophageal distention obstruction of the passage of food through ? and proceeds distally is called ? Harrison’s 17th Ed. 237 Harrison’s 18th Ed. 297 A. Mouth A. Primary peristalsis B. Pharynx B. Secondary peristalsis C. Esophagus C. Tertiary peristalsis D. All of the above D. Any of the above Dysphagia is defined as a sensation of “sticking” or obstruction of the passage of food through the Local distention of esophagus activates secondary peristalsis. It begins at the point of distention mouth, pharynx or esophagus. and proceeds distally as in gastroesophageal reflux. 2 Foreign body sensation localized in the neck is termed as ? 8 Which of the following oesophageal contractions is Harrison’s 18th Ed. 297 nonperistaltic ? A. Odynophagia Harrison’s 18th Ed. 297 B. Globus pharyngeus A. Primary peristalsis C. Transfer dysphagia B. Secondary peristalsis D. Phagophobia C. Tertiary peristalsis A foreign body sensation localized in the neck that does not interfere with swallowing & is sometimes D. Any of the above relieved by swallowing is termed as “Globus pharyngeus”. Tertiary esophageal contractions are nonperistaltic, disordered esophageal contractions that occur spontaneously during fluoroscopic observation. 3 Which of the following is characteristic of oropharyngeal dysphagia ? 9 Which of the following is a part of upper esophageal sphincter Harrison’s 18th Ed. 297 (UES) physiologically ? A. Odynophagia Harrison’s 18th Ed. 298 B. Globus pharyngeus A. Cricopharyngeus muscle C. Transfer dysphagia B. Inferior pharyngeal constrictor D. Phagophobia C. Proximal portion of cervical esophagus Transfer dysphagia frequently results in nasal regurgitation & pulmonary aspiration during swallowing D. All of the above and is characteristic of oropharyngeal dysphagia. Physiologically, UES consists of the cricopharyngeus muscle, the adjacent inferior pharyngeal constrictor, and the proximal portion of the cervical esophagus. 4 Whihc of the following may have a psychogenic cause of dysphagia ? 10 Innervation to the musculature acting on UES to facilitate its Harrison’s 18th Ed. 297 opening during swallowing comes from ? A. Globus pharyngeus Harrison’s 18th Ed. 298 B. Transfer dysphagia A. Fifth cranial nerve C. Phagophobia B. Seventh cranial nerve D. All of the above C. Twelfth cranial nerve Phagophobia refers to fear of swallowing. Refusal to swallow thereby may be psychogenic. D. All of the above UES innervation is derived from the vagus nerve, whereas innervation to the musculature acting on 5 “Deglutitive inhibition” best relates to which of the following ? UES to facilitate its opening during swallowing comes from fifth, seventh & twelfth cranial nerves. Harrison’s 18th Ed. 297 A. Smell 11 Which of the following is involved in keeping UES closed at rest ? B. Mastication Harrison’s 18th Ed. 298 C. Peristaltic contraction of esophagus A. Cricopharyngeus muscle D. Gastroesophageal reflux B. Inferior pharyngeal constrictor Primary peristalsis refers to peristaltic contractions elicited in response to a swallow. It is an C. Proximal portion of cervical esophagus interplay of sequenced inhibition followed by contraction of the entire length of esophageal musculature. The inhibition that precedes peristaltic contraction is called deglutitive inhibition. D. All of the above UES remains closed at rest due to its inherent elastic properties and neurogenically mediated 6 Deglutitive inhibition refers to ? contraction of cricopharyngeus muscle. Harrison’s 18th Ed. 297 A. Complete esophageal obstruction 12 Which of the following is a muscle of upper esophageal B. Difficulty in initiating a swallow sphincter (UES) ? Harrison’s 17th Ed. 237 C. Inhibition that precedes peristaltic contraction A. Cricopharyngeus muscle 390 MCQ’s FOR MEDICAL PROFESSIONALS BY PROF. AJAY MATHUR Gastroenterology B. Inferior pharyngeal constrictor muscles B. Pemphigoid C. Geniohyoid muscle C. Epidermolysis bullosa D. All of the above D. All of the above UES consists of constrictor (cricopharyngeus and inferior pharyngeal constrictor muscles) and Scleroderma, pemphigoid and epidermolysis bullosa can involve the esophagus. dilator muscles (suprahyoid muscles including geniohyoid). 19 Gastric distention-evoked transient lower esophageal 13 Which of the following muscle is involved in opening UES sphincter relaxation (tLESR) is a ? during swallowing ? Harrison’s 17th Ed. 1847 Harrison’s 18th Ed. 298 A. Stretch reflex A. Geniohyoid B. Chemical reflex B. Mylohyoid C. Vasovagal reflex C. Stylohyoid D. All of the above D. Styloglossus Gastric distention - evoked transient lower esophageal sphincter relaxation (tLESR) is a vagovagal UES opening during swallowing is due to relaxation of cricopharyngeus muscle (cessation of vagal reflex. excitation) and simultaneous contraction of suprahyoid & geniohyoid muscles. 20 Agents that increase LES pressure are all except ? 14 Motor dysphagia refers to ? Harrison’s 17th Ed. 1847 Harrison’s 18th Ed. 298 A. Substance P A. Weakness of peristaltic contractions B. Prostaglandin F2 B. Impaired deglutitive inhibition C. Secretin C. Impaired sphincter relaxation D. Gastrin D. All of the above Dysphagia caused by a large bolus or a narrow lumen is called structural dysphagia. Dysphagia due 21 Agents that reduce LES pressure are all except ? to weakness of peristaltic contractions or to impaired deglutitive inhibition causing nonperistaltic Harrison’s 17th Ed. 1847 contractions & impaired sphincter relaxation is called propulsive or motor dysphagia. A. Cholecystokinin 15 Length of adult esophagus is ? B. Secretin Harrison’s 18th Ed. 298 C. Dopamine A. 12 - 16 cm D. Gastrin B. 14 - 20 cm 22 Agents that reduce LES pressure are all except ? C. 16 - 24 cm Harrison’s 17th Ed. 1847 D. 18 - 26 cm A. VIP Adult esophagus is 18 - 26 cm in length. Anatomically, it is divided into cervical esophagus (from B. Calcitonin gene-related peptide (CGRP) pharyngoesophageal junction to suprasternal notch) and thoracic esophagus (upto the diaphragmatic hiatus). C. Prostaglandin E 16 In an adult, esophageal lumen can distend up to ? D. Prostaglandin F2 Harrison’s 17th Ed. 238 A. 4 cm in diameter 23 Agents that reduce LES pressure are all except ? Harrison’s 17th Ed. 1847 B. 6 cm in diameter A. Adenosine C. 8 cm in diameter B. Dopamine D. 10 cm in diameter C. Substance P In adult, esophageal lumen can distend up to 4 cm in diameter. When esophagus cannot dilate beyond 2.5 cm in diameter, dysphagia to normal solid food occurs. Dysphagia is always present D. Nitrates when esophagus cannot distend beyond 1.3 cm. Reduction in LES sphincter pressure occurs with phosphodiesterase-5 inhibitors (sildenafil) use, fatty meals, smoking, and beverages with high xanthine content (tea, coffee, cola), nicotine, beta- 17 Hallmark of oropharyngeal dysphagia is ? adrenergic agonists, dopamine, cholecystokinin, secretin, vasoactive intestinal peptide (VIP), calcitonin Harrison’s 18th Ed. 299 gene–related peptide, adenosine, prostaglandin E, nitric oxide donors (nitrates). LES contraction occurs with GABA-B agonists (baclofen), muscarinic M2 and M3 receptor agonists, alpha-adrenergic A. Food impaction agonists, gastrin, substance P & Pg F 2 B. Odynophagia 24 Which of the following statements about upper esophageal C. Nasal regurgitation sphincter (UES) is false ? D. Hoarseness Harrison’s 16th Ed. 1739 Nasal regurgitation & tracheobronchial aspiration with swallowing are hallmarks of oropharyngeal A. Formed by cricopharyngeus & inferior pharyngeal dysphagia or a tracheoesophageal fistula. constrictor muscles B. These muscles exhibit myogenic tone 18 Which of the following skin diseases may involve the oesophagus ? Harrison’s 18th Ed. 300 C. These muscles receive no inhibitory innervation A. Scleroderma D. Opened by central inhibition of sphincter muscles 391 Gastroenterology MCQ’s MCQ’s FOR FORPROFESSIONALS MEDICAL MEDICAL PROFESSIONALS BY PROF. AJAY MATHUR Cardiology 391 25 Which of the following statements about lower esophageal 31 Esophageal peristalsis is best studied in ? sphincter (LES) is false ? Harrison’s 17th Ed. 1848 Harrison’s 16th Ed. 1739 A. Upright position A. Innervated by parallel sets of parasympathetic excitatory & B. Recumbent position inhibitory pathways C. Lateral position B. Opens in response to activity of inhibitory nerves D. Head down position C. Neurotransmitters of excitatory nerves are acetylcholine, Esophageal peristalsis is best studied in the recumbent position, because in the upright position substance P & nitric oxide barium passage occurs largely by gravity alone. D. Neurotransmitters of inhibitory nerves is VIP 32 Motility pattern of esophagus showing reduced amplitude of 26 The most common esophageal symptom is ? contractions in lower esophagus, peristaltic or simultaneous Harrison’s 18th Ed. 2427 in onset with hypotension of LES is suggestive of ? A. Heartburn Harrison’s 17th Ed. 1849 Figure 286-3 B. Regurgitation A. Scleroderma C. Water brash B. Achalasia D. Globus sensation C. Diffuse esophageal spasm D. None of the above Heartburn is the most common esophageal symptom. It is characterized by a discomfort or burning sensation behind sternum arising from epigastrium and may radiate toward the neck. In scleroderma, thoracic esophagus shows reduced amplitude of contractions, which may be peristaltic or simultaneous in onset & hypotension of LES. 27 Pyrosis is best related to ? Harrison’s 18th Ed. 2427 33 Motility pattern of esophagus showing reduced amplitude of A. Fever contractions in lower esophagus, simultaneous in onset with hypertensive LES nonrelaxing on swallowing is suggestive of ? B. Heartburn Harrison’s 17th Ed. 1849 Figure 286-3 C. Defervescence A. Scleroderma D. Pain B. Achalasia Heartburn, or pyrosis, is characterized by burning retrosternal discomfort. C. Diffuse esophageal spasm 28 Most frequent esophageal cause of chest pain is ? D. None of the above Harrison’s 18th Ed. 2427 In achalasia, lower part of esophagus shows contractions that are reduced in amplitude & simultaneous A. Gastroesophageal reflux in onset. In contrast to scleroderma, LES in achalasia is hypertensive and fails to relax in response to a swallow. B. Diffuse esophageal spasm (DES) C. Achalasia 34 Motility pattern of esophagus showing large amplitude, prolonged and repetitive contractions in lower esophagus, D. Esophageal hypersensitivity syndrome simultaneous in onset is suggestive of ? Gastroesophageal reflux is the most common cause of esophageal chest pain. Harrison’s 17th Ed. 1849 Figure 286-3 A. Scleroderma 29 Odynophagia is unusual in ? B. Achalasia Harrison’s 18th Ed. 2427 A. Pill-induced esophagitis C. Diffuse esophageal spasm B. Nonreflux esophagitis D. None of the above C. Esophageal perforation In diffuse esophageal spasm, lower part of esophagus shows simultaneous-onset, large-amplitude, prolonged, repetitive contractions. D. Uncomplicated reflux esophagitis Odynophagia (painful swallowing) is characteristic of nonreflux esophagitis, herpes & pill-induced 35 Esophageal motility studies are helpful in the diagnosis of all esophagitis. Odynophagia may occur with peptic ulcer of esophagus (Barrett’s ulcer), carcinoma except ? with periesophageal involvement, caustic damage of esophagus, and esophageal perforation. Harrison’s 17th Ed. 1849 Odynophagia is unusual in uncomplicated reflux esophagitis. A. Achalasia 30 Reflex salivary hypersecretion in response to acidification of B. Diffuse esophageal spasm the esophageal mucosa is called ? C. Scleroderma Harrison’s 18th Ed. 2427 D. Mechanical dysphagia A. Water brash Esophageal motility studies are helpful in the diagnosis of esophageal motor disorders (achalasia, B. Salivary brash spasm, and scleroderma) but are of little value in the differential diagnosis of mechanical dysphagia. C. Esophageal brash D. Barret’s brash 36 Which is the most common type of hiatal hernia ? Harrison’s 18th Ed. 2428 Water brash is excessive reflex salivation resulting from a vagal reflex triggered by acidification of the esophageal mucosa. It is not regurgitation which is effortless appearance of gastric or esophageal A. Type I contents in the mouth. B. Type II 392 MCQ’s FOR MEDICAL PROFESSIONALS BY PROF. AJAY MATHUR Gastroenterology C. Type III C. Stomach D. Type IV D. Colon Type I or sliding hiatal hernia comprising ~95% of hiatus hernias. Schatzki ring is one of the most common causes of intermittent food impaction, also called “steakhouse syndrome” as meat is a typical instigator. 37 When colon herniate into the mediastinum, the type of hiatus hernia is ? 43 Which of the following is false about Plummer-Vinson syndrome ? Harrison’s 18th Ed. 2428 Harrison’s 18th Ed. 2429 A. Type I A. Young women B. Type II B. Symptomatic proximal esophageal web C. Type III C. Iron-deficiency anemia D. Type IV D. All of the above With type IV hiatal hernias, viscera other than stomach herniate into mediastinum, most commonly Symptomatic hypopharyngeal webs and iron-deficiency anemia in middle-aged women constitutes the colon. Plummer-Vinson syndrome. 38 Intersection of squamous epithelium of tubular oesophagus 44 Which of the following is false about Zenker’s diverticula ? Harrison’s 18th Ed. 2429 & columnar epithelium of stomach is termed ? Lancet 2009; 373:850 - 61 A. Hypopharyngeal A. W line B. False diverticula B. X line C. Associated with distal obstruction C. Y line D. None of the above D. Z line 45 Which of the following is false about Zenker’s diverticulum ? The intersection of squamous epithelium of the tubular oesophagus & columnar epithelium of Harrison’s 18th Ed. 2429 stomach is termed Z line, because of jagged appearance of the interface. A. Occurs below the Killian’s triangle 39 A lower esophageal mucosal ring is also called ? B. Causes halitosis & regurgitation of saliva & food Harrison’s 18th Ed. 2429 C. Nasogastric intubation may cause perforation A. A ring D. Symptomatic pt’s treated by cricopharyngeal myotomy B. B ring Zenker’s diverticulum appears in natural zone of weakness in posterior hypopharyngeal wall (Killian’s C. C ring triangle). D. D ring 46 Esophageal inlet patch best relates to ? A lower esophageal mucosal ring, also called B ring, is a thin membranous narrowing at the Harrison’s 18th Ed. 2430 squamocolumnar mucosal junction. A. Esophageal atresia 40 Location of ‘Schatzki ring’ is ? B. Benign esophageal tumor Harrison’s 18th Ed. 2429 C. Heterotopic gastric mucosa A. Hypopharyngeal D. Zenker’s diverticula B. Mid esophageal Heterotopic gastric mucosa, also known as an esophageal inlet patch, is an area of gastric type C. Lower esophageal epithelium in proximal cervical esophagus. The inlet patch is due to incomplete replacement of embryonic columnar epithelium with squamous epithelium. D. Any of the above Schatzki ring is a lower esophageal mucosal ring. It is a thin, weblike constriction located at the 47 Which of the following statements about achalasia is false ? squamo-columnar mucosal junction at or near the border of the LES. It may result from GERD or be Harrison’s 18th Ed. 2430-31 congenital in origin. A. Motor disorder of esophageal smooth muscle 41 Schatzki ring invariably produces dysphagia when the lumen B. UES & LES relax normally with swallowing diameter is ? C. Nonperistaltic contractions in esophageal body Harrison’s 18th Ed. 2429 D. Underlying abnormality is loss of intramural neurons A. < 1.3 cm With a swallow, pressure in sphincters falls and a contraction wave starts in pharynx and progresses B. < 2.3 cm down the esophagus. In achalasia, esophageal body loses peristaltic contractions & are replaced by C. < 3.3 cm simultaneous contractions with elevated resting pressure & LES does not relax normally in response to swallowing. Prominent degeneration of nerve cell bodies seen in achalasia. D. < 4.3 cm 48 Which of the following statements about achalasia is false ? Schatzki ring invariably produces dysphagia when the lumen diameter is <1.3 cm. Harrison’s 18th Ed. 2430-31 42 “Steakhouse syndrome” best relates to ? A. Underlying abnormality is loss of intramural neurons Harrison’s 18th Ed. 2429 B. Inhibitory neurons containing substance P are A. Mouth predominantly involved B. Oesophagus C. Inhibitory neurons containing VIP are involved 393 Gastroenterology MCQ’s MCQ’s FOR FORPROFESSIONALS MEDICAL MEDICAL PROFESSIONALS BY PROF. AJAY MATHUR Cardiology 393 D. Presence of GE reflux goes against achalasia C. Hypercontracting LES Inhibitory neurons containing VIP and nitric oxide synthase are predominantly involved. D. Hypertensive LES The hypertensive disorders of esophagus include nutcracker esophagus, hypercontracting LES and 49 Which of the following is not a feature of achalasia ? hypertensive LES. DES is characterized by nonperistaltic contractions. Harrison’s 18th Ed. 2430-31 A. Dysphagia 55 Esophageal contractions are normally peristaltic but hypertensive in ? B. Gastroesophageal reflux Harrison’s 18th Ed. 2432 C. Chest pain A. Nutcracker esophagus D. Regurgitation B. Hypercontracting LES Dysphagia, chest pain, regurgitation and weight loss are the main symptoms of achalasia. Presence C. Hypertensive LES of gastroesophageal reflux goes against achalasia. D. All of the above 50 Chest x-ray in achalasia may show all except ? In nutcracker esophagus, esophageal contractions are normally peristaltic but hypertensive. In Harrison’s 17th Ed. 1850 hypercontracting LES, normal sphincter relaxation is followed by hypertensive contraction. In hypertensive A. Absence of the gastric air bubble LES, basal LES pressure is elevated, but sphincter relaxation and contraction are normal. B. Tubular mediastinal mass beside the aorta 56 “Corkscrew” esophagus is typical of ? C. Air-fluid level in mediastinum in upright position Harrison’s 18th Ed. 2432 D. Atelectasis A. Diffuse esophageal spasm (DES) A chest x-ray shows absence of the gastric air bubble and sometimes a tubular mediastinal mass B. Achalasia beside the aorta. An air-fluid level in the mediastinum in the upright position represents retained food C. Carcinoma esophagus in the esophagus. Barium swallow shows esophageal dilation, and in advanced cases the esophagus may become sigmoid. On fluoroscopy with barium swallow, normal peristalsis is lost in the lower D. GERD two-thirds of the esophagus. The terminal part of the esophagus shows a persistent beaklike narrowing representing the nonrelaxing LES. Barium swallow in DES shows curling or multiple ripples in the wall, sacculations, and pseudodiverticula - termed “corkscrew” esophagus. 51 In CCK test, paradoxical contraction of LES is found in ? Harrison’s 17th Ed. 1850 57 Which of the following contributes to the development of A. Achalasia gastroesophageal reflux disease (GERD) ? Harrison’s 18th Ed. 2433 B. Gastric carcinoma A. Transient LES relaxations C. Scleroderma B. LES hypotension D. Diffuse esophageal spasm C. Anatomic distortion of esophagogastric junction In achalasia, Cholecystokinin (CCK) paradoxically causes contraction of LES (CCK test) because neurally transmitted inhibitory effect of CCK is absent & direct excitatory effect of CCK remains D. All of the above unopposed. 58 Which of the following may have a protective effect in GERD 52 In achalasia, which is the treatment of choice ? patients ? Harrison’s 18th Ed. 2432 Harrison’s 18th Ed. 2433 A. Pneumatic dilatation A. Zollinger-Ellison syndrome B. Esophageal resection with gastric pull-up B. Chronic H. pylori gastritis C. Laparoscopic Heller myotomy of LES C. Hiatus hernia D. Open surgical myotomy of LES D. Scleroderma Laparoscopic Heller myotomy is currently the procedure of choice. Chronic H. pylori gastritis may have a protective effect in GERD by inducing atrophic gastritis with concomitant hypoacidity. 53 Secondary achalasia may be caused by all except ? Harrison’s 17th Ed. 1849 59 Which out of the following conditions has associations with GERD ? A. Ulcerative colitis Harrison’s 18th Ed. 2434 B. Chagas’ disease A. Pulmonary fibrosis C. Eosinophilic gastroenteritis B. Chronic sinusitis D. Neurodegenerative disorders C. Cardiac arrhythmias Secondary achalasia may be caused by gastric carcinoma that infiltrates the esophagus, lymphoma, D. All of the above Chagas’ disease, certain viral infections, eosinophilic gastroenteritis & neurodegenerative disorders. Extraesophageal syndromes that have established association to GERD include chronic cough, 54 Which of the following is not a hypertensive motor disorder of laryngitis, asthma & dental erosions. Other conditions like pharyngitis, chronic bronchitis, pulmonary fibrosis, chronic sinusitis, cardiac arrhythmias, sleep apnea, and recurrent aspiration pneumonia esophagus ? have proposed associations with GERD. Harrison’s 18th Ed. 2432 A. Diffuse esophageal spasm (DES) 60 Multiple esophageal mucosal rings are characteristic of ? Harrison’s 18th Ed. 2434 B. Nutcracker esophagus A. Eosinophilic esophagitis 394 MCQ’s FOR MEDICAL PROFESSIONALS BY PROF. AJAY MATHUR Gastroenterology B. Radiation esophagitis 66 Methods for mucosal ablation in Barrett’s esophagus include C. Corrosive esophagitis all except ? N Engl J Med 2006;354:1403-9 D. Candida Esophagitis A. Electrocautery Multiple mucosal rings (feline esophagus) are characteristic of eosinophilic esophagitis. B. Mucosal stripping 61 Risk of development of esophageal adenocarcinoma in C. Argon-plasma-beam fulguration Barrett’s metaplasia is increased by ? D. Chemical fulguration Harrison’s 18th Ed. 2434 A. 5 fold 67 Methods for mucosal ablation in Barrett’s esophagus include all except ? B. 10 fold N Engl J Med 2006;354:1403-9 C. 15 fold A. Laser photothermal coagulation D. 20 fold B. Heater-probe ablation Risk of development of esophageal adenocarcinoma in Barrett’s metaplasia is increased by 20 fold. C. Hyperbaric oxygen ablation D. Cryotherapy 62 Esophageal mucosal biopsies should be taken at least ? Harrison’s 17th Ed. 1851 68 Photosensitizing agent used in photodynamic therapy for A. 2 cm above the LES Barrett’s esophagus is ? B. 5 cm above the LES N Engl J Med 2006;354:1403-9 C. 7 cm above the LES A. Trigen sodium D. 9 cm above the LES B. Porfimer sodium C. Sugran sodium Mucosal biopsies should be performed at least 5 cm above LES, as esophageal mucosal changes of chronic esophagitis are frequent in the most distal esophagus in otherwise normal individuals. D. Mofetil sodium 63 Which of the following about Bernstein test is false ? 69 Which of the following foods is “refluxogenic” ? Harrison’s 17th Ed. 1851 Harrison’s 18th Ed. 2434 A. Infusion of 0.1 N HCl & saline in esophagus A. Peppermint B. Useful in diagnosing Barrett’s esophagus that is not B. Tomato-based foods endoscopically obvious C. Alcohol C. In symptomatic esophagitis, infusion of acid, but not of saline, D. All of the above reproduces symptoms of heartburn Foods that reduce lower esophageal sphincter pressure are called “refluxogenic”. These include D. Infusion of acid in normal individuals produces no symptom fatty foods, alcohol, spearmint, peppermint, tomato-based foods, coffee and tea. Bernstein test involves infusion of solutions of 0.1 N HCl or normal saline in esophagus. In symptomatic esophagitis, infusion of acid, but not of saline, reproduces the symptoms of heartburn. 70 Absorption of which of the following may be compromised on Infusion of acid in normal individuals usually produces no symptoms. indefinite treatment with PPIs ? Harrison’s 18th Ed. 2435 64 Which of the following statements about Barrett’s esophagus A. Vitamin B12 is false ? Harrison’s 17th Ed. 1852, N Engl J Med 2006;354:1403-9 B. Calcium A. Barrett’s esophagus is an acquired condition C. Iron B. Metaplasia occurs from esophageal columnar to D. All of the above squamous epithelium With indefinite treatment with PPIs, Vitamin B12, calcium, and iron absorption may be compromised and susceptibility to enteric infections, particularly Clostridium difficile colitis increased. C. Complication of severe reflux esophagitis D. Risk factor for esophageal adenocarcinoma 71 Characteristic endoscopic finding of Eosinophilic Esophagitis is ? Metaplasia of esophageal squamous epithelium to columnar epithelium (Barrett’s esophagus) is a Harrison’s 18th Ed. 2435 complication of severe reflux esophagitis. Finding intestinal metaplasia with goblet cells in esophagus is diagnostic of Barrett’s esophagus. A. Multiple esophageal rings B. Linear furrows 65 Which of the following groups is at greatest risk of Barrett’s C. Punctate exudates metaplasia progressing to adenocarcinoma ? Harrison’s 18th Ed. 2434 D. All of the above A. Obese black males in fifth decade of life The characteristic endoscopic findings of Eosinophilic Esophagitis (EoE) include multiple esophageal rings, linear furrows, and punctate exudates. B. Obese white males in fifth decade of life C. Obese black males in sixth decade of life 72 For diagnosis of eosinophilic esophagitis, eosinophils in D. Obese white males in sixth decade of life esophagal mucosa per high-power field should be ? Harrison’s 18th Ed. 2436 Barrett’s metaplasia can progress to adenocarcinoma and the group at greatest risk is obese white males in their sixth decade of life. A. 5 or more 395 Gastroenterology MCQ’s MCQ’s FOR FORPROFESSIONALS MEDICAL MEDICAL PROFESSIONALS BY PROF. AJAY MATHUR Cardiology 395 B. 10 or more 79 Which of the following infections occur only in C. 15 or more immunocompromised patients ? Harrison’s 18th Ed. 2436 D. 20 or more A. Herpes simplex virus (HSV) esophagitis The diagnosis of eosinophilic esophagitis is made histologically, with 15 or more eosinophils per B. Varicella-zoster virus (VZV) esophagitis high-power field in the esophagal mucosa. Normal esophagus contains almost no eosinophils. C. Cytomegalovirus (CMV) esophagitis 73 Preferred treatment of eosinophilic esophagitis is ? D. Candida esophagitis Harrison’s 18th Ed. 2436, Harrison’s 17th Ed. 1854 Cytomegalovirus (CMV) infections occur only in immunocompromised patients, particularly transplant A. H2 receptor blocking agents recipients. B. Nifedipine C. Swallowed fluticasone propionate 80 Serpiginous ulcers in an otherwise normal esophageal mucosa is a feature of ? D. Isosorbide dinitrate Harrison’s 18th Ed. 2436 Treatment of eosinophilic esophagitis consists of a 12-week course of swallowed fluticasone A. Herpes simplex virus (HSV) esophagitis propionate using a metered dose inhaler. Anti human interleukin 5, systemic steroids, montelukast or cromolyn may be useful. B. Varicella-zoster virus (VZV) esophagitis C. Cytomegalovirus (CMV) esophagitis 74 Odynophagia is a characteristic symptom of ? Harrison’s 18th Ed. 2436 D. Candida Esophagitis A. Pill-induced esophagitis Endoscopically, CMV lesions appear as serpiginous ulcers in an otherwise normal mucosa, particularly in the distal esophagus. B. Reflux esophagitis C. Esophageal perforation 81 Ganciclovir is the treatment of choice for ? Harrison’s 18th Ed. 2436 D. Infectious esophagitis A. Herpes simplex virus (HSV) esophagitis Regardless of the infectious agent, odynophagia is a characteristic symptom of infectious esophagitis. B. Varicella-zoster virus (VZV) esophagitis 75 Which of the following is characteristic of Candida C. Cytomegalovirus (CMV) esophagitis esophagitis ? D. Candida Esophagitis Harrison’s 18th Ed. 2436 G a n c i c l o v i r, 5 m g / k g B D I V, i s t h e t r e a t m e n t o f c h o i c e f o r C y t o m e g a l o v i r u s ( C M V ) A. Bleeding esophagitis. B. White plaques with friability C. Perforation 82 Boerhaave’s syndrome refers to ? Harrison’s 18th Ed. 2436 D. Stricture A. Esophagial damage due to instrumentation Candida esophagitis has a characteristic appearance of white plaques with friability. Rarely, Candida esophagitis is complicated by bleeding, perforation, stricture, or systemic invasion. B. Esophagial damage due to vomiting or retching C. Esophagial damage due to external trauma 76 Candida esophagitis can be treated with ? D. None of the above Harrison’s 18th Ed. 2436 A. Fluconazole Boerhaave’s syndrome or spontaneous rupture refers to esophageal rupture caused by increased intraesophageal pressure associated with forceful vomiting or retching. B. Itraconazole C. Amphotericin B 83 Instrumental perforation usually occurs in ? Harrison’s 18th Ed. 2436 D. Any of the above A. Upper esophagus 77 “Volcano-like” oesophageal ulcerations are seen in ? B. Mid esophagus Harrison’s 17th Ed. 1852 C. Lower esophagus A. Candidiasis D. Any of the above B. HSV Instrumental perforation occurs in pharynx or lower esophagus, just above diaphragm in posterolateral C. CMV wall. D. Corrosive poisoning 84 Mallory-Weiss Syndrome can be caused by ? In HSV infection of esophagus, endoscopy shows vesicles & small, discrete, punched-out (“volcano- Harrison’s 18th Ed. 2436 like”) superficial ulcerations with or without a fibrinous exudate. A. Vomiting 78 Herpes simplex virus (HSV) esophagitis is treated with ? B. Retching Harrison’s 18th Ed. 2436 C. Vigorous coughing A. Acyclovir D. All of the above B. Foscarnet Vomiting, retching, or vigorous coughing can cause a nontransmural tear at the gastroesophageal C. Famciclovir junction that is Mallory-Weiss Syndrome. D. All of the above 396 MCQ’s FOR MEDICAL PROFESSIONALS BY PROF. AJAY MATHUR Gastroenterology 85 Which of the following is false about Mallory-Weiss Syndrome ? Harrison’s 18th Ed. 2436-7 293 - Peptic Ulcer Disease A. Usually involves esophageal mucosa B. Upper gastrointestinal bleeding 91 Rumination is best related to ? Harrison’s 18th Ed. 301 C. In most patients bleeding ceases spontaneously A. Nausea D. Respond to vasopressin therapy B. Vomiting Mallory-Weiss Syndrome refers to mucosal tear involving gastric mucosa near the squamocolumnar mucosal junction caused by vomiting, retching or vigorous coughing. Upper gastrointestinal bleeding C. Regurgitation may be severe but in mostly bleeding ceases spontaneously. Respond to vasopressin therapy or angiographic embolization. D. All of the above Rumination refers to repeated voluntary regurgitation of stomach contents, which may be rechewed 86 Treatment with which of the following can reduce esophagitis and reswallowed. during radiation treatment ? Harrison’s 17th Ed. 1853 92 Under normal conditions, frequency of slow wave cycles in A. Cimetidine the stomach is ? Harrison’s 18th Ed. 301 B. Indomethacin A. 3 cycles / minute C. Allopurinol B. 6 cycles / minute D. Sucralfate C. 9 cycles / minute indomethacin treatment may reduce radiation damage of esophagus. D. 12 cycles / minute 87 Patients with alkaline esophagitis are treated with ? 93 Under normal conditions, frequency of slow wave cycles in the Harrison’s 17th Ed. 1852 duodenum is ? A. Cholestyramine Harrison’s 18th Ed. 301 B. Aluminum hydroxide A. 4 cycles / minute C. Sucralfate B. 8 cycles / minute D. All of the above C. 11 cycles / minute Treatment of alkaline esophagitis includes neutralization of bile salts with cholestyramine, aluminum D. 15 cycles / minute hydroxide, or sucralfate. Under normal conditions, distally migrating gut contractions, the slow wave, occur at 3 cycles/ minute in the stomach and 11 cycles/minute in duodenum. 88 Pill-induced esophagitis can be caused by all except ? Harrison’s 18th Ed. 2437 94 Which of the following is termed the chemoreceptor trigger A. Alendronate zone ? B. Ferrous sulfate Harrison’s 18th Ed. 301 C. Doxycycline A. Area postrema D. Medroxyprogesterone B. Nucleus tractus solitarius C. Dorsal vagal nuclei Pill-induced esophagitis mostly occurs with ingestion of doxycycline, tetracycline, quinidine, phenytoin, potassium chloride, ferrous sulfate, nonsteroidal anti-inflammatory drugs (NSAIDs), and D. Phrenic nuclei bisphosphonates. Area postrema, a medullary nucleus, responds to bloodborne emetic stimuli and is termed the 89 Esophageal lesion in systemic sclerosis consist of ? chemoreceptor trigger zone. Harrison’s 18th Ed. 2437 95 Which of the following act on the area postrema ? A. Atrophy of smooth muscle Harrison’s 18th Ed. 301 B. Weakness in lower two-thirds of esophagus A. Uremia C. Incompetence of LES B. Hypoxia D. All of the above C. Ketoacidosis Esophageal lesions in systemic sclerosis consist of atrophy of smooth muscle, manifested by D. All of the above weakness in lower two-thirds of esophagus & incompetence of LES. Uremia, hypoxia, and ketoacidosis act on the area postrema and are emetogenic. 90 “Strawberry gums” is a pathognomonic sign of ? Harrison’s 17th Ed. Chapter 32 96 Neurotransmitter that mediate induction of vomiting in A. Wegener’s granulomatosis labyrinthine disorders is ? Harrison’s 18th Ed. 301 B. Acute myelomonocytic leukemia A. Muscarinic M 1 receptors C. Down’s syndrome B. Histaminergic H2 receptors D. Diabetes mellitus C. Serotonin 5-HT3 receptors Pathognomonic sign of Wegener’s granulomatosis is a red-purplish, granular gingivitis called strawberry gums. D. Dopamine D2 receptors 397 Gastroenterology MCQ’s MCQ’s FOR FORPROFESSIONALS MEDICAL MEDICAL PROFESSIONALS BY PROF. AJAY MATHUR Cardiology 397 Neurotransmitters that mediate induction of vomiting in labyrinthine disorders is vestibular C. Mirtazapine muscarinic M1 and histaminergic H1 receptors, whereas vagal afferent stimuli activate serotonin 5-HT3 receptors. Area postrema is richly served by nerves acting on 5-HT3, M1, H1, and dopamine D. Dimenhydrinate D 2 subtypes. Antiemetic agent ondansetron and granisetron are Serotonin5-HT 3 antagonists. Meclizine & dimenhydrinate are antihistamines. Scopolamine is an anticholinergic drug. 97 Gastroparesis occurs after which of the following ? Harrison’s 18th Ed. 301 103 Which of the following is a combined 5-HT4 agonist and D2 A. Pancreatic adenocarcinoma antagonist ? Harrison’s 18th Ed. 304 B. Mesenteric vascular insufficiency A. Metoclopramide C. Scleroderma B. Domperidone D. All of the above C. Erythromycin Gastroparesis or a delay in gastric emptying of food occurs after vagotomy, with pancreatic adenocarcinoma, with mesenteric vascular insufficiency, or in systemic diseases such as diabetes, D. Octreotide scleroderma, and amyloidosis. Anorexia nervosa, bulimia nervosa, anxiety, and depression may cause significant nausea that may be associated with delayed gastric emptying. Metoclopramide is a combined 5-HT 4 agonist & D2 antagonist. Domperidone is a D 2 antagonist. Erythromycin increases gastroduodenal motility by action on receptors for motilin. Somatostatin analogue octreotide induces propagative small intestinal motor complexes. 98 Gastroparesis does not occur with which of the following ? Harrison’s 18th Ed. 301 104 Which of the following is a Neurokinin NK1 antagonist ? A. Diabetes Harrison’s 18th Ed. 304 B. Amyloidosis A. Aprepitant C. Scleroderma B. Kevetiracetam D. Cyclic vomiting syndrome C. Cyproheptadine Cyclic vomiting syndrome produces periodic discrete episodes of relentless nausea & vomiting and D. Sumatriptan has a strong association with migraine headaches. Cyclic vomiting is most common in children, Aprepitant is a neurokinin NK1 antagonist that has antiemetic & antinausea effects during acute and although adult cases occur in association with rapid gastric emptying & with chronic cannabis use. delayed periods after chemotherapy. 99 Which of the following drugs is a highly emetogenic agent ? 105 Gastric pits of stomach branch into ? Harrison’s 18th Ed. 302 Harrison’s 18th Ed. 2438 A. Digoxin A. 1 or 2 gastric glands B. Oral contraceptives B. 3 or 4 gastric glands C. Cisplatin C. 4 or 5 gastric glands D. Erythromycin D. 6 or 7 gastric glands Acute emesis from intensely emetogenic cisplatin is mediated by 5-HT3 pathways. Microscopic gastric pits (foveolus) of gastric epithelial lining branch into four or five gastric glands made up of highly specialized epithelial cells. 100 What proportion of pregnant women experience nausea in the first trimester ? 106 Which of the following cells is found deepest in the oxyntic Harrison’s 18th Ed. 302 gastric gland ? A. 25 % Harrison’s 18th Ed. 2438 Figure 293-1 A. Mucous neck cells B. 50 % B. Parietal cells C. 70 % C. Endocrine cells D. 90 % D. Chief cells Nausea affects 70% of pregnant women in first trimester. 107 Parietal cell is also known as ? 101 Relief of abdominal pain by emesis is characteristic of ? Harrison’s 18th Ed. 2438 Harrison’s 18th Ed. 302 A. Mucous cell A. Pancreatitis B. Oxyntic cell B. Zenker’s diverticulum C. Endocrine cell C. Gastric obstruction D. Enterochromaffin-like (ECL) cell D. Intestinal obstruction Parietal cell is also known as oxyntic cell and is found in neck, or isthmus, or in oxyntic gland. Relief of abdominal pain by emesis characterizes intestinal obstruction, whereas vomiting has no effect on pancreatitis or cholecystitis pain. 108 Peptic ulcer disease (PUD) occurs due to constant attack on gastroduodenal mucosa by all noxious agents except ? 102 Which of the following antiemetic agent is a Serotonin5-HT3 Harrison’s 18th Ed. 2438 antagonist ? A. Acid Harrison’s 18th Ed. 302 B. Pepsin A. Meclizine C. Bile acids B. Granisetron D. Salivary amylase 398 MCQ’s FOR MEDICAL PROFESSIONALS BY PROF. AJAY MATHUR Gastroenterology 109 PUD occurs due to constant attack on gastroduodenal mucosa 115 Restitution of gastric mucosa means ? by all noxious agents except ? Harrison’s 18th Ed. 2439 Harrison’s 18th Ed. 2438 A. Proliferation of damaged gastric mucosa A. Pancreatic enzymes B. Regeneration of damaged gastric mucosa B. Alcohol C. Migration of normal gastric epithelium to damaged areas C. Virus D. All of the above D. Bacteria Restitution is migration of gastric epithelial cells bordering site of injury when preepithelial barrier is PUD occurs due to constant attack on gastroduodenal mucosa by endogenous noxious agents like breached. acid, pepsin, bile acids, pancreatic enzymes or exogenous substances like medications, alcohol & bacteria. 116 Which of the following statements about restitution is false ? Harrison’s 18th Ed. 2439 110 Which of the following is an element of gastric mucosal defense A. Occurs independent of cell division system ? Harrison’s 18th Ed. 2438 B. Requires uninterrupted blood flow A. Preepithelial C. Requires an alkaline pH in surroundings B. Epithelial D. None of the above C. Subepithelial Restitution occurs independent of cell division and requires uninterrupted blood flow and an alkaline pH in the surrounding environment. D. All of the above Gastric mucosal defense is a 3-level barrier composed of preepithelial, epithelial & subepithelial elements. 117 Restitution of gastric mucosa is modulated by ? Harrison’s 18th Ed. 2439 111 First line of defense of gastric epithelium is ? A. Epidermal growth factor (EGF) Harrison’s 18th Ed. 2438 B. Transforming growth factor alpha (TGF-) A. Mucus layer C. Fibroblast growth factor (FGF) B. Bicarbonate-phospholipid layer D. All of the above C. Phospholipid layer Growth factors like epidermal growth factor (EGF), transforming growth factor (TGF) alpha and D. Mucus-bicarbonate-phospholipid layer basic fibroblast growth factor (FGF) modulate restitution. The first line of defense of gastric epithelium is a mucus-bicarbonate-phospholipid layer that serves as a physicochemical barrier. Surface epithelial cells provide the next line of defense. 118 Gastric epithelial cell regeneration is regulated by ? Harrison’s 18th Ed. 2439 112 Surface epithelial cells generate which of the following ? A. Prostaglandins Harrison’s 18th Ed. 2439 B. Epidermal growth factor (EGF) A. Heat shock proteins C. Transforming growth factor-alpha (TGF-) B. Trefoil factor family (TFF) peptides D. All of the above C. Cathelicidins Larger defects not effectively repaired by restitution require cell proliferation. Epithelial cell regeneration D. All of the above is regulated by prostaglandins, EGF & TGF-alpha. Surface epithelial cells generate heat shock proteins that prevent protein denaturation, trefoil factor family peptides and cathelicidins, which play a role in surface cell protection and 119 Bicarbonate secretion in stomach is stimulated by all except ? regeneration. Harrison’s 16th Ed. 1747 A. Calcium 113 Trefoil factor family (TFF) peptides were formerly known as ? A. C-domain peptides B. Prostaglandins B. D-domain peptides C. VIP C. G-domain peptides D. Luminal acidification D. P-domain peptides 120 Which of the following plays a central role in gastric epithelial Trefoil factor family (TFF) peptides were formerly called P-domain peptides promote wound healing defense / repair ? in the gut through epithelial restitution. TFF peptides are found in mucous membranes of stomach, Harrison’s 18th Ed. 2440 conjunctiva, Brunner’s glands, intestine, salivary glands, uterus, and respiratory tract. A. Mucosal bicarbonate 114 Cathelicidin is best related to ? B. Mucus A. P-domain peptides C. Prostaglandins B. Antimicrobial peptides (AMPs) D. Growth factor C. Angiogenesis 121 Prostaglandins are important in gastric epithelial defense / D. All of the above repair due to all except ? Harrison’s 18th Ed. 2440 Antimicrobial peptides (AMPs) have the capacity to rapidly inactivate infectious agents. The two major AMP families in mammals are the defensins and cathelicidin peptides. A. Release of mucosal bicarbonate and mucus B. Inhibition of parietal cell secretion 399 Gastroenterology MCQ’s MCQ’s FOR FORPROFESSIONALS MEDICAL MEDICAL PROFESSIONALS BY PROF. AJAY MATHUR Cardiology 399 C. Maintaining mucosal lymphatic flow 127 Somatostatin acts by ? Harrison’s 18th Ed. 2440 D. Epithelial cell restitution A. Direct inhibition of parietal cells Pg’s in gastric mucosa play a central role in gastric epithelial defense/repair. Pg regulate release of mucosal bicarbonate & mucus, inhibit parietal cell secretion & maintain mucosal blood flow & B. Decreased histamine release from ECL cells epithelial cell restitution. C. Decreased gastrin release from G cells 122 Key enzyme that controls the rate-limiting step in prostaglandin D. All of the above synthesis is ? Somatostatin is released from endocrine cells in gastric mucosa (D cells) in response to HCl. Harrison’s 18th Ed. 2440 Somatostatin inhibits acid production by direct action on parietal cell, decreased histamine release from enterochromaffin-like (ECL) cells and gastrin release from G cells. A. Thromboxane A2 (TXA2) B. Phospholipase A2 128 Which of the following about parietal cells is false ? C. Cyclooxygenase (COX) Harrison’s 18th Ed. 2440 D. Prostacyclin (PGI2) A. Located in oxyntic gland Key enzyme that controls the rate-limiting step in prostaglandin synthesis is cyclooxygenase B. Does not secrete intrinsic factor (COX). COX is present in two isoforms - COX-1 and COX-2). C. Express receptors for histamine, gastrin & acetylcholine 123 COX-1 is expressed in all except ? D. Express receptors for ligands that inhibit acid production Harrison’s 18th Ed. 2440 Acid-secreting parietal cell also secretes intrinsic factor (IF). A. Stomach 129 Gastric acid production is inhibited by ? B. Platelets Harrison’s 18th Ed. 2440 C. Kidneys A. Prostaglandins D. Liver B. Somatostatin COX-1 is expressed in stomach, platelets, kidneys, and endothelial cells. Gastro-intestinal mucosal C. EGF ulceration & renal dysfunction is due to inhibition of COX-1. D. All of the above 124 COX-2 is expressed in all except ? Gastric acid production is inhibited by prostaglandins, somatostatin & epidermal growth factor (EGF). Harrison’s 18th Ed. 2440 A. Macrophages 130 Which of the following statements about enzyme H+, K+-ATPase B. Leukocytes is false ? Harrison’s 18th Ed. 2441 C. Platelets A. Responsible for generating large concentration of H+ D. Synovial cells B. Membrane-bound protein consisting of  & subunits COX-2, induced by inflammatory stimuli is expressed in macrophages, leukocytes, fibroblasts & synovial cells. C. Active catalytic site is found within  subunit D. Active catalytic site is found within  subunit 125 Which of the following about acid production in stomach is Active catalytic site is found within  subunit. false ? Harrison’s 18th Ed. 2440 131 Which of the following statements about enzyme H+, K+-ATPase A. Basal acid production occurs in a circadian pattern is false ? B. Cholinergic & histaminergic input are the principal Harrison’s 18th Ed. 2441 contributors to basal acid secretion A. Transfers H+ ions from parietal cell cytoplasm to secretory C. Stimulated gastric acid secretion occurs in cephalic, canaliculi in exchange for K+ gastric & intestinal phases B. Located within secretory canaliculus & in nonsecretory D. Cephalic phase stimulates gastric secretion via hormones cytoplasmic tubulovesicles Cephalic phase stimulates gastric secretion via vagus nerve. C. Tubulovesicles are impermeable to K+ D. At rest, 50% of pumps are within secretory canaliculus 126 Which of the following about acid production in stomach is Distribution of proton pumps between nonsecretory vesicles & secretory canaliculus varies according false ? to parietal cell activity. They are recycled back to inactive state in cytoplasmic vesicles once parietal Harrison’s 18th Ed. 2440 cell activation ceases. A. Cephalic phase stimulates gastric secretion via vagus 132 Which of the following about chief cell is false ? B. Gastric phase is activated when food enters stomach Harrison’s 18th Ed. 2441 C. Amino acids stimulate vagus to release gastrin A. Found primarily in gastric fundus D. Intestinal phase is mediated by luminal distention B. Synthesize & secrete pepsinogen Gastric phase is activated once food enters stomach. Amino acids and amines directly stimulate G C. Acid environment converts pepsinogen to pepsin cell to release gastrin, which in turn activate parietal cell. D. Pepsin activity is irreversibly inactivated & denatured at a pH of >=4 400 MCQ’s FOR MEDICAL PROFESSIONALS BY PROF. AJAY MATHUR Gastroenterology Pepsin activity is significantly diminished at a pH of 4 and irreversibly inactivated and denatured at D. Type IV a pH of 7. Type III GU’s occur within 3 cm of the pylorus and are commonly accompanied by duodenal ulcers and normal or high gastric acid production. 133 All of the following about gastric ulcer are true except ? Harrison’s 18th Ed. 2441 139 Helicobacter pylori was discovered by ? A. A break in mucosal surface >5 mm in size N Engl J Med 2005;35:2421 B. Depth to submucosa A. Marshall B & Warren R C. More than half of gastric ulcers occur in males B. Julie Parsonnet & Jennings R D. Peak incidence is in the 4th decade of life C. Banatvala N & Mayo K Gastric & duodenal ulcers are defined as breaks in mucosal surface >5 mm in size with depth to D. Deeks JJ & Feldman RA submucosa. Peak incidence of gastric ulcers is in the sixth decade & more than half of GUs occur in males. 140 Gastric infection with H. pylori can lead to ? Harrison’s 18th Ed. 2442 134 Which of the following statements about GU is false ? A. Peptic ulcer disease (PUD) Harrison’s 18th Ed. 2441 B. Gastric mucosal-associated lymphoid tissue lymphoma A. Can represent malignancy C. Gastric adenocarcinoma B. Benign GUs are found distal to junction between antrum & acid secretory mucosa D. All of the above C. Benign GUs are common in gastric fundus H. pylori plays a role in the development of majority of PUD, gastric mucosal-associated lymphoid tissue (MALT) lymphoma and gastric adenocarcinoma. D. Gastric acid output is normal or decreased in GU Benign GUs are rare in the gastric fundus. 141 Which of the following statements about H. pylori is false ? Harrison’s 18th Ed. 2442 135 ~90% of DU’s occur within what distance from pylorus ? A. It is a gram-positive microaerophilic rod Harrison’s 18th Ed. 2441 B. Found between mucous layer & gastric epithelium A. 3 cm C. Normally, it does not invade gastric epithelial cells B. 5 cm D. S-shaped & contains multiple sheathed flagella C. 7 cm H. pylori is a gram-negative microaerophilic rod found most commonly between the mucous layer D. 9 cm and the gastric epithelium. ~90% of DU’s are located within 3 cm of the pylorus. 142 Which of the following statements about H. pylori is false ? Harrison’s 18th Ed. 2442 136 Which of the following statements about DU is false ? Harrison’s 18th Ed. 2441 A. H. pylori should be eradicated in documented PUD A. Most often occur in first part of duodenum B. No single agent is effective in eradicating H. pylori B. Usually <=1 cm in diameter C. Rifabutin used to treat resistant strains of H. pylori C. Malignant DUs are extremely rare D. H. pylori is implicated in pathogenesis of acute pancreatitis D. Base of ulcer consists of neutrophilic necrosis with 143 pH-gated urea channel in H. pylori bacterium is called ? surrounding fibrosis N Engl J Med 2002:347,1175 Base of DU consists of a zone of eosinophilic necrosis with surrounding fibrosis. Malignant DUs are A. UreI very rare. B. UreJ 137 Type IV Gastric Ulcers are found in ? C. UreK Harrison’s 18th Ed. 2442 D. UreL A. Cardia B. Gastric body 144 Which of the following statements about H. pylori is false ? Harrison’s 18th Ed. 2442 C. Gastric antrum A. Its genome contains 1.65 million base pairs D. Pylorus B. May transform into coccoid dormant form Gastric ulcers are classified based on their location. Type I occur in gastric body, type II occur in antrum, type III occur within 3 cm of pylorus and type IV are found in the cardia. C. Produces urease to convert urea to NH3 and water D. Single strain of H. pylori exist 138 Which of the following Gastric Ulcers has high gastric acid Multiple strains of H. pylori exist. Different diseases related to H. pylori infection can be attributed production ? to different strains with distinct pathogenic features. Harrison’s 18th Ed. 2442 A. Type I 145 Which of the following statements about H. pylori is false ? N Engl J Med 2002:347,1175, Harrison’s 18th Ed. 2442 B. Type II A. 80% population by age 20 years is infected in developing C. Type III countries 401 Gastroenterology MCQ’s MCQ’s FOR FORPROFESSIONALS MEDICAL MEDICAL PROFESSIONALS BY PROF. AJAY MATHUR Cardiology 401 B. Transmission occurs through faeco-oral route 152 Chronic H. pylori infection may lead to ? Harrison’s 18th Ed. 2443 Figure 293-8 C. Infection is associated with chronic active gastritis A. Antral predominant gastritis D. BabA is vital for entry into gastric epithelial cell B. Nonatrophic pangastritis 146 Which of the following statements about H. pylori is false ? C. Corpus predominant atrophic gastritis Harrison’s 18th Ed. 2443 D. All of the above A. Express vacuolating cytotoxin VacA B. Cag A and pic B are virulence factors 153 Asymptomatic H. pylori infection may be a consequence of ? Harrison’s 18th Ed. 2443 Figure 293-8 C. Its LPS has high immunologic activity A. Antral predominant gastritis D. Neutrophil response is strong in acute & chronic H. pylori infection B. Nonatrophic pangastritis C. Corpus predominant atrophic gastritis 147 Which out of the following factors has a central role in H. pylori D. All of the above infection ? N Engl J Med 2002:347,1175 154 Which of the following consequences of chronic H. pylori A. Interleukin-1 infection leads to gastric cancer ? B. Interleukin-2 Harrison’s 18th Ed. 2443 Figure 293-8 C. Interleukin-6 A. Antral predominant gastritis D. Interleukin-8 B. Nonatrophic pangastritis C. Corpus predominant atrophic gastritis 148 Which of the following statements about H. pylori is false ? D. All of the above N Engl J Med 2002:347, 1175 A. Most pathogenic strains contain VacA pathogenicity island Chronic H. pylori infection leading to corpus predominant atrophic gastritis going on to intestinal metaplasia is an important predisposing factor for gastric cancer. B. 5 of its genes are similar to Agrobacterium tumefaciens C. Pathogenicity island proteins are involved in interleukin-8 155 Which of the following consequences of chronic H. pylori production by gastric epithelial cells infection leads to DU ? Harrison’s 18th Ed. 2443 Figure 293-8 D. Pathogenicity island proteins are involved in translocation of A. Antral predominant gastritis CagA from bacterium into host cell B. Nonatrophic pangastritis 149 Adhesins for H. pylori include all except ? C. Corpus predominant atrophic gastritis N Engl J Med 2002:347,1175, Harrison’s 18th Ed. 2442 D. All of the above A. BabA Presence of antral-predominant gastritis is associated with DU formation. B. AlpA C. AlpB 156 Which of the following consequences of chronic H. pylori D. HopY infection leads to MALT lymphoma ? Harrison’s 18th Ed. 2443 Figure 293-8 150 Which of the following statements about H. pylori is false ? A. Antral predominant gastritis Harrison’s 18th Ed. 2442 B. Nonatrophic pangastritis A. H. pylori is usually acquired in childhood C. Corpus predominant atrophic gastritis B. Acute infection causes transient hypochlorhydria D. All of the above C. 80-90% with chronic gastritis will never have symptoms Chronic infection with H. pylori is associated with development of a low-grade B cell lymphoma, D. After eradication, reinfection rates are high gastric MALT lymphoma. (MALT refers to mucosal-associated lymphoid tissue). Reinfection after successful eradication of H. pylori is rare in US. If recurrent infection occurs within first 6 months after completing therapy, most likely explanation is recrudescence and not reinfection. 157 Which of the following about NSAIDs induced disease is false ? Harrison’s 18th Ed. 2444 Table 293-5 151 Which of the following enzymes is not related to H. pylori– A. NSAID-related GUs not accompanied by chronic active gastritis induced gastrointestinal disease ? B. No dose of NSAID is completely safe Harrison’s 18th Ed. 2442 Figure 293-6 C. Misoprostol is useful for active treatment A. Urease D. Selective COX-2 inhibitor useful in prophylactic therapy B. Vac A C. GAD Primary prevention of NSAID-induced ulceration can be accomplished by misoprostol or a PPI. D. Cag A 158 Risk factors that increase morbidity and mortality related to Bacterial enzymes that cause H. pylori - induced gastrointestinal disease are Urease, Vac A & Cag NSAID usage are all except ? A. Harrison’s 18th Ed. 2444 A. Advanced age 402 MCQ’s FOR MEDICAL PROFESSIONALS BY PROF. AJAY MATHUR Gastroenterology B. History of PUD B. Gastrointestinal bleeding C. Concomitant use of iron C. Perforation D. Multisystem disease D. Malignancy Gastric outlet obstruction is the least common ulcer-related complication. 159 Risk factors that increase morbidity and mortality related to NSAID usage are all except ? 165 GUs tend to perforate into the ? Harrison’s 18th Ed. 2444 Harrison’s 18th Ed. 2445 A. High dose NSAIDs A. Spleen B. Multiple NSAIDs B. Pancreas C. Concomitant use of anticoagulants C. Left hepatic lobe D. Spicy food D. Left kidney Established risk factors for serious gastrointestinal complications from NSAIDs include advanced age, history of ulcer, concomitant use of glucocorticoids, high-dose NSAIDs, multiple NSAIDs, GUs tend to penetrate into the left hepatic lobe while DUs tend to penetrate posteriorly into concomitant use of anticoagulants, and serious or multisystem disease. Possible risk factors pancreas. include concomitant infection with H. pylori, cigarette smoking, and alcohol consumption. 166 Non ulcer dyspepsia is typified by ? 160 Disorders that are associated with PUD include all except ? Harrison’s 18th Ed. 2445 Harrison’s 18th Ed. 2444 A. Heartburn A. Systemic mastocytosis B. Upper abdominal pain B. Chronic pulmonary disease C. Abdominal distension C. Chronic renal failure D. Loss of appetite D. Acute pancreatitis NUD (functional dyspepsia / essential dyspepsia) is typified by upper abdominal pain without presence of ulcer. 161 Disorders that are associated with PUD include all except ? Harrison’s 18th Ed. 2444 167 Tests for diagnosing H. pylori include ? A. Cirrhosis Harrison’s 18th Ed. 2446 B. Nephrolithiasis A. Biopsy urease test C. Hyperthyroidism B. Fecal H. pylori antigen test D. 1 antitrypsin deficiency C. 13 C- or 14C-urea breath test Chronic disorders that have a strong association with PUD are systemic mastocytosis, chronic D. All of the above pulmonary disease, chronic renal failure, cirrhosis, nephrolithiasis, and alpha1-antitrypsin deficiency. Those with a possible association are hyperparathyroidism, coronary artery disease, polycythemia Tests for diagnosing H. pylori include serologic testing, 13C- or 14C-urea breath test& fecal H. pylori vera, and chronic pancreatitis. (Hp) antigen test. urinary Hp antigen test & monoclonal antibody stool antigen test appear promising. 162 The typical pain pattern in DU occurs ? 168 Milk-alkali syndrome includes all except ? Harrison’s 18th Ed. 2445 Harrison’s 18th Ed. 2447 A. 15 minutes to 1 hour after a meal A. Hypercalcemia B. 30 minutes to 2 hour after a meal B. Hypophosphatemia C. 60 minutes to 3 hour after a meal C. Renal calcinosis D. 90 minutes to 3 hour after a meal D. Progression to renal insufficiency Typical pain pattern in DU occurs 90 minutes to 3 hours after a meal and is frequently relieved by Milk-alkali syndrome comprises of hypercalcemia, hyperphosphatemia, possible renal calcinosis & antacids or food. Pain that awakes the patient from sleep (between midnight and 3 AM) is the most progression to renal insufficiency. discriminating symptom. In GU, nausea & weight loss occur more commonly & discomfort may be precipitated by food. 169 Structure of H2 receptor antagonists share homology with ? Harrison’s 18th Ed. 2447 163 Most common complication observed in PUD is ? Harrison’s 18th Ed. 2445 A. Pepsin A. Gastrointestinal bleeding B. Secretin B. Perforation C. Gastrin C. Gastric Outlet Obstruction D. Histamine D. Malignancy H2 receptor antagonists (cimetidine, ranitidine, famotidine & nizatidine) have structural homology with histamine. Gastrointestinal bleeding is the most common complication observed in PUD followed by perforation and gastric outlet obstruction. 170 Proton Pump Inhibitors may interfere with absorption of ? Harrison’s 18th Ed. 2447 164 Out of the following, which is the least common ulcer-related complication ? A. Ketoconazole Harrison’s 18th Ed. 2445 B. Iron A. Gastric outlet obstruction C. Digoxin Lansoprazole D. Sucralfate 182 A DU is considered refractory if it fails to heal after how many C. None of the above A. metronidazole & tetracycline. Urinary Hp antigen test Harrison’s 18th Ed. B. 16 weeks B. Rabeprazole 180 Test of choice for documenting eradication of H. 8 weeks Harrison’s 18th Ed. AJAY MATHUR Cardiology 403 D. Quadruple Therapy with a higher incidence of ? Harrison’s 18th Ed. ampicillin. 4 weeks (BSS) include black stools. Ischemia Combination therapy for H. Colloidal bismuth subcitrate (CBS) weeks of therapy ? D. A. All of the above B. Misoprostol 173 PPI containing an imidazopyridine ring instead of a benzimidazole ring is ? D. 2449 The test of choice for documenting eradication is the urea breath test (UBT). 2451 C. 2450 PPIs may interfere with absorption of ketoconazole. 2452 D. A. A. H2 receptor antagonist Long-term acid suppression with PPIs is associated with higher incidence of community-acquired pneumonia. pylori infection ? Hepatic cytochrome P450 can be inhibited by omeprazole & lansoprazole and not by Rabeprazole. 2448 Only PPIs can heal GUs or DUs. 14 days 183 Etiologies of refractory ulcers (GU / DU) include all except ? C. 21 days Harrison’s 18th Ed. Fecal H. A. C. C. pylori was bismuth. 403 Gastroenterology MCQ’s MCQ’s FOR FORPROFESSIONALS MEDICAL MEDICAL PROFESSIONALS BY PROF. 4 weeks 175 Black stools & darkening of tongue are adverse effects of ? B. 2449 C. Esomeprazole B. 2452 Adverse effects with short-term usage of Colloidal bismuth subcitrate (CBS) and bismuth subsalicylate A. Insoluble in water weeks of therapy ? C. 28 days A. 16 weeks Harrison’s 18th Ed. 2449 C. independent of C. Sequential therapy A. Bacterial meningitis D. Biopsy urease test D. Sucralfate Harrison’s 18th Ed. Pantoprazole D. 7 days refractory. B. 2447 C. Lansoprazole + clarithromycin + amoxicillin B. pylori infection should be given C.or 14C-urea breath test 174 Which of the following is false about Sucralfate ? D. To be avoided in chronic renal insufficiency Harrison’s 18th Ed. Gall stones A. 13 C. Bismuth + metronidazole + tetracycline Harrison’s 18th Ed. B. B. Harrison’s 18th Ed. All of the above 177 First triple regimen therapy against H. iron & digoxin. Pantoprazole Harrison’s 18th Ed. pylori is ? B. Amoxicillin + omeprazole + metronidazole 171 Hepatic cytochrome P450 can be inhibited by ? B. PPIs C. 12 weeks for a period of ? D. Complex sucrose salt 181 A GU is considered refractory if it fails to heal after how many B. Tenatoprazole A. pylori antigen test Tenatoprazole is a PPI containing an imidazopyridine ring instead of a benzimidazole ring. Crohn’s disease . independent of whether NSAIDs are discontinued. 2450 D. Proton Pump Inhibitors (PPI) Harrison’s 18th Ed. 2452 D. 2450 pantoprazole and esomeprazole. pylori infection for 14 days provides the greatest efficacy. Rabeprazole First triple regimen therapy against H. Community-acquired pneumonia 179 Which of the following can heal GUs or DUs. Misoprostol D. 2449 GU that fails to heal >12 weeks & a DU that does not heal >8 weeks of therapy is considered A. 12 weeks A. Esomeprazole 178 Which of the following is a treatment regimen for eradication of H. Urinary tract infection whether NSAIDs are discontinued ? Harrison’s 18th Ed. 8 weeks 176 Combination therapy for H. constipation and darkening of tongue. Clarithromycin + omeprazole + metronidazole A. pylori was ? Harrison’s 18th Ed. Triple therapy 172 Long-term acid suppression with PPIs has been associated B. C. 60 % 186 Posterior DU can penetrate into ? D. 50 % D. or ulcers in the absence of H. tuberculosis or syphilis. 20 % Rare etiologies of refractory GU/DU’s include ischemia. and perforation). 2455 D. 2455 A. Syphilis A. sarcoidosis. 01 and 10 years 194 Which of the following suggest the diagnosis of ZES ? Harrison’s 17th Ed. Sarcoidosis A. with up to 30–50% of patients ? having multiple lesions or metastatic disease at presentation. Hypergastrinemia 193 What proportion of gastrinoma are malignant ? C. D. 60 % output leads to peptic ulcer diathesis. Eosinophilic gastroenteritis C. Lymphoma B. 185 Etiologies of refractory ulcers (GU / DU) include all except ? junction of second & third portions of duodenum inferiorly. Stomach Posterior DU can penetrate into pancreas. Harrison’s 18th Ed. All of the above C. 1868 B. Lymph nodes Harrison’s 18th Ed. 80 % Harrison’s 18th Ed. 2455 ulcers presenting with frank complications (bleeding. Has a trophic action on gastric epithelial cells B. Duodenum C. bones. 10 and 30 years A. Stimulates acid secretion through gastrin receptors on parietal cells 195 What proportion of ZES patients have diarrhoea ? B. Liver 187 Which of the following is not a typical feature of Zollinger– Ellison Syndrome (ZES) ? D. and diarrhea. Diarrhea A. Ulcers refractory to standard medical therapy D. obstruction. 40 % lymphoma. amyloidosis. AJAY MATHUR Gastroenterology C. and junction of neck & body of pancreas Harrison’s 18th Ed. 25 % C. Pancreas 192 Which of the following is the most common extrapancreatic B. Less- common extrapancreatic sites include stomach. and lymph nodes. Crohn’s disease. Leprosy 191 What proportion of gastrinoma are found within the B. 2455 from ECL cells A. Cytomegalovirus (CMV) hypothetical gastrinoma triangle ? C. Liver Harrison’s 18th Ed. 2455 A. 2454-5 Duodenal gastrinoma tumors constitute the most common nonpancreatic lesion (50 to75%). colon. Harrison’s 18th Ed. Ulcer presenting with frank complication In ZES. ovaries. Junction of II and III portions of duodenum C. D. Inferior surface of liver D. 50 and 70 years C. The increased gastric acid C. 2455 D. pylori or NSAID ingestion. 40 % a autonomous non-beta cell endocrine tumor (gastrinoma) defines ZES. Ulcerative colitis Gastrin stimulates acid secretion through gastrin receptors on parietal cells and by inducing histamine release from ECL cells. heart. 30 and 50 years B. cytomegalovirus (CMV). All of the above A. 20 % Severe peptic ulcer secondary to gastric acid hypersecretion due to unregulated gastrin release from B. A. liver. and majority of patients are diagnosed between the ages of 30 and 50 years. 2452 medially. Tuberculosis Harrison’s 18th Ed. B. All of the above Gastrinoma should be suspected when ulcers occur unusual locations like II part of duodenum & 189 Which of the following is the action of gastrin ? beyond. eosinophilic gastroenteritis. Gastrin release from beta cell endocrine tumor B. males are more commonly affected than females. A. Ulcer in II part of duodenum & beyond C. D.404 MCQ’s FOR MEDICAL PROFESSIONALS BY PROF. Gastrin also has a trophic action on gastric epithelial cells. ulcer recurrence after acid-reducing surgery. A. Amyloidosis 184 Etiologies of refractory ulcers (GU / DU) include all except ? 190 ‘Gastrinoma triangle’ is formed by all except ? Harrison’s 18th Ed. B. Erosive esophagitis Harrison’s 18th Ed. Colon site of gastrinoma ? C. Stimulates acid secretion by inducing histamine release Harrison’s 18th Ed. 2452 Over 80% of gastrinoma are found within the hypothetical gastrinoma triangle. Junction of neck & body of pancreas Hypothetical gastrinoma triangle is formed by confluence of cystic & common bile ducts superiorly. erosive esophagitis. 2455 D. HIV D. majority of patients are diagnosed between the ages of More than 60% of gastrinoma tumors are considered malignant. 75 % . liver or biliary tree. A. Cystic and common bile ducts B. 2452 Harrison’s 18th Ed. ulcers refractory to standard medical therapy. 80 % 188 In ZES. massive small-bowel obstruction. renal insufficiency. An additional distinguishing feature in ZES patients with MEN I is the higher incidence of gastric carcinoid tumor development (as compared to patients with sporadic C. Hypertension Harrison’s 18th Ed. 25 pg/mL B. >= 1. number & location of gastrinoma provocative test ? C. Magnetic resonance imaging A. Ankylosing arthritis 196 Etiology of the diarrhea in ZES is ? Harrison’s 18th Ed.3 MEN I syndrome (autosomal dominant) involves parathyroid glands (80-90%). and located in the duodenal wall more often than is seen B. 100 pg/mL Harrison’s 18th Ed. Renal insufficiency 208 Which of the following is a favorable prognostic indicator in C. C. 150 pg/mL A. 8 meq/hour 197 Organs involved in MEN I syndrome are all except ? Harrison’s 18th Ed. multiple. A. Disease free period after surgery Harrison’s 18th Ed. Vitiligo B. multiple. < 0. Standard meal test gastrinomas). vitiligo. 2456 D. A. 200 Patients with gastrinoma have gastrin level more than ? Increase in gastrin of >=120 pg within 15 minutes of secretin injection has a sensitivity & specificity Harrison’s 18th Ed. 12 meq/hour A. B. Calcium infusion study in patients with sporadic ZES. Gastrinomas tend to be smaller. 50 pg/mL 207 Which of the following tests has maximum sensitivity in detecting primary gastrinoma ? C. A. 2455 C.6 does not exclude the diagnosis. Long arm of chromosome 11 Harrison’s 18th Ed. 2455 C. Parathyroid D. 2456 D. 2455 BAO/MAO ratio >0. 2455 of >90% for ZES.100%. Pheochromocytoma A. All of the above A. A. pancreas (40-80%). None of the above Most sensitive & specific gastrin provocative test for diagnosis of gastrinoma is secretin study. C. 15 meq/hour B. Rheumotoid arthritis B. Pancreatic enzyme inactivation by acid Elevated fasting gastrin level are due to gastric hypochlorhydria or achlorhydria. Volume overload to the small bowel D. Damage of the intestinal epithelial surface by acid D. 2457 . Short arm of chromosome 12 A. 4 meq/hour B. diabetes mellitus & pheochromocytoma. Short arm of chromosome 11 205 What level of basal gastric pH excludes gastrinoma ? B. Pancreas Harrison’s 18th Ed. B. Calcium infusion study is less sensitive & specific with greater potential for adverse effects. A. Long arm of chromosome 12 B. 2455 D. 405 Gastroenterology MCQ’s MCQ’s FOR FORPROFESSIONALS MEDICAL MEDICAL PROFESSIONALS BY PROF. > 0. etiology of diarrhea is multifactorial.6 is highly suggestive of ZES. Pituitary A.6 Harrison’s 18th Ed. and located in the duodenal wall more often than is seen in patients with sporadic ZES. All gastrinoma patients have a gastrin level >150 . Hypochlorhydria Sensitivity of EUS in Zollinger-Ellison Syndrome is 80 . >= 2 199 Distinguishing feature between MEN I & sprodic ZES is ? D. pancreatic enzyme inactivation by acid & damage of intestinal epithelial surface by acid. 2455 Diarrhea is the next most common clinical manifestation after peptic ulcer and is found in up to 50% of patients of ZES. B.6 and pituitary gland (30-60%). 100 % 202 Elevated fasting gastrin level are due to all except ? Harrison’s 18th Ed.5 The genetic defect in MEN I is in the long arm of chromosome 11 (11q11-q13). D. > 0. Endoscopic ultrasonography (EUS) Harrison’s 18th Ed. Diabetes mellitus ZES ? D. Thyroid 204 What value of BAO / MAO is highly suggestive of ZES ? C. 1759 In ZES. Incidence of gastric carcinoid tumor 206 Which of the following is the most sensitive & specific gastrin B. AJAY MATHUR Cardiology 405 D.200 pg/mL. All of the above 203 Patients with gastrinoma have a BAO level more than ? Harrison’s 16th Ed. Secretin stimulation test Gastrinomas tend to be smaller. resulting from marked volume overload to small bowel. >= 3 Harrison’s 18th Ed. rheumatoid arthritis. Selective arterial secretin injection (SASI) Fasting gastrin levels are <150 pg/mL. C. Octreoscan imaging with 111 In-pentreotide 201 Elevated fasting gastrin level are due to all except ? C. Size. 2455 A basal gastric pH >=3 virtually excludes a gastrinoma. < 0. 2456 Table 293-8 D. but a ratio <0. 2456 C.3 198 Genetic defect in MEN I is in ? D. >= 1 D. AB gastritis refers to a mixed antral/body picture. All of the above D. Billroth I Elderly individuals. Type O gastritis 214 ‘Phlegmonous gastritis’ refers to ? Type A gastritis. A. with antral sparing (autoimmune) and type B is the antral- predominant form (H. Stress ulceration after mechanical ventilation Early phase of chronic gastritis is superficial gastritis in which inflammatory changes are limited to D.406 MCQ’s FOR MEDICAL PROFESSIONALS BY PROF. favorable prognostic indicators include primary duodenal wall tumors. Stress ulceration after severe burns D. Isolated lymph node tumor C. 2458 A. large pancreatic primary tumors (>3 cm). 2457 A. metastatic disease to lymph nodes. > 3. Type AB gastritis PPIs are the treatment of choice for stress ulcer prophylaxis. Type B gastritis C. at times accompanied by necrosis. alcoholics. 217 The final stage of chronic gastritis is ? Harrison’s 18th Ed. pylori–related). Escherichia coli A. 2457 210 Cushing’s ulcer refers to ? A. > 2. Stress ulceration after severe burns D. Type A refers to every 2–3 hours are viable options to avoid stress ulceration.0 B. Rapid growth of 215 Which of the following are affected by Phlegmonous gastritis ? hepatic metastases is also predictive of poor outcome. Stress ulceration after sepsis lamina propria and intact gastric glands.5 D. Poor outcome indicators are shorter disease duration. Intestinal metaplasia C. with destruction of gastric glands. Primary duodenal wall tumor B. Gastric atrophy B. tumor. Sucralfate B. isolated lymph node gastric wall. 2457 B. Stress ulceration after head trauma C. liver. 1756 C. Atrophic gastritis A. Vagotomy D. and undetectable tumor upon surgical exploration. All of the above C.5 with continuous infusion of H2 blockers or liquid antacids administered Chronic gastritis is also classified according to the predominant site of involvement. Vascular congestion of stomach C. Intestinal metaplasia is an important predisposing factor for gastric cancer. Type AB gastritis D. gastric pH should be maintained at ? 218 Antral-predominant form of chronic gastritis is called ? Harrison’s 18th Ed. mucosa is thin and underlying blood vessels can be visualized. and there is a paucity of inflammatory (Curling’s ulcer). > 1. Staphylococci Harrison’s 18th Ed. D. Next stage is atrophic gastritis in which inflammatory infiltrate extends deeper into mucosa. 2458 211 Curling’s ulcer refers to ? A. C. H2 blocker with pernicious anemia ? Harrison’s 18th Ed. 2457 antibodies against parietal cells and IF. Intestinal metaplasia refers to the conversion of gastric glands to small-bowel mucosal glands containing goblet cells. and bone and Cushing’s syndrome. staphylococci. 2457 Harrison’s 18th Ed. and AIDS patients may be affected by Phlegmonous gastritis. 212 To avoid stress ulceration. Elevated gastric acid secretion may be noted in patients with stress ulceration after head trauma (Cushing’s ulcer). fundus and body predominant form . Bacterial infection of stomach B. > 2. 2458 B. All of the above B. Elderly individuals 209 Procedure that provides the lowest rates of peptic ulcer recurrence but has highest complication rate is ? B. also called autoimmune gastritis is associated with pernicious anemia with circulating Harrison’s 18th Ed. Vagotomy in combination with antrectomy 216 Organism associated with Phlegmonous gastritis is ? D. All of the above Bacterial infection of the stomach or phlegmonous gastritis although rare is a potentially life- threatening disorder characterized by marked & diffuse acute inflammatory infiltrates of entire In ZES. infiltrates. Endoscopically. Viral infection of stomach . Superficial gastritis Harrison’s 18th Ed. Escherichia D. Final stage of chronic Elevated gastric acid secretion may be noted in patients with stress ulceration after severe burns gastritis is gastric atrophy in which glandular structures are lost.5 C. higher gastrin levels (>10. Stress ulceration after sepsis coli. AJAY MATHUR Gastroenterology A. Undetectable tumor upon surgical exploration D. 2457 B. Type A gastritis C. Type O gastritis Maintenance of gastric pH >3. AIDS patients A. Harrison’s 18th Ed. Stress ulceration after mechanical ventilation Organisms associated with Phlegmonous gastritis include streptococci. and Haemophilus species. Type A gastritis B.5 A. PPI A. All of the above C. Proteus. 2457 219 Which of the following types of chronic gastritis is associated A. Haemophilus B. Type B gastritis D. 213 Treatment of choice for stress ulcer prophylaxis is ? Harrison’s 18th Ed.000 pg/mL). Stress ulceration after head trauma C. Alcoholics Harrison’s 16th Ed. Billroth II Harrison’s 18th Ed. 2460 Harrison’s 18th Ed. > 200 . 2460 A. D. > 400 . Eosinophilic gastritis B. Tropical sprue 223 Large gastric folds can be seen in ? B. disease is called varioliform gastritis. prostaglandins. >2% B. Celiac sprue Harrison’s 18th Ed. All of the above 230 In a western-type diet. 2460 A. >6% A subgroup of patients with lymphocytic gastritis have thickened folds noted on endoscopy. None of the above Ménétrier’s disease is not considered a form of gastritis. Wilson’s disease C. Cirrhosis A. ZES C. 2460 A.K+-ATPase. Cirrhosis D. Crohn’s disease D. Gastrin receptors B. Hyperplasia of surface & glandular mucous cells D. 227 Steatorrhea is defined as an increase in stool fat excretion of 221 Varioliform gastritis best relates to ? how much of dietary fat intake ? Harrison’s 18th Ed. Sarcoidosis D. Anticholinergic agents B. malabsorption syndromes are associated with steatorrhea. Primary lactase deficiency and pernicious anemia are not associated with steatorrhea. of patients with type A gastritis. PPIs. >5% D. VIPoma A. Crohn’s disease .200 gram / day A.425 gram / day C. 2458 A. tortuous 229 Malabsorption disorder not associated with steatorrhea is ? gastric mucosal folds. and H 2 receptor western-type diet is consumed. 2460 Harrison’s 18th Ed. Prednisone C. 2459 A. It is characterized by large. Enterotoxin-induced traveler’s diarrhea 225 Intestinal absorption is increased in ? B. D. PPIs Diarrhea as a sign is a quantitative increase in stool weight of >200–225 mL gram per day. 2460 Harrison’s 18th Ed. Bacterial overgrowth syndrome C. H2 receptor antagonists D. Histamine receptors D. 228 Malabsorption disorder not associated with steatorrhea is ? 222 Which of the following is false about Ménétrier’s disease ? Harrison’s 18th Ed. Harrison’s 18th Ed. 2459 A. Sarcoidosis Most. H+. Lymphocytic gastritis A. The parietal cell antibody is directed against H+. Celiac sprue B.500 gram / day D. Protein-losing gastropathy B.225 gram / day B. Pernicious anemia B. 231 Which of the following diarrhea would undoubtedly cease during a prolonged fast ? 294 . Primary lactase deficiency Harrison’s 18th Ed. Large gastric mucosal folds in body and fundus C. 2460 C. but not all. >4% C. Jejunal diverticulosis B. 2459 A. where absorption of iron and copper is increased respectively. antagonists yields varying results. Anticholinergics decrease protein loss. > 100 . 407 Gastroenterology MCQ’s MCQ’s FOR FORPROFESSIONALS MEDICAL MEDICAL PROFESSIONALS BY PROF. These Most malabsorption syndrome disorders are associated with an increase in stool fat excretion of folds are often capped by small nodules that contain a central depression or erosion.Disorders of Absorption Harrison’s 18th Ed.K+-ATPase Only clinical malabsorption situations in which absorption is increased are hemochromatosis & Antibodies to parietal cells are detected in >90% of patients with pernicious anemia and in up to 50% Wilson’s disease. Abetalipoproteinemia C. Acetylcholine receptors C. AJAY MATHUR Cardiology 407 220 Parietal cell antibodies are directed against which of the 226 Intestinal absorption is increased in ? following ? Harrison’s 18th Ed. Gastric malignancy D. when a Medical therapy with anticholinergic agents. Hemochromatosis fast. diarrhea as a sign is a quantitative 224 Which of the following decreases protein loss in Ménétrier’s increase in stool water or weight of ? disease ? Harrison’s 18th Ed. Intestinal lymphangiectasia D. 2459 Harrison’s 18th Ed. Granulomatous gastritis C. this form of the >6% of dietary fat intake (steatorrhea). Primary lactase deficiency A. Jejunal diverticulosis Diarrhea secondary to lactose malabsorption in primary lactase deficiency ceases during a prolonged C. prednisone. > 300 . All of the above B. Primary bile acids are cholic acid & chenodeoxycholic acid. Basomedial membrane mOsm/kg (electrolytes account for most of stool osmolality).K + - Harrison’s 18th Ed. C. 5 to 6 L/day 243 Primary functions of bile acids is ? C. All of the above 234 The lengths of the small intestine and colon are ? Na+ pump is located on the basolateral membrane. and intestinal fluid amounts to ? D. 2461 C. 390 . 6 to 7 L/day Harrison’s 18th Ed. Apical membrane D. A. Stool osmolality is assumed to be 300 mosmol/kg H2O.2([Na+] + [K +]) B. D. ~200 cm and ~50 cm.K+ . B.ATPase. 300 mosmol/kg H 2O not in colon D. Secretory function is present in crypts of both small & large 233 Fecal osmotic gap is calculated as ? intestine Gastroenterology 1999. respectively A. A competitive inhibitor of SGLT. Basolateral membrane Osmotic diarrheas are characterized by osmotic gaps >125 mOsm/kg (nonelectrolytes account for most of the osmolality of stool water). Apical membrane D. To promote bile flow The intestine absorbs ~7 to 8 liters of fluid daily. All of the above 235 Effective functional surface area of intestines is about how many times greater than that of a hollow tube ? Active glucose (monosaccharide) absorption & glucose-stimulated Na + absorption require both apical membrane transport protein SGLT (sodium/glucose cotransporter) & basolateral Na +. biliary. 200 mg Intestinal mucosa synthesizes and secretes secretory IgA. respectively B. Secretory IgA bile acids are deoxycholic acid and lithocholic acid. 7 to 8 L/day A. 200 times SGLT is SLC5A. Primary bile acids are synthesized in liver from cholesterol Effective functional surface area is about 600-fold greater than that of a hollow tube due to the B. 2461 D.ATPase in the Na+ pump is located on ? Harrison’s 18th Ed. 275 mosmol/kg H 2O B. Basolateral membrane Lengths of small intestine and colon are ~300 cm and ~80 cm respectively. whereas secretory diarrheas typically have osmotic gaps <50 C. B. 2461 in liver from cholesterol and secondary bile acids are synthesized from primary bile acids in intestine by colonic bacterial enzymes. conjugated to either taurine or glycine to form A. 400 times 241 Which of the following about bile acids is false ? C. Digestive hydrolytic enzymes are present in brush border of villus epithelial cells Stool osmotic gap = 2 x (stool Na + stool K). 600 times Harrison’s 18th Ed. 400 mg 237 Daily salivary. 2461 D. 2461 Na+ through Na+.2([Na+] + [K +]) 239 Na+. 290 . respectively 240 Transport protein SGLT is located on ? B. Primary bile acids are synthesized Harrison’s 18th Ed. 3 to 4 L/day tauro-conjugated or glyco-conjugated bile acids. C. Lithocholic acid is a secondary bile acid immunoglobulin ? Bile acids are not present in the diet but are synthesized in liver. and then secreted into duodenum as bile. gastric. To solubilize cholesterol & phospholipid in gall bladder salivary. 2461 A. 2461 Harrison’s 18th Ed. Gene for A. gastric. To enhance dietary lipid digestion & absorption . 300 mg C. Basomedial membrane D. which expels Na+ and maintains a low intracellular Harrison’s 18th Ed. 250 mosmol/kg H 2O A.116:1461-1463 Villi are present in small intestine but are absent in colon. ~300 cm and ~80 cm. ~250 cm and ~70 cm. A. Villi are present in small intestine & colon B. 325 mosmol/kg H 2O C. and microvilli. 2461 C. Secretory IgM Harrison’s 18th Ed. 190 . D. 500 mg Harrison’s 18th Ed.2([Na+] + [K+]) B. respectively. Secondary bile acids are synthesized from primary bile acids presence of folds. K+ . biliary. Secondary A. B. B. villi (in small intestine). 2461 D. phlorizin exerts a hypoglycemic effect in diabetics. Nutrient digestion & absorption occurs in small intestine but C. AJAY MATHUR Gastroenterology 232 Stool osmolality is ? 238 Which of the following statements is false ? Harrison’s 18th Ed. pancreatic. Secretory IgE A.408 MCQ’s FOR MEDICAL PROFESSIONALS BY PROF. Secretory IgG 242 What quantity of bile acids are synthesized in liver every day ? C. C.2([Na+] + [K +]) A. and intestinal fluid (6 to 7 L/day). respectively Harrison’s 18th Ed. comprising dietary fluid intake (1 to 2 L/day) and B. pancreatic. 2461 ATPase. 2461 About 500 mg bile acids are synthesized in liver daily. 90 . Cholic & deoxycholic acids are primary bile acids 236 Intestinal mucosa synthesizes & secretes which of the following D. ~400 cm and ~100 cm. 800 times A. 2 to 4 times / day cholestyramine & no response to low-fat diet. 7 -hydroxylase A. 2462 Harrison’s 18th Ed. Ileum Patients with greater degrees of ileal disease and/or resection have diarrhea & steatorrhea that do not respond to cholestyramine. fecal bile acid loss is compensated by hepatic Harrison’s 18th Ed. 409 Gastroenterology MCQ’s MCQ’s FOR FORPROFESSIONALS MEDICAL MEDICAL PROFESSIONALS BY PROF. Steatorrhea each meal. C. Responds to low-fat diet Harrison’s 18th Ed. Protein diarrhea C. Half of total fat intake bile acid excretion increased. Table 294-2 D. 2462. It is called bile acid diarrhea. It responds promptly to cholestyramine until ileal size is severely reduced. B. Hepatic bile acid synthesis 255 Which of the following type of fatty acids compose dietary fats ? D. Chloride diarrhea Harrison’s 18th Ed. Responds to low-fat diet 248 How many times bile acid pool is circulated via enterohepatic circulation ? Features of bile acid diarrhea are . response obtained with A. an initial enzyme in cholesterol degradation. 247 The bile acid pool size is approximately ? Harrison’s 18th Ed. Cirrhosis B. 2462 B. Jejunum D. fecal A. Figure 294-1 Features of fatty acid diarrhea are extensive ileal disease. None or mild steatorrhea D. 252 Reabsorption defect in enterohepatic circulation of bile acids 246 Bile acid synthesis is autoregulated by ? is due to ? Harrison’s 18th Ed. Low-fat diet can be effective. steatorrhea is absent or mild. Na +-dependent process exclusively in ileum. C. 7 -hydroxylase B. 7 -hydroxylase D.extent of ileal disease is limited. or six to eight times daily. 12 grams B. Duodenum C. Jejunum active Cl . Figure 294-1 253 Which of the following is not a feature of bile acid diarrhea ? A. Reduced bile acid pool size Bile acid pool size is ~4 grams and is circulated via the enterohepatic circulation about twice during B. Medium-chain fatty acids (MCFAs) . 6 to 8 times / day Harrison’s 18th Ed. 7 -hydroxylase C. Fatty acid diarrhea A. Table 294-2 B. 2462. B. Responds to cholestyramine 249 Daily bile acids excretion in stool (fecal loss) equals ? D. D. steatorrhea (>20 grams) that does not respond to cholestyramine but responds B. 4 to 6 times / day 254 Which of the following is not a feature of fatty acid diarrhea ? C. Bile acid diarrhea proximal small intestine. Normal bile acid pool size C. Harrison’s 18th Ed. Bile acid diarrhea 245 Secondary bile acids are formed in ? Harrison’s 18th Ed. Figure 294-1 synthesis. 2462 A. 2463 A. All of the above 250 Small ileal dysfunction leads to ? Harrison’s 18th Ed. None of the above Harrison’s 18th Ed. AJAY MATHUR Cardiology 409 D. this A. Carbohydrate diarrhea B. Ileal dysfunction caused by Crohn’s disease results in a decrease in bile acid reabsorption in ileum and an increase in the delivery of bile acids to large intestine leading to diarrhea with or without steatorrhea. bile acid pool size is normal. or C. Colon 251 Large ileal dysfunction leads to ? Bile acids are primarily absorbed by an active. Primary biliary cirrhosis C. ileal bile acid absorption reduced. fecal bile acid loss is not compensated by hepatic synthesis. 2462. Responds to cholestyramine D. 2462 Primary functions of bile acids are to promote bile flow. Jejunal diverticulosis D. Ileum cholorrheic enteropathy. Hepatic synthesis D. Ileal bile acid absorption is reduced. Crohn’s disease Bile acid synthesis is autoregulated by 7 alpha-hydroxylase. bile acid pool size is reduced. Increased quantities of bile acids enter colon. Colon proves insufficient resulting in impaired micelle formation and steatorrhea. to solubilize cholesterol and phospholipid in gallbladder and to enhance dietary lipid digestion and absorption by forming mixed micelles in A. Fatty acid diarrhea 244 Bile acids are primarily absorbed “actively” in ? C. Figure 294-1 D. 8 grams A. 2463. 2462. 4 grams Harrison’s 18th Ed. 2463 A relatively small quantity of bile acids (~500 mg) is not absorbed and is excreted in stool daily. Long-chain fatty acids (LCFAs) fecal loss is matched by hepatic bile acid synthesis. This is called fatty acid diarrhea.secretion producing diarrhoea but not steatorrhea because hepatic synthesis of bile acids increases to compensate for the rate of fecal bile acid losses upto a limit. 2463. 16 grams C. Protein diarrhea A. Duodenum Due to limited ileal disease or resection increased amount of bile acids are delivered into colon that stimulate B. B. Colonic bacterial enzymes dehydroxylate bile acids to secondary bile acids. Half of enterohepatic circulation to a low-fat diet. fecal bile acid excretion is increased. 8 to 12 times / day A. 2463.20 Normal lipolysis can be maintained by ~5% of maximal pancreatic lipase secretion. Gastric lipase A. Conjugated bile acids B.e. Mucosal or absorptive phase Mixed micelles are molecular aggregates composed of fatty acids. All of the above D. monoglycerides. Long-chain triglycerides (LCTs) A. Delivery or postabsorptive phase D. pH < 8. lipolysis and micellar formation. 10 . Medium-chain fatty acids (MCFAs) B. 2464 Harrison’s 18th Ed. 2464 261 Lipolysis is initiated in ? A.410 MCQ’s FOR MEDICAL PROFESSIONALS BY PROF. > 12 Pancreatic lipolysis is greatly enhanced by the presence of pancreatic enzyme. Micellar formation Harrison’s 18th Ed.5 A. Table 294-3 A. 16 . Glycerol D. 2464 259 Steatorrhea results due to defect in which phase of dietary lipid assimilation ? A. Triglycerides assimilation ? Harrison’s 18th Ed. Any of the above 267 Which of the following relates best with absorptive phase of lipid digestion-absorption ? The digestive phase has two components. hydrolysis of triglycerides by lipase leads to the 256 Dietary fat is in the form of ? formation of ? Harrison’s 18th Ed. i. Fatty acids 260 “Micellar formation” belongs to which phase of dietary lipid B. 15 % B.30% of total lipolysis occurs in stomach. Jejunum Three types of fatty acids compose fats: long-chain fatty acids (LCFAs).16 D. Assimilation of dietary lipid occurs in intraluminal or digestive phase. Mucosal or absorptive phase D.long chain fatty acids (LCFAs). Free fatty acids B. medium-chain fatty acids D. are present in large amounts in coconut oil. 2464 A. mucosal or absorptive phase 266 Mixed micelles are molecular aggregates composed of all except ? and delivery or postabsorptive phase. Harrison’s 18th Ed. cholesterol. Short-chain fatty acids (SCFAs) B. medium-chain fatty acids facilitates the movement of lipase to triglyceride. hydrolysis of Tg to free fatty acids. 6-8 B. 257 Majority of dietary long chain fatty acids (LCFAs) have carbon 263 Pancreatic lipolysis is greatly enhanced by ? chain lengths of ? Harrison’s 18th Ed.12 C. Duodenum D. Delivery or postabsorptive phase Lipolysis is completed in the duodenum and jejunum by pancreatic lipase. 264 Normal lipolysis can be maintained by what percentage of maximal pancreatic lipase secretion ? 258 Dietary MCFAs have carbon chain lengths of ? Harrison’s 18th Ed. Medium-chain triglycerides (MCTs) or medium-chain fatty acids.. pH < 8 B. colipase. 262 In lipolysis. pH < 7 Harrison’s 18th Ed. 8 . All of the above C. Table 294-3 A. 2463 B. Stomach C. Uptake and reesterification A. glycerol that is bound via Lipolysis i. Any of the above <7. 2463 C. 2463 Harrison’s 18th Ed. An abnormality at any site of this process can cause steatorrhea. Intraluminal or digestive phase D. Intraluminal or digestive phase C. Cholesterol A. Pancreatic lipase B. All of the above Dietary fat is exclusively composed of long-chain triglycerides (LCTs). 2463 Harrison’s 18th Ed. 25 % C. and short-chain fatty acids (SCFAs). 6-8 B. stomach by gastric lipase. 2463 B. C. Formation of chylomicrons . 12 .5 C. phospholipids. which is inactivated by pH D.e. 2463. Iliem (MCFAs). 265 Pancreatic lipase is inactivated at ? Harrison’s 18th Ed. pH < 7. Short-chain fatty acids (SCFAs) C.12 D. 5% A. Colipase C. 8 .10 C. and conjugated bile acids. composed of fatty acids with carbon chain lengths of 8 to 10. 35 % D. ~20 . (MCFAs) & short-chain fatty acids (SCFAs). All of the above D. Majority of dietary LCFAs have carbon chain lengths of 16 or 18. which 3 types of fatty acids compose fats . AJAY MATHUR Gastroenterology C. Harrison’s 18th Ed. Monoglycerides C.0 leading to altered lipolysis. monoglycerides & glycerol by lipase is “initiated” in ester-linkages to three LCFAs. 2463 A. composed of fatty acids with carbon chain lengths of 8 to 10. Micelle formation C. & Fatty acids and monoglycerides are reesterified by a series of enzymatic steps in the endoplasmic their route of exit is via portal vein & not via lymphatics. 270 Chylomicrons contain ? 276 The primary nutrient for colonic epithelial cells is ? Harrison’s 18th Ed. and phospholipids and enter the lymphatics. propionate. B. MCTs do not require pancreatic lipolysis as Tg can be absorbed intact by intestinal D. SCFAs are rapidly absorbed and stimulate colonic Na-Cl and fluid absorption. Lipolysis carbohydrate. None of the above . Micelle formation is not necessary for absorption A. C. Acetate Harrison’s 18th Ed. SCFAs are rapidly absorbed & stimulate colonic Na-Cl & 272 In abetalipoproteinemia. Lipid uptake 278 Carbohydrates in the diet are present in the form of ? Harrison’s 18th Ed. Portal vein A. Propionate A. Cholesterol and Cholesterol ester B. B. Medium-chain triglycerides (MCT) 279 Which of the following statements about carbohydrates is false ? C. Lipolysis. Starch Abetalipoproteinemia. 2464 B. or acanthocytosis is a disorder of impaired synthesis of beta-lipoprotein. 2465 D. Short-chain fatty acids (SCFA) Harrison’s 18th Ed. C. 2464 268 In which form lipids exit from intestinal epithelial cell ? A. and lipid uptake are all normal in patients. Disaccharides (sucrose and lactose) cannot exit from intestinal epithelial cell because of the failure to produce chylomicrons. Butyrate B. cholesterol esters. Systemic vein B. C. -lipoprotein D. Colonic bacterial enzymes 274 Which of the following statements about Medium-chain triglycerides (MCTs) is false ? Uptake and reesterification constitute the absorptive phase of lipid digestion-absorption. 2464 SCFAs are not dietary lipids but are synthesized by colonic bacterial enzymes from nonabsorbed A. Acetate B. Absorbed only in the small intestine Medium-chain triglycerides (MCTs). Harrison’s 18th Ed. respectively. 2465 A. 3. disaccharides (sucrose and lactose). 2464 Carbohydrates in the diet are present in the form of starch. SCFAs present in stool are primarily acetate. All of the above Butyrate is the primary nutrient for colonic epithelial cells & its deficiency may be associated with 271 Reesterified triglyceride exit from intestinal epithelial cell into ? colitis. but reesterified triglyceride B. 2464 A. and 4. reticulum to form triglycerides. All of the above D. Following absorption. whose carbon chain D. the form in which lipid exits from the intestinal epithelial cell. SCFAs are dietary lipids C. All of the above nonabsorbed carbohydrate Chylomicrons are composed of beta-lipoprotein and contain triglycerides. 411 Gastroenterology MCQ’s MCQ’s FOR FORPROFESSIONALS MEDICAL MEDICAL PROFESSIONALS BY PROF. propionate. B. following absorption are not reesterified. Absorption occurs by a Na-dependent process D. 2465 B. Harrison’s 18th Ed. Long-chain triglycerides (LCTs) glucose. & butyrate D. -lipoprotein lengths are 2. Cholesterol Unlike LCTs. Lymphatics 277 Which of the following statements about SCFAs is false ? Harrison’s 18th Ed. All of the above A. All of the above Harrison’s 18th Ed. All of the above C. and butyrate. SCFAs in stool are acetate. 2464 Harrison’s 18th Ed. cholesterol. 2465 269 Chylomicrons are composed of ? A. Phospholipid C. micelle formation. and A. Reesterified triglycerides require formation of chylomicrons for their exit from small-intestinal 275 The SCFA present in stool is ? epithelial cell & their delivery to liver via lymphatics. -lipoprotein The SCFAs present in stool are primarily acetate. not the portal vein. and butyrate. -lipoprotein C. Synthesized by colonic bacterial enzymes from D. Route of exit is via lymphatics C. Absorbed only in the form of monosaccharides are present in large amounts in coconut oil and are used as a nutritional supplement. Reesterified triglyceride exit from epithelial cell A. 2465 D. Harrison’s 18th Ed. Reesterified triglyceride D. Butyrate D. Triglyceride A. Glucose 273 Coconut oil contains mainly ? D. AJAY MATHUR Cardiology 411 D. Propionate C. propionate. Do not require pancreatic lipolysis Harrison’s 18th Ed. do not require chylomicron formation for their exit from intestinal epithelial cells. not reesterified B. 2464 B. Free fatty acids C. Monoglyceride epithelial cell & micelle formation is not necessary for absorption of MCTs. the defect is in ? fluid absorption Harrison’s 18th Ed. whose absorption occurs by a Na-dependent process mediated by the brush border transport protein SGLT1.15 % of all antibiotic-associated diarrhea. TGLT D. Tripeptides C. Symptoms may be similar to IBS Protein is present in food almost exclusively as polypeptides and requires extensive hydrolysis to di- and tripeptides and amino acids before absorption. Fructose C. Fructose A. Cystinuria. Sorbitol D.412 MCQ’s FOR MEDICAL PROFESSIONALS BY PROF. Glucose and galactose are constituents of lactose B. 2466 Harrison’s 18th Ed. None of the above A. 2465 Harrison’s 18th Ed. 2466 290 C. Fructose Harrison’s 18th Ed. 284 Actively transported monosaccharides are all except ? a defect in dibasic amino acid transport. C. MGLT B. Calcium Sugar used in diabetic candy is sorbitol which is only minimally absorbed due to absence of an B. Hartnup syndrome. Sorbitol 291 The proximal small intestine is the site for the absorption of all B. difficile accounts for what percentage of all antibiotic- A. diarrhea.40 % Actively transported monosaccharides are glucose and galactose. galactose malabsorption A.15 % D. 1876 C. 2465 A. Amino acids D. Colipase C. growth retardation and hypoproteinemia.6 D. ~ 40 . ~ 25 . a defect in neutral amino acid transport. Polypeptides A. 2466 281 Which of the following about lactose malabsorption is false ? Harrison’s 18th Ed. 2466 A. Secondary lactase deficiency is seen in celiac sprue D. Most individuals with primary lactase deficiency do not have symptoms.6 B. 2466 C. difficile accounts for ~10 . Galactose B. Hartnup syndrome C.95 % 285 Sugar used in diabetic candy is ? Harrison’s 18th Ed. < 8. 2466 A.6 C. Folic acid D. Lactose malabsorption B. All of the above Fructose is absorbed by the brush border transport protein GLUT 5.75 % D. ~ 10 . 288 Brush border enzyme that converts the proenzyme trypsinogen 282 Congenital absence of SGLT leads to ? to trypsin is ? Harrison’s 18th Ed. Sucrose malabsorption D.GLUT 5 ? is seen in which of the following ? Harrison’s 18th Ed. All of the above B. SGLT C. 287 Protein is present in food almost exclusively as ? Harrison’s 18th Ed. Galactose of the following except ? C. as diarrhea. 2466 D. Primary lactase deficiency patients have severe symptoms C. Enterokinase B. Galactose Harrison’s 17th Ed. Glucose A. Iron intestinal absorptive transport mechanism for sorbitol. C. Harrison’s 18th Ed. ~ 75 . A. Enterokinase deficiency B.116:1461-1463 A. 2465 Gastroenterology 1999. Pepsinogen D. Bile acids . AJAY MATHUR Gastroenterology 280 Transport protein that mediates monosaccharide absorption 286 What value of stool pH is consistent with carbohydrate is ? malabsorption ? Harrison’s 18th Ed. Dipeptides B. is associated with renal calculi and chronic pancreatitis.6 is consistent with carbohydrate malabsorption. AGLT A. 283 Which of the following carbohydrate is absorbed by brush 289 Alterations in protein or amino acid digestion and absorption border transport protein . and subsequently by trypsin. < 7. is characterized by a pellagra-like rash and neuropsychiatric symptoms. Glucose associated diarrhea ? B. a facilitated diffusion process Enterokinase deficiency leads to failure to convert proenzyme trypsinogen to trypsin and is manifested that is not Na-dependent and is distinct from SGLT. Maltose malabsorption C. Amylase Congenital absence of SGLT leads to glucose-galactose or monosaccharide malabsorption Proenzyme trypsinogen is activated to trypsin by the intestinal brush border enzyme enterokinase. < 6. < 5. Cystinuria D. Glucose.6 Carbohydrates are absorbed only in the small intestine and only in the form of monosaccharides Stool pH <5. 68 Co-labeled cobalamin Harrison’s 18th Ed. iron and folic acid are exclusively absorbed by active transport processes in proximal small C. 2466 synthesized & released by gastric parietal cells. Assesses proximal small-intestinal mucosal function B. and Ileal dysfunction. with 58 A. <15% in 24 hours B. All of the above 301 The Schilling test is termed abnormal if oral 58Co-labeled Schilling test is performed to determine the cause for cobalamin malabsorption. where cobalamin is then bound by intrinsic factor (IF). 297 R-binder protein is synthesized in ? Harrison’s 18th Ed. Table e37-1 B. Pernicious anemia C. 78 Co-labeled cobalamin B. <20% in 24 hours D. Egg Schilling test is termed abnormal if oral 58Co-labeled cobalamin excretion in urine is <10% in 24 hours. Chapter e37 303 Co-cobalamin excretion is reduced with intrinsic factor. Chronic pancreatitis. D. Green vegetables C. Chronic pancreatitis D. Blind loop syndrome Dietary cobalamin is bound in stomach to a glycoprotein called R-binder protein. All of the above B. D. Calcium 299 Cobalamin absorption may be abnormal in which of the D. Ileum Schilling test is abnormal in pernicious anemia. blind loop syndrome & ileal disease. <20% in 24 hours C. A. Iron malabsorption Schilling test is performed by administering 58Co-labeled cobalamin orally and collecting urine for 24 hours. 413 Gastroenterology MCQ’s MCQ’s FOR FORPROFESSIONALS MEDICAL MEDICAL PROFESSIONALS BY PROF. Gastric oxyntic cells B. Chapter e37 296 R-binder protein is synthesized in ? Harrison’s 18th Ed. D. to promote its uptake by specific cobalamin receptors on the brush border of ileal enterocytes. Chapter e37 A. Chapter e37. D. cobalamin excretion in urine is ? Harrison’s 18th Ed. Jejunum A. Fruits B. Pernicious anemia A. Ileum Bacterial overgrowth syndromes. and after 5 days of antibiotics in ? Harrison’s 18th Ed. Pancreatic protease enzymes split the cobalamin– A. Salivary glands B. Ileal disease 298 Intrinsic factor is synthesized and released by ? 304 Which of the following is false about urinary D-xylose test ? Harrison’s 18th Ed. Bacterial overgrowth syndromes A. amino acids and lipids are absorbed in ? B. None of the above following ? Harrison’s 18th Ed. Bile acids C. Stomach pancreatic enzymes. Blind loop syndrome C. Meat D. Chapter e37 A. Cobalamin is present primarily in meat. Gastric parietal cells A. It is dependent on normal renal and bladder function. Ileum C. 2467 Harrison’s 18th Ed. Harrison’s 18th Ed. Duodenum D. Chapter e37 Active transport mechanisms for cobalamin & bile acids are present only in ileum. 58 Co-labeled cobalamin 294 Schilling test is performed to determine the cause for ? B. D-Xylose is a pentose carbohydrate . amino acids & lipids are absorbed throughout the small intestine. AJAY MATHUR Cardiology 413 Calcium. Jejunum Cobalamin absorption may be abnormal in Pernicious anemia. Pancreas B. Cobalamin malabsorption C. <10% in 24 hours A. 2466 C. Throughout the small intestine 300 Which isotope is used in Schilling test ? Glucose. IF is a glycoprotein Harrison’s 18th Ed. C. Pernicious anemia 293 Glucose. Gastric G cells 292 Ileum is the absorption site of all of the following except ? Intrinsic factor (IF) is absolutely required for the absorption of cobalamin. Chapter e37 295 Cobalamin is present primarily in ? Harrison’s 18th Ed. which is synthesized in stomach and salivary glands. Chapter e37 A. Chapter e37 A. 88 Co-labeled cobalamin C. Chronic pancreatitis B. Chapter e37 A. Gastric chief cells intestine especially the duodenum. chronic pancreatitis. B. Cobalamin R binder complex to release cobalamin in proximal small intestine. Jejunum C. Achlorhydria. 302 Abnormal Schilling test may be found in all except ? Harrison’s 18th Ed. Pancreas D. Folic acid malabsorption D. Chronic pancreatitis Harrison’s 18th Ed. Cirrhosis of liver D. Celiac disease D. Celiac sprue has also been known as nontropical sprue.357:1731-43 A. C. AJAY MATHUR Gastroenterology C. & oats. Antiendomysial antibody tTG deaminates gliadin. D. Barley D. All of the above A. Myocardial infarction A. Head injury 312 Gliadin (a component of gluten) is present in all except ? Harrison’s 18th Ed. Alcohol-soluble fraction A. Gliadin is a component of gluten present in wheat. quinoa.5 gram excretion) reflects presence of duodenal / jejunal mucosal disease. Immune-mediated disorder C. evidence of malabsorption and histopathologic changes on small- 305 Urinary D-xylose test can be false-positive in ? intestinal biopsy respond to elimination of gluten from diet. Rice B. All of the above In celiac sprue. Heat resistant fraction Harrison’s 18th Ed.414 MCQ’s FOR MEDICAL PROFESSIONALS BY PROF. IgA B. Tissue transaminase A. Gluten sensitive enteropathy “Gluten” refers to the entire protein component of wheat. Tissue transglutaminase C. symptoms. Greek C. 2470 317 Serum antibodies found in celiac sprue include all except ? Harrison’s 18th Ed. 315 Which of the following enzyme in intestine deamidates gliadin peptides ? 309 Which of the following about Celiac disease is false ? N Engl J Med 2007. rye & in Harrison’s 16th Ed. sorghum. Harrison’s 18th Ed. PABA D. teff (an Ethiopian cereal grain). Evidence of malabsorption D. Nontropical sprue C. IgG C. 316 The most sensitive antibody tests for the diagnosis of celiac 310 The word “sprue” comes from which language ? disease are of which class ? N Engl J Med 2007. pleural fluid).357:1731-43 D. D-Xylose D. Triggered by dietary gluten D. All of the above gluten that contains bulk of toxic components. the diet in celiac sprue ? Harrison’s 18th Ed. Gliadin is the alcohol-soluble fraction of D. barley. None of the above Enzyme tissue transglutaminase in intestine deamidates gliadin peptides increasing their Celiac disease is a systemic immune-mediated disorder triggered by dietary gluten in genetically immunogenicity. Rice Urinary D-xylose test is false-positive in large fluid collections in third space (ascites. IgE 311 Which of the following respond to elimination of gluten from Most sensitive antibody tests for diagnosis of celiac disease are of IgA class. Pancreatic exocrine function 313 Which of the following is considered as “safe grains” (gluten- B.357:1731-43 N Engl J Med 2007. Tissue transmurase B. GABA Safe grains (gluten-free) include rice. 2467 A. Pneumonia C. < 12 gm excretion means an abnormal test C. susceptible persons. IgM D. Water soluble fraction B. IgA antigliadin . Pancreatic neural function N Engl J Med 2007. All of the above B. 25 gm of D-xylose & collecting urine for 5 hours B. Millet A. and gluten sensitive enteropathy. A. corn.367:2419-26 A. 1770 smaller amounts in oats.357:1731-43 308 Synonym of Celiac sprue is ? A. Symptoms A. amaranth. Wheat C. D. Tissue transpeptidase D. Ascitis B. Pancreatic endocrine function free) ? C. Systemic disorder B. millet.357:1731-43 N Engl J Med 2012. 2470 A. 1770 C. 2469 B. celiac disease (in children). Histopathologic changes in small-intestinal biopsy Urinary D-xylose test (<4. Corn 307 Which of the following is used in Bentiromide test ? Harrison’s 16th Ed. Oats 306 Bentiromide test is used for ? Gluten is a protein complex. adult celiac disease. 2470 B. English D. Contaminated fraction C. Dutch A. Latin B. All of the above 314 Gliadin is which component of gluten ? N Engl J Med 2007. buckwheat. an uncertain diagnosis ? N Engl J Med 2012. Mostly observed in adults B. Crypt hyperplasia is almost 100% in celiac sprue ? C. Crohn’s disease & gastrinoma with acid 322 Which of the following tests have a high negative predictive hypersecretion. HLA-DQ3 changes seen on duodenal/jejunal biopsy are restricted to mucosa and include absence or reduced C. antigen recognized by antiendomysial antibody test is tissue transglutaminase Harrison’s 18th Ed. Hypoproteinemia . Lymphoma The HLA-DQ2 haplotype (DQA1*0501/DQB1*0201) is expressed in the majority of patients with celiac disease (90%). Enzyme B. lymphoma. HLA-DQ2 or HLA-DQ8 C. IgA anti-tTG antibodies B. At least 39 non-HLA genes that predispose to celiac disease have been identified.367:2419-26. All patients with celiac disease Harrison’s 18th Ed. Harrison’s 18th Ed. IgA anti-tTG antibodies A. 2470 328 Which of the following about “Celiac crisis” is false ? N Engl J Med 2012. All of the above D.367:2419-26 319 Antigen recognized by the antiendomysial antibody test is ? A.357:1731-43 N Engl J Med 2012. One third B. Tropical sprue 321 The HLA-DQ2 haplotype (DQA1*0501/DQB1*0201) is expressed B. HLA-DQ2 A small-intestinal biopsy is required to establish a diagnosis of celiac disease. Increased lymphocytes & plasma cells in lamina propria N Engl J Med 2007.g. Intestinal lymphangiectasis of patients with celiac disease. In another 5% B. 415 Gastroenterology MCQ’s MCQ’s FOR FORPROFESSIONALS MEDICAL MEDICAL PROFESSIONALS BY PROF. None A. Histopathologic features of celiac sprue is seen in tropical sprue. IgA antiendomysial antibodies B. Tissue transaminase C. One half 327 Histopathologic features characteristic of celiac sprue can also C. IgA antiendomysial and IgA anti-tTG antibodies are present suggesting an immunologic component to etiology. A. In celiac sprue. IgA anti-tTG antibodies Harrison’s 18th Ed. Absence of DQ2 excludes the diagnosis of celiac sprue. HLA-DQ2 or HLA-DQ8 C. 2471 express the HLA-DQ2 or HLA-DQ8 allele. enzyme responsible for deamidation of gliadin in lamina propria. 2471 D. HLA-DQ5 In patients with doubtful celiac sprue. whereas it is expressed in one third of the general population. DQ2 & DQ8 haplotypes are necessary but not sufficient for the D. Harrison’s 18th Ed. AJAY MATHUR Cardiology 415 B. eosinophilic enteritis & milk-protein intolerance in children. bacterial overgrowth. IgA antiendomysial antibodies D. IgG anti-tTG antibodies B. B. & antibodies against tissue transglutaminase. Tissue transglutaminase D. Tissue transmurase D. Tissue transpeptidase 325 Which of the following is false about duodenal/jejunal biopsy histopathology in celiac sprue ? In celiac sprue. Eosinophilic enteritis A. IgA anti-tissue transpeptidase the disease is very unlikely to develop in persons who are negative for both HLA-DQ2 and HLA-DQ8. Almost all patients with celiac sprue express HLA-DQ2 be seen in all except ? allele. which means that D. A. IgG anti-tTG antibodies A. since negative 326 Histopathologic features characteristic of celiac sprue can also predictive value of this test is almost 100%.. crypt hyperplasia and villus atrophy and increased lymphocytes and plasma cells in the lamina propria. Milk-protein intolerance in children N Engl J Med 2012.367:2419-26 D. Ulcerative colitis in what proportion of the general population ? C. Reduced height of villi 320 Negative predictive value of which of the following HLA allele B. 2470 B. Such testing has a high negative predictive value. mostly involved in inflammatory and immune responses. Severe diarrhea C. “Celiac serologies” in celiac sprue include serum antibodies like IgA antigliadin. Three fourth be seen in all except ? Harrison’s 18th Ed.357:1731-43. Gastrinoma with acid hypersecretion development of celiac disease. IgG anti-tTG antibodies C.367:2419-26 A. family members of patients with celiac disease). Epithelial tissue A. connective-tissue antibodies 324 Which of the following tests have usefulness in patients with (antireticulin & antiendomysial antibodies). HLA-DQ2 or HLA-DQ8 Serological tests in celiac disease include antigliadin antibodies. Absence of HLA DQ2/DQ8 excludes the diagnosis of celiac disease. 323 Which of the following tests have usefulness in patients with 318 Antiendomysial antibodies are directed against ? IgA deficiency ? N Engl J Med 2007. Crohn’s disease whereas almost all the remaining 5% of patients have at least one of the two genes encoding DQ2 (DQB1*0201 or DQA1*0501). 2470 D. IgA anti-tissue transglutaminase Testing for HLA-DQ2 and HLA-DQ8 may be useful in at-risk persons (e. D. 2470 (tTG). IgA antiendomysial antibodies D. None of the above A. C. Connective-tissue C. HLA-DQ4 height of villi (flat appearance). value in the diagnosis of celiac disease ? N Engl J Med 2012.367:2419-26 A. the HLA-DQ8 haplotype (DQA1*0301/DQB1*0302) is expressed. IgA antiendomysial antibodies C. HLA-DQ2 or HLA-DQ8 typing is useful. Turner’s syndrome (3%). secretion resulting from crypt hyperplasia. C. 1 to 5 mg per day D. Steatorrhea B. Abnormal intraepithelial lymphocytes A. 2471. clonal intraepithelial lymphocytes lacking surface markers malabsorption and endogenous fluid secretion. Hours to days Prevalence of celiac disease is 1. 333 Interval between exposure to gluten and onset of symptoms in 339 Complications of Celiac sprue include all except ? Wheat allergy is ? Harrison’s 18th Ed. IgA deficiency A.367:2419-26 C. D.367:2419-26 C. All of the above B. Any of the above IgA deficiency (9%). Down syndrome.367:2419-26 A. Hours to days C. Secondary lactase deficiency B.367:2419-26 Harrison’s 18th Ed. Any of the above 329 After how many months of a strict gluten free diet. CD8. Clonal intraepithelial lymphocytes without CD3 B. 2471 A. Dermatitis herpetiformis (DH) is ? B. Minutes to hours B. 9 months D. Sjögren’s syndrome. All of the above D. 2471 N Engl J Med 2012. IgA C. 2471. Type 2 is associated with a higher risk of ulcerative jejunoileitis and lymphoma than type 1. 2471 N Engl J Med 2012. N Engl J Med 2012. 10 to 50 mg per day Celiac disease is associated with dermatitis herpetiformis (DH). intestinal ulceration and collagenous sprue. AJAY MATHUR Gastroenterology D. New Codex Alimentarius regulation permits a maximum gluten contamination of 20 ppm in glutenfree products. IgA deficiency A. secondary lactase deficiency. Hours to days Celiac crisis is a rare life-threatening syndrome. 100 to 500 mg per day deficiency. and signs with villous atrophy detected on biopsy despite maintenance of a strict gluten free diet for bile acid malabsorption (due to bile acid–induced fluid secretion in the colon. Minutes to hours A. 337 Celiac sprue may be associated with following diseases except ? Harrison’s 18th Ed.367:2419-26 A. and D. Malignancy A. Autoimmune thyroid disease. N Engl J Med 2002. Steatorrhea A. celiac disease 335 Mechanism of diarrhea in celiac disease is ? is termed refractory ? Harrison’s 18th Ed. Fistulas D. Any of the above Complications of celiac sprue include gastrointestinal and nongastrointestinal neoplasms. the lowest amount of daily gluten that causes damage to celiac intestinal mucosa over time (gluten threshold) A. Bile acid malabsorption C.367:2419-26 A. Weeks to years severe diarrhea. bile acid (abnormal intraepithelial lymphocytes. Weeks to years Hashimoto’s thyroiditis (5%) or other autoimmune diseases (including autoimmune liver diseases. or both). 6 months C. Down’s syndrome (5%). Type 1 diabetes mellitus N Engl J Med 2012. Microscopic colitis. Infertility or recurrent abortion . type 1 diabetes (3 to 16%). N Engl J Med 2012. Minutes to hours D. Rheumatoid arthritis.367:2419-26 A. Clonal intraepithelial lymphocytes without CD8 C. CD3. and metabolic and electrolyte imbalances.5 to 2 times as high among women as among men and is increased among persons who have an affected first-degree relative (10 to 15%). characterized by C. Endogenous fluid secretion Refractory celiac disease can be classified as type 1 (normal intraepithelial lymphocytes) or type 2 Diarrhea in celiac sprue may be secondary to steatorrhea. Chronic pancreatitis B. 12 months The diarrhea in celiac disease may be secondary to steatorrhea (due to changes in jejunal mucosal Refractory celiac disease is diagnosed when there are persistent or recurrent malabsorptive symptoms function). 1000 to 2500 mg per day In celiac disease. mostly observed in children. secondary lactase deficiency (due to changes in jejunal brush border enzymatic function).416 MCQ’s FOR MEDICAL PROFESSIONALS BY PROF. Intestinal ulceration B. 334 Interval between exposure to gluten and onset of symptoms in Gluten sensitivity is ? 340 Complications associated with untreated celiac disease include ? N Engl J Med 2012.367:2419-26 Harrison’s 18th Ed. Weeks to years D. Turner’s syndrome. Metabolic and electrolyte imbalances B. diabetes mellitus type 1. and T-cell receptors. Sjögren’s syndrome. D. 330 Which of the following is true for refractory celiac disease type 2 ? 336 The diarrhea in celiac sprue is due to all except ? N Engl J Med 2012. and IgA nephropathy). Collagenous sprue C. the lowest amount of daily gluten that causes damage to celiac intestinal mucosa 338 Prevalence of celiac disease is increased in which of the over time (gluten threshold) is 10 to 50 mg per day (a 25 gram slice of bread contains ~1. All of the above D. Lipase deficiency C. and endogenous fluid more than 12 months. Hashimoto’s thyroiditis 332 Interval between exposure to gluten and onset of symptoms in B. Turner’s syndrome Celiac disease is ? N Engl J Med 2012.6 grams following conditions ? of gluten).346:181 331 In celiac disease. Bile acid malabsorption D. 3 months B. hypoproteinemia. Cholesterol gall stones hypoparathyroidism. Stasis due to impaired peristalsis Harrison’s 18th Ed. All of the above B. Direct communication between small & large intestine B. Essential fatty acid 350 Which of the following hormones has a role in the treatment of D. Broad-spectrum antibiotics C. Table 294-9 C. Allopurinol D. 352 Diagnosis of the bacterial overgrowth syndrome is done by all 346 Patients of Tropical sprue are rarely found in ? except ? Harrison’s 18th Ed. TSH B. B. Gluten-free diet Enteropathy-associated T-cell lymphoma and adenocarcinoma of the jejunum are rare complications of celiac disease. All of the above Harrison’s 18th Ed. stomatitis in malabsorption syndrome is A. Chronic diarrhea in a tropical environment is most often caused by G. South India A. neurologic disorders. impaired splenic function. Glucocorticoids 341 Endocrine and metabolic disorders that can cause malabsorption syndrome include ? D. cholesterol gallstones and Harrison’s 18th Ed. enteric hyperoxaluria. Yersinia enterocolitica C. Giardia lamblia Bacterial proliferation in bacterial overgrowth syndrome or stagnant bowel syndrome or blind loop D. hyperthyroidism & carcinoid syndrome. Table 294-8 Broad-spectrum antibiotics and folic acid are curative in tropical sprue. Iron D. Table 294-8 gastric hypersecretion of acid occurs. Cholecystokinin A. adrenal insufficiency. VIP Harrison’s 18th Ed. Entamoeba histolytica syndrome is due to stasis caused by impaired peristalsis (functional stasis). Glucagon-like peptide 2 (GLP-2) due to deficiency of ? B. AJAY MATHUR Cardiology 417 B. A. Cholestyramine A. or Southeast Asia. folate Potential effectiveness of hormones like glucagon-like peptide-2 (GLP-2). 2473 344 Glossitis. Vitamin A (EGF) & growth hormone (GH) is being studied in short bowel syndrome. and cancer. All of the above 351 Stagnant bowel or blind loop syndrome refers to ? 345 Chronic diarrhea in a tropical environment is most often caused Harrison’s 18th Ed. D. 2471 B. Low serum cobalamin level B. 2472 Harrison’s 18th Ed. difficile. B. Low serum folate level C. Aspirin 343 Dermatitis in malabsorption syndrome is due to deficiency of ? C. Philippines B. C. Constrictive pericarditis 349 Enteric hyperoxaluria is best treated with ? Harrison’s 18th Ed. Caribbean islands C. A. Philippines and Caribbean islands. Vitamin B12. Zinc C. Hypoparathyroidism B. but is rarely observed in Africa. 2476. 2472 C. Cryptosporidium parvum and Cyclospora cayetanensis. changes in intestinal anatomy (anatomic stasis) or direct communication between small and large intestine. Yersinia enterocolitica. Cryptosporidium parvum D. Codeine Harrison’s 18th Ed. 2473 by all except ? A. an anion-binding resin & calcium are useful in reducing hyperoxaluria. Gastric hypersecretion of acid 342 Circulatory disorders that can cause malabsorption syndrome D. Congestive heart failure B. Carcinoid syndrome Harrison’s 18th Ed. ulcerative jejunoileitis. cheilosis. Colonic diverticulosis Endocrine & metabolic disorders that can cause malabsorption syndrome include diabetes. Vitamin A Cholestyramine. 2472 Complications associated with untreated celiac disease include osteoporosis. B. epidermal growth factor C. Impaired splenic function Tropical sprue is found in South India. Changes in intestinal anatomy A. Table 294-9 D. C. All of the above 347 Treatment of Tropical sprue includes ? Harrison’s 18th Ed. 2472 D. 2476. 2473 A. lamblia. C. A. infertility or recurrent abortion. All of the above A. Hyperthyroidism 348 Diseases that may arise following small-intestinal resection include all except ? C. Hyperoxaluria include ? Following large resections of small intestine. All of the above short bowel syndrome ? Harrison’s 18th Ed. Jamaica. 417 Gastroenterology MCQ’s MCQ’s FOR FORPROFESSIONALS MEDICAL MEDICAL PROFESSIONALS BY PROF. Neurologic disorders D. Increased aerobic &/or anaerobic colonic-type bacteria in jejunal aspirate D. Africa . All of the above C. Mesenteric artery atherosclerosis A. 2475. 2475. low infectivity Harrison’s 18th Ed. Cognitive changes and later dementia are commonest neurologic signs. B. especially with dementia. Multisystem disease via portal vein. Schilling test A. Antibiotics for ~3 weeks false ? C. histolytica trophozoites is C. Chronic pericarditis is false ? C. Harrison’s 16th Ed. AJAY MATHUR Gastroenterology D. steatorrhea. Toxoplasma whippeli loss of protein into gastrointestinal tract. Gal/GalNAc–specific pepsin C. whipplei cannot be grown on culture mediated by ? D. PAS+ macrophages in small intestine A. Recurrence of of IBD ? disease activity. B. Cognitive change Diagnosis of the bacterial overgrowth syndrome is done by a low serum cobalamin level. LCFA D. 1-antitrypsin clearance can be useful in diagnosis Harrison’s 18th Ed. Excess protein loss into gastrointestinal tract In the presence of frequent recurrences. All of the above 356 Which of the following about Whipple’s disease is false ? Harrison’s 18th Ed.Inflammatory bowel disease C. Gal/GalNAc–specific trypsin 357 Which of the following about Whipple’s disease is false ? D. Antibiotics for 1 week per month 359 Which of the following about protein-losing enteropathy is B. Following tetracycline for 5 days.348:16 Presence of T. Drug of second choice is Chloramphenicol B. Schilling test. C. With B. None of the above 363 Which out of the following countries has the highest incidence If TMP/SMX is not tolerated. Supranuclear ophthalmoplegia serum folate level. chloramphenicol is an appropriate second choice.418 MCQ’s FOR MEDICAL PROFESSIONALS BY PROF. weight Harrison’s 18th Ed. Peripheral vascular disease 355 Which of the following statements about Tropheryma whippeli B. which do not exit from intestinal epithelial cells via lymphatics but are delivered to the body A. presence of lymphopenia in a patient with hypoproteinemia supports the presence of increased C. resulting in a relative lymphopenia. Treponema whippeli loss of protein. elevated B. whipplei is a small gram-positive. with low virulence but high infectivity. 2475 loss. 2474 Treatment of hypoproteinemia in intestinal lymphangiectasia is done by low-fat diet and administration of MCTs. 2474 A. 2475 D. Antibiotic therapy has to be prolonged 295 .356:55-66 C. SCFA Hallmark of Whipple’s disease is the presence of PAS-positive macrophages in small intestine. is an extremely poor prognostic sign. Disease recurrence with dementia is a poor prognostic sign D. Schilling test will become normal. Antibiotics continuously A. Greece . Antibiotics until symptoms remit Harrison’s 18th Ed. 2474 Whipple’s disease. 2475 D. B. actinobacterium bacillus. Hemolytic uremic syndrome Harrison’s 18th Ed. 2474 D. T. A. Oculomasticatory or oculofacialskeletal myorhythmia D. Low serum albumin & globulin in absence of renal & hepatic symptoms are present is most effective. Sweden 358 Which of the following is the most common neurologic B. use of antibiotics for 1 week per month whether or not B. Gal/GalNAc–specific capsin Harrison’s 18th Ed. whipplei bacillus outside of macrophages is a more important indicator of active A. Gram-positive B. disease 354 Whipple’s disease is caused by the bacteria named ? C. The current drug of choice is double-strength trimethoprim/sulfamethoxazole for ~1 year. Trenoderma whippeli 360 Which of the following can cause protein-losing enteropathy ? Whipple’s disease is a chronic multisystem disease that presents as diarrhea. Tropheryma whippeli Protein-losing enteropathy is characterized by excess protein loss into gastrointestinal tract. Low virulence. It is caused by bacteria Tropheryma whipplei. 2474 D. whipplei has been grown in culture. Hypothyroidism A. arthralgia & CNS and cardiac problems. A. Thus. Drug of first choice is TMP/SMX for 1 year N Engl J Med 2003. 1776 A. Gal/GalNAc–specific lectin disease than within macrophages. Israel manifestations of classic Whipple’s disease ? N Engl J Med 2007. Actinobacterium 361 Hypoproteinemia in intestinal lymphangiectasia should be treated with ? C. Lymphocytosis supports diagnosis A. Ataxia 353 For frequent recurrences of bacterial overgrowth syndrome. peripheral lymphocytes are also lost via lymphatics. D. C. T. increased levels of aerobic and/or anaerobic colonic-type bacteria in a jejunal aspirate. MCT T. T. whippelii outside macrophages indicates active disease 362 Adherence to colonic mucin by E. Movement disorders of eye which of the following treatment strategies is most effective ? muscles (oculomasticatory or oculofacioskeletal myorhythmia) are considered pathognomonic for Harrison’s 18th Ed. 419 Gastroenterology MCQ’s MCQ’s FOR FORPROFESSIONALS MEDICAL MEDICAL PROFESSIONALS BY PROF. AJAY MATHUR Cardiology 419 D. China UC and CD are associated with Turner’s syndrome and Hermansky-Pudlak syndrome, Wiskott- Aldrich syndrome and chronic granulomatous disease. Immunodeficiency disorders like United States, United Kingdom, Norway and Sweden have the highest rates. hypogammaglobulinemia, selective IgA deficiency and hereditary angioedema also have increased association with IBD. 364 Out of the following, which ethnic group has the highest prevalence of IBD ? 370 All of the following are predisposing factors for UC except ? Harrison’s 18th Ed. 2477 Harrison’s 16th Ed. 1776 A. Jewish A. Jewish ethnicity B. African American B. High socio-economic status C. Hispanic C. Smoking D. Asian D. Hermansky-Pudlak syndrome Prevalence of IBD in Ashkenazi Jews is ~twice that of Israeli-born, Sephardic, or Oriental Jews. 371 If two parents have IBD, what chance does each child has of Prevalence decreases progressively in non-Jewish Caucasian, African-American, Hispanic, and Asian populations. being affected ? Harrison’s 18th Ed. 2477 365 Nonmalignant mortality in IBD cases is maximum during ? A. 36 % Harrison’s 18th Ed. 2477 B. 45 % A. First year of disease C. 66 % B. After five years of disease D. 72 % C. After ten years of disease If two parents have IBD, each child has a 36% chance of being affected. D. After fifteen years of disease Highest mortality is during first years of IBD & in long-duration disease due to risk of colon cancer. 372 Which of the following is not associated with increased risk of UC ? 366 If a patient has IBD, the lifetime risk that a first degree relative Harrison’s 18th Ed. 2477 will be similarly affected is ? A. Urban domicile Harrison’s 18th Ed. 2477 B. High socio-economic status A. 10 % C. Former smoker B. 20 % D. Appendectomy C. 30 % Urban areas, high socioeconomic classes have a higher prevalence of IBD. Former smokers have D. 40 % a 1.7-fold increased risk for UC than people who have never smoked. Appendectomy is protective against UC. Lifetime risk of a first-degree relative of IBD patient is ~10%. If two parents have IBD, each child has a 36% chance of being affected. In twin studies, 58% of monozygotic twins are concordant for CD and 6% are concordant for UC, whereas 4% of dizygotic twins are concordant for CD and none are 373 Disease that are shares genetic risk factors with IBD is ? concordant for UC. Harrison’s 18th Ed. 2478 A. Rheumatoid arthritis 367 Which of the following statements about Ulcerative colitis (UC) and Crohn’s disease (CD) is false ? B. Psoriasis Harrison’s 18th Ed. 2477 C. Systemic lupus erythematosus A. Peak age of onset of UC & CD is 15 - 30 years D. All of the above B. Second peak occurs between 60 - 80 years Diseases and genetic risk factors that are shared with IBD include rheumatoid arthritis (TNFAIP3), C. Appendectomy aggravates UC psoriasis (IL23R,IL12B), ankylosing spondylitis (IL23R), type 1 diabetes mellitus (IL10,PTPN2), asthma (ORMDL3), and systemic lupus erythematosus (TNFAIP3,IL10). D. IBD runs in families 374 NOD1 gene is now known as ? Appendectomy is protective against UC but increases the risk of CD. A. CARD1 368 Which genetic disorder is a predisposing factor for IBD ? B. CARD2 Harrison’s 18th Ed. 2477 C. CARD3 A. Turner’s syndrome D. CARD4 B. Down syndrome C. Patau syndrome 375 NOD2 gene is now known as ? Harrison’s 17th Ed. 1886 D. Edward syndrome A. CARD1 369 IBD is associated with all of the following except ? B. CARD5 Harrison’s 18th Ed. 2477 C. CARD15 A. Turner’s syndrome D. CARD18 B. Down syndrome Gene CARD4 was formerly called NOD1, and CARD15 was formerly called NOD2. CARD15 means C. Selective IgA deficiency caspase-associated recruitment domain containing protein 15, while NOD2 refers to nucleotide oligomerisation domain 2. D. Hereditary angioedema 420 MCQ’s FOR MEDICAL PROFESSIONALS BY PROF. AJAY MATHUR Gastroenterology 376 The disease-related gene of IBD1 on chromosome 16 is ? 382 Which of the following genes is associated with endoplasmic Harrison’s 17th Ed. 1886 reticulum (ER) and metabolic stress ? A. NOD-1 Harrison’s 18th Ed. 2478 B. NOD-2 A. XBP1 C. NOD-3 B. ORMDL3 D. NOD-4 C. OCTN D. All of the above The IBD1 locus encodes NOD2 (also designated CARD 15). Genes that are associated with endoplasmic reticulum (ER) and metabolic stress are XBP1, 377 Which of the following IBD subtype has its loci on chromosome 16 ? ORMDL3, OCTN, which serve to regulate the secretory activity of cells involved in responses to the commensal microbiota such as Paneth and goblet cells and the manner in which intestinal cells Harrison’s 17th Ed. 1886 respond to the metabolic products of bacteria. A. IBD 6 B. IBD 7 383 Which of the following genes is associated with regulation of adaptive immunity ? C. IBD 8 Harrison’s 18th Ed. 2478 D. IBD 9 A. IL23R IBD loci are on chromosomes 16q12 (IBD1), 12q13 (IBD2), 6p13 (IBD3), 14q11 (IBD4), 5q31-33 B. IL12B (IBD5), 19p13 (IBD6), 1p36 (IBD7), 16p (IBD8), 3p (IBD9). C. IL10 378 CARD15 is constitutively expressed in ? D. All of the above Harrison’s 17th Ed. 1886 Genes that are associated with regulation of adaptive immunity are IL23R, IL12B, IL10, PTPN2, A. Paneth cells which regulate the balance between inflammatory and regulatory cytokines. B. Parietal or oxyntic cells 384 Which of the following genes is involved in the development C. G cells and resolution of inflammation ? D. D cells Harrison’s 18th Ed. 2478 Paneth cells are specialised epithelial cells selectively expressed in ileum, located mainly in crypts A. MST1 in close proximity to epithelial stem cells. Paneth cells secrete antibacterial substances. Main antimicrobial factors secreted by Paneth cell include lysozyme, phospholipase A2, trypsin, alpha- B. CCR6 defensins & angiogenins. C. TNFAIP3 379 CARD15 protein is expressed in the cytoplasm of ? D. All of the above Harrison’s 17th Ed. 1886 Genes that are involved in the development and resolution of inflammation are MST1, CCR6, TNFAIP3, PTGER4 and ultimately leukocyte recruitment and inflammatory mediator production. A. Peripheral blood monocytes B. Paneth cells 385 Lack of immune responsiveness of gut mucosal immune C. Dendritic cells system to dietary antigens is best related to ? Harrison’s 18th Ed. 2478 D. All of the above A. Intestinal tolerance CARD15 is expressed by intestinal epithelial cells (Paneth cells, monocytes, macrophages & dendritic cells). B. Gastric tolerance 380 Which of the following is false about CARD15 protein ? C. Oral tolerance Harrison’s 17th Ed. 1886 D. All of the above A. CARD15 was formerly called NOD1 The gut mucosal immune system is normally unreactive to luminal contents due to oral (mucosal) B. Intracellular recognition protein for bacterial components tolerance. In IBD this suppression of inflammation is altered, leading to uncontrolled inflammation. C. Activates nuclear factor kappa B pathway 386 CD4+ T helper (TH) cell that promotes inflammation is ? D. Gain-of-function mutations are associated with CD Harrison’s 18th Ed. 2479 CARD15 protein acts as an intracellular recognition protein for bacterial components such as A. TH1 cells peptidoglycans and activates nuclear factor kappa B pathway which regulates apoptosis and inflammation. Loss-of-function mutations in CARD15 are highly associated with CD B. TH2 cells C. TH17 cells 381 Which of the following genes is associated with innate immunity and autophagy ? D. All of the above Harrison’s 18th Ed. 2478 CD4+ T helper (TH) cells that promote inflammation are of three major types namely TH1 cells, TH2 cells and TH17 cells. A. NOD2 B. ATG16L1 387 Which of the following is a type of CD4+ T cell ? C. IRGM Harrison’s 18th Ed. 2479 D. All of the above A. TH1 cells Genes that are associated with innate immunity and autophagy are NOD2, ATG16L1, IRGM, JAK2, B. TH2 cells STAT3 that function in innate immune cells (both parenchymal and hematopoietic) to respond to and C. TH17 cells clear bacteria, mycobacteria and viruses. D. All of the above 421 Gastroenterology MCQ’s MCQ’s FOR FORPROFESSIONALS MEDICAL MEDICAL PROFESSIONALS BY PROF. AJAY MATHUR Cardiology 421 CD4+ T cells are of three major types : TH1 cells, TH2 cells and TH17 cells. 394 Levels of which of the following is elevated in IBD ? N Engl J Med 2009;361:2066-78 388 TH2 of CD4 + T cells produce all except ? A. Tumor necrosis factor  (TNF-) Harrison’s 18th Ed. 2479 B. Interleukin-1 A. Interferon- (IFN-) C. Interferon- B. Interleukin -4 (IL-4) D. All of the above C. Interleukin -5 (IL-5) Levels of tumor necrosis factor  (TNF-  ), interleukin-1  , interferon-  , and cytokines of the D. Interleukin -13 (IL-13) interleukin-23–Th17 pathway are elevated in IBD. CD4+ T cells are composed of Th1 cells, that produce interferon-gamma (IFN-), and Th2 cells that produce interleukin-4, IL-5 and IL-13. 395 In unstimulated cells, NF-B is found in ? N Engl J Med 1997;336:1067 389 Interferon (IFN) gamma is secreted by which of the following ? A. Nucleus Harrison’s 18th Ed. 2479 B. Cytoplasm A. TH1 cells C. Cell membrane B. TH2 cells D. All of the above C. TH17 cells NF-B (nuclear factor kappa-light-chain-enhancer of activated B cells) is located in the cytosol D. All of the above complexed with the inhibitory protein IB. TH1 cells secrete interferon (IFN) gamma, TH2 cells secrete IL-4, IL-5, IL-13, and TH17 cells secrete IL-17, IL-21. 396 In unstimulated cells, NF- B remains bound to ? N Engl J Med 1997;336:1067 390 Which of the following cells induce transmural granulomatous A. G-B inflammation that resembles CD ? B. H-B Harrison’s 18th Ed. 2479 C. I- B A. TH1 cells D. J- B B. TH2 cells In unstimulated cells, the NF- B dimers are sequestered in cytoplasm by a family of inhibitors, C. TH17 cells called IB (Inhibitor of B). Activation of NF-B is initiated by signal-induced degradation of I B D. All of the above which occurs via activation of a kinase called the IB kinase (IKK). With degradation of IB inhibitor, NF-B complex is freed to enter the nucleus where it activates expression of genes that have DNA- binding sites for NF-B leading to physiological responses like inflammatory or immune response, TH1 cells induce transmural granulomatous inflammation that resembles CD. cell survival response, cellular proliferation and activating expression of its own repressor, IB. IB then re-inhibits NF-B (auto feedback loop). 391 Which of the following cells induce superficial mucosal inflammation resembling UC ? 397 Which of the following can activate NF- B ? Harrison’s 18th Ed. 2479 N Engl J Med 1997;336:1067 A. TH1 cells A. Cytokines B. TH2 cells B. Activators of protein kinase C C. TH17 cells C. Viruses D. All of the above D. All of the above TH2 cells, & related natural killer T cells that secrete IL-13 induce superficial mucosal inflammation NF-B is a rapidly acting primary transcription factor found in all cell types. It is activated activated resembling UC. TH17 cells may be responsible for neutrophilic recruitment. through TLR-4 by cytokines, activators of protein kinase C, bacteria / viruses, stress, UV. 392 The TH1 cytokine pathway is initiated by ? 398 Which of the following is not a probiotic ? Harrison’s 18th Ed. 2479 Harrison’s 18th Ed. 2480 A. IL-4 A. Lactobacillus sp. B. IL-6 B. Taenia suis C. IL-12 C. Campylobacter sp. D. IL-23 D. Saccharomyces boulardii The TH1 cytokine pathway is initiated by IL-12. IL-4 and IL-23, together with IL-6 and TGF-beta, Salmonella sp., Shigella sp., Campylobacter sp., Clostridium difficile are pathogens that may induce TH2 and TH17 cells, respectively. initiate IBD by triggering an inflammatory response. While Faecalibacterium prausnitzii, Lactobacillus, Bifidobacterium, Taenia suis, and Saccharomyces boulardii are probiotics that may inhibit 393 Which of the following is also called “lymphotoxin” ? inflammation. A. TNF- 399 What proportion of UC patients have involvement of whole colon ? B. TNF- Harrison’s 18th Ed. 2480 C. IFN- A. 10 % D. IFN- B. 20 % Tumor-necrosis factor-beta (TNF- ) is also known as lymphotoxin. C. 30 % D. 40 % 422 MCQ’s FOR MEDICAL PROFESSIONALS BY PROF. AJAY MATHUR Gastroenterology ~40-50% of UC patients have disease limited to rectum & rectosigmoid, 30-40% have disease 405 Which of the following statements about pathology of Crohn’s extending beyond sigmoid but not involving whole colon. 20% have a total colitis. disease (CD) is false ? Harrison’s 18th Ed. 2480 400 Backwash ileitis occurs when which part of colon is involved A. Perirectal fistulas, abscesses & anal stenosis common in UC ? Harrison’s 18th Ed. 2480 B. Never involves liver & pancreas A. Ascending colon C. Transmural process B. Transverse colon D. “Cobblestone” appearance on endoscopy C. Descending colon In CD, granulomas are seen in lymph nodes, mesentery, peritoneum, liver & pancreas. D. Whole colon 406 “Cobblestone mucosa” in CD is mainly seen in ? UC is a mucosal disease that almost always involves rectum and extends proximally to involve Harrison’s 18th Ed. 2481 colon in continuity without areas of uninvolved mucosa. When “whole colon” is involved, A. Sigmoid colon inflammation extends 1-2 cm into terminal ileum in 10–20% of patients. This is called backwash ileitis. B. Transverse colon C. Ileum 401 Psychosocial factors can contribute to worsening of symptoms in which of the following illnesses ? D. Rectum Harrison’s 17th Ed. 1887 Unlike UC, CD is a transmural process. CD is segmental with skip areas in the midst of diseased intestine. In CD, stellate ulcerations fuse longitudinally & transversely to demarcate islands of A. IBD mucosa that are histologically normal. This “cobblestone” appearance is characteristic of CD, endoscopically and by barium radiography. Oral mucosal lesions in CD include aphthous stomatitis B. Chronic back pain & “cobblestone” lesions of the buccal mucosa. C. Amyotrophic lateral sclerosis 407 Which of the following statements about pathology of Crohn’s D. All of the above disease (CD) is false ? Harrison’s 18th Ed. 2480 402 Which of the following statements about pathology of Ulcerative colitis (UC) is false ? A. Earliest lesions are aphthoid ulcerations & focal crypt Harrison’s 18th Ed. 2480 abscesses A. Mucosal disease involving rectum and colon B. Caseating granulomas in all bowel wall layers common B. Proximal spread occurs in continuity C. Granulomas can be seen in liver & pancreas C. Sandpaper like appearance of mucosa reflects severe D. Granulomas can be seen in lymph nodes, mesentery, inflammation peritoneum D. Inflammatory polyps (pseudopolyps) are due to epithelial In CD, earliest lesions are aphthoid ulcerations & focal crypt abscesses with loose aggrega-tions of macrophages, forming “noncaseating” granulomas in all layers of bowel wall. regeneration In UC, with “mild inflammation” mucosa is erythematous & has a fine granular surface that looks like 408 In CD, “Creeping fat” is the term given to ? sandpaper. Harrison’s 17th Ed. 1888 A. Excessive enlargement of omentum 403 Which of the following statements about pathology of Ulcerative colitis (UC) is false ? B. Necrosis of omentum Harrison’s 18th Ed. 2480, Figure 295-2 C. Projections of thickened mesentery encasing bowel A. Limited to mucosa & superficial submucosa D. Fatty adhesions B. Crypt architecture of colon is preserved Projections of thickened mesentery encase the bowel is termed as “creeping fat”. C. Basal lymphoplasmacytosis suggests chronicity 409 Which of the following is not a major symptom of UC ? D. Cryptitis and crypt atrophy Harrison’s 18th Ed. 2481 UC process is limited to mucosa and superficial submucosa, with deeper layers unaffected A. Rectal bleeding except in fulminant disease. Histologic features that suggest chronicity are that crypt architecture of colon is distorted and presence of basal plasma cells and multiple basal B. Constipation lymphoid aggregates. C. Crampy abdominal pain 404 Which of the following statements about pathology of Crohn’s D. Tenesmus disease (CD) is false ? The major symptoms of UC are diarrhea, rectal bleeding, tenesmus, passage of mucus, and crampy Harrison’s 18th Ed. 2480 abdominal pain. Other symptoms in moderate to severe disease include anorexia, nausea, vomiting, A. Can affect any part of gastrointestinal tract fever, and weight loss. B. Terminal ileum involved in 90% of patients 410 Which of the following statements about clinical features of C. Rectum is always involved in CD Ulcerative colitis (UC) is false ? Harrison’s 18th Ed. 2481 D. Segmental with skip areas A. Tenesmus is a major symptom Unlike UC, which almost always involves the rectum, the rectum is often spared in CD. B. Severity of symptoms do not correlate with extent of disease 423 Gastroenterology MCQ’s MCQ’s FOR FORPROFESSIONALS MEDICAL MEDICAL PROFESSIONALS BY PROF. AJAY MATHUR Cardiology 423 C. Constipation seen in patients with distal disease 417 Which of the following is false about “fecal calprotectin” ? Harrison’s 17th Ed. 1889 D. Diarrhea is often nocturnal and/or postprandial A. Calcium binding Severity of symptoms correlates with the extent of disease. B. Has antimicrobial properties 411 Which is not a manifestion severe UC ? C. Predict relapses in IBD Harrison’s 18th Ed. 2482, Table 295-4 D. None of the above A. Bowel movements 4-6/day Fecal Calprotectin is a neutrophilic cytosolic protein, first isolated from granulocytes by Fagerhol B. Mean body temperature >37.5ºC and named L1 protein. It was renamed calprotectin upon identification of its calcium binding, antimicrobial properties and resistance to colonic bacterial degradation. Calprotectin, although C. Mean pulse rate > 90 / minute present in blood, enters bowel lumen as part of an inflammatory process. It is positive in stools of IBD and colorectal cancer patients. Fecal calprotectin & lactoferrin is useful in predicting impending D. ESR > 30 mm in first hour clinical relapse in CD & UC patients. In severe UC, bowel movements are >6 per day. 418 Which of the following about investigative features of 412 In severe ulcerative colitis, number of bowel movements per Ulcerative colitis (UC) is false ? day is ? Harrison’s 18th Ed. 2482 Harrison’s 18th Ed. 2482, Table 295-4 A. Barium enema shows fine mucosal granularity A. 2-3 B. “Collar-button” ulcers seen in barium enema B. 3-4 C. Loss of haustration occurs in early disease C. 4-5 D. Colon can become short & narrowed D. >6 Loss of haustration occur in patients with long-standing disease. “Collar-button” ulcers indicate deep ulcerations with penetration into mucosa. 413 Which of the following about investigative features of Ulcerative colitis (UC) is false ? 419 Which of the following statements about complications of Harrison’s 17th Ed. 1889 Ulcerative colitis (UC) is false ? A. CRP levels may rise in active disease Harrison’s 18th Ed. 2482 B. Orosomucoid levels may rise in active disease A. Toxic megacolon means a transverse colon with diameter > C. Leukocytosis is a specific indicator of disease activity 6 cm, with loss of haustration D. Proctitis or proctosigmoiditis rarely cause rise in CRP B. Toxic megacolon can be triggered by electrolyte abnormalities and narcotics In UC, leukocytosis occurs but is not a specific indicator of disease activity. Proctitis or proctosigmoiditis rarely causes a rise in CRP. C. In toxic colitis and severe ulcerations, bowel may perforate without first dilating 414 Which of the following is a highly sensitive & specific marker D. UC patients never develop anal fissures, perianal abscesses for detecting intestinal inflammation ? Harrison’s 18th Ed. 2482 UC patients occasionally develop anal fissures, perianal abscesses, or hemorrhoids, but occurrence of extensive perianal lesions should suggest CD. A. Fecal lactoferrin B. Fecal transferrin 420 Which of the following statements about clinical features of C. Fecal hemolysin Crohn’s disease (CD) is false ? Harrison’s 18th Ed. 2483 D. Fecal reactin A. Can be of fibrostenotic-obstructing or penetrating-fistulous Fecal lactoferrin is a highly sensitive and specific marker of fecal leukocytes and for detecting pattern intestinal inflammation, tested by latex agglutination & enzyme-linked immunosorbent assays. B. Most common site of inflammation is caecum 415 Which of the following about fecal calprotectin is false ? C. Presentation may mimic acute appendicitis Harrison’s 18th Ed. 2482 D. Radiographic “string sign” refers to narrowed intestinal lumen A. Levels correlate well with histologic inflammation In CD, the most common site of inflammation is terminal ileum. B. Predict relapses C. Detects pouchitis 421 Radiographic “String sign” in CD is due to ? D. None of the above Harrison’s 18th Ed. 2483 A. Oedema of bowel wall Fecal calprotectin levels correlate well with histologic inflammation, predict relapses, and detect pouchitis. B. Thickening of bowel wall C. Fibrosis of bowel wall 416 Which of the following is false about “fecal calprotectin” ? Harrison’s 17th Ed. 1889 D. All of the above A. Neutrophilic cytosolic protein Transmural inflammation of CD causes decreased luminal diameter & limited distensibility. Radiographic “string sign” represents long areas of circumferential inflammation & fibrosis causing B. Resistant to colonic bacterial degradation long segments of luminal narrowing. Edema, bowel wall thickening & fibrosis of bowel wall account for the radiographic “string sign” of a narrowed intestinal lumen. C. Formerly called L1 protein D. None of the above 424 MCQ’s FOR MEDICAL PROFESSIONALS BY PROF. AJAY MATHUR Gastroenterology 422 Which of the following is not a feature of pain in CD ? In CD, intestinal malabsorption can cause anemia, hypoalbuminemia, hypocalcemia, Harrison’s 18th Ed. 2483 hypomagnesemia, coagulopathy, hyperoxaluria, vitamin D deficiency and vitamin B12 deficiency. A. Colicky 428 Diarrhea in active CD is caused by ? B. Precedes defecation Harrison’s 18th Ed. 2483 C. Relieved by defecation A. Bacterial overgrowth D. Continues after defecation B. Bile-acid malabsorption Pain is colicky, precedes and is relieved by defecation. C. Intestinal inflammation D. All of the above 423 Right lower quadrant abdominal inflammatory mass in CD is composed of ? Causes of diarrhea in active CD include bacterial overgrowth in obstructive stasis or fistulization, Harrison’s 18th Ed. 2483 bile-acid malabsorption due to diseased or resected terminal ileum & intestinal inflammation with decreased water absorption, increased secretion of electrolytes and decreased rectal compliance. A. Inflamed bowel B. Adherent and indurated mesentery 429 Which of the following about CD is false ? Harrison’s 18th Ed. 2483 C. Enlarged abdominal lymph nodes A. Epigastric pain D. All of the above B. H. pylori negative gastritis Abdominal mass in CD is composed of inflamed bowel, adherent and indurated mesentery, and enlarged abdominal lymph nodes. C. Second part of duodenum more commonly involved D. None of the above 424 Radiographic “string sign” of a narrowed intestinal lumen in CD is due to ? 430 Which of the following statements about investigations of Harrison’s 18th Ed. 2483 Crohn’s disease (CD) is false ? A. Edema of bowel wall Harrison’s 18th Ed. 2483 B. Bowel wall thickening A. Elevated ESR C. Fibrosis of bowel wall B. Elevated CRP D. All of the above C. Leucopenia Edema, bowel wall thickening, and fibrosis of the bowel wall within the mass account for the D. Hypoalbuminemia radiographic “string sign” of a narrowed intestinal lumen. “String sign” represents long areas of Laboratory abnormalities in CD include elevated ESR & CRP, hypoalbuminemia, anemia & leukocytosis. circumferential inflammation and fibrosis, resulting in long segments of luminal narrowing. 425 Factors recognized to exacerbate CD include ? 431 Endoscopic features of Crohn’s disease (CD) is false ? Harrison’s 18th Ed. 2483 Am J Gastroenterol 2009; 104:465-483 A. Intercurrent infections A. Rectal sparing B. Cigarette smoking B. Aphthous ulcerations & skip lesions C. Non-steroidal anti-infl ammatory drugs C. Fistulas D. All of the above D. None of the above Endoscopic features of CD include rectal sparing, aphthous ulcerations, fistulas, and skip lesions. Factors recognized to exacerbate CD include intercurrent infections (both upper respiratory tract & enteric infections, including C. difficile), cigarette smoking & non-steroidal anti-inflammatory drugs. 432 Earliest macroscopic findings of colonic CD is ? 426 Which of the following statements about clinical features of Harrison’s 18th Ed. 2483 Crohn’s disease (CD) is false ? A. Aphthous ulcers Harrison’s 18th Ed. 2483 B. Longitudinal stellate, serpiginous, & linear ulcers A. Pneumaturia may be present C. Fistulas B. Fecaluria may be present D. Polyps C. Enterocutaneous fistulas may be present Earliest macroscopic findings of colonic CD are aphthous ulcers which are small, multiple ulcers D. Enterocholedocal fistulas may be present separated by normal intervening mucosa. Severe inflammation of ileocecal region may lead to microperforation and fistula formation to adjacent bowel, skin, urinary bladder, to abscess cavity in mesentery, or vagina. 433 Which out of the following is the first-line test for evaluation of suspected CD and its complications ? 427 In Crohn’s disease (CD), intestinal malabsorption can cause Harrison’s 18th Ed. 2484 all except ? A. USG Harrison’s 18th Ed. 2483 B. MRI A. Hyperoxaluria C. CT enterography B. Hypocalcemia D. Endoscopy C. Hypomagnesemia CT enterography permits visualization of entire small bowel for inflammation associated with CD by D. Hypokalemia displaying mural hyperenhancement, interloop sinus tracts, mesenteric fat stranding, engorged 425 Gastroenterology MCQ’s MCQ’s FOR FORPROFESSIONALS MEDICAL MEDICAL PROFESSIONALS BY PROF. AJAY MATHUR Cardiology 425 vasa recta and perienteric inflammatory changes. MRI may superior for demonstrating pelvic B. I2 (+) patients more likely to have fibrostenosing disease lesions like ischiorectal abscesses. C. Anti-CBir1 expression is associated with small-bowel disease, 434 Which of the following statements about complications of fibrostenosing, and internal penetrating disease Crohn’s disease (CD) is false ? D. None of the above Harrison’s 18th Ed. 2484 A. Serosal adhesions common Omp C–positive patients are more likely to have internal perforating disease; and I2 positive patients are more likely to have fibrostenosing disease. Anti-Cbir1 expression is associated with small-bowel B. Free perforation common in duodenum disease, fibrostenosing, and internal penetrating disease. C. Systemic glucocorticoid therapy increase risk of 440 Cases of IBD that cannot be categorized as UC or CD are intraabdominal & pelvic abscesses termed ? D. Perianal disease common Harrison’s 18th Ed. 2485 Perforation occur usually in ileum but occasionally in jejunum. A. Indeterminate colitis B. Lymphocytic colitis 435 Which of the following antibody reactivity to antigens is C. Diversion colitis associated with CD ? Harrison’s 18th Ed. 2485 D. Collagenous colitis A. Bacterial flagellin (CBir1) Cases of IBD that cannot be categorized as UC or CD are called indeterminate colitis. IBD cases cannot be distinguished between UC & CD in ~15% of cases. B. Outer membrane porin C (OmpC) C. Bacterial sequence I2 (anti-I2) 441 Which of the following is independently associated with D. All of the above disabling CD after 5 years ? Harrison’s 18th Ed. 2485 Presence of antibody reactivity to antigens like oligomannan (ASCA, anti-Saccharomyces cerevisiae antibody), E. coli outer membrane porin C (OmpC), bacterial (Pseudomonas fluorescens) sequence A. Age at diagnosis below 40 years I2 (anti-I2), and bacterial flagellin (CBir1) are associated with CD. B. ASCA positivity 436 Autoantibody found in cases of UC is ? C. pANCA positivity Harrison’s 18th Ed. 2485 D. Rectal involvement A. Anti dsDNA Features of CD that have been shown to be independently associated with subsequent disabling CD B. cANCA after 5 years are initial requirement for glucocorticoid use, age at diagnosis below 40 years and presence of perianal disease at diagnosis. C. pANCA D. Antimitochondrial 442 All of the following are true for CD except ? Harrison’s 18th Ed. 2486, Table 295-5 pANCA positivity is found in about 60 - 70% of UC patients and 5 - 10% of CD patients. A. Abdominal mass 437 Which of the following statements about serological diagnosis B. pANCA positivity of IBD is false ? C. Response to antibiotics Harrison’s 18th Ed. 2485 D. Recurrence after surgery A. pANCA (+) in 5-10% of UC & 60-70% of CD patients B. 5-15% of Iº relatives of UC patients are pANCA (+) 443 Which of the following can cause proctitis ? Harrison’s 18th Ed. 2486 C. pANCA positivity is associated with pancolitis, early surgery, A. Gonorrhea and primary sclerosing cholangitis B. Chlamydia D. pANCA in CD is associated with colonic disease C. Syphilis pANCA positivity is found in about 60–70% of UC patients and 5–10% of CD patients. D. All of the above 438 Which of the following about ASCA in IBD is false ? Gonorrhea, Chlamydia, and syphilis can cause proctitis. Harrison’s 18th Ed. 2485 A. ASCA is associated with large bowel CD 444 Which of the following can cause watery diarrhea ? Harrison’s 18th Ed. 2486 B. 60-70% of CD, 10-15% of UC patients are ASCA (+) A. Salmonella C. ASCA positivity is associated with increased & early CD complications B. Shigella D. Saccharomyces cerevisiae is Brewer’s or Baker’s yeast. C. C. difficile D. All of the above The presence of both Anti-Saccharomyces cerevisiae antibodies (ASCA) IgG and IgA is highly specific for the presence of Crohn’s disease. ASCA is the most sensitive serologic marker of Apart from above three, the main symptom in Collagenous colitis is chronic watery diarrhea. Crohn’s disease. Saccharomyces cerevisiae is Brewer’s or Baker’s yeast. 439 Which of the following about serology of CD is false ? 445 Parasitic infections that mimic IBD include all except ? Harrison’s 18th Ed. 2486, Table 295-6 Harrison’s 18th Ed. 2485 A. Omp C (+) patients more likely to have internal perforating A. Necator americanus disease B. Trichuris trichiura 426 MCQ’s FOR MEDICAL PROFESSIONALS BY PROF. AJAY MATHUR Gastroenterology C. Strongyloides stercoralis PG occurs years before the onset of bowel symptoms, is independent of bowel disease & respond poorly to colectomy, usually associated with severe disease. D. Enterobius vermicularis Parasitic infections that may mimic IBD include hookworm (Necator americanus), whipworm (Trichuris 452 Which of the following is false about Pyoderma gangrenosum trichiura), Strongyloides stercoralis, Isospora belli and Entamoeba histolytica. (PG) ? Harrison’s 18th Ed. 2487 446 Which of the following IBD patients are at higher risk for developing extraintestinal manifestations ? A. Found on dorsal surface of feet and legs Harrison’s 16th Ed. 1783 B. Usually begins as a macule A. IBD with Selective IgA deficiency C. May be single or multiple B. IBD with granulomas D. Difficult to treat C. Perianal Crohn’s disease PG usually begins as a pustule and then spreads concentrically to rapidly undermine healthy skin. D. IBD with Turner’s syndrome 447 Patients with inflammatory bowel disease are at risk for ? 453 Dermatologic manifestations of IBD include all except ? N Engl J Med 2009;361:2066-78 Harrison’s 18th Ed. 2487 A. Primary sclerosing cholangitis A. Erythema nodosum (EN) B. Ankylosing spondylitis B. Pyoderma gangrenosum (PG) C. Psoriasis C. Pyoderma vegetans D. All of the above D. Erythema marginatum Patients with IBD are at risk for primary sclerosing cholangitis, ankylosing spondylitis, and psoriasis. 454 Dermatologic manifestations of IBD include all except ? Harrison’s 18th Ed. 2487 448 Which of the following is false about collagenous colitis ? A. Pyostomatitis vegetans Harrison’s 18th Ed. 2487 A. Increased subepithelial collagen deposition B. Alopecia B. Male to female ratio is 9:1 C. Perianal skin tags C. Most patients present in 6th or 7th decades D. Psoriasis D. Main symptom is chronic watery diarrhea 455 Which of the following skin lesions is most frequent in CD ? Harrison’s 18th Ed. 2487 Female to male ratio is 9:1. A. Erythema nodosum (EN) 449 Which of the following is false about Erythema nodosum (EN) B. Pyoderma gangrenosum (PG) in IBD ? Harrison’s 18th Ed. 2487 C. Psoriasis A. Correlate with bowel activity D. Perianal skin tags B. Develop after onset of bowel symptoms Perianal skin tags are found in 75 - 80% of patients with CD, especially those with colon involvement. C. Concomitant active peripheral arthritis 456 Peripheral arthritis that develops in IBD patients involves which D. None of the above of the following joints ? EN occurs in ~15% of CD and 10% of UC patients. Attacks correlate with bowel activity, develop Harrison’s 18th Ed. 2488 after onset of bowel symptoms, and frequently have concomitant active peripheral arthritis. A. Small joints of upper & lower extremities 450 Which of the following is false about Erythema nodosum ? B. Large joints of upper & lower extremities Harrison’s 18th Ed. 2487 C. Small joints of upper extremity A. Erythematous tender nodules D. Small joints of lower extremity B. 1-5 cm in diameter Peripheral arthritis of IBD patients is asymmetric, polyarticular and migratory and affects large joints C. Found on anterior surface of lower legs and arms of upper and lower extremities. D. None of the above 457 Rheumatologic manifestations of IBD include all except ? Lesions of EN are hot, red, tender nodules measuring 1–5 cm in diameter and found on anterior Harrison’s 18th Ed. 2488 surface of lower legs, ankles, calves, thighs and arms. A. Peripheral arthritis 451 Which of the following is false about Pyoderma gangrenosum B. Ankylosing spondylitis (AS) (PG) in IBD ? C. Osteopetrosis Harrison’s 18th Ed. 2487 A. May occur years before onset of bowel symptoms D. Sacroiliitis B. Independent of the bowel disease 458 Rheumatologic manifestations of IBD include all except ? C. Respond poorly to colectomy Harrison’s 18th Ed. 2488 D. None of the above A. Hypertrophic osteoarthropathy 427 Gastroenterology MCQ’s MCQ’s FOR FORPROFESSIONALS MEDICAL MEDICAL PROFESSIONALS BY PROF. AJAY MATHUR Cardiology 427 B. Pelvic/femoral osteomyelitis 465 Cardiopulmonary manifestations of IBD include all except ? Harrison’s 18th Ed. 2489 C. Relapsing polychondritis A. Endocarditis / Myocarditis D. Costochondritis B. Pleuropericarditis 459 Which of the following skeletal disease occurs equally in UC C. Interstitial lung disease (ILD) and CD ? Harrison’s 18th Ed. 2488 D. Obstructive lung disease A. Peripheral arthritis In IBD, cardiopulmonary manifestations include endocarditis, myocarditis, pleuropericarditis, & ILD. B. Ankylosing spondylitis (AS) C. Sacroiliitis 466 Most common extraintestinal pulmonary complication of IBD is ? D. All of the above Harrison’s 18th Ed. 2489 Symmetric sacroiliitis occurs equally in UC & CD. Peripheral arthritis and ankylosing spondylitis are A. ILD more common in CD than UC. B. Pneumonitis 460 Ocular manifestations of IBD include all except ? C. Malignancy Harrison’s 18th Ed. 2488 D. Obstructive lung disease A. Conjunctivitis B. Anterior uveitis / iritis 467 Role of sulfasalazine is not clear in which of the following ? Harrison’s 18th Ed. 2489 C. Episcleritis A. Inducing remission in UC D. Retinitis B. Maintaining remission in UC Ocular manifestations of IBD include conjunctivitis, anterior uveitis/iritis, and episcleritis. C. Inducing remission in CD 461 Common hepatobiliary manifestations of IBD include all D. Maintaining remission in CD except ? Sulfasalazine and other 5-ASA agents are effective at inducing and maintaining remission in UC with Harrison’s 18th Ed. 2488 limited role in inducing remission in CD but no clear role in maintenance of CD. The most convincing A. Hepatic steatosis evidence for use of sulfasalazine is treatment of active CD involving colon. B. Pancreatitis 468 Name of the bond linking the sulfa and 5-ASA moieties in C. Cholelithiasis Sulfasalazine is ? Harrison’s 18th Ed. 2489 D. Primary sclerosing cholangitis (PSC) A. Azo bond Pancreatitis is a rare extraintestinal manifestation of IBD and results from duodenal fistulas, ampullary CD, gallstones, PSC, autoimmune pancreatitis and primary CD of the pancreas. B. Thio bond C. Tau bond 462 All are complications of UC except ? Harrison’s 16th Ed. 1780 D. Conn bond A. Hemorrhage Sulfasalazine is broken down in colon by bacterial azo reductases. They cleave the azo bond linking the sulfa and 5-ASA moieties. B. Malignant change C. Intusucception 469 With Sulfasalazine therapy, which of the following should be D. Polyposis supplemented ? Harrison’s 18th Ed. 2489 463 Urologic manifestations of IBD include all except ? A. Iron Harrison’s 18th Ed. 2488 B. Folic acid A. Calculi C. Vitamin B12 B. Ureteral obstruction D. Pyridoxine C. Ileal bladder fistulas Sulfasalazine can also impair folate absorption, and patients should be given folic acid D. Glomerulonephritis supplements. 464 Hypercoagulable state in IBD is due to all except ? 470 Sulfasalazine intolerable side effects are attributable to ? Harrison’s 16th Ed. 1784, Harrison’s 17th Ed. 1894 Harrison’s 18th Ed. 2489 A. Reactive thrombocytosis A. Sulfapyridine moiety B. Increased fibrinopeptide A, factor V, VIII & fibrinogen B. Sulfasialic moiety C. Decreased thromboplastin generation C. Sulfaphenol moiety D. Antithrombin III deficiency D. All of the above In IBD, risk of venous & arterial thrombosis increases even when disease is not active due to Allergic reactions or intolerable side effects of sulfasalazine are attributable to sulfapyridine abnormalities of platelet-endothelial interaction, hyperhomocysteinemia, alterations in coagulation moiety. cascade, impaired fibrinolysis, involvement of tissue factor-bearing microvesicles, disruption of normal coagulation system by autoantibodies, vasculitides and genetic predisposition. 428 MCQ’s FOR MEDICAL PROFESSIONALS BY PROF. AJAY MATHUR Gastroenterology 471 Hypersensitivity reactions to Sulfasalazine include all 477 Which of the following side effects of glucocorticoid therapy except ? in IBD is not related to dose & duration of therapy ? Harrison’s 18th Ed. 2489 Harrison’s 18th Ed. 2490 A. Hepatitis A. Abdominal striae B. Thrombocytosis B. Subcapsular cataract C. Worsening of colitis C. Osteonecrosis D. Reversible sperm abnormalities D. Emotional disturbances Hypersensitivity reactions of sulfasalazine include rash, fever, hepatitis, agranulocytosis, Most of these side effects of glucocorticoids, other than osteonecrosis, are related to dose and hypersensitivity pneumonitis, pancreatitis, worsening of colitis and reversible sperm duration of therapy. abnormalities. 478 First line treatment in active inflammatory, fistulizing and 472 Which out of the following mesalamine formulation has perianal CD is ? ethylcellulose coating ? Harrison’s 18th Ed. 2490 Harrison’s 18th Ed. 2489 A. 5-ASA A. Asacol B. Azathioprine B. Balsalazide C. Metronidazole C. Claversal D. Salicylates D. Pentasa Metronidazole is effective in active inflammatory, fistulous, and perianal CD and may prevent Pentasa is a mesalamine formulation that has an ethylcellulose coating to allow water absorption recurrence after ileal resection. into small beads containing mesalamine. 479 First line treatment in active inflammatory, fistulizing and 473 Which of the following is composed of two 5-ASA radicals perianal CD is ? linked by an azo bond ? Harrison’s 18th Ed. 2490 Harrison’s 18th Ed. 2489 A. 5-ASA A. Asacol B. Azathioprine B. Balsalazide C. Ciprofloxacin C. Claversal D. Salicylates D. Olsalazine Both ciprofloxacin and metronidazole antibiotics can be used as first-line drugs for short periods of Olsalazine is composed of two 5-ASA radicals linked by an azo bond, which is split in the colon by time in active inflammatory, fistulizing and perianal CD. bacterial reduction and two 5-ASA molecules are released. 480 Achilles tendinitis and rupture is associated with which of the 474 5-ASA agents act within what period of time ? following medicines ? Harrison’s 18th Ed. 2489 Harrison’s 18th Ed. 2490 A. 6 - 12 hours A. Azathioprine B. 1 - 7 days B. Glucocorticoids C. 2 - 4 weeks C. Ciprofloxacin D. 4 - 6 weeks D. Metronidazole As a general rule, 5-ASA agents act within 2 - 4 weeks. Ciprofloxacin has recently been associated with Achilles tendinitis and rupture. 475 Treatment of choice in severe UC is ? 481 Active end product of Azathioprine is ? Harrison’s 16th Ed. 1786 Harrison’s 18th Ed. 2490 A. 5-ASA A. 6-MP B. Azathioprine B. Thioinosinic acid C. Metronidazole C. Inosinic acid D. Oral glucocorticoid D. Thioguanic acid Azathioprine is converted to 6-MP, which is metabolized to active end product, thioinosinic acid, an 476 Which of the following is false regarding treatment of IBD ? inhibitor of purine ribonucleotide synthesis and cell proliferation. Harrison’s 18th Ed. 2490 A. Antibiotics have no role in treatment of active or quiescent UC 482 Which of the following drugs is used for postoperative prophylaxis of CD ? B. Glucocorticoids useful in maintenance therapy in UC & CD Harrison’s 18th Ed. 2490 C. 5-ASA effective in inducing remission in UC & CD A. 5-ASA D. 5-ASA useful in maintaining remission in UC B. Azathioprine Glucocorticoids play no role in maintenance therapy in UC or CD. 5-ASA is effective in inducing C. Metronidazole remission in UC & CD & maintaining remission in UC. Its role in remission maintenance in CD is not established. D. Salicylates 6-MP or azathioprine is effective for postoperative prophylaxis of CD. 429 Gastroenterology MCQ’s MCQ’s FOR FORPROFESSIONALS MEDICAL MEDICAL PROFESSIONALS BY PROF. AJAY MATHUR Cardiology 429 483 Which of the following statements about azathioprine use in D. Paresthesias IBD is false ? Creatinine elevation, hypertension, gingival hyperplasia, hypertrichosis, paresthesias, tremors, Harrison’s 18th Ed. 2490 headaches, and electrolyte abnormalities and seizures are common side effects of Cyclosporine A. Pancreatitis as a side effect is completely reversible on therapy. stopping drug 489 Seizures may complicate Cyclosporine therapy if there is ? B. Thiopurine methyltransferase is the enzyme responsible for Harrison’s 18th Ed. 2491 its metabolism A. Hypocholesterolemia C. No increased risk of cancer in IBD patients chronically taking B. Hypercholesterolemia azathioprine C. Hypertriglyceridemia D. Leukopenia is dose-related and delayed D. All of the above IBD patients treated with azathioprine/6-MP are at a fourfold increased risk of developing a lymphoma due to medications, underlying disease, or both. Seizures may complicate CSA therapy if the patient is hypomagnesemic or if serum cholesterol levels are <120 mg/dL. 484 Methotrexate (MTX) inhibits which of the following ? 490 Opportunistic infection that commonly occurs with Harrison’s 18th Ed. 2491 Cyclosporine therapy is ? A. Dihydrofolate reductase Harrison’s 18th Ed. 2491 B. HMG-CoA reductase A. Pneumocystis carnii infection C. Trypanothione reductase B. M. tuberculosis infection D. Fumarate reductase C. Klebsiella infection Methotrexate (MTX) inhibits dihydrofolate reductase, resulting in impaired DNA synthesis. Additional D. Pseudomonas infection anti-inflammatory properties may be related to decreased IL-1 production. Opportunistic infections, most often Pneumocystis carinii pneumonia, may occur with CSA combination immunosuppressive treatment. 485 Potential toxicities of Methotrexate include all except ? Harrison’s 18th Ed. 2491 491 Severe UC patients that are refractory to IV glucocortcoids, A. Leukopenia should be treated with ? B. Hepatic fibrosis Harrison’s 18th Ed. 2491 C. Hypersensitivity pneumonitis A. IV Cyclosporine D. Cholelithiasis B. Oral cyclosporine Potential toxicities of MTX include leukopenia, hepatic fibrosis and hypersensitivity pneumonitis C. Azathioprine (rarely). D. 6 Mercaptopurine 486 Which of the following drugs is a lipophilic peptide ? CSA is given intravenously in severe UC that is refractory to intravenous glucocorticoids. Harrison’s 18th Ed. 2491 A. Azathioprine 492 Tacrolimus has a mechanism of action similar to ? Harrison’s 18th Ed. 2491 B. Methotrexate A. Methotrexate C. Cyclosporine B. Cyclosporine D. Tacrolimus C. Mycophenolate mofetil Cyclosporine (CSA) is a lipophilic peptide with inhibitory effects on both the cellular and humoral immune systems. D. Thalidomide Tacrolimus is a macrolide antibiotic with immunomodulatory properties & mechanism of action 487 Cyclosporine (CSA) acts by inhibition of ? similar to CSA. Harrison’s 18th Ed. 2491 A. IL-1 493 Which of the following drugs is a macrolide antibiotic ? Harrison’s 18th Ed. 2491 B. IL-2 A. Azathioprine C. IFN- B. Methotrexate D. All of the above C. Cyclosporine CSA blocks production of IL-2 by T-helper lymphocytes. CSA binds to cyclophilin, and this complex inhibits calcineurin, a cytoplasmic phosphatase enzyme involved in activation of T cells. CSA also D. Tacrolimus indirectly inhibits B cell function by blocking helper T cells. Tacrolimus is a macrolide antibiotic with immunomodulatory properties similar to CSA. 488 Which of the following is not a side effect of Cyclosporine 494 First biologic therapy approved for Crohn’s disease was ? therapy ? Harrison’s 18th Ed. 2491 Harrison’s 18th Ed. 2491 A. Adalimumab A. Hypertension B. Infliximab B. Gingival hyperplasia C. Natalizumab C. Alopecia 430 MCQ’s FOR MEDICAL PROFESSIONALS BY PROF. AJAY MATHUR Gastroenterology D. Etanercept 501 Which of the following is not an indication of Infliximab therapy ? The first biologic therapy approved for Crohn’s disease was infliximab. Harrison’s 16th Ed. 1787 495 Infliximab is ? A. Steroid dependant CD Harrison’s 18th Ed. 2491 B. Fistulizing CD A. IgG1 chimeric monoclonal antibody against TNF C. Maintenance of remission in CD B. IgG1 fully human monoclonal antibody against TNF D. Mechanical small intestinal obstruction in CD C. IgG4 chimeric monoclonal antibody against TNF 502 Adalimumab is a ? D. P75 TNF receptor fusion protein Harrison’s 18th Ed. 2492 Infliximab is a chimeric mouse-human monoclonal antibody against TNF. It blocks TNF in serum and A. IgG1 chimeric monoclonal antibody against TNF at cell surface and lyses TNF-producing macrophages & T cells. B. IgG1 recombinant human monoclonal antibody against TNF 496 Anti-TNF monoclonal antibody useful in treatment of moderate C. IgG4 chimeric monoclonal antibody against TNF to severely active CD in patients is ? Harrison’s 18th Ed. 2491 D. P75 TNF receptor fusion protein A. Infliximab Less immunogenic than Infliximab, Adalimumab is a recombinant human monoclonal IgG1 antibody containing only human peptide sequences. Adalimumab binds TNF and neutralizes its function by B. Adalimumab blocking the interaction between TNF and its cell-surface receptor. C. Certolizumab pegol 503 4 and Which one of the following specifically blocks Integrin- D. All of the above prevents lymphocyte trafficking to the intestine in IBD ? Anti-TNF monoclonal antibodies, infliximab, adalimumab, and certolizumab pegol, are effective in Harrison’s 18th Ed. 2492 the treatment of moderate to severely active CD in patients who have not responded despite complete & adequate therapy with a corticosteroid or an immunosuppressive agent. A. Anti IL-12 P40 antibody B. Anti gamma interferon 497 Infusion reaction that occur with Infliximab therapy is due to ? C. Natalizumab Harrison’s 18th Ed. 2491 A. Antibody to Infliximab D. Basiliximab B. Concurrent Methotrexate therapy Humanized monoclonal antibody to alpha-4 integrin - Natalizumab inhibits lymphocyte trafficking and is effective in treatment of moderate to severely active CD who have had an inadequate C. Anti ds-DNA antibody response or are unable tolerate conventional CD therapies and anti-TNF monoclonal antibody therapy. D. Antinuclear factor The development of antibodies to infliximab (ATI) is associated with an increased risk of infusion 504 Natalizumab is a ? reactions and a decreased response to treatment. Harrison’s 18th Ed. 2492 A. IgG1 chimeric monoclonal antibody against TNF 498 Which one of the following is considered to be useful additional therapy with treatment of CD with Infliximab ? B. IgG1 recombinant human monoclonal antibody against TNF Harrison’s 18th Ed. 2491 C. IgG4 recombinant humanized immunoglobulin G4 antibody A. Oral steroids against 4 integrin B. 5-ASA D. P75 TNF receptor fusion protein C. Azathioprine Natalizumab is a recombinant humanized immunoglobulin G4 antibody against 4 integrin that is D. Elemental diet effective in the induction and maintenance of remission in CD patients. If infliximab is used episodically for flares, concomitant immunosuppression with AZA, 6-MP or 505 Natalizumab causes progressive multifocal leukoencephalopa- methotrexate in therapeutic doses is recommended to reduce clinical consequences of immunogenicity of chimeric antibodies. SONIC (Study of Biologic and Immunomodulator-Naïve Patients with Crohn’s thy (PML) by reactivation of which virus ? Disease) Trial compared infliximab plus azathioprine, infliximab alone and azathioprine alone in Harrison’s 18th Ed. 2492, Am J Gastroenterol 2009; 104:465.483 moderate-to-severe Crohn’s disease with infliximab plus azathioprine group showing best results and no greater adverse events. A. Hepatitis B B. Human JC polyoma virus 499 Which of the following is associated with infliximab therapy ? Harrison’s 18th Ed. 2491 C. Hepatitis C A. Lymphoma D. Cytomegalovirus (CMV) B. Leukemia Natalizumab reactivates the human JC polyoma virus, which can lead to progressive multifocal leukoencephalopathy (PML). C. New-onset psoriasis D. All of the above 506 Which one of the following helps in intestinal restitution and repair ? 500 Infliximab is not useful in ? Harrison’s 16th Ed. 1747 Harrison’s 17th Ed. 1896 A. TNF A. Crohn’s disease B. EGF B. Rheumatoid arthritis C. GM-CSF C. Ankylosing spondilitis D. IL-12 D. Primary sclerosing cholangitis 431 Gastroenterology MCQ’s MCQ’s FOR FORPROFESSIONALS MEDICAL MEDICAL PROFESSIONALS BY PROF. AJAY MATHUR Cardiology 431 507 Which of the following is false in females with IBD ? Patients should be in remission for 6 months before conceiving. Most CD patients can deliver Harrison’s 18th Ed. 2494 vaginally, but cesarean section may be the preferred route of delivery for patients with anorectal and perirectal abscesses and fistulas to reduce the likelihood of fistulas developing or extending into the A. Normal fertility rates in quiescent IBD episiotomy scar. B. Fallopian tubes scarred by inflammatory process of CD, especially on right side 513 Patients with CD have an increased risk of developing which of the following cancers ? C. Fallopian tubes scarred by inflammatory process of CD, Harrison’s 18th Ed. 2495 especially on left side A. Non-Hodgkin’s lymphoma D. Dyspareunia common B. Hodgkin’s lymphoma Fallopian tubes can be scarred by inflammatory process of CD, especially on right side because of the proximity of terminal ileum. C. Bronchogenic carcinoma D. Carcinoma cervix 508 In IBD, which of following drugs is safe for use in pregnancy ? Harrison’s 18th Ed. 2494 514 Patients with CD have an increased risk of developing which of A. Sulfasalazine the following cancers ? Harrison’s 18th Ed. 2495 B. Mesalamine A. Hodgkin’s lymphoma C. Balsalazide B. Endometrial carcinoma D. All of the above C. Bronchogenic carcinoma Sulfasalazine, mesalamine, and balsalazide are safe for use in pregnancy and nursing, but additional folate supplementation must be given with sulfasalazine. Topical 5-ASA agents are also safe during D. Squamous cell cancers pregnancy and nursing. Glucocorticoids are generally safe for use during pregnancy. Patients with CD may have an increased risk of non-Hodgkin’s lymphoma, leukemia, and myelodysplastic syndromes. Severe chronic, complicated perianal disease in CD patients may be 509 The safe antibiotics to use for short periods in pregnant CD associated with an increased risk of cancer in lower rectum and anal canal (squamous cell cancers). patients are all except ? Harrison’s 18th Ed. 2494 515 Invariably involved site in UC is ? Harrison’s 17th Ed. 1888 A. Ampicillin A. Sigmoid colon B. Cephalosporin B. Transverse colon C. Ciprofloxacin C. Ileum D. Metronidazole D. Rectum Safest antibiotics in CD in pregnancy for weeks, not months are ampicillin and cephalosporin. Metronidazole can be used in the second or third trimester. Ciprofloxacin causes cartilage lesions and should be avoided. 516 Which site is usually spared in CD ? Harrison’s 17th Ed. 1888 510 In treatment of IBD with pregnancy, which of the following drugs A. Sigmoid colon is contraindicated ? B. Transverse colon Harrison’s 18th Ed. 2494 C. Ileum A. 6-Mercaptopurine D. Rectum B. Azathioprine C. Methotrexate In UC rectum is almost always involved. Rectum is often spared in CD. D. Cyclosporine 517 Most common site of involvement in CD is ? Methotrexate is contraindicated in pregnancy and nursing. 6-MP and azathioprine pose minimal or Harrison’s 17th Ed. 1888 no risk during pregnancy. In severe IBD treated with IV CSA during pregnancy, 80% of pregnancies A. Sigmoid colon were successfully completed without development of renal toxicity, congenital malformations, or developmental defects. B. Transverse colon C. Ileum 511 In pregnancy, exacerbation in Crohn’s disease is seen in ? Harrison’s 16th Ed. 36 D. Rectum A. First trimester CD can affect any part of the gastrointestinal tract, from mouth to anus. In 75% of patients with small-intestinal disease, terminal ileum is involved in 90%. B. Second and third trimesters C. Postpartum period 518 Perforation in CD is mainly seen in ? D. All of the above Harrison’s 17th Ed. 1891 A. Sigmoid colon 512 In pregnancy, exacerbation in Ulcerative colitis is seen in ? B. Transverse colon Harrison’s 16th Ed. 36 A. First trimester C. Ileum B. Second trimester D. Rectum C. Third trimester In CD, perforation occurs in 1 - 2% of patients usually in the ileum, but occasionally in jejunum or as a complication of toxic megacolon. D. All of the above 432 MCQ’s FOR MEDICAL PROFESSIONALS BY PROF. AJAY MATHUR Gastroenterology 519 Before making a diagnosis of IBD, which of the following C. CA 19-9 infections should be ruled out ? D. CD30 Harrison’s 17th Ed. 967 A. Campylobacter 526 Gastrointestinal / liver diseases that can produce interstitial B. Arcobacter lung disease include ? Harrison’s 16th Ed. 1555 C. Helicobacter A. Crohn’s disease / ulcerative colitis D. All of the above B. Primary biliary cirrhosis Presentation of Campylobacter enteritis may mimic UC or CD. As Campylobacter enteritis is much more common than UC and CD among young adults, a diagnosis of IBD should not be made until C. Chronic active hepatitis Campylobacter infection has been ruled out. D. All of the above 520 Which virus is a predisposing factor for IBD ? Harrison’s 16th Ed. 1777 Chapter 296. Irritable Bowel Syndrome A. Paramyxovirus B. Orthomyxovirus 527 Which of the following is a criteria for the diagnosis of Irritable C. Arbovirus Bowel Syndrome (IBS) ? D. Hepatitis A Harrison’s 18th Ed. 2496 A. Paris 521 Diarrhoea in CD is due to ? Harrison’s 17th Ed. 1890 B. Rome A. Bacterial overgrowth C. Helsinki B. Bile acid malabsorption D. Warsaw C. Increased electrolyte secretion in bowels Rome II criteria is useful for the diagnosis of IBS. D. All of the above 528 Which of the following is not included in the Rome II criteria Diarrhea in CD is caused by bacterial overgrowth in obstructive stasis or fistulization, bile acid for the diagnosis of IBS ? malabsorption due to a diseased or resected terminal ileum and intestinal inflammation with decreased Harrison’s 18th Ed. 2496 water absorption & increased secretion of electrolytes. A. Improvement of abdominal pain/discomfort with defecation 522 Which of the following is false for colitis-associated colon B. Painless diarrhea or constipation cancer (CAC) ? Harrison’s 16th Ed. 1788 C. Onset associated with a change in frequency of stool A. CACs arise from an adenomatous polyp D. Onset associated with a change in form of stool B. Mean age of CAC is in thirties Rome II diagnostic criteria for IBS includes recurrent abdominal pain or discomfort at least 3 days per month in the last 3 months associated with two or more of the following improvement with C. CAC is distributed uniformly throughout colon defecation, onset associated with a change in frequency of stool and onset associated with a change in form or appearance of stool. D. APC gene mutations occur much later in CAC than sporadic colon cancer (SCC) 529 Which of the following about abdominal pain in IBS is false ? Harrison’s 18th Ed. 2496 523 Ulcerative colitis can be a complication of ? Harrison’s 16th Ed. 980 A. Sleep deprivation is usual A. Primary syphilis B. Exacerbated by eating B. Secondary syphilis C. Improved by passage of flatus or stools C. Tertiary syphilis D. Worsens during the premenstrual & menstrual phases D. All of the abovesyphilis Sleep deprivation is unusual. 524 Antihypertensive drug that can produce acute ulcerative colitis 530 Which of the following about diarrhea in IBS is false ? is ? Harrison’s 18th Ed. 2496 Harrison’s 16th Ed. 1473, Table 230-8 A. Small volume A. Minoxidil B. Nocturnal B. Methyldopa C. Aggravated by emotional stress C. Labetolol D. Aggravated by eating D. Captopril Nocturnal diarrhea or steatorrheal stools do not occur in IBS and malabsorption or weight loss does not occur. Stool volume >200 - 300 mL/day argues against IBS. 525 Which of the following tumour associated proteins can be found in ulcerative colitis ? 531 Microbe involved in the initial infection of post-infectious IBS Harrison’s 17th Ed. 483, Table 77-5 is ? A. CA125 Harrison’s 18th Ed. 2498 B. CD25 A. Campylobacter 433 Gastroenterology MCQ’s MCQ’s FOR FORPROFESSIONALS MEDICAL MEDICAL PROFESSIONALS BY PROF. AJAY MATHUR Cardiology 433 B. Salmonella Chapter 298. Mesenteric Vascular C. Shigella Insufficiency D. All of the above Microbes involved in the initial infection of post-infectious IBS are Campylobacter, Salmonella, and Shigella. 537 Which of the following is a category of intestinal ischemia ? Harrison’s 18th Ed. 2510 532 Which of the following argue against the diagnosis of IBS ? A. Arterioocclusive mesenteric ischemia (AOMI) Harrison’s 18th Ed. 2498 B. Nonocclusive mesenteric ischemia (NOMI) A. Appearance for the first time in old age C. Mesenteric venous thrombosis (MVT) B. Persistent diarrhea after a 48 hour fast D. All of the above C. Nocturnal diarrhea or steatorrheal stools D. All of the above 538 Risk factors for acute intestinal arterial ischemia include ? Harrison’s 18th Ed. 2510 Appearance of the disorder for the first time in old age, progressive course from time of onset, persistent diarrhea after a 48-h fast, and presence of nocturnal diarrhea or steatorrheal stools argue A. Atrial fibrillation against the diagnosis of IBS. B. Recent myocardial infarction 533 Painful constipation is a major complaint in which of the C. Valvular heart disease following ? D. All of the above Harrison’s 18th Ed. 2499 Risk factors for acute intestinal arterial ischemia include atrial fibrillation, recent myocardial infarction, A. Acute intermittent porphyria valvular heart disease, and recent cardiac or vascular catheterization. B. Hypothyroidism 539 Which of the following is a watershed area within the colonic C. Hypoparathyroidism blood supply and common location for intestinal ischemia ? D. All of the above Harrison’s 18th Ed. 2511 Painful constipation is a major complaint in acute intermittent porphyria and lead poisoning. A. Matsumoto point B. Griffiths’ point 534 Which of the following is an antidiarrheal agent ? C. Wyers point Harrison’s 18th Ed. 2500 A. Loperamide D. Mitchell point B. Paregoric 540 Which of the following is a watershed area within the colonic C. Cholestyramine blood supply and common location for intestinal ischemia ? Harrison’s 18th Ed. 2511 D. All of the above A. Shih point 535 Which of the following probiotics has shown significant benefit B. Sudeck’s point in IBS patients ? C. Hsu point Harrison’s 18th Ed. 2500 A. Faecalibacterium prausnitzii D. Sise point B. Lactobacillus Collateral vessels within colon meet at splenic flexure & descending/sigmoid colon. These watershed areas are inherently at risk for decreased blood flow and are known as Griffiths’ point and Sudeck’s C. Bifidobacterium infantis 35624 point respectively and are the most common locations for colonic ischemia. D. Taenia suis 541 Nationality of Jean Riolan was ? Probiotics naturally alter the gut flora. Bifidobacterium infantis 35624 showed significant improvement A. French in the composite score for IBS. B. German 536 Which of the following is a chloride channel activator ? C. Spanish A. Alosetron D. Portugese B. Rifaximin Arch of Riolan is named after Jean Riolan, French anatomist (1580 - 1657). It connects the proximal C. Tegaserod superior mesenteric artery (SMA) or one of its primary branches (middle colic artery) to the proximal inferior mesenteric artery (IMA) or one of its primary branchesand runs close to the root of the D. Lubiprostone mesentery. Lubiprostone is a bicyclic fatty acid that stimulates chloride channels in the apical membrane of intestinal epithelial cells. Chloride secretion induces passive movement of sodium & water into 542 Which of the following is associated with the best prognosis ? bowel lumen & improves bowel function in constipation-predominant IBS patients. Alosetron is a 5- Harrison’s 18th Ed. 2513 HT3 receptor antagonist. Rifaximin is a non-absorbed oral antibiotic. Tegaserod is a 5-HT4 receptor agonists. A. Arterioocclusive mesenteric ischemia (AOMI) B. Nonocclusive mesenteric ischemia (NOMI) C. Mesenteric venous thrombosis (MVT) D. All of the above 434 MCQ’s FOR MEDICAL PROFESSIONALS BY PROF. AJAY MATHUR Gastroenterology 543 Which of the following is not a feature of chronic mesenteric 549 Blood of caudate lobe of liver drains into ? angina ? Diseases of liver & biliary system, Sheila Sherlock 9th Ed. 1 Harrison’s 18th Ed. 2513 A. Right hepatic vein A. Abdominal cramping & pain following ingestion of meal B. Left hepatic vein B. Weight loss C. Middle hepatic vein C. Constipation D. Inferior vena cava D. Chronic diarrhea 550 Right & left lobes of liver are separated inferiorly by ? Patients of chronic intestinal ischemia (intestinal angina) presents with abdominal cramping and pain Diseases of liver & biliary system, Sheila Sherlock 9th Ed. 1 following ingestion of a meal, weight loss and chronic diarrhea. Abdominal pain without weight loss is not chronic mesenteric angina. A. Falciform ligament B. Fissure for ligamentum venosum 544 The gold standard for confirmation of mesenteric arterial occlusion is ? C. Fissure for ligamentum teres Harrison’s 18th Ed. 2513 D. All of the above A. Mesenteric angiography The right and left lobes of liver are separated anteriorly by falciform ligament, posteriorly by fissure B. Duplex ultrasound for ligamentum venosum and inferiorly by fissure for ligamentum teres. C. Magnetic resonance angiography 551 Capacity of gall bladder is about ? D. Abdominal spiral CT Diseases of liver & biliary system, Sheila Sherlock 9th Ed. 3 Gold standard for confirmation of mesenteric arterial occlusion is mesenteric angiography. Duplex A. 20 ml ultrasound evaluation of the mesenteric vessels is used as a screening test for patients with symptoms suggestive of chronic mesenteric ischemia. B. 30 ml C. 40 ml D. 50 ml 301 - Approach to liver disease Gall bladder is a pear shaped bag, ~9 cm long with a capacity of about 50 ml. 545 The contribution of blood supply to liver from hepatic artery 552 Rokitansky-Aschoff sinuses are related to which of the and portal vein is ? following organs ? Harrison’s 18th Ed. 2520 Diseases of liver & biliary system, Sheila Sherlock 9th Ed. 4 A. 50 % and 50 % respectively A. Liver B. 40 % and 60 % respectively B. Pancreas C. 60 % and 40 % respectively C. Gall bladder D. 20 % and 80 % respectively D. Intestine Liver receives a dual blood supply, ~20% of blood flow is oxygen-rich blood from hepatic artery and Rokitansky-Aschoff sinuses are branching evaginations from the lumen into the mucosa and 80% is nutrient-rich blood from the portal vein arising from stomach, intestines, pancreas & spleen. muscularis of the gall bladder. 546 Right lobe of liver is about how many times bigger than the left 553 Which of the following is not a cell type in liver ? lobe ? Harrison’s 18th Ed. 2520 Diseases of liver & biliary system, Sheila Sherlock 9th Ed. 1 A. Hepatocyte A. 2 times B. Rho cells B. 4 times C. Kupffer cells C. 6 times D. Ito cells D. 8 times Hepatocytes constitute two-thirds of mass of liver. The remaining cell types are Kupffer cells (RE system), stellate (Ito or fat-storing) cells, endothelial cells and blood vessels, bile ductular cells and 547 Pressure in the free hepatic vein is about ? supporting structures. Diseases of liver & biliary system, Sheila Sherlock 9th Ed. 1 554 Portal areas form which zone of the hepatic acinus ? A. 3 mm Hg Harrison’s 18th Ed. 2520 B. 6 mm Hg A. Zone 1 C. 9 mm Hg B. Zone 2 D. 12 mm Hg C. Zone 3 548 All are true regarding hepatic venous system except ? D. Zone 4 Diseases of liver & biliary system, Sheila Sherlock 9th Ed. 1 Functionally, liver is organized into acini. Hepatic arterial and portal venous blood enter acinus from A. Hepatic venous blood in ~67% saturated with oxygen portal areas (zone 1) and flow through the sinusoids to terminal hepatic veins (zone 3). Intervening hepatocytes constitute zone 2. B. Hepatic venous blood is usually sterile C. Hepatic veins are seven in number 555 “Central veins” in liver are also called ? Harrison’s 18th Ed. 2520 D. Hepatic veins begin as zone 3 veins A. Terminal hepatic veins 435 Gastroenterology MCQ’s MCQ’s FOR FORPROFESSIONALS MEDICAL MEDICAL PROFESSIONALS BY PROF. AJAY MATHUR Cardiology 435 B. Terminal portal veins B. Liver abscess C. Terminal sinusoidal veins C. Severe venoocclusive disease D. All of the above D. Acute hepatitis. Blood from portal areas is distributed through sinusoids, passing from zone 1 to zone 3 of acinus and Right upper quadrant severe pain is most typical of gallbladder disease, liver abscess, and severe draining into terminal hepatic veins (“central veins”). venoocclusive disease but is an occasional accompaniment of acute hepatitis. 556 In hepatic acinus, secreted bile flows in the which of the 562 Which of the following is the most reliable marker of severity following direction ? of liver disease ? Harrison’s 18th Ed. 2520 Harrison’s 18th Ed. 2522 A. Zone 1 to zone 3 A. Jaundice B. Zone 3 to zone 1 B. Serum Albumin C. Zone 1 to zone 2 C. Prothrombin time D. Zone 2 to zone 1 D. Hepatic enzymes In hepatic acinus, secreted bile flows in the in a countercurrent pattern from zone 3 to zone 1. Jaundice is the hallmark symptom of liver disease & perhaps the most reliable marker of severity. 557 Kupffer cells are best described as ? 563 Jaundice without dark urine indicates which of the following ? Harrison’s 18th Ed. 2520 Harrison’s 18th Ed. 2522 A. Peripheral T lymphocytes A. Hemolytic anemia B. Circulating B lymphocytes B. Gilbert’s syndrome C. Fixed macrophages C. Crigler-Najjar syndrome D. None of the above D. All of the above Kupffer cells lie within sinusoidal vascular space & represent largest group of fixed macrophages in Jaundice without dark urine indicates indirect (unconjugated) hyperbilirubinemia and is typical of the body. hemolytic anemia, Gilbert’s syndrome and Crigler-Najjar syndrome. 558 Hepatocytes produce which of the following ? 564 In Gilbert’s syndrome, jaundice is more noticeable after which Harrison’s 18th Ed. 2520 of the following ? A. Cholesterol Harrison’s 18th Ed. 2522 B. Lecithin A. Fasting C. Phospholipids B. Overnight sleep D. All of the above C. Exposure to sun D. All of the above Hepatocytes produce bile and its carriers i.e. bile acids, cholesterol, lecithin, phospholipids. In Gilbert’s syndrome, jaundice is more noticeable after fasting and with stress. 559 Which out of the following is the more liver-specific symptom ? Harrison’s 18th Ed. 2522 565 Which of the following statements about hepatitis C is false ? A. Nausea Harrison’s 18th Ed. 2522 B. Bloating A. Sexual exposure is a rare mode of spread C. Poor appetite B. Maternal-infant transmission occurs D. Malaise C. No means of prevention of vertical spread D. None of the above Constitutional symptoms of liver disease include fatigue, weakness, nausea, poor appetite, and malaise. More liver-specific symptoms include jaundice, dark urine, light stools, itching, abdominal Vertical spread of hepatitis C occurs uncommonly. Sexual exposure is a common mode of spread of pain, and bloating. hepatitis B and hepatitis C. 560 Most common & most characteristic symptom of liver disease is ? 566 Single most common risk factor for hepatitis C is ? Harrison’s 18th Ed. 2522 Harrison’s 18th Ed. 2522 A. Fatigue A. Blood transfusion B. Nausea B. Injection drug use C. Poor appetite C. Maternal-infant transmission D. Itching D. Sexual exposure Fatigue is the most common & most characteristic symptom of liver disease. It typically arises after Injection drug use is now the single most common risk factor for hepatitis C. activity or exercise & is rarely present or severe in the morning after adequate rest. Fatigue in liver disease is often intermittent and variable in severity from hour to hour and day to day. 567 What quantity of alcohol consumption per day is associated 561 Severe right upper quadrant pain (“liver pain”) is most typical with an increased rate of alcoholic liver disease in men ? of all except ? Harrison’s 18th Ed. 2522 Harrison’s 18th Ed. 2522 A. 10 to 25 grams A. Gallbladder disease B. 25 to 33 grams 436 MCQ’s FOR MEDICAL PROFESSIONALS BY PROF. AJAY MATHUR Gastroenterology C. 33 to 45 grams 574 The nature of vessel involved in spider angiomata is ? Harrison’s 18th Ed. 2523 D. 45 to 62 grams A. Artery Alcohol consumption associated with an increased rate of alcoholic liver disease is probably more than 22 to 30 gram per day in women and 33 to 45 gram in men. B. Arteriole C. Vein 568 CAGE questionnaire is used for ? Harrison’s 18th Ed. 2523 D. Capillary A. Sexual behaviour Spider angiomata are superficial, tortuous arterioles & fill from center outwards & can be pulsatile. B. Cigarette smoking 575 Most reliable physical finding in examining the liver is ? C. Alcohol dependence & abuse Harrison’s 18th Ed. 2523 D. None of the above A. Size One “yes” response of the four questions in CAGE questionnaire should raise suspicion of an alcohol B. Shape use problem, and more than one is a strong indication that alcohol abuse or dependence exists. C. Tenderness 569 In CAGE questionnaire, “C” refers to ? D. Liver edge Harrison’s 18th Ed. 2523, Table 301-2 Most reliable physical finding in examining the liver is hepatic tenderness. A. Continuous B. Cut 576 Which of the following is a test of mental status in hepatic encephalopathy ? C. Craving Harrison’s 18th Ed. 2523 D. Constant A. Trail-making test C = Cut, A = Annoyed, G = Guilty, E = Eyeopener. One “yes” response should raise suspicion of an B. Comparison of signatures alcohol use problem, and more than one is a strong indication that abuse or dependence exists. C. Drawing abstract objects 570 Which of the following is an inherited liver disease ? D. All of the above Harrison’s 18th Ed. 2523 Mental status examination for hepatic encephalopathy include trail-making test, drawing abstract A. Wilson’s disease objects or comparison of a signature to previous ones. B. Hemochromatosis 577 Which of the following is false about trail-making test ? C. 1 antitrypsin (1AT) deficiency Harrison’s 18th Ed. 2523 D. All of the above A. Series of 10 numbered circles 571 Which of the following is an inherited pediatric liver disease ? B. Normal time for connect-the-dot test is 15-30 seconds Harrison’s 18th Ed. 2523 C. Considerable delay means early hepatic encephalopathy A. Familial intrahepatic cholestasis D. None of the above B. Benign recurrent intrahepatic cholestasis Mental status examination by trail-making test consists of a series of 25 numbered circles that the C. Alagille syndrome patient is asked to connect as rapidly as possible using a pencil. Normal range for connect-the-dot test is 15-30 seconds. Delay means early hepatic encephalopathy. D. All of the above 578 Which of the following is not a feature of hepatopulmonary 572 Which of the following statements is false ? syndrome ? Harrison’s 18th Ed. 2523 Harrison’s 18th Ed. 2524 A. Muscle wasting is a sign of advanced liver disease A. Hypoxemia B. Palmar erythema occur in acute liver disease B. Pulmonary arteriovenous shunting C. Jaundice may be detectable with S. bilirubin <2.5 mg/dL C. Orthodeoxia D. None of the above D. Hypercarbia Signs of advanced liver disease include muscle wasting, ascites, edema, bruising, dilated abdominal Hepatopulmonary syndrome is defined by the triad of liver disease, hypoxemia and pulmonary veins, hepatic fetor, asterixis, mental confusion, stupor, and coma. During recovery phase, jaundice arteriovenous shunting. It is characterized by platypnea and orthodeoxia, representing shortness of may be detectable with S. bilirubin <2.5 mg/dL. Spider angiomata & palmar erythema occur in both breath and oxygen desaturation that occur upon assuming an upright position. acute & chronic liver disease. 573 Which of the following about spider angiomata is false ? 579 Slate-gray pigmentation to the skin also occurs with ? Harrison’s 18th Ed. 2524 Harrison’s 18th Ed. 2523 A. Occur in acute & chronic liver disease A. Primary biliary cirrhosis B. Occur during pregnancy B. Sclerosing cholangitis C. Occur only on arms, face & upper torso C. Xanthelasma D. None of the above D. Hemochromatosis A slate-gray pigmentation of the skin occurs in hemochromatosis, if iron levels are high for a Spider angiomata occur only on the arms, face, and upper torso. Spider angiomata and palmar prolonged period. erythema occur in both acute and chronic liver disease and are prominent in cirrhosis, but they can occur in normal individuals and are frequently present during pregnancy. Wilson’s disease 587 Most common causes of chronic liver disease is ? D. ATP9B gene B. Chronic hepatitis B 582 Which of the following statements about Kayser-Fleischer ring is false ? D. Primary biliary cirrhosis C. Bernhard Kayser and Bruno Fleischer were German ophthalmologists. chronic active hepatitis. 2450 A. Primary sclerosing cholangitis A.sclero-corneal junction. hemochromatosis. Failure of this process leads to instability and D. autoimmune hepatitis. serum albumin. 583 The colour of Kayser-Fleischer ring is ? Harrison’s 18th Ed. cholestatic cirrhosis. Table 301-3 581 Kayser-Fleischer rings are a finding in ? A. It may regress or disappear when systemic D. then inferiorly and later becomes circumferential. Hemochromatosis Harrison’s 18th Ed. None of the above B. ATP7B facilitates the transfer of copper into Golgi apparatus where it combines with ceruloplasmin. Palm tree Harrison’s 18th Ed. ATP8B gene A. B. Harrison’s 18th Ed. Prolongation of prothrombin time C. Bleeding from varices A. Density of a KF ring correlates with severity of Wilson’s 588 ERCP is more valuable in evaluating which of the following ? Harrison’s 18th Ed. Other prothrombin time. SGPT Wilson disease is an autosomal recessive disorder caused by mutations in the ATP7B gene. slight decreases in serum albumin. Primary hepatocellular carcinoma Sunflower cataracts & Kayser-Fleischer rings (copper deposits in the outer rim of cornea) are seen C. Found in 95% of patients of Wilson’s disease alcoholic liver disease. Sunflower A. galactosialidosis. 2526 C. Pancreatitis D. Slight decreases in serum albumin Kayser-Fleischer rings consist of a golden-brown copper pigment deposited in Descemet’s membrane at the periphery of the cornea . Rose flower B. Chronic hepatitis C Strumpell disease/Westphal pseudosclerosis. hepatocellular disorders C.86:114 Most common causes of chronic liver disease in general order of frequency are chronic hepatitis C. prolongation of prothrombin time. and mild thrombocytopenia (which is often the first indication of worsening fibrosis). Cryoglobulinemia of chronic hepatitis C in Wilson’s disease. 2524 B. Mild elevations of bilirubin D. 2524 C. AJAY MATHUR Cardiology 437 580 Mucocutaneous vasculitis with palpable purpura is typical of ? B. Golden-brown A. schistosoma infection. 584 Wilson’s disease is due to defect in ? Harrison’s 18th Ed. 437 Gastroenterology MCQ’s MCQ’s FOR FORPROFESSIONALS MEDICAL MEDICAL PROFESSIONALS BY PROF. Primary hepatocellular carcinoma D. All patients with KF rings have neurological manifestations C. primary biliary cirrhosis. alcoholic liver disease. nonalcoholic steatohepatitis. multiple myeloma and intraocular foreign body containing copper. 2524 disease A. A. Primary biliary cirrhosis Harrison’s 18th Ed. It is usually bilateral & appears initially superiorly at 10 to 2 o’clock position. Autoimmune hepatitis B. primary biliary cirrhosis. ascites & hepatic encephalopathy. D. 2524 589 Which of the following is often the first indication of worsening A. 2524 590 Which of the following is not a factor in Child-Pugh A. Yellow-green Harrison’s 18th Ed. ATP6B gene classification of cirrhosis liver ? B. Free circulating Factors included in Child-Pugh classification of cirrhosis liver are serum bilirubin. 2525 Kayser-Fleischer rings occur in Wilson’s disease also called hepatolenticular degeneration/ Westphal. hepatic conditions as listed above that prevent elimination of copper. 591 Child-Pugh score can predict which of the following ? 585 Wilson’s disease can produce a cataract described as ? Harrison’s 18th Ed. Autoimmune hepatitis Br J Ophthalmol 2002. Primary biliary cirrhosis D. Alcoholic liver disease C. 2526 Harrison’s 17th Ed. Christmas tree B. Serum bilirubin D. Spontaneous bacterial peritonitis . Choledocholithiasis (when bilirubin rises acutely above 20mg/dl). B. or intraocular foreign bodies containing copper. D. ATP7B gene Harrison’s 18th Ed. lead to elevated free copper concentrations and K-F ring. Yellow-red fibrosis in liver cirrhosis ? B. copper accumulates in liver cytosol resulting in hepatocyte degeneration and cirrhosis. Hepatic encephalopathy decreased half life of ceruloplasmin and paradoxical ceruloplasmin deficiency. Noninvasive tests that suggest advanced liver fibrosis include mild elevations of bilirubin. 2526 C. chronic hepatitis B. sclerosing cholangitis. Wilsons disease 586 Testing for P-ANCA is for the diagnosis of which of the Mucocutaneous vasculitis with palpable purpura on lower extremities is typical of cryoglobulinemia following ? of chronic hepatitis C. Primary sclerosing cholangitis K-F-like ring has been reported in many other conditions like cryptogenic cirrhosis. Green-brown B. and Wilson disease. 2524. Ascites membrane-bound copper-transporting ATPase. Fatty liver ERCP is more valuable in evaluating ampullary lesions and primary sclerosing cholangitis. Nonalcoholic steatohepatitis C. a C. neonatal hepatitis. A. Mild thrombocytopenia condition is well treated. < 20 % 9 MELD score is a prospectively derived scoring system calculated using three noninvasive variables C. < 25 % 9 . A.438 MCQ’s FOR MEDICAL PROFESSIONALS BY PROF. 7 A. Assessing hepatocellular carcinoma B. aminotransferases. PELD C. None of the above C. spontaneous bacterial peritonitis and need for liver transplantation. 6 Harrison’s 18th Ed. D. Prothrombin time D. Gilbert’s syndrome 592 Decompensation indicates cirrhosis with a Child-Pugh score of ? D. All of the above Harrison’s 18th Ed. is termed ? h Harrison’s 18th Ed. AJAY MATHUR Gastroenterology C. None of the above Model for end-stage liver disease (MELD) score is used for assessing the need for liver transplantation. A. Absence of hemolysis indicates cirrhosis with a Child-Pugh score of 7 or more (Class B). INR. 2527 A. Table 302-1 A.Evaluation of liver function Harrison’s 18th Ed. Decompensation C. Assessing need for liver transplantation D. 2526 600 In isolated unconjugated hyperbilirubinemia. Conjugation of bilirubin in hepatocyte C. or C (10 or above). serum bilirubin and serum creatinine. None of the above R 593 MELD score is used for ? Harrison’s 18th Ed. but transport of conjugated bilirubin into the bile canaliculi. Bilirubin bilirubin are elevated B. Crigler-Najjar syndrome C. < 15 % D. S. S. Prothrombin time 603 Total serum bilirubin correlates with poor outcomes in ? Harrison’s 18th Ed. age. Increased unconjugated bilirubin is rarely due to liver disease 596 Which of the following liver function tests do not measure liver B. Entry of bilirubin in hepatocyte B. In Hemolysis/Gilbert’s syndrome. 2527 D. Table 301-4 Isolated elevation of unconjugated bilirubin is seen in hemolytic disorders. 2526. 4 B. the direct bilirubin is ? B. Child-Pugh class is either A (a score of 5 to 6). urea A. serum albumin. B (7 to 9). D. serum bilirubin is elevated but direct bilirubin is <15%. Albumin D. Healthy patient The Child-Pugh score is calculated by adding the scores of the five factors and can range from 5 to B. 5 598 Which of the following about Gilbert’s syndrome is false ? C. All of the above A. 2526 599 Which of the following is elevated in Gilbert’s syndrome ? V Harrison’s 18th Ed. Conjugated hyperbilirubinemia almost always implies liver function at all ? or biliary tract disease Harrison’s 18th Ed. rate-limiting step in bilirubin metabolism is ? Harrison’s 18th Ed. KELD D. 2530. Transport of conjugated bilirubin into bile canaliculi D. B. Assessing bleeding from varices A. Isolated. alkaline phosphatase. S. All of the above System similar to MELD using bilirubin. In most liver diseases. 2527 D. < 30 % ir 595 System similar to MELD used for children below the age of 12 In isolated unconjugated hyperbilirubinemia. Hemolytic disorders Child-Pugh score is a reliable predictor of bleeding from varices. Need for liver transplantation 597 Isolated elevation of unconjugated bilirubin is seen in ? Harrison’s 18th Ed. 602 Which of the following about liver functions is false ? 302 . Prothrombin time bilirubin.prothrombin time (INR). Aminotransferases d B. SELD B. Rate-limiting step in bilirubin metabolism is not conjugation of bilirubin. Serum bilirubin Harrison’s 18th Ed. 2527 A. 2527 C. Aminotransferases D. of the total U A. Viral hepatitis . There is no bilirubinuria. 2527 - C. Serum creatinine B. both conjugated & unconjugated A. and nutritional status is used for children below the age of 12 is PELD. albumin and prothrombin n time are normal. 594 Variables for calculation of MELD score include all except ? Harrison’s 18th Ed. B. Assessing spontaneous bacterial peritonitis ti e C. 2527 Aminotransferases or alkaline phosphatase do not measure liver function at all. CELD ta A. 2526 601 In liver disease. unconjugated hyperbilirubinemia G 15. Alkaline phosphatase C. Crigler-Najjar & Gilbert’s syndromes. Barbiturates hepatocellular injury like viral hepatitis. cardiac muscle. drug-induced liver disease indicates more severe injury. Erythrocytes Serum bilirubin serves as a prognostic marker in viral hepatitis. > 1. Brain D. C. All of the above B. skeletal muscle. Unconjugated B. Toxin or drug-induced liver injury 605 Which of the following may give a false-positive reading with D. 2529 Phenothiazines may give a false-positive reading with Ictotest tablet. Lungs D. 439 Gastroenterology MCQ’s MCQ’s FOR FORPROFESSIONALS MEDICAL MEDICAL PROFESSIONALS BY PROF. the urine bilirubin clears prior to the serum bilirubin. Serum bilirubin clears prior to urine bilirubin disease. Pancreas AST in alcoholic liver disease is rarely >300 U/L and ALT is often normal. A. Urine and serum bilirubin clear simultaneously 613 Which of the following about alcoholic liver disease is false ? D. Alcoholic hepatitis 610 Least concentration of AST is in ? Harrison’s 18th Ed. AST rarely > 300 U/L 607 Which of the following play a role in detoxification of ammonia ? B. Platelets A low level of ALT in the serum is due to an alcohol-induced deficiency of pyridoxal phosphate. Deficiency of pyridoxal sulphate 609 Aspartate aminotransferase (AST) not found in ? Harrison’s 18th Ed. Skeletal muscle significance in acute hepatocellular disorders B. Deficiency of pyridoxal gluconate B. Pancreas 604 Bilirubin found in urine is ? Harrison’s 18th Ed. None of the above C. Ischemic liver injury bilirubin found in urine is conjugated bilirubin and bilirubinuria implies the presence of liver disease. & erythrocytes in decreasing order of concentration. 2529 D. Leukocytes C. > 2:1 jaundice ? C. lungs. 2528 C. All of the above Ictotest tablet ? Harrison’s 18th Ed. C. pancreas. Any of the above Harrison’s 18th Ed. A. All of the above A. Urine bilirubin clears prior to serum bilirubin An AST:ALT ratio >2:1 is suggestive while a ratio >3:1 is highly suggestive of alcoholic liver B. 2528 B. NSAID’s disease ? Harrison’s 18th Ed. None of the above B. brain. Increases in IgA levels occur in alcoholic liver disease. Spleen D. 2529 A. C. Striated muscles detoxify ammonia by combining it with glutamic acid to form glutamine. A. Liver cell necrosis not required for release of aminotransferases Harrison’s 18th Ed. B. 2527 AST is found in liver. 2528 C. and toxin or drug induced liver injury. alcoholic hepatitisin MELD score. Unconjugated + conjugated of the following disorders ? Harrison’s 18th Ed. Aminotransferases levels of >1000 U/L occur in disorders associated with extensive A. All of the above . Deficiency of pyridoxal chloride C.5:1 606 Which of the following is true in patients recovering from B. Phenothiazines C. C. Aminotransferases are present in serum in low concentrations 608 Aspartate aminotransferase (AST) found in all except ? B. ischemic liver injury. A. Kidneys D. any B. Spleen 615 Low serum ALT in alcoholic liver disease is due to ? D. Drug-induced liver disease A. Increase in IgA levels A. ALT often normal Harrison’s 18th Ed. Antacids 612 What AST:ALT ratio is highly suggestive of alcoholic liver D.5:1 Harrison’s 18th Ed. kidneys. Viral hepatitis Unconjugated bilirubin always binds to albumin in serum and is not filtered by kidney. > 3:1 A. leukocytes. Cartilage 614 Which of the following about aminotransferases is false ? Harrison’s 18th Ed. > 2. Deficiency of pyridoxal phosphate A. 2527 C. Conjugated 611 Elevation of aminotransferases to >1000 U/L occurs in which C. Therefore. D. 2527 D. 2529 Liver converts ammonia to urea which is excreted by kidneys. Striated muscle D. 2529 In patients recovering from jaundice. 2527 Aminotransferase levels of up to 300 U/L are nonspecific and may be found in any type of liver disorder. Erythrocytes D. AJAY MATHUR Cardiology 439 B. Absolute elevation of aminotransferases is of no prognostic A. Lungs Harrison’s 18th Ed. Gamma glutamyl transpeptidase (GGT) A. IgM levels increase in primary biliary cirrhosis phosphatase include all except ? Harrison’s 18th Ed. C. 3 times Gamma globulins (immunoglobulins) are produced by B lymphocytes and alpha and beta globulins are produced in hepatocytes.440 MCQ’s FOR MEDICAL PROFESSIONALS BY PROF. 5’-nucleotidase & GGT are usually elevated in cholestasis. A i D. 2529 A. Liver B. All of the above B. alkaline phosphatase elevations B. Alpha globulin 620 In cholestatic liver disorders. All of the above hepatocytes ? Individuals with blood types O & B can have an elevation of serum alkaline phosphatase after eating Harrison’s 18th Ed. Serum interleukin 1 9 elevation of serum alkaline phosphatase after eating a fatty meal ? B. A. 2529 C. Bone alkaline phosphatase is most susceptible to inactivation by heat. 5’-nucleotidase 622 Serum albumin is synthesized by ? C. C. 5’-nucleotidase 623 Serum albumin has a half-life of ? G Harrison’s 18th Ed. Elevated heat-stable fraction of serum alkaline phosphatase strongly suggests its placental or tumor - source. A. None of the above 627 Which of the following statements is false ? Harrison’s 18th Ed. 2529 A. None of the above A. 2529 D. 2529 C. 2529 Harrison’s 18th Ed. Serum 5'-nucleotidase or GGT are rarely elevated in conditions other than liver disease. 2529 Isolated elevations of serum alkaline phosphatase is seen in Hodgkin’s disease. while GGT is located in the endoplasmic reticulum and in bile duct epithelial cells. Hepatocyte Alkaline phosphatase. 625 Albumin synthesis is inhibited by ? Harrison’s 18th Ed. while increases in IgA levels occur in alcoholic C. 4 times D. > 60 days d 618 Elevated heat-stable fraction of serum alkaline phosphatase 624 What proportion of albumin is degraded per day ? suggests its origin from ? ti e Harrison’s 18th Ed. . Intestine Serum albumin has a half-life of 18 to 20 days with approximately 4% degraded per day. A. Beta globulin are how many times greater than normal ? C. Gamma globulins are increased in chronic liver disease B. 4% B. infiltrative liver diseases and Paget’s disease. Congestive heart failure Harrison’s 18th Ed. 2530 a fatty meal due to influx of intestinal alkaline phosphatase into the blood. CHF. Gamma globulin Harrison’s 18th Ed. Bone n C. B h Prolonged increases in levels of serum cytokines IL-1 &/or tumor necrosis factor inhibit albumin synthesis. Increases in IgM levels are common in primary biliary cirrhosis. Kidney C. Diffuse polyclonal increases in IgG levels are common in autoimmune hepatitis. 18 to 20 days D. 8% U D. Gamma glutamyl transpeptidase (GGT) Harrison’s 18th Ed. AJAY MATHUR Gastroenterology 616 Which of the following enzymes is elevated in cholestasis ? D. A. Alkaline phosphatase hyperthyroidism. Inflammatory bowel disease antigens. 2% A. 28 to 35 days Alkaline phosphatase and 5’-nucleotidase are found in or near the bile canalicular membrane of C. Alkaline phosphatase B. Hodgkin’s disease In cirrhosis. D. Placenta D. B. Hypothyroidism liver disease. some directed against intestinal bacteria because cirrhotic liver fails to clear bacterial B. All of the above Harrison’s 18th Ed. 2529 D. Cholecystokinin 9 Harrison’s 18th Ed. 35 to 45 days V hepatocytes. AB 626 Which of the following serum globulins is not produced by ta D. IgG levels increase in autoimmune hepatitis 621 Conditions causing isolated elevations of serum alkaline C. 6% C. increased serum gamma globulin concentration is due to increased synthesis of antibodies. All of the above A. All of the above B. Lipase r A. B. Intestinal epithelial cells 617 Which of the following is located in endoplasmic reticulum of hepatocytes ? D. 2 times D. 2530 Alkaline phosphatase elevations greater than four times normal occur primarily in cholestatic liver disorders. 2529 Serum albumin is synthesized exclusively by hepatocytes. diabetes. amyloidosis and inflammatory bowel disease. 2529 619 Individuals of which of the following blood group can have an A. All of the above R B. and X depends on vitamin K. Serum globulins Conjugation of bilirubin with glucuronic acid moieties disrupts internal hydrogen bonding that limits C. Biliary excretion A. II D. D. 7 days Harrison’s 18th Ed. 2530 C. Intracellular binding Serum half life of factor VII is 6 hours. Internal sulphate bonding A. measurement of the clotting factors is the single best acute belong to which UGT family ? measure of hepatic synthetic function. VII and X. Harrison’s 18th Ed. 2532. UGT3 A. B.90% of bilirubin is derived from degradation of the hemoglobin of senescent red blood cells. Conjugation following factor ? Harrison’s 18th Ed. V the hemoglobin of senescent red blood cells ? Harrison’s 18th Ed. 2531 C. 2530 C.70 % With the exception of factor VIII (produced by vascular endothelial cells). X and the four common exons.90 % D. 72 hours B. All of the above B. UGT2 Harrison’s 18th Ed. V. bilirubin is kept in solution by binding to glutathione-S-transferases A. 24 hours steps in bilirubin metabolism ? Harrison’s 18th Ed. ~ 100 % 629 Serum half-life of factor VII is ? About 70 . 2531 C. C. All of the above Biosynthesis of factors II. Figure 303-2 A. 6 hours 635 Glutathione-S-transferase is related to which of the following B. ~ 50 . 2530 A. collectively designated the UGT1A1 gene. 2530 A. A. B. IX 637 Aqueous insolubility of bilirubin is due to which of the D. 5 days the following steps in bilirubin metabolism ? D. X following ? Harrison’s 18th Ed. X B.and diglucuronide. UGT1 on vitamin K ? B.Hyperbilirubinemias Harrison’s 18th Ed. II Bilirubin is conjugated with one or two glucuronic acid moieties by a specific UDP- glucuronosyltransferase to form bilirubin mono. V C. Hepatocellular uptake D. Clotting factors aqueous solubility of bilirubin & the resulting glucuronide conjugates are highly soluble in water. A. blood clotting factors are made exclusively in hepatocytes. ~ 30 . Hepatocellular uptake B. VIII A. Intracellular binding 631 Serum prothrombin time does not measure which of the C. 2532 633 Biosynthesis of factors which of the following factors depends A. AJAY MATHUR Cardiology 441 628 Which of the following blood clotting factors is not made by hepatocytes ? 303 . II 634 What proportion of bilirubin is derived from degradation of B. 2531 Serum half life of fibrinogen is 5 days. 2530 After hepatocellular uptake. 2531 Serum prothrombin time collectively measures factors II. 2530 D. 3 days 636 Bilirubin-UDP-glucuronosyltransferase is related to which of C. 441 Gastroenterology MCQ’s MCQ’s FOR FORPROFESSIONALS MEDICAL MEDICAL PROFESSIONALS BY PROF. VII. 2 days formerly called ligandins. C. 639 Human UGT1 gene complex is on which of the following chromosomes ? Harrison’s 18th Ed. 2 . D. Biliary excretion Harrison’s 18th Ed. Internal hydrogen bonding Harrison’s 18th Ed. ~ 70 . Harrison’s 18th Ed. IX. UGT4 B. IX UDP-glucuronosyltransferases (UGT) that conjugate bilirubin belong to the UGT1 family. Serum albumin D. Exon A1 C. Conjugation 630 Serum half-life of fibrinogen is ? D. encode the physiologically critical enzyme bilirubin-UDP-glucuronosyltransferase (UGT1A1). 48 hours A. Internal phosphate bonding 632 Single best acute measure of hepatic synthetic function is ? B. respectively.50 % D. Serum bilirubin 638 UDP-glucuronosyltransferases (UGT) that conjugate bilirubin Because of their rapid turnover. 2532 agents produce defects in bilirubin uptake.associated protein 2 (MRP2) is related Hemolysis results in pure unconjugated hyperbilirubinemia. <= 15 % in exons 2 .and diglucuronides are excreted across canalicular plasma membrane into bile canaliculus by ATP-dependent transport process mediated by a canalicular membrane protein called multidrug D. A. 7 and 10 after birth 644 Hemolysis alone cannot result in a sustained hyperbilirubinemia Immediately after birth. <= 10 % Human UGT1 gene complex is on chromosome 2. All of the above 643 In response to hemolytic stress. Harrison’s 18th Ed. lead poisoning. 6 isolated hemolysis ? Harrison’s 18th Ed. 5 and 7 after birth erythrocyte production. Congenital erythropoietic porphyria D. D. Colorless decreased hepatic bilirubin uptake ? Harrison’s 18th Ed. etc. Urobilinogen undergoes enterohepatic cycling. Since four of these are pseudogenes. Harrison’s 18th Ed. 10 .5 affect all enzymes encoded by the UGT1 complex.). chloramphenicol. 1 and 3 after birth D. Water-soluble 647 Which of the following produces hyperbilirubinemia due to C. Thalassemia major C. 2532 A. 2532 B. A2. bone marrow is capable of a sustained eight fold increase in C. Hepatocellular uptake 646 Ineffective erythropoiesis is seen in all except ? Harrison’s 18th Ed. 2533 C. and gentamicin 9 produce unconjugated hyperbilirubinemia by inhibiting UGT1A1 activity. megaloblastic anemias due to folate or vitamin B12 deficiency. with direct-reacting fraction being <=15% to which of the following steps in bilirubin metabolism ? of total serum bilirubin. Harrison’s 18th Ed. ~4 mg/dL Harrison’s 18th Ed. <= 18 % 640 Multidrug resistance . rifampin and cholecystographic contrast Harrison’s 18th Ed. D. flavaspidic acid. Those C. 8 A. Four fold Harrison’s 18th Ed. C. Placenta ta efficient means to interrupt bilirubin enterohepatic cycling. Crigler-Najjar syndrome R Bilirubin mono. 2532 649 Most neonates develop mild unconjugated hyperbilirubinemia A. Chloramphenicol U urobilinogen. and congenital & acquired dyserythropoietic anemias. 641 Which of the following is false about urobilinogen ? the fraction of total bilirubin production derived from ineffective erythropoiesis is increased. from which some is cleared by kidneys. Biliary excretion C. 5 . Fetal kidney Oral administration of calcium phosphate with or without the lipase inhibitor orlistat may be an C. 3 . Lead poisoning resistance-associated protein 2 (MRP2). AJAY MATHUR Gastroenterology B. Magnesium sulphate 648 Fetal bilirubin is cleared by ? r Harrison’s 18th Ed. A. ~8 mg/dL B. C. Calcium phosphate i A. nine UGT1 isoforms with differing B. bone marrow is capable of increasing erythrocyte production by ? Bilirubin produced by the fetus is cleared by the placenta and eliminated by the maternal liver. 2532 D. ~2 mg/dL 650 What are the peak levels of physiologic neonatal jaundice ? B. congenital erythropoietic porphyria. Two fold between days ? B. Six fold A. Urobilinogen not taken up by liver reaches systemic circulation. Cholecystographic contrast agents cycling ? Apart from Gilbert’s syndrome. Mutations of MRP2 result in Dubin-Johnson syndrome. undeveloped intestinal flora that convert bilirubin to urobilinogen.10 mg/dL Hemolysis alone cannot result in a sustained hyperbilirubinemia of more than ~ 4 mg/dL. 4 645 Direct-reacting fraction is how much of total serum bilirubin in C. most neonates develop mild unconjugated hyperbilirubinemia between days 2 and 5 after birth. V In thalassemia major. Pregnanediol. Gentamicin - 642 Which of the following interrupts bilirubin enterohepatic D. d Harrison’s 18th Ed.442 MCQ’s FOR MEDICAL PROFESSIONALS BY PROF. novobiocin. Calcium carbonate h B. 2533 n D. 2532 A. Undergoes enterohepatic cycling A. Conjugation G B. Mutations in a first exon affect only a single isoform. D. 2532 reaching as much as 70% of the total producing modest degrees of unconjugated hyperbilirubinemia. Pregnanediol “Conjugated bilirubin” is converted by bacterial metabolism in gut to water-soluble colorless B. Fetal liver D. Made from unconjugated bilirubin in gut ti e B. due to incompletely developed hepatic physiologic processes like low levels of more than ? of UGT1A1. 2533 C. <= 12 % substrate specificities are expressed. novobiocin. It contains at least 13 substrate-specific first exons (A1.5 mg/dL D. 2 and 5 after birth In response to hemolytic stress.15 mg/dL . Intracellular binding A. 2533 C. Aluminum hydroxide 9 B. Eight fold B. ~6 mg/dL A. or absolute levels >20 mg/dL. exposed the Crigler-Najjar Syndrome. D. breast milk ? Harrison’s 18th Ed. 2533 657 Which of the following about Crigler-Najjar Syndrome. 2534 C. AJAY MATHUR Cardiology 443 D. D. Crigler-Najjar Syndrome Type IA C. 3 weeks A. Immature blood-brain barrier B.45 mg/dL reduced levels in CN-II and bilirubin concentrations fall with phenobarbital therapy. Deposition in the basal ganglia C. and gentamicin may produce unconjugated hyperbilirubinemia by inhibiting UGT1A1 activity. Crigler-Najjar Syndrome Type IC Pregnanediol. Appears in the neonatal period Peak levels of physiologic neonatal jaundice are typically 5 . Crigler-Najjar Syndrome Type ID In Crigler-Najjar Syndrome Type IB. chloramphenicol. No evidence of hemolysis as mechanisms required for bilirubin disposition mature. 2534 B. Chloramphenicol A. and the causative 654 ‘Breast milk jaundice’ in neonates is due to presence of what in mutation is in the bilirubin-specific exon A1. UGT1A1 in liver is present at A. Kernicterus common D. All of the above B. CN-II is infrequently associated with kernicterus In transient familial neonatal hyperbilirubinemia (Lucey-Driscoll syndrome) there is a UGT1A1 C.0 per million) C. 4 weeks B. 2534 bilirubin concentrations are lower in CN-II. Gentamicin B.1. Type I A. hepatic histology and no hemolysis. 2533 Harrison’s 18th Ed. 45 mg/dL that appears in neonatal period and persists for life. 2534 C. Unconjugated hyperbilirubinemia B. Persists for life D. or 658 Which of the following is false about Crigler-Najjar Syndrome kernicterus ? Type I ? Harrison’s 18th Ed. Type I Crigler-Najjar Syndrome Type II (CN-II) is characterized by marked unconjugated hyperbilirubinemia is false ? with normal conventional hepatic biochemical tests. Bilirubin glucuronides are absent from bile and there is expression of UGT1A1 activity in hepatic tissue. 2534 A. 2533 659 Causative mutation is in the bilirubin-specific exon A1 of which A. Fatty acids B. Carbohydrates C. C. 2533 660 Which of the following is false about Crigler-Najjar Syndrome A. Absent UGT1A1 activity in liver C. UGT1A1 inhibitor in breast milk 661 Crigler-Najjar Syndrome Type I (CN-I) differs from Crigler-Najjar Syndrome. All of the above 656 Which of the following about Crigler-Najjar Syndrome. or kernicterus. Bilirubin is found in the urine 651 Bilibubin levels of physiologic neonatal jaundice return to normal adult concentrations within ? No bilirubin is found in the urine. Rapidly rising unconjugated bilirubin concentration A. UGT1A1 inhibitor in maternal serum Harrison’s 18th Ed. . Low UGT1A1 levels at birth A. 1 week is false ? B. hyperbilirubinemia by inhibiting UGT1A1 activity ? Harrison’s 18th Ed. None of the above 652 Which of the following is false for bilirubin encephalopathy. Autosomal recessive pattern of inheritance D. CN-II responds to phenobarbital inhibitor in maternal serum. 443 Gastroenterology MCQ’s MCQ’s FOR FORPROFESSIONALS MEDICAL MEDICAL PROFESSIONALS BY PROF. novobiocin. D. the defect is limited to bilirubin conjugation. All of the above C. Immunoglobulins Type I ? Harrison’s 18th Ed. Normal serum aminotransferases D. Liver transplantation not helpful In Crigler-Najjar Syndrome Type I. early liver transplantation remains the best hope to prevent brain 655 Lucey-Driscoll syndrome is related to ? injury and death.10 mg/dL. Type I (CN-I) is characterized by unconjugated hyperbilirubinemia of ~20 to infant to the at risk of bilirubin encephalopathy. D. 2533 A.20 mg/dL B. Estrogen glucuronidation is defective Bilirubin conjugation may be inhibited by certain fatty acids that are present in breast milk but not serum of mothers whose infants have excessive neonatal hyperbilirubinemia (breast milk jaundice). Harrison’s 18th Ed. Proteins A. Bile in CN-II contains bilirubin glucuronides mostly monoglucuronides. 2 weeks Harrison’s 18th Ed. Crigler-Najjar Syndrome Type IB D. Harrison’s 18th Ed. Type II (CN-II) is which of the following ? B. Responds to phenobarbital A rapidly rising unconjugated bilirubin concentration. Average Harrison’s 18th Ed. 15 . Average bilirubin concentrations are lower in CN-II D. None of the above D. Pregnanediol variety of Crigler-Najjar Syndrome Type I ? Harrison’s 18th Ed. There is no response to phenobarbital 653 Which of the following drugs may produce unconjugated or any enzyme inducer. 2534 B. Unconjugated hyperbilirubinemia of ~ 20 . Rare (estimated prevalence = 0. Normal alkaline phosphatase Bilibubin levels of physiologic neonatal jaundice return to normal adult concentrations within 2 weeks C.6 . CN-II is infrequently associated with kernicterus. 2535 A. All of the above Dark. Dubin-Johnson Syndrome A. 671 Dark. Liver grossly black D. Intercurrent illness C. Phenobarbital normalizes serum bilirubin B. Crigler-Najjar Syndrome B. 663 Gilbert’s Syndrome is a close clinical entity to which of the following ? 669 Liver is grossly black in appearance in which of the following ? Harrison’s 18th Ed. degree of hyperbilirubinemia may be increased by intercurrent illness. 2535 A. coarsely granular pigment in hepatocytes in Dubin- Johnson Syndrome disappears during ? 9 665 Which of the following drugs used in HIV patients inhibits Harrison’s 18th Ed. Crigler-Najjar Syndrome Type I A. pigment. Benign Recurrent Intrahepatic Cholestasis D. only to reaccumulate slowly after recovery. - because its active metabolite (SN-38) is glucuronidated specifically by bilirubin-UDP- glucuronosyltransferase. Benign recurrent intrahepatic cholestasis (BRIC) V The clinical spectrum of Gilbert’s Syndrome hyperbilirubinemia fades into that of CN-II at serum In DJS. In Dubin-Johnson Syndrome. 2534 Harrison’s 18th Ed. toxicity occurs upon administration of antitumor agent irinotecan (CPT-11) DJS patients have normal serum & biliary bile acid concentrations & do not have pruritus.444 MCQ’s FOR MEDICAL PROFESSIONALS BY PROF. 2535 Harrison’s 18th Ed. Zidovudine i B. 2535 A. which is the conjugated hyperbilirubinemia ? predominant coproporphyrin isomer ? Harrison’s 18th Ed. Pruritus common U D. Coproporphyrin isomer III D. Indinavir C. MRP2 C. 667 Which of the following does not manifest as predominantly 673 In urine from Dubin-Johnson Syndrome patients. Oral contraceptive use D. UGT1A1 activity . Crigler-Najjar Syndrome Type II B. liver is grossly black in appearance. UGT1A1 activity is typically reduced to 10 . Rotor Syndrome C. 2536 A. which has an autosomal recessive pattern of inheritance. Coproporphyrin isomer IV . Conjugated hyperbilirubinemia B. This pigment is thought to be derived from epinephrine metabolites that are not excreted normally. coarsely granular bilirubin concentrations of 5 . oral contraceptive use. All of the above In Gilbert’s Syndrome. Dubin-Johnson Syndrome B. due to accumulation in lysosomes of centrilobular hepatocytes of dark. 672 Mutation in which of the following genes produce the Dubin- Johnson phenotype ? 666 Which of the following are examples of familial defects in Harrison’s 18th Ed. Bilirubinuria present n C. 2535 B. Viral hepatitis r A. d 664 Which of the following drugs is glucuronidated specifically by bilirubin-UDP-glucuronosyltransferase ? 670 Which of the following is false about Dubin-Johnson Syndrome ? ti e Harrison’s 18th Ed. Malaria h C. coarsely granular pigment in hepatocytes in Dubin-Johnson Syndrome disappears during ta bouts of viral hepatitis.35% of normal.8 mg/dL. 2535 UGT1A1 ? A. Lucey-Driscoll syndrom C. Benign recurrent intrahepatic cholestasis (BRIC) D. resulting in hyperbilirubinemia that is most pronounced in patients with preexisting Gilbert’s Syndrome. 2535 Harrison’s 18th Ed. Enfuvirtide D. NTCP B. Tolbutamide C. AJAY MATHUR Gastroenterology 662 Which of the following is false about Gilbert’s Syndrome ? 668 In Dubin-Johnson Syndrome. Aggravated by alcohol use C. Coproporphyrin isomer I B. FIC1 D. HIVprotease inhibitors indinavir and atazanavir inhibit UGT1A1. Acetaminophen B. ABCC2 Harrison’s 18th Ed. Rotor Syndrome C. degree of hyperbilirubinemia Harrison’s 18th Ed. All of the above MRP2 is an ATP-dependent canalicular membrane transporter and mutations in the MRP2 gene produce the Dubin-Johnson phenotype. 2535 B. Conjugated hyperbilirubinemia Harrison’s 18th Ed. Estradiol benzoate A. Progressive familial intrahepatic cholestasis R D. Enteric fever 9 Harrison’s 18th Ed. and bile pigments exhibit a characteristic increase in bilirubin monoglucuronides.10-35 % of normal A. Irinotecan In Gilbert’s Syndrome. Pregnancy D. All of the above D. Rotor syndrome C. 2536 hepatic excretory function ? A. 2534 may be increased by ? A. Benign Recurrent Intrahepatic Cholestasis D. Dubin-Johnson syndrome G B. Coproporphyrin isomer II C. and pregnancy. 2536 C. Rotor Syndrome also a consequence of an FIC1 mutation and may progresses to malnutrition. Hepatitis A virus A. autosomal recessive disorder clinically similar to DJS.October 01. Hepatitis B is a DNA virus. Hepatitis A virion C. Hepatitis B virion D. Laboratory findings include D. Total urinary coproporphyrin excretion normal A. This is C. total coproporphyrin content is normal. Progressive Familial Intrahepatic Cholestasis type 2 Harrison’s 18th Ed. 2536 A. Hepatitis D virus D. Elevations in alkaline phosphatase B. aminotransferase and alkaline phosphatase levels. C. February 04 D. Recurrent attacks of pruritus and jaundice 683 DANE particle is the name given to ? B. C. Hepatitis A A. in contrast to the normal levels seen in DJS. BRIC is an autosomal recessive benign disorder in that it does not lead to cirrhosis or end-stage liver disease. Progressive Familial Intrahepatic Cholestasis type 1 A.Acute viral hepatitis B. Lucey-Driscoll syndrome D. 685 Which of the following viruses is a DNA virus ? 679 Gene FIC1 is mainly expressed in ? Harrison’s 18th Ed. D. Hepatitis C virus C. 676 Which of the following is false about Rotor Syndrome ? Harrison’s 18th Ed. Total urinary coproporphyrin excretion is substantially increased in Rotor syndrome. Harrison’s 18th Ed. 2536 A. Dane and others discovered the HBV virus particle in 1970 by electron microscopy. Crigler-Najjar Syndrome D. in contrast to B. Does not lead to cirrhosis C. 678 Mutation in which of the following genes produce the Benign 684 Which of the following is not a RNA virus ? Recurrent Intrahepatic Cholestasis (BRIC) ? Harrison’s 18th Ed. In urine from DJS patients. A. 2537 Harrison’s 18th Ed. Gilbert’s Syndrome C. Hepatitis B B. Gilbert’s Syndrome Byler disease is also termed as Progressive Familial Intrahepatic Cholestasis (FIC) type 1. Gene FIC1 is mainly expressed strongly in the small intestine but only weakly in the liver. a protein that is essential for normal hepatocellular excretion of phospholipids across the bile canaliculus. Hepatitis B virus B. World Hepatitis Awareness Day . Heart Hepatitis B is a DNA virus. Progressive Familial Intrahepatic Cholestasis type 4 B. FIC1 A. 75% of the coproporphyrin in 680 Byler disease is also known as ? urine is isomer III. Progressive Familial Intrahepatic Cholestasis type 3 A. Progressive FIC type 3 has been associated with a mutation of MDR3. World AIDS Day - Intrahepatic Cholestasis (BRIC) ? December 1. May 19 677 Which of the following is false about Benign Recurrent World Hepatitis Day . MRP2 D. World Cancer Day . Liver B. Progressive Familial Intrahepatic Cholestasis type 2 C. Crigler-Najjar Syndrome B. Liver normal in appearance 682 World Hepatitis Day is observed on ? D. D. 2536 A.S. Small intestine C. but >80% of Harrison’s 18th Ed. Gallbladder visualized on oral cholecystography C.February 04. 2537 Harrison’s 18th Ed. All of the above 681 Mutation of MDR3 gene results in ? 675 Rotor Syndrome is clinically similar to ? Harrison’s 18th Ed. 2536 A. Normal serum aminotransferase levels A. 2537 the total is isomer I. Conjugated hyperbilirubinemia 304 . Hepatitis D D. Hepatitis C C. 445 Gastroenterology MCQ’s MCQ’s FOR FORPROFESSIONALS MEDICAL MEDICAL PROFESSIONALS BY PROF. 2537 Harrison’s 18th Ed. October 01 In Rotor syndrome. Progressive Familial Intrahepatic Cholestasis type 1 674 Which of the following is an autosomal recessive disorder ? B. December 01 nonvisualization that is typical of DJS.May 19. Progressive Familial Intrahepatic Cholestasis type 4 Rotor Syndrome is a benign. Kidney D. Gene FIC1 is mutated in patients with BRIC. and end-stage liver disease during childhood. Normally. Progressive Familial Intrahepatic Cholestasis type 3 D. D. Dubin-Johnson Syndrome B. the gallbladder is visualized on oral cholecystography. Hepatitis D virion elevations in serum conjugated bilirubin. AJAY MATHUR Cardiology 445 Naturally occurring coproporphyrin isomers are I and III. ABCC2 B. . 2536 A. growth retardation. NTCP C. Hepatitis C virion BRIC is characterized by recurrent attacks of pruritus and jaundice. E human hepatitis viruses are RNA viruses. 695 Which of the following has the largest viral genome size ? or by ultraviolet irradiation. But. Table 304-1 A. Harrison’s 17th Ed. HP1 to HP4 C. Formaldehyde C. S.4 kb. ~ 2 weeks C. Harrison’s 18th Ed. Hepatitis E n C. T. AJAY MATHUR Gastroenterology 686 Which of the following about Hepatitis A virus is false ? 692 After acute illness. Hepatitis B Harrison’s 18th Ed. Figure 304-1 Harrison’s 18th Ed. viremia and infectivity diminish rapidly 698 The four overlapping genes of encoding proteins in Hepatitis once jaundice becomes apparent B virus are ? Harrison’s 18th Ed. bile. These are cleaved posttranslationally from the polyprotein product of a 7500-nucleotide genome. Indefinitely Hepatitis A virus is a nonenveloped 27-nm. 2538 D. 2539. 2538 9 A. Hepatitis C D. Hepatitis A Hepatitis A has an incubation period of ~4 weeks. 2537 A. 1937 h A. B. 3 months C. None of the above A. Virus is present in liver. HCV is Hepacivirus and HEV is a Hepevirus.5 kb.7 kb. Hepatitis A 689 Hepatitis A has an incubation period of ? B. the virus is present in liver. ~ 8 weeks Harrison’s 17th Ed. stools. HEV . Liver HAV . 2537. Hepadnaviruses Hepatitis A virus replication is limited to liver. P and X Despite persistence of virus in liver. 1937 - A. SP1 to SP4 B. designated VP1 to VP4. and infectivity diminish rapidly once jaundice becomes apparent. Hepatitis B ta 690 Replication of Hepatitis A virus occurs in ? C. Viral shedding in feces.9. 2538 B. and blood during the late incubation period and acute preicteric phase of illness. Hepatovirus C. ~ 4 weeks r C. 2538 D. Bile C. All of the above Harrison’s 18th Ed. viremia. HDV B. acid. 2537. Blood 697 HBV belongs to which family of viruses ? D.6 kb. incubation period & acute preicteric phase of illness C. U and V . VP1 to VP4 D. C. stools and blood during late resembles viroids and plant satellite viruses.446 MCQ’s FOR MEDICAL PROFESSIONALS BY PROF. Chlorine D. 687 Four capsid polypeptides of Hepatitis A virion are designated 693 Which of the following has the largest virus particle size ? as ? Harrison’s 18th Ed. Hepatitis E 9 B.7.1. B. HDV . Hepatitis A d 688 Inactivation of Hepatitis A virus can be achieved by ? B. 2538 D. bile. Acid resistant B. Hepatitis C virus (55-nm). HCV . Hepatitis A A. Hepatitis D Harrison’s 18th Ed. HBV 3. Hepevirus A.2 kb. Ether resistant C. Hepatitis C B. S. B. B. HAV can be cultivated reproducibly in vitro HBV belongs to hepadnaviruses family of viruses. anti-HAV of the IgG class remains detectable indefinitely. 12 months D. viral shedding in feces. Hepatitis D virus (35–37 nm) and Hepatitis E virus (32–34 nm) D. 2537 for ? A. HAV is a Hepatovirus. After acute illness.7. Figure 304-1 A. Hepatitis C Harrison’s 18th Ed. Hepatitis E A. anti-HAV of IgG class remains detectable Harrison’s 18th Ed. by formaldehyde and chlorine. Hepatitis E R D. Hepatitis C C. Hepatitis D G B. 6 months D. Enveloped Harrison’s 18th Ed. Hepacivirus 691 Which of the following statements about Hepatitis A virus is false ? Harrison’s 18th Ed. All of the above U Inactivation of viral activity can be achieved by boiling for 1 minute. ~ 6 weeks 696 Which of the following has the smallest viral genome size ? i D. WP1 to WP4 694 Which of the following has the smallest virus particle size ? V Hepatitis A virion contains four capsid polypeptides. Hepatitis B virus (42-nm). Ultraviolet irradiation Hepatitis A virus (27-nm). heat. 2537 ti e A. Heat resistant A. type 1 (“hepa” from hepatotrophic and “dna” because it is a DNA virus). and ether-resistant RNA virus in the hepatovirus genus of the picornavirus family. or HBsAg) in the blood of Australian aboriginal people.351:2832-8 A. Blood contains highest concentrations of virus B. who is related to the discovery of Australia C. C gene codes for HBeAg & HBcAg antigen ? A. 27-nm 701 Which of the following is false about HBV genome ? C. Enveloped. C In Hepatitis B virus. Pekin ducks Harrison’s 18th Ed. Figure 304-3 . Baruch Blumberg The S gene codes for the “major” envelope protein. common group-reactive antigen. a 702 Which of the following about Hepatitis B virus is false ? B. 2538 A. M and Z A. Figure 304-3 D. 22-nm B. Partially double-strand and partially single-strand Of the three particulate forms of HBV. 2538. P. 8 B. Breast milk can cause viral transmission In 1988. C gene codes for nucleocapsid proteins .350:1118-29 genotype F. Zuckerman AJ 707 Which particulate form of HBV is most numerous in blood ? In 1965. Harrison’s 18th Ed. S D. hepatitis B virus (HBV) was classified into four genotypes by a sequence divergence in the entire genome exceeding 8%. Baruch Blumberg. A. and designated by capital letters of the alphabet from A to D. X gene codes for HBxAg C. discovered the Australia Harrison’s 18th Ed. AJAY MATHUR Cardiology 447 C. A. Covalently closed circular DNA (cccDNA) is formed in A. 2538 antigen (later known to be Hepatitis B surface antigen. Bouchard MJ which can transactivate the transcription of cellular and viral genes D. 12 C. 8 Genotypes of HBV were defined by a sequence divergence greater than 8% in the entire genome. the most numerous are the 22-nm particles. working at National Institutes of Health (NIH). HBsAg. 4 A. Q. C. double-stranded DNA virus C. Alter MJ D. Primary reservoir is chronically infected people In different HBsAg subdeterminants. Genotype H is phylogenetically closely related to N Engl J Med 2004. S gene codes for HBsAg B. and Y B. C. P gene codes for DNA polymerase 700 Out of the following. 2538. codes for DNA polymerase. None of the above subdeterminants ? Hepatitis B is a non-retroviral virus which uses reverse transcription for its replication. 2538 N Engl J Med 2004. 704 Which of the following about Hepatitis B virus is false ? Later four more were added from E to H. 4 C. A.HBeAg and HBcAg. Risk of development of chronicity is directly related to age at time of infection D. c B. 447 Gastroenterology MCQ’s MCQ’s FOR FORPROFESSIONALS MEDICAL MEDICAL PROFESSIONALS BY PROF. the four overlapping genes encoded by the genome are S. Smallest dsDNA virus known to infect humans D. S. Woodchucks 706 Which of the following statements about overlapping genes in B. O. d C. which appear as spherical or long filamentous forms. 705 Which is the largest overlapping gene in Hepatitis B virus ? Harrison’s 18th Ed. transcription 708 Which is the common group-reactive antigen in different HBsAg D. 42-nm Harrison’s 18th Ed. b N Engl J Med 2004. Ground and tree squirrels Hepatitis B virus is false ? C. Circular genome D. “a” is shared by all HBsAg isolates. 2 hepatocyte nucleus B. Replicates through an RNA intermediate form by reverse envelope protein of HBV and are thought to represent excess viral envelope protein. All of the above A. S. 2538 The P gene of HBV is the largest gene and codes for DNA polymerase. C. Insects can transmit HBV 709 Number of genotypes if Hepatitis B isolates is ? 703 Which of the following about Hepatitis B virus is false ? Harrison’s 18th Ed. Not transmitted by fecal-oral route C. 6 D. The largest gene. P and X. Replication of DNA genome by reverse transcription of RNA intermediate 710 Genotypes of HBV were defined by a sequence divergence greater than what percentage in the entire genome ? B. Most abundant protein in HBV genome is S protein C. X gene codes for HBeAg B. X 699 Hepadnaviruses also infect which of the following ? Harrison’s 18th Ed. 2538 A. All of the above B. 16 D. D.351:2832-8 A. P D. These are antigenically indistinguishable from outer surface or C. Vertical transmission from mother to child possible D. Persistence of HBeAg in serum beyond the first three months of acute infection may be predictive of the development of chronic infection. 2540 Harrison’s 18th Ed. HBeAg has a signal peptide that binds C. HBsAg. Major protein B. Large protein B. D. 2539 721 Persistence of HBeAg in serum beyond how many months of A. and into eight genotypes (A-H) according to overall nucleotide sequence variation of the genome. Anti-HBeAg positivity usually 715 The protein product of pre-S1 plus pre-S2 plus S regions in indicates low infectivity and no replication. HBsAg B. ayw and ayr are readily determined by immunological Harrison’s 18th Ed. B. HBcAg Harrison’s 18th Ed. HBeAg provides a convenient. Large protein Harrison’s 18th Ed. Hepatitis B virus is ? Harrison’s 18th Ed. pre. HBcAg C. add C. AJAY MATHUR Gastroenterology 711 Which of the following is not a major serotype of HBV ? A. HBeAg and HBcAg. All of the above 717 Which of the following is not a nucleocapsid protein ? HBcAg is intracellular. Genotypes of HBV were defined by a sequence divergence > 8% in the 718 Which of the following represents excess virus coat material ? entire genome. All of the above HBsAg-positive serum containing HBeAg is highly infectious and associated with presence of hepatitis B virions than HBeAg-negative or anti-HBe-positive serum. HBcAg C. pre-S2. D. HBsAg D. If translation is initiated at the D. 2540 C. ayw precore region of C gene. . “Middle” protein is the product of pre-S2 + S. “Large” protein is the product of pre-S1 + pre-S2 + S. and they were regarded the phenotypic expression of HBV genotypes. HBsAg serotypes adw. All of the above B. HBV virus is divided into 4 major serotypes (adr. HBeAg B. D. 2539 D. naked HBcAg particles Harrison’s 18th Ed. HBsAg A. 1 month C. qualitative marker of HBV replication and relative infectivity. HBsAg gene is one long open reading frame but contains 3 in frame “start” (ATG) codons that divide the gene into 3 sections. 2538 B. adr and ayr. ayw) based on antigenic epitopes present on its envelope proteins. HBeAg appears transiently and early during acute hepatitis B infection. 2540 B. HBxAg A. Any of the above S1. HBxAg B. ayr The C gene codes for two nucleocapsid proteins. S gene codes for the “major” envelope protein . HBeAg 712 The protein product of S region in Hepatitis B virus is ? C. All HBV isolates of genotype A or B isolates were adw and all genotype D isolates were ayw. HBcAg is the protein product. secreted nucleocapsid protein. Middle protein C. HBsAg. Major protein acute infection is predictive of chronic infection ? Harrison’s 18th Ed. 716 HBsAg is also called ? Harrison’s 18th Ed. HBcAg B. 2539 722 Which of the following particles of HBV do not circulate in the A. HBcAg A. is the product of the S gene of HBV. Therefore. HBsAg Harrison’s 18th Ed. ayr. Major protein D. Large protein D. adw. HBsAg were heterogeneous and covered adw. and S. Isolates of genotype C A. 3 month The envelope protein. 2539 methods. Middle protein A.448 MCQ’s FOR MEDICAL PROFESSIONALS BY PROF. All of the above 720 Which of the following is qualitative marker of HBV replication 714 The protein product of S region plus pre-S2 region in Hepatitis & relative infectivity ? B virus is ? Harrison’s 18th Ed. 2539 C. Middle protein Harrison’s 18th Ed. Large protein A. HBxAg D. All of the above C. 2539 D. Major protein serum ? B. Middle protein nucleocapsid protein ? C. Major protein 719 In Hepatitis B virus. 2539 do not circulate in serum and are not detectable routinely in serum of patients with HBV infection. All of the above A. adr. the protein product is HBeAg. HBeAg A.HBsAg. HBeAg 713 The protein product of S gene in Hepatitis B virus is ? Harrison’s 18th Ed. Middle protein HBeAg is a soluble. it is within an HBsAg coat. 2 month D. All of the above C. When in serum. readily detectable. Large protein it to the smooth endoplasmic reticulum and leads to its secretion into the circulation. unless patient is precore mutant. while if translation begins with the core region of C gene. HBeAg The S gene codes for the “major” envelope protein. adw D. which of the following is a ‘secreted’ B. 2539 A. All of the above D. HBxAg C. Circulating HBsAg follow elevations of SGOT/SGPT A. . None of the above C. Anti-HBs Ag B. HBcAg Hepatitis B x antigen (HBxAg) activates signal-transduction pathways that lead to stimulation of D. HBeAg D. Anti-HBe Ag A. C D. Gene P of the HBV genes is the largest and codes for DNA polymerase which has both DNA- dependent DNA polymerase and RNA-dependent reverse transcriptase activities. HBsAg becomes undetectable 1 . & develop antibodies against HBeAg & HBsAg (anti-HBe & anti-HBs) at later stages. P Circulating HBsAg precedes elevations of serum aminotransferase activity and clinical symptoms D. AJAY MATHUR Cardiology 449 723 Hepatitis B patients contain which of the following circulating 729 Which of the following is transactivated by expression of antibodies ? HBxAg ? Harrison’s 18th Ed. Anti-HBs D. HBeAg A. transcription and HBV DNA replication ? Harrison’s 18th Ed. 2540 B. the first virologic marker detectable in serum is ? A. 2540 Harrison’s 18th Ed. D. the first virologic marker detectable in serum within 1-12 weeks.2 months after the onset of jaundice and rarely 726 Which of the following HBV antigens stimulates HBV reverse persists beyond 6 months. HBxAg Of the 3 antibodies against HBV. 2540 Harrison’s 18th Ed. HBxAg C. clinical association is observed last. 2540 Harrison’s 18th Ed. HBeAg B. HBxAg D. anti-HBs by weeks to months. 2540 B. Anti-HBc appears in serum within the first 1 . None of the above D. 2540 B. Anti-HBe Expression of HBxAg induces programmed cell death (apoptosis). Upon HBV infection. Anti-HBs becomes detectable after HBsAg disappears in serum and remains detectable indefinitely thereafter. 2540 733 Of the following antibodies against HBV.6 weeks and remains detectable during the entire icteric or symptomatic phase of acute hepatitis B and beyond. whereas anti-HBs antibody is detected Because HBxAg transactivation enhances replication of HBV. All of the above Hepatitis B patients contain circulating antibodies against HBcAg (hepatitis B core antigen). 3 months B. HBeAg A. programmed cell death (apoptosis) ? Harrison’s 18th Ed. Transactivation of class I major histocompatibility genes D. HBcAg C. 2540 A. Circulating HBsAg coincide with elevations of SGOT/SGPT B. Anti-HBc A. HBeAg C. HBcAg D. HBxAg HBV reverse transcription and HBV DNA replication. S C. 730 Which of the following statements is true ? Harrison’s 18th Ed. All of the above In typical cases. None of the above 728 Clinical association is observed between expression of which 734 Of the following antibodies against HBV. HBsAg Harrison’s 18th Ed. 9 months D. X by 2 . 449 Gastroenterology MCQ’s MCQ’s FOR FORPROFESSIONALS MEDICAL MEDICAL PROFESSIONALS BY PROF. Transcription & replication of HIV B. HBsAg B. usually 727 Expression of which of the following HBV antigens induces between 8-12 weeks is HBsAg. Anti-HBs B. HBsAg rarely persists beyond how many months ? 725 Which of the following is a nonparticulate protein of HBV ? Harrison’s 18th Ed. HBcAg B. 2540 A. 731 After the onset of jaundice. 2540 A.2 weeks after appearance of HBsAg & preceding between its expression with severe chronic hepatitis and hepatocellular carcinoma. HBsAg Harrison’s 18th Ed. HBcAg B. Transactivation of human interferon gene C. Anti-HBc C. HBsAg C. Anti HBcAg C. Anti-HBe C. 2540 724 Which of the following HBV genes codes for DNA polymerase ? A. which one is detected of the following & severe chronic hepatitis & hepatocellular last ? carcinoma ? Harrison’s 18th Ed. 6 months C. which one is first to appear ? A. Circulating HBsAg precedes elevations of SGOT/SGPT Harrison’s 18th Ed. 2540 732 After infection with HBV. anti-HBc develops first. 1 months A. HBeAg is translated from precore mRNA. using ATG at 1814. beginning within first 1-2 weeks after appearance of HBsAg & preceding detectable levels of anti-HBs by weeks to months. Anti-HBs appears after HBsAg disappears D. HBeAg-positive . anti-HBs & anti-HBc persist indefinitely. Detectable during entire icteric phase C. Presence of IgM anti-HBc A. using ATG at 1814 anti-HBs. and IgM class of anti- HBc antibodies. HBcAg naked core particles do not circulate in serum & therefore. 2540 B. Shorter N-terminus and shorter C-terminal tail D. Core protein is translated from pregenomic mRNA. Anti-HBc is readily demonstrable in C. HBeAg appears concurrently with or shortly after HBsAg. using the ATG codon at 1901 as initiation site. whereas IgG anti-HBc predominates weeks and remains detectable during the entire icteric phase of acute hepatitis B and beyond. beyond 6 months. A. Anti-HBs B. G1896A nonsense mutation in the Harrison’s 18th Ed. Shorter N-terminus and longer C-terminal tail Isolated anti-HBc represent hepatitis B infection in the remote past. A is false ? B. using D. HBeAg appears with HBsAg A. in persons who have recovered from hepatitis B. Anti-HBs D. infection in remote past ? HBeAg is translated from the precore mRNA. HBeAg is not part of the virus particle Harrison’s 18th Ed. All of the above the ATG codon at 1901 as initiation site At times a gap of several weeks or longer may separate disappearance of HBsAg & appearance of B. IgM anti-HBc D. HBsAg C. 739 Isolated presence of which of the following suggests hepatitis 745 Which of the following statements about HBV is false ? B infection in remote past ? A. loss of HBsAg & rise of the anti-HBs antibody indicate recovery from Harrison’s 18th Ed. 2540 precore region specifically prevents translation of HBeAg. Generally. Absence of anti-HBs and HBeAg ? C. G1896A nonsense mutation in precore region specifically has been implicated in development of transfusion-associated hepatitis B. anti-HBs and anti-HBc persist indefinitely. In intermediate stage. Presence of HBsAg After HBsAg disappears. 2540 infection. A. Anti-HBc antibody rises soon after infection A. HBeAg differs from core protein (HBcAg) by a longer N-terminus and shorter C-terminal tail. Any of the above C. IgM anti-HBc predominates D. HBeAg expression is not essential for virus replication B. Any of the above B. Rarely persists beyond 3 months D. Presence of anti-HBs antibody serum.450 MCQ’s FOR MEDICAL PROFESSIONALS BY PROF. patients lose HBeAg. D. Precedes rise in ALT A. C A. 2540 Harrison’s 18th Ed. A. Isolated anti-HBc does not indicate active virus replication but indicates HBV infection in remote past. Anti-HBs remains detectable indefinitely C. Absence of HBsAg 743 Which of the following is false about expression of core protein B. D. IgG anti-HBc 746 HBeAg-negative chronic hepatitis B or e-CHB is characterised by all except ? After HBV infection. All of the above Circulating HBsAg precedes rise in serum aminotransferase levels & clinical symptoms by 2-6 IgM anti-HBc predominates in first 6 months after acute infection. Presence of HBeAg not detectable routinely in serum of patients with HBV infection. 741 Which of the following genotype of HBV is frequent in India ? 736 Which of the following about serology in acute HBV infection A. 2540 C. 2540 A. IgG anti-HBc D. HBsAg-positive for at least 6 months B. anti-HBc may be the only serologic evidence of current or recent HBV infection & blood containing IgM anti-HBc in the absence of HBsAg & anti-HBs C. B Harrison’s 18th Ed. Anti-HBc 744 HBeAg differs from HBcAg by a ? B. HBcAg is B. Anti-HBs A. AJAY MATHUR Gastroenterology 735 Which of the following about HBsAg in acute HBV infection is 740 Recent and remote HBV infections can be distinguished by false ? determination of ? Harrison’s 18th Ed. HBsAg becomes undetectable 1-2 months after onset of jaundice & rarely persists beyond 6 months. Precedes clinical symptoms B. Anti-HBs precedes appearance of anti-HBc 742 Which of the following is false about the serology in first stage of HBV patient ? D. During this “gap” or “window” period. None of the above C. None of the above 738 Presence of which of the following represents hepatitis B Core protein is translated from pregenomic mRNA. antibody to HBsAg (anti-HBs) becomes detectable in serum & remains detectable indefinitely. Anti-HBe B. anti-HBc may persist in circulation longer than anti-HBs. IgM anti-HBc C. Finally. Longer N-terminus and shorter C-terminal tail during the first six months after acute infection. Presence of IgM anti-HBc antibody 737 Which of the following is true during “gap” or “window” period First stage of HBV infection is characterized by presence of HBsAg. HBeAg. HBV DNA may be low or prevents translation of HBeAg undetected. whereas IgG anti-HBc persists beyond six months. In persons who have recovered from hepatitis B. develop anti-HBe antibodies & enter in acute HBV infection ? into clinical remission. Longer N-terminus and longer C-terminal tail C. during which all three forms of HBV circulate. Mediterranean countries Seroconversion from HBeAg-positive to anti-HBe-positive is accompanied by a transient. Re-expression of HBeAg Harrison’s 18th Ed. Undetectable after disappearance of HBsAg of pre-S1 and pre-S2 proteins marking high levels of virus replication and presence of circulating intact virions and detectable HBV DNA. and active liver B. Detectable HBV DNA C. liver histology showing chronic hepatitis with or without cirrhosis). Undetectable after peak rise in ALT HBeAg. In chronic HBV infection. HBeAg B. HBeAg is a D. All of the above C. 2541 B. HBeAg becomes undetectable 748 Which of the following is the qualitative marker of the shortly after peak elevations in aminotransferase activity. HBsAg remains detectable beyond 6 months. Re-expression of HBV DNA A. South America D.e. HBcAg A. HBeAg Harrison’s 18th Ed. 10 % per year A. HBeAg- negative. None of the above qualitative marker & HBV DNA a quantitative marker of replicative phase. HBeAg presence coincides with high levels of virus replication and reflects the presence of circulating intact virions and detectable HBV DNA. HBxAg C. Pre-S1 proteins A. anti-HBe-positive. acute hepatitis-like elevation in aminotransferase activity. Appearance coincides with high viral replication D. Patients with pre S1 mutations D. Patients with core mutations B. 2541 B. 2541 B. before the disappearance of HBsAg. Pre-S2 proteins B. with HBV DNA detectable in serum unamplified assays. Anemia B. Acute elevation in aminotransferase activity B in which region of the world ? Harrison’s 18th Ed. C. None of the above accompanied by re-expression of HBeAg and HBV DNA. 2541 A. nonreplicative HBV infection converting back to replicative infection is D. 751 Spontaneous reactivation of replicative HBV infection from nonreplicative phase is marked by which of the following ? 757 Which of the following is false about severe chronic HBV Harrison’s 18th Ed. North America C. Anti-HBe positive Spontaneous reactivations i. 750 Seroconversion from HBeAg-positive to anti-HBe-positive is accompanied by which of the following ? 756 HBeAg-negative chronic hepatitis with mutations in the precore Harrison’s 18th Ed. 20 % per year B. 2541 B. HBeAg becomes undetectable after peak rise in ALT e-CHB or HBeAg-negative chronic hepatitis B patients are HBsAg-positive for at least 6 months. Anti-HBe becomes detectable before disappearance of HBsAg 747 Which of the following is expressed during periods of peak D. HBsAg Harrison’s 18th Ed. Anti-HBs low to absent Replicative stage of HBV infection is the time of maximal infectivity & liver injury. appears concurrently with or shortly after HBsAg and its appearance coincides expression D. and anti-HBs is either undetectable or detectable at low levels. All of the above C. 2541 754 Which of the following about chronic HBV infection is false ? A. Appears with HBsAg C. HBeAg becomes undetectable before disappearance of HBsAg disease (elevated AST or ALT. Reappearance of IgM anti-HBc B. coinciding with a period of relatively lower infectivity. believed to reflect cell-mediated immune clearance D. 2541 region is now the most frequently encountered form of hepatitis A. Patients with pre S2 mutations Replicative phase of chronic HBV infection converts to a relatively nonreplicative phase at a rate of 10% per year accompanied by seroconversion from HBeAg-positive to anti-HBe-positive. HBsAg detectable > 6 months C. Southeast Asia of virus-infected hepatocytes. 30 % per year C. IgG anti-HBc present D. 749 Replicative phase of chronic HBV infection converts to relatively nonreplicative phase at a rate of ? 755 Replicative chronic hepatitis B in the absence of HBeAg occurs Harrison’s 18th Ed. 2541 D. Hemoglobinuria A. In self-limited HBV infections. All of the above C. Replicative chronic hepatitis B in the absence of HBeAg occurs in patients with precore mutations who cannot synthesize HBeAg. Anti-HBe-positive 752 HBV infections is self-limited if ? Harrison’s 18th Ed. 2541 in which of the following situations ? Harrison’s 18th Ed. Patients with precore mutations C. HBeAg negative D. 451 Gastroenterology MCQ’s MCQ’s FOR FORPROFESSIONALS MEDICAL MEDICAL PROFESSIONALS BY PROF. anti-HBc is primarily of the IgG class. and anti-HBe then becomes detectable. . including intact virions. 2541 infection due to precore region HBV mutant ? A. replicative stage of HBV infection ? Harrison’s 18th Ed. AJAY MATHUR Cardiology 451 C. HBV DNA detectable in serum unamplified assays A. IgM anti-HBc and exacerbations of liver injury. 2541 753 Which of the following about HBeAg in acute HBV infection is false ? A. 40 % per year D. All of the above replication ? Harrison’s 18th Ed. HBeAg-negative chronic hepatitis with mutations in the precore region is now the most frequently encountered form of hepatitis B in Mediterranean countries and in Europe. or both. Single base substitution HDV genome is a small. serves as a template for subsequent genomic RNA synthesis. It is formalin-sensitive. Single amino acid substitution 764 HDV RNA requires which of the following for its replication ? Harrison’s 18th Ed. This is an example of single base substitution. Hepatitis D virus 766 Which of the following is an HDV protein ? Harrison’s 18th Ed. single-strand RNA. 35. B. anti-HDV detected before symptoms appear 760 Which of the following is a member of the genus Deltavirus ? In acute HDV infection. extrahepatic site where Hepatitis B antigens and HBV DNA have been identified include lymph nodes. Defective RNA virus 767 Which of the following hepatitis was earlier called “non-A. Host RNA polymerase I In escape mutants of HBV there occurs a single amino acid substitution. Has antigenic homology with HBV antigens A. . 2542 D. from G to A. Bone marrow 765 Which of the following is false about delta hepatitis virus ? Harrison’s 18th Ed. All of the above C. HDeAg Delta hepatitis agent (HDV) is the only member of the genus Deltavirus. Pancreas B. The antigenomic been identified is ? RNA. 1986 A. HDsAg D. circular. HBcAg B. S replacement of the TGG tryptophan codon by a stop codon (TAG). Harrison’s 18th Ed. non-B hepatitis”. HBeAg D. D. AJAY MATHUR Gastroenterology Molecular variants of HBV may not express typical viral proteins i. except for a small area of the polymerase gene. 1700-nucleotide. California encephalitis virus B. Hepatitis E Before its identification. All of the above 761 Which of the following is false about delta hepatitis virus ? HDV RNA has only one open reading frame. Host RNA polymerase II underwent the procedure for hepatitis B and who were treated with a high-potency human monoclonal anti-HBs preparation. Intracellular replication of HDV RNA can occur without HBV D. a product of the antigenomic Harrison’s 18th Ed. 2541 A. from glycine to arginine at position 145 of the immunodominant “a” determinant common to all subtypes of B. Hepatitis C genome is a 1700-nucleotide. 763 The single-stranded RNA genome of HDV is homologous to an envelope proteins.e. Viral RNA polymerase II HDV RNA requires host RNA polymerase II for its replication via RNA-directed RNA synthesis by 759 Extrahepatic site where Hepatitis B antigen and HBV DNA has transcription of genomic RNA to a complementary antigenomic (plus strand) RNA. 2541 D. Severe chronic HBV infection due to precore region HBV mutantion extent with which gene of HBV ? have detectable HBV DNA (> 105 copies/ml). D. 2542 A. All of the above C. P 758 HBV escape mutants best relate to which of the following ? Harrison’s 18th Ed. Marburg virus A. Viral RNA polymerase I HBsAg. 2542 is incapable of encoding HBeAg. 1989 B. This HBV/a mutant is seen in active and passive immunization. 2542 strand is the only known HDV protein.452 MCQ’s FOR MEDICAL PROFESSIONALS BY PROF. X A. Its C. and anti-HBe but no HBeAg as the mutant virus Harrison’s 18th Ed. which occurs in the second to last codon of pre-C gene at nucleotide 1896 results in the A. 2542 A. HBsAg C. D. C C. anti-HDV is detected 30-40 days after symptoms appear. This change in HBsAg leads to a loss of neutralizing activity by anti-HBs. Harrison’s 18th Ed. and infection pancreas. 762 The delta core of HDV is “encapsidated” by an outer envelope of ? 768 Hepatitis C virus was first identified in which year ? Harrison’s 18th Ed. Hepatitis A HDV is a defective RNA virus that coinfects with and requires the helper function of HBV for its B. Delta antigen bears no antigenic homology with any of the HBV antigens. Any of the above Hepatitis C virus was first identified in 1989. In acute HDV infection. 35. bone marrow. and in liver transplant recipients who C. Duration of HDV infection determined by duration of HBV Although not associated with tissue injury. D. HDV antigen is expressed in hepatocyte nuclei C. Spleen A. 2542 D. Hepatitis C virus was labeled as “non-A. circulating lymphocytes. 1992 C. 1995 D. and delta antigen (HDAg). non- B. The delta core of HDV is “encapsidated” by an outer envelope of HBsAg quite like that of HBV. 2542 B. circular. in turn. B. which prevents the translation of HBeAg. single-strand RNA of negative polarity that is nonhomologous with HBV DNA. Single base addition C. spleen. Hepatitis B replication & expression. A. 1700-nucleotide genome Harrison’s 18th Ed. HDAg C.to 37-nm virus with a hybrid structure. nucleocapsid proteins. Single amino acid addition A.to 37-nm in size B hepatitis” ? C. 2542 B. Lipoprotein of the genus Hepacivirus in the family Flaviviridae. 2543. Harrison’s 18th Ed. p7 D. Dengue virus. 30 % D. NS4 and NS4B. Most sensitive indicator of HCV infection is presence of HCV RNA. Hepatitis B HCV RNA C. 2543. C Harrison’s 18th Ed. HCV however is the only member C.60 nm in diameter B. 4 The three structural genes at the 5' end of hepatitis C virus genome are C . All of the above C. E2 C. 2542 A. NS3. 2543 A. Hepatitis A C. Figure 304-6 776 Number of HCV genotypes identified is ? A. International units (IUs) per milliliter A. NS3 B. p7 is a membrane protein that appears to function as an ion channel. HCV enters hepatocyte via CD81 receptor following ? B. non-A. HCV RNA is the first detectable event. which codes for a serine protease and an RNA helicase. Those with less differences in sequence homology are referred to as quasispecies. 2542 tight junction protein claudin-1. 3 D. E1 B. Hepatitis C virus Harrison’s 18th Ed. preceding alanine aminotransferase (ALT) elevation and the appearance of anti-HCV. West Nile virus. 771 In hepatitis C virus genome. virus genome ? Harrison’s 18th Ed. HCV RNA A. and NS5B. None of the above During acute hepatitis C progressing to chronicity. 40 % Placed adjacent to the structural proteins. 453 Gastroenterology MCQ’s MCQ’s FOR FORPROFESSIONALS MEDICAL MEDICAL PROFESSIONALS BY PROF. Microgram per milliliter B. Viroid B. A. 9600-nucleotide RNA virus C. NS4 C. Till date HCV genotypes identified are 6 as well as >50 subtypes within genotypes. that requires molecular amplification by PCR or transcription-mediated amplification (TMA). E1 A. 20 % C. and E1 and E2 . Figure 304-6 A. Figure 304-7 Harrison’s 18th Ed. 779 Which of the following is the first detectable event during acute hepatitis C progressing to chronicity ? 773 Which of the following is false about HCV ? Harrison’s 18th Ed. Hepatitis C D. 10 % B. 6 nucleocapsid. which codes for a cysteine protease. Dengue virus 775 HCV masquerades as which of the following ? C. CD81 receptor 770 Which of the following is the structural gene in hepatitis C HCV masquerades as a lipoprotein. NS5A. None of the above D. Nucleocapsid protein Members of family Flaviviridae are Yellow fever virus. 40 . Its half-life is 2. 2543 780 “Epidemic. 2543 B.7 hours D. D. 2543. 2543 D.367:1237-44. HCV infection does not induce lasting immunity against N Engl J Med 2012. Figure 304-6 Harrison’s 18th Ed. St. Most sensitive indicator of HCV infection is the presence of B. At 3' end are six nonstructural (NS) regions. 774 Which of the following is false about HCV ? Harrison’s 18th Ed. HCV DNA B. Genotypes differ one from another in sequence homology by 30%. 772 Which of the following nonstructural regions of hepatitis C 778 HCV RNA is reported as ? virus genome codes for RNA-dependent RNA polymerase ? Harrison’s 18th Ed. NS2. C A. Copies per milliliter C. 2543 Harrison’s 18th Ed. AJAY MATHUR Cardiology 453 769 Which of the following is a member of family Flaviviridae ? HCV gains entry into the hepatocyte via the nonliver-specific CD81 receptor and the liver-specific Harrison’s 18th Ed. 2543. Virions per milliliter D. Elevation and appearance of anti-HCV D.which codes for D. which codes for an RNA-dependent RNA polymerase. Yellow fever virus B. Hepatitis E . non-B hepatitis” relates to which of the A. NS5B HCV RNA is reported as international units (IUs) per milliliter.which code for envelope glycoproteins. 2 C. NS5A D. Hepatitis C virus (HCV) and Hepatitis G virus. Louis encephalitis virus. All of the above A. Elevated alanine aminotrans ferase (ALT) C. which of the following functions 777 HCV genotypes differ one from another in sequence homology as an ion channel ? by ? Harrison’s 18th Ed. 2543 reinfection A. E2 B. C. Harrison’s 18th Ed. N Engl J Med 2012. 2543 A. D. cysteine protease (Pro). encodes a structural protein whose function remains undetermined. Hepatitis B Harrison’s 18th Ed. All of the above C. ORF1 B. ORF2 C.367:1237-44 789 Which of the following about HEV is false ? A. 783 ORF1 encodes which of the following nonstructural proteins ? N Engl J Med 2012. 3% A. 2 to 4 weeks B. Dog 782 Which of the following about HEV is false ? N Engl J Med 2012. Helicase (Hel) B. waterborne illness similar to hepatitis A. 2543 C. Hepeviridae HEV is a small (32. 6 to 12 weeks D. Genotypes 3 and 4 are swine viruses. non-A. 0% 786 ORF gene relates to ? B. 2% Harrison’s 18th Ed.367:1237-44 A. non-B C. IgM antibody is undetectable during recovery C. 2543 Contributing to the perpetuation of HEV are animal reservoirs. ORF1 B. 791 Incubation period of acute hepatitis E infection is ? 785 In HEV. H. Hepatitis C A. AJAY MATHUR Gastroenterology Hepatitis E was initially identified in 1980 as “epidemic or enterically transmitted. ORF2 C. 3 to 8 weeks C. ORF1 also encodes three regions of unknown function (Y. enzymatic activities required for viral acute infection. 2543 C. 5% B. Bird In India. and X). Harrison’s 18th Ed. Fish D.to 34-nm). nonenveloped virus with a single-strand. Genotypes 3 and 4 are swine viruses ORF1 encodes nonstructural proteins namely methyl transferase (MT). but both fall rapidly after Largest of three ORFs. ORF3 D. positive-sense RNA genome N Engl J Med 2012. and RNA polymerase (Pol). Hepatitis E 781 In India. Genome is 7. All of the above Both IgM anti-HEV and IgG anti-HEV appear early during acute infection. Genotypes 1 and 2 are human viruses D.454 MCQ’s FOR MEDICAL PROFESSIONALS BY PROF. most common cause of acute hepatitis is ? HEV has three open reading frames (ORF) genes.367:1237-44 N Engl J Med 2012. positive-sense RNA genome C. A. Hepatitis E B. Methyl transferase (MT) N Engl J Med 2012. 2543 A. Swine D. Hepatitis A D.6 kb in length) which contains three partially overlapping open reading frames (ORFs) bracketed by short 5’ and 3’ nontranslated regions. 2543 A. The middle-sized open reading frame 2 (ORF2) gene in HEV encodes the nucleocapsid protein. an infectious. None of the above helicase (Hel). Rhabdoviridae (7. Genome contains three open reading frames (ORFs) B. None of the above D.367:1237-44. Hepatitis C hepatitis”.12 months.6 kb in length A.367:1237-44 B. enzymatic activities required for 790 Which of the following about HEV infection is false ? viral replication ? N Engl J Med 2012. Enveloped virus 788 Which of the following best relates to HEV ? B. D. which of the following genes encode the nucleocapsid N Engl J Med 2012. reaching low levels within 9 . ORF3 D. enterically transmitted HEV is the most common cause of acute hepatitis.367:1237-44. Arenaviridae HEV was the first member to be identified in the Hepeviridae family. All of the above Acute hepatitis E has an incubation period of 3 to 8 weeks.367:1237-44. 2543 C. Hepatitis B . Harrison’s 18th Ed. Single-stranded. 1 to 2 weeks Harrison’s 18th Ed.367:1237-44 protein ? A. HEV replicates in cytoplasm C. HEV RNA is detectable in stool during incubation period A. Four genotypes of HEV have been categorized into two major groups. 784 Which of the following ORF in Hepatitis E virus genome encodes the nonstructural. Hepatitis A 787 Which of the following acts as an animal reservoir contributing to the perpetuation of HEV ? B. Genotypes 1 and 2 are human viruses that cause epidemic hepatitis with waterborne and fecal-oral transmission. replication. most notably in swine. HEV replicates in cytoplasm. D. HEV RNA is detectable in serum during incubation period B. Cysteine protease (Pro) A. 792 Average case fatality rate in acute HEV infections is ? ORF1 encodes nonstructural proteins involved in virus replication. Flaviviridae D. Harrison’s 18th Ed. ORF1 encodes the nonstructural. The smallest ORF3. 800 Which of the following HLA allele has been linked with self- 794 Chronic infection in Hepatitis E has been identified almost limited hepatitis C ? exclusively among ? Harrison’s 18th Ed. 2545 795 Which of the following statements is false ? A.367:1237-44 A. 2544 Harrison’s 18th Ed. Pork eaters D. Autochthonous hepatitis E has frequent serious complications. Most characteristic histologic feature of chronic HBV infection is “ground-glass hepatocyte” due to intracellular 803 Which of the following is the pathognomonic manifestation of accumulation of HBsAg HCV infection ? D. 2544 N Engl J Med 2012. All of the above C. accompanied by fluctuating. or autochthonous hepatitis E. peginterferon indicated B. D. Bell’s palsy. C. None of the above A. neurologic disorders (polyradiculopathy. AJAY MATHUR Cardiology 455 Acute hepatitis E is mostly self-limited without progression to chronic hepatitis. patients receiving cancer chemotherapy. Cytopathic role of virus Harrison’s 18th Ed. Never go into cirrhosis & hepatocellular carcinoma except ? Neonatally acquired HBV infection can may culminate into cirrhosis & hepatocellular carcinoma. but pathognomonic manifestation of HCV. Immunocompromised persons C/C haplotype of the IL28B gene has been linked with self-limited hepatitis C. and HIV-infected persons). Essential mixed cryoglobulinemia (EMC) hepatitis B e antigen (HBeAg) D. HBcAg invites cytolytic T cells to destroy HBV-infected D. Infection with precore genetic mutants of HBV Necrolytic acral erythema is a rare.mediated tissue damage plays a pathogenetic role in the extrahepatic C. All of the above hepatocytes Immune complex .mediated tissue damage A. the average age was more than 60 years. ribavirin. All of the above Harrison’s 18th Ed. mild-to-moderate extrahepatic manifestations of acute hepatitis B ? elevations in serum ALT levels and low or moderate titers of IgG and IgM anti-HEV antibodies. Almost invariable establishment of chronic infection 793 Clinical features of autochthonous hepatitis E include all D. Glomerulonephritis with nephrotic syndrome more likely to remain chronically infected with HBV B. None of the above 802 Which of the following is an extrahepatic manifestation of 796 Which of the following statements is false ? hepatitis B ? Harrison’s 18th Ed. Inactive hepatitis B carriers can have normal liver histology manifestations of acute hepatitis B. peripheral neuropathy. Polyarteritis nodosa B. Necrolytic acral erythema B. Leucocytoclastic vasculitis outcome of HBV infection ? Harrison’s 18th Ed. N Engl J Med 2012. susceptible to antiviral therapy (peginterferon. Chronic hepatitis E is also Chronicity is uncommon and risk of hepatocellular carcinoma is very low. and mental confusion). Acute hepatitis-like illness In endemic. HLA-B*1501 A. HLA-B*5701 B. B. ataxia. Concomitant HDV and HBV infections 804 Mixed cryoglobulinemia (MC) is associated with which of the C. Blood transfusion is a potential but rare route of HEV transmission. Harrison’s 18th Ed. No neurologic complications A. Cryoprecipitable immune complexes B. HDV . Lichen planus (LP) A. Porphyria cutanea tarda 797 Which of the following is associated with a more severe C. C/C haplotype of the IL28B gene D. Immune complex .367:1237-44 A. and men outnumbered women by at least 3 to 1. the GBS. D. Ribavirin. 2544 C. Patients with defects in cellular immune competence are A. 2544 D. Robust host-immune response D. Disease rates highest among older adults 799 HBV infection acquired during adolescence or early adulthood B. Failure to recover is the exception infection is usually subclinical and mild. 2544 B. Chronicity is common including “acute-on-chronic” liver failure. Pre-existing liver disease B. Absence of acute-hepatitis illness fatality rate is ~ 5 %. Blood transfusion recepients C. HCV A. Hepatitis B virus is not directly cytopathic C. Hepatitis E is preventable by vaccination. or a combination of two). Acquisition of immunologic tolerance to HBV C. 2544 B. and chronic hepatitis. Hepatitis E is preventable by vaccination is associated with all except ? Harrison’s 18th Ed. Single nucleotide polymorphism T allele at IL28B locus C. 2545 A. Chronic hepatitis E is characterized 801 Which of the following plays a pathogenetic role in the by the persistence of HEV RNA in serum & stool. Chronic HEV infection has been identified almost exclusively among immunocompromised persons (organtransplant recipients. Autochthonous HEV C. HAV 798 HBV infection in neonatal period is associated with all except ? Harrison’s 18th Ed. Average case B. 2545 A. 455 Gastroenterology MCQ’s MCQ’s FOR FORPROFESSIONALS MEDICAL MEDICAL PROFESSIONALS BY PROF. In liver transplantation for end-stage chronic hepatitis B following ? D. Chronic HBV infection can occur in the absence of serum C. Types I 811 Councilman bodies are best related to ? Harrison’s 18th Ed. Harrison’s 18th Ed. 2545 nationality ? A. Acrocyanosis C. livedo reticularis. Interface hepatitis . 2545 B. 36°C A. D. Plasma cells B. Canadian Councilman bodies are named after American pathologist William Thomas Councilman (1854-1933) 805 Which type of cryoglobulinemia is associated with who discovered them. 37°C infection is due to ? Harrison’s 18th Ed. & chronic liver disease (Brouet classification). HEV D. 34°C C. Types II A. HAV A. 807 Cryoglobulins are immunoglobulins that precipitate at temperatures below ? 813 Ground-glass appearance of the cytoplasm in chronic HBV A. Large hepatocytes with a ground-glass appearance of the cytoplasm may be seen in chronic but not in acute HBV infection. Liver cell regeneration D. peripheral neuropathy. Large lymphocytes D. HCV B. Ground-glass appearance of cytoplasm glomerulonephritis. Acidophilic degeneration of hepatocytes C. 806 Classic triad of cryoglobulinemic syndrome includes all 812 Which of the following is seen in chronic but not in acute HBV except ? infection ? A. Waldenström macroglobulinemia). Small lymphocytes C. Fibrosis C. Others are D. Purpura Harrison’s 18th Ed. the most remarkable histologic feature is ? Harrison’s 18th Ed. Any of the above C. Liver cell damage leads to acidophilic degeneration of hepatocytes called Councilman or apoptotic bodies. Plasma cells & eosinophils are present occasionally. Gradual hepatitis B. Ireland Mixed cryoglobulinemia (MC) is unequivocally associated with HCV. Marked increase in activation of sinusoidal lining cells C. generalized vasculitis. viral infections. Ballooning of hepatocytes D. 2545 Harrison’s 18th Ed. Bile duct lesions is present. 2545 A. Panlobular mononuclear cells infiltration 814 In hepatitis C. 809 Panlobular mononuclear infiltration in viral hepatitis consists 815 Marked cholestasis is a feature of ? “primarily” of ? Harrison’s 18th Ed.456 MCQ’s FOR MEDICAL PROFESSIONALS BY PROF. HEV C. Hepatic cell regeneration D. HBsAg C. HBeAg D. American B. lymphoproliferative diseases ? A. In hepatitis C. British A. HBcAg Cryoglobulins are immunoglobulins that precipitate at temperatures below 37°C and re-dissolve with warming. 816 What was earlier called bridging hepatic necrosis is also 810 William Thomas Councilman (1854-1933) was of which called ? Harrison’s 18th Ed. Lymphoid aggregates Typical morphologic lesions of all types of viral hepatitis are panlobular infiltration with mononuclear cells. arthralgias & weakness. Types III B. 2545 B. Types II & III are mixed & polyclonal cryoglobulinemias D. 2545 B. HDV C. HBxAg Ground-glass appearance of the cytoplasm in chronic HBV infection is due to HBsAg and can be 808 Morphologic lesions of viral hepatitis are all except ? identified histochemically with orcein or aldehyde fuchsin. Arthralgias A. 35°C B. Panlobular mononuclear infiltration in viral hepatitis consists “primarily” of small lymphocytes. Growth arrest and are associated with autoimmune disorders. 2545 A. Weakness B. Apoptosis Type I cryoglobulinemia is monoclonal & associated with lymphoproliferative diseases (multiple myeloma. Kupffer cells hyperplasia & variable cholestasis. Cholestasis B. Hepatic cell necrosis A. the histologic lesion is remarkable for a relative paucity of inflammation. ischemic ulcers. Relative paucity of inflammation D. hepatic cell necrosis. Hepatocyte dropout Classic triad of cryoglobulinemic syndrome consists of purpura. Atrophy of Kupffer cells C. AJAY MATHUR Gastroenterology D. acrocyanosis and hemorrhagic bullae. 2545 B. Eosinophils Marked cholestasis is a common histologic feature of hepatitis E. D. Inflammatory debris C. Destruction hepatitis D. HBcAg C. HBeAg D. HEV antigen D. 823 What is not true for Hepatitis B virus infections ? N Engl J Med 2004. Degenerating liver cells D. bridging hepatic necrosis. HBsAg B. also termed subacute or confluent necrosis or interface hepatitis is observed occasionally. 2546 827 Hepatitis B virus (HBV) chronically infects how many people A. degenerating liver cells that span adjacent portal areas. 2547 B. inflammatory debris. Likelihood of perinatal transmission of HBV correlates with presence of HBeAg. 2546. whereas HBsAg carrier mothers with anti-HBe rarely (10 . 457 Gastroenterology MCQ’s MCQ’s FOR FORPROFESSIONALS MEDICAL MEDICAL PROFESSIONALS BY PROF. HCV antigen B. All are equally infectious C. D. Semen & saliva are infectious though less than serum. 90% of HBeAg- positive mothers but only 10–15% of anti-HBe-positive mothers transmit HBV infection to their 821 Which of the following body fluid from infected persons is offspring. 350 million Hepatitis B virus (HBV) chronically infects over 350 million people worldwide. Saliva A. Risk of chronicity in infected neonates is 90 % B. Hepatitis D C. HBsAg-positive + HBeAg-negative D. infection to their offspring. Percutaneous inoculation Harrison’s 18th Ed. Hepatitis C B. the bridge consists of ? Harrison’s 18th Ed. Hepatitis B A. most infectious ? Harrison’s 18th Ed. Down’s syndrome B. 1938 B. Condensed reticulum B. Perinatal transmission D. 822 Which of the following is the mode of HBV transmission ? Harrison’s 18th Ed. Gradual hepatitis C. portal to central veins. Piecemeal necrosis or limiting plate necrosis is now called "interface hepatitis". Incubation period for acute infection is 45 to 160 days A. All of the above Perinatal transmission occurs primarily in infants born to HBsAg carrier mothers or mothers with In bridging hepatic necrosis. 2547 A. Second trimester of pregnancy B. 2547 828 Prevalence of HBV sero-positivity is more in ? A. AJAY MATHUR Cardiology 457 C. Lepromatous leprosy . Semen worldwide ? Harrison’s 18th Ed. 100 million D. HBsAg-negative + HBeAg-negative C. Any of the above D.351:2832-8 817 What was earlier called piecemeal necrosis is now called ? A. HBxAg Hepatitis A is transmitted almost exclusively by the fecal-oral route. Can be transmitted through breast milk Harrison’s 17th Ed. 824 Perinatal transmission occurs in infants born to HBsAg carrier mothers during ? 818 In bridging hepatic necrosis. HCV & HEV antigens are localized to the HBsAg positive mothers who are HBeAg-positive almost invariably (>90%) transmit hepatitis B cytoplasm. 820 Which of the following hepatitis can be transmitted by fecal- 826 Likelihood of perinatal transmission of HBV correlates with oral route ? the presence of ? Harrison’s 18th Ed. 825 Which of the following mothers almost invariably transmit 819 Which of the following is localized to hepatocyte nucleus ? hepatitis B infection to their offspring ? Harrison’s 18th Ed. the bridge consists of condensed reticulum. or central vein to central vein. Table 304-2 Harrison’s 18th Ed. Most infections occur at the time of delivery and early D. 2547 Harrison’s 18th Ed.15%) infect their offspring. HDV antigen C. while HAV. 2547 A. 2547 B. HAV antigen A. Destruction hepatitis postpartum period & are not related to breast feeding. None of the above D. 2545 Harrison’s 18th Ed. HBsAg-negative + HBeAg-positive HDV antigen is localized to hepatocyte nucleus. 50 million C. All of the above In acute hepatitis. No known animal reservoirs C. There is collapse of the reticulin framework. 250 million HBsAg is identified in almost every body fluid from infected persons. Coupled hepatitis ~10% of HBV infections are acquired in utero. Sexual contact A. Serum B. Coupled hepatitis C. 2545 A. First trimester of pregnancy A. HBsAg-positive + HBeAg-positive B. and acute hepatitis B during third trimester of pregnancy or during the early postpartum period. Interface hepatitis D. Third trimester of pregnancy C. HDV . mean 60-90. B. Table 304-2 D. 2546.3 million for transfusion-associated HCV infection. Hodgkin’s disease C. 835 Which of the following is a presentation of acute viral hepatitis ? Harrison’s 18th Ed.insidious. Most frequent indication for liver transplantation Acute hepatitis B is self-limited in 95 . Lymphopenia 1 . 2549 A.000 B.insidious or acute. Sickle cell anemia B. Bilirubin levels A.69 % B. Alterations in olfaction and taste D. Hepatitis A Apart from the above conditions. Steatorrhea B. Spider angiomas C. Hepatitis E The diagnosis of anicteric hepatitis is based on clinical features & on aminotransferase elevations. HCV . Hepatitis C Bilirubin levels >20 mg/dL persisting late into the course of viral hepatitis is associated with severe D. mean 30.30-180. AJAY MATHUR Gastroenterology C. leukemia. Splenomegaly A. mean 40. All of the above C. while hepatitis C is self-limited in only 15%.89 % A. Table 304-2 B.30-180. Genotype 4 predominates in Egypt. HBV . Spontaneous HBeAg to anti-HBe seroconversion 831 Hepatitis virus with longest incubation period is ? C. genotype 1 is the most common 837 Acute hepatitis-like clinical events in chronic hepatitis B may D. HBV . All of the above D. 2549 Harrison’s 18th Ed. dL) occur in ? Harrison’s 18th Ed. Hepatitis A C. HDV . 2549 833 Viral hepatitis with an insidious onset only is ? A. lepromatous leprosy. risk of acquiring HBV infection from Harrison’s 18th Ed. Hepatitis E Harrison’s 18th Ed. acute 840 Which of the following occur transiently in acute viral 834 Which of the following about presentation of acute viral hepatitis ? hepatitis is false ? Harrison’s 18th Ed. acute clinical exacerbations of chronic hepatitis B may be due to B. C. Worldwide. HCV . HEV . 2549 a blood transfusion is 1 in 230.2 weeks C. HDV superinfection of HCV infection between an infected mother and her infant. mean 60-90. Severe disease Harrison’s 18th Ed. S. genotype 5 is localized to South Africa. 1 in 320.000 D.99 % B. very high serum bilirubin level (20 . Breast-feeding does not increase the risk A. Constitutional symptoms may precede onset of jaundice by B. 2549 829 Risk of acquiring HBV infection from a blood transfusion is ? Harrison’s 18th Ed. CKD patients on dialysis and IDUs. and genotype 6 to Hong Kong. Cervical adenopathy B. 2546. 69 . S.000 836 Acute hepatitis B is self-limited in what proportion of cases ? Because of highly sensitive virologic screening of donor blood.000 C. like glucose-6-phosphate dehydrogenase deficiency and sickle cell anemia. 95 . All of the above D.15-45. Glucose-6-phosphate dehydrogenase deficiency A. Aminotransferase levels Harrison’s 18th Ed. HEV . Alkaline phosphatase levels B. All of the above . 79 . Dark urine & clay-colored stools occur 1 . 1 in 140. D. Breast-feeding increases risk of HCV vertical infection be due to ? Harrison’s 18th Ed. Hepatitis C 838 The diagnosis of anicteric hepatitis is based on ? D. A. Spontaneous reactivation Harrison’s 18th Ed. 2547 C. Hepatitis A C.79 % 830 Which of the following about HCV infection is false ? Harrison’s 18th Ed.000 while it is 1 in 2.99% of infections.14-60. Neutropenia A. polyarteritis nodosa. All of the above 839 In acute hepatitis. Hepatitis E disease. Hepatitis C D. 2549 832 Hepatitis virus with an incubation period of ~2 weeks is ? A. 1 in 50. Accounts for 40% of chronic liver disease D. None of the above Prevalence of 5-20% is found in Down’s syndrome.458 MCQ’s FOR MEDICAL PROFESSIONALS BY PROF.5 days before onset of clinical jaundice D. mean 50. Hodgkin’s disease.insidious or acute. GGT levels C. C. All of the above A. S. Hepatitis B emergence of a precore mutant.30 mg/ Incubation period in days : HAV . S. 2546. 2549 Worldwide. Table 304-2 B. hemolysis. Patients with underlying hemolytic anemia.15-160. genotype 1 is the most common. Hepatitis B D. 1 in 230. 2547 A. also have high serum bilirubin levels (>30 mg/dL) due to superimposed Onset : HAV . 50 .acute. 845 A false-positive test for IgM anti-HBc may be found in patients leucocytoclastic vasculitis. All of the above 843 If levels of HBsAg are too low to be detected during acute HBV In chronic hepatitis B. Porphyria cutanea tarda D. In hepatitis C and D. which of the following establishes its diagnosis ? and hepatocellular carcinoma. IgM anti-HBc-positive. When this Harrison’s 18th Ed. 459 Gastroenterology MCQ’s MCQ’s FOR FORPROFESSIONALS MEDICAL MEDICAL PROFESSIONALS BY PROF. high levels of HBV DNA increase the risk of cirrhosis. which consists of HBsAg alone. Anti-HBs A diffuse but mild elevation of the gamma globulin fraction is common during acute viral hepatitis. HBeAg is invariably present during early acute hepatitis B. All of the above acute phase of viral hepatitis ? After immunization with hepatitis B vaccine. Lichen planus (LP) IgM anti-HBc may be useful to distinguish between acute or recent infection (IgM anti-HBc-positive) D. Hepatitis E A false-positive test for IgM anti-HBc may be encountered in patients with high-titer rheumatoid D. 2550 happens. Frequently associated conditions include porphyria cutanea tarda. All of the above factor. and hepatitis E. IgG anti-HBc-negative A. hepatic decompensation. 844 Which of the following is true in chronic HBV infection ? Harrison’s 18th Ed. IgG anti-HBc A. Anti-HBe Serum IgM level is elevated more characteristically during acute hepatitis A. None of the above acute hepatitis A ? Anti-HBs is rarely detectable in the presence of HBsAg in patients with acute hepatitis B. and heterophil antibody can also be found. Also. IgM anti-HBc-negative. HBeAg testing D. All of the above and chronic HBV infection (IgM anti-HBc-negative. Chronicity and transient steatorrhea. 2551 D. C. Serum IgE A. Nuclear antibody risk of ? C. its of no recognized clinical significance. Complications of hepatitis A include relapsing hepatitis appearing weeks to months after apparent recovery from acute hepatitis. 2552 B. 2552 B. Heterophil antibody Harrison’s 18th Ed. 2550 serologic marker to appear. Well-accepted extrahepatic manifestations of HCV include pruritus. C. 2552 Harrison’s 18th Ed. IgM anti-HBc-positive. presence of IgM anti-HBc establishes its diagnosis. 846 Presence of anti-HBs in the presence of HBsAg in patients with 852 Out of the following. Serum IgM to appear ? C. Fulminant hepatitis . Sickle cell anemia B. It is rare in hepatitis A. microscopic hematuria and minimal proteinuria have been noted. which is the only serologic marker B. IgG anti-HBc-positive Harrison’s 18th Ed. Hepatitis B & D D. sicca syndrome & polyarteritis nodosa. cholestatic hepatitis and rarely fulminant hepatitis. anti-HBs is the only Harrison’s 18th Ed. HBeAg C. AJAY MATHUR Cardiology 459 Neutropenia and lymphopenia are transient and are followed by a relative lymphocytosis. A. high levels of HBV DNA increase the B. 2550 851 Fulminant hepatitis is rare in ? A. lichen planus. 2550 D. IgM anti-HBc 849 Relapsing hepatitis is a feature of ? Harrison’s 18th Ed. Acute hepatitis A C. Hepatic decompensation may be present. fulminant hepatitis is most common in ? acute hepatitis B indicates which of the following ? Harrison’s 18th Ed. with ? Harrison’s 18th Ed. B. High-titer rheumatoid factor A. Hepatitis A C. 2552 B. IgG anti-HBc-positive). IgG anti-HBc-negative C. Fulminant hepatitis is primarily seen in hepatitis B and D. Glucose-6-phosphate dehydrogenase deficiency Harrison’s 18th Ed. IgM & IgG anti-HBc B. Acute hepatitis B D. nuclear antibody. C. A. All of the above A. 2550 A. Acute hepatitis C If levels of HBsAg are too low to be detected during acute HBV infection. antibodies to LKM may be found. 2550 850 Extrahepatic manifestations of HCV include ? A. Hepatitis D superinfection is indicated primarily during follow-up of chronic infection. IgG anti-HBc-positive B. antibodies to smooth muscle and other cell constituents B. and low titers of rheumatoid factor. infection. Imminent HBsAg clearance 841 Which of the following is characteristically elevated during D. Hepatocellular carcinoma D. mild B. Serum IgA Harrison’s 18th Ed. Harrison’s 18th Ed. Hepatitis A A. Cirrhosis During the acute phase of viral hepatitis. All of the above C. Rheumatoid factor 848 In chronic hepatitis B. Anti-HBc 842 Which of the following antibodies may be present during the D. Serum IgG 847 After hepatitis B vaccination. Mixed cryoglobulinemia C. IgM anti-HBc-negative. mixed cryoglobulinemia & necrolytic acral erythema. 2550 A. recommended site of Hepatitis B vaccine is ? Harrison’s 18th Ed. while genetically engineered Hepatitis B infection is ? vaccine is derived from recombinant yeast and consists of HBsAg particles that are nonglycosylated but are otherwise indistinguishable from natural HBsAg. 2553 863 In adults. Hepatitis A vaccines provide adequate protection beginning 4 weeks after a primary inoculation. Hepatitis B B.460 MCQ’s FOR MEDICAL PROFESSIONALS BY PROF. D. HBeAg persists for >3 months vaccine ? B. 2 weeks C. All of the above Down’s syndrome. 3 weeks D. Hepatitis C C. 65 .60 % Harrison’s 18th Ed. HIV infection C. Thigh muscle C. Hepatitis C B. 2552 859 Hepatitis A vaccine is administered by which route ? A. Subcuteneously After acute HCV infection. One month A. 42-nm spherical forms of HBsAg Likelihood of remaining chronically infected after acute HBV infection is high among neonates. 2555 B. Triceps muscle D. Nonglycosylated D. bilirubin and globulin levels fail to normalise within 6-12 months. Chronically hemodialyzed patients B. Hepatitis E D. 2555 A. 22-nm spherical forms of HBsAg C. Deltoid muscle D. Hepatitis E C. D. Hepatitis E C. 853 Fulminant hepatitis is hardly ever seen in ? Harrison’s 18th Ed. 25 . Hepatitis B A. 854 Likelihood of remaining chronically infected after acute HBV infection is high in all except ? 860 The first vaccine for hepatitis B active immunization was Harrison’s 18th Ed. Subcutenuous D. buttocks & limbs & lymphadenopathy.40 % 862 Hepatitis B vaccine is administered ? B. the likelihood of remaining chronically infected approaches 85-90%. D. 2555 A. 2552 prepared from ? A. Hydrolyzed Progression of acute hepatitis to chronic hepatitis is likely if clinical symptoms do not resolve. Gluteal muscle Gianotti-Crosti syndrome or papular acrodermatitis of childhood refers to hepatitis B that presents with anicteric hepatitis. Old Harrison’s 18th Ed. chronically hemodialyzed patients & immunosuppressed patients. Hepatitis A B. AST/ C. immunogenic. Hepatitis B A. 27-nm spherical forms of HBsAg D. 2555 weeks after a primary inoculation ? Harrison’s 18th Ed. 4 weeks Hepatitis B accounts for >50% of fulminant cases of viral hepatitis. noninfectious 22-nm spherical forms of HBsAg derived from plasma of healthy HBsAg carriers. HBeAg persists for >3 months and HBsAg persists for >6 months. 2553 A. Harrison’s 18th Ed.75 % A. 864 After the first dose of Hepatitis B vaccine. 855 Progression of acute to chronic hepatitis is likely if ? Harrison’s 18th Ed. AST/ALT do not normalise within 6-12 months A. 2552 861 Which of the following is the difference between plasma- derived vaccine and genetically engineered Hepatitis B A. Formalin-inactivated vaccines made from strains of HAV attenuated in tissue culture are safe. noninfectious 22-nm spherical forms 856 Likelihood of remaining chronically infected after acute HCV of HBsAg derived from plasma of healthy HBsAg carriers. Three months . including those with HIV infection. 40 . 85 . Heat attenuated Plasma-derived Hepatitis B vaccine is prepared from purified. Intradermally D. HBsAg persists for >6 months Harrison’s 18th Ed. Hepatitis C B. Down’s syndrome A. Intravenous Hepatitis A vaccine is administered intramuscularly. the third dose is 858 Hepatitis A vaccines provide adequate protection how many given after ? Harrison’s 18th Ed. First vaccine for active immunization (1982) was prepared from purified. All of the above B. 2555 C. Intramuscular Fulminant hepatitis is hardly ever seen in hepatitis C. and effectively prevent hepatitis A.90 % B. C. Intravenously 857 Gianotti-Crosti syndrome is best related to ? Harrison’s 18th Ed. 2555 C. Intradermal C. Intramuscularly D. Hepatitis A Harrison’s 18th Ed. 1 week B. Oxidized ALT. 2555 B. AJAY MATHUR Gastroenterology B. nonpruritic papular rash of face. 08 mL / kg A.351:2832-8 D. Neoplasm B vaccine to begin within the first week. Necrolytic acral erythema C. 15 mIU/mL A. 100000 IU per milliliter D. Sexual contact with person who is HBsAg + prior or concomitant reason for liver function abnormality. no other C.351:2832-8 C. Wy's law B.06 mL/ kg. Household members & sexual partners of HBV carriers Harrison’s 18th Ed. 2556 D. None of the above B.04 mL / kg C. Contact with HBsAg + blood / bodily fluid Hy’s Law criteria: >3x ULN AST or ALT + >2x ULN TBL. 10 mIU/mL 875 Wickman's striae are best related to ? B. Infants born to HBsAg + mothers D.351:2832-8 For preexposure prophylaxis against hepatitis B. 5 µg B. 869 HBV inactive carriers have serum HBV DNA level below ? N Engl J Med 2008. 0. Clients & staff of institutions for developmentally disabled A.02 mL / kg D. LP is an inflammatory disease of skin & mucous membranes. Persons at occupational risk Harrison’s 18th Ed. 2556 C. Booster immunizations are not recommended routinely Hepatitis B and HIV occur together ? Harrison’s 16th Ed. Porphyria cutanea tarda D. 0. 20 mIU/mL B. N Engl J Med 2004. None of the above A. 0. lung. Zy's law 870 Hepatitis B immune globulin (HBIG) is used in ? B. Leucocytoclastic vasculitis Booster immunizations are not recommended routinely in immunocompetent persons. 1000 IU per milliliter B.06 mL / kg 873 Leser-Trelat sign mostly points towards which pathology ? Harrison’s 17th Ed. 1111 C. A. dose of HBIG is ? B.359:1486-500 A. has a case fatality rate of 10% to 50% is ? are considered to be inactive carriers. Hy's law A. B. without a hepatic Persons with a serum HBV DNA level below 1000 IU per milliliter and a normal ALT level consistently transplant. papules with an overlying reticulate pattern of white lines called Wickman's striae. three IM injections of hepatitis B vaccine in deltoid. Infection For postexposure HBV prophylaxis. Nutritional deficiency D. 0. and 6 months. Patients on peritoneal dialysis 865 Engerix-B for adults contains what amount of HBsAg in 1 ml. Rezulin Rule is also a prognostic rule of DILI. 1. Adoptees from countries where HBV infection is endemic A. no initial cholestasis (normal AP). polygonal. C. 15 µg all except ? N Engl J Med 2004. dose of HBIG is a single intramuscular dose of HBIG. Recipients of clotting-factor concentrates 866 For postexposure HBV prophylaxis. 2556. Booster recommended if anti-HBs levels are <10 mIU/mL A. administered as soon after exposure as possible and followed by a complete course of hepatitis B. D. Interferon alpha D. Booster D. Six months 871 Groups for whom Hepatitis B vaccine is recommended include D. Ky's law N Engl J Med 2004. Tenofovir antibody level of less than ? Harrison’s 18th Ed. 10000 IU per milliliter 305 . 20 µg A. urinary tract cancers & lymphoid malignancies. A. 50 mIU/mL C. 0. purple. 10 µg 872 Groups for whom Hepatitis B vaccine is recommended include C.Toxic and drug-induced hepatitis C. Twelve months all except ? N Engl J Med 2004. A. Lichen planus (LP) doses are recommended when anti-HBs levels fall to <10 mIU/mL. Travelers spending > 6 weeks in HBV endemic areas B. AJAY MATHUR Cardiology 461 C. Entecavir 868 True nonresponse after proper HBV vaccination means C. Pregnancy is not a contraindication to vaccination 874 Which drug should be included in HAART regimen when B. All of the above . Environmental toxin 867 Which of the following statements about Hepatitis B vaccination is false ? Leser-Trélat sign refers to multiple pruritic seborrheic keratoses of sudden onset. 461 Gastroenterology MCQ’s MCQ’s FOR FORPROFESSIONALS MEDICAL MEDICAL PROFESSIONALS BY PROF. 325 D. All infants not gluteal muscle are recommended at 0. ? C. Rezulin is the trade name of banned drug troglitazone. characterized by pruritic. 2556 with gastrointestinal adenocarcinomas. breast. 1000000 IU per milliliter 876 Name of the prognostic law stating that a pure drug-induced liver injury (DILI) leading to jaundice. Mostly associated Harrison’s 18th Ed.351:2832-8 D. Not dose-dependent 888 In CYP2E1. a capital letter indicating subfamily and another numeral for the individual gene. Akio Suzuki 889 Which of the following about cytochrome P450 is false ? A. Number of isoenzymes in liver D. None of the above B. Martin Klingenberg & David Garfinkel D. Anti LKM 1 878 Phase II reaction in drug metabolism include which of the B. All of the above A. sulfation. B. 2558 compounds like steroids. Oxidation B. Wilhelm Kuhnz & Hille Gieschen D. Lipids B. Morphologic abnormalities reproducible for each toxin B. Monooxygenase reaction biosynthesis and degradation of ? N Engl J Med 2005. followed by a numeral indicating gene family. while phase 2 conjugation enzymes are cytosolic. A. Number of chemical reactions In direct toxic hepatitis. directed against a cytochrome P450 enzyme. Dose-dependent 887 “450” in Cytochrome P450 isoenzymes is related to ? C. Sulfation B. Glucuronidation C. Anti LKM 2 following chemical process ? C. Vitamins C. Harrison’s 18th Ed. Pigment C. Glucuronidation Drug hepatotoxicity may be associated with the appearance of autoantibodies. Reduction 884 Antibody to liver-kidney microsomes associated with drug induced hepatitis is ? C. Hemoprotein 883 Cytochrome P-450 enzymes (CYPs) are important in the B.352:2211-21. Individual gene 882 Cytochrome P450 (CYP) was first discovered in 1954 by ? A. latent period between exposure and liver injury is usually short (often several hours). lipids. Extrahepatic manifestations of hypersensitivity A. Most common reaction catalysed by cytochrome P450 is a monooxygenase reaction. Latent period usually long A. Lipids Cytochrome P450 belongs to a superfamily of hemoproteins. and vitamins. All of the above following chemical process ? Cytochrome P-450 enzymes (CYPs) are important in biosynthesis & degradation of endogenous N Engl J Med 2005. Gene family D. although clinical manifestations may be delayed for 24 . All of the above biosynthesis & degradation of endogenous compounds like ? N Engl J Med 2005.462 MCQ’s FOR MEDICAL PROFESSIONALS BY PROF. Acetylation 885 Cytochrome P-450 enzymes (CYPs) are important in the D. AJAY MATHUR Gastroenterology 877 Phase I reaction in drug metabolism include which of the D. so named for the characteristic spectrophotometric absorption Harrison’s 18th Ed. Harrison’s 18th Ed.352:2211-21. CYP is important for phase I metabolism and are located primarily in endoplasmic reticulum. C.352:2211-21 879 Phase II reactions in drug metabolism include all except ? N Engl J Med 2005. None of the above B.352:2211-21 C. Am Fam Phys 1998. All of the above D. Harrison’s 18th Ed. A. letter ‘2’ indicates ? C. Spectrophotometric absorption peak 881 Which of the following is not a feature of idiosyncratic drug Name cytochrome P450 is derived from the fact that these are colored ('chrome') cellular ('cyto') hepatotoxicity ? proteins. C. Steroids D. 2558 peak formed by absorbance of light at wavelengths near 450 nm when the heme iron is reduced and complexed to carbon monoxide. Protein A. Unpredictability B. Pattern 880 Which of the following is not a feature of direct toxic hepatitis ? Harrison’s 18th Ed. including a class of B.48 hours. 2558 A. Vitamins substrate (RH) while the other oxygen atom is reduced to water: RH + O2 + 2H+ + 2e. Gene subfamily C. Hydrolysis Harrison’s 18th Ed.e. 2558 D.352:2211-21.57:107-16 reduction & hydrolysis (phase I reactions) or glucuronidation. 2558 D. Steroids A. Bernhardt "CYP" stands for cytochrome P450. All of the above A. Number of electron needed for its activity D. None of the above B. Pigment at 450 nm A. insertion of one atom of oxygen into an organic C. acetylation & methylation A. i. Particle (phase II reactions).= ROH + H2O . Hydrolysis 886 The P in P450 stands for ? Drugs may be metabolized by sequential or competitive chemical processes involving oxidation. 2558 D. Predictable regularity The P in P450 stands for "pigment". C. Sulfation antibodies to liver-kidney microsomes. Acetylation D. Anti LKM 3 N Engl J Med 2005. with a "pigment at 450 nm". B. anti-LKM2. CYP 2C19 yielding therapeutic and adverse effects include all except ? C. CYP2C19 903 Major hepatic isoenzyme in metoprolol metabolism is ? CYP1A2 is not expressed in neonates. Dextromethorphan B. AJAY MATHUR Cardiology 463 890 Which of the following enzyme is responsible for metabolism A. Troleandomycin 892 The catalytic activity of CYP2D6 in humans is best assessed by using which of the following drug ? D. Golgi bodies A. Carbamazepine D. the family of cytochrome P450 (CYP) isoforms is present in ? D. Ritonavir C. CYP 3A CYP3A is present in the enterocytes. Cell membrane N Engl J Med 2005. CYP 3A D. CYP 2C9 B. B.352:2211-21 D. A. Saquinavir 891 In liver. CYP 2C19 B. CYP2C19 CYP11B2 is found mainly if not exclusively in the glomerulosa zone of the adrenal gland. CYP3A5 C. CYP1A2 B. CYP2D6 D. Lidocaine A. CYP2D6 B. Lamotrigine 893 Which of the following cytochrome P-450 is present in enterocytes ? 900 Major hepatic isoenzyme involved in warfarin metabolism is ? N Engl J Med 2005. Fluoxetine N Engl J Med 2005. CYP 3A D. Perphenazine A. CYP3A D. CYP 2C19 C. 36 Table 5-1 such as caffeine. CYP2D6 897 Which of the following anti-HIV agent is not a CYP3A inhibitor ? N Engl J Med 2005. Chapter 70 899 Which of the following anticonvulsant agent is not a CYP3A A. Nucleus B. 36 Table 5-1 zone of adrenal gland ? A. CYP3A5 C.352:2211-21 Harrison’s 18th Ed. CYP 2D6 C. Chapter 70 D. making them particularly susceptible to toxicity from drugs Harrison’s 18th Ed. Phenytoin D. Phenobarbital CYP2D6 is second to CYP3A4 in the number of commonly used drugs that it metabolizes. Endoplasmic reticulum Inhibitor ? B. Indinavir CYP3A4 is the most abundant hepatic and intestinal CYP and is also the enzyme responsible for metabolism of the greatest number of drugs in therapeutic use. CYP3A4 D. Clarithromycin D. CYP1A2 A. CYP 2C9 A. Chapter 70 898 Which of the following macrolide antibiotics is not a CYP3A A. 901 Major hepatic isoenzyme involved in phenytoin metabolism is ? 894 Which of the following CYP is found mainly in the glomerulosa Harrison’s 18th Ed. Nevirapine Harrison’s 15th Ed. 463 Gastroenterology MCQ’s MCQ’s FOR FORPROFESSIONALS MEDICAL MEDICAL PROFESSIONALS BY PROF. CYP 2C19 B. CYP 2D6 Harrison’s 15th Ed. Digoxin of most of the drugs used ? B. CYP 3A . Theophylline Harrison’s 18th Ed. Debrisoquin Inducer ? B. CYP2C9 C. 41 C. Azithromycin Harrison’s 15th Ed. 902 Major hepatic isoenzyme in omeprazole metabolism is ? Harrison’s 18th Ed. CYP 2C9 A. CYP11B2 B. C. Erythromycin C. 36 Table 5-1 A. CYP3A5 C.352:2211-21 C. CYP2C19 A. CYP 2D6 D. Amiodarone B.352:2211-21 C. 36 Table 5-1 895 Which of the following CYP is not expressed in neonates ? A. CYP2D6 D. CYP 2C9 896 Drugs having a narrow range between the plasma levels B. CYP 2D6 C. CYP 2D6 B. Rifampin B. Methadone Harrison’s 18th Ed. 2561 911 Major hepatic isoenzyme in quinidine metabolism is ? A. All of the above Harrison’s 18th Ed. Phenobarbital A. Quinidine C. AJAY MATHUR Gastroenterology 904 Major hepatic isoenzyme in tricyclic antidepressants 912 CYP enzyme activity inducers include ? metabolism is ? Harrison’s 18th Ed. CYP 2C19 B. CYP 2C19 Harrison’s 18th Ed. Metronidazole A. CYP 2C19 C. All of the above D. Methylprednisolone Harrison’s 18th Ed. 45 Harrison’s 17th Ed. CYP 2D6 D. Oral contraceptive steroids 910 Major hepatic isoenzyme in lidocaine metabolism is ? B. CYP 2C9 A. Smoking B. CYP 2D6 D. 36 Table 5-1 B. CYP 3A 914 CYP enzyme activity inducers lower plasma levels of which of 906 Major hepatic isoenzyme involved in codeine metabolism is ? the following drugs ? Harrison’s 18th Ed. CYP 2C9 B. Chronic alcohol ingestion C. 36 Table 5-1 D. CYP 2D6 918 Which of the following drugs is associated with moderate to D. Itraconazole 908 Major hepatic isoenzyme in statin metabolism is ? D. 29 Table 5-1 A. CYP 2C19 C. CYP 3A . 36 Table 5-1 C. CYP 3A A. All of the above D. CYP 2C19 917 CYP enzyme activity inducers lower plasma levels of which of C.464 MCQ’s FOR MEDICAL PROFESSIONALS BY PROF. 36 Table 5-1 A. 36 Table 5-1 the following drugs ? A. CYP 2D6 A. Isoniazid C. 45 B. 45 D. CYP 2C9 D. All of the above D. Cyclosporine D. Ketoconazole D. CYP 2C9 Harrison’s 18th Ed. Oxyphenisatin Harrison’s 18th Ed. CYP 2C9 B. CYP 2C9 B. All of the above B. 45 reuptake inhibitors metabolism is ? Harrison’s 18th Ed. 45 C. CYP 2C9 916 CYP enzyme activity inducers lower plasma levels of which of the following drugs ? B. CYP 2D6 the following drugs ? Harrison’s 18th Ed. CYP 2C19 A. 36 Table 5-1 Harrison’s 18th Ed. CYP 3A D. 45 A. CYP 3A C. CYP 2C19 C. Methyldopa A. CYP 3A 913 CYP enzyme activity inducers include ? 905 Major hepatic isoenzyme involved in selective serotonin Harrison’s 18th Ed. All of the above 907 Major hepatic isoenzyme in cyclosporine metabolism is ? 915 CYP enzyme activity inducers lower plasma levels of which of Harrison’s 18th Ed. Mexiletine D. Carbamazepine C. CYP 2C9 B. CYP 2D6 C. Verapamil C. All of the above A. Dexamethasone 909 Major hepatic isoenzyme in phenytoin metabolism is ? C. 36 Table 5-1 A. CYP 3A B. Warfarin B. Phenytoin A. CYP 2D6 D. CYP 3A severe chronic hepatitis ? Harrison’s 18th Ed. CYP 2C19 C. 6 days B. However. Anabolic steroids A. B. >300 µg/mL A. Arsenic intoxication C. Chlorpromazine Blood levels of acetaminophen of >300 µg/mL. All of the above C. 465 Gastroenterology MCQ’s MCQ’s FOR FORPROFESSIONALS MEDICAL MEDICAL PROFESSIONALS BY PROF. Chlorpromazine B. 4 hours after ingestion are predictive severe liver damage. > 60 minutes D. Blood cysts of liver D. >100 µg/mL 920 Syndrome resembling primary biliary cirrhosis can occur B. Cholestyramine A. Methotrexate C. D. Azathioprine B. Acetaminophen Harrison’s 18th Ed. Alcohol induces cytochrome P450 CYP2E1. Quinidine A. Oxyphenisatin Fatal fulminant liver disease is usually associated with ingestion of 25 grams of acetaminophen. 2561 922 Which of the following are associated with angiosarcoma of the liver ? A. 2561 B. Arsenic intoxication D. 2561 931 Which of the following have a role in the management of A. Halothane Harrison’s 18th Ed. 2561 921 Portal hypertension in the absence of cirrhosis may result from A. 25 grams A. of the following ? Harrison’s 18th Ed. All of the above 928 Which of the following is “hepatoprotective” ? Harrison’s 18th Ed. Methyl testosterone C. 2561 D. Tolbutamide 927 Maximal hepatic injury and hepatic failure occurs after how D. 2561 time after ingestion ? A. Glutathione B. 2561 Halothane and methotrexate have been implicated in the development of cirrhosis.4 days A. 20 grams Harrison’s 18th Ed.2 days the use of ? Harrison’s 18th Ed. All of the above 929 N-acetyl-benzoquinone-imine (NAPQI) is best related to which Arsenic intoxication. 2563 . B. Ectopic liver Most of acetaminophen is metabolized by phase II reaction to sulfate & glucuronide metabolites. Halothane liver damage ? Harrison’s 18th Ed. 924 Peliosis hepatis is seen in patients treated with ? 930 Oral activated charcoal or cholestyramine is useless how much Harrison’s 18th Ed.6 days after acetaminophen ingestion. Vitamin A C. > 30 minutes C. 7 . Isoniazid 926 What level of acetaminophen in blood is predictive of severe D. 2563 B. AJAY MATHUR Cardiology 465 919 Which of the following drugs is implicated in the development B. All of the above many days of acetaminophen ingestion ? Harrison’s 18th Ed. ingestion of what amount of acetaminophen ? Harrison’s 18th Ed. 2 . 10 grams of cirrhosis ? C. >150 µg/mL following treatment with ? C. harmless mercapturic acid. or administration of thorium dioxide have been associated with angiosarcoma of the liver. A. D. Exposure to vinyl chloride Maximal hepatic injury & hepatic failure appear 4 . 5 grams acetaminophen hepatotoxicity ? Harrison’s 18th Ed. D. 2561 923 Peliosis hepatis refers to ? A. D. 2561 D. Thorium dioxide Alcohol suppresses hepatic glutathione production. Trauma of liver C. “hepatoprotective” glutathione binds NAPQI to form Peliosis hepatis refers to blood cysts of the liver. industrial exposure to vinyl chloride. >200 µg/mL Harrison’s 18th Ed. 1 .9 days C. 4 . 2561 B. Methyldopa C. > 90 minutes Peliosis hepatis has been observed in some patients treated with anabolic steroids. Carbamazine C. 2561 B. > 120 minutes Oral activated charcoal or cholestyramine to prevent absorption of residual drug is useless if given 925 Fatal fulminant liver disease is usually associated with >30 minutes after acetaminophen ingestion. Activated charcoal Harrison’s 18th Ed. Unilobular liver Phase I reaction by CYP2E1 metabolizes a small amount of acetaminophen to N-acetyl-benzoquinone- imine (NAPQI) which is hepatotoxic. Vinyl chloride D. Cysteine B. 2564 934 Isoniazid hepatotoxicity is enhanced by ? A. administration of sulfhydryl compounds A defect in epoxide hydrolase activity could cause hepatotoxicity due to Phenytoin. 2-pentenoic acid Harrison’s 18th Ed. Erythromycin Harrison’s 18th Ed. Toxic reaction C. 2564 Harrison’s 18th Ed. Halothane is not a direct hepatotoxin C. Valproate A. Harrison’s 18th Ed. Cause severe centrilobular hepatic necrosis D. Harrison’s 18th Ed. -Alkyl-Substituted Anabolic Steroids Harrison’s 18th Ed. hepatotoxicity is attributable to which component of the drug ? B. 2564 . but its metabolite 4-pentenoic acid may be responsible C. 938 Stevens-Johnson syndrome may be a presentation of toxicity due to ? 932 Which of the following about halothane hepatotoxicity is false ? Harrison’s 18th Ed. AJAY MATHUR Gastroenterology A. 17. 2565 C. 2565 D. Valproate B. cysteine. Sulfamethoxazole D. Pyrazinamide D. Phenytoin B. Halothane 942 Drugs producing cholestatic reaction and portal inflammation is ? Harrison’s 18th Ed. Rifampin D. Trimethoprim Sodium valproate is not directly hepatotoxic. None of the above which of the following ? Harrison’s 18th Ed. Acetaminophen If given within 8 hours of ingestion of acetaminophen. 1-pentenoic acid 943 In Trimethoprim-Sulfamethoxazole toxicity. Halothane C. 2564 C. lymphadenopathy. Erythromycin Valproate hepatotoxicity may be ameliorated by IV administration of carnitine. Sulfamethoxazole + Trimethoprim for hepatic injury. N-acetylcysteine C. rash (Stevens-Johnson syndrome or exfoliative dermatitis). or N-acetylcysteine) reduces the severity of hepatic necrosis. Resolves with discontinuation of drug 940 Amiodarone metabolite desethylamiodarone accumulate in D. Oral contraceptive 936 Which metabolite of sodium valproate may be responsible for C. Idiosyncratic reaction B. liver transplantation may be the only option. (cysteamine. 2564 933 Which of the following is false about methyldopa A. 3-pentenoic acid A. These agents act by providing a reservoir of sulfhydryl groups to bind toxic metabolites or by stimulating synthesis and repletion of hepatic glutathione. Hepatocyte mitochondria A. Levoethylamiodarone C. Valproate D. Cross-reactions between halothane & methoxyflurane Aparet from drug induced hepatitis. 2565 935 IV administration of carnitine may be ameliorate hepatotoxicity A. 2563 B. Phenytoin C. Chlorpromazine hepatic injury ? D. Azathioprine A. Acetaminophen A. B. Desmethylamiodarone hepatotoxicity ? Harrison’s 18th Ed. D. If these fail. Cysteamine A. 2564 A. Isoniazid C. The pathologic changes produced by halothane hepatotoxicity are indistinguishable from massive hepatic necrosis resulting from viral hepatitis. 2564 B. fever.466 MCQ’s FOR MEDICAL PROFESSIONALS BY PROF. Hepatocyte lysosomes Harrison’s 18th Ed. rifampin & pyrazinamide. Acetaminophen due to ? B. Halothane D. Bile duct epithelium B. 939 Which of the following is a major metabolite of Amiodarone ? Harrison’s 18th Ed. 4-pentenoic acid B. Desethylamiodarone A. Alcohol C. None of the above 937 A defect in epoxide hydrolase activity could cause The hepatotoxicity with the use of Trimethoprim-Sulfamethoxazole is attributable to the hepatotoxicity due to which drug ? sulfamethoxazole component of the drug. leukocytosis & eosinophilia may manifest in hepatotoxicity due to Phenytoin. 2563 A. Severe centrilobular hepatic necrosis is typical of acetaminophen toxicity. Acetaminophen D. All of the above C. All of the above 941 Toxicity with which of the following produces cholestatic idiosyncratic reaction ? Isoniazid hepatotoxicity is enhanced by alcohol. Idiosyncratic reaction D. Levomethylamiodarone B. Carbamazine B. All of the above D. Congestive heart failure B. 14 Risk of trimethoprim-sulfamethoxazole hepatotoxicity is increased in persons with HIV infection. Deficiency of minerals A. ~9% 949 Classification of chronic hepatitis is based on ? Harrison’s 18th Ed. or stage Harrison’s 18th Ed. ~7% D. Harrison’s 18th Ed. 2567 A. individual scoring is degree of periportal necrosis (max. 2568. degree of intralobular focal necrosis (max. 2565 Staging is based on the degree of fibrosis as categorized on a numerical scale from 0-6 (HAI) or 0- 4 (METAVIR). Autoimmune A. All of the above B. hepatic inflammation & necrosis continue for at least 6 months. C. 2568 D. 10 D. Degree of progression. 947 In chronic hepatitis. 2565 confluent necrosis (max. 2568 C. 8 may result due to an excess of ? Harrison’s 18th Ed. Table 306-2 D. 2568 B. 9 months B. All of the above A. 6 months A. Table 306-2 D. Degree of periportal necrosis In Total Parenteral Nutrition (TPN). Cause 955 Which of the following has prognostic importance among B. its histologic assessment of necroinflammatory increased in persons with ? activity. Chronic renal failure A. or grade adults with chronic hepatitis B ? C. A. is done by examination of liver biopsy. and statins can be used safely in these patients. 12 months C. Hepatic steatosis 951 Histologic activity index (HAI) scoring for fibrosis (stage) is done out of ? C. 4). Severe anemia Harrison’s 18th Ed. 2568. A. 6 946 In Total Parenteral Nutrition (TPN). Other underlying liver diseases Harrison’s 18th Ed. hepatic steatosis. a histologic assessment of necroinflammatory activity. degree of hepatocyte degeneration and focal necrosis within lobule and degree of portal inflammation. C. It includes assessment of degree of periportal necrosis. 4 other underlying liver diseases. ~4% Chronic viral hepatitis. Harrison’s 18th Ed. 90 % HBV infection at birth is associated with clinically silent acute infection but a 90% chance of chronic 948 Chronic hepatitis is due to ? infection. 40 % D. drug-induced chronic hepatitis & autoimmune chronic hepatitis can cause chronicity. or grade and its degree of progression. 4). or B. Drug-induced Harrison’s 18th Ed. Carbohydrate calories B. HIV infection 950 Histologic activity index (HAI) scoring for necroinflammatory activity (grade) is done out of ? B. Chronic hepatitis C B. 6). 20 945 Statin hepatotoxicity is increased in which of the following patients ? Out of a maximum of 18. 2568. AJAY MATHUR Cardiology 467 944 The risk of trimethoprim-sulfamethoxazole hepatotoxicity is Classification of chronic hepatitis is based on its cause. C. ~1% D. 18 D. D. Virus 954 What percentage of HBV infection acquired in immunocompetent young adulthood will become chronic ? B. Table 306-2 C. Fat calories Harrison’s 18th Ed. None of the above A. 3 months Harrison’s 18th Ed. HBV replication . Portal fibrosis carbohydrate calories. C. 75 % In chronic hepatitis. degree of intralobular Harrison’s 18th Ed. steatosis or steatohepatitis may result due to an excess of B. 467 Gastroenterology MCQ’s MCQ’s FOR FORPROFESSIONALS MEDICAL MEDICAL PROFESSIONALS BY PROF. steatosis or steatohepatitis D. Histologic activity. 2565 A.Chronic hepatitis Grade. Degree of portal inflammation 306 . 2567 HBV infection in immunocompetent young adulthood carry a risk of chronicity of ~ 1%. hepatic inflammation and necrosis continue for at least ? 953 What percentage of HBV infection acquired at birth will become Harrison’s 18th Ed. A. 3 Statin hepatotoxicity is “not” increased in patients with chronic hepatitis C. or stage. Intralobular necrosis D. 4) & degree of portal inflammation (max. 10 % C. 2567 chronic ? A. Protein calories 952 Histologic activity index (HAI) scoring for necroinflammatory activity (grade) includes all except ? C. All of the above B. Year 2 replication (HBeAg). Absence of HBeAg in serum N Engl J Med 2008. A. Liver histology A. Presence of HBcAg in liver A. Presence of Anti HBs + anti HBe A. C. HLA DR3 or DR4 markers 961 Which of the following is not a complication of chronic hepatitis C.359:1486-500 B.10 % Serum HBV DNA undetectable by PCR is defined as <300 to 400 copies per milliliter (<1000 copies C. Presence of Anti HBs + IgG anti HBc Harrison’s 18th Ed. 20 IU/mL D. 2569 D. 2568 963 Histologic improvement is defined as a reduction of 2 or more points in the histologic activity index at ? A. DNA. Loss of anti-core antigen IgM in chronic HBV infection is ? Harrison’s 18th Ed. D. & anti- core antigen IgM with development of anti-surface antigen antibody & anti-core antigen IgG (total B. Loss of anti-core antigen IgG A. HBV DNA levels < 103 virions/mL A. < 100 to 200 copies per milliliter year ? B. Almost undetectable levels of HBV DNA Inactive HBV carriers have circulating HBsAg. Level of HBV replication anti-core antigen antibody). and minimal liver injury. e antigen. Circulating HBsAg D. 964 Serum HBV DNA undetectable by PCR is defined as ? N Engl J Med 2008. Undetectable HBeAg infection from that which is acquired through vaccination. undetectable 967 Which of the following phase is seen almost exclusively in HBeAg. C. the appearance of anti-HBe. Generalized vasculitis 956 Replicative phase of chronic HBV infection is characterized D. D. A. 2569 C. Month 6 D. those who acquired HBV infection vertically or during early childhood ? . HBV DNA levels > 105 .15 % / mL for adefovir) at the end of year 1.468 MCQ’s FOR MEDICAL PROFESSIONALS BY PROF. AJAY MATHUR Gastroenterology B.106 virions/ C. D. ~ 20 . AST and ALT B? Harrison’s 18th Ed. normal serum aminotransferase levels. levels of HBV DNA below a threshold of ~10 3 virions/mL. 60 IU/mL mL in serum. Distinctions B. by high infectivity & by accompanying liver injury. by presence of HBcAg in liver. histologic features are of prognostic importance.106 virions/mL 962 HBV DNA can be detected in serum at levels as low as ? C.30 % 965 Immunity to HBV infection is characterized by all except ? ~10-15% patients of HBeAg-reactive chronic hepatitis B convert spontaneously from relatively replicative to nonreplicative infection per year. 2569 C. 80 IU/mL 957 Nonreplicative phase of chronic HBV infection is characterized HBV DNA can be detected in the serum at levels as low as 60 IU/mL. by all except ? Harrison’s 18th Ed. 40 IU/mL Replicative phase is characterized by presence of HBeAg & HBV DNA levels over 105 . All of the above B. Year 1 Nonreplicative phase is characterized by the absence of the conventional serum marker of HBV D. Levels of HBV DNA Immunity to HBV infection is characterized by loss of HBV surface antigen. Immune-complex glomerulonephritis in HBV replication and in histologic category do not always coincide. limited infectivity. < 500 to 1000 copies per milliliter A. Levels of aminotransferase activity 966 Which of the following differentiates natural immunity by D. Presence of HBeAg in serum B. A. All of the above resolved HBV infection from that which is acquired through Level of HBV replication is the most important risk factor for the ultimate development of cirrhosis vaccination ? & HCC in both HBeAg-reactive and HBeAg-negative patients. 2569 D. ~ 5 . Presence of HBcAg in liver C. the absence of intrahepatocytic HBcAg. Month 3 C. Raised serum aminotransferase levels Presence of anti HBs & IgG anti Hbc together differentiates natural immunity through resolved C. and almost undetectable levels of HBV DNA.359:1486-500 958 What percentage of HBeAg-reactive chronic hepatitis B convert spontaneously from replicative to nonreplicative infection per A. ~ 10 . Presence of Anti HBs + IgM anti HBc 960 HBV inactive carriers is characterized by all except ? B. < 300 to 400 copies per milliliter Harrison’s 18th Ed. Loss of HBV e antigen 959 Most important risk factor for development of cirrhosis & HCC C. 2568 Hyperglobulinemia & detectable circulating autoantibodies are distinctly absent in chronic hepatitis B in contrast to autoimmune hepatitis. < 1000 to 2000 copies per milliliter B. ~2-5% D. Presence of anti-HBe B. Hyperglobulinemia Among adults with chronic hepatitis B. which is denoted by isolated anti-surface antigen antibody. Loss of HBV surface antigen B. Leukocytoclastic vasculitis by ? Harrison’s 18th Ed. Liver biopsy shows findings of chronic hepatitis in immune clearance and reactivation phase. B. Ribavirin A. is seen almost exclusively in those who acquired HBV infection vertically or during early childhood. and an acute exacerbation. Inactive carrier phase 974 Which of the following drug is beneficial in severe acute D. Glucocorticoid B. Entecavir B. Reactivation phase hepatitis B ? Harrison’s 17th Ed. 10000 IU per milliliter B.359:1486-500 Except autoimmune thyroiditis. D. HBV infection ? Harrison’s 18th Ed. therapy. 2570 C. although it is not an approved C. Nonsignificant hepatitis is observed in inactive carrier phase. an elevated ALT level. Very young children infected at birth B. 977 IFN-a therapy is not effective in which of the following groups 971 e antibody is positive in which of the following phase of HBV of CHB patients ? infection ? Harrison’s 18th Ed. All of the above e antibody is negative in immune tolerance and immune clearance phase. D. Lancet Infect Dis 2005. 469 Gastroenterology MCQ’s MCQ’s FOR FORPROFESSIONALS MEDICAL MEDICAL PROFESSIONALS BY PROF. AJAY MATHUR Cardiology 469 A. Immune tolerance phase C. Old age D. 1946 Immune tolerance phase. Immune clearance phase Harrison’s 18th Ed. Lamivudine D. IFN therapy has not been effective in very young children infected at birth. All of the above 975 Which of the following is approved for treatment of chronic The strongest predictors of spontaneous e antigen seroconversion are old age. Bone marrow suppression C. A. Elevated ALT level In severe acute hepatitis B. Interferon  A. Inactive carrier phase A. Lancet Infect Dis 2005. lamivudine has been successful. Asian patients with minimal-to-mild ALT elevations. Immune tolerance phase A. lamivudine. A. 5000 IU per milliliter 979 Which of the following is not a ‘Nucleoside analogue’ ? Harrison’s 18th Ed. Immune tolerance phase B. Reactivation phase is also termed as HBV e antigen negative chronic hepatitis. in immunosuppressed persons. the initial phase of chronic HBV infection. Immune clearance phase C. All of the above D. Reactivation phase IFN- was the first approved therapy for chronic hepatitis B. 2569. Inactive carrier phase D. 1000 IU per milliliter B. and tenofovir. 2570. Adefovir C. All of the above C. Immune tolerance phase Persons with a serum HBV DNA level below 1000 IU per milliliter and a normal ALT level consistently are considered to be inactive carriers with a low risk of clinical progression. Autoimmune thyroiditis D. or patients with decompensated chronic hepatitis B. Immune tolerance phase reversible upon dose lowering or cessation of therapy ? B.5:374–82 969 Which of the following phase is also called HBV e antigen A. Adefovir . Alopecia 973 Inactive HBV carriers with a low risk of clinical progression are D. Adefovir A. Lamivudine B. Reactivation phase adefovir dipivoxil. Interferon monotherapy 968 Predictor of spontaneous e antigen seroconversion is ? C. Immune clearance phase B. entecavir. Lancet Infect Dis 2005. Immune clearance phase D. 2570 A. all side effects are reversible upon dose lowering or cessation of IFN. Decompensated chronic hepatitis B D. Inactive carrier phase C. Acute exacerbation indication for lamivudine therapy.5:374–82 the following phase of HBV infection ? A. PEG IFN negative chronic hepatitis ? B. Reactivation phase B. Inactive carrier phase To date. 976 Which of the following drugs was the first to be approved for treatment of chronic HBV infection ? 970 Liver biopsy findings are normal or nonspecific in which of Harrison’s 18th Ed. Lamivudine B. 10000 IU per milliliter A. telbivudine. Numbness & tingling of extremities those with a serum HBV DNA level below ? N Engl J Med 2008. Immune clearance phase C. 2570. Immunosuppressed persons C. 972 HBV DNA is high in which of the following phase of HBV infection ? 978  therapy is not Which of the following side effects of IFN- A.5:374–82 C. Entecavir D. seven drugs have been approved for treatment of chronic hepatitis B: PEG IFN. Entecavir is indicated for patients with ? N Engl J Med 2008. 2571 Factors that are most predictive of a response include a high ALT level.000 IU per milliliter 983 Which of the following antiviral drugs has dual antiviral activity C. Blood transfusion C. Clevudine .fluorinated 986 Which of the following has the highest rates of co-infection cytosine analogue. Anti-HBe seroconversion occurs in minority of patients 987 Markers of successful anti HBV therapy include ? N Engl J Med 2004. 2. 2571. liver D. a low HBV DNA level.359:1492 Long-term monotherapy with lamivudine is associated with methionine-to-valine (M204V) or methionine- to. tenofovir and emtricitabine have antiviral activity against HBV & HIV. Moderate-to-severe necroinflammatory activity or fibrosis favours antiviral therapy. Low HBV DNA level 982 Long-term monotherapy with lamivudine is associated with C. Not be used as monotherapy in HBV/HIV co-infected patients D. Likelihood of HBeAg loss in PEG IFN alfa-2b treated HBeAg-reactive patients is associated with HBV genotype A > B > C > D. All of the above Harrison’s 18th Ed. family history of HCC. A. Lancet Infect Dis 2005. If ALT is <2X with HBV DNA >20. ALT level more than two times upper limit of normal against HBV & HIV ? B. 2571. and should always be combined with at least two A. B. Besides lamivudine. Adefovir 990 For HBeAg-negative chronic HBV infection.6 A.pyrimidine nucleoside analogue. 991 Conversion factor for HBV DNA between international units 985 Which of the following is false for HBV treatment in HIV positive (IU) per milliliter and copies per milliliter is about ? patient ? N Engl J Med 2008. Entecavir C. Infection through heterosexual contacts B. Intravenous drug users A.isoleucine (M204I) mutations. factor that is not predictive of a response to antiviral D. 2570.5:374–82 For HBeAg-reactive chronic HBV. All of the above C. 3. Response to Interferon  is lower B. B. B. Emtricitabine C.6 . Reduction of circulating viral load B. N Engl J Med 2008.cytosine analogue. 300 mg/day should be given when treating HBV/HIV co-infected patients. Low ALT level other anti-HIV agents. ALT level more than two times upper limit of normal aspartate (YMDD) motif of HBV DNA polymerase. Lancet Infect Dis 2005. Moderate-to-severe necroinflammatory activity or fibrosis B.000 IU/mL. YMDD 989 For HBeAg-reactive chronic HBV infection. 100 mg/day C. Emtricitabine D. 2571. Emtricitabine . HBV DNA > 20. B. Lancet Infect Dis 2005. Frequent “Flares” while on HAART Adefovir dipivoxil and Tenofovir are nucleotide analogues.6 B. Emtricitabine D. antiviral therapy is indicated if ALT levels are more than twice the upper limit of normal and HBV DNA >20. or ALT level in high normal range (up to twice the upper limit of normal).5:374–82 A. Seroconversion to anti-HBe antibodies Harrison’s 18th Ed.470 MCQ’s FOR MEDICAL PROFESSIONALS BY PROF. 200 mg/day D.5:374–82 B. and A. Chemokine (C-C motif) receptor 6 mild-to-moderate histologic activity & stage. AJAY MATHUR Gastroenterology C.350:1118-29 981 The inhibitory dose of Lamivudine for treating HBV/HIV co. HBV genotype A>B>C>D Harrison’s 18th Ed. primarily at amino acid 204 in the tyrosine-methionine-aspartate. Co-infection with hepatitis C rare D.5:374–82 A. A.5:374–82 A. Risk factors for progression against HBV & HIV ? D. Lamivudine .359:1492 Lancet Infect Dis 2005. All of the above C.000 IU/mL. Tenofovir C. Lamivudine For HBeAg-negative chronic HBV. RNA-binding motif (RBM) C. Mild-to-moderate histologic activity and stage mutation in which motif of HBV DNA polymerase ? D. 4. Risk factors for progression (older than 40 years.359:1492 984 Which of the following antiviral drugs has dual antiviral activity A. HBV DNA > 20. All of the above biopsy is indicated. Indefinite treatment with nucleoside/nucleotide analogues C.000 IU per milliliter Harrison’s 18th Ed. Anti-HIV & anti-HBV activity C.5:374–82 980 Which of the following statements about lamivudine is false ? A. 300 mg/day 988 In CHB. antiviral therapy is D. Men who have sex with men Harrison’s 18th Ed. 400 mg/day therapy is ? Harrison’s 18th Ed. 2572. antiviral therapy is indicated if ALT levels are more than twice the A. Dose lower for blocking HIV replication than HBV replication D. All of the above indicated for patients with ? N Engl J Med 2008. with HBV & HIV ? Lancet Infect Dis 2005. antiviral therapy D. Entecavir .000 IU/mL. Telbivudine .nucleoside analogue.guanosine analogue polymerase inhibitor. Loss of HBeAg infected patients is ? B. Enfuvirtide upper limit of normal and HBV DNA >20.359:1492 The inhibitory dose of Lamivudine for blocking HBV replication is 100 mg/day. Lancet Infect Dis 2005. 1. other antivirals for hepatitis B should be avoided or used with extreme Harrison’s 18th Ed. PEG IFN D. Adefovir Dipivoxil Adefovir at 30mg has higher antiviral potency. Tenofovir D. Tenofovir D. Adefovir. Tenofovir 997 The most potent of the HBV antivirals is ? Lamivudine should never be used as monotherapy in HBV-HIV infection. 2008 . A. Harrison’s 18th Ed. 2572 A. PEG IFN B. entecavir. Adefovir Patients with HBV genotype A have the highest rate of IFN-induced HBeAg loss. 2572. Entecavir independent of HBV genotype. C. 2577 D. 61-75 caution during pregnancy. PEG IFN. 2005 C. 2005 . Tenofovir D. adefovir somewhat less so. Genotype A 1002 In patients with HBV-HIV infection. 2573 emerges rapidly to both viruses. Lamivudine C. Among the drugs for hepatitis B.Tenofovir (TDF). Therapeutic Advances in Gastroenterology 2008. Telbivudine 1003 Variables that favour the treatment of HBV/HIV co-infected C. entecavir has supplanted lamivudine. Polyethylene glycerol A. 180 µg subcutaneously weekly for one month 1001 Which of the following should be avoided or used with extreme caution during pregnancy ? B. Lamivudine B. AJAY MATHUR Cardiology 471 D. Genotype B be used as monotherapy ? C.Adefovir (ADV).Telbivudine (LDT). 2002 A. 1998 . like syndrome with phosphaturia and proteinuria. D. 5. 61-75 Harrison’s 18th Ed. 999 Which of the following is recommended as first-line therapy in 993 PEG stands for ? Chronic Hepatitis B ? Harrison’s 18th Ed. entecavir. 180 µg subcutaneously weekly for six months A. 1. Genotype D A. Tenofovir disoproxil fumarate. Polyethylene glycol C. Entecavir B. Telbivudine 1992 . Perethylene glycerol B. Adefovir Lamivudine and telbivudine foster the emergence of viral mutations.Entecavir (ETV). B. HBeAg clearance associated with nucleos(t)ide analogues is C. Treatment thresholds in copies/mL are 5 times higher than international units/mL. 471 Gastroenterology MCQ’s MCQ’s FOR FORPROFESSIONALS MEDICAL MEDICAL PROFESSIONALS BY PROF. 2573 A.6 Entecavir is an oral cyclopentyl guanosine analogue polymerase inhibitor and is the most potent of the HBV antivirals. or tenofovir are recommended 994 Pegylated IFN-a was approved by FDA for chronic hepatitis B as first-line therapy. Its high barrier to resistance coupled with its high potency and Conversion factor HBV DNA between international units /mL & copies/mL is ~ 5. Entecavir C. and 995 The recommended regimen of peg IFNa-2a for CHB is ? entecavir (except in lamivudine-experienced patients) and tenofovir rarely at all. All of the above IFN-induced HBeAg loss ? Except for lamivudine. 2000 emergence of viral mutations ? Harrison’s 18th Ed. Entecavir 996 Which of the following HBV genotype has the highest rate of D. Fanconi-like syndrome C. in which year ? Therapeutic Advances in Gastroenterology 2008. Pegylated IFN-a. PEG IFN has supplanted standard IFN. Perethylene glycol D. 2575 B. 2581 Harrison’s 18th Ed. A. 2576 C. 180 µg subcutaneously weekly for one year B. D. 2006 . Tenofovir patients with pegylated interferon are all except ? Lancet Infect Dis 2005. Goodpasteur's syndrome B.Interferon alfa (IFN-a).5:374–82 D. 2003 B.Lamivudine (LAM). because HIV resistance Harrison’s 18th Ed. 180 µg subcutaneously weekly for three months Harrison’s 18th Ed. Telbivudine C. Entecavir B. an acyclic nucleotide analogue similar to adefovir. 992 Nephrotoxicity of adefovir is best related to ? 998 Tenofovir is similar to which of the following ? Therapeutic Advances in Gastroenterology 2008. Adefovir Recommended regimen for peg IFNa-2a in CHB is 180 µg subcutaneously weekly for one year. Adefovir Dipivoxil . 2575 A. All of the above Pegylated interferon (PEG IFN) is a long-acting IFN bound to polyethylene glycol (PEG). Genotype A is most common in North America & Europe.6 (1 IU/mL is ~ 5. It’s potential nephrotoxicity manifests as a Fanconi. All of the above D.6 an excellent safety profile renders entecavir a first-line drug for patients with chronic hepatitis copies/mL). Genotype C Harrison’s 18th Ed. 1. 61-75 1000 Therapy with which of the following is least likely to foster A. and tenofovir has supplanted adefovir. 2002 . Acute glomerulonephritis A. Tenofovir and tenofovir + emtricitabine can be used for treating HBV infection in HBV-HIV co-infected patients. which of the should never B. Figure 306-2 A. level of aminotransferase activity. porphyria cutanea tarda. advanced histologic stage and grade. 12 weeks hemochromatosis. level of HCV RNA. No other epidemiologic or clinical features of RVR refers to undetectable HCV RNA at week 4. 2579 Harrison’s 18th Ed. duration of infection is the most important. Weight loss D. 4 weeks C. Essential mixed cryoglobulinemia D? D. 48 weeks D. Jaundice B. 12 weeks B. HIV infection. and steatohepatitis). 1008 Which of the following is the most common symptom in chronic 1014 Responder status is estimated at what time after institution of hepatitis C ? therapy in Hepatitis C ? Harrison’s 18th Ed. Psychiatric disorders B. Anti-liver kidney microsomal type 1 (LKM1) antibody. Jaundice is rare chronic hepatitis C virus (HCV) infection ? C. HBeAg-positive 1009 Extrahepatic complications unrelated to immune-complex B. ribavirin enhances the 1006 Out of the following. lichen B. 2579. Occasionally. Severity of jaundice B. HIV infection A. increased hepatic iron. . Liver histology A. Out of these. Fever C. 2579. While those unrelated to immune-complex injury are Sjögren’s syndrome. 2579. Adefovir Non-organ specific autoantibodies (NOSA). 2 weeks B. Circulating anti-LKM1 antibodies Harrison’s 18th Ed. presence or absence of jaundice during acute hepatitis) are predictive of eventual outcome. Elevated aminotransferases injury in chronic hepatitis C are all except ? Harrison’s 18th Ed. Jaundice is rare. 2577 Essential mixed cryoglobulinemia is an immune complex–mediated extrahepatic complications of A. Figure 306-2 A. D. chronic hepatitis B. 4 weeks Progression of liver disease in chronic hepatitis C is more likely in older age. 2579 C. Anti-LKM3 D.472 MCQ’s FOR MEDICAL PROFESSIONALS BY PROF. Figure 306-2 C. 72 weeks Fatigue is the most common symptom of chronic hepatitis C. 2579 D. Porphyria cutanea tarda 1004 Which of the following antibody is seen in chronic hepatitis C. Fatigue A. 2578 A. Smooth muscle antibodies (SMA) B. A. concomitant other liver disorders (alcoholic liver disease. Tenofovir Oral guanosine nucleoside ribavirin is ineffective when used alone. HIV status D. genotype 1. Lichen planus Harrison’s 18th Ed. 2579 C. 2 weeks D. Fluctuating aminotransferase levels 1005 Which of the following antibody is prevalent in patients with B. which one is the most important as regards efficacy of IFN by reducing the likelihood of virologic relapse after the achievement of an end- progression of liver disease in chronic hepatitis C ? treatment response (ETR). D. All of the above A. Entecavir infection. 12 weeks Best prognostic indicator in chronic hepatitis C is liver histology. 8 weeks infection. Sjögren’s syndrome D. Obesity B. EVRrefers to 2 log10 HCV RNA reduction by week 12. 1-antitrypsin deficiency. 24 weeks C. Antinuclear (ANA) antibodies 1011 Which of the following enhances the efficacy of IFN ? Harrison’s 18th Ed. Anti-liver kidney microsomal type 1 (LKM1) antibody D. Anti-LKM4 1010 Which of the following in a laboratory feature of chronic hepatitis C ? A distinguishing serologic feature of chronic hepatitis D is the presence in the circulation of anti. Older age 1012 Rapid virologic response (RVR) is estimated at what time after institution of therapy in Hepatitis C ? B. Anti-liver cytosol type 1 (LC1) are found. and obesity. Longer duration of infection Harrison’s 18th Ed. AJAY MATHUR Gastroenterology A. 2578 Harrison’s 18th Ed. 1013 Early virologic response (EVR) is estimated at what time after 1007 Best prognostic indicator in chronic hepatitis C is ? institution of therapy in Hepatitis C ? Harrison’s 18th Ed. chronic hepatitis C (severity of acute hepatitis. Harrison’s 18th Ed. 8 weeks D. type-II diabetes mellitus and metabolic syndrome (including insulin resistance & steatohepatitis). Harrison’s 18th Ed. Anti-LKM2 planus. Ribavirin B. particularly smooth muscle antibodies (SMA) and antinuclear (ANA) antibodies are highly prevalent in patients with chronic hepatitis C virus (HCV) C. Anti-LKM1 chronic hepatitis C. High CD4 counts A. But. 2579 LKM3 directed against uridine diphosphate glucuronosyltransferase. more complex quasispecies diversity. All of the above A. Levels of aminotransferases C. longer duration of C. C. PEG IFN 1023 Which of the following is a side effect of ribavirin therapy ? B. 2580 Harrison’s 18th Ed. 12 weeks A. variants of the IL28B SNP that code for 1016 Interferon therapy results in activation of ? IFN-3 correlate significantly with responsiveness. Five times Harrison’s 18th Ed. 12 weeks D. PEG IFN + Ribavirin A. Moderate to severe hepatitis on liver biopsy C. 2 . 2580 A. All of the above D.000 IU/ml). 2580 D. Harrison’s 18th Ed. 2579. Histologically mild hepatitis Despite a Hepatitis C virion halflife in serum of only 2 . Pruritus D. which results in the intracellular elaboration of genes & their protein products with antiviral properties. Three times 1 should last for ? B. D. All of the above B. 2583. and female gender. B. 2580 C. Agranulocytosis proteins inhibit JAK-STAT signaling pathway. JAKSTAT signal transduction pathway 1022 The most pronounced side effect of ribavirin therapy is ? Harrison’s 18th Ed. Patients with cirrhosis respond less favourably. HLA-B*1501 C. IL28B In studies of patients treated with PEG IFN and ribavirin. 2 .3 hours. All of the above A. Seizure Interferon therapy results in activation of the JAKSTAT signal transduction pathway. Table 306-7 Harrison’s 18th Ed. D. 2580 1019 Duration of IFN-ribavirin therapy have in patients with genotype A. Patients homozygous for the C allele at this locus Harrison’s 18th Ed. Hepatitis C B. and exogenous interferon restores expression of C. while in those with genotypes 2 and 3. 24 weeks C.3 days B. Gout C. standard IFN ? Harrison’s 18th Ed. All of the above 1018 Which of the following variable does not correlate favourably 1024 Halflife in serum of Hepatitis C virion is ? in the IFN-based treatment of chronic hepatitis C ? Harrison’s 18th Ed. 12 weeks B. Genotypes 2 and 3 B. 2579 have the highest frequency of achieving an SVR (80%). histologically 1025 Elimination time of PEG IFN is how many times longer than mild hepatitis and minimal fibrosis. 2 . Low baseline HCV RNA level D. 72 weeks 1027 Antiviral therapy is not recommended in which of the following Combination IFN-ribavirin therapy in chronic hepatitis C with genotype 1 should for 48 weeks. Ribavirin Harrison’s 18th Ed. a 24-week course of therapy suffices. Chemokines C. absence of obesity as well as insulin resistance and type-II diabetes mellitus. Genotypes 1 and 4 C. AJAY MATHUR Cardiology 473 1015 Sustained virologic response (SVR) is estimated at what time 1021 Which of the following is relevant for patients treated with PEG after institution of therapy in Hepatitis C ? IFN and ribavirin ? Harrison’s 18th Ed. 473 Gastroenterology MCQ’s MCQ’s FOR FORPROFESSIONALS MEDICAL MEDICAL PROFESSIONALS BY PROF. Anemia The current standard treatment of chronic hepatitis C is the combination of long acting pegylated IFN (PEG IFN) and ribavirin. those homozygous for the T allele at this locus are least likely to achieve an SVR (25%). the level of HCV is maintained by Patient variables that tend to correlate with sustained virologic responsiveness to IFN-based a high replication rate of 1012 hepatitis C virions per day. age <40. D. 2580 A. 48 weeks C.3 monhs C. 48 weeks and these have elimination times up to sevenfold longer than standard IFNs. 2580 C.3 years D. 2580 A. which is equivalent to 800. Cytokines intermediate level of responsiveness (SVRs in 35%). standard IFNs have now been supplanted by PEG IFNs C. 72 weeks D.3 hours A. Seven times A. Nine times B. Figure 306-2 Harrison’s 18th Ed. patients of chronic hepatitis C ? . and those heterozygous at this locus (C/T) have an A. 2 . 24 weeks For the treatment of chronic hepatitis C. 48 weeks D. 2579 A. low baseline HCV RNA level (<2 million copies/mL. 72 weeks 1026 Standard indication for antiviral therapy of chronic hepatitis C 1020 Duration of IFN-ribavirin therapy have in patients with genotype is ? 2 and 3 should last for ? Harrison’s 18th Ed. Hemolysis interferon-stimulated genes and their antiviral effects. IL15 D. therapy include favorable genotype (genotypes 2 and 3 as opposed to genotypes 1 and 4). All of the above 1017 The current standard treatment of chronic hepatitis C is ? The most pronounced side effect of ribavirin therapy is hemolysis. Detectable HCV RNA (with or without elevated ALT) A. 24 weeks B. Portal/bridging fibrosis on liver biopsy B. SLCO1B1 B. porcine liver D. 2583.5 µg / kg weekly subcutaneously B. A. Measure HCV RNA at 12 weeks 1039 Autoimmune hepatitis is a kind of ? C. Longer duration of treatment D. Exchange transfusion A. 12 weeks D. Lamivudine B. Contraindications to use of interferon or ribavirin D. Table 306-7 D. Adefovir has a slow action & potential nephrotoxicity. PEG IFN -2a is given 180 µg weekly that appears to improve survival. Porphyria cutanea tarda of genotypes 1 and 4 is ? C. Age > 60 years responded to treatment with PEG IFN / ribavirin in chronic B. Persons with severe renal insufficiency A. Extracorporeal liver-assist devices C. 1033 Which of the following is recommended for the success of therapy with PEG IFN / ribavirin in chronic hepatitis C ? Harrison’s 18th Ed. Table 306-7 B. which of the following has been shown D. Lichen planus Harrison’s 18th Ed. However. 180 µg weekly subcutaneously D. telbivudine and PEG IFN -2b is given 1. none of these approaches achieves more than a marginal benefit. Interferon monotherapy 1030 Duration of PEG IFN / ribavirin therapy in chronic hepatitis C C. Telbivudine B. 2584-85 be referred for liver transplantation.5 µg / kg weekly subcutaneously Nucleoside/nucleotide analogues recommended in acute liver failure are lamivudine. C. 2583. Mild hepatitis on liver biopsy hepatitis C ? Harrison’s 18th Ed. Lamivudine A.474 MCQ’s FOR MEDICAL PROFESSIONALS BY PROF. 1946 1031 Dose of PEG IFN  -2a is ? A. 2583. 72 weeks A. AJAY MATHUR Gastroenterology Harrison’s 18th Ed. Table 306-7 For those who have not responded to treatment with PEG IFN / ribavirin in chronicn hepatitis C. Glucocorticoid therapy Harrison’s 18th Ed. ribavirin. 2583.5 µg / kg weekly subcutaneously D. 180 µg weekly intramuscularly cross-perfusion. Orthotopic liver transplantation also has shown excellent results. 24 weeks 1036 Which of the following drug is beneficial in acute hepatitis C ? C. 2579 D. 72 weeks to improve survival ? Harrison’s 17th Ed. Pretreatment liver biopsy Harrison’s 18th Ed. interferon monotherapy (3 million units SC three times a week) is beneficial. Adefovir C. Interferon drugs are contraindicated because they can worsen hepatitis.5 µg / kg weekly subcutaneously C. Decompensated cirrhosis following may be tried : longer duration of treatment. subcutaneously. Meticulous intensive care that includes prophylactic antibiotic coverage is one factor Two PEG IFNs are available: PEG IFN -2b and -2a. Acute hepatitis . Table 306-7 A. 2584 C. Essential mixed cryoglobulinemia 1029 Duration of PEG IFN / ribavirin therapy in chronic hepatitis C B. 2583. Prophylactic antibiotic coverage B.5 µg / kg weekly subcutaneously. Table 306-7 A. Glucocorticoid A. 0. 2. Switching to a different IFN preparation candidates for IFN-based antiviral therapy ? D. All of the above Harrison’s 18th Ed. All of the above 1035 Which of the following associated with hepatitis C may respond to antiviral therapy ? Patients with decompensated cirrhosis are not candidates for IFN-based antiviral therapy but should Harrison’s 18th Ed. 1. entecavir. 2583. exchange transfusion. 2585 D. 180 µg daily subcutaneously C. hemoperfusion and extracorporeal liver-assist devices have not been proved to enhance survival. or B. Table 306-7 1034 Which of the following is advocated in those who have not A. 48 weeks Harrison’s 18th Ed. 48 weeks 1037 In fulminant hepatitis. human cross-circulation. ribavirin. Entecavir D. or both 1028 Which of the following patients of chronic hepatitis C are not C. 1038 Drugs that have been recommended in acute liver failure are 1032 Dose of PEG IFN  -2b is ? all except ? Harrison’s 18th Ed. 12 weeks In acute hepatitis C. Long-acting pegylated interferon plus ribavirin is superior to interferon monotherapy. 3. All of the above A. A. HCV genotype should be determined prior to therapy B. Ribavirin of genotypes 2 and 3 is ? Harrison’s 18th Ed. Pregnancy both and switching to a different IFN preparation. B. All of the above B. plasmapheresis. Interferons are antiproliferative and ribavirin is teratogenic. 180 µg daily intramuscularly Glucocorticoid therapy. Higher doses of either PEG IFN. 2582 Autoimmune hepatitis A. 24 weeks C. All of the above B. higher doses of either PEG IFN. Extrahepatic features of autoimmunity LP autoantibodies). Arthritis D. antibodies to “soluble liver antigen/liver pancreas antigen” (SLA/ A. All of the above A. N Engl J Med 2006. 2586 C. Scurfin is the protein product of FOXP3 gene that is a member of forkhead family of transcription factors. D. HLA-DR3 Autoimmune disorders that occur with increased frequency in autoimmune hepatitis patients are D. All of the above hepatitis ? Harrison’s 18th Ed. 2585 serotype ? A. levels of scurfin are decreased. B. All of the above B. B. cytomegalovirus. . type 1 diabetes mellitus. 2586 B. Associated with a more severe disease course In autoimmune hepatitis. anti-LKM. Atorvastatin C. which can progress to cirrhosis & liver failure. 2586 D. Specific serologic marker for autoimmune liver diseases hepatitis ? Anti-SLA is serologic marker for autoimmune liver diseases. Any of the above A. C.354:54-66. Alpha globulin B. hepatitis viruses. All of the above C. Interferon B. usually with fibrosis. particularly gamma globulin and IgG. immunoregulatory dysfunction of B. FOXP1 gene B. the protein product of FOXP3 gene occurs in autoimmune hepatitis. Rheumatoid arthritis A. CD4+CD24+ regulatory T cells viruses . FOXP2 gene D. 2586 C. 2585. ulcerative colitis. interferon. Cutaneous vasculitis One characteristic laboratory feature of autoimmune hepatitis is a generalized elevation of serum globulins. C. AJAY MATHUR Cardiology 475 B. Hepatitis viruses 1048 In autoimmune hepatitis. FOXP4 gene Oxyphenisatin. All of the above C. Beta globulin 1047 Extrahepatic manifestation of autoimmune hepatitis is ? Harrison’s 18th Ed. antibodies to liver-specific asialoglycoprotein receptor (“hepatic lectin”). None of the above thyroiditis. 1044 Autoimmune disorder that occur with increased frequency in autoimmune hepatitis patients is ? 1050 Type 1 autoimmune hepatitis is associated with which HLA-DR Harrison’s 18th Ed. Chronic hepatitis autoimmune hepatitis ? Harrison’s 18th Ed. membranoproliferative Type I autoimmune hepatitis is associated with HLA-DR3 or HLA-DR4. Harrison’s 17th Ed. extrahepatic features of autoimmunity & seroimmunologic abnormalities B. Most evidence is related to hepatitis C. Target of anti-SLA is a UGA serine tRNA-associated protein complex [tRNP(Ser)Sec]. CD4+CD25+ regulatory T cells 1043 Which of the following drug has been implicated in Immunoregulatory dysfunction characterized by decreased numbers of CD4+CD25+ regulatory T cells pathogenesis of autoimmune hepatitis ? and decreased levels of scurfin. arthritis. Subacute hepatitis 1045 Which of the following autoantibody is present in patients of C. glomerulonephritis.2 to 3. diclofenac. N Engl J Med 2006. autoimmune hemolytic anemia. cutaneous vasculitis and glomerulonephritis. Immunologic abnormalities antigen is false ? D. anti-smooth- muscle antibodies (actin). Anti-LKM C. Epstein–Barr virus A. though autoantibodies do not occur in all cases. or C. Glomerulonephritis D. Antinuclear antibodies (ANAs) Autoimmune hepatitis is a chronic disorder characterized by continuing hepatocellular necrosis & inflammation. Present in nonhepatic autoimmune disorders are prominent & supports an autoimmune process in its pathogenesis.354:54-66 A. and Epstein–Barr virus have been inplicated as initiators of autoimmune hepatitis in genetically predisposed. Antibodies to soluble liver antigen/liver pancreas antigen 1040 Which of the following is prominent in cases of autoimmune D. celiac disease & Sjögren’s syndrome. 1967 A. Measles virus which of the following occurs ? Harrison’s 18th Ed. Gamma globulin A. 2585 Autoantibodies found in autoimmune hepatitis patients are antinuclear antibodies (ANAs). N Engl J Med 2006. All of the above 1042 Which of the following virus has been implicated in pathogenesis of autoimmune hepatitis ? Extrahepatic manifestations of autoimmune hepatitis include arthralgias. Thyroiditis Harrison’s 18th Ed. Target of anti-SLA is tRNP(Ser)Sec 1041 Which class of serum globulins is elevated in autoimmune D. 475 Gastroenterology MCQ’s MCQ’s FOR FORPROFESSIONALS MEDICAL MEDICAL PROFESSIONALS BY PROF. CD8+CD25+ regulatory T cells Measles virus. HLA-DR2 D. A.354:54-66 disease course. Serologic abnormalities 1046 Which of the following statements about anti soluble liver C. HLA-DR1 C. CD8+CD24+ regulatory T cells D.0 times normal. B. FOXP3 gene D. Autoimmune hemolytic anemia B. pemoline. rheumatoid arthritis. is associated with a more severe Harrison’s 18th Ed. In autoimmune hepatitis. Delta globulin B.A. nitrofurantoin. minocycline & atorvastatin can induce hepatocellular injury that mimics autoimmune hepatitis. Minocycline 1049 Scurfin is the protein product of which gene ? A. which are generally 1. methyldopa. it is virtually absent in nonhepatic autoimmune disorders. D. All of the above 1054 Type I autoimmune hepatitis is associated with which of the following antibodies ? Anti-LKM1 antibody seen in type II autoimmune hepatitis is seen in some patients with chronic hepatitis C. 2586 A. SLA/LP autoantibodies Harrison’s 18th Ed. Chronic hepatitis C HLA-DR4 associated autoimmune hepatitis is more common in adults and associated with increased incidence of extrahepatic manifestations. P450 2D6 D. Antinuclear antibodies (ANA) A. Chronic hepatitis D D. Antiactin antibodies alcohol. P450 2C19 hepatitis. 2586.354:54-66 1058 In type II autoimmune hepatitis. Anti–LKM-1 & anti–LC-1 C. P450 3A 1053 Feature of HLA-DR4 associated autoimmune hepatitis include ? In type II autoimmune hepatitis. concurrent other autoimmune diseases. Antimitochondrial antibody (AMA) is the sine qua C. 2587 C. Predominant aminotransferase elevation SLA/LP autoantibodies are the most specific autoantibody identified in type 1 autoimmune hepatitis C. predominant following antibodies ? aminotransferase elevation. and other autoantibodies. and viral inclusions. Antiactin antibodies autoimmune hepatitis ? B. histologic evidence of bile duct injury. HLA DR3 or DR4 markers. A. Common in adults 1059 Anti-LKM antibody seen in Type II autoimmune hepatitis is B. HLA-DRB3 C. rosettes). mitochondrial antibodies. Early-onset directed against which cytochrome ? B. N Engl J Med 2006. severe form of autoimmune C. Features against the diagnosis are predominant alkaline phosphatase elevation. Anti-LKM1 non of primary biliary cirrhosis (PBC) but may be observed in the so-called overlap syndrome with autoimmune hepatitis. Better response to corticosteroid therapy A. Atypical perinuclear antineutrophilic cytoplasmic antibodies 1060 Type III autoimmune hepatitis is associated with which of the following antibodies ? C. Antinuclear antibodies (ANA) B. D. Globulin level elevation 1056 Type II autoimmune hepatitis is associated with which of the Features that favour diagnosis of autoimmune hepatitis include female gender. presence of nuclear. 2586 C. B. Anti-LKM1 characterize type 2 disease. the anti-LKM1 antibody is directed against cytochrome P450 2D6 (CYP2D6). milder disease & a better response to corticosteroid therapy. 2586 features (interface hepatitis.476 MCQ’s FOR MEDICAL PROFESSIONALS BY PROF. Atypical perinuclear antineutrophilic cytoplasmic antibodies Type II autoimmune hepatitisis linked to HLA-DRB1 & HLA-DQB1 haplotypes. Also seen is autoantibody directed against liver cytosol formiminotransferase cyclodeaminase. C. 2586 chronic hepatitis D. In girls and young women A. characteristic histologic Harrison’s 18th Ed. Antinuclear antibodies (ANA) Type I autoimmune hepatitis is associated with circulating ANAs. smooth muscle. Antibodies to soluble liver antigen/liver pancreas antigen atypical perinuclear antineutrophilic cytoplasmic antibodies (pANCA) & SLA/LP autoantibodies. D.354:54-66 1061 Which of the following feature goes against the diagnosis of A. history of hepatotoxic drugs or excessive B. All of the above B. D. the anti-LKM1 antibody is A. 2586 Harrison’s 18th Ed. which often occurs in girls and young women. Anti-LKM2 is seen in drug-induced hepatitis & anti-LKM3 is seen in patients with Harrison’s 18th Ed. All of the above B. LKM1.354:54-66 A. Type II autoimmune hepatitis is associated not with ANA but with anti-LKM1. Antiactin antibodies C. Extrahepatic manifestations also seen in which of the following ? Harrison’s 18th Ed. markers of viral hepatitis. autoantibodies against actin. Antinuclear antibodies (ANA) B. Anti-LKM1 B. Atypical perinuclear antineutrophilic cytoplasmic antibodies . 2586 C. Antiactin antibodies Harrison’s 18th Ed. Drug-induced hepatitis D. HLA-DRB1 A. or such atypical histologic features as fatty infiltration. HLA-DRB2 B. P450 2C9 HLA-DR3–associated disease is more common in the early-onset. Mitochondrial antibodies but is found in only 10 to 30 percent of cases. 2586 D. Female gender D. All of the above A. Antiactin antibodies are more specific for type 1 autoimmune hepatitis. Anti-liver cytosol 1 (anti–LC-1) D. Severe form Harrison’s 18th Ed. plasma cells. & response to A. Harrison’s 18th Ed. 1052 Feature of HLA-DR3 associated autoimmune hepatitis include ? N Engl J Med 2006. a liver-specific 58-kD metabolic enzyme (anti-liver cytosol 1). Antinuclear antibodies (ANA) treatment. iron overload. N Engl J Med 2006. All of the above 1055 Which of the following is the most specific autoantibody Type III autoimmune hepatitis patients lack ANA and anti-LKM1 but have circulating antibodies to identified in type 1 autoimmune hepatitis ? soluble liver antigen/liver pancreas antigen. AJAY MATHUR Gastroenterology 1051 Type 2 autoimmune hepatitis is associated with which HLA 1057 Type II autoimmune hepatitis is associated with which of the haplotype ? following antibodies ? Harrison’s 18th Ed. globulin level elevation. HLA-DRB4 D. 2587 One beer. 12 . alcoholic hepatitis & cirrhosis. Table 307-1 globulin level >= twice normal A. 20 . 40–80 g/d of ethanol produces fatty liver. 2589 B.80 gm / day cyclosporine. Mononuclear-cell infiltrate B.40 gm / day Mainstay of management in autoimmune hepatitis is daily glucocorticoid therapy. Major lesions in the pathology of alcoholic liver disease are fatty liver. 80 gm / day for 5 . 2589. 307 . or one ounce of 80% spirits all contain ~12 grams of alcohol. All of the above Harrison’s 18th Ed. ~ 12 grams 1063 Mainstay of management in autoimmune hepatitis is ? D. Alcohol is a direct hepatotoxin Harrison’s 18th Ed. tacrolimus. None of the above A. what quantity & duration of ethanol consumption causes hepatitis or cirrhosis ? B. 80 . 80 gm / day for 10 . Table 307-1 C. C.10 years D. Alcohol injury does not require malnutrition B.10 years C. 2587 Alcoholic hepatitis is thought to be a precursor to cirrhosis. Patients refractory to this regimen may be treated with C. four ounces of wine.20 years of ethanol consumption causes hepatitis or cirrhosis. NASH . Table 307-1 cirrhosis ? A.18 months A. 3 . 10 . A. 2589 B. Harrison’s 18th Ed. 6 . Alcoholic hepatitis D. Serum AST >= 10 times upper limit of normal 1070 In men. what quantity of ethanol produces fatty liver ? Harrison’s 18th Ed. 15 % D. D.20 gm / day D. 30 % C. 75 % Only 15% of alcoholics develop alcoholic liver disease. D. Men more susceptible to alcoholic liver injury than women A. Harrison’s 18th Ed. 2589.24 months B. Bridging necrosis or multiacinar necrosis on liver biopsy B. 18 . ~ 8 grams Autoimmune hepatitis is characterized by a mononuclear-cell infiltrate (plasma cells + eosinophils) invading the limiting plate (piecemeal necrosis or interface hepatitis). Table 307-1 alcoholic liver disease ? A. 2589. 160 gm / day for 10 .20 years 1065 Therapy for autoimmune hepatitis should continue for at D. Fatty liver C. Cytochrome P450 3A Harrison’s 18th Ed. 2589. AJAY MATHUR Cardiology 477 1062 Which of the following is false about histopathological features C. 1073 Which of the following is related to the genetic risk factors for alcoholic liver disease ? 1067 Which of the following is considered to be a precursor to Harrison’s 18th Ed. 2589. Interface hepatitis or piecemeal necrosis 1068 One beer (12 oz). All of the above C. Cirrhosis D. A. 477 Gastroenterology MCQ’s MCQ’s FOR FORPROFESSIONALS MEDICAL MEDICAL PROFESSIONALS BY PROF. Serum AST >= 5 times upper limit of normal and gamma. Glucocorticoid 1069 In men. Cytochrome P450 2C19 A. All of the above Women are more susceptible to alcoholic liver disease than men. 160 gm / day for 10 . 2589 D. 15% of alcoholics develop alcoholic liver disease C.12 months Harrison’s 18th Ed. Azathioprine alone is not effective in achieving remission.20 years least ? In men. Cytochrome P450 2D6 B. Fatty liver C. 160 gm / day for 5 . Cyclosporine A. Alcoholic hepatitis of liver in autoimmune hepatitis ? D. or mycophenolate mofetil. Table 307-1 C. 45 % Therapy for autoimmune hepatitis should continue for at least 12-18 months.6 months 1071 What proportion of alcoholics develop alcoholic liver disease ? B. Harrison’s 18th Ed. 2587 A. 2587 A. 40 . ~ 14 grams Harrison’s 18th Ed. Mycophenolate mofetil B. ~ 10 grams and fibrosis is present.Alcoholic liver disease 1072 Which of the following statements about alcohol consumption is false ? 1066 Which of the following is the major lesion in the pathology of Harrison’s 18th Ed. Azathioprine B. four ounces of wine or one ounce of 80% C. Sparing of the biliary tree spirits contain how many grams of alcohol ? Harrison’s 18th Ed. Biliary tree is generally spared B.120 gm / day 1064 Severe autoimmune hepatitis is defined as ? In men. Fibrosis in perisinusoidal space of Disse B. portal hypertension. AST : ALT ratio is ? hepatocytes coincides with the location of alcohol dehydrogenase which is the major enzyme Harrison’s 18th Ed. AST:ALT ratio is higher in pericentral hepatocytes than other regions in liver lobule & pericentral zone is more selectively affected in acute alcoholic C. Ballooning degeneration Harrison’s 18th Ed. Acetaldehyde subsequently is metabolized to hepatitis are all except ? acetate via acetaldehyde dehydrogenase. Perivenular fibrosis In alcoholic hepatitis. All of the above B. Zone 3 A. Acetate D. with no lunula (white crescent-shaped area of finger). Acute hepatitis B B. None of the above A. or variceal bleeding can occur in the absence of C.478 MCQ’s FOR MEDICAL PROFESSIONALS BY PROF. D. Fibrotic response 1084 In alcoholic hepatitis. Patients with alcoholic cirrhosis often exhibit clinical features identical to other causes of cirrhosis. Alcohol intake also decreases efficacy of interferon-based antiviral therapy in them. cirrhosis. 40 grams Disse. They converts alcohol to acetaldehyde. cytochrome P4502E1. alcohol dehydrogenase and cytochrome P-450 2E1. Ingestion of 160 gm/day increases risk of developing alcoholic cirrhosis by 25-folds. 20 grams Hallmark of alcoholic hepatitis is hepatocyte injury characterized by ballooning degeneration. 2590. Zone 2 1074 Cytochrome P-450 2E1 converts alcohol to ? C. Harrison’s 18th Ed. Mallory bodies are often present in florid cases but are neither specific nor necessary for D. Free fatty acids central vein i. Mallory bodies A. Weather heat skin Harrison’s 18th Ed. Cheese wind skin 1077 Chronic alcohol ingestion lead to which of the following ? C. Inflammatory response absence of cirrhosis ? Harrison’s 18th Ed. Carbon dioxide Histologically. Ascites Harrison’s 18th Ed. C. D. Zone 1 B. Polymorphonuclear infiltrate 1075 In men. Acute hepatitis C Terry's nails refers to finger/toe nails that have a "ground glass" appearance. malnutrition. centrilobular (perivenular) areas (zone 3 of Rappaport). Variceal bleeding A. D. > 0. the earliest changes in alcoholic hepatitis are located predominantly around the C. A. All of the above In fatty liver secondary to alcohol induced liver injury. > 0. Portal hypertension 1078 Major enzyme responsible for alcohol metabolism is ? B. 2590 responsible for alcohol metabolism. there are two main pathways of alcohol metabolism. Cytochrome P-450 2E1 1080 Histologically. what is the threshold daily alcohol intake necessary to produce pathologic changes of alcoholic hepatitis ? B. Gooseberry skin B.50 associated with progressive liver injury ? Harrison’s 18th Ed. 2590 A. Macrovesicular fat hepatitis. > 0. Acetaldehyde 1081 The hallmark features of hepatocyte injury in alcoholic In liver. A. the earliest changes in alcoholic hepatitis are located predominantly in ? Genepolymorphisms may include alcohol dehydrogenase. Alcohol reductase In alcoholic hepatitis. Threshold for developing alcoholic liver disease in men is an intake of >60 . accumulation of fat within the perivenular 1085 In alcoholic hepatitis. Paper-money skin A. 2589 C. All of the above . No lunula liver disease to cirrhosis in chronic and excessive drinkers ? C. 2590 D. 2589 D. AJAY MATHUR Gastroenterology D. polymorphonuclear infiltrate. Ground glass appearance of fingernails 1076 Which of the following accelerates progression of alcoholic B. Figure 307-1 D.e. A. Alcohol dehydrogenase D. Chronic hepatitis C 1083 Skin texture of cheeks & nasolabial folds in patient with alcohol- Chronic infection with hepatitis C (HCV) is an important comorbidity in the progression of alcoholic related liver disease is called ? liver disease to cirrhosis in chronic and excessive drinkers. All of the above B. All of the above A. spotty necrosis. D. Chronic hepatitis B hyperthyroidism. which of the following can occur in the C.75 A. the AST : ALT ratio is >1. ascites. It frequently occurs in hepatic failure.25 1079 Which of the following hepatic pathologic features may be B. 2590 C. A. DM. Frequent in severe liver disease Harrison’s 18th Ed. CHF. Giant mitochondria D. and those associated with alcoholism (twin studies). 60 grams substantiating diagnosis. 10 grams D. and fibrosis in the perivenular and perisinusoidal space of C. 2589 C. Alcohol oxidase cirrhosis. Autoimmune response B. >1 B.80 gm/day of alcohol for 1082 Which of the following is false about Terry's nails ? 10 years. Nathan B. 2591 A.361:1279-90 1088 Which of the following in ultrasonography indicates serious A. Prothrombin time liver injury with less potential for complete reversal ? B. 2002 C.6 x PT prolongation + total S. 2005 D. Colchicine D. Maddrey D. and TLC. > 5500 / µL The nonspecific TNF inhibitor. 1092 Formula for assessment of prognosis of alcoholic hepatitis is ? 1098 Use of Infliximab in severe alcoholic hepatitis was stopped A. C. 479 Gastroenterology MCQ’s MCQ’s FOR FORPROFESSIONALS MEDICAL MEDICAL PROFESSIONALS BY PROF. Proton pump inhibitors D. C. Portal vein flow reversal C. All of the above 1095 Model for End-Stage Liver Disease (MELD) score was 1089 Discriminant function (DF) formula predicting the outcome of introduced in USA in year ? severe alcoholic hepatitis is named after ? N Engl J Med 2009. AJAY MATHUR Cardiology 479 1086 In alcoholic hepatitis. BUN B. for 4 weeks. It is calculated as 4. 1995 A. > 3500 / µL D. AST & ALT are usually elevated 2-7 fold. PT. Age 1087 Which of the following is a laboratory feature alcoholic fatty B. (GAHS) are all except ? A. Infliximab C.6 x PT prolongation + total S. bilirubin in mg/dL B. 2590 A. Hypertriglyceridemia. bilirubin in mg/dL A. 32 mg/day. Table 307-2 A. 100 IU/L Besides the above ones and also the classical Child-Turcotte-Pugh (CTP) score. Glasgow alcoholic hepatitis score (GAHS) Harrison’s 18th Ed. 1090 Which of the following is the correct formula of Discriminant function (DF) ? 1096 Which of the following is advocated for severe alcoholic Harrison’s 18th Ed. Ascites D. 1971 A. &/or hepatic encephalopathy has been used as a threshold to consider corticosteroid treatment. > 1500 / µL C. bilirubin in mg/dL D. Asymmetric B. BUN. 2590 hepatitis ? Harrison’s 18th Ed. Intraabdominal collaterals MELD score is based on a patient’s prothrombin time. of prothrombin time and serum creatinine to evaluate pretransplantation renal function was introduced as an aid to organ allocation among candidates for liver transplantation. 400 IU/L 1093 Variables included in Glasgow alcoholic hepatitis score In alcoholic hepatitis. serum creatinine & bilirubin. Women with encephalopathy in particular. 2590 D. Serum bilirubin C. MELD score derived from measurements of serum bilirubin. should be given outcome with one month mortality > 50% if only supportive treatment is given. followed by a steroid taper. D. international normalized ratio The discriminant function (DF) formula of Maddrey is based on PT and bilirubin. 2590 A. Hypercholesterolemia D. Glucocorticoids B.6 x [prothombin time . 3. Cushin C. Pentoxifylline A. Model for end-stage liver disease (MELD) score . bilirubin. 1998 B. Combined clinical & laboratory index of University of Toronto due to the increased risk of ? B. A value >32 implies poor Patients with severe alcoholic hepatitis. All of the above Modified Maddrey's discriminant function predicts prognosis in alcoholic hepatitis. All of the above A.control value (seconds)] + serum bilirubin (mg/dl).6 x PT prolongation + total S. All of the above 1094 MELD score is calculated based on all except ? N Engl J Med 2009. Five variables included in GAHS are age. Serum creatinine B. 2591 Harrison’s 18th Ed. Hemoglobin liver ? C. George In 2002. Patients with severe alcoholic hepatitis are defined as a discriminant function >32 or MELD >20. Increased gamma-glutamyl transpeptidase (GGTP) D. or prednisolone. 200 IU/L dimethylarginine (ADMA) score is the most recently proposed predictor of adverse clinical outcome in patients with severe alcoholic hepatitis. 2590. Propylthiouracil B. Bilirubin Harrison’s 18th Ed. > 500 / µL B. pentoxifylline improves survival in severe alcoholic hepatitis was primarily due to a decrease in the development of hepatorenal syndrome. Thiamine C. 300 IU/L D. Serum albumin Harrison’s 18th Ed. 2. 5.361:1279-90 Harrison’s 17th Ed. They are rarely >400 IU/L. 4. bilirubin in mg/dL C. 40 mg/day. Cut off value of 32 prednisone.6 x PT prolongation + total S. 1097 Use of which of the following improves survival in severe 1091 What count of polymorphonuclear cells predicts severe alcoholic hepatitis ? alcoholic hepatitis when discriminant function >32 ? Harrison’s 18th Ed. Glasgow Alcoholic Hepatitis score >= 9 requires treatment. AST and ALT are rarely more than ? C. Pericellular A. D. Destruction of the supporting reticulin network D. Lymphoid tissue 1099 Most liver transplantation centers require alcoholics to have C. D. Activation of the CD 8+ cells In alcoholic cirrhosis. 9 months A. resulting in a mixed micronodular and macronodular cirrhosis. 2592 B. Hemochromatosis Upon activation by factors released by hepatocytes and Kupffer cells. Activation of Kupffer cells alcohol use. Fibril-forming type IV collagen 1106 In cirrhosis liver. B. 2592 C. Periportal B. or periportal. Bone marrow suppression formation of increased amounts of collagen and other components of the extracellular matrix. All of the above Upon successful treatment of chronic hepatitis C. D. Extensive fibrosis D. Infection into ? D. larger nodules may form. < 1 mm Harrison’s 18th Ed. All of the above . Elastic tissue deaths secondary to infection and renal failure.5 mm 1103 Central event leading to hepatic fibrosis in cirrhosis liver is ? B. Alcoholic hepatitis B. Distortion of the vascular bed Chronic alcohol use can produce fibrosis in the absence of accompanying inflammation and/or necrosis. Alcoholic fatty liver A. Insulin Harrison’s 18th Ed. hemochromatosis and alcoholic liver disease liver 1107 Alcohol induced liver injury refers to ? fibrosis can be reversed. Transforming growth factor  (TGF-) A. Erythropoietin B. Centrilobular 1102 The pathologic features of cirrhosis liver result from ? Harrison’s 18th Ed. < 2 mm A. Activation of the hepatic stellate cell D. 1858 C. < 3 mm B. B.480 MCQ’s FOR MEDICAL PROFESSIONALS BY PROF. Fibrosis can be centrilobular. All of the above The cardinal pathologic features reflect irreversible chronic injury of the hepatic parenchyma and 1108 Hepatic fibrosis secondary to chronic alcohol use is ? include extensive fibrosis in association with the formation of regenerative nodules. collapse of the supporting reticulin network 1109 The diameter of nodules in alcoholic cirrhosis is ? with subsequent connective tissue deposition. 2592 A. Jaundice Harrison’s 16th Ed. D. under the influence of cytokines like transforming growth factor  (TGF-). the stellate cell produces ? B. Fibril-forming type II collagen Most transplantation centers currently require patients with a history of alcohol abuse to have documented C. 6 months Harrison’s 16th Ed. Fibril-forming type III collagen abstinence of at least 6 months before undergoing transplantation (“6-month abstinence rule”). Irreversible chronic injury of hepatic parenchyma C. < 0.Cirrhosis cells to produce collagen ? Harrison’s 16th Ed. nodules are usually <3 mm in diameter (micronodular). 1858 1100 Reversal of fibrosis in cirrhosis liver can be achieved with A. the stellate cell assumes a C. AJAY MATHUR Gastroenterology Harrison’s 18th Ed. and nodular regeneration Harrison’s 18th Ed. TNF  treatment in which of the following diseases ? B. 2592 B. Chronic hepatitis C D. Hepatocytes A. 3 months 1105 In cirrhosis liver. activated hepatic stellate cells transform C. Alcoholic liver disease myofibroblast-like conformation and. Fibril-forming type I collagen D. which of the following leads hepatic stellate 308 . Myofibroblasts documented abstinence of at least ? D. D. 2591 The central event leading to hepatic fibrosis is activation of the hepatic stellate cell resulting in the A. 2592 A. 2592 C. Harrison’s 18th Ed. All of the above C. distortion of the vascular bed. 1858 Use of Infliximab in severe alcoholic hepatitis was stopped due to the increased risk of increased A. alcoholic hepatitis and alcoholic cirrhosis. Hepatocyte necrosis C. Harrison’s 18th Ed. produces fibril-forming type I collagen. pericellular. Any of the above C. Seizure 1104 In cirrhosis liver. A. Formation of regenerative nodules Alcohol-induced liver injury includes consequences resulting from chronic alcohol ingestion like alcoholic fatty liver. With cessation of C. Cirrhosis liver 1101 The cardinal pathologic features of cirrhosis liver are ? Harrison’s 18th Ed. 12 months B. 2592 of remaining liver parenchyma. All of the above The pathologic features result from hepatocyte necrosis. hyperlipoproteinaemia. 2592 C. clinical findings include ? Harrison’s 18th Ed. Gastroesophageal varices and splenomegaly are due to portal hypertension. Spider angiomas 1111 Which of the following initiates alcohol metabolism ? Harrison’s 18th Ed. A. Johnson’s cirrhosis C. Hepatic encephalopathy that lead to hepatocyte damage like interference with specific enzyme activities (microtubular formation & hepatic protein trafficking). Gastric alcohol dehydrogenase (ADH) diminished hepatic clearance of the precursor androstenedione leads to gynecomastia. Coagulopathy Ethanol oxidation occurs via ADH to form acetaldehyde which is metabolized to acetate by aldehyde dehydrogenase (ALDH). Cytosolic alcohol dehydrogenase (ADH) D. spider angiomas. Cytosolic alcohol dehydrogenase (ADH) Harrison’s 18th Ed. 2593 C. 2593 cirrhosis encountered in North America. Microsomal ethanol oxidizing system (MEOS) A. the microsomal- D. Spider angiomas B. and peroxisomal catalase. Ascites & hepatic encephalopathy result from both hepatocellular insufficiency & portal hypertension. palmar erythema. and certain metabolic abnormalities are due to loss of functioning are produced that initiate and perpetuate stellate cell activation. Peroxisomal catalase B. parotid and lacrimal gland enlargement. 2592 1120 Which of the following presentation result from both A. 2592 1118 Zieve’s syndrome in alcoholics is best related to ? A. As a result. Dupuytren’s contractures B. 1112 Enzyme system for metabolism of alcohol in the liver is ? Harrison’s 18th Ed. Kupffer cell activation. Splenomegaly 1114 Which out of the following is a highly reactive molecule ? Harrison’s 18th Ed. Clubbing of fingers . Signs of virilization in women A. These are cytosolic ADH. 481 Gastroenterology MCQ’s MCQ’s FOR FORPROFESSIONALS MEDICAL MEDICAL PROFESSIONALS BY PROF. signs of virilization or menstrual irregularities may occur. Lacrimal gland enlargement C. All of the above C. 2592 A. Jaundice Majority of ethanol oxidation occurs via ADH to form acetaldehyde. and C. 2593 B. clubbing of fingers. Xanthoma B. Gilbert’s cirrhosis D. jaundice. All of the above D. Gaucher’s cirrhosis B. Harrison’s 18th Ed. Microsomal-oxidizing system (MEOS) Harrison’s 18th Ed. historically referred to as Laennec’s cirrhosis is the most common type of Harrison’s 18th Ed. Palmar erythema A. Peroxisomal catalase ascites with or without peripheral edema. testicular atrophy and decreased body hair in men. Hemolytic anemia Three enzyme systems metabolise alcohol in liver. 2593 Ethanol is mainly absorbed by small intestine and to a lesser degree through stomach. with the resultant production of hepatocellular mass. profibrogenic cytokines Jaundice. Diarrhoea C. Stomach D. Laennec’s cirrhosis 1116 In alcoholic cirrhosis. Myocardial infarction D. Acetaldehyde Harrison’s 18th Ed. excess collagen and extracellular matrix. splenomegaly. 2593 C. All of the above B. Coagulopathy D. ZS is an acute metabolic condition that can occur during withdrawal from prolonged alcohol abuse. Acetate A. Palmar erythema B. muscle wasting. The underlying cause is liver delipidization. Cytosolic alcohol dehydrogenase (ADH) 1119 Which of the following presentation result from both hepatocellular insufficiency and portal hypertension ? B. Pneumonia oxidizing system (MEOS). All of the above Harrison’s 18th Ed. Small intestine C. coagulopathy. C. In women. 1972 A. clinical findings include all except ? Alcoholic cirrhosis. A. Microsomal-oxidizing system (MEOS) Frequent findings in alcoholic cirrhosis include jaundice. It consists of hemolytic anemia (spur cells and acanthocytes). Peroxisomal catalase A. 2592 C. Gastric alcohol dehydrogenase (ADH) B. flexion contracture of digits are associated with alcoholism but are not specifically related to cirrhosis. 2593 1121 Alcoholic cirrhosis is also called ? Harrison’s 17th Ed. Jaundice C. Large intestine 1117 Which of the following findings are associated with alcoholism but are not specifically related to cirrhosis ? D. and 1113 Ethanol is oxidized to acetaldehyde by ? abdominal pain. Alcohol hepatocellular insufficiency and portal hypertension ? B. edema. western Europe and South America. Increased peripheral formation of estrogen due to D. Ascites D. Muscle wasting Harrison’s 18th Ed. 1115 In alcoholic cirrhosis. AJAY MATHUR Cardiology 481 1110 Ethanol is mainly absorbed by ? B. Dupuytren’s contractures resulting from fibrosis of the palmar fascia with resulting Gastric alcohol dehydrogenase (ADH) initiates alcohol metabolism. Splenomegaly D. Acetaldehyde combines with proteins to form protein-acetaldehyde adducts D. Gynecomastia A. > 0.482 MCQ’s FOR MEDICAL PROFESSIONALS BY PROF. < 400 units C. Direct suppressive effect of alcohol on bone marrow C. fairly “uniform” loss of liver cells. < 300 units B. < 500 units D. but levels > 300 units are unusual. The cell and should prompt one to look for other coincident or complicating factors. 1972 1130 Levels of serum AST in alcoholic cirrhosis are ? Harrison’s 18th Ed. Hypoalbuminemia reflects impairment in hepatic protein synthesis. Psychogenic factors B. which regenerate and form nodules. . acquires a nodular appearance. A. & suppressive effect of alcohol on bone marrow. Deficiency of iron D. Gall bladder stones A. or decreased in size. Hyperglobulinemia due to stimulation of RE system Harrison’s 18th Ed.5 Harrison’s 18th Ed. fibroblasts. Greater clearance of ALT by ethanol A. < 600 units With continued alcohol intake and destruction of hepatocytes. AST levels > ALT are due to ? Harrison’s 18th Ed. Uniform A. Patchy B. loss generally exceeds replacement. Decreased ethanol which may be partially reversed by pyridoxal phosphate. Hyperbilirubinemia C. > 2. Greater production of AST by ethanol 1126 In alcoholic cirrhosis. 1131 In alcoholic liver disease. Coexistent folic acid & vitamin B 12 deficiency D. With continuing hepatocyte destruction and collagen deposition. D. All of the above 1133 Altered albumin/globulin ratio in alcoholic cirrhosis is due In alcoholic cirrhosis. > 1. while serum globulins are increased. Normal In alcoholic liver disease & in contrast to viral hepatitis. with elevated serum alkaline phosphatase levels. deficiency. Harrison’s 18th Ed. None of the above C. Chronic hepatitis B D. 2593 accelerates development of alcoholic cirrhosis ? A. AJAY MATHUR Gastroenterology 1122 Loss of liver cells in alcoholic cirrhosis is ? 1128 Anemia in alcoholic cirrhosis may be due to ? Harrison’s 17th Ed.0 B. Serum AST levels > 300 units Micronodular cirrhosis may result following jejunoileal bypass & thus alcoholic cirrhosis & Hemolytic anemia due to effects of hypercholesterolemia or erythrocyte membranes resulting in micronodular cirrhosis are not synonymous. the liver shrinks in size. All of the above stimulation of reticuloendothelial system. Following jejunoileal bypass B.0 D. coexistent folic acid & vitamin B12 small regenerative nodules. liver may be either enlarged. Levels of serum AST are frequently elevated. 2593 A. hypersplenism. 1124 In cirrhosis liver. Chronic hepatitis C 1132 In alcoholic liver disease. 2593 A. 2593 Concomitant chronic hepatitis C virus (HCV) infection significantly accelerates development of alcoholic cirrhosis. Acute hepatitis B C. 2593 A. Any of the above D. Exceeds replacement B. Alcoholic cirrhosis may progress to macronodular unusual spurlike projections (acanthocytosis) may occur. Lesser clearance of AST by ethanol B. cell loss generally ? Harrison’s 17th Ed. Hyperbilirubinemia is found in association cirrhosis with time. A+B B. which concomitant hepatitis infection Harrison’s 18th Ed. Acute hepatitis A B. Hypoalbuminemia due to impaired hepatic protein synthesis 1127 Anemia in alcoholic cirrhosis may be due to ? B. 1123 Micronodular cirrhosis result from ? 1129 Laboratory finding unusual in alcoholic cirrhosis is ? Harrison’s 17th Ed. and Anemia results from acute & chronic gastrointestinal blood loss. All of the above D. Chronic pancreatitis D. Acute/chronic GI blood loss C. Hemolytic anemia B. serum AST is usually disproportio-nately elevated relative to ALT (AST/ALT ratio >2) due to proportionally greater inhibition of ALT synthesis by C. 2593 A. Elevated serum alkaline phosphatase D. liver size may be ? Harrison’s 17th Ed. Greater inhibition of ALT synthesis by ethanol B. All of the above Alcoholic cirrhosis is characterized by diffuse fine scarring. 1972 C. Lobe specific C.5 C. Equals replacement C. Hypersplenism The serum albumin level is usually depressed. AST / ALT ratio is ? 1125 In alcoholics. normal. while hyperglobulinemia result from nonspecific D. to ? and becomes hard as ‘end-stage’ cirrhosis develops. activated hepatic stellate cells and myofibroblasts appear at the site of injury and deposit collagen forming septa in periportal In alcoholic cirrhosis. > 1. 1972 Harrison’s 18th Ed. Lags replacement A. levels of serum AST are frequently elevated but levels > 300 units are unusual and pericentral zones surrounding remaining liver cells. 2593 A. Following gastrectomy C. 1972 Harrison’s 18th Ed. Enlarged D. and is called micronodular cirrhosis. 2593 A. EBV. copper deposition. autoimmune cholangitis. aspirin. All of the above When clinical status of an otherwise stable cirrhotic patient deteriorates without an obvious 1143 Histopathologic features of chronic cholestasis are all except ? explanation. 2 C. Acamprosate helps restore balance of excitatory & inhibitory B. D. C. Autoimmune cholangitis A. Dilutional hyponatremia D. The term cryptogenic cirrhosis has been used interchangeably with posthepatitic cirrhosis. A. 2593 A. 1 adenosyl methionine decreases proinflammatory cytokines and has survival benefit in alcoholic cirrhosis. syndrome ? Harrison’s 17th Ed. Primary biliary cirrhosis (PBC) an otherwise stable cirrhotic patient include ? Harrison’s 17th Ed. therefore other causes & types of liver disease should be considered. Prerenal azotemia 1140 Which of the following terms are synonymous with posthepatitic cirrhosis ? 1135 Metabolic disturbances seen in alcoholic cirrhosis are all except ? Harrison’s 17th Ed. C. Acamprosate except ? C. sedatives. Hepatocellular carcinoma Causes of chronic cholestatic syndromes are primary biliary cirrhosis (PBC). hypokalemia may occur from increased urinary potassium losses due in part to hyperaldosteronism. Hepatitis C C. aspirin. 1973 Harrison’s 18th Ed. Central hyperventilation leads to respiratory alkalosis. 1860t Harrison’s 18th Ed. clinical diabetes is uncommon. 1974 1137 Complicating conditions that can deteriorate clinical status of A. 4 alcoholism by reducing craving ? A. 1972 B. Tiapride Harrison’s 18th Ed. 2593 D. Infection C. acetaminophen should be used with caution. There may also be chronic portal inflammation. 30 to 35 % Viral infections that lead to chronic liver disease include hepatitis B. Iron deposition A. In patients (~10% of all patients with cirrhosis). . Portal vein thrombosis D. Only 10 to 15% of individuals with excessive 1142 Which of the following is a cause of chronic cholestatic alcohol intake develop cirrhosis. Survival benefit has been reported for S-adenosyl methionine in transformation of hepatocytes. Dietary but this designation should be reserved for those cases in which the etiology of cirrhosis is unknown deficiency and increased urinary losses lead to hypomagnesemia and hypophosphatemia. portal vein thrombosis & hepatocellular carcinoma. steatosis is often present. Cryptogenic cirrhosis C. glucose intolerance due to endogenous insulin resistance may be present. Metabolic alkalosis activating GABA-A receptors. Multilobular cirrhosis B. Diuretics. D. 1973 C. Hyperphosphatemia C. Hepatitis E D. sedatives. B. 1141 Viral infections that can lead to chronic liver disease include 1136 What percentage of individuals with excessive alcohol intake all except ? develop cirrhosis ? Harrison’s 16th Ed. 2593 A. Glucocorticoids are helpful in severe alcoholic hepatitis & D. A. Epstein-Barr virus B. 5 to 10 % B. Copper deposition 1138 Which of the following about alcoholic cirrhosis is true ? B. All of the above D. 2594 Glucocorticoids in moderately large doses for 4 weeks is helpful in patients with severe alcoholic hepatitis and encephalopathy but have no role in the treatment of established alcoholic cirrhosis. Hypomagnesemia B. Diuretics. Respiratory alkalosis neurotransmission in nucleus accumbens by blocking GABA receptors and Glutamate receptors and C. D. Primary sclerosing cholangitis (PSC) B. Prerenal azotemia is also observed in such patients. 10 to 15 % C. 1974 should be looked for. 483 Gastroenterology MCQ’s MCQ’s FOR FORPROFESSIONALS MEDICAL MEDICAL PROFESSIONALS BY PROF. S. and biliary fibrosis. however cirrhosis. Ductopenia is a result of this progressive disease as patients develop cirrhosis. All of the above A. primary sclerosing cholangitis (PSC) and idiopathic adulthood ductopenia. Naltrexone is an opioid antagonist. Naltrexone In patients with HCV genotype 3. Glucose intolerance Tiapride is a dopamine antagonist. Biliary fibrosis encephalopathy Histopathologic features of chronic cholestasis are cholate stasis. Cytomegalovirus A. alcoholic cirrhosis interface activity and chronic lobular inflammation. All of the above genotype ? Harrison’s 18th Ed. Alcoholic cirrhosis should be strongly suspected in patients with a history of prolonged or excessive alcohol intake and physical signs of chronic liver disease. Harrison’s 17th Ed. complicating conditions like infection. 20 to 25 % D. cytomegalovirus. with ascites and dilutional hyponatremia. All of the above C. acetaminophen should be used with caution 1144 Steatosis is often present in patients with which HCV D. 3 1139 Which of the following medications is approved for treating D. Hypokalemia Coarsely nodular cirrhosis and multilobular cirrhosis are terms synonymous with posthepatitic In cirrhosis. xanthomatous B. AJAY MATHUR Cardiology 483 1134 Metabolic disturbances seen in alcoholic cirrhosis are all B. Xanthomatous transformation of hepatocytes Harrison’s 17th Ed. Coarsely nodular cirrhosis A. Hypopigmentation 1154 Coexisting autoimmune disease in primary biliary cirrhosis is ? B. . Fibroadenoma breast In PBC. Peak incidence is in fifth decade of life PBC is frequently associated with a variety of autoimmune disorders. 2595 Harrison’s 16th Ed. Osteomalacia occurs due to diminished vitamin D absorption. Pernicious anemia three are related to the altered cholesterol metabolism seen in PBC.353:1261-73 Harrison’s 17th Ed. hyperpigmentation is evident on ? Harrison’s 18th Ed. Areas of exfoliation and lichenification C. All of the above D. Bone pain B. Protracted elevation of serum lipids. 2595 osteoporosis is common. Autoimmune thyroid disease C. xanthomata & bone pain. AJAY MATHUR Gastroenterology 1145 Nonalcoholic steatohepatitis (NASH) is nowadays diagnosed 1151 Which of the following about primary biliary cirrhosis is true ? as what was earlier diagnosed as ? N Engl J Med 2005. Antimitochondrial antibodies are present in ~90% A. The first C. 1152 Which of the following about primary biliary cirrhosis is true ? 1146 Primary biliary cirrhosis (PBC) is characterized by ? N Engl J Med 2005. Autoantibodies recognize three to five inner mitochondrial D. CREST syndrome C. Pruritus is usually worse at night and is exacerbated by contact C. Cardiac cirrhosis clinical signs appear C. Clinical evidence of sicca syndrome is found in about 75%. Slowly progressive autoimmune disease of liver C.484 MCQ’s FOR MEDICAL PROFESSIONALS BY PROF. pruritus is most bothersome in ? except ? Harrison’s 18th Ed. type 1 diabetes mellitus. and serologic evidence of autoimmune thyroid disease in 1148 Features unique to PBC include all except ? 25% of patients.353:1261-73 A. All of the above In PBC. All of the above 1149 In PBC. such as syndrome of calcinosis. Antimitochondrial antibodies are detectable years before B. Type I diabetes mellitus D. 90% are women between age 35 to 60 years. Xanthelasma Harrison’s 16th Ed. 1861 A. Fatigue is a prominent early symptom. Fibrous obliteration of intrahepatic bile ductules and larger with wool. leads to subcutaneous lipid deposition around the eyes (xanthelasmas) and over joints and tendons (xanthomas). or heat extrahepatic ducts both D.353:1261-73 Harrison’s 18th Ed. D. Renal tubular acidosis D. Fatigue & pruritus are the commonest presenting symptoms A. All of the above steatohepatitis. Fibrous obliteration of larger extrahepatic ducts C. 1153 Associated findings in primary biliary cirrhosis include all 1147 In PBC. Keratoconjunctivitis sicca N Engl J Med 2005. CREST syndrome C. which may be either generalized or limited initially to palms and soles. sclerodactyly. Hyperlipidemia B. 1861 C. autoimmune thyroiditis. Primarily affects men D. Osteomalacia D. hyperpigmentation is evident on trunk and arms and in areas of exfoliation and lichenification. Fibrous obliteration of intrahepatic bile ductules B. Rheumatoid arthritis B. Face B. xanthelasma. Afternoon B. Pruritus precedes onset of jaundice by months to years B. pruritus is most bothersome in the evening. Accelerated Harrison’s 18th Ed. especially cholesterol. 2594 A. Trunk A. 1861 A. pernicious anemia. All of the above D. Morning A. Among patients with symptomatic disease. IgA deficiency B. Cryptogenic cirrhosis C. rheumatoid arthritis. All of the above D. Night D. Most prevalent in northern Europe B. IgA deficiency. and renal tubular acidosis. telangiectasia (CREST). Xanthomata A. Autoimmune hepatitis membrane proteins Many patients who were thought to have cryptogenic cirrhosis in fact have nonalcoholic D. 1861 1150 Which of the following about primary biliary cirrhosis is false ? A. A. 2595 1155 Coexisting autoimmune disease in primary biliary cirrhosis is ? Harrison’s 16th Ed. PBC is characterized by portal inflammation & necrosis of cholangiocytes in small and medium- sized bile ducts. Autoimmune cholangiopathy B. other fabrics. esophageal dysmotility. Evening C. 1974 A. None of the above In PBC. Scleroderma Features unique to PBC include hyperpigmentation. keratoconjunctivitis sicca syndrome (dry eyes and dry mouth). 1156 Coexisting autoimmune disease in primary biliary cirrhosis is ? Harrison’s 16th Ed. the earliest symptom is pruritus. scleroderma. Raynaud’s phenomenon. Other AMA autoantibodies in PBC patients are directed to similar constituents of BCOADC and OGDC and also inhibit their enzymatic function. IgE B. B. 1861 which element should be supplemented ? Harrison’s 16th Ed. Rifampin D. Zinc C. Methotrexate 1160 T cells infiltrating the liver in primary biliary cirrhosis are D. All of the above A. D. Colchicine oxoacid dehydrogenase complex (BCOADC). IgM all except ? Harrison’s 16th Ed. C. All of the above and inhibit the overall enzymatic activity of the PDC. Pyruvate dehydrogenase E2 complex (PDC-E2) 1166 The only established “cure” in the treatment of primary biliary B. 1860 A. IgA A. 1861 D. Should be given with food Antinuclear or smooth-muscle antibodies are present. histological features are similar to PBC. In PBC. Liver transplantation D. fat-soluble vitamins A. branched chain 2. Ursodiol therapy may not prevent ultimate progression of PBC and the only established ‘cure’ is liver transplantation. The AMA titre is negative. Copper B. Cholestyramine. plasmapheresis. B. Serum LDLc C. and 2-oxoglutarate dehydrogenase complex (OGDC). Branched-chain 2-oxo-acid dehydrogenase E2 complex (BCKD- E2) B. Zinc supplementation 1162 Asymptomatic patients of PBC are initially detected by ? may be necessary if night blindness is refractory to vitamin A therapy. Elevated serum alkaline phosphatase levels PBC ? B. B. Branched-chain 2-oxoacid dehydrogenase complex primary biliary cirrhosis ? Harrison’s 16th Ed. As a single dose daily 1158 Antimitochondrial antibody (AMA) found in primary biliary D. Ketoglutaric acid dehydrogenase E2 complex (OGDC-E2) A. azathioprine. Harrison’s 16th Ed. C. 1860 sequestering resin. Harrison’s 16th Ed. All of the above Harrison’s 16th Ed.353:1261-73 A. opiate antagonists (naloxone or naltrexone). Serum VLDLc D. Long term Tacrolimus therapy The major autoantigen in PBC (90%) is 74-kDa E2 component of PDC. hyperlipidemia is common with a striking increase of serum unesterified cholesterol. 1861 D. Ondansetron A circulating IgG antimitochondrial antibody (AMA) is detected in ~90% of patients with PBC and only rarely in other forms of liver disease. and K should be given at regular intervals. IgG 1164 Drugs used to treat pruritus in primary biliary cirrhosis include B. ondansetron. B. Negative AMA 1163 Which of the following is false about ursodiol therapy in PBC ? C. Long term Cyclosporine therapy C. 1861 C. Serum triglycerides B. methotrexate. E3-binding protein (E3-BP) cirrhosis is ? Harrison’s 16th Ed. C. when night blindness is refractory to vitamin A therapy. 1161 Hyperlipidemia seen in primary biliary cirrhosis shows a characteristic rise in ? 1167 In PBC. and ultraviolet light have been tried for control of pruritus with varying results. 1860 1168 Which of the following represent the pathological Stage I of A. 1861 C. All of the above A. 1861 A. N Engl J Med 2005. 485 Gastroenterology MCQ’s MCQ’s FOR FORPROFESSIONALS MEDICAL MEDICAL PROFESSIONALS BY PROF. Ursodeoxycholic acid In PBC. colchicine. 2-oxoglutarate dehydrogenase complex (OGDC) 1165 Which of the following drugs is not used in the treatment of C. Antibodies are directed to a region essential for binding of a lipoic acid cofactor D. Histological features similar to PBC alkaline phosphatase levels during routine screening. AJAY MATHUR Cardiology 485 1157 Which of the following is true for “autoimmune cholangitis” ? D. circulating IgG antimitochondrial autoantibodies (AMA) recognize inner mitochondrial membrane proteins identified as enzymes of the pyruvate dehydrogenase complex (PDC). 1861 C. dihydrolipoamide acetyltransferase. an oral bile salt Harrison’s 16th Ed. 1861 Most patients with PBC are asymptomatic. Selenium In PBC. E. Pyruvate dehydrogenase complex (PDC) hypercholesterolemia. Cobalt D. Elevated AST levels Harrison’s 16th Ed. Serum unesterified cholesterol A. Tetracycline 1159 Which of the following is an autoreactive mitochondrial antigen in primary biliary cirrhosis ? Rifampin. Imatinib mesylate specific for ? Glucocorticoids. Elevated ALT levels A. Antinuclear or smooth-muscle antibodies present Harrison’s 16th Ed. Dose is 13 to 15 mg/kg per day In autoimmune cholangitis. and the disease is initially detected by elevated serum A. None of the above cirrhosis is of which type of immunoglobulin ? Ursodiol is given in doses of 13 to 15 mg/kg per day. Chronic nonsuppurative destructive cholangitis . Cholestyramine D. may be helpful in doses of 12 to 16 gm/day to decrease both pruritus and A. Harrison’s 16th Ed. cyclosporine & tacrolimus are effective. with food and as a single dose daily. Echinococcosis. A. Echinococcosis tissue scars marking stage III. In adults. Biliary atresia. Amioradone B. Lyme’s disease and fibrous obliteration of intrahepatic bile ductules. destruction of hepatic parenchyma. amioradone. At least 3 to 12 months C. Reduction in number of bile ducts and proliferation of smaller D. Hereditary fructose intolerance. Toxoplasmosis. Capillariasis. Primary sclerosing cholangitis Harrison’s 16th Ed. oral contraceptives At least 3 to 12 months is required for biliary obstruction to result in finely nodular secondary biliary (Budd-Chiari). arsenicals. 1860t extrahepatic ducts both A. At least 18 to 36 months Drugs & toxins that can lead to CLD include alcohol. loss of liver cells & expansion of periportal fibrosis into a network of connective C. congenital biliary atresia & cystic fibrosis are common causes of SBC. and progressive fibrosis. None of the above B. It is associated with impaired biliary excretion. 1860t Harrison’s 16th Ed. 1860t A. 1177 Drugs & toxins that can lead to chronic liver disease include 1171 What duration of biliary obstruction is required to result in all except ? secondary biliary cirrhosis (SBC) ? Harrison’s 16th Ed. mild fibrosis & A. Postoperative CBD strictures bile ductules In children. 1860t B. 1860t Harrison’s 16th Ed. D. 1861 A. Renal tubular acidosis D. Malignant tumors of common bile duct or pancreas A. Micronodular or macronodular cirrhosis except ? PBC is divided morphologically into 4 stages. Presence of Lipoprotein X B. 1178 Disorders that can lead to chronic liver disease include all 1172 In children. impaired absorption of fat soluble vitamins and hypoprothrombinemia. chronic pancreatitis or primary sclerosing cholangitis. Cystic fibrosis C. Hemochromatosis C. number of bile ducts are reduced & smaller bile ductules proliferate. Hereditary tyrosinemia. Deficiency of bile salts in intestine leads to moderate steatorrhea and Fanconi’s syndrome. values rarely exceed 150 to 200 units. PBC. Stage I is termed chronic nonsuppurative destructive Harrison’s 16th Ed. 1-antitrypsin deficiency D. Hypoprothrombinemia D. It is a necrotizing inflammatory process of portal triads characterized by destruction of medium & small bile ducts. 1860 Schistosomiasis. Over months to years there is a decrease in B. At least 1 to 3 months B. Jejunoileal bypass C. Wegener’s granulomatosis D. Fibrous obliteration of intrahepatic bile ductules and larger Harrison’s 16th Ed. a dense infiltrate of acute & chronic inflammatory cells. Fibrous obliteration of larger extrahepatic ducts disease include all except ? C. Cystic fibrosis C. gallstones. which of the following is a common cause of SBC ? except ? Harrison’s 16th Ed. Galactosemia A. Gaucher’s disease B. Chronic pancreatitis D.micronodular or macronodular. 1176 Inherited & metabolic disorders that can lead to chronic liver disease include all except ? 1170 Prolonged cholestasis can lead to all of the following except ? Harrison’s 16th Ed. Fanconi’s syndrome Biliary cirrhosis results from injury to or prolonged obstruction of either the intrahepatic or extrahepatic C. Biliary obstruction (chronic) B. Brucellosis occasionally. In stage II. Chronic pancreatitis . 1861 Harrison’s 16th Ed. 1861 A. Graft-versus-host disease 1173 Which of the following rarely causes SBC ? 1179 Disorders that can lead to chronic liver disease include all Harrison’s 16th Ed. Fibrous obliteration of intrahepatic bile ductules 1175 Inherited & metabolic disorders that can lead to chronic liver B. AJAY MATHUR Gastroenterology B. pyrrolidizine alkaloids & antineoplastic agents. loss of liver cells. bile stasis. biliary C. Primary biliary cirrhosis (PBC) is characterized by chronic inflammation D. Alagille’s syndrome. though not specific. Wilson’s disease biliary system. Primary sclerosing cholangitis A. tissue scars 1174 Infections that can lead to chronic liver disease include all D. Patients with malignant tumors of CBD or pancreas rarely survive expansion of periportal fibrosis into a network of connective long enough to develop SBC. Leptospirosis 1169 Secondary biliary cirrhosis (SBC) is characterized by ? Infections that can lead to chronic liver disease include Brucellosis. Elevated liver copper levels C. Oral contraceptives A. Arsenicals D. Decrease in interlobular ducts. Serum aminotransferase > 300 Inherited and metabolic disorders that can lead to chronic liver disease include  1-Antitrypsin An abnormal serum lipoprotein (lipoprotein X) and elevated liver copper levels may be present in deficiency. 1861 except ? A. At least 12 to 18 months D. Galactosemia. Gallstones B. 1860t cholangitis. Testosterone C. and tract obstruction is mostly caused by postoperative strictures. inflammatory infiltrate becomes less prominent. Stage IV represents cirrhosis .486 MCQ’s FOR MEDICAL PROFESSIONALS BY PROF. Gaucher’s disease. Glycogen storage disease. Toxoplasmosis interlobular ducts. Wilson’s disease. Harrison’s 16th Ed. Familial intrahepatic cholestasis (FIC) types 1-3. Secondary biliary cirrhosis (SBC) is the result of longstanding obstruction of the larger extrahepatic ducts. cirrhosis. Serum aminotransferase Hemochromatosis. C. Serum bilirubin concentration Over 50% of patients with PSC also have ulcerative colitis (UC). Hodgekin’s lymphoma B. Cholangitis (PSC) ? Often associated with autoimmune pancreatitis & other fibrosing conditions but not inflammatory Harrison’s 18th Ed. Liver histologic changes D. Jejunoileal bypass. Glucocorticoids 1184 Typical cholangiographic findings in PSC is ? B. Diffuse fibrotic narrowing of biliary tree documented. Riedel’s cirrhosis. once a diagnosis of PSC is established. 2627 A. Immunoglobulin G2 . A. 329. Primary Biliary Cirrhosis with ? Harrison’s 18th Ed. All of the above A. Therefore. Cardiac cirrhosis 1185 Primary or idiopathic sclerosing cholangitis may be associated B. Chronic pancreatitis 1183 Which of the following antibody is seen frequently in Primary D. Antiphospholipid antibody Harrison’s 18th Ed. Cryoglobulins jaundice. Anti-DNA antibody patients usually present with greatly elevated serum alkaline phosphatase levels (mean. Graft-versus-host disease.associated cholangitis Immunoglobulin G4–associated cholangitis is a recently described biliary disease of unknown 1181 Which of the following is seen frequently in Primary Sclerosing etiology that presents with biochemical and cholangiographic features indistinguishable from PSC. 2596 segments of normal or dilated ducts involving both the intrahepatic and extrahepatic biliary tree. 1180 Which of the following about Primary Sclerosing Cholangitis 1186 Which of the following has biochemical & cholangiographic (PSC) is false ? features indistinguishable from PSC ? Harrison’s 18th Ed. 2627 C. Amenorrhoea B. Ulcerative colitis (UC) 1187 Independent predictor of a bad prognosis in PSC is ? Harrison’s 18th Ed. 2596 Harrison’s 18th Ed. Sarcoidosis B.associated cholangitis D. liver histologic changes and splenomegaly. Multiple calculi in biliary tree D. Nonalcoholic fatty liver disease.associated cholangitis B. UDCA B. colonoscopy should be performed to look for evidence of UC. C. Primary or idiopathic sclerosing cholangitis may be associated with autoimmune pancreatitis. which has a cholangiographic appearance similar to that of PSC. AJAY MATHUR Cardiology 487 B. Ductopenia 1188 Which of the following has cholangiographic appearance C. Traumatic biliary injury C. Multifocal stricturing & beading of biliary tree Glucocorticoids. serum bilirubin concentration. Perinuclear antineutrophil cytoplasmic antibody (p-ANCA) 1189 Which of the following is efficacious in PSC ? D. A. Immunoglobulin G3 . Azoospermia A. Fibrous cholangitis (pericholangitis) similar to that of PSC ? Harrison’s 18th Ed. Cyclosporine A. Porphyria D. usually due to infection of the bile duct epithelium with CMV or cryptosporidia. Autoimmune pancreatitis D. methotrexate. Obliteration of extrahepatic biliary tree C. Bile duct proliferation B. Primary biliary cirrhosis C. 800 IU/L). 2596 bowel disease. but the bilirubin is often near normal. Cholangiocarcinoma is a dreaded consequence. Immunoglobulin G4 . None of the above D. Primary biliary multifocal fibrosclerosis syndromes (retroperitoneal. Sarcoidosis. Immunoglobulin G1 . 2627 Perinuclear antineutrophil cytoplasmic antibody (p-ANCA). and cyclosporine have not been shown to be efficacious in PSC. 2627 C. AIDS cholangiopathy Pathologic changes that can occur in PSC show bile duct proliferation as well as ductopenia and fibrous cholangitis (pericholangitis). D. UDCA in high dosage (20 mg/kg) improves serum liver tests. 2596 Independent predictors of a bad prognosis in PSC are age. Riedel’s struma D. A. Methotrexate Harrison’s 18th Ed. None of the above . mediastinal. B. 2627 D. struma or pseudotumor of the orbit. and/or periureteral fibrosis).associated cholangitis C. All of the above 1190 “Nutmeg liver” is the term used to describe liver in ? Typical cholangiographic findings in PSC are multifocal stricturing and beading with intervening Harrison’s 18th Ed. Multifocal fibrosclerosis syndromes C. Glucocorticoids and/or azathioprine are helpful. 2596 C. but an effect on survival has not been C. All of the above 1182 Pathologic change occurs in PSC is ? Harrison’s 18th Ed. it is characterized by elevated serum IgG4 and infiltration of IgG4-positive plasma cells in bile ducts and liver tissue. 2596 AIDS cholangiopathy is a condition. Age D. is positive in ~65% of patients of PSC. 487 Gastroenterology MCQ’s MCQ’s FOR FORPROFESSIONALS MEDICAL MEDICAL PROFESSIONALS BY PROF. Obliteration of intrahepatic biliary tree B. Secondary Biliary Cirrhosis A. A. Primary sclerosing cholangitis. Cystic fibrosis. All of the above Sclerosing Cholangitis (PSC) ? Harrison’s 18th Ed. All of the above Disorders that can lead to chronic liver disease include Biliary obstruction (chronic). Chronic cholestatic syndrome A. These A. These patients do not typically present with B. HVPG is equal to wedged The presence of a firm. Cystic fibrosis D. D C. hepatic sinusoids become dilated & engorged with blood. K prolonged right-sided heart failure ? Harrison’s 18th Ed. cardiac cirrhosis. 2597 1194 Which of the following could lead to cardiac cirrhosis ? Harrison’s 16th Ed. Bilirubin A. 1862 Chiari syndrome.  1 antitrypsin deficiency. Enlarged Sherlock B. Nodules on the surface of liver 1197 Toxicity of which of the following vitamins can cause veno- B. Wilson’s disease C. liver is grossly enlarged. 10 . tender & severe intractable ascites is present. D. mortality associated with each episode 1195 Inherited metabolic liver disease that can progress to cirrhosis of bleeding is ? is ? Harrison’s 18th Ed. Brownish black discolouration of liver A. Jaundice D. Aminotransferases may be normal or 1200 Portal hypertension is defined as elevation of hepatic venous slightly increased with AST usually higher than ALT. along with hepatic B.30 %.40 % C. 1202 Portal vein is formed by the confluence of splenic vein with ? Chiari syndrome ? Harrison’s 18th Ed. & is usually nontender. 1196 Which of the following points against the diagnosis of Budd. Cor pulmonale of long duration (>10 years) D. 1201 In variceal hemorrhage. 2596 1198 Acute Budd-Chiari syndrome has all the features except ? A.50 % D. S.30 % B. B. or cor pulmonale of long duration (>10 years) should suggest abdominal pressure). S. 1862 A. D. > 5 mm Hg D. All of the above Mortality associated with each episode of variceal bleeding is 20 . Alternating red and pale areas occlusive disease of liver ? C. E 1192 Which of the following is false in hepatomegaly due to D. Constrictive pericarditis C. 2596 Sherlock A. Firm A. The mortality rate with rebleeding is 33%. Ascites With prolonged right heart failure. probability of another episode is high. CT abdomen D. > 2 mm Hg A. Tender hepatomegaly B. SGOT B. USG abdomen B. Varices may develop but do not bleed if HVPG is <12 mm Hg. Rebleeding rate without disease. Valvular heart disease B. > 3 mm Hg B. Alkaline phosphatase D. Right-sided heart failure central vein. AJAY MATHUR Gastroenterology In right heart failure. SGPT C. 2597 A. treatment is 70% within 1 year. pressure gradient to ? Harrison’s 18th Ed. Hemochromatosis B. Abdominal pain C. > 4 mm Hg C.488 MCQ’s FOR MEDICAL PROFESSIONALS BY PROF. 20 . A In nutmeg liver. Caudate lobe hypertrophy 1193 Levels of which of the following is characteristically elevated 1199 Initial investigation of choice in suspected Budd-Chiari in cardiac cirrhosis ? syndrome is ? Harrison’s 18th Ed. Hepatic venography or liver biopsy showing the following ? centrilobular congestion & sinusoidal dilatation in absence of right heart failure characterize Budd- Harrison’s 16th Ed. A. 2597 Harrison’s 18th Ed. liver becomes enlarged. 2597 Harrison’s 16th Ed. gross examination of liver shows which of In Budd-Chiari syndrome. Pyramid like elevations on surface of liver Sherlock D. gross examination shows alternating red (congested) & pale (fibrotic) areas. 30 . 40 . Wilson’s an initial episode of variceal bleeding. constrictive pericarditis. enlarged liver with signs of chronic liver disease in a patient with valvular hepatic venous pressure (portal venous pressure) minus free hepatic venous pressure (intra- heart disease. Tender B. Inferior mesenteric vein . Doppler studies C. Centrilobular fibrosis extends outward in a characteristic stellate pattern from C. Centrilobular congestion & sinusoidal dilatation on liver biopsy 1191 In “nutmeg liver”. 1862 A. Intractable ascites ischemia from poor perfusion leading to necrosis of centrilobular hepatocytes with fibrosis in central areas.20 % A. None of the above C. Signs & symptoms of heart failure are notably absent. Even if the patient survives Inherited metabolic liver diseases that can progress to cirrhosis include hemochromatosis. and cystic fibrosis. All of the above Portal hypertension is defined as elevation of hepatic venous pressure gradient (HVPG) to >5 mmHg. Superior mesenteric vein A. firm. MR angiography ALP levels are characteristically elevated in cardiac cirrhosis. the inferior vena cava so that the liver parenchyma is exposed to elevated venous pressures. Post sinusoidal A. 2598 Portal vein is formed by the confluence of superior mesenteric and splenic veins. 2598 Harrison’s 18th Ed. 2598 proportion of patients have esophageal varices ? A. Intrahepatic B. Any of the above D. 2 to 5 mm Hg D. 2598 B. Harrison’s 18th Ed. Any of the above C. One . Portal vein thrombosis C. what Harrison’s 18th Ed. Polycythemia vera A. D. D. 2598 1213 Primary complication of portal hypertension is ? Harrison’s 18th Ed. Budd-Chiari syndrome (BCS) D. All of the above B. Brucellosis 1204 Esophageal varices are present in what percentage of compensated and decompensated cirrhosis ? B. 2598 A. Esophageal varices are present in 30% of patients with compensated cirrhosis and in up to 60% of those with decompensated cirrhosis (with evidence of ascites or encephalopathy). Abdominal trauma A. Venoocclusive disease or abdominal trauma. the D. 40 & 70 % when other nutritional or infectious agents (hepatitis B or C virus) are involved. 30 & 60 % Intrahepatic presinusoidal causes of portal hypertension include congenital hepatic fibrosis and schistosomiasis. Resistance to activated protein C (factor V Leiden) C. Sinusoidal obstruction A. 489 Gastroenterology MCQ’s MCQ’s FOR FORPROFESSIONALS MEDICAL MEDICAL PROFESSIONALS BY PROF. A. and hypersplenism. infection. D. ascites. 20 to 25 mm Hg 1210 ‘Symmers’ clay-pipe stem fibrosis in liver is due to ? Normal pressure in portal vein is 5 to 10 mm Hg because of low vascular resistance in hepatic sinusoids.third D. Pancreatitis B. 10 to 15 mm Hg hepatic cirrhosis is complicated by portal hypertension. AJAY MATHUR Cardiology 489 C. 5 to 10 mm Hg Portal hypertension (>10 mm Hg) mostly results from increased resistance to portal blood flow. essential thrombocythemia. 1863 C. 2598 1205 Which of the following is a posthepatic cause of portal hypertension ? A. hypertension ? Harrison’s 18th Ed. Posthepatic C. 1206 Which of the following is a posthepatic cause of portal 1212 Portal vein thrombosis may develop in ? hypertension ? Harrison’s 18th Ed. All of the above C. All of the above Intrahepatic causes of portal hypertension account for over 95% of cases of portal hypertension and are represented by the major forms of cirrhosis. deficiencies of protein C / protein S / antithrombin III. Schistosomiasis C. All of the above D. Pre sinusoidal obstruction Harrison’s 18th Ed. 1211 Portal vein obstruction may occur in association with ? Harrison’s 18th Ed. Ascites C. Pre sinusoidal 1203 Normal pressure in the portal vein is ? B. All of the above Idiopathic portal vein thrombosis may develop in hypercoagulable states like polycythemia vera. One . 2598 A. Gastroesophageal varices with hemorrhage B. All of the above increased resistance is ? Harrison’s 18th Ed. Splenic vein thrombosis Portal vein obstruction may be idiopathic or occur in association with cirrhosis. Hypersplenism D. Chronic right-sided cardiac congestion D. 2598 B. On screening of histologically confirmed cirrhosis cases. Cirrhosis occurs D.third of patients have esophageal varices. Cirrhosis Harrison’s 18th Ed. increased resistance is usually sinusoidal. Three primary complications of portal hypertension are gastroesophageal varices with hemorrhage. one . Echinococcosis B. resistance to 1207 Which of the following accounts for most cases of portal activated protein C (factor V Leiden) & mutation of prothrombin gene (G20210A). 10 & 40 % C.fourth C. Three . Post sinusoidal obstruction B. Sinusoidal Harrison’s 16th Ed. or antithrombin III B. Prehepatic A. One . Schistosomiasis alone results in pure fibrotic lesions in liver.fourth Chiari syndrome). Budd-Chiari syndrome (BCS) B. 20 & 50 % D. 1208 Budd-Chiari syndrome is an example of ? 1214 On screening of histologically confirmed cirrhosis cases. Venoocclusive disease C. Gastric vein 1209 When cirrhosis is complicated by portal hypertension. pancreatitis. Deficiencies of protein C. It is characteristically periportal (Symmers’ clay pipe–stem fibrosis). . Toxoplasmosis A.half Postsinusoidal obstruction may also occur outside the liver at the level of the hepatic veins (Budd. 1755 A. When C. protein S. Wedged hepatic vein pressure is elevated in postsinusoidal C. 25 % D. 1221 What level of portal hypertension threatens bleeding from 1216 Marker of the presence of cirrhosis in a patient being followed gastroesophageal varices ? for chronic liver disease is ? Harrison’s 18th Ed. 2598 B. Encephalopathy D. it is (Child’s class. is titrated to reduce the hepatic venous pressure gradient (HVPG = wedged hepatic venous In patients with liver disease. Nadolol B.e. Progressive decrease in platelet count B. Wedged hepatic vein pressure is elevated in sinusoidal portal A. Splenomegaly B. 1224 The goal of treatment in patients of portal hypertension is to reduce hepatic venous pressure gradient (HVPG) to ? 1219 In liver disease.490 MCQ’s FOR MEDICAL PROFESSIONALS BY PROF. 1222 Apart from propranolol. while those >5 mm have a 30% risk of bleeding within 2 years). Rectum . <15 mmHg or by 40% from baseline B. with reduced cardiac output. 1223 Doses of propranolol to treat portal hypertension should aim 1218 The risk of variceal hemorrhage is related to ? to reduce the resting pulse rate by ? Harrison’s 18th Ed. Average normal wedged-to-free gradient is 5 mmHg. Appearance of varices B. Wedged-hepatic vein pressure C. factors that predict risk of esophageal variceal bleeding include severity of cirrhosis Wedged hepatic vein pressure is elevated in sinusoidal and postsinusoidal portal hypertension. 2598 A. Carvedilol D. Ascites C. > 10 mm Hg C. 2598 A. 33 % Risk of variceal hemorrhage is related to size of varices (varices <=5 mm in diameter have a 7% risk of bleeding in 2 years. size of varix. Progressive increase in platelet count C. presence of cirrhosis in a patient being followed for chronic liver disease. None of the above In cirrhosis. normal in presinusoidal portal hypertension. or white-nipple spots0 and tense ascites. All of the above D. A low-normal platelet count can be the first clue to progression to cirrhosis. and/or esophageal varices with or without bleeding serve as markers of the subsequent episodes. Cardioesophageal junction Harrison’s 18th Ed. Tense ascites portal hypertension D. bluish color. Appearance of an enlarged spleen B. especially variceal bleeding. and patients Progressive decrease in platelet count serves as a marker of the presence of cirrhosis in a patient with a gradient >12 mmHg are at risk for variceal hemorrhage. AJAY MATHUR Gastroenterology 1215 In cirrhosis. hemorrhage & severity of liver dysfunction (high Child-Pugh score . location of varix. being followed for chronic liver disease. Severity of cirrhosis hypertension B. 2598 Harrison’s 18th Ed. splenomegaly. which other  -adrenergic blocker is 1217 Marker of the presence of cirrhosis in a patient being followed used to reduce portal pressure ? for chronic liver disease is ? Harrison’s 18th Ed. Hepatic encephalopathy D. hematocystic spots. diffuse erythema. > 6 mm Hg A. appearance of enlarged spleen. ascites. 10 % C. Atenolol A. factor predict the risk of esophageal variceal A. red streaks of mucosa overlying varix) have an increased risk of -adrenergic blockade with nonselective agents such as propranolol by 25% is reasonable. Progressive increase in lymphocyte count Wedged and free hepatic vein pressures allow calculation of a wedged-to-free gradient. 1225 “Caput medusae” is best related to ? 1220 Which of the following statements about free and wedged Harrison’s 16th Ed. All of the above Treatment of patients with clinically significant sequelae of portal hypertension. and/or esophageal varices. 2598 Harrison’s 16th Ed. 5% B. especially variceal bleeding. Development of ascites C. 2598 A. which is equivalent to the portal pressure. Severity of liver dysfunction C. <20 mmHg or by 50% from baseline Harrison’s 18th Ed. reduction of resting pulse through of the varices (red wale sign i. > 12 mm Hg D. Size of varices A. development of ascites. 2598 Harrison’s 18th Ed. 2598 B.B or C represents decompensated cirrhosis & is associated with an increased risk of bleeding). development of portal hypertension is revealed Harrison’s 16th Ed. cherry red spots. Progressive decrease in lymphocyte count D. All of the above D. height of wedged-hepatic vein pressure. Wedged hepatic vein pressure is usually normal in bleeding is ? presinusoidal portal hypertension Harrison’s 18th Ed. endoscopic stigmatas (red wale signs. appearance In treatment of portal hypertension. All of the above  -adrenergic blockade with nonselective agents (propranolol or nadolol) reduces portal pressure through vasoconstrictive effects on both splanchnic arterial bed & portal venous system in combination Progressive decrease in platelet count. 1863 A. <12 mmHg or by 20% from baseline C. development of portal hypertension is revealed by the appearance of pressure – free hepatic venous pressure) to <12 mmHg or by 20% from baseline. Such therapy is effective in preventing both a first variceal bleed & encephalopathy. MELD score). Sotalol C. > 8 mm Hg B. encephalopathy. 1864 by the appearance of which of the following ? A. 1863 hepatic vein pressure is false ? A. <6 mmHg or by 10% from baseline D. Peptic ulcers D.1 to 0. 1 1227 Most common cause of upper GI bleeding (UGIB) is ? B. 257 D. 1864 A. blood should be present in GI tract for what C. None of the above B. 1839 C. C. Transjugular intrahepatic portosystemic shunt Harrison’s 17th Ed. B. 4 C. 259. 257 A. Watermelon stomach Intravenous infusion of vasopressin at a rate of 0. Emergency portal-systemic nonselective shunts A.4 U/min results in “generalized C. Hyperactive bowel sounds Decompression procedure to lower portal pressure is accomplished without surgery through percutaneous B. Vasoconstriction in systemic venous system only A. 10 % 1230 Which of the following is false about Dieulafoy’s lesion ? B. 40 % B. Elevated blood urea nitrogen 1236 Hepatic encephalopathy occurs in what proportion of patients D. Windshield stomach . and about 90% of patients with hematochezia are bleeding from the colon. accounting for up to ~50% of cases. B. Transvenous intrahepatic portosystemic shunt 1229 Clues to upper gastrointestinal bleed include ? C. A. 259 C. 14 hours 1235 “TIPS” stands for ? Harrison’s 18th Ed. Large-caliber arteriole beneath gastrointestinal mucosa D. Erosive esophagitis Harrison’s 16th Ed. 2599 depletion and blood proteins absorbed in the small intestine). D. Generalized vasoconstriction 1231 Gastric antral vascular ectasia is the cause of ? B. 2 Harrison’s 17th Ed. 1 Peptic ulcers are the most common cause of UGIB. rectum (hemorrhoids). Ligament of Treitz acute bleeding requires both fluid and blood-product replacement as well as prevention of subsequent bleeding with Endoscopic variceal ligation (EVL). 30 % A. 7 hours C. A. 4 A. hyperactive bowel sounds & elevated BUN level (due to volume Harrison’s 18th Ed. retroperitoneal space & falciform ligament of liver control bleeding acutely ? (periumbilical or abdominal wall collaterals). Endoscopic intervention B. 2598 vessels that radiate from umbilicus toward xiphoid & rib margins (caput medusae). 2 1228 For melena. Washermen stomach D. Transarterial intrahepatic portosystemic shunt A. 259 D. 1841 C. Windmill stomach vasoconstriction” leading to diminished blood flow in the portal venous system. Balloon tamponade Harrison’s 17th Ed. AJAY MATHUR Cardiology 491 C. termed transjugular intrahepatic portosystemic shunt (TIPS). Melena placement of a portal-systemic shunt. Retroperitoneal space Watermelon stomach is seen in gastric antral vascular ectasia. B. Transcuteneous intrahepatic portosystemic shunt Melena indicates that blood has been present in the GI tract for at least 14 hours. 491 Gastroenterology MCQ’s MCQ’s FOR FORPROFESSIONALS MEDICAL MEDICAL PROFESSIONALS BY PROF. Harrison’s 16th Ed. Over 90% of patients with melena are bleeding proximal to 1233 Sengstaken-Blakemore tube has how many lumens ? the ligament of Treitz. Also called persistent caliber artery Harrison’s 16th Ed. 1864 A. Most common on greater curvature of proximal stomach 1237 Intravenous infusion of vasopressin results in ? D. 3 B. Esophageal varices C. Falciform ligament of liver 1232 Which of the following should be the first-line treatment to Major sites of portalsystemic collateral flow are veins around cardioesophageal junction (esophagogastric varices). Somatostatin / octreotide 1226 Upper gastrointestinal tract is upto ? B. Ampulla of Vater Endoscopic intervention should be the first-line treatment to control bleeding acutely. 22 hours A. 2599 D. 1864 Dieulafoy’s lesion is seen most commonly on the lesser curvature of the proximal stomach. Vasoconstriction in portal venous system only Harrison’s 17th Ed. 3 hours Balloon tamponade of bleeding gastroesophageal varices may be accomplished with a triple-lumen (Sengstaken-Blakemore) or four-lumen (Minnesota) tube with esophageal and gastric balloons. 3 duration ? Harrison’s 17th Ed. End of jejunum Upper GI is above the ligament of Treitz. D. Bleeds through a pinpoint mucosal erosion Hepatic encephalopathy occur in 20 % of patients after TIPS. Pylorus D. A. Treatment of C. All of the above after TIPS ? Clues to UGIB include melena. C. D. Gastroduodenal erosions 1234 Minnesota tube has how many lumens ? D. 20 % Harrison’s 17th Ed. Abdominal wall collaterals appear as tortuous epigastric Harrison’s 18th Ed. Hepatocellular cancer Nonselective beta-adrenergic blockade is sometimes effective. Any of the above Serious side effects associated with vasopressin therapy include cardiac and gastrointestinal tract Hepatic hydrothorax is more common on right side due to a rent in the diaphragm with free flow of ischemia. 2599 1247 There is an increased risk for developing which of the following when levels of ascitic fluid proteins are very low ? A. < 6 gram D. are direct splanchnic vasoconstrictors. 2600 A. splenomegaly with variceal hemorrhage 1248 What absolute level of polymorphonuclear leukocytes count suggests the possibility of ? suggests ascitic fluid infection ? Harrison’s 16th Ed. which of the following is curative ? 1249 The recommended amount of sodium per day in the Harrison’s 16th Ed. 1864 Harrison’s 18th Ed. 1242 In absence of cirrhosis. Gastrointestinal tract ischemia B.492 MCQ’s FOR MEDICAL PROFESSIONALS BY PROF. infectious or malignant causes of ascites should be considered. Portal vein thrombosis B. splenomegaly in association with variceal hemorrhage should suggest A high level of red blood cells in the ascitic fluid signifies a traumatic tap or perhaps a hepatocellular the possibility of splenic vein thrombosis. acute renal failure. All of the above In cirrhosis. 1241 Which of the following agents is useful in the treatment of portal hypertensive gastropathy ? Harrison’s 18th Ed. Proton pump inhibitors or other agents useful in the treatment of peptic disease are usually not helpful. Sucralfate B. 1864 A. Beta adrenergic blockers A. “Underfilling” theory D. 1864 is curative. protein concentration of ascitic fluid is low (<1 g/dL). 1865 Harrison’s 18th Ed. Serum ascites-to-albumin gradient A. When the gradient is <1. > 250 / µL In the absence of cirrhosis. Splenic vein thrombosis A. Left side C. Direct splanchnic vasoconstrictors Harrison’s 16th Ed. Spontaneous Bacterial Peritonitis (SBP) Congestive gastropathy due to the venous hypertension is a complication of portal hypertension. terms like exudative or transudative fluid have been replaced. Splenic hamorrhage C. > 200 / µL D. < 4 gram C. Generalized vasoconstrictors 1244 Theories proposed for ascites include ? B. Any of the above D. Serum anion-to-albumin gradient D. Right side B. 2600 A. and hyponatremia. Nitroglycerin / isosorbide dinitrate D. 2600 Harrison’s 16th Ed. < 2 gram B. “Peripheral arterial vasodilation” theory D. Cirrhosis the effectiveness of vasopressin and reduce complications. 1864 management of ascites is ? Harrison’s 18th Ed. octreotide. Portal-systemic shunts C. 1865 C. splenic vein thrombosis should be specifically sought. None of the above B. leading to portal hypertension leading to ascitis. 1240 Concurrent use of which drug may enhance the effectiveness 1246 What does SAAG stand for ? of vasopressin and reduce complications ? Harrison’s 18th Ed. Serum albumin-to-ascites gradient B. AJAY MATHUR Gastroenterology 1238 Somatostatin and octreotide are ? For isolated gastric varices. “Overflow” theory Somatostatin and its analogue. Refractory ascites C. Proton pump inhibitors Harrison’s 18th Ed. TIPS B. Cardiac ischemia A. With the use of serum ascites-to- Concurrent use of venodilators like IV nitroglycerin or isosorbide dinitrate sublingually may enhance albumin gradient (SAAG). the question of ascitic fluid infection should be strongly considered. patients are at increased risk for developing SBP. Serum albumin-to-anion gradient C. 2600 A. cancer or a ruptured omental varix. None of the above C. 1243 For isolated gastric varices due to splenic vein thrombosis.1 g/dL. Octreotide C. gradient between serum albumin level and ascitic fluid albumin level is >1. Acute renal failure C. A. D. H2 receptor blockers A. When the absolute level of polymorphonuclear leukocytes is >250/µL.1 g/dL. When levels of ascitic fluid proteins are very low. ascitic fluid into the thoracic cavity. > 100 / µL B. 2600 B. Hypernatremia D. since splenectomy Harrison’s 16th Ed. Hepatic Encephalopathy D. Somatostatin B. > 150 / µL C. C. All of the above 1239 Serious side effects associated with vasopressin therapy are all except ? 1245 Hepatic hydrothorax is more common on which side ? Harrison’s 16th Ed. Splenectomy . Direct systemic vasoconstrictors A. Bilateral D. Profound renal vasoconstriction 1254 In cirrhosis liver with ascites.2 mg /dL 1255 Which of the following statements about spontaneous bacterial peritonitis (SBP) is false ? B.4 mg /dL A. ~5% B. High urinary sodium excretion (>25 mmol/day) circulation. N Engl J Med 2004. frequency of SBP is significantly increased.5 mg /dL B. once-weekly administration of antibiotics is used as D. Arteriovenous shunting D. Ascites is of recent onset ? D. After resolution. Primary prevention recommended in high-risk cirrhotics A.5 mg /dL. with subsequent bacteremia & infection of ascitic fluid. 2601 B. Bacterial translocation is the presumed mechanism for development of SBP. Type I HRS carries a mortality rate of >90% C. ~ 30 % 1253 In cirrhosis liver with ascites. 1. Splanchnic vasodilation C. SBP frequently recurs. B. B. response to salt restriction alone D. response to salt restriction alone HRS is a type of functional renal failure “without renal pathology” that occurs in ~10% of patients is more likely to occur if ? with advanced cirrhosis or acute liver failure. Empirical coverage for anaerobes is necessary B. 1.350:1646-54 HRS is a unique form of prerenal ARF that complicates advanced cirrhosis and acute liver failure and A. >250 PMN/µL is diagnostic D. ~ 20 % D. Recurrent episodes are relatively uncommon 1250 Ascites may be demonstrated on physical examination when peritoneal fluid accumulation exceeds ? SBP involves translocation of bacteria from intestinal lumen to lymph nodes. Splenic vein thrombosis C. leading to bacteremia and seeding of the ascitic fluid. All of the above N Engl J Med 2009.3 mg /dL Harrison’s 18th Ed. Patients who have C. Prophylactic norfloxacin reduces recurrences Harrison’s 16th Ed. 200 ml B. Ascitic fluid has high concentrations of albumin Most studies & consensus conferences have defined renal failure in cirrhosis as a serum creatinine concentration above 1. 1. with gut A. A. 2601 D. In ascitic fluid.361:1279-90 A. Harrison’s 16th Ed. Normal renal function 1260 Renal failure in cirrhosis is defined as serum creatinine above C. GI bleed prophylaxis for recurrent SBP. 2601. C. None of the above Harrison’s 18th Ed. 1866 70% probability of recurrence at one year. Development of hepatocellular carcinoma D. resulting in profound renal vasoconstriction resulting from extreme underfilling of arterial A. 1252 Factors leading to worsening ascites include all except ? 1258 Hepatorenal syndrome (HRS) occurs in what percentage of Harrison’s 16th Ed. 1866 In HRS. ~ 10 % C. Worsening liver disease A. 1000 ml Harrison’s 18th Ed. All of the above C. kidneys are structurally normal but fail due to splanchnic vasodilation & arteriovenous shunting. Ascites is of recent onset 1259 Which of the following about hepatorenal syndrome is false ? Harrison’s 18th Ed. . Medication noncompliance In patients with variceal hemorrhage. Monomicrobial nonneutrocytic bacterascites is a variant of SBP 1261 Which of the following about hepatorenal syndrome is false ? SBP is characterized by spontaneous infection of ascitic fluid in absence of intraabdominal source Harrison’s 18th Ed. A single organism is typically isolated < 2 gram of sodium per day is the recommended amount in the management of ascites. Underlying liver disease is reversible A. C. Anaerobes are found less frequently D. Develops without obvious primary source of infection D. with an estimated Harrison’s 16th Ed. and prophylaxis against SBP is recommended when a patient presents with upper GI bleeding. 493 Gastroenterology MCQ’s MCQ’s FOR FORPROFESSIONALS MEDICAL MEDICAL PROFESSIONALS BY PROF. Type I HRS is the more aggressive form flora traversing the intestine into mesenteric lymph nodes. Escherichia coli are most commonly isolated is seen in in patients with refractory ascites. AJAY MATHUR Cardiology 493 D. < 8 gram B. Therapy is for 10 to 14 days 1251 Factors leading to worsening ascites include all except ? B. 2601 of infection. 500 ml 1257 Which of the following statements about spontaneous bacterial peritonitis (SBP) is false ? C. 1866 C. 1. HRS is seen in patients with refractory ascites 1256 Which of the following statements about spontaneous bacterial peritonitis (SBP) is false ? D. 2601 A. 1500 ml A. Superimposed infection had an episode(s) of SBP and recovered. 1866 patients with advanced cirrhosis or acute liver failure ? Harrison’s 18th Ed. 2600 C. Type I HRS is the more aggressive form of the disease & carries a mortality rate of >90%. Excessive salt intake D. 1866 A. Precipitating factor can be corrected B. Portal vein thrombosis B. None of the above is more likely to occur if ? Harrison’s 16th Ed. The best therapy for HRS is liver transplantation. 1270 Which of the following statements about hepatorenal syndrome (HRS) is false ? 1264 Which of the following is true for type 2 hepatorenal syndrome ? Harrison’s 16th Ed. Midodrine B. Blood-brain barrier is intact C.350:1646-54 Harrison’s 16th Ed.5 mg/dl in ? the risk of hepatorenal syndrome & improves survival ? N Engl J Med 2009. Hypotension D. short-chain fatty acids.350:1646-54 A. Ammonia is incriminated in its pathogenesis D. Hepatic encephalopathy dopamine is effective B. None of the above A. & phenol are renal vasodilators like dopamine or prostaglandins is ineffective. 1867 C. < 4 weeks B. Many patients have elevated blood ammonia levels Best approach in management of HRS is administration of vasoconstrictor drugs. < 6 weeks C. systemic circulation. Vitamin E Type 1 HRS is characterized by a doubling of serum creatinine level to > 2.361:1279-90 N Engl J Med 2009. which affects not only the kidneys Harrison’s 18th Ed. A. No tendency of serum creatinine to progress over time D. Urinalysis normal A. Harrison’s 16th Ed. Eplerenone C. Midodrine 1273 Which of the following charactetistics about hepatic B. adrenal glands. and liver. Fluctuating neurologic signs D. along with octreotide & IV albumin. Most have refractory ascites B. Hypernatremia D. Common precipitating event is SBP C. Vasodilator therapy with intravenous infusions of low dose A. All of the above 1271 Which of the following statements about hepatorenal syndrome In type 2 hepatorenal syndrome. A common precipitating event is spontaneous bacterial peritonitis (SBP). Kidneys are structurally smaller type 1 hepatorenal syndrome is characterized by progressive oliguria and a rapid rise of the serum creatinine. Octreotide encephalopathy is false ? Harrison’s 16th Ed. most patients have refractory ascites. C. Intravenous albumin A. patients of HRS are treated with midodrine. Behavior & personality changes Currently. Avid sodium retention C. AJAY MATHUR Gastroenterology 1262 Type 1 HRS is characterized by doubling of serum creatinine 1268 Long-term administration of which of the following reduces level to > 2.361:1279-90 A. < 2 weeks A. Diagnosis of hepatic encephalopathy is clinical syndrome ? Harrison’s 18th Ed.361:1279-90 1274 Which of the following statements about hepatic A. Treatment with D. Refractory ascites C. All of the above D. TIPS can improve renal function C. No electroencephalographic changes 1267 Which of the following is a vasoconstrictor drug ? N Engl J Med 2009. Mercaptans. 1867 1265 Type 2 HRS is mainly characterized by ? A. All of the above 1272 Which of the following is false about hepatic encephalopathy ? Type 1 hepatorenal syndrome has severe multiorgan dysfunction. Oliguria without identifiable causes of renal dysfunction D. More common in chronic liver disease B.361:1279-90 B. an -agonist.494 MCQ’s FOR MEDICAL PROFESSIONALS BY PROF. < 8 weeks D. incriminated in its pathogenesis . Norepinephrine A. Increase in serum creatinine is moderate C. increase in serum creatinine (HRS) is false ? is moderate and has no tendency to progress over time. Treatment is usually unsuccessful N Engl J Med 2009. Norfloxacin B. Disturbances in consciousness D. Essential for diagnosis of fulminant hepatic failure 1266 Which of the following treatments have a role in hepatorenal C. Toresamide D. 2601 but also the heart. Rapid rise of serum creatinine B. Progressive oliguria A. Worsening azotemia B. 1269 Which of the following statements about hepatorenal syndrome 1263 Which of the following is true for type 1 hepatorenal syndrome ? (HRS) is false ? N Engl J Med 2004. All of the above C.5 mg/dl in < 2 weeks. 2601 D. Type Long-term oral norfloxacin reduces risk of hepatorenal syndrome & improves survival. Kidneys can be used for renal transplantation C. 2601 B. whereas the clinical course of patients with type 2 hepatorenal syndrome is mainly characterized by refractory ascites. Treatment of choice is liver transplantation D. 1867 A. brain. Renal biopsy is normal B. 1867 N Engl J Med 2004. Terlipressin encephalopathy is false ? Harrison’s 18th Ed. 2 is characterized by a stable or less rapidly progressive course than in type 1. All of the above B. It is a nonabsorbable disaccharide. Rigidity 1287 Which of the following about lactulose is false ? B. Schizophrenia A. AJAY MATHUR Cardiology 495 1275 Which of the following statements about hepatic 1282 Mercaptans are derived from intestinal metabolism of ? encephalopathy is false ? Harrison’s 16th Ed. Superimposed acute viral hepatitis Harrison’s 16th Ed. None of the above A. 1868 D. Methionine concentrations of GABA in the CNS C. 2-3 A. Acute alcohol intoxication 1277 Which of the following is the most common predisposing factor B. Gastrointestinal bleeding B. 4-6 B. B. 1868 D. Deterioration in handwriting 1288 Goal of lactulose therapy is to promote how many soft stools per day ? 1281 Typical smell in fetor hepaticus is due to ? Harrison’s 18th Ed. Reversal of sleep / wake cycle D. Leucine B. Korsakoff’s psychosis for hepatic encephalopathy ? C. Gastrointestinal bleeding B. Bilirubin D. Hypocalcemia Harrison’s 16th Ed.10 D. 6-8 C. 1868 D. Sedative overdose for hepatic encephalopathy ? C. Decreased DTR Harrison’s 18th Ed. Subdural hematoma Harrison’s 16th Ed. EEG changes The mainstay of treatment for encephalopathy is lactulose. Ammonia C. 1868 A. Nonabsorbable D. Consequent catharsis eliminates nitrogenous products in gut that are responsible for the development of encephalopathy. 1868 D.3 soft stools per day. 1. Threonine A. Wilson’s disease A. Excessive magnesium deposition in basal ganglia contribute Harrison’s 16th Ed. 1867 A. Endogenous benzodiazepines act through the GABA receptor D. Alcoholic hepatitis A. . Extensor plantar signs A. 1868 D. Increased dietary protein 1284 Disorders that can mimic the clinical features of hepatic encephalopathy are all except ? C. 2602 C.4-benzodiazepines is isolated from brain tissue of patients with fulminant hepatic failure 1283 For diagnosis of hepatic encephalopathy. Gastrointestinal bleeding B. which of the following tests has most relevance ? D. Computed tomography of brain Harrison’s 16th Ed. Hypoglycemia except ? C. 1867 Harrison’s 16th Ed. Extrahepatic bile duct obstruction 1286 Disorders that can mimic the clinical features of hepatic encephalopathy are all except ? C. Delirium tremens Harrison’s 16th Ed. C. Surgery A. Examination of the cerebrospinal fluid for hepatic encephalopathy ? C. Surgery 1285 Disorders that can mimic the clinical features of hepatic encephalopathy are all except ? C. 1868 its pathogenesis A. 1868 D. Mercaptans B. 1868 D. 2602 Harrison’s 16th Ed. 495 Gastroenterology MCQ’s MCQ’s FOR FORPROFESSIONALS MEDICAL MEDICAL PROFESSIONALS BY PROF. Seizures B. Leads to colonic acidification Harrison’s 16th Ed. Asterixis which results in colonic acidification. All of the above The goal of lactulose therapy is to promote 2 . 1868 D. Injudicious use of CNS-depressing drugs A. Encephalitis A. Reduced levels of consciousness is due to excessive B. Electrolyte disturbances Harrison’s 16th Ed. MRI of brain A. Elevated serum ammonia level 1276 Which of the following is the most common predisposing factor B. Constipation Harrison’s 16th Ed. Isoleucine C. Wernicke’s encephalopathy 1278 Which of the following is the most common predisposing factor B. Meningitis 1279 Neurologic signs in hepatic encephalopathy includes all B. 8 . Disaccharide 1280 Earliest sign of hepatic encephalopathy is ? C. 2602 C. Factor VII N Engl J Med 2008. defined by the triad of liver disease.358:2378-87 B. Decreased synthesis of clotting factors number of dilated vessels. Hypoxemia 1290 Supplementation of which of the following is recommended in B. 2602 The unique striking pathological feature of hepatopulmonary syndrome is gross dilatation of pulmonary precapillary & capillary vessels (15 to 100 µm diameter). All of the above 1300 Which of the following has clinical similarities to hepatopulmonary syndrome ? Coagulopathy is almost universal in patients with cirrhosis. 1869 B. All of the above A. HOCM C. 2602 and a defect in oxygenation. Azithromycin D. Factor V C. coupled with an absolute increase in A.liver disease. diameter of capillary ranges between 8 and B. Factor V syndrome is ? C. There is decreased synthesis of clotting N Engl J Med 2008. Absolute increase in number of dilated vessels Vitamin K–dependent clotting factors are Factors II. Dandy-Walker malformations . D. Rifaximin syndrome. and X. Calcium more when the patient moves from a supine to an upright position (called orthodeoxia). IX. Blue rubber bleb syndrome B. N Engl J Med 2008. pulmonary vascular dilatation. C. Because of a decrease in hepatic mass. Hypercarbia A. 2524. which clotting factor is not reduced ? 1298 Which of the following is not a part of hepatopulmonary Harrison’s 16th Ed. 2602 D. D. Hypoxemia Harrison’s 18th Ed. pleural and pulmonary arteriovenous shunts and portopulmonary venous anastomoses can be seen. Impaired clearance of anticoagulants 15 µm. Orthodeoxia Harrison’s 18th Ed. Pulmonary arteriovenous shunting B. A. Lumefantrine The defect in oxygenation is due to a ventilation.358:2378-87 factors and impaired clearance of anticoagulants. Factor II N Engl J Med 2008. Factor XI C. in a healthy person . Budd-Chiari Syndrome A. VII. Rifaximin is a poorly absorbed rifampin derivative & is highly effective against noninvasive bacterial Harrison’s 18th Ed. Thrombocytopenia due to hypersplenism D. Left atrial myxoma B.perfusion mismatch. C. 279 1291 First clotting factor to be depleted in cirrhosis liver is ? A. Platypnea patients with hepatic encephalopathy ? C. Factor VIII Platypnea (dyspnea in upright position with relief in supine position) is also a feature of left atrial D. Pulmonary vascular dilatation D. 2524 C. Ritonavir Patients with long-standing cirrhosis and portal hypertension are prone to develop the hepatopulmonary C. Factor IX myxoma.5 kPa) or C.& capillary vessels B. and pulmonary arteriovenous shunting. Factor VII D. Factor VII B. Also.496 MCQ’s FOR MEDICAL PROFESSIONALS BY PROF. Pleural & pulmonary arteriovenous shunts D.358:2378-87 D. Factor XI A. Copper Hepatopulmonary syndrome is characterized by platypnea and orthodeoxia. 1297 Platypnea is a clinical presentation of ? Harrison’s 18th Ed. Magnesium Zinc supplementation is at times helpful in patients with hepatic encephalopathy. AJAY MATHUR Gastroenterology 1289 Which of the following has a role in the treatment of hepatic A. Chiari malformation 1295 The triad of hepatopulmonary syndrome includes all except ? Harrison’s 18th Ed. If the partial pressure of oxygen in arterial blood decreases by 5% or more or by 4 mm Hg (0. Gross dilatation of pulmonary pre. Constrictive pericarditis Harrison’s 16th Ed.358:2378-87 pathogens (toxigenic & enteroaggregative E. hypoxemia. representing shortness B. coli). Liver disease C. All of the above 1294 Coagulopathy in liver disease results due to ? Harrison’s 18th Ed. administration of parenteral vitamin K does not improve clotting factors or prothrombin time. Harrison’s 18th Ed. Defect in oxygenation 1293 Reduction in levels of which clotting factor is not worsened by the coincident malabsorption of vitamin K ? Hepatopulmonary syndrome has three components . A. Zinc of breath and oxygen desaturation that occur paradoxically upon assuming an upright position. Factor V A. Factor II 1299 The unique striking pathological feature of hepatopulmonary B. 1292 In hepatic cirrhosis. Patients may have thrombocytopenia from hypersplenism due to portal hypertension. Rifaximin (550 mg twice daily) is very effective in treating encephalopathy without the known side 1296 Hepatopulmonary syndrome is manifested by ? effects of neomycin (renal insufficiency and ototoxicity) or metronidazole (peripheral neuropathy). 1869 syndrome ? A. A. Pulmonary vascular constriction D. he or she may describe worsening dyspnea (platypnea) related to further ventilation perfusion mismatch. Liver disease encephalopathy ? B. Cryptococcus B. porphyria cutanea tarda. HCV A. glycogenesis types 1 and 3. All of the above 1305 Which of the following is a carcinogen ? Most paraneoplastic syndromes in HCC are biochemical abnormalities without associated clinical consequences.365:1118-27 A. A. 777 citrullinemia. All of the above provides support for the hypothesis that blood from the gut must cross liver to prevent pulmonary vascular dilatation. HCV-associated HCC patients tend to have more frequent and advanced cirrhosis. Tannic acid Rare congenital cardiac disorders without liver injury in which either hepatic venous blood flow does C. N Engl J Med 2011. increased thyroxin-binding globulin. some porphyrias. HIV B. ~ 75 % D. autoimmune hepatitis. Aspergillus B. D. carcinoid syndrome. Nocardia 1310 Which of the following is a paraneoplastic syndrome in HCC ? Harrison’s 18th Ed. However.365:1118-27 1309 Factor associated with an increased risk of developing HCC is ? A. 497 Gastroenterology MCQ’s MCQ’s FOR FORPROFESSIONALS MEDICAL MEDICAL PROFESSIONALS BY PROF.365:1118-27 D. 777 B. hypercholesterolemia. Coffee drinking has also been associated with reduced C. ~ 50 % Studies conducted in Japan and southern Europe found that coffee drinking is associated with a reduced risk of hepatocellular carcinoma. Alcoholic liver disease C. alcohol. and orotic aciduria. cryptogenic cirrhosis and NASH/NAFL. infection with hepatitis B virus (HBV) is endemic C. ~ 50 % Hepatocellular carcinoma incidence reaches a peak at approximately 70 years of age. Hepatocellular carcinoma rarely occurs before 40 years of 1307 Worldwide. 778 Hepatocellular Carcinoma A. Pyrrolizidine alkaloids sex characteristics (gynecomastia. testicular atrophy & precocious puberty) & porphyria cutanea tarda. Tea drinking carcinoma have cirrhosis ? N Engl J Med 2011. AJAY MATHUR Cardiology 497 D. chronic HBV infection accounts for what percentage age of all cases of hepatocellular carcinoma ? N Engl J Med 2011.365:1118-27 C. the remainder having chronic active B. autoimmune chronic active hepatitis. Less common causes include hereditary hemochromatosis. Harrison’s 18th Ed. tyrosinemia. 40 years A. HBV Harrison’s 18th Ed. and Wilson’s disease. alpha1-antitrypsin deficiency. ~ 100 % 1302 Which of the following is associated with a reduced risk of hepatocellular carcinoma ? Worldwide. 10 years B. 1306 A typical interval between HCV-associated transfusion and subsequent HCC is approximately ? Harrison’s 18th Ed. Cirrhosis from any cause D. ~ 25 % D. changes in secondary A. Safrole not reach the lung or portal venous blood reaches the inferior vena cava without passing through the liver (Type 1 Abernethy malformation) have clinical similarities to hepatopulmonary syndrome. Candida A. alpha1-antitrypsin deficiency. Alcohol drinking A. chronic HBV infection accounts for ~50% of all cases of hepatocellular carcinoma and virtually all childhood cases. majority (70 to 80%) infection with all except ? of patients with HBV-related hepatocellular carcinoma have cirrhosis. Coffee drinking 1308 What percentage of patients with HBV-related hepatocellular B. alcoholic liver disease & nonalcoholic D. Factor associated with an increased risk of developing HCC include hepatitis. This D. C. Hepatitis B or C chronic infection C. but in HBV-associated HCC. Erythrocytosis D. Rates of liver cancer among men are two to four times as high as the rates among women. Hepatocellular carcinoma incidence reaches a peak at A. . only half the patients have cirrhosis. Hypercalcemia Aflatoxin B1 is a product of the Aspergillus fungus. Nonalcoholic steatohepatitis (NASH) Major risk factors for HCC include infection with HBV or HCV. Hypercholesterolemia leads to hepatocellular carcinoma. Type 1 Abernethy malformation B. Wilson disease. 777 dysfibrinogenemia. hypercalcemia. Harrison’s 18th Ed. They include hypoglycemia.365:1118-27 D. 779 C. ~ 75 % insulin levels and a reduced risk of type 2 diabetes. Highest incidence rates of liver cancer is in regions where hepatitis. D. N Engl J Med 2011. All of the above fatty liver disease. Liver cancer is the fifth most common cancer in men and the A typical interval between HCV-associated transfusion and subsequent HCC is approximately 30 seventh in women years. ~ 25 % approximately 50 years of age B. ~ 100 % 1303 Major risk factors for hepatocellular carcinoma (HCC) include HBV can cause hepatocellular carcinoma in the absence of cirrhosis. 30 years N Engl J Med 2011. Less common association is with 1304 Aflatoxin B1 is related to which of the following pathogen ? primary biliary cirrhosis and several metabolic diseases including hemochromatosis. 20 years 1301 Which of the following statements is false ? C. erythrocytosis. It is a most potent ubiquitous natural chemical carcinogen producing signature mutations in p53 (mutation of arginine to serine at codon 249) and C. Tobacco chewing B. Areas of early arterial enhancement Pittsburgh and by Roy Calne in Cambridge. 779. 10 to 15 1312 Which of the following is a protein induced by vitamin K D. A. Alagille’s syndrome 1316 Child-Pugh scoring system uses how many clinical measures of liver disease ? B. 4 . AJAY MATHUR Gastroenterology 1311 Which of the following estimations are useful in the C. Budd-Chiari syndrome C. 2607. Des-gamma-carboxyprothrombin (DCP) 1317 How many points in Child-Pugh scoring system denote class C. Ultrasonographic imaging B. liver transplantation is now performed routinely worldwide. C. Caroli’s disease A. mental and growth retardation is called ? PET imaging was unsuccessful for the purpose. N Engl J Med 2011. Lens culinaris agglutinin-reactive fraction of AFP (AFP-L3) A disease ? N Engl J Med 2011. Ultrasound D. It may predict for portal vein invasion. PW Angus 1314 Which of the following imaging feature is diagnostic of B. with des-gamma-carboxyprothrombin or lectin-bound alpha-fetoprotein. progressive liver failure. B. areas of early congenital malformations. Caroli’s disease B. Harrison’s 18th Ed.365:1118-27 The Child-Pugh scoring system uses five clinical measures of liver disease. KF Murray A. 780 B.498 MCQ’s FOR MEDICAL PROFESSIONALS BY PROF. Budd-Chiari syndrome B. and positive and negative predictive values for this purpose are unknown.365:1118-27 C. Transarterial catheter embolization D.365:1118-27 D. Transarterial cavity embolization Transarterial chemoembolization (TACE) is useful in intermediate-stage hepatocellular carcinoma. 2606 is associated with high cost as well as possible harm. Byler’s disease A. Byler’s disease N Engl J Med 2011. Liver Transplantation D. JA Fishman N Engl J Med 2011. and In patients with cirrhosis and a focal hepatic mass larger than 2 cm in diameter. 7 to 9 points fetoprotein alone. England. A. 16 to 20 points absence ? A sum of 5 or 6 points Child-Pugh scoring system indicate class A disease. PIVKA-3 A. Table 310-1 1315 Which of the following is of no use in the diagnosis of HCC ? A. and Harrison’s 18th Ed. Magnetic resonance imaging (MRI) Chapter 310. PIVKA-2 N Engl J Med 2011. PIVKA-1 1318 TACE stands for ? B. 2607.365:1118-27 C. or the most severe disease. 5 or 6 points Combined measurement of alpha-fetoprotein.365:1118-27 D. A. Computed tomography (CT) C.365:1118-27 cancer-related symptoms. Their sensitivity. Success measured as 1-year survival has improved from 30% in the 1970s C. and their use Harrison’s 18th Ed. 5 surveillance for hepatocellular carcinoma ? D. 779 10 to 15 points class C. specificity. 7 to 9 points class B. including pulmonary stenosis is arterial enhancement and delayed washout in venous or delayed phase of four-phase multidetector CT or in dynamic contrast-enhanced MRI have high predictive value for HCC. 3 D. Areas of delayed washout to 90% today. Serum alpha-fetoprotein (AFP) B. vascular invasion. D. provide limited additional benefit as compared with the measurement of alpha. later. This protein is increased in as many as 80% of C. called ? Harrison’s 18th Ed. Gadolinium-enhanced MRI D. N Engl J Med 2011. Transarterial cryo embolization PIVKA-2 is a protein induced by vitamin K absence. It is always elevated after Coumadin use. D. PIVKA-4 B. Focal hepatic mass >2 cm in diameter in cirrhotics Pioneered in 1960s by Thomas Starzl at the University of Colorado and. with paucity of bile ducts. All of the above 1320 Arteriohepatic dysplasia. 1313 Which of the following imaging modality is most recommended TACE improves survival among patients with preserved liver function. Thomas Starzl hepatocellular carcinoma ? C. Table 310-1 A. Triphasic CT C. Alagille’s syndrome Harrison’s 18th Ed. particularly those with Child- for hepatocellular carcinoma surveillance ? Pugh class A cirrhosis who do not have extrahepatic metastases. 6 Harrison’s 18th Ed. A. Transarterial chemo embolization HCC patients but may also be elevated in patients with vitamin K deficiency. or prominent Harrison’s 18th Ed. PET imaging 1321 Intrahepatic cholestasis. All of the above 1319 Who pioneered liver transplantation ? CT and MRI are not generally recommended for hepatocellular carcinoma surveillance. All of the above A. at the University of B. 780. < 20 C. Advanced age (>70 years) should be considered a relative contraindication. Active alcohol abuse Cyclosporine is a calcineurin inhibitor (CNI). < 15 B. Caroli’s disease D. < 25 D. with 34 levels ranging between 6 and 40. 2610 B. cells in the bone marrow. Budd-Chiari syndrome UNOS stands for United Network for Organ Sharing. It blocks early activation of T cells & is specific for T cell functions that result from the interaction of T cell with its receptor and that involve the calcium- Absolute contraindications for transplantation include life-threatening systemic diseases. None of the above D. tsukubaensis. United Network for Organ Substitution D. tumor necrosis factor . and active drug Cyclosporine also inhibits B cell functions. Tacrolimus D. which of the following is the most common become available unless the MELD score exceeds 20. 2607. Voluntary donors Tacrolimus is a macrolide lactone antibiotic isolated from a Japanese soil fungus. Nephrotoxicity B. chronic hepatitis C & alcoholic liver disease are the most common indications for liver transplantation. Suicide deaths D. Cyclosporine C. Cyclosporine A. Liver recipients with MELD scores <15 experienced higher posttransplantation mortality rates. Calcineurin inhibitor (CNI) B. The MELD scale is continuous. Neonatal hepatitis D. Activity of cyclosporine leads to inhibition of lymphokine extrahepatic bacterial or fungal infections. gene activation. University of Wisconsin (UW) solution used for preservation C. This process occurs without affecting rapidly dividing or alcohol abuse. United Network for Organ Selection A. United Network for Organ Sharing B. Active drug abuse D. Primary biliary cirrhosis C. 2610 liver transplantation ? Harrison’s 18th Ed. blocking interleukins 2. 2607 Harrison’s 18th Ed. United Network for Organ Surgery C. Nonnucleoside purine metabolism inhibitor C. accounting for over 40% of all adult candidates who undergo the procedure. It was adopted in 2002. 2610 A. Congenital hepatic fibrosis A. 1326 Cadaver donor livers for liver transplantation are procured 1332 Which of the following is a macrolide lactone antibiotic ? primarily from victims of ? Harrison’s 18th Ed. AJAY MATHUR Cardiology 499 1322 Multiple cystic dilatations of the intrahepatic biliary tree is 1328 UNOS stands for ? called ? Harrison’s 18th Ed. Head trauma B. 2608 Harrison’s 18th Ed. Human leukocyte antigen (HLA) matching is not required B. Jail deaths C. Primary sclerosing cholangitis A. Donor organs usually do not 1324 Currently. but present with use of Cyclosporine ? Harrison’s 18th Ed. and other lymphokines. Alagille’s syndrome B. Mycophenolic acid C. Byler’s disease C. Compatibility in ABO blood group is essential A. Hirsutism D. Rapamycin D. Table 310-1 A. uncontrolled dependent signal transduction pathway. Isolated from Streptomyces tsukubaensis A. Tacrolimus B. D. 499 Gastroenterology MCQ’s MCQ’s FOR FORPROFESSIONALS MEDICAL MEDICAL PROFESSIONALS BY PROF. All of the above 1325 Which of the following is not an absolute contraindication for 1331 Which of the following is true about Cyclosporine ? Harrison’s 18th Ed. . Hypertension C. 2608 A. Chronic hepatitis C B. 2607 1330 Which immunosuppressive agent can be given after liver A. Streptomyces Cadaver donor livers for transplantation are procured primarily from victims of head trauma. Mycophenolic acid Currently. 1327 Which of the following about cadaver donor livers for liver 1333 Which of the following side effects are not present with use of transplantation is false ? Tacrolimus. 2609 Harrison’s 18th Ed. and 4. indications for liver transplantation in adults ? Harrison’s 18th Ed. Diabetes mellitus Tacrolimus does not cause hirsutism or gingival hyperplasia. 3. Metastatic malignancy C. Fulminant hepatitis transplantation ? Harrison’s 18th Ed. 1323 The most common indication for transplantation in children 1329 Liver recipients with what MELD scores experienced higher is ? posttransplantation mortality rates ? Harrison’s 18th Ed. Crigler-Najjar disease type I C. Advanced age (>70 years) B. preexisting advanced cardiovascular or pulmonary disease. 2609 A. 2610 Harrison’s 18th Ed. 2608 A. < 30 The most common indication for transplantation in children is biliary atresia. metastatic malignancy. Monoclonal antibodies to T cells D. Biliary atresia B. multiple uncorrectable life-threatening congenital anomalies. thus reducing frequency of posttransplantation systemic infections. 500 MCQ’s FOR MEDICAL PROFESSIONALS BY PROF.7 days. Breast carcinoma Hemolytic uremic syndrome can be associated with cyclosporine. serum amylase remains elevated for ? B. 2632. AJAY MATHUR Gastroenterology 1334 Which of the following is a nonnucleoside purine metabolism so for 3 . Aortic aneurysm C. Approach to the Patient A. 3 . of amylase in ? Harrison’s 18th Ed. All of the above D. 72 hours A. Table 312-2 Harrison’s 18th Ed. < 50 mL / minute B. Trypsinogen. 24 hours B. or OKT3. 2631 1344 Which of the following blood test is reliable for diagnosis of A. ~ 75 % B. 2611 A. D. < 85 mL / minute Harrison’s 18th Ed. Perforated peptic ulcer Mycophenolic acid. and ovarian carcinoma. Hypertriglyceridemia B. inhibitor ? Harrison’s 18th Ed. 1342 Hyperamylasemia is found in which of the following ? Harrison’s 18th Ed. Chymotrypsin D. Pregnancy with Pancreatic Disease B. The newer lipase 1337 Patients with proven pancreatitis have spuriously low levels assays have colipase as a cofactor and are fully automated. None of the above 1338 In acute pancreatitis. Mycophenolic acid B. D. Carcinoma of esophagus B. tacrolimus. A. 2631 failure. All of the above “Tumor” hyperamylasemia is seen in Carcinoma of lung. Cyclosporine B. ~ 25 % Harrison’s 18th Ed. All of the above Harrison’s 18th Ed. 7 . Serum trypsinogen Serum amylase level may be normal if hypertriglyceridemia is present. amylase & lipase are excreted by kidney therefore are elevated in renal failure. Carcinoma of esophagus. Serum amylase C. a nonnucleoside purine metabolism inhibitor derived as a fermentation product D. the serum amylase is usually elevated within 24 hours of onset and remains .14 days In acute pancreatitis.7 days Serum amylase levels are elevated in patients with renal dysfunction when creatinine clearance is C. 9 . Carcinoma of lung A. Serum lipase D. 2632. Acute Pancreatitis C. Table 312-2 Chapter 312. < 100 mL / minute 1339 In acute pancreatitis. OKT3 D. ductal obstruction. Tacrolimus A. Phospholipase A2 Lipase is the single best enzyme to measure for diagnosis of acute pancreatitis. All of the above from several Penicillium species. 1341 Hyperamylasemia is found in which of the following ? 1335 Hemolytic uremic syndrome can be associated with ? Harrison’s 18th Ed. or pseudocyst formation. Cyclosporine Harrison’s 18th Ed. Breast carcinoma C. 2629 1343 The newer lipase assays relate best to which of the following ? A. Trypsin C. Levels usually return to normal within 7 days unless there is pancreatic ductal disruption. Table 312-2 B. 2632 B. ~ 50 % A. Morphine 1336 How much of pancreas must be damaged before maldigestion of fat and protein is manifested ? D.3 days D. < 75 mL / minute A. Diabetic ketoacidosis D. Rapamycin C.9 days < 50 mL/minute. 2632 B. 2610 1340 Hyperamylasemia is found in ? A. 36 hours 1345 Serum amylase levels are elevated when creatinine clearance C. Incomplete ductal obstruction acute pancreatitis in patients with renal failure ? Harrison’s 18th Ed. Pseudocyst formation A. ~ 90 % C. 1 . 2632. Tacrolimus C. 2632 D. serum amylase rises within ? No single blood test is reliable for the diagnosis of acute pancreatitis in patients with renal Harrison’s 18th Ed. 2631 C. All of the above C. 48 hours is less than ? Harrison’s 18th Ed. Colipase > 90% of pancreas must be damaged before maldigestion of fat & protein is manifested. D. 3000 mL of isosmotic alkaline (pH > 8. 1354 The pancreatic secretion contain about ? 1348 Enzyme trypsinogen is present in which of the following ? Harrison’s 18th Ed. serum amylase values are highly specific Harrison’s 17th Ed.Acute and Chronic Pancreatitis secretin stimulation test is abnormal ? Harrison’s 17th Ed. II A.28 ng/mL C. 1357 Which of the following about cholecystokinin (CCK) is false ? Harrison’s 18th Ed. Intestine D. I Harrison’s 18th Ed. serum amylase values > 3 times normal are highly specific.158 ng/mL enzymes & zymogens.1500 ml C. 1355 The pancreatic secretion is ? Harrison’s 18th Ed. CCK evokes an enzyme-rich secretion from pancreas of intestine in acute pancreatitis ? Harrison’s 17th Ed. Gall bladder B. Release of CCK is triggered by essential amino acids B. pH threshold for release of secretin from duodenum & B. 1358 Which of the following statements is false ? Harrison’s 18th Ed. Transverse colon D. 501 Gastroenterology MCQ’s MCQ’s FOR FORPROFESSIONALS MEDICAL MEDICAL PROFESSIONALS BY PROF.98 ng/mL Pancreas secretes 1500 . Three times normal D. Ileum D. 2632 B. Hyposmotic acidic C. Gastric acid is a stimulus for release of secretin Harrison’s 17th Ed. 2002 A. Secretin is a peptide with 27 amino acids A. 2005 B. pancreatic calcification on radiological A. 2634 1351 “Colon cutoff sign” refers to isolated distention of which part A. 1356 Which of the following about secretin is false ? Harrison’s 18th Ed. Duodenum C. 2002 B. 2002 1353 The quantity of pancreatic secretion per day is ? A. 1500 . 28 . 2634 1349 Normal serum trypsinogen level is ? A. 40 enzymes and zymogens D. 2634 1352 In chronic pancreatitis. Isosmotic acidic A. 18 . Descending colon Release of CCK from duodenum & jejunum is triggered by long-chain fatty acids. 10 enzymes and zymogens A. 3000 . and gastric acid itself. About 5000 ml exocrine function has been lost. phenylalanine. 20 enzymes and zymogens B. ~ 40 % B. 2002 B. In acute pancreatitis. Jejunum jejunum is 6. D. 2634 B. methionine). 30 enzymes and zymogens C. ~ 20 % A.3000 ml D.58 ng/mL.5 C. Ascending colon C.58 ng/mL D. 100 . III B. The normal values of serum trypsinogen are 28 . Pancreas C. All of the above Pancreas is the only organ that contains trypsinogen. Four times normal In chronic pancreatitis. essential amino acids (tryptophan. Hyposmotic alkaline B. 2634 Harrison’s 17th Ed. Release of CCK is triggered by short-chain fatty acids A. Sigmoid colon “Colon cutoff sign” refers to isolated distention of transverse colon in acute pancreatitis. 60 . Colon & electrolytes “Sentinel loop” refers to a localized ileus of jejunum in acute pancreatitis. valine. Isosmotic alkaline Harrison’s 17th Ed. AJAY MATHUR Cardiology 501 1346 In acute pancreatitis. 1000 . 1347 How much exocrine function of pancreas must be lost before 313 .4500 ml Secretin stimulation test for assessing pancreatic exocrine function is abnormal when >60% of D. Five times normal vertebra. IV C. 2002 if they are more than ? A. radiographic pancreatic calcification is superimposed on 2nd lumbar D. Release of CCK is triggered by gastric acid C. ~ 60 % C. 2634 1350 “Sentinel loop” refers to a localized ileus of which part of intestine in acute pancreatitis ? A. ~ 15 % Harrison’s 18th Ed. Bile salts stimulate pancreatic secretion examination is superimposed on which lumbar vertebra ? . Secretin stimulates secretion of pancreatic juice rich in water D.0) fluid / day containing ~20 D. Two times normal C. lipolytic. Calcitonin gene . Bile salts activate phospholipase A and cholesterol esterase. 2634 C. Lipolytic . and galanin. and proteolytic enzymes. 2634 Gastric acid is the stimulus for release of secretin which stimulates secretion of pancreatic juice rich in water and electrolytes. A. Proteolytic over pancreatic secretion D. None of the above B. Harrison’s 18th Ed. neuropeptide Y. Vasoactive intestinal peptide (VIP) is a CCK agonist The pancreas secretes amylolytic. Maximal acid output 1369 Enzyme that cleaves lysine-isoleucine bond of trypsinogen to D. 2634 glucagon. Cystic fibrosis transmembrane conductance regulator D. cystic fibrosis transmembrane conductance regulator (CFTR) controls A. Amylolytic B. All of the above A. 20 to 30 mmol/day Lipolytic enzymes secreted by pancreas include lipase. Arginine-Lysine bond Harrison’s 18th Ed. None of the above B. Trypsokinase C. Cholesterol esterase Harrison’s 16th Ed. 2634 1363 Bicarbonate in pancreatic secretion is related to ? A. Zymogens 1362 Which of the following correlates best between stimulation with B. 2634 D. 2634 C. A. Inhibit lipase in isolation 1360 In acini and in ducts. Proteogens secretin and the pancreatic mass ? C. Maximal bicarbonate output form trypsin is ? Harrison’s 18th Ed. Enterokinase A. which hormone causes the cells to add B. Chymogens A. 120 to 300 mmol/day 1366 Which of the following about bile salts is false ? D. Bile salts activate phospholipase A and cholesterol esterase. Bile salts activate phospholipase A & cholesterol esterase A. Colipase binds to lipase & prevents inhibition by bile salts water and bicarbonate to pancreatic fluid ? Harrison’s 18th Ed. Gastrin 1367 Which of the following statements is false ? Harrison’s 18th Ed. B. Threonine-Lysine bond A.secretion by pancreas is dependent secretin and CCK 1365 The lipolytic enzymes secreted by pancreas are ? Harrison’s 18th Ed. Arginine-Threonine bond 1364 Pancreas secretes which of the following enzymes ? C. All of the above 1370 Bond that is cleaved to form trypsin from trypsinogen is ? Harrison’s 18th Ed. Lysine-isoleucine bond chloride and bicarbonate fluxes. Glucagon D.502 MCQ’s FOR MEDICAL PROFESSIONALS BY PROF. peptide YY. D. C. but colipase of pancreatic secretion. A. calcitonin gene–related peptides. Phospholipase A gastric acid is ? C. phospholipase A & cholesterol esterase. Maximal sodium output Proteolytic enzymes are secreted as inactive precursors called zymogens. 1895 D. All of the above 1368 Proteolytic enzymes secreted as inactive precursors are called ? Pancreatic exocrine secretion is influenced by inhibitory neuropeptides like somatostatin. enkephalin. Colipase binds to lipase 1361 Pancreatic exocrine secretion is influenced by ? Harrison’s 18th Ed. 2634 D.related peptides D. About 500 mmol/day Harrison’s 18th Ed. AJAY MATHUR Gastroenterology B. Somatostatin D. H2O and HCO3. pancreastatin. C. B. Maximal chloride output C. 50 to 100 mmol/day C. 2634 In pancreatic ductal cells. binds to lipase and prevents this inhibition. Parasympathetic nervous system exerts significant control C. Lipase 1359 The quantity of bicarbonate from pancreas needed to neutralize B. Neuropeptide Y Bile salts inhibit lipase. Insulin C. 2634 A. Somatostatin D. Duodenokinase Harrison’s 18th Ed. 2634 B. Bile salts inhibit lipase B. B. Secretin Bile salts inhibit lipase in isolation. Colipase in pancreatic secretion binds to lipase and prevents this inhibition. CCK evokes an enzyme-rich secretion from the pancreas. Insulin D. Gastrokinase B. Activate phospholipase A and cholesterol esterase A. 2634 Vagal stimulation leads to release of VIP which is a secretin agonist. pancreatic polypeptide. All of the above C. Amylogens Harrison’s 18th Ed. 2006 1380 Which of the following is the most common cause of acute A. Compared with the anionic isoenzyme. and the alpha1. Risk factors for post-ERCP pancreatitis include minor papilla sphincterotomy.8%). Alcohol abuse A. and endoscopic trainee involvement. AJAY MATHUR Cardiology 503 1371 Enzyme enterokinase is found in ? 1377 Which of the following is the most common PRSS1 mutation ? Harrison’s 18th Ed.. Alpha1. the D. Genetic mutations such as those in CFTR and SPINK1 genes are frequent in HIV-positive patients 1373 Protease inhibitors are found in ? with acute pancreatitis. the function of which can be lost by D. Harrison’s 18th Ed. Stomach B. Duodenal mucosa B. Cause of acute pancreatitis include gallstones (30-60%). seen in most patients with hereditary pancreatitis. pancreatic secretory trypsin inhibitor (PSTI) and serine D. R122K Enterokinase is an enzyme found in duodenal mucosa. It cleaves lysine-isoleucine bond of trypsinogen R122H is the most common PRSS1 mutation observed worldwide.and alpha2-globulin fractions of plasma. Pancreatic acinar cells Harrison’s 18th Ed. Duodenum A.3-3. 1372 Which of the following can lyse and inactivate trypsin ? 1378 CFTR and SPINK1 genetic mutations causing acute pancreatitis Harrison’s 18th Ed. R122A B. hypertriglyceridemia (1. 2635 B. Ileal mucosa D. and enzyme y can also lyse and inactivate trypsin. >2 contrast injections into the 1376 Which of the following is not a type of trypsinogen in human pancreatic duct. The goal is to reduce fasting plasma triglycerides to below 500 mg/ more easily and is more resistant to autolysis. 1374 Kazal type 1 (SPINK1) is best related to ? Harrison’s 17th Ed. dL to prevent the risk of acute pancreatitis. CFTR controls chloride and bicarbonate fluxes. ERCP (5-20%) and drug-related (2-5%). kazal type 1 (SPINK1) that prevents conversion of trypsinogen to trypsin. a specific trypsin inhibitor. Mesotrypsinogen D. R122C C.371:143-52 B. 503 Gastroenterology MCQ’s MCQ’s FOR FORPROFESSIONALS MEDICAL MEDICAL PROFESSIONALS BY PROF. as cationic trypsinogen (PRSS1). > 2 contrast injections into pancreatic duct Pancreas synthesises SPINK1. Gall bladder Harrison’s 18th Ed. Anionic trypsinogen C. In pancreatic ductal cells. Duodenum D.371:143-52 1381 Risk factors for post-ERCP pancreatitis include ? A. All of the above mutation. COPD patients D. Duodenum contains a peptide CCK-releasing factor that is involved in stimulating CCK release. . Jejunal mucosa C. anionic trypsinogen (PRSS2) & Hypertriglyceridemia can cause acute pancreatitis in 1. sphincter of Oddi dysfunction. Serine protease inhibitor C. 1375 SPINK1 is synthesised in ? Lancet 2008. Telotrypsinogen A. Anti-inflammatory cytokine B. Thalassemia patients C. SPINK1 and CFTR mutations together may cause pancreatitis. > 750 mg / dL D. Pancreatic hyperstimulation pancreatitis ? Harrison’s 18th Ed.e. alcohol (15-30%). Drugs Autodigestion of the pancreas is prevented by the packaging of proteases in precursor form and by the synthesis of protease inhibitors. 2635 B. > 250 mg / dL B. chymotrypsin c. > 500 mg / dL C. Pancreas C. Stomach C. prior history of post-ERCP pancreatitis. Sphincter of Oddi dysfunction C.132:1557-1573 A. 2634 are frequent in ? A. Age < 60 years D. All of the above C. are found in the acinar cells.3 .8% of cases when serum triglyceride mesotrypsinogen (PRSS3). Chymotrypsin c A. > 1000 mg / dL Three different trypsinogens in human pancreatic juice have been designated according to their electrophoretic mobility.3. age <60 years. that prevent autodigestion of pancreas. 2634 1379 CCK-releasing factor (CCK-RF) is present in ? A.and alpha2-globulin fractions of plasma B. Jejunum Protease inhibitors. Mutations in PRSS1 gene are to form trypsin. Mesotrypsin Lancet 2008. Alcohol D. 2635 A. HIV-positive patients D. i. Leukemia patients Mesotrypsin. ERCP protease inhibitor. 2635 B. R122H D. Cationic trypsinogen B. Enzyme y B. 2634 Gastroenterology 2007. Pancreatic secretions A.132:1557-1573 1382 What level of hypertriglyceridemia causes acute pancreatitis ? Harrison’s 18th Ed. All of the above C. Gallstones C. pancreatic juice ? Gastroenterology 2007. Gastric mucosa A. All of the above pancreatic secretions. cationic trypsinogen autoactivates levels are usually > 1000 mg/dL. Hypertriglyceridemia can precede & cause pancreatitis phase is due to effects of activated proteolytic enzymes & cytokines released by inflamed pancreas B. 15 to 20% Harrison’s 17th Ed. acts as a required cofactor for LPL in this reaction. Histamine 1385 Deficiency of which of the following have an increased D. Histamine abdominal pain due to acute pancreatitis. Monocyte chemotactic protein (MCP-1) levels measured early in the course of acute pancreatitis are an accurate predictor of severity and death. a determinant of the severity of the inflammatory response. 2636 A. chemoattraction & sequestration of neutrophils in pancreas resulting in intrapancreatic inflammatory reaction. 2636 C. 2009 enzyme activation & acinar cell injury. About 25% A. thiazides and propanolol. MCP-1 Patients with deficiency of apolipoprotein CII have an increased incidence of pancreatitis. 2007 D. Apolipoprotein CII activates lipoprotein lipase. ApoB-100 Harrison’s 18th Ed.504 MCQ’s FOR MEDICAL PROFESSIONALS BY PROF. Vasoactive substances Both LPL & apoC-II deficiency usually present in childhood with recurrent episodes of severe C. Four susceptibility genes have been identified that can increase the susceptibility and/or modify the severity of pancreatic injury in acute pancreatitis. Apolipoprotein CII A. Third A. hypertriglyceridemia 1389 Cathepsin B is best related to ? C. ApoC-I A. MCP-1 2518 G allele is a risk factor for severe acute pancreatitis. ApoC-II B. Second phase involves activation. Phase 2 1394 Which of the following is false about abdominal pain of acute C. pancreatic secretory trypsin inhibitor (SPINK1). Vasoactive substances Drugs that can elevate serum Tg are estrogens. Patients with pancreatitis & hypertriglyceridemia have Harrison’s 18th Ed. MCP-1 levels transferred to circulating chylomicrons from HDL. Triglycerides of chylomicrons are hydrolyzed by LPL. and N291). 1387 What percentage of acute pancreatitis are drug-related ? R122Hm. SPINK1 D. >80% patients of acute pancreatitis do not have on distant organs. Apolipoprotein A-II C. Bradykinin peptides B. Thiazide diuretics Harrison’s 18th Ed. 2636 MCP-1 2518 G allele polymorphism is a gain-of-function promoter that increases MCP-1 expression. Phase 1 B. Progesterone B. CFTR and monocyte chemotactic protein (MCP-1). All of the above D. MCP-1 2518 G allele 1388 Activation. MCP-1 2517 G allele C. Apolipoprotein CII activates lipoprotein lipase. All of the above incidence of pancreatitis ? Harrison’s 18th Ed. Phase 3 pancreatitis ? Harrison’s 18th Ed. 2636 D. Fat necrosis D. Apolipoprotein A-I B. ApoC-II. which is D. 2 to 5% and a promoter of organ failure. vitamin A. Vitamin A 1390 In pancreatitis. and free fatty acids are released. Activation of elastase & phospholipase Fasting Tg levels of < 300 mg/dL pose no risk of pancreatitis. AJAY MATHUR Gastroenterology 1383 Which of the following statements is false ? Pancreatitis evolves in three phases.5 % of acute pancreatitis are drug-related. Phase 4 A. 2636 D. Colicky . chemoattraction & sequestration of neutrophils in D. 1386 Deficiency of which of the following poses an increased incidence of pancreatitis ? 1392 Which of the following susceptibility gene is a determinant of Harrison’s 18th Ed. A. Bradykinin peptides D. MCP-1 2516 G allele ~ 2 . CFTR C. 2635 1391 Which of the following is an accurate predictor of severity & A. 2635 pathologic process of acute pancreatitis. C. MCP-1 may be an important inflammatory mediator in the early Harrison’s 18th Ed. which is important in clearing chylomicrons from bloodstream. Chemoattraction of neutrophils 1384 Drugs that can elevate serum triglycerides include all except ? Harrison’s 18th Ed. B. Beta-blockers A. ApoA-I death when measured early in the course of acute pancreatitis ? B. PRSS1 B. Zymogen activation D. 6 to 12% 1393 Which of the following is a risk factor for severe acute pancreatitis ? C. Fasting Tg levels of < 500 mg/dL pose no risk of pancreatitis B. These are cationic trypsinogen mutations (PRSS1m. B. A. C. Initial phase is characterized by intrapancreatic digestive Harrison’s 17th Ed. 2641 Zymogen activation is mediated by lysosomal hydrolases (cathepsin B) which become co-localized with digestive enzymes in intracellular organelles leading to pancreatic acinar cell injury. A. 2635 severity of inflammatory response in pancreatitis ? Harrison’s 18th Ed. MCP-1 2519 G allele pancreas occur in which phase of pancreatitis ? Harrison’s 18th Ed. 2636 preexisting abnormalities in lipoprotein metabolism A. cellular injury results in liberation of ? C. A. Any of the above 1402 Pearson syndrome is characterized by ? Harrison’s 18th Ed. Intestinal ischemia A. 2636 C. Pancreatitis A. Right-sided D. Subcutaneous fat necrosis D. None of the above C. Pancreatic pseudocyst D. DIC B. Serum amylase is higher in gallstone pancreatitis A. Increased output of adrenal glucocorticoids & catecholamines A. Gallstones with diameter of >8 mm remain in gallbladder. Burns 1400 Cullen’s sign of severe necrotizing pancreatitis is due to ? B. 111. C. 2636 C. Bilateral Gallstones with a diameter of ~ 5 mm can migrate in bile duct & trigger acute pancreatitis. Tissue catabolism of hemoglobin 1395 Abdominal pain due to pancreatitis may have which of the B. Any of the above Pleural effusion in acute pancreatitis is most frequently left-sided. Pyoperitoneum following location ? Harrison’s 18th Ed. Hemoperitoneum . Retroperitoneal abscess D. Cause of hypocalcemia includes a combination of low albumin. ~ 25% of acute pancreatitis will have recurrence A. Epigastric Turner’s sign of severe necrotizing pancreatitis refers to a blue-red-purple or green-brown discoloration of flanks and reflects tissue catabolism of hemoglobin. Table 13-2 C. 420 C. 2636 Abdominal pain of acute pancreatitis is steady & boring in character. Pancreatic carcinoma D. caused by sporadic deletion of several mtDNA genes. 2636 B. 2636 Exudation of blood & plasma proteins into retroperitoneal space due to activated proteolytic enzymes in acute pancreatitis is termed as retroperitoneal burn. AJAY MATHUR Cardiology 505 B. 2636 B. Risk of acute pancreatitis is greater with gallstone <5 mm than larger stones 1397 Pleural effusion in acute pancreatitis is most frequently ? Harrison’s 18th Ed. C. or C. All of the above 1406 Hypocalcemia may occur in which of the following conditions ? Fat necrosis associated with pancreatic disease is secondary to circulating lipases and is seen in Harrison’s 18th Ed. A. hyperphosphatemia. Right Upper Quadrant D. 362 pancreatic carcinoma and acute & chronic pancreatitis. Chronic pancreatitis and increased output of adrenal glucocorticoids and catecholamines. rhabdomyolysis. Chapter e18 1396 Exudation of blood & plasma proteins into retroperitoneal A. Serum lipase higher in alcohol-associated pancreatitis B. D. Thromboembolism D. Retroperitoneal tan Pearson syndrome is characterized by diabetes mellitus from pancreatic insufficiency with pancytopenia & lactic acidosis. All of the above B. and impaired PTH secretion. Located in periumbilical region 1401 Turner’s sign of severe necrotizing pancreatitis is due to ? Harrison’s 18th Ed. Intestinal ischemia pancreatitis. Cystic fibrosis is a cause of recurrent pancreatitis B. 505 Gastroenterology MCQ’s MCQ’s FOR FORPROFESSIONALS MEDICAL MEDICAL PROFESSIONALS BY PROF. More intense in supine position D. tumor lysis. All of the above B. Diabetes mellitus from pancreatic insufficiency space due to activated proteolytic enzymes in acute pancreatitis is termed as ? B. Pancytopenia Harrison’s 18th Ed. A. tissue deposition of calcium. Left Upper Quadrant D. D. 2636 Erythematous skin nodules in acute pancreatitis is due to subcutaneous fat necrosis. A. Retroperitoneal burn 1403 Which of the following about pancreatitis is false ? Harrison’s 18th Ed. Pancreatic isoamylase & lipase remain elevated for 7-14 days Harrison’s 18th Ed. Vasculitis C. Lactic acidosis A. DIC Hypocalcemia may occur with severe tissue injury like burns. 1404 Which of the following about acute pancreatitis is false ? Harrison’s 18th Ed. Tumor lysis Harrison’s 18th Ed. C. Decreased insulin release 1399 Fat necrosis associated with pancreatic disease is seen in ? B. Acute pancreatitis Hyperglycemia in acute pancreatitis is due to decreased insulin release. Left-sided C. increased glucagon release C. None of the above C. All of the above B. Radiates to back Cullen’s sign in severe necrotizing pancreatitis refers to a faint blue discoloration around umbilicus as the result of hemoperitoneum. 2636 1398 Erythematous skin nodules in acute pancreatitis is due to ? A. All of the above 1405 Hyperglycemia in acute pancreatitis is due to ? Harrison’s 18th Ed. Increased glucagon release Harrison’s 18th Ed. D. Retroperitoneal quinsy D. myocardial infarction. 1412 Differential diagnosis of acute pancreatitis include ? 1418 Bedside Index of Severity in Acute Pancreatitis (BISAP) Harrison’s 18th Ed. 3 C. Obesity (BMI > 30) C. BMI > 30. Imrie C. A. mesenteric vascular occlusion. Leukocytosis D. 2637. 2637 A. A. 2639 1419 Noninfectious etiology of systemic inflammatory response A.0 mg/dL D. Table 313-2 peripancreatic fluid collection and necrosis of one-third of A. acute cholecystitis. All of the above 1415 Which of the following determines outcome in majority of difficult to manage cases of acute pancreatitis ? 1409 Risk factor for severity in acute pancreatitis is ? Harrison’s 17th Ed. 5 D. Perforated viscus Harrison’s 18th Ed. 1413 Which of the following is true in diabetic ketoacidosis ? Harrison’s 18th Ed. Imrie & Apache II are multiple factor scoring systems for predicting outcome of acute C-reactive protein > 150 mg/L. 4 C. Gastrointestinal bleeding (>500 mL/day) A. FP abdomen B. Ranson B. 6 BISAP or bedside index of severity in acute pancreatitis includes (B) Blood urea nitrogen (BUN) >22 mg%. Hyperglycemia D. acute intestinal obstruction. Hypoxemia (arterial PO 2 <=60 mm Hg) cholecystitis from acute pancreatitis ? D. incorporates five clinical and laboratory aortic aneurysm. Hct > 44% & admission Ranson. 2637. (I) Impaired mental status. 2637 incorporates how many clinical and laboratory parameters ? A.e. All of the above Acute pancreatitis leading to respiratory failure i. C. renal colic. Elevated serum LDH levels (> 500 U/dL) Serum lipase level is not elevated in DKA. Connective tissue disorders with vasculitis B. AJAY MATHUR Gastroenterology 1407 Markers of poor prognosis in severe pancreatitis is ? D. Hypocalcemia Pain of biliary tract origin is more right-sided or epigastric than periumbilical. APACHE II D. 2637. Hematocrit > 44% C. pneumonia & diabetic ketoacidosis. connective tissue disorders with vasculitis. Pancreatic isoamylase levels not elevated A. Systolic blood pressure < 90 mmHg C. Serum lipase not elevated . Impaired mental status C. 2008 Azotemia is a significant risk factor for mortality. 9 Differential diagnosis of acute pancreatitis includes perforated viscus. dissecting Bedside Index of Severity in Acute Pancreatitis (BISAP).what is the CT severity index ? Harrison’s 18th Ed. pancreatitis. PO2 <= 60 mmHg B. Pancreatitis C. Table 313-2 A. 2008 Harrison’s 18th Ed. All of the above Indicators of a severe attack of pancreatitis are age > 70 years. 2637 Diabetic ketoacidosis is accompanied by abdominal pain & elevated total serum amylase levels. Blood urea nitrogen (BUN) > 22 mg % A. All of the above Harrison’s 18th Ed. 7 1411 Indicators of a severe attack of pancreatitis are all except ? 1417 Multiple factor scoring system for acute pancreatitis is ? Harrison’s 17th Ed. Upper GI Endoscopy 1408 Laboratory studies in acute pancreatitis may show ? C. (S) SIRS: 2/4 present. (A) Age >60 years. 5 D. Radionuclide scanning Harrison’s 18th Ed. 2008 A. (P) Pleural effusion D. Azotemia 1414 Which investigation is most helpful in differentiating acute C. Age > 60 years B.506 MCQ’s FOR MEDICAL PROFESSIONALS BY PROF. ileus is usually absent. B. Pao2 < 60 mmHg pancreas . 7 D. All of the above C. Elevated total serum amylase levels syndrome (SIRS) include which of the following ? Harrison’s 18th Ed. Admission C-reactive protein > 150 mg/L D. Body mass index (BMI) < 25 B. Serum creatinine >2. PO2 < 60 mmHg determines outcome in majority of difficult to manage cases. 2228 B. Dissecting aortic aneurysm A. Table 313-3 B. parameters obtained within the first 24 hours of hospitalization. 2639 B. Age > 70 years A. Serum Lipase B. 2008 Harrison’s 17th Ed. All of the above Harrison’s 17th Ed. Age > 60 years B. Presence of three or more of these factors is associated with substantially increased risk for in-hospital mortality in acute pancreatitis. 1410 Which of the following is not included in the bedside index of severity (BISAP) in acute pancreatitis ? 1416 Abdominal CT of acute pancreatitis patient showed one Harrison’s 18th Ed. Sonography & radionuclide scanning are helpful in diagnosis of cholelithiasis & cholecystitis. Comorbid disease D. Serum lipase D. A.354:2142-50 B. Long-lasting fast A. anaphylaxis. Gall bladder mucus A. VHL C. Trypsinogen activation peptide 1430 Which of the following genes may predict severity of acute D. cardiac 1427 Biliary sludge is made up of ? tamponade. trauma. dissecting or ruptured aortic aneurysm. alcoholism. CT abdomen B. All of the above C. occult hemorrhage. MEN-1 B. Calcium bilirubinate granules Lancet 2008. Old age N Engl J Med 2006.354:2142-50 B.354:2142-50 A. APACHE II score >=8 C.354:2142-50 A. Urine Trypsinogen activation peptide (TAP) B. Cholesterol crystals 1420 SOFA score stands for ? B. 507 Gastroenterology MCQ’s MCQ’s FOR FORPROFESSIONALS MEDICAL MEDICAL PROFESSIONALS BY PROF. RET N Engl J Med 2006. AJAY MATHUR Cardiology 507 B. Abdominal CT & MRI D.354:2142-50 amylase and lipase is ? N Engl J Med 2006. All of the above 1432 Biliary sludge is associated with which of the following ? Lancet 2008. Surgical organ failure assessment Biliary sludge refers to a viscous bile suspension that contains cholesterol crystals and calcium C. tumor-associated lactic acidosis. Ranson’s & APACHE II scores B. C-reactive protein level >150 mg/dl D. Stones in the distal bile duct B. Fluid collections with very high levels of pancreatic enzymes B. burns. Transabdominal ultrasonography A. MRI abdomen C. 371: 143-52 1425 Recurrent pancreatitis in the absence of biliary disease. Gallstones and sludge N Engl J Med 2006. All of the above B. and toxic or metabolic causes suggests ? A. Transabdominal ultrasound D. Trypsinogen-2 C. All of the above 1429 Transabdominal ultrasonography is insensitive for detecting ? N Engl J Med 2006. Genetic susceptibility 1433 Risk factors for post-ERCP pancreatitis include all except ? D. adrenal insufficiency. Serum amylase C.354:2142-50 B. D. Number of cannulation attempts of papilla . PMN elastase >300 µg/liter 1431 Recognized markers of risk of severe acute pancreatitis include D. MRI abdomen C. pulmonary embolism.354:2142-50 1422 Which of the following tests is more specific for the diagnosis of acute pancreatitis ? A. myocardial infarction. Ascites or pleural effusions D. Pancreas divisum C. 371: 143-52 C. 371: 143-52 1426 Test that identifies early pancreatic duct disruption is ? A. MCP-1 A. Female sex A. All of the above D. Bile-duct dilatation A. ERCP D. Total parenteral feeding N Engl J Med 2006. Adrenal insufficiency B. Stones in the proximal bile duct C. Septic organ failure assessment D. Urinary TAP >35 nmol/liter all except ? N Engl J Med 2006.354:2142-50 1424 Pancreatic-duct disruption is suspected when ? N Engl J Med 2006. Any of the above Lancet 2008.354:2142-50 1423 At 24 hours after admission. Pseudocysts C. the most sensitive & specific predictor of severe acute pancreatitis is ? A. Lancet 2008. Trypsinogen-4 pancreatitis ? N Engl J Med 2006. High reticulocyte index D. Elevated C-reactive protein A. CT abdomen C. Distal bile duct obstruction B. Duct-obstructing masses D.354:2142-50 B. 371: 143-52 and drug overdose. Pulmonary embolism C. Symptomatic organ failure assessment bilirubinate granules embedded in strands of gall bladder mucus. Sequential organ failure assessment 1428 Which test is more sensitive for identifying gallstones and sludge and for detecting bile-duct dilatation ? 1421 Test that is more specific for acute pancreatitis than serum N Engl J Med 2006. All of the above Noninfectious etiologies of SIRS include pancreatitis. postcardiopulmonary bypass syndrome. Obesity C. a protease inhibitor is utilized in the management of pancreatitis. ~ 95 % D. Nafamostat D. ~ 10 % A. Metronidazole C. ~ 90 % remove bile duct stones. Antibiotic With single organ system failure. 500 mg thrice daily system failure is ? for 7 days. A. C. It fits in PAF receptors on the surface of cells & blocks activation of these receptors by PAF itself. Pentamidine Necrotizing pancreatitis occurs in ~10% of all patients with acute pancreatitis. Asthma B. Poor emptying of pancreatic duct after opacification C. 50 . such as during hemodialysis therapy A. Glaucoma 1439 In what proportion of acute pancreatitis. oral intake is started by considering all of the pancreatitis is ? following factors except ? Harrison’s 18th Ed. Patient is hungry C. ~ 75 % Risk of acute pancreatitis is higher when ERCP is done to treat Oddi sphincter dysfunction than to D. 2639 A. 1434 The median prevalence of organ failure in necrotizing 1440 In pancreatitis. 2641 A. ~ 50 % . 2640 D. Protease inhibitor C. 5 . Fungicide Platelet activating factor (PAF) enhances polymorphonuclear leukocyte (PMN) superoxide production. 357 C.10 % (HD). disseminated Harrison’s 17th Ed. the disease is self-limited and subsides after opacification. ~ 25 % C. ~ 10 % B.90%) with acute pancreatitis. Ciprofloxacin B. Platelet-activating factor inhibitor D. D. ~ 50 % B. ~ 25 % A. Elevated levels of serum amylase/lipase The median prevalence of organ failure is 54% in necrotizing pancreatitis. ~ 50 % B.20 % C. AJAY MATHUR Gastroenterology D. ~ 10 % pancreatitis ? Harrison’s 18th Ed. plasmapheresis. B. number of cannulation attempts of papilla before success & poor emptying of pancreatic duct In most patients (85 . Organ dysfunction D. ~ 50 % C. Aliskiren pancreatitis ? Nafamostat. and extracorporeal circulation (ECC). all essential components in the pathophysiology of multiple- necrotizing pancreatitis ? organ failure. ~ 25 % A. usually within three to seven days after treatment is instituted. Alopecia Harrison’s 18th Ed. Aztreonam 1436 The mortality in acute pancreatitis with multisystem organ Current recommendation in necrotizing acute pancreatitis is imipenem cilastin. It may cause hyperkalemia. Other risk factors for post-ERCP pancreatitis include young age. 1437 What proportion of patients with acute pancreatitis have CD11b expression & elastase release. ~5% 1442 Lexipafant is best related to ? Harrison’s 18th Ed.20% of patients with acute pancreatitis. 12 . Harrison’s 18th Ed. 1435 The mortality in acute pancreatitis with single organ system failure is ? 1441 Which of the following antibiotic is recommended in Harrison’s 18th Ed.45 % 1444 Besides its use in acute pancreatitis. and cardiopulmonary bypass.70 % Harrison’s 17th Ed. It inhibits aldosterone-induced proteases that activate ENaC by proteolytic cleavage. Peripheral arterial disease A. Tacrolimus 1438 Necrosis is present in what percentage of patients with acute D. ~ 75 % C. Inflammatory changes on CT scan or persistent elevations in serum amylase/lipase may not resolve for weeks to months & should not discourage feeding a hungry asymptomatic patient of pancreatitis. female sex. 2639 A. 2639 necrotizing acute pancreatitis ? Harrison’s 17th Ed. ~5% A. Imipenem cilastin D.508 MCQ’s FOR MEDICAL PROFESSIONALS BY PROF. 2639 Harrison’s 18th Ed. 2010 intravascular coagulation. 2010 Necrosis is present in 12 . ~5% 1443 Which of the following is used in the management of B. Lexipafant is also used in ? D. the disease is self- limited and subsides spontaneously ? C. B. Lexipafant (BB-882) is a potent & specific PAF antagonist. ~ 25 % B. ~ 25 % Inflammatory agent PAF is implicated in the causation of pancreatitis and asthma. ~ 50 % D. the mortality is 3–10% but increases to 47% with multisystem organ failure. Harrison’s 18th Ed. 2010 A. Resolution of abdominal pain B. spontaneously. 25 . 2640 B. Fibrous tissue ~25% of patients who have had an attack of acute pancreatitis have a recurrence. 7–10 days after the onset develop over 4-6 weeks after acute pancreatitis. fluid. 2011 Harrison’s 18th Ed.6 weeks D. occult biliary tract disease should be looked at. hypertriglyceridemia. illness and may fluctuate markedly. ~ 50 % A. 12 to 24 weeks Aprotinin & gabexate mesilate are broad spectrum antiprotease drugs that reduce pancreatic damage but have no effect on mortality rate in pancreatitis. The walls consist of necrotic tissue. 509 Gastroenterology MCQ’s MCQ’s FOR FORPROFESSIONALS MEDICAL MEDICAL PROFESSIONALS BY PROF. Gram-positive bacteria of hematogenous origin D. Do not have an epithelial lining Enteral-feeding with a nasojejunal tube has fewer infectious complications than with total parenteral D. 2 weeks C. PEG C. None of the above nutrition (TPN) and is the preferred method of nutritional support. 2641 B. Pseudocysts of the pancreas are extrapancreatic collections of pancreatic fluid containing pancreatic enzymes and a small amount of debris. All of the above D. 2641 B. 2641 A. enteral feeding helps to maintain integrity of the intestinal tract during severe acute pancreatitis. Their walls consist of necrotic tissue. Feeding with a nasogastric tube A. pancreatic duct stones. Necrotizing pancreatitis is always associated with peripancreatic necrosis involving the fat around the pancreas. Mostly located in body or tail of pancreas B. Granulation tissue D. Gram-negative bacteria of alimentary origin C. pancreatic cancer. Pancreatic cancer B. Any of the above If a cause could not be found in patients with recurrent pancreatitis. granulation tissue. . In contrast to true cysts. Granulation tissue D. and fibrous tissue. hereditary pancreatitis. These include microlithiasis. 2641 1454 The lining of a pancreatic pseudocyst is ? Harrison’s 18th Ed. After 3 to 6 weeks. acute pancreatitis ? Harrison’s 18th Ed. Microlithiasis A. Squamous epithelium B. of acute pancreatitis. most frequently with gram-negative bacteria of alimentary origin. Gabexate mesilate B. pseudocysts do not have an epithelial lining. ~ 75 % B. AJAY MATHUR Cardiology 509 1445 Aprotinin is best related to which of the following drugs ? 1451 “Walled-off Necrosis” occurs how many weeks after necrotizing Harrison’s 18th Ed. ~ 25 % Harrison’s 18th Ed. 2641 1452 Which of the following is false about pancreatic pseudocysts ? A. pancreas divisum. ampullary tumors. 3 . Also. adjacent to pancreas. Necrotic tissue C. Gram-negative bacteria of hematogenous origin Pseudocysts of pancreas are collections of tissue. 1455 After acute pancreatitis. 1448 Which of the following occult biliary tract diseases can lead to granulation tissue. Cuboidal epithelium C. pseudocysts of pancreas develop over a period of ? 1449 Necrotic pancreas becomes secondarily infected with ? Harrison’s 17th Ed. cystic fibrosis. and tumor. Preceded by pancreatitis in 90% of cases A. choledochocele. Lining of a pancreatic pseudocyst is granulation tissue from pancreatic duct leakage. 2641 A. 6 to 12 weeks D. drugs. 1 . 6 weeks Serum amylase level is elevated in 75% of pseudocysts of pancreas at some point during their Pancreatic abscess following acute pancreatitis develops usually after 6 weeks.2 weeks A. stricture. Serum amylase level is mostly normal D.3 weeks B. Lexipafant Harrison’s 18th Ed. 2008 A. Abdominal pain is the usual presenting complaint C. “Walled-off necrosis” refers to this conjoint structure. Calcitonin A. Total parenteral nutrition (TPN) Harrison’s 18th Ed. Pancreas divisum C. 2642 Harrison’s 17th Ed. Gram-positive bacteria of alimentary origin B. sphincter of Oddi dysfunction. 2 . 1447 What proportion of patients of acute pancreatitis have a recurrence ? Harrison’s 18th Ed. the pancreatic necrosis and peripancreatic fat necrosis fuse 1446 Preferred method of nutritional support in patients of together encapsulated by fibrous tissue. 1 to 3 weeks C. 1 weeks B. 2641 B. ~ 90 % C. Enteral-feeding with a nasojejunal tube B. 1456 Which of the following about pseudocysts of pancreas is false ? 1450 Pancreatic abscess following acute pancreatitis develops after ? Harrison’s 18th Ed. 4 . 2640 pancreatitis ? A. Extrapancreatic C. Octreotide C. 3 weeks D. necrotizing pancreatitis is ? Harrison’s 18th Ed. 3 to 6 weeks D.4 weeks C. 2641 1453 Walls of pseudocysts consist of ? A. Collections of pancreatic fluid with pancreatic enzymes D. debris. pancreatic enzymes & blood that Necrotic pancreas becomes secondarily infected in 20-35% of patients. All of the above Pseudocysts of pancreas do not have an epithelial lining. D. and fibrous tissue. All of the above A. Sentinel chronic pancreatitis event peritonitis. Inferior pancreatic duodenal artery B. an internal fistula may develop between the pancreatic duct and the pleural space. Cigarette smoke leads to an increased susceptibility to pancreatic self-digestion area limited by optic disk & macula. 2643 B. Hepatic artery D. Hemorrhage A. Chronic pancreatitis is characterized by irreversible damage to pancreas. 1460 Which of the following statements is false for Purtscher’s 1466 There is a strong association of which of the following and retinopathy ? chronic pancreatitis ? Harrison’s 18th Ed. Obesity Purtscher’s retinopathy in acute pancreatitis is due to occlusion of posterior retinal artery with There is a strong independent. Risk of malignancy C. Pancreatic stellate cells (PSC) C. dose-dependent association of smoking and chronic and recurrent aggregated granulocytes. Splenic artery A. 2012 Harrison’s 18th Ed. Prolonged fasting D. 1459 Which artery most frequently forms ‘Pseudoaneurysm’ in acute pancreatitis ? 1465 Which of the following is best related to chronic pancreatitis ? Harrison’s 18th Ed. Abdominal pain C. >10000 U/L B. 2643 A. 2643 A. Smoking B. inferior pancreatico-duodenal and superior pancreatico-duodenal arteries can be affected. AJAY MATHUR Gastroenterology 1457 Significant number of pancreatic pseudocysts resolve 1463 When main pancreatic duct is disrupted posteriorly. 1461 The fluid in true pancreatic ascites usually has an amylase 1467 Which of the following plays a key role in the development of concentration of ? chronic pancreatitis ? Harrison’s 17th Ed. 1458 Complication of a pancreatic pseudocyst is ? Harrison’s 18th Ed. Cotton wool spots & hemorrhages in optical fundus C. rupture. 2642 1464 Cardinal complications of chronic pancreatitis is ? A. All of the above Splenic artery is most often involved. Acinar epithelial cells D. 2643 A. and predisposes to dysregulation of duct cell CFTR function. Budd-Chiari syndrome Harrison’s 18th Ed. 2643 shift toward fibrogenesis in the case of chronic pancreatitis. Retroperitoneum D. All of the above 1462 Differential diagnosis of pancreatic ascites is ? Pancreatic stellate cells (PSC) play a role in maintaining normal pancreatic architecture that can Harrison’s 18th Ed. All of the above C. Constrictive pericarditis pathogenesis of chronic pancreatitis ? C. Optical fundus shows cotton-wool spots & hemorrhages confined to an acute pancreatitis. >15000 U/L C. 2642 Harrison’s 18th Ed. Superior pancreatic duodenal artery C. Sudden and severe loss of vision B. Sequential chronic pancreatitis event . pressure on other viscera. Islet cells of Langerhans B. C. Tuberculous peritonitis 1468 Which of the following hypothesis describes events in the B. Fluctuating symptomatology B. A. It is a complication of acute pancreatitis D. Pleural space C. Peritoneum B. constrictive pericarditis.510 MCQ’s FOR MEDICAL PROFESSIONALS BY PROF. tuberculous B. >5000 U/L A. internal spontaneously how many weeks after their formation ? fistula may develop between pancreatic duct and ? Harrison’s 18th Ed. steatorrhea. > 4 weeks C. producing a pleural effusion (pancreaticopleural fistula) that is usually left-sided and often massive. >20000 U/L D. and Budd-Chiari syndrome. 2643 A. Sentinel acute pancreatitis event Differential diagnosis of pancreatic ascites includes intraperitoneal carcinomatosis. 2642 Harrison’s 18th Ed. All of the above & abscess. but gastroduodenal. hemorrhage D. Irreversible damage to pancreas D. Sequential acute pancreatitis event D. > 6 weeks D. Rupture Harrison’s 18th Ed. Intravenous drug use C. weight loss & diabetes mellitus. Steatorrhea D. 2642 Harrison’s 18th Ed. > 2 weeks B. Diabetes mellitus Complications of pancreatic pseudocyst are pain. It increases severity in alcohol-induced chronic pancreatitis. Complications of chronic pancreatitis are abdominal pain. > 1 week A. Any of the above Significant number of pancreatic pseudocysts resolve spontaneously >6 weeks after their formation. Abscess B. Due to occlusion of anterior retinal artery A. If the pancreatic duct disruption is posterior. 2643 D. Chronic activity with subsequent new collagen synthesis ? Harrison’s 18th Ed. Malabsorption syndrome In United States. Sjögren’s syndrome. ulcerative colitis. Serum IgG4 pancreatitis 900-fold. effect of trypsin inhibitor.355:2670-6 A. . alcoholism is the most common cause of clinically apparent chronic pancreatitis in adults. 2013 D. Recurrent pancreatitis D.355:2670-6 A. IgG1 being found in greatest amounts and IgG4 the least. and retroperitoneal fibrosis. N Engl J Med 2006. Immunoglobulin G2 C. Sclerosing pancreatitis Proinflammatory cytokines. 1472 In hereditary chronic pancreatitis. 50–75% of patients with AIP present with obstructive jaundice. Acute pancreatitis C. The defect prevents the destruction of trypsinogen and allows it to be resistant to the levels (IgG4). N33S SPINK1 B. Hypergammaglobulinemia B. All of the above B. pancreatic fibrosis with lymphocytic infiltration continual activation of digestive enzymes within the gland causing acute injury and eventually & an absence of pancreatic calcification. Tumefactive pancreatitis 6) as well as oxidant complexes & growth factors are able to induce PSC activity with subsequent C. Growth factors Harrison’s 17th Ed. Isolated autoimmune chronic pancreatitis C. rheumatoid arthritis. antilactoferrin antibodies. D. pancreatitis ? Harrison’s 18th Ed. Cystic fibrosis A. an association with other autoimmune diseases and chronic pancreatitis.355:2670-6 A. All of the above PSCs also possess transforming growth factor (TGF-)–mediated self-activating autocrine pathways that may explain disease progression in chronic pancreatitis even after removal of noxious stimuli. 2644 N Engl J Med 2006. Acute 1469 Which of the following induces pancreatic stellate cells (PSC) B. Nonalcoholic destructive pancreatitis new collagen synthesis. Pancreas divisum D. Rheumatoid arthritis C. 511 Gastroenterology MCQ’s MCQ’s FOR FORPROFESSIONALS MEDICAL MEDICAL PROFESSIONALS BY PROF. normally accounts for only 5–6% of the total IgG in healthy patients but is elevated at least twofold higher than 135 mg/dL in those with AIP. tumor necrosis factor (TNF-). 2643 1476 Autoimmune pancreatitis is frequently associated with ? Harrison’s 18th Ed. N35S SPINK1 D. 2644. Immunoglobulin G3 D. The four IgG subclasses are numbered in combination of two CFTR mutations and an N34S SPINK1 mutation increased the risk of chronic order of their level in serum. Obstructive jaundice B. Chymotrypsin B. a genetic defect that affects the gene encoding for trypsinogen Autoimmune pancreatitis is characterized by the presence of increased serum gammaglobulin was identified. Interleukin 1 (IL-1) A. Autoantibodies against carbonic anhydrase C. Immunoglobulin G4 Presence of an N34S SPINK1 mutation increased the risk of chronic pancreatitis by twentyfold. Tumor necrosis factor (TNF-) B. apparent chronic pancreatitis in children ? Harrison’s 18th Ed. N Engl J Med 2006. All of the above A. mediastinal adenopathy. AJAY MATHUR Cardiology 511 The sentinel acute pancreatitis event (SAPE) hypothesis uniformly describes the events in the 1474 Autoimmune Pancreatitis (AIP) is which form of pancreatitis ? pathogenesis of chronic pancreatitis. become spontaneously activated. 2644 A. and interleukin 6 (IL- B. All of the above 1470 Which of the following plays a role in the self-activating AIP is also referred to as sclerosing pancreatitis. primary biliary sclerosis. Recurrent A. Proinflammatory cytokines D. tumefactive pancreatitis and nonalcoholic destructive autocrine pathways lead to progression in chronic pancreatitis. Hereditary pancreatitis B. 2644 Harrison’s 18th Ed. 2673 A. response to steroid therapy. Sjögren’s syndrome D. A IgG constitutes ~75–85% of total serum immunoglobulin. All of the above D. 2643 C. and to remain activated leading to anticarbonic anhydrase antibodies & rheumatoid factor). presence of autoantibodies (antinuclear antibodies. All of the above In hereditary chronic pancreatitis. AIP is associated with primary sclerosing cholangitis. Pepsin A. while cystic fibrosis is the most frequent cause in children. 2644 1477 Majority of patients with AIP present with ? Harrison’s 18th Ed. N34S SPINK1 C. In the United States. Autoantibodies against lactoferrin D. interleukin 1 (IL-1). Oxidants 1475 Autoimmune Pancreatitis (AIP) is also called ? C. Trypsinogen C. autoimmune thyroiditis. Immunoglobulin G1 B. defect in gene encoding for 1478 Immunologic abnormalities in autoimmune pancreatitis which of the following is found ? include ? Harrison’s 18th Ed. N32S SPINK1 A. Interleukin 6 (IL-6) B. 1473 Which of the following mutation increases the risk of chronic 1479 Serum levels of which of the following immunoglobulin is pancreatitis ? elevated in AIP ? Harrison’s 18th Ed. Transforming growth factor (TGF-) C. 1471 Which of the following is the most frequent cause of clinically tubulointerstitial nephritis. Inflammatory bowel disease D. 2644. This correlates well with the onset of chronic abdominal D. Immunoglobulin G4–associated pneumonitis B. Azathioprine D. Glucocorticoids C. 6-mercaptopurine Alcohol is the most common cause of pancreatic calcification. Vitamin B 12 malabsorption is corrected by oral pancreatic C. Immunoglobulin G4–associated carditis Harrison’s 17th Ed.512 MCQ’s FOR MEDICAL PROFESSIONALS BY PROF. 2645 1491 Which of the following is an uncommon complication of chronic A. It is characterized by a dense inflammatory infiltrate containing IgG4. 2644. insufficiency. < 20 mg/mL C. In chronic pancreatitis. Harrison’s 18th Ed. IgG4-related neuronal disease C.132:1557-1573 AIP ? A. It is associated with autoimmune pancreatitis and is characterized by elevated serum IgG4 and infiltration of IgG4-positive plasma Decrease of serum trypsinogen level to < 20 mg/mL strongly suggests severe pancreatic exocrine cells in bile ducts and liver tissue. islet cell tumors. No history of alcohol abuse or biliary disease Serum amylase & lipase levels are normal. often at the time of presentation in majority (70%) of patients. Immunoglobulin G4–associated neuritis A. 2645 1483 Which of the following drug is useful in AIP ? A. 40 % A. 2645 A. secretin stimulation test is abnormal is evident at very early stages. IgG4-related immunologic disease Harrison’s 18th Ed. secretin stimulation test becomes abnormal when 60% of the pancreatic exocrine function has been lost. Biliary cirrhosis pain. < 40 mg/mL D. Slow progression D. < 60 mg/mL Immunoglobulin G4–associated cholangitis is a biliary disease of unknown etiology with biochemical D. Pancreatic calcification is also seen in severe protein-calorie malnutrition. 2645 B. Autoimmune Pancreatitis B. Severe protein-calorie malnutrition C. Serum lipase IgG4-related systemic disease is a variety of acute tubulointerstitial disorder and a form of Acute Interstitial Nephritis (AIN). . Islet cell tumors A. 80 % C. posttraumatic pancreatitis. Idiopathic chronic pancreatitis Harrison’s 18th Ed. Early onset C. 2014 B. Diffuse pancreatic calcifications on D. Severe protein-calorie malnutrition C. AJAY MATHUR Gastroenterology 1480 Which of the following syndrome is related to IgG4 ? 1486 Which of the following tests is useful in identifying severe Harrison’s 18th Ed. Best diagnostic test is secretin stimulation test B. Alcohol B. Strictures in bile duct D. Severe pancreatic damage enzymes D. 2627 serum trypsinogen levels are ? A. Immunoglobulin G4–associated cholangitis C. < 80 mg/mL & cholangiographic features indistinguishable from PSC. IgG4-related systemic disease A. 2644 B. Fecal elastase D. Diabetic ketoacidosis C. Enlargement at the head of pancreas C. bowel disease.132:1557-1573 B. Gastroenterology 2007. rapid progression & severe pancreatic damage in the absence of a history of alcohol abuse or biliary disease. 20 % pancreatitis ? Harrison’s 18th Ed. Glucocorticoids lead to response. All of the above FP abdomen indicates ~80% damage to pancreas. Deficiencies of fat-soluble vitamins are uncommon 1490 Tropical pancreatitis is characterized by all except ? Gastroenterology 2007. 2644 B. In contrast to PSC. when how much of pancreatic exocrine function is lost ? Harrison’s 18th Ed. Narrowing of pancreatic bile duct D. it is not associated with inflammatory insufficiency. 1488 Absence of pancreatic calcification is a feature of ? 1482 Which of the following is characteristic imaging finding in Harrison’s 18th Ed. diabetic ketoacidosis and coma are uncommon. Fecal elastase levels of < 100 µg per gram of stool strongly suggests severe pancreatic exocrine expressing plasma cells. Hypercalcemic pancreatitis A. D-xylose excretion test C. hereditary pancreatitis. Serum amylase D. IgG4-related pulmonary disease B. Serum amylase & lipase levels are raised A. most patients have impaired glucose tolerance. hypercalcemic 1484 Which of the following is false about chronic pancreatitis ? pancreatitis. Pancreatic cancer D. 2646 B. Idiopathic chronic pancreatitis Harrison’s 18th Ed. Both exocrine & endocrine insufficiency 1485 In chronic pancreatitis. Tropical pancreatitis is characterized by early onset. idiopathic chronic pancreatitis and tropical pancreatitis. 1481 Which of the following syndrome is related to IgG4 ? 1487 Severe pancreatic exocrine insufficiency is obvious when Harrison’s 18th Ed. Gastrointestinal bleeding In chronic pancreatitis. 2369 pancreatic exocrine insufficiency ? A. 60 % B. All of the above 1489 Most common cause of pancreatic calcification is ? Harrison’s 18th Ed. Table 313-6. Idiopathic chronic pancreatitis A. 1493 Increased incidence of pancreatic carcinoma is seen in which of the following ? 1498 The major cause of death in alcoholic CP is ? Harrison’s 18th Ed. In patients with large-duct disease usually from alcohol-induced patients who harbor it. chronic pancreatitis. Human D. Normal fat absorption Macroamylasaemia is a syndrome characterised by formation of large molecular complexes between D. duodenum & enhance fat absorption. Minimal changes on ERCP D. diabetes mellitus & steatorrhea. Abnormal immunoglobulins C. Horse Pancreas divisum is the most common congenital anatomic variant of the human pancreas. 2014 A. Free lipase & enteric-coated proteases C. Severe infection C. Harrison’s 18th Ed. Type IIIc D. UDCA A. . Sphincter of Oddi disorders C. All of the above Octreotide significantly relieves pain in patients with severe chronic pancreatitis refractory to other Pancreas divisum does not predispose to the development of pancreatitis in the great majority of forms of therapy. Octreotide D. Type II C. Pancreatic duct scars D. Harrison’s 18th Ed. 513 Gastroenterology MCQ’s MCQ’s FOR FORPROFESSIONALS MEDICAL MEDICAL PROFESSIONALS BY PROF.000 units of lipase per meal may be necessary to normalize nutritional parameters in malnourished chronic pancreatitis patients. Enteric-coated lipase & free proteases A. They have a tenfold higher risk of pancreatic carcinoma (40% by 70 years). Free proteases enter Diabetes of chronic pancreatitis is classified as type IIIc according to ADA & is characterized by duodenum & evoke a positive feedback control mechanism & enteric-coated lipase open beyond destruction of both insulin & glucagon-producing cells. All of the above Patients with hereditary pancreatitis develop pancreatic calcification. 2646. Severe protein-calorie malnutrition D. 2014 B. 1500 Which of the following does not predispose to the development 1495 Which of the following significantly relieves pain in severe of pancreatitis ? refractory large-duct chronic pancreatitis ? Harrison’s 187th Ed.132:1557-1573 A. 2648 Harrison’s 17th Ed. 2014. Recent data suggests that dosages up to 80. Cow B. Pancreatic enzymes from porcine sources is the cornerstone of pancreatic therapy. infection & malignancy. Pig C. which of the 1497 The ideal pancreatic enzyme preparation is ? following diabetes in found in chronic pancreatitis ? Harrison’s 17th Ed. Clotting factors B. Cardiovascular disease B. Free lipase & free proteases D. Domperidone C. 1501 Macroamylasaemia is characterised by formation of large 1496 Small-duct chronic pancreatitis patients who respond best to molecular complexes between amylase and ? serine proteases are those with ? A. Hereditary pancreatitis C. Haem A. Hypercalcemic pancreatitis B. AJAY MATHUR Cardiology 513 1492 According to the American Diabetes Association. Malignancy D. Pancreas divisum B. 2646 A. Hereditary Pancreatitis B. Annular Pancreas C. Annular pancreas A. 1499 The most common congenital anatomic variant of human 1494 Source of pancreatic enzymes in treatment of chronic pancreas is ? pancreatitis is ? Harrison’s 187th Ed. Enteric-coated lipase & enteric-coated proteases B. ductal decompression has been the therapy of choice. Type I B. Abnormal hormone stimulation test C. 2648 Harrison’s 17th Ed. 2014.000–100. All of the above amylase & abnormal immunoglobulins. 2646 Gastroenterology 2007. Major causes of death in alcoholic CP are cardiovascular disease. 2647 Gastroenterology 2007. Cholestyramine B. Any of the above Ideal pancreatic enzyme preparation is enteric-coated lipase & free proteases.132:1557-1573 A. including surgery. Pancreas Divisum D. Urea Harrison’s 17th Ed. Notes : . ........................................... C 37 .......... D 140 .............. D 184 .......................... C 166 ......... D 15 . B 17 ...... A 12 ....................................... B 9 ..... D 168 ................ C 125 .. B 188 ............... D 191 ...... A 68 ................... D 32 ... A 75 ............ C 16 ................. B 7 ...................... D 172 .. C 38 ....................... B 39 ........... D 182 . C 190 ....................................................... A 92 .................. D 148 .......... B 146 ................................. A 128 ............ C 158 ................................................... C 147 . C 189 ..... D 163 ....................... D 174 ....... D 144 .............. D 93 ................... A 45 .............. D 25 ................................................... D 179 ..... B 134 .................. B 160 ............................. C 105 .. D 41 ............... C 112 ........... A 83 . C 40 ............... D 47 .................. B 81 ............. C 14 .... D 102 ................. A 62 .......... C 55 ......... A 187 . D 8 . B 107 .... D 82 .............................. B 114 . B 74 ...................... D 110 .................... B 79 .............. B 63 ......... C 99 ........ B 71 .............. B 118 ....... D 149 ......... D 177 ............................ D 97 ............ D 76 ....... D 116 ... D 70 ............ B 109 ........... B 23 .. C 156 . C 22 ............................................. C 111 ..............................Answer MCQ’s FOR MEDICAL PROFESSIONALS BY PROF........................................ A 43 ......................................... D 33 .................................... D 106 ............ A 124 .......... C 154 .... D 133 ..... D 61 ...... C 153 ......... D 119 ...... C 186 . D 176 .. B 80 ....... D 181 .............. D 101 .... C 151 ....... A 77 ....... A 73 . B 96 .... D 159 ......................... C 115 .... C 131 ....... D 46 .................................................... D 145 ............ C 21 .............................................. A 122 ................................... D 130 .... C 141 ....... D 3 ........... D 138 ......... B 18 ... AJAY MATHUR 515 ANSWERS GASTROENTEROLOGY 1 ..................................... C 72 . C 113 ... D 30 ................................ D 10 .......................... A 87 ...... B 90 .. B 95 ................................................................................... C 2 ..................... A 167 .................... D 192 .................. A 4 ..... C 152 ......................................... A 180 ................................... C 51 .......... D 108 ..................... D 50 ............................ A 135 ... A 171 ................. D 142 .... C 123 .................... A 64 . A 5 ........... B 13 ... D 127 ..................... C 137 .......... C 178 ..................................... C 52 ... D 150 ..... D 143 ................ D 157 ....................................................................................................... A 28 .................... C 170 .......... C 24 . D 54 ..................... B 139 . B 100 .. A 60 ... A 48 ................................... D 103 .. C 49 ... D 121 ..... D 11 ............................. B 56 .............. C 35 .......... D 98 .................. D 161 .......... D 19 ............... D 164 .. D 42 .. C 36 .......................... A 126 .............................. A 175 ................................... A 173 ..... B 65 ................................ D 91 ... C 6 ................... A 26 ................ B 59 . D 69 ..................... D 27 . B 20 ............ C 78 ......................... D 162 .......... C 183 ....... B 34 ........ D 185 ........ D 129 .............................................. D 89 ........ A 169 .............................. D 155 ............................ A 58 ..................... C 66 ........ C 104 ... A 136 .... A 85 ...... D 53 .... B 94 . A 88 ........... A 117 ............. D 165 ............................. D 44 .... C 84 ................ C 29 ...... D 67 .. C 132 ...... D 120 ........... C 31 . C 57 ........................ A 86 ...... B ......... .. C 316 ..... C 312 ......... D 233 ............... D 306 ... A 332 ............... B 234 ........................ D 232 ...... C 280 ....................... A 292 .... D 367 ........ A 207 .................. D 240 ................................. A 206 ... D 259 ............. A 342 ... D 345 ........... D 213 ..................... B 283 ..... B 263 ....................... A 195 .................. D 235 ................... A 251 .................... C 225 ........ A 317 ... B 358 . A 260 .. B 298 ... D 321 ........ A 296 ............................ D 221 .................................... C 308 ............................................ D 287 . D 343 .......... A 329 ........... D 354 .................. A 212 ......................... C 257 .... B 230 .......... C 286 ............. D 198 ... A 338 .................................... D 375 .. D 310 ... C 281 .......................... D 311 ........................ A 353 ..................... A 288 ............ B 355 .................................... D 224 ........................... D 291 ..... D 252 ........... B 299 .......... D 307 . C 364 ... A 272 ............................. D 275 ... A 253 ..... B 313 .. B 262 ......................... C 273 ................ D 336 ...... D 349 ... A 297 .............. D 215 . B 246 ....... D 362 ............................ C 249 ....... A 293 ... A 219 ............................... A 279 .............. D 228 ................. C 337 .......... C 208 ............. D 374 ........... D 277 .......... A 352 .......... D 372 .......... AJAY MATHUR Answer ANSWERS GASTROENTEROLOGY 193 .......................................... C 265 ...... B 243 ......... D ................................. B 271 .......... A 326 .... D 361 ................. A 203 ...... C 216 ................ D 377 ...... C 269 ................................ A 322 ................. C 261 ........... A 204 ............................................ A 348 ..... D 371 ............................... D 223 .......... A 205 .................... C 344 . D 357 .... C 211 .................. A 320 ......... A 318 ................................................. D 244 . D 383 ........................................ A 278 . A 360 ........... C 241 .. A 256 ............... A 303 .. D 304 ................................ B 217 .... B 290 ................... D 340 ................................................................. C 264 .. B 363 ... D 274 ........................ A 314 .... D 370 ................... D 369 ........................................ A 199 ............. D 248 .......................... D 285 ......... D 237 ......... D 220 ........... D 376 .................. B 384 ........................ D 381 ............... B 202 .. D 294 . C 218 ................................ D 254 . B 301 .. D 236 ...... A 319 .. B 300 .. A 333 .... C 356 ........................ A 282 ..... D 331 .... C 327 ....................... A 268 ..... D 214 .......... C 295 ..... C 276 ...... D 289 .... D 231 ................... A 380 ................. C 197 ................................ A 209 . D 325 .................... D 247 ......... A 242 ... D 226 . A 328 ....................... C 324 ............................ A 365 ................ C 239 ....... C 350 .................... D 200 .......... B 227 ......... D 323 ... A 194 .................... D 255 .................................. B 368 ....................... D 334 ............... B 210 .......... B 305 ... D 302 ........................ A 270 ..... C 351 ......... C 266 .... C 196 ............. B 245 . C 267 .. B 359 .... B 366 ............... C 258 ............................................. B 250 .......................... D 222 ................ D 373 ....... A 238 .......... B 201 .... A 382 ....... C 347 .............. B 379 ................. D 335 ..........................516 MCQ’s FOR MEDICAL PROFESSIONALS BY PROF....................... D 378 ....... B 229 . C 339 ........ D 341 ......... D 284 ...................................... A 309 ..................... A 330 ....... C 315 ........... B 346 ......................... . B 524 . C 465 . D 533 ................ C 575 .......... C 491 ......... B 534 ...... D 453 ..... C 515 ......................................... A 529 .. A 421 .................... D 546 ............................ B 397 ... C 408 .......... D 483 ................................... C 445 ... C 494 .... D 487 ......... D 505 ............................... C 462 ................ B 452 .. D 540 ...... D 566 ....... D 454 .. D 426 ................. D 528 ........ C 409 ..... C 520 ... B 537 ........... B 388 ....... B 435 ...................... B 493 ............................................ C 489 .. B 501 . D 504 ................ D 392 ....... B 519 .... B 559 ..... C 398 ....... D 564 .................................... B 415 ............................... D 413 .......................................... D 499 ... D 497 ..................... C 574 ........................ D 467 ................ C 567 ...................... A 470 ............................................................... D 460 .... C 518 ........... B 439 ......... C 509 ............................. D ........................................................ D 482 .. D 463 . A 565 ........... A 396 .......... A 464 .................. B 560 .. B 394 ... C 512 ...... C 430 . C 511 .... B 407 ............................................................................. A 405 ............ C 391 ..... D 532 ...................... D 444 .............................. D 553 .......... A 573 .... B 513 ........ C 468 ........................................... C 450 ...... D 495 ....... D 448 ..... D 547 .......... C 478 .. B 526 .... B 474 ............................... C 436 .................. A 443 .......... D 525 ................ B 562 .......... A 554 . C 557 ...... C 550 . A 552 . C 535 ........ B 488 ................... A 521 .. B 400 ...................... A 472 ....................... D 404 .................. B 442 ... C 541 ................. B 543 .... D 406 ........... A 502 ... C 434 ............................. D 568 ................................................ B 555 . D 429 ........................... D 548 ........................ A 545 ................. B 484 .................................. A 446 ... A 469 ... C 506 .............. D 418 ............ C 428 ........ D 457 ............................... B 431 ........ D 411 ...................................................... A 473 .. A 403 .. D 455 . D 471 .... A 420 ......... D 563 ....... D 517 ........ A 516 . D 432 .... D 477 ............... D 490 ....... AJAY MATHUR 517 ANSWERS GASTROENTEROLOGY 385 . D 461 ..... C 530 .............. C 416 . D 412 .......... D 476 ....... B 508 ......................... D 419 ...... D 447 ................... B 425 ... D 570 .......... D 402 ...... D 531 ........................ A 561 ............................ A 498 ..... A 395 .. A 500 ............. C 393 ............................................ B 556 .................... A 422 .... B 466 .......... A 522 ....... C 542 ................... A 527 .................................. D 390 ......... A 507 . A 576 ..................... B 571 ............ D 399 ................................................ B 538 .............. B 437 ........... D 414 ........ D 492 ............................................................................................................................. D 451 .......... B 410 ................................................ B 480 . C 438 .... A 523 .......... B 441 .... D 558 ............ C 510 ........ D 386 .......... D 481 . C 496 ................ C 424 .. D 485 .... C 536 .. B 514 .Answer MCQ’s FOR MEDICAL PROFESSIONALS BY PROF........... B 572 ......... A 401 .... C 387 ............... C 389 ..... D 458 .. C 440 ............ D 475 ................................ D 569 .................................... A 544 .... C 417 ............ B 486 ......................................... D 449 ............. D 456 ... D 479 ....... C 539 .... B 423 ...... C 549 ........................ D 459 ............ A 551 ................. B 503 . B 427 ........ D 433 ................... ........ C 658 ...................................... D 629 ............ A 601 ................. B 766 ....... B 711 ... B 647 ........... D 733 .......... A 619 ....... D 595 ................. D 598 .. A 718 ........ A 712 ........................................ A 609 .. D 731 . B 721 .................... C 677 ... C 650 ......... D 734 .............. B 622 ........ C 666 ................................ C 725 ....................... D 755 ..... D 594 . D 688 .................................... A 750 . B 728 ... D 610 ......... D 623 .................... D 685 ...... D 693 ..... C 633 ................. D 698 ..................... B 703 ........................ D 675 ... D 706 ....................... B 669 ....... C 768 .......... C 680 .. D 578 ....... A 748 ... D 738 ............................................ B 716 ............................ C 615 .. B 687 ........................ A 657 ............. D 744 .. B 746 ......... B 683 .................................................................................. A 582 .............. B 637 ..... B 616 ......................... C 649 .............. D 757 . B 587 ......................................... B 656 ............. D 665 ................. B 651 .......................... D 646 ..... C 727 ... C 640 ..... C 756 ....... D 643 .. B 709 ........................... D 631 ......... A 673 .......... C 664 ................ C 737 ......... B 681 . A 707 ...... C 608 ................. C 653 ... D 659 .................... D 679 ............ C 583 ............... A 765 .. B 618 ................................. C 639 ......................................... D 723 .................. C 753 ......................................... C 763 ......... B 682 ........... AJAY MATHUR Answer ANSWERS GASTROENTEROLOGY 577 ................ D 710 ............ C 719 . A 671 .. D 602 ...... C 612 ................................ B 667 .................................... D 581 ... A 591 .... D 758 ........................... D 720 ......... A 730 ........................... D 735 .......... D 600 ...................... D 759 ... A 729 ....... D 625 ............................................ B 676 ................. D 635 .... D 724 ............ B 715 ............... D 611 ........ C 663 .................................... D 624 ... C 678 . C 617 ...................................................... B 628 .................... D 630 ....................... B 636 ......... B 603 ........................................... C 662 .... A 654 ................ D 580 ...... D 645 .............. D 584 ......... D 634 . B 754 ....................... B 642 .518 MCQ’s FOR MEDICAL PROFESSIONALS BY PROF............. D 621 . C 627 ...................... D 672 ...................... A 670 .......... A 764 ......... B 695 ............... B 620 ........................................... C 604 .................................. D 585 ............. A 655 ..... A 745 ............... D 760 .............. A 579 .............................. C 674 ... D 692 .............. B 741 .............. A 607 ..... C 589 ................. D 690 ............ B 596 ............. D 592 ................ A 749 ...... B 751 .... A 726 ....... D 588 . D 614 .................... B 697 ......... D 684 ........ C 668 .... C 747 ........ B 691 .. A 740 .... A 661 ................... A 590 ............................ A 739 ............................. D 714 .. A 694 .... D 708 ............ D 593 ... A 632 ............... B ... C 641 .... D 606 ............ C 605 .. D 586 ........ A 705 .... D 736 ...................... D 761 ... C 704 .............. C 626 ............ D 702 ......... B 699 .............. B 752 .... A 722 ......... D 689 ....... C 599 ...... C 597 ................................... B 732 ......... D 767 ........ C 686 .... C 613 ..... A 762 ............. D 742 .... B 717 .................. A 638 ..................... D 700 ............................................. D 696 ......................... A 701 ..... C 713 ...... D 648 ....................... D 644 .... A 743 ............................. C 660 ............... C 652 ...... ..... A 816 ..... C 773 ........... D 775 .................................... A 771 ....... A 920 ........ D 844 ..... A 845 ..... D 850 ......... D 822 .... D 790 .. A 881 ........... D 897 .. D 957 ................. D 911 ..... D 854 ........... C 861 .. A 814 ............................... A 770 ...................... D 832 . B 863 .............. A 787 ................ D 859 ........... D 904 .. D 862 ............. D 804 ............. C 912 ... C 914 .................... D 831 ............................ B 820 ....... B 918 ............... D 930 ............ D 777 ................... A 941 ....... D 781 ....................... C 906 .............. A 883 ..................... D 946 ..... D 821 ............................. A 795 .. A 839 ....................................... D 958 ................. D 867 . B 955 ...... A 810 ......................... D 942 ......................................... D 827 ... D 802 ................... D 798 ....... D 890 .................... A 785 . D 952 .. C 865 . B 903 ................ B 778 ....... C 791 .. A 933 ......... D 907 ................................ D 953 . D 929 ... D 948 ....... C 895 ...... D 797 . A 927 ................. B 877 ............ C 823 ......................... D 852 .............. B 788 ............ B 846 ......................................................... A 837 ..... A 879 ........ C 792 .................................................................................................... D 796 ....... A 811 ........... D 813 ........................................... C 809 .................... B 905 .................. C 938 .... B 873 ............ D 931 ........ C 851 ...... D 801 ............ D 828 .......................... D 919 .......... D 950 ............................... A 893 .......................... A 923 .......... B 848 .......... D 868 ....... D 878 ........ D 826 ........ B 842 ................ B 884 ...................... A 901 . B 849 .... C 896 .......................... D 772 ............................. A 892 .................. C 799 ................... D 913 ......... D 940 .... D 840 ................................ B 817 .. D 899 ...... C 885 ... D 774 .................................. A 857 .......... C 858 ..... C 876 .... D 864 ..................................... D 928 .. C 959 . C 783 .......... D 943 ... B 793 ........... D 805 ........ A 922 .. D 806 ............................................................................. D 951 ... A 875 ......................... D 786 .. D 818 ..... D 945 .................................... A 926 .................................. B 934 ........................ D 917 ....................... D 947 ............ B 779 ... D 887 .......... A 835 ..................... D 830 ............ D 925 .......... D 915 ...... C 908 .... D 824 ............................... D 869 .... A 932 ... D 866 ............ C 936 . D 949 .......... D 909 .......................... D 939 .......... D 882 ..... A 870 ..... A 886 ............ D 954 ........................ A 819 .......... C 825 .............. D 789 ............. D 871 ...................... B 916 ... D 794 ................ C 856 .................................. C 894 ................. A 855 ...... C 853 ... A 776 ............................... D 838 ...... D 803 ......................... AJAY MATHUR 519 ANSWERS GASTROENTEROLOGY 769 .. B 889 ................................ D 860 .. A 924 ................................... D 829 ........................ C 960 ................ B .... C 843 ....... B 841 ............. D 834 ................................. B 836 ........................ B 956 ..... D 847 . A 784 ....... D 815 .... C 807 ......... C 898 ........ D 812 ...........Answer MCQ’s FOR MEDICAL PROFESSIONALS BY PROF......... A 900 ........ B 780 ...... D 888 ... D 910 ...................... D 921 ................ D 944 ....................... C 935 ....................... B 833 .................. B 782 .......................... D 872 ... D 808 .. C 937 ....... C 891 ......... D 902 ...................... D 874 ............. B 800 ........ D 880 . .................. C 991 .............................. D 1144 ................. D 1132 ............ D 1074 ....... C 1136 ................... C 1036 .... D 1141 .. C 1051 ........ D 983 ... D 1077 ......................... D 966 ............ D 1152 .......................... D 1072 ................. D 1067 ..................................... D 1055 .............. D 1102 ...... A 1008 . C 1122 . A 1011 ................... C 1130 ........ D 1084 ......... D 1065 .... D 1001 ...... B 1104 ............ A 1129 . D 978 ................ C 1049 ........ D 1021 .......... D 1106 ... A 1110 .. C 1025 .. B 996 ................................. D 1033 .................... D 1149 ........ D 962 .... C 1003 .... D 1023 ............................................................. D 1124 ................ C 994 ......... B 1125 ..... C 1127 ..... D 1096 ................................ D 1009 ....... D 1031 .............. D 1075 ......................................................... D 967 ...... A 1020 ...................... D 990 ............ C 1013 .................... D 997 . D 1007 ............... D 1123 .. B 1146 ........... D 1058 .................... C 1017 ................. A 1039 .. C 1054 ...................... A 1090 ...................... A 1128 ............ B 1078 ......... A 1034 ........ A 1002 .... C 1070 .................. C 1082 ...... A 973 ........................................... D 1053 ..................................... B 1015 ........ D 1105 . A 1135 ...... D 1030 ... A 971 ...... A 1006 ... B 1095 .......... B 965 .......... C 1057 ................. B 1091 .............. D 1079 ............. D 1098 ............. B 964 ....... D 1047 .... AJAY MATHUR Answer ANSWERS GASTROENTEROLOGY 961 ... B 1131 .......520 MCQ’s FOR MEDICAL PROFESSIONALS BY PROF.......... D 1019 . C 1089 ......................... D 1134 ..... C 1014 ....... D 1076 ... C 988 ............... C 1093 ............... D 970 .... D 1108 .......... C 1138 ...................... D 1118 ....... D 968 ... D 1085 ..... A 1004 . C 1100 ...................... D 1060 ..... A 1028 ....... D 992 ......... B 1062 .......... C 995 ................... D 1139 ...... C 1088 ... B 1068 ......... C 1061 .. B 1032 ............. D 1094 . D 1147 ................................................................ B 969 ... D 1022 .. C 986 ......... D 979 .... D 1140 .......... B 1092 .......... A 1029 .............. D 1114 ..................... A 993 ........... A 1151 .............................. D 1148 ............... C 1081 .............................. C 1097 . B 1087 ..................... D 1117 . D 982 ............................................................ B 1044 .. B 1142 ........ B 1050 .......... B 977 .... D 1143 .... C 1112 ......................... D 1119 .... C 1073 .............. C 1071 ................... C 985 ............................................ B 1064 . D 980 ............ B 998 ........................... C 974 ..... D 1066 ............................. A 999 ... C 1113 ........................ D 1133 ... A 1145 ....... B 1063 .......... C 1121 ....... C 1041 ...... D 1016 . D 1035 ... D 1037 ............ D 972 .......... C 1024 .. C 1069 ...... A 1010 .................... A 1056 ..... D 1042 ............ C 975 ...... D 1086 . A 963 ................. A 1040 ....... D 981 .......................... D . D 1000 .............. D 1059 ....... B 1038 .................... A 1103 ...... A 989 ...... B 1052 . D 1080 .................... D 1027 ................ D 1109 ............. A 1018 . C 1099 ........... C 984 ..... D 1116 ... C 1150 ... D 1107 . A 1043 .......... C 1101 ............................. A 1137 .......................................................... B 1012 .......... D 1045 . A 1083 ....... B 1046 ............ A 1005 ..................... B 1126 ................ D 1120 ........ D 1115 ................... B 976 ... C 1048 ... D 1111 ............................ D 1026 .. A 987 .................... A 1306 ..... C 1327 . A 1277 ........................ C 1267 ...... D 1176 ... D 1185 ...... B 1263 ........ C 1279 .................. D 1288 ........................ A 1192 ..... B 1233 . B 1216 ...... D 1187 . D 1333 ... B 1162 ..... B 1287 ............ C 1209 ..................... C 1215 ............ C 1236 ...... D 1238 ... A 1166 .......... AJAY MATHUR 521 ANSWERS GASTROENTEROLOGY 1153 ............... D 1328 ..................... B 1259 ... A 1281 ..... C 1258 .......... D 1183 ..... D 1207 ............ B 1235 .......... A 1310 ........ C 1184 ...... D 1156 .............................. C 1338 .... C 1165 ........ A 1336 ............ D 1244 .... D 1227 . D 1171 .... D 1252 ........... D 1250 ... D 1219 ............. A 1222 ... A 1320 ................. A 1315 ................................................. B 1239 ................ D 1186 ..... A 1194 ..... B 1339 ...... C 1321 ......... D 1280 ....... D 1221 ..... D 1264 .... D 1337 . D 1257 .... D 1343 ........ A 1190 .............................................. A 1193 ...... D 1188 ......... B 1344 .... B 1160 ..... D 1296 ........................... A 1308 ........ B 1290 . A 1170 ........................ B 1169 ......... C 1256 ..................... A 1269 .. C 1204 .... C 1157 ...................... D 1292 ... C 1265 .... D 1206 ............. D 1331 ...... B 1325 ........ D 1214 ....... D 1299 .. C 1260 .. A 1294 ... D 1301 ........................................ D 1191 ..................... B 1231 ........................... D 1295 ...... D 1217 .............................. D 1341 ............... D 1243 ..... B 1180 ......... D 1155 ....Answer MCQ’s FOR MEDICAL PROFESSIONALS BY PROF... C 1284 .... C 1175 ...................................................................................... D 1307 ... A 1278 ...... D 1291 ...................... A 1158 .... B 1323 ............ D 1275 ............ D 1289 ...... B 1213 ... B 1205 ....... A 1334 . D 1293 ....... D 1285 ..... B 1199 ............................ D 1212 ...................... B 1270 ....................................... B 1223 .......... D 1168 ..... D 1196 ...................... C 1335 ..... B 1241 ........... D 1237 . A 1248 . D 1197 ..................... A 1234 ..... D 1317 ....... A 1309 ................................ C 1255 ...... B 1172 ....... D 1266 ............... D 1182 .... C 1329 . D 1253 .... B 1203 ...................................... C 1240 ................. D 1249 ................. D 1242 .... D ........ D 1305 ..... A 1229 ........ A 1154 .......... A 1313 .. C 1159 ....... C 1178 ..... D 1181 ......... D 1226 ........................................................ D 1302 ..................... C 1340 ............................... B 1322 .... D 1332 . D 1245 .............................. D 1271 ..................... D 1324 . B 1314 ........ D 1195 ..................... A 1200 . C 1163 .. B 1297 ................. A 1300 ..... D 1298 .. D 1342 ..... D 1283 ........................ A 1179 ............... C 1228 . D 1316 ................ D 1261 ..... B 1268 ..................................... A 1274 .................... B 1201 ............ D 1230 ...................... D 1225 .. B 1311 ... C 1262 ........... D 1273 ......................................................... D 1276 ....................... D 1326 .............................................. B 1164 .... C 1312 ....... D 1232 ............ B 1303 .. D 1211 ... C 1318 .... B 1246 .................... D 1319 ... A 1198 ............................................ C 1224 .. D 1286 ... A 1167 .............. D 1251 ............................ D 1304 .. D 1177 ................... C 1330 ............ B 1173 .... D 1189 .............. C 1272 ..................... D 1218 ....................................... D 1247 .......... B 1254 ........... A 1161 . A 1220 ...... D 1208 ...... D 1202 . C 1174 ...... C 1210 ................................ B 1282 .. ............. D 1431 . A 1426 ........ A 1429 ............. B 1359 ............. A 1402 .... D 1457 ..... D 1437 ................ C 1362 ............................. A 1433 ....................... B 1411 .................................... D 1422 ... C 1421 ............. C 1486 ................. D 1482 ... B 1450 ........ D 1369 ....... C 1497 . C 1454 ...... B 1357 ....... D 1452 .............................. D 1414 ....... D 1424 . B 1466 ...................... C 1388 ............ D 1348 .................. A 1408 ......... D 1377 ......... D 1438 .......... D 1434 .. B 1458 ...... A 1476 .............. A 1492 . B 1381 ..... C 1365 ................. D 1487 . B 1430 ...... D 1471 ...................... B 1382 .... D 1464 ................. D 1427 ...... B 1389 . B 1447 .. D 1463 ...522 MCQ’s FOR MEDICAL PROFESSIONALS BY PROF.................................................................................. A 1400 . B 1448 . B 1469 ...... A 1491 ..... A 1467 ............................................. D 1417 ..... D 1398 ............ D 1355 .. A 1479 .......... D 1453 .............. D 1371 . D 1480 ......... C 1395 ................... A 1442 ...... D 1493 ....... D 1436 ............................ B 1372 . A 1475 . D 1428 ............ C 1391 ........... B 1347 .......................... B 1470 ....... B 1420 ............ C 1477 ... C 1373 ............ D 1405 ............ A 1360 . D 1356 .................... D 1444 .................... D 1415 ....... D 1472 ............................... D 1399 ............................................................... B 1383 ........................................ C 1446 .................. AJAY MATHUR Answer ANSWERS GASTROENTEROLOGY 1345 . B 1367 ........ D 1468 .......... D 1413 ............. D 1374 . D 1445 .. C 1409 . D 1397 ...... D 1449 ...... B 1481 ... D 1406 .......... C 1461 ............................... C ... B 1385 ................................ D 1407 ....................................... D 1501 .. D 1494 ... D 1396 ............ D 1451 ................... B 1443 . D 1432 ................. D 1441 . A 1465 ........................... B 1499 ........ D 1394 .... D 1488 ............ A 1418 .................................. B 1474 ......................... D 1393 ........ D 1423 .................. D 1435 .................................... C 1358 ... B 1378 ............................... D 1354 ................. B 1403 ....... D 1485 ......... A 1364 .. A 1498 ..... A 1370 . C 1455 ................ C 1366 ... A 1350 ................. A 1500 .................. D 1496 .... D 1375 ....... D 1379 .............. A 1460 ... C 1390 ...... D 1440 .. D 1349 ............... D 1484 ..... D 1490 ...... D 1352 ......................................... A 1376 . D 1351 ...... D 1489 .. B 1363 .... B 1380 ....... C 1425 ..... B 1462 ............ D 1439 ................ B 1384 ...... B 1386 ................................ B 1410 .. B 1392 .. B 1495 ....... D 1404 ... C 1346 ............. B 1353 ... A 1416 ...... B 1401 .......... A 1419 .................. B 1483 . C 1361 ....... A 1387 ........................................ D 1459 ................ C 1478 ... D 1368 .......... A 1412 ... C 1473 ............ A 1456 ............. derived from a Greek phrase meaning ? Harrison’s 18th Ed. Harrison’s 18th Ed. Steroid hormones diffuse into circulation as they are synthesized. Steroids. GR B. catecholamines & thyroid hormone). 13 Which of the following is a membrane receptor for hormones ? Harrison’s 18th Ed. insulin. AR. Somatostatin 10 Hormones that are stored in secretory granules before release are all except ? D. retinoic acid. AJAY MATHUR Cardiology 523 B. 2867 A. large proteins (insulin. VDR a Greek phrase meaning “to set in motion”. Steroid hormones B. thyroid hormones. albumin & thyroxine-binding prealbumin (TBPA). Insulin & thyroid hormone receptors are widely distributed. Human chorionic gonadotropin (hCG) B. Luteinizing hormone (LH) Peptide hormones (GnRH. Nuclear receptors bind small molecules that can diffuse across cell membrane like A. vitamin D. PPAR D. ACTH receptors are located exclusively in adrenal cortex C. Thyroid hormones B. Thyroid hormone C. Thyroid hormone receptors are found only in thyroid gland hormone? Hormone receptors are either membrane or nuclear. Vitamin D C. 2868 4 Which of the following is not a ‘large protein’ class of hormone? A. RAR Starling differentiated between endocrine & exocrine secretions. FSH. LH & human chorionic gonadotropin (hCG). B. PTH neural signal stimulates their release. FSH receptors are found only in gonads D. small neuropeptides (GnRH. nuclear receptor family is classified into type 1 receptors (GR. ER. According to their specificities for DNA binding sites. Steroidogenic action regulatory protein D. Tyrosine kinase receptors . MR. T4 & T 3 bind to thyroxine-binding globulin (TBG). Catecholamines A. 523 Endocrinology MCQ’s FOR MCQ’s FOR MEDICAL MEDICAL PROFESSIONALS PROFESSIONALS BY PROF. 2866 Harrison’s 18th Ed. 2869 7 Which of the following is not type 1 of nuclear receptor family ? Harrison’s 18th Ed. 2867 B. Steroidogenic acute regulatory protein C. “To rewind” B. All of the above D. GPCR A. Insulin receptors are widely distributed 3 Which of the following is not a ‘small neuropeptide’ class of D. 2866 A. Thyrotropin-releasing hormone (TRH) C. 2868 B. GH C. All of the above LH & PTH). Thyroxine-binding prealbumin (TBPA) Hormones are divided into 5 major classes . “To set in motion” A. Steroidogenic augmenting regulatory protein StAR (steroidogenic acute regulatory protein) transports cholesterol into mitochondrion upon 6 The glycoprotein hormone family consists of ? stimulation by ACTH and LH in the steroidogenic pathway.amino acid derivatives (dopamine. Albumin D. Steroidogenic active regulatory protein B. 2866 A. Dopamine 9 Which of the following about hormone receptors is false ? Harrison’s 18th Ed. All of the above C. steroids & vitamin D. VDR. GnRH Harrison’s 18th Ed. steroid hormones (cortisol & estrogen) & vitamin derivatives (vitamin A & D). Thyroid-stimulating hormone (TSH) 12 T4 & T3 bind to ? Harrison’s 18th Ed. Amino acid derived & peptide hormones interact with cell-surface membrane receptors. MR Chapter 338. D. “To speak” C. Insulin A. TR 1 The term hormone. Follicle-stimulating hormone (FSH) & LH A. Luteinizing hormone (LH) Harrison’s 18th Ed. Vasopressin C. 2867 B. Glycoprotein hormones are thyroid-stimulating hormone (TSH). Thyroxine-binding globulin (TBG) C. TRH. RAR. “To synchronise” Harrison’s 18th Ed. AR D. GH) are stored in secretory granules before a releasing factor or a D. ACTH receptors are located almost exclusively in adrenal cortex. 5 Which of the following hormone is “lipid-soluble” ? 11 “StAR” refers to ? Harrison’s 18th Ed. PPAR) 2 Which of the following is not an amino acid derivative ? that bind thyroid hormone. Insulin D. Steroids A. Gonadotropin-releasing hormone (GnRH) thyroid hormone. PR) that bind steroids & type 2 receptors (TR. Harrison’s 18th Ed. Somatostatin B. Principles of Endocrinology C. 2866 catecholamines. 2866 8 Which of the following is not type 2 of nuclear receptor family ? A. Membrane receptors bind peptide hormones & Harrison’s 18th Ed. Word “Hormone” is derived from D. 2866 B. PR C. somatostatin & vasopressin). 2868 A. & FSH receptors are found only in gonads. vitamin D & retinoids are lipid-soluble & interact with intracellular nuclear receptors. AJAY MATHUR Endocrinology C. Cytokine receptors B. G q subunits couple to phospholipase C generating diacylglycerol & inositol triphosphate B. GDP) D. Membrane receptors bind peptide hormones & catecholamines. None of the above 20 Intracellular Janus kinases (JAKs) is related to which of the following receptors ? G proteins are a large family that form a heterotrimeric complex composed of various  and subunits. hypothyroidism. and bone morphogenic proteins (BMPs). Growth & differentiation D. 2869 D. G i 22 Smads is related to which of the following membrane receptor C. chronic illness or GH & PRL receptors belong to the cytokine receptor family. They bind guanine nucleotides (GTP. G 21 Serine kinase receptors mediate the action of ? Harrison’s 18th Ed. 2869 Serine kinase receptors mediate the actions of activins. epidermal C. C. All of the above metabolism. All of the above except ? Harrison’s 18th Ed. dopamine). Serine kinase receptors A. Reproduction growth factor (EGF). Serine kinase receptors C. C. müllerian- A. C. G D. GPCR A. MAPK). Tyrosine kinase receptors B. GDP to the nucleus & stimulate expression of target genes. GTP Janus kinases (JAKs) phosphorylate members of signal transduction & activators of transcription (STAT) family and other signaling pathways (Ras. A. 2869 D. A. By alphabetical order The seven transmembrane GPCR family binds large proteins (LH. Growth phase of cell division somatostatin). B. LH receptors & serine kinase receptors. Serine kinase receptors 17 Insulin acts via which of the following membrane receptor for D. the G  subunit is activated and mediates signal transduction through adenylate cyclase or phospholipase C. precocious puberty. malnutrition. Cytokine receptor-linked kinase D. 18 Growth hormone (GH) acts via which of the following membrane receptor for hormones ? 24 Short stature may be due to ? Harrison’s 18th Ed. GPCR B. Maintenance of homeostasis Tyrosine kinase receptors transduce signals for insulin & growth factors like IGF-I.524 MCQ’s FOR MEDICAL PROFESSIONALS BY PROF. PTH). 2869 Membrane receptors for hormones are 7 transmembrane GPCRs. B. Tyrosine kinase receptors Harrison’s 18th Ed. The subunits modulate the activity of the  subunit besides mediating their own effector signaling pathways. Cytokine receptors A. genetic abnormalities affecting epiphyseal growth plates (FGFR3 or SHOX mutations) cause short stature. Hypothyroidism C. Serine kinase receptors 19 The seven transmembrane GPCR family binds all of the following D.binding site & hydrolyzes GTP to GDP. 2870 There are more than a dozen isoforms of G  subunit. Tyrosine kinase receptors play a prominent role in cell growth. catecholamines (epinephrine. TRH 14 G proteins are so named because ? C. GPCR adenylate cyclase that generates second messenger cyclic AMP leading to activation of protein kinase A. GH deficiency B. All of the above Harrison’s 18th Ed. Müllerian-inhibiting substance 16  subunit ? Which of the following is an isoforms of G D. All of the above Harrison’s 18th Ed. Serine kinase receptors Harrison’s 18th Ed. small peptides (TRH. GTP hydrolysis to GDP allows reassociation with the  B. Activated STAT proteins translocate B. Then. C. . and minerals (calcium). cytokine A. platelet-derived growth factor & fibroblast growth factor. Cushing’s syndrome D. maintenance of homeostasis & reproduction. B. 2869 Caenorhabditis elegans sma + mammalian mad). Gq for hormones ? D. GPCR 23 Physiologic function of hormones is ? B. Transforming growth factor subunits and restores the inactive state. Activin A. transforming growth factor . tyrosine kinase receptors. nerve growth factor.AMH). allowing G  to bind GTP & dissociate A. Physiologic functions of hormones are growth & differentiation. PI3-K. 2869 The  subunit contains the guanine nucleotide . All of the above GH deficiency. Harrison’s 18th Ed. G s inhibiting substance (MIS or anti-müllerian hormone . G s stimulates. differentiation & in intermediary D. Cushing’s syndrome. Tyrosine kinase receptors leading to activation of protein kinase C & release of intracellular calcium. Cytokine receptors hormones ? Serine kinase receptors (type I & II subunits) signal through proteins called smads (fusion of Harrison’s 18th Ed. 2870 Harrison’s 18th Ed. 2869 A. 2870 Hormone binding to the receptor induces GDP dissociation. whereas G i  inhibits A. Activins from the complex. 2870 C. Calcium Harrison’s 18th Ed. Tyrosine kinase receptors 15 Which of the following mediates signal transduction through adenylate cyclase or phospholipase C ? C. calcium feedback on PTH.adenohypophysis. SF-1. Guyton’s Textbook of Medical Physiology 11th Ed. Cortisol 32 Which of the following about pituitary gland is false ? Harrison’s 18th Ed. it comprises anatomically & functionally distinct anterior & posterior lobes. as well as gonadotropes. A. Harrison’s 18th Ed. Prop-1. Gonadotrope cell development is further defined by the cell-specific expression A. Extension of posterior pituitary tissue into pars intermedia Harrison’s 18th Ed. gonadal steroids on GnRH-LH/FSH axis and IGF-I on GHRH-GH axis. IGF-I acts on many cells that produce it like chondrocytes. None of the above C. breast epithelium & gonadal cells 35 Pit-1. glucose inhibition of insulin secretion. It is connected to hypothalamus by pituitary (or hypophysial) stalk. and DAX-1 are lineage-specific transcription factors in pluripotential stem cells during embryonic gland is ? differentiation. 921 C. 600 mg Harrison’s 18th Ed. posterior pituitary - A. IGF-I Adrenocorticotropin hormone (ACTH). C. Breast epithelium hypothalamus is called median eminence. Pituitary development Pituitary and Hypothalamus C. 2871 D. Thyroid hormone D. IGF-I & TH stimulate growth. 4 of nuclear receptors. 2872 B. None of the above C. Pit-1 determines cell-specific expression of GH (somatotropes). PRL (lactotropes) Harrison’s 18th Ed. and DAX-1 are related to ? (autocrine regulation). anterior pituitary originates from Rathke’s pouch. Estrogen-mediated stimulation of mid-cycle LH surge D. which connects inferiorly with pituitary stalk. 6 Harrison’s 18th Ed. 7 A. which is an embryonic invagination of the pharyngeal epithelium. while posterior D. Corticotropin upstream transcription B. Located within sella turcica ventral to the diaphragma sella. Pituitary hormones are secreted in pulsatile manner regulatory system ? B. 2872 arteries A. relatively avascular zone called pars intermedia. and leptin feedback on 33 “Median eminence” is best described as ? hypothalamus. Paracrine regulation means factors released by one cell that C. . cortisol on CRH-ACTH axis. Autocrine regulation means action of a factor on the same cell 34 “Tuber cinereum” is best described as ? from which it is produced Guyton’s Textbook of Medical Physiology 11th Ed. C. Lowermost portion of pituitary act on an adjacent cell in the same tissue D. Follicle-stimulating hormone (FSH)& Thyroid-stimulating hormone (TSH). Growth hormone (GH). Embryologically. GH Six major hormones produced by anterior pituitary gland are Prolactin (PRL). Gonadal steroids on GnRH-LH/FSH axis effective transmission of hypothalamic peptide pulses to it without significant systemic dilution. 2876 GH. 2876 27 Which of the following is not a negative hormonal feedback A. 2876 & TSH (thyrotropes). Thyroid hormones D. Sex steroids 31 The pituitary gland weighs about ? Harrison’s 18th Ed. Levels of paracrine & autocrine control factors cannot be A. 921 A. Hypothalamus development Chapter 339. Prop-1 induces the pituitary development of Pit-1-specific lineages. Anteriormost portion of pituitary B. 300 mg pituitary gland ? C. steroidogenic factor (SF-1) & DAX-1. Extension of anterior pituitary tissue into pars intermedia 29 IGF-I acts ? D. Gonadal cells is an extension of hypothalamic tissue into pituitary stalk. Pancreatic development D. 2872 Pituitary gland is also called hypophysis (anterior pituitary . Cortisol on CRH-ACTH axis Major blood supply for anterior pituitary is hypothalamic-pituitary portal plexus which allows D. Chondrocytes neurohypophysis).thyroid hormones on TRH-TSH axis. B. Pit-1. All of the above pituitary originates from a neural tissue outgrowth from hypothalamus. Between these two portions of pituitary is a small. 525 Endocrinology MCQ’s FOR MCQ’s FOR MEDICAL MEDICAL PROFESSIONALS PROFESSIONALS BY PROF. Following are the negative feedback regulatory systems . Leptin D. SF-1. Prop-1. Posterior lobe is innervated by hypothalamic neurons B. Extension of pituitary tissue into pituitary stalk readily measured B. AJAY MATHUR Cardiology 525 25 Which of the following does not stimulate growth ? C. 2871 A. Posterior pituitary is supplied by inferior hypophyseal arteries. Disorders of the Anterior B. 2877 A. Extension of hypothalamic tissue into pituitary stalk D. 100 mg 26 Feedback regulation for which of the following does not involve B. 800 mg B. Thyroid hormones on TRH-TSH axis C. Positive feedback control is estrogen-mediated stimulation of mid-cycle LH surge. All of the above 30 The number of major hormones produced by anterior pituitary Anterior pituitary gland develops from nasopharyngeal Rathke’s pouch. Tuber cinereum C. 5 element (CUTE) & PTX-1 transcription factor play a role in development of corticotrope cells. Lowermost portion of B. Posterior pituitary is supplied by superior hypophyseal Harrison’s 18th Ed. Uppermost portion of pituitary A. Lowermost portion of hypothalamus 28 Which of the following is false ? Harrison’s 18th Ed. whereas sex steroids lead to epiphyseal closure. Luteinizing hormone (LH). IGF-I Pituitary gland weighs ~600 mg. A. with mildly elevated PRL levels. D. Prop-1 is essential for Pit-1 function.3 is the culprit. ACTH Harrison’s 18th Ed. hypoglycemia. Prolactin C. C. Patients of Prader-Willi Syndrome have mental Harrison’s 18th Ed. GH B. All of the above B. 2878 D. obesity & hypogonadotropic hypogonadism. hyperphagia. 2879 D. KAL gene defect on chromosome Xp22. Patients of Laurence-Moon-Bardet-Biedl Syndrome have mental retardation. Seizure . Postpartum period hypogonadism. PROP1 mutations result in combined GH. bilateral visual changes. Gonadotropin ? Harrison’s 18th Ed. Hypotension Laurence-Moon-Bardet-Biedl Syndrome. Visual loss deficiency with obesity. 4000 rad 37 PROP-1 mutations result in deficiency of all except ? D. Pregnant female neurons from hypothalamic olfactory placode to hypothalamus is prevented due to this genetic defect. Low levels of sex steroids (testosterone or estradiol) A. hypertension. PRL & TSH deficiencies B. Unmarried female synthesis. Low LH 46 Sheehan’s syndrome refers to pituitary apoplexy during ? B. adult-onset diabetes mellitus. Color blindness prolonged glucocorticoid treatment. 2879 Kallmann syndrome is due to defective hypothalamic gonadotropin-releasing hormone (GnRH) A. Laurence-Moon-Bardet-Biedl Syndrome apoplexy is ? C. GH deficiency is most common. Embryonic migration of GnRH B. Frohlich Syndrome & Prader-Willi Syndrome have GnRH B. B. 2879 C. cryptorchidism & neurologic abnormalities (mirror movements). Low FSH Harrison’s 18th Ed. Males present with delayed puberty & C. PRL. Pituitary Apoplexy (acute intrapituitary hemorrhage) may occur spontaneously in a preexisting adenoma. Short statured female 39 Which of the following is not a feature of Kallmann syndrome ? D. GnRH Harrison’s 18th Ed. B. CNS hemorrhage & death. diabetes. 5000 rad Harrison’s 18th Ed. Nerve deafness 45 Pituitary apoplexy is associated with ? D. All of the above A. color blindness. renal abnormalities. postpartum (Sheehan’s syndrome). 2878 A. Frohlich Syndrome Harrison’s 18th Ed. Infertile female Harrison’s 18th Ed. nerve A. 2000 rad Autosomal dominant or recessive Pit-1 mutations cause combined GH. hypotension. All of the above A. FSH After cranial irradiation. A. It presents as severe headache with signs of meningeal irritation. Antenatal period D. GH median dose of 50 Gy (5000 rad) directed at skull base. 2879 D. Prader-Willi Syndrome 47 Indication for urgent surgical decompression after pituitary B. TSH & gonadotropin deficiency with preservation of ACTH. 2878 Lymphocytic hypophysitis occurs mainly in pregnant or post-partum women & presents with A. 2877 Up to two-thirds of patients ultimately develop hormone insufficiency after a cranial irradiation A. C. Hypopituitarism occurs over 5 . Females present with primary amenorrhea & failure of secondary sexual development. cleft palate. GH B.15 years & reflects hypothalamic damage rather than primary destruction of pituitary cells. Hyposmia hyperprolactinemia & a pituitary mass on MRI. LH D. 3000 rad resulting in growth failure & hypothyroidism. Sickle cell anemia Harrison’s 18th Ed. Diabetes deafness. All of the above A. optic atrophy. Testosterone 44 Lymphocytic hypophysitis occurs mainly in ? D. Women are treated with cyclic estrogen & progestin. Prolactin 42 What median dose of cranial irradiation can lead to development of hypopituitarism ? C. Precocious puberty Harrison’s 18th Ed.526 MCQ’s FOR MEDICAL PROFESSIONALS BY PROF. ADH B. obesity & central hypogonadism. Resolves after B. A. 2877 retardation. retinal degeneration & central diabetes insipidus. All of the above Fertility may be restored in men with long-term treatment with human chorionic gonadotropin (hCG) or testosterone. C. ACTH 43 Which hormone deficiency is most common after cranial irradiation D. Intra-partum period GnRH deficiency prevents progression through puberty. Gonadotropin 38 Kallmann syndrome is due congenital synthesis defect of ? C. TSH Harrison’s 18th Ed. Hypertension 40 Hormone profile of Kallmann syndrome includes ? C. 41 GnRH deficiency is found in which of the following ? ophthalmoplegia. C. sickle cell anemia or acute shock. Patients of Frohlich Syndrome have hyperphagia. Hypopituitarism is very Harrison’s 18th Ed. Repetitive GnRH administration restores normal pituitary gonadotropin responses. 2879 During development. Pituitary CT / MRI is diagnostic. 2879 Features of Kallmann syndrome include anosmia or hyposmia. 2879 common after apoplexy. followed by gonadotropin & ACTH deficiency. AJAY MATHUR Endocrinology 36 Pit-1 mutations can cause deficiency of ? defects of fingers. C. GH responses to insulin-induced hypoglycemia. Increased B. 2883 Harrison’s 18th Ed. 2882 C. Pituitary carcinomas with include visual field abnormalities. is inactivated by mutation. 2878 Harrison’s 18th Ed. They are often large. 53 “CREB” stands for ? 58 Diabetes insipidus can be caused by ? Harrison’s 18th Ed. severe ophthalmoplegia D. Punched-out lytic bone lesions C. ~ 15 % D. Normal Harrison’s 18th Ed. None of the above Craniopharyngiomas. anterior pituitary dysfunction with tumor size. Metyrapone test pituitary tumors ? Harrison’s 18th Ed. 2879 54 Factors involved in initiation and promotion of pituitary tumors include ? A. Carney syndrome ACTH reserve is most reliably assessed during insulin-induced hypoglycemia. All of the above enough for hormone production. pancreatic islet & pituitary adenomas occurs in autosomal A. Posterior pituitary C. All of the above and ovarian tumours. arginine. Loss of negative-feedback inhibition D. personality changes & cognitive deterioration. Cholesterol response element binding protein A. ~One-third of all adenomas are clinically nonfunctioning & produce no distinct & diabetes insipidus. Harrison’s 18th Ed. Other factors involved 49 ACTH reserve is most reliably assessed by ? in initiation & promotion of pituitary tumors include loss of negative-feedback inhibition & estrogen- mediated or paracrine angiogenesis. Basic fibroblast growth factor (bFGF) C. Monoclonal in origin B. Arginine MENIN allele. cystic. All of the above Harrison’s 18th Ed. Hormone production does not always correlate retardation. growth documented extracranial metastases are very rare. cranial nerve damage. Calcium response element binding protein Patients of pituitary apoplexy with significant or progressive visual loss. Decreased A. Pituitary imaging detects small pituitary lesions in at least 10% of normal individuals. 2880 A. pigmented skin patches. Multiple endocrine neoplasia (MEN) 1 D. 56 Which of the following is not a feature of Hand-Schuller-Christian 51 Pituitary adenomas account for what percentage of all intracranial disease ? neoplasms ? Harrison’s 18th Ed. glands have microadenoma (<10 mm diameter). Cyclic AMP response element binding protein B. L-dopa. is a critical transcription factor for neuronal survival which plays an important role in memory in the hippocampus. Rest can be managed with high- dose glucocorticoids. the tumor-suppressor gene on chromosome 11q13. 527 Endocrinology MCQ’s FOR MCQ’s FOR MEDICAL MEDICAL PROFESSIONALS PROFESSIONALS BY PROF. 2880 A. Estrogen-mediated or paracrine angiogenesis Empty Sella patients usually have normal pituitary function. Insulin-induced hypoglycemia dominant Multiple endocrine neoplasia (MEN) 1 because “MENIN”. Septo-Optic Dysplasia B. AJAY MATHUR Cardiology 527 D. 2880 Genetic predisposition to parathyroid. 2880 A. Hand-Schuller-Christian disease refers to a constellation of diabetes insipidus. Hypothalamus D. ~2% B. Any of the above C. Carney syndrome is due to mutations in R1  regulatory subunit of protein kinase A (PRKAR1A). 25% of all pituitary & typical axillary skin rash. though with time hypopituitarism may develop. They Almost all pituitary adenomas are monoclonal in origin due to somatic mutation leading to increased present frequently before age 20. McCune-Albright syndrome refers to polyostotic fibrous dysplasia. Hormonal hypersecretion is secondary to to cyclic AMP overproduction caused by postzygotic inactivation of GTPase activity of Gs . 2880 A. 2881 B. Bardet-Biedl Syndrome . Testicular. 48 In ‘Empty Sella syndrome’.or plurihormonal neoplasms. CREB (cyclic adenosine monophosphate responsive element-binding) protein. Cancer response element binding protein or loss of consciousness require urgent surgical decompression. Other presentations growth. growth hormone releasing hormone (GHRH) or growth hormone releasing peptides (GHRPs) can be used to assess GH reserve. Anterior pituitary B. Standard ACTH stimulation test B. C. with signs of increased intracranial pressure. All of the above C. Loss of heterozygosity (LOH)of the remaining normal B. Hormone production does not correlate with tumor size D. locally invasive & partially calcified. leads to tumorigenesis. Peripheral rim of pituitary tissue is D. punched-out lytic bone lesions Pituitary adenomas account for ~15% of all intracranial neoplasms. sleep difficulties. 57 Craniopharyngiomas arise near which of the following ? 52 Which of the following is false about pituitary adenomas ? Harrison’s 18th Ed. and GH-secreting pituitary tumors. L-dopa skin pigmentation & myxomas are the characteristic features. ~5% C. adrenal & pituitary adenomas along with spotty C. Basic fibroblast growth factor (bFGF) stimulate pituitary cell mitogenesis. Axillary skin rash D. Pituitary stalk A. Insulin-induced hypoglycemia A. derived from Rathke’s pouch. adrenal D. ~ 20 % Foci of eosinophilic granulomas are a typical feature of Histiocytosis X. At autopsy. Exophthalmos B. Diabetes mellitus A. pituitary functions are usually ? Harrison’s 18th Ed. exophthalmos. McCune-Albright syndrome 50 Which of the following can be used to assess GH reserve ? D. weight gain. CRH test 55 Which of the following is a genetic syndrome associated with B. Pituitary adenomas are benign uni. Benign neoplasms C. arise near the pituitary stalk & extend into suprasellar cistern. 2883 Harrison’s 18th Ed. clinical hypersecretory syndrome. 2883 60 Diabetes insipidus can be caused by ? A. B. Any of the above Harrison’s 18th Ed. 2883 . Ventromedial nuclei 70 Which of the following is an abnormal feature of pituitary MRI ? D.528 MCQ’s FOR MEDICAL PROFESSIONALS BY PROF. along with sweating. 2884 Table 339-7 abnormalities. Compression of intrasellar pituitary tissue B. vasodilation. Central hypothalamus Harrison’s 18th Ed. A. Anterior and preoptic hypothalamus Harrison’s 18th Ed. Convex upper aspect of adult pituitary C. Posterior hypothalamus A. Ventromedial nuclei 59 Diabetes insipidus can be caused by ? Harrison’s 18th Ed. 2883 A. Ventromedial nuclei 62 Blood-borne metastatic deposits in pituitary gland are found mostly in ? D. cardiac. 2884 A. Brain germ-cell tumors Harrison’s 18th Ed. 2883 C. Central hypothalamus A. Bony erosion A. 2878 67 Central osmo-receptors are located in which part of A. B. POMC & its products & gastrointestinal peptides. All of the above hypothalamus ? Harrison’s 18th Ed. Posterior hypothalamus C. Posterior hypothalamus C. Posterior pituitary 68 Elevated serum catecholamine and cortisol levels result due to C. Anterior and preoptic hypothalamus Pituitary metastases occur in ~3% of cancer patients and blood-borne metastatic deposits are found almost exclusively in posterior pituitary. insulin. vomiting & bradycardia. Pituitary stalk lesion in which part of hypothalamus ? D. B. Preoptic nulcei of hypothalamus D. imperforate anus. B. renal & lung disorders is called Pallister-Hall syndrome which is caused by mutations in the carboxy terminus of the GLI3 gene. Anterior and preoptic hypothalamus When an intrasellar mass expands. Pituitary metastases hypothalamus ? B. Craniopharyngiomas. Lateral invasion to impinge on cavernous sinus hypothalamus ? Harrison’s 18th Ed. Hypothalamic gliomas Stimulation of sympathetic neurons with consequent elevation of serum catecholamine & cortisol levels occurs due to lesions of central hypothalamus and are predisposed to cardiac arrhythmias. Hand-Schüller-Christian disease Ventromedial nuclei of hypothalamus contain the energy-satiety responsive melanocortin receptors D. Posterior hypothalamus C. 2883 Central osmo-receptors are located in preoptic nuclei of hypothalamus. All of the above which are influenced by leptin. Hypothalamic hamartomas hypertension & gastric erosions. Slightly heterogeneous soft tissue consistency Harrison’s 18th Ed. Craniopharyngiomas C. Diabetes insipidus is the presenting feature. It is due to lesion in posterior hypothalamus. All of the above B. 2883 D. B. Central hypothalamus A. Amyloidosis A. C. Compression of intrasellar pituitary tissue is the first event. Central hypothalamus Harrison’s 18th Ed. Anterior pituitary associated with polydipsia & hypodipsia. 2878 D. 2878 B. Anterior and preoptic hypothalamus D. B. 2883 D. Rathke’s cleft cysts D. Harrison’s 18th Ed. Posterior hypothalamus C. Hemochromatosis 66 Hyperphagia and obesity results due to lesion in which part of D. Transsphenoidal Surgery A. Rathke’s cysts intrasellar mass ? Pituitary failure secondary to lesions around pituitary due to hamartoma along with craniofacial Harrison’s 18th Ed. Pituitary metastases 69 Which of the following is “rare” in the process of expansion of an D. Damage to them is A. Histiocytosis X C. Ventromedial nuclei 63 Pallister-Hall syndrome is associated with ? Harrison’s 18th Ed. AJAY MATHUR Endocrinology C. Central hypothalamus C. All of the above B. bony erosion & direct brain compression is rare. Invasion of dura to lift the optic chiasm 64 Hyperthermia results due to lesion in which part of C. trauma or inflammation can cause damage to ventromedial nuclei of 61 Diabetes insipidus can be caused by ? hypothalamus leading to hyperphagia & obesity. Sarcoidosis Periodic hypothermia syndrome refers to episodic attacks of hypothermia. Ventromedial nuclei B. Vertical pituitary stalk 65 Periodic hypothermia syndrome results due to lesion in which part of hypothalamus ? D. Pituitary gland height is <8 mm in adults Hemorrhage in anterior & preoptic hypothalamus may cause acute hyperthermia accompanied by paradoxical vasoconstriction & tachycardia. All of the above 72 On MRI. Meningiomas are associated with bony hyperostosis B. Sexual orgasm 77 Which of the following is false about prolactin (PRL) ? C. Normal adult serum PRL levels is 10-20 µg/L in men Harrison’s 18th Ed. Acromegaly 73 On MRI. “pituitary bright spot” is due to high content in the B. All of the above A. Fats C. Circulating half-life of PRL is ~50 minutes. 2887 Table 339-9 D. Posterior pituitary 79 The most common pituitary hormone hypersecretion syndrome C. Cushing’s syndrome B. Pituitary stalk compression B. Bitemporal hemianopia or superior Harrison’s 17th Ed. Primary hypothyroidism causes hyperprolactinemia because of enhanced TRH secretion. Prolactin inhibitory hormone resembles catecholamine dopamine. Phospholipid Harrison’s 18th Ed.6 AM . “pituitary bright spot” is due to ? Harrison’s 18th Ed. Chest stimulation Harrison’s 18th Ed.12 mm. PRL secretion is pulsatile A. Sleep D. damage to hypothalamus or blockage of C. Pregnancy and lactation Harrison’s 18th Ed. Hyperprolactinemia posterior pituitary of ? C. 2204 Table 333-8 bitemporal defects are seen due to pressure of the expanding pituitary tumour because these tracts are located within inferior & posterior part of chiasm. Glucose 80 When PRL level is >200 µg/L. Optic tract D. A. 2887 D. 2887 D. secreted by arcuate nuclei of B. PRL secretory peaks occur during NREM sleep B. where it has an inhibitory effect. Bitemporal hemianopia hypothalamus and decreases prolactin secretion. Predominant central control mechanism is inhibitory D. Primary hypothyroidism When larger pituitary masses (>1 cm) are encountered. None of the above 71 Which of the following is false regarding “nonadenomatous Normal adult serum PRL levels are ~10 to 25 µg/L in women & 10 to 20 µg/L in men. 2887 C. 2887 D. Estrogens & monocular field cuts are prechiasmal. Protein A. Homonymous field defects are postchiasmal A. C. AJAY MATHUR Cardiology 529 Pituitary gland height is ~6 mm in children to 8 mm in adults. 2887 D. It increases during pregnancy & B. Peak Harrison’s 18th Ed. Cirrhosis D. Central control mechanism of PRL is inhibitory. Normal adult serum PRL levels is 10-25 µg/L in women 83 Which of the following causes increase in prolactin level ? B. None of the above A. Optic nerve B. Thus. Circulating half-life of PRL is 50 minutes puberty to ~10 . Anterior pituitary B. Peak serum PRL levels occur between 4 . Craniopharyngiomas may be calcified & are hypodense 78 Systemic disorder associated with hyperprolactinemia is ? C. 1040 75 Loss of red perception is due to pressure on ? Harrison’s 18th Ed. they must be distinguished from nonadenomatous lesions like meningioma. Hypothalamus stimulates A. All of the above 81 “Colostrum” does not contain which of the following ? Guyton’s Textbook of Medical Physiology 11th Ed. A. All of the above 82 Hormonal agents that induce prolactin include ? Loss of red perception is an early sign of optic tract pressure. becomes convex in adolescent & pregnant. PRL secretion pituitary lesions” ? is pulsatile. All of the above A. Tryptophan D. None of the above A. Calcium D. Chronic renal failure B. Loss of red perception production of all anterior pituitary hormones except prolactin. 2884 A. Scotoma hypothalamic-hypophysial portal system increases prolactin secretion. Prolactinoma High phospholipid content of posterior pituitary results in a “pituitary bright spot” on MRI. Upper aspect of adult pituitary is flat or slightly concave. Secondary hyperthyroidism Harrison’s 18th Ed. Optic chiasm C. D. TRH Harrison’s 18th Ed. 2884 Chronic renal failure elevates prolactin by decreasing peripheral prolactin clearance. the most common cause is ? C. with the highest secretory peaks occurring during rapid eye movement sleep. Antiandrogens 76 Which of the following is false about prolactin (PRL) ? C. Lactose B. Gliomas are hyperdense on T2-weighted images Harrison’s 18th Ed. C. Pituitary stalk in both males and females is ? Harrison’s 18th Ed. 2885 A. craniopharyngiomas & gliomas. Ascorbic acid A. 2885 serum PRL levels occur between 4 & 6 AM. Antipsychotic and antidepressant drugs 74 Features of sellar mass lesions involving optic chiasm include ? D. 2884 Table 339-7 Prolactinomas are the most common cause of PRL levels >200 µg/L. 529 Endocrinology MCQ’s FOR MCQ’s FOR MEDICAL MEDICAL PROFESSIONALS PROFESSIONALS BY PROF. B. C. Alpha-methyldopa inhibits dopamine synthesis. Reserpine Female:male ratio for microprolactinomas is 20:1. They do not invade parasellar structures. mixed tumors that secrete prolactin ? Harrison’s 18th Ed.530 MCQ’s FOR MEDICAL PROFESSIONALS BY PROF. Verapamil D. 2888 B. 2888 Harrison’s 18th Ed. Chlorpromazine D. 10:1 C. < 0. Decreased libido C. weight gain & hirsutism. 20:1 D. reduced vertebral bone mineral density due to hypoestrogenemia. the inappropriate discharge of milk-containing fluid from the breast. Haloperidol Galactorrhea. butyrophenones like haloperidol. Alpha-Methyldopa B. < 15 µg/L Harrison’s 18th Ed. 5:1 B. 2889 Harrison’s 18th Ed. ACTH & PRL & rarely TSH & PRL are also seen. 2888 A. > 9 months after childbirth or discontinuation of breastfeeding C. 1:1 A. 2887 Table 339-9 A. whereas it is 1:1 for macroadenomas. Haloperidol manner & levels vary widely. > 12 months after childbirth or discontinuation of breastfeeding D. Metoclopramide C. D. AJAY MATHUR Endocrinology 84 Which of the following produce “drug-induced B. < 20 µg/L A. galactorrhea. > 6 months after childbirth or discontinuation of breastfeeding B.2 cm A. Metoclopramide C. 2888 88 Galactorrhea is present in what percentage of hyperprolactinemic A. Estrogens. fasting morning PRL levels are normally <20 g/L. what is its cutoff 89 Which of the following in not a presenting feature of diameter ? hyperprolactinemia in women ? Harrison’s 18th Ed. Assays on multiple occasions may be needed to document C. < 10 µg/L hyperprolactinemia” by blocking dopamine receptors ? C. 10:1 C. Reserpine 93 The female:male ratio for macroadenomas is ? 87 Which of the following produce “drug-induced Harrison’s 18th Ed. Metoclopramide hyperprolactinemia. < 1.0 cm D. Prolactin secretion occurs in a pulsatile B. Hirsutism Pituitary tumours <1 cm in diameter are termed microadenomas. Metoclopramide C. 2888 A. antiandrogens 94 Which of the following is the least common hormone secreted by & TRH induce prolactin. Chlorpromazine D. D. infertility. 2888 Harrison’s 18th Ed. 5:1 B. GH women ? Harrison’s 18th Ed. “Macroprolactinemia” refers to biologically inactive aggregated forms of circulating prolactin. < 0. Dopamine receptors are blocked by Phenothiazines like chlorpromazine.5 cm B. Males tend to present with larger macroadenomas than females because of delay in appreciating signs & symptoms.8 cm C. Chief presentations of hyperprolactinemia in women are oligomenorrhea / amenorrhea. Mixed tumors secreting combinations of GH & PRL. 2888 A. Weight loss D. Alpha-Methyldopa B. Reduced vertebral bone density B. It is a cause of false elevation of prolactin levels. 96 If fertility is not desired. TSH B. 2887 Table 339-9 A. Verapamil blocks dopamine release. Alpha-Methyldopa B. 2888 hyperprolactinemia” by blocking dopamine release ? Harrison’s 18th Ed. 2888 hyperprolactinemia” by inhibiting dopamine synthesis ? A. < 5 µg/L A. > 3 months after childbirth or discontinuation of breastfeeding Harrison’s 18th Ed. Dopamine agonists . ACTH A. perphenazine. is considered abnormal if it persists for > 6 months after childbirth or discontinuation of breastfeeding. Chlorpromazine Basal. 86 Which of the following produce “drug-induced 92 The female : male ratio for microprolactinomas is ? hyperprolactinemia” by depleting catecholamine ? Harrison’s 18th Ed. Up to 95% 95 For defining “microadenoma” of pituitary gland. decreased libido. Macroadenomas are >1 cm in diameter and are locally invasive & encroach on surrounding structures. All of the above 91 Breast milk secretion is considered abnormal if it persists for ? 85 Which of the following produce “drug-induced Harrison’s 18th Ed. 2887 Table 339-9 D. Up to 90% Tumors arising from lactotrope cells account for ~ half of all functioning pituitary tumors. Up to 80% C. Reserpine is a catecholamine depletor. 20:1 D. what should be the line of management 90 What is the normal basal. < 0. Up to 85% D. None of the above C. fasting morning prolactin level ? of microadenomas ? Harrison’s 18th Ed. thioxanthenes & metoclopramide. 1:1 A. 2890 A. Ventromedial nuclei D. Harrison’s 18th Ed. 2890 C. this fungus synthesizes histamine. Medial preoptic C. Suppressing prolactin secretion C. Harrison’s 18th Ed. Trichosporon asahii A. IGFI. 531 Endocrinology MCQ’s FOR MCQ’s FOR MEDICAL MEDICAL PROFESSIONALS PROFESSIONALS BY PROF. In ~30% of microadenomas. The pituitary GH gene (hGH- N) produces two kinds of GH (22-kDa & 20-kDa). 2890 A. D. All of the above Harrison’s 18th Ed. No treatment somatotrope development. Such asymptomatic patients should be monitored by regular serial prolactin and MRI. Anthony’s fire). 2890 physical stress. Cabergoline has a half-life of 63 . Harrison’s 16th Ed. 2206 B. Quinagolide is a nonergot drug with similar D2 receptor affinity. GH B. Mutations in transcription factors Pit-1 & Prop-1. ACTH C. 2890 treatment of hyperprolactinemia is ? Harrison’s 18th Ed. Acetate B. Suppressing lactotrope cell proliferation 104 The name for pituitary GH gene is ? D. 2890 B. Elevated GH levels occur within an hour of deep sleep onset as well as after exercise. hyperprolactinemia resolves spontaneously. By middle 102 Development & proliferation of somatotrophs is determined by age. feeds back to inhibit GH. the peripheral target hormone for GH. Pancreas C. Sepsis GH secretion is pulsatile. gastrointestinal tract & pancreas. Prophet of Pit-1 (PROP1) 108 GH secretion is induced by all except ? B. Mesylate C. Lysergic acid diethylamide area of the hypothalamus & inhibits GH secretion. both with similar biologic activity. No treatment is advocated. 21 C. Prolactin 107 Elevated GH levels occur in all of the following except ? Harrison’s 18th Ed. GH secretion is reduced in obese which gene ? individuals. Suppressing prolactin synthesis D. SRIF is also expressed in CNS. which control D. hGH-P B. Besides D. trauma & during sepsis. Central Claviceps purpurea is known for epidemics of ergot poisoning (ergotism or St. Estrogen 101 The most abundant anterior pituitary hormone is ? induces GH whereas glucocorticoid excess suppresses GH release. GH is the most secreted hormone from the GH-secreting somatotrope cells and mammosomatotrope cells. hGH-Z C. Lisuride D. Deep sleep Of the six anterior pituitary hormones. GH variant gene D. 2085 which area of hypothalamus ? A.69 hours. where it also acts to inhibit islet hormone secretion. TSH A. Quinagolide GH & related proteins are produced by five genes on chromosome 17. 2087 A. Sulphate GHRH is secreted as discrete spikes that elicit GH pulses. A. with greatest levels at night correlating with onset of sleep. Surgery D. Gastrointestinal tract B. 2889 B. hGH-V 98 Which of the following is a nonergot oral dopamine agonist ? Harrison’s 17th Ed. CNS A. Prophet of Pit-4 (PROP4) C. Radiation Cell-specific Pit-1 nuclear transcription factor is responsible for development of somatotropes & subsequent transcription of GH. if fertility is not 103 Gene that encode GH is located on which chromosome ? desired. 2890 A. estrogen replacement may be given to prevent bone loss. 17 B. However. cause GH deficiency in combination with other pituitary hormone deficiencies. 9 97 Dopamine agonists act by ? Harrison’s 18th Ed. hGH-N A. whereas SRIF sets basal GH tone. Prophet of Pit-3 (PROP3) A. which coexpress PRL with GH. cabergoline & pergolide are ergot alkaloids that have the highest suppressive selectivity for pituitary dopamine D2 Somatostatin [somatotropin-release inhibiting factor (SRIF)] is synthesized in the medial preoptic receptors. Pergolide mesylate C. 106 Somatotropin-release inhibiting factor (SRIF) or somatostatin is expressed by which of the following ? 100 Salt of bromocriptine used as dopamine receptor agonist for Harrison’s 18th Ed. Exercise D. 12 A. AJAY MATHUR Cardiology 531 B. Sporothrix schenckii B. Dopamine . All of the above ergot alkaloids. 99 Fungus that produces “Ergot alkaloids” is ? 105 Somatotropin-release inhibiting factor (SRIF) is synthesized in Harrison’s 16th Ed. Bromocriptine (hGH-V) is expressed by placental syncytiotrophoblast. 2890 B. Prophet of Pit-2 (PROP2) Harrison’s 18th Ed. Bromocriptine. acetylcholine & tyramine. Hortaea werneckii Harrison’s 18th Ed. Obesity C. (LSD) is a synthetic ergot compound & its behavioral effects are mediated by agonist effects at prejunctional or postjunctional 5-HT2 receptors in CNS. Claviceps purpurea C. Pituitary microadenomas are slow growing tumours and only ~5% go on to be macroadenomas. GH secretory rates is ~15% than that during puberty. All of the above D. Cryptate D. 921 C. Liver . All of the above D. 120 Which of the following about serum IGF-I concentration is false ? Harrison’s 18th Ed. C. Promotes sodium. Insulin-like growth factor (IGF-1) Both IGF-I and -II are bound to high-affinity circulating IGF-binding proteins (IGFBPs) that regulate IGF bioactivity. Activated STAT proteins translocate GH secretion is induced by dopamine & apomorphine. Promotes nitrogen retention B. Higher in males than in females Harrison’s 18th Ed. All of the above B. where they modulate expression of GH-regulated target genes. Enhances nitrogen retention Ghrelin. 2891 C. and it serves as the major carrier protein for D. AJAY MATHUR Endocrinology B. 2891 114 Which of the following organs has the greatest number of GH A. Apomorphine C. IGFBP1 113 Somatomedin C is best described as ? Guyton’s Textbook of Medical Physiology 11th Ed. Pancreas 112 Somatostatin is best described as ? B. Lowers cholesterol level D. GH binding induces receptor dimerization. All of the above A. 2891 GH secretion ? Harrison’s 18th Ed. to the nucleus. Gastrointestinal tract B. SSTR2 & SSTR5 subtypes preferentially suppress GH (& TSH) secretion. B. Heart A. Muscle D. Growth hormone releasing hormone (GHRH) C. potassium. Induces protein synthesis A. or octonoylated gastric-derived peptide. C. 2891 D. None of the above circulating IGF-I. 2072 GH impairs glucose tolerance by antagonizing insulin action. Peripheral nervous system 117 Which of the following is an action of IGF-I ? Harrison’s 18th Ed. Somatostatin analogues exert their therapeutic effects A. Levels low in sepsis A. Prolactin inhibitory hormone (PIH) 119 Which fraction of circulating IGF-binding proteins (IGFBP) is a Growth hormone inhibitory hormone (somatostatin) is a single chain of 14 amino acids Inhibits major carrier for circulating IGF-I ? secretion of growth hormone by somatotropes. and lower doses improve insulin sensitivity in patients with severe insulin resistance A. Chondrocytes and diabetes. followed by signaling through JAK/STAT pathway.355:2558-73 D. Growth hormone releasing hormone (GHRH) Major source of circulating IGF-I is liver. 2891 A. Liver Guyton’s Textbook of Medical Physiology 11th Ed. 115 Which of the following is an action of GH ? 109 Which somatostatin receptor subtype preferentially suppresses Harrison’s 18th Ed. Growth hormone inhibitory hormone (GHIH) D. IGF-I is a potent growth & differentiation factor. 2891 Somatostatin binds to five distinct receptor subtypes (SSTR1 to SSTR5). SSTR4 116 Which of the following is an action of GH ? Harrison’s 18th Ed. IGFBP2 A. Lungs A. All of the above C. All of the above A.and insulin-evoked GH release. Heart C. as well as by -adrenergic pathways. -adrenergic blockage D. Gonadal cells Harrison’s 18th Ed. Pancreas D. 924 B. Growth hormone exerts its effect through intermediate substances called Somatomedins . Muscle B. SSTR3 D. Levels high in acromegaly B.also called “Insulin-Like Growth Factors” formed in liver under the influence of GH. Levels of IGFBP3 are GH-dependent. Breast epithelium 118 The major source of circulating IGF-I is from ? C. as well as synthetic agonists of the GHRP receptor stimulate GHRH & also directly stimulate GH release. Stimulates epiphyseal prechondrocyte differentiation N Engl J Med 2006. Levels peak at 16 years of age receptors ? B. IGFBP3 B. and water retention 110 Ghrelin is synthesized mainly in ? C. Growth hormone inhibitory hormone (GHIH) D. IGFBP1 & -2 regulate local tissue IGF action but do not bind circulating IGF-I. While. Corticotropin-releasing hormone (CRH) D. SSTR1 B. both of which are expressed by GH-secreting tumors.  . 2890 A. Impairs glucose tolerance by antagonizing insulin action C. SSTR2 C. All of the above 111 IGF-I acts on ? Harrison’s 16th Ed. B. Stimulates lipolysis through SSTR2 & SSTR5 receptors. Somatostatin Liver & cartilage contain the greatest number of GH receptors. 2891 C.532 MCQ’s FOR MEDICAL PROFESSIONALS BY PROF. Adrenergic blockage induces basal GH and enhances GHRH. Harrison’s 18th Ed. Induces hypoglycemia D. Peripheral tissue IGF-I exerts local paracrine actions that appear to be both dependent & independent of GH. injected IGF-I induces hypoglycemia. High IGF-I levels D. 2891 B.5 cm (girls) from the midparental height. 4 cm/year C. 6 cm/year D. 2892 124 Normal bone age in a child with short stature is suggestive of ? A. GH Harrison’s 18th Ed. IGF-I levels are low with cachexia. Bayley-Pinneau scale Harrison’s 18th Ed. AJAY MATHUR Cardiology 533 Serum IGF-I levels increase during puberty. Though GH levels are increased in sepsis. ~ 40 % As GH secretion is released in pulses. Genetic cartilage dysplasia A. (girls) or 13 (boys). D. ~ 30 % D. and low IGF-I levels. Genetic cartilage dysplasia or growth plate disorder Final height can be predicted using standardized scales (Bayley-Pinneau or Tanner-Whitehouse) D. All of the above or estimated by adding 6. 125 Delayed bone age in a child with short stature is suggestive of ? 131 Treatment with IGF-I is recommended for which of the Harrison’s 18th Ed. with IGF-I bypasses the dysfunctional GH receptor. micropenis. 2892 normal energy production ? Harrison’s 18th Ed. IGF-I. malnutrition & sepsis (hypocaloric states are associated with GH D. 2892 B. 128 Which of the following is false about Laron syndrome ? 122 In later childhood. All of the above Bone age is delayed in patients with all forms of true GH deficiency or GH receptor defects that result in attenuated GH action. Turner syndrome C. 126 Characteristic voice in isolated GH deficiency is ? 132 Which is the “last” hormone to be lost in acquired pituitary Harrison’s 18th Ed. Tanner-Whitehouse scale A. ~ 20 % C. Bone age is delayed by thyroid hormone deficiency.5 cm (boys) or subtracting 6. accelerating growth rates in children with Turner syndrome & chronic renal failure. Secondary sexual development is associated with elevated sex steroids that cause progressive epiphyseal growth plate closure. Decreased circulating GHBP C. increased fat. Growth-promoting process requires caloric energy. 2 cm/year B. Any of the above resistance). vitamins & trace metals & consumes ~10% of normal energy production. Hormonal disorder C. IGF-II B. Midparental height B. 533 Endocrinology MCQ’s FOR MCQ’s FOR MEDICAL MEDICAL PROFESSIONALS PROFESSIONALS BY PROF. Hormonal disorder or systemic disorder Harrison’s 18th Ed. Autosomal recessive D. Glucocorticoid excess inhibits growth. peak at 16 years & then decline. high-pitched In acromegaly. X-linked Skeletal maturation & somatic growth is promoted by hormonal stimuli like GH. Autosomal dominant C. > 3 µg/L A. GH levels normally increase to how much in children ? 123 The growth-promoting process consumes what proportion of Harrison’s 18th Ed. amino acids. voice & a tendancy for hypoglycemia. treatment sex steroid levels (estrogen) induce GHRH-GH-IGF-I axis & directly stimulate epiphyseal growth. The diagnosis is based on normal or high GH is ~6 cm/year in later childhood. Any of the above thyroid hormones. ~ 10 % B. Mean growth velocity partial or complete GH insensitivity & growth failure. mean growth velocity is about ? Harrison’s 18th Ed. GH treatment is moderately effective for High doses of estrogen lead to epiphyseal closure. 2891 A. Peak growth rates occur during midpuberty when bone age is 12 levels. Growth retardation due to mutations of GH receptor D. Cytokines C. Isolated GH deficiency is characterized by short stature. IGF-I levels decrease due to GH resistance. > 9 µg/L D. Chronic renal failure D. > 5 µg/L B. > 7 µg/L C. GH insensitivity A. High-pitched Harrison’s 18th Ed. sex steroids. Low-pitched A. GH is the major determinant of hepatic IGF-I synthesis. Elevated pubertal In patients with GH insensitivity & growth retardation due to mutations of GH receptor. Systemic disorder D. IGF-I concentrations C. Hoarse are higher in females than in males. 2892 Harrison’s 18th Ed. All of the above C. with decreased circulating GHBP. GH deficiency is best assessed by examining the response to exercise. TSH . 2891 hormone deficiency ? A. Growth plate disorder B. 2891 be ? A. 8 cm/year Laron syndrome is caused by mutation defects of GH receptor structure or signaling leading to Linear bone growth rates are very high in infancy & are pituitary dependent. IGF-I levels are invariably high. 121 Hormones promoting growth include all except ? 127 Familial modes of inheritance of isolated GH deficiency may Harrison’s 18th Ed. IGF-I A. Normal or high GH levels B. Malnutrition impairs chondrocyte activity & reduces circulating 130 Final height of children can be predicted by ? IGF-I & IGFBP3 levels. 129 Following provocative stimuli. All of the above C. Harrison’s 18th Ed. 2891 following ? A. 2891 B. 2891 A. 2892 B. insulin-induced hypoglycemia wherein GH levels normally increase to >7 µg/L in children. paracrine growth factors & cytokines. All of the above D. 2894 137 Normal daytime circulating levels of Growth Hormone is ? N Engl J Med 2006. 2893 Harrison’s 18th Ed. N Engl J Med 2006. FSH/LH 139 Familial syndromes associated with acromegaly include ? Sequential order of hormone loss in acquired pituitary hormone deficiency is GH > FSH/LH > TSH > ACTH.355:2558-73 D. GH C. Pancreatic islet cell tumor B.1 U/kg) hypoglycemia. AJAY MATHUR Endocrinology B. 135 Acidophil stem cell pituitary adenoma can lead to ? 142 The most common cause of GHRH-mediated acromegaly is ? Harrison’s 18th Ed. <5 µg/L 140 Which of the following organs is not enlarged in acromegaly ? C. After glucose reduction to ~40 mg/dL.3 µg/liter C. All of the above B. All of the above D. 136 Mammosomatotrope pituitary adenoma can lead to ? 143 Prognathism is due to enlargement of ? Harrison’s 18th Ed. < 0. 2893 A. intracranial thyroid gland enlargement. Abdominal carcinoid tumor D.4 µg/liter D. 2893 A. Acromegaly C.5 µg/liter Acromegaly is associated with an increased risk of colon polyps and colonic malignancy. Acromegaly C. ACTH D. Prostate Generalized visceromegaly occurs in acromegaly. Presence of active neoplasm A.534 MCQ’s FOR MEDICAL PROFESSIONALS BY PROF. macroglossia. hypertension or uncontrolled diabetes & retinopathy. 2894 Harrison’s 18th Ed. Heart 134 Contraindications to GH therapy include ? 141 Which of the following organs is not enlarged in acromegaly ? N Engl J Med 2006. Maxilla B. Galactorrhea B. < 0. Medullary thyroid carcinoma Most common cause of GHRH-mediated acromegaly is a chest or abdominal carcinoid tumor.2 µg/liter B. Liver disease A. Polyps are diagnosed in up to one-third of acromegalic patients. AGHD is defined by a peak GH response to hypoglycemia of <3 µg/L. usually choristomas or neuromas.0. following malignancies ? Harrison’s 18th Ed. and Contraindications to GH replacement therapy include presence of an active neoplasm. 2893 A. Liver B. < 0. Hypothyroidism B.355:2558-73 145 Which of the following regarding acromegaly is false ? Harrison’s 18th Ed. Hypogonadism A. <3 µg/L D. Parotid gland elevated for up to 2 hours. Thyroid gland induced (0. Colonic B.355:2558-73 A. MEN type 1 133 Adult GH deficiency (AGHD) is defined by a peak GH response to hypoglycemia of ? B. and peak GH release occurs at 60 minutes & remains C. Galactorrhea B. All of the above D. Nasal bones D.355:2558-73 A. Pancreatic A. <7 µg/L N Engl J Med 2006. 138 The production of IGF-I is suppressed in ? N Engl J Med 2006. Thyroid D. most individuals experience neuroglycopenic symptoms. Uncontrolled diabetes C. Mandible C.05 . Carney’s syndrome A. McCune-Albright syndrome Harrison’s 18th Ed. PRL & can present with a clinical syndrome of acromegaly. All of the above D. survival is reduced by ~10 years . Hypogonadism A. < 0. Pancreas C. All of the above Mammosomatotrope produce GH. including cardiomegaly. Tongue Most validated test to distinguish pituitary-sufficient patients from those with AGHD is insulin B. 144 Acromegaly is associated with an increased risk of which of the hypogonadism & galactorrhea of varying degrees. D. In patients with acidophilic stem-cell adenomas. Kidney D. & galactorrhea) predominate over the less clinically evident signs of acromegaly. Breast C. <9 µg/L A. Poorly controlled diabetes mellitus C. features of hyperprolactinemia (hypogonadism Excessive GHRH may be secreted by hypothalamic tumors. 2894 A. Often clinically not diagnosed for 10 years or more B. 2893 C.355:2558-73 Harrison’s 18th Ed. ~90% of healthy adults exhibit GH responses >5 µg/L. Without treatment. Intracranial hypertension B. Pheochromocytoma C. 4 AM Octreotide suppresses postprandial gallbladder contractility & delays GB emptying. Dopamine agonists B. AJAY MATHUR Cardiology 535 C. 2896 B. C. Hypertension occur in about 75% of patients 151 Which of the following act by antagonizing endogenous GH D. 535 Endocrinology MCQ’s FOR MCQ’s FOR MEDICAL MEDICAL PROFESSIONALS PROFESSIONALS BY PROF. All of the above A. 2896 In acromegaly. 6 AM patients treated with long-term Octreotide develop echogenic sludge or asymptomatic cholesterol C. -endorphin within 1 to 2 hours of 75 gram oral glucose load. Cholesterol gallstones 156 Peak ACTH secretion occurs at ? C. > 0. GH somatostatin octapeptide analogue that suppresses GH & IGF-I hypersecretion. .4 µg/liter A. B. Coronary heart disease. Octreotide suppresses gastrointestinal motility & secretion. B. met-enkephalin. 2895 suppresses GH secretion particularly those with cosecretion of PRL. > 0. decreased diastolic function & hypertension occur in about 30% of patients. ~ 10 % diagnosis or exclusion of acromegaly and does not correlate with disease severity. ACTH secretion is pulsatile with circadian rhythm. 149 Which of the following is false about Octreotide ? Harrison’s 18th Ed. Hypoglycemia Harrison’s 18th Ed. Diabetes mellitus develops in 25% of patients binding to its receptor ? Harrison’s 18th Ed. Irradiation C. GHRH C. cardiovascular system is most severely affected. survival is C. Consequently. 40 times more potent than native somatostatin A.1 µg/liter 153 Besides ACTH. All of the above preferred primary treatment for most patients ? 154 Besides ACTH. A. All of the above Harrison’s 18th Ed. malignancy & respiratory disease. IGF-I level provides a useful laboratory screening measure when clinical features raise the possibility of acromegaly. IGF-I 152 What proportion of pituitary cell population are ACTH-secreting corticotrope cells ? D. A. Administered subcutaneously B. left ventricular hypertrophy. IL-6 150 Which of the following is a side effect of Octreotide ? D. -endorphin D. Bromocriptine Harrison’s 18th Ed. Met-enkephalin 148 In GH-secreting microadenomas. 2895 ACTH-secreting corticotrope cells constitute ~20% of pituitary cell population. ACTH (39 amino acids) is derived from POMC precursor protein (266 amino acids) that also produces  -lipotropin. peaking at 6 AM & reaching a nadir around midnight. Sandostatin-LAR is a long-acting formulation of octreotide causing GH suppression. POMC precursor protein also produces which of B. A. 2896 D. Tachycardia B.4 µg/L B.  -endorphin.  melanocyte-stimulating hormone (MSH) & corticotropin-like intermediate lobe protein (CLIP). Lanreotide is a cyclic A.  melanocyte-stimulating hormone (MSH) C. serum IGF-I levels are suppressed. Surgical resection Harrison’s 18th Ed.2 µg/liter the following peptides ? C. Lanreotide acromegaly. asymptomatic bradycardia. None of the above C. -lipotropin Diagnosis of acromegaly is confirmed by demonstrating failure of GH suppression to <0. Arginine vasopressin (AVP) D. ~ 40 % Harrison’s 18th Ed. D. 2896 D. D.3 µg/liter Harrison’s 18th Ed. Cabergoline suppresses GH. 8 AM gallstones. Diabetes mellitus develops in 25% of patients with B. including IL-6 & leukemia inhibitory factor. Somatostatin analogues A. All of the above hypersecretion after macroadenoma resection usually necessitates adjuvant or primary medical therapy for these larger tumors. > 0. GH levels remain more than ? D. arginine vasopressin (AVP) & proinflammatory cytokines. which of the following is the D. Overall mortality is increased ~threefold and is due primarily to cardiovascular & cerebrovascular disorders. 2895 155 POMC gene is induced by ? A. The high frequency of GH D. 10 AM hypothyroxinemia & local pain at the injection site. Synthetic somatostatin analogue Harrison’s 18th Ed. It may also produce mild glucose intolerance due to transient insulin suppression. following oral glucose load. 2896 B. Without proper treatment. Corticotropin-like intermediate lobe protein (CLIP) Surgery is the preferred primary treatment for GH secreting microadenomas. 2896 POMC gene is powerfully suppressed by glucocorticoids & induced by CRH. Octreotide acetate reduced by an average of 10 years compared with an age-matched control population. Patients unable to receive or respond to medical treatment can be offered radiation. ~30% of B. 2896 Due to the pulsatility of GH secretion. IGF-II Harrison’s 18th Ed. Cabergoline cardiomyopathy with arrhythmias. POMC precursor protein also produces which of Harrison’s 18th Ed. potentially reducing the deleterious effects of excess endogenous GH. measurement of a single random GH level is not useful for the A. ~ 30 % 147 In acromegaly. > 0. ~ 20 % C. 2895 the following peptides ? A. measurement of which of the Pegvisomant is a GH analogue that antagonizes endogenous GH action by blocking peripheral following is most useful ? GH binding to its receptor. CRH C. Pegvisomant 146 For the diagnosis of acromegaly. 3 mmol/L are evident in ~70% of patients with ectopic ACTH secretion but are seen in <10% of patients with Harrison’s 18th Ed. Harrison’s 18th Ed. 160 Most common cause of cushingoid features is ? glucose intolerance & edema are also more pronounced. Melanocortin-3 receptor D. AVP A. < 8 mm D. Addison’s disease . Renal failure D. All of the above D. 2896 A. < 10 mm ACTH circadian rhymicity is determined by variations in secretory pulse amplitude rather than Most ACTH-secreting pituitary tumors are <5 mm in diameter & about half are undetectable by changes in pulse frequency. 2897 pituitary-dependent Cushing’s disease. 3 162 Hematopoietic features of hypercortisolism include ? B. Eosinopenia compared to those with pituitary ACTH-secreting adenomas. 15 to 20% ACTH-secreting adenomas ? Harrison’s 18th Ed. 164 Primary cause of death in patients of Cushing’s syndrome is ? 158 The receptor for ACTH is ? Harrison’s 18th Ed. Melanocortin-2 receptor C. 2898 Harrison’s 18th Ed. Infections B. < 2 mm B. Ectopic tumor ACTH production D. steroidogenesis by stimulating a cascade of steroidogenic enzymes in adrenal cells. exercise. physical stress. Basal plasma ACTH levels 161 What percentage of all pituitary tumors are ACTH-producing adenomas ? Small pituitary ACTH-secreting adenomas (<2 mm) may go undetected by gadolinium enhancement pituitary MRI. Iatrogenic hypercortisolism C. 2897-98 A. Cardiovascular disease A. Iatrogenic hypercortisolism 166 Most reliable test for confirmation of pituitary ACTH-secreting adenomas is ? C. Lymphopenia Mean basal ACTH levels are about eightfold higher in patients with ectopic ACTH secretion C. All of the above 168 Endocrinopathies that frequently have associated Hematopoietic features of hypercortisolism include leukocytosis. C. 12 B. hypokalemic alkalosis & edema suggests the possibility therapy should not exceed ? of ? Harrison’s 18th Ed. that induces of suicide are also increased. 10 mg daily A. An increased ratio (>2) of inferior petrosal A. A. 20 mg daily B. Ectopic tumor ACTH production Harrison’s 18th Ed. 40 mg daily D. Serum potassium levels <3. Melanocortin-4 receptor Primary cause of death in Cushing’s syndrome is cardiovascular disease. but infections and risk Receptor for ACTH is melanocortin-2 receptor. 2899 A. total daily dose of hydrocortisone replacement hypertension. 2898 ACTH-producing adenomas account for ~10 to 15% of all pituitary tumors. Cortisol-producing adrenal adenoma In ACTH deficiency. hypokalemic alkalosis. acute sensitive MRI. Suicide C. 25 mg daily C. severe myopathy. Melanocortin-1 receptor B. However. 2897 secreting adenomas from ectopic ACTH-secreting tumors. Cortisol-producing adrenal adenoma A. 2898 D. Overnight 1-mg dexamethasone suppression test D.536 MCQ’s FOR MEDICAL PROFESSIONALS BY PROF. Rapid development of features of hypercortisolism associated with skin hyperpigmentation & severe myopathy suggests ectopic production of ACTH. AJAY MATHUR Endocrinology 157 ACTH levels are increased by ? 163 Diameter of most ACTH-secreting pituitary tumors is ? Harrison’s 18th Ed. lymphopenia. 159 In ACTH deficiency. 2896 Harrison’s 18th Ed. B. 2897 C. A. Physical stress B. ACTH levels are increased by AVP. Bilateral inferior petrosal sinus ACTH sampling syndrome. iatrogenic hypercortisolism is the most common cause of cushingoid features. D. 2896 Harrison’s 18th Ed. 24-h urine free cortisol (UFC) Pituitary corticotrope adenomas account for 70% of patients with endogenous causes of Cushing’s B. 8 A. Pituitary corticotrope adenoma B. Hypertension. 5 Harrison’s 18th Ed. 2 to 5% : peripheral vein ACTH confirms pituitary Cushing’s disease. 10 to 15% patients with ectopic ACTH secretion as compared to pituitary D. and hyperpigmentation include ? eosinopenia. Insulin-induced hypoglycemia C. Pituitary corticotrope adenoma B. Leukocytosis D. illness & insulin-induced hypoglycemia. 5 to 10% 167 Mean basal ACTH levels are about how many times higher in C. 165 Rapid development of skin hyperpigmentation. a member of GPCR group. Bilateral inferior petrosal sinus ACTH sampling distinguishes pituitary ACTH- Harrison’s 18th Ed. 2898 A. < 5 mm C. total daily dose of hydrocortisone replacement should not exceed 25 mg daily. 30 to 60 minutes D. Mitotane A. is useful ? Harrison’s 18th Ed. In women. specificity is conferred by the  subunits. GnRH D. Increased skin pigmentation 175 Specificity of gonadotropins. cyproheptadine & etomidate block steroidogenesis and are useful in treatment of Cushing’s disease. Metyrapone inhibits 11  -hydroxylase activity & Mitotane D. which of the following drugs LH & FSH are glycoprotein hormones (like TSH & hCG) & consist of  &  subunits. Ketoconazole 176 Which of the following is the smallest amino-acid peptide ? B. Inhibin B.  D. 10 to 30 minutes C. Purple skin striae B. Hypertension A. In men. All of the above A. Adrenalectomy in the setting of residual corticotrope B. 2899 C. Osteoporosis. 10 % Nelson syndrome & ectopic ACTH syndrome. which in turn elicit LH & FSH pulses. A. ACTH has 39 amino acids derived from POMC precursor protein which has 266 amino acids. Nelson syndrome 174 What percentage of anterior pituitary cells are gonadotrope C. Aminoglutethimide A. is most common ? Harrison’s 18th Ed. Impotence. 2899 Gonadotrope cells comprise ~10% of anterior pituitary cells and produce two gonadotropins LH & FSH. 2899 A. Metyrapone Harrison’s 18th Ed. 2899 D. B. Ketoconazole is an antimycotic drug C. Moon facies. In women. 15 % 169 Which of the following is false about Nelson’s syndrome ? D. whereas activin stimulates FSH synthesis. LH and FSH. Acne 179 Which of the following statements is false ? B. Truncal obesity. GnRH is secreted in discrete pulses every 60 to 120 minutes.  170 In treatment of Cushing’s disease. which are Harrison’s 18th Ed. CRH is a 41-amino-acid hypothalamic peptide 171 In treatment of Cushing’s disease. Pituitary enlargement B. 5 % Endocrinopathies that frequently have associated hyperpigmentation include Addison’s disease. A. Thin skin C. 60 to 120 minutes Aminoglutethimide. D. Radiation therapy may be indicated to prevent the development of Nelson’s syndrome after adrenalectomy. Thin skin. Cyproheptadine B. FSH stimulates ovarian estrogen production D. 2897 Table 339-12 factor  (TGF. 2899 A. None of the above A. Plethora. Activin C. C. 2897 Table 339-12 178 FSH hormone is under control of which of the following ? Harrison’s 18th Ed. Etomidate C. 2899 expressed by separate genes. Mental changes. Hypothalamic GnRH D. Abnormal glucose tolerance. Diabetes mellitus. POMC precursor protein suppresses cortisol hypersecretion by inhibiting 11  -hydroxylase and cholesterol side-chain cleavage enzymes & by destroying adrenocortical cells. All of the above 173 Which out of the following clinical features of Cushing’s Syndrome Inhibin selectively suppresses FSH synthesis. GnRH is most common ? induces FSH secretion. Hyperpigmentation. Ectopic ACTH syndrome cells ? Harrison’s 18th Ed. LH helps maintain corpus luteum Clinical features of Cushing’s Syndrome in order of frequency are : Obesity. Hypothalamic GnRH is a 10-amino-acid peptide.  Nelson’s syndrome is a disorder characterized by rapid pituitary tumor enlargement and increased pigmentation secondary to high ACTH levels. All of the above B. 2899 C. Leydig cells Edema of lower extremities. None of the above . High ACTH levels subunit ? Harrison’s 18th Ed. Hirsutism. Truncal obesity D. Purple skin striae. whereas continuous GnRH exposure induces desensitization. C. Pulsatile mode of GnRH release 172 Which out of the following clinical features of Cushing’s syndrome prime gonadotrope responsiveness. Hypokalemic alkalosis. 2899 D. A. All of the above D. Corticotropin releasing hormone (CRH) that inhibits several P450 enzymes. Acne. Hyperpigmentation A. Hirsutism Harrison’s 18th Ed. which of the following drugs synthesized in paraventricular nucleus. Inhibin & activin are gonadal peptides & are members of transforming growth Harrison’s 18th Ed. 2899 177 GnRH is secreted in discrete pulses every ? Harrison’s 18th Ed. is conferred by which C. The  subunit is useful ? is common to these glycoprotein hormones. 20 % Harrison’s 18th Ed. Bruising. Bruising B. ACTH In Cushing’s disease. AJAY MATHUR Cardiology 537 B. LH induces testosterone synthesis & secretion by Hypertension. C. steroidogenic inhibitors are used in combination with pituitary irradiation to block the adrenal effects of persistently high ACTH levels. ) family. 537 Endocrinology MCQ’s FOR MCQ’s FOR MEDICAL MEDICAL PROFESSIONALS PROFESSIONALS BY PROF. Menstrual disorders.  adenoma tissue predisposes to its development. 5 to 10 minutes B. trilostane. Proximal muscle weakness. Hypothalamic GnRH regulates synthesis & secretion of both LH & FSH. B. 2900 D. Hypogonadism C. Somatotrophe cells TSH-secreting thyrotrope cells comprise 5% of the anterior pituitary cell population. None of the above A. Nal-Glu GnRH hypogonadal adult males ? Harrison’s 18th Ed. 2900 stimulate TSH synthesis & secretion. B. Gonadotrope cells cells. starvation & extreme exercise. Nal-Glu GnRH. Hypothyroidism (FSH > LH) in the setting of a pituitary mass. Pyroglutamyl histidylprolinamide In majority of patients with gonadotrope adenomas. Proglutamyl histidylprolinamide secretion. Testosterone levels are usually low. octreotide and selective GnRH antagonist. Some adenomas express  subunits without FSH or LH. 8 % Harrison’s 18th Ed. but may also be idiopathic. CRH A. 2901 produce small amounts of intact gonadotropins (usually FSH) as well as uncombined  and LH  A. 2900 nervosa. GH-secreting somatotrope cells constitute up B. Lactotrophe cells 189 TSH is structurally related to ? Most clinically nonfunctioning adenomas originate from gonadotrope cells. TRH also stimulates secretion of ? B. If PRL levels are <100 g/L in a patient with pituitary mass. ACTH Harrison’s 18th Ed. Starvation 187 In the treatment of nonfunctioning pituitary tumors. All of the above following reduces adenoma size ? Acquired forms of GnRH deficiency leading to hypogonadotropism are seen in association with anorexia Harrison’s 18th Ed. FSH 184 In patients with gonadotrope adenomas. decreased muscle mass with weakness. 2900 Hypogonadism is the most common presenting feature of adult hypopituitarism. This response is not seen in normal individuals. TRH contains a specific TSH  subunit. Increased gonadotropins (LH > FSH) 191 Besides TSH.538 MCQ’s FOR MEDICAL PROFESSIONALS BY PROF. 2901 C. secondary testicular failure is associated with decreased libido & potency. B. Skin tags anterior pituitary cell population ? Harrison’s 18th Ed. B. administration of which of the following stimulates LH  subunit secretion ? D. D. C. TRH administration stimulates LH  subunit B. even when other pituitary hormones are also deficient. A. A. 2901 In hypogonadal adult males. B. despite the normal or increased LH level. PRL C. Normal GH value due to ? Harrison’s 18th Ed. 2 % soft testes. rarely. GH and FSH  subunits. Addison’s disease 186 Clinically inapparent somatotrope adenoma is best excluded D. B. Fine facial wrinkles C. Stress a nonfunctioning adenoma causing pituitary stalk compression should be considered. Gonadotrope cells comprise about 10% of anterior pituitary cells. stress. C. Hypogonadotropic hypogonadism in these disorders is reversed by removal of the stressful stimulus. 2900 D. AJAY MATHUR Endocrinology 180 Most common presenting feature of adult hypopituitarism is ? D. PRL . These tumors typically Harrison’s 18th Ed. Petechiae Nonfunctioning pituitary tumors respond poorly to dopamine agonists. Normal IGF-II value 181 Acquired GnRH deficiency causing hypogonadotropism can be C. Pentaglutamyl histidylprolinamide 185 Which of the following is false in men with gonadotropin-secreting D. A. Anorexia nervosa Clinically inapparent somatotrope or corticotrope adenomas can be excluded by a normal IGF-I value and normal 24-hour urinary free cortisol levels. Lactotropes comprise about 20% of anterior pituitary C. and characteristic fine facial wrinkles. Low testosterone levels A. 2899 In men. infertility. It shares a common  subunit with these hormones but A. Corticotrope cells to 50% of the total anterior pituitary cells. reduced beard and body hair growth. Tumor secretion may lead to elevated  & FSH  subunits and. gonadotropin-secreting tumors may be diagnosed because of slightly increased gonadotropins A. 12 % A. to increased LH  subunit levels. Dopamine agonists B. All of the above A. 2900 TSH is structurally related to LH & FSH. TSH 190 Hypothalamic tripeptide . Normal IGF-I value B. 2901 D. perhaps reflecting reduced LH bioactivity or the loss of normal LH pulsatility. ACTH Harrison’s 18th Ed. None of the above Harrison’s 18th Ed. A. Pituitary mass Harrison’s 18th Ed. Coarse thickened skin 188 TSH-secreting thyrotrope cells comprise what percentage of D. Octreotide 182 Which of the following is the characteristic skin change in C. Preglutamyl histidylprolinamide tumors ? TRH is a hypothalamic tripeptide (pyroglutamyl histidylprolinamide) that acts through a GPCR to Harrison’s 18th Ed. Hyperprolactinemia by ? Harrison’s 18th Ed. 5 % 183 Most clinically nonfunctioning adenomas originate from ? C. which of the D. 2899 D.TRH is also called ? C. Electrolyte diuresis peaks of secretion & decay. Aquaporin 2 Dopamine agonists are rarely effective for suppressing TSH secretion from TSH-secreting adenomas. 2902 B. GH Harrison’s 18th Ed. LH & FSH are secreted in pulses.000 & osmolarity 50 mosmol/L). Six-membered disulfide ring with tripeptide tail D. 2092. neurophysin & copeptin B. Vasopressin A. GH. Smooth muscle contraction of blood vessels & GI tract 196 Structure of arginine vasopressin (AVP) is ? Harrison’s 18th Ed. 2077. This is called “Water diuresis”. LH have relatively short half-lives (<20 min) with sharp C. ACTH. A. 2903 A. Posterior pituitary gland D. Dopamine agonists B. ACTH 200 Absence of AVP produces which of the following ? C. Posterior pituitary gland Harrison’s 16th Ed. Aquaporin 4 AVP action is mediated via binding to G protein coupled V2 receptors on the serosal surface of the cells that line distal tubule & medullary collecting ducts of kidney. B. D. even though it is secreted in discrete pulses. Potentiates ACTH release by corticotropin-releasing factor B. GnRH. GH A. PTH. All of the above B. 2902 TSH secretion is stimulated by TRH. ACTH. C. 2902 C. causing biochemical hypothyroidism that requires concomitant C. Ionic diuresis 195 Which of the following is not useful in suppressing TSH secretion Without AVP. ACTH Harrison’s 18th Ed. All of the above 194 Which of the following has the longest half life ? Harrison’s 18th Ed. Disorders of the Neurohypophysis 202 At high concentrations. Octreotide markedly suppresses TSH. Seven-membered disulfide ring with tripeptide tail All the above effects are mediated by V1a or V1b receptors that are coupled to phospholipase C. 2096 D. Dopamine AVP secretion is synthesized via a polypeptide precursor that includes AVP. 2902 D.all encoded by a single gene on chromosome 20. a long acting somatostatin analogue. All of the above Harrison’s 18th Ed. All of the above A. activation of adenyl cyclase & insertion into luminal surface of water channels composed of a protein called aquaporin 2. Neurophysin 192 TSH secretion is suppressed by ? C. Water diuresis TSH has a prolonged half-life with relatively constant serum levels. Aquaporin 1 D. 2903 D. D. dopamine. Glucocorticoids 198 ADH is synthesized in which of the following ? D. A. Harrison’s 17th Ed. 2901 D. prolactin. 2096 large volumes (0. All of the above B. GnRH 199 ADH is released by which of the following ? B. GH 197 Which of the following is not synthesized via a polypeptide C. Five-membered disulfide ring with tripeptide tail C. Lanreotide A. Eight-membered disulfide ring with tripeptide tail 203 AV3V region that controls osmolarity is located in ? AVP is a nona-peptide composed of a six-membered disulfide ring and a tripeptide tail. AVP action includes ? Harrison’s 18th Ed. Lanreotide. Hypothalamus D. B. Anterior pituitary gland 193 Which of the following is secreted in pulses ? C. 2903 C. TSH B. C. Anterior pituitary gland Like TSH. 2217 . ACTH precursor ? Harrison’s 18th Ed. 2901 Antidiuretic hormone (ADH) is synthesized in supraoptic nuclei of hypothalamus and released by the posterior pituitary gland. AVP TRH also stimulates the lactotrope cell to secrete PRL. Prolactin (PRL) A. Copeptin A. SRIF & glucocorticoids A. Chapter 340. D. Aquaporin 3 thyroid hormone replacement. GH. Glycogenolysis in liver A.. All of the above C. Octreotide 201 Which of the following is related to the action of AVP ? Harrison’s 18th Ed. cells that line distal tubule & medullary collecting ducts of kidney reabsorb little of in TSH-producing macroadenomas ? the large volume of dilute filtrate that enters from proximal nephron resulting in excretion of very Harrison’s 16th Ed. whereas thyroid hormones. 2087. 539 Endocrinology MCQ’s FOR MCQ’s FOR MEDICAL MEDICAL PROFESSIONALS PROFESSIONALS BY PROF. Hypothalamus suppress TSH by overriding TRH induction. Osmotic diuresis B. Neuroptin Harrison’s 18th Ed.2 mL/kg per min) of maximally dilute urine (specific gravity ~1. SRIF . effectively suppresses TSH. AJAY MATHUR Cardiology 539 B. 295 mosmol/L D. Thirst osmostat is ‘set’ ~5% higher than AVP osmostat. Decreased blood volume B. 2 % B. Cerebellum Osmoreceptors are specialized hypothalamic cells that control AVP secretion through ‘effective’ osmotic pressure of body fluids. During pregnancy. 130 meq/L B. Sodium A. 206 Osmoreceptors are extremely sensitive to small changes in the plasma concentration of which of the following ? 212 AVP clearance is due to degradation in ? Harrison’s 18th Ed. Nausea is an extremely potent stimuli & causes immediate. Anterior pituitary gland A. 2904 A. All of the above C. kidney filters about 180 L/day of plasma. 50 % C. 3 % C. or set point. acute hypoglycemia.540 MCQ’s FOR MEDICAL PROFESSIONALS BY PROF. 80 % D. AJAY MATHUR Endocrinology A. 207 The average threshold. Hypothalamus Harrison’s 18th Ed. Kidneys C. Pregnancy C. 2903 210 Which of the following stimulate AVP secretion ? A. menstrual cycle. Medulla C. decreased arterial pressure and decreased blood volume increase ADH secretion. 135 meq/L C. 2903 B. Acute reductions in blood pressure or volume D. metabolic clearance of AVP is increased three. Placenta D. Third ventricle C. AVP secretion is stimulated by nausea. the set point of osmoregulatory system can be limb of Henle’s loop. 204 Which of the following stimuli increase ADH secretion ? Harrison’s 18th Ed. 2904 A. Menstrual cycle D. 30 % B. 2903 B. In a 70-kg healthy adult. 140 meq/L D. Decreased arterial pressure A. relatively large acute reductions in blood pressure or volume. Pons D. Posterior pituitary gland B. 50. Glucocorticoid deficiency D. Liver B. 95 % Average threshold or set point for AVP release corresponds to a plasma sodium of about 135 meq/L. Set point of osmoregulatory system can be lowered by pregnancy. 5 % Average threshold or set point for AVP release corresponds to a plasma osmolarity of ~ 280 mosmol/ L. ~144 L (80%) is reabsorbed isosmotically in proximal tubule & 8 L (4 . 2903 isosmotically in proximal tubules ? Harrison’s 18th Ed. Kidney B. Emetic stimuli act via emetic center in medulla & can be completely blocked by antiemetics. Nausea C. Urea B. Midbrain D. 2903 osmostat ? Harrison’s 18th Ed. Of this.to 100-fold increases in plasma AVP. 285 mosmol/L B. Most AVP clearance is due to degradation in liver and are insensitive to solutes like urea or glucose.5%) is reabsorbed without solute in descending 209 In healthy adults. 2903 Harrison’s 18th Ed. All of the above In addition to increased blood osmolarity.to four fold due to placental production of an N-terminal peptidase. 205 Osmoreceptors are present in ? Harrison’s 18th Ed. 280 mosmol/L A. . Remainder is diluted to an osmolarity of ~60 mmol/kg by selective reabsorption lowered by ? of sodium & chloride in the ascending limb. smoking and hyperangiotensinemia. for AVP release corresponds to a plasma osmolarity of about ? 213 Thirst osmostat is ‘set’ about how much higher than the AVP Harrison’s 18th Ed. Hypothalamus Harrison’s 18th Ed. Thirst is regulated primarily by an osmostat that is located in anteromedial hypothalamus. kidneys. All of the above C. 4 % D. 208 The average threshold or set point for AVP release corresponds to a plasma sodium of about ? 214 Of the fluid filtered by the kidneys. Increased blood osmolarity Harrison’s 18th Ed. Glucose C. All of the above Osmoreceptors are extremely sensitive to small changes in plasma concentration of sodium but AVP has a half life of 10 to 30 minutes. glucocorticoid deficiency. All of the above D. Posterior pituitary A. 290 mosmol/L C. All of the above AV3V region is located along the anteroventral region of third ventricle and plays a role in controlling osmolarity and ADH secretion. 2903 211 Emetic center is located in ? A. Acute hypoglycemia D. 2903 Figure 340-2 A. what percentage is reabsorbed Harrison’s 18th Ed. 125 meq/L A. 2903 B. G protein–coupled seven-transmembrane (GPCR) reduced to what level of normal ? D. Aquaporin 4 Harrison’s 18th Ed. Enuresis is common B. and neural A. Intercalated cells increase fluid intake on recommendations of health professionals or media. Aquaporin 1 deafness (DIDMOAD)]. enuresis. Central DI B. solute-free water is reabsorbed osmotically through principal cells of collecting A. Nausea D. 80 % 218 Which of the following clinical features of diabetes insipidus is D. C. Uterus receptors ? D. Aquaporin 2 222 In gestational DI. AVP-NPIV neurohypophyseal DI. 2903 Figure 340-2 Wolfram’s syndrome includes diabetes insipidus. solute-free water is reabsorbed osmotically D. Principal cells caused by a reduction in the ‘set’ of osmoregulatory mechanism. Dipsogenic DI is characterized by an inappropriate increase in thirst A. C. Cytokine receptor–linked kinase 223 Urine concentration ceases when secretion or action of AVP is C. thirst & polydipsia. 541 Endocrinology MCQ’s FOR MCQ’s FOR MEDICAL MEDICAL PROFESSIONALS PROFESSIONALS BY PROF. thereby inducing translocation of aquaporin 2 (AQP 2) water channels into the apical membrane resulting in an increase in permeability that permits influx of water that diffuses out of B. 2904 225 In primary or metastatic malignancies. Pituitary DI A. Diabetes mellitus 216 Which of the following aquaporin water channels is located on C. 18 mg/dL . Iatrogenic polydipsia A. AVP-NPIII Primary DI results from agenesis or irreversible destruction of neuro-hypophysis and is also called D. Dipsogenic DI sodium of about 1 meq/L ? B. Neurohypophyseal DI Harrison’s 18th Ed. 100 % false ? When secretion or action of AVP is reduced to ~80 to 85% of normal. Harrison’s 18th Ed. ectopic AVP production results from abnormal expression of which gene ? A. Dehydration is common A. Smoking In diabetes insipidus. 219 Primary diabetes insipidus is also called ? Harrison’s 18th Ed. 2098 A primary deficiency of plasma AVP can also result from increased metabolism by an N-terminal A. Receptor tyrosine kinase aminopeptidase produced by the placenta. 2904 & the rate of output increases to symptomatic levels. B. Psychogenic polydipsia is not associated with thirst & polydipsia is a feature of psychosis. 2904 Aquaporin 2 (AQP 2) water channels are located on the apical membrane. A. Aquaporin 3 AVP comes from ? D. It is an autosomal recessive disorder due to mutations of WFS 1 gene. B. 30 % cyclic AMP. or central DI. Optic atrophy apical membrane of principal cells of the collecting duct ? D. smoking & vasovagal reaction increases plasma AVP to produce profound antidiuresis. It is referred to as gestational DI. AJAY MATHUR Cardiology 541 215 Under AVP influence. Ectopic production of AVP result from abnormal expression of AVP-NPII gene by primary or metastatic 220 Which of the following is a primary polydipsia ? malignancies. AVP-NPII D. Placenta channels are located on the basal-lateral surface and water diffuses out of the cell through them. 2904 Under AVP influence. 2904 226 What value of rise in blood glucose leads to decrease in serum A. All of the above Harrison’s 18th Ed. In iatrogenic polydipsia there is B. 2903 Figure 340-2 called primary polydipsia. All of the above impaired. diabetes mellitus. B. 2904 C. Nausea. Psychogenic polydipsia Harrison’s 18th Ed. 2910 C. AVP-NPI C. Foetus 217 Antidiuretic effect of AVP is mediated via which of the following C. Daily urine volume is >50 mL/kg body weight 224 Which of the following increases plasma AVP levels ? B. All of the above Harrison’s 18th Ed. and/ or nocturia. Serine Kinase Harrison’s 18th Ed. 50 % the cell through AQP 3 and AQP 4 water channels on the basal-lateral surface. B. Any of the above C. AQP 3 and AQP 4 water A. All of the above through which of the following cells ? Secondary deficiency of AVP due to inhibition of AVP secretion by excessive intake of fluids is Harrison’s 18th Ed. 2908 B. optic atrophy. urine concentration ceases Harrison’s 18th Ed. Vasovagal reaction weight/day & urine osmolarity is <300 mosmol/L. Polyuria produces urinary frequency. offending substance that produces deficiency of C. Diabetes insipidus ducts. pituitary DI. All of the above Harrison’s 16th Ed. Mesangial cells 221 Wolfram’s syndrome includes ? D. 2904 Antidiuretic effect of AVP is mediated via a G protein–coupled V2 receptor that increases intracellular A. Urine osmolarity is <300 mosmol/L Harrison’s 18th Ed. due to decreased secretion or action of AVP urine volume is >50 mL/kg body C. Clinical signs of dehydration are uncommon unless fluid intake is D. normal persons and in patients with empty sella who do not have DI. Sepsis B. AJAY MATHUR Endocrinology B. 72 mg/dL DDAVP is a synthetic analogue of AVP that can be given IV or SC. 233 Nephrogenic DI is treated by ? Harrison’s 18th Ed. defined as an excess of water in relation to sodium in extra-cellular fluid.356:2064-72 A. Empty sella who do not have diabetes insipidus C. Oral tablet C. Bronchogenic carcinoma D. Ataxia C. hyperuricemia & hypokalemia. at least at presentation. After the syndrome was described. ADH in humans was found to be arginine vasopressin. All of the above B. It is absent not only in pituitary and nephrogenic DI but also in some release of antidiuretic hormone was lacking and the level of secretion of ADH was deemed “inappropriate”.356:2064-72 C. Due to destruction of neurohypophysis B. 2910 Harrison’s 18th Ed. Pituitary diabetes insipidus A. Amiloride B. 2906 227 Each decrease in serum sodium of 1 meq/L reduces plasma osmolarity by about ? A. Each decrease in serum sodium of 1 meq/L reduces plasma osmolarity by about 2 mosmol/L. Inhibitors of prostaglandin synthesis (indomethacin) are effective. Hyponatremia In MRI of pituitary & hypothalamus of healthy beings. 54 mg/dL D. Oxytocin . Muscle weakness. azotemia. 231 Pituitary bright spot is absent in patients with ? 237 SIADH was first described in patients with ? Harrison’s 18th Ed. 2906 N Engl J Med 2007. 10 mmol/L C. SC injection A. Thiazide diuretic Harrison’s 18th Ed. Paresthesias D. rhabdomyolysis. tachycardia. Hypernatremic C. Hypernatremia is associated with hypovolemia presenting as Central pontine myelinolysis is characterized by quadriparesis. hyperglycemia. DI is usually not present. 8 mmol/L B. 2906 that normally regulate thirst & AVP secretion. 2907 228 Central pontine myelinolysis is characterized by all except ? Harrison’s 18th Ed. Hypovolemia A. posterior pituitary emits a hyperintense C. Hypocalcemia signal in T1-weighted mid-sagittal images. is against nephrogenic DI and strongly SIADH was first described in patients with bronchogenic carcinoma in whom a physiologic stimulus for suggests primary polydipsia. Desmopressin B. Hypomagnesemia pituitary DI. 2906 N Engl J Med 2007. Develop hyponatremia if overhydrated 230 Pituitary bright spot is almost always present in patients with ? Adipsic hypernatremia is caused by agenesis or destruction of the hypothalamic osmoreceptors Harrison’s 18th Ed. 229 In treatment of hyponatremia in SIADH with hypertonic saline. Quadriparesis B. is the vasopressin. Nasal inhalation B. None of the above D. Indomethacin A. Nephrogenic diabetes insipidus B. 1 mosmol/L C. postural hypotension. 36 mg/dL C. ataxia & abnormal extraocular movements. maximum rise in serum sodium level should not be more than ? 235 Which of the following is false about adipsic hypernatremia ? Harrison’s 18th Ed. 4 mosmol/L or amiloride with low-sodium diet. Schizophrenia A normal pituitary bright spot on MRI excludes pituitary DI. pain. 2910 A. Hypokalemia D. 3 mosmol/L Nephrogenic DI does not respond to desmopressin but may be reduced with thiazide diuretic and/ D. 2910 B. Hyperuricemia C. 2 mosmol/L D. 12 mmol/L D. Abnormal extraocular movements Adipsic hypernatremia is a state of chronic or recurrent hypertonic dehydration due to deficiency in osmoregulation of thirst. Hypertonic dehydration D. All of the above D. All of the above C. 2907 A.542 MCQ’s FOR MEDICAL PROFESSIONALS BY PROF. it is small or absent in nephrogenic DI because of high secretion & turnover of Hyponatremia. A. most common electrolyte disorder in hospitalized patients.356:2064-72 A. Serum sodium decreases ~1 meq/L for each 36 mg/dL rise in glucose. 6 mmol/L A. 234 Which of the following is false about adipsic hypernatremia ? Harrison’s 18th Ed. 232 Desmopressin (DDAVP) can be administered by ? 238 Which of the following is an AVP analogue ? Harrison’s 18th Ed. Primary polydipsia A. nasal inhalation or orally. Pituitary diabetes insipidus 236 Most common electrolyte disorder in hospitalized patients is ? B. hyperlipidemia & acute renal failure may also occur. Hypokalemia B. Head trauma C. This “pituitary bright spot” is almost always present in patients with primary polydipsia but is invariably absent or abnormally small in patients with D. All of the above D. Nephrogenic diabetes insipidus N Engl J Med 2007. Nonsteroidal anti-inflammatory drugs. A.356:2064-72 factors like Thyroid transcription factor (TTF) 1. 11-kDa gestation ? B. All of the above 240 Drug that stimulate release of AVP or enhance its action is ? Thyroid gland development is orchestrated by coordinated expression of developmental transcription N Engl J Med 2007. Posterior pituitary A. Basolateral antidepressants. has serious sequelae. Superficial TSH is secreted by the thyrotrope cells of the anterior pituitary gland. 249 The size of thyroid-stimulating hormone (TSH) is ? Harrison’s 18th Ed. Clofibrate. Antipsychotic drugs. D. About 11 weeks’ D. 2912 D. All of the above C. TTF-2 & paired homeobox-8 (PAX-8). Cyclophosphamide. D. All of the above A. Apical Drugs that stimulate release of AVP or enhance its action include Chlorpropramide. TTF-1 (NKX2A) D. Essential Word thyroid is derived from a Greek word thyreos meaning “shield” plus eidos meaning “form”. Shield D.20 grams in weight. 2913 242 What is the normal weight of human throid gland ? A. seizures. Thyroid medullary C cells are neural crest derivatives & produce a calcium-lowering hormone . Anorexia. convulsions A. 2911 B. 2912 C. coma. decerebrate posture & respiratory arrest. D. Vasopressin C. leading to death. 2911 C. About 21 weeks’ to which of the following hormones ? Harrison’s 18th Ed. Lateral Basolateral surface of thyroid follicular cells is apposed to bloodstream and apical surface faces Chapter 341. Headache 245 Which of the following is thyroid gland developmental B. About 16 weeks’ 250 The  subunit of thyroid-stimulating hormone (TSH) is common D. Junction of upper two-third & lower one-thirds D. D. 6 to 12 grams C. Harrison’s 18th Ed. Stalk of pituitary B. 41-kDa C. vomiting transcription factor ? Harrison’s 18th Ed. C. TTF-2 (FKHL15) Severe hyponatremia (serum sodium <125 mmol per liter). Ecstasy. C. 2913 241 Word thyroid is derived from Greek word “thyreos” meaning ? A. Conical shaped C. hallucinations. Anterior pituitary Harrison’s 18th Ed. All of the above D. Tricyclic B. All of the above C. All of the above B. 21-kDa Harrison’s 18th Ed. Human chorionic gonadotropin (hCG) A. 31-kDa B. T3 Harrison’s 18th Ed. 2913 Thyroid hormone synthesis normally begins at about 11 weeks’ gestation. Confusion. Junction of upper one-third & lower two-thirds . Nonsteroidal antiinflammatory drugs Harrison’s 18th Ed. Follicle-stimulating hormone Harrison’s 18th Ed. Carbamazepine 246 Which of the following surface of thyroid follicular cells is in B. especially when the condition develops rapidly (within 48 hours). TSH B. Luteinizing hormone 244 Density of C cells in thyroid gland is greatest at ? B. nausea. Vincristine. SSRIs. 2913 243 Thyroid hormone synthesis normally begins at what time of A. 12 to 20 grams D. A. All of the above B.calcitonin. Junction of upper & lower halves D. Carbamazepine. 30 to 50 grams Normal thyroid is 12 . Narcotics. including confusion. 2911 A. 2908 A. Disorders of the Thyroid the follicular lumen. AJAY MATHUR Cardiology 543 C. 543 Endocrinology MCQ’s FOR MCQ’s FOR MEDICAL MEDICAL PROFESSIONALS PROFESSIONALS BY PROF. About 6 weeks’ C. 2911 B. T4 A. 20 to 30 grams TSH is the most useful physiologic marker of thyroid hormone action. 248 Most useful physiologic marker of thyroid hormone action is ? Harrison’s 18th Ed. Paired homeobox-8 (PAX-8) coma. Nicotine contact with bloodstream ? C. Basal Ifosfamide. At the poles 239 Water intoxication can lead to ? Density of C cells in thyroid gland is greatest at the junction of upper one-third & lower two-thirds. Gland 247 TSH is secreted by the thyrotrope cells of ? Harrison’s 18th Ed. Nicotine. D. 2913 262 Iodine-deficiency data of WHO is based on ? Harrison’s 18th Ed. T4 extracts iodine from circulation mediated by the Na +/I – symporter (NIS) which is expressed at the basolateral membrane of thyroid follicular cells. All of the above D. About 20 minutes NIS is highly expressed in thyroid gland. Based on urinary excretion data. TRH C. The “set-point” in thyroid axis is established by TSH. after what time does peak except ? TSH secretion occur ? Harrison’s 18th Ed. Proteolysis B. Characterized by goiter & sensorineural deafness TSH has a relatively long plasma half-life of 50 minutes. Highest levels occur at night B. AJAY MATHUR Endocrinology 251 The  subunit of thyroid-stimulating hormone is common to 257 T4 & T3 are formed from thyroglobulin by what mechanism ? which of the following hormones ? Harrison’s 18th Ed. 254 Which of the following about TSH secretion is false ? 260 Iodine transporter . 10 minutes Harrison’s 18th Ed. Coupling A. 2913 Harrison’s 18th Ed. glycoprotein hormones like luteinizing hormone. Relatively long plasma half-life B. pendrin. 80 minutes C. Human chorionic gonadotropin (hCG) D.pendrin is located on which surface of thyroid Harrison’s 18th Ed. 259 Na +/I . About 10 minutes C. 2913 B. 2913 B. About 15 minutes D.symporter (NIS) B. human chorionic Reuptake of Tg into thyroid follicular cell allows “proteolysis” & release of newly synthesized T 4 & T3. Lateral Iodine transporter. Iodination C. Due to deficiency of iodine transporter . whereas TSH beta subunit is unique to TSH. 30 minutes A. Bound to albumin Harrison’s 18th Ed. None of the above C. Released in pulsatile manner Harrison’s 18th Ed. Serum levels of thyroid hormones D. Apical C. Mutation of PENDRIN gene causes 256 Single measurement of TSH is adequate for assessing its Pendred syndrome characterized by defective organification of iodine. follicle-stimulating hormone. Urinary excretion of iodine B. Anthropometry C. 2913 cells ? A. None of the above C. 2913 A. Salivary glands B. Testicles D. Basal TSH is released in a pulsatile manner & exhibits diurnal rhythm with highest levels occuring at night. 2913 B.pendrin C. Alpha subunit is common to other thyroid follicle. Iodide uptake is mediated by Na +/I. No diurnal variation A. Symptoms & signs Single measurements of TSH are adequate for assessing its circulating level due to its relatively long plasma half-life. Tg is iodinated on tyrosine residues that are subsequently coupled via an ether linkage. Luteinizing hormone B. 2913 established by ? A. lactating Peak TSH secretion occurs ~15 minutes after administration of exogenous TRH. is located on the apical surface of thyroid cells 255 Plasma half-life of TSH is ? Harrison’s 18th Ed. Exhibits diurnal rhythm A. Thyroid gland D. but low levels are present in the salivary glands. gonadotropin (hCG). goiter & sensorineural deafness. Basolateral D. T3 Ingested iodine is bound to serum albumin. WHO estimates that ~ 2 billion people are iodine-deficient. Thyroid gland A. None of the above Thyroid hormones are derived from a large iodinated glycoprotein thyroglobulin (Tg). D. Non pulsatile release C. 258 Which of the following about ingested iodine is false ? 252 The “set-point” in thyroid axis (endocrine feedback loop) is Harrison’s 18th Ed. 2913 A. . Low protein binding D. circulating level because of ? Harrison’s 18th Ed. None of the above Iodine transporter . Unbound iodine excreted in urine A. TSH D.symporter (NIS) is expressed in all of the following 253 On administration of exogenous TRH. About 5 minutes B. 50 minutes B. breast & placenta. 2913 Harrison’s 18th Ed. After secretion into TSH is a 31-kDa hormone composed of alpha & beta subunits. Follicle-stimulating hormone C. 2913 A. Due to mutation of PENDRIN gene D.pendrin mediates iodine efflux into the lumen.544 MCQ’s FOR MEDICAL PROFESSIONALS BY PROF. Unbound iodine is excreted in urine. 2913 261 Which of the following is false about ‘Pendred syndrome’ ? A. Placenta C. Developmental abnormality of thyroid C. 545 Endocrinology MCQ’s FOR MCQ’s FOR MEDICAL MEDICAL PROFESSIONALS PROFESSIONALS BY PROF. Cubilin B. To delay hormone clearance C. Prolactin A. C. recommended average daily intake of iodine is ? Harrison’s 18th Ed. and 200 µg/day for pregnant women. 200 µg/day D. In patients with underlying autoimmune thyroid disease. transthyretin (TTR. Defects have B. 2913 D. NIS. All of the above Urinary iodine is >10 µg/dL in iodine-sufficient populations. pendrin. Megalin D. thereby avoiding the lysosomal pathway. 90 . children. may modulate hormone delivery to selected tissue sites. > 8 µg/dL C. Copper 270 Factors that influence thryoid hormone synthesis & release include all except ? Concomitant selenium deficiency contributes to the neurologic manifestations of cretinism. recommended average daily intake of 271 Wolff-Chaikoff effect is related to ? iodine is ? Harrison’s 18th Ed. Dimeric protein B. Adaptin thyroglobulin (Tg) is a large (660 kDa) dimeric protein that consists of 2769 amino acids. Consists of 2769 amino acids C. 250 µg/day I (IGF-I). Pendrin A. Tg is taken back into thyroid cell. Albumin D. delay hormone clearance. Iodine deficiency A. where proteolytic cleavage of Tg 269 Congenital hypothyroidism is mostly due to mutations in ? results in hormone release. 100 µg/day B. A. and T 3. hydrogen peroxide generation. 2914 Harrison’s 18th Ed. All of the above D. Mitochondria A. 2914 A. 2914 A. 150 µg/day D. 150 µg/day C. > 10 µg/dL D. 2914 B. 200 µg/day TSH is the dominant hormonal regulator of thyroid gland growth & function. T4 & T 3 are bound to plasma proteins which include thyroxine-binding globulin (TBG). where it is processed in “lysosomes” to release T4 & Plasma-binding proteins increase the pool of circulating hormone. urinary iodine is level is ? 272 T3 & T4 hormones circulate bound to which plasma protein ? Harrison’s 18th Ed. Insulin-like growth factor I (IGF-I) 264 For adults. Harrison’s 18th Ed. 660 kDa size A. 2914 Harrison’s 18th Ed. > 2 µg/dL A. All of the above After coupling. Clathrin C. TPO .120 µg/day for Chaikoff effect. Megalin is a transmembrane glycoprotein that mediates endocytosis of Tg by thyroid cells resulting in its transcytosis. and dehalogenase. Mangnese Majority of congenital hypothyroidism are due to recessive mutations in TPO or Tg. Thyroxine-binding globulin (TBG) B. 100 µg/day C. 267 Thyroglobulin releases T4 & T3 in which structure of thyroid cell ? 273 Function of serum-binding proteins is ? Harrison’s 18th Ed. Golgi apparatus B. 266 In iodine-sufficient populations. Transcytosis of Tg endocytosed from the colloid is thought to be one Harrison’s 18th Ed. 265 For pregnant women. > 5 µg/dL B. Selenium D. epidermal growth factor. 2914 Harrison’s 18th Ed. 2227 A. Excess iodide B. To increase pool of circulating hormone B. All of the above D. 2914 A. Transthyretin (TTR) C. suppressive action of high iodide may persist. 2914 endocytosis by thyroid cells ? Harrison’s 17th Ed. Normal thyroid gland escapes from this inhibitory effect & iodide organification resumes. formerly known as thyroxine-binding prealbumin. 2914 A. None of the above D. Dehalogenase Harrison’s 18th Ed. Lysosome C. 250 µg/day Excess iodide transiently inhibits thyroid iodide organification. NIS manifestations of cretinism ? C. or TBPA) and albumin. 268 Which of the following is false about thyroglobulin (Tg) ? 274 Which of the following receptors mediate thyroglobulin (Tg) Harrison’s 18th Ed. but growth factors produced locally in thyroid gland also influence thyroid hormone synthesis which include insulin-like growth factor D. where the levels have been shown to be increased under conditions with heightened TSH stimulation. a phenomenon known as the Wolff- The recommended average daily intake of iodine is 150 µg/day for adults. Epidermal growth factor B. Cobalt also been identified in TSH-R. Transforming growth factor  (TGF-) C. AJAY MATHUR Cardiology 545 263 Concomitant deficiency of which element contributes to neurologic B. transforming growth factor (TGF-) & endothelins. 2914 of the mechanisms that account for the presence of intact Tg in the circulation. To modulate hormone delivery D. 2915 A. Thyroid Homeostatic mechanisms that regulate the thyroid axis are directed toward maintenance of normal concentrations of unbound hormones. gp360 A. Type III deiodinase D. Familial nature with normal TSH levels suggest this diagnosis. Fasting D. pregnancy. has a relatively low affinity for T 4. It was purified from B. Acute trauma Mutations in TBG. Secretin receptor family A. and kidney. 2915 D. T4 is converted to T3 by deiodinase enzymes Megalin was originally identified as the antigen in Heymann nephritis of rats. Unbound T3 and T4 brown fat. CHF D. 2915 D. TSH levels are decreased B. patients are ? D. liver. brown fat. patients are euthyroid and TSH levels are normal. However. gp280 Acquired TBG excess is seen with estrogen. All of the above 279 Which of the following statements about familial dysalbuminemic hyperthyroxinemia (FDH) is false ? Type III deiodinase inactivates T 4 and T3 and is the most important source of reverse T3 (rT3). and the apolipoprotein thyroid E (apo E) receptor-2. Toll-like receptor family C. TSH 283 Type I deiodinase is located in all except ? Harrison’s 18th Ed. TRH A. sharing common features with LDL receptor. and medications like propylthiouracil. Liver C. brain & Megalin belongs to the LDL-receptor family. Due to mutations in albumin 285 T4 to T3 conversion may be impaired by ? B. Autosomal dominant transmission Harrison’s 18th Ed. 2915 280 Acquired TBG excess occurs due to all except ? A. A. Type III deiodinase inactivates T4 & T3 toward maintenance of normal concentrations of ? D. TTR. Unbound hormone levels (ideally measured by dialysis) are normal in FDH. because B. Kidney Harrison’s 18th Ed. Cirrhosis T 4 to T 3 conversion is impaired by fasting. Massive hemangiomas that express type III deiodinase are a rare cause of hypothyroidism in infants. A. and thyroid gland. Pregnancy C. gp330 281 Which of the following about deiodinases is false ? Harrison’s 18th Ed. 2915 D. B. liver & kidney rat kidney brush border & named gp330 on the basis of molecular weight as estimated by its mobility during gel electrophoresis. B. 2914 Type II deiodinase has a higher affinity for T 4 and is found primarily in the pituitary gland. Hypothyroid B. Type I deiodinase. All of the above C.546 MCQ’s FOR MEDICAL PROFESSIONALS BY PROF. Increased total T4 &/or T3. None of the above A. Type I deiodinase is located in thyroid. 2915 Type I deiodinase is located primarily in thyroid. 2915 Table 341-3 B. LDL- receptor-related protein (LRP). Hyperthyroid 284 Which of the following deiodinases is the most important source of reverse T3 (rT3) ? C. but free hormone levels are normal A. systemic illness or acute trauma. as it may lead to thyrotoxicosis. which is located primarily in thyroid. glucocorticoids. and albumin may increase the binding affinity for T 4 and/or T3 and cause disorders known as euthyroid hyperthyroxinemia or familial dysalbuminemic hyperthyroxinemia (FDH). Protein bound T3 and T4 C. Type II deiodinase is found in pituitary. very-low-density lipoprotein (VLDL) receptor. Prostate 278 In X-linked Thyroxine-binding globulin (TBG) deficiency. and kidney. C. EGF receptor family 282 Which of the following about deiodinases is false ? Harrison’s 18th Ed. All of the above Total T4 and/or T3 are increased but unbound hormone levels are normal. None of the above Harrison’s 18th Ed. 286 T4 to T3 conversion may be impaired by ? Harrison’s 18th Ed. Glucocorticoids B. Type II deiodinase has higher affinity for T4 277 Homeostatic mechanisms regulating thyroid axis are directed C. B. cirrhosis & hepatitis. Hepatitis A. Expression of type II deiodinase allows it to regulate T 3 concentrations locally. LDL-receptor family T 4 is converted to T 3 by deiodinase enzymes. liver. B. 2915 C. Amiodarone A. propranolol. gp310 C. Oral contrast agents Harrison’s 18th Ed. Efforts to normalize total T4 levels should be avoided. AJAY MATHUR Endocrinology 275 Megalin is also known as ? D. Any of the above A. D. Euthyroid Harrison’s 18th Ed. Harrison’s 18th Ed. Type I deiodinase X-linked TBG deficiency is associated with very low levels of total T 4 and T 3. brain. Type I deiodinase has a relatively low affinity for T4 276 Megalin belongs to which of the following ? D. Systemic illness C. amiodarone. C. Type III deiodinase inactivates T 4 and T 3 . B. Type II deiodinase unbound hormone levels are normal. . oral contrast agents. 2915 D. Nuclear hormone receptors D. 1 day 290 Thyroid hormone receptor  (TR   is more abundant than TR B. impaired speech. and heart. TR1 C . 75 % D. Brain B. 2914 Table 341-2 D. 20 % in which of the following organs ? C. high T3. and contractures.2 isoform contains a unique carboxy terminus that precludes thyroid hormone binding. 2915 Harrison’s 17th Ed. and high TSH). secretion ratio from thyroid gland of T3 & T4 is ? Harrison’s 18th Ed. is selectively expressed in the hypothalamus and pituitary. 2915 B. TR-alpha is particularly abundant in brain. 25 % Thyroid hormones bind with high affinity to nuclear thyroid hormone receptors (TR) alpha & beta B. thyroid hormone receptor (TR) & retinoid X receptor (RXR) form heterodimers that bind Allan-Herndon-Dudley syndrome is an X-linked mental retardation condition caused by mutations in the specifically to thyroid hormone response elements (TRE) in promoter regions of target genes. C. 1 : 10 B. Mutations in MCT8 cause neurologic deficits & thyroid function abnormalities (low T4. 2915 Harrison’s 18th Ed. Thyroid hormone reacting elements D. TR2 C. 2916 Harrison’s 18th Ed. Liver Harrison’s 18th Ed.2. TR2 D. Thyroid hormone reccruiting elements A. . AJAY MATHUR Cardiology 547 287 Which of the following is a thyroid hormone (TH) transporter ? 293 Which of the following prevents thyroid hormone binding ? Harrison’s 18th Ed. 50 % . 2916 A. muscle. Heart A. 7 days Harrison’s 18th Ed. TR2 D. Schimke syndrome In nucleus. 1 : 40 The TR-beta2 isoform. TR1 A. Gustavsson syndrome B. 2229 A. Cell membrane hormone receptors C. 1 day Harrison’s 18th Ed. 3 days in which of the following organs ? C. Brooks-Wisniewski-Brown syndrome C. 10 days C. 10 % 291 Thyroid hormone receptor  (TR   is more abundant than TR B. located at 295 Serum T4 has a plasma half-life of ? Xq13. 1 : 30 D. muscle weakness. MCT6 C. 40 % A. 2915 A. Mutations in this gene. TR1 B . kidney. which has a unique amino terminus. Pituitary A. MCT2 A. 2914 Table 341-2 D. Allan-Herndon-Dudley syndrome D. where it plays a role in feedback control of the thyroid axis. 30 % Harrison’s 18th Ed. gonads. whereas TR . 100 % 292 Which of the following plays a role in feedback control of the thyroid axis in hypothalamus and pituitary ? 299 Normally. Thyroid hormone response elements C. Gonads Harrison’s 18th Ed. 547 Endocrinology MCQ’s FOR MCQ’s FOR MEDICAL MEDICAL PROFESSIONALS PROFESSIONALS BY PROF. MCT8 D. Thyroid hormone receptor elements B. MCT4 B. Harrison’s 18th Ed. A. 2914 Table 341-2 Thyroxine (T4) and triiodothyronine (T3) act through nuclear receptors. (MCT8) transporter. 1 : 20 C. monocarboxylate transporter 8 (MCT8) gene. Muscle B. 2915 D. Kidney 298 What fraction of circulating T4 comes directly from thyroid gland ? C. hypotonia. 7 days B. 2914 Table 341-2 289 Thyroid hormones act through ? A.beta expression is relatively high in the pituitary and liver. TR1 B. 2915 D. Its features include moderate-severe mental retardation. Kidney 297 What fraction of circulating T3 comes directly from thyroid gland ? C. 294 Thyroid hormone receptor (TR) binds to which of the following 288 Which of the following X-Linked Syndromes is related to mutation in the promoter region of target genes ? in MCT8 gene ? Harrison’s 18th Ed. 10 days A. 3 days A.expressed in most tissues. All of the above Harrison’s 18th Ed. TR2 Circulating thyroid hormones enter cells by passive diffusion and via the monocarboxylate 8 TR . impair transport of T3 into neurons (elevated free T3 & decreased free T4 in blood). Mitochondrial hormone receptors 296 Serum T3 has a plasma half-life of about ? D. Thyrotoxicosis compensated by increased levels of thyroid hormone. Pregnancy hormone synthesis in 10 . Constipation Harrison’s 16th Ed. Presents as hypothyroidism 307 TSH levels are suppressed in ? Resistance to thyroid hormone (RTH) is an autosomal dominant disorder characterized by elevated Harrison’s 18th Ed. Tamoxifen Harrison’s 18th Ed. Suppressed TSH level (<0. Normal 312 Which of the following is the most common sign of B. U B. the tongue should be extended. 35 D. after B. B. 10 MHz A. TSH-R antibody-mediated 304 TBG is increased due to all except ? D. free T3 and TSH are ? C. C. 2228 D. mild reduction in IQ. V C. Act of swallowing Thyroid gland selectively transports radioisotopes of iodine ( 123I. 2917 thyroid hormone levels & inappropriately normal or elevated TSH.5 MHz r 303 Free T3 index is calculated by ? i Harrison’s 17th Ed. Treated with thyroid replacement drugs imaging ? d Harrison’s 18th Ed. B. oral A. With any central mass above the thyroid. Increased hypothyroidism ? C. 5 MHz 9 C. A. Hypothyroidism A. Bradycardia . First trimester of pregnancy C. Thyroid hormone binding ratio (THBR) x Total T3 A 10-MHz ultrasound probe can detect thyroid nodules and cysts >3 mm. Tachycardia treatment of hyperthyroidism & in response to high doses of glucocorticoids or dopamine. the index corrects B. attention deficit disorder. In effect. Insulin 301 Which of the following statements about resistance to thyroid hormone (RTH) is false ? D.5 MHz 9 especially when the arms are raised (Pemberton’s sign). Elevated free thyroid hormone levels D.548 MCQ’s FOR MEDICAL PROFESSIONALS BY PROF. Autosomal dominant disorder B. TBG is reduced by androgens & in nephrotic syndrome. Individuals with RTH do not exhibit signs & symptoms of hypothyroidism because hormone resistance is partial and is A.1 mU/L) indicates R thyrotoxicosis but are also seen during first trimester of pregnancy (due to hCG secretion). Inborn errors of thyroid hormone synthesis for anomalous total hormone values caused by abnormalities in hormone-protein binding. Impaired metabolic responses to thyroid hormone 308 Which of the following radioisotopes of iodine is used for thyroid D. 131I) and 99mTc pertechnetate. 2917 Neonatal hypothyroidism is due to thyroid gland dysgenesis in 80 . as thyroglossal cysts then move upward. 2918 Table 341-5 Total thyroid hormone levels are elevated when TBG is increased due to estrogens (pregnancy. AJAY MATHUR Endocrinology 300 Which of the following statements about resistance to thyroid 306 TSH levels are elevated in all except ? hormone (RTH) is false ? Harrison’s 18th Ed. hormone therapy. TSH-secreting pituitary tumor D. Thyroid hormone binding ratio (THBR) x Free T3 Harrison’s 18th Ed. First trimester of pregnancy C. and also in TSH-secreting pituitary A. 2. Hormone replacement therapy 311 Which of the following is the most common symptom of C. All of the above Harrison’s 18th Ed. tachycardia & impaired metabolic responses to thyroid hormone. Goiter tumor. Thyroid hormone binding ratio (THBR) / Free T3 A. B. 2917 Clinical features of RTH include goiter. Idiopathic Harrison’s 18th Ed. delayed ti e skeletal maturation. Thyroid hormone resistance B. 7. thyroid hormone resistance & assay artifact. h B. Thyroid gland dysgenesis The product of THBR and total T3 or T 4 provides the free T3 or T4 index. Raising of arms allowing thyroid imaging & quantitation of radioactive tracer fractional uptake. Extrusion of tongue - 309 What frequency of transducer is best for thyroid ultrasonography ? D.15% & is TSH-R antibody-mediated in 5% of affected newborns. Mostly no A. Nephrotic syndrome hypothyroidism ? D. 2229 D. RTH is caused by mutations in the R receptor gene. Hair loss 305 In pregnancy. 2918 Table 341-5 D. tamoxifen). 123 I treatment is indicated. Dopamine G Harrison’s 18th Ed. 2918 Large retrosternal goiters can cause venous distention over the neck and difficulty breathing. C. Any of the above A. 2916 Elevated TSH levels occur in hypothyroidism most commonly. 2916 A. 131 I n Harrison’s 17th Ed. rather inappropriate treatment of mistaken hyperthyroidism should be avoided. B. 125 I 302 Pemberton’s sign is elicited by ? C. Thyroid hormone binding ratio (THBR) / Total T3 310 Neonatal hypothyroidism is most frequently due to ? ta C. 125I. circulating free T4. Decreased Harrison’s 18th Ed.85%. All of the above A. Dry skin contraceptives. 2917 Harrison’s 18th Ed. B. Weight gain with poor appetite A. 2918 D. Due to mutation in TR receptor gene C. to inborn errors of thyroid A. 5 µg increments D. absence of colloid & C. > 9 mU/L A.12. 2921 Harrison’s 18th Ed.5 or 25 µg increments if the TSH is high. 75 . 549 Endocrinology MCQ’s FOR MCQ’s FOR MEDICAL MEDICAL PROFESSIONALS PROFESSIONALS BY PROF. T3 & T4 alternately cells. Inappropriate TSH secretion D. All of the above C. Slow-wave activity on EEG A. Any of the above myoclonus & slow-wave activity on EEG. 2 to 3 months In Hashimoto’s thyroiditis. Activated CD4+ T cells hypothyroidism ? Harrison’s 18th Ed. 2918 Adjustment of levothyroxine dosage is made in 12. atrophy of thyroid follicles accompanied by oxyphil metaplasia. 15 days to 2 months B. daily replacement dose of levothyroxine is usually 1. T3 alone D. 1 month D.150 µg). C. Cholestyramine A. Grand Mal seizure Harrison’s 18th Ed.8 µg/kg body weight C. there is a marked lymphocytic infiltration of thyroid with germinal center formation. 2. Isoniazid D. 2921 C. 2 months 313 Symptoms of hypothyroidism become more readily apparent at TSH levels of ? TSH responses to levothyroxine therapy are gradual & should be measured ~ 2 months after Harrison’s 18th Ed. Poor compliance C. Ferrous sulfate B. > 10 mU/L B.100 µg increments 314 Which of the following is false about Hashimoto’s thyroiditis ? Harrison’s 18th Ed. Lovastatin 318 TSH responses should be measured after what time upon B. patients experience full relief B. 25 . A. 323 Drugs that interfere with T4 absorption / clearance are all except ? 317 Daily replacement dose of levothyroxine is usually ? Harrison’s 18th Ed. Steroid-responsive B. composed of ? Harrison’s 18th Ed. 6 months to 1 year 315 Lymphocytic infiltrate in autoimmune hypothyroidism is Patients experience full relief from symptoms 3 . 2921 D. CD8+ T cells and B cells. 15 days D. > 8 mU/L Harrison’s 18th Ed. A. There is no place for liothyronine alone as long-term replacement due to its short half-life which necessitates multiple dosage per day and is associated with fluctuating T 3 levels.6 µg/kg body weight B. 12. Dry coarse skin B. > 7 mU/L 319 If TSH is high. 316 Which of the following is false about Hashimoto’s encephalopathy ? 322 In patients taking Levothyroxine (> 200 µg/day) with elevated Harrison’s 18th Ed. 2. 1. Peripheral oedema C. 7 days C.6 µg/ kg body weight (typically 100 . D. Diffuse alopecia A. 2921 C. Harrison’s 18th Ed. Thyroid cell destruction is primarily mediated by CD8+ cytotoxic T D.6 months after normal TSH levels are restored. Activated CD8+ T cells A. levothyroxine dosage should be increased by ? B.75 µg increments D. 2918 instituting treatment. 2919 321 Which of the following treatments is least recommended in A. May progress to atrophic thyroiditis A. May occur in autoimmune thyroiditis without hypothyroidism C. T4 alone C. No fibrosis Harrison’s 18th Ed. 6. B cells B. Calcium supplements C. Atrophy of thyroid follicles from symptoms after ? C. 2921 A. 2921 B. Malabsorption Hashimoto’s encephalopathy is a steroid-responsive syndrome associated with TPO antibodies.25 . 1.25 µg increments Symptoms of hypothyroidism become more readily apparent at when TSH is >10 mU/L. Rifampicin . 2919 TSH level suggests the possibility of ? A. AJAY MATHUR Cardiology 549 B. Cimetidine D.2 µg/kg body weight 324 Drugs that interfere with T4 absorption / clearance are all except ? If there is no residual thyroid function.5 .0 µg/kg body weight D. 2921 B. 3 to 6 months mild to moderate fibrosis. Lymphocytic infiltration 320 Upon normalization of TSH levels. T3 & T4 combined Thyroid lymphocytic infiltrate in autoimmune hypothyroidism is composed of activated CD4+. 2921 A. D. Aluminum hydroxide levothyroxine treatment ? Harrison’s 18th Ed. antidepressants).25 µg/day with similar increments every 2 . 2922 A. 2923 Table 341-7 r B. 332 Which of the following is not indicated while treating myxedema coma ? 326 During pregnancy. dose of levothyroxine may need to be ? Harrison’s 18th Ed.550 MCQ’s FOR MEDICAL PROFESSIONALS BY PROF. Inability to maintain convergence for close vision .5 µg/day A. nasogastric tube. Retracted lids causing wide palpebral opening GI bleeding. Diminished frequency of blinking hyponatremia also contribute to the development of myxedema coma. hypoglycemia & dilutional C. 400 µg D. Diminished frequency of blinking B. pneumonia. Anesthetics D. Adolescents C. Decreased R C. Lid retraction or lag D. 50 % - Hyperthyroidism of Graves’ disease is caused by thyroid-stimulating immunoglobulins (TSI) that are synthesized in the thyroid gland as well as in bone marrow & lymph nodes. 2922 A. estrogen therapy & drugs that interfere with T 4 absorption or clearance like cholestyramine. amiodarone. Tremor A. CHF. Lidocaine A. aluminum hydroxide. 300 µg C.5 to 25 µg/day 9 Harrison’s 18th Ed. Broad-spectrum antibiotics C. carbamazepine & phenytoin. ferrous sulfate. 25 to 37. Warm. Sepsis or CVAs. Retracted lids causing wide palpebral opening A. anesthetics. starting dose of levothyroxine is 12. Stellwag’s sign refers to ? D. 2923 Harrison’s 18th Ed. Inability to maintain convergence for close vision C. 12. 2922 A. 2921 Harrison’s 18th Ed. Increased B. Neonates B. von Graefe’s sign refers to ? 330 Myxedema coma is precipitated by ? A. 2923 Table 341-7 329 Myxedema coma almost always occurs in ? Harrison’s 18th Ed.3 months until TSH is normalized. Carbamazepine C. Tachycardia C. Hypotonic intravenous fluids D. All of the above A. hypotonic intravenous fluids should be avoided as they may exacerbate water The dose of levothyroxine may need to be increased by >=50% during pregnancy and returned to retention secondary to reduced renal perfusion & inappropriate vasopressin secretion.5 µg/day C. calcium supplements. Stopped D. 2922 Harrison’s 18th Ed. small-bowel In myxedema coma. 50 to 62. Amiodarone B. Spleen B. such as drugs (sedatives. Heat intolerance and sweating h D. 2922 A. hypoventilation. Diarrhoea ta Elderly patients may require up to 20% less thyroxine than younger patients. Bone marrow U C. Sedatives C. lovastatin. Palpitation i C. Parenteral hydrocortisone B. Lymph nodes D. Lagging of upper eye lid on looking downward Myxedema coma almost always occurs in the elderly & is usually precipitated by factors that impair respiration. Lagging of upper eye lid on looking downward Harrison’s 18th Ed.5 to 50 µg/day B. 335 Which of the following is the most common sign of hyperthyroidism ? Harrison’s 18th Ed. moist skin B. 500 µg of Levothyroxine can be given either as intravenous bolus or by surgery). 328 In elderly patients with CAD. 20 % C. 30 % D. 200 µg B.5 . 37. D. All of the above V In myxedema coma. 500 µg Causes of increased levothyroxine requirements are malabsorption (celiac disease. MI. Weight loss and increased appetite D. B. of most medications is impaired so reduced doses are recommended. the starting dose of levothyroxine is ? 9 334 Which of the following is the most common symptom of Harrison’s 18th Ed. Antidepressants 337 In hyperthyroidism. Metabolism d previous levels after delivery. Phenytoin D. 10 % n B. 2922 hyperthyroidism ? A. External warming G A. In elderly with coronary artery disease. ti e 327 Thyroxine doses in elderly patients should be less than younger 333 TSI are synthesized in all except ? patients by ? Harrison’s 18th Ed. Thyroid gland A. 2922 B. AJAY MATHUR Endocrinology 325 Drugs that interfere with T 4 absorption / clearance are all 331 Initial dose of levothyroxine in myxedema coma given by except ? nasogastric tube is ? Harrison’s 18th Ed. Adults D. rifampicin. Elderly 336 In hyperthyroidism. Exposure to cold. Toxic multinodular goiter D. Inability to maintain convergence for close vision Harrison’s 18th Ed. More A. When using antithyroid drugs.15 mCi 342 Which of the following is not a cause of primary hyperthyroidism ? B. In hyperthyroidism. Dose related B. 21 days C. Inability to maintain convergence for close vision B.25 mCi Harrison’s 17th Ed. 350 Usually.45 mCi B. Lagging of upper eye lid on looking downward B. Retracted lids causing wide palpebral opening C. 2926 339 In hyperthyroidism. Subacute thyroiditis D. B. thyroid function tests & clinical manifestations are reviewed 3 . 1 month B. At least 3 days hyperthyroidism” ? D. as the onset of agranulocytosis is idiosyncratic and abrupt. Retracted lids causing wide palpebral opening C. Moebius’ sign refers to ? A. pretreatment with antithyroid drugs should be done for at least ? D. Less B. Diminished frequency of blinking 346 To avoid thyrotoxic crisis after radioiodine therapy. 2926 A. 3 months C. Any of the above Hyperthyroidism can persist for 2 . increased doses of digoxin are needed. Subacute thyroiditis achieve optimum iodine uptake. second dose of radioiodine can be given after what 344 Agranulocytosis due to anti-thyroid drugs is ? duration in persistent hyperthyroidism ? Harrison’s 18th Ed. Diminished frequency of blinking A. Dalrymple’s sign refers to ? D. 2926 Harrison’s 18th Ed. Toxic adenoma 348 Radioactive 131 I dose used for thyrotoxicosis is ? Harrison’s 18th Ed. Unbound T4 levels C. B. TSH D. 2925 A. 3 months C. 15 mCi . 2233 Table 335-6 C. Toxic adenoma 131 I dosage generally ranges between 185 MBq (5 mCi) to 555 MBq (15 mCi). 551 Endocrinology MCQ’s FOR MCQ’s FOR MEDICAL MEDICAL PROFESSIONALS PROFESSIONALS BY PROF. At least 2 days 341 Which of the following is not a cause of “thyrotoxicosis without C. 340 In hyperthyroidism. Unbound T3 levels B. 25 mCi . Lagging of upper eye lid on looking downward In atrial fibrillation due to thyrotoxicosis. 351 Women can conceive safely how many months after radioiodine 345 In atrial fibrillation due to thyrotoxicosis. 6 months D.4 weeks after starting treatment & the dose is titrated based on unbound T 4 levels. 2926 Harrison’s 18th Ed. AJAY MATHUR Cardiology 551 338 In hyperthyroidism. 1 month B. Duration related C. ~6 months after first dose. 4 to 5 months D. Inability to maintain convergence for close vision Risk of thyrotoxic crisis after radioiodine therapy can be minimized by pretreatment with antithyroid drugs for at least a month before treatment. dose of digoxin is ? treatment ? Harrison’s 18th Ed.35 mCi A. 5 mCi . Any of the above A. 35 mCi . or a larger dose of radioiodine will be necessary. Abadie’s sign refers to ? 347 Antithyroid drugs must be stopped how many days before A. 9 months Persistent hyperthyroidism is treated with a 2nd dose of radioiodine. Propylthiouracil has a prolonged radioprotective effect & is stopped several weeks before radioiodine is given. Retracted lids causing wide palpebral opening Harrison’s 18th Ed. C. Thyrotoxicosis factitia A. 7 days A. 14 days B. Silent thyroiditis C. 2233 Table 335-6 Carbimazole or methimazole must be stopped at least 2 days before radioiodine administration to A. At least 7 days Harrison’s 17th Ed. Idiosyncratic A. 1 month D. Functioning thyroid carcinoma metastases 349 Hyperthyroidism can persist for how long before radioiodine takes full effect ? 343 Titration of doses of anti-thyroid drugs is best based on ? Harrison’s 18th Ed. 2926 D. 3 to 4 months C. All of the above D. Same . 1 to 2 months A. At least 1 day D. 2926 C. Diminished frequency of blinking D. 2 to 3 months B. 2926 A.3 months before radioiodine takes full effect. Involuntary twitching or spasm of LPS muscle administration of radioiodine ? B. 2926 Harrison’s 18th Ed. it is not useful to monitor blood counts prospectively. Dense fibrosis disrupts normal gland architecture that can extend outside thyroid capsule. Low radioiodine uptake C. Amiodarone D. Decreased total & unbound T3 levels A. Low radioiodine uptake G because of relatively low transplacental transfer and its ability to block T 4 to T 3 conversion. 2928 A. Normal TSH levels C. . neck veins or recurrent laryngeal nerves. Raised serum IL-6 levels D. normal ESR and presence of TPO antibodies. Chronic thyroiditis D. 131 I treatment local symptoms due to compression of esophagus. 6 months 358 Postpartum thyroiditis occurs how many months after D. B. First d Harrison’s 18th Ed. lung. low radioiodine uptake and thyroid with normal levels of T4 and TSH. 2929 C. Methimazole Harrison’s 18th Ed. . Amiodarone Despite extensive histologic changes. 2929 Harrison’s 18th Ed. 2926 D. 3 to 6 months 352 Which anti-thyroid drug is preferred in pregnancy with Graves’ C. Normal T4 levels B. All of the above C. 2928 A. 6 to 9 months disease ? Harrison’s 18th Ed. All of the above D. antibodies are negative. B. C. Carbimazole 359 Which of the following is false about silent thyroiditis ? C. IL-2 or amiodarone may develop painless thyroiditis. and orbit. Any of the above Riedel’s thyroiditis is associated with idiopathic fibrosis in retroperitoneum. No thyroid tenderness D. Thyroid dysfunction common 9 thyroiditis ? D. A. Hashimoto’s thyroiditis 361 Which of the following is false about Riedel’s thyroiditis ? - Harrison’s 18th Ed. Riedel’s thyroiditis U B. ta D. 2929 While pregnancy & breast feeding are absolute contraindications to radioiodine treatment. biliary tree. 2929 356 de Quervain’s thyroiditis is a type of ? Harrison’s 18th Ed. Mycobacterial infection beneficial. None of the above D. Presents insidiously as painless goiter with A. Any of the above A. Acute thyroiditis B. 1 to 3 months can conceive safely 6 months after treatment. 9 months pregnancy ? Harrison’s 18th Ed. Subacute thyroiditis C. 363 Which of the following is false about sick euthyroid syndrome ? 357 Which of the following is false about subacute thyroiditis ? Harrison’s 18th Ed. Dense fibrosis of thyroid 9 355 Which of the following can cause acute. Tamoxifen may be C. mediastinum. or viral thyroiditis. Second A. Riedel’s thyroiditis 362 Riedel’s thyroiditis is associated with fibrosis of ? Harrison’s 18th Ed. Tamoxifen therapy beneficial r Harrison’s 18th Ed. subacute or chronic C. High ESR B. AJAY MATHUR Endocrinology C. Painless goiter D. High ESR Propylthiouracil is titrated to lowest effective dose to manage Graves’ disease in pregnancy B. h B.6 months after pregnancy. Carbimazole & methimazole rarely cause fetal aplasia cutis and choanal atresia.552 MCQ’s FOR MEDICAL PROFESSIONALS BY PROF. pregnancy ? V Harrison’s 16th Ed. IL-2 D. 2929 B. Parasitic thyroiditis (Echinococcosis) B. there is a decrease in total & unbound T3 levels (low T3 syndrome) In subacute thyroiditis. 35 360 Which of the following can cause thyroiditis ? A. Radiation thyroiditis after 131I treatment A. thyroid dysfunction is uncommon. diagnosis is confirmed by a high ESR. 2928 Table 341-8 i Riedel’s thyroiditis is seen in middle-aged women. Propylthiouracil Postpartum thyroiditis occurs 3 . Thyroid antibodies present In sick euthyroid syndrome (SES). patients A. Retroperitoneum A. Mediastinum B. 2929 D. All of the above n 354 Which of the following is not a cause of chronic thyroiditis ? Harrison’s 18th Ed. Lung C. Any of the above A. Third B. Presence of TPO antibodies R 353 Hyperthyroidism is most difficult to control in which trimester of Silent thyroiditis features a painless goiter. Subacute Thyroiditis is also termed de Quervain’s thyroiditis. trachea. granulomatous thyroiditis. IFN- ti e C. 2928 Table 341-8 Patients receiving IFN. 76 amino acid A. C. All of the above 375 Size of polypeptide pro opiomelanocortin (POMC) is ? 369 Chemodectomas are derived from ? Harrison’s 18th Ed. leading to increased reverse T 3 (rT 3). 2929 adrenal androgens. 39 amino acid Harrison’s 18th Ed. 341 amino acid C. Unbound T3 levels 371 Which of the following is produced by adrenal cortex ? Harrison’s 18th Ed. then to outer cortical zona glomerulosa. Any of the above C. Increased supply is from outside to inside. Zona reticularis 367 During pregnancy. Iodine A. Sexual differentiation occurs at 7th . Mineralocorticoids ACTH peptide contains 39 amino acids but first 24 are sufficient to elicit a physiologic response. Iron C. Cassava root C.6 gram C. 2930 373 Which of the following is located outermost in adrenal gland ? A. Zinc Harrison’s 18th Ed. Adrenal androgens . Calcium B. mineralocorticoids & Harrison’s 18th Ed. Zona fasciculata D. Adrenal medulla Harrison’s 18th Ed. Adrenal medulla B. D. Chapter 342. 2940 C. 21 amino acid 370 Corticosteroid hormone produced by adrenal cortex is ? B. Harrison’s 18th Ed.11 gram. D. urinary iodide excretion is ? D. A. Cabbage D. Decreased A. 6 . All of the above Harrison’s 18th Ed. 2 . Cauliflower Right suprarenal vein drains directly into vena cava while left drains into left renal vein. 2941 B. Same 374 Which of the following about adrenal glands is false ? D. 2148 A. 553 Endocrinology MCQ’s FOR MCQ’s FOR MEDICAL MEDICAL PROFESSIONALS PROFESSIONALS BY PROF. Adrenals originate from urogenital ridge 368 Which of the following is an environmental goitrogen ? Harrison’s 18th Ed. Initially to subcapsular region. All of the above Corticotropin-releasing hormone (CRH) stimulates cleavage of 241-amino acid polypeptide pro opiomelanocortin (POMC) by pituitary-specific prohormone convertase to produce adrenocorticotropic hormone (ACTH). Reverse T3 levels A. Increased Harrison’s 18th Ed. 2940 C. Left suprarenal vein drains into vena cava B. Postganglionic sympathetic neurons D. Carotid body C. Aldosterone T4 conversion to T3 via peripheral deiodination is impaired.11 gram D. Decreased to adrenal medulla. 11 . 2941 B.18 gram Amiodarone is structurally related to thyroid hormone & contains 39% iodine by weight. 141 amino acid A. 98 amino acid B.9th week of gestation A. Dehydroepiandrosterone D. AJAY MATHUR Cardiology 553 364 Which of the following is increased in sick euthyroid D. 2931 B. 2941 Harrison’s 16th Ed. 441 amino acid D. Cortisol 365 Amiodarone contains which of the following elements ? D. 2930 372 Each normal adrenal gland weighs ? A. TSH levels B. All of the above syndrome ? Corticosteroid hormones produced by adrenal cortex are glucocorticoids. Glucocorticoids D. 2941 A. then to inner zona reticularis and eventually B. Zona glomerulosa C. Any of the above Harrison’s 18th Ed. 18 . thyroid hormone requirements are ? Each normal adrenal gland weighs 6 . Arterial blood A. Disorders of the Adrenal 376 Size of ACTH peptide is ? Cortex Harrison’s 18th Ed. 2941 A. Same B. Total T3 levels B. 2930 Adrenal glands are located above the kidneys and have their own blood supply. C. 241 amino acid B. then to intermediate zona fasciculata. C.31 gram 366 During pregnancy. SF1 and DAX1 are nuclear receptors C. 20-lyase (CYP17A1) 9 for glucocorticoid synthesis. 3-HSD2 Harrison’s 18th Ed. ACTH receptor MC2R (melanocortin C. 2941 C. PKA activation increases import of cholesterol esters. None of the above A. Immune responses B. D. 2940 Figure 342-1 A. 11-hydroxylase (CYP11B1) A. 11-hydroxylase (CYP11B1) d 2 receptor) interacts with MC2R-accessory protein (MRAP) and this complex at adrenocortical cell membrane binds to ACTH. 2940 Figure 342-1 binding). which moves cholesterol from outer to inner mitochondrial membrane. 2942 to androstenedione ? Harrison’s 18th Ed. 11-hydroxylase (CYP11B1) following to bind ACTH ? D. All of the above B.20-lyase (CYP17A1) defined as a cortisol level > 20 µg/dL or an increment of >10 µg/dL over baseline. > 5 µg / dL D. Blood pressure Adrenal steroidogenesis occurs in a zone-specific manner. 2940 Figure 342-1 The standard ACTH stimulation test involves administration of cosyntropin (ACTH 1-24). AJAY MATHUR Endocrinology 377 A normal response in standard ACTH stimulation test is defined A. 2942 G A. B. A normal response is A. 21-hydroxylase (CYP21A2) 378 MC2R (melanocortin 2 receptor) interacts with which of the C. Electrolytes 381 Cholesterol import into mitochondrion is initiated by the action Glucocorticoids modulate intermediary metabolism & immune responses. 21-hydroxylase (CYP21A2) ir 380 Which of the following statements is false ? C. 2942 h A. glucocorticoid synthesis in zona fasciculata. 17-hydroxylase/17. All of the above - D. 3-HSD2 ACTH stimulation generates cyclic AMP (cAMP) which then upregulates protein kinase A (PKA) signaling pathway. D. Adrenal androgen synthesis occurs in zona reticularis Harrison’s 18th Ed. which cleaves cholesterol esters to cholesterol for import into 386 Which of the following enzyme converts 11-deoxycortisol to cortisol ? 9 mitochondrion and increases availability & phosphorylation of CREB (cAMP response element Harrison’s 18th Ed. and adrenal androgen synthesis C. 2942 characteristics (in females). CREB acts in the nucleus and makes available CYP11A1 which D. Vascular volume in inner zona reticularis. Basic structure of steroids is built upon a five-ring nucleus. 3-HSD2 B. MRCP A. MRDP B. 2248 Harrison’s 18th Ed. 21-hydroxylase (CYP21A2) Harrison’s 18th Ed.554 MCQ’s FOR MEDICAL PROFESSIONALS BY PROF. > 20 µg / dL hydroxyprogesterone ? Harrison’s 18th Ed. Mineralocorticoid synthesis occurs in zona glomerulosa D. Mineralocorticoid synthesis occurs in outer zona glomerulosa. 2940 D. 17-hydroxylase/17. CYP11A1 A. A. 21-hydroxylase (CYP21A2) ACTH stimulation is required for initiation of steroidogenesis. 30. Breaks cholesterol esters to cholesterol B. Glucocorticoid synthesis occurs in zona fasciculata ta 387 Action of mineralocorticoids include all except ? C. a process initiated C. increases activity of hormone-sensitive lipase. Three-rings D. and collection of blood samples at 0. 2940 Figure 342-1 C. > 15 µg / dL 383 Which of the following enzyme converts progesterone to 17- D. Increases availability & phosphorylation of CREB C. vascular volume & electrolytes and androgens address secondary sexual Harrison’s 18th Ed. 2940 Figure 342-1 . Five-rings by the action of steroidogenic acute regulatory (StAR) protein. and 60 minutes for cortisol. Six-rings traverses to the mitochondria.20-lyase (CYP17A1) as a cortisol level of ? B. 17-hydroxylase/17. Four-rings All steroidogenic pathways require cholesterol import into the mitochondrion. > 10 µg / dL C. 17-hydroxylase/17.20-lyase (CYP17A1) B.20-lyase (CYP17A1) V D. 11-hydroxylase (CYP11B1) D. 3-HSD2 ti e 379 PKA activation affects steroidogenesis through ? 385 Which of the following enzyme converts 17-hydroxyprogesterone Harrison’s 18th Ed. 17-hydroxylase/17. MRAP 384 Which of the following enzyme converts 17-hydroxyprogesterone to 11-deoxycortisol ? B. a transcription factor that enhances transcription of CYP11A1 and other enzymes required A. 3-HSD2 B. MRBP R Harrison’s 18th Ed. Mineralocorticoids of ? maintain blood pressure. Increase in import of cholesterol esters n A. B. Steroidogenic acute regulatory (StAR) protein rings ? C.25 mg IM or IV. 382 Which of the following enzyme converts pregnenolone to 389 Number of carbon atoms in 17-hydroxycorticosteroids is ? progesterone ? Harrison’s 17th Ed. 0. 11-hydroxylase (CYP11B1) Harrison’s 18th Ed. 21-hydroxylase (CYP21A2) U C. Phosphorylation of CREB 388 Basic structure of steroids is built upon a nucleus with how many B. 2943 390 Number of carbon atoms in 17-ketosteroids is ? A. high-capacity Harrison’s 17th Ed. Steroids D. 19 397 Physiologically active form of cortisol is ? D. VLDL receptor 401 Property of albumin for cortisol binding is ? Harrison’s 17th Ed. Anacortin C.& 17-hydroxylase are expressed in (inner) fasciculata-reticularis cell layers. Cortisol metabolites Harrison’s 17th Ed. Transcortin Harrison’s 18th Ed. B. Triglycerides C. 21 Adrenal steroids contain either 19 or 21 carbon atoms. Normally. 19 21. 2248 393 Basic substrate for steroidogenesis is ? A. low-capacity  2-globulin termed A. All of the above C. Glucocorticoid properties Only the unbound cortisol and its metabolites are filterable at the glomerulus. bioavailable D. < 5 percent 392 Cyclopentenoperhydrophenanthrane nucleus is a constituent B. 18 Phenomenon of zonation exists in adrenal cortex in which selective genes express enzymes that form specific hormones. Low-affinity. B. 20 Harrison’s 18th Ed. Mineralcorticoid properties 398 What percentage of circulating cortisol is in the free. 2248 Plasma has two cortisol-binding systems. Harrison’s 18th Ed. Polysaccharides 400 Property of transcortin or cortisol-binding globulin (CBG) is ? Harrison’s 17th Ed. Adrenocortin A. Pepsin D. Chapter 331 C. Low-affinity. low-capacity Harrison’s 17th Ed. High-affinity. Cortisol metabolites B. 2248 . high-capacity layer of adrenal cortex ? D. Epicortin B. C21 steroids with a hydroxyl group at position 396 Cortisol circulates in the plasma as ? 17 are termed 17-hydroxycorticosteroids. Aldosterone synthase is expressed only in outer (glomerulosa) cell layer. 17 D. high-capacity A. Low-affinity. 2248 D. respectively. High-affinity. 2248 D. 2943 C19 steroids have methyl groups at C-18 & C-19. C. Free cortisol 17-ketosteroids. Growth hormone 399 Cortisol-binding system in plasma is ? Harrison’s 17th Ed. 17-hydroxylase transcortin or cortisol-binding globulin (CBG) and albumin which is a low-affinity. C19 steroids with a ketone group at C-17 are termed A. HDL receptor C. and cortisol metabolites. C. High-affinity. Cholesterol D. C. Aldosterone synthase Harrison’s 17th Ed. Amino acids D. All of the above A. Protein-bound cortisol A. A. 2248 B. Insulin Free cortisol is a physiologically active hormone that is not protein-bound. Androgenic properties Cortisol circulates in the plasma as free cortisol. None of the above A. protein-bound cortisol. < 20 percent of ? Harrison’s 15th Ed. D. which form cortisol & androgen biosynthesis. < 30 percent A. 18 D. AJAY MATHUR Cardiology 555 A. B. low-capacity Uptake of cholesterol by adrenal cortex is mediated by low-density lipoprotein (LDL) receptor. < 40 percent B. high-capacity 395 Which of the following is expressed in outer (glomerulosa) cell C. 21-hydroxylase 402 Cortisol binds to albumin beyond what concentration ? C. Harrison’s 17th Ed. B. Free cortisol Harrison’s 17th Ed. 2943 B. One is a high-affinity. low-capacity 394 Uptake of cholesterol by adrenal cortex is mediated by ? B. All of the above form ? C19 steroids have predominantly androgenic activity. 555 Endocrinology MCQ’s FOR MCQ’s FOR MEDICAL MEDICAL PROFESSIONALS PROFESSIONALS BY PROF. Low-affinity. All of the above B. LDL receptor D. 17 C. 2248 A. <5% of circulating cortisol is free. high-capacity protein. 2248 Cholesterol from diet and from endogenous synthesis is the substrate for steroidogenesis. High-affinity. Protein-bound cortisol 391 C19 steroids have predominantly ? C. low-capacity B. 2248 C. Pregnancy C. between ? Harrison’s 17th Ed. Endometrium B. Androstenedione h Harrison’s 17th Ed. AJAY MATHUR Endocrinology A. Liver C. Promote binding of free cortisol to transcortin B. >75% of circulating aldosterone is normally inactivated by C. 10 and 15 mg During a single passage through liver. All of the above 413 The average daily secretion of androgens by adrenal gland ranges Steroid inactivation occurs mainly in liver. Inhibit binding of free cortisol to transcortin C. 2943 B. 10 to 15 mg A. Bind weakly to circulating plasma proteins With normal salt intake. 11-hydroxyandrostenedione ta B. D. D. Kidney Major androgen secreted by adrenal is dehydroepiandrosterone (DHEA) & its sulfuric acid ester (DHEAS). 2248 n Cortisol metabolites are biologically inactive & bind weakly to circulating plasma proteins. almost all urine 17-ketosteroids are derived from ? Enzyme that regulates cortisol metabolism is 11-hydroxysteroid dehydrogenase (11  -HSD).556 MCQ’s FOR MEDICAL PROFESSIONALS BY PROF. D. Converting cortisone to cortisol 403 The cortisol-binding globulin (CBG) level is increased in ? Harrison’s 17th Ed. All of the above . Harrison’s 17th Ed. Smaller amounts of androstenedione. Testosterone C. 15 and 30 mg conjugation with glucuronic acid. 300 and 600 µg d B. cortisone. 50 to 100 mg C. All of the above 414 In females. Harrison’s 18th Ed. 25 µg/dL 11 -HSD 1 acts as a reductase. 50 and 250 µg A. Adrenal Harrison’s 18th Ed. i A. 45 µg/dL 409 -hydroxysteroid dehydrogenase 2 is ? Function of 11 When concentration of cortisol is > 25 µg/dL. 2943 A. 5 and 10 mg D. > 25 % U Harrison’s 17th Ed. D. 9 D. 10 and 15 µg V Harrison’s 17th Ed. converting inactive cortisone to active glucocorticoid. None of the above 410 Average daily secretion of aldosterone ranges between ? G CBG is increased in high-estrogen states (pregnancy. 2943 A. Converting cortisol to inactive cortisone D. oral contraceptive). Low levels in urine are typical of children with 3-HSD2 ti e deficiency 411 What percentage of circulating aldosterone is normally inactivated D. Ovaries 408 -hydroxysteroid dehydrogenase 1 is ? Function of 11 B. 15 µg/dL D. 30 and 60 mg 412 The major androgen secreted by adrenal gland is ? 9 Harrison’s 17th Ed. cortisol. 5 to 10 mg Harrison’s 18th Ed. A. 2248 A. > 50 % - A. Lung D. > 75 % B. Kidney ~15 . average daily secretion of aldosterone is between 50 & 250 µg. Biologically inactive D. Promote binding of free cortisol to transcortin B.30 mg of androgens are secreted daily. Harrison’s 17th Ed. 5 and 10 µg R 404 Which of the following statements about cortisol metabolites is false ? B. 2248 C. Both of the above 11 -HSD 2 isoform converts cortisol to the inactive metabolite. 11-hydroxyandrostenedione & testosterone are secreted. C. Inhibit binding of free cortisol to transcortin C. 15 to 30 mg B. with a pronounced circadian cycle. Converting inactive cortisone to active cortisol C. 2248 B. 2248 C. 5 µg/dL C. 2248 407 -hydroxysteroid dehydrogenase 1 (11 11 -HSD 1) is primarily expressed in ? A. part of excess binds to albumin. Liver C. Oral contraceptive administration D. 2248 A. 2248 r Daily secretion of cortisol ranges between 15 & 30 mg (8 . Lung D. Dehydroepiandrosterone (DHEA) 406 Steroid inactivation occurs in ? B. Isoform 11 -HSD 1 is primarily expressed in liver.10 mg/m2). > 10 % 405 The daily secretion of cortisol ranges between ? B. None of the above during a single passage through liver ? Harrison’s 17th Ed. Converting cortisol to cortisone A. 2248 A. In female. 557 Endocrinology MCQ’s FOR MCQ’s FOR MEDICAL MEDICAL PROFESSIONALS PROFESSIONALS BY PROF. Catalytic transport C. 2249 425 Which of the following is released in equimolar concentrations with ACTH ? A. almost all urine 17-ketosteroids are derived from the adrenal. Acidophilic cells of posterior pituitary D. Sleep-wake cycle D. C. 2249 A. All of the above Glucocorticoids & mineralocorticoids bind with nearly equal affinity to the mineralocorticoid receptor Release of ACTH from anterior pituitary gland is stimulated by corticotropin-releasing hormone (MR). Free plasma cortisol concentration C. Just prior to waking up POMC is made in brain. a neuropeptide related to CRH. FSH ACTH. ACTH A. 2249 A. Acidophilic cells of anterior pituitary B. 2249 D. ACTH B. All of the above D. All of the above A. 2249 D. All of the above Harrison’s 17th Ed. B. Urocortin. endorphins & melanocyte-stimulating hormones are processed from a larger precursor molecule proopiomelanocortin (POMC). All of the above D. Just prior to waking up D. After meals A. Posterior pituitary C. 416 Which of the following bind to both glucocorticoid & mineralocorticoid receptors ? 423 Major factors controlling ACTH release include ? Harrison’s 17th Ed. 2249 -lipotropin ( -LPT) is released in equimolar concentrations with ACTH. Melanocyte-stimulating hormone C. stress & sleep-wake cycle also control ACTH release. Corticotropin-like intermediate lobe protein (CLIP) A. Metenkephalin Harrison’s 17th Ed. remaining 1/3 comes from testicular androgens. Corticotropin-like intermediate lobe protein (CLIP) 419 POMC (Pro-opiomelanocortin) is made in ? D. depending on the nature of stimulus. 2249 D. Corticotropin-releasing hormone B. Anterior pituitary A. suggesting that they are A. 2249 B. All of the above B. Glucocorticoids A. anterior & posterior pituitary and lymphocytes. Passive diffusion A. Mineralocorticoids B. Lymphocytes 426 The plasma level of ACTH peak ? Harrison’s 17th Ed. Anterior pituitary cleaved enzymatically from the parent POMC before or during secretory process. lipotropins. -Lipotropin 427 The plasma level of ACTH are lowest at ? B. 2248 Harrison’s 17th Ed. Glucocorticoids derived from adrenal metabolites.  -endorphin levels may or may not correlate with circulating levels of ACTH. Leptin D. 2248 422 ACTH is synthesized and stored in ? Harrison’s 17th Ed. All of the above C. (CRH) produced in median eminence of hypothalamus. CRH Plasma ACTH peak just prior to waking and a nadir before sleeping. B. After meals A. 418 POMC is made in ? Harrison’s 17th Ed. -Lipotropin C. Endorphin B. is synthesized & stored in basophilic cells of anterior pituitary. Arginine vasopressin D. 2249 A. CRH C. ACTH. 417 Pro-opiomelanocortin (POMC) is a precursor molecule for ? Harrison’s 17th Ed. 2249 C. Only glucocorticoids bind to the glucocorticoid receptor (GR). IL-6 415 Steroids pass through the cell membrane by ? Harrison’s 17th Ed. Just after waking up 421 The POMC gene is induced by all except ? C. anxiety). Basophilic cells of anterior pituitary C. -endorphin D. a 39-amino-acid peptide. AJAY MATHUR Cardiology 557 DHEA is the major precursor of urinary 17-ketosteroids. Brain Harrison’s 17th Ed. All of the above D. Androgens C. Before sleeping Harrison’s 17th Ed. Before sleeping Harrison’s 17th Ed. Active transport B. Just after waking up 420 Pro-opiomelanocortin (POMC) is a precursor molecule for ? C. 2/3 of urine 17-ketosteroids in male are B. Free plasma cortisol concentration. 2249 424 Neuropeptide ‘Urocortin’ is related to ? Harrison’s 17th Ed. Posterior pituitary B. Basophilic cells of posterior pituitary Steroids diffuse passively through the cell membrane and bind to intracellular receptors. . mimics many of the central effects of CRH (appetite suppression. AT1 receptor B. Cytokine D. 2250 A. AT3 receptor D. surgery. Activation of sympathetic nervous system A. Adrenomedulin V of CRH and arginine vasopressin (AVP) and activation of the sympathetic nervous system. 2249 A. levels of corticosteroid-binding 440 Renin is a ? Harrison’s 17th Ed. All of the above 430 Factors increasing ACTH release include ? d 437 Insulin-tolerance test should not be performed in patients with ? Harrison’s 17th Ed. 434 Suppression of the hypothalamic-pituitary-adrenal axis has been reported with ? 441 Renin acts on ? N Engl J Med 2003. <25 µg per deciliter Normal pulsatile. D. All of the above D. Medroxyprogesterone B. AJAY MATHUR Endocrinology 428 Normal pulsatile. AT1 Harrison’s 17th Ed. All of the above Harrison’s 17th Ed. 2250 - 431 Factors decreasing ACTH release include ? A. Biologic half-life of ACTH in the circulation is <10 minutes. All of the above B. AT1  and AT2.348:727-34 C. Epilepsy C. AT2 B. All of the above C. Corticotropin-releasing hormone 435 What level of cortisol level best identifies persons with clinical B. 2250 globulin ? N Engl J Med 2003. 438 Which of the following is an angiotensin receptor ? Harrison’s 17th Ed. Megestrol acetate by ? D. -endorphin 9 C. 2250 N Engl J Med 2003. 2250 Harrison’s 17th Ed. Syntropin R Stress (pyrogens. Vasopressin B. Sleep-wake cycle A. Ischemic heart disease B. hypoglycemia. Cosyntropin D. 2250 436 Synthetic peptide consisting of the first 24 amino acids of A. Enkephalin 9 D. Exogenous corticosteroid therapy for > 3 weeks A. Angiotensin I . <45 µg per deciliter Harrison’s 17th Ed. i D. Response to eating A. Angiotensinogen B.558 MCQ’s FOR MEDICAL PROFESSIONALS BY PROF.348:727-34 ti e A. Prohormone C. AT4 receptor Most of the effects of angiotensins II & III are mediated by AT1 receptor. AT2 receptor C. < 15 minutes B.348:727-34 G C. exercise & severe emotional trauma) causes the release C. < 30 minutes C. <15 µg per deciliter D. All of the above ACTH & cortisol levels increase in response to eating. Proteolytic enzyme A. < 10 minutes A. Release of CRH corticotropin and used in corticotropin stimulation test is ? B.348:727-34 Harrison’s 17th Ed. Decrease B. <35 µg per deciliter 429 Stress causes which of the following ? D. Remain same D. Severe cortisol deficiency n D. Increase C. < 60 minutes D. circadian pattern of ACTH release is regulated C. Release of AVP N Engl J Med 2003. 2250 ta A. C. Angiotensin II C. All of the above 439 Most effects of angiotensin II are mediated by ? h 432 The biologic half-life of ACTH in the circulation is ? Harrison’s 17th Ed. All of the above Renin is a proteolytic enzyme produced & stored in granules of juxta-glomerular cells surrounding afferent arterioles of glomeruli in kidney. Hormone B. All of the above B. 2250 B. ACTH release is regulated by the level of U free cortisol in plasma. 433 During critical illness. circadian pattern of ACTH release is regulated by CRH.348:727-34 A. Inflammatory cytokines r Angiotensin receptors are AT1 . AT1 A. Free plasma cortisol concentration features of corticosteroid insufficiency ? N Engl J Med 2003. 2 . angiotensin III is a heptapeptide. Inhibiting peripheral glucose uptake C. Gluconeogenesis D. Pulmonary vascular endothelium Harrison’s 17th Ed. 2250 C. Atrial 445 Tissues that have a local renin-angiotensin system and produce natriuretic peptides also inhibit renin release. Eosinophils ACE is present in pulmonary vascular endothelium.20 minutes Macula densa cells are a group of distal convoluted tubular epithelial cells directly opposed to D. Angiotensin II B. B lymphocytes D. Angiotensin III is a heptapeptide D. or aldosterone secretion. sensing renal perfusion pressure & changes in D. Angiotensin II 448 Which of the following is false about ‘juxtaglomerular cells’ ? Harrison’s 17th Ed.45 minutes juxtaglomerular cells. 30 . 2250 451 Glucocorticoids raise the blood glucose level by ? Harrison’s 17th Ed. IL-1 is an endogenous pyrogen. 10 . 2251 B. Increased dietary potassium A. converts Angiotensin I to octapeptide angiotensin D. 2251 A. heart. Angiotensin I is a decapeptide. 446 Angiotensin-converting enzyme (ACE) is present mostly in ? Harrison’s 17th Ed. adrenal cortex and kidney. Atrial natriuretic peptide Harrison’s 17th Ed. whereas decreased potassium D. Brain B. Act as pressure transducers A. Chemoreceptors B. which promotes hepatic glucose synthesis (gluconeogenesis) & hepatic glycogen content. Half-life of renin is 10 . Angiotensin convertig enzyme D. Specialized myoepithelial cells 442 Angiotensinogen is made in ? B. 2250 B. Prostaglandins B. 2250 A.10 minutes D. Vascular tissue Increased dietary intake of potassium decreases renin release. They function as chemoreceptors. Monitor potassium load C. vascular tissue. 2250 B. Release renin B. Lung may be seen as miniature pressure transducers. thereby inhibiting peripheral glucose uptake. 2251 443 Half-life of renin is ? A. Uterus C. . Harrison’s 17th Ed. blood pressure. Placenta D. Kidney Juxtaglomerular cells are specialized myoepithelial cells that cuff the afferent arterioles. Liver D. Cuff efferent arterioles Harrison’s 17th Ed. All of the above A. Kidneys B. Glucocorticoids raise blood glucose level by antagonizing secretion & actions of insulin. 5 . 2251 D. 450 Factor that decreases renin release is ? 444 Tissues that have a local renin-angiotensin system & produce Harrison’s 16th Ed. Angiotensin II is an octapeptide. IL-1 & IL-6 C. All of the above II. None of the above Glucocorticoids inhibit production & action of mediators of inflammation (lymphokines & prostaglandins).20 minutes. IL-1 & IL-6 by macrophages. Angiotensin II exerts negative feedback control on renin release that is independent of alterations in renal blood flow. Directly opposed to juxtaglomerular cells A. All of the above C. AJAY MATHUR Cardiology 559 C. 2250 452 Glucocorticoids deplete ? A. All of the above Tissues that have a local renin-angiotensin system & ability to produce angiotensin II are uterus. placenta. 559 Endocrinology MCQ’s FOR MCQ’s FOR MEDICAL MEDICAL PROFESSIONALS PROFESSIONALS BY PROF. Intestines afferent arteriolar perfusion pressures. Intestines C. Lymphokines A. monitoring sodium (or chloride) load presented to the distal tubule. Suppressing secretion of insulin B. T lymphocytes C. DCT epithelial cells Harrison’s 17th Ed. 2131 angiotensin II include all except ? A. Angiotensin II is an octapeptide C. Renin acts on angiotensinogen which is a circulating  2-globulin produced in liver to form the 449 Which of the following is false about macula densa cells ? decapeptide angiotensin I. They C. 453 Glucocorticoids inhibit all of the following except ? 447 Which of the following is false ? Harrison’s 17th Ed. Ovaries intake increases it. interferon by T lymphocytes. Liver A. Heart A. 2251 Harrison’s 17th Ed. Pancreas D. brain. angiotensin II include all except ? Harrison’s 17th Ed. Angiotensin I is a decapeptide B. Adrenal cortex C.4 minutes C. 2252 Table 336-1 457 Mineralocorticoids act on the epithelium of ? A. All of the above B. Gastrointestinal tract D. 2253 in 3 . 2252 Table 336-1 Harrison’s 17th Ed. atrial natriuretic peptide and Ouabain-like factors inhibit aldosterone biosynthesis. Ouabain-like factors potassium in ? Harrison’s 17th Ed. Neurons in brain . Bradykinin D. AJAY MATHUR Endocrinology 454 Glucocorticoids inhibit all of the following except ? B. Gastrointestinal tract Potassium. -melanocyte-stimulating hormone A. -endorphin Harrison’s 17th Ed. Myocytes Harrison’s 17th Ed. All of the above 463 Which of the following does not stimulate aldosterone Mineralocorticoids are the major determinants of potassium metabolism mediated by the binding of aldosterone to MR in epithelial cells (principal cells in renal cortical collecting duct). 2252 C. 2252 Table 336-1 serotonin. 2252 D. Aldosterone D. Harrison’s 17th Ed. Vasopressin MR is found in nonepithelial cells like neurons in brain.  -endorphin and endothelin stimulate aldosterone biosynthesis. B. There is no escape from A. All of the above A. Sweat glands 465 Adrenal androgen formation is regulated by ? Harrison’s 17th Ed. prostaglandin & leukotriene. D. B. Potassium C. -endorphin B.5 days preventing edema. 2252 Table 336-1 Mineralocorticoids act on epithelium of the salivary ducts. ACTH A. Atrial natriuretic peptide 458 Mineralocorticoids cause reabsorption of sodium in exchange for C. Sodium B. Dopamine D. Renin 461 Primary mechanism that controls aldosterone release is ? Glucocorticoids inhibit production & inflammatory effects of bradykinin. Chloride D. A. Sweat glands D. ACTH 459 Escape phenomenon in kidneys pertains to which hormone ? B. Potassium 455 Cortisol suppresses which of the following ? Harrison’s 17th Ed. 2253 D. Endothelial cells A. serotonin. Potassium ion A. Angiotensin II suppressed by exogenous glucocorticoids. Sodium. Testosterone A. Androstenedione 460 Mineralocorticoid receptor is found in ? C. 11-hydroxyandrostenedione Harrison’s 17th Ed. dopamine. biosynthesis ? Harrison’s 17th Ed. All of the above B. This process is called escape phenomenon. DHEA the potassium-losing effects of mineralocorticoids. T cell growth factor (IL-2) by T lymphocytes. Epithelium of salivary ducts C. Serotonin smooth-muscle cells. 2252 A. sweat glands & gastrointestinal tract to cause reabsorption of sodium in exchange for potassium. myocytes. 2252 C. ACTH. A. All of the above 464 Which of the following inhibit aldosterone biosynthesis ? Harrison’s 17th Ed. C. endothelial cells & vascular C. Gonadotropins Harrison’s 17th Ed. Pituitary POMC D. Salivary ducts C. Atrial natriuretic peptide B. potassium & ACTH. ACTH B. reverse macrophage activation & antagonize action of migration-inhibiting factor (MIF). Adrenal androgens are C. Renin Adrenal androgen formation is regulated by ACTH. All of the above 462 Which of the following does not stimulate aldosterone secretion ? 456 Mineralocorticoids major determinants of the metabolism of ? Harrison’s 17th Ed. Renin-angiotensin system B.560 MCQ’s FOR MEDICAL PROFESSIONALS BY PROF. D. Dopamine B. Hypothalamic CRH Primary mechanisms that control adrenal aldosterone secretion are renin-angiotensin system. Potassium 466 Which of the following is the most potent androgen ? Aldosterone initially causes sodium retention followed by natriuresis to reestablish sodium balance Harrison’s 17th Ed. Endothelin A. Serotonin C. All of the above A. not by gonadotropins. platelet-activating factor. Vasopressin D. 2251 C. 2252 D. Serotonin C. It signifies an “escape” by renal tubules from the sodium-retaining action of aldosterone. 2252 B. >8 µg/dL above baseline B. renin activity is gauged by the generation of angiotensin I during a standardized incubation period. Adrenal tumor D. Sodium restriction B.70% is of adrenal origin. potassium intakes & posture.25 mg) of cosyntropin IV or IM & measurement of plasma cortisol levels before administration. Dietary sodium intake B. ACTH-producing pituitary tumor Plasma level of aldosterone. Pituitary ACTH-secreting tumor 472 Which compound of DHEA is a useful index of adrenal androgen D. androstenedione. sodium restriction or by assumption of upright posture. Exogenous ACTH tumor C. B. DHEA & androstenedione are weak androgens & exert their little DHEA sulfate is formed in gonads. High-sodium diets B. Sodium restriction C. >16 µg/dL above baseline D. >12 µg/dL above baseline C. 2253 dexamethasone suppression test. with age. Urine 17-KS is highest in young adults & decline posture & sodium restriction elevate angiotensin II levels. In normal women. High-potassium diets D. Sulfate Harrison’s 18th Ed. effects via conversion to potent androgen testosterone in extraglandular tissues. Posture D. Chloride B. < 5 µg/dL B. 470 The plasma level of aldosterone is increased by ? 476 Normally. Dietary potassium loading cortisol in normal individuals should be <140 nmol/L (5 µg/dL). Measurement of plasma cortisol levels at 8 AM following oral administration of 1 mg dexamethasone the previous midnight is done. Infants A. 2253 Harrison’s 17th Ed. as Testosterone is a gonadal steroid. Angiotensin III Rapid ACTH stimulation test can be performed at any time of the day. 475 Criterion for a normal response in “Rapid ACTH stimulation test” 469 PRA and active renin correlate less well on ? is a cortisol level of ? Harrison’s 17th Ed. 561 Endocrinology MCQ’s FOR MCQ’s FOR MEDICAL MEDICAL PROFESSIONALS PROFESSIONALS BY PROF. Young adults B. It involves administration D. Old age D. is increased by dietary potassium loading. Angiotensin I C. >18 µg/dL above baseline PRA depends on dietary sodium intake patient’s ambulatory status. PRA & active renin correlate very well on low-sodium nmol/L (>18 µg/dL). dL) above baseline. plasma cortisol level at 8 AM should be ? Harrison’s 17th Ed. 2945 D. 2253 A. and 11-hydroxyandrostenedione. All of the above D. but not of cortisol. 2253 478 Cushing’s syndrome refers to ? A. Plasma angiotensinogen acts as substrate. Upright posture 477 Cushing’s disease refers to ? Harrison’s 18th Ed. 2253 A. Exogenous ACTH tumor C. All of the above Urinary 17-KS originate in adrenal gland or gonad. 2253 Harrison’s 17th Ed. Angiotensin II D. Low-potassium diets C. Pituitary ACTH-secreting tumor . < 15 µg/dL D. 468 Peripheral plasma renin activity (PRA) is gauged by generation 474 Rapid ACTH stimulation test is performed ? of which of the followinng during a standardized incubation Harrison’s 17th Ed. Low-sodium diets A. < 20 µg/dL 471 The plasma level of cortisol is increased by ? The best screening procedure to test pituitary-adrenal suppressibility is the overnight Harrison’s 17th Ed. AJAY MATHUR Cardiology 561 The principal adrenal androgens are DHEA. 2945 B. 8 AM value for plasma A. 2253 B. < 10 µg/dL C. in screening ‘overnight dexamethasone suppression Harrison’s 17th Ed. None of the above A. 2253 A. B. All of the above secretion ? Harrison’s 17th Ed. 90% of urinary 17-KS is derived Angiotensin II levels are influenced by dietary sodium. Upright from adrenal while in men 60 . Late evening B. 473 Urine 17-ketosteroid values are highest in ? 467 Angiotensin II plasma levels are influenced by ? Harrison’s 17th Ed. All of the above of 25 units (0. Dietary potassium loading A. Early morning Harrison’s 17th Ed. Middle age C. Phosphate A. Any time of the day C. In Plasma renin activity (PRA). and the minimal stimulated normal increment of cortisol is >200 nmol/L (>7 g/ diets but less well on high-sodium diets. Afternoon A. Dietary potassium intake C. 2253 test’. Measurement of the sulfate conjugate of DHEA is a useful index of adrenal androgen secretion. 30 & 60 minutes after administration. PRA has a diurnal rhythm with Criterion for a normal rapid ACTH stimulation test response is a stimulated cortisol level of >500 peak values in morning & a nadir in afternoon. 2253 period ? A. None of the above C. Upright posture C. All of the above C. Caused by nonpituitary tumors secreting ACTH /CRH B. Cardiac myxomas Traditionally. or excessive glucocorticoid treatment. A. adrenal tumor. Children Harrison’s 18th Ed. Centripetal obesity Most cases of Ectopic ACTH syndrome are associated with primitive small cell (oat cell) bronchogenic carcinoma. CREB Typical signs & symptoms of Cushing’s syndrome may be absent or minimal with ectopic ACTH production & hypokalemic alkalosis is a prominent manifestation. C. 2945 Harrison’s 16th Ed. unilateral café-au-lait spots. Von Hippel–Lindau syndrome 490 Which of the following is typical of Cushing’s syndrome ? Mutations in a regulatory subunit of PKA (PRKAR1A) are found in primary pigmented nodular Harrison’s 17th Ed. 2945 A. Hypokalemic alkalosis A. or bronchial adenomas. medullary carcinoma of B. AJAY MATHUR Endocrinology D. and PPNAD. and precocious puberty. 2255 adrenal disease (PPNAD) as part of Carney’s complex. Vasopressin B. Adrenal macronodular hyperplasia C. 2255 483 Primary pigmented nodular adrenal disease (PPNAD) is best related to which of the following ? A. pancreas. Hirschsprung disease D. A. 2945 syndrome is caused by activating mutations in GNAS-1 (guanine nucleotide binding protein alpha stimulating activity polypeptide 1). PKA & cause bilateral adrenal hyperplasia. ovary. Absence of normal fall of plasma cortisol at midnight 482 In ACTH-independent macronodular hyperplasia (AIMAH). only basophilic ACTH-producing pituitary tumor is defined as Cushing’s disease. Carney’s complex Decreased bone mineralization is particularly pronounced in children. serotonin. All of the above C. Adrenal neoplasia The most common cause of Cushing’s syndrome is iatrogenic administration of steroids. Ectopic production of CRH results in clinical. Oat cell bronchogenic carcinoma B. 2945 . Plasma cortisol > 5 µg/dL after standard low-dose C. Urine free cortisol > 50 µg/day B. vasopressin. basophilic pituitary ACTH-secreting tumor. C. All of the above D. Luteinizing hormone A. Moon facies 484 Which of the following is a feature of Carney’s complex ? B. hyperlentiginosis. Typical s/s of Cushing’s syndrome present Harrison’s 18th Ed.562 MCQ’s FOR MEDICAL PROFESSIONALS BY PROF. 2135 A. 2946 Harrison’s 18th Ed. McCune-Albright Harrison’s 18th Ed. Sertoli’s cell tumors D. It features 480 Which of the following can cause ‘Ectopic ACTH syndrome’ ? polyostotic fibrous dysplasia. B. carcinoid tumors of thymus. A. Elderly B. Adults A. D. Carcinoid tumors of thymus. 2255 myxomas. or gastric inhibitory peptide (GIP). In ACTH-independent macronodular hyperplasia (AIMAH). biochemical. Serotonin C. Proximal myopathy syndrome ? 487 Which of the following is suggestive of Cushing’s syndrome ? Harrison’s 18th Ed. Hirsutism 481 Which of the following is a cause of ACTH-independent Cushing’s D. pancreas. C. Activation of these 489 Decreased bone mineralization due to use of steroids is receptors results in upregulation of PKA signaling with a subsequent increase in cortisol production. D. Crow-Fukase Syndrome C. These include receptors for luteinizing hormone. All of the above A. Primary pigmented nodular adrenal disease (PPNAD) B. All of the above A. Bilateral adrenal hyperplasia 485 McCune-Albright syndrome is caused by activating mutations in which of the following ? C. interleukin-1. Sertoli’s cell tumors. 2945 B. particularly pronounced in ? Harrison’s 17th Ed. Adrenal micronodular dysplasia D. StAR McCune-Albright syndrome can cause ACTH-independent Cushing’s syndrome. ovary 486 Which of the following is the commonest feature of ‘Cushing’s C. All of the above of the following receptor is ectopically expressed in the adrenal gland ? 488 Most common cause of Cushing’s syndrome is ? Harrison’s 18th Ed. Buffalo hump Harrison’s 18th Ed. which D. Hyperlentiginosis Cushing’s syndrome refers to all causes of excess cortisol like exogenous ACTH tumor. and radiologic features indistinguishable from those caused by hypersecretion of pituitary ACTH. 2945 D. GNAS-1 Ectopic ACTH syndrome is caused by nonpituitary tumors that secrete either ACTH and/or CRH B. All of the above 479 Which of the following is false about ‘Ectopic ACTH syndrome’ ? Carney’s complex is an autosomal dominant multiple neoplasia condition associated with cardiac Harrison’s 17th Ed. McCune-Albright syndrome dexamethasone suppression test D. Iatrogenic administration of steroids B. Hypertension thyroid. ACTH-independent macronodular hyperplasia (AIMAH) A. 2255 Table 336-3 D. Medullary carcinoma of thyroid syndrome’ ? Harrison’s 17th Ed. adrenal cortisol excess occurs due to ectopic expression of receptors not usually found in adrenals. 2949 Harrison’s 17th Ed. Truncal obesity B. Hypertension A. < 8 µg/dL 500 Which of the following is false about primary aldosteronism ? C. High-dose dexamethasone C. Less radiologic evidence for adrenal pathology all except ? D. 2256 hyposecretion of renin (low PRA levels) that fails to increase appropriately during volume depletion (upright posture. Size > 4 . Principal drug for treatment of ACC is mitotane (o. Diastolic hypertension <25 nmol/day (10 µg/day) or of plasma cortisol to fall to <140 nmol/L (5 µg/dL) after a standard low- dose dexamethasone suppression test (0. < 10 µg/dL Harrison’s 17th Ed. headaches. after a standard low-dose dexamethasone D. > 150 µg/day Chemical adrenalectomy can be done with ketoconazole or mitotane or aminoglutethimide or metyrapone or Mifepristone. Low unenhanced CT values (<10 HU) area (buffalo hump). None of the above Harrison’s 17th Ed. left ventricular hypertrophy disproportionate to level of BP. > 5 µg/day B. Mifepristone D. ~80% of patients with primary aldosteronism Normal ACTH levels are <14 pmol/L i. typical striae. 2256 A. < 16 µg/dL A. AJAY MATHUR Cardiology 563 C. and hypersecretion of aldosterone that does not suppress A. 563 Endocrinology MCQ’s FOR MCQ’s FOR MEDICAL MEDICAL PROFESSIONALS PROFESSIONALS BY PROF. 2260 D. Edema is characteristically absent they exhibit an “escape” phenomenon from the sodium-retaining aspects of mineralocorticoids. Rarely. Diastolic hypertension without edema A. < 60 pg/mL B. B. B.e. 493 In Cushing’s syndrome. All of the above suppression test. Harrison’s 17th Ed.5 mg every 6 hour for 48 hours). Hyposecretion of renin (low PRA levels) that do not increase B.p’-DDD). urinary cortisol to fall to ? Harrison’s 17th Ed. < 10 µg/day B. soft tissue calcifications visible on CT & high unenhanced CT attenuation values (>10 HU). A. polyuria. < 40 µg/day response to volume expansion 494 In Cushing’s syndrome. < 120 pg/mL A. They constitute ~80% of patients . Mitotane B.6 cm). 2258 Patients with bilateral hyperplasia are unlikely to have hypokalemia & have lower levels of A. < 5 µg/dL appropriately in response to volume expansion. Ketoconazole C. None of the above 495 Normal plasma ACTH level is ? Hypersecretion of aldosterone causes hypokalemia. 2260 Harrison’s 17th Ed. Broad violaceous cutaneous striae C. < 80 pg/mL 501 Which of the following is false about bilateral cortical nodular C. Hypokalemia In Cushing’s syndrome. diastolic hypertension. 2256 A. supraclavicular fat pads & mesenteric bed (truncal obesity). 2949 A. <60 pg/mL. Tumor inhomogeneity Hypercortisolism promotes deposition of adipose tissue in upper face (moon facies). myopathy & virilizing signs are more suggestive of Cushing’s syndrome. > 25 µg/day C. Easy bruising. Hypokalemia less likely 496 Features suggestive of adrenocortical carcinoma (ACC) include C. < 100 pg/mL hyperplasia ? Harrison’s 17th Ed. These are not curative but effective as long as chronically administered A 24-hour urine free cortisol level of >140 nmol/day (50 µg/day) is suggestive of Cushing’s syndrome. interscapular D. 492 What value of a 24-hour urine free cortisol is suggestive of 498 Chemical adrenalectomy can be done by ? Cushing’s syndrome ? Harrison’s 18th Ed. Osteoporosis B. Features suggestive of malignancy include large size (>4 . All of the above C. Obesity D. Ketoconazole A. definitive diagnosis is established by failure of urinary cortisol to fall to B. plasma cortisol to fall to ? Criteria for the diagnosis of primary aldosteronism are diastolic hypertension without edema. Metyrapone C. sodium depletion). Metyrapone Signs & symptoms of hypercortisolism (obesity. in selected patients. after a standard low-dose dexamethasone 499 Criteria for the diagnosis of primary aldosteronism include ? suppression test. hypertension.6 cm aldosterone & less radiologic evidence for adrenal pathology. 491 Which of the following features of Cushing’s syndrome is considered more specific ? 497 Principal drug for the treatment of adrenocortical carcinoma Harrison’s 17th Ed. All of the above D. Soft tissue calcifications visible on CT D. 2260 D. No edema administration means 2 mg every 6 hour for 48 hours or 8-mg overnight. 2255 (ACC) is ? Harrison’s 18th Ed. D. < 30 µg/day C. osteoporosis & diabetes) are nonspecific. Mitotane B. 2256 polydipsia. > 50 µg/day D. Harrison’s 17th Ed. episternal fatty tumors and mediastinal widening secondary to fat accumulation occur. Hypersecretion of aldosterone that does not suppress in D. < 20 µg/day during volume depletion C. irregular margins & tumor inhomogeneity. isomer of DDT. Chromosome 12 Harrison’s 17th Ed. 2262 A. there is an increased incidence of chronic lymphocytic thyroiditis. Cirrhosis C. whereas hypertension associated with aldosterone-producing tumors is usually improved or cured by removal of the adenoma. nephrotic syndrome and congestive heart failure.or hyperthyroidism. Eplerenone D. Left ventricular hypertrophy (LVH) C. type 1 diabetes mellitus. hypo. 2262 B.564 MCQ’s FOR MEDICAL PROFESSIONALS BY PROF. an appropriately increased production of aldosterone in response to activation of renin-angiotensin system occurs. Hypo. insufficiency appears ? Harrison’s 17th Ed. Underlying edema disorder B. Hypertension associated with idiopathic bilateral nodular hyperplasia 508 What percentage of adrenal glands are destroyed before adrenal does not usually benefit from bilateral adrenalectomy. D. pernicious anemia. 21-hydroxylase (CYP21A2) B. mainly tuberculosis but also histoplasmosis. Chromosome 18 B. Pernicious anemia A. 2263 506 Secondary aldosteronism is a feature of ? A. Secondary aldosteronism is present edematous states like cirrhosis. D. Congestive heart failure In Addison’s disease. Activation of sympathetic system B. 2260 C. 513 Mutant gene in Type I polyglandular syndrome is on ? Harrison’s 17th Ed. > 25 % aldosterone secretion ? B. which of the following also occur with 505 Secondary aldosteronism occurs in association with ? increased frequency ? Harrison’s 17th Ed. which of the following also occur with disorder. nontropical sprue & myasthenia gravis. Bartter & Gitelman syndromes C. CYP19 D. All of the above ovarian failure. All of the above Secondary aldosteronism occurs with accelerated phase of hypertension or an underlying edema 512 In Addison’s disease. Activation of parasympathetic system C. 2262 B. 511 In Addison’s disease. Spironolactone C. premature D. Stimulus for aldosterone release is arterial hypovolemia and/or hypotension. Type 1 diabetes mellitus D. RSPO1 C. 2263 502 What ratio of serum aldosterone to PRA suggests autonomy of A. AJAY MATHUR Endocrinology with primary aldosteronism. All of the above Specific adrenal antigens to which autoantibodies may be directed include 21-hydroxylase (CYP21A2). > 30 509 Adrenals is a frequent site for which of the following chronic Ratio of serum aldosterone to plasma renin activity if high (>30) strongly suggests autonomy of aldosterone secretion in all normokalemic/hypokalemic and difficult-to-control hypertensive granulomatous diseases ? patients. which must involve >90% of C. > 10 D. Stroke 514 “C” in APECED gene stands for ? D. Coccidioidomycosis B. Secondary hyperaldosteronism occurs without edema or hypertension in Bartter & Gitelman increased frequency ? syndromes. Triamterene The adrenal is a frequent site for chronic granulomatous diseases. 2262 A. alopecia. 2263 . > 75 % A. Accelerated phase of hypertension A. 2263 507 As compared to essential hypertension. > 15 Addison’s disease results from progressive destruction of adrenals. Chromosome 21 C. 2263 Harrison’s 17th Ed. 2262 B. > 20 the glands before adrenal insufficiency appears. SOX9 A. Harrison’s 17th Ed. 2263 Harrison’s 17th Ed. Albuminuria D. vitiligo. Harrison’s 17th Ed. 2263 A. Chromosome 16 A. All of the above C. Myasthenia gravis B. Chronic lymphocytic thyroiditis B. > 50 % Harrison’s 17th Ed. patients with primary aldosteronism have a higher incidence of ? A. Activation of renin-angiotensin system D. Histoplasmosis A.or hyperthyroidism Harrison’s 17th Ed. Premature ovarian failure C. All of the above D. Tuberculosis 503 Drug that is of use in primary aldosteronism is ? Harrison’s 17th Ed. In secondary aldosteronism. coccidioidomycosis& cryptococcosis. Nephrotic syndrome D. > 90 % B. All of the above Harrison’s 17th Ed. All of the above 510 Specific adrenal antigens to which autoantibodies may be directed 504 Increased production of aldosterone occurs in secondary include ? aldosteronism in response to ? Harrison’s 17th Ed. All of the above C. Darkening of areolae of nipples A. All of the above Type I polyglandular syndrome is caused by mutations in the autoimmune polyendocrinopathy candidiasis ectodermal dystrophy (APECED) gene located on chromosome 21q22. 2263 C. Angiotensin I A. 2264 515 Mutant gene in Type II polyglandular syndrome is on ? A. > 24 µg/dL C. Bluish-black patches on mucous membranes B. cortisol level at 60 minutes after cosyntropin should be > 18 µg/dL. 2264 A. cosyntropin should be ? Harrison’s 17th Ed. Ketoconazole be distinguished by measuring aldosterone levels from the same blood samples. Phenytoin If rapid ACTH stimulation test is abnormal. 524 Which of the following is given in the treatment of adrenal insufficiency ? Harrison’s 17th Ed. by measuring ? alopecia. D. 250 µg B. DHEA B. 565 Endocrinology MCQ’s FOR MCQ’s FOR MEDICAL MEDICAL PROFESSIONALS PROFESSIONALS BY PROF. One third B. 150 µg A. Harrison’s 17th Ed. AJAY MATHUR Cardiology 565 A. upon ACTH stimulation testing. Normocytic anemia B.30 mg/day. and premature gonadal failure may also be associated. two-thirds of the dose is taken in morning & remaining one-third is taken in late 520 Hematologic abnormalities seen in adrenal insufficiency is ? afternoon. > 6 µg/dL ? B. phenytoin. 2264 516 Type I polyglandular autoimmune syndrome consists of all except A. Two third D. Weakness C. Fludrocortisone A. dose of cosyntropin is ? Harrison’s 17th Ed. Moderate eosinophilia D. Salt craving Replacement therapy in adrenal insufficiency aims to correct both glucocorticoid & mineralocorticoid D. Diarrhea deficiencies by hydrocortisone and fludrocortisone respectively. Angiotensin II B. In primary adrenal D. Persistent tanning following sun exposure C. Chronic mucocutaneous candidiasis In adrenal insufficiency. Three fourth Persistent tanning following sun exposure is an early sign in Addison’s disease. plasma ACTH and associated peptides (  -LPT) are elevated. Calcinosis C. Renin 517 Which of the following may cause or potentiate adrenal insufficiency ? B. 50 µg Harrison’s 17th Ed. Pigmentation of mucous membranes D. chronic active hepatitis. 500 µg C. All of the above insufficiency. To simulate normal diurnal adrenal rhythm. 2263 A. Adrenal insufficiency D. Medications like rifampin. Harrison’s 17th Ed. primary hypothyroidism. cortisol level at 60 minutes after HLA alleles B8 and DR3.3. D. > 18 µg/dL B. 250 µg of cosyntropin is given IM or IV. Candidiasis B. Chromosome 8 In rapid ACTH stimulation test. In secondary adrenal insufficiency. then primary and secondary adrenal insufficiency can C. Carcinoma A. Chromosome 10 Type II polyglandular syndrome is associated with a mutant gene on chromosome 6 as well as with 522 In rapid ACTH stimulation test. All of the above C. ketoconazole. 2264 A. 519 Which of the following is an early sign of hyperpigmentation in Addison’s disease ? 525 What proportion of total glucocorticoid dose should be given in Harrison’s 17th Ed. 2264 518 Which of the following is the most frequent presentation in adrenal insufficiency ? A. 2264 . megestrol & opiates may cause or potentiate adrenal insufficiency. Hydrocortisone Harrison’s 17th Ed. One half C. Pernicious anemia. Chromosome 6 D. the aldosterone increment will be normal (> 5 ng/dL). Relative lymphocytosis C. Chromosome 4 C. 2263 morning ? Harrison’s 17th Ed. Dark freckles D. 2263 Table 336-7 B. DHEA may improve quality of life & bone mineral density. Rifampin D. Myasthenia gravis Combination of parathyroid & adrenal insufficiency & chronic mucocutaneous candidiasis 523 Primary & secondary adrenal insufficiency can be distinguished constitutes type I polyglandular autoimmune syndrome. In females with adrenal insufficiency and low androgen levels. Parathyroid insufficiency C. Dose of hydrocortisone for most adults is 20 . 521 In rapid ACTH stimulation test. > 12 µg/dL Harrison’s 17th Ed. Aldosterone Harrison’s 17th Ed. Caif-au-lait D. 2263 B. 10 µg/dL B. Continuous hydrocortisone infusion maintains plasma cortisol constantly at stress levels of 30 µg/dL. . <= 10 µg/dL C. 2266 insufficiency ? A. 2266 Harrison’s 17th Ed. All of the above C. 536 What value of a random cortisol level is indicative of relative 530 Acute adrenocortical insufficiency may result from ? adrenal insufficiency ? Harrison’s 17th Ed. Coagulation disorder A. levels. Pregnancy C. None of the above D. have low ACTH cortisol levels following administration of 250 µg of cosystropin as an IV bolus. > 24 µg/dL C. > 10 mg/day B. 30 µg/dL D. Low ACTH level C. during pregnancy. 2265 Harrison’s 17th Ed. 2265 A. 2266 A. mineralocorticoid administration is unnecessary at hydrocortisone doses fractions of cortisol rise in circulation & target tissues. Subnormal cortisol production during acute severe illness A.566 MCQ’s FOR MEDICAL PROFESSIONALS BY PROF. diurnal variation is abolished & unbound In Addison’s disease. 20 µg/dL C. None of the above Relative adrenal insufficiency is defined as a <9 µg/dL increment between peak & baseline Patients with secondary adrenocortical hypofunction are not hyperpigmented. Unbound fractions of cortisol rises C. 2265 relative adrenal insufficiency ? A. of >100 mg/day because of the mineralocorticoid effects of hydrocortisone at such dosages. <= 20 µg/dL 531 Acute adrenocortical insufficiency may result from ? 537 What value of a random cortisol level excludes the diagnosis of Harrison’s 17th Ed. 2265 B. Rapid withdrawal of steroids B. Ineffective replacement therapy in adrenal insufficiency C. > 100 mg/day During critical illnesses cortisol levels rise 4 . All of the above D. 2265 A. > 34 µg/dL D. have near-normal level of aldosterone secretion. 2265 A. Severe dehydration. during fever. Pseudomonas septicemia Harrison’s 17th Ed. Adrenal insufficiency on withdrawal of long term steroids Harrison’s 17th Ed. vein thrombosis. rifampin. 527 In Addison’s disease. and hyperkalemia are characteristic of severe mineralocorticoid insufficiency and favor a diagnosis of primary adrenocortical insufficiency. Meningococcal septicemia A. the dose of hydrocortisone 532 During stress. Kept the same A.6 fold. as a complication of venography. < 9 µg/dL C. Hyponatremia D. < 6 µg/dL B. those on mitotane. mineralocorticoid administration is 533 Which of the following is false in acutely ill patients ? unnecessary at hydrocortisone doses of ? Harrison’s 17th Ed. 2266 B. hyponatremia. evidence of multiple hormone deficiencies. Hyperkalemia Subnormal cortisol production during acute severe illness has been termed “functional” or “relative” D. 534 Functional or relative adrenal insufficiency is ? 528 Which of the following favors a diagnosis of primary adrenocortical Harrison’s 17th Ed. dose of hydrocortisone should be doubled. No hyperpigmentation B. phenytoin. anticoagulant therapy or a coagulation disorder. Cortisol levels rise four. > 80 mg/day D. following idiopathic adrenal cortisol level >34 g/dL usually excludes the diagnosis of relative adrenal insufficiency. <= 5 µg/dL B. congenital adrenal hyperplasia. increment of less than what between peak & baseline cortisol levels defines relative adrenal 529 Which of the following is false about secondary adrenocortical insufficiency ? insufficiency ? Harrison’s 17th Ed. rapid withdrawal of steroids from those on chronic steroid therapy. < 12 µg/dL D. AJAY MATHUR Endocrinology 526 In adrenal insufficiency. Dehydration C. <= 15 µg/dL D. Anticoagulant therapy B. < 3 µg/dL A. 2264 Harrison’s 17th Ed. > 40 mg/day C. 2266 Harrison’s 17th Ed. > 54 µg/dL Acute adrenocortical insufficiency may result from septicemia with Pseudomonas or meningococcemia (Waterhouse-Friderichsen syndrome). ketoconazole. 40 µg/dL During periods of intercurrent illness (fever). Failure to take replacement therapy in adrenal insufficiency B. 535 Following cosystropin. plasma cortisol levels should be constantly should be ? maintained at ? Harrison’s 17th Ed. bilateral A random cortisol level of <=15 µg/dL is indicative of relative adrenal insufficiency and a random adrenal hemorrhage in newborn due to birth trauma. Tripled C. Doubled B. All of the above adrenal insufficiency.to sixfold A. Near-normal aldosterone secretion D. > 44 µg/dL D. Quadrupled D. Diurnal variation is abolished B. 20-lyase (CYP17) A. 20 years B. With or without aldosterone deficiency A. > 45 nmol/L Isolated aldosterone deficiency with normal cortisol production occurs in hyporeninism. 365: 2125–36 B. > 35 nmol/L D. Hypogonadism C. 17-hydroxyprogesterone C. > 25 nmol/L C. Severe postural hypotension C. 365: 2125–36 Harrison’s 18th Ed. 365: 2125–36 A. Prednisolone C. 17-hydroxylase/17. with or D. 11-hydroxylase (CYP11B1) C. Lancet 2005. 8 A. 542 The 21-hydroxylase gene is located on chromosome ? Lancet 2005. . 2962 A. Cortisol deficiency 546 CYP17 deficiency is characterized by ? Harrison’s 17th Ed. 3-HSD2 The most common form of CAH (95% of cases) is a result of impairment of CYP21A2. Triamcinolone 21-hydroxylase gene is located on chromosome 6p21·3 within HLA histocompatibility complex.20-lyase (CYP17) C. Hypertension CAH caused by deficiency of 21-hydroxylase is characterised by cortisol deficiency. 2269 Table 336-11 B.Yupic Eskimos of Alaska (one in 280) & French island of La Réunion (one in 2100). Adolescents The highest rates of classic CAH occur in two geographically isolated populations . and androgen excess. Hypertension & hypokalemia occur because of impaired conversion of 11-deoxycorticosterone to corticosterone. a potent mineralocorticoid. and hypertension. CYP11B1 deficiency causes a “hypertensive” variant of CAH. High rates have also been D. and in severe postural hypotension. Androstenedione A. 365: 2125–36 548 Which of the following has the longest half-life ? A. CYP21A2 (CYP21B) & an inactive pseudogene CYP21A1P (CYP21A. 17-hydroxylase/17. Prednisone C. as an inherited biosynthetic defect. AJAY MATHUR Cardiology 567 538 Isolated aldosterone deficiency occurs in ? 544 What value of stimulated concentration of 17-hydroxyprogeste- Harrison’s 17th Ed. Therapy in CAH is daily administration of glucocorticoids to suppress pituitary ACTH secretion. in pretectal disease of CNS. normal < 3 nmol/L at 3 days In pheochromocytoma. 539 The most common form of CAH is due to impairment of ? 545 “Hypertensive” variant of Congenital adrenal hyperplasia (CAH) Harrison’s 17th Ed. Adults reported in Brazil (one in 7500) and the Philippines (one in 7000). although the tumors can occur in full-term infant) in a randomly timed blood sample is diagnostic of classic 21-hydroxylase from early childhood until late in life. deficiency. 40 years D. 12 D. CYP17 deficiency is characterized by hypogonadism. Brazil C. hypokalemia. 2266 rone is diagnostic of 21-hydroxylase deficiency ? A. 10 B. and suppressed plasma renin activity. following removal of aldosterone-secreting adenomas. USA C. Philippines A. It causes decreased production of cortisol & shunting of precursors into the mineralocorticoid pathway with 541 The highest rates of classic CAH occur in ? hypokalemic alkalosis. Lancet 2005. resulting in the 540 CAH caused by deficiency of 21-hydroxylase is characterised by ? accumulation of 11-deoxycorticosterone. Children D. 3-HSD2 D. in whom hydrocortisone is usually used. mean age at diagnosis is about 40 years. Infants B. 567 Endocrinology MCQ’s FOR MCQ’s FOR MEDICAL MEDICAL PROFESSIONALS PROFESSIONALS BY PROF. Androgen excess B. 2267 B. 21-hydroxylase (CYP21A2) B. Methylprednisolone D. Protracted heparin administration Lancet 2005. during A corticotropin stimulated concentration of 17-hydroxyprogesterone higher than 45 nmol/L is protracted heparin therapy. 11-hydroxylase (CYP11B1) D. There are two highly homologous 21-hydroxylase genes resulting from ancestral duplication : an active gene. All of the above without aldosterone deficiency. CYP21P). 2267 is due to deficiency of ? Harrison’s 17th Ed. 11-deoxycortisol B. 30 years C. > 15 nmol/L B. Hypokalemia D. hypertension. Pregnenolone D. All of the above C. 6 Harrison’s 17th Ed. 21-hydroxylase (CYP21A2) B. 50 years A very high concentration of 17-hydroxyprogesterone (>242 nmol/L. 2267 B. 365: 2125–36 A. Prednisone is the drug of choice except in infants. diagnostic of 21-hydroxylase deficiency. prednisone is the drug of choice in all except ? Harrison’s 17th Ed. Pheochromocytoma 543 A very high concentration of which of the following is diagnostic of classic 21-hydroxylase deficiency ? 549 In pheochromocytoma. Alaska 547 In CAH. Pretectal disease of nervous system A. 2267 A. mean age at diagnosis is about ? Lancet 2005. Chapter 343. ALl of the above D. These percentages are higher in the inherited syndromes. Tumors arising from parasympathetic nervous system B. Headache A. Paroxysmal hypertension D. positional changes. Glucagon cause of autosomal recessive Parkinson’s disease. 2963 PARK2 encodes parkin. Norepinephrine D. Catecholamine-producing tumors in head & neck A. which may secrete little or no catecholamines. SDHC. an E3 ubiquitin protein ligase. 2962 of most sporadic pheochromocytomas & paragangliomas is unknown. All of the above A. Wilson’s disease C. Hallervorden-Spatz disease D. 10% are extraadrenal & 10% are malignant. Frontotemporal dementia Harrison’s 18th Ed. Opiates ligase ? B. Mutations in parkin appear to be the major A. Parkinson’s Disease 561 Drugs that can induce paroxysmal attacks in pheochromocytoma include ? D.366:665-675 B. 3320 C. All of the above 560 Drugs that can induce paroxysmal attacks in pheochromocytoma include ? The VHL protein is a component of a ubiquitin E3 ligase. Ubiquitin E3 ligase Paroxysmal attacks in pheochromocytoma are precipitated by surgery. 2962 common symptom ? A. AJAY MATHUR Endocrinology 550 The “rule of tens” for pheochromocytomas states ? 556 In pheochromocytoma. IV Methyldopa 555 Classic triad of pheochromocytoma consists of all except ? C. Cytosolic kinase D. < 1 hour 552 Germ-line mutations in which of the following can cause inherited B. C. exercise. Profuse sweating B. Histamine Harrison’s 18th Ed. Unknown etiology C. Association with episodic or sustained hypertension. All of the above Harrison’s 18th Ed. or SDHD genes. 2963 554 Which of the following diseases is related to E3 ubiquitin protein A. Tricyclic antidepressants Harrison’s 18th Ed. metoclopramide). C. < 3 hour pheochromocytoma ? Harrison’s 18th Ed. VHL Paroxysmal attacks in pheochromocytoma generally last less than an hour. A. headaches & profuse sweating. 2962 D. urination (bladder pheochromocytomas) & medications (tricyclic antidepressants. makes pheochromocytoma a likely diagnosis. Dopamine . < 6 hour A. Palpitations B. None of the above D. 2962 C. Positional changes NF1. Epinephrine Classic triad of pheochromocytoma consists of episodes of palpitations.568 MCQ’s FOR MEDICAL PROFESSIONALS BY PROF. B. which of the following is the least common 551 Which of the following about paraganglioma is false ? symptom ? Harrison’s 18th Ed. Headache D. RET D. opiates. C. SDHB. B. Sweating C. Exercise 553 The VHL protein is a component of ? C. < 12 hour B. as well as tumors that 558 Paroxysmal attacks in pheochromocytoma generally last for ? arise from parasympathetic nervous system. ~ 10 % are bilateral Lancet 2005. NF1 559 Paroxysmal attack in pheochromocytoma is precipitated by ? D. Hypertension 562 Which of the following is secreted most by pheochromocytoma ? Harrison’s 18th Ed. 2962 Lancet 2005. VHL. B. Flushing The “rule of tens” for pheochromocytomas states that ~10% are bilateral. 2962 ~25% of patients of pheochromocytoma are inherited due to germ-line mutations in the RET. Harrison’s 18th Ed. Mitochondrial kinase pregnancy. Anxiety Paraganglioma refers to catecholamine-producing tumors in head & neck. All of the above A. 2963 C. Etiology Harrison’s 18th Ed. 2962 D. ~ 10 % are extraadrenal A. which of the following is the most Harrison’s 18th Ed. Adrenocorticotropin A. ~ 10 % are malignant B. A. All of the above B. 557 In pheochromocytoma. Palpitation B. Pregnancy Harrison’s 18th Ed. Orthostatic hypotension D. Sustained hypertension C. All of the above C.366:665-675 A. Plasma catecholamines C. 50 and 100 µg/day A. 150 and 250 µg/day C. metanephrines & chromagranin A. 131I-MIBG is also used in treatment of malignant phaeochromocytoma Fractionated metanephrines & catecholamines are the most sensitive of these. All of the above D.or 123 I- metaiodobenzylguanidine (MIBG). Urinary VMA B.or 123I-metaiodobenzylguanidine (MIBG) A. Doxazosin C. AJAY MATHUR Cardiology 569 D. 3. 2150 Lancet 2005. labetalol & sympathomimetics. Levodopa Lancet 2005. a secretory product of endocrine cells. Methyldopa A. 18 F-fluorodopamine PET D. 131 I-MIBG 568 False-positive increases in catecholamine excretion result from ? 575 Which of the following is an -adrenoceptor blocker ? Harrison’s 16th Ed. Plasma-free metanephrines C. Epinephrine is virtually never increased in extraadrenal pheochromocytomas. Chromagranin A C. 111In-somatostatin analogues. 2 mg administered for at least how many days prior to surgery ? . 18 F-dopa (or dopamine) C. emission tomography (PET).3 mg C. 2963 diagnosis of pheochromocytoma ? A. 569 Endocrinology MCQ’s FOR MCQ’s FOR MEDICAL MEDICAL PROFESSIONALS PROFESSIONALS BY PROF. 10 and 50 µg/day Lancet 2005. 573 Radioactive tracer used to localize phaeochromocytomas is ? Harrison’s 18th Ed. L-methylated metabolites C. Catecholamines A. All of the above phaeochromocytoma tumor can be localized by radioactive tracers like 131 I. Sympathomimetics B. 100 and 150 µg/day B. 1.3 mg B. D. Urinary-fractionated metanephrines D. 111 In-somatostatin analogues B. using 200-mCi doses at monthly intervals. Labetalol C. 9 mg 563 Plasma and urinary metanephrine measure which metabolite of 570 Upper limit of normal of total metanephrine excretion per day catecholamines ? is ? Harrison’s 18th Ed.3 mg 564 Medication that increase catecholamines is ? 571 Which out of the following has maximum sensitivity in the Harrison’s 18th Ed. 2963 Harrison’s 16th Ed. Plasma-free metanephrines B. E-methylated metabolites A. doxazosin & urapidil. 2150 A. All of the above C. prazosin. 5 mg Pheochromocytomas & paragangliomas synthesize & store catecholamines (norepinephrine.366:665-675 B. metanephrines & catecholamines are commonly used for initial testing. All of the above D. Homovanillic acid (HVA) B. Harrison’s 16th Ed. Metanephrines C.366:665-675 B. All of the above D. 2963 Table 343-2 A. VMA Plasma tests for pheochromocytoma include estimation of catecholamines. Prazosin B. 2963 566 Urinary test for pheochromocytoma is estimation of ? Harrison’s 18th Ed.3 mg D. 2. 7 mg & dopamine). 2150 the diagnosis of phaeochromocytomas ? A. Urinary-fractionated metanephrines Medications that increase catecholamines are levodopa. Catecholamines D. epinephrine C. phenoxybenzamine should be A. 2150 576 In pheochromocytoma. S-methylated metabolites B. MRI D. D. 0. Labetalol A. over three to six cycles. 567 Upper limit of normal for total urinary catecholamines is ? 574 Which of the following provides better diagnostic sensitivity in Harrison’s 16th Ed. CT C. Metanephrines B. Urapidil D. or 18F-dopa (or dopamine) positron- Urinary tests for VMA.366:665-675 A. Levodopa B.366:665-675 A. VMA 565 Plasma test for pheochromocytoma is estimation of ? 572 Which out of the following has maximum specificity in the Harrison’s 17th Ed. All of the above 569 Upper limit of normal of VMA excretion per day is ? -adrenoceptor blockers are phenoxybenzamine. 131 I. 2271 diagnosis of pheochromocytoma ? Lancet 2005. Plasma catecholamines D. O-methylated metabolites D. Testicular tumour B. 2271 B. 583 Sipple’s syndrome is also called ? Harrison’s 16th Ed. Located in adrenals Harrison’s 18th Ed. which also occur in the brain stem & spinal cord. Adequate alpha adrenergic blockade with phenoxybenzamine generally requires 10 . Pancreatic islet cell tumour C. most commonly A. It typically lacks B. Both types of MEN 2 are caused by mutations in RET (rearranged in transfection) that encodes a tyrosine kinase. Lungs D. pigmented café au lait macules (CALM). 2964 D. Paragangliomas with SDHB mutations 585 Multiple endocrine neoplasia type 2B is characterized by ? C.14 days. pancreatic islet cell tumors. A. endolymphatic sac tumors (ELSTs) of the inner ear. Bones C. 2964 Term malignant pheochromocytoma is restricted to tumors with distant metastases. All of the above A. Medullary thyroid carcinoma (MTC) B. 10 to 14 days NF1 gene on chromosome 17 causes von Recklinghausen’s disease. Insulin-like growth factor (IGF) I A. B.366:665-675 A. Hyperparathyroidism higher risk for malignant disease ? Lancet 2005. Pheochromocytoma C. 14 to 21 days gene & encodes a neurofibromin which modulates signal transduction through the ras GTPase pathway. Axillary freckling of skin Pheochromocytomas in MEN 2 are benign. pheochromocytoma & multiple mucosal neuromas. Lisch nodules refers to hamartomas of the iris. All of the above Harrison’s 18th Ed. Lancet 2005. located in the adrenals. Large size (5 cm) MEN 2A is characterized by MTC. Lisch nodules of iris and pseudoarthrosis of the tibia. 18 C. with a typical final dose of 20 . freckling of syndrome ? non-sun-exposed skin of axilla. Harrison’s 18th Ed. 5 to 10 days D. MEN type 1A Lancet 2005. 2965. D. and vincristine. bone or liver. suggesting a vascular pathway of spread. Liver 584 Multiple endocrine neoplasia type 2A is characterized by ? D. Harrison’s 18th Ed. Lisch nodules of iris may be symptomatic before MTC. C. Pheochromocytoma 578 Which of the following features of phaeochromocytoma have a C. Prolapse VHL is an autosomal dominant disorder that predisposes to retinal & cerebellar hemangioblastomas. cystadenomas of the epididymis & broad ligament. Multiple mucosal neuromas Harrison’s 18th Ed. 17 B. 2964 A. Medullary thyroid carcinoma (MTC) to lungs. 2964 B. Renal clear-cell carcinoma 581 Lisch nodules of iris best relates to which of the following ? B. Bilateral B. B. Other features of VHL are clear cell renal D. Pheochromocytoma 579 Averbuch’s chemotherapy protocol for treatment of malignant pheochromocytoma consists of all except ? C. Retinal hemiangioblastoma Harrison’s 18th Ed.30 mg three times per day. All of the above A. Benign 580 Classic features of neurofibromatosis include ? B. Methotrexate hyperparathyroidism.366:665-675. 2 to 5 days C. MEN type 1B A. 2965 Harrison’s 17th Ed. MTC may be symptomatic before pheochromocytoma A.366:665-675 D.570 MCQ’s FOR MEDICAL PROFESSIONALS BY PROF. Dacarbazine MEN 2B includes MTC. 2964 D. MEN type 2B C. Hamartoma D. Increased plasma/urinary dopamine and dopa Harrison’s 18th Ed. MEN type 2A B. Cyclophosphamide D. 2149 577 Malignant phaeochromocytoma can metastasize to ? A. Dysplasia carcinomas. 2964 repeated every 21 days for three to six cycles. 2272 A. 6 . All of the above A. Café au lait spots D. cyclophosphamide. Pigmentation C. Harrison’s 18th Ed. 2964 C. NF1 gene is a tumor-suppressor D. Vincristine 586 Which of the following is not a feature of pheochromocytomas in MEN 2 ? Averbuch’s chemotherapy protocol includes dacarbazine. pheochromocytoma & hyperparathyroidism. AJAY MATHUR Endocrinology Harrison’s 17th Ed. and multiple pancreatic or renal cysts. and bilateral. 12 A. 588 VHL gene encodes an E3 ubiquitin ligase that regulates expression 582 NF1 gene that causes von Recklinghausen’s disease is on which of ? chromosome ? Harrison’s 18th Ed. All of the above 587 Which of the following is not a feature of Von Hippel-Lindau Classic features of Neurofibromatosis type 1 (NF 1) or Von Recklinghausen’s Disease include multiple benign Schwann cell neurofibromas. SDHC C. Bilaterality variations that have a frequency of at least 1%. and SDHD (PGL1) predispose to three of the paraganglioma syndromes. 2148 C. PGL1 600 Which of the following may cause hypertension & increased B. Missense B. 2149 A. 2150 591 Mutations of which of the following do not predispose to A. None of the above . 2965 D. Celiac ganglia A. SDHA D. Hypoxia-inducible factor-1 (HIF-1) B. C. Sialolithiasis A. Most commonly. Urea of encephalopathy. followed by PGL4. Changes from a purine to Harrison’s 16th Ed. C. All of the above D. PGL3 Harrison’s 16th Ed. B. Solitary lesions are right sided B. elevated level of amylase is due to ? Harrison’s 16th Ed. Adrenomedullin C. SDHC 599 To assess adequacy of collected urine sample. 2965 Harrison’s 16th Ed. Inferior mesenteric ganglia C. 2149 a pyrimidine. Tissue-specific transcription factor A. FSH receptors are found only in gonads C. Neuropeptide Y Succinate dehydrogenase (SDH) is formed by four subunits (A . Creatinine 592 Which is most frequent paraganglioma syndrome ? C. AJAY MATHUR Cardiology 571 B. which induces angiogenesis. If the DNA sequence change occurs in a coding region and alters an amino acid. 2148 A. 2150 Mutations of SDHA do not predispose to paraganglioma tumors but cause Leigh disease. Multicentricity abnormal protein segment of variable length before termination of translation occurs at a stop codon (nonsense mutation). ACTH receptors are located exclusively in adrenal cortex B. Mutations of SDHB (PGL4). but not of PGL3. Potassium A. All of the above 590 Mutations of which of the following gene causes paraganglioma syndrome 1 (PGL1) ? 597 Which of the following is a hypotensive peptide ? Harrison’s 18th Ed. Point A. Superior mesenteric ganglia B. Transition C. PGL4 A. Substitutions are called transitions if 596 Features that suggest familial pheochromocytoma include ? a purine or pyrimidine is replaced by another purine or pyrimidine base. Damaged pulmonary endothelium paraganglioma tumors ? B. Tumors are not innervated D. Age of onset < 30 years D. Adrenal. Erythropoietin D. reading frame alterations result in an B. or vice versa.D). Loss of VHL is associated with increased expression of vascular endothelial growth C. 2069 A. Von Hippel-Lindau syndrome VHL gene encodes an E3 ubiquitin ligase that regulates expression of hypoxia-inducible factor- 1 (HIF-1). Posterior fossa tumors PGL1 is most frequent. extraadrenal abdominal & B. D. SDHC (PGL3). it is called a missense mutation. None of the above Patients with pheochromocytoma predominantly have missense mutations in VHL gene. which of the D. Associated pancreatitis Harrison’s 18th Ed. SDHD D. PGL2 excretion of catecholamines/catecholamine metabolites ? C. 2965 C. SDHB C. Mutations involving single nucleotides are referred to as point mutations. Subarachnoid hemorrhage thoracic pheochromocytomas are components of PGL1 & PGL4. are called transversions. Frameshift D. 2151 D. a form A. Polymorphisms are sequence A. SDHD following should also be determined ? Harrison’s 16th Ed. Small nucleotide deletions or insertions cause a shift of the codon reading frame (frameshift). 2148 Harrison’s 16th Ed. Endothelin B. Neurofibromatosis type 1 factor (VEGF). 598 In pheochromocytoma. Highly vascular C. Diencephalic or autonomic epilepsy 593 Abdominal extraadrenal pheochromocytoma are located in D. SDHA A. Basic fibroblast growth factor (bFGF) 594 Hereditary phaeochromocytomas occur in ? Harrison’s 16th Ed. All of the above B. All of the above association with ? 601 Which of the following statements is false ? Harrison’s 16th Ed. PGL3 is rare. Sodium Harrison’s 18th Ed. MEN type 2 D. Insulin & thyroid hormone receptors are widely distributed D. All of the above 589 Type of mutation in VHL gene in pheochromocytoma is ? 595 Which of the following is false about pheochromocytoma ? Harrison’s 18th Ed. 571 Endocrinology MCQ’s FOR MCQ’s FOR MEDICAL MEDICAL PROFESSIONALS PROFESSIONALS BY PROF. 2965 Harrison’s 16th Ed. SDHB B. Inhibition of acid secretion C. None of the above D. GH B. Skeletal muscle excitability D. Wasting C. Autosomal recessive B. 2228 Harrison’s 16th Ed. Carcinoid syndrome C. 2233 A. Weight loss B. All of the above D. 2228 A. Glucagon D. Watery D. Weight loss B. Alkalosis 614 ‘Pancreatic cholera’ is due to overproduction of ? 606 Which of the following is not caused by ‘VIP’ hormone ? Harrison’s 16th Ed. Serum calcium rarely elevated at birth B. Lung tumour C. Henin A. TERC A. Weakness 613 Tumor-suppressor protein encoded by MEN1 gene is ? 605 Which of the following is a part of WDHA syndrome ? Harrison’s 16th Ed. Medullary thyroid carcinoma (MTC) B. -human chorionic gonadotropin C. None of the above D. pituitary & pancreatic islet C. 2232 Harrison’s 16th Ed. Dementia C. 2234 Harrison’s 16th Ed. Hypokalemia D. ‘W’ stands for ? Harrison’s 16th Ed. Systemic mastocytosis D. Mucosal neuromas D.572 MCQ’s FOR MEDICAL PROFESSIONALS BY PROF. Pheochromocytoma C. Pancreatic endocrine tumour B. VIP A. TRIM37 D. Hyperparathyroidism is most common manifestation D. Glucagon C. TSH 608 Diseases that cause secretory large-volume diarrhea include all 616 Mutations of which of the following genes occurs in MEN2 ? except ? Harrison’s 16th Ed. Phenoxybenzamine C. Ghrelin C. Tenin C. All of the above 603 Which of the following hormones is proteolytically derived from 611 Which of the following is false about MEN1 ? larger precursor polypeptides ? Harrison’s 16th Ed. Chromogranin B B. Gastrinomas B. AJAY MATHUR Endocrinology 602 Which of the following drugs block catecholamine synthesis ? 610 GRFomas are found as ? Lancet 2005. Menin B. Gastrin B. RET 609 Nonfunctional pancreatic endocrine tumours secrete all except ? 617 Which of the following is not a feature of MEN type 2A ? Harrison’s 16th Ed.366:665-675 Harrison’s 16th Ed. Prolactin C. 2231 Harrison’s 16th Ed. Small intestinal carcinoids D. PTH B. VIP D. Hyperparathyroidism . Methyl-paratyrosine (metirosine) B. ACTH D. Vasodilatory effects 615 Which of the following hormones is most commonly produced 607 Which of the following regarding VIPoma’s is false ? by pituitary tumors in MEN1 ? Harrison’s 16th Ed. 2068 A. 2233 A. Increased urine calcium excretion A. Insulin D. SPINK5 B. Also called Wermer’s syndrome A. 2229 Harrison’s 16th Ed. 2231 Harrison’s 16th Ed. Also called Verner-Morrison syndrome A. Parathyroid hyperplasia C. All of the above 612 Which of the following is false about MEN1 ? 604 In WDHA syndrome. 2229 A. 2229 A. Doxazosin A. 2232 Harrison’s 16th Ed. Small-intestinal chloride secretion B. Chromogranin A A. 2228 A. Hyperglycemia & hypercalcemia frequent C. Neoplasia of parathyroid. 2228 A. GRFomas C. Senin D. Stool volume of < 700 mL/day rules out VIPoma B. Increased glucose production A. Congenital Rubella B. Prader-Willi syndrome. cytomegalovirus and coxsackie have been implicated in 619 Subvariant of MEN2A include ? pancreatic islet destruction. 2233 Harrison’s 18th Ed. Thiazides B. as opposed to earlier criteria such as age of onset or type of therapy. -adrenergic agonists. Klinefelter syndrome Type 2 DM is a heterogeneous group of disorders characterized by variable degrees of insulin C. Age of onset D. 622 Type 2 diabetes mellitus is characterized by all except ? diazoxide. Nicotinic acid Harrison’s 18th Ed. C. 2969 Table 344-1 C. porphyria. Pyrantel pamoate C. impaired insulin secretion. Medullary thyroid carcinoma (MTC) A. Autosomal dominant inheritance 630 Out of the following countries. 2968 vacor. which one has the highest incidence B. Down’s syndrome. D. EBV D. Huntington’s chorea. myotonic dystrophy. Turner syndrome resistance. Impairment of insulin secretion A. Type IB Harrison’s 18th Ed. 2969 Table 344-1 A. Hyperparathyroidism D. and impairment in insulin secretion. All of the above A. phenytoin. Harrison’s 16th Ed. Phenytoin 621 Type of diabetes mellitus resulting from autoimmune beta cell destruction is ? C. Diazoxide D. Pheochromocytoma B. Individuals C. All of the above B. Beta blockers Harrison’s 18th Ed. Mucosal neuromas C. all except ? Klinefelter’s syndrome. 2968 B. thyroid hormone. Insulin resistance 629 Syndromes associated with DM are all except ? B. clozapine & Harrison’s 18th Ed. All of the above 627 Drugs that can cause DM include ? Harrison’s 18th Ed. Clozapine with type 1B DM lack immunologic markers indicative of an autoimmune destructive process of beta cells. 2969 Table 344-1 D. glucocorticoids. CMV C. Diabetes Mellitus 626 Drugs that can cause DM include ? Harrison’s 18th Ed. All of the above B. Cutaneous lichen amyloidosis Harrison’s 18th Ed. 2968 Moon-Biedl syndrome. Protease inhibitors Type 1A DM results from autoimmune beta cell destruction causing insulin deficiency. Early onset hyperglycemia Harrison’s 18th Ed. All of the above Drugs that can cause DM include pentamidine. Phenytoin Chapter 344. Coxsackie Viral infections like congenital rubella. Down syndrome D. Thyroid hormone A. Pentamidine D. protease inhibitors. Pathogenic process leading to hyperglycemia C. 573 Endocrinology MCQ’s FOR MCQ’s FOR MEDICAL MEDICAL PROFESSIONALS PROFESSIONALS BY PROF. Hirschsprung disease A. -interferon. thiazides. 2969 Table 344-1 620 Most recent classification of diabetes mellitus is based on ? A. Beta-adrenergic agonists B. Anti-insulin receptor antibodies B. Scandinavia . Familial medullary thyroid carcinoma (FMTC) 625 All of the following drugs can cause DM except ? B. Cri-du-chat syndrome 623 Maturity onset diabetes of the young (MODY) is characterized by Genetic syndromes associated with diabetes are Wolfram’s syndrome. Type II A. nicotinic acid. early onset of hyperglycemia. Autosomal recessive inheritance of type 1 DM ? C. D. Type IA 628 Drugs that can cause DM include ? B. AJAY MATHUR Cardiology 573 618 Which of the following is not a feature of MEN type 2B ? 624 All of the following viruses can cause DM except ? Harrison’s 16th Ed. China Maturity onset diabetes of the young (MODY) is characterized by autosomal dominant inheritance. Turner’s syndrome. 2233 A. Friedreich’s ataxia. A. 2969 Table 344-1 “Pathogenic process leading to hyperglycemia” is the basis of classification of DM. 2969 Table 344-1 C. and increased glucose production. A. Laurence- Harrison’s 18th Ed. 2969 D. They develop insulin deficiency by unknown mechanisms & are ketosis prone. Type of therapy D. All of the above C. Alpha-interferon D. Impaired insulin secretion Harrison’s 18th Ed. 2969 Table 344-1 C. Japan B. A. 632 After 2 hours of 75-gram oral glucose load. Proinsulin D. Disulfide A. United States of America C. >45 yrs .every three years D. Impaired fasting glucose (IFG) is defined when fasting plasma glucose level are between 100 to 125 mg/dL. 125 & 180 mg/dL D. 2971 A. preproinsulin. Type 1 DM B.every year American Diabetes Association (ADA) recommends screening all individuals >45 years every 3 years. 2 hours after a 75-gram oral glucose load. Cardiovascular diseases D. 633 After 2 hours of 75-gram oral glucose load. 2970 C. Proinsulin C. 21 & 29 amino acids C. Japan and China have a lower rate. 100 & 180 mg/dL A. 2971 C. C-peptide B. C peptide B. Arthropathy Proinsulin cleavage generates C peptide and A and B chains of insulin. 2971 A. 126 & 180 mg/dL B. AJAY MATHUR Endocrinology D. 160 & 200 mg/dL D. 2971 634 Individuals with impaired fasting glucose are at increased risk of A. 80 & 125 mg/dL B. Renal failure 641 The A and B chains of insulin contain how many amino acids respectively ? 635 Individuals with impaired fasting glucose (IFG) are at increased Harrison’s 18th Ed. 2971 A. 140 & 199 mg/dL C. C peptide developing which of the following condition ? B. >50 yrs . what is meant by “fasting state” ? 637 86-amino-acid precursor polypeptide of insulin is called ? Harrison’s 18th Ed. 21 & 30 amino acids A. . Proinsulin B. >35 yrs . giving rise to proinsulin. B chain of insulin A. 631 In the ADA criteria for diagnosis of DM. dL. US. Type 2 DM C. C. 2970 Table 344-2 Harrison’s 18th Ed. 29 & 21 amino acids D. Glucagon D. Amino acid Harrison’s 18th Ed. Insulin D. Insulin is initially synthesized as a single-chain 86-amino-acid precursor polypeptide. Preinsulin C. No caloric intake for last 8 hours C. No caloric intake for last 12 hours D. They are connected by disulfide bonds. Pancreatitis D. 100 & 125 mg/dL C. 2970 Harrison’s 18th Ed. No oral intake since last 8 hours A. Calcium B. 2970 A. D. 640 Cleavage of proinsulin generates which of the following ? Harrison’s 18th Ed. Neuropathy D. Insulin C. A chain of insulin Harrison’s 18th Ed. 2970 A. impaired fasting 639 Which of the following is structurally related to insulin-like growth glucose is defined when fasting plasma glucose level are between factors I and II ? Harrison’s 18th Ed.every three years Scandinavia has the highest incidence of type 1 DM. impaired glucose 638 Proteolytic processing of “preproinsulin” results in the formation tolerance is defined when plasma glucose levels are between ? of ? Harrison’s 18th Ed. No oral intake for last 12 hours B.every three years A & B chains of insulin contain 21 & 30 amino acids respectively. 30 & 21 amino acids B. All of the above B. Hydrogen 636 American Diabetes Association (ADA) recommends screening for DM in which of the following ? B.574 MCQ’s FOR MEDICAL PROFESSIONALS BY PROF. >40 yrs . 125 & 200 mg/dL Proinsulin is structurally related to insulin-like growth factors I and II. Preproinsulin A. Harrison’s 18th Ed. Hypothyroidism 642 The A and B chains of insulin are connected by which of the Individuals with IFG or IGT are at substantial risk for developing type 2 DM & cardiovascular following bond ? disease. 2971 ? Harrison’s 18th Ed. All of the above Impaired glucose tolerance (IGT) is defined as plasma glucose levels between 140 and 199 mg/ Proteolytic processing of “preproinsulin” removes aminoterminal signal peptide. Preproinsulin Fasting state is defined as no “caloric” intake for at least 8 hours. 2971 risk of developing ? Harrison’s 18th Ed. 2972 Glucose levels > 70 mg/dL stimulate insulin synthesis by enhancing protein translation & processing. SHP-2. 2971 C. Brain D. Metabolism of glucose-6-phosphate leads to inhibition of K+ channel inducing beta cell membrane depolarization which opens voltage-dependent calcium channels and causes insulin secretion. GLUT 4 A. Liver B. IRS proteins 648 Beta cell membrane depolarization that stimulates insulin secretion B. Immunologic factors B. 30 mg% Once insulin is secreted into the portal venous system. 40 mg% C. About 50% of secreted insulin is degraded by liver C. 653 Which of the following is false about insulin secretion ? 647 Which of the following acts as a rate-limiting step that controls Harrison’s 18th Ed. 2971 Figure 344-4 645 Frederick G. who discovered insulin was a ? A. A. GLUT 1 B. GLUT 4 Major portion of postprandial glucose is utilized by skeletal muscle through insulin-stimulation. D. 50 mg% 650 Which of the following glucose transporters is crucial for glucose D.48:2270-2288 B. C. PI-3-kinase A. 70 mg% uptake by skeletal muscle and fat ? Harrison’s 18th Ed. Genetic factors Harrison’s 18th Ed. Stimulation of ATP-sensitive Ca++ channel actions of insulin [GrB-2. Meals induce large bursts of insulin secretion B. Biochemist D. GLUT 2 644 Word insulin is derived from a Latin word “insula” meaning ? C. Stimulation of ATP-sensitive K+ channel D. Pyruvate B. 25 % . p110 & phosphatidylinositol-3’-kinase (PI-3-kinase)]. Environmental factors A. 2971 D. 2971 B. None of the above D. which promotes translocation of intracellular vesicles containing GLUT4 glucose transporter to the plasma membrane. 655 Destruction of pancreatic beta cells in Type 1A DM develops as a result of which of the following ? 649 What proportion of insulin in portal system is degraded by the Harrison’s 18th Ed. All of the above 654 All of the following are involved in insulin signal transduction in Glucose phosphorylation by “glucokinase” is the rate-limiting step that controls glucose-regulated skeletal muscles except ? insulin secretion. Protected Glucose transporter . Sulphar receptor C. Secreted in a pulsatile pattern Harrison’s 18th Ed. B. GLUT4 is crucial for glucose uptake by skeletal muscle & fat. D. 75 % A. Inhibition of ATP-sensitive K+ channel Insulin receptor has intrinsic tyrosine kinase activity & interacts with insulin receptor substrates (IRS & Shc) proteins. GLUT 3 Clinical Chemistry 2002. 2972 liver ? A. 2971 A. Banting. GLUT 2 C. 575 Endocrinology MCQ’s FOR MCQ’s FOR MEDICAL MEDICAL PROFESSIONALS PROFESSIONALS BY PROF. GLUT 3 D. Sucrose receptor B. Sulfonylurea receptor A.GLUT4 is translocated to cell surface following activation of PI-3-kinase pathway B. 2971 glucose-regulated insulin secretion ? A. 2972 Figure 344-5 A. Skeletal muscle A. Sugar receptor Clinical Chemistry 2002. GLUT 1 B. 2972 Harrison’s 18th Ed. Shc proteins by opening calcium channels is done by ? Harrison’s 18th Ed. 10 % C. ~50% is degraded by the liver. Glucose is transported into the beta cell by the GLUT2 glucose transporter. Physiologist C. Island D. AJAY MATHUR Cardiology 575 643 Insulin synthesis is stimulated by glucose levels above ? C. Inhibition of ATP-sensitive Ca++ channel Insulin increases glucose transport through PI-3-kinase & Cbl pathway. 2971 A. Glucokinase D. SOS. Adipose tissue C. Harrison’s 18th Ed. Physician 652 Major portion of postprandial glucose is utilized by ? 646 Glucose is transported into the pancreatic  cells via ? Harrison’s 18th Ed. p65. A number of “docking” proteins bind to these cellular proteins & initiate metabolic C. High land 651 “SUR” stands for ? Harrison’s 18th Ed. PAX-3 B. Orthopedic surgeon SUR stands for sulfonylurea receptor & is the binding site for drugs that act as insulin secretagogues. Glucose-6-phosphate C. 50 % Harrison’s 18th Ed. Projected which is stimulated by insulin.48:2270-2288 D. 2973 transporter (ZnT-8). Beta cells 667 Which of the following autoantigens is not found is type I DM ? Harrison’s 18th Ed. B. 100 % A. Alpha cells B. 40 % B. Inflammatory process abates Progressive impairment in insulin release due to gradual decline in beta cell mass results in type 1 diabetes when ~80% of the beta cell mass is destroyed. B. delta produces somatostatin and PP produces pancreatic Harrison’s 18th Ed. 20 and 50% C. 2973 A. Alpha cells Type 1A DM develops as a result of the synergistic effects of genetic. PP cells B. Delta cells A. 12 665 In pathogenesis of type 1 DM. Pancreatic islets are infiltrated by lymphocytes time glycemic control is achieved with modest doses of insulin or. Neutrophils B. environmental. 2973 660 Glucagon is produced by ? Harrison’s 18th Ed. None of the above A. Pancreatic 661 Somatostatin is produced by ? islet molecules targeted by the autoimmune process include insulin. the inflammatory process abates. a “honeymoon” phase may ensue during which A. islet destruction is mediated by T lymphocytes rather than islet autoantibodies. A. AJAY MATHUR Endocrinology D. Harrison’s 18th Ed. islet destruction is mediated by ? The major susceptibility gene for type 1A DM is located in the HLA region on chromosome 6. 2973 A. and B. 2973 C. Beta cell death could B. honeymoon period is ? Harrison’s 18th Ed. and a beta cell–specific zinc Harrison’s 18th Ed. 2973 After the initial clinical presentation of type 1A DM. 40 and 60% In pathogenesis of type 1 DM. All of the above diabetes results ? Islet cell type alpha produces glucagon. the islets become atrophic. glutamic acid decarboxylase (GAD). all of the following are true except ? D. 2973 D. C. Delta cells D. type 1A DM are all except ? Harrison’s 18th Ed. C. which of the C. All of the above A. IA-2 / ICA-512 process ? D. Insulin Harrison’s 18th Ed. T lymphocytes 659 Concordance of type 1A DM in identical twins ranges between ? B. after all beta cells are destroyed. 2973 D. GAD-65 D. A. Glutamic acid decarboxylase (GAD) B. Beta cells C. Increased insulin requirement after marriage After all beta cells are destroyed in type 1A DM. Eosinophils A. 2972 Figure 344-6 polypeptide. B lymphocytes Harrison’s 18th Ed. insulitis occurs (lymphocytic infiltration of pancreatic islets). 6 apoptosis & direct CD8+ T cell cytotoxicity may also contribute. interferon. 2973 . Islets become atrophic C. rarely. D. 60 % 663 In type 1A DM. Integrin C. insulin is not needed. Direct CD8+ mediated cytotoxicity A. All islet cell types are involved 658 Major susceptibility gene for type 1 DM is on chromosome ? C. 80 % following statements is false ? Harrison’s 18th Ed. 50 and 90% 666 Pancreatic islet molecules targeted in autoimmune process in The concordance of type 1A DM in identical twins ranges between 30 and 70%. All these three though functionally and embryologically similar to beta cells and express most of the same proteins as beta cells. PP cells Pancreatic islet molecules targeted by autoimmune process include insulin. Insulin requirement is nil or modest 664 In pathogenesis of type 1 DM. ICA-512/IA-2. glutamic acid decarboxylase (GAD). ZnT-8 D. D. are spared from the autoimmune process. ICA-512/IA-2 (homology with tyrosine phosphatases). GLUR 662 Which of the following islet cell type is spared from the autoimmune C. and IL-1. and immunologic markers disappear. Insulin A. B. 10 and 40% D. PP cells 656 What percentage of beta cell mass is destroyed when type 1 D. 5 be due to toxic effects of TNF. Immunologic markers disappear 657 In relation to type 1A DM. and phogrin (insulin secretory granule protein). Alpha cells B. 2972 D. 2973 C. Glycemic control achieved by OHA C. 1 In type 1 DM.576 MCQ’s FOR MEDICAL PROFESSIONALS BY PROF. Weight gain after insulin treatment Harrison’s 18th Ed. Release of TNF & IL-1 play a role Harrison’s 18th Ed. Formation of nitric oxide metabolites. Delta cells immunologic factors that ultimately destroy the pancreatic beta cells.. DQB1*0302 B. ~ 40 % D. Insulin resistance occur simultaneously with insulin secretory C. Both of the above D. Both DR3 & DR4 Adipocytes secrete nonesterified free fatty acids. which of the following is true ? except ? Harrison’s 18th Ed. 672 Which of the following gene is considered “protective” for type 678 In type 2 DM. D. DQB1*0602 is considered “protective” for type 1 diabetes. 2974 671 DQB1*0302 gene is associated with which haplotype ? A. 70 . Most individuals with type 1A DM have the HLA DR3 and/or DR4 haplotype. 2071 B. risk of developing diabetes is ? known genetic risk for type 1 diabetes mellitus ? Harrison’s 18th Ed. ~ 20 % B. DQB1*0302 & DQB1*0201 are most strongly associated with type 1 DM. None of the above C. Any of the above Haplotypes DQA1*0301. Insulin secretory defects precede insulin resistance B.50 % C. Both of the above B.30 % B. DR3 C. The concordance of type 2 DM in identical twins is between 70 and 90%.90 % Haplotypes that carry a DQB1*0302 gene are strongly associated with type 1 diabetes. whereas D. leptin. Molecular mimicry between microbial proteins & host tissues is reported in rheumatic fever. 2973 A. 2974 Harrison’s 18th Ed. 670 Presence of which haplotype in one individual confers the highest 676 If both parents have type 2 DM. Insulin resistance precedes insulin secretory defects A. Multiple sclerosis Increased hepatic glucose output predominantly accounts for increased FPG levels. Presence of both DR3 & DR4 haplotypes in one individual confers a 20-fold increased risk for type 1 diabetes 677 Adipocytes secrete which of the following ? Harrison’s 18th Ed. 30 . DQA1*0102 C. 668 Haplotypes most strongly associated with type 1 DM include all 674 In type 2 DM. the risk approaches 40%.70 % D. DQB1*0602 D. Leptin Harrison’s 18th Ed. Both DR3 & DR4 D. 2973 A. None of the above Individuals with both parents having type 2 DM. 50 . insulin resistance precedes insulin secretory defects and diabetes develops only if insulin secretion becomes inadequate. 577 Endocrinology MCQ’s FOR MCQ’s FOR MEDICAL MEDICAL PROFESSIONALS PROFESSIONALS BY PROF. DQB1*0201 D. Decreased peripheral utilization of glucose C. None of the above Haplotype DQA1*0102. TNF. All of the above decreased peripheral glucose usage results in postprandial hyperglycemia. Adiponectin B. All of the above C. increased postprandial plasma glucose is predominantly due to ? 673 Molecular mimicry between microbial proteins & host tissues has Harrison’s 18th Ed. 2974 been reported in ? A. DQB1*0602 D. DQB1*0302 defects D. DQB1*0602 is extremely rare in individuals with type 1 DM & provide protection from type 1 DM. DR4 C. rheumatoid arthritis & multiple sclerosis. Decreased peripheral utilization of glucose A. retinol-binding protein 4. Rheumatoid arthritis D. 2973 Harrison’s 18th Ed. 2974 A. Increased hepatic glucose output B. type IA-2/ICA-512. DR3 B. ~ 10 % A. DQA1*0301 B. In type 2 DM. ICAs serve as a marker of autoimmune process of type 1A DM.. 2974 A. DR4 D. DQB1*0202 A. 1 diabetes mellitus. 2973 Harrison’s 18th Ed. Type 1 diabetes mellitus C. 10 . resistin. Resistin A. 679 In type 2 DM. insulin. increased fasting plasma glucose is predominantly 1A diabetes mellitus ? due to ? Harrison’s 18th Ed. DQB1*0302 A. Increased hepatic glucose output Harrison’s 17th Ed. 669 The strongest single association with type 1 DM is with which haplotype ? 675 Concordance of type 2 DM in identical twins is between ? Harrison’s 18th Ed. DQB1*0402 B. 2973 B. DQB1*0402 C. . DQB1*0502 C. 2974 Harrison’s 18th Ed. None of the above and adiponectin). ~ 30 % C. AJAY MATHUR Cardiology 577 Islet cell autoantibodies (ICAs) are directed at pancreatic islet molecules such as GAD. A. Harrison’s 17th Ed. NSAIDs A. Acarbose B. Islet cell autoantibodies Harrison’s 18th Ed. Dietary fat HNF are expressed in liver.578 MCQ’s FOR MEDICAL PROFESSIONALS BY PROF. MODY 4 is a rare variant caused by mutations in insulin promoter factor (IPF) 1. Autosomal dominant C. Acanthosis nigricans Harrison’s 18th Ed. Ataxia telangiectasia B.  cells also secrete ? Harrison’s 18th Ed. Hyperandrogenism D. independent of the effects of obesity. Pancreatic islets A. 2975 B. Ptyalin D. Hyperglycemia MODY 2 is due to mutations in the glucokinase gene. promote glucose production by liver and impair beta cell function. defect. B. 2975 A. 2976 D. It 689 All are risk factors for type 2 DM except ? is a major component of amyloid fibrils found in islets of patients with type 2 diabetes. Amylin Diabetes Prevention Program (DPP) demonstrated that metformin prevented or delayed diabetes by 31% compared to placebo. D. Receptor defects B. Impair beta cell function D. 2975 C. All of the above Postreceptor defects play the predominant role in insulin resistance. Harrison’s 18th Ed. Chronic hyperglycemia C. Pravastatin B. 2975 is being used in treating both type 1 and type 2 DM. Hypoglycemia all are transmitted as autosomal dominant disorders. All of the above C. 2282 ? A. 2158. 687 All are proved to delay or prevent DM except ? 681 Elevated levels of free fatty acids can cause which of the following Harrison’s 16th Ed. 2976 . Kidney B. Chronic hyperglycemia impairing islet function C. All of the above D. Therefore. Hypertension 683 “Glucose toxicity” refers to ? B. 2976 685 Which of the following can worsen islet function ? A. All of the above A. Ramipril and pravastatin reduced the number of new cases of diabetes. Although insulin receptor PCOS affects premenopausal women & is characterized by chronic anovulation. Promote glucose production by liver C. thiazolidinediones. MODY 1 is due to mutation in glucokinase gene A. Impair glucose utilization in skeletal muscle B. but are not approved for this purpose. 2282 Elevated levels of free fatty acids can impair glucose utilization in skeletal muscle. Metformin 682 Other than insulin. Monogenic Chronic hyperglycemia paradoxically impairs islet function (glucose toxicity) and leads to a worsening of hyperglycemia. C. Chronic anovulation C. hyperandrogenism levels and tyrosine kinase activity in skeletal muscle are reduced in T2DM. Lipase Pancreatic beta cells cosecrete islet amyloid polypeptide (IAPP) or amylin along with insulin. PCOD A. Ramipril C. MODY 4 is due to mutation in IPF-1 684 All can cause islet cell dysfunction except ? Harrison’s 16th Ed. Hyperlipidemia 691 Hepatocyte nuclear transcription factor (HNF) is expressed in ? D. All of the above Chronic hyperglycemia impairs islet function (“glucose toxicity”) so does elevation of free fatty 692 MODY 1 is caused by mutations in ? acid levels (“lipotoxicity”) and dietary fat. and B. Life style modification Harrison’s 18th Ed. A. pancreatic islets and kidney. Its analogue Harrison’s 18th Ed. 2975 C. AJAY MATHUR Endocrinology 680 Which of the following play predominant role in insulin 686 Which of the following is a feature of Polycystic ovary syndrome resistance ? (PCOS) ? Harrison’s 18th Ed.` Postreceptor defects C. Seizures in hyperglycemia 690 All of the following are true about MODY except ? Harrison’s 18th Ed. 2974 Harrison’s 18th Ed. 2158 D. Harrison’s 17th Ed. Acute hyperglycemia impairing islet function D. All of the above 688 All are proved to delay or prevent DM except ? Harrison’s 16th Ed. 2158. these are not a primary with insulin resistance that increases the risk for type 2 DM. patients may also have renal absorption abnormalities and renal cysts. C. Prereceptor defects A. 2974 A. Elevation of free fatty acid levels D. D. Liver Harrison’s 18th Ed. Six different variants of MODY have been identified so far. Studies in Finnish and Chinese populations noted that acarbose. and orlistat prevent or delay type 2 DM. Metformin B. Amylase metformin. Atorvastatin D. Affects premenopausal women B. All of the above dL)/18 + BUN/2. Acute pancreatitis B. 579 Endocrinology MCQ’s FOR MCQ’s FOR MEDICAL MEDICAL PROFESSIONALS PROFESSIONALS BY PROF. decreased ratio of insulin to glucagon promotes which of the following ? A. Salivary D. the elevated amylase is usually of which origin ? B. peripheral). 1. HNF-4 alpha C. glucose (mg/dL)/18 + BUN/2. [2 x (S. All of the above hydroxybutyrate. This enzyme is crucial for regulating fatty acid transport into the mitochondria. [4 x (S. and HNF-1beta.8] Harrison’s 18th Ed. Blood Urea C. [3 x (S. 703 Which of the following drug may cause false-positive reactions in the nitroprusside test used to detect urine ketones ? 697 In DKA. MODY 3. is seen most frequently in children.8] B.6 meq/L C. Cocaine A. Adults D. All of the above 704 Which of the following drug may cause false-positive reactions in 698 In DKA.2 meq/L A.II Precipitating events in DKA are inadequate insulin administration. HNF-1 alpha D. Children C. Acidemia C. elevation in serum amylase is due to ? Harrison’s 17th Ed.4 meq/L B. Acetoacetate 696 In DKA. Captopril A. 2977 693 Cerebral edema in DKA is seen most frequently in ? A. respectively. through 702 Nitroprusside reaction does not detect which of the following ? activation of enzyme carnitine palmitoyltransferase I. glucose (mg/dL)/18 + BUN/2. Cerebral infarction Harrison’s 18th Ed. Gluconeogenesis C. Na+ + K+) + P. 2977 694 In liver. 1. 2977 C. 2976 B. AJAY MATHUR Cardiology 579 A. [4 x (S.6 meq). 1. Infection D. Beta-hydroxybutyrate A. 2977 A. All of the above 699 For every 100 mg/dL rise in serum glucose. D. All of the above B. 2977 B. serum sodium is reduced by 1. Atorvastatin B. infection. Aspirin C. Na+ + K+) + P. HMG CoA reductase C. Harrison’s 18th Ed. Harrison’s 17th Ed. 2283 Harrison’s 16th Ed. Acetoacetate B.I D. Acetone Harrison’s 18th Ed. infarction (cerebral. an extremely serious complication of DKA. hyperglucagonemia alters hepatic metabolism to favor ketone body formation. cocaine & pregnancy. 2283 Harrison’s 18th Ed. Hypothyroidism C. Azithromycin B.8] A. CPT . in DKA the amylase is usually of salivary origin and thus is not diagnostic of pancreatitis. In DKA. CPT . and ketone 701 Which of the following is not a precipitating event in DKA ? body formation in liver. 2284 A. All of the above 700 Serum osmolality is calculated by which of the following Cerebral edema. Pancreatic Acetoacetate is preferentially detected by the commonly used nitroprusside ketosis detection reagent. Glycogenolysis D. Acyl CoA transferase coronary. formula ? Harrison’s 18th Ed. Nitroprusside reaction only detects acetoacetate and acetone and not beta- D. serum sodium is MODY 1. glucose (mg/dL)/18 + BUN/2. Harrison’s 18th Ed. HNF-1 beta Interference from acetoacetate may falsely elevate the serum creatinine measurement in DKA.6 mmol/L (1. Na+ + K+) + P. HNF-1alpha. Penicillamine A. Na+ + K+) + P. 2976 Table 344-5 695 Enzyme mediating increased ketone body formation in DKA is ? A. mesenteric. All of the above D. Pregnancy B.0 meq/L Harrison’s 18th Ed. D. 1897 A. Hyperglycemia B. and MODY 5 are caused by mutations in the hepatocyte nuclear transcription reduced by ? factors (HNF) 4alpha. Ketone body formation Serum osmolality is calculated by [2 x (serum sodium + serum potassium) + plasma glucose (mg/ D.8] C.8] The decreased ratio of insulin to glucagon promotes gluconeogenesis. Hypertriglyceridemia . interference from which of the following may falsely the nitroprusside test used to detect urine ketones ? elevate serum creatinine measurement? Harrison’s 17th Ed. glycogenolysis. Gastric C. D. Elderly For every 100 mg/dL rise in serum glucose. 1. glucose (mg/dL)/18 + BUN/2. 2977 B. where beta oxidation and conversion to ketone bodies occur. maintain plasma glucose in 200 to 250 mg/dL range & insulin infusion should be continued. if present. Reduced hepatic glucose release B. the decline in plasma glucose 713 In treatment of DKA. D. Pioglitazone B. 2978 in DKA ? Harrison’s 18th Ed. moderate ketonuria. 2978 Harrison’s 17th Ed. 2979 for the chloride salt. and rehydration. Rehydration C. > 3. acidosis and ketosis resolve more slowly than hyperglycemia. is secondary to starvation. 2976 Table 344-4 A. glucose should be added to 0. with insulin therapy. 2978 A. > 3. Total-body stores of sodium. > 3. which of the following resolves 714 Which of the following drugs should be withheld well before first ? intravenous contrast administration ? Harrison’s 18th Ed. Chloride Initial use of lactated Ringer’s IV solution reduces hyperchloremia that occurs with normal saline. 150 mg/dL C. potassium phosphate or acetate can be substituted Harrison’s 18th Ed. 2978 Table 344-6 D. following insulin therapy. Magnesium 706 In DKA. Potassium D. glucose should be added to 0. Normal saline A. 708 With insulin therapy in DKA. All of the above A. All of the above D. chloride. 300 mg% A. following insulin therapy. Glipizide In DKA.1 meq/L D. Any of the above 716 In HHS. 100 mg/dL B. phosphorous & magnesium are reduced in DKA. Metformin C. 2978 Harrison’s 18th Ed.45% saline infusion to related to volume expansion. Phosphate C. Acetate D. 500 mg% C. Acidosis A. 0. potassium. Reduced hepatic glucose release B.580 MCQ’s FOR MEDICAL PROFESSIONALS BY PROF. 200 mg/dL D. the blood glucose level is usually above ? 709 In DKA. Lactated Ringer’s IV solution B. 2978 A. Magnesium Hyperglycemia usually improves at a rate of 75 to 100 mg/dL per hour as a result of insulin- mediated glucose disposal. Sodium C. Insulin-mediated glucose disposal A. very large changes occur in the serum Harrison’s 18th Ed. 2978 levels of all except ? A. D. 707 In DKA. 2976 Table 344-4 B. 600 mg% D. Oliguria Harrison’s 18th Ed.45% saline Harrison’s 18th Ed. which salt of potassium should be avoided for Harrison’s 18th Ed. Hyperglycemia C. hyperglycemia improves due to ? 712 Total-body stores which of the following elements is not reduced Harrison’s 18th Ed. 2976 Table 344-4 replacement ? Harrison’s 18th Ed. Starvation 710 In DKA. is secondary to ? To reduce the amount of chloride administered. Insulin-mediated glucose disposal A. Cephalosporin B. Acarbose D.3 meq/L In DKA. All of the above C. moderate ketonuria. . Ketosis B. 715 In hyperglycemic hyperosmolar state (HHS). All of the above D. Chloride C.0 meq/L In HHS. 705 Which of the following intravenous solution is preferred for initial use in DKA to reduce hyperchloremia ? 711 During treatment of DKA. Any of the above C. AJAY MATHUR Endocrinology C. reduced hepatic glucose release. > 3.3 meq/L. A. insulin should not be administered until potassium level B. Sodium B. Infection is ? C. All of the above D.2 meq/L Captopril or penicillamine may cause false-positive nitroprusside test used to detect urine ketones. Rehydration C. 2300 A. insulin should not be administered until potassium level is >3. 250 mg/dL The decline in the plasma glucose within the first 1 to 2 hours may be more rapid and is mostly When plasma glucose reaches 250 mg/dL. 400 mg% B. if present. Calcium D.45% saline within the first 1 to 2 hours is mostly related to ? infusion when plasma glucose level is around ? Harrison’s 18th Ed. Chloride B. 5 times B. vomiting B. vomiting. lethargy. Macular edema is a microvascular chronic complication of diabetes mellitus. 581 Endocrinology MCQ’s FOR MCQ’s FOR MEDICAL MEDICAL PROFESSIONALS PROFESSIONALS BY PROF. Mortality higher in HHS than DKA C. 10 times C. The appearance of neovascularization in response to retinal hypoxia is the hallmark of proliferative 722 “AGE” stands for ? diabetic retinopathy. what is the probability A. Cotton wool spots electronegativity compared with hydrogen (which is set at 0). 50 % Increased intracellular glucose leads to the formation of advanced glycosylation end products C. In HHS. 25 times Individuals with DM are 25 times more likely to become legally blind than individuals without DM. Normal PCO2 but when increased. Microalbuminuria B. 2982 B. 2976 Table 344-4 C. Kumamoto study D. Neovascularization The redox potential is a measure (in volts) of the affinity of a substance for electrons. or coma. 725 Which of the following trial was not for the study of chronic 719 Which of the following statements is false ? complications of diabetes mellitus ? Harrison’s 18th Ed. Hydrogen A.3 724 In the Diabetes Control and Complications Trial (DCCT). 2979 Harrison’s 18th Ed. None of the above D. A. 720 Which of the following is not a nonvascular chronic complication 726 Individuals with DM are how much more likely to become legally of DM ? blind than individuals without DM ? Harrison’s 18th Ed. C. Blot hemorrhages D. 15 times D. 2981 Harrison’s 18th Ed. Glaucoma B. 75 % (AGEs) via the nonenzymatic glycosylaton of intra. Aldose reductase . Glucose between 600-1200 mg % Intracellular glucose is predominantly metabolized by phosphorylation and subsequent glycolysis. Advanced glycosylation end products A. the following complication showed maximum reduction with improved glycemic control ? 718 Which of the following symptom is absent in HHS ? Harrison’s 18th Ed. pH < 7. which of In contrast to DKA. HHS patient is usually young B. 100 % 723 Intracellular glucose is converted to sorbitol by which enzyme ? Nonproliferative retinopathy is found in almost all individuals who have had DM for >20 Harrison’s 18th Ed. K+. its D. D. HHS has a substantially higher mortality than DKA. 2979 A. 2982 C. Anti glycosylation end products Harrison’s 18th Ed. Harrison’s 18th Ed. Tuskegee Study C. D. Cataracts C. Clinical nephropathy C. Nausea. 2980 728 Individuals who have had DM for >20 years. Abdominal pain C. 2981 A. Cl- D. United Kingdom Prospective Diabetes Study (UKPDS) Typical patient of HHS is an elderly type 2 DM individual. Kussmaul respiration D. All of the above DCCT demonstrated that improvement of glycemic control reduced nonproliferative & proliferative In HHS. Oxygen Harrison’s 18th Ed. Nitrogen B. nausea. microalbuminuria (39%). Helium C. Normal Na+. Periodontal disease D. Dehydration in HHS > DKA A. 2981 Harrison’s 18th Ed. clinical nephropathy (54%) & neuropathy (60%). abdominal pain & Kussmaul respirations characteristic of DKA are notably retinopathy (47%). with a several week history of polyuria. weight Tuskegee Study (1932–1972) was a prospective study of 431 African-American men with loss & diminished oral intake that culminates in mental confusion. Phosphoenolpyruvate carboxykinase B. 25 % D. AJAY MATHUR Cardiology 581 717 All are features of hyperglycemic hyperosmolar state except ? B. 721 The redox potential is zero for ? 727 Which of the following is the hallmark of “proliferative” diabetic retinopathy ? A. absent. Retinal vascular microaneurysms C. Fructose-1. Retinopathy A. 2980 Table 344-7 A. Associated glycosylation end products B. some glucose is converted to sorbitol by the enzyme aldose reductase.6-bisphosphatase A. 2980 years. & dehydration are more pronounced than in DKA. Diabetes Control and Complications Trial (DCCT) B. Phosphofructokinase Harrison’s 18th Ed. Activated glycosylation end products of detecting nonproliferative retinopathy ? B. fluid losses seropositive latent syphilis of >3 years’ duration. Neuropathy D. Macular edema A.and extracellular proteins. acidosis and ketonemia are absent or mild. Neovascular glaucoma near normalization of glycemia. III A. 20 to 200 µg/mg creatinine D. Diabetic macular edema 736 In type 2 DM. Distal symmetric polyneuropathy 733 In a 24 hour collected urine sample. 738 Leading cause of death in diabetic individuals on dialysis is ? Harrison’s 18th Ed. Mexilitine A. 40 to 400 mg/day common in DM ? Harrison’s 18th Ed. 2179 B. 2983 B. VI B. Any of the above 739 The most common form of diabetic neuropathy is ? During the first 6 to 12 months of improved glycemic control. Calcium channel blockers C. independent of blood pressure control. 10 to 100 mg/day The most common form of diabetic neuropathy is distal symmetric polyneuropathy. preferred drug for blood pressure control is ? Harrison’s 18th Ed. Suicide C. ACE inhibitors (in types 1 and 2 DM) and ARBs (in type 2 DM) slow the progression of diabetic renal disease. A. 2982 B. Mononeuropathy Harrison’s 18th Ed. None of the above 737 In type 2 DM. 2984 734 In a spot urine collection. 40 to 400 µg/mg creatinine Microalbuminuria is defined as 30 to 299 mg/day in a 24-hour collection or 30 to 299 µg/mg 741 Which of the following antiarrhythmic drug is useful in the creatinine in a spot collection. ARBs A. 24 & 28 weeks of gestation Drug-specific benefit in diabetic nephropathy. Administration of ACE inhibitors or ARBs N Engl J Med 2004. Calcium channel blockers D. Diabetic polyradiculopathy as ? C. established diabetic retinopathy may Harrison’s 18th Ed. 2982 B. Remains the same D. Atherosclerosis established diabetic retinopathy ? Harrison’s 18th Ed. II Harrison’s 18th Ed. 30 to 299 µg/mg creatinine Involvement of the third cranial nerve is most common in DM and manifests as diplopia. 30 to 300 mg/day 740 Involvement of which of the following cranial nerves is most D. VII C. 2984 transiently worsen. Dyselectrolytemia B. 10 to 100 µg/mg creatinine C. 2983 is ? N Engl J Med 2004. Mononeuropathy multiplex A. Treatment of dyslipidemia B. Diuretics Harrison’s 16th Ed. Procainamide microalbuminuria to overt nephropathy include all except ? Harrison’s 18th Ed. Loss of capillary endothelial cells C. Cotton-wool spots Interventions effective in slowing progression from microalbuminuria to overt nephropathy include C. 2166 735 Interventions effective in slowing progression from A. Adenosine . AJAY MATHUR Endocrinology 729 The earliest clinical signs of diabetic retinopathy is ? B. Beta blockers C. Strict blood pressure control C. D. preferred drug for blood pressure control is ? 730 Lesion that occurs early in histopathology of diabetic retinopathy Harrison’s 18th Ed. microalbuminuria is defined as ? A. strict blood pressure control. has been D. 20 to 200 mg/day C.350:48-58 A. Improves C. Administration of beta blockers A. Dot intraretinal hemorrhages D. Proliferation of capillary endothelial cells D.350:48-58 C. Selective loss of pericytes from retinal capillaries B. Beta blockers B. treatment of chronic. 2982 D.582 MCQ’s FOR MEDICAL PROFESSIONALS BY PROF. D. microalbuminuria is defined B. painful diabetic neuropathy ? Harrison’s 16th Ed. ARBs B. 2983 731 Screening for gestational diabetes mellitus is recommended for pregnant women between ? A. A. Worsens Atherosclerosis is the leading cause of death in diabetic individuals on dialysis D. Infection A. 2984 732 During the first 6 to 12 months of improved glycemic control. 16 & 20 weeks of gestation D. Current recommendations advise screening for glucose intolerance between weeks 24 and 28 of pregnancy in women with high risk for GDM. administration of ACE inhibitors or ARBs and treatment of dyslipidemia. 12 & 14 weeks of gestation C. ACE inhibitors A. 32 & 36 weeks of gestation shown for ACE inhibitors in type 1 DM and ARBs in type 2 DM. B. Myositis Orthostatic hypotension due to diabetic autonomic neuropathy may respond to fludrocortisone. Gabapentin 748 Which of the following should receive first priority in treating B. 2986 C. Lower VLDLc 743 Which of the following drug is useful in orthostatic hypotension Order of priorities in treatment of hyperlipidemia in DM is to lower LDLc. Bile acid binding resins Erectile dysfunction and retrograde ejaculation are very common in DM and may be one of the earliest signs of diabetic neuropathy. due to diabetic autonomic neuropathy ? Harrison’s 18th Ed. Clonidine Harrison’s 18th Ed. clonidine. Increased Tg and reduced HDLc of chronic. All of the above 745 Drugs that improve symptoms of gastroparesis in DM include all except ? Bile acid binding resins should not be used if hypertriglyceridemia is present. C. More in men than women diabetics ? Harrison’s 18th Ed. 2985 752 Which of the following infections occur almost exclusively in A. Erectile dysfunction 750 In the treatment of hypertriglyceridemia. Increased LDLc Chronic. Metoclopramide B. B. desipramine. death rate among men and women in type 2 DM ? Harrison’s 18th Ed. lamotrigine. Gastroparesis should not be used ? Harrison’s 18th Ed. octreotide and yohimbine. central adrenergic antagonists & vasodilators are glucose. Erythromycin interacts with motilin receptor and may promote gastric emptying. emphysematous infections of gall bladder 747 Most common pattern of dyslipidemia in DM is ? & urinary tract. D. Domperidone 751 All of the following drugs are glucose-neutral except ? Harrison’s 18th Ed. Phenytoin hyperlipidemia in DM ? C. Emphysematous infections of gall bladder & urinary tract D. 2984 levels. Harrison’s 18th Ed. Rhinocerebral mucormycosis C. nortriptyline. All of the above B. C. imipramine or selective serotonin norepinephrine reuptake inhibitors. Loperamide A. 2985 A. Lower triglycerides carbamazepine. calcium channel blockers. Fibric acid derivative D. Skin rash 744 Earliest sign of diabetic neuropathy is ? Combination therapy with an HMG CoA reductase inhibitor and fibric acid derivative increases the possibility of myositis. carbamazepine and capsaicin cream. painful diabetic neuropathy ? Most common pattern of dyslipidemia is hypertriglyceridemia & reduced HDL cholesterol Harrison’s 18th Ed. 2988 B. painful diabetic neuropathy may respond to tricyclic antidepressants. C. “Malignant” or invasive otitis externa After controlling for all known cardiovascular risk factors. A. 2986 C. Rare infections like rhinocerebral mucormycosis. Raise HDLc Chronic. which of the following B. Diabetic diarrhea is treated symptomatically with loperamide and may respond to octreotide. -adrenergic blockers. 2986 B. More in women than men A. Midodrine acid derivative increases the chances of ? B. Increased Tg 742 Which of the following antiepileptic drug is useful in the treatment D. raise HDLc & lower triglycerides.  -blockers & thiazide diuretics increase insulin 746 Which of the following statements is correct for cardiovascular resistance. 583 Endocrinology MCQ’s FOR MCQ’s FOR MEDICAL MEDICAL PROFESSIONALS PROFESSIONALS BY PROF. phenytoin & carbamazepine. Dementia midodrine. phenytoin. 2985 749 Combination therapy of HMG CoA reductase inhibitor and fibric A. Erythromycin C. Carbamazepine Harrison’s 18th Ed. Harrison’s 18th Ed. Equal in men and women B. Central adrenergic antagonists bethanechol. gabapentin. Lower LDLc Chronic. Gall stones D. 2985 A. pregabalin. NSAIDS. painful diabetic neuropathy may respond to gabapentin. All of the above A. type 2 DM increases the cardiovascular D.and lipid-neutral. All of the above death rate twofold in men and fourfold in women. D. B. 2985 . ACE inhibitors. domperidone). Ibutilide A. ACE inhibitors C. Octreotide A. Anhidrosis of the lower extremities A. D. Distal sensory loss B. and “malignant” or invasive otitis externa are seen almost exclusively in diabetics. Reduced HDLc mexilitine. Beta blockers D. HMG CoA reductase inhibitors D. 2986 D. C. painful diabetic neuropathy may respond to amitriptyline. AJAY MATHUR Cardiology 583 D. Harrison’s 18th Ed. gabapentin. Calcium channel blockers Drugs that promote gastric emptying include dopamine agonists (metoclopramide. Any of the above C. 45 mg/dL D. None of the above In patients who have achieved glycemic goal. 15 mg/dL A. or lispro provides prandial insulin.15 % C. Short-acting insulin B. 115 mg/dL B. 10 . 2994 A. Aspart C. glargine) supply basal insulin. ultralente. AJAY MATHUR Endocrinology 753 A 1% rise in HbA1C translates into how much increase in mean 759 Mixing of intermediate and short-acting insulin formulations alters glucose level ? the insulin absorption profile of ? Harrison’s 17th Ed. hemolytic anemias. 135 mg/dL D. Both of the above D. . All of the above D. 2992 Harrison’s 18th Ed. pre-supper glucose is a function of morning long- acting insulin and bedtime glucose is a function of the pre-supper. 2296 Harrison’s 18th Ed. Prior evening long-acting insulin B. Most physicians use lispro. Mixing of intermediate and short-acting insulin formulations may alter the insulin absorption profile especially of the short-acting insulins. HbA1C approximates the following mean plasma glucose values: 6% = Intermediate. Ultralente C. 758 Plasma glucose values are higher than whole blood values by ? 764 Oral glucose lowering agents that is effective in type 1 DM is ? Harrison’s 16th Ed. Alpha glucosidase inhibitors Plasma glucose values are 10-15% higher than whole blood values. Globulin B. Ketone C.20 % D. Morning long-acting insulin D. Short + Intermediate B. hemoglobinopathies. Twice per year B. 2995 A. 2296 Harrison’s 18th Ed. Glargine insulin B. Pre-supper short-acting insulin Depending on the assay methodology. None of the above A 1% rise in the A1C translates into a 35 mg/dL increase in the mean glucose level.584 MCQ’s FOR MEDICAL PROFESSIONALS BY PROF. regular. 756 Which disease may interfere with the HbA1C result ? 762 Fasting glucose is primarily determined by ? Harrison’s 18th Ed. No insulin regimen reproduces the precise insulin secretory pattern of the pancreatic islet. ADA recommends 761 Which of the following insulin regimen reproduces the precise measurement of the HbA1C how frequently ? insulin secretory pattern of the pancreatic islet ? Harrison’s 18th Ed. Intermediate acting insulin C. Short + long C. 757 Which other glycated compound can be measured for determining 763 Which of the following insulins is used in continuous glycemic control ? subcutaneous insulin infusion (CSII) ? Harrison’s 18th Ed. 25 mg/dL B. Oral glucose lowering agents with the exception of alpha-glucosidase inhibitors and an amylin analogue are ineffective in type 1 DM. short-acting insulin. Lispro D. or insulin aspart in CSII. lente. Intermediate + long D. 2993 A. Four times per year D.10 % B. 2172 Table 323-9 Harrison’s 18th Ed. Biguanides C. Hemolytic anemia B. 8% = 205 mg/dL. pre-lunch glucose interfere with the HbA1C result. ADA recommends HbA1C estimation twice a year. Glargine D. Uremia C. Glulisine In standardized assays. glulisine. NPH B. Thrice per year C. 2992 Harrison’s 18th Ed. 2994 A. 7% = 170 mg/dL. whereas 135 mg/dL. 754 In standardized assays. Lispro Fructosamine assay (measuring glycated albumin) reflects glycemic status over prior 2 weeks but there are no studies to indicate whether such assays accurately predict the complications of DM. aspart. 2993 A. is a function of morning short-acting insulin. Sulfonylureas B. HbA1C of 6% approximates what level of mean plasma glucose value ? 760 Basal insulin is provided by which of the following insulins ? Harrison’s 17th Ed. 125 mg/dL C. and uremia may Fasting glucose is primarily determined by the prior evening long-acting insulin. Morning short acting insulin C. All of the above D. 2-5% A. Thiazolidinediones D. 2993 A. Albumin A. 105 mg/dL A. 5 . Hemoglobinopathies A. Once per year A. 15 . 35 mg/dL C. 2992 Harrison’s 18th Ed. 755 In patients who have achieved glycemic goal.or long-acting insulins (NPH. Pancreatic beta cells B. Gemfibrozil Dipeptidyl peptidase 4 is the enzyme responsible for the degradation of GLP-1 and GIP.351:1106-18 B. Incretin stimulation of beta cells causes them to secrete more insulin in response to the same amount of blood 767 “PPAR” stands for ? glucose. Insulin B. Snake venom B. GIP (glucose-dependent insulinotropic peptide. Dipeptidyl peptidase (DPP) 2 A. Skeletal muscle C. Insulin secretagogues C. and adipose tissue. This has widespread expression throughout GI tract. It is mostly excreted unchanged by kidneys. Insulin glulisine Highest expression of PPAR  is in the skin. 2995 A. the heart. Scorpion venom Peroxisome-proliferator-activated receptors (PPARs) are a subfamily of 48-member nuclear- receptor superfamily & regulate gene expression in response to ligand binding. Plasma insulin response to glucose taken orally is more than N Engl J Med 2004. Pancreas The incretins are hormones that work to augment glucose-stimulated insulin secretion. AJAY MATHUR Cardiology 585 765 Weight gain does not occur with which of the following drugs for 771 Expression of PPAR is lowest in which of the following tissues ? type 2 DM ? N Engl J Med 2004. Dipeptidyl peptidase (DPP) 4 C. (TZDs). also known PPAR  is a transcription factor that is activated by certain fatty acids. Brain N Engl J Med 2005. Heart C. where it has a key role in regulation of normal adipocyte differentiation & epithelium of small intestine. pancreas. Peroxisome-proliferator-activated receptor B. Exenatide is a peptide GLP-1 receptor agonist that was originally isolated from lizard venom. Lizard venom C. Dipeptidyl peptidase (DPP) 3 B. All of the above a cell-surface and circulating-peptidase enzyme and is also known as CD26 (a T-cell activating antigen). All of the above B. 585 Endocrinology MCQ’s FOR MCQ’s FOR MEDICAL MEDICAL PROFESSIONALS PROFESSIONALS BY PROF. Spider venom D. Skin 776 Which of the following is a ‘long acting’ insulin analogue ? B. Skeletal muscles D. Insulin lispro D. thymus gland. PPAR  such as the liver. Liver intravenously is more than when taken orally C. Metformin reduces fasting plasma glucose and insulin levels.351:1106-18 involves active tubular secretion via OAT-3 and so renal function should be monitored. It N Engl J Med 2004. Whereas the isoform PPAR 1 is expressed in most tissues. prostanoids & as gastric inhibitory peptide) and GLP-1 (glucagon-like peptide-1). Incretins are proliferation as well as fatty acid uptake & storage. Sitagliptin (Januvia) is the first on the market in UK and was licensed in April 2007. Adipose tissue A. Preventing degradation of incretin hormones by blocking action of DPP 4 has led to the creation of a new class of drugs known as ‘gliptins’. 774 ‘Incretin effect’ refers to ? 768  is expressed in all except ? PPAR A. Peroxisome. PPAR  2 is specific and delays stomach emptying. A. Its expression is low in tissues that express predominantly promotes modest weight loss. 766 PPAR- is found at highest levels in ? 772 Which of the following is true for “incretin” hormones ? Harrison’s 18th Ed. Adipose tissue B. as well as in the vascular wall. Increase insulin secretion B. bezafibrate. the major protease that breaks down GIP and GLP-1. N Engl J Med 2004. Fenofibrate C. and macrophages. and vascular endothelium. Glucose in small intestine stimulates incretin release. 2995 A. and skeletal muscle.351:1106-18 when administered intravenously A. Adipose tissue A. heart. Insulin aspart . 775 The major protease that breaks down GIP and GLP-1 is ? 769  agonist ? Which of the following is a PPAR A. It is a once-daily oral dose for combination with metformin or 770  is expressed in which of the following tissues ? PPAR glitazones and not yet indicated for monotherapy. Macrophages D. Peroxisome-promoter-activated receptor D.351:1106-18 773 Exenatide was originally isolated from ? A. Muscle D. Adipocytes B. C. DPP 4 is D. kidneys. Decrease insulin secretion A. Peroxisome-protein-activated receptor A. ciprofibrate & gemfibrozil act as full or partial PPAR agonists.352:174-83 C.351:1106-18 Harrison’s 18th Ed. Thiazolidinediones D. There are two main incretin hormones in humans. improves the lipid profile. Dipeptidyl peptidase (DPP) 1 N Engl J Med 2004. Peroxisome-producer-activated receptor C. Insulin detemir D. Prevents insulin degradation C. Plasma insulin response to glucose administered B. Over response to insulin D. Augments insulin degradation D. Metformin PPAR  is expressed most abundantly in adipose tissue but is also found in pancreatic beta cells. Liver C. GLP-1 inhibits glucagon secretion thiazolidinediones. carried through the circulation to their target tissue like pancreatic beta cells. Under response to insulin PPAR  is expressed in the liver. and muscle. Fibrates like fenofibrate. brain. Both hormones are secreted by endocrine L cells located in the for adipose tissue. Bezafibrate D. It is proliferator-activated receptor  (PPAR  ) is a transcription factor activated by thiazolidinediones resistant to degradation by DPP-4. 3002 A. if hyperglycemia persists. 28 & 36 Insulin secretagogues. Selenium D. screening for glucose Harrison’s 18th Ed. Hypoglycemic symptoms 781 Most crucial period of glycemic control during pregnancy is ? C. Calcium C. General anesthesia B. alpha-glucosidase inhibitors & thiazolidinediones do not directly cause hypoglycemia. 3 & 12 B. Hypoglycemia B. 2995 Harrison’s 16th Ed. Glucose at high levels is a teratogen to Harrison’s 18th Ed. All of the above D. 8 to 12 hours Harrison’s 16th Ed. B. biguanides. Pyruvate . which occur with greater frequency in fetuses of diabetic mothers. Intrapartum 787 Hepatic glycogen stores are sufficient to maintain plasma glucose levels for ? Most crucial period of glycemic control is soon after fertilization. Bulimia D. 35 C. Chromium C. Pregnancy is associated with insulin resistance Chapter 345. Around 7 months D. Soon after fertilization Whipple’s triad for diagnosis of hypoglycemia includes symptoms consistent with hypoglycemia. 12 & 24 C. Around 3 months low plasma glucose concentration and relief of symptoms after plasma glucose level is raised. All of the above Chromium potentiates action of insulin in those with impaired glucose tolerance by increasing insulin receptor mediated signaling. History of diabetes mellitus effects of insulin results in macrosomia. 24 to 48 hours A. 4 to 6 hours 782 Gestational diabetes is best treated with ? B. Thiazolidinediones C. Sulfonylurea D. Macrosomia is due to anabolic effects of insulin except ? Pregnancy is associated with marked insulin resistance. Folate D. 780 Which of the following statements is false ? Harrison’s 18th Ed. Iron B. Thiazolidinediones C. Rich food sources of chromium include yeast. All of the above D. Lipodystrophy. Insulin crosses placenta 786 For diagnosis of hypoglycemia. alpha-glucosidase inhibitors. A. C. 778 Which of the following drugs is used as monotherapy for type 2 DM ? 784 In women with high risk for GDM. 2995 intolerance should be done between which weeks of pregnancy ? Harrison’s 18th Ed. 72 to 96 hours B. The risk of fetal malformations Harrison’s 18th Ed.  glucosidase inhibitors B. Biguanides A. B. 3003 is increased 4 to 10 times in individuals with uncontrolled DM at the time of conception. thiazolidinediones. readily crosses placenta. 35 A. Metformin A. Current recommendations advise screening for glucose intolerance between weeks 24 and 28 of pregnancy in women with high risk for GDM. High glucose levels are teratogenic to fetus C. Whipple’s triad consists of all D. B. Oral glucose-lowering agents have not been approved for use during pregnancy. Low plasma glucose level Harrison’s 18th Ed. Folate supplementation reduces the incidence of fetal neural tube defects. Lipodystrophy C. 3002 A. Lactate of gestational diabetes are needed before it supplants insulin as treatment agent of choice. Metformin Hepatic glycogen stores are sufficient to maintain plasma glucose levels for 8 to 12 hours. C. Relief of symptoms after plasma glucose level is raised A. AJAY MATHUR Endocrinology 777 Which of the following drugs does not directly cause 783 Supplementation of which of the following in pregnancy with hypoglycemia ? diabetes mellitus reduces risk of fetal neural tube defects ? Harrison’s 18th Ed. exenatide & insulin are approved for monotherapy of type 2 DM. Zinc A. 3002 D. Alpha glucosidase inhibitors B. may be generalized in the genetic condition leprechaunism. 3003 developing fetus. meat & grain products. Insulin does not cross placenta. Generalized lipodystrophy is associated with severe insulin resistance and is accompanied by acanthosis nigricans and dyslipidemia. Insulin D. 24 & 28 D. 779 Which of the following elements potentiates action of insulin ? Harrison’s 17th Ed. More data on safety & efficacy of glyburide for management A. 3003 Therapy for GDM involves MNT & insulin. 3002 A.586 MCQ’s FOR MEDICAL PROFESSIONALS BY PROF. or the loss of subcutaneous fat tissue. Anabolic & growth A. 449 785 Leprechaunism is related to ? Harrison’s 18th Ed. Any of the above 788 Precursors required for gluconeogenesis include ? Harrison’s 18th Ed. Vitamin B12 Biguanides. Copper B. Cytoplasmic protein D. N Engl J Med 2004. 795 Which of the following GLUT is found in placenta ? N Engl J Med 1999. 2. Hunger C. tremor. 3005 A. Attenuated sympathetic neural response transporter ? B. Increased Epinephrine GLUT-3 is distributed in neurons and placenta. GLUT-5 D. Nuclear protein C. GLUT-4 D. 3006 794 Which of the following GLUT is a fructose transporter ? A.341:248 A. D. Muscle provides lactate. GLUT-3 epinephrine responses cause hypoglycemia unawareness. autosomal recessive Fanconi . >2 weeks of scrupulous avoidance of hypoglycemia B. 587 Endocrinology MCQ’s FOR MCQ’s FOR MEDICAL MEDICAL PROFESSIONALS PROFESSIONALS BY PROF. Acetyl CoA D. Triglycerides in adipose tissue are broken down into glycerol. GLUT-1 D. with absent counterregulatory glucagon responses. 3. Carotid body C. All of the above GLUT-5 is distributed in small intestine. reduced autonomic responses. All of the above D. GLUT-2 A.Bickel syndrome characterized by hepatic & renal glycogen accumulation. GLUT-3 C. . Insulin clearance is reduced GLUT-2 mutations cause the rare. It is characteristically a fructose transporter and has a very low affinity for glucose. GLUT-2 Periods of relative or absolute therapeutic insulin excess leading to falling glucose concentrations. Seizure D. GLUT-1 A. nephropathy & impaired utilization of glucose & galactose. Harrison’s 18th Ed. >1 week of scrupulous avoidance of hyperglycemia A. Insulin sensitivity is increased B. loss of consciousness. Decreased Insulin C. All of the above B. Free fatty acids generate acetyl CoA for gluconeogenesis. 4. GLUT-2 B.341:248 Harrison’s 18th Ed. Loss of consciousness Mutations in GLUT-1 are associated with intractable seizures resulting from a reduction in glucose Neuroglycopenic symptoms are a direct result of CNS neuronal glucose deprivation that include transport across the blood–brain barrier. D. Endogenous glucose production is reduced following alcohol ingestion. 3004 Table 345-2 B. GLUT-2 C. and paresthesia.341:248 789 In hypoglycemia. GLUT-3 D. 3005 A. Fatigue B. GLUT-3 B.Bickel syndrome is caused by mutations in which of the following glucose transporters ? 791 Hypoglycemia following alcohol ingestion is due to ? N Engl J Med 1999. GLUT-4 C. reduced C. Hepatic portal vein B. GLUT-3 C. Endogenous glucose production is reduced D. GLUT-4 Syndrome of hypoglycemia unawareness can be reversed by >2 weeks of scrupulous avoidance of hypoglycemia. and anxiety while cholinergic symptoms are sweating. seizure. Mitochondrial protein 799 Which of the following is true about syndrome of hypoglycemia Group of glucose transporters (GLUT) consists of five homologous trans-membrane proteins. kidney. GLUT-4 D. confusion. and 5 that are encoded by distinct genes. GLUT-4 800 Syndrome of hypoglycemia unawareness can be reversed by ? GLUT-4 is the main insulin-responsive glucose transporter.350:2272-9 793 Which of the following GLUT is an insulin-responsive glucose A. fatigue. 797 Fanconi . It is a high-affinity glucose transporter. Influx of exogenous glucose is reduced C. behavioral changes. 798 Falling arterial glucose concentrations are sensed by ? 792 GLUT is best described as a ? N Engl J Med 2004. brain. Transmembrane protein B.341:248 C. >1 week of scrupulous avoidance of hypoglycemia N Engl J Med 1999. hunger. Brain A. AJAY MATHUR Cardiology 587 C. Attenuated adrenomedullary response N Engl J Med 1999. Hypoglycemia-induced autonomic responses include adrenergic symptoms like palpitations. GLUT-2 B. GLUT-1 Harrison’s 18th Ed. GLUT-1 A. Increased Cortisol 796 Blood-to-brain glucose transport is facilitated by ? 790 Neuroglycopenic symptoms of hypoglycemia include all except ? N Engl J Med 2004. >2 weeks of scrupulous avoidance of hyperglycemia C.341:248 B. adipose cells and muscle. Loss of neurogenic warning symptoms A. Ethanol blocks gluconeogenesis but not glycogenolysis.350:2272-9 Harrison’s 18th Ed. which of the following is the first physiologic response to falling plasma glucose levels ? A. alanine.350:2272-9 N Engl J Med 1999. Increased Glucagon D. unawareness ? GLUT-1. located primarily in muscle cells & adipocytes. pyruvate. sperm. 30 gram B. Apart from insulin and sulfonylureas. Stimulates gluconeogenesis A. sepsis and renal failure are causes of fasting 810 In urgent treatment of hypoglycemia. Initially it causes insulin release with hypoglycemia. Fasting plasma insulin >= 6 µU/mL A. of hypoglycemia ? Harrison’s 18th Ed. Alpha-glucosidase inhibitor Diazoxide has a hyperglycemic effect due to inhibition of insulin release from pancreas. Stimulates insulin secretion B. None of the above Despite of hypoglycemia.588 MCQ’s FOR MEDICAL PROFESSIONALS BY PROF. Bulimia . High plasma C-peptide level A. 3009 805 Somogyi effect is best understood as ? A. Alimentary hypoglycemia D. Renal failure Sulfonylureas stimulate endogenous insulin and can be best detected by measuring drug levels D. Fasting plasma C-peptide >= 0. 20 gram A. 3008 803 Which of the following is a cause of fasting hypoglycemia ? A.6 ng/mL B. 3007 Harrison’s 17th Ed. All of the above C. 3007 B. post gastric surgery (alimentary hypoglycemia). Sepsis D. Diazoxide B. High plasma sulfonylurea level B. Small cell cancer of lung C. High plasma insulin level Harrison’s 18th Ed. AJAY MATHUR Endocrinology 801 Which of the following statements about postprandial (reactive) 807 Endogenous hyperinsulinism is diagnosed by ? hypoglycemia is false ? Harrison’s 18th Ed. D. Adrenocortical tumors D. Salicylates C. Postprandial (reactive) hypoglycemia occurs only after meals and is self-limited. D. Pentamidine A. 813 Bilateral nontender parotid enlargement occurs with all except ? and later predisposes to diabetes mellitus. endogenous glucose production is impaired due to hepatic hypoperfusion. Occurs only after meals B. 10 gram Harrison’s 18th Ed. 3008 802 Which of the following might help in the treatment of postprandial A. plasma insulin & C-peptide levels are high in endogenous hyperinsulinism. Metformin D. Breast cancer D. Ethacrynic acid A. what should be the hypoglycemia. Self-limited C. as well C. A. Ethanol use C. All of the above 809 Factitious hypoglycemia due to self-ingestion of sulfonylurea is Alpha-glucosidase inhibitor delays carbohydrate digestion and glucose absorption from intestine best diagnosed by ? and may be useful in reactive hypoglycemia. All of the above C. Stimulates glycogenolysis Joslin’s Diabetes mellitus 13th Ed. In sepsis. adrenocortical tumors and carcinoids can cause fasting hypoglycemia due to 811 What is the primary mechanism of action of glucagon in treatment overproduction of insulin-like growth factor (IGF) II that through insulin or IGF-I receptors. All of the above D. Harrison’s 18th Ed. Diabetes mellitus B. Hepatoma C. Causes include enzymatic defects in carbohydrate metabolism like hereditary fructose intolerance and 808 Which of the following inhibits insulin release from pancreas ? galactosemia. All of the above D. Post hypoglycemic hyperglycemia D. All of the above in plasma or urine. Salicylates and Harrison’s 17th Ed. Quinine B. 2308 A. All of the above 806 Use of which of the following drugs can cause hypoglycemia ? 812 Stiff-Person Syndrome is associated with all except ? Harrison’s 16th Ed. 138 Harrison’s 17th Ed. 2309 C. Hypertension Pentamidine is toxic to the pancreatic beta cell. Glucagon as an extrapancreatic (catecholamine-induced) effect. Post hyperglycemic hypoglycemia C. ethanol use. and cytokine induced glucose utilization is increased. 220 sulfonamides can cause hypoglycemia. reasonable initial dose of oral glucose ? Harrison’s 18th Ed. Fasting plasma glucose <= 45 mg/dL C. Furosemide (reactive) hypoglycemia ? B. D. Quinine stimulates insulin secretion. Harrison’s 18th Ed. 3007 B. 40 gram C. Thiazide Harrison’s 17th Ed. All of the above Hepatoma. 499 B. 2307 A. Nocturnal hypoglycemia Glucagon primarily acts by stimulating glycogenolysis. 3009 804 Fasting hypoglycemia occurs with which of the following ? A. Carcinoids Reasonable initial dose of oral glucose is 20 grams for urgent oral treatment of hypoglycemia. Diabetes D. Digital sclerosis D. granulomatous inflammation and obliterative endarteritis. None of the above urinary obstruction & chronic infection. following diseases ? Harrison’s 17th Ed. Hypopituitarism B. Convexity of sole 815 Which of the following drugs can cause weight loss ? C. 824 Which of the following is a cause of chronic renal disease with 817 Which of the following is false about necrobiosis lipoidica ? normal kidney size ? Harrison’s 17th Ed. Calluses over metatarsal heads D. or insulin. cirrhosis. Diabetes mellitus C. Glenohumeral joint enlargement ? C. 2181 C. levodopa and ACE inhibitors. All of the above Lesions of necrobiosis lipoidica are found primarily on the shins (90%). Chronic pancreatitis A. atrophy. 2108 Table 317-3 D. Serotonin reuptake inhibitors B. All of the above A. Diabetes mellitus C. Type 1 diabetes mellitus C. and drugs (e. telangiectasias. Indium 111 labeled immunoglobulin G Acanthosis nigricans is associated with obesity and insulin resistance. HIV-associated renal disease B. Knee. HIV/ Harrison’s 17th Ed. A. hip & ankle D. 826 Lipid abnormalities in patients with type 2 diabetes mellitus include all except ? 819 Most frequent cause of neuropathic joint disease is ? Harrison’s 17th Ed. Amyloidosis D. 2180 A. Tarsometatarsal joint 816 Acanthosis nigricans is associated with ? Harrison’s 17th Ed. Features include central yellow color. Found on thigh A. Tuberculosis B. Metformin 822 Lisfranc fracture-dislocation is related to ? Harrison’s 17th Ed. 1986 B. AJAY MATHUR Cardiology 589 B. 2181 C. Amyloidosis C. and lipoatrophic forms). Diabetes mellitus 821 Which of the following describes “Rocker foot” ? Harrison’s 17th Ed. 2181 B. Diabetes A. ACE inhibitors A. C. All of the above C. It may reflect an endocrinopathy such as acromegaly. Contractures Emphysematous pyelonephritis & cystitis almost always occur in diabetic patients. Glenohumeral joint Medications that can cause weight loss are antibiotics. Elevated plasma triglycerides A. Obliterative endarteritis B. often with D. serotonin reuptake inhibitors. Stein-Leventhal syndrome B. 589 Endocrinology MCQ’s FOR MCQ’s FOR MEDICAL MEDICAL PROFESSIONALS PROFESSIONALS BY PROF. 2424 Harrison’s 17th Ed. Hepatic cirrhosis B. Telangiectasias C. Technetium bone scan resistant diabetes mellitus (type A.g. 256 Table 41-1 D. Acromegaly neuropathic joint disease from osteomyelitis ? Harrison’s 17th Ed. 1826 818 Which of the following is false about ‘Prayer hand deformity’ ? A. type B. All of the above C. MRI C. Elbow & wrist D. Elevated dense LDL B. Erythematous border D. Cushing’s syndrome A. hip & ankle 814 Which of the following is not a cause of bilateral parotid gland B. All of the above B. 332 Harrison’s 16th Ed. Leprosy B. 1660 A. D. Cirrhosis D. Thyroxine 820 Joint most commonly affected in diabetes mellitus is ? Bilateral nontender parotid enlargement occurs with diabetes mellitus. and erythematous border. polycystic ovary syndrome. nonsteroidal anti-inflammatory drugs. Tarsal & tarsometatarsal joint A. Indium 111 labeled white blood cells D. iodide. propylthiouracil). Knee. Elbow & wrist Harrison’s 17th Ed. Elevated plasma LDL-C . metformin.. HIV Harrison’s 16th Ed. Osteophyte protrusion from top of foot Harrison’s 17th Ed. 2181 AIDS. Biopsy shows necrobiosis of 825 Emphysematous pyelonephritis is most common in which of the collagen. Patients with insulin-dependent diabetes mellitus may develop digital sclerosis & contractures (prayer hand deformity). bulimia. 325 823 Which of the following tests is not useful in differentiating A. Cushing’s syndrome. Meningomyelocele C. Insulin-like factor 3 (INSL3) 837 In males. 3010 Patients with type I diabetes mellitus generally do not have hyperlipidemia. AJAY MATHUR Endocrinology D. 2228 C. 3010 D. All of the above 838 In males sexual maturation process is dependent on ? 832 Insulin-like factor 3 (INSL3) acts via receptor termed ? Harrison’s 18th Ed. Interstitial cells of androgens from zona reticularis. Sex-related gene on the Y chromosome 828 Somatostatinoma syndrome consists of all except ? Harrison’s 16th Ed. which acts via a receptor termed Great (G protein–coupled receptor affecting testis descent). 3010 Harrison’s 18th Ed. 3010 C. During puberty.8 years of age when adrenal gland begins to produce more C. FSH D. Gonadotropin-releasing hormone (GnRH) secretion from Testicular descent through inguinal canal is controlled by Leydig cell production of insulin-like factor hypothalamus induces puberty through GPR54. metastin. Ghost D. 2311 Sertoli cells produce müllerian inhibiting substance (MIS) which regresses müllerian structures A. B. Sertoli cells markedly elevated levels of VLDL & chylomicrons. 3010 A. Diabetes mellitus D. All of the above B. Hyperchlorhydria 835 Formation of prostate & external male genitalia is induced by ? D. 2310 During embryonic development. elevated dense LDL. Hyperuricemia 834 “SRY” stands for ? C. Testosterone D. and mature into sperm. LH 830 Testicular descent through inguinal canal is controlled by ? Harrison’s 17th Ed. Gap C. D. Interstitial cells 827 Syndrome X is characterized by all except ? D. 3010 A. 12 and 14 years B. differentiate. Patients with lipodystrophy with severe insulin resistance have B. Diabetic ketoacidosis is accompanied by hypertriglyceridemia. the principal site of dehydroepiandrosterone (DHEA) production. All of the above Harrison’s 16th Ed. Leptin C. DHT also stimulates prostate & facial hair growth & initiates recession of temporal hairline. Great B. Harrison’s 18th Ed.elevated A. Dihydrotestosterone (DHT) and Male Reproductive System C. Increased HDL cholesterol A. . Leydig cells 836 Growth & differentiation of Wolffian duct structures is induced B. All of the above D. Hyperinsulinemia (fallopian tube. C. uterus & upper segment of vagina). Disorders of the Testes B. ligand. B. Elevated plasma LDL-C levels usually are not a feature of diabetes mellitus. Sex-regulating gene on the Y chromosome B. Dehydroepiandrosterone (DHEA) A. C. 10 and 12 years A. a G protein–coupled receptor binds an endogenous 3 (INSL3). Good DHEA from adrenal gland induces adrenarche. LH 829 Testosterone is produced by ? Conversion of testosterone to DHT leads to growth of external genitalia & pubic hair. Diarrhea & steatorrhea Harrison’s 18th Ed. Interstitial cells A. Sertoli cells by ? Harrison’s 18th Ed. Leptin hormone from adipose cells plays a permissive role in the onset of puberty. In the adult. Insulin-like factor 1 (INSL1) and the development of secondary sex characteristics.590 MCQ’s FOR MEDICAL PROFESSIONALS BY PROF. testosterone promotes somatic growth A. Decreased HDL-C 833 Mullerian inhibiting substance (MIS) is produced by ? Harrison’s 18th Ed. decreased HDL-C. Gall-bladder disease SRY stands for sex-related gene on the Y chromosome. 6 and 8 years 831 Insulin-like factor 3 (INSL3) is produced by ? B. testosterone and dihydrotestosterone (DHT) induce the wolffian duct and virilization of the external genitalia. Insulin-like factor 4 (INSL4) A. Sex-requiring gene on the Y chromosome B. Testosterone Chapter 346. 3010 D. 8 and 10 years Harrison’s 18th Ed. Leydig cells D. FSH D. DHT Leydig cells of the testes produce testosterone and germ cells are nurtured by Sertoli cells to divide. Sertoli cells Adrenarche usually occurs between 6 . Type II diabetes mellitus causes dyslipidemia . C. testosterone is necessary for spermatogenesis and stimulation of libido & normal sexual function. adrenarche usually occurs between age ? Harrison’s 18th Ed. Gonadotropin-releasing hormone (GnRH) B. Insulin-like factor 2 (INSL2) C. Leydig cells plasma Tg. 3010 A. 3010 C. Hypertriglyceridemia Harrison’s 18th Ed. Sex-restricting gene on the Y chromosome A. Steroid active regulatory protein A. 95% of circulating testosterone is derived from testicular production which is 3 . Inhibin B is produced by Sertoli cells B. CYP17 C. Progesterone Harrison’s 18th Ed. Pregnenolone every ? B. 6 to 10 mL five major enzymatic steps involved in testosterone synthesis. 3 to 10 mg/day B. Most circulating DHT comes from peripheral conversion of B. None of the above C. Steroid amplifying regulatory protein D. CYP17 GnRH is released in discrete pulses ~ every 2 hours.10 mg/day. 25 A. 3012 A. CYP11A1 D. 3010 C. 17-Hydroxysteroid dehydrogenase 2 spermatogenesis & production of inhibin B. CYP11A1 it has a longer half-life. 4 A.3 hours in normal men. None of the above enzymatic steps are involved ? 852 What proportion of testosterone is “unbound” ? Harrison’s 18th Ed. CYP19 842 Which of the following statements is false ? D. In males. daily production of estradiol comes from peripheral Cholesterol is a 27-carbon molecule. 3012 A. resulting in corresponding pulses of LH & FSH. FSH acts on Sertoli cell to regulate D. 3011 Harrison’s 18th Ed. 5-reductase 841 In normal men. 26 testosterone C. 17-hydroxylase & 17. LH pulses occur about every ? B. 13 to 30 mg/day D. FSH acts on Sertoli cells A. 3 . FSH is less pulsatile than LH because B. 4 hours A. 3011 Figure 346-3 840 Hypothalamic GnRH is released in discrete pulses approximately A. Steroid acute regulatory protein C. A small amount of DHT is secreted directly by testis D.3 hours C. 8 to 20 mg/day C. 10 to 12 mL D. Steroid associated regulatory protein B. 0. 5 B. which acts to selectively suppress pituitary FSH. 3013 C.20-lyase reactions are catalyzed by CYP17. 844 The number of carbon atoms in a cholesterol molecule is ? Harrison’s 18th Ed. 3011 Figure 346-3 D. 3011 A. 3 Harrison’s 18th Ed. 28 C. 3 hours 847 Testosterone is converted to dihydrotestosterone (DHT) by ? Harrison’s 18th Ed. 843 “StAR” stands for ? Harrison’s 18th Ed. 3011 B. 3011 Figure 346-3 851 Which of the following statements is false ? Harrison’s 18th Ed. growth spurt occurs at a testicular volume of ? B. 17-OH-Progesterone B. 3011 C. CYP19 LH acts on Leydig cell to stimulate testosterone synthesis. Androstenedione A. 27 B.4 hours 848 Pregnenolone is formed from cholesterol through cleavage by ? Harrison’s 18th Ed. 4 to 6 mL D. 591 Endocrinology MCQ’s FOR MCQ’s FOR MEDICAL MEDICAL PROFESSIONALS PROFESSIONALS BY PROF. conversion of testosterone & androstenedione 845 In testosterone synthesis from cholesterol. 17-Hydroxysteroid dehydrogenase 2 Harrison’s 18th Ed. 1 hours D. 4 . AJAY MATHUR Cardiology 591 839 In males.5 to 3 % . 20 to 40 mg/day StAR stands for steroid acute regulatory protein. Half . 1 . 15 to 20 mL 846 Which of the following is converted by 17  -HSD3 to testosterone ? Growth spurt occurs at a testicular volume of about 10 . 17-Hydroxylase Harrison’s 18th Ed. LH acts on Leydig cells Harrison’s 18th Ed. 3011 850 Daily testicular secretion of circulating testosterone is ? Harrison’s 18th Ed. 17-Hydroxysteroid dehydrogenase 2 A. C. C. 17-Hydroxysteroid dehydrogenase 3 B. LH pulses occur about every 1 . 3011 Figure 346-3 D.6 hours A. Harrison’s 18th Ed. 3011 849 Which of the following acts as a catalyst ? A. 2 hours C.1 hour D. how many major D.12 mL. 2 A. In men. Liver 863 Which of the following serves as stem cells capable of self- B. Insulin A. Insulin B. Inhibin B AR is structurally related to nuclear receptors for estrogen. DHT. 100 meters C. 3012 D. Estrogen A. testosterone is converted enzymatically to all except ? Harrison’s 18th Ed. Androgens Harrison’s 18th Ed. None of the above Harrison’s 18th Ed. Short arm of X chromosome D. Elongation of cytoplasm 858 Androgen receptor (AR) is structurally related to the nuclear D. 400 meters 855 Sex hormone binding globulin (SHBG) concentrations are D. 3012 861 Total length of seminiferous tubules is about ? A. After glucuronidation or sulfation. obesity. 3012 C. Shedding of an acrosome C. Testosterone .3% of circulating testosterone is not bound to sex hormone binding globulin (SHBG) & albumin. Type C spermatogonia 857 In liver. Formation of a tail receptors for ? Harrison’s 18th Ed. insulin& nephrotic syndrome. A. Nephrotic syndrome B. 3012 D. 3012 D. Aging B. 3--androstanediol A. 3012 is a network of progressively larger efferent ducts that ultimately form epididymis. Distributed in both cytoplasm & nucleus D. 300 meters D. C. Short arm of chromosome 9 853 Sex hormone binding globulin (SHBG) concentrations are D. Type D spermatogonia A. B. Some degradation occurs in prostate & skin. C. Long arm of chromosome 9 decreased by all except ? AR is encoded by a gene on the long arm of X chromosome. 3012 A. Androstenedione 864 Which of the following is false about spermiogenesis ? C. and 3  - androstanediol. Estrogen administration B. Androgens 860 Which of the following about androgen receptor is false ? B. Testosterone binds to AR with half the affinity of DHT 854 Sex hormone binding globulin (SHBG) concentrations are decreased by all except ? D.592 MCQ’s FOR MEDICAL PROFESSIONALS BY PROF. testosterone is converted into androsterone. Hyperthyroidism A. Estrogen B. chronic inflammatory illnesses & aging are associated with high SHBG concentrations. B. 5 to 8 % 859 Gene encoding androgen receptor (AR) is located on ? Harrison’s 18th Ed. elongation of cytoplasm & formation of a tail. they are excreted by the kidneys. Glucocorticoids 865 Sertoli cells produce which of the following ? C. Androsterone A pool of type A spermatogonia serve as stem cells capable of self-renewal. A. Prostate Harrison’s 18th Ed. Type B spermatogonia C. Seminiferous tubules are ~ 600 meters in length 7 form ~two-thirds of testis volume. acquisition of an acrosome. Estrogens. closed loops with both ends emptying into rete testis which Harrison’s 18th Ed. B. AJAY MATHUR Endocrinology B. 600 meters increased by all except ? Seminiferous tubules are convoluted. glucocorticoids & progesterone. 3012 Spermiogenesis is a differentiation process involving chromatin condensation. Skeletal muscle renewal ? C. 15 to 30 % B. Type D spermatogonia 856 Testosterone is metabolized predominantly in ? Harrison’s 18th Ed. 3012 B. 3012 C. 3012 D. 11 to 13 % A. Skin A. All of the above A. Harrison’s 18th Ed. Is a ligand-regulated transcription factor C. Obesity Harrison’s 18th Ed. A. hyperthyroidism. B. 3012 C. Type A spermatogonia Testosterone is metabolized predominantly in liver. etiocholanolone. Type B spermatogonia SHBG concentrations are decreased by androgens. Hyperthyroidism 862 Primary spermatocytes are derived from ? Harrison’s 18th Ed. Chromatin condensation In the liver. Nephrotic syndrome C. Type C spermatogonia D.5 . Dihydrotestosterone (DHT) Harrison’s 18th Ed. Type A spermatogonia D. 3012 Primary spermatocytes are derived from type B spermatogonia. Progesterone Harrison’s 18th Ed. Long arm of X chromosome Only 0. 40% motility.5 cm in length. AJAY MATHUR Cardiology 593 C. 50 to 180 ng/dL Harrison’s 18th Ed.5 . 3013 D. IV administration of 100 µg of GnRH defect have sperm count of < 10 x 10 6/mL & 10% motility. D. In Klinefelter syndrome testicular volume is markedly reduced D. Elevated LH indicates primary defect at testicular level C. Arm span is less than height . 40 to 65 mL D. Early morning erections The GnRH test is performed by measuring LH and FSH concentrations at baseline and at 30 & 60 B. The spermatozoa spend an additional 21 days in epididymis. None of the above Harrison’s 18th Ed. Acceptable response is a twofold increase in LH & FSH A. Testicular volume is best assessed by using a Prader orchidometer. Measure LH & FSH at 30 & 60 min Harrison’s 18th Ed. ~50 million B. which corresponds to a volume of 12 . 3012 A. FSH B. 3013 In men with a low testosterone level. Harrison’s 18th Ed. Elevated LH level indicates a primary defect at testicular level. < 10 x 10 6/mL & 30% motility level. Increased FSH indicates damage to seminiferous tubules D. Testicular length D.5 cm Normal adult testes produce >100 million sperm per day. C. 867 Normal adult testes produces how many sperms per day ? 874 Length of normal testes ranges from ? Harrison’s 18th Ed. 1. takes about how many days ? Harrison’s 18th Ed. 3013 A. Increased FSH suggests damage to the seminiferous tubules.25 x 10 6/mL & normal motility C.25 mL. < 10 x 10 6/mL & 10% motility 877 Which of the following is false about GnRH stimulation testing ? Harrison’s 18th Ed. < 10 x 10 6/mL & 40% motility (low or inappropriately normal LH) hypogonadism. 3013 A. All of the above C. 15 . Testes range from 3. 875 Normal adult testicular volume is about ? 868 Which of the following statements is false ? Harrison’s 18th Ed. Testicular area D.0 cm C. ~75 million C. 3013 872 In eunuchoid proportions. 25 to 40 mL C. C. Testicular volume C. Advanced age does not influence testicular 869 Mild male factor infertility is defined as a sperm count of ? size. Any of the above Eunuchoid proportions are defined as an arm span >2 cm greater than height & suggest that 866 Process of formation of mature spermatozoa from germ cells androgen deficiency occurred before epiphyseal fusion. Testicular density Complete differentiation process into mature sperm requires 74 days. 593 Endocrinology MCQ’s FOR MCQ’s FOR MEDICAL MEDICAL PROFESSIONALS PROFESSIONALS BY PROF.0 cm D. Measure LH & FSH at baseline 871 Which of the following is an androgen-dependent event ? C.0 to 3. 5 to 12 mL A. Low or inappropriately normal LH level suggests a defect at the hypothalamic-pituitary B. Arm span is equal to height D. 3. Frequency of masturbation or intercourse D. All of the above 878 Range of testosterone in healthy young men is ? Harrison’s 18th Ed. 3013 Harrison’s 18th Ed. an LH level can distinguish primary (high LH) versus secondary A. Arm span is greater than height Sertoli cells produce inhibin B. 3012 873 “Prader orchidometer” is used to measure ? A. < 10 x 10 6/mL & 20% motility D. 180 to 225 ng/dL A. is reduced with seminiferous tubule damage. 12 to 25 mL B. ~100 million D.5 to 5. Inhibin B levels are reduced with seminiferous tubule damage 870 Severe male factor infertility is defined as a sperm count of ? D.15 x 10 6/mL & 20 . 3013 Men with mild male factor infertility have sperm count of 15 . 3013 B. a Sertoli cell product that suppresses FSH. 44 days A. 74 days B. A minimally acceptable response is a twofold LH increase and a 50% FSH increase.15 x 10 6/mL & normal motility A. B. None of the above Testicular volume is best assessed by Prader orchidometer.10 x 106/mL & normal motility 876 Which of the following statements is false ? Harrison’s 18th Ed. which of the following is true ? A. 3012 Harrison’s 18th Ed. an inhibitor of FSH. 3. 20 . 6 moderate male factor infertility have 10 .20 x 10 6/mL & normal motility B. Varicocele is more common on left side C.5. 10 . 24 days Harrison’s 18th Ed. Frequency & intensity of sexual thoughts minutes after IV administration of 100 g of GnRH. 3013 B.0 to 4. 3013 B. 2. 94 days C.20 x 10 /mL & normal motility). Advanced age does not influence testicular size B. where they undergo further maturation & capacitation. ~25 million A. 5 . while those with a severe A. Inhibin B.5 cm B.5 to 2. Inhibin B 888 Treatment options for familial male-limited precocious puberty B. androgen receptor antagonists (flutamide) & aromatase inhibitors (anastrazole). 72 & 120 hours 887 Which of the following disorders is also called testotoxicosis ? C. Ketoconazole D. None of the above Bioavailable testosterone refers to unbound testosterone + testosterone loosely bound to albumin. Because pituitary priming has occurred. All of the above 883 Normal semen ejaculate volume is ? Harrison’s 18th Ed. 3013 (CPP) ? A. Anastrazole D. D.3 to 0. 3015 C. Androgens from the testis or the adrenal are high. is estimated to detect the presence of testes in prepubertal boys with cryptorchidism. 300 to 1000 ng/dL D. Telangiectasia 884 Puberty in boys before what age is considered precocious ? D. GnRH elicits LH & FSH responses typical of those seen in puberty or in adults. 2 to 6 mL 889 Which of the following skin lesion is characteristic of McCune- C.05 to 0. 8 to 10 mL A. Testosterone bound to albumin C. hypothyroidism.3 % 886 Which of the following is not a cause of gonadotropin-independent B. an increase in testosterone to >150 B. 0.594 MCQ’s FOR MEDICAL PROFESSIONALS BY PROF. 10 years . Unbound testosterone + testosterone bound to albumin A.5 to 3 % A. 3015 D. hCG-secreting tumors D. Flutamide Measurement of MIS. hCG -secreting tumor. It is an autosomal dominant prepubertal boys with cryptorchidism ? disorder due to activating mutations in LH receptor leading to increased testosterone production. B. AJAY MATHUR Endocrinology C. C. 3015 testosterone concentration A. A.8 % precocious puberty in boys ? Harrison’s 18th Ed. In prepubertal boys. 12 years Testosterone concentration in healthy young men ranges from 300 to 1000 ng/dL. 3014 Treatment options for familial male-limited precocious puberty include inhibitors of testosterone synthesis (ketoconazole). McCune-Albright syndrome D. Harrison’s 18th Ed. 2. 9 years melanocytes during embryonic development. Elevated gonadotropin levels D. Only 0. a Sertoli cell product. Measure testosterone levels at 0. Café au lait spots of >50% and >15% normal morphology. Total testosterone Harrison’s 18th Ed. heterosexual. Congenital adrenal hyperplasia Administer 1500 to 4000 IU of hCG intramuscularly. Testosterone A. C. Hyperthyroidism Most circulating testosterone is bound to SHBG & to albumin. 11 years D. 225 to 300 ng/dL C. 879 Bioavailable testosterone refers to ? 885 Which of the following is false about central precocious puberty Harrison’s 18th Ed. Harrison’s 18th Ed. activating LH receptor mutation A. acceptable response is a doubling of Harrison’s 18th Ed.3% of circulating testosterone is unbound or “free”. McCune-Albright syndrome 881 Which of the following about hCG stimulation test is false ? Harrison’s 18th Ed. McCune-Albright syndrome. Precocity can be isosexual or SHBG levels can affect total testosterone levels. Less common in boys than in girls D. Müllerian inhibiting substance (MIS) include ? Harrison’s 18th Ed. 3014 Table 346-1 C. 3014 A. 3013 A.5 . Administer 150 to 400 IU of hCG intramuscularly & exogenous androgens. CPP is caused by premature activation of GnRH pulse generator and is characterized by gonadotropin 880 What proportion of circulating testosterone is unbound or free ? levels that are inappropriately elevated for age. Alterations in Puberty in boys before age 9 years is considered precocious.6 mL & contains sperm counts of >20 million/mL. Isosexual precocity can be gonadotropin-dependent or gonadotropin-independent. 5 to 8 mL Albright syndrome ? Harrison’s 18th Ed. Congenital adrenal hyperplasia D. In adult men. Erythroderma The normal ejaculate volume is 2 . 48.5 to 8 % B. 0. All of the above B. Familial male-limited precocious puberty ng/dL indicates the presence of testicular tissue C. 0. Testosterone levels are elevated and LH is suppressed. 3014 Gonadotropin-independent causes of precocious puberty in boys are congenital adrenal hyperplasia. Scarring alopecia Harrison’s 18th Ed. 3014 Cafe-au-lait spots are characteristic skin lesions and reflect the onset of somatic mutations in A. 3014 B. Gonadotropin-Dependent C. Unbound testosterone B. C.24. 1 to 3 mL B. with a motility B. None of the above 882 Which of the following is measured to detect presence of testes in Familial male-limited precocious puberty is also called testotoxicosis. B. Constitutional delay B. Hypergonadotropic hypogonadism secondary to primary C. Harrison’s 18th Ed. Autosomal recessive 895 Which of the following categories of delayed puberty is more C. AJAY MATHUR Cardiology 595 890 In McCune-Albright syndrome. 3017 D. Low LH D. Autosomal dominant common in girls than in boys ? D. 3016 A. 3016 A. leading to constitutive activation of adenylyl cyclase and stimulates testosterone production leading to gonadotropin-independent precocious puberty. 3016 precocious puberty. Sertoli cell tumors in the testis Constitutional delay should be suspected when there is a family history. Testolactone 899 What testicular length generally indicates that the child has entered puberty ? Long-acting GnRH analogues can be used to suppress gonadotropins in gonadotropin-dependent Harrison’s 18th Ed. spironolactone is a weak androgen antagonist and testolactone and letrozole are aromatase inhibitor.5 cm B. Functional hypogonadotropic hypogonadism caused by A. Marijuana smoking D. Long-acting GnRH analogues Testotoxicosis or familial male-limited precocious puberty is an autosomal dominant disorder. Short stature B. Spironolactone D. Small testes gonadal failure D. 595 Endocrinology MCQ’s FOR MCQ’s FOR MEDICAL MEDICAL PROFESSIONALS PROFESSIONALS BY PROF. Autosomal dominant A. 3015 A. Mutations impair GTP activity of Harrison’s 18th Ed. 18 A. Constitutional delay of growth and puberty 901 Kallmann syndrome is due to mutation in which gene ? Harrison’s 18th Ed. Positive family history 892 Breast enlargement in prepubertal boys can result from ? Harrison’s 18th Ed. X-linked 893 In children with gonadotropin-independent precocious puberty. Harrison’s 18th Ed. But. 898 Familial hypogonadotropic hypogonadism is transmitted most producing tumors in adrenal gland. Hypogonadotropic hypogonadism due to genetic or acquired A. 16 900 Kallmann syndrome is transmitted as ? Harrison’s 18th Ed. 3015 C.5 cm generally indicates that the child has entered puberty. > 3. Gsgamma subunit hypogonadism is more common in girls than in boys. Familial aromatase excess C. which of the following is not useful ? B. evidence of delayed bone age & short stature. Autosomal recessive Harrison’s 18th Ed. 3015 C. Gsdelta subunit 896 Which of the following is the most common cause of delayed McCune-Albright syndrome is caused by somatic activating mutations in G s subunit that links puberty ? G protein-coupled receptors to intracellular signaling pathways.5 cm Harrison’s 18th Ed. > 2. mutation occurs in which of the C. Functional hypogonadotropic hypogonadism due to systemic 891 Which of the following is false about congenital adrenal hyperplasia (CAH) ? illness or malnutrition Harrison’s 18th Ed. D. X-linked Puberty is delayed in boys if it has not ensued by age 14 years. D. 3015 B. C. > 2. Gsalpha subunit gonadal failure B. > 1. Sertoli cell tumors in testis. 15 C. 3017 B. Ketoconazole D.5 cm A. Autosomal codominant B. Chronic ACTH stimulation defects in hypothalamic-pituitary region B. Autosomal codominant Harrison’s 18th Ed. KAL1 systemic illness or malnutrition . None of the above 897 Constitutional delay in puberty should be suspected when there Boys with CAH if untreated with adequate glucocorticoid suppression of ACTH can develop premature is ? virilization because of excessive androgen production by adrenal gland. A. 3016 the Gs protein. functional hypogonadotropic C. Gsbeta subunit Delayed puberty is more common in boys than in girls. B. Hypergonadotropic hypogonadism secondary to primary A. estrogen. B. or exogenous commonly as ? estrogens or androgens and germ cell tumors that secrete hCG. Hypogonadotropic hypogonadism caused by genetic or following ? acquired defects in the hypothalamic-pituitary region Harrison’s 18th Ed. All of the above Breast enlargement in prepubertal boys can result from familial aromatase excess. 3015 D. LH is low & testes are small. 14 Testicular size >2.0 cm 894 Puberty is considered delayed in boys if it has not happend by the age of ? C. 3016 D. Ketoconazole inhibits steroidogenesis. All of the above C. Delayed bone age A. A. marijuana smoking. All of the above A. Polydactyly Harrison’s 18th Ed. absence of spermatogenesis. 3018 Harrison’s 18th Ed. and neurologic abnormalities including mirror movements. C. cause severe obesity & pubertal arrest because of hypothalamic GnRH Azoospermia is the rule in men with Klinefelter syndrome (47. Mental retardation Klinefelter syndrome is the most common chromosomal disorder associated with testicular dysfunction & male infertility. Decrease in SHBG levels anosmin. mental retardation. renal defects. C. Decreased GnRH secretion 902 KAL1 gene encodes which of the following ? C. GPR54 A. Kalmin Mechanism of marijuana-induced hypogonadism is decreased GnRH secretion. Neurokinin Harrison’s 18th Ed. Androgen deficiency D. 3018 D. Pituitary adenoma 904 Prader-Willi syndrome is characterized by all except ? B. SHBG levels decrease due to inhibitory effect of increased circulating insulin resulting in lower total testosterone levels but not free testosterone levels. KAL2 908 Mechanism of marijuana-induced hypogonadism is ? Harrison’s 18th Ed. 3018 B. and paraplegia. 3017 C. 3017 A. Decreased Testosterone D. leading to undervirilization & gynecomastia. 1 in 100000 live-born males C. Higher estradiol levels D. Tall stature Harrison’s 18th Ed. It occurs in ~ 1 in 1000 live-born males. 907 Laurence-Moon syndrome is characterized by all except ? Harrison’s 18th Ed. mental retardation. a protein that mediates migration of neural progenitors of the olfactory bulb & GnRH-producing neurons. D. . Short stature GnRH secretion & mildly increasing PRL secretion. DAX1 B. Anorchia syndrome 905 Laurence-Moon syndrome is transmitted as ? B. Increased estradiol Laurence-Moon syndrome is an autosomal recessive disorder characterized by obesity. Small hands and feet C. All of the above B. pituitary is D. B. All of the above C. These individuals have GnRH deficiency and variable B. Myotonic dystrophy Harrison’s 18th Ed.XXY). Testicular histology shows deficiency. Autosomal recessive D. which encodes A. Recessive mutations of leptin. Hemochromatosis A. Lean and thin constitution Pituitary adenomas can cause hypogonadism by extension into suprasellar region & impairing C. All of the above hypogonadism. Hypotonic musculature 910 Which of the following is a cause of hypogonadism ? C. Autosomal codominant 912 The occurrence of Klinefelter syndrome is ? 906 Laurence-Moon syndrome is characterized by all except ? Harrison’s 18th Ed. None of the above 903 Prader-Willi syndrome is characterized by all except ? Harrison’s 18th Ed. diabetes insipidus. hypogonadism. Hypogonadism D. Harrison’s 18th Ed. 3017 913 Which of the following is a feature of Klinefelter syndrome ? A. 1 in 1000000 live-born males D. polydactyly & retinitis pigmentosa. or its receptor. Prader-Willi syndrome is a genomic imprinting 911 Hypogonadism can be caused by ? disorder. Testosterone is decreased & estradiol is increased. D. spinobulbar muscular atrophy. 3017 In obesity. AJAY MATHUR Endocrinology B. 1 in 1000 live-born males A. 3017 C. Anosmin marijuana users is caused by plant estrogens in crude preparations. Migrin 909 Which of the following is false in obesity ? D. Normal free testosterone levels combinations of anosmia or hyposmia. Estradiol levels are elevated due A. X-linked C. 3018 A. Prader-Willi syndrome is characterized by obesity. short stature & small hands & feet. Small hands and feet affected more than testis by excessive iron deposition leading to hypogonadism. Short stature D. Craniopharyngioma should be suspected when sellar mass. Mental retardation A. Hypogonadism C. 3018 Kallmann syndrome is an X-linked disorder due to mutations in KAL1 gene. hypotonic musculature. Obesity to aromatization of testosterone to estradiol in adipose tissue. Retinitis pigmentosa A. hypogonadism coexist. Craniopharyngioma Harrison’s 18th Ed. Obesity B. 3017 A. In hemochromatosis. Gynecomastia in B.596 MCQ’s FOR MEDICAL PROFESSIONALS BY PROF. Autosomal dominant Neurologic diseases associated with altered testicular function are myotonic dystrophy. 3017 D. 1 in 10000 live-born males B. Klinefelter syndrome B. Cytokine and/or glucocorticoid effects Harrison’s 18th Ed. Number of Leydig cells is increased. Gynecomastia C. Mental retardation B. 3018 C. Normal sized testes Viral orchitis may be caused by mumps virus. which of the following is not found ? 927 Which of the following inhibits testosterone synthesis ? Harrison’s 16th Ed. All of the above A. permanent androgen deficiency is uncommon. 20 rad A. Gynecomastia Radiation dose of 20 rad damages spermatogonia resulting in increased FSH & LH levels. Spironolactone A. 3018 C. Higher dose (80 rad) causes azoospermia. 2218 D. All of the above B. SF1 Harrison’s 18th Ed. After therapeutic radiation. XXY Harrison’s 16th Ed. D. Infertility group B arboviruses. 2218 A. 46. Inflammatory bowel disease D. 2218 Men with Klinefelter syndrome are at increased risk of breast cancer. 3018 D. Phenotypic females B. 75 % C. Short stature C. Breast cancer D. 47. 40 rad B. WT1 lung cancer but reduced risk of prostate cancer. SOX9 Harrison’s 18th Ed. Non-Hodgkin’s lymphoma C. WT1 . X fragments 926 At what dose of radiation. WT1 916 Hormone profile in Klinefelter syndrome includes all except ? B. 2215 B. 45. oligospermia or azoospermia develops ? 919 Turner syndrome is characterized by all except ? Harrison’s 18th Ed. 2216 Harrison’s 18th Ed.XXYY A. Eunuchoid features 925 Orchitis occurs in what percentage of adult men with mumps ? Harrison’s 18th Ed. 46. Lung cancer 922 Gonadal dysgenesis & primary adrenal failure occurs due to mutations in which of the following genes ? D.XXY B. All of the above 921 Gonadal dysgenesis & renal dysfunction occurs due to mutations 928 Which of the following blocks androgen action ? in which of the following genes ? Harrison’s 18th Ed. Increased FSH D. Hypertension A. Ketoconazole C. Echovirus Harrison’s 18th Ed. AJAY MATHUR Cardiology 597 914 Men with Klinefelter syndrome are at a reduced risk of ? B. 3018 D. 3019 A. 3019 Harrison’s 16th Ed. 10 % Harrison’s 16th Ed. 60 rad C. 80 rad D.XY 923 Gonadal dysgenesis & campomelic dysplasia occurs due to mutations in which of the following genes ? C. Increased LH 924 Viral orchitis may be caused by ? C. echovirus. 45. 2215 A. SF1 Harrison’s 18th Ed. Gynecomastia D. All of the above B. 47. non-Hodgkin’s lymphoma & A. Marijuana D. B. 597 Endocrinology MCQ’s FOR MCQ’s FOR MEDICAL MEDICAL PROFESSIONALS PROFESSIONALS BY PROF. Mumps virus 917 Clinical features of Klinefelter syndrome includes all except ? B. Decreased testosterone A. Spironolactone B. lymphocytic choriomeningitis virus & C. 3019 Harrison’s 16th Ed.XX D.X C. 3018 C. 3018 918 Which of the following is the most frequent karyotype in Turner syndrome (TS) ? A. Chronic pancreatitis C. 50 % B. D. All of the above B. Decreased estradiol Harrison’s 18th Ed. Osteoporosis B. SOX9 915 Genotype in classic form of Klinefelter syndrome is ? C. Prostate cancer Harrison’s 16th Ed. Lymphocytic choriomeningitis virus A. 25 % A. 48. Primary amenorrhea D.X Orchitis occurs in as many as one-fourth of adult men with mumps. 920 In an adult female with Turner syndrome. SOX9 A. SF1 A. Colour blindness D. 931 In Carney syndrome. 3019 A. 3019 Harrison’s 18th Ed. Pituitary B. Cushing’s disease in these patients are located around nose & mouth. 3019 B. and breast. Chapter 337 Sertoli cell tumor in men is a feature of the autosomal dominant Peutz-Jeghers syndrome. Newborn B. and testicular. Myxoma C. 553 C. Ratio of estrogen/androgen is increased. True gynecomastia is associated with glandular breast tissue that is >4 cm in diameter & often tender. 2081 Harrison’s 15th Ed. With aging D. Mucocutaneous pigmentation D. PRKA1C C. Sertraline A. D. Hypernephroma C. and oral cavity. PRKA1A A. Ketoconazole B. 3019 936 Drugs that can cause gynecomastia include all except ? A. All of the above C. mutation occurs in which of the following gene ? 937 Which of the following conditions do not cause galactorrhea ? Harrison’s 16th Ed. ketoconazole inhibits androgen synthesis. > 1 cm . increases estrogen and chemotherapy directly inhibits spermatogenesis. All of the above gynecomastia ? Harrison’s 18th Ed. and spironolactone inhibits androgen action. 938 Which of the following drugs do not cause galactorrhoea ? Harrison’s 15th Ed. 3019 B. All of the above Acromegaly occurs in about 20% of patients of Carney complex. Prolactin B.598 MCQ’s FOR MEDICAL PROFESSIONALS BY PROF. Flat foot D. B. > 3 cm D. 302. 929 Pituitary tumors in Carney complex secrete which of the following hormones ? 935 Gynecomastia occurs as a normal physiologic phenomenon in ? Harrison’s 18th Ed. AJAY MATHUR Endocrinology B. 930 Which of the following tumors occur in Carney complex ? Harrison’s 18th Ed. spironolactone blocks androgen action. GH A. adrenal. Lentigines A. A. Metoclopramide Harrison’s 18th Ed. Paroxetine Harrison’s 16th Ed. Hypothyroidism 939 Which of the following drugs do not cause galactorrhoea ? Harrison’s 15th Ed. Verapamil C. Quinidine D. ACTH C. Adrenal A. Vasopressin D. Leydig cell tumors in men galactorrhoea ? D. PRKA1D D. Hypothyroidism 934 What should be the diameter of glandular breast tissue in true D. Chapter 337 A. Intestinal polyps 940 Which of the following endocrine conditions can cause C. Metformin B. Hyperthyroidism C. Bronchogenic carcinoma B. Methyldopa A. marijuana Gynecomastia means enlargement of male breast glandular tissue & not excess adipose tissue. Reserpine 933 Which of the following is not a feature of Peutz-Jeghers syndrome ? B. Testicular Harrison’s 18th Ed. > 4 cm Ketoconazole inhibits testosterone synthesis. Ketoconazole Carney complex is characterized by spotty skin pigmentation. Ovarian sex cord stromal tumors Harrison’s 15th Ed. hands and feet. myxomas. During puberty C. Choriocarcinoma D. > 2 cm C. Spironolactone and pituitary tumors. Prevalence of gynecomastia increases with age & BMI because of increased aromatase activity in adipose tissue that converts adrenal and gonadal androgens and its precursors to estrogen. skin. Glandular breast tissue is firmer. Chapter 337 Carney syndrome is characterized by myxomas of heart. contains fibrous-like cords & is often tender. All of the above D. Pancreatic carcinoma Carney syndrome patients have mutations in the R1  regulatory subunit of protein kinase A (PRKAR1A) on chromosome 17q. Chapter 337 932 Which of the following is a feature of Carney syndrome ? A. PRKA1B B. Digitalis acts directly as estrogenic substance. Digitalis C. Atenolol B. Gastrointestinal polyps may become malignant. 3019 A. Marijuana C. Calcium channel antagonists D. 2215 . Anastrazole D.Y A. HIV-protease inhibitors 944 Drug-induced gynecomastia is due to all except ? C. Rifampin B. Table 71-2 956 Wolffian structures include all except ? A. Chapter 337 D. Angiotensin-converting enzyme inhibitors A. Abnormal cardiac function A. Ketoconazole 950 Gynecomastia is not caused by ? Harrison’s 15th Ed. Metoclopramide B.XXXY 947 Drug that causes gynecomastia by acting directly as estrogenic C. Heparin 945 Drug-induced gynecomastia is due to all except ? C. Abnormal kidney function 949 Galactorrhea is not caused by ? Harrison’s 15th Ed. Tricyclic antidepressants B. Phytoestrogen Harrison’s 16th Ed. 599 Endocrinology MCQ’s FOR MCQ’s FOR MEDICAL MEDICAL PROFESSIONALS PROFESSIONALS BY PROF. All of the above Harrison’s 15th Ed. Penicillamine B. Beta blockers B. Metoclopramide A. Ovarian follicles 948 Which of the following drugs is known to cause anaphylaxis and angioedema ? D. Theophylline A. All of the above A. Methyldopa 942 Drug-induced gynecomastia is due to all except ? C. Digitalis 943 Drug-induced gynecomastia is due to all except ? C. Chlorthalidone D. Chapter 337 D. All of the above B. Clomiphene B. Streptomycin A.XXY D. Phenytoin Harrison’s 15th Ed. Leydig cells D. All of the above B. Peutz-Jegher syndrome viable ? Harrison’s 16th Ed. 2214 C. Chapter 337 D. 48. Ranitidine A. Fadrozole 946 Excess estrogen production may be caused by tumors in C. Cimetidine B. Fromestane association with ? Harrison’s 16th Ed. Carney complex A. Domperidone D. 2193 C. 2192 D. Calcium channel blocking agents 952 Hyperkalemia is not caused by ? Harrison’s 15th Ed. Chapter 337 D. Antiretroviral agents A. All of the above B. Sertoli cell 954 Fetuses with which of the following chromosomal pattern are not B. Lithium Harrison’s 15th Ed. Spironolactone A. 45. Table 71-2 C. Sibutramine 953 Which of the following is an aromatase inhibitor ? Harrison’s 16th Ed. ACE inhibitors D. Table 71-2 C. Intravenous immune globulin men with ? C.X substance is ? Harrison’s 16th Ed. Methyldopa B. Sertoli cells C. Digitalis A. ACE inhibitors Harrison’s 16th Ed. Abnormal liver function B. Testosterone Harrison’s 15th Ed. Oral contraceptive 955 Anti-mullerian hormone (AMH) is secreted by ? B. 2214 C. ACE inhibitors A. 47. Omeprazole 951 Hyperglycemia is not caused by ? Harrison’s 15th Ed. AJAY MATHUR Cardiology 599 941 Gynecomastia of digitalis ingestion occurs most commonly in B. Clomiphene A. Table 71-2 C. Insulin Harrison’s 15th Ed. Chapter 337 D. Table 71-2 C. Tricyclic antidepressants Harrison’s 15th Ed. 45. 2192 D. Prior infectious orchitis A. FSH D. 3021 957 Mullerian ducts forms all except ? Harrison’s 16th Ed. Cancer chemotherapeutic agents Tamoxifen is an antiestrogen. All of the above C. Marfan syndrome Harrison’s 18th Ed. Klinefelter syndrome D. A. > 50 ng/dL . 3021 hyperprolactinemia. Tamoxifen ? B. Chronic illness Incidence of autosomal dominant diseases like achondroplasia. Evening C. Excessive exercise Apert’s syndrome increases in offspring of men who are advanced in age (sporadic missense mutations). Gradual rise of LH levels with age Harrison’s 18th Ed. Harrison’s 16th Ed. radiation to testes. torsion. Harrison’s 18th Ed. what D. Testosterone levels decrease with age B. C. cancer chemotherapy. Noon B. Early-morning A. Hemochromatosis 959 Aromatase inhibitors include all except ? D. Epididymides B. and substance abuse. C. Midnight D. > 150 ng/dL B. < 400 ng/dL 969 Human menopausal gonadotropin (hMG) contains ? Harrison’s 18th Ed. < 800 ng/dL B. Klinefelter syndrome C. hemochromatosis.600 MCQ’s FOR MEDICAL PROFESSIONALS BY PROF. Urogenital swelling B. 3021 D. trauma. All of the above A. Uterus C. LH 963 What level of total testosterone excludes testosterone deficiency ? C. Hemochromatosis Harrison’s 18th Ed. chronic illness. None of the above A. Lower vagina Suspicion of androgen deficiency prompts measurement of total testosterone. 3021 D. < 300 ng/dL B. 2215 965 Which of the following is not a cause of primary testicular failure ? A. 3020 infection. Uncorrected cryptorchidism level of total testosterone denotes testosterone deficiency ? Common causes of acquired secondary hypogonadism include space-occupying lesions of the sella. Testolactone A. infectious orchitis. uncorrected cryptorchidism. Urogenital sinus A. Marfan syndrome & B. Testolactone is a relatively weak aromatase inhibitor. > 250 ng/dL C. excessive exercise. 2193 966 Which of the following is not a cause of primary testicular failure A. Substance abuse 962 In a patient with symptoms or signs of androgen deficiency. Fromestane B. Anastrazole Harrison’s 18th Ed. preferably in morning. None of the above A. Genital tubercle Harrison’s 18th Ed. 3022 C. 2215 A. 3021 B. Prior infectious orchitis Harrison’s 16th Ed. polyposis coli. A. AJAY MATHUR Endocrinology A. Achondroplasia B. FSH & LH Harrison’s 18th Ed. Seminal vesicles D. Prostate 964 Samples for testosterone estimation are obtained in ? Harrison’s 18th Ed. Space-occupying lesions of sella 961 Which of the following diseases increase in occurrence in offspring B. 958 Prostate develops from ? Early-morning testosterone level >350 ng/dL makes diagnosis of androgen deficiency unlikely. 3021 D. Prior infectious orchitis D. Hyperprolactinemia of men who are advanced in age ? C. 3021 C. Upper vagina D. anorchia syndrome & myotonic dystrophy. Fallopian tubes B. < 600 ng/dL A. SHBG levels are higher in older men 967 Which of the following is not a cause of acquired hypogonadotropic hypogonadism ? C. Vasa deferentia C. HIV infection D. 3021 D. HIV Harrison’s 18th Ed. Polyposis coli 968 Which of the following is not a cause of acquired hypogonadotropic hypogonadism ? C. Hyperprolactinemia 960 Which of the following statements about age related hormone status is false ? Common causes of primary testicular dysfunction include Klinefelter syndrome. Total testosterone level <300 ng/dL in association with symptoms is evidence of testosterone deficiency. > 350 ng/dL D. 3024 A. both pubertal development & spermatogenesis can be Drugs perceived to be muscle-building or performance-enhancing include growth hormone. Sleep apnea Gas chromatography-mass spectrometry. Measurements of LH B. Blood C. high- Testosterone should not be given in prostate cancer. 3027 Harrison’s 18th Ed. Add FSH C. FSH A. abuse. All of the above In documented GnRH deficiency. or recombinant hFSH. LH B. Harrison’s 18th Ed. All of the above 973 After 6 months of hCG alone therapy. D. Urine B. Bone marrow failure B. if testosterone levels are in Commonly used androgenic steroids by athletes include testosterone esters. 3025 A. Pruritis Erythrocytosis is the most frequent adverse event reported in testosterone trials in middle- 972 When drugs are used to restore spermatogenesis. High-normal range B. 977 Most frequent adverse event reported in testosterone trials in 971 Which of the following is useful in restoring spermatogenesis ? middle-aged and older men is ? Harrison’s 18th Ed. All of the above D. All of the above D. Rise in PSA levels B. IGF-1 B. Increasing doses of single androgenic steroid C. hCG C. Human menopausal gonadotropin (hMG) is purified from urine of postmenopausal women and Testosterone should be measured 3 months after initiating therapy to assess adequacy of therapy. insulin. contains FSH & LH. IGF- successfully induced by pulsatile administration of low doses of GnRH. Athletes use increasing doses of multiple steroids in a practice known as stacking. liquid chromatography-mass spectrometry. Low-normal range 978 Androgenic steroids used by athletes is ? Harrison’s 18th Ed. AJAY MATHUR Cardiology 601 970 Human menopausal gonadotropin (hMG) is derived from which 976 Test for assessing the adequacy of testosterone replacement is ? of the following of postmenopausal women ? Harrison’s 18th Ed. to men with resolution mass spectrometry and tandem mass spectrometry are of use in detecting androgen baseline hematocrit >=50% and sleep apnea syndrome. stacking refers to ? Harrison’s 18th Ed. Cholestasis D. . Measurements of FSH C. Increasing doses of multiple androgenic steroids D. Add LH B. which of stanozolol. methandienone & methenolol. Withdrawal symptoms of androgenic steroids B. Any of the above D. 981 Method for detecting androgen abuse is ? 975 Androgen therapy can be given in all except ? Harrison’s 18th Ed. 3022 A. Insulin C. This response requires 1. Gas chromatography-mass spectrometry A. High-resolution mass spectrometry C. 3022 A. Stanozolol D. 3022 Harrison’s 18th Ed. 3022 979 In athletes. Nandrolone C. Any of the above C. Measurements of testosterone levels A. 3024 Harrison’s 18th Ed. 3026 A. Erythrocytosis C. Saliva D. nandrolone. 3026 A. Documented GnRH deficiency A. 3022 Harrison’s 18th Ed. 601 Endocrinology MCQ’s FOR MCQ’s FOR MEDICAL MEDICAL PROFESSIONALS PROFESSIONALS BY PROF. normal pituitary & testicular function. benign prostatic hypertrophy. Endometriosis D. Liquid chromatography-mass spectrometry B. highly purified urinary hFSH. the following should be done ? Harrison’s 18th Ed. ephedrine and thyroxine. mid-normal range and sperm concentrations are low. Normal pituitary function B. Rage reactions with of androgenic steroids FSH is available as hMG. Increase dose of hCG A. 3026 B. All of the above D. amphetamine. Any of the above LH & FSH estimation are not useful in assessing adequacy of testosterone replacement. All of the above D. Hereditary angioedema C. Methandienone Drugs used to restore spermatogenesis should bring testosterone levels into the mid-normal range. testosterone aged and older men and is also the most frequent cause of treatment discontinuation in these levels should be raised to ? trials. clenbuterol. Mid-normal range A. 974 Which of the following is a mandatory requirement of GnRH therapy ? 980 Muscle-building or performance-enhancing agents include ? Harrison’s 18th Ed. Normal testicular function C. Thyroxine D. During the first 10 days after birth B. and Contraception C. Infant mortality rate is the number of infant deaths per 1000 live births. C. Urinary testosterone to serum testosterone ratio B. but are termed abortuses for purposes D. 500 grams 992 Infant mortality rate is the number of infant deaths per B.602 MCQ’s FOR MEDICAL PROFESSIONALS BY PROF. 985 Birth rate is expressed as ? A. Urinary 13C : 12C ratio in testosterone A. 1000 grams 988 Late neonatal death refers to ? C. Ratios > 6 suggest exogenous testosterone use. Number of stillbirths + neonatal deaths per 10000 total births Exogenous testosterone use is confirmed by estimating 13C : 12C ratio in testosterone by isotope Perinatal mortality rate is number of stillbirths + neonatal deaths per 1000 total births. C. 9 months of age D. A. 12 months of age 984 Fetuses weighing less than what are termed abortuses ? Infant death refers to all deaths of liveborn infants from birth to 12 months of age. Number of neonatal deaths per 10000 live births D. 2500 grams 987 Early neonatal death refers to ? Very low-birthweight is a newborn whose weight is less than 1500 grams. 3 months of age Chapter 347. 1000 live births D. 3027 990 Perinatal mortality rate refers to ? A. Infertility. 15 to 44 years 994 Very low-birthweight is a newborn whose weight is less than ? C. AJAY MATHUR Endocrinology 982 Illicit testosterone use is detected by measurement of ? D. 991 Infant death refers to all deaths of liveborn infants from birth to ? A. Number of live births per 1000 population 993 Low-birthweight is a newborn whose weight is less than ? A. 500 grams B. 500 grams D. 1500 grams D. Number of live births per 1000000 population D. Urinary C : C ratio in testosterone 9 10 D. Death after 7 days but before 29 days . Number of neonatal deaths per 1000 live births Exogenous testosterone administration increases urinary testosterone glucuronide excretion & consequently testosterone to epitestosterone ratio. 100000 live births of vital statistics. Number of neonatal deaths per 10000 total births D. ratio combustion mass spectrometry. 15 to 48 years B. 500 grams D. 1000 grams Fertility rate of females is the number of live births per 1000 females aged 15 through 44 years. Death after 7 days but before 42 days Harrison’s 18th Ed. 6 months of age System. 3027 Early neonatal death refers to death of a liveborn neonate during the first 7 days after birth. 100 live births C. 1000 grams C. Urinary epitestosterone 989 Neonatal mortality rate refers to ? C. Urinary C : C ratio in testosterone 10 11 C. 10000 live births Fetuses weighing < 500 grams are not considered as births. Death after 7 days but before 21 days Extremely low-birthweight is a newborn whose weight is less than 1000 grams. During the first 7 days after birth than ? A. 15 to 40 years B. Number of live births per 100000 population C. C. A. Number of neonatal deaths per 100 live births D. Urinary testosterone to epitestosterone ratio A. Death after 7 days but before 14 days D. Number of stillbirths + neonatal deaths per 1000 total births C. 1000 grams C. Urinary 11C : 12C ratio in testosterone B. Number of neonatal deaths per 1000 total births B. The Female Reproductive B. 1500 grams D. During the first day after birth B. B. Number of live births per 10000 population B. 600 grams A. 1500 grams A. 2500 grams B. A. 2500 grams 986 Fertility rate of females is calculated between what age range ? Low-birthweight is a newborn whose weight is less than 2500 grams. During the first 3 days after birth 995 Extremely low-birthweight is a newborn whose weight is less C. Late A. Synthetic testosterone has a lower 13C : 12 C ratio than endogenously produced testosterone. 750 grams B. 15 to 46 years A. Number of neonatal deaths per 100000 live births 983 Illicit testosterone use is confirmed by measurement of ? Harrison’s 18th Ed. Urinary testosterone neonatal death refers to death after 7 days but before 29 days. 260 to 294 days 1003 What percentage of women suffer from depression during D. In all pregnant women. the new onset of hypertension (BP >140/90 D. Heart rate increases by about 10 beats per minute during the third trimester. B. 1000 live births D. Harrison’s 17th Ed. Estrogens have stimulatory actions on cellular immunity D. 130 / 80 mmHg 999 Abortus is fetus or embryo removed or expelled from uterus maximally before ? B. 10 % pregnancy ? C. 603 Endocrinology MCQ’s FOR MCQ’s FOR MEDICAL MEDICAL PROFESSIONALS PROFESSIONALS BY PROF.e. or renal disease. Harrison’s 16th Ed. and proteinuria in preeclampsia ? A. A. 130 / 90 mmHg A. B. measurement of BP should be performed D. Supine B. AJAY MATHUR Cardiology 603 996 Term neonate is a neonate born after how many days of pregnancy ? C. 31 A. 20 weeks or less and weighing less than 500 grams. 259 days Harrison’s 17th Ed. because for many the lateral recumbent position is associated with a blood pressure lower than that recorded in sitting position. 16 weeks of gestation C. 44 Abortus is a fetus or embryo removed or expelled from uterus during the first half of gestation i. 1002 Which of the following statements is false ? 1008 Which of the following contributes to endothelial dysfunction. Women awaken from anesthesia faster than men given the same A.000 live births. Sitting 1000 Maternal mortality ratio is the number of maternal deaths that result from reproductive process per ? C. 22 weeks of gestation in which position ? Harrison’s 17th Ed. Adaptive immune responses are more robust in women than B. 20 % C. cardiac output increases by ? C. 31 hypertension. what value of blood pressure is considered as D. 40 % A. 18 weeks of gestation D. Standing A. 5 % 997 Preterm neonate is a neonate born before how many days of B. 5 to 7 % in men C. Any of the above B. a BP of 140/90 mmHg is considered as abnormally elevated & is associated with increase in perinatal morbidity & mortality. Diagnosis of hypertension requires D. None of the above mmHg) and proteinuria (>300 mg/day) after 20 weeks of gestation. 31 Term neonate is a neonate born anytime after 37 completed weeks of gestation and up until 42 completed weeks of gestation (260 to 294 days). Women have lower frequency of adverse drug reactions than men C. fms-like tyrosine kinase 1 doses of anesthetics B. chronic hypertension. 140 / 90 mmHg C.7% of all pregnant women develop preeclampsia. 100000 live births be performed in the sitting position. 245 to 284 days schizophrenia B. Women require lower doses of neuroleptics to control A. Androgens have inhibitory actions on cellular immunity ~5 . 15 to 18 % C. Hypertension during pregnancy is usually caused by preeclampsia. 40 % B. 1000000 live births measurement of two elevated BP. 20 weeks of gestation 1006 In all pregnant women. Maternal mortality ratio is the number of maternal deaths that result from the reproductive process per 100. 140 / 80 mmHg B. measurement of BP should C. 294 days Increase in CO is is due to increase in stroke volume. 252 to 290 days D. 296 days 1005 During pregnancy. at least 6 hours apart. Women have lower total body water than men Harrison’s 17th Ed. gestational hypertension. beginning with day 295. 25 % A. 12 to 15 % B. 30 % 998 Post-term neonate is a neonate born on or after how many days of pregnancy ? D. 261 days A. 248 days D. 44 A. 2 to 3 % A. 44 B. 295 days C. 10000 live births During pregnancy. A. 10 % Preterm neonate is a neonate born before 37 completed weeks (259th day). fms-like tyrosine kinase 2 . 297 days abnormally elevated and is associated with a marked increase in perinatal morbidity and mortality ? Postterm neonate is a neonate born anytime after completion of 42nd week. 252 days 1004 In pregnancy. 44 D. 1007 What percentage of pregnant women suffer from preeclampsia ? 1001 Which of the following statements is false ? Harrison’s 17th Ed. 44 Harrison’s 16th Ed. 268 to 298 days pregnancy and postpartum period ? Harrison’s 16th Ed. Multiparity Definitive treatment of preeclampsia is delivery of fetus & placenta. Chronic hypertension 1017 Drug that can be used to manage hypertension in severe preeclampsia is ? 1011 Risk factors for development of preeclampsia include all except ? Harrison’s 17th Ed. coma). Labetalol A. Near-normal blood pressures C. Multiple gestation 1018 Drug of choice for prevention & treatment of eclamptic seizures D. D. 28 weeks the risk of stroke at near-normal blood pressures. Severe proteinuria C. Presence of platelet dysfunction & coagulation disorders further increases the risk of stroke. None of the above following in central nervous system ? Harrison’s 17th Ed. a naturally occurring vascular endothelial growth factor antagonist. D. Factor V Leiden mutation Aggressive management of blood pressures > 160/110 mmHg in severe preeclampsia reduces the 1012 Risk factor for development of preeclampsia is ? risk of cerebrovascular accidents. factor V Leiden mutation. AJAY MATHUR Endocrinology C. 160 / 110 mmHg 1020 Pregnant women with chronic hypertension are at increased risk Severe preeclampsia is the presence of new-onset hypertension and proteinuria accompanied by CNS dysfunction (headaches. Hemolysis Excessive placental secretion of a soluble fms-like tyrosine kinase 1. 32 weeks 1010 Risk factors for development of preeclampsia include all except ? C. 44 in blood flow to fetus. a history of renal disease or chronic hypertension. New-onset hypertension Magnesium sulfate is better than phenytoin & diazepam and is the treatment of choice for prevention & treatment of eclamptic seizures. prior history of preeclampsia. 44 D. the blood pressure A. and decreased secretion of placental growth factor may B.604 MCQ’s FOR MEDICAL PROFESSIONALS BY PROF. 44 Harrison’s 17th Ed. 44 D. 44 B. 45 injury (ALT > 2 x the upper limits of normal). hepatocellular Harrison’s 17th Ed. Any of the above D. B. 36 weeks Harrison’s 17th Ed. Extremes of maternal age (>35 years or <15 years) C. “H” stands for ? Harrison’s 17th Ed. Calcium channel blocker C. Hypertension range blood pressures 1016 For women with severe preeclampsia. A.000/L). Any of the above A. Hepatitis 1009 Stroke in preeclampsia may occur at ? D. low platelets) syndrome is a special subgroup of A. Dopamine receptors C. B. Glutamate receptors A. C. Any of the above after how many weeks of gestation ? Harrison’s 16th Ed. seizures. angiotensinogen gene T235. delivery is recommended D. 44 D. 44 A. Central nervous system dysfunction 1019 Magnesium prevents seizures by interacting with which of the D. diabetes mellitus. thrombocytopenia (platelet count < 100. or disseminated intravascular coagulation. 44 1014 For the diagnosis of “severe preeclampsia”. delivery reduces mother’s morbidity but exposes fetus to C. N-methyl-D-aspartate (NMDA) receptors B. For pregnant women with B. 44 Risk factors for the development of preeclampsia include nulliparity. oliguria or renal failure. Angiotensin-converting enzyme (ACE) inhibitors as well as angiotensin- A. B. IV labetalol or hydralazine and calcium channel blockers are drugs most commonly used. Diabetes mellitus preeclampsia before 37 weeks’ gestation. Antiphospholipid antibody syndrome C. severe proteinuria (>5 g/day). Low BMI D. B. Hydralazine C. Sub-normal blood pressures severe preeclampsia which increases morbidity & mortality. History of renal disease risk of premature delivery. blurred vision. Magnesium sulfate A. Calcium channel blocker 1013 To define severe preeclampsia. All of the above is ? Harrison’s 17th Ed. hypertension & proteinuria observed in preeclampsia. fms-like tyrosine kinase 3 1015 In HELLP syndrome. GABA receptors must be above ? Harrison’s 17th Ed. Hydralazine B. which of the following is false ? Harrison’s 17th Ed. 140 / 100 mmHg Magnesium prevents seizures by interacting with N-methyl-D-aspartate (NMDA) receptors in CNS. G20210A prothrombin gene mutation. antiphospholipid antibody syndrome & multiple gestation. extremes of maternal age A. Intrauterine growth restriction . Hypertension Harrison’s 17th Ed. 33 Preeclampsia is associated with abnormalities of cerebral circulatory autoregulation. 130 / 90 mmHg D. Prior history of preeclampsia B. pulmonary edema. 44 HELLP (hemolysis. Angiotensinogen gene T235 receptor blockers should be avoided in the second and third trimesters of pregnancy because of their adverse effects on fetal development. fms-like tyrosine kinase 4 A. B. obesity. marked elevations of BP (>160/110 for all except ? mmHg). Labetalol (>35 years or <15 years). which increase A. elevated liver enzymes. 120 / 80 mmHg C. Hemorrhage contribute to endothelial dysfunction. Raised BP should be reduced slowly to avoid hypotension & a decrease Harrison’s 17th Ed. Prothrombin G20210A mutation (heterozygotes & homozygotes) & methylenetetrahydrofolate reductase C677T mutation (homozygotes) also 1021 Which of the following drugs is most commonly used for treatment increases risk of DVT. DVT occurs much more commonly in ? D. Warfarin therapy is contraindicated This is the development of elevated blood pressure during pregnancy or in the first 24 hours post C. Left arm Harrison’s 17th Ed. About 50 % C. About 25 % A. Fasting plasma glucose is lower by 15 . warfarin is teratogenic Harrison’s 17th Ed. Fasting plasma acetoacetate higher 1026 Genetic mutations associated with DVT during pregnancy include ? D. 46 When DVT occurs in the postpartum period. Mental retardation D. fasting plasma glucose is lower by 15 . the blood pressure is elevated Warfarin therapy is contraindicated in the 1st trimester due to its association with fetal during which trimester of pregnancy ? chondrodysplasia punctata. Right arm A. 1 . LMWH or unfractionated heparin can be used for anticoagulation D.& homozygotes) C.6 months Mitral stenosis is the valvular disease most likely to cause death during pregnancy. LMWH therapy for 7 . Left leg B. Prophylaxis against thrombosis with low-molecular-weight D. 78 associated with ? Harrison’s 17th Ed. D. Methylenetetrahydrofolate reductase C677T mutation D. 46 D. superimposed preeclampsia & abruptio Activated protein C resistance caused by factor V Leiden mutation increases risk of DVT. Fasting plasma hydroxybutyrate higher In pregnant women.20 mg/dL than in nonpregnant state due to use of glucose by fetus. Post partum hemorrhage (homozygotes) Pregnancy complicated by chronic essential hypertension is associated with intrauterine D. Superimposed preeclampsia B. In pregnancy. Hydralazine contemplating pregnancy ? Harrison’s 16th Ed. About 33 % B. 45 mental retardation. 34 B. During pregnancy. C. Third A. About 75 % D. Abruptio placenta C. labetalol & nifedipine are the most commonly used medications for the treatment C. Mitral regurgitation for ? C. A. DVT can occur in postpartum period 1024 Valvular disease most likely to cause death during pregnancy is ? Warfarin is not contraindicated in breast-feeding women. Factor V Leiden mutation hydroxybutyrate & acetoacetate levels may rise to 2 . . All of the above 1023 In “Gestational Hypertension”. due to compression of left iliac vein by iliac artery and the uterus. 6 . Mitral regurgitation.20 mg/dL B.4 times the normal after a fast. Warfarin is contraindicated in breast-feeding women partum in the absence of preexisting chronic hypertension and other signs of preeclampsia. 46 Pregnancy is a state of accelerated starvation & ketosis. None of the above Harrison’s 17th Ed. 9 . Plasma A. First 1029 Which of the following about DVT in pregnancy is false ? B. aortic regurgitation & stenosis are well tolerated during pregnancy.9 months 1025 In pregnant women. Mitral stenosis 1030 When DVT occurs in the postpartum period.6 months. increased perinatal mortality. AJAY MATHUR Cardiology 605 B. Prophylaxis with unfractionated heparin is recommended of chronic hypertension in pregnancy. In 2nd & 3rd trimesters. A. 45 1027 Which of the following is false for women with artificial valves A. 46 A. pulmonary placenta. Any time during pregnancy B. Warfarin be stopped & heparin initiated prior to conception C. All of the above growth restriction. Second Harrison’s 17th Ed. Beta blocker B.3 months B.12 months Harrison’s 17th Ed. None of the above 1022 What percentage of all aortic dissections in women under 40 years of age occur during pregnancy ? 1028 Warfarin therapy during the first trimester of pregnancy is Harrison’s 16th Ed. Aortic regurgitation A. 45 & should be avoided. Aortic stenosis Harrison’s 17th Ed. embolism & severe preeclampsia during pregnancy. warfarin may cause fetal optic atrophy and Harrison’s 17th Ed. A.10 days may be followed by warfarin therapy for 3 . Fetal optic atrophy C. Prothrombin G20210A mutation (hetero. 46 D. Labetalol A. 605 Endocrinology MCQ’s FOR MCQ’s FOR MEDICAL MEDICAL PROFESSIONALS PROFESSIONALS BY PROF. warfarin is given B. of chronic hypertension in pregnancy ? Harrison’s 17th Ed. 3 . Fetal chondrodysplasia punctata B. 46 C. C. Right leg 1031 Which of the following is false in pregnancy ? C. D. Diuretic heparin (LMWH) is not recommended Alpha methyldopa. DVT occurs more commonly in left leg than in right leg. First trimester of pregnancy D. Metabolic acidosis In pregnancy. renal agenesis & ventricular 1040 Circulating levels of which of the following remains unaltered by septal defect. Metabolic alkalosis D. fasting blood glucose should be maintained at <105 mg/dL with no values >140 mg/dL to reduce congenital anomalies like sacral agenesis. 1. Thyroxine-binding globulin (TBG) what level of serum glucose measured at 60 minutes is considered D. <160 mg/dL Harrison’s 17th Ed. Any of the above A. <190 mg/dL Harrison’s 17th Ed. Third trimester of pregnancy normal ? Harrison’s 17th Ed. Beta blockers Harrison’s 17th Ed. <140 mg/dL 1043 Hyperthyroidism is most difficult to control in ? Harrison’s 17th Ed. 46 D. 47 C. <200 mg/dL A. <126 mg/dL C. 180 mg/dL C. 126 mg/dL A. 47 For diagnosis of gestational diabetes. <100 mg/dL 1044 Hyperthyroidism is easiest to control in ? B. Free T4 normal ? B. maternal insulin resistance is due to ? After a 100-gram oral glucose challenge in a pregnant woman. Total T3 1035 After a 100-gram oral glucose challenge in a pregnant woman. > 140 mg/dL production. 46 A. Methimazole what level of serum glucose measured at 120 minutes is considered C. pregnancy ? Harrison’s 17th Ed. <165 mg/dL B. <145 mg/dL A. 46 In pregnancy. <100 mg/dL Methimazole crosses placenta more than propylthiouracil & is associated with fetal aplasia cutis. C. 190. > 160 mg/dL 1033 Pregnancy complicated by diabetes mellitus. All of the above normal ? Harrison’s 17th Ed. 46 plasma glucose concentrations are <105. Placental production of a growth hormone variant hour postprandial glucose concentrations is ? C. 46 1039 Which of the following is a common feature of pregnancy ? Harrison’s 16th Ed. B. AJAY MATHUR Endocrinology 1032 During pregnancy. All of the above A. 140 mg/dL B. aim should be to D. Second trimester of pregnancy .2 and 3 hour Harrison’s 17th Ed. estrogen-induced increase in thyroxine-binding globulin causes increase in circulating levels of total T3 & total T4. > 180 mg/dL keep all blood glucose levels below ? Harrison’s 17th Ed. serum glucose is A. & TSH remain unaltered A. Thyroid stimulating hormone (TSH) A. All of the above B. <165 mg/dL A. <100 mg/dL D. C. B. 47 D. Normal range of circulating levels of free T4. 35 D. normally fasting. B. B. respectively. 165. Second trimester of pregnancy what level of serum glucose measured at 180 minutes is considered C. Respiratory acidosis B. Beta blockers & saturated solution of potassium iodide may be necessary in emergency. Free T3 Harrison’s 17th Ed. what level of serum glucose measured at 60 minutes is considered A. Total T4 measured at 60 minutes. Saturated solution of potassium iodide normal ? D. growth hormone variant & placental lactogen. Propylthiouracil 1036 After a 100-gram oral glucose challenge in a pregnant woman. free T3. 270 A. <160 mg/dL 1042 Treatment of choice of hyperthyroidism in pregnancy is ? C. <140 mg/dL 1041 Circulating levels of which of the following is increased by pregnancy ? D. Placental production of placental lactogen Harrison’s 16th Ed. and 145 mg/dL. <126 mg/dL Harrison’s 17th Ed. > 120 mg/dL Metabolic changes in pregnancy includes maternal insulin resistance caused by placental steroid B. 47 D. Placental production of steroids 1038 Insulin therapy in gestational diabetes should be started when 2- B. Values <140 mg/dL are normal. Respiratory alkalosis C. caudal dysplasia.606 MCQ’s FOR MEDICAL PROFESSIONALS BY PROF. A. 47 1034 After a 50-gram oral glucose challenge in a pregnant woman. <126 mg/dL C. 200 mg/dL D. 46 C. B. after a 50 gram oral glucose challenge. <140 mg/dL by pregnancy. First trimester of pregnancy 1037 After a 100-gram oral glucose challenge in a pregnant woman. Differential diagnosis should include immune thrombocytopenia. entrapment of lateral femoral cutaneous nerve. Worsen B. Lateral femoral cutaneous nerve A. dopamine-agonist therapy). Urinary tract tuberculosis A. AJAY MATHUR Cardiology 607 C. immune globulin soon after birth (within first 72 hours). Any of the above Intrahepatic cholestasis of pregnancy is a 3rd trimester event manifesting as profound pruritus. 1045 During pregnancy with hypothyroidism. 47 Harrison’s 17th Ed. Third trimester C. the dose of thyroxine required to keep the TSH in the normal range rises. Hyperglycemia D. Ulnar nerve D. Any of the above D. 139 A. Improve C. All of the above During pregnancy. 48 Harrison’s 16th Ed. 48 1047 Migraine during pregnancy may ? Harrison’s 17th Ed. 48 B. 158 A. neuroleptics. Benign gestational thrombocytopenia B. Gall bladder disease exacerbation increased by 30% as soon as pregnancy is diagnosed & then adjusted by serial measurement of is due to pregnancy-induced alteration in the metabolism of bile and fatty acids. None of the above D. Group B streptococcus . It Thrombocytopenia occurs commonly during pregnancy (benign gestational thrombocytopenia). Increased A. Sleep deprivation B. retention of dead fetus. Decreased B. Hepatitis B vaccine B. Remain same D. Ulcerative colitis shows Increased thyroxine requirement occurs as early as 5th week of pregnancy. Headache appearing during pregnancy is usually migraine which may worsen. birth control pills. Any of the above Acute fatty liver of pregnancy is characterized by markedly increased levels of bilirubin & ammonia & hypoglycemia. thyroxine dose is exacerbation in first trimester and during early postpartum period. Asymptomatic bacteriuria B. Hepatitis B immune globulin A. Gall bladder disease D. All of the above Screening for asymptomatic bacteriuria with nitrite/leukocyte esterase strips is indicated for high- risk pregnant women (sickle cell trait or a history of UTI). Urinary tract fungal infection C. or be unaffected by pregnancy. Median nerve Harrison’s 17th Ed. hyperthyroidism or antiphospholipid syndrome. Hypothyroidism D. Common peroneal nerve Hyperthyroidism is most difficult to control in the first trimester of pregnancy and easiest to control Manifests as pain and numbness in the lateral aspect of thigh without weakness. 607 Endocrinology MCQ’s FOR MCQ’s FOR MEDICAL MEDICAL PROFESSIONALS PROFESSIONALS BY PROF. Immune thrombocytopenia C. 48 Harrison’s 16th Ed. Crohn’s disease B. 1053 Acute fatty liver of pregnancy is characterized by all except ? Harrison’s 17th Ed. sepsis. Any of the above D. is associated with increased fetal mortality. improve. Markedly increased levels of ammonia B. abruptio placenta & amniotic fluid embolism. Ulcerative colitis C. 47 A. Crohn’s disease shows exacerbation in second and third trimesters. Second trimester B. Pregnancy C. 47 1056 Intrauterine amniotic infection is most often caused by ? A. 1054 Infants born to mothers who are carriers of HBsAg should also receive ? 1048 Chorea may be a manifestation of all except ? Harrison’s 17th Ed. Birth control pills B. Same C. Preeclampsia D. 1052 Intrahepatic cholestasis of pregnancy happens during ? 1046 Thrombocytopenia during pregnancy may be due to ? Harrison’s 17th Ed. the dose of thyroxine 1051 Which of the following diseases show exacerbation during required to keep TSH in normal range is ? pregnancy ? Harrison’s 17th Ed. Hypoglycemia C. It is due to in the third trimester. Hepatitis B Ig + Hepatitis B vaccine C. Pregnancy D. 47 A. preeclampsia. 1055 Screening of pregnant woman with nitrite/leukocyte esterase urine 1049 Symptoms of “Restless legs syndrome” (RLS) can be exacerbated strips is done for ? by ? Harrison’s 17th Ed. Antiphospholipid syndrome Infants born to mothers who are carriers of hepatitis B surface antigen should receive hepatitis B Acute or subacute chorea is due to toxins (excess levodopa. 47 A. 1050 “Meralgia paresthetica” is caused by entrapment of ? Harrison’s 17th Ed. First trimester A. Any of the above D. pregnancy (chorea gravidarum). Markedly increased levels of bilirubin A. 48 Harrison’s 17th Ed. Third trimester of pregnancy C. Caffeine C. All of the above D. TSH. 28 % Harrison’s 17th Ed. 48 A. Regardless of the mode of delivery.608 MCQ’s FOR MEDICAL PROFESSIONALS BY PROF. recent infection with high maternal viral load. 76 % . Zidovudine therapy before & during delivery D. 49 C. in decreasing order of frequency. 12 B. Harrison’s 17th Ed. hemolytic anemia. preterm delivery. clindamycin is presence of other genital tract infections (syphilis or herpes). 3035 1062 Which of the following does not increase the risk of mother-to- child transmission of HIV ? A. Eating undercooked meat months of unprotected sexual intercourse ? D. 49 B. Pulmonary embolism Rubella virus is a known teratogen. Breast-feeding may transmit HIV 1058 Severe CMV disease in the newborn is characterized by all to the newborn and is therefore contraindicated for HIV-infected mothers. Iron deficiency anemia D. microcephaly. Petechiae B. Antepartum period A. except ? Harrison’s 17th Ed. C. obstetric hemorrhage. especially in women with a viral load >1000 copies/mL. low Intrauterine amniotic infection is mostly caused by Escherichia coli & group B streptococcus. pulmonary embolism. 3035 A. Chlamydia Exposures which increase risk of mother-to-child transmission include vaginal delivery. maternal CD4+ T cell count. In penicillin-allergic patients. Zidovudine treatment to neonate at birth Severe cytomegalovirus infection in newborn is characterized by petechiae. 48 B. 42 % A. Breast feeding D. prolonged length of membrane rupture. hypertension. All of the above & jaundice. Preterm delivery D. cardiovascular conditions including A. Staphylococcus C. Tetracycline Infertility is defined as the inability to conceive after 12 months of unprotected sexual intercourse. recommended. Second trimester is ? Harrison’s 17th Ed. Sex with multiple partners peripartum cardiomyopathy & ectopic pregnancy. Cesarean section is associated with additional risk reduction 1059 Risk of fetal anomalies in congenital rubella is highest in ? compared to vaginal delivery. Maternal bleeding D. Spiramycin D. C. Chorioretinitis Harrison’s 17th Ed. 48 Harrison’s 17th Ed. 58 % B. Zidovudine (ZDV) administered during pregnancy and labor and to the newborn reduces the risk of vertical transmission by 70%. Mechanisms of vertical transmission is infection after rupture of membranes & direct contact of fetus with infected secretions or blood from maternal genital tract. 48 1064 Which of the following reduces the risk of vertical transmission of HIV infection ? A. First-trimester rubella carries a high risk of fetal anomalies. Jaundice Majority of mother-to-child (vertical) transmission of HIV-1 occur during intrapartum period. First trimester 1065 Most common causes of maternal death in United States today B. Macrocephaly A. Cesarean section C. Trauma to fetal skin C. 3 1061 Triple-drug therapy for placental/fetal toxoplasmosis includes all B. 37 Most common causes of maternal death in US today are. Cesarean section C. Ectopic pregnancy D. AJAY MATHUR Endocrinology B. and the Penicillin G & ampicillin are the drugs of choice. maternal bleeding. B. Pyrimethamine 1067 Infertility can be attributed primarily to female factors in what D. hepatitis. Intracranial calcifications B. Chorioretinitis. prolonged labor. 9 A. sepsis. Sulfonamide percentage of infertile couples ? Harrison’s 18th Ed. trauma to fetal skin. 37 C. Hepatosplenomegaly C. Postpartum period C. and purpura may also develop. Obstetric hemorrhage 1060 Most primary infections of toxoplasmosis in US come from ? Harrison’s 16th Ed. Hemolytic anemia may be present. B. Third trimester A. 1063 Majority of cases of mother-to-child (vertical) transmission of 1057 Severe CMV disease in newborn is characterized by all HIV-1 occur during ? except ? Harrison’s 17th Ed. Equally in all three trimesters B. Intrapartum period B. Hepatitis C. 6 except ? Harrison’s 16th Ed. intrapartum ZDV should be provided. Homosexuality 1066 Infertility is defined as the inability to conceive after how many C. coli D. Hypertension D. risk decreases later in pregnancy. Use of tampoons Harrison’s 18th Ed. intracranial calcifications. 48 A. hepatosplenomegaly D. 280 A. All of the above C. followed by development of pubic hair (pubarche) & later by development reserve” ? of axillary hair (adrenarche). motility and 1072 Condition that leads to early menarche is ? normal sperm morphology percentage of ? Harrison’s 16th Ed. An FSH level < 10 IU/mL on cycle day 3 predicts adequate ovarian oocyte reserve. > 36%. All of the above hypothalamic insensitivity to feedback by steroids that leads to increased secretion of gonadotropins & to menarche. 3033 C. D. Elevated levels of FSH A critical combination of total body weight & percent body fat is associated with development of D. Percent body fat A. 18 months response to clomiphene. 1073 Which of the following determine age of menarche ? 1079 Which of the following is true in males with “primary gonadal Harrison’s 16th Ed. < 13 million/mL. < 9% respectively C. FSH level < 5 IU/mL on cycle day 1 1070 Average time between the beginning of breast development and onset of menses i. FSH level < 10 IU/mL on cycle day 3 B. Malnutrition C.e. > 48 million/mL. Active participation in Normal male fertility is associated with sperm counts of > 48 million/mL. . Nutrition B. and < 9% normal morphology. Harrison’s 16th Ed. and is unexplained 1074 Evidence of ovulation is provided by which of the following ? in ~17% of couples. > 3 ng/mL C. All of the above level should be ? Harrison’s 16th Ed. < 32%. Chronic debilitating disease 1078 Male subfertility is associated with sperm count. > 63%. 3033 B. > 0. Active participation in sports B. All of the above C. < 18 million/mL. > 6% respectively A. Basal body temperature charts A. All of the above At age 10 to 11. Total body weight Harrison’s 16th Ed. Subfertility is seen with sperm counts of < 13 million/mL. with a motility of > 63%. > 12% respectively B. Development of pubic hair (pubarche) D. < 9% respectively D. Urinary ovulation predictor kits 1068 Secondary sexual characteristics in girls include ? Harrison’s 18th Ed. 280 Harrison’s 16th Ed. < 9% respectively B. 24 months Average time between beginning of breast development & onset of menses (menarche) is 2 years. > 6% respectively C. > 48 million/mL. Harrison’s 16th Ed. < 32%. Obesity D. malnutrition & chronic debilitating disease delay menarche. 280 B. 1077 Normal male fertility is associated with sperm count. < 32%. with > 12% showing normal morphology. FSH level < 5 IU/mL on cycle day 3 Harrison’s 18th Ed. D. > 48 million/mL. > 2 ng/mL B. FSH level < 10 IU/mL on cycle day 1 A. sports. 6 months D. female factors in 58%. motility of < 32%. Development of breast buds (thelarche) C. > 63%. 609 Endocrinology MCQ’s FOR MCQ’s FOR MEDICAL MEDICAL PROFESSIONALS PROFESSIONALS BY PROF. which of the A. an estrogen antagonist. Elevated levels of LH D. Thelarche C. Low testosterone level with raised levels of LH & FSH indicate primary gonadal deficiency. Adrenarche 1075 For confirmation of ovulation. They are development of 1076 Which of the following predicts adequate “ovarian oocyte the breast buds (thelarche). Development of axillary hair (adrenarche) Evidence of ovulation is provided by urinary ovulation predictor kits (preovulatory gonadotropin surge). AJAY MATHUR Cardiology 609 Infertility can be attributed primarily to male factors in 25%. 2198 A. mid-luteal phase progesterone D. Mid-luteal phase progesterone level B. 3033 B. Obesity C. menarche is ? B. or a mid-luteal phase progesterone rise which is usually > 3 ng/mL. < 23 million/mL. < 33 million/mL. 279 A.5 ng/mL following appears first ? Harrison’s 18th Ed. 12 months Evaluation of ovarian reserve is done by measurement of FSH on day 3 of menstrual cycle or in C. > 36%. basal body temperature charts. 2198 deficiency” ? A. < 32%. motility and normal sperm morphology percentage of ? 1071 Conditions that can delay menarche include all except ? Harrison’s 16th Ed. > 1 ng/mL A. < 9% respectively Obese girls have earlier menarche than girls with normal body weight. > 48 million/mL. 279 A. Pubarche D. Chronic debilitating disease D. Low testosterone level C. 279 1069 Among the secondary sexual characteristics in girls. Malnutrition B. > 12% respectively D. Active participation in sports A. first secondary sexual characteristics appear in girls. 2198 Harrison’s 16th Ed. Levonorgestrel B. All of the above D. 280 A. Deficiency of fructose in ejaculate B. Low prolactin level In addition to prevention of pregnancy. Deficiency of galactose in ejaculate D. Hess syndrome C. 2 to 6 months D. tubal ligation reduces the risk of ovarian cancer. Changing cervical mucus B. Low FSH level D. Gonadotropins B. Norgestimate Harrison’s 16th Ed. Urinary tract infections B. Norethindrone A. Low LH level C. AJAY MATHUR Endocrinology 1080 Which of the following is false in males with “secondary 1086 Tubal ligation reduces the risk of ? hypogonadism” ? Harrison’s 16th Ed. Deficiency of glucose in ejaculate A. Suppressing ovulation A. 281 B. Bacterial endocarditis C. 281 Harrison’s 16th Ed. Intracorporeal sperm injection D. All of the above Oral contraceptive pills act by suppressing ovulation. 1088 Copper IUDs inhibit pregnancy primarily through ? 1082 Inspissated secretions precluding normal sperm transport is seen Harrison’s 16th Ed. None of the above D. All of the above Clomiphene citrate is a nonsteroidal estrogen antagonist that increases FSH & LH levels by IUD should not be used in women at high risk for development of STDs or in women at high risk blocking estrogen negative feedback in hypothalamus. changing cervical mucus & altering endometrium. Nonsteroidal estrogen antagonist A. Clomiphene D. Young’s syndrome D. possibly Decreased spermatogenesis due to hypothalamic or pituitary disease show low testosterone & by limiting the upward migration of potential carcinogens. IUDs) or by release of progestins (Progestasert). Altering endometrium C. 280 A. Pulsatile GnRH C. 280 Harrison’s 16th Ed. Intracavitary sperm injection C. often associated with an abnormality of cystic fibrosis transmembrane regulator (CFTR) gene. 280 Harrison’s 16th Ed. Spermicidal effect B. Cervical cancer C. 282 in ? A. 1084 Which of the following drugs has no risk of ovarian 1090 Oral contraceptive pills act by ? Harrison’s 16th Ed. 282 A. 1083 Which of the following about “Clomiphene citrate” is false ? 1089 IUD should not be used in women at high risk for ? Harrison’s 16th Ed. All of the above D. Desogestrel C. 282 A. for bacterial endocarditis. decreased spermatogenesis with low LH & FSH levels. Intracytoplasmic sperm injection D. 1087 After vasectomy. Increases FSH levels B. Bloom syndrome Intrauterine devices (IUDs) inhibit pregnancy primarily through a spermicidal effect caused by a sterile inflammatory reaction produced by the presence of a foreign body in the uterine cavity (copper Young’s syndrome.610 MCQ’s FOR MEDICAL PROFESSIONALS BY PROF. development of azoospermia may be delayed for 2 to 6 months. 6 to 9 months Congenital absence of vas deferens is diagnosed by deficiency of fructose in the ejaculate. Alteration in endometrial lining Harrison’s 16th Ed. Inhibition of implantation C. Pulsatile GnRH restores ovulation in hypothalamic amenorrhea and carries virtually no risk of ovarian hyperstimulation. Ovarian cancer A. 1091 Which of the following progestins is most androgenic ? Harrison’s 16th Ed. can preclude normal sperm transport. 280 A. 282 1085 “ICSI” stands for ? A. Pneumonia D. Norgestimate . 282 hyperstimulation ? Harrison’s 16th Ed. Deficiency of sucrose in ejaculate C. characterized by inspissated secretions. Breast cancer D. Increases LH levels C. Bowen syndrome B. 15 days to 3 months B. 282 ICSI stands for intracytoplasmic sperm injection. development of azoospermia may be delayed 1081 Congenital absence of the vas deferens can be diagnosed by ? for ? Harrison’s 16th Ed. Endometrial cancer B. None of the above 1092 Which of the following progestins is least androgenic ? Harrison’s 16th Ed. It is After vasectomy. Low testosterone level B. A. 1 to 4 months C. 72 hours which menstrual bleeding generally occurs. Levonorgestrel is most androgenic of the progestins & should be venous thromboembolic events. Fixed-dose estrogen-progestin combination Harrison’s 16th Ed. 2 weeks A. 1 year C. 16 % D. 1097 After discontinuation of Depo-Provera. 8 % D. Fixed-dose estrogen-progestin combination A. Progestin only B. Microdose Progestin D. Long-term progestin Norplant requires surgical insertion & is effective for up to 5 years afer insertion. fertility usually returns 1103 Which of the following is part of a reversible male after ? contraceptive ? Harrison’s 16th Ed. Levonorgestrel C. 12 to 18 months C. contraception. 1094 Combination oral contraceptive is administered daily for ? 1100 Unprotected intercourse without regard to time of month carries Harrison’s 16th Ed. (norgestral. Fixed-dose estrogen-progestin combination A. 6 to 12 months B. 4 weeks C. Mifepristone D. may occur. AJAY MATHUR Cardiology 611 B. Ethinyl estradiol and Levonorgestrel B. Fifth week . Levonorgestrel C. 7 years Major oral contraceptives are fixed-dose estrogen-progestin combination. 282 A. 24 hours D. 2 to 6 months A. desogestrel. All of the above Microdose progestin-only minipill is appropriate for women with cardiovascular disease or for Ethinyl estradiol + levonorgestrel. norgestimate. 3 weeks B. Phasic estrogen progestin combination A. Injectable progestin-based contraceptives Synthetic progestins like norethindrone. primitive ovary can be which of the following contraceptive formulation ? distinguished from the testis ? Harrison’s 16th Ed. 18 to 24 months D. All of the above C. Phasic estrogen progestin combination C. All of the above Intramuscular Depo-Provera is effective for 3 months. 2 % C. Long-acting testosterone B. Microdose Progestin only C. 282 A. Any of the above D. 3 years D. Progestin-only pills are administered continuously. Fixed-dose estrogen-progestin combination Harrison’s 16th Ed. 1093 Oral contraceptive formulations include all except ? 1099 Contraception by injectable Norplant is effective for what Harrison’s 16th Ed. norethisterone) is part of an effective male contraceptive or male pill. 282 what incidence of pregnancy ? A. Estrogens only C. Norethindrone B. desonorgestrel. 282 Harrison’s 16th Ed. 25 % 1095 Which of the following oral contraceptive formulation is administered continuously ? 1101 Emergency contraceptives should be used within how many hours Harrison’s 16th Ed. Phasic estrogen progestin combination B. 282 B. 12 hours C. Gestodene D. 282 Harrison’s 16th Ed. 282 Harrison’s 16th Ed. 282 after unprotected intercourse ? A. 282 duration ? A. gestodene and drospirenone Major advantage of injectable progestin-based contraceptives is lack of increased arterial & have a less androgenic profile. levonorgestrel and mifepristone are approved for postcoital women who cannot tolerate synthetic estrogens. Progestin only B. 2198 A. Progestin D. GnRH antagonist C. 282 B. 36 hours Combination formulations are given daily for 3 weeks followed by a week of no medication during D. Phasic estrogen progestin combination A. phasic estrogenprogestin combination & progestin only. 1098 Increased incidence of gallbladder disease is seen after use of 1104 In which week of developing foetus. 282 B. But increased gallbladder disease & decreased bone density avoided in hyperandrogenic states. 611 Endocrinology MCQ’s FOR MCQ’s FOR MEDICAL MEDICAL PROFESSIONALS PROFESSIONALS BY PROF. 1 week Harrison’s 16th Ed. 1096 Which of the following oral contraceptive formulation is appropriate for women with cardiovascular disease ? 1102 Which of the following is used as emergency contraceptive ? Harrison’s 16th Ed. 5 years D. Return of fertility after discontinuation may Combination of a long-acting testosterone preparation with GnRH antagonist or a progestin be delayed for up to 12 to 18 months. Unprotected intercourse without regard to time of month carries 8% incidence of pregnancy which can be reduced to 2% by the use of emergency contraceptives within 72 hours of unprotected intercourse. Shorter Harrison’s 16th Ed. cycle averages 28 ± 3 days & mean duration of flow is 4 ± 2 days. 28 ± 5 days D.000 In women of reproductive age. the number peaking at about 7 million oogonia by 5th to 6th month of gestation. 2198 C. pattern of menstrual cycles is variable. Normal duration B.000 C. Luteal phase D. and only 1 . ~ 400. Gonads exist in an undifferentiated state until the seventh week of fetal life. 4 ± 4 days A. 2200 1110 Which of the following statements about FSH is false ? Harrison’s 16th Ed. 28 ± 4 days C. 2200 A. 5th to 6th month of gestation B. Normal duration B. Secretory phase Preceding menopause. 2199 B. 2200 1109 Luteal phase of menstrual cycle is also called ? A. Secretory or luteal phase of menstrual cycle is progesterone driven later phase. Harrison’s 16th Ed. 2200 A. Follicular phase D. Harrison’s 16th Ed. interval between menses becomes ? Harrison’s 16th Ed. Longer A.000 D. 1111 Length of menstrual cycle is defined as ? Harrison’s 16th Ed. Plasma levels of progesterone fall B.000 1114 Menstrual cycles at menarche and near the onset of menopause Germ cells decrease in number through atresia so that approximately 1 million remain at birth.000 at menarche& only a few remain at menopause.612 MCQ’s FOR MEDICAL PROFESSIONALS BY PROF. ~ 2 million C. 2199 B. From onset of one menstrual bleeding episode to onset of next C. ~ 200. oogonia are no longer present in the ovary. All of the above increase . FSH secretion is inhibited progressively as estrogen levels D. AJAY MATHUR Endocrinology B. are ? 400. Secretory phase Longer menstrual cycles (usually anovulatory) occur at menarche and near the onset of menopause. at which time the primitive ovary can be distinguished from the testis. Pregnancy Length of the menstrual cycle is defined as the time from the onset of one menstrual bleeding episode to onset of the next. ~ 600. Proliferative phase C. but the interval between menses D. mean duration of menstrual flow averages ? At birth. 2198 A. 4 ± 2 days 1107 How many germ cells are present in ovary at menarche ? B. None of the above Proliferative or follicular phase of menstrual cycle is estrogen driven intital phase. Menarche D. Inhibin selectively suppresses FSH B. 2200 1105 Maximum number of germ cells are present in ovary at ? Harrison’s 16th Ed. From end of one menstrual bleeding episode to onset of next A. None of the above usually becomes shorter. From onset of one menstrual bleeding episode to end of next D. Seventh week levels of estrogen D. ~ 3 million D. ~ 1 million B. 2200 A. Sixth week D. 4 ± 3 days Harrison’s 16th Ed. 4 ± 5 days B. Proliferative phase C. D. ~ 800. Birth C. Shorter 1108 Follicular phase of menstrual cycle is also called ? Harrison’s 16th Ed. 1112 In an adult female. LH secretion is suppressed maximally by sustained high C. menstrual cycle averages ? Harrison’s 16th Ed. Circulating levels of FSH increase C. 2325 A. D. Longer A. 28 ± 2 days A. 1115 Preceding menopause. Feedback of estrogen involves hypothalamus & pituitary C. 2200 1106 How many germ cells are present in ovary at birth ? Harrison’s 17th Ed. Plasma levels of estrogen fall A. 1116 Which of the following is true at the end of a menstrual cycle ? Harrison’s 16th Ed. 28 ± 3 days B. Ovary contains a finite number of germ cells. Any of the above C. Eighth week LH secretion is suppressed maximally by sustained low levels of estrogen and is enhanced by a rising level of estradiol denoting a positive feedback.2 million germ cells remain. ~ 4 million 1113 In an adult female. From end of one menstrual bleeding episode to end of next B. as follicular recruitment is hastened by increases in FSH. Any of the above C. A. Plasma progesterone increases 1129 Menopause is the consequence of ? D. D. C. After midcycle Glandular growth of the endometrium is is mediated by estrogen during the proliferative phase. 2200 1121 Plasma progesterone level begins to rise at what time ? A. Just after ovulation development of the next follicle (usually in the contralateral ovary) & the next menstrual cycle. Harrison’s 17th Ed. Resistance of ovaries to gonadotropins of the luteal phase? B. D. 2200 B. 16 to 23 hours menstrual bleeding. Contralateral ovary Harrison’s 16th Ed. 13 to 15 days 1124 Following one menstrual cycle. Just prior to midcycle C. Any of the above Plasma progesterone level begins to rise just prior to midcycle & facilitates the positive feedback 1128 Which of the following about menopause is false ? action of estradiol on LH secretion. Intense vasospasm occurs in the spiral venules after the onset of the LH surge ? Harrison’s 16th Ed. 5 to 7 days C. 2200 A. Intense vasospasm occurs in endometrial capillaries A. 2334 A. 2200 C. None of the above B. Tortuosity of the glands Harrison’s 16th Ed. At midcycle D. Inhibin A levels are low in the follicular phase 1119 Ovulation from the dominant follicle occurs how many hours after the LH peak ? B. 2200 C. At the time of ovulation Near the end of luteal phase. Inhibin A levels reach a peak in the luteal phase Harrison’s 16th Ed. 12 to 14 hours Inhibin A levels are low in the follicular phase but reach a peak in the luteal phase. Just before ovulation D. 2200 the “midcycle LH surge” ? A. 2200 C. Any of the ovaries B. Glandular secretion C. 16 to 23 hours are increased in the follicular phase & low in the luteal phase. Smoking accelerates menopausal transition by 2 years A. Diagnosed retrospectively >12 months of amenorrhea of luteal phase ? Harrison’s 16th Ed. plasma estradiol levels begin to rise as the result of Harrison’s 16th Ed. Both ovaries A. Inhibin B levels reach a peak in the luteal phase B. development of the next follicle is usually in the ? ~ 8 to 10 days prior to midcycle LH surge. 613 Endocrinology MCQ’s FOR MCQ’s FOR MEDICAL MEDICAL PROFESSIONALS PROFESSIONALS BY PROF. None of the above Harrison’s 17th Ed. None of the above C. D. Plasma progesterone decreases B. Any of the above 1125 Which of the following statements about “Inhibin” is false ? Just before ovulation. AJAY MATHUR Cardiology 613 1117 Plasma estradiol levels begin to rise about how many days before Harrison’s 16th Ed. 10 to 12 hours D. Plasma FSH decreases D. Inhibin B levels C. 2200 C. 2200 A. estradiol secretion reaches a peak and then falls. Permanent cessation of menstruation 1122 Which of the following is false about hormone status at the onset B. Inhibin B levels are increased in the follicular phase A. except ? Harrison’s 16th Ed. 11 to 13 days D. D. endometrial desquamation and C. 2200 estradiol formation by the granulosa cells of the dominant follicle. Same ovary 1118 Estradiol secretion reaches a peak at ? B. 8 to 10 days D. Glandular growth of the endometrium B. 2334 1123 Which of the following is false about hormone status near the end A. All of the above B. Intense vasospasm occurs in the spiral arterioles 1120 Ovulation from the dominant follicle occurs how many hours B. Harrison’s 16th Ed. 2200 B. progesterone & estrogen levels fall & FSH levels begin to rise to initiate C. 12 to 14 hours Intense vasospasm caused by locally synthesized prostaglandins occurs in spiral arterioles supplying blood to endometrium causing ischemic necrosis. Plasma estrogen decreases A. Curling of the spiral arterioles A. Exhaustion of ovarian follicles . 24 to 38 hours 1127 Secretory phase of menstrual cycle is characterized by all Ovulation from the dominant follicle occurs 16 to 23 hours after the LH peak and 24 to 38 hours after the onset of the LH surge when the follicular wall ruptures in the area of the stigma. Plasma FSH begins to rise Harrison’s 16th Ed. 10 to 12 hours D. Plasma LH decreases Menopause is the permanent cessation of menstruation due to loss of ovarian follicular function. Harrison’s 16th Ed. 24 to 38 hours 1126 Menstrual bleeding is due to ? Follicular rupture & ovulation occurs 16 to 23 hours after the LH peak. Same B. A. B. 36 Harrison’s 17th Ed. Estrone B. 35 Harrison’s 17th Ed. Hyperestrogenic. Static doses of estrogen & progestin eliminate vasomotor symptoms & restore regular cyclicity. 6 C. All of the above Harrison’s 17th Ed. and Process of exhaustion of ovarian follicles begins in intrauterine life & terminates at menopause. 2334 B. 20 to <30 mIU/mL. < 20 mIU/mL In perimenopause. 2334 Perimenopausal women with irregular or heavy menses or hormonally related symptoms benefit with low- A. 40 B. They reduce volume of menstrual flow. . 2334 A. fair. uterine polyps. as well as estrogen feedback. Shorter Harrison’s 17th Ed. Longer A. None of the above due to ? In menopause. 2334 A. 9 D. and poor likelihood of achieving A. hyperprogestagenic environment D. Estradiol levels fall more than estrone levels C. respectively. hypoprogestagenic environment that leads to an increased incidence of endometrial hyperplasia or carcinoma. B. 6 for perimenopausal menorrhagia in women who smoke or have cardiovascular risk factors. anovulatory cycles produce a hyperestrogenic. Progestin-only formulations or medroxyprogesterone injections are an alternative B. 12 Estradiol is secreted by ovaries and is the most potent estrogen.8 years. 2334 1137 Which of the following statements about hormone levels is false as regards menopause ? A. 45 C. Hyperestrogenic. Estriol C. Nonsteroidal anti-inflammatory agents menses ? D. Most estrone is formed by Menopause is diagnosed retrospectively >12 months of amenorrhea. AJAY MATHUR Endocrinology C. Smoking accelerates the extraglandular conversion of androstenedione in peripheral tissues. Acceleration in luteal phase 1138 Which of the following FSH levels on day 3 of menstrual cycle C. leiomyoma observed among women of perimenopausal age. Estriol (16-hydroxyestradiol) is the main estrogen in urine and arises from 16- hydroxylation of estrone and estradiol. 24 1139 Which of the following treatments is useful in perimenopausal women ? C. Hypoestrogenic. hyperprogestagenic environment D. 30 to < 40 mIU/mL 1133 “Perimenopause” time period precedes final cessation of menses D. Resistance of pituitary to estradiol D. 2334 reflecting peripheral aromatization of adrenal & ovarian androgens. 1130 Menopause is diagnosed retrospectively after how many months 1136 Predominant estrogen formed in menopausal women is ? of amenorrhea ? Harrison’s 17th Ed. 3 B. with mean duration of 4 years. 2334 FSH measurement aid in assessing fertility. 2334 D. 20 to < 30 mIU/mL C. 9 D. intermenstrual intervals shorten significantly D. Any of the above C. Nonhormonal strategies include NSAIDs or endometrial ablation. B. when fertility wanes and menstrual cycle irregularity increases. 3 dose combined oral contraceptives. C. Harrison’s 17th Ed. Progestin-only formulations 1134 “Perimenopause” time period extends up till what time after final C. FSH levels increase more than LH levels D. intermenstrual intervals shorten significantly (typically by 3 days) due to an accelerated follicular phase. although some estrone is also menopausal transition by 2 years. >= 40 mIU/mL by a mean of how many months ? Harrison’s 17th Ed. All of the above In perimenopause. hypoprogestagenic environment C. until the first year after cessation of menses. 30 1135 Perimenopause is characterized by which of the following ? B.614 MCQ’s FOR MEDICAL PROFESSIONALS BY PROF. Reduced inhibin secretion 1132 In perimenopause. Hypoestrogenic. 12 1140 Premature ovarian failure or premature menopause applies to women who cease menstruating before age of ? Perimenopause refers to the time period preceding menopause. 2204 A. Low-dose combined oral contraceptives Onset of perimenopause precedes the final menses by 2 . None of the above D. 48 A. 2334 D. Estradiol A. None of the above A. Levels of <20 mIU/mL. 12 pregnancy. whereas estrone levels are relatively preserved. interval between menses is usually ? Harrison’s 17th Ed. hypoprogestagenic environment Term premature ovarian failure or premature menopause applies to women who cease menstruating before the age of 40 years. 1131 Preceding menopause. produced by ovary. and >=30 mIU/mL measured on day 3 of the cycle indicate a good. estradiol levels fall markedly. Acceleration in follicular phase B. Acceleration in follicular & luteal phase indicate a good likelihood of achieving pregnancy ? Harrison’s 17th Ed. Harrison’s 16th Ed. FSH levels increase more than LH because of the loss of inhibin. 2334 Harrison’s 17th Ed. 2211 C. 2211 A. compared to only 1% of women assigned to placebo. D. Use of a progestin eliminates these risks. Lowers triglycerides C. Endometrial cancer A. Increased risk of fracture A.8 IU/L Postmenopausal Estrogen/Progestin Interventions (PEPI) trial found 24% of women assigned to C. Prolactin A. 2212 Harrison’s 16th Ed.8 1151 In the Heart and Estrogen/progestin Replacement Study (HERS). Breast cancer genitourinary symptoms of menopause ? Harrison’s 16th Ed. adequate calcium (1000 to 1500 mg/day) & vitamin D (400 to 800 IU/ Harrison’s 16th Ed. Vitamin D Estrogens. < 3. B. AJAY MATHUR Cardiology 615 1141 Which hormone is secreted in a pulsatile manner ? D. 300 to 500 mg/day Following hormones are secreted in a pulsatile manner . Calcium D. D. < 1.8 IU/L FSH levels that are persistently > 40 IU/L are diagnostic of ovarian failure. Evaluating women with suspected ovarian failure 1149 Which of the following is a selective estrogen receptor modulator ? B. 400 to 800 IU/day . IU/L suggests hypogonadotropic hypogonadism. All of the above 1153 Which of the following is a “primary” prevention trial for assessing 1147 Dose of vitamin D given to reduce the risk of osteoporosis-related efficacy of hormone therapy in postmenopausal women ? fractures is ? Harrison’s 16th Ed. 2212 D. LH Harrison’s 16th Ed.FSH. Residronate 1143 What value of FSH level is diagnostic of ovarian failure ? C. 2211 C. Urinary bladder cancer C. 200 to 500 IU/day C. GH and prolactin. Vitamin E In the Heart & Estrogen/progestin Replacement Study (HERS). produces all except ? Harrison’s 16th Ed. In diagnosis of hypogonadotropic hypogonadism A. A. Raloxifene Harrison’s 16th Ed. Vitamin D Exogenous estrogen lowers plasma LDL cholesterol & Lp(a). ACTH. 2201 C. All of the above B. Clonidine D. and an LH value < 0. Heart and Estrogen/progestin Replacement Study (HERS) A. < 0. All of the above B.8 IU/L unopposed estrogen for 3 years developed atypical endometrial hyperplasia. 2212 1146 Which of the following reduces the risk of osteoporosis-related A. All of the above A. Colorectal cancer B. 615 Endocrinology MCQ’s FOR MCQ’s FOR MEDICAL MEDICAL PROFESSIONALS PROFESSIONALS BY PROF. 2201 day) intakes reduces the risk of osteoporosis-related fractures. 2201 D. 4 years of combination therapy was associated with a 27% increase in breast cancer risk.8 IU/L D. raises HDL cholesterol levels and triglyceride levels. > 10 IU/L Raloxifene is a selective estrogen receptor modulator (SERM). Estrogen therapy C. 2210 B. Estrogen C. venlafaxine. Lowers lipoprotein(a) B. 800 to 1500 IU/day Harrison’s 16th Ed. Alendronate D. All of the above B. TSH. 100 to 150 mg/day D. Lowers LDL cholesterol fractures ? Harrison’s 16th Ed. > 30 IU/L 1150 Postmenopausal Estrogen/Progestin Interventions trial found ? Harrison’s 16th Ed. 2201 A. < 2. > 20 IU/L C. FSH 1148 Dose of oral calcium given to reduce the risk of osteoporosis- related fractures is ? B. D. 1000 to 1500 mg/day 1142 Plasma gonadotropin measurements are of value in ? Increased physical activity. a premalignant lesion. clonidine. Raises HDL cholesterol A. In diagnosis of polycystic ovarian syndrome (PCOS) Harrison’s 16th Ed. 2212 1145 Which of the following is not useful in controlling vasomotor and A. Increased risk of breast cancer 1144 What value of LH suggests hypogonadotropic hypogonadism ? B. > 40 IU/L A. Estrogen Replacement and Atherosclerosis (ERA) trial C. 100 to 300 IU/day B. LH. gonadotropin-releasing hormone C. 500 to 750 mg/day (GnRH). Increased risk of endometrial cancer Harrison’s 16th Ed. Women’s Health Initiative trial (WHI) B. D. vitamin E are effective for controlling vasomotor and 1152 Exogenous estrogen administration in postmenopausal women genitourinary symptoms in menopause. 2211 B. risk of which of the following cancers was increased ? Harrison’s 16th Ed. Plasma or urine assays of hCG CAN detect pregnancies 8 to 10 days after ovulation. 1 to 3 days B. 366: 409–21 D. A. 2202 In WHI. C. Neck first missed menstrual period.1 years of estrogen-only therapy did not increase risk.8 ng/mL D. Progestagenic C. > 2. ovary secretes androstenedione. Chest 1163 Which of the following is not a feature of cervical mucus that D. Urinary incontinence A. Cervical mucus A. 2201 Lancet 2005.616 MCQ’s FOR MEDICAL PROFESSIONALS BY PROF. 366: 409–21 A. women assigned to receive combination hormones for an average of 5. testosterone & dehydroepiandrosterone.  In WHI. Copious B. 5 to 7 days feeling of heat in felt over ? Lancet 2005. 2212 C. Androgenic 1166 Under normal conditions. sudden C. 2202 C. 3 minutes D. 8 to 10 days A. Oestriol C. Testosterone Lancet 2005. Trophoblastic cells of the placenta 1154 Estrogen-progestin therapy reduces risk of ? B. All of the above Presence of viscous cervical mucus that does not stretch or fern and of predominantly intermediate cells on vaginal cytology or demonstration of a secretory epithelium in an endometrial biopsy 1158 Vaginal oestrogen does not improve which of the following ? during the luteal phase is typical of progesterone secretion. 2201 1156 The average hot flush lasts about ? Lancet 2005. 366: 409–21 C. although no benefit was seen with 7 years of estrogen-only therapy. 2 minutes C. Gallbladder disease 1162 By plasma or urine hCG assays. Breast cancer C.  likely to develop breast cancer than women assigned to placebo. before the B. Vaginal dryness 1165 What plasma progesterone level suggests successful ovulation B. Estradiol valerate D. > 0.3 ng/mL Lancet 2005. Estrogen in the Prevention of Reinfarction Trial (ESPRIT) C. AJAY MATHUR Endocrinology D. estrogen-progestin was associated with a significant 44% reduction in colorectal cancer over a 5. Abundant cornified squamous epithelial cells B.6 ng/mL 1159 Tibolone has which of the following actions ? C. 366: 409–21 A. All of the above B. 3 to 5 days 1155 In hot flush. Coronary heart disease many days after ovulation ? Harrison’s 16th Ed. the ovary secretes which of the D. B. Oestrogenic Plasma progesterone level > 10 µmol/L (> 3 ng/mL) suggests successful ovulation and adequate corpus luteum function. 2202 1160 Phyto-oestrogens have oestrogen-like effects because of A. Pyknotic nuclei D. Lancet 2005.0 ng/mL A. Mature vaginal epithelial cells B. Androstenedione preferential binding to which oestrogen receptor ? B. the WHI showed a trend toward a reduction in breast cancer risk with 1161 Human chorionic gonadotropin (hCG) is secreted by ? estrogen alone. Clear D. All of the above following androgen ? Harrison’s 16th Ed. pregnancy can be detected how D. All of the above B. > 3. a characteristic manifestation of menopause.6-year period. All of the above indicates adequate estrogen production ? Harrison’s 16th Ed. All of the above B. Face hCG is secreted by trophoblastic cells of placenta into maternal plasma & excreted in urine. Exhibiting arborization or ferning C. 2202 A. 4 minutes 1164 Vaginal cytology findings that confirm the presence of adequate 1157 Oral oestrogen preparation used in HRT include ? estrogen levels include all except ? Harrison’s 16th Ed. . 366: 409–21 D. Uterine myocardium Harrison’s 16th Ed.6 years were 24% more D. > 1. Thick A. Colorectal cancer D.  D. 366: 409–21 A. Predominantly intermediate cells C. Oestrone sulphate A. Vaginal atrophy and adequate corpus luteum function ? Harrison’s 16th Ed. 1 minutes B. but 7.  Under normal conditions. Indeed. Dehydroepiandrosterone A. although it is unclear whether this finding would pertain to formulations of estrogen other than conjugated equine estrogens or to treatment durations longer than 7 years. Harrison’s 16th Ed. ependymomas. All of the above D. Precocious pseudopuberty B. McCune-Albright sequence of pubertal development. 2202 Table 326-1 A. Ovarian teratomas C. tuberous sclerosis & neurofibromatosis. 2202 A. Estrogen D. and estrogen-containing medications. Russell-Silver syndrome. Head injury the age of ? C. virilization in girls or Ovarian cysts or tumors that secrete estrogen (granulosa-theca cell tumors) are the most frequent feminization in boys. 2202 A. hypothyroidism. None of the above Gonadotropin-independent precocious pseudopuberty occurs when girls undergo feminization as a consequence of enhanced estrogen formation but do not ovulate or develop cyclic menses. D. 11-hydroxylase (CYP11B1) C. 1170 Which of the following is a type of “isosexual precocious puberty in girls” ? Harrison’s 16th Ed. Gonadotropin-dependent A. None of the above D. including increased secretion of gonadotropins & ovulatory syndrome. meningitis. True precocious puberty A. Progesterone Primary hypothyroidism is sometimes associated with increased secretion of FSH. LH B. 1178 Hormone responsible for development of precocious 1172 Which of the following is of use in treatment of true precocious pseudopuberty in primary hypothyroidism is ? puberty ? Harrison’s 16th Ed. 2145 Harrison’s 16th Ed. FSH A. C. All of the above A. GnRH C. 1173 Which of the following can cause true precocious puberty ? 1179 Which of the following can be a feature of incomplete isosexual Harrison’s 16th Ed. 11 years 1174 Most common form of congenital adrenal hyperplasia (CAH) is 1168 Puberty is considered precocious if menarche occurs before the due to impairment of ? age of ? Harrison’s 16th Ed. Anovulatory cycles C. McCune-Albright syndrome B. Enhanced estrogen formation B. Early normal pubertal development C. All of the above Gonadotropin-dependent true precocious puberty is characterized by an early but otherwise normal Precocious pseudopuberty can be caused by ovarian tumors. 9 years 10% causes are due to organic brain diseases like brain tumors (hypothalamic gliomas. Russell-Silver syndrome D. Heterosexual precocity occurs when sexual characteristics are not according to genetic sex i. AJAY MATHUR Cardiology 617 1167 Puberty is considered precocious if breast budding begins before B. head injury. Regular cyclic menses D. Primary Hypothyroidism C. 10 years astrocytomas. estrogen secretion. Dysgerminomas B. Incomplete isosexual precocity C. Cystadenomas Feminization in girls or virilization in boys are termed isosexual precocity. High levels of TSH caused by hypothyroidism may also stimulate FSH receptor. 2202 A. encephalitis. inducing ovarian GnRH analogues suppress gonadotropins & inhibit estrogen synthesis thus blocking precocious puberty. 8 years Constitutional or idiopathic precocious puberty accounts for 90% cases of true precocious puberty. 10 years D. CYP21A2 (21-Hydroxylase) A. hydrocephalus.e. adrenal tumors. Neurofibromatosis Harrison’s 16th Ed. 3-HSD2 D. Granulosa-theca cell tumors D. 2202 Harrison’s 16th Ed. 2202 precocity ? Harrison’s 16th Ed. Ovulatory menstrual cycles B. 9 years C. Puberty is precocious if breast budding begins before 8 years of or if menarche occurs before age 9. Encephalitis . menstrual cycles. B. 617 Endocrinology MCQ’s FOR MCQ’s FOR MEDICAL MEDICAL PROFESSIONALS PROFESSIONALS BY PROF. All of the above D. Gonadotropin-independent A. Feminization in boys C. Both of the above D. GnRH analogues B. 8 years B. 2202 Harrison’s 16th Ed. Virilization in girls B. 2202 D. 11 years Most common form of CAH (95%) is a result of impairment of CYP21A2. 17-hydroxylase (CYP17) B. germinomas & hamartomas). 1175 Which of the following is false about precocious pseudopuberty ? 1169 Heterosexual precocity is defined as ? Harrison’s 16th Ed. 2202 A. Gonadotropins C. 1171 Which of the following is false about true precocious puberty ? 1177 Precocious pseudopuberty can be caused by ? Harrison’s 16th Ed. 2202 A. 2202 1176 Most frequent cause of precocious pseudopuberty is ? Harrison’s 16th Ed. 2202 Harrison’s 16th Ed. cause of precocious pseudopuberty. isosexual precocity ? Harrison’s 16th Ed. Premature pubarche B. & predictable. 2203 B. 2202 1186 Which of the following is false about hypomenorrhea ? Harrison’s 16th Ed. 2203 A. duration or hypothalamic-pituitary-ovarian patterns required for ovulatory cycles. D. 150 to 185 mL C. 2203 1187 Polymenorrhea is defined when regular menstruation occurs by A. It requires no treatment & patients enter puberty around the average time. 27 Polymenorrhea refers to occurrence of regular menstruation more frequently than 21 days. Interruption of normal sequence of follicular & luteal phases A. 2203 1188 Which of the following about dysfunctional uterine bleeding is false ? A. Regular. Duration ovarian patterns C.618 MCQ’s FOR MEDICAL PROFESSIONALS BY PROF. cyclic. Regular. 35 to 80 mL A. 2203 in which of the following from a normal menstrual cycle ? A. 2203 A. Estrogen breakthrough bleeding C. 2203 Harrison’s 16th Ed. 23 D. predictable menstruation characterized by spotting or light bleeding is termed hypomenorrhea and is due to obstruction of the outflow tract as from intrauterine synechiae or 1181 Which of the following is recommended treatment of incomplete scarring of the cervix. Unpredictable in amount C. amount from the pattern observed during a normal menstrual cycle. Treatment with estrogens A. Permanent disruption of synchronous hypothalamic-pituitary- B. Cyclic B. Unpredictablein duration Virilization in a prepubertal female is usually due to congenital adrenal hyperplasia or to androgen D. 2203 B. 2203 B. Premature pubarche C. 200 to 230 mL D. Premature adrenarche A. Predictable D. 2203 A. 1190 Most common type of primary dysfunctional uterine bleeding is 1184 Average blood loss during a menstrual cycle is ? due to ? Harrison’s 16th Ed. regular. Painful secretion by an ovarian or adrenal tumor. Unpredictable menstruation C. 21 C. None of the above Clitoromegaly is a feature of syndromes of virilization. Abnormalities of the uterus Harrison’s 16th Ed. Frequency C. Premature adrenarche B. Spotting or light bleeding D. Treatment with glucocorticoids how many days apart ? Harrison’s 16th Ed. Unpredictable in onset D. 25 Usually. All of the above D. Clitoromegaly D. 2203 B. Amount D. 1182 Virilization in a prepubertal female can be due to ? Harrison’s 16th Ed. Most common type of DUB is estrogen breakthrough bleeding 1185 Normal menstrual bleeding with ovulatory cycles is characterized 1191 Primary amenorrhea is failure of menarche by age of ? by all except ? Harrison’s 16th Ed. cyclic D. Premature thelarche A. 80 to 135 mL B. Androgen secretion by adrenal tumor B. No treatment C. 1180 Which of the following is not a feature of incomplete isosexual It is frequently associated with discomfort (dysmenorrhea). Premature thelarche Harrison’s 16th Ed. Congenital adrenal hyperplasia Harrison’s 16th Ed. None of the above Average blood loss during a normal menstrual cycle is 35 to 80 mL. 1189 Which of the following is the cause of dysfunctional uterine bleeding ? 1183 Abnormal uterine bleeding refers to a bleeding pattern that differs Harrison’s 16th Ed. incomplete isosexual precocity is self-limited & resolves spontaneously. Painless In incomplete isosexual precocity. Estrogen withdrawal bleeding B. premature development of a single pubertal event occurs. Any of the above C. Androgen secretion by ovarian tumor A. AJAY MATHUR Endocrinology A. Progesterone breakthrough bleeding D. 13 . Spontaneous C. Any of the above C. cyclic. Treatment with ACTH B. precocity ? Harrison’s 16th Ed. Any of the above Dysfunctional uterine bleeding occurs in women who have a transient disruption of synchronous Abnormal uterine bleeding is defined as any bleeding pattern that differs in frequency. Normal menstrual bleeding with ovulatory cycles is spontaneous. 3 months A. 12 months D. 2204 A. Type 2 diabetes mellitus its effects on insulin resistance & plasma lipids leading to increased possibility of cardiovascular disease. Galactorrhoea deficiency ? Harrison’s 16th Ed.XX) subjects are women with mullerian agenesis Harrison’s 16th Ed. Ovary D. 16 1192 Secondary amenorrhea is failure of menses for ? 1198 DiGeorge syndrome consists of which of the following ? Harrison’s 16th Ed. which of the following systems is affected ? 17  -Hydroxylase deficiency is an autosomal recessive disorder characterized by primary N Engl J Med 2005. Enteropathy A. irrespective of presence or absence of secondary sexual characteristics. AJAY MATHUR Cardiology 619 B. enteropathy and is X-linked. C. 2204 B. Primary hypogonadism C.352:1223-36 Destruction of endometrium due to vigorous curettage is called Asherman’s syndrome. Karyotype in women with Mayer-Rokitansky-Kuster-Hauser syndrome is 46. Hypothyroidism Harrison’s 16th Ed.XX C. Blindness 1195 The tissue affected in Asherman’s syndrome is ? C. 2203 A. 46. 2237 i. 2237 Harrison’s 16th Ed. Primary hypogonadism “Syndrome XX” is the term proposed for PCOD since it involves the metabolic millieu of the body through C. . Sexual infantilism D. A. Developed secondary sex characteristics IPEX syndrome consists of immunodysregulation. A. Panhypopituitarism B. 14 D. Primary amenorrhea Published in 1935 in Am J Obstet Gynecol. Vitiligo B. 2237 Harrison’s 16th Ed. and a characteristic polycystic appearance to their ovaries was mentioned. 46. which D.XX B. Vitiligo D. Müllerian agenesis or Mayer-Rokitansky-Kuster-Hauser syndrome is associated with mutations in WNT4 gene. Endometrium Congenital rubella syndrome consists of type 1 diabetes mellitus & hypothyroidism. All of the above B.352:1223-36 amenorrhea. 619 Endocrinology MCQ’s FOR MCQ’s FOR MEDICAL MEDICAL PROFESSIONALS PROFESSIONALS BY PROF. 2204 C. Hypoparathyroidism B. Hypertension 1203 In woman with polycystic ovary syndrome. Hypoparathyroidism D. 1199 Wolfram’s syndrome consists of which of the following ? 1193 Karyotype in Mayer-Rokitansky-Kuster-Hauser syndrome is ? Harrison’s 16th Ed. Obesity B. 6 months B. obesity. and hypertension. hirsutism. observations in seven women with amenorrhea. 48. All of the above C. D. type 1 diabetes mellitus & panhypopituitarism. Congenital diabetes insipidus C.XX D.e. 45. 15 Kearns-Sayre syndrome consists of hypoparathyroidism. including cyclic ovulation are present. Metabolic health Harrison’s 16th Ed.XX karyotype D. 2237 1194 Which of the following is false about Mayer-Rokitansky-Kuster. Amenorrhea is failure of menarche by age 15. A. Polyendocrinopathy Hauser syndrome ? Harrison’s 16th Ed. absence or hypoplasia of the vagina. 9 months C. Panhypopituitarism C. A. primary hypogonadism. Type 1 diabetes mellitus D. Cardiovascular health A. Vagina of the following was not noted ? N Engl J Med 2005. Autosomal dominant D.XX Wolfram’s syndrome consists of congenital diabetes insipidus & diabetes mellitus. C. polyendocrinopathy. sexual infantilism. Immunodysregulation B. Phenotypically normal females C. Panhypopituitarism B. or absence of menstruation for 6 months in a woman with previous periodic menses. Hirsutism A.XX. 2204 A. Hyperthyroidism A. 2237 C. Amenorrhea 1196  -hydroxylase Which of the following is not a feature of 17 B. Reproductive system 1197 Features of Kearns-Sayre syndrome include all except ? B. D. Panhypopituitarism DiGeorge syndrome is hypoparathyroidism due to glandular agenesis & mucocutaneous candidiasis. Female secondary sex characteristics with normal ovarian function. 1200 IPEX syndrome consists of which of the following ? Harrison’s 16th Ed. Abnormal ovarian function 1201 Congenital rubella syndrome consists of ? Mayer-Rokitansky-Kuster-Hauser syndrome (46. Cervix uteri 1202 In the original observations of Stein and Leventhal in 1935. 47. C. copus luteum is not formed thus causing estrogen . Presence of corpora albicans 1214 In PCOS.352:1223-36 C. Multiple follicular cysts FSH levels are characteristically reduced in plasma of PCOS patients due to negative feedback of estrogens on pituitary. C. Increased formation of extraglandular estrogen D.352:1223-36 D.352:1223-36 Obesity along with elevated adrenal androgens lead to increased formation of extraglandular estrogen which stimulates LH secretion and depresses FSH secreation by pituitary. All of the above 1213 Which of the following is false in PCOS ? N Engl J Med 2005. Androgenic alopecia 1216 Chronic anovulation with estrogen present is found in all except Acanthosis nigricans is the manifestation of hyperinsulinemia.352:1223-36 C.00 C. C.352:1223-36 B. Obesity SHBG is synthesized by liver under the influence of androgens & insulin. Hyperandrogenism 1211 In most women with PCOS. Hypothyroidism A. LH levels are increased in PCOD due to increased pulse frequency of GnRH A. First child birth N Engl J Med 2005. Decreased fertility D. Oligoovulation or anovulation PCOD symptoms begin around menarche which comes at the expected time. Due to decreased levels B. Fewer than nine menses per year A.352:1223-36 D.75 N Engl J Med 2005. 1. Early and stimulation by excess extraglandular estrogen. B. Isolated hypogonadotropic hypogonadism N Engl J Med 2005. Brenner tumors 1209 Cutaneous manifestations of hyperandrogenemia in polycystic ovary syndrome include all except ? B. Hyperplastic theca and stroma D. Hyperandrogenemia may occur after variable time thereafter.75 B. Increased LH pulse frequency Harrison’s 16th Ed. Increased pulse frequency of hypothalamic GnRH 1207 Which of the following is not a feature of ovary in PCOD ? B. unbound testosterone is available in plasma leading to hyperandrogenemia & hyperandrogenism. Reduced hepatic synthesis of sex hormone . Hirsutism D. menarche occurs ? D. Any of the above A. Dysfunctional uterine bleeding D. Oligomenorrhea or amenorrhea C. which of the following play a role ? (SHBG) Harrison’s 16th Ed. Menarche deficiency B. Adult-onset adrenal hyperplasia due to partial 21-hydroxylase A. Hyperinsulinemia B. Krukenberg tumors B.25 A. 1. Elevated adrenal androgens of SHBG. Late N Engl J Med 2005. Acne Isolated hypogonadotropic hypogonadism is a condition of chronic anovulation with estrogen C. Kallmann syndrome N Engl J Med 2005. 2205 D. 2. 2205 1208 Anovulatory cycles in PCOD may cause ? A.352:1223-36 B. Increased FSH pulse frequency A.620 MCQ’s FOR MEDICAL PROFESSIONALS BY PROF. LH to FSH ratio in plasma is characteristically greater Corpora albicans is the degenerating corpus luteum which is not formed in PCOD due to anovulation than ? Harrison’s 16th Ed.binding globulin PCOD. Sclerotic ovary D. Fewer than four menses per year B. Fewer than ten menses per year B. Reduced level of luteinizing hormone Harrison’s 16th Ed. All of the above A. D. A. Following oral contraceptive pill intake A. 2205 A. ? Harrison’s 16th Ed. Acanthosis nigricans absent. Increased frequency of LH pulses D.progesterone imbalance. Synergistic action of insulin with LH 1206 For the initiation and perpetuation of chronic anovulation in C. 2205 1210 Symptoms of polycystic ovary syndrome usually begin around ? A. First sexual contact . excess of free. At the expected time 1205 Oligomenorrhea refers to ? C. Fewer than six menses per year 1212 Enhanced androgen production in PCOS is due to ? N Engl J Med 2005. Harrison’s 16th Ed. Oligomenorrhoea B. AJAY MATHUR Endocrinology 1204 Which of the following is not a feature of PCOS ? C. 0. All of the above 1215 Chronic anovulation with estrogen present is found in all except ? Due to anovulation. 2205 Characteristically. 2204-5 C. 338:1129 Estrogen is anti-androgenic. 2205 C. Carbamazepine A. Carbamazepine A. Human menopausal gonadotropin (hMG) “Moebius sequence” ? N Engl J Med 1998. Lithium Harrison’s 16th Ed. Misoprostol B. Vaginal carcinoma C. Clomiphene promotes ovulation. Estrogen suppresses LH and thus ovarian androgen production. 1218 If pregnancy is desired by a woman suffering from PCOS. 2205 A. Lithium 1222 Which of the following drugs has antiandrogen effects ? C. there is increased prevalence of endometrial hyperplasia and carcinoma due to the Insulin-sensitizing drugs improve fertility in women with PCOS. Valproic acid C. A. Human menopausal gonadotropin (hMG) 1228 Teratogenic effect of which of the following drugs produces C. Carbamazepine N Engl J Med 2005. GnRH (gonadorelin) “Dandy–Walker syndrome” ? D. Medroxyprogesterone acetate D. All of the above D. All of the above In PCOS. analogue of spironolactone with antimineralocorticoid and antiandrogenic activities. Warfarin A. Warfarin B.352:1223-36 Harrison’s 16th Ed. persistent stimulation of endometrial tissue by estrogen without progesterone-induced inhibition of proliferation and differentiation to secretory endometrium. Clomiphene D. Estrogen also enhances hepatic Kallmann syndrome is Isolated hypogonadotropic hypogonadism associated with defects of smell production of SHBG thereby reducing free and unbound plasma testosterone. Tetracycline D. Estrogen 1229 Teratogenic effect of which of the following drugs produces “aplasia C. Angiotensin-converting–enzyme inhibitors 1221 FSH secretion in PCOS can be enhanced by ? B. 621 Endocrinology MCQ’s FOR MCQ’s FOR MEDICAL MEDICAL PROFESSIONALS PROFESSIONALS BY PROF. Drospirenone 1226 Teratogenic effect of which of the following drugs produces Medroxyprogesterone acetate treatment prevents development of endometrial hyperplasia and carcinoma by bringing about withdrawal menses. 1225 Teratogenic effect of which of the following drugs produces 1219 If a woman having PCOS is not hirsute and does not desire “Neural-tube defects” ? pregnancy. Tetracycline All the three drugs induce ovulation. 2205 A. Thalidomide B. Lithium Harrison’s 16th Ed. All of the above D. Angiotensin-converting–enzyme inhibitors treatment of choice is ? B. treatment of choice is ? N Engl J Med 1998. Carbamazepine D. Clomiphene 1227 Teratogenic effect of which of the following drugs produces D. Combined estrogen-progestogen therapy C. Medroxyprogesterone acetate B. Cyproterone acetate D. 2205 “Neural-tube defects” ? N Engl J Med 1998.338:1129 A. A. 1217 If pregnancy is desired by a woman suffering from PCOS. Misoprostol B. Gonadorelin C. Clomiphene D. Metformin B. Combined estrogen-progestogen therapy C. Warfarin B. Cystic teratomas Cyproterone acetate is antiandrogenic by inhibiting androgens from binding to androgen receptors. AJAY MATHUR Cardiology 621 C. 1224 Teratogenic effect of which of the following drugs produces ovulation can be induced with which of the following drugs ? Harrison’s 16th Ed. Purified FSH (Urofollitropin) B. All of the above D. which 1223 Chronic anovulation is associated with an increased risk of ? of the following drugs is useful ? N Engl J Med 2005. Thalidomide C. Cyproterone acetate D. 2205 C. A. Angiotensin-converting–enzyme inhibitors B. Human menopausal gonadotropin (hMG) A.338:1129 Harrison’s 16th Ed. All of the above N Engl J Med 1998. “Ebstein’s anomaly” ? N Engl J Med 1998. Thiazolidinediones C.352:1223-36 A. Drospirenone is an due to a single gene defect in the X-linked KAL gene.338:1129 1220 If a woman having PCOS is hirsute and does not desire pregnancy.338:1129 Clomiphene citrate is antiestrogenic and induces rise in FSH and LH. Cervical carcinoma B. Endometrial carcinoma A. Drospirenone cutis” ? . Mucous cystadenomas D. AJAY MATHUR Endocrinology N Engl J Med 1998. Webbed neck C. Dandy–Walker malformations B. Hydantoin C. 8-dehydrocholesterol N Engl J Med 1998. 25- C. since they can be synthesized endogenously. Vitamin D from animal sources is vitamin D3 includes all except ? C. 2374 1235 Fetal hydantoin syndrome caused by teratogenic effect of “Phenytoin” includes all except ? A. Methimazole C. Misoprostol A. Spontaneous abortion 1241 The half-life of 25(OH)D is approximately ? Harrison’s 17th Ed.3 weeks.338:1129 Vitamin D & its metabolites are hormones & hormone precursors rather than vitamins. A. Nonsteroidal antiinflammatory drugs 1238 Vitamin D is better related to which of the following ? Harrison’s 17th Ed. Vitamin D from plant sources is vitamin D2 1234 Retinoid embryopathy caused by teratogenic effect of “Isotretinoin” B. Enzyme 1232 Teratogenic effect of which of the following drugs produces D.2 weeks A.3 weeks B.6 weeks C. Carbamazepine D. 2373 D. Nonsteroidal antiinflammatory drugs B. 1 . Both have equivalent biologic activity & are activated equally by vitamin D hydroxylases.338:1129 A. Hearing loss D. 5-dehydrocholesterol D. Nonsteroidal antiinflammatory drugs B. Hypoplasia of nails Vitamin D from plant sources is vitamin D 2 whereas that from animal sources is vitamin D 3. 1 .338:1129 A. 2373 B. Vitamin C. Micrognathia 1242 The formation of the mature Vitamin D hormone occurs in ? 1236 Fetal warfarin syndrome caused by teratogenic effect of “Warfarin” Harrison’s 17th Ed. 2 . D2 has more biologic activity than D3 N Engl J Med 1998. Angiotensin-converting enzyme inhibitors A. a photochemical cleavage results in formation of vitamin D from 7-dehydrocholesterol. Agenesis of corpus callosum A.338:1129 N Engl J Med 1998. 2374 includes all except ? A. Diethylstilbestrol 1240 Which of the following statements about Vitamin D is false ? Harrison’s 17th Ed. Danazol 1239 UV radiation of skin leads to synthesis of Vitamin D from ? Harrison’s 17th Ed. B. Epicanthus D.338:1129 B. All of the above “Masculinization of female fetuses” ? N Engl J Med 1998. 4 . Angiotensin-converting–enzyme inhibitors C. Webbed neck The half-life of 25(OH)D is ~ 2 .338:1129 . Nonsteroidal antiinflammatory drugs In response to ultraviolet radiation of skin.12 hours N Engl J Med 1998. Valproic acid C. Thalidomide D.622 MCQ’s FOR MEDICAL PROFESSIONALS BY PROF. 6-dehydrocholesterol 1233 Teratogenic effect of which of the following drugs produces “renal C.338:1129 A. 25(OH)D is major circulating &storage form of vitamin D A. Optic-nerve blindness B. Hormone D. Carbamazepine D. 7-dehydrocholesterol tubular dysgenesis” ? D. 2373 C. Methyl mercury B. D. Misoprostol B. Methimazole C. Liver N Engl J Med 1998. Warfarin 1237 Fetal Minamata disease is caused by the teratogenic effect of 1230 Teratogenic effect of which of the following drugs produces which of the following drug ? “necrotizing enterocolitis” ? N Engl J Med 1998. Angiotensin-converting–enzyme inhibitors B. Thymic defects hydroxyvitamin D [25(OH)D] is the major circulating & storage form of vitamin D. Valproic acid 1231 Teratogenic effect of which of the following drugs produces Vitamin D “decreased skull ossification” ? N Engl J Med 1998.338:1129 D. Warfarin A.338:1129 A. D. Short nose with broad nasal bridge C. Diethylstilbestrol A. Carbamazepine B. Nerves B. Phenytoin D. 2247 B. Endoplasmic reticulum Harrison’s 16th Ed. 2248 D. > 50 mg/day . Serum bone-specific alkaline phosphatase to all except ? Harrison’s 16th Ed. 2246 C. Serum bone sialoprotein Harrison’s 16th Ed. Cell membrane D. Urine hydroxyproline B. C. AJAY MATHUR Cardiology 623 B. 25(OH)D3 D.25(OH)2 D3 Harrison’s 16th Ed. 2247 A. Hypoparathyroidism D. All of the above A. All of the above A. Nuclear 1256 In patients whose 1  hydroxylation is impaired. 2247 D. Keratinocytes B.25(OH)2 D 1257 What level of urinary calcium excretion predisposes to D. All of the above in adults include all except ? Harrison’s 16th Ed. Veins A. Sarcoidosis D. Cells of loop of Henle of Vitamin D3 ? C. Mitochondrial following vitamin D analogues is given ? D. Tuberculosis 1255 Radiologic feature of osteomalacia ‘Looser’s zones’ is related to ? C. 1. All of the above 1244 Biochemical markers of bone formation include all except ? Harrison’s 16th Ed. 1  hydroxyvitamin D2 B. 25(OH)D2 C. Any of the above C. 2271 1251 1. Serum osteocalcin A. Isoniazid Harrison’s 16th Ed. 2247 C. 2273 A. Isoniazid A.25(OH)2 D has an antiproliferative effect on ? Harrison’s 16th Ed. Berylliosis Harrison’s 16th Ed. Skin A. Distal convoluted tubule cells Harrison’s 16th Ed. 2246 C. Kidney 1250 Active transport of calcium across enterocytes is a function of ? Harrison’s 16th Ed. All of the above nephrolithiasis ? Harrison’s 16th Ed. Arteries 1248 Vitamin D receptor (VDR) is what kind of a receptor ? Harrison’s 16th Ed. Calbindin 9K D. 1. Liver disease Harrison’s 16th Ed. Serum osteocalcin B. Proximal convoluted tubule cells 1254 Which of the following drugs causes impaired 25-hydroxylation B. Lithium D. Ketoconazole B. 2273 1252 Accelerated loss of vitamin D due to increased metabolism is due A. 2247 C. ICaC 1243 Drugs associated with increased risk of generalized osteoporosis D. 2247 A. Serum bone sialoprotein A. Protamine C. Vitamin D receptor (VDR) has maximum affinity for ? B. Barbiturates C. 2246 D. 2249 A. 2248 A. 623 Endocrinology MCQ’s FOR MCQ’s FOR MEDICAL MEDICAL PROFESSIONALS PROFESSIONALS BY PROF. Serum tartrate-resistant acid phosphatase 1253 Impaired 1  hydroxylation leading to improper activation of vitamin D is due to all except ? B. ECaC The formation of the mature Vitamin D hormone occurs in the kidney after second hydroxylation. Rifampin 1245 Biochemical markers of bone resorption include all except ? D. Gonadotropin-releasing hormone agonists B. Ketoconazole A. All of the above B. Rifampin 1247  hydroxylase is produced in the granulomas of ? 1 Harrison’s 16th Ed. Prostate cancer cells D. Phenytoin B. Renal failure 1246  hydroxylase is expressed in ? 25(OH)D-1 C. Dihydrotachysterol (DHT) 1249 Normally. Breast cancer cells C. which of the C. Aluminum A. Lymph nodes B. Serum propeptide of type I procollagen C. D. Warmth over underlying pagetic lesion Urinary and serum deoxypyridinoline. 3137 C. TNFRSF11D B. 3137 1260 Principal abnormality in Paget disease is ? A.355:593-600 A. Thiazides diuretics Osteoclast-mediated resorption of bone takes place in scalloped spaces called Howship’s lacunae where the osteoclasts are attached through a specific integrin to components of the bone matrix such as osteopontin. 3137 A. Serum C-telopeptide C. Deoxypyridinoline B. Serum AST B. Urine or serum bone resorption markers include pyridinoline. Urinary Oxypyridinoline A. Its homozygous deletion causes juvenile Paget disease which is characterized C. which of the following is the most Harrison’s 18th Ed. Serum N-telopeptide B. 5-8 D. and C-telopeptide levels are products of type I collagen degradation. Pain is the most common presenting symptom. 1263 Inheritance of Juvenile Paget’s disease is ? Harrison’s 18th Ed. Pagetic osteoclasts are large. Osteoclast progenitors D. Pamidronate D. deoxypyridinoline. > 250 mg/day 1264 Howship’s lacunae is related to which of the following ? 1258 Burnett’s syndrome is related to ? Harrison’s 17th Ed. 3136 1269 Which of the following is an intravenous bisphosphonate ? N Engl J Med 2006. 2408 Harrison’s 16th Ed.624 MCQ’s FOR MEDICAL PROFESSIONALS BY PROF. C. 3138 common presenting symptom ? A. Urinary pyridinoline B. All of the above D. > 175 mg/day by uncontrolled osteoclastic differentiation & resorption. Osteoclast development A. Chapter 355. Diplopia D.12 1266 Which of the following is a marker of bone formation ? Harrison’s 18th Ed. N-telopeptide. 3136 A. 1268 Which of the following is not a urine or serum bone resorption 1262 Which of the following is a marker of bone resorption ? marker ? N Engl J Med 2006. TNFRSF11B Harrison’s 18th Ed. Increased size of osteoclasts D. Urinary hydroxyproline C. All of the above D. Autosomal dominant B. > 100 mg/day TNFRSF11B gene encodes osteoprotegrin with an autosomal dominant pattern of inheritance with variable penetrance. AJAY MATHUR Endocrinology B. Osteoclast-mediated resorption of bone B. Aluminum Intoxication B. 3137 A. Autosomal recessive A. All of the above Serum ALP and urinary hydroxyproline levels are markers of bone formation & resorption respectively. Paget’s Disease and Other 1265 Gene which encodes osteoprotegrin is ? Dysplasias of Bone Harrison’s 18th Ed. Vitamin A Intoxication D. Olpadronate C. Principal abnormality in Paget disease is the increased number and activity of osteoclasts. Paresthesias C. 3137 A. 1-3 D. Osteoblast progenitors C. while serum markers of bone formation are ALP and osteocalcin. TNFRSF11C A. 8 . Urinary deoxypyridinoline C. Any of the above . Milk-Alkali Syndrome C. 3-5 TNFRSF11B gene encodes osteoprotegrin. Increased number of osteoclasts B. N-telopeptide and C-telopeptide). Pain B. TNFRSF11A 1259 Number of nuclei in the normal osteoclast is ? B. X linked C. Serum deoxypyridinoline Harrison’s 18th Ed. Osteocalcin C. N-telopeptide D.355:593-600 Harrison’s 18th Ed. Increased activity of osteoclasts C. Zoledronic acid B. C-telopeptide Markers of bone resorption are urinary deoxypyridinoline & cross-linked N-telopeptide of type I collagen. Serum ALP Harrison’s 18th Ed. 1267 Which of the following is a product of type I collagen degradation ? 1261 In Paget disease of bone. 2259 A. All of the above D. D. Transfer A. Etidronate N Engl J Med 2012. and systemic iron needs. Storage Harrison’s 18th Ed. alendronate. Iron regulatory protein D. All of above C. Pamidronate 1277 Which of the following is the first step in iron metabolism ? D. absorption. Iron synthesis protein 1275 Which of the following is the major clinical manifestation of Enterocyte iron content regulates iron absorption through its effects on iron regulatory protein (IRP) hemochromatosis ? types 1 & 2 and their subsequent effects on messenger RNAs (mRNAs) encoding DMT1. Risedronate from enterocytes to plasma occurs through the basolateral transporter ferroportin. with axial migration of hips & functional flexion contracture. frame” vertebra. None of the above B. Etidronate Second-generation oral bisphosphonates include tiludronate. and hepatocytes (iron storage & endocrine regulation). reticuloendothelial macrophages (iron storage & recycling). porotic & radiodense lesions of ilium with whorls of coarse trabeculation. Risedronate C. Reticuloendothelial macrophages that of ? D. Ivory vertebra Regulation of each step of iron metabolism (reduction. Diffuse radiodense enlargement of a vertebra is referred to as “ivory vertebra. AJAY MATHUR Cardiology 625 D. and transfer) is mediated D.366:348-59 A. intracellular iron levels. Vertebral cortical thickening C. Hypoxia-inducible factor 2 (HIF-2) D. arthritis. Reduction B. Hypoxia-inducible factor 2 (HIF-2) C. Picture frame vertebra D. They are duodenal enterocytes A. Hypoxia-inducible factor 2 (HIF-2) In Pagets disease od bone. and HIF-2 ) or stability (with respect to A. A. N Engl J Med 2006. 2409 Four major cell types determine body iron content & distribution. Diabetes mellitus DMT1). 3139 D. Vertebral cortical thickening of the superior & inferior end plates creates a “picture inducible factor 2 (HIF-2 ) & subsequent changes in transcription of DMT1 & ferroportin. diabetes mellitus. All of the above Harrison’s 17th Ed. Systemic iron needs B. D. and HIF-2 . Absorption 1272 Which of the following is a 1st generation oral bisphosphonate ? C. Erythroid precursors 1271 Potencies of various bisphosphonates are expressed relative to C. Alendronate membrane. B. 625 Endocrinology MCQ’s FOR MCQ’s FOR MEDICAL MEDICAL PROFESSIONALS PROFESSIONALS BY PROF. N Engl J Med 2012. ferroportin. Intracellular iron levels A. Alendronate (dietary iron absorption). Thickened & sclerotic ileopectinal line B.366:348-59 A.366:348-59 1273 Radiographic finding of Paget disease include ? A. and risedronate. 1274 Brim sign refers to ? Harrison’s 18th Ed. 3137 1279 Enterocyte tension regulates iron absorption through ? N Engl J Med 2012. Oxygen tension in enterocytes Harrison’s 18th Ed. erythroid precursors (iron utilization). storage. Osteoporosis circumscripta C.” Pelvic radiographs show disruption or fusion of sacroiliac joints. Risedronate sodium A. Tiludronate Dietary iron is absorbed primarily by duodenal enterocytes. After the iron is reduced at the apical B. Hemochromatosis C. 3137 B. 1278 Which of the following regulates iron metabolism ? N Engl J Med 2012. Thickening of diploic skull areas A. Iron metabolism protein Chapter 357. ferritin. All of the above Major clinical manifestation of hemochromatosis are cirrhosis of liver. All of the above by signals reflecting oxygen tension in enterocytes. Iron absorption protein B. Etidronate disodium 1276 Which of the following cell types determine body iron content & distribution ? B.366:348-59 C.366:348-59 Potencies of various bisphosphonates are expressed relative to that of etidronate. zoledronic acid & olpadronate. Duodenal enterocytes D. All of the above C. B. Diffuse radiodense enlargement of a vertebra D. Iron is stored as ferritin and is lost on sloughing of senescent enterocyte. Harrison’s 18th Ed. Cardiomyopathy . Hypoxia-inducible factor 2 (HIF-2) B. thickening of diploic areas. 3162 ferritin. Hypogonadotropic hypogonadism Intravenous bisphosphonates are pamidronate. skull radiographs show regions of “cotton wool” or osteoporosis circumscripta. it is taken into the cell through the divalent metal transporter 1 (DMT1). enlargement & sclerosis of a portion or all of one or more Enterocyte tension regulates iron absorption through its effects on transcription factor hypoxia- skull bones. Tiludronate disodium N Engl J Med 2012. The IRPs bind to sequences (iron-responsive elements [IREs]) that influence mRNA translation (with respect to ferroportin. 1270 Which of the following is an intravenous bisphosphonate ? cardiomyopathy. Export of iron C. and hypogonadotropic hypogonadism. thickened & sclerotic ileopectinal line (Brim sign). and 1280 Enterocyte iron content regulates iron absorption through ? softening with protrusio acetabuli.355:593-600 A. The IRE-IRP system increases ferritin mRNA translation in response to cellular iron. C. This non-transferrin bound iron (NTBI) is readily taken B. 12 the circulation. In duodenal enterocyte. D. Non-transferrin bound iron (NTBI) A. The consequent iron retention in duodenal enterocytes decreases A. Non-transferrin bound iron (NTBI) dietary iron absorption. Hepcidin up by hepatocytes & cardiomyocytes. At equilibrium. Transferrin 1288 Hepatocytes are an important site of iron storage in the form of ? N Engl J Med 2012. Relative proportion of these ferritin chains in the complex varies in various tissues. iron export from RE cells is C. Type III Reticuloendothelial cells serve as the major hepcidin-regulated iron storehouse. B.366:348-59 D. Type II C. Citrate 1289 Which of the following is called as “hypoferremia hormone” ? N Engl J Med 2012. Ferroportin C. Hepcidin of incoming ferrous iron.366:348-59 D. RE cells release ~25 mg of iron per day. Recycling endosomes A. Type I D. Acetate Hepatocytes are an important site of iron storage in the form of ferritin. Entry of iron-bound transferrin into cells 1290 Hepatocellular hepcidin production is regulated by ? C. iron status. Inflammation Erythroid precursor cells regulate intake of transferrin-bound iron by altering expression of surface B. Phagocytosis of senescent erythrocytes N Engl J Med 2012. Hepcidin blocks release of iron from enterocytes & reticuloendothelial macrophages by degrading the iron exporter ferroportin.366:348-59 D. Liver 1291 Hepcidin is which type of acute-phase protein ? B. A fraction of circulating NTBI is redox-active & is designated labile plasma iron. Excess uptake of iron as NTBI produces oxidant-mediated cellular injury. Normal transferrin saturation is ~30%). Hepcidin binds to iron exporter ferroportin & induces its degradation. NTBI C.366:348-59 hepcidin. and oxygen tension. B. Sulphate A. NTBI is a major contributor to iron loading in hepatocytes under conditions of elevated transferrin saturation.366:348-59 1292 Hepcidin has structural properties similar to those of ? A. 24 1282 Plasma iron transport protein is ? Ferritin is an iron-storage protein complex composed of 24 ferritin monomers of two subtypes - N Engl J Med 2012. Heavy-chain ferritin has ferroxidase activity. Type IV represent the most dynamic iron compartment. Ferroportin When transferrin becomes highly saturated. D.366:348-59 D. Erythropoietic activity D.366:348-59 D. additional iron released into circulation is bound to low-molecular-weight compounds like citrate. which is required for the efficient oxidation A.366:348-59 A. Transferrin 1283 Non transferrin bound iron (NTBI) is best related to ? N Engl J Med 2012. Reticuloendothelial macrophages A. compartment ? erythropoietic activity. Ferroportin C.626 MCQ’s FOR MEDICAL PROFESSIONALS BY PROF. B. Intestinal cells N Engl J Med 2012. Iron retention in RE macrophages decreases iron turnover. Hemosidrin C. Hepcidin is a type II acute-phase protein that mediates hypoferremia associated with infection and inflammation. whereas light-chain ferritin promotes efficient nucleation and mineralization. Iron status transferrin receptor 1 (TfR1). As the pool of circulating transferrin iron is <3 mg. N Engl J Med 2012.366:348-59 D. Labile plasma iron A. Ferritin Iron released from enterocytes basolateral membrane binds to free sites on the plasma iron transport protein transferrin. Hemosidrin 1284 Transferrin receptor 1 (TfR1) is best related to ? Hepatocytes serve a central role in iron homeostasis as the site of regulated production of hormone N Engl J Med 2012. 18 mediated by ferroportin and regulated by hepcidin. Ferritin A. Hepcidin functions as the “hypoferremia hormone” by down-regulating ferroportin-mediated release of iron into circulation. B. All of the above 1285 Which of the following represents the most dynamic iron Hepatocellular hepcidin production is regulated by signals reflecting inflammation. Hepcidin Reticuloendothelial cells obtain most of their iron from phagocytosis of senescent erythrocytes. Labile plasma iron N Engl J Med 2012. Ferritin 1287 Ferritin is composed of how many ferritin monomers ? N Engl J Med 2012. Circulating transferrin iron N Engl J Med 2012. Ferroportin A. 1286 Reticuloendothelial cells obtain most of their iron from ? N Engl J Med 2012.366:348-59 B. Leukotriene B4 . RE cells D. 6 Systemic regulation of iron absorption is mediated by hormone hepcidin. Butyrate C.366:348-59 C. thus decreasing transfer of iron from enterocytes to B. TfR1 mediates entry of iron-bound transferrin (ferri-transferrin) into recycling endosomes.366:348-59 heavy & light chains. Transferrin cannot be used for production of heme which requires transferrin-bound iron. DMT1 A. turning over ~10 times per day. Bone marrow B. B. AJAY MATHUR Endocrinology 1281 Systemic regulation of iron absorption is mediated by ? C. Aceruloplasminemia. largely mediated by ? N Engl J Med 2012. Dysmetabolic iron overload A. Symptomatic organ involvement begins in midlife & Bone Morphogenetic Protein (BMP)/SMAD pathway. Hepatitis C. and results in a progressive increase of plasma ferritin. Hemojuvelin (HJV) atransferrinemia. HFE & TfR2 syndrome. 3162 (C282Y) is the most common mutation in HFE gene responsible for hemochromatosis. AJAY MATHUR Cardiology 627 B. Chronic hemolytic Harrison’s 18th Ed. and certain enveloped viruses. Ferroportin (FPN) B. protegrin. HFE mutation H63D results in a substitution of histidine to aspartic acid at codon 63. C126Y 1294 Hemochromatosis due to hepcidin mutation is referred to as ? A homozygous G to A mutation resulting in a cysteine to tyrosine substitution at position 282 Harrison’s 18th Ed. and hematologic anomalies are not usually seen. Bone Morphogenetic Protein (BMP)/SMAD pathway 1297 Which of the following about hemochromatosis is false ? D. linking body stores with intestinal iron absorption. C. Porphyria cutanea tarda. Type 1 B. All of the above which of the following ? Acquired iron overload states include Thalassemia major. 3162 A. 3162 B. Congenital atransferrinemia 1302 Hepcidin responds to changes in body-iron requirements through D. A. C. Type 2B which of the following ? Harrison’s 18th Ed.366:348-59 1299 Which of the following is the most common mutation in A. D. C282Y The inflammatory signal up-regulating hepcidin expression is largely mediated by interleukin-6. progressive accumulation of iron increases plasma iron. Sideroblastic anemia. Porphyria cutanea tarda intestinal iron absorption is ? Harrison’s 18th Ed. mucosal absorption of iron is greater than body requirements and amounts to 4 mg/day or more. Iron oxidase hephaestin (Heph) . E-selectin 1298 HFE gene is located on which of the following chromosome ? Harrison’s 18th Ed. Plasma iron Type 1 Hemochromatosis is HFE-related. C. Type 2A 1300 In hemochromatosis. erythropoiesis is not jeopardized. Iron oxidase hephaestin (Heph) 1296 Which of the following is a secondary iron overload state ? C. 4 D. Thalassemia major Hepcidin is the crucial molecule in iron metabolism. 7 HFE stands for High Iron Fe. Transfusional and parenteraliron overload. histatin. Hepatitis C A. granulysin. One involves HFE & TfR2. Type 2A is non-HFE-related caused by hemojuvelin B. 3163 Although iron metabolism is abnormal. 3163 anemias. HFE mutation related hemochromatosis results in lack of cell surface D. Congenital B. progressive accumulation of iron increases C. None of the above (Fe2+) state by ? Harrison’s 18th Ed. Interleukin-6 A. Hepcidin Harrison’s 18th Ed. Saturation of transferrin mutations and is called Juvenile hemochromatosis (type 2A). Duodenal cytochrome B (DcytB) A. Other antimicrobial peptides include cathelin. while the other involves hemojuvelin (HJV) B. H95D D. HFE gene is located on short arm of chromosome 6 at location 6p21. and probiotics. In hemochromatosis. 627 Endocrinology MCQ’s FOR MCQ’s FOR MEDICAL MEDICAL PROFESSIONALS PROFESSIONALS BY PROF. Nonalcoholic steatohepatitis.3 1293 The inflammatory signal up-regulating hepcidin expression is and encodes human hemochromatosis protein also known as the HFE protein. Sideroblastic anemia 1301 Crucial molecule in iron metabolism that links body stores with B. All of the above expression of HFE (due to the C282Y mutation). In hemochromatosis. 3163 C. Defensins B. B. Defensins are a family of cysteine-rich polypeptides with broad antimicrobial C. Lymphotoxin C. All of the above B. Ferroportin (FPN) D. Autosomal recessive through two mechanisms. Aceruloplasminemia Alcohol suppresses hepatic hepcidin secretion. SLC11A3. Leptin hemochromatosis ? Harrison’s 18th Ed. A. 1295 Which of the following is a secondary iron overload state ? saturation of transferrin. Type 3 A. Granulocyte-macrophage colony-stimulating factor B. Dietary iron overload. 3162 C. Post-portacaval shunting. D. Plasma ferritin is due to mutated transferrin receptor 2 TFR2 while type 4 hemochromatosis is due to mutated ferroportin 1 gene. Neonatal iron overload. Type 3 hemochromatosis C. Cathepsin G A. Harrison’s 18th Ed. 5 Hepcidin protein was originally identified as an antimicrobial peptide with structural properties similar to defensins. 3163 D. All of the above N Engl J Med 2004. fungi. secretory leukoprotease inhibitor. 3162 D.350:2383-97 Hepcidin responds to changes in body-iron requirements by signals mediated by diferric transferrin A. Alcoholic cirrhosis. H63D C. Type 2B is non-HFE-related caused by hepcidin mutation and is called Juvenile hemochromatosis (type 2B). Hematologic anomalies are not usually seen 1303 Dietary inorganic iron is reduced from ferric (Fe3+) iron to ferrous D. 6 activity against bacteria. 0. Sternoclavicular joints D. extensor aspects of lower forearms. Diabetes mellitus (65%). Upregulation of divalent metal transporter (DMT1) Harrison’s 18th Ed. Liver Ingestion of large doses of ascorbic acid promotes iron absorption. Cardiac involvement B. Second & third metacarpophalangeal joints C. Ferroportin B. are usually the first joints involved in hemochromatosis. 3 mg C. Plasma iron C. TfR2. and hemojuvelin. 4 mg In hemochromatosis. neck. 3164 1304 Hepcidin binds to which of the following ? Harrison’s 18th Ed. Wrist A. skin pigmentation. Reduced TfR1-mediated iron uptake by intestinal crypt cell 1311 Ingestion of which of the following promotes iron absorption ? C. 3165 increases intestinal iron absorption A. D. 3165 C. Skin A. Arthropathy (25-50%). 3163 A. 5 mg on face. Pancreas Harrison’s 18th Ed. Total iron-binding capacity D. diabetes C. Hypogonadism B. responds 1310 Which of the following is most common in hemochromatosis ? to signals in liver mediated by HFE. This feature helps to distinguish chondrocalcinosis associated with Iron moves from enterocyte to the circulation via a process requiring basolateral iron exporter hemochromatosis from the idiopathic form. arthritis. Harrison’s 18th Ed. 1308 Which of the following is the least common in 1314 Which of the following is the strongest predictor of hemochromatosis ? hemochromatosis expression in individuals homozygous for Harrison’s 18th Ed. characteristic metallic or slate-gray skin pigmentation referred to as bronzing results from increased melanin and iron in the dermis. Ascorbic acid D. 0. Diabetes mellitus C. 3163 D.2 mg Liver is usually the first organ to be affected in hemochromatosis. B. Hemosidrin D. In the circulation. as well as in scars. Transferrin C. 0. 3165 B. heart disease. 3164 1313 An increase of 1 mg/L in serum ferritin level reflects an increase A. D. Increased iron in the epidermis of about how much iron in body stores ? B. ferroportin (FPN) and iron oxidase hephaestin (Heph). B. and hypogonadism should suggest hemochromatosis.3 mg in >95% of symptomatic patients. Hypoparathyroidism A. and hypoparathyroidism are rare manifestations. A. Decreased iron in the epidermis Harrison’s 18th Ed. and hepatomegaly is present B. 3164 A. Heart 1312 1 mL blood contains approximately how much iron ? C. Carotene hemochromatosis ? D. Serum ferritin .628 MCQ’s FOR MEDICAL PROFESSIONALS BY PROF. DMT1 Dietary inorganic iron traverses the brush border membrane of duodenal enterocytes via DMT1 1309 Which of the following is usually the first joint involved in after reduction of ferric (Fe 3+) iron to the ferrous (Fe2+) state by duodenal cytochrome B (DcytB). iron binds to plasma transferring. Calcium Harrison’s 18th Ed. transferrin receptor 1 (TfR1) hypothyroidism. All of the above Joints of hands. in turn. especially second & third metacarpophalangeal joints. resulting in hemochromatosis.5 mg iron. An increase of 1 mg/L in serum ferritin level reflects an increase of about 5 mg iron in body stores. dorsa of the hands. Decreased iron in the dermis B. Adrenal insufficiency 1305 Which of the following is false about HFE-related hemochromatosis C. Cardiac involvement (15%). 3165 D. hemojuvelin and hepcidin lead to decreased hepcidin release and increased iron absorption. and genital regions. hypogonadism due to impairment of hypothalamic-pituitary function by iron deposition occurs in both sexes and may antedate other clinical features. First metatarsophalangeal joint B. None of the above B.5 mg 1307 Bronzing or characteristic metallic or slate-gray skin pigmentation 1 mL blood contains approximately 0. 0. AJAY MATHUR Endocrinology C. HFE protein forms a complex with 2-microglobulin & In hemochromatosis. 3165 TfR2. Mutations in genes encoding HFE. Hepcidin is a liver-derived peptide that represses basolateral iron transport in intestine & iron release from macrophages and other cells by binding to ferroportin. Hepcidin. Duodenal cytochrome B (DcytB) Frequency of organ involvement in hemochromatosis is hepatomegaly (95%). in hemochromatosis is due to ? Harrison’s 18th Ed. hemochromatosis ? Harrison’s 18th Ed. The association of hepatomegaly. Hypothyroidism ? Harrison’s 18th Ed.4 mg mellitus. 3164 C282Y mutation ? A. Arthropathy Harrison’s 18th Ed. 2 mg D. lower legs. Hepatomegaly A. Flavinoids 1306 Which is usually the first organ to be affected in C. Increased iron in the dermis A. Pigmentation is diffuse but more pronounced D. Adrenal insufficiency. Gastrointestinal D. 3167 Also. 4 B. 3167 C. B. In homozygotes with early. B. Nonalcoholic steatosis (NASH) C. asymptomatic hemochromatosis it si between 1 and 2. < 0. Porphyrias C. D. PCT is the most common porphyria.250 mg iron. Porphyria cutanea tarda (PCT) is a sporadic disorder. D.200 mg conversion of glycine & succinyl-CoA to heme ? Harrison’s 18th Ed. Harrison’s 18th Ed. 3167 D.specific (ALAS2) isozymes. Serum ferritin level <1000 mg/L B. No hepatomegaly A. AJAY MATHUR Cardiology 629 D. and disfigurement. 12 1319 Which of the following is a sporadic metabolic disorder ? Enzymes involved in heme biosynthesis are encoded by nine genes. B. peripheral motor neuropathy. which is usually a manifestation Harrison’s 18th Ed. Cutaneous sensitivity to sunlight results from excitation of excess porphyrins in A. or X-linked traits. The second enzyme.5 Harrison’s 18th Ed. Variegate porphyria (VP) 1315 Which of the following indicates risk of severe fibrosis in a C282Y Porphyrias are either hepatic or erythropoietic. It is hepatic and presents with blistering cutaneous photosensitivity.0 B.2 1322 Erythropoietic porphyrias usually present with ? B. < 0. Porphyria cutanea tarda (PCT) . 9 D. 50 . Skin There is virtually no risk of severe fibrosis in a C282Y homozygous hemochromatosis subject with C. Normal serum alanine amino transaminase values 1321 Major manifestation of the acute hepatic porphyrias is ? Harrison’s 18th Ed. or in the case of congenital erythropoietic porphyria (CEP). 3167 C. normal serum alanine amino transaminase values. 3167 C. 2 One 500 mL unit of blood contains 200 .100 mg 1323 Heme biosynthesis involves how many enzymatic steps in the C. Rrest of the porphyrias are inherited deaminase) either as autosomal dominant. 629 Endocrinology MCQ’s FOR MCQ’s FOR MEDICAL MEDICAL PROFESSIONALS PROFESSIONALS BY PROF. Hereditary coproporphyria (HCP) homozygous for the C282Y mutation. Harrison’s 18th Ed. Acute intermittent porphyria (AIP) Harrison’s 18th Ed. subjects homozygous for C282Y are at increased risk of breast & colorectal cancer. Chronic HCV infection 1324 Enzymes involved in heme biosynthesis are encoded by how D. A. Skin manifestations C.75 A. 3167 aminolevulinate synthase (ALA-synthase) has two genes. 100 . D. 20 . In symptomatic hemochromatosis it is > 2. < 1. Gastrointestinal manifestations D. scarring. 200 . 5’-aminolevulinate synthase (ALA-synthase) D. leading to cell damage. 6 1318 HFE mutations are related to which of the following diseases ? Harrison’s 18th Ed. Variegate porphyria (VP) B. and up to 25 g iron or more may have to be removed. Acute intermittent porphyria (AIP) Serum ferritin level >1000 mg/L is the strongest predictor of disease expression among individuals C. even in utero as nonimmune Harrison’s 18th Ed.25 mg the skin by long-wave ultraviolet light. 3165 A. 7 Chapter 358. D. The first enzyme 5?- Harrison’s 18th Ed. Neurologic manifestations Normal hepatic iron index is < 1. None of the above B. Hereditary coproporphyria (HCP) A. Neurologic serum ferritin level <1000 mg/L. 5’-aminolevulinate dehydratase (ALA-dehydratase) Porphyrias are metabolic disorders due to deficiency of a specific enzyme in heme biosynthetic C.250 mg A. 3167 It has two genes that encode unique housekeeping (ALAS1) and erythroid. no hepatomegaly. and no excess alcohol intake. third is A. 8 A. Endocrinal 1316 Normal hepatic iron index is ? Major manifestations of acute hepatic porphyrias are neurologic (neuropathic abdominal pain. 3165 of the erythropoietic porphyrias. 3166 hydrops fetalis. A. < 0. depending on the primary site of overproduction and homozygous subject ? accumulation of their respective porphyrin precursors. Porphyria cutanea tarda (PCT) 1325 The first enzyme in the heme biosynthesis pathway is ? B. autosomal recessive. 4 C. Hydroxymethylbilane synthase (HMB-synthase or PBG- pathway. A. Uroporphyrinogen III synthase (URO-synthase) 1320 The most common porphyria is ? The first enzyme in the heme biosynthesis pathway is 5?-aminolevulinate synthase (ALA-synthase). and mental disturbances). 3167 D. 5?-aminolevulinate dehydratase (ALA-dehydratase). C-reactive protein B. Endocrinal manifestations 1317 One 500 mL unit of blood contains how much iron ? Erythropoietic porphyrias usually present with cutaneous photosensitivity at birth or in early childhood. Porphyria cutanea tarda (PCT) Heme biosynthesis involves eight enzymatic steps in the conversion of glycine & succinyl-CoA to heme. All of the above many genes ? Harrison’s 18th Ed. myoglobin.acute intermittent porphyria (AIP). Hemoglobin synthesis in erythroid precursor cells accounts for approximately 85% of daily heme synthesis in humans. precursors. Glycine Ferrous iron is inserted into protoporphyrin IX to form heme. B. fifth is uroporphyrinogen decarboxylase (URO-decarboxylase). and ALA-dehydratase porphyria (ADP) present during adult life with acute attacks of neurologic manifestations and elevated levels of one or both of the porphyrin C. Feces D. Coproporphyrinogen (COPRO’gen) III A. and the cytochrome P450 enzymes (CYPs). Patients with ADP have also deaminase) presented in infancy and adolescence. Variegate porphyria (VP) Harrison’s 18th Ed. 3169 D. Heme is required for a variety of hemoproteins such as hemoglobin. catalyzes the condensation of glycine. All of the above A. Cytochrome P450 enzymes (CYPs) coproporphyrinogen (COPRO’gen) III. Succinyl coenzyme A 1335 Which of the following hepatic porphyrias present during adult D. AJAY MATHUR Endocrinology Hydroxymethylbilane synthase (HMB-synthase or PBG-deaminase). 3167 C. 3167 C. Harrison’s 18th Ed. catalyzes decarboxylation of two of the four propionic acid groups to form the two vinyl groups of protoporphyrinogen (PROTO’gen) IX. A. and turn over more rapidly than many other hemoproteins. hereditary coproporphyria B. HMB. catalyzes the manifestations ? condensation of two molecules of ALA to form PBG. ALA-synthase. 3169 D. or feces is required to confirm or exclude the A. All of the above the sixth enzyme. 3 A. All of the above 1334 Ferrochelatase is also called ? Harrison’s 18th Ed. variegate porphyria (VP). Harrison’s 18th Ed. 5’-aminolevulinate synthase (ALA-synthase) Four of the five hepatic porphyrias . Hydroxymethylbilane synthase (HMB-synthase or PBG- deaminase) 1332 Which of the following enters mitochondrion via transporter D. Urine D. sixth is COPRO-oxidase. Porphobilinogen (PBG) B. D. 3169 A. Hydroxymethylbilane synthase (HMB-synthase or PBG. All of the above life ? The first enzyme. 3169 The first and last three enzymes in the pathway are located in the mitochondrion. ALA and PBG. which are especially abundant in the liver endoplasmic reticulum. Hemoglobin D. erythrocytes. C. Plasma PROTO-oxidase oxidizes PROTO IX’gen to protoporphyrin IX by the removal of six hydrogen atoms to form porphyrin (oxidized form). and are thus classified as acute porphyrias. 3169 Laboratory measurement of porphyrin precursors like 5?-aminolevulinic acid (ALA) and porphobilinogen (PBG) in urine. Propionic acid groups of porphyrin precursors is done in which of the following ? Harrison’s 18th Ed. to form ALA. Hereditary coproporphyria (HCP) two molecules of ALA to form PBG ? C. Harrison’s 18th Ed. plasma. 5’-aminolevulinate dehydratase (ALA-dehydratase) Hydroxymethylbilane synthase (HMB-synthase or PBG-deaminase) catalyzes head-to-tail condensation of four PBG molecules to form the linear tetrapyrrole. Ferrous iron atoms B. Acute intermittent porphyria (AIP) 1330 Which of the following enzymes catalyzes the condensation of B. and eighth is ferrochelatase (heme synthetase or HMB ? protoheme ferrolyase). Protoheme synthase Harrison’s 18th Ed. 1 1326 Which of the following enzymes in the heme biosynthesis pathway is located in mitochondrion ? B. a tetracarboxylate porphyrinogen. Uroporphyrinogen III synthase (URO-synthase) 1336 Which of the following hepatic porphyrias have cutaneous The second enzyme. fourth is Uroporphyrinogen III 1331 Head-to-tail condensation of how many PBG molecules forms synthase (URO-synthase). 4 B. D. such as by the removal of ? the mitochondrial respiratory cytochromes. 3169 A. Protoheme ferrolyase A. Acetic acid side chains 1328 To confirm the diagnosis of porphyria. A. Heme decarboxylase diagnosis of porphyria. This reaction is catalyzed by eighth B. respiratory cytochromes. Pyridoxal phosphate enzyme. Hepatocytes 1333 PROTO-oxidase oxidizes PROTO IX’gen to protoporphyrin IX account for most of the rest. catalyzes the sequential removal of four carboxyl groups from acetic acid side chains of URO’gen III to form C. primarily for the synthesis of CYPs. Protoporphyrinogen (PROTO’gen) IX B. Coproporphyrinogen (COPRO’gen) III enters mitochondrion via a specific transporter. 3167 C. where COPRO-oxidase. a decarboxylate porphyrinogen. Hydrogen atoms A. 2 Harrison’s 18th Ed. Myoglobin The fifth enzyme. Heme synthase 1329 Which of the following is involved in the formation of ALA ? C.630 MCQ’s FOR MEDICAL PROFESSIONALS BY PROF. 3170 phosphate and succinyl coenzyme A. C. 5?-aminolevulinate dehydratase (ALA-dehydratase). seventh is PROTO-oxidase. laboratory measurement B. 5’-aminolevulinate dehydratase (ALA-dehydratase) (HCP). 3170 . ABCB6. 5’-aminolevulinate synthase (ALA-synthase) D. uroporphyrinogen decarboxylase (URO-decarboxylase). whereas the other four are in the cytosol. Uroporphyrinogen III synthase (URO-synthase) ABCB6 ? Harrison’s 18th Ed. C. activated by pyridoxal Harrison’s 18th Ed. Uroporphyrinogen (URO’gen) III 1327 Heme is required for synthesis of which of the following ? Harrison’s 18th Ed. ferrochelatase (also known as heme synthetase or protoheme ferrolyase). Variegate porphyria (VP) A. 3171 A. with URO-decarboxylase mutations) and develops after exposure to halogenated aromatic hydrocarbons. and urinary ALA excretion is 20 .100 mg/day (normal is 1 . Phenytoin A substantial increase in PBG may be due to AIP. or VP. All of the above Porphyria cutanea tarda (PCT) is the most common of porphyrias. heme albumin (hematin reconstituted 1340 Which of the following is false about acute intermittent porphyria with human albumin). All of the above B. 3176 B. Autosomal recessive Harrison’s 18th Ed. 3173 1346 Long-term risk of which of the following is increased in AIP ? A. AIP is especially common in Scandinavia & Great Britain. Almost always latent before puberty C. Heme arginate Urinary PBG is virtually always increased during acute attacks of AIP. D. All of the above normal level of HMB-synthase activity. Intravenous hemin elevations of porphyrins in bone marrow and erythrocytes and present with cutaneous photosensitivity. . 3172 A. Urinary A. Congenital erythropoietic porphyria (CEP) PBG excretion during an attack is 50 . Barbiturates D. AIP 1345 In AIP. or heme arginate is infused daily for 4 days. 1. B. B. excretion of ALA & PBG decrease dramatically after ? Harrison’s 18th Ed. (AIP) ? Harrison’s 18th Ed.200 mg/day (normal is 0 . It can be either sporadic (type Attacks of AIP can be provoked by infections. Erythropoietic protoporphyria (EPP) C. Harrison’s 18th Ed. Heme albumin D. X-linked protoporphyria (XLP) 1343 In AIP. Intravenous calcium D. Harrison’s 18th Ed. Intravenous amono acids 1338 A substantial increase in PBG is seen in ? In AIP. Abdominal pain is the most common symptom D. C. 3173 A. presents with blistering skin lesions.congenital erythropoietic porphyria (CEP) & erythropoietic protoporphyria (EPP). with no URO-decarboxylase mutations) or familial (type 2. HCP Harrison’s 18th Ed. 3 . Due to half-normal level of HMB-synthase activity A. peripheral neuropathy is due to axonal degeneration rather than due to demyelinization. Hepatic URO-decarboxylase C. 3173 C. Porphyria cutanea tarda (PCT) 1342 Which of the following is false about acute intermittent porphyria B. All of the above B. All of the above C. HCP A. VP B. Intravenous glucose The erythropoietic porphyrias . Hepatic URO-decarboxylase is deficient in all types of PCT. HCP. 1347 Which of the following is deficient in all types of porphyria cutanea tarda (PCT) ? 1341 Attacks of AIP can be provoked by ? Harrison’s 18th Ed. and VP and is not substantially increased in any other medical condition. Urinary retention is characteristic The fifth hepatic disorder. 3176 D. COPRO-oxidase D. Surgery C.4 mg/kg of heme in the form of lyophilized hematin. excretion of ALA & PBG decrease dramatically after intravenous hemin.4 mg/day). 3173 C. Infections B. Lyophilized hematin C. All of the above A. Ethanol D. 3177 Harrison’s 18th Ed. Harrison’s 18th Ed. C. All of the above In AIP. 3170 ALA & PBG levels are substantially increased in plasma & urine during acute attacks. Clonazepam of ? may be safer than phenytoin or barbiturates. D. AIP 1344 In AIP. Hypertension C. heme is given in the form of ? Harrison’s 18th Ed. XLP are characterized by B. Hepatocellular carcinoma Acute intermittent porphyria (AIP) is an autosomal dominant hepatic porphyria resulting from half. Chronic renal disease D. Common in Scandinavia & Great Britain B. Hereditary coproporphyria (HCP) (AIP) ? Harrison’s 18th Ed. Clonazepam D. A. 3177 B. AJAY MATHUR Cardiology 631 A. surgery and ethanol. HCP. 631 Endocrinology MCQ’s FOR MCQ’s FOR MEDICAL MEDICAL PROFESSIONALS PROFESSIONALS BY PROF. D. VP A. treatment of seizures is preferred with the use of ? B. porphyria cutanea tarda (PCT). including the recently described X-linked form. Peripheral neuropathy is due to axonal demyelinization HCP and VP also may have cutaneous manifestations similar to PCT. AIP is almost always latent before puberty. ALA & PBG levels increased in plasma & urine during acute 1337 Which of the following erythropoietic porphyrias present with attacks cutaneous photosensitivity ? In AIP. C.7 mg/day). Any of the above 1339 Urinary PBG is virtually always increased during acute attacks Treatment of seizures is difficult because most antiseizure drugs can exacerbate AIP. Hydroxyurea D. Hereditary Coproporphyria (HCP) A. Urinary COPRO III D. 3178 B. 3179 Harrison’s 18th Ed. China A.10% of normal. EPP is the D. Variegate Porphyria (VP) D. after PCT. Skin friability & small white papules termed milia are common on back of hands and fingers. Congenital erythropoietic porphyria (CEP) is also called Günther disease. AJAY MATHUR Endocrinology 1348 Which of the following is not an autosomal dominant hepatic 1354 Diagnosis of Hereditary Coproporphyria (HCP) is confirmed by porphyria ? which of the following ? Harrison’s 18th Ed. COPRO-oxidase Susceptibility factors in inherited UROD mutations in type 2 PCT include hepatitis C. All of the above D. . Hepatitis C A. 3177 A. Fecal protoporphyrin B. In PCT. HIV. 3178 Harrison’s 18th Ed. 3177 Harrison’s 18th Ed. Variegate Porphyria (VP) B. 3178 A. All of the above 1352 URO-decarboxylase activity in liver. Brazil B. & scarring. where 3 of every 1000 whites have the disorder. HIV B. South Africa D. erythrocytes. Erythropoietic Protoporphyria (EPP) B. 3178 A. Chloroquine phosphate Erythropoietic Protoporphyria (EPP) results from deficient activity of ferrochelatase. Hemochromatosis gene (HFE) mutations frequent B. 1357 Which of the following is increased in Variegate porphyria 1351 Which of the following statements about PCT is false ? (VP) ? Harrison’s 18th Ed. B. URO-decarboxylase activity is 1359 Which of the following is the most common erythropoietic markedly deficient. Chronic liver disease present C. elevated iron levels. Congenital Erythropoietic Porphyria (CEP) B. 3177 A. X-Linked Sideroblastic Anemia (XLSA) A. 1350 In PCT. Type 2 Porphyria cutanea tarda C. erythrocytes is very 1358 Which of the following is called Günther disease ? significantly decreased in ? Harrison’s 18th Ed. 1349 Susceptibility factor in type 2 PCT is ? 1355 Variegate porphyria (VP) results from the deficient activity of ? Harrison’s 18th Ed. Pilia D. with typical levels of 3 . Hepatoerythropoietic porphyria (HEP) C. 3179 1353 Which of the following is useful in the treatment of porphyria cutanea tarda ? A. Neurologic features are absent A. III. Milia B. Elevated iron levels C. Congenital Erythropoietic Porphyria (CEP) A. Type 1 Porphyria cutanea tarda B. sometimes marked increases in photosensitivity and hepatocellular damage.632 MCQ’s FOR MEDICAL PROFESSIONALS BY PROF. In HEP. leaving areas of atrophy Missense mutation R59W is common mutation in PPOX gene. Hepatoerythropoietic porphyria (HEP) D. URO-decarboxylase activity in liver. Plasma porphyrins D. which distinguishes it from other porphyrias. low dose of chloroquine phosphate (125 mg) twice weekly) should be given. because standard doses can induce transient. 3177 Harrison’s 18th Ed. 3177 Harrison’s 18th Ed. Quinine most common erythropoietic porphyria in children and. blistering skin lesions appear on back of hands that rupture & crust. PROTO-oxidase D. porphyria in children ? Harrison’s 18th Ed. Plasma porphyrin C. 3178 A. skin friability & small white papules on back of hands & 1356 Variegate porphyria (VP) is particularly common in ? fingers is called ? Harrison’s 18th Ed. Urinary PBG C. Hepatoerythropoietic porphyria (HEP) D. Erythropoietic Protoporphyria (EPP) C. None of the above D. Australia C. Urinary ALA B. and estrogens. the second most common porphyria in adults. Cryslia VP is particularly common in South Africa. oxidase. URO-synthase C. Pentamidine C. and cultured skin fibroblasts in type 2 PCT is approximately 50% of normal in affected individuals. excess VP is an autosomal dominant hepatic porphyria that results from the deficient activity of PROTO- alcohol. In PCT. X-Linked Sideroblastic Anemia (XLSA) Harrison’s 18th Ed. Trichia C. Increased fecal porphyrins Hepatoerythropoietic porphyria (HEP) is an autosomal recessive form of porphyria that results from Diagnosis of HCP is confirmed by increased fecal porphyrins consisting almost entirely of COPRO the marked systemic deficiency of URO-decarboxylase activity with clinical symptoms in childhood. Hepatoerythropoietic porphyria (HEP) C. B. clinical manifestations of Wilson disease are caused by copper toxicity that primarily involve liver and brain. N-type ATPase medical therapy. Hepatoerythropoietic porphyria (HEP) D. Serum aspartate transferase (AST). Hasford prognostic system C. 3188 A. Dystonia B. 3188 Chapter 360. Brain B. 3189 D. AJAY MATHUR Cardiology 633 1360 Which of the following is an autosomal recessive disorder ? B. Nazer prognostic index Harrison’s 17th Ed. B. Ascorbic acid As the disease progresses. All of the above A. B. Friedreich’s Ataxia. Lung A. . P-type ATPase 1370 Which of the following is associated with pes cavus ? D. S-type ATPase A. 3188 Sensory abnormalities and muscular weakness are not features of Wilson’s disease. Friedreich’s Ataxia causes retention of copper in liver leading to liver disease. 3188 D. 3189 D. Hematin A. D. CMT1x P-type ATPase is required to export copper from hepatocyte. M-type ATPase Nazer prognostic index includes Serum bilirubin. All of the above B. Heart accompanied by ? Harrison’s 18th Ed. C. Halsted prognostic index 1364 Which of the following best relates to Wilson disease ? D. Incoordination 1361 Wilson disease is caused by mutations in ? C. International Prognostic Index (IPI) Excess hepatic copper due to ATP7B protein deficiency is bound to metallothionein. Kayser-Fleischer rings in cornea & elevated hepatic copper level. Wilson’s Disease A. All of the above 1366 Movement disorder in Wilson’s Disease is ? Harrison’s 18th Ed. B. Pancreas 1368 In Wilson’s disease. All of the above Harrison’s 18th Ed. Central Core A. Orthostatic hypotension & sweating abnormalities Wilson disease is an autosomal recessive disorder caused by mutations in ATP7B gene. Sensory abnormalities involvement of ? D. Erythropoietic Protoporphyria (EPP) urinary excretion of copper. C. Patients with scores >9 should be considered immediately for liver transplantation. Parkinsonism 1362 Clinical manifestations of Wilson disease are primarily due to C. ATP7B gene C. Tremor Harrison’s 18th Ed. 3179 C. incoordination & tremor. Pyridoxine C. Cynacobalamin Harrison’s 18th Ed. leading to neurologic & psychiatric disease. Elevated hepatic copper level A. Metallothionein Pyridoxine in doses of 25 mg/day should always be given with penicillamine. ATP9B gene A. Serum ferritin 1369 Which of the following best relates to Wilson’s disease ? C. increased C. Patients with scores <7 can usually be managed with B. ATP8B gene 1367 Which of the following is not a feature of Wilson’s Disease ? Harrison’s 18th Ed. 633 Endocrinology MCQ’s FOR MCQ’s FOR MEDICAL MEDICAL PROFESSIONALS PROFESSIONALS BY PROF. 3188 Pes cavus is associated with CMT1x. 3188 D. Thiamine 1363 Which of the following best relates to Wilson disease ? D. Increased urinary excretion of copper Harrison’s 18th Ed. 2449 A. Abetalipoproteinemia Harrison’s 18th Ed. ATP6B gene Three main movement disorders in Wilson’s Disease are dystonia. Reduced blood ceruloplasmin level Disease and Plantar Fasciitis. Nemaline Myopathy. Memory loss Harrison’s 18th Ed. A. Nemaline Myopathy 1365 Diagnosis of Wilson’s Disease is done by finding ? D. penicillamine treatment should always be C. Congenital Erythropoietic Porphyria (CEP) Diagnosis of Wilson’s Disease includes demonstration of reduced blood ceruloplasmin level. Mutations in ATP7B in Wilson disease B. and Prolongation of prothrombin time (seconds). B. Notes : . ......... D 140 ....... D 168 . B 56 .......... B 101 ....... D 5 ................ B 129 ................ D 167 ............. C 95 .......................... B 40 . D 163 ........... B 138 ............ B 91 ................... D 47 .... B . D 125 ........ D 115 .......................... D 90 ........ C 160 ........ A 131 ............................ C 51 ......... C 59 ..................... D 13 . D 137 ......................................................................... B 73 ........... D 79 ..... D 126 .. C 153 ..... D 111 ............. C 17 .......... B 109 ........ D 113 ................................................. D 34 .... C 44 ..................... D 69 ........................... A 96 ...................... D 146 ............ D 49 ................................................................. A 37 .......... A 134 ...................... D 97 ....................................... B 133 ......... B 6 .. B 143 ............ D 89 ....................... C 151 ..................... D 124 .. B 46 .. C 39 ........... AJAY MATHUR 635 ANSWERS ENDOCRINOLOGY 1 ...... D 83 ...... A 114 ..... D 54 ................... B 149 .......... C 116 . B 172 .... A 155 ....... D 103 ....................... D 87 ... D 105 ...... C 171 .. A 122 ....... C 142 ............ D 156 ....... B 16 .......... B 76 ......................................... C 174 ...... D 24 .... D 57 ......................................................... A 93 ............... C 60 ................................................................. A 72 ....................... D 38 .................... C 148 .. B 8 ............................... B 99 ...Answer MCQ’s FORMCQ’s FOR MEDICAL MEDICAL PROFESSIONALS PROFESSIONALS BY PROF............. D 26 .. D 42 ...... C 21 .................. D 25 ............................... D 36 ........................... D 7 ......... B 63 ................ D 27 ................................... B 22 ...... A 162 ....................................... C 141 ...... D 157 ............................. C 78 ......................................... D 70 .......... C 123 .. D 166 ........ C 2 . C 132 ..... D 65 ........... B 61 .... D 52 .. C 104 ........................... C 102 ......... C 15 ........................... D 106 ............... C 161 ....... A 86 ................................... D 135 ................................ A 136 ...... D 119 .............. D 58 .................. D 121 .................................... A 55 .................. B 29 .............. B 94 .......... B 173 ................... D 100 ............................................... D 71 . B 12 ...................... D 112 ......... D 164 ................. D 53 ..................... D 67 ...... A 139 ................... B 19 ... A 33 ...... C 150 . D 88 .. B 82 ............... B 170 ...... A 4 ............ B 92 ......................... D 50 .. D 144 .... D 107 .............................................. A 20 ....... D 84 . C 130 ............... C 118 ............... B 64 ... D 3 ..................... D 9 ........................ C 127 ................ B 108 ... D 41 ... C 75 ....... D 11 ........ C 169 ................ A 152 ................................. D 81 . D 74 . C 28 .... D 10 ........... D 48 . A 62 ................... A 154 ................................. D 98 ...... D 18 . C 110 ..... A 117 ............................................................ C 31 ....... D 165 ..... D 32 ................................... B 120 ................................ D 45 ....... B 14 ......... D 159 ......... C 80 ................................. D 68 ........................................ B 147 ..................... C 158 ................................... A 85 ................. B 30 ... D 145 ..... D 128 ..... C 43 ............ A 77 . C 66 ............... D 35 ..................................... C 23 ... . A 200 .......................... A 224 ............ D 268 ....... B 328 ................. D 178 ..... D 222 ..................... B 209 ................. A 219 ........... D 311 ......... A 306 . D 206 ................... D 301 . D 308 . D 241 .................. B 189 ............... C 259 .......... B 293 ........ C 322 ... C 226 ....... A 334 .. C 228 ........ D 330 ............ D 295 ............. D 287 .... B 285 ...... C 341 ...................... D 315 ...... A 298 ............................................................................ D 208 ........ D 288 ....................................... C 257 ............... C 218 ... D 195 ..... A 251 .. A 332 ... C 320 ........... B 265 .......................... B 245 ................ B 183 .................... D 344 .. C 304 ............................. D 316 ........ A 289 .... B 190 ...................................... D 337 .... D 297 .... B 286 . B 318 .......... AJAY MATHUR Answer ANSWERS ENDOCRINOLOGY 175 . C 197 . D 248 ............................................ B 199 ............. D 302 ........... A 182 .................... C 246 ......... B 256 .... C 277 ........ B 281 ...... D 261 ............. A 305 .............. C 247 ... D 279 ................................... C 307 ... C 335 .......................... C 323 ................ D 250 ...................................... C 282 .... C 180 .... B 309 ................ C 191 . A 227 ............................. D 329 ............ C 266 .... A 223 ......... D 263 ......... D 267 ...................................................... C 193 ............ D 242 .. C 317 . A 196 .. D 233 ..... B 225 .......................................... B 254 .............. A 214 ......................... B 194 .................... D 188 ............ C 243 ............ C 198 ..... B 203 ... C 303 ............. D 294 ................... D 262 ....... A 235 ...... B 326 ........... B 300 ........................................... C 343 ............. C 284 .... A 184 ............. D 185 ..... D 327 .... D 179 .. D 253 ............................................ B 348 .............. A 192 . A 340 .................... B 271 ... D 216 ................. D 258 ... C 312 ........... C 336 .............. A 215 ............................................................... D 210 .. A 244 ...................................... A 346 ... A 201 ..................... D 249 ............... C 313 ...... D 283 ....... A 291 ........... D 269 ....... B 270 . D 260 ... A 276 ............................ C 278 ..... D 310 ........ D 221 ..... A 213 .. A 205 .................... C 333 .............. A 325 ..................... A 264 . C 324 ........................................ B 299 .. D 186 .... D 207 ....................636 MCQ’s FOR MEDICAL PROFESSIONALS BY PROF... D 231 ............. B 211 ..... A 240 ................... D 202 .. D 239 ....................... D 342 ..... B 217 ....................................... D 319 ............................ A 236 .......................... D 321 . B 275 ............. B 331 ........... A 339 .............. D 290 ....................... C 176 ... C 292 .. B 272 ............................ D 232 .......... B 230 .. C 296 ........................................... B 204 ..... C 212 . D 238 ............................................ B 273 ....................................................................... A 345 .......... D 314 ..... D 187 ............................. C 237 ....... C 252 .. B 177 .................................... B 229 ..... A 234 ..... D 274 .. D 280 ......... D 338 ......... B 255 . A ... A 220 .......................... A 181 .............................................. D 347 ........................... ... D 355 . D 503 ...... A 488 .................. A 395 ............................ D 392 ....... B 354 ................................. A 494 ................. C 432 ................ D 361 ............................................... D 506 .. D 474 ............................................................................ A 457 . D 360 . A 428 ....... A 519 .. D 389 ............................. C 434 ... C 376 .................... A 416 ....... A 421 ................ B 404 . D 351 .......... D 452 ............... B 451 ...... A 437 ......... B 443 ....... D 481 .. D 449 ..... B 502 ...... A 454 ........... D 463 ... C 359 ................. D 466 ... D 493 ................... C 512 ....................... A 381 .................................................................... A 469 ............................. D 425 ...... D 478 ......................... D 356 ......................................................... D 484 ..................... D 418 .. D 433 ........... D 501 ............ D 483 .... D 460 ....... D 386 .......... D 499 ... A ...... D 496 .... C 424 .. C 521 .. C 419 ............... B 477 .... A 470 .................... D 368 ........... C 492 . C 439 .. D 479 ........................................... C 522 .................... D 371 .......... A 407 ........................... D 366 ................. C 383 ................. B 430 . C 380 ........... D 413 ................ A 436 . B 405 ............................. A 426 ....... D 399 . A 414 ..... A 505 .... B 485 .............. A 462 ................ A 382 ................. D 411 ..... C 374 ............................................... D 510 ... A 423 . C 442 ................................................................................................... C 393 ..................................... B 450 .............. D 467 ................................. A 429 ............................ D 476 ........... A 435 ...... A 353 ................ D 406 ......................... D 375 ......... A 465 ............................. D 448 . A 441 ... D 397 ......................................................... B 387 .................... D 504 .............. D 447 .. D 362 .. C 456 . AJAY MATHUR 637 ANSWERS ENDOCRINOLOGY 349 ..... A 475 ............... D 487 .................Answer MCQ’s FORMCQ’s FOR MEDICAL MEDICAL PROFESSIONALS PROFESSIONALS BY PROF................................... D 352 ...... D 391 ..... D 367 ... A 412 .................. A 464 ..................... D 514 . D 473 ........ D 458 ................................................................. A 350 ... D 489 ................................................. B 453 ......... A 455 ...... C 390 .... A 445 ............ A 461 ................... D 363 ..... B 500 .......... A 373 ........................ D 400 ............ D 509 ............ D 491 ... D 409 ....... D 511 ............ C 379 ........................ D 459 .... D 440 .......... A 402 . B 497 ......................... A 444 . B 438 ... B 394 ....... C 417 .... A 468 ........ B 401 .......... D 486 ............................ A 471 . D 513 ......... D 372 .. D 446 ........ B 384 .......... D 482 .... A 408 ...... D 495 ...... B 422 ................................. D 358 ................ A 396 .................... C 377 ..... B 378 ..................................... A 518 .. B 385 .... D 498 ........ A 420 .......................... A 507 ................... C 415 . B 516 .. A 388 .. A 427 ............... D 480 . B 365 ........ C 403 . A 410 .......... B 369 . D 490 ..................... C 431 .... A 515 .... D 364 .......................... D 520 ..................... D 517 .... A 370 .. C 472 ...... C 508 .................. A 357 ...................... B 398 ........................ ... C 676 .......... C 660 ............................ B 568 .......... D 573 ... D 586 ... B 540 .. A 650 ........................... C 561 ............................638 MCQ’s FOR MEDICAL PROFESSIONALS BY PROF........... D 604 ............. C 578 .......... A 681 .... D 629 .............................. B 608 ..... B 677 ............................................... B 535 ............... A 591 . A 570 ...... B 524 ............... C 659 ............ D 639 ................ D 556 ....... A 585 ....... D 546 . D 559 .... B 637 .... A 694 ....... A 621 ......................................... C 615 .. C 661 ............................ A 582 ...... D 683 ....................... D 579 ............................ C 687 .......................................... C 685 ................................... D 607 .................. D 529 .............................. D 531 ... B 551 ..................................... D 656 ....... B 552 ... A 534 ..... C 603 .. C 690 ............................ C 635 ...... B 555 ........... D 630 .......... C 663 . B 696 ........... D 542 ................ B 527 ...... D 613 ......... C 671 . D 622 .......... C 678 .. D 666 ....... C 554 ................ C 572 .... C 605 ... D 657 .. D 580 .... D 651 ....................... D 619 ..................... D 567 ..... D 569 .............. D 682 . AJAY MATHUR Answer ANSWERS ENDOCRINOLOGY 523 ..... D 537 ............. B 668 ............. C 646 .... C 665 ................................... D 547 ..... C 564 ... D 575 ................ D 609 .... B 539 ........ A 689 ...... A 670 .... D 636 ... D 638 ... C 598 ............ D 669 .......................... C 686 ................................. B 628 .................... D 560 ..................................... D 532 ................... C 674 ......... D 684 ....... C 633 ..................... A 576 ................................. D 658 .................... B 662 ...................... A 600 ... A 649 . B 584 ........ D 581 ... B 599 .. D 617 .................... B 538 ......... D 648 ......... C 653 ....... A 627 ............................................................................. C 688 .............. A 530 ................... D 601 .................... A 549 ...................... A ..... D 602 .... D 614 ..... D 553 ........... B 642 ............................ D 528 ..... D 593 ........ D 625 ............................................................... D 679 . D 543 ................................ A 620 ............................... B 654 ..................................... D 557 .. C 634 .......... B 631 ........... A 548 . D 550 ................ D 577 .......... C 695 .......... A 571 ............ D 545 ..... C 616 .......... C 574 .... A 587 ......... C 526 ...................... B 618 ...... B 664 ........... C 596 . B 644 ............................................. B 667 .... B 610 ...... D 533 ... D 655 ... C 583 ......... D 541 .................. D 595 .. D 647 . C 536 ..................... C 673 . D 562 .... B 525 ...................... D 597 ...... C 544 . C 611 ............ B 566 ............... B 652 ............ D 588 ......... D 692 ........ D 589 .............. B 640 ................. B 626 ................. D 624 ................ D 592 .................... C 612 .................................. D 672 ............ D 558 ............................. D 594 ................................. D 632 .......... C 565 ................................. D 606 .... D 641 ...... B 680 ................... B 563 . D 623 .............. A 643 ............................................................................ B 675 ................ D 691 ....... D 645 ................ B 693 ..................... D 590 ...... . A 773 ................. C 741 ................. A 777 ......................... D 714 ............................ B 779 ................................................................................................................. C 736 . D 716 ............. C 813 ..... A 866 .. D 852 ... A 846 .. D 832 ..................... D 849 ............ B 726 ........ B 775 ........ D 856 ........... A 705 .. C 808 .................... D 792 .. A 788 ............... A 765 .... AJAY MATHUR 639 ANSWERS ENDOCRINOLOGY 697 ....... A 843 .. A 733 .......... A 811 ........... C 855 .... C 838 ...... A 767 .. C 763 . A 791 ................................ B 774 .......................... A 759 ......... C 824 ........ D 742 .............. B 784 ....................... A 752 .. A 815 ........................... A 861 . B 826 .......... B 720 ................ B 850 ................ D 701 ........ A 732 ... B 854 ...................... B 816 ................................................ D 698 ....... D 794 .... C 756 ................... D 753 ......... D 830 . C 760 . A 810 .. D 739 .... C 800 ............................................. D 728 ..... A 805 ................................... D 799 ................................................. A 821 ........................... D 717 ..... B 769 ............ D 781 ........... C 812 .... A 738 ........ C 700 ................. C 829 ... B 754 ........................... D 841 ............ D 835 .. B 870 ....... A 706 ............ D 771 .. C 809 ................. D 723 ......... A 803 . D 755 .......................... C 787 .............. C 848 ................ A 721 ............ A 789 ................... D 785 .............................. B 814 ................. D 735 ....... D 807 ...... A 768 .......... A 745 ................................... A 703 ..... A 749 ............... B 834 ........... A 712 ..................... D 798 ......................................... C 859 ............................... A 797 ............... D 783 ...... B 796 ..... D 836 . A 818 .................................... A 825 .................. D 793 ........ A 719 . C 734 ........ D 851 ..................... D 828 .................................... C 722 ......... C 751 .. B 863 .... D 790 ........................ B 770 ........... B 778 ......... D 842 .... C 699 ..................................... D 730 ...... C 709 .... D 804 ......................... B 864 ......... A 860 ...... A 858 .............. B 743 ........ C 731 .. B 802 ............... C 762 ....... D 817 ....... D 772 ......... B 780 ........... C 820 .......... D 837 ......... B 704 ................ D 833 ............. D 806 ........... B 801 ........ D 776 .... D 827 ............... D 847 ........... C 725 ....................................... A 758 .......... D 845 ........................ A 708 ....... B 862 . B 750 .................. B 819 ............ A 727 . B 766 ........................ B 748 ........... D ........... D 823 .............. D 822 ................Answer MCQ’s FORMCQ’s FOR MEDICAL MEDICAL PROFESSIONALS PROFESSIONALS BY PROF........................................ D 844 ............ A 865 .... D 724 ............. A 729 .... C 857 ......................... C 868 .. C 782 ............ D 853 ........................................................ D 744 .............. A 795 ....... C 761 ........... B 713 .......... C 737 ........... D 757 ..... B 718 ............. A 840 ........................... D 707 ....... C 764 . B 869 ....... B 710 ............ B 715 .... D 747 ........................................... A 740 . A 786 ....... B 839 ............... B 831 ..... D 867 ........................................ D 746 .............. C 702 .......... C 711 ................................ ... D 892 ........ D 971 ... C 1002 ............. B 916 ................ D 926 . B 1015 ............. D 1009 .............. B 904 .................. A 988 ................... A 1037 .... D 968 ........ D 989 ... A 899 .......... D 953 ........ C 967 ............ B 895 .......... B 1001 ...... C 890 ..................... D 1004 ..... B 952 .... C 1026 ..................................... D 921 ........ B 1035 .......... B 877 ........ D 907 ..... A 913 .................. AJAY MATHUR Answer ANSWERS ENDOCRINOLOGY 871 ...... B 918 ........................ C 1029 ........ D 1041 . D 976 ............................ D 982 .... D 981 ................................ B 987 ..... C 878 ........................ D 1022 .................................. D 985 .. D 883 . D 917 ........... A 950 ............ D 1033 ............... A 1005 ....... D 920 .......... B 946 . A 919 ....... B 974 ....................... D 986 ........ D 1042 .................. A 960 ...640 MCQ’s FOR MEDICAL PROFESSIONALS BY PROF.... C 1023 . D 888 ... B 889 .... C 962 . C 951 .. A 1012 .............. D 873 .. B 1018 .................... C 1011 .............................. D 942 ................ C 1028 ....................... D 1030 . C 909 ...................... D 997 ..................... D 979 ............... D 973 ........ D 955 ............................... D 980 ..................................................... B 1019 ..................................... A 944 ................. D 1010 ................................... B 934 .......... B 924 ...... C 928 ....... A 957 .. B 915 .................... D 914 .. C 885 ...... A 977 .. A 925 ............................................................... A 941 ................. C 998 ..... C 1016 ......................................... D 943 .......... D 963 .............................. B 1027 ................ C 1017 ................ D 964 ................................ A 898 ............. C 893 ......... D 1007 .... C 1040 ............. D 887 .... C 972 ....... B 1024 .................... D 990 ...... A 1014 .............................................. A 910 ............. D 939 ................ D 983 ................ D 900 .................................. A 902 .... A 947 .................. D 959 .. D 1000 ................................ D 1034 ................. C 891 ......................... B 922 ...................... D 940 ....................................... C 949 ................ D 975 .......... B 931 ... C .. D 879 .......... A 923 ................ B 984 .. A 958 ...... B 908 ............................ D 938 . D 906 . D 882 .. C 994 ................................. B 886 .......... C 1008 .................................................. D 897 ............................................. D 876 ......... D 966 ...... D 905 . D 978 ......... B 872 ... B 1036 .... B 912 .............. D 992 ............... C 932 . B 1032 . A 881 ..... A 993 . A 884 ...... D 948 .......... A 894 . D 1020 .... B 965 ........................ B 1006 ........... A 999 .............. B 954 ..... A 929 ... B 956 ............. A 970 ............ D 1043 .. D 996 ................................ C 945 .. D 903 ....................... B 1038 .. D 896 ........................... B 1021 ........ C 1025 .... D 1031 .................. B 911 ............................. C 927 ... A 1044 ........ C 875 .......... C 935 . A 930 ......... B 933 ............. D 936 ...................................... C 969 ................. D 1003 .................... D 995 ...... B 937 ................ A 880 .............. C 901 .......... A 1039 ............ D 874 .... A 991 ...... A 961 ........ A 1013 ......................... ................... C 1128 .................. D 1095 .... A 1167 ........... C 1169 ................... B 1189 .......... D 1202 ..... D 1151 ...... B 1153 ... B 1048 ....................... B 1118 ............................................................. D 1181 . C 1148 ... C 1086 ............... A 1074 ...... D 1163 ........... D 1211 ... D 1076 ...... D 1139 ....................... C 1187 ......... C 1198 .... A 1088 .................. C 1140 .......................... D 1099 ..... A 1155 ..................................... D 1050 ......... C 1077 ................. B 1136 ............ B 1158 ....................... B 1094 ..... D 1130 .... D 1080 ........ D 1093 ................. A 1115 .. D 1175 .. B 1084 ........ D 1159 ..... B 1071 ...... D 1078 ....... D 1063 ........... D 1103 ............ C 1123 .... C 1178 ..... A 1134 .... D 1209 ...... D 1194 ......... C 1065 .......... A 1052 ... A 1083 . A 1161 ............................. B 1129 ......... C 1126 ...... A 1066 ............................... D 1157 . B 1117 .. C 1069 ................................................................ A 1203 ........................ C 1205 .. B 1079 ....... D 1186 ............ C 1116 . C 1096 ............... AJAY MATHUR 641 ANSWERS ENDOCRINOLOGY 1045 ......................................................................................................... A 1192 .. D 1064 .. C 1081 .. A 1183 ........ B 1106 ...... A 1072 ........... A 1213 ....... A 1091 ...... D 1199 ............................... C 1133 ........... D 1060 ........... D 1056 ...... C 1193 . A 1107 .. A 1196 ................................................. D 1145 ................... B 1092 ........ D 1119 ......... B 1051 .... D 1170 .. D 1075 . D 1082 ...................... B 1049 ...... D 1154 ................... D 1127 ............... D 1212 ........... D 1208 ............................ D 1206 ............. B 1090 . D 1174 ...................... B 1143 ... D 1062 .... A 1137 .............. D 1200 ............ B 1087 ............ D 1102 ... A 1135 .................. B 1114 ...... D 1097 ......... B 1215 ...... A 1201 ..... C 1138 ............ D 1122 .. D 1188 .......................... A 1054 ..... C 1176 ..... C 1120 . D 1166 ..... A 1173 ......... B 1197 ........ D 1177 .............. B 1110 .... D 1108 ... A 1098 ..... B 1179 ...... D 1152 ......... D 1162 ...................... C 1055 ......................... B 1058 .............. D 1195 .......... A 1121 .................. B 1146 .................... C 1053 ........ A 1144 .. C 1101 ............ C 1089 ........... A 1150 ..... A 1160 ... C 1164 .... D 1214 .. B 1046 ............ D 1204 .. D 1109 ..... B 1141 .................. A 1142 ........... A 1216 .................. D 1112 .............. C 1125 ............................... B 1113 .................... D 1073 ................ D 1131 .................... D 1217 .... D 1184 ............... A 1180 ..............Answer MCQ’s FORMCQ’s FOR MEDICAL MEDICAL PROFESSIONALS PROFESSIONALS BY PROF. D 1132 .................... D 1068 .... D 1104 ............ C 1111 . D ............................ C 1156 ......................... C 1124 ....... B 1182 ........... C 1057 ............... D 1168 ... C 1100 ............... D 1171 ......... D 1059 ................. D 1210 ........ C 1047 ....... A 1190 .................... C 1061 ........... D 1149 ............ D 1070 ............................ A 1085 ..... B 1218 . D 1185 ... A 1147 ................................. A 1207 ..... D 1172 .. D 1191 .. B 1067 ........ B 1165 .......................... D 1105 .......... ........... C 1282 ................... C 1328 .............. A 1335 ........... B 1313 . A 1369 .................. A 1290 ...... C 1361 .... A 1339 ....... C 1338 ...... D 1366 ......... A 1229 ...... A 1348 .. D 1357 ............. A 1247 .................. C 1283 .. A 1355 ........... B 1284 ....... D 1234 ..... C 1240 .. D 1344 .... C 1272 ......... B 1359 ................................... C 1362 ................. A ... D 1241 . C 1269 ......................... D 1250 ............................... D 1358 ......... D 1326 ...................................... D 1280 .............. C 1278 ...... A 1295 .. D 1356 ....................... D 1341 ......... D 1316 ................ A 1252 ............................................. C 1311 ........ C 1309 .................. A 1259 ... D 1346 ......................................... C 1312 .... D 1345 .... D 1264 ........................................ A 1340 .. C 1353 ..................... D 1221 ... AJAY MATHUR Answer ANSWERS ENDOCRINOLOGY 1219 ............. D 1299 ...................................... B 1263 .... D 1342 ................................... C 1232 ..... A 1315 ... D 1231 .. C 1238 .................... B 1297 ..................... D 1279 ................... A 1235 ......... B 1267 ..... C 1223 ........... D 1347 ................ D 1305 ......... B 1224 ... C 1254 ........... D 1334 ......... D 1301 . C 1337 .................... B 1294 ....................... C 1314 ... A 1310 ............. A 1248 . C 1220 ....... D 1296 ..... B 1249 ..... A 1275 ....... C 1237 ... B 1303 ............ B 1293 ..... D 1368 ....................... B 1273 ... D 1281 ..... B 1243 ... D 1236 ........... B 1246 ............. D 1329 ..... D 1370 .... A 1354 ....642 MCQ’s FOR MEDICAL PROFESSIONALS BY PROF... B 1287 ........... B 1228 ........... B 1260 ...... C 1270 ...................... D 1225 ................................... C 1327 ................... B 1351 .......... D 1257 ..... C 1274 .......... A 1333 ........ D 1222 ... B 1266 ....... C 1336 . B 1320 ...................... D 1365 .... D 1298 ........... A 1308 .................. C 1343 .................. B 1318 ....... C 1350 .......................... D 1302 ....... B 1271 ........... D 1352 ......... A 1349 .... D 1230 ....... D 1245 ........... C 1291 ......................... D 1360 ......... D 1319 ........... B 1292 ........ A 1306 .......... D 1289 .............. B 1261 .............................. C 1330 ................ D 1242 . D 1251 ............ B 1277 ................ D 1304 ....... D 1226 ......................................... D 1364 . D 1300 . C 1244 ............. C 1332 ........................ D 1285 .......... C 1323 ........ B 1288 ....... C 1268 ...... D 1239 ............................. D 1276 ............. D 1325 .. D 1307 ............. D 1324 .... B 1255 .... B 1322 .... A 1317 .................. C 1253 . D 1363 . D 1321 .... C 1233 .................. D 1256 ............ B 1262 .............. D 1367 ............... B 1258 .............. C 1227 ....................................... D 1331 ....... C 1265 .................. D 1286 ........ Paratonia C. the antigravity muscles (upper-limb flexors and lower-limb extensors). Increased tone varies irregularly D. ~ one-half C. One-third Chapter 366. has a sudden release after reaching a maximum and predominantly affects and >1015 synapses. ~ one-third B. 182 A. AJAY MATHUR Cardiology 643 A. 100 million C. Affects flexors & extensors equally D. ~ 1010 B. Increased muscle tone C. 3224 Harrison’s 18th Ed. Two-third D. 10 million B. Rigidity B. Anti-gravity stretch Over one-third of 23. 5 Nissl bodies are composed of ? 12 Which of the following about ‘Paratonia’ is false ? A. Affects antigravity muscles C. Endoplasmic reticulum A. Active stretch B. All of the above D. Gravitational stretch D. Spasticity Each mature brain is composed of 100 billion neurons. 182 B. . Passive stretch C. 643 Neurology MCQ’s FORMCQ’s FOR MEDICAL MEDICAL PROFESSIONALS PROFESSIONALS BY PROF. system. Results from Harrison’s 16th Ed. Also called gegenhalten C. 3224 A. Cerebellum Harrison’s 18th Ed. microtubule-associated Rigidity is increased tone that is present throughout the range of motion and affects flexors and protein tau (MAPT) in frontotemporal dementia & alpha-synuclein in Parkinson’s disease represent extensors equally. Familial Alzheimer’s disease A. ~ one-fourth A. Three-fourth 1 What proportion of 23. 182 B. None of the above 6 Synaptic glomeruli are found in ? Harrison’s 16th Ed.000 genes encoded in human genome are expressed in human nervous system ? 8 Tone is the resistance of a muscle to ? Harrison’s 18th Ed. 182 Harrison’s 18th Ed. None of the above 7 What fraction of genes encoded in human genome is expressed in the nervous system ? Paratonia (gegenhalten) is increased tone that varies irregularly related to the degree of relaxation. 9 Increased muscle tone is found in ? 2 Each mature brain is composed of how many neurons ? Harrison’s 18th Ed. None of the above D. None of the above Mutations of amyloid precursor protein in familial Alzheimer’s disease. Frontotemporal dementia B. 10 billion D. B. Present throughout range of motion B. Affects flexors & extensors equally C. Present throughout range of motion D. Lateral geniculate body B. 4 Which of the following is a monogenic disorder ? 11 Which of the following about ‘Rigidity’ is false ? Harrison’s 18th Ed.000 genes encoded in human genome are expressed in human nervous Tone is the resistance of a muscle to passive stretch. Rigidity occurs with certain extrapyramidal disorders such as Parkinson’s monogenic causes of common phenotypes. Parkinson’s disease C. All of the above D. 3224 Harrison’s 18th Ed. Velocity-dependent A. disease. 182 A. Lysosomes C. 2339 disease of frontal lobes. ~ 1025 Spasticity refers to an increase in tone associated with disease of upper motor neurons. Golgi complex B. several million miles of axons and dendrites. Results from disease of frontal lobes D. ~ 1020 D. Occurs with extrapyramidal disorders D. One-half Biology of Neurologic Diseases C. 3224 A. All of the above C. 100 billion 10 Which of the following about ‘Spasticity’ is false ? 3 Each mature brain is composed of how many synapses ? Harrison’s 18th Ed. Spasticity A. Sudden release at maximum B. ~ 1015 C. Mitochondria Harrison’s 18th Ed. 182 Harrison’s 18th Ed. It is present throughout the range of motion and affects flexors & extensors equally. 176 13 Which of the following about ‘Paratonia’ is false ? A. Olfactory bulb A. is velocity-dependent. ~ three-fourth D. All of the above Weakness with decreased tone is called flaccidity. or leg is due to a lesion in ? B. 2 to 10 % 24 Which of the following is false about myopathic weakness ? B. 183 Figure 22-1 A.644 MCQ’s FOR MEDICAL PROFESSIONALS BY PROF. Vestibulospinal pathway D.  motor neurons receive direct excitation from Renshaw cell 18 What proportion of pyramidal axons do not decussate and remain interneurons ipsilateral ? Harrison’s 18th Ed. Due to disorders of muscle fibers D. Bulbospinal system is also called extrapyramidal upper motor neuron system. Reticulospinal pathway D. Posterior limb of internal capsule D. Absent tendon stretch reflex suggests involvement of spindle In UMN lesions.30% remain ipsilateral in anterior spinal cord. Due to defect in neuromuscular junctions At cervicomedullary junction. Visible twitch Harrison’s 18th Ed. Harrison’s 18th Ed. most pyramidal axons decussate into contralateral cortico-spinal C. Decreased tone B. It occurs with disorders of motor units which Descending ventromedial bulbospinal pathways include tectospinal. vestibulospinal. Descending ventrolateral bulbospinal pathways. Reticulospinal pathway Fasciculations are visible or palpable twitch within a muscle due to spontaneous discharge of a motor D. 183 Figure 22-1 A. arm. Tectospinal pathway C. Distribution of weakness is distal 19 Pyramidal axons innervate most densely which of the following Distribution of myopathic weakness is typically proximal. reticulospinal pathways. afferent fibers D. 182 22 Which of the following is false about lower motor neuron weakness ? A. 182 21 All of the following are part of descending ventromedial bulbospinal pathways except ? A. facilitate distal limb muscles. Harrison’s 18th Ed. Fasciculations signify anterior horn cell disease 17 Upper motor neurons have their cell bodies in which layer of Loss of -motor neurons does not cause weakness but decreases muscle tone and attenuates the primary motor cortex ? stretch reflexes elicited on examination. Normal movement rhythmicity is maintained C. motor neurons innervate intrafusal muscle fibers C. Of hand muscles 25 “Pure motor” hemiparesis of the face. 183 Figure 22-2 C. None of the above C. 183 Figure 22-1 -motor neurons receive direct inhibition from Renshaw cell interneurons. Spontaneous discharge of a motor unit B. Table 22-1 C. or Brodmann’s area 4) and in premotor & supplemental motor cortex (area 6). Of face muscles B. Layer IV Harrison’s 18th Ed. weakness ? Harrison’s 18th Ed. Of foot muscles Harrison’s 18th Ed. 10 to 30 % A. Of axial muscles A. 184 C. In EMG. Layer V A. AJAY MATHUR Neurology 14 Which of the following about ‘flaccidity’ is false ? 20 Descending ventromedial bulbospinal pathways include ? Harrison’s 18th Ed. Cerebral peduncle Pyramidal neurons innervate most densely the LMN’s of hand muscles. 183 Figure 22-1 B. Palpable twitch A. A. Rubrospinal pathway unit. Affects ability to perform rapid repetitive movements B. Due to loss of  neurons C. Lower motor neuron weakness is most lower motor neurons ? profound distally. C. which originate predominantly in red nucleus 16 Which of the following is false about upper motor neuron (rubrospinal pathway). Layer III 23 Which of the following is false about motor neurons ? B.  motor neuron receives direct excitatory input from Cell bodies of UMN’s is in layer V of primary motor cortex (precentral gyrus. 5 to 15 % Harrison’s 18th Ed. and consists of a single lower motor neuron and all of the muscle fibers that are innervated by it. Axial movements spared A. None of the above D. but 10 . Vestibulospinal pathway C. Upper pons . These pathways influence axial & proximal muscles and are involved in the maintenance of posture and integrated movements of the limbs and trunk. Layer VI B. Due to loss of  motor neurons D. size of each motor unit action potential is reduced tract of lateral spinal cord. 182 Harrison’s 18th Ed. 183 Figure 22-1 A. 182. 20 to 35 % B. Disorder of motor unit C. Weakness A. motor neurons innervate extrafusal muscle fibers D. distal muscle groups are affected more severely than proximal ones. Proximal muscle groups affected more than distal Harrison’s 18th Ed. D. corticomotoneurons & primary muscle spindle afferents D. 182 B. Tectospinal pathway B. 15 Which of the following is false about fasciculation ? Harrison’s 18th Ed. 30 Primary sensory & motor areas constitute what percentage of 36 Which of the following is preserved in Broca’s aphasia ? cerebral cortex ? Harrison’s 18th Ed. 20 % D. 203 28 Cerebral cortex of human brain contains approximately how many A. Repetition of spoken language 32 Anomia refers to ? B. 203 D. All of the above D. 645 Neurology MCQ’s FORMCQ’s FOR MEDICAL MEDICAL PROFESSIONALS PROFESSIONALS BY PROF. Anomia B. Comprehension A. 3. Wernicke’s aphasia B. Naming C. 1. 205 A. A. Alexia Harrison’s 18th Ed. 202 A. Harrison’s 18th Ed. Deficit of naming “Pure motor” hemiparesis of face. Paraphasia C. Any of the above A. 5 billion C. cerebral peduncle or upper pons. 203 Table 26-1 Harrison’s 18th Ed. Anterior horn cell disorders 33 Anomic aphasia is the single most common language disturbance B. agraphia (or dysgraphia) is used to describe an acquired deficit in the spelling or grammar of written language. Aphemia 31 A language disturbance occurring after a right hemisphere lesion in a right hander is called ? B. 203 Table 26-1 Harrison’s 18th Ed. 15 % C. 203 C. Cortical venous thrombosis C.5 m2 Comprehension. 184 the wrong word is called paraphasia. Head trauma B. Crossed aphasia D.5 m2 B. Impaired comprehension A. AJAY MATHUR Cardiology 645 D. Sequence aphasia Aphemia is an acute onset of severely impaired fluency. 0. Peripheral neuropathies Harrison’s 18th Ed. Ipsilateral aphasia D. and Alzheimer’s disease. Acute hydrocephalus 34 Inability to read aloud or comprehend single words and simple D.5 m 2. arm. 205 Language disturbance occurring after a right hemisphere lesion in a right hander is called crossed aphasia. Disorder of initiating skilled / learned movement Harrison’s 18th Ed. 202 A. repetition of spoken language and naming are impaired in Wernicke’s aphasia. 184 D. Any of the above 26 Paraparesis can arise due to ? Deficit of naming or anomia is the single most common finding in aphasic patients. Naming with the wrong word D. Human cerebral cortex contains ~20 billion neurons spread over an area of 2. Anomia A. metabolic encephalopathy. No purposeful speech . A. Alzheimer’s disease of cerebral hemispheres ? Harrison’s 18th Ed. Naming with Harrison’s 18th Ed. Metabolic encephalopathy 27 Acute paraparesis can be due to which of the following diseases C. All of the above A. Repetition of spoken language B. Any of the above B. Fluency is preserved or increased. Naming C. 20 billion Alexia describes an inability to either read aloud or comprehend single words and simple sentences. Naming a different object capsule. 203 C. 10 billion D. Agraphia Harrison’s 18th Ed. 10 % B. 2. Retro aphasia 38 Apraxia refers to ? Harrison’s 18th Ed. or leg is due to lesion in posterior limb of the internal C. B.5 m2 D. Superior sagittal sinus thrombosis Anomic aphasia is the single most common language disturbance seen in head trauma. Fluency D. Comprehension A. 25 % 37 Fluency is severely impaired in ? Primary sensory & motor areas constitute 10% of the cerebral cortex. D. Cauda equina syndrome seen in ? C. 50 billion 35 Which of the following is impaired in Wernicke’s aphasia ? 29 Cerebral cortex of human brain has an area of ? Harrison’s 18th Ed. C. Alexia neurons ? B. D. Any of the above sentences is called ? Harrison’s 18th Ed. Repetition of spoken language B.5 m2 C. 202 A. C. Extrapyramidal dysfunction D.fourth Harrison’s 17th Ed. corticobasal degeneration. Aphasia prosopagnosia & visual object agnosia consist of bilateral infarctions in the territory of posterior cerebral arteries.half 42 Paratonic rigidity or gegenhalten results from disease of ? D. 168 D. 206 A. Visual object agnosia 40 Gerstmann’s Syndrome includes all except ? C. Parkinson’s disease Harrison’s 17th Ed.646 MCQ’s FOR MEDICAL PROFESSIONALS BY PROF. Right-left confusion D. A. Acalculia Face & object recognition deficits are known as prosopagnosia & visual object agnosia. Occipital lobes A. Aprosodia Harrison’s 18th Ed. It is a disorder of planning & initiating a skilled or learned A. Simultagnosia 51 Pill-rolling tremor is a characteristic feature of ? D. Parietal lobes Harrison’s 18th Ed. Inferior parietal lobule in non-dominant hemisphere A. 206 twisting movements and abnormal posture. Corticobasal degeneration 45 Bálint’s syndrome may result from ? D. One .third C. Hypoglycemia Harrison’s 18th Ed. Prosopagnosia A. Temporal lobe seizures 41 In isolated Gerstmann’s syndrome. All of the above A. 147 A. B. progressive supranuclear palsy. Progressive supranuclear palsy D. 193 D. Ill sustained muscle contractions Paratonia (or gegenhalten) is increased tone that varies irregularly in a manner that may seem B. extrapyramidal. Sagittal sinus thrombosis A. Frontal lobes ~ One-third of all human cerebral cortex is located in the frontal lobes. B. One . Corticobasal degeneration Harrison’s 17th Ed. Aprosodia A. Primary progressive aphasia (PPA) movement unrelated to a significant motor or sensory deficit. It often has a genetic basis. All of the above Harrison’s 18th Ed. Multiple system atrophy in the center of gaze with more peripheral information. None of the above 46 Face recognition deficit is termed as ? Harrison’s 18th Ed. Lesions in B. Inferior parietal lobule in dominant hemisphere 48 What proportion of all human cerebral cortex is located in the C. Abnormal posture D. Primary progressive aphasia (PPA) B. and primary pallidal degeneration. AJAY MATHUR Neurology 39 In apraxia. complex motor deficit is attributed to ? A. Oculomotor apaxia atrophy. 210 Gerstmann tetrad comprises of acalculia. Progressive supranuclear palsy scanning of the environment (oculomotor apraxia) and in accurate manual reaching toward visual targets (optic ataxia) and simultanagnosia and reflects an inability to integrate visual information B. 209 Apraxia refers to a complex motor deficit that cannot be attributed to pyramidal. C. Often has a genetic basis 43 Statements uttered in a monotone are termed ? Dystonia is a disorder characterized by sustained muscle contractions. Cerebellar dysfunction D. Finger anomia 47 Cause of transient global amnesia is ? Harrison’s 18th Ed. Optic ataxia Freezing gait is a feature of Parkinson’s disease. Three . Pure Gerstmann syndrome is without aphasia. 206 D. Prosopagnosia B. 193 C. cerebellar. Repetitive twisting movements related to the degree of relaxation. Superior parietal lobule in non-dominant hemisphere frontal lobes ? Harrison’s 18th Ed. Temporal lobes 49 Characteristics of dystonia include all except ? C. multiple system B. Pyramidal dysfunction C. 210 D. it usually results from disease of the frontal lobes. 168 . finger anomia and right-left confusion. resulting in repetitive Harrison’s 18th Ed. Anomic aphasia 50 Freezing gait is a feature of ? Harrison’s 18th Ed. All of the above B. B. is present throughout the range of motion. C. A. alexia. Multiple system atrophy 44 Which symptom can occur in Bálint’s syndrome ? C. 205 B. C. Migraine B. the damage is in ? C. TIA in the posterior cerebral territory Harrison’s 18th Ed. Atypical forms of Alzheimer’s disease B. One . and affects flexors and extensors equally.fourth Isolated Gerstmann’s syndrome results from damage to inferior parietal lobule (angular gyrus) in the left hemisphere. All of the above C. 193 Bálint’s syndrome is a state of severe spatial disorientation involving deficits in orderly visuomotor A. or sensory dysfunction. Superior parietal lobule in dominant hemisphere D. 191 Hyperesthesia means pain or increased sensitivity in response to touch. 3 mm B. Imbalance Harrison’s 18th Ed. Hyperalgesia Hemisection of spinal cord produces Brown-Séquard syndrome. Sensory ataxia simultaneous stimulation & tests for graphesthesia and stereognosis. 5 mm C. motor function & deep tendon reflexes. Fear of pain Small-fiber polyneuropathies are characterized by burning. encompasses all the phenomena described by hyperesthesia. Sensory ataxia 59 Cortical sensation are tested by ? Harrison’s 18th Ed. None of the above C. Position sense . Burning D. Absent tendon reflexes A. painful dysesthesias with reduced pinprick & thermal sensation but sparing of proprioception. but once felt. Like. Loss of sensory function (diminished or absent feeling) is termed negative phenomena experienced 61 Which of the following is false about small-fiber polyneuropathy ? as numbness and abnormal finding on sensory examination. 647 Neurology MCQ’s FORMCQ’s FOR MEDICAL MEDICAL PROFESSIONALS PROFESSIONALS BY PROF. Hyperesthesia C. imbalance. 187 B. Numbness C. Absent tendon reflexes C. Tingling B. 188 53 “Astasia-abasia” best relates to ? A. Bilateral simultaneous stimulation A. 188 54 Which of the following is not a positive sensory symptom ? Harrison’s 18th Ed. AJAY MATHUR Cardiology 647 52 Term “gait apraxia” refers to ? B. 1 mm A. minimum distance in 2-PD test ? Harrison’s 18th Ed. All of the above C. 193 C. D. 194 B. with absent pain & temperature D. 256 Hz Harrison’s 18th Ed. Psychogenic gait disorder Commonly used tests of cortical function are two-point discrimination. Warm or cold stimuli 62 Which of the following is false about large-fiber polyneuropathy ? D. Sparing of motor function B. 186 C. Vibration & position sense deficits 56 Allodynia refers to ? B. 191 55 Hyperesthesia means pain or increased sensitivity in response to? A. Any of the above B. painful dysesthesias Harrison’s 18th Ed. All of the above sensation contralaterally & loss of proprioceptive sensation & power ipsilaterally below the lesion. Nonpainful response to painful stimulus absent tendon reflexes & variable motor dysfunction but preservation of most cutaneous sensation. Cautious gait C. Pain D. Cerebellar gait ataxia Sense of vibration is tested with a tuning fork that vibrates at 128 Hz. allodynia. is unduly painful. C. Loss of proprioceptive sensation & power ipsilaterally B. With hyperpathia. 186 B. Burning. Touch C. 7 mm D. Cautious gait D. Hyperpathia. Pinprick & temperature 58 Sense of vibration is tested with a tuning fork that vibrates at ? Harrison’s 18th Ed. Frontal gait disorder D. Graphesthesia B. 128 Hz C. 512 Hz A. Pain on imagination D. Harrison’s 18th Ed. 191 57 Hyperpathia includes which of the following ? A. Light touch A. Pricking Normal individuals can distinguish separation of points by 3 mm distance in two-point discrimination test. Harrison’s 18th Ed. elicitation of a painful sensation 63 Which of the following is false about Brown-Séquard syndrome ? by application of a vibrating tuning fork. Hysterical gait disorders with odd gyrations of posture & wastage of muscular energy (astasia- abasia). Cerebellar gait ataxia D. 186 A. A. Sparing of proprioception A. Two-point discrimination Harrison’s 18th Ed. Painful response to nonpainful stimulus Large-fiber polyneuropathies are characterized by vibration & position sense deficits. C. Harrison’s 18th Ed. 191 A. Absent pain & temperature sensation contralaterally A. a broad term. bilateral D. extreme slow motion & dramatic fluctuations over time are seen in somatoform disorders 60 Normal individuals can distinguish separation of points by what & conversion reaction. the threshold for a sensory stimulus is increased and perception 64 Which of the following sensory loss occurs in syringomyelia ? is delayed. Allodynia D. Painful response to nonpainful stimulus is termed allodynia. touch localization. Hemisection of the spinal cord Harrison’s 18th Ed. All of the above Harrison’s 18th Ed. 186 B. and hyperalgesia. None of the above B. Cooperation of patient Headache D. 134 A. Colour of nails 69 In adults. Radiation down the back & into the legs C. 280 . D. 292 .280 mm H2O Pain-producing cranial structures include scalp. ~ 150 ml C. falx 71 Normal CSF osmolarity is ? cerebri and proximal segments of large pial arteries. pial veins. Choroid plexus B. 285 . Have high energy demands A.7. EP-3 is essential for fever. 120 . 7. Harrison’s 16th Ed. 7. EP-2 Harrison’s 18th Ed. Dural sinuses C. Produce relatively high force 68 Which of the following is labeled as the “Fifth vital sign” ? Harrison’s 17th Ed. A. position sense and vibration appreciation are preserved.7.70 mm H 2O B.34 B. All of the above D. causes fibers ? pansensory loss contralaterally. 145 deficiency syndrome & Cisplatin toxicity can also cause it. normal CSF volume is ? 76 Which cranial structure is pain-insensitive ? Harrison’s 18th Ed. Poor in oxidative enzymes A. 191 Harrison’s 17th Ed.7. 191 B.44 In Lhermitte’s sign.285 mOsm/L . choroid plexus. Falx cerebri D. and much of the brain parenchyma are not pain-producing. 7.37 C. Skin hue C. Produce relatively low force C. A.120 mm H2O C. Ventricular ependyma A. Cervical spinal cord D. 70 C. Light touch. A 11 A. ~ 200 ml D. Choroid plexus B.40 . 112 Harrison’s 17th Ed.7. AJAY MATHUR Neurology D. Cervical spinal cord C.648 MCQ’s FOR MEDICAL PROFESSIONALS BY PROF. Flexion of the neck A. Ventricular ependyma. Tegmentum of pons & midbrain 75 Which of the following statements is false about Type II muscle D. ~ 250 ml 77 Which cranial structure is pain-sensitive ? 70 Normal CSF pressure is ? Harrison’s 18th Ed. cervical spondylosis or recent irradiation to the cervical region.180 mm H2O D. Scalp A. It is seen in patients with a cervical lesion affecting posterior columns like 73 Which receptor for PGE2 in brain is essential for fever ? multiple sclerosis.31 . Tegmentum of pons & midbrain Out of the 4 receptors for PGE 2.302 mOsm/L 65 Which of the following is related to Lhermitte’s sign ? 72 Normal CSF pH is ? Harrison’s 18th Ed. where lemniscal & spinothalamic tracts merge. Thalamus D. Rich in glycolytic enzymes B. D.40 D. Have low energy demands B.297 mOsm/L appreciation. Vitamin B 12 Harrison’s 18th Ed. 7. ~ 100 ml B. EP-3 A. A 11 A. 20 . Vibration B. Lateral medulla 74 Which of the following statements is false about Type I muscle A lesion in tegmentum of pons & midbrain. Pain D. Rich in mitochondria 67 The site of lesion in Déjerine-Roussy syndrome is in ? Harrison’s 18th Ed.37 . EP-1 66 Pansensory loss contralaterally is produced by a lesion in ? B. Lateral medulla fibers ? Harrison’s 16th Ed. Pial veins C. 80 . None of the above B. A 11 A. Electric shock like sensation B. Thalamus Elevation of PGE2 in brain starts the process of raising hypothalamic set point for core temperature. C. Harrison’s 17th Ed. flexion of neck leads to an electric shock like sensation that radiates down the back and into the legs.292 mOsm/L In syringomyelia there is a dissociated sensory loss with impairment of pinprick & temperature C. dural sinuses. 50 . Falx cerebri D. EP-4 B. 112 Harrison’s 17th Ed. 134 Lesion affecting VPL nucleus of thalamus produces a syndrome of thalamic pain called Déjerine- Roussy syndrome. 298 . middle meningeal artery. A 11 A.34 . 191 C. 113 A. Activators of migraine are referred to as triggers. DRD2 C. brain tumors. and 5-HT1F receptors and are less potent at the 5- Relative frequency of various types of primary headache is migraine 16%. which one is the 86 Triptans are least potent agonist of ? most common ? Harrison’s 18th Ed. Pregnancy D. Temporal arteritis Mutations involving the Ca v2. vascular disorders 1%. 80 Of the following cause of primary headache. head injury located on both blood vessels and nerve terminals. 4%. tRNALeu(UUR) B. 89 A. 5-HT1B receptors B. or other whereas secondary headaches are those caused by exogenous disorders. 112 Table 14-1 A. 5-HT1A receptors A. 112 Table 14-1 A. excess stress. subarachnoid hemorrhage <1% and brain tumor 0. which are Relative frequency of various types of secondary headache is systemic infection 63%. Exertional Triptans are potent agonists of 5-HT1B. Subarachnoid hemorrhage D. tension-type 69%. Tension-type D. 5-HT1F receptor D. 115 Harrison’s 18th Ed. Sleep B. Menstruation C. physical exertion. Pregnancy C. Migraine B. . All of the above D. Tension-type B. Hunger Primary headaches are those in which headache and its associated features are the disorder in itself. Vomiting that precedes appearance of headache by weeks is highly characteristic of posterior fossa Mutations in neuronal voltage-gated sodium channel SCN1A cause FHM 3. 112 Table 14-1 Harrison’s 17th Ed. Cluster headache B. 5-HT1D receptor C. idiopathic stabbing 2% and exertional 1%. None of the above D. 5-HT1C receptors C. bright lights. subarachnoid hemorrhage & brain tumor. Mutations in Na +-K +ATPase ATP1A2 gene causes FHM 2. cluster 0. 5-HT1B receptor B. sounds. AJAY MATHUR Cardiology 649 78 Which of the following is a type of primary headache ? 84 Migraine activators include all except ? Harrison’s 18th Ed. tRNALeu(UUR) A. lack of or excess sleep & alcohol or other chemical are systemic infection. 115 highly characteristic of ? Harrison’s 18th Ed. 649 Neurology MCQ’s FORMCQ’s FOR MEDICAL MEDICAL PROFESSIONALS PROFESSIONALS BY PROF. Examples of secondary headache changes. Examples of primary headache afferent stimulation. vascular disorders. 5-HT1A receptor A. tension-type. HT 1A receptor. head injury.1 (P/Q) type voltage-gated calcium channel CACNA1A gene cause familial hemiplegic migraine 1 (FHM 1). Exhiliration D. Cluster C.1%. All of the above Migraine & other primary headache types are not vascular headaches.1%. idiopathic stabbing & exertional. stimulation. Idiopathic stabbing A. 96 A. Subarachnoid hemorrhage C. Posterior fossa brain tumors D. Cluster D. hormonal fluctuations during menses. All of the above D. Systemic infection B. 113 Harrison’s 16th Ed. CACNL 1A4 C. Meningitis B. 5-HT 1D. Migraine A. stormy weather or barometric pressure are migraine. hunger. Exertional B. Brain tumor Antimigraine efficacy of triptans relates to their ability to stimulate 5-HT 1B/1D receptors. DRD2 D. 115 Harrison’s 18th Ed. Migraine A. 96 A. 5-HT1E receptors D. 112 Table 14-1 Harrison’s 17th Ed. 88 Which of the following gene has been incriminated in the causation 82 Which of the following is not a vascular headache ? of migraine ? Harrison’s 18th Ed. Systemic infection C. cluster. Tension-type headache C. Vascular disorders C. 89 Which of the following gene has been incriminated in the causation of Familial hemiplegic migraine (FHM) ? 83 Vomiting that precedes the appearance of headache by weeks is Harrison’s 18th Ed. Red wine B. CACNA1A B. Tension-type C. 87 Antimigraine efficacy of the triptans is due to their ability to 81 Of the following cause of secondary headache. Lack of sleep 79 Which of the following is not a type of primary headache ? 85 Migraine deactivators include all except ? Harrison’s 18th Ed. These include glare. which one is the stimulate ? most common ? Harrison’s 18th Ed. D. Propranolol and amitriptyline are ineffective C. Nausea or vomiting TTH patients have headaches without accompanying features like nausea. Unruptured aneurysms 101 Which of the following is false about cluster headache ? 95 Chronic head-pain syndrome best relates to ? Harrison’s 18th Ed. C. SUNCT B. Migraine B. Primary stabbing headache A. Brain tumors D. C. None of the above B. 120 A. Globus pallidum primary stabbing headache & hypnic headache. Selective serotonin reuptake inhibitors Harrison’s 18th Ed. Putamen Short-lasting headaches without prominent cranial autonomic syndromes are trigeminal neuralgia. All of the above 102 Which of the following is false about cluster headache ? Term tension-type headache (TTH) is commonly used to describe a chronic head-pain syndrome Lancet 2005. Migraine Diagnosis Assessment Score 98 Trigeminal autonomic cephalalgias include all except ? Harrison’s 18th Ed. maintained for several hours 100 Which of the following is false about cluster headache ? to days. 122 93 Electrical stimulation of which part of brain results in migraine- like headache ? A. Cluster headache B. Men are affected more than women A. Amitriptyline 91 MIDAS stands for ? B. Photophobia. Cluster headache D. Tension-type headache (TTH) C. Temporal lobe C. Anti-inflammatory agents D. D. 122 MIDAS stands for Migraine Disability Assessment Score meant to assess the extent of a patient’s disease and disability. A. Certain foods or emotional factors precipitate pain C. Benzodiazepines A. Acupuncture B. bandlike discomfort. Circadian rhythmicity of painful attacks 96 Which of the following is an accompanying feature of tension. 122 Harrison’s 18th Ed. 86 A. 115 Figure 14-1 B. throbbing & aggravation with movement. Paroxysmal hemicrania A. A. 5HT 1B/1D receptor agonists 99 Short-lasting headaches without prominent cranial autonomic and dopamine receptor antagonists. osmophobia. Lithium is not beneficial D. Migraine Disease Assessment Score D. Ipsilateral autonomic features B. syndromes include ? Harrison’s 18th Ed. Migraine 92 Drugs effective in treatment of migraine are ? Harrison’s 18th Ed. Headache phonophobia. AJAY MATHUR Neurology 90 In acephalgic migraine. 5HT1B/1D receptor agonists Trigeminal autonomic cephalalgias (TACs) includes cluster headache. Nausea. relieved by sleep are sugestive of ? Harrison’s 18th Ed. Midbrain in the region of dorsal raphe D. Cluster headache C. Hypothalamic dysfunction has no role in acute attacks . 94 Headaches that build up over hours. vomiting. Autonomic symptoms type headache ? Harrison’s 18th Ed. All of the above C. Vertigo may be prominent. Migraine Disability Assessment Score For chronic TTH. which of the following is least prominent? B.650 MCQ’s FOR MEDICAL PROFESSIONALS BY PROF. Harrison’s 18th Ed. Drugs effective in the treatment of migraine are anti-inflammatory agents. vomiting D. photophobia. B. Severe unilateral pain A. with nausea or vomiting. Migraine B. 366: 843–55 characterized by bilateral tight. which is the only proven treatment ? Patients with acephalgic migraine have recurrent neurologic symptoms. 120 C. Migraine Drug Assessment Score D. phonophobia Harrison’s 18th Ed. 116 C. paroxysmal hemicrania & SUNCT (short-lasting unilateral neuralgiform headache attacks with conjunctival injection & tearing). Hypnic headache B. Pain is strictly unilateral A. 119 C. Aggravation with movement A. C. 122 Harrison’s 16th Ed. D. 122 but with little or no headache. Vertigo 97 For chronic TTH. Pain affects the same side in subsequent episodes B. or nasal congestion). Dopamine receptor antagonists TAC features are short-lasting attacks of head pain with cranial autonomic symptoms (lacrimation. 116 C. On-off vulnerability to alcohol D. Trigeminal neuralgia Harrison’s 18th Ed. Recurrent neurologic symptoms D. amitriptyline is the only proven treatment. A. All of the above conjunctival injection. 651 Neurology MCQ’s FORMCQ’s FOR MEDICAL MEDICAL PROFESSIONALS PROFESSIONALS BY PROF. Valsalva manuevre B. Equal male : female ratio C. Cyproheptadine C. Brain tumors carbamazepine offer modest benefit. 7 days per month C. Relief by jugular vein compression D. Ipsilateral conjunctival injection and lacrimation D. Lamotrigine Harrison’s 18th Ed. Worsened by jugular vein compression B. C. All of the above Harrison’s 18th Ed. 124 A. Cluster headache Most effective treatment for prevention of SUNCT is lamotrigine. Cyproheptadine A. Prednisone A. Methysergide D. Periodicity 105 Drugs used for preventive treatment of cluster headache include all except ? 112 Which of the following is false about lumbar puncture headache ? Harrison’s 18th Ed. Unruptured aneurysms . Epidural blood patch is effective treatment D. 124 A. Unilateral pain A. Very frequent short-lasting attacks B. Anterior communicating artery aneurysm B. Post head injury 115 Headache on rising in the morning or nocturnal headache is characteristic of ? C. Lithium B. 3 days per month B. Poorly controlled hypertension tearing) can be seen with posterior fossa or pituitary lesions. Headache on most if not all days C. D. AJAY MATHUR Cardiology 651 103 Which of the following is most useful during acute attack of 110 Chronic daily headache (CDH) means headache for how many cluster headache ? days per month ? Lancet 2005. Lithium B. Very frequent short-lasting attacks D. A pain-free interval B. A. 125 106 Which of the following is a feature of paroxysmal hemicrania ? Harrison’s 18th Ed. All of the above 114 Which of the following is not a feature of lumbar puncture headache ? 107 Which of the following is a feature of SUNCT ? Harrison’s 18th Ed. 122 Harrison’s 18th Ed. Hyperventilation C. gabapentin and C. methysergide. All of the above Low CSF volume headache is positional (recumbency improves headache within minutes). 15 days per month 104 Which of the following is a feature of cluster headache ? 111 Which of the following is a feature of new daily persistent headache Harrison’s 18th Ed. 122 (NDPH) ? A. Obstructive sleep apnea SUNCT (short-lasting unilateral neuralgiform headache attacks with conjunctival injection and B. Aggravated by abdominal compression Oral glucocorticoids. 126 D. Hypertension A. 113 Which of the following is false about lumbar puncture headache ? Harrison’s 18th Ed. lithium are useful for chronic form of cluster headache. 10 days per month D. Migraine D. Begins when the patient sits or stands upright B. Ipsilateral conjunctival injection and lacrimation B. abdominal binder & oral theophylline provide temporary relief. 108 Secondary SUNCT is seen in ? occipitofrontal. Worsened by head shaking A. All of the above C. 125 B. 124 rather than low CSF pressure. Lack of a response to indomethacin D. Symptoms result from low CSF volume Harrison’s 18th Ed. Raised CSF pressure headache 109 Most effective treatment for prevention of SUNCT is ? D. 124 A. 114 C. Prednisone 116 Sleep disruption and early morning headaches that improve during the day are characteristics of ? B. 100% oxygen inhalation A. Is dramatically positional D. 366: 843–55 Harrison’s 18th Ed. Posterior fossa or pituitary lesions Harrison’s 18th Ed. Usually begins within 48 hours B. 125 Harrison’s 18th Ed. Incidence is between 10 and 30 % C. 125 A. Relief by abdominal compression C. Periodicity Harrison’s 18th Ed. Excellent response to indomethacin D. Topiramate. 124 A. All of the above D. Location is bitemporal C. Dramatically positional A. Lack of refractory period to triggering C. begins within 48 hours following lumbar puncture (incidence 10-30%) & caffeine. At least three attacks per month Abulia is a milder form of akinetic mutism characterized by mental & physical slowness with diminished ability to initiate activity. 90 B. stupor. Basilar migraine Harrison’s 18th Ed. drowsiness and coma are precise narrative descriptions of the level of arousal A. Ventral pons 122 Out of the following. 90 127 At bedside. Full recovery after the episode is the rule 126 Which of the following drugs for treatment of acute migraine can 119 Methysergide may cause retroperitoneal or cardiac valvular be administered by nasal route ? fibrosis when used for more than ? Harrison’s 18th Ed. Obtundation Harrison’s 16th Ed. Migraine D. Akinetic mutism . Timolol Harrison’s 16th Ed. what should A. Verapamil B. 120 A. Cluster headache C. Coma 120 Lower-half headache or facial migraine called ? Harrison’s 16th Ed. or obtundation. At least three attacks per day C. 90 A. Raeder’s syndrome B. All of the above D. Vomiting D. Episodes begin with total blindness B.30 minutes C. 124 Drugs used for prophylactic treatment of migraine include all 130 Which of the following occurs as part of a major psychosis ? except ? Harrison’s 18th Ed. 2247 Harrison’s 18th Ed. It is usually the result of damage to frontal lobes and its D. Dental trauma C. 120 A. Sumatriptan B. Locked-in state C. Zolmitriptan C. Metoclopramide C. AJAY MATHUR Neurology 117 “Fortification spectrum” is characteristic of ? A. Dihydroergotamine A. 12 months Methysergide may cause retroperitoneal or cardiac valvular fibrosis when it is used for > 6 months. 2247 123 For instituting prophylactic treatment of migraine. Akinetic mutism Harrison’s 16th Ed. B. Cyproheptadine D. Risk of fibrosis is about 1:1500 and is likely to reverse after drug is stopped. 6 months C.652 MCQ’s FOR MEDICAL PROFESSIONALS BY PROF. Sodium valproate A. Medial thalamic nuclei accompanying severe migraine ? C. 2247 D. Photophobia D. At least three attacks per year connections. Catatonia B. Lethargy C. ataxia) persist for 20 . 3 months B. Hypertension 128 Akinetic mutism results from damage in ? Harrison’s 18th Ed. Temporal lobe Harrison’s 16th Ed. Chapter 274. Occurs only in males D. Symptoms (vertigo. 9 months D. Giant cell arteritis 125 Drugs used for prophylactic treatment of migraine include all except ? 118 Which of the following is false about Bickerstaff’s migraine ? Harrison’s 18th Ed. Semicoma 121 Common precipitant of carotidynia attacks is ? D. which of the following terms is not ambiguous and is preferred ? A. Drowsiness D. Parkinsonism be the frequency of attacks ? B. Vertigo 129 Abulia describes a milder form of ? Harrison’s 18th Ed. At least three attacks per week D. which is the commonest symptom B. Cervical spondylosis and of the type of responses evoked by various stimuli and are preferable to ambiguous terms such as lethargy. 2247 B. Any of the above B. 90 A. semicoma. 93 A. C. All of the above A. Pseudotumour cerebri D. Carotidynia A. Sumatriptan C. Scalp tenderness Akinetic mutism results from damage in regions of medial thalamic nuclei or frontal lobes (particularly orbitofrontal surfaces) or from extreme hydrocephalus. Methysergide B. 118 Table 14-5 Harrison’s 18th Ed. Amitriptyline A. Opthalmoplegic migraine C. 120 Harrison’s 16th Ed. 91 At bedside. Abulia D. D. 134 “Locked-in state” like condition may be seen in ? Harrison’s 18th Ed. Medulla oblongata C. Ventral pons N Engl J Med 2001. All of the above C. Midbrain D. Loss of vertical & adduction movements of eyes C. Catatonia Metabolic derangements that can lead to suppression of reticulocerebral function are hypoglycemia. 2247 A. 2247 138 The term ‘coma dépassé’ means ? N Engl J Med 2001. Coma in children In catatonia. 653 Neurology MCQ’s FORMCQ’s FOR MEDICAL MEDICAL PROFESSIONALS PROFESSIONALS BY PROF. Loss of light reaction B. which is the last part of brain to cease to function ? A. anoxia.344:1215 A. Abulia A. As brain death occurs. Pharmacologic neuromuscular blockade B.” or catalepsy). Dynamic radionuclide brain scan Coma results due to damage in reticular activating system (RAS) at the level of upper midbrain or C. Pons N Engl J Med 2001. AJAY MATHUR Cardiology 653 B. Uremia A. the patient retains the posture in which they have been placed by the examiner (“waxy flexibility. and hepatic failure. Pupillary enlargement A. Harrison’s 18th Ed. Locked-in syndrome C. uremia. Irreversible coma C.344:1215 B. 132 “Waxy flexibility” is a feature of which of the following ? Harrison’s 18th Ed. 133 “Locked-in state” results from damage in ? Harrison’s 18th Ed.344:1215 C. All of the above D. Hypothermia D. Babinski signs C. brainstem reflexes & respiration & whose EEGs were flat. Pons . Akinetic mutism B. “hollow-skull sign” is a finding in which of the following ? B.344:1216 B. 2247 A. On the other hand. Hepatic failure B. Harrison’s 18th Ed. EEG D. Locked-in state Catatonia occurs as part of a major psychosis (schizophrenia or major depression). Hysteria B. if pupillary light Babinski signs and hypertonicity of the limbs present in akinetic mutism. Medulla oblongata descending motor (corticospinal and corticobulbar) pathways. Medial thalamic nuclei A. 2247 Harrison’s 18th Ed. D. Mollaret & Goulon introduced the term coma dépassé (irreversible coma) in 23 comatose patients who lost consciousness. Coma during pregnancy D. Medulla oblongata 141 In the tests to confirm brain death. Cerebral angiography its projections. Drug intoxication 135 Coma results due to damage in reticular activating system (RAS) D. a dynamic radionuclide brain scan shows no intracranial filling - reticulocerebral function is ? the so called hollow-skull sign. patients lose their reflexes in a rostral-to-caudal direction. In 1959. Guillain-Barré syndrome N Engl J Med 2001. Any of the above C. Hypoglycemia 142 Which part of brain is damaged in “locked-in syndrome” ? N Engl J Med 2001. 137 Which of the following finding suggests that the lesion is in the upper brainstem ? 131 Catatonia is differentiated from akinetic mutism by ? Harrison’s 18th Ed. Locked-in state D. Pons “Locked-in state” results from an infarction or hemorrhage of ventral pons that transects all D. Mobility Pupillary enlargement with loss of light reaction and loss of vertical and adduction movements of Catatonia is differentiated from akinetic mutism by clinical evidence of cerebral damage such as the eyes suggests that the lesion is in the upper brainstem. Transcranial Doppler measurements 136 Metabolic derangement that can lead to suppression of When brain death has occurred. and medulla oblongata is the last part of brain to cease to function. Blinking D. the lesion is more widespread or the cause of coma is metabolic suppression of the cerebral hemispheres. Critical illness neuropathy A. Temporal lobe B. 2247 140 Misdiagnosis of brain death is possible in ? A. Lower midbrain A. Upper midbrain B. All of the above at the level of ? Misdiagnosis of brain death is possible in locked-in syndrome. Cerebral cortex C. 2247 A. Level of awakening B.344:1215 B. reaction and eye movements are intact. 2247 139 As brain death occurs. Catatonia C. hypothermia or drug intoxication. Any of the above C. 2248 B. 50 mL B. azotemia. Foraminal herniation with the reticular formation. Epileptic Coma 151 Coma occurs when horizontal displacement of the pineal Transtentorial herniation refers to part of brain being displaced from supratentorial to infratentorial calcification occurs by ? compartment through the tentorial opening. PCO 2 A. destruction of large portions of both cerebral hemispheres and suppression of reticulo- cerebral function by drugs. Central transtentorial herniation The locked-in syndrome (pseudocoma) is a consequence of destruction of base of pons mostly caused by acute embolus to basilar artery. A. Central fever C. transfalcial. 2248 C. > 9 mm projections. Consciousness persists because tegmentum. D. 2248 compress the opposite cerebral peduncle thus eliciting Kernohan-Woltman sign. and >9 mm with coma. Temporal transtentorial herniation B. calcification occurs by ? Harrison’s 18th Ed. 2248 D. Central transtentorial herniation D. or hepatic failure. A. All of the above 153 Volume of CSF within ventricles and surrounding the brain and Various forms of brain herniations are transtentorial (temporal & central). 2248 A. 1632 147 Cingulate gyrus is likely to be involved in which of the following A. causing compression of medulla that may result in respiratory arrest and death. or swallow (damaged C. Midbrain Harrison’s 18th Ed. spinal cord is about ? Harrison’s 16th Ed. Transfalcial D. Systemic blood pressure 146 Brain herniation can be ? B. Destruction of large portions of both cerebral hemispheres Harrison’s 18th Ed. Foraminal herniation Harrison’s 16th Ed. cerebellar tonsils move downward into foramen magnum. or hepatic failure B. 50 mL brain herniations ? Harrison’s 18th Ed. Thalamus A. 2248 144 Kernohan-Woltman sign best relates to ? Harrison’s 18th Ed. 1 to 3 mm 145 Kernohan-Woltman sign refers to ? B. pH and PCO 2 . 3 to 5 mm Harrison’s 18th Ed. anoxia. 6 to 8 mm A. 100 mL 148 Cerebellar tonsils are likely to be involved in which of the following brain herniations ? C. horizontal displacement of pineal calcification of 3 to 5 mm is associated with C. Consequent lateral displacement of midbrain may Harrison’s 18th Ed. All of the above C.654 MCQ’s FOR MEDICAL PROFESSIONALS BY PROF. Temporal transtentorial herniation C. > 9 mm D. grimace. 100 mL A. 150 Stupor occurs when horizontal displacement of the pineal azotemia. 3 to 5 mm D. A. 143 The principal cause of coma is ? Harrison’s 16th Ed. Lesions that damage substantial portion of RAS calcification occurs by ? B. Transfalcial herniation corticospinal & corticobulbar tracts). > 9 mm B. pH Harrison’s 18th Ed. 6 to 8 mm C. Patient cannot move limbs. anoxia. 150 mL B. 2248 C. toxins. 1 to 3 mm A. Bilateral nystagmus 152 Cerebral blood flow is principally influenced by all except ? Kernohan-Woltman sign refers to a Babinski response and hemiparesis contralateral to the original Harrison’s 16th Ed. Transfalcial herniation 154 Cerebral blood volume is about ? D. Transtentorial herniation D. 1632 hemiparesis due to compression of the opposite cerebral peduncle. foraminal. 6 to 8 mm Principal causes of coma are lesions that damage the reticular activating system (RAS) or its D. the cingulate gyrus is displaced under the falx and across the midline. D. CBF increases with C. D. 2248 C. or metabolic derangements such as hypoglycemia. 1625 149 Drowsiness occurs when horizontal displacement of the pineal A. 150 mL . 1 to 3 mm hypoglycemia. Contraleteral pupillary dilatation in brain mass lesion D. Suppression of thalamocerebral function by drugs. PO 2 B. but upper rostral mesencephalic structures involved in voluntary blinking and vertical eye movements remain intact. AJAY MATHUR Neurology C. In foraminal herniation. in acutely appearing masses. Hypoglycemia B. Hemiparesis contralateral to original hemiparesis drowsiness. 6 to 8 mm with stupor. toxins. Waxing and waning levels of consciousness On CT/MRI. Transtentorial CBF is strongly influenced by systemic blood pressure. 1632 In transfalcial herniation. A. Foraminal hypercapnia and acidosis and decreases with hypocapnia and alkalosis. 3 to 5 mm B. is not affected. 200 mL C. Sepsis Cerebral neurons are fully dependent on cerebral blood flow (CBF) and the related delivery of C. the serum osmolarity is generally more B. Oxygen consumption is 3. 2249 D. 5 mg per 100 g/min C. 3 min B. 118 mmol/L D. 1. 25 mL per 100 g/min A. 2248 convulsions ? Harrison’s 18th Ed. 2. 2248 165 Hypothermia itself causes coma only when the temperature is ? Harrison’s 18th Ed. oxygen stores last for approximately ? Cushing response refers to marked hypertension in a case of coma that may be secondary to a Harrison’s 18th Ed. 2249 166 Cushing response best relates to ? A. most often after cerebral hemorrhage or head injury. >280 mosmol/L B. thrombotic thrombocytopenic purpura. 45 mL per 100 g/min 164 Disorder that occlude small blood vessels throughout the brain D. < 31°C 160 After the cessation of blood flow. 155 Cerebral blood flow (CBF) in gray matter is about ? 162 Sodium levels below how much is associated with coma and Harrison’s 18th Ed. 2249 rapid rise in intracranial pressure (ICP). 50 mL per 100 g/min B. 2 min A. 4 min C. the serum osmolarity is generally >350 mosmol/L. 4. >320 mosmol/L A. 2248 convulsions. CBF approximates 75 mL per 100 gram/minute in gray matter and 30 mL per . Hypothermia 161 After the cessation of cerebral blood flow. All of the above D.5 mL per 100 g/min D. Hypertension C. 75 mL per 100 g/min C. 8 to 10 seconds Harrison’s 18th Ed.5 mL per 100 g/min Disorders that occlude small blood vessels throughout the brain causing bihemispheral damage are cerebral malaria. < 32°C D. 115 mmol/L 156 Cerebral blood flow (CBF) in white matter is about ? Sodium levels <125 mmol/L induce confusion & <115 mmol/L are associated with coma & Harrison’s 18th Ed. 3 mg per 100 g/min B. 159 Glucose utilization of brain tissue is ? Harrison’s 18th Ed. A. 1 min Harrison’s 18th Ed. 3 to 5 seconds 167 Hypotension is characteristic of coma due to ? B. 655 Neurology MCQ’s FORMCQ’s FOR MEDICAL MEDICAL PROFESSIONALS PROFESSIONALS BY PROF. 2249 A. Cerebral malaria A.5 mL per 100 gram/minute. 2249 C. 10 seconds after cessation of blood flow. 30 mL per 100 g/min Harrison’s 18th Ed. 40 mL per 100 g/min A. 200 mL 100 gram/minute in white matter (mean of 55 mL per 100 gram/minute). 20 mL per 100 g/min than ? C. 55 mL per 100 g/min causing bihemispheral damage is ? Harrison’s 18th Ed. 120 mmol/L C. 35 mL per 100 g/min In hyperosmolar coma. and hyperviscosity. AJAY MATHUR Cardiology 655 D. and oxygen stores last 8 to Cerebral blood volume is also ~150 mL. 100 mL per 100 g/min D. 3. 2249 B. 2249 A. >350 mosmol/L B. 1 mg per 100 g/min A. 25 mL per 100 g/min D.5 mL per 100 g/min B. and glucose utilization is 5 mg per 100 gram/minute. Profound hypothyroidism D. C. 2248 A. < 34°C B. 125 mmol/L B. Hyperviscosity C. Alcohol intoxication oxygen and glucose. A. 2249 158 Oxygen consumption of brain tissue is ? Harrison’s 18th Ed. 7 mg per 100 g/min D. >300 mosmol/L 157 Mean cerebral blood flow (CBF) is about ? Harrison’s 18th Ed. energy for approximately ? Harrison’s 18th Ed. 10 mL per 100 g/min 163 In hyperosmolar coma. brain stores of glucose provide Hypothermia itself causes coma only when the temperature is <31°C (87. Thrombotic thrombocytopenic purpura B.8°F). glucose provide energy for ~2 minutes after blood flow is interrupted. 20 to 30 seconds B.5 mL per 100 g/min C. Convulsion D. 12 to 15 seconds A. 2248 C. Cyanosis D. Brain stores of About 150 mL of CSF is present within the ventricles and surrounding the brain and spinal cord. < 33°C C. if these brainstem activities. spontaneous and elicited eye movements. bilateral asterixis is a certain sign of ? 176 Reaction to light is difficult to appreciate in pupils ? Harrison’s 18th Ed. < 4 mm in diameter D. internal hemorrhage. 169 Multifocal myoclonus indicates which of the following ? Harrison’s 18th Ed. Temporal lobe damage Multifocal myoclonus almost always indicates a metabolic disorder (uremia. Head injury D. 2250 B. bilateral asterixis is a certain sign of metabolic encephalopathy or drug intoxication. Extension of elbows & wrists with pronation of arm Harrison’s 18th Ed. diencephalon produces which of the following ? Harrison’s 18th Ed. AJAY MATHUR Neurology D. Severe thalamic damage D. corneal responses. coma is due to bilateral hemispheral disease.656 MCQ’s FOR MEDICAL PROFESSIONALS BY PROF. Arm extension with flaccid legs C. bilaterally dilated & unreactive pupils. Meningococcemia B. indicates severe midbrain damage. are preserved. compression by a supratentorial mass. Subarachnoid hemorrhage C. 2250 Harrison’s 18th Ed. Extension of elbows & wrists with pronation of arm A. Unilateral miosis D. Basal ganglia B. Uremia Harrison’s 18th Ed. 2251 Harrison’s 18th Ed. anoxia). Septicemia C. Subhyaloid hemorrhages on funduscopic examination indicate subarachnoid hemorrhage. 2249 A. Flexion of elbows & wrists with supination of arm A. Reaction to light is often difficult to appreciate in pupils <2 mm in diameter. Drug intoxication A. Medulla oblongata Hypotension is characteristic of coma from alcohol or barbiturate intoxication. Cerebral malaria B. myocardial infarction. Corneal responses C. Severe pontine damage Flexion of elbows and wrists and supination of arm (decortication) suggests bilateral damage rostral to the midbrain. 2250 Harrison’s 18th Ed. 2250 A. drug D. subhyaloid hemorrhage suggests ? Harrison’s 18th Ed. Haloperidol intoxication A. Flexion of elbows & wrists with supination of arm B. Midbrain C. normally reactive & round pupils of midsize (2. Prion disease B. 174 Which of the following is not a brainstem reflex ? 168 On fundoscopic examination. Occipital lobe damage intoxication (lithium or haloperidol). 2250 A. Hydrocephalus B. Severe cortical damage C. Spontaneous and elicited eye movements B. 2250 B. All of the above . usually from indicates damage to motor tracts in midbrain or caudal diencephalon. Metabolic encephalopathies A. Reactive and bilaterally small In coma. Combination of arm extension with leg flexion or flaccid legs is associated with lesions in pons. Jaw jerk D. 170 In a drowsy & confused patient. or a prion disease.5 . Any of the above C. Pontine damage D. Midbrain damage C. All of the above Brainstem reflexes include pupillary responses to light. Arm extension with leg flexion B. particularly pupillary reactions and eye movements. 2250 A. Oval & slightly eccentric pupil C. Arm extension with flaccid legs D. 171 Bilateral damage rostral to the midbrain produces which of the 177 A transitional sign that accompanies early midbrain–third nerve following ? compression is ? Harrison’s 18th Ed. Pupillary responses to light A. As a rule. and respiratory pattern. or Addisonian crisis. sepsis. All of the above D. In coma. Thalamic hemorrhage C. Arm extension with leg flexion C. Pons D. profound hypothyroidism. < 3 mm in diameter C. 2250 Harrison’s 18th Ed. 173 Combination of arm extension with leg flexion or flaccid legs is 179 Pupils are smaller in size in which of the following ? associated with lesions in ? Harrison’s 18th Ed. < 2 mm in diameter B. Pupil > 6 mm is diameter B. whereas extension of elbows and wrists with pronation (decerebration) In coma. 2250 175 Normally reactive & round pupils of midsize excludes ? A.5 mm) essentially excludes midbrain damage. 2250 178 Bilaterally dilated & unreactive pupils indicate ? A. an oval & slightly eccentric pupil is a transitional sign that accompanies early midbrain– 172 Bilateral damage to motor tracts in midbrain or caudal third nerve compression. Bleeding diathesis D. Severe midbrain damage D. < 5 mm in diameter In a drowsy and confused patient. either primary or secondary to compression. horizontal eye movements are dependent on medial longitudinal fasciculus 185 “Doll’s eye” oculocephalic reflex is indicative of which of the (MLF) that interconnects sixth & contralateral third nerve nuclei. Oculovestibular & oculocephalic reflexes provide essentially the same information. Temporal radiation Reflex conjugate. Harrison’s 18th Ed. 2250. Eyes look towards hemispheral or a brainstem lesion Harrison’s 18th Ed. Eyes look away from hemispheral lesion & toward a C. The movements. Superior colliculus D. Reduced cortical influence on brainstem & damaged same” is for which of the following ? Harrison’s 18th Ed. Respiratory responses brainstem lesion D. midbrain & third nerve function are tested by pupillary reaction to light. Tonic deviation of head B. pontine function by spontaneous & reflex eye movements & corneal responses. Reduced cortical influence on brainstem & intact brainstem pathways B. Narcotic overdose The oculocephalic reflexes. following ? Harrison’s 18th Ed. D. Cerebellum “Ocular dipping” is a slower. Pupillary reaction to light D. Normal cortical influence on brainstem & intact brainstem Harrison’s 18th Ed. Bilateral pontine damage A. Figure 274-3 B. Spontaneous and reflex eye movements D. Figure 274-3 A. Tonic deviation of both eyes C. Corneal responses D. Normal cortical influence on brainstem & damaged brainstem C. Cerebellar damage C. 2251 Harrison’s 18th Ed. Pontine lesions A. 2251 A. In coma. C. Harrison’s 18th Ed. Figure 274-3 C. Corneal responses “Ocular bobbing” describes a brisk downward and slow upward movement of the eyes associated with loss of horizontal eye movements and is diagnostic of bilateral pontine damage. called somewhat inappropriately “doll’s eyes” (which refers more accurately to the reflex elevation of the eyelids with flexion of the neck). Bilateral temporal lobe infarct B. Eyes look away from hemispheral or a brainstem lesion 188 Pontine function is tested by ? D. Pontine hemorrhage medulla. Tubercular meningitis C. Diffuse cortical anoxic damage 190 Reflex conjugate. AJAY MATHUR Cardiology 657 180 Pupils are smaller in size in which of the following ? D. usually from D. 2251 191 Acronym “COWS” that refers to ”cold water opposite. elicited by moving the head from side to side or vertically and observing B. arrhythmic downward movement followed by a faster upward move- ment in patients with normal reflex horizontal gaze indicating diffuse cortical anoxic damage. The ability to elicit them therefore indicates both reduced cortical influence on the brainstem and intact brainstem pathways. Medial longitudinal fasciculus (MLF) C. Pupillary reaction to light lesion B. Respiratory & pharyngeal responses thrombosis of the basilar artery. Spontaneous and reflex eye movements B. 657 Neurology MCQ’s FORMCQ’s FOR MEDICAL MEDICAL PROFESSIONALS PROFESSIONALS BY PROF. C. Pharyngeal responses C. depend on the integrity of the ocular motor nuclei and their interconnecting tracts that extend from the midbrain to the pons and C. Barbiturate overdose eye movements in the direction opposite to the head movement. Direction of nystagmus pathways . 2250. Spontaneous and reflex eye movements Eyes turn down & inward with thalamic hemorrhage & upper midbrain lesions. All of the above in the awake patient. warm water A. are normally suppressed D. ICSOL A. SAH B. 2250. Figure 274-3 Conjugate horizontal ocular deviation to one side indicates damage to pons on the opposite side A. Corneal responses 183 “Ocular bobbing” is diagnostic of ? D. 2250. 2251 A. 2251 brainstem pathways A. and medullary function by respiratory 184 “Ocular dipping” is diagnostic of ? and pharyngeal responses. Bilateral occipital infarct 189 Medullary function is tested by ? Harrison’s 18th Ed. Respiratory & pharyngeal responses 182 Which of the following statements about eyes in a state of coma is correct ? 187 Pontine function is tested by ? Harrison’s 18th Ed. Eyes look toward hemispheral lesion & away from brainstem A. The rule therefore is “eyes look toward a hemispheral lesion and away from a brainstem lesion”. All of the above Harrison’s 18th Ed. Pupillary reaction to light C. indicating that coma is caused by a lesion or 181 Eyes turn down & inward with which of the following lesion ? dysfunction in the cerebral hemispheres. 2251 pathways A. 2251 B. 2251 A. “Wrong-way eyes” refers to eyes that may on a rare occasion B. horizontal eye movements are dependent on ? Harrison’s 18th Ed. Reflex eye movements turn paradoxically away from the side of a deep hemispheral lesion. Thalamic lesions Harrison’s 18th Ed. Frontal lobe lesions 186 Midbrain and third nerve function are tested by ? B. Medullary lesions B. Spontaneous eye movements or to frontal lobe on the same side. Deep coma D. 2251 pontomesencephalic lesions ? A. which of the following may not be detected by CT scan ? 197 Which of the following respiratory pattern is seen in Harrison’s 18th Ed.658 MCQ’s FOR MEDICAL PROFESSIONALS BY PROF.2 g/dL B. Awake state Cheyne-Stokes respiration in its classic cyclic form signifies bihemispheral damage or metabolic 193 Corneal reflex depends on the integrity of pontine pathways suppression and commonly accompanies light coma. 0. Fifth and opposite sided seventh cranial nerve A. Medulla Most cases of coma (and confusion) are metabolic or toxic in origin. After a brief C. Medullary function C. which of these is the first with stupor ? to disappear ? Harrison’s 18th Ed. Tachypnea the opposite direction. Any of the above In nonhabituated patients. (Acronym “COWS” refers to direction of nystagmus .5 g/dL D. 2251 Harrison’s 18th Ed. 0. Midbrain function A. Corneal responses B. Agonal gasps are the result of lower brainstem (medullary) damage and are recognized as the terminal respiratory pattern of severe brain damage. AJAY MATHUR Neurology D. Metabolic or toxic origin C.”cold water opposite. 2251 A. Hemorrhage A. Acute brainstem infarction Harrison’s 18th Ed. 203 In a case of coma. 2251 B. 2251 198 Cheyne-Stokes respiration in its classic cyclic form is seen in A. Cheyne-Stokes respiration elicits contraversive eye movements. 2251 B. deep breathing is usually seen in metabolic acidosis but may also occur with pontomesencephalic lesions. between ? Harrison’s 18th Ed. Fifth and same sided seventh cranial nerve Harrison’s 18th Ed. All of the above A. Lymphoma of CNS D. 202 Most cases of coma (and confusion) are due to ? 196 Agonal gasps are the result of damage to ? Harrison’s 18th Ed. warm water same. 2251 A. what level of ethanol is associated 195 When CNS-depressant drugs are given. 0.3 g/dL C. Reflex eye movements C. result is tonic deviation of both eyes to the side of cool-water irrigation and nystagmus in D.3 g/dL) is associated with stupor. 194 Corneal reflex in conjunction with reflex eye movements is a 200 In nonhabituated patients. 2251 Harrison’s 18th Ed. regular Head rotation (oculocephalic reflex) or caloric stimulation of the labyrinths (oculovestibular reflex) B. Light coma C.2 g/dL) causes impaired mental activity and a level of >65 mmol/L (0. thus functional or hysterical coma is likely. 2251 199 Tachypnea occurs in which of the following conditions ? A.3 g/dL B. 2251 A.4 g/dL C. Frontal lobes are functioning which of the following conditions ? Harrison’s 18th Ed. Sleep Presence of corrective nystagmus indicates that the frontal lobes are functioning and connected to the brainstem. 2251 Harrison’s 18th Ed. Cortex B. Fungal infection of CNS D. 0. All of the above D.5 g/dL Corneal reflex in conjunction with reflex eye movements is a useful test of pontine function. 0. Kussmaul or rapid. 2251 . 0. All of the above C. CNS-depressant drugs diminish or eliminate the corneal responses soon after reflex eye movements are paralyzed but before the pupils become unreactive to light. Midbrain C. Kussmaul breathing latency. 192 Presence of corrective nystagmus indicates which of the following ? Harrison’s 18th Ed. All of the above Corneal reflex depends on the integrity of pontine pathways between fifth (afferent) & both seventh (efferent) cranial nerves. Functional or hysterical coma is likely B.2 g/dL A. Non-reactivity of pupils to light D. D. Fifth and both seventh cranial nerves B.4 g/dL D. Pontine function B. Shallow. 0. 0. Tuberculosis of CNS C. Pons D. Any of the above C. Tumor B. Tachypnea occurs with lymphoma of the CNS. generally an ethanol level of 43 mmol/L (0.”) Loss of induced conjugate ocular movements indicates brainstem damage. Hydrocephalus D. slow. Oculovestibular test is performed by irrigating the external auditory canal with cool water in order to induce convection currents in labyrinths. Frontal lobes are connected to the brainstem A. 201 In nonhabituated patients. what level of ethanol causes impaired useful test of ? mental activity ? Harrison’s 18th Ed. 2252 coma. Cerebral infarction . vomiting. Pontine hemorrhage Harrison’s 18th Ed. Encephalitis Conditions that cause sudden coma include drug ingestion. hysteria or catatonia. epilepsy. Widespread fast beta activity response abnormalities. 2252 A. EEG is useful in which of the following conditions ? B. Harrison’s 18th Ed. trauma. vomiting. 2251 D. Infarction in middle cerebral artery territory (diazepines. 2252 Normal alpha activity on EEG is found in locked-in syndrome. 2251 C. Thalamic hemorrhage 204 In coma. Widespread variable 8. A. or to prion (Creutzfeldt-Jakob) disease. cardiac arrest. Hyperventilation D. Acute hydrocephalus Harrison’s 18th Ed. posturing. Herpesvirus encephalitis Occipital headache. 2252 metabolic coma. Delta or triphasic waves in the frontal regions Neurologic prodrome or warning spells. 659 Neurology MCQ’s FORMCQ’s FOR MEDICAL MEDICAL PROFESSIONALS PROFESSIONALS BY PROF. All of the above Bilateral hemisphere infarction. barbiturates) as a cause of Harrison’s 18th Ed. Subarachnoid hemorrhage C.to 12-Hz activity 212 Neurologic prodrome or warning spells are a feature of which of D. Thalamic hemorrhage 207 Alpha coma results from ? B. 2252 C. eye movement and corneal B. 2252 to adjacent brain may not be detected. and asymmetric limb paresis are features of basilar artery thrombosis. Hypoglycemia Sudden onset. Delta or triphasic waves in the frontal regions C. Sagittal sinus thrombosis D. Subarachnoid hemorrhage B. Subdural hematoma 214 Vomiting is a feature of which of the following ? Alpha coma results from pontine or diffuse cortical damage and is associated with a poor prognosis. mechanical 210 Gaze paresis is a feature of which cerebrovascular disease ? shearing of axons (closed head trauma). Normal alpha activity which of the following ? Harrison’s 18th Ed. Acute hydrocephalus D. ocular C. to herpesvirus encephalitis. Pontine hemorrhage Harrison’s 18th Ed. Infarction in middle cerebral artery territory A. Subarachnoid hemorrhage A. loss of reflex eye movements and corneal responses. Widespread variable 8. Cerebellar hemorrhage A. Prion (Creutzfeldt-Jakob) disease hemorrhage. Normal alpha activity the following ? Predominant high-voltage slowing (delta or triphasic waves) in the frontal regions is typical of Harrison’s 18th Ed. Encephalitis C. 2251 B. Harrison’s 18th Ed. 2252 C. meningitis.to 12-Hz activity 215 Precipitous coma after headache and vomiting is a feature of D. Thalamic hemorrhage 208 Which of the following is typical of locked-in syndrome ? Harrison’s 18th Ed. acute brainstem infarction. 2252 205 Which of the following is typical of metabolic coma ? A. dysarthria. Widespread fast beta activity D. Basilar artery thrombosis 206 Which of the following is typical of coma due to sedative drugs B. cerebral hemorrhage. barbiturates) ? C. 2252 B. pinpoint pupils. All of the above C. gaze paresis. and excessive sweating are features of pontine hemorrhage. encephalitis. Widespread fast beta activity D. AJAY MATHUR Cardiology 659 B. A. D. hyperventilation. Subarachnoid hemorrhage B. A. Thalamic hemorrhage 209 Which of the following condition causes sudden coma ? B.to 12-Hz activity 213 Hyperventilation and excessive sweating are a feature of which D. Basilar artery embolism Precipitous coma after headache and vomiting is a feature of subarachnoid hemorrhage. All of the above 211 Asymmetric limb paresis is a feature of which cerebrovascular The EEG is useful in metabolic or drug-induced states and is diagnostic when coma is due to disease ? clinically unrecognized seizure. Cerebellar hemorrhage A. Pontine damage bobbing. Subarachnoid hemorrhage B. Cerebellar hemorrhage A. and inability to stand are features of cerebellar C. D. C. C. Widespread variable 8. Normal alpha activity cerebrovascular disease ? Widespread fast beta activity implies sedative drugs (diazepines. diplopia. D. Subarachnoid hemorrhage B. Cerebellar hemorrhage A. Acute hydrocephalus B. Basilar artery thrombosis Harrison’s 18th Ed. A. and subdural hematoma isodense Harrison’s 18th Ed. Pontine hemorrhage Harrison’s 18th Ed. Clinically unrecognized seizure D. or basilar artery embolism. Delta or triphasic waves in the frontal regions C. sagittal sinus thrombosis. Pontine hemorrhage D. Absent Babinski signs D.40 mmHg A. 50 . 221 In a valid apnea testing. 2252 A. Subarachnoid hemorrhage myelopathy ? Harrison’s 18th Ed. the pituitary gland. Typhoid fever 226 In normal CNS. 30 .e. In a routine brain CT study.50 mmHg B. carcinomatous and lymphomatous meningitis cause meningeal irritation with or without fever. radiation dose is normally between 2 and 5 mSv (millisievert). Choroid plexus ta B. 40 . Fat embolism B. cholesterol embolism. 3241 220 Which of the following is a feature of brain death ? Harrison’s 18th Ed. 228 Rise in serum creatinine of at least 1 mg/dL within how many Chapter 3 68. MR angiography (MRA) - Harrison’s 18th Ed. Infection/abscess d lateralizing cerebral signs ? D. V B. 3240 n C. Diabetes insipidus C. 48 hours . 6 to 12 mSv D. radiation dose is normally about ? Harrison’s 18th Ed. 2253 227 In a routine brain CT study. CT contrast Harrison’s 18th Ed. apnea is confirmed if no respiratory effort is observed in the presence of a sufficiently elevated Pco 2 i. 2252. which of the following structure lacks blood- 9 D. Cholesterol embolism D. Cerebellar hemorrhage 223 Which of the following investigation is recommended in D. AJAY MATHUR Neurology 216 Acute hydrocephalus accompanies particularly which of the A. Eclampsia brain barrier (BBB) ? ir Harrison’s 18th Ed. Immunosuppressed with focal findings ti e Harrison’s 18th Ed. Dura C.2 to 1 mSv B. choroid plexus. Perfusion CT (pCT) U 219 Cellular content of the CSF is not normal in which of the following ? C. 0. Fat embolism 9 cord. and dura lack blood-brain barrier (BBB) and enhance after contrast administration. Table 368-1 Fat embolism. 2252. All of the above except ? R Harrison’s 18th Ed. Table 368-1 D. Carcinomatous meningitis 224 MRI + contrast is the investigation of choice for all of the following D. encephalitis. 60 . 22 to 50 mSv To confirm brain death. 2 to 5 mSv C. CT angiography (CTA) D. Herpes simplex encephalitis 225 Which of the following is the most sensitive technique for detecting B. MRI with contrast C. Table 274-1 C. Thrombotic thrombocytopenic purpura acute ischemic stroke of brain ? Harrison’s 18th Ed. Malaria Diffusion MR is the most sensitive technique for detecting acute ischemic stroke of brain or spinal B. Neoplasm (primary or metastatic) without focal or lateralizing cerebral or brainstem signs. All of the above In normal CNS.660 MCQ’s FOR MEDICAL PROFESSIONALS BY PROF. 2252. usually A. Pituitary gland h A. Heart rate unresponsive to atropine B. MRI A. CT contrast Harrison’s 18th Ed. Neuroimaging in hours of contrast administration defines contrast nephropathy ? Harrison’s 18th Ed. 3240. 3241 A. 2252 A. and with an excess of WBCs or RBCs in CSF. Table 368-1 The syndrome of acute hydrocephalus accompanies particularly subarachnoid hemorrhage. 3240. C. All of the above D. 12 hours 222 Which of the following investigation is recommended in meningeal C. Thalamic hemorrhage C. CT or MRI shows no mass lesion. MRI B. White matter disorders 218 Which of the following diseases cause focal brainstem or C. CT (noncontrast) following ? B. 6 hours B. abscesses and prion diseases. Pituitary apoplexy A. 50 . Table 274-1 A. Diffusion MR A. 3241 Neurologic Disorders A.70 mmHg D. Pco2 should be at least ? Harrison’s 18th Ed. 24 hours disease ? Harrison’s 18th Ed. Table 274-1 D. MRI with contrast G C. CT (noncontrast) 217 Which of the following diseases cause meningeal irritation ? B.60 mmHg. 3240.60 mmHg C. All of the above B. A. Ilmenite The American College of Radiology suggests using an estimated glomerular filtration rate (eGFR) B.75(2):95-111 A. preexisting renal disease (serum creatinine exceeding 2 mg/dL).2 mL/kg body weight administered intravenously. A. Hypertension D. diabetes mellitus.25.24 % 239 Which of the following is best related to gadolinium contrast Severe allergic reactions occur in 0. 3243 237 Which of the following countries has deposits of monazite A. Sillimanite 231 Use of which of the following may reduce the incidence of contrast Gadolinium is produced both from monazite and bastnäsite. use of bicarbonate & acetylcysteine D.732 236 Gadolinium is produced both from which of the following C. 3241 D. 661 Neurology MCQ’s FORMCQ’s FOR MEDICAL MEDICAL PROFESSIONALS PROFESSIONALS BY PROF. Nephrogenic diabetes insipidus (NDI) 233 Which of the following is not related to magnetic resonance imaging (MRI) ? B. 3243 Approximate dose of gadolinium is 0. 60 mL/min/1. AJAY MATHUR Cardiology 661 229 Risk factors for contrast nephropathy include all except ? C. Between ankles and thighs Harrison’s 18th Ed. and energy (echo) in the form of radiofrequency (Rf) waves of a specific frequency. Chelated to DTPA 230 What is the eGFR threshold below which iodinated contrast should not be given ? D. Gd A. dehydration. A.18 % Symbol of Gadolinium is Gd with atomic number 64 and atomic weight of 157. B. B. Brazil D. 45 mL/min/1.732 B. Paramagnetic substance C. and chronic hemorrhage. South Africa Apart from hydration and reduction in dose of contrast media. Gadolinium has the greatest ability to capture thermal neutrons. 0. 0. India C. Radiofrequency (Rf) waves 240 Nephrogenic systemic fibrosis (NSF) involves which of the MRI is a complex interaction between hydrogen protons in biologic tissues. Advanced age (>80 years) 235 Which of the following statements about gadolinium is false ? Harrison’s 18th Ed. Upper extremities 234 T2W images are more sensitive than T1W images to all of the following except ? B. Trunk A. infarction. Gl B. Oxygen inhalation A.nephrogenic systemic fibrosis (NSF) between 5 and 75 days following exposure. 0.732 A. Edema D. 3244 Risk factors for contrast nephropathy include advanced age (>80 years). Solitary kidney T2-weighted (T2W) images are more sensitive than T1-weighted (T1W) images to edema. while T1W imaging is more sensitive to subacute hemorrhage and fat-containing structures. Harrison’s 18th Ed. Fat-containing structures . sixfold lower than agents ? with ionic media. Ga receiving nonionic media ? Harrison’s 18th Ed. a static magnetic field following parts of the body most ? (the magnet). Osmolar diuretics B. 3243 B. 75 mL/min/1. Static magnetic field complication . Diabetes mellitus demyelination. Approximate dose 2 mL/kg IV Harrison’s 18th Ed.732 minerals ? D. 238 Symbol of Gadolinium is ? 232 Severe allergic reactions occur in what proportion of patients A. Demyelination Harrison’s 18th Ed. D. solitary kidney. Acetylcysteine sands ? B.73 2 as a threshold below which iodinated contrast should not be given without C. D. concurrent use of nephrotoxic medication or chemotherapeutic agents. Monazite serious consideration of the potential for contrast nephropathy. nephropathy ? Harrison’s 18th Ed. 3243 C. Nephrogenic Syndrome of Inappropriate Antidiuresis Harrison’s 18th Ed. Face B.12 % D. 90 mL/min/1. Infarction A. Nephrotic syndrome B. Zircon of 45 mL/min/1. All of the above may reduce the incidence of contrast nephropathy. Dynamic magnetic field Patients with renal insufficiency exposed to gadolinium contrast agents may develop a rare C.04 % C. C. 3244 C. and high contrast dose. D. Nephrogenic systemic fibrosis (NSF) A. Gm C. Cleveland Clinic Journal of Medicine 2008. 3244 A.04% of patients receiving nonionic media. Hydrogen protons in biologic tissues D. Loop diuretics C. Heavy-metal element paraproteinemia. 0. “to invade” . other disorders that can cause thickening & hardening of skin of extremities & trunk B. Correctable or avoidable circumstance found in the umbilical cord. AJAY MATHUR Neurology 241 Nephrogenic systemic fibrosis (NSF) involves which of the B. an analogue of glucose. It is taken up by cells competitively with 2-deoxyglucose. Etiology or cellular substrate are not considered. and in the vascular endothelium. Early childhood & late adulthood C. A.75(2):95-111 D. ~5 . but usually with lower extremity A. 3251 A. Anti nuclear antibodies G 248 Highest incidence of seizure is in which of the following age B. Immunohistochemically CD34 reactivity in fibroblast-like cells Harrison’s 18th Ed. By definition. Proliferation of dermal spindle cells is in early childhood and late adulthood. term partial seizures is no longer used and subcategories of “simple focal seizures” and “complex focal seizures” have been 246 Seizure is derived from a Latin word “sacire” that means ? eliminated. 1965 Besides NSF. Anti-SCL70 antibodies R Harrison’s 18th Ed. 2498 ta D. Trunk is involved less commonly than legs and arms.75(2):95-111 ~5–10% percentage of population will have at least one seizure in lifetime and the highest incidence A. underlying process. the International League against Epilepsy (ILAE) classification is based on clinical Harrison’s 18th Ed. 3251 D. Middle age d 243 Which of the following is characteristic and pathognomonic of NSF ? D. ~3 . Biochemical abnormalities D. 2-[ F]fluoro-2-deoxy-d-glucose 19 B. and eosinophilic fasciitis. Rheumatoid factor groups ? C.15 % A. Eosinophilic fasciitis Harrison’s 17th Ed. does not have epilepsy. B. the most frequently used moiety is 2-[ 18F]fluoro-2-deoxy- d-glucose (FDG). 2-[16F]fluoro-2-deoxy-d-glucose B. Upper extremity involvement occurs frequently. Between ankles and thighs C. In the new classification system (International League against Epilepsy (ILAE) Commission on Classification and Terminology. Upper extremities Seizure is derived from Latin word “sacire” meaning “to take possession of”. Thick collagen bundles with surrounding clefts n 249 Which of the following is false about epilepsy ? C. Cellular abnormalities In Positron Emission Tomography (PET). the bone marrow (as pluripotential hematopoietic stem cells). ~1 . All of the above A.75(2):95-111 D.Seizures and Epilepsy Focal seizures are associated with structural abnormalities of brain. “to silence” Cleveland Clinic Journal of Medicine 2008. None of the above A. 3248 features of seizures and associated EEG findings.10 % Cleveland Clinic Journal of Medicine 2008. Cells expressing CD34 are normally C. 1995 Tomography (PET) is ? In 1981. Systemic sclerosis 250 In which year did the International League Against Epilepsy (ILAE) B. Face lifetime ? Harrison’s 18th Ed.662 MCQ’s FOR MEDICAL PROFESSIONALS BY PROF. B. All of the above 9 244 Disorder that causes thickening & hardening of skin of extremities Epilepsy is a clinical phenomenon & is said to be present when a person has recurrent seizures due to a chronic. 2005–2009). B. Scleromyxedema h classify seizure disorders ? C. there are exception though. Harrison’s 18th Ed. scleromyxedema. C. Single seizure U D. Infancy V There is no laboratory biomarker for NSF. Structural abnormalities of brain D. ~10 . 2-[17F]fluoro-2-deoxy-d-glucose 251 Focal seizures are associated with ? Harrison’s 18th Ed. “to take possession of” A. i A.75(2):95-111 seizures. C. All of the above 369 . 1978 include systemic sclerosis or scleroderma.3 % disease. The face is typically spared. epilepsy is two or more UNPROVOKED r Cleveland Clinic Journal of Medicine 2008. “to destroy” following parts of the body least ? C. Acute - identification of CD34 reactivity in the fibroblast-like cells. 3251 A.5 % 242 Which of the following is a laboratory biomarker for NSF ? C. 3251 NSF typically presents between ankles and the thighs (symmetric. D. 3251 C. 1981 245 Most frequently used radionuclide moiety in Positron Emission D. B. Old age ti e Cleveland Clinic Journal of Medicine 2008. Absence of mucin and elastic fibers A characteristic and almost pathognomonic staining profile is the immunohistochemical B. Trunk 247 What percentage of population will have at least one seizure in D. A person with a single seizure or recurrent seizures due to correctable & trunk is ? 9 or avoidable circumstances. progresses to involve the entire lower extremities). 2-[ F]fluoro-2-deoxy-d-glucose 18 A. This is called a “Jacksonian march”. <= 2. or structural abnormalities. EEG shows generalized. A.0 per second B.0 per second localized paresis in the involved region for minutes to many hours (Todd’s paralysis). patients with atypical B. symmetric. <= 4. it is termed simple partial seizure. A. 3252 263 The most common seizure type resulting from metabolic A. Hyperventilation provokes EEG discharges & seizures C. Absence Seizures background B. Consciousness or unconsciousness A. ~ 60 . spike- B. abnormal discharges may spread to other wider areas. Exercise D. KCNQ2 C. D. biochemical. 3-Hz spike-and-wave Harrison’s 18th Ed. Following focal motor seizure.20 % D. Behavioral arrest denotes beginning of ictal phase absence seizures have ? Harrison’s 18th Ed. ~ 10 . slow spike-and-wave pattern with a 255 Which of the following about complex partial seizures is false ? frequency of <= 2. ~ 50 . EEG finding of generalized. Epilepsia partialis continua Harrison’s 18th Ed. EEG discharge begin & end suddenly on an abnormal A. beginning with an aura. Consciousness returns as suddenly as it was lost C. CHRNA4 B. Hyperventilation provokes these EEG discharges. Absence or presence of aura B. Structural abnormality A. confusion or postictal aphasia may be present. Unilateral or bilateral 259 What percentage of patients of typical absence seizures will have B. Focal motor seizure may continue for hours or days (epilepsia partialis continua). 3252 ~ 60-70% patients of absence seizures have a spontaneous remission during adolescence. Harrison’s 17th Ed. 3252 C. 257 Which of the following is false about absence seizures ? Harrison’s 18th Ed. 3252 Table 369-2 A. Reterograde amnesia A. Jacksonian march 260 In atypical absence seizures.5 per second In focal motor seizures.40 % If consciousness is fully preserved during seizure. as well as other abnormal EEG activity. <= 3. Harrison’s 17th Ed. X-linked dominant classic lissencephaly is associated with severe mental retardation and seizures in males. Any of the above B. ~ 30 . All of the above A. Todd’s paralysis and-wave pattern has a frequency of ? C. If consciousness is impaired. expressed primarily in frontal lobes directly regulates microtubule normal & have no memory of the event or no postictal confusion. Lapse of consciousness of longer duration D.5 per second. Grand Mal seizures . 3252 D. 3252 C. C. <= 3. subcortical band heterotopia with more subtle findings in females. brief loss of consciousness with loss of postural control A. 663 Neurology MCQ’s FORMCQ’s FOR MEDICAL MEDICAL PROFESSIONALS PROFESSIONALS BY PROF. 2499 A. C. Biochemical abnormality 258 Which of the following will trigger a seizure in most patients with untreated absence seizures ? C.70 % 254 Which of the following is a feature of focal motor seizure ? Harrison’s 18th Ed. Less abrupt onset and cessation Complex partial seizures are characterized by focal seizure activity with transient unconsciousness. patient may have C. Myoclonic Seizures D. Begins with an aura 261 When compared to typical absence seizures. the seizure is termed complex partial seizure. Patients stare & cease normal activity for a few seconds. Sudden. Breath holding 253 Main difference between simple partial and complex partial seizure D. Lack of sleep is ? Hyperventilation for 3 minutes will trigger a seizure in most patients with untreated absence seizures. Automatisms B. 262 Which of the following gene for with various epilepsy syndromes 256 Which of the following is false about absence seizures ? is associated with severe mental retardation ? Harrison’s 18th Ed. polymerization and bundling.60 % D. 3251 discharge that begins and ends suddenly. Cellular abnormality B. Following seizure anterograde D. after initiation from a restricted region. symmetric. SCN1B D.5 per second EEG in atypical absence seizures shows a generalized. Hyperventilation Generalized seizures may result from cellular. No postictal confusion D. Typically lasts for only seconds B. 3252 Harrison’s 18th Ed. All of the above amnesia. Doublecortin Absence seizures are characterized by sudden. Focal or generalised a spontaneous remission during adolescence ? Harrison’s 17th Ed. superimposed on a “normal” EEG background. Behavioral arrest or motionless stare denotes beginning of ictal phase C. return immediately to Doublecortin (Xq21-24). 2498 A. Less response to anticonvulsants which marks the onset of the period of amnesia & automatisms. AJAY MATHUR Cardiology 663 252 Generalized seizures are associated with ? The EEG hallmark of typical absence seizures is a generalized. 3-Hz spike-&-wave Harrison’s 18th Ed. 2499 C. brief lapses of consciousness “without” loss of postural control. Usually begin in childhood (4 to 8 yrs) derangements is ? B. Low-amplitude activity interrupted by slow waves the most common seizure type resulting from metabolic derangements. Bladder or bowel incontinence C. Ictal phase usually lasts no more than 1 minute. Increase in blood pressure A. Any of the above d A. 3253 A.664 MCQ’s FOR MEDICAL PROFESSIONALS BY PROF. 3253 273 Giant slow waves relate best with which of the following ? Harrison’s 18th Ed. 2498 D. High-amplitude activity interrupted by slow waves B. Muscles of expiration and larynx 270 Which of the following is false about atonic seizures ? C. Muscular flaccidity D. Primary generalized seizure Initial phase of Grand Mal seizure is tonic contraction of muscles throughout the body. Post . N Engl J Med 1999. 3253 Harrison’s 18th Ed. high-amplitude activity is typically interrupted by slow waves to create a spike-and-wave A. Atonic seizures are R usually seen in association with known epilepsy syndromes. 3253. 3253 272 Pathologic myoclonus is mostly seen in association with ? A. Metabolic disorders C. Epileptic spasms D. Atypical absence seizures D. initial tonic phase evolves into clonic phase after 10 . Seen in association with known epilepsy syndromes G Harrison’s 18th Ed. Atypical absence seizures In Grand Mal seizure. polyspike discharges. Muscles of expiration and pharynx A. spasms. Consciousness is briefly impaired cry”. Respiration is impaired.20 seconds. They are also C. Epileptic spasms C. A. 3253 A. Myoclonic seizure B. Muscles of inspiration and pharynx Harrison’s 18th Ed. EEG in epileptic spasms shows hypsarrhythmias (diffuse. Low-amplitude activity interrupted by fast waves 264 “Ictal cry” is related to tonic contraction of which of the following ? EEG during the tonic phase of Grand Mal seizure shows a progressive increase in generalized low- Harrison’s 18th Ed. All of the above 9 A marked enhancement of sympathetic tone during ictal phase of Grand Mal seizure leads to increases in heart rate. Ictal phase V C. Tonic B. 60 . Atonic seizures A. Degenerative CNS diseases D. 3253 voltage fast activity. C. Morning A. blood pressure. Pathologic myoclonus is commonly seen with metabolic disorders. giant slow waves with a chaotic background of irregular. Secondary generalized seizure ti e contraction of muscles of expiration and larynx at the onset produces a loud moan or “ictal cry”. Atonic seizures B. 3253 Atonic seizures are characterized by sudden loss of postural muscle tone lasting 1-2 seconds. C. and pupillary size. 268 Postictal phase in Grand Mal seizures is characterized by all 274 “Electrodecremental response” in EEG is a feature of ? except ? Harrison’s 18th Ed. Irritability B. Excessive salivation “Electrodecremental response” or a marked suppression of EEG background is a feature of epileptic Postictal phase in Grand Mal seizures is characterized by unresponsiveness. 120 . multifocal spikes and sharp waves). In clonic phase. High-amplitude activity interrupted by fast waves Grand Mal seizures are the main seizure type in ~10% of all persons with epilepsy. 30 . 267 In Grand Mal seizure. followed by generalized high-amplitude. but there is usually no postictal confusion.ictal phase Consciousness is briefly impaired.340:1565 Harrison’s 18th Ed.20 seconds A. initial tonic phase evolves into clonic phase r after ? i Harrison’s 18th Ed. Constriction of pupils C. Muscles of inspiration and larynx pattern. 3253 h A. 3253 D. 10 . degenerative CNS diseases 9 or anoxic brain injury.ictal phase 271 Myoclonic seizure is a type of ? Harrison’s 17th Ed. Increase in heart rate Harrison’s 18th Ed. Pre . AJAY MATHUR Neurology D.60 seconds ta B. Atonic Seizures B. D. 3253 U B. Afternoon . muscular flaccidity. Impaired respiration - B. Myoclonic seizure C. Significant postictal confusion 265 Ictal cry is observed in which phase of Grand Mal seizure ? D. 275 Seizures in Juvenile myoclonic epilepsy (JME) are most frequent 269 Which of the following is typical of EEG of clonic phase of Grand in ? Mal seizure ? Harrison’s 18th Ed. excessive salivation and bladder or bowel incontinence. which of the following does not occur ? D. Simple partial seizure 266 During tonic phase of Grand Mal seizure.180 seconds D. Anoxic brain injury D. Complex partial seizure n Harrison’s 18th Ed. B. B.120 seconds C. Sudden loss of postural muscle tone Tonic contraction of muscles of expiration and larynx at the onset will produce a loud moan or “Ictal B. Small temporal horn D. Mutations in KCNQ2 (20q13. AJAY MATHUR Cardiology 665 C. Named after Canadian neurologist Juhn A. Consciousness is preserved. 3255 Table 369-3 A. It has autosomal dominant inheritance. Mesial temporal lobe epilepsy (MTLE) D. 3253 B. All of the above Harrison’s 18th Ed. Cerebral functions are localized to which hemisphere seizures in adolescents (12 . Autosomal dominant inheritance Harrison’s 18th Ed. It is B. Sodium amobarbital is injected into one cerebral hemisphere at a time. also known as Janz syndrome.18 years) who have normal intellectual function. mutation occurs in the SCN1B (19q12.1) gene for the voltage-gated sodium channel which modifies gating and inactivation properties of the C. Infants are normal with no neurologic or metabolic abnormality. 3254 Table 369-2 Harrison’s 18th Ed. IQ level characterized by onset of myoclonic seizures. CSTB mutation causes PME. 3253 D. 3254 Table 369-2 Triad of Lennox-Gastaut syndrome includes multiple seizure types. slow (<3 Hz) spike-and-wave A. None of the above C. On MRI. D. EEG shows slow (<3 Hz) spike-&-wave discharges 282 Which of the following about Generalized epilepsy with febrile C. Normal MRI potassium channels on chromosomes 20q. Evening 281 Intracranial amobarbital or Wada test is performed for ? D. Juvenile myoclonic epilepsy (JME) channel. multiple seizure types include generalized tonic. B. 285 Gene responsible for progressive myoclonus epilepsy (PME) or 280 In Mesial temporal lobe epilepsy (MTLE). atonic. namely speech and memory. Sleep pattern and can be provoked by sleep deprivation. gene for D. Harrison’s 18th Ed. 3253 for certain vital cognitive functions. Multiple seizure types (Wada) test is a feature of MTLE. the MRI finding include Unverricht-Lundborg disease is ? all except ? Harrison’s 18th Ed. (BFNC) is false ? Harrison’s 18th Ed. generalized tonic–clonic seizures. Atonic plus at least one other type of seizure (absence. CHRNA4 278 Most common syndrome associated with focal seizures with B. May have variable seizure types not associated with fever Harrison’s 18th Ed. Material-specific memory deficits on intracranial amobarbital A. 665 Neurology MCQ’s FORMCQ’s FOR MEDICAL MEDICAL PROFESSIONALS PROFESSIONALS BY PROF. The aim is to determine which side of the brain is responsible Harrison’s 18th Ed. temporal lobe.3) B. the characteristic MRI finding is ? A. Generalized tonic-clonic “Generalized epilepsy with febrile seizures plus” is a genetic syndrome consisting of febrile seizures B. ataxia and MRI findings in Mesial Temporal Lobe Epilepsy Syndrome include small hippocampus. Night A. EPM2A B. atonic and atypical absence seizures. 3254 Table 369-2 clonic. They are most frequent in morning D. 3254 Table 369-2 279 In Mesial temporal lobe epilepsy (MTLE). Hippocampal calcification C. KCNQ2 Harrison’s 18th Ed. Circulation adequacy of cerebral hemispheres Syndrome of JME. Onset in first week of life A. Mutations have been identified in KCNQ2. Small hippocampus B. accounts for ~7% of all cases of epilepsy. CSTB C. Hippocampal sclerosis D. generalized spike-wave & polyspike-wave discharges. 1940). A. LGI1 B. C. this test is used 276 Which of the following statements about Lennox-Gastaut syndrome to establish which cerebral functions are localized to which hemisphere. SCN1B dyscognitive features is ? C. characteristic & essential finding of MTLE is hippocampal sclerosis. enlarged temporal horn and hippocampal sclerosis. Autosomal recessive inheritance A. or afebrile generalized tonic–clonic seizures). 3253 A. Increased signal intensity in frontotemporal region BFNC has autosomal dominant inheritance with onset in 1st week of life. small progressive cognitive decline. Lennox-Gastaut syndrome In Generalized epilepsy with febrile seizures plus (GEFS+). Atypical absence seizures 283 Which of the following gene is associated with Generalized epilepsy with febrile seizures plus (GEFS+) ? In children with Lennox-Gastaut syndrome. . Presents with febrile seizures 277 Seizure types in Lennox-Gastaut syndrome include all except ? C. SCN1B D. Small temporal lobe C. myoclonic. LGI1 A. B. Formerly it was known as “psychomotor epilepsy” (Gibbs et al. with EEG findings of rapid. Hippocampal sclerosis Progressive myoclonus epilepsy (PME) (Unverricht-Lundborg disease) has autosomal recessive inheritance with age of onset between 6-15 years presenting as myoclonic seizures. & absence C. It is also known as the “intracarotid sodium amobarbital procedure” (ISAP) and is performed prior to ablative surgery for is true ? epilepsy and prior to tumor resection. Myoclonic D. All of the above Mesial temporal lobe epilepsy (MTLE) is the most common syndrome associated with focal seizures 284 Which of the following about Benign familial neonatal convulsions with dyscognitive features. Wada (University of British Columbia). Mutations in other sodium channel subunits (SCN1A and SCN2A) and GABAA receptor subunit (GABRG2 and GABRA1) have been reported. Impaired cognitive function seizures plus (GEFS+) is false ? D. Autosomal dominant inheritance discharges on EEG and impaired cognitive function. haematobium A. Gene mution in SCN1B 294 Which of the following schistosomes is associated with associated Gene mution occurs in EPM2A (6q24). S. A. japonicum D. Age of onset usually between 10 and 25 years species ? Harrison’s 18th Ed. Wall attacks i 297 Which of the following is false about seizures due to inborn 290 West’s syndrome includes ? h errors of metabolism ? A. Infants and children (>1 month and <12 years) D. r D. 1756 Harrison’s 18th Ed. Febrile convulsions B. Auditory agnosia or “word deafness” B. Myoclonic epilepsies due to storage diseases B. CHRNA4 G D. Hearing tests show normal hearing. 1 and 3 months D. S. 3255 A.Laforin influences glycogen metabolism. D. Polyglucosan intracellular inclusion bodies D. 9 and 18 months and may have epileptic seizures usually at night. S. 3 and 9 months A childhood disorder. 3254 Table 369-2 D. haematobium - 289 Episodic dyscontrol syndrome is also called ? C. Age of onset . d B. Normal hearing 299 Febrile seizures have a peak incidence between ? Harrison’s 18th Ed. Young adults (18 . Early cryptogenic focal epilepsy C. S. 1756 Autosomal dominant nocturnal frontal lobe epilepsy (ADNFLE) occurs during childhood & presents as A. 1756 287 Gene responsible for Autosomal dominant nocturnal frontal lobe A.6 to 19 years C. Infantile spasms Harrison’s 18th Ed. Gene responsible is CHRNA4 (20q13.666 MCQ’s FOR MEDICAL PROFESSIONALS BY PROF. haematobium C. mansoni B. mental retardation & hypsarrhythmic EEG pattern. S. All of the above C. nighttime seizures with prominent motor movements. S. intercalatum Harrison’s 18th Ed. Time attacks CNS schistosomiasis occurs as Jacksonian epilepsy due to S. japonicum infection. Type of idiopathic lateral temporal lobe epilepsy 296 Which of the following schistosomes belongs to the urinary n B. S. AJAY MATHUR Neurology 286 Which of the following is false about Progressive myoclonus 293 Which of the following schistosomes causes Jacksonian epilepsy ? epilepsy (Lafora’s disease) ? Harrison’s 18th Ed. S. with transverse myelitis ? Harrison’s 18th Ed. mekongi B. All of the above B. Severe myoclonic epilepsy in infancy A. mekongi A. S.35 years) 292 Characteristics of Landau-Kleffner syndrome include ? Pyridoxine (vitamin B6) deficiency is an important cause of neonatal seizures. Gene mutation in LGI1 (10q24) U A. mansoni 9 C. Hypsarrhythmic EEG B. 3254 Table 369-2 A. EPM2A R C. S. Autosomal recessive inheritance B.3 days after birth D. The protein tyrosine phosphatase (PTP) . in LKS there occurs gradual or sudden loss of ability to understand and use spoken language (auditory agnosia or “word deafness”). None of the above 291 Dravet syndrome best relates to which of the following ? 298 Pyridoxine deficiency causes seizures during ? Harrison’s 18th Ed. S. Idiosyncratic side effect to Valproic acid common West syndrome is a triad of infantile spasms. S. Rage attacks D. haematobium infection leads to dysuria. Occur once regular feeding begins C. mansoni 288 Which of the following about Autosomal dominant partial epilepsy ti e C. SCN1B V Harrison’s 18th Ed. 1756 C.18 years) D. S. None of the above B. S. . Neonatal period (<1 month) C. 3255 ta B. All children with LKS have abnormal EEG C.2). japonicum epilepsy (ADNFLE) is ? B. Occur typically 2 . Adolescents (12 . Ground attacks & S. Mental retardation A. japonicum brief. mekongi D. intercalatum A. 3256 C. S. mekongi 9 B. frequency and hematuria. S. intercalatum with auditory features (ADPEAF) is false ? Harrison’s 18th Ed. CSTB 295 Which of the following schistosomes is associated with associated with transverse myelitis ? D. 3254 Table 369-2 C. haematobium infections have been associated with transverse myelitis. Abnormal electrical brain activity A. japonicum A. S. S. 3256 A. Aspirin D. All of the above Antimalarials . Chronic seizures are associated with all forms of stroke.18 years) Ion-channel mutations “channelopathies” are characterized by paroxysmal episodes of neurologic D.18 years) D. Hemorrhagic stroke C. Acyclovir C. Repeated seizure activity Harrison’s 18th Ed. Gamma-aminobutyric acid (GABA) receptors C. 309 Which of the following antimicrobial / antiviral can cause epilepsy ? 302 Primary epilepsy syndromes present during which of the following Harrison’s 18th Ed. familial hemiplegic migraine. Adolescents (12 . Rising phase of temperature curve D. while complex febrile seizures have a risk of 2 . Duration > 15 minutes 308 Which of the following analgesics can cause epilepsy ? B. A. 18 and 24 months 306 Which of the following bronchodilators cause epilepsy ? Harrison’s 18th Ed. Quinine B. and the long-QT syndrome.18 years) D. 3257 Table 369-5 Febrile seizures usually occur between 3 months and 5 years of age and have a peak incidence between 18 and 24 months. Diclofenac sodium C. Isoniazid Febrile seizure is likely to occur during the rising phase of the temperature curve. Familial hemiplegic migraine A. Salbutamol 300 Febrile seizure is likely to occur during ? Harrison’s 18th Ed. Terbutaline A. Young adults (18 . Episodic ataxia Harrison’s 18th Ed. 3257 Table 369-5 D. 3256 Table 369-4 B. Nimusilide epilepsy. Pyrazinamide 301 Which of the following is false about complex febrile seizures ? D. Adolescents (12 . 3257 Table 369-5 303 Acute seizure in older adults is mostly due to ? A. Thrombotic stroke D. Neonatal period (<1 month) B. 305 Hypocalcemia or hypomagnesemia are a cause of seizure disorder 312 The initial bursting activity in seizure is caused by ? in which of the following age groups ? Harrison’s 18th Ed. Embolic stroke B. Ethambutol Harrison’s 18th Ed. periodic paralysis. Young adults (18 . or by focal features. All of the above D. Infants and children (>1 month and <12 years) C. Long-QT syndrome B.35 years) 310 Which of the following antimalarials can cause seizure ? Harrison’s 18th Ed. Simple febrile seizures are not associated with an increase in the risk of developing D. All of the above D. 3256 A. Focal features B. Quinolones Harrison’s 18th Ed.chloroquine and mefloquine can cause seizure.35 years) or cardiac dysfunction. Declining phase of temperature curve Harrison’s 18th Ed. Rifampin C. Artemether C. 3256 Table 369-4 A. 667 Neurology MCQ’s FORMCQ’s FOR MEDICAL MEDICAL PROFESSIONALS PROFESSIONALS BY PROF. Ipratropium bromide B. 3257 Table 369-5 C. Not associated with increased risk of developing epilepsy A. AJAY MATHUR Cardiology 667 D. Neonatal period (<1 month) B. Infants and children (>1 month and <12 years) D. 3257 Table 369-5 age groups ? A. B. Infants and children (>1 month and <12 years) C. duration >15 minutes. include ? N Engl J Med 2003. Ganciclovir B. 3256 C. Opening of voltage-dependent sodium (Na+) channels B. Adolescents (12 . Tramadol Complex febrile seizures are characterized by repeated seizure activity. Acute seizures occurring at the time of stroke are seen more often with embolic rather than 311 Diseases due to ion-channel mutations “channelopathies” hemorrhagic or thrombotic stroke. All of the above C. Peak of temperature curve 307 Which of the following anti-tubercular drug cause epilepsy ? C. Theophylline B.35 years) . Mefloquine Harrison’s 18th Ed. Any of the above A. 3256 Harrison’s 18th Ed. include episodic ataxia.349:1257-66 304 Seizure disorder is not “idiopathic” in which of the following age groups ? A. Neonatal period (<1 month) C. Young adults (18 . Influx of extracellular calcium (Ca2+) A. Halofantrine D.5 %. 3256 A. electrolyte or metabolic derangements. If a forearm pronates then Hippocampus consists of 3 major regions: subiculum. Phenytoin C. Lamotrigine D. hypoxia. Arginine B. B. 3258 C. 3258 (Ca2+). 3255. Glycine V Harrison’s 18th Ed. Benzodiazepine D. Sleep deprivation Ethosuximide & valproic acid act by inhibiting T-type Ca++ channels in thalamic neurons. Benzodiazepine D. Valproic acid . 3258 A. AMPA A. acute infection. hippocampus proper (Ammon’s horn) & dentate the person is said to have pronator drift on that side. Valproic acid G C. Subiculum ta Precipitating factors of epilepsy include sleep deprivation. One that acts by inhibiting voltage-gated Ca++ channels is phenytoin A. Opening of voltage-dependent sodium (Na+) channels A. which leads to the opening of voltage-dependent sodium (Na +) channels. GABA is an inhibitory neurotransmitter in adult CNS Harrison’s 18th Ed. carbamazepine. gyrus. L-tryptophan A. depending on cell type. topiramate & zonisamide. Tiagabine mediated by gamma-aminobutyric acid (GABA) receptors or potassium (K +) channels. gabapentin & tiagabine increase availability of GABA. Dentate gyrus 317 Pronator drift is an indicator of ? D. 3258 A. All of the above D. or B. CA3.668 MCQ’s FOR MEDICAL PROFESSIONALS BY PROF. Benzodiazepine glutaminergic receptors are excitatory. CA 2 C. Gabapentin 314 Which of the following is a co-agonist of glutamate at the NMDA receptor ? D. 3257 is ? A. This is followed by a hyperpolarizing afterpotential A. Phenytoin C. All of the above Pronator drift is a pathologic sign and indicates UMN type weakness. The initiation phase is characterized by two concurrent events in an aggregate of neurons: (1) high-frequency 320 Antiepileptic drug that acts by increasing the availability of GABA bursts of action potentials and (2) hypersynchronization. Valproic acid 315 Which of the following about gamma-aminobutyric acid (GABA) 322 Antiepileptic drug that acts by inhibiting voltage-gated Ca++ n is false ? channels is ? A. Subiculum is the transition zone from the three to six layered cortex. None of the above C. AJAY MATHUR Neurology 313 Hyperpolarizing afterpotential during seizure is mediated by ? 319 Antiepileptic drug that acts by increasing the availability of GABA Harrison’s 18th Ed. Lamotrigine C. 9 lamotrigine. Valproic acid 9 316 Which of the following is a precipitant of epilepsy ? Harrison’s 18th Ed. Person is asked to extend arms by 90 degrees. All of the above Harrison’s 18th Ed.e. 3257 Antiepileptic drugs that act by inhibiting Na+ dependent action potentials are phenytoin. Ammon’s horn alcohol or illicit drug use/withdrawal. systemic diseases. Benzodiazepines & barbiturates potentiate r GABA receptor function. CA 3 D. Gamma-aminobutyric acid (GABA) receptors B. GABAB receptors are located presynaptically B. NMDA C. there is a seizure initiation phase and a seizure propagation phase. C. Lower motor neuron weakness 324 Which of the following is a part of Ammon’s horn ? A. NMDA C. Phenytoin d D. Hormonal changes with menstrual cycle h D. drugs that lower the seizure threshold. while valproic acid. Tyrosine 321 Antiepileptic drug that acts by decreasing glutamate release is ? B. Muscle wasting C.. 3258 U B. and generation of repetitive action potentials. i B. Phenytoin B. All of the above 323 Which of the following is a part of hippocampus ? A. Sensory neuropathy D. Lamotrigine - D. influx of Na +. Hippocampus & dentate gyrus have a three layered cortex. CA1 B. 318 Antiepileptic drug that acts by potentiation of GABA receptor function is ? 325 Which of the following is a glutamate receptor ? Harrison’s 18th Ed. Benzodiazepine D. Important regions of hippocampus proper include CA1. Upper motor neuron weakness B. Valproic acid Focal seizure activity can begin in a very discrete region of cortex and then spread to neighboring regions. All of the above R A. Influx of extracellular calcium (Ca2+) Harrison’s 18th Ed. All of the above C. The bursting activity is caused by a is ? relatively long-lasting depolarization of the neuronal membrane due to influx of extracellular calcium Harrison’s 18th Ed. CA 2. GABAA receptors are located postsynaptically A. Psychological or physical stress while lamotrigine acts by decreasing glutamate release. supinate forearms. fever. C. 3258 B. i. D. Lamotrigine ti e Glycine is a co-agonist of glutamate at the (N-methyl D-aspartate) NMDA receptor. close eyes and hold the position. Kainate (Ka) B. 333 In EEG. but not asleep. which of the following signifies an encephalopathy ? A. (parieto-occipital). Chloral hydrate Harrison’s 18th Ed. Postsynaptic potentials D. Most prominent when subject is quietly relaxed with eyes closed. that undergoes amplitude attenuation with eye opening or mental alerting activities. Beta rhythm D. 100 microvolt D. Beta rhythm B. benzodiazepines and chloral hydrate increase beta activity.3-dihydro-5-methyl-3-oxo-4-isoxazolepropanoic acid (AMPA). AMPA B. activity. A. D. 1000 microvolt GABA A receptor agonists are barbiturates and benzodiazepines. Diffuse slow activity signifies an encephalopathy. theta (4-7+ Hz). None of the above The rhythmic activity normally recorded represents the postsynaptic potentials of vertically oriented 335 Which of the following about alpha rhythm is false ? pyramidal cells of the cerebral cortex. Benzodiazepine C. which is the slowest frequency in EEG ? C. Delta rhythm . NMDA D. Beta C. Alpha rhythm A. 328 In EEG. Delta Alpha rhythm is 8 and 12 Hz. calibration marker shows what level of voltage as 327 Which of the following is a GABAB agonist ? standard ? A. The calibration marker shows one second and GABA B receptors agonist is baclofen and exacerbates hyperexcitability and seizures. is visualized on the EEG as a “jitteriness” of the trace. They suppress seizure activity. 30 . Glutamate receptors are postsynaptic. All of the above B. such as components of alpha & high frequency “jittery” activity in the beta range. Baclofen A. Beta Beta activity is a fast rhythm of 13 per second and greater. Benzodiazepines 330 Out of the following. None of the above C. D. AMPA & kainate agonists induce seizure activity. 326 Antagonists to which of the following glutamate receptor 332 Which rhythm produces an underlying undulation of EEG ? suppresses seizure activity ? A. waxing and waning. AJAY MATHUR Cardiology 669 Major excitatory neurotransmitter is amino acid glutamate. Theta rhythm C. Alpha rhythm is also called Berger rhythm. usually in the frequency range of about 20-50 Hz. rhythmic activity of vertically oriented pyramidal cells of 334 Normal EEG background voltages are in the range of ? cerebral cortex normally recorded represents ? A. 20 . EEG waveforms are divided into four major frequency bands: delta (0-3+ Hz). These allow ion influx upon activation by It is called the posterior dominant rhythm or alpha rhythm. Higher on right side of the brain B. Chapter e45 B. Delta 337 In EEG. All of the above Delta activity produces an underlying undulation of EEG. Alpha rhythm C. Hz. Diffuse fast frequency C. Localized slow frequency 331 “Posterior dominant rhythm” is also called ? B. A symmetrical rhythm is observed over the posterior head regions during relaxed wakefulness with Ionotropic subclasses are alpha-amino-2. Barbiturate B. 10 microvolt C. as in adults it has a frequency of 8-13 glutamate. Theta D.100 microvolts B. Kainate (Ka) C. Theta D. Barbiturates. Presynaptic potentials C. C. All of the above quietly relaxed with eyes closed. whereas their antagonists suppress seizure in the other bands. All of the above D. 1 microvolt B. kainate receptors (Ka) & N-methyl-D-aspartate (NMDA).20 microvolts Harrison’s 18th Ed. Barbiturates B.150 microvolts C. with a posterior maximum. alpha (8-13 Hz). and beta (>14 Hz). but not asleep 329 In EEG. 10 . It can spread into any electrodes of 10-20 system except Fp1 & Fp2. Chapter e45 A. 100-microvolt standards. It is most prominent when subject is D. They are named in Greek order of historical discovery rather than low 336 Which of the following medications increase  activity in EEG ? to high frequency numbers Muscle artifact is high amplitude irregular activity. Diffuse slow frequency B.and Postsynaptic potentials Normal EEG background voltages are in the range of 30-100 microvolts. Alpha D. Chapter e45 B. Localized fast frequency Harrison’s 18th Ed. eyes closed. Riding on top of delta activity may be frequencies NMDA. 669 Neurology MCQ’s FORMCQ’s FOR MEDICAL MEDICAL PROFESSIONALS PROFESSIONALS BY PROF. A.50 microvolts A. which of the following rhythm is faster than alpha rhythm ? Harrison’s 18th Ed. Each EEG channel is a record of voltage over time. Alpha rhythm usually is higher on right side of brain. Chapter e45 A. Difference of Pre. Waxing and waning A. 40 . Theta rhythm Alterations of brain function results in abnormal slow frequency activity in EEG. Delta rhythm D. FLAIR A. Multifocal VEP is likely to be more sensitive than routine VEP. 3260 348 Factors associated with a high risk of recurrent seizures include A. To provoke abnormalities. Chapter 360 B. P200 response 340 Usually. 346 In visual evoked potentials (VEPs). Hypothermia complexes. Sharp waves 345 Electrocerebral silence is observed in ? C. sleep deprivation. In epileptic. Fluid-augmented inversion radiography Harrison’s 16th Ed. cm. 12 hours of scalp. which of the following suggest epileptiform activity ? C. Harrison’s 18th Ed. P100 response B. Visual cortex D. Chapter e45 N Engl J Med 2001. P400 response A. P100 is restored. 342 Which of the following about EEG is false ? Harrison’s 18th Ed. left- D. All of the above sided placements by odd numbers FLAIR has increased sensitivity for detection of abnormalities of cortical architecture.344:1146 D. Chapter e45 D. ~ 6 sq. cm. Onset of seizure disorder during childhood 343 Which of the following is useful in the evaluation of a patient with D. because EEG shortly In acute severe optic neuritis. Fluid-attenuated inversion radiation Encephalopathy ? B. Anterior to optic chiasm A. Photic stimulation C. and hyperventilation (Activating procedures). B. P300 response episode ? N Engl J Med 2001. VEPs may be normal. Spikes D. clinical importance is ? D. AJAY MATHUR Neurology 338 In EEG. Spike-wave complexes Harrison’s 18th Ed. Alpha rhythm attenuates with eye opening C. None of the above C. sharp waves and spike-wave B. EEG should be recorded occurs and visual acuity improves. cm. 3260 350 Delta activity in EEG is seen in which stage of Hepatic A. C. ~ 12 sq. Any of the above A. or subclinical optic neuritis. A. Chapter e45 A. Fluid-attenuated inversion recovery B. Stage I . It happens in irreversible brain damage. ~ 9 sq. EEG abnormalities D. VEP findings are helpful in indicating previous during sleep. Optic chiasm B. A. Sleep B.670 MCQ’s FOR MEDICAL PROFESSIONALS BY PROF. Presence of epileptiform activity is not specific for epilepsy all except ? Harrison’s 15th Ed. All of the above VEPs are most useful in detecting dysfunction of the visual pathways anterior to the optic chiasm. 3260 349 Which of the following statements is false about EEG ? Harrison’s 18th Ed. EEG is performed after how many hours after a seizure C. PET C. In cortical blindness. and midline placements by Z. 48 hours 347 VEPs are most useful in detecting dysfunction of visual pathways at the level ? 341 EEG should include recordings during ? Harrison’s 18th Ed. 344 FLAIR stands for ? Harrison’s 18th Ed. SPECT B. Right-sided placements are indicated by even numbers. 6 hours Visual evoked potentials (VEPs) are elicited by monocular stimulation with a reversing checkerboard pattern and are recorded from the occipital region in the midline and on either side B. cm. the component of major C. hypothermic patients or with drug overdose. there is a reduction in amplitude of EEG until activity cannot be detected. Normal EEG shows posteriorly situated 8-13 Hz  rhythm B. C. Chapter e45 Each EEG electrode can detect synchronous activity generated by ~ 6 sq. The component of major clinical importance is P100 response. 24 hours D. Midline placements are indicated by M to evaluate patients with medically refractory seizures. Seizures that require more than one antiepileptic drug epilepsy ? Harrison’s 18th Ed. Known structural lesion C. It is customary to perform EEG 48 hours or more after a suspected seizure. Fluid-attenuated inversion radiography A. photic stimulation. All of the above how much area of surface cerebral cortex ? In electrocerebral silence.344:1146 A. Irreversible brain damage Epileptiform activity characteristic of epilepsy includes spikes. Hyperventilation D. of cortex. A normal EEG implies a better prognosis B. left-sided placements by odd numbers. P100 is frequently lost or grossly attenuated and as clinical recovery after a seizure may give misleading findings. Drug overdose 339 Each EEG electrode can detect synchronous activity generated by D. cm. Right-sided electrode placements are indicated by even numbers. 1868 A. Optic radiation C. initial interictal EEG is normal in ~ 60% A. ~ 3 sq. Positron emission tomography (PET) & single photon emission computed tomography (SPECT) are used D. a positive peak having a latency of approximately 100 ms. Valproic acid D. Carbamazepine B. D. Ethosuximide A. Rufinamide. Anti-epileptic drugs useful in Lennox-Gastaut syndrome include Lamotrigine. 3262 Table 369-8 352 The specific order of electrode pairs is called ? A. an Hepatic encephalopathy is associated with triphasic waves. whereas psychogenic seizures are not. Abnormal EEG C. 3262 Table 369-8 353 Which of the following statements about therapeutic range of A. Gabapentin 355 “Reflex epilepsy” best relates to ? 362 Which of the following anti-epileptic drugs is preferred in Lennox- Harrison’s 18th Ed. Peripheral WBC count also increase significantly after a generalized seizure. Prelage C. Lamotrigine 354 Estimation of which of the following helps to distinguish between organic and psychogenic seizures ? C. Stage III D. Ethosuximide 40–120 µg/ml. Serum vasopressin levels B. seizures presenting as status epilepticus.20 µg/ml 360 Which of the following is the first-line medication in myoclonic D. periodic lateralized epileptiform discharges (PLEDs) is typical of herpes simplex encephalitis. 671 Neurology MCQ’s FORMCQ’s FOR MEDICAL MEDICAL PROFESSIONALS PROFESSIONALS BY PROF. Physical exertion C. Oxcarbazepine is not established. Specific music or an individual’s voice B. Phenobarbital 10 . Therapeutic range for Gabapentin. Montage B. Ethosuximide D. Valproic acid B. Carbamazepine C. Stage II C. Valproic acid A. Carbamazepine 15 . Serum growth hormone levels Harrison’s 18th Ed.40 µg/ml seizures ? Harrison’s 18th Ed.20 µg/ml C. Lamotrigine Most generalized seizures and many complex partial seizures are accompanied by rises in serum C. Lennox-Gastaut syndrome). Felbamate. Abnormal neurologic examination B. Valproic acid A. Serum insulin levels 361 Which of the following is not a first-line medication in seizures ? B.150 µg/ml D. Carbamazepine C. Phenobarbital C. A. 3262 Table 369-8 Harrison’s 16th Ed. absence seizures ? Harrison’s 18th Ed. 351 In EEG. Ethosuximide D. strong family history of seizures. or an individual’s voice are categorized as “reflex epilepsy”.g. Herpes simplex encephalitis C. Harrison’s 18th Ed. abnormal EEG. 3262 Table 369-8 Therapeutic range of Carbamazepine is 4–12 µg/ml. Carbamazepine prolactin during the immediate 30 minute postictal period. Jakob-Creutzfeldt disease D. AJAY MATHUR Cardiology 671 B. Hepatic encephalopathy 358 Which of the following is the first-line medication in typical absence In EEG. All of the above D. Levetiracetam. Stage IV Risk factors associated with recurrent seizures include an abnormal neurologic examination. Valproic acid 50 . Valproic acid antiepileptic drugs is false ? Harrison’s 16th Ed. Ethosuximide C. Primidone 5–12 µg/ ml. Carbamazepine Harrison’s 18th Ed.. music. Consure D. 3262 Gastaut syndrome ? A. Sleep deprivation A. 3262 Table 369-8 C. 3261 D. seizures ? Harrison’s 18th Ed. Lamotrigine B. Valproic acid D. usually when they are part of a mixed-seizure syndrome (e. Serum prolactin levels A. Lamotrigine B. Lamotrigine. Lamotrigine A. Primark 359 Which of the following is the first-line medication in atypical The specific order of electrode pairs is called a montage. Gabapentin D. Corpus callosotomy has been shown to be effective for disabling tonic or 356 Risk factors associated with recurrent seizures include ? atonic seizures. Alcohol intake Harrison’s 18th Ed. Phenytoin 10 . Lamotrigine Seizures that are induced by highly specific stimuli such as a video game monitor. Seizures presenting as status epilepticus . Topiramate. 3262 Table 369-9 B. 3262 A. 1868 A. postictal Todd’s paralysis. Subacute sclerosing panencephalitis (SSPE) B. periodic lateralized epileptiform discharges (PLEDs) is 357 Which of the following is the first-line medication in focal seizures ? typical of ? Harrison’s 18th Ed. 2368 B. All of the above Valproic acid is the treatment of choice for juvenile myoclonic epilepsy. 3262 Harrison’s 17th Ed. Carbamazepine can cause leukopenia. All of the above During pregnancy. or hepatotoxicity 374 Which of the following anti-epileptic medications can significantly 368 Treatment of choice for juvenile myoclonic epilepsy is ? antagonize effects of oral contraceptives ? N Engl J Med 1999. 3266 A. 2510 A. 20 % B. Topiramate D. Gabapentin A. and atonic seizures. 4 months C. All of the above Most common surgical procedure for temporal lobe epilepsy is resection of anteromedial temporal lobe. Phenobarbital A. 372 Most common surgical procedure for patients with temporal lobe epilepsy involves resection of ? 366 Carbamazepine and phenytoin are contraindicated in which of Harrison’s 18th Ed. phenobarbital. to Stevens-Johnson syndrome if unrecognized and if the medication is not discontinued immediately. 10 % A. Lamotrigine D. Levetiracetam C. Gabapentin B. 371 What percentage of patients with epilepsy do not respond to inducing effects ? treatment with a single antiepileptic drug ? Harrison’s 18th Ed. Anterolateral temporal lobe B. Myoclonic seizures D. aplastic anemia. topiramate. Topiramate D. Valproic acid B. Carbamazepine. ~ two-third Enzyme-inducing anti-epileptic drugs (phenytoin. of oral contraceptives via enzyme induction. Aplastic anemia C. Posteromedial temporal lobe C. oxcarbazepine. ~ one-fourth B. clonazepam. All of the above D. 365 Which of the following anti-epileptic medications have enzyme. 3269. Phenobarbital B. tonic. 3 months B. 3267 the following epilepsies ? A. seizure frequency will remain unchanged in ~50% of women. myoclonic. Carbamazepine. primidone & topiramate are useful. N Engl J Med 1999. 3264 Table 369-9 Harrison’s 18th Ed. 3269 367 Which of the following is the adverse effect of carbamazepine ? Harrison’s 18th Ed. increase in 30%. Carbamazepine B. 30 % C. Valproic acid A. Phenytoin A.672 MCQ’s FOR MEDICAL PROFESSIONALS BY PROF.340:1565 Harrison’s 18th Ed. of women ? Harrison’s 18th Ed. 3262 A. Juvenile myoclonic epilepsy (JME) B. oxcarbazepine. Acetazolamide C. 3262 Table 369-9 Harrison’s 17th Ed. Anteromedial temporal lobe Harrison’s 18th Ed. and phenytoin can worsen certain types of generalized seizures. 2507 A. Leukopenia B. ~ one-third C. phenobarbital. carbamazepine. and topiramate can significantly antagonize the effects acetazolamide. and decrease in 20%. This can be extremely severe and lead Most recurrences occur in the first 3 months after discontinuing therapy. 373 During pregnancy. Hepatotoxicity D. 3266. Absence seizures C. AJAY MATHUR Neurology 363 Which of the following anti-epileptic drugs does not have 369 Which of the following anti-epileptic medications rarely cause significant drug interactions ? skin rashes ? Harrison’s 18th Ed. 3269 A. Lamotrigine D. 5 months D. About one-third of epileptic patients do not respond to treatment with a single antiepileptic drug. but lamotrigine. Posterolateral temporal lobe D. Lamotrigine B. Primidone C. Divalproex sodium C. phenytoin. seizure frequency decreases in what percentage including absence. ~ one-half D. Phenytoin C. 40 % D. All of the above 364 Most recurrences of epilepsy occur how many months after 370 Which of the following anti-epileptic drug causes skin rashes ? discontinuing therapy ? Harrison’s 18th Ed. . 6 months Lamotrigine causes skin rash during initiation of therapy. Levetiracetam D. Divalproex sodium B.340:1565 A. Gabapentin C. and primidone) cause a transient and reversible deficiency of vitamin K–dependent clotting factors in about 50% of newborn infants. which of the following leads to irreversible neuronal injury ? C. Vomiting Harrison’s 18th Ed. 1624 B. It may result from brainstem-mediated effects of seizures on Harrison’s 16th Ed. May result from brainstem-mediated effects of seizures on cardiac rhythms or pulmonary function D. Impaired consciousness in the interictal period menses. and 5% for valproic acid. 2512 D. All of the above 379 In an awake patient of uremic encephalopathy. Phenytoin A. No effect B. Continuous seizures There may be an increase in seizure frequency around menses due to effects of estrogen or progesterone B. Acetazolamide Harrison’s 18th Ed. Primary generalized seizure C. Cardiorespiratory dysfunction 376 Ratio of drug concentration in breast milk relative to serum is B.2 minutes to be repeated once if no response is obtained after 5 minutes. Valproate 377 One of the most specific signs of a metabolic encephalopathy is ? D. Phenobarbital 382 In generalized convulsive status epilepticus (GCSE). Its subtypes include generalized convulsive status Harrison’s 16th Ed. C. Metabolic derangements Harrison’s 16th Ed. Increase seizure frequency Harrison’s 16th Ed. 40% for carbamazepine. Valproic acid B. 2372 epilepticus (GCSE) and nonconvulsive status epilepticus (persistent absence seizures or focal seizures). D. A. 1625 cardiac rhythms or pulmonary function. Carbamazepine Harrison’s 18th Ed. 3268 A. Repetitive. Phenobarbital 383 Which of the following is the first medicine that is recommended for generalized tonic-clonic status epilepticus in adults? C. Partial seizure B. Any tonically held posture C. the typical 385 Which of the following is false about “Sudden unexpected death movement abnormality is ? in epileptic patients (SUDEP) ? Harrison’s 16th Ed. Prednisolone B. Decrease seizure frequency A. discrete seizures on neuronal excitability or changes in antiepileptic drug levels due to altered protein binding. Partial seizure catamenial epilepsy ? C. 3268 Figure 369-3 D. 1624 Lorazepam is recommended for generalized tonic-clonic status epilepticus in adults in doses of A. Absence seizure D. Absence seizure A. Carbamazepine Harrison’s 18th Ed. High doses of tricyclic antidepressants may lower the seizure threshold. Seizure inhibitors (SSRIs) on seizures ? D. 673 Neurology MCQ’s FORMCQ’s FOR MEDICAL MEDICAL PROFESSIONALS PROFESSIONALS BY PROF. Voluntary limb motion Selective serotonin reuptake inhibitors (SSRIs) typically have no effect on seizures.15 mg/kg IV over 1 . 3268 378 Asterixis can be found in ? A. 15% for phenytoin.1 . Beta blockers 381 Which of the following statements about status epilepticus is false ? C. D. Lorazepam Ratio of drug concentration in breast milk relative to serum is ~80% for ethosuximide. Tonically held tongue D. Affects young people with convulsive seizures B. 40–60% for B. A. AJAY MATHUR Cardiology 673 375 Ratio of drug concentration in breast milk relative to serum is Status epilepticus refers to continuous seizures or repetitive. discrete seizures with impaired maximum with which of the following antiepileptic drug ? consciousness in the interictal period.0. 3267 D. Phenobarbital Harrison’s 16th Ed. Myoclonic jerking and tremor A. Ethosuximide B. Fosphenytoin phenobarbital. B. Phenytoin A. Primary generalized seizure Harrison’s 17th Ed. All of the above A. Myoclonic jerking and tremor 386 Which of the following drug is effective as adjunctive therapy in B. Diuretics A. None of the above 380 Typical movement abnormality seen with use of antipsychotic Sudden unexpected death in epileptic patients (SUDEP) affects young people with convulsive drugs such as lithium. Confusion 384 Which of the following is the effect of selective serotonin reuptake C. phenothiazines. May be nonconvulsive . Any of the above C. Tends to occur at night C. Bilateral Asterixis 0. Acetazolamide (250-500 mg/d) may be effective as adjunctive therapy when started 7-10 days before C. 1625 Harrison’s 18th Ed. 3268 D. or butyrophenones is ? seizures and tends to occur at night. 2372 D. Hyperthermia minimum with which of the following antiepileptic drug ? C. 230 Harrison’s 16th Ed. Calcium B. 397 Which of the following statements about amaurosis fugax is false ? 391 In the first 3 months after TIA. ~ 60 . D. Trauma A. Emboli from a proximal arterial source D. SAPS II Score . 3280 A. 1-5% B. 3280 A. 392 Which of the following estimates risk of stroke following TIA ? 398 Most neurologic events occur within what duration after a TIA ? Harrison’s 18th Ed. 393 Which of the following is not a part of ABCD Scoring system for 387 By definition. 3280 A. Transient ischemic attack of retina A. AJAY MATHUR Neurology C. Associated with antiphospholipid antibody syndrome D. ABCD Score A. Cerebrovascular Diseases D.50 % Emboli causing amaurosis fugax are composed of cholesterol (Hollenhorst plaque). ~ 12 minutes C. IPI score Chapter 370. Modified Child-Pugh classification is a staging system for cirrhosis. All of the above occurs from emboli to central retinal artery of one eye indicating carotid stenosis or local ophthalmic artery disease as the cause. All of the above C. with most events occurring in first 2 days. < 24 hours ABCD Scoring system for TIA includes age.60 % D. Even in absence of neurologic signs & symptoms. clinical symptoms (unilateral weakness. Typically the neurologic signs and symptoms of a TIA last for 5 to 15 minutes but. Briefer duration than migraine A. 3280 Table 370-5 Harrison’s 18th Ed. ~ 1 minute B. Emboli from the heart Amaurosis fugax or a transient ischemic attack of the retina or transient monocular blindness. Not followed by headache C.35 % C. 10 . infarcts of brain occur in 15 . The average duration of a TIA is ~12 minutes.15% in first 3 months. Cluster headache Harrison’s 18th Ed. ~ 56 minutes Amaurosis fugax usually results from an embolus stuck in a retinal arteriole. Cholesterol A. It can be a part of Risk of stroke after TIA is ~10 . 394 Amaurosis Fugax is best related to ? Harrison’s 18th Ed. speech disturbance without weakness).50% of TIAs. 230 388 Focal brain ischemia or infarction is usually caused by ? A. 396 Emboli causing amaurosis fugax are composed of ? 390 Brain infarction occurs in what percentage of of TIAs even when Harrison’s 18th Ed. Thrombosis of cerebral vessels C. high-altitude neurologic events. ~ 35 minutes D. 3270 B. duration and diabetes. Platelet-fibrin debris C. 3270 A. Diabetes D. 230 neurologic signs and symptoms are absent ? Harrison’s 18th Ed. Most common source is atherosclerotic plaque in carotid artery or aorta. 3 . ~ 10 . 15 . or platelet-fibrin debris. 395 Amaurosis Fugax is the result of ? 389 The average duration of a TIA is ? Harrison’s 18th Ed. Newly proposed definition classifies those with new brain infarction as ischemic strokes regardless of whether symptoms persist.15 % B. risk of stroke is ? Harrison’s 18th Ed. First 2 days B. Occurs in both eyes B. calcium. < 12 hours D. Age A. ~ 20 . 2379 A.20 % D.85 % Amaurosis fugax occurs in only one eye (painless monocular loss of vision). Modified Child-Pugh classification APACHE (acute physiology and chronic health evaluation) system and SAPS (simplified acute physiology score) system are severity-of-illness (SOI) scoring systems. 3280 Table 370-5 Harrison’s 18th Ed. ~ 40 . Focal brain ischemia or infarction is caused by thrombosis of cerebral vessels or by emboli from a proximal arterial source or heart. Hypertension C. 231 Harrison’s 18th Ed. Any of the above D. must last for <24 hours. by definition. Vascular spasm C. Any of the above D. Hysterical conversion reaction B. < 6 hours C.20 % C. blood pressure (SBP & DBP). International Prognostic Ischemic stroke Index (IPI) is for NHL. Sex B.674 MCQ’s FOR MEDICAL PROFESSIONALS BY PROF. < 1 hour B. Emboli B. Complete occlusion of the central retinal artery produces arrest of blood flow and a milky retina with a cherry-red fovea. neurologic signs and symptoms of TIA last for less TIA ? than ? Harrison’s 18th Ed. Surrounding the core region of infarction D. 4 to 10 minutes D. Any of the above Conditions that may mimic stroke include hypoglycemia. All of the above D. 399 Conditions that may mimic stroke include ? Harrison’s 18th Ed. Hyperglycemia In ischemic penumbra. 3271 A. Seizures D. 3271 A. Collateral channels The discrepancy between the region of poor perfusion and diffusion deficit is called diffusion. what will happen to Harrison’s 18th Ed. penumbra is the goal of revascularization therapies. 401 When cerebral blood flow falls to <16 to 18 mL/100 gram per minute. First 7 days B. Mechanical D. Tissue surrounding the core region of infarction is ischemic but reversibly dysfunctional and is called as the ischemic penumbra. 2513 A. 403 In MRI. Will continue to remain the same D. AJAY MATHUR Cardiology 675 B. Lower initial blood pressure A. 30 seconds to 1 minute A. 1 hour A. brain tumor. and a deficit that remits suggests ischemia. mechanism of cell death is ? Harrison’s 18th Ed. or worsening of symptoms after onset favor hemorrhage. Saving the ischemic Systemic hypotension & hypoxia reduce substrate delivery to vulnerable brain tissue.343:710 405 “Ischemic penumbra” is located at ? Harrison’s 18th Ed. Because collateral blood flow within the ischemic brain is blood pressure dependent. Will infarct C. 402 Which of the following is false about ischemic penumbra ? 408 Events that can cause “secondary brain insults” include all except ? Harrison’s 18th Ed. All of the above C. brain infarction occurs in 1 hour. Cerebral arteries C. Will improve B. Hypoxia C. Perfusion defect Harrison’s 18th Ed. 3271 Harrison’s 16th Ed. Apoptosis B. Reversibly dysfunctional B. 2 hour B. seizures & hyperglycemia can increase cellular metabolism. hypertensive encephalopathy. Hypoglycemia A. Diffusion deficit A. Hypertensive encephalopathy B.18 mL/100 gram/minute. 3 hour C. higher initial blood pressure. Migraine C. apoptotic cellular death occurs days to weeks later. D. None of the above D. 4 hour D. 3271 Harrison’s 17th Ed. > 140 / 90 mm Hg . 1 to 4 minutes B. 10 to 15 minutes C. 3270. BP should be lowered if it is ? Harrison’s 18th Ed. First 30 days D. 675 Neurology MCQ’s FORMCQ’s FOR MEDICAL MEDICAL PROFESSIONALS PROFESSIONALS BY PROF. 3271 ischemic penumbra ? Harrison’s 18th Ed. 3272 B. Ischemic tissue around core region of infarction A. Fever. 3295 Figure 370-16 409 BP should not be lowered acutely in acute ischemic stroke becaue it affects blood flow in ? A. which of the following is a measure of ischemic penumbra ? Harrison’s 18th Ed. seizures. 3271 410 In acute ischemic stroke. Any of the above Most neurologic events occur in first 2 days after TIA. migraine & metabolic encephalopathy. All of the above perfusion mismatch and is a measure of ischemic penumbra. Toxicity due to free radicals When cerebral blood flow falls to <16 . Imaged by perfusion-diffusion imaging with MRI C. Within the core region of infarction C. Center of the core region of infarction B. how long would it take for brain infarction ? 407 In ischemic penumbra. Necrotic C. 400 A fall in cerebral blood flow to zero causes death of brain tissue within ? 406 If no improvement in cerebral flow occurs. Risk of stroke is ~10% in first 3 months. Higher initial blood pressure C. Any of the above A fall in cerebral blood flow to zero causes death of brain tissue within 4 to 10 minutes. 2374 A. Clinically these events are known as secondary brain insults because they lead to exacerbation of the primary brain injury. cellular death occurs by apoptosis days to weeks later. Diffusion-perfusion mismatch B. The ischemic penumbra will eventually infarct if no change in flow occurs. Systemic hypertension B. hemorrhage rather than ischemia ? Harrison’s 18th Ed. First 15 days C. 3272 A. Cerebral veins D. Fluctuating blood pressure D. A more depressed level of consciousness. In Ischemic penumbra. N Engl J Med 2000. blood pressure 404 Which of the following favour the diagnosis of cerebral should not be lowered acutely. Systolic blood pressure . glistening. Calcium B. 421 Cerebral perfusion pressure (CPP) is defined as ? Harrison’s 18th Ed. Avoiding psychological stress The National Institute of Neurological Disorders and Stroke (NINDS) recombinant tPA (rtPA) Stroke Restraint therapy i. 3274 Harrison’s 18th Ed. ECASS B. Seizure disorder may precede Todd’s paresis. immobilizing unaffected side improves hemiparesis following stroke. All of the above In acute ischemic stroke. It is a sign of severe atherosclerosis & is seen in a blood vessel of retina on ophthalmoscopy as a bright. 1. All of the above American ophthalmologist Dr Robert Hollenhorst (1913-2008) who first described Hollenhorst plaque Time of stroke onset is defined as the time patient’s symptoms began or time patient was last seen as a cholesterol embolus that originated from an atheromatous plaque in a more proximal vessel. A patient who awakens with stroke has the onset defined as when they went to bed. Time the patient was last seen as normal C. Trial of Organon 10172 in Acute Stroke Treatment (TOAST). AJAY MATHUR Neurology B.6 D. Metabolic encephalopathy Prolyse in Acute Cerebral Thromboembolism (PROACT) II trial found benefit for intraarterial pro.3 mg/kg B. 3273 A. Aspirin D. and may not be present at subsequent visits. 10% as a bolus. 160 mg/day C. failed to show any benefit over aspirin. 3274 A. 75 mg/day B. an investigational low- myocardial ischemia or if BP is >185/110 mm Hg or if thrombolytic therapy is anticipated. 3274 A. Stems of ACA. Day 4 . Glycoprotein IIb/IIIa In acute ischemic stroke. cerebral edema peaks on day 2 .9 mg/kg C. 1632 Harrison’s 18th Ed. NINDS D. Preventing reinfarction B. Study used IV rtPA (0. Cholesterol A. Time the patient’s symptoms began B. 2374 A. 3293 415 In Chinese Acute Stroke Trial (CAST).e. MCA. Time to bed in those who awaken with stroke D. patients with ischemic stroke received 160 mg/day of aspirin for up to 4 weeks. 422 Which of the following is responsible for autoregulation of cerebral 416 TOAST trial investigated which of the following in acute ischemic blood flow ? stroke ? Harrison’s 16th Ed. 0. in patients with ischemic stroke within 3 hours of onset. 3272 Harrison’s 18th Ed. Checking the risk factors D.intracranial pressure A. Diastolic blood pressure . 300 mg/day D. ATLANTIS C.3 & cause mass effect for ~10 days. Seizure B. 3274 A. > 150 / 100 mm Hg B. 3273 Harrison’s 16th Ed. Checking the precipitating factors C. Checking the risk factors B. refractile plaque at bifurcation of a retinal arteriole. Mean arterial pressure (MAP) ÷ intracranial pressure D. concomitant The U. PROACT II A.2 mg/kg D. Migraine D. Mean arterial pressure . usually internal carotid artery.intracranial pressure C. PCA A. Day 1 A.1 mg/kg A. Brain tumor C. Prolonging brain’s tolerance to ischemia C. > 185 / 110 mm Hg D. 412 The established dose of rtPA to be administered within 3 hours of acute ischemic stroke is ? 418 Restraint therapy is best related to ? Harrison’s 18th Ed. 625 mg/day Cerebral perfusion pressure (CPP) is defined as the mean systemic arterial pressure (MAP) minus the intracranial pressure (ICP). Day 2 . Use of subcutaneous unfractionated heparin versus aspirin was tested in International Stroke Trial (IST).9 mg/kg to a 90-mg max. as normal. In CAST. Bacteria D.S. dose of aspirin was ? Harrison’s 18th Ed. > 165 / 110 mm Hg C. Day 5 . molecular-weight heparin. 3273 A. Low-molecular-weight heparin .3 B. 3272 417 Neuroprotection is the concept related to ? Harrison’s 18th Ed. 0. Platelet-fibrin debris C. These may break up & move. 411 In acute ischemic stroke.676 MCQ’s FOR MEDICAL PROFESSIONALS BY PROF.intracranial pressure B. then the remainder over 60 min) suggesting that physical therapy can recruit unused neural pathways (reprogramming brain). BP should be lowered if there is malignant hypertension.5 C. urokinase for acute middle cerebral artery (MCA) occlusions up to 6th hour following onset of stroke. Neuroprotection refers to providing a treatment that prolongs brain’s tolerance to ischemia. Unfractionated heparin C. Recruiting unused neural pathways D. cerebral edema peaks on ? Harrison’s 18th Ed. 419 Emboli in ‘Hollenhorst plaque’ are composed of ? 413 The time of acute ischemic stroke onset is defined as ? Harrison’s 18th Ed. 414 Which of the following studied the value of thrombolytics via an intraarterial route in acute ischemic stroke ? 420 Todd’s paresis is related to ? Harrison’s 18th Ed. 0. 1632 A. 1634. 425 Cerebral blood volume is about ? 431 What proportion of all strokes are due to small-vessel disease ? Harrison’s 16th Ed. 428 Which of the following convey a poor prognosis after a severe 434 Emboli of what size are large enough to occlude stem of MCA ? hypoxic-ischemic insult ? Harrison’s 18th Ed. Absence of pupillary light reflex day 3 after injury . 1634 Harrison’s 18th Ed. 1632 A. 3274 A. Onset B. 3274 Harrison’s 16th Ed. 288 Harrison’s 18th Ed. At 24 hours D.1 mm A. All of the above Absence of pupillary light reflex or absence of a motor response to pain on day 3 following injury. 430 What proportion of all ischemic strokes are due to carotid 424 Volume of CSF within ventricles and surrounding the brain and atherosclerosis ? Harrison’s 18th Ed. 427 Selective persistent memory deficits after brief cardiac arrest is 433 Which of the following vessels is infrequently involved in due to injury to ? embolization from heart ? Harrison’s 16th Ed. with maximum neurologic deficit at the onset. Shock A. Cerebral cortex A.2 mm B. PCO 2 D. 0. Systemic blood pressure B. ~ 40 % About 150 mL of CSF is present within the ventricles and surrounding the brain and spinal cord. Small-vessel strokes account for ~20% of all strokes. 429 What proportion of all ischemic strokes are due to cardioembolism ? 423 Cerebral blood flow is principally influenced by all except ? Harrison’s 18th Ed. or one of their branches. 100 mL C. ~ 30 % D. the PCA. 150 mL D. Asphyxiation B. 3276 A. 200 mL D. Hippocampus C. Anterior cerebral artery (ACA) territory is involved infrequently. 1635 A. Emboli from heart most often lodge in MCA. pH and PCO 2 . Embolic strokes tend to be sudden in onset. ~ 30 % C. 1632 Harrison’s 18th Ed. ~ 20 % B. PO 2 Cardioembolism is responsible for ~20% of all ischemic strokes. 677 Neurology MCQ’s FORMCQ’s FOR MEDICAL MEDICAL PROFESSIONALS PROFESSIONALS BY PROF. 1 . Hypothalamus D. 150 mL C. 3275 spinal cord is about ? Harrison’s 16th Ed. At 12 hours C. ~ 10 % A. Anterior cerebral artery (ACA) B. Branches of posterior cerebral artery Selective persistent memory deficits after brief cardiac arrest is due to injury to hippocampal CA1 neurons. Microcirculation D. 50 mL A. Middle cerebral artery (MCA) C. Myoclonic status epilepticus B. 50 mL B. ~ 20 % B. CBF increases with hypercapnia and acidosis and decreases with hypocapnia and alkalosis. ~ 20 % C. Bilateral absence of somatosensory evoked response C. ~ 40 % D. All of the above D. 3274 Harrison’s 16th Ed. Carbon monoxide poisoning C. pH C. Circle of Willis C. AJAY MATHUR Cardiology 677 B. 3274 A. 432 Sudden onset of neurologic dysfunction in embolic stroke is 426 “Histotoxic hypoxia” is produced by ? usually maximum at ? Harrison’s 18th Ed. Basal ganglia B. 200 mL 10% of ischemic stroke are due to carotid atherosclerosis. ~ 10 % B. ~ 40 % D.5 . At 48 hours Hypoxia produced by carbon monoxide and cyanide poisoning is termed histotoxic hypoxia. Cerebral blood volume is also ~150 mL. Any of the above D. Posterior cerebral artery (PCA) D. bilateral absence of early cortical somatosensory evoked response (SSEPs) and myoclonic status Cerebral blood flow (CBF) remains relatively constant over a wide range of blood pressures due to epilepticus after a severe hypoxic-ischemic insult convey a poor prognosis. ~ 30 % C. 100 mL B. physiologic autoregulation that occur in microcirculation. CBF is strongly influenced by systemic blood pressure. ~ 10 % A. 30 % C. congestive Lacunar infarction refers to infarction following atherothrombotic or lipohyalinotic occlusion of a heart failure (1). Nonvalvular atrial fibrillation C. 3276 Harrison’s 18th Ed. 446 Lacunar infarcts range in size from ? 440 NASCET and ECST trials advocate carotid endarterectomy when Harrison’s 18th Ed. 3275 A. 50 % D. 20 to 200 µm C. 3276 A. diabetes (1). 3277 Table 370-3 Harrison’s 18th Ed. 2 . Lateral ventricular wall Lacunes in Latin mean “lake”. 6 weeks B. 10 to 100 µm B. 3 mm to 20 mm C. Harrison’s 18th Ed. Recent transmural MI involving anteroapical ventricular wall is a source of cerebral emboli. 3276 carotid artery stenosis is more than ? A. Rheumatic heart disease with atrial fibrillation C. Stress D. Anteroapical ventricular wall B. 3 . 3279 436 CHADS2 score is calculating risk of stroke in ? A. 445 The principal risk factor for lacunar infarcts is ? 439 Most common source of artery-to-artery embolic stroke is ? Harrison’s 18th Ed. Hypertension B. Congenital anomaly Harrison’s 18th Ed. Age A. Mechanical heart value 443 In lacunar infarction. dilated cardiomyopathy. Cigarette smoking C. hypertension (1). 8 weeks C. All of the above C. Carotid bifurcation C. Nonrheumatic atrial fibrillation D. small artery (30 to 300 µm) in brain. Vertebral arteries Hypertension and age are the principal risk factors for lacunar infarcts. 30 to 300 µm D. D. Patent foramen ovale Lipohyalinosis appears as an eosinophilic deposit in the connective tissue of the vessel wall. what size of a small artery is occluded in 437 Maximum points in CHADS2 score is for ? brain ? Harrison’s 18th Ed. Diabetes mellitus B. 1 mm to 10 mm A. 1 mm to 5 mm Harrison’s 18th Ed. 3276.3 mm North American Symptomatic Carotid Endarterectomy Trial (NASCET) & European Carotid Surgery Trial (ECST) showed benefit for carotid endarterectomy when stenosis of >70%. Embolism A. Deficiet A. D. and stroke or TIA (2). 2 weeks Harrison’s 18th Ed.4 mm Emboli of the size of 3 . Stone D. Mitral valve prolapse D. mitral stenosis. 3275 A. mechanical valve. Mitral valve prolapse is not a source of emboli unless prolapse is severe. 90 % . Lake B. Recent MI Carotid endarterectomy is most beneficial when performed within 2 weeks of symptom onset. 3274 B. deep cerebral infarcts". River C. Inferior ventricular wall C. D. lacunae are "small. 3277 Table 370-3 B. Stroke or TIA D. Posterior ventricular wall D. 70 % Lacunar infarcts range in size from 3 mm to 20 mm. 2 mm to 10 mm B. Others are mural thrombus. 3276 Harrison’s 18th Ed. 441 Carotid endarterectomy is most beneficial within how many weeks of symptom onset ? 435 Which of the following is the most common cause of cerebral Harrison’s 18th Ed. D. Aortic arch B. 438 Which of the following location of recent transmural myocardial 444 “Lacune” in Latin means ? infarction is mostly the source of cerebral emboli ? Harrison’s 18th Ed. Lipohyalinosis B. AJAY MATHUR Neurology C.4 mm are large enough to occlude stem of MCA.678 MCQ’s FOR MEDICAL PROFESSIONALS BY PROF. Nonrheumatic atrial fibrillation is the most common cause of cerebral embolism. myocardial infarction. Common carotid D. Congestive heart failure C. Mitral valve prolapse Benefits more men than women and those >75 years. Hypertension A. 3282 B. 4 weeks A. Clinicaly. 40 to 400 µm CHADS2 score is calculated by summing of points for age >75 years (1). Carotid bifurcation atherosclerosis is the most common source of artery-to-artery embolus. 442 The underlying pathology in lacunar stroke is ? bacterial endocarditis. 3283 cardioembolism ? A. Sickle cell anemia D. Emissary veins venous sinuses is false ? D. and posterior part of the C. They also receive some of the diploic veins and some small veins from the middle ear. Superior sagittal sinus Cerebral veins contain 70% of the total cerebral blood volume. Envoy veins 457 Which of the following statements about thrombosis of cerebral C. Ataxic hemiparesis due to infarct in base of pons C. another classic source of septic thrombosis.4:2-17 Harrison’s 18th Ed. Thin walls B. no muscle fibers. Systemic malignancy B.4:2-17 C. Brainstem 458 The most common pattern of clinical presentations of CVT is ? Semin Cerebrovasc Dis Stroke 2004. thrombosis occurs in > one sinus Superior sagittal sinus (SSS) drains major part of the cortex. which explains the occurrence of SSS B.4:2-17 B. 90 % C. None of the above A. Petrosal sinuses C. Superior petrosal sinus B. especially cranial nerves III. 70 % B. Sphenoparietal sinus C. and no valves. thereby permitting both dilation & reversal of the direction of blood flow when the sinus into which they drain is occluded. brainstem. AJAY MATHUR Cardiology 679 447 Which of the following is not a lacunar syndrome ? 453 Which of the following is a feature of superficial cerebral veins or Harrison’s 18th Ed. cerebral vein. Polycythemia vera A. Messenger veins N Engl J Med 2005. Representative veins sinus. Dysarthria and clumsy hand due to infarction in medulla D. VI. No valves D. Isolated intracranial hypertension The lateral sinuses (LS) drain blood from the cerebellum. Factor V Leiden mutation D. D. Transverse sinus 451 Veins that connect superior sagittal sinus with the scalp veins are Because of the close relationship of the petrosal sinuses to cranial nerves.4:2-17 internal capsule A. Pure sensory stroke due to infarct in ventrolateral thalamus B. Superior and inferior ophthalmic veins B. Subacute diffuse encephalopathy cerebral hemispheres.4:2-17 mainly by a fifth nerve palsy for the superior petrosal sinus and by a sixth nerve palsy for the inferior A. All of the above B. VII. the genu of the internal capsule. Great vein of Galen D. thrombosis of petrosal called ? sinuses (with or without thrombosis of transverse sinus) may produce cranial nerve palsies.352:1791-8 A. 3278 Semin Cerebrovasc Dis Stroke 2004. Focal deficits D. Hypoplasia & atresia of transverse sinuses frequent thrombosis after scalp infection or minor head trauma. 50 % A. All of the above Cavernous sinuses drain the blood flow from the orbits through the superior & inferior ophthalmic Besides increasing risk of venous thrombosis. 30 % manifests as cranial nerve palsies ? Semin Cerebrovasc Dis Stroke 2004. Libman-Sacks endocarditis B. It also receives diploic veins. . V. Straight sinus C. Antiphospholipid syndrome A.4:2-17 D. 3278 A. A prothrombotic risk factor or a direct cause is identified in ~ 452 Lateral sinuses (LS) drain blood from which of the following ? 85 % of patients Semin Cerebrovasc Dis Stroke 2004. Middle cerebral vein D. Straight sinus D. and VIII. 449 Which of the following can be a cause of embolic stroke ? 455 The cavernous sinuses empty into which of the following ? Harrison’s 18th Ed. Thrombosis of the petrosal sinuses is characterized Semin Cerebrovasc Dis Stroke 2004. Protein S deficiency and homocysteinemia may veins. Superior sagittal sinus Systemic lupus erythematosus with Libman-Sacks endocarditis can be a cause of embolic stroke.4:2-17 456 Thrombosis of which of the following venous sinuses mostly A. Posterior part of cerebral hemispheres A. Pure motor hemiparesis due to infarct in posterior limb of the Semin Cerebrovasc Dis Stroke 2004. 679 Neurology MCQ’s FORMCQ’s FOR MEDICAL MEDICAL PROFESSIONALS PROFESSIONALS BY PROF. In most patients. 3276 cortical veins ? A. Cerebellum B.4:2-17 A. All of the above Lacunar syndrome dysarthria and a clumsy hand or arm due to infarction in the ventral pons or in The superficial cerebral veins or cortical veins have thin walls. Nephrotic syndrome C. No muscle fibers C. C. 448 Which of the following increases risk of venous thrombosis and arterial thrombosis ? 454 Which of the following drain into cavernous sinuses ? Semin Cerebrovasc Dis Stroke 2004. B. The cavernous sinuses empty into both the superior and the inferior petrosal sinuses and ultimately into the internal jugular veins. Homocysteinemia C. 450 Cerebral veins contain how much of the total cerebral blood volume ? Semin Cerebrovasc Dis Stroke 2004. and from the anterior part of the base of brain by the sphenoparietal sinus and the middle cause arterial thromboses as well. themselves connected to scalp veins by emissary veins. Severe headache B. Thrombophilia is diagnosed C.680 MCQ’s FOR MEDICAL PROFESSIONALS BY PROF. present in >90 % of adult patients. It is also seen C. Antiprothrombin G20210 mutation N Engl J Med 2005. deficiencies of proteins C and S. 460 Classic picture of deep cerebral venous thrombosis is ? Semin Cerebrovasc Dis Stroke 2004. Acetazolamide D. Acute coma. Thrombin G20210 mutation B. Ophthalmoplegia C. All of the above A. hemiparesis. Definitive treatment symptoms such as delirium. The same radiologic signs have been described on MRI. Corticosteroids should be avoided. MR-venography B. Dense triangle sign A. Conventional x-ray angiography The most common pattern of clinical presentations of CVT (~75%) is characterized by the presence CT imaging is normal in patients with venous sinus thrombosis unless an intracranial venous of focal signs such as focal deficits or partial seizures. the “empty delta sign” (filling of collateral veins in SSS wall after contrast injection. Focal deficits D. or mannitol. but the venous sinus occlusion is readily visualized using MR. Glycerol Classic picture of deep cerebral venous thrombosis is that of an acute coma with decerebration C. One year for sinus thrombosis. vitamin K antagonists are given for what length of time after a first episode of sinus thrombosis ? C. Intravenous thrombolysis. 3278 thrombotic event within one year.5. 3278 continued indefinitely if ? Harrison’s 18th Ed. palsies Semin Cerebrovasc Dis Stroke 2004. and dehydration. Thrombosis of lateral sinus B. with a target INR of 2. glycerol. Thrombosis of the deep venous system. Any neurological symptoms such as hemorrhage has occurred. inflammatory bowel disease. antithrombin III deficiency. Some patients with CVT carry more than one congenital thrombophilia factors. and 461 Venous sinus thrombosis pertains to which of the following ? surgical decompression are useful. 464 Which of the following is the brain CT finding in a case of superior 459 The most frequent but least specific symptom of sinus thrombosis sagittal sinus thrombosis ? is ? Semin Cerebrovasc Dis Stroke 2004. the straight sinus and its branches. Thrombosis of sagittal sinus 466 Recurrent sinus thrombosis within one year occurs in what percentage of patients ? C. with behavioral Intracranial hypertension is common in CVT and should be treated immediately. AJAY MATHUR Neurology D.or CT- aphasia. 3278 thrombogenic risk. it occurs as a complication of pregnancy and postpartum period. factor V Leiden mutation D.352:1791-8 D. and amnesia can occur with CVT. there is concern about their Harrison’s 18th Ed. They increase the risk of systemic infections and are of no proven value in CVT. Hypercoagulable state is found B. Prothrombin G20210 mutation 467 Usually. and mutism. Thalamic edema is the imaging hallmark. CT A. asymptomatic. 2 % Thrombosis of lateral or sagittal sinus or of small cortical veins fall under the category of venous B. Normal brain CT scan does not rule out CVT. amnesia. or longer in the presence of predisposing factors. Intracranial hypertension hypertension management in CVT ? B. contrasting with the nonenhancement of clot inside the thrombosed sinus). All of the above D. Partial seizures B. Partial seizures Brain CT findings in CVT include the “dense triangle sign” (occlusion of SSS by fresh clot on The most frequent but least specific symptom of sinus thrombosis is severe headache. which can be the only manifestation of sinus is establishing reperfusion of the venous structures (Heparin. which is noncontrast CT). Six months Women who take oral contraceptives and have the prothrombin G20210 mutation are at a high risk B. intracranial infections (meningitis). 463 Which of the following investigation is normal in patients with 468 Anticoagulation in patients with venous sinus thrombosis is often venous sinus thrombosis ? Harrison’s 18th Ed. hemianopsia.4:2-17 C. sickle cell anemia. 3278 A. CVT can be totally venography or conventional x-ray angiography. Acute painful ophthalmoplegia D. and the “cord sign” (visualization of a thrombosed cortical vein on noncontrast CT). Empty delta sign B. Patient has a h/o coma . Good pain control and thermal regulation.4:2-17 465 Use of which of the following has no proven value in intracranial A. A. 4 % sinus thrombosis. Life long (resistance to activated protein C). acetazolamide. Cord sign C. surgical ventricular drainage. decerebration & extrapyramidal hypertonia A. C. and homocysteinemia. Five years with increased incidence in patients with laboratory-confirmed thrombophilia including polycythemia. Progressive cerebellar incoordination & cranial N.352:1791-8 A. A. Corticosteroids mutism and dementia when resolving. Elevating head of the bed by 30° improves venous drainage. Mannitol and extrapyramidal hypertonia leading to death within a few days. Antithrombin G20210 mutation A. vitamin coagulation work-up should be systematically performed in patients of CVT.4:2-17 N Engl J Med 2005.352:1791-8 D. Extensive etiologic and complete Optimal duration of oral anticoagulant treatment after the acute phase is unknown. Thrombosis of small cortical veins N Engl J Med 2005. often bilateral thalamic lesions. causes centrally located. 8 % sinus thrombosis in women who take oral contraceptives ? Recurrent sinus thrombosis occurs in 2 % of patients and ~4 % of patients have an extracranial Harrison’s 18th Ed. K antagonists are given for six months after a first episode of sinus thrombosis. Also. Moreover. thrombosis. CT-venography C. thrombectomy). 6 % 462 Which of the following mutation carries a high risk of venous D. or complicated by akinetic D. Usually. None of the above Homunculus means ‘little man’ in Latin. In somatotropic representations. 472 Which of the following is a common cause of stroke in children ? 479 Hypothesis of homunculus organization of fibers in internal capsule Harrison’s 18th Ed. B. Fibromuscular dysplasia C. Adults Lacunar infarcts mainly occur in basal ganglia. 3278 A. & centrum semiovale. Anterior cerebral artery Treatment with anticoagulant drugs leads to a moderate benefit for patients with CVST compared D. Adjacent regions on cortex correspond on adjacent areas on D. Superior thalamic radiation 474 Which of the following is false about Temporal (giant cell) arteritis ? C. Sickle cell anemia body surface Sickle cell anemia (SS disease) is a common cause of stroke in children.108 A. If an underlying hypercoagulable state is not found. but mortality after treatment with anticoagulant drugs is still 5% to 10%. 15 % to 20 % C. Aneurysms of brain blood vessels of <300 µm diameter A. lacunar infarcts are most frequent in ? 470 Sickle cell anemia (SS disease) is a common cause of stroke in ? A. White matter of internal capsule A. Kawasaki disease cerebral venous sinus thrombosis (CVST) is ? D. Rarely causes stroke A. Corpus callosum Harrison’s 18th Ed. Transcranial Doppler ultrasonography Charcot-Bouchard miliary aneurysms (French physicians Jean-Martin Charcot and Charles-Joseph Bouchard) are aneurysms of brain small blood vessels (<300 µm diameter). regardless of the presence of intracranial Harrison’s 16th Ed. Cerebellum C. putamen. Associated with chronic hypertension C.70:105 . thalamus and white matter of internal D. cerebral gyri & spinal cord & are rare in gray matter of cerebral surface.6 months then switch over to aspirin. None of the above D. Cortico-pontine D. 681 Neurology MCQ’s FORMCQ’s FOR MEDICAL MEDICAL PROFESSIONALS PROFESSIONALS BY PROF. Hypercoagulable disorders B. Involvement of renal arteries is common A. 3278 means ? A. Sensory & motor fibers have same orientation B. Temporal arteritis if thrombophilia is diagnosed. Polyarteritis nodosa J Neurol Neurosurg Psychiatry 2001. 3278 B. MR-venography B. Functional mapping & lesion studies have shown that 473 Which of the following about fibromuscular dysplasia is false ? primary motor & somatosensory cortices are so arranged that adjacent regions on cortex correspond Harrison’s 18th Ed. Foramen of Magendie . pons. Old capsule. Internal carotid artery C. 3278 on adjacent areas on body surface. 3278 D. vitamin K antagonists (VKAs) are given for 3 . They occasionally occur in cerebellum. Patient has a h/o seizure 475 Which is the most common form of vasculitis that affects the carotid artery ? Intravenous heparin reduces morbidity & mortality. Children D. CT-venography C. AJAY MATHUR Cardiology 681 D. While. 2379 hemorrhage. Middle cerebral artery D. Affects carotid or vertebral arteries B. 478 Which of the following is false about Charcot-Bouchard miliary 471 Which of the following investigation is useful in predicting stroke aneurysm ? in childhood in SS disease carriers ? Harrison’s 18th Ed. Infants C. Occurs mainly in women 480 Which of the following tracts is not present in the posterior limb of internal capsule ? C. 2 % to 5 % 476 Thalamoperforator branches arise from ? B. corpus callosum & visual radiations. Anticoagulation is often continued indefinitely A. Foremen of Lushka Temporal (giant cell) arteritis rarely causes stroke as internal carotid artery is usually not inflamed. Corticospinal tract A. Affects elderly persons Anterior thalamic radiation is part of anterior limb of internal capsule. Charcot-Bouchard aneurysms can predict the chances of stroke in childhood in SS disease carriers. D. 477 Out of the following. Posterior cerebral artery with placebo treatment. 10 % to 15 % B. saccular Documenting high-velocity blood flow within MCAs using transcranial Doppler ultrasonography aneurysms (berry aneurysms) occur in larger-sized blood vessels. Gray matter of cerebral surface B. Temporal (giant cell) arteritis A. Conventional x-ray angiography D. B. May cause blindness 481 Genu of the internal capsule is at the level of ? D. Located in brainstem B. Sensory & motor fibers have different orientation C. Anterior thalamic radiation Harrison’s 18th Ed. 5 % to 10 % A. None of the above B. B. Takayasu’s arteritis 469 Mortality after treatment with anticoagulant drugs in patients of C. are most often located in the brainstem and are associated with chronic hypertension. arms are always medial to legs except in primary sensiromotor cortex & in posterior columns. Sickle cell anemia (SS disease) is a common cause of stroke in children. Affects internal & external carotid system C. A. 3309 C. Fabry disease. hereditary endotheliopathy with retinopathy. They are cerebral autosomal. and extensive B. None of the above 487 Which of the following is categorized under monogenic disorders Binswanger’s disease is also called chronic progressive subcortical encephalopathy. Foramen of Monro autosomal-recessive arteriosclerosis with subcortical infarcts and leukoencephalopathy (CARASIL). 482 Hippocampus-fornix-anterior thalamus-cingulate gyrus-cingulum bundle-perihippocampal cortex-hippocampus . 3279 called ? A. Notch-3 A. occlusion of ? Harrison’s 18th Ed. Genetic testing is available. Moyamoya disease D. CADASIL family characterized by epidermal growth factor repeats in its extracellular domain. nephropathy. familial MoyaMoya disease. 490 Which of the following is a monogenic ischemic stroke syndrome ? Harrison’s 18th Ed. Notch-1 leukoencephalopathy is called ? B. Hafez’s circuit B. retinopathy. All of the above Harrison’s 18th Ed. and stroke (HERNS).this circuit is 488 Which of the following is false about CADASIL ? Harrison’s 18th Ed. CADASIL C. Binswanger’s disease B. Moyamoya disease Stroke due to monogenic disorders represent <1% of all stroke patients. 2379 D. Cerebral autosomal dominant arteriopathy. None of the above D. D. Foramen magnum pseudoxanthoma elasticum. Presents as small-vessel strokes. Distal internal carotid artery B. Peter’s ciircuit D. because it does not clearly identify a single entity. All of the above Moyamoya disease is an occlusive disease involving large intracranial arteries. Notch-2 Harrison’s 18th Ed. progressive dementia. It is one of the most common causes of dementia. Notch-4 B. All of the above A. CARASIL symmetric white matter changes in MRI is suggestive of which of the following ? C. 3279 489 Which of the following is a monogenic ischemic stroke syndrome ? Harrison’s 18th Ed. Ehlers-Danlos syndrome type IV. 3279 A. Distal vertebral artery A. Binswanger’s disease dominant arteriopathy with subcortical infarcts and leukoencephalopathy (CADASIL). HERNS Harrison’s 16th Ed. Moderate (MELAS) D. HERNS D. Autosomal recessive condition C. vascular inflammation is ? Harrison’s 18th Ed. Distal Basilar artery B. 485 In Moyamoya disease.682 MCQ’s FOR MEDICAL PROFESSIONALS BY PROF. 3280 A. Lenticulostriate arteries (anterior circulation) develop a rich collateral circulation around the occlusive Other monogenic ischemic stroke syndromes include cerebral autosomal recessive arteriopathy lesion. 3280 A. Acute disseminated encephalomyelitis (ADEM) B. 3279 491 Which of the following disorders can mimic Multiple Sclerosis ? Harrison’s 18th Ed. CADASIL D. Mitochondrial encephalopathy with lactic acidosis and stroke C. Needle CADASIL (cerebral autosomal dominant arteriopathy with subcortical infarcts and leukoencephalopathy) is a monogenic ischemic stroke syndrome. progressive dementia D. Parkinson’s circuit C. Pin head C. Papez’s circuit A. CADASIL 493 Small-vessel strokes. Notch-3 is a member of a highly conserved gene C. AJAY MATHUR Neurology C. Harrison’s 18th Ed. with subcortical infarcts and leukoencephalopathy (CARASIL) and hereditary endotheliopathy. B. Severe C. CARASIL C. Cotton ball B. Mild B. vascular inflammation is absent. which on conventional x-ray angiography appears like a “puff of smoke” (moyamoya in Japanese). None of the above C. 3402 Table 380-4 A. D. Extensive symmetric white matter changes in MRI CADASIL (cerebral autosomal dominant arteriopathy with subcortical infarcts and 483 Moyamoya disease results from progressive stenosis and leukoencephalopathy) is an autosomal dominant inherited disorder. 3280 484 ‘Moyamoya’ in Japanese language means ? Harrison’s 18th Ed. and stroke (HERNS). nephropathy. and leukoencephalopathy (CADASIL) 486 CADASIL is due to mutations in which gene ? D. Moyamoya disease A. Marfan syndrome. Onset is in IV or V decade of life B. especially distal internal carotid artery and stem of middle and anterior cerebral arteries. cerebral . subcortical In Moyamoya disease. infarcts. Neurofibromatosis type 1. Absent A. The term Binswanger’s disease should be used with as a cause of stroke ? caution. Binswanger’s disease CADASIL is caused by mutations in Notch-3 gene. 3278 492 Chronic progressive subcortical arteriosclerotic A. Puff of smoke D. Aspirin 501 Which of the following is the principal side effect of dipyridamole ? B. Skin rash 496 Which of the following is false about Stroke Prevention by Aggressive Reduction in Cholesterol Levels (SPARCL) trial ? D. Aspirin Harrison’s 18th Ed. A. anticoagulation with B. 3281 CADASIL (cerebral autosomal dominant arteriopathy with subcortical infarcts and A. Ticlopidine C. Calcium channel blockers and PRoFESS trials ? C. Statins reduce it by 16-30%. may be beneficial in acute period. 80 mg/day. Thiazide diuretics 500 Which of the following drug was studied in the ESPS II. MATCH D. 3282 D. A. 3281 D. . Clopidogrel B. 3281 Dipyridamole with aspirin combination drug was studied in three trials. Aspirin D. ~ 67 % In chronic nonvalvular. 3281 C. Angiotensin receptor blockers A. Dipyridamole & Aspirin are thiazide diuretics. Diarrhea D. 3282 B. Dipyridamole decade of life. it has been abandoned because of its adverse effects like diarrhea. Neutropenia Harrison’s 18th Ed. Done on patients with recent stroke or TIA B. It is caused by one of several mutations in Notch-3 gene. 3281 504 In ACTIVE-A trial. 3281 The principal side effect of dipyridamole is headache. It found that patients with low LDL (<130 mg/dL) caused C. But. 25 % 497 Which of the following is a primary prevention trial advocating use of statins to reduce the risk of stroke ? Antiplatelet agents produce an overall relative reduction in risk of nonfatal stroke of about 25-30% and of all vascular events of about 25%. Ticlopidine leukoencephalopathy) is an inherited disorder that presents as small-vessel strokes. Clopidogrel & Aspirin D. Ticlopidine Though Ticlopidine is more effective than aspirin.VKA) 498 Which of the following platelet antiaggregation agents is least reduces the risk of cerebral embolism by ~67% both for primary & secondary prevention of stroke preferred to prevent atherothrombotic events ? or TIA. Clopidogrel A. None of the above A. In diabetics. which of the following drugs was not used ? A. skin rash. Drugs of value B. 5 % SPARCL (Stroke Prevention by Aggressive Reduction in Cholesterol Levels) trial showed benefit in B. Harrison’s 18th Ed. ESPRIT B. Ticlopidine & Aspirin Lowering BP to normotensive levels reduces risk of stroke in hypertensives with TIA. ~ 17 % D. Dipyridamole B. nonrheumatic AF. Headache C. ~ 27 % Justification for the Use of Statins in Prevention: An Intervention Trial Evaluating Rosuvastatin (JUPITER) was a primary prevention trial. and extensive symmetric white matter changes visualized by MRI in the fourth or fifth B. CAPRIE warfarin reduces risk of cerebral embolism by ? Harrison’s 18th Ed. Statins the risk of stroke in hypertensives with TIA ? These three trials concluded that use of clopidogrel in combination with aspirin is not recommended Harrison’s 18th Ed. Clopidogrel & Aspirin 494 Out of the following. 3281 A. 3282 C. JUPITER A. which drug is of least importance in reducing D. ACE inhibitors & ARBs. MATCH B. Primary stroke occurrence was reduced by 51% with no increase in rates of intracranial hemorrhage. ~ 47 % by elevated C-reactive protein benefitted by daily use of rosuvastatin. Dose of atorvastatin used was 80 mg/day Harrison’s 18th Ed. A. Pioglitazone Harrison’s 18th Ed. pioglitazone is effective. nonrheumatic atrial fibrillation. Clopidogrel C. 683 Neurology MCQ’s FORMCQ’s FOR MEDICAL MEDICAL PROFESSIONALS PROFESSIONALS BY PROF. C. AJAY MATHUR Cardiology 683 C. D. Statins & Aspirin 495 Which of the following is related to the trial ‘SPARCL’ ? Harrison’s 18th Ed. Angiotensin-converting enzyme inhibitors Harrison’s 18th Ed. Secondary stroke reduction 502 What is the overall ‘relative’ reduction in risk of all vascular events with antiplatelet agents ? C. neutropenia and thrombotic thrombocytopenic purpura (TTP). C. SPARCL 503 In chronic nonvalvular. progressive dementia. None of the above and CHARISMA trials ? Harrison’s 18th Ed. warfarin (a vitamin K antagonist . The risk of a major bleeding complication is 1–3% per year. 3281 for stroke prevention. CADASIL 499 Which of the following drug was studied in the CAPRIE. Harrison’s 18th Ed. 10 % secondary stroke reduction for patients with recent stroke or TIA who were prescribed atorvastatin. C. Atorvastatin A. 18 % D. Outer globus pallidus C. 3284 B. 3284 D. Meta-analysis of NASCET and ECST trials showed that endarterectomy is most Cortical branches of MCA supply the lateral surface of hemisphere except for frontal pole and a beneficial when performed within 2 weeks of symptom onset. CREST.684 MCQ’s FOR MEDICAL PROFESSIONALS BY PROF. homonymous hemianopia occur in both instances. Branches of the 508 SAPPHIRE. B. C. Occipital pole convolution intracranial atherosclerosis. large-vessel B. ECST 511 Cortical branches of MCA supply which of the following ? C. A. 3284 In Atrial Fibrillation Clopidogrel Trial with Irbesartan for Prevention of Vascular Events (ACTIVE- A) trial. ~ 12 % B. A. Use of atherothrombotics in TIA/stroke except ? C. ECST found harm for patients with stenosis <30% treated surgically. Irbesartan 510 Occlusion of proximal MCA is most often due to ? Harrison’s 18th Ed. Putamen B. Anticoagulation in stroke 514 In a right handed person. 3283 Harrison’s 18th Ed. Intracranial atherothrombosis 505 Which of the following trails does not address carotid C. in patients >75 years. ACAS cardiac) rather than intracranial atherothrombosis. Anosognosia Stenting and Angioplasty with Protection in Patients at High Risk for Endarterectomy (SAPPHIRE) trial was done in high risk patients with symptomatic carotid stenosis >50% or asymptomatic C. A. Anterior limb of internal capsule D. Stem of MCA Stroke syndromes are divided into large vessel stroke within anterior circulation. 3284 of facial weakness. Small-vessel disease of either vascular bed Harrison’s 18th Ed. patients who cannot take anticoagulants. Warfarin-Aspirin Symptomatic Intracranial Disease (WASID) study demonstrated no benefit of warfarin over aspirin in patients with symptomatic C. 507 What is the annual stroke rate in the natural history of symptomatic outer globus pallidus. North American Symptomatic Carotid Endarterectomy Trial (NASCET) and the European Carotid Surgery Trial (ECST) showed a substantial benefit for surgery in patients D. Superior division of MCA D. Proximal superior division of MCA . Extracranial-to-intracranial (EC-IC) bypass surgery A. and neglect are found. B. ~ 13 % C. None of the above Harrison’s 18th Ed. posterior limb of internal capsule. ~2% B. occlusion of right MCA leads to all B. NASCET A. Dysarthria Stenting (ICSS) trial also studied symptomatic patients treated with stents or endarterectomy. Large vessel stroke within anterior circulation 515 Which of the following artery supplying dominant hemisphere is B. All of the above A. Body of caudate nucleus Proximal MCA (M1 segment) gives rise to penetrating lenticulostriate arteries that supply putamen. anosognosia. temporal and occipital pole convolutions (supplied by PCA). clopidogrel plus aspirin was compared to aspirin A. D. Harrison’s 18th Ed. ~ 10 % 513 Temporal cortex is supplied by ? Harrison’s 18th Ed. ~ 11 % A. Lower temporal pole convolution over aspirin for secondary prevention of stroke. and lower appear to benefit more than women. Penetrating lenticulostriate arteries of MCA stroke within posterior circulation and small-vessel disease of either vascular bed. Global aphasia occurs when the dominant hemisphere is involved. AJAY MATHUR Neurology D. 3284 D. Constructional apraxia stenosis >80% by either stenting combined with a distal emboli-protection device or endarterectomy. and ICSS pertain to which of the following ? inferior division supply inferior parietal and temporal cortex. Global aphasia B. Frontal pole Warfarin-Aspirin Reinfarction Stroke Study (WARSS) study found no benefit of warfarin sodium B. ACA The natural history of asymptomatic carotid stenosis is ~2% per year stroke rate. WARSS Harrison’s 18th Ed. adjacent corona radiata & most of caudate carotid stenosis ? nucleus. 3284 A. Large-vessel stroke within posterior circulation occluded in Wernicke’s aphasia without weakness ? C. and superior division supply the Harrison’s 18th Ed. None of the above with a symptomatic carotid stenosis of >=70%. PCA MCA divides into superior & inferior divisions (M2 branches) in sylvian fissure. Inferior division of MCA C. Carotid artery stenting and angioplasty Harrison’s 18th Ed. When nondominant hemisphere is affected. ~1% A. while symptomatic patients experience a 13% per year risk of stroke. 3283 A. Contralateral hemiplegia. 3284 D. constructional apraxia. ~3% C. Carotid Revascularization Endarterectomy versus Stenting Trial (CREST) and International Carotid D. 506 What is the annual stroke rate in the natural history of asymptomatic 512 Lenticulostriate arteries from proximal MCA (M1 segment) supply carotid stenosis ? all except ? Harrison’s 18th Ed. 3283 frontal and superior parietal cortex. Embolus alone. ~4% D. Dysarthria is common because Harrison’s 18th Ed. 509 Which of the following is a category of stroke syndromes ? hemianesthesia. and men strip along the superomedial border of frontal and parietal lobes (supplied by ACA). 3282 Occlusion of proximal MCA or one of its major branches is mostly due to embolus (artery-to-artery. Similarly. Vasospasm endarterectomy ? D. B. Contralateral hemiplegia A. B. Vasospasm Johann Otto Leonhardt Heubner (1843-1926) described this artery in 1872. . Posterior cerebral artery artery at the level of ? A. Homonymous hemianopia C. 3285 Occlusion of a lenticulostriate vessel of MCA produces lacunar stroke within the internal capsule A. during surgical clipping of aneurysms of ICA. Genu C. common anterior cerebral trunk & azygos pericallosal artery. Internal carotid artery posterior part (temporal cortex) of dominant hemisphere is probably involved. and inferior part of the head of caudate nucleus. 3284 term arteria termatica of Wilder in 1885. Posterior limb of internal capsule C. Genu of internal capsule Occlusion of a single A2 segment of ACA results in profound abulia. B. inferior division of MCA supplying A. Inferior division of MCA 522 “Arteria termatica of Wilder” is a branch of ? If a fluent (Wernicke’s) aphasia occurs without weakness. anterior hypothalamus. Globus pallidus and putamen 525 Anterior choroidal artery arises from ? Harrison’s 18th Ed. Retrolentiform part The complete syndrome of anterior choroidal artery occlusion consists of contralateral hemiplegia. Olfactory trigone Harrison’s 18th Ed. 3285 520 “Recurrent artery of Heubner” supplies which part of internal capsule ? A. Hemianesthesia B. and iatrogenic occlusion improper clip placement during repair of aneurysms near the anterior communicating artery. Body of caudate nucleus 517 Which of the following artery is occluded in a case of pure motor stroke contralateral to the lesion ? D. 3284 The A1 segment of anterior cerebral artery (ACA) gives rise to deep penetrating branches that A. Also known as the unpaired pericallosal stem artery. Internal carotid artery Lacunar infarction affecting the globus pallidus and putamen often produce few clinical signs. anterior perforate substance. Middle cerebral artery B. B. All of the above D. Anterior cerebral artery 519 Anterior cerebral artery (ACA) arises from the Internal carotid D. but parkinsonism and hemiballismus occur. hemianesthesia and homonymous hemianopia. Posterior cerebral artery C. Internal carotid artery 527 Anterior choroidal strokes are usually the result of ? Harrison’s 18th Ed. AJAY MATHUR Cardiology 685 D. 3285 B. Anterior cerebral artery A. 521 “Recurrent artery of Heubner” is a branch of ? A. Embolus C. B. B. Urinary incontinence very few clinical signs ? D. through which geniculocalcarine fibers pass partly. Optic chiasma C. All of the above A. 3284 A. Amygdala C. bilateral pyramidal signs with paraparesis and urinary incontinence. Anterior cerebral artery 516 Which of the following artery supplying the nondominant C. B. Bilateral pyramidal signs with paraparesis 518 Lacunar infarction affecting which of the following areas produces C. 3285 D. amygdala. Inferior division of MCA A. Abulia producing pure motor stroke or sensory-motor stroke contralateral to the lesion. Proximal superior division of MCA branches of the A1 segment of ACA ? Harrison’s 18th Ed. Lenticulostriate Recurrent artery of Heubner (RAH) D. Anterior hypothalamus Harrison’s 18th Ed. Anterior limb B. Proximal superior division of MCA 524 Occlusion of a single A2 segment of anterior cerebral artery (ACA) results in which of the following ? D. B. Posterior limb D. Inferior division of MCA Harrison’s 18th Ed. Posterior cerebral artery without weakness ? Wilder was the first to describe fusion of both A2 segments to form a single artery & introduced the Harrison’s 18th Ed. Thrombosis D. He also described syphilitic endarteritis (Heubner's disease). Middle cerebral artery C. Stem of MCA supply the anterior limb of internal capsule. None of the above supplies parts of basal ganglia and anterior limb of internal capsule and can be damaged with Anterior choroidal strokes are usually the result of in situ thrombosis. 3285 D. Penetrating lenticulostriate arteries of MCA C. Anterior limb of internal capsule Hemineglect or spatial agnosia without weakness indicates that the inferior division of MCA in the nondominant hemisphere is occluded. 685 Neurology MCQ’s FORMCQ’s FOR MEDICAL MEDICAL PROFESSIONALS PROFESSIONALS BY PROF. Middle cerebral artery hemisphere is occluded in a case of hemineglect or spatial agnosia D. Penetrating lenticulostriate arteries of MCA 523 Which of the following is not supplied by the deep penetrating C. Optic nerve 526 Complete syndrome of anterior choroidal artery occlusion consists of ? D. Stem of MCA unpaired anterior cerebral artery. All of the above Harrison’s 18th Ed. Anterior clinoid process Anterior choroidal artery arises from ICA & supplies posterior limb of internal capsule & white matter posterolateral to it. A. the basilar artery. Interpeduncular fossa C. Hemiplegia is localized to the cerebral peduncle. Pontomedullary junction A. and medial temporal and occipital lobes. 3286 of red nucleus or dentatorubrothalamic tract. Hermann Weber. pupillary dilatation. 541 Fascicles from III CN nuclei run forward & laterally through ? 534 Occlusion of artery of Percheron produces ? A. Pontomedullary junction B. Basilar artery A. C. 3286 A. Medial temporal lobes 539 Complete oculomotor nerve lesion results in all except ? C. PCA arising from MCA 535 Weber’s syndrome results due to occlusion of ? Harrison’s 18th Ed. A. 3287 B. A. All of the above 531 Basilar artery divides into 2 posterior cerebral arteries at ? III nerve palsy with contralateral ataxia is called Claude’s syndrome. 3287 Anatomic configuration of PCA arising from ICA is called fetal posterior cerebral artery (20%). Abulia A. Cerebral peduncle Basilar artery divides into two posterior cerebral arteries in the interpeduncular fossa. AJAY MATHUR Neurology 528 “Fetal posterior cerebral artery” refers to ? B. Vertebral artery B. C. Harrison’s 18th Ed. Temporal arteritis B. Lower mid-brain D. midbrain. Absent PCA Occlusion of artery of Percheron produces paresis of upward gaze. medulla. Takayasu’s arteritis 536 Claude’s syndrome results due to occlusion of ? Harrison’s 18th Ed. 3287 C. Loss of pupillary & accommodation reflexes paired posterior cerebral arteries. Foramen magnum 537 In Claude’s syndrome. loss of 533 The artery of Percheron arises from ? pupillary and accommodation reflexes and ‘lateral’ deviation of the eye. ataxia is due to involvement of ? Harrison’s 18th Ed. Interpeduncular fossa B. Posterior cerebral artery 530 The vertebral arteries join to form the basilar artery at ? D. Internal capsule D. Drowsiness Harrison’s 18th Ed. Upper mid-brain C. Lower medulla oblongata A. Pons Thalamogeniculate. Occipital lobes A. subthalamus. Substantia nigra Harrison’s 18th Ed. Kawasaki disease A. Corpus callosum D. Cerebellum C. Middle cerebral artery Bilateral common carotid artery occlusions at their origin may occur in Takayasu’s arteritis. and the C. All of the above B. Medulla oblongata proximal P1 segment of PCA. III CN supplies all extra- ocular muscles except lateral rectus and superior oblique. Anterior cerebral artery 529 Bilateral common carotid artery occlusions at their origin may B. Red nucleus Vertebral arteries join to form basilar artery at the pontomedullary junction. German physician. A. Basilar artery Harrison’s 18th Ed. 3286 C. Medical deviation of the eye thalamus. Foramen magnum 538 In Weber’s syndrome. Hippocampus B. pons. hemiplegia is due to involvement of ? B. 3287 B. D. D. Single PCA D. Pyramidal decussation 532 Posterior circulation supplies which of the following ? Harrison’s 18th Ed. Posterior cerebral artery A. Basilar artery C. Red nuclei A. Complete oculomotor (III) nerve lesion results in ipsilateral ptosis. C. Posterior cerebral artery Harrison’s 18th Ed. Percheron and posterior choroidal arteries are the penetrating branches of the D. Polyarteritis nodosa B.686 MCQ’s FOR MEDICAL PROFESSIONALS BY PROF. 3287 C. Ipsilateral ptosis D. 3287 A. 1863. Superior cerebellar peduncle D. 3286 III nerve palsy with contralateral hemiplegia is called Weber’s syndrome. Pupillary dilatation The posterior circulation is composed of the paired vertebral arteries. hippocampus. Anterior cerebral artery 540 Edinger-Westphal nucleus located in ? B. 3286 D. Ataxia is due to involvement Harrison’s 18th Ed. Middle cerebral artery occur in ? C. Anterior cerebral artery D. Paresis of upward gaze . drowsiness & abulia. It supplies the cerebellum. Lower medulla oblongata Harrison’s 18th Ed. All of the above B. PCA arising from internal carotid artery (ICA) D. Palinopia Harrison’s 18th Ed. patient is unaware of blindness. Ocular ataxia C. 687 Neurology MCQ’s FORMCQ’s FOR MEDICAL MEDICAL PROFESSIONALS PROFESSIONALS BY PROF. It runs forward parallel to the posterior communicating artery and enters the orbit through the A. Simultagnosia 544 Which of the following is false about “Thalamic Dejerine-Roussy syndrome” ? D. 3288 D. whereas lesions involving both upper & lower parts of mid-brain are associated with pupillary dilatation. Blindness A. Peduncular hallucinosis 551 Branches from which segment of vertebral artery supply brainstem B. B. The upper branch supplies the levator palpebrae superioris and superior rectus muscles. patients experience persistence of a visual image for several minutes despite gazing A. 3287 After emerging from mid-brain. 1 All the above presentations occur due to occlusion of distal PCA causing infarction of hippocampus. 3288 D. C4 545 Occlusion of the posterior cerebral artery can produce which of D. searing or burning pain in the affected areas. Asimultanagnosia oblique muscle conveys the preganglionic parasympathetic fibers to the ciliary ganglion from which the post-ganglionic parasympathetic fibers arise to supply the ciliary muscle and the muscles D. Acute disturbance in memory Harrison’s 18th Ed. and the inferior oblique muscle. Lesion of Harrison’s 18th Ed. III cranial nerve nuclei are located in midbrain. Bilateral occlusion of distal PCA magnum ? Harrison’s 18th Ed. Olivary nuclei Anton’s syndrome is due to bilateral infarction in the distal PCAs. B. Contralateral hemisensory loss at another scene. None of the above A. B. 3 D. Subthalamic nucleus A. Substantia nigra B. The nerve to the inferior C. Smell B. 3287 (palinopsia) or an inability to synthesize whole of an image (asimultanagnosia). Because memory has bilateral representation. Bilateral infarction in distal PCAs produces cortical blindness (blindness with preserved pupillary light reaction). AJAY MATHUR Cardiology 687 C. B. C. Dentate nuclei D. 3287 lower mid-brain affects extraocular muscles but spares pupils. C. C7 Thalamic Dejerine-Roussy syndrome consists of contralateral hemisensory loss followed later by agonizing. contralateral hemiballismus may occur. this defect clears soon. Balint’s syndrome is a disorder of the orderly visual scanning of the environment. 3287 D. Contralateral homonymous hemianopia. It produces cortical blindness with preserved pupillary light reaction. A. Hearing 542 Lower branch of occulomotor nerve supplies all of the following extraocular muscles except ? C. Medial rectus D. Inferior oblique 548 Which symptom can occur in Balint’s syndrome ? Harrison’s 18th Ed. the inferior rectus. Oculomotor apaxia the medial rectus. Red nucleus 549 Experiencing persistence of a visual image for several minutes despite gazing at another scene is called ? C. 3288 . A. Thalamus Harrison’s 18th Ed. Bilateral occlusion of vertebral artery D. All of the above of the iris (pupilloconstrictor fibers). Peduncular hallucinosis means visual hallucinations of brightly colored scenes & objects. Fascicles from the III CN nuclei 547 In Anton’s syndrome. Responds poorly to analgesics foramen at ? Harrison’s 18th Ed. C. 2 medial temporal & occipital lobes. Optic ataxia If the subthalamic nucleus is involved. dividing into an upper and a lower branch. usually resulting 543 Hemiballismus is due to damage to which of the following ? from infarctions secondary to low flow in “watershed” between distal PCA & MCA territories. Colour vision B. All of the above A. The lower branch supplies three muscles: B. May respond to carbamazepine or gabapentin. A. Optic ataxia superior orbital fissure. C3 the following ? The first (V1) segment of vertebral artery extends from its origin to its entrance into the sixth or fifth Harrison’s 18th Ed. III CN lies between posterior cerebral artery and the superior cerebellar artery. except lateral rectus and superior oblique. 3287 transverse vertebral foramen. 3287 Fourth (V4) segment of vertebral artery courses upward from foramen magnum to join the other vertebral artery and forms the basilar artery. Agonizing pain in affected area later 550 The vertebral artery enters into the transverse cervical vertebral C. Bilateral occlusion of distal MCA 552 Which segment of the vertebral artery pierce the dura at the foramen C. macular sparing and cerebellum ? C. They consist of Edinger-Westphal nucleus located in upper mid-brain supplying fibers to pupils and motor nucleus located in lower mid-brain supplying extra-ocular muscles. 3287 In palinopia. Bilateral occlusion of distal ACA supply the brainstem and cerebellum. C6 poorly to analgesics. Inferior rectus D. There occurs persistence of a visual image for several minutes despite gazing at another scene Harrison’s 18th Ed. Symptoms are persistent and respond B. Only the fourth segment gives rise to branches that A. Levator palpebrae superioris In Anton’s syndrome. 4 546 Anton’s syndrome results due to occlusion of ? Harrison’s 18th Ed. patient is often unaware of ? run forward & laterally through red nuclei and converge at the inter-peduncular fossa. paralysis of arm & leg. Hemiparesis Due to damage to descending tract and nucleus of fifth nerve. paralysis of palate & vocal cord. or “subclavian steal. Hoarseness D. Nucleus and tractus solitarius disease rarely narrows second and third segments of vertebral artery. Dysphagia. superior. 4 B. diplopia. vertigo. vertigo. On the D. 3289 Figure 370-10 Harrison’s 18th Ed. Loss of taste in lateral medullary syndrome is due to lesion in nucleus & tractus solitarius on side of 554 “Subclavian steal” is best related to ? lesion. 3 A. ptosis.688 MCQ’s FOR MEDICAL PROFESSIONALS BY PROF. middle or inferior lateral medullary arteries. fibromuscular dysplasia. Diplopia C. Contralateral pyramidal tract A. 1 558 Which of the following is false for lateral medullary syndrome ? Harrison’s 18th Ed. 3288 diminished gag reflex occur due to damage to issuing fibers of IX & X nerves. Falling to side of lesion A. decreased sweating) occurs on side of lesion in lateral They are vertebral. encroachment by osteophytic spurs within the D. A. Numbness of ipsilateral face C. Vestibular nucleus direction of blood flow in the ipsilateral vertebral artery. Impaired touch over opposite half of face C. Harrison’s 18th Ed. Raynauds phenomenon lateral medullary syndrome is due to lesion in ? B. Contralateral medial lemniscus B. which of the following structures to lesion in Vestibular nucleus. 3289 Figure 370-10 A. Upper limb clubbing A. vomiting in A. Superior lateral medullary artery C. Numbness of ipsilateral arm. Horner’s syndrome (miosis. Numbness of contralateral limbs D. Descending sympathetic tract D. Vertigo B. Paralysis with atrophy of half tongue due to ipsilateral XII nerve occurs. nausea.” D. All of the above C. and. All of the above D. Vestibular nucleus D. Atheromatous C. Consists of four segments Harrison’s 18th Ed. 3289 Figure 370-10 B. 3288 560 Nystagmus. Oscillopsia D. Cuneate and gracile nuclei opposite side. oscillopsia. or leg in lateral medullary A. 3289 Figure 370-10 561 Numbness of ipsilateral arm. vomiting in lateral medullary syndrome is due 555 In medial medullary syndrome. trunk. AJAY MATHUR Neurology A. Nystagmus C. Arises from innominate artery on right side 559 Loss of taste in lateral medullary syndrome is due to lesion in ? B. there is reversal in the C. pain. oscillopsia. Cerebellar hemisphere If the subclavian artery is occluded proximal to the origin of vertebral artery. producing posterior circulation TIAs. nystagmus. Contralateral Horner’s syndrome Occlusion of any of the five vessels is responsible for producing lateral medullary syndrome. . trunk. Vertigo B. Atherothrombotic lesion has predilection for V3 segment A. Posterior inferior cerebellar B. Horner’s syndrome B. Cuneate and gracile nuclei vertebral foramina. diplopia. due to involvement of pyramidal tract & medial lemniscus. Exercise of the ipsilateral arm may increase demand on vertebral flow. All of the above B. oscillopsia. hoarseness. Posterior circulation TIA Harrison’s 18th Ed. numbness. vertigo. and gracile nuclei. 2 C. D. 556 Occlusion of which of the following artery is responsible for 562 Which of the following is not a part of lateral medullary (or producing lateral medullary syndrome ? Wallenberg’s) syndrome ? Harrison’s 18th Ed. 3289 Figure 370-10 B. is involved ? Harrison’s 18th Ed. Vestibular nucleus Atherothrombotic lesions have a predilection for V1 & V4 segments of vertebral artery. Face is spared. Vertebral A. Loss of taste D. posterior inferior cerebellar. 557 Which of the following is false for lateral medullary syndrome ? 563 Which of the following is not a feature of vertebral artery occlusion ? Harrison’s 18th Ed. Spinocerebellar tract Nystagmus. Strabismus Third segment of vertebral artery (V3) passes through transverse foramen and circles around the arch of the atlas to pierce dura at foramen magnum. 3289 Figure 370-10 C. Nucleus and tractus solitarius Medial medullary syndrome is due to occlusion of vertebral artery or branch of vertebral or lower basilar artery. Nucleus of fifth nerve C. medullary syndrome due to damage to descending sympathetic tract. Dysphagia 553 Which of the following statements about vertebral artery is false ? Due to damage to vestibular nucleus. diplopia. Nucleus of fifth nerve D. Harrison’s 18th Ed. None of the above B. or leg in lateral medullary syndrome is due to lesion in cuneate impaired tactile & proprioceptive sense over half the body occur. rarely. Ipsilateral twelfth nerve syndrome is due to lesion in ? Harrison’s 18th Ed. 3289 Figure 370-10 Harrison’s 18th Ed. but this region is subject to dissection. 3288 A. impaired sensation over half the face occurs on the side of lesion. nausea. nausea & vomiting occur in lateral medullary syndrome. 3289 Figure 370-10 C. 2386 C. Cerebral infarction Occlusion of anterior inferior cerebellar artery produces lateral inferior pontine syndrome. diplopia. Anterior inferior cerebellar artery Occlusion of vertebral artery can produce medial and lateral and total unilateral medullary syndrome. Medial medullary syndrome 573 “Insular ribbon sign” is an earliest indicator of ? Harrison’s 16th Ed. All of the above 567 Which of the following syndromes occur due to occlusion of Cerebral infarction infarct may not be seen reliably for 24 to 48 hours.a branch C. Harrison’s 18th Ed. Lateral midpontine syndrome 574 Which of the following is used to visualize extra. Superior cerebellar artery Constellation of bilateral long tract signs (sensory and motor) with signs of cranial nerve and cerebellar dysfunction is suggestive of complete basilar artery occlusion. Basilar artery Harrison’s 18th Ed. 2384. Lateral superior pontine syndrome A. Cerebral infarction for first 24 to 48 hours D. 2383. All of the above 569 Corticobulbar and corticospinal tracts are damaged bilaterally MRI with fat saturation is used to visualize extra.or intracranial arterial dissection as it images due to occlusion of ? clotted blood within dissected vessel wall. 3290 C. Total unilateral medullary syndrome B. 3292 Figure 370-13 A. Anterior inferior cerebellar artery D. Any of the above the following vessel ? Harrison’s 18th Ed. Complete middle cerebral artery occlusion A. Medial inferior pontine syndrome A. Upper basilar artery C. D. Harrison’s 18th Ed. D. Subarachnoid hemorrhage D. MRI with fat saturation Lateral superior pontine syndrome occurs due to occlusion of superior cerebellar artery . Complete basilar occlusion leads to a constellation of bilateral long tract signs (sensory nerve and cerebellar dysfunction is suggestive of ? and motor) with signs of cranial nerve and cerebellar dysfunction. 3289 Figure 370-10 A. Total unilateral medullary syndrome arteries ? Harrison’s 18th Ed. Hemiparesis is not a feature of vertebral artery occlusion. All of the above 568 Which of the following syndrome occurs due to occlusion of “Insular ribbon sign” is caused by edema within insular cortex and basal ganglia and is an earliest superior cerebellar artery ? indicator of cerebral infarction in MCA territory. Meningitis C. dysarthria. 2386 B. A. AJAY MATHUR Cardiology 689 Embolic occlusion or thrombosis of a V4 segment causes ischemia of the lateral medulla. 689 Neurology MCQ’s FORMCQ’s FOR MEDICAL MEDICAL PROFESSIONALS PROFESSIONALS BY PROF. Basilar artery constellation of vertigo. Small cortical infarcts Lateral pontomedullary syndrome is due to occlusion of vertebral artery. Complete posterior cerebral artery occlusion following arteries ? Harrison’s 16th Ed. D. Lateral medullary syndrome 571 Deafness is produced by occlusion of which of the following C. B. Harrison’s 18th Ed. Lateral superior pontine syndrome due to occlusion of superior cerebellar artery sometimes produces Horner’s syndrome. The B. Complete basilar artery occlusion 570 Horner’s syndrome is produced by occlusion of which of the B. 3289 D. Vertebral artery images ? B.or intracranial arterial dissection ? B. Vertebral artery D. 3289 Figure 370-10 anterior inferior cerebellar artery produces Horner’s syndrome. All of the above A. Complete anterior cerebral artery occlusion B. Anterior inferior cerebellar artery (or Wallenberg’s) syndrome. C. Ischemic strokes in posterior fossa C. The last one being a combination of medial and lateral syndromes. Superior cerebellar artery 564 Constellation of bilateral long tract signs with signs of cranial Basilar artery or lone vertebral artery occlusion can damage corticobulbar and corticospinal tracts bilaterally. MR angiography D. All of the above C. Superior cerebellar artery B. All of the above 565 Which of the following syndromes occur due to occlusion of In lateral medullary syndrome due to occlusion of vertebral artery. Vertebral artery . Horner’s syndrome occurs due vertebral artery ? to damage to descending sympathetic tract. Medial superior pontine syndrome Harrison’s 18th Ed. Anterior inferior cerebellar artery A. dysphagia. and ipsilateral Horner’s syndrome is called the lateral medullary C. Lateral inferior pontine syndrome due to occlusion of Harrison’s 18th Ed. Harrison’s 18th Ed. 3288 A. Lateral inferior pontine syndrome B. Medial medullary syndrome B. 3290 Figure 370-11 A. Anterior cerebral artery 566 Lateral pontomedullary syndrome is due to occlusion of which of D. CT may fail to show anterior inferior cerebellar artery ? small ischemic strokes in posterior fossa and small infarcts on cortical surface. Also. hoarseness. 3291 C. numbness of the ipsilateral face and contralateral limbs. 3289 Figure 370-10 572 Which of the following is not reliably detected by CT radiographic A. D. MRI with IV gadolinium contrast of basilar artery. B. Decreases maximum frequency of occurrence ? D. 3293 A. mean arterial blood pressure should be ? The most common sites of hypertensive intraparenchymal hemorrhage are basal ganglia (esp. 1 to 10 minutes Intraparenchymal hemorrhage is the most common type of intracranial hemorrhage. Globus pallidus A. the eyes deviate towards B. Nicardipine Putamen is the most common site for hypertensive intraparenchymal hemorrhage. Hemorrhage presents as abrupt rupture of ? onset of focal neurologic deficit. Labetalol 582 In hypertensive intraparenchymal hemorrhage. 30 to 90 minutes 578 Which of the following is the most important causes of D. Any of the above Harrison’s 18th Ed. < 150 mm Hg hypertensive intraparenchymal hemorrhage ? Harrison’s 18th Ed. Increases 577 Out of the following hemorrhagic stroke subtypes. 3294 A. Hypertension 584 Which of the following statements about intracerebral hemorrhages B. Hypertension. C. Any of the above B. Remain in the center D. 3294 C. All of the above Harrison’s 18th Ed. A. Esmolol into the ventricular space. Medulla oblongata 575 In intraparenchymal hemorrhage. Seizures are common ischemic stroke. < 140 mm Hg A. Almost always occur in awake state D. D.690 MCQ’s FOR MEDICAL PROFESSIONALS BY PROF. AJAY MATHUR Neurology B. Abrupt onset of focal neurologic deficit Advanced age and heavy alcohol consumption increase the risk of intraparenchymal hemorrhage. Cerebral amyloid angiopathy is false ? Harrison’s 18th Ed. 10 to 30 minutes cerebral amyloid angiopathy cause the majority of intraparenchymal hemorrhage. Harrison’s 18th Ed. Cocaine use A. Aneurysm of artery A. < 130 mm Hg B. trauma and B. Seizures are uncommon. if blood dissects C. labetalol or esmolol). Basal ganglia . 3294 A. Towards the side of hemiparesis Hypertensive intraparenchymal hemorrhage usually results from spontaneous rupture of a small penetrating artery deep in the brain. Focal deficit worsens steadily over 30- Harrison’s 18th Ed. 3293 D. blood may dissect into the ventricular space which A. Subdural 583 Most hypertensive intraparenchymal hemorrhages develop over C. Cerebellum Intracranial hemorrhage C. Intraparenchymal substantially ‘increases’ morbidity and may cause hydrocephalus. which has the C. Putamen 576 Which of the following drugs is useful in lowering blood pressure C. and SAH are all associated with cocaine use. Caudate nucleus in intraparenchymal hemorrhage ? Harrison’s 18th Ed. Intracerebral hemorrhage. Cocaine use is one of the most important causes in young (<45 years). It accounts for ~10% of all strokes and is associated with a 50% case fatality rate. A. Penetrating artery 585 In hypertensive putaminal hemorrhage. thalamus. Heavy alcohol consumption B. 3294 C. putamen). 3294 Most hypertensive intraparenchymal hemorrhages develop over 30 to 90 minutes. which is the most common site for B. contralateral hemiparesis is the sentinel sign. cerebellum and pons. 3294 In hypertensive intraparenchymal hemorrhage. Remains the same Intraparenchymal hemorrhage is exacerbated by acutely elevated BP & should be lowered to a mean arterial BP of <130 mm Hg by nonvasodilating IV drugs (nicardipine. Focal deficit worsens steadily over 30 to 90 minutes 579 Hypertensive intraparenchymal hemorrhage is due to spontaneous Intracerebral hemorrhages almost always occur in awake state. < 160 mm Hg 581 Out of the following. 90 to 180 minutes intraparenchymal hemorrhage in the young ? Harrison’s 18th Ed. Bifurcation of artery which side ? Harrison’s 18th Ed. the morbidity ? D. C. C. 3295 C. Eyes deviate Harrison’s 18th Ed. Subarachnoid Harrison’s 18th Ed. Rotate upwards 580 Which of the following is not a common site for hypertensive intraparenchymal hemorrhage ? In putaminal hypertensive hemorrhage. A. B. 3294 90 minutes with a diminishing level of consciousness & signs of raised ICP (headache/vomiting). Epidural what period of time ? D. B. Thalamus D. Away from the side of hemiparesis D. Pons D. 3294 away from the side of hemiparesis. Thalamus C. if there is In cerebellar hemorrhage. the location lateral gaze toward the side of hemorrhage. Pontine hemorrhage B. Pons B. Drowsiness Presentation of pontine hemorrhage include sudden onset. A. 3295 Cerebellar hemorrhages develop over several hours & are characterized by occipital headache. Thalamus A. Putamen In pontine hemorrhages. Occipital hemorrhage C. Paresis of conjugate lateral gaze toward side of hemorrhage visual field defect. 3295 596 Which of the following is uncommon in lobar hemorrhages ? A. Forced deviation of eyes to opposite side Thalamic hemorrhages by extension inferiorly into upper midbrain may lead to homonymous B. Pontine hemorrhage is in ? Harrison’s 18th Ed. with parietal hemorrhage it is hemisensory loss and with frontal hemorrhage 590 Excessive sweating (hyperhidrosis) is a feature of which of the it is arm weakness. C. D. Cerebellum C. Seizure A. None of the above 589 In hypertensive intraparenchymal hemorrhage. 3295 D. Arterioles . Ataxia of gait 587 In thalamic hemorrhage. 3295 A. Pons 592 Cerebellar hemorrhages are characterized by ? Harrison’s 18th Ed. Major neurologic deficit with occipital hemorrhage is hemianopia. Putamen Harrison’s 18th Ed. 3295 B. 3295 D. D. Stiff neck loss of reflex eye movements and corneal responses. 3295 D. unequal pupils with absence of light reaction. Frontal hemorrhage turning (doll’s head or oculocephalic maneuver). B. Usually develop over several hours paralysis of vertical gaze. Prominent sensory deficit involving all modalities is usually present. dizziness or vertigo may be prominent. Absence of convergence 594 Which of the following about cerebellar hemorrhage is false ? C. ipsilateral Horner’s syndrome. Cortical neurologic signs other than ? Harrison’s 18th Ed. 3295 B. B. Vomiting D. Occipital headache C. there may be no other neurologic signs other than gait ataxia. Proprioceptive 593 In mild cases of cerebellar hemorrhage. and retraction nystagmus. Occipital headache Thalamic hemorrhages may extend inferiorly into upper midbrain and cause deviation of eyes downward and inward. Ipsilateral Horner’s syndrome Harrison’s 18th Ed. the location of hemorrhage C. Gait ataxia Thalamic hemorrhages also produce contralateral hemiplegiadue to involvement of adjacent internal capsule. Basal ganglia hemorrhage A. absence of convergence. Thalamic hemorrhage downward & inward deviation of eyes. severe hypertension and excessive sweating. skew deviation with eye opposite hemorrhage displaced downward & medially. Unequal pupils with absence of light reaction In mild cases of cerebellar hemorrhage. 3295 D. with left temporal hemorrhage it is aphasia and delirium. B. there is impairment of reflex horizontal eye movements evoked by head D. 3295 595 Which of the following lobar hemorrhage causes arm weakness ? A. posturing. or an ipsilateral sixth nerve palsy. deep coma with quadriplegia usually occurs over a few minutes with prominent decerebrate rigidity. 3295 D. 691 Neurology MCQ’s FORMCQ’s FOR MEDICAL MEDICAL PROFESSIONALS PROFESSIONALS BY PROF. there may be no other C. Focal headaches C. Repeated vomiting 588 Which of the following is true in thalamic hemorrhage ? Harrison’s 18th Ed. AJAY MATHUR Cardiology 691 586 In hypertensive intraparenchymal hemorrhage. 3295 angiopathy ? Harrison’s 18th Ed. following CVA’s ? Harrison’s 18th Ed. if there is a B. and ataxia of gait. B. Cerebellar hemorrhage A. of hemorrhage is in ? Harrison’s 18th Ed. Basal ganglia hemorrhage Harrison’s 18th Ed. 591 Decerebrate rigidity is most frequent which of the following 597 Which of the following is mainly affected in cerebral amyloid hypertensive intraparenchymal hemorrhage ? Harrison’s 18th Ed. Exteroceptive repeated vomiting. All of the above A. Parietal hemorrhage In pontine hemorrhages. Cerebellum A. Most patients with lobar hemorrhages have focal headaches and more than half vomit or are drowsy. All of the above Harrison’s 18th Ed. Left temporal hemorrhage D. All of the above A. There is paresis of conjugate impairment of doll’s head or oculocephalic maneuver. Repeated vomiting C. B. Cerebellar hemorrhage C. which of the following sensory modality is affected ? D. ocular bobbing. Thalamic hemorrhage A. forced deviation of eyes to opposite side. pinpoint pupils (1 mm) reactive to light. Stiff neck and seizures are uncommon. hyperventilation. advanced age. > 3 cm Intracranial hemorrhages associated with anticoagulant therapy & hematologic disorders can Most cerebellar hematomas >3 cm in diameter require surgical evacuation. D. > 25 mL B. Fresh-frozen plasma B. < 60 mL C. 602 Which of the following worsens the prognosis in intracerebral hemorrhage ? 608 Largest AVMs are most frequently found in ? Harrison’s 17th Ed. > 60 mL D. Cocaine use A. 2392 Harrison’s 18th Ed. 3295 Harrison’s 18th Ed. All of the above 598 Cerebral amyloid angiopathy affects mainly ? 604 What volume of supratentorial hematomas due to hypertensive Harrison’s 18th Ed. Adults B. AJAY MATHUR Neurology B. 3298 A. Vitamin K C. Dissection of the internal carotid or vertebral arteries C. 3298 A. All of the above D. Children A. 603 Which is the agent of first choice in reversing warfarin induced 609 Bleeding or other symptoms from AVMs are most common coagulopathy in intracerebral hemorrhage ? between the ages of ? Harrison’s 18th Ed. 2534 A. Elderly C. Prothrombin complex concentrate infusion A. 2534 Harrison’s 18th Ed. Ventriculostomy Glucocorticoids are not helpful for the edema from intracerebral hematoma. 3295 intracerebral hemorrhage have a good prognosis ? Harrison’s 17th Ed. location within posterior fossa & depressed Largest AVMs are found most frequently in posterior half of hemispheres. Advanced age B. Posterior half of hemispheres C. Lobar hemorrhage C. Pontine hemorrhage C. All of the above D. 10 and 30 years C. 605 What volume of supratentorial hematomas due to hypertensive intracerebral hemorrhage have a poor prognosis ? 599 Cerebral amyloid angiopathy cause which of the following ? Harrison’s 16th Ed. They may also be found level of consciousness at initial presentation worsens prognosis in intracerebral hemorrhage. Spinal cord Extension into ventricular system. Moyamoya disease B. > 2 cm D. 3295 A. Any of the above C. < 120 mL Cerebral amyloid angiopathy is a disease of the elderly in which arteriolar degeneration occurs and amyloid is deposited in the walls of the cerebral arteries. All of the above D.5 cm C. Location within the posterior fossa C. 30 and 50 years . 600 Intracranial hemorrhages associated with anticoagulant therapy 606 What diameter of cerebellar hematoma will require surgical occur in ? evacuation ? Harrison’s 18th Ed. Intraparenchymal hemorrhage B. < 90 mL D. 3298 A. occur at any location and may present as multiple intracerebral hemorrhages. an intermediate prognosis and >60 mL. Anterior half of hemispheres B. Extension into the ventricular system A. Supratentorial hematomas due to Cerebral amyloid angiopathy causes both single and recurrent lobar hemorrhages and is the most hypertensive intracerebral hemorrhage with volumes <30 mL have a good prognosis 30 to 60 mL. Skin and mucous membrane bleeding is usually evident and offers a diagnostic clue. Subarachnoid hemorrhage Volume and location of hematoma determine prognosis. 3298 Harrison’s 18th Ed. 3295 A. > 40 mL C. Venules D. < 30 mL B. Brainstem D.5 cm B. 1 and 10 years B. Glucocorticoids B. Any location D. Thalamic hemorrhage B. 3298 Harrison’s 18th Ed. 607 Which of the following is not helpful for the edema from 601 Which of the following is a cause of stroke in young ? intracerebral hematoma ? Harrison’s 18th Ed. > 10 mL A. Lobar hemorrhage A. Capillaries Agent of first choice in reversing warfarin induced coagulopathy in intracerebral hemorrhage is prothrombin complex concentrate followed by fresh-frozen plasma & vitamin K. > 1. > 0. Osmotic agents C. a poor prognosis during initial hospitalization. common cause of lobar hemorrhage in elderly. in brainstem and spinal cord. Cerebellar hemorrhage D. Induced hyperventilation D.692 MCQ’s FOR MEDICAL PROFESSIONALS BY PROF. Harrison’s 18th Ed. Skin and 620 Which of these is a natural anticoagulant ? mucous membrane bleeding is usually evident and offers a diagnostic clue. D. Arteriovenous malformation A. CST3. 3299 B. Pons D. Conventional x-ray angiography is the gold standard for evaluating amyloid angiopathies ? precise anatomy of AVM. Stroke tions from a dural artery to dural sinus. Pulmonary embolus Capillary telangiectasias are true capillary malformations in normal brain structure. Dilute Russell viper venom time (dRVVT) D. C. 3298 D. BRI A. Mid brain APP. C. All of the above B. Bleeding rarely produces mass effect or significant symptoms. Subarachnoid A. Deep vein thrombosis (DVT) B. Symptomatic cerebral vasospasm is rare D. 3299. N Engl J Med 2002. 3298 coagulation test results ? Disease-a-Month 2003. Antibodies against B2G1 which of the following ? Harrison’s 18th Ed. Cerebellum antibody syndrome is ? Capillary telangiectasias are true capillary malformations located typically in pons and deep Harrison’s 16th Ed. Anticardiolipin antibodies B. All of the above C. N Engl J Med 2002. Hypertension D. BRI genes cause autosomal dominant amyloid angiopathies. Dural arteriovenous fistulas C. Activated protein C A. 1964 D. All of the above Intracranial hemorrhages associated with anticoagulant therapy can occur at any location. 693 Neurology MCQ’s FORMCQ’s FOR MEDICAL MEDICAL PROFESSIONALS PROFESSIONALS BY PROF. Cocaine abuse include ? C. None of the above Disease-a-Month 2003. Smaller lesions have a higher hemorrhage rate KRIT1 and CCM2 have been identified as genes causing cavernous angiomas. B. Disease-a-Month 2003. All of the above A. APP 611 Which of the following is a common location of capillary telangiectasias in brain ? B.49:696-741 A. 1964 cerebral white matter as in hereditary hemorrhagic telangiectasia (Osler-Rendu-Weber) syndrome. Dural AV fistulas are acquired connec.347:1711 610 Which of the following statements about AVM bleed is false ? A. A. Intraparenchymal B. AJAY MATHUR Cardiology 693 D. Pons & deep cerebral white matter are typical locations. APP A. Capillary telangiectasias 621 Sneddon syndrome consists of ? D. NOTCH3 Harrison’s 18th Ed. Any of the above D. Hereditary hemorrhagic telangiectasia (Osler-Rendu. Symptomatic cerebral vasospasm is rare. Livido reticularis C. 3298 B. A. 50 and 75 years 615 Which of the following gene is responsible in causing cavernous angiomas ? Bleeding or other symptoms from AVMs are most common between the ages of 10 & 30 years. KRIT1 C. Annexin V B. Capillary telangiectasias Harrison’s 16th Ed. Smaller lesions have a higher hemorrhage rate. Cavernous angiomas are tufts of capillary sinusoids in deep hemispheric white matter & brainstem with no normal intervening neural structures. Cavernous angiomas D. CST3 Harrison’s 18th Ed. Medulla oblongata 617 Primary class of antibody that is associated with antiphospholipid D. BRI B. Activated partial thromboplastin time (aPTT) B.347:1711 A. Prothrombin time C.49:696-741 614 Which of the following is not an acquired vascular lesions ? A. 3298 C.49:696-741 Capillary telangiectasias are true capillary malformations. D. Placental vascular thrombosis Weber) syndrome is an example. Lupus anticoagulant 612 Intracranial hemorrhage located usually in brainstem is due to C. Basal ganglion C. Hypertensive hemorrhage 618 Thrombotic events associated with antiphospholipid antibodies B. MRI is gold standard for evaluating anatomy of AVM 616 Which of the following genes can cause autosomal dominant Hemorrhage is mainly intraparenchymal. Anticoagulant-related intracerebral hemorrhages evolve slowly over 24 to 48 hours. All of the above 613 Intracranial hemorrhage due to a coagulopathy occurs in which of the following locations ? 619 The lupus anticoagulants prolong which of the following Harrison’s 18th Ed. All of the above . Beta2-Gp1 Harrison’s 18th Ed. Hemorrhage is mainly intraparenchymal C. 30% in first month. MCA bifurcation C. Anterior communicating artery A.49:696-741 C. Middle of basilar artery perimesencephalic cisterns ? Harrison’s 17th Ed. 627 What percentage of adults harbor intracranial aneurysms ? Harrison’s 17th Ed. 30 % seen at ? C. > 0. 626 Which of the following is the most common cause of SAH ? Harrison’s 17th Ed. the rate of rebleeding is about 20% B. Bleeding time 630 Most common location of an intracranial aneurysm is ? Harrison’s 17th Ed. > 1. 1 % of major arteries of circle of Willis ? Harrison’s 17th Ed. Anterior communicating artery Subarachnoid hemorrhage (SAH) B. Benign Harrison’s 17th Ed. Middle of basilar artery or capillary source. 1st D. All of the above 631 Most common location of an intracranial aneurysm is ? Harrison’s 17th Ed. Posterior communicating artery B.0 cm Disease-a-Month 2003. D. aPTT D. 1727 B.5 cm B. 8 or more weeks of gestation in first 2 weeks.49:696-741 D. 1726 632 Most common location of an intracranial aneurysm is ? A. None of the above C. 1727 623 Which of the following is the preferred test for diagnosis of A. > 1. 4th 628 In SAH. 2 % C. 3rd Autopsy & angiography studies show that ~2% of adults harbor intracranial aneurysms. Most result from infected emboli due to bacterial endocarditis. 1726 633 Mycotic aneurysms are best related to ? A. 20 % . 4 % B. the rate of rebleeding in the first month is about ? Mycotic aneurysms due to infected emboli are usually located distal to the first bifurcation of major Harrison’s 17th Ed. Extension into subarachnoid space from a primary B. 1726 634 Mycotic aneurysms are usually located distal to which bifurcation A. Diabetes mellitus intracerebral hemorrhage C. Posterior communicating artery 625 Which of the following is false about idiopathic SAHs localized to D. 6 or more weeks of gestation D. A.5 cm antiphospholipid antibody ? B. 50 % A. Top of basilar artery Some idiopathic SAHs are localized to perimesencephalic cisterns and are benign with a venous D. Terminal internal carotid artery C.49:696-741 A. if the patient survives but aneurysm is not obliterated. KCT Berry aneurysms >2. Posterior communicating artery D. Hypertension C. Trauma After head trauma.5 cm A. Angiography is unrevealing. the most common cause of SAH is rupture of a saccular aneurysm. Recurrent arterial thrombosis B. > 2. Bacterial endocarditis D. Mycotic aneurysms are usually located distal to the first bifurcation of major arteries of the circle of Willis. Idiopathic D. 1727 B. AJAY MATHUR Neurology 622 Obstetric complications in antiphospholipid syndrome are usually B. 40 % Disease-a-Month 2003.694 MCQ’s FOR MEDICAL PROFESSIONALS BY PROF. 1727 A. 4 or more weeks of gestation 629 An aneurysm is termed ‘Giant’ when its diameter is ? Harrison’s 17th Ed. dRVVT D. Vertebral artery D. Venous or capillary source A. The three most common locations of berry aneurysms are the terminal internal carotid artery. Thrombocytopenia D. 1727 624 Antiphospholipid syndrome is a disorder characterized by ? Disease-a-Month 2003. and top of basilar artery. MCA bifurcation. 2nd C. 3 % A. Anterior communicating artery C. 1727 B. Bleeding from a vascular anomaly A. Spontaneous abortions C. 1726 arteries of the circle of Willis. C. 10 or more weeks of gestation In SAH. C.5 cm in diameter are called giant aneurysms. Rupture of a saccular aneurysm Harrison’s 17th Ed. Angiography is unrevealing B. 5 % C. with pupillary dilatation. Posterior inferior cerebellar artery aneurysm Most patients first complain of headache upon regaining consciousness. Anterior cerebral artery aneurysm The site of rupture of a berry aneurysm most often is the dome. and visual field defects can occur with an anterior cerebral artery aneurysm. 1727 D. 1727 D. Temporal C. Any of the above C. Posterior communicating artery aneurysm B. A. >7 mm in diameter B. 1727 636 What percentage of all intracranial aneurysms are multiple ? A. Anterior inferior cerebellar artery aneurysm C. MCA aneurysm In SAH. Any of the above A. many at mirror sites bilaterally. 1727 B. Anterior cerebral artery aneurysm D. 1727 D. mostly on the circle of Willis. 647 Pain in or behind the eye and in the low temple suggests what 640 The location of headache in SAH is ? location of an aneurysm ? Harrison’s 17th Ed. Posterior communicating artery aneurysm Those >7 mm in diameter and those at the top of the basilar artery and at the origin of the posterior B. Pain in or behind the eye and in the low temple can occur with A. Acalculia A. Anterior cerebral artery aneurysm 638 The site of rupture of a berry aneurysm is ? D. Anterior inferior cerebellar artery aneurysm 639 Patients first complaint upon regaining consciousness in SAH is ? 646 Occipital and posterior cervical pain suggests what location of an Harrison’s 17th Ed. Aneurysm in the cavernous sinus D. 65 % D. AJAY MATHUR Cardiology 695 635 Anterior circulation accounts for what percentage of all B. 1727 A. 45 % hematoma large enough to produce mass effect resulting in hemiparesis. Hemiparesis an expanding MCA aneurysm. At the origin of posterior communicating artery Harrison’s 17th Ed. Dome B. Blurred vision Harrison’s 17th Ed. Top of the neck A. Generalized D. 10 % D. Anterior cerebral artery aneurysm D. III cranial nerve palsy associated with pupillary dilatation. Posterior communicating artery aneurysm Harrison’s 17th Ed. Posterior communicating artery aneurysm C. VI cranial nerve palsy may indicate an aneurysm aneurysmal rupture include all except ? in cavernous sinus. Thirst B. Frontal A. 695 Neurology MCQ’s FORMCQ’s FOR MEDICAL MEDICAL PROFESSIONALS PROFESSIONALS BY PROF. Occipital and posterior cervical pain may signal a posterior inferior cerebellar artery or anterior Harrison’s 17th Ed. Aneurysm in the cavernous sinus D. 1727 inferior cerebellar artery aneurysm. 20 % D. D. Abulia Harrison’s 17th Ed. Posterior communicating artery aneurysm 637 Which of the following characteristics of a berry aneurysm make B. Aneurysm in the cavernous sinus communicating artery are at greater risk of rupture. 85 % 642 III cranial nerve palsy. 1727 aneurysm ? A. Anterior inferior cerebellar artery aneurysm A. 1727 B. 25 % ACA or MCA bifurcation aneurysms may rupture into adjacent brain or subdural space and form a B. Aneurysm in the cavernous sinus it at greater risk of rupture ? C. 40 % 643 VI cranial nerve palsy suggests what location of an aneurysm ? Harrison’s 17th Ed. . loss of ipsilateral light reflex and focal pain above or behind the eye suggests what ~85 % of aneurysms occur in the anterior circulation. Aphasia intracranial aneurysms ? C. loss of ipsilateral light reflex and focal pain above or behind the eye. often with neck stiffness and vomiting. Posterior communicating artery aneurysm C. Headache A. At the top of basilar artery 644 Occipital and posterior cervical pain suggests what location of an aneurysm ? C. Anterior cerebral artery aneurysm B. 1727 ~20% of patients have multiple aneurysms. Anterior cerebral artery aneurysm Harrison’s 17th Ed. Aneurysm in the cavernous sinus C. Nausea C. C. 1727 645 Visual field defects suggests what location of an aneurysm ? A. 1727 B. headache is usually generalized. C. may occur with an expanding aneurysm at the junction of posterior 641 Common deficits that result due to middle cerebral bifurcation communicating artery and internal carotid artery. Occipital B. 1727 Harrison’s 17th Ed. location of an aneurysm ? Harrison’s 17th Ed. Anterior inferior cerebellar artery aneurysm Harrison’s 17th Ed. Aneurysm in the cavernous sinus A. aphasia and abulia. Base of the neck Harrison’s 17th Ed. focal vasospasm is preceded by decline in mental status. Headache A.354:387-96 D. 7 days 650 In a case of SAH who also has third-nerve palsy. 1727 Harrison’s 17th Ed. which artery is possibly involved ? N Engl J Med 2006. Posterior communicating artery C. Anterior communicating artery B. Hydrocephalus C.354:387-96 C. Bacterial meningitis B.696 MCQ’s FOR MEDICAL PROFESSIONALS BY PROF. 1727 C. Decline in mental status C. Posterior communicating artery D.354:387-96 peak in the first 7 days. Anterior communicating artery Chronic hydrocephalus may develop weeks to months after SAH and manifest as gait difficulty. Posterior communicating artery Delayed vasospasm is due to direct effects of clotted blood & its breakdown products on artery. > 0. B. HIV dementia C. Clotted blood & its breakdown products on artery A. 1 day C. Day 7 B. which artery is possibly involved ? N Engl J Med 2006. Subarachnoid space A. Thalamus D. which artery is D. Hyperviscosity visuospatial neglect. and hyponatremia. Harrison’s 17th Ed. Anterior communicating artery C. Day 14 C. > 0. incontinence. Vomiting D. Hypertension 652 In a case of SAH who also has aphasia. A.5 C. Gray matter C. Any of the above 660 In most cases of SAH. most often at 7 days. Anterior communicating artery D. focal vasospasm is preceded by ? 654 Clinical scale of Hunt and Hess is used in ? Harrison’s 17th Ed. Anterior cerebral artery Signs of cerebral ischemia appear 4 to 14 days after SAH. Seizure B. Middle cerebral artery Harrison’s 17th Ed.3 A.354:387-96 A.2 Harrison’s 17th Ed. Posterior communicating artery 657 Chronic hydrocephalus after SAH manifests as ? Harrison’s 17th Ed. hydrocephalus. Anterior cerebral artery 658 Delayed vasospasm in SAH is due to ? D. Middle cerebral artery B. vasospasm. All of the above B. There are four major causes of delayed neurologic deficits in SAH. Vasospasm D.4 B. 649 ‘Wide-neck aneurysms’ are those in which the ratio of neck diameter to that of the largest dome is ? 656 Peak incidence of rerupture of an untreated aneurysm following N Engl J Med 2006. Middle cerebral artery 659 Signs of cerebral ischemia due to vasospasm following SAH is most often on which day ? 653 In a case of SAH who also has retinal and subhyaloid hemorrhage. Incontinence 651 In a case of SAH who also has bilateral weakness in legs or C. Hyponatremia Aneurysms with small ruptures & leaks of blood into subarachnoid space are sentinel bleeds. Day 1 N Engl J Med 2006. Initial clinical manifestations of SAH can be graded by Hunt-Hess or World Federation of Neurosurgical Societies (WFNS) classification. 1727 which artery is possibly involved ? A. AJAY MATHUR Neurology 648 ‘Sentinel bleed’ is the term used for small aneurysmal rupture 655 Which of the following is not a cause of delayed neurologic deficit into ? in SAH ? Harrison’s 17th Ed. or impaired mentation. Gait difficulty D. Impaired mentation abulia. 1727 Harrison’s 17th Ed. 3 days D. 1727. All of the above A. Subarachnoid hemorrhage D. White matter B. 1727 A. Multiple sclerosis In most SAH cases. C. > 0. Infection B. D. 1727 A. 1727 B. N Engl J Med 2006. 14 days possibly involved ? Incidence of rerupture of an untreated aneurysm in the first month following SAH is ~30% with the N Engl J Med 2006. Day 3 A. .354:387-96 B. Anterior cerebral artery A.354:387-96 SAH is within ? A. > 0. They are rerupture. hemiparesis. Anterior cerebral artery D. or B. 1633 A. > 0. which of the following ECG finding is unusual ? 671 Which of the following drugs is most suited to lower raised blood Harrison’s 17th Ed. In general. Subarachnoid blood C. > 3 x 3 mm C.1 mm B. Inappropriate secretion of vasopressin Harrison’s 17th Ed. Esmolol C. 1729 basal cisterns ? Harrison’s 17th Ed. > 1 mm C. > 4 x 3 mm D. Secretion of brain natriuretic factors B. 1 to 4 weeks C. 1728 A. Nicardipine D. 1729 B. 1728 at ? Harrison’s 16th Ed. Vasospasm causing symptomatic ischemia & infarction C. or esmolol. Gold C. agents such as nicardipine. 1 to 2 weeks C. Vasospasm following SAH causes symptomatic ischemia and infarction 663 Incidence of symptomatic vasospasm in MCA & ACA is high and is a major cause of delayed morbidity and death.25 mm A. acute hydrocephalus. < 20 mm Hg subarachnoid clots >5 x 3 mm in basal cisterns or layers of blood >1 mm thick in cerebral fissures. “Peaked” or deeply inverted symmetric T waves B. > 5 x 3 mm High ICP refractory to treatment is a poor prognostic sign. cerebral perfusion pressure should be maintained at 60 .354:387-96 Harrison’s 17th Ed. < 10 mm Hg D. . 2389 secondary to all except ? A. 40 . > 0. Parenchymal hematoma D. 662 Incidence of symptomatic vasospasm in MCA & ACA is high 668 Which of the following are poor prognostic signs in SAH ? when early CT scans show subarachnoid clots of what size in Harrison’s 17th Ed. Steel D. coils are made of ? pressure in patients with acute SAH ? Harrison’s 17th Ed. 1 to 4 days A. 1 to 3 weeks B. 1729 B. Systemic hypertension Hyponatremia following SAH or “cerebral salt-wasting syndrome” develop in first 2 weeks due to D. AJAY MATHUR Cardiology 697 661 Hyponatremia following SAH is due to ? 667 Intracranial hypertension following aneurysmal rupture occurs Harrison’s 16th Ed. Any of the above Prolonged QRS. 1729 A. It clears over 1 to 2 weeks. 1728.6 mm B. Nifedipine C. labetalol. intracranial pressure should be maintained Harrison’s 17th Ed. > 0. All of the above D. parenchymal hematoma.60 mm Hg D. High ICP refractory to treatment A. Rerupture of untreated aneurysm in 1st month of SAH B. N Engl J Med 2006. 672 Which of the following drugs is most suited to lower raised blood 666 In endovascular treatment of aneurysms. Platinum A. Labetalol B. Intracranial hypertension following aneurysmal rupture occurs secondary to subarachnoid blood. or loss of vascular autoregulation.50 mm Hg D. Rerupture is associated with a 60% mortality and poor outcome. when early CT scans show blood layer of what thickness in the cerebral fissures ? 669 In treatment of SAH. 664 Xanthochromic CSF in SAH usually lasts for ? 670 In treatment of SAH. Secretion of atrial natriuretic factors A. Any of the above Newer endovascular technique involves placing platinum coils within aneurysm via a femoral Patients with acute SAH should have blood pressure lowered to a normal range with nonvasodilating artery catheter to enhance thrombosis & over time is walled-off from circulation. Prolonged QRS complex Harrison’s 17th Ed. Increased QT interval A. CT scans less reliably predict vasospasm in vertebral. 30 . 1728 maintained at ? Harrison’s 17th Ed. < 15 mm Hg High incidence of symptomatic vasospasm in MCA & ACA is observed when early CT scans show D. 697 Neurology MCQ’s FORMCQ’s FOR MEDICAL MEDICAL PROFESSIONALS PROFESSIONALS BY PROF. Amlodipine D.70 mm Hg Xanthochromic CSF in SAH peaks in intensity at 48 hours and lasts for 1 to 4 weeks. In treatment of SAH.70 mm Hg.40 mm Hg B. increased QT interval & prominent “peaked” or deeply inverted symmetric T waves occur secondary to intracranial hemorrhage. 1729 A. Loss of vascular autoregulation inappropriate secretion of vasopressin & secretion of atrial and brain natriuretic peptides. basilar or posterior cerebral arteries. Serious ventricular dysrhythmias C. Serious ventricular dysrhythmias are unusual. Silver B. cerebral perfusion pressure (CPP) should be Harrison’s 17th Ed. All of the above C. Nicardipine D. 50 . It should not be treated with free-water restriction as this may increase risk of stroke. > 2 x 3 mm B. < 5 mm Hg C. 665 Secondary to intracranial hemorrhage. ICP should be maintained at <20 mm Hg. 60 . 1728 pressure in patients with acute SAH ? A. withdrawal. A. Coma due to accidental ingestion of barbiturates 682 Pathological process in Alzheimer’s disease (AD) begins in ? B. Comorbid diseases medical therapy. 1729 371 . Harrison’s 16th Ed. Age intracranial aneurysm and subarachnoid hemorrhage include ? The single strongest risk factor for dementia is increasing age. Harrison’s 16th Ed.The common forms of dementia are progressive. 1633 while cholinergic signaling is critical for attention and memory functions. N Engl J Med 2006. Conversion reaction . AJAY MATHUR Neurology 673 “Triple-H” therapy in SAH refers to all except ? Harrison’s 17th Ed. and disinhibition. Hyperventilation A. Parietal neocortex which of the following ? AD begins in entorhinal cortex. Hemodilution 679 Which of the following does not appear in the definition of dementia ? C. Coma due to barbiturates given for induction of anesthesia Harrison’s 18th Ed. Impairment in activities of daily living 674 In SAH. Hypothermia A. Geographical location to bring PaCO 2 level to 30-35 mm Hg. 3300 C. Ventriculostomy 683 Which of the following is the single strongest risk factor for B. serotonergic. 3301 Table 371-1 B. judgment and problem solving are also affected. Hydrocephalus pathway ? Harrison’s 18th Ed. All of the above C. Acquired “Triple-H” therapy in SAH includes hypertension. Cortical venous thrombosis 681 Behavior and mood are modulated by which of the following B. calculation. High-dose barbiturates for refractory elevated ICP A. Posterior temporal neocortex 677 Emergent treatment of elevated ICP is most quickly achieved by D. 3300 C. Intravenous nitroglycerine 680 Which of the following is most commonly affected in dementia ? B.698 MCQ’s FOR MEDICAL PROFESSIONALS BY PROF. Intravenous phenylephrine B. A. Entorhinal cortex D. but neurotoxic. 1729 of activities of daily living. A. Central pontine myelinolysis A. hyponatremia of several days’ duration ? Neuropsychiatric and social deficits may also develop in dementia syndromes resulting in depression. 2390 A. D.354:387-96 A. Deterioration in cognitive abilities by volume expansion. 3300 D. Language Memory is the most common cognitive ability lost with dementia. which drug is then used ? It is defined as an acquired deterioration in cognitive abilities that impairs the successful performance Harrison’s 17th Ed. Intraarterial papaverine is an effective vasodilator. Other mental faculties like 675 Which of the following is a complication of correction of marked language. hallucinations. Intoxications 678 Heritable connective-tissue disorders associated with presence of D. visuospatial ability. posterior temporal & parietal neocortex Harrison’s 16th Ed.Dementia A. Visuospatial ability D. delusions. then involves hippocampus. 3300 C. C. Depression C. Hippocampus High-dose barbiturate therapy for refractory elevated ICP is termed “pentobarb coma”. Serotonergic pathway Care must be taken not to correct serum sodium too quickly in patients with marked hyponatremia of several days’ duration as central pontine myelinolysis may occur. Ehlers–Danlos syndrome (type IV) A. if symptomatic vasospasm persists despite optimal D. Calculation If symptomatic vasospasm persists despite optimal medical therapy. C. High-dose barbiturates Harrison’s 18th Ed. 1633 and eventually causes a relatively diffuse degeneration throughout cerebral cortex. C. C. EACA A. All of the above 676 “Pentobarb coma” refers to ? Behavior and mood are modulated by noradrenergic. Hyperventilation dementia ? C. Noradrenergic pathway D. Schizophrenia D. Hypertension B. Vasodilators like verapamil & nicardipine do not last > 8-24 hours. Hypervolemia Harrison’s 18th Ed. Any of the above B. agitation. intraarterial vasodilators & C. Pseudoxanthoma elasticum B. hemodilution and hypervolemia and is achieved B. 3300 D. All of the above B. Race / ethnicity Emergent treatment of elevated ICP is most quickly achieved by intubation and hyperventilation B. and dopaminergic pathways. Intraarterial papaverine Harrison’s 18th Ed. Memory percutaneous transluminal angioplasty are considered. Dopaminergic pathway D. insomnia. Polycystic kidney disease 684 Pseudodementia is best related to ? Harrison’s 18th Ed. 3300 Harrison’s 18th Ed. Mini-mental status examination (MMSE) is a brief screening tool to confirm the presence of cognitive impairment & to follow progression of dementia. 689 Unexplained falls is a feature of ? 695 Cognitive domain includes which of the following ? Harrison’s 18th Ed. scoring is done out of ? Harrison’s 18th Ed. naming. All of the above Progressive supranuclear palsy (PSP) is associated with unexplained falls. Executive function C. alcohol dependence. 3303 Harrison’s 18th Ed. Hippocampal atrophy C. 685 Which of the following is not a potentially reversible dementia ? thyroid dysfunction. Vitamin B12 deficiency . Alzheimer’s disease B. 696 Which of the following is best related to visuospatial disorientation ? 690 Dementia with peripheral neuropathy suggests ? Harrison’s 18th Ed. language comprehension. Subdural effusion D. 198-199 Harrison’s 18th Ed. Campbell C. and visuospatial skills. Cortical basal degeneration C. 694 Mini Mental Status Examination (MMSE) provides information 688 Rapid progression of the dementia with motor rigidity & myoclonus regarding ? suggests which of the following ? Harrison’s 18th Ed. 3303 A. Lyme disease pseudodementia. Normal-pressure hydrocephalus A. executive function. AJAY MATHUR Cardiology 699 D. factors that predict progression from MCI to AD include a memory deficit MMSE contains tests of orientation. Dementia associated with depression (pseudodementia). All of the above Dementia with a peripheral neuropathy suggests vitamin B12 deficiency. Prion disease C. and conversion reaction are potentially reversible while that with HIV is not. presence of an apolipoproteine4 (Apoe4) and (orientation & 3-word recall). Lyme disease. Alzheimer’s disease A. Depression 691 In Mini Mental Status Examination (MMSE). startle-sensitive myoclonus suggests CJD. and figure copying. due to drug. 699 Neurology MCQ’s FORMCQ’s FOR MEDICAL MEDICAL PROFESSIONALS PROFESSIONALS BY PROF. 20 C. 693 MMSE is named after ? 687 Rapid progression of dementia with motor rigidity & myoclonus Harrison’s 18th Ed. Folstein D. Harrison’s 18th Ed. Creutzfeldt-Jakob disease Folstein mini-mental status examination (MMSE) is an easy. All of the above D. Rapid progression of dementia with motor rigidity & myoclonus suggests a prion disease. subacute combined degeneration. 3303 Table 371-5 B. Frontotemporal dementia D. axial rigidity. Alzheimer’s disease B. Visuospatial tasks D. 3301 A. 50 medication. Memory B. Thyroid dysfunction Severely depressed or anxious individuals may appear demented. hydrocephalus. Ventricular system dilatation B. standardized screening examination of cognitive function esp. Dialysis dementia (aluminum) B. 40 Potentially reversible dementias are depression. Adrenal insufficiency and Cushing’s syndrome. All of the above In neuroimaging studies. pellagra. family history of dementia. It is a 30-point test of cognitive function. 196 A. Cortical basal degeneration D. Frontotemporal dementia C. Parkinson’s disease Mini Mental Status Examination (MMSE) can provide information regarding orientation. and vertical gaze deficits. Orientation B. 3303 Table 371-5 A. Frontotemporal dementia B. 30 D. akinetic-mute state & specific for the diagnosis of dementia. D. 3303 suggests ? Harrison’s 18th Ed. Cognitive domains include memory. Visuospatial skills C. and narcotic poisoning. dysphagia. visuospatial tasks and language. schizophrenia. working and episodic memory. or vasculitis. The test is 85% sensitive and 85% Rapid progression of dementia in association with diffuse motor rigidity. 3301 A.5 SD from normal. HIV B. small hippocampal volumes + posterior-predominant cortical atrophy. 239 Harrison’s 18th Ed. working memory (spell world backwards). Samuels A. Alcoholism C. a phenomenon called C. Diffuse white matter disease A. Progressive supranuclear palsy D. Orientation A. episodic memory >1. language. Blumenthal B. Optic ataxia A. Language B. 686 Which of the following can predict progression from mild cognitive impairment (MCI) to Alzheimer’s disease (AD) ? 692 MMSE contains tests for ? Harrison’s 18th Ed. D. 3301 Table 371-1 A. heavy metal intoxication. Working memory C. Episodic memory D. and supplementary motor area A. which explains why episodic memory deficits are the most 700 Which of the following is false about “Digit span forward” test ? common cognitive deficits that follow these disorders. Test of attention 706 The process of memory retrieval requires which of the following cerebral lobes ? B. Retrieval (decoding or recall) called digit span backward which is a test of working memory. Ocular apraxia B. Working A. 2394 Harrison’s 16th Ed. retention (storage or consolidation). Episodic A. Remote B. Harrison’s 16th Ed. Harrison’s 18th Ed. Parietal lobe Harrison’s 18th Ed. cerebellum. Episodic B. Parietal lobe More permanent stores of words. All of the above Bilateral parietal lesions produce Bálint’s syndrome. while normal adults can generally repeat five digits backward. C. and physiologic changes in synapses in Memory is divided into three major types . which is characterized by impaired eye-hand Episodic memory is tested by asking a patient to recall three words after 3 to 5 minutes. Prefrontal lobe Working memory lasts for <30 seconds. All of the above D. D. and remote. and visuospatial disorientation (simultanagnosia). this brain region accompany new episodic memories. Simultanagnosia C. Remote D. with the clinician gradually increasing the number of to-be-retained digits. It is tested by asking the patient to recall digits backwards. 699 Which of the following types of memory is tested by asking the patient to recall digits backwards ? 705 Most common cognitive deficits that follow seizures. None of the above maintenance. < 20 seconds B. Hippocampal complex is vulnerable to metabolic insults such as seizures. Normal capacity is five numbers while the process of retrieval requires frontal lobes. 2394 Most common bedside test of working memory involves asking patients to repeat a series of digits A. hypoxia is ? A. AJAY MATHUR Neurology B. 2393 A. Remote Hippocampal complex is critical for episodic memory. Remote memory deficits Working memory is highly vulnerable to distraction. All of the above 702 Which of the following types of memory is tested by asking the Memory function includes registration (encoding or acquisition). 2393 require which of the following ? A. D. Working memory deficits C. Remote C. None of the above B. Working B. None of the above Harrison’s 16th Ed. Asking patient to repeat digits in same order told A. historic facts or names Harrison’s 16th Ed. requiring attention and vigilance for its D. Clinician may also ask the patient to repeat digits in reverse order. episodic.working. dates. 2394 B. 2394 B. coordination (optic ataxia). hypoxia. Episodic memory deficits D. Episodic D. B. Chapter e9 D. Asking patient to repeat digits in reverse order 707 Memory function includes which of the following ? D. and neurodegenerative processes. facts or names require left anterior temporal cortex. Test of working memory The process of memory encoding is dependent upon frontal lobes and hippocampal complex.700 MCQ’s FOR MEDICAL PROFESSIONALS BY PROF. All of the above D. Temporal lobe 701 Which of the following is false about “Digit span backward” test ? C. hypoglycemia. < 10 seconds A. 703 Hippocampal complex is critical for which of the following types of memory ? 697 Which of the following is not a major type of memory ? Harrison’s 16th Ed. Retention (storage or consolidation) is a test of attention. stabilization (consolidation). Frontal lobe D. Episodic C. 2394 708 The basal ganglia. Chapter e9 A. < 30 seconds C. Asking the patient to repeat digits in the same order as they were delivered is called digit span forward which B. Registration (encoding or acquisition) orally. Left anterior temporal cortex D. Sequential C. All of the above C. difficulty initiating voluntary eye movements (ocular apraxia). Hippocampal complex C. dates. This is C. Working are critical for ? . Capacity for digit span forward is typically six numbers. Working Harrison’s 16th Ed. and retrieval (decoding or recall). 2393 hypoglycemia. 698 Working memory lasts for what duration ? 704 More permanent stores of words. Normal capacity is six numbers Harrison’s 16th Ed. < 5 seconds Harrison’s 16th Ed. All of the above A. patient to recall three words after 3 to 5 minutes? Harrison’s 16th Ed. 2394 C. or driving a car. category generation (“name as A. Declarative (explicit) memory 709 The medial temporal lobes. are critical for ? D. All of the above A. B. Semantic memory D. Episodic memory A. It is highly dependent upon working memory. Semantic memory many animals as you can in one minute”). Amgdala C. Episodic is prototypical declarative explicit memory. getting dressed. Procedural memory A. Language Harrison’s 16th Ed. Executive function principles & rules (number of days in a week) ? D. 2394 A. Working 711 Inferolateral temporal lobes are critical for ? N Engl J Med 2005. Entorhinal region D. All of the above C. 2394 Episodic memory requires dorsomedial nucleus of thalamus and medial temporal lobes. Procedural memory skill or procedure like riding a bicycle. Semantic memory ganglia. Episodic A. Tasks requiring patient to recall a long list of words or a series of pictures after a predetermined delay will demonstrate B. Episodic memory consciously retrieved to be remembered. Dorsomedial nucleus of thalamus B. Amygdala automatic and do not require conscious implementation. 2394 In AD the early deficits involve verbal or visual episodic memory. AJAY MATHUR Cardiology 701 Harrison’s 16th Ed. C. 2394 Semantic memory contains unchanging facts. Hippocampus B. Procedural memory Executive function refers to mental activity involved in planning. Medial temporal lobes Procedural (implicit) memory is involved in learning and retaining a skill or procedure such as riding a bicycle. cerebellum. Working memory 718 Which of the following is not a part of the components of medial 712 Prefrontal cortex. Perirhinal region 713 Prefrontal cortex. A. Declarative (explicit) memory deficits. and regulating behavior. D.352:692-9 B. Semantic memory 715 Which kind of memory is involved in remembering facts about C. All of the above N Engl J Med 2005. D. getting dressed. and sensory cortex. Procedural (implicit) memory D. Wernicke’s area are critical for ? N Engl J Med 2005. B. Semantic memory 716 Which kind of memory is involved in learning and retaining a C. B. 2394 A. 3304 C. Working memory Harrison’s 16th Ed. 701 Neurology MCQ’s FORMCQ’s FOR MEDICAL MEDICAL PROFESSIONALS PROFESSIONALS BY PROF.352:692-9 including the entorhinal. Procedural (implicit) memory parahippocampus. and rules (number of days A. Episodic memory in a week). Phonologic working memory D. Injury to anterior temporal neocortex leads to loss of semantic memory. Spatial working memory B. Working memory D. Semantic memory B. Episodic memory 719 In AD the early deficits involve which class of memory ? B. including the hippocampus and C. Procedural memory C. Declarative (explicit) memory to thiamine deficiency ? C. Procedural (implicit) memory Harrison’s 16th Ed. perirhinal. Abilities stored in procedural memory become C. Episodic memory 714 Which memory is involved in remembering unchanging facts. Episodic memory C. visual-association areas are critical for ? Components of medial temporal lobe memory system include hippocampus and adjacent cortex. Broca’s area.352:692-9 Declarative (explicit) memory refers to facts about world & past personal events that must be A. A. Harrison’s 16th Ed. or driving D.352:692-9 temporal lobe memory system ? Harrison’s 16th Ed. and parahippocampal regions. Semantic memory Harrison’s 18th Ed. basal A. N Engl J Med 2005. Working memory a car? Harrison’s 16th Ed. initiating. Remote B. Procedural memory the world and past personal events ? D. associations. All of the above . 2394 Procedural (implicit) memory involves centers outside the hippocampus such as amygdala. Semantic memory 710 Which of the following is damaged in Korsakoff’s syndrome due B. and visuoconstructive ability. C. principles. 2394 D. Basal ganglia 717 Executive memory function is highly dependent upon which of Dorsomedial nucleus of thalamus is damaged in Korsakoff’s syndrome due to thiamine deficiency the following ? resulting in loss of episodic memory. Alzheimer’s disease 726 EEG is helpful in the diagnosis of which of the following ? Harrison’s 18th Ed. Events just before ictus are most vulnerable to dissolution etiology for dementia.351:56-67 724 On neuroimaging studies. Fronto temporal dementia D. DLB. Galantamine D. Schizophrenia the cerebral cortex ? Harrison’s 16th Ed. extensive white matter abnormalities A. Cortical basal degeneration D. FTD. parkinsonism. None of the above Harrison’s 18th Ed. or prion disorders. Alzheimer’s disease A. Pittsburgh Compound-A 732 The most common clinical disorder disrupting semantic memory is ? B. Alzheimer’s disease . Vascular etiology 731 Ribot’s law states that ? D. Rivastigmine A. Gait 730 Which of the following drugs used in the management of dementia disturbance is common in vascular dementia.351:56-67 Harrison’s 18th Ed. Memantine D. All of the above B. 3303 brain amyloid associated with amyloid angiopathy or neuritic plaques of AD. Creutzfeldt-Jakob disease N Engl J Med 2005. Frontotemporal dementia An autosomal dominant family history is found in HD and in familial forms of AD.352:692-9 On neuroimaging studies. Fronto temporal dementia A. A+B (AD). 3305 C. 729 Which of the following drugs used in management of dementia is not a cholinesterase inhibitor ? 723 Early prominent gait disturbance is a feature of ? N Engl J Med 2004. Creutzfeldt-Jakob disease B. Galantamine B. PD/DLB. Donepezil A. Harrison’s 18th Ed. extensive white matter abnormalities correlate best with a vascular A. 3306 A. Donepezil B. Galantamine C. is an early feature of Dementia with Lewy bodies. Vascular dementia Early prominent gait disturbance with only mild memory loss suggests vascular dementia.351:56-67 A. Dementia with Lewy bodies D. Pittsburgh Compound-C A. Nucleus accumbens has been replaced by an impostor). Fronto temporal dementia A. or “periodic complexes”). which of the following is used ? D.702 MCQ’s FOR MEDICAL PROFESSIONALS BY PROF. 2394 C. 3303 A. AJAY MATHUR Neurology 720 An autosomal dominant family history of dementia can be found D. Memantine C. degenerative diseases ? Harrison’s 18th Ed.delusion that a familiar person has been replaced by an impostor. Pittsburgh Compound-B N Engl J Med 2005. Fronto temporal dementia C. 3304 A. delirium. Remote memories are most resistant 725 In amyloid imaging of brain for diagnosis of Alzheimer’s disease C. Alzheimer’s disease D. C. Creutzfeldt-Jakob disease 721 Early presence of visual hallucinations is a feature of which of the Electroencephalogram (EEG) is rarely helpful except in CJD (repetitive bursts of diffuse high- following dementias ? amplitude sharp waves. Creutzfeldt-Jakob disease D. Table 371-4 728 Agents useful for reducing the signs of dementia include ? N Engl J Med 2004. 3304 B. Huntington’s disease B. 3303. Thalamus Diagnosis of DLB is suggested by early presence of visual hallucinations. Alzheimer’s disease C. B. All of the above Capgras syndrome . D. Nucleus basalis of Meynert 722 Capgras syndrome is an early feature in which of the following Nucleus basalis of Meynert is the major source of cholinergic input to the cerebral cortex. A. Alzheimer’s disease B. rapid-eye-movement (REM) sleep disorder or Capgras syndrome (delusion that a familiar person C. Rivastigmine C. Pittsburgh Compound-D in which of the following ? Amyloid imaging with radioligands like Pittsburgh Compound-B (PiB) and 18F-AV-45 help in detecting Harrison’s 18th Ed. Dementia with Lewy bodies C. or normal-pressure hydrocephalus (NPH). Dementia with Lewy bodies B. Donepezil correlate best with ? Harrison’s 18th Ed. Rivastigmine B.352:692-9 C. is a NMDA-receptor antagonist ? N Engl J Med 2004. Globus pallidus D. Creutzfeldt-Jakob disease 727 Which of the following is the major source of cholinergic input to B. 6 . 8 . but the course can range from 1 to 25 years. Johnson C. 3305 suffered from Alzheimer’s disease ? A.5 B. Positive family history C. 703 Neurology MCQ’s FORMCQ’s FOR MEDICAL MEDICAL PROFESSIONALS PROFESSIONALS BY PROF.19 December 1915) in a patient at Frankfurt asylum named Mrs. Olivopontocerebellar degeneration D. Spain A. years (roughly 12% per year). Myoclonic jerks C. 1. vegetables.352:692-9 ? Harrison’s 18th Ed. 3305 Harrison’s 18th Ed.0 C. Old age B. red wine & diabetes. Lyndon B. and superficial conversation may remain Alzheimer’s disease surprisingly intact. resting tremor. 3305 A.5 D. Multiple sclerosis Term mild cognitive impairment (MCI) is applied when memory loss falls 1. Alzheimer’s disease A. routine behavior. Lateral frontal cortex D. very low educational attainment. 4 . 3306 Harrison’s 18th Ed. 2 . Gerald Ford D. diminished serum folic acid.6 years B. Use of NSAID’s reduces risk of AD. low dietary intake of fruits. Ronald Reagan The typical duration of AD is 8 to 10 years. 1864 . Hippocampus A. ~ 50 % D. ~ 30 % B. which of the following remains intact N Engl J Med 2005. 3305 A. Any of the above D. Germany B.0 C. Resting tremor D. All of the above . 740 Which of the following is rare in AD ? 734 Alois Alzheimer belonged to which nation ? Harrison’s 18th Ed. elevated homocysteine & cholesterol levels. social graces. positive family history. Medial temporal lobes B. Huntington’s disease C. Lateral and medial parietal lobes C.3 years A. Superficial conversation D. Hyperactive tendon reflexes B. rhythmic. D. 737 Cognitive changes of AD characteristically begin with ? 743 Risk factors for AD are ? Harrison’s 18th Ed. Female gender D. 3305 A. beginning with memory impairment and with concussion. 1. Richard Nixon B. Social graces B.8 years C. All of the above In AD. 736 What percentage of mild cognitive impairment individuals will progress to Alzheimer’s disease (AD) in 4 years ? 742 Neuroimaging studies in AD may show atrophy in ? Harrison’s 18th Ed. Parkinsons’s disease B. 0. mercury & viruses. female gender. Parkinsons’s disease D. Memory impairment A. Poland C. Language deficit B. AJAY MATHUR Cardiology 703 B. 3305 A. 741 The typical duration of Alzheimer’s disease is ? 735 Which of the following President of United States of America Harrison’s 18th Ed. 3306 Harrison’s 18th Ed. Nucleus basalis of Meynert C. ~ 20 % A. Auguste Deter. ~ 40 % C.5 standard deviations below normal on standardized memory tests. going on to language and visuospatial deficits. as well as lateral and medial parietal lobes and lateral frontal cortex. Delusion Alzheimer’s disease (AD) was first described by German psychiatrist Alois Alzheimer (14 June Patients of AD often look parkinsonian but rarely have a high-amplitude. Sweden D. 3305 A. 2. Progressive supranuclear palsy 733 Disorders affecting procedural memory include all except ? 739 In the middle stages of AD. hypertension. 738 Term mild cognitive impairment (MCI) is applied when memory loss falls how many standard deviations below normal on 744 Most severe pathology in Alzheimer’s disease is found in ? standardized memory tests ? Harrison’s 18th Ed. Temporal cortex B. past history of head trauma Cognitive changes of AD follow a characteristic pattern. All of the above Risk factors for AD are old age.10 years D. neuroimaging studies (CT and MRI) show atrophy is distributed throughout the medial Approximately 50% of patients with mild cognitive impairment (MCI) will progress to AD over 4 temporal lobes. All of the above In middle stages of AD. exposure to aluminum. Visuospatial deficit C. Routine behavior C. 3306 751 Which of the following products of Amyloid precursor protein A. twisted neurofilaments in neuronal cytoplasm that represent abnormally A. A  40 D. 3307 Figure 371-4 NFTs are silver-staining. Found in neuronal cytoplasm transmembrane protein. D.42 amino acids C. A amyloid In AD.  secretase (lateral septum). All of the above B. 3307 Figure 371-4 C. All of the above Harrison’s 18th Ed. Visuospatial deficits A. can lead to A. Cleavage of APP by  and  secretases results in two products i. A  42 747 Which of the following clinical features of Alzheimer’s disease occurs in the later phases of the disease ? Soluble amyloid fibrils (oligomers) lead to dysfunction of neural cells in AD. Apo 4. Derived proteolytically from amyloid precursor protein 755 Neurofibrillary tangles (NFTs) in Alzheimer’s disease is characterized by all except ? D. All of the above B. Misfolded A  42 molecules may be the most toxic form of this protein. Accumulation of oligomers eventually leads N Engl J Med 2004. Harrison’s 18th Ed. increased production B. Amyloid precursor protein (APP) is catabolized by . A. Deterioration of language 753 In Alzheimer’s disease. Amnesic type of memory impairment B. Paired helical & twisted neurofilaments 750 A peptide results from cleavage of amyloid precursor protein C. All of the above A. . the neuritic plaque contains ? C. 3306 A  amyloid is a protein of 39-42 amino acids that is derived proteolytically from a larger A.  746 Which of the following statements is true about microscopic secretase acts on  secretase product to produce nontoxic P3 peptide. Gait disturbances B. A  peptide results from cleavage of APP by  -and -secretases. 2541 C. Similarly. neuritic plaque is surrounded by ? Harrison’s 17th Ed. P3 acetyltransferase and nicotinic cholinergic receptors in cerebral cortex due to degeneration of cholinergic neurons in nucleus basalis of Meynert. 3306 D. A42 D. B. or ischemic infarctions.  secretase . A  36 D. and  secretases. Large lobar hemorrhages 749 Which of the following statements is true about A amyloid in C.  antichymotrypsin. microglia & macrophages. B.  secretase product and  secretase product. temporal cortex & nucleus basalis of Meynert C. B. microscopic examination of brain shows neuritic plaques containing A  amyloid. It is a protein of 39 . termed amyloid angiopathy.349:1056-63 C. in cerebral cortex are decreased ? Harrison’s 18th Ed. neuritic plaque core is surrounded by debris of degenerating neurons. C. Apo 4 C. All of the above levels in brainstem nuclei like locus coeruleus. All of the above Harrison’s 18th Ed. Acetylcholine (APP) is neurotoxic ? B. 3306 D.704 MCQ’s FOR MEDICAL PROFESSIONALS BY PROF. Debris of degenerating neurons D. All of the above 745 In Alzheimer’s disease.351:56-67 to formation of neuritic plaques. All of the above A. the last being a major source of cholinergic input to cerebral cortex. Neuritic plaques contain A amyloid 752 In familial early-onset Alzheimer’s disease. AJAY MATHUR Neurology Most severe pathology is found in hippocampus. Ischemic infarctions Alzheimer’s disease ? D. Excess production of A42 findings in Alzheimer’s disease ? is a key initiator of cellular damage in Alzheimer’s disease. B.  secretase acts on secretase product to produce nontoxic A40 and toxic A42 peptide. its synthetic enzyme choline C. Serotonin A. silver- staining neurofibrillary tangles (NFTs) in neuronal cytoplasm. 3306 A. proteoglycans. 3306 Accumulation of A  amyloid in cerebral arterioles. B. 3307. which of the following neurotransmitters APP is a membrane-spanning protein that is processed A  amyloid which is deposited in neuritic plaques.  antichymotrypsin 754 In Alzheimer’s disease. Silver staining neurofibrillary tangles in cytoplasm of which peptide is most neurotoxic ? Harrison’s 18th Ed. Microhemorrhages Neuritic plaques contain a central core that includes A amyloid. Neuritic plaques contains A amyloid microhemorrhages.e. Norepinephrine Harrison’s 18th Ed. and accumulation of A  amyloid in arterial walls of cerebral blood vessels.  secretase phosphorylated tau protein and appear as paired helical filaments by electron microscopy. A40 AD is associated with decreased levels of acetylcholine. A  38 In AD. N Engl J Med 2003. amyloid precursor protein (APP). Microglia 748 In Alzheimer’s disease. Macrophages Harrison’s 18th Ed. amyloid angiopathy may lead to ? Harrison’s 18th Ed. Gold staining Harrison’s 18th Ed. large lobar hemorrhages. Represent abnormally phosphorylated tau protein (APP) by ? D. A amyloid in arterial walls of cerebral blood vessels A. There is also reduction in norepinephrine D. Apo  4 chromosome 1 and encodes a protein called STM2 (seven transmembrane domains). None of the above C. Presenilin-2 (PS-2) Presenilin-1 (PS-1) gene implicated in the pathogenesis of Alzheimer’s disease is present on D. AJAY MATHUR Cardiology 705 756 Tau protein is characterized by all except ? S182 and STM2 are cytoplasmic neuronal proteins. 2 B. A ‘hyperphosphorylated’ state of tau impairs its capacity to bind to microtubules. 705 Neurology MCQ’s FORMCQ’s FOR MEDICAL MEDICAL PROFESSIONALS PROFESSIONALS BY PROF. 3 C. PSP and Cortical Basal Degeneration (CBD). 3307 B. D. 3307 is protective ? A. Apo  gene chromosome 14 and encodes a protein called S182. Presenilin-1 (PS-1) D. All of the above Presenilin-2 (PS-2) gene implicated in the pathogenesis of Alzheimer’s disease is present on Apo  4 allele has a strong association with sporadic and late-onset familial cases of AD. Chromosome 9 Harrison’s 18th Ed. highly penetrant fashion. A. N Engl J Med 2001. which allelic variant of apolipoprotein  Harrison’s 18th Ed. D. 3306 are homologous to a cell-trafficking protein . 3307 D. All of the above C. Functions to assemble and stabilize microtubules Familial Alzheimer’s disease (FAD) ? C. Chromosome 21 C. Progressive supranuclear palsy B. 4 . 761 Which of the following about S182 & STM2 is false ? 767 For Alzheimer’s disease. Widely expressed throughout nervous system A. Presenilin-1 (PS-1) Tau () is a microtubule-associated protein that assembles & stabilizes microtubules that convey C. 3307 familial & sporadic forms of Alzheimer’s disease ? A. 3307 B. 760 Presenilin-2 (PS-2) gene implicated in the pathogenesis of 766 Age of onset of sporadic & familial forms of AD is modulated by Alzheimer’s disease is present on which chromosome ? which allelic variants of apolipoprotein  ? Harrison’s 18th Ed. Presenilin-2 (PS-2) cell organelles. Alzheimer’s disease is present on which chromosome ? Harrison’s 18th Ed. Chromosome 21 758 APP gene implicated in the pathogenesis of Alzheimer’s disease Apo  gene implicated in the pathogenesis of Alzheimer’s disease is present on chromosome 19. it is hypophosphorylated Harrison’s 18th Ed. Chromosome 14 B. Chromosome 1 A. 4 D. Presenilin-1 (PS-1). Presenilin-1 (PS-1) & Presenilin-2 (PS-2) D. 3307 764 Gene implicated in pathogenesis of Alzheimer’s disease is ? A. 3306 763 Apo  gene implicated in the pathogenesis of Alzheimer’s disease A. All of the above AD genes include APP. disorders ? Harrison’s 18th Ed. Homologous to cell-trafficking protein . widely expressed in the nervous system and Harrison’s 18th Ed. Chromosome 18 B. It is a microtubule associated protein 762 Mutations in which of the following gene causes early-onset B. Chromosome 11 A.344:1516 A. Chromosome 9 Harrison’s 18th Ed. Progressive supranuclear palsy may represent a tau A. APP pathologic process B. 3307. Chromosome 14 C.sel 12 B. Chromosome 19 Tau plays a major role in the pathogenesis of FTD. Chromosome 21 D. 3307 B. APP C. Frontotemporal dementia A. 3307 Harrison’s 18th Ed. 3 D. Chromosome 11 A. glycoproteins through neuron. APP C. Chromosome 1 C. D. Chromosome 11 D. remains the single most important biologic marker associated with risk for AD. Apo  gene Mutations in Presenilin-1 (PS-1) gene cause an early-onset AD (onset before age 60) transmitted 757 Tau protein has pathological significance in which of the following in an autosomal dominant. 3307 B. In Alzheimer’s disease. Cytoplasmic neuronal proteins Harrison’s 18th Ed. is present on which chromosome ? Harrison’s 18th Ed. Apo  gene APP gene implicated in the pathogenesis of Alzheimer’s disease is present on chromosome 21.sel 12. Chromosome 21 C. 2 C. Chromosome 11 B. 759 Presenilin-1 (PS-1) gene implicated in the pathogenesis of Alzheimer’s disease is present on which chromosome ? 765 Mutations in which gene is the most common cause of late-onset Harrison’s 18th Ed. Presenilin-2 (PS-2) and Apo  gene. Apo  gene on chromosome 19 leads to late onset familial and sporadic forms of AD. donepezil. AD 775 Which of the following drugs is contraindicated in patients with B. Rivastigmine There is evidence that the Apo 2 allele maybe “protective” against Alzheimer’s disease. D. Rivastigmine 779 EEG studies are most useful for the diagnosis of ? N Engl J Med 2001.349:1056-63 (PICALM) A. B. Galantamine MRI are unique diagnostic features of CJD. Rivastigmine D. FTD Parkinson’s disease ? C. Tacrine Tarenflurbil and Semagacestat are gamma secretase inhibitors. Donepezil C. Vaccines 772 Which of the underementioned drug for Alzheimer’s disease is a C. and galantamine is inhibition of cholinesterase. All of the above Tacrine (tetrahydroaminoacridine). hepatotoxicity ? Harrison’s 18th Ed. Alzheimer’s disease ? N Engl J Med 2003. Their use is being explored in AD treatment with a hypothesis that they would diminish the production of A 42. Alzheimer’s disease Pharmacologic action of donepezil. 3308 with cholinesterase inhibitors and is not approved for mild AD. Galantamine C. Rivastigmine A. Tacrine B. Turner syndrome D. Clusterin (CLU) 776 Which of the following drugs is associated with an increased risk B. Klienfelter syndrome C. DLB N Engl J Med 2003.706 MCQ’s FOR MEDICAL PROFESSIONALS BY PROF. rivastigmine. Rivastigmine Recent genome-wide association studies have implicated the clusterin (CLU). APO 3 A. Estrogen analogues hallmarks of AD if they survive beyond age 40. CLU may play a role in synapse turnover. 3308 Adults with trisomy 21 (Down’s syndrome) consistently develop the typical neuropathologic A. All of the above C. 3308 771 Adults with which of the following consistently develop typical neuropathologic hallmarks of AD ? A. B. Gamma secretase inhibitors cholinesterase inhibitor ? Harrison’s 18th Ed. APO 2 channels ? Harrison’s 18th Ed.349:1056-63 D. AJAY MATHUR Neurology D. 3308 A. Donepezil Harrison’s 18th Ed. rivastigmine. Down’s syndrome 778 Tarenflurbil and Semagacestat are ? D. Donepezil C. B. All of the above Harrison’s 18th Ed. Parkinson’s disease cholinesterase inhibitor ? D. A. Donepezil C.344:1516. Galantamine Apo  4 allele is not associated with FTD. APO 4 B. Complement component (3b/4b) receptor 1 (CR1) B. PICALM participates in clathrin-mediated endocytosis & CR1 may be involved in amyloid clearance through complement 777 Which of the following drug has been discarded due to pathway. It is used along Harrison’s 18th Ed. All of the above C. Risperidone genes in AD pathogenesis. None of the above C. Haloperidol Harrison’s 18th Ed. Phosphatidylinositol-binding clathrin assembly protein of stroke ? N Engl J Med 2003. 3308 Abnormal periodic EEG discharges & cortical / basal ganglia abnormalities on diffusion-weighted A. Harrison’s 17th Ed. Frontotemporal dementia 773 Which of the underementioned drug for Alzheimer’s disease is a C. All of the above A. 3308 B. or CJD. Rivastigmine 770 Which of the following gene is implicated in AD pathogenesis ? D. Donepezil .349:1056-63 774 Which of the following acts on N-methyl-D-aspartate (NMDA) A. Galantamine C. B. D. CJD A. 3307 B. and complement component (3b/4b) receptor 1 (CR1) D. Memantine Apo  4 allele is an important risk factor for AD. B. HMG-CoA reductase inhibitors A. and galantamine act by inhibition of 768 Carrying of which APO  allele nearly doubles the lifetime risk of cholinesterase leading to increase cerebral Ach levels.Jakob disease Harrison’s 18th Ed. Donepezil D. Galantamine 769 Apo 4 allele associated with which of the following diseases ? Memantine acts by blocking overexcited N-methyl-D-aspartate (NMDA) channels. Creutzfeldt . 3308 D. DLB. phosphatidylinositol- binding clathrin assembly protein (PICALM). 2542 D. TAR divided protein 43 Harrison’s 18th Ed. B. TAR dense protein 43 B. D. 3310 D. HD C. Seizure protein (chromosome 9) and charged multivesicular body protein 2b (CHMP2b) genes (chromosome 3) also lead to rare autosomal dominant forms of familial FTD. Astrocytic plaque White matter diseases that also present with dementia include adult metachromatic leukodystrophy B. Britain 787 Primary progressive aphasia is related to ? C. A. Symmetrical axial rigidity The most common autosomal dominantly inherited mutations causing FTD involve the MAPT C. All of the above 784 In FTD. PSP begins with falls and vertical supranuclear gaze paresis or nonfluent aphasia and progresses to symmetrical rigidity and dementia. Behavioral symptoms predominate in early stages A. mutations occur in which of the following genes ? Progressive supranuclear palsy (PSP) is a degenerative disease that involves the brainstem. TAR depleted protein 43 A. 3310 D. Marked atrophy of temporal and/or frontal lobes single entity. Basal ganglia FTD begins in V to VII decades. behavioral C. Limbic structures symptoms predominate in early stages of FTD. progressive multifocal leukoencephalopathy (JC virus infection) & C. 3311 C. insular. Mutations in valosin-containing D. All of the above A. Cholinergic system is relatively spared Term Binswanger’s disease should be used with caution. Unlike AD. GRN A. D. and/or temporal cortex. 3310 basal ganglia. FTD . North America A. Begins in fifth to seventh decade 789 Progressive supranuclear palsy (PSP) involves which of the following ? B. Distinguishing anatomic hallmark of Harrison’s 18th Ed. Motor neuron disease Harrison’s 18th Ed. inability to look down. 707 Neurology MCQ’s FORMCQ’s FOR MEDICAL MEDICAL PROFESSIONALS PROFESSIONALS BY PROF. White skin tags B. PSP C. CBD 782 Which of the following white matter diseases also present with dementia ? D. MAPT Harrison’s 18th Ed. D. More common in men than women Harrison’s 18th Ed. 3309 The left-hemisphere presentation of FTD is called primary progressive aphasia with nonfluent and semantic variants. 3309 TDP-43 stands for TAR DNA binding protein 43. 3309 FTD is a focal atrophy of frontal. All of the above B. 3310 A. 785 TDP-43 stands for ? Harrison’s 18th Ed. All of the above Pick’s disease is characterized by selective involvement of anterior frontal & temporal neocortex 783 Which of the following statements about Frontotemporal dementia and pathologically by intraneuronal cytoplasmic inclusions (Pick bodies) which are argyrophilic. Dementia (causing loss of function in tau molecule) or GRN genes (altering progranulin protein gene). Brainstem D. A. 3311 C. Intraneuronal cytoplasmic inclusions (arylsulfatase A deficiency). Frequent unexplained and sometimes spectacular falls are common secondary to a combination of axial rigidity. Memory and visuospatial skills relatively spared Leukoaraiosis or subcortical arteriosclerotic leukoencephalopathy or Binswanger’s disease is a diffuse white matter disease which presents with dementia with lacunar infarctions found on MRI. Harrison’s 18th Ed. limbic structures. Cholinergic system is spared in FTD. TAR DNA binding protein 43 Harrison’s 18th Ed. (FTD) is false ? staining positively with the Bielschowsky silver method and also with immunostaining for tau. both on chromosome 17 and both associated with parkinsonian features. Spain Harrison’s 18th Ed. 3301 791 Globose tangles best relate to ? A. Ballooned neurons cerebral autosomal dominant arteriopathy with subcortical infarcts & leukoencephalopathy (CADASIL). It is more common in men than women. Asia B. AJAY MATHUR Cardiology 707 780 “Leukoaraiosis” refers to ? D. None of the above 781 Cerebral vascular disease is a common cause of dementia in ? In FTD. White lines around umblicus 786 Which of the following statements about frontotemporal dementia C. Valosin 790 PSP begins with which of the following ? B. because it does not clearly identify a C. Diffuse white matter disease is false ? Harrison’s 18th Ed. and bad judgment. B. 3311 D. White nails A. FTD Cerebral vascular disease is a more common cause of dementia in Asia than in Europe & North America. 3311 B. Adult metachromatic leukodystrophy B. CADASIL Harrison’s 18th Ed. Frequent falls D. Harrison’s 18th Ed. A. Progressive multifocal leukoencephalopathy 788 Which of the following best relates to Pick bodies ? C. None of the above B. A. memory & visuospatial skills are relatively spared. and selected areas of cortex. Australia B. falls and REM sleep behavior disorder. Striatum B. degenerative brain disorder characterized by chorea. 3312 B. Cell membrane protein A. causal gene and an accurate genetic test for HD is available. which of the following is 796 In DLB. Nuclear membrane protein D. Hippocampus D. AD Huntingtin gene (HTT) encodes the highly conserved cytoplasmic protein huntingtin which is present in all neurons. Great Britain 792 “Alien hand” phenomenon is a feature of ? Harrison’s 18th Ed. 799 Huntington’s disease (HD) is caused by mutations in the Huntington’s gene on the ? 793 Glial plaques with tau inclusions are pathognomonic of ? Harrison’s 18th Ed. Chorea D. 3312 Lewy Bodies (DLB) ? Harrison’s 18th Ed. Huntington’s disease (HD) B. Auditory hallucinations HD is an autosomal dominant. 800 “Huntingtin” is a ? 794 Which of the following is rarely familial ? Harrison’s 16th Ed. 3312 Harrison’s 17th Ed. In FTD. DLB is characterized by visual hallucinations. FTD C. Mamillary bodies Neuropathology of HD consists of widespread cerebral atrophy with prominent involvement of Key neuropathologic feature of DLB is the presence of Lewy bodies throughout the cortex. Dementia C. D. Huntington’s gene is single. an American physician in 1872. CBD Huntington’s disease (HD) was described by George Huntington. amygdala. Amygdala B. Substantia nigra C. 798 George Huntington belonged to which country ? A. 802 In Huntington’s disease (HD). Cerebellum C. CBD D. behavioral disturbance & frontal executive disorder in IV or V decade. All of the above D. Memory is frequently not impaired. Glial plaques with tau inclusions are pathognomonic of CBD. 3312 A. Dementia with Lewy Bodies (DLB) A. parkinsonism. 3312 hydrocephalus (NPH) ? Harrison’s 18th Ed. Lewy bodies are found in ? predominantly atrophied ? Harrison’s 18th Ed. Long arm of chromosome 5 D. Creutzfeldt-Jakob disease (CJD) D. there occurs accumulation of hyperphosphorylated tau within neurons and glia called globose tangles. 797 Which of the following about Lewy bodies is false ? 803 Which of the following is not a feature of normal-pressure Harrison’s 18th Ed. PSP B. Nuclear protein C. Seizure B. DLB D. 3312 A. Cerebral cortex A. Abnormal gait B. AD In cortical basal degeneration (CBD). CBD is rarely familial. AD HD is caused by mutations in the Huntington’s gene on the short arm of chromosome 4. CBD D. Urinary incontinence . Stain with periodic acid–Schiff (PAS) & ubiquitin B. America A. 3312 Harrison’s 18th Ed.20 nm long with surrounding amorphous D. apraxia of one arm and hand is called the alien hand. cingulated cortex. Short arm of chromosome 4 A. They are composed of straight neurofilaments 7 . fluctuating alertness. 2415 Harrison’s 18th Ed. and substantia nigra. Symmetrical axial rigidity C. Intraneuronal cytoplasmic inclusions A. CBD Lewy bodies are intraneuronal cytoplasmic inclusions that stain with periodic acid-Schiff (PAS) & ubiquitin. 2545 A. 795 Which of the following is a characteristic feature of Dementia with 801 Chorea is a characteristic feature of which of the following ? Harrison’s 18th Ed. AJAY MATHUR Neurology C. Visual hallucinations D. Cytoplasmic protein B. HD B. Cortical basal degeneration (CBD) C. 3312 A. Long arm of chromosome 4 B. striatum and cerebral cortex. Composed of straight neurofilaments 7 to 20 nm long C. Short arm of chromosome 5 C. 3312 A. AD material. PSP C. DLB C. Canada C. None of the above D.708 MCQ’s FOR MEDICAL PROFESSIONALS BY PROF. along (Korsakoff’s psychosis) or even death. Isolated vasculitis of CNS . Posterior columns A. Thiamine (vitamin B 1) deficiency damages thalamus. Caudate nucleus A. Elevated plasma levels of A amyloid D. 3312 810 Thiamine deficiency damages which of the following ? A. Mammillary bodies D. Italian C. Corpus callosum C. FTD A. 3313 Harrison’s 18th Ed. favors NPH. Periaqueductal grey matter of midbrain of corpus callosum. corticospinal tracts. Peripheral nerves C. Midline cerebellum In NPH. Thalamus B. Dilation of the lateral ventricle C. Elevation of the floor of third ventricle A. & diplopia due to ophthalmoplegia. Stretching of corpus callosum Harrison’s 18th Ed. damage occurs to ? 807 Dementia & seizures in Marchiafava-Bignami disease is related to ? Harrison’s 18th Ed. neurologic damage is related to ? Harrison’s 18th Ed. Primary neoplasms of CNS A. Short strides D. 813 In Vitamin B12 deficiency. Enlargement of the aqueduct B. Midline cerebellum MRI feature of normal-pressure hydrocephalus (NPH) include dilation of lateral ventricle. 808 Which of the following degenerates in Marchiafava-Bignami disease ? 814 In Vitamin B12 deficiency. All of the above D. 805 Which of the following is not a MRI feature of Normal-pressure hydrocephalus ? 811 In Thiamine deficiency. 3313 Figure 371-10 correlates most closely with memory loss ? A. Substantia nigra D. neuroimaging shows enlarged lateral ventricles (hydrocephalus) with little or no cortical D. 3313 A. 3313 Harrison’s 18th Ed. Deficiency of S-adenosyl methionine C. NPH B. damage occur to posterior columns. Thalamus C. All of the above In Vitamin B 12 deficiency. ataxia. Paraneoplastic syndrome D. Prolonged untreated thiamine deficiency A. Corticospinal tracts B. Diplopia mild to moderate dementia and urinary incontinence. External hip rotation B. Drinkers of red wine D. periaqueductal grey matter of midbrain. All of the above D. Metastatic neoplasms of CNS B. AJAY MATHUR Cardiology 709 Clinical triad of normal-pressure hydrocephalus (NPH) includes abnormal gait (ataxic or apractic). Mammillary bodies D. 709 Neurology MCQ’s FORMCQ’s FOR MEDICAL MEDICAL PROFESSIONALS PROFESSIONALS BY PROF. mammillary bodies. Seizure C. 806 Which of the following is a feature of gait abnormality in 812 Which of the following is related to Korsakoff’s psychosis ? NPH ? Harrison’s 18th Ed. Clinical presentation of Wernicke’s encephalopathy include confusion. damage to dorsomedial thalamus correlates most closely with memory loss. 3313 Harrison’s 18th Ed. 3313 B. CJD C. depression of the floor of third ventricle. 3313 A. 3314 Harrison’s 18th Ed. All of the above Prolonged untreated thiamine deficiency results in irreversible dementia / amnestic syndrome Characteristic “magnetic” gait with external hip rotation. low foot clearance and short strides. Inhibition of -aminolevulinic acid dehydrase B. AD Harrison’s 18th Ed. 3313 B. 809 Which of the following is not related to Wernicke’s 815 Limbic encephalitis best relates to ? encephalopathy ? Harrison’s 18th Ed. 3313 A. Ataxia C. peripheral nerves and trochlear & abducens nuclei. with prominent truncal sway or instability. damage to which of the following Harrison’s 18th Ed. Mechanism of neurologic damage in Vitamin B12 deficiency is related to deficiency of S-adenosyl methionine which is required for methylation of myelin phospholipids. Dorsomedial thalamus C. Amnestic syndrome C. peripheral nerves and cerebral myelinated fibers (dementia). All of the above atrophy with a patent aqueduct of Sylvius suggesting communicating hydrocephalus. All of the above Marchiafava-Bignami disease consists of dementia and seizures with degeneration of corpus callosum in male Italian drinkers of red wine. In thiamine deficiency. Low foot clearance C. Confusion B. midline cerebellum. Male B. Irreversible dementia B. stretching D. 804 Cortical atrophy is not a feature of ? Harrison’s 18th Ed. enlargement of the patent aqueduct (communicating hydrocephalus). 3313 A. D. 3314 Other Movement Disorders A. D. Globus pallidus 819 “Dementia pugilistica” is related to which of the following ? D. C. D. All of the above A.339:1049 A. and tremulousness that may progress to choreoathetosis. Several preparations containing Mucuna pruriens (Kapikachu) were descrided for treatment of patients with Kampavata. Aluminum poisoning described in Ayureda (Charaka Samhita) centuries ago. poor memory. Chronic arsenic intoxication compound present in Mucuna pruriens is levodopa. Subthalamic nucleus common and associated with severe and generalized EEG changes. Chronic mercury poisoning B. 3317 D. postural instability. Rest tremor Transient global amnesia is characterized by sudden onset of a severe episodic memory deficit B. and. Alzheimer’s disease (AD) Recurrent head trauma in professional boxers may lead to dementia. later. Substantia nigra Harrison’s 18th Ed. Speech arrest and myoclonic jerks are B. in persons over age 50 years. 2548 Cardinal features of PD include rest tremor. Gait impairment following disease ? Harrison’s 17th Ed. The major active C. agitation. ataxia. A. Chronic lead poisoning N Engl J Med 1998. Corticobasal degeneration (CBD) is termed limbic encephalitis in which confusion. Chronic lead poisoning Principles of Neurology. sleep dysfunction. B. Recurrent head trauma in professional boxers A. Nondopaminergic features (as they do not fully respond to dopaminergic therapy) include freezing A. Working B. seizures. Aluminum poisoning 822 Paralysis agitans was described in Ayureda (Charaka Samhita) C. C. Jack Harrison’s 18th Ed. 3314 D. Nonconvulsive seizure disorder nucleus in the “basal ganglia”. Chronic arsenic intoxication centuries ago by the name of ? D. 3315 Parkinson’s disease (PD) is the second commonest neurodegenerative disease. Chronic lead poisoning 823 The first name of Parkinson was ? Mees’ lines are transverse white lines of the fingernails seen in chronic arsenic intoxication. speech difficulty.Parkinson’s Disease (PD) and Harrison’s 18th Ed. called the “punch drunk” B. lethargy and stupor. rigidity. George A. Kampavata C. myoclonus is a feature of which of the D. Bradykinesia 821 Along with dementia. Jatharagni 817 Mees’ lines are best related to ? Harrison’s 18th Ed.710 MCQ’s FOR MEDICAL PROFESSIONALS BY PROF. emotional changes. AJAY MATHUR Neurology A paraneoplastic syndrome of dementia associated with occult carcinoma (small cell lung cancer) C. C. agitation. Claustrum memory loss. Parkinson’s disease (PD) syndrome or dementia pugilistica. Masked facies in the setting of an emotional stimulus or physical exertion. Adams RD. 3317 Table 372-1 D. 2538. Creutzfeldt . B. 3314 Corpus Luysii is the name given to subthalamic nucleus (STN) which is the only glutamatergic A. and dementia. cognitive impairment. Wilson’s disease (WD) 820 In Transient global amnesia (TGA). Chronic mercury poisoning B.Jakob disease . 3314 D. All of the above 816 Which of the following chronic metal exposure can lead to choreoathetosis ? 372 . Chronic metal exposure Harrison’s 18th Ed. Chronic mercury poisoning Paralysis agitans described by James Parkinson is an independent description of Kampavata (Kampa:tremor. Aluminum poisoning Parkinson disease was first described by James Parkinson in 1817. Episodic 826 Which of the following is not included in the cardinal features of PD ? C. peripheral neuropathy. 57 Dialysis dementia syndrome due to aluminum overexposure is associated with confusion. 5th Ed. B. C. and frank dementia may occur. A. James myoclonic jerks ? C. SCA 3 (spinocerebellar ataxia) types is lost ? Harrison’s 18th Ed. sensory alterations. exceeded only by Alzheimer’s disease (AD). Remote Harrison’s 18th Ed. Transient global amnesia (TGA) 825 Which of the following is commonest neurodegenerative disease ? C. mood disorders. A. Chronic arsenic intoxication 824 Corpus Luysii is the name given to ? D. Vata:metabolic derangement predisposing to neurologic and mental diseases) B. Neuronal cerebrolipofuscinoses of gait. John 818 Which of the following chronic metal exposure can lead to B. bradykinesia and gait impairment. Adhyashana Chronic mercury poisoning produces dementia. which of the following memory D. aphasia. Shalmali A. autonomic disturbances. K. Decreased traffic in indirect striato-pallidal pathway B. A. and intracytoplasmic Harrison’s 18th Ed. brain diagnosis criteria C. metabolic imaging (PET/SPECT) shows which of the Pathologically. U. particularly in posterior putamen. AJAY MATHUR Cardiology 711 827 Non-motor features of PD include all except ? C. Subthalamic nucleus (STN) . spinal cord. Increased traffic in direct striato-nigral pathway 837 Diagnosis of atypical Parkinsonism is favoured by all except ? C. and 835 Commonest cause of familial PD is due to mutations of ? neurons of the olfactory system. Peripheral autonomic nervous system D. 3317 C. Symptom complex of Parkinsonism reflect damage to different components of basal ganglia. and peripheral autonomic Harrison’s 18th Ed. which LRRK2 mutation is the commonest ? In PD. Increased activity in GPi Harrison’s 18th Ed. loss of smell (anosmia) and disturbances of autonomic function. LRRK3 gene A. 832 Which of the following is not a component of basal ganglia ? Pathologically. Gly2018Ser 831 Which of the following is mostly responsible for signs of C. All of the above 834 In PD. Because striatal outflow is inhibitory to the thalamus (GABA).K. pathology begins in peripheral autonomic nervous system. sleep disturbances. 3317 A. Olfactory system Mutations of LRRK2 gene are the commonest cause of familial PD and are responsible for ~1% of C.K. brain bank criteria is the revised criteria for the clinical diagnosis of PD. Harrison’s 18th Ed. A. Raphe nuclei of the brainstem photon emission computed tomography (SPECT) shows reduced uptake of striatal dopaminergic markers. and an aggressive clinical course. 3317 thalamus network shows decreased activity in GPi with increased activity in thalamus. Globus pallidus pars externa (GPe) Harrison’s 18th Ed. brain support criteria B. A. Early gait impairment Striatal dopamine denervation results in increased traffic in indirect striato-pallidal pathway and decreased traffic in direct pathway. Locus coeruleus (LC) D. with Gly2019Ser being the commonest. Loss of smell 833 Which of the following is the revised criteria for the clinical Non-motor aspects of PD include depression and anxiety. brain store criteria Harrison’s 18th Ed. LRRK1 gene 830 Pathological changes in PD begin in ? B. Cognitive impairment nucleus (STN). 3317. U. increased activity in the GPi with decreased activity D. Absence of rest tremor striatal outflow stemming from the increased activity of STN and ultimately GPi/SNr neurons. 3317 proteinaceous inclusions known as Lewy bodies. D. Cholinergic neurons are present in nucleus basalis of Meynert (NBM). neurodegeneration occurs without Lewy bodies. Figure 366-5 Six different LRRK2 mutations have been linked to PD. Table 372-1 D. subthalamic B. A. globus pallidus pars externa (GPe). Reduced uptake in posterior putamen 829 Cholinergic neurons are present in ? B.K. All of the above 836 As a cause of PD. Gly2019Ser parkinsonism ? D. globus pallidus pars interna (GPi) & SNc. 3321 nucleus of vagus nerve in lower brainstem. cerebral hemispheres. reduced striatal dopamine. there is a decrease in the ability of D. B. U. no asymmetry. Sleep disturbances Basal ganglia are subcortical nuclei that include striatum (putamen & caudate nucleus). Decreased ability of thalamus to activate frontal cortex Harrison’s 18th Ed. Claustrum A. Decreased activity in thalamus A. cognitive impairment. dopamine neurons are affected in midstage A. 3318 nervous system. poor or no response to levodopa. Asymmetry thalamus to activate frontal cortex leading to signs of parkinsonism. reverse of what is seen in PD. Gly2020Ser Harrison’s 17th Ed. LRRK4 gene B. Dorsal motor nucleus of vagus nerve in lower brainstem typical sporadic cases of the disease. 3317 A. Degeneration of dopaminergic neurons in SNc C. brain bank criteria 828 Pathological hallmark feature of PD is ? B. 2557. 3317 C. All of the above A. Reduced striatal dopamine D. Thus. the hallmark features of PD are degeneration of dopaminergic neurons in the following ? substantia nigra pars compacta (SNc). olfactory system & dorsal motor Harrison’s 18th Ed. LRRK2 gene Harrison’s 18th Ed. D. U. Atypical Parkinsonism is characterized by early speech and gait impairment. All of the above in thalamus. Norepinephrine neurons are present in locus coeruleus (LC) and serotonin neurons in raphe nuclei of the brainstem. Nucleus basalis of Meynert (NBM) Imaging of brain dopamine system in PD with positron emission tomography (PET) or single- C. 711 Neurology MCQ’s FORMCQ’s FOR MEDICAL MEDICAL PROFESSIONALS PROFESSIONALS BY PROF. 3318 D. Early speech impairment B. Metabolic imaging of basal ganglia/ Harrison’s 18th Ed. Downstream consequence of this is increased activity in C.K. Intracytoplasmic proteinaceous inclusions (Lewy bodies) U. diagnosis of PD ? sensory abnormalities and pain. Loss of taste D. and then spreads in a sequential manner to affect the upper brainstem and cerebral hemispheres. C. All of the above B. Gly2017Ser disease. Striatum (putamen & caudate nucleus) B. absence of rest tremor. for ? Harrison’s 18th Ed. MSA A. 3319 846 Factors linked to a reduced incidence of PD include all except ? Harrison’s 18th Ed. Coffee drinking B. 80s D. 70s C. Corticobasal ganglionic degeneration C. PSP B. 3318. MSA A. Table 372-2 C.6 tetrahydropyridine (MPTP) D. 3319 A. Drugs Multiple-system atrophy (MSA) manifests as a combination of parkinsonian. Cerebrovascular disease D. Tau A. UCHL-1 C. consumption of well water. causes a PD-like syndrome in addicts. 60s B. 2550 A. PSP B. exposure to pesticides.6-tetrahydropyridine) is a byproduct of the illicit manufacture of a heroin-like drug.2. Alcohol use Pathologically.2. Rural living D. and estrogen replacement in postmenopausal women. autonomic features. and the course of the illness ranges 840 Glial cytoplasmic inclusions (GCIs) characteristic of MSA stain between 10 and 25 years.712 MCQ’s FOR MEDICAL PROFESSIONALS BY PROF. Apamin B. SNc B. All of the above D. MSA is characterized by degeneration of SNc.4. Wilson’s disease 849 Toxins that can produce secondary Parkinsonism include all except ? B. 50s A. MPTP toxicity can be prevented by coadministration of a MAO-B inhibitor that Harrison’s 18th Ed.5. 2549 Harrison’s 18th Ed. dopa. Cerebellar features B. and rural living. 3319 848 Which of the following neurotoxin produces parkinsonism ? Harrison’s 18th Ed. Autonomic features D. 845 Peak age of onset of Sporodic Parkinson’s disease (PD) is in which decade of life ? 839 Degeneration of which of the following is found in MSA ? Harrison’s 17th Ed. 3320 A. 3320. Striatum C. head injury. term MSA-p is used and when cerebellar features are predominant. 841 “Hot cross buns” sign relates to which of the following ? 847 Environmental risk factors for PD include all except ? Harrison’s 18th Ed. a mitochondrial toxin that is selectively taken up by. Cyanide responsive dystonia and pantothenate kinase (PANK)–associated neurodegeneration (formerly known as Hallervorden-Spatz disease). Sensory features C. Exposure to pesticides B. 3320 blocks its conversion to the toxic pyridinium ion MPP +. When parkinsonian features are predominant. and damages. AJAY MATHUR Neurology 838 Manifestations of Multiple-system atrophy (MSA) include all except ? 844 Which of the following is the cause of PD in majority ? Harrison’s 18th Ed. Sporadic B. 3320 A. 843 Parkinsonism is a feature of ? dopamine neurons. Inferior olivary nuclei D. cerebellum & inferior olivary nuclei coupled with characteristic glial cytoplasmic inclusions (GCIs) that stain for alpha. Manganese Parkinsonism is a feature of Wilson’s disease. All of the above D. Urban living The pontine “hot cross buns” sign is a MRI feature of MSA-c. Huntington’s disease Harrison’s 18th Ed. Risk factors include a positive family history. striatum. MPTP is transported to CNS. All of the above Peak age of onset of PD is in the 60s (range is 35 to 85 years). MPTP (1-methyl-4-phenyl-1. MPTP D. Corticobasal ganglionic degeneration C. Botulinum C. Huntington’s disease (Westphal variant). 842 “Hummingbird sign” relates to which of the following ? Harrison’s 18th Ed. smoking. 3320 A. A. where it is metabolized to form MPP +. Parkin D. and Most PD cases occur sporadically (85–90%) and are of unknown cause. Consumption of well water C. Use of NSAIDs D. Smoking C. D. Alpha-synuclein B. cerebellar. C. Harrison’s 17th Ed. 3319 Harrison’s 18th Ed. Carbon dioxide . Genetic C. Factors linked to a reduced incidence of PD include coffee drinking. 1-Methyl-1. male gender. anti-inflammatory drugs. Parkinsonian features A. Hallervorden-Spatz disease A. term MSA-c is used. All of the above B. Tetanospasmin The “hummingbird sign” on midsagittal MRI images is a feature of progressive supranuclear palsy (PSP) and is due to atrophy of midbrain with relative preservation of pons. use of nonsteroidal synuclein. 3318 Harrison’s 18th Ed. striatum Drugs that can produce secondary PD include neuroleptics (typical antipsychotics). PARK 2 A.thalamus PARK10 is a late-onset PD susceptibility gene. alpha methyldopa. PARK 2 B. PARK2 encodes parkin Drugs that can cause secondary parkinsonism include metoclopramide. 3322. Figure 372-5 like early age of onset & rapid progression of symptoms. dopamine-depleting agents (reserpine. Cyanide. parkin. Table 366-2 A. Striatum . Ethanol C. Methanol B.subthalamic nucleus D.subthalamic nucleus 856 Which of the following gene leads to an autosomal recessive B. Globus pallidus externa . All of the above . 3320 Table 372-4 Harrison’s 18th Ed. lithium carbonate. Valproic acid pathways ? Harrison’s 18th Ed. PARK4 & PARK5 genes lead to an autosomal dominant form of PD with atypical features Harrison’s 18th Ed. Fluoxetine B. and lithium. mutation leading to its abnormal aggregation. 3318. PARK 1 B.6 tetrahydropyridine (MPTP). Lewy body is eosinophilic cytoplasmic inclusion that contains neurofilaments and -synuclein 853 Drugs that can produce secondary PD include all except ? D. PARK2 and PARK7 lead to autosomal recessive PD C. Globus pallidus externa . Methyldopa B. AJAY MATHUR Cardiology 713 850 Toxins that can produce secondary PD include all except ? Harrison’s 18th Ed. Table 366-2 Harrison’s 17th Ed. PARK 1 A. Manganese.4. Striatum . PARK 5 C.thalamus C.subthalamic nucleus C. Carbon disulfide. Striatum . Subthalamic nucleus . tetrabenazine). D. 857 Which of the following statements about familial forms of PD is 851 Drug that can cause secondary parkinsonism is ? false ? Harrison’s 18th Ed. D. 3320 Table 372-4 A.thalamus D. Globus pallidus interna . Clonidine C. PARK 1 Harrison’s 18th Ed. PARK 5 D. PARK1 and PARK5 lead to autosomal dominant PD B. antiemetics (compazine. Hexane produce parkinsonism. Subthalamic nucleus . All of the above 861 Which of the following pathways is excitatory ? 855 Which of the following gene leads to an autosomal dominant Harrison’s 18th Ed.Globus pallidus interna D. Figure 372-5 form of PD ? Harrison’s 18th Ed. Globus pallidus interna . Chlorperazine C. Phenytoin A. PARK 4 C. metoclopramide). Globus pallidus interna . Carbon monoxide.2. PARK 10 C. amiodarone. 3319-3320 Harrison’s 18th Ed. A. PARK5 encodes synuclein D. Table 372-2 A. Globus pallidus externa . 713 Neurology MCQ’s FORMCQ’s FOR MEDICAL MEDICAL PROFESSIONALS PROFESSIONALS BY PROF.Globus pallidus externa B. Lithium B. Substantia nigra compacta . Carbon disulfide PARK2 and PARK7 genes lead to autosomal recessive disorders with atypical features like juvenile forms of parkinsonism. All of the above D. 1-Methyl-1. PARK2 encodes tetrabenazine. 2552 A. some atypical C.subthalamic nucleus A. Figure 372-5 C.Globus pallidus externa form of PD ? C. Characteristic histopathologic feature is Lewy body D. an E3 ubiquitin protein ligase. 852 Drugs that can produce secondary PD include all except ? 858 Which of the following is false for Familial Parkinson’s disease ? Harrison’s 17th Ed. PARK 7 B. Amiodarone A. fluoxetine. PARK1 encodes alpha-synuclein.Globus pallidus interna D.thalamus 854 Which of the following gene is a late-onset PD susceptibility gene ? 860 Gamma aminobutyric acid (GABA) is the neurotransmitter for Harrison’s 17th Ed. 2553. Mutations in -synuclein and ubiquitin carboxy-terminal B. Methanol. Lithium carbonate 859 Glutamate is the neurotransmitter for which of the following B. 2553. Metoclopramide A. 3322. 2553 which of the following pathways ? A. Carbon monoxide D. Reserpine hydroxylase L1 (UCH-L1) C. Globus pallidus externa . Figure 372-5 B. valproic acid. 3322. chlorperazine. 3322. None of the above Harrison’s 17th Ed. Globus pallidus interna . All of the above 862 Which of the following pathways is inhibitory ? PARK1. 3320 Table 372-4 A.Substantia nigra reticulata antipsychotics. All of the above D. 2550 865 Most disabling motor feature of PD is ? Harrison’s 17th Ed. first distally. 872 Which of the following symptom precedes onset of PD ? Harrison’s 17th Ed. 2553 Festinating gait is a classic sign of parkinsonism due to a combination of flexed posture and loss of postural reflexes. 2551 866 Which of the following is false about ‘rest tremor’ in PD ? Harrison’s 17th Ed.behavioral disorder (RBD) A. Stooped posture Anosmia. 2550 A. Festinating gait 875 Which of the following is not a feature of essential tremor ? Harrison’s 17th Ed. Depression B. ipsilaterally. AJAY MATHUR Neurology 863 Which of the following is false in PD ? 869 In Latin "festino" means ? Harrison’s 18th Ed. Bradykinesia D. Appear bilaterally. 2550 A. Rest tremor. Three cardinal signs of PD are rest tremor. Rigidity C. rigidity & bradykinesia. Increased firing of neurons in GPi C. Push test Neuropsychiatric symptoms of later stages of PD may be direct result of PD or accompanying C. Swing test D. Early age of onset 868 Term used to describe gait in PD is ? B. Antalgic gait Predictors of dementia in PD include late age of onset. Significant postural instability & falls in first years of illness A. The patient accelerates to “catch up” with body’s center of gravity. Figure 366-1 C. Rapid eye movement . involving the digits and wrist with a “pill-rolling” character. presence of severe depression. Anosmia A. Micrographia B. Pigeon gait D. and 873 Cause of neuropsychiatric symptoms in PD can be ? occasionally to the leg before crossing to the other side. All of the above B. dopamine denervation causes increased firing of neurons in STN & GPi. Excessive inhibition of thalamus D. consequent reduced activation of cortical motor systems & 870 Which of the following strongly suggests a diagnosis other than development of parkinsonian features. Rest tremor C. Bradykinesia Significant postural instability and falls in the first years of the illness strongly suggest a diagnosis D. "to freeze" B. Spares head and neck Restless legs and rapid eye movement–behavioral disorder (RBD) often preceed the onset of PD. akinetic rigid phenotype. 2550 B. PD ? Harrison’s 17th Ed. All of the above D. 2551 but spares the head and neck. Tremor is present in 85% of 871 Manifestation that may be present long before the onset of motor patients with true PD and diagnosis of PD is difficult if tremor is absent. D. Frequency of 4 to 6 Hz B. at a frequency of 4 to 6 Hz. Increased firing of neurons in STN B. persistent hallucinations. May appear in lips. 2550. Scissor gait D. Akinetic rigid phenotype Harrison’s 17th Ed. Accompanying Alzheimer’s disease (AD) 867 Testing for postural instability in PD is done by ? B. Rigidity D. tongue and jaw Harrison’s 17th Ed. Freezing of gait C. DLB. Presence of severe depression A. postural instability is one of the most disabling feature. A. tongue and jaw D. typically appears unilaterally. It may appear later in lips. resulting in excessive inhibition of thalamus. Tremor usually spreads proximally. All of the above D. signs in PD include ? Harrison’s 17th Ed. A.714 MCQ’s FOR MEDICAL PROFESSIONALS BY PROF. Bilaterality . Persistent hallucinations B. Masked facies other than PD like Progressive Supranuclear Palsy. 3321 A. C. Accompanying cortical dementia with Lewy bodies (DLB) Harrison’s 17th Ed. depression and sleep disorders may be present long before the onset of motor signs. 2550 864 Which of the following is most frequently present in patients with true PD ? A. "to swing" D. Proximal and ipsilateral tremors B. None of the above In Latin "festino" means "to hurry". Hypophonia C. Pull test Alzheimer’s disease. Side effect of pharmacotherapy A. Dystonia involving distal arm or leg Harrison’s 17th Ed. In PD. Rest tremor B. The most disabling motor feature of PD is bradykinesia. 2551 with the “pull test”. Lift test 874 Predictors of dementia in PD include all except ? In advanced PD. "to hurry" C. Sleep disorders C. A. and advanced stages of disease. 2550 C. first distally C. It can be tested in office Harrison’s 17th Ed. cortical dementia with Lewy bodies or as a side effect of its drugs. "to fall" A. 2554 Harrison’s 17th Ed. 70 to 90 % Dopamine agonists readily cross the blood-brain barrier and act directly on postsynaptic dopamine Symptoms develop when striatal dopamine depletion reaches 50 to 70 % of normal. Rigidity D. 2557 diagnosis ? Harrison’s 17th Ed. Abnormal posture B. Progressive supranuclear palsy (PSP) C. Parkinson’s disease A. orthostatic hypotension. intrastriatal infusion of neurotrophic factors and exercise may be beneficial. All of the above 881 Which of the following symptoms respond poorly to therapy in Four areas of brain are not protected by the blood-brain barrier. 2554 Harrison’s 17th Ed. coenzyme Q10. D1 type A.primarily D2 type. Pramipexole C. parkinsonian features are compounded by early appearance of hallucinations or drug- induced hallucinations and disturbances in arousal and behavior. autonomic dysfunction &balance difficulties respond poorly to symptomatic therapy. Aggravation with alcohol Cognitive symptoms. Pineal gland C. pramipexole. Postural dependency D. Levodopa A. Cortical dementia with Lewy bodies (DLB) D. and 884 Which of the following is false for Dyskinesias in PD ? dysarthria’ favours which of the following diagnosis ? Harrison’s 17th Ed. Multiple system atrophy (MSA) D. 2553 A. median eminence of hypothalamus and area postrema. rigidity & abnormal posture respond well. and ropinirole are promising neuroprotection The development of early imbalance and falls suggests progressive supranuclear palsy (PSP). Area postrema D. or disease modification agents in PD. Selegiline B. orthostatic hypotension and dysarthria suggest at the beginning or end of the dose (diphasic dyskinesias). Occur as a peak dose effect B. and postural dependency and significant relief with alcohol. Bromocryptine B. Bradykinesia A. Alzheimer’s disease Selegiline in addition to a mild symptomatic effect has a neuroprotective function. 7: 70 A. Multiple system atrophy (MSA) Dyskinesias refer to choreiform and dystonic movements that can occur as a peak dose effect or The development of early urinary incontinence. Rigidity D. All of the above Rasagiline. Amantadine D. multiple system atrophy (MSA). A. 30 to 50 % C. AJAY MATHUR Cardiology 715 B. 883 Which of the following drugs have the potential of a neuroprotective 877 ‘Early imbalance & falls’ favours the diagnosis of ? therapy in PD ? Harrison’s 18th Ed. These areas include posterior pituitary gland. Occur at the beginning or end of the dose C. 3324 Harrison’s 17th Ed. Area postrema PD ? (a paired circumventricular organ & vomiting center in medulla) is not covered by BBB because it Harrison’s 17th Ed. In DLB. High doses of coenzyme Q 10. bradykinesia. 885 Dopamine agonists act directly on which of the following 879 Parkinsonian symptoms develop when striatal dopamine postsynaptic dopamine receptor ? depletion reaches how much of normal ? Harrison’s 17th Ed. Cortical dementia with Lewy bodies (DLB) D. 2553 A. higher frequency (8 to 10 Hz). 2552 A. pineal gland. D4 type D. Frequency of 8 to 10 Hz C. 2553 A. Parkinson’s disease B. receptors . Choreiform and dystonic movements A. Posterior pituitary gland B. Ropinirole D. 878 ‘Early urinary incontinence. 2553 J Neuroinflammation 2010. Essential tremor (ET) presentation includes bilaterality. Tremor C. While. Abnormal posture . Selegiline C. Balance difficulties C. 715 Neurology MCQ’s FORMCQ’s FOR MEDICAL MEDICAL PROFESSIONALS PROFESSIONALS BY PROF. None of the above D. 882 Which of the following drugs has neuroprotective function in PD 876 ‘Early appearance of hallucinations’ favours which of the following ? Harrison’s 17th Ed. D2 type B. 10 to 30 % B. Wilson ’s disease C. Parkinson’s disease B. Cognitive symptoms B. 880 Which of the following symptoms respond poorly to therapy in 886 Which of the following area of the brain is not protected by the PD ? blood-brain barrier (BBB) ? Harrison’s 17th Ed. D3 type C. Area postrema triggers nausea & vomiting to prevent further ingestion of toxins. tremor. Progressive supranuclear palsy (PSP) B. 50 to 70 % D. selegiline. hypophonia. Cortical dementia with Lewy bodies (DLB) C. 2553 senses toxins in blood that other parts of brain are protected from. Pramipexole A. 3321 B. Ropinirole C. 3323 A. They do not functionally D. and dopamine. C. 3323 D. ropinirole and rotigotine. It is metabolized peripherally by dopa-decarboxylase D. Pergolide hydroxytryptamine. Cycloserine B. All of the above A.716 MCQ’s FOR MEDICAL PROFESSIONALS BY PROF. Norepinephrine Pergolide is associated with asymptomatic valvular disease on chronic administration. Rotigotine Harrison’s 18th Ed. Dopamine Harrison’s 18th Ed. Apomorphine 889 Which of the following statements about Levodopa is false ? B. Levodopa D. Levodopa Intramitochondrial enzyme MA0 is responsible for the deactivation of norepinephrine. Selegiline B. 5-hydroxytryptamine 891 Which of the following is not a ‘Dopamine agonist’ ? C. Bromocriptine 898 Which of the following is a monoamine oxidase (MAO) B inhibitor ? D. 3323 entacapone). 2556 A. Chapter e26 Harrison’s 18th Ed. Pramipexole 897 MA0 is responsible for the deactivation of ? D. cabergoline. Levodopa B. Pergolide A. C. D. Clarithromycin . Cabergoline Selegiline & rasagiline are selective & irreversible monoamine oxidase (MAO) B inhibitor that block central dopamine metabolism & increase its synaptic concentration. Ropinirole Harrison’s 18th Ed. A. catechol-O-methyltransferase (COMT) and monoamine oxidase. Furazolidone A. 10 mg/day 890 Which of the following antiparkinsonian drug is associated with B. Pergolide Mostly. Dopamine does not cross blood-brain barrier (BBB) and second generation non-ergot alkaloids like pramipexole. 3321 C. 3323 B. Rotigotine A. Carbidopa C. Levodopa is a dopamine precursor Dopamine agonists include ergot alkaloids like bromocriptine. Pramipexole Harrison’s 18th Ed. Pergolide D. B. All of the above A. Harrison’s 17th Ed. 3323 888 Levodopa can be combined with ? A. Domperidone is a peripheral dopamine-blocking agent 894 Which of the following is administered as a transdermal patch ? D. C. AJAY MATHUR Neurology 887 Which of the following statements is false ? C. Pramipexole 892 Which of the following dopamine agonist is an ergot derivative ? Harrison’s 18th Ed. levodopa is used in combination with severe “off” episodes ? peripheral decarboxylase inhibitor (carbidopa or benserazide) or COMT inhibitor (tolcapone or Harrison’s 18th Ed. 25 mg/day valvular disease ? Harrison’s 17th Ed. 75 mg/day of carbidopa is necessary to block peripheral levodopa decarboxylation into dopamine. All of the above inhibit MAO-A. 2555 C. It is metabolized by COMT & MAO 896 What amount of carbidopa is necessary to block peripheral levodopa Levodopa is a precursor of dopamine and is metabolized centrally & peripherally by dopa- decarboxylation into dopamine ? decarboxylase. Entacapone C. Ropinirole A. Pramipexole Harrison’s 18th Ed. lisuride and pergolide B. It is not metabolized centrally by dopa-decarboxylase Apomorphine is a dopamine agonist but must be administered parenterally (SC/continuous infusion) and is used as a rescue agent for the treatment of severe “off” episodes. All of the above 895 Which of the following is used as a rescue agent for treatment of To prevent peripheral metabolism of levodopa to dopamine & development of nausea & vomiting due to activation of dopamine receptors in area postrema. Entacapone Rotigotine is administered as a once-daily transdermal patch. Is a precursor of dopamine D. Rotigotine B. C. Bromocriptine C. 3321 D. Levodopa C. 75 mg/day B. Benserazide D. 893 Which of the following dopamine agonist is a nonergot derivative 899 Which of the following is a monoamine oxidase (MAO) inhibitor ? ? Harrison’s 18th Ed. Pramipexole B. None of the above Harrison’s 18th Ed. 50 mg/day A. 3324 A. 5- B. Isocarboxazid the effects of levodopa by blocking enzymatic degradation of levodopa and dopamine. Cleveland Clinic Journal of Medicine 2010:77. Its side effects are livido reticularis. Linezolid inhibitor. Selegiline concomitantly with serotonergic drugs (SSRI). MAO-A preferentially metabolizes serotonin (5-HT) & NE. They augment A. Selegiline is selective for MAO-B. MAO-B preferentially metabolizes trace amines. 3324 D. Amikacin B. Pergolide Harrison’s 18th Ed. Pergolide Harrison’s 18th Ed. Alcohol effect (tolcapone and entacapone) due to accumulation of a metabolite. weight gain. Substantia nigra. discoloration of urine can be seen with both COMT inhibitors A. serotonin & dopamine) & exogenous amines (dietary tyramine). pars compacta ratio is 50%:50%. destroying its function forever. Wine effect A. Ventrolateral thalamus D. isocarboxazid. isocarboxazid & tranylcypromine. Intestine 910 Deep brain stimulation (DBS) for PD primarily targets which of B. Tranylcypromine Harrison’s 18th Ed. Selegiline Inhibition of MAO-A prevents metabolism of tyramine in gut. Amantadine MAO exists in two subtypes. Moclobemide is a reversible MAO A. phenelzine and tranylcypromine. Bromocriptine 901 Which of the following is not a MAO inhibitor ? D. Brain the following ? Harrison’s 18th Ed. in intestine. Capreomycin Harrison’s 18th Ed. Phenelzine .860 A. is the only oral agent that reduces dyskinesia while improving parkinsonian features. Isocarboxazid Furazolidone inhibits monoamine oxidase (MAO) gradually over several days. Diet effect 908 Livedo reticularis is the side effect of ? C. Entacapone 902 With MAO inhibitors. B. ratio is 80%:20% & in peripheral adrenergic neurons. including phenethylamine. which have only MAO-B. 3324 Though not of clinical concern. Clorgyline is selective for MAO-A. 717 Neurology MCQ’s FORMCQ’s FOR MEDICAL MEDICAL PROFESSIONALS PROFESSIONALS BY PROF. D. AJAY MATHUR Cardiology 717 D. B. Phenelzine 907 Discoloration of urine can be due to which of the following C. Rifapentine 906 Which of the following is a Catechol O-methyltransferase (COMT) C. Pergolide 903 MAO catabolizes which of the following ? Cleveland Clinic Journal of Medicine 2010:77. 3324 D. 3324 MAO inhibitors include selegiline.860 Amantadine is a weak NMDA-receptor antagonist. 3325 C. aged meats & red wine). C. Pergolide 904 Which of the following have only MAO-B ? Amantadine. and impaired cognitive function. leading to a potentially fatal B. Liver B. 3324 Entacapone and tolcapone are catechol O-methyltransferase (COMT) inhibitors. B. 3324 Linezolid is a weak MAO inhibitor and can be associated with the serotonin syndrome when given A. Amantadine hypertensive reaction known as a “cheese effect” as it can be precipitated by foods rich in tyramine (some cheeses. Selegiline are irreversible enzyme inhibitors that binds to MAO covalently. MAO-A & MAO-B C. All of the above A. The last three A. Entacapone and is a MAOI anti-tuberculous drug. C. tyramine-induced hypertensive crisis is C. Subthalamic nucleus (STN) MAO-A is the major enzyme outside of brain. 1378 Nonselective MAO inhibitors are phenelzine. Tetrabenazine drugs ? D. Bromocriptine D. Clorgyline D. A & B. Cheese effect Harrison’s 18th Ed. Bromocriptine named as ? D. Motor cortex DBS for PD primarily targets subthalamic nucleus (STN) or internal segment of globus pallidus 905 Which of the following is a selective MAO-A inhibitor ? (GPi). C. Cleveland Clinic Journal of Medicine 2010:77. D. with the exception of platelets and lymphocytes. Norepinephrine B. A. Clarithromycin inhibitor ? D. Platelets A. C. Ratio of MAO-A to MAO-B in human brain is 25%:75%.860 A. Iproniazid was the first antidepressant introduced B. Bromocriptine metabolize DA & tyramine. B. an antiviral agent. Selegiline MAO is a flavin-containing enzyme critical for regulating neurotransmitter levels by catabolizing endogenous monoamines (NE. ratio is 90%:10%. Serotonin 909 The only oral agent that reduces dyskinesia while improving C. Moclobemide 900 Which of the following is a monoamine oxidase (MAO) inhibitor ? Harrison’s 18th Ed. whereas in liver. B. Dopamine parkinsonian features is ? Harrison’s 18th Ed. 3327 A. Risperidone B. rhythmic manner. All of the above D. All of the above Clinical phenotype of MSA can be striatonigral degeneration (SND). 920 Cerebellar disease is characterized by which of the following tremor ? 914 Bell’s reflex refers to ? Harrison’s 18th Ed. Rest tremor A. 2414 A. . Depression & adduction of eyes on attempted lid closure D. Tau B. Kinetic tremor C. Postural tremor D. olivopontocerebellar atrophy (OPCA) & progressive autonomic failure (PAF). Lithium B. 3327 C. 2414 A. 3327 B. Elevation & abduction of eyes on attempted lid closure C. 2413 Harrison’s 18th Ed. Akinetic tremor incorporated into host genome. Seizure D. 919 Essential tremor (ET) is characterized by which of the following 913 Which of the following is a clinical phenotype of MSA ? tremor ? Harrison’s 16th Ed. Olivopontocerebellar atrophy (OPCA) B. Bell’s reflex refers to elevation & abduction of eyes on attempted lid closure. Rest tremor B.718 MCQ’s FOR MEDICAL PROFESSIONALS BY PROF. Postural tremor C. is not C. Kinetic tremor B.to 12-Hz postural & kinetic tremor affecting the upper extremities mostly. All of the above D. -synuclein D. Elevation & adduction of eyes on attempted lid closure B. 3328 disorders ? A. 3326 A. or on actively reaching for a target (kinetic tremor). electrolyte abnormalities. Arms Harrison’s 18th Ed. olanzapine and aripiprazole are not well tolerated by most patients with PD due to a D. Head B. 922 Which part of the body is affected most in essential tremor (ET) ? 916 Which of the following best relate to hyperkinetic movement Harrison’s 18th Ed. AJAY MATHUR Neurology 911 In gene therapy for PD. Harrison’s 18th Ed. 3328 which of the following ? Harrison’s 16th Ed. postural or action tremors usually of no clinical consequence seen in anxiety. It can be most prominent at rest (rest tremor). All of the above Enhanced physiologic tremors (EPT) are mild. and cerebellar disease by an intention or kinetic tremor. AAV1 B. or alcohol. on assuming a posture (postural drug is preferred ? tremor). Kinetic tremor Gene therapy involves viral vector delivery of DNA of a therapeutic protein to specific target regions. All of the above D. Quetiapine 918 PD is characterized by which of the following tremor ? Harrison’s 18th Ed. Kinetic tremor D. which of the following is used as viral D. All of the above higher incidence of drug-induced parkinsonism (DIP) and akathisia. AAV2 917 Which of the following is not a kind of tremor ? Harrison’s 18th Ed. D. Postural tremor Quetiapine is recommended because it lacks the risk of agranulocytosis associated with clozapine. Involuntary C. lithium. AAV4 B. Valproate A. Amyloid C. Olanzapine C. Larynx Essential tremor (ET) is the most common movement disorder characterized by 6. Postural tremor 912 In patients of PD with psychotic symptoms. Striatonigral degeneration (SND) A. AAV3 A. Depression & abduction of eyes on attempted lid closure PD is characterized by a resting tremor. Alcohol C. high-frequency. A. 3327 Harrison’s 16th Ed. 3325 or in combination. hyperthyroidism. which of the following Tremor consists of alternating contractions of agonist & antagonist muscles in an oscillating. neurofibrillary tangles are histochemically positive for 921 Enhanced physiologic tremor (EPT) can be seen with the use of ? Harrison’s 18th Ed. 915 In PSP. The AAV2 virus is used as viral vector as it does not cause inflammatory response. Voluntary B. Legs C. Rest tremor D. Clozapine A. and is associated with long-lasting transgene expression. essential tremor (ET) by a postural tremor. fatigue. PSP is characterized by deposition of neurofibrillary tangles histochemically valproate. Pathologically. positive for tau and negative for amyloid or -synuclein. Progressive autonomic failure (PAF) C. Risperidone. Passive vector ? Hyperkinetic movement disorders are characterized by involuntary movements occuring in isolation Harrison’s 18th Ed. 3327 A. Rest tremor C. retrocollis). spasmodic dysphonia. 3329 D. Tricyclic antidepressants exclude Dopa responsive dystonia (DRD) or the Segawa variant (DYT5). Myoclonic dystonia 933 Dystonia can occur as a part of which neurodegenerative condition ? Harrison’s 18th Ed. Lubag form of dystonia-parkinsonism A. 3328 D. 3329 mitochondrial encephalopathies. 3329 Co-contraction of agonist and antagonist muscles is a fundamental feature of dystonia and characteristically present during attempted voluntary movement. glucocorticoids. PLOD1 gene Myoclonic dystonia (DYT11) results from a mutation in epsilon-sarcoglycan gene on chromosome 7q21. Dopa responsive dystonia (DRD) A. CBGD. Dopa responsive dystonia (DRD) principally from the Panay Island. C. None of the above parkinsonism ? Harrison’s 18th Ed. Exacerbated by stress and fatigue 932 Which of the following is is an x-linked recessive dystonia D. Primidone Most common forms of dystonia are in adults . LINGO1 gene accompanied by psychiatric disturbances. C. TNXB gene Harrison’s 18th Ed. Lubag form of dystonia-parkinsonism (DYT3). A. Idiopathic torsion dystonia (ITD) B. Wilson’s disease D. Levodopa Harrison’s 18th Ed. Serotonin reuptake blockers in favour of ? Alcohol consumption reduces ET. A. Torticollis 925 Drugs that have usefulness in treatment of essential tremors include all except ? C. Serotonin reuptake blockers A. C. with Harrison’s 18th Ed. Lithium . Appear during attempted voluntary movement C. Huntington’s disease and fatigue and attenuated by sensory inputs [touching affected body part (geste antagoniste)].blepharospasm. 3329 Association with LINGO1 gene is found in patients with young-onset ET. Gabapentin 931 Which of the following is a combination of oromandibular dystonia (OMD) and blepharospasm ? Primidone and propranolol are the first-line treatments for ET. cervical dystonias (torticollis. B. Linkage analysis has confirmed the mode of inheritance and localized the disease gene to the proximal long arm of the x-chromosome. Surgical therapies targeting the VIM nucleus of the A. include benzodiazepines. Dystonia is exacerbated by stress A. 3329 acid. The combination of lower facial and jaw dystonia is called Meige’s syndrome. and Harrison’s 18th Ed. AJAY MATHUR Cardiology 719 923 All of the following can aggravate essential tremors except ? C. Myoclonic dystonia 924 Association with which of the following gene is found in patients C. Meige’s syndrome Harrison’s 18th Ed. Co-contraction of agonist and antagonist muscles Meige syndrome is a combination of oromandibular dystonia (OMD) and blepharospasm. PD. tricyclic antidepressants. B. C. All of the above 928 Any patient suspected of having a childhood-onset dystonia should Dystonia plus syndromes refer to dystonia occuring as a part of neurodegenerative conditions like receive a trial of ? HD. HD Idiopathic torsion dystonia (ITD) or Oppenheim’s dystonia is a primary dystonia of childhood- onset with autosomal dominant inheritance linked to a mutation in DYT1 gene located on chromosome B. B. Lubag form of dystonia parkinsonism 927 Mutation in DYT1 gene on chromosome 9q34 results in which of C. Dopa responsive dystonia (DRD) thalamus can be very effective for severe and drug-resistant cases. Leigh’s disease the following dystonia ? D. COL1A1 gene 930 Which of the following is a focal dystonia ? D. lithium. Idiopathic torsion dystonia (ITD) 926 Which of the following statements is false about dystonia ? C. thyroxin. frequently B. 3328 D. PD 9q34 resulting in a trinucleotide GAG deletion with loss of one of a pair of glutamic acid residues in the protein torsin A. Alcohol 929 Dystonia frequently accompanied by psychiatric disturbances goes D. Additional useful medications. Stress and drugs that can aggravate any tremor include valproic Harrison’s 18th Ed. Dystonia is usually not the dominant neurologic feature. and botulinum toxin injections to affected muscle groups. Valproic acid D. 3329 or without the primary agents. 3328 D. B. All of the above A. 3328 D. 3328 Lubag is an x-linked recessive dystonia parkinsonism that affects Filipino men originating A. Valproate B. gabapentin. PSP. Idiopathic torsion dystonia (ITD) C. topiramate. Propranolol anterocollis. Blepharospasm B. Lubag form of dystonia-parkinsonism with young-onset ET ? Harrison’s 18th Ed. Wilson’s disease. methylxanthines. Lubag form of dystonia-parkinsonism A. Valproic acid Any patient suspected of having a childhood-onset dystonia should receive a trial of levodopa to B. All of the above Harrison’s 18th Ed. oromandibular dystonia. A. Meige’s syndrome Harrison’s 18th Ed. B. 719 Neurology MCQ’s FORMCQ’s FOR MEDICAL MEDICAL PROFESSIONALS PROFESSIONALS BY PROF. It typically manifests as a combination of dystonia and myoclonic jerks. beta-adrenergic agonists. and serotonin reuptake blockers. CNS stimulants. Gait disturbance all except ? B. 3331 B. Canada A. Parkinsonian syndrome A. causal gene and an accurate genetic test for HD is available. Alzheimer’s disease 942 Most common systemic disorder that causes chorea is ? Harrison’s 18th Ed. All of the above Anticholinergic drugs are the most effective forms of treatment for generalized primary dystonia. 3332 C. Progressive atrophy of caudate nuclei 943 In hemiballismus. B. Harrison’s 18th Ed. America 944 Which of the following best relates to hemiballismus ? C. Harrison’s 18th Ed. an American physician in 1872. and spasticity emerge. Autosomal dominant neuroferritinopathy D. Dopamine-blocking agents B. 3330 of proximal limb muscles on one side of the body. metabolic alterations. Dementia C. 3331 C. 3332 D. Neurodegenerative diseases with brain iron accumulation (NBIA) manifesting with chorea include 936 Westphalt variant of Huntington’s disease (HD) relates to which of autosomal dominant neuroferritinopathy. Sydenham’s chorea (St. B. C. with many medications (anticonvulsants. AJAY MATHUR Neurology 934 Most effective treatment for generalized primary dystonia is ? 940 Sydenham’s choreas responds to which of the following ? Harrison’s 18th Ed. Great Britain (STN) and rarely in putamen. most often the lesion is in ? C. Short arm of chromosome 5 A. Polycythemia rubra vera 937 Which of the following is a typical MRI finding in Huntington’s Systemic lupus erythematosus is the most common systemic disorder that causes chorea. Myoclonus B. Short arm of chromosome 4 945 Which of the following is a neurobehavioral disorder ? B. Any of the above Huntington’s gene on the ? Hemiballismus is a violent form of chorea composed of wild. rigidity. lithium) and paraneoplastic syndromes associated with anti-CRMP-5 or anti-Hu A. chorea is reduced & dystonia. Choreas Disease (HD) ? can also be seen with hyperthyroidism. 3332 D. 3330 941 Disorders having excess brain iron accumulation on MRI include A. Valproic acid C. C. 3330 A. Chorea may recur in later life. C. 3330 polycythemia rubra vera. Orthostatic hypotension B. Vitus’ dance) is of acute onset and responds to dopamine-blocking agents. and aceruloplasminemia. Essential tremor (ET) D. Progressive atrophy of caudate nuclei and consequent B. estrogens. 3330 Most common cause of hemiballismus is a lesion (infarct or hemorrhage) of subthalamic nucleus A. nonpatterned. Huntington’s disease (HD) . Systemic lupus erythematosus (SLE) HD can present as an akinetic-rigid or parkinsonian syndrome (Westphal variant). Pedunculopontine nucleus Harrison’s 18th Ed. HIV disease. Subthalamic nucleus 938 George Huntington belonged to which country ? D. Atrophy of the midbrain A. 3331 A. 3312. gait disturbance. autosomal recessive pantothenate-kinase-associated neurodegeneration (PKAN. HIV disease D. and oculomotor abnormalities are common features. Carbamazepine D. Oculomotor abnormalities A. Iron the following ? accumulation in globus pallidus provides on MRI the characteristic “eye of the tiger” appearance. Australia Harrison’s 18th Ed. Cerebellum enlargement of lateral ventricles is visualized by MRI. flinging. myoclonus. Sjögren’s syndrome. Hypointensity of the caudate head antibodies. valproic acid. 935 Which of the following is most characteristic feature of HD ? Harrison’s 18th Ed.720 MCQ’s FOR MEDICAL PROFESSIONALS BY PROF. 3329 Harrison’s 18th Ed. B. Chorea Huntington’s disease (HD) was described by George Huntington. Athetosis Huntington’s gene is single. Choreoathetosis 939 Huntington’s disease (HD) is caused by mutations in the D. Propranolol B. Long arm of chromosome 5 B. A. Anticholinergic drugs D. bradykinesia. involuntary choreiform movements. C. Iron accumulation in striatum Harrison’s 18th Ed. and carbamazepine. D. Primidone A. Wilson’s disease HD is caused by mutations in the Huntington’s gene on the short arm of chromosome 4. Globus pallidus HD predominantly strikes striatum. Hallervorden-Spatz disease HD is characterized by rapid. large-amplitude movements Harrison’s 18th Ed. particularly in association with pregnancy (chorea gravidarum) or treatment with sex hormones. Long arm of chromosome 4 Harrison’s 18th Ed. Valproic acid C. Chorea Harrison’s 18th Ed. Hyperthyroidism C. semipurposeful. cocaine. As disease D. Aceruloplasminemia progresses. Hallervorden-Spatz disease). Neuroacanthocytosis Dysarthria. 721 Neurology MCQ’s FORMCQ’s FOR MEDICAL MEDICAL PROFESSIONALS PROFESSIONALS BY PROF. 947 Which of the following is a kind of vocal tic ? Harrison’s 18th Ed. Asterixis is a negative myoclonus B. depression Harrison’s 18th Ed. Rapid. 956 Myoclonus can result with the use of which of the following 950 Myoclonus can be seen in association with pathology in ? drugs ? Harrison’s 18th Ed. and obsessive-compulsive disorder. All of the above Drugs can be associated with parkinsonism & hyperkinetic movement disorders like phenytoin Myoclonus can be seen in association with pathology in cortical. All of the above D. and coprolalia (expression of obscene words). 3333 Harrison’s 18th Ed. fluphenazine. Clonidine A. in association with voluntary movement (action myoclonus) altered mental status. Lactic acid D. Phenytoin B. 12 weeks Clonidine. 951 Which of the following is useful in the treatment of myoclonus ? cimetidine. of a twitch due to a brief loss of muscle activity (asterixis in hepatic failure). labile blood pressure. jerky movement of <100 msec. Negative myoclonus consists elevated creatine kinase levels. subcortical. dystonia). lips. Interferes with normal movement C. Myoclonus is not a feature of NMS but of serotonin syndrome. Lactic dehydrogenase C. Valproic acid 957 Restless Legs Syndrome was first described by ? Harrison’s 18th Ed. 3333 A. Anxiety. clonazepam. It is typically generalized in children and focal in adults like blepharospasm. attention deficit hyperactivity disorder. encephalopathy. duration due to single or repetitive muscle Neuroleptic malignant syndrome (NMS) is characterized by muscle rigidity. Creatine kinase B. Valproic acid. neuroleptics (haloperidol. 3333 B. elevated temperature. AJAY MATHUR Cardiology 721 D. Levetiracetam Tourette’s Syndrome (TS) is a neurobehavioral disorder named after French neurologist Georges D. Guanfacine. 3332 952 Which of the following is an example of Dystonia ? A. 3332 Harrison’s 18th Ed. Atypical neuroleptics (risperidone. Echolalia Harrison’s 18th Ed. olanzapine. Tetrabenazine helpful in refractory cases D. Tourette’s Syndrome (TS) C. tachycardia. and fentanyl (dyskinesias). depression. All of the above C. 946 Associated behavioral disturbance with Tourette’s Syndrome (TS) include ? Harrison’s 18th Ed. or oromandibular dystonia. oral contraceptives (dyskinesia). chorea. primidone and levetiracetam may be effective. 3333 B. Attention deficit hyperactivity disorder A. Dystonia is the most common acute hyperkinetic drug reaction. carbamazepine (tics and dystonia). tremor. None of the above Vocal tics include grunting. 4 weeks C. myoclonus). buspirone (akathisia. Myoclonic jerks differ from tics in that they interfere with normal movement and are not suppressible. Blepharospasm C. Coprolalia B. hyperthermia. Cortical region A. Fluoxetine C. Piracetam A. 955 Levels of which of the following is markedly elevated in neuroleptic 949 Which of the following about myoclonus is false ? malignant syndrome (NMS) ? Harrison’s 18th Ed. fluoxetine (myoclonus. 3332 953 Which of the following about tardive dyskinesia (TD) is false ? A. Suppressible D. adrenergics (tremor). All of the above C. and tongue mostly involved D. All of the above D. and markedly or in response to an external stimulus (reflex or startle myoclonus). lithium. tricyclic antidepressants regions and associated with hypoxemic damage. 3333 A. 6 weeks D. (dyskinesias. palilalia (repeating one’s own words). All of the above Myoclonus is a rapid. duration A. Palilalia A. Choreiform movements C. It occur spontaneously. piracetam. Guanfacine B. myoclonus). All of the above Associated behavioral disturbances with Tourette’s Syndrome (TS) include anxiety. Christopher Ross . diazoxide. Spinal cord region C. torticollis. and digoxin. 3332 Harrison’s 18th Ed. 3334 B. dystonia. jerky movement of <100 msec. or pimozide) are useful in treatment of TS. Obsessive-compulsive disorder B. Subcortical region B. myoclonus). renal failure. 954 Use of metoclopramide for more than how many weeks increases 948 Effective medication in the treatment of Tourette’s Syndrome is ? the risk of tardive dyskinesia (TD) ? Harrison’s 18th Ed. methadone. and neurodegeneration. dyskinesias. tremor. Olanzapine C. Treatment primarily consists of treating the underlying condition or removing offending agent. ziprasidone) and classical FDA has warned that use of metoclopramide for more than 12 weeks increases the risk of TD. echolalia (repeating other people’s words). discharges. 2 weeks B. Torticollis D. 3332 A. 3333 A. Oromandibular dystonia D. Harrison’s 18th Ed. All of the above Gilles de la Tourette. Mouth. Buspirone D. or spinal cord (chorea. caudate & pallidum. 3334 A.722 MCQ’s FOR MEDICAL PROFESSIONALS BY PROF. About half of WD patients (esp. Trientine rare for WD patients with neurologic features not to have KF rings. Psychiatric disorders C. younger patients) manifest with liver abnormalities. 3334 In WD. Micheal Emre 963 Which of the following marks the onset of neurologic C. ferritin deficiency. None of the above C. MRI shows symmetric hyperintensity on T2-weighted images in putamen. A. James Hardy A. 100 % B. Tetrathiomolybdate Kayser-Fleischer (KF) rings are seen in nearly all WD patients with neurologic features. bilateral. Best detected by slit-lamp examination B. Represent deposition of copper in Descemet’s membrane 968 Which of the following statements about Penicillamine is false ? Harrison’s 18th Ed. psychiatric. Dysarthria Harrison’s 18th Ed. None of the above examination. B. Peripheral neuropathy C. They are best detected by slit-lamp D. 3334 D. High levels of urinary copper 959 Which of the following in Wilson’s disease can manifest alone ? D. Any of the above Harrison’s 18th Ed. Worsens symptoms in initial stages of therapy C. Thomas Willis manifestations in WD ? Harrison’s 18th Ed. All of the above A. Caudate C. 3334 Neurologic onset usually manifests in the second decade with tremor and rigidity. 3334 D. Pallidum 960 Kayser-Fleischer (KF) rings are seen in nearly what proportion of WD patients with hepatic presentations ? D. low levels of blood copper and ceruloplasmin and high levels of urinary copper are found A. and asymmetric. 3334 In WD. Dystonia 958 Secondary RLS may be associated with ? D. Should be coadministered with pyridoxine Kayser-Fleischer rings (KF rings) represent the deposition of copper in Descemet’s membrane producing brownish discoloration of peripheral cornea. Neurologic disorders but normal levels do not exclude the diagnosis. 40 % Harrison’s 18th Ed. Hyperintensity of putamen on T2-weighted MRI images 961 Kayser-Fleischer (KF) rings are seen in nearly what proportion of WD patients with neurologic features ? D. Low levels of blood copper Secondary RLS may be associated with pregnancy. 3334 Liver biopsy with demonstration of high copper levels is the gold standard for diagnosis of WD. which of the following is false ? Harrison’s 18th Ed. 20 % 966 Which of the following is the gold standard for the diagnosis of B. Tremor century by English physician Thomas Willis. Increases copper excretion D. It is very C. 3334 B. 100 % A. All of the above Harrison’s 18th Ed. A. Renal failure 964 In WD. and B. Brownish discoloration of the peripheral cornea A. 40 % WD ? C. and liver disorders. 20 % 967 Which of the following blocks the absorption of copper ? B. anemia. Penicillamine D. Liver disorders 965 In WD. Penicillamine increases copper excretion. Zinc 962 Which of the following statements about Kayser-Fleischer (KF) Tetrathiomolybdate blocks absorption of copper. D. Parkinsonism . Low levels of blood ceruloplasmin peripheral neuropathy. C. Tremor is A. renal failure. Putamen may manifest with neurologic. 3334 D. Pregnancy usually in upper limbs. AJAY MATHUR Neurology B. C. B. alone or in combination. Harrison’s 18th Ed. None of the above Harrison’s 18th Ed. Generalized atrophy on CT brain scan C. B. 80 % A. Bradykinesia Restless legs syndrome (RLS) is a neurologic disorder and was first described in seventeenth B. Kayser-Fleischer (KF) rings Kayser-Fleischer (KF) rings are seen in 80% of WD patients with hepatic presentations. Trientine rings is false ? & zinc are useful drugs for maintenance therapy. MRI shows symmetric hyperintensity on T2-weighted images in ? D. B. 3334 A. 3334 C. 80 % Harrison’s 18th Ed. 3334 Wilson’s disease (WD) is an autosomal recessive inherited disorder of copper metabolism that A. 969 Astasia-abasia is best related to ? Harrison’s 18th Ed. High copper levels in liver biopsy Harrison’s 18th Ed. Joint position & vibration sense is diminished in lower limbs. Psychogenic movement disorder C. B. Cerebellar degeneration A. Dystonia walking and do poorly in dark. Stance destabilized by eye closure D. Harrison’s 18th Ed. Falls in daily life is a late Harrison’s 17th Ed. Patient looks down at their feet when B. Gait impairment 975 Pill-rolling tremor is specific for ? Harrison’s 18th Ed. difficulty 972 Which of the following is the commonest etiology of gait disorders ? in maintaining balance on turning (an early feature) and tandem gait. 195 A. C. Sensory deficits 979 Sensory ataxia is characterized by ? B. Psychogenic B. erratic foot placement. All of the above Difficulty with gait initiation leads to a freezing gait seen in PD. Cerebellar gait ataxia 970 Sensory information for postural control is primarily generated C. corticobasal degeneration & primary pallidal degeneration. Nystagmus D. as if walking on a slippery surface. and a motion. shuffling along A. Parkinson’s disease C. 193 A.Ataxic Disorders A. Progressive supranuclear palsy D. Poor balance B. All of the above Harrison’s 18th Ed. Parkinson’s disease . Look down at their feet when walking 973 Stiff-legged gait category includes which of the following ? C. Vestibular system C. nystagmus (vestibulo-oculomotor sign) & poor balance (impairment of vestibulo-spinal function). Parkinsonism Harrison’s 18th Ed. MSA and CBD frequently present with axial stiffness. 723 Neurology MCQ’s FORMCQ’s FOR MEDICAL MEDICAL PROFESSIONALS PROFESSIONALS BY PROF. 193 B. All of the above A. 193 D. Progressive supranuclear palsy C. Loss of two of the three pathways B. Gait apraxia Sensory information for postural control is primarily generated by visual system. Freezing Gait A. Vertigo 974 Causes of freezing gait include ? Harrison’s 18th Ed. 193 D. Proprioceptive receptors in muscle spindles & joints 977 Which of the following relate to frontal gait disorders ? Harrison’s 18th Ed. Joint position & vibration sense diminished in lower limbs Harrison’s 18th Ed. Stiff-person syndrome 980 Which of the following is a feature of vestibular disorder ? D. gait disorder must be viewed as the product of a C. A. Nystagmus A. Clinically. Difficulty in maintaining balance when turning C. Unable to walk tandem heel to toe D. conversion reaction. D. postural instability. AJAY MATHUR Cardiology 723 B. Narrow base of support Cautious gait refers to describe a patient who walks with an abbreviated stride & lowered center of mass. 193 373 . short stride. All of the above Astasia-abasia refers to odd gyrations of posture with wastage of muscular energy. D. Chapter 24 event. Corticobasal degeneration Feature of vestibular disorders are vertigo (subjective appreciation or illusion of movement). Cautious Gait 978 Cerebellar gait ataxia is characterized by all except ? B. Psychogenic gait disorder Harrison’s 18th Ed. All of the above A. Frontal Gait Disorder B. Lower body parkinsonism neurologic deficit and a functional adaptation. 192 Frontal gait disorder or “gait apraxia” features wide base of support. 193 C. Visual system floor and difficulty with starts & turns. Cerebellar degeneration B. and dramatic fluctuations over time observed in patients with somatoform disorders and shuffling gait but lack the characteristic pill-rolling tremor of Parkinson’s disease. 193 C. Falls is a late event Cerebellar gait ataxia is characterized by a wide base of support. 981 Symptoms & signs of true cerebellar ataxia consist of all except ? multiple system atrophy. Stiff-Legged Gait Harrison’s 18th Ed. All of the above C. Sensory ataxia B. Corticobasal degeneration D. vestibular system and by proprioceptive receptors in the muscle spindles and joints. stance is destabilized by eye closure. D. All of the above 971 “As if walking on a slippery surface” best relates to ? Difficulty with gait initiation is referred to as “slipping clutch” syndrome & lower body parkinsonism. “Slipping clutch” syndrome is compromises standing balance. Frontal gait disorder by ? D. Hand incoordination A. This disorder is both common and nonspecific. Spastic gait In sensory ataxia. 976 “Gait apraxia” is characteristic of ? Harrison’s 18th Ed. Harrison’s 18th Ed. 195 A. extreme slow Patients of PSP. 193 B. progressive supranuclear palsy. Hodgkin’s disease C. Inherited ataxia Acute & reversible ataxia can result from intoxication with alcohol. Breast cancer C. 3336 C. B. 195 frontopontocerebellar pathway from rostral frontal lobe. Patients with progressive supranuclear A. All of the above C. and with Hodgkin’s disease is anti-Tr. chaotic saccades that occur in all directions of gaze frequently associated with myoclonus & ataxia. 3336 991 Paraneoplastic cerebellar ataxia is associated with ? A. 3336 Table 373-1 D. 3336 Demyelinating disease multiple sclerosis presents with focal and ipsilateral cerebellar signs of subacute duration (days to weeks). Neuroblastoma B. Small-cell lung cancer B. 3336 B. All of the above A. in adults and neuroblastoma in children. Breast cancer C. small- cell lung cancer (anti-PQ type voltage-gated calcium channel). Hypothyroidism Hypothyroidism must always be considered as a readily treatable and reversible form of chronic symmetric gait ataxia. C. A. Cerebellar ataxia presents as gait B. Phenytoin 989 Which of the following is not a cause of symmetric ataxia ? C. Cerebellar pathology Patients with hereditary ataxia or alcoholic cerebellar degeneration do not complain of dizziness. Ovarian cancer A. lung cancer and neuroblastoma. Small-cell lung cancer D. In sensory ataxia. Lung cancer D. Lung cancer A. Lithium Subacute development of ataxia of gait may be due to vitamin B 1 and B12 deficiency and hyponatremia. B1 deficiency B. Multiple sclerosis D. B12 deficiency 990 Which of the following drugs can cause symmetric ataxia ? Harrison’s 18th Ed. Hyponatremia A. Gait freezing in Parkinson’s disease results in a forward fall. and Hodgkin’s disease (anti-Tr). All of the above 982 Symptoms & signs of true cerebellar ataxia consist of all except ? True cerebellar ataxia result from involvement of cerebellum. Alcohol palsy fall over backwards. with small-cell lung cancer is anti-PQ type voltage-gated calcium channel. All of the above Opsoclonus is a disorder of eye movement characterized by involuntary. Autoantibodies related to D. Nystagmus 988 Patients with which of the following disorder fall over backwards ? B. Barbiturates 985 Anti-Tr autoantibody is related to which of the following ? D. Breast cancer Harrison’s 18th Ed. Hodgkin’s disease Paraneoplastic cerebellar ataxia may present with various tumor states. lithium & barbiturates. Meningovascular syphilis 984 Which of the following may lead to development of ataxia of gait ? Harrison’s 18th Ed. 3336 B. 986 Paraneoplastic Opsoclonus-Myoclonus Syndrome is related to which of the following ? 992 Paraneoplastic syndrome associated with myoclonus and Harrison’s 18th Ed. Cerebellum Ataxia due to vestibular nerve or labyrinthine disease results in gait disorder associated with dizziness. 2565 D.724 MCQ’s FOR MEDICAL PROFESSIONALS BY PROF. . All of the above B. Breast cancer Harrison’s 18th Ed. Neuroblastoma D. A. phenytoin. light-headedness or perception of movement. Patients with lesions of vestibular system have lateral pulsion & toppling falls. C. C. Tremor with movement B. visual blurring due to nystagmus. Patients with somatosensory deficits 983 Acute and reversible ataxia can occur due to which of the following ? Harrison’s 18th Ed. B. hand incoordination and tremor with movement. spinocerebellar pathways and Harrison’s 18th Ed. 3336 opsoclonus occurs with ? A. When the cause of Paraneoplastic syndrome associated with myoclonus and opsoclonus occurs with breast cancer opsoclonus-myoclonus is paraneoplastic. 195 C. Spinocerebellar pathway impairment. Lesions of the vestibular system D. All of the above Harrison’s 18th Ed. Progressive supranuclear palsy D. Scanning speech Harrison’s 18th Ed. AJAY MATHUR Neurology C. All of the above breast and ovarian cancers is anti-Yo. 3335 Patients with cerebellar pathology may lean & topple toward side of lesion. tumors involved are usually cancer of lung and breast (anti-Ri). C. True cerebellar ataxia is devoid of vertiginous complaints. Romberg sign 987 True cerebellar ataxia result from the involvement of ? Harrison’s 17th Ed. Frontopontocerebellar pathway imbalance worsens when visual input is removed (Romberg sign). Tremor with movement A. Paraneoplastic cerebellar ataxia is associated with breast and ovarian cancers (anti-Yo). Phenytoin D. Lithium Harrison’s 18th Ed. scanning speech. Dizziness A. D. Harrison’s 18th Ed. SCA12 is linked to an untranslated CAG repeat. 30 1001 Which clinical phenotype of spinocerebellar ataxia has been D. Machado-Joseph Disease (MJD) Clinical phenotype of of spinocerebellar ataxia (SCA2) has been described in patients from Cuba B. C. Spinocerebellar ataxia 1 995 About what number of glutamines due to expanded polyglutamine B. SCA6.GCC repeat. termed ataxins. Harrison’s 18th Ed. Length of nucleotide repeat correlates with severity of disease. Other triplet repeat expansions diseases include Fragile X-syndrome (FRAXA) . Spinocerebellar ataxia 3 Harrison’s 18th Ed. Expanded polyarginine proteins Friedreich ataxia (FRDA1) is GAA repeat autosomal recessive congenital ataxia. All of the above autosomal dominant ataxia ? Besides Machado-Joseph Disease (MJD). Dentatorubral-pallidoluysian atrophy (DRPLA) 1002 Which out of the following is the most common inherited D. Spinocerebellar ataxia 6 B. and SCA17 are caused by CAG triplet 999 Which of the following Trinucleotide Repeat Disorders has repeat expansions in different genes. Friedreich’s Ataxia Nucleotide repeat expansion disorders are associated with an increase in number of nucleotide repeats above a threshold. Dentatorubropallidoluysian atrophy (DRPLA) during cell division. Spinocerebellar ataxia 6 Harrison’s 18th Ed. 3341 dominant inheritance. Expanded polyserine proteins C. Friedreich ataxia (FRDA1) C. Fragile X-syndrome (FRAXE) . 3341 common autosomal dominant ataxia. Expanded polyleucine proteins 1000 Which of the following was also called olivopontocerebellar CAG encodes glutamine & expanded CAG triplet repeat expansions in SCA’s result in expanded atrophy ? polyglutamine proteins. SCA6 B.dynamic mutation. Decreasingly severe genotype in next generation C. 3343 C. and disease manifests severely and at an earlier age D.GAA repeat. SCA1. 10 D. SCA8 is due to an untranslated CTG repeat expansion. 725 Neurology MCQ’s FORMCQ’s FOR MEDICAL MEDICAL PROFESSIONALS PROFESSIONALS BY PROF. Spinocerebellar ataxia 1 Atrophy (SBMA or Kennedy’s Disease). They impair neuronal functions. alter efficiency of ubiquitin-proteosome system of protein turnover A. 3341 B. that produce a toxic gain of function with autosomal Harrison’s 18th Ed. . SCA17 In subsequent generations. Spinocerebellar ataxia 6 repeats” is false ? In most populations. 20 SCA1 was previously referred to as olivopontocerebellar atrophy. SCA3 [Machado-Joseph disease (MJD)]. Length correlates with severity of disease B. Spinocerebellar ataxia 2 myotonica (DM) . Spinocerebellar ataxia 7 A. Dentorubral pallidoluysiane atrophy (DRPLA) B. 3341 A. A. Spinocerebellar ataxia 3 997 Which of the following about increase in number of “nucleotide D. D. Expanded polyglutamine proteins D. 3342 disease. SCA7. C.CTG repeat and Friedreich ataxia (FRDA1) . and SCA10 is caused by an untranslated autosomal recessive inheritance ? pentanucleotide repeat. Spinocerebellar ataxia 2 ataxins are potentially toxic to neurons ? C.CGG repeat. 3341 Harrison’s 18th Ed. Leads to anticipation A. Machado-Joseph Disease (MJD) or spinocerebellar ataxias 3 is the most Harrison’s 18th Ed. Repeat length increases from one generation to the next. Increasingly severe genotype in next generation D. AJAY MATHUR Cardiology 725 993 Which of the following is caused by an untranslated 998 Dynamic mutation refers to ? pentanucleotide repeat ? Harrison’s 18th Ed. Machado-Joseph Disease (MJD) D. Friedreich’s Ataxia is the most common form of inherited ataxia. 40 described in patients from India ? Harrison’s 18th Ed. A. Episodic ataxia -phenomenon called anticipation. SCA2. Spinocerebellar ataxia 2 996 Which of the following is a CAG repeat disease ? C. Spinocerebellar ataxia type 1 (SCA1) 994 Ataxins are ? Harrison’s 18th Ed. Spinocerebellar ataxia 1 and induce neuronal apoptosis. SCA7 C. Huntington’s disease and India. Increasingly severe phenotype in next generation B. dentatorubral-pallidoluysian atrophy (DRPLA) and X-chromosomal Spinobulbar Muscular A. 3341 D. Dystrophia B. B. other CAG repeat diseases include Huntington’s Harrison’s 18th Ed. C. None of the above B. Repeats alter gene regulatory sequences and tends to further expand C. 3341 A. 503 A. Repeat length increases from one generation to next 1003 Most common form of inherited ataxia is ? Harrison’s 18th Ed. comprising 1/2 of all hereditary ataxias. Machado Joseph disease (MJD) A. 3341 Expanded polyglutamine ataxins with more than ~40 glutamines are potentially toxic to neurons. SCA10 D. Decreasingly severe phenotype in next generation A. expanded repeat may increase further in length & result in an increasingly Autosomal spinocerebellar ataxias (SCAs) include SCA types 1 through SCA28. dentatorubropallidoluysian atrophy (DRPLA) & episodic ataxia (EA) types 1 and 2. 503. severe phenotype . Blue-yellow C. type II MJD . Type I MJD SCA5 (autosomal dominant type 5) maps to chromosome 11 & presentation includes ataxia & dysarthria. Atrophin 1 DRPLA is due to unstable CAG triplet repeats in the open reading frame of a gene “atrophin” located 1010 Which of the following is spared in Machado-Joseph Disease on chromosome 12p12-ter. Green-yellow B. Quadrupin D. 3342 A. Machado-Joseph Disease Harrison’s 18th Ed. Type I MJD Harrison’s 18th Ed. SCA1 D. It represents a unique spectrum of multiple system degenerations resembling Huntington’s disease. None of the above 1013 The color blindness in SCA7 is for which colour ? Harrison’s 18th Ed.dystonia type D. choreoathetosis & dementia. Type II MJD A. AJAY MATHUR Neurology 1004 Amyotrophic lateral sclerosis .amyotrophic lateral sclerosis–parkinsonism–dystonia type). 3342 1007 Which is the most common clinical type of MJD ? Harrison’s 18th Ed. Type II is the most common form of MJD. Type I MJD . SCA3 Harrison’s 18th Ed. None of the above spinocerebellar ataxias (SCAs) ? Harrison’s 18th Ed. Type III MJD B. Cardinal features in children are mental retardation. 1014 Haw River syndrome relates to which of the following ? Harrison’s 18th Ed. Type III MJD . 3342 B. Dentate nucleus of cerebellum presentation is in which clinical type of MJD ? Sparing of the inferior olives distinguishes MJD from other dominantly inherited ataxias. Inferior olives presentation similar to Huntington disease (HD) ? Harrison’s 16th Ed. Type III MJD D. 3342 A. D. 5 years C. 3342 A. myoclonus.726 MCQ’s FOR MEDICAL PROFESSIONALS BY PROF.ataxic type. 3342 1015 Gene related to Dentatorubropallidoluysian Atrophy is ? A. SCA1 type of MJD ? B. All of the above C.ataxic-amyotrophic type. 3342 B. Friedreich’s Ataxia B. Type III MJD 1012 ‘Retinal pigmentary degeneration’ occurs in which of the following D.parkinsonism . (MJD) ? Harrison’s 18th Ed. Pars compacta of substantia nigra . and epilepsy. None of the above B. Type I MJD B. MJD patients retain full intellectual function. D. Any of the above D. 2422 C. Episodic Ataxia C. MJD ? Harrison’s 18th Ed. Cardinal features in adults are ataxia. visual abnormalities first appear as blue-yellow color blindness and proceed to frank visual loss with macular degeneration. Dentatorubropallidoluysian atrophy (DRPLA) A. SCA2 Harrison’s 18th Ed. SCA7 C. 3342 1008 The mean age of onset of symptoms in MJD is ? A. atrophy and dentatorubropallidoluysian atrophy (DRPLA). 25 years five generations of an African-American family in rural North Carolina. 1009 Patients retain full intellectual function in which clinical type of ATN1 (DRPLA) is the only gene associated with DRPLA. All of the above In SCA7. SCA2 1005 Ataxic type presentation is seen in which type of MJD ? C. spinocerebellar The mean age of onset of symptoms in MJD is 25 years. Dentatin C. 2420 B. Machado-Joseph disease B. Type II MJD C. Corpus striatum A. MJD is of three clinical types. a neurodegenerative disease that is primarily reported in Japan. Harrison’s 18th Ed. 15 years Haw River Syndrome (HRS) is an autosomal dominant neurodegenerative disease that has affected D. behavioral changes. 10 years D. 3342 1006 Ataxic-amyotrophic type presentation is seen in which clinical A. Type II MJD D. 3342 D. Type I MJD 1011 Which of the following spinocerebellar ataxias (SCAs) has pure B. Simplin Usually. Type I MJD C. SCA1 C. 3342 1016 Which of the following spinocerebellar ataxias (SCAs) has A. Type II MJD cerebellar presentation ? Harrison’s 16th Ed. Type III MJD A. Red-yellow A. B. Type II MJD C. SCA5 A. Type III MJD SCA7 is distinguished from all other SCAs by the presence of retinal pigmentary degeneration. occur spontaneously or B. chorea. in part by signaling in a retrograde fashion from postsynaptic to the presynaptic cell. 2527 1017 Which of the following is not a potassium ion channelopathy ? A. Generalized epilepsy with febrile convulsions plus 1019 Which of the following is not a calcium ion channelopathy ? 1026 Which of the following is false about neurotransmitter receptors ? Harrison’s 18th Ed. SCA3 C. Spinocerebellar ataxia (SCA) Episodes of ataxia in EA 1 & 2. Enkephalin A. 3225 Table 366-1 Harrison’s 18th Ed. Nitric oxide C. Benign neonatal familial convulsions D. after whom D. Lithium D. C. exercise. Electroconvulsive therapy (ECT) Harrison’s 16th Ed. Hyperkalemic periodic paralysis D. Jervell & Lange-Nielsen syndrome 1025 Which of the following neurotransmitter function by signaling in a retrograde fashion . AJAY MATHUR Cardiology 727 B. Spinobulbar muscular atrophy (SBMA) A. 2420 1027 In episodic ataxia. Metabotropic receptors interact with G proteins C. 2420 Symptoms & episodic attacks of episodic ataxia type 1 & 2 are responsive to acetazolamide A.from postsynaptic to presynaptic cell ? Episodicataxia-2 is a calcium ion channelopathy. B. Hunger B. SCA17 D. 370. Kinetics of ionotropic receptor effects are fast Ionotropic receptors are multiple-subunit structures. Kinetics of ionotropic receptor effects are fast (<1 ms) whereas metabotropic dominant mutations include all except ? receptors function over longer periods. 2420 A. D. Hypokalemic periodic paralysis Nitric oxide and carbon monoxide are gases that also function as neurotransmitters. Wilson’s disease 1029 What was the nationality of Nicholaus Friedreich. British 1023 Neurologic genetic disorders inherited as Mendelian autosomal recessive include all except ? B. Ataxia telangiectasia SCAs & HD show broad phenotypic overlap including ataxia. Exercise 1022 Neurologic genetic disorders inherited in Mendelian autosomal dominant mutations include all except ? C. Familial Hyperkalemic periodic paralysis (HYPP) A. 3342 C. Episodic ataxia-1 C. 3226 A. Autosomal dominant progressive deafness D. or excessive fatigue. Ataxia telangiectasia 1028 Episodic ataxia episodes are precipitated by ? C. SCA2 B. spasticity. Neuropeptide Y B. Fragile X syndrome B. with gait imbalance & slurring of speech. Friedreich’s ataxia (FA) C. Tuberous sclerosis are precipitated by sudden movement. Friedreich’s ataxia (FA) . Substance P Harrison’s 18th Ed. 3342 D. Meals A. Charcot-Marie-Tooth disease 1A (CMT 1A) Harrison’s 18th Ed. Similar phenotypes have been observed in SCA17. Later three symptoms in a hereditary context should 1024 Neurologic genetic disorders inherited as X-linked recessive traits favor a genetic test for HD. Ionotropic receptors are single subunits D. 3225 Table 366-1 B. Paramyotonia congenita C. Harrison’s 18th Ed. Charcot-Marie-Tooth disease 1A (CMT 1A) Friedreich’s ataxia has been named ? A. French Harrison’s 16th Ed. Sleep Harrison’s 16th Ed. 2420 D. Duchenne muscular dystrophy (DMD) Harrison’s 18th Ed. Spinocerebellar ataxia-6 B. Dystrophia myotonica (DM) (AZM) or or anticonvulsants. Acetazolamide dominant mutations include all except ? D. Familial Alzheimer’s Disease Harrison’s 18th Ed. symptoms and episodic attacks of ataxia A. dementia & mood disorders. include all except ? Harrison’s 16th Ed. Parkinsonian features. Harrison’s 16th Ed. 727 Neurology MCQ’s FORMCQ’s FOR MEDICAL MEDICAL PROFESSIONALS PROFESSIONALS BY PROF. Episodic ataxia-2 D. dystonia. Ionotropic receptors are direct ion channels B. Tuberous sclerosis D. Friedreich’s ataxia (FA) B. Familial hemiplegic migraine C. Huntington’s Disease (HD) responsive to ? B. 3226 1018 Which of the following is not a sodium ion channelopathy ? A. whereas metabotropic receptors are composed 1020 Neurologic genetic disorders inherited in Mendelian autosomal of single subunits only. Episodic ataxia-2 A. stress. 3225 Table 366-1 B. excitement. Amyotrophic lateral sclerosis (ALS) A. Atypical antipsychotic drugs 1021 Neurologic genetic disorders inherited in Mendelian autosomal C. Wilson’s disease C. acute intermittent porphyria.1. Antiretroviral therapy neuropathy B. 4 Harrison’s 18th Ed. Iron chelators and antioxidants are potentially harmful Friedreich’s ataxia is an autosomal recessive congenital ataxia and is caused by a mutation in D. Diphtheria C. Presents before 25 years of age D. amyloidosis. the They strongly predispose to neoplasia because of the rapid acquisition of additional mutations mutation decreases the transcription of the gene) through induction of a heterochromatin structure. Lepromatous leprosy Cardiomegaly. Diabetes mellitus is found in 20%. lead & dapsone. the mutation causes gene silencing (i. brachial neuropathy. Friedreich’s ataxia tendon reflexes are found. Extensor plantar responses (with normal tone in trunk & extremities). Lepromatous leprosy Harrison’s 18th Ed. symmetric hypertrophy. absence of deep B. multifocal motor neuropathy Primary pathology in Friedreich’s ataxia is in spinal cord (sclerosis and degeneration in (MMN).900 GAA repeats (normal 7 . All of the above D. pes equinovarus and progressive scoliosis common. Pes cavus. Iron chelators and antioxidants are potentially harmful. Lower extremities more severely involved than upper 1034 Which of the following is a cause of pure sensory neuropathy ? C. 3343 1038 Which of the following about Friedreich’s ataxia is false ? A. Ataxia telangiectasia C. 3343 C. B. murmurs & conduction defects reported in ~90% of patients. 2665 Harrison’s 18th Ed. the condition in 1860s. patients have 200 . spinocerebellar tracts. None of the above gene FXN (formerly known as X25) that codes for frataxin. diabetic small-fiber neuropathy. C. 1032 Classic form of Friedreich’s ataxia is mapped to chromosome ? Harrison’s 18th Ed. Severe mental retardation Harrison’s 17th Ed. 6 B. Bloom’s syndrome B. 1036 Which of the following is a cause of pure motor neuropathy ? 1030 Primary site of pathology in Friedreich’s ataxia is ? Harrison’s 17th Ed. 3343 DNA mismatch/repair ? Harrison’s 18th Ed. located on chromosome 9q13-q21. Cisplatin neuropathy D.e. Musculoskeletal deformities are common A. All of the above Causes of predominantly motor neuropathies include GBS. All of the above The classic form of Friedreich’s ataxia is mapped to 9q13-q21. Ataxin 2 C. Mutant gene frataxin contains expanded GAA triplet repeats in the first intron. All of the above D. xeroderma pigmentosum. German D. 3343 A. vitamin B12 neuropathy. All of the above 1040 Which of the following about Friedreich’s ataxia is false ? Harrison’s 18th Ed. Acute intermittent porphyria B. All of the above D. dorsal root ganglion neuropathy due to spinal muscular atrophy. Altered protein is called frataxin. Spinal cord B. Fabry’s disease & Friedreich’s ataxia is named after the German physician Nicholaus Friedreich. lepromatous leprosy. and hereditary nonpolyposis colon cancer (HNPCC). Cardiac involvement occurs in 90% of patients neuropathy associated with Sjögren syndrome. Frataxin mRNA and frataxin protein are low in Friedreich’s ataxia. 2665 D. Instead.1.. 3343 B. Diabetic small-fiber neuropathy Mental retardation is uncommon. GBS A. Spanish A painful sensory neuropathy is an early feature of hereditary sensory neuropathies. 9 C. Frataxin A. Tangier Disease (TD). Flexor plantar responses Harrison’s 17th Ed. Hereditary nonpolyposis colon cancer (HNPCC) D. sensory A. lateral corticospinal tracts and posterior columns). pyridoxine toxicity & cisplatin neuropathy. A. who first described dysautonomia. this mutation does not result Bloom’s syndrome. cells and peripheral nerves (loss of large myelinated fibers). High incidence of diabetes mellitus 1035 Painful sensory neuropathy is an early feature of ? C. Loss of vibratory & proprioceptive sensation occurs. Lower extremities are more severely involved than upper. Diabetes mellitus A. CIDP. Ataxin 1 B. C. in the production of abnormal frataxin proteins. idiopathic sensory neuropathy. Frataxin is a mitochondrial protein involved in iron homeostasis. Frataxin is a mitochondrial protein for iron homeostasis C. Excess oxidized intramitochondrial iron D. Peripheral nerves D. Because the defect is located on an intron (which is Genetic abnormalities of DNA mismatch/repair include Fanconi anemia. removed from the mRNA transcript between transcription and translation). ataxia telangiectasia. B. diphtheria. Pyridoxine toxicity Friedreich’s ataxia presents before 25 years of age. In Friedreich’s ataxia. 3343 Causes of pure sensory neuropathy include Friedreich’s ataxia. 2665 D.728 MCQ’s FOR MEDICAL PROFESSIONALS BY PROF.22). AJAY MATHUR Neurology C. 5 A. diabetic lumbosacral radiculoplexus neuropathy (diabetic amyotrophy). Mitochondrial iron accumulation due to loss of the iron transporter coded 1033 Which of the following is a large-fiber sensory neuropathy ? by the mutant frataxin gene results in oxidized intramitochondrial iron resulting in oxidation of cellular Harrison’s 18th Ed. Absence of deep tendon reflexes A. 3343 components & irreversible cell injury. Dorsal root ganglion cells C. 488 A. Amyloidosis . 1037 Mutant gene in classic form of Friedreich’s ataxia is ? 1031 Which of the following diseases is due to genetic abnormalities of Harrison’s 18th Ed. Friedreich’s ataxia 1039 Which of the following about Friedreich’s ataxia is false ? B. Due to mutations in gene for  tocopherol transfer protein 1050 Which of the following is false about Ataxia Telangiectasia ? B. These patients have an impaired ability to bind vitamin E into the VLDL produced and secreted by the liver. ATM gene is homologous to genes involved in DNA repair & control of cell cycle A. Defects in MTP result in impairment of formation and secretion of VLDL in liver (abetalipoproteinemia). Autosomal recessive Harrison’s 17th Ed. Also. 3343 A. None of the above B. Ascorbic acid 1048 Mutation of which of the following gene causes a variant of AT (AT-like disease) ? D. C. This defect results in a deficiency of delivery of vitamin E to tissues. Nijmegen breakage syndrome (NBS1) lymphoid system is ? C. Present in first decade of life D. Splenomegaly 1046 Gene responsible for Ataxia Telangiectasia is found on which D. . Ataxia-telangiectasia gene . 2571 D. Chapter e39 Idebenone. All of the above 1044 Which of the following is false regarding ataxia with vitamin E AT causes a B cell immunodeficiency (low IgA. CHD7 Harrison’s 18th Ed. Generalised lymphadenopathy Other names for ataxia-telangiectasia are A-T. STAT5b A. Ataxia-telangiectasia mutated B. 729 Neurology MCQ’s FORMCQ’s FOR MEDICAL MEDICAL PROFESSIONALS PROFESSIONALS BY PROF. Chromosome 11 1052 Telangiectasia in Ataxia Telangiectasia usually occur on ? B. Cerebellar SOL B. 3343 D. Louis-Bar syndrome. neck and extremities. It is caused by a defect in microsomal triglyceride transfer B. ATM. All of the above Ataxia Telangiectasia is inherited as an autosomal recessive disorder and offending gene is found on the long arm of chromosome 11 at 11q22-23. Poorly developed or absent thymus gland B. Abetalipoproteinemia 1049 AT causes which of the following ? D. Mutations in the ATM gene give rise to defects in meiosis as well as increasing susceptibility to damage from ionizing radiation. MRE11 1043 Which of the following is a feature of Bassen-Kornzweig syndrome ? C. Diabetes mellitus A variant of AT (AT-like disease) is caused by mutation of the MRE11 gene. 3343 Harrison’s 18th Ed. 669 Table 83–3 A. a short-chain analogue of coenzyme Q10 when used at an early stage reduces A. Pott’s disease A. Harrison’s 18th Ed. 496 A. neoplasms of lymphatic and reticuloendothelial system 1045 Ataxia Telangiectasia is also known as ? D. Louis-Bar syndrome 1051 In Ataxia Telangiectasia (AT). B. Cardiomyopathy C. D. Idebenone C. Harrison’s 18th Ed. cerebello-oculocutaneous. Chromosome 14 C.ATM is on chromosome 11 and ATM stands for ataxia-telangiectasia mutated. Desferrioxamine checkpoints. SBDS progression of cerebellar manifestation. B. Intracranial D. Telangiectasia in AT usually occur on the white portion of eye (bulbar conjunctiva) but may also 3-kinase-like enzyme involved in cellular responses to stress. Ataxia-telangiectasia magnified Neuroimaging reveals thinning of the cervical spinal cord. Obstructive hydrocephalus D. All of the above Harrison’s 18th Ed. Telangiectasia. Hypothyroidism Harrison’s 18th Ed. Chromosome 13 B. Low IgA Bassen-Kornzweig syndrome is an autosomal recessive disorder in which a person is unable to fully absorb dietary fats through intestines. Low antibody production the central & peripheral nervous system. AT is associated with a progressive T cell immunodeficiency. 2571 C. High incidence of recurrent pulmonary infections and resulting in a deficiency of vitamin E in peripheral tissues. DNA damage and cell cycle control. B. Ataxia Telangiectasia Syndrome.the gene for Ataxia Telangiectasia is ? Harrison’s 18th Ed. IgG2 deficiency. AJAY MATHUR Cardiology 729 1041 Which of the following MRI finding is typical of Friedreich’s ataxia ? 1047 Full name of ATM . Chapter e39 A. as VLDL is the transport molecule for vitamin E. It controls the production of a phosphatidylinositol. Nail bed D. None of the above A. Brainstem atrophy A. including C. Lymphedema chromosome ? Poorly developed or absent thymus gland is the most consistent defect of the lymphoid system. Ataxia-telangiectasia morphed D. the most consistent defect of the B. Progressive telangiectatic lesions with deficits in cerebellar function & nystagmus AVED is due to mutations in the gene for alpha-tocopherol transfer protein (  TTP). be found on the ears. Spinal cord atrophy C. and low antibody production) that deficiency (AVED) ? often requires immunoglobulin replacement therapy. Bulbar conjunctiva C. Ataxia-telangiectasia manifested C. C. Chromosomal localization is on 8q13 A. Chromosome 12 A. IgG2 deficiency protein (MTP) gene. Lou Gehrig’s disease Harrison’s 17th Ed. ATM encodes a protein that is predominantly confined to nucleus of cells and that remains constant throughout all stages of the cell cycle and participate in cell’s responses to 1042 Which of the following drugs is of use in Friedreich’s ataxia ? genomic damage. the structures that are left intact C. Amyotrophic lateral sclerosis of Ataxia Telangiectasia ? B. All of the above C. Parasympathetic neurons in sacral spinal cord Motor Neurone Disease (MND). or Onuf) that innervate the sphincters of the bowel and bladder. All of the above Since mtDNA is transmitted exclusively by the mother. Cardiovascular event involvement of upper motor neurons is ? Harrison’s 18th Ed. Ceruloplasmin level in blood D. Subacute combined degeneration of cord A. Respiratory paralysis 1058 Chronic sporadic motor neuron disease with predominant D. 3345 A. after the French doctor Jean-Martin Charcot. The French refer to it as Maladie de Charcot. D. A. 3347 Table 374-2 motility remain unaffected. All of the above 1057 Acute sporadic motor neuron diseases include ? In ALS. Sensory apparatus 374 .730 MCQ’s FOR MEDICAL PROFESSIONALS BY PROF. include ? Harrison’s 18th Ed. Cryofibrinogen C. as do the parasympathetic neurons in the sacral spinal cord (the A. cerebellar ataxia. Herpes zoster and Coxsackie virus. . B. Motor neurons required for ocular Harrison’s 18th Ed. Entire sensory apparatus. 3347 Table 374-2 In ALS. Centers for cognitive processes D. Herpes zoster 1063 In ALS. the cause leading to death is ? C. In Multifocal motor neuropathy with D. Sensory apparatus D. Multiple sclerosis include ? Harrison’s 18th Ed. predominantly upper motor neurons are involved. the cause leading to death is respiratory paralysis. 2571 B. All of the above C. 2571 B. Affected child has normal intelligence C. there is selectivity of involvement. 3346 D. Multifocal motor neuropathy with conduction block false ? Harrison’s 17th Ed. Depression and suicide C. Hypersensitivity to ionizing radiation D. Myoclonus epilepsy with ragged red fibers (MERRF) D. Infections Acute sporadic motor neuron diseases include Poliomyelitis. Insulin-resistant diabetes mellitus associated with anti-insulin antibodies occurs in patients with 1060 Diseases that affect only upper motor neurons innervating the ataxia-telangiectasia. and familial spastic paraplegia (FSP) affect only upper motor neurons innervating the brainstem and spinal cord. All of the above 1056 Lou Gehrig’s disease is the other name of ? A remarkable feature of ALS is the selectivity of neuronal cell death. the structures that are left intact inheritance and can be excluded when paternal transmission of the disease occ urs in the family. who first wrote about ALS in 1869. Myasthenia gravis 1062 In amyotrophic lateral sclerosis. Primary lateral sclerosis (PLS) A. primary lateral sclerosis (PLS).344:1688 regulatory mechanisms for the control and coordination of movement and the components of brain that are needed for cognitive processes remain intact. Motor neuropathy with paraproteinemia B. Pseudobulbar palsy Harrison’s 17th Ed. Apolipoprotein A-1 Alpha-fetoprotein levels in the blood are elevated (> 10 ng/ml) in >95% of patients of AT. N Engl J Med 2001. In Primary lateral sclerosis. Kearns-Sayre syndrome C. Predisposition to malignancy In Amyotrophic lateral sclerosis. All of the above A. AJAY MATHUR Neurology 1053 Which of the following laboratory tests is useful in the diagnosis A. People in England and Australia call ALS as B. Motor-predominant peripheral neuropathies B. Amyotrophic lateral sclerosis B. 3347 Table 374-2 1054 Which of the following statements about Ataxia-telangiectasia is A. C. AFP is 1059 Chronic sporadic motor neuron disease with predominant produced only during fetal development. brainstem and spinal cord are ? Harrison’s 18th Ed. mitochondrial cytopathies show maternal 1061 In amyotrophic lateral sclerosis. involvement of lower motor neurons are ? Harrison’s 18th Ed. a hall-of-fame baseball player for New York Yankees who suffered with ALS. Coxsackie virus Harrison’s 18th Ed. Motor neuropathy with paraproteinemia or cancer A. AT causes large telangiectatic lesions of face. within motor system. predominantly lower motor neurons are affected. Affected child has normal or above normal intelligence. Multifocal motor neuropathy with conduction block A. Familial spastic paraplegia (FSP) B. Mitochondrial encephalopathy with lactic acidosis and stroke- like episodes (MELAS) Pseudobulbar palsy. Primary lateral sclerosis C. 3345 D. B. None of the above conduction block. Centers for control & coordination of movement Other Motor Neuron Diseases C. Motor neuropathy with paraproteinemia or cancer and Motor-predominant peripheral neuropathies. Motor neurons for ocular motility ALS is often called Lou Gehrig’s disease after Lou Gehrig (1903-1941). D. both upper & lower motor neurons are involved. Alpha-fetoprotein level in blood Chronic sporadic MND with predominant involvement of UMN is primary lateral sclerosis. Poliomyelitis nucleus of Onufrowicz. 3345 1055 Hereditary forms of mitochondrial cytopathies include ? A.Amyotrophic Lateral Sclerosis and B. immunologic defects & hypersensitivity to ionizing radiation. None of the above microgliosis. B. 3349 D.& polyclonal antibodies to ganglioside GM1 Riluzole is approved for ALS. Motor neuron disease fatal course. within a highly polymorphic region of androgen receptor is associated with spinal & bulbar muscular atrophy (Kennedy disease). Chronic lead poisoning D. thyrotoxicosis. AJAY MATHUR Cardiology 731 1064 Which of the following is not a feature of ALS ? 1070 Which of the following is false for Kennedy’s Disease ? Harrison’s 18th Ed. Azithromycin 1071 Werdnig-Hoffmann Disease is the name given to ? Harrison’s 18th Ed. Amyotrophic lateral sclerosis 1076 Which of the following is false about ‘Multifocal Motor Neuropathy B. and multiple sclerosis. 3347 B. None of the above A. 3350 B. Absence of pain or of sensory changes A. Elevated serum titers of mono. . These patients may show evidence of partial 1065 Which of the following antibiotics has neuroprotective properties ? androgen insensitivity in adolescence or adulthood (gynecomastia). Multiple sclerosis with Conduction Block’ (MMCB) ? C. Thyrotoxicosis D. Normal roentgenographic studies of spine C. A. N Engl J Med 2003. 3350 1066 Multifocal motor neuropathy with conduction block (MMCB) evolves in association with which of the following ? A. chronic lead poisoning. Tuberculosis C. or alpha-glucosidase deficiency. EP studies distinguish SMA III from myopathic syndromes. Subacute combined degeneration of cord D. None of the above Absence of pain or of sensory changes. Subacute combined degeneration of cord Harrison’s 18th Ed. brain injury. Vancomycin C. Myasthenia gravis 1074 Which of the following is false about spinal muscular atrophy ? Harrison’s 18th Ed. androgen receptor. cervical or foramen magnum tumors. normal bowel and bladder function. Minocycline A. 3350 D. 3346 Harrison’s 18th Ed. Minocycline inhibits ischemia-induced up-regulation of nitric oxide synthase. Chronic childhood SMA (SMA II) begins later in childhood & evolves with slowly B. Trinucleotide (CAG) repeat expansion. Neuroprotection by minocycline has been observed in Huntington’s disease. 3350 D/D of ALS includes cervical spinal cord compression. 3347 B. All of the above Harrison’s 18th Ed. 3350 A. Spiramycin B. SMA III Minocycline is a second-generation tetracycline with neuroprotective properties.348:1365-75 A. SMA III B. 3350 1068 Differential diagnosis of amyotrophic lateral sclerosis is ? A. SMA III feature of ? D. Early onset A. and reactive D. Parkinson’s disease. 731 Neurology MCQ’s FORMCQ’s FOR MEDICAL MEDICAL PROFESSIONALS PROFESSIONALS BY PROF. and normal cerebrospinal fluid (CSF) all favor ALS. Multiple sclerosis progressive course. SMA I Harrison’s 18th Ed. Gynecomastia and reduced fertility D. SMA II 1067 Intraneuronal inclusions “Bunina bodies” are a pathognomonic C. lower motor axonal neuropathy mimicking ALS which sometimes evolves in A. All of the above 1075 Which of the following is false about spinal muscular atrophy ? Harrison’s 18th Ed. C. Locus on chromosome 5 B. Compression of cervical spinal cord C. SMA II D.a glutamate antagonist is useful in treatment of ? C.344:1688 Infantile SMA or SMA I is known as Werdnig-Hoffmann disease with earliest onset & most rapidly A. Lower motor neuron disorder C. normal roentgenographic Azoospermia & male-factor infertility occurs in association with mild loss of function mutations in studies of the spine. SMA I association with lymphoma or multiple myeloma. None of the above N Engl J Med 2001. Juvenile SMA (SMA III or Kugelberg-Welander disease) manifests in late childhood & has a slow course. Electrophysiologic evidence of denervation Harrison’s 18th Ed. Syphilis 1073 Which out of the following carries the worst prognosis ? D. Myasthenia gravis A. 1072 Kugelberg-Welander disease is the name given to ? Harrison’s 18th Ed. 3350 MMCB is a diffuse. None of the above C. SMA II A. cervical spondylosis. None of the above C. Riluzole reduces excitotoxicity by diminishing glutamate release. SMA I C. Raised proteins in cerebrospinal fluid (CSF) D. from a mean of 22 repeats to greater than 40 repeats. ALS. cervicomedullary junction tumors. X linked spinobulbar muscular atrophy B. caspase 1. B. Lymphoma D. Evidence of denervation atrophy on muscle biopsy 1069 Riluzole . Genetic lower motor neuron disease Harrison’s 18th Ed. Extensive loss of large motor neurons hexosaminidase A. Normal bowel and bladder function B. 1082 Protein restricted to PNS myelin is which of the following ? Harrison’s 16th Ed. Sexual function preserved B. A. Gene implicated is spastin & atlastin in dominantly the central nervous system in all except ? inherited and childhood-onset dominant FSP respectively. Long survival Connections between cerebral cortex & autonomic centers in brainstem coordinate autonomic outflow with higher mental functions. Third cranial nerve A. Postganglionic sympathetic ANS . P0 B. GM 1 A. P0 1077 Which of the following is true for Fazio-Londe syndrome ? C. GM 1 D.732 MCQ’s FOR MEDICAL PROFESSIONALS BY PROF. Third sacral nerve B. most adult-onset FSP are transmitted as an autosomal dominant trait. seventh. Preganglionic parasympathetic ANS B. 3351 1081 Protein restricted to CNS myelin is which of the following ? A. In its pure form. T1 and L4 segments D. Juvenile onset C. T1 and L1 segments Harrison’s 16th Ed. AJAY MATHUR Neurology B. Proteolipid protein (PLP) B. ninth & tenth cranial nerves as well as C. 3350 C. 3351 (FSP) ? Harrison’s 18th Ed. Brainstem B. Degeneration of corticospinal tracts Harrison’s 18th Ed. 3351 (FSP) in adults ? Harrison’s 18th Ed. 3351 occurs in FSP. Cerebral cortex A. Fragile X syndrome D. Eleventh cranial nerve Harrison’s 18th Ed. Myelin oligodendrocyte glycoprotein (MOG) Harrison’s 18th Ed. Spastin and atlastin gene mutations common C. spinal cord between ? Harrison’s 18th Ed. None of the above B. Presents in third or fourth decade C. Involves musculature innervated by brainstem mainly 375 . Second sacral nerve A. Pelizaeus-Merzbacher syndrome Preganglionic neurons of PNS leave CNS in third. T1 and L2 segments B. Proteolipid protein (PLP) A. 2341 C. 1085 Preganglionic neurons of parasympathetic nervous system leave the central nervous system in all except ? 1079 Which of the following is false for Familial Spastic Paraplegia Harrison’s 18th Ed. 3350 D. None of the above 1086 Preganglionic neurons of parasympathetic nervous system leave In FSP. FSP has long survival because respiratory function is spared. Spinal cord C. PMP22 D. sexual function tends to be preserved. Sixth cranial nerve B. Progressive spastic weakness in lower extremities D. 1084 Connections between cerebral cortex and autonomic centers occurs in ? 1078 Which of the following is false for Familial Spastic Paraplegia Harrison’s 18th Ed. Myelin oligodendrocyte glycoprotein (MOG) C. most severe at more caudal levels in spinal cord. All of the above Autonomic Nervous System Fazio-Londe syndrome is an ALS variant of juvenile onset that involves mainly the musculature innervated by the brainstem.Disorders of the D. ALS variant B. Proteolipid protein (PLP) D. 2341 1088 Acetylcholine is the neurotransmitter for all except ? Harrison’s 18th Ed. T1 and L3 segments C. 3350 A. Rett syndrome 1087 Preganglionic neurons of sympathetic nervous system exit the Pelizaeus-Merzbacher disease is a widespread disorder of CNS myelin. No corticospinal tract signs 1083 Protein in both CNS & PNS myelin is ? Harrison’s 16th Ed. Respond to intravenous immunoglobulin therapy A. Postganglionic parasympathetic ANS D. X linked transmission B. D. All of the above D. 3350 A. Tenth cranial nerve D. GM 1 Preganglionic neurons of sympathetic nervous system exit spinal cord between first thoracic and the second lumbar segments. Degeneration of corticospinal tracts D. 2341 A. Tenth cranial nerve 1080 Which of the following is a disorder of CNS myelin ? B. Prader-Willi syndrome the second & third sacral nerves. Seventh cranial nerve C. Preganglionic sympathetic ANS C. P0 C. 3351 A. Preganglionic parasympathetic ANS C. Insulin Harrison’s 18th Ed. Glucagon A. Tearing D. Late phase II results in LMN flaccid bladder (large volumes. All of the above D. All of the above LMN bladder dysfunction. Bowel & bladder function B. Beta adrenergic blockers 1100 The ‘Valsalva ratio’ is defined as ? Harrison’s 18th Ed. 733 Neurology MCQ’s FORMCQ’s FOR MEDICAL MEDICAL PROFESSIONALS PROFESSIONALS BY PROF. urinary frequency & overflow incontinence). All of the above C. Bladder volume is a useful test for distinguishing between Harrison’s 18th Ed. Persistent A. orgasm 1097 Heart rate variation with deep breathing (respiratory sinus 1091 With parasympathetic activation. a constant expiratory pressure of 40 mmHg is maintained for A. dopamine agonists. Upper motor neuron lesion leads to spastic bladder 15 seconds while measuring changes in heart rate and beat-to-beat BP. Diuretics peripheral arteriolar vasoconstriction and increased cardiac adrenergic tone results in a temporary BP overshoot and phase IV bradycardia (mediated by baroreceptor reflex). Norepinephrine (NE) is the 1096 OH is defined as. a sustained neurotransmitter of postganglionic sympathetic neurons. antidepressants. barbiturates & calcium channel blocking agents. Venous return and cardiac output return to normal in phase IV. Increased bowel motility B. narcotics. a constant expiratory pressure of how A. except for cholinergic neurons innervating drop in SBP and DBP of ? eccrine sweat glands. phenothiazines. Phase I D.8 seconds after the onset of maneuver. Sleep much is maintained for 15 seconds ? Harrison’s 18th Ed. Measurement of post-void residual bladder volume is useful test to distinguish between UMN & D. which of the following does not A. Penile erection C. In early phase II. Ejaculation. Lower motor neuron lesion leads to flaccid bladder 1099 Fall in BP occurs in which phase of Valsalva maneuver ? C. Sweating HRDB or Heart rate variation with deep breathing (respiratory sinus arrhythmia) is abolished by atropine but is unaffected by sympathetic blockade (propranolol). antihypertensives. D. Preganglionic sympathetic ANS D. phases I & III are mechanical & related to changes in intrathoracic & intraabdominal pressure. AJAY MATHUR Cardiology 733 1089 Norepinephrine (NE) is the neurotransmitter for ? A. Maximum phase 2 tachycardia divided by minimum phase 4 Harrison’s 18th Ed. Calcium channel blockers A. Sweating three minutes of standing. 3352 Table 375-1 C. 2429 1098 In Valsalva maneuver. None of the above B. Postganglionic sympathetic ANS ethanol. 3352 BP (adrenergic). Ethanol C. 20 and 10 mm Hg respectively Harrison’s 18th Ed. Phenothiazines C. 40 mm Hg urinary bladder is false ? Valsalva maneuver assesses the integrity of baroreflex control of heart rate (parasympathetic) & Harrison’s 18th Ed. 2578 1090 With sympathetic activation. Pupils A. 10 mm Hg C. Atropine A. 3355 UMN and LMN bladder dysfunction A. Increased bladder sphincter tone OH is defined as a sustained drop in systolic (>=20 mmHg) or diastolic (>=10 mmHg) BP within C. 1092 Which of the following is regulated by autonomic circuits ? Harrison’s 16th Ed. 30 mm Hg 1093 Which of the following statements about autonomic control of D. 3354 . Beta blockers C. 20 and 20 mm Hg respectively occur ? B. 3352 Table 375-1 A. D. Late phase II begins with a progressive rise in BP towards or above baseline. Acetylcholine (ACh) is the preganglionic neurotransmitter for both divisions of ANS as well as postganglionic neurotransmitter of parasympathetic neurons. 3351 B. B. With subject supine. 3355 occur ? Harrison’s 18th Ed. Digoxin B. Increased bowel motility D. Harrison’s 17th Ed. insulin. 3354 bradycardia . Out of four phases of Valsalva maneuver response. B. which of the following does not arrhythmia) is abolished by administration of ? Harrison’s 18th Ed. Postganglionic parasympathetic ANS Drugs that may cause OH are diuretics. 3355 B. 10 and 5 mm Hg respectively B. counteracted by a combination of reflex tachycardia and except ? increased total peripheral resistance. 20 mm Hg D. reduced venous return 1094 Drugs that can cause Orthostatic hypotension (OH) include all results in a fall in stroke volume and BP. within 3 minutes of standing up. 3355 D. 10 and 10 mm Hg respectively A. PNS disease C. Maximum phase 1 tachycardia divided by minimum phase 4 1095 Drugs that can cause Orthostatic hypotension (OH) include all bradycardia except ? B. Increased total peripheral resistance arrests the BP drop ~5 Harrison’s 18th Ed. Barbiturates B. Early phase II Brain & spinal cord disease above lumbar level results in UMN or spastic bladder. 3356-57 1104 If both QSART and TST are absent. 3357 A. Parkinson’s disease 1102 Which of the following is a test for sweating ? Harrison’s 18th Ed. 1106 Multiple system atrophy (MSA) comprises autonomic failure and 1112 Autoantibodies against which of the following is present in which of the following ? autoimmune autonomic neuropathy (AAN) ? Harrison’s 18th Ed. Measurement of galvanic skin responses cord lesion above which level ? D. A postganglionic lesion is present if both QSART and TST show absent sweating. Thermoregulatory sweat test (TST) 1109 Autonomic dysreflexia is common in patients with traumatic spinal C. Maximum phase 3 tachycardia divided by minimum phase 4 Multiple system atrophy (MSA) comprises of autonomic failure (OH and/or a neurogenic bladder) bradycardia combined with either striatonigral degeneration (Shy-Drager syndrome) or sporadic olivopontocerebellar atrophy. None of the above 1110 Neuromuscular junction disorders accompanied by autonomic A reduced or absent response indicates a lesion of the postganglionic sudomotor axon. Postganglionic sudomotor axon B. B. Shy-Drager syndrome C. Postganglionic sudomotor axon D. All of the above B. ~ 14 In a preganglionic lesion. D. A3 AChR B. B. Preganglionic parasympathetic ANS D. ~5 B. QSART is intact but TST shows anhidrosis. Multiple system atrophy (MSA) C. Valsalva ratio is defined as the maximum phase II tachycardia Harrison’s 18th Ed. Horner’s syndrome A. Dementia with Lewy bodies (DLB) Sweating is induced by release of acetylcholine (ACh) from sympathetic postganglionic fibers. Preganglionic sympathetic ANS 1108 Autonomic dysfunction is a common feature in ? C. 3356 divided by minimum phase IV bradycardia. 3356 Harrison’s 18th Ed. chronic alcoholism. Wernicke-Korsakoff syndrome B. Guillain-Barré syndrome C. 3357 A. C3 AChR D. Pre & postganglionic sudomotor axon C. Quantitative sudomotor axon reflex test (QSART) Autonomic dysfunction is a feature in dementia with Lewy bodies. Neuromuscular junction disorders with autonomic involvement include botulism and Lambert-Eaton syndrome. lesion is in ? onset of diabetes mellitus ? Harrison’s 18th Ed.734 MCQ’s FOR MEDICAL PROFESSIONALS BY PROF. Lesion of pre & postganglionic sudomotor axon traumatic spinal cord lesion above the C6 level. T12 A. All of the above A. C6 1103 A reduced or absent response in ‘Quantitative sudomotor axon reflex test (QSART)’ indicates ? B. 3356 D. Preganglionic sudomotor axon C. Lesion of preganglionic sudomotor axon D. Levodopa-induced dyskinesia uncommon A. Postganglionic sympathetic ANS A. D. Lesion of postganglionic sudomotor axon Markedly increased autonomic discharge or autonomic dysreflexia is common in patients with C. L2 B. amyloidosis. ~ 10 D. porphyria. Chronic alcoholism B. None of the above D. Autonomic neuropathy typically begins ~10 years after the onset of DM. Pre & postganglionic sudomotor axon Peripheral neuropathies as in diabetes mellitus. Maximum phase 4 tachycardia divided by minimum phase 4 bradycardia 1107 Parkinsonism seen in multiple system atrophy features which of the following ? Autonomic function during the Valsalva maneuver can be measured using beat-to-beat blood pressure or heart rate changes. T3 Harrison’s 18th Ed. Postganglionic parasympathetic ANS Harrison’s 18th Ed. 1111 Autonomic neuropathy typically begins how many years after the 1105 If QSART is intact but TST shows anhidrosis. All of the above Harrison’s 18th Ed. the lesion is in ? A. Porphyria Harrison’s 18th Ed. 3356 Harrison’s 18th Ed. Not responsive to levodopa Harrison’s 18th Ed. B3 AChR C. Shapiro’s syndrome . 3356 A. A. The ratio reflects cardiovagal function. 3356 B. 3355 C. AJAY MATHUR Neurology C. No rest tremor 1101 Sweating is induced by release of acetylcholine from ? B. ~7 C. Preganglionic sudomotor axon A. involvement include ? Harrison’s 18th Ed. 3355 D. Botulism A. MSA & Parkinson’s disease. and D. None of the above Guillain-Barré syndrome lead to chronic autonomic insufficiency. 3356 C. 3358 Harrison’s 18th Ed. 3357 A. A. All of the above autoimmune autonomic neuropathy (AAN). toxins & drugs. Harrison’s 18th Ed. 3357 A. SPTLC2 B. Involvement of postganglionic sympathetic neurons 1120 Neuroleptic malignant syndrome is seen in psychotic patients treated with ? B. Glucosyl ceramide C. AJAY MATHUR Cardiology 735 D. CRPS type II is a regional pain syndrome that 1118 Cause of autonomic storm is ? develops after injury to a specific peripheral nerve. focal edema may occur alone or in combination but must be present for diagnosis. minor shoulder or limb injury. Sudomotor abnormalities C. an exaggerated pain response to a painful stimulus. IKBKAP A. Autonomic neuropathy . 735 Neurology MCQ’s FORMCQ’s FOR MEDICAL MEDICAL PROFESSIONALS PROFESSIONALS BY PROF. Amphetamine blocks ACh release. sudomotor abnormalities. Acetylcholine Harrison’s 18th Ed. Dermatan sulfate A. Norepinephrine 1119 Drug that can produce autonomic storm is ? B. Women affected five times more often than men B. Limb injury The defective gene in HSAN III is IKBKAP. Phenothiazine neurons that results in low supine plasma NE levels & noradrenergic supersensitivity. Serotonin A. or of SPTLC1 gene produce higher-than-normal levels of glucosyl ceramide triggering apoptosis. All of the above in heart rate to >120 beats/minute or an increase of 30 beats/minute with standing that subsides on sitting or lying down. Women are affected approximately five times more often than men. Phenylpropanolamine In Pure Autonomic Failure (PAF) there is primary involvement of postganglionic sympathetic C. Increase in heart rate of > 30 beats/min with standing A. SPTLC4 D. Heparan sulfate D. Stroke D. SPTLC3 C. Autonomic storm is an acute state of sustained sympathetic 1113 Botulinum toxin blocks release of which of the following ? surge. 3357 pheochromocytoma. usually a major nerve trunk. Acute autonomic syndrome may be either acute autonomic failure (acute AAN syndrome) or a state of sympathetic overactivity. 3359 A. hyperthermia & hypertension in psychotic Tachycardia Syndrome (POTS) ? patients treated with phenothiazines. CRPS type I is a regional pain syndrome that usually develops after tissue trauma (myocardial infarction. Vasomotor dysfunction. Cocaine Botulinum toxin binds presynaptically to cholinergic nerve terminals and after uptake into cytosol C. Symptomatic orthostatic hypotension 1121 Allodynia refers to ? Harrison’s 18th Ed. Occurs between 15 and 50 years of age C. cocaine. All of the above 1114 Which of the following statements about Pure Autonomic Failure Drugs and toxins that can produce autonomic storm include phenylpropanolamine. 3358 B. Focal edema D. 1117 The defective gene in HSAN III is ? 1123 CRPS type II develops after ? Harrison’s 18th Ed. Noradrenergic supersensitivity A. Perception of nonpainful stimulus as painful C. stroke). All of the above B. Brain and spinal cord injury B. Exaggerated pain response to a painful stimulus D. Cells from HSAN I patients affected by mutation Pain is the primary clinical feature of CRPS. 1116 Hereditary sensory and autonomic neuropathy I (HSAN I) is best 1122 Which of the following must be present for the diagnosis of related to which of the following ? complex regional pain syndrome (CRPS) ? Harrison’s 18th Ed. Myocardial infarction C. 3358 C. 3358 C. Low supine plasma NE levels Harrison’s 18th Ed. D3 AChR C. and Harrison’s 18th Ed. Phenylpropanolamine D. Sphingomyelin B. D. 3358 A. tetanus and botulinum. Vasomotor dysfunction B. Spontaneous pain POTS is characterized by symptomatic orthostatic intolerance (not OH) and by either an increase D. Any of the above 1115 Which of the following is false about Postural Orthostatic Neuroleptic malignant syndrome features muscle rigidity. Any of the above SPTLC is an enzyme in the regulation of ceramide. Amphetamine D. Harrison’s 18th Ed. and Allodynia refers to the perception of a nonpainful stimulus as painful while hyperpathia refers to most develop the syndrome between the ages of 15 and 50. Injury to a specific peripheral nerve B. 3359 A. Causes are brain & spinal cord injury. None of the above B. Harrison’s 18th Ed. autonomic neuropathy. D. (PAF) is false ? amphetamine & tricyclic antidepressants. Pheochromocytoma Autoantibodies against ganglionic ACh receptor (A3 AChR) are present in the serum of patients of D. 3358 Harrison’s 18th Ed. Paroxysmal & lancinating D. Glossopharyngeal neuralgia B. PH-tic syndrome Trigeminal neuralgia is characterized by excruciating paroxysms of pain in lips. 3360 1133 Which of the following drugs are used in cases of trigeminal A. in the distribution of the ophthalmic division of the fifth nerve. Cluster-tic D. Superficial stabbing quality C. Maxillary division A. Pterygoid B. 3361 B. or chin and. Inferior cerebellar artery B. and bouts tend to persist for weeks or months before remitting spontaneously. Superior cerebellar artery A. 3361 Table 376-1 A. Onset before age 50 years C. Twenty four to forty eight hours C. Atypical features such as onset before age Remissions may be long-lasting. When cluster headache is associated with trigeminal neuralgia. Ophthalmic division treating trigeminal neuralgia ? Harrison’s 17th Ed. syndrome is known as cluster-tic. Posterior auricular A. Tongue C. Lips B. objective signs of sensory loss are found and baclofen. but in most patients the disorder ultimately recurs. very rarely. 2583 B. Objective sensory loss In trigeminal neuralgia. Most cases of trigeminal neuralgia are not MS-related. Mandibular division C. Posterior cerebral artery Compression of trigeminal nerve root by superior cerebellar artery or a tortuous vein causes 1124 Motor part of trigeminal (fifth cranial) nerve innervates which of trigeminal neuralgia. Occurs both day and night The trigeminal (fifth cranial) nerve supplies sensation to the skin of face and anterior half of the head. or nonparoxysmal pain should raise concerns that MS could be responsible. All of the above 1125 Trigeminal neuralgia is rare in which distribution of fifth nerve ? 1131 When cluster headache is associated with trigeminal neuralgia. Orbicularis oculi neuralgia ? Harrison’s 18th Ed. objective sensory loss. D. None of the above D. 3360 the syndrome is known as ? Harrison’s 18th Ed. AJAY MATHUR Neurology C. Microvascular decompression An essential feature of trigeminal neuralgia is that objective signs of sensory loss cannot be demonstrated on examination. D. cheek. 3361 A. Baclofen Characteristic feature of Trigeminal neuralgia is the presence of trigger zones. Phenytoin D. pain lasts for about ? 1132 Which of the following point towards trigeminal neuralgia being Harrison’s 18th Ed. Mandibular division B. Onset is D. Bilateral symptoms D. 3360 1134 Which of the following is the most widely used procedure in A. gums. 50 years. in ? Harrison’s 18th Ed. Gamma knife radiosurgery 1129 Which of the following artery causes compression of the trigeminal 1135 Gasserian ganglion lesions causing trigeminal nerve disorders nerve root to cause trigeminal neuralgia ? include all except ? Harrison’s 18th Ed. Harrison’s 18th Ed. 1127 In trigeminal neuralgia. Face neuralgia ? Harrison’s 18th Ed. Forehead A. Half an hour to six hours B. Trigeminal neuroma B. Ophthalmic division A. the following muscle ? Harrison’s 18th Ed.736 MCQ’s FOR MEDICAL PROFESSIONALS BY PROF. Injection of glycerol in Meckel’s cave D. Five to ten minutes A. 3360 Harrison’s 18th Ed. 3360 related to multiple sclerosis ? A. D. phenytoin 1128 In trigeminal neuralgia. 1126 In trigeminal neuralgia. All of the above typically sudden. Herpes zoster .Cranial Nerve Disorders D. 3360 B. 3361 B. Carbamazepine C. lips. Sternocleidomastoid C. Few seconds to a minute or two Harrison’s 18th Ed. Maxillary division B. or tongue. pain seldom lasts more than a few seconds or a minute or two. trigger zones are least frequent in ? Harrison’s 18th Ed. Radiofrequency thermal rhizotomy C. Tic Douloureux C. typically on the face. oxcarbazepine. 3360 1130 Which of the following is characteristic of pain of trigeminal A. Its motor part innervates the masseter & pterygoid masticatory muscles. bilateral symptoms. None of the above C. lamotrigine. Anterior choroidal artery 376 . All of the above Drug therapy for trigeminal neuralgia include carbamazepine. 351:1323-31 A.351:1323-31 B. Eclampsia 1140 Which of the following may be caused by multiple sclerosis or D. Ramsay Hunt syndrome A. All of the above 1145 Causes of acquired peripheral facial weakness include all except ? N Engl J Med 2004. maximal weakness being attained by ? A. 2 days A.351:1323-31 1136 Peripheral trigeminal nerve lesion can be due to ? A. Older age A. Guillain Barré syndrome Facial myokymia may be caused by MS or follow GBS. Recipients of inactivated intranasal influenza vaccine C. Lyme disease Harrison’s 17th Ed. D. Collagen-vascular diseases D. Complete facial weakness . Sarcoidosis 1143 For how many days. rather than primary infection A. Guillain-Barré syndrome C. Hemifacial spasm B. All of the above 1142 Poor prognostic factors in a case of Bell’s palsy are all except ? 1150 Which of the following statements about central weakness of the N Engl J Med 2004. Lyme disease D. 24 hours D. 6 hours C. HIV infection C.351:1323-31 C. Disease probably reflects virus reactivation from latency in geniculate ganglion. 2586 1148 Recurrent or bilateral facial palsy is found in all except ? A. 2584 B. Sarcoidosis C. Tetanus D. Rheumatoid arthritis C. 3 days B.351:1323-31 1137 Which of the following can cause trismus ? A. Ramsay Hunt syndrome in immunocompetent people 1141 Poor prognostic factors in a case of Bell’s palsy are all except ? 1149 Recurrent or bilateral facial palsy is found in ? N Engl J Med 2004. Diabetes mellitus C. 4 days C. Sjögren’s syndrome D.351:1323-31 1138 In Bell’s palsy. Facial hemiatrophy C. A. Hypertension A. Blepharospasm N Engl J Med 2004. Hypertension B. Anemia ipsilateral cerebral hemisphere D. Lesion above level of contralateral facial nucleus B. AJAY MATHUR Cardiology 737 C. electrical studies reveal no changes in D. 3361 Table 376-1 B. Parotid-nerve tumors is false ? C. Infection involved facial muscles in Bell’s palsy ? N Engl J Med 2004. 12 hours D. Pain other than in the ear B. HSV-1 is the cause of most cases of Bell’s palsy B. Sarcoidosis B. Lymphoma B. 48 hours 1146 Causes of acquired peripheral facial weakness include all except ? N Engl J Med 2004. 737 Neurology MCQ’s FORMCQ’s FOR MEDICAL MEDICAL PROFESSIONALS PROFESSIONALS BY PROF. Dental trauma A. All of the above follow Guillain-Barré syndrome ? Harrison’s 17th Ed. Hypertension B. Sjögren ’s syndrome 1139 Which of the following statements about causation of Bell’s palsy B. All of the above 1144 Causes of acquired peripheral facial weakness include all except ? N Engl J Med 2004. Phenothiazine D. HSV-1 DNA appear to be specific to Bell’s palsy 1147 Causes of acquired peripheral facial weakness include ? N Engl J Med 2004. None of the above B. Facial myokymia A. Facial nucleus innervating lower face receives fibers from C. Impairment of taste D.351:1323-31 D. Eclampsia N Engl J Med 2004.351:1323-31 The onset of Bell’s palsy is abrupt and maximal weakness occurs by 48 hours as a general rule. Hypothyroidism A. 1 day Harrison’s 18th Ed. Diabetes mellitus Harrison’s 18th Ed.351:1323-31 N Engl J Med 2004. Lesions at base of brain D. 3361 B. Ludwig’s angina C. Amyloidosis D.351:1323-31 A. Myasthenia gravis C.351:1323-31 unilateral lower facial area is false ? N Engl J Med 2004. 351:1323-31 N Engl J Med 2004. Facial nerve encircles nucleus of sixth cranial nerve C. Lesions proximal to geniculate ganglion have permanent loss B. Facial nerve in stylomastoid foramen lacrimation. Cerebellopontine angle 1162 Decompression surgery should not be performed ? N Engl J Med 2004. Aberrant regeneration of nerve fibres is the cause of D. Exit point of internal auditory canal A. Facial nerve in internal auditory canal proximal to or initiate salivary. Hyperacusis results from paralysis of stapedius muscle A. Cerebellopontine angle both cerebral hemispheres D. subcortical region A. Middle of internal auditory canal N Engl J Med 2004. Pons Harrison’s 18th Ed. contralateral hemiparesis. 1151 Which of the following about facial palsy is false ? nystagmus. and taste intact ? N Engl J Med 2004.351:1323-31 A. Peripheral fibers of nervus intermedius portion of facial nerve B. and taste intact. Unilateral lesion in cortex produces contralateral voluntary central-type facial paralysis and contralateral hemiplegia 1157 Locate the lesion if there is ipsilateral peripheral facial weakness.351:1323-31 A. contralateral hemiparesis and spasticity ? 1160 House Brackmann grading system is used for assessing ? N Engl J Med 2004. Salivary function 1155 Locate the lesion if there is ipsilateral peripheral facial weakness. Facial nerve distal to internal auditory canal and geniculate 1153 Which of the following about facial nerve is false ? ganglion N Engl J Med 2004. N Engl J Med 2004. 7 days after the onset of paralysis 1156 Locate the lesion if there is ipsilateral peripheral facial weakness.351:1323-31 C. nystagmus ? C. Facial nerve distal to internal auditory canal & geniculate ganglion 1154 Locate the lesion if there is contralateral central facial weakness. nystagmus. tinnitus. All of the above D. 21 days after the onset of paralysis N Engl J Med 2004.351:1323-31 D. Cerebellopontine angle C. Facial nerve exits in midpons involving geniculate ganglion D. Pons B.351:1323-31 N Engl J Med 2004.351:1323-31 sensory loss. AJAY MATHUR Neurology C. 1161 Facial nerve is compressed at its narrowest point.351:1323-31 A. subcortical region B. ataxia. salivation. Recurrent facial paralysis . Parotid function C. D. 28 days after the onset of paralysis A. 14 days after the onset of paralysis lacrimation. tinnitus. 1152 Which of the following about facial nerve is false ? nystagmus. ipsilateral abducens palsy. Facial nucleus is located in caudal pons B. Cortex. motor root of 7th nerve. and taste likely to be involved. salivation. None of the above C. Exit point of facial nerve from pons B. facial numbness. Facial nucleus innervating upper face receives fibers from C. ataxia. Pons of taste and are unable to produce tears C. Cortex. Pons B. Facial nerve in stylomastoid foramen C. 3363 A. lacrimation.738 MCQ’s FOR MEDICAL PROFESSIONALS BY PROF. and mucous secretion involving geniculate ganglion D. salivation. subcortical region 1163 The Melkersson-Rosenthal syndrome consists of ? B. Cerebellopontine angle C. and taste intact. B. Entrance to meatal foramen ophthalmoparesis ? B. All of the above A. salivation. enter internal auditory meatus lacrimation.351:1323-31 A. All of the above D. Facial nerve is predominantly motor A. which is ? lacrimation. Pons C. Facial nerve in internal auditory canal proximal to or D. lacrimation intact but salivation and taste impaired. hearing loss ? N Engl J Med 2004. A. subcortical region D.351:1323-31 B. lacrimal. and nervus intermedius 1159 Locate the lesion if there is ipsilateral peripheral facial weakness. Cortex. Cortex. salivation. tinnitus. None of the above 1158 Locate the lesion if there is ipsilateral peripheral facial weakness.351:1323-31 D. 8th nerve. and taste are intact. Facial nerve in internal auditory canal proximal to or syndrome of crocodile tears involving geniculate ganglion D. Hearing function D. hearing loss ? N Engl J Med 2004. Sensory cells located in geniculate ganglion continue distally as chorda tympani nerve A. Facial muscle function B. Cerebellopontine angle C.351:1323-31 N Engl J Med 2004. Carotid-cavernous fistula Harrison’s 18th Ed. Cones function under daylight conditions pontocerebellar angle syndrome ? C. Rods specialized for color perception Harrison’s 18th Ed. 224 C. 6 1167 Multiple cranial neuropathy can be caused by ? Harrison’s 18th Ed. Rods operate in dim illumination 1165 Which of the following cranial nerves is not involved in B. Pain is intense & paroxysmal. 5 C. AJAY MATHUR Cardiology 739 B. 1 million B. Ipsilateral ptosis C. Wegener’s granulomatosis 1175 Which of the following feature of Horner’s syndrome is inconstant ? B. All of the above A. Tonic pupil D. 5 million D. plication of tongue of unknown cause. 100 million 1164 Which of the following is false about Glossopharyngeal Neuralgia ? 1171 About how many cones are present in human retina ? Harrison’s 18th Ed. Benign disorder in healthy young women 1169 Which of the following is true for Tolosa-Hunt syndrome ? Harrison’s 18th Ed. 75 million D. originates on one side 1172 Which of the following statements about vision is false ? of throat (tonsillar fossa) and may radiate to the ear (tympanic branch of IX). 3364 Harrison’s 18th Ed. Meningitis Harrison’s 18th Ed. 2 million 1166 Which of the following cranial nerves is not involved in jugular C. 10 million D. C. Miosis D. 50 million Melkersson-Rosenthal syndrome consists of recurrent facial paralysis. Amyloidosis Tolosa-Hunt syndrome is an idiopathic granulomatous disorder that responds to glucocorticoids. Diabetes mellitus 1176 Which of the following is false regarding Adie’s syndrome ? B. 3364 Table 376-2 D. 3 million foramen syndrome ? Harrison’s 18th Ed. XII B. XII B. Anhidrosis 1168 Multiple cranial neuropathy can be caused by ? Harrison’s 18th Ed. All of the above B. 3365 D. All of the above . All of the above B. 224 A. Enlarging saccular aneurysms A. VII 1173 How many fibers are present in each optic nerve ? Harrison’s 18th Ed. VIII A. 225 C. 224 C. 3364 Table 376-2 D. 3365 D. 224 C. Meningioma B. Idiopathic granulomatous disorder A. 1 million D. recurrent and eventually permanent facial (labial) edema and less constantly. Behcet’s disease Harrison’s 18th Ed. Weak or absent tendon reflexes in lower extremities C. 3 million C. Fovea is packed exclusively with cones A. X Harrison’s 18th Ed. 739 Neurology MCQ’s FORMCQ’s FOR MEDICAL MEDICAL PROFESSIONALS PROFESSIONALS BY PROF. 225 C. None of the above A. There is no motor or sensory deficit A. Spasms of pain may Harrison’s 18th Ed. 224 be initiated by swallowing or coughing. IX 1174 Each eye is moved by how many extraocular muscles ? B. 3365 D. 4 million A. 10 million Glossopharyngeal neuralgia though uncommon resembles trigeminal neuralgia & involves IX & sometimes portions of vagus cranial nerves. V B. None of the above D. Plication of tongue A. 225 C. Pain may radiate from throat to ear B. All of the above B. Involves IX & portions of X cranial nerves A. 8 A. Spasms of pain may be initiated by coughing C. Shy-Drager syndrome D. Sarcoidosis A. D. XI A. Aneurysm of carotid artery 1177 Tonic pupils is associated with ? B. Facial (labial) edema 1170 About how many rods are present in human retina ? Harrison’s 18th Ed. Diabetes mellitus C. 4 D. Inferior oblique B. 226 Harrison’s 18th Ed. None of the above D. Herpes zoster infection C. Bilateral optic disc swelling B. None of the above Harrison’s 18th Ed. 239 B. Ishihara color plates is used to detect red-green color D. Aneurysm D. Medial rectus 1190 Which of the following can produce ophthalmoplegia ? Harrison’s 18th Ed. Lateral gaze palsy A. Tumour blindness 1187 Foville’s syndrome includes all except ? 1180 Marcus Gunn pupil indicates ? Harrison’s 18th Ed. 239 B. Benedikt’s syndrome is ipsilateral III nerve palsy and A. Stroke 1184 Which of the following is false about III nerve palsy ? D. direct pupillary response will be weaker than consensual pupillary response evoked by shining a light into other eye. If the retina or optic nerve Harrison’s 18th Ed. Due to ventral pontine injury swinging flashlight test. Mastoiditis D. Palsy results in hypertropia B. Supranuclear vertical gaze disorder C. Also known as dorsal midbrain syndrome contralateral tremor. 238 A. Lambert-Eaton myasthenic syndrome C. Weber’s syndrome is ipsilateral III nerve palsy with C. Nothnagel’s syndrome is ipsilateral III palsy and contralateral 1192 Which of the following is false for Parinaud’s Syndrome ? cerebellar ataxia Harrison’s 18th Ed. Green B. Miller Fisher syndrome 1183 Total palsy of oculomotor nerve leaves the eye ? D. chorea. AJAY MATHUR Neurology 1178 In retina. Red & green cone pigments are encoded on X chromosome A. Infarction B. Mutations of blue cone pigment are common C. Down and in 1191 Most common cause of internuclear ophthalmoplegia is ? Harrison’s 18th Ed. Up and in B. Down and out A. Lateral gaze palsy 1181 Which of the following is false regarding ‘Papilledema’ ? C. Hydrocephalus from aqueductal stenosis contralateral hemiparesis D. Intracranial tumor D. Concussion trauma Harrison’s 18th Ed. 239 C. Deafness 1182 Third cranial nerve innervates all of the following extraocular C. 238 A. 240 B. Diabetes mellitus D. Contralateral hemiparesis A. 232 D. Yellow C. Superior oblique C. 238 A. Palsy results in excyclotorsion C. 239 is only partially injured. It is an extremely useful sign in retrobulbar optic neuritis & optic nerve diseases. Retrobulbar optic neuritis B. Blue D. Up and out C. B. Blue cone pigment is encoded on chromosome 7 B. 225 A. All of the above 1188 Which of the following is false about Millard-Gubler syndrome ? An eye with no light perception has no pupillary response to direct light stimulation. This relative afferent pupillary defect (Marcus Gunn pupil) can be elicited with the A. Head tilt test is diagnostic D. None of the above . Ipsilateral abducens palsy muscles except ? D. 238 A. Multiple sclerosis C. Red A. Ipsilateral facial palsy B. Due to raised intracranial pressure 1189 Which of the following is false about Gradenigo’s syndrome ? Harrison’s 18th Ed. Contralateral hemiparesis C. None of the above 1179 Which of the following is false about colour vision ? 1186 Isolated trochlear nerve palsy occurs due to all except ? Harrison’s 18th Ed. Ipsilateral facial palsy Harrison’s 18th Ed. 226 Harrison’s 18th Ed. Herniation C. Visual acuity is generally affected by papilledema B. Superior rectus A. which of the following is not a class of cones ? 1185 Which of the following is false about Trochlear Nerve ? Harrison’s 18th Ed. 238 A. Giant cell (temporal) arteritis D. 239 Harrison’s 18th Ed. and athetosis B. All of the above Harrison’s 18th Ed. Contralateral hemianesthesia D.740 MCQ’s FOR MEDICAL PROFESSIONALS BY PROF. Transient visual obscuration A. 240 C. paired laminae. T12 . L1 Upper cervical . 3367 B.Diseases of the Spinal Cord A. Knee disease B. Periosteum of vertebrae 1197 Sacral spinal cord segments correspond to which vertebral body ? B.T12 1203 Pain-sensitive structures in spine include all except ? C. T11 D. T12 . 32 of ? D.2 levels higher. Beevor’s sign Intervertebral disks are composed of a central gelatinous nucleus pulposus surrounded by a tough C. Sacral region Harrison’s 18th Ed. Cervical region B. D. 741 Neurology MCQ’s FORMCQ’s FOR MEDICAL MEDICAL PROFESSIONALS PROFESSIONALS BY PROF. S4 nerve root A. 108 Harrison’s 17th Ed.L1 annulus fibrosus of the intervertebral disk. S1 nerve root . S3 nerve root Harrison’s 18th Ed. A. 3366 Table 377-2 C.T12 to L1. Setting sun sign cartilaginous ring. 3 Adult spinal cord is ~46 cm (18 inches) long. are located. 7 1195 The spinal cord has how many segments ? D. 107 D. The nucleus pulposus of the intervertebral disk is not pain-sensitive. facet joints. Annulus fibrosus of intervertebral disk A. B. the annulus fibrosis. S1 nerve root B. B. Thoracic region 1194 What is the length of adult spinal cord ? C. 31 1202 Disk herniation at L4-L5 level commonly produces compression C.2 to 3 levels higher. 10 % B. epidural veins. ~ 52 cm A. 108 A. Harrison’s 17th Ed. 107 D. 1204 Patrick sign is used to demonstrate pain due to ? lower thoracic . D. None of the above A. C. where neurons that innervate upper & lower extremities. 20 % A. B. ~ 46 cm Harrison’s 17th Ed. Collier’s sign 1200 Intervertebral disks are largest in ? Harrison’s 17th Ed. 15 % Harrison’s 18th Ed. ~ 40 cm 1201 Each vertebral arch gives rise to how many processes ? C. T8-T10 Disk herniation at L4-L5 produces compression of S1 nerve root. each defined by exiting ventral motor root & entering dorsal sensory root. Segmental signs D. respectively. Band of altered sensation Patrick sign refers to elicitation of hip pain by internal and external rotation at the hip with the knee and hip in flexion. 3366 D. T12 Pain-sensitive structures in the spine include the periosteum of the vertebrae. C. Nucleus pulposus of intervertebral disk Harrison’s 18th Ed. Ankle disease C. 33 Harrison’s 17th Ed. Hip disease A. L5 nerve root A. dura.T10 to T12 and sacral . All of the above 1205 In supine position. T10 . Lumbar spine disease 1198 Uppermost level of a spinal cord lesion can be localized by ? Harrison’s 18th Ed. 3366 Seven processes arise from each vertebral arch . L1 A. 3366 Table 377-2 D.paired pedicles. lower cervical . and the posterior longitudinal ligament. S2 nerve root 1196 Lumbar spinal cord segments correspond to which vertebral body ? C. paired transverse A. Pineal region tumors D. 107 377 .L1 Harrison’s 17th Ed. AJAY MATHUR Cardiology 741 1193 Which of the following has no relation with Parinaud’s Syndrome ? B. passive flexion of extended leg at the hip 1199 Intervertebral disks are responsible for what percentage of spinal stretches which of the following ? column length ? Harrison’s 17th Ed. Sensory level C. Disks are responsible for 25% of spinal column length. lumbar . 25 % B. 5 regions. 30 processes and one spinous process. 9 Harrison’s 18th Ed.1 level higher. Dura B. ~ 34 cm Intervertebral disks are largest in cervical & lumbar regions where movements of spine are greatest. oval in shape and enlarged in cervical & lumbar B. D.same as cord level. 107 Spinal cord has 31 segments. 107 A. upper thoracic . 742 MCQ’s FOR MEDICAL PROFESSIONALS BY PROF. T11 . T8 . osteoporosis. B. Paralyze movements of foot and ankle Cervical angina syndrome refers to pain in neck that is due to coronary artery ischemia. Weakness of flexion of thigh Extension & lateral rotation of neck narrows ipsilateral intervertebral foramen resulting in radicular B.T12 1209 Which of the following spinal root value innervates axilla ? Lesions at T9-T10 paralyze the lower. 3367 D. Weakness of leg extension at knee 1211 Cervical angina syndrome relates best with ? D. Weakness in finger extensors Reverse SLR stretches the L2-L4 nerve roots and the femoral nerve. T2 D. B. passive flexion of the extended leg at the hip stretches the L5 and S1 nerve roots and the sciatic nerve. 3367 C. All of the above A. B. 3367 D. 115 D. Weakness in finger and wrist flexors Tethered cord syndrome presents as a progressive cauda equina disorder or myelopathy in a B. T4 1210 Spurling’s sign relates to ? Harrison’s 17th Ed. All of the above A. 116 Lesions at L2 . Coronary artery ischemia 1218 Lesions at L5-S1 cause all except ? C. Extreme extension of neck Harrison’s 18th Ed. weakness is found only in finger and wrist extensors and triceps. A. AJAY MATHUR Neurology C.T10 osteodystrophy. T1 Harrison’s 18th Ed. All of the above A. 115 Table 16-4 movement of the umbilicus when the abdominal wall contracts (Beevor’s sign). B. Extension & lateral rotation of neck Dermatome at nipples is T4 and at umbilicus is T10. T5 A. Weakness of flexion at knee . ptosis & facial hypohidrosis) may accompany cervical cord lesion at any level. Any cervical level A. Acromegaly 1215 Beevor’s sign is positive in spinal cord lesion at ? B. 110 D. resulting in upward Harrison’s 17th Ed. 3367 D. Absent patellar reflex Harrison’s 17th Ed. B. Extreme flexion of neck A. Presents as progressive cauda equina disorder At C7 lesion.L4 cause all except ? C. T2 A.T8 D. Perineal or perianal pain 1214 Lesions at C8 produces ? C. Sciatic nerve 1212 Horner’s syndrome accompanies cervical cord lesion at ? Harrison’s 18th Ed. 111 At C8. Low-lying conus (below L1-L2) Harrison’s 18th Ed. All of the above B. Weakness in supination 1208 Lumbar spinal stenosis can be caused by ? Harrison’s 17th Ed. C. hypoparathyroidism. 3367 C. L2 nerve root Horner’s syndrome (miosis. L4 nerve root 1213 Lesions at C7 produce all except ? C. Long and thickened filum terminale A. finger and wrist flexion are impaired. C6 C. D. C. Weakness in wrist extensors D. weaken leg extension A. T3 C. A. Weakness in wrist extensors C. Weakness in brachioradialis 1207 Which of the following is false about Tethered cord syndrome ? Harrison’s 17th Ed. Femoral nerve Harrison’s 18th Ed. Weakness in finger extensors young adult who complains of perineal or perianal pain. Neuroimaging studies show a low-lying conus (below L1-L2) and a short & thickened filum terminale. and Paget’s disease. Hypoparathyroidism A. T3 B. B. B. renal C. but not the upper abdominal muscles. Weakness in triceps A. Subclavian steal syndrome Harrison’s 18th Ed. Vertebrobasilar syndrome at knee and abolish the patellar reflex. 109 D.T9 Lumbar spinal stenosis can be caused by epidural lipomatosis. Renal osteodystrophy Harrison’s 18th Ed. C7 1206 Reverse SLR sign stretches which of the following ? Harrison’s 17th Ed.L4 spinal cord levels paralyze flexion & adduction of thigh. 3367 D. T7 . Flexion & lateral rotation of neck 1217 Lesions at L2 . acromegaly. C5 With the patient lying flat. Weakness of abduction of thigh symptoms (Spurling’s sign). C8 1216 Sensory dermatome at the level of nipples is ? B. 3367 D. T9 . Hip flexion is spared D. Contralateral loss pain & temperature Harrison’s 16th Ed. Posterior column D.S5 D. Thoracic B. variable areflexia in the lower extremities & relative sparing of bowel & bladder function. Ipsilateral weakness Harrison’s 18th Ed. Lumbar D. Lateral corticospinal tract C. Impotence 1228 In lateral spinothalamic tracts. Sacral D. Cervical A. Asymmetric leg weakness D. Bilateral saddle anesthesia 1229 In lateral corticospinal tracts. 1226 In spinal cord. S4 . Bilateral segmental signs C. 3368 Figure 377-1 A. are nearest midline ? Harrison’s 18th Ed. Bladder and bowel dysfunction 1232 Brown-Sequard syndrome is characterized by ? Harrison’s 18th Ed. 3368 Figure 377-1 1220 The conus syndrome is characterized by all except ? A. Sacral + coccygeal segments C. S4 . Contralateral loss of pain & temperature sensation B. Extensors Conus syndrome consists of bilateral saddle anesthesia (S3-S5). 1231 Brown-Sequard hemicord syndrome features all except ? Harrison’s 18th Ed. Bladder and bowel dysfunction D. S1 . A. AJAY MATHUR Cardiology 743 C. Asymmetric leg weakness D. S1 . and extension of thigh D. S2 . Lateral spinothalamic tract B. comprising of the lower sacral and single coccygeal segments. Absent anal reflex C.S2 A.S4 B. 3367 cord are placed anteriorly ? Harrison’s 18th Ed. sacral fibres are placed laterally in all except ? 1219 Conus medullaris comprises of ? Harrison’s 18th Ed. Thoracic A. Bulbocavernosus (S2-S4) & anal (S4-S5) reflexes are absent. 3368 Figure 377-1 Harrison’s 18th Ed. 3367 B. Lower sacral + coccygeal segments D. Sensation over anterolateral thighs is spared Lesions at L5 . Weakness of flexion of thigh B. Variable areflexia in lower extremities D. asymmetric leg weakness & sensory loss. Thoracic C. fibres from which part of the body conus medullaris is the tapered caudal termination of the spinal cord. Flexors D. None of the above and abolish the ankle jerks (S1). S3 .S5 C. 3367 1224 Cauda equina lesion is characterized by all except ? A. 3367 A. 3367 B. Absence of cranial nerve signs B. fibres from which part of the body are nearest midline ? 1221 The conus syndrome is characterized by all except ? Harrison’s 18th Ed. Proximal B.S5 C. Ipsilateral loss of propioception 1225 Which of the following is false about cauda equina syndrome ? C. Asymmetric sensory loss in leg D. 3368 Figure 377-1 Harrison’s 18th Ed. 743 Neurology MCQ’s FORMCQ’s FOR MEDICAL MEDICAL PROFESSIONALS PROFESSIONALS BY PROF. Ventral spinothalamic tract A. fibres from which part of the body 1222 The root value of bulbocavernosus reflex is ? are nearest midline ? Harrison’s 18th Ed.S4 B. 3367 Lesions of cauda equina are characterized by low back & radicular pain. Bilateral saddle anesthesia C. prominent bladder & bowel dysfunction (urinary retention & incontinence with lax anal tone). flexion at knee. Cervical B. Ipsilateral loss of joint position & vibratory sensation A. 137 D. Distal C. Lower lumbar + sacral + coccygeal segments B. and impotence. 3367 A. Absent bulbocavernosus reflex B. All of the above A. S3 .S2 A. Sphincters are affected .S1 paralyse movements of the foot & ankle. Radicular pain C. Absent ankle jerk C. Sacral 1223 The root value of anal reflex is ? 1230 Which of the following motor neurons in anterior horn of spinal Harrison’s 18th Ed.S5 D. 3368 Figure 377-1 A. Sacral C. S2 . None of the above 1227 In spinal posterior columns. Cervical Harrison’s 18th Ed. Lumbar C. 3367 Harrison’s 18th Ed. Lumbar B. Muscle strength is largely preserved. Myokymia Harrison’s 16th Ed. Infection 1234 Diseases of the cerebral hemispheres that produce acute C. Distal limb muscles most severely affected 1233 Hemiparesis due to a cortical lesion is suggested by all except ? C. Loss of pain & temperature sense with preservation of JPS & A. Sparing of sensation in perineal & sacral areas . Weakness affects nonantigravity muscles C. 3367 C. Destruction spares posterior columns. Anterior spinal artery syndrome B. Seizures A. Myasthenia gravis Harrison’s 18th Ed. Muscle stiffness at rest & during sleep B. Loss of temperature sense with preservation of pain sense D. Retained vibration & position sensation A. 3367 D. Manifestation of hypocalcemia extramedullary lesions ? C. 138 A. Loss of JPS & vibration sense with preservation of pain & by ? temperature sense Harrison’s 16th Ed. C. Typical suboccipital pain spreading Harrison’s 16th Ed. Disorders of visual-spatial integration 1241 Central cord syndrome results from all except ? C. Loss of pain sense with preservation of temperature sense C. 137 B. & finally contralateral arm. then contralateral leg. lower neck & upper trunk. syringomyelia.744 MCQ’s FOR MEDICAL PROFESSIONALS BY PROF. central cord syndrome produces arm weakness out of proportion to leg weakness and “dissociated” sensory loss (loss of pain & temperature sense with preservation of light touch. All of the above In cervical cord. Segmental signs. and vibration sense over shoulders. Foramen magnum syndrome C. Trauma. Acute hydrocephalus 1242 Dissociated sensory loss in central cord syndrome means ? D. 138 A. Serum antibody against glutamic acid decarboxylase present Compressive lesions near foramen magnum produce weakness of ipsilateral shoulder & arm followed by weakness of ipsilateral leg. 138 to neck & shoulders is present. Early sacral sensory loss C. Myasthenia gravis D. with contralateral loss of pain and Harrison’s 16th Ed. Associated with cancers D. sensory & autonomic functions are lost below the level of lesion 1237 Which of the following is a cause of episodic generalized weakness ? Harrison’s 16th Ed. 137 D. Central cord syndrome A. 3367 D. Harrison’s 16th Ed. such as radicular pain. 3367 Harrison’s 16th Ed. Trauma B. Motor neuron disease 1243 Which of the following is false about anterior spinal artery syndrome ? B. Lambert-Eaton myasthenic syndrome Anterior spinal artery syndrome is due to infarction of spinal cord due to its occlusion or diminished flow.. Motor. an 1239 Which of the following is false about ‘Tetany’ ? “around the clock” pattern that may begin in any of 4 limbs. Motor. AJAY MATHUR Neurology Brown-Sequard hemicord syndrome consists of ipsilateral weakness (corticospinal tract) and loss 1240 Which of the following is false about ‘Neuromyotonia’ ? of joint position and vibratory sense (posterior column). 3367 1235 Acute monoparesis due to focal cortical ischemia is characterized A. 136 D. Not associated with sensory impairment or pain D. Polymyositis/dermatomyositis A. 138 temperature sense (spinothalamic tract) one or two levels below the lesion. Vibration and position sensation are retained. All of the above Harrison’s 18th Ed. Apraxia Harrison’s 18th Ed. Manifestation of respiratory alkalosis Harrison’s 18th Ed. Radicular pain B. Due to occlusion of anterior spinal artery D. 1236 Weakness limited to respiratory muscles is due to ? joint position. None of the above A. 137 C. Multiple sclerosis the level of lesion. All of the above D. “Pure motor” hemiparesis B. or loss of a deep tendon reflex. Involves jaw C. tumors & anterior spinal artery ischemia B. None of the above B. A. None of the above A. Tingling around mouth 1245 Which of the following features does not favour the diagnosis of B. D. are unilateral. All of the above 1244 “Around the clock” pattern of weakness is seen in ? 1238 Which of the following is false about Stiff-Person Syndrome ? Harrison’s 18th Ed. sensory and autonomic functions are lost below C. Muscle stiffness & superimposed spasms B. Superior sagittal sinus or cortical venous thrombosis cause central cord syndrome. Unpaired anterior cerebral artery ischemia Central cord syndrome is due to damage to gray matter nerve cells & crossing spinothalamic tracts near the central spinal canal. A. Anterior spinal artery ischemia A. Weakness is predominantly distal vibration sense B. All of the above B. Syringomyelia paraparesis include ? Harrison’s 16th Ed. muscle atrophy. 3370 1248 Which of the following malignant tumors usually do not metastasize to “thoracic” spinal cord ? A. Harrison’s 18th Ed. Early corticospinal tract signs A. progressive spinal cord syndrome weakness in legs (corticospinal tract) indicate extramedullary lesions. Movement D. or low-grade Epidural compression causes warning signs of neck or back pain. prostate. Spastic weakness in legs B. Arteries Most cases of spinal epidural abscess are due to Staphylococcus aureus.single anterior spinal artery & paired posterior spinal arteries. Progressive limb weakness B. Ependymomas A. Sparing of sensation in perineal & sacral areas B. fungi and tuberculosis are other important causes. AJAY MATHUR Cardiology 745 D. Coughing Spinal cord is supplied by 3 arteries . Streptococcus most common bacteria A.Artery of Adamkiewicz arises at ? Harrison’s 18th Ed. Midline dorsal pain Harrison’s 18th Ed. Myelopathy without antecedent symptoms Primary intramedullary tumors of spinal cord include ependymomas. Breast C. Initial symptom is radicular pain Prominent radicular pain with early sacral sensory loss (lateral spinothalamic tract) & spastic C. kidney. Low-grade astrocytoma C. due to laminated configuration of spinothalamic tract with sacral fibers outermost. A. Gram-negative bacilli. Slow-growing & benign D.L5 . C6 B. A. Sensory loss C. bladder disturbances and astrocytomas in adults. coughing or sneezing & awakens patients at night. Spastic weakness in legs C. 3368 C. Harrison’s 18th Ed. Neck or back pain B. C. 3370 Metastases from prostate & ovarian cancer occur disproportionately in sacral & lumbar vertebrae. Fever A. 3371 1250 Pain in neoplastic epidural compressive myelopathy worsens A. 1247 Which of the following features favours noncompressive 1253 Which of the following is a primary intramedullary tumor ? myelopathy ? Harrison’s 18th Ed. T4 . Posterior root D. Asymmetric. T11 . Radicle Intramedullary lesions produce poorly localized burning pain & spare sensation in perineal and D. often in cervical region. 2 Harrison’s 18th Ed. corticospinal tract signs appear later. L4 . 3 A. Radicular pain Harrison’s 18th Ed. D. Nerves D. 1 with ? B. All of the above D. Any of the above sacral areas (“sacral sparing”). Ovary Clinical triad of spinal epidural abscess consists of midline dorsal pain. Symptoms usually begin with radicular intramedullary lesions ? sensory symptoms followed by an asymmetric.T5 1251 Which of the following is false about intradural mass lesions ? Harrison’s 18th Ed. Capillaries 1256 The spinal cord is supplied by how many arteries ? Batson’s plexus is a network of veins along the anterior epidural space. All of the above 1257 Anterior spinal artery feeder . Lymphoma Almost any malignant tumor can metastasize to spinal column with breast. fever & progressive limb weakness. D. sensory symptoms that precede development of paralysis. 3368 A. Kidney D. 1254 Clinical triad of spinal epidural abscess consists of all except ? Harrison’s 18th Ed. progressive spinal cord syndrome. Infections of skin is a risk factor 1249 Batson’s plexus is composed of ? B. C. Bladder disturbances C. 1255 Which of the following about spinal epidural abscess is false ? lymphoma & plasma cell dyscrasia being frequent. Harrison’s 18th Ed. Anterior root C. 745 Neurology MCQ’s FORMCQ’s FOR MEDICAL MEDICAL PROFESSIONALS PROFESSIONALS BY PROF. Noncompressive etiologies produce myelopathy without antecedent symptoms. Sensory symptoms preceding paralysis D. lung. 3370 Harrison’s 18th Ed. They present as central cord or hemicord syndromes. Hemangioblastomas B. D.L2 A. 3369 B. hemangioblastomas. 4 B. 3367 1252 Intradural neurofibromas typically arise near ? A. Commonly due to hematogenous spread Harrison’s 18th Ed. 3371 Pain in neoplastic epidural compressive myelopathy typically worsens with movement. anaerobes. Sneezing D. Thoracic cord is most commonly involved. Lumbar puncture is not recommended B. Veins streptococcus. 3368 C. None of the above 1246 Which of the following features favours the diagnosis of Most intradural mass lesions are slow-growing & benign. 3368 B. 3369 C. Intradural neurofibromas typically arise near the posterior root. vertebral D. Epstein-Barr virus (EBV) Recurrent episodes of myelitis are due to immune-mediated disease like demyelinating disease. 3371 In sarcoid myelopathy. All of the above C. HSV . Preservation of sphincter control B. Sharp midline or D. D. greatest ischemic risk of cord infarction D. Sjogren’s syndrome A. Cytomegalovirus (CMV) 1263 Immune-mediated disorders causing acute transverse HSV-2 causes recurrent sacral myelitis along with outbreaks of genital herpes mimicking MS. Paraplegia or quadriplegia 1266 Typical neurologic manifestation of sarcoidosis is ? B. Hypotension from any cause B. Sparing of vibration & position sense A. D. Antiphospholipid antibodies C. Sjogren’s syndrome D.L2 D. C. Dissecting aortic aneurysm A. SLE D. and C. Typical neurologic manifestation of sarcoidos are cranial neuropathy.T4 of spinal cord mimicing tumor is observed ? Harrison’s 18th Ed. All of the above 1258 Posterior spinal arteries become less distinct below ? Immune-mediated myelitides include Sjögren’s syndrome. AJAY MATHUR Neurology Anterior spinal artery is fed by radicular vessels that arise at C6. greatest ischemic risk is at T3 . profound hypotension from any cause. Sarcoidosis With systemic hypotension. 3371 1268 Recurrent sacral myelitis which mimic multiple sclerosis is due A. SLE B. NMO is a demyelinating syndrome of severe myelopathy with optic neuritis which is bilateral & may precede or follow myelitis by weeks or months. T3 . Midlumbar level Harrison’s 18th Ed. Herpes zoster . A. Midthoracic level 1264 Antiphospholipid antibodies are found in which of the following immune-mediated disorders ? C. Herpes zoster C. dissociated sensory loss affecting pain & temperature sense but sparing vibration & position sense. and loss of sphincter control. SLE Posterior spinal arteries supply posterior columns & become less distinct below midthoracic B. Behcet’s syndrome 1259 With systemic hypotension. mixed connective tissue disease. SLE. SLE to ? B. at an upper thoracic level. All of the above B. Behcet’s syndrome 1260 Which of the following is not a feature of anterior cord syndrome ? Harrison’s 18th Ed. myelopathies (ATM) include ? Harrison’s 18th Ed. a large edematous swelling B. hypothalamic involvement or meningeal enhancement visualized by MRI. 3372 B. A. NMO is also associated with SLE. L5 . and vasculitis with perinuclear antineutrophilic cytoplasmic (p-ANCA) antibodies. at T11 . RA is at the level of ? Harrison’s 18th Ed. Sjogren’s syndrome level. or sarcoid or to infection with HSV type 2. T11 . antiphospholipid antibodies and with other connective tissue diseases. Hypothalamic involvement Anterior cord syndrome is due to acute infarction in the territory of anterior spinal artery that C. C6 1265 In which of the following myelopathies. 1262 Recurrent episodes of myelitis are usually due to ? Harrison’s 18th Ed. 3371 Myelitis occurs in SLE associated with antiphospholipid antibodies. 3372 D.T4 and also at boundary zones between anterior & posterior spinal artery territories. Sarcoid Harrison’s 18th Ed. 3372 C.746 MCQ’s FOR MEDICAL PROFESSIONALS BY PROF. Connective tissue diseases Spinal cord infarction results from aortic atherosclerosis. C. vasculitis related to collagen vascular disease. Aortic atherosclerosis Harrison’s 18th Ed. dissecting aortic aneurysm. All of the above radiating back pain localized to the area of ischemia is frequent. Midcervical level B. Midsacral level A. 3372 1269 Acyclovir is useful in treatment of myelitis caused by which of the following viruses ? A. A. Meningeal enhancement visualized by MRI presents as paraplegia or quadriplegia. Loss of pain & temperature sense Harrison’s 18th Ed. 3371 Behçet’s syndrome. Behcet’s syndrome most consistently. All of the above artery occlusion or dissection in neck. 3372 C. 3371 1267 Neuromyelitis optica (NMO) is associated with ? A.2 D. 1261 Spinal cord infarction is associated with ? Harrison’s 18th Ed.S1 A. SLE Harrison’s 18th Ed. cardiogenic emboli. Harrison’s 18th Ed. an edematous swelling of spinal cord may mimic tumor.2 A. 3372 B. 3372 C. and surgical interruption of aortic aneurysms. Infection with HSV. Cranial neuropathy D.L2 (artery of Adamkiewicz). D. Associated with Chiari type 1 malformations 1279 Which of the following is false about tabes dorsalis ? D.14 days. Dissociated sensory loss in upper limbs C. HSV myelitis C. Tabes dorsalis B. AJAY MATHUR Cardiology 747 B. sensory. Nitrous oxide inhalation B. 747 Neurology MCQ’s FORMCQ’s FOR MEDICAL MEDICAL PROFESSIONALS PROFESSIONALS BY PROF. C. None of the above Harrison’s 18th Ed. 3375 1273 Which of the following is false about syringomyelia ? A. HSV & EBV myelitis are treated with IV acyclovir or oral valacyclovir for 10 . Loss of deep tendon reflexes B. Ataxia A. Areflexic weakness in upper limbs D. 3373 B. Tabes Dorsalis C. B. 3375 C. Flaccid paraplegia B. sensory & bladder/bowel disturbances. Fleeting & repetitive lancinating pains in legs Harrison’s 18th Ed. Romberg’s sign D. Adrenomyeloneuropathy 1272 Which of the following is false about syringomyelia ? B. Paraparesis Harrison’s 18th Ed. 3374 1281 Which of the following polyradiculopathy may simulate tabes A. 3374 A. Non-Hodgkin’s lymphoma D. All of the above A. and bladder/bowel disturbances A. The responsible gene encodes ADLP. Arteriovenous malformations 1278 Which of the following condition producing myelopathy identical B. All of the above C. Elevated serum levels of homocysteine D. D. position sense & vibration appreciation. Tabes Dorsalis Harrison’s 18th Ed. Adrenomyeloneuropathy B. None of the above B. 3374 D. Acute abdominal pain with vomiting B. CMV polyradiculopathy produces motor. None of the above C. low levels of serum copper and low level of serum ceruloplasmin. involvement is typically bilateral but asymmetric & C. EBV polyradiculopathy In chronic progressive myelopathy of MS. Mixed motor & sensory neuropathy A. There occurs impairment of pinprick & temperature appreciation but relative preservation of light touch. Progressive thoracic myelopathy to subacute combined degeneration ? C. degeneration of cord ? Harrison’s 18th Ed. Adrenal insufficiency D. 3375 Harrison’s 18th Ed. Bilateral dorsalis ? B. Loss of deep tenson reflexes in legs spinothalamic tract involvement occurs in syringomyelia). Motor. Elevated levels of very long chain fatty acids C. None of the above Harrison’s 18th Ed. Symptoms begin in adolescence or early adulthood Nitrous oxide inhalation producing a myelopathy identical to subacute combined degeneration. 3375 Dissociated sensory loss can reflect spinothalamic tract involvement in spinal cord (bilateral A. Diabetic polyradiculopathy 1275 Which of the following is false about subacute combined 1282 Which of the following is false about adrenomyeloneuropathy ? degeneration of cord ? Harrison’s 18th Ed. A. None of the above 1270 Which of the following is not a type of chronic myelopathy ? 1277 Which of the following condition is virtually identical to subacute Harrison’s 18th Ed. None of the above Adrenomyeloneuropathy is associated with a mixed motor & sensory neuropathy with spastic 1276 Which of the following is false about subacute combined paraplegia in adults associated with elevated circulating levels of very long chain fatty acids and adrenal insufficiency. Impaired position & vibratory sense C. 3374 A. Romberg’s sign 1274 Which of the following is false about chronic myelopathy of multiple sclerosis ? D. Asymmetric Harrison’s 18th Ed. None of the above Harrison’s 18th Ed. Babinski sign C. 3374 . 3373 serum levels of B12. EBV B. Hypocupric myelopathy 1271 Foix-Alajouanine syndrome best relates with ? Hypocupric myelopathy is virtually identical to subacute combined degeneration but with normal Harrison’s 18th Ed. Familial Spastic Paraplegia A. Familial Spastic Paraplegia D. Subacute combined degeneration of cord D. Syringomyelia A. HSV A. Spasticity and weakness of the legs 1280 Which of the following is false about tabes dorsalis ? D. 3373 C. Acquired or developmental D. Macrocytic red blood cells C. Elevated serum levels of methylmalonic acid Herpes zoster. 3372 combined degeneration ? Harrison’s 18th Ed. ~ 6 months D. C7 D. loss of biceps reflex signifies lesion in the root ? 1287 In cervical radiculopathy.353:392-9 B. C8 C.353:392-9 infraspinatus muscles signify lesion in which root ? A. pain in the region of medial scapular border. ~ 12 months The prospects for recovery from an acute destructive spinal cord lesion fade after ~6 months.748 MCQ’s FOR MEDICAL PROFESSIONALS BY PROF. C7 B. C6 A. C5 A. C5 N Engl J Med 2005. C8 Lathyrism due to ingestion of chick peas containing the excitotoxin -N-oxalylaminoalanine (BOAA). C7 B. C6 D. C5 C. finger N Engl J Med 2005. C6 D. N Engl J Med 2005. C8 C. weakness of biceps. C8 1293 In cervical radiculopathy. C7 D. C7 D. C6 A. C5 C. C8 C. C5 N Engl J Med 2005. C5 C. N Engl J Med 2005. C7 D. loss of supinator reflex signifies lesion in the root ? 1286 In cervical radiculopathy. C7 B. C5 N Engl J Med 2005.353:392-9 Harrison’s 18th Ed. ~ 1 month B.353:392-9 signifies lesion in the root ? A.353:392-9 B. C6 D. C6 A. C6 D. C6 B. C8 . supraspinatus & N Engl J Med 2005. 1292 In cervical radiculopathy.353:392-9 wrist extensor muscles signify lesion in the root ? A. Peas N Engl J Med 2005. brachioradialis. C8 C. C8 C. Barley C. C5 N Engl J Med 2005. C6 D. loss of triceps reflex signifies lesion in the root ? 1288 In cervical radiculopathy. C7 B. weakness of deltoid. lateral upper arm to elbow signifies lesion in the root ? 1289 In cervical radiculopathy. C6 A. C5 C. C8 D. sensory loss over posterior forearm 1284 The prospects for recovery from an acute destructive spinal cord and third finger signifies lesion in the root ? lesion fade after ? N Engl J Med 2005. C7 D. Chick peas A. C8 1295 In cervical radiculopathy.353:392-9 extensor muscles signify lesion in the root ? A.353:392-9 B. C5 C. ~ 3 months C. sensory loss over fifth finger signifies lesion in the root ? 1285 In cervical radiculopathy. C7 B. sensory loss over thumb and index Harrison’s 18th Ed. 1291 In cervical radiculopathy. weakness of thumb flexors. 3375 A. C7 D.353:392-9 B. sensory loss over lateral upper arm N Engl J Med 2005. wrist flexor.353:392-9 B. abductors. Soyabean B. C6 D.353:392-9 B. 3375 finger signifies lesion in the root ? A. C8 1294 In cervical radiculopathy. C8 1296 In cervical radiculopathy. AJAY MATHUR Neurology 1283 Lathyrism is due to ingestion of faulty ? 1290 In cervical radiculopathy. C6 A.353:392-9 intrinsic hand muscles signify lesion in the root ? A. C7 C. weakness of triceps. C5 C. C5 N Engl J Med 2005. 353:392-9 C. Pancoast syndrome D. Thoracic outlet syndrome C. All of the above 1310 In transverse myelitis. C8 B. Used for diagnosis of Thoracic outlet syndrome C. Acute brachial-plexus neuritis thumb and index finger signifies lesion in the root ? C. Referred somatic pain from the neck D. Subacute D. C7 A. Chronic 1302 Differential diagnosis of cervical radiculopathy include ? Transverse myelitis is a heterogeneous group of acute or subacute inflammatory disorders with N Engl J Med 2005.353:392-9 D.353:392-9 D. Intermittent paresthesia. C5 B.353:392-9 C. bowel. Pancoast syndrome A. Carpal tunnel syndrome N Engl J Med 2005. Acute brachial-plexus neuritis A. pain in the region of medial scapula. and sexual) spinal cord dysfunction. Median nerve entrapment 1298 In cervical radiculopathy. First 6 months after the event A. Disorders of rotator cuff & shoulder N Engl J Med 2010. Thoracic outlet syndrome Transverse myelitis syndrome most often occurs as an autoimmune phenomenon after an infection B. pain in the region of lateral forearm.353:392-9 D. B. Inflammatory disorder B.363:564-72 A. an underlying systemic autoimmune disease.353:392-9 D. acquired demyelinating disease (multiple sclerosis) or idiopathic (15 .353:392-9 ? N Engl J Med 2010. Median nerve entrapment D. First 3 months after the event N Engl J Med 2005. AJAY MATHUR Cardiology 749 1297 In cervical radiculopathy. Acquired demyelinating disease A. Ulnar nerve entrapment posterior arm. B. Ulnar nerve entrapment 1299 In cervical radiculopathy.353:392-9 B. Underlying systemic autoimmune disease N Engl J Med 2005. most commonly in C5 region B. Disorders of the rotator cuff and shoulder C.363:564-72 C. pain in the region of shoulder. ulnar C. C6 1305 Hypoesthesia and weakness in medial three digits and opponens C. Pancoast syndrome A. Opposite of Valsalva maneuver C. most of the recovery occurs over ? N Engl J Med 2010. C7 pollicis occurs in ? N Engl J Med 2005. Thoracic outlet syndrome side of forearm. Autoimmune phenomenon after infection or vaccination D. Foraminal compression test in cervical radiculopathy D. Direct infection 1303 Differential diagnosis of cervical radiculopathy include ? C. Acute brachial-plexus neuritis A. All of the above B. Thoracic outlet syndrome in the root ? N Engl J Med 2005. First 12 months after the event . Done by rapid flexion and extension of fingers while the arms 1300 Spurling maneuver is ? are abducted at 90° and externally rotated 90° N Engl J Med 2005. C6 N Engl J Med 2005.363:564-72 1304 Tinel’s sign and Phalen’s maneuver best relate to ? A.353:392-9 A. Peripheral entrapment neuropathies B.353:392-9 D. C5 1306 Hypoesthesia and weakness in the fourth and fifth digits and B. C7 A. C6 thumb adductor occurs in ? N Engl J Med 2005. Peripheral entrapment neuropathies 1309 Transverse myelitis syndrome “most often” occurs due to ? B. fifth finger signifies lesion in the root ? N Engl J Med 2005. Acute C. A. Herpes zoster or vaccination or as a result of a direct infection. sensory and autonomic (bladder. Other name of hyperventilation Transverse Myelitis D. 749 Neurology MCQ’s FORMCQ’s FOR MEDICAL MEDICAL PROFESSIONALS PROFESSIONALS BY PROF. Breath holding test 1301 Parsonage . dorsum of forearm & third finger signifies lesion C.Turner syndrome refers to ? 1308 Which of the following does not relate well with transverse myelitis N Engl J Med 2005. It is a kind of provocation test D. C5 1307 Which of the following statements about Roo’s test is false ? B.353:392-9 motor. C8 B. C8 A. Pancoast syndrome A.30%) C. C. Course of illness nuclear antigen are present in ? A well-defined truncal sensory level.363:564-72 and autonomic spinal cord dysfunction. 4 hours and 21 days after symptom onset D. A. Good 1318 In transverse myelitis. C. No identifiable underlying cause 1311 Pathological hallmark of transverse myelitis within the spinal cord is ? D. Motor spinal cord dysfunction and microglial activation.363:564-72 lesions enhance with intravenous gadolinium administration. with varying degrees of demyelination. Astroglial activation 1321 MRI findings in multiple sclerosis include ? D. Axonal injury patients with multiple sclerosis or neuromyelitis optica. Hypotonia A. In acute phase.363:564-72 D.363:564-72 characteristic of transverse myelitis. clinical deficits progress to nadir between B.363:564-72 D. Sensory spinal cord dysfunction In spinal cord. Lesions enhance with intravenous gadolinium 1314 Which of the following best distinguishes myelopathy from cerebral D. None of the above N Engl J Med 2010. C. Lesions in idiopathic transverse A. Transverse myelitis or neuromyelitis optica associated with other diseases usually have clinically significant residual neurologic deficits. C. Concomitant MRI evidence of demyelinating brain lesions 1316 Which of the following is false about idiopathic transverse myelitis ? D. hypotonia and areflexia. A. Focal collections of eosinophils 1317 Presentation in transverse myelitis include ? N Engl J Med 2010. 4 hours and 14 days after symptom onset recovery. First 18 months after the event N Engl J Med 2010. pathological hallmark of transverse myelitis is the presence of focal collections of lymphocytes and monocytes. systemic lupus erythematosus. Multiple sclerosis 1315 Which of the following is responsible for the Lhermitte’s sign in B. Lesions located in the periphery of the cord C. Normal MRI should prompt a reconsideration of the diagnosis of myelopathy in favor of other disorders of the central or peripheral nervous B. A. Babinski signs 1320 Serum autoantibodies like antinuclear antibody or extractable D. All of the above lesions and peripheral neuropathies ? MRI finding of more than one intrinsic cord lesion is characteristic of myelitis.e. B. All of the above A. Focal collections of neutrophils myelitis but that are not associated with cerebrospinal fluid abnormalities or abnormalities detected on MRI and that have no identifiable underlying cause. Neuromyelitis optica transverse myelitis ? C. AJAY MATHUR Neurology D. Focal collections of basophils A. Microglial activation N Engl J Med 2010. improvement may continue for a year or longer. All of the above is ? Diagnostic criteria for transverse myelitis include bilateral (not necessarily symmetric) sensorimotor N Engl J Med 2010.363:564-72 A. B. Demyelination Serum autoantibodies like antinuclear antibody or extractable nuclear antigen may be present in B.363:564-72 D. distinguishes myelopathy from cerebral lesions & peripheral neuropathies.363:564-72 D.involuntary dystonic contractions of limb or trunk muscles. C. No cerebrospinal fluid abnormalities Most of the recovery in transverse myelitis cases occurs in the first 3 months after the event. N Engl J Med 2010. Systemic lupus erythematosus N Engl J Med 2010.750 MCQ’s FOR MEDICAL PROFESSIONALS BY PROF. 4 hours and 7 days after symptom onset Transverse myelitis associated with multiple sclerosis may have a substantial or even complete B. It is the only B. All of the above . axonal injury and astroglial B. Seizure myelitis usually span at least two vertebral segments. No MRI abnormalities C. Although. Sensory loss 1319 Which of the following is true for MRI findings characteristic of myelitis ? C. Areflexia B. Demyelination is responsible for the paroxysmal tonic spasms i. Focal collections of lymphocytes & monocytes C. Sensory level system. Worst A.363:564-72 or lost. 4 hours and 28 days after symptom onset 1313 “Spinal shock” consists of all except ? Progression to nadir of clinical deficits between 4 hours and 21 days after symptom onset is N Engl J Med 2010.363:564-72 Patients with possible idiopathic transverse myelitis have clinical events consistent with transverse A. below which the sensation of pain & temperature is altered N Engl J Med 2010. Short lesions spanning less than two vertebral segments down the spine or limbs with neck flexion. Intrinsic cord lesions span at least two vertebral segments recognized predictor of a poor outcome in transverse myelitis. Autonomic spinal cord dysfunction 1312 Prognosis in transverse myelitis associated with multiple sclerosis D. Bad ? C. More than one intrinsic cord lesions “Spinal shock” refers to a combination of severe weakness.363:564-72 Demyelination is responsible for the Lhermitte’s sign that refers to the paresthesias that radiate A. Worse N Engl J Med 2010. B. Severe weakness N Engl J Med 2010. < 90 neurologic events. Decreased Harrison’s 17th Ed. Meningiomas C. whereas multiple sclerosis and neuromyelitis optica. Primary CNS lymphoma IgG index is a measure of intrathecal synthesis of immunoglobulin and is calculated by the C. Medulloblastoma C.363:564-72 Headaches due to increased ICP are usually holocephalic. Postvaccination transverse myelitis 1330 What level of ‘Karnofsky performance status’ have a poor prognosis in patients with brain tumor ? C. All of the above CSF contains malignant cells in leptomeningeal metastases. meningiomas for 25%. High-grade glioma defined by a lesion that spans three or more vertebral segments on MRI. Seizure N Engl J Med 2010.60% of primary brain tumors. Tumors of the Nervous System 1332 Which of the following is associated with increased risk for deep vein thrombosis & pulmonary embolism ? Harrison’s 17th Ed. < 70 Postinfectious. Postinfectious transverse myelitis B. D. vomiting. < 100 1325 What happens to the IgG index in transverse myelitis ? Karnofsky performance scale is meant to assess patients with brain tumors. Pituitary adenoma Gliomas & primary CNS lymphomas are associated with increased risk for DVT & PE because these tumors secrete procoagulant factors. 3382 and concomitant MRI evidence of demyelinating brain lesions. or B. dementia. Such lesions tend to be symmetric and situated centrally within the cord (involving both gray and white matter) and may B. Glial tumors B. .spectrum disorders are relapsing diseases that are associated with a high risk of future attacks of transverse myelitis and other C. Medulloblastoma following formula . Develop rapidly. Leptomeningeal metastases D. Lesions situated centrally within the cord 1328 Brain tumor frequently associated with hemorrhage is ? D. Progressive focal neurologic deficits 1322 MRI findings in Neuromyelitis optica include ? B. Metastatic melanoma extend into the brain stem. aCL 1329 Features of headache due to increased ICP include all except ? C. 3382 D. Harrison’s 18th Ed. C. 2602 C. Normal 1331 CSF has higher probability of containing malignant cells in ? B. A score >=70 indicates N Engl J Med 2010. 3382 A. AJAY MATHUR Cardiology 751 Multiple sclerosis is associated with short lesions. aquaporin-4. spanning fewer than two vertebral segments. Symmetric lesions nonfocal neurologic disorder like headache. B. Frontal Neuromyelitis optica is specifically associated with the serum autoantibody marker NMO-IgG. Idiopathic transverse myelitis Harrison’s 17th Ed. 1327 Brain tumors usually present with ? located in the periphery of the cord. seizure. schwannomas for 10%. aquaporin-4 Harrison’s 18th Ed.363:564-72 C. Beta2GPI melanoma and choriocarcinoma. Increased A. Frequency more than once a day 1324 Which of the following is not a monophasic syndrome ? D. Glial tumors account for 50 . Nonfocal neurologic disorder A. Astrocytic water channel. affecting mainly white matter (“partial” transverse myelitis). Primary and Metastatic neuroectodermal tumors (medulloblastoma). and hiccups. All of the above N Engl J Med 2010. 2602 1326 Which is the most common primary CNS tumor ? Harrison’s 18th Ed.(CSF IgG ÷ serum IgG) ÷ (CSF albumin ÷ serum albumin).363:564-72 Brain tumors frequently associated with hemorrhage include high-grade gliomas. primary CNS lymphoma and primitive Chapter 379. 2601 Neuromyelitis optica is strongly associated with longitudinally extensive transverse myelitis. & other CNS tumors for the remainder. A. personality change or gait disorder.363:564-72 that patient is ambulatory & independent in self-care activities. All of the above D. C. A. 751 Neurology MCQ’s FORMCQ’s FOR MEDICAL MEDICAL PROFESSIONALS PROFESSIONALS BY PROF. metastatic A. All of the above Harrison’s 17th Ed. subside quickly N Engl J Med 2010. B. Anti-LKM3 A. 2602 D. A. Choriocarcinoma 1323 Serum autoantibody marker NMO-IgG targets ? D. Leptomeningeal metastases A. Longitudinally extensive lesions spanning three or more D. D. postvaccination and idiopathic forms of transverse myelitis are usually monophasic B. Any of the above vertebral segments Brain tumors may present with subacute progression of a focal neurologic deficit. Primary CNS lymphoma B. Any of the above B. < 80 syndromes. A. causing nausea. Multiple sclerosis A. Episodic which targets the astrocytic water channel. Schwannomas D. C. Werner syndrome Oligodendrogliomas respond well to systemic combination chemotherapy with procarbazine. Turcot syndrome C. from the filum terminale of the spinal cord and appears in the lumbosacral region. Malignant glioma C. 3p D. Pleiomorphic xanthoastrocytoma C. gliomas. Cervical region B. Benzene A. 2602 A. 3387 A. Medulloblastoma D. Endothelial proliferation C. Carmustine (BCNU) B. Hypercellularity A. 3384 tumors includes all except ? A. pancytopenia and lactic acidosis. the only established risk factors are exposure to ionizing Proliferation index (mitotic activity) can be determined with monoclonal antibody Ki-67 which radiation (meningiomas. Hemangioblastoma Subependymal giant cell astrocytoma mostly occurs in tuberous sclerosis. 1341 Which of the following is the preferred drug for high-grade astrocytomas ? 1335 Nervous system neoplasm seen in tuberous sclerosis is ? Harrison’s 18th Ed. Nuclear and cytoplasmic atypia B. Endothelial proliferation & necrosis are strong predictors In adults. Nuclear and cytoplasmic atypia B. which typically arises of aggressive behavior. myxopapillary histologic type is the most frequent ependymoma. Glioblastoma multiforme D. Ionizing radiation C. Turcot syndrome D. Werner syndrome D. 4p Oligodendrogliomas with deletions of chromosome 1p always respond to chemotherapy. AJAY MATHUR Neurology 1333 Well-documented environmental risk factor for the development 1339 The proliferation index of brain tumours can be determined by a of gliomas is ? monoclonal antibody termed ? Harrison’s 18th Ed. Li-Fraumeni syndrome C. Pearson syndrome Pilocytic astrocytoma (spindle-shaped cells) is the most common childhood brain tumor and is typically benign. 2602 Harrison’s 18th Ed. most common childhood brain tumor is ? 1334 Hereditary syndromes associated with an increased risk of brain Harrison’s 18th Ed. Lomustine A. 2602 Harrison’s 18th Ed. 1p B. ependymomas of spinal canal are mostly located in ? Harrison’s 17th Ed. von Hippel-Lindau syndrome lomustine. nuclear & cytoplasmic atypia. Ki-66 B. Vincristine C. or to temozolomide. 3386 1336 Pheochromocytoma is associated with which of the following hereditary syndrome ? A. Ki-67 C. 3383 Harrison’s 17th Ed. All of the above Histologic features of higher grade are hypercellularity. Temozolomide A. Astrocytoma B. endothelial proliferation. Vincristine D. Subependymal giant cell astrocytoma B. and vincristine (PCV). 3384 Table 379-2 A. 3384 Table 379-2 B. Lumbosacral region D. . lymphoma). and schwannomas) and immunosuppression (primary CNS recognizes a histone protein expressed in proliferating but not quiescent cells. Li-Fraumeni syndrome B. Mitotic activity D. Hypercellularity A. Necrosis C. Ki-68 D. 1340 Out of the following. 1338 Which of the following is a robust predictor of aggressive behavior of brain tumours ? 1344 In adults. Cadmium B. Temozolomide B. 1337 Which of the following is a robust predictor of aggressive behavior 1343 Oligodendrogliomas with deletions of which chromosome always of brain tumours ? respond to chemotherapy ? Harrison’s 17th Ed. mitotic activity & necrosis. 1342 Systemic combination chemotherapy (PCV) for oligodendrogliomas includes all except ? Harrison’s 18th Ed. 3386 A. Ki-69 For majority of primary brain tumors. D. Mitotic activity D. Thoracic region C.752 MCQ’s FOR MEDICAL PROFESSIONALS BY PROF. 3385 Harrison’s 18th Ed. 3384 Table 379-2 A. Lomustine (CCNU) C. Ozone D. Pilocytic astrocytoma Harrison’s 18th Ed. Procarbazine Harrison’s 18th Ed. Pearson syndrome is a mitochondrial disease consisting of pancreatic insufficiency. 2p C. Harrison’s 17th Ed. Shagreen patches B. Iris growing cystic tumors. B. Conjunctiva which hormone ? Hamartomas of the iris are called Lisch nodules. Frontal cortex A. 3390 A. Cerebellum Harrison’s 18th Ed. & encodes Harrison’s 18th Ed. 3389 B. Ca. C. II Tuberous sclerosis is characterized by cutaneous lesions. cerebellar. Its mutation causes von Recklinghausen’s metastases ? disease. Calcitonin A. Adenoma sebaceum 1346 Cranial nerve where Schwannoma never occur is ? B. Occipital cortex B. II D. lung 1349 Bourneville’s disease is also called ? B. Hippocampus Harrison’s 18th Ed. Seizures . Tuberous sclerosis 1357 Breast cancer has a propensity to metastasize to ? Harrison’s 18th Ed. Ca. 3390 C. Lentigo A. 3390 Tuberous sclerosis is also called Bourneville’s disease. Secondary syphilis D. 3388 D. Neurofibromatosis Type 1 C. 3390 D. fifth cranial nerve is the second most frequent site. 3390 C. seizures. Primary CNS lymphoma 1356 Breast cancer has a propensity to metastasize to ? Harrison’s 18th Ed. Cerebellum 1347 Lisch nodules relate best to ? C. 3388 C. 3389 D. Tuberous sclerosis Lung cancers (primary & metastatic) are the most common origin of brain metastases. 4 D. B. It encodes protein neurofibromin. seen in tuberous sclerosis. 9 Erythropoietin produced by hemangioblastomas results in polycythemia. Cutaneous lesions Breast cancer (esp. Retina are myelinated by oligodendroglia rather than by Schwann cells. III In tuberous sclerosis. 2607 Schwannomas are most frequent in the vestibular division of VIII cranial nerve. Meningioma D. ductal carcinoma) has a propensity to metastasize to the cerebellum and the posterior pituitary gland. Von Hippel–Lindau syndrome A. Ocular lens VHL syndrome consists of retinal. 17 1355 Which of the following is the most common origin of brain Tumor-suppressor NF1 gene on chromosome 17. Parathyroid hormone (PTH) 1348 NF1 gene is on chromosome ? Harrison’s 18th Ed. prostate B. B. Deafness Harrison’s 18th Ed. 3390 D. XI 1353 Hemangioblastomas in Von Hippel-Lindau syndrome are seen in ? Harrison’s 17th Ed. and spinal hemangioblastomas. Posterior pituitary gland C. D. 2607 A. earliest cutaneous sign is an ash leaf spot. Schwannomas are not found in optic & olfactory nerves as they A. thyroid A. C.shaped hypopigmented macules Harrison’s 18th Ed. or merlin. Erythropoietin B. Anterior pituitary gland 1350 Shagreen patches are best related to ? B. Anterior pituitary gland Shagreen patches are yellowish thickenings of skin over lumbosacral region of the back. Mental retardation A. All of the above A. Hypothalamus 1351 Tuberous sclerosis is characterized by all except ? D. 753 Neurology MCQ’s FORMCQ’s FOR MEDICAL MEDICAL PROFESSIONALS PROFESSIONALS BY PROF. schwannomin. C. Ca. Ca. and mental retardation. A. 1 C. V 1352 Cutaneous lesions in tuberous sclerosis include all except ? Harrison’s 18th Ed. Spinal cord Harrison’s 18th Ed. Ash leaf . III C. NF2 gene is on chromosome 22q. AJAY MATHUR Cardiology 753 1345 Schwannoma most frequently involves which cranial nerve ? C. 3390 neurofibromin 2. Retina 1354 Hemangioblastomas in Von Hippel-Lindau syndrome produce D. which are slowly B. Antidiuretic hormone C. ovary Harrison’s 18th Ed. B. A. Ependymoma D. VIII A. VIII D. Juvenile pilocytic astrocytoma A. and trauma. Liver D. Lung C. Brain abscess B. Bone B. AJAY MATHUR Neurology 1358 Which of the following cancers rarely metastasize to brain ? 1365 Which of the following is a germ cell tumor ? Harrison’s 18th Ed. All of the above patients have no evidence of an associated systemic illness. primary brain tumors. 3387 380 . Inflammation 1362 In vegetative state which of the following presists ? B. 3390 Harrison’s 16th Ed. Aggregates of T and B cells D.e. Astrocytic proliferation (gliotic areas) B. Ovarian cancer B. tuberculosis. predominantly T cells and macrophages. Immune-mediated preferential destruction of CNS myelin 1361 Differential diagnosis of ring-enhancement lesions include ? B. Pleura A. Relative sparing of axons is typical of MS. sarcoidosis. Demyelination Harrison’s 16th Ed. Radiation necrosis C. Any of the above Surviving oligodendrocytes partially remyelinate surviving naked axons. All of the above Prostate & ovarian cancer & Hodgkin’s disease rarely metastasize to brain. Yawning D. Remyelination of surviving naked axons C. 2607 1369 New MS lesions begin with ? Harrison’s 18th Ed. Radiologically. 3387 C. Astrocytic proliferation B. demyelination.. and gliosis (scarring) C. . 1366 Breast cancer that metastasizes to which of the following tends not to metastasize to brain ? 1359 Differential diagnosis of ring-enhancement lesions include ? Harrison’s 16th Ed. New MS lesions begin with perivenular cuffing by inflammatory mononuclear cells. Stroke Demyelinating disorders are immune-mediated conditions characterized by preferential destruction C.754 MCQ’s FOR MEDICAL PROFESSIONALS BY PROF. 1625 C. 3395 D. Harrison’s 18th Ed. All of the above 1360 Differential diagnosis of ring-enhancement lesions include ? Harrison’s 18th Ed. Glioblastoma multiforme D. 2455 A. Coughing Multiple sclerosis is characterized by a triad of inflammation. Swallowing with neuronal loss specially in advanced cases. disseminated in time and space). hemorrhage. Peripheral nervous system (PNS) is spared Harrison’s 18th Ed. radiation necrosis. Choriocarcinoma D. Toxoplasmosis D. No evidence of associated systemic illness A. Yolk sac tumor C. Primary CNS lymphoma D. primary CNS lymphoma. Perivenular cuffing by inflammatory mononuclear cells D. Hodgkin’s disease C. 1364 Brain tumor metastases that spread to spinal cord via CSF pathways are termed ? 1370 “Shadow plaques” refer to ? Harrison’s 16th Ed. The peripheral nervous system (PNS) is spared. Hemorrhage of central nervous system (CNS) myelin. 3395 A. D. 1363 Which out of the following is a clinically more aggressive tumor ? Harrison’s 17th Ed. Gliosis B. Infiltration of nervous system by B lymphocytes Grade IV GBM is a clinically aggressive tumor. Pleiomorphic xanthoastrocytoma C. 3387 A. Drop metastases C. All of the above Lesions of MS typically occur at different times and in different CNS locations (i. Pin metastases A. demyelinating lesions.Multiple Sclerosis and Other A. Remyelination A. 2458 Harrison’s 18th Ed. Confluent metastases D. None of the above B. 3395 A. 2455 Harrison’s 18th Ed. Prostate cancer A. producing “shadow plaques”. 3395 A. and most D. Disruption of blood-brain barrier (BBB) C. Primary brain tumors 1367 Which of the following is false about demyelinating disorders ? Harrison’s 18th Ed. Subependymal giant cell astrocytoma B. 1368 Triad in multiple sclerosis (MS) includes all except ? granulomas. Demyelinating lesions C. All of the above D. Teratoma B. Granulomas Demyelinating Diseases B. All of the above A. Hanging metastases B. toxoplasmosis. stroke. ring-enhancing lesions includes brain abscess. 3397 Table 380-2 1375 Highest known prevalence for MS is in ? Harrison’s 18th Ed. 1372 Which of the following is best related to pathophysiology of MS ? 1378 Which of the following races are inherently at higher risk for MS ? Harrison’s 18th Ed. MHC on chromosome 8 B. September D. 3397 Table 380-2 1376 Deficiency of which of the following vitamins is associated with A. Ataxia C. born in November. Paresthesias A. February B. AJAY MATHUR Cardiology 755 1371 Nerve conduction in myelinated axons occurs in a saltatory manner 1377 In northern hemisphere. 2462 Harrison’s 18th Ed. Opposite is true for southern hemisphere. Movement-induced muscle spasms Vitamin D deficiency is associated with an increase in MS risk. In In northern hemisphere. Fiji 1382 Which out of the following is the most common initial symptoms Highest known prevalence for MS is in Orkney Islands (Scotland) (250 per 100. continuous propagation of nerve action potentials through the in the genome ? demyelinated segment. 3395 Harrison’s 18th Ed. Theiler virus A. All of the above In MS. A C. MHC on chromosome 6 1373 Risk of developing MS is highest ? Harrison’s 18th Ed. ~ 110 meters / second D. conduction blocks occur. ~ 90 meters / second C. 755 Neurology MCQ’s FORMCQ’s FOR MEDICAL MEDICAL PROFESSIONALS PROFESSIONALS BY PROF. due to demyelination. of MS ? Harrison’s 18th Ed. ~10% cases begin <18 years of age 1381 Which out of the following is the most common initial symptoms Prevalence rates of MS increase at higher latitudes. If a spouse has MS Risk of developing MS is highest if an identical twin has MS (1 in 3). If a parent has MS Major histocompatibility complex (MHC) on chromosome 6 is by far the strongest MS susceptibility region in the genome. D D. MHC on chromosome 9 C. Tasmania D. 1380 Which virus produces demyelinating disease having lesions resembling lesions of MS ? 1374 Which of the following is false about multiple sclerosis ? Harrison’s 16th Ed. 3396 A. 3396 A. Diplopia B. resting axon membrane becomes hyperpolarized due to exposure of voltage-dependent potassium channels. Age of onset is between 20 and 40 years C. With demyelination sodium channels redistribute along the naked axon allowing 1379 Which of the following is the strongest MS susceptibility region slow. Sodium channels are concentrated at the nodes of Ranvier. 3395 A. the myelin sheath are exposed. C Harrison’s 18th Ed. Africans C. Redistribution of potassium channels along naked axon A. Thereafter. Middle east B. Epstein-Barr D. . rather than saltatory. Prevalence decreases at higher latitudes D. D. 3396 Table 380-1 B. Thrice more common in women than men B. Asians D. Diplopia Harrison’s 18th Ed. Bladder dysfunction C. This is called “month-of-birth effect”. babies born in which month are at a velocity of ? significantly more likely to develop MS ? Harrison’s 18th Ed. 3395 Figure 380-1 Harrison’s 18th Ed. continuous propagation occurs at ~1 m/sec. If a sibling has MS D. E 1383 Which of the following is characteristic of MS ? D. 3396 A. Redistribution of chloride channels along naked axon D. babies born in May are significantly more likely to develop MS than those unmyelinated nerves. Redistribution of calcium channels along naked axon C. Harrison’s 18th Ed. Axons that are normally buried underneath Whites are inherently at higher risk for MS than Africans or Asians. Vertigo D. 3395 A. Whites B. Varicella B. ~ 50 meters / second A. Orkney Islands C. Sensory loss an increase in MS risk ? B. December Nerve conduction in myelinated axons occurs in saltatory manner at a velocity of ~70 m/sec. 3397 A. ~ 70 meters / second B. 3396 A. Rubella C. If an identical twin has MS C. Ongoing deficiency may increase the relapse rate in established MS.000). Optic neuritis B. May C. Redistribution of sodium channels along naked axon B. MHC on chromosome 7 A. of MS ? Harrison’s 18th Ed. Radiates down the back into legs B. 3398 A. 3398 Harrison’s 18th Ed. optic neuritis (ON) is least likely to presents as ? Harrison’s 18th Ed. “one and a half” syndrome (horizontal gaze palsy + an occurring in <20% of patients. Slowed information processing Bilateral internuclear ophthalmoplegia (INO) is particularly suggestive of MS. Periorbital pain often precedes or with ipsilateral loss of taste or retroauricular pain. 3398 Harrison’s 18th Ed. 3398 B. typically induced by flexion of neck. Unilateral visual blurring may occur during a hot shower or with physical exercise (Uhthoff’s symptom). Cerebellar tremors A.756 MCQ’s FOR MEDICAL PROFESSIONALS BY PROF. 1390 Which of the following was once thought to be characteristic of 1384 Which of the following is strongly suggestive of MS ? MS ? Harrison’s 18th Ed. Relapsing/remitting MS (RRMS) . 1391 In MS. Bilateral internuclear ophthalmoplegia (INO) B. Uveitis is uncommon Facial weakness due to a MS lesion in pons resembles idiopathic Bell’s palsy but is not associated Visual symptoms are generally monocular but may be bilateral. or sleep disturbances. All of the above D. which of the following statements is false ? Harrison’s 18th Ed. Sensory level Lhermitte’s symptom is an electric shock like sensation. Acquired pendular nystagmus D. Paresthesias C. Hypesthesia D. Complete loss of light perception 90% of patients of MS experience fatigue which is exacerbated by elevated temperatures. Electric shock . 1394 Which of the following is false about Lhermitte’s symptom ? Harrison’s 18th Ed. Depression B. 3398 A. (desaturation) in the central field of vision. 90 % Bladder dysfunction is present in >90% of MS patients. fatigue can be exacerbated by ? 1385 In MS. ‘Uhthoff’s symptom’ is related to ? Harrison’s 18th Ed. D. 3398 B. by spasticity. Periorbital pain precedes or accompanies visual loss C. 3398 1388 Which of the following about sensory symptoms in MS is false ? A. Heat sensitivity B. dimness. Memory loss D. Visual symptoms A. Decreased color perception D. disturbances include horizontal gaze palsy. Weakness is of UMN type accompanied Ataxia usually manifests as cerebellar tremors. Episodic ataxia D. 3398 Harrison’s 18th Ed. that D. AJAY MATHUR Neurology B. 3398 A. Horizontal gaze palsy A. Acquired pendular nystagmus D. bandlike cause it. Monocular visual loss C. Dimness C. Other gaze Euphoria (elevated mood) was once thought to be characteristic of MS but is actually uncommon. Sleep disturbances C. Optic neuritis (ON) presents as diminished visual acuity. Vitamin B 12 deficiency syndrome and Cisplatin toxicity can also Sensory symptoms in MS include paresthesias. Funduscopic examination may be normal or reveal optic disc swelling (papillitis). Afferent pupillary defect is usually present D. Evoked by neck flexion A. accompanies visual loss. or decreased color perception depression. unpleasant sensations. Hyperthyroid tremors C. Facial myokymia of MS patients ? Harrison’s 18th Ed. 75 % Heat sensitivity refers to neurologic symptoms produced by elevation of body’s core temperature. Movement C. there is complete loss of light perception. Diminished visual acuity B. Rarely. Exercise-induced weakness of limbs B. Resembles idiopathic Bell’s palsy A. Gait apraxia Exercise-induced weakness of limbs is characteristic of MS. Not associated with ipsilateral loss of taste sensation B. None of the above radiates down the back into legs. 3398 Harrison’s 18th Ed. 65 % D.like sensation Harrison’s 18th Ed. Euphoria B. Lhermitte’s symptom can also occur with other disorders of cervical spinal cord (cervical spondylosis). Visual symptoms are generally bilateral B. hyperreflexia & Babinski signs. “One and a half” syndrome C. which of the following is the commonest clinical type of 1389 Ataxia in MS usually manifests as ? MS ? Harrison’s 18th Ed. Pallor of optic disc (optic atrophy) commonly follows ON. Elevated temperatures A. INO) & acquired pendular nystagmus. Impaired attention C. 50 % C. Not associated with retroauricular pain C. 1392 In MS. hypesthesia. Uveitis is uncommon. facial weakness is characterised by ? 1386 In MS. 1393 In MS. Afferent pupillary defect is usually present. 3398 A. sensation of tightness around torso. All of the above D. 1395 At onset. 3398 1387 Symptoms of bladder dysfunction are present in what percentage A. None of the above C. All of the above A. Vitamin B12 deficiency. are gait-impaired and occupationally disabled. Those with truncal C. Lesions in the anterior corpus callosum are frequent in MS and rare in vascular disease. A. Relapsing/remitting MS (RRMS) D. C. and leukoencephalopathy (CADASIL). parkinsonism. Sarcoid D.5 have RRMS and are generally not disabled. Secondary progressive MS (SPMS) Clinical features that point towards favorable prognosis in MS are ON or sensory symptoms at onset. D. Any of the above include all except ? Harrison’s 18th Ed. Sarcoid. Propionyl-CoA carboxylase 1405 Most patients with Kurtzke Expanded Disability Status Score B. Progressive/relapsing MS (PRMS) 1400 Which of the following is uncommon in MS ? Kurtzke Expanded Disability Status Score (EDSS) is a measure of neurologic impairment in MS. 3399 A. Tropical spastic paraparesis (HTLV I/II infection). Parkinson disease are oriented perpendicular to ventricular surface. chorea. episodic loss of consciousness. Seizures C. fever.5 have ? Harrison’s 18th Ed. C. D. Argininosuccinate lyase (EDSS) scores < 3. A. Systemic lupus erythematosus. Secondary progressive MS (SPMS) B. Creutzfeldt-Jakob disease Harrison’s 18th Ed. isolated dementia. 3402 D. severe muscular atrophy. Alzheimer’s disease 1399 Which of the following is a marker of axonal integrity ? D. Progressive/relapsing MS (PRMS) D. All of the above Harrison’s 18th Ed. glioma. Stroke & ischemic cerebrovascular A. 3400 Most patients with EDSS scores <3. 3402 of >100 mg/dL in CSF should raise doubts about MS. AJAY MATHUR Cardiology 757 B. 757 Neurology MCQ’s FORMCQ’s FOR MEDICAL MEDICAL PROFESSIONALS PROFESSIONALS BY PROF. Pyramidal symptoms prognosis ? C. 20 % B. B. 3401 with complete recovery over weeks to months. action tremor. Glatiramer acetate 1401 Disorders that can mimic multiple sclerosis are all except ? D. B. patients never develop neurologic disability & maintain a benign course. metachromatic leukodystrophy).5 have progressive MS (SPMS or PPMS). Age < 40 years at onset A. subcortical infarcts. HSV-1 encephalitis Relapsing/remitting MS (RRMS) accounts for 85% of MS cases at onset. Mild CSF pleocytosis (> 5 cells/µL) D. HIV infection. A. Secondary progressive MS (SPMS) N-acetyl aspartate. Dawson’s fingers refers to lesions as seen in MRI that A. peripheral neuropathy. 3399 Lyme disease. Primary progressive MS (PPMS) C. Dawson’s fingers 1404 Kurtzke Expanded Disability Status Score (EDSS) is used for ? Pleocytosis of >75 cells/µL. Antiphospholipid antibody syndrome (APLA). Mitochondrial encephalopathy with lactic acidosis and stroke (MELAS). Vitamin B12 deficiency . Syphilis. Neoplasms (lymphoma. 10 % A. Relapsing/remitting MS (RRMS) disease. CSF protein concentration of > 100 mg/dL In benign variant of MS. pyramidal symptoms or progressive disease are likely to be disabled. or coma. Primary progressive MS (PPMS) Vascular malformations. Progressive/relapsing MS (PRMS) 1402 Clinical features that have a more favorable prognosis in MS D. Behçet’s disease. headache. Syphilis C. Proton magnetic resonance spectroscopic C. 3399 Figure 380-3 C. N-acetyl aspartate A. B. Mitoxantrone seizures. Harrison’s 18th Ed. Congenital leukodystrophies 1396 Secondary progressive MS (SPMS) always begins as ? (adrenoleukodystrophy. Chorea 1406 Which of the following is used for acute attacks of MS ? Harrison’s 18th Ed. Lesions in the anterior corpus callosum Likelihood of having benign MS is <20%. Glucocorticoids Uncommon or rare symptoms in MS are aphasia. 5 % Harrison’s 18th Ed. Primary progressive MS (PPMS) imaging (MRSI) can quantitate it. meningioma). Sjögren’s syndrome. which is a marker of axonal integrity. 2 % 1398 Which of the following does not favour the diagnosis of MS ? B. 3402 C. Optic neuritis or sensory symptoms at onset 1397 Which of the following clinical type of MS has a more favorable B. Relapsing/remitting MS (RRMS) B. 3402 Table 380-4 Glucocorticoids are used to manage first attacks or acute exacerbations of MS. Primary progressive MS (PPMS) ataxia. or a protein concentration Harrison’s 18th Ed. C. N-Acetylglutamate synthase B. <2 relapses in 1st year of illness & minimal impairment after 5 years. 3399 D. Disorders that can mimic multiple sclerosis are acute disseminated encephalomyelitis (ADEM). presence of polymorphonuclear leukocytes. 3401 Secondary progressive MS (SPMS) always begins as RRMS. Ischemic optic neuropathy. Multiple sclerosis perivenous demyelination. D. Vasculitis. and corresponding to pathologic pattern of B. Between attacks. Harrison’s 18th Ed. Progressive/relapsing MS (PRMS) 1403 Likelihood of having benign MS is about ? Relapsing/remitting MS (RRMS) is characterized by discrete attacks that evolve over days to weeks Harrison’s 18th Ed. Cerebral autosomal dominant arteriopathy. Women Harrison’s 18th Ed. patients are neurologically stable. Aphasia EDSS scores >5. Patients with A. C. Isolated dementia Untreated patients are followed closely with periodic brain MRI scans. 3408 1413 Mitoxantrone is indicated for use in ? A. IFN 1a D. IFN  1417 In neuromyelitis optica. Mitoxantrone C. C. Glatiramer acetate (GA) D. Neurologic status abnormal between MS attacks awaits FDA approval. Worsening CSF IgG index 1408 Which of the following drugs is used for patients with progressive C. None of the above 1409 Untreated patients of MS should be followed with ? Harrison’s 18th Ed. Mitoxantrone (MTX) Mitoxantrone is indicated in SPMS. PRMS. Mitoxantrone B. cerebellar & cognitive involvement. PRMS Harrison’s 18th Ed. C. 3405 A highly specific autoantibody directed against water channel protein aquaporin-4 is seen in >50% A. the course of disease D. Aquaporin-4 is localized to foot processes of astrocytes in close apposition to endothelial surfaces. IFN 1a who have failed other treatments. IFN--1a 1418 Autoantibody against which of the following is seen in B. Natalizumab C. Natalizumab A.-1a (IM). patients with NMO do not experience brainstem. 3408 B. 3405 & the brain MRI is typically normal. Acute Disseminated Encephalomyelitis (ADEM) D. Mitoxantrone Harrison’s 18th Ed. 3406 No currently available therapies have shown any promise for treating PPMS at this time. Kurtzke Expanded Disability Status Score (EDSS) A. IFN--1b. 3402 C. IFN--1b neuromyelitis optica ? C. Glatiramer acetate 1419 In Marburg’s variant of multiple sclerosis. Cladribine A. Glatiramer acetate (SC). 3405 Disease-Modifying Therapies for Relapsing Forms of MS (RRMS. IFN--1a patients of NMO. AJAY MATHUR Neurology 1407 Disease modifying therapies for RRMS include ? B. and mitoxantrone. MRI lesions >3 mm in diameter disability in MS ? D. Glucocorticoid between MS attacks. Worsening RRMS A. IFN. Subacute A. Hyperacute Harrison’s 18th Ed. or glatiramer acetate as first-line therapy. 3408 C. Glatiramer acetate A. Fingolimod is ? Harrison’s 18th Ed.758 MCQ’s FOR MEDICAL PROFESSIONALS BY PROF. Aquaporin-1 B. Glatiramer acetate D. 3406 D. Brainstem C. and worsening RRMS. Periodic brain MRI scans B. C. Harrison’s 18th Ed. which of the following is involved ? Harrison’s 18th Ed. D. None of the above 1411 Which of the following is administered by IV infusion ? In contrast to MS. Mitoxantrone should not be used as a first-line agent in either RRMS or relapsing SPMS. Cognitive functions Relapsing forms of MS are treated with IFN. 3405 B. Glatiramer acetate Harrison’s 18th Ed. Aquaporin-3 1412 Which of the following is administered orally ? D. All of the above 1414 Worsening RRMS is defined as ? Harrison’s 18th Ed. B. A. B.-1b (SC). Neuromyelitis optica (NMO) D. Over three or more spinal cord segments are involved. Plasma exchange 1415 Which of the following therapies is effective in treating PPMS ? C. Involvment of more sites of CNS Harrison’s 18th Ed. 3402 Worsening RRMS is defined as patients whose neurologic status remains significantly abnormal A. Aquaporin-4 Harrison’s 18th Ed. 3405 A. Mitochondrial encephalopathy 1410 Which of the following is the first-line therapy for Relapsing/ remitting MS ? Neuromyelitis optica (NMO) is also called Devic’s syndrome. 3408 D. SPMS with Exacerbations) include IFN--1a. Glucocorticoids Mitoxantrone is an immune suppressant generally reserved for patients with progressive disability B. Harrison’s 18th Ed. All of the above B. Natalizumab (IV infusion). fingolimod. Evoked Potential testing Harrison’s 18th Ed. A. Cerebellum D. glatiramer acetate & natalizumab. SPMS . B. IV Ig A. Aquaporin-2 IFN. C. Objective clinical CNS examination 1416 Devic’s syndrome refers to which of the following ? B. All of the above D. Hallmark of ADEM is presence of widely scattered small foci of perivenular inflammation & demyelination. D. A. Meningitis C. Astroglial scarring 1428 ADEM is differentiated from MS by ? 1422 Most explosive form of acute disseminated encephalomyelitis Harrison’s 18th Ed. Abscess. influenza. Hip joint . and Empyema 1424 ADEM can follow infections with all except ? Harrison’s 18th Ed. Rubella B. Bilateral involvement of optic nerve 1423 Which of the following is false about acute disseminated B. Legionella D. All of the above A. Complete transverse myelopathy encephalomyelitis (ADEM) ? C. 759 Neurology MCQ’s FORMCQ’s FOR MEDICAL MEDICAL PROFESSIONALS PROFESSIONALS BY PROF. coma. Seizures C. Measles A. Lesions are vasculitic Harrison’s 18th Ed. Encephalitis. Monophasic course Harrison’s 18th Ed. Drowsiness or coma B Acute hemorrhagic leukoencephalitis C. Metachromatic leukodystrophy 1429 ADEM is differentiated from MS by ? Acute hemorrhagic leukoencephalitis is the most explosive form of ADEM. Varicella B. Plaque is a chronic inflammatory. Mumps moves ? Harrison’s 18th Ed. Primary targets are myelin sheath & oligodendrocyte All forms of ADEM result from a cross-reactive immune response to infectious agent or vaccine D. seizures. simultaneous onset of B. 1426 Which CNS antigens correlates with development of ADEM ? Harrison’s 18th Ed. Hypertrophic astrocytes In ADEM due to chickenpox. mumps. complete transverse myelopathy. Mumps C. 3408 A.354:942-55 B. Postinfectious encephalomyelitis D. Thrombophlebitis 1425 ADEM can follow infections with all except ? Focal bacterial. 3409 (ADEM) is called ? Harrison’s 18th Ed. varicella (chickenpox).354:942-55 Harrison’s 18th Ed.Meningitis. parainfluenza. Bruton’s tyrosine kinase (BTK) B. plasma cells. loss of myelin and oligodendrocytes B. myelin-laden C. Encephalitis B. D. Indistinct margin. Tropical spastic paraparesis D. Infectious mononucleosis virus A. Abscess D. Fulminant Postvaccinal encephalomyelitis may follow administration of smallpox vaccine & rabies vaccine. Mycoplasma macrophages D. Hepatitis B 1431 While eliciting Brudzinski’s sign which part of body is passively C. 3408 1430 Cerebritis is best related to ? Harrison’s 18th Ed. 3410 D. Measles infiltration. Postinfectious encephalomyelitis is associated with infection with measles virus. infectious mononucleosis viruses and Mycoplasma. C. Postvaccinal encephalomyelitis disseminated symptoms & signs. Brain encephalomyelitis) or infection (postinfectious encephalo-myelitis). meningismus. Gray matter lesions are known to occur that triggers an inflammatory demyelinating response. Polyphasic course Features that suggest ADEM rather than MS are monophasic course. A. 3408 or abscess. drowsiness. Simultaneous onset of disseminated symptoms & signs A. 3410 A. intense perivascular A. Myelin basic protein (MBP) A. AJAY MATHUR Cardiology 759 C. fungal or parasitic infections involving brain tissue are classified as either cerebritis Harrison’s 18th Ed. cerebellar involvement is often conspicuous. Widespread axonal damage. Mitochondrial encephalopathy B. Perivenular inflammation & demyelination ADEM has a monophasic course & is associated with antecedent immunization (postvaccinal 381 . 3408 D. 1421 Which of the following about acute lesions of multiple sclerosis is false ? 1427 Cerebellar involvement in ADEM is common in infections due to ? N Engl J Med 2006. Mycoplasma C. Chronic rubella. Stromal-cell derived factor (SDF) C. Autoantibodies to MBP and to other myelin antigens have been detected in the CSF from many patients with ADEM. 3408 A. Vascular cell adhesion molecule (VCAM) N Engl J Med 2006. 3409 & hemorrhagic & the clinical course is devastating. Acute MS (Marburg’s variant) is a fulminant demyelinating process that in some cases progresses inexorably to death within 1–2 years. Varicella B. When brain tissue is directly injured by a viral infection the disease is referred to as encephalitis. Plaque lesions develop in white matter D. depending on the presence or absence of a capsule. hypercellularity. 3408 1420 Which of the following about multiple sclerosis is false ? A. bilateral optic nerve involvement. demyelinating lesion C. < 0. Pentraxin A. H. B.6 D. Cytotoxic edema B. toxic effects of antimicrobial agents (high-dose A. group B streptococci (~15%). including properdin. < 0. All of the above acquired bacterial meningitis is ? Harrison’s 18th Ed. D. N. Cathelin B. Carcinomatous meningitis D. Ankle joint A. > 100 mm H 2O N. 3412 1440 In bacterial meningitis. B. 3414 B. Streptococcus pneumoniae 1438 Over 90% of patients of bacterial meningitis have a CSF opening C. Tuberculous meningitis C. Toxic effects of antimicrobial agents 1432 In adults. All of the above C.8 Normal CSF contains few white blood cells (WBCs) and relatively small amounts of complement proteins and immunoglobulins. Galectin-1 1439 Cushing reflex in raised ICP relates to ? B. A. Properdin Harrison’s 18th Ed. < 0. 3413 Organisms responsible for community-acquired bacterial meningitis are S. meningitidis (~25%). CSF/serum glucose ratio in bacterial meningitis is <0.6 The combination of interstitial.4 C. < 0. C.4 is seen in ? Harrison’s 18th Ed. normal CSF ? Harrison’s 18th Ed. are C. 1436 Which of the following is not a feature of classic clinical triad of meningitis ? 1442 CSF / serum glucose ratio of <0. 3412 is ? A.8 CSF glucose concentration is low when CSF/serum glucose ratio is <0. < 0. 3413 Harrison’s 18th Ed. Interstitial edema Harrison’s 18th Ed. All of the above The classic clinical triad of meningitis is fever. 3411 More than 90% of patients will have a CSF opening pressure >180 mmH2O. All of the above C. tuberculous. Immunoglobulins B. cerebral anoxia or less commonly. Decreased level of Besides bacterial meningitis. vomiting & photophobia are also common complaints.4 is also seen in fungal. Complement proteins A. Headache B. White blood cells (WBCs) Harrison’s 18th Ed.6. C. D. > 160 mm H 2O 1433 Individuals with deficiencies of which of the following are highly D. A. Listeria monocytogenes (~10%) & H.4 in ~60% of patients. Fungal meningitis B. CSF / serum glucose ratio is ? A. < 0.760 MCQ’s FOR MEDICAL PROFESSIONALS BY PROF. Nuchal rigidity D. AJAY MATHUR Neurology B. and carcinomatous meningitis. Vasogenic edema A. 3410 Generalized seizure activity & status epilepticus in bacterial meningitis may be due to hyponatremia. D.4 C. 3413 C. . Hyponatremia Brudzinski’s sign is positive when passive flexion of neck results in spontaneous flexion of hips & B. Bradycardia D. consciousness occurs in >75% of patients. CSF/serum glucose ratio <0. meningitidis penicillin). Irregular respirations highly susceptible to meningococcal infections. vasogenic. 1435 Raised intracranial pressure in bacterial meningitis is due to ? 1441 CSF glucose concentration is low when CSF/serum glucose ratio Harrison’s 18th Ed. All of the above 1434 Which of the following is present in relatively small amounts in Cushing reflex includes bradycardia. and cytotoxic edema leads to raised ICP and coma. Fever A. organism most commonly responsible for community. 3413 D. Vomiting C. Knee joint 1437 Generalized seizure and status epilepticus in bacterial meningitis C. Hypertension Individuals with deficiencies of any of the complement components. < 0. 3414 B. pneumoniae (~50%). Nausea. influenzae Harrison’s 18th Ed. Cerebral anoxia knees.5 D.5 D. 3414 A. > 180 mm H 2O susceptible to meningococcal infections ? Harrison’s 18th Ed. > 120 mm H 2O influenzae (<10%). < 0. Neck may be due to ? Harrison’s 18th Ed. headache & nuchal rigidity. Group B streptococci pressure of ? D. hypertension & irregular respirations. Dexamethasone therapy is unlikely to be of significant benefit if started > 6 hours after antimicrobial therapy has been initiated. 6 hour C. aeruginosa ? many hours after antimicrobial therapy ? Harrison’s 18th Ed. 761 Neurology MCQ’s FORMCQ’s FOR MEDICAL MEDICAL PROFESSIONALS PROFESSIONALS BY PROF. 3417 A. 1 hour A. Penicillin G Vogt-Koyanagi-Harada syndrome refers to uveomeningitic syndromes. 3416 B. Cefotaxime B. adult dose of Harrison’s 18th Ed. the goal is to begin antibiotic therapy within 60 minutes of patient’s arrival Rationale for giving dexamethasone 20 minutes before antibiotic therapy is that it inhibits production in the emergency room. Drug-induced hypersensitivity meningitis acute meningitis for coverage of L. Along with antibiotic therapy C. Gram-negative bacterial meningitis Harrison’s 18th Ed. Neck C. 3416 Table 381-3 Harrison’s 18th Ed. 3415 Harrison’s 18th Ed. 600 mg twice a day for 2 days Skin rash in RMSF typically begins in wrist & ankles. Close contacts of meningococcal meningitis patient should receive chemoprophylaxis with rifampin. > 3 hours C. Uveomeningitic syndromes C. Cryptococcus C. of TNF by macrophages & microglia only if it is administered before these cells are activated by endotoxin. . Coccidioides immitis and Treponema pallidum. Vancomycin can be given by intraventricular route neoformans. Ampicillin C. Mycobacterium tuberculosis 1452 Which of the following is false about vancomycin therapy in S. Any of the above In bacterial meningitis. > 6 hours D. Meningitis associated with sarcoid B. 3416 B. 450 mg once a day for 1 days C. azithromycin or ceftriaxone. In meningococcal meningitis chemoprophylaxis. Ceftazidime C. skin rash typically begins in ? 1450 In meningococcal meningitis chemoprophylaxis. > 1 hour B. pneumoniae meningitis ? B. Carcinomatous meningitis A. 3415 B. 3417 A. Treponema pallidum A. Antibiotic of choice if cephalosporin MIC is >1 µg/mL D. 3416 C. Tuberculous meningitis B. > 12 hours Meningitis due to P. monocytogenes ? Harrison’s 18th Ed. monocytogenes. Histoplasma capsulatum. Azithromycin D. Cryptococcus neoformans Harrison’s 18th Ed. It spreads distally & proximally rapidly to involve palms & soles. 3415 1451 Which antibiotic should be added to the empirical regimen in A. None of the above D. Any of the above Limulus amebocyte lysate assay detects gram-negative endotoxin in CSF & is diagnostic of gram- negative bacterial meningitis. 3415 rifampin is ? A. 1445 Vogt-Koyanagi-Harada syndrome refers to ? Harrison’s 18th Ed. Rifampin C. aeruginosa should be treated with ceftazidime. Metronidazole 1446 Which of the following causes a subacutely evolving meningitis ? Ampicillin should be added to the empirical regimen for coverage of L. Before antibiotic therapy B. Carcinomatous meningitis C. ciprofloxacin. None of the above 1447 In bacterial meningitis. Wrists and ankles A. AJAY MATHUR Cardiology 761 1443 Limulus amebocyte lysate assay in CSF is diagnostic of ? 1449 Close contacts of a patient of meningococcal meningitis should Harrison’s 18th Ed. After antibiotic therapy D. Face and scalp Harrison’s 18th Ed. 3 hour B. D. All of the above B. cefepime. Ciprofloxacin D. 12 hour D. 450 mg twice a day for 2 days D. 1444 In Rocky Mountain spotted fever (RMSF). the goal is to begin antibiotic therapy within what time of patient’s arrival ? 1453 When should dexamethasone be given in acute bacterial meningitis ? Harrison’s 18th Ed. Harrison’s 18th Ed. Rifampin produces synergistic effect to vancomycin Subacutely evolving meningitis is caused by Mycobacterium tuberculosis. Chest and abdomen B. D. Fungal meningitis A. 3415 A. 600 mg once a day for 1 days D. 3414 receive chemoprophylaxis with ? A. or meropenem. Nafcillin D. 1448 Which of the following cephalosporin is preferred in CNS infection 1454 Benefit of dexamethasone therapy is not significant if started how with P. Ceftriaxone A. rifampin is given as a 2-day regimen of 600 mg every 12 hour for 2 days in adults. Fever with ? may be accompanied by malaise. especially in patients with infections due to echovirus 9. Reduced glucose concentration B. AJAY MATHUR Neurology 1455 Benefits of dexamethasone therapy are most striking in patients Viral meningitis presents as fever. HSV-2 C. Arboviruses A. abdominal pain. should prompt consideration of alternative diagnoses. Common sequelae of bacterial meningitis include decreased intellectual function. Adenoviruses rarely cause viral meningitis. 3417 Mild degree of lethargy or drowsiness may occur but more profound alterations in consciousness like stupor. a normal glucose concentration. Risk of death from bacterial meningitis increases with decreased level of consciousness on admission. Echovirus 9 B. D. 3417 B. Marked confusion A. Meningococcal meningitis A. Harrison’s 18th Ed. L. Enteroviruses Harrison’s 18th Ed. Normal or slightly elevated protein . echoviruses. nausea and vomiting. onset of seizures within 24 hours of admission. Fever B. Decreased intellectual function C. A. and human enteroviruses 68 to 71. West Nile virus 1460 Which of the following is uncommon in viral meningitis ? C. 1464 PMNs may predominate in CSF of viral meningitis in the first 48 hours of illness in which of the following infections ? Harrison’s 18th Ed.frontal or retroorbital 1456 Mortality is higher in meningitis due to ? C. seizures. Normal or mildly elevated pressure (100 . meningeal irritation and an inflammatory CSF profile. Seizures A.25 to 500 cells/µL 1458 Common sequelae of bacterial meningitis include all except ? Harrison’s 18th Ed. All of the above genus are members of the family Picornaviridae and include the coxsackieviruses. Visual loss Examination of CSF is the most important laboratory test in the diagnosis of viral meningitis. B. Viruses belonging to the Enterovirus D. a normal or slightly elevated D. meningitidis eyes & mild teminal nuchal rigidity. Lymphocytic pleocytosis . Headache . Seizures or other focal neurologic signs or symptoms C. Echovirus 9 B. 15% for that due to L. hearing loss & dizziness & gait disturbances. Gram-negative bacterial meningitis B. CSF glucose concentration of < 40 mg/dL D. A. Kernig’s and Brudzinski’s signs D. 3417 D. All of the above A. headache. monocytogenes and 20% for S. CSF protein concentration of > 300 mg/dL Enteroviruses account for > 85 % of aseptic meningitis cases. Meningeal irritation meningitis due to which of the following ? Harrison’s 18th Ed. such as Kernig’s and Brudzinski’s signs. young age (infancy) & age >50. N. Mumps causing viral meningitis ? Harrison’s 18th Ed. presence of shock and/or need for mechanical ventilation.350 mm H2O) C. The typical profile is a lymphocytic pleocytosis (25 to 500 cells/µL). Echovirus 9 . meningitidis or group B streptococci. pneumoniae 1462 Which of the following viruses is a less common cause of viral Mortality is 3 . Mumps 1465 Cell counts of several thousand per microliter are seen in viral D. 3418 D. anorexia. CSF glucose concentration <40 mg/dL and CSF protein >300 mg/dL. influenzae. signs of increased ICP. Arboviruses & enteroviruses are frequent during summer/early fall. coma. N.7% for meningitis caused by H. Harrison’s 18th Ed. meningitis ? Harrison’s 18th Ed. 3418 Viral meningitis A. 3418 A. Seizures D. West Nile virus 1459 Which of the following viruses has a nonseasonal prevalence in C. LCMV C. LCMV & mumps viruses cause viral meningitis during winter months. 3418 HSV & HIV viral meningitis have no seasonal predominance. and/or diarrhea. 3419 1457 Predictors of increased mortality in bacterial meningitis include ? A. monocytogenes suggest involvement of the brain parenchyma and do not occur in uncomplicated viral meningitis. Focal neurological deficits Harrison’s 18th Ed. or marked confusion. Headache 1466 Cell counts of several thousand per microliter are seen in viral C. 3417 D.762 MCQ’s FOR MEDICAL PROFESSIONALS BY PROF. influenzae Headache in viral meningitis is usually frontal or retroorbital with photophobia. S. polioviruses. Adenoviruses B. 3417 B. delay in initiation 1463 Typical CSF profile in viral meningitis includes all except ? of treatment. memory impairment. HSV meningitis due to which of the following ? Harrison’s 18th Ed. Evidence of severe meningeal irritation. myalgia.20 to 80 mg/dL A. Hearing loss protein concentration (20 to 80 mg/dL). Arboviruses A. Mumps Harrison’s 18th Ed. 3417 D. pneumoniae. Arboviruses PMNs may predominate in the first 48 hours of illness. or mumps. H. 3417 C. Staphylococcal meningitis B. pain on moving B. Onset of seizures within 24 hours of admission C. is generally absent. and a normal or mildly elevated opening pressure (100 to 350 mm H2O). West Nile virus or Eastern equine encephalitis virus. Pneumococcal meningitis 1461 Which of the following is common in viral meningitis ? Harrison’s 18th Ed. But. mumps. 763 Neurology MCQ’s FORMCQ’s FOR MEDICAL MEDICAL PROFESSIONALS PROFESSIONALS BY PROF. 3418 A. 3418 Harrison’s 18th Ed. HIV A. West Nile virus (WNV) B. Neopterin B. Throat washings C.50 ºC D. 3418 B. although it may be decreased For West Nile virus (WNV). CMV A. 1475 Oligoclonal bands in CSF is not found in which of the following 1469 In CSF. listerial. CMV The CSF glucose concentration is typically normal in viral infections. West Nile virus B.40 ºC C. Feces Harrison’s 18th Ed.g. . 3418 A. lymphocytic pleocytosis with low glucose concentration should suggest which of the following ? 1476 Oligoclonal bands are found in which of the following ? Harrison’s 18th Ed. Lyme borreliosis D. or HSV infections. HSV B. Lymphocytic choriomeningitis virus (LCMV) C. lymphocytic pleocytosis with low glucose concentration conditions ? should suggest which of the following ? Harrison’s 18th Ed. Echovirus 9 C. leukopenia. PCR amplification of viral nucleic acid ‘Mollaret’s meningitis’ appear to be due to HSV. mumps & D. 1468 Low CSF glucose concentration is seen in viral meningitis due to 1474 Serologic studies are a crucial diagnostic tool for which of the which of the following ? following viruses ? Harrison’s 18th Ed. although cell counts of several Beyond 24 hours. arboviruses & LCMV in blood.70 ºC Total CSF cell count in viral meningitis is typically 25 to 500/µL. in 10 to 30% of cases due to mumps as well as in cases due to LCMV. Blood A. . 1471 Which of the following is the most important method for 1477 ‘Mollaret’s meningitis’ is due to ? Harrison’s 18th Ed. . EBV infection . HIV infection A. Lymphocytic choriomeningitis virus (LCMV) Enteroviruses & adenoviruses may be found in feces. EBV C. especially with infections due to lymphocytic choriomeningitis virus (LCMV) and mumps virus. VZV D. SSPE & progressive rubella panencephalitis. Mumps C. Tuberculous meningitis C. 2478 A. 3418 A. West Nile virus B. and adenoviruses in throat washings. All of the above D. 1473 Enteroviruses may be found in all except ? Harrison’s 18th Ed. Syphilis C. Fungal meningitis B. CSF specimens for viral isolation are stored in a (-)70°C freezer. Arboviruses CMV. . enterovirus. 2-microglobulin C. 1470 In CSF. HSV C. West Nile virus D. Multiple sclerosis B. 1478 Aseptic meningitis occuring in winter. 3419 diagnosing CNS viral infections ? Harrison’s 18th Ed. All of the above Oligoclonal bands in CSF are not seen with arbovirus. CMV D. 3419 Harrison’s 16th Ed. Progressive rubella panencephalitis D. LDH D. 3418 Harrison’s 18th Ed. Arboviruses D. syphilis & Lyme borreliosis. mumps. HSV infections B. HSV-2 infection A. thrombocytopenia and CSF 1472 At what temperature should CSF be stored for culture for diagnosis pleocytosis suggest the diagnosis of which infection ? of viral infection (> 24 hours) ? Harrison’s 18th Ed. All of the above As a rule.20 ºC B. thousand per microliter are occasionally seen. in urine and enteroviruses. Urine B. HTLV type I. 3418 Harrison’s 18th Ed.. 3418 A. EBV D. serologic studies are a crucial diagnostic tool. Neoplastic meningitis B. seen in infections with HIV. sarcoid. Tuberculous meningitis D. neoplastic meningitis). Sarcoid A. VZV. Amplification of viral-specific DNA or RNA from CSF using PCR amplification is the single most important method for diagnosing CNS viral infections. with a history of exposure to house mice excreta. AJAY MATHUR Cardiology 763 B. Subacute sclerosing panencephalitis (SSPE) C. 3419 1467 Low CSF glucose concentration is seen in viral meningitis due to which of the following ? A. a lymphocytic pleocytosis with a low glucose concentration should suggest fungal. or tuberculous meningitis or noninfectious disorders (e. Listerial meningitis C. Oligoclonal bands are encountered in multiple sclerosis. Myoclonic by ? Harrison’s 18th Ed. EBV infection Harrison’s 18th Ed. orchitis. 3421 1481 A case of viral meningitis complicated by acute cerebellar ataxia A. St. VZV can produce acute cerebellar 1487 Viral “hemorrhagic” encephalitis can be seen in ? ataxia. alopecia. rash. D. Colorado tick fever virus Harrison’s 18th Ed.764 MCQ’s FOR MEDICAL PROFESSIONALS BY PROF. hand-foot-mouth disease. C. California encephalitis virus A. oophoritis. pleurodynia. hand-foot-mouth Harrison’s 18th Ed. HSV-2 infection D. parotitis. Severe EBV infection D. California encephalitis virus C. especially in males. oophoritis. Arbovirus infections typically occur in summer months. so A. Occur in summer months. herpangina. 3420 disease. Western equine encephalitis virus). Eastern equine encephalitis virus should prompt for infection with ? B. Enterovirus infection Most important viruses causing sporadic cases of encephalitis in immunocompetent adults are HSV-1. West Nile virus (WNV) B. Enterovirus infection B. LCMV infection A. AJAY MATHUR Neurology C. HSV-1 A. LCMV infection which is also seen with HSV. Western equine encephalitis virus Harrison’s 17th Ed. Colorado tick fever virus and California encephalitis virus. 1484 “Pleconaril” is efficacious against which of the following infections ? 1479 A case of viral meningitis with exanthemata. LCMV infection D. HSV-2 infection C. LCMV infection Epidemics of encephalitis are caused by arboviruses of which there are several groups like Alphaviruses (Eastern equine encephalitis virus. reflecting transmission through infected insect vectors. Any of the above A. LaCrosse virus). The presence of viral encephalitis ? orchitis. Arbovirus infection ~20% of patients with encephalitis have RBC’s (>500/µL) in CSF tap (hemorrhagic encephalitis) C. myopericarditis. Flaviviruses D. All of the above individuals with a history of exposure to house mice (Mus musculus) excreta. especially in children. 3421 1482 A case of viral meningitis along with orchitis. LaCrosse virus D. and in D. 1485 Sporadic cases of encephalitis in immunocompetent adults are 1480 A case of viral meningitis occuring in clusters during summer mostly caused by ? Harrison’s 18th Ed. Severe VZV infection LCMV infection should be considered when aseptic meningitis occurs in late fall or winter. herpangina. 3420 C. pancreatitis. in both endemic & epidemic form. 2629 A. Mumps C. 3420 D. West Nile virus (WNV) B. HSV-2 infection B. VZV CNS infection (WNV. and A. 3419 C. Powassan virus). Clinical meningitis occurs in 5% of patients with parotitis. HSV-2 infection hemorrhagic conjunctivitis suggest the diagnosis of which infection ? B. but only 50% of Harrison’s 18th Ed. Physical findings may include exanthemata. Arbovirus infection C. Enterovirus infection Viral Encephalitis Enteroviruses are the most common cause of viral meningitis. Arboviral meningitis should be considered when clusters of meningitis cases occur in a restricted geographic region during the summer or early fall. Epidemics of encephalitis are caused by arboviruses. EBV infection Pleconaril still an experimental drug has shown efficacy against a variety of enteroviral infections. Absence (petit mal) a documented history of previous infection excludes this diagnosis. 3421 should prompt for infection with ? Harrison’s 18th Ed. or myopericarditis. . and hemorrhagic conjunctivitis. VZV and EBV. HSV-2 infection D. pulmonary infiltrates. HSV-1 or -2 Virtually every possible type of focal neurologic disturbance can occur in viral encephalitis. marked CSF pleocytosis (>1000 cells/µL) and 2 and in cases of severe EBV or VZV infection. LCMV infection B. A. 3419 A. Laboratory clues to the diagnosis of LCMV include Oral or intravenous acyclovir may be of benefit in patients with meningitis caused by HSV-1 or - leukopenia. parotitis. Mumps infection confers lifelong immunity. 1486 Which of the following is a member of Flaviviruses ? Harrison’s 18th Ed. 3421 patients with meningitis have associated parotitis. B. parotitis. thrombocytopenia. Mumps infection 1488 Which of the following seizure type occurs in a case of severe Mumps meningitis occurs in late winter or early spring. myopericarditis. hypoglycorrachia (<30%). abnormal LFT. VZV meningitis should be suspected in the presence of concurrent chickenpox or shingles though up to 40% of VZV meningitis occur in the absence of rash. and Bunyaviruses (California encephalitis virus serogroup. Louis encephalitis virus C. Harrison’s 18th Ed. All of the above B. Louis encephalitis virus. Tonic-clonic (grand mal) 1483 Acyclovir therapy is of benefit in patients with meningitis caused C. LCMV infection D. St. Arbovirus infection D. or elevations in serum lipase and amylase are suggestive of the diagnosis. pleurodynia. HSV pancreatitis should prompt for infection with ? B. Occipital 1500 PCR amplification of viral nucleic acid for diagnosis of viral encephalitis caused by rabies can be done from ? . Focal findings in MRI D. HSV infection with which of the following viruses ? D. areas of hemorrhagic infarction reflect C. infection with which of the following can C. rabies. Presentation is therefore with prominent movement disorders (tremor. Rabies Harrison’s 18th Ed. Invariably negative CSF cultures Acute ascending paralysis resembling Guillain-Barre’ syndrome but associated with CSF pleocytosis C. WNV vasculopathy rather than a true encephalitis. C. CSF HSV PCR has a sensitivity of ~98%. 3421 A. Enterovirus viral encephalitis ? D. IgM present as acute ascending paralysis resembling Guillain-Barre’ D. 3423 Sensitivity is ~98% & specificity is ~94% of CSF HSV PCR. Most sensitive CSF HSV PCR can occur with HIV infection. WNV causing viral encephalitis ? D. 3421 basal ganglia & brainstem rather than cortex. Frontal D. Table 381-5 C. CSF lymphocytic pleocytosis with areas of increased T2 signal abnormality in brainstem. A. Cultures are invariably negative of the brain in viral encephalitis caused by rabies ? in cases of HSV-1 encephalitis. HIV infection 1493 Which of the following about HSV-1 encephalitis is false ? B. and brainstem rather than B. 3422 A. A. although ~90% will encountered in viral encephalitis ? have abnormalities in the temporal lobe. VZV movements & cranial nerve deficits. EBV acute poliomyelitis-like areflexic paralysis. 3422 D. hemiparesis. VZV 1491 CSF PCR is most sensitive for which of the following viruses C. 3422. WNV A. Mycoplasma Harrison’s 18th Ed. involuntary B. None of the above 1499 Abnormalities in MRI are found in which of the following regions Focal findings in encephalitis raises the possibility of HSV encephalitis. B. B. Harrison’s 18th Ed. 3423 Atypical lymphocytes in CSF may be seen in EBV infection and less commonly with other viruses A. Hypothalamus A. Enteroviruses Harrison’s 18th Ed. Ataxia cortex indicate infection with which of the following viruses ? C. Aphasia 1495 In MRI of viral encephalitis. Leptospira A. which cortical lobe of brain is most affected in HSV encephalitis ? B. D. All of the above B. and WNV infection. Parietal hippocampus & hypothalamus are seen. ataxia. Listeria B. 3421 Patients with VZV encephalitis show areas of hemorrhagic infarction pointing towards a CNS A. HSV and enteroviruses. All of the above seen in ? MRI abnormalities of patients with WNV encephalitis involve deep brain structures like thalamus. D. HSV Most common focal findings in viral encephalitis are aphasia. Patients with WNV infection can also present with A. Harrison’s 17th Ed. IgA Bartonella infection is the most common bacterial infection mimicking viral encephalitis. All of the above Harrison’s 18th Ed. HSV like CMV. B. IgE syndrome ? Demonstration of WNV intrathecally synthesised IgM antibodies is diagnostic of WNV encephalitis. abnormalities involving deep brain structures like thalamus. AJAY MATHUR Cardiology 765 1489 Which of the following focal neurological findings is commonly ~10% of patients with PCR-documented HSV encephalitis have normal MRI. 765 Neurology MCQ’s FORMCQ’s FOR MEDICAL MEDICAL PROFESSIONALS PROFESSIONALS BY PROF. 3422 D. All of the above B. Temporal In rabies. VZV Harrison’s 18th Ed. B. myoclonus) or Parkinsonism. Hippocampus Harrison’s 18th Ed. basal ganglia. Harrison’s 18th Ed. Harrison’s 18th Ed. Bartonella 1492 Presence in CSF of which class of antibody against WNV indicates WNV encephalitis ? C. Hemiparesis Harrison’s 18th Ed. Brainstem 1494 In MRI. All of the above A. 2632 They do not cross blood-brain barrier. WNV 1490 “Atypical lymphocytes” in CSF of viral encephalitis are commonly D. IgG 1498 In viral encephalitis. CMV 1496 In MRI of viral encephalitis. 3422 C. C. HSV 1497 Which of the following bacterial infection most commonly mimic C. seizures) (1%). VZV. EBV C. Hemolysis A. Biphosphate B. 21 days D. Cranial nerves VII and VIII are most frequently involved in syphilis. & EBV encode enzyme. 1503 In adults suffering from HSV encephalitis. 3425 A. D. . Foscarnet D. 3425 A. pallidum invasion of CNS ? Harrison’s 18th Ed. Candida Adults should receive a dose of 10 mg/kg of acyclovir IV 8 hourly (30 mg/kg/day) for at least 14 days. Thrombocytopenia C. and EBV that phosphorylates acyclovir to Ganciclovir & foscarnet (often used in combination) are effective in CMV-related CNS infections. Pyrimidine kinase B. GI toxicity (7%) & neurotoxicity (lethargy. Cidofovir acyclovir to produce acyclovir-5`-monophosphate. 5 days A. days. with resulting anemia. Cryptococcus neoformans D. 3`deoxyguanosine Harrison’s 18th Ed. duration of intravenous 1509 Incidence & severity of sequelae of viral encephalitis depends on acyclovir therapy should be for a minimum period of ? which of the following ? Harrison’s 18th Ed. Hemolysis. 1511 Which of the following cranial nerve is most frequently involved 1505 Which of the following is not a side effect of IV acyclovir ? in T. Foscarnet HSV. 14 days D. Elevations in BUN & creatinine B. Histoplasma capsulatum C. Monophosphate A. All of the above 1501 Enzyme in HSV. 10 days B. 3 days A. hallucinations. Amount of HSV DNA in CSF at time of presentation D. Deoxypyrimidine kinase A. CSF infections ? Harrison’s 18th Ed. Venezuelan equine encephalitis virus Infected host cell enzymes phosphorylate acyclovir-5`-monophosphate to form a triphosphate derivative which acts as an antiviral agent by inhibiting viral DNA polymerase & by causing In Eastern equine encephalitis virus infection. 3424 encephalitis is more in infections with ? Harrison’s 18th Ed. Cidofovir PCR amplification of viral nucleic acid from CSF and saliva or tears may also enable diagnosis. California encephalitis virus D. Triphosphate C. produce acyclovir-5` monophosphate is ? Cidofovir may provide an alternative in patients who fail to respond to ganciclovir and foscarnet. disorientation. Neurotoxicity D. duration of initiation of therapy. Tears B. agitation. ? Harrison’s 18th Ed. VII D. Ganciclovir C. 3425 C. that phosphorylates C. premature termination of nascent viral DNA chains. California & Venezuelan equine encephalitis. Age of patient B. Eastern equine encephalitis virus B. VZV. Saliva A. 3425 B. 3425 A.766 MCQ’s FOR MEDICAL PROFESSIONALS BY PROF. Coccidioides immitis B. 3422 1506 Which of the following drug is effective in CMV-related CNS A. 2634 A. ~80% of survivors have severe neurologic sequelae. Neonates with HSV encephalitis receive 20 mg/kg of acyclovir 8 hourly (60 mg/kg/day) for at least 21 The most common pathogen causing fungal meningitis is C. thrombocytopenia (6%). 10 days C. deoxypyrimidine (thymidine) kinase. 2`deoxyguanosine 1507 Which of the following drugs is useful for treatment of WNV encephalitis ? B. tremors. is a major side effect of IV ribavarin therapy. obtundation. neoformans. XII Complications of acyclovir therapy are elevations in BUN & creatinine levels (5%). Any of the above D. Clinical outcome following treatment correlates with amount of HSV DNA in CSF intravenous acyclovir therapy should be for a minimum period of at presentation. Level of consciousness at time of initiation of therapy C. 3424 1510 Most common pathogen causing fungal meningitis is ? Harrison’s 18th Ed. All of the above C. confusion. 14 days C. 3424 Harrison’s 17th Ed. Severe sequelae are unusual in EBV. 3424 Harrison’s 18th Ed. None of the above 1502 Which of the following metabolite of acyclovir acts as an antiviral agent by inhibiting viral DNA polymerase ? 1508 Incidence & severity of sequelae in patients surviving viral Harrison’s 18th Ed. II B. D. 7 days B. Harrison’s 18th Ed. All of the above Incidence & severity of sequelae are directly related to age of patient & level of consciousness at 1504 In neonates suffering from HSV encephalitis. AJAY MATHUR Neurology Harrison’s 18th Ed. 3424 A. III C. Ganciclovir D. CSF is acellular with markedly elevated gamma globulin level (>20% of total CSF protein).6 years D. . If there is a pellicle in CSF or a cobweb-like clot on surface of fluid. Term cerebritis refers to describe a nonencapsulated brain abscess. C. CSF A. Elevated CSF antimeasles antibody level leukoencephalopathy ? D. 3428 In PML. 1519 Which of the following is not a manifestation of subacute sclerosing 1513 In tuberculous meningitis. Isoprinosine Harrison’s 18th Ed. First tube B. which of the following is not involved ? A. Midbrain 1521 CSF finding in SSPE is ? D. EEG D. 2 . Positive CSF PCR for JCV DNA prolongs survival with clinical improvement. CSF FTA-ABS B. Elevated gamma globulin level 1515 Which of the following is false about progressive multifocal C. 3428 1514 In progressive multifocal leukoencephalopathy. 3428 A. Only known manifestation of JC virus infection present. suppurative infection within brain parenchyma. Ataxia D. multifocal areas of demyelination distributed throughout the brain but sparing spinal cord A. AFB is demonstrated in smear of clot / pellicle. Foscarnet B. CSF antimeasles antibody levels are invariably elevated & oligoclonal antimeasles antibodies are B. Occurs in immunosuppressed population In SSPE. 3428 A. Cytarabine A. Any of the above Signs of a CNS viral infection . alone or in combination with intrathecal or intraventricular -interferon C. No capsule A. Amaurosis fugax Patients of progressive multifocal leukoencephalopathy present with visual deficits. Middle tube C. Fever and headache A. All of the above Harrison’s 18th Ed. typically surrounded by a vascularized capsule. Progressive intellectual deterioration B. 767 Neurology MCQ’s FORMCQ’s FOR MEDICAL MEDICAL PROFESSIONALS PROFESSIONALS BY PROF. CSF VDRL A. Last tube D. which sample of CSF is best for panencephalitis ? detecting acid-fast bacilli (AFB) ? Harrison’s 18th Ed.4 years C. Frontal cortex C. AJAY MATHUR Cardiology 767 1512 Negative result of which of the following does not rule out 1518 Subacute sclerosing panencephalitis occurs after a latent interval neurosyphilis ? of how many years after measles infection ? Harrison’s 17th Ed. 1520 Which of the following diagnostic modalities are useful in SSPE ? Harrison’s 18th Ed. Serologic studies for JC virus 1523 Which of the following is true for the capsule of a brain abscess ? Positive CSF PCR for JCV DNA with typical MRI lesions in appropriate clinical setting is diagnostic of PML.8 years after measles infection.8 years A negative CSF VDRL does not rule out neurosyphilis. MHA-TP C. Fibrous 1517 Typical visual deficit seen in progressive multifocal B. 3427 C. Vascularized leukoencephalopathy is ? Harrison’s 18th Ed. 2634 A. Cortical blindness D. Focal and/or generalized seizures C. Acyclovir 1516 Which of the following is useful for the diagnosis of progressive multifocal leukoencephalopathy ? B. 3427 C. CT scan of brain Isoprinosine (100 mg/kg/day). MRI B. In TBM. 1 .fever & headache do not occur in SSPE. 3428 Harrison’s 17th Ed. 3428 A. EEG Harrison’s 18th Ed. 3427 A. Achromatopia D. 3427 B. Spinal cord Harrison’s 18th Ed.2 years B. typically homonymous hemianopia. B. out neurosyphilis. Harrison’s 18th Ed. Homonymous hemianopia Brain abscess is a focal. All of the above D. last tube of CSF collected at LP is the best tube to send for a smear for AFB. 2634 Harrison’s 18th Ed. All of the above C. 4 . Cerebellum D. D. Late manifestation of AIDS 1522 Which of the following drugs is used in the treatment of SSPE ? D. 6 . Acellular and optic nerves are seen. C. A negative CSF FTA-ABS or MHA-TP rules SSPE occurs after a latent interval of 6 . None of the above Harrison’s 18th Ed. Any of the above B. Streptococcus C. Should not be performed C. All of the above 1528 When should a lumbar puncture performed in patients suspected Cysticerci appearing as cystic lesions or as enhancing lesions in the brain parenchyma or in the subarachnoid space at the convexity of the cerebral hemispheres should be treated with anticysticidal of brain abscess ? therapy. D. 3429 treatment with anticysticidal drugs ? Harrison’s 17th Ed. Amebiasis D. superior sagittal sinus Headache and earache are the most frequent symptoms of transverse sinus thrombosis. On day 10 B. Superior sagittal sinus thrombosis D. Taenia solium B. As early as possible 1534 The superior sagittal sinus drains into ? Harrison’s 18th Ed. thrombosis & acute disseminated encephalomyelitis. Predilection for MCA territory C. 2638 A. 3428 B. Staphylococcus Anticonvulsant therapy is continued for at least 3 months after resolution of the brain abscess. 3428 A. Cysticerci appearing as enhancing lesions in subarachnoid D. Headache A. Multiple D. Transverse sinus C. At the junction of gray & white matter B. subdural empyema. Sigmoid sinus D. E. Bacterial meningitis A. 1527 Most common symptom of brain abscess is ? 1533 Which of the following findings on neuroimaging calls for Harrison’s 18th Ed. 3429 A. Encapsulated Most common manifestation of neurocysticercosis is new-onset partial seizures with or without secondary generalization. D. 1526 Which of the following is false about hematogenous brain 1532 Most common manifestation of neurocysticercosis is ? abscesses ? Harrison’s 17th Ed. Cavernous sinus thrombosis Conditions that can cause headache. All of the above D. Neurocysticercosis A. 3433 B. and seizure activity include ? 1535 Headache & earache are the most frequent symptoms of ? Harrison’s 18th Ed. Cavernous sinus herniation. Headache D. coli 1531 Most common parasitic disease of the CNS worldwide is ? 1525 In immunocompromised hosts the most common pathogen that Harrison’s 17th Ed. Internal jugular vein thrombosis C. New-onset generalized seizures B. Nocardia D. 2637 causes brain abscess is ? Harrison’s 18th Ed. 3429 Harrison’s 18th Ed. Transverse sinus thrombosis B. Klebsiella D. Seizure C. 1529 Conditions that can cause headache. Cysticerci appearing as cystic lesions in brain B. focal neurologic signs & seizures are brain abscess. 3429 A. 3 Harrison’s 18th Ed. 3434 A. fever. AJAY MATHUR Neurology 1524 In immunocompetent individuals the most common pathogen Harrison’s 18th Ed. 1536 Gradinego’s syndrome is due to thrombosis in which of the 1530 Anticonvulsant therapy is continued for at least how many months following intracranial sinuses ? after resolution of brain abscess ? . E. Internal jugular vein LP should not be performed in those with known or suspected focal intracranial infections (abscess / empyema) as CSF analysis contributes nothing to diagnosis or therapy. viral meningoencephalitis. 3429 that causes brain abscess is ? A. Schistosomiasis C. Focal neurologic deficit C. bacterial meningitis. On day 3 A. Toxoplasmosis B. 2637 Harrison’s 18th Ed. Focal neurologic deficit space Most common symptom in patients with a brain abscess is headache. Hematogenous abscesses are often poorly encapsulated. Superior sagittal sinus drains into the transverse sinuses which becomes sigmoid sinus before draining into internal jugular vein.768 MCQ’s FOR MEDICAL PROFESSIONALS BY PROF. 6 A. coli Neurocysticercosis is the most common parasitic disease of the CNS worldwide. Fever B. fever. 9 B. Mycobacterium tuberculosis C. Harrison’s 18th Ed. New-onset partial seizures A. Cysticerci appearing as enhancing lesions in brain C. focal neurologic signs. Acute disseminated encephalomyelitis C. & LP increases the risk of D. 12 C. Superior sagittal sinus thrombosis B. Transverse sinus thrombosis Harrison’s 18th Ed. Persistent CSF WBC count > 5/µL A. sulfonamides. Arachnoid villi 1547 Anti-Ro / SSA positivity is associated with ? C. Chronic & persistent or recurrent & discrete headache B. IV & VI cranial nerves B. Syphilis . sarcoid or CNS lymphoma. CNS lymphoma sinus ? D. Viral meningitis 1545 Which of the following is a feature of Behçet’s syndrome ? B. 3434 Uveitis is a feature of Vogt-Koyanagi-Harada syndrome. 3438 Transverse sinus thrombosis may present with otitis media. Raynaud’s phenomenon CSF is resorbed by arachnoid villi projecting into superior sagittal sinus. C. A. CNS sarcoidosis III. Internal jugular vein 1543 Uveitis is a feature of ? D. 3439. 3435 A. Improvement occurs after discontinuation of drug. Sarcoid 1537 Which of the following does not pass through the cavernous C. A. Choroid plexus B. Behçet’s syndrome 1548 Most useful empirical therapy for chronic meningitis is ? C. 769 Neurology MCQ’s FORMCQ’s FOR MEDICAL MEDICAL PROFESSIONALS PROFESSIONALS BY PROF. Aphthous oral lesions D. Ophthalmic & maxillary branches of V cranial nerve 1544 Iridocyclitis is a feature of ? C. Wegener’s granulomatosis Harrison’s 18th Ed. Congenital heart block D. Ventricular-peritoneal shunt B. Middle cerebral artery Harrison’s 18th Ed. Transverse sinus Exposure to ibuprofen. isoniazid. All of the above 1541 Recurrent meningitis is a feature of all except ? Harrison’s 18th Ed. Neurocysticercosis D. C. Systemic lupus erythematosus B. All of the above B. IV & VI and ophthalmic & maxillary branches of V cranial nerve. Mollaret’s meningitis A. Table 382-3 Congenital heart block in newborn due to damage to developing conducting system of heart results from transfer of anti-Ro antibody from mother to child. 3438 B. Mumps A. Internal carotid artery A. All of the above C. AJAY MATHUR Cardiology 769 Harrison’s 18th Ed. Sensorineural hearing loss B. 2108 D. III. Table 382-3 A. Sigmoid sinus recurrent episodes with recurrent exposure. 3440 D. Wegener’s granulomatosis D. All of the above A. genital ulcers and hypopyon suggest Behçet’s syndrome. B. sixth nerve palsy & retroorbital or A. phenazopyridine lead to B. Drug hypersensitivity Most useful empirical therapy for chronic meningitis is administration of glucocorticoids rather than antituberculous therapy. 3434 Iridocyclitis is a feature of Behçet’s syndrome. fatal levels of raised ICP can occur without enlarged ventricles. along with internal carotid artery pass through the cavernous sinus. Lyme disease C. Vogt-Koyanagi-Harada syndrome B. A. Virchow-Robin spaces are arachnoid cuffs surrounding blood vessels that penetrate brain tissue. 3438 C. Vogt-Koyanagi-Harada syndrome facial pain (Gradinego’s syndrome). All of the above Harrison’s 18th Ed. Cavernous sinus thrombosis A. Cavernous sinus Harrison’s 18th Ed. tolmetin. Virchow-Robin spaces Harrison’s 17th Ed. Behçet’s syndrome 1538 Retroorbital pain is a feature of ? Harrison’s 18th Ed. A. All of the above 1539 Chronic meningitis is diagnosed when ? Aphthous oral lesions. Antituberculous therapy B. 3438 D. Cryptococcal meningitis 1540 Which of the following is best related to resorbption of CSF ? D. 3439. ciprofloxacin. 3435 In cryptococcal meningitis. Harrison’s 18th Ed. Antifungal medication 1542 Recurrent meningitis is a feature of ? C. D. Hypopyon & pain on eye movement. 3434 D. Chronic benign lymphocytic meningitis Harrison’s 18th Ed. CNS sarcoidosis D. Glucocorticoids Harrison’s 18th Ed. Genital ulcers Headache of viral meningitis is usually frontal or retroorbital and is often associated with photophobia C. C. C. > 4 weeks duration 1546 Fatal levels of raised ICP can occur without enlarged ventricles in ? Harrison’s 18th Ed. Familial CJD (fCJD) Harrison’s 18th Ed. Short arm of chromosome 12 Prions cause scrapie in sheep & goats & Creutzfeldt-Jakob disease in humans. CSF is nearly always abnormal in CJD 1561 Pathologic hallmarks of Creutzfeldt-Jakob disease (CJD) is ? Harrison’s 18th Ed. Gerstmann-Straussler-Scheinker disease (GSS) A. PrP D 1549 Scrapie agent is synonym of ? Harrison’s 18th Ed.Prion Diseases B. PrP gene (PRNP) is located on ? C. 3441 C. Devoid of nucleic acid D. B. Prion diseases result from accumulation of PrP Sc Accidental transmission of CJD to humans can occur with corneal transplantation. PrP C D. PrP gene is designated PRNP and is located on the short arm of chromosome 20. Prions reproduce by binding to PrPC 1551 The fundamental event underlying prion diseases is ? B. 3444 Prions reproduce by binding to normal. Astrocytic gliosis Harrison’s 18th Ed. GSS disease & FFI are dominantly inherited prion diseases caused by mutations in PrP gene. 3442 Table 383-1 D. -to- structural transition in PrP n D. PrP 27-30 retains prion infectivity and polymerizes into amyloid. Sporadic disorders r 9 D. 3442 Table 383-1 D. A. CSF is nearly always normal but may show protein elevation & rarely mild pleocytosis. 3441 C. Spongiform degeneration 1555 Which of the following is the precursor of PrP ? Sc B. Implantation of duramater grafts A. genetic & sporadic disorders. Pathologic hallmarks of CJD are spongiform degeneration and (hGH) and human pituitary gonadotropin. Gerstmann-Straussler-Scheinker disease (GSS) ta Harrison’s 18th Ed. C. PrP D fCJD. A. Lack of inflammatory response . Fatal familial insomnia (FFI) 9 B. 3442 D. astrocytic gliosis D. 3445. Familial CJD (fCJD) h illness present with such wide spectrum clinical manifestations. B. PrP 27-30 is a fragment of PrPC B. 3441 U -to. duramater / pericardium graft). 3445 In CJD. 3446 C. All of the above 1559 Inherited prion disease caused by mutations in PrP gene is ? i Harrison’s 18th Ed. Prion 1556 In humans.770 MCQ’s FOR MEDICAL PROFESSIONALS BY PROF. PRNP A. PrP Sc D. -to- structural transition in PrPSC PrP 27-30 is a fragment of PrP Sc. Fatal familial insomnia (FFI) A. Genetic disorders Sporadic CJD is the most common prion disorder in humans (~85%). Myelin Harrison’s 18th Ed. 3441 Table 383-1 C. Infectious proteins In humans. 1553 Disease causing isoform of prion is ? B. PrP Sc has less  helix & high amount of  structure ti e A. Short arm of chromosome 20 G A. No other single etiology A. Short arm of chromosome 14 C. -to- structural transition in PrPSC 1558 Which is the most common prion disorder in humans ? Harrison’s 18th Ed. PrP Sd 1560 Iatrogenic CJD can be caused by ? Harrison’s 18th Ed. Amyloid PrP is the precursor of PrP Sc C B. All of the above B. cellular isoform of prion protein (PrP ) & stimulating C conversion of PrP C into disease-causing isoform (PrPSc). C. None of the above A. 3441. 3441 C. None of the above V A. Corneal transplantation 1554 Which of the following statements about prions is false ? B. AJAY MATHUR Neurology A. 3443 & Table 383-1 D. contaminated human growth hormone C. Cause degeneration of CNS R 1557 Which of the following about prions is false ? C. PrP C is rich in  helix & has little  structure d Harrison’s 18th Ed. -to- structural transition in PrP D. generated by truncation of the NH 2-terminus by limited digestion with proteinase K. Contaminated EEG electrode implantation Harrison’s 18th Ed. C. 3441 Prion diseases manifest as infectious. Infectious disorders D. Lacks nucleic acid Harrison’s 18th Ed. Sporadic CJD (sCJD) 1552 Prion diseases may manifest as ? - B. PrP 27-30 383 . PrP C C. contaminated EEG electrode implantation. Short arm of chromosome 16 1550 Which of the following about prions is false ? Harrison’s 18th Ed. structural transition in prion protein (PrP ) is the fundamental event underlying prion diseases C A. All of the above B. All of the above 1570 Which of the following is false for Hashimoto’s encephalopathy ? Harrison’s 18th Ed. D. A. Tonic-clonic seizures 1564 Which of the following about myoclonus in CJD is false ? C. Subacute progressive encephalopathy cryptococcal encephalitis. elevated ESR. 3445 C. or the myoclonic epilepsy disorder Unverricht-Lundborg disease. Gerstmann-Straussler-Scheinker disease (GSS) 1562 “Florid plaques” is characteristic feature of ? Harrison’s 18th Ed. B. Leukocytosis C. 3445 C. 3445 Dementia with myoclonus can be due to Alzheimer’s disease (AD). Harrison’s 18th Ed. PNS D. Dementia occurs in terminal phase of illness. Persists during sleep B. CJD B. 2500 Clinical abnormalities in CJD are confined to the CNS. sCJD (Creutzfeldt-Jakob disease) D. Creutzfeldt-Jakob disease (CJD) D. Harrison’s 16th Ed. 3445 B. Fever D. These are composed of a central core of PrP amyloid. NaHCo 3 A.Peripheral Neuropathy D. is elicited by loud sounds or bright lights.12 months D. None of the above 1571 Sterilization for Creutzfeldt-Jakob disease (CJD) contaminated In CJD. Familial CJD (fCJD) B. 2500 A.24 months Autoclaving at 134°C for 5 hours or treatment with 2 N NaOH for several hours is recommended for C. C. All of the above Harrison’s 16th Ed. Elicited by loud sounds Myoclonus & periodic triphasic complexes on EEG. high titers of antithyroglobulin or antithyroid peroxidase (antimicrosomal) antibodies and improvement with glucocorticoid therapy point towards C. Alzheimer’s disease (AD) D. NaOH Harrison’s 18th Ed. Familial CJD (fCJD) B. materials is done by ? Myoclonus persists during sleep. Early ataxia & visual hallucinations with a prominent psychiatric prodrome point towards vCJD. vCJD B. AJAY MATHUR Cardiology 771 D. CNS C. A. iCJD 1569 Insomnia & dysautonomia are a feature of ? Harrison’s 18th Ed. ANS 1573 Stress protein 14-3-3 is elevated in the CSF of all except ? D. sCJD B. fCJD Ataxia is a prominent & presenting feature in GSS disease. 3445 Pathologic hallmarks of CJD are spongiform degeneration.48 months sterilization of prions. D. with dementia occurring late in the disease course. leukocytosis. 24 . or pleocytosis in CSF point towards another etiology to explain 1574 Peripheral nerves are composed of which of the following elements ? patient’s CNS dysfunction. Fatal familial insomnia (FFI) A. 3446 1565 Most patients with CJD survive for what length of time after onset A. Unverricht-Lundborg disease FFI is characterized by insomnia & dysautonomia. vCJD D. All of the above C. myoclonus is neither specific nor confined to CJD and tends to occur later in the course. Gerstmann-Straussler-Scheinker disease (GSS) 1563 Dementia with myoclonus is seen in ? Harrison’s 18th Ed. Stroke B. 3445 C. Fluctuations in severity A. Pleocytosis in CSF 384 . 3445 Presence of “florid plaques” is a characteristic feature of vCJD. A. Multi-infarct dementia 1567 Which of the following is a finding in CJD ? Harrison’s 18th Ed. All of the above B. Herpes simplex virus encephalitis 1566 Clinical abnormalities in CJD involve ? Harrison’s 18th Ed. surrounded by vacuoles in a pattern suggesting petals on a flower. 771 Neurology MCQ’s FORMCQ’s FOR MEDICAL MEDICAL PROFESSIONALS PROFESSIONALS BY PROF. Hcl of clinical signs & symptoms ? B. None of the above Fever.12 months after the onset of clinical signs and symptoms. Harrison’s 18th Ed. dementia with Lewy bodies. Fatal familial insomnia (FFI) A.60 months 1572 Stress protein 14-3-3 is elevated in the CSF of ? Most patients with CJD survive for 6 . 6 . A. Periodic triphasic complexes on EEG Harrison’s 18th Ed. Multi-infarct dementia A. 12 . astrocytic gliosis & lack of inflammatory response. All of the above 1568 Ataxia is a prominent & presenting feature in ? Harrison’s 18th Ed. C. 48 . 3445 C. Elicited by bright lights Hashimoto’s encephalopathy. Herpes simplex virus encephalitis B. 3448 . AD A. 3445 D. Autonomic D. Motor neuron disease h Harrison’s 18th Ed. All of the above B. Motor C. Spinal muscular atrophy (SMA) B12. hereditary neuropathy with liability to pressure palsies (HNPP). 1575 Which of the following is not a major class of nerves ? 1581 Which of the following is a possibility in asymmetric distal weakness Harrison’s 18th Ed. vitamin E. Guillain-Barré syndrome loss with or without distal weakness ? B. Sarcoidosis D. CMV). Those without upper motor neuron findings is seen in progressive muscular B. Large myelinated plexopathy due to diabetes mellitus. Multifocal CIDP C. diabetes mellitus and other metabolic disorders. cryptogenic or idiopathic sensory polyneuropathy (CSPN). amyloidosis. without sensory loss ? Harrison’s 18th Ed. motor and autonomic elements. Diabetes mellitus B. 3451 Table 384-2 A. Large unmyelinated A. C. idiopathic - B. 25 m/sec 1582 Which of the following is a possibility in asymmetric proximal d B. multifocal motor neuropathy. 75 m/sec Harrison’s 18th Ed. AJAY MATHUR Neurology A. Hereditary neuropathy with liability to pressure palsies n 1577 Sensory fibers are of which type ? D. A. Sensory B. Any of the above weakness (UMN) without sensory loss ? 9 Harrison’s 18th Ed. large myelinated. 3448 D. amyloidosis. 3451 Table 384-2 C. infectious (leprosy. All of the above Harrison’s 18th Ed. Diabetes mellitus r B. Small myelinated C. Meningeal carcinomatosis Motor axons are usually large myelinated fibers & conduct impulses at ~ 50 m/sec. Guillain-Barré syndrome B. HNA). 3451 Table 384-2 1580 Which of the following is a possibility in symmetric distal sensory A. Chronic inflammatory demyelinating polyneuropathy (CIDP) D. toxins. multifocal C. Small unmyelinated C. cryoglobulinemia. while and distal areflexia with upper motor neuron findings ? the smaller-diameter myelinated and unmyelinated fibers transmit pain and temperature sensation. 1576 Motor axons conduct impulses at the speed of about ? hepatitis B or C.772 MCQ’s FOR MEDICAL PROFESSIONALS BY PROF. All of the above unmyelinated. 3451 Table 384-2 B. Progressive muscular atrophy A. small myelinated. meningeal carcinomatosis or lymphomatosis. hereditary plexopathy (HNPP. Small myelinated B. Multifocal motor neuropathy i 1578 Proprioception & vibratory sensation are conducted to the brain by ? C. Diabetes mellitus D. and small D. Peripheral nerves are composed of sensory. and others) are a possibility. inflammatory 1585 Which of the following is a possibility in symmetric weakness demyelinating polyneuropathy (GBS and CIDP) are a possibility. Lyme. Smaller-diameter unmyelinated fibers acquired motor axonopathy. drugs. Charcot-Marie-Tooth disease Symmetric sensory loss and distal areflexia with upper motor neuron findings is seen in Vitamin C. 100 m/sec A. Copper deficiency A. hereditary leukodystrophies (adrenomyeloneuropathy) D. tumor Harrison’s 18th Ed. All of the above 1584 Which of the following is a possibility in symmetric sensory loss Large-diameter sensory fibers conduct proprioception and vibratory sensation to the brain. 3451 Table 384-2 A. hereditary (Charcot-Marie-Tooth. D. Small unmyelinated 1583 Which of the following is a possibility in asymmetric distal D. Diabetes mellitus In the scenario of symmetric proximal and distal weakness with sensory loss. Smaller-diameter myelinated fibers atrophy. 50 m/sec and distal weakness with sensory loss ? C. 3448 infiltration V A. Harrison’s 18th Ed. HIV. vasculitis. 3448 with sensory loss with involvement of multiple nerves ? A. and copper deficiency with combined system degeneration with peripheral neuropathy. Asymmetric distal weakness with sensory loss with involvement of multiple nerves may occur in R multifocal CIDP. juvenile monomelic amyotrophy (Hirayama disease). Vitamin E deficiency Harrison’s 18th Ed. Large myelinated Harrison’s 18th Ed. Large-diameter sensory fibers ta Asymmetric distal weakness without sensory loss with upper motor neuron findings is seen in motor neuron disease. All of the above In the scenario of symmetric distal sensory loss with or without distal weakness. sarcoid. Spinal muscular atrophy (SMA) C. Spinal muscular atrophy Harrison’s 18th Ed. small myelinated or 9 small unmyelinated. 3448 U Asymmetric proximal and distal weakness with sensory loss can be seen in polyradiculopathy or A. 3451 Table 384-2 ti e D. Vitamin B12 deficiency 1579 Which of the following is a possibility in symmetric proximal and distal weakness with sensory loss ? B. Hereditary neuropathy with liability to pressure palsies G Nerves can be subdivided into three major classes: large myelinated. 3451 Table 384-2 Sensory fibers may be any of the three types namely. Chronic inflammatory demyelinating polyneuropathy (CIDP) . Absence of fibrillations C. Low-amplitude potentials B. deficiency. or sexual dysfunction. GBS B. 3452 A. prolonged distal latencies and late 1588 Majority of neuropathies are predominantly ? potentials. 3448 1594 Most common cause of sensory ganglionopathies is ? A. Lyme disease atypical CMT D. Harrison’s 18th Ed. Porphyria C. D. 3450 B. Hammer toes C. Neuromuscular junction abnormality A. Nickel Fixatives that contain zinc that can lead to copper deficiency. Diabetes mellitus B. All of the above D. 3448 A. Motor neuropathy 1592 Fixatives containing which element can lead to copper deficiency ? Harrison’s 18th Ed. conduction velocities. HIV infection. Motor Harrison’s 18th Ed. Sensory B. Sural nerve is most commonly 1590 Which of the following is the cause of combined system biopsied because it is a pure sensory nerve and biopsy will not result in loss of motor function. Common peroneal nerve C. Harrison’s 18th Ed. 3449 1596 Which of the following physical finding favours diagnosis of CMT ? Harrison’s 18th Ed. AJAY MATHUR Cardiology 773 C. 3452 C. 773 Neurology MCQ’s FORMCQ’s FOR MEDICAL MEDICAL PROFESSIONALS PROFESSIONALS BY PROF. 3449 B. vasculitis. 3448 A. Harrison’s 18th Ed. degeneration with neuropathy ? Harrison’s 18th Ed. Relatively preserved distal latencies D. along with fibrillations suggest an axonal neuropathy. copper Wasting & weakness of distal muscles of legs lead to inverted champagne bottle like appearance. 3450 A. heat intolerance or any bowel. Autonomic C. 3449 D. Guillain-Barré syndrome (GBS) Harrison’s 18th Ed. 1591 Which of the following neuropathies does not present acutely or subacutely ? . Myopathy B. 3448 Harrison’s 18th Ed. Sensory A. HIV infection C. A relapsing course is seen in CIDP and porphyria. Superficial peroneal nerve D. segmental demyelination. a motor neuropathy. suggests combined system degeneration with neuropathy as seen in vitamin B12 deficiency. High arched feet (pes cavus) D. and radiculopathies or autonomic dysfunction. All of the above D. Hammer toes & pes cavus are common & indicate onset of CMT in early life. neuromuscular junction abnormality. D. Diabetes Symmetric weakness without sensory loss with proximal and distal weakness is seen in spinal muscular atrophy and with distal weakness seen in hereditary motor neuropathy (“distal” SMA) or C. Vitamin B12 deficiency A. Inverted champagne bottle appearanceof legs B. Aluminium C. 1587 Heat intolerance is a symptom due to which of the following abnormalities ? 1593 Which of the following is not a EDx finding in axonal neuropathy ? Harrison’s 18th Ed. Autonomic B. severe hepatic disease & adrenomyeloneuropathy. Combined C. Vitamin B12 deficiency Majority of neuropathies are predominantly sensory in nature. Nerve biopsies should be done only if NCS studies are abnormal. or myopathy can be the cause. Motor neuron disease A. Diabetes mellitus Harrison’s 18th Ed. Lead D. and absence of fibrillations imply a primary demyelinating neuropathy. Diabetes mellitus D. Chronic inflammatory demyelinating polyneuropathy (CIDP) A. bladder. Zinc If the patient has only weakness without any evidence of sensory or autonomic dysfunction. Low-amplitude potentials with relatively preserved distal latencies. both acute form (AIDP) also known as Guillain-Barré syndrome (GBS) and the D. Sural nerve Symmetric proximal & distal weakness is the hallmark of acquired immune demyelinating polyneuropathies. the diagnosis can be ? related to diabetes or Lyme disease. All of the above Symmetric distal sensory neuropathy. Copper deficiency B. Relatively preserved conduction velocities Symptoms of autonomic neuropathy are fainting spells or orthostatic lightheadedness (orthostatic Nerve conduction studies (NCS) are helpful in differentiating between axonal degeneration or fall in blood pressure without an appropriate increase in heart rate). Superficial lateral cuteneous nerve chronic inflammatory demyelinating polyneuropathy (CIDP). Relatively preserved amplitudes. Any of the above C. Motor A. slow conduction velocities. Sjögren syndrome 1589 Symmetric proximal and distal weakness is the hallmark of ? Most common causes of sensory ganglionopathies presenting as severe sensory ataxia are Sjögren syndrome and paraneoplastic neuropathy. Porphyria 1586 If the patient has only weakness without any evidence of sensory Neuropathies with acute and subacute presentations include GBS. and late potentials. Charcot-Marie-Tooth disease 1595 Most commonly biopsied nerve is ? B. with symmetric upper motor neuron involvement. Contains cytoplasm B. 1606 Pelizaeus Merzbacher disease is caused by mutations in gene for ? 1600 Which of the following is the role of PMP22 ? Harrison’s 17th Ed. 2480 Harrison’s 17th Ed. congenital around the axon. Initiation of myelin spirals 1607 Which of the following is called tomaculous neuropathy ? B. Chronic distal sensory and motor neuropathy glycoprotein (MOG) while that in PNS myelin are P 0. Myelin oligodendrocyte glycoprotein (MOG) i C. caused by 1601 Which of the following is the role of P0 ? ta mutations in gene for proteolipid protein (PLP) that promotes extracellular compaction between adjacent myelin lamellae. 2662 which of the following ? Harrison’s 17th Ed. 2662 A. DéJerine-Sottas Syndrome (DSS) B. Mucopolysaccharides B. PMP22 1608 Which of the following is false about Charcot-Marie-Tooth Disease (CMT) ? C. Congenital hypomyelinating neuropathies (CHN) n C. diagnosis of CMT can be made with important for transport of metabolic substances across myelin sheath and for metabolic maintenance confidence. DéJerine-Sottas Syndrome (DSS) D. Positive family history C. 2662 A. Charcot-Marie-Tooth 2 (CMT2) axonal neuropathy D.774 MCQ’s FOR MEDICAL PROFESSIONALS BY PROF. or P0) gene can produce DSS. Regulation & growth of Schwann cells r C. Initiation of myelin spirals 9 B. Early age of onset B. Tomaculae are sausage-shaped bodies that indicate segmental Harrison’s 17th Ed. Congenital Hypomyelinating Neuropathy (CHN) C. Regulation & growth of Schwann cells Harrison’s 17th Ed. None of the above D. myelin-associated glycoprotein (MAG) and GQ1b. Maintenance of myelin sheaths A. 1598 Mutations in myelin protein zero (MPZ. P0 protein B. PMP22 and Cx32. Proteolipid protein (PLP) Harrison’s 17th Ed. 2663 C. 2480 Figure 360-1 9 Harrison’s 17th Ed. Roussy-Lévy syndrome D. P0 protein B. A. Myelin compaction B. Congenital Hypomyelinating Neuropathy (CHN) 1602 Which of the following proteins is restricted to CNS myelin ? Autosomal dominant disorder Hereditary neuropathy with liability to pressure palsies (HNPP) is also called tomaculous neuropathy. or CMT2. Noncompacted regions of myelin A. AJAY MATHUR Neurology 1597 Diagnosis of CMT is favoured by which of the following 1603 Which of the following is false about Schmidt-Lanterman incisures ? observation ? The Journal of Neuroscience 2005. Glycogen V D. If EDx findings indicate a demyelinating process. Myelin basic protein (MBP) h D. or P0) gene can produce 1604 Palpably thickened nerves in CMT1 is due to increase in ? Harrison’s 17th Ed. SLI are symptoms. Long standing symptoms D. Single oligodendrocyte ensheaths multiple CNS axons B. Myelin-associated glycoprotein (MAG) A. - hypomyelinating neuropathies (CHN) and Roussy-Lévy syndrome. and long standing membrane or noncompacted cytoplasmic regions (Schmidt-Lanterman incisures or SLI). 2480 demyelination. 3452 A. It is a protein-rich material U D. All of the above Schwann cells form myelin sheath around peripheral nerve axons as either highly compacted Diagnosis of CMT is favoured by an early age of onset. Hereditary neuropathy with liability to pressure palsies PMP22 plays a role in initiation of myelin spirals. C.25(13):3259 -3269 Harrison’s 18th Ed. control C. Proteins present in both CNS & PNS are GM1. Mutations in myelin protein zero (MPZ. d 1605 Which of the following is false about myelin ? 1599 Infantile-onset or severe childhood forms of CMT include ? ti e Harrison’s 17th Ed. Harrison’s 17th Ed. Proteolipid protein (PLP) A. CHN. 2662 A. All of the above Myelin is a lipid-rich material formed by a spiraling process of the membrane of myelinating cell Infantile-onset or severe childhood forms of CMT include Déjerine-Sottas syndrome (DSS). CMT does not reduce the life span . All of the above Pelizaeus-Merzbacher syndrome is a dysmyelinating X-linked allelic disorder of CNS. increased collagen between layers of supernumerary Schwann cells leads to palpably thickened nerves. positive family history. Maintenance of myelin sheaths D. Roussy-LéVy Syndrome of thickness. It speeds impulse conduction A. P 0 is important in myelin compaction. & longitudinal growth of sheath. D. Heritable neuromuscular disorder Protein restricted to CNS myelin are proteolipid protein (PLP) and myelin oligodendrocyte B. Hyaline G A. Each Schwann cell myelinates a single PNS axon C. All of the above In CMT1. Transport metabolic substances across myelin sheath C. stability & maintenance of myelin sheaths. Collagen R C. 2662 D. Déjerine-Sottas syndrome (DSS) B. regulation & growth of Schwann cells. 2662 A. 4:1 recessive. CMT2 is an axonal polyneuropathy D. 1:1 C. Pes cavus & hammer toes indicate that neuropathy dates from early life. X-linked Demyelinating forms of CMT are classified as CMT1. Harrison’s 18th Ed. which helps in distinguishing CMT from acquired forms of neuropathy in which sensory symptoms usually predominate. Nerve conduction velocities A. CMT4 CMT1. 2663 A. < 48 m/sec. People D. Any of the above with CMT generally do not complain of numbness or tingling. Intermediate B. 3452 polyneuropathy. CMT1 is a demyelinating polyneuropathy neuropathy. None of the above A. 775 Neurology MCQ’s FORMCQ’s FOR MEDICAL MEDICAL PROFESSIONALS PROFESSIONALS BY PROF. 3452 B. 3:1 Various subtypes of CMT are classified according to the nerve conduction velocities and predominant pathology (demyelination or axonal degeneration). 2:1 D. Inheritance pattern B. None of the above 1609 Which of the following is the most common type of hereditary Charcot–Marie–Tooth disease type 1 is most common inherited demyelinating sensorimotor neuropathy ? neuropathy. Any of the above C. CMT2 and CMT3 have autosomal dominant inheritance (with a few exceptions). Demyelinating is ? Harrison’s 18th Ed. Demyelinating B. Predominant pathology 1616 The ratio of occurrence of CMT1:CMT2 is approximately ? Harrison’s 18th Ed. Charcot-Marie-Tooth (CMT) disease B. Autosomal dominant C. Frequent falls C. Numbness or tingling A. 3452 B. D. 3452. It is the most common type of hereditary neuropathy. CMT is usually transmitted as autosomal dominant trait. 3452 A. CMT1 is the most common form of hereditary neuropathy. CMT1 C. Any of the above By definition. 1620 Which of the following manifestation prompts suspicion of CMT1 ? 1614 Which of the following about Charcot–Marie–Tooth disease is Harrison’s 18th Ed. 3454 1613 Inheritance of CMT1 is ? A. or X-linked). Harrison’s 17th Ed. 3452 B. D. Autosomal dominant C. Harrison’s 17th Ed. 3452 Harrison’s 18th Ed. Axonal C. < 68 m/sec. Autosomal recessive D. CMT3 Charcot-Marie-Tooth (CMT) disease is a syndrome of several genetically distinct disorders. Autosomal recessive D. Both are autosomal dominant. All of the above most of these are X-linked. B. Footdrop B. and the specific mutated genes. 1618 CMT2 is which form of CMT ? Harrison’s 18th Ed. Hereditary Sensory and Autonomic Neuropathy (HSAN) C. 1611 In CMT1. Type 1 is a demyelinating Harrison’s 18th Ed. It is a chronic distal sensory & motor B. < 38 m/sec. X-linked CMT1 usually present in the first to third decade of life with distal leg weakness (footdrop). D. whereas type 2 is an axonal sensory polyneuropathy. 3452 (HNPP) A. C. Hereditary Neuropathy with Liability to Pressure Palsies Harrison’s 18th Ed. inheritance pattern (autosomal dominant. C. motor conduction velocities in the arms are slowed to ? 1617 CMT1 is which form of CMT ? Harrison’s 18th Ed. Inheritance of CMT1 is autosomal dominant. 1619 Which of the following manifestation prompts suspicion of CMT1 ? Harrison’s 18th Ed. followed by type 2. Nerve conduction velocities (NCVs) intermediate between CMT1 & CMT2 are classified as “intermediate CMT” and D. < 58 m/sec. as X-linked-dominant CMT in ~10%.354:2584-92 . CMT does not reduce life span & rarely involves respiratory muscles. A. motor conduction velocities in the arms are slowed to < 38 m/sec in CMT1 and are greater than 38 m/sec in CMT2. AJAY MATHUR Cardiology 775 D. Intermediate D. All of the above C. and axonal forms as CMT2. Hereditary Neuralgic Amyotrophy (HNA) 1615 Which of the following is an autosomal recessive neuropathy ? B. 3454 false ? N Engl J Med 2006. CMT4 1610 Various subtypes of CMT are classified according to ? (genetically heterogenic) is an autosomal recessive neuropathy. A. 2663 1612 Transmission of Charcot-Marie-Tooth (CMT) neuropathy diseases A. Rare autosomal recessive forms (CMT4) have an early onset & are more severe than dominant types. Bed sore Harrison’s 18th Ed. The ratio of CMT1:CMT2 is ~2:1. CMT2 D. Axonal A. CMT1 and 2 are autosomal dominant CMT is the most common heritable neuromuscular disorder. 3452. 9 CMT1A is caused by a 1. Mutated CMT1B is due to a mutation in the myelin protein zero (MPZ. All of the above Connexins are gap junction structural proteins that are important in cell-to-cell communication. Deletion in PMP22 C. Atrophy of muscles below knee (particularly anterior compartment). ATP-binding cassette transporter 1 (ABC1) gene B. Atrophy of muscles below knee D. Mitofusin 2 (MFN2) gene C. Connexin 42 C. 3454 Harrison’s 18th Ed.776 MCQ’s FOR MEDICAL PROFESSIONALS BY PROF. i 22 gene rather than two. AJAY MATHUR Neurology A. Peroneal . whereas in HNPP. 1A subtype C. CMT type 2 B. CMT2A Harrison’s 18th Ed. Weakness in distribution of single peripheral nerves C. Connexin 48 D. Supernumerary. None of the above Most common cause of CMT2 is a mutation in the gene for mitofusin 2 (MFN2) which is localized Hereditary Neuropathy with Liability to Pressure Palsies (HNPP) is an autosomal dominant disorder. or ERG-2. Onion bulb appearance biopsy can distinguish them. In CMT1. 3455 A. Early growth response gene (ERG2) B. 1627 DéJerine-Sottas syndrome is similar to ? Harrison’s 17th Ed. Reduced sensation to all modalities B. Cabbage appearance D. that pair across adjacent cells. or P 0) gene. where it regulates the mitochondrial network architecture by CMT1A is associated with duplication in chromosome 17p11. Nerve biopsies show evidence of repeated demyelination & remyelination. to the outer mitochondrial membrane. Membranins B. 3454 B. Schwann cells attempting to remyelinate axons give a characteristic “onion bulb” appearance. Spannins A. 22 gene. gene is GJB1. 1625 CMT2 is caused by mutations for which gene ? 1631 Which of the following about HNPP is false ? Harrison’s 18th Ed. Duplication of PMP22 gene - B. CMT type 1 A. 3455 Harrison’s 18th Ed. CMT type 4 D. which are also the genes responsible for CMT1. 3455 V 1622 The most common form of CMT1 is ? A. Connexins C. Affected individuals have only one copy of PMP-22 gene B. Gap junctions provide direct neuron-neuron electrical conduction and also create openings for the r 22) gene containing chromosome 17p11. Mulberry appearance C. CMT type 3 C. Transthyretin (TTR) gene D. Myelin basic protein (MBP) Mutations in connexin 32 are responsible for X-linked form of CMT disease (CMT1X). representing 70% of cases. leads to inverted champagne bottle legs. Myelin protein zero (MPZ) d A. 1626 DéJerine-Sottas disease is also called ? 1632 Most commonly involved nerve in HNPP is ? Harrison’s 18th Ed. Potato peel appearance G DéJerine-Sottas syndrome & Congenital hypomyelinating neuropathy are indistinguishable. 3454 A. Reduced muscle stretch reflexes C. All of the above D. affected individuals have only one copy of the PMP-22 gene. 3454 B. Alpha-galactosidase gene A. 2A subtype ti e D. h 1624 Charcot-Marie-Tooth disease (CMT) type 1B is caused by mutations 1630 Mutation of which gene of gap junction protein is responsible for in gene for ? X-linked Charcot-Marie-Tooth disease ? ta Harrison’s 18th Ed. MPZ. diffusion of ions and metabolites between cells. expressed by Schwann cells are connexins. 2663 A.2-12 resulting in patients having three copies of the PMP. 3455 Harrison’s 17th Ed. Connexin 32 C. 2B subtype D. 3454 Table 384-4 1621 In CMT1. Muscle stretch reflexes are absent or reduced. All of the above C. 1628 DéJerine-Sottas syndrome is caused by mutations of ? Harrison’s 18th Ed. Myelin oligodendrocyte glycoprotein (MOG) D. Nerve D. 2663. Symptoms precipitated by trivial compression of nerve D. 3455 U Harrison’s 18th Ed. Gap junctions membrane-spanning proteins. Congenital hypomyelinating neuropathy (CHN) B. PMP22 Harrison’s 18th Ed. 1629 Gap junctions consist of membrane-spanning proteins called ? 1623 CMT 1A subtype is caused by ? Harrison’s 18th Ed. Roussy-LéVy Syndrome C. Harrison’s 18th Ed. CMT2B A. All of the above CMT3 was originally described by Dejerine and Sottas as a hereditary demyelinating sensorimotor polyneuropathy presenting in infancy or early childhood. 3455 A. reduced sensation to all modalities is apparent on examination. characteristic nerve biopsy appearance is ? A. P0 protein B.5-megabase (Mb) duplication within peripheral myelin protein-22 (PMP. Connexin 26 B.2 resulting in an extra copy of PMP- fusion of mitochondria. 2C subtype Most cases of CMT3 are caused by point mutations in the genes for PMP-22. Proteolipid protein (PLP) A. n CMT1A is the most common subtype of CMT1. concentrically R arranged. Translocation in PMP22 9 D. parietal and tubular epithelia. and brain. Mitofusin 2 (MFN2) gene A. Premature atherosclerosis D. All of the above B. HSAN 2 Angiokeratomas in Fabry’s disease are punctate. and Harrison’s 18th Ed. Mitofusin 2 (MFN2) are easily seen on electron microscopy. 3191 Roussy-LéVy syndrome is a combination of demyelinating CMT with postural and action tremor. Lumbosacral plexus D. Deafness and lenticular opacities. Harrison’s 18th Ed. Harrison’s 18th Ed. flat or slightly raised. heart. FIngers of hand A. B. Brachial plexus Fabry disease is caused by mutations in alpha-galactosidase gene that leads to accumulation of C. HSAN 3 usually symmetric. Septin 9 (SEPT9) A. Blindness 1642 Angiokeratomas in Fabry’s disease are best seen in ? D. gross foot and hand deformities. Transthyretin (TTR) HSAN1A is caused by mutations in serine palmitoyltransferase long-chain base 1 (SPTLC1) gene. HSAN 1 D. Zebra bodies 1637 Hereditary Sensory and Autonomic Neuropathy1A (HSAN1A) is B. 3455 1634 Which of the following is best related to Hereditary Neuralgic A. what else characterizes Roussy. HSAN 4 1643 Which of the following is related to Fabry’s disease ? Hereditary sensory neuropathies (HSN) are also called hereditary sensory & autonomic neuropathies (HSANs). HSAN 1 is most common. Nape of the neck Harrison’s 17th Ed. Action tremor Fabry’s disease manifests with angiokeratomas (telangiectatic skin lesions). Spina bifida Harrison’s 18th Ed. X-linked dominant disorder B. Between umbilicus and knees 1636 Riley-Day syndrome is also called ? B. bone loss. and C. Cornea verticillata A. acroparesthesia and small-vessel disease of kidney. HSAN 3 is also called Riley-Day syndrome. Refsum disease Harrison’s 18th Ed. Reddish-purple maculopapular lesions C. AJAY MATHUR Cardiology 777 B. Serine palmitoyltransferase long-chain base 1 (SPTLC1) Renal biopsy reveals small clear vacuoles containing globotriaosylceramide in glomeruli. All of the above HNPP presents as recurrent focal entrapment neuropathy (numbness & weakness) most frequently HSAN1 is associated with degeneration of small myelinated & unmyelinated nerve fibers leading in peroneal then ulnar. Mitofusin 2 (MFN2) gene C. Maltese cross caused by mutations in ? Harrison’s 18th Ed. recurrent osteomyelitis. deep dermal ulcerations. Peroneal Neuropathy A. 1644 Footdrop is a prominent feature of ? Harrison’s 18th Ed. 3455 B. dark red to blue-black. D. 3191 sensory loss in the distribution of brachial plexus. A. A. corneal B. None of the above HNA is caused by mutations in septin 9 (SEPT9). Acroparesthesia A. 1633 Hereditary Neuralgic Amyotrophy (HNA) is caused by mutations in ? 1639 Which of the following about Fabry Disease is false ? Harrison’s 18th Ed. Charcot joints. Radial C. Median D. Transthyretin (TTR) gene B. weakness. Septins may be important in formation of the 1640 Fabry disease is caused by mutations in ? neuronal cytoskeleton and have a role in cell division. ATP-binding cassette transporter 1 (ABC1) gene B. Transthyretin (TTR) gene D. 3456 1638 Which of the following is a feature of HSAN1 ? A. 777 Neurology MCQ’s FORMCQ’s FOR MEDICAL MEDICAL PROFESSIONALS PROFESSIONALS BY PROF. D. These vacuoles of electron-dense materials in parallel arrays (zebra bodies) C. Nose B. Recurrent osteomyelitis C. D. radial & median nerves. 3455 A. Angiokeratomas (telangiectatic skin lesions) 1635 Apart from demyelinating CMT. 3455 Harrison’s 18th Ed. C. Charcot joints ceramide trihexoside in nerves and blood vessels. Out of five subtypes. and amputated digits. 3455 C. 3455 C. Septin 9 (SEPT9) D. They do not blanch with pressure. They are most dense between umbilicus and knees “bathing suit area”. to severe loss of pain & temperature sensation. 2663 D. Ulnar B. hypohidrosis. Gross foot and hand deformities D. 2663 C. Urinalysis in Fabry’s disease may reveal oval fat bodies and birefringent glycolipid globules under polarized light (Maltese cross). Hyperhidrosis LéVy syndrome ? C. Deep dermal ulcerations . Deep dermal ulcerations 1641 Manifestations of Fabry disease include all except ? HNA is an autosomal dominant disorder characterized by recurrent attacks of pain. 2350 A. ATP-binding cassette transporter 1 (ABC1) gene Harrison’s 18th Ed. Corneal and lenticular opacities Harrison’s 17th Ed. Alpha-galactosidase gene Amyotrophy (HNA) ? B. Anosmia U B. Presence of retinopathy Tangier disease is caused by mutations in the ATP-binding cassette transporter 1 (ABC1) gene. CMT1 A. C. 3457 C. Diabetic lumbosacral radiculoplexus neuropathy mononeuropathies. Very low-density lipoproteins (HDL) Harrison’s 18th Ed. apolipoprotein A1. Oxalic acid 1651 Which of the following tests is useful in porphyria ? D. 2664 A. D. Refsum disease is treated by removing phytanic precursors (phytols: fish oils. B. Multiple sclerosis Harrison’s 18th Ed. slowly progressive symmetric polyneuropathy predominantly in legs. 2664 caused by mutations in the genes for transthyretin (TTR). Presence of nephropathy triacylglycerol levels are increased. 3456 D. Kidney transplantation r 9 D. cerebellar ataxia. which leads to markedly reduced levels of high-density lipoprotein (HDL) cholesterol levels while C. poorly controlled DM B. peripheral neuropathy ? Harrison’s 18th Ed. hereditary coproporphyria (HCP). All of the above Harrison’s 18th Ed. Phytanic acid Harrison’s 17th Ed. Increased urinary excretion of aminolevulinic acid Refsum disease (autosomal recessive) is caused by mutations in the gene that encodes for phytanoyl-CoA alpha-hydroxylase (PAHX). coproporphyrin G 1646 The classic tetrad of Refsum disease include all except ? D. Retinitis pigmentosa Harrison’s 18th Ed. Gelosin (FAP 4) ataxia. retinitis pigmentosa. All of the above 1649 Presentation of Tangier disease resembles those of ? Risk factors for the development of diabetic neuropathy include long-standing. Acute intermittent porphyria (AIP) D. Normal CSF protein concentration B. Variegate porphyria (VP) 1645 Refsum disease is caused by defective oxidation of ? Harrison’s 18th Ed. A. abnormal pain/temperature C. D. All of the above B. All of the above A. All of the above A. Amyotrophic lateral sclerosis A. syndactyly. shortening of the fourth toe. IVIg 1648 Severe deficiency of which of the following occurs in Tangier i Disease (TD) ? Because the liver produces much of the body’s TTR. All of the above 1647 Presentation of Refsum disease include ? Familial amyloid polyneuropathy (FAP) is phenotypically and genetically heterogeneous and is n Harrison’s 17th Ed. ichthyosis. High-density lipoproteins (HDL) 1654 Presence of which of the following is a risk factors for diabetic ta B. 3456 R 1652 Mutations in which of the following genes is responsible for A. Diabetic sensorimotor polyneuropathy (DSPN) 1650 Which of the following forms of porphyria are associated with Diabetic Distal Symmetric Sensory and Sensorimotor Polyneuropathy (DSPN) is the most common form of diabetic neuropathy. 3456 . This mutations lead to elevation of serum phytanic B. poorly controlled Harrison’s 18th Ed. 3456 A. and elevated CSF protein concentration. Transthyretin (FAP 1 & 2) d D. Cerebellar ataxia A.778 MCQ’s FOR MEDICAL PROFESSIONALS BY PROF. 3457 D. Oculomotor (III or VI nerve) neuropathy perception but preserved position/vibration in the arms. Hereditary coproporphyria (HCP) C. A. anosmia. D. Increased 24-hour fecal excretion of protoporphyrin & fats) from diet. AJAY MATHUR Neurology C. Formic acid Three forms of porphyria are associated with peripheral neuropathy . retinitis pigmentosa. Increased urinary excretion of porphobilinogen acid levels and accumulation of phytanic acid in central and peripheral nervous systems. Cerebellar ataxia Harrison’s 18th Ed. and variegate porphyria (VP).acute intermittent porphyria (AIP). Guillain-Barré syndrome 1655 Which of the following is the most common form of diabetic B. 3456 DM and presence of retinopathy & nephropathy. Low-density lipoproteins (HDL) neuropathy ? C. Diabetic truncal radiculoneuropathy (DTRN) Tangier disease is a rare autosomal recessive disorder that can present as asymmetric multiple B.. Liver transplantation deafness. dairy products. Plasmapheresis 9 Features of Refsum disease include sensorimotor demyelinating neuropathy. The majority of patients with FAP have mutations in the TTR gene.e. Long-standing. Glycolic acid C. Liver transplantation halts disease progression of FAP. Sensorineural deafness 1653 Which of the following modalities halts disease progression in familial amyloid neuropathy ? - C. 3457 D. Peripheral neuropathy familial amyloid neuropathy ? V B. and ruminant C. and a pseudo-syringomyelia pattern with dissociated sensory loss (i. Syringomyelia neuropathy ? C. h Harrison’s 18th Ed. cardiomyopathy and cataracts. sensorineural B. 3457 D. Apolipoprotein A1 (FAP 3) ti e The classic tetrad of Refsum disease include peripheral neuropathy. liver transplantation has been used to treat FAP related to TTR mutations. Triglycerides A. or gelsolin. cerebellar C. Sensory & autonomic symptoms are more prominent than motor. A. 3458 Thus. patients of diabetic radiculoplexus neuropathy present with severe pain in low back. & motor nerve involvement B. This C. All of the above Harrison’s 18th Ed. Diabetic truncal radiculoneuropathy A. Leprosy hip. Diplopia B. K +. and thigh in one leg. autonomic. 779 Neurology MCQ’s FORMCQ’s FOR MEDICAL MEDICAL PROFESSIONALS PROFESSIONALS BY PROF. Bilateral ptosis sequence of events leads to axonal loss & demyelination. 1667 Which of the following is related to insulin treatment for diabetes ? 1661 Asymmetric abrupt-onset diabetic neuropathy is ? Harrison’s 17th Ed. IV legs and into the fingers and arms. Harrison’s 17th Ed. Intense retroorbital pain Autonomic neuropathy is typically seen in combination with DSPN. All of the above A. Buttocks A. unilateral ptosis & restriction of medial gaze & upgaze. III DSPN manifests as sensory loss beginning in the toes that gradually progresses over time up the B. IV in various combinations. Pain 1666 Trigeminal neuropathy occurs in which of the following illnesses ? B.& large-fiber sensory. 2656 D. Atrophy & weakness of proximal & distal muscles in the affected leg become apparent within a few days or weeks. ATPase activity. VI 1658 Which of the following has relevance in the pathogenesis of D. Sensory loss Harrison’s 18th Ed. 2657 D. fourth nerve palsies. Bell’s palsy is common in older diabetics. Restriction of medial gaze & upgaze 1659 Diabetic autonomic neuropathy is typically seen in combination Diabetic III nerve palsy is abrupt in onset. AJAY MATHUR Cardiology 779 1656 Sensory loss in DSPN begins in ? D. Abrupt in onset DSPN could be caused by an increased neuronal glucose conversion to sorbitol by aldose reductase B. Glutamine Harrison’s 17th Ed. Small. B. Diabetic amyotrophy B. Systemic sclerosis (Scleroderma) D. Toes Harrison’s 17th Ed. and an ischemic lesion tends to involve the center of the fascicle. Diabetic lumbosacral radiculoplexus neuropathy C. a seventh nerve palsy is most common. and. double vision. Sorbitol decreases levels of myo-inositol and phosphoinositides. Cranial neuropathy . Nose diabetic lumbosacral radiculoplexus neuropathy (DLSRPN) & oculomotor (III or VI nerve) neuropathy. III DSPN is a mixed neuropathy with small. less frequently. VII diabetic sensorimotor polyneuropathy (DSPN) ? In regard to cranial mononeuropathies. protein kinase C. 3457 Asymmetric abrupt-onset diabetic neuropathies include diabetic truncal radiculoneuropathy (DTRN). Betaine palsy ? C. leading to a decrease in diacylglycerol. C. VI 1657 Which of the following nerves are involved in diabetic D. Dilated pupil D. Harrison’s 18th Ed. Intense retroorbital pain using NADPH as a coenzyme. Myo-inositol 1664 Which of the following is not a feature of diabetic third nerve B. C.6 months 1660 Initial presentation in diabetic amyotrophy or Bruns-Garland Pupil is nearly always spared in diabetic III nerve palsy as pupillomotor fibers are present on the Syndrome is ? outer layers of III nerve fascicle. Neuropathy is often accompanied by severe weight D. Fingers 1662 Which is the most frequently involved cranial nerve in diabetes ? C. Sjögren syndrome Typically. All of the above A. Muscle weakness A. heralded by intense retroorbital pain. C. All of the above loss. with ? Harrison’s 18th Ed. Polyradiculoneuropathies A. 3458 C. DSPN palsy ? B. followed by third Harrison’s 17th Ed. 2657 Harrison’s 17th Ed. A. A. B. Motor Harrison’s 18th Ed. Abrupt in onset D.and large-fiber sensory 5 months. D. 3458 C. diabetic third nerve palsies are characteristically pupil-sparing. Improve spontaneously in 3 . VII sensorimotor polyneuropathy (DSPN) ? Cranial neuropathy occurs in those over 50 years and who already have evidence of DSPN. Diabetic neuropathic cachexia Harrison’s 18th Ed. Cranial neuropathies B. 2656 nerve. 3458 D. 2656 A. Autonomic 1663 Which is the most frequently involved cranial nerve in diabetes ? C. 2656 Abducens (sixth) nerve is most often affected. 3457 1665 Which of the following is not a feature of diabetic third nerve A. sixth nerve. C. Diabetic truncal radiculoneuropathy (DTRN) B. Spontaneous recovery typically occurs within 3 - A. and Na + . polyradiculopathy (CMV-related). Sarcoidosis the initial infection ? C. 3459 1675 Lyme disease is caused by ? A. 3 or 4 months Barré syndrome in ? ti e Harrison’s 18th Ed. VI Insulin neuritis is a painful neuropathy seen with initiation of insulin treatment for diabetes. CMV infection C. Mononeuropathy C. Scalp D. 3460 V D. A. groin. Any of the above 9 C. Borrelia recurrentis C. Insulin neuritis B. III A. Polyradiculopathy - 1671 Mycobacterium leprae causes ? Harrison’s 17th Ed. of the body. is seen in patients with AIDS. Tuberculosis D. Distal symmetric polyneuropathy (DSP) B. and peroneal nerves 1668 Carpal tunnel syndrome can be a presentation of which of the are involved in leprosy. greater auricular nerve in neck. AJAY MATHUR Neurology C. 2661 A. Familial amyloid polyneuropathy (FAP) Harrison’s 18th Ed. multiple D. severe sensorimotor polyneuropathy can resemble Guillain- C. Plexopathy i mononeuropathies (vasculitis. 1674 Which of the following cranial nerves is involved in leprosy ? 1680 Anti-HIV drug not associated with neuropathy is ? Harrison’s 18th Ed. inflammatory demyelinating polyneuropathy (GBS & CIDP). Lepromatous B. following illnesses ? Harrison’s 18th Ed. Porphyria 1676 Diphtheritic neuropathy develops after how many months after R B. All of the above Harrison’s 18th Ed. Tuberculoid A. Nitrofurantoin A generalized polyneuropathy may manifest 2 or 3 months following the initial diphtheritic infection. 1 or 2 months B. D. Borderline C. All of the above C. Neuropathies are most common in DSP is the most common form of peripheral neuropathy associated with HIV infection and usually patients with borderline leprosy. Mononeuropathy multiplex D. autonomic neuropathy. Distal symmetric polyneuropathy (DSP) A. M. 3460 A. median & ulnar nerves. Groin A. Herpes B. and sensory ganglionitis. a spirochete. Inflammatory Bowel Disease n 1677 Which of the following is the presentation of nerve involvement C. Lymphoma In leprosy. Arsenic neuropathy Harrison’s 18th Ed. 2 or 3 months d 1670 Acute. 2661 Harrison’s 18th Ed. CMV-related). sensory loss does not occur in all except ? 1679 Lumbosacral polyradiculopathy in advanced HIV/AIDS is due to ? Harrison’s 17th Ed. All of the above D. h 1678 Most common form of peripheral neuropathy associated with HIV 1672 Peripheral nerves are affected in which type of leprosy ? infection is ? ta Harrison’s 18th Ed. V D. Borrelia hermsii D. B. 4 or 5 months A. sensory loss spares midline of trunk anteriorly. Autonomic neuropathy D. All of the above A. 3460 Harrison’s 17th Ed. Hypothyroidism A. axilla & scalp . Sensory ganglionitis Peripheral nerves may be affected in all three types of leprosy. Polyneuropathy Major presentations of peripheral neuropathy associated with HIV infection include distal symmetric r polyneuropathy (DSP). 1673 In leprosy. 3460 B. 3459 D.780 MCQ’s FOR MEDICAL PROFESSIONALS BY PROF. Inflammatory demyelinating polyneuropathy C. Pinnae of ears C.the warmer regions Lumbosacral polyradiculopathies are usually due to CMV infection in advanced HIV/AIDS. XI V & VII cranial nerves. Uremic neuropathy B. Borrelia burgdorferi D. 3459 Lyme disease is caused by infection with Borrelia burgdorferi. 3460 A. 3460 A. Uremic Neuropathy in HIV infection ? U D. All of the above C. 3460 Harrison’s 18th Ed. Zidovudine . leprae causes mononeuropathy multiplex affecting peripheral nerves in cooler regions of body. Borrelia turicatae 1669 Clinical presentation can mimic Guillain-Barré syndrome in ? G Harrison’s 18th Ed. Inflammatory demyelinating polyneuropathy 9 B. Axilla B. 2661 B. stomach. pyridoxamine. Motor neuropathy C. Pyridoxal kinase Harrison’s 17th Ed. Thalidomide toxicity B. A. All of the above A. Harrison’s 17th Ed. Prophylactic administration of pyridoxine 100 mg/day can prevent this neuropathy. Autonomic neuropathy . disulfiram and pyridoxine are potentially neurotoxic drugs. D. pyridoxal. 2659 A. Hu protein C. 3461 B. All of the above 1691 What dose of Pyridoxine (Vitamin B6) can cause toxicity ? 1685 Which of the following may cause optic neuropathy ? Harrison’s 18th Ed. 100 mg / day A. AJAY MATHUR Cardiology 781 B. Paclitaxel A. 3461 A. Cisplatin toxicity 1682 Paraneoplastic encephalomyelitis/sensory neuronopathy (PEM/ B. 2659 A. 2659 A. 106 mg / day B. Cyclin B1 pyridoxine deficiency and neuropathy ? Harrison’s 18th Ed. Vincristine B. All of the above At high doses of Pyridoxine (116 mg/day). Carcinoma ovary Small-fiber sensation (pain & temperature) and motor strength are spared in cisplatin toxicity. Sensory ataxia B. isoniazid. C. Carcinoma stomach 1688 “Dying back” axonal neuropathy is a feature of ? Most common malignancy causing neuropathy is lung cancer. Gephyrin 1690 Which of the following is inhibited by isoniazid (INH) resulting in D. patients can develop a severe sensory neuropathy with 1686 Which of the following is toxic to dorsal root ganglia neurons ? dysesthesias and sensory ataxia. They cause a sensory greater than motor length-dependent axonal neuropathy or neuronopathy/ganglionopathy. Pyridoxal 5'-phosphate 1684 Predominantly motor neuropathy is caused by ? C. 1687 Which of the following is not a feature of cisplatin toxicity ? dideoxyinosine & stavudine are neurotoxic and can cause a painful sensory neuropathy. A. Chloramphenicol B. lamivudine and abacavir are not associated with neuropathy. Vincristine toxicity Harrison’s 18th Ed. Harrison’s 17th Ed. Hearing loss 1681 Most common malignancy causing neuropathies is ? Harrison’s 18th Ed. Disulfiram 1683 Autoantigen found in sera / CSF of patients with paraneoplastic C. 3463 Polyclonal antineuronal antibodies (IgG) directed against Hu antigen are found in sera or CSF in the majority of patients with paraneoplastic PEM/SN. and their 5'-phosphate derivatives. Small cell lung carcinoma D. Lung cancer C. rectum. Dapsone neuropathy INH inhibits pyridoxal phosphokinase. 781 Neurology MCQ’s FORMCQ’s FOR MEDICAL MEDICAL PROFESSIONALS PROFESSIONALS BY PROF. 3461 D. Dicrease in motor strength C. Lead neuropathy D. Dideoxycytidine. Lamivudine D. All of the above Zidovudine. B. Neuropathies also complicate Harrison’s 17th Ed. Isoniazid B. Thalidomide C. producing a dose-related large-fiber sensory neuropathy (neuronopathy). 3466 B. Carcinoma breast Paclitaxel affects microtubule assembly. 2659 carcinoma of breast. 116 mg / day D. Sensory neuropathy C. ovaries. causing disruption of axonal transport and a “dying back” axonal neuropathy. Amphiphysin Nitrofurantoin. Carcinoma ovary D. Nitrous oxide D. Paclitaxel toxicity SN) usually complicates which of the following malignancies ? C. 2659 1692 Most common presentation of lead poisoning is ? A. D. Ethambutol C. Pyridoxal phosphokinase B. Cisplatin Harrison’s 18th Ed. C. colon. Carcinoma stomach 1689 Which of the following is a potentially neurotoxic drug ? Harrison’s 18th Ed. Carcinoma breast D. 3462 Harrison’s 17th Ed. Abacavir Cisplatin is toxic to dorsal root ganglia neurons. A. 3462 Paraneoplastic encephalomyelitis/sensory neuronopathy (PEM/SN) usually complicates small cell lung carcinoma. Vincristine neuropathy Pyridoxine (Vitamin B6) refers to a family of compounds that include pyridoxine. 110 mg / day C. Lhermitte’s sign A. Pyridoxine PEM/SN is ? Harrison’s 18th Ed. Pyridoxal phosphate B. resulting in pyridoxine deficiency and consequent neuropathy. D. 10 C. 9 B. Sensation is generally preserved. Motor neuropathy can also D. Mee’s lines are seen in arsenic toxicity and following thallium poisoning. All of the above Serum coproporphyrin level is elevated in lead poisoning. Mercury Harrison’s 17th Ed. Presentation of vitamin E deficiency resemble those of Friedreich’s ataxia. Index finger 1697 Drug that acts as pyridoxine (Vitamin B6) antagonist is ? C. 2501 1696 Erythrocyte transketolase activity is reduced in the blood of which A.T1 . Friedreich’s ataxia D. 1693 In lead poisoning. C5 . Isoniazid 1704 Systemic diseases associated with carpal tunnel syndrome B. Subacute combined degeneration of spinal cord C. C. Middle finger Harrison’s 17th Ed. 2660 Harrison’s 16th Ed. Varicella zoster virus (VZV) C. 2660 A. Riboflavin in ? Harrison’s 16th Ed. Painful sensory neuropathy B. 1694 Mee’s lines are related to which of the following ? Harrison’s 18th Ed. serum levels of which of the following is elevated ? 1699 Strachan’s syndrome is characterized by ? Harrison’s 18th Ed. Prolactin D. Erythrocyte transketolase activity and the percentage increase in activity (in vitro) following the addition of A. Ceruloplasmin B. Orogenital dermatitis C. the metabolites that accumulate when cobalamin- dependent reactions are blocked. Thiamine (Vitamin B1) deficiency A. 3466 Harrison’s 17th Ed. Raised levels of homocysteine C. All of the above C. Cobalamin (Vitamin B12) deficiency Most common presentation of lead poisoning is an encephalopathy. Vitamin E deficiency D. Cycloserine include ? C. B. Pellagra D. 2660 D. 3467 Harrison’s 17th Ed. Tabes dorsalis Mee’s lines are transverse lines at the base of fingernails & toenails. Coproporphyrin C. Thiamine (Vitamin B1) C. 3468 B. Hypothyroidism Isoniazid. All of the above A. Low serum cobalamin levels A. 3467 1700 Clinical features of vitamin E deficiency resemble those of ? A. Vitamin B6 deficiency is most commonly seen in patients treated with isoniazid or hydralazine. Arsenic A. Pyridoxine (Vitamin B6) deficiency B. Alpha fetoprotein A. 2660 B. Lead B. Thumb thiamine pyrophosphate (TPP) may be more accurate & reliable. Raised levels of methylmalonic acid B. 2501 D. All of the above Serum methylmalonic acid and homocysteine.782 MCQ’s FOR MEDICAL PROFESSIONALS BY PROF. 1695 Diagnosis of vitamin B12 deficiency is made by ? 1701 Which of the following damage neurons in dorsal root ganglion ? Harrison’s 18th Ed. Diabetes mellitus D. Measurement of xanthurenic acid after tryptophan loading can help confirm the diagnosis of pyridoxine (Vitamin B 6) deficiency. C6 . Encephalopathy C. Pyridoxine (Vitamin B6) D. 1702 The number of tendons in carpal tunnel is ? Harrison’s 16th Ed. 8 A. amblyopia and deafness. 7 nutritional neuropathy ? Harrison’s 18th Ed. Strachan’s syndrome is characterized by a painful sensory neuropathy associated with orogenital dermatitis. cycloserine and penicillamine act as pyridoxine antagonists by combining to aldehyde B. Rheumatoid arthritis moiety of vitamin B6. 2501 Blood & urine assays for thiamine are not reliable for diagnosis of deficiency. Autonomic nervous system can be affected. All of the above 1698 Measurement of xanthurenic acid after tryptophan loading helps in the diagnosis of ? 1705 The root value of median nerve is ? Harrison’s 17th Ed. All of the above A. nocturnal paresthesia is common D. are elevated.T1 B. Amblyopia and deafness D. Cisplatin B. 2502 A. Riboflavin deficiency occur. Penicillamine Harrison’s 16th Ed. Cobalamin (Vitamin B12) 1703 In carpal tunnel syndrome (CTS). All of the above D. AJAY MATHUR Neurology D. 2660 A. T1 A. C6 . C8. ‘Cubital tunnel’ ? Harrison’s 18th Ed. Slowing of nerve conduction velocity (NCV) B. Repair of fracture of clavicle hammer is tapped over wrist (Tinel’s sign) or wrist is flexed for 30 . 3469 Harrison’s 16th Ed. Lateral Femoral Cutaneous Neuropathy B. middle. T1 A. C8. neuralgic C. Shoulder C. Radial Neuropathy A. All of the above C. lateral. Radial nerve Harrison’s 18th Ed. (Phalen’s sign). with two divisions 1718 Example of motor neuronopathies include ? (anterior & posterior) per trunk. 783 Neurology MCQ’s FORMCQ’s FOR MEDICAL MEDICAL PROFESSIONALS PROFESSIONALS BY PROF. 3469 1715 Which of the following surgical procedures is the most common cause of brachial plexopathy ? A. Dispersion of evoked compound action potentials C. AJAY MATHUR Cardiology 783 C. C7 . Brachial Plexus 1706 The root value of ulnar nerve is ? B. 3469 1714 Out of the following. Cord Ulnar neuropathy at the elbow is called ”Cubital Tunnel Syndrome”. three trunks (upper. NeuroAIDS Brachial plexus is composed of five roots. digit. This is seen in cubital tunnel syndrome. Syphilis D. & lower). Ulnar Neuropathy at the Elbow C. Ulnar nerve A. Peroneal Neuropathy D. Harrison’s 16th Ed. Median nerve B. C7 . Three trunks B. Two divisions per trunk C. Median sternotomy In carpal tunnel syndrome (CTS). Conduction block & prolongation of distal latencies D. 3470 B. Most common pattern of IBPN involves the upper trunk or a single or multiple mononeuropathies 1709 Which of the following signs relate to diagnosis of a primarily involving the suprascapular. T1 D. Froment sign indicates thumb adductor weakness and consists of flexion of thumb at the Most common surgical procedures associated with brachial plexopathy as a complication are interphalangeal joint when attempting to oppose the thumb against lateral border of the second those that involve median sternotomies.T1 D. T1 1713 Pain in immune-mediated brachial plexus neuropathy (IBPN) is 1707 The root value of radial nerve is ? at ? Harrison’s 16th Ed. 3470 A. Radiation-Induced Plexopathy A. Leprosy B. Division Ulnar nerve passes through the condylar groove between the medial epicondyle and the olecranon. All of the above C. Sarcoidosis C. D. Repair of fracture of neck of humerus D. Trunks divide into three cords (medial. D. 1716 Which of the following is false about electrodiagnostic features of 1710 Meralgia paresthetica relates best to ? demyelination ? Harrison’s 18th Ed.T1 Immune-mediated brachial plexus neuropathy (IBPN) is also called acute brachial plexitis. Four roots A. Tinel’s sign Harrison’s 18th Ed.T1 C.T1 amyotrophy and Parsonage . C5 . 2509 . Three cords D. 2507 A. or axillary nerves. Peroneal Neuropathy B. 3470 Harrison’s 16th Ed. 2502 C. Lumbosacral Plexus Harrison’s 16th Ed.Turner syndrome. None of the above The neuropathy affecting lateral femoral cutaneous nerve is also known as meralgia paresthetica. Trunk D.T1 1712 Parsonage-Turner syndrome relates best with ? Harrison’s 18th Ed.60 sec. Froment sign B. C8. 3470 D. C5 .T1 B. C6 . which is most commonly involved in IBPN ? A. mononeuropathy ? Harrison’s 18th Ed. Root C. Hand 1708 Which of the following nerve is involved if the site of lesion is IBPN usually presents with an acute onset of severe pain in the shoulder region. C7 . Neck B. 2506 A. sensation of tingling can be reproduced when a percussion D. long thoracic. Arm D. 3470 B. & posterior). 2502 Harrison’s 18th Ed. Phalen’s sign A. Reduction of subluxated shoulder C. 1717 Mononeuropathy multiplex syndrome may be a manifestation of 1711 Which of the following is false for Brachial Plexus ? all except ? Harrison’s 18th Ed. Small-fiber neuropathies present with paresthesias A. In HIV. Porphyria C. HIV D. Sjögren’s syndrome 1720 Peripheral neuropathy includes disorders of ? Harrison’s 17th Ed. Axonal Harrison’s 17th Ed. Amyloidosis B. Lead intoxication C. All of the above C. 2653 Table 379-5 B. 1898 A. All of the above Harrison’s 17th Ed. dorsal A. 2651 B. Lead neuropathy B. dorsal or ventral nerve roots. Small-fiber sensory neuropathies 1731 Which of the following is a recurrent neuropathy ? Harrison’s 17th Ed. Cisplatin neuropathy B. All of the above Harrison’s 17th Ed. Peripheral nerves C. Toxic C. cranial nerves (except I & II) & other sensory. Porphyria 1727 Which of the following is a neuronal neuropathy ? D. B. D. 1869 Harrison’s 17th Ed. AJAY MATHUR Neurology A. Lepromatous leprosy 385 . Pyridoxine toxicity C. Guillain-Barré syndrome 1723 Term ataxic-neuropathy is used for ? D.366:2294-304 1725 Which of the following is not a large-fiber sensory neuropathy ? A. brachial or lumbosacral plexus. Diphtheria neuropathy following neuropathies ? Harrison’s 17th Ed. Refsum’s disease D. All of the above Harrison’s 17th Ed. 2651 A. Neuronal B. 2653 Table 379-3 A. Sjögren’s syndrome C. All of the above C. Guillain-Barré syndrome root ganglia. HIV neuropathy 1721 Electrodiagnostic studies (EDx) help to diagnose which of the D. Lower-motor form of amyotrophic lateral sclerosis C. All of the above Harrison’s 17th Ed. Large-fiber neuropathies present as gait disturbance B. Large-fiber sensory neuropathies A. length-dependent axonal 1722 Which of the following is false about peripheral neuropathy ? degeneration of sensory fibers without nerve-fiber regeneration occurs. None of the above C. HIV & antiretroviral therapy neuropathy D. Friedreich’s ataxia 1732 Guillain-Barré syndrome was first described in which year ? N Engl J Med 2012. 2653 Table 379-4 Peripheral neuropathy includes disorders of peripheral nerves. Demyelinating A. 2509 B. Porphyria C. 2653 Table 379-4 A. Cisplatin toxicity A. Multifocal motor neuropathy (MMN) C. 2653 Table 379-5 B. Dorsal or ventral nerve roots D.784 MCQ’s FOR MEDICAL PROFESSIONALS BY PROF. Autonomic neuropathies C. Motor-predominant neuropathies B. Pyridoxine toxicity B. Diphtheritic neuropathy A. 2651 1729 Which of the following is not a axonal neuropathy ? A. 1916 B. Fabry’s disease . autonomic or mixed nerves. Harrison’s 17th Ed. All of the above Most toxic neuropathies are distal axonal degenerations. 2653 Table 379-3 A. All of the above Harrison’s 17th Ed. Poliomyelitis D. Hereditary spinal muscular atrophies 1726 Which of the following is a motor-predominant neuropathy ? D. demyelinating or neuronal.Guillain-Barré syndrome (GBS) C. motor. 2653 Table 379-3 B. 2653 Table 379-3 A. Dapsone intoxication D. Hereditary neuropathy with pressure palsies (HNPP) 1724 Which of the following is not a small-fiber sensory neuropathy ? D. 2653 Table 379-4 B. Sensory symptoms precede motor symptoms 1730 Which of the following is a acute onset neuropathy ? Harrison’s 17th Ed. Diphtheria D. Acute intermittent porphyria 1719 Example of motor neuronopathies include ? Harrison’s 16th Ed. Dorsal root ganglia 1728 Which of the following is not a demyelinating neuropathy ? D. Charcot-Marie-Tooth 2 (CMT2) Electrodiagnostic studies help to classify neuropathy into either axonal. All of the above C. 5-fold higher risk for GBS than females. GBS with severe bulbar 1733 Miller Fisher syndrome was first reported in which year ? & facial paralysis associated with cytomegalovirus (CMV) infection & anti-GM2 antibodies & N Engl J Med 2012. All of the above onset. Miller Fisher syndrome (MFS) D. characterized by ophthalmoplegia. Acute motor sensory axonal neuropathy (AMSAN) D. 3474 B. If bladder dysfunction is a prominent & early in the course. Brazilian Males are at 1. 80 % C. Acute sensory & motor axonal neuropathy 1956 as a likely variant of Guillain–Barré syndrome. CMV 1737 In GBS. Antecedent infection or reinfection with Campylobacter jejuni. 20 % A. Acute axonal motor disorder Miller Fisher syndrome. ? Harrison’s 18th Ed. characterized by acute areflexic paralysis with albuminocytologic D. 3473 1743 Which of the following vaccinations could predispose to Guillain- A. ~70% of GBS occur 1-3 weeks after an acute infectious process (respiratory or gastrointestinal). Charles Miller Fisher was a Canadian stroke specialist. 4 weeks A. 1934 C. clinical worsening reaches a plateau in how many weeks or Epstein-Barr virus & Mycoplasma pneumoniae have been identified. Lower cranial nerves are frequently involved A. Polyradiculoneuropathy C. influenza vaccines and older-type rabies vaccine 1738 Limited or regional GBS syndromes include ? are implicated as a trigger of GBS. Limited or regional GBS syndromes are Miller Fisher syndrome. 10 % Harrison’s 18th Ed. Human herpes virus D. Fever & constitutional symptoms are absent at onset syndrome include ? Harrison’s 18th Ed. AJAY MATHUR Cardiology 785 D. Bladder dysfunction is a prominent feature C. Acute motor axonal neuropathy (AMAN) 1744 Guillain-Barre syndrome occurs more frequently in patients with B. Acute motor axonal neuropathy (AMAN) C. Older type rabies vaccine Clinical worsening stops and GBS patient reaches a plateau almost always within 4 weeks of D. A. Transient D. Miller Fisher syndrome (MFS) In Europe & North America ~95% of GBS are acute inflammatory demyelinating 1734 Feature not typical of Guillain-Barre syndrome is ? polyradiculoneuropathy. ophthalmoplegia with anti-GQ1b antibodies (severe motor-sensory GBS). 12 weeks C. Hodgkin’s disease . GBS is unlikely. ~5% are AMAN & AMSAN. All of the above dissociation (elevated protein in CSF and normal cell counts). Acute onset 1740 Nationality of Charles Miller Fisher was ? B. of onset ? Harrison’s 18th Ed. There is no increased risk of GBS with meningococcal vaccinations. 3474 A. 50 % B. 8 weeks B. human herpes virus infection. 3473 1742 Organisms involved in antecedent infections in Guillain-Barre A. 3473 B. was described in 1916. because the CSF of affected patients showed albuminocytologic dissociation. N Engl J Med 2012. 785 Neurology MCQ’s FORMCQ’s FOR MEDICAL MEDICAL PROFESSIONALS PROFESSIONALS BY PROF.366:2294-304 acute pandysautonomia. 1966 B. 3473 A. 3473 A. Acute motor sensory axonal neuropathy (AMSAN) following diseases except ? Harrison’s 18th Ed. 1923 1739 Which of the following is the most common form of GBS ? B. Acute inflammatory demyelinating polyradiculoneuropathy D. Influenza vaccine D. Harrison’s 18th Ed. Males & females are at equal risk A. 3473 1741 Which of the following is a demyelinating disorder ? A.366:2294-304 B. 2667. 3473 C. Autoimmune B. All of the above Facial diparesis is present in 50% of GBS patients. 1943 Harrison’s 18th Ed. Immunizations with swine influenza vaccine. British D. American D. Cytomegalovirus Presence of fever & constitutional symptoms at onset cast doubt on the diagnosis of GBS. 1 week Barre syndrome ? Harrison’s 18th Ed. Adults are more frequently affected than children. ataxia & areflexia. Swine influenza vaccine C. Acute motor axonal neuropathy (AMAN) and acute motor sensory axonal neuropathy (AMSAN) 1736 Which of the following is not typical of Guillain-Barre syndrome subtypes are axonal variants of Guillain-Barré syndrome (GBS). Campylobacter jejuni C. All of the above bladder dysfunction may occur in severe GBS. 1735 Facial diparesis is present in what proportion of GBS patients ? Harrison’s 18th Ed. Harrison’s 17th Ed. was reported in C. 3474. D. pure sensory forms. 1956 A. Miller Fisher syndrome (MFS) Guillain–Barré syndrome. Canadian C. DTRs disappear within first few days of onset B. Harrison’s 18th Ed. Escape of activated B cells from Peyer’s patches into A. None of the above following ? N Engl J Med 2012. ataxia. jejuni 1752 Which of the following gangliosides is strongly expressed at the B. IgG titers are highest early in the course C. Harrison’s 18th Ed. an antiganglioside antibody is common in GBS (20 . which cross-react with GT1a. Bickerstaff’s brain-stem encephalitis 1749 Which of the following is most relevant to Gangliosides ? B. Lectin D. Abducens nerve . Systemic lupus erythematosus D. leading to membrane-attack complex (MAC) A. Anti-GQ1b antibodies are found in > 90% patients formation. a critical event in the development of GBS is the escape of activated B cells from Peyer’s patches into regional lymph nodes.366:2294-304 by C. 1751 Which of the following is a central nervous system variant of 1745 Which of the following statements is false about GBS ? Miller Fisher syndrome ? Harrison’s 18th Ed. D. and pharyngeal–cervical–brachial weakness. 3474 Figure 385-1 A. 3474 N Engl J Med 2012. Bickerstaff’s brain-stem encephalitis C. Miller Fisher syndrome N Engl J Med 2012. None of the above D. Anti-Hu–Associated Paraneoplastic Encephalitis (PEM) B.50%). All of the above B. GM1 C. Bickerstaff’s brain-stem encephalitis (acute ophthalmoplegia. D. where voltage-gated 1747 Which of the following is false about Miller Fisher syndrome ? sodium (Nav) channels are localized. C. All of the above Harrison’s 18th Ed. Bickerstaff’s brain-stem encephalitis. Anti GM4 1754 The anti-GQ1b antibody syndrome includes which of the following ? Anti GM1. GD1a. Mannose Gangliosides are composed of a ceramide attached to one or more sugars (hexoses) and contain 1755 GQ1b is strongly expressed in which of the following cranial sialic acid (N-acetylneuraminic acid) linked to an oligosaccharide core. A. HIV-seropositive GM1. which can lead to nerve-conduction failure and muscle weakness. GM1 extraocular motor nerve involvement. its incomplete forms (acute ophthalmoparesis and acute ataxic neuropathy).786 MCQ’s FOR MEDICAL PROFESSIONALS BY PROF. GQ1b In the immunopathogenesis of GBS. N Engl J Med 2012. 3475 (Kv) channels are present at paranodes and juxtaparanodes respectively. C. GT1a. GT1a GBS ? D. Anti GM1 Fisher syndrome. jejuni infection. acute ophthalmoparesis. Areflexia of limbs without weakness 1753 IgG autoantibodies to GQ1b cross-react with which of the D. All of the above 1746 Which of the following is the critical event in the development of Antibodies against ganglioside GQ 1b have also been detected in patients with Bickerstaff’s brainstem encephalitis. HIV-seropositive individuals C. B. B cells recognize glycoconjugates on C. Post–infectious brainstem encephalitis D. Activation of B cells T cell nodes of Ranvier ? N Engl J Med 2012. GT1a D. Bickerstaff’s encephalitis is characterized by ophthalmoplegia and ataxia but is GBS ? also accompanied by pyramidal and sensory tract findings and cerebrospinal fluid pleocytosis. Analogous to experimental allergic neuritis (EAN) B. All of the above GBS also occurs more frequently in patients with lymphoma (Hodgkin’s disease). individuals and in patients with SLE. and its CNS variant. This results in disappearance of Nav clusters and detachment of paranodal myelin. IgG anti-GM1 or anti- GD1a autoantibodies bind to the nodal axolemma. GM1 regional lymph nodes B. particularly in those preceded N Engl J Med 2012. Gangliosides are important nerves ? components of the peripheral nerves. Anti GM2 impaired consciousness after an infectious episode). Anti GM3 D. and B. and titers of IgG are highest early in the course. 3475 Table 385-2 IgG autoantibodies to GQ1b. Oculomotor nerve N Engl J Med 2012. GT1a D. Both cellular & humoral immune mechanisms involved A.366:2294-304 Anti-GQ1b IgG antibodies are found in >90% of patients with MFS. Heparan The anti-GQ1b antibody syndrome includes the Miller Fisher syndrome.366:2294-304 C. B. AJAY MATHUR Neurology B.366:2294-304 1750 Which of the following is a Ganglioside ? A. GD1a C. All of the above Gangliosides GM1 and GD1a are strongly expressed at the nodes of Ranvier. Contactin associated protein (Caspr) & voltage-gated potassium Harrison’s 18th Ed.366:2294-304 C. Anti-GQ1b antibodies are not found in other forms of GBS unless there is A. and GQ1b are gangliosides. Ceramide acute ataxic neuropathy. Molecular mimicry mechanism C. Trochlear nerve A. GD1a 1748 Which of the following antiganglioside antibody is common in C.366:2294-304 A. Chronic myeloid leukemia B. Pharyngeal–cervical–brachial weakness A.366:2294-304 B. are strongly associated with the Miller A. All of the above C. 3476 1765 Which of the following statements about Gullain-Barre syndrome A. basis for flaccid paralysis & sensory disturbance is conduction block. 3476 Harrison’s 17th Ed. Ataxia B. Apoptosis D. C. AJAY MATHUR Cardiology 787 D. Severe proximal motor & sensory axonal damage is a poor prognostic sign 1760 What length of time after the first motor symptoms in GBS. GQ1a B. None of the above 1758 In CSF of GBS patient. Lyme polyradiculitis. GQ1d Tau & 14-3-3 protein levels are elevated early in some cases of GBS. Weakness progressively increases for > 2 months D. botulism. Areflexia of limbs without weakness CSF is often normal when symptoms have been present for 48 hours. Symmetrical weakness in proximal muscles D. West Nile virus. GQ1c D. Glucocorticoids are effective in GBS poisonings with organophosphates.352:1343-56 C. All of the above 1762 Which of the following is not found in Miller Fisher syndrome ? Harrison’s 17th Ed. Chymotrypsin C. No pleocytosis A. 3477 B. Tau A. Glucocorticoids have not been found to be effective in GBS. D. Glossopharyngeal and vagus nerves strongly express GT1a and GQ1b. often with pupillary paralysis. Weakness of limbs Harrison’s 18th Ed. Secondary axonal degeneration usually occurs in severe cases. All of the above Harrison’s 18th Ed. C. as well as muscle spindles in the limbs. CSF is normal when symptoms are present for < 6 hours B. ~2 weeks after first motor symptoms. Miller Fisher subtype of GBS CMV polyradiculitis. Anti GQ1b IgG antibodies are found in >90% of patients with Differential diagnosis of Guillain-Barré syndrome include transverse myelitis & neuromyelitis optica. By the end of first week the level of protein is usually elevated without accompanying pleocytosis. A. Hypokalaemia (GBS) is false ? Harrison’s 18th Ed. Combination of IVIg & plasmapheresis offers no advantage possibly accounting for dysphagia. and abducens nerves. Ophthalmoplegia often with pupillary paralysis suggests an alternative diagnosis. or arsenic. Ophthalmoplegia A. Apolipoprotein E D. Porphyria A. A course of A. B. 3475 IVIg is administered as five daily infusions for a total dose of 2 g/kg body weight. Tau increases in CSF may reflect axonal damage & predict a residual deficit. Conduction block plasmapheresis usually consists of 40–50 mL/kg plasma exchange (PE) four to five times over a week. 2667 In demyelinating forms of GBS. All of the above B. Not associated with impaired sensation 1761 Which of the following about treatment of GBS is false ? . 14 days characterized by all except ? N Engl J Med 2005. 2667 C. basis for flaccid paralysis & sensory III week disturbance is ? D. 1759 Differential diagnosis of Guillain-Barré syndrome includes ? Harrison’s 18th Ed. porphyria. myasthenia gravis. critical illness neuropathy or myopathy. paralytic shellfish poisoning. severe hypophosphatemia. Miller Fisher syndrome (MFS) presents as rapidly evolving ataxia & areflexia of limbs without weakness. thallium. GQ1b C. whereas GBS does not. Sustained CSF pleocytosis C. 30% of patients with GBS B. 30 days A. 3477 A. High-dose IVIg & plasmapheresis are equally effective GQ1b is strongly expressed in the oculomotor. 60 days B. Neurodegeneration require ventilatory assistance. and ophthalmoplegia. 3477 A. 787 Neurology MCQ’s FORMCQ’s FOR MEDICAL MEDICAL PROFESSIONALS PROFESSIONALS BY PROF. CIDP responds to glucocorticoids. Polymyositis C. immunotherapy is no longer effective ? Glucocorticoids have not been found to be effective in GBS. Symmetrical weakness in distal muscles Each day counts in the treatment of GBS. After successful treatment of GBS. poliomyelitis. 2667 A. MFS is strongly associated with antibodies to ganglioside GQ1b. increase in which of the following reflects 1764 MFS is strongly associated with which of the following axonal damage & predicts a residual deficit ? antibodies ? Harrison’s 18th Ed. CSF protein elevated by end of first week 1763 Which of the following is false about Miller Fisher syndrome ? Harrison’s 17th Ed. vasculitic neuropathy. None of the above Harrison’s 18th Ed. immunotherapy is no longer effective. Escape of activated B cells from Peyers patches into regional lymph nodes D. D. Rapidly evolving ataxia D. Harrison’s 18th Ed. trochlear. C. C. relapse may occur in II or 1756 In demyelinating GBS. 3 days 1766 Classic chronic inflammatory demyelinating polyneuropathy is B. Elevated CSF proteins B. 3476 D. Amyloid precursor protein (APP) B. Areflexia of limbs 1757 CSF findings in Guillain-Barre syndrome include all except ? C. Guillain-Barré syndrome C. Three weeks Controlled studies have shown that high-dose IVIg. 3478 A. methotrexate. HIV infection. HIV multiple sclerosis. and glucocorticoids are all more effective C. N Engl J Med 2005. 1 grams per kilogram over 5 days A. is present in >50% of patients. Kawasaki’s syndrome of conduction block is a certain sign of an acquired demyelinating process.345:747 C. myasthenia gravis. Motor and sensory deficits C. Reduced motor-nerve conduction velocity 1773 Intravenous immune globulin has been established to be C. 2684 B.345:747. Hypothyroidism N Engl J Med 2001.345:747 D. IgA class C. Absent F-wave latencies A. Interference with activation of complement & cytokines bowel disease and lymphoma. In addition. Drop of 20% or more in peak-to-peak amplitude A. 3478. D. PE & glucocorticoids may benefit from treatment with azathioprine. diabetes mellitus.352:1343-56 B. PE.345:747 A. and conduction block as the principal features. Chronic inflammatory demyelinating polyneuropathy of Classic chronic inflammatory demyelinating polyneuropathy ? C. and chronic demyelinating polyneuropathy. distal and temporal dispersion of CMAPs. GVHD prevention in allogeneic BM transplants loss. antibodies to gangliosides Weakness of the limbs is usually symmetric in CIDP. Five weeks 1770 Lewis-Sumner Syndrome is ? 1776 Most natural autoantibodies in adult serum are of ? Harrison’s 18th Ed. Guillain-Barré syndrome. 2 grams per kilogram over 5 days C. Multifocal Motor Neuropathy C. Two weeks D. Presence B. All of the above A. Asymmetrical presentation of symptoms Intravenous immune globulin has replaced plasma exchange as the treatment of choice in GBS D. IgM class B. IVIg is also useful in selected patients with graft-versus-host disease. Partial motor-nerve conduction block B. It is a variant of CIDP in which discrete peripheral nerves are involved. Plasma exchange (PE) N Engl J Med 2001. cyclosporine. IVIg is useful for prevention of tissue damage in certain inflammatory Harrison’s 18th Ed.366:2294-304 1771 Lewis-Sumner Syndrome is characterized by all except ? A.352:1343-56 D. D. Prolonged distal motor latencies efficacious in ? N Engl J Med 2001. Provision of antiidiotypic antibodies 1769 Which of the following treatments is effective in CIDP ? D. N Engl J Med 2005.788 MCQ’s FOR MEDICAL PROFESSIONALS BY PROF. but can be strikingly asymmetric in Lewis. and anti-CD20 (rituximab). Corticosteroid-resistant dermatomyositis Edx findings in CIDP reveal variable degrees of conduction slowing. N Engl J Med 2005.352:1343-56 N Engl J Med 2001. AJAY MATHUR Neurology 1767 Which of the following electrodiagnostic findings is not a feature B. 2 grams per kilogram over 3 days B. Elevated CSF protein content D. Immune globulin is given at a total dose of 2 grams per kilogram of body weight over a period of 5 days. amyloidosis. collagen vascular disease (SLE). prolonged distal latencies. IgG class Neuropathy B. Harrison’s 18th Ed. Multifocal Acquired Demyelinating Sensory and Motor A. Change of <15% in duration between proximal & distal site stimulation . 1778 Which of the following is true regarding definition of partial Sumner syndrome. N Engl J Med 2012. N Engl J Med 2005. SLE C. Modulation of expression & function of Fc receptors In all patients with presumptive CIDP. Four weeks than placebo.352:1343-56 1772 Intravenous immune globulin has been established to be A. inflammatory B. Distal Acquired Demyelinating Symmetric Neuropathy D. All of the above D. High-dose IVIg 1775 The half-life of infused immune globulin in immunocompetent persons is ? B. C. B.345:747 A. 3477. Evidence of axonal C. All of the above B. Motor neuron disease 1777 Which of the following about total dose of Intravenous immune Lewis-Sumner syndrome is also called multifocal acquired demyelinating sensory and motor globulin is true ? (MADSAM) neuropathy. Myasthenia gravis Harrison’s 18th Ed. All of the above 1768 Chronic inflammatory demyelinating polyneuropathy may be IVIg blocks reticuloendothelial cell function and immune complex clearance in immune associated with all of the following except ? thrombocytopenia. 3477. presumably secondary to demyelination. cyclophosphamide. Poor response to intravenous immune globulin in presence of patients who are not able to walk unaided. it is also reasonable to exclude vasculitis. All of the above Harrison’s 18th Ed. A. 3477 syndromes such as Kawasaki disease and as Ig replacement therapy for certain types of immunoglobulin deficiencies. chronic hepatitis. Diabetes mellitus 1774 The mode of action of immune globulin involves ? D. 3477. Patients who fail therapy with IVIg. 1 grams per kilogram over 3 days Harrison’s 18th Ed. nerve conduction block ? Harrison’s 18th Ed. Drop of 20% or more in negative peak area efficacious in ? N Engl J Med 2001. Glucocorticoids A. associated with sites of persistent B. 1-beta. MG is a neuromuscular disorder characterized by weakness & fatigability of skeletal muscles. 3480 B. Magnesium 386 . 1-beta. Decrease in their size D. Potassium Most patients with Multifocal motor neuropathy (MMN) respond to high-dose IVIg. 1delta or 1epsilon B. All of the above In MG. Acetylcholine receptor consists of five subunits . 1-alpha. Sodium B. ACh is released normally Harrison’s 18th Ed. and 1-gamma or epsilon arranged around a central pore. 3480 A. Immune globulin therapy A. Plasma exchange (PE) cation gains rapid entry ? C. & widened synaptic space. Plasma exchange (PE) Harrison’s 16th Ed. 1-delta.352:1343-56 D. 2-alpha. 1-beta. 3480 MG ? A. 1-alpha. B. reduced number of AChRs (stippling). 789 Neurology MCQ’s FORMCQ’s FOR MEDICAL MEDICAL PROFESSIONALS PROFESSIONALS BY PROF. Oxidation A. Relative sparing of sensory fibres C. Harrison’s 18th Ed. Refractory patients may respond to rituximab or cyclophosphamide. Hydrolysis C. Normal nerve terminal . 3478. C. 2-delta. Cardiac muscle D.352:1343-56 A. 1-alpha. 3480 D. Methotrexate 2beta. N Engl J Med 2005. Flattened. Symmetric weakness In MG. C. 1gamma. Calcium D. Phosphorylation D. 1-delta. and 1-gamma or epsilon 1780 Which of the following is effective in the treatment of Multifocal Motor Neuropathy ? Structure of AChR consists of five subunits . Smooth muscle C. A partial motor-conduction block at multiple sites Harrison’s 18th Ed. underlying pathogenetic defect is decrease in number of available AChRs at neuromuscular junctions due to antibody-mediated autoimmune attack. 1-delta. simplified postsynaptic folds. Increased presynaptic rundown B. Pathologic antibodies are IgG & are T cell dependent 1781 Which of the following is effective in the treatment of Multifocal C. chiefly sodium. Cyclophosphamide Harrison’s 18th Ed. 1-delta. 3480 acetylcholine receptors (AChRs) is ? A. which of the following B. Acetylcholine Harrison’s 18th Ed. N Engl J Med 2005. Sensory fibers are relatively spared. Reduction B.1alpha. N Engl J Med 2005. None of the above A. 1-beta. Skeletal muscle B. 3480 Figure 386-1 B. 2518 C. flattened. D. Acetylcholine receptor (AChR) A.2-alpha. 3478. Alteration in their shape Amount of ACh released per impulse normally declines on repeated activity (presynaptic rundown). AJAY MATHUR Cardiology 789 D. and 1-gamma or epsilon focal motor conduction block in the same nerve trunks. and 1-gamma or epsilon Multifocal motor neuropathy (MMN) presents as slowly progressive motor weakness & atrophy evolving over years in the distribution of selected nerve trunks. and >75% of all patients are male.352:1343-56 D. simplified postsynaptic folds C. the Neuromuscular Junction 1788 Acetylcholinesterase (AChE) rapidly terminates the action of ACh by ? 1782 Myasthenia gravis (MG) is a neuromuscular disorder involving ? Harrison’s 18th Ed. D. 1790 In MG. Widened synaptic space 1779 Which of the is false about Multifocal Motor Neuropathy ? Harrison’s 18th Ed. In MG. 3480 Harrison’s 18th Ed. 3478. Glucocorticoids and PE are not effective. Methotrexate A. Corticosteroids 1786 Which of the following about Myasthenia gravis is false ? B. All of the above B. Absence of anti AChR antibodies does not exclude diagnosis Motor Neuropathy ? Harrison’s 18th Ed. Decrease in number of available ACh receptors A. and 2-gamma or epsilon D. the autoimmune antibody is targetted against ? 1784 Which of the following is false about neuromuscular junction in Harrison’s 18th Ed.Myasthenia Gravis and Diseases of ACh combines with binding sites on alpha subunits of AChR to open the channel permitting rapid entry of cations. Yhymus is abnormal in ~75% of patients with MG. Mycophenolate mofetil is useful A. 1789 Which of the following about myasthenia gravis is false ? 1783 At neuromuscular junctions in myasthenia gravis. this normal rundown along with decreased efficiency of NM transmission results in weakness or myasthenic fatigue. the defect in Harrison’s 18th Ed. NM junction has a normal nerve terminal. All of the above Acetylcholinesterase (AChE) rapidly terminates the action of acetylcholine (ACh) by hydrolysis. 3480 A. Thymus abnormal in ~ 75 % of patients C. Starts in distal arm muscles 1785 Structure of AChR consists of ? D. The arms are affected more frequently than the legs. Corticosteroids 1787 With the opening of the channel in AChR. Decrease in their number C. 2-beta. Optic nerve glioma A. Hyperthyroidism B. Pure red cell aplasia & extraocular muscles are involved early in MG. as both lung recoil & passive chest wall recoil are normal. reduced strength due to impaired neuromuscular transmission results in type II respiratory failure i. Atkin D. Caught between the low TLC and the elevated RV. as respiratory muscle strength is insufficient. 50 % D. D. alveolar hypoventilation and inability to eliminate carbon dioxide C. in a ratio of ~3:2. All of the above is false ? Harrison’s 18th Ed. both airways resistance (R aw) and DL CO are normal. None of the above B. the pathogenic antibodies are IgG & are T cell dependent. Harrison’s 18th Ed. Type IV B. Limb weakness is proximal and asymmetric 1794 Thymoma along with which of the following is called Good’s D. 426 C. 387 1793 Thymus is abnormal in what proportion of patients with MG ? A. 2200 turnover of AChRs by its rapid endocytosis. 100 % Harrison’s 18th Ed. D. anti-AChR antibodies reduce the number of available AChRs B. 25 % C. IgE 1799 Myasthenia gravis is related to which of the following ? In MG. 3480 C. Diplopia is intermittent & variable Harrison’s 18th Ed. which of the following spirometric statement D. C. Endocytosis of the receptors In MG. Chap. FVC and FEV1 are reduced as “innocent bystanders. Bullous Pemphigoid . particularly lids A. Deep tendon reflexes diminished syndrome ? Women are affected more frequently than men. Pupils are always normal 1796 Thymoma and / or myasthenia gravis is associated with which of the following ? B. Myasthenia gravis Thymoma with hypogammaglobulinemia also is called Good’s syndrome. C. 237 D. specific anti-AChR autoimmune antibody is targetted against Acetylcholine receptor (AChR). 3480 D. Orbital cellulitis B. Tomaszek gravis ? The staging system for thymoma was developed by Masaoka and colleagues. thymus is “hyperplastic” though not A. B. may serve as a source of autoantigen B. Blockade of the active site of AChR and RV is elevated.790 MCQ’s FOR MEDICAL PROFESSIONALS BY PROF. Hypogammaglobulinemia C. Harrison’s 18th Ed. Limb weakness is proximal & asymmetric. Diplopia & ptosis are common initial complaints. C. Ulcerative colitis A. TLC is low B. 238 A. Cranial muscles. Type II 1792 Pathogenic anti-AChR antibody in myasthenia gravis is ? Harrison’s 18th Ed. Fluctuating ptosis may be present.” As airway size and the lung C. Lids & extraocular muscles involved early and trigger the autoimmune reaction within thymus gland. 2093 In MG. Normal FRC 1791 In MG. and damage to postsynaptic muscle membrane by the antibody + complement. Pemphigus Vulgaris D. Paraneoplastic Pemphigus D. In 65%. All of the above C. 75 % 1800 Which of the following about myasthenia gravis is false ? D. M component on electrophoretic analysis A. 3480 B. Deep tendon reflexes are preserved. variable. Elevated RV Harrison’s 18th Ed. Falkson Fluctuating ptosis that worsens late in the day is typical of myasthenia gravis. which bear AChRs on their surface. FRC remains normal. Masaoka 1802 Which of the following is an eye related finding in myasthenia D. Damage to postsynaptic muscle membrane vasculature are unaffected. Neuromyelitis optica (NMO) Harrison’s 18th Ed. 3480 Thymus is abnormal in 75% of patients with MG. Normal FVC A. IgG In myasthenia gravis. Pupils are always normal. Acetylcholinesterase (AChE) 1797 In myasthenia gravis. Type III A. Systemic lupus erythematosus 1801 Fluctuating ptosis that worsens late in the day is typical of ? Harrison’s 18th Ed. Glaucoma 1795 Staging system for thymomas is named after ? C. Pemphigus Foliaceus MG is a major cause of diplopia which is intermittent.e. 10% of patients have thymic tumors (thymomas). A. IgA D. blockade of the active binding site of AChR. Muscle-like cells within the thymus (myoid cells). Low TLC at NM junctions by which mechanism ? C. All of the above 1798 Type of respiratory failure in MG is usually ? Anti-AChR antibodies reduce the number of available AChRs at NM junctions by accelerated Harrison’s 18th Ed. IgM effectively. and not confined to any single ocular motor nerve distribution. Women more affected than men necessarily enlarged. Type I B. B. A. AJAY MATHUR Neurology C. Diplopia is not confined to any single ocular motor nerve A. 24 hours before testing. 3481 Harrison’s 18th Ed. ~ 45 % C. > 30 . ~ 99 % in repetitive nerve stimulation for evidence of MG. No HMF neurologic defect D. Proximal muscle groups myasthenic patients ? Harrison’s 18th Ed. if muscle weakness remains restricted to extraocular muscles for 3 years (ocular MG). Antibodies to muscle-specific tyrosine kinase (MuSK) are present in ~40% of AChR antibody- 1813 It is best to test which of the following muscles in repetitive nerve negative patients with generalized MG. All of the above A. Muscle-specific tyrosine kinase A. ~ 40 % hyperkalemia. 3481 Harrison’s 18th Ed. electric shocks are delivered at a rate of two . 3481 A.45 % C. but a negative test does not exclude MG. Membrane-specific tyrosine kinase C. Multiple sclerosis). Not present in MG limited to ocular muscles D. None of the above D. ~ 10 % Causes of episodic generalized weakness include Electrolyte disturbances (hypokalemia. hypernatremia. metabolic defects of muscle). 1804 MG muscle weakness will not become generalized if it remains 1810 Anti MuSK antibodies are present in what percentage of AChR restricted to extraocular muscles for ? antibody-positive MG patients ? Harrison’s 18th Ed. None of the above MuSK antibodies are rarely present in AChR antibody-positive MG patients or in patients with MG MG is characterized by typical distribution of skeletal muscle weakness & fatigability. 185 Table 22-2 patients. Lambert-Eaton myasthenic syndrome antibody-negative patients with generalized MG ? C. Lambert-Eaton myasthenic syndrome). ~ 80 % Neuromuscular junction disorders (Myasthenia gravis. 50 % C. B. it is unlikely to become generalized. 3481 A. 791 Neurology MCQ’s FORMCQ’s FOR MEDICAL MEDICAL PROFESSIONALS PROFESSIONALS BY PROF. 3 years D. 25 % C. ~ 85 % It is best to test weak muscles or proximal muscle groups in repetitive nerve stimulation for evidence of MG. 1811 Anti MuSK antibodies are present in what percentage of patients with MG limited to ocular muscles ? 1805 Which of the following is false about MG ? Harrison’s 18th Ed. Multiple sclerosis Harrison’s 18th Ed. ~ 50 % C. 80 % D. None of the above Presence of anti-AChR antibodies is diagnostic of MG.75 % D.30 % B. C. hypercalcemia. Muscle disorders (periodic paralyses. 50 % D. 1812 Which of the following statements about antibodies to muscle- 1806 In seronegative MG. No impairment of sensation C. Myasthenia gravis 1809 Anti-MuSK antibodies are found in what percentage of AChR B. Interosseous muscles B. 3481 Harrison’s 18th Ed. Not present in AChR antibody-positive patients C. ~ 100 % Central nervous system disorders (Transient ischemic attacks of the brainstem. hypermagnesemia). A. Rare B. Rare A. > 50 . loss of reflexes or impairment of sensation or other neurologic function. ~ 65 % D. 3481 Harrison’s 18th Ed. ~ 25 % B. > 20 . No loss of reflexes B. ~ 75 % D. 25 % B. AJAY MATHUR Cardiology 791 1803 Cause of episodic generalized weakness is ? Anti-AChR antibodies are detectable in ~85% of all MG patients & in only ~50% of ocular MG Harrison’s 18th Ed. 2 years C. Anti-AChE medication must be stopped 6 . 1 year B. ~ 99 % 1814 In myasthenic patients. hypophosphatemia. rapid reduction of what proportion in 1808 Anti-AChR antibodies are detectable in what percentage of ocular amplitude of evoked responses is noted ? MG patients ? Harrison’s 18th Ed. 3481 D. without limited to ocular muscles. Found in ~40% of AChR antibody-negative generalized MG B. 3481 A. antibodies can be found against ? specific kinase (MuSK) is false ? Harrison’s 18th Ed. 6 months A. 3481 1807 Anti-AChR antibodies are detectable in what percentage of all A. Distal muscle groups A.15 % A. Transient global cerebral ischemia. 80 % In MG. D. D. stimulation for evidence of MG ? Harrison’s 18th Ed. Any of the above C. 3481 B. ~40% of AChR antibody-negative patients with generalized MG have anti-MuSK antibodies. 3481 A. Myasthenia-specific tyrosine kinase B. > 10 . hyponatremia. ready for IV use if nausea. CMS should be suspected when symptoms of myasthenia begin in infancy or childhood & AChR antibody tests are consistently negative. Proximal muscles of lower limbs most commonly affected Harrison’s 18th Ed. Penicillamine B. 3481 C. Presynaptic nerve terminal D. Autoantibodies against P/Q type calcium channels 1819 Which of the following statements about congenital myasthenic syndromes (CMS) is false ? B. 3. Amyotrophic lateral sclerosis history of autoimmune disorder. AJAY MATHUR Neurology or three per second to the appropriate nerves. asthma. Graves’ disease. Nondepolarizing muscle relaxants (D-Tubocurarine. D. CMS that involve AChR. quinidine.4 Diaminopyridine (3. Diazepam B.4-DAP) helpful B.15% in the amplitude of the evoked responses. Penicillamine Anticholinesterase Test ? B. Alterations in function of presynaptic nerve terminal or in various subunits syndrome (LEMS) ? of the AChR or AChE have been identified in different forms of CMS. Hyperthyroidism or hypothyroidism. Creutzfeldt-Jakob disease (CJD) 1823 Which of the following is false about Lambert-Eaton myasthenic False-positive Edrophonium anticholinesterase test occur in occasional patients with amyotrophic syndrome (LEMS) ? lateral sclerosis. 3481 Disorders associated with Myasthenia Gravis are thymoma. atracurium). diabetes. Hodgkin lymphoma ulcer. Lambert-Eaton myasthenic syndrome (LEMS) . 1825 Muscle testing reveals “jerky release” or “give-away weakness” in which of the following ? 1820 Drugs that can exacerbate weakness in myasthenic patients are all Harrison’s 18th Ed. pancuronium. Treatment of LEMS involves plasmapheresis & D. Quinolones (ciprofloxacin. family A. hypertension. C. vecuronium. peptic B. Incremental responses on repetitive nerve stimulation B. Atropine (0. Diazepam A. Aminoglycoside antibiotics Harrison’s 18th Ed. 1821 Myasthenia gravis may be “induced” by ? fasciculations. thymus hyperplasia. Procainamide Edrophonium anticholinesterase test ? Harrison’s 18th Ed. experience autonomic changes such as dry mouth and impotence. diarrhea. Aminoglycoside antibiotics 1815 Which of the following drug must be kept at hand while conducting C. Patients with LEMS have depressed or C. In Harrison’s 18th Ed. A. kanamycin). medical treatment for other conditions may exacerbate myasthenia gravis. Ptosis & diplopia occur in ~70% of patients.  subunit of AChR is affected in ~75% of cases immunosuppression. Local anesthetics (Procaine. Normal deep tendon reflexes A. 3481 D. Procainamide A. Symptoms begin in infancy or childhood LEMS is caused by autoantibodies directed against P/Q type calcium channels at the motor nerve terminals impairing release of ACh & are detected in ~85% of LEMS patients. All of the above Edrophonium anticholinesterase test ? Harrison’s 18th Ed. Hashimoto’s thyroiditis A longer acting drug like neostigmine (orally) may be used for anticholinesterase test. Adrenaline Xylocaine). Procainamide. Neostigmine azithromycin). and have D. syncope or bradycardia develop following IV edrophonium. Botulinum toxin. 3482 except ? A. Small cell lung C. Plasmapheresis ineffective A. 3481 C. 3483 Table 386-3 D.4-Diaminopyridine (3. levofloxacin. Various subunits of AChE absent reflexes. Graves’ disease C. Harrison’s 18th Ed. lupus erythematosus. occult infection. Various subunits of AChR LEMS is a presynaptic disorder of NM junction. ofloxacin. Magnesium. Penicillamine.6 mg) should be drawn up in a syringe. chloroquine. Atropine D. osteoporosis. All of the above A. Atropine Drugs that may exacerbate MG include aminoglycosides (streptomycin. Harrison’s 18th Ed. and action potentials are recorded from muscles. Associated small-cell carcinoma of lung Harrison’s 18th Ed. tobramycin. Adrenaline 1822 Disorders associated with myasthenia gravis include ? Harrison’s 18th Ed. 3482 A. 3. and in placebo-reactors. Systemic lupus erythematosus 1817 Which of the following diseases can cause false-positive D. AChR antibody tests are consistently negative carcinoma is common in LEMS. gatifloxacin). 3481 C. Beta-blockers (Propranolol.4-DAP) & pyridostigmine. Tuberculosis. obesity are disorders that may interfere with therapy. rheumatoid arthritis. Table 387-11 1816 Apart from Edrophonium which drug can be used for A. as it permits more time for detailed evaluation of strength. B. Harrison’s 18th Ed. C. Quinine derivatives (Quinine. Neostigmine C. macrolides (erythromycin. Charcot-Marie-Tooth disease D. All of the above mefloquine). gastrointestinal bleeding. B. 3486 Table 386-4 myasthenic patients there is a rapid reduction of > 10 . renal disease. metoprolol).792 MCQ’s FOR MEDICAL PROFESSIONALS BY PROF. salivation.  subunit is affected in ~75%. Hashimoto’s thyroiditis. Congenital myasthenic syndromes (CMS) are disorders of neuromuscular junction that are not 1824 Which of the following is false about Lambert-Eaton myasthenic autoimmune but are due to genetic mutations in which virtually any component of the neuromuscular junction may be affected. Due to genetic mutations D. Proximal muscles of lower limbs are most commonly affected. All of the following incremental responses on repetitive nerve stimulation. 3482 A. Presynaptic disorder of neuromuscular junction 1818 Which of the following component of the neuromuscular junction may be affected in congenital myasthenic syndromes (CMS) ? B. atenolol. skin disorders. Cyclosporine C. 3484 1831 Which of the following produce clinical improvement within a A. Intravenous immunoglobulin (IVIg) D. D. Cephalosporins A. but thymus enlargement in a patient >40 years is highly suspicious of thymoma. if immediate improvement is essential. 3484 generalized MG between ? Harrison’s 18th Ed. cyclosporine or tacrolimus produce 1826 Which of the following cause weeakness of somatic musculature clinical improvement within a period of 1. because C. salivation 1834 Which of the following is used in MG patients refractory to optimal and nausea. Botulism Harrison’s 18th Ed. Cyclophosphamide presynaptic nerve terminal. Terfenadine B. 7 . Allopurinol Harrison’s 18th Ed. high-dose cyclophosphamide may induce long-lasting benefit by “rebooting” the immune system. High-dose cyclophosphamide In patients taking azathioprine. 3484 C. Tacrolimus A. > 10 years B. Mycophenolate mofetil C. Thymic shadow on CT is normal in young adulthood. Neurasthenia B. Glucocorticoids due to a common degradation pathway. 1833 Beneficial effect of azathioprine takes how long to appear ? 1827 Muscarinic side effects of anticholinesterase medication include Harrison’s 18th Ed. glucocorticoids. Hyperthyroidism A. Abdominal cramps C. All of the above For the intermediate term goal of improvement. abdominal cramps. Nausea Beneficial effects of azathioprine & mycophenolate mofetil begin after many months (up to a year). severe bone marrow depression may occur. All of the above B. allopurinol should never be used to treat hyperuricemia. 1835 Drug of choice for long-term treatment of myasthenic patients is ? 1829 By consensus. 1 . 18 months D. 3482 1832 Which of the following immunosuppressive agents is preferred A. Glucocorticoids D.3 months. 3483 A. Lambert-Eaton myasthenic syndrome (LEMS) (toxicity & cost) in treatment of MG ? B. Paracetamol 1830 If immediate improvement is essential. Tacrolimus Muscle testing reveals “jerky release” or “give-away weakness” characteristic of nonorganic disorders like Neurasthenia . 3 to 6 months C. > 40 years Patient with MG refractory to optimal treatment with conventional immunosuppressive agents. due to presynaptic neuromuscular junction abnormality ? Harrison’s 18th Ed. > 20 years C. High-dose cyclophosphamide B. Azathioprine D. 3484 all except ? A. 40 and 65 years D. 3484 C. Mycophenolate mofetil B. which out of the following drugs It is the consensus that thymectomy should be carried out in all patients with generalized MG who should not be used ? are between puberty and at least 55 years of age. Muscarinic side effects of anticholinesterase medication are diarrhea. Botulism Harrison’s 18th Ed. Tacrolimus D. Intravenous immunoglobulin (IVIg) 1837 Which of the following may be used as indications of adequacy of In MG.3 months ? . thymectomy should be done in all patients with Harrison’s 18th Ed. Congenital myasthenic syndromes (CMS) A. Cyclosporine C. Thymectomy D.a myasthenia-like fatigue syndrome without an organic basis. treatment with immunosuppressive agents ? Harrison’s 18th Ed. 3484 A. 30 and 55 years C. 50 and 75 years 1836 In patients taking azathioprine. 3484 1828 Thymoma is suspected if thymus is enlarged after ? Harrison’s 18th Ed. 793 Neurology MCQ’s FORMCQ’s FOR MEDICAL MEDICAL PROFESSIONALS PROFESSIONALS BY PROF. D. bacterial toxin interferes with release of acetylcholine from D. Azathioprine A. Harrison’s 18th Ed. Diarrhea B. Cyclosporine C. Dry mouth D. Increase in hemoglobin period of 1 . Puberty and 55 years B. > 30 years D. IVIg is given or plasmapheresis instituted. azathioprine dosage ? Harrison’s 18th Ed. AJAY MATHUR Cardiology 793 B. 3485 A.3 months B. 3484 C. which of the following is used to manage MG ? B. 3483 A.21 days Harrison’s 18th Ed. Mycophenolate mofetil CMS are not due to autoimmune but due to genetic mutations presynaptic nerve terminal or in subunits of AChR or AChE. In Botulism. 3489 A. Statins 1841 Clinical findings of a myopathy include all except ? Harrison’s 18th Ed. D. amyotrophic lateral The most common cause of myasthenic crisis is intercurrent infection. All of the above D. Hyperparathyroidism C. and some forms of inclusion body myopathy. Hyperthyroidism B. Paramyotonia congenita A. 3490 Table 387-3 387 .Muscular Dystrophies A. l-Tryptophan. Emetine. hyperparathyroidism. Amyotrophic lateral sclerosis C. Patient’s AChR antibody level also provides clinically valuable confirmation of the effectiveness of treatment. of ptosis on upward gaze. Chronic myopathies C. late-onset nemaline myopathy. Fibromyalgia 1842 Which of the following is not a cause of intermittent weakness of muscles ? D. sclerosis. cause activity-related muscle breakdown accompanied by myoglobinuria. Depressed reflexes 1847 Which of the following is a painful muscle condition ? D. Methadone. hyperkalemic. Preserved sensation Harrison’s 18th Ed. 3485 A. 3487 Painful muscle conditions not associated with muscle weakness include fibromyalgia and A. Paramyotonia congenita Clinical findings of a myopathy are proximal. Danazol. Time to development of ptosis on upward gaze D. It occurs in disorders of Most useful clinical tests to assess MG patient’s clinical status include forward arm abduction neuromuscular transmission and in disorders altering energy production (glycolysis. Cocaine. appearing as light-brown. 3489 A. Polymyositis Harrison’s 18th Ed. syndrome” ? Harrison’s 18th Ed.794 MCQ’s FOR MEDICAL PROFESSIONALS BY PROF. Takayasu’s arteritis . Carnitine palmitoyltransferase deficiency Harrison’s 18th Ed. and time to development metabolism. Intercurrent infection D. 3485 A. or mitochondrial energy production). fatty acid utilization (carnitine palmitoyltransferase D. Cyclophosphamide acid utilization (carnitine palmitoyltransferase deficiency). forced vital capacity. 1840 Which of the following is useful to assess MG patient’s clinical 1845 Pathologic fatigability occurs in which of the following ? status ? Harrison’s 18th Ed. 1846 Drugs that cause true myalgia include ? Harrison’s 18th Ed. is useful in the treatment of anti.to dark-brown-colored urine. 3485 C. 3487 1839 Most common cause of myasthenic crisis is ? A. chronic myopathies. Gold. Heroin. 3487 D. 3487 1838 Which of the following drug is useful in the treatment of anti. Decrease in ESR Disorders causing intermittent muscle weakness include myasthenia gravis. Statins. All of the above Pathologic fatigability refers to inability to maintain or sustain a force. fatty C. a monoclonal antibody that depletes CD20 B cells. Cyclosporine. All of the above A. All of the above Harrison’s 18th Ed. and metabolic energy deficiencies of C. Increase of RBC mean corpuscular volume deficiency). Labetalol. Labetalol C. Amyotrophic lateral sclerosis B. Reduction of lymphocytes to <1000/µL and/or an increase of MCV may be used as indications of 1843 Which of the following is accompanied by myoglobinuria ? adequacy of azathioprine dosage. range of eye movements. Myasthenia gravis MuSK antibody positive MG ? B. and some mitochondrial myopathies. Carnitine palmitoyltransferase deficiency B. 3487 Harrison’s 18th Ed. A. Drug underdose Dropped head syndrome indicates selective neck extensor muscle weakness. B.). All of the above D. Myasthenia gravis polymyalgia rheumatica. Hypothyroidism C. focal myositis. Paramyotonia congenita rheumatica ? D. Azathioprine Rituximab. AJAY MATHUR Neurology B. Disorders of neuromuscular transmission A. Forward arm abduction time (5 min) B. lipid time (up to a full 5 min. Polymyositis 1848 Which of the following is associated with polymyalgia C. Increase in hematocrit glycolysis (myophosphorylase deficiency). Forced vital capacity C. 1844 Which of the following can have a presentation of “dropped head MuSK antibody positive MG. Zidovudine. Symmetric limb weakness C. symmetric limb weakness (arms or legs) with preserved reflexes and sensation. Rituximab The states of metabolic energy deficiencies of glycolysis (myophosphorylase deficiency). Myasthenia gravis Harrison’s 18th Ed. periodic paralyses (hypokalemic. Harrison’s 18th Ed. and paramyotonia congenita). B. B. d-Penicillamine. Neuromuscular diseases associated with this pattern of weakness include myasthenia gravis. Proximal limb weakness Drugs that cause true myalgia include Cimetidine. Disorders of defects in glycolysis B. Cimetidine B. Tacrolimus D. Birth A. patients of DMD are predisposed to serious. Lactic dehydrogenase (LDH) Harrison’s 18th Ed. Danish B. Creatine kinase (CK) 1857 In DMD.5 years C.2 years B. 3494 D. aspartate aminotransferase (AST). Most common gene mutation is a deletion. Less often. Neck flexors Creatine kinase (CK). Duchenne Muscular Dystrophy (DMD) 1860 Which of the following statements about serum CK levels in DMD B. Granulomatosis with polyangiitis (Wegener’s) A. 1 . Mostly due to gene duplication B. 12 1849 Muscle cramps often occur in which of the following neurogenic D. 3494 D. Neck extensors glutamyl transferase (GGT) has a liver origin and is not found in muscle. Dystrophin gene is one of the largest identified human genes (>2000 kb) on the C. and lactic dehydrogenase (LDH) are enzymes sharing an origin in both muscle and liver. Harrison’s 18th Ed. alanine aminotransferase (ALT). also referred to as cranial arteritis or temporal arteritis. 3489 Patients with Duchenne dystrophy are typically in wheelchair by age of 12. sometimes fatal pulmonary infections. 795 Neurology MCQ’s FORMCQ’s FOR MEDICAL MEDICAL PROFESSIONALS PROFESSIONALS BY PROF. with predilection for proximal limb muscles and neck was ? flexors. Decline late in the disease . and trunk muscles. Other causes of death include aspiration of food and acute gastric dilation. Polish 1858 Which of the following about DMD is false ? B. 3494 Table 387-2 C. By age 16–18 years. Facioscapulohumeral Muscular Dystrophy B. Dystrophin is on X chromosome A. German A. 3 . Serum CK levels are abnormal at birth C. but it usually becomes apparent between ages 3 & 5. C. A. Duchenne’s dystrophy is caused by a gene duplication or point mutation. 3490 Weakness is in the hip. Distal limb muscles B. Acute gastric dilation D. 3491 Duchenne’s muscular dystrophy ? Harrison’s 18th Ed. Knee extensor muscles D. which of the following muscle groups are preferrentially B. 3494 C. C. Pulmonary infection B. 10 disorder of medium. Radiculopathies Harrison’s 18th Ed. may accompany polymyalgia rheumatica. They are not a feature of most primary muscle diseases. thigh. D. Emery-Dreifuss Muscular Dystrophy A. Gamma-glutamyl transferase (GGT) A. D. 3494 A. 14 disorders ? Harrison’s 18th Ed. and polyneuropathies. 8 Giant cell arteritis. Invariably elevated D. Harrison’s 18th Ed. aldolase. Henoch-Schönlein purpura 1855 Patients with Duchenne dystrophy are typically in a wheelchair by C. Aldolase involved ? C. Hip muscles Muscle cramps often occur in neurogenic disorders. Meryon 1853 Duchenne Muscular dystrophy is present at ? 1859 Which of the following is an uncommon cause of death in Harrison’s 18th Ed. Anterior compartment of leg muscles radiculopathies.and large-sized arteries. A. 5 . 3494 A. especially motor neuron disease. Gowers DMD is caused by a mutation of gene encoding 427-kDa dystrophin protein on the inner surface of sarcolemma. Limb-girdle Muscular Dystrophy is false ? Harrison’s 18th Ed. Motor neuron disease 1856 Gowers’ sign is due to weakness of which group of muscles ? B. Cardiac 1854 ‘Gowers’ maneuver’ is diagnostic of ? cause of death is uncommon despite the presence of a cardiomyopathy in almost all patients. Dystrophin is the largest identified human gene 1852 Who discovered dystrophin gene ? C. Gamma. Anterior abdominal muscles 1851 Nationality of Guillaume Benjamin Amand Duchenne de Boulogne Loss of muscle strength is progressive. 3494 C. A. All of the above B. Little D. AJAY MATHUR Cardiology 795 B. French Harrison’s 18th Ed. C. D.7 years D. Kunkel short arm of X chromosome (Xp21). Polyneuropathies A. is an inflammatory B. Aspiration of food C. Cardiac cause (cardiomyopathy) Duchenne dystrophy is present at birth. Due to mutation in dystrophin gene D. Temporal arteritis the age of ? Harrison’s 18th Ed. Posterior compartment of leg muscles 1850 Which of the following enzymes is not found in muscles ? Gowers’ sign refers to the inability to get up from the floor without climbing up the extremities. 5 years D. Myotonic Dystrophy 1872 In which of the following gastrocnemius muscles are preferentially affected at onset ? Becker’s muscular dystrophy is a less severe and 10 times less frequent form of X-linked recessive Harrison’s 18th Ed. Emerin C.352:2413-23 to F-actin at its amino terminus and to beta-dystroglycan at the carboxyl terminus. 3493 Table 387-7 D. the key defect in DMD due to abnormal dystrophin B. None of the above C. Calpain activity is regulated by calpastatin 1862 In essence. 3495 Harrison’s 18th Ed. Laing distal myopathy A. Calpain-4 1863 Which of the following significantly slows progression of Duchenne’s dystrophy ? 1870 LGMD2B is due to defect in ? Harrison’s 18th Ed. D. Calpain-2 glycoprotein complexes weakens sarcolemma. 3494 C. Calpain-3 leading to muscle fiber necrosis and eventually muscular dystrophy. F-actin D. causing membrane tears and a cascade of events C. Glucocorticoids B. Cyclosporine C. 1 . Harrison’s 18th Ed. Multiple sclerosis B. Dysferlin B. Physical weakening of the sarcolemma D. Emery-Dreifuss Muscular Dystrophy D. A. Limb-girdle muscular dystrophy type 2A (LGMD2A) C. Walker-Warburg syndrome A. Welander distal myopathy 1865 Most patients with Becker’s muscular dystrophy first experience B. Harrison’s 18th Ed. 3493 Table 387-7 1864 Becker’s Muscular Dystrophy resembles which of the following A. Calpains are involved in apoptosis protein is ? Harrison’s 18th Ed. Calpain-1 Dystrophin-glycoprotein complex confer stability to the sarcolemma. AJAY MATHUR Neurology D.2 years In dysferlinopathies (LGMD2B) there is a predilection for early atrophy of gastrocnemius muscles. serum CK levels are invariably elevated (20 to 100 times). Disruption of dystrophin- B. Telethonin D. Alpha-dystroglycan C. Arginine proteases . Myotubular myopathy B. 1867 Tissue-specific calpains have been implicated in ? 1861 Dystrophin binds to which of the following in sarcolemma ? N Engl J Med 2005. Dystrophin binds N Engl J Med 2005. Miyoshi myopathy B. Laminin B. Leucine proteases but decline late in the disease because of inactivity & loss of muscle mass. Diabetes A. 5 . Laing distal myopathy A. IVIg D. Abnormal extracellular matrix (ECM) A. Miyoshi myopathy is unique in that gastrocnemius muscles are preferentially affected at onset. Methionine proteases In DMD. None of the above B. 3491 C. Duchenne’s muscular dystrophy D. Cysteine proteases C. Calpain is a cytoplasmic protease A. All of the above D.796 MCQ’s FOR MEDICAL PROFESSIONALS BY PROF. 3495 C. Miyoshi myopathy B. Tibial muscular dystrophy (Udd) difficulties at what age ? Harrison’s 18th Ed. Tibial muscular dystrophy (Udd) Harrison’s 18th Ed. Caveolin-3 1868 Which of the following statements about Calpain is false ? Dystrophin is part of a large complex of sarcolemmal proteins and glycoproteins. 3 . 3499 Duchenne’s muscular dystrophy. C.15 years 1873 Which of the following muscular dystrophy has its onset at birth ? Most patients with Becker’s dystrophy first experience difficulties between ages 5 and 15 years. Disruption of Golgi apparatus 1869 LGMD2A is caused by gene mutations in ? C. Welander distal myopathy muscular dystrophy ? B. Levels are abnormal at birth D. Azathioprine A. significantly slows progression of Duchenne’s 1871 Which of the following is a variant of LGMD2B ? dystrophy for up to 3 years. Abnormal mitochondria Harrison’s 18th Ed. Muscle-eye-brain disease N Engl J Med 2005.75 mg/kg per day). 3493 Table 387-7 A. 3497 Table 387-8 1866 Calpains are Ca++ dependent ? A. Titin Glucocorticoids (prednisone 0. 3494 A.352:2413-23 Harrison’s 18th Ed. Limb-Girdle Muscular Dystrophy Miyoshi’s myopathy is variant of LGMD2B with calf muscles affected at onset C. Birth D.352:2413-23 B. A. Mexiletine C. Emery-Dreifuss Muscular Dystrophy D. Tibial muscular dystrophy A. 3496 A. wrist extensors & muscles weakness of anterior compartment muscles of legs are severely & WWS. vision is impaired. ALT. Kearns-Sayre Syndrome (KSS) . 1878 Which of the following is not a congenital myopathy ? Harrison’s 18th Ed. C-terminal mutations of the RYR1 gene (ryanodine receptor gene on chromosome 19q) predispose to this complication. All of the above 1879 Patients with which of the following congenital myopathies are Fatty acids and glucose are are the two principal sources of energy for skeletal muscles. aldolase and LDH originate in both muscle & liver. Flecainide Serum CK level is is very elevated in Miyoshi myopathy. weakness begins in is not found in muscle. Muscle-eye-brain (MEB) disease C. with relative sparing of deltoid muscles. 3501 A. Amiodarone D. All of the above C. Centronuclear (myotubular) myopathy B. Nemaline (rod) myopathy B. 1874 Which of the following muscles is relatively spared in Facioscapulohumeral (FSH) muscular dystrophy ? 1880 Severe brain impairment is seen in which of the following Harrison’s 18th Ed. 3499 1884 Which of the following is a principal source of energy for skeletal A. 3500 Table 387-9 Harrison’s 18th Ed. Biceps B. Facial A. -glutamyl transferase (GGT) D. Congenital muscular dystrophy with severe brain impairment are FCMD. Triceps C. All of the above Muscle-eye-brain disease. Malignant hyperthermia during anesthesia may occur in central core disease. Facioscapulohumeral (FSH) Muscular Dystrophy B. Nonanka distal myopathy B. Central core disease muscles ? B. Walker-Warburg syndrome is more severe. 1881 Which of the following enzyme is not found in muscle ? 1875 Which of the following distal myopathy begins in hands ? Harrison’s 18th Ed. Biceps. Miyoshi myopathy C. onset is in 5th decade. Fatty acid Miyoshi myopathy is a type of distal myopathy. Leigh’s syndrome B. Amono acid D. Sotalol Phenytoin & mexiletine are the preferred antimyotonia drugs. In other distal myopathies. Centronuclear (myotubular) myopathy Merosin deficiency. Aspartate aminotransferase (AST) A. Scapular winging is common. Myotonic dystrophy C. but not in FCMD. 3498 B. Alanine aminotransferase (ALT) B. Walker-Warburg syndrome (WWS) D. with death by 1 year. In MEB triceps. Lifespan is normal. Lactic acid fingerprint body myopathy. Harrison’s 18th Ed. hands and slowly progresses to involve distal lower extremities. MEB disease & WWS. 1882 “Hatchet-faced” appearance is typical of which of the following 1876 Which of the following distal myopathies begin in lower limbs ? condition ? Harrison’s 18th Ed. Central core disease A. Weakness of shoulder girdles brings patient to the doctor. Limb-Girdle Muscular Dystrophy 1877 Serum CK is elevated most in which of the following distal Patients with myotonic dystrophy have a typical “hatchet-faced” appearance due to temporalis. Minicore myopathy (multi-minicore disease). masseter and facial muscle atrophy and weakness. AJAY MATHUR Cardiology 797 D. 3490 Harrison’s 18th Ed. Origin of GGT is liver and In autosomal dominant Welander’s distal myopathy. Fukitin-related protein deficiency. Nonanka distal myopathy B. Deltoid D. Tibial muscular dystrophy A. Nonanka distal myopathy D. Tibial muscular dystrophy C. 3501 C. Miyoshi myopathy C. Miyoshi myopathy Enzymes AST. 3496 A. Walker-Warburg syndrome are congenital muscular dystrophies that have an onset at birth. Frontal baldness is also a characteristic myopathies ? feature. Fukuyama congenital muscular dystrophy. 3498 congenital muscular dystrophies ? Harrison’s 18th Ed. 797 Neurology MCQ’s FORMCQ’s FOR MEDICAL MEDICAL PROFESSIONALS PROFESSIONALS BY PROF. 3499 1885 Alpha-glucosidase or acid maltase deficiency is also called ? Harrison’s 18th Ed. All of the above D. C. Lactic dehydrogenase (LDH) C. affected. and sarcotubular myopathy are also congenital myopathies. Welander distal myopathy 1883 Which of the following is an “antimyotonia drug” ? Harrison’s 18th Ed. All of the above Facial weakness is the initial manifestation in Facioscapulohumeral (FSH) Muscular Dystrophy. Nemaline (rod) myopathy Harrison’s 18th Ed. serum CK is only slightly increased. D. D. Welander distal myopathy B. 3500 Table 387-9 A. 3499 A. D. susceptibile to malignant hyperthermia ? Harrison’s 18th Ed. Fukuyama congenital muscular dystrophy (FCMD) B. Miyoshi myopathy A. Clinically. WWS is the most severe congenital muscular dystrophy. Beta-enolase deficiency C. phosphoglycerate mutase deficiency (type X glycogenosis). with onset of symptoms in the first 3 months of life.798 MCQ’s FOR MEDICAL PROFESSIONALS BY PROF. Phosphoglycerate mutase deficiency or acid maltase deficiency ? D. Fatty acids are derived from triglycerides in muscle fibers 1887 Which of the following about Pompe’s disease is false ? Harrison’s 18th Ed. phosphofructokinase deficiency (type VII glycogenosis). 3502 A. Outer side of inner mitochondrial membrane D. To enter mitochondria. phosphoglycerate kinase deficiency (type IX glycogenosis). and beta-enolase deficiency. 3502 D. 3502 converted to an “activated fatty acid. Acyl-CoA must be linked with carnitine by enzyme carnitine palmitoyltransferase (CPT) I for 1889 Which of the following is called McArdle’s disease ? transport into mitochondria. C. All of the above A. Acyl-CoA 1888 Which of the following is a disorder of glycolysis causing exercise D. 3502 D. Inner side of inner mitochondrial membrane In McArdle’s disease. Inner side of outer mitochondrial membrane C. 3502 There are three clinical forms of alpha-glucosidase. Phosphoglycerate kinase deficiency C. Autosomal recessive disorder D. Infantile form deficiency are inherited as autosomal recessive disorders. Cystinosin V glycogenosis). 3502 D. Phosphofructokinase deficiency 1886 Which out of the following is the most common alpha-glucosidase. Outer side of inner mitochondrial membrane Myophosphorylase deficiency is also called McArdle’s disease and is the most common of the D. Carnitine palmitoyltransferase (CPT) I glycolytic defects are associated with recurrent myoglobinuria. Pompe’s Disease 1891 Which of the following is a X-linked recessive disorder ? Harrison’s 18th Ed. a fatty acid must first be converted to ? C. disorder with mental retardation is the usual clinical presentation in phosphoglycerate kinase deficiency. Outer side of outer mitochondrial membrane C. Carnitine palmitoyltransferase (CPT) I is present on inner side of outer mitochondrial membrane. AJAY MATHUR Neurology C. 3501 C. McArdle’s disease A. Seizure allowing transport of acyl-CoA into the mitochondrial matrix for beta-oxidation. Outer side of outer mitochondrial membrane B. Enzyme replacement tt. anemia accompany deficiencies of phosphofructokinase & phosphoglycerate kinase. Carnitine lactate dehydrogenase deficiency (glycogenosis type XI). 3501 Myophosphorylase deficiency. 1890 Seizure disorder with mental retardation is a feature of which of the following ? 1896 Enzyme carnitine palmitoyltransferase (CPT) II is present on ? Harrison’s 18th Ed. C. 3502 1895 Enzyme carnitine palmitoyltransferase (CPT) I is present on ? A. These D. phosphofructokinase deficiency & phosphoglycerate mutase A. A. Beta-enolase deficiency D. Caused by mutations of the alpha-glucosidase gene 1893 To enter mitochondria. Phosphofructokinase deficiency A. Myophosphorylase deficiency Harrison’s 18th Ed. Childhood form has X-linked recessive inheritance. Enoyl–acyl carrier Enzyme replacement therapy (ERT) with IV recombinant human alpha-glucosidase is beneficial B. or acid maltase deficiency (type II glycogenosis). Amino-acyl-tRNA intolerance ? Oxidation of fatty acids occurs in mitochondria. 3502 Harrison’s 18th Ed. None of the above A. Lactate dehydrogenase deficiency B. Hemolytic Carnitine is removed by CPT II. Adult form 1892 Which of the following statements about lipid as energy source is D. Myophosphorylase deficiency 1894 Acyl-CoA must be linked to which of the following for transport B. A. B. Harrison’s 18th Ed. beneficial in infantile-onset type Harrison’s 18th Ed. Phosphoglycerate kinase deficiency B. Fatty acids are derived from circulating VLDL in blood B. Inner side of inner mitochondrial membrane glycolytic defects associated with exercise intolerance. Inner side of outer mitochondrial membrane D. The infantile form is the most common. Myophosphorylase deficiency Alpha-glucosidase or acid maltase deficiency is also called Pompe’s Disease. C. Lactate dehydrogenase deficiency Harrison’s 18th Ed. 1897 Myoglobinuria in CPT II deficiency is precipitated by ? Harrison’s 18th Ed. Phosphoglycerate kinase deficiency Harrison’s 18th Ed. an enzyme attached to the inside of inner mitochondrial membrane. exercise tolerance is enhanced by warm-up or brief periods of rest. 3502 . Acyl-NADH in infantile-onset type. Phosphofructokinase deficiency B. 3502 B. Phosphofructokinase deficiency into mitochondria ? C. Oxidation of fatty acids occurs in mitochondria A. None of the above false ? Harrison’s 18th Ed. Acyl-CoA dehydrogenase Disorders of glycolysis causing exercise intolerance include myophosphorylase deficiency (type B. None of the above B. C.” acyl-CoA. a fatty acid must first be Harrison’s 18th Ed. Myophosphorylase deficiency A. C. optic atrophy. 799 Neurology MCQ’s FORMCQ’s FOR MEDICAL MEDICAL PROFESSIONALS PROFESSIONALS BY PROF. All of the above 1903 Which of the following is false about progressive external CPT II deficiency is much more common in men than women (5:1) and is the most common ophthalmoplegia (PEO) ? recognizable cause of recurrent myoglobinuria. All of the above 1902 Which of the following about KSS is false ? Characteristic features include myoclonic epilepsy. CSF protein is normal. Diplopia B. cerebellar ataxia. C. Harrison’s 18th Ed. starlike staining properties. AJAY MATHUR Cardiology 799 A. Addison’s disease A. Diabetes mellitus (MERRF) is ? Harrison’s 18th Ed. All of the above 1900 Which of the following about Kearns-Sayre Syndrome (KSS) is false ? In chromosome 10q–related CPEO. Sporadic disorder B. A. which of the following is abnormal in a muscle cell ? C. Carnitine palmitoyltransferase (CPT) I D. It co-localizes with mtDNA and is critical for lifetime maintenance of mitochondrial integrity. Spontaneous deletion occur in spermatozoa B. Its gene product is twinkle. Directly inherited from cytoplasm of oocyte mainly D. It codes for 22 transfer RNAs. C. and pigmentary retinopathy. 15q22–26 1899 Which of the following is a mitochondrial myopathy ? D. All of the above Harrison’s 18th Ed. 3503 1907 Characteristic feature of myoclonic epilepsy with ragged red fibers A. a point of distinction from myasthenia gravis. 3503 Autosomal dominant form of CPEO is linked to loci on 3 chromosomes . 15q22–26 D. 3503 named for its punctate. Thus. The disease is caused by single mtDNA deletions arising spontaneously in the ovum or zygote. 3503 C. CPEO. Onset before 20 years of age B. Onset is usually after puberty Harrison’s 18th Ed. 4q35 DNA of mitochondria is directly inherited from the cytoplasm of gametes. fasting or infections. Muscle pain and myoglobinuria can be precipitated Harrison’s 18th Ed.CNS syndromes & pure myopathy. Prolonged exercise D. A. Ragged red fibers are prominently seen in muscle biopsy. or cerebellar ataxia. circular molecule comprising 16. mainly oocyte. Other more variable features include dementia. Caused by single mtDNA deletions 1908 To be called ragged red fiber. Skeletal muscle . A. 10q24 & 15q22–26. Cerebellar ataxia D. plus one or more of the following features: complete heart block. Progressive muscle weakness Diabetes mellitus is a cardinal sign of mitochondrial disorders and occurs in 13% of KSS patients. C. 3503 base pairs. A. Spontaneous deletion occur in ovum or zygote Harrison’s 18th Ed. Myoclonic epilepsy C. 4q35 B. and progressive muscle Harrison’s 18th Ed. Triad of clinical findings in KSS C. Onset is usually after puberty. Due to mtDNA mutations 1898 Which of the following is false about human mitochondrial DNA B. disorders ? Harrison’s 18th Ed. Sperm B. Myoadenylate deaminase includes onset before age 20.977 bp of contiguous mtDNA. 3502 .569 Harrison’s 18th Ed. mitochondrial genes are derived almost exclusively from mother. and diabetes mellitus. usually in the absence of diplopia. and 13 polypeptides of the respiratory chain enzymes. 3502 D.4q35. Pure myopathy Harrison’s 18th Ed. Double-stranded circular molecule PEO is caused by nuclear DNA mutations affecting mtDNA copy number & integrity. Inherited in a Mendelian fashion (mtDNA) ? C. skeletal muscle . D. hearing loss. Acid maltase In CPEO. peripheral neuropathy. Genetics of mitochondrial diseases differ from genetics of chromosomal disorders. a mutation affects the gene encoding mtDNA polymerase 1901 Which of the following is a cardinal sign of mitochondrial (POLG). 3503 weakness in a limb-girdle distribution. Hyperaldosteronism C. 2 ribosomal RNAs. mutations occur in C10orf2 gene. Chronic progressive external ophthalmoplegia (CPEO) B. 3503 D. cerebrospinal fluid protein >100 mg/dL. Pigmentary retinopathy A. It is inherited B. 3503 B. Human dominant form of CPEO ? mitochondrial DNA (mtDNA) consists of a double-strand. None of the above 1904 Which of the following chromosome is affected in autosomal Each mitochondrion possesses a DNA genome that is distinct from that of nuclear DNA. Infections D. A. Hypoparathyroidism B.CNS syndromes 1905 “Twinkle” is the gene product of ? C. varying degrees of ptosis & weakness of extraocular muscles are seen. Fasting KSS is a sporadic disorder. POLG In CPEO mapped to chromosome 15q. Ragged red fibers seen in muscle biopsy A. 10q24 contributes very little. All of the above A. 10q24 Mitochondrial myopathies include chronic progressive external ophthalmoplegia (CPEO). D. an enzyme important in mtDNA replication. Codes for transfer & ribosomal RNAs & polypeptides in a Mendelian fashion. CPEO 1906 Enzyme important in mtDNA replication is ? Harrison’s 18th Ed. The most common deletion removes 4. 3503 by prolonged exercise. Lactic Acidosis. the term stroke like is appropriate because cerebral lesions do not conform to a strictly due to mutations in voltage-sensitive. onset is before age 20. Hyperkalemic periodic paralysis 1909 Which of the following is caused by maternally inherited point mutations of mitochondrial tRNA gene ? C. Mannitol is the preferred D. Pompe’s Disease A. and Stroke like Episodes 1916 Hypokalemic periodic paralysis type 1 is caused by mutation in (MELAS) is false ? which of the following gene ? Harrison’s 18th Ed. Cell membrane In MELAS. Hypokalemic periodic paralysis B. 3504 Table 387-10 numbers of abnormal mitochondria (enlarged. preferred vehicle for seen mainly in occipital & parietal lobes. 3504 B. administration of IV potassium is ? Harrison’s 18th Ed. IN MELAS. Cerebral lesions conform to a vascular distribution A. Hyperkalemic periodic paralysis MERRF & MELAS are caused by maternally inherited point mutations of mitochondrial tRNA C. 3503 D. hemianopia. Mitochondria D. Serum lactic acid is depolarizes when potassium levels are low. Middle age C. and cerebral angiography does not show lesions of the major cerebral blood vessels. CALCL1A3 D. bizarrely shaped with crystalline inclusions) highlighted with modified trichrome stain. SCN3A . A3243G point mutation in tRNALeu(UUR) B. CALCL1A2 C. GDW Harrison’s 18th Ed. Kearns-Sayre Syndrome (KSS) 1915 Which of the following is a potassium channelopathy ? B. Golgi apparatus C. 3505 1912 Which of the following is caused by maternally inherited point A.2 . 3504 C. 3256T point mutation in tRNALeu(UUR) B. MERRF Harrison’s 18th Ed. 3504 Encephalopathy. 3504 A. D. Muscle biopsies show ragged red fibers D. Paramyotonia congenita genes. Focal lesions in frontal lobes C. Cerebral angiography essentially normal with onset after age 25 is almost never due to periodic paralyses. All of the above B. SCN2A B. A. Lactic Acidosis. neuroimaging shows basal ganglia calcification. Paramyotonia congenita Harrison’s 18th Ed. Andersen-Tawil Syndrome 1910 Which of the following about Mitochondrial Myopathy. MERRF & MELAS are caused by maternally inherited point mutations of mitochondrial tRNA genes. Encephalopathy. SCN1A Harrison’s 18th Ed. 3252G point mutation in tRNALeu(UUR) C. Nucleus 1914 Which of the following is a calcium channelopathy ? Term ragged red fibers was coined by Olson (1972) to describe muscle fibers with significant Harrison’s 18th Ed. and cortical blindness. Childhood Harrison’s 18th Ed. Vascular territories are not respected Onset of hypokalemic periodic paralysis (Hypokpp) occurs at adolescence. Basal ganglia calcification D. D. 1911 Which of the following about Mitochondrial Myopathy. typically elevated. every 30 minutes. Adulthood B. Mitochondrial DNA depletion syndrome (MDS) vehicle for administration of IV potassium as glucose solution may further reduce serum potassium levels. 3505 MELAS ? A. CALCL1A1 B. Adolescence A. type 2 is due to mutations in voltage-sensitive sodium channel gene (SCN4A). MELAS Treatment of hypokalemic periodic paralysis is oral KCl (0. 3504 Table 387-10 C.0. Strict vascular territories are not respected. AJAY MATHUR Neurology A. Mannitol B. Pompe’s Disease Should IV therapy become necessary (swallowing problems or vomiting).4 mmol/kg). Serum lactic acid is elevated C. and Stroke like Episodes (MELAS) is false ? A. Kearns-Sayre Syndrome (KSS) D. CALCL1A4 HypoKPP type 1 is inherited as an autosomal dominant disorder with incomplete penetrance. 3503 A. Like KSS. CALCL1A3. Focal lesions mimiking infarction are 1918 In hypokalemic periodic paralysis. Partial motor or generalized seizures B. 1917 Onset of hypokalemic periodic paralysis (HypoKPP) is at ? Harrison’s 18th Ed. CSF protein is increased. It is In MELAS. 3504 Harrison’s 18th Ed. 1919 Hyperkalemic periodic paralysis (HyperKPP) is caused by mutation in which of the following gene ? 1913 Which of the following point mutation is most common in Harrison’s 18th Ed. The muscle cell Cerebral insults cause hemiparesis. C. Muscle biopsies show ragged red fibers. HypoKPP vascular distribution. Hypokalemic periodic paralysis D. 3271C point mutation in tRNALeu(UUR) C. Episodic weakness D. Seizures are partial motor or generalized.800 MCQ’s FOR MEDICAL PROFESSIONALS BY PROF. A3243G point mutation in tRNA Leu(UUR) is the most common (80%). Ringer lactate A. skeletal muscle calcium channel gene. Andersen-Tawil Syndrome A. Normal saline mutations of mitochondrial tRNA gene ? B. Becker’s disease Fluorinated glucocorticoids like triamcinolone. Paramyotonia congenita D. Diabetes mellitus Harrison’s 18th Ed. Fibrates C. 1930 In inflammatory myopathy. EMG is typically normal. Vitamin D Harrison’s 18th Ed. 3509 . Becker’s disease sepsis. HMG-CoA reductase inhibitors (statins). Vitamin E related myopathy A. All classes of lipid-lowering agents have been implicated in muscle toxicity. 3506 D. Zidovudine or prominent low-set ears. including fibrates (clofibrate. SCN4A 1925 “Acute quadriplegic” myopathy best relates to ? Harrison’s 18th Ed. Betamethasone 1921 Long QT syndrome is a feature of which of the following ? Harrison’s 18th Ed. A. Glucocorticoids 1923 Characteristic prolongation of relaxation phase of muscle stretch reflexes in hypothyroidism is best observed in ? D. Hyperparathyroidism caused by which of the following medications ? D. betamethasone and dexamethasone pose a high risk for myopathy. A. 3506 Zidovudine myopathy produces muscle atrophy affecting thigh & calf muscles. C. Dermatomyositis. Supinator reflex 388 . ventricular ectopy. Knee reflex and Inclusion Body Myositis Relaxation phase of muscle stretch reflexes is characteristically prolonged in hypothyroidism and is best observed at the ankle or biceps brachii reflexes. Harrison’s 18th Ed.1) gene that heighten muscle cell excitability. D-penicillamine C. B. Thomsen’s disease D. 3508 A. Vitamin D Harrison’s 18th Ed. 3509 myopathy ? A. 3508 Myotonia in chloride channel disorders . Myasthenia gravis and polymyositis can also occur with D-penicillamine. 3507 B. Dermatomyositis (DM) A. hypertelorism. All of the above myositis (IBM). dermatomyositis (DM) and inclusion body D. Vitamin D related myopathy Harrison’s 18th Ed. Triceps reflex penicillamine. micrognathia. D-penicillamine C. All of the above 1926 Which of the following glucocorticoid pose a high risk for myopathy ? In paramyotonia congenita (PC).Thomsen’s disease and Becker’s disease is worsened by cold and improved by activity.Polymyositis. 3506 C. Inclusion body myositis (IBM) B. HMG-CoA reductase inhibitors (statins) D. clinodactyly. All of the above C. 3506 Drug-related inflammatory or antibody-mediated myopathy is most consistently caused by D- A. D. C. It is caused by mutations of the inwardly rectifying potassium channel (Kir 2. and dysmorphic features (short stature. bidirectional ventricular arrhythmias). B. 801 Neurology MCQ’s FORMCQ’s FOR MEDICAL MEDICAL PROFESSIONALS PROFESSIONALS BY PROF. Dexamethasone A. Hyperthyroidism shows ragged red fibers with minimal inflammation. even when there is minimal clinical evidence of muscle disease. Lipid-lowering agent related myopathy 1920 Attacks of muscle weakness are induced by cold in which of the following ? C. 3508 episodic weakness. Paramyotonia congenita D. Triamcinolone B. Zidovudine In hypothyroidism. Glucocorticoids 1922 Serum CK level is most conspicuously elevated in which of the following endocrine myopathies ? D. Hypothyroidism 1928 Drug-related inflammatory or antibody-mediated myopathy is C. Andersen-Tawil Syndrome 1927 Which of the following medication can caise mitochondrial myopathy with ragged red fibers ? Autosomal dominant Andersen-Tawil Syndrome is a potassium channel disorder characterized by Harrison’s 18th Ed. Glucocorticoid related myopathy B. 3507 HyperKPP is an autosomal dominant disorder due to mutations of voltage-gated sodium channel SCN4A gene. small A. scoliosis. Biceps reflex C. AJAY MATHUR Cardiology 801 D. which of the following muscles is not involved ? Harrison’s 18th Ed. and broad forehead). gemfibrozil). Thomsen’s disease Glucocorticoid myopathy occurs with chronic treatment or as “acute quadriplegic” myopathy secondary to high-dose IV glucocorticoid use. the attacks of weakness are cold-induced or occur spontaneously. 1929 Which of the following is an inflammatory myopathy ? 1924 Which class of the following lipid-lowering agents causes Harrison’s 18th Ed. Polymyositis (PM) Harrison’s 18th Ed. cardiac arrhythmias (long QT. niacin (nicotinic acid) & ezetimibe. Muscle biopsy A. B. serum CK level is elevated (up to 10 times normal). Acute quadriplegic myopathy can also occur in B. All of the above B. Niacin (nicotinic acid) Inflammatory myopathies include polymyositis (PM). B. B. nonerosive arthritis. Fine-motor movements affected late in the course D. 1937 Patients with overlap syndrome of dermatomyositis and systemic sclerosis have which antinuclear antibody ? 1931 Which of the following about inclusion body myositis is false ? Harrison’s 18th Ed. 1938 Antibody directed against histidyl-transfer RNA synthetase is 1932 Extramuscular manifestations are associated with which of the called ? following inflammatory myopathies ? Harrison’s 18th Ed. Colon cancer (parasitic polymyositis) ? Harrison’s 18th Ed. or IBM. CD4/MHC-I complex C. involvement of extraocular & facial muscles. Falling is common Patients with overlap syndrome of DM & systemic sclerosis may have anti-PM/Scl antinuclear Fine-motor movements depend on strength of distal muscles are affected late in the course of PM antibody directed against a nucleolar-protein complex. ocular muscles are spared. biochemical muscle disorder. or Sjögren’s D. All of the above C. humoral immune mechanisms are implicated. colon cancer. Nonerosive arthritis Polymyositis is a subacute inflammatory myopathy of adults. incidence of ovarian cancer.mediated Harrison’s 18th Ed. 3510 1939 Inflammatory myopathy patients with anti-Jo-1 antibodies have A. All of the above history of exposure to myotoxic drugs or toxins. Quadriceps muscle C. Thrice more frequent in men than women B. None of the above B. but fairly early in IBM. 3510 Table 388-1 A. Inflammatory myopathy patients with anti-Jo-1 antibodies have an increased frequency of interstitial lung disease. Anti-Ro antibody B. Anti-Jo-1 antibody A. melanoma. Polymyositis B. V sign A. Ovarian cancer B. Anti-Zic antibody D. 3511 Harrison’s 18th Ed. Anti-Hu antibody C. Ocular muscles D. A. Cysticerci 1936 Which of the following is rare with dermatomyositis (DM) ? Harrison’s 18th Ed. 3511 C. Toxoplasma In DM. cytokines secreted include all except ? by activated T cells & macrophages induce its expression. All of the above Antibody directed against cytoplasmic histidyl-transfer RNA synthetase is called anti-Jo-1. family history of neuromuscular disease. Dermatomyositis C. 3513 D. Gottron rash B. Rheumatoid arthritis Protozoa (toxoplasma. Anti-IBM/Scl B. 3509 A. cestodes (cysticerci). Sjögren’s syndrome B. diagnosis of inflammatory myopathy should be questioned. and not in PM or IBM. In PM. All of the above C. Trypanosoma C. CD28/MHC-I complex 1935 Most common tumors associated with dermatomyositis (DM) MHC-I expression is absent from sarcolemma of normal muscle fibers. T cell . neurogenic disease. Shawl sign B. Involvement of extraocular & facial muscles Harrison’s 18th Ed. All of the above D. Raynaud’s phenomenon. In inflammatory myopathy. B. breast cancer. 3511 D. AJAY MATHUR Neurology A. 3510 1940 Which of the following is characteristic of polymyositis ? Harrison’s 18th Ed. Systemic lupus erythematosus . Disease progression is slow but steady. All of the above A.802 MCQ’s FOR MEDICAL PROFESSIONALS BY PROF. Antibodies to cytoplasmic signal-recognition particles (SRP) are also found in PM. 3511 A. & DM. signs of rheumatoid arthritis. 3511 cytotoxicity plays a key role. SLE. Raynaud’s phenomenon C. Inclusion Body Myositis D. CD20/MHC-I complex D. Hodgkin lymphoma A. and nematodes (trichinae) may produce focal or diffuse inflammatory myopathy known as parasitic polymyositis. D. Skin rash an increased frequency of ? B. Anti-PM/Scl A. Neck-flexor muscles syndrome are very rare in DM. muscular dystrophy. in PM & IBM. If these muscles are affected. Following are typically absent - rash. and non-Hodgkin lymphoma is increased. CD8/MHC-I complex D. endocrinopathy. Pharyngeal muscles An overlap syndrome occurs in patients with DM who also have manifestations of systemic sclerosis or mixed connective tissue disease. 1934 Which of the following is a feature of dermatomyositis ? Harrison’s 18th Ed. 1933 Which of the following is present in polymyositis ? Harrison’s 18th Ed. May have asymmetric muscle weakness pattern C. CD8/ MHC-I complex is characteristic for histologic diagnosis of PM. THus. Anti-DM/Scl C. Breast cancer 1941 Which of the following parasites can lead to inflammatory myopathy C. while in DM. Interstitial lung disease D. 3511 Harrison’s 18th Ed. trypanosoma). Endocrinopathy A. By contrast. Scleroderma Psychiatric disorders are central nervous system diseases characterized by disturbances in B. and socialization. Intravenous immunoglobulin (IVIg) myositis known as tropical polymyositis or pyomyositis. All of the above A. with onset prior to B. D. and symbolic or imaginative play. and alleles B. C. Mycophenolate mofetil Staphylococcus aureus. Emotion nasogastric suction receiving parenteral hyperalimentation. Mood Disorders PM ? D.Biology of Psychiatric Disorders B. Simvastatin 1952 Alleles of which of the following genes is associated with both ASDs and schizophrenia ? D-Penicillamine & procainamide produce a true myositis resembling PM. FMR1 and TSC1&2 can cause mental retardation without ASDs. AZT causes a mitochondrial Harrison’s 18th Ed. language as used in social communication. or provastatin. Hypophosphatemia C. Plasmapheresis 1943 Acute painless muscle weakness with myoglobinuria may occur Neither plasmapheresis nor leukapheresis appears to be effective in PM & DM. 1 month Mutations in MeCP2. with ? Harrison’s 18th Ed. 2000 rheumatoid arthritis. Schizophrenia 1946 Which of the following drugs produce true myositis resembling C. simvastatin. Rheumatoid arthritis emotion. cognition. Neurexin 1 1947 In PM & DM. motivation. Socialization Harrison’s 18th Ed. Any of the above characterized by disturbances in ? Harrison’s 18th Ed. A. Autism spectrum Disorders (ASD) cytochrome oxidase . All of the above Harrison’s 18th Ed. 12 months Harrison’s 18th Ed. 2004 A. Azathioprine D. B. 3513 A. 803 Neurology MCQ’s FORMCQ’s FOR MEDICAL MEDICAL PROFESSIONALS PROFESSIONALS BY PROF. 3522 In IBM. Primary biliary cirrhosis 1950 DSM-IVTR was released in which year ? D. 3517 C. Streptococcus. Virus A. Substance Use Disorders Harrison’s 18th Ed. 3513 living. B. Toxin B. 3522 myopathy. text revision (DSM-IVTR) was released in year 2000. MeCP2 B. 2002 Harrison’s 18th Ed. Cognition 1944 D-penicillamine is used in which of the following diseases ? C. Parasite 1948 Which of the following is ineffective in PM & DM ? B. Colchicine age 3) with relatively preserved cognitive functioning and language skills have Asperger’s syndrome. Inclusion Body Myositis 1951 Asperger’s syndrome best relates to ? D. C. AJAY MATHUR Cardiology 803 1942 Tropical polymyositis is produced by ? Efficacy of prednisone is noted by objective increase in muscle strength & activities of daily Harrison’s 18th Ed. and primary biliary cirrhosis. 3522 Acute painless muscle weakness with myoglobinuria may occur with prolonged hypokalemia. A. C. 3517 D. Yersinia. Hypomagnesemia 1949 Psychiatric disorders are central nervous system diseases D.negative fibers are found. usually in chronic alcoholics or in patients on A. objective increase in muscle strength occurs how many months after prednisone treatment ? C. Amphotericin B Individuals with autism-like symptoms (delays or abnormal functioning in social interactions. Toxic noninflammatory myopathy is caused by clofibrate. B. especially when combined with cyclosporine or gemfibrozil. lovastatin. A. TSC 1 & 2 Harrison’s 18th Ed. 3514 A. B. which occurs by 3rd month. or anaerobic bacteria may produce a suppurative C. 3523 . DM responds better than PM. 2001     -amyloid deposits on muscle biopsy are found in ? C. Polymyositis Diagnostic and Statistical Manual of Mental Disorders of the American Psychiatric Association. Hypokalemia 390 . 3509 D. 6 months 1953 Major symptom cluster seen in schizophrenia is ? D. FMR1 A. 3522 D-penicillamine chelates copper and is used in Wilson’s disease. It is also used in scleroderma. 3 months of neurexin 1 is associated with both ASDs and schizophrenia. Dermatomyositis 4th edition. Procainamide D. 3512 Table 388-2 D. or hypophosphatemia and hypomagnesemia. C.  -amyloid deposits within vacuolated muscle fibers & abnormal mitochondria with A. Bacteria Harrison’s 18th Ed. All of the above Harrison’s 18th Ed. A. Cerebral hemisphere asymmetry B. ‘Functional’ psychosis in schizophrenia is ? N Engl J Med 2003.804 MCQ’s FOR MEDICAL PROFESSIONALS BY PROF. 3 % Three major symptom clusters seen in schizophrenia are positive (hallucinations & delusions). 1955 The best-established neuropathologic finding in schizophrenia 1961 Which of the following about schizophrenia is false ? is ? Harrison’s 18th Ed. 3542 Harrison’s 18th Ed. negative (blunted affect. Late adolescence D. asocial behavior. Amygdala 1957 Which of the following term was used to address schizophrenia B. Early childhood C. 45 years 1956 Which of the following is related to schizophrenia ? Mostly. None of the above A. AJAY MATHUR Neurology A. Rate of completed suicide is about 10% C.349:1738-49 A.349:1738-49 D. Middle age Positive symptoms cluster seen in schizophrenia responds to current antipsychotic drugs that D. was replaced by Eugen Bleuler who called those closely related diseases characterized not by deterioration of intellect but by splitting of cognitive sides schizophrenia individuals ? of personality from the affective or emotional sides by the name of schizophrenia. Equal prevalence rates in men & women A. It occurs in all ethnic populations with a worldwide lifetime prevalence of ~1 %. Kraepelin E Cortical layers II and III are thinner than normal in schizophrenia. impoverished speech. Schizophrenia is a sporadic or familial mental disorder of yet unknown etiology. Dementia with Lewy bodies C. Enlargement of lateral ventricles D. Reduced number of gyri & sulci C. 2 % D. Peak age of onset is in III decade of life B. Negative A. Anterior & medial nuclei of thalamus in past ? C. a term used by Emil Kraepelin. Negative A. 35 years D. A. None of the above D. cluster seen in schizophrenia ? Harrison’s 18th Ed. Dorsolateral prefrontal cortex (DLPFC) A. All of the above C. 3523 B. Frontotemporal dementia D. Positive 1959 Worldwide lifetime prevalence of schizophrenia is ? Harrison’s 18th Ed. Ventricular enlargement B. D. Dementia praecox 1964 Which of the following cortical layers is thinner than normal in schizophrenia ? D. Kohn R 1965 Which of the following is not a structural abnormality in brain of Dementia praecox. Multiple-personality 1963 Area of the brain that are predominantly and consistently affected C. Cognitive B. 3542 B. All of the above C. Schizophrenia presents typically in late adolescence and early adulthood as a lifelong phenomenon. Old age generally lack efficacy for negative and cognitive symptoms. Maurer D C. people do not get schizophrenia after the age of ? ventricles of the cerebral hemispheres accompanied by a reduction in cortical thickness. and D. 1 % C. people do not get schizophrenia after the age of 45 years. Conversion reaction N Engl J Med 2003. Split personality B. All of the above B. II A. 15 years B. 4 % cognitive symptoms (disabling deficits in working memory & cognitive control of behavior). and diminished motivation).349:1738-49 1958 Out of the following. Absence of glial proliferation . Sachs GS D. Increase in cortical thickness The best-established neuropathologic finding in schizophrenia is enlargement of the lateral 1962 Mostly. N Engl J Med 2003. 25 years Schizophrenia C. Positive Harrison’s 18th Ed. 3523 A. who performed original research in A.349:1738-49 were the first to suggest that schizophrenia is an organic brain disease with significant cognitive deficits. I schizophrenia ? B. Schizophrenia affects males and 1954 Current antipsychotic drugs are efficacious for which symptom females roughly equally. Kraepelin and Bleuler N Engl J Med 2003. 3523 1960 Schizophrenia presents typically in ? A. Cognitive B. IV B. V C. GABA Dopamine receptor subtypes D1 and D5 are excitatory while D2. Lysergic acid diethylamide (LSD) 1973 Dopaminergic fibers in brain arise in which of the following ? D. Reduction in neuropil 1972 Which of the following is a dopamine pathway in the brain ? N Engl J Med 2003. kainate. Striatum propionic acid). Hyperdopaminergia in mesocortical neurons N Engl J Med 2003. Thalamus 1977 Which of the following is an ionotropic glutamate receptor ? Serotonergic neurons in the CNS are found primarily in the midline raphe nuclei. AMPA 1971 Principal site for synthesis of norepinephrine in human brain is ? B.349:1738-49 C. C. nigrostriatal & tuberoinfundibular. D3 A. D2 1968 The most prevalent excitatory transmitter in brain is ? C.349:1738-49 A. D3 and D4 are inhibitory.349:1738-49 D. Decreased volume of Nucleus accumbens A. D3 N Engl J Med 2003. Mesolimbic 1966 Which of the following recreational drugs can mimic symptoms B. mesocortical. pregnant mothers tend to deliver children who have an increased incidence of schizophrenia later in their life. Mesocortical of schizophrenia in healthy individuals ? N Engl J Med 2003. while D2 are subcortical. 2721 Neurotransmitter is -amino-butyric acid (GABA) is mostly inhibitory at the synapses. AJAY MATHUR Cardiology 805 C. Midline raphe nuclei D. D2 1969 Most prevalent inhibitory transmitter in brain is ? N Engl J Med 2003. Nigrostriatal A. Plague epidemic Dopaminergic fibers in brian arise in the ventral tegmental area (VTA).349:1738-49 brain stem from the midbrain to the medulla. Nuclear holocaust in Japan D. This observation stems from the experiences of the “Dutch A. D1 receptors tend to be distributed in cortical regions. Thalamus 1978 AMPA-receptor subunits are derived from which of the following Locus coeruleus is the principal site for synthesis of norepinephrine (NE) that has an excitatory genes ? effect on most of the brain. B.349:1738-49 C. None of the above B.349:1738-49 . Locus coeruleus Ionotropic glutamate receptors (iGluRs) are AMPA (a-amino-3-hydroxy-5-methyl-4-isoxazole C. Glutamate D3 receptors are mainly present in nucleus accumbens while D4 receptors express themselves in frontal cortex. D. A. All of the above B. B. Serotonin D. Amygdala 1967 Which of the following is related to the increased occurrence of schizophrenia ? B. C. N Engl J Med 2003. Kainate N Engl J Med 2003. Glutamate 1975 Which of the following dopamine receptor subtype is most C. Dopamine N Engl J Med 2003. Dopamine 1976 Schizophrenia is characterized by ? Harrison’s 17th Ed. C.349:1738-49 During severe famine. Hypodopaminergia in mesocortical neurons 1970 Serotonergic neurons in the CNS are found primarily in ? B.349:1738-49 Most prevalent transmitter in the human brain is glutamate. All of the above N Engl J Med 2003. A. Nucleus accumbens B. D1 hunger winter” during 1944 – 1945.349:1738-49 C. D. Dutch hunger winter 1974 Which of the following dopamine receptor subtype is excitatory ? N Engl J Med 2003. A. NMDA A. Hippocampus N Engl J Med 2003. Amphetamine D. Ventral tegmental area (VTA) A. Serotonin abundant in nucleus accumbens ? D. Striatum hyperdopaminergia in mesolimbic neurons. D1 B. Tsunami D. 805 Neurology MCQ’s FORMCQ’s FOR MEDICAL MEDICAL PROFESSIONALS PROFESSIONALS BY PROF. It is mostly excitatory at synapses. located in the N Engl J Med 2003. Hypodopaminergia in mesolimbic neurons A. GABA D. D4 B. All of the above B.349:1738-49 D. Phencyclidine (PCP) Dopamine pathways in the brain are mesolimbic.349:1738-49 C. Locus coeruleus Schizophrenia is characterized by hypodopaminergia in mesocortical neurons and C.349:1738-49 C. D4 A. and NMDA. Midline raphe nuclei D. D. negative symptomatology exists in the absence of schizophrenia ? delusions. D-serine 1986 Which of the following is false about the negative symptoms of 1980 Serotonin is produced from ? schizophrenia ? N Engl J Med 2003. 1982 GABA is synthesized from ? N Engl J Med 2003. AJAY MATHUR Neurology A. NR2D genes. Schizoaffective disorder . any relevant medical disease. Glutamic acid symptomatology exists in the absence of delusions. All of the above C. HD. Altered short term memory B. Respond inadequately to drug treatment C. 3543 B. All of the above C. Frontotemporal dementia 1979 Glycine co-agonist essential for glutamatergic transmission is ? N Engl J Med 2003. Paranoid action of glutamic acid decarboxylase (GAD). Atonic D. Negative symptoms carry a poor long-term outcome and a respond inadequately to drug treatment. GluR4 D.349:1738-49 1988 In which of the following subtypes of schizophrenia. Arginine A.349:1738-49 1987 Which of the following is not a schizophrenia subtype ? A. Catatonic D. NMDA receptor subunits are encoded like features ? by NR1. 3543 Catatonic subtype patients have profound changes in motor activity. 3543 C. negative A. Perturbations of perception Harrison’s 18th Ed. AD. or motor disturbance. 3542 A. Alzheimer’s disease B. Perturbations of thinking A. Kainate receptors are derived from GluR5. NR2A. Catatonic C. Delusions C. Bizzare psychosis 1990 Patients with symptoms of schizophrenia and independent periods B. Melanin FTD. Amphetamine Harrison’s 18th Ed. Frequent mood disturbances of mood disturbance are called ? C. Disorganized GABA is synthesized from glutamic acid by removal of alpha-carboxl group from glutamte by the C. 3543 C. negativism & echolalia or echopraxia. temporal-lobe epilepsy. KA2 genes. hallucinations. Residual Harrison’s 18th Ed. GluR7 1985 Which of the following diseases can begin with schizophrenia- AMPA-receptor subunits are derived from GluR1. Disorganized There are no pathognomonic features of schizophrenia. Harrison’s 17th Ed. Develop prior to positive symptoms D. Dementia with Lewy bodies (DLB).349:1738-49 Harrison’s 18th Ed. Four subtypes of schizophrenia are catatonic. Glycine B. Butyric acid Harrison’s 18th Ed. or leukoencephalopathy can begin with schizophrenia-like features. hallucinations. GluR5 Features that preclude the diagnosis of schizophrenia are significant mood symptoms. vascular dementia. None of the above Serotonin is produced by decarboxylation & hydroxylation of L-tryptophan. None of the above B. Residual 1983 Which of the following is a pathognomonic feature of In residual-type subtype of schizophrenia. disorganized. LSD (lysergic acid diethylamide) A. NR2B. L-tryptophan B. 3543 A. NR2C. Residual Hallucinogen LSD (lysergic acid diethylamide) is a 5-HT agonist. D. Histidine C. GluR7 and KA1. D. paranoid. 1981 Which of the following hallucinogen is a 5-HT agonist ? N Engl J Med 2003.806 MCQ’s FOR MEDICAL PROFESSIONALS BY PROF. Huntington’s disease A. C. 3542 A. GluR2. Hypervigilance Harrison’s 18th Ed. Perturbations of language 1989 Echolalia or echopraxia os a feature of which of the following subtypes of schizophrenia ? B. A. GluR6 & hallucinations of schizophrenia are more complex & bizarre than those of dementia. Paranoid 1984 Which of the following is not a typical feature of schizophrenia ? D. GluR3 and GluR4 genes. or motor disturbance ? B. Sialic acid A.349:1738-49 B. Harrison’s 18th Ed. Ketamine B. 2721 A. Serenine D. Negative symptoms develop prior to positive symptoms. Tyrosine C. evidence of substance abuse and drug reaction. Catatonic D. GluR6. Carry a poor long-term outcome B. and residual. Leucine D. Paranoid D. 10 % positive symptoms of schizophrenia ? B. ~ 50 % C. winter birth. 40 % C. Medial temporal lobes (MTL) D. Schizocognitive disorder C. C. Olanzapine. 3543 antipsychotic drug ? Harrison’s 18th Ed. C. 3544 Table 391-14 A. DTNBP1. DAOA. Clonidine 1999 Term neuroleptic in Greek means ? B. 1996 If both parents are affected with schizophrenia. DISC1 B. and ~50% in monozygotic twins. and mild perceptual distortions). to have schizoaffective disorder. D. and procaine derivatives. Aripiprazole D. Quetiapine. Reelin.349:1738-49 C. Amygdala 1993 Which out of the following prescription medications is frequently associated with symptoms of schizophrenia ? Reduction in volume of superior temporal gyrus correlate with positive symptoms of schizophrenia while MTL reductions correlate with memory impairment. Brown–Peterson procedure and the Letter-Number Sequencing (LNS) task are Schizophrenia candidate genes include DISC1. eccentric behavior. RGS4. Aripiprazole and Amisulpride.349:1738-49 C. Perphenazine. 3543 A. 3543 A. 807 Neurology MCQ’s FORMCQ’s FOR MEDICAL MEDICAL PROFESSIONALS PROFESSIONALS BY PROF. Chlorpromazine Risk factors for schizophrenia include urban birth. NRG1 C. AJAY MATHUR Cardiology 807 B. 20 % N Engl J Med 2003. the incidence of schizophrenia is ? Harrison’s 18th Ed. Haloperidol decanoate. To clasp the neuron Most common prescription medications associated with symptoms of schizophrenia are clonidine. Schizophreniform disorder B. ~ 75 % D. are Clozapine. NRG1. Perphenazine B. PANSS C. ~ 100 % First-generation or “typical” antipsychotic agents include Chlorpromazine. Ziprasidone. 30 % A. Reelin D. the risk for offspring is 40%. migration. Amisulpride C. increasing paternal age. Superior temporal gyrus (STG) About 10% of schizophrenic patients commit suicide.349:1738-49 Harrison’s 18th Ed. 20 % . Risperidone. 1997 Which of the following scales is used for severity of clinical symptoms of schizophrenia ? 1991 Which of the following genes is associated with schizophrenia ? N Engl J Med 2003. 80 % Patients with symptoms of schizophrenia and independent periods of mood disturbance are termed If both parents are affected with schizophrenia. C. To straighten the neuron D. Lorazepam N Engl J Med 2003. Harrison’s 17th Ed. Schizopolar disorder D. 2721 2000 Which of the following is not a typical antipsychotic drug ? A. prenatal nutritional deficiency & intrauterine exposure to viral infection. Intrauterine exposure to viral infection. the risk for offspring is ? 2002 Agranulocytosis is a side effect of which of the following Harrison’s 18th Ed. To thin the neuron quinacrine. Harrison’s 18th Ed. ~ 25 % B. Ziprasidone A. Trifluoperazine. BAS B. 3544 Table 391-14 1995 Among monozygotic twins. To energise the neuron 1994 Risk factors for schizophrenia include all except ? Term neuroleptic in Greek means “to clasp the neuron”. 3543 1998 Reduction in volume of which of the following correlate with A. the incidence of schizophrenia is ~10% in dizygotic twins of affected individuals. 2001 Which of the following is not an atypical antipsychotic drug ? Harrison’s 18th Ed. Haloperidol C. Prenatal nutritional deficiency B. 3544 Table 391-14 B. Extrapyramidal Symptom Rating Scale (ESRS) and the Barnes Akathisia Scale (BAS) are used to assess movement disorders that occur with antipsychotic 1992 What percentage of schizophrenic patients commit suicide ? medication of schizophrenia. ESRS A. Risperidone D. Thiothixene and Haloperidol. Thiothixene fetal hypoxia. LNS D. Tizanidine A. Schizotypal and schizoid personality disorders refers to individuals who show a lifetime pattern of social & interpersonal deficits (inability to form close interpersonal relationships. maternal-fetal Rhesus blood-group incompatibility. Summer birth Harrison’s 18th Ed. Reserpine B. Flupentixol decanoate and Risperidone microspheres. 40 % B. Harrison’s 18th Ed. measures of verbal working memory. Second-generation or “atypical” antipsychotic agents Among twins. Hippocampus D. All of the above Positive and Negative Syndrome Scale (PANSS) is used for severity of clinical symptoms of schizophrenia. 60 % D. Increasing paternal age A. D. 3524 A. Intramuscular depot preparations available are Fluphenazine decanoate. Ziprasidone C. 2005 In schizophrenia. Quetiapine Bipolar disorder (episodes of mania & depression) is one of the most heritable of psychiatric illnesses. 3525 use of antipsychotic agents if given early in the syndrome ? A. D. Ziprasidone Mefloquine should not be prescribed to patients with depression. if 2014 Deep brain stimulation (DBS) of which of the following elevates given early in the syndrome. Posttraumatic stress disorder (PTSD) B. Day-night reversal B. 3543 A. and hypertriglyceridemia ? Harrison’s 18th Ed. All of the above In UK. 3543 A. 10% per month D. Seizure disorder C. it is mandatory for all patients receiving Clozapine to register with Clozaril Patient Monitoring 2010 Which of the following anti-malarials should not be prescribed to Services (CPMS) that monitors patient’s hematological profile. if antipsychotic medications are completely 2011 Dysthymia best relates to ? discontinued. the relapse rate is ? Harrison’s 18th Ed. Tongue C. Lumefantrine A. Subgenual area 24 A. In severe cases trunk. Bipolar disorder A. Quetiapine D. Depressive disorder A. Subgenual area 25 . until eventually almost all patients undergo relapse. Quinine Harrison’s 18th Ed. 3525 2008 Which of the following is most commonly involved in tardive A. mood in normal and depressed individuals ? Harrison’s 18th Ed. Myocarditis D. Subgenual area 23 dyskinesia (TD) ? Harrison’s 18th Ed. Insomnia D. 2013 Neurovegetative symptoms best relate to ? 2007 Which of the following may reduce tardive dyskinesia with the Harrison’s 18th Ed. 2003 In UK. Clozapine D. patients of schizophrenia ? Harrison’s 17th Ed. Bipolar disorder B. 3544 B. weight gain & hypertriglyceridemia more than Ziprasidone. Schizophrenia C. Posttraumatic stress disorder (PTSD) C. 2012 Which of the following is one of the most heritable of psychiatric 2006 Use of which of the following is least likely to cause hyperglycemia. 3525 Harrison’s 18th Ed. olanzapine & quetiapine cause hyperglycemia. Hematological profile A. None of the above psychosis. Depressive disorder A. Calcium Vitamin E may reduce abnormal involuntary movements with the use of antipsychotic agents.808 MCQ’s FOR MEDICAL PROFESSIONALS BY PROF. Weight gain B. D. Vitamin E C. Clozapine. 3524 Harrison’s 18th Ed. AJAY MATHUR Neurology A. 1% per month B. schizophrenia. Lenticular opacities C. Schizophrenia D. the relapse rate is 60% within 6 months or 10% per month. 5% per month C. Primaquine C. generalized anxiety disorder. 2723 2004 Which of the following antipsychotic drug is unlikely to increase prolactin levels ? A. Schizophrenia D. 20% per month Mood disorders can be either depressive or bipolar disorders. Limbs C. Depressive disorder Harrison’s 18th Ed. with genetic risk of 80%. Respiratory muscles D. illnesses ? weight gain. Aripiprazole C. Mefloquine B.349:1738-49 Harrison’s 17th Ed. Sleep fragmentation C. Clozaril Patient Monitoring Services (CPMS) monitors patient’s ? 2009 Sleep distrubance in chronic schizophrenia includes ? N Engl J Med 2003. Depressive disorders include the major & minor depressive disorders and dysthymia If antipsychotic medications are completely discontinued in schizophrenia. Trunk B. limbs & respiratory muscles may be affected. 3545 B. lips & tongue. 2723 A. Bipolar disorder B. Vitamin B12 D. Olanzapine D. Chlorpromazine TD comprises of choreiform movements involving mouth. 3544 B. Clozapine C. and seizure disorder. Zinc Symptoms of depression are referred to as neurovegetative symptoms. Clozapine B. Olanzapine B. Subgenual area 26 2020 Which of the following is a serotonin-norepinephrine reuptake inhibitor (SNRI) ? Deep brain stimulation (DBS) of either nucleus accumbens or subgenual area 25 elevates mood Harrison’s 18th Ed. any underlying personality disorder (axis II). Duloxetine C. general psychosocial functioning (axis V). Serotonin-norepinephrine reuptake inhibitor (SNRI) D. Panic disorder B. 3531 Table 391-3 A. General medical condition B. psychosocial and environmental problems (axis IV). 3533 Table 391-6 raphe. Obsessive-compulsive disorder C. Sertraline 2015 General medical condition leading to mental disorders is Fluoxetine. Amoxapine B. Trazodone Agoraphobia occurs commonly in patients with panic disorder. Bupropion C. Axis III B. 3531 Table 391-3 A. because of possible hyperadrenergic effects. 2018 Agoraphobia commonly accompanies ? Harrison’s 18th Ed. Phenelzine D. because of the risk of serotonin syndrome. Venlafaxine 391 . Anxiety is defined as a subjective sense of unease. Presence or absence D. Desvenlafaxine. 3529 A. cholecystokinin tetrapeptide (CCK-4). 3531 Table 391-3 A. Trazodone IV sodium lactate. 3531 Table 391-3 B. Mirtazapine D. All of the above D. Anxiety disorder A. 3531. Phenelzine D. Any underlying personality disorder A. axis III disorders refer to ? D. Amoxapine B. Panic attack B. Tricyclic antidepressants (TCA) The revised fourth edition for use by primary care physicians of the Diagnostic and Statistical Manual (DSM-IV-PC) provides the current multiaxial system of classification. or with TCAs. A. All of the above D. Venlafaxine. Mirtazapine are mixed norepinephrine/serotonin reuptake inhibitors and receptor blockers. 809 Neurology MCQ’s FORMCQ’s FOR MEDICAL MEDICAL PROFESSIONALS PROFESSIONALS BY PROF. classification ? Harrison’s 18th Ed. All of the above of a major mental disorder (axis I). Amitriptyline C. All of the above Anxiety disorders are the most prevalent psychiatric illnesses. sertraline are selective serotonin reuptake inhibitors (SSRIs). A. Cholecystokinin tetrapeptide (CCK-4) C. Presence or absence of a major mental disorder 2022 MAOIs should not be used concomitantly with ? Harrison’s 18th Ed. Selective serotonin reuptake inhibitors (SSRIs) C. Axis I tolerated. 3539 Table 391-3 B. and overall rating of MAOIs should not be used concomitantly with SSRIs. dread. Anxiety A. Psychosocial and environmental problems C. 3529 2021 Which of the following tricyclic antidepressants (TCA) is best A. Nortriptyline D. Intravenous infusion of sodium lactate A. 2019 Which of the following can evoke an attack of panic disorder ? 2025 Extrapyramidal symptoms can appear with the use of ? Harrison’s 18th Ed. Duloxetine. 3529 2024 Which of the following is a monoamine oxidase inhibitor (MAOI) ? Harrison’s 18th Ed. Fluoxetine B. Paroxetine D. paroxetine.Mental Disorders C. 3530 Harrison’s 18th Ed. general medical condition (axis III). It is an acquired irrational fear of being in places where one might feel trapped or unable to escape. alpha2-adrenergic antagonist yohimbine. Imipramine 2016 In DSM-IV-PC. Isocarboxazid. Oxazepam . Carbon dioxide inhalation B. Agents that block serotonin reuptake can prevent attacks. and carbon dioxide inhalation evoke a panic attack in panic disorder patients by activating a 2026 Which of the following anxiolytic drugs has no active metabolite ? pathway involving noradrenergic neurons in locus coeruleus and serotonergic neurons in dorsal Harrison’s 18th Ed. Phenelzine. or foreboding. Doxepin Harrison’s 18th Ed. Bupropion C. All of the above D. Axis II A. 3529 reuptake inhibitors and receptor blocker ? Harrison’s 18th Ed. Obsessive-compulsive disorder C. and denoted by which of the following in the multiaxial system of venlafaxine is a serotonin-norepinephrine reuptake inhibitor (SNRI). AJAY MATHUR Cardiology 809 D. especially by elderly ? Harrison’s 18th Ed. Venlafaxine B. and Transdermal selegiline are MAOI. 3530 in normal and depressed individuals. Tranylcypromine. Axis IV C. 2017 Subjective sense of unease best relates to ? 2023 Which of the following is a mixed norepinephrine/serotonin Harrison’s 18th Ed. Whisky B. Harrison’s 18th Ed. 4 . C. 3532 A. 3 .4 weeks A. C. does not interact with benzodiazepine receptors or alcohol. Clonazepam A. neuroleptic or benzodiazepine is beneficial and in severe cases deep brain stimulation is effective. Thoughts D. flurazepam. and Triazolam have no active metabolites. Buspirone C. avoidance of cheese and wine. Wine A. Harrison’s 18th Ed. Temazepam 2032 Obsessive-compulsive disorder (OCD). Alprazolam 2035 Which of the following is useful in the treatment of obsessive- C. may reduce fasting plasma glucose. B. obsession pertains to ? Harrison’s 18th Ed. Fluoxetine off shorter-acting benzodiazepines. chlordiazepoxide. 2 . 1 . Tourette’s Syndrome D. TCAs 2031 Which of the following anticonvulsants with GABAergic properties C. Fluvoxamine 2030 Which of the following is a nonbenzodiazepine anxiolytic agent ? D. Evening . and clonazepam. Shorter-acting benzodiazepines are alprazolam and oxazepam. Caudate nucleus D. & globus pallidus. B. C. and has no abuse or A. Actions Oxazepam. Divalproex Harrison’s 18th Ed. All of the above Harrison’s 18th Ed. Temazepam. 3532 nucleus. Amygdala Benzodiazepines should not be prescribed for >4 . pregabalin. MAOI disinhibition potential. does not produce tolerance or dependence. fluoxetine. Afternoon and divalproex are beneficial in anxiety-related syndromes. oxcarbazepine. Clonazepam. Alprazolam. Clonazepam Harrison’s 18th Ed. Morning Anticonvulsants with GABAergic properties like gabapentin. Diazepam. Behaviour D. Hippocampus D. B. Diazepam skill learning. 3536 Buspirone is a nonbenzodiazepine anxiolytic agent. Gabapentin carbohydrate craving. 3533 MAOIs can induce hypoglycemia and weight gain. Caudate nucleus is involved in acquisition & maintenance of habit & A. Wilson’s Disease D. augmentation with buspirone. It is nonsedating. Myoclonus Patients taking monoamine oxidase inhibitors (MAOIs) need to maintain a low-tyramine diet i. Clonazepam carbohydrate craving ? Harrison’s 18th Ed. Coffee following clinical conditions ? Harrison’s 18th Ed. fluvoxamine. 3535 C. All of the above 2027 Which of the following is to be avoided when monoamine oxidase Obsessive-compulsive disorder (OCD) is characterized by obsessive thoughts and compulsive inhibitors (MAOIs) are used ? behaviors that impair everyday functioning.e. Clomipramine Longer-acting benzodiazepines are diazepam. Pregabalin 2037 In patients with depression. like MAOIs. AJAY MATHUR Neurology B. It is more difficult to taper patients B. Flurazepam 2036 Which of the following can produce hyperglycemia and D. Both are more common in many weeks to avoid risk of abuse and dependence ? males and in first-born children. 3536 D. and Buspirone have active metabolites. and sertraline are useful in the treatment of OCD. But. In A. caudate Harrison’s 18th Ed. Lorazepam. Tobacco 2033 Obsessive-compulsive disorder (OCD) is related to which of the B. tiagabine. 3530 A. All of the above A.6 weeks because of the development of tolerance and the risk of abuse and dependence. 3535 C. while TCAs can produce hyperglycemia and A. mood is worse in ? C. All of the above B. Parkinson’s disease 2028 Benzodiazepines should not be prescribed for more than how Family studies show an aggregation of OCD with Tourette’s disorder. C. SSRIs. 3533 Clomipramine. Chlordiazepoxide. All of the above Harrison’s 18th Ed. SSRIs is beneficial in anxiety-related syndromes ? D. Mamillary bodies 2029 Which of the following is not a long-acting benzodiazepine ? Anatomical localization of obsessive-compulsive behavior is to orbital frontal cortex. Flurazepam compulsive disorder (OCD) ? D. B.2 weeks 2034 Which of the following part of the brain is responsible for B. 3535 A. Diazepam treatment-resistant cases. requires several weeks to take effect and thrice-daily dosing. 3535 C. C. B.3 weeks acquisition and maintenance of habit and skill learning ? Harrison’s 18th Ed.810 MCQ’s FOR MEDICAL PROFESSIONALS BY PROF.6 weeks B. decreased inhibitory response of glucocorticoids to dexamethasone and blunted response of D. 811 Neurology MCQ’s FORMCQ’s FOR MEDICAL MEDICAL PROFESSIONALS PROFESSIONALS BY PROF. Metafluoxetine B. AJAY MATHUR Cardiology 811 D. which also normalizes with antidepressant treatment. Hyperpyrexia weeks. 0 to 60 half-life. TCA N Engl J Med 2011. 3537 hypomania ? A. increase in adrenal size. Agitation depression ? Harrison’s 18th Ed. None of the above B.364:51-9 B. D. Increase in adrenal size 2045 Response to treatment of depression should be evaluated after how many months ? C. panic. hyperpyrexia. 3539 dexamethasone A. and potentially death. months. Increase in REM density A. Deep brain stimulation (DBS) 2039 Neuroendocrine abnormality that reflect neurovegetative signs & D. Isofluoxetine episodes of either depression or mania in a given year ? B. One or more D. No current compound combines rapid onset of action. an increase in REM density. Decrease in REM sleep N Engl J Med 2011. 3538 2048 Term rapid cycling is used for patients who have how many A. Myoclonus . 2040 Sleep abnormality in patients with major depression is ? 2046 Which of the following distinguish bipolar I mania from bipolar II Harrison’s 18th Ed. 3537 (VNS) and Deep brain stimulation (DBS) are useful in managing treatment resistant depression. Vagus nerve stimulation Harrison’s 18th Ed. 2042 Principal active metabolite of Fluoxetine is ? Harrison’s 18th Ed. or agitation significantly increases near-term suicidal risk in depressed patients. All of the above C. 0 to 20 D. Three or more Principal active metabolite of Fluoxetine is Norfluoxetine. Night B. MAOI A. Norfluoxetine A. Hypotension 2038 Presence of which of the following significantly increases near. or marked 2041 Which of the following is an ideal antidepressant ? impairment) from bipolar II hypomania. Transcranial magnetic stimulation (TMS). a meaningful relationship between dose & blood level.364:51-9 C. Major depression is also associated with an upregulation Regardless of the modality of treatment for depression. 3538 mania in a given year. Abdominal cramping Major depression is defined as depressed mood on a daily basis for a minimum duration of 2 C. Severity of elevated mood Patients with major depression show a decrease in rapid eye movement (REM) sleep onset (REM D. 3538 A. MAOIs used concomitantly with SSRIs can lead to serotonin syndrome that results from term suicidal risk in depressed patients ? hyperstimulation of brainstem 5HT 1A receptors and is characterized by myoclonus. C. 3539 C. The greater severity of elevated mood and associated functional disability distinguish bipolar I mania (which is characterized by psychosis. Anxiety 2044 Which of the following is useful in managing treatment-resistant B. Panic A. Transcranial magnetic stimulation (TMS) D. D. Decreased inhibitory response of glucocorticoids to Harrison’s 18th Ed. the need for urgent care or hospitalization. A. 1 month D. Voluntarily produced physical symptoms of illness D. 2 month Neuroendocrine abnormalities that reflect the neurovegetative signs & symptoms of depression C. and. Harrison’s 18th Ed. All of the above B. 3536 A. 0 to 80 Montgomery and Åsberg Depression Rating Scale [MADRS] ranges from 0 to 60. Two or more C. 3540 C. and safety in overdose. Intentionally produced deficit symptoms latency). SSRI 2047 Montgomery and Åsberg Depression Rating Scale (MADRS) ranges from ? B. Decrease in stage IV delta slow-wave sleep B. All of the above symptoms of depression is ? Electroconvulsive therapy. a low side effect profile. All of the above B. abdominal cramping. 3 month include increased cortisol and corticotropin-releasing hormone (CRH) secretion. Four or more 2043 Which of the following is false about serotonin syndrome ? Term rapid cycling is used for patients who have four or more episodes of either depression or Harrison’s 18th Ed. 0 to 40 There is no ideal antidepressant. a decrease in stage IV delta slow-wave sleep. hypertension. response should be evaluated after 2 of proinflammatory cytokines. Parfluoxetine Harrison’s 18th Ed. Vagus nerve stimulation (VNS) Presence of anxiety. A. minimal interaction with other drugs. Increased cortisol & CRH secretion B. Suicidal tendancy C. Harrison’s 18th Ed. D. Patients with depression often notice a diurnal variation in mood. Antidepressant treatment normalises these abnormalities. agitation. 6 month thyroid-stimulating hormone (TSH) level to infusion of thyroid-releasing hormone (TRH). worse in morning hours. moderate C. 3540 Table 391-11 D.4 µg/mL B. or severe factitious illness for a B. Lithium carbonate Formal diagnostic criteria for somatization disorder requires the recording of at least four pain. skin eruptions. 0. A. C. & 1 pseudoneurologic symptom depressed phase ? C. 3540 Table 391-11 D. Sodium valproate gastrointestinal.350 µg/mL 2059 Which of the following is required for the diagnosis of borderline D. 4 pain.525 µg/mL personality disorder ? N Engl J Med 2011. Weight loss In conversion disorder. A. 2 GI. one sexual. chronic. which of the following is true to fulfill A. 1 sexual. weight gain. 200 . Intentionally produced deficit symptoms mania is ? Harrison’s 18th Ed. 2 GI.42 µg/mL 2060 Most distinctive characteristic of patients with BPD is ? N Engl J Med 2011. 3542 2055 In somatization disorder. 3541 Table 391-13 C.1.364:2037-42 2053 Therapeutic blood levels of carbamazepine is ? Harrison’s 18th Ed. Inflated self-esteem or grandiosity C.2 meq / L Harrison’s 18th Ed. lithium is initiated at 300 mg bid or tid. deficit symptoms are not intentionally produced or simulated and involve B. Affective dysregulation C. C. C. Conversion disorder Harrison’s 18th Ed. Hypochondriasis 2052 Therapeutic blood levels of valproic acid is ? C.4 .8 . All of the above A.2 meq/L. Sodium valproate D. 2 sexual. nausea. Factitious disorder 3 days to achieve blood levels of 0.12 µg/mL C. 50 . 2 GI. Harrison’s 18th Ed. Interpersonal hypersensitivity A. 1 sexual. AJAY MATHUR Neurology 2049 In bipolar disorders.225 µg/mL desire for some external reward such as a narcotic medication or disability reimbursement. Valproic acid is better for patients who experience rapid cycling (> 4 episodes a year) or who 2061 Which of the following is not a feature of personality disorders ? present with a mixed or dysphoric mania. diarrhea. 380 . Dose is titrated every 2 . Characteristic patterns of thinking & feeling the diagnostic criteria ? B.22 µg/mL D. 12 . Hypersensitivity to rejection B. 2. Decreased need for sleep A. and one pseudoneurologic symptom. 4 . 100 . Significant functional impairment A. Simulated deficit symptoms A. as is lamotrigine in the depressed phase. All of the above D. Sensory 2050 Which of the following is not a side effect of lithium therapy ? C. two B. B. Olanzapine D. which of the following is effective in the B. Impulsivity B. & 2 pseudoneurologic symptom Harrison’s 18th Ed. deficit symptoms involve ? Harrison’s 18th Ed. 0. Polyuria 2057 Which of the following best relates to hypochondriasis ? Harrison’s 18th Ed. & 2 pseudoneurologic symptom . 3540 Table 391-11 A. Lithium carbonate C. patient consciously & voluntarily produces physical symptoms of illness.3.5 .2 meq / L 2058 Munchausen’s syndrome best relates to ? D. edema. 2 pain. & 1 pseudoneurologic symptom A. 3 pain.364:2037-42 2054 Which of the following drugs is appropriate for patients who experience rapid cycling ? A. 3 pain.125 µg/mL Munchausen’s syndrome is characterized by dramatic. 3540 D.8 meq / L In factitious disorder (malingering). 1.812 MCQ’s FOR MEDICAL PROFESSIONALS BY PROF. Essential feature in hypochondriasis is a belief of serious medical illness that persists B. Belief of serious medical illness 2051 Therapeutic blood levels of lithium in the treatment of acute C. All of the above A.4 .2. Relative flexibility Harrison’s 18th Ed. polyuria. 3542 Lithium carbonate is the mainstay of treatment in bipolar disorder.2 meq / L despite reassurance & appropriate medical evaluation. 1 . 25 . 3541 B. C. alopecia. 2 GI. Voluntarily produced physical symptoms of illness B. Psychological Harrison’s 18th Ed. 3542 In the treatment of acute mania. Lamotrigine their fearful preoccupation with expected abandonment. Olanzapine Most distinctive characteristics of patients with BPD are their hypersensitivity to rejection and D. 3540 D. A. 3542 Side effects of lithium include gastrointestinal discomfort. Lamotrigine 2056 In conversion disorder.1.8 . 2 sexual. Sodium valproate & olanzapine are equally effective in acute mania. Antithyroid effect D. Excessive involvement in pleasurable activities Harrison’s 18th Ed.0. Motor B. and antithyroid effect. Alopecia motor or sensory function & psychological factors that initiate or exacerbate medical presentation. Subjective distress A. N Engl J Med 2005. 3539.353:1819-34 B. Hippocampus D. excessively emotional. Overactivity of acetylcholine. Anterior and posterior cingulate cortex C. dopamine & GABA D.353:1819-34 B.353:1819-34 A. Suicide Personality disorders are characteristic patterns of thinking. Depression with melancholic features A.353:1819-34 B. Heart disease are relatively inflexible & cause significant functional impairment or subjective distress. Duloxetine B. D. Anxiety disorders C. Cluster B A. Generalized weakness D. Any of the above D. histrionic & narcissistic types (impulsive behavior. 2063 Paranoid personality type is grouped in which personality cluster and substance P disorder ? Harrison’s 18th Ed. All of the above disorder ? Harrison’s 18th Ed. Cluster C B. 50 system ? N Engl J Med 2005. Cluster C C. Epinephrine breakdown A. All of the above C. 813 Neurology MCQ’s FORMCQ’s FOR MEDICAL MEDICAL PROFESSIONALS PROFESSIONALS BY PROF. 3542 2070 Disordered neurocircuitry in depression is observed in which of A. Any of the above C. which of the following statements is true ? B. Absence of sweating 2075 Mirtazapine blocks ? D. Ventral.353:1819-34 C. schizoid. Any of the above C. 3542 2069 Which of the following about depression is false ? N Engl J Med 2005. Schizophrenic disorders D. Histamine H1 receptors . Depression in which physical symptoms or pain are prominent B. 3542 2071 Disordered neurocircuitry in depression is observed in which of A. 30 D. Cluster A A. Alpha2-adrenergic autoreceptors 2068 As regards risk of problems in depressed patients. 2072 Tricyclic antidepressants have greater efficacy than SSRIs in ? compulsive personality types. Insula 2064 Antisocial personality type is grouped in which personality cluster D.353:1819-34 2066 Depressive episodes that occur in conjunction with manic episodes A. Medial thalamus Personality disorders are grouped into 3 overlapping clusters. feeling & interpersonal behavior that B. All of the above Cluster B disorders include antisocial. medial. erratic). Stroke 2062 Obsessive-compulsive personality types is grouped in which D. Difficulty in swallowing Harrison’s 18th Ed. Bupropion are called ? N Engl J Med 2005.353:1819-34 A. Cluster B A. Cluster A includes paranoid (unjustified pervasive mistrust & suspiciousness). dependent & obsessive. Milnacipran C.353:1819-34 B. Dopamine breakdown B. Deficiencies of serotonin. Cluster C includes avoidant.353:1819-34 A. Severe major depressive disorder 2065 In Mini Mental Status Examination. All of the above C. Major depressive disorders N Engl J Med 2005. Higher levels of N-acetyl aspartate C. norepinephrine. Persistent. N Engl J Med 2005. and dorsolateral prefrontal cortex D. Venlafaxine A. Cluster C B. Cluster A the following structures ? N Engl J Med 2005. AJAY MATHUR Cardiology 813 D. corticotropin-releasing factor. 20 C. Bipolar disorders 2074 Monoamine oxidase (MAO) is the enzyme involved in ? D. and schizotypal personality disorders. Elevated cortisol levels B. Cluster B B. All of the above personality cluster disorder ? Harrison’s 18th Ed. Cluster A the following structures ? N Engl J Med 2005. C. Norepinephrine breakdown 2067 “Dysthymia” is the term used for ? N Engl J Med 2005. Ventral striatum C. residual depressive symptoms C. borderline. 40 2073 Which of the following drugs has no direct action on the serotonin D. scoring is done out of ? Harrison’s 16th Ed. 2397 B.353:1819-34 A. Serotonin 5-HT2A and 5-HT3 receptors N Engl J Med 2005. 3540 2081 Which Tricyclic antidepressant carries the greatest risk in overdose ? A. B. Anticholesterolemic drugs 2085 Which of the following SSRIs does not inhibit one or more C. Side effects include polyuria & weight gain contraindicated in patients with serious cardiovascular risk factors. Paroxetine B. B. Severe major depressive disorder with psychotic features All SSRIs impair sexual function. C. Impulsivity and impairment in judgment 2080 Which amino acid is the precursor of serotonin ? C. anticholesterolemics.814 MCQ’s FOR MEDICAL PROFESSIONALS BY PROF. Antimicrobials With the exception of citalopram & escitalopram. Amitriptyline Harrison’s 18th Ed. antiparkinsonian medications. or by treatment with amantadine. It increases noradrenergic & serotonergic neurotransmission through a blockade of central alpha 2-adrenergic receptors and postsynaptic 5HT 2 and 5HT3 receptors. Antiarrhythmic drugs cytochrome P450 enzymes ? D. Medication resistance 2084 Which of the following about Mirtazapine is false ? D. Transient blindness A. 3538 D. All of the above 2086 Mania in its pure form is associated with ? Iatrogenic depression can be induced by antihypertensives. 3538 Mirtazapine is a TCA. B. each of the SSRIs inhibits one or more cytochrome C. D. decreased need for sleep. Olanzapine B. weekend B. systemic analgesics. Obsession for cleanliness Harrison’s 18th Ed. 3540 Table 391-11. Imipramine D. 3539 A. 3540 Overdoses of tricyclic agents can be lethal.353:1819-34 D. and anticonvulsants. Sexual dysfunction can be ameliorated by lowering dose. All of the above C. Sodium valproate A. Decreased need for sleep Harrison’s 16th Ed. 3536 C. Buspirone N Engl J Med 2005. which in its pure form is associated with increased psychomotor activity. All of the above B. Lithium exerts an antithyroid effect . C. Tryptophan D. Carbamazepine/Oxcarbazepine & Lamotrigine. or bupropion. 2554 A. Tetracyclic antidepressant 2078 Depressive symptoms can be caused by which of the following group of drugs ? B. Lithium Akathisia refers to an inner sense of restlessness and anxiety. Leucine Bipolar disorder patients suffer from recurrent attacks of mania. All of the above A. Strongly antihistaminic A. B. TCAs are A. Systemic analgesics P450 enzymes. bethanechol. It is A. buspirone. Inner sense of restlessness and anxiety 2076 Which of the following are classified as mood stabilizers ? C. D. Lithium carbonate Harrison’s 18th Ed. Increased psychomotor activity B. Sertraline 2079 Iatrogenic depression can be caused by which of the following group of drugs ? B. 3539 A. 3536 C.353:1819-34 D. Desipramine C. All of the above A. Arginine 2087 Drug for treatment of bipolar disorders include ? Harrison’s 18th Ed. Does not inhibit cytochrome P450 enzymes B. Glucocorticoids D. Fluoxetine A. AJAY MATHUR Neurology D. A. Valproic Acid. excessive social extroversion. All of the above Harrison’s 18th Ed. Antihypertensive drugs D. 3539 glucocorticoids. Lamotrigine C. Divalproex 2083 SSRI induced sexual dysfunction can be ameliorated by ? Harrison’s 18th Ed. N Engl J Med 2005. Phobias for high pitched sounds also strongly antihistaminic and may produce sedation. and grandiose irritable mood. antiarrhythmics. All of the above Harrison’s 18th Ed. Isoleucine impulsivity and impairment in judgment. Increases noradrenergic & serotonergic neurotransmission Harrison’s 18th Ed. antimicrobials. 3538 B. Bethanechol 2077 ECT can be a first-line treatment for patients who have ? C. Citalopram Harrison’s 18th Ed. All of the above 2082 Akathisia refers to ? Harrison’s 18th Ed. Harrison’s 18th Ed. with desipramine carrying the greatest risk. Amantadine Mood Stabilizers include Lithium. Nortriptyline 2088 Which of the following about lithium therapy is false ? D. Psychomotor retardation drug holidays. Decreased rates of myocardial infarction Harrison’s 18th Ed. None of the above gallstones and vascular & Alzheimer’s dementias. Sweat A. or a poorly tolerated medication supervenes in a mildly demented patient 2098 Alcohol is devoid of which of the following ? B. 2724 D. All of the above Alcohol produces modest effects on folate (folacin or folic acid). Abulia C. All of the above B. All of the above A. Mouth and esophagus Lithium also has a prophylactic effect in prevention of recurrent mania and. 3546 D.Alcohol and Alcoholism D. Increase in serum lactate D. Akathesia Two pathways operate to metabolise alcohol in body.moderate increase in serum lactate & increased  -hydroxybutyrate/lactate ratio. which is then rapidly destroyed by 2092 Beclouded dementia refers to ? aldehyde dehydrogenase (ALDH) in cytosol and mitochondria to either Acetyl CoA or Acetate. 3546 D. 3546 Figure 392-1 Harrison’s 16th Ed. Pyridoxine (B6) C. Alcohol ketoacidosis patients show an increase in serum ketones with mild increase in glucose but a large anion gap. None of the above 2097 In liver. Lungs 2090 Mild form of akinetic mutism is called ? B. D. Abulia Between 2% and 10% of ethanol is excreted directly through lungs. urine. Syncope Harrison’s 18th Ed. Proteins C. Transient confusion & drowsiness with febrile infection Harrison’s 18th Ed. effects ? Harrison’s 17th Ed. Adducted thumbs A. from stomach & large bowel in modest amounts. Excreted mainly by kidneys A. A. alcohol can have which of the following benficial nicotinic acid (niacin. B. First pathway (~80%) occurs in cell cytosol where alcohol dehydrogenase (ADH) produces acetaldehyde. Nicotinic acid (niacin. thiamine (B 1). pyridoxine (B 6). stroke. a mild . Agnosia D. Acute medical problem. Mental retardation Alcohol is devoid of nutrients such as minerals. Greater part C. B3) 392 . 1625 D. 815 Neurology MCQ’s FORMCQ’s FOR MEDICAL MEDICAL PROFESSIONALS PROFESSIONALS BY PROF. AJAY MATHUR Cardiology 815 C. Alcohol dehydrogenase (ADH) A. 2724 2100 Which of the following is false about alcohol ketoacidosis ? A. No prophylactic effect in prevention of recurrent mania 2095 The major site of absorption of alcohol is ? Harrison’s 17th Ed. Aphasia 2099 Alcohol affects levels of which of the following vitamins ? C. 3546 C. 1625 Second pathway (~20%) is microsomal ethanol-oxidizing system or MEOS in smooth endoplasmic reticulum. proteins and vitamins. Catatonia B. 2094 At low doses. in prevention of recurrent depression. 1625 C. Decreased rates of gallstones B. Harrison’s 16th Ed. Agnosia D. Aldehyde dehydrogenase (ALDH) B. or sweat. Large bowel A. Minerals D. Decreased rates of stroke A. Microsomal ethanol-oxidizing system (MEOS) C. All of the above D. Awake but unresponsive state A. Increase in -hydroxybutyrate / lactate ratio Beneficial effects of low doses of alcohol are decreased rates of myocardial infarction. 3546 B. Awake but cannot produce speech B. . Urine Harrison’s 16th Ed. which of the following is responsible for metabolism of 2091 Hypomobile and mute syndrome associated with a major psychosis alcohol ? is ? Harrison’s 18th Ed. 1625 A. Vitamin A B. Chapter 359 D. Stomach C. B. Akathesia is metabolized to acetaldehyde in liver. D. B. to a lesser extent. Damage of the frontal lobes C. Vitamins 2093 MASA syndrome includes all except ? Harrison’s 15th Ed. Damage of the medial thalamic nuclei Alcohol is absorbed from mucous membranes of mouth & esophagus in small amounts. B 3) and vitamin A levels. The major site of absorption is the proximal portion of small intestine. Proximal small intestine 2089 Akinetic mutism can be caused by ? Harrison’s 16th Ed. All of the above A. Large anion gap C. Hydrocephalus 2096 Alcohol is excreted directly through which of the following ? Harrison’s 18th Ed. All of the above C. 3546 Alcohol impairs gluconeogenesis in liver. Dopamine 2111 Korsakoff’s syndrome in alcoholics is predisposed by the D. 2102 “Legal intoxication” in USA requires a blood alcohol concentration of at least ? 2108 Which of the following occurs in chronic alcoholism ? Harrison’s 18th Ed. Alcohol tolerance involves compensatory mechanisms like metabolic or pharmacokinetic tolerance. 3547 A. Encephalopathy D. Receptors B. Dopamine & opiate systems are involved with reinforcement. 3546 Harrison’s 18th Ed. 40 mg/dL A. Breast cancer in women Alcohol acutely enhances actions at  -aminobutyric acid A (GABA A) receptors and inhibits N- methyl-D-aspartate (NMDA) receptors. All of the above “Legal intoxication” in USA requires a blood alcohol concentration of at least 80 . Ophthalmoparesis B. Ataxia C. -aminobutyric acid (GABA) C. Alcoholics may experience episodes of temporary anterograde amnesia. increases lactate production and decreases oxidation A. 80 mg/dL B. 120 mg/dL C. ~ 30 mg/dL C. sustained use of alcohol and potential B. intoxication & many symptoms of alcohol withdrawal.100 mg/dL. 3547 dopamine. Harrison’s 18th Ed. ataxia & encephalopathy. All of the above deficiency of ? Neurochemical systems in brain influenced by alcohol are g-aminobutyric acid (GABA). 3547 2104 Alcohol affects which of the following neurochemical systems in the brain ? A. C. reduced rapid eye movement (REM) and deep sleep. Repeated exposure to ethanol leads to alcohol-induced hepatitis. Increases oxidation of fatty acids 2106 Which of the following is false about actions of alcohol ? D. Caudate nucleus 2112 Alcohol can cause all except which of the following ? D. receptors & transporters. A. Acetylketolase deficiency sedation. 3547 Harrison’s 18th Ed. Nucleus basalis of Meynert includes retrograde & anterograde amnesia. Impairs gluconeogenesis in liver Ethanol inhibits inhibitory neurons in midbrain ventral tegmental area (VTA) leading to increased dopamine release in nucleus accumbens. some aspects of craving. 3547 A. All of the above C. AJAY MATHUR Neurology 2101 One typical alcohol drink produces blood levels of ? 2107 Compensatory mechanism in alcohol tolerance is ? Harrison’s 18th Ed. ~ 40 mg/dL D. Increases serotonin actions Harrison’s 18th Ed. Korsakoff’s syndrome A.359:715-21 Wernicke’s syndrome refers to ophthalmoparesis. Inhibition of uptake of adenosine 2113 Alcohol increases the risk of which of the following cancer ? D.359:715-21 B. Exacerbation of sleep apnea D. Glutamate D. Both are due to thiamine deficiency. Tryptophan B. 2110 Korsakoff’s syndrome in alcoholics is due ot the deficiency of ? Harrison’s 18th Ed. GABA and glutamate are involved with alcohol stimulation. Metabolic or pharmacokinetic tolerance B. Transporters C. Paraketolase deficiency relapse after prolonged abstinence in dependent person. Deficiency in REM sleep C. All of the above Harrison’s 18th Ed. Alcohol inhibits GABAA receptors of fatty acids leading to increased fat accumulation in liver cells. Blockade of dopamine in nucleus accumbens can B. Behavioral tolerance D. and opiate systems. perivenular sclerosis and cirrhosis. All of the above Blood values of ~20 mg/dL result from ingestion of one typical drink. B. 160 mg/dL D. 3548 A. B. Semiketolase deficiency N Engl J Med 2008. especially in those with transketolase deficiency. Oral and esophageal cancer . Nucleus accumbens C. Increases lactate production terminate rewarding effects of addictive drugs. Methionine N Engl J Med 2008. ~ 20 mg/dL B. Transketolase deficiency 2105 Which of the following is related to alcohol-use disorders ? D. Thiamine A. 2103 Intoxicating effect of alcohol is due to its action on ? Harrison’s 18th Ed. Alcohol inhibits NMDA receptors C. 3546 2109 Which of the following are features of Wernicke’s syndrome ? Harrison’s 18th Ed. All of the above Intoxicating effect of alcohol is due to its action on several neurotransmitters. exacerbated sleep apnea & prominent and disturbing dreams. C.816 MCQ’s FOR MEDICAL PROFESSIONALS BY PROF. Increases fat accumulation in liver cells Harrison’s 18th Ed. reward. glutamate. Episode of temporary anterograde amnesia B. All of the above D. cellular or pharmacodynamic tolerance and behavioral tolerance. 3547 A. Cellular or pharmacodynamic tolerance C. Neurotransmitters A. ~ 10 mg/dL A. B. 3548 A. 3549 B. Mental confusion B. and some modest cognitive deficits. Increase in cortisol levels B. 2729 Harrison’s 18th Ed.150 mg/day. Disulfiram (250 mg/day) is an ALDH inhibitor. aberrant palmar confusion. D. small teeth with faulty enamel. 3-methoxymorphine C. Fluctuating levels of consciousness Fetal alcohol syndrome can include facial changes with epicanthal eye folds. 2116 Features of ‘Fetal alcohol syndrome’ include all except ? 2122 Delirium in delirium tremens includes all except ? Harrison’s 18th Ed. B. Small teeth with faulty enamel Alcohol withdrawal symptoms begin within 5 . cardiac atrial or ventricular septal defects. 3549 Harrison’s 18th Ed. All of the above 2120 Blood tests useful in identifying individuals consuming six or Alcohol consumption increases risk of breast cancer (in women). Harrison’s 18th Ed. 3552 A. Inhibition of vasopressin secretion C. Decrease in serum triiodothyronine (T3) D. more standard drinks per day include ? Harrison’s 18th Ed. Finding of >25 % decrease in heart rarte at times of leisure 8 or more drinks per day are  -glutamyl transferase (GGT) (>35 U). 1-2 Harrison’s 18th Ed. 3549 A.Opioid Drug Abuse and D. -glutamyl transferase (GGT) Harrison’s 18th Ed. Serum uric acid (>7 mg/dL) B. Other drugs under investigation are selective 5-HT3 receptor antagonist ondansetron. 3550 2114 “Holiday heart” refers to ? A. Finding of >10 % decrease in SBP at times of leisure (CDT) (>20 U/L). 2-methoxymorphine B. can occur after a drinking 2121 Alcohol withdrawal symptoms peak in intensity on day ? binge in individuals with no other evidence of heart disease (holiday heart syndrome”). poorly formed ear Delirium tremens (DTs) is a state of intense acute alcohol withdrawal. AJAY MATHUR Cardiology 817 C. Disulfuram D. agitation. inhibition of . Naltrexone B. Paroxysmal tachycardia after a drinking binge in individuals D. nalmafene (opioid receptor antagonist) and cannabinol receptor antagonist ramonibant. Agitation C. 1-methoxymorphine A. All of the above Naltrexone is a mu opioid receptor antagonist given in doses of 50 . a lower IQ. Low birth weight topiramate. Hyperactive behavior 393 . 2-3 A. 817 Neurology MCQ’s FORMCQ’s FOR MEDICAL MEDICAL PROFESSIONALS PROFESSIONALS BY PROF. Polydactyly C. Rectal cancer vasopressin secretion at rising blood alcohol concentrations & enhanced secretion at falling blood alcohol concentrations. All of the above Dependence Features of fetal alcohol spectrum disorder (FASD) include low birth weight. It produces Harrison’s 18th Ed. 3549 an unpleasant reaction in presence of alcohol due to rapidly rising blood levels of acetaldehyde. Epicanthal eye folds C. 3551 A. Lower IQ C. Carbohydrate-deficient transferrin (CDT) A. Dilation of all heart chambers in chronic alcoholics C. Acamprosate in doses of 2 gm/day disorder (FASD) ? inhibits the actions of NMDA receptors. and tachypnoea). All of the above without heart disease Laboratory tests that are likely to be abnormal in persons with regular alcohol consumption of 6 to C. anticonvulsant A. overactivity (tachycardia. Aberrant palmar crease B. and fluctuating levels of consciousness) associated with a tremor & autonomic crease & limitation in joint movement. Poorly formed concha D. and microcephaly with mental retardation. hyperactive behavior.10 hours of decreasing ethanol intake. All of the above D. Ellis-van Creveld syndrome A. and improve by day 4 or 5. 3551 2115 Features of ‘Fetal alcohol syndrome’ include all except ? A. All of the above Codeine chemically is 3-methoxymorphine. Trisomy 13 (D) C. Atrial or ventricular arrhythmias. reversible decrease in serum T 4 and T 3. Microglossia intensity on day 2 or 3. It includes delirium (mental concha. Hormonal changes during heavy drinking include an increase in cortisol levels. 2117 Polydactyly is a feature of ? 2123 Medication helpful in alcoholic rehabilitation include ? Harrison’s 16th Ed. 4-5 C. 3-4 B. 3548 B. Hallucinations D. 4-methoxymorphine D. especially paroxysmal tachycardia. 2124 The pharmacological term for codeine is ? 2119 Hormonal changes during heavy drinking include ? Harrison’s 17th Ed. Atrial or ventricular septal defect A. Laurence Moon Biedl Bardet syndrome B. peak in D. high-normal MCVs (>=91 µm 3) and serum uric acid (>7 mg/dL). It decreases 2118 Which of the following is a feature of fetal alcohol spectrum activity in dopamine-rich ventral tegmental reward system. Acamprosate C. carbohydrate-deficient transferrin D. Microcephaly with mental retardation D. 1382 Harrison’s 18th Ed. oral. esophageal and rectal cancers. hypertension. buprenorphine. Naltrexone C. CMT1x C. Propoxyphene Harrison’s 17th Ed. 2729 A. Kappa C. Friedreich’s Ataxia D. Pes cavus is associated with CMT1x. Harrison’s 17th Ed. Central Core Disease and Plantar Fasciitis. Inhaling the vapor C. Hydromorphone A. levallorphan.818 MCQ’s FOR MEDICAL PROFESSIONALS BY PROF. Tramadol B. Nalmefene B. D. 2730 C. D. 2729 A. 2730 Harrison’s 17th Ed. Papaver somnifitide C. luteinizing 2129 Pure opiate antagonists are all of the following except ? hormone & thyrotropin. nalmefene and naltrexone. Slowest onset & least 2127 The purely synthetic opioids include all except ? intense effects occur after oral consumption. -aminobutyric acid (GABA) pathway D. It increases prolactin levels. 2730 A. Inhibits release of luteinizing hormone C. Inhibits release of corticotropin-releasing factor (CRF) D. Diacetylmorphine (heroin) C. 2730 A. Naloxone 2134 Which of the following is associated with pes cavus ? B. Little less efficient absorption is after smoking or inhaling the vapor (“chasing the dragon”). Enkephalins 2133 Acute opioid administration causes all of the following except ? B. Nalorphine. Papaver somnicuram A. fentanyl. Nalorphine A. methadone. propoxyphene. Meperidine Most rapid & pronounced effects of opioids occur through IV administration. 2730 C. 2729 Harrison’s 17th Ed. AJAY MATHUR Neurology 2125 Morphine & codeine are derived from the juice of ? 2130 Which of the following is not an ‘Opioid Receptor’ ? Harrison’s 17th Ed.   and   . Dynorphins A. Harrison’s 17th Ed. All of the above cyclazocine. Harrison’s 17th Ed. tramadol. and pentazocine. . butorphanol. Delta D. Decreases release of thyrotropin Body’s endogenous opioid peptides are enkephalins. Oral consumption D. All of the above B. Friedreich’s Ataxia. Papaver somnitrium B. C. 2126 The semisynthetic drugs produced from morphine or thebane 2131 The least intense effects of opioids occur after ? molecules include all except ? Harrison’s 17th Ed. diphenoxylate. Intravenous administration A. endorphins & dynorphins and are natural ligands for opioid receptors. buprenorphine & pentazocine have mixed agonist & antagonist properties. Purely synthetic opioids include meperidine. Papaver somniferum D. Meperidine 2132 Euphoria & rewarding effects of opioids are due to stimulation of ? B. Smoking B. Nemaline Myopathy. Dopaminergic pathway diacetylmorphine (heroin) and oxycodone. Endorphins Harrison’s 17th Ed. All of the above Euphoria and rewarding effects of opioids are due to stimulation of dopaminergic pathways 2128 Endogenous opioid peptide includes ? originating in midbrain & terminating in nucleus accumbens. Decreases release of prolactin Acute opioid administration inhibits release of corticotropin-releasing factor (CRF). Nociceptin or orphanin is another receptor. Gamma Morphine & codeine are derived from the juice of poppy Papaver somniferum. Hydromorphone B. Glutamate pathway Semisynthetic drugs produced from morphine or thebane molecules include hydromorphone. Nemaline Myopathy Pure opiate antagonists include naloxone. 2729 A. Mu B. Oxycodone D. Opioid Receptor types include . D. . C 23 ......... D 137 ........... A 52 ................... D 191 ..... C 128 ............... A 245 .......... D 89 . B 183 ..................... B 154 .... C 39 ................................ AJAY MATHUR 819 ANSWERS NEUROLOGY 1 .................. D 58 ..... A 211 ...................................... D 249 ................................................ A 229 ... B 14 ........... C 26 ............... D 188 .. D 185 ................ C 98 ... C 195 . A 156 .. B 46 ............. D 261 ... B 48 ................... A 167 .................. B 230 ................................................. A 215 ...... B 166 ...... B 254 ............. D 38 ............... B 172 ..... D 30 .. C 175 .......... C 118 ... A 8 .............. D 65 .................... A 82 ......................... D 262 ................... C 133 ..... B 50 .... C 115 .... A 79 ........... D 95 ............................. B 240 ...... D 233 ............................................. B 173 ............................................. D 120 .................... D 145 .........Answer MCQ’s FORMCQ’s FOR MEDICAL MEDICAL PROFESSIONALS PROFESSIONALS BY PROF............. A 32 . D 127 .............. A 64 .. D 263 ................... A 44 ............. D 60 .............................. D 57 ................................................... B 121 ...... D 159 ....... D 208 ................. B 24 ........ C 199 ..... C 252 . D 108 ....... C 101 ......... C 150 .................................. B 164 ............. C 31 ............... A 220 ............ C 242 ........... C 21 .......... A 87 ............................................... B 232 ...... D 151 .......... D 184 ......... D 106 ......................... B 176 ........... C 129 ............. C 189 ...... C 74 .. C 135 ................... B 114 . B 77 .. D 102 ..... C 223 ...................... D 123 ... D 267 .............. B 140 ......... B 93 ...................... C 192 ... D 122 ....... D 49 .......................... B 119 ........... C 97 ....... D 209 .... D 70 .............. A 250 ............................ B 206 ...... D 37 .. C 124 .... D 259 ......... B 239 ............ A 217 ... D 197 .... D 179 .......... D 92 ....... A 139 ............................................. B 222 ................ D 36 .......................... C 146 .... A 221 .......................................................................... C 12 ......................... D 80 .. D 55 ....................... B 3 ............................. D 253 ........... B 205 .... C 168 ........ C 29 ...................... A 256 .............. A 117 ..... D 144 ..... A 75 .................. D 51 . B 228 ................... A 243 .... D 147 ........................ C 204 ............................ D 110 ......... A 257 ... C 15 ................ D 210 ................... D 42 ......................... D 270 ..... D 56 ....... A 148 . C 5 .. D 219 ................. D 2 .... A 103 ............................................ D 152 ............. D 91 ..... B 40 ............. B 134 .... D 212 .. D 149 ........... C ............ C 153 ..... A 45 ....................... A 231 ......... C 198 ...................... C 200 ............................ D 225 .......................................................................... D 4 ....... B 10 ............ D 96 ............... B 67 ................................ D 181 ....... A 113 ........ D 11 . C 88 .. B 105 ........ D 54 .... C 238 .. B 174 .............. D 160 .................... D 84 ......... B 216 ............. D 90 ...... B 194 .......................... A 43 ...... A 142 ......................... C 203 .............. D 169 ... B 136 . D 141 .............................. B 16 ......... A 126 ................................... D 214 ....... C 76 ... B 143 .... D 268 .......................................... D 180 . D 35 ........................ D 234 ......... D 193 .. B 41 .............. D 104 .......................................................................... A 7 ............. C 170 ............ B 258 .. B 112 ...... B 53 .. A 227 ..... C 247 ........... A 131 ............................................... C 255 ............ D 171 ............................... D 18 ................................................................. B 125 ....... B 99 . B 213 ......... A 81 . A 235 ........ D 78 ..................... C 116 .. D 130 ........................................ D 25 ............. D 17 ..... D 224 ...... C 201 ......... B 165 ... B 138 ......... D 47 .............. D 157 ... D 59 .................................................. D 68 ............................. C 163 ...... B 246 ................ D 207 ........... A 27 . D 155 .................................................................................... C 161 ........................ C 158 ........... C 236 ........ D 69 . C 73 ....... B 269 ..... D 13 ..... C 190 ... C 33 ....... D 66 ................................................ B 132 .............. D 107 .............. D 265 .................. B 244 ... C 9 .. D 260 .......... B 186 ............................................................ B 187 ..... C 111 . D 237 .................... D 72 ...................................... D 28 ............................. C 177 ...... C 266 .................. D 71 .......... B 85 ................ A 178 ........ D 22 ................................... A 109 ........... C 62 ... A 61 ................................... D 196 ...... C 218 ............... B 264 ....................... D 20 ..................... C 63 ..... B 162 .................. B 34 ..................... D 202 .................................... B 226 ... B 86 .................................. D 241 ............. C 19 .... D 6 ............. D 182 .............................. D 248 ...... A 94 ..... B 251 ............. A 100 ....................... B 83 ...................... ... D 292 ...................... B 460 .......... A 501 . A 392 ................. C 408 ... D 395 ..................... A 486 ................... A 355 ...................... B 439 .......... A 296 .......... D 342 ........................ B 484 ......... C 496 ............................................ C 526 ................ A 391 .... D 394 .......................... C 312 ... B 362 ........... C 378 ..................................... D 532 ...... D 479 ..... C 531 ................. A 477 ..................... D 291 . B 305 ........................ B 539 ....................... D 387 ........... D 433 ................ A 512 .. C 306 ..................... D 377 ....... C 399 ...... C 353 .... B 449 .... A 345 . D 308 ...................... D 498 .................... A 376 ............................................ C 524 ........ C 299 ............. C 525 ......... D 458 ................ D 287 .............. B 328 . B 513 ......... B 340 .... A 398 ......... B 507 . A 397 ............... D 427 ................ D 302 ... A 404 . B 293 ..... A 481 .... B 282 ...... D 521 .................... D 432 .................... D 506 ................................................... A 464 ............ B 462 ... B 489 ............ B 276 ..... A 508 ................. D 447 .................... C 319 ........ A 367 ............................ B 309 .... B 416 ................ A 338 ........................ B 303 ........ D 423 ............................. B 396 . D 441 ........ C 284 .............................. B 438 ....... C 325 .... D 380 ... B 341 .... D 499 ............. D 516 ............................... A 504 ...... C 487 ..................................... B 298 ......................... A 437 . A 425 ............... C 280 .......... D 414 ... B 434 .............................................. D 426 ........... A 295 ... A 428 ...... C 313 ....... D 300 ........ D 455 ........... D 307 .......... D 375 ................ B 314 ...... D 482 .......................... D 473 ....................... A 343 .............. C 346 ..................... D 491 .......... D 517 ..... D 368 .. A 500 ........... B 474 ....................................................... D 410 ............. C 322 ............. D 317 ....... B 366 ........ C 540 .. C 339 .................... C 400 ................. D 386 .. D 344 .. B 277 ............... D 381 ....... D 534 .................. A 323 .............. A 374 ...................... B 442 ............................... B 347 .......... C 382 ........... C 450 .. D 389 ................ A 424 ....................... C 471 ............. C 463 ................ B 402 ........ D 407 . D 370 ...... B 515 ......................... A 413 .......... A 443 ............................. D 419 ..... A 490 ............................ D 321 ................................ D 315 .........820 MCQ’s FOR MEDICAL PROFESSIONALS BY PROF......... B 324 ......................... B 520 ........... C 318 ........................ C 470 ......... A 301 ........ A 422 ......................................... A 316 ... D 335 .... C 448 ........................................................ A 290 ....................... D 371 .................. C 294 ................. B 476 .......... D 444 .................................. B 514 ................................... A 436 ................... D 333 .... D 518 .... D 390 . D 283 ..... A 403 ....... A 503 ........... C 454 ............... C 472 .. A 332 ........ C 288 ............... C 326 ........................ C 467 ...................... C 488 .... C 311 .......................... A 349 ........ C 363 .. C 275 ...................... D 411 ................ D 369 ...... A 446 ........ A 453 .......... B 359 ..... D 523 ..... A 459 .......... D 356 .. C 330 ...... D 383 ........ D 388 ................................ A 465 ............ C 457 .... A 357 ...... D 278 ..... D 327 .... C 469 ........................... D 331 ............ D 354 ........................ D 274 .................. D 502 .................. D 337 ........................................... B 373 .............. A 405 .......... D 329 .... A 478 .... B 297 . C 304 ............................ D 279 ..... C 529 ..... B 522 . D 509 . D 536 ............ A 320 ...... C 485 .................................................... A 421 ...... A 452 . D 415 .... C 535 . C 538 ...... A 530 ....... D 401 ................... D 468 . B 440 ... D 364 ............... D 497 .......................................................................... C 528 ............ C 493 . D 435 .... C 358 .. C 281 ....... D 310 .... A 483 .................. A 409 ................. A 286 ......................... A .......... B 519 ....... D 412 .................................................. D 430 ........ A 350 ................................................. C 495 .......... D 361 ............ C 379 ......... B 336 ............... B 537 .... D 360 .......... C 505 .... D 492 ................................................................. A 417 ... B 480 ......... D 365 .... C 466 ... D 385 .......................................... D 510 ........................... A 511 ...................... A 352 ............................................................ D 406 ... A 351 .... C 273 .. C 431 ................. B 418 .......... C 445 ..... A 420 . A 348 ......... A 475 .......... D 285 .......... A 372 .................. B 456 ........ B 451 ............................ B 494 .. A 289 ... B 384 .................. B 533 .. AJAY MATHUR Answer ANSWERS NEUROLOGY 271 ....................... A 527 ........ D 272 ...... C 429 ................................................ A 461 ...................... A 334 ... C 393 .................... ........... C 761 .................... D 732 ..................... A 648 ................................................................... D 803 .......... C 753 .. A 679 .. B 595 ..... AJAY MATHUR 821 ANSWERS NEUROLOGY 541 . A 652 ..................... B 577 ......... D 560 . C 783 . D 792 ....................... C 699 ............. D 639 . A 578 ...................... A 700 ....... C 677 .......... B 644 ...... C 668 ....... D 685 .......... D 789 ............................... B 586 ........................ A 543 ............................................................... D 686 ....................... C 720 ........................................... A 805 .................................. B 688 ................... D 651 ................ C 602 ..... D 620 .................. D 551 ................... D 702 .......... B 574 ............... C 681 ................ A 630 ........................... B 748 .......... C 712 .. C 553 .......... C 604 .................. D 786 ... D 600 ...... D 588 .... D 689 ........ C 547 ..................................... C 659 . D 662 .......... A 687 .................. A 778 .... D 734 ............. D 655 .......................... C 731 ............ D 661 ........... C 623 ..................................... D 594 ........... D 708 .......................... D 745 ............. B 614 ..................... D 641 ........... D 548 ..................... D 589 . D 558 .... D 739 .............................. C 605 .. D 698 .......... B 599 .. C 613 ... D 674 ............. B 581 . C 570 ..... B 627 ........ D 590 ....... B 684 . A 563 ...... A 755 ........... D 724 ....... C 636 .......... D 793 ... B 719 .............................. D 646 .. D 680 ........................................ D 791 .......................................... D 601 ... D 714 .... A 649 .... B 779 .................. C 598 ...... D 746 .................. A 642 .... C 785 ...... D 660 .............. C 804 ........................... A 759 .... D 694 ..... D 565 ............... D 692 ..... A 790 ........................ B 750 . C 569 .................. C 716 .... D ....... A 656 .... D 650 . C 643 ... D 806 ......... C 571 . D 583 ........ B 619 . D 781 ... D 632 ............................. A 561 .... B 722 ..................... A 796 .............................................................. D 593 .............................. D 788 ......... A 638 ........... C 629 ..................... D 633 .......... D 608 ............................. B 760 ............. C 690 ..........................Answer MCQ’s FORMCQ’s FOR MEDICAL MEDICAL PROFESSIONALS PROFESSIONALS BY PROF.................... C 618 .... D 663 .. D 773 ........................ D 582 ............................................. B 742 ... D 794 .... D 752 ................. C 683 ....................... C 704 ............................. D 715 ............................... D 691 . C 542 . B 585 ....... D 610 .. C 678 ........................... B 807 ..... A 727 ..... A 744 ..... D 775 ...... C 808 .... C 612 ............ C 603 ....................... D 728 ............. D 562 ..... D 707 .... C 703 ............................ A 667 ... C 634 . D 640 ... A 795 ................................................ D 713 .. D 766 ....... A 557 ........ D 666 ............. D 546 ........ C 675 ... D 572 ................... B 653 ....................................... D 780 ......................... D 765 ........... C 579 ................ C 705 .............. D 645 ............................................................... B 576 ...... D 726 ... D 709 .. B 733 .... D 607 ................ A 611 ............ A 622 . D 757 .... D 758 .. D 706 .... D 802 .......... A 671 . C 592 ............................. D 637 ...... D 559 .................. D 647 ...... D 552 ........................ C 725 .................. D 800 ... D 669 ............. C 769 ............................... D 723 ............. A 711 ........ C 556 ............ C 635 ..... B 747 ... D 617 .... D 798 ...... D 682 .......... D 737 ....................... D 606 ..................... D 743 ........................ B 756 ........................................................ A 575 ............................ A 751 ................................................. D 549 .............. A 729 ................... A 545 .... A 754 ..... A 762 ...... D 771 . B 654 ... C 730 ........ C 657 ............. B 770 .... D 670 ............................................. D 718 .......... D 596 ... C 597 ......... A 616 ... B 763 . B 736 .......... B 631 .................................. D 772 ........................... D 810 ................. B 564 .............. B 673 ............ C 721 ...... C 741 ... D 767 ..... B 695 ......................... B 672 ...................... D 555 ................... D 658 ................................ B 749 ..... D 784 ............ D 774 .................. C 554 ......... D 710 ......... D 740 ........... C 544 .. D 768 .......................... D 573 .................. B 567 ................... D 799 ...... D 738 ........................... D 664 ....... C 776 ....... A 566 .... A 777 ................................ C 568 ... C 801 ........ D 809 .. B 764 ....... C 591 .......... A 693 . D 797 ............................... B 676 ...................................................... C 550 ............... A 624 . D 787 ....................... B 587 ...................... A 609 ...... D 717 ................. C 628 ...................... A 696 ........... C 580 .... D 625 ...... C 584 .................. A 782 .. D 615 ............... D 735 ................................. B 626 ... D 665 .............................................. D 697 .. D 621 ................................... C 701 ..... ................................. B 901 ...... B 1046 ......... D 1037 ................. B 880 ................................. D 998 .... A 917 ............... A 944 ................... A 870 . D 1003 ............................ B 842 ............... C 924 ... D 896 .. D 837 ............ A 1055 ...... B 977 .......... B 841 ........... D 853 .......................... B 887 ........ B 922 .............. B 885 .... D 992 .................... D 906 ................ D 888 ........... B 1048 ............. D 979 ........................ C 1051 ....................................................................... C 876 ................. A 849 .............. D 1076 ........ B 895 ...... D 1024 ... D 1042 ........... D 892 .................. D 975 ......................... B 867 .. C 970 ...... A 984 .... B 952 ........................ D 932 .... C 964 ................... D 1009 ......... C 925 ...... D 846 ............ D 995 ..................... A 909 ........................................ D 899 .. B 980 ........................ B 935 .... C 1038 .. B 974 .... B 1059 . D 1075 ........................ D 1001 ......................................... D 971 .. D 820 ........ A 963 ..... C 921 . A 828 .............. D 916 ...... B 865 .. D 838 . C 883 .................. C 986 ................. A 960 .................. A 1017 .. D 1032 ............................. C 862 ....................... D 900 ...... D 852 .......... A 967 .............................. B 1043 ....................... C 1074 ............................. B 871 ........... A 847 ........... D 1036 .. D 851 .. C 1068 .............. D 855 ............................. B 868 .............................. D 879 .................................. A 843 ...... B 1008 ............. C 962 .... C 1058 .............................. B 972 ........ D 1065 ... D 875 ......... C 1014 ... C 872 ........... D 812 ...... D 938 ..... A 822 ..... C 1002 ............. C 950 ......... C 988 ......................... D 928 . D 1035 ........................................................... B 859 ......................................... C 860 .......... D 937 ........................................ C 910 ....... D 1054 ................. D 1041 ..... A 1049 ......... D 1010 . C 939 . D 907 ..... D 877 ... D 918 ... D 993 .................... D 1011 . A 878 ... C 949 ............................ D 1019 ... D 996 ...... D 840 ......................... D 858 .... D 920 ... D 891 ........ D 850 ................... D 897 ........ AJAY MATHUR Answer ANSWERS NEUROLOGY 811 . B 930 ........ D 999 ... B 1052 ................ C 914 . D 833 ................... C 881 ....................................... C 839 ........... C 1021 .................. C 1023 ............................ A 1045 .. B 954 ............. D 966 ........................ D 1069 ........ B 983 ... B 1062 .. B 863 .............. D 1040 ...... D 1030 ............... D 929 . D 844 ........ A 989 ........... C 1072 ........................... D 908 .... D 834 ..................... B 981 . D 830 ........................ A 854 ........... A 861 ............................. D 1057 ....................... A 926 ......................... D 903 ....... C 1063 . D 1070 ......... C 927 ...... A 827 .............. C 973 . A 1016 ....... D 831 .............. C 1034 ... B ...................... B 1066 .......... D 958 . B 869 ................................. D 1018 ........................ D 990 .................... A 1000 .......... D 1022 ................ D 1060 ... B 955 ......... B 961 ..... D 1029 ... D 1028 .... D 857 .. C 1026 ............... D 931 ......................................................................... B 1039 ....... D 882 .... A 832 ........................................ B 934 .................................... D 978 ........... B 956 .................... A 856 ............ D 1080 ............... D 1015 ............................ D 1006 ........... A 1033 ......... C 1071 . C 816 .............................. C 817 . C 987 ........... B 965 ......................... D 994 ........................ C 976 ................... D 1056 . D 1007 .................. C 898 ........ D 835 .... D 905 ....... B 1050 ....... D 1061 ... D 1064 ....... D 884 ..................... D 1044 ....................... C 1027 .............. B 1012 ......... D 873 .... C 829 ............. D 836 ..... C 815 ................ A 1020 .............. A 889 ........ B 969 ... B 951 .......... A 940 . B 874 .......... A 957 ....... B 911 ... C 902 . D 1031 ......... D 948 .... D 1053 ..... A 1078 ....... D 1004 ....... A 823 ............................... A 945 ....... D 825 ............................................................. C 904 ...... D 1077 .... B 824 ................................................................................... C 943 ................... B 890 ...................... D 923 . C 819 ..... A 845 . D 913 ....................................................................................... D 1025 .................. D 942 . B 959 ......... A 886 ................ C 1067 ..... D 826 ..... C 946 ......... B 1073 .............. C 893 .........................822 MCQ’s FOR MEDICAL PROFESSIONALS BY PROF............. A 919 .. D 933 ......................... C 1047 ..... B 953 ................. D 991 ......................... B 915 .. A 936 ... D 1013 ...... C 1005 ... B 982 .............. C 848 ......... A 821 ................... D 864 ........ C 912 ... C 968 . D 1079 .. D 866 ............. D 985 ..... A 813 ............ D 814 ........... D 997 ........................... B 818 .............. D 941 ..... C 947 ................ D 894 ........... ...... C 1343 .... A 1174 ......... C 1263 ... D 1147 ............. D 1124 ........ C 1289 ............. C 1094 . D 1288 .......... D 1272 ........................................ B 1262 ......... D 1110 ............. D 1134 .... C 1231 ............. D 1112 ....... C 1109 .......... C 1216 ... C 1336 ............... D 1138 . A 1273 ..... C 1340 . D 1305 ......... D 1167 .... A 1091 ...... B 1335 ...... B 1125 .................Answer MCQ’s FORMCQ’s FOR MEDICAL MEDICAL PROFESSIONALS PROFESSIONALS BY PROF............. C 1300 .... A 1119 . D 1276 ............................................................................................. D 1348 ..... B 1200 ......... C 1223 ............ D 1234 .......... D 1232 . B 1185 ...... A 1218 ....... B 1256 .... C 1242 ...................... B 1304 .... D 1175 ...... A 1330 .. B 1140 ......... C 1186 ........................... C 1118 ........ A 1102 ......... D 1281 . D 1197 ... C 1188 .. B 1254 . B 1287 ................................................................................. B 1303 .... C 1329 . A 1168 .... D 1098 ... B 1283 ...... B 1148 .......... D 1347 ....................... D 1230 ................................. B 1328 .... A 1162 ... A 1247 .................. A 1086 ................ D 1146 . C 1294 . A 1179 . C 1239 ...... D 1314 ..... C 1260 ... C 1156 ........ D 1194 ........ D 1326 .......................... A 1116 ................................... A 1106 ....... B 1241 ........ B 1248 ................... C 1302 ..... AJAY MATHUR 823 ANSWERS NEUROLOGY 1081 ... D 1325 .. B 1342 ........................ C 1290 . D 1284 .......... D 1192 ...... A 1334 ............ A 1245 ................................................ B 1123 ....... D 1292 ........ D 1308 . A 1225 .. D 1202 ...... B 1115 .................................................. D 1096 .. C 1214 . A 1341 ........................ A 1313 ........... D 1184 . B 1310 .................. D 1104 . D 1221 .................................... B 1149 ........ D 1163 .......... D 1318 .... A 1160 ......................... D 1164 ........................................ C 1139 .. A 1171 ........................................... D 1295 ............. A 1170 .... D 1244 ........................................ D 1320 .. D 1252 ........ D 1270 ............. D 1121 .... A 1150 . D 1311 .... C 1151 ..... D 1243 ....... B 1249 .................................. C 1315 ............. C .............. D 1316 .. D 1191 .... D 1298 .......... C 1201 ....... D 1322 ............................................ C 1187 .......... D 1092 .. D 1173 ......... B 1145 ............. B 1131 ............ B 1207 .. B 1233 ............................... D 1143 .. B 1158 ..... B 1228 . A 1122 ........ D 1107 ................. D 1103 .... D 1333 . B 1286 ......... A 1278 .. D 1344 ...... D 1275 ................... C 1114 ...................... A 1199 . D 1250 . A 1135 .... C 1113 ......... C 1266 .......... D 1317 .. A 1154 .......... C 1169 .......... D 1280 ...................... B 1327 ....... B 1331 ................................. D 1235 .................................................... A 1205 ................ D 1299 ... B 1155 ...................................... C 1301 ................ B 1195 .......... D 1204 ............................................. B 1240 ................. A 1088 ....... B 1238 ...... C 1293 ............................... A 1157 ... D 1190 .. D 1268 ............. B 1172 . D 1133 .. B 1087 ...... D 1095 ...... D 1097 .... D 1203 .. A 1345 ..... B 1132 .... A 1206 ....... A 1338 .......... D 1319 ....................... A 1255 ......... D 1152 ...................................... D 1277 ... C 1165 . D 1212 .......... A 1274 ..... D 1271 .... C 1176 ....................................... D 1237 ............ B 1089 ...... B 1267 .... B 1090 ....................... D 1159 ..... D 1093 ..... D 1083 ..................... D 1100 .................. D 1193 ..... D 1211 ......... D 1227 ........ A 1166 ............... A 1309 ................ D 1324 ....................... C 1246 ..... D 1306 ...... D 1213 ............................... D 1279 ... C 1217 .................. D 1222 ................... B 1265 ..... B 1130 ........ A 1105 ...... D 1084 .... C 1219 ................................................ D 1209 ..... C 1099 .... C 1180 ...... C 1261 ... B 1144 .................... D 1215 ......................................... B 1189 ......................................... D 1312 ................................ A 1236 .............. D 1229 ......................... D 1177 ............. B 1108 . B 1127 ............... A 1126 ..... C 1161 .. D 1282 ... C 1220 ............... C 1224 ................... B 1339 . D 1307 ........ C 1120 ...................... D 1183 .................. C 1258 ........... B 1297 ... D 1296 ....................... C 1332 ........................................... D 1337 . D 1226 . B 1129 .... D 1196 . C 1291 ..... A 1082 ................... D 1182 ............................................... D 1285 ......................... A 1142 ........... D 1321 . D 1257 ........ D 1136 . D 1128 ... C 1264 . D 1208 .............. A 1346 ........... D 1178 ............ C 1101 .......................... D 1269 ..................... D 1251 ..... A 1350 ....................... A 1085 .. D 1323 .. A 1117 ... C 1349 .......................... D 1210 .. D 1181 .............. B 1141 ........... A 1111 ..... A 1259 ...................... D 1198 ... D 1153 ... D 1253 ... D 1137 ..... ..... A 1572 ... D 1534 .... B 1369 . D 1374 .. D 1375 ..................... D 1543 ....... D 1544 .... AJAY MATHUR Answer ANSWERS NEUROLOGY 1351 .................. D 1410 ............. D 1386 .. D 1619 ...... B 1449 ........ D 1527 ............... A 1455 ...................................................... D 1379 . D 1367 ............................... D 1436 . C 1518 ............ D 1554 .... C 1592 . D ..................... A 1580 .. C 1510 ....... A 1354 .. A 1603 .... D 1406 ........ A 1488 ...... C 1546 ..... C 1511 .... D 1548 ......... D 1595 ............ C 1561 ......................... C 1537 .............................................. B 1585 .......... D 1517 .................... D 1357 .......................... A 1530 ...... D 1601 ............. D 1445 ................... C 1414 ......... B 1478 .. D 1376 ........................... B 1568 ................. D 1515 ............ D 1617 ............ A 1496 .......................... D 1456 .... D 1584 .............................. D 1605 ........... D 1473 ............. C 1598 ......... B 1567 .. C 1503 ...................................... D 1589 .............. A 1468 . D 1439 ...................... D 1599 ........................ C 1614 .... C 1453 ............. D 1407 . B 1536 ......................... C 1363 ...... C 1461 .... B 1613 ... D 1482 ..... D 1495 .......... D 1597 .... D 1366 .. B 1539 .................... C 1359 . A 1424 .... C 1499 ... A 1418 ............. D 1377 ...... B 1489 ...... A 1435 ........... B 1600 ....... C 1419 ........ A 1431 ...... C 1371 ................. D 1529 ............ D 1566 .. D 1405 ... D 1545 ..... A 1427 ............... A 1575 ................ D 1484 ........................824 MCQ’s FOR MEDICAL PROFESSIONALS BY PROF............. C 1512 ................... C 1513 .... D 1480 ...................... B 1562 . A 1576 .... A 1426 ............ D 1462 ............................ A 1498 ...... D 1358 ........... D 1388 ......... D 1353 . B 1615 .................. D 1565 ...... B 1506 .. D 1356 ................................. B 1447 ... A 1611 ............... A 1550 .................. A 1441 .......... B 1522 .. B 1467 .................................... B 1420 ....... C 1399 ......................................................... C 1438 .................... B 1428 ....................... B 1364 ................................................. D 1608 ... D 1583 ....................... C 1523 ................. B 1429 ......... D 1516 ...... D 1563 ....... D 1532 ....... D 1612 ... A 1450 ..................... D 1574 ......... D 1360 ........................................... D 1403 .............. A 1400 ....... B 1541 ...... D 1573 ... D 1408 ........... B 1620 . D 1604 ........ A 1451 . A 1423 .. B 1594 .......................... A 1557 ................ C 1444 ......... D 1483 ... B 1525 ..... D 1448 ....................................... C 1607 ...... A 1492 ........ C 1507 .......................... A 1577 ................ A 1442 . B 1616 .... B 1587 ........................ D 1502 ......... D 1391 . D 1457 ..... D 1501 ........ C 1481 ................... C 1558 ....... D 1390 ................................ D 1479 ......... C 1547 ................ C 1396 .. D 1560 ..................... C 1582 .......................... D 1551 ................................... A 1393 ................ A 1569 .............. D 1509 ................. B 1394 .................. C 1368 .. D 1549 ......... B 1411 ........ D 1452 ............................. B 1470 ..... A 1564 ............ D 1446 ................................... D 1490 ... D 1437 ............... D 1430 ............... B 1415 .... A 1596 ................... D 1381 .. D 1505 ............. D 1384 ... D 1609 .... D 1487 ....................... B 1542 .... D 1588 . A 1581 .......... D 1578 . D 1458 . D 1514 ............. D 1454 ..... A 1610 .......................... D 1474 ......... B 1383 ........................ D 1606 .. B 1361 . C 1540 ................................. C 1591 ......... D 1370 ..... A 1387 .............. D 1372 ...... D 1412 ................................................. D 1469 ..... C 1570 ....... A 1389 ............... C 1464 ...... A 1553 ..... D 1526 ...................................... A 1519 ............ A 1434 ... B 1443 ....... C 1486 . C 1476 ........ D 1521 . C 1380 .. B 1421 ........... C 1556 .. B 1352 .. D 1504 ..... D 1433 . C 1409 .. D 1466 ......... A 1535 ............. D 1559 ... A 1440 ...... C 1586 .......................... B 1472 ...... D 1533 ....................... D 1460 ..................... D 1528 .............. A 1355 ... C 1555 ........ D 1463 ...................... D 1373 . A 1579 ........ B 1417 . C 1618 ........... D 1602 ............. A 1459 ............ D 1590 ............................................ D 1494 ... B 1477 .................... D 1538 ................................ B 1401 .................... B 1500 ...... A 1382 ................. D 1432 ... B 1402 . C 1413 ........ D 1398 .......... D 1571 ................ D 1404 .... B 1471 .................................... C 1385 ... C 1395 . B 1416 ......................... A 1465 .... B 1365 ..................... A 1425 ...... C 1508 ............ C 1493 . D 1552 . B 1422 ................... A 1485 ........ D 1524 ...... C 1593 .................. D 1491 ... D 1362 ..... C 1475 ..... C 1531 . C 1378 ....................... B 1392 ....... D 1497 ....................................... C 1397 ........... B 1520 . ................. A 1687 ......................... D 1806 .......... D 1699 . B 1843 ... B 1759 ............... B 1835 .............. D 1804 ........ D 1871 ................................. C 1833 ................................... B 1719 ................ D 1737 ...... D 1749 ............ D 1695 ............... A 1727 ............................... A 1874 ... C 1884 ....................... A 1851 ........... B 1682 ..................... A 1821 .................. D 1761 ..... A 1756 .......... D 1829 . C 1783 .................... C 1869 ............................. D 1626 ......... D 1776 ... C 1709 ....... D 1730 ............ A 1659 ................... D 1814 .................. A 1824 .... C 1710 .............. D 1763 ........ D 1683 . D 1857 ........ D 1625 ........ D 1870 . A 1846 ....................................... B 1809 ............. D 1725 ................. A 1680 ..... B 1883 ............... A 1651 ......... A 1877 ..................................... C 1712 .... D 1622 ............... C 1654 ....... B 1782 ......... C 1780 ......... A 1723 .... D 1647 ............... C ........................ D 1666 ....... A 1856 ........... D 1800 .... B 1627 .. A 1757 .. D 1802 ..... A 1731 ............... D 1889 .. A 1704 .. D 1652 .................................. C 1810 ........ B 1770 .... B 1758 ..... B 1880 ............... D 1715 ................ D 1772 .... D 1684 .. D 1742 .................................................. D 1873 ............ B 1686 ............................................... D 1722 ......................................................... D 1875 .................. C 1834 .. A 1630 ............... D 1691 . C 1631 .......... D 1653 ......................... A 1677 ........... C 1791 ...... C 1624 ...... D 1799 ....... A 1815 .. C 1872 ........ A 1668 ................ A 1685 ......................... B 1724 ..... D 1774 ........ B 1861 .......... B 1639 .... B 1887 . B 1721 ... D 1808 .. C 1637 .... A 1858 . D 1865 ............... A 1801 ..... AJAY MATHUR 825 ANSWERS NEUROLOGY 1621 ................... D 1740 .. C 1852 ................................ D 1657 ......... D 1744 ............. B 1779 ...................... A 1860 ....................... D 1812 ........ B 1633 ...... A 1784 .... D 1788 ......... D 1847 ............. B 1716 ......... D 1636 ......... B 1841 ..... C 1762 ............................ D 1635 .. D 1751 ................... D 1643 ............... C 1773 . C 1701 .......... D 1767 .................. D 1706 .................. D 1714 . D 1688 ............................ D 1766 . D 1822 ...................... C 1777 ..................... B 1660 ... C 1628 ...................... B 1669 .......... B 1728 ........... B 1743 ................ C 1645 ...... D 1735 ........................... D 1717 .. D 1664 ................ D 1697 ......... B 1881 .... D 1819 ............. D 1867 . C 1778 ........ D 1702 .. A 1831 .. C 1670 .. D 1848 ............................... C 1674 ......................... A 1855 ... D 1850 ........ B 1853 ....... A 1640 .... B 1820 ............................................ D 1656 .. D 1700 ..................... D 1729 ..................... C 1718 ....................................... A 1832 .......... D 1864 .................................. C 1826 ... C 1786 .. D 1792 ..... B 1720 ........................... D 1754 .............................. D 1813 . A 1638 .... D 1882 ........ D 1748 ............... A 1785 ...... A 1678 . B 1733 ........... A 1726 ..... C 1738 ... C 1707 ................. D 1836 ........ C 1623 ............. C 1661 .... C 1886 .................... D 1781 ............. D 1648 ............ C 1868 ....... D 1863 . D 1736 .......... D 1771 .................................. A 1655 .............................. B 1837 ....... D 1811 ....... C 1681 .............. D 1676 .... C 1878 .... A 1693 ...... B 1698 .. B 1658 ............ B 1805 .... C 1838 ................... D 1795 .................................... D 1876 ..................... D 1690 ............. D 1764 ........... B 1679 .... A 1752 ...................... D 1644 .. C 1798 ........ A 1859 ............... C 1794 ................. A 1646 ................... A 1803 ........................ B 1789 . D 1816 .... D 1732 ........................ A 1741 .. B 1750 .... D 1888 ...... A 1703 . B 1694 .. C 1807 ............ D 1790 .............. A 1828 .................... A 1665 .................. B 1839 . B 1768 ...................................................Answer MCQ’s FORMCQ’s FOR MEDICAL MEDICAL PROFESSIONALS PROFESSIONALS BY PROF...... D 1765 . C 1747 . D 1830 ... A 1827 ..... C 1672 ...... A 1866 ... D 1649 . D 1885 ........................ D 1823 ..... A 1629 ... B 1879 ......... B 1632 . B 1739 .................. D 1769 . A 1662 ................ A 1642 ...... D 1849 ........................... C 1760 ... D 1663 ............... D 1713 ......... D 1842 . D 1787 ........ A 1862 ......................... D 1817 .......................... B 1745 ........... A 1667 ................................ D 1689 .. C 1840 ....................... B 1675 . D 1711 . C 1671 .... C 1708 . D 1818 ... A 1793 ... D 1692 .......... D 1734 ..... D 1634 .. B 1753 .... D 1650 . B 1854 ......... D 1641 .......................... D 1696 ........................ D 1845 ..................... B 1796 . A 1705 ............. D 1844 .................. D 1775 ............................ D 1746 .......................... D 1890 . A 1797 . B 1755 . D 1673 .. C 1825 ........................ ........... D 1998 ..... D 2036 ....... B 2110 .......... D 1979 .. D 1910 ....... D 1944 ....... B 2009 ................... C 2060 ................. D 1894 ................... B 1911 .. C 2111 .................. A 2118 ......... B 2045 ... D 2086 .............. D 1901 . B 1950 ... D 1977 .... D 2071 . AJAY MATHUR Answer ANSWERS NEUROLOGY 1891 .......................... D 2104 .................... B 1940 ........... A 2004 .. D 1966 .............. A 1956 . B 2075 ............. C 2078 . D 1899 .... A 1958 ....................... D 2132 ..... B 2051 ................. A 2021 .. D 2076 ....................................... D 1963 .... D 1943 ............... C 1990 .............. D 2068 .. C 2050 ....... C 2061 ............. D 1928 . D 1994 ... D 2083 ............... A 2027 .................. D 1960 .. D 2033 . B 1967 .................... D 2001 ... D 2067 ............. A 2019 ... B 1987 ...... D 1988 ................. D .......... A 2077 .. A 1970 ......................... C 2107 .. D 2113 . B 1905 ................ D 1921 ........ A 2034 ... A 1982 ................................... B 2041 ........................ D 2010 . D 1964 ........ D 2127 . A 1999 ... A 1913 ... B 2130 .... D 1907 ............................ A 2023 ..... D 2116 ... D 2128 ......................... D 1941 ............................................................. C 2069 ............... D 1895 ....................... A 2037 ....................... A 1983 . D 1925 .. D 2024 ..... D 2031 ........... D 2089 ....... D 2064 ......................... D 1919 ............ D 1952 ...... A 2097 .... D 2028 ....... B 1931 ...................... C 1976 ...... C 1903 . D 2049 ............. B 2026 . A 1959 ....... D 1951 .. B 2102 .................................................................... D 1937 .................................. D 2007 .............................. C 2072 .. D 2012 ......... D 2084 ....... A 2038 ..... D 1965 ............................................. A 1946 ...................... D 2054 ...................... B 2096 ......... D 1906 ........................... A 2058 ... B 2074 .................................... A 2015 ................ A 2014 ................. A 2112 ..... B 1923 ...826 MCQ’s FOR MEDICAL PROFESSIONALS BY PROF..... A 2062 .............. D 2100 .. B 2109 . C 2092 ........................ D 1915 . B 2016 ........ C 2059 ............. A 2108 .................. D 2121 ... D 2006 .... C 2087 ....... B 1975 ..... A 1955 .. C 1932 ...... B 2057 ....................... D 1902 ... A 2103 .......................... D 1920 ........................... D 1993 ........... D 1980 ..... C 1991 ...... C 1939 ............... B 1968 ............ B 1973 ........... D 2093 .... D 2094 .... B 2063 ..... A 2125 ........ A 2053 . A 2056 ......... D 1934 ........ C 2020 .......... C 1916 .. A 2039 ........ A 2126 . C 1898 ........................... D 1971 .. B 2114 . D 2122 ................................. D 1909 ............... B 1954 ............ C 2003 ............. D 2134 .... B 1985 ............ B 2098 ........... A 1948 ........ B 2032 ........... D 2131 . D 2120 .................. D 1936 ........... D 1989 ... B 1897 ................ A 2095 ....... D 2101 ................................................................................. B 2043 .......................... D 1947 ................ B 2046 ....... B 2090 .................................................. C 1908 ....................... D 2073 . D 2129 ........ C 2105 ............. B 2013 ....... D 2011 .... D 1969 .. C 2018 .... D 1986 ................... C 2042 ......................................... A 1996 ...... B 2066 . B 1992 ........................ B 1922 ............................. C 2133 ...... C 1900 ..... C 1892 . D 2088 ..... B 1926 ..... B 1962 . A 2035 ............... D 1929 . D 1930 . A 1927 ...... D 1917 ............... C 1933 ......... B 2047 ......... A 2070 .. A 2052 .. B 1957 ... B 1935 ...................................................... D 2008 ......... D 1945 ......................... A 1912 ..... D 1949 ................. C 2055 .... B 1997 .. D 1974 . A 2082 ............... D 2117 .................... B 2065 ... B 1953 ................................... D 2022 .......... B 2025 . B 2044 .. D 2030 ............... D 1918 ................. D 2115 ............ A 1972 ............... D 1981 ................ B 2005 ... C 1961 ... D 1942 .... C 2000 ............................................ A 2048 ................ C 1914 .......................... D 1893 ................. D 2080 ................. B 2106 .... B 2099 .... D 1978 ................ A 2119 .......................... B 2029 ........................... B 2040 . C 2079 ......... C 2002 ........................... D 2123 .. D 2017 ............ D 1924 .... A 1995 .............. C 2091 ... D 2085 ........ C 1938 ......... B 2081 ............................ D 1896 ............. D 1904 ........... D 2124 . D 1984 ....................... 361 C. G0 D. G2 A.40000 genes mRNA are called ? Harrison’s 16th Ed.366:249–60 7 Albright hereditary osteodystrophy consists of ? A. Ability to form embryo & trophoblast of placenta Harrison’s 16th Ed. 362 10 Spacing regions between the exons that are spliced out of precursor A. None of the above C. 362 B. Ability to differentiate into all cells of all germ layers A. Exon C. Cistron 3 Which of the following is false about codons ? D. Ability to generate one cell type C. Ability to differentiate into all cells of all germ layers A. 362 B. 360 9 Portion of genes that are eventually spliced together to form A. Ability to differentiate into all cells of all germ layers Harrison’s 16th Ed. 362 C. guanine & cytosine RNAs during RNA processing are called ? Harrison’s 16th Ed. Short stature D. DNA is composed of adenine. Adenine is paired to thymidine A. G1 12 Which of the following defines ‘Pluripotency’ ? Lancet 2005. Brachydactyly D. Short arm of chromosome is termed “p” C. Ability to differentiate into limited cell lineages appropriate to 4 Resting state cells are in which phase ? the location Harrison’s 16th Ed. RNA is more stable than DNA C. Ability to form embryo & trophoblast of placenta D. Short stature the location C. Cistron D. G0 B. There are more codons than amino acids B. Embryonic stem cells . Ability to generate one cell type B. Ability to differentiate into a limited cell lineages appropriate to B. Ability to differentiate into a limited cell lineages appropriate to 5 Which phase in cell mitosis is called critical checkpoint ? the location Harrison’s 16th Ed. AJAY MATHUR Cardiology 827 B. G3 A. Intron D. G3 B. Ability to differentiate into a limited cell lineages appropriate to A. 827 Miscellaneous MCQ’s FORMCQ’s FOR MEDICAL MEDICAL PROFESSIONALS PROFESSIONALS BY PROF. G1 13 Which of the following defines ‘Multipotency’ ? C. Human genome contains ~ 30000 . Hypogonadism Lancet 2005. Protein translation Genetics C. 362 A. Ability to differentiate into all cells of all germ layers D.366:249–60 B. Resistance to parathyroid hormone C.366:249–60 C. Ability to generate one cell type D. Guanine is paired to cytosine B. Triplet of bases 11 Which of the following defines ‘Totipotency’ ? Lancet 2005. Human DNA consist of ~3 billion base pairs (bp) of DNA per haploid genome A. Recon 2 Which of the following statements is false ? Harrison’s 16th Ed. Posttranslational modifications D. All of the above 15 Which of the following is an example of ‘Totipotent cell’ ? Lancet 2005. Obesity the location B. Ability to form embryo & trophoblast of placenta 6 Prader-Willi syndrome consists of all except ? B.366:249–60 D. Ability to form embryo & trophoblast of placenta C. Ability to generate one cell type A. Specifies a particular amino acid A. 361 D. Exons C. 362 B.366:249–60 8 Gene expression requires ? Harrison’s 16th Ed. Deafness 14 Which of the following defines ‘Unipotency’ ? D. All of the above 1 Which of the following statements is false ? Harrison’s 16th Ed. 361 A. Recon Harrison’s 16th Ed. Fertilised oocyte or zygote A. All DNA encodes genes B. G2 Lancet 2005. thymidine. mRNA processing B. Introns D. Unipotent 29 Syngeneic transplantation refers transplantation of ? 20 Cells in Inner Cell Mass (ICM) of human blastocyst are ? A. 375 A. Pluripotent C. Genetically identical marrow A. Aplastic anemia 22 In humans. Ectoderm B. First trimester C. Any of the above D. time from fertilization to implantation in uterine wall 31 Which of the following is a mitochondrial disease ? is about ? Harrison’s 16th Ed. None of the above D. Ectoderm C. somatic stem cells D.366:249–60 A. Type II pneumocyte C. Simultaneously with implantation B. Ectoderm 17 Which of the following is an example of ‘Multipotent cell’ ? D. Ectoderm Lancet 2005. Endoderm B. Mesoderm D. Mesoderm A. Third Trimester D. Type II pneumocyte A. Unipotent 30 Diseases commonly treated with autologous hematopoietic stem- 21 When does ICM form in blastocyst ? cell transplantation include all except ? A. Germ-cell tumors D. Adult. Endoderm during pregnancy ? B. Prior to implantation A. somatic stem cells 24 Thyroid is formed from ? D. Leber’s optic atrophy C. Own stored marrow B.366:249–60 D. Endoderm A. All of the above . All of the above D. Pluripotent C. Embryonic stem cells 25 Eyes and ears are formed from ? C. 7 days B.128 cells in morula is ? A. All of the above 18 Which of the following is an example of ‘Unipotent cell’ ? 27 Urinary bladder and urethra are formed from ? Lancet 2005. After implantation C. Genetically non-identical marrow C. somatic stem cells C. None of the above D. Adult. MELAS syndrome B. Genetically identical marrow A. Embryonic stem cells B. Fertilised oocyte or zygote B.366:249–60 A. Multipotent D. All of the above 23 Liver is formed from ? 32 When are Screening tests for Neural Tube defects (NTD) done A.828 MCQ’s FOR MEDICAL PROFESSIONALS BY PROF. Endoderm B. All of the above Lancet 2005. 14 days C. Adult. Adult. 21 days D. Type II pneumocyte 28 Allogeneic transplantation refers to transplantation of ? 19 Each of the 32 . Neuroblastoma B. Own stored marrow B. Fertilised oocyte or zygote B. Type II pneumocyte D. 3 days A. Totipotent B. All of the above C. AJAY MATHUR Miscellaneous C. Multipotent D. Second Trimester D. All of the above A. Mesoderm C. Genetically non-identical marrow C. Ovarian cancer C. Endoderm B. Mesoderm B. Mesoderm 16 Which of the following is an example of ‘Pluripotent cell’ ? C. Kearns-Sayre syndrome (KSS) D. somatic stem cells A. Fertilised oocyte or zygote 26 Kidney tubules are formed from ? A. Embryonic stem cells C. Ectoderm D. Totipotent B. high hCG. 100 : 50 A. alpha fetoprotein (AFP) is synthesized by ? 41 Which type of cell division ensures that humans have the same A. Meiosis D. Fetal Lung A. Binary fission D. uE3 & Inhibin-A B. 100 : 100 48 Down syndrome in humans is due to ? D. Down syndrome C. Placental tissue C. Karyotype B. Cistron D. uE3 & Inhibin-A in the formation of zygote ? B. High AFP. Budding 34 Which of the following are “Second trimester serum markers” used for prenatal screening of Down syndrome ? 42 How many chromosomes does each sperm and ova contribute at A. 829 Miscellaneous MCQ’s FORMCQ’s FOR MEDICAL MEDICAL PROFESSIONALS PROFESSIONALS BY PROF. 23 pairs of chromosome D. what is the chance of having 46 Inheritance of skin colour is an example of ? an affected child if two unaffected people who carry one copy of A. Mosaic D. Proband 49 The parents of a colour blind daughter would be ? C. Translocation C. low unconjugated estriol & high inhibin-A homologous chromosomes is known as ? B. CAG B. hCG C. Inversion D. A. hCG. 4 A. AFP. low unconjugated estriol & high inhibin-A A. Low AFP. Fetal Liver number of chromosomes (46) in each generation? B. Genetic lead A. Muton 38 In an autosomal recessive disorder. Heterozygosity D. high hCG. low unconjugated estriol & high inhibin-A C. hCG & Inhibin-A A. high hCG. CGA D. 46 chromosomes D. 4 C. High AFP. Umbilical Cord B. 50 : 50 C. 7 B. Mitosis C. Colour blind father & carrier mother D. 25 % D. Polygenic inheritance mutated gene (carriers) each ? B. Duplication 36 Mutation on IT15 gene causing Huntington’s disease (HD) is located on which chromosome ? 44 Chromosome number in somatic cells of Drosophila is ? A. 0 % 47 Which condition is caused by a mutation that involves an entire chromosome rather than a single gene ? 39 If a women is suffering from any X-linked recessive disorder. Trisomy B. Monosomy B. Replicon B. Recon C. Mendelian inheritance A. Thalessemia then what is the chance of her passing the mutant gene to her son B. AFP. Autosomal inheritance C. 23 chromosomes C. Monogenic inheritance B. 2 B. 100 % C. 50 % D. 46 pairs of chromosome 35 In maternal serum. pregnancy suspected of Down syndrome will show ? 43 The exchange of chromosomal segments between non- A. X C. Hemophilia (XY) and daughter (XX) respectively? A. Phenylketonuria B. Carrier father & colour blind mother . Nullisomy 40 What is the technical name given to the affected individual through whom a family with a genetic disorder is analysed ? C. hCG. GGG A. 50 : 30 D. High AFP. Triploidy A. GAA C. 6 D. high hCG. 21 D. 8 37 Huntington’s disease is associated with which triplet repeat ? 45 DNA segment that participates in ‘crossing over’ is called ? A. AJAY MATHUR Cardiology 829 33 During pregnancy. low unconjugated estriol & low inhibin-A B. or singly when it is present on ? 59 In electrophoresis. BRCA1 D. XXY 56 What is the situation called in which two different alleles for a genetic trait are both expressed ? B. Template DNA D. DNA molecule will move towards ? A. DNA printing D. 50 % of daughters will be colour blind A. Neutral D. Anode C. Karyotype 51 In humans. Telomerase 62 Which syndrome results when the genetic constitution is XO ? A. XY A. Mosaic 46. AJAY MATHUR Miscellaneous C. Cathode B. None of the parents are colour blind C. Zero onset and more severe disease than the parent. Autosomes A. Klinefelter syndrome C. Backcross 64 Which of the following does not associate with Klinefelter D. 50 % of sons will be colour blind chromosomes’ ? B. Plasmid DNA D. Pentrance D. Dominant D. Cytomorphics D. Topoimerase D. X chromosome of female B. Satellite DNA 63 The number of ‘Barr bodies’ in a person suffering from Turner syndrome is ? 55 The name of the mechanism wherein a grandchild has an earlier A. Three C. Segregation from messenger RNA template ? A. Duplicate gene A. Mitochondrial DNA C. Thumb printing C. DNA Ligase C. None 52 Gene which results in a non-viable progeny in homozygous 60 Process of identification of multiple specific alleles on a person’s condition is known as ? DNA to produce a unique identifier for that person is called ? A. One A. Lethal gene B. XY C. Codominance C. 46. Complementary DNA B. XXY trisomy B. Blue printing 53 Enzyme that influences DNA synthesis that binds to one strand of 61 Which is the largest known gene in human genome ? DNA in 3' to 5' direction. Two B. a recessive gene can express either in homozygous condition. Familial syndrome ? A. Satellite DNA 50 Which of the following is true when a colour blind man marries the daughter of a colour blind person ? 58 What is the name for the study of ‘physical appearance of A. 47. DNA Polymerase B. Propagation . All sons will be colour blind B. Cytogenetics C. Cytokinesis D.830 MCQ’s FOR MEDICAL PROFESSIONALS BY PROF. who had earlier onset than the grandparent is ? B. MEN1 C. Both parents are colour blind B. Turner syndrome 54 Which of the following DNA is present only in female gametes ? A. Finger printing B. None of the above D. Marfan syndrome D. Some sons and some daughters will be colour blind C. X chromosome of male C. using single-stranded DNA as a template A. Nuclear DNA B. Epistatic gene D. Linked gene C. Plasmid DNA D. CFTR B. Down syndrome B. Dystrophin is ? A. Autosomal pair separate during meiosis and randomly distribute to the germ cells ? 57 Which amongst the following DNA is synthesized in laboratories A. Anticipation C. Recessive 65 Through which biological process do two pieces of a chromosome D. G0 B. Telophase have ? 74 The formation of Chiasma is seen during ? A. the base is Cytosine the other helix will D. Hypotonia 75 A single cell gives rise to 32 cells by dividing mitotically. Diplotene B. Cytoplasm B.4 Å B. 4 D. G2 phase 77 How many cells will be produced if it divides mitotically 6 times D. Prophase C. S stage ? 69 Formation of cell plate is completed in which phase of cell cycle ? A. Kinds of bond C. Same amount of DNA & half number of chromosomes C. Phosphate is halved is known as ? B. Meiotic II 80 tRNA is found in ? D. Meiotic I D. Membranes A. Telomere D. None of the above C. 8 67 DNA replication takes place in which phase of cell cycle ? D. Purines B. 34 Å C. Leptotene 66 All are classical features of Down syndrome except ? A. G0 phase those of parent cell B. Loss of memory B. G2 A. 64 B. Uracil A. Metaphase 82 If on one helix of DNA. Growth period B. 3. G1 76 The two cells formed during mitosis contain ? C. All of the above 78 Two daughter cells produced at the end of meiotic I will have ? 70 Period between two successive mitotic divisions is ? A. Same amount of DNA & different set of chromosomes as A. Cell division C. 5 C. 200 Å A. 128 C. G1 phase D. Half amount of DNA & same number of chromosomes A. Interphase C. AJAY MATHUR Cardiology 831 C. Cleavage B. Nucleus C. RNA and of parent enzymes take place in cell cycle during ? C. 831 Miscellaneous MCQ’s FORMCQ’s FOR MEDICAL MEDICAL PROFESSIONALS PROFESSIONALS BY PROF. How B. S B. Half amount of DNA & same set of chromosomes as those of parent cell D. Growth phase D. Zygotene D. 20 Å 73 The chromosomes get aligned at the equator during ? B. Same amount of DNA & same number of chromosomes D. Guanine . 256 D. Synthetic period 79 DNA and RNA molecule differ in which of the following ? 71 The phase of cell division during which the chromosome number A. Mitochondria 72 Chromatids of a chromosome are attached at a point called ? B. Mitotic phase B. Centromere 81 Width of a DNA molecule is ? D. Separation C. Same amount of DNA & same set of chromosomes as those 68 The formation of structural & functional proteins. Centrosome A. Pachytene D. Mental retardation many times has it undergone mitotic division ? C. Interphase A. Centriole C. Sugar A. Mitosis C. 16 A. Anaphase D. Gap period D. Half amount of DNA & half number of chromosomes B. Palmar crease A. 32 A. Zygotene D. Kallmann syndrome 90 Which amongst the following is the most common ‘Robertsonian 98 Which of the following is not a ‘X-linked’ disorder ? translocation’ reported in patients ? Harrison’s 16th Ed. X-linked Dominant B. 16 B. rob(13q14q) A. Cytosine C. but in which of the A. Occurs due to mutation 85 What proportion of genes is common between ‘First cousins’ ? A.832 MCQ’s FOR MEDICAL PROFESSIONALS BY PROF. Hereditary nonpolyposis colon cancer D. AJAY MATHUR Miscellaneous C. rob(15q16q) 83 In a RNA molecule. 390 A. Angelman syndrome A. what A. Telophase A. Charcot-Marie-Tooth syndrome type 1A D. G6PD deficiency C. 1/4 D. Birds D. Amphibians D. Duchenne & Becker muscular dystrophy . Maternal non-disjunction. DiGeorge syndrome 86 Assuming that hemolytic jaundice in man is due to a dominant gene. non-disjunction D. 12 to 16 89 At which stage of Meiosis I. Pachytene B. at second meiotic division B. Reptiles C. Thymine C. Hemophilia A B. is caused due to error in ? Harrison’s 16th Ed. 1/10 D. Male child normally gets these genes from ? B. 4 to 6 B. Paternal non-disjunction. 1 96 Chorionic villus sampling (CVS) is usually done at which week of gestation ? 88 Trisomy 18 (Edward’s syndrome). Guanine A. what are the chances A. 1/16 C. Beckwith-Wiedemann syndrome B. None of the above 84 Duchenne & Becker muscular dystrophy is an example of what kind of inheritance ? 92 Genes controlling secondary sexual characters are located on X A. Thymine B. Only father C. 379 A. Autosomal Dominant chromosome. X-linked Recessive C. rob(14q21q) B. Autosomal Recessive A. 379 87 In a family has 4 children. Post zygotic. 0 C. Cytosine D. rob(21q21q) D. 3 boys and 1 girl. 12 A. Uracil following XX is for male and XY is for female ? B. 1/2 D. at first meiotic division A. the chromosomes form a ‘Quadrivalent’ 97 Which of the following is not a ‘X-linked’ disorder ? ? Harrison’s 16th Ed. Only mother D. Prader-Willi syndrome proportion of the children would develop hemolytic disease ? B. Angelman syndrome D. the base adenine compliments with ? 91 Normally XX is for female and XY is for male. 1/20 95 Amniocentesis is usually done at which week of gestation ? Harrison’s 16th Ed. 6 to 9 C. 9 to 12 D. 1/4 A. 20 C. 390 A. Prader-Willi syndrome C. 1/4 D. Both the parents D. 1/8 B. Maternal non-disjunction. but only 10% of people actually develop disease. Diplotene C. 1/2 C. Adrenoleukodystrophy B. 8 that the next baby will be a girl ? B. If a 94 Which of the following is not a microdeletion syndrome ? heterozygous man marries a homozygous normal female. at first meiotic division C. DiGeorge syndrome C. 1/2 93 Which of the following is not a microdeletion syndrome ? B. > 200 / µL 103 In a HIV-infected individual. Human metapneumoviruses 104 The earliest documented case of HIV infection in humans was D. Dr. < 250 / µL D. 1506 HIV-infected individual with a CD4+ T cell count of <200/µL has AIDS by definition. RNA virus D. Kallmann syndrome B. Gaetan Dugas C. David Ho C. A3 D. Zaire B. Any of the above Chapter 189. < 200 / µL C. With accompanying illness/history of acute HIV infection D. 1506 Table 189-1 B. Fauci 100 Which of the following is not an ‘autosomal recessive’ disorder ? 107 Who is known as “patient zero” by AIDS researchers ? Harrison’s 16th Ed. Anthony S. > 500 / µL C. Uganda A. Dr. British Medical Bulletin HIV-1 & HIV-2 are cytopathic RNA viruses. Zoonotic infection N Engl J Med 2001. 1506 C. Congo 112 Which of the following is false about HIV virus ? B. 21-Hydroxylase deficiency C. CD4+ T cell count is ? Harrison’s 18th Ed. Wilson’s disease D. And. Journal of Clinical Virology 99 Which of the following is not an ‘autosomal recessive’ disorder ? Harrison’s 16th Ed. A. > 400 / µL B. Familial breast and ovarian cancer A. regardless of A. HIV-1 & HIV-2 are zoonotic infections. Polycystic kidney disease B. when which of the 101 Which of the following is not an ‘autosomal dominant’ disorder ? following conditions is present ? Harrison’s 16th Ed.344:1764-72 D. AJAY MATHUR Cardiology 833 C. 390 106 Who is credited with the discovery of virus that causes AIDS ? A. 1 Antitrypsin deficiency D. Retroviridae the presence of symptoms or opportunistic diseases. Any of the above 102 Which of the following is not an ‘autosomal dominant’ disorder ? 109 Clinical category in asymptomatic HIV infection with CD4+ T cell Harrison’s 16th Ed. C. Morbidity and Mortality Weekly Report D. Kallmann syndrome C. Ryan White A. 390 Harrison’s 18th Ed. Hemochromatosis B. HIV / AIDS 110 In Clinical Category A1.350:1872-80 and the subfamily of lentiviruses. Long QT syndrome 108 Documented HIV infection is Category A. Lentiviruses cell-mediated immunity (category C). The Lancet D. 1507 Table 189-2 A. Dr. 390 count <200/µL is ? A. Cytopathic virus 105 AIDS was first reported in which of the following ? C. 1506 B. Hereditary nonpolyposis colon cancer C. Wilson’s disease A. A2 D. Arthur Ashe D. in anyone with HIV infection who develops one of the HIV-associated diseases considered to be indicative of a severe defect in B. Zambia Harrison’s 18th Ed. Familial Parkinson’s disease C. Persistent generalized lymphadenopathy C. Coronaviruses identified in 1959 in ? The etiologic agent of AIDS is HIV which belongs to the family of human retroviruses (Retroviridae) N Engl J Med 2004. Hyperkalemic periodic paralysis D. None of the above A. 21-Hydroxylase deficiency B. Clayton A. Earvin “Magic” Johnson B. 833 Miscellaneous MCQ’s FORMCQ’s FOR MEDICAL MEDICAL PROFESSIONALS PROFESSIONALS BY PROF. Dr. . Hypertrophic cardiomyopathy C. A1 C. 1506 Table 189-1 A. Asymptomatic HIV infection B. > 300 / µL A. Malignant hyperthermia A. < 300 / µL 111 HIV belongs to the subfamily of ? D. Robert Gallo B. Buck D. AIDS is present if CD4+ T cell count is ? Harrison’s 18th Ed. < 350 / µL Harrison’s 18th Ed. Marfan syndrome Harrison’s 18th Ed. Familial Mediterranean fever B. 390 A. 45-kDa protein A. 65-kDa protein . A. Rhesus macaque 121 The glycoprotein 41 subunit of HIV envelope protein has which D. Monomer A.160 nm C. 1507 120 Which are the two major envelope proteins of HIV ? A.200 nm A. 1507 A. 35-kDa protein Harrison’s 18th Ed. C. D. 72 spikes D. 10 to 20 percent 115 The diameter of mature HIV virion is ? B. 96 spikes 126 CD4 molecule is a ? Harrison’s 18th Ed. Glycoprotein 24 119 Lipid bilayer of HIV mature virion includes aproximately ? Journal of Clinical Virology 2005 C. gp 100 C.351:743 Harrison’s 18th Ed. All of the above 122 What percentage of homology exists between HIV-2 DNA and HIV- The Pan troglodytes troglodytes species of chimpanzees is the natural reservoir of HIV-1 (M and 1 DNA ? N groups) and source of original human infection.CD4 molecule. gp 80 B.180 nm 123 Which of the following statements about HIV-2 is true ? Harrison’s 18th Ed. pH independent virus 124 Which viral-envelope glycoprotein of HIV-1 is required for 117 The HIV envelope protein is a ? attachment of virions to the CD4 cellular receptors ? N Engl J Med 2004. 20 to 40 percent Journal of Clinical Virology 2005 C. gp 120 D. Sooty mangabey monkeys proteins . AJAY MATHUR Miscellaneous 113 HIV-1 came to humans from ? C. 1506 A. gp41 penetrates the plasma membrane & then coils upon itself to bring virion & target cell together. Spherical 125 Which component of HIV is first inserted into the membrane of B. 60 to 80 percent B. 1506 B. Glycoprotein 160 B.120 nm D.834 MCQ’s FOR MEDICAL PROFESSIONALS BY PROF.350:1872-80 C. HIV-2 is more closely related phylogenetically N Engl J Med 2004. 1506 D. Trimer C. 1508 D.350:1872-80 to simian immunodeficiency virus (SIV) found in sooty mangabeys than it is to HIV-1. Rhesus macaque D. All of the above D. Glycoprotein 41 B. All of the above of the following component ? N Engl J Med 2004. Helical region 1 B. Helical region 2 C. Glycoprotein 160 and glycoprotein 39 subunits A Chimpanzee HIV virion is an icosahedral structure with numerous external spikes formed by two major envelope B. HIV-2 has vpu gene and not vpx gene 116 HIV is a ? B. gp 140 Replication cycle of HIV begins with the high-affinity binding of the gp120 protein via a portion of its 118 Nucleocapsid of HIV virion is ? V1 region near N terminus to its receptor on host cell surface . 55-kDa protein B. 24 spikes D. 140 . Upon binding to CD4. C. 60 spikes After binding of viral envelope gp120 to CD4 molecule. 180 . Acidic pH dependent virus D. HIV-2 does not have vpx and vpu gene B. A. Glycoprotein 120 and glycoprotein 39 subunits D. HIV-2 has both vpx and vpu gene A. Journal of Clinical Virology 2005 gp120 undergoes confor-mational change that facilitates binding to CCR5 & CXCR4 co-receptors. HIV-2 has vpx gene and not vpu gene N Engl J Med 2004. 1507 D. Dimer B. 100 . N-terminal hydrophobic domain termed fusion peptide A Chimpanzee B. 160 . Cylinder-shaped Harrison’s 18th Ed.external gp120 and transmembrane gp41. Alkaline pH dependent virus The major difference between the genomes of HIV-1 and HIV-2 is the fact that HIV-2 lacks the vpu C. Cone-shaped the target cell ? C. Elliptical-shaped A.351:744 114 HIV-2 originated from ? Harrison’s 18th Ed. Tetramer D. Neutral pH dependent virus gene and has a vpx gene not contained in HIV-1. Sooty mangabey monkeys C. Glycoprotein 160 and glycoprotein 41 subunits Harrison’s 18th Ed. Glycoprotein 120 A. 40 to 60 percent A. Glycoprotein 120 and glycoprotein 41 subunits C. D. G protein–coupled receptors properties ? Harrison’s 18th Ed. 1507 B. composed of viral RNA and viral enzymes and 138 HIV virus preintegration complex docks to nuclear membrane surrounded by a capsid protein coat. All of the above D. Neither A. viral reverse the virus ? transcriptase enzyme catalyzes reverse transcription of genomic RNA into DNA and protein coat Harrison’s 18th Ed. Vif D. 1507 is counteracted by ? A. Level of plasma viremia directly reflects virus production in lymphoid tissue. lymphoid tissues are the major anatomic sites for the establishment and propagation of HIV infection. the preintegration complex. Lymphoid tissue C. HIV-1 vpu D. A. HIV-1 Nef B. 1507 134 HIV viral uncoating involves which of the viral proteins ? A. 1507 135 Which of the following HIV gene products possess antiapoptotic A. SIGN that bind with high affinity to HIV gp120 envelope protein. Capsid protein coat C. HIV protects itself from APOBEC through its viral protein Vif that targets APOBEC. Nef D. 835 Miscellaneous MCQ’s FORMCQ’s FOR MEDICAL MEDICAL PROFESSIONALS PROFESSIONALS BY PROF. All of the above D. Vpr 130 Which of the following cells express C-type lectin receptors on 136 HIV virus replicates mainly in ? Harrison’s 17th Ed. Dendritic cells Regardless of the portal of entry of HIV. A subset of T lymphocytes Harrison’s 18th Ed. and not in blood. 137 The HIV viral RNA genome is converted into a full-length double- 131 Preintegration complex is composed of ? stranded DNA by viral RT by a process called ? Harrison’s 18th Ed. HIV-1 vpr CD4 molecule is a 55-kDa protein found on a subset of T lymphocytes that are responsible for helper function in immune system. causing hypermutation of HIV genomes. is released into the cytoplasm of the target cell. When dendritic cells engage with CD4+ T cells. APOBEC proteins bind to nascent reverse transcripts and deaminate viral cytidine. During reverse transcription of genomic RNA into DNA C. and is directed by ? Journal of Clinical Virology 2005 132 Protein coat of the preintegration complex opens ? Harrison’s 18th Ed. 1507 Journal of Clinical Virology 2005 A. Bone marrow B. 1527 B. Retrotranscription B. HIV-1 vpu C. HIV-1 Vif C. CXCR4 A. Macrophages D. HIV-1 vif D. Before reverse transcription of genomic RNA into DNA B. Dendritic / Langerhans cells B. 1153 their surface ? Harrison’s 18th Ed. Blood A. All of the above C. dendritic/Langerhans cells. A+B B. 128 Major co-receptors for HIV-1 is ? Harrison’s 18th Ed. Deletion Following fusion. Monocytes / Macrophages A. one of which is called DC. Nef The two major co-receptors for HIV-1 are CCR5 and CXCR4. Tyrosine kinase receptors A. Virus replication occurs mainly in lymphoid tissue Certain dendritic cells express C-type lectin receptors on their surface. 1507 A. Monocytes C. 1508 opens to release double-stranded HIV-DNA. Tat D. All of the above 129 CCR5 & CXCR4 belong to which family of cellular receptors ? Harrison’s 18th Ed. CCR5 Journal of Clinical Virology 2005 B. AJAY MATHUR Cardiology 835 127 CD4 molecule is found on the surface of ? 133 Antiretroviral effect of the cellular protein APOBEC3G (or CEM15) Harrison’s 18th Ed. T lymphocytes B. Viral enzymes B. Viral RNA A. Duplication D. MA C. After reverse transcription of genomic RNA into DNA D. Cytokine receptors B. gag . Envelope C. It is also expressed on surface of monocytes/macrophages & APOBEC family of cellular proteins inhibits progression of virus infection after virus has entered the cell. None of the above 139 Which of the following HIV-1 genes encode structural proteins of As the preintegration complex traverses the cytoplasm to reach the nucleus. Both receptors belong to the family of seven-transmembrane-domain G protein–coupled cellular receptors. HIV-1 vpr A. Transcription C. HIV virus is presented to CD4+ T cells. Serine kinase receptors C. 1507 A. nor B C. HIV-1 Nef D. vif. None of the above pol encodes the enzymes responsible for protease processing of viral proteins. gag A. 1508 149 Which of the following is not a subtype of HIV-1 M group ? Harrison’s 18th Ed. env B.group M (major). HIV-1 N D. Viral IN A. HIV-1 O env encodes the envelope glycoproteins. B. . CRF01_AE and CRF02_AG. D. Viral KN D. pol A. group O (outlier). where the core acquires its external envelope. All of the above C. All of the above form the core of the virion ? Budding of progeny virion occurs through specialized regions in lipid bilayer of host cell membrane Harrison’s 18th Ed. A B. Viral FN C. 1509 A. as a number of major & minor circulating recombinant forms (CRFs). Viral EN B. nef. B C. env A. N D. 1508 known as lipid rafts. All of the above D. All of the above D. C. E D. D. 144 HIV viral linear double-stranded DNA in the preintegration 150 Which of the following about circulating recombinant forms (CRFs) complex is inserted into the host chromosome by ? is false ? Journal of Clinical Virology 2005 Harrison’s 18th Ed. glycoproteins ? Harrison’s 18th Ed. All of the above B. CRF01_AE is predominant in southeast Asia C. a relatively rare viral form found originally in Cameroon. N C. B.836 MCQ’s FOR MEDICAL PROFESSIONALS BY PROF. pol Harrison’s 18th Ed. 1509 A. HIV-1 is most closely related to viruses isolated from chimpanzees & gorillas. pol 145 Lipid rafts is related to ? Harrison’s 18th Ed. pol B. G and two CRFs. 1508 infections in world ? Harrison’s 18th Ed. HIV-1 M C. Generated by infection with two subtypes B. There are three groups of HIV-1 . None of the above From cytoplasm HIV viral DNA enters the nucleus through nuclear pore where it is integrated into CRFs are generated by infection in one patient with two subtypes that recombine & create a host cell chromosomes (introns of active genes & regional hotspots) through the action of virally different virus. gag the natural reservoir of ? Harrison’s 18th Ed. Host cell membrane HIV-1 has genes that encode the structural proteins of virus. O 141 Which of the following HIV-1 gene encodes enzymes responsible for reverse transcription & integration ? 147 Which of the following HIV-1 groups is responsible for most HIV Harrison’s 18th Ed. and France and group 142 Which of the following HIV-1 gene encodes the envelope N. env C. Gabon. CRF02_AG is predominant in west & central Africa D. nef C. H. first identified in a Cameroonian woman with AIDS. L D. AE virus or CRF01_AE is common in southeast Asia & is often called as E. vpr & vpu which code for proteins involved in the modification of the host cell to enhance virus growth and the regulation of viral gene The HIV-1 M group comprises nine subtypes. as well expression. 1509 A. All of the above B. C. 1508 148 The chimpanzee subspecies Pan troglodytes troglodytes is not A. 1509 C. AJAY MATHUR Miscellaneous B. reverse transcription and integration. encoded enzyme integrase (IN). Host nuclear membrane 140 Which of the following HIV-1 gene that encodes the proteins that D. or clades. HIV-1 O group is most closely related to viruses found in Cameroonian gorillas. C. F. M B. CRF02_AG is common in west & central Africa. which is responsible for most of the infections in the world. HIV-1 cell membrane D. J and K. F HIV-1 contains six genes named tat. expression ? Harrison’s 18th Ed. 1508 C. gag 146 Which of the following is not a HIV-1 group ? B. G. O D. A. Seven strains account for the majority of HIV infections globally: HIV-1 subtypes A. None of the above 143 Which of the HIV-1 genes code for proteins involved in modification The chimpanzee subspecies Pan troglodytes troglodytes is the natural reservoir of the HIV-1 M of host cell to enhance virus growth and regulation of viral gene and N groups. rev. env A. designated A. rev B. D. M gag encodes the proteins that form the core of the virion (including p24 antigen) C. tat A. 1509 B. A B. F1 & F2. the most common mode of infection with HIV is heterosexual transmission. 156 HIV is transmitted by all except ? Harrison’s 18th Ed. subcuteneous puncture (“skin popping”) casual contact or by a mosquito bite. Canada. Use of oral contraceptives C. STDs with genital ulcerations strongly associated with HIV B. D 159 Sexual transmission of HIV is rare if the infected partner has a Subtype C viruses (of the M group) are the most common form worldwide. Adolescent girls B. >50% of infections contraceptives is associated with increased incidence of HIV infection due to drug-induced changes are caused by subtype C. Intravenous puncture B. certain countries D. AJAY MATHUR Cardiology 837 151 Which of the following HIV subtypes are closely related to each 157 Worldwide. Penis & urethral orifice are exposed relatively briefly to infected vaginal fluid. Blood and blood products C. 2500 copies per milliliter A. western Europe and Australia. All of the above HIV is transmitted by both homosexual & heterosexual contact. C. Subcuteneous puncture C. 3500 copies per milliliter B. 1513 Harrison’s 18th Ed. Mosquito bite D. A Haemophilus ducreyi & herpes simplex virus are causes of genital ulcerations linked to HIV transmission. Infected mothers to infants D. 1510 Harrison’s 18th Ed. B C. 837 Miscellaneous MCQ’s FORMCQ’s FOR MEDICAL MEDICAL PROFESSIONALS PROFESSIONALS BY PROF. Sexual contact A. A B. B transmission C. D transmission is rare when infected partner has a plasma level of <1700 copies of HIV RNA per milliliter. Neisseria gonorrhoeae. Sexual D. D Male circumcision is associated with a lower risk of HIV infection among men. the most common mode of infection with HIV is ? other ? Harrison’s 18th Ed. B&D Worldwide. Adolescent girls are more susceptible to infection due to an immature genital tract with increased cervical ectopy or exposed columnar epithelium. 1513 154 Which subtype of M group of HIV-1 is most prevalent in India ? A. Asia. Receptive anal intercourse is a much less efficient mode of in South America. D risk of HIV transmission Subtype B viruses are the predominant viruses seen in United States. Infected mothers to infants C. Lack of circumcision A. 1509 A. HIV infection ? Saharan Africa ? Harrison’s 18th Ed. HIV-1 M group subtypes A & F are now subdivided into A1 & A2. There is no evidence that HIV is transmitted by Parenteral transmission of HIV can occur by IV puncture. CRF01_AE accounts for most infections in south & southeast when plasma HIV RNA levels are highest & in advanced disease as the viral set point increases. and by infected mothers to infants or via breast milk. 158 Which of the following about HIV transmission is false ? Harrison’s 18th Ed. C D. C D. 1513 A. 4500 copies per milliliter C. C Chief predictor of heterosexual transmission of HIV is the level of plasma viremia. Chlamydia trachomatis. Heterosexual transmission B. 1510 161 Parenteral transmission of HIV occurs by ? Harrison’s 18th Ed. Intramuscular puncture D. B D. The rate of HIV transmission per coital act is highest during early stage of HIV infection Subtype C is prevalent in India. A&C C. A&B B. All of the above D. 1510 prolonged exposure to infected seminal fluid of the vaginal. by blood & blood products. Homosexual transmission A. . B nonulcerative inflammatory STDs and are associated with an increased risk of HIV transmission. Oral sex is a much less efficient mode of transmission of HIV than is receptive anal intercourse. 1700 copies per milliliter Harrison’s 18th Ed. or intramuscular puncture (“muscling”). HIV transmission than oral sex 153 Which subtype of M group of HIV-1 is most prevalent worldwide ? Male-to-female HIV transmission is more efficient than female-to-male transmission due to Harrison’s 18th Ed. Nonulcerative inflammatory STDs associated with increased D. B&C D. C C. 1510 female-to-male transmission A. and Trichomonas vaginalis produce B. 1510 A. Subtypes B & D are too close to be separate subtypes & should be considered sub-subtypes. A C. 1510 B. home to ~two-thirds of all individuals living with HIV/AIDS. Use of oral In sub-Saharan Africa. plasma level of HIV RNA less than ? Harrison’s 18th Ed. Blood and blood products B. A. 1510 152 Which subtype of M group of HIV-1 is prevalent in USA ? A. Male-to-female HIV transmission is more efficient than Harrison’s 18th Ed. 160 Which of the following is strongly associated with a higher risk of 155 Which subtype of M group of HIV-1 is most prevalent in sub. Treponema pallidum. in cervical mucosa. cervical mucosa and endometrium. 35 % . plasma-derived hepatitis B vaccine and Rh o immune globulin have not been associated with transmission of HIV infection. vaginal secretions.364:1943-54 Higher rates of transmission is associated high maternal levels of plasma viremia. C.55 % 166 Eclipse phase of HIV-1 infection generally lasts for ? D. A. C. Monocyte-derived macrophages are generally poor targets for infection Harrison’s 18th Ed. CD4 T cells B. Tissues used in organ transplantation most commonly in ? C.30% in nonimmune individuals and ~1. Third trimester 165 Risk of HIV transmission following skin puncture from a needle D. packed red blood cells. Delivery Hyperimmune  globulin. submucosa.3%.25 % blood from person with HIV infection is ~0. B. sputum. 7 to 21 days 172 The best-documented factor that is associated with higher rates C. or by breast-feeding. High maternal levels of plasma viremia 167 Which of the following is a poor target for HIV-1 infection ? D. Hyperimmune  globulin 169 HIV infection can be transmitted from an infected mother to her B. and draining lymphoreticular tissues. Plasma-derived hepatitis B vaccine A. Breast milk A. pleural fluid. 48 to 72 days Harrison’s 18th Ed. transmission of HIV infection ? pericardial fluid and amniotic fluid are considered potentially infectious.838 MCQ’s FOR MEDICAL PROFESSIONALS BY PROF.25 % in industrialized countries. ~ 0. 35 . C. Low maternal CD4+ T cell counts N Engl J Med 2011. 1513 delivery.2 % 171 Without prophylactic antiretroviral therapy. Perinatal period contaminated with HIV (+) blood is ? Harrison’s 18th Ed. D. cerebrospinal fluid. 1515 Risk of HIV transmission following skin puncture from a needle or a sharp object contaminated with A. Transmission of HIV through intact skin has not been documented. Hepatitis B immune globulin fetus during ? Harrison’s 18th Ed. Rho immune globulin administration Harrison’s 18th Ed. Risk of hepatitis B virus (HBV) infection following percutaneous exposure is ~6 B. tears. 1514 C. All of the above B. 1515 D. Semen used in artificial insemination 170 Maternal-fetal / infant transmission of HIV infection to fetus occurs B. Leukocytes A. 1514 HIV can be transmitted to fetus as early as first and second trimester of pregnancy but maternal transmission to fetus occurs most commonly in the perinatal period. sweat. This eclipse phase B. STDs during pregnancy generally lasts 7 to 21 days. 1515 C. The procedures C. Monocyte-derived macrophages 173 Rate of mother-to-baby transmission is 0% in women with HIV C. 25 . Packed red blood cells 168 Which of the following is considered potentially infectious ? B.4 % Harrison’s 18th Ed. Feces D. Second trimester Semen used in artificial insemination and tissues used in organ transplantation can transmit HIV. Breast-feeding involved in processing these products either inactivate or remove the virus. peritoneal fluid. A. AJAY MATHUR Miscellaneous 162 Transfusions of which of the following is capable of transmitting CD4 T cells & Langerhans’ cells are the first targets of HIV-1 virus.364:1943-54 Without prophylactic antiretroviral therapy. 1515 A. A. probability of HIV transmission from mother to infant/ A. Other dendritic cells play an HIV infection ? important accessory role. platelets.75 % N Engl J Med 2011. None of the above 163 Administration of which of the following is not associated with Semen. ~ 0. First trimester B.1 % B. Harrison’s 18th Ed. saliva.3 % countries ? D. Urine Transfusions of whole blood. All of the above B. Pregnancy D. D. 55 . Feces. Submucosal dendritic cells A. as HIV-1 is replicating in the mucosa. ~ 0. A. the virus cannot be detected in plasma. Platelets Harrison’s 18th Ed. 21 to 42 days of maternal-fetal/infant transmission is ? D. All of the above 164 Transmission of HIV can occur due to all except ? HIV infection can be transmitted from an infected mother to her fetus during pregnancy. 1513 A. nasal secretions.35 % and 15 . Saliva factors and plasma are all capable of transmitting HIV infection. 1 to 7 days fetus in developing countries is 25 . hepatitis B immune globulin. during Harrison’s 18th Ed. 1515 D. and vomitus are not considered potentially infectious. 1513 as compared with CD4 T cells. < 1000 per milliliter of blood . urine. 10 . leukocytes.8% in hepatitis C virus (HCV) infection. synovial fluid. Presence of chorioamnionitis at delivery Immediately after exposure & transmission. ~ 0. Langerhans’ cells RNA copies less than ? Harrison’s 18th Ed. what is the probability of HIV transmission from mother to infant/fetus in developing C. concentrates of clotting C. B. Can trap HIV & mediate transinfection of CD4+ T cells perinatal transmission of HIV.50. All of the above A. Express C-type lectin receptors (DC-SIGN) Zidovudine treatment of HIV-infected pregnant women from the beginning of second trimester through delivery and of infant for 6 weeks following birth decreases rate of intrapartum and C. ~50% are female.001 . ~ 10 % D. Harrison’s 18th Ed. D.350:1872-80 via breast-feeding is ? A. Expressed by Langerhans cells Harrison’s 18th Ed.3% with 10. low maternal CD4+ T cell counts & maternal vitamin A deficiency. ~ 33 % Rate of mother-to-baby transmission was 0% among women with <1000 copies/mL of blood of HIV RNA. DC-SIGN binds with high affinity to HIV envelope gp120 A. CD8+ C. < 10000 per milliliter of blood 179 What proportion of HIV/AIDS patients are females ? Harrison’s 18th Ed. Heparan sulfate proteoglycans . Zidovudine C. persistent infection are also hallmarks of HIV infection. Secretory leucokine protease inhibitor (SLPI) Secretory leukocyte protease inhibitor (SLPI) blocks HIV infection.350:1872-80 C. Facilitates CD4-mediated infection of dendritic cells B. 183 Receptor that is abundantly expressed by macrophages facilitating HIV infection is ? 177 Risk factor that increases the likelihood of transmission of HIV N Engl J Med 2004. Submandibular saliva reduces 185 Which of the following is essential for efficient fusion & entry of HIV infectivity by stripping gp120 from surface of virions. Avoidance of breast-feeding N Engl J Med 2004.350:1872-80 transmission of HIV ? A. All of the above A. ~ 66 % 174 Increased mother-to-child HIV transmission is correlated with ? Among people living with HIV/AIDS.000 copies/mL. None of the above transmission. 21.350:1872-80 D.000 copies/mL. All of the above DC-SIGN (dendritic cell-specific intercellular adhesion molecule 3-grabbing nonintegrin) is expressed on dendritic cells. 1519 A. Capture antigens in bone marrow D. 1519 Risk factors for mother-to-child transmission of HIV via breast-feeding include detectable levels of HIV A. 1515 B. Moderately susceptible to HIV infection because they A. 1516 C. Obstetric management that minimizes exposure of infant to maternal blood & genital secretions and avoidance of breast-feeding also reduce D.6% with >100. CXCR4 & CCR5 B. Secretory lymphocyte protease inhibitor (SLPI) HIV is an RNA virus whose hallmark is the reverse transcription of its genomic RNA to DNA by the B. Langerhans cells belong to the family of Dendritic cells HIV (+) pregnant women seen by health care provider for the first time at or near the time of delivery can be given zidovudine with lamivudine.9% with C. DC-SIGN allows efficient infection by aggregating virus on C.100. Aberrant immune activation in breast milk. < 1000000 per milliliter of blood B. Harrison’s 18th Ed. 16. presence of mastitis. ~ 50 % 50.6% with 1000 . 1515 B. Profound immunodeficiency Harrison’s 18th Ed. Transport captured antigens to lymphoid organs 175 Recommendation to reduce perinatal transmission of HIV is ? C. 1515 D. Closer HLA match between mother and child N Engl J Med 2004. Profound immunodeficiency resulting from a progressive C. D.000 copies/mL. Lamivudine D. Nevirapine surface of dendritic cells D. HIV is an RNA virus whose hallmark is the reverse transcription of its genomic RNA to DNA by the enzyme reverse transcriptase. Cesarean section delivery 181 Which of the following about Dendritic cells is false ? C.001 . 1515 D. All of the above B. < 100000 per milliliter of blood A. Nevirapine can be given as a single dose to mother at the onset of labor and a single dose to infant within 72 hours of birth. CD4 molecule . and 40. Aberrant immune activation and Chronic. 839 Miscellaneous MCQ’s FORMCQ’s FOR MEDICAL MEDICAL PROFESSIONALS PROFESSIONALS BY PROF. CD90 B.10. 30. persistent infection 178 Soluble salivary factor that inhibits HIV is ? C.000 copies/mL.Syndecan Harrison’s 18th Ed. Zidovudine treatment from II trimester onwards express low levels of CD4. Secretory lymphokine protease inhibitor (SLPI) quantitative & qualitative deficiency of helper T cells. Chronic. 1515 A. All of the above 184 Which of the following is the hallmark of HIV infection ? Harrison’s 18th Ed. AJAY MATHUR Cardiology 839 B. Secretory leukocyte protease inhibitor (SLPI) enzyme reverse transcriptase. Maternal vitamin A deficiency D. DC-SIGN A. Low maternal CD4+ T cell counts 180 Which one of the following statements about ‘Dendritic cells’ is false ? B. Detectable levels of HIV in breast milk C. Presence of mastitis D. Digest captured antigens & display resulting peptides to T cells Harrison’s 18th Ed. Play a role in initiation of HIV infection B. Disruption & lysis of HIV-infected cells HIV-1 into its target cells ? occurs due to hypotonicity of oral secretions. 182 Which of the following about DC-SIGN is false ? 176 Which of the following ARV is used to reduce perinatal N Engl J Med 2004. Prolonged interval between membrane rupture & delivery A. ~ 120 . latent or otherwise. 1523 C. A. HBV and HCV infections. extensive glycosylation of the envelope & conformational masking of neutralizing epitopes. Harrison’s 18th Ed.90 minutes 188 Slope of HIV disease progression correlate with the set point of the level of steady-state plasma viremia after how many months C. 1522 196 With ARV therapy. HBV infection Harrison’s 18th Ed. 1523 B. which 191 Mechanism by which HIV evades neutralizing responses is ? serves as the primary cellular receptor for HIV. ~ 6 months 194 What quantity of HIV virions are produced and cleared from the C. disappearance of clones of CD8+ CTLs. All of the above CD4 + T cell is defined phenotypically by the presence on its surface of CD4 molecule. Levels of viremia generally increase and level of CD4+ T cells in the blood decreases as disease progresses. exist in lymphoid tissue. 1521 D. High rate of replication & continual mutation of virus A. the immune system ? Harrison’s 18th Ed. 1522 . ~ 10 % Envelope proteins gp120 and gp41 are the principal targets of neutralizing antibodies against HIV. CCR5 and the continual mutation of virus. decrease in virus replication occurs in ? A. Viral set point at ~1 year has important prognostic implications for the B. Any of the above Harrison’s 18th Ed. D. ~ 50 % D. Conformational masking of neutralizing epitopes Harrison’s 18th Ed. They are selection D.840 MCQ’s FOR MEDICAL PROFESSIONALS BY PROF. C. ~ 1013 – 10 14 virions 189 Chronicity associated with persistent virus replication is seen in ? Through studies of relatively steady level of plasma viremia and of infected cells. Serum sickness like symptoms D. Lymphoid tissue 187 Acute HIV syndrome symptoms resemble which of the B. ~ 7 days C. 1521 C. Selection of mutants that escape control by CTLs Harrison’s 18th Ed. Lymph nodes D. it is shown that ~10 10 . ~ 90 . Flu-like symptoms Reservoirs of HIV infected cells. HIV-infected individuals who have a low set point at 6 months to 1 year C. ~ 1011 – 10 12 virions progression of HIV disease. Acute mononucleosis-like symptoms 193 Half-life of a circulating HIV virion is ? D. CXCR4 of HIV. 1523 D. 1523 Acute mononucleosis-like symptoms are well correlated with the presence of HIV viremia. ~ 2 days D. sequestration of infected cells in immunologically privileged sites like CNS and formation of a pool of latently infected cells.10 11 virions are produced and cleared from the circulation each day.120 minutes of infection ? D. ~ 30 . Extensive glycosylation of the envelope experience the “acute HIV syndrome” ? C. D. HCV infection A. All of the above C. Hypervariability in primary sequence of the envelope 186 What proportion of individuals with primary HIV infection B. ~ 21 days 190 What is the mechanism of HIV infection to evade elimination by The minimal duration of the HIV-1 replication cycle in vivo is ~2 days. ~ 25 % employs three mechanisms to evade neutralizing responses: hypervariability in primary sequence of the envelope. AJAY MATHUR Miscellaneous B. Peripheral blood Initial burst of viremia brings about several processes that help HIV survive. ~ 3 months B. A. 1521 Half-life of a circulating virion is ~30–60 min and that of productively infected cells is 1 day. C. Spleen C. HIV uses two major co-receptors for fusion and entry Harrison’s 18th Ed. ~ 14 days Chronicity associated with persistent virus replication is seen in HIV. ~ 12 months circulation each day ? Harrison’s 18th Ed. 1522 A. All of the above of mutants that escape control by CD8+ cytolytic T lymphocytes (CTLs). Disappearance of clones of CD8+ CTLs B. high rate of virus replication . CNS A. ~ 75 % 192 Which of the following acts as a latent reservoir of HIV infected ~ 50 % of individuals with primary HIV infection experience the “acute HIV syndrome”. Peripheral blood following ? Harrison’s 18th Ed.CCR5 and CXCR4 (seven-transmembrane-domain G protein–coupled family of receptors). 1522 A. ~ 60 . D.180 minutes Harrison’s 18th Ed. activation of apoptosis. ~ 1010 – 10 11 virions Level of steady-state viremia is called the viral set point which correlate with the slope of HIV disease progression. ~ 1012 – 10 13 virions progress to AIDS much more slowly than individuals whose set point is very high. elimination of memory CD4+ T cells in GALT.60 minutes B. Harrison’s 18th Ed. peripheral blood & CNS. All of the above B. They have cells ? high levels of viremia. HIV B. All of the above A. HIV infection 195 Minimal duration of the HIV-1 replication cycle in vivo is ? B. downregulation of HLA class I molecules on the surface of HIV-infected cells by the Nef protein C. A. All of the above B. ~ 18 months A. Previously. Infected with HIV for a long period (>10 years) B. By parenteral route CD4+ T cell counts in normal range. Received ARV therapy Harrison’s 18th Ed. 1524 and CCR5 is the major co-receptor for R5 or macrophage-tropic strains of HIV. ~ 3 years 204 HIV individuals co-infected with which of the following have B. A. C. A. 1525 C. Regardless of the portal of entry of HIV. All of the above are by definition long-term survivors. Long-term nonprogressors D. 1522 heterozygosity for RANTES-28G mutation. HLA B*3701 . All of the above replication ? The earliest burst of virus replication occurs in gut-associated lymphoid tissue (GALT) associated Harrison’s 17th Ed. but detectable levels of plasma viremia A. A. 841 Miscellaneous MCQ’s FORMCQ’s FOR MEDICAL MEDICAL PROFESSIONALS PROFESSIONALS BY PROF. In general. 1524 mortality than HIV-infected individuals without GBV-C infection. ~ 5 years lower mortality ? C. who had not received ARV therapy. homozygosity for SDF1-3’A mutation and Harrison’s 18th Ed. 1528 C. ~ 15 years A. Heterozygosity for SDF1-3’A mutation D. lymphoid tissues are the major anatomic sites for the establishment and propagation of HIV infection. Heterozygosity for CCR5-32 deletion D. Mother to infant transmission node biopsy. 1552 D. CD4+ T cell counts in the normal range 206 Lymphoid tissues are the major anatomic sites for establishment & propagation of HIV infection when portal of entry of HIV is ? C. < 100 copies per milliliter Harrison’s 18th Ed. HCV in the developed world is ~10 years. Gut-associated lymphoid tissue (GALT) 202 Which of the following plays a direct role in restriction of virus D. HBV The median time from primary HIV infection to the development of AIDS in untreated individuals B. SDF-1 is the natural ligand for CXCR4 D. HLA B*4701 in lymph nodes. CCR5 is the major co-receptor for R5 or macrophage-tropic infection. < 300 / µL A. < 50 copies per milliliter 207 Earliest burst of virus replication occurs in ? B. RANTES (CCL5) expression is the natural ligand for CCR5 Harrison’s 18th Ed. confirming that lymphoid tissue is the main site of HIV replication and the main source of plasma viremia. with CD4+ T cell counts ? Harrison’s 18th Ed. 1524 D. Heterozygosity for CCR2-64I mutation CDC case definition of AIDS includes all HIV-infected individuals with CD4+ T cell counts < 200 / µL. AJAY MATHUR Cardiology 841 Decrease in plasma viremia after ARV therapy correlates closely with a decrease in virus replication B. < 150 copies per milliliter A. < 200 / µL include all except ? B. HLA B*5701 D. 1543 tissue. < 200 copies per milliliter B. Heterozygosity for RANTES-28G mutation 198 Median time from primary HIV infection to development of AIDS Host genetic factors that delay the progression of HIV disease include heterozygosity for CCR5- in untreated patients in developed world is ? 32 deletion. 1524 203 Host genetic factors that delay the progression of HIV disease A. ~ 10 years Harrison’s 18th Ed. normal-appearing lymphoid tissue architecture on lymph C. HLA B*6701 197 CDC case definition of AIDS includes all HIV-infected individuals HLA B*5701 or HLA B*2705 alleles plays a direct role in restriction of virus replication. 1153 with marked depletion of CD4+ T cells. A. A. strains of HIV 200 Which of the following is not a characteristic feature of long-term D. the reverse is not always true. None of the above nonprogressor ? SDF-1 is the natural ligand for CXCR4. normal immune function. Blood Elite nonprogressors have HIV RNA levels < 50 copies per milliliter. 1528 C. C. 5 205 Which of the following statements is false ? B. 15 A. relatively low detectable levels of plasma viremia. Virus replication occurs mainly in lymphoid 201 Elite nonprogressors have HIV RNA levels of ? tissue and not in blood and level of plasma viremia reflects virus production in lymphoid Harrison’s 18th Ed. both humoral (neutralizing antibodies) and cell-mediated (HIV-specific CTLs). 10 Harrison’s 18th Ed. < 250 / µL Harrison’s 18th Ed. B.15 years. however. Peripheral lymphoid tissue D. heterozygosity for CCR2-64I mutation. it was 10 . 20 B. All of the above alive for how many years after initial infection ? Individuals co-infected with HIV and hepatitis G virus also called GB virus C (GBV-C) have lower Harrison’s 18th Ed. < 350 / µL B. GB virus C 199 Individuals are considered to be long-term survivors if they remain D. long-term nonprogressors manifested robust HIV-specific immune responses. By sexual route Characteristic features of long-term nonprogressor are : infected with HIV for a long period (>=10 years). C. RANTES is the natural ligand for CCR5 Individuals are considered to be long-term survivors if they remain alive for 20 years after initial C. C. Relatively low. Epstein-Barr virus (EBV) D. the trapped virions serve as ? Harrison’s 18th Ed. HSV types 1 and 2 210 Which of the following help follicular dendritic cells trap HIV ? D. thus driving further virus replication. Aberrantly activated immune system A. B. HIV replication in blood replication ? Harrison’s 18th Ed. Interleukin  (IL-1) HIV-infected individuals. Tat which upregulate virus replication in infected cells. A. Malaria In follicular or germinal centers of lymph nodes. 1527 D. 1525 Nef has been shown to possess antiapoptotic properties. Secretion of Tumor necrosis factor  to apoptosis include all except ? D. Harrison’s 18th Ed. Tat C. Tat & Vpr. Vpr 212 As the HIV disease progresses. 1525 nematodes is associated with a heightened state of immune activation that facilitates HIV replication. apoptosis is seen in “bystander” cells such as CD8+ T cells and B cells as well system in HIV infection ? as in CD4+ T cells. Nef happen ? Harrison’s 18th Ed. Cytomegalovirus (CMV) Persistence of trapped virus after the transition from acute to chronic infection reflects a steady state whereby trapped virus turns over and is replaced by fresh virions. which of the following events D. Autoimmune-like phenomenon C. Macrophages 215 Which is the most common opportunistic infection in HIV-infected C. All of the above 213 Which of the following is false regarding activation of immune In HIV infection. Vpr 211 In HIV infection. Follicular dendritic cells (FDCs) individuals ? Harrison’s 18th Ed. Infestation with nematodes Harrison’s 18th Ed. Env secretion of proinflammatory cytokines such as interleukin-1 . Germinal centers of lymph nodes disrupt 218 In HIV infection. Secretion of Interleukin -6 217 HIV viral gene products associated with enhanced susceptibility C. AJAY MATHUR Miscellaneous 208 Which of the following serves as a continual activator of CD4+ T D. which are continually produced to a C. trapped virions serve as a persistent source of cellular activation. Monocytes B. CD8+ T cells As HIV disease progresses. Mycobacterium tuberculosis is probably the most common opportunistic infection in A. Secretion of Interleukin  HIV viral gene products associated with enhanced susceptibility to apoptosis include Env. A. All of the above A. D. 1525 A. Mycobacterium tuberculosis lymphocytes in the germinal center. C. All of the above A. HIV replication in GALT 214 Which of the following can enhance cellular activation and HIV B. extracellular virions are trapped on follicular dendritic cells (FDCs) which form a fine network of long. 1526 C. Autoimmune phenomenon . human herpesvirus (HHV) 6. Complement to apoptosis include ? Harrison’s 18th Ed. architecture of germinal centers lymph nodes show disruption and eventually B. fingerlike processes that envelop B. All of the above B. trapping efficiency of lymphoid tissue diminishes & FDCs undergo cell death. HBV. therefore deworming of the infected host results in a decrease in plasma viremia. All of the above B. Trapped HIV virus A. apoptosis is seen in ? B. 1527 C. D. Harrison’s 18th Ed. 1525 D. Trapping efficiency of lymphoid tissue diminishes Harrison’s 18th Ed. The trapped virus. FDCs undergo cell death A.842 MCQ’s FOR MEDICAL PROFESSIONALS BY PROF. adenovirus. cytomegalovirus (CMV). 1527 In HIV infection. Chronically activated immune system Harrison’s 18th Ed. Tumor necrosis factor  (TNF-) 216 HIV viral gene products associated with enhanced susceptibility C. Secretion of proinflammatory & immunoregulatory cytokines cells ? Lymph node hyperplasia occurs particularly early in the course of disease. B. All of the above Co-infection with HSV types 1 and 2. C. virions that have bound complement components on their surfaces attach to the surface of FDCs via interactions with complement receptors. Lymph node hyperplasia in later course of disease B. serves as a continual activator of CD4+ T cells. Infestation with Harrison’s 18th Ed. B. All of the above A. tumor necrosis factor. 1527 B. and HTLV-I upregulates HIV expression. Epstein- 209 Trapping of HIV is best related to which of the following ? Barr virus (EBV). resulting in A. Env In follicular or germinal centers of the lymph nodes. 1526 219 Immune reconstitution inflammatory syndrome (IRIS) is an ? A. CD4+ T cells D. and IL-6 B. B cells get “burnt out”. 1525 Globally. All of the above Harrison’s 18th Ed. 1526 D. HBV greater or lesser degree in individual patients. either as whole virion or shed envelope. Increased intracranial pressure beta). 1528 of HIV by blocking the binding of HIV virus to its co-receptors. SDF-1 is the natural ligand for CXCR4. IL-6 T cell counts ? D. Regulated upon activation normal T-cell expressed & secreted B. Stromal cell deficient factor B. D. CXC chemokines B. TNF-alpha. Hypothyroidism 228 Which of the following has a HIV-inducing effect at viral transcription level in NF-kB-independent manner ? IRIS may present anywhere from 2 weeks to 2 years after initiation of HAART and can include localized lymphadenitis. RANTES CC-chemokines macrophage inflammatory protein (MIP)-1alpha (CCL3). CX3C chemokines C. < 100 cells / µL colony-stimulating factor (GM-CSF). Globulin binding proteins Stromal cell derived factor (SDF) 1 inhibits infection by & spread of X4 strains by blocking the D.354:610-21 230 RANTES stands for ? N Engl J Med 1998. Type III hypersensitivity reactions alpha. Stromal cell derived factor A. B. IL- 6. < 200 cells / µL 227 Which of the following cytokines suppress HIV replication ? D. A. Stromal cell dominant factor C. Macrophage inflammatory protein (MIP)-1 HIV viruse ? C. IL-6 A. CC chemokines A. and IL-6 are the most consistent & potent inducers A.338:438 A. and MIP-1 (CCL4) inhibit infection of R5 strains Harrison’s 18th Ed. 1556 D. RANTES Harrison’s 18th Ed. 843 Miscellaneous MCQ’s FORMCQ’s FOR MEDICAL MEDICAL PROFESSIONALS PROFESSIONALS BY PROF. Harrison’s 18th Ed. Harrison’s 18th Ed.354:610-21 B. < 50 cells / µL of HIV expression. Interferon-alpha therapy with CD4+ T cell counts < 50 cells / µL. TNF-alpha 222 Underlying mechanism of IRIS is a phenomenon similar to ? B. IL-3. 1528 IRIS is an autoimmune-like phenomenon characterized by paradoxical deterioration of preexisting. GM-CSF and IFN-gamma regulate HIV expression mainly by posttranscriptional mechanisms. Any of the above of HIV expression ? Harrison’s 18th Ed. AJAY MATHUR Cardiology 843 C. A. Released upon activation normal T-cell expressed & secreted C. C. IL-1beta 220 IRIS is most common in patients starting ARV therapy with CD4+ C. MIP-1 (CCL3). increased intracranial pressure. IL-1beta. TNF-beta. Type II hypersensitivity reactions HIV-inducing effect of IL-beta occurs at viral transcription level in an NF-kB-independent manner. Viral load decreases with increases in CD4+ T cell counts. All of the above Harrison’s 18th Ed. the CC-chemokine receptor CCR5. SDF-1 (CCL5) inhibit infection by and spread of R5 HIV-1 strains. 224 Which is the largest family of chemotactic cytokines or chemokines ? N Engl J Med 2006. Reduced upon activation normal T-cell expressed & secreted D. IL-4. TNF-alpha untreated or partially treated opportunistic infections occurs in individuals started on ARV therapy. Apoptotic phenomenon 226 Which of the following cytokines are consistent & potent inducers D. MIP-1beta (CCL4) & RANTES B. IL-6 A. pulmonary infiltrates. and Graves’ disease. Free proteins binding of the HIV virus to its co-receptor CXC-chemokine receptor CXCR4. which activates NF-kB proteins that function as transcriptional activators of HIV expression. Other cytokines that induce HIV expression include IL-1. 1556 Proinflammatory cytokines TNF-alpha. IL-6. depending on the system involved. TNF- C. C. XCL1 chemokine D. IL-1beta Harrison’s 18th Ed. IL-10 & IFN-gamma can either induce or suppress HIV expression. Receptor upon activation normal T-cell expressed & secreted 225 Which of the following is a CC chemokine ? CC-chemokines RANTES (CCL5). Heparin binding proteins C. 1528 D. IL-2. and granulocyte-macrophage B. IL-beta Harrison’s 18th Ed. macrophage colony-stimulating factor (M-CSF). Macrophage inflammatory protein (MIP)-1 231 Which of the following cytokine inhibits both R5 and X4 strains of B. Alpha defensin . prolonged fever. Uveitis D. B. 1528 Immune reconstitution inflammatory syndrome (IRIS) is most common in patients starting ARV A. They are the natural ligands for CCR5. Type IV hypersensitivity reactions 229 SDF stands for ? Underlying mechanism of IRIS is a phenomenon similar to type IV hypersensitivity reactions. C. whereas transforming growth factor-beta (TGF- B. Type I hypersensitivity reactions D. IL-12. 1556 C. Stromal cell dependent factor N Engl J Med 2006. Albumin binding proteins D. < 350 cells / µL Harrison’s 18th Ed. 1528 uveitis. Pulmonary infiltrates Interferon-alpha and -beta suppress HIV replication. TNF-alpha 221 Presentation of IRIS includes all except ? C. A. 1528 223 Chemokines are ? A. IFN-gamma B. All of the above A. 1528 B. T cell receptor emptying cycles C. 5R4X viruses D. X4 virus B. Inhibin A. X4R5 viruses C. CCR5 viruses B.354:610-21 B. CX3C chemokines C. Chromosome 6 Harrison’s 18th Ed. Serologic specificity B. 2686 Harrison’s 18th Ed. Chromosome 9 A. 1528 A. CXC chemokines D. fuse with. increased turnover of which lymphocyte is seen ? Harrison’s 18th Ed. CR4 viruses Human major histocompatibility complex (MHC). Sequence-based subtype C. Integrin The alpha defensin family of cytokines inhibits both R5 and X4 viruses. 1528 C.350:1872-80 245 Which of the following is the transmitting strain of HIV ? A. R5 viruses D. R5 virus . CC C. All of the above D. CX3C chemokines T cell receptor excision circles (TRECs) are a byproduct of T cell development & represent episomal fragments of DNA that are excised during T cell receptor gene rearrangement. CX2C chemokines D. D. D. CXC D. 237 In WHO nomenclature. is a 4-megabase (Mb) region on chromosome 6 (6p21. TRECs D. Strains of HIV that utilize CCR5 as a co-receptor are Harrison’s 18th Ed. 235 Lymphotactin is a member of which chemokine family ? 242 Strains of HIV that utilize CCR5 as a co-receptor are called ? N Engl J Med 2006. 1528 A. Chromosome 3 243 Strains of HIV that utilize CXCR4 as a co-receptor are called ? B. XCL1 chemokine D. 1092 N Engl J Med 2006.844 MCQ’s FOR MEDICAL PROFESSIONALS BY PROF. T cell receptor excision circles B.3). R5X4 virus C. All of the above HIV-1 utilizes two major co-receptors along with CD4 to bind to. CXC chemokines B. 236 HLA complex is located on ? These co-receptors are CCR5 and CXCR4. CXC chemokines C. R5 virus A. R5X4 viruses B. Fusin Harrison’s 18th Ed. and enter target cells. T cell receptor emptying circles 234 Fractalkine is a member of which chemokine family ? B. All of the above C. XCR4 viruses (HLA) complex. X4 viruses whose products are critical for immunologic specificity & transplantation histocompatibility. AJAY MATHUR Miscellaneous D. R4 viruses D. Locus A. A. XCL1 Harrison’s 18th Ed. B lymphocytes B. 1528 233 Interleukin-8 is the prototype member of which chemokine A. XCL1 chemokine increase following initiation of ARV therapy. All of the above D. Catalin 232 Which of the following is not a family of chemotactic cytokines or 239 Which of the following is called syncytium-inducing HIV virus ? chemokines ? Harrison’s 16th Ed. CX 3 C 241 TRECs refer to ? D. It is densely packed with expressed genes of which the best known are HLA class I & class II genes.354:610-21 Harrison’s 18th Ed. CR5 viruses C. Strains of HIV that utilize CXCR4 are referred to as X4 viruses.354:610-21 A. CD8+ T lymphocytes A. CC chemokines A. All of the above C. 1528 A.354:610-21 A. 2685 referred to as R5 viruses. HLA class I alleles are given a single 244 Strains of HIV that utilize both CCR5 and CXCR4 as co-receptors designation that indicates ? are referred to as ? Harrison’s 18th Ed. CD4+ T lymphocytes family ? N Engl J Med 2006. N Engl J Med 2004. CX3C chemokine 240 In HIV. T cell receptor excision cycles N Engl J Med 2006. Chromosome 12 B. CC chemokines C. CC chemokines B. also called human leukocyte antigen C. R54X viruses 238 The original name of CXCR4 was ? HIV virus strains that utilize both CCR5 and CXCR4 are referred to as R5X4 viruses. necessary for the normal activation of T cells ? Harrison’s 18th Ed. CCR8. All of the above C. 1529 Figure 189-25 B. D. ~ 60 % C. such C. and CD45RO. CD20 ligand A. there is a transition 252 Individuals with which of the following are protected against HIV from R5 to X4 virus ? infection ? Harrison’s 18th Ed. SDF-1 SDF-1 is the natural ligand for CXCR4. All of the above In ~40% of HIV-infected individuals. other co-receptors for HIV entry also include CCR3. MIP-1beta (CCL4) CD4+ T cells lacking expression of CD28 do not respond normally to activation signals and may express markers of terminal activation including HLA-DR. CCR2 B. RANTES-28G mutation D. AJAY MATHUR Cardiology 845 B. HLA-DR Harrison’s 18th Ed. macrophages & microglial cells of brain. All of the above D. 845 Miscellaneous MCQ’s FORMCQ’s FOR MEDICAL MEDICAL PROFESSIONALS PROFESSIONALS BY PROF. C as tetanus toxoid and influenza. All of the above CD4+ T cells from HIV-infected individuals express abnormally low levels of CD40 ligand which Apart from CCR5 and CXCR4. Heterozygous for the CCR2-64I mutation C. D. R5 virus B. CD38. MIP-1alpha (CCL3) D. 1529 Harrison’s 17th Ed. ~ 40 % B. and MIP-1beta (CCL4). there is a transition to a predominantly X4 virus that is associated with a relatively rapid progression of disease. may contribute to dysregulation of B cell function. CD38 A. ~ 80 % D. CD80 ligand D. 255 Markers of terminal activation include ? MIP-1alpha (CCL3). They are more efficient in infecting monocytes/ disease. 253 One of the first immune defects detected in HIV disease is ? Harrison’s 18th Ed. A D. 1157 A. CD38 B. 1537 A. Defect in response to remote recall antigens A. Homozygous for CCR5-32 deletion B. 1529 248 The natural chemokine ligands for HIV co-receptor CCR5 are all except ? A. It blocks the entry of X4 viruses. 1529 249 Natural chemokine ligands for HIV co-receptor CXCR4 is ? A. CXCR6 (Bonzo/STRL33/TYMSTR). Decreased IFN- production in response to antigens Harrison’s 18th Ed. MIP-1alpha (CCL3) D. ~ 20 % A. T registry cells . CD45RO B. CD20 Harrison’s 18th Ed. D. 1529 Figure 189-25 B. CCR3 C. RANTES (CCL5) C. Harrison’s 18th Ed. 256 Which of the following contribute to the dysregulation of B cell function seen in HIV disease ? 250 Which of the following is also a co-receptors for HIV ? Harrison’s 18th Ed. D 254 Which of the following is a major co-stimulatory molecule C subtypes. 1534 A. 1528 C. Response to mitogenic stimulation B. 1528 Harrison’s 18th Ed. CD60 ligand C. T regulatory cells A. CCR2. They block entry of R5 viruses. B One of the first abnormalities to be detected is a defect in response to remote recall antigens. 1529 Harrison’s 18th Ed. 1529 247 Which of the following subtypes. RANTES (CCL5) C. Defective T cell cloning & colony-forming efficiencies almost never switch from CCR5 to CXCR4 tropism ? B. It is expressed by CD4+ T cells and is reduced during HIV infection. or clades of HIV-1 M group viruses almost never switch from CCR5 to CXCR4 tropism. SDF-1 The natural chemokine ligands for HIV co-receptor CCR5 are the CC-chemokines RANTES (CCL5). R5X4 virus C. BOB/GPR15. GPR1 D. MIP-1beta (CCL4) CD28 is a major co-stimulatory molecule necessary for the normal activation of T cells. All of the above Transmitting virus is almost invariably an R5 virus that predominates during early stages of HIV R5 is the macrophage-tropic strains of HIV. CD28 A. X4 virus B. 257 T-regs refers to ? 251 Which of the following is the macrophage-tropic strain of HIV ? Harrison’s 18th Ed. or clades of HIV-1 M group virus A. X4 virus C. CD45RO C. CX 3CR1(V28) and GPR1. CD40 ligand B. R5X4 virus D. 246 In what percentage of HIV-infected individuals. Naive CD4+ T cells D. gp120 sequences B. Free gp120 258 Which cells constitute a long-term reservoir of potentially infectious D. B Lymphocyte Elevated levels of monocyte chemotactic protein (MCP)1 in the brain and CSF correlate best with D.350:1872-80 B. Activated CD4+ T cells HIV preferentially infects activated CD4+ T cells including HIV-specific CD4+ T cells. C. Activated CD4+ T cells developed a phenotype of HLA-DR+/CD38+ had a more aggressive course and a poorer prognosis.350:1872-80 B. Polish Anti-MHC class II antibodies in HIV infection can lead to elimination of MHC class II bearing cells Moritz Kaposi was a Hungarian dermatologist (1872). D.846 MCQ’s FOR MEDICAL PROFESSIONALS BY PROF. HLA-DR-/CD38+ B. Microglial cells B.342:1027 Harrison’s 17th Ed. via antibody-dependent cellular cytotoxicity (ADCC). Neurons A. Antibody-derived cellular cytotoxicity C. Proliferating CD4+ T cells Main cell types infected by HIV in brain in vivo are perivascular macrophages & the microglial cells. 1536 Figure 189-26 N Engl J Med 2000. Uninfected CD4+ T cells higher CD4+/CD8+ T cell ratios. German B.350:1872-80 B. Memory CD4+ T cells C. ADCC involves the killing of HIV-expressing cells by NK cells armed with specific antibodies directed against HIV antigens. Vasoactive intestinal peptide (VIP) N Engl J Med 2004. Memory CD4+ T cells C. AJAY MATHUR Miscellaneous C. 1530 A.350:1872-80 CD4+ T cells if uninfected cells had bound free gp120. Macrophage the presence and degree of HIV encephalopathy. 1158 A. This phenomenon is called bystander killing. Proliferating CD4+ T cells HIV infected individuals whose CD8+ T cells developed a phenotype of HLA-DR+/CD38– following seroconversion had stabilization of their CD4+ T cell counts. gp160 sequences C. Italian C. CD8+ T-cell D. IL-1 B. Antigen-derived cellular cytotoxicity D. Monocyte chemotactic protein (MCP)1 A. CD8+ T cells 265 HIV infected individuals whose CD8+ T cells developed which B. 269 Which of the following variant of Kaposi’s sarcoma is AIDS 263 Strain differences in single-cell killing are determined by ? associated ? N Engl J Med 2000. Strain differences in single-cell killing are determined largely by gp120 sequences. Oligodendrocytes N Engl J Med 2004. Endemic C. Antibody-dependent cellular cytotoxicity A. CD4+ T cells phenotype after seroconversion have a more aggressive course C. All of the above C. Presence of these T-reg cells correlates with lower viral loads and B. T recognizing cells A. 1537 D. Naive CD4+ T cells D.342:1027 A. Astrocytes C. CD4+ T-cell C. Epidemic Epidemic Kaposi’s sarcoma is AIDS associated. Hungarian D. B Lymphocyte and a poorer prognosis ? D. 1534 260 HIV replicates most efficiently in ? A. All of the above virus ? Anti-gp120 antibodies that participate in ADCC killing of HIV-infected cells might also kill uninfected N Engl J Med 2004. T regenerating cells 264 Which of the following relate to bystander killing ? Harrison’s 18th Ed. 267 Elevated levels of which of the following in the brain & CSF correlate best with the presence & degree of HIV encephalopathy ? 261 Pool of infected quiescent T cells called “latent reservoir” is Harrison’s 18th Ed. HLA-DR+/CD38+ 259 Predominantly infected cell population in HIV infection is ? N Engl J Med 2004. gp41 sequences A. Macrophage Harrison’s 18th Ed. HLA-DR-/CD38- C. 266 The main cell type infected with HIV in the brain is ? Harrison’s 18th Ed. A. Classic B. 262 ADCC refers to ? 268 Moritz Kaposi had which nationality ? Harrison’s 18th Ed. 1535 formed by ? A. HLA-DR+/CD38- A. Anti-gp120 antibodies T-regs refers to T regulatory cells that are involved in dampening aberrant immune activation that propagates HIV replication. A significant percentage of CD4+ T cells (particularly memory cells) in the gut-associated lymphoid tissue (GALT) is depleted early after infection. All of the above D. Antigen-dependent cellular cytotoxicity B. whereas those whose CD8+ T cells D. . Immunosuppression or transplantation-associated D. proliferation of spindle cells & endothelial cells. but lymph C. Lymphoma A. All of the above hemosiderin-laden macrophages are seen. and viscera. Harrison’s 18th Ed. Yellow 278 Which of the following about Kaposi’s sarcoma is false ? Purple-colored papules and plaques are seen in Kaposi’s sarcoma. or a swollen lymph node (a marker of good prognosis). Angioproliferative disease IFN. Bacillary angiomatosis B. Epidermal cells B. Body cavity lymphoma B. Skin B. and Cytomegalovirus are also called human herpesvirus (HHV) 3. 1564 B. Coxsackievirus D. Herpes simplex virus C. 1564 A. Neoplastic sarcoma C. D. All of the above A. even with normal CD4+ T cell count. 1433 Table 177-1 D. 4 and 5 respectively. 280 Differential diagnosis of KS includes ? 274 Human herpes virus 8 (HHV-8) is related to ? Harrison’s 18th Ed. Cytomegalovirus 277 Multiple vascular nodules in Kaposi’s sarcoma appear in ? Harrison’s 18th Ed. None of the above C. Human herpes virus 8 (HHV-8) Histologically. 847 Miscellaneous MCQ’s FORMCQ’s FOR MEDICAL MEDICAL PROFESSIONALS PROFESSIONALS BY PROF. Black in skin. Viscera Harrison’s 18th Ed. All of the above D. Harrison’s 18th Ed. Varicella-zoster virus 281 Which of the following is used in the treatment of KS ? HHV-8 is a  -herpesvirus related to EBV and herpesvirus saimiri. Initial lesion may be a swollen lymph node B. 1565 Harrison’s 18th Ed. Multiple myeloma family ? C. All of the above C. & D. Cytomegalovirus B. Helicobacter pylori D. 1564 Coxsackievirus belongs to Picornaviridae family of RNA viruses. 1565 name of ? A. May appear at any stage of HIV infection 272 Which of the following best relates to Kaposi’s sarcoma ? B. Human herpesvirus type 8 is believed to be the cause of most cases of Kaposi’s sarcoma. Epstein-Barr virus (EBV) C. A. 1565 A. Monoclonal gammopathy of undetermined significance Harrison’s 18th Ed. D. Vinblastine 275 Kaposi’s sarcoma (KS) is a ? Harrison’s 18th Ed. Varicella-zoster virus HHV-8. extravasation of RBCs. Herpes simplex virus D. 1564 A. 279 In KS. HHV-8 is also referred to as Kaposi’s sarcoma–associated herpesvirus (KSHV). GI tract and lung are most commonly affected by KS. in KS. AJAY MATHUR Cardiology 847 270 Which of the following does not belong to the Herpesviridae B. Lesions can occur in virtually every organ. KS may present at any stage of HIV infection. Cutaneous mycobacterial infections C. Human herpes virus 7 (HHV-7) D. 1535 A. mucous membranes. 1564 D. Spindle cells A. histologically which of the following cells proliferate ? 273 Kaposi’s sarcoma–associated herpesvirus (KSHV) is the other Harrison’s 18th Ed. Besides the above three. multiple myeloma and monoclonal B. Koebner phenomenon + Harrison’s 18th Ed. Cytomegalovirus of trauma (Koebner phenomenon). Varicella-zoster virus. Lymphoproliferative disease D. 1535 A. Epstein- Barr virus. C. 1536 276 Human herpes virus 8 (HHV-8) causes ? Harrison’s 18th Ed. All of the above C. liposomal doxorubicin & paclitaxel are also useful. All of the above A. Human herpes virus 6 (HHV-6) C. at least not in its early 282 Antibodies to HIV almost invariably appear within how many stages. Purple Kaposi’s sarcoma is a multicentric neoplasm consisting of multiple vascular nodules appearing B. Sebaceous cells Harrison’s 18th Ed. Brown nodes. Human herpes virus 8 (HHV-8) Clinically. can cause body cavity lymphoma. has ARV activity. in addition to KS. Epstein-Barr virus gammopathy of undetermined significance. 2 A. discoloration on oral mucosa. 1535 B. Mucous membranes 271 Colour of papules & plaques seen in Kaposi’s sarcoma is ? C. IFN- A. Granulomatous disease KS is an angioproliferative disease and not a true neoplastic sarcoma. 8 . Liposomal daunorubicin B. weeks of primary infection ? Harrison’s 18th Ed. Initial lesion may be a reddish-purple skin nodule in sun-exposed areas with tendancy to occur in areas D. gp160 291 “Opsonize” in Greek means ? Harrison’s 16th Ed. gp20 A. Hyperglycosylation of envelope protein is termed as the glycan shield. recent influenza vaccination. To be deformed 286 Low-molecular-weight regulatory proteins encoded by the HIV 292 Nucleic acid testing can detect HIV infection within ? genes include ? Harrison’s 18th Ed. But. 6 days A. gag C. p51 B. 160-kDa genes of HIV is conclusive evidence of HIV infection ? Harrison’s 18th Ed. Hyperphosphorylation of envelope protein D. All of the above D. nef D. Hyperglycosylation of envelope protein The products of the pol gene are p31. tat C. gp120. p66 C. HIV escapes these neutralizing antibodies by addition of carbohydrate moieties that interfere with envelope recognition 285 Which of the following is an envelope protein of HIV ? by neutralizing antibodies. and acute viral infections. 18 days C. followed by antibodies to structural or gag protein p24 and gag precursor p55. Hepatic disease C. 12 D. the gag protein p17. To prepare for eating Envelope proteins of HIV are gp160. p17 B. Harrison’s 18th Ed. All of the above C. 1538 B. 698 B. rev. 289 V3 loop region is present in ? 283 The first HIV antibodies detected is directed against ? Harrison’s 18th Ed. 288 The HIV envelope glycoproteins gp120 and gp41 are derived hepatic disease. gp41 B. vpu. 290 Mechanism of immune escape of HIV infection from initial 284 Which of the following is the product of pol gene ? neutralizing antibodies is ? Harrison’s 18th Ed. 162-kDa B. 12 days B. 1536 A. Recent influenza vaccination D. Type-specific neutralizing p24 gag are followed by antibodies to outer envelope glycoprotein (gp120). 161-kDa A. 16 The envelope of HIV consists of an outer envelope glycoprotein with a molecular mass of 120 kDa and a transmembrane glycoprotein with a molecular mass of 41 kDa. gp120 C. 22 days D. 1538 B. the products of the pol gene (p31 and p66). pol . All of the above The interval between HIV infection & detection (window period) is 22 days for antibody testing.848 MCQ’s FOR MEDICAL PROFESSIONALS BY PROF. p31 A. vif. p88 and gp41. 1536 A. Acute viral infections D. gp160 D. infection. 163-kDa D. 1536 tests is ? A. from a precursor with a molecular mass of ? Harrison’s 17th Ed. 1536 A. gp120 A. p17 Harrison’s 18th Ed. and antibodies are directed to the V3 loop region. 1536 A. 16 Low-molecular-weight regulatory proteins encoded by HIV genes include vpr. 1538 Harrison’s 18th Ed. V3 region gp41.613 region) or toward a First antibodies detected in HIV are those directed against the immunodominant region of envelope hypervariable region in gp120 molecule known as V3 loop region (amino acids 303 . Hypersulphation of envelope protein C. 1536 Harrison’s 18th Ed. AJAY MATHUR Miscellaneous C. 1163 294 Western blot demonstrating antibodies to which of the major A. autoantibodies. p55 B. vpr B. Hyperamination of envelope protein B. gp41 Antienvelope antibodies are directed toward gp41 (amino acids 579 . Within first 6 months of HIV infection neutralizing antibodies appear. tat & nef. days with p24 antigen testing and 12 days with nucleic acid testing. p24 A. These are initially synthesized Antibodies to HIV usually appear within 6 weeks & almost invariably within 12 weeks of primary as a 160-kDa precursor that is cleaved by cellular proteases. All of the above The only HIV viral proteins that elicit neutralizing antibodies are envelope proteins gp120 & gp41. Antibodies to is a major site for the development of mutations that lead to variants of HIV.338). gp41 C. To destroy D. p24 C. 287 HIV viral protein that elicit neutralizing antibody is ? 293 Factor associated with false-positive enzyme immunoassay (EIA) Harrison’s 18th Ed. p55 D. D. p51 and p66. To swell C. Factors associated with false-positive EIA tests are antibodies to class II antigens. gp41 B. 1536 Harrison’s 18th Ed. All of the above A. pol. Harrison’s 18th Ed. Patients with CD4+ counts <200/µL are at high risk of disease from P. suspicion that it may a false-positive test result ? m y c o b a c t e r i a o f t h e M . Every 3 . p 33 A. < 450 / µL C. 1541 C. 1541 D. 6 months C. Every month D. EIA with Western blot D. plasma or saliva. 1 month Harrison’s 18th Ed. jiroveci and should start prophylaxis treatment. HIV RNA by bDNA and HIV RNA by NASBA. exist against ? Harrison’s 18th Ed.6 months thereafter. 1538 A. Immune complex dissociated p24 antigen capture assay B. Branched DNA (bDNA) D. Every week D. env B. absence of the p31 band (pol gene product) should raise suspicion that it may be a false-positive test result. Immune complex dissociated p24 antigen capture assay Criteria established by the US FDA for a positive Western blot state that a result is considered positive if antibodies exist to two of the three HIV proteins: p24. Herpes zoster D. C. 1538 303 Patients with CD4+ T cell counts <50/µL are at high risk of disease A. CMV C. p24 and gp120/160 A. A. DNA PCR is used by research laboratories. and nucleic acid sequence based 295 Criteria by US FDA for a positive Western blot is if antibodies amplification (NASBA). jiroveci Western blot should be repeated in 1 month to determine whether or not the indeterminate pattern is a pattern in evolution. Herpes simplex virus infection (HSV) C. 296 When found indeterminate. EIA C. All of the above C. 297 OraQuick Rapid HIV-1 antibody test can be done with ? Harrison’s 18th Ed. Western blot should be repeated after how much time ? 302 Patients with CD4+ T cell counts <200/µL are at high risk of Harrison’s 18th Ed. g o n d i i a n d s h o u l d s t a r t prophylaxis treatment. < 500 / µL Tests for direct detection of HIV are Immune complex dissociated p24 antigen capture assay. All of the above and env) is conclusive evidence of infection with HIV. Candidal esophagitis D. p 31 304 Patients with HIV infection should have CD4+ T cell measurements B. All of the above 298 In Western Blot. Every fortnight In Western Blot. Saliva B. Any of the above C.6 months 299 Test for direct detection of HIV is ? Harrison’s 18th Ed. 2 months A. < 400 / µL 300 Test that measure HIV RNA in plasma of patients with HIV infection is ? D. gp41 and gp120/160 B. gp41 and gp120/160. HIV RNA by PCR. 1538 disease from ? A. 1538 301 “Gold standard” for the diagnosis of HIV infection is ? A. avium complex (MAC) D. gondii OraQuick Rapid HIV-1 antibody test can be done on blood. P. 3 months B. D. Mycobacteria of M. 1541 B. Blood from ? Harrison’s 18th Ed. AJAY MATHUR Cardiology 849 C. a v i u m c o m p l e x ( M A C ) a n d / o r T. p 32 at the time of diagnosis and ? Harrison’s 18th Ed. p 34 B. 1541 B. < 350 / µL Harrison’s 18th Ed. branched DNA (bDNA). HIV RNA by bDNA 305 What level of CD4 T cell count is an indication for consideration C. Commercially available tests that measure HIV RNA in plasma of patients with HIV infection are reverse transcriptase PCR (RT-PCR). B. D. Reverse transcriptase PCR (RT-PCR) CD4 T cell count <350/µL is an indication for consideration of initiating ARV therapy . All of the above C. Western blot D. 849 Miscellaneous MCQ’s FORMCQ’s FOR MEDICAL MEDICAL PROFESSIONALS PROFESSIONALS BY PROF. 1540 Table 189-7 Patients with HIV infection should have CD4+ T cell measurements performed at the time of diagnosis and every 3 . p24 and gp41 Harrison’s 18th Ed. Plasma A. Nucleic acid sequence based amplification (NASBA) Western blot demonstrating antibodies to products of all three of the major genes of HIV (gag. T. 1540 A. A positive EIA with a confirmatory Western blot is the “gold standard” for diagnosis of HIV infection. 1540 B. absence of which of the following band raises P a t i e n t s w i t h C D 4 + T c e l l c o u n t s < 5 0 / µ L a r e a t h i g h r i s k o f d i s e a s e f r o m C M V. HIV RNA by NASBA of initiating ARV therapy ? Harrison’s 18th Ed. Tat.2 weeks effectiveness of therapy.351 A. 20 % Nef has been shown to possess antiapoptotic properties.338:1281 Harrison’s 17th Ed. > 100000 315 Antibodies to HIV appear almost invariably within how many ARV therapy is considered in patients with >100. AJAY MATHUR Miscellaneous 306 What percentage of decline in CD4+ T cell count is an indication 312 Enfuvirtide binds to which glycoprotein to inhibit fusion of HIV for considering a change in therapy ? with the target cell ? Harrison’s 18th Ed. > 15 % C.850 MCQ’s FOR MEDICAL PROFESSIONALS BY PROF. weeks of primary infection ? Harrison’s 18th Ed. vpr C.3 weeks 311 Effective ARV therapy implies a plasma HIV RNA level of ? D. < 100 copies per milliliter D. 1542 Effective ARV therapy implies a plasma HIV RNA level of < 50 copies per milliliter. and p7 D. Glycoprotein 41 C. > 25 % D. 16 weeks C. levels of HIV RNA should be monitored every 3 . 2 . 1542 B. 2 weeks D. Glycoprotein 120 D. 8 weeks syndrome ? Following initiation / change in ARV therapy. 1541 N Engl J Med 2004. 15 % D.30 % 310 During ARV therapy. p24. p17. This level of virus is achieved within 6 months of the initiation of effective treatment. plasma HIV RNA levels should be monitored B. 2 months 317 Acute HIV syndrome occurs how many weeks after primary infection ? C. 5 % B. > 1000 B. p34. 3 months Harrison’s 18th Ed. plasma HIV RNA levels should be monitored every ? A. < 350 copies per milliliter primary infection which coincides with a burst of plasma viremia. 1537 of 200/µL ? Harrison’s 18th Ed. C. > 50000 D. It is less common in those infected B. A. A. p24. 12 weeks B. 1542 4 weeks till a new steady-state level of HIV RNA appears. 8 weeks A. 4 weeks 316 What percentage of HIV infected patients experience acute HIV D. and Vpr enhance susceptibility to apoptosis. 20 . 1536 309 Following initiation / change in ARV therapy.6 weeks after A. 1542 D. Glycoprotein 24 B. vif B. > 10 % B. 314 Cleavage of the gag polyprotein produces three large proteins 308 ARV therapy is considered in patients with how many copies of named ? HIV RNA per milliliter ? N Engl J Med 1998. nef A. 1168 A. 1 .000 copies of HIV RNA per milliliter. A CD4+ T cell percent of 15 is comparable to a CD4+ T cell count of 200/µL. < 50 copies per milliliter 318 Which of the following about acute HIV syndrome is false ? Harrison’s 18th Ed. vpu D. persist for one to several weeks. Env. and p27 A. 3 .4 months to evaluate the continuing B. 1543 50 . 2 weeks Harrison’s 18th Ed.90 % A. > 10000 C. and p7 C. 313 HIV virus with defect in which of the following gene may lead to “long-term nonprogressors” ? 307 What percent of CD4+ T cell is comparable to a CD4+ T cell count Harrison’s 18th Ed. C. 6 months A. 1542 D. Presentation resembles acute infectious mononucleosis . and p7 B. 10 . features similar to those of to acute infectious mononucleosis. p17. 10 % C. 1541 A. p24. Month B. >5% A. Glycoprotein 160 A decline in CD4+ T cell count of >25% is an indication for considering a change in therapy. p27. < 200 copies per milliliter by injection drug use than those infected by sexual contact.6 weeks Harrison’s 18th Ed. 70 . plasma HIV RNA levels should be monitored every Harrison’s 18th Ed.70% of individuals with HIV infection experience an acute clinical syndrome ~3 . < 1 week During therapy.70 % Harrison’s 18th Ed. 1 week C.50 % every ? C. 50 . p17. severe & life-threatening complications (opportunistic infection & neoplasms) of HIV infection occur. Chlamydophila pneumoniae 319 Median time from primary HIV infection to development of AIDS 325 Most common cause of pneumonia in HIV infection is ? in untreated individuals in developed world is ? Harrison’s 18th Ed. < 50 / µL D. Maxillary C. Therapy for TB is generally D. ~ 15 years In HIV/AIDS. None of the above D. Clindamycin / primaquine C. Streptococcus pneumoniae D. Rifabutin is substituted for rifampin in those receiving HIV protease inhibitors A. Colony number . ~ 5 years C. < 400 / µL B. CD4+ T cell counts is < 200 / µL or a CD4 % <15. ~ 200 cells / µL For patients who cannot tolerate TMP/SMX. Sphenoid HIV infection increases the risk of developing active TB by a factor of 100. Aerosolized pentamidine A. influenzae) & the unicellular fungus P. pneumoniae Although variable. 328 Agents for prophylaxis for PCP include ? 322 Average CD4+ T cell count of an HIV-infected patient at the time of Harrison’s 18th Ed. 1547 Harrison’s 17th Ed. Disease is also frequently seen in ethmoid. the resulting state of immunodeficiency is severe enough & & clindamycin/primaquine. 1543 A. All of the above D. jiroveci. < 300 / µL A. ~ 1 year B. None of the above C. average rate of pneumonia prophylaxis when CD4+ T cell counts is ? CD4+ T cell decline is ? Harrison’s 18th Ed. pneumonia is caused most commonly by encapsulated organisms (S. Haemophilus influenzae C. sphenoid and frontal sinuses. Pseudomonas aeruginosa A. HIV infection increases risk of active TB by 100x HIV infection ? Harrison’s 18th Ed. Atovaquone B. Rifabutin should be substituted for rifampin in patients receiving HIV protease inhibitors or nonnucleoside reverse transcriptase inhibitors. 330 Runyon classification of Nontuberculous Mycobacteria (NTM) is 324 Most common cause of pneumonia in HIV infection is ? based on ? Harrison’s 18th Ed. ~ 125 / µL per year All patients of HIV infection must receive pneumocystis pneumonia (PCP) prophylaxis when their During asymptomatic period of HIV infection. Trimethoprim / sulfamethoxazole A. dapsone/trimethoprim When CD4+ T cell count falls to <200/µL. Chapter 160 A. Staphylococcus aureus B. ~ 75 / µL per year C. 326 All patients of HIV infection must receive pneumocystis 320 During asymptomatic period of HIV infection. < 350 / µL C. Rifabutin is substituted for rifampin in those receiving NNRTI’s B. < 200 / µL B. average rate of CD4+ T cell decline is ~50/µL per year. 1548 323 Which of the following sinuses are most commonly involved in A. 1547 death is ? A. 1547 Harrison’s 18th Ed. Neisseria meningitidis D. All of the above C. < 200 / µL Treatment options for PCP or disseminated pneumocystosis are TMP/SMX. Dapsone + pyrimethamine + leucovorin Harrison’s 17th Ed. ~ 100 cells / µL D. AJAY MATHUR Cardiology 851 B. for untreated patients is ~10 years. Ethmoid D. 321 Immunodeficiency is severe enough when CD4+ T cell count falls 327 Treatment for PCP or disseminated pneumocystosis is ? to ? Harrison’s 18th Ed. ~70% develop lymphadenopathy C. aerosolized pentamidine and atovaquone. Klebsiella pneumoniae C. ~ 300 cells / µL + leucovorin. ~ 100 / µL per year D. Average CD4+ T cell count of an HIV-infected patient at the time of death is just over 300 cells/ µL. 1546 B. ~ 50 cells / µL C. 1547 Harrison’s 18th Ed. IV pentamidine is the treatment of choice for severe disease. < 100 / µL C. 329 Which of the following regarding TB in HIV patient is false ? Harrison’s 18th Ed. Dapsone / trimethoprim B. ~ 50 / µL per year B. < 300 / µL D. 1169 B. 1543 A. Maxillary sinuses are most commonly involved in HIV infection. 1547 Harrison’s 18th Ed. 1542 A. 851 Miscellaneous MCQ’s FORMCQ’s FOR MEDICAL MEDICAL PROFESSIONALS PROFESSIONALS BY PROF. median length of time from initial infection to development of clinical disease & H. Mycoplasma pneumoniae A. Opportunistic infections may occur B. PCP prophylaxis include dapsone + pyrimethamine D. Frontal the same in HIV-infected patient as in HIV-negative patient. ~ 10 years D. esophagitis associated with a single large ulcer occurs in CMV infection. 1549 Harrison’s 18th Ed. Entecavir D. The most common form of heart disease in HIV HBV co-infection. as well as hyperinsulinemia & hyperglycemia. Colony size D. fortuitum. A. Clarithromycin A. Adefovir / Tenofovir C. & G viruses is common. M. avium B. 335 Which of the following about oral hairy leukoplakia is false ? Harrison’s 18th Ed. Indinavir & atazanavir may cause mild . 1554 C. All of the above 336 Esophagitis associated with a single large ulcer is typical of ? Harrison’s 18th Ed. Chapter 160 A. C. fortuitum D. marinum Hepatitis A virus infection is not seen with an increased frequency in patients with HIV infection. HSV A. Aphthous ulcers C. Colony shape In HIV infection. hepatic necrosis. Found along lateral borders of tongue Harrison’s 18th Ed. Hepatitis D virus D. Indinavir C. Hepatitis C C. Hepatitis B virus B. which is the most common 339 Which of the following drugs is used in the treatment of HIV and clinically significant cardiac finding ? HBV co-infection ? Harrison’s 18th Ed. Nonbacterial thrombotic endocarditis B. kansasii C. total cholesterol & apolipoprotein B. Thrush A. Candida B. 337 Which of the following hepatitis virus infections are seen with increased frequency in patients with HIV infection ? 331 Which of the following is a rapidly growing NTM species ? Harrison’s 18th Ed. or adefovir/tenofovir and entecavir are used in the treatment of HIV and is the most common clinically significant finding. 1548 Harrison’s 18th Ed. CMV 342 Which of the following ARV drug is a thymidine analogue ? Harrison’s 18th Ed. 1549 Harrison’s 18th Ed. emtricitabine. or amikacin is added Patients with HIV infection co-infected with hepatitis G virus have a decreased rate of progression to in patients with extensive disease. M. Zidovudine . and M. Hepatitis G virus Therapy for MAC is clarithromycin with ethambutol. Increased total cholesterol D. M. patients is coronary heart disease. 338 HIV infection co-infected with which of the following virus has a 332 Therapy for MAC include all except ? decreased rate of progression to AIDS ? Harrison’s 18th Ed. Left atrial myxoma C. All of the above Due primarily to HIV infection. and aphthous Nevirapine is associated with fatal fulminant & cholestatic hepatitis. 333 Due primarily to HIV infection. None of the above C. Increased apolipoprotein B D. M. 1552 Harrison’s 17th Ed. Rifabutin D. HSV infection is associated with multiple small ulcers. Associated with florid replication of EBV B. Emtricitabine D. KS C. Hepatitis B B. D. 1550 Features of lipodystrophy in HIV patients receiving HAART are elevations in plasma triglycerides. Dilated cardiomyopathy D.852 MCQ’s FOR MEDICAL PROFESSIONALS BY PROF. AJAY MATHUR Miscellaneous B. Rapidly growing NTM species are M. abscessus. 1553 B. Not considered a premalignant condition A. ulcerans. chelonae. AIDS. All of the above D. Hepatitis A A. dilated cardiomyopathy with CHF (HIV-associated cardiomyopathy Lamivudine. 1552 A. and M. ciprofloxacin. Ethambutol B. 1549 341 Features of lipodystrophy in HIV patients receiving HAART are ? A. avium. E. failure. 1553 A. All of the above D. Hepatitis C virus C. 340 A syndrome similar to Gilbert’s syndrome appears with the use of 334 Oral lesions in HIV infection include ? which of the following ARV drug ? Harrison’s 18th Ed. Pyrazinamide C.moderate elevations in serum bilirubin in a syndrome similar to Gilbert’s syndrome. Emtricitabine B. Colony pigmentation Runyon classification of Nontuberculous Mycobacteria (NTM) is based on colony pigmentation. D. Rifabutin. hairy leukoplakia (EBV). & hepatic ulcers. M. M. Pericardial effusion A. Co-infection with hepatitis B. 1552 A. Tenofovir Oral lesions in HIV infection include thrush (Candida). marinum. Increased plasma triglycerides C. kansasii. M. Hairy leukoplakia B. while slow. growing species include M. Nevirapine B. Agnosia C. CSF findings are abnormal in ~ 90 % of patients (subcortical). Reduced hematocrit condition ? Harrison’s 18th Ed. CSF findings are by ? abnormal in what percentage of patients ? Harrison’s 18th Ed. Nevirapine HIV infection ? Harrison’s 18th Ed. apraxia. 1559 B. ~ 25 % B. All of the above D. ~ 70 % D. detection of viral RNA (~75%). HIV-infected individuals with the E4 allele for apo E are at increased risk for AIDS encephalopathy It is also called “subcortical dementia”. Atorvastatin elevated CSF protein (35%). 1559 Harrison’s 18th Ed. All of the above is ? Harrison’s 18th Ed. 853 Miscellaneous MCQ’s FORMCQ’s FOR MEDICAL MEDICAL PROFESSIONALS PROFESSIONALS BY PROF. 1556 of HIV infection i. and agnosia are uncommon in HIV encephalopathy. Invasive cervical cancer A characteristic feature of zidovudine therapy is an elevated mean corpuscular volume (MCV). All of the above 346 HIV-infected individuals with which of the following are at 352 Which of the following is uncommon in AIDS dementia complex ? increased risk for AIDS encephalopathy & peripheral Harrison’s 18th Ed. Generalized wasting D. . A. ApoA-I B. AIDS dementia complex or HIV encephalopathy PGL is defined as presence of enlarged lymph nodes (>1 cm) in two or more extrainguinal sites for B. VII cranial nerve most often involved or atorvastatin. 1556 351 Which of the following is included as an AIDS-defining A. AJAY MATHUR Cardiology 853 B. Increased rouleaux formation B. ~ 50 % C. A. Behavioral abnormalities C. and peripheral neuropathy. Motor abnormalities B. Elevated mean corpuscular volume (MCV) A. motor and behavioral abnormalities. B. Efavirenz A. 1554 CSF abnormalities in HIV infection include pleocytosis (50-65%). Histoplasmosis C. 1534 Table 189-5 A. Dementia A. Normal glucose level in CSF 344 Persistent generalized lymphadenopathy is defined enlarged lymph C. 1559 Most commonly used drug for lipodystrophy in HIV patients receiving HAART is either gemfibrozil A. rare following development of AIDS. Resolves spontaneously within 2 . 1559 neuropathy ? A. Nicotinic acid Harrison’s 18th Ed. Apraxia B. 353 HIV encephalopathy or AIDS dementia complex is characterized 347 During asymptomatic phase of HIV infection. > 6 months Harrison’s 18th Ed. Evidence of intrathecal synthesis of anti-HIV antibodies Harrison’s 18th Ed. B. Lamivudine 348 Which of the following is the most common CSF abnormality in C.e. 1558 A. > 1 month B. also called HIV-associated dementia or AIDS dementia complex. aphasia. > 9 months A. B. C. AIDS dementia complex is characterized by dementia During asymptomatic phase of HIV infection. Others may cause drug interactions. Cholestyramine 349 Which of the following about aseptic meningitis in HIV infection C. ApoB-100 C. All of the above D. Reactivation of Chagas’ disease 345 Typical hematological laboratory finding in Zidovudine therapy D. Ezetimibe is false ? D. ApoJ In contrast to “cortical” dementia (Alzheimer’s disease). Cryptococcus neoformans meningitis >3 months without an obvious cause. D. All of the above C. Aphasia Harrison’s 18th Ed. ApoE4 allele D. Pleocytosis Zidovudine and Stavudine are thymidine analogues. 1558 D. without an obvious D. Elevated CSF protein receiving HAART is ? D. More common following development of AIDS cause for ? Aseptic meningitis in HIV infection is immune-mediated and is seen in any but the very late stages Harrison’s 18th Ed.4 weeks nodes in two or more extrainguinal sites. ~ 90 % Usually a late complication of HIV infection. Detection of viral RNA 343 Most commonly used drug for lipodystrophy in HIV patients C. > 3 months 350 Which of the following is included as an AIDS-defining condition ? C. and evidence of intrathecal synthesis of anti-HIV antibodies (90%). 1559 D. Trypanosoma cruz zidovudine-induced myopathy. 1563 A. D. 1563 Harrison’s 18th Ed. Perivascular hemorrhage and exudate in retina D. keratitis. C. Kaposi’s sarcoma D. Chorioretinitis 356 Type of spinal cord disease seen in patients with AIDS is ? B. All of the above 362 Most common abnormal findings on funduscopic examination in advanced HIV infection is ? JC virus. 1562 In contrast to dementia due to toxic/metabolic encephalopathies. Cotton-wool spots C. JC virus D. All of the above A. Cytoplasmic bodies Harrison’s 18th Ed. Toxoplasma gondii Zidovudine interfere with the function of mitochondrial polymerases. Chronic arachnoiditis D. 1564 C. C. Mitochondrial abnormalities A. Acute inflammatory demyelinating polyneuropathy neoplastic diseases in HIV infection ? B. Retinal atrophy B. Mild mania induced myopathy ? Harrison’s 18th Ed. None of the above D. 1562 365 HAART has had little effect on frequency of which of the following A. Transverse myelitis B. a human polyomavirus that is the etiologic agent of progressive multifocal Harrison’s 18th Ed. 1559 D. Pure sensory ataxia Most common abnormal findings in optical fundus is cotton-wool spots. 1561 D. AJAY MATHUR Miscellaneous 354 Which of the following is not a behavioral abnormality in HIV B. significant changes in level of alertness are absent in HIV encephalopathy. Cotton-wool spots A. Red ragged fibers are a histologic hallmark of B. 358 Which of the following about myelopathy in AIDS is false ? 364 PORN stands for ? Harrison’s 18th Ed. Non-Hodgkin’s lymphoma D. Perivascular hemorrhage and exudate in retina B. Red ragged fibers B. Significant changes in level of alertness C. 1562 Harrison’s 18th Ed. They are not associated with visual loss & remain stable or improve over time.854 MCQ’s FOR MEDICAL PROFESSIONALS BY PROF. Proximal sensory polyneuropathy A. State of agitation 361 Which of the following is the histologic hallmark of zidovudine- D. Its toxicity is dose-dependent & reversible following discontinuation of drug. and iritis. Do not respond well to ARV drugs C. Apathy HIV-associated myopathy is due to HIV infection itself. Progressive ocular retinal necrosis B. A. Nemaline rod bodies 355 Etiologic agent of progressive multifocal leukoencephalopathy (PML) is ? C. Painful ocular retinal necrosis D. Mononeuritis multiplex Harrison’s 18th Ed. Vacuolar myelopathy D. 1562 HAART has had little effect on human papilloma virus (HPV)-associated malignancies and AIDS- associated systemic lymphomas. Painful outer retinal necrosis 359 Most common peripheral neuropathy in patients with HIV infection In HIV. retinal detachment after retinal atrophy in areas of prior inflammation may occur. Tabes dorsalis The characteristic in HIV infection with CMV retinitis is perivascular hemorrhage & exudate in Vacuolar myelopathy in patients with AIDS is pathologically similar to subacute combined retina due to necrotic inflammatory process and the visual loss is irreversible. B. 1562 A. Paresthesias & dysesthesias of lower extremities surface of retina with irregular edge. zidovudine & generalized wasting syndrome. Zidovudine encephalopathy ? C. Chorioretinitis A. Distal sensory polyneuropathy B. They represent areas of retinal ischemia secondary to microvascular disease. All of the above 363 Characteristic abnormal findings on funduscopic examination in 357 Vacuolar myelopathy in patients with AIDS is pathologically similar HIV infection with CMV retinitis is ? to ? Harrison’s 18th Ed. Generalized wasting syndrome Harrison’s 18th Ed. Human papilloma virus associated malignancies Most common peripheral neuropathy in HIV infection is a distal sensory polyneuropathy. Rhegmatogenous degeneration of cord. 1562 C. Progressive outer retinal necrosis occurs due to HSV & varicella zoster virus with pain. A. Present in ~20% of patients with AIDS A. A. Subacute combined degeneration of cord C. All of the above 360 HIV-associated myopathy is due to ? Harrison’s 18th Ed. Progressive outer retinal necrosis C. 90% patients have dementia B. is ? Harrison’s 18th Ed. HIV infection . 1563 leukoencephalopathy (PML) which is the only known clinical manifestation of JC virus infection. Retinal atrophy Harrison’s 18th Ed. These are hard white spots on C. Indinavir increased by ? N Engl J Med 1998. 1569 N Engl J Med 1998. Viral protease enzyme inhibitors D. Viral integrase enzyme inhibitors 378 Drug to be avoided with HIV protease inhibitors is ? D. Absolute CD4+ T cell count are <300/µL or <20% of total T cells on at least two occasions 6 weeks apart. Midazolam 371 A high-fat meal increases the bioavailability of all except ? C. Grade III or IV immunoblastic lymphoma B. Indinavir 369 Which of the following category is a viral reverse transcriptase D. lamivudine. 1569 B. Ritonavir Most common HPV genotypes in the general population are 16 and 18. In HIV-infected population genotypes 56 and 53 predominate. abacavir & emtricitabine). Ritonavir FDA-approved reverse transcriptase inhibitors include nucleoside analogues (zidovudine. 8 and 5 A. 1567 C.338:1281 A. Saquinavir In ICL. 855 Miscellaneous MCQ’s FORMCQ’s FOR MEDICAL MEDICAL PROFESSIONALS PROFESSIONALS BY PROF. N Engl J Med 1998. Saquinavir B. Nelfinavir hypergammaglobulinemia. All of the above A. nucleotide analogue 377 The combination of ritonavir and rifabutin is associated with an (tenofovir) & nonnucleoside reverse transcriptase inhibitors (nevirapine. Viral reverse transcriptase enzyme inhibitors C. Indinavir D. Nelfinavir 379 Area under the plasma concentration-time curve of Saquinavir is C. 18 and 16 B. HIV-2. Nelfinavir C. Hypergammaglobulinemia 375 Which of the following PI must be stored in a refrigerator ? N Engl J Med 1998. Saquinavir B.338:1281 D. Nucleotide analogues B. Cisapride N Engl J Med 1998. Burkitt’s lymphoma C. Nelfinavir . Peripheral neuropathy A. Indinavir A. Ig levels are normal or decreased while HIV-infected individuals have B. Ritonavir A. Uveitis Harrison’s 18th Ed. 373 Which of the following protease inhibitors (PI) has the highest cerebrospinal fluid penetration ? N Engl J Med 1998. HTLV-I. 1566 A. Saquinavir lymphocytopenia (ICL) is false ? B. delavirdine & efavirenz). 56 and 53 D. Ritonavir B. C. Saquinavir B. Nonnucleoside reverse transcriptase inhibitors C.338:1281 Viral reverse transcriptase enzyme inhibitors were the first class of drugs licensed for treatment A. Nelfinavir C. Indinavir C. Saquinavir B. Ritonavir capsules A. B. increased incidence of rifabutin-associated toxicity particularly ? N Engl J Med 1998. Nelfinavir Harrison’s 18th Ed. or HTLV-II C. Viral entry inhibitors N Engl J Med 1998. 32 and 22 C.338:1281 367 Lymphoma seen in patients with HIV infection is ? Harrison’s 18th Ed. with a desiccant ? Burkitt’s lymphoma.338:1281 368 Which of the following statements about idiopathic CD4+ T A. All of the above D. Ritonavir inhibitor ? 376 Which of the following PI contains alcohol ? Harrison’s 18th Ed. 1567 N Engl J Med 1998. Renal failure C. Opportunistic diseases occur A. didanosine.338:1281 370 First class of drugs licensed for treatment of HIV infection was ? A. All of the above D. AJAY MATHUR Cardiology 855 366 Most common HPV genotype in HIV-infected population is ? 372 Which of the following PI is not protein-bound in plasma ? Harrison’s 18th Ed. Absolute CD4+ T cell count of <300/µL D. & primary CNS lymphoma. zalcitabine.338:1281 A.338:1281 D. stavudine. Cardiomyopathy B. All of the above 374 Which of the following PI must be stored in an airtight container Three categories of lymphoma seen with HIV infection are grade III or IV immunoblastic lymphoma. Nucleoside analogues A. Indinavir D. Terfenadine of HIV infection. No evidence of HIV-1.338:1281 D. Primary CNS lymphoma D. 355:173-81 B. Nephrolithiasis Harrison’s 18th Ed. Ritonavir B. Saquinavir C. 1579 392 Which of the drugs require dose adjustment in patients with A. Ototoxicity C.355:173-81 B. Lamivudine B.350:1023 388 Which of the following is a fusion inhibitor ? A. Stavudine . Efavirenz an intravenous formulation ? N Engl J Med 2006.344:1766 in patients with renal insufficiency ? A. Ritonavir D. 1191 A. Indinavir B. Tenofovir C. Saquinavir A. Nelfinavir C. Zidovudine D. Emtricitabine nucleoside analogue. Didanosine Harrison’s 18th Ed. Delavirdine C. Indinavir hepatic impairment ? N Engl J Med 2006. 393 ‘Lactic acidosis syndrome’ is a serious adverse effect of which of 386 The first HIV-1 protease inhibitors to be licensed was ? the following antiretroviral drug ? Harrison’s 18th Ed.350:1023 389 Which of the following antiretroviral medication is available as A.355:173-81 B. Abacavir D. D. Lamivudine 385 First protease inhibitor used along with zidovudine and Tenofovir is contraindicated in patients with renal impairment. Glaucoma B. lamivudine & indinavir was the first “triple combination” effective on D. Abacavir C. Atazanavir C.856 MCQ’s FOR MEDICAL PROFESSIONALS BY PROF. Indinavir N Engl J Med 2003. Lamivudine 383 First drug approved by US FDA for treatment of AIDS is ? D. 381 Which of the following is not a Protease inhibitor ? N Engl J Med 2004. 1199 387 Which of the following protease inhibitors is best-tolerated ? A. Nevirapine A. Stavudine C. Nelfinavir Saquinavir is among the best-tolerated protease inhibitors. 1574 390 Which of the following drugs does not require dose adjustment A. All of the above HIV replication. Saquinavir A. Indinavir Indinavir was the first protease inhibitor used in combination with dual nucleoside therapy. Fosamprenavir D.348:2228-38 B. Nelfinavir Saquinavir was the first HIV-1 protease inhibitors to be licensed. 1990 B. Ritonavir B. Efavirenz D. 380 The most important side effect of Indinavir is ? Harrison’s 17th Ed. Stavudine C. Abacavir C. Delavirdine A. 1579 N Engl J Med 2006.355:173-81. Lamivudine 384 Zidovudine was approved by US FDA for treatment of AIDS in ? 391 Which of the following drugs does not require dose adjustment N Engl J Med 2001. All of the above D. Harrison’s 17th Ed. Zalcitabine B. 1987 A. D. Indinavir C. AJAY MATHUR Miscellaneous B. Didanosine Zidovudine (AZT) was the first drug approved for the treatment of HIV infection and is the prototype C. 1579 B. Indinavir D. 1995 D.355:173-81 C. Ritonavir C. Saquinavir A. lamivudine in “triple combination therapy” for HIV was ? Harrison’s 18th Ed. Tenofovir 382 Which of the following is not a NNRTI ? N Engl J Med 2004. Zalcitabine N Engl J Med 2006. Combination of zidovudine. Zidovudine D. Ritonavir The main side effects of indinavir are nephrolithiasis. 1983 N Engl J Med 2006. Enfuvirtide D. Zidovudine in patients with renal insufficiency ? B. Peripheral neuropathy A. Didanosine B. Zidovudine C. Lamivudine A. 1578 A. Therefore. 1578 Harrison’s 18th Ed. Etravirine C. Indinavir B. Tenofovir A. which one is the weakest ? Harrison’s 18th Ed. Stavudine C. Nucleoside analogues C. serious side effects are more varied with nucleoside analogues & include mitochondrial damage that can lead to hepatic nonnucleoside inhibitors of HIV-1 reverse transcriptase ? steatosis. Lamivudine B. Abacavir C. All of the above 404 Which of the following results in inhibition of cytochrome P450 398 Among the nucleoside analogues licensed for the treatment of action ? Harrison’s 18th Ed. Zalcitabine is probably the weakest. Nevirapine 397 Pancreatitis is the adverse effect of which of the following ? Harrison’s 18th Ed.355:173-81. Zidovudine D. lactic acidosis. Tenofovir D. didanosine & zalcitabine. Nonnucleoside reverse transcriptase inhibitors are quite selective for the HIV-1 reverse transcriptase. 1579 analogue ? Harrison’s 18th Ed. 402 In patients who are HLA-B5701. 1578 A. lamivudine. 405 Which of the following increases the activity of 399 Action of zidovudine is antagonised by ? glucuronyltransferases ? Harrison’s 18th Ed. Saquinavir is also metabolized by cytochrome P450 system. emtricitabine and tenofovir are used to treat hepatitis B infection in HIV infection. Abacavir D. 1579 HIV infection. Its use results in large increases in plasma concentrations of drugs metabolized by Among nucleoside analogues (zidovudine. All of the above D. 857 Miscellaneous MCQ’s FORMCQ’s FOR MEDICAL MEDICAL PROFESSIONALS PROFESSIONALS BY PROF. stavudine is a thymidine analogue. Didanosine B. 395 Which of the following antiretroviral medications is also active 401 Abacavir-resistant strains of HIV are also resistant to ? against hepatitis B virus (HBV) ? Harrison’s 18th Ed. Delavirdine A. Zidovudine C.Johnson syndrome or toxic epidermal necrosis is a serious adverse effect of which of the following antiretroviral drug ? 400 Which of the following is the best tolerated & least toxic nucleoside N Engl J Med 2006. Didanosine B. Lamivudine is the best tolerated and least toxic nucleoside analogues. AJAY MATHUR Cardiology 857 B. Harrison’s 17th Ed. These two drugs are antagonistic in vitro and in vivo and should not be given together. abacavir this pathway. Didanosine B. All of the above Abacavir-resistant strains of HIV are typically also resistant to lamivudine. Harrison’s 18th Ed. zalcitabine. Emtricitabine In HIV infection Stevens-Johnson syndrome may occur due to nonnucleoside reverse transcriptase inhibitors nevirapine. Ritonavir C. 1578 Harrison’s 18th Ed. All of the above Abacavir hypersensitivity occurs more in patients who are HLA-B57. & emtricitabine) licensed for the treatment of HIV infection.355:173-81. Nonnucleoside reverse transcriptase inhibitors B. 1198 A. didanosine. the nucleoside and nucleotide analogues inhibit a variety of DNA polymerization 403 Vivid dreams is a side effect of which of the following reactions in addition to those of HIV-1 reverse transcriptase. Lamivudine B. Zalcitabine C. 1578 N Engl J Med 2006. 1569 A. Emtricitabine D. 394 Stevens . Stavudine C. Tenofovir . Stavudine Some patients treated with efavirenz complain of vivid dreams. Emtricitabine Ritonavir has a high affinity for several isoforms of cytochrome P450 thereby inhibiting cytochrome P450 action. and efavirenz. 1579 A. Tenofovir A. Zalcitabine D. Abacavir D. Emtricitabine C. 1578 A. Lamivudine. Didanosine A. Didanosine B. Zalcitabine C. peripheral neuropathy & and pancreatitis. All of the above D. delavirdine. Harrison’s 18th Ed. which of the following can produce 396 Which of the following reverse transcriptase inhibitors are quite hypersensitivity ? selective for the HIV-1 reverse transcriptase ? Harrison’s 18th Ed. stavudine. Like zidovudine. Nelfinavir D. Nucleotide analogues D. 1572 Table 189-20 B. Emtricitabine Lactic acidosis is most commonly seen with nucleoside analogue reverse transcriptase inhibitors. D. Stavudine A. Didanosine C. Efavirenz B. Nevirapine D. C.2 months 410 Molecule of which of the following contain a sulfonamide moiety ? Harrison’s 18th Ed. C.150/µL. C. Every 1 . All of the above Harrison’s 18th Ed. 1579 Enfuvirtide interferes with the fusion of viral & cellular membranes by binding to HR1 region in gp41 subunit of the HIV-1 envelope. efavirenz. 1580 B. 1 . Ritonavir A. 1581 D. 1580 D. Tenofovir B. Every 2 .4 months during therapy. DNA chip hybridization. 1580 D. Efavirenz 413 Which of the following ARV is an integrase inhibitor ? C. B. A. Raltegravir 407 Indinavir levels are increased during concurrent therapy with ? Harrison’s 18th Ed. Patients started on ARV should remain on ARV therapy Harrison’s 18th Ed. Immediate treatment of every HIV-infected individual upon 408 Total cholesterol & triglyceride levels do not increase with which diagnosis may not be prudent of the following protease inhibitors ? C.4 months protease inhibitors ? D. gp41 D. 3 . Atazanavir 416 Plasma HIV RNA levels & CD4+ T lymphocyte counts should be D. thus decreasing the levels of drugs metabolized by this pathway. Maraviroc C.3 months 409 Prolongations of PR interval occurs with which of the following C.3 months A. Fosamprenavir B. 4 . or nevirapine A. Enfuvirtide A. Nelfinavir monitored how frequently during therapy ? Atazanavir is associated with prolongations of PR interval. a 1 log (tenfold) reduction C. Nelfinavir A. Ritonavir 414 Guidelines for the use of ARV therapy include ? D. Indinavir 415 Following the initiation of ARV therapy. Dideoxynucleotide sequencing Harrison’s 18th Ed. gp160 D.858 MCQ’s FOR MEDICAL PROFESSIONALS BY PROF. Enfuvirtide C. Raltegravir D. Every 4 . 1580 C. 1580 B. Indinavir milliliter and a rise in CD4+ T cell count of 100 . Darunavir Molecule of Darunavir contain a sulfonamide moiety. DNA chip hybridization A. D. gp120 Ritonavir may increase the activity of glucuronyltransferases. 1580 A. Fosamprenavir Plasma HIV RNA levels & CD4+ T lymphocyte counts should be monitored every 3 . Atazanavir in plasma HIV RNA levels is expected within ? Harrison’s 18th Ed. eventually a decline in plasma HIV RNA levels to <50 copies per B. Rifabutin B. p24 406 Indinavir levels are decreased during concurrent therapy with ? Harrison’s 18th Ed. 1579 C. 1581 Levels of indinavir are decreased during concurrent therapy with rifabutin.2 months Total cholesterol & Tg levels do not increase as much with atazanavir as with other protease inhibitors. 1582 A. 2 . 1582 411 Which of the following drugs is a CCR5 antagonist ? A. All of the above D. Maraviroc is a CCR5 antagonist that interferes with HIV binding at the stage of co-receptor engagement. Ketoconazole D.2 months. Tenofovir Following initiation of ARV therapy one should expect a 1 log (tenfold) reduction in plasma HIV RNA levels within 1 . All of the above A. delavirdine.4 months B. Harrison’s 18th Ed. Nevirapine Harrison’s 18th Ed. Atazanavir C. or line probe assays. Tipranavir D. Nelfinavir B. Indinavir 412 Enfuvirtide is best related to which of the following ? Harrison’s 18th Ed. Maraviroc B. 417 Genotyping of HIV quasispecies may be done by ? Harrison’s 18th Ed. or ritonavir. Should not be used as monotherapy for HIV infection and increased during concurrent therapy with ketoconazole.6 months Harrison’s 18th Ed. All of the above Genotyping of HIV quasispecies may be done through dideoxynucleotide sequencing. Every 3 .6 months C. Line probe assays B. Delavirdine Raltegravir is an inhibitor of the viral enzyme integrase. AJAY MATHUR Miscellaneous B. . All of the above A. B. 0% with insertive anal intercourse & receptive vaginal intercourse. some D. ~ 6. Mycobacterial interference repeat units C. Mycobacterial interspersed representative units B. bovis.4 % 426 Robert Koch won the Nobel Prize in which year ? N Engl J Med 2012.09 % Chapter 165.361:1768-75 A. africanum 422 The estimated risk of HIV transmission with sharing needles for D. Swollen glands C. tuberculosis Estimated per-contact risk of HIV transmission from sexual exposure are 1 to 30% with receptive anal intercourse. . MIRUs. 36 hours B. 1984 C.1 to 1. None of the above except ? A. 1915 transmission from sexual exposure ? N Engl J Med 2009. 0. 6 weeks Risk of HIV infection following a percutaneous exposure to HIV-contaminated blood is ~0. 0. 1982 A. N Engl J Med 2009. ~ 0. Wasting away C.1 % Harrison’s 18th Ed.1 to 1. B. tuberculosis complex includes all of the following species D. 1920 A. Tuberculosis C.361:1768-75 A. Mycobacterial interspersed repeat units C.09%). 48 hours C.361:1768-75 D. His manuscript was published in the Berliner Klinische Wochenschrift.0% with insertive vaginal intercourse C. ~ 1. >= 1500 copies per milliliter containing identical repeat units & others containing repeats that vary slightly in sequence & length.9 % D. 1910 421 Which of the following is false about per-contact risk of HIV C.2 % A.1 to 10. >= 1000 copies per milliliter Genome of M. 1583 B.67% per needle-sharing contact. AJAY MATHUR Cardiology 859 418 The first case of HIV transmission from a patient to health care 424 Greater benefit of postexposure HIV prophylaxis is obtained when worker was reported in ? it is initiated within ? Harrison’s 18th Ed. Harrison’s 18th Ed.349:1149-56 423 Source HIV-positive patients have a viral load of ? N Engl J Med 2009. N Engl J Med 2009.3 % B. ~ 0. M. ~ 0.0% with receptive vaginal intercourse 427 M. 1983 B. 1 to 30% with receptive anal intercourse Robert Koch received the 1905 Nobel Prize. tuberculosis contains many mycobacterial interspersed repeat units (MIRUs). 4 weeks D.67 % per needle-sharing contact A. >= 100 copies per milliliter B.361:1768-75 A.67 % per needle-sharing contact 428 Tuberculosis also called “phthisis” which in Greek means ? B. & B. 2 weeks C. ~ 0. Dark pigmentation of skin D. ~ 0. 1582 N Engl J Med 2009. followed by gel electrophoresis.67 % per needle-sharing contact D. tuberculosis means ? N Engl J Med 2003. 1985 D.4 % C. 429 MIRUs in genome of M. bovis 0. with the use of a polymerase-chain-reaction (PCR) assay.361:1768-75 425 Postexposure ARV prophylaxis (PEP) is given for ? A. 72 hours D.361:1768-75 A. 0. ~ 1. tuberculosis complex includes M. M.1 to 10. ~ 0. africanum & M. Greater 419 The risk of HIV infection following a percutaneous exposure to benefit of PEP is obtained when initiated within 36 hours after exposure as compared with 72 hours HIV-contaminated blood is ? after exposure.67 % per needle-sharing contact B. M. A. xenopi injection-drug use is ? M. 96 hours Postexposure prophylaxis should be initiated as rapidly as possible after exposure to HIV. ~ 0. ~ 0. 1905 B. C. Mycobacterial interpolated repeat units A. tuberculosis.0% with insertive vaginal intercourse. >= 500 copies per milliliter D. Chronic cough The estimated risk of transmission associated with sharing needles for injection-drug use is approximately 0. N Engl J Med 2009. 859 Miscellaneous MCQ’s FORMCQ’s FOR MEDICAL MEDICAL PROFESSIONALS PROFESSIONALS BY PROF. M.3% D. MIRU genotyping categorizes the number & size of the repeats in each of 12 independent Source HIV-positive patients have a high viral load of >= 1500 copies/mL.367:931-6 Splashes of infectious material to mucous membranes (conjunctivae or oral mucosa) or broken skin may transmit HIV infection (risk per exposure is 0. microti. 1583. M. Harrison’s 18th Ed. 1583. ~ 3. 8 weeks Recommendations postexposure ARV prophylaxis (PEP) are that a combination of two NRTI’s for 420 The risk of HIV infection following a mucous membrane exposure less severe exposures & combination of two NRTI’s plus a third drug for more severe exposures to HIV-contaminated blood is ? be given for 4 weeks. M.04 % B. Complement activation Nocardia and Rhodococcus. Aerobic bacterium B. tuberculosis comprises Harrison’s 18th Ed. Cryptosporidium Harrison’s 18th Ed. 2000 genes C. and other cell-wall lipids. C. As a result. 1343 B. and the protozoa Isospora and Cryptosporidium. M. tuberculosis is a rod-shaped. 4000 genes D. Nocardia surface molecules ? Harrison’s 18th Ed. Legionella micdadei B. Phosphatidyl-inositol-3-phosphate [(PI(3)P)] and lipoarabinomannan. 431 Acid fastness of M. Phosphatidyl-inositol-1-phosphate [(PI(1)P)] D. The complete genome sequence of M. All of the above A. Production of liposomal enzymes C. A. tuberculosis 439 Phagosome-lysosome fusion is related to ? C. Mycolic acids B. an essential component of phago-lysosome biogenesis at the phagosomal membrane. 1343 D. A large proportion of genes are devoted to the production of enzymes involved in cell wall metabolism. Arabinogalactan lysosome biogenesis ? C. C. A. 1340 devoted to ? A. peptidoglycan C. Lipoarabinomannan (LAM) A. arabinogalactan. Rod-shaped Harrison’s 18th Ed. Ca2+/calmodulin pathway D. All of the above A. Complement receptors C. Spore-forming A. tuberculosis is false ? 436 Complete genome sequence of M. Mannose receptor Microorganisms other than mycobacteria that display some acid fastness include species of B. LAMP1 Lipoarabinomannan present in mycobacterial cell wall is involved in the pathogen-host interaction B. Production of enzymes involved in cell wall metabolism D. Rab7 . Glycogenolysis D. EEA1 (early endosome antigen 1) and facilitates the survival of M. Mycolic acids in cell wall A. Tyrosine in cell wall B.complement receptors. 1340 B. PI(3)P within macrophages ? dependent recruitment of the late endosomal markers Hrs (hepatocyte growth factor regulated Harrison’s 18th Ed. Legionella micdadei.5 µm by 3 µm. Arabinogalactan 441 Which of the following is a PI(3)P binding protein ? C. tuberculosis Mycobacterial prevent the accumulation of phosphatidyl-inositol-3-phosphate [(PI(3)P)]. tuberculosis comprises 4043 genes encoding 3993 proteins & 50 genes encoding RNAs. Isospora mannose receptor. 3000 genes D. tuberculosis bacterium is is due to high content of ? 437 Large proportion of the genes in M. Rhodococcus A. 1340 tyrosine kinase substrate) and EEA1 (early endosome antigen 1) via PI(3)P is abolished and phsgosomes fail to fuse with lysosomes. tuberculosis genome are Harrison’s 18th Ed. Measures about 0. long-chain cross- linked fatty acids. 1340 Mycobacteria bind to the following macrophage cell-surface molecules . 5000 genes M.860 MCQ’s FOR MEDICAL PROFESSIONALS BY PROF. immunoglobulin GFc  receptor and type A scavenger receptors. 1340 about ? A. Type A scavenger receptors 434 Which of the following is a component of mycobacterial cell wall ? Lipoarabinomannan (LAM) inhibits intracellular increase of Ca 2+ thereby impairing Ca2+/calmodulin Harrison’s 18th Ed. Phosphatidyl-inositol-4-phosphate [(PI(4)P)] 435 Which of the following facilitates the survival of M. B. 1343 D. 1340 438 Mycobacteria bind to which of the following macrophage cell- A. 1340 pathway which is essential in phagosome-lysosome fusion & eventually arresting phagosome maturation. non-spore-forming. 1340 B. 432 Which of the following microorganisms display acid fastness ? Harrison’s 18th Ed. Phenolic acids in cell wall Harrison’s 18th Ed. D. tuberculosis within macrophages.5 µm by 3 µm C. Phosphatidyl-inositol-2-phosphate [(PI(2)P)] Molecules in the mycobacterial cell wall include mycolic acid. thin aerobic bacterium measuring 0. A. Mycolic acids 440 Which of the following is an essential component of phago- B. Production of proline-rich proteins Acid fastness is due mainly to the organisms’ high content of mycolic acids. Mannose receptor D. Lipoarabinomannan Harrison’s 18th Ed. Immunoglobulin GFc receptor 433 Which of the following microorganisms display acid fastness ? D. All of the above C. AJAY MATHUR Miscellaneous 430 Which of the following about M. All of the above Harrison’s 18th Ed. All of the above B. Glucosamine in cell wall C. other routes of transmission of tubercle bacilli are through the skin or the placenta. Rupture of cell wall of bacteria by antibody A. 861 Miscellaneous MCQ’s FORMCQ’s FOR MEDICAL MEDICAL PROFESSIONALS PROFESSIONALS BY PROF. HIV infection B. M. Intravenous drug use C. 45 % C. 445 Among infected persons. destroy the bacteria. self-healed. Nonspecific activation of macrophages B. Jejunoileal bypass D. while at older ages the opposite is true. 65 % M. 446 Conditions known to increase risk of active tuberculosis among 452 Mycobacterium tuberculosis is a slow-growing bacteria that persons infected with tubercle bacilli include ? divides every ? Harrison’s 18th Ed. and two PI(3)P binding proteins EEA1 and Hrs. H39R early adulthood. which are aerosolized by coughing. 698 Harrison’s 18th Ed. Thus. in ? tuberculosis is able to survive within macrophage. 48 .60 hours C. Middle age B. Children 451 Best characterized strain of M. the enzymes are never activated within macrophage and therefore. Sneezing B. LAMP1.90 hours D.48 hours B.20 hours. 448 5-year mortality rate among sputum smear positive cases of 442 M. v-ATPase.20 hours A. Placenta B. All of the above C. AJAY MATHUR Cardiology 861 D. The best characterized strain of Mycobacterium tuberculosis is H37R. Harrison’s 18th Ed. tuberculosis A. Old age C. rates among women are higher than those among men. Moderately acidic D. Late adolescence and early adulthood A. opsonized (Greek. the incidence of tuberculosis is highest internal environment of the macrophage is never acidified. 35 % B. 72 . Mildly acidic B. Silicosis B. Coughing A. 16 . Aerosolized droplet nuclei resides and survives is ? A. Skin C . All of the above Mycobacterium tuberculosis is a slow-growing bacteria that divides every 16 . 55 % D. Severely acidic D. Coating by antibody & complement in preparation for phagocytosis C. M. H36R C. < 20 µm in diameter C. 1342 450 Internal environment of macrophage where M. H38R Among infected persons. tuberculosis is most commonly transmitted from a patient with infectious pulmonary tuberculosis The 5-year mortality rate among sputum smear positive cases of untreated tuberculosis is 65%. < 10 µm in diameter B. 444 Route of transmission of tubercle bacilli is ? Harrison’s 18th Ed. Invasion of macrophages by bacteria D. Slow generation time B. 1342 A. the incidence of tuberculosis is highest during late adolescence and D. tuberculosis droplet nuclei may be aerosolized by ? untreated tuberculosis is ? Harrison’s 18th Ed. Old. tuberculosis can only replicate & grow within a host organism. to other persons by droplet nuclei. 36 . 449 Opsonization refers to ? 443 What is the size of the M. All of the above D. Normally. None of the above Apart from aerosolized droplet nuclei. H37R D. Gastrectomy and jejunoileal bypass surgery A. Speaking C. fibrotic lesions Mycobacterial phagosomes are characterized by the absence of late endosomal or lysosomal D. Phagolysosome formation . 1342 Table 165-1 Mycobacterium tuberculosis ? A. sneezing. or speaking. tuberculosis is ? B. 1342 Harrison’s 18th Ed. tuberculosis resides in a compartment of macrophages that does not contain lysosomal markers. 1342 A. v-ATPase C. In the age group of 25 to 34 years. M. Its vacuolar membrane lacks host vacuolar ATPase that is needed for vesicle acidification. Mental retardation markers like Rab7. < 40 µm in diameter Although tissue macrophages & polymorphonuclear leukocytes (PMNs) are capable of killing Droplet nuclei <10 µm in diameter may remain suspended in air for several hours and may gain microorganisms without help. M. 1342 Table 165-1 A. tuberculosis phagosome inhibits production of phosphatidylinositol 3-phosphate (PI3P). most phagosomes for membrane sorting & maturation including phagolysosome formation. < 30 µm in diameter D. tuberculosis infecting droplets ? Harrison’s 16th Ed. they function much more efficiently when pathogens are first direct access to terminal air passages when inhaled. which would important being HIV co-infection. “to prepare for eating”) by components of complement system such as C3b &/ or by antibodies. PI3P earmarks Several conditions favor the development of active tuberculosis among infected individuals. 447 Conditions known to increase risk of active tuberculosis among persons infected with tubercle bacilli include all except ? 453 Which of the following best relates to the pathogenicity of Harrison’s 18th Ed. Lysosomal enzymes are only effective in acidic environments. 1342 A. 1995 D. . Macrophage-activating response Strains of the Beijing/W genotype family are associated with higher mortality rates. Haarlem family. tuberculosis Beijing isolates. NRAMP1 (chromosome 2q) may play a role in innate nonimmune resistance to infection with M. First. Tissue-damaging response D. 1-2 C. AJAY MATHUR Miscellaneous C. Fusion between phagosomes & lysosomes D.mediated phenomenon) and second a tissue-damaging response (delayed-type hypersensitivity (DTH) reaction). rpoV genes result in loss of virulence. a macrophage-activating CMI response (T cell . 10 % A. several genes. Lower mortality rates B. IS6110 sequence is not found in NTM. Recruitment of immature monocyte-derived macrophages C.mediated phenomenon” ? Harrison’s 18th Ed. 15 % B. Gibralter Phagosome maturation arrest best relates to pathogenicity of M. These help keep vacuoles away from bactericidal and antigen processing organelles in infected macrophages. 1343 456 Beijing family of M. 20 % C. tuberculosis strains was described in 1995.862 MCQ’s FOR MEDICAL PROFESSIONALS BY PROF. katG gene encodes for catalase/peroxidase enzymes that D. 1990 reaction” ? Harrison’s 18th Ed. tuberculosis strains was described in year ? A. rpoV. 1343 C. 1343 B. Macrophage-activating response Revised National Tuberculosis Control Program was begun in 1993. 1342 susceptibility to tuberculosis ? Harrison’s 18th Ed. such as M. The erp gene. 1993 A. Defects in these two genes result in loss of virulence. Highest density found in Beijing A. katG B. Mycobaterial B. tuberculosis and development of disease. erp D. LAM prevents increases of Ca 2+ surge. EEA1 and syntaxin 6 are necessary components of the lysosomal compartment. Defects in katG. Fusion between phagosomes & lysosomes C. 2-4 D. 462 Which of the following is a “T cell . 457 Revised National Tuberculosis Control Program started in ? 463 Which of the following is a “delayed-type hypersensitivity (DTH) A. 1995 B. tuberculosis infection. BCG-induced immunological defense may protect against most M. 1984 B. tuberculosis genotype family ? D. two host responses to M. Haarlem prevents the interaction of a kinase and downstream recruitment of EEA1. 25 % D. tuberculosis genotype families are Beijing family.mediated phenomenon resulting in activation of Beijing family of M. tuberculosis strains but not against selected D. 455 Gene thought to confer virulence to M. rpoV C. 1990 C. tuberculosis microorganisms reach the alveoli ? 460 Which of the following gene in humans plays a role in determining Harrison’s 18th Ed. 1343 B. tuberculosis infection ? Harrison’s 18th Ed. the X clade. rpoV A. 1998 C. 454 What fraction of inhaled droplet nuclei containing M. Largest genotype family of M. D. macrophages capable of killing & digesting tubercle bacilli. tuberculosis. also contributes to virulence.4 weeks after M. tuberculosis are katG. Phagosome maturation arrest 459 Which of the following is not a M. erp. A. Tissue-damaging response 458 Beijing/W genotype family of M. Multibanded IS6110 restriction fragment length polymorphism 464 Caseous necrosis of involved tissues is the result of ? (RFLP) patterns Harrison’s 18th Ed. rpoV is the main sigma factor initiating transcription of Cell mediated immunity (CMI) & humoral immunity begin 2 . tuberculosis is ? Harrison’s 18th Ed. tuberculosis is associated with About 2-4 weeks after infection. Recruitment of immature monocyte-derived macrophages D. erp B. NRAMP1 Majority of inhaled TB bacilli are trapped in upper airways and expelled by ciliated mucosal cells. 4-6 Genes that confer virulence to M. Product of rpoB gene is RNA polymerase. tuberculosis develop to inhibit all except ? mycobacterial growth. Latino-American phagosomal maturation arrest is by LAM-mediated disruption of Ca 2+ calmodulin dependent C. All of the above C. Mediterranean regulation of PI3P & EEA1 on phagosomes. Macrophage-activating response C. Tissue-damaging response genotypes (called escape variants). The tissue-damaging response destroys unactivated macrophages containing multiplying bacilli and causes caseous necrosis of involved tissues. M. D. 6-8 protect against oxidative stress. encoding a protein required for multiplication. In humans. katG many weeks after M. 2000 Macrophage-activating response is a T cell . tuberculosis B. about 10% reach the alveoli. Resistance to osmotic lysis A. Recruitment of immature monocyte-derived macrophages B. Fusion between phagosomes & lysosomes A. 1343 461 Cell mediated immunity (CMI) & humoral immunity begin how A. 1343 A. Latino-American and Mediterranean family. They are less affected by malnutrition. Due to endogenous reactivation of latent infection B. Hemorrhage in a tubercular cavity D. Lymphedema C. None of the above Alveolar macrophages. Seen in children Harrison’s 18th Ed. All of the above D. Postprimary disease is due to endogenous reactivation of latent infection. progressively debilitating course of A. Exasperate 469 CFP10 stands for ? Lancet Infect Dis 2005. Superior segments of lower lobes frequently involved D. Pulmonary artery aneurysm secondary to tuberculosis M tuberculosis are commercially available in ELISA & ELISPOT formats with higher sensitivity than tuberculin skin test (TST). Aspergilloma formation in an old cavity D. Alveolar macrophages A. None of the above Soft cheese or caseous necrosis may also be observed in neoplasms. reactivation. monocytes. Any of the above Growth of M. tuberculosis is inhibited within necrotic environment by low oxygen tension & low pH. Neoplasms A. Dendritic cells C. from rupture of a dilated vessel in a cavity (Rasmussen’s aneurysm) or from aspergilloma formation in an old cavity. A small calcified TB nodule in lung C.5:415-30. Corrosion C. 1345 A. 1345 D. C. wall of a cavity. Consumption D. Reactivation tuberculosis C. Nitric oxide C. Any of the above Primary pulmonary tuberculosis results from an initial infection with tubercle bacilli. Localized to apical & posterior segments of upper lobes C. Cytoplasmic filtrate protein 10 A. Monocytes B. Rupture of a dilated vessel in a cavity lymphadenopathy C. the lung lesion may heal spontaneously and appear as a small calcified nodule termed Ghon lesion at a later date. Localized to middle and lower lung zones A. 1345 A. 466 Growth of M. It can lead to hemoptysis. Dilated vessel in a tubercular cavity D. It is usually localized to apical and posterior segments of the upper lobes & superior segments of lower lobes are frequently involved. Harrison’s 18th Ed. Lesion is central & accompanied by hilar or paratracheal B. or secondary tuberculosis. tuberculosis antigens to T lymphocytes. Early secretory antigenic target 6 A. 1343 D. progressively debilitating course of postprimary pulmonary tuberculosis is also called “consumption”. B. HIV infection & BCG vaccination. 1345 C. Aortic aneurysm secondary to tuberculosis Newer in-vitro T-cell based interferon gamma assays using early secretory antigenic target 6 (ESAT6) & culture filtrate protein 10 (CFP10) that are absent in BCG but are major targets of T-cell response to B. 1344 Harrison’s 18th Ed. 467 Which of the following plays a role in processing & presenting M. Secondary tuberculosis D. Cryo filtrate protein 10 475 Rasmussen’s aneurysm refers to ? Harrison’s 18th Ed. Epidemic secretory antigenic target 6 Harrison’s 18th Ed. AJAY MATHUR Cardiology 863 465 Caseous necrosis may also be observed in ? 471 What is Ghon lesion ? Harrison’s 18th Ed. and dendritic cells are critical in processing & presenting M. 468 ESAT6 stands for ? Lancet Infect Dis 2005. tuberculosis is false ? Harrison’s 18th Ed. 863 Miscellaneous MCQ’s FORMCQ’s FOR MEDICAL MEDICAL PROFESSIONALS PROFESSIONALS BY PROF. tuberculosis is inhibited by ? Harrison’s 18th Ed. 1343 474 The process of chronic. Episodic secretory antigenic target 6 B. Monocytes & macrophages produce nitric oxide which has antimycobacterial activity.5:415-30. Culture filtrate protein 10 The process of chronic. Decay A. 1344 472 Post primary pulmonary tuberculosis is also called ? Harrison’s 18th Ed. 1345 476 Massive hemoptysis in pulmonary TB is due to ? A. Calcified paratracheal lymph node B. 1345 B. The lesion forming Massive hemoptysis in pulmonary TB may be a consequence of erosion of a blood vessel in the after infection is usually “peripheral” and accompanied by hilar or paratracheal lymphadenopathy. Essential secretory antigenic target 6 postprimary pulmonary tuberculosis is also called ? B. Post primary pulmonary tuberculosis is also called adult-type. Lesion heals spontaneously D. All of the above D. Adult-type tuberculosis B. Burns B. Erosion of blood vessel in wall of a cavity C. Low oxygen tension A. Low pH B. tuberculosis antigens to T lymphocytes ? 473 Which of the following about postprimary pulmonary tuberculosis Harrison’s 18th Ed. All of the above D. Harrison’s 18th Ed. 1345 A. Dilatation of a major bronchus 470 Which of the following statements about primary pulmonary A dilated vessel in a tubercular cavity is called Rasmussen’s aneurysm. It is often seen in children and is frequently localized to middle and lower lung zones. . Cell filtrate protein 10 C. In primary pulmonary tuberculosis. 1344 is false ? Harrison’s 18th Ed. the extrapulmonary sites most commonly involved in tuberculosis are the lymph nodes. peritoneum & pericardium. Caseating epithelioid cell granulomas D. AJAY MATHUR Miscellaneous 477 In pulmonary TB. Full meal the following sites ? Harrison’s 18th Ed. Kikuchi-Fujimoto disease 483 In what percentage of tubercular lymph-node biopsy are AFB seen ? D. 50 % . 957 Features of a typical tuberculous lymphadenitis include areas of tuberculous granulation tissue A. Healed calcified lesion in lung parenchyma & hilar lymph nodes with full-blown caseating epithelioid cell granulomas. B.864 MCQ’s FOR MEDICAL PROFESSIONALS BY PROF. 80 % areas during inspiration especially after ? D. mesangial proliferative glomerulonephritis and also nephrotic syndrome). Kikuchi-Fujimoto disease B. Intermittent Lymph-node tuberculosis mostly presents at posterior cervical and supraclavicular sites (scrofula). detectable rales can be heard over involved areas during inspiration especially after coughing. A. Langerhans’ giant cells In TB. It can precede or occur in association with SLE. AFB are seen in up to 80% of cases. Pleura Renal involvement is its only systemic manifestation (membranous glomerulonerphritis. Lung A. benign. 1346 Mikulicz’s disease is a benign lymphoepithelial lesion presenting as benign enlargement of A. Bones & joints 489 Benign lymphoepithelial lesion is best related to ? A. Genitourinary tract D. Axilla 478 In TB. Remittent 485 Features of a typical tuberculous lymphadenitis include ? C. Inguinal A. Meninges C. meninges. which is the least commonly involved site B. Any of the above B. 25 % parotid and/or lacrimal glands sometimes associated with Sjögren’s syndrome. Bones & joints & submandibular lymphadenopathy. C. None of the above B. Genitourinary tract 487 The only systemic manifestation of Kimura’s disease is ? D. presents as fever. Necrotizing lymphadenitis D. Renal in extrapulmonary tuberculosis ? C. bones and joints. Lying prone 484 Lymph-node tuberculosis most commonly presents at which of C. Cardiac 481 Out of the following. B. Exercise B. 1346 D. Continuous A. peripheral eosinophilia and elevated IgE level. Calcification of pleura 486 Which of the following about Kikuchi-Fujimoto disease is false ? A. Mikulicz’s disease In order of frequency. fever is ? Harrison’s 18th Ed. which is the least commonly involved site 488 Histiocytic necrotizing lymphadenitis is best related to ? in extrapulmonary tuberculosis ? A. Genitourinary tract change glomerulonephritis. CNS Harrison’s 18th Ed. Posterior cervical C. Kimura’s disease C. minimal C. self-limited disease. 1346 D. 482 Out of the following. diffuse proliferative glomerulonephritis. C. B. Tuberculous lymphadenitis Harrison’s 18th Ed. All of the above Harrison’s 16th Ed. pleura. Kimura’s disease A. Pleura hepatosplenomegaly. 100 % Harrison’s 18th Ed. 1346 D. 1346 D. Tuberculous lymphadenitis C. 1346 B. the fever is often low-grade and intermittent in up to 80% of cases. B. Lymphocytic infiltrates 479 What is Ranke complex ? D. Can occur in association with SLE A. detectable rales can be heard over involved C. Anterior cervical In pulmonary TB. and B. B. genitourinary tract. predominantly occurs in young women. self-limited disease 480 Which is the most commonly involved site in extrapulmonary C. Lymph nodes Features of Kikuchi-Fujimoto disease are necrotizing lymphadenitis. cervical lymphadenopathy. Meninges Kimura’s disease is a benign allergic inflammatory disorder of unknown cause endemic in Orientals. Bones & joints A. Coughing A. 1345 In fine-needle aspiration or surgical biopsy of TB lymph node. Langerhans’ giant cells & lymphocytic infiltrates. Predominantly in old males tuberculosis ? Harrison’s 18th Ed. Fibrocavitatory lung lesion C. Benign. B. Peritoneum D. It typically presents as non-tender preauricular D. Mikulicz’s disease Harrison’s 18th Ed. Preferential involvement of epididymis C. Hematogenous spread B. Upper thoracic In tuberculous pleural fluid. Tuberculosis of bones & joints Tuberculous empyema is the result of rupture of a tubercular cavity into pleural space. Lymph-node tuberculosis disease. tuberculosis preferentially affects the epididymis which may form an external 492 Which of the following about tuberculous pleural fluid is false ? fistula.5%. Mononuclear cells Harrison’s 18th Ed. TB involvement of pleura ~20%. 491 Human Herpes Virus 8 (HHV8) is associated with ? 497 In males. All of the above and multicentric Castleman’s Disease. which of the following is most frequently A. genital tuberculosis preferentially affects which of the following ? A. 1347 B. Normal to low glucose concentration A. Ankle joint 493 Which of the following cells is rare in tuberculous pleural fluid ? Weight-bearing joints are affected most commonly in skeletal tuberculosis. Scrofula 501 Pathogenetic mechanism of gastrointestinal tuberculosis is ? C. Harrison’s 18th Ed. May result in restrictive lung disease Harrison’s 18th Ed. hematogenous spread or ingestion of milk from cows with bovine tuberculosis. Castleman’s disease B. Due to a bronchopleural fistula from pulmonary lesion frequent ? C. D. Cervical D. Swallowing of sputum with direct seeding B. upper thoracic spine is the most common site of spinal tuberculosis. 1347 Tuberculosis of bones & joints ~10%. Lower thoracic Harrison’s 18th Ed. 1348 D. pH ~ 7. Orchitis and prostatitis may also develop. It may result in severe pleural fibrosis & restrictive lung C. Advanced cavitary pulmonary TB Harrison’s 18th Ed. It may involve larynx. Primary Effusion lymphoma A. Kikuchi-Fujimoto disease Calcifications and ureteral strictures on IVP and documentation of culture-negative pyuria in acidic urine raises the suspicion of tuberculosis. Involvement of spine Harrison’s 18th Ed. Lower lumbar 494 Which of the following about tuberculous empyema is true ? In children. C. Mesothelial cells A. A. 1348 D. Spine D. A. 865 Miscellaneous MCQ’s FORMCQ’s FOR MEDICAL MEDICAL PROFESSIONALS PROFESSIONALS BY PROF. WBC count usually 500 to 6000/µL affected ? B. Prostate Human Herpes Virus 8 (HHV8) is associated with Kaposi’s sarcoma. Genitourinary tuberculosis B. 1347 is more frequent than hip and knee. Hematogenous spread Tuberculosis of upper airways is a complication is nearly always a complication of advanced cavitary pulmonary tuberculosis. pH generally > 7. All of the above 496 Which of the following suggest tuberculosis as a cause of genitourinary disease ? Pathogenetic mechanism of gastrointestinal tuberculosis may be swallowing of sputum with direct Harrison’s 18th Ed. Any of the above A.4 B. 1347 seeding. Testes D. Knee joint normal to low glucose concentration. A. mononuclear cells C. D. Gastrointestinal tuberculosis 3. Later. Kimura’s disease D. a C. 1348 C.3. Kaposi’s sarcoma Harrison’s 18th Ed. Primary Effusion lymphoma D. Multicentric Castleman’s Disease B. or of a bronchopleural fistula from a pulmonary lesion. Calcifications and ureteral strictures on IVP B. Due to rupture of a tubercular cavity into pleural space 500 Of the extrapulmonary TB cases in US. and detectable WBC (500 to 6000/µL). Ingestion of milk from tubercular cows D. All of the above B. 1347 and upper lumbar vertebrae are usually affected in adults. In male patients. neutrophils may predominate in early stage. 1346 498 In skeletal tuberculosis. Lower thoracic are the typical finding. Neutrophils 499 Most common site of spinal tuberculosis in adults is ? B. All of the above D. All of the above C. . Tuberculosis of CNS ~5%. Protein concentration > 50 % of that in serum Harrison’s 18th Ed. 1347 C. Gastrointestinal tuberculosis 495 Tuberculosis of upper airways is a complication of ? Lymph-node tuberculosis >40%. Harrison’s 18th Ed. Epididymis C. which is the least B. All of the above A. Genitourinary tuberculosis ~15%. Castleman’s disease is also referred to as angiofollicular hyperplasia. Culture-negative pyuria in acidic urine A. AJAY MATHUR Cardiology 865 490 Angiofollicular hyperplasia is best related to ? A. D. Hip joint The tubercular pleural fluid is an exudate with a protein concentration >50% of that in serum. Mesothelial cells are generally rare or absent. pharynx and epiglottis. Mikulicz’s disease C. 511 Which of the following is false about ‘Lupus vulgaris’ ? 505 In miliary tuberculosis. Any of the above Nonreactive miliary tuberculosis is an acute septicemic form. chest radiography reveals a miliary reticulonodular pattern that is more Tissue specimen intended for tubercular culture should not be put in formaldehyde. Negative TST Clinical manifestations of miliary tuberculosis are protean. Most lesions occur in head and neck area C. 510 Pancytopenia is common in which of the following ? Harrison’s 18th Ed. Increased frequency of sputum-smear negativity B. Atypical radiographic findings C. Cryptic miliary tuberculosis A. Formaldehyde D. Most lesions occur in head and neck area and are red-brown plaques with a yellow- Lesions in miliary tuberculosis are yellowish granulomas. Choroidal tubercles D. multiple necrotic but nongranulomatous (“nonreactive”) lesions are detected. . Amoebic colitis D. 1349 Harrison’s 18th Ed. Nonreactive miliary tuberculosis B. 1348 B. 1349 A. Miliary tuberculosis Harrison’s 18th Ed. Ideally exposed C. and ultimately meningeal involvement preceding death. and In tuberculosis in HIV-infected patients there is increased frequency of sputum-smear negativity. Black D. lymphadenopathy are frequent physical findings. Histopathologically. Neonates Harrison’s 18th Ed. Overpenetrated B. ulcerations & fistulae may simulate Crohn’s disease. Underpenetrated A. Rectum Cryptic miliary tuberculosis is seen in elderly. 507 Miliary reticulonodular pattern is more easily seen on chest 513 Tissue specimen intended for tubercular culture should not be radiography that is ? put in ? Harrison’s 18th Ed. All of the above C. Children A. which one is pathognomonic of miliary 512 Which of the following statements about tuberculosis in HIV- tuberculosis ? infected patients is false ? Harrison’s 18th Ed. 1349 504 The mechanism of tuberculous peritonitis is ? A. 1350 A. Hepatomegaly. With intestinal-wall involvement in tuberculosis. White Lupus vulgaris is a form of cutaneous tuberculosis that is seen in previously infected and sensitized individuals. Direct spread of tubercle bacilli from intraabdominal organ D. anemia. 1349 gastrointestinal tuberculosis ? A. Any portion of the gastrointestinal tract may be affected by tuberculosis. Any of the above In miliary tuberculosis. 1349 Harrison’s 18th Ed. Choroidal tubercles in eye examination is atypical radiographic findings (lower-zone infiltrates without cavity formation). Elderly C. the presentation 508 HRCT finding in miliary tuberculosis is termed as ? may simulate which of the following ? A. due to massive hematogenous dissemination of tubercle bacilli. Terminal ileum and cecum are the sites most commonly involved. Saline B. granuloma formation in late stages and negative TST.866 MCQ’s FOR MEDICAL PROFESSIONALS BY PROF. 1349 Harrison’s 18th Ed. Duodenum C. Pancytopenia is common. the colour of granuloma lesions is ? Harrison’s 18th Ed. brown color on diascopy. 1 to 2 mm in diameter resembling millet seeds. A form of cutaneous tuberculosis A. Anal canal intermittent fever. It has a chronic course characterized by mild D. easily seen on underpenetrated film. None of the above D. Glazed glass appearance Harrison’s 18th Ed. AJAY MATHUR Miscellaneous 502 With intestinal-wall involvement in tuberculosis. Ulcerative colitis C. 1349 A. Sandstorm appearance C . 1348 B. Any of the above D. Appendicitis Coalescing nodules result into patchy irregular opacities and HRCT shows this variation effectively D. White froth appearance A. lack of classic pathognomonic of miliary tuberculosis. Snowstorm appearance B. Hepatomegaly A. Yellow C. 509 Cryptic miliary tuberculosis is mostly seen in ? 503 Which of the following is most commonly involved in Harrison’s 18th Ed. Adults B. Crohn’s disease and has been described as “snowstorm appearance”. Brown B. Tuberculous peritonitis occurs either by direct spread of tubercle bacilli from ruptured lymph nodes and intraabdominal organs or hematogenous seeding. Lack of classic granuloma formation in early stages D. Hematogenous seeding D. splenomegaly. 1348 B. Alcohol C. Splenomegaly B. Seen in previously infected and sensitized individuals B. Direct spread of tubercle bacilli from ruptured lymph nodes C. 506 Out of the following. Cecum D. Lymphadenopathy C. virtually any radiographic pattern from a normal film or a solitary pulmonary nodule to diffuse alveolar infiltrates can be encountered. Pyrazinamide empyema. was termed as ? Harrison’s 18th Ed. RT24 D. Tuberculostearic acid (TBSA) is the characteristic fatty acid of acid-fast mycobacterial cell wall of the order Actinomycetales. RT22 B. B. who often do not expectorate D. WHO and UNICEF later sponsored large- scale production of a master batch of PPD. Values <1. D. B. parapneunomic effusions and mesothelioma. 1351 516 Which of the following culture medium is used to grow A. Throat swab A. ADA1 is found in all cells and ADA 2 reflects monocyte/macrophage activation. Isoniazid . 1034 Adenosine deaminase (ADA) catalyzes the conversion of adenosine to inosine and is relaeased A. the ratio of bromide in serum to A. measles. specimen for culture can be obtained from ? ZN staining requires 10 5 bacilli / ml. pyogenic meningitis and hypothyroidism. whilst those with empyema and parainfective effusions have B. AJAY MATHUR Cardiology 867 514 Which of the following is used for staining AFB ? 520 Which of the following radiographic pattern is considered Harrison’s 18th Ed. Patients with TB A. termed RT23. Adenosine deaminase (ADA) B. C. Upper lobe disease with infiltrates and cavities D. ADA 4 uric acid level ? Harrison’s 17th Ed. Kirschner’s For the diagnosis of primary pulmonary tuberculosis in children. and made it available for general use. Early-morning gastric lavage B. Bromide partition test C. None of the above 515 Ziehl-Neelsen staining requires how many bacilli / ml ? Although “classic” radiographic picture of pulmonary tuberculosis is upper lobe disease with infiltrates A. All of the above C. Clinical gout is rare.Egg based (LJ. 522 Master batch of PPD produced on directions of WHO and UNICEF Agar based (Middlebrook 7H10 or 7H11) and Liquid based (Kirschner’s. ADA is also found in inflammatory fluids associated with RA. Its incidence is reduced by concurrent rifampin therapy. Harrison’s 18th Ed. 867 Miscellaneous MCQ’s FORMCQ’s FOR MEDICAL MEDICAL PROFESSIONALS PROFESSIONALS BY PROF. Lysine IGRAs stands for IFN. (Adenosine + H 2O + ADA = Inosine + NH3 +ADA). Nasal secretion mycobacteria ? B. Detection of TBSA in CSF by use of gas chromatography-mass spectrometry 523 IGRAs stands for ? has proven to be a very rapid. Gas chromatography of mycobacterial fatty acids (Tuberculostearic acid) D. 102 bacilli / ml and cavities. It may be false +ve in herpes simplex. listeria. Harrison’s 18th Ed. ADA 3 525 Which of the following antituberculosis medication causes elevated D. Glycine Harrison’s 17th Ed. lymphoma. Inosine 524 Which of the following antituberculosis medication causes elevated C. RT23 C. The partition of bromide ion between serum and CSF after a loading dose reflects the integrity of the blood brain barrier. No radiographic pattern can be considered pathognomonic. Ratio of ADA 1 / ADAp <0. 1351 517 Biochemical marker test for diagnosis of tuberculosis is ? A. B. Auramine-rhodamine staining Harrison’s 18th Ed. Release Assays. IFN- Related Assays that in CSF can be estimated. 103 bacilli / ml C. Middlebrook 7H10 D. Solitary pulmonary nodule D. 105 bacilli / ml who often do not expectorate sputum. IFN- Response Assays C. All of the above sputum. Petragnani and ATS). 1350 pathognomonic for pulmonary tuberculosis ? A. All of the above PPD-S was developed by Seibert and Glenn in 1941. specimens from early-morning gastric lavage may yield positive cultures. B. Streptomycin A. IFN- Release Assays A. Ziehl-Neelsen staining B.42 is a good indicator of TB pleurisy. Homocysteine uric acid level ? D. 1350 B. 1351 Either by direct chemical measurement or by using an isotopic tracer. mumps. 104 bacilli / ml 521 For the diagnosis of primary pulmonary tuberculosis in children. ADA 2 C.6 are characteristic of TBM. IFN- Requiring Assays 518 Adenosine deaminase (ADA) catalyzes the conversion of adenosine to ? D. sensitive. Kinyoun staining A. 1034 Enzyme ADAp or “total pleural ADA level” is typically found at high concentrations in pleural TB. and specific test for tuberculous meningitis. C. Rifampin pleurisy have predominantly ADA 2. Lowenstein .Jensen (LJ) C. Isoniazid 519 Patients with TB pleurisy have which type of ADA ? D. ADA 1 Hyperuricemia is a common adverse effect of pyrazinamide therapy. RT21 A. Three types of culture media are used to grow mycobacteria . Faeces C. Normal film C. Rifampin by lymphocytes and macrophages during the cellular immune response. B. Ethambutol mainly ADA1. Middlebrook 7H9). All of the above . All of the above B. 3 TU N Engl J Med 2002. Its toxicity include intense gastrointestinal D. Two or more A. At least three A. C.347:1420-5 531 Which of the following Anti HIV drugs should not be used with A. Para-Aminosalicylic Acid (PAS) D. 1036 B. Diabetes mellitus D.349:1149-56 B. At least five C.349:1149-56 B. 1992 C. Nelfinavir 539 Which of the following is true for Category 2 tuberculosis? C. All of the above 526 Which of the following antitubercular medication causes hypothyroidism ? 533 Conditions that confer predisposition to progression from latent Harrison’s 17th Ed. All of the above intolerance. Head or neck cancer Harrison’s 17th Ed. AJAY MATHUR Miscellaneous D. Resistance to rifampin Adverse effects of ethambutol includes hyperuricemia but is usually asymptomatic. Ethionamide A. All of the above C. treatment failed N Engl J Med 2003. serious neurologic reactions. Patient has extrapulmonary TB & is seriously ill A. tuberculosis by impairing folate synthesis. patient seriously ill N Engl J Med 2003. Chronic renal failure Ethionamide is most useful in treatment of MDR tuberculosis. Ethambutol 535 What quantity of intradermal injection of purified protein derivative is given in Mantoux test ? PAS inhibits the growth of M. Capreomycin C. a positive sputum smear is defined as D. Persons coinfected with HIV and M. N Engl J Med 2003. Nelfinavir D. Streptomycin C. Positive sputum smear rifabutin ? B. Negative sputum smear. Positive sputum smear. 1991 D. hypersensitivity reactions & hypothyroidism. Ritonavir 538 Which of the following is true for Category 1 tuberculosis ? D. Nevirapine N Engl Med 2002. Resistance to isoniazid D.347:1421 530 Which of the following Anti HIV drugs should not be used with rifabutin ? A. Streptomycin B. tuberculosis ? A. 1036 tubercular infection to active disease include ? A. 5 TU B. Resistance to at least isoniazid and rifampin D.868 MCQ’s FOR MEDICAL PROFESSIONALS BY PROF.347:1421 529 Which of the following Anti HIV drugs can be used with rifabutin ? A. patient relapsed 532 Multidrug-resistant tuberculosis is defined as ? B. At least four B. Indinavir D. Delavirdine N Engl Med 2002. Positive sputum smear. At least two N Engl J Med 2003.349:1149-56 C. Nelfinavir D. tuberculosis C.349:1149-56 528 Revised National Tuberculosis Control Program was begun in A. 4 TU A. 1994 N Engl J Med 2002. Indinavir C. Three or more B.347:1420-5 C. Organ transplantation A.349:1149-56 folate synthesis of M. patient treated after default A. 2 TU which year ? B. Isoniazid B.347:1420-5 D. reversible hepatitis.349:1149-56 C. 534 Conditions that confer predisposition to progression from latent tubercular infection to active disease include ? 527 Which of the following antitubercular drug acts by impairing N Engl J Med 2003. Four or more C. 1993 536 A negative sputum smear with pulmonary tuberculosis is defined as how many new sputum smears negative for AFB ? D. One or more N Engl J Med 2003. Injection-drug use B. Indinavir C. Rifampin N Engl J Med 2003. Positive sputum smear. Ethionamide D. Nevirapine 537 Without sputum culture. Nevirapine A.349:1149-56 B. All of the above how many new sputum smears positive for AFB ? N Engl J Med 2002. D. and linezolid. Two or more consecutive months C. tuberculosis culture how many months of anti-TB treatment ? PAS and fluoroquinolones. capreomycin. the concept of short-course chemotherapy started. 1354 A. isoniazid (1951). pyrazinamide and ethambutol. 5 accurately by ? Harrison’s 18th Ed. Pyrazinamide was first used in the 1950s.367:931-6 541 A negative sputum smear is defined as at least how many new A.2005). 1354 of patients with resistance to most of first. 1352 C. tuberculosis culture after three months of anti-TB treatment. cycloserine (1952). Rutgers University N Engl Med 2002. 15 days consecutively 549 Which of the following is not a first-line antitubercular agent ? Harrison’s 18th Ed. amikacin. patient not seriously ill C. Any of the above pulmonary tuberculosis is ? “Expire” cases were those who died during anti-TB treatment. discovered streptomycin. Waksman.347:1420-5 548 Which of the following anti-tubercular drug was developed after A. along with Albert Schatz (1920 . Second-line drugs are streptomycin. Pyrazinamide 544 “Failure” cases are those with persistent sputum (+) AFB &/or M. Harrison’s 18th Ed. First-line agents for treatment of tuberculosis are isoniazid. A. All of the above A. Rifampin “Default” cases were those with interruption of treatment for at least two consecutive months. grisein. Type of drugs used 545 “Expire” cases are those ? D. para- 543 A tuberculosis patient is called “Defaulter” if treatment has been aminosalicylic acid (1948). candicidin. Length of treatment C. Ethambutol “Relapse” cases were those who had bacteriologic evidence of recurrence of M. 30 days consecutively A. Other drugs of unproven efficacy that have been used in the treatment Harrison’s 18th Ed. Patient has extrapulmonary TB but is not seriously ill 547 Which of the following places is related to the firt documentation D. amoxicillin/ clavulanic acid. neomycin. 1352 D. Total number of doses taken “Failure” cases are those with persistent sputum (+) AFB &/or M. N Engl J Med 2002. In early 1970s. D. D. Interferon-gamma level B. Isoniazid D. Para-aminosalicylic acid (PAS) Bacteriologic evaluation is the preferred method of monitoring response to treatment for tuberculosis. C. ethionamide. Para-aminosalicylic acid (PAS) A. 1 B. Thiacetazone C. cycloserine. 1352 B. candidin. 2 B. streptomycin was used for treatment of tuberculosis.347:1420-5 C. 869 Miscellaneous MCQ’s FORMCQ’s FOR MEDICAL MEDICAL PROFESSIONALS PROFESSIONALS BY PROF. Streptomycin D. streptothricin. fradicin. ADA level A. tuberculosis relapse ? N Engl Med 2002. Cycloserine The years in which various anti-tubercular drugs were developed are thiacetazone (1948). For this discovery. pyrazinamide (1952). and ethambutol (1962). University of Utrecht C. None of the above C. Streptomycin C. Isoniazid . Who died after anti-TB treatment 551 The preferred method of monitoring response to treatment for D. Pyrazinamide N Engl Med 2002. 3 It was at Rutgers University that Selman Waksman (1888 .and second-line agents include clofazimine.347:1421 A. Patient previously treated for TB A. Bacteriologically positive TB rifampin ? N Engl J Med 2012. rifampin (1957). X-Ray chest 546 Which of the following anti-tubercular drugs was used first to B. 3 550 A full course of therapy (completion of treatment) is defined more C. B. rifampin.1973) discovered several antibiotics. B.347:1420-5 In mid-1940s. Yale University sputum smears negative for AFB ? B. AJAY MATHUR Cardiology 869 540 Which of the following is true for Category 3 tuberculosis? D. 542 Which of the following is false for M. Who died during anti-TB treatment doses taken than by the length of treatment. Columbia University A. Who died without receiving anti-TB treatment A full course of therapy (completion of treatment)is defined more accurately by the total number of B. including actinomycin. 45 days consecutively B. Pyrazinamide D. Patient declared cured or completed treatment B. kanamycin. clavacin. All of the above of streptomycin as anti-tubercular drug ? N Engl J Med 2012. Waksman received the Nobel Prize for Medicine in 1952. 6 A. tuberculosis infection during the follow-up period after completion of anti-TB treatment. Negative sputum smear and isoniazid were the next drugs. Abnormal radiograph. ethionamide interrupted for ? (1956). C.367:931-6 B. Bacteriologic evaluation treat tuberculosis ? Harrison’s 18th Ed. 4 and others. 2 D. with the discovery of Rifampin. 2 months A. 20 months D. if the patient develops gouty arthritis. Occurence of Immune reconstitution inflammatory syndrome (IRIS) . 553 With the recommended ATT regimen. 1354 A. 12 months B. Rifampin With the recommended regimen. Pyrazinamide should be stopped of the second month of treatment. 5 months C. Fourth month D. 557 Treatment failure should be suspected when a patient’s sputum 563 Which of the following statements about tuberculosis treatment smears remain positive beyond ? in HIV-infected patients is false ? Harrison’s 17th Ed. 1 months A.347:1420-5 Harrison’s 18th Ed. Ethambutol To prevent isoniazid-related neuropathy. 15 days A. treatment failure and drug Optimal treatment duration recommended when TB bacilli is resistant to all of the first-line agents resistance should be suspected. Pyrazinamide D. 1016 Harrison’s 18th Ed. 2 months B. Isoniazid B. First month A. Life long When a patient’s sputum cultures remain positive at >=3 months. 4 months B. virtually all patients will 559 Autoimmune thrombocytopenia is the adverse effect of which of have negative sputum cultures at the end of ? the following anti-tubercular drug ? Harrison’s 18th Ed. 1354 A. Rifampin C. 1352 tubercular drug ? Harrison’s 18th Ed. Ethambutol . First month A.if patient has autoimmune thrombocytopenia C.if patient develops gouty arthritis B.if patient has optic neuritis D. Individuals who develop autoimmune thrombocytopenia secondary to rifampin therapy should not receive the drug thereafter. Any of the above D. 15 months C. Pyrazinamide D. 3 months patients C. 1354 Harrison’s 18th Ed. Pantothenic acid D. 1355 A. Ethionamide C. Drug interactions between ART and Isoniazid AFB smears positive after 5 months are indicative of treatment failure. 1354 A. is 20 months with a combination of four second-line drugs. Third month C. Rifampin . Ascorbic acid B. 555 Patient is considered as ‘Defaulter’ if ATT treatment has been 561 Which of the following is an indication for permanent interrupted for what duration ? discontinuation of the drug ? N Engl Med 2002. One month B. Pyrazinamide D. AJAY MATHUR Miscellaneous 552 With the recommended ATT regimen. 1354 Harrison’s 18th Ed. almost all patients should be culture-negative with recommended ATT regimen. 3 months C.870 MCQ’s FOR MEDICAL PROFESSIONALS BY PROF. Rifampin C. 1354 A. >80% of patients will have negative sputum cultures at the end Hyperuricemia and arthralgia may be caused by pyrazinamide. Optic neuritis with ethambutol is an indication for permanent discontinuation of this drug. >80% of patients will have 558 Hyperuricemia is the adverse effect of which of the following anti- negative sputum cultures at the end of ? tubercular drug ? Harrison’s 18th Ed. Increased frequency of paradoxical reactions D. All of the above 556 Treatment failure & drug resistance should be suspected when a 562 Optimal treatment duration recommended when TB bacilli is patient’s sputum cultures remain positive at or beyond ? resistant to all of the first-line agents is ? Harrison’s 18th Ed. Standard treatment is equally efficacious in HIV (-) & HIV (+) B. 554 To prevent isoniazid-related neuropathy. Isoniazid B. Isoniazid B. pyridoxine (10 to 25 mg/day) should be added to the antitubercular regimen. Ethambutol By the end of 3rd month. 1354 A. Second month B. 4 months D. Two or more consecutive months C. including one injectable agent. 1354 Harrison’s 18th Ed. D. which of the following is helpful ? 560 Optic neuritis is the adverse effect of which of the following anti- Harrison’s 18th Ed. Thiamine A. Pyrazinamide . Third month C. Pyridoxine C. Fourth month D. Second month B. D. Previously sensitized CD4+ & CD8+ T lymphocytes for first 2 months. Common in patients with advanced immunosuppression 570 Which of the following anti-tubercular drug interacts with glucocorticoids ? B. 2 months refers to exacerbations in symptoms. with scar formation and syndrome (IRIS) is false ? healing within 3 months. Children with tuberculous meningitis 572 Tuberculin skin test (TST) was first introduced in ? B. D. 947 Table 149-2 C. 1344 C. capreomycin. Pyrazinamide A. Previously sensitized macrophages Delayed-type hypersensitivity reaction (DTH) to M. Drug resistance is expected of ? Harrison’s 18th Ed. Monotherapy Exacerbations in symptoms. 1942 tuberculous meningitis. Children with bone or joint tuberculosis A. and development of rifampin monoresistance. tuberculin skin test (TST) to diagnose latent tuberculosis is the oldest pregnancy ? diagnostic test in use. Following antimycobacterial agents have no or minimal effects on other drugs: amikacin. M. Cases of active tuberculosis have strongly positive TST C. A. 4 months 564 Which of the following about immune reconstitution inflammatory The local tissue response following BCG vaccination begins 2 to 3 weeks. Host resistance Development of drug-resistant TB is invariably the result of monotherapy (failure to prescribe at least 566 9 to 12 month antitubercular regimen is still recommended for ? two drugs to which tubercle bacilli are susceptible or of the patient to take properly prescribed Harrison’s 18th Ed. Streptomycin B. Pyrazinamide IRIS is more common among patients with advanced immunosuppression and extrapulmonary tuberculosis. It is due to an immune response elicited by antigens released from killed bacilli are D. and laboratory or radiographic manifestations of tuberculosis termed paradoxical reactions have been associated with the administration of HAART regimens. Cellular mechanisms responsible for TST reactivity are related mainly to previously Harrison’s 18th Ed. 1 month of paradoxical reactions. Rifampin Harrison’s 18th Ed. Any of the above C. signs. 1956 567 Which of the following anti-tubercular drug is contraindicated in First introduced in 1890. AJAY MATHUR Cardiology 871 Standard ATT treatment is efficacious in HIV (+) & HIV (-) patients. M. 1890 D. Due to immune response elicited by antigens released from A. Harrison’s 18th Ed. 565 “Paradoxical reaction” to ATT occur when ? Harrison’s 18th Ed. signs and laboratory or radiographic manifestations of C. 1915 American Academy of Pediatrics recommends that children with bone and joint tuberculosis. azithromycin. bovis & was first administered to humans in 1921. A. D. Streptomycin is contraindicated in pregnancy because it causes eighth-cranial-nerve damage in the fetus. Isoniazid killed bacilli during effective chemotherapy B. HAART regimen is administered A. There is increased frequency A. Previously sensitized CD4+ T lymphocytes D. 1356 A. Genetic factors C. 871 Miscellaneous MCQ’s FORMCQ’s FOR MEDICAL MEDICAL PROFESSIONALS PROFESSIONALS BY PROF. C. D. streptomycin. 3 months tuberculosis associated with administration of ART. Virulence of tubercular bacteria D. Ethambutol during effective chemotherapy. bovis 574 Which of the following statements is false ? B. 1355 571 Development of drug-resistant tuberculosis is invariably the result A. ethambutol. TST-positive persons are less susceptible to a new M. None of the above . drug interactions between antiretroviral therapy and rifamycins. Retreatment of defaulters is begun B. intercellulare A. 1355 A. Isoniazid 573 Cellular mechanism responsible for tuberculin skin test (TST) reactivity is ? B.4:761-76 C. tuberculosis is the basis of tuberculin skin test 568 BCG was derived from an attenuated strain of ? (TST). which are attracted to the skin-test site. tuberculosis Harrison’s 18th Ed. Common in patients with extrapulmonary tuberculosis Harrison’s 16th Ed. WHO recommends routine use of pyrazinamide in pregnant women. B. pyrazinamide. All of the above B. Children with miliary tuberculosis Lancet Infect Dis 2004. 1356 D. Latent or active tuberculosis may not confer fully protective 569 Scar formation & healing following BCG vaccination occurs immunity within ? Harrison’s 18th Ed. 1356 therapy). or miliary tuberculosis receive 9-12 months of treatment. 1354 B. None of the above C. 1344 C. Any of the above tuberculosis infection BCG was derived from an attenuated strain of M. Previously sensitized CD8+ T lymphocytes Pregnant women should receive ATT for 9 months with isoniazid & rifampin supplemented by ethambutol C. immune reconstitution inflammatory syndrome (IRIS) B. D. Harrison’s 18th Ed. M. Rifampin D. 1356 sensitized CD4+ T lymphocytes. Amikacin B. Ribs In Mantoux test. D. distinguishing between NTM & M tuberculosis infection. Sternum Tuberculosis and brucellosis affect the spine more often than other bones. streptomycin. All of the above Antimycobacterials that have no / minimal effects on other drugs are amikacin. AJAY MATHUR Miscellaneous 575 In Mantoux test. All of the above B. diagnosis of active C. D. a cutoff positive PPD skin test is ? Diskitis is early in tuberculosis while it is late in brucellosis. 10 mm A. Lancet Infect Dis 2004. 1357 compression and psoas abscess are rare. in brucellosis. Any of the above C. C. and previously untreated persons whose chest C. Streptomycin tuberculosis. 20 mm C. All of the above B. Rifampin D. the spinal canal Harrison’s 18th Ed. Diskitis is late C. 5 mm Harrison’s 18th Ed. Brucellosis For persons with a very low risk of developing tuberculosis if infected. infected. 10 mm B. Ethambutol Positive reactions for close contacts of infectious cases. Streptomycin is drug of choice in initial therapy for tularemia. Ulceration B. Plague D. ethambutol & pyrazinamide. 5 mm 584 Which of the following antimycobacterial agents have no or B. Rifampin D. Thoracic spine is 576 For persons with a very low risk of developing tuberculosis if involved most commonly in tuberculous spondylitis (Pott’s disease). Pyrazinamide 580 Interferon-gamma assays is useful for ? Four first-line essential antituberculous agents are rifampin. a cutoff positive PPD skin test is ? Harrison’s 18th Ed. isoniazid. 1032 D. 10 mm minimal effects on other drugs ? C. Induration 581 Tuberculosis affects which bone more frequently ? Harrison’s 18th Ed. Pyrazinamide radiograph is consistent with healed tuberculosis are defined as an area of induration >=5 mm in diameter. Metaphyses of long bones D. 15 mm Harrison’s 17th Ed. reactions are read at 48 to 72 hours as the transverse diameter in millimeters of “induration”. 1373-75 C. Lancet Infect Dis 2004. Distinguishing between NTM & M tuberculosis infection A.4:761-76 A. All of the above considered as a positive reaction. Site mostly affected is dorsolumbar spine B. 15 mm C. Streptomycin A high level of interferon-gamma production is presumed to be indicative of tuberculosis infection. Isoniazid Interferon-gamma assays is useful for diagnosis of latent tuberculosis. Diagnosis of active tuberculosis Harrison’s 18th Ed. 579 Interferon-gamma is produced by ? capreomycin. 1299 B. Harrison’s 18th Ed. 5 mm A. 1357 Harrison’s 18th Ed. a cutoff 583 Which of the following is false about tubercular vertebral positive PPD skin test is ? osteomyelitis ? Harrison’s 18th Ed. 15 mm B. 1347 B. persons with HIV infection. 1376 C. ethambutol. Also. azithromycin. 20 mm A. A.872 MCQ’s FOR MEDICAL PROFESSIONALS BY PROF. Spinal canal compression common D. 577 For close contacts of infectious tuberculosis cases. 1298 Table 157-1 A.4:761-76 A. Diagnosis of latent tuberculosis 586 Which of the following is a “first-line supplemental anti- tuberculous agent” ? B. prediction of reactivation disease among those with latent the following is measured ? tuberculosis. 20 mm D. transverse diameter in millimeters of which of tuberculosis infection & previous BCG vaccination. The diameter of erythema is not considered. D. a cutoff of 15 mm is D. glanders & brucellosis. and monitoring treatment response. Macrophages 585 Which of the following is not a “first-line essential anti-tuberculous B. Psoas abscess likely 578 In HIV infection. 1357 582 Streptomycin is the drug of choice in initial therapy for ? A. Isoniazid Interferon-gamma assays are based on the principle that T cells of individuals sensitised with tuberculosis antigens produce interferon gamma when they reencounter mycobacterial antigens. T cells agent” ? Harrison’s 18th Ed. Erythema A. & pyrazinamide. Tularemia C. Spine C. plague. persons receiving drugs that suppress the immune system. serving as a correlate of protective immunity & for assessment of vaccine efficacy. 1357 A. differentiating between M . Dendritic cells A. D. and immunosuppression of infrequent. Streptomyces victorii A. 1374 bacilli & bactericidal against rapidly multiplying organisms. Saliva C. 1374 Harrison’s 18th Ed. It causes hepatocellular injury when given in combination with isoniazid or pyrazinamide. and HIV infection as well as in women who are Rifampin is a fat-soluble and has intracellular & extracellular bactericidal activity. 1374 597 Which of the following is false about rifampin ? Harrison’s 18th Ed. 591 Isoniazid-associated hepatitis increases in ? Harrison’s 18th Ed. All of the above D. Blocks RNA synthesis C. 1374 A. 590 Isoniazid-associated hepatitis increases in ? 596 Which of the following is best related to rifampin ? Harrison’s 18th Ed. kansasii. Rifampin C. and streptomycin. sputum and tears to a red-orange color. 1374 B. Daily alcohol consumption B. 1377 A. HIV infection A. Seven times thrombocytopenia. Isoniazid D. Ethionamide C. Rifampin turns urine. Pyrazinamide 595 Which of the following body fluids is turned red-orange in color 589 Isoniazid is bactericidal against ? by rifampin ? Harrison’s 18th Ed. Injured bacilli C. All of the above Rifampin is a fat-soluble complex macrocyclic antibiotic. Amikacin Rifampin is a semisynthetic derivative of Amycolatopsis rifamycinica formerly known as Second-line antituberculous drugs are para-aminosalicylic acid (PAS). 1374 A. M. 1375 of normal ? A. 1374 A. Streptomyces mediterranei C. extracellularly & intracellularly ? Harrison’s 18th Ed. D. Urine B. 873 Miscellaneous MCQ’s FORMCQ’s FOR MEDICAL MEDICAL PROFESSIONALS PROFESSIONALS BY PROF. Legionella spp. Rapidly multiplying bacilli B. Streptomyces perii D. Streptomyces mediterranei. Two times C. Skin rash Harrison’s 18th Ed. Streptomyces griseus B. rifapentine. Age A. concomitant rifampin administration. Five times Most common adverse effect of rifampin is gastrointestinal upset. All of the above D. capreomycin. . daily alcohol consumption. Macrocyclic antibiotic C. Skin rash. All of the above Isoniazid is bacteriostatic against resting bacilli and bactericidal against rapidly multiplying organisms. both extracellularly and intracellularly. synthesis by specifically binding & inhibiting DNA-dependent RNA polymerase. marinum. and fluoroquinolones. Sputum D. 592 Isoniazid should be discontinued when there is asymptomatic 598 Most common adverse effect of rifampin is ? elevation of ALT level that exceed how many times the upper limit Harrison’s 18th Ed. Inhibits DNA-dependent RNA polymerase D. cycloserine. Thrombocytopenia C. Gastrointestinal upset B. Concomitant rifampin administration D. Resting bacilli A. both A. kansasii A. amikacin. It blocks RNA pregnant or in postpartum period. 1373 Harrison’s 18th Ed. 1374 Harrison’s 18th Ed.. Three times D. Pregnant women B. M. Legionella spp. M. and M. Levofloxacin B. Ethambutol Rifampin is also active against some gram-positive & gram-negative bacteria. 1373 B. Sulphonylurea C. 587 Which of the following is not a “second-line anti-tuberculous 593 Rifampin is a semisynthetic derivative of ? drug” ? Harrison’s 18th Ed. All of the above C. kanamycin. upper limit of normal along with hepatitis symptoms or jaundice. Pyrazinamide CDC & ATS recommend that isoniazid should be discontinued when there occurs an asymptomatic elevation of ALT level that exceeds five times upper limit of normal or when ALT is three times the First-line “supplemental” agents include rifabutin. Ethambutol D. Water-soluble Isoniazid-associated hepatitis is idiosyncratic and increases in incidence with age. hemolytic anemia. ethionamide. marinum B. Trimethoprim D. AJAY MATHUR Cardiology 873 D. Intracellular & extracellular bactericidal activity B. Hepatitis A. saliva. 594 Rifampin is also active against ? 588 Which of the following drugs is bacteriostatic against resting Harrison’s 18th Ed. Aminosalicylic acid B. Postpartum period C. Rifabutin 602 Retrobulbar optic neuritis as a complication of ethambutol therapy Yellow skin discoloration “pseudojaundice” may be rifabutin’s adverse effect. HIV nonnucleoside reverse transcriptase inhibitors. clarithromycin. central scotoma. Streptomycin ineffective in the acidic environment within the macrophage. 1375 A. B. . kansasii. Vertigo warfarin. Harrison’s 18th Ed. 1375 A. 24 hours D. DNA-dependent RNA polymerase & fluconazole block hepatic metabolism of rifabutin with consequent increases in its serum levels. Pyrazinamide Harrison’s 18th Ed. Streptomycin A. Digoxin loss & vestibular dysfunction). Red A. 1375 Adverse effects of Streptomycin include renal toxicity (nonoliguric renal failure). Loss of ability to see green D. Central scotoma C. Ethambutol Retrobulbar optic neuritis due to ethambutol therapy results in reduced visual acuity. Isoniazid Ethambutol is a water-soluble compound (derivative of ethylenediamine). Pyrazinamide C. Ethambutol only against mycobacteria like M. Yellow D. M. 1376 A. cyclosporine. C. B. tuberculosis ? acuity. M. Clarithromycin (not azithromycin) A. Rifampin B. 62 hours D. results in loss of vision for which colour ? Harrison’s 18th Ed. Optic neuritis with associated visual loss is reversible. All of the above 610 Which of the following antitubercular drug acts only against Retrobulbar optic neuritis involves papillomacular bundle of fibers & results in reduced visual extracellular M. 1376 Harrison’s 18th Ed. Pyrazinamide is bactericidal to slowly metabolizing organisms located within the acidic environment but recovery may take >6 months. All of the above B. methadone. Ethambutol A. oral contraceptives. 1375 608 Which of the following antitubercular drug is not bacteriostatic ? Harrison’s 18th Ed. Reduced visual acuity B. Amikacin B. Hearing loss Rifampin is a potent inducer of hepatic microsomal enzymes and decreases half-life of digoxin. Primary mechanism of action is inhibition of an arabinosyltransferase that mediates the C. 45 hours Among first-line drugs. and MAC organisms. AJAY MATHUR Miscellaneous 599 Rifampin decreases half-life of which of the following drugs ? D. D. ethambutol is the least potent against M. rash and drug fever. Pyrazinamide D. Isoniazid 606 Mean serum half-life of Rifabutin is ? B. Harrison’s 18th Ed. Tinnitus D. Streptomycin A. All of the above 600 Among the first-line drugs. Prednisone 605 Ototoxicity in patients receiving streptomycin include ? C. and loss of ability to see green. prednisone.874 MCQ’s FOR MEDICAL PROFESSIONALS BY PROF. Pyrazinamide B. vertigo. 1375 45 hours. and tinnitus. and loss of ability to see green. 78 hours 601 Primary mechanism of action of ethambutol relates best to ? Slow clearance of rifabutin via hepatic metabolism & renal excretion results in mean serum half-life of Harrison’s 18th Ed. Blue B. 1035 C. B. which one is the least potent against M. tuberculosis. marinum. eosinophilia. Ethionamide C. Isoniazid Harrison’s 17th Ed. central scotoma. All of the above D. Rifampin Streptomycin and Amikacin are bactericidal for rapidly dividing extracellular mycobacteria but is C. 603 Retrobulbar optic neuritis as a complication of ethambutol therapy 609 Which of the following antitubercular drug is bactericidal ? can lead to ? Harrison’s 18th Ed. ototoxicity (hearing A. C. tuberculosis ? Ototoxicity in patients receiving streptomycin include both hearing loss and vestibular dysfunction like loss of balance. that is bacteriostatic B. Arabinosyltransferase Harrison’s 18th Ed. Pyrazinamide polymerization of arabinose into arabinogalactan within mycobacterial cell wall. 1376 C. tuberculosis. All of the above A.to 5-hour half-life of rifampin. which is much longer than 3. HIV protease inhibitors. Catalase-peroxidase reaction 607 “Pseudojaundice” is an adverse effect of ? C. Isoniazid D. HIV protease inhibitors Harrison’s 18th Ed. Ribosome A. 1376 D. of phagocyte or caseous granuloma. 1376 604 Perioral paresthesia and eosinophilia are adverse effects of which A. and quinidine. 1376 D. perioral paresthesia. Green C. Rifampin of the following drugs ? B. Rifampin Harrison’s 18th Ed. RNA polymerase (rpoB) gene. 1032. 621 “DOTS” stands for ? Three selected codons (rpoB531. 1375 of risk when used in pregnancy ? A. and katG315) allowes identification of 90% of MDR- N Engl Med 2002. 1376 in dosage in patients with hepatic disease ? A. Political commitment Pyrazinamide is not available in a parenteral formulation. pncA gene encodes pyrazinamidase. B. ethambutol resistance) & pncA (pyrazinamide resistance). rpoB526. Rifabutin D. Directly observed treatment . DOTS does not include specific therapy for patients with drug-resistant tuberculosis. rpoB D. rpoV B. uninterrupted supply of all essential drugs. 1034-5 C. B. rpoB (RIF resistance). erp D. All strains of M. katG B. Rifapentine Nine genes are known to be linked to resistance to first-line drugs: katG. pncA C. Pyrazinamide A. Pyrazinoic acid . A. aphC. Directly observed treatment . Kidney B. Directly observed treatment . Table 161-1 B. Liver A. rpoV A. rpoV A. Rifampin D. Ethionamide B. case detection by sputum microscopy. Isoniazid 622 Which of the following is not an element of WHO DOTS strategy C. embB B. 1032. erp Pyrazinamide is a prodrug & is converted by tubercle bacillus mycobacterial pyrimidase to the Resistance to rifampin results from spontaneous point mutations that alter the beta subunit of the active form pyrazinoic acid.sure B. hydrazide of isonicotinic acid. 1375 D. katG Harrison’s 17th Ed.safe A. 1376 Harrison’s 18th Ed.365:1239-45 D. All of the above (INH resistance). Pyrazoic acid DOTS consists of five main elements: political commitment. Pyranoic acid and overall programme performance. Specific therapy for patients with drug-resistant TB Harrison’s 18th Ed. 614 Resistance to ethambutol is related to which gene ? 620 Which of the following antimycobacterial agents has no evidence Harrison’s 18th Ed. pncA C. All of the above Resistance to pyrazinamide is due to loss of pyrazinamidase activity preventing conversion of pyrazinamide to pyrazinoic acid. Rifampin C. D. Rifampin C. Isoniazid C. 1034 A. Rifabutin D. Streptomycin ? Lancet 2005. Pyrazine acid directly observed therapy of a standard short-course regimen. Pyrazinamide D. Case detection by sputum microscopy 616 Active form of Pyrazinamide is ? C. Isoniazid is the or the promoter of a two-gene locus known as inhA. Standard recording and reporting system A. and a standard recording and reporting system that allows assessment of treatment results C. 1375 A. rpoB B. All of the above Most isoniazid-resistant strains have amino acid changes in the catalase-peroxidase gene (katG) Like isoniazid and pyrazinamide. Directly observed treatment . AJAY MATHUR Cardiology 875 611 Resistance to rifampin results from spontaneous point mutations 617 Pyrazinamide is converted to its active form by ? related to which of the following genes ? Harrison’s 18th Ed.short course for parenteral use ? Harrison’s 17th Ed. katG A.347:1420-5 TB cases. rpoV C. 613 Resistance to pyrazinamide is related to which gene ? 619 Which of the following antimycobacterial agents need no change Harrison’s 18th Ed. Skin C.supervised 615 Which of the following anti-tuberculous agents is not available B. Rifapentine D. inhA. Table 161-1 B. rpoB C. 618 Which of the following anti-tuberculous agent is a derivative of 612 Resistance to isoniazid is related to which gene ? isonicotinic acid ? Harrison’s 18th Ed. ethionamide is a derivative of isonicotinic acid. Tubercle bacillus D. hepatic or renal function. and kasA D. embB (that encodes for arabinosyltransferase. katG Harrison’s 17th Ed. 1374 Harrison’s 17th Ed. 875 Miscellaneous MCQ’s FORMCQ’s FOR MEDICAL MEDICAL PROFESSIONALS PROFESSIONALS BY PROF. bovis are Adjustment of dosage of Rifabutin is not needed in elderly patients and in patients with reduced naturally resistant to pyrazinamide and have a point substitution within the pncA gene. rpsL and rrs (streptomycin STR resistance). 337:a1110 B.359:563-74.337:a1110 A. Tuberculosis Tuberculosis that is resistant to any first line antituberculosis drug is called drug resistant tuberculosis. 2846 BMJ 2008. brucellosis. rifampicin. 2023 B. a phenomenon known as primary resistance. All of the above D. or kanamycin). transmission of drug-resistant strains to uninfected individuals (transmitted resistance). fluoroquinolone & a second line injectable agent Poncet’s disease is a reactive symmetric form of polyarthritis that affects persons with visceral or disseminated tuberculosis. Resistance to at least INH & rifampicin C. cycloserine. Resistance to at least INH. Prothionamide C. A. < 8 % D.876 MCQ’s FOR MEDICAL PROFESSIONALS BY PROF. 627 Poncet’s disease refers to ? 633 Extensively drug resistant tuberculosis (XDR-TB) refers to ? Harrison’s 18th Ed. Anterior uveitis in disseminated tuberculosis B. but interacting processes . tuberculosis ? 634 Which of the following is not an antitubercular drug ? A. BMJ 2008. 2023 A. 631 Drug resistant tuberculosis refers to ? interstitial nephritis. 1346 to new patients leading to drug resistance at the onset is called ? A. Any of the above A patient with acquired drug resistance may transmit resistant strains to another individual. capreomycin. lymphoma. anatomic abnormality. Resistance to any first line Anti TB drug A. Brucellosis C. Resistance to at least INH. Resistance to at least INH. D. Resistance to at least INH. rifampicin. Mycobacterium tuberculosis or fungi. and symptoms resolve with Extensively drug resistant tuberculosis (XDR-TB) refers to tuberculosis that is resistant to at least isoniazid antituberculous therapy. tuberculosis Harrison’s 18th Ed. Terizidone D. nephrocalcinosis. and thioacetazone. Tuberculosis that is resistant to at least isoniazid and rifampicin is called Multidrug resistant coccidioidomycosis. U. All of the above second line injectable agent Bilateral hilar adenopathy is observed in sarcoidosis. < 5 % C. AJAY MATHUR Miscellaneous 623 Tubercular markers in pleural fluid include ? 629 Phenomenon of transmission of drug resistant M. 1718 630 The global prevalence in new patients of MDR-TB is ? A. Meningitis in disseminated tuberculosis D. Resistance to at least INH. and rifampicin and also to a fluoroquinolone and a second line injectable agent (amikacin. trachomatis. All of the above C. All of the above The global prevalence in new patients of MDR-TB is still below 3%. Adenosine deaminase > 45 IU/L A. Posterior uveitis in disseminated tuberculosis C. Transmitted resistance A. Amplified resistance B. Adhered resistance D. In-situ resistance B. urealyticum B. < 3 % C. rifampicin & fluoroquinolone D. Lymphoma second line injectable agent C. fluoroquinolone & a D. N Engl J Med 2008. rifampicin. rifampicin & fluoroquinolone C. Coccidioidomycosis B. trachomatis A. Mycobacterium tuberculosis D. Interferon  > 140 pg/mL B. strains of resistance to more drugs during treatment (amplified resistance). Resistance to at least INH & rifampicin A. tuberculosis. para-aminosalicylic acid. tuberculosis (MDR-TB). Pyuria in the absence of bacteriuria (sterile pyuria) may indicate infection with C. No mycobacteria are found in the joints. and bronchogenic carcinoma. C. fluoroquinolone & a B. . Bronchogenic carcinoma D. All of the above 632 Multidrug resistant tuberculosis (MDR-TB) refers to ? 626 Bilateral hilar adenopathy on chest x-ray indicates ? BMJ 2008.337:a1110 Harrison’s 16th Ed. Resistance to at least INH & rifampicin B. infection with ? Harrison’s 16th Ed. Thioacetazone of wild-type to drug-resistant cases (acquired resistance). mL and positive polymerase chain reaction (PCR) for TB DNA.Conversion D. vesicoureteral reflux. Polyarthritis in disseminated tuberculosis A. rifampicin & fluoroquinolone D. tuberculosis isolates that 628 Which of the following drug resistance is observed in M. Acquired resistance BMJ 2008. rifampin. Positive polymerase chain reaction (PCR) for TB DNA C. Sterile pyuria may be found in noninfectious urologic conditions such as calculi. interferon gamma > 140 pg/ Subsequent transmission leads to tuberculosis in new patients that is drug resistant at the onset. and progressive acquisition by drug-resistant Second line oral antitubercular drugs are ethionamide. Sarcoidosis C. prothionamide. Resistance to at least INH. terizidone. and members of three of the six classes of secondline drugs. Resistance to any first line Anti TB drug B. Cyclozidone Drug resistance epidemics are caused by three independent. OR extensively drug-resistant tuberculosis was defined as M. are resistant to isoniazid. urealyticum.337:a1110. or polycystic disease. Resistance to any first line Anti TB drug 625 Bilateral hilar adenopathy on chest x-ray indicates ? Harrison’s 16th Ed. TB markers in pleural fluid include adenosine deaminase > 45 IU/L. U. Primary resistance C. < 1 % B. Primary resistance includes natural resistance (not associated with contact to drug) and the resistance occurring as a result of exposure to a drug- 624 Pyuria in the absence of bacteriuria (sterile pyuria) indicates resistant strain from another patient. He won the Nobel Prize (Physiology or Medicine) in 1902. Laveran A single sporozoite eventually may produce 10. D. Hepatic parenchymal cell . Morton (1696) presented the first detailed description of the clinical picture 635 Anopheles in Greek means ? of malaria & of its treatment with cinchona. P. without + ophelos. Konradsen Harrison’s 18th Ed. 3 . falciparum A. Utzinger A. Dense forest B. Dangerous from cinchona bark & Pelletier & Caventou (1820) in France extracted pure quinine alkaloids. 1688 by Anopheles mosquitoes and proposed an exerythrocytic stage in the life cycle. or hypnozoites. Liver parenchymal cells D. ovale RBCs & multiply 6. Bad air A. some of the intrahepatic forms do not divide immediately but 641 Who first employed the tincture of the cinchona bark for treating remain dormant from 3 weeks to a year or longer before reproduction begins. 1688 In 1898.000 B. 1689 C. vivax and P. showed that human malaria could only be transmitted Harrison’s 18th Ed. falciparum D.to 20 fold every ? Italian zoology professor. Covell and Shute (1948). Garnham. P. 100 to 1000 B. Giovanni Grassi D. P. Useless 642 Inoculation of plasmodial sporozoites by female anopheline mosquito and asexual reproduction in hepatic parenchymal cells D. 640 Name malaria is derived from Italian “mala . C. Garnham liver cells multiply 6. 1688 B.aria” meaning bad air (poisonous vapours of swamps). 1688 A. Thompson (1650) introduced this “Jesuits’ bark” to England: Its first recorded use was by Dr John Metford of Northampton (1656). Preerythrocytic schizogony A. ~ 150 / µL The four species of Plasmodium that attack humans are P. ~ 100 / µL D. 12 . P. These dormant forms. malariae 645 Symptomatic stage of the malarial infection begins when parasite 639 Which of the following causes benign quartan malaria ? density is about ? Harrison’s 18th Ed. Gize (1816) studied extraction of crystalline quinine A. Laveran numbering ? Harrison’s 18th Ed. Black C. Huan del Vego Chapter 210. Giovanni Grassi D. ovale infections. A. ~ 10 / µL B. which they named quinine and cinchonine. Gilroy 647 ‘Ookinete’ stage in life cycle of Plasmodium is found in ? B.000 to >30.24 hours 638 Which of the following causes malignant tertian malaria ? C. B. Any of the above Name malaria comes from Italian “mala .to 20 fold every 48-72 hours. P. Windy C. vivax Swollen infected liver cell bursts to discharge motile merozoites into bloodstream. These invade C. Huan del Vego first employed the tincture of cinchona bark for treating malaria. vivax & P. 3000 to 8000 A.aria” meaning ? 646 Hypnozoites of P. P. AJAY MATHUR Cardiology 877 D. advantage. Harrison’s 18th Ed.6 hours by Short. Ronald Ross 644 Merozoites liberated into bloodstream due to bursting of infected D. use). P. are the cause of the relapses. Intrahepatic schizogony 636 Who demonstrated the existence of Plasmodium in the stomach of the Anopheles mosquito ? B. later confirmed A. who found Plasmodium vivax in human liver.000 to >30. Anopheles mosquito C. ~ 50 / µL C.000 daughter merozoites. Any of the above B. Giovanni Grassi. 48 . Malaria In 1640. When parasites reach densities of ~50/µL of blood. malaria ? A. In P. ovale are found in ? A. 877 Miscellaneous MCQ’s FORMCQ’s FOR MEDICAL MEDICAL PROFESSIONALS PROFESSIONALS BY PROF.72 hours A. ovale C. P. 1688 A. Stagnation Harrison’s 18th Ed. P. RBC D. ovale (benign tertian malaria). 1000 to 3000 637 Who showed that human malaria could only be transmitted by Anopheles mosquitoes ? C. vivax B. & P. Suffering is called ? Word Anopheles is derived from Greek & it means useless (an-. Preerythrocytic merogony B. falciparum (malignant tertian malaria). Ronald Ross 643 A single sporozoite eventually may produce daughter merozoites D. vivax and P. B. Ronald Ross demonstrated the existence of Plasmodium in the stomach of the Anopheles mosquito. symptomatic stage of the infection begins. 10. malariae (benign quartan malaria). Garnham C. malariae D. All of the above C. Merozoite A.800 Constant. All of the above 655 Duffy blood-group antigen Fya or Fyb is related to ? 649 Intraerythrocytic life cycle is of about 48-hour duration for all Harrison’s 18th Ed. P. P. Oocyst expands by asexual division until it bursts to liberate motile sporozoites which migrate in hemolymph to salivary gland of mosquito to await inoculation into another human at the next feeding. Sporozoite Harrison’s 18th Ed. ovale is relatively unusual outside ? 650 Malaria in human beings is caused by ? Harrison’s 18th Ed. Direct effects of RBC invasion B. Trophozoites . AJAY MATHUR Miscellaneous B. vivax D. vivax malaria. New Guinea. 1000 . ovale is relatively unusual outside Africa. This is termed unstable transmission. mesoendemic (11-50%). 1688 A. 1691 652 Infected RBC’s ruptures to release how many daughter merozoites A.75 % B. 6 . merozoites rapidly invade erythrocytes and become trophozoites. 1688 Harrison’s 18th Ed. Global transmission A. and D. frequent. Most West Africans have Duffy-negative FyFy phenotype & are resistant to P. erratic.30 C. 1688 B. ovale D. P. vivax C. 100 . By the end of 48-hour intraerythrocytic life cycle (72 hour for P. Destruction by the asexual parasite C. Perpetual transmission C. falciparum C. P. merozoites invade erythrocytes to A. 51 . Mosquito’s salivary gland D. which of the following is an invasive form ? 648 Intraerythrocytic life cycle is of about 48-hour duration for ? A.878 MCQ’s FOR MEDICAL PROFESSIONALS BY PROF.3000 symptomatic disease may occur at all ages. P. 657 Hypoendemicity means parasitemia rates of ? 651 Schizont appears at which of the following time in the life cycle of Harrison’s 18th Ed. malariae D. Following schizogony RBC’s ruptures to release 6 . All of the above C. which penetrates and encysts in the mosquito’s gut wall. D. malariae. Mosquito’s midgut C. falciparum predominates in Africa. 1688 C. P. and ookinete.300 D. End of intrahepatic life cycle Endemicity is defined in terms of parasitemia rates or palpable-spleen rates in children aged 2 . vivax B. Perennial transmission ? Harrison’s 18th Ed. 659 Transmission of malaria is directly proportional to ? Harrison’s 18th Ed. and Haiti. Beginning of intraerythrocytic life cycle D. falciparum A. vivax merozoite is mediated via Duffy blood-group antigen Fy a or Fyb (RBC surface receptor). frequent. P. Schizonts C. year-round malarial infection is called ? Harrison’s 18th Ed. 1688 B. In hypoendemic areas. into an ookinete. 11 . parasite has consumed nearly all hemoglobin & grown to occupy most of the RBC. Ookinete B. Stable transmission B. Beginning of intrahepatic life cycle C. 658 Constant. It is then called a schizont. P.9 years D. P. All of the above P. P. Sporozoites D. Indian subcontinent C. hyperendemic (51-75%) & holoendemic (>75%). 654 In Plasmodium life cycle. each potentially capable of invading a new RBC and repeating the cycle.50 % A. Oceania D. Density of vector become ? B. malariae is found in sub-Saharan Africa. P. (Number of human bites per day per mosquito)2 Harrison’s 18th Ed. P. 1690 the malarial parasite ? A. P. > 75 % C. (Probability of mosquito’s surviving for 1 day)10 A. Africa A. merozoite. 1688 A. Any of the above Male & female gametocytes form a zygote in female anopheline mosquito midgut which matures After entry into the bloodstream. < 10 % Harrison’s 18th Ed. asexual parasite and the host’s reaction.30 daughter merozoites. P. vivax is more common in Central Malaria in human beings is caused by the direct effects of RBC invasion and destruction by the America. 1691 653 After entry into bloodstream. falciparum B. malariae Attachment of P. Central America B. End of intraerythrocytic life cycle as hypoendemic (<10%). 300 . ovale Three distinct invasive forms in Plasmodium life cycle are sporozoite. year-round malarial infection is termed stable transmission. 1688 except ? Harrison’s 18th Ed. or focal. where transmission is low. 656 P. Red blood cells (RBCs) B. full protective immunity is not acquired. ovale C. Intraerythrocytic life cycle is of 72-hour duration for P. 1688 B. malariae). Host’s reaction D. falciparum infection include intercellular adhesion molecule 1 (ICAM-1) in brain. Agglutination C. falciparum infected RBCs adhering to other Harrison’s 18th Ed. Chondroitin sulfate B Harrison’s 18th Ed. Cytoadherence Harrison’s 18th Ed. and agglutination are central to the pathogenesis of D. the mosquito must survive for >7 days. falciparum infected RBCs adhere to uninfected RBCs to form rosettes. > 5 days In P. 12 .15 hours Harrison’s 18th Ed. host defense mechanism like splenic processing and filtration. Erythrocyte-binding protein 165 (EBA-165) Harrison’s 18th Ed. 1691 C. Rosette In P. 8 . PfEMP4 C. Erythrocyte-binding protein 175 (EBA-175) A. 5 . which of the following microbial ligand interacts with host receptor Glycophorin A ? 661 Malarial parasite’s life cycle that takes place within the mosquito Harrison’s 17th Ed. Hemophyte B. Intercellular adhesion molecule 1 (ICAM-1) 662 In Africa. falciparum D. > 7 days To transmit malaria. 665 Cytoadherence in P. AJAY MATHUR Cardiology 879 D.10 hours 660 To transmit malaria. Agglutination C. 1691 B. 879 Miscellaneous MCQ’s FORMCQ’s FOR MEDICAL MEDICAL PROFESSIONALS PROFESSIONALS BY PROF. falciparum infections.10 days D. 24 . falciparum infected RBCs adhering to uninfected Harrison’s 18th Ed.cytoadherence. Process of P. PfEMP1 B. Anopheles gambiae Vascular receptors that mediate cytoadherence in P. 1691 subsequent inoculation (sporogony) lasts 8 . Erythrocyte-binding protein 195 (EBA-195) C. Number of sporozoite (+) mosquito bites/year C. D. Potentially toxic heme is detoxified by polymerization D. All of the above C. strain-specific erythrocyte membrane adhesive protein (PfEMP1) that mediates attachment to receptors on venular and capillary endothelium .48 hours C. 1691 C. A. 36 . membrane protuberances or “knobs” appear on RBC’s surface 12 . 1691 parasitized RBCs is called ? A. Hemosiderin A.12 days 668 Vascular receptor that mediate cytoadherence in P. 1691 RBCs is called ? A. P. CD36 A. . falciparum malaria ? Harrison’s 18th Ed. All of the above to biologically inert hemozoin or the malaria pigment. the mosquito must survive for ? B. 1691 B.30 days. 667 In Plasmodium falciparum infection. 5 . PfEMP3 B. A.30 hours A. 8 . the most effective mosquito vector of malaria is ? B. PfEMP2 A. > 3 days D. Number of sporozoite (+) mosquito bites/person The processes of cytoadherence. falciparum infections. All of the above antigenically variant. They result in the sequestration of RBCs containing mature forms of the parasite in vital organs where they interfere with microcirculatory flow and metabolism and avoid Entomologic inoculation rate is the number of sporozoite-positive mosquito bites per person per year. rosetting. falciparum infected RBCs adhering to other parasitized RBCs is called agglutination. PfEMP1 Harrison’s 18th Ed. square of the number of human Harrison’s 18th Ed. Rosette Malarial parasite consumes hemoglobin. 3 . Cytoadherence D. 1691 B. Number of sporozoite (+) mosquito bites/person/year D. 1691 663 The entomologic inoculation rate is ? A. chondroitin sulfate B in placenta & CD36 in most other organs. Hemozoin C. Anopheles stephensi D.from gametocyte ingestion to Harrison’s 18th Ed.15 hours after the cell’s invasion. Anopheles costalis D. All of the above 666 In P. “knobs” appear on RBC’s surface how many hours after the cell’s invasion ? Transmission of malaria is directly proportional to density of the vector. C. Anopheles sinensis 669 Which of the following processes is central to the pathogenesis of The most effective mosquito vector of malaria in Africa is Anopheles gambiae. and tenth power of the probability of the mosquito’s surviving for 1 day. D. Rosetting A.7 days C. falciparum infections. falciparum infection is mainly due to ? 671 Process of P.30 days infection include all except ? Malarial parasite’s life cycle that takes place within the mosquito . 664 Malaria pigment is also called ? 670 Process of P. Cytoadherence D. 1691 B. Table 114-1 (sporogony) lasts ? A. > 1 day B. Hemoverdin Harrison’s 18th Ed. membrane protuberances or “knobs” extrude a high-molecular-weight. Erythrocyte-binding protein 185 (EBA-185) B. Agglutination B. Number of sporozoite (+) mosquito bites falciparum malaria. 1691 bites per day per mosquito. 5 days. ovale In classic malarial paroxysms. falciparum. Malarial sequele 679 The level of parasitemia in P. IgM B. C. P. All of the above A. P. ovale death from falciparum malaria ? Harrison’s 18th Ed. vivax phosphate dehydrogenase (G6PD) deficiency confer protection against death from falciparum malaria. vivax. falciparum D. A. Sickle cell disease B. P. 50 hours for P. C. 1692 D. C. P. P. hereditary ovalocytosis. P. malariae. falciparum. malariae 681 Which of the following genetic disorders confers protection against D. for P. P.880 MCQ’s FOR MEDICAL PROFESSIONALS BY PROF. falciparum Sickle cell disease. malariae attacks old cells and produces parasitemia that is seldom >2%. ovale and brown- black in P. P. falciparum is 5. 5 % 673 Which of the following has the least duration of intrahepatic phase ? Harrison’s 18th Ed. P. vivax parasitemia. IgG 676 Pigment colour of which of the following is “black” ? Harrison’s 18th Ed. vivax is 8 days. 1689 Table 210-1 C. P. sequestration does not occur. 1691 Number of merozoites released per infected hepatocyte is 30000 for P. & rigors is irregular at first. 1689 Table 210-1 C. A. 1692 Pigment color is black in P. IgG. AJAY MATHUR Miscellaneous 672 Premunition refers to ? B. P. malariae D. P. malariae 682 In malarial immune individuals. P. chills. malariae Harrison’s 18th Ed. P. 1691 C. ovale B. In falciparum malaria it may never become regular. Ovalocytosis 675 Which of the following has the longest duration of erythrocytic cycle ? C. malariae A. fever spikes. P. falciparum . P. malariae is 15 days. malariae D. A. falciparum D. ovale following increase ? Harrison’s 18th Ed. 10 % B. P. 1691 The state of malarial infection without illness is called premunition. A. P. P. serum levels of which of the D. A. and IgA. falciparum falciparum can invade erythrocytes of all ages and may be associated with very high levels of B. for P. vivax B. 48 hours for P. G6PD deficiency Harrison’s 18th Ed. 1012 parasites infected hepatocyte ? In malaria due to P. hemoglobins C & E. falciparum 684 Death rate in adults with coma due to falciparum malaria is ? Harrison’s 18th Ed. and glucose-6- B. P. malariae seldom exceeds ? D. vivax. vivax D. vivax. 106 parasites Duration of intrahepatic phase for P. 1692 678 Which of the following has preference for older RBC’s ? A. ovale show a marked predilection for young RBCs. Malarial prodrome C. falciparum . ovale and 72 hours for P. and all stages Harrison’s 18th Ed. P. 15000 for P. C. vivax. 10000 for P. P. yellow-brown in P. thalassemias. 1691 B. Infection without illness D. vivax Immune malarial individuals have a polyclonal increase in serum levels of IgM. dark brown in P. 1010 parasites 674 Which of the following releases least number of merozoites per D. P. P. 108 parasites 9 days and for P. P. malariae A. P. vivax Harrison’s 18th Ed. 1689 Table 210-1 D. ovale and 15000 for P. ovale. vivax. P. vivax 680 Total proportion of infected RBC’s of >2% corresponds to ? C. although much of this antibody is unrelated to protection. P. P. 1689 Table 210-1 of the parasite’s development are evident on PBF. ~ 10 % A. ~5% Harrison’s 18th Ed. P. ovale is B. falciparum B. vivax. 2 % B. All of the above B. 1692 Duration of erythrocytic cycle is 48 hours for P. malariae. malariae. ovale 677 Which of the following is not a “benign” human malaria ? Harrison’s 18th Ed. P. ovale A. P. and P. C. and P. P. malariae. Repeated malarial infection Harrison’s 18th Ed. ovale 683 The fever never becomes regular in ? Harrison’s 18th Ed. 8 % A. P. ovale. 1691 C. IgA A. acute renal failure & acute pulmonary edema are unusual. Diffuse symmetric encephalopathy D. 1693 15% among children despite treatment. 1694. Convulsions. Convulsions. Convulsions A. All of the above 693 Which of the following is “uncommon” among children with severe malaria ? Hypoglycemia in malaria is due to failure of hepatic gluconeogenesis & increase in consumption of glucose by host and malaria parasites. 1695 insulin secretion. C. ~ 15 % D. Anuria Harrison’s 18th Ed. Anaerobic glycolysis in tissues 685 Which of the following about cerebral malaria is false ? B. rare in children Harrison’s 18th Ed. focal neurologic signs are unusual. Serum Bilirubin D. Failure of hepatic gluconeogenesis C. Abdominal and cremasteric reflexes are absent by the parasites. PO 2 696 Which of the following is “uncommon” among pregnant women with severe malaria ? D. ~ 20 % 691 In severe malaria. Focal neurologic signs are unusual Plasma concentrations of bicarbonate or lactate are the best biochemical prognosticators in C. pH D. coma 688 Poor prognostic laboratory features in severe falciparum malaria B. None of the above C. B. A. Hypoglycemia are all except ? C. AJAY MATHUR Cardiology 881 C. 1695 D. 1694 Table 210-3 D. Metabolic acidosis Acidosis results from accumulation of organic acids and carries poor prognosis. Hyperventilation B. 1693 Convulsions. Acute pulmonary edema Harrison’s 18th Ed. Use of quinine & quinidine D. coma. Renal failure severe malaria ? B. 1695 D. 1693 C. Severe anemia Harrison’s 18th Ed. 1693 686 Hypoglycemia in malaria is due to ? A. Hypoglycemia B. Failure of hepatic & renal lactate clearance A. 1694 Table 210-3 D. 1692 B. More common in adults. A. severe falciparum malaria is false ? Harrison’s 18th Ed. Severe anemia A. Hypoglycemia Harrison’s 18th Ed. Bicarbonate concentration C. hypoglycemia. Signs of meningeal irritation are absent. PCO 2 Congenital malaria occurs in <5% of newborns whose mothers are infected. PCO 2 Harrison’s 18th Ed. Hyperthermia 694 Which of the following is “uncommon” among children with severe malaria ? C. 881 Miscellaneous MCQ’s FORMCQ’s FOR MEDICAL MEDICAL PROFESSIONALS PROFESSIONALS BY PROF. Table 210-4 690 Which of the following is the best biochemical prognosticator in A. Deep jaundice 689 Which of the following is the best biochemical prognosticator in severe malaria ? D. lactic acidosis is caused by ? Death rates in cerebral malaria with coma due to falciparum malaria is ~20% among adults and Harrison’s 18th Ed. Hypoglycemia C. Increased consumption of glucose by host & parasite D. Pout reflex is uncommon severe malaria. Cerebral malaria is a diffuse symmetric encephalopathy. Lactic acidosis is caused by the combination of anaerobic glycolysis in tissues where sequestered parasites interfere with microcirculatory flow. Renal cortical necrosis never develops B. metabolic acidosis & severe anemia are relatively common among A. Seizures A. Metabolic acidosis A. Abdominal 692 Which of the following statements about renal impairment in and cremasteric reflexes are absent. Lactate concentration . B. Acidosis 695 Which of the following is “uncommon” among children with severe malaria ? C. pH children with severe malaria. Lactate production by parasites Harrison’s 18th Ed. coma 687 Poor prognostic clinical features in severe falciparum malaria are B. Hypoglycemia all except ? C. Pulmonary edema B. Hypercalcemia A. Quinine & quinidine are powerful stimulants of pancreatic Harrison’s 18th Ed. and a failure of hepatic and renal lactate clearance. Eyes may be divergent and a pout reflex is common. Manifests as acute tubular necrosis A. whereas deep jaundice. Hyperuricemia Harrison’s 18th Ed. Acute renal failure Harrison’s 18th Ed. 1692 C. All of the above B. hypovolemia. lactate production D. 1693 C. All of the above Malaria can be transmitted by blood transfusion. Hyperactive malarial splenomegaly after the peak of asexual parasites ? 9 Harrison’s 18th Ed. Phagocytosed malarial pigment in >5% of neutrophils D. All of the above film should be of uneven thickness. dried thoroughly and stained without fixing. vivax 707 At low levels of parasitemia. PfHRP2 703 Which of the following about blood smear prepared for visualising malarial parasite is false ? C. Thick blood film should be of even thickness A. hepatocyte injury & cholestasis. or Leishman’s stain can also be used. 700 Tropical Splenomegaly is also called ? - Harrison’s 18th Ed. P. Glucocorticoids diagnosis of malaria in patients with low-level parasitemia. Bone-marrow suppression dried. Harrison’s 18th Ed. 1695 d Harrison’s 18th Ed. Antimalarial agents Of the Romanowsky stains. Blood transfusion A. Wright’s 702 Quartan malarial nephropathy usually responds poorly to C. 2 D. falciparum . 4 h 701 Which of the following can cause nephrotic syndrome ? In P. Giemsa at pH 7. 1696 A. gametocytemia peaks how many weeks A. 1 C. Thin blood smear should be fixed in anhydrous methanol Rapid. falciparum infections results from hemolysis. peripheral schizontaemia. sharing of needles by infected injection drug users. 704 Patients with what count of parasite are at increased risk of dying ? 698 Severe jaundice in P. C. or organ transplantation. a poor prognosis is indicated by a predominance of more mature P. Anemia results from accelerated RBC removal by the spleen. Giemsa’s D. PfHRP4 A. > 102 parasites/µL A. Thin blood smear should be rapidly air- C. staining of malaria parasite with B. Sharing of needles by infected drug addicts C. PfHRP1 B. Hyperreactive malarial splenomegaly 9 B. Wright’s. obligatory RBC destruction at parasite schizogony. Cytotoxic drugs 708 Specific antibody-based diagnostic stick or card test for malaria D. P. Hemolysis B. Staining of parasites with fluorescent dye acridine orange allows more rapid B. In severe malaria. 1696 Harrison’s 18th Ed. Thick smear should be stained without fixing B. Harrison’s 18th Ed. falciparum specific antigen ? Harrison’s 18th Ed. 1696 Quartan nephropathy usually responds poorly to treatment with either antimalarial agents or glucocorticoids and cytotoxic drugs. 1694 A. sensitive & specific antibody-based diagnostic stick or card tests detect P. > 104 parasites/µL G C. All of the above detect which P. P. Leishman’s treatment with ? D. PfHRP3 Harrison’s 18th Ed. simple. Direct destruction of parasitised RBC’s D. RBC’s in tail of thin blood smear should be examined Lancet 2005. a poor prognosis is indicated by ? Harrison’s 18th Ed. 1696 C. 365: 1487-98.2 is preferred. D. V 699 Malaria can be transmitted by ? 705 In severe malaria. Thick blood D. though nonimmune patients may Severe jaundice in P. gametocytemia peaks 1 week after the peak of asexual parasites. 1695 ta A. Hypertrophic malarial splenomegaly A. > 105 parasites/µL D. and partially immune persons may tolerate parasitemia levels many times higher with only minor symptoms. 1695 D. falciparum infections is due to ? Harrison’s 18th Ed. P. All of the above R Patients with > 105 parasites/µL are at increased risk of dying. Level of parasitemia is expressed as number of parasitized RBC’s per 1000 RBCs. needle-stick injury. malariae A. Hepatocyte injury C. Destruction of non-parasitised RBC’s Both thin and thick blood smears should be examined. Organ transplantation B. All of the above n D. 1695 A. AJAY MATHUR Miscellaneous 697 Severe anaemia in malaria is due to ? B. > 103 parasites/µL B. 3 i Tropical splenomegaly is also called Hyperreactive malarial splenomegaly (HMS). Hypercongestive malarial splenomegaly r C. 1696 B. and stained. > 20 % of parasites with visible pigment in PBF ti e B. falciparum B. fixed in anhydrous methanol. Cholestasis D. Presence of circulating schizonts in PBF C. A. die with lower counts. Field’s.882 MCQ’s FOR MEDICAL PROFESSIONALS BY PROF. 1695 706 In P. and ineffective erythropoiesis. ovale which of the following stains allows more rapid diagnosis ? Harrison’s 18th Ed. falciparum infections. RBCs in tail of the film should then be examined. falciparum infections. falciparum U parasites (>20% parasites with visible pigment) in PBF or by presence of phagocytosed malarial pigment in >5% of neutrophils and. Fluorescent dye acridine orange Harrison’s 18th Ed. All of the above Harrison’s 18th Ed. D. artemisinin is also called qinghaosu. 1700 Table 210-7 D. Atovaquone-proguanil acts on liver schizonts of all four Plasmodial species The main active metabolite of artesunate is dihydroartemisinin. P. Dihydroartemisinin 712 Which of the following statements is false ? C. Artesunate A.359:603-12 B. Artemisia genia N Engl J Med 2008. Primaquine kills quiescent C. Elevated levels of antithrombin III N Engl Med 2011. B. P. raised CRP. Hepatic stage schizonticides . normocytic anemia. P. 1700 Table 210-7 A.365:1073 711 Which of the following is a blood-stage schizonticide ? A. artemisinin is also called ? D. Project 521 plasma viscosity. Atovaquone D.S/AS02A is an experimental vaccine against ? Lancet 2005. P. Primaquine kills quiescent hypnozoites C. Tetrahydroartemisinin A. Doxycycline D. raised ESR. Mefloquine and Chloroquine. Guanil Schizonts are multinucleated stages of parasites that undergo mitotic division within host cells. P. preventing clinical manifestations of malaria infection. Artemether is oil-based whereas artesunate is water soluble. normal TLC. 1697 In China. Qingli . Hepatitis C B. Qingshi Harrison’s 18th Ed. Quinine D. Project 522 710 Trophozoites and schizonts of Plasmodium knowlesi is C.proguanil C. histidine-rich protein 2 (PfHRP2) or lactate dehydrogenase antigens in blood samples. Cycloguanil hepatocytes and act on liver schizonts of all four species of plasmodia. B. Project 524 A. Guanyldehyde 713 RTS. Trihydroartemisinin N Engl J Med 2008. Lumefantrine C. vivax of P. ovale N Engl Med 2011. Reduced platelet count project that developed artemisinins or qinghaosu derivatives ? D. Primaquine 715 In China. 365: 1487–98 D. Monohydroartemisinin They interrupt schizogony within red cells. Hepatitis A A. raised A. falciparum ? Harrison’s 18th Ed. Pentaguanil hypnozoites while Atovaquone-proguanil does not act on hypnozoites. Atovaquone . Normochromic. Artemisia annua A. Artemisia gripe B. Atovaquone-proguanil does not act on hypnozoites 719 Active metabolite of ‘Proguanil’ is ? Harrison’s 18th Ed.atovaquone-proguanil & primaquine kill malaria parasites within B. Elevated C-reactive protein 716 Chinese government scientists worked on which secret C. normocytic anemia B. falciparum C. 883 Miscellaneous MCQ’s FORMCQ’s FOR MEDICAL MEDICAL PROFESSIONALS PROFESSIONALS BY PROF. P. ovale A.365:1073 Laboratory findings in malaria include normochromic. Doxycycline. Mefloquine 718 Active metabolite of artesunate and artemether is ? D. 1700 Table 210-7 B. P. 1700 Table 210-7 721 Which antimalarial drug kills all stages of gametocyte development A. 365: 1487–98 720 Which antimalarial drug has good absorption ? Harrison’s 18th Ed. B. malariae 717 Researchers in China isolated the active artemisinin compounds from the plant named ? D. falciparum Chinese government scientists worked on secret “Project 523” that developed artemisinins or qinghaosu derivatives. B. Halofantrine C. reduced platelet count. Qingchao 709 Which laboratory finding is not found in malaria ? D. Halofantrine B. B. P. C. Qinghaosu PfHRP2-based tests may remain positive for several weeks after acute infection. A. reduced antithrombin III even in mild infection.359:603-12 D. Leishmania D. Artesunate 714 Quinine kills the gametocytes of all except ? Harrison’s 18th Ed. AJAY MATHUR Cardiology 883 specific. Project 523 morphologically similar to which of the following ? Lancet 2005. A. Artemisia duova C. 1700 Table 210-7 Blood-stage schizonticides are Atovaquone-proguanil. malariae B. vivax C. Malaria C. None of the above A. or Blantyre coma scale <3/5 in children. Polymerase chain reaction D. 1700 Table 210-7 A. > 100 gram A. respiratory distress (>32 breaths per minute) ti e A. > 75 gram malaria. Plasmodium LDH test A. 2 days D. Failure of hepatic gluconeogenesis h D. 75 & 76: 29-47 D. 1691 C. tricyclic antidepressants. Positive PfHRP1 test U 725 In quartan malaria. 1700 Table 210-7 730 What level of parasitemia is associated with an increased risk of A.2 mg/mL D. > 15 % A. blood bicarbonate <15 mmol/L. > 10 gram B. All of the above A. 1698 Table 210-5 Harrison’s 18th Ed. Asexual parasites/200 WBCs x 40 B. terfenidine. 1698 Table 210-5 C. Parasitized RBCs (%) x Hematocrit x 1256 i Harrison’s 18th Ed. B. PCR is used for genotyping and speciation in mixed infections. 1227 For quinine. 1698 Table 210-5 D. V Indicators of severe malaria are Glasgow coma scale <11/15 in adults. 0. chloroquine. 1-Acid Glycoprotein (1-AGP) Erythrocytapheresis in which the RBC fraction is removed by apheresis & plasma. haematocrit <20% and Pf parasitaemia >100000/µL. Acridine orange staining C. mefloquine. 1693 C.15 mg/L provides optimal therapeutic range for its A. > 80000 parasites/µL b Halofantrine should not be used by patients with long ECG QT c intervals or known conduction R disturbances or by those taking drugs that may affect ventricular repolarization like quinidine. Stimulation of pancreatic insulin secretion ta C. neuroleptics. 1-Acid Glycoprotein (1-AGP) platelet fractions are returned to patient is also used with good effect in severe disease. Quinine causes hypoglycemia due to ? B. >2% effect in severe malaria and does not cause serious toxicity. 25 to 35 mg/mL parasitemia levels of ? Harrison’s 16th Ed. Decreased glucagon secretion 733 Parasite count/µL can be calculated by ? Harrison’s 18th Ed. Tricyclic antidepressants severe malaria ? B.884 MCQ’s FOR MEDICAL PROFESSIONALS BY PROF. All of the above B. > 40000 parasites/µL C. 2 to 4 mg/mL C. Mefloquine D. 4 days Harrison’s 18th Ed. 723 Halofantrine is not be used with which of the following drug ? Harrison’s 18th Ed. A. even if the blood smear is negative ? n Harrison’s 18th Ed.6 to 1. Quinine C. > 10 % British Medical Bulletin 2005. blood urea nitrogen >17 mmol/L. 1-Acid Glycoprotein (1-AGP) D. Parasitized RBCs (%) x Platelet count x 1256 B. asexual Pf Harrison’s 18th Ed. visible 724 Chloroquine can cause retinopathy when its cumulative dose exceeds ? d jaundice and Pf parasitaemia >100000/µL. Asexual parasites/200 WBCs x 30 A. shock (SBP<70 mmHg or core-skin temperature difference >10°C). plasma glucose <2·2 mmol/L. Asexual parasites/200 WBCs x 20 Harrison’s 18th Ed. regular fever patterns are evident every ? B. Chloroquine B. antiarrhythmics. All of the above Antibody & polymerase chain reaction tests have no role in diagnosis of malaria. 1700 Table 210-7 parasitaemia >10%. Positive PfHRP2 test Harrison’s 18th Ed. Chloroquine . or astemizole. 1701 C. 8 to 15 mg/mL 734 Exchange transfusion for severe malaria is indicated for D. total plasma concentrations of 8 . 5 days 9 A. > 100000 parasites/µL quinine. Terfenadine Harrison’s 18th Ed. > 50 gram 731 Which of the following test indicates that the illness is falciparum C. AJAY MATHUR Miscellaneous 722 Halofantrine should not be used with which of the following 729 Which of the following tests have no role in the diagnosis of malaria ? antimalarial drug ? Harrison’s 18th Ed. Asexual parasites/200 WBCs x 10 727 Optimal therapeutic range for quinine in severe malaria is ? B. 1702 1-Acid Glycoprotein (a1-AGP or Orosomucoid) is the main quinine binding plasma protein. Positive PfHRP3 test - A. >5% 728 Main quinine binding plasma protein is ? C. 1698 Table 210-5 D. PfHRP2 B. 1698 Table 210-5 D. Astemizole A. Parasitized RBCs (%) x Reticulocyte count x 1256 A. D. leukocyte & B. 1-Acid Glycoprotein (1-AGP) 735 Which of the following antimalarials is safe in pregnancy ? Harrison’s 18th Ed. > 20000 parasites/µL G D. Positive PfHRP4 test B. However. Parasitized RBCs (%) x Hemoglobin x 1256 r 726 In malaria. C. 3 days 732 Parasite count/µL can be calculated by ? 9 C. Parenteral artesunate is a water-soluble artemisinin derivative and is the drug of choice in treating severe malaria. Tinnitus A. Mechanism remains unknown. Artesunate D. 745 Which of the following statements about falciparum malaria in 739 In endemic areas. 12 . 8 . doxycycline.55 mg/L Repeat blood smears should be performed at least every 12-24 hours for 2 days if the first smears Total plasma concentrations of 8 . second- 738 Which of the following about blackwater fever is false ? line treatment is given for what duration ? British Medical Bulletin 2005.g. 1701 A. 3 .24 hours for 2 days C. 12 .15 mg/L C. 25 % C. 1702 everywhere ? A. All of the above Cinchonism comprising tinnitus. Nil A. 18 . or clindamycin is effective.15 mg/L for quinine is effective & does not cause serious are negative and malaria is strongly suspected. uncomplicated falciparum malaria everywhere. 50 % D. Chloroquine B. malariae) except in Indonesia and Papua New Guinea. Primaquine B. 3 days B.6 seconds C. Artemether and artemotil (arteether) are oil-based formulations. Mefloquine To prevent resistance. Artemether B. maintenance doses of quinine Despite chloroquine resistance. Doxycycline C. high-tone deafness. 1702 Harrison’s 18th Ed. 1701 Harrison’s 18th Ed.24 hours for 1 days B. 1702 Harrison’s 18th Ed. In some it artesunate or quinine plus tetracycline. 7 days D. Combination drugs (two or more) must be used simultaneously and must In endemic areas. 36 . repeat 742 What is the total plasma concentrations of quinine at which it is blood smears should be performed at least every ? effective & does not cause serious toxicity ? Harrison’s 18th Ed. 1 . and dysphoria. P. ovale. 1698 A. The initial doses should never be reduced. QTc interval >0. Quinine C.6 hours for 1 day A. Artemisinin B. 1701 A. 885 Miscellaneous MCQ’s FORMCQ’s FOR MEDICAL MEDICAL PROFESSIONALS PROFESSIONALS BY PROF. Quinine administration B. Artemotil (arteether) C. the initial dose of quinine should be reduced by ? Asia is ? Harrison’s 18th Ed. vomiting. 10 days Blackwater fever reflects the presence of massive haemolysis leading to haemoglobinuria especially As second-line treatment for recrudescence following first-line therapy. None of the above D. malarias (P. 1701 A. Should not be treated with single drug Harrison’s 18th Ed. 747 In drug resistance areas. 737 Treatment of choice for the “benign” human malarias in southern 743 In renal failure. Combination drugs should have different modes of action C. quinine can be given with all of the following except ? 741 Indication for slowing infusion rates of quinidine is ? Harrison’s 18th Ed. Massive intravascular haemolysis & haemoglobinuria A. Atovaquone-proguanil D. chloroquine remains the treatment of choice for “benign” human should be reduced by 30-50%. a 7-day course of either after quinine or artemisinin derivatives administration. 744 For recrudescence of malaria following first-line therapy. vivax. nausea. 746 Cinchonism comprises of all of the following except ? 740 Which of the following is a water-soluble antimalarial ? Harrison’s 18th Ed. P. Mefloquine If patient remains seriously ill or is in acute renal failure for >2 days. Glucose-6-phosphate dehydrogenase (G6PD) deficiency C. may be associated with an underlying haemolytic tendency.6 mg/L B. e. Tetracycline A. 1702 A. 1701 A. Should be treated with drug combinations B. 12 . 10 % B. Artesunate D. Total plasma levels >8 µg/mL . QRS widening beyond 25% of baseline D. Artemisinin-based combinations C. Diarrhoea D. 75 & 76: 29-47 Harrison’s 18th Ed. All of the above 736 If the first PBF is negative & malaria is strongly suspected. High-tone deafness C. toxicity. Dysphoria B. None of the above D. AJAY MATHUR Cardiology 885 B. WHO recommends which of the following as endemic areas is false ? first-line treatment for uncomplicated falciparum malaria Harrison’s 18th Ed. G6PD deficiency. 1698 Harrison’s 18th Ed. Chloroquine D.31 mg/L D. PF malaria should be treated with drug combinations and not with single drugs in endemic areas. 5 days C.24 hours for 3 days D. WHO recommends Artemisinin-based combinations as first-line treatment for have different modes of action. doxycycline. In drug. All of the above Dihydroartemisinin-napthoquine-trimethoprim. In multidrug- resistant Pf malaria. Parasitemia has not cleared by 7 days A. Parasitemia has not cleared by 2 days 758 Malarial chemoprophylaxis should be continued for what duration B. Dawn Artemisinin combination regimens now constitute first-line treatment for Pf malaria. Repeat the same dose Harrison’s 18th Ed. Mefloquine 754 Peak feeding time for mosquitoes is ? Harrison’s 18th Ed. doxycycline. Throughout the night R sensitive areas.dusk. ta hypotension associated with malaria. 1702 A. All of the above h Recommendations for prophylaxis include drugs effective against resistant P. 21 days with which of the following ? Harrison’s 18th Ed. dawn and throughout the night. the dose D. and quinine exacerbate the orthostatic atovaquone-proguanil. Artesunate C. Other combinations are Dihydroartemisinin-piperaquine. 10 days C. Pyrimethamine n D. mefloquine. Dusk D. Quinine ir C. 3 week D. 1703 B. Doxycycline 753 Primaquine is given for how many days to eradicate persistent C. falciparum like All the antimalarial quinolines like chloroquine. 4 week . Sulfadoxine / pyrimethamine A. Mefloquine C. 1703 A. Doxycycline & primaquine should not be given to pregnant women. atovaquone-proguanil. Parasitemia has not cleared by 10 days B. mefloquine. Give parenteral preparation B. artesunate-amodiaquine or artesunate- sulfadoxine/pyrimethamine can be used. 2 week If the level of parasitemia does not fall to <25% of admission value in 48 hours or if parasitemia has not cleared by 7 days. 1702 D. Does not fall < 25 % of admission value in 6 hours A. 749 If a malaria patient vomits within one hour of administration of d any oral antimalarial drug. C. 1702. D. Lumefantrine B. 365: 1487-98 To eradicate persistent liver stages & prevent relapse (radical treatment). Primaquine D. 1 week D. Artesunate D. 1702 malaria in travellers ? ti e A. Quinine D. or clindamycin is effective. 756 Which of the following drugs is used for prophylaxis against - 750 Which of the following antimalarial drugs exacerbate the malaria in travellers ? orthostatic hypotension associated with malaria ? Harrison’s 18th Ed. Clindamycin A. what should be done ? 755 Which of the following drugs is used for prophylaxis against Harrison’s 18th Ed. Pyrimethamine 9 B. 5 days As second-line treatment for recrudescence following first-line therapy. AJAY MATHUR Miscellaneous B. Quinine U If a malaria patient vomits within one hour of administration of any oral antimalarial drug. C. Does not fall < 25 % of admission value in 48 hours D. Artesunate should be repeated. Clarithromycin liver stages & to prevent relapse ? Harrison’s 18th Ed. V Prevention of malaria includes avoidance of exposure to mosquitoes at their peak feeding times . Doxycycline Mefloquine is the only drug advised for pregnant women traveling to areas with drug-resistant 752 Antimalarial drug resistance is likely if ? malaria. 757 Which antimalarial drug is used for chemoprophylaxis for 751 Antimalarial drug resistance is likely if parasitemia? pregnant women ? Harrison’s 18th Ed. Harrison’s 18th Ed. A. 1703 9 Harrison’s 18th Ed. Parasitemia has not cleared by 5 days after leaving the endemic area ? Harrison’s 18th Ed.886 MCQ’s FOR MEDICAL PROFESSIONALS BY PROF. a 7-day course of either artesunate or quinine plus tetracycline. Mefloquine B. primaquine should be given daily for 14 days after ruling out G6PD deficiency. drug resistance is likely. 1703 C. 14 days 748 Artemisinin derivative in combination regimens can be combined D. Mefloquine C. or primaquine. B. Does not fall < 25 % of admission value in 24 hours C. 1702 Harrison’s 18th Ed. Does not fall < 25 % of admission value in 12 hours B. Change to another oral preparation C. 1702 A. Reduce the dose A. Lancet 2005. Doxycycline A. artemether-lumefantrine or artesunate-mefloquine should be used. mefloquine. Chloroquine B. 1703 C. All of the above G B. Travelers should start taking antimalarial drugs 2 days to 1-2 weeks before departure so that any untoward reactions can be detected and so that therapeutic antimalarial blood concentrations will be present when needed.366:1423-32 Harrison’s 18th Ed. South East Asia B. 770 Which of the following is best related to dengue spread ? N Engl J Med 2012. Culex tarsalis C. 1705 B. D. Dengue fever 760 Malarial chemoprophylaxis should be started for what duration 766 Which of the following statements about Dengue fever is false ? before arriving in the endemic area ? N Engl J Med 2012. Intermittent treatment with pyrimethamine-sulfadoxine An estimated 50 million infections per year occur across ~100 countries. PATH A. spread into Asia through D. Atovaquone Dengue emerged from Africa during slave trade in 15th to 19th centuries. 2 week C. 1703 A. Amodiaquine D. 1705 B. All of the above C. Aedes triseriatus A. 887 Miscellaneous MCQ’s FORMCQ’s FOR MEDICAL MEDICAL PROFESSIONALS PROFESSIONALS BY PROF. Dichlorodiphenyltrichloroethane B. 1622 763 Which of the following antimalarial drug has “sporonticidal A. Program for Appropriate Technology D. Indian Ocean. Dichlorodiphenyltrichloroethylene D. Trade of tires from used vehicles C. 100 A. 769 Which of the following is related to Dengue ? N Engl J Med 2012. 1622 764 Insecticide DDT refers to ? A. Harrison’s 18th Ed. except if atovaquone-proguanil or primaquine is taken.366:1423-32 761 Chemoprophylaxis with which of the following antimalarial drugs A.366:1423-32 762 Which of the following antimalarial drug is associated with a high risk of agranulocytosis ? A. a secondary vector. Doxycycline C. Global warming D. MACEPA Harrison’s 18th Ed. Mefloquine D. explains the dispersal of eggs Insecticide DDT refers to Dichlorodiphenyltrichloroethane. Transmitted to humans by mosquitoes C. Proguanil D. None of the above D. 1703 B. D. AJAY MATHUR Cardiology 887 759 Which malarial chemoprophylactic agent can be discontinued 765 Which of the following programmes is related to the control of malaria ? after 1 week of leaving the endemic area ? A. Chloroquine C. It spread globally with the advent of increased travel & trade in the past 50 years. Harrison’s 18th Ed. None of the above in Health (PATH) and Medicines for Malaria Venture (MMV) are programmes for the control of malaria. Aedes albopictus activity” ? Harrison’s 18th Ed. Culex tritaeniorhynchus B. Mefloquine commercial exchanges in 18th & 19th centuries. Mefloquine Dihydrofolate reductase inhibitor proguanil inhibit development in mosquito (sporontocidal activity). Aedes albopictus. When used continuously. and parts of Europe. Proguanil C. Latin America A. 3 week D. . & immature forms of these arboviral vectors into new territories. All of the above 768 Dengue is said to have emerged from ? N Engl J Med 2012. particularly trade of tires from used vehicles. 50 can be given to pregnant women at risk in endemic areas ? Harrison’s 18th Ed. Dichlorodipropyltrichloroethylene Globalization of trade. 767 Globally. These have significant activities against liver stage of infection (causal prophylaxis) & can be discontinued 1 week after departure from endemic area. Systemic viral infection B. 1703 B. Urban migration A. Atovaquone In recent years. Australia C. Amodiaquine D. has dramatically expanded Dengue spread from Asia to United States.366:1423-32. MMV B. Textile trade B. Antimalarial prophylaxis should continue for 4 weeks after the traveler has left endemic area. about how many countries are affected by dengue ? N Engl J Med 2012. Self-limited illness A. Atovaquone-proguanil Malaria Control and Evaluation Partnership in Africa (MACEPA). Proguanil C. amodiaquine is associated with a high risk of agranulocytosis and hepatotoxicity and should not be used for prophylaxis. 1 week B. Dichlorodipropyltrichloroethane C. systemic viral infection transmitted to humans by mosquitoes. 150 B.366:1423-32. 200 C. 4 week Dengue is a self-limited. Africa Harrison’s 18th Ed. Typically breeds near human habitation dengue as severe dengue ? V B. Dengue fever A. Most common arboviral disease worldwide D. and a spontaneous recovery phase. D. Typhoid fever 782 Which of the following is not a phase of dengue fever ? N Engl J Med 2012. headache. Keratan sulfate A. B. All of the above glycocalyx. AJAY MATHUR Miscellaneous 771 Aedes aegypti is an efficient vector of ? Cross-reactive but non-neutralizing anti-dengue antibodies from previous infection bind to new Harrison’s 18th Ed. Relapsing fever C. Critical phase around defervescence A. 1 to 2 days U 780 Which of the following is false about dengue fever ? B. and ascites. 14 to 21 days B. or both.888 MCQ’s FOR MEDICAL PROFESSIONALS BY PROF. T cell non-reactivity apparent around the time of defervescence (days 4 and 7 of the illness). Type 3 776 Biphasic “saddleback” fever is suggestive of ? D. platelet destruction & consumption of coagulation factors leading B. Dengue virus 778 Which of the following is best related to dengue ? D. 1632 D. 1622 infecting serotype & enhance viral uptake of monocytes and macrophages. D. Initial afebrile phase 777 Which of the following is central to pathogenesis of dengue fever B. aegypti is false ? detected in children with severe infection. Yellow fever C. Lyme disease potential to cause DHF/DSS.366:1423-32 C. Systemic vascular leak syndrome becomes C. Zoonotic illness D. Dengue fever A. 150 Table 17-1 Type 2 (especially Southeast Asian) is more dangerous than other dengue serotypes with more A. Yellow fever causing endothelial dysfunction. Bites during the day A. severe bleeding. Dengue is the most common arboviral disease worldwide. retroorbital & back pain. 1632 C. A. h B. Severe bleeding ti e Aedes aegypti mosquitotypically breeds near human habitation. Spontaneous recovery phase B. evidenced by increasing hemoconcentration. Type 1 Severe myalgia seen in Dengue fever is named “break-bone fever”. a critical phase around the time of defervescence. Typhoid fever ta Harrison’s 18th Ed. All of the above N Engl J Med 2012.hemorrhagic manifestations. Chikungunya virus to plasma leakage and hemorrhagic manifestations. Other presentations are sudden B. and platelet counts <100. Initial febrile phase and DHF/DSS ? Harrison’s 18th Ed. Dengue Preliminary data suggest that in dengue fever. accumulation of serosal fluid sufficient Harrison’s 18th Ed. Macrophage/monocyte infection Three phases of dengue fever are an initial febrile phase. C. Severe organ impairment bites during the day. a key G structural element of the glycocalyx. Plasma leakage resulting in shock d D. All of the above 774 Incubation period of Dengue fever is ? n Severe dengue refers to plasma leakage resulting in shock. Urban vector 9 Incubation period of Dengue fever is 2 . None of the above 9 775 “Break-bone fever” is suggestive of ? r Harrison’s 18th Ed. pleural effusions. Serotype 1 followed by serotype 2 is more dangerous than serotype 4 followed by serotype 2. Type 2 onset of fever. Protective antiviral antibodies D.7 days. hypoproteinemia. Type 4 Harrison’s 18th Ed. Patients have a triad of symptoms A. 10 to 14 days A.366:1423-32 A. 1622 Viral hemorrhagic fevers are zoonotic illnesses caused by viruses that reside in arthropod urban i vectors. Both the virus itself and dengue NS1 are known to adhere to heparan sulfate.000/µL. 2 to 7 days Harrison’s 18th Ed. Breeds using relatively fresh water N Engl J Med 2012. R Harrison’s 18th Ed. C. transient disruption in the function of endothelial glycocalyx layer occurs bringing about crucial change in the filtration characteristics of the D. Sphingomyelin C. C. Relapsing fever 781 Which infecting serotype of dengue virus is most dangerous ? C. A. 1622 779 Which of the following condition is required to label a case of A. and severe organ impairment.366:1423-32 D. All of the above . 1632 - C. Japanese encephalitis D. Heparan sulfate 772 Which of the following is a Flavivirus ? Harrison’s 18th Ed. 1622 Table 196-1 B. using relatively fresh water. Lyme disease . None of the above B. 1622 to cause respiratory distress. and increased urinary heparan sulfate excretion has been 773 Which of the following about A. and C. This antibody- dependent enhancement results in an amplified cascade of cytokines & complement activation. evidence of plasma leakage. Ceramide B. Twice through illness. Bone marrow D. a high or 789 Main risk factor for development of DHF / DSS is ? increasing hematocrit level that is concurrent with a rapid decrease in the platelet count. 150 Table 17-1 788 Which of the following is a risk factor for severe dengue ? A. Susceptibility drops after 12 years of age B. All of the above disease. and coxsackie virus - focalglomerulonephritis. beginning on trunk & spreads centrifugally to extremities and face. MPGN 791 In dengue fever. Endocarditis . high body-mass index. Age and genetic background of patient D. measles . B. Liver Harrison’s 18th Ed.mesangioproliferative glomerulonephritis. serosal N Engl J Med 2005. Palm Susceptibility to DHF/DSS drops after 12 years of age.366:1423-32 A.endocapillary proliferative glomerulonephritis. Focal glomerulonephritis Harrison’s 18th Ed. Degree of viremia 784 Which of the following about DHF/DSS is false ? Harrison’s 18th Ed. malnutrition is protective. with nadir platelet counts below B. is also an epidemiologic risk factor for severe D. Females are more often affected than males 790 Petechiae DHF/DSS are best visualized in ? C. Malnutrition is protective Harrison’s 18th Ed. A second rash may appear C. 1632 Main risk factor for development of dengue hemorrhagic fever & dengue shock syndrome is secondary infection with another serotype. Endocapillary proliferative glomerulonephritis A.366:1423-32 786 After an infected mosquito bites person. the dengue virus replicates in ? A. 792 In dengue fever. If the pulse pressure narrows to 20 mm Hg or less. followed by rapid improvement during the recovery phase. 1628 D. Regional lymph nodes C. virus strain. accompanied by signs of peripheral vascular collapse. Face disseminated through lymphatic system & blood to other tissues. A. Axillae D. D.353:924-32 B. 150 Table 17-1.MPGN. parvovirus . females are more often affected than males. Thrice during recovery phase. Diffuse proliferative nephritis B. Macular N Engl J Med 2005. increasingly severe abdominal pain. 50 % Viral infections and their respective glomerular lesions include: CMV . Pustular C. 25 % D. None of the above A. Warning signs that clinically significant vascular leakage may be developing in dengue patient include persistent vomiting.353:924-32 effusions. illnesses ? Harrison’s 18th Ed. after defervescence. Nape of the neck 785 Dengue hemorrhagic fever causes which of the following ? Harrison’s 18th Ed. 100 % dengue hemorrhagic fever . Epstein-Barr virus . the kind of skin rash is ? Harrison’s 18th Ed. Female sex developing in dengue patient include ? C. Rocky Mountain spotted fever N Engl J Med 2012. Moderate to severe thrombocytopenia is common. virus replicates in regional lymph nodes and is A. female sex. midway B. AJAY MATHUR Cardiology 889 783 Warning signs that clinically significant vascular leakage may be B. All of the above A.366:1423-32 D. Papular A. dengue shock syndrome is A. Extremities 787 A person may be infected with dengue virus for a maximum of ? C. two sequential infections by different serotypes. B. 150 Table 17-1 C. 2353 Petechiae DHF/DSS are best visualized in axillae. ranging from mild maculopapular rash to severe. A. Young age B.mesangioproliferative glomerulonephritis. High body-mass index N Engl J Med 2012. Strain and serotype of the infecting virus diagnosed. Pulse pressure of 20 mm Hg or less and phospholipase C epsilon 1 genes as risk factors for severe dengue. Once In dengue fever. i. N Engl J Med 2012. it is maculopapular rash. High or increasing hematocrit level and genetic variants of the human major-histocompatibility-complex class I-related sequence B C. D. C. Secondary infection with another serotype 20000 per microliter often observed during the critical phase. mucosal bleeding. C. 150 Table 17-1 After an infected mosquito bites a person. petechiae on extremities may occur. tender hepatomegaly.353:924-32 D. skin rash occurs in 50% of cases. To start with its a diffuse flushing. Trunk N Engl J Med 2005. Neck A. Maculopapular B. Persistent vomiting Epidemiologic studies have identified young age. Four times Infection with one Dengue serotype confers long-term immunity only to that serotype and therefore 794 Purpuric eruptions may occur in which of the following persons may be infected up to four times. Spleen 793 In dengue fever. and lethargy or restlessness. Secondary infection. 75 % endocapillary proliferative glomerulonephritis.MPGN. Groin B. 889 Miscellaneous MCQ’s FORMCQ’s FOR MEDICAL MEDICAL PROFESSIONALS PROFESSIONALS BY PROF. the skin rash begins on ? D. itchy lesion suggesting leukocytoclastic vasculitis that resolves with desquamation over a period of 1 to 2 weeks. mumps .e. skin rash occurs in what proportion of cases ? B. C. and rickettsial infections (tick typhus) from southern Africa. and bacterial infection. and invasion (trichinellosis. typhoid fever from disease. Yellow fever C. 1048 Table 123-3 A. Cysticercosis C. but severe muscle pain is the hallmark ta Flaviviridae (yellow fever in Africa and South America and dengue in Asia. Caribbean A. particularly in which region ? Harrison’s 18th Ed. disseminated gonococcal infection. Flaviviridae r C. purpura B. and the of pleurodynia (coxsackievirus B). All of the above Lassa fever and Ebola and Marburg virus infections are also transmitted from person to person. 804 Positive tourniquet test for capillary fragility is ? 798 Which of the following virus infection is also transmitted from N Engl J Med 2005. or coxsackievirus B infection) or parasitic renal syndrome in Asia).40 % G D. Africa D. All of the above D. 1028 the upper arm to ? N Engl J Med 2005. cysticercosis. dengue fever. mortality rates are ? Harrison’s 18th Ed. Marburg virus infections C. dengue was acquired most often V from ? 802 Which of the following can be transmitted by blood transfusion ? Harrison’s 18th Ed. and yellow fever vaccine virus can be transmitted by blood U southern Asia. Filoviridae (Ebola and Marburg virus infections in Africa). dengue. Arenaviridae (Lassa h fever in Africa). Below diastolic BP for 5 minutes C. 5 or more petechiae appear in a 1 sq. 1068 A. If dengue shock syndrome develops. and cutaneous small-vessel C. mortality can reach 40%. 1028 infections ? Harrison’s 18th Ed.20%. AJAY MATHUR Miscellaneous C. 1028 D. viral hemorrhagic fever. In dengue hemorrhagic fever. Dengue D. Lassa fever Purpuric eruptions may occur in Rocky Mountain spotted fever. mortality rates are 10 . All of the above i D. Between systolic and diastolic BP for 5 minutes D. Dengue fever 800 Vectors for most viral fevers are found in rural areas except ? Harrison’s 18th Ed. chikungunya virus. Ebola virus infections B. D. 15 or more petechiae appear in a 1 sq. Southeast Asia B. Bunyaviridae 9 B. Above systolic BP for 5 minutes B. it was found that malaria was Malaria. babesiosis. dengue & yellow fever are exceptions.70 % Over the past decade. 957 d Harrison’s 18th Ed. Hantavirus hemorrhagic fever with renal syndrome D. or toxoplasmosis). incidence of dengue has increased particularly in Caribbean. trichinellosis. Babesiosis ti e B. All of the above n Cooperative study by GeoSentinel of febrile returned travelers. Chagas disease C. Dengue fever 795 Over the past decade. rat-bite fever. 1044 801 In dengue hemorrhagic fever. dengue from Southeast Asia and Caribbean. 60 . Latin America. - 797 Viral hemorrhagic fevers are caused by which group of viruses ? 803 Severe muscle pain is the hallmark of which of the following Harrison’s 18th Ed. Coxsackievirus B D. Hantavirus hemorrhagic fever with renal syndrome vasculitis (leukocytoclastic vasculitis).890 MCQ’s FOR MEDICAL PROFESSIONALS BY PROF. 10 or more petechiae appear in a 1 sq. Bunyaviridae (Rift Valley fever in Africa or hantavirus hemorrhagic fever with Myalgia occurs with viral infection (influenza. Southeast Asia C. Americas). Latin America C. endocarditis. Influenza B. 1028 A. variant Creutzfeldt-Jakob acquired most often from Africa. 20 or more petechiae appear in a 1 sq. dengue. 2 . 796 In febrile returned travelers. R and Southeast Asia.10 % B. inch C. Latin America B. 10 . inch patch on forearm after deflation of BP cuff. Dengue Viral hemorrhagic fevers are caused by four major groups of viruses.353:924-32 person to person ? Harrison’s 18th Ed. inch A. All of the above A. Africa. Southern Asia A. inch B. enteroviral petechial rash. The tourniquet test is performed by inflating a BP cuff on upper arm to a point midway between systolic & diastolic blood pressures for five minutes. All of the above D. Arenaviridae 9 A. transfusion. Anaplasma phagocytophilum. Chagas disease. Dengue fever Positive tourniquet test for capillary fragility is positive if 20 or more petechiae appear in a 1 sq. 30 . . 1028 A.20 % C. epidemic typhus. Lassa fever A. 799 Which of the following virus infection is also transmitted from person to person ? 805 Tourniquet test is performed by inflating a blood pressure cuff on Harrison’s 18th Ed. incidence of dengue has increased Vectors for most viral fevers are found in rural areas. Ebola virus infections fulminans. acute & chronic meningococcemia. inch D. Lassa fever B.353:924-32 A. thrombotic thrombocytopenic purpura/hemolytic-uremic syndrome. human parvovirus B19 infection. 811 Which of the following is false about primary infection of dengue 817 Which of the following antiviral drug has been evaluated in patients virus ? with confirmed dengue virus infection ? N Engl J Med 2005. All of the above 808 Which of the following statements about dengue shock syndrome Dengue can be ruled out if symptoms begin > 2 weeks after traveler has left endemic area. Hemorrhagic manifestations without capillary leakage also constitute dengue hemorrhagic fever B. Docosanolf B. Rapid. Carbazochrome C. Absence of high titers of IgM early in disease course N Engl J Med 2005. virusexpressed soluble nonstructural protein 1 (NS1). at about the time of defervescence. Sudden change from fever to hypothermia may be the first 812 Criterion for secondary infection of dengue virus is ? N Engl J Med 2005. IgM A. Balapiravir C. Objective evidence of plasma leakage A. Antiviral agents 810 Which of the following statements about IgM capture ELISA used for diagnosis of dengue fever is false ? D. IgM antibodies are detectable for 3 to 6 months. weak pulse with a narrowing pulse pressure 814 Which of the following is a laboratory diagnostic option in a B. C.353:924-32 816 Drugs that have shown some antiviral activity against dengue in A.353:924-32 N Engl J Med 2012. N Engl J Med 2005. which of the following is false ? D. All of the above early in the course of disease and should be performed 4-5 days after onset of symptoms. 891 Miscellaneous MCQ’s FORMCQ’s FOR MEDICAL MEDICAL PROFESSIONALS PROFESSIONALS BY PROF. interferon alfa and 6-azauridine have shown some antiviral activity in vitro. IgM antibodies appear 4 to 5 days after onset of fever B. Culture of the virus seroconversion is a confirmatory finding in patients in whom dengue is a possible diagnosis. Corticosteroids A confirmed diagnosis is established by culture of the virus. Capillary leakage. Ribavirin.353:924-32 C. B. Hemorrhagic manifestations B.353:924-32 C. All of the above A. Negative early in the course of disease vitro is ? B. IgM seroconversion 809 A confirmed diagnosis of dengue fever is established by ? D. Interferon alfa antibodies remain detectable for life C.353:924-32. Duration of shock is short & mortality rate is ~40 % A. Detection of nonstructural protein 1 (NS1) Dengue shock syndrome in travelers is uncommon. Profound hypotension patient with suspected dengue infection ? N Engl J Med 2012. 6-azauridine The most commonly used test for the diagnosis of dengue is IgM capture ELISA. B. Fever that persists for > 10 days C. Other flavivirus infections never test positive A. or serologic assays.353:924-32 A. All of the above N Engl J Med 2005. whereas IgG B.000 per cubic millimeter 813 Which of the following rules out the diagnosis of dengue ? N Engl J Med 2005. Increase in dengue-specific IgM and IgG antibodies A. None of the above fever (DHF) is false ? Primary infections are characterized by an increase in dengue-specific IgM antibodies 4 to 5 N Engl J Med 2005. None of the above N Engl J Med 2005. Zanamivir . It is negative D. polymerase-chain-reaction (PCR) tests. Other flavivirus infections & rheumatoid factor may lead to false positive test. Often associated with a marked decrease in platelet count B.353:924-32 clinical warning sign A. Presence of high titers of IgM early in disease course 807 For the diagnosis of DHF. Serologic assays management of dengue ? N Engl J Med 2005. Polymerase-chain-reaction (PCR) tests 815 Which of the following therapeutic agents have a role in C. D. Detection of viral nucleic acid D. Platelet count of < 100. B.366:1423-32 C. Presence of high titers of IgG early in disease course D. IgG antibodies appear 7 to 10 days after onset of fever C. C. Fever (DSS) is false ? that persists >10 days usually rules out dengue. 1028 Presence of high titers of IgG early in the course of disease indicates secondary infection. Leucocytosis Hemorrhagic manifestations without capillary leakage do not constitute dengue hemorrhagic fever. accompanied by hemorrhagic manifestations antibodies are detectable for three to six months. Rheumatoid factor may lead to false positive assay N Engl J Med 2005.353:924-32 Detection of viral nucleic acid. whereas IgG antibodies remain detectable for life. Harrison’s 18th Ed. Dengue shock syndrome in travelers is common B. Ribavirin D. Absence of high titers of IgG early in disease course Plasma leakage develops 4 to 7 days after onset of disease. or IgM A. Plasma leakage develops about a week later after defervescence C. AJAY MATHUR Cardiology 891 806 Which of the following statements about dengue hemorrhagic D.353:924-32 days after onset of fever and by an increase in IgG antibodies only after 7 to 10 days.366:1423-32 A. If symptoms begin > 2 weeks after leaving endemic area D.353:924-32 D. A. as is death from dengue. AJAY MATHUR Miscellaneous D. Salmonellae produce acid on glucose fermentation. 1274 A. Enterobacteriaceae family The infectious dose of Salmonella infections is 10 3 . 704 Most Salmonella serotypes are named for the city where they were identified. Gram negative Currently. 1274 C.3 by 0. None of the above dengue viruses in A. Typhoid Harrison’s 18th Ed. 103 to 106 colony-forming units Harrison’s 18th Ed. there is no evidence in favor of the use of any specific therapeutic agent for dengue. 3 to 4 days Somatic O antigen is related to lipopolysaccharide (LPS) cell-wall components. A rise in the hematocrit of 20% indicates 826 H antigen of Salmonella is related to ? considerable plasma loss. 106 to 108 colony-forming units A. None of the above Harrison’s 18th Ed. M-cells C.oral prednisolone.6 µm.366:1423-32 Salmonellae are gram negative. 1274 A. None of the above D. Ehrlichia . Surface antigen C. aegypti mosquito C. Rise in PCV of 20 % indicates considerable plasma loss C. The laboratory where they were identified Harrison’s 16th Ed.106 colony-forming units. Intramuscular injections are avoided B. non-spore forming. All of the above C. while the H antigen D. Lipopolysaccharide (LPS) cell-wall components B. A. The person who identified them In small intestine. suggesting the possibility of induction of widespread biologic resistance to D. 1274 820 Incubation period of Chikungunya virus is ? A. Emerging technologies include mosquito genetic transformation B. Gram-negative bacilli D.892 MCQ’s FOR MEDICAL PROFESSIONALS BY PROF. facultatively anaerobic bacilli that measure 2 A. 1234 B. Flagellar antigen D. Produce acid on glucose fermentation For mosquito control. salmonellae penetrate mucous layer of gut & traverse intestinal layer through phagocytic microfold (M) cells within Peyer’s patches. Surface antigen A. Flagellar antigen B. Facultatively anaerobic bacilli control ? D.353:924-32 Harrison’s 18th Ed. A. None of the above cells known as ? Harrison’s 18th Ed. 1274 D. 1274 Recent trials have assessed chloroquine. B. A.4 . P-cells Harrison’s 18th Ed. Aspirin and NSAID’s are avoided A. tuberculosis . The city where they were identified C. N-cells 822 Most Salmonella serotypes are named after ? D. choleraesuis (six subspecies) and S. N Engl J Med 2012. Wolbachia. Taxonomic name for organism that causes enteric fever is Salmonella choleraesuis 828 S. Anaplasma C. destruction of breeding sites is generally the most economically and environmentally sound approach. C. Do not produce cytochrome oxidase infected A. The river 829 Which of the following have an ability to survive inside a macrophage ? D. Lipopolysaccharide (LPS) cell-wall components Harrison’s 17th Ed. and balapiravir in patients with confirmed dengue virus infection. aegypti populations. Further trials of statins and other antiviral drugs are planned. aegypti are partially resistant to dengue virus infection and can invade natural A. Wolbachia 824 Which of the following is false about Salmonellae ? Harrison’s 18th Ed. Harrison’s 18th Ed. Neorickettsia. Intra-muscular injections should not be given as they may cause large hematomas. 819 Which of the following statements about management of dengue fever is false ? 825 Somatic O antigen of Salmonella is related to ? N Engl J Med 2005. 4 to 5 days relates to flagella of bacteria. aegypti. aegypti populations. full taxonomic A. Foscarnet 823 Which of the following is false about Salmonellae ? Harrison’s 18th Ed. 109 to 1012 colony-forming units B. bongori. M. C. 1274 Genus of gram-negative bacilli within Enterobacteriaceae family consists of S. The strategy involves embryonic introduction of strains of the obligate intracellular bacterium wolbachia into A. B. 827 Infectious dose of Salmonella varies from ? Chapter 153. 1 to 2 days C.0. B. reduce nitrates & do not produce cytochrome oxidase. B. 102 to 103 colony-forming units 821 Which of the following is false about ‘Salmonellae’ ? B. 1274 A. Non-spore forming 818 Which of the following is best related to A. 2 to 3 days D. According to current Salmonella nomenclature system. Reduce nitrates and introduction of Wolbachia to limit multiplication rates. L-cells designation Salmonella enterica subspecies enterica serotype Typhimurium can be shortened to Salmonella serotype Typhimurium or simply Salmonella Typhimurium. All of the above Aspirin & NSAID’s are best avoided as they can increase bleeding. 1620. typhi invades gut mucosa through specialised antigen-sampling D. ampicillin and trimethoprim. 1275 A. S. 1276 Harrison’s 18th Ed. 831 Incubation period for S. typhi is high during ? 6%). vomiting (18%). Toxoplasma gondii D. typhi is higher for all except ? D. infants. All of the above A. 1276 Harrison’s 18th Ed. Rose spots Yield of blood cultures is high (90%) during 1st week of infection & decreases to 50% by 3rd week. Jejunum D. and C. B. Bone marrow B. Difficult to detect in dark-skinned patients Rose spots are a faint. B. Second week A. 1275 A. Trimethoprim D. Abdominal pain C. typhi enters the body through which part of the gut ? A. 1276 fever. B. stool or intestinal secretions is required. Hepatosplenomegaly Physical findings in enteric fever include coated tongue (51-56%). Third week B. All of the above For definitive diagnosis of enteric fever isolation of S. Vomiting 841 Rate of chronic carriers of S. Appears at the end of second week B. rose spots (30%). All of the above D. hepatosplenomegaly (3-6%) & relative bradycardia at peak of high Harrison’s 18th Ed. Men GI symptoms in enteric fever include anorexia (55%). 3 to 7 days Harrison’s 16th Ed. Humans A. diarrhea (22-28%) and constipation (13-16%). Typhi & S. splenomegaly (5. 3 to 28 days C. Rose spots C. 840 Yield of stool cultures for S. All of the above 836 Which of the following is false about Rose spots in enteric 830 Hosts for salmonellae include ? fever ? Harrison’s 18th Ed. Blood A. Multidrug-resistant (MDR) strains of S. First week Harrison’s 18th Ed. bone marrow. Cholelithiasis 835 Which of the following is false about Rose spots in enteric fever ? Harrison’s 18th Ed. Fourth week C. Anorexia Stool cultures become positive during III week of infection in untreated patients. Duodenum C. abdominal pain (30-40%). 1276 enteric fever ? A. 897 B. Colon on inoculum size and host’s health & immune status. Humans are the only hosts of S. Dogs C. Ampicillin C. blanching. abdominal tenderness (4-5%). B. Paratyphi serotypes A. Abdominal tenderness D. D. and persons with biliary abnormalities or concurrent gall bladder infection with Schistosoma haematobium. Second week in enteric fever ? C. 3 to 14 days A. 1276 D. Typhi in enteric fever can be obtained 832 Plasmid-mediated resistance was observed with which of the from ? following drugs ? Harrison’s 18th Ed. 1276 D. rose spots. Typhi or S. 1276 A. 1275 837 S. typhi ranges from ? Harrison’s 18th Ed. Maculopapular rash C. Salmon-colored Chronic carriage of S typhi is more common among women. 3 to 21 days B. All of the above D. typhi C. 839 Yield of blood cultures for S. Patients can have two or three crops of lesions C. A. in ~30% of patients at the end of 1st week & resolves after 2-5 days. Blanching . Third week Harrison’s 18th Ed. 1276 B. nausea (18-24%). salmon-colored. Carcinoma of gall bladder A. AJAY MATHUR Cardiology 893 B. Gall bladder infection with S. Chloramphenicol B. Located on face and limbs D. Paratyphi from blood. Difficult to detect in dark skinned patients. 893 Miscellaneous MCQ’s FORMCQ’s FOR MEDICAL MEDICAL PROFESSIONALS PROFESSIONALS BY PROF. 838 Specimens for culture of S. Monkey B. Typhi averages 10-14 days but ranges from 3 to 21 days depending D. First week 834 Which of the following is the commonest gastrointestinal symptom B. Salmonella can be cultured from punch biopsies of lesions D. Terminal ileum The incubation period for S. Typhi contain plasmids encoding resistance to chloramphenicol. Salmonella can be cultured from punch biopsies of these lesions. Coated tongue C. haematobium C. maculopapular rash primarily on trunk & chest. Diarrhea Harrison’s 18th Ed. typhi is high during ? 833 Which of the following is the commonest physical finding in Harrison’s 18th Ed. B. 3. 849 Minimum age for vaccination for Ty21a is ? Harrison’s 18th Ed. Vi is bound to a nontoxic recombinant protein. 6 years B. relapse rate in enteric fever is ? C. 16 weeks D. Whole-cell vaccine A. AJAY MATHUR Miscellaneous 842 Which of the following drugs is ineffective in treating enteric C. 6 months C. Lancet 2005. 366: 749-62 A. ~2% Currently. B. 1278 A. Vi-rEPA D. Whole-cell vaccine C. Whole-cell vaccine B. 6 years A. 1276. Whole-cell vaccine urine for more than a year. 1277 Ty21a is an oral live attenuated S. 1 years duration in chronic carriage of Salmonella ? B. Typhi vaccine (given on days 1. 3 months Minimal age for vaccination for Vi CPS is 2 years. 4 years A. with a booster every 2 years). 4 years Harrison’s 18th Ed.894 MCQ’s FOR MEDICAL PROFESSIONALS BY PROF. Vi CPS is a parenteral vaccine consisting of purified Vi polysaccharide from the bacterial B. HLA-DQB1*0401/2 . Ty21a B. there is no licensed vaccine for paratyphoid fever. 1278 antibiotic. Whole-cell vaccine A. Aminoglycosides acetone-killed whole-cell vaccine is available only for use by the US military. A. 1277 C. B. First generation cephalosporins 5 years). 1278 Lancet 2005. All of the above 848 Which of the following S. Ty21a B. 899 D. 2 years stools for ? C. Vi-rEPA 853 Which of the following S. All of the above 1st & 2nd generation cephalosporins & aminoglycosides are ineffective in treating clinical infections. D. Ty21a 845 In immunocompetent patients. In Vi-rEPA vaccine. 6 weeks D. ViCPS In Vi-rEPA vaccine. Ty21a D. ~ 10 % 852 Vi-rEPA vaccine is best related to which of the following ? D. with a booster every A. 1278 A. Cold urticaria Harrison’s 18th Ed. 1278 Chronic asymptomatic carriers are defined as individuals who excrete S. C. Vi is bound to a nontoxic recombinant protein that is identical to Pseudomonas aeruginosa exotoxin A. & 7. ViCPS fever ? D. 12 weeks Minimal age for vaccination for Ty21a is 6 years. D. 1 years 844 Chronic carriers is defined as excretion of S. Xanthomas 846 Which of the following typhoid vaccines is given orally ? B. 366: 749-62 D. HLA-DQB1*0201-3 B. typhi in their stools or A. 5. Hypergammaglobulinemia C. 1278 A. 24 weeks 850 Minimum age for vaccination for Vi CPS is ? Patients with chronic carriage of Salmonella are treated for 4-6 weeks with an appropriate oral Harrison’s 18th Ed. typhi vaccines has best 3-year cumulative 847 Which of the following is an attenuated S. Ty21a C. > 2 year Harrison’s 18th Ed. The old parenteral whole-cell typhoid/ paratyphoid A and B vaccine is no longer licensed. None of the above A. typhi vaccine ? efficacy ? Harrison’s 18th Ed. Vi-rEPA Harrison’s 18th Ed. typhi in urine or B. Second generation cephalosporins capsule (given in 1 dose. ~5% C. 1278 843 Appropriate oral antibiotic treatment should be given for what A. ~ 20 % Harrison’s 18th Ed. ViCPS D. > 1 year 851 Which of the following vaccines is licensed for paratyphoid fever ? D. An C. 3 years Harrison’s 18th Ed. 1278 Harrison’s 18th Ed. Pseudomonas aeruginosa exotoxin A B. ViCPS C. largely because of significant side effects. typhi vaccines has higher incidence of 854 Which of the following HLA-linked genes is associated with side effects ? protection against complicated typhoid fever ? Harrison’s 18th Ed. ViCPS Harrison’s 16th Ed. 6 weeks. 5599 C. Norwegian Spirochaetales include three genera pathogenic for humans . 859 Chloramphenicol was introduced for the treatment of typhoid 865 Endemic syphilis is also called ? fever in ? Harrison’s 18th Ed. T. typhi O and S.347:1770 A. pallidum A. HLA-DRB1*12 C. T. ~ 7 days A.60 weeks HLADRB1* 0301/6/8. complicated typhoid fever. HLA-DRQB1*04 B. endemicum B. pertenue B. and tumour necrosis factor a (TNFA*2-308) were associated with susceptibility to typhoid fever. Agglutination of H antigens appears as loose or wooly Treponema pallidum causes venereal syphilis. All of the above D.Leptospira (Leptospirosis). Incubation period rarely TNFA*1(-308) were associated with lower risk. typhi O and S. 861 Median incubation period in humans of primary lesion of syphilis 855 Number of predicted genes in a complete DNA sequence of a is ? multidrug resistant S. All of the above D. 1380 N Engl J Med 2002. Leptospira A. German B.24 weeks HLA-linked genes play an important role in determining susceptibility or resistance to typhoid. pallidum A. French C. 4599 B. 1948 B. 6 . Algeria and died in Paris on (Relapsing fever & Lyme disease) and Treponema (Treponematoses). Chloramphenicol was introduced for the treatment of typhoid fever in 1948. 863 Which Treponema subspecies causes ‘Yaws’ ? 857 Salmonella typhi strain used to prepare S. T. 895 Miscellaneous MCQ’s FORMCQ’s FOR MEDICAL MEDICAL PROFESSIONALS PROFESSIONALS BY PROF. T. Treponema C. 864 Which Treponema subspecies causes ‘Pinta’ ? Harrison’s 18th Ed. Syphilis Harrison’s 18th Ed. 6599 D. British D. 1961 D. 862 Spirochaetale that is pathogenic for humans is ? Harrison’s 18th Ed. T. None of the above pallidum subspecies endemicum causes endemic syphilis or bejel. 1380 858 Which of the following about Widal test is false ? A. 1380 antigens is ? A. 1929) was a French physician. T. Number of predicted genes in a complete DNA sequence of a multidrug resistant S typhi isolate CT18 is 4599. 12 . Highest serum dilution producing positive agglutination is B. D. 24 . 1001 Salmonella typhi strain used to prepare S.6 weeks .12 weeks D. Bejel A. 801 C. pertenue taken as titre C. HLA-DQB1*0201-3. njovera or skerljevo and is caused by T. Borrelia B. ~ 21 days C. Agglutination of O antigen appears as “matt” or “carpet” D. and HLA-DRQB1*04 and HLA-DQB1*0401/2 and After an incubation period averaging 2 . typhi isolate CT18 is ? Harrison’s 18th Ed. pallidum subspecies endemicum. Borrelia Georges-Fernand-Isidor Widal. 1966 Endemic syphilis is also called bejel. 1862 in Dellys. stf 860 Primary lesion of syphilis appear how many weeks after infection ? B. mof A. T. ~ 28 days D. tpr Harrison’s 18th Ed. 7599 The median incubation period in humans is ~21 days. 1954 C. typhi H antigens is 901. Siti B. 1380 A. 1380 C. 701 B. January 14. Dichuchwa C. endemicum C. typhi H Harrison’s 18th Ed. 866 PCR amplification of which gene can differentiate etiologic agents of venereal syphilis. yaws and bejel ? Chapter 169. a primary lesion appears. 2 . dichuchwa. T. T. HLA-DRB1*12 is associated with protection against exceeds 6 weeks. T. AJAY MATHUR Cardiology 895 C. (born on March 9. pallidum subspecies pertenue causes yaws. ~ 14 days B. 1380 856 The nationality of Georges-Fernand-Isidor Widal was ? A. carateum D. 366: 749-62 A. D. 1381 Lancet 2005. siti. 901 D. carateum causes pinta. carateum C. T. Begins as a single painless papule Harrison’s 18th Ed. B. 1039 C.896 MCQ’s FOR MEDICAL PROFESSIONALS BY PROF. Recurrent genital herpes B. C. 1381 The most common cause of genital ulceration in most developing countries is genital herpes. 1381 Molecular signatures that can differentiate the causative agents of venereal syphilis. Rabbits 874 Mucocutaneous manifestations of secondary syphilis appear how D. But the accompanying lymphadenopathy may persist for months. pallidum are grown in ? C. 50% infectious dose for intradermal inoculation in humans is 57 organisms. AJAY MATHUR Miscellaneous D. 1381 B. pallidum are grown in rabbits. Chancroid Harrison’s 18th Ed. A. pallidum can penetrate intact mucous membranes D.1000. None of the above 876 Which of the following is not a presentation of tertiary syphilis ? Harrison’s 18th Ed. 4 . yaws & bejel have been identified by polymerase chain reaction amplification of tpr genes & restriction A.6 weeks 868 The most common cause of genital ulceration in most developing B.24 weeks B. Cardiovascular syphilis B. Genital herpes 875 Secondary lesions subside within what time ? D. Appears within 1 week of lesion onset . Symptomatic neurosyphilis C.16 weeks A. T. 500 . C. ~ 16 . 1381 B.8 weeks countries is ? Harrison’s 17th Ed. 357 organisms syphilis (ascending aorta aneurysm) & symptomatic neurosyphilis (tabes dorsalis and paresis). 1381 C. 12 .6 weeks 867 Virulent strains of T.18 weeks A.12 weeks 869 Which of the following about T. Blood from patient of incubating syphilis is infectious Secondary lesions subside within 2 . 257 organisms Most common types of tertiary disease are gumma (benign granulomatous lesion). Gumma A. 6 .4 weeks digestion. 877 Which of the following about primary chancre is false ? Harrison’s 18th Ed. 57 organisms C. 1381 A. C.8 weeks after chancre heals. Cow many weeks after the chancre heals ? Harrison’s 18th Ed. A. 500000 . 2 .100000 879 Which of the following is false about inguinal lymph-adenopathy D. Multiple primary lesions more common among men with B. 6 . A. Chancroid D. 1382 The average inoculum of infectious organisms for naturally acquired syphilis is 500 . 106 / gram concurrent HIV infection C.1000 D. xqr 873 Primary lesion at the site of inoculation usually persists for ? Harrison’s 18th Ed.6 weeks B.6 weeks and then heals B. ~ 3 . 1382 871 What is the treponeme concentration in tissues before a clinical lesion appears ? A. 5000 . Incubation period rarely exceeds 6 weeks D.10000 Donovanosis is usually seen as a granulomatous ulcer that is painless but friable. 2 . cardiovascular D. ~ 6 . Horse spontaneously. Herpes simplex virus infection 872 What is the average inoculum of infectious organisms for naturally B. A. 18 . Lymphogranuloma venereum (LGV) Harrison’s 18th Ed. 107 / gram D.1000000 that accompanies primary syphilitic lesion ? Harrison’s 18th Ed.6 weeks and the infection enters a latent stage which is detectable only by serologic testing. 50000 . 1382 Treponeme concentration in tissues of 10 7/gram produces a a clinical lesion. 12 . 10 / gram 8 878 Which of the following genital lesion is painless ? Harrison’s 18th Ed. 1380 D.12 weeks Harrison’s 18th Ed. 105 / gram C.28 weeks B. ~ 10 . C. pallidum is false ? Harrison’s 18th Ed. Mice Primary lesion at the site of inoculation usually persists for 4 . Donovanosis acquired syphilis ? C. 157 organisms D. None of the above D. Syphilis Mucocutaneous manifestations of secondary syphilis usually appear ~6 . 1381 Virulent strains of T. Cartilaginous consistency on palpation A. 1381 870 50% infectious dose for intradermal inoculation in humans is ? A.16 weeks A. Condylomata lata Harrison’s 18th Ed. 20 years clinical features with secondary syphilis ? 890 Which of the following comes last in the clinical spectrum of Harrison’s 18th Ed. 5 . painless and bilateral. Psoriasis Harrison’s 18th Ed. General paresis plaques with trailing scale. Meningeal syphilis C. 1383 B. C. 405 symptomatic neurosyphilis ? A. with normal CSF examination & absence of clinical manifestations of syphilis indicate a diagnosis of latent syphilis. Involve oral or genital mucosa 889 Meningovascular syphilis manifests at what time after after infection ? C. optic neuritis and the classic B. B. None of the above A. Alopecia Harrison’s 18th Ed. Lichen planus A. is firm. Painful Harrison’s 18th Ed. are painless silver-gray erosion surrounded by a red periphery. poorly circumscribed patches of alopecia with a “moth-eaten” appearance are a A.10 years for meningovascular syphilis. None of the above syphilis ? Harrison’s 18th Ed. Pupillary abnormalities B. Unusually high serum alkaline phosphatase C. Optic atrophy occurs in tabes. 15 . Initial lesions are nonpruritic macules A. More than one form is present simultaneously 887 Which of the following is false about hepatic involvement in D. C. Silver-gray erosion surrounded by red periphery A. B. 1382 Skin rash of secondary syphilis consists of macular. onset of symptoms are <1 year after infection for meningeal syphilis. 1383 D. A.15 years 884 Which of the following papulosquamous disease shares many D. No cholestasis 882 Which of the following is a highly infectious lesion in syphilis ? Harrison’s 18th Ed. Tabes dorsalis Clinical features of Pityriasis rosea like rash often preceded by herald patch. Cheese spread appearance of lesion onset. Weight loss C. Positive serologic tests for syphilis In secondary syphilis. at 20 years for general paresis & at 25 . vulva. poorly circumscribed patches of alopecia seen in C. 897 Miscellaneous MCQ’s FORMCQ’s FOR MEDICAL MEDICAL PROFESSIONALS PROFESSIONALS BY PROF. Iritis or uveitis Harrison’s 18th Ed. Absence of clinical manifestations of syphilis 883 Which of the following is false about mucous patches in secondary syphilis ? D. affecting trunk and eruption lines up in skin folds giving a “fir tree” like appearance shares many clinical features with secondary syphilis in which palms & soles In symptomatic neurosyphilis. Dermatophytosis C. oval to round D. Superficial mucosal erosions B. Unusually high serum alkaline phosphatase without cholestasis with some hepatocellular damage are common. Wild pluck appearance Harrison’s 18th Ed. Usually asymptomatic on the trunk & proximal extremities which progress to papular lesions that involve palms & soles. Optic neuritis 881 Which of the following is false about skin rash of secondary syphilis ? C. Eye lesion manifestation of secondary stage of syphilis. 1382 D. Moth eaten appearance Inguinal lymphadenopathy that accompanies the primary syphilitic lesion appears within 1 week B. 1382 Positive serologic tests for syphilis. 1382 D. Retinitis A.10 years Superficial mucosal erosions or mucous patches of secondary syphilis involve oral or genital mucosa. Chancre Hepatic involvement is common in syphilis though asymptomatic. and scrotum. papulosquamous. All of the above Harrison’s 18th Ed. Mucous patch A. Pityriasis rosea B. Salt and pepper appearance 880 Lues maligna refers to ? D. Papular lesions involve palms & soles iritis or uveitis. 1382 Severe necrotic skin lesions of secondary syphilis are called lues maligna. 1382 Scattered. Meningovascular syphilis D. 10 . at 5 are spared. 407 D. nonsuppurative. B. Nonsuppurative secondary stage of syphilis is termed as ? Harrison’s 18th Ed. Lues maligna C.30 years for tabes dorsalis. Condylomata lata 888 Diagnosis of latent syphilis is established by ? Harrison’s 18th Ed. Normal CSF examination C. Nonpruritic macules initially Ocular findings in secondary syphilis include pupillary abnormalities. 1 . AJAY MATHUR Cardiology 897 B. . papular. . Skin lesion 886 Which of the following ocular finding is classic of secondary syphilis ? D.5 years B. papules enlarge to form highly infectious lesions called condylomata lata in perianal region. More than one form is present simultaneously. Painless A. C. and pustular syphilides. B. A. Unilateral 885 Scattered. 1383 D. Stroke in MCA territory of a young adult B. 2595 B. vomiting. cranial nerve involvement. Decrease in recent memory B. Dorsal roots C. nausea. Syphilitic aneurysms are saccular & do not lead to dissection delusions. impaired position & vibratory sense. 1383 A. Areflexia A. Cardinal signs of tabes are loss of reflexes in legs. 1383 901 Which of the following is false about syphilitic aneurysms ? A. Painful sensory neuropathy Harrison’s 18th Ed. ~90% of syphilitic aneurysms are located in ascending aorta or aortic paresis ? arch. Occurs in 15 . Acute abdominal pain C. and speech. 1383 899 Visceral crisis in tabes refers to ? A. These cardiovascular manifestations are due to endarteritis obliterans of vasa vasorum. 1384 A. ataxic wide-based gait & footslap. Illusions. Subacute encephalitic prodrome C. seizures. affect. Changes in mental status C. 895 Stroke in meningovascular syphilis is preceded by ? Harrison’s 18th Ed. Syphilitic periaortitis & mesoaortitis damage elastic fibers with 896 Which of the following is not a manifestation of general weakening of aortic wall. judgment. Acute vomiting C. Meningeal syphilis A. AJAY MATHUR Miscellaneous 891 Patients presenting with uveitis or iritis frequently have ? 897 Which of the following structures is involved in tabes dorsalis ? Harrison’s 18th Ed. hallucinations). Loss of position sensation C. Endarteritis obliterans of vasa vasorum C. Meningovascular syphilis B. D. Secondary syphilis A.898 MCQ’s FOR MEDICAL PROFESSIONALS BY PROF. insomnia & psychological abnormalities. Due to endarteritis obliterans of vasa vasorum subacute encephalitic prodrome consisting of headache. Illusions. and insight). Argyll Robertson pupil C. involve ascending aorta and do not lead to dissection. Tabes dorsalis C. 2595 B. 1384 B. All of the above cerebral artery in a relatively young adult. 1384 B. Harrison’s 18th Ed. neck stiffness. Acute abdominal pain with vomiting Meningeal syphilis may be concurrent with or may follow secondary syphilis. Romberg’s sign Most common presentation of meningovascular syphilis is stroke syndrome involving middle D. Cranial nerve involvement A. delusions. All of the above D. Tabes dorsalis is a late manifestation of syphilis with presentation due to demyelination of posterior columns. Seizures B. eye ( Argyll Robertson pupils). delusions. Tertiary syphilis B. and changes in mental status. Romberg’s sign and. paresthesia. dorsal roots & dorsal root ganglia. Skin eruption B. Frequently lead to dissection Gradually progressive vascular syndrome (stroke) in meningovascular syphilis is preceded by a D. Tabes dorsalis is a late manifestation of syphilis that presents as Argyll Robertson pupil. hallucinations C. bilateral Argyll Robertson pupils (fail to constrict to light but accommodate). Loss of reflexes in legs C. Areflexia D. 1383 900 Cardinal sign of tabes is ? A. deep pain & temperature sensations. Visual disturbances A. 894 Most common presentation of meningovascular syphilis is ? Harrison’s 18th Ed. calculations. Symptoms usually appear 10 to 40 years after infection D. General paresis D. intellect (a decrease in recent memory and in the capacity for orientation. Involves ascending aorta D. Dorsal root ganglia D. which supply blood to large vessels. All of the above Meningeal syphilis may may present with headache. reflexes (hyperactive). Ataxic wide-based gait B.30% of patients. Argyll Robertson pupils 902 Which of the following about cardiovascular syphilis is false ? Harrison’s 18th Ed. sensorium (illusions. Radiological sign of cardiovascular syphilis is appearance of linear calcification of ascending aorta. bladder disturbances. vertigo. D. 892 Meningeal syphilis may present with ? Harrison’s 18th Ed. impotence. 1383 Harrison’s 18th Ed. Charcot’s joints & perforating ulceration of feet. . Argyll Robertson pupils Harrison’s 17th Ed. Primary syphilis Harrison’s 17th Ed. Linear calcification of descending aorta on chest x-ray Manifestations of general paresis reflect widespread late parenchymal damage that include personality. 1384 A. Saccular aneurysm C. 893 Meningeal syphilis may be concurrent with or may follow ? Harrison’s 18th Ed. Any of the above C. All of the above Patients presenting with uveitis or iritis frequently have meningeal syphilis. hallucinations A. Impaired position & vibratory sense D. Posterior columns B. 1383 898 Which of the following is a manifestation of Tabes dorsalis ? Harrison’s 18th Ed. areflexia & loss of position. Syphilitic aneurysms are usually saccular. Any of the above Acute abdominal pain with vomiting in tabes is termed as visceral crisis. Before 16th week of pregnancy A.40 years after infection are uncomplicated aortitis. “Saber” molars D. osteitis and A. and leukocytosis. Petechiae C. Crusting of the lips D. Bald tongue Harrison’s 18th Ed. Before 24th week of pregnancy B. Aortic stenosis A. saccular aneurysm of ascending aorta and coronary ostial stenosis. AJAY MATHUR Cardiology 899 903 Which is the commonest valvular lesion in syphilis ? 909 In syphilis. 1384. Hairy tongue A. 905 Which is the earliest sign of congenital syphilis ? 911 In congenital syphilis. Loss of eyebrows B. Peg-shaped Adequate treatment of the mother before 16th week of pregnancy prevents fetal damage & treatment Features of Hutchinson’s teeth are central notching. Centrally notched B. anemia (34%). 912 Which of the following is a feature of syphilis ? Harrison’s 18th Ed. Bilateral knee effusions are known as Clutton’s joints. Clutton’s joint is a late manifestation of aortic regurgitation. Nano shins C. 899 Miscellaneous MCQ’s FORMCQ’s FOR MEDICAL MEDICAL PROFESSIONALS PROFESSIONALS BY PROF. Frontal bossing periostitis. congenital syphilis that typically develops between 8 & 15 years of age. Bilateral knee effusions D. Anemia Harrison’s 16th Ed. multiple. adequate treatment of syphilis mother could prevent 910 Which of the following is false for upper central incisors in syphilis congenital syphilis ? (Hutchinson’s teeth) ? Harrison’s 18th Ed. 1384. Bone changes C. Hepatosplenomegaly D. thrombocytopenia. 981 Most common early manifestations are bone changes (61%) including osteochondritis. Lower central incisors D. Pes cavus C. Severe hepatitis B. 1384 Harrison’s 18th Ed. C. the affected molars are called “mulberry” molars (sixth-year molars with followed by other mucocutaneous lesions which include bullae (syphilitic pemphigus). ‘Rhagades’ refers to ? 908 Which of the following is a feature of syphilis ? Harrison’s 16th Ed. jaundice (30%). mucous patches & condylomata lata. Lymphadenopathy 913 Abnormal facies in congenital syphilis include all except ? D. superficial desquamation. vesicles. Cooper shins Neonatal death is usually due to pulmonary hemorrhage. or “snuffles” which is soon In congenital syphilis. Papulosquamous lesions D. Rhinitis A. 1384 Harrison’s 18th Ed. pallidum is detectable in 22% of infected B. Aortic regurgitation B. the affected molars are called ? Harrison’s 18th Ed. All of the above . Pulmonary hemorrhage 914 Anterior tibial bowing in syphilis is called ? Harrison’s 18th Ed. Raspberry tongue C. Saber shins D. 1384 A. Before 28th week of pregnancy C. Widely spaced C. Superficial desquamation B. widely spaced. peg-shaped upper central of mother before third trimester adequately treats the infected fetus. Linear scars at angles of mouth and nose A. Saddle nose neonates. “Mulberry” molars B. Weber shins hepatitis. Clutton’s joints B. Argyll Robertson pupils D. Mitral regurgitation C. Avascular necrosis of hip joint Cardiovascular manifestations that come up 10 . “Saddle” molars C. 268 A. 1384 Harrison’s 18th Ed. Any of the above C. Hutchinson’s teeth C. petechiae. Strawberry tongue B. Clutton’s joints refer to ? Harrison’s 18th Ed. Secondary bacterial infection A. Small pinna of ear Harrison’s 18th Ed. poorly developed cusps). 2846 A. incisors. 1384 B. 273 Table 32-4 906 Which out of the following is the most common early manifestation of congenital syphilis ? A. or severe D. Frozen shoulder B. 904 Till what time. 1384. 1384 A. secondary bacterial infection. Before 32nd week of pregnancy D.6 weeks after birth) is usually rhinitis. 981 Harrison’s 18th Ed. All of the above Earliest sign of congenital syphilis (2 . Tricuspid regurgitation D. 1384 B. 268 A. CNS invasion by T. hepatosplenomegaly (50%). and papulosquamous lesions. 915 In syphilis. Poorly developed maxillae 907 Neonatal death in congenital syphilis is usually due to ? D. Small pinna of ear C. lymphadenopathy (32%). All of the above D. pallidum ? Harrison’s 18th Ed. 90% of those with secondary syphilis. Azithromycin A. All of the above D.900 MCQ’s FOR MEDICAL PROFESSIONALS BY PROF. CSF pleocytosis C. Unusually high serum alkaline phosphatase inhibit T. Cholestasis following medicines can be used ? D. Aminoglycosides & spectinomycin A. 984 919 ‘Prozone phenomenon’ is best related to ? Harrison’s 18th Ed. Parenteral drug use A. pinta and endemic syphilis. pallidum (MHA-TP) D. Early latent syphilis B. Harrison’s 18th Ed. Cephalosporins Harrison’s 16th Ed. 1388 A. 982 A. Elevated levels of CSF protein A. They measure IgG & IgM against 925 Which of the following tests is not useful in following the response cardiolipin-lecithin-cholesterol antigen complex. provides the most T. AJAY MATHUR Miscellaneous 916 Which of the following tests is not useful in detection of T. 1386 C. 1384 A. 981 Sulfonamides and quinolones are inactive against T. None of the above C. 1387 . Ceftriaxone Harrison’s 18th Ed. D. Typhoid B. Quantitative RPR titer B. CSF VDRL titer B. sensitive index of response to treatment of neurosyphilis ? Harrison’s 18th Ed. 1384 C. Harrison’s 16th Ed. 1385 A. Fluorescent treponemal antibody-absorbed (FTA-ABS) test C. Rapid plasmin reagin (RPR test) Jarisch-Herxheimer reaction occurs in ~50% of patients with primary syphilis. D. Histologically different form viral hepatitis 922 For penicillin-allergic patients with early syphilis. Tetracyclines D. to therapy in patients of syphilis ? Harrison’s 16th Ed. Microhemagglutination assay for T. All of the above B. 1386 B. Endemic syphilis D. 1387 917 Serologic tests for syphilis are reactive in ? Harrison’s 18th Ed. Secondary syphilis A. Immunofluorescence or immunohistochemical methods Ceftriaxone & Azithromycin are effective for early syphilis. Cephalosporins Harrison’s 16th Ed. Quinolones D. doxycycline or tetracycline is recommended. Serodia TP-PA test Nontreponemal antibody tests for syphilis are RPR & VDRL tests. and a lower proportion of persons with later-stage disease. Serum pH D. C. 982 A. All of the above 924 Jarisch-Herxheimer Reaction is most common following treatment Both nontreponemal & treponemal serologic test for syphilis are reactive in any treponemal infection of ? including yaws. Malaria 921 Which of the following drugs is inactive against T. Genital herpes Harrison’s 18th Ed. Fluorescein-conjugated polyclonal antitreponemal antibody 923 Which of the following in CSF examination. FTA-ABS and agglutination tests C. A. Sulfonamides 928 Which of the following is false about syphilitic hepatitis ? D. pallidum cannot be detected by culture. which of the C. Serum dilution B. Recent trauma B. Doxycycline pallidum ? B. Primary syphilis 918 Which of the following is not a treponemal test ? Harrison’s 18th Ed. Pinta C. Yaws B. None of the above Harrison’s 18th Ed. pallidum. HIV infection A. Spectinomycin C. Erythromycin nontreponemal serologic tests for syphilis ? D. Recent viral illness or immunization 920 Which of the following drugs is ineffective in the treatment of acquired syphilis ? B. Dark-field microscopic examination For penicillin-allergic patients with syphilis. pallidum only in very large doses B. Serum temperature C. Aging C. Culture D. 1385 B. Serum colour 926 Which of the following is not a cause of False-Positive Reactions VDRL test that is nonreactive or weakly reactive with undiluted serum but is positive at higher in Nontreponemal Serologic Tests for Syphilis ? serum dilutions is called prozone phenomenon. Quantitative VDRL titer A. Tetracyclines 927 Which of the following is not a cause of false-positive reactions in C. 1393 disease . 1048 A.multiple sclerosis (MS). Multiple sclerosis (MS) Leptospires traverse the interstitial spaces of kidney.10 days of clinical illness. Veterinarians C.30 days). 2 to 30 days A. 1395 A. Pancreas 934 The incubation period of leptospirosis is in the range of ? Harrison’s 18th Ed. Oral mucosa C. All of the above C. Rats 939 In kidney. Epidermis D. D. Sewage workers 931 Which of the following about Leptospires is false ? D. Urinary epithelium A. 1393 B. cattle. Birds B. 2 to 40 days B. CNS syphilis & vitamin B 12 deficiency. Wall of small blood vessels 932 The most important reservoir of Leptospires is ? Harrison’s 18th Ed. Most important sources of transmission to humans are rats. Direct contact with tissue D. Borrelia A. Eye D. Medical Laboratory technicians Harrison’s 18th Ed. Conjunctival mucosa A. Direct contact with urine B. Coiled. A. 1393 D. All of the above 936 Transmission of leptospires is rare through ? Combination of optic neuritis & spastic ataxic paraparesis suggest optic nerve & spinal cord Harrison’s 18th Ed. 1393 Harrison’s 18th Ed. Can be seen by dark-field microscopic examination A. 901 Miscellaneous MCQ’s FORMCQ’s FOR MEDICAL MEDICAL PROFESSIONALS PROFESSIONALS BY PROF. Leptospirosis C. Cat Harrison’s 18th Ed. Leptospira 937 Occupational groups at high risk of leptospirosis are all except ? Harrison’s 18th Ed. 942 Which of the following is conspicuous during immune phase of 935 In the host. 2484 D. thin. Direct contact with blood Chapter 171. Slaughterhouse workers D. Intestinal flora Harrison’s 17th Ed. 1393 941 Even when antibodies are formed. Leptospires can be isolated from D. and pigs. AJAY MATHUR Cardiology 901 929 Combination of optic neuritis & spastic ataxic paraparesis suggests B. D. Eye C. leptospires persist for weeks / months at all sites in host except ? A. Glomerulonephritis Leptospirosis is a zoonotic disease. Stain poorly Harrison’s 17th Ed. Kidney Leptospires infect humans usually through conjunctival mucosa and possibly oral or tonsillar C. Leptospires persist in ? leptospirosis ? Harrison’s 18th Ed. C. All of the above 933 Leptospires infect humans usually through ? In leptospirosis. 1394 C. Saliva A. None of the above B. Treponema B. Nails Harrison’s 17th Ed. 1393 B. Hair follicle B. Renal tubules A. highly motile organisms with hooked ends 938 All forms of leptospires damage ? B. acute & chronic inflammation within kidney is associated with acute tubular Harrison’s 18th Ed. Interstitial nephritis D. Intact skin 940 The organ invariably involved in leptospirosis is ? B. 1393 necrosis & interstitial nephritis. Brain The incubation period averages 5 . Perinephritis dogs. or abraded skin. Muscle blood during the first 3 . Hepatic ducts which of the following ? C. and become adherent to proximal C. from where they are excreted in the urine. Vitamin B12 deficiency renal tubular brush border. A. penetrate basement membrane of proximal B. punctured. Fish A. Liver mucosa or through macerated. Human-to-human transmission 930 Which of the following is Spirochaetale ? Human-to-human transmission does not occur.14 days (range 2 . cross through proximal renal tubule epithelial cells. 2 to 50 days C. D. Jarisch-Herxheimer reaction . 1392 A. Proximal renal tubules D. Harrison’s 18th Ed. 1049 C. 2 to 20 days Harrison’s 18th Ed. leptospires causes ? B. CNS syphilis renal tubules. 1049 C. Scarlet fever A. Peripheral neuropathy B. Serum lactic dehydrogenase Harrison’s 17th Ed. Korthoff medium A. Serum bilirubin D. Patchy alveolar pattern D. 1050 946 Temperature-pulse dissociation (relative bradycardia) occurs A. 1395 C. Viral hepatitis Harrison’s 17th Ed. Cephalosporin B. Hemorrhagic diathesis medium is useful ? Harrison’s 18th Ed. All of the above B. AJAY MATHUR Miscellaneous B. 1050 B. Ellinghausen-McCullough-Johnson-Harris (EMJH) medium 944 The most common clinical finding in leptospirosis is ? B. Jaundice B. Middle lobes 956 Accentuated petechiae in body folds (Pastia’s lines) is D. Fourth week of illness . Doxycycline Harrison’s 17th Ed. All of the above A. Upper lobes D. 1396 Table 171-1 947 Most common radiographic finding in severe leptospirosis is ? A. First week of illness B. 1395 B. 17 days Harrison’s 18th Ed. Typhoid fever D. 1050 C. Lower lobes C. Dengue A. 1050 C. Malaria in ? B. Third week of illness D. Second week of illness C. Erythromycin B. Serum alkaline phosphatase C. 24 days A. Hantavirus infection B. Cotrimoxazole C. Renal dysfunction 951 For isolation of leptospires from body fluids or tissues. Indirect hemagglutination test Harrison’s 18th Ed. Hemolytic anemia 950 Tests with diagnostic value in leptospirosis include ? Harrison’s 17th Ed. 1394 D. Muscle tenderness C. IV penicillin G Harrison’s 18th Ed. 1393 D. Lymphadenopathy 952 Leptospires do not remain viable in anticoagulated blood after ? D. Skin rash Harrison’s 18th Ed.902 MCQ’s FOR MEDICAL PROFESSIONALS BY PROF. Kawasaki disease B. Streptococcal toxic shock syndrome C. antibodies are generally detectable by ? Harrison’s 18th Ed. Aseptic meningitis A. which C. 1174 949 In leptospirosis. All of the above Harrison’s 18th Ed. 1395 945 Test that helps to differentiate leptospirosis from viral hepatitis A. Staphylococcal toxic shock syndrome D. ELISA 943 Severe Leptospirosis (Weil’s Syndrome) is characterized by ? C. 1396 Table 171-1 948 Radiographic abnormalities in severe leptospirosis most often affect ? A. Erythromycin A. Tetracycline A. Hilar lymphadenopathy 955 Drug indicated for chemoprophylaxis against leptospirosis D. Brucellosis C. 27 days B. 1394 A. 11 days is ? B. Pleural effusion C. Microscopic agglutination test (MAT) D. Fletcher medium Harrison’s 18th Ed. Leptospirosis 954 Which of the following drugs is not effective in severe cases of leptospirosis ? D. Ampicillin C. Serum creatine phosphokinase 953 Which of the following illness has a strong similarity in epidemiology and clinical presentation with leptospirosis ? D. All of the above A. 1394 C. Fever with conjunctival suffusion D. All of the above is ? Harrison’s 18th Ed. All of the above characteristic of ? Harrison’s 18th Ed. 45 A. III and IV B. Increased vascularity B. 961 Age of onset of RA is most frequent during which decade of life ? 967 Earliest lesion in rheumatoid synovitis is ? Harrison’s 17th Ed. after which it plateaus until the age of 75 and then decreases. Bone C. PTPN22 A. Cartilage damage C. 2084 A.344:907 risk for other autoimmune diseases ? Harrison’s 17th Ed. and HLA- DR7 (DR 1*0701) may protect against the development of RA. Involves peripheral joints in a symmetric distribution D. AJAY MATHUR Cardiology 903 963 Women are affected by RA approximately how many times more Chapter 321. 2083 957 Rheumatoid arthritis (RA) is a chronic disease of ? A. Four times B. involving peripheral joints in a symmetric distribution. D. 2083 B. Cartilage damage & bone erosions Class II major histocompatibility complex allele HLA-DR4 (DR  1*0401) is a major genetic risk D. All of the above D. 75 D. 2084 A. 2083 B. 2083 Harrison’s 17th Ed. None of the above factors for RA. HLA-DR2 (DR 1*1501). Multisystem 964 Which of the following is a major genetic risk factor for RA ? Rheumatoid arthritis (RA) is a chronic multisystem disease of unknown cause. Except for PADI4. Harrison’s 18th Ed.CD4+ T cells predominate over CD8+ T cells. 960 Synovial membrane in rheumatoid arthritis is characterized 966 Which of the following susceptibility genes for RA does not convey by ? N Engl J Med 2001. HLA-DR5 (DR1*1101) C. PADI4 (enzyme involved in conversion of citrulline to The potential of the synovial inflammation to is the hallmark of the disease. 2084 A. FcRL3 C. and CTLA4 (a molecule involved in regulation of T cell activation) are regarded as susceptibility genes for RA. 962 The incidence of RA decreases after the age of ? Harrison’s 18th Ed. increased vascularity & infiltrate of inflammatory cells (CD4+ T cells). Joints D. T lymphocyte B. Increase in size of synovial lining cells The age of onset of RA is most frequent during fourth and fifth decades of life. 2742 A. Five times C. V and VI D. FcRL3 B. Microvascular injury & an increase in the number of synovial lining cells are the earliest lesions in rheumatoid synovitis. B lymphocyte C. Three times A. CTLA4 D. Persistent inflammatory synovitis C. 965 Which of the following is regarded as susceptibility gene for RA ? Harrison’s 17th Ed. 2738 968 Predominant infiltrating cell in RA is ? Harrison’s 17th Ed. CTLA4 C. these genes also appear to convey risk for other autoimmune diseases. All of the above D. 2741 958 Which of the following is false about established RA ? A. Dendritic cells The incidence of RA increases between 25 and 55 years of age. II and III A. HLA-DR2 (DR1*1501) Harrison’s 17th Ed. Hyperplasia A. 55 B. Predominant infiltrating cell in RA is T lymphocyte . bone erosions & disturbing joint integrity. Two times Harrison’s 17th Ed. . Muscle Women are affected by RA ~three times more often than men. HLA-DR4 (DR1*0401) B. PTPN22 B. Bone erosions D. arginine in proteins). 65 C. Increase in number of synovial lining cells C. Characteristic feature of established RA is persistent inflammatory synovitis causing cartilage damage. 903 Miscellaneous MCQ’s FORMCQ’s FOR MEDICAL MEDICAL PROFESSIONALS PROFESSIONALS BY PROF. HLA-DR3 (DR1*0301) A. Macrophages D. HLA-DR3 (DR 1*0301). Synovial inflammation B. Infiltrate of inflammatory cells C. 2084 959 The hallmark of rheumatoid arthritis is ? Harrison’s 18th Ed. Decrease in size of synovial lining cells D. Decrease in number of synovial lining cells B. IV and V C. HLA-DR5 (DR 1*1101). Systemic manifestations PTPN22 (a phosphatase involved in antigen receptor signaling in lymphocytes). FcRL3 (a molecule involved in regulating B cell activation). PADI4 Synovial membrane in rheumatoid arthritis is characterized by hyperplasia. Rheumatoid Arthritis often than men ? Harrison’s 17th Ed. 972 Which of the following about rheumatoid arthritis is false ? 979 Interleukin-4 inhibits the production of ? Harrison’s 17th Ed. TNF  970 Enzymes like collagenase & cathepsins are produced by ? D.344:907 D. an inflammatory cytokine. Hallmark is synovial inflammation C. Interleukin-1 B. 2084 on immune. All of the above B. A.73 & 74:71-82 C.344:907 D. synovial proliferation & cartilage & bone damage as well as systemic D. Type 1 TNF receptor (p65) & type 2 TNF receptor (p85) . activity of chemokines & cytokines result in synovial tissue inflammation.350:2167-79 1 & TNF-. CD4+ memory T cells & CD8+ T cells express early activation antigen CD69. Type 1 TNF receptor (p50) & type 2 TNF receptor (p70) C. Synovial fluid inflammation A. Systemic manifestations of RA B. Interleukin-10 In rheumatoid synovitis. Interleukin-8 N Engl J Med 2004. Adalimumab A. interleukin-6 & TNF are key cytokines that drive inflammation in rheumatoid arthritis. Interleukin-1 B. Interleukin-1 D. Interleukin-10 and interleukin-4 are antiinflammatory cytokines. CD4+ and CD8+ T cells express the TNF. type 1 TNF receptor (p55) & type 2 TNF receptor (p75). A. Interleukin-4 inhibits production of interleukin-6 & interleukin-8. Activated synovial fibroblasts 977 Rheumatoid factor is produced by ? C. C. TNF  manifestations of RA. synovial fibroblasts are activated to produce collagenase & cathepsins that degrade components of articular matrix particularly in the lining layer & at the interface with bone & cartilage. CD59 arthritis ? C. Immune mediated disease A. Dendritic cell 971 Activity of chemokines & cytokines result in which of the features of rheumatoid synovitis ? Activated CD4+ T cells stimulate B cells to produce immunoglobulins. All of the above British Medical Bulletin 2005. B cell B. Etanercept 980 Which of the following is a chemokine ? B. 2084 N Engl J Med 2001. Interleukin-8 D.350:2167-79 B. Cytopathic T-lymphocyte-associated antigen-4 B.344:907 A. Cytopathic T-lymphocyte-associated antibody-4 C. called chemokines. Activated mast cells B. 2084 A. T cell In RA. CD69 N Engl J Med 2001. found Harrison’s 17th Ed. attract inflammatory cells to travel to the synovial joint. Interleukin-6 In rheumatoid synovium. It is a co-stimulatory molecule involved in the interaction between T lymphocyte and antigen-presenting cell. including rheumatoid factor. Activated monocytes Some cytokines. 2084 Interleukin-1. None of the above D. AJAY MATHUR Miscellaneous 969 In rheumatoid synovium. CD79 A. CD49 976 Which of the following cytokines drive inflammation in rheumatoid B. exerts its effects through early activation antigen called ? binding to two cell surface receptors. Osteoclasts A. B. Interleukin-6 974  . Damage initiated by CD4+ T cells B. All of the above 973  blocking drugs used for rheumatoid arthritis are ? TNF- Interleukin-4 inhibits activation of type 1 helper T cells leading to decreased production of interleukin- N Engl J Med 2004.344:907 B. Type 1 TNF receptor (p60) & type 2 TNF receptor (p80) CTLA-4 stands for Cytotoxic T-lymphocyte-associated antigen-4.73 & 74:71-82 A. A. All of the above C.350:2167-79 D. Type 1 TNF receptor (p55) & type 2 TNF receptor (p75) D. synovial fluid inflammation. Cytotoxic T-lymphocyte-associated antigen-4 975  are ? The two receptors of TNF- N Engl J Med 2004. Interleukin-1 C. All of the above Harrison’s 17th Ed. Cytotoxic T-lymphocyte-associated antibody-4 A. Interleukin-6 D. D. is a central component in the cascade of cytokines induced in RA. Macrophages C.904 MCQ’s FOR MEDICAL PROFESSIONALS BY PROF. inflammatory & endothelial cells. C. T lymphocytes 981 CTLA-4 stands for ? D. Infliximab British Medical Bulletin 2005. These include interleukin 8 (IL-8) and RANTES. Synovial tissue inflammation 978 Which of the following is an antiinflammatory cytokine ? N Engl J Med 2001. is released by ? TNF- C. Macrophage D. Interleukin-10 A. Interleukin-6 C. Activated mesenchymal stromal cells N Engl J Med 2001. Harrison’s 17th Ed. None of the above 990 Common location of rheumatoid nodule is ? 984 In RA. It is helpful in rheumatoid arthritis. which of the following is the characteristic deformity of RA tends to spare central nervous system directly. C. Renal 988 In RA. Renal vasculitis is rare. distal interphalangeal joints are rarely involved. 2086 993 RA tends to spare which of the following systems ? A. hyperextension of proximal interphalangeal A. Palisading macrophages B. Achilles tendon B. Wrist joints D. Peripheral nervous system D. “Z” deformity Harrison’s 17th Ed. Elbow joint rheumatoid nodules dramatically ? B. D. Penicillamine 987 Part of the axial skeleton that is not spared in RA is ? In some patients. 2087 B. Achilles tendon. Psoriatic arthritis. rheumatoid nodules (focal vasculitis) consist of a central zone of necrotic material. Multiple sclerosis and hyperextension of first interphalangeal joint & flexion of first metacarpophalangeal joint. 2087 B. B. Aspirin D. High titers of antibodies to CCP Harrison’s 17th Ed. Hypertrophy of synovium C. Ankylosing spondylitis 989 Which of the following about extra-articular manifestations of RA D. 2086 991 Histologically. Increased mortality A. A. AJAY MATHUR Cardiology 905 982 TNF blockade causes deterioration of which of the following D. which of the following joints is rarely involved ? Harrison’s 17th Ed. Necrotic material C. None of the above B. Bone C. All of the above D. flexion contracture of proximal interphalangeal joints & extension of distal interphalangeal joints (Boutonnière deformity). Knee joint Harrison’s 17th Ed. Thoracic spine A. Symmetric joint involvement is more typical Patients of RA with extra-articular manifestations have high titers of autoantibodies to Fc component of IgG (rheumatoid factor) or with antibodies to CCP. Central nervous system C. Thickening of the joint capsule D. Rheumatoid arthritis joints with compensatory flexion of distal interphalangeal joints (swan-neck deformity). Cervical spine Harrison’s 17th Ed. Lumbar spine B. Psoriasis is false ? Harrison’s 17th Ed. Morning stiffness of >1 hour is an invariable feature D. 985 In RA. proximal ulna. ankylosing spondylitis. RA patients with severe extraarticular C. 2086 C. Wrist joint B. 986 Baker’s cyst is related to which of the following joint ? Harrison’s 17th Ed. Cardiovascular system Axial involvement is usually limited to the upper cervical spine. 2087 Harrison’s 17th Ed. Boutonnière deformity B. joint swelling results from ? Harrison’s 17th Ed. D. Histologically. None of the above Common locations of rheumatoid nodule include olecranon bursa. D. Sacral spine C. Ankle joint A. Olecranon bursa A. & occiput. 905 Miscellaneous MCQ’s FORMCQ’s FOR MEDICAL MEDICAL PROFESSIONALS PROFESSIONALS BY PROF. psoriasis. Chloroquine C. Distal interphalangeal joints B. 2086 992 Treatment with which of the following can increase the number of A. Harrison’s 17th Ed. Methotrexate Baker’s cyst is extension of inflamed synovium into popliteal space causing pain & swelling behind knee. central zone of rheumatoid nodules consists of ? Harrison’s 17th Ed. Metacarpophalangeal joints C. All of the above inflammatory disease ? Characteristic changes of hand in RA include radial deviation at wrist with ulnar deviation of digits with British Medical Bulletin 2005. a midzone of palisading macrophages and an outer zone of granulation tissue. Accumulation of synovial fluid B. 2087 A. Occiput C. Granulation tissue D. and Crohn’s disease. High titers of rheumatoid factor 983 Which of the following about RA is false ? B. Proximal interphalangeal joints A. 2086 A. treatment with methotrexate can increase the number of nodules dramatically. Swan-neck deformity A. Calcified tissue In RA. Cartilage . hand ? Harrison’s 18th Ed.73 & 74:71-82 palmar subluxation of proximal phalanges (“Z” deformity). Pain originates predominantly from joint capsule manifestations have an increased mortality. 2087 C. 2087 TNF blockade causes deterioration of Multiple sclerosis. 2738 994 Scleromalacia perforans is related to involvement of which of the following organ in RA ? A. AJAY MATHUR Miscellaneous C. Trypnosomiasis B. Systemic lupus erythematosus A. ceruloplasmin and C-reactive protein are elevated in RA and RA is associated with an increased incidence of lymphoma. Felty’s syndrome consists of chronic RA. correlate well with disease activity and the likelihood of progressive joint damage. 2087 A. Tendon B. neutropenia. Antibodies to calpastatin Thinning & perforation of globe due to scleritis which involves deeper layers of eye is termed as D. 2088 A. Wegeners granulomatosis C. Erythrocyte sedimentation rate A. Low socioeconomic status or education level Besides RA rheumatoid factor may be found in SLE. Eye C. C-reactive protein C. PMN leucocytosis A. Juxta-articular osteopenia C. markedly elevated ESR. Early age at onset D. Sjögren’s syndrome. Soft tissue swelling B. Antibodies to citrullinated proteins D. All of the above Acute-phase reactants like ESR. Features that are correlated with a greater likelihood of developing joint abnormalities or disabilities in RA are the presence of >20 inflamed joints. Loss of articular cartilage D. chronic liver disease. 2088 1007 Features that correlate with a greater likelihood of developing A. Mesothelioma D. especially large B cell lymphoma. presence of functional disability. Malaria C. Antibodies to fibronectin scleromalacia perforans. . Markedly elevated erythrocyte sedimentation rate C. 2088 Harrison’s 16th Ed. Antibodies to filaggrin HLA-DR 1*0401 or -DR *0404. Thrombocytosis D.906 MCQ’s FOR MEDICAL PROFESSIONALS BY PROF. Infectious mononucleosis B. hepatitis B. 1972 presence of comorbid conditions. SABE. when present. persistent inflammation. Presence of HLA-DRB1*0401 or DRB*0404 leprosy. reflects severe systemic RA disease. visceral leishmaniasis. Eosinophilia B. anemia & thrombocytopenia. 2088 Harrison’s 17th Ed. Bone erosions 998 Rheumatoid factor may be present in all except ? 1005 Which of the following is the earliest radiological sign in RA ? Harrison’s 17th Ed. Visceral leishmaniasis A. 2088 Harrison’s 17th Ed. tuberculosis. Joint effusion D. Lymphoma C. Chronic rheumatoid arthritis B. 1973 A. High titers of serum rheumatoid factor C. Basophilia C. Presence of more than 10 inflamed joints B. Presence of rheumatoid nodules 1000 Rheumatoid factor may be present in all except ? Harrison’s 17th Ed. Ceruloplasmin B. Cataract B. Juxta-articular osteopenia C. Tuberculosis A. sarcoidosis. splenomegaly. interstitial pulmonary fibrosis. low socioeconomic status / educational level. 2088 B. Subacute bacterial endocarditis joint abnormalities or disabilities in RA include all except ? Harrison’s 17th Ed. advanced age at onset. syphilis. Neutropenia D. Harrison’s 16th Ed. infectious mononucleosis. or presence of A. Syphilis C. Sarcoidosis D. Splenomegaly C. 1003 Acute-phase reactants elevated in RA include ? 996 RA is associated with an increased incidence of ? Harrison’s 17th Ed. HIV/AIDS D. 1002 Severe systemic RA disease is reflected by ? 995 Felty’s syndrome consists of all except ? Harrison’s 17th Ed. Leprosy B. Formation of spicules B. Lymphadenopathy Eosinophilia. Interstitial pulmonary fibrosis A. radiographic evidence of 1001 Additional autoantibodies found in RA are all except ? bone erosions. All of the above D. 1973 A. 2087 A. Radiographic evidence of bone erosions D. Loose bodies 999 Rheumatoid factor may be present in all except ? 1006 Features that correlate with a greater likelihood of developing Harrison’s 17th Ed. 2088 Harrison’s 16th Ed. high titers of serum rheumatoid factor or anti-CCP antibodies. 2088 joint abnormalities/disabilities in RA include all except ? Harrison’s 17th Ed. Hepatitis B D. D. 997 Rheumatoid factor may be present in all except ? 1004 Which of the following is the earliest radiological sign in RA ? Harrison’s 17th Ed. Hepatitis C C. Sjogren’s syndrome B. presence of rheumatoid nodules. schistosomiasis and malaria. Type 1 diabetes mellitus C. Involves small & large joints in upper & lower limbs B. High titers of rheumatoid factor.79(4):249 C. Sustained disease activity of > 6 months duration B. Glucocorticoid use A. 2088 been reported in ? A. aureus D. 1 . 1500 C. None of the above B. Reactive inflammation. High titers of haptoglobin B. 2088 1019 Upper airway obstruction in RA is due to ? A. 2088 D. Decreased concentrations of glucose 1014 Which of the following about RA is false ? C. 2088 is ? A. Hemophilus radiographic evidence of erosions at the time of initial evaluation. S. 1649 B. High titers of histatin C. Rheumatoid arthritis D. Rheumatoid arthritis D. Multiple sclerosis D. A. 2153 C. Disability 1017 Distal interphalangeal joints are involved in ? B. and multiple sclerosis. 1012 Which of the following statements about RA is false ? Harrison’s 17th Ed. AJAY MATHUR Cardiology 907 1008 Which of the following does not signify poor prognosis in RA ? 1015 Molecular mimicry between microbial proteins & host tissues has Harrison’s 17th Ed. Streptococci RA patients with high titers of rheumatoid factor. Presence of subcutaneous nodules is diagnostic A. Morning stiffness lasting > 1 hour Harrison’s 17th Ed. vasculitic Harrison’s 17th Ed. 3 .10 years C. Subcutaneous nodules infective arthritis most often with ? Harrison’s 17th Ed. Radiographic evidence of erosions at time of initial evaluation A. 2088 Upper airway obstruction in RA is due to Crico-arytenoid arthritis. Sparing of axial skeleton except cervical spine 1020 In RA. Juxtaarticular bone demineralization A. rheumatoid arthritis. Osteoarthritis D. Nil Harrison’s 17th Ed. All of the above B. All of the above 1009 Features of patients with RA that have poor prognostic significance Molecular mimicry between microbial proteins and host tissues has been reported in type 1 diabetes are all except ? mellitus. Crico-arytenoid arthritis D. 2133 B.7 years. All of the above 1013 Which of the following about RA is false ? Harrison’s 17th Ed.3 years A. Harrison’s 17th Ed. Cervical spine involvement C. glucocorticoid therapy & frequent breakdown of rheumatoid nodules. aureus. Elevated cholesterol Harrison’s 17th Ed.most often S. and low socioeconomic or educational status. Psoriatic Factors correlated with early death include disability. disease duration or severity. 907 Miscellaneous MCQ’s FORMCQ’s FOR MEDICAL MEDICAL PROFESSIONALS PROFESSIONALS BY PROF. High titers of rheumatoid factor Harrison’s 17th Ed. Presence of rheumatoid factor & anti-CCP antibodies 1021 Which of the following is true about Rheumatoid factor ? B. because of 1010 Factors correlated with early death in RA include ? chronically inflamed joints. 2170 C. Antibody against Fc portion of immunoglobulin A D. A. Systemic vasculitis C. Inflammatory synovial fluid with increased PMNLs Cleveland Clinic J of Med 2012. All of the above A. CRP & haptoglobin have a worse prognosis. 2088 A. 7 . anti-CCP (cyclic citrullinated peptide) antibodies. Decreased concentrations of complement D. Age at onset Harrison’s 17th Ed. D. Muscular atrophy D. Bilateral symmetric inflammatory polyarthritis A. Antibody against Fc portion of immunoglobulin E . age at onset. Systemic lupus erythematosus The median life expectancy of persons with RA is shortened by 3 . exudative pericardial fluid has ? B. D. Pseudomonas Patients with RA have highest incidence of infective arthritis . All of the above 1011 Median life expectancy of RA patients is shortened by ? 1018 Most common disorder associated with relapsing polychondritis Harrison’s 17th Ed. 2071 B. None of the above C. CRP & haptoglobin 1016 Patients with rheumatoid arthritis have the highest incidence of B. D. Foot joints more frequently affected than hand joints Harrison’s 17th Ed. persistent C. glucocorticoid use. Sjögren’s syndrome Most common disorder associated with relapsing polychondritis is Systemic vasculitis. 2088 ulcers. None of the above B. as do patients with subcutaneous nodules or C. Sustained disease activity of >1 year’s duration has a poor outcome. High titers of C-reactive protein A.7 years B. Neither test varies with the B. Etanercept D. or D-penicillamine. Malaria B. all except ? Harrison’s 17th Ed. All of the above 1029 Concurrent administration of which of the following may diminish Rheumatoid factor testing suffers from low specificity. It is a pro-drug C. Lysine folinic acid diminishes frequency of its side effects. is an antibody against Fc portion of immunoglobulin G. D.350:2167-79 RA patients often produce autoantibodies directed against proteins and peptides containing amino A. and aging. Hepatitis C methotrexate therapy ? Harrison’s 17th Ed. Iron A. Systemic lupus erythematosus improvement thereafter. N Engl J Med 2004. 2090 D. C. first described in 1940. N Engl J Med 2004. C. D. Causes hypertension activity of RA. osteomyelitis. TNF-neutralizing agents include infliximab. SjÖgren syndrome Maximal improvement is observed after six months of methotrexate therapy. 2090 B. the rate limiting enzyme for de novo synthesis of pyrimidine nucleotides. Appear earlier than rheumatoid factor Its active metabolite is A77 1726. Concurrent administration of folic acid or B. primary biliary cirrhosis. An oral antimetabolite acid citrulline. 1026 Disease-Modifying Antirheumatic Drugs (DMARDs) in RA include Usual daily dose is 20 mg. Tryptophan Harrison’s 17th Ed. Sulfasalazine 1022 Rheumatoid factor can be detected in ? Cleveland Clinic J of Med 2012. D-penicillamine B. B. Azathioprine C. since it can be detected in bacterial the frequency of some side effects of methotrexate ? endocarditis. 2090. 2090 . sulfasalazine. Hepatic adverse effects D. B. SjÖgren syndrome. 2090 1032 Which of the following is a TNF-neutralizing agent ? A. Omega 3 fatty acids modification of which amino acid ? C.908 MCQ’s FOR MEDICAL PROFESSIONALS BY PROF. Rheumatoid factor positivity and anti-CCP antibody positivity are each associated with more severe RA. malaria. tuberculosis. All of the above A. C. 2090 arthropathy.79(4):249 D. Isoleusine 1030 Which of the following is false about Leflunomide ? D. Tuberculosis 1028 Maximal improvement is observed after how many months of C. Arginine Methotrexate is an antimetabolite & antifolate drug. Leflunomide is safe in pregnancy A. AJAY MATHUR Miscellaneous C.79(4):249 Methotrexate has emerged as the DMARD of choice especially in individuals with risk factors for development of bone erosions or persistent synovitis of >3 months’ duration.79(4):249 C. 2090. etanercept and adalimumab. Does not vary with the activity of RA 1031 Which of the following is false about Leflunomide ? D. A. Higher anti-CCP antibody titer was associated with increased disease activity and inversely correlated with remission. Blocks pyrimidine-synthesis pathway who later develop RA and may appear earlier than rheumatoid factor. gold salts. D-penicillamine D. Bacterial endocarditis D. Citrulline is generated in an inflammatory environment by the modification of the amino acid arginine by the enzyme peptidylarginine deiminase. postvaccination Harrison’s 17th Ed.350:2167-79 Anti-CCP antibody has been found in sera up to 10 years before the onset of joint symptoms in patients A. 3 A. Infliximab C. Gold compounds A. Found in sera years before the onset of joint symptoms Leflunomide is an oral antimetabolite pro-drug which inhibits dihydro-orotate dehydrogenase. 1 1023 Rheumatoid factor can be detected in ? B. Antibody against Fc portion of immunoglobulin G Harrison’s 16th Ed. Gold compounds Rheumatoid factor.79(4):249 D. Pyridoxine 1024 Citrulline is generated in an inflammatory environment by the B. Methotrexate B. A. most rheumatologists use which one as the first drug ? Harrison’s 17th Ed. Folic acid or folinic acid Cleveland Clinic J of Med 2012. with little additional C. 6 B. Usual daily dose is 200 mg. Competitive inhibitor of dihydro-orotate dehydrogenase 1025 Which of the following about Anti-CCP antibody is false ? Cleveland Clinic J of Med 2012. hepatitis C (with or without cryoglobulinemia). C. especially in those also positive for rheumatoid factor. 1027 Out of the following Disease-Modifying Antirheumatic Drugs in 1033 Which of the following is a IL-1-neutralizing agent ? RA. Leflunomide is teratogenic. All of the above A. Methotrexate Harrison’s 17th Ed. All of the above Combinations of DMARDs appear to be more effective than single agents in controlling the signs and symptoms of RA. systemic lupus erythematosus. Adalimumab DMARDs include methotrexate. None of the above Harrison’s 17th Ed. hydroxychloroquine. 2 Cleveland Clinic J of Med 2012. 1975 D. Any of the above regulates IL-1 production through the formation of a macromolecular complex termed the inflammasome. None of the above risk of other infections. Less effective than Anti-TNF therapy D. Muckle-Wells syndrome (MWS). the gene encoding cryopyrin (or NALP3). Infliximab C. IL-1-neutralizing action A. Interfering with T cell activation B. TNF-neutralizing agent + methotrexate Familial cold autoinflammatory syndrome (FCAS). It inhibits T cell activation by competitively inhibiting co-stimulation of T cells that results from interaction of T cell . 2090 A. Adalimumab B. Patients with all three cryopyrinopathies show a dramatic response to daily injections 1041 Anti-TNF therapy can lead to ? of anakinra. Etanercept A. Its therapy can be repeated at 6. 2090 C. Human anti-cytoplasmic antibodies 1038 Immunosuppressive and cytotoxic drug used in RA is ? B. Reactivation of dormant tuberculosis Harrison’s 17th Ed. Etanercept D. TNF-neutralizing action Harrison’s 17th Ed. Adalimumab C. Given to RA patients who have failed anti-TNF therapy. Leflunomide D. It can be combined with methotrexate. and adalimumab is a 1034 Which of the following is a hereditary febrile syndrome ? fully human antibody to TNF with no rodent components. AJAY MATHUR Cardiology 909 A. All of the above Immunosuppressive and cytotoxic drugs used in RA are leflunomide. TNF-neutralizing action B. azathioprine and cyclophosphamide. These human anti-chimaeric antibodies C. Abatacept is a fusion protein consisting of CTLA4 (cytotoxic T-lymphocyte-associated antigen- 4) and the Fc portion of IgG1. worsening of severe heart failure. Infliximab 1039 Which of the following is a fully human antibody to TNF ? Harrison’s 17th Ed. Use of methotrexate with infliximab reduced this antibody production. Development of demyelinating CNS disease B. and neonatal- onset multisystem inflammatory disease (NOMID) are hereditary febrile syndromes caused by C. demyelinating CNS disease and increased risk of 1036 Rituximab acts by ? lymphomas (more with infliximab). Muckle-Wells syndrome (MWS) in RA ? Harrison’s 17th Ed. Familial cold autoinflammatory syndrome (FCAS) 1040 Which of the following drug regimens provides greatest benefit B. month intervals when circulating B cells return. Features of MWS are urticarial rash. antibodies against it are produced. Etanercept D.73 & 74:71-82 contraindications to Anti-TNF therapy. Anakinra C. the IL-1 receptor antagonist. IL-1-neutralizing action C. Cyclosporine During infliximab therapy. 1043 Infliximab was originally called ? 1037 Abatacept acts by ? British Medical Bulletin 2005. 2090 B.expressed CD28 & CD80/86 expressed by antigen. Neonatal-onset multisystem inflammatory disease (NOMID) A. Harrison’s 17th Ed.73 & 74:71-82 Harrison’s 17th Ed. 909 Miscellaneous MCQ’s FORMCQ’s FOR MEDICAL MEDICAL PROFESSIONALS PROFESSIONALS BY PROF. TNF-neutralizing agent + methotrexate + DMARD mutations in CIAS1. TNF-neutralizing agent alone D. Cryopyrin D. Increased risk of lymphoma A. abdominal pain. Anakinra is a recombinant IL-1 receptor antagonist that binds to the cell-bound IL-1 receptor and inhibits the binding of endogenous D. Azathioprine (HACA) result in reduction of its efficacy. Depleting B cells C. Human anti-cytoskeleton antibodies Harrison’s 17th Ed. All of the above B. Human anti-chimaeric antibodies A. A. Harrison’s 17th Ed. Anakinra Rituximab is a chimeric antibody directed against the B-lymphocyte cell surface marker CD20 D. None of the above IL-1. 2090 1042 Which of the following is administered subcuteneously ? A. cyclosporine. Infliximab and Rituximab are given IV. Depleting B cells Infliximab was originally called CA2. 2090 B. CA1 A. 2090 B. 1044 HACA stands for ? presenting cells. Chapter 323 A. CA4 D. Should not be combined with Anti-TNF therapy TNF blocking therapies is associated with reactivation of latent Mycobacterium tuberculosis and D. All of the above (CD20 is not present on all plasma cells or on very early B cells).. limb pain. infliximab is a chimeric mouse/human monoclonal antibody to TNF. . fevers. CA2 B. Etanercept is a fusion protein consisting of two p75 TNF receptors linked to Fc portion of human IgG1. Human anti-conjugation antibodies B. conjunctivitis and sensorineural hearing loss. Interfering with T cell activation D. Harrison’s 17th Ed. It is reserved for patients who have failed Anti-TNF therapy or those who have British Medical Bulletin 2005. It has been used effectively to treat adult Still’s disease and Muckle–Wells syndrome. Can be given as monotherapy or with methotrexate C. D. All of the above C. arthritis. CA3 C. Adalimumab IL-beta and IL-1alpha are two related proinflammatory cytokines. 2090 C. induction of autoantibodies (ANAs anti dsDNA) and autoimmune diseases (SLE). 2090 1035 Which of the following about Anakinra is false ? A. which includes three genera of animal C. Domestic dog Rhabdos means “rodlike”. Peripheral nervous system 1053 Rabies is not found in mammals in which of the following regions C. 1611 D. foxes. Antarctica 1048 Rabies manifests most often as ? C.910 MCQ’s FOR MEDICAL PROFESSIONALS BY PROF. Arenaviridae A. wolves. 1156 D. Wolves virus. 1611 C. Positive polarity C. Domestic dog C. The first step in viral replication is synthesis of full-length copies (postive strands) of the viral D. Matrix protein gene In Greek. November 12 Rabies is an RNA virus. Peripheral neuritis 1054 Sylvatic rabies is propagated by ? Harrison’s 16th Ed. Arctic Rabies is an acute viral disease of the central nervous system (CNS) B. Mokola virus. 1050 Which of the following is not a characteristic of rabies virus ? Harrison’s 18th Ed. phosphoprotein (P) and a large 1046 In Greek. All of the above A. the domestic animal most commonly infected with rabies is the cat. Glycoprotein C. Bats “bullet” shape are classified in the Rhabdoviridae family. and bats. Rabies of ? B. Rabies virus genome consists of 11. A. Bullet shaped with rabies is ? Harrison’s 18th Ed. October 11 D. Cats viruses. viruses with a nonsegmented. All of the above A. negative-stranded RNA genomes. Rabies virus belongs to the order Mononegavirales. B. lyssa means rabies. Polymerase protein gene 1047 Rabies is a disease of ? Harrison’s 18th Ed. Ephemerovirus and Vesiculovirus. matrix protein (M). Duvenhage virus. Bullet Harrison’s 18th Ed. Coronaviridae Sylvatic rabies is propagated by skunks. Central nervous system of the world ? Harrison’s 18th Ed. 1611 1045 World Rabies Day is on ? A. Autonomic nervous system B. enveloped. glycoprotein (G). C.932 nucleotides and encodes five proteins . 1611 B. single-strand RNA virus of negative polarity. Single-strand RNA B. Nucleocapsid gene D. All of the above D. 1611 D. 1611 1056 In United States. Genus Lyssavirus includes rabies virus. the domestic animal most commonly infected A. August 05 B. Tasmania Harrison’s 18th Ed. Wolve genome. Enveloped A. 1611 Each animal reservoir harbors one or more distinct rabies virus variants that can be distinguished by the sequence of the nucleocapsid gene. Meningitis Rabies is found in mammals in all regions of the world except Antarctica. Within this group. Bat D. B. raccoons. 1611 D. Wolves 1049 Rabies virus belongs to which family of viruses ? C. Encephalitis C. Flaviviridae C. Matrix protein B. AJAY MATHUR Miscellaneous 1051 Rabies virus genome encodes which of the follwoing Rabies proteins ? Harrison’s 18th Ed. Rhabdoviridae 1055 The main reservoir of rabies throughout the world is ? Harrison’s 18th Ed. Newzealand A. Nucleoprotein A. . Animal A. The main reservoir of rabies throughout the world is the domestic dog. In the United States. Death B. Foxes Rabies manifests most often as encephalitis. WHO Technical Report Series 931 1052 Rabies virus variants can be distinguished by the sequence A. B. Bats Harrison’s 18th Ed. viruses with a distinct B. Lagos bat. Lyssavirus.nucleocapsid (N). All of the above A. 1611 D. September 07 C. European bat virus 1 & 2 and Australian bat D. Glycoprotein gene D. mongooses. word “lyssa” means ? RNA polymerase protein (L). Cat Rabies virus is bullet shaped. Bunina bodies C. Negri bodies are cytoplasmic inclusions In CNS.150 mm/day A. 1058 Upon inoculation of skin by rabies virus-laden saliva. 60 days to >1 year B. Purkinje cells of cerebellum After entering sensory and motor neurons. Dr. A. Abdomen Rabies virus binds to nicotinic acetylcholine receptors at neuromuscular junctions. Ammon’s horn C. Neural cell adhesion molecule Rates of infection and mortality from rabies are highest from bites on face. Ln/Ps B.400 mm/day B. Autonomic nerves D. Neuronal death D. Lewy bodies Incubation period of rabies is exceedingly variable. 50 . Degenerative changes C. There are inflammatory changes. Mn/Ps C. liver. Kn/Ps A. 1156 Harrison’s 18th Ed. Face D. skeletal muscles. Dorsal spinal ganglia B. lungs. The most characteristic pathologic finding of rabies in CNS is the formation of eosinophilic 1061 Rabies virus spreads centripetally to CNS at a rate of ? cytoplasmic inclusions called Negri bodies within neurons. From CNS. skin. Negri bodies are most commonly present in Purkinje cells of cerebellum and in pyramidal cells in 1062 In CNS. 911 Miscellaneous MCQ’s FORMCQ’s FOR MEDICAL MEDICAL PROFESSIONALS PROFESSIONALS BY PROF. All of the above mm/day or ~250 mm/day via fast axonal transport to the spinal cord or brainstem. Absence of Negri bodies excludes rabies D. Negri bodies D. neuroanatomic connections. 21 days to >1 year A. rabies virus passes centrifugally along sensory and autonomic nerves to other tissues like salivary glands. 20 days to >1 year Harrison’s 18th Ed. adrenal medulla. Hypothalamus D. Negri bodies are found in ? A. initial viral replication occurs in ? 1064 From CNS. 1612 rabies are called ? A. 1060 The incubation period of rabies is ? 1066 Eosinophilic cytoplasmic inclusions within neurons of CNS in Harrison’s 18th Ed. Norepinephrine B. Cortical gray matter C. Negri bodies can also be found in neurons of salivary glands or tongue. Serotonin C. and less frequently seen in cortical neurons and in brainstem. 1612 C. Adelchi Negri reported round or oval inclusions within cytoplasm of nerve cells of animals infected with Harrison’s 16th Ed. nn/Ps D. 1612 on ? Harrison’s 16th Ed. Acetylcholine A. initial viral replication occurs within striated muscle cells at the site of inoculation.400 D. Venous sinuses B. 1157 A. intermediate from bites and p75NTR neurotrophin receptor may be receptors for rabies virus. Dopamine D. Rare in infections due to laboratory variants of rabies virus . Gray matter B. 1612 B.Ln/ Once the virus enters the CNS. Harrison’s 18th Ed. 1612 1067 In brain. All of the above It is observed that a particular bat-associated rabies virus is implicated in most human cases . but there are few degenerative changes involving neurons and little evidence of neuronal death. Blood B. AJAY MATHUR Cardiology 911 1057 Which particular bat-associated rabies virus is implicated in most 1063 Which of the following is least evident in the gray matter infected human cases ? by the rabies virus ? Harrison’s 16th Ed. 400 . Skin C.700 mm/day C. 1612 A. 1157 rabies. 1 . Inflammatory changes B. 1612 B. Motor nerves After inoculation of rabies virus through skin. Striated muscle cells D. 100 . 1157 via ? Harrison’s 18th Ed. Hands and arms B. kidneys. and acetylcholine receptor blockade inhibits rabies virus attachment. which is associated with silver-haired and eastern pipistrelle bats. Nissel bodies D. Legs C. rabies virus replicates almost exclusively within ? hippocampus. Epidermal cells of skin A. CSF A. it spreads rapidly throughout the gray matter via established Ps (Lasionycteris noctivagans). C. 90 days to >1 year C. from 20 days to 90 days but may be >1 year.50 mm/day Harrison’s 18th Ed. White matter 1068 Which of the following about Negri bodies is false ? Harrison’s 18th Ed. 1612 A. In 1903. rabies virus spreads centripetally at a rate of 100 . and heart. rabies virus replicates almost exclusively within the gray matter. rabies virus passes centrifugally to other tissues Harrison’s 16th Ed. on hands & arms and lowest from bites on legs. 1059 Which neurotransmitter receptors is implicated in rabies virus attachment and internalization ? 1065 Rates of infection & mortality from rabies are highest from bites Harrison’s 18th Ed. 2482 A. 1613 D. Represent nucleocapsids of rabies virus C. D. Coma/death A. Cranial nerve palsies Two acute neurologic forms of rabies seen in humans are encephalitic (furious) in 80% & paralytic in 20%. Sphincter involvement by 50 to 80% of patients. with mononuclear B. Diaphragmatic Pathology studies show mild inflammatory changes in the CNS in rabies. Harrison’s 18th Ed. 20 % A. Paresthesia / fasciculations at the site of inoculation Harrison’s 18th Ed. violent. Acute neurologic phase 1076 “Dumb rabies” is similar in presentation to ? C. AJAY MATHUR Miscellaneous D. 40 % B. Pharyngeal inflammatory infiltration in the leptomeninges. 1613 is common. including C. 40 days 1073 Paralytic presentation during acute neurologic phase of rabies is The median period of survival after the onset of symptoms is 4 days. Brainstem A. 1615 . Hydrophobia or aerophobia are probably B. Vanishing nodules of which of the following muscles ? C. Microglial nodules Harrison’s 18th Ed. Rabies Absence of Negri bodies does not exclude the diagnosis of rabies. Japanese encephalitis virus) 1080 Full-thickness skin biopsy sample for the antemortem diagnosis of rabies is obtained from ? B. B. C. Harrison’s 16th Ed. D. and parenchyma. Early and prominent muscle weakness Prodromal period usually lasts 2 to 10 days. Flaviviruses (WNV. 5 % 1079 Which of the following is not a presentation of rabies ? Harrison’s 16th Ed. but sensory involvement is usually mild. 1157 B. A. Harrison’s 18th Ed. increased deep tendon reflexes. 1157 D. Amaurosis fugax Harrison’s 18th Ed. B. and brainstem signs is suggestive of ? extensor plantar responses. acute neurologic phase (Encephalitic and B. pharyngeal. Sphincter involvement Harrison’s 18th Ed. 1612 HSV. 1612 Rabies may present as an ascending paralysis resembling the Landry Guillain-Barre´ syndrome A. 60 % C. Landry Guillain-Barre´ syndrome 1071 Prodromal symptom suggestive of rabies is ? D. 1157 D. maximum period B. All of the above 1072 Hydrophobia or aerophobia in encephalitic rabies is due to Paralytic rabies is characterized by early and prominent muscle weakness. 30 days nucleus ambiguus resulting in exaggerated defense reflexes that protect the respiratory tract. Gangrene at the site of inoculation A. Chronic neurologic phase Harrison’s 16th Ed.the painful. 10 days Rabies encephalitis results in early brainstem involvement. violent and involuntary contraction B. seen in what percentage of cases ? unless artificial supportive measures are instituted. symptoms or 1069 Babes nodules are best related to ? neuroimaging abnormalities may be infected by flaviviruses (WNV. All of the above Patients with rapidly progressive encephalitis and prominent brainstem signs. D. 20 days due to dysfunction of infected brainstem neurons that normally inhibit inspiratory neurons near C. D. The prodromal symptom suggestive of rabies is the B. Prodrome cases of rabies. Apoptotic neurons 1075 Hydrophobia is due to painful. HSV Harrison’s 18th Ed. 1614 D. rabies or L. often beginning in the involvement of neurons in ? bitten extremity and spreading to produce quadriparesis and facial weakness. is the rule. Amotrophic lateral sclerosis paralytic) and coma/death. Japanese encephalitis virus). Purkinje cells of cerebellum of survival after onset of symptoms is ? C. without artificial supportive measures.prodrome. A. Autonomic nervous system dysfunction C. perivascular regions. accessory Harrison’s 18th Ed. Priapism and spontaneous ejaculation 1074 Features of rapidly progressive encephalitis and prominent Evidence of upper motor neuron paralysis. Cortex 1078 In rabies. D.reported C. Pyramidal cells in hippocampus Harrison’s 16th Ed. Facial weakness complaint of paresthesia and/or fasciculations at or around the site of inoculation of virus . Vesicles at the site of inoculation 1077 Paralytic rabies is characterized by ? C. involuntary contraction of the diaphragmatic. Degenerative neurons A. with weakness. monocytogenes. 1614 Table 195-1 respiratory. All of the above 1070 Phases of clinical rabies are all except ? Hydrophobia .912 MCQ’s FOR MEDICAL PROFESSIONALS BY PROF. and laryngeal muscles initiated by swallowing liquids is seen in ~50% of A. with a maximum of 20 days. 1614 A. Lower motor neuron paralysis D. Multiple sclerosis Clinical rabies has three general phases . Laryngeal microglial nodules called Babes nodules. Oozing of blood at the site of inoculation (dumb rabies or rage tranquille). 1088 Human rabies postexposure prophylaxis (PEP) consists of five doses of purified inactivated rabies vaccine given on days ? 1082 Which of the following does not constitute an exposure to rabies ? Harrison’s 18th Ed. Thigh Rabies PEP includes local wound care. schedule of preexposure rabies vaccination is ? B. serum neutralizing antibody titers become 1081 Which of the following is the most sensitive test for demonstrating detectable within ? Harrison’s 18th Ed. 2 . 7. Simply touching a rabid animal or person does not constitute an exposure to rabies. 1615 A. 20 IU/kg B. or Purified Chick Embryo Cell Vaccine (PCEC) injections because administration of HDCV in this area results in lower neutralizing antibody titers. and 21 or 28 D.4 weeks. and 90 are produced and passive immunization may be counterproductive. 0. 0. Reverse transcription polymerase chain reaction (RT-PCR) D. 40 IU/kg C. 7. Full-thickness skin biopsy sample from nape of the neck with at least 10 hair follicles is taken for diagnosis of rabies. Indirect fluorescent antibody test B. 913 Miscellaneous MCQ’s FORMCQ’s FOR MEDICAL MEDICAL PROFESSIONALS PROFESSIONALS BY PROF. 14. 14 and 28 A. and 28 days. Rapid fluorescent focus inhibition test (RFFIT) C. Infectious aerosol of rabies virus D. 1616 1092 Semple vaccine is best related to ? A. Cow spinal cords Pasteur vaccine was made out of rabbit spinal cords. and 21 or 28. 1 . The gluteal area should never be used for Human Diploid Cell Vaccine (HDCV). Local wound care A. 7 days B. ACIP recommends preexposure rabies vaccination of 1-mL doses of modern cell culture vaccine administered intramuscularly on days 0. Ocular contact with contaminated material 1-mL doses of purified inactivated rabies vaccine are given on 0. 1089 For adults. 1616 Harrison’s 16th Ed. Site of inoculation C. 1616 1090 According to Advisory Committee on Immunization Practices A. 1616 A. 4 . When a previously immunized individual 1085 Dose of Rabies immune globulin (RIG) is ? is exposed to rabies. Days 0. 14. Days 0.6 weeks C. Rabies Vaccine Adsorbed (RVA). Gluteal D. 21 and 28 C. the presence of rabies virus nucleic acid in fresh saliva can be confirmed or ruled out. Direct fluorescent antibody (DFA) test After immunization. 7. two booster doses of vaccine should be administered on days 0 and 3. AJAY MATHUR Cardiology 913 A. 14. Days 0. 14 and 90 B. 10 IU/kg A. Nape of the neck B. Sheep C. Ear lobule RIG is purified from serum of hyperimmunized human donors (HRIG) and are better tolerated than equine-derived preparations (ERIG dose 40 IU/kg). or PCEC 1083 Rabies postexposure prophylaxis (PEP) includes ? injections ? Harrison’s 18th Ed. serum neutralizing antibody titers become detectable within 2 . 0. 3. Contamination of mucous membranes with infectious material C. Rabbit spinal cords D. 3. 3. Active immunization B. Human D. 6 . 7. RIG Harrison’s 18th Ed. 1616 Harrison’s 18th Ed. 5 days A. and 21 or 28 All previously unvaccinated persons should be passively immunized with rabies immune globulin (RIG). Administration in anterolateral 1084 Rabies immune globulin (RIG) should not be administered how aspect of thigh is acceptable in children. A. 7. 1086 RIG in derived from ? Harrison’s 18th Ed. 3. 21 and 90 D. 1087 After immunization. 3. 3 days Harrison’s 18th Ed. 5 IU/kg 1091 Pasteur vaccine was made out of ? B.4 weeks B. active & passive immunization. many days after the first vaccine dose ? Harrison’s 18th Ed. 1616 C. Abdomen C. which is the preferred site for HDCV. Passive immunization C. 1616 should not be administered to previously vaccinated persons.2 weeks A. 1 day (ACIP). With the exquisitely sensitive RT-PCR. 7. Mouse spinal cords Dose of RIG is 20 IU/kg and should be injected at site of bite and any remaining after infiltration of the bite site should be administered intramuscularly at a distant site. Horse spinal cords C. RVA. Deltoid B. 7. Days 0. RIG should not be administered 7 days after the first vaccine dose as endogenous antibodies D. Touching a rabid animal or person B. Dog B. D. Scalp D. All of the above D. Coonoor . 1158 A. Cow A. and 90 C.8 weeks D. 1616 rabies virus specific antigen/antibody ? Harrison’s 18th Ed. 0. 7. All of the above 1098 Lichenoid eruptions are seen with all except ? 1106 “Mat telangiectasia” are commonly found at all of the following Harrison’s 16th Ed. All of the above 1095 Which of the following is a papulosquamous disease ? 1103 Which of the following is a feature of ‘Poikiloderma’ ? Harrison’s 16th Ed. Chennai D. Face Shelf life of Semple vaccine is six months. Scleroderma C. Abdomen 1099 Which of the following is false about secondary syphilis ? Harrison’s 16th Ed. 298 A. Twelve months A. 298 (CRI) at Kasauli in 1911.and hyperpigmentation B. 298 B. Sulfonylureas Harrison’s 16th Ed. Nonscarring alopecia C. Hands D. 298 D. 296 1107 Which of the following is not a cause of “Periungual telangiectasia” ? A. ACE inhibitors A. AJAY MATHUR Miscellaneous B. Psoriasis A. 296 sites except ? A. ACE inhibitors B. HIV infection C. Kasauli C. Dermatomyositis B. Mumbai 1100 Which of the following is a cause of nonscarring alopecia ? Semple vaccine is named after Dr David Semple. Androgenetic alopecia 1094 Shelf life of Semple vaccine is ? C. Lichen planus A. A. All of the above 1096 Which of the following is known to exacerbate psoriasis ? 1104 Poikiloderma is a feature of all except ? Harrison’s 16th Ed. Brains of adult mouse Harrison’s 16th Ed. All of the above D. 300 A. All of the above D. Telangiectasias D. Carcinoid syndrome D. Extremities Skin manifestations in medical disorders D. Oral mucosa D. Venous hypertension B. Reticulated hypo. Lithium A. B. Mastocytosis D. Tinea capitis B. Brains of adult horse D. 296 Harrison’s 16th Ed. Brains of adult sheep C. None of the above D. British director of Central Research institute Harrison’s 16th Ed. Hyperthyroidism A. Involves palms & soles A. Wrinkling due to epidermal atrophy C. Metronidazole D. Face C. Nine months 1102 Erythema marginatum is seen primarily on ? Harrison’s 16th Ed. Thiazides B.914 MCQ’s FOR MEDICAL PROFESSIONALS BY PROF. Lupus erythematosus . Trunk C. Pityriasis rosea C. Tinea B. Telogen effluvium B. Xeroderma pigmentosum 1097 Pityriasis rosea like drug eruptions are seen with all except ? 1105 “Mat telangiectasia” is characteristic of ? Harrison’s 16th Ed. Metronidazole C. Brains of adult cow 1101 Which of the following is not a cause of nonscarring alopecia ? D. Thiazides C. Hypopituitarism B. Beta blockers A. All of the above C. 300 B. Hypothryoidism 1093 Semple vaccine is manufactured from ? B. Three months D. Beta blockers B. Six months C. 296 Harrison’s 16th Ed. 298 Semple vaccine is manufactured from brains of adult sheep. Cutaneous T cell lymphoma C. A. 298 A. Red-brown papules Harrison’s 16th Ed. 296 Harrison’s 16th Ed. 915 Miscellaneous MCQ’s FORMCQ’s FOR MEDICAL MEDICAL PROFESSIONALS PROFESSIONALS BY PROF. Cyclophosphamide 1112 Which of the following is not a cause of acneiform eruptions ? Harrison’s 16th Ed. 300 1116 Acanthosis nigricans is associated with all except ? A. Vogt-Koyanagi-Harada syndrome Harrison’s 16th Ed. TSST-1 1114 Which of the following is a cause of hyperpigmentation ? D. 300 1120 Erythema multiforme (EM) may be due to ? A. Pregnancy related A. Deafness (sensorineural) 1110 Which of the following diseases is associated with vitiligo ? Harrison’s 16th Ed. Hypothyroidism D. All of the above C. 303 B. 302 B. Acromegaly C. Optic neuritis D. 303 A. 302 B. All of the above 1113 Which of the following is a cause of hyperpigmentation ? Harrison’s 16th Ed. Chlorpromazine C. Scleroderma A. Congenital sensorineural hearing loss Harrison’s 16th Ed. Barbiturates D. Piebaldism C. All of the above C. Endocrinopathies D. 299 1118 Which of the following is not a part of POEMS syndrome ? A. Calcium B. Dermatomyositis C. Idiopathic Harrison’s 16th Ed. 303 1121 Which of the following toxins is associated with Staphylococcal A. Whipple’s disease A. 304 C. Scleroderma A. Folic acid deficiency Harrison’s 16th Ed. Hemochromatosis Harrison’s 16th Ed. Carbamazepine C. Broad nasal root B. Exfoliatin Harrison’s 16th Ed. Mycoplasma pneumoniae D. 303 . Hereditary hemorrhagic telangiectasia B. Porphyria cutanea tarda 1122 Gottron’s sign is characteristic of ? B. 300 hyperpigmentation” ? A. Increased interpupillary distance A. Lithium A. Dermatomyositis Harrison’s 16th Ed. 304 B. All of the above 1108 “Spider angioma” can be ? Harrison’s 16th Ed. Penicillins D. Carcinoid syndrome A. Chloroquine D. M-protein 1111 Which of the following is not a feature of Waardenburg’s syndrome ? 1119 Which of the following drugs does not produce “Diffuse Harrison’s 16th Ed. Mastocytosis C. Dermatitis herpetiformis C. Lymphotoxin C. Ataxia-telangiectasia B. Phenytoin C. Iodides C. Stein-Leventhal syndrome 1109 “Papular telangiectasia” is a feature of ? Harrison’s 16th Ed. 300 1117 Which of the following is not a part of LEOPARD syndrome ? A. Pellagra A. AJAY MATHUR Cardiology 915 B. Melanoma-associated leukoderma B. All of the above B. Herpes simplex virus D. Anabolic steroids Harrison’s 16th Ed. Cicatricial pemphigoid 1115 Which of the following can cause toxic epidermal necrolysis ? D. Cushing’s syndrome D. 316 C. All of the above B. Busulfan D. 303 B. EEG abnormalities D. Stevens-Johnson syndrome D. Pulmonary stenosis D. Hereditary hemorrhagic telangiectasia Harrison’s 16th Ed. Polyneuropathy C. Cirrhosis liver related B. Aflatoxin B1 D. Lentigines C. Vitamin B12 deficiency scalded-skin syndrome (SSSS) ? B. Carbamazepine B. 305 A. Sulfonamides 1130 Increased beta lipoproteins result in which of the following types of xanthoma ? B. 307 D. > 20 % A. Sebaceous carcinoma 1126 Which of the following urticarias is typically present in B. Emotion Harrison’s 16th Ed. > 30 % B. Lymphocytic leukemia Harrison’s 16th Ed. All of the above A. All of the above B. Hyperthyroidism can lead to pseudoxanthoma elasticum (PXE) like skin changes ? C. Chloroquine A. 307 B. >5% 1128 “Button-hole”sign is typical of ? B.916 MCQ’s FOR MEDICAL PROFESSIONALS BY PROF. Multiple myeloma D. Endometrial carcinoma Harrison’s 16th Ed. 306 C. Mastocytosis 1132 Long-term administration of which of the following drugs B. Erythema infectiosum B. 305 D. Beta blockers 1125 Cholinergic urticaria is precipitated by ? B. Angiofibromas 1136 Which of the following has the highest risk of severe adverse C. Gout (Tophi) C. Rheumatoid nodules D. Rubeola C. Shins Harrison’s 16th Ed. Colon carcinoma Erythropoietic protoporphyria ? C. Exercise 1133 Patients with Torre syndrome can have carcinomas of ? C. Systemic lupus erythematosus B. Hepatitis B Harrison’s 16th Ed. Calcium channel blockers Harrison’s 16th Ed. Solar urticaria B. Olecranon D. Phenytoin A. Kawasaki’s disease A. 307 D. 321 D. D-penicillamine B. 305 A. Lamotrigine 1129 Yellow colored cutaneous papules/plaques are seen in ? Harrison’s 16th Ed. All of the above Harrison’s 16th Ed. All of the above Harrison’s 16th Ed. Xanthelasma D. 322 A. All of the above A. All of the above A. Toxic shock syndrome 1131 Lesions of necrobiosis lipoidica in diabetes are found on ? Harrison’s 16th Ed. Lamotrigine Harrison’s 16th Ed. AJAY MATHUR Miscellaneous 1123 Coronary aneurysm secondary to arteritis in which of the following C. Heat D. Waldenstrom’s macroglobulinemia 1127 Urticarial vasculitis is a feature of ? C. Phenobarbitone C. Tendon xanthomas . Cold urticaria 1134 Monoclonal cryoglobulinemia is associated with ? Harrison’s 16th Ed. Sjogren’s syndrome 1135 What percentage of epidermal detachment defines Toxic epidermal necrolysis (TEN) ? C. 322 A. IV Vancomycin A. Plane xanthomas disease ? D. Lipomas Harrison’s 16th Ed. Helix of ear C. High-osmolality radiocontrast media B. Prepatellar bursae 1124 Urticarial lesions can be seen in which of the following diseases ? D. Juvenile rheumatoid arthritis Harrison’s 16th Ed. 305 C. 307 C. Cholinergic urticaria A. > 10 % Harrison’s 16th Ed. Pseudoxanthoma elasticum 1137 “Red man syndrome” is caused by ? D. 307 B. Neurofibromas cutaneous reactions ? D. 305 D. All of the above A. All of the above A. Diabetes (Necrobiosis lipoidica) D. 307 C. Carcinoid syndrome . Friedreich’s Ataxia 1142 Which of the following is not a “metabolic photosensitivity C. Polycythemia B. pigmentation and scaling of the skin. B. weight loss. Aluminum poisoning D. Nemaline Myopathy. fixed drug reactions occur within ? 1144 “Shagreen patches” refers to ? Harrison’s 16th Ed. Pemphigus vulgaris 1139 After initiating drug therapy. Chronic lead poisoning 1141 Fitzpatrick Classification is used for ? Harrison’s 16th Ed. Porphyria cutanea tarda . 917 Miscellaneous MCQ’s FORMCQ’s FOR MEDICAL MEDICAL PROFESSIONALS PROFESSIONALS BY PROF. CMT1x D. B. Porphyria A. Dermatomyositis C. Pellagra D. 325 Pes cavus is associated with CMT1x. IFN- A. 2 days A.transverse A. Dermatomyositis white lines of fingernails. 7 days B. Chronic arsenic intoxication D. All of the above C. Erythropoietic porphyria C. 322 Harrison’s 16th Ed. IFN- B. AJAY MATHUR Cardiology 917 1138 Alopecia is a common complication of ? 1143 “Ash leaf spot” is the earliest cutaneous sign in ? Harrison’s 16th Ed. Pellagra D. 300 A. All of the above Harrison’s 16th Ed. IL-2 D. 322 Harrison’s 16th Ed. 300 A.transverse white lines of fingernails is suggestive Harrison’s 16th Ed. 325 Chronic arsenic intoxication can produce confusion and memory loss associated with nausea. 14 days C. Ungual and gingival fibromas C. Adenoma sebaceum B. Friedreich’s Ataxia. 21 days D. Central Core A. Chronic mercury poisoning C. peripheral neuropathy. Stevens-Johnson syndrome B. 2405 B. Connective tissue nevi D. 322 of which of the following ? A. Pemphigus vulgaris Harrison’s 16th Ed. Toxic epidermal necrolysis A. Sunburn sensitivity 1146 Which of the following is associated with pes cavus ? C. Nemaline Myopathy disease” ? D. Granulocyte colony-stimulating factor C. Tuberous sclerosis B.sporadic Disease and Plantar Fasciitis. Fibrous plaques of the forehead 1140 Nikolsky’s sign is seen in ? 1145 Mee’s lines . and Mee’s lines . Notes : . ........... A 129 .......... B 148 ................... A 54 ..........Answer MCQ’s FORMCQ’s FOR MEDICAL MEDICAL PROFESSIONALS PROFESSIONALS BY PROF..... D 39 ..... C 105 ......... D 111 ............... C 25 ..... C 124 ... A 83 ............ A 106 ...................... A 5 ......................... D 141 ........... A 67 ............................... A 52 .. C 144 .......... C 26 . A 44 . C 31 ............... C 50 ..................... D 15 ..................................... B 126 ........................ A 80 .......... AJAY MATHUR 919 ANSWERS MISCELLANEOUS 1 .................. D 91 ........ D 109 ........................... A 115 ... D 92 ... A 20 ........ A 13 ............ C 123 .................................... A 145 ........ C 98 ........................ C 120 ........... B 93 ............................. A 46 . D 69 .................... C 103 ........ B 57 ........ D 135 ...... C 47 ........... A 68 ..... B 51 ............ A 150 ... A 85 ......... C 122 ........................ A 87 . C 112 ..... B 104 .......................................... A 61 ....................................... B 45 ....... A 48 .................. D 7 ........... D ..................... C 37 ......................... B 96 .. D 75 ........................ D 19 ........ B 101 ............. C 134 .................. C 118 ................ B 136 .............. D 55 ....... B 21 ...................... C 53 .......... B 79 ............. A 138 .......... C 24 .......................... C 110 ................................ B 95 ............. A 81 ....... A 108 .................. A 40 ..................... B 66 ................................................ D 28 . B 65 ................... B 139 ......... C 121 .............. D 43 .................. C 2 ... A 71 .............. A 82 ............................. B 30 .................... A 116 ............................................. D 64 .......... D 32 .... B 35 .... A 90 ........................... B 100 ................... C 149 ................. C 63 ......... B 143 ................................ A 78 ...... C 42 .............. C 142 ... A 113 ...... D 133 ............. A 9 ..................... D 56 . D 27 ..... C 18 ........ A 59 ........... A 49 ......... A 128 .... C 11 ............. D 146 .......... C 4 .................. C 97 ..... B 17 .................................... A 34 ............ A 88 .. B 107 ................................... D 72 .... C 38 ............. A 29 ......................................... D 89 ................... A 36 ............................. C 12 ...... B 117 ........................................... D 3 .. A 74 ............... A 84 .. A 131 .................... C 127 ......... B 77 .............. B 62 .................. D 33 ..................... D 73 ................ C 132 ............................................................ A 60 .... A 58 . A 99 .......................... C 76 ......................... A 86 ........ B 70 ................................ D 137 .... B 130 ....................... B 94 .................. D 6 .......................... B 114 ................................ D 125 ............ B 41 ............................ C 119 .............. C 147 .. A 14 .......... A 140 ........ A 22 ................... A 16 ...... C 8 ............ B 102 ........................................ A 23 ... D 10 ........ ........................... C 257 .................... D 199 .................... D 277 ................................ D 201 ......... B 192 ........................ D 193 ................... A 218 ........... D 214 .................. D 186 .. C 256 .. D 159 .............. D 251 .................920 MCQ’s FOR MEDICAL PROFESSIONALS BY PROF............................... B 271 ........................ C 225 ....................................... B 258 .... C 197 . C 229 ............ D 209 .. B 288 ......................... C 228 ...... A 248 .. A 184 ........ B 279 .................... A 205 .. A 255 .... D 241 ............................... C 259 ...... A 245 . D 162 .... B 155 ............. D 194 ................ D 210 . C 223 .... C 300 .......................................... D 161 ..... D 289 ............. B 221 ........ C 278 ........... C 158 ..................... B 261 .. A 244 ............. B 191 ................................ C 295 .. D 173 ................................ D 260 .. C 178 .................................... D 273 ... D 166 . C 291 . B 253 ............................. B 254 .................................... A 172 .............................. A 222 ............. D 215 ........... D 200 ............................. A 287 .............. C 238 .......... B 283 ........ A 270 ..... C 235 ........... D 250 ........... A 219 .. D 181 .............. D 230 .... D 285 ..... D 262 ........ A 220 ......... B 292 . C 190 .... A 290 ......... C 234 .................... C 233 .... D 265 .......................... C 227 ......... B 240 .. C 298 . A 174 ................ C 232 . D 243 ...... D 224 ... D 176 ..... D 153 . C 267 . A 252 .... D 242 ..... D 202 ... B 168 .......... B 281 ............................ A 198 ....... C 213 ........... B 264 ............. A 164 ........... A 207 ................ C 189 ....... C 272 ....................... D 274 ..... D 171 ......................................... D 188 ....... C 211 .... C 266 ...... B 196 ......................... D 185 ........ D 275 ............... A 208 ........... B 299 ... A 226 ....... D 236 ................ D 247 ........... AJAY MATHUR Answer ANSWERS MISCELLANEOUS 151 ........... D 156 ..................... D 160 . D 163 .............................. D 175 .............. C 293 ... A 263 ........................ C 212 .. A 249 ............ C 165 .... C 204 ....... C 297 ......................... C 180 ..... D 195 .......................................... D 216 ................................................................................. D 154 .................................................... A 284 ............ C 167 ................. D 170 .............. A 182 .. D 157 .............. D 239 .... D 231 ............................................. D 169 .... D 237 ........... D 152 .......................................... D 280 .. D ............ D 217 ........ B 177 ........................... D 246 ... D 268 .. C 179 .................... D 294 .......... B 203 .................. D 187 ......... A 282 .. A 296 ......... D 206 ...... D 183 .......................................... A 276 ...................... A 269 ................................ A 286 ............................. ...... C 441 .. A 367 ..................... D 377 ........... B 321 ...... D 420 ............................. C 396 ....................................... B 351 .............. B 445 ......................... C 430 .......................................................... D 331 ....... D 435 ................. D 305 ................................. B 436 ................................ D 318 ...... C 340 ........... A 416 ....................... D 391 . B 307 ....... C 427 ............. D 327 ............. C 357 ....... A 338 ... A 395 ............................................................. B 379 ..................................... D 375 ....................... C 325 .................. D 352 .... B 365 ........................................................... D 439 ..... D 426 .......................... A 415 .. C 380 ..... D 310 . C 350 ..... A 368 . C 344 ..... B 337 ...... D 308 ............... D 442 . D 446 ................... C 332 ... B 389 . D 343 ................. A 449 .... D 403 ..... D 400 .......... A 386 .......... D 381 .. A 345 ............................... D 342 ...... B 409 ......... C 398 ............ D 401 .................................................... D 328 ...... D 385 ......... D 388 ..... C 387 .. B 313 ........................ D 322 ... A 440 .. B 369 ... B 306 ..... D 422 .......... A 405 ................. A 410 ......... D 314 .... C 315 ........... D 333 ... C 334 ........................ A 411 . A 348 ................ B 317 ..................................... D ...... D 329 ................................... A 421 ......... D 364 ................ D 346 ............................. D 383 ........ D 363 ........ D 336 .................................. C 312 ......... D 362 ............... D 320 ..Answer MCQ’s FORMCQ’s FOR MEDICAL MEDICAL PROFESSIONALS PROFESSIONALS BY PROF.... B 450 ............... B 382 .... A 402 ............... D 376 ............................ C 371 ..... C 341 ............................... C 397 ........ A 330 ........ B 339 .. C 428 .. C 393 .......... D 374 ................................ D 303 ............................... D 419 ...... D 378 ... A 304 ........................ D 324 ... A 302 ...... B 438 ................................. D 418 ................. D 431 ......... C 349 ......................... B 361 ........... D 373 ................... D 353 ..................... D 404 .... D 417 ..................... D 347 ............ B 423 . B 370 ........ A 425 ... D 366 ..... A 412 .............................. D 394 ................................ D 384 .... A 447 ....... B 424 ......... D 372 ..... C 316 ... C 444 .......... C 443 ... AJAY MATHUR 921 ANSWERS MISCELLANEOUS 301 .......................... D 406 ... A 408 ................... C 434 .................... D 360 ..... C 407 .............................................. C 433 ................ D 432 .......... C 356 .... C 311 . D 392 .................... C 429 ............. C 326 ................ D 359 ................................... B 414 .... B 390 . B 413 ........ C 335 ........ D 358 ...... D 323 ......... A 437 ............. D 448 ........................................ A 319 ....... D 355 ....... C 399 ................................ D 354 . D 309 .. ..................... A 515 ......................................... C 557 ........... C 518 ....... C 495 .................. B 459 ................... C 481 .................. C 579 ................ D 589 ........ C 520 .............. B 572 .... B 570 ... A 494 ............... D 565 ...... B 482 ........................... B 544 ..... D 489 .... C 462 ..... AJAY MATHUR Answer ANSWERS MISCELLANEOUS 451 .............. B 590 ............ D 541 ................ B 512 ........ C 535 ....... A 571 ... C 538 ...... D 460 ................... B 561 ....................... D 471 ........... C 587 ..... C 533 ................... C 508 ............................... B 486 ............ A 505 ..... A 503 ................................. D 540 ................. B 543 ....... A 506 . C 468 ...................................... D 531 . B 465 ........... D 564 .. C 583 .......................................... C 573 .... D 545 .. A 574 ........................................................... A 499 .... B 509 ...................... D 470 ..................... A 454 ......................... D 466 . C 478 ..................................................... D 599 .............. C 463 ......... D 586 ............................. C ...... B 550 ...... D 542 ...... D 554 .. A 514 ....................... D 458 ................................. A 600 ............ C 556 ..................................................................... D 534 ..................... C 513 .922 MCQ’s FOR MEDICAL PROFESSIONALS BY PROF..................... D 473 ................ B 551 ........... D 563 . B 528 ..................... B 476 ................................................................................... C 555 ............................... D 568 ..... C 493 ..... D 591 ........... B 549 ........ A 483 ..... B 569 ......... C 580 ............ B 530 . D 480 ................................ A 490 ................. D 521 .......... D 492 .............. C 469 ... B 464 ...... C 581 ...... D 501 ................... D 558 .............. C 452 .... B 455 ..................................... D 517 ....... A 479 .... D 539 ... A 523 .......................... D 585 .......... D 510 ................. B 576 .... C 537 .................................... D 467 ......................... D 516 . A 504 .............. A 453 .............. C 553 .. A 597 ................. D 475 ... A 532 ... D 526 .............. C 566 ............ D 519 ..... C 511 .......... C 487 .. D 491 ............... D 560 .............. C 546 .................. C 474 ........ D 502 ......................... D 567 ......... D 456 .............. A 575 ........ D 529 .... A 598 . D 488 .......... C 596 ............... B 562 ................ A 593 .......... C 500 ..... D 498 ......................... B 496 ...... C 594 .......... C 461 ...... A 457 .... D 497 ................. C 524 ... D 559 . D 536 ................. D 527 ...... C 547 ........ A 522 .. A 477 .... A 484 ................ D 582 ........................ C 588 ..... B 595 ...................... C 552 ........ D 592 ...... C 472 .... B 507 .... D 548 .... D 485 ................... C 578 .... D 525 ............. B 584 .......... B 577 ............................................ ............... C 623 ....... A 678 ........... C 635 ..... A 613 ............................ D 676 ............ B 608 ............ D 610 ....... C 710 .......................... D 685 ........ C 603 ... D 624 ......................... C 715 ..... A 662 ............. C 746 .. D 666 .... D 638 ...................... D 653 ...................................................................... C 683 ..... B 602 ............... B 744 ........................... B 740 ....... B 618 .................. D 622 . A 690 .....Answer MCQ’s FORMCQ’s FOR MEDICAL MEDICAL PROFESSIONALS PROFESSIONALS BY PROF.. D 674 ........... C 647 ............... A 656 . A 625 ........................................... A 652 ........ C 639 .. A 663 .... C 703 ......... C 612 .. B 733 ....... B 607 ...................... D 706 ........... A 637 ............ A 619 ........... D 651 ...................... C 661 ....... D 748 .. D 628 .......... AJAY MATHUR 923 ANSWERS MISCELLANEOUS 601 ....... D 736 .................... D 667 .............................. C 671 ........................... C 726 .... C 620 . D 727 .................. A 682 ...................................................................... A 731 .......... A 701 ...... B 657 ........ B 750 ......................... B 742 ... D 694 ................................................. C 659 ......... B 691 ........................ D 640 . D 709 ............... D 749 ........ D 708 . D 723 ....... A 681 ............ D 745 ... B 631 .. C 728 .................................... D 675 .............................. C 695 ....... B 692 ............ D 737 . C 735 ... D 632 .... D 730 ................. D 609 ..................................... B 645 .... C 741 .... C 616 ...... D 747 ... D 713 ....................... D 668 ...... C 633 ...... B 670 ...... B 743 . D 648 ......... C 720 ............. A 704 ........ B 655 .......................... D 650 . D 724 ............ D 621 ................... C 672 ............. D 739 ................................................ B 712 .. B 699 . A 698 ............................ B 615 .... C 629 .. D 649 . B 654 .... C 660 .. B 688 ........ D 664 .... D 617 .... B 665 ...... A 604 ................. A 696 ...... D 734 .................... C 626 ... C 669 ................... D ... B 680 ....................................................... B 644 ............ D 641 . C 718 ........................................... D 729 ............... A 697 ..................... D 673 .................... D 693 ...... D 707 ........................... D 684 .................. D 606 .............. D 689 .......................................................... B 646 ..................... C 725 .......... C 719 . D 722 ......... C 636 ......... C 721 . C 658 ....................... B 687 ........................... D 642 ...................... A 611 ...... D 605 ................................. D 679 .......... D 686 ..... D 705 ...................... D 716 .. C 738 .... C 700 .... D 643 ............................ C 627 ... D 717 ............................. A 677 ................. D 702 ....... D 630 . D 614 .................................................................... B 714 ... D 711 ...................... D 634 ............. D 732 ......... .. D 883 ........................ D 804 ............ B 853 ......... D 863 ........................... AJAY MATHUR Answer ANSWERS MISCELLANEOUS 751 ..... B 845 ............................ A 767 .......... C 784 .. A 858 .................. D 771 ........ B 899 ............... A 866 .. B 842 ............ D 813 .. D 834 .......... D 887 ... C 777 . A 823 ................. A 793 . A 811 . D 832 ...... D 824 ........................ C 896 ..................... B 792 ........ A 873 ....... D 840 ....................... B 814 ....... D 774 ............. D 801 ... A 837 ........... C 766 . D 833 .................................................................... D 890 ......................... D 808 ... C 851 ........................... D 874 .... B 871 ........................ D 805 ...... D 829 . D 763 ...... C 815 ........................................................ C 877 . A 884 .............................. C 760 ....... D 812 .......... A 761 ............ A 803 .... D 846 .. D 764 ....................... B 831 .... D 768 ........ D 860 ......................... D 857 ... A 880 ............. D 849 ...... A 782 ........ A 807 ................ A 868 ... D 841 ..... D 844 . B 872 ..................... D 820 ...................................... C 818 ......... B 891 .................. D 800 .... D 879 ..... B 881 ..... D 876 ... B 821 ............................ B 799 .............. A 859 ....................... B 875 ................ D 894 ...... D 753 ................ D 854 ..................... B 772 ..................................... D 839 .................................................. C 843 ...... C 830 . B 769 ................. D 867 ............. B 754 ....................... C 759 ................................... D 757 ................................ D 776 ... D 786 .................... B 802 .... A 780 ............... A 752 .......... D 848 ........................ D 822 ................. A 794 ................. D 836 . C 762 . D 773 ..... D 809 ........... D 889 ................ D 827 ................... B 826 ............. C 886 ....................... B 765 .......... B 852 ....... D 838 ................... A 781 ... C 869 ...... A 861 ..... D 779 ........................... A 855 ............................... C 795 .................................................. C 817 ....... C 865 .......................... D 783 .......... B 806 .................. B 888 ............ C 856 .. A 885 ................................................................. C 770 .. B 898 .... D 790 ... C 835 .. C 810 ......... C 892 ....................................................................... A 864 ................. C 862 ..... D 825 ..... C 778 . A 878 ................. A 895 .. B 787 ..... C 893 ............ A 788 .... C 758 ..... C 775 ........ D 797 .... B 816 .. D 798 .......... A 897 ............. D .. D 791 ............... A 850 ....................................... A 900 ................................... C 870 ...... B 755 .............. D 819 ................ C 828 . B 847 ................. C 882 ......... D 796 ........924 MCQ’s FOR MEDICAL PROFESSIONALS BY PROF............... A 789 .... C 756 .................... A 785 ............................ D 987 .. D 902 ........ A 993 ..... C 946 ................. B 1000 ......... A 936 ............ A 937 ....... C 994 .... A 969 ................... D 991 ...................... D 1039 ........................ D 1050 ................................. D 982 . B 1032 ..... B 988 ...... C 1046 ............ A 929 ........ A 922 ... A 944 ................................... B 921 ......... C 948 .............. A 1049 ................................ D 998 ........ C 1005 ... D 996 .... A 931 ....... D 907 ...... D 1024 ................................... D 938 ................. C 920 ................... D 1033 ......................... AJAY MATHUR 925 ANSWERS MISCELLANEOUS 901 .......... D 956 ................... A 941 ......... C 986 ................ D 962 .. D 1040 .... D 932 ........... C 1036 .................. D 911 ......................................................................... C 952 ... C 1025 .................................................... D 978 ................................................... B 1021 . C 927 . D 914 ............ B 959 .. D 1015 ..................... C 958 ..................... A 930 ................................................................... D 966 ... D 908 ............... A 973 . A 1029 ..... D 985 ........ C 935 .................. C 963 ................. A 980 ....... D 912 ..... D 933 . A 984 ..... D 1038 ................................................. C 905 .... C 1003 ......... A 1031 ...... B 995 ... D 904 ..... D 1014 ............. A 1002 ............... D 947 ......................... A 957 .. D 917 ... D 942 ........ A 1043 ..... C 989 ........... D 997 ............................. D 1008 ................ D 1020 ..... B 974 ........ C 1030 .... D 1042 ...................... D 983 ................ B 965 ..... D 1004 ............. D 1019 ..... D . B 924 .. D 909 .. C 925 ................ C 950 .......... A 1012 ............................... D 1001 ......... D 1026 ......... C 967 ........ D 1034 ......... C 1017 .................... D 990 .................. C 970 ................................. D 1048 .......................................... D 1047 ..... D 1037 ................ B 939 ............. B 1027 ......... B 1016 ....Answer MCQ’s FORMCQ’s FOR MEDICAL MEDICAL PROFESSIONALS PROFESSIONALS BY PROF....................................... D 951 ....... A 945 .......... D 971 .......... C 1044 .... D 961 .. C 915 ................................... A 1041 .... D 1028 .. C 953 .............. D 976 .... C 913 .... D 1045 ....... A 960 ....... D 955 .............. B 949 ... D 968 .. B 1007 ....................... D 954 ........................ D 918 ... C 926 ..................... C 923 .............. D 906 ............ B 972 ........... B 992 . A 903 ................ C 1022 ....... D 999 ................................. A 1018 ............................. A 981 ... D 1013 ............ C 934 ... B 1011 ............ B 1010 .......... D 979 ........... C 943 .. A 1006 .... B 919 ........................... C 1023 ....... C 916 .................................... B 928 .......................... D 1009 ... D 975 .............. D 1035 .................... A 977 ......... D 910 ... A 964 . A 940 ........ ................................... C 1122 .................. A 1077 .............................. A 1084 ....... B 1104 ................ D 1066 .... A 1087 .............. D 1108 ................... B 1078 ....... D 1133 ..... C 1115 ... D 1083 .................. D 1076 .... D 1120 ..................... A 1085 ... C 1081 ........................ D 1138 ..... D 1060 .. D 1145 ........................................... D 1053 . A 1105 . D 1109 .............. B 1142 ..................... D 1125 .. B 1074 ..........926 MCQ’s FOR MEDICAL PROFESSIONALS BY PROF...... D 1124 ....................................................................................................... D 1075 .. D 1139 .... A 1069 ..................... D 1123 ..... D 1111 .... A 1113 .... D 1056 ....... B 1117 . D 1128 ......... B 1127 ......... C 1110 . C 1091 ...... D ......................... D 1072 ....... D 1092 .......... C 1086 .... A 1107 ........ B 1098 ... B 1143 ....... A 1080 ... A 1062 ......................... B 1130 . B 1116 .. A 1052 ............................. D 1058 .. D 1079 ... A 1064 ............................ B 1136 ........... D 1057 ....... C 1106 ..... D 1135 ..... C 1054 ...... B 1112 ....... B 1082 .......................... D 1061 ..................... C 1090 .... D 1146 .. C 1126 ...................................... C 1088 ..... C 1129 .... C 1132 .... A 1065 ... D 1100 .. B 1067 .......... D 1102 ...... C 1071 .... C 1094 ......................... D 1097 ... D 1073 . C 1144 ............. D 1131 .......... A 1114 .................................. B 1141 ........... D 1055 ............... A 1093 ............. D 1140 ....... A 1118 .................................. C 1137 .............. D 1099 .. AJAY MATHUR Answer ANSWERS MISCELLANEOUS 1051 .. D 1059 ................................ C 1089 ... B 1103 .... C 1095 ....................... D 1121 ......................... B 1068 . B 1119 ...... C 1101 ............ C 1063 ............ A 1134 ... C 1096 ..... C 1070 ........
Copyright © 2024 DOKUMEN.SITE Inc.